Fundamentals of Physics [10 ed.] 9781118230718, 111823071X

The 10th edition of Halliday, Resnick and Walkers Fundamentals of Physics provides the perfect solution for teaching a 2

2,649 210 50MB

English Pages 1232 [1450] Year 2013

Report DMCA / Copyright

DOWNLOAD FILE

Polecaj historie

Fundamentals of Physics [10 ed.]
 9781118230718, 111823071X

Citation preview

MATHEMATICAL FORMULAS* Quadratic Formula If ax2 # bx # c " 0, then x "

Derivatives and Integrals

nx n(n $ 1)x2 # # ... 1! 2!

(x2 ( 1)

" " "

:

Let u be the smaller of the two angles between a and b . :

Then :

:

a ! b " b ! a " axbx # ayby # azbz " ab cos u :

!

ˆi : : : : a ' b " $b ' a " ax bx

!

ˆj ay by

! !

ay az ax $ ˆj " ˆi by bz bx

!

kˆ az bz

dx 2x2 # a2

ex dx " ex

" ln(x # 2x2 # a2)

x dx 1 "$ 2 (x2 # a2)3/2 (x # a2)1/2 dx x " 2 2 (x2 # a2)3/2 a (x # a2)1/2

Cramer’s Rule

! !

az ax # kˆ bz bx

Two simultaneous equations in unknowns x and y,

!

ay by

a1x # b1 y " c1 and a2x # b2 y " c2, have the solutions

" (aybz $ by az)iˆ # (azbx $ bzax)jˆ # (axby $ bxay)kˆ

x"

! !

y"

! !

:

|a ' b | " ab sin u :

Trigonometric Identities

cos a # cos b " 2 cos

# b) cos

1 2 (a

! !

"

c1b2 $ c2b1 a1b2 $ a2b1

! !

"

a1c2 $ a2c1 . a1b2 $ a2b1

c1 c2

b1 b2

a1 a2

b1 b2

and

sin a % sin b " 2 sin 12(a % b) cos 12(a & b) 1 2 (a

cos x dx " sin x

d x e " ex dx

Products of Vectors

:

sin x dx " $cos x

d cos x " $sin x dx

Binomial Theorem (1 # x)n " 1 #

" " "

d sin x " cos x dx

$b % 1b2 $ 4ac 2a

$ b)

*See Appendix E for a more complete list.

a1 c1 a2 c2

a1 b1 a2 b2

SI PREFIXES* Factor

Prefix

Symbol

Factor

Prefix

Symbol

1024 1021 1018 1015 1012 109 106 103 102 101

yotta zetta exa peta tera giga mega kilo hecto deka

Y Z E P T G M k h da

10–1 10–2 10–3 10–6 10–9 10–12 10–15 10–18 10–21 10–24

deci centi milli micro nano pico femto atto zepto yocto

d c m m n p f a z y

*In all cases, the first syllable is accented, as in ná-no-mé-ter.

E

X

T

E

N

D

E

D

FUNDAMENTALS OF PHYSICS

T

E

N

T

H

E

D

I

T

I

O

N

This page intentionally left blank

E

X

T

E

N

D

E

D

Halliday & Resnick

FUNDAMENTALS OF PHYSICS

T

E

N

T

H

E

D

I

T

I

O

N

J EAR L WALK E R CLEVELAND STATE UNIVERSITY

EXECUTIVE EDITOR Stuart Johnson SENIOR PRODUCT DESIGNER Geraldine Osnato CONTENT EDITOR Alyson Rentrop ASSOCIATE MARKETING DIRECTOR Christine Kushner TEXT and COVER DESIGNER Madelyn Lesure PAGE MAKE-UP Lee Goldstein PHOTO EDITOR Jennifer Atkins COPYEDITOR Helen Walden PROOFREADER Lilian Brady SENIOR PRODUCTION EDITOR Elizabeth Swain COVER IMAGE © 2007 CERN This book was set in 10/12 Times Ten by cMPreparé, CSR Francesca Monaco, and was printed and bound by Quad Graphics.The cover was printed by Quad Graphics. This book is printed on acid free paper.

Copyright © 2014, 2011, 2008, 2005 John Wiley & Sons, Inc. All rights reserved. No part of this publication may be reproduced, stored in a retrieval system or transmitted in any form or by any means, electronic, mechanical, photocopying, recording, scanning or otherwise, except as permitted under Sections 107 or 108 of the 1976 United States Copyright Act, without either the prior written permission of the Publisher, or authorization through payment of the appropriate per-copy fee to the Copyright Clearance Center, Inc. 222 Rosewood Drive, Danvers, MA 01923, website www.copyright.com. Requests to the Publisher for permission should be addressed to the Permissions Department, John Wiley & Sons, Inc., 111 River Street, Hoboken, NJ 07030-5774, (201)748-6011, fax (201)748-6008, or online at http://www.wiley.com/go/permissions. Evaluation copies are provided to qualified academics and professionals for review purposes only, for use in their courses during the next academic year.These copies are licensed and may not be sold or transferred to a third party. Upon completion of the review period, please return the evaluation copy to Wiley. Return instructions and a free of charge return shipping label are available at www.wiley.com/go/returnlabel. Outside of the United States, please contact your local representative. Library of Congress Cataloging-in-Publication Data Walker, Jearl Fundamentals of physics / Jearl Walker, David Halliday, Robert Resnick—10th edition. volumes cm Includes index. ISBN 978-1-118-23072-5 (Extended edition) Binder-ready version ISBN 978-1-118-23061-9 (Extended edition) 1. Physics—Textbooks. I. Resnick, Robert. II. Halliday, David. III. Title. QC21.3.H35 2014 530—dc23 2012035307 Printed in the United States of America 10 9 8 7 6 5 4 3 2 1

B

R

I

E

F

C

O

N

T

E

N

T

S

V O L U M E 1

V O L U M E 2

1 Measurement

21 Coulomb’s Law

2 Motion Along a Straight Line

22 Electric Fields

3 Vectors

23 Gauss’ Law

4 Motion in Two and Three Dimensions

24 Electric Potential

5 Force and Motion—I

25 Capacitance

6 Force and Motion—II

26 Current and Resistance

7 Kinetic Energy and Work

27 Circuits

8 Potential Energy and Conservation of Energy

28 Magnetic Fields

9 Center of Mass and Linear Momentum

29 Magnetic Fields Due to Currents

10 Rotation

30 Induction and Inductance

11 Rolling, Torque, and Angular Momentum

31 Electromagnetic Oscillations and Alternating

12 Equilibrium and Elasticity

Current

13 Gravitation

32 Maxwell’s Equations; Magnetism of Matter

14 Fluids

33 Electromagnetic Waves

15 Oscillations

34 Images

16 Waves—I

35 Interference

17 Waves—II

36 Diffraction

18 Temperature, Heat, and the First Law of

37 Relativity

Thermodynamics

38 Photons and Matter Waves

19 The Kinetic Theory of Gases

39 More About Matter Waves

20 Entropy and the Second Law of Thermodynamics

40 All About Atoms 41 Conduction of Electricity in Solids 42 Nuclear Physics 43 Energy from the Nucleus 44 Quarks, Leptons, and the Big Bang

Appendices / Answers to Checkpoints and Odd-Numbered Questions and Problems / Index v

C

O

N

T

E

1 Measurement 1 1-1 MEASURING THINGS, INCLUDING LENGTHS What Is Physics? 1 Measuring Things 1 The International System of Units 2 Changing Units 3 Length 3 Significant Figures and Decimal Places

1-2 TIME Time

1

3-3 MULTIPLYING VECTORS Multiplying Vectors

QUESTIONS

55

What Is Physics? 62 Position and Displacement

PROBLEMS

56

4 Motion in Two and Three Dimensions 4-1 POSITION AND DISPLACEMENT 62

4

57

62

63

6

4-2 AVERAGE VELOCITY AND INSTANTANEOUS VELOCITY 8

PROBLEMS

Average Velocity and Instantaneous Velocity

8

2 Motion Along a Straight Line 13 2-1 POSITION, DISPLACEMENT, AND AVERAGE VELOCITY What Is Physics? 13 Motion 14 Position and Displacement 14 Average Velocity and Average Speed

Instantaneous Velocity and Speed

2-3 ACCELERATION

Average Acceleration and Instantaneous Acceleration

13

Projectile Motion

18

4-6 RELATIVE MOTION IN ONE DIMENSION

Relative Motion in Two Dimensions

23

REVIEW & SUMMARY

23

27

30

29

29

QUESTIONS

3 Vectors 40 3-1 VECTORS AND THEIR COMPONENTS What Is Physics? 40 Vectors and Scalars 40 Adding Vectors Geometrically Components of Vectors 42

81

80

80

QUESTIONS

82

5 Force and Motion—I 94 5-1 NEWTON’S FIRST AND SECOND LAWS

27

Graphical Integration in Motion Analysis

31

PROBLEMS

40

PROBLEMS

84

32

5-2 SOME PARTICULAR FORCES

41

102

102

5-3 APPLYING NEWTON’S LAWS 46

94

What Is Physics? 94 Newtonian Mechanics 95 Newton’s First Law 95 Force 96 Mass 97 Newton’s Second Law 98

Some Particular Forces

3-2 UNIT VECTORS, ADDING VECTORS BY COMPONENTS 46

78

26

2-6 GRAPHICAL INTEGRATION IN MOTION ANALYSIS REVIEW & SUMMARY

78

4-7 RELATIVE MOTION IN TWO DIMENSIONS

20

Free-Fall Acceleration

76

76

Relative Motion in One Dimension

2-5 FREE-FALL ACCELERATION

68

70

Uniform Circular Motion

18

Constant Acceleration: A Special Case Another Look at Constant Acceleration

67

70

4-5 UNIFORM CIRCULAR MOTION

15

20

2-4 CONSTANT ACCELERATION

64

65

4-3 AVERAGE ACCELERATION AND INSTANTANEOUS ACCELERATION 4-4 PROJECTILE MOTION

2-2 INSTANTANEOUS VELOCITY AND SPEED

Unit Vectors

50

50

REVIEW & SUMMARY

6

Acceleration

S

Adding Vectors by Components 46 Vectors and the Laws of Physics 47

5

REVIEW & SUMMARY

vi

T

5

1-3 MASS Mass

N

106

Newton’s Third Law 106 Applying Newton’s Laws 108 REVIEW & SUMMARY

114

QUESTIONS

114

PROBLEMS

116

CONTE NTS

6 Force and Motion—II 6-1 FRICTION 124

8-4 WORK DONE ON A SYSTEM BY AN EXTERNAL FORCE

124

Work Done on a System by an External Force

What Is Physics? 124 Friction 124 Properties of Friction 127

8-5 CONSERVATION OF ENERGY Conservation of Energy

The Drag Force and Terminal Speed

130

6-3 UNIFORM CIRCULAR MOTION

133

Uniform Circular Motion

QUESTIONS

200

9 Center of Mass and Linear Momentum 9-1 CENTER OF MASS 214

QUESTIONS

PROBLEMS

PROBLEMS

139

140

214

What Is Physics? 214 The Center of Mass 215

9-2 NEWTON’S SECOND LAW FOR A SYSTEM OF PARTICLES 7 Kinetic Energy and Work 7-1 KINETIC ENERGY 149

Newton’s Second Law for a System of Particles

149

9-3 LINEAR MOMENTUM

Work 151 Work and Kinetic Energy

Collision and Impulse

155

159

159

Elastic Collisions in One Dimension

162

162

166

166

QUESTIONS

169

PROBLEMS

8 Potential Energy and Conservation of Energy 8-1 POTENTIAL ENERGY 177 What Is Physics? 177 Work and Potential Energy 178 Path Independence of Conservative Forces Determining Potential Energy Values 181

179

8-2 CONSERVATION OF MECHANICAL ENERGY Conservation of Mechanical Energy

187

240

240

9-9 SYSTEMS WITH VARYING MASS: A ROCKET 170

Systems with Varying Mass: A Rocket

187

241

241

REVIEW & SUMMARY 243

QUESTIONS

10 Rotation 257 10-1 ROTATIONAL VARIABLES

257

245

PROBLEMS

246

177

What Is Physics? 258 Rotational Variables 259 Are Angular Quantities Vectors?

264

10-2 ROTATION WITH CONSTANT ANGULAR ACCELERATION Rotation with Constant Angular Acceleration

184

8-3 READING A POTENTIAL ENERGY CURVE Reading a Potential Energy Curve

184

237

237

9-8 COLLISIONS IN TWO DIMENSIONS Collisions in Two Dimensions

233

233

9-7 ELASTIC COLLISIONS IN ONE DIMENSION

Work Done by a General Variable Force

REVIEW & SUMMARY 168

230

Momentum and Kinetic Energy in Collisions Inelastic Collisions in One Dimension 234

7-5 WORK DONE BY A GENERAL VARIABLE FORCE

Power

230

9-6 MOMENTUM AND KINETIC ENERGY IN COLLISIONS

156

7-4 WORK DONE BY A SPRING FORCE Work Done by a Spring Force

226

Conservation of Linear Momentum

Work Done by the Gravitational Force

225

226

9-5 CONSERVATION OF LINEAR MOMENTUM

152

7-3 WORK DONE BY THE GRAVITATIONAL FORCE

7-6 POWER

224

9-4 COLLISION AND IMPULSE 151

220

220

Linear Momentum 224 The Linear Momentum of a System of Particles

What Is Physics? 149 What Is Energy? 149 Kinetic Energy 150

7-2 WORK AND KINETIC ENERGY

202

130

133

REVIEW & SUMMARY 138

195

195

REVIEW & SUMMARY 199

6-2 THE DRAG FORCE AND TERMINAL SPEED

191

192

10-3 RELATING THE LINEAR AND ANGULAR VARIABLES Relating the Linear and Angular Variables

266

266

268

268

vii

viii

CONTE NTS

10-4 KINETIC ENERGY OF ROTATION Kinetic Energy of Rotation

271

10-5 CALCULATING THE ROTATIONAL INERTIA Calculating the Rotational Inertia

10-6 TORQUE Torque

Equilibrium 327 The Requirements of Equilibrium The Center of Gravity 330

271

273

12-2 SOME EXAMPLES OF STATIC EQUILIBRIUM

273

Some Examples of Static Equilibrium

277

12-3 ELASTICITY

278

10-7 NEWTON’S SECOND LAW FOR ROTATION Newton’s Second Law for Rotation

279

Work and Rotational Kinetic Energy 285

REVIEW & SUMMARY

PROBLEMS

286

11 Rolling, Torque, and Angular Momentum 295 11-1 ROLLING AS TRANSLATION AND ROTATION COMBINED What Is Physics? 295 Rolling as Translation and Rotation Combined

11-3 THE YO-YO

287

338

QUESTIONS

343

13 Gravitation 354 13-1 NEWTON’S LAW OF GRAVITATION What Is Physics? 354 Newton’s Law of Gravitation

295

355

Gravitation and the Principle of Superposition

13-3 GRAVITATION NEAR EARTH’S SURFACE

298

Gravitation Near Earth’s Surface

298

Gravitation Inside Earth

363

11-4 TORQUE REVISITED

Gravitational Potential Energy

303

Angular Momentum

Planets and Satellites: Kepler’s Laws

305

Newton’s Second Law in Angular Form

Satellites: Orbits and Energy

307

307

Einstein and Gravitation

310

The Angular Momentum of a System of Particles 310 The Angular Momentum of a Rigid Body Rotating About a Fixed Axis

11-8 CONSERVATION OF ANGULAR MOMENTUM Conservation of Angular Momentum

12 Equilibrium and Elasticity 12-1 EQUILIBRIUM 327 327

327

376

QUESTIONS

What Is Physics? 386 What Is a Fluid? 386 Density and Pressure 387

317

QUESTIONS

374

14 Fluids 386 14-1 FLUIDS, DENSITY, AND PRESSURE

312

317

318

374

319

PROBLEMS

377

311

312

11-9 PRECESSION OF A GYROSCOPE

What Is Physics?

REVIEW & SUMMARY

371

371

13-8 EINSTEIN AND GRAVITATION

11-7 ANGULAR MOMENTUM OF A RIGID BODY

320

14-2 FLUIDS AT REST Fluids at Rest

388

389

14-3 MEASURING PRESSURE Measuring Pressure

368

369

13-7 SATELLITES: ORBITS AND ENERGY

11-6 NEWTON’S SECOND LAW IN ANGULAR FORM

REVIEW & SUMMARY

364

364

13-6 PLANETS AND SATELLITES: KEPLER’S LAWS

305

Precession of a Gyroscope

359

362

13-5 GRAVITATIONAL POTENTIAL ENERGY 302

357

357

360

13-4 GRAVITATION INSIDE EARTH

301

11-5 ANGULAR MOMENTUM

345

354

The Yo-Yo 302

Torque Revisited

PROBLEMS

343

13-2 GRAVITATION AND THE PRINCIPLE OF SUPERPOSITION

295

11-2 FORCES AND KINETIC ENERGY OF ROLLING The Kinetic Energy of Rolling The Forces of Rolling 299

332

282

282

QUESTIONS

332

338

Indeterminate Structures Elasticity 339

279

10-8 WORK AND ROTATIONAL KINETIC ENERGY REVIEW & SUMMARY

329

392

392

386

PROBLEMS

378

ix

CONTE NTS

14-4 PASCAL’S PRINCIPLE Pascal’s Principle

16-4 THE WAVE EQUATION

393

The Wave Equation

393

14-5 ARCHIMEDES’ PRINCIPLE Archimedes’ Principle

16-5 INTERFERENCE OF WAVES

394

458

The Principle of Superposition for Waves Interference of Waves 459

395

14-6 THE EQUATION OF CONTINUITY

456

456

398

Ideal Fluids in Motion 398 The Equation of Continuity 399

16-6 PHASORS

14-7 BERNOULLI’S EQUATION

16-7 STANDING WAVES AND RESONANCE

Bernoulli’s Equation

Phasors

401

401

REVIEW & SUMMARY

405

QUESTIONS

15 Oscillations 413 15-1 SIMPLE HARMONIC MOTION

405

421

17-3 INTERFERENCE

424

15-5 DAMPED SIMPLE HARMONIC MOTION

428

472

PROBLEMS

506

482

485

485

Sources of Musical Sound

17-6 BEATS Beats

432

488

489

PROBLEMS

436

492

493

496

497

17-7 THE DOPPLER EFFECT The Doppler Effect

498

499

17-8 SUPERSONIC SPEEDS, SHOCK WAVES

444

Supersonic Speeds, Shock Waves

What Is Physics? 445 Types of Waves 445 Transverse and Longitudinal Waves 445 Wavelength and Frequency 446 The Speed of a Traveling Wave 449

16-2 WAVE SPEED ON A STRETCHED STRING

PROBLEMS

482

17-5 SOURCES OF MUSICAL SOUND

432

434

471

479

Intensity and Sound Level

430

QUESTIONS

Wave Speed on a Stretched String

QUESTIONS

470

17-4 INTENSITY AND SOUND LEVEL

430

15-6 FORCED OSCILLATIONS AND RESONANCE

16 Waves—I 444 16-1 TRANSVERSE WAVES

423

Interference

Pendulums 425 Simple Harmonic Motion and Uniform Circular Motion

434

Traveling Sound Waves

423

15-4 PENDULUMS, CIRCULAR MOTION

467

17-2 TRAVELING SOUND WAVES

421

An Angular Simple Harmonic Oscillator

REVIEW & SUMMARY

Standing Waves 465 Standing Waves and Resonance

465

What Is Physics? 479 Sound Waves 479 The Speed of Sound 480

419

15-3 AN ANGULAR SIMPLE HARMONIC OSCILLATOR

Forced Oscillations and Resonance

462

17 Waves—II 479 17-1 SPEED OF SOUND

413

15-2 ENERGY IN SIMPLE HARMONIC MOTION

Damped Simple Harmonic Motion

406

462

REVIEW & SUMMARY

What Is Physics? 414 Simple Harmonic Motion 414 The Force Law for Simple Harmonic Motion Energy in Simple Harmonic Motion

PROBLEMS

458

REVIEW & SUMMARY

504

503

503

QUESTIONS

505

18 Temperature, Heat, and the First Law of Thermodynamics 18-1 TEMPERATURE 514 452

452

16-3 ENERGY AND POWER OF A WAVE TRAVELING ALONG A STRING 454 Energy and Power of a Wave Traveling Along a String

454

What Is Physics? 514 Temperature 515 The Zeroth Law of Thermodynamics Measuring Temperature 516

515

18-2 THE CELSIUS AND FAHRENHEIT SCALES The Celsius and Fahrenheit Scales

518

518

514

x

CONTE NTS

18-3 THERMAL EXPANSION Thermal Expansion

Change in Entropy 585 The Second Law of Thermodynamics

520

520

18-4 ABSORPTION OF HEAT

20-2 ENTROPY IN THE REAL WORLD: ENGINES

522

Entropy in the Real World: Engines

Temperature and Heat 523 The Absorption of Heat by Solids and Liquids

524

18-5 THE FIRST LAW OF THERMODYNAMICS

528

REVIEW & SUMMARY

QUESTIONS

19 The Kinetic Theory of Gases 19-1 AVOGADRO’S NUMBER 549

PROBLEMS

540

541

Ideal Gases

598

598

QUESTIONS

602

21 Coulomb’s Law 21-1 COULOMB’S LAW

Charge Is Quantized

551

Charge Is Conserved

554

19-4 TRANSLATIONAL KINETIC ENERGY

557

22 Electric Fields 630 22-1 THE ELECTRIC FIELD 630

557

558

19-6 THE DISTRIBUTION OF MOLECULAR SPEEDS

560

The Molar Specific Heats of an Ideal Gas

The Electric Field Due to a Point Charge 564

19-8 DEGREES OF FREEDOM AND MOLAR SPECIFIC HEATS

The Adiabatic Expansion of an Ideal Gas

576

The Electric Field Due to Line of Charge

What Is Physics? 584 Irreversible Processes and Entropy

584

The Electric Field Due to a Charged Disk

PROBLEMS

577

583

REVIEW & SUMMARY

650

645

645

22-7 A DIPOLE IN AN ELECTRIC FIELD A Dipole in an Electric Field

643

643

22-6 A POINT CHARGE IN AN ELECTRIC FIELD A Point Charge in an Electric Field

638

638

22-5 THE ELECTRIC FIELD DUE TO A CHARGED DISK

571

20 Entropy and the Second Law of Thermodynamics 20-1 ENTROPY 583

636

22-4 THE ELECTRIC FIELD DUE TO A LINE OF CHARGE

571

QUESTIONS

633

The Electric Field Due to an Electric Dipole 568

568

19-9 THE ADIABATIC EXPANSION OF AN IDEAL GAS

633

22-3 THE ELECTRIC FIELD DUE TO A DIPOLE 635

564

Degrees of Freedom and Molar Specific Heats A Hint of Quantum Theory 570

QUESTIONS

22-2 THE ELECTRIC FIELD DUE TO A CHARGED PARTICLE

561

19-7 THE MOLAR SPECIFIC HEATS OF AN IDEAL GAS

622

What Is Physics? 630 The Electric Field 631 Electric Field Lines 631

558

575

624

621

REVIEW & SUMMARY

REVIEW & SUMMARY

PROBLEMS

621

554

The Distribution of Molecular Speeds

623

619

619

Pressure, Temperature, and RMS Speed

Mean Free Path

604

612

21-3 CHARGE IS CONSERVED

19-3 PRESSURE, TEMPERATURE, AND RMS SPEED

19-5 MEAN FREE PATH

PROBLEMS

609

21-2 CHARGE IS QUANTIZED

550

Translational Kinetic Energy

603

609

What Is Physics? 610 Electric Charge 610 Conductors and Insulators Coulomb’s Law 613

549

What Is Physics? 549 Avogadro’s Number 550

19-2 IDEAL GASES

20-4 A STATISTICAL VIEW OF ENTROPY

534

534

538

596

REVIEW & SUMMARY

18-6 HEAT TRANSFER MECHANISMS

595

Entropy in the Real World: Refrigerators The Efficiencies of Real Engines 597

A Statistical View of Entropy

590

590

20-3 REFRIGERATORS AND REAL ENGINES

A Closer Look at Heat and Work 528 The First Law of Thermodynamics 531 Some Special Cases of the First Law of Thermodynamics 532

Heat Transfer Mechanisms

588

647

648

QUESTIONS

651

PROBLEMS

652

CONTE NTS

23 Gauss’ Law 659 23-1 ELECTRIC FLUX 659

25 Capacitance 25-1 CAPACITANCE

What Is Physics 659 Electric Flux 660

What Is Physics? 717 Capacitance 717

23-2 GAUSS’ LAW

25-2 CALCULATING THE CAPACITANCE

664

Gauss’ Law 664 Gauss’ Law and Coulomb’s Law

666

25-4 ENERGY STORED IN AN ELECTRIC FIELD

Applying Gauss’ Law: Cylindrical Symmetry

Capacitor with a Dielectric Dielectrics: An Atomic View

673

673

QUESTIONS

PROBLEMS

677

679

What Is Physics? 685 Electric Potential and Electric Potential Energy

Equipotential Surfaces 690 Calculating the Potential from the Field

686

738

26-2 CURRENT DENSITY 690

QUESTIONS

738

PROBLEMS

739

Potential Due to a Charged Particle 694 Potential Due a Group of Charged Particles

26-4 OHM’S LAW

697

698

698

24-6 CALCULATING THE FIELD FROM THE POTENTIAL

Electric Potential Energy of a System of Charged Particles

24-8 POTENTIAL OF A CHARGED ISOLATED CONDUCTOR

703

706

706 708

758

PROBLEMS

710

760

760

763

27 Circuits 771 27-1 SINGLE-LOOP CIRCUITS

701

QUESTIONS

Power in Electric Circuits Semiconductors 762 Superconductors 763 REVIEW & SUMMARY

701

24-7 ELECTRIC POTENTIAL ENERGY OF A SYSTEM OF CHARGED PARTICLES 703

Potential of Charged Isolated Conductor

756

26-5 POWER, SEMICONDUCTORS, SUPERCONDUCTORS

24-5 POTENTIAL DUE TO A CONTINUOUS CHARGE DISTRIBUTION Potential Due to a Continuous Charge Distribution

752

753

Ohm’s Law 756 A Microscopic View of Ohm’s Law

697

Calculating the Field from the Potential

749

Resistance and Resistivity 694

695

24-4 POTENTIAL DUE TO AN ELECTRIC DIPOLE

Current Density

748

26-3 RESISTANCE AND RESISTIVITY

691

24-3 POTENTIAL DUE TO A CHARGED PARTICLE

707

735

What Is Physics? 745 Electric Current 746

24-2 EQUIPOTENTIAL SURFACES AND THE ELECTRIC FIELD

REVIEW & SUMMARY

REVIEW & SUMMARY

735

26 Current and Resistance 745 26-1 ELECTRIC CURRENT 745

24 Electric Potential 685 24-1 ELECTRIC POTENTIAL 685

Potential Due to an Electric Dipole

733

Dielectrics and Gauss’ Law

675

731

731

25-6 DIELECTRICS AND GAUSS’ LAW

675

728

728

25-5 CAPACITOR WITH A DIELECTRIC

23-6 APPLYING GAUSS’ LAW: SPHERICAL SYMMETRY 677

Energy Stored in an Electric Field

671

671

23-5 APPLYING GAUSS’ LAW: PLANAR SYMMETRY

Applying Gauss’ Law: Spherical Symmetry

723

Capacitors in Parallel and in Series 724

668

668

Applying Gauss’ Law: Planar Symmetry

719

720

25-3 CAPACITORS IN PARALLEL AND IN SERIES

23-4 APPLYING GAUSS’ LAW: CYLINDRICAL SYMMETRY

REVIEW & SUMMARY

717

Calculating the Capacitance

23-3 A CHARGED ISOLATED CONDUCTOR A Charged Isolated Conductor

717

QUESTIONS

764

771

What Is Physics? 772 “Pumping” Charges 772 Work, Energy, and Emf 773 Calculating the Current in a Single-Loop Circuit Other Single-Loop Circuits 776 Potential Difference Between Two Points 777

774

PROBLEMS

765

xi

xii

CONTE NTS

27-2 MULTILOOP CIRCUITS Multiloop Circuits

29-5 A CURRENT-CARRYING COIL AS A MAGNETIC DIPOLE

781

A Current-Carrying Coil as a Magnetic Dipole

781

27-3 THE AMMETER AND THE VOLTMETER The Ammeter and the Voltmeter

27-4 RC CIRCUITS RC Circuits

REVIEW & SUMMARY

788

788

789

28 Magnetic Fields

QUESTIONS

793

PROBLEMS

793

795

803 :

28-1 MAGNETIC FIELDS AND THE DEFINITION OF B 804

Induced Electric Fields

30-5 SELF-INDUCTION

810

Self-Induction

811

28-4 A CIRCULATING CHARGED PARTICLE

RL Circuits

28-6 MAGNETIC FORCE ON A CURRENT-CARRYING WIRE

REVIEW & SUMMARY

827

820

30-9 MUTUAL INDUCTION Mutual Induction

824

825 827

PROBLEMS

837

29-2 FORCE BETWEEN TWO PARALLEL CURRENTS Force Between Two Parallel Currents

29-3 AMPERE’S LAW Ampere’s Law

842

844

844

29-4 SOLENOIDS AND TOROIDS Solenoids and Toroids

848

848

890

893

QUESTIONS

893

PROBLEMS

31 Electromagnetic Oscillations and Alternating Current 31-1 LC OSCILLATIONS 903

895

903

What Is Physics? 904 LC Oscillations, Qualitatively 904 The Electrical-Mechanical Analogy 906 LC Oscillations, Quantitatively 907

29 Magnetic Fields Due to Currents 836 29-1 MAGNETIC FIELD DUE TO A CURRENT 836 What Is Physics? 836 Calculating the Magnetic Field Due to a Current

829

889

889

890

REVIEW & SUMMARY

QUESTIONS

887

887

Energy Density of a Magnetic Field

Torque on a Current Loop 822

The Magnetic Dipole Moment

883

30-8 ENERGY DENSITY OF A MAGNETIC FIELD

820

822

28-8 THE MAGNETIC DIPOLE MOMENT

882

Energy Stored in a Magnetic Field

818

28-7 TORQUE ON A CURRENT LOOP

881

30-7 ENERGY STORED IN A MAGNETIC FIELD

817

Magnetic Force on a Current-Carrying Wire

879

879

881

30-6 RL CIRCUITS

814

814

28-5 CYCLOTRONS AND SYNCHROTRONS Cyclotrons and Synchrotrons

Inductors and Inductance

809

28-3 CROSSED FIELDS: THE HALL EFFECT

874

875

30-4 INDUCTORS AND INDUCTANCE

808

871

871

30-3 INDUCED ELECTRIC FIELDS

Crossed Fields: Discovery of the Electron

A Circulating Charged Particle

856

865

30-2 INDUCTION AND ENERGY TRANSFERS

803

28-2 CROSSED FIELDS: DISCOVERY OF THE ELECTRON

Crossed Fields: The Hall Effect

What Is Physics 864 Two Experiments 865 Faraday’s Law of Induction Lenz’s Law 868

Induction and Energy Transfers

What Is Physics? 803 What Produces a Magnetic Field? : The Definition of B 804

PROBLEMS

855

30 Induction and Inductance 864 30-1 FARADAY’S LAW AND LENZ’S LAW 864

788

REVIEW & SUMMARY

QUESTIONS

854

851

851

31-2 DAMPED OSCILLATIONS IN AN RLC CIRCUIT Damped Oscillations in an RLC Circuit

842

910

911

31-3 FORCED OSCILLATIONS OF THREE SIMPLE CIRCUITS Alternating Current 913 Forced Oscillations 914 Three Simple Circuits 914

31-4 THE SERIES RLC CIRCUIT The Series RLC Circuit

921

921

912

xiii

CONTE NTS

31-5 POWER IN ALTERNATING-CURRENT CIRCUITS Power in Alternating-Current Circuits

31-6 TRANSFORMERS Transformers

Reflection and Refraction

930

QUESTIONS

933

934

PROBLEMS

What Is Physics? 941 Gauss’ Law for Magnetic Fields

946

34-2 SPHERICAL MIRRORS

950

34-4 THIN LENSES

953

Thin Lenses

961

961 964

QUESTIONS

965

PROBLEMS

33 Electromagnetic Waves 972 33-1 ELECTROMAGNETIC WAVES 972

33-2 ENERGY TRANSPORT AND THE POYNTING VECTOR 33-3 RADIATION PRESSURE 33-4 POLARIZATION Polarization

985

983

985

983

1033

QUESTIONS

1036

981

1037

What Is Physics? Light as a Wave

1038

1047 1048

Diffraction 1053 Young’s Interference Experiment

1053

1054

35-3 INTERFERENCE AND DOUBLE-SLIT INTENSITY 980

PROBLEMS

35 Interference 1047 35-1 LIGHT AS A WAVE 1047

35-2 YOUNG’S INTERFERENCE EXPERIMENT

What Is Physics? 972 Maxwell’s Rainbow 973 The Traveling Electromagnetic Wave, Qualitatively 974 The Traveling Electromagnetic Wave, Quantitatively 977

Energy Transport and the Poynting Vector

967

1030

1030

REVIEW & SUMMARY

959

Radiation Pressure

1023

34-6 THREE PROOFS

959

1020

1020

34-5 OPTICAL INSTRUMENTS

957

REVIEW & SUMMARY

1016

1023

Optical Instruments

32-8 FERROMAGNETISM Ferromagnetism

Spherical Refracting Surfaces 952

957

32-7 PARAMAGNETISM

1001

1010

34-3 SPHERICAL REFRACTING SURFACES

Magnetism and Electrons Magnetic Materials 956

PROBLEMS

1000

1014

Spherical Mirrors 1015 Images from Spherical Mirrors

949

32-5 MAGNETISM AND ELECTRONS

Paramagnetism

QUESTIONS

999

What Is Physics? 1010 Two Types of Image 1010 Plane Mirrors 1012

943

950

Diamagnetism

997

998

34 Images 1010 34-1 IMAGES AND PLANE MIRRORS

947

32-6 DIAMAGNETISM

33-7 POLARIZATION BY REFLECTION REVIEW & SUMMARY

943

32-3 DISPLACEMENT CURRENT

32-4 MAGNETS

941

942

32-2 INDUCED MAGNETIC FIELDS

Displacement Current Maxwell’s Equations

935

996

996

Polarization by Reflection

32 Maxwell’s Equations; Magnetism of Matter 32-1 GAUSS’ LAW FOR MAGNETIC FIELDS 941

Induced Magnetic Fields

990

991

33-6 TOTAL INTERNAL REFLECTION

930

Total Internal Reflection

REVIEW & SUMMARY

Magnets

33-5 REFLECTION AND REFRACTION

927

927

Coherence 1059 Intensity in Double-Slit Interference

1060

35-4 INTERFERENCE FROM THIN FILMS Interference from Thin Films

REVIEW & SUMMARY

1063

1064

35-5 MICHELSON’S INTERFEROMETER Michelson’s Interferometer

1059

1070

1071

1072

QUESTIONS

1072

PROBLEMS

1074

xiv

CONTE NTS

36 Diffraction 1081 36-1 SINGLE-SLIT DIFFRACTION

38 Photons and Matter Waves 1153 38-1 THE PHOTON, THE QUANTUM OF LIGHT

1081

1153

What Is Physics? 1081 Diffraction and the Wave Theory of Light 1081 Diffraction by a Single Slit: Locating the Minima 1083

What Is Physics? 1153 The Photon, the Quantum of Light

36-2 INTENSITY IN SINGLE-SLIT DIFFRACTION

1086

The Photoelectric Effect

Intensity in Single-Slit Diffraction 1086 Intensity in Single-Slit Diffraction, Quantitatively

1088

38-3 PHOTONS, MOMENTUM, COMPTON SCATTERING, LIGHT INTERFERENCE 1158

36-3 DIFFRACTION BY A CIRCULAR APERTURE

1090

Photons Have Momentum 1159 Light as a Probability Wave 1162

Diffraction by a Circular Aperture

Diffraction by a Double Slit

Electrons and Matter Waves

Gratings: Dispersion and Resolving Power

Schrödinger’s Equation

1101

1166

1167

38-6 SCHRÖDINGER’S EQUATION

1101

1164

1165

38-5 ELECTRONS AND MATTER WAVES

1098

36-6 GRATINGS: DISPERSION AND RESOLVING POWER

X-Ray Diffraction

1156

The Birth of Quantum Physics

1098

36-7 X-RAY DIFFRACTION

1155

38-4 THE BIRTH OF QUANTUM PHYSICS

1094

1095

36-5 DIFFRACTION GRATINGS Diffraction Gratings

38-2 THE PHOTOELECTRIC EFFECT

1091

36-4 DIFFRACTION BY A DOUBLE SLIT

1154

1170

1170

38-7 HEISENBERG’S UNCERTAINTY PRINCIPLE

1104

Heisenberg’s Uncertainty Principle

1172

1173

1104

REVIEW & SUMMARY

1107

QUESTIONS

37 Relativity 1116 37-1 SIMULTANEITY AND TIME DILATION

1107

PROBLEMS

1108

The Relativity of Length

1116

REVIEW & SUMMARY

1143

1181

An Electron in a Finite Well 1133

1191

1192

1195

1195

39-4 TWO- AND THREE-DIMENSIONAL ELECTRON TRAPS More Electron Traps 1197 Two- and Three-Dimensional Electron Traps

1134

39-5 THE HYDROGEN ATOM 1137

QUESTIONS

1187

39-3 AN ELECTRON IN A FINITE WELL

1131

1197

1200

1201

The Hydrogen Atom Is an Electron Trap 1202 The Bohr Model of Hydrogen, a Lucky Break 1203 Schrödinger’s Equation and the Hydrogen Atom 1205

A New Look at Momentum 1138 A New Look at Energy 1138 REVIEW & SUMMARY

PROBLEMS

1187

Wave Functions of a Trapped Electron

1135

37-6 MOMENTUM AND ENERGY

1180

39-2 WAVE FUNCTIONS OF A TRAPPED ELECTRON

1133

37-5 DOPPLER EFFECT FOR LIGHT Doppler Effect for Light

What Is Physics? 1186 String Waves and Matter Waves Energies of a Trapped Electron

1129

The Lorentz Transformation 1129 Some Consequences of the Lorentz Equations

The Relativity of Velocities

QUESTIONS

1179

1176

1176

39 More About Matter Waves 1186 39-1 ENERGIES OF A TRAPPED ELECTRON 1186

1126

37-4 THE RELATIVITY OF VELOCITIES

1174

Tunneling Through a Potential Barrier

1125

37-3 THE LORENTZ TRANSFORMATION

Reflection from a Potential Step

1174

38-9 TUNNELING THROUGH A POTENTIAL BARRIER

What Is Physics? 1116 The Postulates 1117 Measuring an Event 1118 The Relativity of Simultaneity 1120 The Relativity of Time 1121

37-2 THE RELATIVITY OF LENGTH

38-8 REFLECTION FROM A POTENTIAL STEP

1144

PROBLEMS

1145

REVIEW & SUMMARY

1213

QUESTIONS

1213

PROBLEMS

1214

xv

CONTE NTS

40 All About Atoms 1219 40-1 PROPERTIES OF ATOMS 1219

42-2 SOME NUCLEAR PROPERTIES

What Is Physics? 1220 Some Properties of Atoms 1220 Angular Momentum, Magnetic Dipole Moments

42-3 RADIOACTIVE DECAY

Some Nuclear Properties

40-2 THE STERN-GERLACH EXPERIMENT The Stern-Gerlach Experiment

1226

40-3 MAGNETIC RESONANCE

1229

Magnetic Resonance

Radioactive Decay

1222

Alpha Decay

1229

40-4 EXCLUSION PRINCIPLE AND MULTIPLE ELECTRONS IN A TRAP The Pauli Exclusion Principle 1230 Multiple Electrons in Rectangular Traps

1231

40-5 BUILDING THE PERIODIC TABLE

1234

Building the Periodic Table

Nuclear Models

1236

QUESTIONS

What Is Physics? 1252 The Electrical Properties of Solids Energy Levels in a Crystalline Solid Insulators 1254 Metals 1255

1246

PROBLEMS

1247

1297

1297

QUESTIONS

1300

PROBLEMS

1301

1302

What Is Physics? 1309 Nuclear Fission: The Basic Process 1310 A Model for Nuclear Fission 1312

The Nuclear Reactor

1316

1316

43-3 A NATURAL NUCLEAR REACTOR A Natural Nuclear Reactor

1254

1320

1320

43-4 THERMONUCLEAR FUSION: THE BASIC PROCESS Thermonuclear Fusion: The Basic Process 1261

Thermonuclear Fusion in the Sun and Other Stars

41-3 THE p-n JUNCTION AND THE TRANSISTOR

43-6 CONTROLLED THERMONUCLEAR FUSION

1265

Controlled Thermonuclear Fusion REVIEW & SUMMARY

1268

QUESTIONS

42 Nuclear Physics 1276 42-1 DISCOVERING THE NUCLEUS

1276

1322

1322

43-5 THERMONUCLEAR FUSION IN THE SUN AND OTHER STARS

Semiconductors 1262 Doped Semiconductors 1263

What Is Physics? 1276 Discovering the Nucleus 1276

1296

43-2 THE NUCLEAR REACTOR

1253

41-2 SEMICONDUCTORS AND DOPING

1271

1296

43 Energy from the Nucleus 1309 43-1 NUCLEAR FISSION 1309

41 Conduction of Electricity in Solids 1252 41-1 THE ELECTRICAL PROPERTIES OF METALS 1252

REVIEW & SUMMARY

1295

1295

REVIEW & SUMMARY

1237

Lasers and Laser Light 1241 How Lasers Work 1242

The p-n Junction 1266 The Junction Rectifier 1267 The Light-Emitting Diode (LED) The Transistor 1270

Radioactive Dating

42-8 NUCLEAR MODELS

1240

1245

42-6 RADIOACTIVE DATING

Measuring Radiation Dosage

1234

X Rays and the Ordering of the Elements

REVIEW & SUMMARY

1292

1292

42-7 MEASURING RADIATION DOSAGE

40-6 X RAYS AND THE ORDERING OF THE ELEMENTS 40-7 LASERS

1230

1289

1289

42-5 BETA DECAY Beta Decay

1286

1286

42-4 ALPHA DECAY

1226

1279

1280

1272

PROBLEMS

1272

1329

1324

1326

1326

QUESTIONS

1329

44 Quarks, Leptons, and the Big Bang 1334 44-1 GENERAL PROPERTIES OF ELEMENTARY PARTICLES What Is Physics? 1334 Particles, Particles, Particles An Interlude 1339

1335

44-2 LEPTONS, HADRONS, AND STRANGENESS The Leptons

1343

1324

1343

PROBLEMS

1334

1330

xvi

CONTE NTS

The Hadrons 1345 Still Another Conservation Law The Eightfold Way 1347

APPENDICES

44-3 QUARKS AND MESSENGER PARTICLES The Quark Model 1349 Basic Forces and Messenger Particles

44-4 COSMOLOGY

A The International System of Units (SI) A-1 B Some Fundamental Constants of Physics A-3 C Some Astronomical Data A-4 D Conversion Factors A-5 E Mathematical Formulas A-9 F Properties of The Elements A-12 G Periodic Table of The Elements A-15

1346

1349

1352

1355

A Pause for Reflection 1355 The Universe Is Expanding 1356 The Cosmic Background Radiation Dark Matter 1358 The Big Bang 1358 A Summing Up 1361 REVIEW & SUMMARY

1362

ANSWERS

to Checkpoints and Odd-Numbered Questions and Problems 1357

QUESTIONS

I N D E X I-1

1362

PROBLEMS

1363

AN-1

P

R

E

F

A

C

E

WHY I WROTE THIS BOOK Fun with a big challenge. That is how I have regarded physics since the day when Sharon, one of the students in a class I taught as a graduate student, suddenly demanded of me, “What has any of this got to do with my life?” Of course I immediately responded, “Sharon, this has everything to do with your life—this is physics.” She asked me for an example. I thought and thought but could not come up with a single one.That night I began writing the book The Flying Circus of Physics (John Wiley & Sons Inc., 1975) for Sharon but also for me because I realized her complaint was mine. I had spent six years slugging my way through many dozens of physics textbooks that were carefully written with the best of pedagogical plans, but there was something missing. Physics is the most interesting subject in the world because it is about how the world works, and yet the textbooks had been thoroughly wrung of any connection with the real world. The fun was missing. I have packed a lot of real-world physics into Fundamentals of Physics, connecting it with the new edition of The Flying Circus of Physics. Much of the material comes from the introductory physics classes I teach, where I can judge from the faces and blunt comments what material and presentations work and what do not. The notes I make on my successes and failures there help form the basis of this book. My message here is the same as I had with every student I’ve met since Sharon so long ago: “Yes, you can reason from basic physics concepts all the way to valid conclusions about the real world, and that understanding of the real world is where the fun is.” I have many goals in writing this book but the overriding one is to provide instructors with tools by which they can teach students how to effectively read scientific material, identify fundamental concepts, reason through scientific questions, and solve quantitative problems. This process is not easy for either students or instructors. Indeed, the course associated with this book may be one of the most challenging of all the courses taken by a student. However, it can also be one of the most rewarding because it reveals the world’s fundamental clockwork from which all scientific and engineering applications spring. Many users of the ninth edition (both instructors and students) sent in comments and suggestions to improve the book. These improvements are now incorporated into the narrative and problems throughout the book. The publisher John Wiley & Sons and I regard the book as an ongoing project and encourage more input from users. You can send suggestions, corrections, and positive or negative comments to John Wiley & Sons or Jearl Walker (mail address: Physics Department, Cleveland State University, Cleveland, OH 44115 USA; or the blog site at www.flyingcircusofphysics.com). We may not be able to respond to all suggestions, but we keep and study each of them.

WHAT’S NEW? Modules and Learning Objectives “What was I supposed to learn from this section?” Students have

asked me this question for decades, from the weakest student to the strongest. The problem is that even a thoughtful student may not feel confident that the important points were captured while reading a section. I felt the same way back when I was using the first edition of Halliday and Resnick while taking first-year physics. To ease the problem in this edition, I restructured the chapters into concept modules based on a primary theme and begin each module with a list of the module’s learning objectives. The list is an explicit statement of the skills and learning points that should be gathered in reading the module. Each list is following by a brief summary of the key ideas that should also be gathered. For example, check out the first module in Chapter 16, where a student faces a truck load of concepts and terms. Rather than depending on the student’s ability to gather and sort those ideas, I now provide an explicit checklist that functions somewhat like the checklist a pilot works through before taxiing out to the runway for takeoff. xvii

xviii

PR E FACE

Links Between Homework Problems and Learning Objectives In WileyPLUS, every question and prob-

lem at the end of the chapter is linked to a learning objective, to answer the (usually unspoken) questions, “Why am I working this problem? What am I supposed to learn from it?” By being explicit about a problem’s purpose, I believe that a student might better transfer the learning objective to other problems with a different wording but the same key idea. Such transference would help defeat the common trouble that a student learns to work a particular problem but cannot then apply its key idea to a problem in a different setting.

Rewritten Chapters My students have continued to be challenged by several key chapters and by

spots in several other chapters and so, in this edition, I rewrote a lot of the material. For example, I redesigned the chapters on Gauss’ law and electric potential, which have proved to be tough-going for my students. The presentations are now smoother and more direct to the key points. In the quantum chapters, I expanded the coverage of the Schrödinger equation, including reflection of matter waves from a step potential. At the request of several instructors, I decoupled the discussion of the Bohr atom from the Schrödinger solution for the hydrogen atom so that the historical account of Bohr’s work can be bypassed. Also, there is now a module on Planck’s blackbody radiation.

New Sample Problems and Homework Questions and Problems Sixteen new sample problems have been added to the chapters, written so as to spotlight some of the difficult areas for my students. Also, about 250 problems and 50 questions have been added to the homework sections of the chapters. Some of these problems come from earlier editions of the book, as requested by several instructors.

Video Illustrations In the eVersion of the text available in

WileyPLUS, David Maiullo of Rutgers University has created video versions of approximately 30 of the photographs and figures from the text. Much of physics is the study of things that move and video can often provide a better representation than a static photo or figure.

Online Aid WileyPLUS is not just an online grading pro-

gram. Rather, it is a dynamic learning center stocked with many different learning aids, including just-in-time problem-solving tutorials, embedded reading quizzes to encourage reading, animated figures, hundreds of sample problems, loads of simulations and demonstrations, and over 1500 videos ranging from math reviews to mini-lectures to examples. More of these learning aids are added every semester. For this 10th edition of HRW, some of the photos involving motion have been converted into videos so that the motion can be slowed and analyzed. These thousands of learning aids are available 24/7 and can be repeated as many times as desired. Thus, if a student gets stuck on a homework problem at, say, 2:00 AM (which appears to be a popular time for doing physics homework), friendly and helpful resources are available at the click of a mouse.

LEARNINGS TOOLS

A

When I learned first-year physics in the first edition of Halliday and Resnick, I caught on by repeatedly rereading a chapter. These days we better understand that students have a wide range of learning styles. So, I have produced a wide range of learning tools, both in this new edition and online in WileyPLUS:

Animations of one of the key figures in each chapter.

Here in the book, those figures are flagged with the swirling icon. In the online chapter in WileyPLUS, a mouse click begins the animation. I have chosen the figures that are rich in information so that a student can see the physics in action and played out over a minute or two

PR E FACE

instead of just being flat on a printed page. Not only does this give life to the physics, but the animation can be repeated as many times as a student wants.

Videos I have made well over 1500 instructional videos, with more coming each semester. Students can watch me draw or type on the screen as they hear me talk about a solution, tutorial, sample problem, or review, very much as they would experience were they sitting next to me in my office while I worked out something on a notepad. An instructor’s lectures and tutoring will always be the most valuable learning tools, but my videos are available 24 hours a day, 7 days a week, and can be repeated indefinitely. • Video tutorials on subjects in the chapters. I chose the subjects that challenge the students the most, the ones that my students scratch their heads about. • Video reviews of high school math, such as basic algebraic manipulations, trig functions, and simultaneous equations. • Video introductions to math, such as vector multiplication, that will be new to the students. • Video presentations of every Sample Problem in the textbook chapters . My intent is to work out the physics, starting with the Key Ideas instead of just grabbing a formula. However, I also want to demonstrate how to read a sample problem, that is, how to read technical material to learn problem-solving procedures that can be transferred to other types of problems. • Video solutions to 20% of the end-of chapter problems. The availability and timing of these solutions are controlled by the instructor. For example, they might be available after a homework deadline or a quiz. Each solution is not simply a plug-and-chug recipe. Rather I build a solution from the Key Ideas to the first step of reasoning and to a final solution. The student learns not just how to solve a particular problem but how to tackle any problem, even those that require physics courage. • Video examples of how to read data from graphs (more than simply reading off a number with no comprehension of the physics).

Problem-Solving Help I have written a large number of resources for WileyPLUS designed to help build the students’ problem-solving skills.

• Every sample problem in the textbook is available online in both reading and video formats. • Hundreds of additional sample problems. These are available as standalone resources but (at the discretion of the instructor) they are also linked out of the homework problems. So, if a homework problem deals with, say, forces on a block on a ramp, a link to a related sample problem is provided. However, the sample problem is not just a replica of the homework problem and thus does not provide a solution that can be merely duplicated without comprehension. • GO Tutorials for 15% of the end-of-chapter homework problems. In multiple steps, I lead a student through a homework problem, starting with the Key Ideas and giving hints when wrong answers are submitted. However, I purposely leave the last step (for the final answer) to the student so that they are responsible at the end. Some online tutorial systems trap a student when wrong answers are given, which can generate a lot of frustration. My GO Tutorials are not traps, because at any step along the way, a student can return to the main problem. • Hints on every end-of-chapter homework problem are available (at the discretion of the instructor). I wrote these as true hints about the main ideas and the general procedure for a solution, not as recipes that provide an answer without any comprehension.

xix

xx

PR E FACE

Evaluation Materials • Reading questions are available within each online section. I wrote these so that they do not require analysis or any deep understanding; rather they simply test whether a student has read the section. When a student opens up a section, a randomly chosen reading question (from a bank of questions) appears at the end. The instructor can decide whether the question is part of the grading for that section or whether it is just for the benefit of the student. • Checkpoints are available within most sections. I wrote these so that they require analysis and decisions about the physics in the section. Answers to all checkpoints are in the back of the book.

Checkpoint 1 Here are three pairs of initial and final positions, respectively, along an x axis. Which pairs give a negative displacement: (a) $3 m, #5 m; (b) $3 m, $7 m; (c) 7 m, $3 m?

• All end-of-chapter homework Problems in the book (and many more problems) are available in WileyPLUS. The instructor can construct a homework assignment and control how it is graded when the answers are submitted online. For example, the instructor controls the deadline for submission and how many attempts a student is allowed on an answer. The instructor also controls which, if any, learning aids are available with each homework problem. Such links can include hints, sample problems, in-chapter reading materials, video tutorials, video math reviews, and even video solutions (which can be made available to the students after, say, a homework deadline). • Symbolic notation problems that require algebraic answers are available in every chapter. • All end-of-chapter homework Questions in the book are available for assignment in WileyPLUS. These Questions (in a multiple choice format) are designed to evaluate the students’ conceptual understanding.

Icons for Additional Help When worked-out solutions are provided either in print or electronically for certain of the odd-numbered problems, the statements for those problems include an icon to alert both student and instructor as to where the solutions are located. There are also icons indicating which problems have GO Tutorial, an Interactive LearningWare, or a link to the The Flying Circus of Physics. An icon guide is provided here and at the beginning of each set of problems.

Tutoring problem available (at instructor’s discretion) in WileyPLUS and WebAssign SSM • – •••

Worked-out solution available in Student Solutions Manual Number of dots indicates level of problem difficulty

WWW Worked-out solution is at ILW

Interactive solution is at

http://www.wiley.com/college/halliday

Additional information available in The Flying Circus of Physics and at flyingcircusofphysics.com

VERSIONS OF THE TEXT To accommodate the individual needs of instructors and students, the ninth edition of Fundamentals of Physics is available in a number of different versions. The Regular Edition consists of Chapters 1 through 37 (ISBN 9781118230718). The Extended Edition contains seven additional chapters on quantum physics and cosmology, Chapters 1–44 (ISBN 9781118230725). Volume 1 –– Chapters 1–20 (Mechanics and Thermodynamics), hardcover, ISBN 9781118233764 Volume 2 –– Chapters 21–44 (E&M, Optics, and Quantum Physics), hardcover, ISBN 9781118230732

PR E FACE

INSTRUCTOR SUPPLEMENTS Instructor’s Solutions Manual by Sen-Ben Liao, Lawrence Livermore National Laboratory. This manual provides worked-out solutions for all problems found at the end of each chapter. It is available in both MSWord and PDF.

Instructor Companion Site http://www.wiley.com/college/halliday • Instructor’s Manual This resource contains lecture notes outlining the most important topics of each chapter; demonstration experiments; laboratory and computer projects; film and video sources; answers to all Questions, Exercises, Problems, and Checkpoints; and a correlation guide to the Questions, Exercises, and Problems in the previous edition. It also contains a complete list of all problems for which solutions are available to students (SSM,WWW, and ILW). • Lecture PowerPoint Slides These PowerPoint slides serve as a helpful starter pack for instructors, outlining key concepts and incorporating figures and equations from the text. • Classroom Response Systems (“Clicker”) Questions by David Marx, Illinois State University. There are two sets of questions available: Reading Quiz questions and Interactive Lecture questions.The Reading Quiz questions are intended to be relatively straightforward for any student who reads the assigned material.The Interactive Lecture questions are intended for use in an interactive lecture setting. • Wiley Physics Simulations by Andrew Duffy, Boston University and John Gastineau, Vernier Software. This is a collection of 50 interactive simulations (Java applets) that can be used for classroom demonstrations. • Wiley Physics Demonstrations by David Maiullo, Rutgers University. This is a collection of digital videos of 80 standard physics demonstrations. They can be shown in class or accessed from WileyPLUS. There is an accompanying Instructor’s Guide that includes “clicker” questions. • Test Bank For the 10th edition, the Test Bank has been completely over-hauled by Suzanne Willis, Northern Illinois University. The Test Bank includes more than 2200 multiple-choice questions. These items are also available in the Computerized Test Bank which provides full editing features to help you customize tests (available in both IBM and Macintosh versions). • All text illustrations suitable for both classroom projection and printing.

Online Homework and Quizzing. In addition to WileyPLUS, Fundamentals of Physics, tenth edition,

also supports WebAssignPLUS and LON-CAPA, which are other programs that give instructors the ability to deliver and grade homework and quizzes online. WebAssign PLUS also offers students an online version of the text.

STUDENT SUPPLEMENTS Student Companion Site. The web site http://www.wiley.com/college/halliday was developed specifically for Fundamentals of Physics, tenth edition, and is designed to further assist students in the study of physics. It includes solutions to selected end-of-chapter problems (which are identified with a www icon in the text); simulation exercises; tips on how to make best use of a programmable calculator; and the Interactive LearningWare tutorials that are described below.

Student Study Guide (ISBN 9781118230787) by Thomas Barrett of Ohio State University. The Student Study Guide consists of an overview of the chapter’s important concepts, problem solving techniques and detailed examples.

Student Solutions Manual (ISBN 9781118230664) by Sen-Ben Liao, Lawrence Livermore National

Laboratory. This manual provides students with complete worked-out solutions to 15 percent of the problems found at the end of each chapter within the text. The Student Solutions Manual for the 10th edition is written using an innovative approach called TEAL which stands for Think, Express, Analyze, and Learn. This learning strategy was originally developed at the Massachusetts Institute of Technology and has proven to be an effective learning tool for students. These problems with TEAL solutions are indicated with an SSM icon in the text.

xxi

xxii

PR E FACE

Interactive Learningware. This software guides students through solutions to 200 of the end-of-chapter problems. These problems are indicated with an ILW icon in the text. The solutions process is developed interactively, with appropriate feedback and access to error-specific help for the most common mistakes.

Introductory Physics with Calculus as a Second Language: (ISBN 9780471739104) Mastering

Problem Solving by Thomas Barrett of Ohio State University. This brief paperback teaches the student how to approach problems more efficiently and effectively. The student will learn how to recognize common patterns in physics problems, break problems down into manageable steps, and apply appropriate techniques. The book takes the student step by step through the solutions to numerous examples.

A C K N O W L E D G M E N T S

A great many people have contributed to this book. Sen-Ben Liao of Lawrence Livermore National Laboratory, James Whitenton of Southern Polytechnic State University, and Jerry Shi, of Pasadena City College, performed the Herculean task of working out solutions for every one of the homework problems in the book. At John Wiley publishers, the book received support from Stuart Johnson, Geraldine Osnato and Aly Rentrop, the editors who oversaw the entire project from start to finish. We thank Elizabeth Swain, the production editor, for pulling all the pieces together during the complex production process. We also thank Maddy Lesure for her design of the text and the cover; Lee Goldstein for her page make-up; Helen Walden for her copyediting; and Lilian Brady for her proofreading. Jennifer Atkins was inspired in the search for unusual and interesting photographs. Both the publisher John Wiley & Sons, Inc. and Jearl Walker would like to thank the following for comments and ideas about the recent editions: Jonathan Abramson, Portland State University; Omar Adawi, Parkland College; Edward Adelson, The Ohio State University; Steven R. Baker, Naval Postgraduate School; George Caplan, Wellesley College; Richard Kass, The Ohio State University; M. R. Khoshbin-e-Khoshnazar, Research Institution for Curriculum Development & Educational Innovations (Tehran); Craig Kletzing, University of Iowa, Stuart Loucks, American River College; Laurence Lurio, Northern Illinois University; Ponn Maheswaranathan, Winthrop University; Joe McCullough, Cabrillo College; Carl E. Mungan, U. S. Naval Academy, Don N. Page, University of Alberta; Elie Riachi, Fort Scott Community College; Andrew G. Rinzler, University of Florida; Dubravka Rupnik, Louisiana State University; Robert Schabinger, Rutgers University; Ruth Schwartz, Milwaukee School of Engineering; Carol Strong, University of Alabama at Huntsville, Nora Thornber, Raritan Valley Community College; Frank Wang, LaGuardia Community College; Graham W. Wilson, University of Kansas; Roland Winkler, Northern Illinois University; William Zacharias, Cleveland State University; Ulrich Zurcher, Cleveland State University. Finally, our external reviewers have been outstanding and we acknowledge here our debt to each member of that team. Maris A. Abolins, Michigan State University Edward Adelson, Ohio State University Nural Akchurin, Texas Tech Yildirim Aktas, University of North Carolina-Charlotte Barbara Andereck, Ohio Wesleyan University Tetyana Antimirova, Ryerson University Mark Arnett, Kirkwood Community College Arun Bansil, Northeastern University Richard Barber, Santa Clara University Neil Basecu, Westchester Community College Anand Batra, Howard University Kenneth Bolland, The Ohio State University Richard Bone, Florida International University Michael E. Browne, University of Idaho Timothy J. Burns, Leeward Community College Joseph Buschi, Manhattan College Philip A. Casabella, Rensselaer Polytechnic Institute Randall Caton, Christopher Newport College

N. John DiNardo, Drexel University Eugene Dunnam, University of Florida Robert Endorf, University of Cincinnati F. Paul Esposito, University of Cincinnati Jerry Finkelstein, San Jose State University Robert H. Good, California State University-Hayward Michael Gorman, University of Houston Benjamin Grinstein, University of California, San Diego John B. Gruber, San Jose State University Ann Hanks, American River College Randy Harris, University of California-Davis Samuel Harris, Purdue University Harold B. Hart, Western Illinois University Rebecca Hartzler, Seattle Central Community College John Hubisz, North Carolina StateUniversity Joey Huston, Michigan State University David Ingram, Ohio University

Roger Clapp, University of South Florida

Shawn Jackson, University of Tulsa Hector Jimenez, University of Puerto Rico

W. R. Conkie, Queen’s University Renate Crawford, University of Massachusetts-Dartmouth

Sudhakar B. Joshi, York University Leonard M. Kahn, University of Rhode Island

Mike Crivello, San Diego State University Robert N. Davie, Jr., St. Petersburg Junior College Cheryl K. Dellai, Glendale Community College

Sudipa Kirtley, Rose-Hulman Institute

Eric R. Dietz, California State University at Chico

Peter F. Koehler, University of Pittsburgh

Leonard Kleinman, University of Texas at Austin Craig Kletzing, University of Iowa

xxiii

xxiv

ACKNOWLE DG M E NTS

Arthur Z. Kovacs, Rochester Institute of Technology Kenneth Krane, Oregon State University Hadley Lawler, Vanderbilt University Priscilla Laws, Dickinson College Edbertho Leal, Polytechnic University of Puerto Rico Vern Lindberg, Rochester Institute of Technology Peter Loly, University of Manitoba James MacLaren, Tulane University Andreas Mandelis, University of Toronto Robert R. Marchini, Memphis State University Andrea Markelz, University at Buffalo, SUNY Paul Marquard, Caspar College David Marx, Illinois State University Dan Mazilu, Washington and LeeUniversity James H. McGuire, Tulane University David M. McKinstry, Eastern Washington University Jordon Morelli, Queen’s University

Eugene Mosca, United States Naval Academy Eric R. Murray, Georgia Institute of Technology, School of Physics James Napolitano, Rensselaer Polytechnic Institute Blaine Norum, University of Virginia Michael O’Shea, Kansas State University Patrick Papin, San Diego State University Kiumars Parvin, San Jose State University Robert Pelcovits, Brown University Oren P. Quist, South Dakota State University Joe Redish, University of Maryland Timothy M. Ritter, University of North Carolina at Pembroke Dan Styer, Oberlin College Frank Wang, LaGuardia Community College Robert Webb, Texas A&M University Suzanne Willis, Northern Illinois University Shannon Willoughby, Montana State University

C

H

A

P

T

E

R

1

Measurement 1-1 MEASURING THINGS, INCLUDING LENGTHS Learning Objectives After reading this module, you should be able to . . .

1.01 Identify the base quantities in the SI system. 1.02 Name the most frequently used prefixes for SI units.

1.03 Change units (here for length, area, and volume) by using chain-link conversions. 1.04 Explain that the meter is defined in terms of the speed of light in vacuum.

Key Ideas ● Physics is based on measurement of physical quantities.

Certain physical quantities have been chosen as base quantities (such as length, time, and mass); each has been defined in terms of a standard and given a unit of measure (such as meter, second, and kilogram). Other physical quantities are defined in terms of the base quantities and their standards and units. ● The unit system emphasized in this book is the International

System of Units (SI). The three physical quantities displayed in Table 1-1 are used in the early chapters. Standards, which must be both accessible and invariable, have been established for these base quantities by international agreement.

These standards are used in all physical measurement, for both the base quantities and the quantities derived from them. Scientific notation and the prefixes of Table 1-2 are used to simplify measurement notation. ● Conversion of units may be performed by using chain-link

conversions in which the original data are multiplied successively by conversion factors written as unity and the units are manipulated like algebraic quantities until only the desired units remain. ● The meter is defined as the distance traveled by light

during a precisely specified time interval.

What Is Physics? Science and engineering are based on measurements and comparisons. Thus, we need rules about how things are measured and compared, and we need experiments to establish the units for those measurements and comparisons. One purpose of physics (and engineering) is to design and conduct those experiments. For example, physicists strive to develop clocks of extreme accuracy so that any time or time interval can be precisely determined and compared. You may wonder whether such accuracy is actually needed or worth the effort. Here is one example of the worth: Without clocks of extreme accuracy, the Global Positioning System (GPS) that is now vital to worldwide navigation would be useless.

Measuring Things We discover physics by learning how to measure the quantities involved in physics. Among these quantities are length, time, mass, temperature, pressure, and electric current. We measure each physical quantity in its own units, by comparison with a standard. The unit is a unique name we assign to measures of that quantity—for example, meter (m) for the quantity length. The standard corresponds to exactly 1.0 unit of the quantity. As you will see, the standard for length, which corresponds 1

2

CHAPTE R 1 M EASU R E M E NT

to exactly 1.0 m, is the distance traveled by light in a vacuum during a certain fraction of a second. We can define a unit and its standard in any way we care to. However, the important thing is to do so in such a way that scientists around the world will agree that our definitions are both sensible and practical. Once we have set up a standard—say, for length—we must work out procedures by which any length whatever, be it the radius of a hydrogen atom, the wheelbase of a skateboard, or the distance to a star, can be expressed in terms of the standard. Rulers, which approximate our length standard, give us one such procedure for measuring length. However, many of our comparisons must be indirect. You cannot use a ruler, for example, to measure the radius of an atom or the distance to a star. Base Quantities. There are so many physical quantities that it is a problem to organize them. Fortunately, they are not all independent; for example, speed is the ratio of a length to a time. Thus, what we do is pick out—by international agreement—a small number of physical quantities, such as length and time, and assign standards to them alone. We then define all other physical quantities in terms of these base quantities and their standards (called base standards). Speed, for example, is defined in terms of the base quantities length and time and their base standards. Base standards must be both accessible and invariable. If we define the length standard as the distance between one’s nose and the index finger on an outstretched arm, we certainly have an accessible standard—but it will, of course, vary from person to person. The demand for precision in science and engineering pushes us to aim first for invariability. We then exert great effort to make duplicates of the base standards that are accessible to those who need them.

Table 1-1 Units for Three SI Base Quantities Quantity

Unit Name

Length Time Mass

meter second kilogram

Unit Symbol m s kg

Table 1-2 Prefixes for SI Units Factor

Prefixa

1024 1021 1018 1015 1012 109 106 103 102 101 10$1 10!2 10!3 10!6 10!9 10!12 10$15 10$18 10$21 10$24

yottazettaexapetateragigamegakilohectodekadecicentimillimicronanopicofemtoattozeptoyocto-

Symbol Y Z E P T G M k h da d c m m n p f a z y

a The most frequently used prefixes are shown in bold type.

The International System of Units In 1971, the 14th General Conference on Weights and Measures picked seven quantities as base quantities, thereby forming the basis of the International System of Units, abbreviated SI from its French name and popularly known as the metric system. Table 1-1 shows the units for the three base quantities—length, mass, and time—that we use in the early chapters of this book. These units were defined to be on a “human scale.” Many SI derived units are defined in terms of these base units. For example, the SI unit for power, called the watt (W), is defined in terms of the base units for mass, length, and time. Thus, as you will see in Chapter 7, 1 watt " 1 W " 1 kg ! m2/s3,

(1-1)

where the last collection of unit symbols is read as kilogram-meter squared per second cubed. To express the very large and very small quantities we often run into in physics, we use scientific notation, which employs powers of 10. In this notation,

and

3 560 000 000 m " 3.56 ' 109 m

(1-2)

0.000 000 492 s " 4.92 ' 10$7 s.

(1-3)

Scientific notation on computers sometimes takes on an even briefer look, as in 3.56 E9 and 4.92 E–7, where E stands for “exponent of ten.” It is briefer still on some calculators, where E is replaced with an empty space. As a further convenience when dealing with very large or very small measurements, we use the prefixes listed in Table 1-2. As you can see, each prefix represents a certain power of 10, to be used as a multiplication factor. Attaching a prefix to an SI unit has the effect of multiplying by the associated factor. Thus, we can express a particular electric power as 1.27 ' 109 watts " 1.27 gigawatts " 1.27 GW

(1-4)

1-1 M EASU R I NG TH I NGS, I NCLU DI NG LE NGTHS

or a particular time interval as 2.35 ' 10$9 s " 2.35 nanoseconds " 2.35 ns.

(1-5)

Some prefixes, as used in milliliter, centimeter, kilogram, and megabyte, are probably familiar to you.

Changing Units We often need to change the units in which a physical quantity is expressed. We do so by a method called chain-link conversion. In this method, we multiply the original measurement by a conversion factor (a ratio of units that is equal to unity). For example, because 1 min and 60 s are identical time intervals, we have 1 min " 1 and 60 s

60 s " 1. 1 min

Thus, the ratios (1 min)/(60 s) and (60 s)/(1 min) can be used as conversion factors. This is not the same as writing 601 " 1 or 60 " 1; each number and its unit must be treated together. Because multiplying any quantity by unity leaves the quantity unchanged, we can introduce conversion factors wherever we find them useful. In chain-link conversion, we use the factors to cancel unwanted units. For example, to convert 2 min to seconds, we have

# 160mins $ " 120 s.

2 min " (2 min)(1) " (2 min)

(1-6)

If you introduce a conversion factor in such a way that unwanted units do not cancel, invert the factor and try again. In conversions, the units obey the same algebraic rules as variables and numbers. Appendix D gives conversion factors between SI and other systems of units, including non-SI units still used in the United States. However, the conversion factors are written in the style of “1 min " 60 s” rather than as a ratio. So, you need to decide on the numerator and denominator in any needed ratio.

Length In 1792, the newborn Republic of France established a new system of weights and measures. Its cornerstone was the meter, defined to be one ten-millionth of the distance from the north pole to the equator. Later, for practical reasons, this Earth standard was abandoned and the meter came to be defined as the distance between two fine lines engraved near the ends of a platinum–iridium bar, the standard meter bar, which was kept at the International Bureau of Weights and Measures near Paris. Accurate copies of the bar were sent to standardizing laboratories throughout the world. These secondary standards were used to produce other, still more accessible standards, so that ultimately every measuring device derived its authority from the standard meter bar through a complicated chain of comparisons. Eventually, a standard more precise than the distance between two fine scratches on a metal bar was required. In 1960, a new standard for the meter, based on the wavelength of light, was adopted. Specifically, the standard for the meter was redefined to be 1 650 763.73 wavelengths of a particular orange-red light emitted by atoms of krypton-86 (a particular isotope, or type, of krypton) in a gas discharge tube that can be set up anywhere in the world. This awkward number of wavelengths was chosen so that the new standard would be close to the old meter-bar standard.

3

4

CHAPTE R 1 M EASU R E M E NT

By 1983, however, the demand for higher precision had reached such a point that even the krypton-86 standard could not meet it, and in that year a bold step was taken. The meter was redefined as the distance traveled by light in a specified time interval. In the words of the 17th General Conference on Weights and Measures: The meter is the length of the path traveled by light in a vacuum during a time interval of 1/299 792 458 of a second.

This time interval was chosen so that the speed of light c is exactly c " 299 792 458 m/s.

Table 1-3 Some Approximate Lengths Measurement Distance to the first galaxies formed Distance to the Andromeda galaxy Distance to the nearby star Proxima Centauri Distance to Pluto Radius of Earth Height of Mt. Everest Thickness of this page Length of a typical virus Radius of a hydrogen atom Radius of a proton

Length in Meters 2 ' 1026 2 ' 1022 4 ' 1016 6 ' 1012 6 ' 106 9 ' 103 1 ' 10$4 1 ' 10$8 5 ' 10$11 1 ' 10$15

Measurements of the speed of light had become extremely precise, so it made sense to adopt the speed of light as a defined quantity and to use it to redefine the meter. Table 1-3 shows a wide range of lengths, from that of the universe (top line) to those of some very small objects.

Significant Figures and Decimal Places Suppose that you work out a problem in which each value consists of two digits. Those digits are called significant figures and they set the number of digits that you can use in reporting your final answer. With data given in two significant figures, your final answer should have only two significant figures. However, depending on the mode setting of your calculator, many more digits might be displayed. Those extra digits are meaningless. In this book, final results of calculations are often rounded to match the least number of significant figures in the given data. (However, sometimes an extra significant figure is kept.) When the leftmost of the digits to be discarded is 5 or more, the last remaining digit is rounded up; otherwise it is retained as is. For example, 11.3516 is rounded to three significant figures as 11.4 and 11.3279 is rounded to three significant figures as 11.3. (The answers to sample problems in this book are usually presented with the symbol " instead of % even if rounding is involved.) When a number such as 3.15 or 3.15 ' 103 is provided in a problem, the number of significant figures is apparent, but how about the number 3000? Is it known to only one significant figure (3 ' 103)? Or is it known to as many as four significant figures (3.000 ' 103)? In this book, we assume that all the zeros in such given numbers as 3000 are significant, but you had better not make that assumption elsewhere. Don’t confuse significant figures with decimal places. Consider the lengths 35.6 mm, 3.56 m, and 0.00356 m. They all have three significant figures but they have one, two, and five decimal places, respectively.

Sample Problem 1.01 Estimating order of magnitude, ball of string The world’s largest ball of string is about 2 m in radius. To the nearest order of magnitude, what is the total length L of the string in the ball?

ball’s builder most unhappy. Instead, because we want only the nearest order of magnitude, we can estimate any quantities required in the calculation.

KEY IDEA

Calculations: Let us assume the ball is spherical with radius R " 2 m. The string in the ball is not closely packed (there are uncountable gaps between adjacent sections of string). To allow for these gaps, let us somewhat overestimate

We could, of course, take the ball apart and measure the total length L, but that would take great effort and make the

5

1-2 TI M E

the cross-sectional area of the string by assuming the cross section is square, with an edge length d " 4 mm. Then, with a cross-sectional area of d2 and a length L, the string occupies a total volume of

d 2L " 4R3, or

L"

4R3 4(2 m)3 " d2 (4 ' 10$3 m)2 " 2 ' 10 6 m & 106 m " 103 km.

V " (cross-sectional area)(length) " d2L. This is approximately equal to the volume of the ball, given by 43)R3, which is about 4R3 because p is about 3. Thus, we have the following

(Answer)

(Note that you do not need a calculator for such a simplified calculation.) To the nearest order of magnitude, the ball contains about 1000 km of string!

Additional examples, video, and practice available at WileyPLUS

1-2

TIME

Learning Objectives After reading this module, you should be able to . . .

1.05 Change units for time by using chain-link conversions.

1.06 Use various measures of time, such as for motion or as determined on different clocks.

Key Idea ● The second is defined in terms of the oscillations of light emitted by an atomic (cesium-133) source. Accurate time

signals are sent worldwide by radio signals keyed to atomic clocks in standardizing laboratories.

Time Time has two aspects. For civil and some scientific purposes, we want to know the time of day so that we can order events in sequence. In much scientific work, we want to know how long an event lasts. Thus, any time standard must be able to answer two questions: “When did it happen?” and “What is its duration?” Table 1-4 shows some time intervals. Any phenomenon that repeats itself is a possible time standard. Earth’s rotation, which determines the length of the day, has been used in this way for centuries; Fig. 1-1 shows one novel example of a watch based on that rotation. A quartz clock, in which a quartz ring is made to vibrate continuously, can be calibrated against Earth’s rotation via astronomical observations and used to measure time intervals in the laboratory. However, the calibration cannot be carried out with the accuracy called for by modern scientific and engineering technology. Table 1-4 Some Approximate Time Intervals Measurement

Time Interval in Seconds

Lifetime of the proton (predicted) Age of the universe Age of the pyramid of Cheops Human life expectancy Length of a day a

3 ' 1040 5 ' 1017 1 ' 1011 2 ' 109 9 ' 104

Measurement

Time Interval in Seconds

Time between human heartbeats Lifetime of the muon Shortest lab light pulse Lifetime of the most unstable particle The Planck timea

8 ' 10 2 ' 10$6 1 ' 10$16 $1

1 ' 10$23 1 ' 10$43

This is the earliest time after the big bang at which the laws of physics as we know them can be applied.

Steven Pitkin

Figure 1-1 When the metric system was proposed in 1792, the hour was redefined to provide a 10-hour day. The idea did not catch on. The maker of this 10-hour watch wisely provided a small dial that kept conventional 12-hour time. Do the two dials indicate the same time?

6

CHAPTE R 1 M EASU R E M E NT

Difference between length of day and exactly 24 hours (ms)

+4

+3

+2

+1

1980

1981

1982

Figure 1-2 Variations in the length of the day over a 4-year period. Note that the entire vertical scale amounts to only 3 ms (" 0.003 s).

To meet the need for a better time standard, atomic clocks have been developed. An atomic clock at the National Institute of Standards and Technology (NIST) in Boulder, Colorado, is the standard for Coordinated Universal Time (UTC) in the United States. Its time signals are available by shortwave radio (stations WWV and WWVH) and by telephone (303-499-7111). Time signals (and related information) are also available from the United States Naval Observatory at website http://tycho.usno.navy.mil/time.html. (To set a clock extremely accurately at your particular location, you would have to account for the travel time required for these signals to reach you.) Figure 1-2 shows variations in the length of one day on Earth over a 4-year period, as determined by comparison with a cesium (atomic) clock. Because the variation displayed by Fig. 1-2 is seasonal and repetitious, we suspect the rotating Earth when there is a 1983 difference between Earth and atom as timekeepers. The variation is due to tidal effects caused by the Moon and to large-scale winds. The 13th General Conference on Weights and Measures in 1967 adopted a standard second based on the cesium clock: One second is the time taken by 9 192 631 770 oscillations of the light (of a specified wavelength) emitted by a cesium-133 atom.

Atomic clocks are so consistent that, in principle, two cesium clocks would have to run for 6000 years before their readings would differ by more than 1 s. Even such accuracy pales in comparison with that of clocks currently being developed; their precision may be 1 part in 1018 — that is, 1 s in 1 ' 1018 s (which is about 3 ' 1010 y).

1-3

MASS

Learning Objectives After reading this module, you should be able to . . .

1.07 Change units for mass by using chain-link conversions.

1.08 Relate density to mass and volume when the mass is uniformly distributed.

Key Ideas ● The density * of a material is the mass per unit volume:

Mass The Standard Kilogram The SI standard of mass is a cylinder of platinum and iridium (Fig. 1-3) that is kept at the International Bureau of Weights and Measures near Paris and assigned, by

Figure 1-3 The international 1 kg standard of mass, a platinum–iridium cylinder 3.9 cm in height and in diameter.

*"

Courtesy Bureau International des Poids et Mesures. Reproduced with permission of the BIPM.

● The kilogram is defined in terms of a platinum–iridium standard mass kept near Paris. For measurements on an atomic scale, the atomic mass unit, defined in terms of the atom carbon-12, is usually used.

m . V

7

1-3 MASS

international agreement, a mass of 1 kilogram. Accurate copies have been sent to standardizing laboratories in other countries, and the masses of other bodies can be determined by balancing them against a copy. Table 1-5 shows some masses expressed in kilograms, ranging over about 83 orders of magnitude. The U.S. copy of the standard kilogram is housed in a vault at NIST. It is removed, no more than once a year, for the purpose of checking duplicate copies that are used elsewhere. Since 1889, it has been taken to France twice for recomparison with the primary standard.

A Second Mass Standard The masses of atoms can be compared with one another more precisely than they can be compared with the standard kilogram. For this reason, we have a second mass standard. It is the carbon-12 atom, which, by international agreement, has been assigned a mass of 12 atomic mass units (u). The relation between the two units is 1 u " 1.660 538 86 ' 10$27 kg,

(1-7)

Table 1-5 Some Approximate Masses Mass in Kilograms

Object Known universe Our galaxy Sun Moon Asteroid Eros Small mountain Ocean liner Elephant Grape Speck of dust Penicillin molecule Uranium atom Proton Electron

1 ' 1053 2 ' 1041 2 ' 1030 7 ' 1022 5 ' 1015 1 ' 1012 7 ' 107 5 ' 103 3 ' 10$3 7 ' 10$10 5 ' 10$17 4 ' 10$25 2 ' 10$27 9 ' 10$31

with an uncertainty of %10 in the last two decimal places. Scientists can, with reasonable precision, experimentally determine the masses of other atoms relative to the mass of carbon-12. What we presently lack is a reliable means of extending that precision to more common units of mass, such as a kilogram.

Density As we shall discuss further in Chapter 14, density r (lowercase Greek letter rho) is the mass per unit volume:

*"

m . V

(1-8)

Densities are typically listed in kilograms per cubic meter or grams per cubic centimeter. The density of water (1.00 gram per cubic centimeter) is often used as a comparison. Fresh snow has about 10% of that density; platinum has a density that is about 21 times that of water.

Sample Problem 1.02

Density and liquefaction

A heavy object can sink into the ground during an earthquake if the shaking causes the ground to undergo liquefaction, in which the soil grains experience little friction as they slide over one another. The ground is then effectively quicksand. The possibility of liquefaction in sandy ground can be predicted in terms of the void ratio e for a sample of the ground: e"

Vvoids . Vgrains

KEY IDEA The density of the sand rsand in a sample is the mass per unit volume — that is, the ratio of the total mass msand of the sand grains to the total volume Vtotal of the sample:

*sand " (1-9)

Here, Vgrains is the total volume of the sand grains in the sample and Vvoids is the total volume between the grains (in the voids). If e exceeds a critical value of 0.80, liquefaction can occur during an earthquake. What is the corresponding sand density r sand? Solid silicon dioxide (the primary component of sand) has a density of *SiO2 " 2.600 ' 10 3 kg/m3.

msand . Vtotal

(1-10)

Calculations: The total volume Vtotal of a sample is Vtotal " Vgrains # Vvoids. Substituting for Vvoids from Eq. 1-9 and solving for Vgrains lead to Vtotal Vgrains " (1-11) . 1#e

8

CHAPTE R 1 M EASU R E M E NT

From Eq. 1-8, the total mass msand of the sand grains is the product of the density of silicon dioxide and the total volume of the sand grains: msand " *SiO2Vgrains.

Substituting *SiO " 2.600 ' 10 3 kg/m3 and the critical value of e " 0.80, we find that liquefaction occurs when the sand density is less than 2

(1-12)

*sand "

Substituting this expression into Eq. 1-10 and then substituting for Vgrains from Eq. 1-11 lead to

*sand "

*SiO2 Vtotal *SiO2 " . Vtotal 1 # e 1#e

2.600 ' 10 3 kg/m3 " 1.4 ' 103 kg/m3. 1.80 (Answer)

A building can sink several meters in such liquefaction.

(1-13)

Additional examples, video, and practice available at WileyPLUS

Review & Summary Measurement in Physics Physics is based on measurement

successively by conversion factors written as unity and the units are manipulated like algebraic quantities until only the desired units remain.

of physical quantities. Certain physical quantities have been chosen as base quantities (such as length, time, and mass); each has been defined in terms of a standard and given a unit of measure (such as meter, second, and kilogram). Other physical quantities are defined in terms of the base quantities and their standards and units.

Length The meter is defined as the distance traveled by light during a precisely specified time interval. Time The second is defined in terms of the oscillations of light emitted by an atomic (cesium-133) source. Accurate time signals are sent worldwide by radio signals keyed to atomic clocks in standardizing laboratories.

SI Units The unit system emphasized in this book is the International System of Units (SI). The three physical quantities displayed in Table 1-1 are used in the early chapters. Standards, which must be both accessible and invariable, have been established for these base quantities by international agreement. These standards are used in all physical measurement, for both the base quantities and the quantities derived from them. Scientific notation and the prefixes of Table 1-2 are used to simplify measurement notation.

Mass The kilogram is defined in terms of a platinum – iridium standard mass kept near Paris. For measurements on an atomic scale, the atomic mass unit, defined in terms of the atom carbon-12, is usually used. Density The density r of a material is the mass per unit volume:

Changing Units Conversion of units may be performed by us-

*"

ing chain-link conversions in which the original data are multiplied

m . V

(1-8)

Problems Tutoring problem available (at instructor’s discretion) in WileyPLUS and WebAssign SSM • – •••

Worked-out solution available in Student Solutions Manual

WWW Worked-out solution is at

Number of dots indicates level of problem difficulty

ILW

Interactive solution is at

http://www.wiley.com/college/halliday

Additional information available in The Flying Circus of Physics and at flyingcircusofphysics.com

Module 1-1 Measuring Things, Including Lengths •1 SSM Earth is approximately a sphere of radius 6.37 ' 106 m. What are (a) its circumference in kilometers, (b) its surface area in square kilometers, and (c) its volume in cubic kilometers? •2 A gry is an old English measure for length, defined as 1/10 of a line, where line is another old English measure for length, defined as 1/12 inch. A common measure for length in the publishing business is a point, defined as 1/72 inch. What is an area of 0.50 gry2 in points squared (points2)? •3

The micrometer (1 mm) is often called the micron. (a) How

many microns make up 1.0 km? (b) What fraction of a centimeter equals 1.0 mm? (c) How many microns are in 1.0 yd? •4 Spacing in this book was generally done in units of points and picas: 12 points " 1 pica, and 6 picas " 1 inch. If a figure was misplaced in the page proofs by 0.80 cm, what was the misplacement in (a) picas and (b) points? •5 SSM WWW Horses are to race over a certain English meadow for a distance of 4.0 furlongs. What is the race distance in (a) rods and (b) chains? (1 furlong " 201.168 m, 1 rod " 5.0292 m, and 1 chain " 20.117 m.)

9

PROB LE M S

••6 You can easily convert common units and measures electronically, but you still should be able to use a conversion table, such as those in Appendix D. Table 1-6 is part of a conversion table for a system of volume measures once common in Spain; a volume of 1 fanega is equivalent to 55.501 dm3 (cubic decimeters). To complete the table, what numbers (to three significant figures) should be entered in (a) the cahiz column, (b) the fanega column, (c) the cuartilla column, and (d) the almude column, starting with the top blank? Express 7.00 almudes in (e) medios, (f) cahizes, and (g) cubic centimeters (cm3).

Table 1-6 Problem 6 cahiz 1 cahiz " 1 fanega " 1 cuartilla " 1 almude " 1 medio "

1

fanega

cuartilla

almude

12 1

48 4 1

144 12 3 1

medio 288 24 6 2 1

••8 Harvard Bridge, which connects MIT with its fraternities across the Charles River, has a length of 364.4 Smoots plus one ear. The unit of one Smoot is based on the length of Oliver Reed Smoot, Jr., class of 1962, who was carried or dragged length by length across the bridge so that other pledge members of the Lambda Chi Alpha fraternity could mark off (with paint) 1-Smoot lengths along the bridge. The marks have been repainted biannually by fraternity pledges since the initial measurement, usually during times of traffic congestion so that the police cannot easily interfere. (Presumably, the police were originally upset because the Smoot is not an SI base unit, but these days they seem to have accepted the unit.) Figure 1-4 shows three parallel paths, measured in Smoots (S), Willies (W), and Zeldas (Z). What is the length of 50.0 Smoots in (a) Willies and (b) Zeldas? 212

32

0

258 216

60

S W Z

Figure 1-4 Problem 8. ••9 Antarctica is roughly semicircular, with a radius of 2000 km (Fig. 1-5). The average thickness of its ice cover is 3000 m. How many cubic centimeters of ice does Antarctica contain? (Ignore the curvature of Earth.) 2000 km 3000 m

Figure 1-5 Problem 9.

•11 For about 10 years after the French Revolution, the French government attempted to base measures of time on multiples of ten: One week consisted of 10 days, one day consisted of 10 hours, one hour consisted of 100 minutes, and one minute consisted of 100 seconds. What are the ratios of (a) the French decimal week to the standard week and (b) the French decimal second to the standard second? •12 The fastest growing plant on record is a Hesperoyucca whipplei that grew 3.7 m in 14 days. What was its growth rate in micrometers per second?

••7 ILW Hydraulic engineers in the United States often use, as a unit of volume of water, the acre-foot, defined as the volume of water that will cover 1 acre of land to a depth of 1 ft. A severe thunderstorm dumped 2.0 in. of rain in 30 min on a town of area 26 km2. What volume of water, in acre-feet, fell on the town?

0

Module 1-2 Time •10 Until 1883, every city and town in the United States kept its own local time. Today, travelers reset their watches only when the time change equals 1.0 h. How far, on the average, must you travel in degrees of longitude between the time-zone boundaries at which your watch must be reset by 1.0 h? (Hint: Earth rotates 360° in about 24 h.)

•13 Three digital clocks A, B, and C run at different rates and do not have simultaneous readings of zero. Figure 1-6 shows simultaneous readings on pairs of the clocks for four occasions. (At the earliest occasion, for example, B reads 25.0 s and C reads 92.0 s.) If two events are 600 s apart on clock A, how far apart are they on (a) clock B and (b) clock C? (c) When clock A reads 400 s, what does clock B read? (d) When clock C reads 15.0 s, what does clock B read? (Assume negative readings for prezero times.) 312 25.0

125

92.0

512 200

290

142

A (s) B (s) C (s)

Figure 1-6 Problem 13. •14 A lecture period (50 min) is close to 1 microcentury. (a) How long is a microcentury in minutes? (b) Using percentage difference "

approximation # actual $ actual $ 100,

find the percentage difference from the approximation. •15 A fortnight is a charming English measure of time equal to 2.0 weeks (the word is a contraction of “fourteen nights”). That is a nice amount of time in pleasant company but perhaps a painful string of microseconds in unpleasant company. How many microseconds are in a fortnight? •16 Time standards are now based on atomic clocks. A promising second standard is based on pulsars, which are rotating neutron stars (highly compact stars consisting only of neutrons). Some rotate at a rate that is highly stable, sending out a radio beacon that sweeps briefly across Earth once with each rotation, like a lighthouse beacon. Pulsar PSR 1937 # 21 is an example; it rotates once every 1.557 806 448 872 75 % 3 ms, where the trailing %3 indicates the uncertainty in the last decimal place (it does not mean %3 ms). (a) How many rotations does PSR 1937 # 21 make in 7.00 days? (b) How much time does the pulsar take to rotate exactly one million times and (c) what is the associated uncertainty?

10

CHAPTE R 1 M EASU R E M E NT

•17 SSM Five clocks are being tested in a laboratory. Exactly at noon, as determined by the WWV time signal, on successive days of a week the clocks read as in the following table. Rank the five clocks according to their relative value as good timekeepers, best to worst. Justify your choice.

that range, give the lower value and the higher value, respectively, for the following. (a) How many cubic meters of water are in a cylindrical cumulus cloud of height 3.0 km and radius 1.0 km? (b) How many 1-liter pop bottles would that water fill? (c) Water has a density of 1000 kg/m3. How much mass does the water in the cloud have?

Clock

••27 Iron has a density of 7.87 g/cm3, and the mass of an iron atom is 9.27 ' 10$26 kg. If the atoms are spherical and tightly packed, (a) what is the volume of an iron atom and (b) what is the distance between the centers of adjacent atoms?

Sun.

Mon.

Tues.

Wed.

Thurs.

Fri.

Sat.

A

12:36:40

12:36:56

12:37:12

12:37:27

12:37:44

12:37:59

12:38:14

B

11:59:59

12:00:02

11:59:57

12:00:07

12:00:02

11:59:56

12:00:03

C

15:50:45

15:51:43

15:52:41

15:53:39

15:54:37

15:55:35

15:56:33

D

12:03:59

12:02:52

12:01:45

12:00:38

11:59:31

11:58:24

11:57:17

E

12:03:59

12:02:49

12:01:54

12:01:52

12:01:32

12:01:22

12:01:12

••18 Because Earth’s rotation is gradually slowing, the length of each day increases: The day at the end of 1.0 century is 1.0 ms longer than the day at the start of the century. In 20 centuries, what is the total of the daily increases in time? •••19 Suppose that, while lying on a beach near the equator watching the Sun set over a calm ocean, you start a stopwatch just as the top of the Sun disappears. You then stand, elevating your eyes by a height H " 1.70 m, and stop the watch when the top of the Sun again disappears. If the elapsed time is t " 11.1 s, what is the radius r of Earth? Module 1-3 Mass •20 The record for the largest glass bottle was set in 1992 by a team in Millville, New Jersey — they blew a bottle with a volume of 193 U.S. fluid gallons. (a) How much short of 1.0 million cubic centimeters is that? (b) If the bottle were filled with water at the leisurely rate of 1.8 g/min, how long would the filling take? Water has a density of 1000 kg/m3. •21 Earth has a mass of 5.98 ' 1024 kg.The average mass of the atoms that make up Earth is 40 u. How many atoms are there in Earth? •22 Gold, which has a density of 19.32 g/cm3, is the most ductile metal and can be pressed into a thin leaf or drawn out into a long fiber. (a) If a sample of gold, with a mass of 27.63 g, is pressed into a leaf of 1.000 mm thickness, what is the area of the leaf? (b) If, instead, the gold is drawn out into a cylindrical fiber of radius 2.500 mm, what is the length of the fiber? •23 SSM (a) Assuming that water has a density of exactly 1 g/cm3, find the mass of one cubic meter of water in kilograms. (b) Suppose that it takes 10.0 h to drain a container of 5700 m3 of water. What is the “mass flow rate,” in kilograms per second, of water from the container? ••24 Grains of fine California beach sand are approximately spheres with an average radius of 50 +m and are made of silicon dioxide, which has a density of 2600 kg/m3. What mass of sand grains would have a total surface area (the total area of all the individual spheres) equal to the surface area of a cube 1.00 m on an edge? ••25 During heavy rain, a section of a mountainside measuring 2.5 km horizontally, 0.80 km up along the slope, and 2.0 m deep slips into a valley in a mud slide. Assume that the mud ends up uniformly distributed over a surface area of the valley measuring 0.40 km ' 0.40 km and that mud has a density of 1900 kg/m3. What is the mass of the mud sitting above a 4.0 m2 area of the valley floor? ••26 One cubic centimeter of a typical cumulus cloud contains 50 to 500 water drops, which have a typical radius of 10 mm. For

••28 A mole of atoms is 6.02 ' 1023 atoms. To the nearest order of magnitude, how many moles of atoms are in a large domestic cat? The masses of a hydrogen atom, an oxygen atom, and a carbon atom are 1.0 u, 16 u, and 12 u, respectively. (Hint: Cats are sometimes known to kill a mole.) ••29 On a spending spree in Malaysia, you buy an ox with a weight of 28.9 piculs in the local unit of weights: 1 picul " 100 gins, 1 gin " 16 tahils, 1 tahil " 10 chees, and 1 chee " 10 hoons. The weight of 1 hoon corresponds to a mass of 0.3779 g. When you arrange to ship the ox home to your astonished family, how much mass in kilograms must you declare on the shipping manifest? (Hint: Set up multiple chain-link conversions.) ••30 Water is poured into a container that has a small leak. The mass m of the water is given as a function of time t by m " 5.00t0.8 $ 3.00t # 20.00, with t , 0, m in grams, and t in seconds. (a) At what time is the water mass greatest, and (b) what is that greatest mass? In kilograms per minute, what is the rate of mass change at (c) t " 2.00 s and (d) t " 5.00 s? •••31 A vertical container with base area measuring 14.0 cm by 17.0 cm is being filled with identical pieces of candy, each with a volume of 50.0 mm3 and a mass of 0.0200 g.Assume that the volume of the empty spaces between the candies is negligible. If the height of the candies in the container increases at the rate of 0.250 cm/s, at what rate (kilograms per minute) does the mass of the candies in the container increase? Additional Problems 32 In the United States, a doll house has the scale of 1!12 of a real house (that is, each length of the doll house is 121 that of the real house) and a miniature house (a doll house to fit within a doll house) has the scale of 1!144 of a real house. Suppose a real house (Fig. 1-7) has a front length of 20 m, a depth of 12 m, a height of 6.0 m, and a standard sloped roof (vertical triangular faces on the ends) of height 3.0 m. In cubic meters, what are the volumes of the corresponding (a) doll house and (b) miniature house?

3.0 m

6.0 m

20 m 12 m

Figure 1-7 Problem 32.

PROB LE M S

33 SSM A ton is a measure of volume frequently used in shipping, but that use requires some care because there are at least three types of tons: A displacement ton is equal to 7 barrels bulk, a freight ton is equal to 8 barrels bulk, and a register ton is equal to 20 barrels bulk. A barrel bulk is another measure of volume: 1 barrel bulk " 0.1415 m3. Suppose you spot a shipping order for “73 tons” of M&M candies, and you are certain that the client who sent the order intended “ton” to refer to volume (instead of weight or mass, as discussed in Chapter 5). If the client actually meant displacement tons, how many extra U.S. bushels of the candies will you erroneously ship if you interpret the order as (a) 73 freight tons and (b) 73 register tons? (1 m3 " 28.378 U.S. bushels.) 34 Two types of barrel units were in use in the 1920s in the United States. The apple barrel had a legally set volume of 7056 cubic inches; the cranberry barrel, 5826 cubic inches. If a merchant sells 20 cranberry barrels of goods to a customer who thinks he is receiving apple barrels, what is the discrepancy in the shipment volume in liters? 35 An old English children’s rhyme states, “Little Miss Muffet sat on a tuffet, eating her curds and whey, when along came a spider who sat down beside her. . . .” The spider sat down not because of the curds and whey but because Miss Muffet had a stash of 11 tuffets of dried flies. The volume measure of a tuffet is given by 1 tuffet " 2 pecks " 0.50 Imperial bushel, where 1 Imperial bushel " 36.3687 liters (L). What was Miss Muffet’s stash in (a) pecks, (b) Imperial bushels, and (c) liters? 36 Table 1-7 shows some old measures of liquid volume. To complete the table, what numbers (to three significant figures) should be entered in (a) the wey column, (b) the chaldron column, (c) the bag column, (d) the pottle column, and (e) the gill column, starting from the top down? (f) The volume of 1 bag is equal to 0.1091 m3. If an old story has a witch cooking up some vile liquid in a cauldron of volume 1.5 chaldrons, what is the volume in cubic meters?

Table 1-7 Problem 36

1 wey " 1 chaldron " 1 bag " 1 pottle " 1 gill "

wey

chaldron

bag

pottle

gill

1

10/9

40/3

640

120 240

37 A typical sugar cube has an edge length of 1 cm. If you had a cubical box that contained a mole of sugar cubes, what would its edge length be? (One mole " 6.02 ' 1023 units.) 38 An old manuscript reveals that a landowner in the time of King Arthur held 3.00 acres of plowed land plus a livestock area of 25.0 perches by 4.00 perches. What was the total area in (a) the old unit of roods and (b) the more modern unit of square meters? Here, 1 acre is an area of 40 perches by 4 perches, 1 rood is an area of 40 perches by 1 perch, and 1 perch is the length 16.5 ft. 39 SSM A tourist purchases a car in England and ships it home to the United States. The car sticker advertised that the car’s fuel consumption was at the rate of 40 miles per gallon on the open road.

11

The tourist does not realize that the U.K. gallon differs from the U.S. gallon: 1 U.K. gallon " 4.546 090 0 liters 1 U.S. gallon " 3.785 411 8 liters. For a trip of 750 miles (in the United States), how many gallons of fuel does (a) the mistaken tourist believe she needs and (b) the car actually require? 40 Using conversions and data in the chapter, determine the number of hydrogen atoms required to obtain 1.0 kg of hydrogen. A hydrogen atom has a mass of 1.0 u. 41 SSM A cord is a volume of cut wood equal to a stack 8 ft long, 4 ft wide, and 4 ft high. How many cords are in 1.0 m3? 42 One molecule of water (H2O) contains two atoms of hydrogen and one atom of oxygen.A hydrogen atom has a mass of 1.0 u and an atom of oxygen has a mass of 16 u, approximately. (a) What is the mass in kilograms of one molecule of water? (b) How many molecules of water are in the world’s oceans, which have an estimated total mass of 1.4 ' 1021 kg? 43 A person on a diet might lose 2.3 kg per week. Express the mass loss rate in milligrams per second, as if the dieter could sense the second-by-second loss. 44 What mass of water fell on the town in Problem 7? Water has a density of 1.0 ' 103 kg/m3. 45 (a) A unit of time sometimes used in microscopic physics is the shake. One shake equals 10$8 s. Are there more shakes in a second than there are seconds in a year? (b) Humans have existed for about 106 years, whereas the universe is about 1010 years old. If the age of the universe is defined as 1 “universe day,” where a universe day consists of “universe seconds” as a normal day consists of normal seconds, how many universe seconds have humans existed? 46 A unit of area often used in measuring land areas is the hectare, defined as 104 m2. An open-pit coal mine consumes 75 hectares of land, down to a depth of 26 m, each year. What volume of earth, in cubic kilometers, is removed in this time? 47 SSM An astronomical unit (AU) is the average distance between Earth and the Sun, approximately 1.50 ' 108 km. The speed of light is about 3.0 ' 108 m/s. Express the speed of light in astronomical units per minute. 48 The common Eastern mole, a mammal, typically has a mass of 75 g, which corresponds to about 7.5 moles of atoms. (A mole of atoms is 6.02 ' 1023 atoms.) In atomic mass units (u), what is the average mass of the atoms in the common Eastern mole? 49 A traditional unit of length in Japan is the ken (1 ken " 1.97 m). What are the ratios of (a) square kens to square meters and (b) cubic kens to cubic meters? What is the volume of a cylindrical water tank of height 5.50 kens and radius 3.00 kens in (c) cubic kens and (d) cubic meters? 50 You receive orders to sail due east for 24.5 mi to put your salvage ship directly over a sunken pirate ship. However, when your divers probe the ocean floor at that location and find no evidence of a ship, you radio back to your source of information, only to discover that the sailing distance was supposed to be 24.5 nautical miles, not regular miles. Use the Length table in Appendix D to calculate how far horizontally you are from the pirate ship in kilometers.

12

CHAPTE R 1 M EASU R E M E NT

51 The cubit is an ancient unit of length based on the distance between the elbow and the tip of the middle finger of the measurer. Assume that the distance ranged from 43 to 53 cm, and suppose that ancient drawings indicate that a cylindrical pillar was to have a length of 9 cubits and a diameter of 2 cubits. For the stated range, what are the lower value and the upper value, respectively, for (a) the cylinder’s length in meters, (b) the cylinder’s length in millimeters, and (c) the cylinder’s volume in cubic meters? 52 As a contrast between the old and the modern and between the large and the small, consider the following: In old rural England 1 hide (between 100 and 120 acres) was the area of land needed to sustain one family with a single plough for one year. (An area of 1 acre is equal to 4047 m2.) Also, 1 wapentake was the area of land needed by 100 such families. In quantum physics, the cross-sectional area of a nucleus (defined in terms of the chance of a particle hitting and being absorbed by it) is measured in units of barns, where 1 barn is 1 ' 10$28 m2. (In nuclear physics jargon, if a nucleus is “large,” then shooting a particle at it is like shooting a bullet at a barn door, which can hardly be missed.) What is the ratio of 25 wapentakes to 11 barns? 53 SSM An astronomical unit (AU) is equal to the average distance from Earth to the Sun, about 92.9 ' 106 mi. A parsec (pc) is the distance at which a length of 1 AU would subtend an angle of exactly 1 second of An angle of arc (Fig. 1-8). A light-year (ly) exactly 1 second is the distance that light, trav1 pc eling through a vacuum with a speed of 186 000 mi/s, would 1 AU 1 pc cover in 1.0 year. Express the Earth – Sun distance in (a) Figure 1-8 Problem 53. parsecs and (b) light-years. 54 The description for a certain brand of house paint claims a coverage of 460 ft2/gal. (a) Express this quantity in square meters per liter. (b) Express this quantity in an SI unit (see Appendices A and D). (c) What is the inverse of the original quantity, and (d) what is its physical significance? 55 Strangely, the wine for a large wedding reception is to be served in a stunning cut-glass receptacle with the interior dimensions of 40 cm ' 40 cm ' 30 cm (height). The receptacle is to be initially filled to the top. The wine can be purchased in bottles of the sizes given in the following table. Purchasing a larger bottle instead of multiple smaller bottles decreases the overall cost of the wine. To minimize the cost, (a) which bottle sizes should be purchased and how many of each should be purchased and, once the receptacle is filled, how much wine is left over in terms of (b) standard bottles and (c) liters? 1 standard bottle 1 magnum " 2 standard bottles 1 jeroboam " 4 standard bottles 1 rehoboam " 6 standard bottles 1 methuselah " 8 standard bottles 1 salmanazar " 12 standard bottles 1 balthazar " 16 standard bottles " 11.356 L 1 nebuchadnezzar " 20 standard bottles

56 The corn–hog ratio is a financial term used in the pig market and presumably is related to the cost of feeding a pig until it is large enough for market. It is defined as the ratio of the market price of a pig with a mass of 3.108 slugs to the market price of a U.S. bushel of corn. (The word “slug” is derived from an old German word that means “to hit”; we have the same meaning for “slug” as a verb in modern English.) A U.S. bushel is equal to 35.238 L. If the corn–hog ratio is listed as 5.7 on the market exchange, what is it in the metric units of price of 1 kilogram of pig ? price of 1 liter of corn (Hint: See the Mass table in Appendix D.) 57 You are to fix dinners for 400 people at a convention of Mexican food fans. Your recipe calls for 2 jalapeño peppers per serving (one serving per person). However, you have only habanero peppers on hand. The spiciness of peppers is measured in terms of the scoville heat unit (SHU). On average, one jalapeño pepper has a spiciness of 4000 SHU and one habanero pepper has a spiciness of 300 000 SHU. To get the desired spiciness, how many habanero peppers should you substitute for the jalapeño peppers in the recipe for the 400 dinners? 58 A standard interior staircase has steps each with a rise (height) of 19 cm and a run (horizontal depth) of 23 cm. Research suggests that the stairs would be safer for descent if the run were, instead, 28 cm. For a particular staircase of total height 4.57 m, how much farther into the room would the staircase extend if this change in run were made? 59 In purchasing food for a political rally, you erroneously order shucked medium-size Pacific oysters (which come 8 to 12 per U.S. pint) instead of shucked medium-size Atlantic oysters (which come 26 to 38 per U.S. pint). The filled oyster container shipped to you has the interior measure of 1.0 m ' 12 cm ' 20 cm, and a U.S. pint is equivalent to 0.4732 liter. By how many oysters is the order short of your anticipated count? 60 An old English cookbook carries this recipe for cream of nettle soup: “Boil stock of the following amount: 1 breakfastcup plus 1 teacup plus 6 tablespoons plus 1 dessertspoon. Using gloves, separate nettle tops until you have 0.5 quart; add the tops to the boiling stock. Add 1 tablespoon of cooked rice and 1 saltspoon of salt. Simmer for 15 min.” The following table gives some of the conversions among old (premetric) British measures and among common (still premetric) U.S. measures. (These measures just scream for metrication.) For liquid measures, 1 British teaspoon " 1 U.S. teaspoon. For dry measures, 1 British teaspoon " 2 U.S. teaspoons and 1 British quart " 1 U.S. quart. In U.S. measures, how much (a) stock, (b) nettle tops, (c) rice, and (d) salt are required in the recipe? Old British Measures

U.S. Measures

teaspoon " 2 saltspoons dessertspoon " 2 teaspoons tablespoon " 2 dessertspoons teacup " 8 tablespoons breakfastcup " 2 teacups

tablespoon " 3 teaspoons half cup " 8 tablespoons cup " 2 half cups

C

H

A

P

T

E

R

2

Motion Along a Straight Line 2-1 POSITION, DISPLACEMENT, AND AVERAGE VELOCITY Learning Objectives After reading this module, you should be able to …

2.01 Identify that if all parts of an object move in the same direction and at the same rate, we can treat the object as if it were a (point-like) particle. (This chapter is about the motion of such objects.) 2.02 Identify that the position of a particle is its location as read on a scaled axis, such as an x axis. 2.03 Apply the relationship between a particle’s displacement and its initial and final positions.

2.04 Apply the relationship between a particle’s average velocity, its displacement, and the time interval for that displacement. 2.05 Apply the relationship between a particle’s average speed, the total distance it moves, and the time interval for the motion. 2.06 Given a graph of a particle’s position versus time, determine the average velocity between any two particular times.

Key Ideas ● The position x of a particle on an x axis locates the particle

with respect to the origin, or zero point, of the axis. ● The position is either positive or negative, according to which side of the origin the particle is on, or zero if the particle is at the origin. The positive direction on an axis is the direction of increasing positive numbers; the opposite direction is the negative direction on the axis. ● The displacement - x of a particle is the change in its

position: -x " x2 $ x 1. Displacement is a vector quantity. It is positive if the particle has moved in the positive direction of the x axis and negative if the particle has moved in the negative direction.



● When a particle has moved from position x1 to position x2

during a time interval -t " t2 $ t1, its average velocity during that interval is -x x2 $ x1 . " vavg " -t t2 $ t1 ● The algebraic sign of vavg indicates the direction of motion (vavg is a vector quantity). Average velocity does not depend on the actual distance a particle moves, but instead depends on its original and final positions. ● On a graph of x versus t, the average velocity for a time in-

terval -t is the slope of the straight line connecting the points on the curve that represent the two ends of the interval. ● The average speed savg of a particle during a time interval -t depends on the total distance the particle moves in that time interval: total distance savg " . -t

What Is Physics? One purpose of physics is to study the motion of objects—how fast they move, for example, and how far they move in a given amount of time. NASCAR engineers are fanatical about this aspect of physics as they determine the performance of their cars before and during a race. Geologists use this physics to measure tectonic-plate motion as they attempt to predict earthquakes. Medical researchers need this physics to map the blood flow through a patient when diagnosing a partially closed artery, and motorists use it to determine how they might slow sufficiently when their radar detector sounds a warning. There are countless other examples. In this chapter, we study the basic physics of motion where the object (race car, tectonic plate, blood cell, or any other object) moves along a single axis. Such motion is called one-dimensional motion. 13

14

CHAPTE R 2 M OTION ALONG A STRAIG HT LI N E

Motion The world, and everything in it, moves. Even seemingly stationary things, such as a roadway, move with Earth’s rotation, Earth’s orbit around the Sun, the Sun’s orbit around the center of the Milky Way galaxy, and that galaxy’s migration relative to other galaxies. The classification and comparison of motions (called kinematics) is often challenging.What exactly do you measure, and how do you compare? Before we attempt an answer, we shall examine some general properties of motion that is restricted in three ways. 1. The motion is along a straight line only. The line may be vertical, horizontal, or slanted, but it must be straight. 2. Forces (pushes and pulls) cause motion but will not be discussed until Chapter 5. In this chapter we discuss only the motion itself and changes in the motion. Does the moving object speed up, slow down, stop, or reverse direction? If the motion does change, how is time involved in the change? 3. The moving object is either a particle (by which we mean a point-like object such as an electron) or an object that moves like a particle (such that every portion moves in the same direction and at the same rate). A stiff pig slipping down a straight playground slide might be considered to be moving like a particle; however, a tumbling tumbleweed would not.

Position and Displacement Positive direction Negative direction –3

–2

–1

0

1

2

3

x (m)

Origin

Figure 2-1 Position is determined on an axis that is marked in units of length (here meters) and that extends indefinitely in opposite directions. The axis name, here x, is always on the positive side of the origin.

To locate an object means to find its position relative to some reference point, often the origin (or zero point) of an axis such as the x axis in Fig. 2-1. The positive direction of the axis is in the direction of increasing numbers (coordinates), which is to the right in Fig. 2-1. The opposite is the negative direction. For example, a particle might be located at x " 5 m, which means it is 5 m in the positive direction from the origin. If it were at x " $5 m, it would be just as far from the origin but in the opposite direction. On the axis, a coordinate of $5 m is less than a coordinate of $1 m, and both coordinates are less than a coordinate of #5 m. A plus sign for a coordinate need not be shown, but a minus sign must always be shown. A change from position x1 to position x2 is called a displacement -x, where -x " x2 $ x1.

(2-1)

(The symbol -, the Greek uppercase delta, represents a change in a quantity, and it means the final value of that quantity minus the initial value.) When numbers are inserted for the position values x1 and x2 in Eq. 2-1, a displacement in the positive direction (to the right in Fig. 2-1) always comes out positive, and a displacement in the opposite direction (left in the figure) always comes out negative. For example, if the particle moves from x1 " 5 m to x2 " 12 m, then the displacement is -x " (12 m) $ (5 m) " #7 m. The positive result indicates that the motion is in the positive direction. If, instead, the particle moves from x1 " 5 m to x2 " 1 m, then -x " (1 m) $ (5 m) " $4 m. The negative result indicates that the motion is in the negative direction. The actual number of meters covered for a trip is irrelevant; displacement involves only the original and final positions. For example, if the particle moves from x " 5 m out to x " 200 m and then back to x " 5 m, the displacement from start to finish is -x " (5 m) $ (5 m) " 0. Signs. A plus sign for a displacement need not be shown, but a minus sign must always be shown. If we ignore the sign (and thus the direction) of a displacement, we are left with the magnitude (or absolute value) of the displacement. For example, a displacement of -x " $4 m has a magnitude of 4 m.

2-1 POSITION, DISPL ACE M E NT, AN D AVE RAG E VE LOCITY

Figure 2-2 The graph of x(t) for an armadillo that is stationary at x " $2 m. The value of x is $2 m for all times t.

This is a graph of position x versus time t for a stationary object.

15

x (m) +1 –1 0 –1

1

2

3

Same position for any time.

t (s)

4

x(t)

Displacement is an example of a vector quantity, which is a quantity that has both a direction and a magnitude. We explore vectors more fully in Chapter 3, but here all we need is the idea that displacement has two features: (1) Its magnitude is the distance (such as the number of meters) between the original and final positions. (2) Its direction, from an original position to a final position, can be represented by a plus sign or a minus sign if the motion is along a single axis. Here is the first of many checkpoints where you can check your understanding with a bit of reasoning. The answers are in the back of the book.

Checkpoint 1 Here are three pairs of initial and final positions, respectively, along an x axis. Which pairs give a negative displacement: (a) $3 m, #5 m; (b) $3 m, $7 m; (c) 7 m, $3 m?

Average Velocity and Average Speed A compact way to describe position is with a graph of position x plotted as a function of time t—a graph of x(t). (The notation x(t) represents a function x of t, not the product x times t.) As a simple example, Fig. 2-2 shows the position function x(t) for a stationary armadillo (which we treat as a particle) over a 7 s time interval. The animal’s position stays at x " $2 m. Figure 2-3 is more interesting, because it involves motion. The armadillo is apparently first noticed at t " 0 when it is at the position x " $5 m. It moves

A

x (m)

This is a graph of position x versus time t for a moving object.

At x = 2 m when t = 4 s. Plotted here.

4 3 2

x(t)

1 0 –1

1

2

–5 3

4

0

t (s)

2 4s

x (m)

–2 –3

It is at position x = –5 m when time t = 0 s. Those data are plotted here. –5 0s

0

2

x (m)

–4 –5

At x = 0 m when t = 3 s. Plotted here. –5

0 3s

2

x (m)

Figure 2-3 The graph of x(t) for a moving armadillo. The path associated with the graph is also shown, at three times.

16

CHAPTE R 2 M OTION ALONG A STRAIG HT LI N E

toward x " 0, passes through that point at t " 3 s, and then moves on to increasingly larger positive values of x. Figure 2-3 also depicts the straight-line motion of the armadillo (at three times) and is something like what you would see. The graph in Fig. 2-3 is more abstract, but it reveals how fast the armadillo moves. Actually, several quantities are associated with the phrase “how fast.” One of them is the average velocity vavg, which is the ratio of the displacement -x that occurs during a particular time interval -t to that interval: vavg "

-x x2 $ x1 . " -t t2 $ t1

(2-2)

The notation means that the position is x1 at time t1 and then x2 at time t2. A common unit for vavg is the meter per second (m/s). You may see other units in the problems, but they are always in the form of length/time. Graphs. On a graph of x versus t, vavg is the slope of the straight line that connects two particular points on the x(t) curve: one is the point that corresponds to x2 and t2, and the other is the point that corresponds to x1 and t1. Like displacement, vavg has both magnitude and direction (it is another vector quantity). Its magnitude is the magnitude of the line’s slope. A positive vavg (and slope) tells us that the line slants upward to the right; a negative vavg (and slope) tells us that the line slants downward to the right. The average velocity vavg always has the same sign as the displacement -x because -t in Eq. 2-2 is always positive. Figure 2-4 shows how to find vavg in Fig. 2-3 for the time interval t " 1 s to t " 4 s. We draw the straight line that connects the point on the position curve at the beginning of the interval and the point on the curve at the end of the interval.Then we find the slope -x/-t of the straight line. For the given time interval, the average velocity is vavg "

6m " 2 m/s. 3s

Average speed savg is a different way of describing “how fast” a particle moves. Whereas the average velocity involves the particle’s displacement -x, the average speed involves the total distance covered (for example, the number of meters moved), independent of direction; that is, savg "

total distance . -t

(2-3)

Because average speed does not include direction, it lacks any algebraic sign. Sometimes savg is the same (except for the absence of a sign) as vavg. However, the two can be quite different.

A

This is a graph of position x versus time t.

x (m) 4 3 2

Figure 2-4 Calculation of the average velocity between t " 1 s and t " 4 s as the slope of the line that connects the points on the x(t) curve representing those times. The swirling icon indicates that a figure is available in WileyPLUS as an animation with voiceover.

To find average velocity, first draw a straight line, start to end, and then find the slope of the line.

End of interval

1 0 –1 –2 –3 x(t) –4 –5

Start of interval

vavg = slope of this line rise ∆x = ___ = __ run ∆t

1

2

3

4

t (s) This vertical distance is how far it moved, start to end: ∆x = 2 m – (–4 m) = 6 m

This horizontal distance is how long it took, start to end: ∆t = 4 s – 1 s = 3 s

17

2-1 POSITION, DISPL ACE M E NT, AN D AVE RAG E VE LOCITY

Sample Problem 2.01 Average velocity, beat-up pickup truck

(a) What is your overall displacement from the beginning of your drive to your arrival at the station? KEY IDEA Assume, for convenience, that you move in the positive direction of an x axis, from a first position of x1 " 0 to a second position of x2 at the station. That second position must be at x2 " 8.4 km # 2.0 km " 10.4 km. Then your displacement -x along the x axis is the second position minus the first position. Calculation: From Eq. 2-1, we have -x " x2 $ x1 " 10.4 km $ 0 " 10.4 km.

(Answer)

Thus, your overall displacement is 10.4 km in the positive direction of the x axis. (b) What is the time interval -t from the beginning of your drive to your arrival at the station? KEY IDEA

Calculation: Here we find vavg "

" 16.8 km/h % 17 km/h.

(d) Suppose that to pump the gasoline, pay for it, and walk back to the truck takes you another 45 min. What is your average speed from the beginning of your drive to your return to the truck with the gasoline? KEY IDEA Your average speed is the ratio of the total distance you move to the total time interval you take to make that move. Calculation: The total distance is 8.4 km # 2.0 km # 2.0 km " 12.4 km. The total time interval is 0.12 h # 0.50 h # 0.75 h " 1.37 h. Thus, Eq. 2-3 gives us savg "

10

KEY IDEA From Eq. 2-2 we know that vavg for the entire trip is the ratio of the displacement of 10.4 km for the entire trip to the time interval of 0.62 h for the entire trip.

Slope of this line gives average velocity.

Station

g

Walkin

8 g

Position (km)

-xdr . vavg,dr " -tdr Rearranging and substituting data then give us

(Answer) " 0.12 h # 0.50 h " 0.62 h. (c) What is your average velocity vavg from the beginning of your drive to your arrival at the station? Find it both numerically and graphically.

(Answer)

Driving ends, walking starts.

12

-t " -tdr # -twlk

12.4 km " 9.1 km/h. 1.37 h

x

Calculations: We first write

So,

(Answer)

To find vavg graphically, first we graph the function x(t) as shown in Fig. 2-5, where the beginning and arrival points on the graph are the origin and the point labeled as “Station.”Your average velocity is the slope of the straight line connecting those points; that is, vavg is the ratio of the rise (-x " 10.4 km) to the run (-t " 0.62 h), which gives us vavg " 16.8 km/h.

We already know the walking time interval -twlk (" 0.50 h), but we lack the driving time interval -tdr. However, we know that for the drive the displacement -xdr is 8.4 km and the average velocity vavg,dr is 70 km/h. Thus, this average velocity is the ratio of the displacement for the drive to the time interval for the drive.

-xdr 8.4 km -tdr " " " 0.12 h. vavg,dr 70 km/h

10.4 km -x " -t 0.62 h

6 4

Driv in

You drive a beat-up pickup truck along a straight road for 8.4 km at 70 km/h, at which point the truck runs out of gasoline and stops. Over the next 30 min, you walk another 2.0 km farther along the road to a gasoline station.

How far: ∆ x = 10.4 km

2

0

0

0.2

0.4 Time (h)

0.6

t

How long: ∆t = 0.62 h Figure 2-5 The lines marked “Driving” and “Walking” are the position – time plots for the driving and walking stages. (The plot for the walking stage assumes a constant rate of walking.) The slope of the straight line joining the origin and the point labeled “Station” is the average velocity for the trip, from the beginning to the station.

Additional examples, video, and practice available at WileyPLUS

18

CHAPTE R 2 M OTION ALONG A STRAIG HT LI N E

2-2 INSTANTANEOUS VELOCITY AND SPEED Learning Objectives After reading this module, you should be able to . . .

2.07 Given a particle’s position as a function of time, calculate the instantaneous velocity for any particular time.

Key Ideas ● The instantaneous velocity (or simply velocity) v of a moving particle is -x dx v " lim " , - t : 0 -t dt where -x " x2 $ x1 and -t " t2 $ t1.

2.08 Given a graph of a particle’s position versus time, determine the instantaneous velocity for any particular time. 2.09 Identify speed as the magnitude of the instantaneous velocity. ● The instantaneous velocity (at a particular time) may be

found as the slope (at that particular time) of the graph of x versus t. ● Speed is the magnitude of instantaneous velocity.

Instantaneous Velocity and Speed You have now seen two ways to describe how fast something moves: average velocity and average speed, both of which are measured over a time interval -t. However, the phrase “how fast” more commonly refers to how fast a particle is moving at a given instant—its instantaneous velocity (or simply velocity) v. The velocity at any instant is obtained from the average velocity by shrinking the time interval -t closer and closer to 0. As -t dwindles, the average velocity approaches a limiting value, which is the velocity at that instant: v " lim

-t : 0

-x dx " . -t dt

(2-4)

Note that v is the rate at which position x is changing with time at a given instant; that is, v is the derivative of x with respect to t. Also note that v at any instant is the slope of the position – time curve at the point representing that instant. Velocity is another vector quantity and thus has an associated direction. Speed is the magnitude of velocity; that is, speed is velocity that has been stripped of any indication of direction, either in words or via an algebraic sign. (Caution: Speed and average speed can be quite different.) A velocity of #5 m/s and one of $5 m/s both have an associated speed of 5 m/s. The speedometer in a car measures speed, not velocity (it cannot determine the direction).

Checkpoint 2 The following equations give the position x(t) of a particle in four situations (in each equation, x is in meters, t is in seconds, and t . 0): (1) x " 3t $ 2; (2) x " $4t 2 $ 2; (3) x " 2/t 2; and (4) x " $2. (a) In which situation is the velocity v of the particle constant? (b) In which is v in the negative x direction?

Sample Problem 2.02 Velocity and slope of x versus t, elevator cab Figure 2-6a is an x(t) plot for an elevator cab that is initially stationary, then moves upward (which we take to be the positive direction of x), and then stops. Plot v(t). KEY IDEA We can find the velocity at any time from the slope of the x(t) curve at that time.

Calculations: The slope of x(t), and so also the velocity, is zero in the intervals from 0 to 1 s and from 9 s on, so then the cab is stationary. During the interval bc, the slope is constant and nonzero, so then the cab moves with constant velocity. We calculate the slope of x(t) then as -x 24 m $ 4.0 m "v" " #4.0 m/s. -t 8.0 s $ 3.0 s

(2-5)

19

2-2 I NSTANTAN EOUS VE LOCITY AN D SPE E D

x

Position (m)

25

x = 24 m at t = 8.0 s

20 15 x = 4.0 m at t = 3.0 s

5 0

∆x

x(t)

10

b

a 0

1

2

3

∆t 4

5 6 Time (s) (a)

v b

4 Velocity (m/s)

d

c

7

8

t

9

Slopes on the x versus t graph are the values on the v versus t graph.

Slope of x(t)

c

v(t)

3 2 1

Figure 2-6 (a) The x(t) curve for an elevator cab that moves upward along an x axis. (b) The v(t) curve for the cab. Note that it is the derivative of the x(t) curve (v " dx/dt). (c) The a(t) curve for the cab. It is the derivative of the v(t) curve (a " dv/dt). The stick figures along the bottom suggest how a passenger’s body might feel during the accelerations.

Acceleration (m/s2)

0

3 2 1 0 –1 –2 –3 –4

a 0

d 1

2

3

4

5 6 Time (s) (b)

7

8

b 1

2

3

4

a(t) 5

6

t

Slopes on the v versus t graph are the values on the a versus t graph.

a Acceleration a

9

7

c 8

d 9

t

Deceleration

What you would feel. (c)

The plus sign indicates that the cab is moving in the positive x direction. These intervals (where v " 0 and v " 4 m/s) are plotted in Fig. 2-6b. In addition, as the cab initially begins to move and then later slows to a stop, v varies as indicated in the intervals 1 s to 3 s and 8 s to 9 s. Thus, Fig. 2-6b is the required plot. (Figure 2-6c is considered in Module 2-3.) Given a v(t) graph such as Fig. 2-6b, we could “work backward” to produce the shape of the associated x(t) graph (Fig. 2-6a). However, we would not know the actual values for x at various times, because the v(t) graph indicates only changes in x. To find such a change in x during any in-

terval, we must, in the language of calculus, calculate the area “under the curve” on the v(t) graph for that interval. For example, during the interval 3 s to 8 s in which the cab has a velocity of 4.0 m/s, the change in x is -x " (4.0 m/s)(8.0 s $ 3.0 s) " #20 m.

(2-6)

(This area is positive because the v(t) curve is above the t axis.) Figure 2-6a shows that x does indeed increase by 20 m in that interval. However, Fig. 2-6b does not tell us the values of x at the beginning and end of the interval. For that, we need additional information, such as the value of x at some instant.

Additional examples, video, and practice available at WileyPLUS

20

CHAPTE R 2 M OTION ALONG A STRAIG HT LI N E

2-3 ACCELERATION Learning Objectives After reading this module, you should be able to . . .

2.10 Apply the relationship between a particle’s average acceleration, its change in velocity, and the time interval for that change. 2.11 Given a particle’s velocity as a function of time, calculate the instantaneous acceleration for any particular time.

2.12 Given a graph of a particle’s velocity versus time, determine the instantaneous acceleration for any particular time and the average acceleration between any two particular times.

Key Ideas ● Average acceleration is the ratio of a change in velocity -v to the time interval -t in which the change occurs:

aavg "

-v . -t

The algebraic sign indicates the direction of aavg.

● Instantaneous acceleration (or simply acceleration) a is the

first time derivative of velocity v(t) and the second time derivative of position x(t): d 2x dv a" " . dt dt2 ● On a graph of v versus t, the acceleration a at any time t is the slope of the curve at the point that represents t.

Acceleration When a particle’s velocity changes, the particle is said to undergo acceleration (or to accelerate). For motion along an axis, the average acceleration aavg over a time interval -t is aavg "

v2 $ v1 -v " , t2 $ t1 -t

(2-7)

where the particle has velocity v1 at time t1 and then velocity v2 at time t2. The instantaneous acceleration (or simply acceleration) is a"

dv . dt

(2-8)

In words, the acceleration of a particle at any instant is the rate at which its velocity is changing at that instant. Graphically, the acceleration at any point is the slope of the curve of v(t) at that point.We can combine Eq. 2-8 with Eq. 2-4 to write a"

d dv " dt dt

# dxdt $ " ddt x . 2

2

(2-9)

In words, the acceleration of a particle at any instant is the second derivative of its position x(t) with respect to time. A common unit of acceleration is the meter per second per second: m/(s ! s) or m/s2. Other units are in the form of length/(time ! time) or length/time2. Acceleration has both magnitude and direction (it is yet another vector quantity). Its algebraic sign represents its direction on an axis just as for displacement and velocity; that is, acceleration with a positive value is in the positive direction of an axis, and acceleration with a negative value is in the negative direction. Figure 2-6 gives plots of the position, velocity, and acceleration of an elevator moving up a shaft. Compare the a(t) curve with the v(t) curve — each point on the a(t) curve shows the derivative (slope) of the v(t) curve at the corresponding time. When v is constant (at either 0 or 4 m/s), the derivative is zero and so also is the acceleration. When the cab first begins to move, the v(t)

2-3 ACCE LE RATION

curve has a positive derivative (the slope is positive), which means that a(t) is positive. When the cab slows to a stop, the derivative and slope of the v(t) curve are negative; that is, a(t) is negative. Next compare the slopes of the v(t) curve during the two acceleration periods. The slope associated with the cab’s slowing down (commonly called “deceleration”) is steeper because the cab stops in half the time it took to get up to speed. The steeper slope means that the magnitude of the deceleration is larger than that of the acceleration, as indicated in Fig. 2-6c. Sensations. The sensations you would feel while riding in the cab of Fig. 2-6 are indicated by the sketched figures at the bottom. When the cab first accelerates, you feel as though you are pressed downward; when later the cab is braked to a stop, you seem to be stretched upward. In between, you feel nothing special. In other words, your body reacts to accelerations (it is an accelerometer) but not to velocities (it is not a speedometer). When you are in a car traveling at 90 km/h or an airplane traveling at 900 km/h, you have no bodily awareness of the motion. However, if the car or plane quickly changes velocity, you may become keenly aware of the change, perhaps even frightened by it. Part of the thrill of an amusement park ride is due to the quick changes of velocity that you undergo (you pay for the accelerations, not for the speed). A more extreme example is shown in the photographs of Fig. 2-7, which were taken while a rocket sled was rapidly accelerated along a track and then rapidly braked to a stop. g Units. Large accelerations are sometimes expressed in terms of g units, with 1g " 9.8 m/s2

(g unit).

(2-10)

(As we shall discuss in Module 2-5, g is the magnitude of the acceleration of a falling object near Earth’s surface.) On a roller coaster, you may experience brief accelerations up to 3g, which is (3)(9.8 m/s2), or about 29 m/s2, more than enough to justify the cost of the ride. Signs. In common language, the sign of an acceleration has a nonscientific meaning: positive acceleration means that the speed of an object is increasing, and negative acceleration means that the speed is decreasing (the object is decelerating). In this book, however, the sign of an acceleration indicates a direction, not Figure 2-7 Colonel J. P. Stapp in a rocket sled as it is brought up to high speed (acceleration out of the page) and then very rapidly braked (acceleration into the page).

Courtesy U.S. Air Force

21

22

CHAPTE R 2 M OTION ALONG A STRAIG HT LI N E

whether an object’s speed is increasing or decreasing. For example, if a car with an initial velocity v " $25 m/s is braked to a stop in 5.0 s, then aavg " #5.0 m/s2. The acceleration is positive, but the car’s speed has decreased. The reason is the difference in signs: the direction of the acceleration is opposite that of the velocity. Here then is the proper way to interpret the signs: If the signs of the velocity and acceleration of a particle are the same, the speed of the particle increases. If the signs are opposite, the speed decreases.

Checkpoint 3 A wombat moves along an x axis. What is the sign of its acceleration if it is moving (a) in the positive direction with increasing speed, (b) in the positive direction with decreasing speed, (c) in the negative direction with increasing speed, and (d) in the negative direction with decreasing speed?

Sample Problem 2.03 Acceleration and dv/dt A particle’s position on the x axis of Fig. 2-1 is given by x " 4 $ 27t # t 3, with x in meters and t in seconds. (a) Because position x depends on time t, the particle must be moving. Find the particle’s velocity function v(t) and acceleration function a(t). KEY IDEAS (1) To get the velocity function v(t), we differentiate the position function x(t) with respect to time. (2) To get the acceleration function a(t), we differentiate the velocity function v(t) with respect to time. Calculations: Differentiating the position function, we find v " $27 # 3t 2,

(Answer)

with v in meters per second. Differentiating the velocity function then gives us a " #6t, (Answer) with a in meters per second squared.

t=3s v=0 a pos reversing (c)

(b) Is there ever a time when v " 0? Calculation: Setting v(t) " 0 yields 0 " $27 # 3t 2,

(d )

−50 m

0 4m t=1s v neg a pos slowing (b)

(Answer)

Thus, the velocity is zero both 3 s before and 3 s after the clock reads 0. (c) Describe the particle’s motion for t , 0.

t=4s v pos a pos speeding up x

which has the solution t " %3 s.

Reasoning: We need to examine the expressions for x(t), v(t), and a(t). At t " 0, the particle is at x(0) " #4 m and is moving with a velocity of v(0) " $27 m/s — that is, in the negative direction of the x axis. Its acceleration is a(0) " 0 because just then the particle’s velocity is not changing (Fig. 2-8a). For 0 ( t ( 3 s, the particle still has a negative velocity, so it continues to move in the negative direction. However, its acceleration is no longer 0 but is increasing and positive. Because the signs of the velocity and the acceleration are opposite, the particle must be slowing (Fig. 2-8b). Indeed, we already know that it stops momentarily at t " 3 s. Just then the particle is as far to the left of the origin in Fig. 2-1 as it will ever get. Substituting t " 3 s into the expression for x(t), we find that the particle’s position just then is x " $50 m (Fig. 2-8c). Its acceleration is still positive. For t . 3 s, the particle moves to the right on the axis. Its acceleration remains positive and grows progressively larger in magnitude. The velocity is now positive, and it too grows progressively larger in magnitude (Fig. 2-8d).

Figure 2-8 Four stages of the particle’s motion.

Additional examples, video, and practice available at WileyPLUS

t=0 v neg a=0 leftward motion (a)

23

2-4 CONSTANT ACCE LE RATION

2-4 CONSTANT ACCELERATION Learning Objectives After reading this module, you should be able to . . .

2.13 For constant acceleration, apply the relationships between position, displacement, velocity, acceleration, and elapsed time (Table 2-1).

2.14 Calculate a particle’s change in velocity by integrating its acceleration function with respect to time. 2.15 Calculate a particle’s change in position by integrating its velocity function with respect to time.

Key Ideas ● The following five equations describe the motion of a particle with constant acceleration:

v " v0 # at,

x $ x0 " v0t #

1 2 at , 2

1 (v # v)t, 2 0 These are not valid when the acceleration is not constant. v2 " v 20 # 2a(x $ x0),

x $ x0 "

x $ x0 " vt $

1 2 at . 2

Constant Acceleration: A Special Case Position

x(t)

x0

Slopes of the position graph are plotted on the velocity graph. v v(t)

t

Slope of the velocity graph is plotted on the acceleration graph.

(2-11)

As a check, note that this equation reduces to v " v0 for t " 0, as it must. As a further check, take the derivative of Eq. 2-11. Doing so yields dv/dt " a, which is the definition of a. Figure 2-9b shows a plot of Eq. 2-11, the v(t) function; the function is linear and thus the plot is a straight line. Second Basic Equation. In a similar manner, we can rewrite Eq. 2-2 (with a few changes in notation) as x $ x0 t$0

Slope = a

0

(b)

Here v0 is the velocity at time t " 0 and v is the velocity at any later time t. We can recast this equation as

vavg "

t

v0

v $ v0 . t$0

v " v0 # at.

Slope varies

0

(a)

(c)

Acceleration

a " aavg "

x

Velocity

In many types of motion, the acceleration is either constant or approximately so. For example, you might accelerate a car at an approximately constant rate when a traffic light turns from red to green. Then graphs of your position, velocity, and acceleration would resemble those in Fig. 2-9. (Note that a(t) in Fig. 2-9c is constant, which requires that v(t) in Fig. 2-9b have a constant slope.) Later when you brake the car to a stop, the acceleration (or deceleration in common language) might also be approximately constant. Such cases are so common that a special set of equations has been derived for dealing with them. One approach to the derivation of these equations is given in this section. A second approach is given in the next section. Throughout both sections and later when you work on the homework problems, keep in mind that these equations are valid only for constant acceleration (or situations in which you can approximate the acceleration as being constant). First Basic Equation. When the acceleration is constant, the average acceleration and instantaneous acceleration are equal and we can write Eq. 2-7, with some changes in notation, as

a

0

a(t) Slope = 0 t

Figure 2-9 (a) The position x(t) of a particle moving with constant acceleration. (b) Its velocity v(t), given at each point by the slope of the curve of x(t). (c) Its (constant) acceleration, equal to the (constant) slope of the curve of v(t).

24

CHAPTE R 2 M OTION ALONG A STRAIG HT LI N E

and then as x " x0 # vavgt,

(2-12)

in which x0 is the position of the particle at t " 0 and vavg is the average velocity between t " 0 and a later time t. For the linear velocity function in Eq. 2-11, the average velocity over any time interval (say, from t " 0 to a later time t) is the average of the velocity at the beginning of the interval (" v0) and the velocity at the end of the interval (" v). For the interval from t " 0 to the later time t then, the average velocity is vavg " 12 (v0 # v).

(2-13)

Substituting the right side of Eq. 2-11 for v yields, after a little rearrangement, vavg " v0 # 12 at.

(2-14)

Finally, substituting Eq. 2-14 into Eq. 2-12 yields x $ x0 " v0 t # 12 at 2.

(2-15)

As a check, note that putting t " 0 yields x " x0, as it must. As a further check, taking the derivative of Eq. 2-15 yields Eq. 2-11, again as it must. Figure 2-9a shows a plot of Eq. 2-15; the function is quadratic and thus the plot is curved. Three Other Equations. Equations 2-11 and 2-15 are the basic equations for constant acceleration; they can be used to solve any constant acceleration problem in this book. However, we can derive other equations that might prove useful in certain specific situations. First, note that as many as five quantities can possibly be involved in any problem about constant acceleration — namely, x $ x0, v, t, a, and v0. Usually, one of these quantities is not involved in the problem, either as a given or as an unknown. We are then presented with three of the remaining quantities and asked to find the fourth. Equations 2-11 and 2-15 each contain four of these quantities, but not the same four. In Eq. 2-11, the “missing ingredient” is the displacement x $ x0. In Eq. 2-15, it is the velocity v. These two equations can also be combined in three ways to yield three additional equations, each of which involves a different “missing variable.” First, we can eliminate t to obtain v 2 " v 20 # 2a(x $ x0).

(2-16)

This equation is useful if we do not know t and are not required to find it. Second, we can eliminate the acceleration a between Eqs. 2-11 and 2-15 to produce an equation in which a does not appear: Table 2-1 Equations for Motion with Constant Accelerationa

x $ x0 " 12(v0 # v)t.

(2-17)

Finally, we can eliminate v0, obtaining

Equation Number

Equation

Missing Quantity

x $ x0 " vt $ 12 at 2.

2-11 2-15 2-16 2-17 2-18

v " v0 # at x $ x0 " v0 t # 12at 2 v 2 " v 20 # 2a(x $ x0) x $ x0 " 12(v0 # v)t x $ x0 " vt $ 12at 2

x $ x0 v t a v0

Note the subtle difference between this equation and Eq. 2-15. One involves the initial velocity v0; the other involves the velocity v at time t. Table 2-1 lists the basic constant acceleration equations (Eqs. 2-11 and 2-15) as well as the specialized equations that we have derived.To solve a simple constant acceleration problem, you can usually use an equation from this list (if you have the list with you). Choose an equation for which the only unknown variable is the variable requested in the problem. A simpler plan is to remember only Eqs. 2-11 and 2-15, and then solve them as simultaneous equations whenever needed.

a

Make sure that the acceleration is indeed constant before using the equations in this table.

(2-18)

2-4 CONSTANT ACCE LE RATION

25

Checkpoint 4 The following equations give the position x(t) of a particle in four situations: (1) x " 3t $ 4; (2) x " $5t 3 # 4t 2 # 6; (3) x " 2/t 2 $ 4/t; (4) x " 5t 2 $ 3. To which of these situations do the equations of Table 2-1 apply?

Sample Problem 2.04 Drag race of car and motorcycle A popular web video shows a jet airplane, a car, and a motorcycle racing from rest along a runway (Fig. 2-10). Initially the motorcycle takes the lead, but then the jet takes the lead, and finally the car blows past the motorcycle. Here let’s focus on the car and motorcycle and assign some reasonable values to the motion. The motorcycle first takes the lead because its (constant) acceleration am " 8.40 m/s2 is greater than the car’s (constant) acceleration ac " 5.60 m/s2, but it soon loses to the car because it reaches its greatest speed vm " 58.8 m/s before the car reaches its greatest speed vc " 106 m/s. How long does the car take to reach the motorcycle? KEY IDEAS We can apply the equations of constant acceleration to both vehicles, but for the motorcycle we must consider the motion in two stages: (1) First it travels through distance xm1 with zero initial velocity and acceleration am " 8.40 m/s2, reaching speed vm " 58.8 m/s. (2) Then it travels through distance xm2 with constant velocity vm " 58.8 m/s and zero acceleration (that, too, is a constant acceleration). (Note that we symbolized the distances even though we do not know their values. Symbolizing unknown quantities is often helpful in solving physics problems, but introducing such unknowns sometimes takes physics courage.) Calculations: So that we can draw figures and do calculations, let’s assume that the vehicles race along the positive direction of an x axis, starting from x " 0 at time t " 0. (We can

choose any initial numbers because we are looking for the elapsed time, not a particular time in, say, the afternoon, but let’s stick with these easy numbers.) We want the car to pass the motorcycle, but what does that mean mathematically? It means that at some time t, the side-by-side vehicles are at the same coordinate: xc for the car and the sum xm1 # xm2 for the motorcycle. We can write this statement mathematically as (2-19)

xc " xm1 # xm2.

(Writing this first step is the hardest part of the problem. That is true of most physics problems. How do you go from the problem statement (in words) to a mathematical expression? One purpose of this book is for you to build up that ability of writing the first step — it takes lots of practice just as in learning, say, tae-kwon-do.) Now let’s fill out both sides of Eq. 2-19, left side first. To reach the passing point at xc, the car accelerates from rest. From Eq. 2-15 (x $ x0 " v0t # 12at 2), with x0 and v0 " 0, we have xc " 12act 2.

(2-20)

To write an expression for xm1 for the motorcycle, we first find the time tm it takes to reach its maximum speed vm, using Eq. 2-11 (v " v0 # at). Substituting v0 " 0, v " vm " 58.8 m/s, and a " am " 8.40 m/s2, that time is tm " "

vm am

(2-21)

58.8 m/s " 7.00 s. 8.40 m/s2

To get the distance xm1 traveled by the motorcycle during the first stage, we again use Eq. 2-15 with x0 " 0 and v0 " 0, but we also substitute from Eq. 2-21 for the time. We find

# av $ " 12

xm1 " 12amt 2m " 12am

Figure 2-10 A jet airplane, a car, and a motorcycle just after accelerating from rest.

m

m

2

v 2m . am

(2-22)

For the remaining time of t $ tm , the motorcycle travels at its maximum speed with zero acceleration. To get the distance, we use Eq. 2-15 for this second stage of the motion, but now the initial velocity is v0 " vm (the speed at the end of the first stage) and the acceleration is a " 0. So, the distance traveled during the second stage is xm2 " vm(t $ tm) " vm(t $ 7.00 s).

(2-23)

CHAPTE R 2 M OTION ALONG A STRAIG HT LI N E

To finish the calculation, we substitute Eqs. 2-20, 2-22, and 2-23 into Eq. 2-19, obtaining 1 2 2 act

"

1 v2m # vm(t $ 7.00 s). 2 am

(2-24)

This is a quadratic equation. Substituting in the given data, we solve the equation (by using the usual quadratic-equation formula or a polynomial solver on a calculator), finding t " 4.44 s and t " 16.6 s. But what do we do with two answers? Does the car pass the motorcycle twice? No, of course not, as we can see in the video. So, one of the answers is mathematically correct but not physically meaningful. Because we know that the car passes the motorcycle after the motorcycle reaches its maximum speed at t " 7.00 s, we discard the solution with t ( 7.00 s as being the unphysical answer and conclude that the passing occurs at t " 16.6 s.

(Answer)

Figure 2-11 is a graph of the position versus time for the two vehicles, with the passing point marked. Notice

that at t " 7.00 s the plot for the motorcycle switches from being curved (because the speed had been increasing) to being straight (because the speed is thereafter constant). 1000

800 Car passes motorcycle

600 x (m)

26

Motorcycle 400

200

0

Car

Acceleration ends

0

5

10 t (s)

15

20

Figure 2-11 Graph of position versus time for car and motorcycle.

Additional examples, video, and practice available at WileyPLUS

Another Look at Constant Acceleration* The first two equations in Table 2-1 are the basic equations from which the others are derived. Those two can be obtained by integration of the acceleration with the condition that a is constant. To find Eq. 2-11, we rewrite the definition of acceleration (Eq. 2-8) as dv " a dt. We next write the indefinite integral (or antiderivative) of both sides:

" " dv "

a dt.

Since acceleration a is a constant, it can be taken outside the integration. We obtain

"

"

dv " a

or

dt

v " at # C.

(2-25)

To evaluate the constant of integration C, we let t " 0, at which time v " v0. Substituting these values into Eq. 2-25 (which must hold for all values of t, including t " 0) yields v0 " (a)(0) # C " C. Substituting this into Eq. 2-25 gives us Eq. 2-11. To derive Eq. 2-15, we rewrite the definition of velocity (Eq. 2-4) as dx " v dt and then take the indefinite integral of both sides to obtain

" " dx "

v dt.

*This section is intended for students who have had integral calculus.

2-5 FR E E-FALL ACCE LE RATION

27

Next, we substitute for v with Eq. 2-11:

" " dx "

(v0 # at) dt.

Since v0 is a constant, as is the acceleration a, this can be rewritten as

"

" "

dx " v0 dt # a t dt.

Integration now yields

x " v0t # 12 at 2 # C/,

(2-26)

where C/ is another constant of integration. At time t " 0, we have x " x0. Substituting these values in Eq. 2-26 yields x0 " C/. Replacing C/ with x0 in Eq. 2-26 gives us Eq. 2-15.

2-5 FREE-FALL ACCELERATION Learning Objectives After reading this module, you should be able to . . .

2.16 Identify that if a particle is in free flight (whether upward or downward) and if we can neglect the effects of air on its motion, the particle has a constant

downward acceleration with a magnitude g that we take to be 9.8 m/s2. 2.17 Apply the constant-acceleration equations (Table 2-1) to free-fall motion.

Key Ideas ● An important example of straight-line motion with constant

acceleration is that of an object rising or falling freely near Earth’s surface. The constant acceleration equations describe this motion, but we make two changes in notation:

(1) we refer the motion to the vertical y axis with #y vertically up; (2) we replace a with $g, where g is the magnitude of the free-fall acceleration. Near Earth’s surface, g " 9.8 m/s2 " 32 ft/s2.

Free-Fall Acceleration If you tossed an object either up or down and could somehow eliminate the effects of air on its flight, you would find that the object accelerates downward at a certain constant rate. That rate is called the free-fall acceleration, and its magnitude is represented by g. The acceleration is independent of the object’s characteristics, such as mass, density, or shape; it is the same for all objects. Two examples of free-fall acceleration are shown in Fig. 2-12, which is a series of stroboscopic photos of a feather and an apple. As these objects fall, they accelerate downward — both at the same rate g. Thus, their speeds increase at the same rate, and they fall together. The value of g varies slightly with latitude and with elevation. At sea level in Earth’s midlatitudes the value is 9.8 m/s2 (or 32 ft/s2), which is what you should use as an exact number for the problems in this book unless otherwise noted. The equations of motion in Table 2-1 for constant acceleration also apply to free fall near Earth’s surface; that is, they apply to an object in vertical flight, either up or down, when the effects of the air can be neglected. However, note that for free fall: (1) The directions of motion are now along a vertical y axis instead of the x axis, with the positive direction of y upward. (This is important for later chapters when combined horizontal and vertical motions are examined.) (2) The free-fall acceleration is negative — that is, downward on the y axis, toward Earth’s center — and so it has the value $g in the equations.

© Jim Sugar/CORBIS

Figure 2-12 A feather and an apple free fall in vacuum at the same magnitude of acceleration g. The acceleration increases the distance between successive images. In the absence of air, the feather and apple fall together.

28

CHAPTE R 2 M OTION ALONG A STRAIG HT LI N E

The free-fall acceleration near Earth’s surface is a " $g " $9.8 m/s2, and the magnitude of the acceleration is g " 9.8 m/s2. Do not substitute $9.8 m/s2 for g.

Suppose you toss a tomato directly upward with an initial (positive) velocity v0 and then catch it when it returns to the release level. During its free-fall flight (from just after its release to just before it is caught), the equations of Table 2-1 apply to its motion. The acceleration is always a " $g " $9.8 m/s2, negative and thus downward. The velocity, however, changes, as indicated by Eqs. 2-11 and 2-16: during the ascent, the magnitude of the positive velocity decreases, until it momentarily becomes zero. Because the tomato has then stopped, it is at its maximum height. During the descent, the magnitude of the (now negative) velocity increases.

Checkpoint 5 (a) If you toss a ball straight up, what is the sign of the ball’s displacement for the ascent, from the release point to the highest point? (b) What is it for the descent, from the highest point back to the release point? (c) What is the ball’s acceleration at its highest point?

Sample Problem 2.05 Time for full up-down flight, baseball toss

Ball

In Fig. 2-13, a pitcher tosses a baseball up along a y axis, with an initial speed of 12 m/s.

y

v = 0 at highest point

(a) How long does the ball take to reach its maximum height? KEY IDEAS (1) Once the ball leaves the pitcher and before it returns to his hand, its acceleration is the free-fall acceleration a " $g. Because this is constant, Table 2-1 applies to the motion. (2) The velocity v at the maximum height must be 0. Calculation: Knowing v, a, and the initial velocity v0 " 12 m/s, and seeking t, we solve Eq. 2-11, which contains those four variables. This yields t"

0 $ 12 m/s v $ v0 " 1.2 s. " a $9.8 m/s2

(Answer)

(b) What is the ball’s maximum height above its release point? Calculation: We can take the ball’s release point to be y0 " 0. We can then write Eq. 2-16 in y notation, set y $ y0 " y and v " 0 (at the maximum height), and solve for y.We get 0 $ (12 m/s)2 v 2 $ v 20 " " 7.3 m. y" 2a 2($9.8 m/s2)

(Answer)

(c) How long does the ball take to reach a point 5.0 m above its release point? Calculations: We know v0, a " $g, and displacement y $ y0 " 5.0 m, and we want t, so we choose Eq. 2-15. Rewriting it for y and setting y0 " 0 give us y " v0 t $ 12 gt 2,

During ascent, a = –g, speed decreases, and velocity becomes less positive

Figure 2-13 A pitcher tosses a baseball straight up into the air. The equations of free fall apply for rising as well as for falling objects, provided any effects from the air can be neglected.

or

During descent, a = –g, speed increases, and velocity becomes more negative y=0

5.0 m " (12 m/s)t $ (12)(9.8 m/s2)t 2.

If we temporarily omit the units (having noted that they are consistent), we can rewrite this as 4.9t 2 $ 12t # 5.0 " 0. Solving this quadratic equation for t yields t " 0.53 s and t " 1.9 s.

(Answer)

There are two such times! This is not really surprising because the ball passes twice through y " 5.0 m, once on the way up and once on the way down.

Additional examples, video, and practice available at WileyPLUS

2-6 G RAPH ICAL I NTEG RATION I N M OTION ANALYSIS

29

2-6 GRAPHICAL INTEGRATION IN MOTION ANALYSIS Learning Objectives After reading this module, you should be able to . . .

2.19 Determine a particle’s change in position by graphical integration on a graph of velocity versus time.

2.18 Determine a particle’s change in velocity by graphical integration on a graph of acceleration versus time.

Key Ideas ● On a graph of acceleration a versus time t, the change in

the velocity is given by v1 $ v0 "

"

t1

● On a graph of velocity v versus time t, the change in the

position is given by a dt.

x1 $ x0 "

t0

The integral amounts to finding an area on the graph:

"

t1

t0

a dt "

#

"

t1

v dt,

t0

where the integral can be taken from the graph as

"

$

area between acceleration curve . and time axis, from t0 to t1

t1

v dt "

t0

between velocity curve . #area and time axis, from t to t $ 0

1

Graphical Integration in Motion Analysis Integrating Acceleration. When we have a graph of an object’s acceleration a versus time t, we can integrate on the graph to find the velocity at any given time. Because a is defined as a " dv/dt, the Fundamental Theorem of Calculus tells us that v1 $ v0 "

"

t1

t0

(2-27)

a dt.

The right side of the equation is a definite integral (it gives a numerical result rather than a function), v0 is the velocity at time t0, and v1 is the velocity at later time t1.The definite integral can be evaluated from an a(t) graph,such as in Fig.2-14a.In particular,

"

t1

t0

a dt "

acceleration curve #areaandbetween $. time axis, from t to t 0

1

(2-28)

If a unit of acceleration is 1 m/s2 and a unit of time is 1 s, then the corresponding unit of area on the graph is (1 m/s2)(1 s) " 1 m/s, which is (properly) a unit of velocity. When the acceleration curve is above the time axis, the area is positive; when the curve is below the time axis, the area is negative. Integrating Velocity. Similarly, because velocity v is defined in terms of the position x as v " dx/dt, then x1 $ x0 "

"

t1

t0

"

t0

$

#

Area

(2-29)

v dt,

where x0 is the position at time t0 and x1 is the position at time t1. The definite integral on the right side of Eq. 2-29 can be evaluated from a v(t) graph, like that shown in Fig. 2-14b. In particular, t1

a

area between velocity curve v dt " and time axis, from t to t . 0 1

(2-30)

If the unit of velocity is 1 m/s and the unit of time is 1 s, then the corresponding unit of area on the graph is (1 m/s)(1 s) " 1 m, which is (properly) a unit of position and displacement. Whether this area is positive or negative is determined as described for the a(t) curve of Fig. 2-14a.

t0

t

t1

This area gives the change in velocity.

(a) v Area t0

t1

t

This area gives the change in position.

(b)

Figure 2-14 The area between a plotted curve and the horizontal time axis, from time t0 to time t1, is indicated for (a) a graph of acceleration a versus t and (b) a graph of velocity v versus t.

30

CHAPTE R 2 M OTION ALONG A STRAIG HT LI N E

Sample Problem 2.06 Graphical integration a versus t, whiplash injury “Whiplash injury” commonly occurs in a rear-end collision where a front car is hit from behind by a second car. In the 1970s, researchers concluded that the injury was due to the occupant’s head being whipped back over the top of the seat as the car was slammed forward. As a result of this finding, head restraints were built into cars, yet neck injuries in rearend collisions continued to occur. In a recent test to study neck injury in rear-end collisions, a volunteer was strapped to a seat that was then moved abruptly to simulate a collision by a rear car moving at 10.5 km/h. Figure 2-15a gives the accelerations of the volunteer’s torso and head during the collision, which began at time t " 0. The torso acceleration was delayed by 40 ms because during that time interval the seat back had to compress against the volunteer. The head acceleration was delayed by an additional 70 ms. What was the torso speed when the head began to accelerate?

Combining Eqs. 2-27 and 2-28, we can write v1 $ v0 "

Calculations: We know that the initial torso speed is v0 " 0 at time t0 " 0, at the start of the “collision.” We want the torso speed v1 at time t1 " 110 ms, which is when the head begins to accelerate.

(2-31)

areaA " 0. From 40 ms to 100 ms, region B has the shape of a triangle, with area areaB " 12(0.060 s)(50 m/s2) " 1.5 m/s. From 100 ms to 110 ms, region C has the shape of a rectangle, with area areaC " (0.010 s)(50 m/s2) " 0.50 m/s. Substituting these values and v0 " 0 into Eq. 2-31 gives us v1 " 2.0 m/s " 7.2 km/h.

(Answer)

Comments: When the head is just starting to move forward, the torso already has a speed of 7.2 km/h. Researchers argue that it is this difference in speeds during the early stage of a rear-end collision that injures the neck. The backward whipping of the head happens later and could, especially if there is no head restraint, increase the injury. a

100

50

Head 50

Torso

(b)

B

A 40

0

1

For convenience, let us separate the area into three regions (Fig. 2-15b). From 0 to 40 ms, region A has no area:

or

We can calculate the torso speed at any time by finding an area on the torso a(t) graph.

a (m/s2)

0

v1 $ 0 " 0 # 1.5 m/s # 0.50 m/s,

KEY IDEA

(a)

acceleration curve $. #areaandbetween time axis, from t to t

40

80 t (ms)

120

160

The total area gives the change in velocity.

C 100 110

t

Figure 2-15 (a) The a(t) curve of the torso and head of a volunteer in a simulation of a rear-end collision. (b) Breaking up the region between the plotted curve and the time axis to calculate the area.

Additional examples, video, and practice available at WileyPLUS

Review & Summary Position The position x of a particle on an x axis locates the particle with respect to the origin, or zero point, of the axis.The position is either positive or negative, according to which side of the origin the particle is on, or zero if the particle is at the origin. The positive direction on an axis is the direction of increasing positive numbers; the opposite direction is the negative direction on the axis.

Average Velocity When a particle has moved from position x1

Displacement The displacement -x of a particle is the change

The algebraic sign of vavg indicates the direction of motion (vavg is a vector quantity). Average velocity does not depend on the actual distance a particle moves, but instead depends on its original and final positions. On a graph of x versus t, the average velocity for a time interval -t is the slope of the straight line connecting the points on the curve that represent the two ends of the interval.

in its position: -x " x2 $ x1.

(2-1)

Displacement is a vector quantity. It is positive if the particle has moved in the positive direction of the x axis and negative if the particle has moved in the negative direction.

to position x2 during a time interval -t " t2 $ t1, its average velocity during that interval is vavg "

-x x $ x1 " 2 . -t t2 $ t1

(2-2)

31

QU ESTIONS

Average Speed The average speed savg of a particle during a time interval -t depends on the total distance the particle moves in that time interval: total distance (2-3) . savg " -t

Instantaneous Velocity The instantaneous velocity (or simply velocity) v of a moving particle is

a"

dv d 2x " . dt dt 2

(2-8, 2-9)

On a graph of v versus t, the acceleration a at any time t is the slope of the curve at the point that represents t.

Constant Acceleration The five equations in Table 2-1 describe the motion of a particle with constant acceleration:

-x dx " , v " lim -t : 0 -t dt

(2-4)

where -x and -t are defined by Eq. 2-2. The instantaneous velocity (at a particular time) may be found as the slope (at that particular time) of the graph of x versus t. Speed is the magnitude of instantaneous velocity.

Average Acceleration Average acceleration is the ratio of a change in velocity -v to the time interval -t in which the change occurs: aavg "

and the second time derivative of position x(t):

-v . -t

(2-7)

The algebraic sign indicates the direction of aavg.

Instantaneous Acceleration Instantaneous acceleration (or simply acceleration) a is the first time derivative of velocity v(t)

(2-11)

v " v0 # at, x $ x0 " v0t #

1 2 2 at ,

(2-15)

v2 " v20 # 2a(x $ x0), x $ x0 "

1 2 (v0

(2-16)

# v)t,

(2-17)

1 2 2 at .

(2-18)

x $ x0 " vt $

These are not valid when the acceleration is not constant.

Free-Fall Acceleration An important example of straightline motion with constant acceleration is that of an object rising or falling freely near Earth’s surface. The constant acceleration equations describe this motion, but we make two changes in notation: (1) we refer the motion to the vertical y axis with #y vertically up; (2) we replace a with $g, where g is the magnitude of the free-fall acceleration. Near Earth’s surface, g " 9.8 m/s2 (" 32 ft/s2).

Questions v

1 Figure 2-16 gives the velocity of a particle moving on an x axis. What are (a) the initial and (b) the final directions of travel? (c) Does the particle stop momentarily? (d) Is the acceleration positive or negative? (e) Is it constant or varying?

t

2 Figure 2-17 gives the acceleration a(t) of a Chihuahua as it chases Figure 2-16 Question 1. a German shepherd along an axis. In which of the time periods indicated does the Chihuahua move at constant speed? a

t

Figure 2-17 Question 2.

A B

C

3 Figure 2-18 shows four paths along which objects move from a starting point to a final point, all in the same time interval. The paths pass over a grid of equally spaced straight lines. Rank the paths according to (a) the average velocity of the objects and (b) the average speed of the objects, greatest first.

D

E

F

G

H

1 2 4

3

Figure 2-18 Question 3.

4 Figure 2-19 is a graph of a particle’s position along an x axis versus time. (a) At time t " 0, what

x

is the sign of the particle’s position? Is the particle’s velocity positive, negative, or 0 at (b) t " 1 s, (c) t " 2 s, and (d) t " 3 s? (e) How many times does the particle go through the point x " 0? 5 Figure 2-20 gives the velocity of a particle moving along an axis. Point 1 is at the highest point on the curve; point 4 is at the lowest point; and points 2 and 6 are at the same height. What is the direction of travel at (a) time t " 0 and (b) point 4? (c) At which of the six numbered points does the particle reverse its direction of travel? (d) Rank the six points according to the magnitude of the acceleration, greatest first.

0

1

2

t (s)

4

Figure 2-19 Question 4. v 1 2

6 3

t

5 4

Figure 2-20 Question 5.

v 6 At t " 0, a particle moving along an x axis is at position x0 " $20 m. The signs of the particle’s initial velocity v0 (at time t0) and constant acceleration a are, respectively, for four situations: (1) #, #; (2) #, $; (3) $, #; (4) $, $. In 0 which situations will the particle (a) stop momentarily, (b) pass through the origin, and (c) never pass through the origin?

7 Hanging over the railing of a bridge, you drop an egg (no initial velocity) as you throw a second egg downward. Which curves in Fig. 2-21

3

A

B

t

C D G

F

E

Figure 2-21 Question 7.

32

CHAPTE R 2 M OTION ALONG A STRAIG HT LI N E

give the velocity v(t) for (a) the dropped egg and (b) the thrown egg? (Curves A and B are parallel; so are C, D, and E; so are F and G.) 8 The following equations give the velocity v(t) of a particle in four situations: (a) v " 3; (b) v " 4t 2 # 2t $ 6; (c) v " 3t $ 4; (d) v " 5t 2 $ 3. To which of these situations do the equations of Table 2-1 apply?

2

11 Figure 2-23 shows that a particle moving along an x axis undergoes three periods of acceleration. Without written computation, rank the acceleration periods according to the increases they produce in the particle’s velocity, greatest first.

3 Acceleration a

9 In Fig. 2-22, a cream tangerine is thrown directly upward past three evenly spaced windows of equal heights. Rank the windows according to (a) the average speed of the cream tangerine while passing them, (b) the time the cream tangerine takes to pass them, (c) the magnitude of the acceleration of the cream tangerine while passing them, and (d) the change -v in the speed of the cream tangerine during the passage, greatest first.

1

apple’s release, the balloon is accelerating upward with a magnitude of 4.0 m/s2 and has an upward velocity of magnitude 2 m/s. What are the (a) magnitude and (b) direction of the acceleration of the apple just after it is released? (c) Just then, is the apple moving upward or downward, or is it stationary? (d) What is the magnitude of its velocity just then? (e) In the next few moments, does the speed of the apple increase, decrease, or remain constant?

Figure 2-22 Question 9.

10 Suppose that a passenger intent on lunch during his first ride in a hot-air balloon accidently drops an apple over the side during the balloon’s liftoff. At the moment of the

(3) (1) (2) Time t

Figure 2-23 Question 11.

Problems Tutoring problem available (at instructor’s discretion) in WileyPLUS and WebAssign SSM • – •••

Worked-out solution available in Student Solutions Manual

WWW Worked-out solution is at

Number of dots indicates level of problem difficulty

ILW

Interactive solution is at

http://www.wiley.com/college/halliday

Additional information available in The Flying Circus of Physics and at flyingcircusofphysics.com

Module 2-1 Position, Displacement, and Average Velocity •1 While driving a car at 90 km/h, how far do you move while your eyes shut for 0.50 s during a hard sneeze?

“Cogito ergo zoom!” (I think, therefore I go fast!). In 2001, Sam Whittingham beat Huber’s record by 19.0 km/h. What was Whittingham’s time through the 200 m?

•2 Compute your average velocity in the following two cases: (a) You walk 73.2 m at a speed of 1.22 m/s and then run 73.2 m at a speed of 3.05 m/s along a straight track. (b) You walk for 1.00 min at a speed of 1.22 m/s and then run for 1.00 min at 3.05 m/s along a straight track. (c) Graph x versus t for both cases and indicate how the average velocity is found on the graph.

••7 Two trains, each having a speed of 30 km/h, are headed at each other on the same straight track. A bird that can fly 60 km/h flies off the front of one train when they are 60 km apart and heads directly for the other train. On reaching the other train, the (crazy) bird flies directly back to the first train, and so forth. What is the total distance the bird travels before the trains collide?

•3 SSM WWW An automobile travels on a straight road for 40 km at 30 km/h. It then continues in the same direction for another 40 km at 60 km/h. (a) What is the average velocity of the car during the full 80 km trip? (Assume that it moves in the positive x direction.) (b) What is the average speed? (c) Graph x versus t and indicate how the average velocity is found on the graph.

••8 Panic escape. Figure 2-24 shows a general situation in which a stream of people attempt to escape through an exit door that turns out to be locked. The people move toward the door at speed vs " 3.50 m/s, are each d " 0.25 m in depth, and are separated by L " 1.75 m. The L L L arrangement in Fig. 2-24 occurs at time t " 0. (a) At what average rate does the layer of people at the door d d d increase? (b) At what time Locked does the layer’s depth reach door 5.0 m? (The answers reveal Figure 2-24 Problem 8. how quickly such a situation becomes dangerous.)

•4 A car moves uphill at 40 km/h and then back downhill at 60 km/h. What is the average speed for the round trip? •5 SSM The position of an object moving along an x axis is given by x " 3t $ 4t 2 # t 3, where x is in meters and t in seconds. Find the position of the object at the following values of t: (a) 1 s, (b) 2 s, (c) 3 s, and (d) 4 s. (e) What is the object’s displacement between t " 0 and t " 4 s? (f) What is its average velocity for the time interval from t " 2 s to t " 4 s? (g) Graph x versus t for 0 0 t 0 4 s and indicate how the answer for (f) can be found on the graph. •6 The 1992 world speed record for a bicycle (human-powered vehicle) was set by Chris Huber. His time through the measured 200 m stretch was a sizzling 6.509 s, at which he commented,

••9 ILW In 1 km races, runner 1 on track 1 (with time 2 min, 27.95 s) appears to be faster than runner 2 on track 2 (2 min, 28.15 s). However, length L2 of track 2 might be slightly greater than length L1 of track 1. How large can L2 $ L1 be for us still to conclude that runner 1 is faster?

PROB LE M S

••10 To set a speed record in a measured (straight-line) distance d, a race car must be driven first in one direction (in time t1) and then in the opposite direction (in time t2). (a) To eliminate the effects of the wind and obtain the car’s speed vc in a windless situation, should we find the average of d/t1 and d/t2 (method 1) or should we divide d by the average of t1 and t2? (b) What is the fractional difference in the two methods when a steady wind blows along the car’s route and the ratio of the wind speed vw to the car’s speed vc is 0.0240? ••11 You are to drive 300 km to an interview. The interview is at 11!15 A.M. You plan to drive at 100 km/h, so you leave at 8!00 A.M. to allow some extra time. You drive at that speed for the first 100 km, but then construction work forces you to slow to 40 km/h for 40 km. What would be the least speed needed for the rest of the trip to arrive in time for the interview? •••12 Traffic shock wave. An abrupt slowdown in concentrated traffic can travel as a pulse, termed a shock wave, along the line of cars, either downstream (in the traffic direction) or upstream, or it can be stationary. Figure 2-25 shows a uniformly spaced line of cars moving at speed v " 25.0 m/s toward a uniformly spaced line of slow cars moving at speed vs " 5.00 m/s. Assume that each faster car adds length L " 12.0 m (car length plus buffer zone) to the line of slow cars when it joins the line, and assume it slows abruptly at the last instant. (a) For what separation distance d between the faster cars does the shock wave remain stationary? If the separation is twice that amount, what are the (b) speed and (c) direction (upstream or downstream) of the shock wave? L

d

L

v

d

L

Car

L

Buffer

L

vs

Figure 2-25 Problem 12. •••13 ILW You drive on Interstate 10 from San Antonio to Houston, half the time at 55 km/h and the other half at 90 km/h. On the way back you travel half the distance at 55 km/h and the other half at 90 km/h. What is your average speed (a) from San Antonio to Houston, (b) from Houston back to San Antonio, and (c) for the entire trip? (d) What is your average velocity for the entire trip? (e) Sketch x versus t for (a), assuming the motion is all in the positive x direction. Indicate how the average velocity can be found on the sketch. Module 2-2 Instantaneous Velocity and Speed •14 An electron moving along the x axis has a position given by x " 16te$t m, where t is in seconds. How far is the electron from the origin when it momentarily stops? •15 (a) If a particle’s position is given by x " 4 $ 12t # 3t 2 (where t is in seconds and x is in meters), what is its velocity at t " 1 s? (b) Is it moving in the positive or negative direction of x just then? (c) What is its speed just then? (d) Is the speed increasing or decreasing just then? (Try answering the next two questions without further calculation.) (e) Is there ever an instant when the velocity is zero? If so, give the time t; if not, answer no. (f) Is there a time after t " 3 s when the particle is moving in the negative direction of x? If so, give the time t; if not, answer no. •16 The position function x(t) of a particle moving along an x axis is x " 4.0 $ 6.0t 2, with x in meters and t in seconds. (a) At what time and (b) where does the particle (momentarily) stop? At what (c) negative time and (d) positive time does the particle pass through the origin? (e) Graph x versus t for the range $5 s to #5 s. (f) To shift the curve rightward on the graph, should we include the

33

term #20t or the term $20t in x(t)? (g) Does that inclusion increase or decrease the value of x at which the particle momentarily stops? ••17 The position of a particle moving along the x axis is given in centimeters by x " 9.75 # 1.50t 3, where t is in seconds. Calculate (a) the average velocity during the time interval t " 2.00 s to t " 3.00 s; (b) the instantaneous velocity at t " 2.00 s; (c) the instantaneous velocity at t " 3.00 s; (d) the instantaneous velocity at t " 2.50 s; and (e) the instantaneous velocity when the particle is midway between its positions at t " 2.00 s and t " 3.00 s. (f) Graph x versus t and indicate your answers graphically. Module 2-3 Acceleration •18 The position of a particle moving along an x axis is given by x " 12t 2 $ 2t 3, where x is in meters and t is in seconds. Determine (a) the position, (b) the velocity, and (c) the acceleration of the particle at t " 3.0 s. (d) What is the maximum positive coordinate reached by the particle and (e) at what time is it reached? (f) What is the maximum positive velocity reached by the particle and (g) at what time is it reached? (h) What is the acceleration of the particle at the instant the particle is not moving (other than at t " 0)? (i) Determine the average velocity of the particle between t " 0 and t " 3 s. •19 SSM At a certain time a particle had a speed of 18 m/s in the positive x direction, and 2.4 s later its speed was 30 m/s in the opposite direction. What is the average acceleration of the particle during this 2.4 s interval? •20 (a) If the position of a particle is given by x " 20t $ 5t 3, where x is in meters and t is in seconds, when, if ever, is the particle’s velocity zero? (b) When is its acceleration a zero? (c) For what time range (positive or negative) is a negative? (d) Positive? (e) Graph x(t), v(t), and a(t). ••21 From t " 0 to t " 5.00 min, a man stands still, and from t " 5.00 min to t " 10.0 min, he walks briskly in a straight line at a constant speed of 2.20 m/s. What are (a) his average velocity vavg and (b) his average acceleration aavg in the time interval 2.00 min to 8.00 min? What are (c) vavg and (d) aavg in the time interval 3.00 min to 9.00 min? (e) Sketch x versus t and v versus t, and indicate how the answers to (a) through (d) can be obtained from the graphs. ••22 The position of a particle moving along the x axis depends on the time according to the equation x " ct 2 $ bt 3, where x is in meters and t in seconds.What are the units of (a) constant c and (b) constant b? Let their numerical values be 3.0 and 2.0, respectively. (c) At what time does the particle reach its maximum positive x position? From t " 0.0 s to t " 4.0 s, (d) what distance does the particle move and (e) what is its displacement? Find its velocity at times (f) 1.0 s, (g) 2.0 s, (h) 3.0 s, and (i) 4.0 s. Find its acceleration at times (j) 1.0 s, (k) 2.0 s, (l) 3.0 s, and (m) 4.0 s. Module 2-4 Constant Acceleration •23 SSM An electron with an initial velocity v0 " 1.50 ' 10 5 m/s enters a region of length L " 1.00 cm where it is electrically acceler- Nonaccelerating Accelerating region region ated (Fig. 2-26). It emerges with 6 v " 5.70 ' 10 m/s. What is its acL celeration, assumed constant? •24 Catapulting mushrooms. Certain mushrooms launch their spores by a catapult mechanism. As water condenses from the air onto a spore that is attached to

Path of electron

Figure 2-26 Problem 23.

CHAPTE R 2 M OTION ALONG A STRAIG HT LI N E

•25 An electric vehicle starts from rest and accelerates at a rate of 2.0 m/s2 in a straight line until it reaches a speed of 20 m/s. The vehicle then slows at a constant rate of 1.0 m/s2 until it stops. (a) How much time elapses from start to stop? (b) How far does the vehicle travel from start to stop? •26 A muon (an elementary particle) enters a region with a speed of 5.00 ' 10 6 m/s and then is slowed at the rate of 1.25 ' 1014 m/s2. (a) How far does the muon take to stop? (b) Graph x versus t and v versus t for the muon. •27 An electron has a constant acceleration of #3.2 m/s2. At a certain instant its velocity is #9.6 m/s. What is its velocity (a) 2.5 s earlier and (b) 2.5 s later? •28 On a dry road, a car with good tires may be able to brake with a constant deceleration of 4.92 m/s2. (a) How long does such a car, initially traveling at 24.6 m/s, take to stop? (b) How far does it travel in this time? (c) Graph x versus t and v versus t for the deceleration. •29 ILW A certain elevator cab has a total run of 190 m and a maximum speed of 305 m/min, and it accelerates from rest and then back to rest at 1.22 m/s2. (a) How far does the cab move while accelerating to full speed from rest? (b) How long does it take to make the nonstop 190 m run, starting and ending at rest? •30 The brakes on your car can slow you at a rate of 5.2 m/s2. (a) If you are going 137 km/h and suddenly see a state trooper, what is the minimum time in which you can get your car under the 90 km/h speed limit? (The answer reveals the futility of braking to keep your high speed from being detected with a radar or laser gun.) (b) Graph x versus t and v versus t for such a slowing. •31 SSM Suppose a rocket ship in deep space moves with constant acceleration equal to 9.8 m/s2, which gives the illusion of normal gravity during the flight. (a) If it starts from rest, how long will it take to acquire a speed one-tenth that of light, which travels at 3.0 ' 108 m/s? (b) How far will it travel in so doing? •32 A world’s land speed record was set by Colonel John P. Stapp when in March 1954 he rode a rocket-propelled sled that moved along a track at 1020 km/h. He and the sled were brought to a stop in 1.4 s. (See Fig. 2-7.) In terms of g, what acceleration did he experience while stopping? •33 SSM ILW A car traveling 56.0 km/h is 24.0 m from a barrier when the driver slams on the brakes. The car hits the barrier 2.00 s later. (a) What is the magnitude of the car’s constant acceleration before impact? (b) How fast is the car traveling at impact? ••34 In Fig. 2-27, a red car and a green car, identical except for the color, move toward each other in adjacent lanes and parallel to an x axis. At time t " 0, the red car is at xr " 0 and the green car is at xg " 220 m. If the red car has a constant velocity of 20 km/h, the cars pass each other at x " 44.5 m, and if it has a constant velocity of 40 km/h, they pass each other at x " 76.6 m. What are (a) the initial velocity and (b) the constant acceleration of the green car?

Green car

xr Red car

xg

x

Figure 2-27 Problems 34 and 35. ••35 Figure 2-27 shows a red car and a green car that move toward each other. Figure 2-28 is a graph of their motion, showing the positions xg0 " 270 m and xr0 " $35.0 m at time t " 0. The green car has a constant speed of 20.0 m/s and the red car begins from rest. What is the acceleration magnitude of the red car?

xg 0 x (m)

the mushroom, a drop grows on one side of the spore and a film grows on the other side. The spore is bent over by the drop’s weight, but when the film reaches the drop, the drop’s water suddenly spreads into the film and the spore springs upward so rapidly that it is slung off into the air. Typically, the spore reaches a speed of 1.6 m/s in a 5.0 mm launch; its speed is then reduced to zero in 1.0 mm by the air. Using those data and assuming constant accelerations, find the acceleration in terms of g during (a) the launch and (b) the speed reduction.

0 xr 0 0

12 t (s)

Figure 2-28 Problem 35.

••36 A car moves along an x axis through a distance of 900 m, starting at rest (at x " 0) and ending at rest (at x " 900 m). Through the first 14 of that distance, its acceleration is #2.25 m/s2. Through the rest of that distance, its acceleration is $0.750 m/s2. What are (a) its travel time through the 900 m and (b) its maximum speed? (c) Graph position x, velocity v, and acceleration a versus time t for the trip. ••37 Figure 2-29 depicts the motion x (m) of a particle moving along an x axis with a constant acceleration. The fig- xs ure’s vertical scaling is set by xs " 6.0 m. What are the (a) magnitude and (b) direction of the particle’s acceleration? ••38 (a) If the maximum acceleration t (s) that is tolerable for passengers in a 0 1 2 2 subway train is 1.34 m/s and subway stations are located 806 m apart, what Figure 2-29 Problem 37. is the maximum speed a subway train can attain between stations? (b) What is the travel time between stations? (c) If a subway train stops for 20 s at each station, what is the maximum average speed of the train, from one start-up to the next? (d) Graph x, v, and a versus t for the interval from one start-up to the next. ••39 Cars A and B move in xs the same direction in adjacent lanes.The position x of car A is given in Fig. 2-30, from time t " 0 to t " 7.0 s. The figure’s vertical scaling is set by xs " 32.0 m.At t " 0, car B is at x " 0 1 2 3 4 5 6 7 0, with a velocity of 12 m/s and t (s) a negative constant acceleraFigure 2-30 Problem 39. tion aB. (a) What must aB be such that the cars are (momentarily) side by side (momentarily at the same value of x) at t " 4.0 s? (b) For that value of aB, how many times are the cars side by side? (c) Sketch the position x of car B versus time t on Fig. 2-30. How many times will the cars be side by side if the magnitude of acceleration aB is (d) more than and (e) less than the answer to part (a)? x (m)

34

••40 You are driving toward a traffic signal when it turns yellow. Your speed is the legal speed limit of v0 " 55 km/h; your best deceleration rate has the magnitude a " 5.18 m/s2.Your best reaction time to begin braking is T " 0.75 s. To avoid having the front of your car enter the intersection after the light turns red, should you brake to a stop or continue to move at 55 km/h if the distance to

PROB LE M S

35

•46 Raindrops fall 1700 m from a cloud to the ground. (a) If they were not slowed by air resistance, how fast would the drops be moving when they struck the ground? (b) Would it be safe to walk outside during a rainstorm?

vs ••41 As two trains move along a track, their conductors suddenly notice that they are headed toward each other. 0 t (s) 2 4 6 Figure 2-31 gives their velocities v as functions of time t as the conductors slow the trains. The Figure 2-31 Problem 41. figure’s vertical scaling is set by vs " 40.0 m/s. The slowing processes begin when the trains are 200 m apart.What is their separation when both trains have stopped?

•47 SSM At a construction site a pipe wrench struck the ground with a speed of 24 m/s. (a) From what height was it inadvertently dropped? (b) How long was it falling? (c) Sketch graphs of y, v, and a versus t for the wrench.

•••42 You are arguing over a cell phone while trailing an unmarked police car by 25 m; both your car and the police car are traveling at 110 km/h. Your argument diverts your attention from the police car for 2.0 s (long enough for you to look at the phone and yell, “I won’t do that!”). At the beginning of that 2.0 s, the police officer begins braking suddenly at 5.0 m/s2. (a) What is the separation between the two cars when your attention finally returns? Suppose that you take another 0.40 s to realize your danger and begin braking. (b) If you too brake at 5.0 m/s2, what is your speed when you hit the police car? •••43 When a high-speed passenger train traveling at 161 km/h rounds a bend, the engineer is shocked to see that a locomotive has improperly entered onto the track from a siding and is a distance D " 676 m ahead (Fig. 2-32). The locomotive is moving at 29.0 km/h. The engineer of the high-speed train immediately applies the brakes. (a) What must be the magnitude of the resulting constant deceleration if a collision is to be just avoided? (b) Assume that the engineer is at x " 0 when, at t " 0, he first spots the locomotive. Sketch x(t) curves for the locomotive and high-speed train for the cases in which a collision is just avoided and is not quite avoided.

D High-speed train

Locomotive

Figure 2-32 Problem 43. Module 2-5 Free-Fall Acceleration •44 When startled, an armadillo will leap upward. Suppose it rises 0.544 m in the first 0.200 s. (a) What is its initial speed as it leaves the ground? (b) What is its speed at the height of 0.544 m? (c) How much higher does it go? •45 SSM WWW (a) With what speed must a ball be thrown vertically from ground level to rise to a maximum height of 50 m? (b) How long will it be in the air? (c) Sketch graphs of y, v, and a versus t for the ball. On the first two graphs, indicate the time at which 50 m is reached.

•48 A hoodlum throws a stone vertically downward with an initial speed of 12.0 m/s from the roof of a building, 30.0 m above the ground. (a) How long does it take the stone to reach the ground? (b) What is the speed of the stone at impact? •49 SSM A hot-air balloon is ascending at the rate of 12 m/s and is 80 m above the ground when a package is dropped over the side. (a) How long does the package take to reach the ground? (b) With what speed does it hit the ground? ••50 At time t " 0, apple 1 is dropped from a bridge onto a roadway beneath the bridge; somewhat later, apple 2 is thrown down from the same height. Figure 2-33 gives the vertical positions y of the apples versus t during the falling, until both apples have hit the roadway. The scaling is set by ts " 2.0 s. With approximately what speed is apple 2 thrown down?

y

v (m/s)

the intersection and the duration of the yellow light are (a) 40 m and 2.8 s, and (b) 32 m and 1.8 s? Give an answer of brake, continue, either (if either strategy works), or neither (if neither strategy works and the yellow duration is inappropriate).

0

ts

Figure 2-33 Problem 50. ••51 As a runaway scientific bal- v loon ascends at 19.6 m/s, one of its instrument packages breaks free of a harness and free-falls. Figure 2-34 0 t (s) 2 4 6 8 gives the vertical velocity of the package versus time, from before it breaks free to when it reaches the ground. (a) What maximum height above the break-free point does it Figure 2-34 Problem 51. rise? (b) How high is the break-free point above the ground? ••52 A bolt is dropped from a bridge under construction, falling 90 m to the valley below the bridge. (a) In how much time does it pass through the last 20% of its fall? What is its speed (b) when it begins that last 20% of its fall and (c) when it reaches the valley beneath the bridge? ••53 SSM ILW A key falls from a bridge that is 45 m above the water. It falls directly into a model boat, moving with constant velocity, that is 12 m from the point of impact when the key is released. What is the speed of the boat? ••54 A stone is dropped into a river from a bridge 43.9 m above the water. Another stone is thrown vertically down 1.00 s after the first is dropped. The stones strike the water at the same time. (a) What is the initial speed of the second stone? (b) Plot velocity versus time on a graph for each stone, taking zero time as the instant the first stone is released.

CHAPTE R 2 M OTION ALONG A STRAIG HT LI N E

••55 SSM A ball of moist clay falls 15.0 m to the ground. It is in contact with the ground for 20.0 ms before stopping. (a) What is the magnitude of the average acceleration of the ball during the time it is in contact with the ground? (Treat the ball as a particle.) (b) Is the average acceleration up or down? v

vA

••66 In a forward punch in karate, the fist begins at rest at the waist and is brought rapidly forward until the arm is fully extended. The speed v(t) of the fist is given in Fig. 2-37 for someone skilled in karate. The vertical scaling is set by vs " 8.0 m/s. How far has the fist moved at (a) time t " 50 ms and (b) when the speed of the fist is maximum?

1 __ v 3 A

••58 An object falls a distance h from rest. If it travels 0.50h in the last 1.00 s, find (a) the time and (b) the height of its fall. (c) Explain the physically unacceptable solution of the quadratic equation in t that you obtain. ••59 Water drips from the nozzle of a shower onto the floor 200 cm below. The drops fall at regular (equal) intervals of time, the first drop striking the floor at the instant the fourth drop begins to fall. When the first drop strikes the floor, how far below the nozzle are the (a) second and (b) third drops? ••60 A rock is thrown vertically upward from ground level at time t " 0.At t " 1.5 s it passes the top of a tall tower, and 1.0 s later it reaches its maximum height.What is the height of the tower? •••61 A steel ball is dropped from a building’s roof and passes a window, taking 0.125 s to fall from the top to the bottom of the window, a distance of 1.20 m. It then falls to a sidewalk and bounces back past the window, moving from bottom to top in 0.125 s. Assume that the upward flight is an exact reverse of the fall. The time the ball spends below the bottom of the window is 2.00 s. How tall is the building? •••62 A basketball player grabbing a rebound jumps 76.0 cm vertically. How much total time (ascent and descent) does the player spend (a) in the top 15.0 cm of this jump and (b) in the bottom 15.0 cm? (The player seems to hang in the air at the top.) •••63 A drowsy cat spots a flowerpot that sails first up and then down past an open window. The pot is in view for a total of 0.50 s, and the top-to-bottom height of the window is 2.00 m. How high above the window top does the flowerpot go? y

50

t (ms)

100

140

Figure 2-37 Problem 66. ••67 When a soccer as ball is kicked toBare ward a player and the player deflects the ball by “headHelmet ing” it, the acceleration of the head dur2 4 6 0 ing the collision can t (ms) be significant. Figure 2-38 gives the measFigure 2-38 Problem 67. ured acceleration a(t) of a soccer player’s head for a bare head and a helmeted head, starting from rest. The scaling on the vertical axis is set by as " 200 m/s2. At time t " 7.0 ms, what is the difference in the speed acquired by the bare head and the speed acquired by the helmeted head? ••68 A salamander of the genus Hydromantes captures prey by launching its tongue as a projectile: The skeletal a2 part of the tongue is shot forward, unfolding the rest of the tongue, until the outer a1 portion lands on the prey, sticking to it. Figure 2-39 0 10 20 30 40 shows the acceleration magt (ms) nitude a versus time t for the Figure 2-39 Problem 68. acceleration phase of the launch in a typical situation. The indicated accelerations are vs a2 " 400 m/s2 and a1 " 100 m/s2. What is the outward speed of the tongue at the end of the acceleration phase? v (m/s)

y (m)

s

0

a (m/s2)

0

d ••57 To test the quality yB yA of a tennis ball, you drop it onto the floor from a Figure 2-35 Problem 56. height of 4.00 m. It rebounds to a height of 2.00 m. If the ball is in contact with the floor for 12.0 ms, (a) what is the magnitude of its average acceleration during that contact and (b) is the average acceleration up or down?

•••64 A ball is shot vertically upward from the surface of another planet. A plot of y versus t for the ball is shown in Fig. 2-36, where y is the height of the ball above its starting point and t " 0 at the instant the ball is shot. The figure’s vertical scaling is set by ys " 30.0 m. What are the magnitudes of (a) the free-fall acceleration on the planet and (b) the initial velocity of the ball?

vs

y

a (m/s2)

••56 Figure 2-35 shows the speed v versus height y of a ball tossed directly upward, along a y axis. Distance d is 0.40 m. The speed at height yA is vA. The speed at height yB is 13vA. What is speed vA?

Module 2-6 Graphical Integration in Motion Analysis •65 Figure 2-15a gives the acceleration of a volunteer’s head and torso during a rear-end collision. At maximum head acceleration, what is the speed of (a) the head and (b) the torso?

v (m/s)

36

0 0

1

2 3 t (s)

4

Figure 2-36 Problem 64.

5

••69 ILW How far does the runner whose velocity–time graph is shown in Fig. 2-40 travel in 16 s? The figure’s vertical scaling is set by vs " 8.0 m/s.

0

4

8 t (s)

12

Figure 2-40 Problem 69.

16

37

PROB LE M S

Additional Problems 71 In an arcade video game, a spot is programmed to move across the screen according to x " 9.00t $ 0.750t 3, where x is distance in centimeters measured from the left edge of the screen and t is time in seconds. When the spot reaches a screen edge, at either x " 0 or x " 15.0 cm, t is reset to 0 and the spot starts moving again according to x(t). (a) At what time after starting is the spot instantaneously at rest? (b) At what value of x does this occur? (c) What is the spot’s acceleration (including sign) when this occurs? (d) Is it moving right or left just prior to coming to rest? (e) Just after? (f) At what time t . 0 does it first reach an edge of the screen? 72 A rock is shot vertically upward from the edge of the top of a tall building. The rock reaches its maximum height above the top of the building 1.60 s after being shot. Then, after barely missing the edge of the building as it falls downward, the rock strikes the ground 6.00 s after it is launched. In SI units: (a) with what upward velocity is the rock shot, (b) what maximum height above the top of the building is reached by the rock, and (c) how tall is the building? 73 At the instant the traffic light turns green, an automobile starts with a constant acceleration a of 2.2 m/s2. At the same instant a truck, traveling with a constant speed of 9.5 m/s, overtakes and passes the automobile. (a) How far beyond the traffic signal will the automobile overtake the truck? (b) How fast will the automobile be traveling at that instant? 74 A pilot flies horizontally at 1300 km/h, at height h " 35 m above initially level ground. However, at time t " 0, the pilot begins to fly over ground sloping upward at angle u " 4.3° (Fig. 2-41). If the pilot does not change the airplane’s heading, at what time t does the plane strike the ground? θ h

Figure 2-41 Problem 74. 75 To stop a car, first you require a certain reaction time to begin braking; then the car slows at a constant rate. Suppose that the total distance moved by your car during these two phases is 56.7 m when its initial speed is 80.5 km/h, and 24.4 m when its initial speed is 48.3 km/h. What are (a) your reaction time and (b) the magnitude of the acceleration? 76 Figure 2-42 shows part of a street where traffic flow is to be controlled to allow a platoon of cars to move smoothly along the street. Suppose that the platoon leaders have just ONE WAY

2

1 D12

Figure 2-42 Problem 76.

3 D23

reached intersection 2, where the green appeared when they were distance d from the intersection. They continue to travel at a certain speed vp (the speed limit) to reach intersection 3, where the green appears when they are distance d from it. The intersections are separated by distances D23 and D12. (a) What should be the time delay of the onset of green at intersection 3 relative to that at intersection 2 to keep the platoon moving smoothly? Suppose, instead, that the platoon had been stopped by a red light at intersection 1. When the green comes on there, the leaders require a certain time tr to respond to the change and an additional time to accelerate at some rate a to the cruising speed vp. (b) If the green at intersection 2 is to appear when the leaders are distance d from that intersection, how long after the light at intersection 1 turns green should the light at intersection 2 turn green? 77 SSM A hot rod can accelerate from 0 to 60 km/h in 5.4 s. (a) What is its average acceleration, in m/s2, during this time? (b) How far will it travel during the 5.4 s, assuming its acceleration is constant? (c) From rest, how much time would it require to go a distance of 0.25 km if its acceleration could be maintained at the value in (a)? 78 A red train traveling at 72 km/h and a green train traveling at 144 km/h are headed toward each other along a straight, level track. When they are 950 m apart, each engineer sees the other’s train and applies the brakes. The brakes slow each train at the rate of 1.0 m/s2. Is there a collision? If so, answer yes and give the speed of the red train and the speed of the green train at impact, respectively. If not, answer no and give the separation between the trains when they stop. 79 At time t " 0, a rock climber accidentally allows a piton to fall freely from a high point on the rock wall to the valley below him. Then, after a short delay, his climbing partner, who is 10 m higher on the 0 1 2 wall, throws a piton downt (s) ward. The positions y of the pitons versus t during the Figure 2-43 Problem 79. falling are given in Fig. 2-43. With what speed is the second piton thrown? y

•••70 Two particles move along an x axis. The position of particle 1 is given by x " 6.00t2 # 3.00t # 2.00 (in meters and seconds); the acceleration of particle 2 is given by a " $8.00t (in meters per second squared and seconds) and, at t " 0, its velocity is 20 m/s. When the velocities of the particles match, what is their velocity?

3

80 A train started from rest and moved with constant acceleration. At one time it was traveling 30 m/s, and 160 m farther on it was traveling 50 m/s. Calculate (a) the acceleration, (b) the time required to travel the 160 m mentioned, (c) the time required to attain the speed of 30 m/s, and (d) the distance moved from rest to the time the train had a speed of 30 m/s. (e) Graph x versus t and v versus t for the train, from rest. 81 SSM A particle’s acceleration along an x axis is a " 5.0t, with t in seconds and a in meters per second squared. At t " 2.0 s, a (m/s2) its velocity is #17 m/s. What is as its velocity at t " 4.0 s? 82 Figure 2-44 gives the acceleration a versus time t for a particle moving along an x axis. The a-axis scale is set by as " 12.0 m/s2. At t " $2.0 s, –2 the particle’s velocity is 7.0 m/s. What is its velocity at t " 6.0 s?

0

2

4

Figure 2-44 Problem 82.

6

t (s)

38

CHAPTE R 2 M OTION ALONG A STRAIG HT LI N E

83 Figure 2-45 shows a simple device for measuring your reaction time. It consists of a cardboard strip marked with a scale and two large dots. A friend holds the strip vertically, with thumb and forefinger at the dot on the right in Fig. 2-45. You then position your thumb and forefinger at the other dot (on the left in Fig. 2-45), being careful not to touch the strip. Your friend releases the strip, and you try to pinch it as soon as possible after you see it begin to fall. The mark at the place where you pinch the strip gives your reaction time. (a) How far from the lower dot should you place the 50.0 ms mark? How much higher should you place the marks for (b) 100, (c) 150, (d) 200, and (e) 250 ms? (For example, should the 100 ms marker be 2 times as far from the dot as the 50 ms marker? If so, give an answer of 2 times. Can you find any pattern in the answers?) Reaction time (ms)

91 A rock is dropped from a 100-m-high cliff. How long does it take to fall (a) the first 50 m and (b) the second 50 m? 92 Two subway stops are separated by 1100 m. If a subway train accelerates at #1.2 m/s2 from rest through the first half of the distance and decelerates at $1.2 m/s2 through the second half, what are (a) its travel time and (b) its maximum speed? (c) Graph x, v, and a versus t for the trip. 93 A stone is thrown vertically upward. On its way up it passes point A with speed v, and point B, 3.00 m higher than A, with speed 1 2 v. Calculate (a) the speed v and (b) the maximum height reached by the stone above point B. 94 A rock is dropped (from rest) from the top of a 60-m-tall building. How far above the ground is the rock 1.2 s before it reaches the ground?

84 A rocket-driven sled running on a straight, level track is used to investigate the effects of large accelerations on humans. One such sled can attain a speed of 1600 km/h in 1.8 s, starting from rest. Find (a) the acceleration (assumed constant) in terms of g and (b) the distance traveled. 85 A mining cart is pulled up a hill at 20 km/h and then pulled back down the hill at 35 km/h through its original level. (The time required for the cart’s reversal at the top of its climb is negligible.) What is the average speed of the cart for its round trip, from its original level back to its original level? 86 A motorcyclist who is moving along an x axis directed toward the east has an acceleration given by a " (6.1 $ 1.2t) m/s2 for 0 0 t 0 6.0 s. At t " 0, the velocity and position of the cyclist are 2.7 m/s and 7.3 m. (a) What is the maximum speed achieved by the cyclist? (b) What total distance does the cyclist travel between t " 0 and 6.0 s? 87 SSM When the legal speed limit for the New York Thruway was increased from 55 mi/h to 65 mi/h, how much time was saved by a motorist who drove the 700 km between the Buffalo entrance and the New York City exit at the legal speed limit? 88 A car moving with constant acceleration covered the distance between two points 60.0 m apart in 6.00 s. Its speed as it passed the second point was 15.0 m/s. (a) What was the speed at the first point? (b) What was the magnitude of the acceleration? (c) At what prior distance from the first point was the car at rest? (d) Graph x versus t and v versus t for the car, from rest (t " 0). 89 SSM A certain juggler usually tosses balls vertically to a height H. To what height must they be tossed if they are to spend twice as much time in the air?

v (m/s)

vs

95 SSM An iceboat has a constant velocity toward the east when a sudden gust of wind causes the iceboat to have a constant acceleration toward the east for a period of 3.0 s. A plot of x versus t is shown in Fig. 2-47, where t " 0 is taken to be the instant the wind starts to blow and the positive x axis is toward the east. (a) What is the acceleration of the iceboat during the 3.0 s interval? (b) What is the velocity of the iceboat at the end of the 3.0 s interval? (c) If the acceleration remains constant for an additional 3.0 s, how far does the iceboat travel during this second 3.0 s interval?

x (m)

250

200

150

100

50 0

Figure 2-45 Problem 83.

90 A particle starts from the origin at t " 0 and moves along the positive x axis. A graph of the velocity of the particle as a function of the time is shown in Fig. 2-46; the v-axis scale is set by vs " 4.0 m/s. (a) What is the coordinate of the particle at t " 5.0 s? (b) What is the velocity of the particle at t " 5.0 s? (c) What is

the acceleration of the particle at t " 5.0 s? (d) What is the average velocity of the particle between t " 1.0 s and t " 5.0 s? (e) What is the average acceleration of the particle between t " 1.0 s and t " 5.0 s?

30 25 20 15 10 5 0 0

0.5

1

1.5 t (s)

2

2.5

3

Figure 2-47 Problem 95. 96 A lead ball is dropped in a lake from a diving board 5.20 m above the water. It hits the water with a certain velocity and then sinks to the bottom with this same constant velocity. It reaches the bottom 4.80 s after it is dropped. (a) How deep is the lake? What are the (b) magnitude and (c) direction (up or down) of the average velocity of the ball for the entire fall? Suppose that all the water is drained from the lake. The ball is now thrown from the diving board so that it again reaches the bottom in 4.80 s. What are the (d) magnitude and (e) direction of the initial velocity of the ball? 97 The single cable supporting an unoccupied construction elevator breaks when the elevator is at rest at the top of a 120-m-high building. (a) With what speed does the elevator strike the ground? (b) How long is it falling? (c) What is its speed when it passes the halfway point on the way down? (d) How long has it been falling when it passes the halfway point? 98 Two diamonds begin a free fall from rest from the same height, 1.0 s apart. How long after the first diamond begins to fall will the two diamonds be 10 m apart?

0

1

2

3 4 t (s)

5

Figure 2-46 Problem 90.

6

99 A ball is thrown vertically downward from the top of a 36.6m-tall building. The ball passes the top of a window that is 12.2 m above the ground 2.00 s after being thrown. What is the speed of the ball as it passes the top of the window?

PROB LE M S

100 A parachutist bails out and freely falls 50 m. Then the parachute opens, and thereafter she decelerates at 2.0 m/s2. She reaches the ground with a speed of 3.0 m/s. (a) How long is the parachutist in the air? (b) At what height does the fall begin? 101 A ball is thrown down vertically with an initial speed of v0 from a height of h. (a) What is its speed just before it strikes the ground? (b) How long does the ball take to reach the ground? What would be the answers to (c) part a and (d) part b if the ball were thrown upward from the same height and with the same initial speed? Before solving any equations, decide whether the answers to (c) and (d) should be greater than, less than, or the same as in (a) and (b). 102 The sport with the fastest moving ball is jai alai, where measured speeds have reached 303 km/h. If a professional jai alai player faces a ball at that speed and involuntarily blinks, he blacks out the scene for 100 ms. How far does the ball move during the blackout? 103 If a baseball pitcher throws a fastball at a horizontal speed of 160 km/h, how long does the ball take to reach home plate 18.4 m away? 104 A proton moves along the x axis according to the equation x " 50t # 10t 2, where x is in meters and t is in seconds. Calculate (a) the average velocity of the proton during the first 3.0 s of its motion, (b) the instantaneous velocity of the proton at t " 3.0 s, and (c) the instantaneous acceleration of the proton at t " 3.0 s. (d) Graph x versus t and indicate how the answer to (a) can be obtained from the plot. (e) Indicate the answer to (b) on the graph. (f) Plot v versus t and indicate on it the answer to (c). 105 A motorcycle is moving at 30 m/s when the rider applies the brakes, giving the motorcycle a constant deceleration. During the 3.0 s interval immediately after braking begins, the speed decreases to 15 m/s. What distance does the motorcycle travel from the instant braking begins until the motorcycle stops? 106 A shuffleboard disk is accelerated at a constant rate from rest to a speed of 6.0 m/s over a 1.8 m distance by a player using a cue. At this point the disk loses contact with the cue and slows at a constant rate of 2.5 m/s2 until it stops. (a) How much time elapses from when the disk begins to accelerate until it stops? (b) What total distance does the disk travel? 107 The head of a rattlesnake can accelerate at 50 m/s2 in striking a victim. If a car could do as well, how long would it take to reach a speed of 100 km/h from rest? 108 A jumbo jet must reach a speed of 360 km/h on the runway for takeoff. What is the lowest constant acceleration needed for takeoff from a 1.80 km runway? 109 An automobile driver increases the speed at a constant rate from 25 km/h to 55 km/h in 0.50 min. A bicycle rider speeds up at a constant rate from rest to 30 km/h in 0.50 min. What are the magnitudes of (a) the driver’s acceleration and (b) the rider’s acceleration? 110 On average, an eye blink lasts about 100 ms. How far does a MiG-25 “Foxbat” fighter travel during a pilot’s blink if the plane’s average velocity is 3400 km/h? 111 A certain sprinter has a top speed of 11.0 m/s. If the sprinter starts from rest and accelerates at a constant rate, he is able to reach his top speed in a distance of 12.0 m. He is then able to maintain this top speed for the remainder of a 100 m race. (a) What is his time for the 100 m race? (b) In order to improve his time, the sprinter tries to decrease the distance required for him to reach his

39

top speed. What must this distance be if he is to achieve a time of 10.0 s for the race? 112 The speed of a bullet is measured to be 640 m/s as the bullet emerges from a barrel of length 1.20 m.Assuming constant acceleration, find the time that the bullet spends in the barrel after it is fired. 113 The Zero Gravity Research Facility at the NASA Glenn Research Center includes a 145 m drop tower.This is an evacuated vertical tower through which, among other possibilities, a 1-m-diameter sphere containing an experimental package can be dropped. (a) How long is the sphere in free fall? (b) What is its speed just as it reaches a catching device at the bottom of the tower? (c) When caught, the sphere experiences an average deceleration of 25g as its speed is reduced to zero.Through what distance does it travel during the deceleration? 114 A car can be braked to a stop from the autobahn-like speed of 200 km/h in 170 m. Assuming the acceleration is constant, find its magnitude in (a) SI units and (b) in terms of g. (c) How much time Tb is required for the braking? Your reaction time Tr is the time you require to perceive an emergency, move your foot to the brake, and begin the braking. If Tr " 400 ms, then (d) what is Tb in terms of Tr, and (e) is most of the full time required to stop spent in reacting or braking? Dark sunglasses delay the visual signals sent from the eyes to the visual cortex in the brain, increasing Tr. (f) In the extreme case in which Tr is increased by 100 ms, how much farther does the car travel during your reaction time? 115 In 1889, at Jubbulpore, India, a tug-of-war was finally won after 2 h 41 min, with the winning team displacing the center of the rope 3.7 m. In centimeters per minute, what was the magnitude of the average velocity of that center point during the contest? 116 Most important in an investigation of an airplane crash by the U.S. National Transportation Safety Board is the data stored on the airplane’s flight-data recorder, commonly called the “black box” in spite of its orange coloring and reflective tape. The recorder is engineered to withstand a crash with an average deceleration of magnitude 3400g during a time interval of 6.50 ms. In such a crash, if the recorder and airplane have zero speed at the end of that time interval, what is their speed at the beginning of the interval? 117 From January 26, 1977, to September 18, 1983, George Meegan of Great Britain walked from Ushuaia, at the southern tip of South America, to Prudhoe Bay in Alaska, covering 30 600 km. In meters per second, what was the magnitude of his average velocity during that time period? 118 The wings on a stonefly do not flap, and thus the insect cannot fly. However, when the insect is on a water surface, it can sail across the surface by lifting its wings into a breeze. Suppose that you time stoneflies as they move at constant speed along a straight path of a certain length. On average, the trips each take 7.1 s with the wings set as sails and 25.0 s with the wings tucked in. (a) What is the ratio of the sailing speed vs to the nonsailing speed vns? (b) In terms of vs, what is the difference in the times the insects take to travel the first 2.0 m along the path with and without sailing? 119

The position of a particle as it moves along a y axis is given by y " (2.0 cm) sin (pt/4),

with t in seconds and y in centimeters. (a) What is the average velocity of the particle between t " 0 and t " 2.0 s? (b) What is the instantaneous velocity of the particle at t " 0, 1.0, and 2.0 s? (c) What is the average acceleration of the particle between t " 0 and t " 2.0 s? (d) What is the instantaneous acceleration of the particle at t " 0, 1.0, and 2.0 s?

C

H

A

P

T

E

R

3

Vectors 3-1 VECTORS AND THEIR COMPONENTS

Learning Objectives

After reading this module, you should be able to . . .

3.01 Add vectors by drawing them in head-to-tail arrangements, applying the commutative and associative laws. 3.02 Subtract a vector from a second one. 3.03 Calculate the components of a vector on a given coordinate system, showing them in a drawing.

3.04 Given the components of a vector, draw the vector and determine its magnitude and orientation. 3.05 Convert angle measures between degrees and radians.

Key Ideas ● Scalars, such as temperature, have magnitude only. They are specified by a number with a unit (10°C) and obey the rules of arithmetic and ordinary algebra. Vectors, such as displacement, have both magnitude and direction (5 m, north) and obey the rules of vector algebra. ● Two vectors

vector : a along the coordinate axes are found by dropping perpendicular lines from the ends of : a onto the coordinate axes. The components are given by ax " a cos u

:

a and b may be added geometrically by drawing them to a common scale and placing them head to tail. The vector connecting the tail of the first to the head of the : second is the vector sum : s . To subtract b from : a , reverse the : : : direction of b to get $ b ; then add $ b to : a . Vector addition is commutative and obeys the associative law. :

● The (scalar) components ax and ay of any two-dimensional

and

ay " a sin u,

where u is the angle between the positive direction of the x axis and the direction of : a . The algebraic sign of a component indicates its direction along the associated axis. Given its components, we can find the magnitude and orientation of the vector : a with ay . a " 2a 2x # a 2y and tan 1 " ax

What Is Physics? Physics deals with a great many quantities that have both size and direction, and it needs a special mathematical language — the language of vectors — to describe those quantities. This language is also used in engineering, the other sciences, and even in common speech. If you have ever given directions such as “Go five blocks down this street and then hang a left,” you have used the language of vectors. In fact, navigation of any sort is based on vectors, but physics and engineering also need vectors in special ways to explain phenomena involving rotation and magnetic forces, which we get to in later chapters. In this chapter, we focus on the basic language of vectors.

Vectors and Scalars A particle moving along a straight line can move in only two directions. We can take its motion to be positive in one of these directions and negative in the other. For a particle moving in three dimensions, however, a plus sign or minus sign is no longer enough to indicate a direction. Instead, we must use a vector. 40

3-1 VECTORS AN D TH E I R COM PON E NTS

A vector has magnitude as well as direction, and vectors follow certain (vector) rules of combination, which we examine in this chapter. A vector quantity is a quantity that has both a magnitude and a direction and thus can be represented with a vector. Some physical quantities that are vector quantities are displacement, velocity, and acceleration. You will see many more throughout this book, so learning the rules of vector combination now will help you greatly in later chapters. Not all physical quantities involve a direction. Temperature, pressure, energy, mass, and time, for example, do not “point” in the spatial sense. We call such quantities scalars, and we deal with them by the rules of ordinary algebra. A single value, with a sign (as in a temperature of $40°F), specifies a scalar. The simplest vector quantity is displacement, or change of position. A vector that represents a displacement is called, reasonably, a displacement vector. (Similarly, we have velocity vectors and acceleration vectors.) If a particle changes its position by moving from A to B in Fig. 3-1a, we say that it undergoes a displacement from A to B, which we represent with an arrow pointing from A to B.The arrow specifies the vector graphically. To distinguish vector symbols from other kinds of arrows in this book, we use the outline of a triangle as the arrowhead. In Fig. 3-1a, the arrows from A to B, from A/ to B/, and from A2 to B2 have the same magnitude and direction. Thus, they specify identical displacement vectors and represent the same change of position for the particle. A vector can be shifted without changing its value if its length and direction are not changed. The displacement vector tells us nothing about the actual path that the particle takes. In Fig. 3-1b, for example, all three paths connecting points A and B correspond to the same displacement vector, that of Fig. 3-1a. Displacement vectors represent only the overall effect of the motion, not the motion itself.

41

B'

A' B" B A" A

(a)

B

A

(b)

Figure 3-1 (a) All three arrows have the same magnitude and direction and thus represent the same displacement. (b) All three paths connecting the two points correspond to the same displacement vector.

Adding Vectors Geometrically Suppose that, as in the vector diagram of Fig. 3-2a, a particle moves from A to B and then later from B to C. We can represent its overall displacement (no matter what its actual path) with two successive displacement vectors, AB and BC. The net displacement of these two displacements is a single displacement from A to C. We call AC the vector sum (or resultant) of the vectors AB and BC. This sum is not the usual algebraic sum. In Fig. 3-2b, we redraw the vectors of Fig. 3-2a and relabel them in the way that we shall use from now on, namely, with an arrow over an italic symbol, as in : a . If we want to indicate only the magnitude of the vector (a quantity that lacks a sign or direction), we shall use the italic symbol, as in a, b, and s. (You can use just a handwritten symbol.) A symbol with an overhead arrow always implies both properties of a vector, magnitude and direction. We can represent the relation among the three vectors in Fig. 3-2b with the vector equation :

(3-1)

s " a # b,

:

:

B Actual path C Net displacement is the vector sum

A

(a)

b

a

To add a and b, draw them head to tail.

:

which says that the vector : s is the vector sum of vectors : a and b . The symbol # in Eq. 3-1 and the words “sum” and “add” have different meanings for vectors than they do in the usual algebra because they involve both magnitude and direction. : Figure 3-2 suggests a procedure for adding two-dimensional vectors : a and b : geometrically. (1) On paper, sketch vector a to some convenient scale and at the : proper angle. (2) Sketch vector b to the same scale, with its tail at the head of vec: tor a , again at the proper angle. (3) The vector sum : s is the vector that extends : from the tail of : a to the head of b . Properties. Vector addition, defined in this way, has two important proper: ties. First, the order of addition does not matter. Adding : a to b gives the same

s

(b)

This is the resulting vector, from tail of a to head of b.

Figure 3-2 (a) AC is the vector sum of the vectors AB and BC. (b) The same vectors relabeled.

42

CHAPTE R 3 VECTORS :

result as adding b to : a (Fig. 3-3); that is,

b

a

Vector sum

a+b

Start

:

:

:

(commutative law).

a#b"b#: a

(3-2)

Finish

b+a a

b

You get the same vector result for either order of adding vectors.

Second, when there are more than two vectors, we can group them in any order : : as we add them. Thus, if we want to add vectors : a , b , and : c , we can add : a and b : first and then add their vector sum to : c . We can also add b and : c first and then : add that sum to a . We get the same result either way, as shown in Fig. 3-4. That is, :

:

: (a # b) # : c ": a # (b # : c)

b a

a

a+b

a+b c

c

+ (a

a+

a+

b+c

b+c

(3-3)

You get the same vector result for any order of adding the vectors.

:

Figure 3-3 The two vectors : a and b can be added in either order; see Eq. 3-2.

(associative law).

c

c)

c

+ b)

b+

+ (b

:

Figure 3-4 The three vectors : a , b , and : c can be grouped in any way as they are added; see Eq. 3-3. :

:

The vector $b is a vector with the same magnitude as b but the opposite direction (see Fig. 3-5). Adding the two vectors in Fig. 3-5 would yield

–b

:

:

b # ($b ) " 0.

b :

:

:

Figure 3-5 The vectors b and $b have the same magnitude and opposite directions.

:

Thus, adding $b has the effect of subtracting b . We use this property to define : : the difference between two vectors: let d " : a $ b . Then :

:

:

d": a$b": a # ($b )

(vector subtraction);

:

(3-4)

:

that is, we find the difference vector d by adding the vector $b to the vector : a. Figure 3-6 shows how this is done geometrically. As in the usual algebra, we can move a term that includes a vector symbol from one side of a vector equation to the other, but we must change its sign. For example, if we are given Eq. 3-4 and need to solve for : a , we can rearrange the equation as :

:

d#b": a

b

(a) Note head-to-tail arrangement for –b addition d=a–b a

:

Checkpoint 1 :

:

The magnitudes of displacements : a and b are 3 m and 4 m, respectively, and : c ": a # b. : : Considering various orientations of a and b , what are (a) the maximum possible magnitude for : c and (b) the minimum possible magnitude?

Components of Vectors

(b) : :

:

a " d # b.

:

Remember that, although we have used displacement vectors here, the rules for addition and subtraction hold for vectors of all kinds, whether they represent velocities, accelerations, or any other vector quantity. However, we can add only vectors of the same kind. For example, we can add two displacements, or two velocities, but adding a displacement and a velocity makes no sense. In the arithmetic of scalars, that would be like trying to add 21 s and 12 m.

–b

a

or

:

Figure 3-6 (a) Vectors a , b , and $b . : (b) To subtract vector b from vector : a, : add vector $b to vector : a.

Adding vectors geometrically can be tedious. A neater and easier technique involves algebra but requires that the vectors be placed on a rectangular coordinate system. The x and y axes are usually drawn in the plane of the page, as shown

43

3-1 VECTORS AN D TH E I R COM PON E NTS

ax " a cos u

and ay " a sin u,

(3-5)

where u is the angle that the vector : a makes with the positive direction of the x axis, and a is the magnitude of : a . Figure 3-7c shows that : a and its x and y components form a right triangle. It also shows how we can reconstruct a vector from its components: we arrange those components head to tail. Then we complete a right triangle with the vector forming the hypotenuse, from the tail of one component to the head of the other component. Once a vector has been resolved into its components along a set of axes, the components themselves can be used in place of the vector. For example, : a in Fig. 3-7a is given (completely determined) by a and u. It can also be given by its components ax and ay. Both pairs of values contain the same information. If we know a vector in component notation (ax and ay) and want it in magnitude-angle notation (a and u), we can use the equations a " 2a 2x # a 2y

and tan 1 "

ay ax

This is the y component of the vector. y

y

a

ay

x

ax

O

ay

θ

θ

ax

(a)

x

O (b)

This is the x component of the vector. The components and the vector form a right triangle. (c)

a

ay

θ ax

Figure 3-7 (a) The components ax and ay of a . (b) The components are unchanged if vector : the vector is shifted, as long as the magnitude and orientation are maintained. (c) The components form the legs of a right triangle whose hypotenuse is the magnitude of the vector.

This is the x component of the vector.

y (m)

θ

bx = 7 m

O

x (m)

b

This is the y component of the vector.

(3-6)

to transform it. In the more general three-dimensional case, we need a magnitude and two angles (say, a, u, and f) or three components (ax, ay, and az) to specify a vector.

a

b y = –5 m

in Fig. 3-7a. The z axis comes directly out of the page at the origin; we ignore it for now and deal only with two-dimensional vectors. A component of a vector is the projection of the vector on an axis. In Fig. 3-7a, for example, ax is the component of vector : a on (or along) the x axis and ay is the component along the y axis. To find the projection of a vector along an axis, we draw perpendicular lines from the two ends of the vector to the axis, as shown. The projection of a vector on an x axis is its x component, and similarly the projection on the y axis is the y component. The process of finding the components of a vector is called resolving the vector. A component of a vector has the same direction (along an axis) as the vector. In Fig. 3-7, ax and ay are both positive because : a extends in the positive direction of both axes. (Note the small arrowheads on the components, to indicate their direction.) If we were to reverse vector : a , then both components would be negative : and their arrowheads would point toward negative x and y. Resolving vector b in Fig. 3-8 yields a positive component bx and a negative component by. In general, a vector has three components, although for the case of Fig. 3-7a the component along the z axis is zero. As Figs. 3-7a and b show, if you shift a vector without changing its direction, its components do not change. Finding the Components. We can find the components of : a in Fig. 3-7a geometrically from the right triangle there:

:

Figure 3-8 The component of b on the x axis is positive, and that on the y axis is negative.

Checkpoint 2 a are proper to determine that vector? In the figure, which of the indicated methods for combining the x and y components of vector : y

y

ax ay

a

(a)

ax

x

(b)

ax

x ay

a

y

y

ay

ay

a

(c)

ax

x

x a

y

ay

a

y x

ax

a

ax (d)

(e)

x ay

(f )

44

CHAPTE R 3 VECTORS

Sample Problem 3.01 Adding vectors in a drawing, orienteering In an orienteering class, you have the goal of moving as far (straight-line distance) from base camp as possible by making three straight-line moves. You may use the followa , 2.0 km due east ing displacements in any order: (a) : : (directly toward the east); (b) b , 2.0 km 30° north of east (at an angle of 30° toward the north from due east); c , 1.0 km due west. Alternatively, you may substitute (c) : : : c for : c . What is the greatest distance either $b for b or $: you can be from base camp at the end of the third displacement? (We are not concerned about the direction.) a, Reasoning: Using a convenient scale, we draw vectors : : : : b , c , $b , and $: c as in Fig. 3-9a. We then mentally slide the vectors over the page, connecting three of them at a time : in head-to-tail arrangements to find their vector sum d . The tail of the first vector represents base camp. The head of the third vector represents the point at which you stop. : The vector sum d extends from the tail of the first vector to the head of the third vector. Its magnitude d is your distance from base camp. Our goal here is to maximize that base-camp distance. We find that distance d is greatest for a head-to-tail : c . They can be in any a , b , and $: arrangement of vectors :

a a b

b 30°

–b

c

d=b+a–c

This is the vector result for adding those three vectors in any order.

–c

Scale of km 0

1

–c

2

(a)

(b)

Figure 3-9 (a) Displacement vectors; three are to be used. (b) Your distance from base camp is greatest if you undergo : displacements : c , in any order. a , b , and $:

order, because their vector sum is the same for any order. (Recall from Eq. 3-2 that vectors commute.) The order shown in Fig. 3-9b is for the vector sum :

:

d"b#: a # ($: c ).

Using the scale given in Fig. 3-9a, we measure the length d of this vector sum, finding d " 4.8 m.

(Answer)

Sample Problem 3.02 Finding components, airplane flight A small airplane leaves an airport on an overcast day and is later sighted 215 km away, in a direction making an angle of 22° east of due north. This means that the direction is not due north (directly toward the north) but is rotated 22° toward the east from due north. How far east and north is the airplane from the airport when sighted? y

200

Distance (km)

d

P

215 k

100

m

22°

0

100 Distance (km)

We are given the magnitude (215 km) and the angle (22° east of due north) of a vector and need to find the components of the vector. Calculations: We draw an xy coordinate system with the positive direction of x due east and that of y due north (Fig. 3-10). For convenience, the origin is placed at the airport. (We don’t have to do this. We could shift and misalign the coordinate system but, given a choice, why make the prob: lem more difficult?) The airplane’s displacement d points from the origin to where the airplane is sighted. : To find the components of d , we use Eq. 3-5 with u " 68° (" 90° $ 22°): dx " d cos u " (215 km)(cos 68°) " 81 km

θ 0

KEY IDEA

x

Figure 3-10 A plane takes off from an airport at the origin and is later sighted at P.

dy " d sin u " (215 km)(sin 68°) " 199 km % 2.0 ' 102 km.

(Answer) (Answer)

Thus, the airplane is 81 km east and 2.0 ' 102 km north of the airport.

Additional examples, video, and practice available at WileyPLUS

3-1 VECTORS AN D TH E I R COM PON E NTS

45

Problem-Solving Tactics Angles, trig functions, and inverse trig functions Tactic 1: Angles—Degrees and Radians Angles that are measured relative to the positive direction of the x axis are positive if they are measured in the counterclockwise direction and negative if measured clockwise. For example, 210° and $150° are the same angle. Angles may be measured in degrees or radians (rad). To relate the two measures, recall that a full circle is 360° and 2p rad. To convert, say, 40° to radians, write 403

Quadrants II III

Tactic 4: Measuring Vector Angles The equations for cos u and sin u in Eq. 3-5 and for tan u in Eq. 3-6 are valid only if the angle is measured from the positive direction of leg opposite θ hypotenuse leg adjacent to θ hypotenuse

leg opposite θ tan θ = leg adjacent to θ

Hypotenuse

IV

+1 sin –90°

0

90°

180°

270°

360°

270°

360°

270°

360°

–1

(a)

Tactic 3: Inverse Trig Functions When the inverse trig functions sin$1, cos$1, and tan$1 are taken on a calculator, you must consider the reasonableness of the answer you get, because there is usually another possible answer that the calculator does not give. The range of operation for a calculator in taking each inverse trig function is indicated in Fig. 3-12. As an example, sin$1 0.5 has associated angles of 30° (which is displayed by the calculator, since 30° falls within its range of operation) and 150°. To see both values, draw a horizontal line through 0.5 in Fig. 3-12a and note where it cuts the sine curve. How do you distinguish a correct answer? It is the one that seems more reasonable for the given situation.

cos θ =

I

2) rad " 0.70 rad. 3603

Tactic 2: Trig Functions You need to know the definitions of the common trigonometric functions — sine, cosine, and tangent — because they are part of the language of science and engineering. They are given in Fig. 3-11 in a form that does not depend on how the triangle is labeled. You should also be able to sketch how the trig functions vary with angle, as in Fig. 3-12, in order to be able to judge whether a calculator result is reasonable. Even knowing the signs of the functions in the various quadrants can be of help.

sin θ =

IV

Leg opposite θ

θ Leg adjacent to θ

Figure 3-11 A triangle used to define the trigonometric functions. See also Appendix E.

+1 cos –90°

0

90°

180°

–1

(b)

tan

+2 +1 –90°

0

90°

180°

–1 –2

(c)

Figure 3-12 Three useful curves to remember. A calculator’s range of operation for taking inverse trig functions is indicated by the darker portions of the colored curves.

the x axis. If it is measured relative to some other direction, then the trig functions in Eq. 3-5 may have to be interchanged and the ratio in Eq. 3-6 may have to be inverted. A safer method is to convert the angle to one measured from the positive direction of the x axis. In WileyPLUS, the system expects you to report an angle of direction like this (and positive if counterclockwise and negative if clockwise).

Additional examples, video, and practice available at WileyPLUS

46

CHAPTE R 3 VECTORS

3-2 UNIT VECTORS, ADDING VECTORS BY COMPONENTS Learning Objectives After reading this module, you should be able to . . .

3.06 Convert a vector between magnitude-angle and unitvector notations. 3.07 Add and subtract vectors in magnitude-angle notation and in unit-vector notation.

3.08 Identify that, for a given vector, rotating the coordinate system about the origin can change the vector’s components but not the vector itself.

Key Ideas ● Unit vectors ˆi,

ˆj, and k ˆ have magnitudes of unity and are directed in the positive directions of the x, y, and z axes, respectively, in a right-handed coordinate system. We can write a vector : a in terms of unit vectors as

ˆ are the vector components of : in which axiˆ, ayjˆ, and azk a and ax, ay, and az are its scalar components. ● To add vectors in component form, we use the rules

rx " ax # bx

ˆ, a " axˆi # ayjˆ # azk

:

ry " ay # by

rz " az # bz.

:

Here a and b are the vectors to be added, and : r is the vector sum. Note that we add components axis by axis. :

Unit Vectors The unit vectors point along axes. y

ˆj ˆk

ˆi

x

z

Figure 3.13 Unit vectors iˆ, ˆj, and kˆ define the directions of a right-handed coordinate system.

A unit vector is a vector that has a magnitude of exactly 1 and points in a particular direction. It lacks both dimension and unit. Its sole purpose is to point — that is, to specify a direction. The unit vectors in the positive directions of the x, y, and z axes are labeled iˆ, jˆ , and kˆ, where the hat ˆ is used instead of an overhead arrow as for other vectors (Fig. 3-13). The arrangement of axes in Fig. 3-13 is said to be a right-handed coordinate system. The system remains right-handed if it is rotated rigidly. We use such coordinate systems exclusively in this book. Unit vectors are very useful for expressing other vectors; for example, we can : express : a and b of Figs. 3-7 and 3-8 as :

a " axˆi # ay ˆj

(3-7)

b " bxˆi # by ˆj.

(3-8)

:

and

These two equations are illustrated in Fig. 3-14. The quantities axˆi and ayˆj are vectors, called the vector components of : a . The quantities ax and ay are scalars, called the scalar components of : a (or, as before, simply its components). This is the y vector component. y

y

a y ˆj

a

θ

θ

b xˆi

O

x

b

a xˆi

O

Figure 3-14 (a) The vector components a . (b) The vector components of vector : : of vector b .

(a)

x

This is the x vector component.

b y ˆj

(b)

Adding Vectors by Components We can add vectors geometrically on a sketch or directly on a vector-capable calculator. A third way is to combine their components axis by axis.

47

3-2 U N IT VECTORS, ADDI NG VECTORS BY COM PON E NTS

To start, consider the statement :

:

(3-9)

r ": a # b, :

which says that the vector r is the same as the vector (a # b ). Thus, each : : component of : r must be the same as the corresponding component of (a # b ): :

:

rx " ax # bx

(3-10)

ry " ay # by

(3-11)

rz " az # bz.

(3-12)

In other words, two vectors must be equal if their corresponding components are : equal. Equations 3-9 to 3-12 tell us that to add vectors : a and b , we must (1) resolve the vectors into their scalar components; (2) combine these scalar components, axis by axis, to get the components of the sum : r ; and (3) combine the components of : r to get : r itself. We have a choice in step 3. We can express : r in unit-vector notation or in magnitude-angle notation. This procedure for adding vectors by components also applies to vector : : subtractions. Recall that a subtraction such as d " : a $ b can be rewritten as an : : : : : addition d " a # ($b). To subtract, we add a and $b by components, to get dx " ax $ bx, where

dy " ay $ by,

and dz " az $ bz,

d " dxˆi # dyˆj # dzkˆ.

:

(3-13) y

Checkpoint 3 (a) In the figure here, what are the signs of the x : : components of d1 and d2? (b) What are the signs of : : the y components of d1 and d2? (c) What are the : : signs of the x and y components of d1 # d2?

d1

d2

x y

Vectors and the Laws of Physics So far, in every figure that includes a coordinate system, the x and y axes are parallel to the edges of the book page. Thus, when a vector : a is included, its components ax and ay are also parallel to the edges (as in Fig. 3-15a). The only reason for that orientation of the axes is that it looks “proper”; there is no deeper reason. We could, instead, rotate the axes (but not the vector : a ) through an angle f as in Fig. 3-15b, in which case the components would have new values, call them a/x and a/y. Since there are an infinite number of choices of f, there are an infinite number of different pairs of components for : a. Which then is the “right” pair of components? The answer is that they are all equally valid because each pair (with its axes) just gives us a different way of describing the same vector : a ; all produce the same magnitude and direction for the vector. In Fig. 3-15 we have and

a " 2a x2 # a y2 " 2a/x 2 # a/y2 u " u/ # f.

ay

a

O

θ ax

x (a)

Rotating the axes changes the components but not the vector. y y'

(3-14) (3-15)

The point is that we have great freedom in choosing a coordinate system, because the relations among vectors do not depend on the location of the origin or on the orientation of the axes. This is also true of the relations of physics; they are all independent of the choice of coordinate system. Add to that the simplicity and richness of the language of vectors and you can see why the laws of physics are almost always presented in that language: one equation, like Eq. 3-9, can represent three (or even more) relations, like Eqs. 3-10, 3-11, and 3-12.

a a'y

θ'

x'

a'x

φ

O

x

(b)

a and its Figure 3-15 (a) The vector : components. (b) The same vector, with the axes of the coordinate system rotated through an angle f.

48

CHAPTE R 3 VECTORS

Sample Problem 3.03 Searching through a hedge maze A hedge maze is a maze formed by tall rows of hedge. After entering, you search for the center point and then for the exit. Figure 3-16a shows the entrance to such a maze and the first two choices we make at the junctions we encounter in moving from point i to point c. We undergo three displacements as indicated in the overhead view of Fig. 3-16b: d1 " 6.00 m d2 " 8.00 m

11 " 40° 12 " 30°

d3 " 5.00 m

13 " 0°,

Calculations: To evaluate Eqs. 3-16 and 3-17, we find the x and y components of each displacement. As an example, the components for the first displacement are shown in Fig. 3-16c. We draw similar diagrams for the other two displacements and then we apply the x part of Eq. 3-5 to each displacement, using angles relative to the positive direction of the x axis: dlx " (6.00 m) cos 40° " 4.60 m d2x " (8.00 m) cos ($60°) " 4.00 m d3x " (5.00 m) cos 0° " 5.00 m. Equation 3-16 then gives us

where the last segment is parallel to the superimposed x axis. When we reach point c, what are the magnitude and : angle of our net displacement d net from point i?

dnet, x " #4.60 m # 4.00 m # 5.00 m " 13.60 m. Similarly, to evaluate Eq. 3-17, we apply the y part of Eq. 3-5 to each displacement:

KEY IDEAS :

(1) To find the net displacement d net, we need to sum the three individual displacement vectors: :

:

:

dly " (6.00 m) sin 40° = 3.86 m d2y " (8.00 m) sin ($60°) = $6.93 m

:

d net " d 1 # d 2 # d 3.

(2) To do this, we first evaluate this sum for the x components alone, dnet,x " dlx # d2x # d3x, (3-16)

d3y " (5.00 m) sin 0° " 0 m. Equation 3-17 then gives us dnet, y " #3.86 m $ 6.93 m # 0 m " $3.07 m.

and then the y components alone,

:

dnet,y " d1y # d2y # d3y.

(3-17)

:

(3) Finally, we construct d net from its x and y components.

Next we use these components of d net to construct the vector as shown in Fig. 3-16d: the components are in a head-totail arrangement and form the legs of a right triangle, and

y

y

a Three

First vector

vectors

d1

a

u2 d2

u1

i

b d 3

i

b

(a)

d1

x (b)

c

d1x

Net vector i

x

(c)

y

c

d1y

dnet,x

x dnet,y

dnet c (d)

Figure 3-16 (a) Three displacements through a hedge maze. (b) The displacement vectors. (c) The first displacement vector and its components. (d) The net displacement vector and its components.

3-2 U N IT VECTORS, ADDI NG VECTORS BY COM PON E NTS

the vector forms the hypotenuse. We find the magnitude and : angle of d net with Eq. 3-6. The magnitude is dnet " 2d2net,x # d2net,y

(3-18)

" 2(13.60 m)2 # ($3.07 m)2 " 13.9 m.

(Answer)

To find the angle (measured from the positive direction of x), we take an inverse tangent: dnet,y 1 " tan$1 (3-19) dnet,x

$

#

" tan$1

m " $12.7°. # –3.07 13.60 m $

(Answer)

The angle is negative because it is measured clockwise from positive x. We must always be alert when we take an inverse

49

tangent on a calculator. The answer it displays is mathematically correct but it may not be the correct answer for the physical situation. In those cases, we have to add 180° to the displayed answer, to reverse the vector. To check, we always need to draw the vector and its components as we did in Fig. 3-16d. In our physical situation, the figure shows us that 1 " $12.7° is a reasonable answer, whereas $12.7° # 180° " 167° is clearly not. We can see all this on the graph of tangent versus angle in Fig. 3-12c. In our maze problem, the argument of the inverse tangent is $3.07/13.60, or $0.226. On the graph draw a horizontal line through that value on the vertical axis. The line cuts through the darker plotted branch at $12.7° and also through the lighter branch at 167°. The first cut is what a calculator displays.

Sample Problem 3.04 Adding vectors, unit-vector components Figure 3-17a shows the following three vectors: a " (4.2 m)iˆ $ (1.5 m)jˆ, b " ($1.6 m)iˆ # (2.9 m)jˆ ,

: :

c " ($3.7 m)jˆ.

and

:

What is their vector sum : r which is also shown? y 3

b

To add these vectors, find their net x component and their net y component.

2 1 –3

–2

–1

1

2

3

x

4

–1

a

–3

–2

1

–1

2.6iˆ 2

–1 r

–2

Then arrange the net components head to tail. 3

4

x

–2.3jˆ

Similarly, for the y axis, ry " ay # by # cy " $1.5 m # 2.9 m $ 3.7 m " $2.3 m.

This is the result of the addition.

Figure 3-17 Vector : r is the vector sum of the other three vectors.

(Answer)

where (2.6 m)iˆ is the vector component of : r along the x axis and $(2.3 m)jˆ is that along the y axis. Figure 3-17b shows one way to arrange these vector components to form : r. (Can you sketch the other way?) We can also answer the question by giving the magnitude and an angle for : r . From Eq. 3-6, the magnitude is r " 2(2.6 m)2 # ($2.3 m)2 % 3.5 m

–3 (b)

rx " ax # bx # cx " 4.2 m $ 1.6 m # 0 " 2.6 m.

r " (2.6 m)iˆ $ (2.3 m)jˆ,

c

y

Calculations: For the x axis, we add the x components of : a, : b , and : c , to get the x component of the vector sum : r:

:

r

(a)

We can add the three vectors by components, axis by axis, and then combine the components to write the vector sum : r.

We then combine these components of : r to write the vector in unit-vector notation:

–2 –3

KEY IDEA

(Answer)

and the angle (measured from the #x direction) is $2.3 m (Answer) 1 " tan$1 " $413, 2.6 m where the minus sign means clockwise.

Additional examples, video, and practice available at WileyPLUS

#

$

50

CHAPTE R 3 VECTORS

3-3 MULTIPLYING VECTORS Learning Objectives After reading this module, you should be able to . . .

3.09 Multiply vectors by scalars. 3.10 Identify that multiplying a vector by a scalar gives a vector, taking the dot (or scalar) product of two vectors gives a scalar, and taking the cross (or vector) product gives a new vector that is perpendicular to the original two. 3.11 Find the dot product of two vectors in magnitude-angle notation and in unit-vector notation. 3.12 Find the angle between two vectors by taking their dot product in both magnitude-angle notation and unit-vector notation.

3.13 Given two vectors, use a dot product to find how much of one vector lies along the other vector. 3.14 Find the cross product of two vectors in magnitudeangle and unit-vector notations. 3.15 Use the right-hand rule to find the direction of the vector that results from a cross product. 3.16 In nested products, where one product is buried inside another, follow the normal algebraic procedure by starting with the innermost product and working outward.

Key Ideas ● The product of a scalar s and a vector

v is a new vector whose magnitude is sv and whose direction is the same as that of : v if s is positive, and opposite that of : v if s is negative. To divide : v by s, multiply : v by 1/s. :

● The scalar (or dot) product of two vectors :

ten a " b and is the scalar quantity given by :

:

a and b is writ-

:

:

a " b " ab cos 4,

:

:

in which 4 is the angle between the directions of : a and b . A scalar product is the product of the magnitude of one vector and the scalar component of the second vector along the direction of the first vector. In unit-vector notation, ˆ )!(bxˆi # byjˆ # bzk ˆ ), a " b " (axˆi # ayjˆ # azk

:

:

which may be expanded according to the distributive law. : : Note that : a. a" b"b":

● The vector (or cross) product of two vectors

:

a and b is written : a ' b and is a vector : c whose magnitude c is given by :

:

c " ab sin 4, in which 4 is the smaller of the angles between the directions : of : a and b . The direction of : c is perpendicular to the plane : defined by : a and b and is given by a right-hand rule, as shown : : in Fig. 3-19. Note that : a ' b " $( b ' : a ). In unit-vector notation, ˆ ) ' (bxˆi # byjˆ # bzk ˆ ), a ' b " (axˆi # ayjˆ # azk

:

:

which we may expand with the distributive law. ● In nested products, where one product is buried inside an-

other, follow the normal algebraic procedure by starting with the innermost product and working outward.

Multiplying Vectors* There are three ways in which vectors can be multiplied, but none is exactly like the usual algebraic multiplication. As you read this material, keep in mind that a vector-capable calculator will help you multiply vectors only if you understand the basic rules of that multiplication.

Multiplying a Vector by a Scalar If we multiply a vector : a by a scalar s, we get a new vector. Its magnitude is the product of the magnitude of : a and the absolute value of s. Its direction is the direction of : a if s is positive but the opposite direction if s is negative. To divide : a by s, we multiply : a by 1/s.

Multiplying a Vector by a Vector There are two ways to multiply a vector by a vector: one way produces a scalar (called the scalar product), and the other produces a new vector (called the vector product). (Students commonly confuse the two ways.) *This material will not be employed until later (Chapter 7 for scalar products and Chapter 11 for vector products), and so your instructor may wish to postpone it.

3-3 M U LTI PLYI NG VECTORS

The Scalar Product :

:

The scalar product of the vectors : a and b in Fig. 3-18a is written as : a " b and defined to be : :

a " b " ab cos f,

(3-20) :

where a is the magnitude of : a , b is the magnitude of b , and 4 is the angle between : : a and b (or, more properly, between the directions of : a and b ). There are actually two such angles: 4 and 360° $ 4. Either can be used in Eq. 3-20, because their cosines are the same. Note that there are only scalars on the right side of Eq. 3-20 (including the : value of cos 4). Thus : a " b on the left side represents a scalar quantity. Because of : : the notation, a " b is also known as the dot product and is spoken as “a dot b.” A dot product can be regarded as the product of two quantities: (1) the magnitude of one of the vectors and (2) the scalar component of the second vector along the direction of the first vector. For example, in Fig. 3-18b, : a has a scalar : component a cos 4 along the direction of b ; note that a perpendicular dropped : : from the head of : a onto b determines that component. Similarly, b has a scalar : component b cos 4 along the direction of a . :

If the angle 4 between two vectors is 0°, the component of one vector along the other is maximum, and so also is the dot product of the vectors. If, instead, 4 is 90°, the component of one vector along the other is zero, and so is the dot product.

Equation 3-20 can be rewritten as follows to emphasize the components: : :

a " b " (a cos f)(b) " (a)(b cos f).

(3-21)

The commutative law applies to a scalar product, so we can write : :

:

a " b " b ": a.

When two vectors are in unit-vector notation, we write their dot product as a " b " (axˆi # ayˆj # azkˆ) "(bxˆi # byˆj # bzkˆ),

: :

(3-22)

which we can expand according to the distributive law: Each vector component of the first vector is to be dotted with each vector component of the second vector. By doing so, we can show that : :

a " b " axbx # ayby # azbz.

(3-23)

a

φ b

(a) Component of b along direction of a is b cos φ

Multiplying these gives the dot product. Figure 3-18 (a) Two vectors : a : and b , with an angle f between them. (b) Each vector has a component along the direction of the other vector.

Or multiplying these gives the dot product.

φ

a b Component of a along direction of b is a cos φ (b)

51

52

CHAPTE R 3 VECTORS

Checkpoint 4 :

:

Vectors C and D have magnitudes of 3 units and 4 units, respectively. What is the : : : : angle between the directions of C and D if C " D equals (a) zero, (b) 12 units, and (c) $12 units?

The Vector Product :

:

The vector product of : a and b , written : a ' b , produces a third vector : c whose magnitude is c " ab sin f,

(3-24) :

where f is the smaller of the two angles between : a and b . (You must use the smaller of the two angles between the vectors because sin f and sin(360° $ f) : a ' b is also known as the cross differ in algebraic sign.) Because of the notation, : product, and in speech it is “a cross b.” :

:

:

If : a and b are parallel or antiparallel, : a ' b " 0. The magnitude of : a ' b , which can : : : be written as !a ' b ! , is maximum when : a and b are perpendicular to each other. :

: The direction of : c is perpendicular to the plane that contains a and b . : : : Figure 3-19a shows how to determine the direction of c " a ' b with what is : a and b tail to tail without altering known as a right-hand rule. Place the vectors : their orientations, and imagine a line that is perpendicular to their plane where they meet. Pretend to place your right hand around that line in such a way that : a into b through the smaller angle between them. Your your fingers would sweep : c. outstretched thumb points in the direction of : The order of the vector multiplication is important. In Fig. 3-19b, we are : : c/" b ' : a , so the fingers are placed to sweep b determining the direction of : : into a through the smaller angle. The thumb ends up in the opposite direction c ; that is, c/ " $: from previously, and so it must be that : :

:

: b': a " $(a ' b ).

(3-25)

In other words, the commutative law does not apply to a vector product. In unit-vector notation, we write : a ' b " (axˆi # ay ˆj # az kˆ) ' (bxˆi # by ˆj # bz kˆ),

:

(3-26)

which can be expanded according to the distributive law; that is, each component of the first vector is to be crossed with each component of the second vector. The cross products of unit vectors are given in Appendix E (see “Products of Vectors”). For example, in the expansion of Eq. 3-26, we have axˆi ' bxˆi " axbx( ˆi ' ˆi ) " 0, because the two unit vectors ˆi and ˆi are parallel and thus have a zero cross product. Similarly, we have axˆi ' by ˆj " axby( ˆi ' ˆj ) " axby kˆ. In the last step we used Eq. 3-24 to evaluate the magnitude of ˆi ' ˆj as unity. (These vectors ˆi and ˆj each have a magnitude of unity, and the angle between them is 90°.) Also, we used the right-hand rule to get the direction of ˆi ' ˆj as being in the positive direction of the z axis (thus in the direction of kˆ).

3-3 M U LTI PLYI NG VECTORS

53

Continuing to expand Eq. 3-26, you can show that : a ' b " (ay bz $ byaz)ˆi # (azbx $ bz ax)ˆj # (axby $ bx ay)kˆ.

:

(3-27)

A determinant (Appendix E) or a vector-capable calculator can also be used. To check whether any xyz coordinate system is a right-handed coordinate system, use the right-hand rule for the cross product ˆi ' ˆj " kˆ with that system. If your fingers sweep ˆi (positive direction of x) into ˆj (positive direction of y) with the outstretched thumb pointing in the positive direction of z (not the negative direction), then the system is right-handed.

Checkpoint 5 :

:

Vectors C and D have magnitudes of 3 units and 4 units, respectively. What is the an: : : : gle between the directions of C and D if the magnitude of the vector product C ' D is (a) zero and (b) 12 units?

A c

a b

b

b

(a)

b a

a

a

c#

(b) :

Figure 3-19 Illustration of the right-hand rule for vector products. (a) Sweep vector : a into vector b with the fingers of your right hand. : : : Your outstretched thumb shows the direction of vector : a ' b. c ": a ' b . (b) Showing that b ' : a is the reverse of :

54

CHAPTE R 3 VECTORS

Sample Problem 3.05 Angle between two vectors using dot products : What is the angle 4 between : a " 3.0ˆi $ 4.0ˆj and b " $2.0ˆi # 3.0kˆ? (Caution: Although many of the following steps can be bypassed with a vector-capable calculator, you will learn more about scalar products if, at least here, you use these steps.)

We can separately evaluate the left side of Eq. 3-28 by writing the vectors in unit-vector notation and using the distributive law: : : a " b " (3.0ˆi $ 4.0ˆj ) "($2.0ˆi # 3.0kˆ) " (3.0ˆi ) "($2.0ˆi ) # (3.0ˆi ) "(3.0kˆ) # ($4.0ˆj ) "($2.0ˆi ) # ($4.0ˆj ) "(3.0kˆ).

KEY IDEA The angle between the directions of two vectors is included in the definition of their scalar product (Eq. 3-20): : :

a " b " ab cos f.

We next apply Eq. 3-20 to each term in this last expression. The angle between the unit vectors in the first term (ˆi and ˆi ) is 0°, and in the other terms it is 90°.We then have

(3-28)

: :

a " b " $(6.0)(1) # (9.0)(0) # (8.0)(0) $ (12)(0) " $6.0.

Calculations: In Eq. 3-28, a is the magnitude of a , or :

a "23.02 # ($4.0)2 " 5.00,

(3-29)

Substituting this result and the results of Eqs. 3-29 and 3-30 into Eq. 3-28 yields

:

$6.0 " (5.00)(3.61) cos f,

and b is the magnitude of b , or b " 2($2.0)2 # 3.02 " 3.61.

(3-30)

so

$6.0 " 1093 %1103. (5.00)(3.61)

4 " cos$1

(Answer)

Sample Problem 3.06 Cross product, right-hand rule z

In Fig. 3-20, vector : a lies in the xy plane, has a magnitude of 18 units, and points in a direction 250° from the positive di: rection of the x axis. Also, vector b has a magnitude of 12 units and points in the positive direction of the z axis.What : is the vector product : c ": a ' b?

a

KEY IDEA When we have two vectors in magnitude-angle notation, we find the magnitude of their cross product with Eq. 3-24 and the direction of their cross product with the right-hand rule of Fig. 3-19. Calculations: For the magnitude we write c " ab sin f " (18)(12)(sin 90°) " 216.

(Answer)

To determine the direction in Fig. 3-20, imagine placing the fingers of your right hand around a line perpendicular to the : plane of : a and b (the line on which : c is shown) such that : your fingers sweep : a into b . Your outstretched thumb then

Sweep a into b.

b

c=a

b

This is the resulting vector, perpendicular to both a and b.

250° 160° y

x

Figure 3-20 Vector c (in the xy plane) is the vector (or cross) : product of vectors : a and b . :

gives the direction of : c . Thus, as shown in the figure, : c lies in the xy plane. Because its direction is perpendicular to the direction of : a (a cross product always gives a perpendicular vector), it is at an angle of 250° $ 90° " 160° from the positive direction of the x axis.

Sample Problem 3.07 Cross product, unit-vector notation If : a " 3ˆi $ 4ˆj and b " $2ˆi # 3kˆ, what is : c ": a ' b? :

:

KEY IDEA When two vectors are in unit-vector notation, we can find their cross product by using the distributive law.

Calculations: Here we write c " (3ˆi $ 4ˆj ) ' ($2ˆi # 3kˆ)

:

" 3ˆi ' ($2ˆi ) # 3ˆi ' 3kˆ # ($4ˆj ) ' ($2ˆi ) # ($4ˆj ) ' 3kˆ.

(Answer)

55

R EVI EW & SU M MARY

We next evaluate each term with Eq. 3-24, finding the direction with the right-hand rule. For the first term here, the angle f between the two vectors being crossed is 0. For the other terms, f is 90°. We find c " $6(0) # 9($ ˆj ) # 8($kˆ) $ 12ˆi " $12ˆi $ 9 ˆj $ 8 kˆ.

:

(Answer)

:

This vector : c is perpendicular to both : a and b , a fact you : can check by showing that : c": a = 0 and : c " b = 0; that is, there : is no component of : c along the direction of either : a or b . In general: A cross product gives a perpendicular vector, two perpendicular vectors have a zero dot product, and two vectors along the same axis have a zero cross product.

Additional examples, video, and practice available at WileyPLUS

Review & Summary Scalars and Vectors Scalars, such as temperature, have magnitude only. They are specified by a number with a unit (10°C) and obey the rules of arithmetic and ordinary algebra. Vectors, such as displacement, have both magnitude and direction (5 m, north) and obey the rules of vector algebra. :

Adding Vectors Geometrically Two vectors : a and b may be added geometrically by drawing them to a common scale and placing them head to tail. The vector connecting the tail of the first to the head of the second is the vector sum : s . To : : : subtract b from : a , reverse the direction of b to get $ b ; then : add $ b to : a . Vector addition is commutative :

:

:

(3-2)

a a#b"b#:

:

:

: (a # b) # : c ": a # (b # : c ).

ry " ay # by

rz " az # bz.

(3-10 to 3-12)

:

Here : a and b are the vectors to be added, and : r is the vector sum. Note that we add components axis by axis.We can then express the sum in unit-vector notation or magnitude-angle notation. Product of a Scalar and a Vector The product of a scalar s and a vector : v is a new vector whose magnitude is sv and whose direction is the same as that of : v if s is positive, and opposite that of : v if s is negative. (The negative sign reverses the vector.) To divide : v by s, multiply : v by 1/s.

(3-3)

Components of a Vector The (scalar) components ax and ay of any two-dimensional vector : a along the coordinate axes are found by dropping perpendicular lines from the ends of : a onto the coordinate axes. The components are given by ax " a cos u and

ay " a sin u,

(3-5)

where u is the angle between the positive direction of the x axis and the direction of : a . The algebraic sign of a component indicates its direction along the associated axis. Given its components, we can find the magnitude and orientation (direction) of the vector : a by using and tan 1 "

ay ax

a " axˆi # ay ˆj # az kˆ ,

: :

a " b " ab cos f,

(3-20) :

in which f is the angle between the directions of a and b . A scalar product is the product of the magnitude of one vector and the scalar component of the second vector along the direction of the : : first vector. Note that : a " b " b" : a, which means that the scalar product obeys the commutative law. In unit-vector notation, :

a " b " (axˆi # ay ˆj # az kˆ ) "(bxˆi # by ˆj # bz kˆ ),

: :

(3-22)

which may be expanded according to the distributive law. The Vector Product The vector (or cross) product of two vectors : : a and b is written : a ' b and is a vector : c whose magnitude c is given by c " ab sin f, (3-24) :

(3-6)

Unit-Vector Notation Unit vectors ˆi , ˆj, and kˆ have magnitudes of unity and are directed in the positive directions of the x, y, and z axes, respectively, in a right-handed coordinate system (as defined by the vector products of the unit vectors). We can write a vector : a in terms of unit vectors as :

rx " ax # bx

The Scalar Product The scalar (or dot) product of two vectors : a : : and b is written : a " b and is the scalar quantity given by

and obeys the associative law

a " 2a 2x # a 2y

Adding Vectors in Component Form To add vectors in component form, we use the rules

(3-7)

in which axˆi , ay ˆj , and az kˆ are the vector components of : a and ax, ay, and az are its scalar components.

in which f is the smaller of the angles between the directions of : a : and b . The direction of : c is perpendicular to the plane : defined by : a and b and is given by a right-hand rule, as shown in : : Fig. 3-19. Note that : a ' b " $( b ' : a ), which means that the vector product does not obey the commutative law. In unit-vector notation, : a ' b " (axˆi # ay ˆj # az kˆ ) ' (bxˆi # by ˆj # bz kˆ ),

:

which we may expand with the distributive law.

(3-26)

56

CHAPTE R 3 VECTORS

Questions y

1 Can the sum of the magnitudes of two vectors ever be equal to the magnitude of the sum of the same two vectors? If no, why not? If yes, when? 2 The two vectors shown in Fig. 3-21 lie in an xy plane. What are the signs of the x and y components, respec: : : : tively, of (a) d1 # d2, (b) d1 $ d2, and : : (c) d2 $ d1?

:

6 Describe two vectors : a and b such that :

a#b ": c (a) :

:

d1

:

:

:

:

d3 " (8 m)iˆ.

d2 " (6 m)ˆj, :

:

z

:

y

y

v

(c )

x

z

y

(1)

(2)

(d )

(e )

Figure 3-23 Question 5.

(3)

:

10 Figure 3-25 shows vector A and four other vectors that have the same magnitude but differ in orientation. (a) Which of those other four vectors : have the same dot product with A ? (b) Which have a negative dot product : with A ?

θ

θ

A

θ

θ

C E

r " 2ˆi $ 3ˆj # 2kˆ : s " 3ˆi # 5ˆj $ 3kˆ. :

:

ax " 3

ay " 3

cx " $3

cy " $3

bx " $3

by " 3

dx " 3

dy " $3.

13 Which of the following are correct (meaningful) vector expressions? What is wrong with any incorrect expression? :

:

:

(a) A " (B " C) :

:

:

:

:

:

:

:

:

:

(f) A # (B ' C)

:

(g) 5 # A

:

(h) 5 # (B " C)

(c) A " (B ' C) (f )

B

D

11 In a game held within a threeFigure 3-25 Question 10. dimensional maze, you must move your game piece from start, at xyz coordinates (0, 0, 0), to finish, at coordinates ($2 cm, 4 cm, $4 cm). The game piece can undergo only the displacements (in centimeters) given below. If, along the way, the game piece lands at coordinates ($5 cm, $1 cm, $1 cm) or (5 cm, 2 cm, $1 cm), you lose the game. Which displacements and in what sequence will get your game piece to finish?

(b) A ' (B " C) y

F

12 The x and y components of four vectors : a , b,: c , and d are given below. For which vectors will your calculator give you the correct angle u when you use it to find u with Eq. 3-6? Answer first by examining Fig. 3-12, and then check your answers with your calculator.

y x

x

Figure 3-24 Question 9.

:

z

z

z

:

(b )

z

v

x

p " $7ˆi # 2ˆj $ 3kˆ : q " 2ˆi $ ˆj # 4kˆ

z

y

v

x z

y F

F

:

y

x

:

v ' B) and : v is perpendicular to B, then what is the 9 If F " q(: : direction of B in the three situations shown in Fig. 3-24 when constant q is (a) positive and (b) negative?

x

x

:

:

:

Figure 3-21 Question 2.

5 Which of the arrangements of axes in Fig. 3-23 can be labeled “right-handed coordinate system”? As usual, each axis label indicates the positive side of the axis.

(a )

:

a "b ": a ": c , must b equal : c? 8 If :

:

z

and a2 # b2 " c2.

a#b ": c (c) :

c ), does (a) : a # ($ d ) " : c # ($ b ), (b) : a" 7 If d " a # b # ($: : : : : : : : ($ b ) # d # c , and (c) c # ($ d ) " a # b ?

4 Equation 3-2 shows that the addition of two vectors : a and b is commutative. Does that mean subtraction is commutative, so that : : : a? a$b "b$:

y

:

:

:

x

The pit is at coordinates (8 m, 6 m). If a team member putts the ball into or through the pit, the member is automatically transferred to Florida State University, the arch rival. What sequence of displacements should a team member use to avoid the pit and the school transfer?

x

and a # b " c;

(b) a # b " a $ b ;

d2

3 Being part of the “Gators,” the University of Florida golfing team y must play on a putting green with an Hole alligator pit. Figure 3-22 shows an overhead view of one putting chalGator lenge of the team; an xy coordinate pit system is superimposed. Team members must putt from the origin to the hole, which is at xy coordinates (8 m, x 12 m), but they can putt the golf ball using only one or more of the folFigure 3-22 Question 3. lowing displacements, one or more times: d1 " (8 m)iˆ # (6 m)jˆ,

:

:

:

(d) A ' (B ' C) :

(e) A # (B " C)

:

:

:

:

:

(i) 5 # (B ' C) :

:

:

:

(j) (A " B) # (B ' C)

PROB LE M S

57

Problems Tutoring problem available (at instructor’s discretion) in WileyPLUS and WebAssign SSM • – •••

Worked-out solution available in Student Solutions Manual

WWW Worked-out solution is at

Number of dots indicates level of problem difficulty

ILW

Interactive solution is at

http://www.wiley.com/college/halliday

Additional information available in The Flying Circus of Physics and at flyingcircusofphysics.com

Module 3-1 Vectors and Their Components •1 SSM What are (a) the x component and (b) the y component of a vector : a in the xy plane if its direction is 250° y counterclockwise from the positive direction of the x axis and its magnitude is 7.3 m? r θ

•2 A displacement vector : r in the xy plane is 15 m long and directed at angle u " 30° in Fig. 3-26. Determine (a) the x component and (b) the y component of the vector.

x

Figure 3-26 Problem 2.

:

•3 SSM The x component of vector A is $25.0 m and the y component is #40.0 m. (a) What is the magni: : tude of A ? (b) What is the angle between the direction of A and the positive direction of x? •4 Express the following angles in radians: (a) 20.0°, (b) 50.0°, (c) 100°. Convert the following angles to degrees: (d) 0.330 rad, (e) 2.10 rad, (f) 7.70 rad. •5 A ship sets out to sail to a point 120 km due north. An unexpected storm blows the ship to a point 100 km due east of its starting point. (a) How far and (b) in what direction must it now sail to reach its original destination? •6 In Fig. 3-27, a heavy piece of machinery is raised by sliding it a distance d " 12.5 m along a plank oriented at angle u " 20.0° to the horizontal. How far is it moved (a) vertically and (b) horizontally?

d

θ

•7 Consider two displacements, Figure 3-27 Problem 6. one of magnitude 3 m and another of magnitude 4 m. Show how the displacement vectors may be combined to get a resultant displacement of magnitude (a) 7 m, (b) 1 m, and (c) 5 m. Module 3-2 Unit Vectors, Adding Vectors by Components •8 A person walks in the following pattern: 3.1 km north, then 2.4 km west, and finally 5.2 km south. (a) Sketch the vector diagram that represents this motion. (b) How far and (c) in what direction would a bird fly in a straight line from the same starting point to the same final point? •9

Two vectors are given by b " ($1.0 m)iˆ # (1.0 m)jˆ # (4.0 m)kˆ.

:

:

:

In unit-vector notation, find (a) : a # b , (b) : a $ b , and (c) a third : vector : c such that : a$b#: c " 0. •10 Find the (a) x, (b) y, and (c) z components of the sum : r of : the displacements : c and d whose components in meters are cx " 7.4, cy " $3.8, cz " $6.1; dx " 4.4, dy " $2.0, dz " 3.3. :

a # b if •11 SSM (a) In unit-vector notation, what is the sum : : a " (4.0 m)ˆi # (3.0 m)ˆj and b " ($13.0 m)ˆi # (7.0 m)ˆj ? What : are the (b) magnitude and (c) direction of : a # b?

:

•13 A person desires to reach a point that is 3.40 km from her present location and in a direction that is 35.0° north of east. However, she must travel along streets that are oriented either north – south or east – west. What is the minimum distance she could travel to reach her destination? •14 You are to make four straight-line moves over a flat desert floor, starting at the origin of an xy coordinate system and ending at the xy coordinates ($140 m, 30 m). The x component and y component of your moves are the following, respectively, in meters: (20 and 60), then (bx and $70), then ($20 and cy), then ($60 and $70). What are (a) component bx and (b) component cy? What are (c) the magnitude and (d) the angle (relative to the positive direction of the x axis) of the overall displacement? WWW The two vec•15 SSM ILW : tors : a and b in Fig. 3-28 have equal magnitudes of 10.0 m and the angles are 11 " 30° and 12 " 105°. Find the (a) x and (b) y components of their vector sum : r , (c) the magnitude of : r, and (d) the angle : r makes with the positive direction of the x axis.

y

b

θ2

a

•16 For the displacement vectors θ1 : x : a " (3.0 m)iˆ # (4.0 m)jˆ and b " O : : ˆ ˆ (5.0 m)i # ($2.0 m)j , give a # b in Figure 3-28 Problem 15. (a) unit-vector notation, and as (b) a magnitude and (c) an angle (rela: tive to ˆi ). Now give b $ : a in (d) unit-vector notation, and as (e) a magnitude and (f) an angle. :

: : ILW Three vectors a , b , and c each have a magnitude of •17 50 m and lie in an xy plane. Their directions relative to the positive direction of the x axis are 30°, 195°, and 315°, respectively. What are : (a) the magnitude and (b) the angle of the vector : a#b#: c , and : (c) the magnitude and (d) the angle of : a$b#: c ? What are the : (e) magnitude and (f) angle of a fourth vector d such that : : : : (a # b ) $ ( c # d ) " 0? :

a " (4.0 m)iˆ $ (3.0 m)jˆ # (1.0 m)kˆ

:

and

•12 A car is driven east for a distance of 50 km, then north for 30 km, and then in a direction 30° east of north for 25 km. Sketch the vector diagram and determine (a) the magnitude and (b) the angle of the car’s total displacement from its starting point.

:

:

:

•18 In the sum A # B " C, vector A has a magnitude of 12.0 m and is angled 40.0° counterclockwise from the #x direction, and vec: tor C has a magnitude of 15.0 m and is angled 20.0° counterclockwise from the $x direction. What are (a) the magnitude and (b) the : angle (relative to #x) of B? •19 In a game of lawn chess, where pieces are moved between the centers of squares that are each 1.00 m on edge, a knight is moved in the following way: (1) two squares forward, one square rightward; (2) two squares leftward, one square forward; (3) two squares forward, one square leftward. What are (a) the magnitude and (b) the angle (relative to “forward”) of the knight’s overall displacement for the series of three moves?

58

CHAPTE R 3 VECTORS

••20 An explorer is caught in a whiteout (in which the snowfall is so thick that the ground cannot be distinguished from the sky) while returning to base camp. He was supposed to travel due north for 5.6 km, but when the snow clears, he discovers that he actually traveled 7.8 km at 50° north of due east. (a) How far and (b) in what direction must he now travel to reach base camp?

an ant’s displacement from the nest (find it in the figure) if the ant enters the trail at point A? What are the (c) magnitude and (d) angle if it enters at point B? b

a

••21 An ant, crazed by the Sun on a hot Texas afternoon, darts over an xy plane scratched in the dirt. The x and y components of four consecutive darts are the following, all in centimeters: (30.0, 40.0), (bx, $70.0), ($20.0, cy), ($80.0, $70.0). The overall displacement of the four darts has the xy components ($140, $20.0). What are (a) bx and (b) cy? What are the (c) magnitude and (d) angle (relative to the positive direction of the x axis) of the overall displacement?

c

e

:

F: 5.00 m at $75.03

:

H: 6.00 m at $2103

••23 If B is added to C " 3.0iˆ # 4.0jˆ , the result is a vector in the : positive direction of the y axis, with a magnitude equal to that of C. : What is the magnitude of B? :

:

••24 Vector A , which is directed along an x axis, is to be added : to vector B, which has a magnitude of 7.0 m. The sum is a third vector that is directed along the y axis, with a magnitude that is 3.0 : : times that of A . What is that magnitude of A ? ••25 Oasis B is 25 km due east of oasis A. Starting from oasis A, a camel walks 24 km in a direction 15° south of east and then walks 8.0 km due north. How far is the camel then from oasis B? ••26 What is the sum of the following four vectors in (a) unitvector notation, and as (b) a magnitude and (c) an angle? : A " (2.00 m)iˆ # (3.00 m)jˆ

:

C " ($4.00 m)iˆ # ($6.00 m)jˆ

D: 5.00 m, at $2353

B: 4.00 m, at #65.03

:

: : d1 # d2

: : 5d3, d1

••27 If " what are, in unit-vector

: d2

: and d3 " $ " : : notation, (a) d1 and (b) d2?

2iˆ # 4jˆ , then

••28 Two beetles run across flat sand, starting at the same point. Beetle 1 runs 0.50 m due east, then 0.80 m at 30° north of due east. Beetle 2 also makes two runs; the first is 1.6 m at 40° east of due north. What must be (a) the magnitude and (b) the direction of its second run if it is to end up at the new location of beetle 1? ••29 Typical backyard ants often create a network of chemical trails for guidance. Extending outward from the nest, a trail branches (bifurcates) repeatedly, with 60° between the branches. If a roaming ant chances upon a trail, it can tell the way to the nest at any branch point: If it is moving away from the nest, it has two choices of path requiring a small turn in its travel direction, either 30° leftward or 30° rightward. If it is moving toward the nest, it has only one such choice. Figure 3-29 shows a typical ant trail, with lettered straight sections of 2.0 cm length and symmetric bifurcation of 60°. Path v is parallel to the y axis. What are the (a) magnitude and (b) angle (relative to the positive direction of the superimposed x axis) of

r

o

B

q

j

i

s v x

t

u w

Figure 3-29 Problem 29. Here are two vectors:

••30

a " (4.0 m)iˆ $ (3.0 m)jˆ

b " (6.0 m)iˆ # (8.0 m)jˆ .

:

and

What are (a) the magnitude and (b) the angle (relative to ˆi ) of : a? : What are (c) the magnitude and (d) the angle of b ? What are (e) : the magnitude and (f) the angle of : a # b ; (g) the magnitude and : : (h) the angle of b $ a ; and (i) the magnitude and ( j) the angle of : : : a $ b ? (k) What is the angle between the directions of b $ : a : : and a $ b ? ••31 In Fig. 3-30, a vector : a with a magnitude of 17.0 m is directed at angle 1 " 56.0° counterclockwise from the #x axis. What are the components (a) ax and (b) ay of the vector? A second coordinate system is inclined by angle 1/ " 18.0° with respect to the first. What are the components (c) a/x and (d) a/y in this primed coordinate system? y

y' ay

:

: 3d3,

p

y

: :

n

k

g

f

:

:

G: 4.00 m at #1.20 rad

l

h

d

••22 (a) What is the sum of the following four vectors in unitvector notation? For that sum, what are (b) the magnitude, (c) the angle in degrees, and (d) the angle in radians? E: 6.00 m at #0.900 rad

m A

a

a'y x'

θ'

a'x

θ O

θ'

ax

x

Figure 3-30 Problem 31. z •••32 In Fig. 3-31, a cube of edge length a sits with one corner at the origin of an xyz coordinate system. A a body diagonal is a line that extends y from one corner to another through a the center. In unit-vector notation, a what is the body diagonal that extends x from the corner at (a) coordinates (0, Figure 3-31 Problem 32. 0, 0), (b) coordinates (a, 0, 0), (c) coordinates (0, a, 0), and (d) coordinates (a, a, 0)? (e) Determine the

PROB LE M S

59

:

angles that the body diagonals make with the adjacent edges. (f) Determine the length of the body diagonals in terms of a.

ponent of b; and (e) the x component and (f) the y component of : c ? If : : : c " pa # qb, what are the values of (g) p and (h) q?

Module 3-3 Multiplying Vectors •33 For the vectors in Fig. 3-32, with a " 4, b " 3, and c " 5, what are (a) the magnitude and (b) the direction : of : a ' b, (c) the magnitude and (d) the di- y rection of : a': c , and (e) the magnitude : c and (f) the direction of b ' : c ? (The z axis b is not shown.)

••44

•34 Two vectors are presented as : : a " 3.0iˆ # 5.0jˆ and b " 2.0iˆ # 4.0jˆ . Find : : : : : : : # b ) " b , and (a) a ' b , (b) a " b , (c) (a : (d) the component of a along the direc: tion of b . (Hint: For (d), consider Eq. 3-20 and Fig. 3-18.)

x

a

Figure 3-32 Problems 33 and 54.

:

v " 2.0iˆ # 4.0jˆ # 6.0kˆ

:

F " 4.0iˆ $ 20jˆ # 12kˆ. :

:

What then is B in unit-vector notation if Bx " By? Additional Problems :

:

:

45 Vectors A and B lie in an xy plane. A has magnitude 8.00 and : angle 130°; B has components Bx " $7.72 and By " $9.20. (a) : : : : What is 5A " B? What is 4A ' 3B in (b) unit-vector notation and (c) magnitude-angle notation with spherical coordinates (see : Fig. 3-34)? (d) What is the angle between the directions of A and : : 4A ' 3B? (Hint: Think a bit before you resort to a calculation.) : What is A # 3.00kˆ in (e) unit-vector notation and (f) magnitudeangle notation with spherical coordinates? z φ

: : •36 If d1 " 3iˆ $ 2jˆ # 4kˆ and d2 " $5iˆ # 2jˆ $ kˆ, then what is : : : : (d1 # d2) " (d1 ' 4d2)? •37 Three vectors are given by : a " 3.0iˆ # 3.0jˆ $ 2.0kˆ, : : ˆ ˆ ˆ ˆ b " $1.0i $ 4.0j # 2.0k, and c " 2.0i # 2.0jˆ # 1.0kˆ. Find (a) : : : : a " (b ' : c ), (b) : a " (b # : c ), and (c) : a ' (b # : c ). :

and

:

•35 Two vectors, : r and : s , lie in the xy plane. Their magnitudes are 4.50 and 7.30 units, respectively, and their directions are 320° and 85.0°, respectively, as measured counterclockwise from the positive x axis. What are the values of (a) : r ': r ": s and (b) : s?

••38

:

: In the product F " qv ' B, take q " 2,

y θ

x

Figure 3-34 Problem 45.

:

For the following three vectors, what is 3C " (2A ' B)? : A " 2.00iˆ # 3.00jˆ $ 4.00kˆ : B " $3.00iˆ # 4.00jˆ # 2.00kˆ :

: C " 7.00iˆ $ 8.00jˆ :

••39 Vector A has a magnitude of 6.00 units, vector B has a mag: : nitude of 7.00 units, and A " B has a value of 14.0. What is the angle : : between the directions of A and B? :

••40 Displacement d1 is in the yz plane 63.0° from the positive direction of the y axis, has a positive z component, and has a mag: nitude of 4.50 m. Displacement d2 is in the xz plane 30.0° from the positive direction of the x axis, has a positive z component, and has : : : : magnitude 1.40 m. What are (a) d1 " d2, (b) d1 ' d2, and (c) the an: : gle between d1 and d2? ••41 SSM ILW WWW Use the definition of scalar product, : : : a " b " axbx # ayby # azbz to cala " b " ab cos 1, and the fact that : culate the angle between the two vectors given by : a " 3.0iˆ # : ˆ ˆ ˆ ˆ ˆ 3.0j # 3.0k and b " 2.0i # 1.0j # 3.0k. ••42 In a meeting of mimes, mime 1 goes through a displacement : d1 " (4.0 m)iˆ # (5.0 m)jˆ and mime 2 goes through a displacement : : : : : d2 " ($3.0 m)iˆ # (4.0 m)jˆ . What are (a) d1 ' d2, (b) d1 " d2, : : : (c) (d1 # d2) " d2, and (d) the com: y ponent of d1 along the direction of c : d2? (Hint: For (d), see Eq. 3-20 and Fig. 3-18.) ••43 SSM ILW The three vectors in Fig. 3-33 have magnitudes a " 3.00 m, b " 4.00 m, and c " 10.0 m and angle 1 " 30.0°. What are (a) the x component and (b) the y component of : a ; (c) the x component and (d) the y com-

b θ a

Figure 3-33 Problem 43.

x

46 Vector : a has a magnitude of 5.0 m and is directed east. : Vector b has a magnitude of 4.0 m and is directed 35° west of due : a # b? north. What are (a) the magnitude and (b) the direction of : : : What are (c) the magnitude and (d) the direction of b $ a ? (e) Draw a vector diagram for each combination. :

:

:

47 Vectors A and B lie in an xy plane. A has magnitude 8.00 : and angle 130°; B has components Bx " $7.72 and By " $9.20. What are the angles between the negative direction of the y axis : and (a) the direction of A , (b) the direction of the product : : : : A ' B, and (c) the direction of A ' (B # 3.00kˆ)? :

48 Two vectors : a and b have the components, in meters, ax " 3.2, ay " 1.6, bx " 0.50, by " 4.5. (a) Find the angle between : the directions of : a and b . There are two vectors in the xy plane that : are perpendicular to a and have a magnitude of 5.0 m. One, vector : : c , has a positive x component and the other, vector d , a negative x component. What are (b) the x component and (c) the y component of vector : c , and (d) the x component and (e) the y component : of vector d ? 49 SSM A sailboat sets out from the U.S. side of Lake Erie for a point on the Canadian side, 90.0 km due north. The sailor, however, ends up 50.0 km due east of the starting point. (a) How far and (b) in what direction must the sailor now sail to reach the original destination? :

:

50 Vector d1 is in the negative direction of a y axis, and vector d2 is in the positive direction of an x axis. What are the directions of : : (a) d2 /4 and (b) d1 /($4)? What are the magnitudes of products (c) : : : : d1 " d2 and (d) d1 " (d2 /4)? What is the direction of the vector result: : : : ing from (e) d1 ' d2 and (f) d2 ' d1? What is the magnitude of the vector product in (g) part (e) and (h) part (f)? What are the (i) : : magnitude and (j) direction of d1 ' (d2/4)?

60

CHAPTE R 3 VECTORS

51 Rock faults are ruptures along which opposite faces of rock have slid past each other. In Fig. 3-35, points A and B coincided before the rock in the foreground slid down to the right. The net dis9: placement AB is along the plane of the fault. The horizontal compo9: 9: nent of AB is the strike-slip AC. The component of AB that is directed down the plane of the fault is the dip-slip AD. (a) What is the 9: magnitude of the net displacement AB if the strike-slip is 22.0 m and the dip-slip is 17.0 m? (b) If the plane of the fault is inclined at angle 9: 4 " 52.0° to the horizontal, what is the vertical component of AB ? Strike-slip C

Dip-slip

:

59 A has the magnitude 12.0 m and is angled 60.0° counterclockwise from the positive direction of the x axis of an xy coordinate : system. Also, B " (12.0 m)iˆ # (8.00 m)jˆ on that same coordinate system. We now rotate the system counterclockwise about the origin : by 20.0° to form an x/y/ system. On this new system, what are (a) A : and (b) B, both in unit-vector notation? : : 60 If : a $ b " 2: c, : a # b " 4: c , and : c " 3iˆ # 4jˆ , then what are : : (a) a and (b) b ? :

61 (a) In unit-vector notation, what is : r ": a$b#: c if : : a " 5.0ˆi # 4.0ˆj $ 6.0kˆ, b " $2.0ˆi # 2.0ˆj # 3.0kˆ, and : c " 4.0ˆi # 3.0ˆj # 2.0kˆ? (b) Calculate the angle between : r and the positive z : axis. (c) What is the component of : a along the direction of b ? (d) : What is the component of : a perpendicular to the direction of b but : : in the plane of a and b ? (Hint: For (c), see Eq. 3-20 and Fig. 3-18; for (d), see Eq. 3-24.)

B

A

:

58 A vector d has a magnitude of 2.5 m and points north. What : are (a) the magnitude and (b) the direction of 4.0d ? What are (c) : the magnitude and (d) the direction of $3.0d ?

D

φ Fault plane

Figure 3-35 Problem 51. Here are three displacements, each measured in meters: : : " 4.0iˆ # 5.0jˆ $ 6.0kˆ, d2 " $1.0iˆ # 2.0jˆ # 3.0kˆ, and d3 " : : : 4.0iˆ # 3.0jˆ # 2.0kˆ. (a) What is : r " d1 $ d2 # d3? (b) What is the angle between : r and the positive z axis? (c) What is the compo: : nent of d1 along the direction of d2? (d) What is the component of : : : d1 that is perpendicular to the direction of d2 and in the plane of d1 : and d2? (Hint: For (c), consider Eq. 3-20 and Fig. 3-18; for (d), consider Eq. 3-24.) 52

: d1

:

53 SSM A vector : a of magnitude 10 units and another vector b of magnitude 6.0 units differ in directions by 60°. Find (a) the scalar product of the two vectors and (b) the magnitude of the vec: tor product : a ' b. 54 For the vectors in Fig. 3-32, with a " 4, b " 3, and c " 5, calcu: : late (a) : a " b , (b) : a ": c , and (c) b " : c. 55 A particle undergoes three successive displacements in a : : plane, as follows: d1, 4.00 m southwest; then d2, 5.00 m east; and : finally d3, 6.00 m in a direction 60.0° north of east. Choose a coordinate system with the y axis pointing north and the x axis pointing : east. What are (a) the x component and (b) the y component of d1? : What are (c) the x component and (d) the y component of d2? : What are (e) the x component and (f) the y component of d3? Next, consider the net displacement of the particle for the three successive displacements. What are (g) the x component, (h) the y component, (i) the magnitude, and ( j) the direction of the net displacement? If the particle is to return directly to the starting point, (k) how far and (l) in what direction should it move? 56 Find the sum of the following four vectors in (a) unit-vector notation, and as (b) a magnitude and (c) an angle relative to #x. :

P: 10.0 m, at 25.0° counterclockwise from #x : Q: 12.0 m, at 10.0° counterclockwise from #y : R: 8.00 m, at 20.0° clockwise from $y :

S : 9.00 m, at 40.0° counterclockwise from $y

: : : 57 SSM If B is added to A, the result is 6.0ˆi # 1.0ˆj . If B is subtracted : : from A, the result is $4.0ˆi # 7.0ˆj .What is the magnitude of A?

62 A golfer takes three putts to get the ball into the hole. The first putt displaces the ball 3.66 m north, the second 1.83 m southeast, and the third 0.91 m southwest. What are (a) the magnitude and (b) the direction of the displacement needed to get the ball into the hole on the first putt? 63

Here are three vectors in meters: : d1

" $3.0iˆ # 3.0jˆ # 2.0kˆ

: d2

" $2.0iˆ $ 4.0jˆ # 2.0kˆ

: d3

" 2.0iˆ # 3.0jˆ # 1.0kˆ . :

:

:

:

:

:

What results from (a) d1 " (d2 # d3), (b) d1 " (d2 ' d3), and : : : (c) d1 ' (d2 # d3)? 64 SSM WWW A room has dimensions 3.00 m (height) ' 3.70 m ' 4.30 m. A fly starting at one corner flies around, ending up at the diagonally opposite corner. (a) What is the magnitude of its displacement? (b) Could the length of its path be less than this magnitude? (c) Greater? (d) Equal? (e) Choose a suitable coordinate system and express the components of the displacement vector in that system in unit-vector notation. (f) If the fly walks, what is the length of the shortest path? (Hint: This can be answered without calculus. The room is like a box. Unfold its walls to flatten them into a plane.) 65 A protester carries his sign of protest, starting from the origin of an xyz coordinate system, with the xy plane horizontal. He moves 40 m in the negative direction of the x axis, then 20 m along a perpendicular path to his left, and then 25 m up a water tower. (a) In unit-vector notation, what is the displacement of the sign from start to end? (b) The sign then falls to the foot of the tower. What is the magnitude of the displacement of the sign from start to this new end? :

66 Consider : a in the positive direction of x, b in the positive di: rection of y, and a scalar d. What is the direction of b /d if d is : (a) positive and (b) negative? What is the magnitude of (c) : a"b : : and (d) a " b /d? What is the direction of the vector resulting from : : (e) : a ' b and (f) b ' : a ? (g) What is the magnitude of the vector product in (e)? (h) What is the magnitude of the vector product in : (f)? What are (i) the magnitude and (j) the direction of : a ' b /d if d is positive?

PROB LE M S

67 Let ˆi be directed to the east, ˆj be directed to the north, and kˆ be directed upward. What are the values of products (a) ˆi " kˆ, (b) ($kˆ) " ($ˆj), and (c) ˆj " ($ˆj)? What are the directions (such as east or down) of products (d) kˆ ' ˆj, (e) ($ˆi) ' ($ˆj), and (f) ($kˆ) ' ($ˆj)? 68 A bank in downtown Boston is robbed (see the map in Fig. 3-36). To elude police, the robbers escape by helicopter, making three successive flights described by the following displacements: 32 km, 45° south of east; 53 km, 26° north of west; 26 km, 18° east of south. At the end of the third flight they are captured. In what town are they apprehended? N

BOSTON and Vicinity 5

Salem

10 km

Bank

Medford

Waltham

Wellesley

:

Lynn

Arlington

Newton Brookline

Winthrop BOSTON

Massachusetts Bay

Framingham Dedham

:

:

a lies in the yz plane 63.03 from the positive direction 74 Vector : of the y axis, has a positive z component, and has magnitude 3.20 : units. Vector b lies in the xz plane 48.03 from the positive direction of the x axis, has a positive z component, and has magnitude 1.40 : : : units. Find (a) : a " b , (b) : a ' b , and (c) the angle between : a and b . 75 Find (a) “north cross west,” (b) “down dot south,” (c) “east cross up,” (d) “west dot west,” and (e) “south cross south.” Let each “vector” have unit magnitude. :

Walpole

Figure 3-36 Problem 68. P

P At time t1

:

77 A man goes for a walk, starting from the origin of an xyz coordinate system, with the xy plane horizontal and the x axis eastward. Carrying a bad penny, he walks 1300 m east, 2200 m north, and then drops the penny from a cliff 410 m high. (a) In unit-vector notation, what is the displacement of the penny from start to its landing point? (b) When the man returns to the origin, what is the magnitude of his displacement for the return trip? :

At time t2

Figure 3-37 Problem 69.

78 What is the magnitude of : a ' (b ' : a ) if a " 3.90, b " 2.70, and the angle between the two vectors is 63.0°? :

79 In Fig. 3-38, the magnitude of : a is 4.3, the magnitude of b is 5.4, and 4 " 46°. Find the area of the triangle contained between the two vectors and the thin diagonal line.

70 A woman walks 250 m in the direction 30° east of north, then 175 m directly east. Find (a) the magnitude and (b) the angle of her final displacement from the starting point. (c) Find the distance she walks. (d) Which is greater, that distance or the magnitude of her displacement? :

:

76 A vector B, with a magnitude of 8.0 m, is added to a vector A, which lies along an x axis. The sum of these two vectors is a third vector that lies along the y axis and has a magnitude that is twice : : the magnitude of A. What is the magnitude of A?

Quincy Weymouth

69 A wheel with a radius of 45.0 cm rolls without slipping along a horizontal floor (Fig. 3-37). At time t1, the dot P painted on the rim of the wheel is at the point of contact between the wheel and the floor. At a later time t2, the wheel has rolled through one-half of a revolution. What are (a) the magnitude and (b) the angle (relative to the floor) of the displacement of P?

72 A fire ant, searching for hot sauce in a picnic area, goes : through three displacements along level ground: d l for 0.40 m southwest (that is, at 45° from directly south and from directly : : west), d 2 for 0.50 m due east, d 3 for 0.60 m at 60° north of east. Let the positive x direction be east and the positive y direction be north. What are (a) the x component and (b) the y compo: nent of d l? Next, what are (c) the x component and (d) the y : component of d 2? Also, what are (e) the x component and (f) : the y component of d 3? What are (g) the x component, (h) the y component, (i) the magnitude, and (j) the direction of the ant’s net displacement? If the ant is to return directly to the starting point, (k) how far and (1) in what direction should it move? : 73 Two vectors are given by : a " 3.0iˆ # 5.0ˆj and b " 2.0iˆ # 4.0ˆj. : a ' b , (b) : a " b , (c) (a # b ) " b , and (d) the component of Find (a) : : : a along the direction of b .

Woburn

Lexington

61

71 A vector d has a magnitude 3.0 m and is directed south. What : are (a) the magnitude and (b) the direction of the vector 5.0 d ? What : are (c) the magnitude and (d) the direction of the vector $2.0 d ?

b f

a

Figure 3-38 Problem 79.

C

H

A

P

T

E

R

4

Motion in Two and Three Dimensions 4-1 POSITION AND DISPLACEMENT Learning Objectives After reading this module, you should be able to . . .

4.01 Draw two-dimensional and three-dimensional position vectors for a particle, indicating the components along the axes of a coordinate system. 4.02 On a coordinate system, determine the direction and

magnitude of a particle’s position vector from its components, and vice versa. 4.03 Apply the relationship between a particle’s displacement vector and its initial and final position vectors.

Key Ideas ● The location of a particle relative to the origin of a coordi: nate system is given by a position vector r , which in unitvector notation is

one or two angles for orientation, or by its vector or scalar components. ● If a particle moves so that its position vector changes from : r1

r " xiˆ # yjˆ # zkˆ .

:

Here xˆi , yjˆ, and zkˆ are the vector components of position vector : r , and x, y, and z are its scalar components (as well as the coordinates of the particle).

to : r 2, the particle’s displacement -: r is -: r ": r2 $ : r 1.

The displacement can also be written as

● A position vector is described either by a magnitude and

-: r " (x2 $ x1)iˆ # ( y2 $ y1)jˆ # (z2 $ z1)kˆ " -xiˆ # -yjˆ # -zkˆ .

What Is Physics? In this chapter we continue looking at the aspect of physics that analyzes motion, but now the motion can be in two or three dimensions. For example, medical researchers and aeronautical engineers might concentrate on the physics of the two- and three-dimensional turns taken by fighter pilots in dogfights because a modern high-performance jet can take a tight turn so quickly that the pilot immediately loses consciousness. A sports engineer might focus on the physics of basketball. For example, in a free throw (where a player gets an uncontested shot at the basket from about 4.3 m), a player might employ the overhand push shot, in which the ball is pushed away from about shoulder height and then released. Or the player might use an underhand loop shot, in which the ball is brought upward from about the belt-line level and released. The first technique is the overwhelming choice among professional players, but the legendary Rick Barry set the record for free-throw shooting with the underhand technique. Motion in three dimensions is not easy to understand. For example, you are probably good at driving a car along a freeway (one-dimensional motion) but would probably have a difficult time in landing an airplane on a runway (threedimensional motion) without a lot of training. In our study of two- and three-dimensional motion, we start with position and displacement. 62

63

4-1 POSITION AN D DISPL ACE M E NT

Position and Displacement

To locate the particle, this is how far parallel to z.

One general way of locating a particle (or particle-like object) is with a position vector : r , which is a vector that extends from a reference point (usually the origin) to the particle. In the unit-vector notation of Module 3-2, : r can be written r " xiˆ # yjˆ # zkˆ ,

This is how far parallel to y.

(4-1)

:

where xˆi , yˆj , and zkˆ are the vector components of r and the coefficients x, y, and z are its scalar components. The coefficients x, y, and z give the particle’s location along the coordinate axes and relative to the origin; that is, the particle has the rectangular coordinates (x, y, z). For instance, Fig. 4-1 shows a particle with position vector :

y

This is how far parallel to x. (2 m)jˆ (–3 m)iˆ

ˆ (5 m)k

r " ($3 m)iˆ # (2 m)jˆ # (5 m)kˆ

r

and rectangular coordinates ($3 m, 2 m, 5 m). Along the x axis the particle is 3 m from the origin, in the $iˆ direction. Along the y axis it is 2 m from the origin, in the #jˆ direction. Along the z axis it is 5 m from the origin, in the #kˆ direction. As a particle moves, its position vector changes in such a way that the vector always extends to the particle from the reference point (the origin). If the posi: tion vector changes—say, from : r 1 to r 2 during a certain time interval—then the : particle’s displacement - r during that time interval is :

:

x

O

:

:

- r " r 2 $ r 1.

z

Figure 4-1 The position vector : r for a particle is the vector sum of its vector components.

(4-2)

Using the unit-vector notation of Eq. 4-1, we can rewrite this displacement as - r " (x2 iˆ # y2 jˆ # z2 kˆ ) $ (x1iˆ # y1 jˆ # z1 kˆ ) :

or as

: - r " (x2 $ x1)iˆ # (y2 $ y1)jˆ # (z2 $ z1)kˆ ,

(4-3) :

where coordinates (x1, y1, z1) correspond to position vector r 1 and coordinates : (x2, y2, z2) correspond to position vector r 2 . We can also rewrite the displacement by substituting -x for (x2 $ x1), -y for (y2 $ y1), and -z for (z2 $ z1): -: r " -xiˆ # -y jˆ # -zkˆ .

(4-4)

Sample Problem 4.01 Two-dimensional position vector, rabbit run A rabbit runs across a parking lot on which a set of coordinate axes has, strangely enough, been drawn. The coordinates (meters) of the rabbit’s position as functions of time t (seconds) are given by and

x " $0.31t 2 # 7.2t # 28

(4-5)

y " 0.22t 2 $ 9.1t # 30.

(4-6)

(a) At t " 15 s, what is the rabbit’s position vector : r in unitvector notation and in magnitude-angle notation?

position vector : r . Let’s evaluate those coordinates at the given time, and then we can use Eq. 3-6 to evaluate the magnitude and orientation of the position vector. Calculations: We can write r (t) " x(t)iˆ # y(t)jˆ .

:

(We write r (t) rather than r because the components are : functions of t, and thus r is also.) At t " 15 s, the scalar components are x " ($0.31)(15)2 # (7.2)(15) # 28 " 66 m

KEY IDEA The x and y coordinates of the rabbit’s position, as given by Eqs. 4-5 and 4-6, are the scalar components of the rabbit’s

(4-7)

:

:

and so

y " (0.22)(15)2 $ (9.1)(15) # 30 " $57 m, r " (66 m)iˆ $ (57 m)jˆ ,

:

(Answer)

64

CHAPTE R 4 M OTION I N TWO AN D TH R E E DI M E NSIONS

y (m)

y (m)

40

40

To locate the rabbit, this is the x component.

20 –41° 20

0

40

60

80

20

x (m)

–60 (b)

This is the y component.

which is drawn in Fig. 4-2a. To get the magnitude and angle of : r , notice that the components form the legs of a right triangle and r is the hypotenuse. So, we use Eq. 3-6: r " 2x2 # y2 " 2(66 m)2 # ($57 m)2 and

1 " tan$1

#

60

80

x (m)

5s

10 s

–40

r

–60

" 87 m,

40

–20

–40

(a)

20

0

–20

Figure 4-2 (a) A rabbit’s position vector : r at time t " 15 s. The scalar components of : r are shown along the axes. (b) The rabbit’s path and its position at six values of t.

t=0s

25 s

15 s 20 s

This is the path with various times indicated.

Check: Although u " 139° has the same tangent as $41°, the components of position vector : r indicate that the desired angle is 139° $ 180° " $41°. (b) Graph the rabbit’s path for t " 0 to t " 25 s.

(Answer)

$

y $57 m " tan$1 " $413 . (Answer) x 66 m

Graphing: We have located the rabbit at one instant, but to see its path we need a graph. So we repeat part (a) for several values of t and then plot the results. Figure 4-2b shows the plots for six values of t and the path connecting them.

Additional examples, video, and practice available at WileyPLUS

4-2 AVERAGE VELOCITY AND INSTANTANEOUS VELOCITY Learning Objectives After reading this module, you should be able to . . .

4.04 Identify that velocity is a vector quantity and thus has both magnitude and direction and also has components. 4.05 Draw two-dimensional and three-dimensional velocity vectors for a particle, indicating the components along the axes of the coordinate system.

4.06 In magnitude-angle and unit-vector notations, relate a particle’s initial and final position vectors, the time interval between those positions, and the particle’s average velocity vector. 4.07 Given a particle’s position vector as a function of time, determine its (instantaneous) velocity vector.

Key Ideas :

● If a particle undergoes a displacement - r

its average velocity

: vavg

in time interval -t, for that time interval is :

: vavg

-r " . -t

:

● As -t is shrunk to 0, vavg reaches a limit called either the

velocity or the instantaneous velocity : v: d: r : v" , dt

which can be rewritten in unit-vector notation as v " vx ˆi # vy ˆj # vzkˆ ,

:

where vx " dx/dt, vy " dy/dt, and vz " dz/dt. ● The instantaneous velocity

:

v of a particle is always directed along the tangent to the particle’s path at the particle’s position.

4-2 AVE RAG E VE LOCITY AN D I NSTANTAN EOUS VE LOCITY

Average Velocity and Instantaneous Velocity If a particle moves from one point to another, we might need to know how fast it moves. Just as in Chapter 2, we can define two quantities that deal with “how fast”: average velocity and instantaneous velocity. However, here we must consider these quantities as vectors and use vector notation. If a particle moves through a displacement -: r in a time interval -t, then its average velocity : v avg is displacement average velocity " , time interval :

: v avg

or

"

-r . -t

(4-8)

:

This tells us that the direction of v avg (the vector on the left side of Eq. 4-8) must : be the same as that of the displacement - r (the vector on the right side). Using Eq. 4-4, we can write Eq. 4-8 in vector components as -x ˆ -y ˆ -z ˆ -xiˆ # -yjˆ # -zkˆ (4-9) " i # j # k. -t -t -t -t For example, if a particle moves through displacement (12 m)iˆ # (3.0 m)kˆ in 2.0 s, then its average velocity during that move is : v avg

: v avg

"

"

-: r (12 m)iˆ # (3.0 m)kˆ " " (6.0 m/s)iˆ # (1.5 m/s)kˆ . -t 2.0 s

That is, the average velocity (a vector quantity) has a component of 6.0 m/s along the x axis and a component of 1.5 m/s along the z axis. When we speak of the velocity of a particle, we usually mean the particle’s instantaneous velocity : v at some instant. This : v is the value that : v avg approaches in the limit as we shrink the time interval -t to 0 about that instant. Using the language of calculus, we may write : v as the derivative :

:

v"

dr . dt

(4-10)

Figure 4-3 shows the path of a particle that is restricted to the xy plane. As the particle travels to the right along the curve, its position vector sweeps to the right. During time interval -t, the position vector changes from : r 1 to : r 2 and the : particle’s displacement is - r . To find the instantaneous velocity of the particle at, say, instant t1 (when the particle is at position 1), we shrink interval -t to 0 about t1. Three things happen as we do so. (1) Position vector : r 2 in Fig. 4-3 moves toward : r 1 so that -: r shrinks

As the particle moves, the position vector must change.

y

Tangent 1 :

Figure 4-3 The displacement - r of a particle during a time interval -t, from position 1 with position vector : r 1 at time t1 to position 2 r 2 at time t2. The tangent with position vector : to the particle’s path at position 1 is shown.

∆r r1

2

This is the displacement.

r2 Path

O

x

65

66

CHAPTE R 4 M OTION I N TWO AN D TH R E E DI M E NSIONS :

:

toward zero. (2) The direction of - r /-t (and thus of v avg) approaches the direction of the line tangent to the particle’s path at position 1. (3) The average : : velocity v avg approaches the instantaneous velocity v at t1. : : In the limit as -t : 0, we have v avg : v and, most important here, : v avg takes : on the direction of the tangent line. Thus, v has that direction as well: v of a particle is always tangent to the The direction of the instantaneous velocity : particle’s path at the particle’s position. :

The result is the same in three dimensions: v is always tangent to the particle’s path. : To write Eq. 4-10 in unit-vector form, we substitute for r from Eq. 4-1: :

v"

dx ˆ dy ˆ dz ˆ d (xiˆ # yjˆ # zkˆ ) " i # j # k. dt dt dt dt

This equation can be simplified somewhat by writing it as v " vx iˆ # vy jˆ # vz kˆ ,

:

(4-11)

where the scalar components of : v are vx "

dx dy , vy " , and dt dt

vz "

dz . dt

(4-12)

:

For example, dx/dt is the scalar component of v along the x axis. Thus, we can find : : the scalar components of v by differentiating the scalar components of r . : Figure 4-4 shows a velocity vector v and its scalar x and y components. Note that : v is tangent to the particle’s path at the particle’s position. Caution: When a position vector is drawn, as in Figs. 4-1 through 4-3, it is an arrow that extends from one point (a “here”) to another point (a “there”). However, when a velocity vector is drawn, as in Fig. 4-4, it does not extend from one point to another. Rather, it shows the instantaneous direction of travel of a particle at the tail, and its length (representing the velocity magnitude) can be drawn to any scale. The velocity vector is always tangent to the path. y Tangent vy

v

vx

Figure 4-4 The velocity : v of a particle, along with the scalar components of : v.

These are the x and y components of the vector at this instant.

Path O

Checkpoint 1 The figure shows a circular path taken by a particle. : If the instantaneous velocity of the particle is v " ˆ ˆ (2 m /s)i $ (2 m /s)j , through which quadrant is the particle moving at that instant if it is traveling (a) clockwise and (b) counterclockwise around the circle? For both cases, draw : v on the figure.

x y

x

67

4-3 AVE RAG E ACCE LE RATION AN D I NSTANTAN EOUS ACCE LE RATION

Sample Problem 4.02 Two-dimensional velocity, rabbit run For the rabbit in the preceding sample problem, find the velocity : v at time t " 15 s. KEY IDEA

v " 2v 2x # v 2y " 2($2.1 m /s)2 # ($2.5 m /s)2 " 3.3 m /s

and

We can find : v by taking derivatives of the components of the rabbit’s position vector. Calculations: Applying the vx part of Eq. 4-12 to Eq. 4-5, we find the x component of : v to be vx "

1 " tan$1

(Answer)

20

0

20

40

60

80

x (m)

–20 –40

(4-14)

At t " 15 s, this gives vy " $2.5 m/s. Equation 4-11 then yields v " ($2.1 m /s)iˆ # ($2.5 m /s)jˆ ,

$

40

d dy " (0.22t 2 $ 9.1t # 30) dt dt

:

#

y (m)

(4-13)

" 0.44t $ 9.1.

" tan$1

Check: Is the angle $130° or $130° # 180° " 50°?

At t " 15 s, this gives vx " $2.1 m/s. Similarly, applying the vy part of Eq. 4-12 to Eq. 4-6, we find vy "

vx

$2.5 m /s $2.1 m /s

" tan$1 1.19 " $1303.

d dx " ($0.31t 2 # 7.2t # 28) dt dt

" $0.62t # 7.2.

(Answer)

vy

(Answer)

which is shown in Fig. 4-5, tangent to the rabbit’s path and in the direction the rabbit is running at t " 15 s. To get the magnitude and angle of : v , either we use a vector-capable calculator or we follow Eq. 3-6 to write

x

–60

These are the x and y components of the vector at this instant.

v

–130°

Figure 4-5 The rabbit’s velocity : v at t " 15 s.

Additional examples, video, and practice available at WileyPLUS

4-3 AVERAGE ACCELERATION AND INSTANTANEOUS ACCELERATION Learning Objectives After reading this module, you should be able to . . .

4.08 Identify that acceleration is a vector quantity and thus has both magnitude and direction and also has components. 4.09 Draw two-dimensional and three-dimensional acceleration vectors for a particle, indicating the components. 4.10 Given the initial and final velocity vectors of a particle and the time interval between those velocities, determine

the average acceleration vector in magnitude-angle and unit-vector notations. 4.11 Given a particle’s velocity vector as a function of time, determine its (instantaneous) acceleration vector. 4.12 For each dimension of motion, apply the constantacceleration equations (Chapter 2) to relate acceleration, velocity, position, and time.

Key Ideas ● If a particle’s velocity changes from

: v1

-t, its average acceleration during -t is :

a avg "

● As

: v2

to : v 2 in time interval

: $: v1 -v " . -t -t

-t is shrunk to 0, : aavg reaches a limiting value called

:

either the acceleration or the instantaneous acceleration a : : dv : . a" dt ● In unit-vector notation, : a " ax iˆ # ay jˆ # azkˆ , where ax " dvx /dt, ay " dvy /dt, and az " dvz /dt.

68

CHAPTE R 4 M OTION I N TWO AN D TH R E E DI M E NSIONS

Average Acceleration and Instantaneous Acceleration When a particle’s velocity changes from : v 1 to : v 2 in a time interval -t, its average : acceleration a avg during -t is average change in velocity , " acceleration time interval : :

or

a avg "

: : -v v2 $ v1 " . -t -t

(4-15)

If we shrink -t to zero about some instant, then in the limit : a avg approaches the instantaneous acceleration (or acceleration) : a at that instant; that is, :

:

a"

dv . dt

(4-16)

If the velocity changes in either magnitude or direction (or both), the particle must have an acceleration. We can write Eq. 4-16 in unit-vector form by substituting Eq. 4-11 for : v to obtain :

a" "

d (vx iˆ # vy jˆ # vz kˆ ) dt dvx dvy dvz iˆ # jˆ # kˆ . dt dt dt

We can rewrite this as a " ax iˆ # ay jˆ # az kˆ,

:

(4-17)

where the scalar components of : a are

ax "

dvx , dt

ay "

dvy , dt

and

dvz . dt

az "

(4-18)

: To find the scalar components of : a , we differentiate the scalar components of v . : Figure 4-6 shows an acceleration vector a and its scalar components for a particle moving in two dimensions. Caution: When an acceleration vector is drawn, as in Fig. 4-6, it does not extend from one position to another. Rather, it shows the direction of acceleration for a particle located at its tail, and its length (representing the acceleration magnitude) can be drawn to any scale.

y

These are the x and y components of the vector at this instant. ax ay

Figure 4-6 The acceleration : a of a particle and the scalar components of : a.

a

Path O

x

4-3 AVE RAG E ACCE LE RATION AN D I NSTANTAN EOUS ACCE LE RATION

69

Checkpoint 2 Here are four descriptions of the position (in meters) of a puck as it moves in an xy plane: (1) x " $3t 2 # 4t $ 2 and y " 6t 2 $ 4t (3) : r " 2t 2 iˆ $ (4t # 3)jˆ (2) x " $3t 3 $ 4t and y " $5t 2 # 6

(4) : r " (4t 3 $ 2t)iˆ # 3jˆ

Are the x and y acceleration components constant? Is acceleration : a constant?

Sample Problem 4.03 Two-dimensional acceleration, rabbit run For the rabbit in the preceding two sample problems, find the acceleration : a at time t " 15 s.

has the same tangent as $35° but is not displayed on a calculator, we add 180°: $35° # 180° " 145°. (Answer)

KEY IDEA

This is consistent with the components of : a because it gives a vector that is to the left and upward. Note that : a has the same magnitude and direction throughout the rabbit’s run because the acceleration is constant. That means that we could draw the very same vector at any other point along the rabbit’s path (just shift the vector to put its tail at some other point on the path without changing the length or orientation). This has been the second sample problem in which we needed to take the derivative of a vector that is written in unit-vector notation. One common error is to neglect the unit vectors themselves, with a result of only a set of numbers and symbols. Keep in mind that a derivative of a vector is always another vector.

We can find : a by taking derivatives of the rabbit’s velocity components. Calculations: Applying the ax part of Eq. 4-18 to Eq. 4-13, we find the x component of : a to be d dvx " ($0.62t # 7.2) " $0.62 m /s2. dt dt Similarly, applying the ay part of Eq. 4-18 to Eq. 4-14 yields the y component as ax "

ay "

d dvy " (0.44t $ 9.1) " 0.44 m /s2. dt dt

We see that the acceleration does not vary with time (it is a constant) because the time variable t does not appear in the expression for either acceleration component. Equation 4-17 then yields a " ($0.62 m /s2)iˆ # (0.44 m /s2)jˆ ,

:

(Answer)

which is superimposed on the rabbit’s path in Fig. 4-7. To get the magnitude and angle of : a , either we use a vector-capable calculator or we follow Eq. 3-6. For the magnitude we have a " 2a 2x # a 2y " 2($0.62 m /s2)2 # (0.44 m /s2)2 " 0.76 m/s2.

(Answer)

For the angle we have

1 " tan$1

ay ax

y (m) 40 20

0

20

40

60

80

–20 –40

a

145° x

–60

" tan$1

#

$

0.44 m/s2 " $353. $0.62 m/s2

However, this angle, which is the one displayed on a calculator, indicates that : a is directed to the right and downward in Fig. 4-7. Yet, we know from the components that : a must be directed to the left and upward. To find the other angle that

x (m)

These are the x and y components of the vector at this instant. Figure 4-7 The acceleration : a of the rabbit at t " 15 s. The rabbit happens to have this same acceleration at all points on its path.

Additional examples, video, and practice available at WileyPLUS

70

CHAPTE R 4 M OTION I N TWO AN D TH R E E DI M E NSIONS

4-4 PROJECTILE MOTION Learning Objectives After reading this module, you should be able to . . .

4.13 On a sketch of the path taken in projectile motion, explain the magnitudes and directions of the velocity and acceleration components during the flight.

4.14 Given the launch velocity in either magnitude-angle or unit-vector notation, calculate the particle’s position, displacement, and velocity at a given instant during the flight. 4.15 Given data for an instant during the flight, calculate the launch velocity.

Key Ideas ● In projectile motion, a particle is launched into the air with a

● The trajectory (path) of a particle in projectile motion is par-

speed v0 and at an angle u0 (as measured from a horizontal x axis). During flight, its horizontal acceleration is zero and its vertical acceleration is $g (downward on a vertical y axis).

abolic and is given by

● The equations of motion for the particle (while in flight) can

if x0 and y0 are zero.

be written as

y " (tan 10)x $

gx2 , 2(v0 cos 10)2

● The particle’s horizontal range R, which is the horizontal

x $ x0 " (v0 cos 10)t, y $ y0 " (v0 sin 10)t $ 12 gt 2, vy " v0 sin 10 $ gt, v 2y " (v0 sin 10 )2 $ 2g(y $ y0).

distance from the launch point to the point at which the particle returns to the launch height, is v2 R " 0 sin 210. g

Projectile Motion We next consider a special case of two-dimensional motion: A particle moves in a vertical plane with some initial velocity : v 0 but its acceleration is always the freefall acceleration : g , which is downward. Such a particle is called a projectile (meaning that it is projected or launched), and its motion is called projectile motion. A projectile might be a tennis ball (Fig. 4-8) or baseball in flight, but it is not a duck in flight. Many sports involve the study of the projectile motion of a ball. For example, the racquetball player who discovered the Z-shot in the 1970s easily won his games because of the ball’s perplexing flight to the rear of the court. Our goal here is to analyze projectile motion using the tools for twodimensional motion described in Module 4-1 through 4-3 and making the assumption that air has no effect on the projectile. Figure 4-9, which we shall analyze soon, shows the path followed by a projectile when the air has no effect. The projectile is launched with an initial velocity : v 0 that can be written as : v0

" v0x iˆ # v0y jˆ .

(4-19)

The components v0x and v0y can then be found if we know the angle u0 between : v0 and the positive x direction: v0x " v0 cos u0 Richard Megna/Fundamental Photographs

Figure 4-8 A stroboscopic photograph of a yellow tennis ball bouncing off a hard surface. Between impacts, the ball has projectile motion.

and

v0y " v0 sin u0.

(4-20)

r and velocity During its two-dimensional motion, the projectile’s position vector : a is constant and always v change continuously, but its acceleration vector : vector : directed vertically downward.The projectile has no horizontal acceleration. Projectile motion, like that in Figs. 4-8 and 4-9, looks complicated, but we have the following simplifying feature (known from experiment): In projectile motion, the horizontal motion and the vertical motion are independent of each other; that is, neither motion affects the other.

71

4-4 PROJ ECTI LE M OTION

A

y

O

+

y Vertical motion



Horizontal motion This vertical motion plus this horizontal motion produces this projectile motion.

v0y O

O

x

Projectile motion

v0

v0y

Vertical velocity v0x

y

θ0

Launch velocity Launch angle

x

v0x

O

Launch

Launch

y

y

vy

vy

Speed decreasing

O

O

v

vx

x

vx

x

O

Constant velocity y

y vy = 0

Stopped at maximum height

vx O

O

x

Constant velocity

x

O

y

y vx

Speed increasing

vy

vy

vx O

v

vy = 0

x

O

v

x

O

Constant velocity y

y

vx vy

O

Constant velocity

x

vx O

vy

θ v

Figure 4-9 The projectile motion of an object launched into the air at the origin of a coordinate system and with launch velocity : v 0 at angle u0. The motion is a combination of vertical motion (constant acceleration) and horizontal motion (constant velocity), as shown by the velocity components.

x

72

CHAPTE R 4 M OTION I N TWO AN D TH R E E DI M E NSIONS

This feature allows us to break up a problem involving two-dimensional motion into two separate and easier one-dimensional problems, one for the horizontal motion (with zero acceleration) and one for the vertical motion (with constant downward acceleration). Here are two experiments that show that the horizontal motion and the vertical motion are independent.

Two Golf Balls Figure 4-10 is a stroboscopic photograph of two golf balls, one simply released and the other shot horizontally by a spring.The golf balls have the same vertical motion, both falling through the same vertical distance in the same interval of time. The fact that one ball is moving horizontally while it is falling has no effect on its vertical motion; that is, the horizontal and vertical motions are independent of each other.

A Great Student Rouser

Richard Megna/Fundamental Photographs

Figure 4-10 One ball is released from rest at the same instant that another ball is shot horizontally to the right. Their vertical motions are identical.

The ball and the can fall the same distance h.

Checkpoint 3

M

r

g o-

Ze

p

h at

In Fig. 4-11, a blowgun G using a ball as a projectile is aimed directly at a can suspended from a magnet M. Just as the ball leaves the blowgun, the can is released. If g (the magnitude of the free-fall acceleration) were zero, the ball would follow the straight-line path shown in Fig. 4-11 and the can would float in place after the magnet released it. The ball would certainly hit the can. However, g is not zero, but the ball still hits the can! As Fig. 4-11 shows, during the time of flight of the ball, both ball and can fall the same distance h from their zero-g locations. The harder the demonstrator blows, the greater is the ball’s initial speed, the shorter the flight time, and the smaller the value of h.

v " 25iˆ $ 4.9jˆ (the x axis is horizontal, the At a certain instant, a fly ball has velocity : y axis is upward, and : v is in meters per second). Has the ball passed its highest point?

Can h

G

Figure 4-11 The projectile ball always hits the falling can. Each falls a distance h from where it would be were there no free-fall acceleration.

The Horizontal Motion Now we are ready to analyze projectile motion, horizontally and vertically. We start with the horizontal motion. Because there is no acceleration in the horizontal direction, the horizontal component vx of the projectile’s velocity remains unchanged from its initial value v0x throughout the motion, as demonstrated in Fig. 4-12. At any time t, the projectile’s horizontal displacement x $ x0 from an initial position x0 is given by Eq. 2-15 with a " 0, which we write as x $ x0 " v0x t. Because v0x " v0 cos u0, this becomes x $ x0 " (v0 cos u0)t.

(4-21)

The Vertical Motion The vertical motion is the motion we discussed in Module 2-5 for a particle in free fall. Most important is that the acceleration is constant. Thus, the equations of Table 2-1 apply, provided we substitute $g for a and switch to y notation. Then, for example, Eq. 2-15 becomes y $ y0 " v0yt $ 12gt 2 " (v0 sin 10)t $ 12gt 2,

(4-22)

where the initial vertical velocity component v0y is replaced with the equivalent v0 sin u0. Similarly, Eqs. 2-11 and 2-16 become vy " v0 sin u0 $ gt (4-23) and

v2y " (v0 sin 10)2 $ 2g(y $ y0).

(4-24)

73

4-4 PROJ ECTI LE M OTION

As is illustrated in Fig. 4-9 and Eq. 4-23, the vertical velocity component behaves just as for a ball thrown vertically upward. It is directed upward initially, and its magnitude steadily decreases to zero, which marks the maximum height of the path. The vertical velocity component then reverses direction, and its magnitude becomes larger with time.

The Equation of the Path We can find the equation of the projectile’s path (its trajectory) by eliminating time t between Eqs. 4-21 and 4-22. Solving Eq. 4-21 for t and substituting into Eq. 4-22, we obtain, after a little rearrangement, y " (tan 10)x $

gx 2 2(v0 cos 10)2

(trajectory).

(4-25)

This is the equation of the path shown in Fig. 4-9. In deriving it, for simplicity we let x0 " 0 and y0 " 0 in Eqs. 4-21 and 4-22, respectively. Because g, u0, and v0 are constants, Eq. 4-25 is of the form y " ax # bx2, in which a and b are constants. This is the equation of a parabola, so the path is parabolic.

The Horizontal Range The horizontal range R of the projectile is the horizontal distance the projectile has traveled when it returns to its initial height (the height at which it is launched). To find range R, let us put x $ x0 " R in Eq. 4-21 and y $ y0 " 0 in Eq. 4-22, obtaining R " (v0 cos u0)t

Jamie Budge

Figure 4-12 The vertical component of this skateboarder’s velocity is changing but not the horizontal component, which matches the skateboard’s velocity. As a result, the skateboard stays underneath him, allowing him to land on it.

0 " (v0 sin 10)t $ 12gt 2.

and

Air reduces height ...

Eliminating t between these two equations yields 2v20 R" sin 10 cos 10. g

v0

Using the identity sin 2u0 " 2 sin u0 cos u0 (see Appendix E), we obtain R"

v20 sin 210. g

... and range.

y II I 60°

(4-26)

This equation does not give the horizontal distance traveled by a projectile when the final height is not the launch height. Note that R in Eq. 4-26 has its maximum value when sin 2u0 " 1, which corresponds to 2u0 " 90° or u0 " 45°. The horizontal range R is maximum for a launch angle of 45°.

However, when the launch and landing heights differ, as in many sports, a launch angle of 45° does not yield the maximum horizontal distance.

Figure 4-13 (I) The path of a fly ball calculated by taking air resistance into account. (II) The path the ball would follow in a vacuum, calculated by the methods of this chapter. See Table 4-1 for corresponding data. (Based on “The Trajectory of a Fly Ball,” by Peter J. Brancazio, The Physics Teacher, January 1985.)

Table 4-1 Two Fly Ballsa Path I (Air)

The Effects of the Air We have assumed that the air through which the projectile moves has no effect on its motion. However, in many situations, the disagreement between our calculations and the actual motion of the projectile can be large because the air resists (opposes) the motion. Figure 4-13, for example, shows two paths for a fly ball that leaves the bat at an angle of 60° with the horizontal and an initial speed of 44.7 m/s. Path I (the baseball player’s fly ball) is a calculated path that approximates normal conditions of play, in air. Path II (the physics professor’s fly ball) is the path the ball would follow in a vacuum.

x

Range Maximum height Time of flight a

98.5 m

Path II (Vacuum) 177 m

53.0 m

76.8 m

6.6 s

7.9 s

See Fig. 4-13. The launch angle is 60° and the launch speed is 44.7 m/s.

74

CHAPTE R 4 M OTION I N TWO AN D TH R E E DI M E NSIONS

Checkpoint 4 A fly ball is hit to the outfield. During its flight (ignore the effects of the air), what happens to its (a) horizontal and (b) vertical components of velocity? What are the (c) horizontal and (d) vertical components of its acceleration during ascent, during descent, and at the topmost point of its flight?

Sample Problem 4.04 Projectile dropped from airplane In Fig. 4-14, a rescue plane flies at 198 km/h (" 55.0 m/s) and constant height h " 500 m toward a point directly over a victim, where a rescue capsule is to land. (a) What should be the angle f of the pilot’s line of sight to the victim when the capsule release is made?

y v0 O

x

Tr

φ

aje

h

Lin

eo

KEY IDEAS Once released, the capsule is a projectile, so its horizontal and vertical motions can be considered separately (we need not consider the actual curved path of the capsule). Calculations: In Fig. 4-14, we see that f is given by 4 " tan$1

x , h

x $ x0 " (v0 cos u0)t.

(4-28)

Here we know that x0 " 0 because the origin is placed at the point of release. Because the capsule is released and not shot from the plane, its initial velocity : v 0 is equal to the plane’s velocity. Thus, we know also that the initial velocity has magnitude v0 " 55.0 m/s and angle u0 " 0° (measured relative to the positive direction of the x axis). However, we do not know the time t the capsule takes to move from the plane to the victim. To find t, we next consider the vertical motion and specifically Eq. 4-22: 1 2 2 gt .

(4-29)

Here the vertical displacement y $ y0 of the capsule is $500 m (the negative value indicates that the capsule moves downward). So, $500 m " (55.0 m/s)(sin 03)t $ 12 (9.8 m/s2)t 2.

(4-30)

Solving for t, we find t " 10.1 s. Using that value in Eq. 4-28 yields x $ 0 " (55.0 m/s)(cos 0°)(10.1 s), (4-31) or

x " 555.5 m.

f si

ry

gh

t

θ v

Figure 4-14 A plane drops a rescue capsule while moving at constant velocity in level flight. While falling, the capsule remains under the plane.

(4-27)

where x is the horizontal coordinate of the victim (and of the capsule when it hits the water) and h " 500 m. We should be able to find x with Eq. 4-21:

y $ y0 " (v0 sin 10)t $

cto

Then Eq. 4-27 gives us 555.5 m (Answer) " 48.03. 500 m (b) As the capsule reaches the water, what is its velocity : v?

4 " tan$1

KEY IDEAS (1) The horizontal and vertical components of the capsule’s velocity are independent. (2) Component vx does not change from its initial value v0x " v0 cos u0 because there is no horizontal acceleration. (3) Component vy changes from its initial value v0y " v0 sin u0 because there is a vertical acceleration. Calculations: When the capsule reaches the water, vx " v0 cos u0 " (55.0 m/s)(cos 0°) " 55.0 m/s. Using Eq. 4-23 and the capsule’s time of fall t " 10.1 s, we also find that when the capsule reaches the water, vy " v0 sin u0 $ gt " (55.0 m/s)(sin 0°) $ (9.8 m/s2)(10.1 s) " $99.0 m/s. Thus, at the water v " (55.0 m /s)iˆ $ (99.0 m /s)jˆ .

:

(Answer)

From Eq. 3-6, the magnitude and the angle of : v are v " 113 m/s and

Additional examples, video, and practice available at WileyPLUS

u " $60.9°.

(Answer)

75

4-4 PROJ ECTI LE M OTION

Sample Problem 4.05 Launched into the air from a water slide One of the most dramatic videos on the web (but entirely fictitious) supposedly shows a man sliding along a long water slide and then being launched into the air to land in a water pool. Let’s attach some reasonable numbers to such a flight to calculate the velocity with which the man would have hit the water. Figure 4-15a indicates the launch and landing sites and includes a superimposed coordinate system with its origin conveniently located at the launch site. From the video we take the horizontal flight distance as D " 20.0 m, the flight time as t " 2.50 s, and the launch angle as 10 " 40.0°. Find the magnitude of the velocity at launch and at landing.

we know that the horizontal velocity component vx is constant during the flight and thus is always equal to the horizontal component v0x at launch. We can relate that component, the displacement x $ x0, and the flight time t " 2.50 s with Eq. 2-15:

KEY IDEAS

That is a component of the launch velocity, but we need the magnitude of the full vector, as shown in Fig. 4-15b, where the components form the legs of a right triangle and the full vector forms the hypotenuse. We can then apply a trig definition to find the magnitude of the full velocity at launch: v0x , cos10 " v0 and so v0x 8.00 m/s " v0 " cos u0 cos 40$

(1) For projectile motion, we can apply the equations for constant acceleration along the horizontal and vertical axes separately. (2) Throughout the flight, the vertical acceleration is ay " $g " $9.8 m/s and the horizontal acceleration is ax " 0. Calculations: In most projectile problems, the initial challenge is to figure out where to start. There is nothing wrong with trying out various equations, to see if we can somehow get to the velocities. But here is a clue. Because we are going to apply the constant-acceleration equations separately to the x and y motions, we should find the horizontal and vertical components of the velocities at launch and at landing. For each site, we can then combine the velocity components to get the velocity. Because we know the horizontal displacement D " 20.0 m, let’s start with the horizontal motion. Since ax " 0,

x $ x0 " v0xt # 12axt 2.

(4-32)

Substituting ax " 0, this becomes Eq. 4-21. With x $ x0 " D, we then write 20 m " v0x(2.50 s) # 12 (0)(2.50 s)2 v0x " 8.00 m/s.

" 10.44 m/s % 10.4 m/s.

(Answer)

Now let’s go after the magnitude v of the landing velocity. We already know the horizontal component, which does not change from its initial value of 8.00 m/s. To find the vertical component vy and because we know the elapsed time t " 2.50 s and the vertical acceleration ay " $9.8 m/s2, let’s rewrite Eq. 2-11 as

y

vy " v0y # ayt

v0 θ0 x Launch

and then (from Fig. 4-15b) as

Water pool

(4-33)

vy " v0 sin 10 # ayt.

Substituting ay " $g, this becomes Eq. 4-23.We can then write vy " (10.44 m/s) sin (40.0$) $ (9.8 m/s2)(2.50 s)

D (a)

v0

Launch velocity

θ0 v0x

(b)

v0y

" $17.78 m/s. v0x θ0

Landing velocity

v

Now that we know both components of the landing velocity, we use Eq. 3-6 to find the velocity magnitude: vy

(c)

Figure 4-15 (a) Launch from a water slide, to land in a water pool. The velocity at (b) launch and (c) landing.

v " 2v 2x # v 2y

" 2(8.00 m/s)2 # ($17.78 m/s)2 " 19.49 m/s2 % 19.5 m/s.

Additional examples, video, and practice available at WileyPLUS

(Answer)

76

CHAPTE R 4 M OTION I N TWO AN D TH R E E DI M E NSIONS

4-5 UNIFORM CIRCULAR MOTION Learning Objectives After reading this module, you should be able to . . .

4.16 Sketch the path taken in uniform circular motion and explain the velocity and acceleration vectors (magnitude and direction) during the motion.

4.17 Apply the relationships between the radius of the circular path, the period, the particle’s speed, and the particle’s acceleration magnitude.

Key Ideas ● If a particle travels along a circle or circular arc of radius r at

constant speed v, it is said to be in uniform circular motion and has an acceleration : a of constant magnitude v2 a" . r : The direction of a is toward the center of the circle or circular

The acceleration vector always points toward the center. v v

a

a

The velocity vector is always tangent to the path.

a

arc, and : a is said to be centripetal. The time for the particle to complete a circle is 2) r T" . v T is called the period of revolution, or simply the period, of the motion.

Uniform Circular Motion A particle is in uniform circular motion if it travels around a circle or a circular arc at constant (uniform) speed. Although the speed does not vary, the particle is accelerating because the velocity changes in direction. Figure 4-16 shows the relationship between the velocity and acceleration vectors at various stages during uniform circular motion. Both vectors have constant magnitude, but their directions change continuously. The velocity is always directed tangent to the circle in the direction of motion. The acceleration is always directed radially inward. Because of this, the acceleration associated with uniform circular motion is called a centripetal (meaning “center seeking”) acceleration. As we prove next, the magnitude of this acceleration : a is

v

Figure 4-16 Velocity and acceleration vectors for uniform circular motion.

a"

v2 r

(centripetal acceleration),

(4-34)

where r is the radius of the circle and v is the speed of the particle. In addition, during this acceleration at constant speed, the particle travels the circumference of the circle (a distance of 2pr) in time T"

2) r v

(period).

(4-35)

T is called the period of revolution, or simply the period, of the motion. It is, in general, the time for a particle to go around a closed path exactly once.

Proof of Eq. 4-34 To find the magnitude and direction of the acceleration for uniform circular motion, we consider Fig. 4-17. In Fig. 4-17a, particle p moves at constant speed v around a circle of radius r. At the instant shown, p has coordinates xp and yp. Recall from Module 4-2 that the velocity : v of a moving particle is always tangent to the particle’s path at the particle’s position. In Fig. 4-17a, that means : v is perpendicular to a radius r drawn to the particle’s position. Then the angle u that : v makes with a vertical at p equals the angle u that radius r makes with the x axis.

77

4-5 U N I FOR M CI RCU L AR M OTION y

The scalar components of : v are shown in Fig. 4-17b. With them, we can write the velocity : v as v " vx iˆ # vy jˆ " ($v sin 1)iˆ # (v cos 1)jˆ.

:

v

p

(4-36) r

Now, using the right triangle in Fig. 4-17a, we can replace sin u with yp /r and cos u with xp /r to write vxp ˆ vy : j. v " $ p iˆ # (4-37) r r

#

θ xp

$ # $

To find the acceleration : a of particle p, we must take the time derivative of this equation. Noting that speed v and radius r do not change with time, we obtain

#

$ #

: dv v dyp ˆ " $ i# a" dt r dt

:

$

v dxp ˆ j. r dt

#

$ #

$

x

y v

(4-38)

v2 v2 a" $ cos 1 iˆ # $ sin 1 jˆ. r r

yp

(a)

Now note that the rate dyp /dt at which yp changes is equal to the velocity component vy. Similarly, dxp /dt " vx, and, again from Fig. 4-17b, we see that vx " $v sin u and vy " v cos u. Making these substitutions in Eq. 4-38, we find :

θ

θ vy vx

x

(4-39)

This vector and its components are shown in Fig. 4-17c. Following Eq. 3-6, we find a"

2a 2x

#

a 2y

(b)

v2 v2 v2 " 2(cos 1)2 # (sin 1)2 " 11 " , r r r

y

as we wanted to prove. To orient : a , we find the angle f shown in Fig. 4-17c: tan 4 "

ax

ay $(v /r) sin 1 " " tan 1. ax $(v2/r) cos 1 2

a

φ

ay x

Thus, f " u, which means that : a is directed along the radius r of Fig. 4-17a, toward the circle’s center, as we wanted to prove.

Checkpoint 5

(c)

An object moves at constant speed along a circular path in a horizontal xy plane, with the center at the origin. When the object is at x " $2 m, its velocity is $(4 m/s) ˆj. Give the object’s (a) velocity and (b) acceleration at y " 2 m.

Figure 4-17 Particle p moves in counterclockwise uniform circular motion. (a) Its position and velocity : v at a certain instant. (b) Velocity : a. v . (c) Acceleration :

Sample Problem 4.06 Top gun pilots in turns “Top gun” pilots have long worried about taking a turn too tightly. As a pilot’s body undergoes centripetal acceleration, with the head toward the center of curvature, the blood pressure in the brain decreases, leading to loss of brain function. There are several warning signs. When the centripetal acceleration is 2g or 3g, the pilot feels heavy. At about 4g, the pilot’s vision switches to black and white and narrows to “tunnel vision.” If that acceleration is sustained or increased, vision ceases and, soon after, the pilot is unconscious — a condition known as g-LOC for “g-induced loss of consciousness.” What is the magnitude of the acceleration, in g units, of a pilot whose aircraft enters a horizontal circular turn with a velocity of : vi " (400ˆi # 500ˆj) m/s and 24.0 s later leaves the turn with a velocity of : v f " ($400ˆi $ 500 ˆj) m/s?

KEY IDEAS We assume the turn is made with uniform circular motion. Then the pilot’s acceleration is centripetal and has magnitude a given by Eq. 4-34 (a " v2/R), where R is the circle’s radius. Also, the time required to complete a full circle is the period given by Eq. 4-35 (T " 2pR/v). Calculations: Because we do not know radius R, let’s solve Eq. 4-35 for R and substitute into Eq. 4-34. We find 2)v . T To get the constant speed v, let’s substitute the components of the initial velocity into Eq. 3-6: a"

v " 2(400 m/s)2 # (500 m/s)2 " 640.31 m/s.

78

CHAPTE R 4 M OTION I N TWO AN D TH R E E DI M E NSIONS

To find the period T of the motion, first note that the final velocity is the reverse of the initial velocity. This means the aircraft leaves on the opposite side of the circle from the initial point and must have completed half a circle in the given

24.0 s. Thus a full circle would have taken T " 48.0 s. Substituting these values into our equation for a, we find a"

2)(640.31 m/s) " 83.81 m/s2 % 8.6g. 48.0 s

(Answer)

Additional examples, video, and practice available at WileyPLUS

4-6

RELATIVE MOTION IN ONE DIMENSION

Learning Objective After reading this module, you should be able to . . .

4.18 Apply the relationship between a particle’s position, velocity, and acceleration as measured from two reference

frames that move relative to each other at constant velocity and along a single axis.

Key Idea ● When two frames of reference A and B are moving relative

to each other at constant velocity, the velocity of a particle P as measured by an observer in frame A usually differs from that measured from frame B. The two measured velocities are related by

: v PA

:

:

" v PB # v BA,

where : v BA is the velocity of B with respect to A. Both observers measure the same acceleration for the particle: :

a PA " : a PB.

Relative Motion in One Dimension

Frame B moves past frame A while both observe P. y

y Frame A

Frame B P vBA xBA

xPB

x x xPA = xPB + xBA

Figure 4-18 Alex (frame A) and Barbara (frame B) watch car P, as both B and P move at different velocities along the common x axis of the two frames. At the instant shown, xBA is the coordinate of B in the A frame. Also, P is at coordinate xPB in the B frame and coordinate xPA " xPB # xBA in the A frame.

Suppose you see a duck flying north at 30 km/h. To another duck flying alongside, the first duck seems to be stationary. In other words, the velocity of a particle depends on the reference frame of whoever is observing or measuring the velocity. For our purposes, a reference frame is the physical object to which we attach our coordinate system. In everyday life, that object is the ground. For example, the speed listed on a speeding ticket is always measured relative to the ground. The speed relative to the police officer would be different if the officer were moving while making the speed measurement. Suppose that Alex (at the origin of frame A in Fig. 4-18) is parked by the side of a highway, watching car P (the “particle”) speed past. Barbara (at the origin of frame B) is driving along the highway at constant speed and is also watching car P. Suppose that they both measure the position of the car at a given moment. From Fig. 4-18 we see that xPA " xPB # xBA.

(4-40)

The equation is read: “The coordinate xPA of P as measured by A is equal to the coordinate xPB of P as measured by B plus the coordinate xBA of B as measured by A.” Note how this reading is supported by the sequence of the subscripts. Taking the time derivative of Eq. 4-40, we obtain d d d (xPA) " (xPB) # (x ). dt dt dt BA Thus, the velocity components are related by vPA " vPB # vBA.

(4-41)

This equation is read: “The velocity vPA of P as measured by A is equal to the

4-6 R E L ATIVE M OTION I N ON E DI M E NSION

79

velocity vPB of P as measured by B plus the velocity vBA of B as measured by A.” The term vBA is the velocity of frame B relative to frame A. Here we consider only frames that move at constant velocity relative to each other. In our example, this means that Barbara (frame B) drives always at constant velocity vBA relative to Alex (frame A). Car P (the moving particle), however, can change speed and direction (that is, it can accelerate). To relate an acceleration of P as measured by Barbara and by Alex, we take the time derivative of Eq. 4-41: d d d (vPA) " (vPB) # (v ). dt dt dt BA Because vBA is constant, the last term is zero and we have aPA " aPB.

(4-42)

In other words, Observers on different frames of reference that move at constant velocity relative to each other will measure the same acceleration for a moving particle.

Sample Problem 4.07 Relative motion, one dimensional, Alex and Barbara In Fig. 4-18, suppose that Barbara’s velocity relative to Alex is a constant vBA " 52 km/h and car P is moving in the negative direction of the x axis. (a) If Alex measures a constant vPA " $78 km/h for car P, what velocity vPB will Barbara measure? KEY IDEAS

Calculation: The initial velocity of P relative to Alex is vPA " $78 km/h and the final velocity is 0. Thus, the acceleration relative to Alex is 0 $ ($78 km/h) 1 m/s v $ v0 " t 10 s 3.6 km/h 2 (Answer) " 2.2 m/s .

aPA "

We can attach a frame of reference A to Alex and a frame of reference B to Barbara. Because the frames move at constant velocity relative to each other along one axis, we can use Eq. 4-41 (vPA " vPB # vBA) to relate vPB to vPA and vBA. Calculation: We find vPB " $130 km/h.

(c) What is the acceleration aPB of car P relative to Barbara during the braking? KEY IDEA

$78 km/h " vPB # 52 km/h. Thus,

to relate the acceleration to the initial and final velocities of P.

(Answer)

Comment: If car P were connected to Barbara’s car by a cord wound on a spool, the cord would be unwinding at a speed of 130 km/h as the two cars separated. (b) If car P brakes to a stop relative to Alex (and thus relative to the ground) in time t " 10 s at constant acceleration, what is its acceleration aPA relative to Alex? KEY IDEAS To calculate the acceleration of car P relative to Alex, we must use the car’s velocities relative to Alex. Because the acceleration is constant, we can use Eq. 2-11 (v " v0 # at)

To calculate the acceleration of car P relative to Barbara, we must use the car’s velocities relative to Barbara. Calculation: We know the initial velocity of P relative to Barbara from part (a) (vPB " $130 km/h). The final velocity of P relative to Barbara is $52 km/h (because this is the velocity of the stopped car relative to the moving Barbara). Thus, $52 km/h $ ($130 km/h) 1 m/s v $ v0 " t 10 s 3.6 km/h 2 " 2.2 m/s . (Answer)

aPB "

Comment: We should have foreseen this result: Because Alex and Barbara have a constant relative velocity, they must measure the same acceleration for the car.

Additional examples, video, and practice available at WileyPLUS

80

CHAPTE R 4 M OTION I N TWO AN D TH R E E DI M E NSIONS

4-7 RELATIVE MOTION IN TWO DIMENSIONS Learning Objective After reading this module, you should be able to . . .

4.19 Apply the relationship between a particle’s position, velocity, and acceleration as measured from two reference

frames that move relative to each other at constant velocity and in two dimensions.

Key Idea : v PA

● When two frames of reference A and B are moving relative

to each other at constant velocity, the velocity of a particle P as measured by an observer in frame A usually differs from that measured from frame B. The two measured velocities are related by

P y rPB vBA

rBA Frame A

:

where v BA is the velocity of B with respect to A. Both observers measure the same acceleration for the particle: :

: a PA " a PB.

Relative Motion in Two Dimensions

y

rPA

": v PB # : v BA,

x

Frame B x

Figure 4-19 Frame B has the constant two-dimensional velocity : v BA relative to frame A. The position vector of B relative to A is : r BA. The position vectors of particle P are : r PB r PA relative to A and : relative to B.

Our two observers are again watching a moving particle P from the origins of refer: ence frames A and B, while B moves at a constant velocity v BA relative to A. (The corresponding axes of these two frames remain parallel.) Figure 4-19 shows a certain instant during the motion. At that instant, the position vector of the origin of B : relative to the origin of A is : r BA.Also, the position vectors of particle P are r PA rela: tive to the origin of A and r PB relative to the origin of B. From the arrangement of heads and tails of those three position vectors, we can relate the vectors with :

:

:

r PA " r PB # r BA.

(4-43)

By taking the time derivative of this equation, we can relate the velocities : v PA and : v PB of particle P relative to our observers: : v PA

": v PB # : v BA.

(4-44)

By taking the time derivative of this relation, we can relate the accelerations : a PA and : a PB of the particle P relative to our observers. However, note that because : v BA is constant, its time derivative is zero. Thus, we get :

a PA " : a PB.

(4-45)

As for one-dimensional motion, we have the following rule: Observers on different frames of reference that move at constant velocity relative to each other will measure the same acceleration for a moving particle. Sample Problem 4.08 Relative motion, two dimensional, airplanes In Fig. 4-20a, a plane moves due east while the pilot points the plane somewhat south of east, toward a steady wind that blows to the northeast. The plane has velocity : v PW relative to the wind, with an airspeed (speed relative to the wind) of 215 km/h, directed at angle u south of east. The wind has velocity : v WG relative to the ground with speed 65.0 km/h, directed 20.0° east of north. What is the magnitude of the velocity : v PG of the plane relative to the ground, and what is 1?

KEY IDEAS The situation is like the one in Fig. 4-19. Here the moving particle P is the plane, frame A is attached to the ground (call it G), and frame B is “attached” to the wind (call it W). We need a vector diagram like Fig. 4-19 but with three velocity vectors. Calculations: First we construct a sentence that relates the three vectors shown in Fig. 4-20b:

R EVI EW & SU M MARY

velocity of plane velocity of plane velocity of wind " # relative to ground relative to wind relative to ground. (PG) (PW) (WG)

This is the plane's actual direction of travel.

N

vPG

This relation is written in vector notation as : v PG

": v PW # : v WG.

(4-46)

We need to resolve the vectors into components on the coordinate system of Fig. 4-20b and then solve Eq. 4-46 axis by axis. For the y components, we find

E

θ

This is the plane's orientation.

N

(a)

Solving for u gives us

vPG

y

(65.0 km/h)(cos 20.03) " 16.53. 215 km/h

vWG

This is the wind direction.

0 " $(215 km/h) sin u # (65.0 km/h)(cos 20.0°).

1 " sin$1

20°

vPW

vPG,y " vPW,y # vWG,y or

81

θ

(Answer)

vWG

vPW x

Similarly, for the x components we find

The actual direction is the vector sum of the other two vectors (head-to-tail arrangement).

vPG,x " vPW,x # vWG,x. Here, because : v PG is parallel to the x axis, the component vPG,x is equal to the magnitude vPG. Substituting this notation and the value u " 16.5°, we find vPG " (215 km/h)(cos 16.5°) # (65.0 km/h)(sin 20.0°) " 228 km/h. (Answer)

(b)

Figure 4-20 A plane flying in a wind.

Additional examples, video, and practice available at WileyPLUS

Review & Summary Position Vector The location of a particle relative to the ori:

gin of a coordinate system is given by a position vector r , which in unit-vector notation is r " xiˆ # yjˆ # zkˆ .

:

Displacement If a particle moves so that its position vector changes from : r 1 to : r 2, the particle’s displacement -: r is -r "

: r2

$

: r 1.

(4-2)

The displacement can also be written as -: r " (x2 $ x1)iˆ # ( y2 $ y1)jˆ # (z2 $ z1)kˆ " -xˆi # -yˆj # -zkˆ.

:

v"

(4-1)

Here x ˆi , y ˆj , and z kˆ are the vector components of position vector : r, and x, y, and z are its scalar components (as well as the coordinates of the particle). A position vector is described either by a magnitude and one or two angles for orientation, or by its vector or scalar components.

:

As -t in Eq. 4-8 is shrunk to 0, : v avg reaches a limit called either the velocity or the instantaneous velocity : v:

(4-4)

v " vx ˆi # vy ˆj # vzkˆ ,

:

cle undergoes a displacement - r in time interval -t, its average velocity : v avg for that time interval is : v avg

-: r . " -t

(4-8)

(4-11)

where vx " dx /dt, vy " dy /dt, and vz " dz /dt. The instantaneous velocity : v of a particle is always directed along the tangent to the particle’s path at the particle’s position.

Average Acceleration and Instantaneous Acceleration If a particle’s velocity changes from : v 1 to : v 2 in time interval -t, its average acceleration during -t is a avg "

: v2

: $: v1 -v " . -t -t

(4-15)

As -t in Eq. 4-15 is shrunk to 0, : a avg reaches a limiting value called a: either the acceleration or the instantaneous acceleration :

Average Velocity and Instantaneous Velocity If a parti:

(4-10)

which can be rewritten in unit-vector notation as

:

(4-3)

d: r , dt

:

a"

In unit-vector notation,

d: v . dt

a " ax iˆ # ay jˆ # azkˆ ,

:

where ax " dvx /dt, ay " dvy /dt, and az " dvz /dt.

(4-16) (4-17)

82

CHAPTE R 4 M OTION I N TWO AN D TH R E E DI M E NSIONS

Projectile Motion Projectile motion is the motion of a particle

that is launched with an initial velocity : v 0. During its flight, the particle’s horizontal acceleration is zero and its vertical acceleration is the free-fall acceleration $g. (Upward is taken to be a positive direction.) If : v 0 is expressed as a magnitude (the speed v0) and an angle u0 (measured from the horizontal), the particle’s equations of motion along the horizontal x axis and vertical y axis are x $ x0 " (v0 cos u0)t,

(4-21)

y $ y0 " (v0 sin 10)t $ 12gt 2,

(4-22)

vy " v0 sin u0 $ gt,

(4-23)

v 2y " (v0 sin 10)2 $ 2g(y $ y0).

(4-24)

The trajectory (path) of a particle in projectile motion is parabolic and is given by gx2 , (4-25) y " (tan 10)x $ 2(v0 cos 10)2 if x0 and y0 of Eqs. 4-21 to 4-24 are zero. The particle’s horizontal range R, which is the horizontal distance from the launch point to the point at which the particle returns to the launch height, is v2 R " 0 sin 210. g

(4-26)

Uniform Circular Motion If a particle travels along a circle or circular arc of radius r at constant speed v, it is said to be in uniform circular motion and has an acceleration : a of constant magnitude a"

v2 . r

(4-34)

The direction of : a is toward the center of the circle or circular arc, and : a is said to be centripetal. The time for the particle to complete a circle is 2)r . (4-35) T" v T is called the period of revolution, or simply the period, of the motion.

Relative Motion When two frames of reference A and B are moving relative to each other at constant velocity, the velocity of a particle P as measured by an observer in frame A usually differs from that measured from frame B.The two measured velocities are related by : v PA

": v PB # : v BA,

(4-44)

v BA is the velocity of B with respect to A. Both observers where : measure the same acceleration for the particle: :

a PB. a PA " :

(4-45)

Questions 1 Figure 4-21 shows the path taken by a skunk foraging for trash food, from initial point i. The skunk took the same time T to go from each labeled point to the next along its path. Rank points a, b, and c according to the magnitude of the average velocity of the skunk to reach them from initial point i, greatest first.

twice as long as at 45º. Does that result mean that the air density at high altitudes increases with altitude or decreases? a

i

b

c

Figure 4-21 2 Figure 4-22 shows the initial posiQuestion 1. tion i and the final position f of a particle. What are the (a) initial position : vector : r i and (b) final position vector rf , both in unit-vector notation? (c) What is the x component of displacement -: r? y 3m 2m 1m 4m

(a)

(b)

(c)

Figure 4-23 Question 5.

3m z

5 Figure 4-23 shows three situations in which identical projectiles are launched (at the same level) at identical initial speeds and angles. The projectiles do not land on the same terrain, however. Rank the situations according to the final speeds of the projectiles just before they land, greatest first.

i

x

4m

4 You are to launch a rocket, from just above the ground, with one of the following initial velocity vectors: (1) : v 0 " 20iˆ # 70jˆ , v 0 " $20iˆ $ 70jˆ . In (2) : v 0 " $20iˆ # 70jˆ, (3) : v 0 " 20iˆ $ 70jˆ, (4) : your coordinate system, x runs along level ground and y increases upward. (a) Rank the vectors according to the launch speed of the projectile, greatest first. (b) Rank the vectors according to the time of flight of the projectile, greatest first.

5m

f

3m

Figure 4-22 Question 2. 3 When Paris was shelled from 100 km away with the WWI long-range artillery piece “Big Bertha,” the shells were fired at an angle greater than 45º to give them a greater range, possibly even

6 The only good use of a fruitcake is in catapult practice. Curve 1 in Fig. 4-24 gives the height y of a catapulted fruitcake versus the angle u between its velocity vector and its acceleration vector during flight. (a) Which of the lettered points on that curve corresponds to the landing of the fruitcake on the ground? (b) Curve 2 is a similar plot for the same

y 2 1

A

B

θ

Figure 4-24 Question 6.

QU ESTIONS

launch speed but for a different launch angle. Does the fruitcake now land farther away or closer to the launch point? 7 An airplane flying horizontally at a constant speed of 350 km/h over level ground releases a bundle of food supplies. Ignore the effect of the air on the bundle. What are the bundle’s initial (a) vertical and (b) horizontal components of velocity? (c) What is its horizontal component of velocity just before hitting the ground? (d) If the airplane’s speed were, instead, 450 km/h, would the time of fall be longer, shorter, or the same? 8 In Fig. 4-25, a cream tangerine is thrown up past windows 1, 2, and 3, which are identical in size and regularly spaced vertically. Rank those three windows according to (a) the time the cream tangerine takes to pass them and (b) the average speed of the cream tangerine during the passage, greatest first. The cream tangerine then moves down past windows 4, 5, and 6, which are identical in size and irregularly spaced horizontally. Rank those three windows according to (c) the time the cream tangerine takes to pass them and (d) the average speed of the cream tangerine during the passage, greatest first.

3

4

83

11 Figure 4-28 shows four tracks (either half- or quarter-circles) that can be taken by a train, which moves at a constant speed. Rank the tracks according to the magnitude of a train’s acceleration on the curved portion, greatest first. 1 2 3 4

Figure 4-28 Question 11. 12 In Fig. 4-29, particle P is in uniform circular motion, centered on the origin of an xy coordinate system. (a) At what values of u is the vertical component ry of the position vector greatest in magnitude? (b) At what values of u is the vertical component vy of the particle’s velocity greatest in magnitude? (c) At what values of u is the vertical component ay of the particle’s acceleration greatest in magnitude? y

2

5 P r

1

6

θ

x

Figure 4-25 Question 8. 9 Figure 4-26 shows three paths for a football kicked from ground level. Ignoring the effects of air, rank the paths according to (a) time of flight, (b) initial vertical velocity component, (c) initial horizontal velocity component, and (d) initial speed, greatest first.

1

2

13 (a) Is it possible to be accelerating while traveling at constant speed? Is it possible to round a curve with (b) zero acceleration and (c) a constant magnitude of acceleration? 14 While riding in a moving car, you toss an egg directly upward. Does the egg tend to land behind you, in front of you, or back in your hands if the car is (a) traveling at a constant speed, (b) increasing in speed, and (c) decreasing in speed?

3

Figure 4-26 Question 9. 10 A ball is shot from ground level over level ground at a certain initial speed. Figure 4-27 gives the range R of the ball versus its launch angle u0. Rank the three lettered points on the plot according to (a) the total flight time of the ball and (b) the ball’s speed at maximum height, greatest first. R a

Figure 4-29 Question 12.

b

15 A snowball is thrown from ground level (by someone in a hole) with initial speed v0 at an angle of 45° relative to the (level) ground, on which the snowball later lands. If the launch angle is increased, do (a) the range and (b) the flight time increase, decrease, or stay the same? 16 You are driving directly behind a pickup truck, going at the same speed as the truck. A crate falls from the bed of the truck to the road. (a) Will your car hit the crate before the crate hits the road if you neither brake nor swerve? (b) During the fall, is the horizontal speed of the crate more than, less than, or the same as that of the truck? 17

c

θ0

Figure 4-27 Question 10.

At what point in the path of a projectile is the speed a minimum?

18 In shot put, the shot is put (thrown) from above the athlete’s shoulder level. Is the launch angle that produces the greatest range 45°, less than 45°, or greater than 45°?

84

CHAPTE R 4 M OTION I N TWO AN D TH R E E DI M E NSIONS

Problems Tutoring problem available (at instructor’s discretion) in WileyPLUS and WebAssign SSM • – •••

Worked-out solution available in Student Solutions Manual

WWW Worked-out solution is at

Number of dots indicates level of problem difficulty

Interactive solution is at

ILW

http://www.wiley.com/college/halliday

Additional information available in The Flying Circus of Physics and at flyingcircusofphysics.com

vector on a right-handed coordinate system. •2 A watermelon seed has the following coordinates: x " $5.0 m, y " 8.0 m, and z " 0 m. Find its position vector (a) in unit-vector notation and as (b) a magnitude and (c) an angle relative to the positive direction of the x axis. (d) Sketch the vector on a right-handed coordinate system. If the seed is moved to the xyz coordinates (3.00 m, 0 m, 0 m), what is its displacement (e) in unit-vector notation and as (f) a magnitude and (g) an angle relative to the positive x direction? •3 A positron undergoes a displacement -: r " 2.0iˆ $ 3.0jˆ # 6.0kˆ , ending with the position vector : r " 3.0jˆ $ 4.0kˆ , in meters. What was the positron’s initial position vector?

and (b) angle of the one with the least magnitude and the (c) magnitude and (d) angle of the one with the greatest magnitude?

20°

•••10 The position vector 0° r " 5.00tiˆ # (et # ft2)jˆ locates a 10 20 particle as a function of time t. Vector : r is in meters, t is in seconds, –20° and factors e and f are constants. t (s) Figure 4-31 gives the angle u of the particle’s direction of travel as a Figure 4-31 Problem 10. function of t (u is measured from the positive x direction). What are (a) e and (b) f, including units? θ

Module 4-1 Position and Displacement r " (5.0 m)iˆ $ •1 The position vector for an electron is : (3.0 m)jˆ # (2.0 m)kˆ . (a) Find the magnitude of : r . (b) Sketch the

:

••4 The minute hand of a wall clock measures 10 cm from its tip to the axis about which it rotates. The magnitude and angle of the displacement vector of the tip are to be determined for three time intervals. What are the (a) magnitude and (b) angle from a quarter after the hour to half past, the (c) magnitude and (d) angle for the next half hour, and the (e) magnitude and (f) angle for the hour after that?

Module 4-3 Average Acceleration and Instantaneous Acceleration •11 The position : r of a particle moving in an xy plane is given : by r " (2.00t3 $ 5.00t)iˆ # (6.00 $ 7.00t4)jˆ , with : r in meters and t in seconds. In unit-vector notation, calculate (a) : r , (b) : v , and (c) : a for t " 2.00 s. (d) What is the angle between the positive direction of the x axis and a line tangent to the particle’s path at t " 2.00 s?

Module 4-2 Average Velocity and Instantaneous Velocity •5 SSM A train at a constant 60.0 km/h moves east for 40.0 min, then in a direction 50.0° east of due north for 20.0 min, and then west for 50.0 min. What are the (a) magnitude and (b) angle of its average velocity during this trip? •6 An electron’s position is given by : r " 3.00t iˆ $ 4.00t2jˆ # 2.00kˆ ,

•12 At one instant a bicyclist is 40.0 m due east of a park’s flagpole, going due south with a speed of 10.0 m/s. Then 30.0 s later, the cyclist is 40.0 m due north of the flagpole, going due east with a speed of 10.0 m/s. For the cyclist in this 30.0 s interval, what are the (a) magnitude and (b) direction of the displacement, the (c) magnitude and (d) direction of the average velocity, and the (e) magnitude and (f) direction of the average acceleration?

with t in seconds and : r in meters. (a) In unit-vector notation, what is the electron’s velocity : v (t)? At t " 2.00 s, what is : v (b) in unitvector notation and as (c) a magnitude and (d) an angle relative to the positive direction of the x axis? •7 An ion’s position vector is initially : r " 5.0iˆ $ 6.0jˆ # 2.0kˆ , : ˆ ˆ ˆ and 10 s later it is r " $2.0i # 8.0j $ 2.0k , all in meters. In unitvector notation, what is its : v avg during the 10 s? ••8 A plane flies 483 km east from city A to city B in 45.0 min and then 966 km south from city B to city C in 1.50 h. For the total trip, what are the (a) magnitude and (b) direction of the plane’s displacement, the (c) magnitude y (m) and (d) direction of its aver50 age velocity, and (e) its averD age speed? ••9 Figure 4-30 gives the 25 path of a squirrel moving about on level ground, from point A (at time t " 0), to 0 points B (at t " 5.00 min), C (at t " 10.0 min), and finally D (at t " 15.0 min). Consider the –25 average velocities of the squirrel from point A to each of the other three points. Of them, –50 what are the (a) magnitude

A

x (m) 50

25 C

B

Figure 4-30 Problem 9.

•13 SSM A particle moves so that its position (in meters) as a function of time (in seconds) is : r " iˆ # 4t 2jˆ # tkˆ . Write expressions for (a) its velocity and (b) its acceleration as functions of time. v " 4.0iˆ $ 2.0jˆ # 3.0kˆ and then •14 A proton initially has : 4.0 s later has : v " $2.0iˆ $ 2.0jˆ # 5.0kˆ (in meters per second). For : that 4.0 s, what are (a) the proton’s average acceleration a avg in unit: vector notation, (b) the magnitude of a avg, and (c) the angle between : a avg and the positive direction of the x axis? ••15 SSM ILW A particle leaves the origin with an initial velocity : v " (3.00iˆ ) m/s and a constant acceleration : a " ($1.00iˆ $ 2 ˆ 0.500j ) m/s . When it reaches its maximum x coordinate, what are its (a) velocity and (b) position vector? ••16 The velocity : v of a particle moving in the xy plane is : given by v " (6.0t $ 4.0t2)iˆ # 8.0jˆ , with : v in meters per second and t (> 0) in seconds. (a) What is the acceleration when t " 3.0 s? (b) When (if ever) is the acceleration zero? (c) When (if ever) is the velocity zero? (d) When (if ever) does the speed equal 10 m/s? ••17 A cart is propelled over an xy plane with acceleration components ax " 4.0 m/s2 and ay " $2.0 m/s2. Its initial velocity has components v0x " 8.0 m/s and v0y " 12 m/s. In unit-vector notation, what is the velocity of the cart when it reaches its greatest y coordinate? ••18 A moderate wind accelerates a pebble over a horizontal xy plane with a constant acceleration : a " (5.00 m/s2)iˆ # (7.00 m/s2)jˆ .

PROB LE M S

ˆi What are the (a) magniAt time t " 0, the velocity is (4.00 m/s)i. tude and (b) angle of its velocity when it has been displaced by 12.0 m parallel to the x axis? •••19 The acceleration of a particle moving only on a horizontal a is in meters per secondxy plane is given by : a " 3tiˆ # 4tjˆ , where : squared and t is in seconds. At t " 0, the position vector : r " (20.0 m)iˆ # (40.0 m)jˆ locates the particle, which then has the velocity vector : v " (5.00 m/s)iˆ # (2.00 m/s)jˆ . At t " 4.00 s, what are (a) its position vector in unit-vector notation and (b) the angle between its direction of travel and the positive direction of the x axis? •••20 In Fig. 4-32, particle A moves along the line y " 30 m with a constant velocity : v of magnitude 3.0 m/s and parallel to the x axis. At the instant particle A passes the y axis, particle B leaves the origin with a zero initial speed and a constant acceleration : a of magnitude 0.40 m/s2. What angle u between : a and the positive direction of the y axis would result in a collision?

θ

d

Figure 4-33 Problem 27.

••28 In Fig. 4-34, a stone is projected at a cliff of height h with an initial speed of 42.0 m/s directed at angle u0 " 60.0° above the horizontal. The stone strikes at A, 5.50 s after launching. Find (a) the height h of the cliff, (b) the speed of the stone just before impact at A, and (c) the maximum height H reached above the ground.

y v

A

••27 ILW A certain airplane has a speed of 290.0 km/h and is diving at an angle of 1 " 30.0° below the horizontal when the pilot releases a radar decoy (Fig. 4-33). The horizontal distance between the release point and the point where the decoy strikes the ground is d " 700 m. (a) How long is the decoy in the air? (b) How high was the release point?

85

θ a

x

B

Module 4-4 Projectile Motion •21 A dart is thrown horizontally with an initial speed of 10 m/s toward point P, the bull’s-eye on a dart board. It hits at point Q on the rim, vertically below P, 0.19 s later. (a) What is the distance PQ? (b) How far away from the dart board is the dart released?

A

H

Figure 4-32 Problem 20. θ0

h

Figure 4-34 Problem 28. ••29 A projectile’s launch speed is five times its speed at maximum height. Find launch angle 10.

•22 A small ball rolls horizontally off the edge of a tabletop that is 1.20 m high. It strikes the floor at a point 1.52 m horizontally from the table edge. (a) How long is the ball in the air? (b) What is its speed at the instant it leaves the table?

••30 A soccer ball is kicked from the ground with an initial speed of 19.5 m/s at an upward angle of 45°. A player 55 m away in the direction of the kick starts running to meet the ball at that instant. What must be his average speed if he is to meet the ball just before it hits the ground?

•23 A projectile is fired horizontally from a gun that is 45.0 m above flat ground, emerging from the gun with a speed of 250 m/s. (a) How long does the projectile remain in the air? (b) At what horizontal distance from the firing point does it strike the ground? (c) What is the magnitude of the vertical component of its velocity as it strikes the ground?

••31 In a jump spike, a volleyball player slams the ball from overhead and toward the opposite floor. Controlling the angle of the spike is difficult. Suppose a ball is spiked from a height of 2.30 m with an initial speed of 20.0 m/s at a downward angle of 18.00°. How much farther on the opposite floor would it have landed if the downward angle were, instead, 8.00°?

•24 In the 1991 World Track and Field Championships in Tokyo, Mike Powell jumped 8.95 m, breaking by a full 5 cm the 23-year long-jump record set by Bob Beamon. Assume that Powell’s speed on takeoff was 9.5 m/s (about equal to that of a sprinter) and that g " 9.80 m/s2 in Tokyo. How much less was Powell’s range than the maximum possible range for a particle launched at the same speed?

••32 You throw a ball toward a wall at speed 25.0 m/s and at angle 10 " 40.0° above the horizontal (Fig. 4-35). The wall is distance d " θ0 22.0 m from the release point of the d ball. (a) How far above the release point does the ball hit the wall? Figure 4-35 Problem 32. What are the (b) horizontal and (c) vertical components of its velocity as it hits the wall? (d) When it hits, has it passed the highest point on its trajectory?

•25 The current world-record motorcycle jump is 77.0 m, set by Jason Renie. Assume that he left the take-off ramp at 12.0º to the horizontal and that the take-off and landing heights are the same. Neglecting air drag, determine his take-off speed. •26 A stone is catapulted at time t " 0, with an initial velocity of magnitude 20.0 m/s and at an angle of 40.0° above the horizontal. What are the magnitudes of the (a) horizontal and (b) vertical components of its displacement from the catapult site at t " 1.10 s? Repeat for the (c) horizontal and (d) vertical components at t " 1.80 s, and for the (e) horizontal and (f) vertical components at t " 5.00 s.

••33 SSM A plane, diving with constant speed at an angle of 53.0° with the vertical, releases a projectile at an altitude of 730 m. The projectile hits the ground 5.00 s after release. (a) What is the speed of the plane? (b) How far does the projectile travel horizontally during its flight? What are the (c) horizontal and (d) vertical components of its velocity just before striking the ground? ••34 A trebuchet was a hurling machine built to attack the walls of a castle under siege. A large stone could be hurled against a wall to break apart the wall. The machine was not placed near the

86

CHAPTE R 4 M OTION I N TWO AN D TH R E E DI M E NSIONS

wall because then arrows could reach it from the castle wall. Instead, it was positioned so that the stone hit the wall during the second half of its flight. Suppose a stone is launched with a speed of v0 " 28.0 m/s and at an angle of u0 " 40.0°. What is the speed of the stone if it hits the wall (a) just as it reaches the top of its parabolic path and (b) when it has descended to half that height? (c) As a percentage, how much faster is it moving in part (b) than in part (a)? ••35 SSM A rifle that shoots bullets at 460 m/s is to be aimed at a target 45.7 m away. If the center of the target is level with the rifle, how high above the target must the rifle barrel be pointed so that the bullet hits dead center? ••36 During a tennis match, a player serves the ball at 23.6 m/s, with the center of the ball leaving the racquet horizontally 2.37 m above the court surface. The net is 12 m away and 0.90 m high. When the ball reaches the net, (a) does the ball clear it and (b) what is the distance between the center of the ball and the top of the net? Suppose that, instead, the ball is served as before but now it leaves the racquet at 5.00° below the horizontal. When the ball reaches the net, (c) does the ball clear it and (d) what now is the distance between the center of the ball and the top of the net? ••37 SSM WWW A lowly high diver pushes off horizontally with a speed of 2.00 m/s from the platform edge 10.0 m above the surface of the water. (a) At what horizontal distance from the edge is the diver 0.800 s after pushing off? (b) At what vertical distance above the surface of the water is the diver just then? (c) At what horizontal distance from the edge does the diver strike the water?

v (m/s)

vb ••38 A golf ball is struck at ground level. The speed of the golf ball as a function of the time is shown in Fig. 4-36, where t " 0 at the instant the ball is struck. The scaling on the vertical axis is set by va va " 19 m/s and vb " 31 m/s. 0 1 2 3 4 5 t (s) (a) How far does the golf ball travel horizontally beFigure 4-36 Problem 38. fore returning to ground level? (b) What is the maximum height above ground level attained by the ball?

••39 In Fig. 4-37, a ball is thrown leftward from the left edge of the roof, at height h above the ground. The ball hits the ground 1.50 s later, at distance d " 25.0 m from the building and at angle u " 60.0° with the horizontal. (a) Find h. (Hint: One way is to reverse the motion, as if on video.) What h are the (b) magnitude and (c) θ angle relative to the horizontal of the velocity at which the ball d is thrown? (d) Is the angle Figure 4-37 Problem 39. above or below the horizontal? ••40 Suppose that a shot putter can put a shot at the worldclass speed v0 " 15.00 m/s and at a height of 2.160 m. What horizontal distance would the shot travel if the launch angle 10 is (a) 45.00° and (b) 42.00°? The answers indicate that the angle of 45°, which maximizes the range of projectile motion, does not maximize the horizontal distance when the launch and landing are at different heights.

••41 Upon spotting an inInsect on twig sect on a twig overhanging water, an archer fish squirts water drops at the d insect to knock it into the water (Fig. 4 -38). Although the fish sees the φ insect along a straight-line path at angle f and distance d, a drop must be Archer fish launched at a different angle u0 if its parabolic path is to intersect the Figure 4-38 Problem 41. insect. If f " 36.0° and d " 0.900 m, what launch angle u0 is required for the drop to be at the top of the parabolic path when it reaches the insect? ••42 In 1939 or 1940, Emanuel Zacchini took his humancannonball act to an extreme: After being shot from a cannon, he soared over three Ferris wheels and into a net (Fig. 4-39). Assume that he is launched with a speed of 26.5 m/s and at an angle of 53.0°. (a) Treating him as a particle, calculate his clearance over the first wheel. (b) If he reached maximum height over the middle wheel, by how much did he clear it? (c) How far from the cannon should the net’s center have been positioned (neglect air drag)?

v0

3.0 m

18 m

3.0 m

θ0

Net

23 m R

Figure 4-39 Problem 42. ••43 ILW A ball is shot from the ground into the air. At a height of 9.1 m, its velocity is : v " (7.6iˆ # 6.1jˆ ) m/s, with ˆi horizontal and ˆj upward. (a) To what maximum height does the ball rise? (b) What total horizontal distance does the ball travel? What are the (c) magnitude and (d) angle (below the horizontal) of the ball’s velocity just before it hits the ground? ••44 A baseball leaves a pitcher’s hand horizontally at a speed of 161 km/h.The distance to the batter is 18.3 m. (a) How long does the ball take to travel the first half of that distance? (b) The second half? (c) How far does the ball fall freely during the first half? (d) During the second half? (e) Why aren’t the quantities in (c) and (d) equal? ••45 In Fig. 4-40, a ball is launched with a velocity of magnitude 10.0 m/s, at an angle of 50.0° to the horizontal. The launch point is at the base of a ramp of horizontal length d1 " 6.00 m and height d2 " 3.60 m. A plateau v0 d2 is located at the top of the Ball ramp. (a) Does the ball land on d1 the ramp or the plateau? When it lands, what are the (b) magFigure 4-40 Problem 45. nitude and (c) angle of its displacement from the launch point? ••46 In basketball, hang is an illusion in which a player seems to weaken the gravitational acceleration while in midair. The illusion depends much on a skilled player’s ability to rapidly shift

PROB LE M S

••47 SSM WWW A batter hits a pitched ball when the center of the ball is 1.22 m above the ground. The ball leaves the bat at an angle of 45° with the ground. With that launch, the ball should have a horizontal range (returning to the launch level) of 107 m. (a) Does the ball clear a 7.32-m-high fence that is 97.5 m horizontally from the launch point? (b) At the fence, what is the distance between the fence top and the ball center?

θ h

vys

y

x

0

xs

(a) –vys

x (m) (b)

Figure 4-43 Problem 52. •••53 In Fig. 4-44, a baseball is hit at a height h " 1.00 m and then caught at the same height. It travels alongside a wall, moving up past the top of the wall 1.00 s after it is hit and then down past the top of the wall 4.00 s later, at distance D " 50.0 m farther along the wall. (a) What horizontal distance is traveled by the ball from hit to catch? What are the (b) magnitude and (c) angle (relative to the horizontal) of the ball’s velocity just after being hit? (d) How high is the wall?

d

Figure 4-41 Problem 48.

•••49 SSM A football kicker can give the ball an initial speed of 25 m/s. What are the (a) least and (b) greatest elevation angles at which he can kick the ball to score a field goal from a point 50 m in front of goalposts whose horizontal bar is 3.44 m above the ground? •••50 Two seconds after being projected from ground level, a projectile is displaced 40 m horizontally and 53 m vertically above its launch point. What are the (a) horizontal and (b) vertical components of the initial velocity of the projectile? (c) At the instant the projectile achieves its maximum height above ground level, how far is it displaced horizontally from the launch point? •••51 A skilled skier knows to jump upward before reaching a downward slope. Consider a jump in which the launch speed is v0 " 10 m/s, the launch angle is u0 " 11.3°, the initial course is approximately flat, and the steeper track has a slope of 9.0°. Figure 4-42a shows a prejump that allows the skier to land on the top portion of the steeper track. Figure 4-42b shows a jump at the edge of the steeper track. In Fig. 4-42a, the skier lands at approximately the launch level. (a) In the landing, what is the angle f between the skier’s path and the slope? In Fig. 4-42b, (b) how far below the launch level does the skier land and (c) what is f? (The greater fall and greater f can result in loss of control in the landing.)

(a)

(b) Figure 4-42 Problem 51.

•••52 A ball is to be shot from level ground toward a wall at distance x (Fig. 4-43a). Figure 4-43b shows the y component vy of the ball’s velocity just as it would reach the wall, as a function of that

D

h

h

Figure 4-44 Problem 53. •••54 A ball is to be shot from level ground with a certain speed. Figure 4-45 shows the range R it will have versus the launch angle u0. The value of u0 determines the flight time; let tmax represent the maximum flight time. What is the least speed the ball will have during its flight if u0 is chosen such that the flight time is 0.500tmax?

200 R (m)

••48 In Fig. 4-41, a ball is thrown up onto a roof, landing 4.00 s later at height h " 20.0 m above the release level. The ball’s path just before landing is angled at u " 60.0° with the roof. (a) Find the horizontal distance d it travels. (See the hint to Problem 39.) What are the (b) magnitude and (c) angle (relative to the horizontal) of the ball’s initial velocity?

distance x. The scaling is set by vys " 5.0 m/s and xs " 20 m. What is the launch angle?

vy (m/s)

the ball between hands during the flight, but it might also be supported by the longer horizontal distance the player travels in the upper part of the jump than in the lower part. If a player jumps with an initial speed of v0 " 7.00 m/s at an angle of u0 " 35.0°, what percent of the jump’s range does the player spend in the upper half of the jump (between maximum height and half maximum height)?

87

100

0

θ0

Figure 4-45 Problem 54.

•••55 SSM A ball rolls horizontally off the top of a stairway with a speed of 1.52 m/s. The steps are 20.3 cm high and 20.3 cm wide. Which step does the ball hit first? Module 4-5 Uniform Circular Motion •56 An Earth satellite moves in a circular orbit 640 km (uniform circular motion) above Earth’s surface with a period of 98.0 min. What are (a) the speed and (b) the magnitude of the centripetal acceleration of the satellite? •57 A carnival merry-go-round rotates about a vertical axis at a constant rate. A man standing on the edge has a constant speed of 3.66 m/s and a centripetal acceleration : a of magnitude 1.83 m/s2. Position vector : r locates him relative to the rotation axis. (a) What is the magnitude of : a is dir ? What is the direction of : r when : rected (b) due east and (c) due south? •58 A rotating fan completes 1200 revolutions every minute. Consider the tip of a blade, at a radius of 0.15 m. (a) Through what distance does the tip move in one revolution? What are (b) the

88

CHAPTE R 4 M OTION I N TWO AN D TH R E E DI M E NSIONS

tip’s speed and (c) the magnitude of its acceleration? (d) What is the period of the motion? •59 ILW A woman rides a carnival Ferris wheel at radius 15 m, completing five turns about its horizontal axis every minute. What are (a) the period of the motion, the (b) magnitude and (c) direction of her centripetal acceleration at the highest point, and the (d) magnitude and (e) direction of her centripetal acceleration at the lowest point? •60 A centripetal-acceleration addict rides in uniform circular motion with radius r " 3.00 m. At one instant his acceleration is : a " (6.00 m/s2)iˆ # ($4.00 m/s2)jˆ. At that instant, what are the values of (a) : v ": a and (b) : r ': a? •61 When a large star becomes a supernova, its core may be compressed so tightly that it becomes a neutron star, with a radius of about 20 km (about the size of the San Francisco area). If a neutron star rotates once every second, (a) what is the speed of a particle on the star’s equator and (b) what is the magnitude of the particle’s centripetal acceleration? (c) If the neutron star rotates faster, do the answers to (a) and (b) increase, decrease, or remain the same? •62 What is the magnitude of the acceleration of a sprinter running at 10 m/s when rounding a turn of radius 25 m? ••63 At t1 " 2.00 s, the acceleration of a particle in counterclockwise circular motion is (6.00 m/s2)ˆi # (4.00 m/s2)ˆj. It moves at constant speed. At time t2 " 5.00 s, the particle’s acceleration is (4.00 m/s2)ˆi # ($6.00 m/s2)ˆj. What is the radius of the path taken by the particle if t2 $ t1 is less than one period? ••64 A particle moves horizontally in uniform circular motion, over a horizontal xy plane. At one instant, it moves through the point at coordinates (4.00 m, 4.00 m) with a velocity of $5.00ˆi m/s and an acceleration of #12.5ˆj m/s2. What are the (a) x and (b) y coordinates of the center of the circular path? ••65 A purse at radius 2.00 m and a wallet at radius 3.00 m travel in uniform circular motion on the floor of a merry-go-round as the ride turns. They are on the same radial line. At one instant, the acceleration of the purse is (2.00 m/s2)ˆi # (4.00 m/s2)ˆj. At that instant and in unit-vector notation, what is the acceleration of the wallet? ••66 A particle moves along a circular path over a horizontal xy coordinate system, at constant speed.At time t1 " 4.00 s, it is at point (5.00 m, 6.00 m) with velocity (3.00 m/s)ˆj and acceleration in the positive x direction. At time t2 " 10.0 s, it has velocity ($3.00 m/s)ˆi and acceleration in the positive y direction. What are the (a) x and (b) y coordinates of the center of the circular path if t2 $ t1 is less than one period? •••67 SSM WWW A boy whirls a stone in a horizontal circle of radius 1.5 m and at height 2.0 m above level ground. The string breaks, and the stone flies off horizontally and strikes the ground after traveling a horizontal distance of 10 m. What is the magnitude of the centripetal acceleration of the stone during the circular motion? •••68 A cat rides a merry-go-round turning with uniform circular motion. At time t1 " 2.00 s, the cat’s velocity is : v1 " (3.00 m/s)iˆ # (4.00 m/s)jˆ , measured on a horizontal xy coordinate system. At t2 " 5.00 s, the cat’s velocity is : v 2 " ($3.00 m/s)iˆ # ($4.00 m/s)jˆ . What are (a) the magnitude of the cat’s centripetal acceleration and (b) the cat’s average acceleration during the time interval t2 $ t1, which is less than one period?

Module 4-6 Relative Motion in One Dimension •69 A cameraman on a pickup truck is traveling westward at 20 km/h while he records a cheetah that is moving westward 30 km/h faster than the truck. Suddenly, the cheetah stops, turns, and then runs at 45 km/h eastward, as measured by a suddenly nervous crew member who stands alongside the cheetah’s path. The change in the animal’s velocity takes 2.0 s. What are the (a) magnitude and (b) direction of the animal’s acceleration according to the cameraman and the (c) magnitude and (d) direction according to the nervous crew member? •70 A boat is traveling upstream in the positive direction of an x axis at 14 km/h with respect to the water of a river. The water is flowing at 9.0 km/h with respect to the ground. What are the (a) magnitude and (b) direction of the boat’s velocity with respect to the ground? A child on the boat walks from front to rear at 6.0 km/h with respect to the boat. What are the (c) magnitude and (d) direction of the child’s velocity with respect to the ground? ••71 A suspicious-looking man runs as fast as he can along a moving sidewalk from one end to the other, taking 2.50 s. Then security agents appear, and the man runs as fast as he can back along the sidewalk to his starting point, taking 10.0 s. What is the ratio of the man’s running speed to the sidewalk’s speed? Module 4-7 Relative Motion in Two Dimensions •72 A rugby player runs with the ball directly toward his opponent’s goal, along the positive direction of an x axis. He can legally pass the ball to a teammate as long as the ball’s velocity relative to the field does not have a positive x component. Suppose the player runs at speed 4.0 m/s relative to the field while he passes the ball with velocity : v BP relative to himself. If : v BP has magnitude 6.0 m/s, what is the smallest angle it can have for the pass to be legal? ••73 Two highways intersect as shown in Fig. 4-46. At the instant shown, a police car P is distance dP " 800 m from the intersection and moving at speed vP " 80 km/h. Motorist M is distance dM " 600 m from the intersection and moving at speed vM " 60 km/h. y

M

dM

vM P vP

x

dP

Figure 4-46 Problem 73. (a) In unit-vector notation, what is the velocity of the motorist with respect to the police car? (b) For the instant shown in Fig. 4-46, what is the angle between the velocity found in (a) and the line of sight between the two cars? (c) If the cars maintain their velocities, do the answers to (a) and (b) change as the cars move nearer the intersection?

PROB LE M S

••75 SSM A train travels due south at 30 m/s (relative to the ground) in a rain that is blown toward the south by the wind. The path of each raindrop makes an angle of 70° with the vertical, as measured by an observer stationary on the ground. An observer on the train, however, sees the drops fall perfectly vertically. Determine the speed of the raindrops relative to the ground. ••76 A light plane attains an airspeed of 500 km/h. The pilot sets out for a destination 800 km due north but discovers that the plane must be headed 20.0° east of due north to fly there directly. The plane arrives in 2.00 h. What were the (a) magnitude and (b) direction of the wind velocity? ••77 SSM Snow is falling vertically at a constant speed of 8.0 m/s. At what angle from the vertical do the snowflakes appear to be falling as viewed by the driver of a car traveling on a straight, level road with a speed of 50 km/h? ••78 In the overhead view of N Fig. 4-47, Jeeps P and B race P along straight lines, across flat E terrain, and past stationary borθ1 A der guard A. Relative to the θ2 guard, B travels at a constant speed of 20.0 m/s, at the angle u2 " 30.0°. Relative to the guard, B P has accelerated from rest at a Figure 4-47 Problem 78. constant rate of 0.400 m/s2 at the angle u1 " 60.0°.At a certain time during the acceleration, P has a speed of 40.0 m/s. At that time, what are the (a) magnitude and (b) direction of the velocity of P relative to B and the (c) magnitude and (d) direction of the acceleration of P relative to B? ••79 SSM ILW Two ships, A and B, leave port at the same time. Ship A travels northwest at 24 knots, and ship B travels at 28 knots in a direction 40° west of south. (1 knot " 1 nautical mile per hour; see Appendix D.) What are the (a) magnitude and (b) direction of the velocity of ship A relative to B? (c) After what time will the ships be 160 nautical miles apart? (d) What will be the bearing of B (the direction of B’s position) relative to A at that time? ••80 A 200-m-wide river flows due east at a uniform speed of 2.0 m/s. A boat with a speed of 8.0 m/s relative to the water leaves the south bank pointed in a direction 30° west of north. What are the (a) magnitude and (b) direction of the boat’s velocity relative to the ground? (c) How long does the boat take to cross the river? •••81 Ship A is located 4.0 km north and 2.5 km east of ship B. Ship A has a velocity of 22 km/h toward the south, and ship B has a velocity of 40 km/h in a direction 37° north of east. (a) What is the velocity of A relative to B in unit-vector notation with ˆi toward the east? (b) Write an expression (in terms of ˆi and ˆj) for the position of A relative to B as a function of t, where t " 0 when the ships are in the positions described above. (c) At what time is the separation between the ships least? (d) What is that least separation? •••82 A 200-m-wide river has a uniform flow speed of 1.1 m/s through a jungle and toward the east. An explorer wishes to

leave a small clearing on the south bank and cross the river in a powerboat that moves at a constant speed of 4.0 m/s with respect to the water. There is a clearing on the north bank 82 m upstream from a point directly opposite the clearing on the south bank. (a) In what direction must the boat be pointed in order to travel in a straight line and land in the clearing on the north bank? (b) How long will the boat take to cross the river and land in the clearing? Additional Problems 83 A woman who can row a boat at 6.4 km/h in still water faces a long, straight river with a width of 6.4 km and a current of 3.2 km/h. Let iˆ point directly across the river and ˆj point directly downstream. If she rows in a straight line to a point directly opposite her starting position, (a) at what angle to iˆ must she point the boat and (b) how long will she take? (c) How long will she take if, instead, she rows 3.2 km down the river and then back to her starting point? (d) How long if she rows 3.2 km up the river and then back to her starting point? (e) At what angle to iˆ should she point the boat if she wants to cross the river in the shortest possible time? (f) How long is that shortest time? 84 In Fig. 4-48a, a sled moves in the negative x direction at constant speed vs while a ball of ice is shot from the sled with a velocity : v 0 " v0xiˆ # v0yjˆ relative to the sled. When the ball lands, its horizontal displacement -xbg relative to the ground (from its launch position to its landing position) is measured. Figure 4-48b gives -xbg as a function of vs. Assume the ball lands at approximately its launch height. What are the values of (a) v0x and (b) v0y? The ball’s displacement -xbs relative to the sled can also be measured. Assume that the sled’s velocity is not changed when the ball is shot. What is -xbs when vs is (c) 5.0 m/s and (d) 15 m/s? 40 y Ball

vs

Sled x

(a)

∆xbg (m)

••74 After flying for 15 min in a wind blowing 42 km/h at an angle of 20° south of east, an airplane pilot is over a town that is 55 km due north of the starting point. What is the speed of the airplane relative to the air?

89

0

–40

10

20

vs (m/s) (b)

Figure 4-48 Problem 84. 85 You are kidnapped by political-science majors (who are upset because you told them political science is not a real science). Although blindfolded, you can tell the speed of their car (by the whine of the engine), the time of travel (by mentally counting off seconds), and the direction of travel (by turns along the rectangular street system). From these clues, you know that you are taken along the following course: 50 km/h for 2.0 min, turn 90° to the right, 20 km/h for 4.0 min, turn 90° to the right, 20 km/h for 60 s, turn 90° to the left, 50 km/h for 60 s, turn 90° to the right, 20 km/h for 2.0 min, turn 90° to the left, 50 km/h for 30 s. At that point, (a) how far are you from your starting point, and (b) in what direction relative to your initial direction of travel are you?

90

CHAPTE R 4 M OTION I N TWO AN D TH R E E DI M E NSIONS

86 A radar station detects an airplane approaching directly from the east. At first observation, the airplane is at distance d1 " 360 m from the station and at angle u1 " 40° above the horizon (Fig. 4-49). The airplane is tracked through an angular change -u " 123° in the vertical east – west plane; its distance is then d2 " 790 m. Find the (a) magnitude and (b) direction of the airplane’s displacement during this period.

moment, the effects of air on the bomb’s travel. (b) What would be the time of flight? (c) Would the effect of the air increase or decrease your answer in (a)?

A

Airplane d2

∆θ

h

d1

d

θ1

W

Figure 4-51 Problem 91.

Figure 4-49 Problem 86. 87 SSM A baseball is hit at ground level. The ball reaches its maximum height above ground level 3.0 s after being hit. Then 2.5 s after reaching its maximum height, the ball barely clears a fence that is 97.5 m from where it was hit. Assume the ground is level. (a) What maximum height above ground level is reached by the ball? (b) How high is the fence? (c) How far beyond the fence does the ball strike the ground? 88 Long flights at midlatitudes in the Northern Hemisphere encounter the jet stream, an eastward airflow that can affect a plane’s speed relative to Earth’s surface. If a pilot maintains a certain speed relative to the air (the plane’s airspeed), the speed relative to the surface (the plane’s ground speed) is more when the flight is in the direction of the jet stream and less when the flight is opposite the jet stream. Suppose a round-trip flight is scheduled between two cities separated by 4000 km, with the outgoing flight in the direction of the jet stream and the return flight opposite it. The airline computer advises an airspeed of 1000 km/h, for which the difference in flight times for the outgoing and return flights is 70.0 min.What jet-stream speed is the computer using? 89 SSM A particle starts from the origin at t " 0 with a velocity of 8.0ˆj m/s and moves in the xy plane with constant acceleration (4.0ˆi # 2.0ˆj ) m/s2. When the particle’s x coordinate is 29 m, what are its (a) y coordinate and (b) speed?

θ0

92 An astronaut is rotated in a horizontal centrifuge at a radius of 5.0 m. (a) What is the astronaut’s speed if the centripetal acceleration has a magnitude of 7.0g? (b) How many revolutions per minute are required to produce this acceleration? (c) What is the period of the motion? 93 SSM Oasis A is 90 km due west of oasis B. A desert camel leaves A and takes 50 h to walk 75 km at 37° north of due east. Next it takes 35 h to walk 65 km due south. Then it rests for 5.0 h. What are the (a) magnitude and (b) direction of the camel’s displacement relative to A at the resting point? From the time the camel leaves A until the end of the rest period, what are the (c) magnitude and (d) direction of its average velocity and (e) its average speed? The camel’s last drink was at A; it must be at B no more than 120 h later for its next drink. If it is to reach B just in time, what must be the (f) magnitude and (g) direction of its average velocity after the rest period? 94 Curtain of death. A large metallic asteroid strikes Earth and quickly digs a crater into the rocky material below ground level by launching rocks upward and outward. The following table gives five pairs of launch speeds and angles (from the horizontal) for such rocks, based on a model of crater formation. (Other rocks, with intermediate speeds and angles, are also launched.) Suppose that you are at x " 20 km when the asteroid strikes the ground at time t " 0 and position x " 0 (Fig. 4-52). (a) At t " 20 s, what are the x and y coordinates of the rocks headed in your direction from launches A through E? (b) Plot these coordinates and then sketch a curve through the points to include rocks with intermediate launch speeds and angles.The curve should indicate what you would see as you look up into the approaching rocks. Launch

d1 h1

B

E

Radar dish

90 At what initial speed must the basketball player in Fig. 4-50 throw the ball, at angle u0 " 55° above the horizontal, to make the foul shot? The horizontal distances are d1 " 1.0 ft and d2 " 14 ft, and the heights are h1 " 7.0 ft and h2 " 10 ft.

θ0

h2

91 During volcanic erupd2 tions, chunks of solid rock Figure 4-50 Problem 90. can be blasted out of the volcano; these projectiles are called volcanic bombs. Figure 4-51 shows a cross section of Mt. Fuji, in Japan. (a) At what initial speed would a bomb have to be ejected, at angle u0 " 35° to the horizontal, from the vent at A in order to fall at the foot of the volcano at B, at vertical distance h " 3.30 km and horizontal distance d " 9.40 km? Ignore, for the

A B C D E

Speed (m/s)

Angle (degrees)

520 630 750 870 1000

14.0 16.0 18.0 20.0 22.0

y

You x (km) 0

10

Figure 4-52 Problem 94.

20

PROB LE M S

91

B 95 Figure 4-53 shows the straight path of a particle across an xy coordinate system as the particle is accelerated from rest during time interval -t1. The ac- y celeration is constant. The xy coordinates for point A A are (4.00 m, 6.00 m); those for point B are (12.0 x m, 18.0 m). (a) What is the ratio ay /ax of the accelerFigure 4-53 ation components? (b) What are the coordinates of the particle if the motion is continued for another Problem 95. interval equal to -t1?

104 A ball is thrown horizontally from a height of 20 m and hits the ground with a speed that is three times its initial speed. What is the initial speed?

96 For women’s volleyball the top of the net is 2.24 m above the floor and the court measures 9.0 m by 9.0 m on each side of the net. Using a jump serve, a player strikes the ball at a point that is 3.0 m above the floor and a horizontal distance of 8.0 m from the net. If the initial velocity of the ball is horizontal, (a) what minimum magnitude must it have if the ball is to clear the net and (b) what maximum magnitude can it have if the ball is to strike the floor inside the back line on the other side of the net?

r" 106 The position vector for a proton is initially : 5.0iˆ $ 6.0jˆ # 2.0kˆ and then later is : r " $2.0iˆ # 6.0jˆ # 2.0kˆ , all in meters. (a) What is the proton’s displacement vector, and (b) to what plane is that vector parallel?

97 SSM A rifle is aimed horizontally at a target 30 m away. The bullet hits the target 1.9 cm below the aiming point. What are (a) the bullet’s time of flight and (b) its speed as it emerges from the rifle? 98 A particle is in uniform circular motion about the origin of an xy coordinate system, moving clockwise with a period of 7.00 s. At one instant, its position vector (measured from the origin) is : r " (2.00 m)iˆ $ (3.00 m)jˆ . At that instant, what is its velocity in unit-vector notation? 99 In Fig. 4-54, a lump of wet putty moves in uniform circular moWheel tion as it rides at a radius of 20.0 cm on the rim of a wheel rotating counh Putty terclockwise with a period of 5.00 ms. The lump then happens to fly off d the rim at the 5 o’clock position (as Figure 4-54 Problem 99. if on a clock face). It leaves the rim at a height of h " 1.20 m from the floor and at a distance d " 2.50 m from a wall. At what height on the wall does the lump hit? 100 An iceboat sails across the surface of a frozen lake with constant acceleration produced by the wind. At a certain instant the boat’s velocity is (6.30ˆi $ 8.42ˆj ) m/s. Three seconds later, because of a wind shift, the boat is instantaneously at rest. What is its average acceleration for this 3.00 s interval? 101 In Fig. 4-55, a ball is shot divc rectly upward from the ground with an initial speed of v0 " 7.00 m/s. v0 Simultaneously, a construction elevaBall tor cab begins to move upward from the ground with a constant speed of Figure 4-55 Problem 101. vc " 3.00 m/s. What maximum height does the ball reach relative to (a) the ground and (b) the cab floor? At what rate does the speed of the ball change relative to (c) the ground and (d) the cab floor? 102 A magnetic field forces an electron to move in a circle with radial acceleration 3.0 ' 10 14 m/s2. (a) What is the speed of the electron if the radius of its circular path is 15 cm? (b) What is the period of the motion? 103 In 3.50 h, a balloon drifts 21.5 km north, 9.70 km east, and 2.88 km upward from its release point on the ground. Find (a) the magnitude of its average velocity and (b) the angle its average velocity makes with the horizontal.

105 A projectile is launched with an initial speed of 30 m/s at an angle of 60° above the horizontal. What are the (a) magnitude and (b) angle of its velocity 2.0 s after launch, and (c) is the angle above or below the horizontal? What are the (d) magnitude and (e) angle of its velocity 5.0 s after launch, and (f) is the angle above or below the horizontal?

y

107 A particle P travels with constant speed on a circle of radius r " 3.00 m (Fig. 4-56) and completes one revolution in 20.0 s. The particle P r passes through O at time t " 0. State the following vectors in magnitudeangle notation (angle relative to the positive direction of x). With respect to O, find the particle’s position vecx tor at the times t of (a) 5.00 s, (b) O 7.50 s, and (c) 10.0 s. (d) For the Figure 4-56 Problem 107. 5.00 s interval from the end of the fifth second to the end of the tenth second, find the particle’s displacement. For that interval, find (e) its average velocity and its velocity at the (f) beginning and (g) end. Next, find the acceleration at the (h) beginning and (i) end of that interval. 108 The fast French train known as the TGV (Train à Grande Vitesse) has a scheduled average speed of 216 km/h. (a) If the train goes around a curve at that speed and the magnitude of the acceleration experienced by the passengers is to be limited to 0.050g, what is the smallest radius of curvature for the track that can be tolerated? (b) At what speed must the train go around a curve with a 1.00 km radius to be at the acceleration limit? 109 (a) If an electron is projected horizontally with a speed of 3.0 ' 10 6 m/s, how far will it fall in traversing 1.0 m of horizontal distance? (b) Does the answer increase or decrease if the initial speed is increased? 110 A person walks up a stalled 15-m-long escalator in 90 s. When standing on the same escalator, now moving, the person is carried up in 60 s. How much time would it take that person to walk up the moving escalator? Does the answer depend on the length of the escalator? 111 (a) What is the magnitude of the centripetal acceleration of an object on Earth’s equator due to the rotation of Earth? (b) What would Earth’s rotation period have to be for objects on the equator to have a centripetal acceleration of magnitude 9.8 m/s2? 112 The range of a projectile depends not only on v0 and 10 but also on the value g of the free-fall acceleration, which varies from place to place. In 1936, Jesse Owens established a world’s running broad jump record of 8.09 m at the Olympic Games at Berlin (where g " 9.8128 m/s2). Assuming the same values of v0 and 10, by how much would his record have differed if he had competed instead in 1956 at Melbourne (where g " 9.7999 m/s2)?

92

CHAPTE R 4 M OTION I N TWO AN D TH R E E DI M E NSIONS

113 Figure 4-57 shows the path taken by a drunk skunk over level ground, from initial point i to final point f. The angles are 11 " 30.0°, 12 " 50.0°, and 13 " 80.0°, and the distances are d1 " 5.00 m, d2 " 8.00 m, and d3 " 12.0 m. What are the (a) magnitude and (b) angle of the skunk’s displacement from i to f?

120 A sprinter running on a circular track has a velocity of constant magnitude 9.20 m/s and a centripetal acceleration of magnitude 3.80 m/s2. What are (a) the track radius and (b) the period of the circular motion?

y d2

θ3

θ2 d1

d3

θ1 i

x

114 The position vector : r of a particle moving in the xy plane is : r " 2tiˆ # 2 sin[()/4 rad/s)t] jˆ , with f : Figure 4-57 Problem 113. r in meters and t in seconds. (a) Calculate the x and y components of the particle’s position at t " 0, 1.0, 2.0, 3.0, and 4.0 s and sketch the particle’s path in the xy plane for the interval 0 0 t 0 4.0 s. (b) Calculate the components of the particle’s velocity at t " 1.0, 2.0, and 3.0 s. Show that the velocity is tangent to the path of the particle and in the direction the particle is moving at each time by drawing the velocity vectors on the plot of the particle’s path in part (a). (c) Calculate the components of the particle’s acceleration at t " 1.0, 2.0, and 3.0 s. 115 An electron having an initial horizontal velocity of magnitude 1.00 ' 10 9 cm/s travels into the region between two horizontal metal plates that are electrically charged. In that region, the electron travels a horizontal distance of 2.00 cm and has a constant downward acceleration of magnitude 1.00 ' 10 17 cm/s2 due to the charged plates. Find (a) the time the electron takes to travel the 2.00 cm, (b) the vertical distance it travels during that time, and the magnitudes of its (c) horizontal and (d) vertical velocity components as it emerges from the region. 116 An elevator without a ceiling is ascending with a constant speed of 10 m/s. A boy on the elevator shoots a ball directly upward, from a height of 2.0 m above the elevator floor, just as the elevator floor is 28 m above the ground. The initial speed of the ball with respect to the elevator is 20 m/s. (a) What maximum height above the ground does the ball reach? (b) How long does the ball take to return to the elevator floor? 117 A football player punts the football so that it will have a “hang time” (time of flight) of 4.5 s and land 46 m away. If the ball leaves the player’s foot 150 cm above the ground, what must be the (a) magnitude and (b) angle (relative to the horizontal) of the ball’s initial velocity? 118 An airport terminal has a moving sidewalk to speed passengers through a long corridor. Larry does not use the moving sidewalk; he takes 150 s to walk through the corridor. Curly, who simply stands on the moving sidewalk, covers the same distance in 70 s. Moe boards the sidewalk and walks along it. How long does Moe take to move through the corridor? Assume that Larry and Moe walk at the same speed. 119 A wooden boxcar is moving along a straight railroad track at speed v1. A sniper fires a bullet (initial speed v2) at it from a high-powered rifle. The bullet passes through both lengthwise walls of the car, its entrance and exit holes being exactly opposite each other as viewed from within the car. From what direction, relative to the track, is the bullet fired? Assume that the bullet is not deflected upon entering the car, but that its speed decreases by 20%. Take v1 " 85 km/h and v2 " 650 m/s. (Why don’t you need to know the width of the boxcar?)

121 Suppose that a space probe can withstand the stresses of a 20g acceleration. (a) What is the minimum turning radius of such a craft moving at a speed of one-tenth the speed of light? (b) How long would it take to complete a 90° turn at this speed? 122 You are to throw a ball with a speed of 12.0 m/s at a target that is height h = 5.00 m above the level at which you release the ball (Fig. 4-58). You want the ball’s velocity to be horizontal at the instant it reaches the target. (a) At what angle 1 above the horizontal must you throw the ball? (b) What is the horizontal distance from the release point to the target? (c) What is the speed of the ball just as it reaches the target? 123 A projectile is fired with an initial speed v0 = 30.0 m/s from level ground at a target that is on the ground, at distance R = 20.0 m, as shown in Fig. 4-59. What are the (a) least and (b) greatest launch angles that will allow the projectile to hit the target?

Target

h

θ

Figure 4-58 Problem 122. High trajectory v0

Low trajectory v0 R

Figure 4-59 Problem 123.

124 A graphing surprise. At time t = 0, a burrito is launched from level ground, with an initial speed of 16.0 m/s and launch angle 10. Imagine a position vector : r continuously directed from the launching point to the burrito during the flight. Graph the magnitude r of the position vector for (a) 10 = 40.0° and (b) 10 = 80.0°. For 10 = 40.0°, (c) when does r reach its maximum value, (d) what is that value, and how far (e) horizontally and (f) vertically is the burrito from the launch point? For 10 = 80.0°, (g) when does r reach its maximum value, (h) what is that value, and how far (i) horizontally and (j) vertically is the burrito from the launch point? 125 A cannon located at sea level fires a ball with initial speed 82 m/s and initial angle 45°. The ball lands in the water after traveling a horizontal distance 686 m. How much greater would the horizontal distance have been had the cannon been 30 m higher? 126 The magnitude of the velocity of a projectile when it is at its maximum height above ground level is 10.0 m/s. (a) What is the magnitude of the velocity of the projectile 1.00 s before it achieves its maximum height? (b) What is the magnitude of the velocity of the projectile 1.00 s after it achieves its maximum height? If we take x = 0 and y = 0 to be at the point of maximum height and positive x to be in the direction of the velocity there, what are the (c) x coordinate and (d) y coordinate of the projectile 1.00 s before it reaches its maximum height and the (e) x coordinate and (f) y coordinate 1.0 s after it reaches its maximum height? 127 A frightened rabbit moving at 6.00 m/s due east runs onto a large area of level ice of negligible friction. As the rabbit slides across the ice, the force of the wind causes it to have a constant acceleration of 1.40 m/s2, due north. Choose a coordinate system with the origin at the rabbit’s initial position on the ice and the positive x axis directed toward the east. In unit-vector notation, what are the rabbit’s (a) velocity and (b) position when it has slid for 3.00 s?

PROB LE M S

128 The pilot of an aircraft flies due east relative to the ground in a wind blowing 20.0 km/h toward the south. If the speed of the aircraft in the absence of wind is 70.0 km/h, what is the speed of the aircraft relative to the ground? 129 The pitcher in a slow-pitch softball game releases the ball at a point 3.0 ft above ground level.A stroboscopic plot of the position of the ball is shown in Fig. 4-60, where the readings are 0.25 s apart and the ball is released at t = 0. (a) What is the initial speed of the ball? (b) What is the speed of the ball at the instant it reaches its maximum height above ground level? (c) What is that maximum height?

y (ft)

10

5 t=0 0

10

20 x (ft)

30

40

Figure 4-60 Problem 129. 130 Some state trooper departments use aircraft to enforce highway speed limits. Suppose that one of the airplanes has a speed of 135 mi/h in still air. It is flying straight north so that it is at all times directly above a north–south highway. A ground observer tells the pilot by radio that a 70.0 mi/h wind is blowing but neglects to give the wind direction. The pilot observes that in spite of the wind the plane can travel 135 mi along the highway in 1.00 h. In other words, the ground speed is the same as if there were no wind. (a) From what direction is the wind blowing? (b) What is the heading of the plane; that is, in what direction does it point? 131 A golfer tees off from the top of a rise, giving the golf ball an initial velocity of 43.0 m/s at an angle of 30.0° above the horizontal. The ball strikes the fairway a horizontal distance of 180 m from the tee. Assume the fairway is level. (a) How high is the rise above the fairway? (b) What is the speed of the ball as it strikes the fairway? 132 A track meet is held on a planet in a distant solar system. A shot-putter releases a shot at a point 2.0 m above ground level. A stroboscopic plot of the position of the shot is shown in Fig. 4-61,

y (m)

10

5 t=0 0

5

10

15 x (m)

20

Figure 4-61 Problem 132.

25

30

93

where the readings are 0.50 s apart and the shot is released at time t = 0. (a) What is the initial velocity of the shot in unit-vector notation? (b) What is the magnitude of the free-fall acceleration on the planet? (c) How long after it is released does the shot reach the ground? (d) If an identical throw of the shot is made on the surface of Earth, how long after it is released does it reach the ground? 133 A helicopter is flying in a straight line over a level field at a constant speed of 6.20 m/s and at a constant altitude of 9.50 m. A package is ejected horizontally from the helicopter with an initial velocity of 12.0 m/s relative to the helicopter and in a direction opposite the helicopter’s motion. (a) Find the initial speed of the package relative to the ground. (b) What is the horizontal distance between the helicopter and the package at the instant the package strikes the ground? (c) What angle does the velocity vector of the package make with the ground at the instant before impact, as seen from the ground? 134 A car travels around a flat circle on the ground, at a constant speed of 12.0 m/s. At a certain instant the car has an acceleration of 3.00 m/s2 toward the east. What are its distance and direction from the center of the circle at that instant if it is traveling (a) clockwise around the circle and (b) counterclockwise around the circle? 135 You throw a ball from a cliff with an initial velocity of 15.0 m/s at an angle of 20.0° below the horizontal. Find (a) its horizontal displacement and (b) its vertical displacement 2.30 s later. 136 A baseball is hit at Fenway Park in Boston at a point 0.762 m above home plate with an initial velocity of 33.53 m/s directed 55.0° above the horizontal. The ball is observed to clear the 11.28-m-high wall in left field (known as the “green monster”) 5.00 s after it is hit, at a point just inside the left-field foulline pole. Find (a) the horizontal distance down the left-field foul line from home plate to the wall; (b) the vertical distance by which the ball clears the wall; (c) the horizontal and vertical displacements of the ball with respect to home plate 0.500 s before it clears the wall. 137 A transcontinental flight of 4350 km is scheduled to take 50 min longer westward than eastward. The airspeed of the airplane is 966 km/h, and the jet stream it will fly through is presumed to move due east. What is the assumed speed of the jet stream? 138 A woman can row a boat at 6.40 km/h in still water. (a) If she is crossing a river where the current is 3.20 km/h, in what direction must her boat be headed if she wants to reach a point directly opposite her starting point? (b) If the river is 6.40 km wide, how long will she take to cross the river? (c) Suppose that instead of crossing the river she rows 3.20 km down the river and then back to her starting point. How long will she take? (d) How long will she take to row 3.20 km up the river and then back to her starting point? (e) In what direction should she head the boat if she wants to cross in the shortest possible time, and what is that time?

C

H

A

P

T

E

R

5

Force and Motion—I 5-1 NEWTON’S FIRST AND SECOND LAWS Learning Objectives After reading this module, you should be able to . . .

5.01 Identify that a force is a vector quantity and thus has both magnitude and direction and also components. 5.02 Given two or more forces acting on the same particle, add the forces as vectors to get the net force. 5.03 Identify Newton’s first and second laws of motion. 5.04 Identify inertial reference frames. 5.05 Sketch a free-body diagram for an object, showing the

object as a particle and drawing the forces acting on it as vectors with their tails anchored on the particle. 5.06 Apply the relationship (Newton’s second law) between the net force on an object, the mass of the object, and the acceleration produced by the net force. 5.07 Identify that only external forces on an object can cause the object to accelerate.

Key Ideas ● The velocity of an object can change (the object can accelerate) when the object is acted on by one or more forces (pushes or pulls) from other objects. Newtonian mechanics relates accelerations and forces. ● Forces are vector quantities. Their magnitudes are defined in terms of the acceleration they would give the standard kilogram. A force that accelerates that standard body by exactly 1 m/s2 is defined to have a magnitude of 1 N. The direction of a force is the direction of the acceleration it causes. Forces are combined according to the rules of vector algebra. The net force on a body is the vector sum of all the forces acting on the body. ● If there is no net force on a body, the body remains at rest if

it is initially at rest or moves in a straight line at constant speed if it is in motion. ● Reference frames in which Newtonian mechanics holds are

called inertial reference frames or inertial frames. Reference frames in which Newtonian mechanics does not hold are called noninertial reference frames or noninertial frames.

● The mass of a body is the characteristic of that body that

relates the body’s acceleration to the net force causing the acceleration. Masses are scalar quantities. : ● The net force Fnet on a body with mass m is related to the body’s acceleration : a by :

:

Fnet " ma , which may be written in the component versions Fnet,x " max Fnet,y " may and

Fnet,z " maz.

The second law indicates that in SI units 1 N " 1 kg!m/s2. ● A free-body diagram is a stripped-down diagram in which

only one body is considered. That body is represented by either a sketch or a dot. The external forces on the body are drawn, and a coordinate system is superimposed, oriented so as to simplify the solution.

What Is Physics? We have seen that part of physics is a study of motion, including accelerations, which are changes in velocities. Physics is also a study of what can cause an object to accelerate. That cause is a force, which is, loosely speaking, a push or pull on the object. The force is said to act on the object to change its velocity. For example, when a dragster accelerates, a force from the track acts on the rear tires to cause the dragster’s acceleration. When a defensive guard knocks down a quarterback, a force from the guard acts on the quarterback to cause the quarterback’s backward acceleration. When a car slams into a telephone pole, a force on the car from the 94

5-1 N EWTON’S FI RST AN D SECON D L AWS

pole causes the car to stop. Science, engineering, legal, and medical journals are filled with articles about forces on objects, including people. A Heads Up. Many students find this chapter to be more challenging than the preceding ones. One reason is that we need to use vectors in setting up equations— we cannot just sum some scalars. So, we need the vector rules from Chapter 3. Another reason is that we shall see a lot of different arrangements: objects will move along floors, ceilings, walls, and ramps. They will move upward on ropes looped around pulleys or by sitting in ascending or descending elevators. Sometimes, objects will even be tied together. However, in spite of the variety of arrangements, we need only a single key idea (Newton’s second law) to solve most of the homework problems. The purpose of this chapter is for us to explore how we can apply that single key idea to any given arrangement. The application will take experience—we need to solve lots of problems, not just read words. So, let’s go through some of the words and then get to the sample problems.

Newtonian Mechanics The relation between a force and the acceleration it causes was first understood by Isaac Newton (1642 –1727) and is the subject of this chapter. The study of that relation, as Newton presented it, is called Newtonian mechanics. We shall focus on its three primary laws of motion. Newtonian mechanics does not apply to all situations. If the speeds of the interacting bodies are very large —an appreciable fraction of the speed of light — we must replace Newtonian mechanics with Einstein’s special theory of relativity, which holds at any speed, including those near the speed of light. If the interacting bodies are on the scale of atomic structure (for example, they might be electrons in an atom), we must replace Newtonian mechanics with quantum mechanics. Physicists now view Newtonian mechanics as a special case of these two more comprehensive theories. Still, it is a very important special case because it applies to the motion of objects ranging in size from the very small (almost on the scale of atomic structure) to astronomical (galaxies and clusters of galaxies).

Newton’s First Law Before Newton formulated his mechanics, it was thought that some influence, a “force,” was needed to keep a body moving at constant velocity. Similarly, a body was thought to be in its “natural state” when it was at rest. For a body to move with constant velocity, it seemingly had to be propelled in some way, by a push or a pull. Otherwise, it would “naturally” stop moving. These ideas were reasonable. If you send a puck sliding across a wooden floor, it does indeed slow and then stop. If you want to make it move across the floor with constant velocity, you have to continuously pull or push it. Send a puck sliding over the ice of a skating rink, however, and it goes a lot farther. You can imagine longer and more slippery surfaces, over which the puck would slide farther and farther. In the limit you can think of a long, extremely slippery surface (said to be a frictionless surface), over which the puck would hardly slow. (We can in fact come close to this situation by sending a puck sliding over a horizontal air table, across which it moves on a film of air.) From these observations, we can conclude that a body will keep moving with constant velocity if no force acts on it. That leads us to the first of Newton’s three laws of motion: Newton’s First Law: If no force acts on a body, the body’s velocity cannot change; that is, the body cannot accelerate.

95

96

CHAPTE R 5 FORCE AN D M OTION—I

In other words, if the body is at rest, it stays at rest. If it is moving, it continues to move with the same velocity (same magnitude and same direction). F

Force a :

Figure 5-1 A force F on the standard kilogram gives that body an acceleration : a.

Before we begin working problems with forces, we need to discuss several features of forces, such as the force unit, the vector nature of forces, the combining of forces, and the circumstances in which we can measure forces (without being fooled by a fictitious force). Unit. We can define the unit of force in terms of the acceleration a force would give to the standard kilogram (Fig. 1-3), which has a mass defined to be exactly 1 kg. Suppose we put that body on a horizontal, frictionless surface and pull horizontally (Fig. 5-1) such that the body has an acceleration of 1 m/s2. Then we can define our applied force as having a magnitude of 1 newton (abbreviated N). If we then pulled with a force magnitude of 2 N, we would find that the acceleration is 2 m/s2. Thus, the acceleration is proportional to the force. If the standard body of 1 kg has an acceleration of magnitude a (in meters per second per second), then the force (in newtons) producing the acceleration has a magnitude equal to a. We now have a workable definition of the force unit. Vectors. Force is a vector quantity and thus has not only magnitude but also direction. So, if two or more forces act on a body, we find the net force (or resultant force) by adding them as vectors, following the rules of Chapter 3. A single force that has the same magnitude and direction as the calculated net force would then have the same effect as all the individual forces. This fact, called the principle of superposition for forces, makes everyday forces reasonable and predictable. The world would indeed be strange and unpredictable if, say, you and a friend each pulled on the standard body with a force of 1 N and somehow the net pull was 14 N and the resulting acceleration was 14 m/s2. In this book, forces are most often represented with a vector symbol such as : : F, and a net force is represented with the vector symbol Fnet.As with other vectors, a force or a net force can have components along coordinate axes.When forces act only along a single axis, they are single-component forces. Then we can drop the overhead arrows on the force symbols and just use signs to indicate the directions of the forces along that axis. The First Law. Instead of our previous wording, the more proper statement of Newton’s First Law is in terms of a net force: :

Newton’s First Law: If no net force acts on a body (Fnet " 0), the body’s velocity cannot change; that is, the body cannot accelerate.

There may be multiple forces acting on a body, but if their net force is zero, the body cannot accelerate. So, if we happen to know that a body’s velocity is constant, we can immediately say that the net force on it is zero.

Inertial Reference Frames Newton’s first law is not true in all reference frames, but we can always find reference frames in which it (as well as the rest of Newtonian mechanics) is true. Such special frames are referred to as inertial reference frames, or simply inertial frames. An inertial reference frame is one in which Newton’s laws hold.

For example, we can assume that the ground is an inertial frame provided we can neglect Earth’s astronomical motions (such as its rotation).

5-1 N EWTON’S FI RST AN D SECON D L AWS

That assumption works well if, say, a puck is sent sliding along a short strip of frictionless ice — we would find that the puck’s motion obeys Newton’s laws. However, suppose the puck is sent sliding along a long ice strip extending from the north pole (Fig. 5-2a). If we view the puck from a stationary frame in space, the puck moves south along a simple straight line because Earth’s rotation around the north pole merely slides the ice beneath the puck. However, if we view the puck from a point on the ground so that we rotate with Earth, the puck’s path is not a simple straight line. Because the eastward speed of the ground beneath the puck is greater the farther south the puck slides, from our ground-based view the puck appears to be deflected westward (Fig. 5-2b). However, this apparent deflection is caused not by a force as required by Newton’s laws but by the fact that we see the puck from a rotating frame. In this situation, the ground is a noninertial frame, and trying to explain the deflection in terms of a force would lead us to a fictitious force. A more common example of inventing such a nonexistent force can occur in a car that is rapidly increasing in speed. You might claim that a force to the rear shoves you hard into the seat back. In this book we usually assume that the ground is an inertial frame and that measured forces and accelerations are from this frame. If measurements are made in, say, a vehicle that is accelerating relative to the ground, then the measurements are being made in a noninertial frame and the results can be surprising.

Checkpoint 1 :

Which of the figure’s six arrangements correctly show the vector addition of forces F1 : : and F2 to yield the third vector, which is meant to represent their net force Fnet?

F1

F1

F2

(a)

F1

F2

(b)

F2

(c)

F2

F1 (d)

F1 (e)

F1

F2

(f )

F2

Mass From everyday experience you already know that applying a given force to bodies (say, a baseball and a bowling ball) results in different accelerations. The common explanation is correct: The object with the larger mass is accelerated less. But we can be more precise. The acceleration is actually inversely related to the mass (rather than, say, the square of the mass). Let’s justify that inverse relationship. Suppose, as previously, we push on the standard body (defined to have a mass of exactly 1 kg) with a force of magnitude 1 N. The body accelerates with a magnitude of 1 m/s2. Next we push on body X with the same force and find that it accelerates at 0.25 m/s2. Let’s make the (correct) assumption that with the same force, mX a " 0, m0 aX

97

N W

E S

(a)

Earth's rotation causes an apparent deflection. (b)

Figure 5-2 (a) The path of a puck sliding from the north pole as seen from a stationary point in space. Earth rotates to the east. (b) The path of the puck as seen from the ground.

98

CHAPTE R 5 FORCE AN D M OTION—I

and thus mX " m0

a0 1.0 m/s2 " (1.0 kg) " 4.0 kg. aX 0.25 m/s2

Defining the mass of X in this way is useful only if the procedure is consistent. Suppose we apply an 8.0 N force first to the standard body (getting an acceleration of 8.0 m/s2) and then to body X (getting an acceleration of 2.0 m/s2). We would then calculate the mass of X as mX " m0

a0 8.0 m/s2 " (1.0 kg) " 4.0 kg, aX 2.0 m/s2

which means that our procedure is consistent and thus usable. The results also suggest that mass is an intrinsic characteristic of a body—it automatically comes with the existence of the body. Also, it is a scalar quantity. However, the nagging question remains: What, exactly, is mass? Since the word mass is used in everyday English, we should have some intuitive understanding of it, maybe something that we can physically sense. Is it a body’s size, weight, or density? The answer is no, although those characteristics are sometimes confused with mass. We can say only that the mass of a body is the characteristic that relates a force on the body to the resulting acceleration. Mass has no more familiar definition; you can have a physical sensation of mass only when you try to accelerate a body, as in the kicking of a baseball or a bowling ball.

Newton’s Second Law All the definitions, experiments, and observations we have discussed so far can be summarized in one neat statement: Newton’s Second Law: The net force on a body is equal to the product of the body’s mass and its acceleration.

In equation form, :

: F net " ma

(Newton’s second law).

(5-1)

Identify the Body. This simple equation is the key idea for nearly all the homework problems in this chapter, but we must use it cautiously. First, we must : be certain about which body we are applying it to. Then Fnet must be the vector sum of all the forces that act on that body. Only forces that act on that body are to be included in the vector sum, not forces acting on other bodies that might be involved in the given situation. For example, if you are in a rugby scrum, the net force on you is the vector sum of all the pushes and pulls on your body. It does not include any push or pull on another player from you or from anyone else. Every time you work a force problem, your first step is to clearly state the body to which you are applying Newton’s law. Separate Axes. Like other vector equations, Eq. 5-1 is equivalent to three component equations, one for each axis of an xyz coordinate system: Fnet, x " max,

Fnet, y " may,

and

Fnet, z " maz.

(5-2)

Each of these equations relates the net force component along an axis to the acceleration along that same axis. For example, the first equation tells us that the sum of all the force components along the x axis causes the x component ax of the body’s acceleration, but causes no acceleration in the y and z directions. Turned around, the acceleration component ax is caused only by the sum of the

5-1 N EWTON’S FI RST AN D SECON D L AWS

force components along the x axis and is completely unrelated to force components along another axis. In general, The acceleration component along a given axis is caused only by the sum of the force components along that same axis, and not by force components along any other axis.

Forces in Equilibrium. Equation 5-1 tells us that if the net force on a body is : zero, the body’s acceleration a " 0. If the body is at rest, it stays at rest; if it is moving, it continues to move at constant velocity. In such cases, any forces on the body balance one another, and both the forces and the body are said to be in equilibrium. Commonly, the forces are also said to cancel one another, but the term “cancel” is tricky. It does not mean that the forces cease to exist (canceling forces is not like canceling dinner reservations). The forces still act on the body but cannot change the velocity. Units. For SI units, Eq. 5-1 tells us that 1 N " (1 kg)(1 m/s2) " 1 kg !m/s2.

(5-3)

Some force units in other systems of units are given in Table 5-1 and Appendix D. Diagrams. To solve problems with Newton’s second law, we often draw a free-body diagram in which the only body shown is the one for which we are summing forces. A sketch of the body itself is preferred by some teachers but, to save space in these chapters, we shall usually represent the body with a dot. Also, each force on the body is drawn as a vector arrow with its tail anchored on the body. A coordinate system is usually included, and the acceleration of the body is sometimes shown with a vector arrow (labeled as an acceleration). This whole procedure is designed to focus our attention on the body of interest. Table 5-1 Units in Newton’s Second Law (Eqs. 5-1 and 5-2) System

Force

Mass

Acceleration

SI CGSa Britishb

newton (N) dyne pound (lb)

kilogram (kg) gram (g) slug

m/s2 cm/s2 ft/s2

1 dyne " 1 g !cm/s2.

a

1 lb " 1 slug !ft/s2.

b

External Forces Only. A system consists of one or more bodies, and any force on the bodies inside the system from bodies outside the system is called an external force. If the bodies making up a system are rigidly connected to one an: other, we can treat the system as one composite body, and the net force F net on it is the vector sum of all external forces. (We do not include internal forces — that is, forces between two bodies inside the system. Internal forces cannot accelerate the system.) For example, a connected railroad engine and car form a system. If, say, a tow line pulls on the front of the engine, the force due to the tow line acts on the whole engine – car system. Just as for a single body, we can relate the net ex: : ternal force on a system to its acceleration with Newton’s second law, F net " ma , where m is the total mass of the system.

Checkpoint 2

3N

5N

The figure here shows two horizontal forces acting on a block on a frictionless floor. If a third horizon: : tal force F3 also acts on the block, what are the magnitude and direction of F3 when the block is (a) stationary and (b) moving to the left with a constant speed of 5 m/s?

99

100

CHAPTE R 5 FORCE AN D M OTION—I

Sample Problem 5.01 One- and two-dimensional forces, puck Here are examples of how to use Newton’s second law for a puck when one or two forces act on it. Parts A, B, and C of Fig. 5-3 show three situations in which one or two forces act on a puck that moves over frictionless ice along an x axis, in one-dimensional motion. The puck’s mass is m " 0.20 kg. : : Forces F1 and F2 are directed along the axis and have : magnitudes F1 " 4.0 N and F2 " 2.0 N. Force F3 is directed at angle u " 303 and has magnitude F3 " 1.0 N. In each situation, what is the acceleration of the puck?

A F1

x

The horizontal force causes a horizontal acceleration.

(a) Puck

F1

x

This is a free-body diagram.

(b)

KEY IDEA

B :

In each situation we can relate the acceleration a to the net : force Fnet acting on the puck with Newton’s second law, : : . However, because the motion is along only the x Fnet " ma axis, we can simplify each situation by writing the second law for x components only: Fnet, x " max.

F2

These forces compete. Their net force causes a horizontal acceleration.

(c) F2

(5-4)

The free-body diagrams for the three situations are also given in Fig. 5-3, with the puck represented by a dot.

F1

x

This is a free-body diagram.

x

Only the horizontal component of F3 competes with F2.

x

This is a free-body diagram.

(d) C

Situation A: For Fig. 5-3b, where only one horizontal force acts, Eq. 5-4 gives us

F2

F1 " max,

θ

F3 (e)

which, with given data, yields ax "

F1 x

F1 4.0 N " " 20 m/s2. m 0.20 kg

F2

(Answer)

The positive answer indicates that the acceleration is in the positive direction of the x axis. Situation B: In Fig. 5-3d, two horizontal forces act on the : : puck, F1 in the positive direction of x and F2 in the negative direction. Now Eq. 5-4 gives us

F3

θ

(f )

Figure 5-3 In three situations, forces act on a puck that moves along an x axis. Free-body diagrams are also shown.

dimensional.) Thus, we write Eq. 5-4 as

F1 $ F2 " max, which, with given data, yields 4.0 N $ 2.0 N F1 $ F2 " " 10 m/s2. m 0.20 kg (Answer) Thus, the net force accelerates the puck in the positive direction of the x axis. ax "

F3, x $ F2 " max.

From the figure, we see that F3,x " F3 cos u. Solving for the acceleration and substituting for F3,x yield F3,x $ F2 F3 cos 1 $ F2 " m m (1.0 N)(cos 303) $ 2.0 N " " $5.7 m/s2. 0.20 kg

ax "

(Answer)

:

Situation C: In Fig. 5-3f, force F3 is not directed along the direction of the puck’s acceleration; only x component F3, x : is. (Force F3 is two-dimensional but the motion is only one-

(5-5)

Thus, the net force accelerates the puck in the negative direction of the x axis.

Additional examples, video, and practice available at WileyPLUS

101

5-1 N EWTON’S FI RST AN D SECON D L AWS

Sample Problem 5.02 Two-dimensional forces, cookie tin Here we find a missing force by using the acceleration. In the overhead view of Fig. 5-4a, a 2.0 kg cookie tin is accelerated at 3.0 m/s2 in the direction shown by : a , over a frictionless horizontal surface. The acceleration is caused by three : horizontal forces, only two of which are shown: F 1 of magni: tude 10 N and F 2 of magnitude 20 N. What is the third force : F 3 in unit-vector notation and in magnitude-angle notation?

x components: Along the x axis we have F3,x " max $ F1, x $ F2,x " m(a cos 503) $ F1 cos($1503) $ F2 cos 903. Then, substituting known data, we find F3,x " (2.0 kg)(3.0 m/s2) cos 503 $ (10 N) cos($1503) $ (20 N) cos 903 " 12.5 N.

KEY IDEA :

The net force F net on the tin is the sum of the three forces and is related to the acceleration : a via Newton’s second law : : (Fnet " ma ). Thus, :

:

:

: , F 1 # F 2 # F 3 " ma

y components: Similarly, along the y axis we find F3, y " may $ F1, y $ F2, y " m(a sin 503) $ F1 sin($1503) $ F2 sin 903 " (2.0 kg)(3.0 m/s2) sin 503 $ (10 N) sin($1503)

(5-6)

$ (20 N) sin 903

which gives us

" $10.4 N. :

:

:

: F 3 " ma $ F 1 $ F 2.

(5-7)

Vector: In unit-vector notation, we can write F 3 " F3, x ˆi # F3, y ˆj " (12.5 N)ˆi $ (10.4 N)ˆj % (13 N)ˆi $ (10 N)ˆj . (Answer) :

Calculations: Because this is a two-dimensional problem, : we cannot find F 3 merely by substituting the magnitudes for the vector quantities on the right side of Eq. 5-7. Instead, we : : : : must vectorially add ma , $F1 (the reverse of F 1), and $F 2 : (the reverse of F 2), as shown in Fig. 5-4b. This addition can be done directly on a vector-capable calculator because we know both magnitude and angle for all three vectors. However, here we shall evaluate the right side of Eq. 5-7 in terms of components, first along the x axis and then along the y axis. Caution: Use only one axis at a time.

We can now use a vector-capable calculator to get the mag: nitude and the angle of F 3 . We can also use Eq. 3-6 to obtain the magnitude and the angle (from the positive direction of the x axis) as F3 " 2F 23,x # F 23,y " 16 N and

1 " tan$1

F3,y

F3, x

" $403.

y

These are two of the three horizontal force vectors.

F2

y

This is the resulting horizontal acceleration vector.

– F1

a 50°

30°

– F2

ma

x

F1 (a)

We draw the product of mass and acceleration as a vector.

(b)

x

F3

Then we can add the three vectors to find the missing third force vector.

Figure 5-4 (a) An overhead view of two of three horizontal forces that act on a cookie : : : tin, resulting in acceleration : , $F1, a . F 3 is not shown. (b) An arrangement of vectors ma : : and $F2 to find force F3. Additional examples, video, and practice available at WileyPLUS

(Answer)

102

CHAPTE R 5 FORCE AN D M OTION—I

5-2 SOME PARTICULAR FORCES Learning Objectives After reading this module, you should be able to . . .

5.08 Determine the magnitude and direction of the gravitational force acting on a body with a given mass, at a location with a given free-fall acceleration. 5.09 Identify that the weight of a body is the magnitude of the net force required to prevent the body from falling freely, as measured from the reference frame of the ground. 5.10 Identify that a scale gives an object’s weight when the measurement is done in an inertial frame but not in an accelerating frame, where it gives an apparent weight.

Key Ideas

:

● A gravitational force Fg on a body is a pull by another body.

In most situations in this book, the other body is Earth or some other astronomical body. For Earth, the force is directed down toward the ground, which is assumed to be an inertial : frame. With that assumption, the magnitude of Fg is Fg " mg, where m is the body’s mass and g is the magnitude of the free-fall acceleration. ● The weight W of a body is the magnitude of the upward force

needed to balance the gravitational force on the body. A body’s weight is related to the body’s mass by W " mg.

5.11 Determine the magnitude and direction of the normal force on an object when the object is pressed or pulled onto a surface. 5.12 Identify that the force parallel to the surface is a frictional force that appears when the object slides or attempts to slide along the surface. 5.13 Identify that a tension force is said to pull at both ends of a cord (or a cord-like object) when the cord is taut.

:

● A normal force FN is the force on a body from a surface

against which the body presses. The normal force is always perpendicular to the surface. ● A frictional force

:

f is the force on a body when the body slides or attempts to slide along a surface. The force is always parallel to the surface and directed so as to oppose the sliding. On a frictionless surface, the frictional force is negligible. ● When a cord is under tension, each end of the cord pulls

on a body. The pull is directed along the cord, away from the point of attachment to the body. For a massless cord (a cord with negligible mass), the pulls at both ends of the cord have the same magnitude T, even if the cord runs around a massless, frictionless pulley (a pulley with negligible mass and negligible friction on its axle to oppose its rotation).

Some Particular Forces The Gravitational Force :

A gravitational force F g on a body is a certain type of pull that is directed toward a second body. In these early chapters, we do not discuss the nature of this force and usually consider situations in which the second body is Earth. Thus, when we : speak of the gravitational force F g on a body, we usually mean a force that pulls on it directly toward the center of Earth — that is, directly down toward the ground. We shall assume that the ground is an inertial frame. Free Fall. Suppose a body of mass m is in free fall with the free-fall acceleration of magnitude g. Then, if we neglect the effects of the air, the only force acting : on the body is the gravitational force Fg. We can relate this downward force and : : downward acceleration with Newton’s second law (F " ma ). We place a vertical y axis along the body’s path, with the positive direction upward. For this axis, Newton’s second law can be written in the form Fnet,y " may , which, in our situation, becomes $Fg " m($g) or

Fg " mg.

(5-8)

In words, the magnitude of the gravitational force is equal to the product mg.

103

5-2 SOM E PARTICU L AR FORCES

At Rest. This same gravitational force, with the same magnitude, still acts on the body even when the body is not in free fall but is, say, at rest on a pool table or moving across the table. (For the gravitational force to disappear, Earth would have to disappear.) We can write Newton’s second law for the gravitational force in these vector forms: : Fg

: " $Fg ˆj " $mg ˆj " mg ,

mL

mR

FgL = mLg

FgR = mRg

(5-9)

where ˆj is the unit vector that points upward along a y axis, directly away from the ground, and : g is the free-fall acceleration (written as a vector), directed downward.

Weight The weight W of a body is the magnitude of the net force required to prevent the body from falling freely, as measured by someone on the ground. For example, to keep a ball at rest in your hand while you stand on the ground, you must provide an upward force to balance the gravitational force on the ball from Earth. Suppose the magnitude of the gravitational force is 2.0 N. Then the magnitude of your upward force must be 2.0 N, and thus the weight W of the ball is 2.0 N. We also say that the ball weighs 2.0 N and speak about the ball weighing 2.0 N. A ball with a weight of 3.0 N would require a greater force from you — namely, a 3.0 N force — to keep it at rest. The reason is that the gravitational force you must balance has a greater magnitude — namely, 3.0 N. We say that this second ball is heavier than the first ball. Now let us generalize the situation. Consider a body that has an acceleration : a of zero relative to the ground, which we again assume to be an inertial frame. : Two forces act on the body: a downward gravitational force F g and a balancing upward force of magnitude W. We can write Newton’s second law for a vertical y axis, with the positive direction upward, as

Figure 5-5 An equal-arm balance. When the device is in balance, the gravitational force : FgL on the body being weighed (on the left : pan) and the total gravitational force FgR on the reference bodies (on the right pan) are equal. Thus, the mass mL of the body being weighed is equal to the total mass mR of the reference bodies.

Fnet,y " may. In our situation, this becomes W $ Fg " m(0) or

W " Fg

(weight, with ground as inertial frame).

(5-10) (5-11)

This equation tells us (assuming the ground is an inertial frame) that Scale marked in either weight or mass units

The weight W of a body is equal to the magnitude Fg of the gravitational force on the body.

Substituting mg for Fg from Eq. 5-8, we find W " mg

(weight),

(5-12)

which relates a body’s weight to its mass. Weighing. To weigh a body means to measure its weight. One way to do this is to place the body on one of the pans of an equal-arm balance (Fig. 5-5) and then place reference bodies (whose masses are known) on the other pan until we strike a balance (so that the gravitational forces on the two sides match). The masses on the pans then match, and we know the mass of the body. If we know the value of g for the location of the balance, we can also find the weight of the body with Eq. 5-12. We can also weigh a body with a spring scale (Fig. 5-6). The body stretches a spring, moving a pointer along a scale that has been calibrated and marked in

Fg = mg

Figure 5-6 A spring scale. The reading is proportional to the weight of the object on the pan, and the scale gives that weight if marked in weight units. If, instead, it is marked in mass units, the reading is the object’s weight only if the value of g at the location where the scale is being used is the same as the value of g at the location where the scale was calibrated.

104

CHAPTE R 5 FORCE AN D M OTION—I

either mass or weight units. (Most bathroom scales in the United States work this way and are marked in the force unit pounds.) If the scale is marked in mass units, it is accurate only where the value of g is the same as where the scale was calibrated. The weight of a body must be measured when the body is not accelerating vertically relative to the ground. For example, you can measure your weight on a scale in your bathroom or on a fast train. However, if you repeat the measurement with the scale in an accelerating elevator, the reading differs from your weight because of the acceleration. Such a measurement is called an apparent weight. Caution: A body’s weight is not its mass. Weight is the magnitude of a force and is related to mass by Eq. 5-12. If you move a body to a point where the value of g is different, the body’s mass (an intrinsic property) is not different but the weight is. For example, the weight of a bowling ball having a mass of 7.2 kg is 71 N on Earth but only 12 N on the Moon. The mass is the same on Earth and Moon, but the free-fall acceleration on the Moon is only 1.6 m/s2.

The Normal Force If you stand on a mattress, Earth pulls you downward, but you remain stationary. The reason is that the mattress, because it deforms downward due to you, pushes up on you. Similarly, if you stand on a floor, it deforms (it is compressed, bent, or buckled ever so slightly) and pushes up on you. Even a seemingly rigid concrete floor does this (if it is not sitting directly on the ground, enough people on the floor could break it). : The push on you from the mattress or floor is a normal force F N. The name comes from the mathematical term normal, meaning perpendicular: The force on you from, say, the floor is perpendicular to the floor. When a body presses against a surface, the surface (even a seemingly rigid one) : deforms and pushes on the body with a normal force FN that is perpendicular to the surface.

Figure 5-7a shows an example. A block of mass m presses down on a table, : deforming it somewhat because of the gravitational force F g on the block. The : table pushes up on the block with normal force F N. The free-body diagram for the : : block is given in Fig. 5-7b. Forces F g and F N are the only two forces on the block and they are both vertical. Thus, for the block we can write Newton’s second law for a positive-upward y axis (Fnet, y " may) as FN $ Fg " may. y

The normal force is the force on the block from the supporting table.

Normal force FN

Block

FN

Block x

The gravitational force on the block is due to Earth's downward pull.

Fg (a)

The forces balance.

Fg

(b) :

Figure 5-7 (a) A block resting on a table experiences a normal force FN perpendicular to the tabletop. (b) The free-body diagram for the block.

5-2 SOM E PARTICU L AR FORCES

105

From Eq. 5-8, we substitute mg for Fg, finding FN $ mg " may. Then the magnitude of the normal force is FN " mg # may " m(g # ay)

(5-13)

for any vertical acceleration ay of the table and block (they might be in an accelerating elevator). (Caution: We have already included the sign for g but ay can be positive or negative here.) If the table and block are not accelerating relative to the ground, then ay " 0 and Eq. 5-13 yields (5-14)

FN " mg.

Checkpoint 3 :

In Fig. 5-7, is the magnitude of the normal force FN greater than, less than, or equal to mg if the block and table are in an elevator moving upward (a) at constant speed and (b) at increasing speed?

Friction If we either slide or attempt to slide a body over a surface, the motion is resisted by a bonding between the body and the surface. (We discuss this bonding more in : the next chapter.) The resistance is considered to be a single force f , called either the frictional force or simply friction. This force is directed along the surface, opposite the direction of the intended motion (Fig. 5-8). Sometimes, to simplify a situation, friction is assumed to be negligible (the surface, or even the body, is said to be frictionless).

Tension When a cord (or a rope, cable, or other such object) is attached to a body and : pulled taut, the cord pulls on the body with a force T directed away from the body and along the cord (Fig. 5-9a). The force is often called a tension force because the cord is said to be in a state of tension (or to be under tension), which means that it is being pulled taut. The tension in the cord is the magnitude T of the force on the body. For example, if the force on the body from the cord has magnitude T " 50 N, the tension in the cord is 50 N. A cord is often said to be massless (meaning its mass is negligible compared to the body’s mass) and unstretchable. The cord then exists only as a connection between two bodies. It pulls on both bodies with the same force magnitude T,

T

T

T

T

The forces at the two ends of the cord are equal in magnitude.

(a)

T

(b )

T

(c )

Figure 5-9 (a) The cord, pulled taut, is under tension. If its mass is negligible, the cord : pulls on the body and the hand with force T , even if the cord runs around a massless, frictionless pulley as in (b) and (c).

Direction of attempted slide f :

Figure 5-8 A frictional force f opposes the attempted slide of a body over a surface.

106

CHAPTE R 5 FORCE AN D M OTION—I

even if the bodies and the cord are accelerating and even if the cord runs around a massless, frictionless pulley (Figs. 5-9b and c). Such a pulley has negligible mass compared to the bodies and negligible friction on its axle opposing its rotation. If the cord wraps halfway around a pulley, as in Fig. 5-9c, the net force on the pulley from the cord has the magnitude 2T.

Checkpoint 4 The suspended body in Fig. 5-9c weighs 75 N. Is T equal to, greater than, or less than 75 N when the body is moving upward (a) at constant speed, (b) at increasing speed, and (c) at decreasing speed?

5-3 APPLYING NEWTON’S LAWS Learning Objectives After reading this module, you should be able to . . .

5.14 Identify Newton’s third law of motion and third-law force pairs. 5.15 For an object that moves vertically or on a horizontal or inclined plane, apply Newton’s second law to a free-body diagram of the object.

Key Ideas

:

● The net force Fnet on a body with mass m is related to the body’s :

acceleration a by

:

: Fnet " ma ,

5.16 For an arrangement where a system of several objects moves rigidly together, draw a free-body diagram and apply Newton’s second law for the individual objects and also for the system taken as a composite object.

:

● If a force FBC acts on body B due to body C, then there is

a force

: FCB

on body C due to body B: : : FBC " $FCB.

which may be written in the component versions Fnet,x " max Fnet,y " may and

The forces are equal in magnitude but opposite in directions.

Fnet,z " maz.

Newton’s Third Law Book B

Crate C

(a) FBC

FCB B

C (b)

The force on B due to C has the same magnitude as the force on C due to B.

Figure 5-10 (a) Book B leans against crate : C. (b) Forces FBC (the force on the book : from the crate) and F CB (the force on the crate from the book) have the same magnitude and are opposite in direction.

Two bodies are said to interact when they push or pull on each other — that is, when a force acts on each body due to the other body. For example, suppose you position a book B so it leans against a crate C (Fig. 5-10a). Then the book and : crate interact: There is a horizontal force FBC on the book from the crate (or due : to the crate) and a horizontal force FCB on the crate from the book (or due to the book). This pair of forces is shown in Fig. 5-10b. Newton’s third law states that Newton’s Third Law: When two bodies interact, the forces on the bodies from each other are always equal in magnitude and opposite in direction.

For the book and crate, we can write this law as the scalar relation FBC " FCB

(equal magnitudes)

or as the vector relation :

:

FBC " $FCB

(equal magnitudes and opposite directions),

(5-15)

where the minus sign means that these two forces are in opposite directions. We can call the forces between two interacting bodies a third-law force pair. When

5-3 APPLYI NG N EWTON’S L AWS

107

Cantaloupe FCE

Cantaloupe C

FEC

Table T

Earth

These are third-law force pairs.

Earth E (c)

(a)

F CT

These forces just happen to be balanced.

F CT (normal force from table) F TC F CE (gravitational force) (b)

Figure 5-11 (a) A cantaloupe lies on a table that stands on Earth. (b) The forces on : : the cantaloupe are FCT and FCE. (c) The third-law force pair for the cantaloupe – Earth interaction. (d) The third-law force pair for the cantaloupe – table interaction.

any two bodies interact in any situation, a third-law force pair is present. The book and crate in Fig. 5-10a are stationary, but the third law would still hold if they were moving and even if they were accelerating. As another example, let us find the third-law force pairs involving the cantaloupe in Fig. 5-11a, which lies on a table that stands on Earth. The cantaloupe interacts with the table and with Earth (this time, there are three bodies whose interactions we must sort out). Let’s first focus on the forces acting on the cantaloupe (Fig. 5-11b). Force : : FCT is the normal force on the cantaloupe from the table, and force FCE is the gravitational force on the cantaloupe due to Earth. Are they a third-law force pair? No, because they are forces on a single body, the cantaloupe, and not on two interacting bodies. To find a third-law pair, we must focus not on the cantaloupe but on the interaction between the cantaloupe and one other body. In the cantaloupe – Earth interaction (Fig. 5-11c), Earth pulls on the cantaloupe with a gravitational force : : FCE and the cantaloupe pulls on Earth with a gravitational force FEC. Are these forces a third-law force pair? Yes, because they are forces on two interacting bodies, the force on each due to the other.Thus, by Newton’s third law, :

:

FCE " $FEC

(cantaloupe – Earth interaction).

Next, in the cantaloupe – table interaction, the force on the cantaloupe from : : the table is FCT and, conversely, the force on the table from the cantaloupe is FTC (Fig. 5-11d). These forces are also a third-law force pair, and so :

:

FCT " $FTC

(cantaloupe – table interaction).

Checkpoint 5 Suppose that the cantaloupe and table of Fig. 5-11 are in an elevator cab that begins to : : accelerate upward. (a) Do the magnitudes of FTC and FCT increase, decrease, or stay the same? (b) Are those two forces still equal in magnitude and opposite in direction? : : (c) Do the magnitudes of FCE and FEC increase, decrease, or stay the same? (d) Are those two forces still equal in magnitude and opposite in direction?

(d)

So are these.

108

CHAPTE R 5 FORCE AN D M OTION—I

Applying Newton’s Laws The rest of this chapter consists of sample problems. You should pore over them, learning their procedures for attacking a problem. Especially important is knowing how to translate a sketch of a situation into a free-body diagram with appropriate axes, so that Newton’s laws can be applied.

Sample Problem 5.03 Block on table, block hanging Figure 5-12 shows a block S (the sliding block) with mass M " 3.3 kg. The block is free to move along a horizontal frictionless surface and connected, by a cord that wraps over a frictionless pulley, to a second block H (the hanging block), with mass m " 2.1 kg. The cord and pulley have negligible masses compared to the blocks (they are “massless”). The hanging block H falls as the sliding block S accelerates to the right. Find (a) the acceleration of block S, (b) the acceleration of block H, and (c) the tension in the cord. Q What is this problem all about? You are given two bodies — sliding block and hanging block — but must also consider Earth, which pulls on both bodies. (Without Earth, nothing would happen here.) A total of five forces act on the blocks, as shown in Fig. 5-13: 1. The cord pulls to the right on sliding block S with a force of magnitude T. 2. The cord pulls upward on hanging block H with a force of the same magnitude T. This upward force keeps block H from falling freely. 3. Earth pulls down on block S with the gravitational force : FgS, which has a magnitude equal to Mg. 4. Earth pulls down on block H with the gravitational force : FgH, which has a magnitude equal to mg. :

5. The table pushes up on block S with a normal force FN. There is another thing you should note. We assume that the cord does not stretch, so that if block H falls 1 mm in a Sliding block S M

Frictionless surface m

Hanging block H

FN

Block S T

M

FgS

T m FgH

Figure 5-13 The forces acting on the two blocks of Fig. 5-12.

certain time, block S moves 1 mm to the right in that same time. This means that the blocks move together and their accelerations have the same magnitude a. Q How do I classify this problem? Should it suggest a particular law of physics to me? Yes. Forces, masses, and accelerations are involved, and : they should suggest Newton’s second law of motion, F net " : . That is our starting key idea. ma Q If I apply Newton’s second law to this problem, to which body should I apply it? We focus on two bodies, the sliding block and the hanging block. Although they are extended objects (they are not points), we can still treat each block as a particle because every part of it moves in exactly the same way. A second key idea is to apply Newton’s second law separately to each block. Q What about the pulley? We cannot represent the pulley as a particle because different parts of it move in different ways. When we discuss rotation, we shall deal with pulleys in detail. Meanwhile, we eliminate the pulley from consideration by assuming its mass to be negligible compared with the masses of the two blocks. Its only function is to change the cord’s orientation. :

Figure 5-12 A block S of mass M is connected to a block H of mass m by a cord that wraps over a pulley.

Block H

: Q OK. Now how do I apply F net " ma to the sliding block? Represent block S as a particle of mass M and draw all the forces that act on it, as in Fig. 5-14a. This is the block’s free-body diagram. Next, draw a set of axes. It makes sense

109

5-3 APPLYI NG N EWTON’S L AWS

because block H accelerates in the negative direction of the y axis). We find T $ mg " $ma. (5-20)

y y FN M

T FgS

Now note that Eqs. 5-18 and 5-20 are simultaneous equations with the same two unknowns, T and a. Subtracting these equations eliminates T. Then solving for a yields

a m

x Sliding block S

T x FgH

a

Hanging block H

a"

m g. M#m

(5-21)

Substituting this result into Eq. 5-18 yields (a)

(b)

T"

Figure 5-14 (a) A free-body diagram for block S of Fig. 5-12. (b) A free-body diagram for block H of Fig. 5-12.

to draw the x axis parallel to the table, in the direction in which the block moves. Q Thanks, but you still haven’t told me how to apply : : to the sliding block. All you’ve done is explain F net " ma how to draw a free-body diagram. You are right, and here’s the third key idea: The : : expression F net " Ma is a vector equation, so we can write it as three component equations: Fnet, y " May

Fnet, z " Maz

(5-16)

in which Fnet,x, Fnet,y, and Fnet,z are the components of the net force along the three axes. Now we apply each component equation to its corresponding direction. Because block S does not accelerate vertically, Fnet, y " May becomes FN $ FgS " 0

or

FN " FgS.

(5-17)

Thus in the y direction, the magnitude of the normal force is equal to the magnitude of the gravitational force. No force acts in the z direction, which is perpendicular to the page. In the x direction, there is only one force component, which is T. Thus, Fnet, x " Max becomes T " Ma.

(5-18)

This equation contains two unknowns, T and a; so we cannot yet solve it. Recall, however, that we have not said anything about the hanging block. :

: Q I agree. How do I apply Fnet " ma to the hanging block? We apply it just as we did for block S: Draw a free-body : : diagram for block H, as in Fig. 5-14b.Then apply Fnet " ma in component form. This time, because the acceleration is along the y axis, we use the y part of Eq. 5-16 (Fnet, y " may) to write

T $ FgH " may.

(5-22)

Putting in the numbers gives, for these two quantities, a"

Fnet,x " Max

Mm g. M#m

(5-19)

We can now substitute mg for FgH and $a for ay (negative

2.1 kg m g" (9.8 m/s2) M#m 3.3 kg # 2.1 kg

" 3.8 m/s2 and T "

(Answer)

(3.3 kg)(2.1 kg) Mm g" (9.8 m/s2) M#m 3.3 kg # 2.1 kg

" 13 N.

(Answer)

Q The problem is now solved, right? That’s a fair question, but the problem is not really finished until we have examined the results to see whether they make sense. (If you made these calculations on the job, wouldn’t you want to see whether they made sense before you turned them in?) Look first at Eq. 5-21. Note that it is dimensionally correct and that the acceleration a will always be less than g (because of the cord, the hanging block is not in free fall). Look now at Eq. 5-22, which we can rewrite in the form T"

M mg. M#m

(5-23)

In this form, it is easier to see that this equation is also dimensionally correct, because both T and mg have dimensions of forces. Equation 5-23 also lets us see that the tension in the cord is always less than mg, and thus is always less than the gravitational force on the hanging block.That is a comforting thought because, if T were greater than mg, the hanging block would accelerate upward. We can also check the results by studying special cases, in which we can guess what the answers must be. A simple example is to put g " 0, as if the experiment were carried out in interstellar space. We know that in that case, the blocks would not move from rest, there would be no forces on the ends of the cord, and so there would be no tension in the cord. Do the formulas predict this? Yes, they do. If you put g " 0 in Eqs. 5-21 and 5-22, you find a " 0 and T " 0. Two more special cases you might try are M " 0 and m : 5.

Additional examples, video, and practice available at WileyPLUS

110

CHAPTE R 5 FORCE AN D M OTION—I

Sample Problem 5.04 Cord accelerates box up a ramp Many students consider problems involving ramps (inclined planes) to be especially hard. The difficulty is probably visual because we work with (a) a tilted coordinate system and (b) the components of the gravitational force, not the full force. Here is a typical example with all the tilting and angles explained. (In WileyPLUS, the figure is available as an animation with voiceover.) In spite of the tilt, the key idea is to apply Newton’s second law to the axis along which the motion occurs. In Fig. 5-15a, a cord pulls a box of sea biscuits up along a frictionless plane inclined at angle u " 30.03. The box has mass m " 5.00 kg, and the force from the cord has magnitude T " 25.0 N. What is the box’s acceleration a along the inclined plane? KEY IDEA The acceleration along the plane is set by the force components along the plane (not by force components perpendi-

A

cular to the plane), as expressed by Newton’s second law (Eq. 5-1). Calculations: We need to write Newton’s second law for motion along an axis. Because the box moves along the inclined plane, placing an x axis along the plane seems reasonable (Fig. 5-15b). (There is nothing wrong with using our usual coordinate system, but the expressions for components would be a lot messier because of the misalignment of the x axis with the motion.) After choosing a coordinate system, we draw a freebody diagram with a dot representing the box (Fig. 5-15b). Then we draw all the vectors for the forces acting on the box, with the tails of the vectors anchored on the dot. (Drawing the vectors willy-nilly on the diagram can easily lead to errors, especially on exams, so always anchor the tails.) : Force T from the cord is up the plane and has magni: tude T " 25.0 N. The gravitational force Fg is downward (of

y

Figure 5-15 (a) A box is pulled up a plane by a cord. (b) The three forces acting on the : box: the cord’s force T, the gravitational force : : Fg, and the normal force FN. (c)–(i) Finding the force components along the plane and perpendicular to it. In WileyPLUS, this figure is available as an animation with voiceover.

Normal force FN

Cord

T

The box accelerates.

Cord's pull

θ

90° − θ

θ

(c)

(d)

mg

θ mg cos θ

Parallel component of Fg

(e)

(f )

y

The net of these forces determines the acceleration.

These forces merely balance.

x T

FN

x

mg sin θ

mg sin θ

( g)

mg cos θ

( h)

θ

Hypotenuse

Fg

θ

Gravitational force

(b)

Perpendicular component of Fg

This is also. 90° − θ θ

Fg

θ (a)

This is a right triangle.

x

(i)

Adjacent leg (use cos θ )

Opposite leg (use sin θ )

111

5-3 APPLYI NG N EWTON’S L AWS

course) and has magnitude mg " (5.00 kg)(9.80 m/s2) " 49.0 N. That direction means that only a component of the force is along the plane, and only that component (not the full force) affects the box’s acceleration along the plane. Thus, before we can write Newton’s second law for motion along the x axis, we need to find an expression for that important component. Figures 5-15c to h indicate the steps that lead to the expression. We start with the given angle of the plane and work our way to a triangle of the force components (they are the legs of the triangle and the full force is the hypotenuse). Figure 5-15c shows that the angle between the : ramp and Fg is 903 $ u. (Do:you see a right triangle there?) Next, Figs. 5-15d to f show Fg and its components: One component is parallel to the plane (that is the one we want) and the other is perpendicular to the plane. Because the perpendicular component is perpendicular, : the angle between it and Fg must be u (Fig. 5-15d). The component we want is the far leg of the component right triangle. The magnitude of the hypotenuse is mg (the magnitude of the gravitational force). Thus, the component we want has magnitude mg sin u (Fig. 5-15g). We have one more force to consider, the normal force : FN shown in Fig. 5-15b. However, it is perpendicular to the

plane and thus cannot affect the motion along the plane. (It has no component along the plane to accelerate the box.) We are now ready to write Newton’s second law for motion along the tilted x axis: Fnet,x " max. The component ax is the only component of the acceleration (the box is not leaping up from the plane, which would be strange, or descending into the plane, which would be even stranger). So, let’s simply write a for the acceleration along the : plane. Because T is in the positive x direction and the component mg sin u is in the negative x direction, we next write T $ mg sin u " ma. Substituting data and solving for a, we find a " 0.100 m/s2.

When F1 is horizontal, the acceleration is 3.0 m/s2. F1 θ

ax (m/s2)

3

x

(a)

2 1 0 0°

θ (b)

KEY IDEAS (1) The horizontal acceleration ax depends on the net horizontal force Fnet, x, as given by Newton’s second law. (2) The net horizontal force is the sum of the horizontal compo: : nents of forces F1 and F2 . : F2 : F1

Calculations: The x component of is F2 because the vector is horizontal. The x component of is F1 cos 1. Using these expressions and a mass m of 4.00 kg, we can write Newton’s : second law ( F net " m : a ) for motion along the x axis as F1 cos u # F2 " 4.00ax.

(5-25)

From this equation we see that when angle u " 903, F1 cos u is zero and F2 " 4.00ax. From the graph we see that the

(Answer)

The result is positive, indicating that the box accelerates up the inclined plane, in the positive direction of the tilted x axis. If : we decreased the magnitude of T enough to make a " 0, the box would move up the plane at constant speed.And if we de: crease the magnitude of T even more, the acceleration would be negative in spite of the cord’s pull.

Sample Problem 5.05 Reading a force graph Here is an example of where you must dig information out of a graph, not just read off a number. In Fig. 5-16a, two forces are applied to a 4.00 kg block on a frictionless floor, : but only force F1 is indicated. That force has a fixed magnitude but can be applied at an adjustable angle 1 to the posi: tive direction of the x axis. Force F2 is horizontal and fixed in both magnitude and angle. Figure 5-16b gives the horizontal acceleration ax of the block for any given value of u from 03 to 903. What is the value of ax for u " 1803?

(5-24)

90°

When F1 is vertical, the acceleration is 0.50 m/s2.

Figure 5-16 (a) One of the two forces applied to a block is shown. Its angle u can be varied. (b) The block’s acceleration component ax versus u.

corresponding acceleration is 0.50 m/s2. Thus, F2 " 2.00 N : and F2 must be in the positive direction of the x axis. From Eq. 5-25, we find that when u " 03, F1 cos 03 # 2.00 " 4.00ax.

(5-26)

From the graph we see that the corresponding acceleration is 3.0 m/s2. From Eq. 5-26, we then find that F1 " 10 N. Substituting F1 " 10 N, F2 " 2.00 N, and u " 1803 into Eq. 5-25 leads to ax " $2.00 m/s2. (Answer)

Additional examples, video, and practice available at WileyPLUS

112

CHAPTE R 5 FORCE AN D M OTION—I

Sample Problem 5.06 Forces within an elevator cab Although people would surely avoid getting into the elevator with you, suppose that you weigh yourself while on an elevator that is moving. Would you weigh more than, less than, or the same as when the scale is on a stationary floor? In Fig. 5-17a, a passenger of mass m " 72.2 kg stands on a platform scale in an elevator cab. We are concerned with the scale readings when the cab is stationary and when it is moving up or down. (a) Find a general solution for the scale reading, whatever the vertical motion of the cab. KEY IDEAS (1) The reading is equal to the magnitude of the normal force : FN on the passenger from the scale. The only other force act: ing on the passenger is the gravitational force Fg, as shown in the free-body diagram of Fig. 5-17b. (2) We can relate the forces on the passenger to his acceleration : a by using : : Newton’s second law (F net " ma ). However, recall that we can use this law only in an inertial frame. If the cab accelerates, then it is not an inertial frame. So we choose the ground to be our inertial frame and make any measure of the passenger’s acceleration relative to it. Calculations: Because the two forces on the passenger and his acceleration are all directed vertically, along the y axis in Fig. 5-17b, we can use Newton’s second law written for y components (Fnet, y " may) to get FN $ Fg " ma or

FN " Fg # ma.

This tells us that the scale reading, which is equal to normal force magnitude FN, depends on the vertical acceleration. Substituting mg for Fg gives us FN " m(g # a)

for any choice of acceleration a. If the acceleration is upward, a is positive; if it is downward, a is negative. (b) What does the scale read if the cab is stationary or moving upward at a constant 0.50 m/s? KEY IDEA For any constant velocity (zero or otherwise), the acceleration a of the passenger is zero. Calculation: Substituting this and other known values into Eq. 5-28, we find FN " (72.2 kg)(9.8 m/s2 # 0) " 708 N.

(5-27)

FN

Passenger

(b)

(Answer)

This is the weight of the passenger and is equal to the magnitude Fg of the gravitational force on him. (c) What does the scale read if the cab accelerates upward at 3.20 m/s2 and downward at 3.20 m/s2? Calculations: For a " 3.20 m/s2, Eq. 5-28 gives FN " (72.2 kg)(9.8 m/s2 # 3.20 m/s2) " 939 N, (Answer) FN " (72.2 kg)(9.8 m/s2 $ 3.20 m/s2) " 477 N.

(a)

(5-28)

and for a " $3.20 m/s2, it gives

y

Fg

(Answer)

These forces compete. Their net force causes a vertical acceleration.

Figure 5-17 (a) A passenger stands on a platform scale that indicates either his weight or his apparent weight. (b) The free-body : diagram for the passenger, showing the normal force F N on him : from the scale and the gravitational force Fg.

(Answer)

For an upward acceleration (either the cab’s upward speed is increasing or its downward speed is decreasing), the scale reading is greater than the passenger’s weight. That reading is a measurement of an apparent weight, because it is made in a noninertial frame. For a downward acceleration (either decreasing upward speed or increasing downward speed), the scale reading is less than the passenger’s weight. (d) During the upward acceleration in part (c), what is the magnitude Fnet of the net force on the passenger, and what is the magnitude ap,cab of his acceleration as measured in the : : frame of the cab? Does Fnet " ma p,cab ? Calculation: The magnitude Fg of the gravitational force on the passenger does not depend on the motion of the passenger or the cab; so, from part (b), Fg is 708 N. From part (c), the magnitude FN of the normal force on the passenger during

113

5-3 APPLYI NG N EWTON’S L AWS

the upward acceleration is the 939 N reading on the scale.Thus, the net force on the passenger is Fnet " FN $ Fg " 939 N $ 708 N " 231 N,

(Answer)

during the upward acceleration. However, his acceleration ap,cab relative to the frame of the cab is zero. Thus, in the noninertial frame of the accelerating cab, Fnet is not equal to map,cab, and Newton’s second law does not hold.

Sample Problem 5.07 Acceleration of block pushing on block Some homework problems involve objects that move together, because they are either shoved together or tied together. Here is an example in which you apply Newton’s second law to the composite of two blocks and then to the individual blocks. : In Fig. 5-18a, a constant horizontal force Fapp of magnitude 20 N is applied to block A of mass mA " 4.0 kg, which pushes against block B of mass mB " 6.0 kg. The blocks slide over a frictionless surface, along an x axis. (a) What is the acceleration of the blocks? :

Serious Error: Because force Fapp is applied directly to block A, we use Newton’s second law to relate that force to the acceleration : a of block A. Because the motion is along the x axis, we use that law for x components (Fnet, x " max), writing it as Fapp " mAa. :

However, this is seriously wrong because Fapp is not the only horizontal force acting on block A. There is also the : force FAB from block B (Fig. 5-18b).

B Fapp

A

x

This force causes the acceleration of the full two-block system.

:

Dead-End Solution: Let us now include force FAB by writing, again for the x axis, Fapp $ FAB " mAa. :

(We use the minus sign to include the direction of FAB .) Because FAB is a second unknown, we cannot solve this equation for a. Successful Solution: Because of the direction in which force : Fapp is applied, the two blocks form a rigidly connected system. We can relate the net force on the system to the acceleration of the system with Newton’s second law. Here, once again for the x axis, we can write that law as Fapp " (mA # mB)a, :

where now we properly apply Fapp to the system with total mass mA # mB. Solving for a and substituting known values, we find a"

Fapp mA # mB

"

20 N 4.0 kg # 6.0 kg

" 2.0 m/s2. (Answer)

Thus, the acceleration of the system and of each block is in the positive direction of the x axis and has the magnitude 2.0 m/s2. :

(b) What is the (horizontal) force FBA on block B from block A (Fig. 5-18c)?

(a)

KEY IDEA Fapp

A

FAB

x

(b)

B FBA

x (c)

These are the two forces acting on just block A. Their net force causes its acceleration.

This is the only force causing the acceleration of block B. :

Figure 5-18 (a) A constant horizontal force F app is applied to block A, which pushes against block B. (b) Two horizontal forces act on block A. (c) Only one horizontal force acts on block B.

We can relate the net force on block B to the block’s acceleration with Newton’s second law. Calculation: Here we can write that law, still for components along the x axis, as FBA " mBa, which, with known values, gives FBA " (6.0 kg)(2.0 m/s2) " 12 N. : FBA

(Answer)

Thus, force is in the positive direction of the x axis and has a magnitude of 12 N.

Additional examples, video, and practice available at WileyPLUS

114

CHAPTE R 5 FORCE AN D M OTION—I

Review & Summary Newtonian Mechanics The velocity of an object can change (the object can accelerate) when the object is acted on by one or more forces (pushes or pulls) from other objects. Newtonian mechanics relates accelerations and forces.

A free-body diagram is a stripped-down diagram in which only one body is considered.That body is represented by either a sketch or a dot. The external forces on the body are drawn, and a coordinate system is superimposed, oriented so as to simplify the solution.

Force Forces are vector quantities. Their magnitudes are de-

Some Particular Forces A gravitational force Fg on a body

fined in terms of the acceleration they would give the standard kilogram. A force that accelerates that standard body by exactly 1 m/s2 is defined to have a magnitude of 1 N. The direction of a force is the direction of the acceleration it causes. Forces are combined according to the rules of vector algebra. The net force on a body is the vector sum of all the forces acting on the body.

Newton’s First Law If there is no net force on a body, the body remains at rest if it is initially at rest or moves in a straight line at constant speed if it is in motion.

Inertial Reference Frames Reference frames in which Newtonian mechanics holds are called inertial reference frames or inertial frames. Reference frames in which Newtonian mechanics does not hold are called noninertial reference frames or noninertial frames.

Mass The mass of a body is the characteristic of that body that relates the body’s acceleration to the net force causing the acceleration. Masses are scalar quantities. :

Newton’s Second Law The net force Fnet on a body with :

mass m is related to the body’s acceleration a by :

: , Fnet " ma

(5-1)

which may be written in the component versions Fnet, x " max

Fnet, y " may

and

Fnet, z " maz.

(5-2)

The second law indicates that in SI units

:

is a pull by another body. In most situations in this book, the other body is Earth or some other astronomical body. For Earth, the force is directed down toward the ground, which is assumed to be : an inertial frame. With that assumption, the magnitude of Fg is

where m is the body’s mass and g is the magnitude of the free-fall acceleration. The weight W of a body is the magnitude of the upward force needed to balance the gravitational force on the body. A body’s weight is related to the body’s mass by

1 N " 1 kg !m/s .

(5-12)

W " mg. : FN

A normal force is the force on a body from a surface against which the body presses. The normal force is always perpendicular to the surface. : A frictional force f is the force on a body when the body slides or attempts to slide along a surface. The force is always parallel to the surface and directed so as to oppose the sliding. On a frictionless surface, the frictional force is negligible. When a cord is under tension, each end of the cord pulls on a body. The pull is directed along the cord, away from the point of attachment to the body. For a massless cord (a cord with negligible mass), the pulls at both ends of the cord have the same magnitude T, even if the cord runs around a massless, frictionless pulley (a pulley with negligible mass and negligible friction on its axle to oppose its rotation). :

Newton’s Third Law If a force FBC acts on body B due to body C, then there is a force

2

(5-8)

Fg " mg,

(5-3)

: FCB

: FBC

on body C due to body B: :

" $FCB.

Questions 1 Figure 5-19 gives the free-body diagram for four situations in which an object is pulled by several forces across a frictionless floor, as seen from overhead. In which situations does the acceleration : a of the object have (a) an x component and (b) a y comy

a by naming ponent? (c) In each situation, give the direction of : either a quadrant or a direction along an axis. (Don’t reach for the calculator because this can be answered with a few mental calculations.)

y 7N

6N

3N 5N

2N 4N (1)

y

x

3N

2N

6N 5N

x 4N

2N 4N (2)

y

4N

3N

2N x

3N

5N 5N

(3)

Figure 5-19 Question 1.

3N

(4)

4N

x

115

QU ESTIONS

2

Two horizontal forces, : F1

" (3 N)iˆ $ (4 N)jˆ

and

pull a banana split across a frictionless lunch counter. Without using a calculator, determine which of the vectors in the free-body diagram of : Fig. 5-20 best represent (a) F 1 and : (b) F 2. What is the net-force component along (c) the x axis and (d) the y axis? Into which quadrants do (e) the net-force vector and (f) the split’s acceleration vector point? :

:

: F2

" $(1 N)iˆ $ (2 N)jˆ y 1

4 2

3

x

7

6

8

3 In Fig. 5-21, forces F 1 and F 2 are applied to a lunchbox as it slides at constant velocity over a frictionless floor. We are to decrease angle u without changing the : magnitude of F 1. For constant velocity, should we increase, decrease, : or maintain the magnitude of F 2?

5

Figure 5-20 Question 2. F1

θ

F2

7 July 17, 1981, Kansas City: The newly opened Hyatt Regency is packed with people listening and dancing to a band playing favorites from the 1940s. Many of the people are crowded onto the walkways that hang like bridges across the wide atrium. Suddenly two of the walkways collapse, falling onto the merrymakers on the main floor. The walkways were suspended one above another on vertical rods and held in place by nuts threaded onto the rods. In the original design, only two long rods were to be used, each extending through all three walkways (Fig. 5-24a). If each walkway and the merrymakers on it have a combined mass of M, what is the total mass supported by the threads and two nuts on (a) the lowest walkway and (b) the highest walkway? Apparently someone responsible for the actual construction realized that threading nuts on a rod is impossible except at the ends, so the design was changed: Instead, six rods were used, each connecting two walkways (Fig. 5-24b). What now is the total mass supported by the threads and two nuts on (c) the lowest walkway, (d) the upper side of the highest walkway, and (e) the lower side of the highest walkway? It was this design that failed on that tragic night—a simple engineering error.

:

4 At time t " 0, constant F begins Figure 5-21 Question 3. to act on a rock moving through deep space in the +x direction. (a) For time t . 0, which are possible functions x(t) for the rock’s position: (1) x " 4t $ 3, (2) x " $4t 2 # 6t $ 3, (3) x " 4t 2 # 6t $ 3? (b) : For which function is F directed opposite the rock’s initial direction of motion?

Rods

Nuts

5 Figure 5-22 shows overhead views of four situations in which forces act on a block that lies on a frictionless floor. If the force magnitudes are chosen properly, in which situations is it possible that the block is (a) stationary and (b) moving with a constant velocity? F1 F2

(1)

F1

(2)

F1

F3

(3)

F2

F1

F2

(4)

Walkways

(a)

(b)

Figure 5-24 Question 7. 8 Figure 5-25 gives three graphs of velocity component vx(t) and three graphs of velocity component vy(t). The graphs are not to scale. Which vx(t) graph and which vy(t) graph best correspond to each of the four situations in Question 1 and Fig. 5-19? vx

vx

vx

F2 F3

t

t

t

Figure 5-22 Question 5. 6 Figure 5-23 shows the same breadbox in four situations where horizontal forces are applied. Rank the situations according to the magnitude of the box’s acceleration, greatest first.

(a)

(b)

vy 3N

6N

58 N

(a) 13 N

vy

t

20 N (d )

Figure 5-23 Question 6.

t

t

25 N

43 N

(c )

vy

60 N (b)

15 N

(c)

(d)

(e)

Figure 5-25 Question 8.

(f )

116

CHAPTE R 5 FORCE AN D M OTION—I :

9 Figure 5-26 shows a train of four blocks being pulled across a : frictionless floor by force F . What total mass is accelerated to the : right by (a) force F , (b) cord 3, and (c) cord 1? (d) Rank the blocks according to their accelerations, greatest first. (e) Rank the cords according to their tension, greatest first.

10 kg

Cord 1

3 kg

Cord 2

5 kg

Cord 3

2 kg

F 32 on block 3 from block 2? (d) Rank the blocks according to : : their acceleration magnitudes, greatest first. (e) Rank forces F , F 21, : and F 32 according to magnitude, greatest first. :

11 A vertical force F is applied to a block of mass m that lies on : a floor. What happens to the magnitude of the normal force F N on the block from the floor as magnitude F is increased from zero if : force F is (a) downward and (b) upward?

F

12 Figure 5-28 shows four choices for the direction of a force of magnitude F to be applied to a block on an inclined plane. The directions b are either horizontal or vertical. (For choice b, the force is not a enough to lift the block off the c plane.) Rank the choices according to the magnitude of the normal 30° d force acting on the block from the plane, greatest first. Figure 5-28 Question 12.

Figure 5-26 Question 9. 10 Figure 5-27 shows three blocks being pushed across a frictionless : floor by horizontal force F . What total mass is accelerated to the right : : by (a) force F , (b) force F 21 on block 2 from block 1, and (c) force

10 kg

5 kg F 1

2 kg 2

3

Figure 5-27 Question 10.

Problems Tutoring problem available (at instructor’s discretion) in WileyPLUS and WebAssign SSM • – •••

Worked-out solution available in Student Solutions Manual

WWW Worked-out solution is at

Number of dots indicates level of problem difficulty

ILW

Interactive solution is at

http://www.wiley.com/college/halliday

Additional information available in The Flying Circus of Physics and at flyingcircusofphysics.com

Module 5-1 Newton’s First and Second Laws •1 Only two horizontal forces act on a 3.0 kg body that can move over a frictionless floor. One force is 9.0 N, acting due east, and the other is 8.0 N, acting 623 north of west. What is the magnitude of the body’s acceleration? •2 Two horizontal forces act on a 2.0 kg chopping block that can slide over a frictionless kitchen counter, which lies in an xy plane. : One force is F 1 " (3.0 N)iˆ # (4.0 N)jˆ . Find the acceleration of the chopping block in unit-vector notation when the other force is : : (a) F 2 " ($3.0 N)iˆ # ($4.0 N)jˆ , (b) F 2 " ($3.0 N)iˆ # (4.0 N)jˆ , : and (c) F 2 " (3.0 N)iˆ # ($4.0 N)jˆ . •3 If the 1 kg standard body has an acceleration of 2.00 m/s2 at 20.03 to the positive direction of an x axis, what are (a) the x component and (b) the y component of the net force acting on the body, and (c) what is the net force in unit-vector notation? ••4 While two forces act on it, a y particle is to move at the constant velocity : v " (3 m/s)iˆ $ (4 m/s)jˆ . One : of the forces is F1 " (2 N)iˆ # ˆ F1 ($6 N)j . What is the other force? θ1 ••5 Three astronauts, propelled x θ3 by jet backpacks, push and guide a F2 120 kg asteroid toward a processing F3 dock, exerting the forces shown in Fig. 5-29, with F1 " 32 N, F2 " 55 N, Figure 5-29 Problem 5. F3 " 41 N, u1 " 303, and u3 " 603. What is the asteroid’s acceleration (a) in unit-vector notation and as (b) a magnitude and (c) a direction relative to the positive direction of the x axis?

••6 In a two-dimensional tug-ofwar, Alex, Betty, and Charles pull horizontally on an automobile tire at the angles shown in the overhead view of Fig. 5-30. The tire remains stationary in spite of the three pulls. : Alex pulls with force FA of magnitude 220 N, and Charles pulls with : force FC of magnitude 170 N. Note : that the direction of FC is not given. What is the magnitude of Betty’s : force FB? ••7 SSM There are two forces on the 2.00 kg box in the overhead view of Fig. 5-31, but only one is shown. For F1 " 20.0 N,a " 12.0 m/s2,and u " 30.03, find the second force (a) in unit-vector notation and as (b) a magnitude and (c) an angle relative to the positive direction of the x axis.

Alex Charles

Betty

137°

Figure 5-30 Problem 6. y F1

θ

••8 A 2.00 kg object is subjected to a three forces that give it an acceleration : a " $(8.00 m/s2)iˆ # (6.00 m/s2)jˆ. If Figure 5-31 Problem 7. two of the three forces are : : F1 " (30.0 N)iˆ # (16.0 N)jˆ and F2 " $(12.0 N)iˆ # (8.00 N)jˆ, find the third force. ••9 A 0.340 kg particle moves in an xy plane according to x(t) " $15.00 # 2.00t $ 4.00t3 and y(t) " 25.00 # 7.00t $ 9.00t2, with x and y in meters and t in seconds. At t " 0.700 s, what are

x

117

PROB LE M S

reading on the scale? (This is the way by a deli owner who was once a physics major.)

••10 A 0.150 kg particle moves along an x axis according to x(t) " $13.00 # 2.00t # 4.00t2 $ 3.00t3, with x in meters and t in seconds. In unit-vector notation, what is the net force acting on the particle at t " 3.40 s?

Spring scale

I

GE

SA

M

O

SA

I

A

GE

Spring scale N

GE

I

I

SA

A

SA

LA

O

N

A

LA

M

(a)

N

O

GE

M

LA

Figure 5-34 Problem 15.

4 2 0

(b)

(c)

vx (m/s)

–2

LA

:

•••12 Two horizontal forces F 1 and F 2 act on a 4.0 kg disk that slides over frictionless ice, on which an xy coordinate system is laid : out. Force F 1 is in the positive direction of the x axis and has a mag: nitude of 7.0 N. Force F 2 has a magnitude of 9.0 N. Figure 5-32 gives the x component vx of the velocity of the disk as a function of time t during the sliding. What is the angle between the constant di: : rections of forces F 1 and F 2?

N

A

:

Spring scale

O

••11 A 2.0 kg particle moves along an x axis, being propelled by a variable force directed along that axis. Its position is given by x " 3.0 m # (4.0 m/s)t # ct 2 $ (2.0 m/s3)t 3, with x in meters and t in seconds. The factor c is a constant. At t " 3.0 s, the force on the particle has a magnitude of 36 N and is in the negative direction of the axis. What is c?

M

(a) the magnitude and (b) the angle (relative to the positive direction of the x axis) of the net force on the particle, and (c) what is the angle of the particle’s direction of travel?

1

2

3

••16 Some insects can walk below Rod a thin rod (such as a twig) by hang- Leg Tibia ing from it. Suppose that such an in- joint θ sect has mass m and hangs from a horizontal rod as shown in Fig. 5-35, with angle u " 403. Its six legs are all under the same tension, and the leg Figure 5-35 Problem 16. sections nearest the body are horizontal. (a) What is the ratio of the tension in each tibia (forepart of a leg) to the insect’s weight? (b) If the insect straightens out its legs somewhat, does the tension in each tibia increase, decrease, or stay the same?

t (s)

–4

Figure 5-32 Problem 12. Module 5-2 Some Particular Forces •13 Figure 5-33 shows an arrangement in which four disks are suspended by cords. The longer, top cord loops over a frictionless pulley and pulls with a force of magnitude 98 N on the wall to which it is attached.The tensions in the three shorter cords are T1 " 58.8 N, T2 " 49.0 N, and T3 " 9.8 N. What are the masses of (a) disk A, (b) disk B, (c) disk C, and (d) disk D? •14 A block with a weight of 3.0 N is at rest on a horizontal surface. A 1.0 N upward force is applied to the block by means of an attached vertical string. What are the (a) magnitude and (b) direction of the force of the block on the horizontal surface?

A T1 B T2 C T3 D

Figure 5-33 Problem 13.

•15 SSM (a) An 11.0 kg salami is supported by a cord that runs to a spring scale, which is supported by a cord hung from the ceiling (Fig. 5-34a). What is the reading on the scale, which is marked in SI weight units? (This is a way to measure weight by a deli owner.) (b) In Fig. 5-34b the salami is supported by a cord that runs around a pulley and to a scale. The opposite end of the scale is attached by a cord to a wall. What is the reading on the scale? (This is the way by a physics major.) (c) In Fig. 5-34c the wall has been replaced with a second 11.0 kg salami, and the assembly is stationary. What is the

Module 5-3 Applying Newton’s Laws •17 SSM WWW In Fig. 5-36, let the mass of the block be 8.5 kg and the angle 1 be 303. Find (a) the tension in the cord and (b) the normal force acting on the block. (c) If the cord is cut, find the magnitude of the resulting acceleration of the block.

m

less

ion

t Fric

θ

•18 In April 1974, John Figure 5-36 Problem 17. Massis of Belgium managed to move two passenger railroad cars. He did so by clamping his teeth down on a bit that was attached to the cars with a rope and then leaning backward while pressing his feet against the railway ties. The cars together weighed 700 kN (about 80 tons). Assume that he pulled with a constant force that was 2.5 times his body weight, at an upward angle u of 303 from the horizontal. His mass was 80 kg, and he moved the cars by 1.0 m. Neglecting any retarding force from the wheel rotation, find the speed of the cars at the end of the pull.

118

CHAPTE R 5 FORCE AN D M OTION—I

•19 SSM A 500 kg rocket sled can be accelerated at a constant rate from rest to 1600 km/h in 1.8 s. What is the magnitude of the required net force? •20 A car traveling at 53 km/h hits a bridge abutment. A passenger in the car moves forward a distance of 65 cm (with respect to the road) while being brought to rest by an inflated air bag. What magnitude of force (assumed constant) acts on the passenger’s upper torso, which has a mass of 41 kg? :

•21 A constant horizontal force Fa pushes a 2.00 kg FedEx package across a frictionless floor on which an xy coordinate system has been drawn. Figure 5-37 gives the package’s x and y velocity components versus time t. What are the (a) magnitude and (b) direc: tion of Fa? vx (m/s)

•24 There are two horizontal F1 x forces on the 2.0 kg box in the overhead view of Fig. 5-38 but only one (of magnitude F1 " 20 N) is shown. Figure 5-38 Problem 24. The box moves along the x axis. For each of the following values for the acceleration ax of the box, find the second force in unit-vector notation: (a) 10 m/s2, (b) 20 m/s2, (c) 0, (d) $10 m/s2, and (e) $20 m/s2. •25 Sunjamming. A “sun yacht” is a spacecraft with a large sail that is pushed by sunlight. Although such a push is tiny in everyday circumstances, it can be large enough to send the spacecraft outward from the Sun on a cost-free but slow trip. Suppose that the spacecraft has a mass of 900 kg and receives a push of 20 N. (a) What is the magnitude of the resulting acceleration? If the craft starts from rest, (b) how far will it travel in 1 day and (c) how fast will it then be moving? •26 The tension at which a fishing line snaps is commonly called the line’s “strength.”What minimum strength is needed for a line that is to stop a salmon of weight 85 N in 11 cm if the fish is initially drifting at 2.8 m/s? Assume a constant deceleration.

10

5

0

1

2

3

t (s)

vy (m/s) 0

t (s) 0

1

2

3

–5

–10

Figure 5-37 Problem 21. •22 A customer sits in an amusement park ride in which the compartment is to be pulled downward in the negative direction of a y axis with an acceleration magnitude of 1.24g, with g " 9.80 m/s2. A 0.567 g coin rests on the customer’s knee. Once the motion begins and in unit-vector notation, what is the coin’s acceleration relative to (a) the ground and (b) the customer? (c) How long does the coin take to reach the compartment ceiling, 2.20 m above the knee? In unit-vector notation, what are (d) the actual force on the coin and (e) the apparent force according to the customer’s measure of the coin’s acceleration? •23 Tarzan, who weighs 820 N, swings from a cliff at the end of a 20.0 m vine that hangs from a high tree limb and initially makes an angle of 22.03 with the vertical. Assume that an x axis extends horizontally away from the cliff edge and a y axis extends upward. Immediately after Tarzan steps off the cliff, the tension in the vine is 760 N. Just then, what are (a) the force on him from the vine in unit-vector notation and the net force on him (b) in unit-vector notation and as (c) a magnitude and (d) an angle relative to the positive direction of the x axis? What are the (e) magnitude and (f) angle of Tarzan’s acceleration just then?

•27 SSM An electron with a speed of 1.2 ' 10 7 m/s moves horizontally into a region where a constant vertical force of 4.5 ' 10$16 N acts on it. The mass of the electron is 9.11 ' 10$31 kg. Determine the vertical distance the electron is deflected during the time it has moved 30 mm horizontally. •28 A car that weighs 1.30 ' 10 4 N is initially moving at 40 km/h when the brakes are applied and the car is brought to a stop in 15 m. Assuming the force that stops the car is constant, find (a) the magnitude of that force and (b) the time required for the change in speed. If the initial speed is doubled, and the car experiences the same force during the braking, by what factors are (c) the stopping distance and (d) the stopping time multiplied? (There could be a lesson here about the danger of driving at high speeds.) •29 A firefighter who weighs 712 N slides down a vertical pole with an acceleration of 3.00 m/s2, directed downward. What are the (a) magnitude and (b) direction (up or down) of the vertical force on the firefighter from the pole and the (c) magnitude and (d) direction of the vertical force on the pole from the firefighter? •30 The high-speed winds around a tornado can drive projectiles into trees, building walls, and even metal traffic signs. In a laboratory simulation, a standard wood toothpick was shot by pneumatic gun into an oak branch. The toothpick’s mass was 0.13 g, its speed before entering the branch was 220 m/s, and its penetration depth was 15 mm. If its speed was decreased at a uniform rate, what was the magnitude of the force of the branch on the toothpick? ••31 SSM WWW A block is projected up a frictionless inclined plane with initial speed v0 " 3.50 y m/s. The angle of incline is F1 1 " 32.03. (a) How far up the plane does the block go? (b) How long θ1 x does it take to get there? (c) What is its speed when it gets back to the θ2 F2 bottom? ••32 Figure 5-39 shows an overhead view of a 0.0250 kg lemon half and

Figure 5-39 Problem 32.

119

PROB LE M S

two of the three horizontal forces that act on it as it is on a frictionless : table. Force F 1 has a magnitude of 6.00 N and is at 11 " 30.03. Force : F 2 has a magnitude of 7.00 N and is at 12 " 30.03. In unit-vector notation, what is the third force if the lemon half (a) is stationary, (b) has the constant velocity : v " (13.0iˆ $ 14.0jˆ ) m/s, and (c) has the varying velocity : v " (13.0tiˆ $ 14.0t jˆ ) m/s2, where t is time? ••33 An elevator cab and its load have a combined mass of 1600 kg. Find the tension in the supporting cable when the cab, originally moving downward at 12 m/s, is brought to rest with constant acceleration in a distance of 42 m. m

••34 In Fig. 5-40, a crate of mass m " 100 kg is pushed at constant speed up a frictionless ramp (1 " 30.03) by a horizontal force : : F . What are the magnitudes of (a) F and (b) the force on the crate from the ramp?

F

θ

Figure 5-40 Problem 34. ••35 The velocity of a 3.00 kg parti: 2 cle is given by v " (8.00tiˆ + 3.00t jˆ ) m/s, with time t in seconds. At the instant the net force on the particle has a magnitude of 35.0 N, what are the direction (relative to the positive direction of the x axis) of (a) the net force and (b) the particle’s direction of travel? ••36 Holding on to a towrope moving parallel to a frictionless ski slope, a 50 kg skier is pulled up the slope, which is at an angle of 8.03 with the horizontal. What is the magnitude Frope of the force on the skier from the rope when (a) the magnitude v of the skier’s velocity is constant at 2.0 m/s and (b) v " 2.0 m/s as v increases at a rate of 0.10 m/s2? ••37 A 40 kg girl and an 8.4 kg sled are on the frictionless ice of a frozen lake, 15 m apart but connected by a rope of negligible mass. The girl exerts a horizontal 5.2 N force on the rope. What are the acceleration magnitudes of (a) the sled and (b) the girl? (c) How far from the girl’s initial position do they meet? ••38 A 40 kg skier skis directly down a frictionless slope angled at 103 to the horizontal. Assume the skier moves in the negative direction of an x axis along the slope. A wind force with component Fx acts on the skier. What is Fx if the magnitude of the skier’s velocity is (a) constant, (b) increasing at a rate of 1.0 m/s2, and (c) increasing at a rate of 2.0 m/s2? ••39 ILW A sphere of mass 3.0 ' 10$4 kg is suspended from a cord. A steady horizontal breeze pushes the sphere so that the cord makes a constant angle of 373 with the vertical. Find (a) the push magnitude and (b) the tension in the cord. ••40 A dated box of dates, of mass 5.00 kg, is sent sliding up a frictionless ramp at an angle of 1 to the horizontal. Figure 5-41 gives, vx (m/s) 4 2

0

1

2

–2 –4

Figure 5-41 Problem 40.

3

t (s)

as a function of time t, the component vx of the box’s velocity along an x axis that extends directly up the ramp. What is the magnitude of the normal force on the box from the ramp? ••41 Using a rope that will snap if the tension in it exceeds 387 N, you need to lower a bundle of old roofing material weighing 449 N from a point 6.1 m above the ground. Obviously if you hang the bundle on the rope, it will snap. So, you allow the bundle to accelerate downward. (a) What magnitude of the bundle’s acceleration will put the rope on the verge of snapping? (b) At that acceleration, with what speed would the bundle hit the ground? ••42 In earlier days, horses pulled barges down canals in the manner shown in Fig. 5-42. Suppose the horse pulls on the rope with a force of 7900 N at an angle of u " 183 to the direction of motion of the barge, which is headed straight along the positive direction of an x axis. The mass of the barge is 9500 kg, and the magnitude of its acceleration is 0.12 m/s2. What are the (a) magnitude and (b) direction (relative to positive x) of the force on the barge from the water? θ

Figure 5-42 Problem 42. ••43 SSM In Fig. 5-43, a chain consisting of five links, each of mass 0.100 kg, is lifted vertically with constant acceleration of magnitude a " 2.50 m/s2. Find the magnitudes of (a) the force on link 1 from link 2, (b) the force on link 2 from link 3, (c) the force on link 3 from link 4, and (d) the force on link 4 from link 5. Then find the magni: tudes of (e) the force F on the top link from the person lifting the chain and (f) the net force accelerating each link.

F 5 4

a

3 2 1

••44 A lamp hangs vertically from a cord in a deFigure 5-43 scending elevator that decelerates at 2.4 m/s2. (a) If the tension in the cord is 89 N, what is the lamp’s Problem 43. mass? (b) What is the cord’s tension when the elevator ascends with an upward acceleration of 2.4 m/s2? ••45 An elevator cab that weighs 27.8 kN moves upward. What is the tension in the cable if the cab’s speed is (a) increasing at a rate of 1.22 m/s2 and (b) decreasing at a rate of 1.22 m/s2? ••46 An elevator cab is pulled upward by a cable. The cab and its single occupant have a combined mass of 2000 kg. When that occupant drops a coin, its acceleration relative to the cab is 8.00 m/s2 downward. What is the tension in the cable? ••47 The Zacchini family was renowned for their human-cannonball act in which a family member was shot from a cannon using either elastic bands or compressed air. In one version of the act, Emanuel Zacchini was shot over three Ferris wheels to land in a net at the same height as the open end of the cannon and at a range of 69 m. He was propelled inside the barrel for 5.2 m and launched at an angle of 533. If his mass was 85 kg and he underwent constant acceleration inside the barrel, what was the magnitude of the force propelling him? (Hint: Treat the launch as though it were along a ramp at 533. Neglect air drag.)

120

CHAPTE R 5 FORCE AN D M OTION—I

••48 In Fig. 5-44, elevator cabs A and B are connected by a short cable and can be pulled upward or lowered by the cable above cab A. Cab A has mass 1700 kg; cab B has mass 1300 kg.A 12.0 kg box of catnip lies on the floor of cab A. The tension in the cable connecting the cabs is 1.91 ' 10 4 N. What is the magnitude of the normal force on the box from the floor?

A

••49 In Fig. 5-45, a block of mass m " 5.00 kg is pulled along a horizontal frictionless floor by a cord B that exerts a force of magnitude F " 12.0 N at an angle u " 25.03. (a) What is the magnitude of the block’s acceleration? (b) The force magnitude F is Figure 5-44 slowly increased. What is its value just before the Problem 48. block is lifted (completely) off the floor? (c) What is the magnitude of the block’s acceleration just before it is lifted (completely) off the floor?

θ F

m

Figure 5-45 Problems 49 and 60.

••51 Figure 5-47 shows two blocks connected by a cord (of negligible mass) that passes over a frictionless pulley (also of negligible mass). The arrangement is known as Atwood’s machine. One block has mass m1 " 1.30 kg; the other has mass m2 " m1 2.80 kg. What are (a) the magnitude of the blocks’ acceleration and (b) the tension in the cord? ••52 An 85 kg man lowers himself to the ground from a height of 10.0 m by holding onto a rope that runs over a frictionless pulley to a 65 kg sandbag. With what speed does the man hit the ground if he started from rest?

m2

T2

m3

Figure 5-48 Problem 53.

T2

T4

Figure 5-49 Problem 54. m1 ••55 SSM ILW WWW Two blocks are in contact on a frictionless table. A horizonm2 F tal force is applied to the larger block, as shown in Fig. 5-50. (a) If m1 " 2.3 kg, m2 " 1.2 kg, and F " 3.2 N, find the magnitude of the force between the two Figure 5-50 blocks. (b) Show that if a force of the same Problem 55. magnitude F is applied to the smaller block but in the opposite direction, the magnitude of the force between the blocks is 2.1 N, which is not the same value calculated in (a). (c) Explain the difference.

T3

••56 In Fig. 5-51a, a constant horizontal force Fa is applied to block A, which pushes against block B with a 20.0 N force directed : horizontally to the right. In Fig. 5-51b, the same force Fa is applied to block B; now block A pushes on block B with a 10.0 N force directed horizontally to the left. The blocks have a combined mass of 12.0 kg. What are the magnitudes of (a) their acceleration in : Fig. 5-51a and (b) force Fa? A

B

B

Fa

A

Fa (a)

(b)

Figure 5-51 Problem 56.

m2

Figure 5-47 Problems 51 and 65.

••53 In Fig. 5-48, three connected blocks are pulled to the right on a horizontal frictionless table by a force of magnitude T3 " 65.0 N. If m1 " 12.0 kg, m2 " 24.0 kg, and m3 " 31.0 kg, calculate (a) the magnitude of the system’s acceleration, (b) the tension T1, and (c) the tension T2. T1

m4

m3

m1

:

••50 In Fig. 5-46, three ballot A boxes are connected by cords, one of which wraps over a pulley having B negligible friction on its axle and C negligible mass. The three masses are mA " 30.0 kg, mB " 40.0 kg, Figure 5-46 Problem 50. and mC " 10.0 kg. When the assembly is released from rest, (a) what is the tension in the cord connecting B and C, and (b) how far does A move in the first 0.250 s (assuming it does not reach the pulley)?

m1

••54 Figure 5-49 shows four penguins that are being playfully pulled along very slippery (frictionless) ice by a curator. The masses of three penguins and the tension in two of the cords are m1 " 12 kg, m3 " 15 kg, m4 " 20 kg, T2 " 111 N, and T4 " 222 N. Find the penguin mass m2 that is not given.

••57 ILW A block of mass m1 " 3.70 kg on a frictionless plane inclined at angle 1 " 30.03 is connected by a cord over a massless, frictionless pulley to a second block of mass m2 " 2.30 kg (Fig. 5-52). What are (a) the magnitude of the acceleration of each block, (b) the direction of the acceleration of the hanging block, and (c) the tension in the cord?

m1

m2

θ

Figure 5-52 Problem 57. ••58 Figure 5-53 shows a man sitting in a bosun’s chair that dangles from a massless rope, which runs over a massless, frictionless pulley and back down to the man’s hand. The combined mass of man and chair is 95.0 kg. With what force magnitude must the man pull on the rope if he is to rise (a) with a constant velocity and

PROB LE M S

(b) with an upward acceleration of 1.30 m/s2? (Hint: A free-body diagram can really help.) If the rope on the right extends to the ground and is pulled by a co-worker, with what force magnitude must the coworker pull for the man to rise (c) with a constant velocity and (d) with an upward acceleration of 1.30 m/s2? What is the magnitude of the force on the ceiling from the pulley system in (e) part a, (f ) part b, (g) part c, and (h) part d? ••59 SSM A 10 kg monkey climbs up a massless rope that runs over a frictionless tree limb and back down to a 15 kg package on the ground (Fig. 5-54). (a) What is the magnitude of the least acceleration the monkey must have if it is to lift the package off the ground? If, after the package has been lifted, the monkey stops its climb and holds onto the rope, what are the (b) magnitude and (c) direction of the monkey’s acceleration and (d) the tension in the rope?

121

the axis, with a speed of 3.0 m/s. What are its (a) speed and (b) direction of travel at t " 11 s? Fx (N) 6

0 2

4

6

8

10

t (s) 12

–4

Figure 5-55 Problem 63. Figure 5-53 Problem 58.

••60 Figure 5-45 shows a 5.00 kg block being pulled along a frictionless floor by a cord that applies a Bananas force of constant magnitude 20.0 N but with an angle u(t) that varies with time. When angle u " 25.03, at Figure 5-54 Problem 59. what rate is the acceleration of the block changing if (a) u(t) " (2.00 ' 10$2 deg/s)t and (b) u(t) " $(2.00 ' 10$2 deg/s)t? (Hint: The angle should be in radians.) ••61 SSM ILW A hot-air balloon of mass M is descending vertically with downward acceleration of magnitude a. How much mass (ballast) must be thrown out to give the balloon an upward acceleration of magnitude a? Assume that the upward force from the air (the lift) does not change because of the decrease in mass. •••62 In shot putting, many athletes elect to launch the shot at an angle that is smaller than the theoretical one (about 423) at which the distance of a projected ball at the same speed and height is greatest. One reason has to do with the speed the athlete can give the shot during the acceleration phase of the throw. Assume that a 7.260 kg shot is accelerated along a straight path of length 1.650 m by a constant applied force of magnitude 380.0 N, starting with an initial speed of 2.500 m/s (due to the athlete’s preliminary motion). What is the shot’s speed at the end of the acceleration phase if the angle between the path and the horizontal is (a) 30.003 and (b) 42.003? (Hint: Treat the motion as though it were along a ramp at the given angle.) (c) By what percent is the launch speed decreased if the athlete increases the angle from 30.003 to 42.003? •••63 Figure 5-55 gives, as a function of time t, the force component Fx that acts on a 3.00 kg ice block that can move only along the x axis. At t " 0, the block is moving in the positive direction of

•••64 Figure 5-56 shows a box of mass m2 " 1.0 kg on a frictionless plane inclined at angle u " 303. It is connected by a cord of negligible mass to a box of mass m1 " 3.0 kg on a horizontal frictionless surface. The pulley is frictionless and massless. (a) If the : magnitude of horizontal force F is 2.3 N, what is the tension in the : connecting cord? (b) What is the largest value the magnitude of F may have without the cord becoming slack? m1

F m2

θ

Figure 5-56 Problem 64. •••65 Figure 5-47 shows Atwood’s machine, in which two containers are connected by a cord (of negligible mass) passing over a frictionless pulley (also of negligible mass). At time t " 0, container 1 has mass 1.30 kg and container 2 has mass 2.80 kg, but container 1 is losing mass (through a leak) at the constant rate of 0.200 kg/s. At what rate is the acceleration magnitude of the containers changing at (a) t " 0 and (b) t " 3.00 s? (c) When does the acceleration reach its maximum value? •••66 Figure 5-57 shows a section of a cable-car system. The maximum permissible mass of each car with occupants is 2800 kg. The cars, riding on a support cable, are pulled by a second cable attached to the support tower on each car. Assume that the cables Support cable Pull cable

θ

Figure 5-57 Problem 66.

122

CHAPTE R 5 FORCE AN D M OTION—I

are taut and inclined at angle u " 353. What is the difference in tension between adjacent sections of pull cable if the cars are at the maximum permissible mass and are being accelerated up the incline at 0.81 m/s2? •••67 Figure 5-58 shows three blocks attached by cords that loop over frictionless pulleys. Block B lies on a frictionless table; the masses are mA " 6.00 kg, mB " 8.00 kg, and mC " 10.0 kg. When the blocks are released, what is the tension in the cord at the right?

B

C

A

Figure 5-58 Problem 67.

•••68 A shot putter launches a 7.260 kg shot by pushing it along a straight line of length 1.650 m and at an angle of 34.10° from the horizontal, accelerating the shot to the launch speed from its initial speed of 2.500 m/s (which is due to the athlete’s preliminary motion). The shot leaves the hand at a height of 2.110 m and at an angle of 34.103, and it lands at a horizontal distance of 15.90 m. What is the magnitude of the athlete’s average force on the shot during the acceleration phase? (Hint: Treat the motion during the acceleration phase as though it were along a ramp at the given angle.) Additional Problems 69 In Fig. 5-59, 4.0 kg block A and 6.0 kg block B are connected by : a string of negligible mass. Force FA " (12 N)iˆ acts on block A; : force FB " (24 N)iˆ acts on block B.What is the tension in the string? FA

A

FB

B

x

Figure 5-59 Problem 69.

70 An 80 kg man drops to a concrete patio from a window 0.50 m above the patio. He neglects to bend his knees on landing, taking 2.0 cm to stop. (a) What is his average acceleration from when his feet first touch the patio to when he stops? (b) What is the magnitude of the average stopping force exerted on him by the patio? 71 SSM Figure 5-60 shows a box of dirty money (mass m1 " 3.0 kg) on a frictionless plane inclined at angle 11 " 303. The box is connected via a cord of negligible mass to a box of laundered money (mass m2 " 2.0 kg) on a frictionless plane inclined at angle u2 " 603. The pulley is frictionless and has negligible mass. What is the tension in the cord?

m1

θ1

m2

θ2

Figure 5-60 Problem 71. 72 Three forces act on a particle that moves with unchanging ve: locity : v " (2 m/s)iˆ $ (7 m/s)jˆ . Two of the forces are F1 " (2 N)iˆ # : (3 N)jˆ # ($2 N)kˆ and F 2 " ($5 N)iˆ # (8 N)jˆ # ($2 N)kˆ . What is the third force?

73 SSM In Fig. 5-61, a tin of antioxidants (m1 " 1.0 kg) on a frictionless inclined surface is connected to a tin of corned beef (m2 " 2.0 kg). The pulley is massless and frictionless. An upward force of magnitude F " 6.0 N acts on the corned beef tin, which has a downward acceleration of 5.5 m/s2. What are (a) the tension in the connecting cord and (b) angle b? 74 The only two forces acting on a body have magnitudes of 20 N and 35 N and directions that differ by 803. The resulting acceleration has a magnitude of 20 m/s2. What is the mass of the body? 75 Figure 5-62 is an overhead view of a 12 kg tire that is to be pulled by three horizontal ropes. One rope’s force (F1 " 50 N) is indicated. The forces from the other ropes are to be oriented such that the tire’s acceleration magnitude a is least. What is that least a if (a) F2 " 30 N, F3 " 20 N; (b) F2 " 30 N, F3 " 10 N; and (c) F2 " F3 " 30 N?

m1

β

m2

F

Figure 5-61 Problem 73.

x F1

Figure 5-62 Problem 75.

M

m

F

76 A block of mass M is pulled along a horizontal frictionless surface by a rope of mass m, as shown Figure 5-63 Problem 76. : in Fig. 5-63. A horizontal force F acts on one end of the rope. (a) Show that the rope must sag, even if only by an imperceptible amount. Then, assuming that the sag is negligible, find (b) the acceleration of rope and block, (c) the force on the block from the rope, and (d) the tension in the rope at its midpoint. 77 SSM A worker drags a crate across a factory floor by pulling on a rope tied to the crate. The worker exerts a force of magnitude F " 450 N on the rope, which is inclined at an upward angle u " 383 to the horizontal, and the floor exerts a horizontal force of magnitude f " 125 N that opposes the motion. Calculate the magnitude of the acceleration of the crate if (a) its mass is 310 kg and (b) its weight is 310 N. :

78 In Fig. 5-64, a force F of magF m2 nitude 12 N is applied to a FedEx m1 box of mass m2 " 1.0 kg. The force is directed up a plane tilted by u " θ 373. The box is connected by a cord to a UPS box of mass m1 " 3.0 kg Figure 5-64 Problem 78. on the floor. The floor, plane, and pulley are frictionless, and the masses of the pulley and cord are negligible. What is the tension in the cord? 79 A certain particle has a weight of 22 N at a point where g " 9.8 m/s2. What are its (a) weight and (b) mass at a point where g " 4.9 m/s2? What are its (c) weight and (d) mass if it is moved to a point in space where g " 0? 80 An 80 kg person is parachuting and experiencing a downward acceleration of 2.5 m/s2. The mass of the parachute is 5.0 kg. (a)

PROB LE M S

What is the upward force on the open parachute from the air? (b) What is the downward force on the parachute from the person? 81 A spaceship lifts off vertically from the Moon, where g " 1.6 m/s2. If the ship has an upward acceleration of 1.0 m/s2 as it lifts off, what is the magnitude of the force exerted by the ship on its pilot, who weighs 735 N on Earth? 82 In the overhead view of Fig. 5-65, five forces pull on a box of mass m " 4.0 kg. The force magnitudes are F1 " 11 N, F2 " 17 N, F3 " 3.0 N, F4 " 14 N, and F5 " 5.0 N, and angle u4 is 303. Find the box’s acceleration (a) in unit-vector notation and as (b) a magnitude and (c) an angle relative to the positive direction of the x axis.

y F5

F4

θ4 F1

x

F3 F2

Figure 5-65 Problem 82. 83 SSM A certain force gives an object of mass m1 an acceleration of 12.0 m/s2 and an object of mass m2 an acceleration of 3.30 m/s2. What acceleration would the force give to an object of mass (a) m2 $ m1 and (b) m2 # m1?

123

acceleration in g units? (c) What force is required for the acceleration? (d) If the engines are shut down when 0.10c is reached (the speed then remains constant), how long does the ship take (start to finish) to journey 5.0 light-months, the distance that light travels in 5.0 months? 91 SSM A motorcycle and 60.0 kg rider accelerate at 3.0 m/s2 up a ramp inclined 103 above the horizontal. What are the magnitudes of (a) the net force on the rider and (b) the force on the rider from the motorcycle? 92 Compute the initial upward acceleration of a rocket of mass 1.3 ' 10 4 kg if the initial upward force produced by its engine (the thrust) is 2.6 ' 10 5 N. Do not neglect the gravitational force on the rocket. 93 SSM Figure 5-66a shows a mobile hanging from a ceiling; it consists of two metal pieces (m1 " 3.5 kg and m2 " 4.5 kg) that are strung together by cords of negligible mass. What is the tension in (a) the bottom cord and (b) the top cord? Figure 5-66b shows a mobile consisting of three metal pieces. Two of the masses are m3 " 4.8 kg and m5 " 5.5 kg. The tension in the top cord is 199 N. What is the tension in (c) the lowest cord and (d) the middle cord?

:

84 You pull a short refrigerator with a constant force F across a : greased (frictionless) floor, either with F horizontal (case 1) or with : F tilted upward at an angle u (case 2). (a) What is the ratio of the refrigerator’s speed in case 2 to its speed in case 1 if you pull for a certain time t? (b) What is this ratio if you pull for a certain distance d? 85 A 52 kg circus performer is to slide down a rope that will break if the tension exceeds 425 N. (a) What happens if the performer hangs stationary on the rope? (b) At what magnitude of acceleration does the performer just avoid breaking the rope? 86 Compute the weight of a 75 kg space ranger (a) on Earth, (b) on Mars, where g " 3.7 m/s2, and (c) in interplanetary space, where g " 0. (d) What is the ranger’s mass at each location? 87 An object is hung from a spring balance attached to the ceiling of an elevator cab. The balance reads 65 N when the cab is standing still. What is the reading when the cab is moving upward (a) with a constant speed of 7.6 m/s and (b) with a speed of 7.6 m/s while decelerating at a rate of 2.4 m/s2? 88 Imagine a landing craft approaching the surface of Callisto, one of Jupiter’s moons. If the engine provides an upward force (thrust) of 3260 N, the craft descends at constant speed; if the engine provides only 2200 N, the craft accelerates downward at 0.39 m/s2. (a) What is the weight of the landing craft in the vicinity of Callisto’s surface? (b) What is the mass of the craft? (c) What is the magnitude of the free-fall acceleration near the surface of Callisto? 89 A 1400 kg jet engine is fastened to the fuselage of a passenger jet by just three bolts (this is the usual practice). Assume that each bolt supports one-third of the load. (a) Calculate the force on each bolt as the plane waits in line for clearance to take off. (b) During flight, the plane encounters turbulence, which suddenly imparts an upward vertical acceleration of 2.6 m/s2 to the plane. Calculate the force on each bolt now. 90 An interstellar ship has a mass of 1.20 ' 10 6 kg and is initially at rest relative to a star system. (a) What constant acceleration is needed to bring the ship up to a speed of 0.10c (where c is the speed of light, 3.0 ' 108 m/s) relative to the star system in 3.0 days? (b) What is that

m3

m1 m2

m5

(a)

(b)

Figure 5-66 Problem 93. 94 For sport, a 12 kg armadillo runs onto a large pond of level, frictionless ice. The armadillo’s initial velocity is 5.0 m/s along the positive direction of an x axis. Take its initial position on the ice as being the origin. It slips over the ice while being pushed by a wind with a force of 17 N in the positive direction of the y axis. In unitvector notation, what are the animal’s (a) velocity and (b) position vector when it has slid for 3.0 s? 95 Suppose that in Fig. 5-12, the masses of the blocks are 2.0 kg and 4.0 kg. (a) Which mass should the hanging block have if the magnitude of the acceleration is to be as large as possible? What then are (b) the magnitude of the acceleration and (c) the tension in the cord? 96 A nucleus that captures a stray neutron must bring the neutron to a stop within the diameter of the nucleus by means of the strong force. That force, which “glues” the nucleus together, is approximately zero outside the nucleus. Suppose that a stray neutron with an initial speed of 1.4 ' 10 7 m/s is just barely captured by a nucleus with diameter d " 1.0 ' 10$14 m. Assuming the strong force on the neutron is constant, find the magnitude of that force. The neutron’s mass is 1.67 ' 10$27 kg. :

97 If the 1 kg standard body is accelerated by only F 1 " : (3.0 N)iˆ # (4.0 N)jˆ and F 2 " ($2.0 N)iˆ # ($6.0 N)jˆ , then what : is Fnet (a) in unit-vector notation and as (b) a magnitude and (c) an angle relative to the positive x direction? What are the (d) magnitude and (e) angle of : a?

C

H

A

P

T

E

R

6

Force and Motion—II 6-1 FRICTION Learning Objectives After reading this module, you should be able to . . .

6.01 Distinguish between friction in a static situation and a kinetic situation. 6.02 Determine direction and magnitude of a frictional force.

6.03 For objects on horizontal, vertical, or inclined planes in situations involving friction, draw free-body diagrams and apply Newton’s second law.

Key Ideas :

● When a force F

:

:

tends to slide a body along a surface, a frictional force from the surface acts on the body. The frictional force is parallel to the surface and directed so as to oppose the sliding. It is due to bonding between the body and the surface. If the body does not slide, the frictional force is a static : frictional force f s. If there is sliding, the frictional force is a : kinetic frictional force f k.

● The magnitude of f s has a maximum value

● If a body does not move, the static frictional force f s and :

frictional force rapidly decreases to a constant value fk given by

:

the component of F parallel to the surface are equal in magni: tude, and f s is directed opposite that component. If the component increases, fs also increases.

f s,max given by

fs,max " msFN, where ms is the coefficient of static friction and FN is the mag: nitude of the normal force. If the component of F parallel to the surface exceeds fs,max, the body slides on the surface. ● If the body begins to slide on the surface, the magnitude of the

fk " mkFN, where mk is the coefficient of kinetic friction.

What Is Physics? In this chapter we focus on the physics of three common types of force: frictional force, drag force, and centripetal force. An engineer preparing a car for the Indianapolis 500 must consider all three types. Frictional forces acting on the tires are crucial to the car’s acceleration out of the pit and out of a curve (if the car hits an oil slick, the friction is lost and so is the car). Drag forces acting on the car from the passing air must be minimized or else the car will consume too much fuel and have to pit too early (even one 14 s pit stop can cost a driver the race). Centripetal forces are crucial in the turns (if there is insufficient centripetal force, the car slides into the wall). We start our discussion with frictional forces.

Friction Frictional forces are unavoidable in our daily lives. If we were not able to counteract them, they would stop every moving object and bring to a halt every rotating shaft. About 20% of the gasoline used in an automobile is needed to counteract friction in the engine and in the drive train. On the other hand, if friction were totally absent, we could not get an automobile to go anywhere, and we could not walk or ride a bicycle. We could not hold a pencil, and, if we could, it would not write. Nails and screws would be useless, woven cloth would fall apart, and knots would untie. 124

6-1 FR ICTION

125

Three Experiments. Here we deal with the frictional forces that exist between dry solid surfaces, either stationary relative to each other or moving across each other at slow speeds. Consider three simple thought experiments: 1. Send a book sliding across a long horizontal counter. As expected, the book slows and then stops. This means the book must have an acceleration parallel to the counter surface, in the direction opposite the book’s velocity. From Newton’s second law, then, a force must act on the book parallel to the counter surface, in the direction opposite its velocity. That force is a frictional force. 2. Push horizontally on the book to make it travel at constant velocity along the counter. Can the force from you be the only horizontal force on the book? No, because then the book would accelerate. From Newton’s second law, there must be a second force, directed opposite your force but with the same magnitude, so that the two forces balance. That second force is a frictional force, directed parallel to the counter. 3. Push horizontally on a heavy crate. The crate does not move. From Newton’s second law, a second force must also be acting on the crate to counteract your force. Moreover, this second force must be directed opposite your force and have the same magnitude as your force, so that the two forces balance. That second force is a frictional force. Push even harder. The crate still does not move. Apparently the frictional force can change in magnitude so that the two forces still balance. Now push with all your strength. The crate begins to slide. Evidently, there is a maximum magnitude of the frictional force. When you exceed that maximum magnitude, the crate slides. Two Types of Friction. Figure 6-1 shows a similar situation. In Fig. 6-1a, a block : : rests on a tabletop, with the gravitational force Fg balanced by a normal force FN. : In Fig. 6-1b, you exert a force F on the block, attempting to pull it to the left. In re: sponse, a frictional force f s is directed to the right, exactly balancing your force. : The force f s is called the static frictional force. The block does not move.

There is no attempt at sliding. Thus, no friction and no motion.

Frictional force = 0 Fg

(a)

Force F attempts sliding but is balanced by the frictional force. No motion. Force F is now stronger but is still balanced by the frictional force. No motion. Figure 6-1 (a) The forces on a stationary block. (b–d) An external : force F , applied to the block, is balanced by a static frictional force : f s. As F is increased, fs also increases, until fs reaches a certain maximum value. (Figure continues)

FN

Frictional force = F

fs

F Fg

(b) FN

fs

F

FN

fs

F Fg (d)

Frictional force = F

Fg

(c)

Force F is now even stronger but is still balanced by the frictional force. No motion.

A

FN

Frictional force = F

CHAPTE R 6 FORCE AN D M OTION—I I

Finally, the applied force has overwhelmed the static frictional force. Block slides and accelerates.

Figure 6-1 (Continued) (e) Once fs reaches its maximum value, the block “breaks away,” accelerating suddenly in the direc: tion of F . (f ) If the block is now to move with constant velocity, F must be reduced from the maximum value it had just before the block broke away. (g) Some experimental results for the sequence (a) through (f ). In WileyPLUS, this figure is available as an animation with voiceover.

To maintain the speed, weaken force F to match the weak frictional force.

a

FN

F

fk

Weak kinetic frictional force

Fg (e) v FN

fk

F

Same weak kinetic frictional force

Fg (f )

Static frictional force can only match growing applied force.

Magnitude of frictional force

126

0 (g)

Maximum value of fs fk is approximately constant

Kinetic frictional force has only one value (no matching).

Breakaway Time

Figures 6-1c and 6-1d show that as you increase the magnitude of your : applied force, the magnitude of the static frictional force f s also increases and the block remains at rest. When the applied force reaches a certain magnitude, however, the block “breaks away” from its intimate contact with the tabletop and accelerates leftward (Fig. 6-1e). The frictional force that then opposes the motion : is called the kinetic frictional force f k . Usually, the magnitude of the kinetic frictional force, which acts when there is motion, is less than the maximum magnitude of the static frictional force, which acts when there is no motion. Thus, if you wish the block to move across the surface with a constant speed, you must usually decrease the magnitude of the applied force once the block begins to move, as in Fig. 6-1f. As an example, Fig. 6-1g shows the results of an experiment in which the force on a block was slowly increased until breakaway occurred. Note the reduced force needed to keep the block moving at constant speed after breakaway. Microscopic View. A frictional force is, in essence, the vector sum of many forces acting between the surface atoms of one body and those of another body. If two highly polished and carefully cleaned metal surfaces are brought together in a very good vacuum (to keep them clean), they cannot be made to slide over each other. Because the surfaces are so smooth, many atoms of one surface contact many atoms of the other surface, and the surfaces cold-weld together instantly, forming a single piece of metal. If a machinist’s specially polished gage blocks are brought together in air, there is less atom-to-atom contact, but the blocks stick firmly to each other and can be separated only by means of a wrenching motion. Usually, however, this much atom-to-atom contact is not possible. Even a highly polished metal surface is far from being flat on the atomic scale. Moreover, the surfaces of everyday objects have layers of oxides and other contaminants that reduce cold-welding. When two ordinary surfaces are placed together, only the high points touch each other. (It is like having the Alps of Switzerland turned over and placed down on the Alps of Austria.) The actual microscopic area of contact is much less than the apparent macroscopic contact area, perhaps by a factor of 104. Nonetheless,

6-1 FR ICTION

many contact points do cold-weld together. These welds produce static friction when an applied force attempts to slide the surfaces relative to each other. If the applied force is great enough to pull one surface across the other, there is first a tearing of welds (at breakaway) and then a continuous re-forming and tearing of welds as movement occurs and chance contacts are made (Fig. 6-2). : The kinetic frictional force fk that opposes the motion is the vector sum of the forces at those many chance contacts. If the two surfaces are pressed together harder, many more points cold-weld. Now getting the surfaces to slide relative to each other requires a greater applied : force: The static frictional force fs has a greater maximum value. Once the surfaces are sliding, there are many more points of momentary cold-welding, so the : kinetic frictional force fk also has a greater magnitude. Often, the sliding motion of one surface over another is “jerky” because the two surfaces alternately stick together and then slip. Such repetitive stick-and-slip can produce squeaking or squealing, as when tires skid on dry pavement, fingernails scratch along a chalkboard, or a rusty hinge is opened. It can also produce beautiful and captivating sounds, as in music when a bow is drawn properly across a violin string.

Properties of Friction Experiment shows that when a dry and unlubricated body presses against a surface : in the same condition and a force F attempts to slide the body along the surface, the resulting frictional force has three properties: :

Property 1. If the body does not move, then the static frictional force fs and the : component of F that is parallel to the surface balance each other. They are : : equal in magnitude, and fs is directed opposite that component of F . Property 2.

:

The magnitude of fs has a maximum value fs,max that is given by fs,max " msFN,

(6-1)

where ms is the coefficient of static friction and FN is the magnitude of the normal force on the body from the surface. If the magnitude of the compo: nent of F that is parallel to the surface exceeds fs,max, then the body begins to slide along the surface. Property 3. If the body begins to slide along the surface, the magnitude of the frictional force rapidly decreases to a value fk given by fk " mkFN,

(6-2)

where mk is the coefficient of kinetic friction. Thereafter, during the sliding, a : kinetic frictional force fk with magnitude given by Eq. 6-2 opposes the motion. The magnitude FN of the normal force appears in properties 2 and 3 as a measure of how firmly the body presses against the surface. If the body presses harder, then, by Newton’s third law, FN is greater. Properties 1 and 2 are worded : in terms of a single applied force F , but they also hold for the net force of several applied forces acting on the body. Equations 6-1 and 6-2 are not vector equations; : : the direction of fs or fk is always parallel to the surface and opposed to the at: tempted sliding, and the normal force FN is perpendicular to the surface. The coefficients ms and mk are dimensionless and must be determined experimentally. Their values depend on certain properties of both the body and the surface; hence, they are usually referred to with the preposition “between,” as in “the value of ms between an egg and a Teflon-coated skillet is 0.04, but that between rock-climbing shoes and rock is as much as 1.2.” We assume that the value of mk does not depend on the speed at which the body slides along the surface.

127

(a)

(b)

Figure 6-2 The mechanism of sliding friction. (a) The upper surface is sliding to the right over the lower surface in this enlarged view. (b) A detail, showing two spots where cold-welding has occurred. Force is required to break the welds and maintain the motion.

128

CHAPTE R 6 FORCE AN D M OTION—I I

Checkpoint 1 A block lies on a floor. (a) What is the magnitude of the frictional force on it from the floor? (b) If a horizontal force of 5 N is now applied to the block, but the block does not move, what is the magnitude of the frictional force on it? (c) If the maximum value fs,max of the static frictional force on the block is 10 N, will the block move if the magnitude of the horizontally applied force is 8 N? (d) If it is 12 N? (e) What is the magnitude of the frictional force in part (c)?

Sample Problem 6.01 Angled force applied to an initially stationary block This sample problem involves a tilted applied force, which requires that we work with components to find a frictional force. The main challenge is to sort out all the components. Figure 6-3a shows a force of magnitude F " 12.0 N applied to an 8.00 kg block at a downward angle of u " 30.03. The coefficient of static friction between block and floor is ms " 0.700; the coefficient of kinetic friction is mk " 0.400. Does the block begin to slide or does it remain stationary? What is the magnitude of the frictional force on the block?

ond law as FN $ mg $ F sin u " m(0),

(6-4)

FN " mg # F sin u.

(6-5)

which gives us

Now we can evaluate fs,max " msFN: fs,max " ms (mg # F sin u) " (0.700)((8.00 kg)(9.8 m/s2) # (12.0 N)(sin 303)) " 59.08 N.

KEY IDEAS (1) When the object is stationary on a surface, the static frictional force balances the force component that is attempting to slide the object along the surface. (2) The maximum possible magnitude of that force is given by Eq. 6-1 ( fs,max " msFN). (3) If the component of the applied force along the surface exceeds this limit on the static friction, the block begins to slide. (4) If the object slides, the kinetic frictional force is given by Eq. 6-2 ( fk " mkFN). Calculations: To see if the block slides (and thus to calculate the magnitude of the frictional force), we must compare the applied force component Fx with the maximum magnitude fs,max that the static friction can have. From the triangle of components and full force shown in Fig. 6-3b, we see that

(6-6)

Because the magnitude Fx (" 10.39 N) of the force component attempting to slide the block is less than fs,max (" 59.08 N), the block remains stationary. That means that the magnitude fs of the frictional force matches Fx. From Fig. 6-3d, we can write Newton’s second law for x components as (6-7)

Fx $ fs " m(0), fs " Fx " 10.39 N % 10.4 N.

and thus

y x

u

Block

(Answer)

Fx u

F

F (b)

(a)

Fy

Fx " F cos u " (12.0 N) cos 303 " 10.39 N.

FN

(6-3)

From Eq. 6-1, we know that fs,max " msFN, but we need the magnitude FN of the normal force to evaluate fs,max. Because the normal force is vertical, we need to write Newton’s second law (Fnet,y " may) for the vertical force components acting on the block, as displayed in Fig. 6-3c. The gravitational force with magnitude mg acts downward. The applied force has a downward component Fy " F sin u. And the vertical acceleration ay is just zero. Thus, we can write Newton’s sec-

Block Fy Fg (c)

fs

Fx (d)

Figure 6-3 (a) A force is applied to an initially stationary block. (b) The components of the applied force. (c) The vertical force components. (d) The horizontal force components.

Additional examples, video, and practice available at WileyPLUS

129

6-1 FR ICTION

Sample Problem 6.02 Sliding to a stop on icy roads, horizontal and inclined Some of the funniest videos on the web involve motorists sliding uncontrollably on icy roads. Here let’s compare the typical stopping distances for a car sliding to a stop from an initial speed of 10.0 m/s on a dry horizontal road, an icy horizontal road, and (everyone’s favorite) an icy hill. (a) How far does the car take to slide to a stop on a horizontal road (Fig. 6-4a) if the coefficient of kinetic friction is mk " 0.60, which is typical of regular tires on dry pavement? Let’s neglect any effect of the air on the car, assume that the wheels lock up and the tires slide, and extend an x axis in the car’s direction of motion.

v0

v=0

µ = 0.60 x – x0 (a) y FN

This is a free-body diagram of the forces on the car.

y

Normal force supports the car. Car

FN fk

x

mg sin u

fk

KEY IDEAS (1) The car accelerates (its speed decreases) because a horizontal frictional force acts against the motion, in the negative direction of the x axis. (2) The frictional force is a kinetic frictional force with a magnitude given by Eq. 6-2 ( fk " mkFN), in which FN is the magnitude of the normal force on the car from the road. (3) We can relate the frictional force to the resulting acceleration by writing Newton’s second law (Fnet,x " max) for motion along the road. Calculations: Figure 6-4b shows the free-body diagram for the car.The normal force is upward, the gravitational force is downward, and the frictional force is horizontal. Because the frictional force is the only force with an x component, Newton’s second law written for motion along the x axis becomes $fk " max.

(6-8)

Substituting fk " mkFN gives us $mkFN " max.

(6-9)

From Fig. 6-4b we see that the upward normal force balances the downward gravitational force, so in Eq. 6-9 let’s replace magnitude FN with magnitude mg. Then we can cancel m (the stopping distance is thus independent of the car’s mass—the car can be heavy or light, it does not matter). Solving for ax we find ax " $mkg.

(6-10)

Because this acceleration is constant, we can use the constant-acceleration equations of Table 2-1. The easiest choice for finding the sliding distance x $ x0 is Eq. 2-16 (v2 " v20 # 2a(x $ x0)), which gives us v 2 $ v 20 x $ x0 " . 2ax

(6-11)

Substituting from Eq. 6-10, we then have x $ x0 "

v2 $ v20 . $2mkg

(6-12)

Frictional force opposes the sliding.

Fg

Gravitational force mg cos u u pulls downward.

(b)

u Fg

(c)

Figure 6-4 (a) A car sliding to the right and finally stopping after a displacement of 290 m. A free-body diagram for the car on (b) a horizontal road and (c) a hill.

Inserting the initial speed v0 " 10.0 m/s, the final speed v " 0, and the coefficient of kinetic friction mk " 0.60, we find that the car’s stopping distance is x $ x0 " 8.50 m % 8.5 m.

(Answer)

(b) What is the stopping distance if the road is covered with ice with mk " 0.10? Calculation: Our solution is perfectly fine through Eq. 6-12 but now we substitute this new mk, finding x $ x0 " 51 m.

(Answer)

Thus, a much longer clear path would be needed to avoid the car hitting something along the way. (c) Now let’s have the car sliding down an icy hill with an inclination of u " 5.003 (a mild incline, nothing like the hills of San Francisco). The free-body diagram shown in Fig. 6-4c is like the ramp in Sample Problem 5.04 except, to be consistent with Fig. 6-4b, the positive direction of the x axis is down the ramp. What now is the stopping distance? Calculations: Switching from Fig. 6-4b to c involves two major changes. (1) Now a component of the gravitational force is along the tilted x axis, pulling the car down the hill. From Sample Problem 5.04 and Fig. 5-15, that down-the-hill component is mg sin u, which is in the positive direction of the x axis in Fig. 6-4c. (2) The normal force (still perpendicular to the road) now balances only a component of the gravitational

x

130

CHAPTE R 6 FORCE AN D M OTION—I I

force, not the full force. From Sample Problem 5.04 (see Fig. 5-15i), we write that balance as FN " mg cos u. In spite of these changes, we still want to write Newton’s second law (Fnet,x " max) for the motion along the (now tilted) x axis. We have

and

$fk # mg sin u " max, $mkFN # mg sin u " max, $mkmg cos u # mg sin u " max.

now give us ax " $mkg cos u # g sin u " $(0.10)(9.8 m/s2) cos 5.003 # (9.8 m/s2) sin 5.003 " $0.122 m/s2.

(6-13)

Substituting this result into Eq. 6-11 gives us the stopping distance hown the hill: x $ x0 " 409 m % 400 m,

Solving for the acceleration and substituting the given data

(Answer)

which is about 14 mi! Such icy hills separate people who can do this calculation (and thus know to stay home) from people who cannot (and thus end up in web videos).

Additional examples, video, and practice available at WileyPLUS

6-2 THE DRAG FORCE AND TERMINAL SPEED Learning Objectives After reading this module, you should be able to . . .

6.04 Apply the relationship between the drag force on an object moving through air and the speed of the object.

6.05 Determine the terminal speed of an object falling through air.

Key Ideas ● When there is relative motion between air (or some other : fluid) and a body, the body experiences a drag force D that opposes the relative motion and points in the direction in : which the fluid flows relative to the body. The magnitude of D is related to the relative speed v by an experimentally determined drag coefficient C according to

of a cross section taken perpendicular to the relative velocity : v). ● When a blunt object has fallen far enough through air, the :

:

magnitudes of the drag force D and the gravitational force Fg on the body become equal. The body then falls at a constant terminal speed vt given by

D " 12C*Av 2,

vt " A

where r is the fluid density (mass per unit volume) and A is the effective cross-sectional area of the body (the area

2Fg CrA

.

The Drag Force and Terminal Speed A fluid is anything that can flow—generally either a gas or a liquid. When there is a relative velocity between a fluid and a body (either because the body moves through the fluid or because the fluid moves past the body), the body experiences : a drag force D that opposes the relative motion and points in the direction in which the fluid flows relative to the body. Here we examine only cases in which air is the fluid, the body is blunt (like a baseball) rather than slender (like a javelin), and the relative motion is fast enough so that the air becomes turbulent (breaks up into swirls) behind the : body. In such cases, the magnitude of the drag force D is related to the relative speed v by an experimentally determined drag coefficient C according to D " 12C*Av 2,

(6-14)

6-2 TH E DRAG FORCE AN D TE R M I NAL SPE E D

131

Table 6-1 Some Terminal Speeds in Air Object

Terminal Speed (m/s)

Shot (from shot put) Sky diver (typical) Baseball Tennis ball Basketball Ping-Pong ball Raindrop (radius " 1.5 mm) Parachutist (typical) a

145 60 42 31 20 9 7 5

95% Distancea (m) 2500 430 210 115 47 10 6 3

This is the distance through which the body must fall from rest to reach 95% of its terminal speed.

Based on Peter J. Brancazio, Sport Science, 1984, Simon & Schuster, New York. Karl-Josef Hildenbrand/dpa/Landov LLC

where r is the air density (mass per volume) and A is the effective cross-sectional area of the body (the area of a cross section taken perpendicular to the velocity : v ). The drag coefficient C (typical values range from 0.4 to 1.0) is not truly a constant for a given body because if v varies significantly, the value of C can vary as well. Here, we ignore such complications. Downhill speed skiers know well that drag depends on A and v 2. To reach high speeds a skier must reduce D as much as possible by, for example, riding the skis in the “egg position” (Fig. 6-5) to minimize A. : Falling. When a blunt body falls from rest through air, the drag force D is directed upward; its magnitude gradually increases from zero as the speed of the : : body increases. This upward force D opposes the downward gravitational force Fg on the body. We can relate these forces to the body’s acceleration by writing Newton’s second law for a vertical y axis (Fnet,y " may) as D $ Fg " ma,

(6-15)

where m is the mass of the body. As suggested in Fig. 6-6, if the body falls long enough, D eventually equals Fg. From Eq. 6-15, this means that a " 0, and so the body’s speed no longer increases. The body then falls at a constant speed, called the terminal speed vt . To find vt , we set a " 0 in Eq. 6-15 and substitute for D from Eq. 6-14, obtaining 1 2 2 C*Av t

which gives

vt "

$ Fg " 0,

2Fg . A C*A

Figure 6-5 This skier crouches in an “egg position” so as to minimize her effective cross-sectional area and thus minimize the air drag acting on her.

As the cat's speed increases, the upward drag force increases until it balances the gravitational force.

(6-16)

Table 6-1 gives values of vt for some common objects. According to calculations* based on Eq. 6-14, a cat must fall about six floors to reach terminal speed. Until it does so, Fg . D and the cat accelerates downward because of the net downward force. Recall from Chapter 2 that your body is an accelerometer, not a speedometer. Because the cat also senses the acceleration, it is frightened and keeps its feet underneath its body, its head tucked in, and its spine bent upward, making A small, vt large, and injury likely. However, if the cat does reach vt during a longer fall, the acceleration vanishes and the cat relaxes somewhat, stretching its legs and neck horizontally outward and *W. O. Whitney and C. J. Mehlhaff, “High-Rise Syndrome in Cats.” The Journal of the American Veterinary Medical Association, 1987.

Falling body

Fg (a)

D D

Fg

(b)

Fg

(c)

Figure 6-6 The forces that act on a body falling through air: (a) the body when it has just begun to fall and (b) the freebody diagram a little later, after a drag force has developed. (c) The drag force has increased until it balances the gravitational force on the body. The body now falls at its constant terminal speed.

132

CHAPTE R 6 FORCE AN D M OTION—I I

Steve Fitchett/Taxi/Getty Images

Figure 6-7 Sky divers in a horizontal “spread eagle” maximize air drag.

straightening its spine (it then resembles a flying squirrel). These actions increase area A and thus also, by Eq. 6-14, the drag D. The cat begins to slow because now D . Fg (the net force is upward), until a new, smaller vt is reached. The decrease in vt reduces the possibility of serious injury on landing. Just before the end of the fall, when it sees it is nearing the ground, the cat pulls its legs back beneath its body to prepare for the landing. Humans often fall from great heights for the fun of skydiving. However, in April 1987, during a jump, sky diver Gregory Robertson noticed that fellow sky diver Debbie Williams had been knocked unconscious in a collision with a third sky diver and was unable to open her parachute. Robertson, who was well above Williams at the time and who had not yet opened his parachute for the 4 km plunge, reoriented his body head-down so as to minimize A and maximize his downward speed. Reaching an estimated vt of 320 km/h, he caught up with Williams and then went into a horizontal “spread eagle” (as in Fig. 6-7) to increase D so that he could grab her. He opened her parachute and then, after releasing her, his own, a scant 10 s before impact. Williams received extensive internal injuries due to her lack of control on landing but survived.

Sample Problem 6.03 Terminal speed of falling raindrop A raindrop with radius R " 1.5 mm falls from a cloud that is at height h " 1200 m above the ground. The drag coefficient C for the drop is 0.60. Assume that the drop is spherical throughout its fall. The density of water rw is 1000 kg/m3, and the density of air ra is 1.2 kg/m3. (a) As Table 6-1 indicates, the raindrop reaches terminal speed after falling just a few meters. What is the terminal speed?

sity ra and the water density rw, we obtain vt "

2Fg A Cra A

"A

"

8Rrwg 8pR3rwg "A A 3Cra)R2 3Cra

(8)(1.5 ' 10$3 m)(1000 kg/m3)(9.8 m/s2) (3)(0.60)(1.2 kg/m3)

" 7.4 m/s % 27 km/h.

(Answer)

Note that the height of the cloud does not enter into the calculation.

KEY IDEA The drop reaches a terminal speed vt when the gravitational force on it is balanced by the air drag force on it, so its acceleration is zero. We could then apply Newton’s second law and the drag force equation to find vt , but Eq. 6-16 does all that for us. Calculations: To use Eq. 6-16, we need the drop’s effective cross-sectional area A and the magnitude Fg of the gravitational force. Because the drop is spherical, A is the area of a circle (pR2) that has the same radius as the sphere. To find Fg, we use three facts: (1) Fg " mg, where m is the drop’s mass; (2) the (spherical) drop’s volume is V " 43pR3; and (3) the density of the water in the drop is the mass per volume, or rw " m/V. Thus, we find Fg " Vrwg " 43pR3rwg. We next substitute this, the expression for A, and the given data into Eq. 6-16. Being careful to distinguish between the air den-

(b) What would be the drop’s speed just before impact if there were no drag force? KEY IDEA With no drag force to reduce the drop’s speed during the fall, the drop would fall with the constant free-fall acceleration g, so the constant-acceleration equations of Table 2-1 apply. Calculation: Because we know the acceleration is g, the initial velocity v0 is 0, and the displacement x $ x0 is $h, we use Eq. 2-16 to find v: v " 22gh " 2(2)(9.8 m/s2)(1200 m) " 153 m/s % 550 km/h. (Answer) Had he known this, Shakespeare would scarcely have written, “it droppeth as the gentle rain from heaven, upon the place beneath.” In fact, the speed is close to that of a bullet from a large-caliber handgun!

Additional examples, video, and practice available at WileyPLUS

6-3 U N I FOR M CI RCU L AR M OTION

133

6-3 UNIFORM CIRCULAR MOTION Learning Objectives After reading this module, you should be able to. . .

6.06 Sketch the path taken in uniform circular motion and explain the velocity, acceleration, and force vectors (magnitudes and directions) during the motion. 6.07 ldentify that unless there is a radially inward net force (a centripetal force), an object cannot move in circular motion.

6.08 For a particle in uniform circular motion, apply the relationship between the radius of the path, the particle’s speed and mass, and the net force acting on the particle.

Key Ideas ● If a particle moves in a circle or a circular arc of radius R at

constant speed v, the particle is said to be in uniform circular motion. It then has a centripetal acceleration : a with magnitude given by v2 a" . R

● This acceleration is due to a net centripetal force on the

particle, with magnitude given by mv 2 , F" R : where m is the particle’s mass. The vector quantities : a and F are directed toward the center of curvature of the particle’s path.

Uniform Circular Motion From Module 4-5, recall that when a body moves in a circle (or a circular arc) at constant speed v, it is said to be in uniform circular motion. Also recall that the body has a centripetal acceleration (directed toward the center of the circle) of constant magnitude given by a"

v2 R

(centripetal acceleration),

(6-17)

where R is the radius of the circle. Here are two examples: 1. Rounding a curve in a car. You are sitting in the center of the rear seat of a car moving at a constant high speed along a flat road. When the driver suddenly turns left, rounding a corner in a circular arc, you slide across the seat toward the right and then jam against the car wall for the rest of the turn. What is going on? While the car moves in the circular arc, it is in uniform circular motion; that is, it has an acceleration that is directed toward the center of the circle. By Newton’s second law, a force must cause this acceleration. Moreover, the force must also be directed toward the center of the circle. Thus, it is a centripetal force, where the adjective indicates the direction. In this example, the centripetal force is a frictional force on the tires from the road; it makes the turn possible. If you are to move in uniform circular motion along with the car, there must also be a centripetal force on you. However, apparently the frictional force on you from the seat was not great enough to make you go in a circle with the car. Thus, the seat slid beneath you, until the right wall of the car jammed into you. Then its push on you provided the needed centripetal force on you, and you joined the car’s uniform circular motion. 2. Orbiting Earth. This time you are a passenger in the space shuttle Atlantis. As it and you orbit Earth, you float through your cabin. What is going on? Both you and the shuttle are in uniform circular motion and have accelerations directed toward the center of the circle. Again by Newton’s second law, centripetal forces must cause these accelerations. This time the centripetal forces are gravitational pulls (the pull on you and the pull on the shuttle) exerted by Earth and directed radially inward, toward the center of Earth.

134

CHAPTE R 6 FORCE AN D M OTION—I I

In both car and shuttle you are in uniform circular motion, acted on by a centripetal force — yet your sensations in the two situations are quite different. In the car, jammed up against the wall, you are aware of being compressed by the wall. In the orbiting shuttle, however, you are floating around with no sensation of any force acting on you. Why this difference? The difference is due to the nature of the two centripetal forces. In the car, the centripetal force is the push on the part of your body touching the car wall. You can sense the compression on that part of your body. In the shuttle, the centripetal force is Earth’s gravitational pull on every atom of your body. Thus, there is no compression (or pull) on any one part of your body and no sensation of a force acting on you. (The sensation is said to be one of “weightlessness,” but that description is tricky. The pull on you by Earth has certainly not disappeared and, in fact, is only a little less than it would be with you on the ground.) Another example of a centripetal force is shown in Fig. 6-8. There a hockey puck moves around in a circle at constant speed v while tied to a string looped around a central peg. This time the centripetal force is the radially inward pull on the puck from the string. Without that force, the puck would slide off in a straight line instead of moving in a circle. Note again that a centripetal force is not a new kind of force. The name merely indicates the direction of the force. It can, in fact, be a frictional force, a gravitational force, the force from a car wall or a string, or any other force. For any situation: A centripetal force accelerates a body by changing the direction of the body’s velocity without changing the body’s speed.

From Newton’s second law and Eq. 6-17 (a " v 2/R), we can write the magnitude F of a centripetal force (or a net centripetal force) as F"m

v2 R

(magnitude of centripetal force).

(6-18)

Because the speed v here is constant, the magnitudes of the acceleration and the force are also constant. However, the directions of the centripetal acceleration and force are not constant; they vary continuously so as to always point toward the center of the circle. For this reason, the force and acceleration vectors are sometimes drawn along a radial axis r that moves with the body and always extends from the center of the circle to the body, as in Fig. 6-8. The positive direction of the axis is radially outward, but the acceleration and force vectors point radially inward.

v

r

Puck T

String R

The puck moves in uniform circular motion only because of a toward-thecenter force.

Figure 6-8 An overhead view of a hockey puck moving with constant speed v in a circular path of radius R on a horizontal frictionless surface. The centripetal force on the : puck is T , the pull from the string, directed inward along the radial axis r extending through the puck.

6-3 U N I FOR M CI RCU L AR M OTION

135

Checkpoint 2 As every amusement park fan knows, a Ferris wheel is a ride consisting of seats mounted on a tall ring that rotates around a horizontal axis. When you ride in a Ferris wheel at constant speed, what are the directions of your acceleration : a and the : normal force FN on you (from the always upright seat) as you pass through (a) the highest point and (b) the lowest point of the ride? (c) How does the magnitude of the acceleration at the highest point compare with that at the lowest point? (d) How do the magnitudes of the normal force compare at those two points?

Sample Problem 6.04 Vertical circular loop, Diavolo

Photograph reproduced with permission of Circus World Museum

Largely because of riding in cars, you are used to horizontal circular motion. Vertical circular motion would be a novelty. In this sample problem, such motion seems to defy the gravitational force. In a 1901 circus performance, Allo “Dare Devil” Diavolo introduced the stunt of riding a bicycle in a loopthe-loop (Fig. 6-9a). Assuming that the loop is a circle with radius R " 2.7 m, what is the least speed v that Diavolo and his bicycle could have at the top of the loop to remain in contact with it there?

KEY IDEA We can assume that Diavolo and his bicycle travel through the top of the loop as a single particle in uniform circular motion. Thus, at the top, the acceleration : a of this particle must have the magnitude a " v2/R given by Eq. 6-17 and be directed downward, toward the center of the circular loop. Calculations: The forces on the particle when it is at the top of the loop are shown in the free-body diagram of Fig 6-9b. : The gravitational force Fg is downward along a y axis; so is the : normal force FN on the particle from the loop (the loop can push down, not pull up); so also is the centripetal acceleration of the particle. Thus, Newton’s second law for y components (Fnet,y " may) gives us $FN $ Fg " m($a) and

y Diavolo and bicycle a

FN

$

v2 . R

(6-19)

If the particle has the least speed v needed to remain in contact, then it is on the verge of losing contact with the loop (falling away from the loop), which means that FN " 0 at the top of the loop (the particle and loop touch but without any normal force). Substituting 0 for FN in Eq. 6-19, solving for v, and then substituting known values give us

(a)

The normal force is from the overhead loop.

#

$FN $ mg " m $

Fg

The net force provides the toward-the-center acceleration.

(b)

Figure 6-9 (a) Contemporary advertisement for Diavolo and (b) free-body diagram for the performer at the top of the loop.

v " 2gR " 2(9.8 m/s2)(2.7 m) " 5.1 m/s.

(Answer)

Comments: Diavolo made certain that his speed at the top of the loop was greater than 5.1 m/s so that he did not lose contact with the loop and fall away from it. Note that this speed requirement is independent of the mass of Diavolo and his bicycle. Had he feasted on, say, pierogies before his performance, he still would have had to exceed only 5.1 m/s to maintain contact as he passed through the top of the loop.

Additional examples, video, and practice available at WileyPLUS

136

CHAPTE R 6 FORCE AN D M OTION—I I

Sample Problem 6.05 Car in flat circular turn :

Upside-down racing: A modern race car is designed so that the passing air pushes down on it, allowing the car to travel much faster through a flat turn in a Grand Prix without friction failing. This downward push is called negative lift. Can a race car have so much negative lift that it could be driven upside down on a long ceiling, as done fictionally by a sedan in the first Men in Black movie? Figure 6-10a represents a Grand Prix race car of mass m " 600 kg as it travels on a flat track in a circular arc of radius R " 100 m. Because of the shape of the car and the : wings on it, the passing air exerts a negative lift FL downward on the car. The coefficient of static friction between the tires and the track is 0.75. (Assume that the forces on the four tires are identical.)

Radial calculations: The frictional force f s is shown in the free-body diagram of Fig. 6-10b. It is in the negative direction of a radial axis r that always extends from the center of curvature through the car as the car moves. The force produces a centripetal acceleration of magnitude v 2/R. We can relate the force and acceleration by writing Newton’s second law for components along the r axis (Fnet,r " mar) as

(a) If the car is on the verge of sliding out of the turn when its speed is 28.6 m/s, what is the magnitude of the negative : lift FL acting downward on the car?

Vertical calculations: Next, let’s consider the vertical forces : on the car. The normal force FN is directed up, in the positive direction of the y axis in Fig. 6-10b. The gravitational : : : force Fg " mg and the negative lift FL are directed down. The acceleration of the car along the y axis is zero. Thus we can write Newton’s second law for components along the y axis (Fnet,y " may) as

KEY IDEAS 1. A centripetal force must act on the car because the car is moving around a circular arc; that force must be directed toward the center of curvature of the arc (here, that is horizontally). 2. The only horizontal force acting on the car is a frictional force on the tires from the road. So the required centripetal force is a frictional force.

#

$fs " m $

$

v2 . R

Substituting fs,max " msFN for fs leads us to msFN " m

# $

v2 . R

# +v R $ g$ (28.6 m/s) " (600 kg) # $ 9.8 m/s $ (0.75)(100 m) 2

FL " m

s

2

" 663.7 N % 660 N.

y

v

FN

r Center a

FL

Track-level view of the forces

(b)

Normal force: helps support car Car r

fs

R

(a)

(6-22)

Combining results: Now we can combine our results along the two axes by substituting Eq. 6-22 for FN in Eq. 6-21. Doing so and then solving for FL lead to

Friction: toward the center

The toward-thecenter force is the frictional force.

(6-21)

FN $ mg $ FL " 0, FN " mg # FL.

or

3. Because the car is not sliding, the frictional force must : be a static frictional force f s (Fig. 6-10a). 4. Because the car is on the verge of sliding, the magnitude fs is equal to the maximum value fs,max " msFN, where FN : is the magnitude of the normal force FN acting on the car from the track.

fs

(6-20)

Fg

Gravitational force: pulls car downward

Negative lift: presses car downward

Figure 6-10 (a) A race car moves around a flat curved track at constant speed v. The frictional : force f s provides the necessary centripetal force along a radial axis r. (b) A free-body diagram (not to scale) for the car, in the vertical plane containing r.

2

(Answer)

137

6-3 U N I FOR M CI RCU L AR M OTION

(b) The magnitude FL of the negative lift on a car depends on the square of the car’s speed v 2, just as the drag force does (Eq. 6-14). Thus, the negative lift on the car here is greater when the car travels faster, as it does on a straight section of track. What is the magnitude of the negative lift for a speed of 90 m/s?

Substituting our known negative lift of FL " 663.7 N and solving for FL,90 give us FL,90 " 6572 N % 6600 N.

(Answer)

Upside-down racing: The gravitational force is, of course, the force to beat if there is a chance of racing upside down: Fg " mg " (600 kg)(9.8 m/s2)

KEY IDEA

" 5880 N. 2

FL is proportional to v . Calculations: Thus we can write a ratio of the negative lift FL,90 at v " 90 m/s to our result for the negative lift FL at v " 28.6 m/s as FL,90 (90 m/s)2 " . FL (28.6 m/s)2

With the car upside down, the negative lift is an upward force of 6600 N, which exceeds the downward 5880 N. Thus, the car could run on a long ceiling provided that it moves at about 90 m/s (" 324 km/h " 201 mi/h). However, moving that fast while right side up on a horizontal track is dangerous enough, so you are not likely to see upside-down racing except in the movies.

Sample Problem 6.06 Car in banked circular turn This problem is quite challenging in setting up but takes only a few lines of algebra to solve. We deal with not only uniformly circular motion but also a ramp. However, we will not need a tilted coordinate system as with other ramps. Instead we can take a freeze-frame of the motion and work with simply horizontal and vertical axes. As always in this chapter, the starting point will be to apply Newton’s second law, but that will require us to identify the force component that is responsible for the uniform circular motion. Curved portions of highways are always banked (tilted) to prevent cars from sliding off the highway. When a highway is dry, the frictional force between the tires and the road surface may be enough to prevent sliding. When the highway is wet, however, the frictional force may be negligible, and banking is then essential. Figure 6-11a represents a car

v

of mass m as it moves at a constant speed v of 20 m/s around a banked circular track of radius R " 190 m. (It is a normal car, rather than a race car, which means that any vertical force from the passing air is negligible.) If the frictional force from the track is negligible, what bank angle u prevents sliding? KEY IDEAS :

Here the track is banked so as to tilt the normal force FN on : the car toward the center of the circle (Fig. 6-11b). Thus, FN now has a centripetal component of magnitude FNr , directed inward along a radial axis r. We want to find the value of the bank angle u such that this centripetal component keeps the car on the circular track without need of friction.

The toward-thecenter force is due to the tilted track.

y FN FNy θ

r R

a

Track-level view of the forces

Fg

(b)

Car r

FNr

θ

(a)

Tilted normal force supports car and provides the towardthe-center force.

The gravitational force pulls car downward.

Figure 6-11 (a) A car moves around a curved banked road at constant speed v. The bank angle is exaggerated for clarity. (b) A free-body diagram for the car, assuming that friction between tires and road is zero and that the car lacks negative lift. The radially inward component FNr of the normal force (along radial axis r) provides the necessary centripetal force and radial acceleration.

138

CHAPTE R 6 FORCE AN D M OTION—I I

Radial calculation: As Fig. 6-11b shows (and as you : should verify), the angle that force FN makes with the vertical is equal to the bank angle u of the track. Thus, the radial component FNr is equal to FN sin u. We can now write Newton’s second law for components along the r axis (Fnet,r " mar) as

#

$FN sin u " m $

$

v2 . R

(6-23)

We cannot solve this equation for the value of u because it also contains the unknowns FN and m. Vertical calculations: We next consider the forces and acceleration along the y axis in Fig. 6-11b. The vertical component of the normal force is FNy " FN cos u, the gravita: tional force Fg on the car has the magnitude mg, and the acceleration of the car along the y axis is zero. Thus we can

write Newton’s second law for components along the y axis (Fnet,y " may) as FN cos u $ mg " m(0), from which FN cos u " mg.

(6-24)

Combining results: Equation 6-24 also contains the unknowns FN and m, but note that dividing Eq. 6-23 by Eq. 6-24 neatly eliminates both those unknowns. Doing so, replacing (sin u)/(cos u) with tan u, and solving for u then yield v2 1 " tan$1 gR " tan$1

(20 m/s)2 " 123. (9.8 m/s2)(190 m)

(Answer)

Additional examples, video, and practice available at WileyPLUS

Review & Summary :

Friction When a force F tends to slide a body along a surface, a frictional force from the surface acts on the body. The frictional force is parallel to the surface and directed so as to oppose the sliding. It is due to bonding between the atoms on the body and the atoms on the surface, an effect called cold-welding. If the body does not slide, the frictional force is a static : frictional force f s. If there is sliding, the frictional force is a kinetic : frictional force f k. :

1. If a body does not move, the static frictional force f s and the : component of F parallel to the surface are equal in magnitude, : and f s is directed opposite that component. If the component increases, fs also increases. : 2. The magnitude of f s has a maximum value fs,max given by fs,max " msFN,

(6-1)

where ms is the coefficient of static friction and FN is the magni: tude of the normal force. If the component of F parallel to the surface exceeds fs,max, the static friction is overwhelmed and the body slides on the surface. 3. If the body begins to slide on the surface, the magnitude of the frictional force rapidly decreases to a constant value fk given by fk " mkFN,

(6-2)

where mk is the coefficient of kinetic friction.

Drag Force When there is relative motion between air (or :

some other fluid) and a body, the body experiences a drag force D that opposes the relative motion and points in the direction in : which the fluid flows relative to the body. The magnitude of D is

related to the relative speed v by an experimentally determined drag coefficient C according to D " 12C*Av 2,

(6-14)

where r is the fluid density (mass per unit volume) and A is the effective cross-sectional area of the body (the area of a cross section taken perpendicular to the relative velocity : v ).

Terminal Speed When a blunt object has fallen far enough : through air, the magnitudes of the drag force D and the gravita: tional force Fg on the body become equal. The body then falls at a constant terminal speed vt given by vt " A

2Fg CrA

.

(6-16)

Uniform Circular Motion If a particle moves in a circle or a circular arc of radius R at constant speed v, the particle is said to be in uniform circular motion. It then has a centripetal acceleration : a with magnitude given by v2 (6-17) a" . R This acceleration is due to a net centripetal force on the particle, with magnitude given by mv 2 (6-18) F" , R :

where m is the particle’s mass. The vector quantities : a and F are directed toward the center of curvature of the particle’s path. A particle can move in circular motion only if a net centripetal force acts on it.

QU ESTIONS

139

Questions 1 In Fig. 6-12, if the box is stationθ ary and the angle u between the horx : izontal and force F is increased F somewhat, do the following quantiFigure 6-12 Question 1. ties increase, decrease, or remain the same: (a) Fx; (b) fs; (c) FN; (d) fs,max? (e) If, instead, the box is sliding and u is increased, does the magnitude of the frictional force on the box increase, decrease, or remain the same? :

2 Repeat Question 1 for force F angled upward instead of downward as drawn. :

F1 3 In Fig. 6-13, horizontal force F1 of magnitude 10 N is applied to a box on a floor, but the box does not F2 slide. Then, as the magnitude of ver: Figure 6-13 Question 3. tical force F2 is increased from zero, do the following quantities increase, decrease, or stay the same: (a) the magnitude of the frictional : : force f s on the box; (b) the magnitude of the normal force FN on the box from the floor; (c) the maximum value fs,max of the magnitude of the static frictional force on the box? (d) Does the box eventually slide?

4 In three experiments, three different horizontal forces are applied to the same block lying on the same countertop. The force magnitudes are F1 " 12 N, F2 " 8 N, and F3 " 4 N. In each experiment, the block remains stationary in spite of the applied force. Rank the forces according to (a) the magnitude fs of the static frictional force on the block from the countertop and (b) the maximum value fs,max of that force, greatest first. 5 If you press an apple crate against a wall so hard that the crate cannot slide down the wall, what is the direction of (a) the static : frictional force f s on the crate from the wall and (b) the normal : force FN on the crate from the wall? If you increase your push, what happens to (c) fs, (d) FN, and (e) fs,max? 6 In Fig. 6-14, a block of mass m is held stationary on a ramp by the frictional force on : it from the ramp. A force F , directed up the ramp, is then applied to the block and gradually increased in magnitude from zero. During the increase, what happens to the direction and magnitude of the frictional force on the block?

F

θ

Figure 6-14 Question 6. :

7 Reconsider Question 6 but with the force F now directed : down the ramp. As the magnitude of F is increased from zero, what happens to the direction and magnitude of the frictional force on the block? 8 In Fig. 6-15, a horizontal force of 100 N is to be applied to a 10 kg slab that is initially stationary on a frictionless floor, to accelerate the slab. A 10 kg block lies on top of the slab; the coefficient of friction m between the block and the slab is not known, and the Block Slab

Figure 6-15 Question 8.

100 N

block might slip. In fact, the contact between the block and the slab might even be frictionless. (a) Considering that possibility, what is the possible range of values for the magnitude of the slab’s acceleration aslab? (Hint: You don’t need written calculations; just consider extreme values for m.) (b) What is the possible range for the magnitude ablock of the block’s acceleration? 9 Figure 6-16 shows the overhead view of the path of an amusement-park ride that travels at constant speed through five circular arcs of radii R0, 2R0, and 3R0. Rank the arcs according to the magnitude of the centripetal force on a rider traveling in the arcs, greatest first. 2 1

3

5

4

Figure 6-16 Question 9. 10 In 1987, as a Halloween stunt, two sky divers passed a pumpkin back and forth between them while they were in free fall just west of Chicago. The stunt was great fun until the last sky diver with the pumpkin opened his parachute. The pumpkin broke free from his grip, plummeted about 0.5 km, ripped through the roof of a house, slammed into the kitchen floor, and splattered all over the newly remodeled kitchen. From the sky diver’s viewpoint and from the pumpkin’s viewpoint, why did the sky diver lose control of the pumpkin? 11 A person riding a Ferris wheel moves through positions at (1) the top, (2) the bottom, and (3) midheight. If the wheel rotates at a constant rate, rank these three positions according to (a) the magnitude of the person’s centripetal acceleration, (b) the magnitude of the net centripetal force on the person, and (c) the magnitude of the normal force on the person, greatest first. 12 During a routine flight in 1956, test pilot Tom Attridge put his jet fighter into a 203 dive for a test of the aircraft’s 20 mm machine cannons. While traveling faster than sound at 4000 m altitude, he shot a burst of rounds. Then, after allowing the cannons to cool, he shot another burst at 2000 m; his speed was then 344 m/s, the speed of the rounds relative to him was 730 m/s, and he was still in a dive. Almost immediately the canopy around him was shredded and his right air intake was damaged. With little flying capability left, the jet crashed into a wooded area, but Attridge managed to escape the resulting explosion. Explain what apparently happened just after the second burst of cannon rounds. (Attridge has been the only pilot who has managed to shoot himself down.) 13 A box is on a ramp that is at angle u to the horizontal. As u is increased from zero, and before the box slips, do the following increase, decrease, or remain the same: (a) the component of the gravitational force on the box, along the ramp, (b) the magnitude of the static frictional force on the box from the ramp, (c) the component of the gravitational force on the box, perpendicular to the ramp, (d) the magnitude of the normal force on the box from the ramp, and (e) the maximum value fs,max of the static frictional force?

140

CHAPTE R 6 FORCE AN D M OTION—I I

Problems Tutoring problem available (at instructor’s discretion) in WileyPLUS and WebAssign SSM • – •••

Worked-out solution available in Student Solutions Manual

WWW Worked-out solution is at

Number of dots indicates level of problem difficulty

ILW

Interactive solution is at

http://www.wiley.com/college/halliday

Additional information available in The Flying Circus of Physics and at flyingcircusofphysics.com

Module 6-1 Friction •1 The floor of a railroad flatcar is loaded with loose crates having a coefficient of static friction of 0.25 with the floor. If the train is initially moving at a speed of 48 km/h, in how short a distance can the train be stopped at constant acceleration without causing the crates to slide over the floor? •2 In a pickup game of dorm shuffleboard, students crazed by final exams use a broom to propel a calculus book along the dorm hallway. If the 3.5 kg book is pushed from rest through a distance of 0.90 m by the horizontal 25 N force from the broom and then has a speed of 1.60 m/s, what is the coefficient of kinetic friction between the book and floor?

over ground softened by rain. When the desert dried out, the trails behind the stones were hard-baked in place. According to measurements, the coefficient of kinetic friction between the stones and the wet playa ground is about 0.80. What horizontal force must act on a 20 kg stone (a typical mass) to maintain the stone’s motion once a gust has started it moving? (Story continues with Problem 37.)

•3 SSM WWW A bedroom bureau with a mass of 45 kg, including drawers and clothing, rests on the floor. (a) If the coefficient of static friction between the bureau and the floor is 0.45, what is the magnitude of the minimum horizontal force that a person must apply to start the bureau moving? (b) If the drawers and clothing, with 17 kg mass, are removed before the bureau is pushed, what is the new minimum magnitude? •4 A slide-loving pig slides down a certain 353 slide in twice the time it would take to slide down a frictionless 353 slide. What is the coefficient of kinetic friction between the pig and the slide? •5 A 2.5 kg block is initially at rest on a horizontal surface. A : : horizontal force F of magnitude 6.0 N and a vertical force P are then applied to the block (Fig. 6-17). The coefficients of friction for the block and surface are ms " 0.40 and mk " 0.25. Determine the magnitude of the frictional force acting on the block if the magni: tude of P is (a) 8.0 N, (b) 10 N, and (c) 12 N. P F

Figure 6-17 Problem 5. •6 A baseball player with mass m " 79 kg, sliding into second base, is retarded by a frictional force of magnitude 470 N. What is the coefficient of kinetic friction mk between the player and the ground? •7 SSM ILW A person pushes horizontally with a force of 220 N on a 55 kg crate to move it across a level floor. The coefficient of kinetic friction between the crate and the floor is 0.35. What is the magnitude of (a) the frictional force and (b) the acceleration of the crate? •8 The mysterious sliding stones. Along the remote Racetrack Playa in Death Valley, California, stones sometimes gouge out prominent trails in the desert floor, as if the stones had been migrating (Fig. 6-18). For years curiosity mounted about why the stones moved. One explanation was that strong winds during occasional rainstorms would drag the rough stones

Jerry Schad/Photo Researchers, Inc.

Figure 6-18 Problem 8. What moved the stone? •9 A 3.5 kg block is pushed along a horizontal floor by a force : F of magnitude 15 N at an angle the horizontal u " 403 with (Fig. 6-19). The coefficient of kinetic friction between the block and the floor is 0.25. Calculate the magnitudes of (a) the frictional force on the block from the floor and (b) the block’s acceleration.

θ F

Figure 6-19 Problems 9 and 32.

y

F

•10 Figure 6-20 shows an initially θ stationary block of mass m on a x floor. A force of magnitude 0.500mg is then applied at upward angle u " Figure 6-20 Problem 10. 203. What is the magnitude of the acceleration of the block across the floor if the friction coefficients are (a) ms " 0.600 and mk " 0.500 and (b) ms " 0.400 and mk " 0.300? •11 SSM A 68 kg crate is dragged across a floor by pulling on a rope attached to the crate and inclined 158 above the horizontal. (a) If the coefficient of static friction is 0.50, what minimum force magnitude is required from the rope to start the crate moving? (b) If mk " 0.35, what is the magnitude of the initial acceleration of the crate? •12 In about 1915, Henry Sincosky of Philadelphia suspended himself from a rafter by gripping the rafter with the thumb of each

141

PROB LE M S

hand on one side and the fingers on the opposite side (Fig. 6-21). Sincosky’s mass was 79 kg. If the coefficient of static friction between hand and rafter was 0.70, what was the least magnitude of the normal force on the rafter from each thumb or opposite fingers? (After suspending himself, Sincosky chinned himself on the rafter and then moved hand-over-hand along the rafter. If you do not think Sincosky’s grip was remarkable, try to repeat his stunt.) •13 A worker pushes horizontally on a 35 kg crate with a force of magnitude 110 N. The coefficient of static friction between the crate and the floor is 0.37. (a) What is the value of fs,max under the circumstances? (b) Does the crate move? (c) What is the frictional force on the crate from the floor? (d) Suppose, next, that a second worker Figure 6-21 pulls directly upward on the crate to help out. What is the least vertical pull that will allow the Problem 12. first worker’s 110 N push to move the crate? (e) If, instead, the second worker pulls horizontally to help out, what is the least pull that will get the crate moving? •14 Figure 6-22 shows the cross Joint with ice section of a road cut into the side of a mountain. The solid line AA/ repB A' resents a weak bedding plane along F which sliding is possible. Block B directly above the highway is sepaθ rated from uphill rock by a large A crack (called a joint), so that only Figure 6-22 Problem 14. friction between the block and the bedding plane prevents sliding. The mass of the block is 1.8 ' 10 7 kg, the dip angle u of the bedding plane is 243, and the coefficient of static friction between block and plane is 0.63. (a) Show that the block will not slide under these circumstances. (b) Next, water seeps into the joint and ex: pands upon freezing, exerting on the block a force F parallel to AA/. What minimum value of force magnitude F will trigger a slide down the plane? •15 The coefficient of static friction between Teflon and scrambled eggs is about 0.04. What is the smallest angle from the horizontal that will cause the eggs to slide across the bottom of a Teflon-coated skillet? F ••16 A loaded penguin sled weighing 80 N rests on a plane inclined at angle u " 203 to the horizontal (Fig. 6-23). Between the sled and the plane, the coefficient of static θ friction is 0.25, and the coefficient of Figure 6-23 kinetic friction is 0.15. (a) What is : Problems 16 and 22. the least magnitude of the force F, parallel to the plane, that will prevent the sled from slipping down the plane? (b) What is the minimum magnitude F that will start the sled moving up the plane? (c) What value of F is required to x move the sled up the plane at conP stant velocity? :

••17 In Fig. 6-24, a force P acts on a block weighing 45 N. The block is

θ

Figure 6-24 Problem 17.

initially at rest on a plane inclined at angle u " 153 to the horizontal. The positive direction of the x axis is up the plane. Between block and plane, the coefficient of static friction is ms " 0.50 and the coefficient of kinetic friction is mk " 0.34. In unit-vector notation, what is the frictional force on the block from the plane when : P is (a) ($5.0 N)ˆi , (b) ($8.0 N)ˆi , and (c) ($15 N)ˆi ? ••18 You testify as an expert witness in a case involving an accident in which car A slid into the rear of car B, which was stopped at a red light along a road headed down a hill (Fig. 6-25). You find that the slope of the hill is u " 12.03, that the cars were separated by distance d " 24.0 m when the driver of car A put the car into a slide (it lacked any automatic anti-brake-lock system), and that the speed of car A at the onset of braking was v0 " 18.0 m/s. With what speed did car A hit car B if the coefficient of kinetic friction was (a) 0.60 (dry road surface) and (b) 0.10 (road surface covered with wet leaves)? v0

B

A

d

θ

Figure 6-25 Problem 18. y ••19 A 12 N horizontal force F pushes a block weighing 5.0 N F against a vertical wall (Fig. 6-26). x The coefficient of static friction between the wall and the block is 0.60, and the coefficient of kinetic friction Figure 6-26 Problem 19. is 0.40. Assume that the block is not moving initially. (a) Will the block move? (b) In unit-vector notation, what is the force on the block from the wall? :

••20 In Fig. 6-27, a box of mC Cheerios (mass mC " 1.0 kg) and a mW F box of Wheaties (mass mW " 3.0 kg) are accelerated across a horizontal surface by a horizontal force Figure 6-27 Problem 20. : F applied to the Cheerios box. The magnitude of the frictional force on the Cheerios box is 2.0 N, and the magnitude of the frictional force on the Wheaties box is : 4.0 N. If the magnitude of F is 12 N, what is the magnitude of the force on the Wheaties box from the Cheerios box? ••21 An initially stationary box of sand is to be pulled across a floor by means of a cable in which the tension should not exceed 1100 N. The coefficient of static friction between the box and the floor is 0.35. (a) What should be the angle between the cable and the horizontal in order to pull the greatest possible amount of sand, and (b) what is the weight of the sand and box in that situation? ••22 In Fig. 6-23, a sled is held on an inclined plane by a cord pulling directly up the plane. The sled is to be on the verge of moving up the plane. In Fig. 6F 28, the magnitude F required of F2 the cord’s force on the sled is plotted versus a range of values for the coefficient of static fric- F1 tion ms between sled and plane: µs F1 " 2.0 N, F2 " 5.0 N, and m2 " µ2 0 0.50. At what angle u is the plane Figure 6-28 Problem 22. inclined?

CHAPTE R 6 FORCE AN D M OTION—I I

••23 When the three blocks in Fig. 6-29 are released from rest, they accelerate with a magnitude of 0.500 m/s2. Block 1 has mass M, block 2 has 2M, and block 3 has 2M. What is the coefficient of kinetic friction between block 2 and the table? ••24 A 4.10 kg block is pushed along a floor by a constant applied force that is horizontal and has a magnitude of 40.0 N. Figure 6-30 gives the block’s speed v versus time t as the block moves along an x axis on the floor. The scale of the figure’s vertical axis is set by vs " 5.0 m/s. What is the coefficient of kinetic friction between the block and the floor? ••25 SSM WWW Block B in Fig. 6-31 weighs 711 N. The coefficient of static friction between block and table is 0.25; angle u is 303; assume that the cord between B and the knot is horizontal. Find the maximum weight of block A for which the system will be stationary.

2

1

3

Figure 6-29 Problem 23.

C

Frictionless, massless pulley

A

vs

B 0

0.5 t (s)

Figure 6-30 Problem 24.

Knot

θ

B

Figure 6-34 Problem 29.

1.0

A

••26 Figure 6-32 shows three crates being pushed over a concrete Figure 6-31 Problem 25. : floor by a horizontal force F of magnitude 440 N. The masses of the m1 crates are m1 " 30.0 kg, m2 " 10.0 m3 m2 kg, and m3 " 20.0 kg. The coefficient of kinetic friction between the floor F and each of the crates is 0.700. (a) What is the magnitude F32 of the Figure 6-32 Problem 26. force on crate 3 from crate 2? (b) If the crates then slide onto a polished floor, where the coefficient of kinetic friction is less than 0.700, is magnitude F32 more than, less than, or the same as it was when the coefficient was 0.700? ••27 Body A in Fig. 6-33 weighs 102 N, and body B weighs 32 N. The coefficients of friction between A and the incline are ms " 0.56 and mk " 0.25. Angle u is 403. Let the positive direction of an x axis be up the incline. In unit-vector notation, what is the acceleration of A if A is initially (a) at rest, (b) moving up the incline, and (c) moving down the incline?

••29 In Fig. 6-34, blocks A and B have weights of 44 N and 22 N, respectively. (a) Determine the minimum weight of block C to keep A from sliding if ms between A and the table is 0.20. (b) Block C suddenly is lifted off A. What is the acceleration of block A if mk between A and the table is 0.15?

v (m/s)

142

Frictionless, massless pulley

A B

θ

Figure 6-33 Problems 27 and 28.

••28 In Fig. 6-33, two blocks are connected over a pulley. The mass of block A is 10 kg, and the coefficient of kinetic friction between A and the incline is 0.20. Angle u of the incline is 303. Block A slides down the incline at constant speed. What is the mass of block B? Assume the connecting rope has negligible mass. (The pulley’s function is only to redirect the rope.)

••30 A toy chest and its contents have a combined weight of 180 N. The coefficient of static friction between toy chest and floor is 0.42. The child in Fig. 6-35 attempts to move the chest across the floor by pulling on an attached rope. (a) If u is 423, what is the mag: nitude of the force F that the child must exert on the rope to put the chest on the verge of moving? (b) Write an expression for the magnitude F required to put the chest on the verge of moving as a function of the angle u. Determine (c) the value of u for which F is a minimum and (d) that minimum magnitude.

θ

Figure 6-35 Problem 30. ••31 SSM Two blocks, of weights 3.6 N and 7.2 N, are connected by a massless string and slide down a 303 inclined plane. The coefficient of kinetic friction between the lighter block and the plane is 0.10, and the coefficient between the heavier block and the plane is 0.20. Assuming that the lighter block leads, find (a) the magnitude of the acceleration of the blocks and (b) the tension in the taut string. ••32 A block is pushed across a floor by a constant force that is applied at downward angle u (Fig. 6-19). Figure 6-36 gives the acceleration magnitude a versus a range of values for the coefficient of kinetic friction mk between block and floor: a1 " 3.0 m/s2, mk2 " 0.20, and mk3 " 0.40. What is the value of u? a1

a

0

µk2

µk3

–a1

Figure 6-36 Problem 32.

µk

PROB LE M S

•••33 SSM A 1000 kg boat is traveling at 90 km/h when its engine : is shut off. The magnitude of the frictional force f k between boat and water is proportional to the speed v of the boat: fk " 70v, where v is in meters per second and fk is in newtons. Find the time required for the boat to slow to 45 km/h. m2 F •••34 In Fig. 6-37, a slab of mass m m1 " 40 kg rests on a frictionless µ = 0 1 x floor, and a block of mass m2 " 10 Figure 6-37 Problem 34. kg rests on top of the slab. Between block and slab, the coefficient of static friction is 0.60, and the coefficient of kinetic friction is 0.40. A : horizontal force F of magnitude 100 N begins to pull directly on the block, as shown. In unit-vector notation, what are the resulting accelerations of (a) the block and (b) the slab?

m •••35 ILW The two blocks (m " 16 kg and M " 88 kg) in Fig. 6-38 are F not attached to each other. The coefM ficient of static friction between the blocks is ms " 0.38, but the surface Frictionless beneath the larger block is frictionless. What is the minimum magnitude Figure 6-38 Problem 35. : of the horizontal force F required to keep the smaller block from slipping down the larger block?

Module 6-2 The Drag Force and Terminal Speed •36 The terminal speed of a sky diver is 160 km/h in the spreadeagle position and 310 km/h in the nosedive position. Assuming that the diver’s drag coefficient C does not change from one position to the other, find the ratio of the effective cross-sectional area A in the slower position to that in the faster position. ••37 Continuation of Problem 8. Now assume that Eq. 6-14 gives the magnitude of the air drag force on the typical 20 kg stone, which presents to the wind a vertical cross-sectional area of 0.040 m2 and has a drag coefficient C of 0.80. Take the air density to be 1.21 kg/m3, and the coefficient of kinetic friction to be 0.80. (a) In kilometers per hour, what wind speed V along the ground is needed to maintain the stone’s motion once it has started moving? Because winds along the ground are retarded by the ground, the wind speeds reported for storms are often measured at a height of 10 m. Assume wind speeds are 2.00 times those along the ground. (b) For your answer to (a), what wind speed would be reported for the storm? (c) Is that value reasonable for a high-speed wind in a storm? (Story continues with Problem 65.) ••38 Assume Eq. 6-14 gives the drag force on a pilot plus ejection seat just after they are ejected from a plane traveling horizontally at 1300 km/h. Assume also that the mass of the seat is equal to the mass of the pilot and that the drag coefficient is that of a sky diver. Making a reasonable guess of the pilot’s mass and using the appropriate vt value from Table 6-1, estimate the magnitudes of (a) the drag force on the pilot # seat and (b) their horizontal deceleration (in terms of g), both just after ejection. (The result of (a) should indicate an engineering requirement: The seat must include a protective barrier to deflect the initial wind blast away from the pilot’s head.) ••39 Calculate the ratio of the drag force on a jet flying at 1000 km/h at an altitude of 10 km to the drag force on a propdriven transport flying at half that speed and altitude. The density

143

of air is 0.38 kg/m3 at 10 km and 0.67 kg/m3 at 5.0 km. Assume that the airplanes have the same effective cross-sectional area and drag coefficient C. ••40 In downhill speed skiing a skier is retarded by both the air drag force on the body and the kinetic frictional force on the skis. (a) Suppose the slope angle is u " 40.03, the snow is dry snow with a coefficient of kinetic friction mk " 0.0400, the mass of the skier and equipment is m " 85.0 kg, the cross-sectional area of the (tucked) skier is A " 1.30 m2 , the drag coefficient is C " 0.150, and the air density is 1.20 kg/m3. (a) What is the terminal speed? (b) If a skier can vary C by a slight amount dC by adjusting, say, the hand positions, what is the corresponding variation in the terminal speed? Module 6-3 Uniform Circular Motion •41 A cat dozes on a stationary merry-go-round in an amusement park, at a radius of 5.4 m from the center of the ride. Then the operator turns on the ride and brings it up to its proper turning rate of one complete rotation every 6.0 s. What is the least coefficient of static friction between the cat and the merry-go-round that will allow the cat to stay in place, without sliding (or the cat clinging with its claws)? •42 Suppose the coefficient of static friction between the road and the tires on a car is 0.60 and the car has no negative lift. What speed will put the car on the verge of sliding as it rounds a level curve of 30.5 m radius? •43 ILW What is the smallest radius of an unbanked (flat) track around which a bicyclist can travel if her speed is 29 km/h and the ms between tires and track is 0.32? •44 During an Olympic bobsled run, the Jamaican team makes a turn of radius 7.6 m at a speed of 96.6 km/h. What is their acceleration in terms of g? ••45 SSM ILW A student of weight 667 N rides a steadily rotating Ferris wheel (the student sits upright). At the : highest point, the magnitude of the normal force FN on the student from the seat is 556 N. (a) Does the student feel “light” or “heavy” : there? (b) What is the magnitude of FN at the lowest point? If the wheel’s speed is doubled, what is the magnitude FN at the (c) highest and (d) lowest point? ••46 A police officer in hot pursuit drives her car through a circular turn of radius 300 m with a constant speed of 80.0 km/h. Her mass is 55.0 kg.What are (a) the magnitude and (b) the angle (relative to vertical) of the net force of the officer on the car seat? (Hint: Consider both horizontal and vertical forces.) ••47 A circular-motion addict of mass 80 kg rides a Ferris wheel around in a vertical circle of radius 10 m at a constant speed of 6.1 m/s. (a) What is the period of the motion? What is the magnitude of the normal force on the addict from the seat when both go through (b) the highest point of the circular path and (c) the lowest point? ••48 A roller-coaster car at an amusement park has a mass of 1200 kg when fully loaded with passengers. As the car passes over the top of a circular hill of radius 18 m, assume that its speed is not changing. At the top of the hill, what are the (a) magnitude FN and (b) direction (up or down) of the normal force on the car from the track if the car’s speed is v " 11 m/s? What are (c) FN and (d) the direction if v " 14 m/s?

144

CHAPTE R 6 FORCE AN D M OTION—I I

••49 In Fig. 6-39, a car is driven at constant speed over a circular hill and then into a circular valley with the same radius. At the top of the hill, the normal force on the driver from the car seat is 0. The driver’s mass is 70.0 kg. What is the magnitude of the normal force on the driver from the seat when the car passes through the bottom of the valley? Radius Radius

Figure 6-39 Problem 49. ••50 An 85.0 kg passenger is made to move along a circular path of radius r " 3.50 m in uniform circular motion. (a) Figure 6-40a is a plot of the required magnitude F of the net centripetal force for a range of possible values of the passenger’s speed v. What is the plot’s slope at v " 8.30 m/s? (b) Figure 6-40b is a plot of F for a range of possible values of T, the period of the motion. What is the plot’s slope at T " 2.50 s? F

F

v (a)

T

dv in the speed with r held constant, and (c) a variation dT in the period with r held constant? Bolt

••55 A bolt is threaded onto one end of a thin horizontal rod, and Rod the rod is then rotated horizontally about its other end. An engineer monitors the motion by flashing a Strobed positions strobe lamp onto the rod and bolt, adjusting the strobe rate until the Figure 6-42 Problem 55. bolt appears to be in the same eight places during each full rotation of the rod (Fig. 6-42). The strobe rate is 2000 flashes per second; the bolt has mass 30 g and is at radius 3.5 cm. What is the magnitude of the force on the bolt from the rod? ••56 A banked circular highway curve is designed for traffic moving at 60 km/h. The radius of the curve is 200 m. Traffic is moving along the highway at 40 km/h on a rainy day. What is the minimum coefficient of friction between tires and road that will allow cars to take the turn without sliding off the road? (Assume the cars do not have negative lift.) ••57 A puck of mass m " 1.50 kg slides in a circle of radius r " 20.0 cm on a frictionless table while attached to a hanging cylinder of mass M " 2.50 kg by means of a cord that extends through a hole in the table (Fig. 6-43). What speed keeps the cylinder at rest?

m

r

(b)

Figure 6-40 Problem 50. ••51 SSM WWW An airplane is flying in a horizontal circle at a speed of 480 km/h (Fig. 6-41). If its wings are tilted at angle u " 403 to the horizontal, what is the radius of the circle in which the plane is flying? Assume that the required force is provided entirely by an “aerodynamic lift” that is perpendicular to the wing surface.

M

Figure 6-43 Problem 57.

θ

••52 An amusement park Figure 6-41 Problem 51. ride consists of a car moving in a vertical circle on the end of a rigid boom of negligible mass. The combined weight of the car and riders is 5.0 kN, and the circle’s radius is 10 m. At the top of the circle, what are the (a) magnitude FB and (b) direction (up or down) of the force on the car from the boom if the car’s speed is v " 5.0 m/s? What are (c) FB and (d) the direction if v " 12 m/s? ••53 An old streetcar rounds a flat corner of radius 9.1 m, at 16 km/h. What angle with the vertical will be made by the loosely hanging hand straps? ••54 In designing circular rides for amusement parks, mechanical engineers must consider how small variations in certain parameters can alter the net force on a passenger. Consider a passenger of mass m riding around a horizontal circle of radius r at speed v. What is the variation dF in the net force magnitude for (a) a variation dr in the radius with v held constant, (b) a variation

••58 Brake or turn? Figure 644 depicts an overhead view of a car’s path as the car travels toward a wall. Assume that the driver begins to brake the car when the distance to Car path the wall is d " 107 m, and take the car’s mass as m " 1400 kg, its initial d speed as v0 " 35 m/s, and the coeffiWall cient of static friction as ms " 0.50. Figure 6-44 Assume that the car’s weight is disProblem 58. tributed evenly on the four wheels, even during braking. (a) What magnitude of static friction is needed (between tires and road) to stop the car just as it reaches the wall? (b) What is the maximum possible static friction fs,max? (c) If the coefficient of kinetic friction between the (sliding) tires and the road is mk " 0.40, at what speed will the car hit the wall? To avoid the crash, a driver could elect to turn the car so that it just barely misses the wall, as shown in the figure. (d) What magnitude of frictional force would be required to keep the car in a circular path of radius d and at the given speed v0, so that the car moves in a quarter circle and then parallel to the wall? (e) Is the required force less than fs,max so that a circular path is possible?

PROB LE M S

•••59 SSM ILW In Fig. 6-45, a 1.34 kg ball is connected by means of two massless strings, each of length L " 1.70 m, to a vertical, rotating rod. The strings are tied to the rod with separation d " 1.70 m and are taut. The tension in the upper string is 35 N. What are the (a) tension in the lower : string, (b) magnitude of the net force Fnet on the ball, and (c) speed of the ball? (d) : What is the direction of Fnet?

L d L

63 In Fig. 6-49, a 49 kg rock climber is climbing a “chimney.” The coefficient of static friction between her shoes and the rock is 1.2; between her back and the rock is 0.80. She has reduced her push against the rock until her back and her shoes are on the verge of slipping. (a) Draw a free-body diagram of her. (b) What is the magnitude of her push against the rock? (c) What fraction of her weight is supported by the frictional force on her shoes?

Rotating rod

Additional Problems Figure 6-45 60 In Fig. 6-46, a box of ant aunts (total Problem 59. mass m1 " 1.65 kg) and a box of ant uncles (total mass m2 " 3.30 kg) slide down an inclined plane while attached by a massless rod parallel to the plane. The angle of incline is u " 30.0°. The coefficient of kinetic friction between the aunt box and the incline is m1 " 0.226; that between the uncle box and the incline is m2 " 0.113. Compute (a) the tension in the rod and (b) the magnitude of the common acceleration of the two boxes. (c) How would the answers to (a) and (b) change if the uncles trailed the aunts?

Figure 6-49 Problem 63. 64 A high-speed railway car goes around a flat, horizontal circle of radius 470 m at a constant speed. The magnitudes of the horizontal and vertical components of the force of the car on a 51.0 kg passenger are 210 N and 500 N, respectively. (a) What is the magnitude of the net force (of all the forces) on the passenger? (b) What is the speed of the car?

m1

m2

θ

Figure 6-46 Problem 60. 61 SSM A block of mass mt " 4.0 kg is put on top of a block of mass mb " 5.0 kg. To cause the top block to slip on the bottom one while the bottom one is held fixed, a horizontal force of at least 12 N must be applied to the top block. The assembly of blocks is now placed on a horizontal, frictionless table (Fig. 6-47). Find the mag: nitudes of (a) the maximum horizontal force F that can be applied to the lower block so that the blocks will move together and (b) the resulting acceleration of the blocks. mt mb

145

F

Figure 6-47 Problem 61. 62 A 5.00 kg stone is rubbed across the horizontal ceiling of a cave passageway (Fig. 6-48). If the coefficient of kinetic friction is 0.65 and the force applied to the stone is angled at u " 70.0°, what must the magnitude of the force be for the stone to move at constant velocity? F

θ Stone

Figure 6-48 Problem 62.

65 Continuation of Problems 8 and 37. Another explanation is that the stones move only when the water dumped on the playa during a storm freezes into a large, thin sheet of ice. The stones are trapped in place in the ice. Then, as air flows across the ice during a wind, the air-drag forces on the ice and stones move them both, with the stones gouging out the trails. The magnitude of the air-drag force on this horizontal “ice sail” is given by Dice " 4CicerAicev 2, where Cice is the drag coefficient (2.0 ' 10$3), r is the air density (1.21 kg/m3), Aice is the horizontal area of the ice, and v is the wind speed along the ice. Assume the following: The ice sheet measures 400 m by 500 m by 4.0 mm and has a coefficient of kinetic friction of 0.10 with the ground and a density of 917 kg/m3. Also assume that 100 stones identical to the one in Problem 8 are trapped in the ice. To maintain the motion of the sheet, what are the required wind speeds (a) near the sheet and (b) at a height of 10 m? (c) Are these reasonable values for high-speed winds in a storm? 66 In Fig. 6-50, block 1 of mass m1 " 2.0 kg and block 2 of mass m2 " 3.0 kg are connected by a string of negligible mass and are initially held in place. Block 2 is on a frictionless surface tilted at u " 303. The coefficient of kinetic friction between block 1 and the horizontal surface is 0.25. The pulley has negligible mass and friction. Once they are released, the blocks move. What then is the tension in the string? m1 m2

θ

Figure 6-50 Problem 66.

146

CHAPTE R 6 FORCE AN D M OTION—I I

67 In Fig. 6-51, a crate slides down an inclined right-angled trough. The coefficient of kinetic friction between the crate and the trough is mk. What is the acceleration of the crate in terms of mk, u, and g?

72 A box of canned goods slides down a ramp from street level into the basement of a grocery store with acceleration 0.75 m/s2 directed down the ramp. The ramp makes an angle of 403 with the horizontal. What is the coefficient of kinetic friction between the box and the ramp?

90°

θ

Figure 6-51 Problem 67. 68 Engineering a highway curve. If a car goes through a curve too fast, the car tends to slide out of the curve. For a banked curve with friction, a frictional force acts on a fast car to oppose the tendency to slide out of the curve; the force is directed down the bank (in the direction water would drain). Consider a circular curve of radius R " 200 m and bank angle u, where the coefficient of static friction between tires and pavement is ms. A car (without negative lift) is driven around the curve as shown in Fig. 6-11. (a) Find an expression for the car speed vmax that puts the car on the verge of sliding out. (b) On the same graph, plot vmax versus angle u for the range 03 to 503, first for ms " 0.60 (dry pavement) and then for ms " 0.050 (wet or icy pavement). In kilometers per hour, evaluate vmax for a bank angle of u " 103 and for (c) ms " 0.60 and (d) ms " 0.050. (Now you can see why accidents occur in highway curves when icy conditions are not obvious to drivers, who tend to drive at normal speeds.) :

69 A student, crazed by final exams, uses a force P of magnitude 80 N and angle u " 703 to push a 5.0 kg block across the ceiling of his room (Fig. 6-52). If the coefficient of kinetic friction between the block and the ceiling is 0.40, what is the magnitude of the block’s acceleration? P

θ

Figure 6-52 Problem 69. 70 Figure 6-53 shows a conical pendulum, in which the bob (the small object at the lower end of the cord) moves in a horizontal circle at constant speed. (The cord sweeps out a cone as the bob rotates.) The bob has a mass of 0.040 kg, the string has length L " 0.90 m and negligible mass, and the bob follows a circular path of circumference 0.94 m. What are (a) the tension in the string and (b) the period of the motion? 71 An 8.00 kg block of steel is at rest on a horizontal table. The coefficient of static friction between the block and the table is 0.450. A force is to be applied to the block.

To three significant figures, what is the magnitude of that applied force if it puts the block on the verge of sliding when the force is directed (a) horizontally, (b) upward at 60.03 from the horizontal, and (c) downward at 60.0° from the horizontal?

73 In Fig. 6-54, the coefficient of kinetic friction between the block and inclined plane is 0.20, and angle u is 603. What are the (a) magnitude a and (b) direction (up or down the plane) of the block’s acceleration if the block is sliding down the plane? What are (c) a and (d) the direction if the block is sent sliding up the plane?

θ

Figure 6-54 Problem 73.

74 A 110 g hockey puck sent sliding over ice is stopped in 15 m by the frictional force on it from the ice. (a) If its initial speed is 6.0 m/s, what is the magnitude of the frictional force? (b) What is the coefficient of friction between the puck and the ice? 75 A locomotive accelerates a 25-car train along a level track. Every car has a mass of 5.0 ' 10 4 kg and is subject to a frictional force f " 250v, where the speed v is in meters per second and the force f is in newtons. At the instant when the speed of the train is 30 km/h, the magnitude of its acceleration is 0.20 m/s2. (a) What is the tension in the coupling between the first car and the locomotive? (b) If this tension is equal to the maximum force the locomotive can exert on the train, what is the steepest grade up which the locomotive can pull the train at 30 km/h? 76 A house is built on the top of a hill with a nearby slope at angle u " 453 (Fig. 6-55). An engineering study indicates that the slope angle should be reduced because the top layers of soil along the slope might slip past the lower layers. If the coefficient of static friction between two such layers is 0.5, what is the least angle f through which the present slope should be reduced to prevent slippage?

New slope Original slope

φ θ

Figure 6-55 Problem 76.

Cord

77 What is the terminal speed of a 6.00 kg spherical ball that has a radius of 3.00 cm and a drag coefficient of 1.60? The density of the air through which the ball falls is 1.20 kg/m3.

L

Bob

Figure 6-53 Problem 70.

r

78 A student wants to determine the coefficients of static friction and kinetic friction between a box and a plank. She places the box on the plank and gradually raises one end of the plank. When the angle of inclination with the horizontal reaches 303, the box starts to slip, and it then slides 2.5 m down the plank in 4.0 s at constant acceleration. What are (a) the coefficient of static friction and (b) the coefficient of kinetic friction between the box and the plank?

PROB LE M S

79 SSM Block A in Fig. 6-56 has mass mA " 4.0 kg, and block B has mass mB " 2.0 kg. The coefficient of kinetic friction between block B and the horizontal plane is mk " 0.50.The inclined plane is frictionless and at angle u " 30°. The pulley serves only to change the direction of the cord connecting the blocks. The cord has negligible mass. Find (a) the tension in the cord and (b) the magnitude of the acceleration of the blocks. Frictionless, massless pulley

A

mB B

µk

mA

θ

Figure 6-56 Problem 79. 80 Calculate the magnitude of the drag force on a missile 53 cm in diameter cruising at 250 m/s at low altitude, where the density of air is 1.2 kg/m3. Assume C " 0.75. 81 SSM A bicyclist travels in a circle of radius 25.0 m at a constant speed of 9.00 m/s. The bicycle – rider mass is 85.0 kg. Calculate the magnitudes of (a) the force of friction on the bicycle from the road and (b) the net force on the bicycle from the road. 82 In Fig. 6-57, a stuntman drives a car (without negative lift) over the top of a hill, the cross section of R which can be approximated by a circle of radius R " 250 m. What is Figure 6-57 Problem 82. the greatest speed at which he can drive without the car leaving the road at the top of the hill? 83 You must push a crate across a floor to a docking bay. The crate weighs 165 N. The coefficient of static friction between crate and floor is 0.510, and the coefficient of kinetic friction is 0.32. Your force on the crate is directed horizontally. (a) What magnitude of your push puts the crate on the verge of sliding? (b) With what magnitude must you then push to keep the crate moving at a constant velocity? (c) If, instead, you then push with the same magnitude as the answer to (a), what is the magnitude of the crate’s acceleration? :

y

84 In Fig. 6-58, force F is applied θ to a crate of mass m on a floor x where the coefficient of static friction between crate and floor is ms. F Angle u is initially 03 but is graduFigure 6-58 Problem 84. ally increased so that the force vector rotates clockwise in the figure. During the rotation, the magnitude F of the force is continuously adjusted so that the crate is always on the verge of sliding. For ms " 0.70, (a) plot the ratio F/mg versus u and (b) determine the angle uinf at which the ratio approaches an infinite value. (c) Does lubricating the floor increase or decrease uinf, or is the value unchanged? (d) What is uinf for ms " 0.60? 85 In the early afternoon, a car is parked on a street that runs down a steep hill, at an angle of 35.03 relative to the horizontal. Just then the coefficient of static friction between the tires and the street surface is 0.725. Later, after nightfall, a sleet storm hits the area, and the coefficient decreases due to both the ice and a chemi-

147

cal change in the road surface because of the temperature decrease. By what percentage must the coefficient decrease if the car is to be in danger of sliding down the street? 86 A sling-thrower puts a stone (0.250 kg) in the sling’s pouch (0.010 kg) and then begins to make the stone and pouch move in a vertical circle of radius 0.650 m. The cord between the pouch and the person’s hand has negligible mass and will break when the tension in the cord is 33.0 N or more. Suppose the slingthrower could gradually increase the speed of the stone. (a) Will the breaking occur at the lowest point of the circle or at the highest point? (b) At what speed of the stone will that breaking occur? 87 SSM A car weighing 10.7 kN and traveling at 13.4 m/s without negative lift attempts to round an unbanked curve with a radius of 61.0 m. (a) What magnitude of the frictional force on the tires is required to keep the car on its circular path? (b) If the coefficient of static friction between the tires and the road is 0.350, is the attempt at taking the curve successful? F 88 In Fig. 6-59, block 1 of mass θ m1 " 2.0 kg and block 2 of mass m1 m2 m2 " 1.0 kg are connected by a string of negligible mass. Block 2 is : Figure 6-59 Problem 88. pushed by force F of magnitude 20 N and angle u " 353. The coefficient of kinetic friction between each block and the horizontal surface is 0.20. What is the tension in the string?

89 SSM A filing cabinet weighing 556 N rests on the floor. The coefficient of static friction between it and the floor is 0.68, and the coefficient of kinetic friction is 0.56. In four different attempts to move it, it is pushed with horizontal forces of magnitudes (a) 222 N, (b) 334 N, (c) 445 N, and (d) 556 N. For each attempt, calculate the magnitude of the frictional force on it from the floor. (The cabinet is initially at rest.) (e) In which of the attempts does the cabinet move? 90 In Fig. 6-60, a block weighing 22 N is held at : rest against a vertical wall by a horizontal force F of magnitude 60 N. The coefficient of static friction F between the wall and the block is 0.55, and the coefficient of kinetic friction between them is 0.38. In : six experiments, a second force P is applied to the block and directed parallel to the wall with these magnitudes and directions: (a) 34 N, up, (b) 12 N, up, (c) 48 N, up, (d) 62 N, up, (e) 10 N, down, and Figure 6-60 (f) 18 N, down. In each experiment, what is the Problem 90. magnitude of the frictional force on the block? In which does the block move (g) up the wall and (h) down the wall? (i) In which is the frictional force directed down the wall? 91 SSM A block slides with constant velocity down an inclined plane that has slope angle 1. The block is then projected up the same plane with an initial speed v0. (a) How far up the plane will it move before coming to rest? (b) After the block comes to rest, will it slide down the plane again? Give an argument to back your answer. 92 A circular curve of highway is designed for traffic moving at 60 km/h. Assume the traffic consists of cars without negative lift. (a) If the radius of the curve is 150 m, what is the correct angle of banking of the road? (b) If the curve were not banked, what would be the minimum coefficient of friction between tires and road that would keep traffic from skidding out of the turn when traveling at 60 km/h?

148

CHAPTE R 6 FORCE AN D M OTION—I I

93 A 1.5 kg box is initially at rest on a horizontal surface when at : t " 0 a horizontal force F " (1.8t)iˆ N (with t in seconds) is applied to the box. The acceleration of the box as a function of time t is given by : a " 0 for 0 0 t 0 2.8 s and : a " (1.2t $ 2.4)iˆ m/s2 for t . 2.8 s. (a) What is the coefficient of static friction between the box and the surface? (b) What is the coefficient of kinetic friction between the box and the surface? 94 A child weighing 140 N sits at rest at the top of a playground slide that makes an angle of 253 with the horizontal. The child keeps from sliding by holding onto the sides of the slide. After letting go of the sides, the child has a constant acceleration of 0.86 m/s2 (down the slide, of course). (a) What is the coefficient of kinetic friction between the child and the slide? (b) What maximum and minimum values for the coefficient of static friction between the child and the slide are consistent with the information given here? 95 In Fig. 6-61 a fastidious worker pushes directly along the handle of : a mop with a force F . The handle is at an angle u with the vertical, and ms and mk are the coefficients of static and kinetic friction between the head of the mop and the floor. F θ Ignore the mass of the handle and assume that all the mop’s mass m is in its head. (a) If the mop head Figure 6-61 Problem 95. moves along the floor with a constant velocity, then what is F? (b) Show that if u is less than a cer: tain value u0, then F (still directed along the handle) is unable to move the mop head. Find u0. 96 A child places a picnic basket on the outer rim of a merrygo-round that has a radius of 4.6 m and revolves once every 30 s. (a) What is the speed of a point on that rim? (b) What is the lowest value of the coefficient of static friction between basket and merry-go-round that allows the basket to stay on the ride? 97 SSM A warehouse worker exerts a constant horizontal force of magnitude 85 N on a 40 kg box that is initially at rest on the horizontal floor of the warehouse. When the box has moved a distance of 1.4 m, its speed is 1.0 m/s. What is the coefficient of kinetic friction between the box and the floor? 98 In Fig. 6-62, a 5.0 kg block is sent sliding up a plane inclined at : u " 37° while a horizontal force F of magnitude 50 N acts on it. The coefficient of kinetic friction between block and plane is 0.30. What are the (a) magnitude and (b) direction (up or down the plane) of the block’s acceleration? The block’s initial speed is 4.0 m/s. (c) How far up the plane does the block go? (d) When it reaches its highest point, does it remain at rest or slide back down the plane?

F

θ

Figure 6-62 Problem 98.

99 An 11 kg block of steel is at rest on a horizontal table. The coefficient of static friction between block and table is 0.52. (a) What is the magnitude of the horizontal force that will put the block on the verge of moving? (b) What is the magnitude of a force acting upward 603 from the horizontal that will put the block on the verge of moving? (c) If the force acts downward at 603 from the horizontal, how large can its magnitude be without causing the block to move? 100 A ski that is placed on snow will stick to the snow. However, when the ski is moved along the snow, the rubbing warms and partially melts the snow, reducing the coefficient of kinetic friction and promoting sliding. Waxing the ski makes it water repellent and reduces friction with the resulting layer of water. A magazine reports that a new type of plastic ski is especially water repellent and that, on a gentle 200 m slope in the Alps, a skier reduced his top-to-bottom time from 61 s with standard skis to 42 s with the new skis. Determine the magnitude of his average acceleration with (a) the standard skis and (b) the new skis. Assuming a 3.03 slope, compute the coefficient of kinetic friction for (c) the standard skis and (d) the new skis. 101 Playing near a road construction site, a child falls over a barrier and down onto a dirt slope that is angled downward at 353 to the horizontal. As the child slides down the slope, he has an acceleration that has a magnitude of 0.50 m/s2 and that is directed up the slope. What is the coefficient of kinetic friction between the child and the slope? 102 A 100 N force, directed at an angle u above a horizontal floor, is applied to a 25.0 kg chair sitting on the floor. If u " 03, what are (a) the horizontal component Fh of the applied force and (b) the magnitude FN of the normal force of the floor on the chair? If u " 30.03, what are (c) Fh and (d) FN? If u " 60.03, what are (e) Fh and (f) FN? Now assume that the coefficient of static friction between chair and floor is 0.420. Does the chair slide or remain at rest if u is (g) 03, (h) 30.03, and (i) 60.03? 103 A certain string can withstand a maximum tension of 40 N without breaking. A child ties a 0.37 kg stone to one end and, holding the other end, whirls the stone in a vertical circle of radius 0.91 m, slowly increasing the speed until the string breaks. (a) Where is the stone on its path when the string breaks? (b) What is the speed of the stone as the string breaks? 104 A four-person bobsled (total mass " 630 kg) comes down a straightaway at the start of a bobsled run.The straightaway is 80.0 m long and is inclined at a constant angle of 10.23 with the horizontal. Assume that the combined effects of friction and air drag produce on the bobsled a constant force of 62.0 N that acts parallel to the incline and up the incline. Answer the following questions to three significant digits. (a) If the speed of the bobsled at the start of the run is 6.20 m/s, how long does the bobsled take to come down the straightaway? (b) Suppose the crew is able to reduce the effects of friction and air drag to 42.0 N. For the same initial velocity, how long does the bobsled now take to come down the straightaway? 105 As a 40 N block slides down a plane that is inclined at 253 to the horizontal, its acceleration is 0.80 m/s2, directed up the plane. What is the coefficient of kinetic friction between the block and the plane?

C

H

A

P

T

E

R

7

Kinetic Energy and Work 7-1 KINETIC ENERGY Learning Objectives After reading this module, you should be able to . . .

7.01 Apply the relationship between a particle’s kinetic energy, mass, and speed.

7.02 Identify that kinetic energy is a scalar quantity.

Key Idea ●

The kinetic energy K associated with the motion of a particle of mass m and speed v, where v is well below the speed of light, is K " 12 mv2

(kinetic energy).

What Is Physics? One of the fundamental goals of physics is to investigate something that everyone talks about: energy. The topic is obviously important. Indeed, our civilization is based on acquiring and effectively using energy. For example, everyone knows that any type of motion requires energy: Flying across the Pacific Ocean requires it. Lifting material to the top floor of an office building or to an orbiting space station requires it. Throwing a fastball requires it. We spend a tremendous amount of money to acquire and use energy. Wars have been started because of energy resources. Wars have been ended because of a sudden, overpowering use of energy by one side. Everyone knows many examples of energy and its use, but what does the term energy really mean?

What Is Energy? The term energy is so broad that a clear definition is difficult to write. Technically, energy is a scalar quantity associated with the state (or condition) of one or more objects. However, this definition is too vague to be of help to us now. A looser definition might at least get us started. Energy is a number that we associate with a system of one or more objects. If a force changes one of the objects by, say, making it move, then the energy number changes. After countless experiments, scientists and engineers realized that if the scheme by which we assign energy numbers is planned carefully, the numbers can be used to predict the outcomes of experiments and, even more important, to build machines, such as flying machines. This success is based on a wonderful property of our universe: Energy can be transformed from one type to another and transferred from one object to another, but the total amount is always the same (energy is conserved). No exception to this principle of energy conservation has ever been found. Money. Think of the many types of energy as being numbers representing money in many types of bank accounts. Rules have been made about what such money numbers mean and how they can be changed. You can transfer money numbers from one account to another or from one system to another, perhaps 149

150

CHAPTE R 7 KI N ETIC E N E RGY AN D WOR K

electronically with nothing material actually moving. However, the total amount (the total of all the money numbers) can always be accounted for: It is always conserved. In this chapter we focus on only one type of energy (kinetic energy) and on only one way in which energy can be transferred (work).

Kinetic Energy Kinetic energy K is energy associated with the state of motion of an object. The faster the object moves, the greater is its kinetic energy. When the object is stationary, its kinetic energy is zero. For an object of mass m whose speed v is well below the speed of light, K " 12 mv2

(kinetic energy).

(7-1)

For example, a 3.0 kg duck flying past us at 2.0 m/s has a kinetic energy of 6.0 kg !m2/s2; that is, we associate that number with the duck’s motion. The SI unit of kinetic energy (and all types of energy) is the joule (J), named for James Prescott Joule, an English scientist of the 1800s and defined as 1 joule " 1 J " 1 kg !m2/s2.

(7-2)

Thus, the flying duck has a kinetic energy of 6.0 J.

Sample Problem 7.01 Kinetic energy, train crash In 1896 in Waco, Texas, William Crush parked two locomotives at opposite ends of a 6.4-km-long track, fired them up, tied their throttles open, and then allowed them to crash head-on at full speed (Fig. 7-1) in front of 30,000 spectators. Hundreds of people were hurt by flying debris; several were killed. Assuming each locomotive weighed 1.2 ' 10 6 N and its acceleration was a constant 0.26 m/s2, what was the total kinetic energy of the two locomotives just before the collision?

We can find the mass of each locomotive by dividing its given weight by g: 1.2 ' 10 6 N " 1.22 ' 10 5 kg. 9.8 m/s2 Now, using Eq. 7-1, we find the total kinetic energy of the two locomotives just before the collision as m"

K " 2(12 mv2) " (1.22 ' 10 5 kg)(40.8 m/s)2 " 2.0 ' 10 8 J. (Answer) This collision was like an exploding bomb.

KEY IDEAS (1) We need to find the kinetic energy of each locomotive with Eq. 7-1, but that means we need each locomotive’s speed just before the collision and its mass. (2) Because we can assume each locomotive had constant acceleration, we can use the equations in Table 2-1 to find its speed v just before the collision. Calculations: We choose Eq. 2-16 because we know values for all the variables except v: v2 " v20 # 2a(x $ x0). With v0 " 0 and x $ x0 " 3.2 ' 10 3 m (half the initial separation), this yields v2 " 0 # 2(0.26 m/s2)(3.2 ' 10 3 m), or

v " 40.8 m/s " 147 km/h.

Courtesy Library of Congress

Figure 7-1 The aftermath of an 1896 crash of two locomotives.

Additional examples, video, and practice available at WileyPLUS

7-2 WOR K AN D KI N ETIC E N E RGY

151

7-2 WORK AND KINETIC ENERGY Learning Objectives After reading this module, you should be able to . . .

7.03 Apply the relationship between a force (magnitude and direction) and the work done on a particle by the force when the particle undergoes a displacement. 7.04 Calculate work by taking a dot product of the force vector and the displacement vector, in either magnitude-angle or unit-vector notation.

7.05 If multiple forces act on a particle, calculate the net work done by them. 7.06 Apply the work–kinetic energy theorem to relate the work done by a force (or the net work done by multiple forces) and the resulting change in kinetic energy.

Key Ideas ● Work W is energy transferred to or from an object via a force acting on the object. Energy transferred to the object is positive work, and from the object, negative work. : ● The work done on a particle by a constant force F during : displacement d is : :

W " Fd cos f " F " d

(work, constant force), :

in which f is the constant angle between the directions of F : and d . : : ● Only the component of F that is along the displacement d can do work on the object.

● When two or more forces act on an object, their net work is the sum of the individual works done by the forces, which is also equal to the work that would be done on the object by : the net force F net of those forces. ● For a particle, a change -K in the kinetic energy equals the net work W done on the particle: -K " Kf $ Ki " W (work – kinetic energy theorem),

in which Ki is the initial kinetic energy of the particle and Kf is the kinetic energy after the work is done. The equation rearranged gives us Kf " Ki # W.

Work If you accelerate an object to a greater speed by applying a force to the object, you increase the kinetic energy K (" 12 mv2) of the object. Similarly, if you decelerate the object to a lesser speed by applying a force, you decrease the kinetic energy of the object. We account for these changes in kinetic energy by saying that your force has transferred energy to the object from yourself or from the object to yourself. In such a transfer of energy via a force, work W is said to be done on the object by the force. More formally, we define work as follows: Work W is energy transferred to or from an object by means of a force acting on the object. Energy transferred to the object is positive work, and energy transferred from the object is negative work.

“Work,” then, is transferred energy; “doing work” is the act of transferring the energy. Work has the same units as energy and is a scalar quantity. The term transfer can be misleading. It does not mean that anything material flows into or out of the object; that is, the transfer is not like a flow of water. Rather, it is like the electronic transfer of money between two bank accounts: The number in one account goes up while the number in the other account goes down, with nothing material passing between the two accounts. Note that we are not concerned here with the common meaning of the word “work,” which implies that any physical or mental labor is work. For example, if you push hard against a wall, you tire because of the continuously repeated muscle contractions that are required, and you are, in the common sense, working. However, such effort does not cause an energy transfer to or from the wall and thus is not work done on the wall as defined here. To avoid confusion in this chapter, we shall use the symbol W only for work and shall represent a weight with its equivalent mg.

152

CHAPTE R 7 KI N ETIC E N E RGY AN D WOR K

Work and Kinetic Energy Finding an Expression for Work Let us find an expression for work by considering a bead that can slide along a frictionless wire that is stretched along a horizontal x axis (Fig. 7-2). A constant : force F , directed at an angle f to the wire, accelerates the bead along the wire. We can relate the force and the acceleration with Newton’s second law, written for components along the x axis: Fx " max, (7-3) :

where m is the bead’s mass. As the bead moves through a displacement d , the force changes the bead’s velocity from an initial value : v0 to some other value : v. Because the force is constant, we know that the acceleration is also constant. Thus, we can use Eq. 2-16 to write, for components along the x axis, v2 " v20 # 2ax d.

(7-4)

Solving this equation for ax, substituting into Eq. 7-3, and rearranging then give us 1 2 2 mv

1

$ 2 mv20 " Fx d.

(7-5)

The first term is the kinetic energy Kf of the bead at the end of the displacement d, and the second term is the kinetic energy Ki of the bead at the start. Thus, the left side of Eq. 7-5 tells us the kinetic energy has been changed by the force, and the right side tells us the change is equal to Fxd. Therefore, the work W done on the bead by the force (the energy transfer due to the force) is (7-6)

W " Fxd.

If we know values for Fx and d, we can use this equation to calculate the work W. To calculate the work a force does on an object as the object moves through some displacement, we use only the force component along the object’s displacement. The force component perpendicular to the displacement does zero work.

From Fig. 7-2, we see that we can write Fx as F cos f, where f is the angle : : between the directions of the displacement d and the force F . Thus, W " Fd cos f

A

This component does no work.

Small initial kinetic energy F

F Wire

φ Bead :

Figure 7-2 A constant force F directed at : angle f to the displacement d of a bead on a wire accelerates the bead along the wire, changing the velocity of the bead from : v . A “kinetic energy gauge” v0 to : indicates the resulting change in the kinetic energy of the bead, from the value Ki to the value Kf . In WileyPLUS, this figure is available as an animation with voiceover.

This component does work.

x

Ki

(7-7)

(work done by a constant force).

This force does positive work on the bead, increasing speed and kinetic energy.

φ

x

v0

F

φ

Larger final kinetic energy

F Kf

φ v

Displacement d

7-2 WOR K AN D KI N ETIC E N E RGY

153

We can use the definition of the scaler (dot) product (Eq. 3-20) to write : :

W " F"d

(7-8)

(work done by a constant force),

:

where F is the magnitude of F . (You may wish to review the discussion of scaler products in Module 3-3.) Equation 7-8 is especially useful for calculating the : : work when F and d are given in unit-vector notation. Cautions. There are two restrictions to using Eqs. 7-6 through 7-8 to calculate work done on an object by a force. First, the force must be a constant force; that is, it must not change in magnitude or direction as the object moves. (Later, we shall discuss what to do with a variable force that changes in magnitude.) Second, the object must be particle-like. This means that the object must be rigid; all parts of it must move together, in the same direction. In this chapter we consider only particle-like objects, such as the bed and its occupant being pushed in Fig. 7-3. Signs for Work. The work done on an object by a force can be either positive work or negative work. For example, if angle f in Eq. 7-7 is less than 903, then cos f is positive and thus so is the work. However, if f is greater than 903 (up to 1803), then cos f is negative and thus so is the work. (Can you see that the work is zero when f " 903?) These results lead to a simple rule. To find the sign of the work done by a force, consider the force vector component that is parallel to the displacement: A force does positive work when it has a vector component in the same direction as the displacement, and it does negative work when it has a vector component in the opposite direction. It does zero work when it has no such vector component.

Units for Work. Work has the SI unit of the joule, the same as kinetic energy. However, from Eqs. 7-6 and 7-7 we can see that an equivalent unit is the newtonmeter (N !m). The corresponding unit in the British system is the foot-pound (ft !lb). Extending Eq. 7-2, we have 1 J " 1 kg "m2/s2 " 1 N "m " 0.738 ft "lb.

(7-9)

Net Work. When two or more forces act on an object, the net work done on the object is the sum of the works done by the individual forces. We can calculate the net work in two ways. (1) We can find the work done by each force : and then sum those works. (2) Alternatively, we can first find the net force F net of those forces. Then we can use Eq. 7-7, substituting the magnitude Fnet for F : : and also the angle between the directions of F net and d for f. Similarly, we can : : use Eq. 7-8 with F net substituted for F .

Work–Kinetic Energy Theorem Equation 7-5 relates the change in kinetic energy of the bead (from an initial Ki " 12 mv20 to a later Kf " 12 mv2) to the work W (" Fxd) done on the bead. For such particle-like objects, we can generalize that equation. Let -K be the change in the kinetic energy of the object, and let W be the net work done on it. Then -K " Kf $ Ki " W, (7-10) which says that

#

$ #

$

change in the kinetic net work done on " . energy of a particle the particle

F

We can also write Kf " Ki # W, which says that kinetic energy after kinetic energy the net . " # #the net work is done$ #before the net work$ #work done$

(7-11)

Figure 7-3 A contestant in a bed race. We can approximate the bed and its occupant as being a particle for the purpose of calculating the work done on them by the force applied by the contestant.

154

CHAPTE R 7 KI N ETIC E N E RGY AN D WOR K

These statements are known traditionally as the work – kinetic energy theorem for particles. They hold for both positive and negative work: If the net work done on a particle is positive, then the particle’s kinetic energy increases by the amount of the work. If the net work done is negative, then the particle’s kinetic energy decreases by the amount of the work. For example, if the kinetic energy of a particle is initially 5 J and there is a net transfer of 2 J to the particle (positive net work), the final kinetic energy is 7 J. If, instead, there is a net transfer of 2 J from the particle (negative net work), the final kinetic energy is 3 J.

Checkpoint 1 A particle moves along an x axis. Does the kinetic energy of the particle increase, decrease, or remain the same if the particle’s velocity changes (a) from $3 m/s to $2 m/s and (b) from $2 m/s to 2 m/s? (c) In each situation, is the work done on the particle positive, negative, or zero?

Sample Problem 7.02 Work done by two constant forces, industrial spies Figure 7-4a shows two industrial spies sliding an initially : stationary 225 kg floor safe a displacement d of magnitude : 8.50 m. The push F 1 of spy 001 is 12.0 N at an angle of 30.03 : downward from the horizontal; the pull F 2 of spy 002 is 10.0 N at 40.03 above the horizontal. The magnitudes and directions of these forces do not change as the safe moves, and the floor and safe make frictionless contact. (a) What is the net work done on the safe by forces : during the displacement d ?

: F1

and

: F2

KEY IDEAS (1) The net work W done on the safe by the two forces is the sum of the works they do individually. (2) Because we can treat the safe as a particle and the forces are constant in both magnitude and direction, we can use either Eq. 7-7 : : (W " Fd cos f) or Eq. 7-8 (W " F " d ) to calculate those works. Let’s choose Eq. 7-7. Calculations: From Eq. 7-7 and the free-body diagram for : the safe in Fig. 7-4b, the work done by F 1 is

Safe d

(a)

W2 " F2d cos f2 " (10.0 N)(8.50 m)(cos 40.03) " 65.11 J.

F1

Fg (b)

(b) During the displacement, what is the work Wg done on the : safe by the gravitational force F g and what is the work WN : done on the safe by the normal force FN from the floor? KEY IDEA Because these forces are constant in both magnitude and direction, we can find the work they do with Eq. 7-7. Calculations: Thus, with mg as the magnitude of the gravitational force, we write Wg " mgd cos 903 " mgd(0) " 0

(Answer)

WN " FNd cos 903 " FNd(0) " 0.

(Answer)

We should have known this result. Because these forces are perpendicular to the displacement of the safe, they do zero work on the safe and do not transfer any energy to or from it. (c) The safe is initially stationary. What is its speed vf at the end of the 8.50 m displacement?

Thus, the net work W is W " W1 # W2 " 88.33 J # 65.11 J " 153.4 J % 153 J.

F2 40.0° 30.0°

Figure 7-4 (a) Two spies move a floor safe through a displacement : d . (b) A free-body diagram for the safe.

and

:

and the work done by F 2 is

Only force components parallel to the displacement do work. FN

W1 " F1d cos f1 " (12.0 N)(8.50 m)(cos 30.03) " 88.33 J,

Spy 002

Spy 001

(Answer)

During the 8.50 m displacement, therefore, the spies transfer 153 J of energy to the kinetic energy of the safe.

KEY IDEA The speed of the safe changes because its kinetic energy is : : changed when energy is transferred to it by F 1 and F 2 .

155

7-3 WOR K DON E BY TH E G RAVITATIONAL FORCE

Calculations: We relate the speed to the work done by combining Eqs. 7-10 (the work–kinetic energy theorem) and 7-1 (the definition of kinetic energy):

done is 153.4 J. Solving for vf and then substituting known data, we find that 2W 2(153.4 J) " A m A 225 kg " 1.17 m/s.

vf "

W " Kf $ Ki " 12 mv2f $ 12 mv2i . The initial speed vi is zero, and we now know that the work

(Answer)

Sample Problem 7.03 Work done by a constant force in unit-vector notation During a storm, a crate of crepe is sliding across a slick, : oily parking lot through a displacement d " ($3.0 m)iˆ while a steady wind pushes against the crate with a force : F " (2.0 N)iˆ # ($6.0 N)jˆ . The situation and coordinate axes are shown in Fig. 7-5.

The parallel force component does negative work, slowing the crate. y

:

x

(a) How much work does this force do on the crate during the displacement?

Figure 7-5 Force F slows a : crate during displacement d .

KEY IDEA

Thus, the force does a negative 6.0 J of work on the crate, transferring 6.0 J of energy from the kinetic energy of the crate.

Because we can treat the crate as a particle and because the wind force is constant (“steady”) in both magnitude and direction during the displacement, we can use either Eq. 7-7 (W " : : Fd cos f) or Eq. 7-8 (W " F " d ) to calculate the work. Since : : we know F and d in unit-vector notation, we choose Eq. 7-8. Calculations: We write W " F " d " [(2.0 N)iˆ # ($6.0 N)jˆ ] " [($3.0 m)iˆ ]. : :

Of the possible unit-vector dot products, only iˆ "iˆ, ˆj " ˆj, and ˆk "kˆ are nonzero (see Appendix E). Here we obtain W " (2.0 N)($3.0 m)iˆ "iˆ # ($6.0 N)($3.0 m)ˆj "iˆ " ($6.0 J)(1) # 0 " $6.0 J.

(Answer)

d

F

(b) If the crate has a kinetic energy of 10 J at the beginning : : of displacement d , what is its kinetic energy at the end of d ? KEY IDEA Because the force does negative work on the crate, it reduces the crate’s kinetic energy. Calculation: Using the work – kinetic energy theorem in the form of Eq. 7-11, we have Kf " Ki # W " 10 J # ($6.0 J) " 4.0 J.

(Answer)

Less kinetic energy means that the crate has been slowed.

Additional examples, video, and practice available at WileyPLUS

7-3 WORK DONE BY THE GRAVITATIONAL FORCE Learning Objectives After reading this module, you should be able to . . .

7.07 Calculate the work done by the gravitational force when an object is lifted or lowered.

Key Ideas ● The work Wg done by the gravitational force

:

F g on a particle-like object of mass m as the object moves through a : displacement d is given by Wg " mgd cos f, : Fg

7.08 Apply the work–kinetic energy theorem to situations where an object is lifted or lowered.

done by the gravitational force and the change -K in the object’s kinetic energy by -K " Kf $ Ki " Wa # Wg. If Kf " Ki, then the equation reduces to

:

in which f is the angle between and d . ● The work Wa done by an applied force as a particle-like object is either lifted or lowered is related to the work Wg

Wa " $Wg, which tells us that the applied force transfers as much energy to the object as the gravitational force transfers from it.

156

CHAPTE R 7 KI N ETIC E N E RGY AN D WOR K

Work Done by the Gravitational Force

Kf

v

Fg

We next examine the work done on an object by the gravitational force acting on it. Figure 7-6 shows a particle-like tomato of mass m that is thrown upward with initial speed v0 and thus with initial kinetic energy Ki " 12 mv20. As the tomato : rises, it is slowed by:a gravitational force F g; that is, the tomato’s kinetic energy decreases because F g does work on the tomato as it rises. Because we can treat the tomato as a particle, we can use Eq. 7-7 (W " Fd cos f) to express the work : done during a displacement d . For the force magnitude F, we use mg as the mag: : nitude of Fg . Thus, the work Wg done by the gravitational force Fg is

The force does negative work, decreasing speed and kinetic energy.

d

Fg Ki

v0

Wg " mgd cos f

Fg :

Figure 7-6 Because the gravitational force F g acts on it, a particle-like tomato of mass m thrown upward slows from velocity : v0 to : velocity : v during displacement d . A kinetic energy gauge indicates the resulting change in the kinetic energy of the tomato, from Ki (" 12 mv20) to Kf (" 12 mv2).

:

For a rising object, force is directed opposite the displacement d , as indicated in Fig. 7-6. Thus, f " 1803 and Wg " mgd cos 1803 " mgd($1) " $mgd.

Upward displacement

F

Does positive work

Fg

Does negative work

Object

(a)

Object

F

Fg

d

Does negative work Does positive work

Downward displacement :

Figure 7-7 (a) An applied force F lifts an : object. The object’s displacement d makes an angle f " 1803 with the gravitational : force Fg on the object. The applied force does positive work on the object. (b) An : applied force F lowers an object. The dis: placement d of the object makes an angle : f " 03 with the gravitational force Fg. The applied force does negative work on the object.

(7-14)

The plus sign tells us that the gravitational force now transfers energy in the amount mgd to the kinetic energy of the falling object (it speeds up, of course).

Work Done in Lifting and Lowering an Object :

Now suppose we lift a particle-like object by applying a vertical force F to it. During the upward displacement, our applied force does positive work Wa on the object while the gravitational force does negative work Wg on it. Our applied force tends to transfer energy to the object while the gravitational force tends to transfer energy from it. By Eq. 7-10, the change -K in the kinetic energy of the object due to these two energy transfers is -K " Kf $ Ki " Wa # Wg,

(7-15)

in which Kf is the kinetic energy at the end of the displacement and Ki is that at the start of the displacement. This equation also applies if we lower the object, but then the gravitational force tends to transfer energy to the object while our force tends to transfer energy from it. If an object is stationary before and after a lift (as when you lift a book from the floor to a shelf), then Kf and Ki are both zero, and Eq. 7-15 reduces to Wa # Wg " 0 or

(b)

(7-13)

The minus sign tells us that during the object’s rise, the gravitational force acting on the object transfers energy in the amount mgd from the kinetic energy of the object. This is consistent with the slowing of the object as it rises. After the object has reached its maximum height and is falling back down, : : the angle f between force F g and displacement d is zero. Thus, Wg " mgd cos 03 " mgd(#1) " #mgd.

d

(7-12)

(work done by gravitational force).

: Fg

Wa " $Wg.

(7-16)

Note that we get the same result if Kf and Ki are not zero but are still equal. Either way, the result means that the work done by the applied force is the negative of the work done by the gravitational force; that is, the applied force transfers the same amount of energy to the object as the gravitational force transfers from the object. Using Eq. 7-12, we can rewrite Eq. 7-16 as Wa " $mgd cos f

(work done in lifting and lowering; Kf " Ki), : Fg

:

(7-17)

with f being the angle between and d . If the displacement is vertically upward (Fig. 7-7a), then f " 1803 and the work done by the applied force equals mgd.

157

7-3 WOR K DON E BY TH E G RAVITATIONAL FORCE

If the displacement is vertically downward (Fig. 7-7b), then f " 03 and the work done by the applied force equals $mgd. Equations 7-16 and 7-17 apply to any situation in which an object is lifted or lowered, with the object stationary before and after the lift. They are independent of the magnitude of the force used. For example, if you lift a mug from the floor to over your head, your force on the mug varies considerably during the lift. Still, because the mug is stationary before and after the lift, the work your force does on the mug is given by Eqs. 7-16 and 7-17, where, in Eq. 7-17, mg is the weight of the mug and d is the distance you lift it. Sample Problem 7.04 Work in pulling a sleigh up a snowy slope In this problem an object is pulled along a ramp but the object starts and ends at rest and thus has no overall change in its kinetic energy (that is important). Figure 7-8a shows the situation. A rope pulls a 200 kg sleigh (which you may know) up a slope at incline angle u " 303, through distance d " 20 m. The sleigh and its contents have a total mass of 200 kg. The snowy slope is so slippery that we take it to be frictionless. How much work is done by each force acting on the sleigh? KEY IDEAS (1) During the motion, the forces are constant in magnitude and direction and thus we can calculate the work done by each with Eq. 7-7 (W " Fd cos f) in which f is the angle between the force and the displacement. We reach the same : : result with Eq. 7-8 (W " F " d ) in which we take a dot product of the force vector and displacement vector. (2) We can relate the net work done by the forces to the change in kinetic energy (or lack of a change, as here) with the work–kinetic energy theorem of Eq. 7-10 (-K " W). Calculations: The first thing to do with most physics problems involving forces is to draw a free-body diagram to organize our thoughts. For the sleigh, Fig.7-8b is our free-body dia: : gram, showing the gravitational force F g, the force T from the : rope, and the normal force FN from the slope.

The angle f between the displacement and this force component is 1803. So we can apply Eq. 7-7 to write Wg " Fgxd cos 1803 " (980 N)(20 m)($1) "$1.96 ' 104 J.

The negative result means that the gravitational force removes energy from the sleigh. The second (equivalent) way to get this result is to use : the full gravitational force F g instead of a component. The : : angle between F g and d is 1203 (add the incline angle 303 to 903). So, Eq. 7-7 gives us Wg " Fgd cos 1203 " mgd cos 1203 " (200 kg)(9.8 m/s2)(20 m) cos 1203 "$1.96 ' 104 J.

0 " WN # Wg # WT " 0 $ 1.96 ' 104 J # WT WT " 1.96 ' l04 J.

and

(Answer)

d

(Answer)

Work Wg by the gravitational force. We can find the work done by the gravitational force in either of two ways (you pick the more appealing way). From an earlier discussion about ramps (Sample Problem 5.04 and Fig. 5-15), we know that the component of the gravitational force along the slope has magnitude mg sin u and is directed down the slope. Thus the magnitude is Fgx " mg sin u " (200 kg)(9.8 m/s2) sin 303 " 980 N.

(Answer)

Work WT by the rope’s force. We have two ways of calculating this work. The quickest way is to use the work–kinetic energy theorem of Eq. 7-10 (-K " W), where the net work W done by the forces is WN # Wg # WT and the change -K in the kinetic energy is just zero (because the initial and final kinetic energies are the same—namely, zero). So, Eq. 7-10 gives us

Work WN by the normal force. Let’s start with this easy calculation. The normal force is perpendicular to the slope and thus also to the sleigh’s displacement. Thus the normal force does not affect the sleigh’s motion and does zero work. To be more formal, we can apply Eq. 7-7 to write WN " FNd cos 903 " 0.

(Answer)

(a)

θ

FN

x

T

Does positive work

Does negative work mg sinu

mg cosu (b)

u

Fg

Figure 7-8 (a) A sleigh is pulled up a snowy slope. (b) The freebody diagram for the sleigh.

158

CHAPTE R 7 KI N ETIC E N E RGY AN D WOR K

Instead of doing this, we can apply Newton’s second law for motion along the x axis to find the magnitude FT of the rope’s force. Assuming that the acceleration along the slope is zero (except for the brief starting and stopping), we can write Fnet,x " max,

This is the magnitude. Because the force and the displacement are both up the slope, the angle between those two vectors is zero. So, we can now write Eq. 7-7 to find the work done by the rope’s force: WT " FT d cos 03 " (mg sin 303)d cos 03

FT $ mg sin 303 " m(0),

to find

" (200 kg)(9.8 m/s2)(sin 303)(20 m) cos 03 " 1.96 ' 104 J.

FT " mg sin 303.

(Answer)

Sample Problem 7.05 Work done on an accelerating elevator cab An elevator cab of mass m " 500 kg is descending with speed vi " 4.0 m/s when its supporting cable begins to slip, allowing it to fall with constant acceleration : a": g /5 (Fig. 7-9a). (a) During the fall through a distance d " 12 m, what is the : work Wg done on the cab by the gravitational force F g? KEY IDEA We can treat the cab as a particle and thus use Eq. 7-12 (Wg " mgd cos f) to find the work Wg. Calculation: From Fig. 7-9b, we see that the angle between : : the directions of Fg and the cab’s displacement d is 03. So, Wg " mgd cos 03 " (500 kg)(9.8 m/s2)(12 m)(1) " 5.88 ' 10 4 J % 59 kJ.

(Answer)

(b) During the 12 m fall, what is the work WT done on the : cab by the upward pull T of the elevator cable?

Elevator cable y

Figure 7-9 An elevator cab, descending with speed vi, suddenly begins to accelerate downward. (a) It moves through a dis: placement d with constant acceleration : a": g/5. (b) A freebody diagram for the cab, displacement included.

Cab

d

T

Does negative work

Fg

Does positive work

a

(a)

(b)

Caution: Note that WT is not simply the negative of Wg because the cab accelerates during the fall. Thus, Eq. 7-16 (which assumes that the initial and final kinetic energies are equal) does not apply here.

KEY IDEA

(c) What is the net work W done on the cab during the fall?

We can calculate work WT with Eq. 7-7 (W " Fd cos f) by first writing Fnet,y " may for the components in Fig. 7-9b.

Calculation: The net work is the sum of the works done by the forces acting on the cab:

Calculations: We get

W " Wg # WT " 5.88 ' 10 4 J $ 4.70 ' 10 4 J T $ Fg " ma.

(7-18)

Solving for T, substituting mg for Fg, and then substituting the result in Eq. 7-7, we obtain WT " Td cos f " m(a # g)d cos f.

(7-19)

Next, substituting $g/5 for the (downward) acceleration a and then 1803 for the angle f between the directions of : : forces T and mg , we find

#

WT " m $

$

g 4 # g d cos 4 " mgd cos 4 5 5

(Answer)

(d) What is the cab’s kinetic energy at the end of the 12 m fall? KEY IDEA The kinetic energy changes because of the net work done on the cab, according to Eq. 7-11 (Kf " Ki # W). Calculation: From Eq. 7-1, we write the initial kinetic energy as Ki " 12mv2i . We then write Eq. 7-11 as Kf " Ki # W " 12 mv2i # W

4 " (500 kg)(9.8 m/s2)(12 m) cos 1803 5 " $4.70 ' 10 4 J % $47 kJ.

" 1.18 ' 10 4 J % 12 kJ.

" 12(500 kg)(4.0 m/s)2 # 1.18 ' 10 4 J (Answer)

" 1.58 ' 10 4 J % 16 kJ.

Additional examples, video, and practice available at WileyPLUS

(Answer)

7-4 WOR K DON E BY A SPR I NG FORCE

159

7-4 WORK DONE BY A SPRING FORCE Learning Objectives After reading this module, you should be able to . . .

7.09 Apply the relationship (Hooke’s law) between the force on an object due to a spring, the stretch or compression of the spring, and the spring constant of the spring. 7.10 Identify that a spring force is a variable force. 7.11 Calculate the work done on an object by a spring force by integrating the force from the initial position to the final

position of the object or by using the known generic result of that integration. 7.12 Calculate work by graphically integrating on a graph of force versus position of the object. 7.13 Apply the work–kinetic energy theorem to situations in which an object is moved by a spring force.

Key Ideas ● The force

:

Fs from a spring is : Fs

:

" $kd

(Hooke’s law),

:

where d is the displacement of the spring’s free end from its position when the spring is in its relaxed state (neither compressed nor extended), and k is the spring constant (a measure of the spring’s stiffness). If an x axis lies along the spring, with the origin at the location of the spring’s free end when the spring is in its relaxed state, we can write Fx " $kx

● A spring force is thus a variable force: It varies with the displacement of the spring’s free end. ● If an object is attached to the spring’s free end, the work Ws

done on the object by the spring force when the object is moved from an initial position xi to a final position xf is Ws " 12 kx2i $ 12 kx2f . If xi " 0 and xf " x, then the equation becomes Ws " $12 kx2.

(Hooke’s law).

Work Done by a Spring Force We next want to examine the work done on a particle-like object by a particular type of variable force —namely, a spring force, the force from a spring. Many forces in nature have the same mathematical form as the spring force. Thus, by examining this one force, you can gain an understanding of many others.

x=0 Fx = 0

Block attached to spring x

0 (a)

The Spring Force Figure 7-10a shows a spring in its relaxed state —that is, neither compressed nor extended. One end is fixed, and a particle-like object — a block, say — is attached to the other, free end. If we stretch the spring by pulling the block to the right as in Fig. 7-10b, the spring pulls on the block toward the left. (Because a spring force acts to restore the relaxed state, it is sometimes said to be a restoring force.) If we compress the spring by pushing the block to the left as in Fig. 7-10c, the spring now pushes on the block toward the right. : To a good approximation for many springs, the force F s from a spring is pro: portional to the displacement d of the free end from its position when the spring is in the relaxed state. The spring force is given by :

:

F s " $kd

(Hooke’s law),

(7-20)

which is known as Hooke’s law after Robert Hooke, an English scientist of the late 1600s. The minus sign in Eq. 7-20 indicates that the direction of the spring force is always opposite the direction of the displacement of the spring’s free end. The constant k is called the spring constant (or force constant) and is a measure of the stiffness of the spring. The larger k is, the stiffer the spring; that is, the larger k is, the stronger the spring’s pull or push for a given displacement. The SI unit for k is the newton per meter. In Fig. 7-10 an x axis has been placed parallel to the length of the spring, with the origin (x " 0) at the position of the free end when the spring is in its relaxed

x positive Fx negative F s

d

x

x

0 (b) d

Fs x

x negative Fx positive x

0 (c)

Figure 7-10 (a) A spring in its relaxed state. The origin of an x axis has been placed at the end of the spring that is attached to a : block. (b) The block is displaced by d , and the spring is stretched by a positive amount : x. Note the restoring force F s exerted by the spring. (c) The spring is compressed by a negative amount x. Again, note the restoring force.

160

CHAPTE R 7 KI N ETIC E N E RGY AN D WOR K

state. For this common arrangement, we can write Eq. 7-20 as Fx " $kx

(Hooke’s law),

(7-21)

where we have changed the subscript. If x is positive (the spring is stretched toward the right on the x axis), then Fx is negative (it is a pull toward the left). If x is negative (the spring is compressed toward the left), then Fx is positive (it is a push toward the right). Note that a spring force is a variable force because it is a function of x, the position of the free end. Thus Fx can be symbolized as F(x). Also note that Hooke’s law is a linear relationship between Fx and x.

The Work Done by a Spring Force To find the work done by the spring force as the block in Fig. 7-10a moves, let us make two simplifying assumptions about the spring. (1) It is massless; that is, its mass is negligible relative to the block’s mass. (2) It is an ideal spring; that is, it obeys Hooke’s law exactly. Let us also assume that the contact between the block and the floor is frictionless and that the block is particle-like. We give the block a rightward jerk to get it moving and then leave it alone. As the block moves rightward, the spring force Fx does work on the block, decreasing the kinetic energy and slowing the block. However, we cannot find this work by using Eq. 7-7 (W " Fd cos f) because there is no one value of F to plug into that equation—the value of F increases as the block stretches the spring. There is a neat way around this problem. (1) We break up the block’s displacement into tiny segments that are so small that we can neglect the variation in F in each segment. (2) Then in each segment, the force has (approximately) a single value and thus we can use Eq. 7-7 to find the work in that segment. (3) Then we add up the work results for all the segments to get the total work. Well, that is our intent, but we don’t really want to spend the next several days adding up a great many results and, besides, they would be only approximations. Instead, let’s make the segments infinitesimal so that the error in each work result goes to zero. And then let’s add up all the results by integration instead of by hand. Through the ease of calculus, we can do all this in minutes instead of days. Let the block’s initial position be xi and its later position be xf. Then divide the distance between those two positions into many segments, each of tiny length -x. Label these segments, starting from xi, as segments 1, 2, and so on. As the block moves through a segment, the spring force hardly varies because the segment is so short that x hardly varies. Thus, we can approximate the force magnitude as being constant within the segment. Label these magnitudes as Fx1 in segment 1, Fx2 in segment 2, and so on. With the force now constant in each segment, we can find the work done within each segment by using Eq. 7-7. Here f " 1803, and so cos f " $1. Then the work done is $Fx1 -x in segment 1, $Fx2 -x in segment 2, and so on. The net work Ws done by the spring, from xi to xf , is the sum of all these works: Ws " ' $Fxj -x,

(7-22)

where j labels the segments. In the limit as -x goes to zero, Eq. 7-22 becomes Ws "

"

xf

xi

(7-23)

$Fx dx.

From Eq. 7-21, the force magnitude Fx is kx. Thus, substitution leads to Ws "

"

xf

xi

$kx dx " $k

"

xf

xi

x dx

" ($12 k)[x2]xxfi " ($12 k)(x2f $ x2i ).

(7-24)

7-4 WOR K DON E BY A SPR I NG FORCE

Multiplied out, this yields Ws " 12 kx2i $ 12 kx2f

(7-25)

(work by a spring force).

This work Ws done by the spring force can have a positive or negative value, depending on whether the net transfer of energy is to or from the block as the block moves from xi to xf . Caution: The final position xf appears in the second term on the right side of Eq. 7-25. Therefore, Eq. 7-25 tells us: Work Ws is positive if the block ends up closer to the relaxed position (x " 0) than it was initially. It is negative if the block ends up farther away from x " 0. It is zero if the block ends up at the same distance from x " 0.

If xi " 0 and if we call the final position x, then Eq. 7-25 becomes Ws " $12 kx2

(work by a spring force).

(7-26)

The Work Done by an Applied Force Now suppose that we displace the block along the x axis while continuing to apply a : force Fa to it. During the displacement, our applied force does work Wa on the block while the spring force does work Ws. By Eq. 7-10, the change -K in the kinetic energy of the block due to these two energy transfers is -K " Kf $ Ki " Wa # Ws ,

(7-27)

in which Kf is the kinetic energy at the end of the displacement and Ki is that at the start of the displacement. If the block is stationary before and after the displacement, then Kf and Ki are both zero and Eq. 7-27 reduces to Wa " $Ws.

(7-28)

If a block that is attached to a spring is stationary before and after a displacement, then the work done on it by the applied force displacing it is the negative of the work done on it by the spring force.

Caution: If the block is not stationary before and after the displacement, then this statement is not true.

Checkpoint 2 For three situations, the initial and final positions, respectively, along the x axis for the block in Fig. 7-10 are (a) $3 cm, 2 cm; (b) 2 cm, 3 cm; and (c) $2 cm, 2 cm. In each situation, is the work done by the spring force on the block positive, negative, or zero?

Sample Problem 7.06 Work done by a spring to change kinetic energy When a spring does work on an object, we cannot find the work by simply multiplying the spring force by the object’s displacement. The reason is that there is no one value for the force—it changes. However, we can split the displacement up into an infinite number of tiny parts and then approximate the force in each as being constant. Integration sums the work done in all those parts. Here we use the generic result of the integration. In Fig. 7-11, a cumin canister of mass m " 0.40 kg slides across a horizontal frictionless counter with speed v " 0.50 m/s.

The spring force does negative work, decreasing speed and kinetic energy.

v

k Frictionless d

Stop

First touch

Figure 7-11 A canister moves toward a spring.

m

161

162

CHAPTE R 7 KI N ETIC E N E RGY AN D WOR K

It then runs into and compresses a spring of spring constant k " 750 N/m. When the canister is momentarily stopped by the spring, by what distance d is the spring compressed?

Calculations: Putting the first two of these ideas together, we write the work – kinetic energy theorem for the canister as

KEY IDEAS

Substituting according to the third key idea gives us this expression:

1. The work Ws done on the canister by the spring force is related to the requested distance d by Eq. 7-26 (Ws " $12 kx2) , with d replacing x. 2. The work Ws is also related to the kinetic energy of the canister by Eq. 7-10 (Kf $ Ki " W). 3. The canister’s kinetic energy has an initial value of K " 1 2 2 mv and a value of zero when the canister is momentarily at rest.

Kf $ Ki " $12 kd 2.

0 $ 12 mv2 " $12 kd 2. Simplifying, solving for d, and substituting known data then give us d"v

m 0.40 kg " (0.50 m/s) A k A 750 N/m

" 1.2 ' 10$2 m " 1.2 cm.

(Answer)

Additional examples, video, and practice available at WileyPLUS

7-5 WORK DONE BY A GENERAL VARIABLE FORCE Learning Objectives After reading this module, you should be able to . . .

7.14 Given a variable force as a function of position, calculate the work done by it on an object by integrating the function from the initial to the final position of the object, in one or more dimensions. 7.15 Given a graph of force versus position, calculate the work done by graphically integrating from the initial position to the final position of the object.

7.16 Convert a graph of acceleration versus position to a graph of force versus position. 7.17 Apply the work–kinetic energy theorem to situations where an object is moved by a variable force.

Key Ideas ● When

:

the force F on a particle-like object depends on : the position of the object, the work done by F on the object while the object moves from an initial position ri with coordinates (xi, yi, zi) to a final position rf with coordinates (xf, yf, zf) must be found by integrating the force. If we assume that component Fx may depend on x but not on y or z, component Fy may depend on y but not on x or z, and component Fz may depend on z but not on x or y, then the

work is W" ● If

"

xf

xi

F x dx #

"

yf

yi

F y dy #

"

zf

zi

F z dz.

:

F has only an x component, then this reduces to W"

"

xf

xi

F(x) dx.

Work Done by a General Variable Force One-Dimensional Analysis Let us return to the situation of Fig. 7-2 but now consider the force to be in the positive direction of the x axis and the force magnitude to vary with position x. Thus, as the bead (particle) moves, the magnitude F(x) of the force doing work on it changes. Only the magnitude of this variable force changes, not its direction, and the magnitude at any position does not change with time.

163

7-5 WOR K DON E BY A G E N E RAL VAR IAB LE FORCE

Figure 7-12a shows a plot of such a one-dimensional variable force. We want an expression for the work done on the particle by this force as the particle moves from an initial point xi to a final point xf . However, we cannot use Eq. 7-7 : (W " Fd cos f) because it applies only for a constant force F . Here, again, we shall use calculus. We divide the area under the curve of Fig. 7-12a into a number of narrow strips of width -x (Fig. 7-12b). We choose -x small enough to permit us to take the force F(x) as being reasonably constant over that interval. We let Fj,avg be the average value of F(x) within the jth interval. Then in Fig. 7-12b, Fj,avg is the height of the jth strip. With Fj,avg considered constant, the increment (small amount) of work -Wj done by the force in the jth interval is now approximately given by Eq. 7-7 and is

Work is equal to the area under the curve. F(x)

(a)

In Fig. 7-12b, -Wj is then equal to the area of the jth rectangular, shaded strip. To approximate the total work W done by the force as the particle moves from xi to xf, we add the areas of all the strips between xi and xf in Fig. 7-12b: W " '-Wj " 'Fj,avg -x.

-x : 0

'Fj,avg - x.

x

F(x) ∆Wj

(7-30)

Equation 7-30 is an approximation because the broken “skyline” formed by the tops of the rectangular strips in Fig. 7-12b only approximates the actual curve of F(x). We can make the approximation better by reducing the strip width -x and using more strips (Fig. 7-12c). In the limit, we let the strip width approach zero; the number of strips then becomes infinitely large and we have, as an exact result, W " lim

xf

We can approximate that area with the area of these strips.

(7-29)

-Wj " Fj,avg -x.

0 xi

Fj, avg

(b)

0 xi

∆x

xf

x

We can do better with more, narrower strips. F(x)

(7-31)

This limit is exactly what we mean by the integral of the function F(x) between the limits xi and xf . Thus, Eq. 7-31 becomes W"

"

xf

xi

F(x) dx

(work: variable force).

(7-32)

(c)

If we know the function F(x), we can substitute it into Eq. 7-32, introduce the proper limits of integration, carry out the integration, and thus find the work. (Appendix E contains a list of common integrals.) Geometrically, the work is equal to the area between the F(x) curve and the x axis, between the limits xi and xf (shaded in Fig. 7-12d).

0 xi

∆x

xf

x

For the best, take the limit of strip widths going to zero. F(x)

W

Three-Dimensional Analysis Consider now a particle that is acted on by a three-dimensional force F " Fx ˆi # Fy ˆj # Fz kˆ, :

(7-33)

in which the components Fx, Fy, and Fz can depend on the position of the particle; that is, they can be functions of that position. However, we make three simplifications: Fx may depend on x but not on y or z, Fy may depend on y but not on x or z, and Fz may depend on z but not on x or y. Now let the particle move through an incremental displacement dr: " dx ˆi # dyˆj # dzkˆ. (7-34) :

The increment of work dW done on the particle by F during the displacement dr: is, by Eq. 7-8, : dW " F " dr: " Fx dx # Fy dy # Fz dz. (7-35)

(d)

0 xi

xf

x

Figure 7-12 (a) A one-dimensional force : F (x) plotted against the displacement x of a particle on which it acts. The particle moves from xi to xf . (b) Same as (a) but with the area under the curve divided into narrow strips. (c) Same as (b) but with the area divided into narrower strips. (d) The limiting case. The work done by the force is given by Eq. 7-32 and is represented by the shaded area between the curve and the x axis and between xi and xf .

164

CHAPTE R 7 KI N ETIC E N E RGY AN D WOR K :

The work W done by F while the particle moves from an initial position ri having coordinates (xi , yi , zi) to a final position rf having coordinates (xf , yf , zf) is then W"

"

rf

ri

dW "

"

xf

xi

Fx dx #

"

yf

Fy dy #

yi

"

zf

zi

Fz dz.

(7-36)

:

If F has only an x component, then the y and z terms in Eq. 7-36 are zero and the equation reduces to Eq. 7-32.

Work–Kinetic Energy Theorem with a Variable Force Equation 7-32 gives the work done by a variable force on a particle in a onedimensional situation. Let us now make certain that the work is equal to the change in kinetic energy, as the work – kinetic energy theorem states. Consider a particle of mass m, moving along an x axis and acted on by a net force F(x) that is directed along that axis. The work done on the particle by this force as the particle moves from position xi to position xf is given by Eq. 7-32 as W"

"

xf

xi

F(x) dx "

"

xf

xi

ma dx,

(7-37)

in which we use Newton’s second law to replace F(x) with ma. We can write the quantity ma dx in Eq. 7-37 as dv (7-38) dx. ma dx " m dt From the chain rule of calculus, we have

and Eq. 7-38 becomes

dv dx dv dv " " v, dt dx dt dx ma dx " m

dv v dx " mv dv. dx

Substituting Eq. 7-40 into Eq. 7-37 yields W"

"

vf

vi

mv dv " m

"

vf

vi

" 12 mv2f $ 12 mv2i .

(7-39)

(7-40)

v dv (7-41)

Note that when we change the variable from x to v we are required to express the limits on the integral in terms of the new variable. Note also that because the mass m is a constant, we are able to move it outside the integral. Recognizing the terms on the right side of Eq. 7-41 as kinetic energies allows us to write this equation as W " Kf $ Ki " -K, which is the work – kinetic energy theorem.

Sample Problem 7.07 Work calculated by graphical integration In Fig. 7-13b, an 8.0 kg block slides along a frictionless floor as a force acts on it, starting at x1 " 0 and ending at x3 " 6.5 m. As the block moves, the magnitude and direction of the force varies according to the graph shown in Fig. 7-13a. For

example, from x " 0 to x " 1 m, the force is positive (in the positive direction of the x axis) and increases in magnitude from 0 to 40 N. And from x " 4 m to x " 5 m, the force is negative and increases in magnitude from 0 to 20 N.

165

7-5 WOR K DON E BY A G E N E RAL VAR IAB LE FORCE

(Note that this latter value is displayed as $20 N.) The block’s kinetic energy at x1 is K1 " 280 J. What is the block’s speed at x1 " 0, x2 " 4.0 m, and x3 " 6.5 m?

F (N) 40

KEY IDEAS (1) At any point, we can relate the speed of the block to its kinetic energy with Eq. 7-1 (K ! 12mv2). (2) We can relate the kinetic energy Kf at a later point to the initial kinetic Ki and the work W done on the block by using the work– kinetic energy theorem of Eq. 7-10 (Kf $ Ki " W). (3) We can calculate the work W done by a variable force F(x) by integrating the force versus position x. Equation 7-32 tells us that W!

"

2

4

xi

(a) v1

F

0

2

Calculations: The requested speed at x ! 0 is easy because we already know the kinetic energy. So, we just plug the kinetic energy into the formula for kinetic energy: K1 ! 12mv21, 280 J ! 12(8.0 kg)v21, and then (Answer)

As the block moves from x ! 0 to x ! 4.0 m, the plot in Figure 7-13a is above the x axis, which means that positive work is being done on the block. We split the area under the plot into a triangle at the left, a rectangle in the center, and a triangle at the right. Their total area is

! 120 J. This means that between x ! 0 and x ! 4.0 m, the force does 120 J of work on the block, increasing the kinetic energy and speed of the block. So, when the block reaches x ! 4.0 m, the work–kinetic energy theorem tells us that the kinetic energy is

! 280 J " 120 J ! 400 J.

v3

F

4

x (m)

6

Figure 7-13 (a) A graph indicating the magnitude and direction of a variable force that acts on a block as it moves along an x axis on a floor, (b) The location of the block at several times.

Again using the definition of kinetic energy, we find K2 " 12mv22, and then

400 J " 12(8.0 kg)v22, v2 " 10 m/s.

(Answer)

This is the block’s greatest speed because from x " 4.0 m to x " 6.5 m the force is negative, meaning that it opposes the block’s motion, doing negative work on the block and thus decreasing the kinetic energy and speed. In that range, the area between the plot and the x axis is 1 2 (20

N)(1 m) # (20 N)(1 m) # 12(20 N)(0.5 m) " 35 N!m " 35 J.

This means that the work done by the force in that range is $35 J. At x " 4.0, the block has K " 400 J. At x " 6.5 m, the work–kinetic energy theorem tells us that its kinetic energy is K3 " K2 # W

N)(1 m) " (40 N)(2 m) " 12(40 N)(1 m) ! 120 N#m

K2 ! K1 " W

v2

(b)

F(x) dx.

v1 ! 8.37 m/s % 8.4 m/s.

x (m)

6

!20

xf

We don’t have a function F(x) to carry out the integration, but we do have a graph of F(x) where we can integrate by finding the area between the plotted line and the x axis. Where the plot is above the axis, the work (which is equal to the area) is positive. Where it is below the axis, the work is negative.

1 2 (40

0

" 400 J $ 35 J " 365 J. Again using the definition of kinetic energy, we find K3 " 12mv23, 365 J " 12(8.0 kg)v23, and then v3 " 9.55 m/s % 9.6 m/s.

(Answer)

The block is still moving in the positive direction of the x axis, a bit faster than initially.

Additional examples, video, and practice available at WileyPLUS

166

CHAPTE R 7 KI N ETIC E N E RGY AN D WOR K

Sample Problem 7.08 Work, two-dimensional integration When the force on an object depends on the position of the object, we cannot find the work done by it on the object by simply multiplying the force by the displacement. The reason is that there is no one value for the force—it changes. So, we must find the work in tiny little displacements and then add up all the work results. We effectively say, “Yes, the force varies over any given tiny little displacement, but the variation is so small we can approximate the force as being constant during the displacement.” Sure, it is not precise, but if we make the displacements infinitesimal, then our error becomes infinitesimal and the result becomes precise. But, to add an infinite number of work contributions by hand would take us forever, longer than a semester. So, we add them up via an integration, which allows us to do all this in minutes (much less than a semester). : Force F ! (3x2 N)ˆi " (4 N)ˆj , with x in meters, acts on a particle, changing only the kinetic energy of the particle. How much work is done on the particle as it moves from coordinates (2 m, 3 m) to (3 m, 0 m)? Does the speed of the particle increase, decrease, or remain the same?

KEY IDEA The force is a variable force because its x component depends on the value of x. Thus, we cannot use Eqs. 7-7 and 7-8 to find the work done. Instead, we must use Eq. 7-36 to integrate the force. Calculation: We set up two integrals, one along each axis: W!

"

3

2

3x2 dx "

"

3

0

4 dy ! 3

"

2

3

x2 dx " 4

"

3

0

dy

! 3[13x3]32 " 4[ y]03 ! [33 $ 23] " 4[0 $ 3] (Answer)

! 7.0 J.

The positive result means that energy is transferred to the : particle by force F . Thus, the kinetic energy of the particle increases and, because K ! 12mv2, its speed must also increase. If the work had come out negative, the kinetic energy and speed would have decreased.

Additional examples, video, and practice available at WileyPLUS

7-6 POWER Learning Objectives After reading this module, you should be able to . . .

7.18 Apply the relationship between average power, the work done by a force, and the time interval in which that work is done. 7.19 Given the work as a function of time, find the instantaneous power.

7.20 Determine the instantaneous power by taking a dot product of the force vector and an object’s velocity vector, in magnitude-angle and unit-vector notations.

Key Ideas ● The power due to a force is the rate at which that force does work on an object. ● If the force does work W during a time interval %t, the aver-

age power due to the force over that time interval is Pavg !

W . %t

● Instantaneous power is the instantaneous rate of doing work:

dW . dt : ● For a force F at an angle f to the direction of travel of the instantaneous velocity : v , the instantaneous power is P!

:

P ! Fv cos & ! F " : v.

Power The time rate at which work is done by a force is said to be the power due to the force. If a force does an amount of work W in an amount of time %t, the average power due to the force during that time interval is Pavg !

W %t

(average power).

(7-42)

7-6 POWE R

167

The instantaneous power P is the instantaneous time rate of doing work, which we can write as P!

dW dt

(7-43)

(instantaneous power).

Suppose we know the work W(t) done by a force as a function of time. Then to get the instantaneous power P at, say, time t ! 3.0 s during the work, we would first take the time derivative of W(t) and then evaluate the result for t ! 3.0 s. The SI unit of power is the joule per second. This unit is used so often that it has a special name, the watt (W), after James Watt, who greatly improved the rate at which steam engines could do work. In the British system, the unit of power is the foot-pound per second. Often the horsepower is used. These are related by

and

1 watt ! 1 W ! 1 J/s ! 0.738 ft #lb/s

(7-44)

1 horsepower ! 1 hp ! 550 ft #lb/s ! 746 W.

(7-45)

Inspection of Eq. 7-42 shows that work can be expressed as power multiplied by time, as in the common unit kilowatt-hour. Thus, 1 kilowatt-hour ! 1 kW #h ! (103 W)(3600 s) ! 3.60 ' 106 J ! 3.60 MJ.

(7-46)

Perhaps because they appear on our utility bills, the watt and the kilowatt-hour have become identified as electrical units. They can be used equally well as units for other examples of power and energy. Thus, if you pick up a book from the floor and put it on a tabletop, you are free to report the work that you have done as, say, 4 ' 10$6 kW #h (or more conveniently as 4 mW #h). We can also express the rate at which a force does work on a particle (or particle-like object) in terms of that force and the particle’s velocity. For a particle that is moving along a straight line (say, an x axis) and is acted on by a : constant force F directed at some angle f to that line, Eq. 7-43 becomes P!

F cos f dx dW ! ! F cos f dt dt

or

# dxdt $,

P ! Fv cos f.

(7-47) : :

Reorganizing the right side of Eq. 7-47 as the dot product F " v , we may also write the equation as :

P ! F": v

(instantaneous power).

(7-48)

:

For example, the truck in Fig. 7-14 exerts a force F on the trailing load, which : has velocity : v at some instant. The instantaneous power due to F is the rate at : which F does work on the load at that instant and is given by Eqs. 7-47 and 7-48. Saying that this power is “the power of the truck” is often acceptable, but keep in mind what is meant: Power is the rate at which the applied force does work.

Checkpoint 3 A block moves with uniform circular motion because a cord tied to the block is anchored at the center of a circle. Is the power due to the force on the block from the cord positive, negative, or zero?

© Reglain/ZUMA

Figure 7-14 The power due to the truck’s applied force on the trailing load is the rate at which that force does work on the load.

168

CHAPTE R 7 KI N ETIC E N E RGY AN D WOR K

Sample Problem 7.09 Power, force, and velocity Here we calculate an instantaneous work—that is, the rate at which work is being done at any given instant rather than averaged over a time interval. Figure 7-15 shows constant forces : : F 1 and F 2 acting on a box as the box slides rightward across a : frictionless floor. Force F 1 is horizontal, with magnitude 2.0 N; : force F 2 is angled upward by 60( to the floor and has magnitude 4.0 N.The speed v of the box at a certain instant is 3.0 m/s. What is the power due to each force acting on the box at that instant, and what is the net power? Is the net power changing at that instant? KEY IDEA

:

Calculation: We use Eq. 7-47 for each force. For force F 1, at angle f1 ! 180( to velocity : v , we have P1 ! F1v cos f1 ! (2.0 N)(3.0 m/s) cos 180( ! $6.0 W.

(Answer) : F1

This negative result tells us that force is transferring energy from the box at the rate of 6.0 J/s. : For force F 2 , at angle f 2 ! 60( to velocity : v , we have P2 ! F2v cos f 2 ! (4.0 N)(3.0 m/s) cos 60( ! 6.0 W.

(Answer) :

We want an instantaneous power, not an average power over a time period. Also, we know the box’s velocity (rather than the work done on it). Negative power. (This force is removing energy.)

60°

F1

Frictionless :

Pnet ! P1 " P2

Positive power. (This force is supplying energy.)

F2

This positive result tells us that force F 2 is transferring energy to the box at the rate of 6.0 J/s. The net power is the sum of the individual powers (complete with their algebraic signs): ! $6.0 W " 6.0 W ! 0,

(Answer)

which tells us that the net rate of transfer of energy to or from the box is zero. Thus, the kinetic energy (K ! 12 mv2) of the box is not changing, and so the speed of the box will : : remain at 3.0 m/s. With neither the forces F 1 and F 2 nor the : velocity v changing, we see from Eq. 7-48 that P1 and P2 are constant and thus so is Pnet.

v

:

Figure 7-15 Two forces F1 and F2 act on a box that slides rightward across a frictionless floor. The velocity of the box is : v.

Additional examples, video, and practice available at WileyPLUS

Review & Summary Kinetic Energy The kinetic energy K associated with the motion of a particle of mass m and speed v, where v is well below the speed of light, is K ! 12 mv2

(7-1)

(kinetic energy).

Work Work W is energy transferred to or from an object via a force acting on the object. Energy transferred to the object is positive work, and from the object, negative work. Work Done by a Constant Force The work done on a par:

:

ticle by a constant force F during displacement d is : :

W ! Fd cos & ! F " d

(work, constant force),

in which Ki is the initial kinetic energy of the particle and Kf is the kinetic energy after the work is done. Equation 7-10 rearranged gives us

Work Done by the Gravitational Force The work Wg :

done by the gravitational force F g on a particle-like object of mass : m as the object moves through a displacement d is given by Wg ! mgd cos f, in which f is the angle between

(7-7, 7-8) :

:

in which f is the constant angle between the directions of F and d . : : Only the component of F that is along the displacement d can do work on the object. When two or more forces act on an object, their net work is the sum of the individual works done by the forces, which is also equal to the work that would be done on the : object by the net force F net of those forces.

Work and Kinetic Energy For a particle, a change %K in the kinetic energy equals the net work W done on the particle: %K ! Kf $ Ki ! W (work – kinetic energy theorem),

(7-10)

(7-11)

Kf ! Ki " W.

: Fg

(7-12)

:

and d .

Work Done in Lifting and Lowering an Object The work Wa done by an applied force as a particle-like object is either lifted or lowered is related to the work Wg done by the gravitational force and the change %K in the object’s kinetic energy by %K ! Kf $ Ki ! Wa " Wg.

(7-15)

If Kf ! Ki , then Eq. 7-15 reduces to Wa ! $Wg,

(7-16)

which tells us that the applied force transfers as much energy to the object as the gravitational force transfers from it.

169

QU ESTIONS :

Spring Force The force Fs from a spring is : Fs

:

! $kd

(7-20)

(Hooke’s law),

:

where d is the displacement of the spring’s free end from its position when the spring is in its relaxed state (neither compressed nor extended), and k is the spring constant (a measure of the spring’s stiffness). If an x axis lies along the spring, with the origin at the location of the spring’s free end when the spring is in its relaxed state, Eq. 7-20 can be written as Fx ! $kx

must be found by integrating the force. If we assume that component Fx may depend on x but not on y or z, component Fy may depend on y but not on x or z, and component Fz may depend on z but not on x or y, then the work is W!

"

xf

xi

F x dx "

A spring force is thus a variable force: It varies with the displacement of the spring’s free end.

Work Done by a Spring Force If an object is attached to the spring’s free end, the work Ws done on the object by the spring force when the object is moved from an initial position xi to a final position xf is (7-25) Ws ! 12 kx2i $ 12 kx2f .

yf

F y dy "

yi

"

zf

(7-36)

F z dz.

zi

:

If F has only an x component, then Eq. 7-36 reduces to

(7-21)

(Hooke’s law).

"

W!

"

xf

xi

(7-32)

F(x) dx.

Power The power due to a force is the rate at which that force does work on an object. If the force does work W during a time interval %t, the average power due to the force over that time interval is Pavg !

W . %t

(7-42)

Instantaneous power is the instantaneous rate of doing work:

If xi ! 0 and xf ! x, then Eq. 7-25 becomes Ws ! $12 kx2.

(7-26) :

Work Done by a Variable Force When the force F on a particle:

like object depends on the position of the object, the work done by F on the object while the object moves from an initial position ri with coordinates (xi, yi, zi) to a final position rf with coordinates (xf , yf , zf)

P!

dW . dt

(7-43)

:

For a force F at an angle f to the direction of travel of the instantaneous velocity : v , the instantaneous power is :

P ! Fv cos & ! F " : v.

(7-47, 7-48)

Questions 1 Rank the following velocities according to the kinetic energy a particle will have with each velocity, greatest first: (a) : v ! 4iˆ " 3jˆ , (b) : v ! 5iˆ , v ! $4iˆ " 3jˆ , (c) : v ! $3iˆ " 4jˆ , (d) : v ! 3iˆ $ 4jˆ , (e) : and (f) v ! 5 m/s at 30( to the horizontal. 2 Figure 7-16a shows two horizontal forces that act on a block that is sliding to the right across a frictionless floor. Figure 7-16b shows three plots of the block’s kinetic energy K versus time t. Which of the plots best corresponds to the following three situations: (a) F1 ! F2, (b) F1 ) F2, (c) F1 * F2? K 1 2 F1 (a)

F2

y

y

vf = 5 m/s vi = 6 m/s

vi = 4 m/s

x

(a)

x

x

vf = 3 m/s

(b)

(c)

5 The graphs in Fig. 7-18 give the x component Fx of a force acting on a particle moving along an x axis. Rank them according to the work done by the force on the particle from x ! 0 to x ! x1, from most positive work first to most negative work last. Fx

Fx t

F1

F1 x1

Figure 7-16 Question 2. :

3 Is positive or negative work done by a constant force F on a par: : ticle during a straight-line displacement d if (a) the angle between F : : : and d is 30(; (b) the angle is 100(; (c) F ! 2iˆ $ 3jˆ and d ! $4iˆ ? 4 In three situations, a briefly applied horizontal force changes the velocity of a hockey puck that slides over frictionless ice. The overhead views of Fig. 7-17 indicate, for each situation, the puck’s initial speed vi, its final speed vf , and the directions of the corresponding velocity vectors. Rank the situations according to the work done on the puck by the applied force, most positive first and most negative last.

vi = 2 m/s

Figure 7-17 Question 4.

3 (b)

vf = 4 m/s

y

(a)

x1

x

–F 1

(b)

Fx

F1 x1

(c)

–F 1

–F 1 Fx

F1

Figure 7-18 Question 5.

x

x

x1 (d)

–F 1

x

170

CHAPTE R 7 KI N ETIC E N E RGY AN D WOR K

6 Figure 7-19 gives the x component Fx of a force that can act on a particle. If the particle begins at rest at x ! 0, what is its coordinate when it has (a) its greatest kinetic energy, (b) its greatest speed, and (c) zero speed? (d) What is the particle’s direction of travel after it reaches x ! 6 m?

plot in Fig. 7-21c (for kinetic energy K versus time t) best corresponds to which plot in Fig. 7-21b?

F2 F1 Fx

9 Spring A is stiffer than spring B (kA ) kB). The spring force of which spring does more work if the springs are compressed (a) the same distance and (b) by the same applied force?

x (m) 1 2 3 4 5 6 7 8 –F1

10 A glob of slime is launched or dropped from the edge of a cliff. Which of the graphs in Fig. 7-22 could possibly show how the kinetic energy of the glob changes during its flight?

–F2

Figure 7-19 Question 6.

K

7 In Fig. 7-20, a greased pig has a choice of three frictionless slides along which to slide to the ground. Rank the slides according to how much work the gravitational force does on the pig during the descent, greatest first.

K

K

t

t

(a)

(b)

(c)

F3

x

K

A

B

E

F

t

(b)

11 In three situations, a single force acts on a moving particle. Here are the velocities (at that instant) and the forces: : (1) : v ! ($4iˆ ) m/s, F ! (6iˆ $ 20jˆ ) N; (2) : v ! (2iˆ $ 3jˆ ) m/s, : : F ! ($2jˆ " 7kˆ) N; (3) : v ! ($3iˆ " jˆ ) m/s, F ! (2iˆ " 6jˆ ) N. Rank the situations according to the rate at which energy is being transferred, greatest transfer to the particle ranked first, greatest transfer from the particle ranked last.

t

C

D

t (h)

12 Figure 7-23 shows three arrangements of a block attached to identical springs that are in their relaxed state when the block is centered as shown. Rank the arrangements according to the magnitude of the net force on the block, largest first, when the block is displaced by distance d (a) to the right and (b) to the left. Rank the arrangements according to the work done on the block by the spring forces, greatest first, when the block is displaced by d (c) to the right and (d) to the left.

F4

(a) vx

t (g)

Figure 7-22 Question 10.

8 Figure 7-21a shows four situations in which a horizontal force acts on the same block, which is initially at rest. The force magnitudes are F2 ! F4 ! 2F1 ! 2F3. The horizontal component vx of the block’s velocity is shown in Fig. 7-21b for the four situations. (a) Which plot in Fig. 7-21b best corresponds to which force in Fig. 7-21a? (b) Which F2

K

t (f )

Figure 7-20 Question 7.

F1

(d)

K

t (a)

(c)

K

(e)

t

t

(b)

K

K

H

G

(c)

(1)

Figure 7-21 Question 8.

(2)

(3)

Figure 7-23 Question 12.

Problems Tutoring problem available (at instructor’s discretion) in WileyPLUS and WebAssign SSM • – •••

Worked-out solution available in Student Solutions Manual

WWW Worked-out solution is at

Number of dots indicates level of problem difficulty

ILW Interactive solution is at Additional information available in The Flying Circus of Physics and at flyingcircusofphysics.com

Module 7-1

Kinetic Energy

•1 SSM A proton (mass m ! 1.67 ' 10$27 kg) is being accelerated along a straight line at 3.6 ' 10 15 m/s2 in a machine. If the proton has an initial speed of 2.4 ' 10 7 m/s and travels 3.5 cm, what then is (a) its speed and (b) the increase in its kinetic energy?

http://www.wiley.com/college/halliday

•2 If a Saturn V rocket with an Apollo spacecraft attached had a combined mass of 2.9 ' 105 kg and reached a speed of 11.2 km/s, how much kinetic energy would it then have? •3 On August 10, 1972, a large meteorite skipped across the atmosphere above the western United States and western Canada,

171

PROB LE M S

•4 An explosion at ground level leaves a crater with a diameter that is proportional to the energy of the explosion raised to the 13 power; an explosion of 1 megaton of TNT leaves a crater with a 1 km diameter. Below Lake Huron in Michigan there appears to be an ancient impact crater with a 50 km diameter. What was the kinetic energy associated with that impact, in terms of (a) megatons of TNT (1 megaton yields 4.2 ' 1015 J) and (b) Hiroshima bomb equivalents (13 kilotons of TNT each)? (Ancient meteorite or comet impacts may have significantly altered the climate, killing off the dinosaurs and other life-forms.) ••5 A father racing his son has half the kinetic energy of the son, who has half the mass of the father. The father speeds up by 1.0 m/s and then has the same kinetic energy as the son. What are the original speeds of (a) the father and (b) the son? ••6 A bead with mass 1.8 ' 10$2 kg is moving along a wire in the positive direction of an x axis. Beginning at time t ! 0, when the bead passes through x ! 0 with speed 12 m/s, a constant force acts on the bead. Figure 7-24 indicates the bead’s position at these four times: t0 ! 0, t1 ! 1.0 s, t2 ! 2.0 s, and t3 ! 3.0 s. The bead momentarily stops at t ! 3.0 s. What is the kinetic energy of the bead at t ! 10 s? t0

t1

0

5

t2 10 x (m)

t3 15

20

Figure 7-24 Problem 6. Module 7-2 Work and Kinetic Energy •7 A 3.0 kg body is at rest on a frictionless horizontal air track : when a constant horizontal force F acting in the positive direction of an x axis along the track is applied to the body.A stroboscopic graph of the position of the body as it slides to the right is shown in Fig. 7: 25. The force F is applied to the body at t ! 0, and the graph records the position of the body at 0.50 s intervals. How much work is done : on the body by the applied force F between t ! 0 and t ! 2.0 s?

ity of 4.0 m/s in the positive x direction and some time later has a velocity of 6.0 m/s in the positive y direction. How much work is done on the canister by the 5.0 N force during this time? •10 A coin slides over a frictionless plane and across an xy coordinate system from the origin to a point with xy coordinates (3.0 m, 4.0 m) while a constant force acts on it. The force has magnitude 2.0 N and is directed at a counterclockwise angle of 100( from the positive direction of the x axis. How much work is done by the force on the coin during the displacement? ••11 A 12.0 N force with a fixed orientation does work on a particle as the particle moves through the three-dimensional dis: placement d ! (2.00iˆ $ 4.00jˆ " 3.00kˆ ) m. What is the angle between the force and the displacement if the change in the particle’s kinetic energy is (a) "30.0 J and (b) $30.0 J? Ws ••12 A can of bolts and nuts is pushed 2.00 m along an x axis by a broom along the greasy (frictionless) floor of a car repair shop in a version of shuffleboard. Figure 7-26 gives the work W done on the can by the constant horizontal force 0 1 2 from the broom, versus the can’s pox (m) sition x. The scale of the figure’s vertical axis is set by Ws ! 6.0 J. (a) Figure 7-26 Problem 12. What is the magnitude of that force? (b) If the can had an initial kinetic energy of 3.00 J, moving in the positive direction of the x axis, what is its kinetic energy at the end of the 2.00 m?

W (J)

much like a stone skipped across water. The accompanying fireball was so bright that it could be seen in the daytime sky and was brighter than the usual meteorite trail. The meteorite’s mass was about 4 ' 10 6 kg; its speed was about 15 km/s. Had it entered the atmosphere vertically, it would have hit Earth’s surface with about the same speed. (a) Calculate the meteorite’s loss of kinetic energy (in joules) that would have been associated with the vertical impact. (b) Express the energy as a multiple of the explosive energy of 1 megaton of TNT, which is 4.2 ' 1015 J. (c) The energy associated with the atomic bomb explosion over Hiroshima was equivalent to 13 kilotons of TNT. To how many Hiroshima bombs would the meteorite impact have been equivalent?

••13 A luge and its rider, with a total mass of 85 kg, emerge from a downhill track onto a horizontal straight track with an initial speed of 37 m/s. If a force slows them to a stop at a constant rate of 2.0 m/s2, (a) what magnitude F is required for the force, (b) what distance d do they travel while slowing, and (c) what work W is done on them by the force? What are (d) F, (e) d, and (f) W if they, instead, slow at 4.0 m/s2? ••14 Figure 7-27 shows an overhead view of three horizontal forces acting on a cargo canister that was initially stationary but now moves across a frictionless floor. The force magnitudes are F1 ! 3.00 N, F2 ! 4.00 N, and F3 ! 10.0 N, and the indicated angles are u2 ! 50.0( and u3 ! 35.0(. What is the net work done on the canister by the three forces during the first 4.00 m of displacement?

y

F3

θ3

F1

x

θ2 F2

Figure 7-27 Problem 14.

•8 A ice block floating in a river is pushed through a displacement : d ! (15 m)iˆ $ (12 m)jˆ along a straight embankment by rushing wa: ter, which exerts a force F ! (210 N)iˆ $ (150 N)jˆ on the block. How much work does the force do on the block during the displacement?

••15 Figure 7-28 shows three forces applied to a trunk that moves leftward by 3.00 m over a frictionless floor. The force magnitudes are F1 ! 5.00 N, F2 ! 9.00 N, and F3 ! 3.00 N, and the indicated angle is u ! 60.0(. During the displacement, (a) what is the net work done on the trunk by the three forces and (b) does the kinetic energy of the trunk increase or decrease?

•9 The only force acting on a 2.0 kg canister that is moving in an xy plane has a magnitude of 5.0 N. The canister initially has a veloc-

••16 An 8.0 kg object is moving in the positive direction of an x axis. When it passes through x ! 0, a constant force directed

t=0 0

0.5 s

1.0 s 0.2

1.5 s 0.4 x (m)

2.0 s 0.6

0.8

Figure 7-25 Problem 7.

F2

θ

F1

F3

Figure 7-28 Problem 15.

172

CHAPTE R 7 KI N ETIC E N E RGY AN D WOR K

along the axis begins to act on it. K (J) Figure 7-29 gives its kinetic energy K versus position x as it moves K 0 from x ! 0 to x ! 5.0 m; K0 ! 30.0 J. The force continues to act. What is v when the object moves back through x ! $3.0 m?

the box to move up a frictionless ramp at constant speed. How : much work is done on the box by F a when the box has moved through vertical distance h ! 0.150 m?

0

x (m)

5

Module 7-3 Work Done by Figure 7-29 Problem 16. the Gravitational Force •17 SSM WWW A helicopter lifts a 72 kg astronaut 15 m vertically from the ocean by means of a cable. The acceleration of the astronaut is g/10. How much work is done on the astronaut by (a) the force from the helicopter and (b) the gravitational force on her? Just before she reaches the helicopter, what are her (c) kinetic energy and (d) speed? •18 (a) In 1975 the roof of Montreal’s Velodrome, with a weight of 360 kN, was lifted by 10 cm so that it could be centered. How much work was done on the roof by the forces making the lift? (b) In 1960 a Tampa, Florida, mother reportedly raised one end of a car that had fallen onto her son when a jack failed. If her panic lift effectively raised 4000 N (about 14 of the car’s weight) by 5.0 cm, how much work did her force do on the car?

••20 A block is sent up a frictionless ramp along which an x axis extends upward. Figure 7-31 gives the kinetic energy of the block as a function of position x; the scale of the figure’s vertical axis is set by Ks ! 40.0 J. If the block’s initial speed is 4.00 m/s, what is the normal force on the block?

Fr

d

θ

Ks

1 x (m)

2

Figure 7-31 Problem 20. ••21 SSM A cord is used to vertically lower an initially stationary block of mass M at a constant downward acceleration of g/4. When the block has fallen a distance d, find (a) the work done by the cord’s force on the block, (b) the work done by the gravitational force on the block, (c) the kinetic energy of the block, and (d) the speed of the block.

•28 During spring semester at MIT, residents of the parallel buildings of the East Campus dorms battle one another with large catapults that are made with surgical hose mounted on a window frame. A balloon filled with dyed water is placed in a pouch attached to the hose, which is then stretched through the width of the room. Assume that the stretching of the hose obeys Hooke’s law with a spring constant of 100 N/m. If the hose is stretched by 5.00 m and then released, how much work does the force from the hose do on the balloon in the pouch by the time the hose reaches its relaxed length? ••29 In the arrangement of Fig. 7-10, we gradually pull the block from x ! 0 to x ! "3.0 cm, where it is stationary. Figure 7-35 gives

••22 A cave rescue team lifts an injured spelunker directly upward and out of a sinkhole by means of a motor-driven cable. The lift is performed in three stages, each requiring a vertical distance of 10.0 m: (a) the initially stationary spelunker is accelerated to a speed of 5.00 m/s; (b) he is then lifted at the constant speed of 5.00 m/s; (c) finally he is decelerated to zero speed. How much work is done on the 80.0 kg rescuee by the force lifting him during each stage? φ :

••23 In Fig. 7-32, a constant force F a of magnitude 82.0 N is applied to a 3.00 kg shoe box at angle & ! 53.0(, causing

Figure 7-34 Problem 25.

•27 A spring and block are in the arrangement of Fig. 7-10.When the block is pulled out to x ! "4.0 cm, we must apply a force of magnitude 360 N to hold it there.We pull the block to x ! 11 cm and then release it. How much work does the spring do on the block as the block moves from xi ! "5.0 cm to (a) x ! "3.0 cm, (b) x ! $3.0 cm, (c) x ! $5.0 cm, and (d) x ! $9.0 cm?

Figure 7-30 Problem 19.

0

•••25 In Fig. 7-34, a 0.250 kg block of cheese lies on the floor of a 900 kg elevator cab that is being pulled upward by a cable through distance d1 ! 2.40 m and then through distance d2 ! 10.5 m. (a) Through d1, if the normal force on the block from the floor has constant magnitude FN ! 3.00 N, how much work is done on the cab by the force from the cable? (b) Through d2, if the work done on the cab by the (constant) force from the cable is 92.61 kJ, what is the magnitude of FN?

Module 7-4 Work Done by a Spring Force •26 In Fig. 7-10, we must apply a force of magnitude 80 N to hold the block stationary at x ! $2.0 cm. From that position, we then slowly move the block so that our force does "4.0 J of work on the spring–block system; the block is then again stationary. What is the block’s position? (Hint: There are two answers.)

K ( J)

••19 In Fig. 7-30, a block of ice slides down a frictionless ramp at angle + ! 50( while an ice worker pulls on : the block (via a rope) with a force F r that has a magnitude of 50 N and is directed up the ramp. As the block slides through distance d ! 0.50 m along the ramp, its kinetic energy increases by 80 J. How much greater would its kinetic energy have been if the rope had not been attached to the block?

••24 In Fig. 7-33, a horizontal force : F a of magnitude 20.0 N is applied to a d 3.00 kg psychology book as the book slides a distance d ! 0.500 m up a frictionless ramp at angle u ! 30.0(. (a) y log During the displacement, what is the net ho Fa : yc s P work done on the book by F a, the graviθ tational force on the book, and the norFigure 7-33 Problem 24. mal force on the book? (b) If the book has zero kinetic energy at the start of the displacement, what is its speed at the end of the displacement?

W (J)

Ws

Fa

Figure 7-32 Problem 23.

Figure 7-35 Problem 29.

0

1

2 x (cm)

3

PROB LE M S

K (J)

••30 In Fig. 7-10a, a block of mass Ks m lies on a horizontal frictionless surface and is attached to one end of a horizontal spring (spring con0 0 0.5 1 1.5 2 stant k) whose other end is fixed. x (m) The block is initially at rest at the Figure 7-36 Problem 30. position where the spring is unstretched (x ! 0) when a con: stant horizontal force F in the positive direction of the x axis is applied to it. A plot of the resulting kinetic energy of the block versus its position x is shown in Fig. 7-36. The scale of the figure’s vertical : axis is set by Ks ! 4.0 J. (a) What is the magnitude of F ? (b) What is the value of k?

••32 Figure 7-37 gives spring force Fx versus position x for the spring – block arrangement of Fig. 710. The scale is set by Fs ! 160.0 N. We release the block at x ! 12 cm. How much work does the spring do on the block when the block moves from xi ! "8.0 cm to (a) x ! "5.0 cm, (b) x ! $5.0 cm, (c) x ! $8.0 cm, and (d) x ! $10.0 cm?

Fx Fs

–2

–1

0

•36 A 5.0 kg block moves in a straight line on a horizontal frictionless surface under the influence of a force that varies with position as shown in Fig. 7-39.The scale of the figure’s vertical axis is set by Fs ! 10.0 N. How much work is done by the force as the block moves from the origin to x ! 8.0 m?

1

2

Figure 7-37 Problem 32.

•••33 The block in Fig. 7-10a lies on a horizontal frictionless surface, and the spring constant is 50 N/m. Initially, the spring is at its relaxed length and the block is stationary at position x ! 0. Then an applied force with a constant magnitude of 3.0 N pulls the block in the positive direction of the x axis, stretching the spring until the block stops. When that stopping point is reached, what are (a) the position of the block, (b) the work that has been done on the block by the applied force, and (c) the work that has been done on the block by the spring force? During the block’s displacement, what are (d) the block’s position when its kinetic energy is maximum and (e) the value of that maximum kinetic energy? Module 7-5 Work Done by a General Variable Force •34 ILW A 10 kg brick moves along an x axis. Its acceleration as a function of its position is shown in Fig. 7-38. The scale of the figure’s vertical axis is set by as ! 20.0 m/s2. What is the net work performed on the brick by the force causing the acceleration as the brick moves from x ! 0 to x ! 8.0 m?

0 –Fs

2

4 Position (m)

Figure 7-39 Problem 36.

0

2

4

6

8

x (m)

–as

Figure 7-40 Problem 37. ••38 A 1.5 kg block is initially at rest on a horizontal frictionless surface when a horizontal force along an x axis is applied to the block. : The force is given by F (x) ! (2.5 $ x2)iˆ N, where x is in meters and the initial position of the block is x ! 0. (a) What is the kinetic energy of the block as it passes through x ! 2.0 m? (b) What is the maximum kinetic energy of the block between x ! 0 and x ! 2.0 m? : ••39 A force F ! (cx $ 3.00x2)iˆ acts on a particle as the parti: cle moves along an x axis, with F in newtons, x in meters, and c a constant.At x ! 0, the particle’s kinetic energy is 20.0 J; at x ! 3.00 m, it is 11.0 J. Find c. ••40 A can of sardines is made to move along an x axis from x ! 0.25 m to x ! 1.25 m by a force with a magnitude given by F ! exp($4x2), with x in meters and F in newtons. (Here exp is the exponential function.) How much work is done on the can by the force? ••41 A single force acts on a 3.0 kg particle-like object whose position is given by x ! 3.0t $ 4.0t2 " 1.0t3, with x in meters and t in seconds. Find the work done by the force from t ! 0 to t ! 4.0 s. •••42 Figure 7-41 shows a cord attached to a cart that can slide along a frictionless horizontal rail aligned along an x axis. The left

as

a (m/s2)

y

T

h 00

2

4 x (m)

6

8

Figure 7-38 Problem 34.

8

as

x (cm)

–Fs

Fs

••37 Figure 7-40 gives the accel: eration of a 2.00 kg particle as an applied force F a moves it from rest along an x axis from x ! 0 to x ! 9.0 m.The scale of the figure’s vertical axis is set by as ! 6.0 m/s2. How much work has the force done on the particle when the particle reaches (a) x ! 4.0 m, (b) x ! 7.0 m, and (c) x ! 9.0 m? What is the particle’s speed and direction of travel when it reaches (d) x ! 4.0 m, (e) x ! 7.0 m, and (f) x ! 9.0 m?

a (m/s2)

••31 SSM WWW The only force acting on a 2.0 kg body as it moves along a positive x axis has an x component Fx ! $6x N, with x in meters. The velocity at x ! 3.0 m is 8.0 m/s. (a) What is the velocity of the body at x ! 4.0 m? (b) At what positive value of x will the body have a velocity of 5.0 m/s?

•35 SSM WWW The force on a particle is directed along an x axis and given by F ! F0(x/x0 $ 1). Find the work done by the force in moving the particle from x ! 0 to x ! 2x0 by (a) plotting F(x) and measuring the work from the graph and (b) integrating F(x).

Force (N)

the work that our force does on the block. The scale of the figure’s vertical axis is set by Ws ! 1.0 J. We then pull the block out to x ! "5.0 cm and release it from rest. How much work does the spring do on the block when the block moves from x i ! "5.0 cm to (a) x ! "4.0 cm, (b) x ! $2.0 cm, and (c) x ! $5.0 cm?

173

x x2

x1

Figure 7-41 Problem 42.

174

CHAPTE R 7 KI N ETIC E N E RGY AN D WOR K

end of the cord is pulled over a pulley, of negligible mass and friction and at cord height h ! 1.20 m, so the cart slides from x1 ! 3.00 m to x2 ! 1.00 m. During the move, the tension in the cord is a constant 25.0 N. What is the change in the kinetic energy of the cart during the move?

: di ! (3.00 m)iˆ $ (2.00 m)jˆ " (5.00 m)kˆ to a final position of : d f ! $(5.00 m)iˆ " (4.00 m)jˆ " (7.00 m)kˆ in 4.00 s. Find (a) the work done on the object by the force in the 4.00 s interval, (b) the average power due to the force during that interval, and (c) the an: : gle between vectors d i and d f .

Module 7-6 Power •43 SSM A force of 5.0 N acts on a 15 kg body initially at rest. Compute the work done by the force in (a) the first, (b) the second, and (c) the third seconds and (d) the instantaneous power due to the force at the end of the third second.

•••52 A funny car accelerates from rest through a measured track distance in time T with the engine operating at a constant power P. If the track crew can increase the engine power by a differential amount dP, what is the change in the time required for the run?

•44 A skier is pulled by a towrope up a frictionless ski slope that makes an angle of 12( with the horizontal. The rope moves parallel to the slope with a constant speed of 1.0 m/s. The force of the rope does 900 J of work on the skier as the skier moves a distance of 8.0 m up the incline. (a) If the rope moved with a constant speed of 2.0 m/s, how much work would the force of the rope do on the skier as the skier moved a distance of 8.0 m up the incline? At what rate is the force of the rope doing work on the skier when the rope moves with a speed of (b) 1.0 m/s and (c) 2.0 m/s? •45 SSM ILW A 100 kg block is pulled at a constant speed of 5.0 m/s across a horizontal floor by an applied force of 122 N directed 37( above the horizontal. What is the rate at which the force does work on the block? •46 The loaded cab of an elevator has a mass of 3.0 ' 103 kg and moves 210 m up the shaft in 23 s at constant speed. At what average rate does the force from the cable do work on the cab? ••47 A machine carries a 4.0 kg package from an initial position : of d i ! (0.50 m)iˆ " (0.75 m)jˆ " (0.20 m)kˆ at t ! 0 to a final posi: tion of d f ! (7.50 m)iˆ " (12.0 m)jˆ " (7.20 m)kˆ at t ! 12 s. The constant force applied by the machine on the package is : F ! (2.00 N)iˆ " (4.00 N)jˆ " (6.00 N)kˆ . For that displacement, find (a) the work done on the package by the machine’s force and (b) the average power of the machine’s force on the package. ••48 A 0.30 kg ladle sliding on a horizontal frictionless surface is attached to one end of a horizontal spring (k ! 500 N/m) whose other end is fixed. The ladle has a kinetic energy of 10 J as it passes through its equilibrium position (the point at which the spring force is zero). (a) At what rate is the spring doing work on the ladle as the ladle passes through its equilibrium position? (b) At what rate is the spring doing work on the ladle when the spring is compressed 0.10 m and the ladle is moving away from the equilibrium position? ••49 SSM A fully loaded, slow-moving freight elevator has a cab with a total mass of 1200 kg, which is required to travel upward 54 m in 3.0 min, starting and ending at rest. The elevator’s counterweight has a mass of only 950 kg, and so the elevator motor must help. What average power is required of the force the motor exerts on the cab via the cable? ••50 (a) At a certain instant, a particle-like object is acted on by a : force F ! (4.0 N)iˆ $ (2.0 N)jˆ " (9.0 N)kˆ while the object’s veloc: ity is v ! $(2.0 m/s)iˆ " (4.0 m/s)kˆ . What is the instantaneous rate at which the force does work on the object? (b) At some other time, the velocity consists of only a y component. If the force is unchanged and the instantaneous power is $12 W, what is the velocity of the object? ••51 A force F ! (3.00 N)iˆ " (7.00 N)jˆ " (7.00 N)kˆ acts on a 2.00 kg mobile object that moves from an initial position of :

Additional Problems 53 Figure 7-42 shows a cold package of hot dogs sliding rightward across a frictionless floor through a distance d ! 20.0 cm while three forces act on the package. Two of them are horizontal and have the magnitudes F1 ! 5.00 N and F2 ! 1.00 N; the third is angled down by u ! 60.0( and has the magnitude F3 ! 4.00 N. (a) For the 20.0 cm displacement, what is the net work done on the package by the three applied forces, the gravitational force on the package, and the normal force on the package? (b) If the package has a mass of 2.0 kg and an initial kinetic energy of 0, what is its speed at the end of the displacement? d F2

F1

θ F3

Figure 7-42 Problem 53. 54 The only force acting on a Fx (N) 2.0 kg body as the body moves along Fs an x axis varies as shown in Fig. 7-43. 1 2 3 4 5 x (m) 0 The scale of the figure’s vertical axis is set by Fs ! 4.0 N. The velocity of the body at x ! 0 is 4.0 m/s. (a) What –Fs is the kinetic energy of the body at Figure 7-43 Problem 54. x ! 3.0 m? (b) At what value of x will the body have a kinetic energy of 8.0 J? (c) What is the maximum kinetic energy of the body between x ! 0 and x ! 5.0 m? 55 SSM A horse pulls a cart with a force of 40 lb at an angle of 30( above the horizontal and moves along at a speed of 6.0 mi/h. (a) How much work does the force do in 10 min? (b) What is the average power (in horsepower) of the force? 56 An initially stationary 2.0 kg object accelerates horizontally and uniformly to a speed of 10 m/s in 3.0 s. (a) In that 3.0 s interval, how much work is done on the object by the force accelerating it? What is the instantaneous power due to that force (b) at the end of the interval and (c) at the end of the first half of the interval? 57 A 230 kg crate hangs from the end of a rope of length L ! 12.0 m. You push horizontally on the crate with a : varying force F to move it distance d ! 4.00 m to the side (Fig. 7-44). (a) What is : the magnitude of F when the crate is in this final position? During the crate’s displacement, what are (b) the total

L

F d

Figure 7-44 Problem 57.

175

PROB LE M S

work done on it, (c) the work done by the gravitational force on the crate, and (d) the work done by the pull on the crate from the rope? (e) Knowing that the crate is motionless before and after its displacement, use the answers to (b), (c), and (d) to find the work your force : F does on the crate. (f) Why is the work of your force not equal to the product of the horizontal displacement and the answer to (a)? 58 To pull a 50 kg crate across a horizontal frictionless floor, a worker applies a force of 210 N, directed 20( above the horizontal. As the crate moves 3.0 m, what work is done on the crate by (a) the worker’s force, (b) the gravitational force, and (c) the normal force? (d) What is the total work? W0

W (J)

:

59 A force F a is applied to a bead as the bead is moved along a straight wire through displacement "5.0 cm. The mag: nitude of F a is set at a certain value, but : the angle f between F a and the bead’s displacement can be chosen. Figure 7-45 : gives the work W done by F a on the bead for a range of f values; W0 ! 25 J. : How much work is done by F a if f is (a) 64( and (b) 147(?

0

φ

Figure 7-45 Problem 59.

60 A frightened child is restrained by her mother as the child slides down a frictionless playground slide. If the force on the child from the mother is 100 N up the slide, the child’s kinetic energy increases by 30 J as she moves down the slide a distance of 1.8 m. (a) How much work is done on the child by the gravitational force during the 1.8 m descent? (b) If the child is not restrained by her mother, how much will the child’s kinetic energy increase as she comes down the slide that same distance of 1.8 m? : 61 How much work is done by a force F ! (2x N)iˆ " (3 N)jˆ , with x in meters, that moves a particle from a position : ri ! (2 m)iˆ " (3 m)jˆ to a position : r f ! $(4 m)iˆ $ (3 m)jˆ ? 62 A 250 g block is dropped onto a relaxed vertical spring that has a spring constant of k ! 2.5 N/cm (Fig. 7-46).The block becomes attached to the spring and compresses the spring 12 cm before momentarily stopping. While the spring is being compressed, what work is done on the block by (a) the gravitational force on it and (b) the spring force? (c) What is the speed of the block just before it hits the spring? (Assume that friction is negligible.) (d) If the speed at impact is doubled, what is the maximum compression of the spring?

65 In Fig. 7-47, a cord runs around two massless, frictionless pulleys. A canister with mass m ! 20 kg hangs from one pulley, and you exert a : force F on the free end of the cord. : (a) What must be the magnitude of F if you are to lift the canister at a constant speed? (b) To lift the canister by 2.0 cm, how far must you pull the free end of the cord? During that lift, what is the work done on the canister by (c) your force (via the cord) and m F (d) the gravitational force? (Hint: When a cord loops around a pulley Figure 7-47 Problem 65. as shown, it pulls on the pulley with a net force that is twice the tension in the cord.) 66 If a car of mass 1200 kg is moving along a highway at 120 km/h, what is the car’s kinetic energy as determined by someone standing alongside the highway? 67 SSM A spring with a pointer attached is hanging next to a scale marked in millimeters. Three different packages are hung from the spring, in turn, as shown in Fig. 7-48. (a) Which mark on the scale will the pointer indicate when no package is hung from the spring? (b) What is the weight W of the third package?

mm

mm

0

mm

0

0

30 40 60

W

110 N 240 N

Figure 7-48 Problem 67. Figure 7-46 Problem 62.

63 SSM To push a 25.0 kg crate up a frictionless incline, angled at 25.0( to the horizontal, a worker exerts a force of 209 N parallel to the incline. As the crate slides 1.50 m, how much work is done on the crate by (a) the worker’s applied force, (b) the gravitational force on the crate, and (c) the normal force exerted by the incline on the crate? (d) What is the total work done on the crate? 64 Boxes are transported from one location to another in a warehouse by means of a conveyor belt that moves with a constant speed of 0.50 m/s. At a certain location the conveyor belt moves for 2.0 m up an incline that makes an angle of 10( with the horizontal, then for 2.0 m horizontally, and finally for 2.0 m down an incline that makes an angle of 10( with the horizontal. Assume that a 2.0 kg box rides on the belt without slipping. At what rate is the force of the conveyor belt doing work on the box as the box moves (a) up the 10( incline, (b) horizontally, and (c) down the 10( incline?

68 An iceboat is at rest on a frictionless frozen lake when a sudden wind exerts a constant force of 200 N, toward the east, on the boat. Due to the angle of the sail, the wind causes the boat to slide in a straight line for a distance of 8.0 m in a direction 20( north of east. What is the kinetic energy of the iceboat at the end of that 8.0 m? 69 If a ski lift raises 100 passengers averaging 660 N in weight to a height of 150 m in 60.0 s, at constant speed, what average power is required of the force making the lift? : 70 A force F ! (4.0 N)iˆ " cjˆ acts on a particle as the particle : goes through displacement d ! (3.0 m)iˆ $ (2.0 m)jˆ . (Other forces also act on the particle.) What is c if the work done on the particle : by force F is (a) 0, (b) 17 J, and (c) $18 J? 71 A constant force of magnitude 10 N makes an angle of 150( (measured counterclockwise) with the positive x direction as it acts on a 2.0 kg object moving in an xy plane. How much work is done on the object by the force as the object moves from the origin to the point having position vector (2.0 m)ˆi $ (4.0 m)ˆj ?

176

CHAPTE R 7 KI N ETIC E N E RGY AN D WOR K

72 In Fig. 7-49a, a 2.0 N force is applied to a 4.0 kg block at a downward angle u as the block moves rightward through 1.0 m across a frictionless floor. Find an expression for the speed vf of the block at the end of that distance if the block’s initial velocity is (a) 0 and (b) 1.0 m/s to the right. (c) The situation in Fig. 7-49b is similar in that the block is initially moving at 1.0 m/s to the right, but now the 2.0 N force is directed downward to the left. Find an expression for the speed vf of the block at the end of the 1.0 m distance. (d) Graph all three expressions for vf versus downward angle u for u ! 0( to u ! 90(. Interpret the graphs.

θ

θ

F

F

(a)

rected along the x axis and has the x component Fax ! 9x $ 3x2, with x in meters and Fax in newtons. The case starts at rest at the position x ! 0, and it moves until it is again at rest. (a) Plot the : work F a does on the case as a function of x. (b) At what position is the work maximum, and (c) what is that maximum value? (d) At what position has the work decreased to zero? (e) At what position is the case again at rest? 79 SSM A 2.0 kg lunchbox is sent sliding over a frictionless surface, in the positive direction of an x axis along the surface. Beginning at time t ! 0, a steady wind pushes on the lunchbox in the negative direction of the x axis. Figure 7-51 shows the position x of the lunchbox as a function of time t as the wind pushes on the lunchbox. From the graph, estimate the kinetic energy of the lunchbox at (a) t ! 1.0 s and (b) t ! 5.0 s. (c) How much work does the force from the wind do on the lunchbox from t ! 1.0 s to t ! 5.0 s?

(b)

3

:

73 A force F in the positive direction of an x axis acts on an object moving along the axis. If the magnitude of the force is F ! 10e$x/2.0 : N, with x in meters, find the work done by F as the object moves from x ! 0 to x ! 2.0 m by (a) plotting F(x) and estimating the area under the curve and (b) integrating to find the work analytically. 74 A particle moves along a straight path through displacement : : d ! (8 m)iˆ " cjˆ while force F ! (2 N)iˆ $ (4 N)jˆ acts on it. (Other forces also act on the particle.) What is the value of c if the work : done by F on the particle is (a) zero, (b) positive, and (c) negative? 75 SSM What is the power of the force required to move a 4500 kg elevator cab with a load of 1800 kg upward at constant speed 3.80 m/s? 76 A 45 kg block of ice slides down a frictionless incline 1.5 m long and 0.91 m high. A worker pushes up against the ice, parallel to the incline, so that the block slides down at constant speed. (a) Find the magnitude of the worker’s force. How much work is done on the block by (b) the worker’s force, (c) the gravitational force on the block, (d) the normal force on the block from the surface of the incline, and (e) the net force on the block? 77 As a particle moves along an x axis, a force in the positive direction of the axis acts on it. Figure 7-50 shows the magnitude F of the force versus position x of the particle. The curve is given by F ! a/x2, with a ! 9.0 N #m2. Find the work done on the particle by the force as the particle moves from x ! 1.0 m to x ! 3.0 m by (a) estimating the work from the graph and (b) integrating the force function. 12

F (N)

10 8 6 4 2 0

0

1

2 x (m)

3

4

Figure 7-50 Problem 77. 78 A CD case slides along a floor in the positive direction of an : x axis while an applied force F a acts on the case. The force is di-

x (m)

Figure 7-49 Problem 72. 2 1

1

2

3

4 t (s)

5

6

7

8

Figure 7-51 Problem 79. 80 Numerical integration. A breadbox is made to move along an x axis from x ! 0.15 m to x ! 1.20 m by a force with a magnitude given by F ! exp($2x2), with x in meters and F in newtons. (Here exp is the exponential function.) How much work is done on the breadbox by the force? 81 In the block–spring arrangement of Fig. 7-10, the block’s mass is 4.00 kg and the spring constant is 500 N/m. The block is released from position xi ! 0.300 m. What are (a) the block’s speed at x ! 0, (b) the work done by the spring when the block reaches x ! 0, (c) the instantaneous power due to the spring at the release point xi , (d) the instantaneous power at x ! 0, and (e) the block’s position when the power is maximum? 82 A 4.00 kg block is pulled up a frictionless inclined plane by a 50.0 N force that is parallel to the plane, starting from rest. The normal force on the block from the plane has magnitude 13.41 N. What is the block’s speed when its displacement up the ramp is 3.00 m? 83 A spring with a spring constant of 18.0 N/cm has a cage attached to its free end. (a) How much work does the spring force do on the cage when the spring is stretched from its relaxed length by 7.60 mm? (b) How much additional work is done by the spring force when the spring is stretched by an additional 7.60 mm? : 84 A force F ! (2.00iˆ " 9.00jˆ " 5.30kˆ ) N acts on a 2.90 kg object that moves in time interval 2.10 s from an initial position : r2 ! r 1 ! (2.70iˆ $ 2.90jˆ " 5.50kˆ) m to a final position : ($4.10iˆ " 3.30jˆ " 5.40kˆ) m. Find (a) the work done on the object by the force in that time interval, (b) the average power due to the force during that time interval, and (c) the angle between vectors : r 2. r 1 and : : 85 At t ! 0, force F ! ($5.00iˆ " 5.00jˆ " 4.00kˆ ) N begins to act on a 2.00 kg particle with an initial speed of 4.00 m/s. What is the particle’s speed when its displacement from the initial point is : d ! (2.00iˆ " 2.00jˆ " 7.00kˆ ) m?

C

H

A

P

T

E

R

8

Potential Energy and Conservation of Energy 8-1

POTENTIAL ENERGY

Learning Objectives After reading this module, you should be able to . . .

8.01 Distinguish a conservative force from a nonconservative force. 8.02 For a particle moving between two points, identify that the work done by a conservative force does not depend on which path the particle takes.

8.03 Calculate the gravitational potential energy of a particle (or, more properly, a particle–Earth system). 8.04 Calculate the elastic potential energy of a block–spring system.

Key Ideas ● A force is a conservative force if the net work it does on

a particle moving around any closed path, from an initial point and then back to that point, is zero. Equivalently, a force is conservative if the net work it does on a particle moving between two points does not depend on the path taken by the particle. The gravitational force and the spring force are conservative forces; the kinetic frictional force is a nonconservative force. ● Potential energy is energy that is associated with the con-

figuration of a system in which a conservative force acts. When the conservative force does work W on a particle within the system, the change %U in the potential energy of the system is %U ! $W. If the particle moves from point xi to point xf, the change in the potential energy of the system is

"

xf

%U ! $

xi

F(x) dx.

● The potential energy associated with a system consisting of

Earth and a nearby particle is gravitational potential energy. If the particle moves from height yi to height yf , the change in the gravitational potential energy of the particle–Earth system is %U ! mg(yf $ yi) ! mg %y. ● If the reference point of the particle is set as yi ! 0 and the

corresponding gravitational potential energy of the system is set as Ui ! 0, then the gravitational potential energy U when the particle is at any height y is U(y) ! mgy. ● Elastic potential energy is the energy associated with the

state of compression or extension of an elastic object. For a spring that exerts a spring force F ! $kx when its free end has displacement x, the elastic potential energy is U(x) ! 12 kx 2. ● The reference configuration has the spring at its relaxed

length, at which x ! 0 and U ! 0.

What Is Physics? One job of physics is to identify the different types of energy in the world, especially those that are of common importance. One general type of energy is potential energy U. Technically, potential energy is energy that can be associated with the configuration (arrangement) of a system of objects that exert forces on one another. 177

178

CHAPTE R 8 POTE NTIAL E N E RGY AN D CONSE RVATION OF E N E RGY

This is a pretty formal definition of something that is actually familiar to you. An example might help better than the definition: A bungee-cord jumper plunges from a staging platform (Fig. 8-1). The system of objects consists of Earth and the jumper. The force between the objects is the gravitational force. The configuration of the system changes (the separation between the jumper and Earth decreases — that is, of course, the thrill of the jump). We can account for the jumper’s motion and increase in kinetic energy by defining a gravitational potential energy U. This is the energy associated with the state of separation between two objects that attract each other by the gravitational force, here the jumper and Earth. When the jumper begins to stretch the bungee cord near the end of the plunge, the system of objects consists of the cord and the jumper. The force between the objects is an elastic (spring-like) force. The configuration of the system changes (the cord stretches). We can account for the jumper’s decrease in kinetic energy and the cord’s increase in length by defining an elastic potential energy U. This is the energy associated with the state of compression or extension of an elastic object, here the bungee cord. Physics determines how the potential energy of a system can be calculated so that energy might be stored or put to use. For example, before any particular bungee-cord jumper takes the plunge, someone (probably a mechanical engineer) must determine the correct cord to be used by calculating the gravitational and elastic potential energies that can be expected. Then the jump is only thrilling and not fatal. Rough Guides/Greg Roden/Getty Images, Inc.

Figure 8-1 The kinetic energy of a bungeecord jumper increases during the free fall, and then the cord begins to stretch, slowing the jumper.

Work and Potential Energy In Chapter 7 we discussed the relation between work and a change in kinetic energy. Here we discuss the relation between work and a change in potential energy. Let us throw a tomato upward (Fig. 8-2). We already know that as the tomato rises, the work Wg done on the tomato by the gravitational force is negative because the force transfers energy from the kinetic energy of the tomato. We can now finish the story by saying that this energy is transferred by the gravitational force to the gravitational potential energy of the tomato – Earth system. The tomato slows, stops, and then begins to fall back down because of the gravitational force. During the fall, the transfer is reversed: The work Wg done on the tomato by the gravitational force is now positive — that force transfers energy from the gravitational potential energy of the tomato – Earth system to the kinetic energy of the tomato. For either rise or fall, the change %U in gravitational potential energy is defined as being equal to the negative of the work done on the tomato by the gravitational force. Using the general symbol W for work, we write this as (8-1)

%U ! $W.

Negative work done by the gravitational force

Figure 8-2 A tomato is thrown upward. As it rises, the gravitational force does negative work on it, decreasing its kinetic energy. As the tomato descends, the gravitational force does positive work on it, increasing its kinetic energy.

Positive work done by the gravitational force

179

8-1 POTE NTIAL E N E RGY

This equation also applies to a block – spring system, as in Fig. 8-3. If we abruptly shove the block to send it moving rightward, the spring force acts leftward and thus does negative work on the block, transferring energy from the kinetic energy of the block to the elastic potential energy of the spring – block system. The block slows and eventually stops, and then begins to move leftward because the spring force is still leftward. The transfer of energy is then reversed — it is from potential energy of the spring – block system to kinetic energy of the block.

x 0

(a)

Conservative and Nonconservative Forces Let us list the key elements of the two situations we just discussed: 1. The system consists of two or more objects. 2. A force acts between a particle-like object (tomato or block) in the system and the rest of the system. 3. When the system configuration changes, the force does work (call it W1) on the particle-like object, transferring energy between the kinetic energy K of the object and some other type of energy of the system. 4. When the configuration change is reversed, the force reverses the energy transfer, doing work W2 in the process. In a situation in which W1 ! $W2 is always true, the other type of energy is a potential energy and the force is said to be a conservative force. As you might suspect, the gravitational force and the spring force are both conservative (since otherwise we could not have spoken of gravitational potential energy and elastic potential energy, as we did previously). A force that is not conservative is called a nonconservative force. The kinetic frictional force and drag force are nonconservative. For an example, let us send a block sliding across a floor that is not frictionless. During the sliding, a kinetic frictional force from the floor slows the block by transferring energy from its kinetic energy to a type of energy called thermal energy (which has to do with the random motions of atoms and molecules). We know from experiment that this energy transfer cannot be reversed (thermal energy cannot be transferred back to kinetic energy of the block by the kinetic frictional force). Thus, although we have a system (made up of the block and the floor), a force that acts between parts of the system, and a transfer of energy by the force, the force is not conservative. Therefore, thermal energy is not a potential energy. When only conservative forces act on a particle-like object, we can greatly simplify otherwise difficult problems involving motion of the object. Let’s next develop a test for identifying conservative forces, which will provide one means for simplifying such problems.

Path Independence of Conservative Forces The primary test for determining whether a force is conservative or nonconservative is this: Let the force act on a particle that moves along any closed path, beginning at some initial position and eventually returning to that position (so that the particle makes a round trip beginning and ending at the initial position). The force is conservative only if the total energy it transfers to and from the particle during the round trip along this and any other closed path is zero. In other words: The net work done by a conservative force on a particle moving around any closed path is zero.

We know from experiment that the gravitational force passes this closedpath test. An example is the tossed tomato of Fig. 8-2. The tomato leaves the launch point with speed v0 and kinetic energy 12 mv20. The gravitational force acting

x 0

(b)

Figure 8-3 A block, attached to a spring and initially at rest at x ! 0, is set in motion toward the right. (a) As the block moves rightward (as indicated by the arrow), the spring force does negative work on it. (b) Then, as the block moves back toward x ! 0, the spring force does positive work on it.

180

CHAPTE R 8 POTE NTIAL E N E RGY AN D CONSE RVATION OF E N E RGY

b

1 a

2 (a)

b

1 a

The force is conservative. Any choice of path between the points gives the same amount of work.

2

And a round trip gives a total work of zero.

(b)

Figure 8-4 (a) As a conservative force acts on it, a particle can move from point a to point b along either path 1 or path 2. (b) The particle moves in a round trip, from point a to point b along path 1 and then back to point a along path 2.

on the tomato slows it, stops it, and then causes it to fall back down. When the tomato returns to the launch point, it again has speed v0 and kinetic energy 1 2 2 mv0. Thus, the gravitational force transfers as much energy from the tomato during the ascent as it transfers to the tomato during the descent back to the launch point. The net work done on the tomato by the gravitational force during the round trip is zero. An important result of the closed-path test is that: The work done by a conservative force on a particle moving between two points does not depend on the path taken by the particle.

For example, suppose that a particle moves from point a to point b in Fig. 8-4a along either path 1 or path 2. If only a conservative force acts on the particle, then the work done on the particle is the same along the two paths. In symbols, we can write this result as Wab,1 ! Wab,2, (8-2) where the subscript ab indicates the initial and final points, respectively, and the subscripts 1 and 2 indicate the path. This result is powerful because it allows us to simplify difficult problems when only a conservative force is involved. Suppose you need to calculate the work done by a conservative force along a given path between two points, and the calculation is difficult or even impossible without additional information. You can find the work by substituting some other path between those two points for which the calculation is easier and possible.

Proof of Equation 8-2 Figure 8-4b shows an arbitrary round trip for a particle that is acted upon by a single force. The particle moves from an initial point a to point b along path 1 and then back to point a along path 2. The force does work on the particle as the particle moves along each path. Without worrying about where positive work is done and where negative work is done, let us just represent the work done from a to b along path 1 as Wab,1 and the work done from b back to a along path 2 as Wba,2. If the force is conservative, then the net work done during the round trip must be zero: and thus

Wab,1 " Wba,2 ! 0, Wab,1 ! $Wba,2.

(8-3)

In words, the work done along the outward path must be the negative of the work done along the path back. Let us now consider the work Wab,2 done on the particle by the force when the particle moves from a to b along path 2, as indicated in Fig. 8-4a. If the force is conservative, that work is the negative of Wba,2: Wab,2 ! $Wba,2.

(8-4)

Substituting Wab,2 for $Wba,2 in Eq. 8-3, we obtain Wab,1 ! Wab,2, which is what we set out to prove.

Checkpoint 1 The figure shows three paths connecting points a : and b. A single force F does the indicated work on a particle moving along each path in the indicated direction. On the basis of this information, is force : F conservative?

–60 J a 60 J 60 J

b

181

8-1 POTE NTIAL E N E RGY

Sample Problem 8.01 Equivalent paths for calculating work, slippery cheese The main lesson of this sample problem is this: It is perfectly all right to choose an easy path instead of a hard path. Figure 8-5a shows a 2.0 kg block of slippery cheese that slides along a frictionless track from point a to point b. The cheese travels through a total distance of 2.0 m along the track, and a net vertical distance of 0.80 m. How much work is done on the cheese by the gravitational force during the slide?

The gravitational force is conservative. Any choice of path between the points gives the same amount of work. a

a

KEY IDEAS (1) We cannot calculate the work by using Eq. 7-12 (Wg ! mgd cos f). The reason is that the angle f between the : directions of the gravitational force F g and the displacement : d varies along the track in an unknown way. (Even if we did know the shape of the track and could calculate f along it, : the calculation could be very difficult.) (2) Because Fg is a conservative force, we can find the work by choosing some other path between a and b — one that makes the calculation easy. Calculations: Let us choose the dashed path in Fig. 8-5b; it consists of two straight segments. Along the horizontal segment, the angle f is a constant 90(. Even though we do not know the displacement along that horizontal segment, Eq. 7-12 tells us that the work Wh done there is Wh ! mgd cos 90( ! 0. Along the vertical segment, the displacement d is 0.80 m : : and, with Fg and d both downward, the angle f is a constant 0(. Thus, Eq. 7-12 gives us, for the work Wv done along the

(a)

vertical part of the dashed path, Wv ! mgd cos 0( ! (2.0 kg)(9.8 m/s2)(0.80 m)(1) ! 15.7 J. :

The total work done on the cheese by Fg as the cheese moves from point a to point b along the dashed path is then W ! Wh " Wv ! 0 " 15.7 J % 16 J.

Here we find equations that give the value of the two types of potential energy discussed in this chapter: gravitational potential energy and elastic potential energy. However, first we must find a general relation between a conservative force and the associated potential energy. Consider a particle-like object that is part of a system in which a conservative : force F acts. When that force does work W on the object, the change %U in the potential energy associated with the system is the negative of the work done. We wrote this fact as Eq. 8-1 (%U ! $W). For the most general case, in which the force may vary with position, we may write the work W as in Eq. 7-32: xf

xi

F(x) dx.

(Answer)

This is also the work done as the cheese slides along the track from a to b.

Determining Potential Energy Values

"

(b)

Figure 8-5 (a) A block of cheese slides along a frictionless track from point a to point b. (b) Finding the work done on the cheese by the gravitational force is easier along the dashed path than along the actual path taken by the cheese; the result is the same for both paths.

Additional examples, video, and practice available at WileyPLUS

W!

b

b

(8-5)

This equation gives the work done by the force when the object moves from point xi to point xf , changing the configuration of the system. (Because the force is conservative, the work is the same for all paths between those two points.)

182

CHAPTE R 8 POTE NTIAL E N E RGY AN D CONSE RVATION OF E N E RGY

Substituting Eq. 8-5 into Eq. 8-1, we find that the change in potential energy due to the change in configuration is, in general notation,

"

xf

%U ! $

xi

(8-6)

F(x) dx.

Gravitational Potential Energy We first consider a particle with mass m moving vertically along a y axis (the positive direction is upward). As the particle moves from point yi to point yf , : the gravitational force Fg does work on it. To find the corresponding change in the gravitational potential energy of the particle – Earth system, we use Eq. 8-6 with two changes: (1) We integrate along the y axis instead of the x axis, because the gravitational force acts vertically. (2) We substitute $mg for the force symbol F, : because Fg has the magnitude mg and is directed down the y axis. We then have

"

yf

%U ! $

yi

"

yf

($mg) dy ! mg

yi

yf

()

dy ! mg y , yi

which yields %U ! mg(yf $ yi) ! mg %y.

(8-7)

Only changes %U in gravitational potential energy (or any other type of potential energy) are physically meaningful. However, to simplify a calculation or a discussion, we sometimes would like to say that a certain gravitational potential value U is associated with a certain particle – Earth system when the particle is at a certain height y. To do so, we rewrite Eq. 8-7 as U $ Ui ! mg(y $ yi).

(8-8)

Then we take Ui to be the gravitational potential energy of the system when it is in a reference configuration in which the particle is at a reference point yi . Usually we take Ui ! 0 and yi ! 0. Doing this changes Eq. 8-8 to U( y) ! mgy

(8-9)

(gravitational potential energy).

This equation tells us: The gravitational potential energy associated with a particle – Earth system depends only on the vertical position y (or height) of the particle relative to the reference position y ! 0, not on the horizontal position.

Elastic Potential Energy We next consider the block – spring system shown in Fig. 8-3, with the block moving on the end of a spring of spring constant k. As the block moves from point xi to point xf , the spring force Fx ! $kx does work on the block. To find the corresponding change in the elastic potential energy of the block – spring system, we substitute $kx for F(x) in Eq. 8-6. We then have

"

xf

%U ! $

xi

or

($kx) dx ! k

"

xf

xi

xf

()

x dx ! 12k x2 ,

%U ! 12 kx2f $ 12 kx2i .

xi

(8-10)

To associate a potential energy value U with the block at position x, we choose the reference configuration to be when the spring is at its relaxed length and the block is at xi ! 0. Then the elastic potential energy Ui is 0, and Eq. 8-10

183

8-1 POTE NTIAL E N E RGY

becomes U $ 0 ! 12 kx 2 $ 0,

which gives us

U(x) ! 12 kx 2

(8-11)

(elastic potential energy).

Checkpoint 2 A particle is to move along an x axis from x ! 0 to x1 while a conservative force, directed along the x axis, acts on the particle. The figure shows three situations in which the x component of that force varies with x. The force has the same maximum magnitude F1 in all three situations. Rank the situations according to the change in the associated potential energy during the particle’s motion, most positive first.

F1

F1 x1 x1

(1)

x1 (2)

(3)

–F1

Sample Problem 8.02 Choosing reference level for gravitational potential energy, sloth Here is an example with this lesson plan: Generally you can choose any level to be the reference level, but once chosen, be consistent. A 2.0 kg sloth hangs 5.0 m above the ground (Fig. 8-6).

the ground, (3) at the limb, and (4) 1.0 m above the limb? Take the gravitational potential energy to be zero at y ! 0.

(a) What is the gravitational potential energy U of the sloth – Earth system if we take the reference point y ! 0 to be (1) at the ground, (2) at a balcony floor that is 3.0 m above

Once we have chosen the reference point for y ! 0, we can calculate the gravitational potential energy U of the system relative to that reference point with Eq. 8-9.

6

3

1

5

2

0

0

KEY IDEA

Calculations: For choice (1) the sloth is at y ! 5.0 m, and U ! mgy ! (2.0 kg)(9.8 m/s2)(5.0 m) ! 98 J.

(Answer)

For the other choices, the values of U are

3

0

–2

–3

(2) U ! mgy ! mg(2.0 m) ! 39 J, (3) U ! mgy ! mg(0) ! 0 J, (4) U ! mgy ! mg($1.0 m) ! $19.6 J % $20 J.

(Answer)

(b) The sloth drops to the ground. For each choice of reference point, what is the change %U in the potential energy of the sloth – Earth system due to the fall? KEY IDEA

0 (1)

–3 (2)

–5 (3)

–6 (4)

Figure 8-6 Four choices of reference point y ! 0. Each y axis is marked in units of meters. The choice affects the value of the potential energy U of the sloth – Earth system. However, it does not affect the change %U in potential energy of the system if the sloth moves by, say, falling.

The change in potential energy does not depend on the choice of the reference point for y ! 0; instead, it depends on the change in height %y. Calculation: For all four situations, we have the same %y ! $5.0 m. Thus, for (1) to (4), Eq. 8-7 tells us that %U ! mg %y ! (2.0 kg)(9.8 m/s2)($5.0 m) ! $98 J.

Additional examples, video, and practice available at WileyPLUS

(Answer)

184

CHAPTE R 8 POTE NTIAL E N E RGY AN D CONSE RVATION OF E N E RGY

8-2 CONSERVATION OF MECHANICAL ENERGY Learning Objectives After reading this module, you should be able to . . .

8.05 After first clearly defining which objects form a system, identify that the mechanical energy of the system is the sum of the kinetic energies and potential energies of those objects.

8.06 For an isolated system in which only conservative forces act, apply the conservation of mechanical energy to relate the initial potential and kinetic energies to the potential and kinetic energies at a later instant.

Key Ideas ● The mechanical energy Emec of a system is the sum of its

kinetic energy K and potential energy U: Emec ! K " U.

● An isolated system is one in which no external force causes energy changes. If only conservative forces do work within an isolated system, then the mechanical energy Emec of the

system cannot change. This principle of conservation of mechanical energy is written as K2 " U2 ! K1 " U1, in which the subscripts refer to different instants during an energy transfer process. This conservation principle can also be written as %Emec ! %K " %U ! 0.

Conservation of Mechanical Energy The mechanical energy Emec of a system is the sum of its potential energy U and the kinetic energy K of the objects within it: Emec ! K " U

(mechanical energy).

(8-12)

In this module, we examine what happens to this mechanical energy when only conservative forces cause energy transfers within the system — that is, when frictional and drag forces do not act on the objects in the system. Also, we shall assume that the system is isolated from its environment; that is, no external force from an object outside the system causes energy changes inside the system. When a conservative force does work W on an object within the system, that force transfers energy between kinetic energy K of the object and potential energy U of the system. From Eq. 7-10, the change %K in kinetic energy is %K ! W

(8-13)

and from Eq. 8-1, the change %U in potential energy is %U ! $W.

(8-14)

Combining Eqs. 8-13 and 8-14, we find that %K ! $%U.

(8-15)

In words, one of these energies increases exactly as much as the other decreases. We can rewrite Eq. 8-15 as ©AP/Wide World Photos

In olden days, a person would be tossed via a blanket to be able to see farther over the flat terrain. Nowadays, it is done just for fun. During the ascent of the person in the photograph, energy is transferred from kinetic energy to gravitational potential energy. The maximum height is reached when that transfer is complete. Then the transfer is reversed during the fall.

K2 $ K1 ! $(U2 $ U1),

(8-16)

where the subscripts refer to two different instants and thus to two different arrangements of the objects in the system. Rearranging Eq. 8-16 yields K2 " U2 ! K1 " U1

(conservation of mechanical energy).

In words, this equation says: of K and U for the sum of K and U for ! , #theanysum state of a system $ #any other state of the system$

(8-17)

185

8-2 CONSE RVATION OF M ECHAN ICAL E N E RGY

when the system is isolated and only conservative forces act on the objects in the system. In other words: In an isolated system where only conservative forces cause energy changes, the kinetic energy and potential energy can change, but their sum, the mechanical energy Emec of the system, cannot change.

This result is called the principle of conservation of mechanical energy. (Now you can see where conservative forces got their name.) With the aid of Eq. 8-15, we can write this principle in one more form, as %Emec ! %K " %U ! 0. (8-18) The principle of conservation of mechanical energy allows us to solve problems that would be quite difficult to solve using only Newton’s laws: When the mechanical energy of a system is conserved, we can relate the sum of kinetic energy and potential energy at one instant to that at another instant without considering the intermediate motion and without finding the work done by the forces involved.

Figure 8-7 shows an example in which the principle of conservation of mechanical energy can be applied: As a pendulum swings, the energy of the v = +vmax

All kinetic energy v v

v

U

U

K (h)

Figure 8-7 A pendulum, with its mass concentrated in a bob at the lower end, swings back and forth. One full cycle of the motion is shown. During the cycle the values of the potential and kinetic energies of the pendulum – Earth system vary as the bob rises and falls, but the mechanical energy Emec of the system remains constant. The energy Emec can be described as continuously shifting between the kinetic and potential forms. In stages (a) and (e), all the energy is kinetic energy. The bob then has its greatest speed and is at its lowest point. In stages (c) and (g), all the energy is potential energy. The bob then has zero speed and is at its highest point. In stages (b), (d), ( f ), and (h), half the energy is kinetic energy and half is potential energy. If the swinging involved a frictional force at the point where the pendulum is attached to the ceiling, or a drag force due to the air, then Emec would not be conserved, and eventually the pendulum would stop.

K

U

(a)

K (b)

v=0

v=0

The total energy does not change (it is conserved).

All potential energy U

K

All potential energy U

(g)

K (c)

v = –vmax

v

U

K (f )

v

U v

K (d)

All kinetic energy U

K (e)

186

CHAPTE R 8 POTE NTIAL E N E RGY AN D CONSE RVATION OF E N E RGY

pendulum – Earth system is transferred back and forth between kinetic energy K and gravitational potential energy U, with the sum K " U being constant. If we know the gravitational potential energy when the pendulum bob is at its highest point (Fig. 8-7c), Eq. 8-17 gives us the kinetic energy of the bob at the lowest point (Fig. 8-7e). For example, let us choose the lowest point as the reference point, with the gravitational potential energy U2 ! 0. Suppose then that the potential energy at the highest point is U1 ! 20 J relative to the reference point. Because the bob momentarily stops at its highest point, the kinetic energy there is K1 ! 0. Putting these values into Eq. 8-17 gives us the kinetic energy K2 at the lowest point: K2 " 0 ! 0 " 20 J

or

K2 ! 20 J.

Note that we get this result without considering the motion between the highest and lowest points (such as in Fig. 8-7d) and without finding the work done by any forces involved in the motion.

Checkpoint 3

A

The figure shows four situations — one in which an initially stationary block is dropped and three in which the B B block is allowed to slide B down frictionless ramps. (1) (2) (3) (4) (a) Rank the situations according to the kinetic energy of the block at point B, greatest first. (b) Rank them according to the speed of the block at point B, greatest first.

B

Sample Problem 8.03 Conservation of mechanical energy, water slide The huge advantage of using the conservation of energy instead of Newton’s laws of motion is that we can jump from the initial state to the final state without considering all the intermediate motion. Here is an example. In Fig. 8-8, a child of mass m is released from rest at the top of a water slide, at height h ! 8.5 m above the bottom of the slide. Assuming that the slide is frictionless because of the water on it, find the child’s speed at the bottom of the slide.

The total mechanical energy at the top is equal to the total at the bottom.

h

KEY IDEAS (1) We cannot find her speed at the bottom by using her acceleration along the slide as we might have in earlier chapters because we do not know the slope (angle) of the slide. However, because that speed is related to her kinetic energy, perhaps we can use the principle of conservation of mechanical energy to get the speed. Then we would not need to know the slope. (2) Mechanical energy is conserved in a system if the system is isolated and if only conservative forces cause energy transfers within it. Let’s check. Forces: Two forces act on the child. The gravitational force, a conservative force, does work on her. The normal force on her from the slide does no work because its direction at any point during the descent is always perpendicular to the direction in which the child moves.

Figure 8-8 A child slides down a water slide as she descends a height h.

System: Because the only force doing work on the child is the gravitational force, we choose the child – Earth system as our system, which we can take to be isolated. Thus, we have only a conservative force doing work in an isolated system, so we can use the principle of conservation of mechanical energy. Calculations: Let the mechanical energy be Emec,t when the child is at the top of the slide and Emec,b when she is at the bottom. Then the conservation principle tells us Emec,b ! Emec,t .

(8-19)

8-3 R EADI NG A POTE NTIAL E N E RGY CU RVE

To show both kinds of mechanical energy, we have Kb " Ub ! Kt " Ut , 1 2 2 mv b

or

" mgyb !

1 2 2 mv t

(8-20)

" mgyt .

Dividing by m and rearranging yield v2b ! v 2t " 2g(yt $ yb). Putting vt ! 0 and yt $ yb ! h leads to vb ! 12gh ! 1(2)(9.8 m/s2)(8.5 m) ! 13 m/s.

(Answer)

187

This is the same speed that the child would reach if she fell 8.5 m vertically. On an actual slide, some frictional forces would act and the child would not be moving quite so fast. Comments: Although this problem is hard to solve directly with Newton’s laws, using conservation of mechanical energy makes the solution much easier. However, if we were asked to find the time taken for the child to reach the bottom of the slide, energy methods would be of no use; we would need to know the shape of the slide, and we would have a difficult problem.

Additional examples, video, and practice available at WileyPLUS

8-3 READING A POTENTIAL ENERGY CURVE Learning Objectives After reading this module, you should be able to . . .

8.07 Given a particle’s potential energy as a function of its position x, determine the force on the particle. 8.08 Given a graph of potential energy versus x, determine the force on a particle. 8.09 On a graph of potential energy versus x, superimpose a line for a particle’s mechanical energy and determine the particle’s kinetic energy for any given value of x.

8.10 If a particle moves along an x axis, use a potentialenergy graph for that axis and the conservation of mechanical energy to relate the energy values at one position to those at another position. 8.11 On a potential-energy graph, identify any turning points and any regions where the particle is not allowed because of energy requirements. 8.12 Explain neutral equilibrium, stable equilibrium, and unstable equilibrium.

Key Ideas ● If we know the potential energy function U(x) for a system in which a one-dimensional force F(x) acts on a particle, we can find the force as

F(x) ! $

dU(x) . dx

● If U(x) is given on a graph, then at any value of x, the force F(x) is the negative of the slope of the curve there and the

kinetic energy of the particle is given by K(x) ! Emec $ U(x), where Emec is the mechanical energy of the system. ● A turning point is a point x at which the particle reverses its motion (there, K ! 0). ● The particle is in equilibrium at points where the slope of the U(x) curve is zero (there, F(x) ! 0).

Reading a Potential Energy Curve Once again we consider a particle that is part of a system in which a conservative force acts. This time suppose that the particle is constrained to move along an x axis while the conservative force does work on it. We want to plot the potential energy U(x) that is associated with that force and the work that it does, and then we want to consider how we can relate the plot back to the force and to the kinetic energy of the particle. However, before we discuss such plots, we need one more relationship between the force and the potential energy.

Finding the Force Analytically Equation 8-6 tells us how to find the change %U in potential energy between two points in a one-dimensional situation if we know the force F(x). Now we want to

188

CHAPTE R 8 POTE NTIAL E N E RGY AN D CONSE RVATION OF E N E RGY

go the other way; that is, we know the potential energy function U(x) and want to find the force. For one-dimensional motion, the work W done by a force that acts on a particle as the particle moves through a distance %x is F(x) %x. We can then write Eq. 8-1 as %U(x) ! $W ! $F(x) %x.

(8-21)

Solving for F(x) and passing to the differential limit yield F(x) ! $

dU(x) dx

(one-dimensional motion),

(8-22)

which is the relation we sought. We can check this result by putting U(x) ! 12 kx 2, which is the elastic potential energy function for a spring force. Equation 8-22 then yields, as expected, F(x) ! $kx, which is Hooke’s law. Similarly, we can substitute U(x) ! mgx, which is the gravitational potential energy function for a particle – Earth system, with a particle of mass m at height x above Earth’s surface. Equation 8-22 then yields F ! $mg, which is the gravitational force on the particle.

The Potential Energy Curve Figure 8-9a is a plot of a potential energy function U(x) for a system in which a particle is in one-dimensional motion while a conservative force F(x) does work on it. We can easily find F(x) by (graphically) taking the slope of the U(x) curve at various points. (Equation 8-22 tells us that F(x) is the negative of the slope of the U(x) curve.) Figure 8-9b is a plot of F(x) found in this way.

Turning Points In the absence of a nonconservative force, the mechanical energy E of a system has a constant value given by U(x) " K(x) ! Emec.

(8-23)

Here K(x) is the kinetic energy function of a particle in the system (this K(x) gives the kinetic energy as a function of the particle’s location x). We may rewrite Eq. 8-23 as K(x) ! Emec $ U(x).

(8-24)

Suppose that Emec (which has a constant value, remember) happens to be 5.0 J. It would be represented in Fig. 8-9c by a horizontal line that runs through the value 5.0 J on the energy axis. (It is, in fact, shown there.) Equation 8-24 and Fig. 8-9d tell us how to determine the kinetic energy K for any location x of the particle: On the U(x) curve, find U for that location x and then subtract U from Emec. In Fig. 8-9e for example, if the particle is at any point to the right of x5, then K ! 1.0 J. The value of K is greatest (5.0 J) when the particle is at x2 and least (0 J) when the particle is at x1. Since K can never be negative (because v2 is always positive), the particle can never move to the left of x1, where Emec $ U is negative. Instead, as the particle moves toward x1 from x2, K decreases (the particle slows) until K ! 0 at x1 (the particle stops there). Note that when the particle reaches x1, the force on the particle, given by Eq. 8-22, is positive (because the slope dU/dx is negative). This means that the particle does not remain at x1 but instead begins to move to the right, opposite its earlier motion. Hence x1 is a turning point, a place where K ! 0 (because U ! E) and the particle changes direction. There is no turning point (where K ! 0) on the right side of the graph. When the particle heads to the right, it will continue indefinitely.

189

8-3 R EADI NG A POTE NTIAL E N E RGY CU RVE

A U (J) U(x)

This is a plot of the potential energy U versus position x.

Force is equal to the negative of the slope of the U(x) plot.

6

Strong force, +x direction

F (N)

5 +

4 3

x1

2 1 x2

x3

x4

x

x5

U (J), Emec (J)

U(x) Emec = 5.0 J

6

The difference between the total energy and the potential energy is the U(x) kinetic energy K. Emec = 5.0 J

6

5

5

4

4

3

3

2

2

K

1

1 x1

x2

x3

x4

x

x5

x1

(d)

At this position, K is zero (a turning point). The particle cannot go farther to the left. U (J), Emec (J)

Emec = 5.0 J

x2

x3

x4

x5

x

For either of these three choices for Emec, the particle is trapped (cannot escape left or right).

U (J), Emec (J)

At this position, K is greatest and the particle is moving the fastest.

6

6

5

5 K = 5.0 J at x2

4

4 K = 1.0 J at x > x5

3

(e)

x

x5

Mild force, –x direction

(b)

The flat line shows a given value of the total mechanical energy Emec.

U (J), Emec (J)

(c)

x3 x4

– x1

(a)

x2

3

2

2

1

1 x1

x2

x3

x4

x5

x

(f )

x1

x2

x3

x4

x5

x

Figure 8-9 (a) A plot of U(x), the potential energy function of a system containing a particle confined to move along an x axis. There is no friction, so mechanical energy is conserved. (b) A plot of the force F(x) acting on the particle, derived from the potential energy plot by taking its slope at various points. (c)–(e) How to determine the kinetic energy. ( f ) The U(x) plot of (a) with three possible values of Emec shown. In WileyPLUS, this figure is available as an animation with voiceover.

190

CHAPTE R 8 POTE NTIAL E N E RGY AN D CONSE RVATION OF E N E RGY

Equilibrium Points Figure 8-9f shows three different values for Emec superposed on the plot of the potential energy function U(x) of Fig. 8-9a. Let us see how they change the situation. If Emec ! 4.0 J (purple line), the turning point shifts from x1 to a point between x1 and x2. Also, at any point to the right of x5, the system’s mechanical energy is equal to its potential energy; thus, the particle has no kinetic energy and (by Eq. 8-22) no force acts on it, and so it must be stationary. A particle at such a position is said to be in neutral equilibrium. (A marble placed on a horizontal tabletop is in that state.) If Emec ! 3.0 J (pink line), there are two turning points: One is between x1 and x2, and the other is between x4 and x5. In addition, x3 is a point at which K ! 0. If the particle is located exactly there, the force on it is also zero, and the particle remains stationary. However, if it is displaced even slightly in either direction, a nonzero force pushes it farther in the same direction, and the particle continues to move. A particle at such a position is said to be in unstable equilibrium. (A marble balanced on top of a bowling ball is an example.) Next consider the particle’s behavior if Emec ! 1.0 J (green line). If we place it at x4, it is stuck there. It cannot move left or right on its own because to do so would require a negative kinetic energy. If we push it slightly left or right, a restoring force appears that moves it back to x4. A particle at such a position is said to be in stable equilibrium. (A marble placed at the bottom of a hemispherical bowl is an example.) If we place the particle in the cup-like potential well centered at x2, it is between two turning points. It can still move somewhat, but only partway to x1 or x3.

U(x) (J)

Checkpoint 4 The figure gives the potential energy function U(x) for a system in which a particle is in onedimensional motion. (a) Rank regions AB, BC, and CD according to the magnitude of the force on the particle, greatest first. (b) What is the direction of the force when the particle is in region AB?

5 3 1 A

B

x

CD

Sample Problem 8.04 Reading a potential energy graph A 2.00 kg particle moves along an x axis in one-dimensional motion while a conservative force along that axis acts on it. The potential energy U(x) associated with the force is plotted in Fig. 8-10a. That is, if the particle were placed at any position between x ! 0 and x ! 7.00 m, it would have the plotted value of U. At x ! 6.5 m, the particle has velocity v:0 ! ($4.00 m/s) iˆ . (a) From Fig. 8-10a, determine the particle’s speed at x1 ! 4.5 m. KEY IDEAS (1) The particle’s kinetic energy is given by Eq. 7-1 (K ! 12mv 2). (2) Because only a conservative force acts on the particle, the mechanical energy Emec (! K " U) is conserved as the particle moves. (3) Therefore, on a plot of U(x) such as Fig. 8-10a, the kinetic energy is equal to the difference between Emec and U.

Calculations: At x ! 6.5 m, the particle has kinetic energy K0 ! 12mv 20 ! 12(2.00 kg)(4.00 m/s)2 ! 16.0 J. Because the potential energy there is U ! 0, the mechanical energy is Emec ! K0 " U0 ! 16.0 J " 0 ! 16.0 J. This value for Emec is plotted as a horizontal line in Fig. 8-10a. From that figure we see that at x ! 4.5 m, the potential energy is U1 ! 7.0 J . The kinetic energy K1 is the difference between Emec and U1 : K1 ! Emec $ U 1 ! 16.0 J $ 7.0 J ! 9.0 J. Because K1 ! 12 mv 21 , we find v1 ! 3.0 m/s. (b) Where is the particle’s turning point located?

(Answer)

8-4 WOR K DON E ON A SYSTE M BY AN EXTE R NAL FORCE

20

The turning point is where the force momentarily stops and then reverses the particle’s motion. That is, it is where the particle momentarily has v ! 0 and thus K ! 0.

16

Calculations: Because K is the difference between Emec and U , we want the point in Fig. 8-10a where the plot of U rises to meet the horizontal line of Emec, as shown in Fig. 8-10b. Because the plot of U is a straight line in Fig. 8-10b, we can draw nested right triangles as shown and then write the proportionality of distances 20 $ 7.0 16 $ 7.0 , ! d 4.0 $ 1.0

Emec = 16 J

U ( J)

KEY IDEA

K1

0

1

4 x (m)

5

6

(c) Evaluate the force acting on the particle when it is in the region 1.9 m * x * 4.0 m.

U ( J) 20 Turning point

7

1

d

4

x (m)

The kinetic energy is zero at the turning point (the particle speed is zero).

(b)

KEY IDEA The force is given by Eq. 8-22 (F(x) ! $dU(x)/dx): The force is equal to the negative of the slope on a graph of U(x). Calculations: For the graph of Fig. 8-10b, we see that for the range 1.0 m * x * 4.0 m the force is F!$

7

(a)

16

(Answer)

Kinetic energy is the difference between the total energy and the potential energy.

K0

7

which gives us d ! 2.08 m. Thus, the turning point is at x ! 4.0 m $ d ! 1.9 m.

191

20 J $ 7.0 J ! 4.3 N. 1.0 m $ 4.0 m

(Answer)

Figure 8-10 (a) A plot of potential energy U versus position x. (b) A section of the plot used to find where the particle turns around.

Thus, the force has magnitude 4.3 N and is in the positive direction of the x axis. This result is consistent with the fact that the initially leftward-moving particle is stopped by the force and then sent rightward.

Additional examples, video, and practice available at WileyPLUS

8-4 WORK DONE ON A SYSTEM BY AN EXTERNAL FORCE Learning Objectives After reading this module, you should be able to . . .

8.13 When work is done on a system by an external force with no friction involved, determine the changes in kinetic energy and potential energy.

8.14 When work is done on a system by an external force with friction involved, relate that work to the changes in kinetic energy, potential energy, and thermal energy.

Key Ideas ● Work W is energy transferred to or from a system by means of an external force acting on the system. ● When more than one force acts on a system, their net work is the transferred energy. ● When friction is not involved, the work done on the system and the change %Emec in the mechanical energy of the system are equal: W ! %Emec ! %K " %U.

● When a kinetic frictional force acts within the system, then

the thermal energy Eth of the system changes. (This energy is associated with the random motion of atoms and molecules in the system.) The work done on the system is then W ! %Emec " %Eth. ● The change %Eth is related to the magnitude fk of the frictional

force and the magnitude d of the displacement caused by the external force by %Eth ! fkd.

192

CHAPTE R 8 POTE NTIAL E N E RGY AN D CONSE RVATION OF E N E RGY

System

Work Done on a System by an External Force In Chapter 7, we defined work as being energy transferred to or from an object by means of a force acting on the object. We can now extend that definition to an external force acting on a system of objects.

Positive W (a)

Work is energy transferred to or from a system by means of an external force acting on that system.

System

Negative W (b)

Figure 8-11 (a) Positive work W done on an arbitrary system means a transfer of energy to the system. (b) Negative work W means a transfer of energy from the system.

Figure 8-11a represents positive work (a transfer of energy to a system), and Fig. 8-11b represents negative work (a transfer of energy from a system). When more than one force acts on a system, their net work is the energy transferred to or from the system. These transfers are like transfers of money to and from a bank account. If a system consists of a single particle or particle-like object, as in Chapter 7, the work done on the system by a force can change only the kinetic energy of the system. The energy statement for such transfers is the work – kinetic energy theorem of Eq. 7-10 (%K ! W); that is, a single particle has only one energy account, called kinetic energy. External forces can transfer energy into or out of that account. If a system is more complicated, however, an external force can change other forms of energy (such as potential energy); that is, a more complicated system can have multiple energy accounts. Let us find energy statements for such systems by examining two basic situations, one that does not involve friction and one that does.

No Friction Involved To compete in a bowling-ball-hurling contest, you first squat and cup your hands under the ball on the floor. Then you rapidly straighten up while also pulling your hands up sharply, launching the ball upward at about face level. During your upward motion, your applied force on the ball obviously does work; that is, it is an external force that transfers energy, but to what system? To answer, we check to see which energies change. There is a change %K in the ball’s kinetic energy and, because the ball and Earth become more separated, there is a change %U in the gravitational potential energy of the ball – Earth system. To include both changes, we need to consider the ball – Earth system. Then your force is an external force doing work on that system, and the work is W ! %K " %U, (8-25) Your lifting force transfers energy to kinetic energy and potential energy.

W

or Ball–Earth system

∆Emec = ∆K + ∆U

W ! %Emec

(work done on system, no friction involved),

(8-26)

where %Emec is the change in the mechanical energy of the system. These two equations, which are represented in Fig. 8-12, are equivalent energy statements for work done on a system by an external force when friction is not involved.

Friction Involved :

Figure 8-12 Positive work W is done on a system of a bowling ball and Earth, causing a change %Emec in the mechanical energy of the system, a change %K in the ball’s kinetic energy, and a change %U in the system’s gravitational potential energy.

We next consider the example in Fig. 8-13a. A constant horizontal force F pulls a block along an x axis and through a displacement of magnitude d, increasing the block’s velocity from : v 0 to : v . During the motion, a constant kinetic frictional : force f k from the floor acts on the block. Let us first choose the block as our system and apply Newton’s second law to it. We can write that law for components along the x axis (Fnet, x ! max) as F $ fk ! ma.

(8-27)

8-4 WOR K DON E ON A SYSTE M BY AN EXTE R NAL FORCE

The applied force supplies energy. The frictional force transfers some of it to thermal energy. v0

fk

So, the work done by the applied force goes into kinetic energy and also thermal energy. Block–floor system

v

F

∆Emec x

W ∆Eth

d (b)

(a)

:

Figure 8-13 (a) A block is pulled across a floor by force F while a kinetic frictional : : force f k opposes the motion. The block has velocity : v 0 at the start of a displacement d : and velocity v at the end of the displacement. (b) Positive work W is done on the : block – floor system by force F , resulting in a change %Emec in the block’s mechanical energy and a change %Eth in the thermal energy of the block and floor.

Because the forces are constant, the acceleration : a is also constant. Thus, we can use Eq. 2-16 to write v2 ! v 20 " 2ad. Solving this equation for a, substituting the result into Eq. 8-27, and rearranging then give us Fd ! 12 mv2 $ 12 mv20 " fk d

(8-28)

or, because 21 mv 2 $ 12 mv 20 ! %K for the block, (8-29)

Fd ! %K " fk d.

In a more general situation (say, one in which the block is moving up a ramp), there can be a change in potential energy. To include such a possible change, we generalize Eq. 8-29 by writing Fd ! %Emec " fk d.

(8-30)

By experiment we find that the block and the portion of the floor along which it slides become warmer as the block slides. As we shall discuss in Chapter 18, the temperature of an object is related to the object’s thermal energy Eth (the energy associated with the random motion of the atoms and molecules in the object). Here, the thermal energy of the block and floor increases because (1) there is friction between them and (2) there is sliding. Recall that friction is due to the cold-welding between two surfaces. As the block slides over the floor, the sliding causes repeated tearing and re-forming of the welds between the block and the floor, which makes the block and floor warmer. Thus, the sliding increases their thermal energy Eth. Through experiment, we find that the increase %Eth in thermal energy is equal to the product of the magnitudes fk and d: %Eth ! fk d

(increase in thermal energy by sliding).

(8-31)

Thus, we can rewrite Eq. 8-30 as Fd ! %Emec " %Eth. :

(8-32)

Fd is the work W done by the external force F (the energy transferred by the force), but on which system is the work done (where are the energy transfers made)? To answer, we check to see which energies change. The block’s mechanical energy

193

194

CHAPTE R 8 POTE NTIAL E N E RGY AN D CONSE RVATION OF E N E RGY

changes, and the thermal energies of the block and floor also change. Therefore, the : work done by force F is done on the block–floor system.That work is W ! %Emec " %Eth

(work done on system, friction involved).

(8-33)

This equation, which is represented in Fig. 8-13b, is the energy statement for the work done on a system by an external force when friction is involved.

Checkpoint 5 In three trials, a block is pushed Trial F Result on Block’s Speed by a horizontal applied force across a floor that is not frictiona 5.0 N decreases less, as in Fig. 8-13a. The magnib 7.0 N remains constant tudes F of the applied force and c 8.0 N increases the results of the pushing on the block’s speed are given in the table. In all three trials, the block is pushed through the same distance d. Rank the three trials according to the change in the thermal energy of the block and floor that occurs in that distance d, greatest first.

Sample Problem 8.05 Work, friction, change in thermal energy, cabbage heads A food shipper pushes a wood crate of cabbage heads (total mass m ! 14 kg) across a concrete floor with a constant : horizontal force F of magnitude 40 N. In a straight-line displacement of magnitude d ! 0.50 m, the speed of the crate decreases from v0 ! 0.60 m/s to v ! 0.20 m/s. :

(a) How much work is done by force F , and on what system does it do the work?

be friction and a change %Eth in thermal energy of the crate and the floor. Therefore, the system on which the work is done is the crate – floor system, because both energy changes occur in that system. (b) What is the increase %Eth in the thermal energy of the crate and floor? KEY IDEA

KEY IDEA

:

:

Because the applied force F is constant, we can calculate the work it does by using Eq. 7-7 (W ! Fd cos &). Calculation: Substituting given data, including the fact that : : force F and displacement d are in the same direction, we find W ! Fd cos f ! (40 N)(0.50 m) cos 0( ! 20 J.

(Answer)

Reasoning: To determine the system on which the work is done, let’s check which energies change. Because the crate’s speed changes, there is certainly a change %K in the crate’s kinetic energy. Is there friction between the floor and the : crate, and thus a change in thermal energy? Note that F and the crate’s velocity have the same direction. Thus, if there is : no friction, then F should be accelerating the crate to a greater speed. However, the crate is slowing, so there must

We can relate %Eth to the work W done by F with the energy statement of Eq. 8-33 for a system that involves friction: W ! %Emec " %Eth.

(8-34)

Calculations: We know the value of W from (a). The change %Emec in the crate’s mechanical energy is just the change in its kinetic energy because no potential energy changes occur, so we have %Emec ! %K ! 12 mv 2 $ 12 mv 20. Substituting this into Eq. 8-34 and solving for %Eth, we find %Eth ! W $ (12 mv 2 $ 12 mv 20) ! W $ 12 m(v2 $ v 20) ! 20 J $ 12(14 kg)[(0.20 m/s)2 $ (0.60 m/s)2] ! 22.2 J % 22 J. (Answer) Without further experiments, we cannot say how much of this thermal energy ends up in the crate and how much in the floor. We simply know the total amount.

Additional examples, video, and practice available at WileyPLUS

8-5 CONSE RVATION OF E N E RGY

195

8-5 CONSERVATION OF ENERGY Learning Objectives After reading this module, you should be able to . . .

8.15 For an isolated system (no net external force), apply the conservation of energy to relate the initial total energy (energies of all kinds) to the total energy at a later instant. 8.16 For a nonisolated system, relate the work done on the system by a net external force to the changes in the various types of energies within the system.

8.17 Apply the relationship between average power, the associated energy transfer, and the time interval in which that transfer is made. 8.18 Given an energy transfer as a function of time (either as an equation or a graph), determine the instantaneous power (the transfer at any given instant).

Key Ideas ● The total energy E of a system (the sum of its mechanical energy and its internal energies, including thermal energy) can change only by amounts of energy that are transferred to or from the system. This experimental fact is known as the law of conservation of energy. ● If work W is done on the system, then

W ! %E ! %Emec " %Eth " %Eint.

● The power due to a force is the rate at which that force transfers energy. If an amount of energy %E is transferred by a force in an amount of time %t, the average power of the force is %E Pavg ! . %t ● The instantaneous power due to a force is

If the system is isolated (W ! 0), this gives P!

%Emec " %Eth " %Eint ! 0 and

Emec,2 ! Emec,1 $ %Eth $ %Eint,

where the subscripts 1 and 2 refer to two different instants.

On a graph of energy E versus time t, the power is the slope of the plot at any given time.

Conservation of Energy We now have discussed several situations in which energy is transferred to or from objects and systems, much like money is transferred between accounts. In each situation we assume that the energy that was involved could always be accounted for; that is, energy could not magically appear or disappear. In more formal language, we assumed (correctly) that energy obeys a law called the law of conservation of energy, which is concerned with the total energy E of a system. That total is the sum of the system’s mechanical energy, thermal energy, and any type of internal energy in addition to thermal energy. (We have not yet discussed other types of internal energy.) The law states that The total energy E of a system can change only by amounts of energy that are transferred to or from the system.

The only type of energy transfer that we have considered is work W done on a system by an external force. Thus, for us at this point, this law states that W ! %E ! %Emec " %Eth " %Eint,

dE . dt

(8-35)

where %Emec is any change in the mechanical energy of the system, %Eth is any change in the thermal energy of the system, and %Eint is any change in any other type of internal energy of the system. Included in %Emec are changes %K in kinetic energy and changes %U in potential energy (elastic, gravitational, or any other type we might find). This law of conservation of energy is not something we have derived from basic physics principles. Rather, it is a law based on countless experiments.

196

CHAPTE R 8 POTE NTIAL E N E RGY AN D CONSE RVATION OF E N E RGY

Scientists and engineers have never found an exception to it. Energy simply cannot magically appear or disappear.

Isolated System If a system is isolated from its environment, there can be no energy transfers to or from it. For that case, the law of conservation of energy states: The total energy E of an isolated system cannot change.

Tyler Stableford/The Image Bank/Getty Images

Figure 8-14 To descend, the rock climber must transfer energy from the gravitational potential energy of a system consisting of him, his gear, and Earth. He has wrapped the rope around metal rings so that the rope rubs against the rings. This allows most of the transferred energy to go to the thermal energy of the rope and rings rather than to his kinetic energy.

Many energy transfers may be going on within an isolated system — between, say, kinetic energy and a potential energy or between kinetic energy and thermal energy. However, the total of all the types of energy in the system cannot change. Here again, energy cannot magically appear or disappear. We can use the rock climber in Fig. 8-14 as an example, approximating him, his gear, and Earth as an isolated system. As he rappels down the rock face, changing the configuration of the system, he needs to control the transfer of energy from the gravitational potential energy of the system. (That energy cannot just disappear.) Some of it is transferred to his kinetic energy. However, he obviously does not want very much transferred to that type or he will be moving too quickly, so he has wrapped the rope around metal rings to produce friction between the rope and the rings as he moves down. The sliding of the rings on the rope then transfers the gravitational potential energy of the system to thermal energy of the rings and rope in a way that he can control. The total energy of the climber – gear – Earth system (the total of its gravitational potential energy, kinetic energy, and thermal energy) does not change during his descent. For an isolated system, the law of conservation of energy can be written in two ways. First, by setting W ! 0 in Eq. 8-35, we get %Emec " %Eth " %Eint ! 0

(isolated system).

(8-36)

We can also let %Emec ! Emec,2 $ Emec,1, where the subscripts 1 and 2 refer to two different instants — say, before and after a certain process has occurred. Then Eq. 8-36 becomes Emec,2 ! Emec,1 $ %Eth $ %Eint. (8-37) Equation 8-37 tells us: In an isolated system, we can relate the total energy at one instant to the total energy at another instant without considering the energies at intermediate times.

This fact can be a very powerful tool in solving problems about isolated systems when you need to relate energies of a system before and after a certain process occurs in the system. In Module 8-2, we discussed a special situation for isolated systems — namely, the situation in which nonconservative forces (such as a kinetic frictional force) do not act within them. In that special situation, %Eth and %Eint are both zero, and so Eq. 8-37 reduces to Eq. 8-18. In other words, the mechanical energy of an isolated system is conserved when nonconservative forces do not act in it.

External Forces and Internal Energy Transfers An external force can change the kinetic energy or potential energy of an object without doing work on the object — that is, without transferring energy to the object. Instead, the force is responsible for transfers of energy from one type to another inside the object.

197

8-5 CONSE RVATION OF E N E RGY

Her push on the rail causes a transfer of internal energy to kinetic energy.

F

φ

v

F φ v0

Ice (a)

v

d

(b)

x (c)

Figure 8-15 (a) As a skater pushes herself away from a railing, the force on her from : the railing is F . (b) After the skater leaves the railing, she has velocity v: . (c) External : force F acts on the skater, at angle f with a horizontal x axis. When the skater goes : through displacement d , her velocity is changed from v:0 (! 0) to v: by the horizontal : component of F .

Figure 8-15 shows an example. An initially stationary ice-skater pushes away from a railing and then slides over the ice (Figs. 8-15a and b). Her kinetic energy : increases because of an external force F on her from the rail. However, that force does not transfer energy from the rail to her. Thus, the force does no work on her. Rather, her kinetic energy increases as a result of internal transfers from the biochemical energy in her muscles. Figure 8-16 shows another example. An engine increases the speed of a car with four-wheel drive (all four wheels are made to turn by the engine). During the acceleration, the engine causes the tires to push backward on the road sur: face. This push produces frictional forces f that act on each tire in the forward : direction. The net external force F from the road, which is the sum of these fric: tional forces, accelerates the car, increasing its kinetic energy. However, F does not transfer energy from the road to the car and so does no work on the car. Rather, the car’s kinetic energy increases as a result of internal transfers from the energy stored in the fuel. : In situations like these two, we can sometimes relate the external force F on an object to the change in the object’s mechanical energy if we can simplify the situation. Consider the ice-skater example. During her push through distance d in Fig. 8-15c, we can simplify by assuming that the acceleration is constant, her : speed changing from v0 ! 0 to v. (That is, we assume F has constant magnitude F and angle f.) After the push, we can simplify the skater as being a particle and neglect the fact that the exertions of her muscles have increased the thermal energy in her muscles and changed other physiological features. Then we can apply Eq. 7-5 (12 mv2 $ 12 mv20 ! Fxd) to write K $ K0 ! (F cos f)d, or

%K ! Fd cos f.

(8-38)

If the situation also involves a change in the elevation of an object, we can include the change %U in gravitational potential energy by writing %U " %K ! Fd cos f.

acom

(8-39)

The force on the right side of this equation does no work on the object but is still responsible for the changes in energy shown on the left side.

Power Now that you have seen how energy can be transferred from one type to another, we can expand the definition of power given in Module 7-6. There power is

f

f

Figure 8-16 A vehicle accelerates to the right using four-wheel drive. The road exerts four frictional forces (two of them shown) on the bottom surfaces of the tires. Taken together, these four forces make up : the net external force F acting on the car.

198

CHAPTE R 8 POTE NTIAL E N E RGY AN D CONSE RVATION OF E N E RGY

defined as the rate at which work is done by a force. In a more general sense, power P is the rate at which energy is transferred by a force from one type to another. If an amount of energy %E is transferred in an amount of time %t, the average power due to the force is Pavg !

%E . %t

(8-40)

Similarly, the instantaneous power due to the force is P!

dE . dt

(8-41)

Sample Problem 8.06 Lots of energies at an amusement park water slide Figure 8-17 shows a water-slide ride in which a glider is shot by a spring along a water-drenched (frictionless) track that takes the glider from a horizontal section down to ground level. As the glider then moves along ground-level track, it is gradually brought to rest by friction. The total mass of the glider and its rider is m ! 200 kg, the initial compression of the spring is d ! 5.00 m, the spring constant is k ! 3.20 ' 103 N/m, the initial height is h ! 35.0 m, and the coefficient of kinetic friction along the ground-level track is mk ! 0.800. Through what distance L does the glider slide along the ground-level track until it stops? KEY IDEAS Before we touch a calculator and start plugging numbers into equations, we need to examine all the forces and then determine what our system should be. Only then can we decide what equation to write. Do we have an isolated system (our equation would be for the conservation of energy) or a system on which an external force does work (our equation would relate that work to the system’s change in energy)? Forces: The normal force on the glider from the track does no work on the glider because the direction of this force is always perpendicular to the direction of the glider’s displacement. The gravitational force does work on the glider, and because the force is conservative we can associate a potential energy with it. As the spring pushes

k m!0

h

Figure 8-17 A spring-loaded amusement park water slide.

L mk

on the glider to get it moving, a spring force does work on it, transferring energy from the elastic potential energy of the compressed spring to kinetic energy of the glider. The spring force also pushes against a rigid wall. Because there is friction between the glider and the ground-level track, the sliding of the glider along that track section increases their thermal energies. System: Let’s take the system to contain all the interacting bodies: glider, track, spring, Earth, and wall. Then, because all the force interactions are within the system, the system is isolated and thus its total energy cannot change. So, the equation we should use is not that of some external force doing work on the system. Rather, it is a conservation of energy. We write this in the form of Eq. 8-37: Emec,2 ! Emec,1 $ %Eth.

(8-42)

This is like a money equation: The final money is equal to the initial money minus the amount stolen away by a thief. Here, the final mechanical energy is equal to the initial mechanical energy minus the amount stolen away by friction. None has magically appeared or disappeared. Calculations: Now that we have an equation, let’s find distance L. Let subscript 1 correspond to the initial state of the glider (when it is still on the compressed spring) and subscript 2 correspond to the final state of the glider (when it has come to rest on the ground-level track). For both states, the mechanical energy of the system is the sum of any potential energy and any kinetic energy. We have two types of potential energy: the elastic potential energy (Ue ! 12kx2) associated with the compressed spring and the gravitational potential energy (Ug ! mgy) associated with the glider’s elevation. For the latter, let’s take ground level as the reference level. That means that the glider is initially at height y ! h and finally at height y ! 0. In the initial state, with the glider stationary and elevated and the spring compressed, the energy is Emec,1 ! K1 " Ue1 " Ug1 ! 0 " 12kd2 " mgh.

(8-43)

199

R EVI EW & SU M MARY

In the final state, with the spring now in its relaxed state and the glider again stationary but no longer elevated, the final mechanical energy of the system is Emec,2 ! K2 " Ue2 " Ug2 ! 0 " 0 " 0.

(8-44)

Let’s next go after the change %Eth of the thermal energy of the glider and ground-level track. From Eq. 8-31, we can substitute for %Eth with fkL (the product of the frictional force magnitude and the distance of rubbing). From Eq. 6-2, we know that fk ! mkFN, where FN is the normal force. Because the glider moves horizontally through the region with friction, the magnitude of FN is equal to mg (the upward force matches the downward force). So, the friction’s theft from the mechanical energy amounts to %Eth ! mkmgL.

(8-45)

(By the way, without further experiments, we cannot say how much of this thermal energy ends up in the glider and how much in the track. We simply know the total amount.)

Substituting Eqs. 8-43 through 8-45 into Eq. 8-42, we find 0 ! 21kd2 " mgh $ mkmgL, and L! !

(8-46)

h kd2 " mk 2mkmg (3.20 ' 103 N/m)(5.00 m)2 35 m " 2(0.800)(200 kg)(9.8 m/s2) 0.800

! 69.3 m.

(Answer)

Finally, note how algebraically simple our solution is. By carefully defining a system and realizing that we have an isolated system, we get to use the law of the conservation of energy. That means we can relate the initial and final states of the system with no consideration of the intermediate states. In particular, we did not need to consider the glider as it slides over the uneven track. If we had, instead, applied Newton’s second law to the motion, we would have had to know the details of the track and would have faced a far more difficult calculation.

Additional examples, video, and practice available at WileyPLUS

Review & Summary Conservative Forces A force is a conservative force if the net work it does on a particle moving around any closed path, from an initial point and then back to that point, is zero. Equivalently, a force is conservative if the net work it does on a particle moving between two points does not depend on the path taken by the particle. The gravitational force and the spring force are conservative forces; the kinetic frictional force is a nonconservative force.

Potential Energy A potential energy is energy that is associated

cle is at any height y is U(y) ! mgy.

(8-9)

Elastic Potential Energy Elastic potential energy is the energy associated with the state of compression or extension of an elastic object. For a spring that exerts a spring force F ! $kx when its free end has displacement x, the elastic potential energy is U(x) ! 12 kx 2.

(8-11)

with the configuration of a system in which a conservative force acts. When the conservative force does work W on a particle within the system, the change %U in the potential energy of the system is

The reference configuration has the spring at its relaxed length, at which x ! 0 and U ! 0.

(8-1)

Mechanical Energy The mechanical energy Emec of a system

%U ! $W.

If the particle moves from point xi to point xf , the change in the potential energy of the system is

"

xf

%U ! $

xi

F(x) dx.

(8-6)

Gravitational Potential Energy The potential energy associated with a system consisting of Earth and a nearby particle is gravitational potential energy. If the particle moves from height yi to height yf , the change in the gravitational potential energy of the particle–Earth system is %U ! mg(yf $ yi) ! mg %y.

(8-7)

If the reference point of the particle is set as yi ! 0 and the corresponding gravitational potential energy of the system is set as Ui ! 0, then the gravitational potential energy U when the parti-

is the sum of its kinetic energy K and potential energy U: Emec ! K " U.

(8-12)

An isolated system is one in which no external force causes energy changes. If only conservative forces do work within an isolated system, then the mechanical energy Emec of the system cannot change. This principle of conservation of mechanical energy is written as K2 " U2 ! K1 " U1,

(8-17)

in which the subscripts refer to different instants during an energy transfer process. This conservation principle can also be written as %Emec ! %K " %U ! 0. (8-18)

Potential Energy Curves If we know the potential energy function U(x) for a system in which a one-dimensional force F(x)

200

CHAPTE R 8 POTE NTIAL E N E RGY AN D CONSE RVATION OF E N E RGY

acts on a particle, we can find the force as F(x) ! $

dU(x) . dx

(8-22)

If U(x) is given on a graph, then at any value of x, the force F(x) is the negative of the slope of the curve there and the kinetic energy of the particle is given by (8-24)

K(x) ! Emec $ U(x),

where Emec is the mechanical energy of the system. A turning point is a point x at which the particle reverses its motion (there, K ! 0). The particle is in equilibrium at points where the slope of the U(x) curve is zero (there, F(x) ! 0).

Work Done on a System by an External Force Work W is energy transferred to or from a system by means of an external force acting on the system. When more than one force acts on a system, their net work is the transferred energy. When friction is not involved, the work done on the system and the change %Emec in the mechanical energy of the system are equal: (8-26, 8-25)

W ! %Emec ! %K " %U.

When a kinetic frictional force acts within the system, then the thermal energy Eth of the system changes. (This energy is associated with the random motion of atoms and molecules in the system.) The work done on the system is then W ! %Emec " %Eth.

(8-33)

The change %Eth is related to the magnitude fk of the frictional force and the magnitude d of the displacement caused by the external force by %Eth ! fkd. (8-31)

Conservation of Energy The total energy E of a system (the sum of its mechanical energy and its internal energies, including thermal energy) can change only by amounts of energy that are transferred to or from the system. This experimental fact is known as the law of conservation of energy. If work W is done on the system, then W ! %E ! %Emec " %Eth " %Eint.

(8-35)

If the system is isolated (W ! 0), this gives

and

%Emec " %Eth " %Eint ! 0

(8-36)

Emec,2 ! Emec,1 $ %Eth $ %Eint,

(8-37)

where the subscripts 1 and 2 refer to two different instants.

Power The power due to a force is the rate at which that force transfers energy. If an amount of energy %E is transferred by a force in an amount of time %t, the average power of the force is Pavg !

%E . %t

(8-40)

The instantaneous power due to a force is P!

dE . dt

(8-41)

Questions 1 In Fig. 8-18, a horizontally moving block can take three frictionless routes, differing only in elevation, to reach the dashed finish line. Rank the routes according to (a) the speed of the block at the finish line and (b) the travel time of the block to the finish line, greatest first. Finish line (1)

v

(2) (3)

Figure 8-18 Question 1. 2 Figure 8-19 gives the potential energy function of a particle. (a) Rank regions AB, BC, CD, and DE according to the magni-

U(x) (J)

8 6 5 3 1 0

A B

C

D

E

F

Figure 8-19 Question 2.

G

H

x

tude of the force on the particle, greatest first. What value must the mechanical energy Emec of the particle not exceed if the particle is to be (b) trapped in the potential well at the left, (c) trapped in the potential well at the right, and (d) able to move between the two potential wells but not to the right of point H? For the situation of (d), in which of regions BC, DE, and FG will the particle have (e) the greatest kinetic energy and (f ) the least speed? –30 3 Figure 8-20 shows one direct path and four indirect paths from 32 10 –6 20 point i to point f. Along the direct –4 f i path and three of the indirect paths, 15 –10 7 40 2 only a conservative force Fc acts on –30 a certain object. Along the fourth indirect path, both Fc and a nonconFigure 8-20 Question 3. servative force Fnc act on the object. The change %Emec in the object’s mechanical energy (in joules) in going from i to f is indicated along each straight-line segment of the indirect paths. What is %Emec (a) from i to f along the direct path and (b) due to Fnc along the one path where it acts?

4 In Fig. 8-21, a small, initially stationary block is released on a frictionless ramp at a height of 3.0 m. Hill heights along the ramp are as shown in the figure. The hills have identical circular tops, and the block does not fly off any hill. (a) Which hill is the first the block cannot cross? (b) What does the block do after failing to cross that hill? Of the hills that the block can cross, on which hill-

201

QU ESTIONS

top is (c) the centripetal acceleration of the block greatest and (d) the normal force on the block least? 3.5 m (4)

2.5 m (3)

1.5 m 3.0 m (2)

0.5 m (1)

Figure 8-21 Question 4. 5 In Fig. 8-22, a block slides from A to C along a frictionless ramp, and then it passes through horizontal region CD, where a frictional force acts on it. Is the block’s kinetic energy increasing, decreasing, or constant in (a) region AB, (b) region BC, and (c) region CD? (d) Is the block’s mechanical energy increasing, decreasing, or constant in those regions?

A

Rod Cylinder

Rope

Figure 8-24 Question 7.

8 In Fig. 8-25, a block slides along a track that descends through distance h. The track is frictionless except for the lower section. There the block slides to a stop in a certain distance D because of friction. (a) If we decrease h, will the block now slide to a stop in a distance that is greater than, less than, or equal to D? (b) If, instead, we increase the mass of the block, will the stopping distance now be greater than, less than, or equal to D?

D

9 Figure 8-26 shows three situations involving a plane that is not frictionless and a block sliding along the plane. The block begins with the same speed in all three situations and slides until the kinetic frictional force has stopped it. Rank the situations according to the increase in thermal energy due to the sliding, greatest first.

B

6 In Fig. 8-23a, you pull upward on a rope that is attached to a cylinder on a vertical rod. Because the cylinder fits tightly on the rod, the cylinder slides along the rod with considerable friction. Your force does work W ! "100 J on the cylinder – rod – Earth system (Fig. 8-23b). An “energy statement” for the system is shown in Fig. 8-23c: the kinetic energy K increases by 50 J, and the gravitational potential energy Ug increases by 20 J. The only other change in energy within the system is for the thermal energy Eth. What is the change %Eth?

Work W

Rope

W = +100 J System's energies:

System

Cylinder

∆K = +50 J ∆Ug = +20 J ∆Eth = ?

Earth (b)

Figure 8-25 Question 8.

D

Figure 8-22 Question 5.

(a)

Block

h

C

Rod

descends, it pulls on a block via a second rope, and the block slides over a lab table. Again consider the cylinder – rod – Earth system, similar to that shown in Fig. 8-23b. Your work on the system is 200 J. The system does work of 60 J on the block. Within the system, the kinetic energy increases by 130 J and the gravitational potential energy decreases by 20 J. (a) Draw an “energy statement” for the system, as in Fig. 8-23c. (b) What is the change in the thermal energy within the system?

(c)

Figure 8-23 Question 6. 7 The arrangement shown in Fig. 8-24 is similar to that in Question 6. Here you pull downward on the rope that is attached to the cylinder, which fits tightly on the rod. Also, as the cylinder

(1)

(2)

(3)

Figure 8-26 Question 9. 10 Figure 8-27 shows three plums that are launched from the same level with the same speed. One moves straight upward, one is launched at a small angle to the vertical, and one is launched along a frictionless incline. Rank the plums according to their speed when they reach the level of the dashed line, greatest first. 11 When a particle moves from f to i and from j to i along the paths shown in Fig. 8-28, and in the indicated directions, a conservative : force F does the indicated amounts of work on it. How much work is : done on the particle by F when the particle moves directly from f to j?

(1) (2)

(3)

Figure 8-27 Question 10.

"20 J

i

f

20 J

Figure 8-28 Question 11.

j

202

CHAPTE R 8 POTE NTIAL E N E RGY AN D CONSE RVATION OF E N E RGY

Problems Tutoring problem available (at instructor’s discretion) in WileyPLUS and WebAssign SSM • – •••

Worked-out solution available in Student Solutions Manual

WWW Worked-out solution is at

Number of dots indicates level of problem difficulty

ILW

Interactive solution is at

http://www.wiley.com/college/halliday

Additional information available in The Flying Circus of Physics and at flyingcircusofphysics.com

Module 8-1 Potential Energy •1 SSM What is the spring constant of a spring that stores 25 J of elastic potential energy when compressed by 7.5 cm? •2 In Fig. 8-29, a single frictionless roller-coaster car of mass m ! 825 kg tops the first hill with speed v0 ! 17.0 m/s at height h ! 42.0 m. How much work does the gravitational force do on the car from that point to (a) point A, (b) point B, and (c) point C? If the gravitational potential energy of the car – Earth system is taken to be zero at C, what is its value when the car is at (d) B and (e) A? (f) If mass m were doubled, would the change in the gravitational potential energy of the system between points A and B increase, decrease, or remain the same?

v0

A

First hill h

B h h/2

C

Figure 8-29 Problems 2 and 9. •3 You drop a 2.00 kg book to a friend who stands on the ground at distance D ! 10.0 m below. If your friend’s outstretched hands are at distance d ! 1.50 m above the ground (Fig. 8-30), (a) how much work Wg does the gravitational force do on the book as it drops to her hands? (b) What is the change "U in the gravitational potential energy of the book – Earth system during the drop? If the gravitational potential energy U of that system is taken to be zero at ground level, what is U (c) when the book is released and (d) when it reaches her hands? Now take U to be 100 J at ground level and again find (e) Wg, (f) "U, (g) U at the release point, and (h) U at her hands.

D

d

Figure 8-30 Problems 3 and 10.

•4 Figure 8-31 shows a ball with mass m ! 0.341 kg attached to the end of a thin rod with length L ! 0.452 m and negligible mass. The other end of the rod is pivoted so that the ball can move in a vertical circle. The rod is held horizontally as shown and then given enough of a downward push to cause the ball to swing down and around and just reach the vertically up position, with zero speed there. How much work is done on the ball by the gravitational force from the initial point

L

Figure 8-31 Problems 4 and 14.

to (a) the lowest point, (b) the highest point, and (c) the point on the right level with the initial point? If the gravitational potential energy of the ball – Earth system is taken to be zero at the initial point, what is it when the ball reaches (d) the lowest point, (e) the highest point, and (f) the point on the right level with the initial point? (g) Suppose the rod were pushed harder so that the ball passed through the highest point with a nonzero speed. Would "Ug from the lowest point to the highest point then be greater than, less than, or the same as it was when the ball stopped at the highest point? Ice •5 SSM In Fig. 8-32, a 2.00 g ice flake flake is released from the edge of a hemispherical bowl whose radius r r is 22.0 cm. The flake – bowl contact is frictionless. (a) How much work is done on the flake by the gravitational force during the flake’s descent to the bottom of the bowl? (b) What is the change in the potential energy of the flake – Earth sysFigure 8-32 Problems 5 tem during that descent? (c) If that and 11. potential energy is taken to be zero at the bottom of the bowl, what is its value when the flake is released? (d) If, instead, the potential energy is taken to be zero at the release point, what is its value when the flake reaches the bottom of the bowl? (e) If the mass of the flake were doubled, would the magnitudes of the answers to (a) through (d) increase, decrease, or remain the same?

••6 In Fig. 8-33, a small block of P mass m ! 0.032 kg can slide along the frictionless loop-the-loop, with loop radius R ! 12 cm. The block is released from rest at point P, at height h ! 5.0R above the bottom h of the loop. How much work does R the gravitational force do on the Q block as the block travels from point R P to (a) point Q and (b) the top of the loop? If the gravitational potenFigure 8-33 Problems 6 tial energy of the block – Earth sysand 17. tem is taken to be zero at the bottom of the loop, what is that potential energy when the block is (c) at point P, (d) at point Q, and (e) at the top of the loop? (f) If, instead of merely being released, the block is given some initial speed downward along the track, do the answers to (a) through (e) increase, decrease, or remain the same? ••7 Figure 8-34 shows a thin rod, of length L ! 2.00 m and negligible mass, that can pivot about one end to rotate in a vertical circle. A ball of mass m ! 5.00 kg is attached to the other end. The rod is pulled aside to angle u0 ! 30.0# and released with initial velocity : v0 ! 0 . As the ball descends to its lowest point, (a) how much work does the gravitational force do on it and (b) what is the change in the gravitational potential energy of

PROB LE M S

the ball – Earth system? (c) If the gravitational potential energy is taken to be zero at the lowest point, what is its value just as the ball is released? (d) Do the magnitudes of the answers to (a) through (c) increase, decrease, or remain the same if angle u0 is increased?

203

L must the ramp have if the truck is to stop (momentarily) along it? (Assume the truck is a particle, and justify that assumption.) Does the minimum length L increase, decrease, or remain the same if (b) the truck’s mass is decreased and (c) its speed is decreased?

θ0 L

••8 A 1.50 kg snowball is fired from a cliff 12.5 m high. The snowball’s initial velocity is m 14.0 m/s, directed 41.0# above the horizontal. (a) How much work is done on the snowball v0 by the gravitational force during its flight to the flat ground below the cliff? (b) What is Figure 8-34 the change in the gravitational potential enProblems 7, 18, ergy of the snowball – Earth system during and 21. the flight? (c) If that gravitational potential energy is taken to be zero at the height of the cliff, what is its value when the snowball reaches the ground? Module 8-2 Conservation of Mechanical Energy •9 In Problem 2, what is the speed of the car at (a) point A, (b) point B, and (c) point C? (d) How high will the car go on the last hill, which is too high for it to cross? (e) If we substitute a second car with twice the mass, what then are the answers to (a) through (d)? •10 (a) In Problem 3, what is the speed of the book when it reaches the hands? (b) If we substituted a second book with twice the mass, what would its speed be? (c) If, instead, the book were thrown down, would the answer to (a) increase, decrease, or remain the same? •11 SSM WWW (a) In Problem 5, what is the speed of the flake when it reaches the bottom of the bowl? (b) If we substituted a second flake with twice the mass, what would its speed be? (c) If, instead, we gave the flake an initial downward speed along the bowl, would the answer to (a) increase, decrease, or remain the same? •12 (a) In Problem 8, using energy techniques rather than the techniques of Chapter 4, find the speed of the snowball as it reaches the ground below the cliff. What is that speed (b) if the launch angle is changed to 41.0# below the horizontal and (c) if the mass is changed to 2.50 kg? •13 SSM A 5.0 g marble is fired vertically upward using a spring gun. The spring must be compressed 8.0 cm if the marble is to just reach a target 20 m above the marble’s position on the compressed spring. (a) What is the change "Ug in the gravitational potential energy of the marble – Earth system during the 20 m ascent? (b) What is the change "Us in the elastic potential energy of the spring during its launch of the marble? (c) What is the spring constant of the spring? •14 (a) In Problem 4, what initial speed must be given the ball so that it reaches the vertically upward position with zero speed? What then is its speed at (b) the lowest point and (c) the point on the right at which the ball is level with the initial point? (d) If the ball’s mass were doubled, would the answers to (a) through (c) increase, decrease, or remain the same? •15 SSM In Fig. 8-35, a runaway truck with failed brakes is moving downgrade at 130 km/h just before the driver steers the truck up a frictionless emergency escape ramp with an inclination of u ! 15#. The truck’s mass is 1.2 $ 104 kg. (a) What minimum length

L

θ

Figure 8-35 Problem 15. ••16 A 700 g block is released from rest at height h0 above a vertical spring with spring constant k ! 400 N/m and negligible mass. The block sticks to the spring and momentarily stops after compressing the spring 19.0 cm. How much work is done (a) by the block on the spring and (b) by the spring on the block? (c) What is the value of h0? (d) If the block were released from height 2.00h0 above the spring, what would be the maximum compression of the spring? ••17 In Problem 6, what are the magnitudes of (a) the horizontal component and (b) the vertical component of the net force acting on the block at point Q? (c) At what height h should the block be released from rest so that it is on the verge of losing contact with the track at the top of the loop? (On the verge of losing contact means that the normal force on the block from the track has just then become zero.) (d) Graph the magnitude of the normal force on the block at the top of the loop versus initial height h, for the range h ! 0 to h ! 6R. ••18 (a) In Problem 7, what is the speed of the ball at the lowest point? (b) Does the speed increase, decrease, or remain the same if the mass is increased? ••19 Figure 8-36 shows an 8.00 kg stone at rest on a spring. The spring is compressed 10.0 cm by the stone. (a) What is the spring k constant? (b) The stone is pushed down an additional 30.0 cm and released. What is the elastic potential energy of the compressed spring just before that release? (c) What is Figure 8-36 the change in the gravitational potential enProblem 19. ergy of the stone – Earth system when the stone moves from the release point to its maximum height? (d) What is that maximum height, measured from the release point? ••20 A pendulum consists of a 2.0 kg stone swinging on a 4.0 m string of negligible mass. The stone has a speed of 8.0 m/s when it passes its lowest point. (a) What is the speed when the string is at 60# to the vertical? (b) What is the greatest angle with the vertical that the string will reach during the stone’s motion? (c) If the potential energy of the pendulum – Earth system is taken to be zero at the stone’s lowest point, what is the total mechanical energy of the system? ••21 Figure 8-34 shows a pendulum of length L ! 1.25 m. Its bob (which effectively has all the mass) has speed v0 when the cord makes an angle u0 ! 40.0# with the vertical. (a) What is the speed of the bob when it is in its lowest position if v0 ! 8.00 m/s? What is the least value that v0 can have if the pendulum is to swing down and then up (b) to a horizontal position, and (c) to a vertical position with the cord remaining straight? (d) Do the answers to (b) and (c) increase, decrease, or remain the same if u0 is increased by a few degrees?

204

CHAPTE R 8 POTE NTIAL E N E RGY AN D CONSE RVATION OF E N E RGY

••22 A 60 kg skier starts from rest at height H ! 20 m above the end of a ski-jump ramp (Fig. 8-37) and leaves the ramp at angle u ! 28#. Neglect the effects of air resistance and assume the ramp is frictionless. (a) What is the maximum height h of his jump above the end of the ramp? (b) If he increased his weight by putting on a backpack, would h then be greater, less, or the same?

H

End of ramp h

θ

••29 SSM WWW In Fig. 8-42, a block of mass m ! 12 kg is released from rest on a frictionless incline of angle ' ! 30#. Below the block is a spring that can be compressed 2.0 cm by a force of 270 N. The block momentarily stops when it compresses the spring by 5.5 cm. (a) How far does the block move down the incline from its rest position to this stopping point? (b) What is the speed of the block just as it touches the spring?

Figure 8-37 Problem 22. ••23 ILW The string in Fig. 8-38 is L ! 120 cm long, has a ball attached to one end, and is fixed at its other end. The distance d from the fixed end to a fixed peg at point P is 75.0 cm. When the initially stationary ball is released with the string horizontal as shown, it will swing along the dashed arc. What is its speed when it reaches (a) its lowest point and (b) its highest point after the string catches on the peg?

L

d P

r

Figure 8-38 Problems 23 and 70.

••24 A block of mass m ! 2.0 kg is dropped from height h ! 40 cm onto a spring of spring constant k ! 1960 N/m (Fig. 8-39). Find the maximum distance the spring is compressed. ••25 At t ! 0 a 1.0 kg ball is thrown from a tall tower with : v ! (18 m/s)iˆ & (24 m/s)jˆ. What is "U of the ball – Earth system between t ! 0 and t ! 6.0 s (still free fall)? : ••26 A conservative force F ! (6.0x % 12)iˆ N, where x is in meters, acts on a particle moving along an x axis. The potential energy U associated with this force is assigned a value of 27 J at x ! 0. (a) Write an expression for U as a function of x, with U in joules and x in meters. (b) What is the maximum positive potential energy? At what (c) negative value and (d) positive value of x is the potential energy equal to zero? ••27 Tarzan, who weighs 688 N, swings from a cliff at the end of a vine 18 m long (Fig. 8-40). From the top of the cliff to the bottom of the swing, he descends by 3.2 m.The vine will break if the force on it exceeds 950 N. (a) Does the vine break? (b) If no, what is the greatest force on it during the swing? If yes, at what angle with the vertical does it break?

••28 Figure 8-41a applies to the spring in a cork gun (Fig. 8-41b); it shows the spring force as a function of the stretch or compression of the spring. The spring is compressed by 5.5 cm and used to propel a 3.8 g cork from the gun. (a) What is the speed of the cork if it is released as the spring passes through its relaxed position? (b) Suppose, instead, that the cork sticks to the spring and stretches it 1.5 cm before separation occurs.What now is the speed of the cork at the time of release?

m

h

Force (N) 0.4 0.2 –4 –2

2 4 –0.2

x (cm)

–0.4 (a) Compressed spring

Cork x

0 (b)

Figure 8-41 Problem 28.

m

••30 A 2.0 kg breadbox on a fricθ tionless incline of angle u ! 40# is connected, by a cord that runs over a Figure 8-42 Problems 29 pulley, to a light spring of spring conand 35. stant k ! 120 N/m, as shown in Fig. 8-43. The box is released from rest when the spring is unstretched. Assume that the pulley is massless and frictionless. (a) What is the speed of the box when it has moved 10 cm down the incline? (b) How far down the incline from its point of release does the box slide before momentarily stopping, and what are the (c) magnitude and (d) direction (up or down the incline) of the box’s acceleration at the instant the box momentarily stops?

k

Figure 8-39 Problem 24.

Figure 8-40 Problem 27.

θ

Figure 8-43 Problem 30. ••31 ILW A block with mass m ! 2.00 kg is placed against a spring on a frictionless incline with angle ' ! 30.0# (Fig. 8-44). (The block is not attached to the spring.) The spring, with spring constant k ! 19.6 N/cm, is compressed 20.0 cm and then released. (a) What is the elastic potential energy of the compressed spring? (b) What is the change in the gravitational potential energy of the m block – Earth system as the block moves from the release point to its highest point on the incline? (c) k How far along the incline is the θ highest point from the release Figure 8-44 Problem 31. point?

205

PROB LE M S

values: UA ! 9.00 J, UC ! 20.00 J, and UD ! 24.00 J. The particle is released at the point where U forms a “potential hill” of “height” UB ! 12.00 J, with kinetic energy 4.00 J. What is the speed of the particle at (a) x ! 3.5 m and (b) x ! 6.5 m? What is the position of the turning point on (c) the right side and (d) the left side?

•••33 In Fig. 8-46, a spring with k ! 170 N/m is at the top of a frictionless incline of angle ' ! 37.0#. The lower end of the incline is distance D ! 1.00 m from the end of the spring, which is at its relaxed D length. A 2.00 kg canister is pushed against the spring until the spring is θ compressed 0.200 m and released from rest. (a) What is the speed of Figure 8-46 Problem 33. the canister at the instant the spring returns to its relaxed length (which is when the canister loses contact with the spring)? (b) What is the speed of the canister when it reaches the lower end of the incline? •••34 A boy is initially seated on the top of a hemispherical ice mound of radius R ! 13.8 m. He begins to slide down the ice, with a negligible initial speed (Fig. 8-47). Approximate the ice as being frictionless. At what height does the boy lose contact with the ice?

R

Figure 8-47 Problem 34.

•••35 In Fig. 8-42, a block of mass m ! 3.20 kg slides from rest a distance d down a frictionless incline at angle u ! 30.0# where it runs into a spring of spring constant 431 N/m. When the block momentarily stops, it has compressed the spring by 21.0 cm. What are (a) distance d and (b) the distance between the point of the first block – spring contact and the point where the block’s speed is greatest? •••36 Two children are playing a game in which they try to hit a small box on the floor with a marble fired from a spring-loaded gun that is mounted on a D table. The target box is horiFigure 8-48 Problem 36. zontal distance D ! 2.20 m from the edge of the table; see Fig. 8-48. Bobby compresses the spring 1.10 cm, but the center of the marble falls 27.0 cm short of the center of the box. How far should Rhoda compress the spring to score a direct hit? Assume that neither the spring nor the ball encounters friction in the gun. •••37 A uniform cord of length 25 cm and mass 15 g is initially stuck to a ceiling. Later, it hangs vertically from the ceiling with only one end still stuck. What is the change in the gravitational potential energy of the cord with this change in orientation? (Hint: Consider a differential slice of the cord and then use integral calculus.) Module 8-3 Reading a Potential Energy Curve ••38 Figure 8-49 shows a plot of potential energy U versus position x of a 0.200 kg particle that can travel only along an x axis under the influence of a conservative force. The graph has these

UD UC

UB

UA

0

1

2

3

4 5 x (m)

6

7

8

9

Figure 8-49 Problem 38. ••39 Figure 8-50 shows a UC plot of potential energy U verUB sus position x of a 0.90 kg particle that can travel only along an x axis. (Nonconservative forces UA are not involved.) Three values are UA ! 15.0 J, UB ! 35.0 J, and UC ! 45.0 J. The particle is 2 4 6 x (m) released at x ! 4.5 m with an initial speed of 7.0 m/s, headed Figure 8-50 Problem 39. in the negative x direction. (a) If the particle can reach x ! 1.0 m, what is its speed there, and if it cannot, what is its turning point? What are the (b) magnitude and (c) direction of the force on the particle as it begins to move to the left of x ! 4.0 m? Suppose, instead, the particle is headed in the positive x direction when it is released at x ! 4.5 m at speed 7.0 m/s. (d) If the particle can reach x ! 7.0 m, what is its speed there, and if it cannot, what is its turning point? What are the (e) magnitude and (f) direction of the force on the particle as it begins to move to the right of x ! 5.0 m? U (J)

Figure 8-45 Problem 32.

U (J)

••32 In Fig. 8-45, a chain is held on a frictionless table with onefourth of its length hanging over the edge. If the chain has length L ! 28 cm and mass m ! 0.012 kg, how much work is required to pull the hanging part back onto the table?

••40 The potential energy of a diatomic molecule (a two-atom system like H2 or O2) is given by U!

A B % 6, r 12 r

where r is the separation of the two atoms of the molecule and A and B are positive constants. This potential energy is associated with the force that binds the two atoms together. (a) Find the equilibrium separation — that is, the distance between the atoms at which the force on each atom is zero. Is the force repulsive (the atoms are pushed apart) or attractive (they are pulled together) if their separation is (b) smaller and (c) larger than the equilibrium separation? •••41 A single conservative force F(x) acts on a 1.0 kg particle that moves along an x axis. The potential energy U(x) associated with F(x) is given by U(x) ! %4x e%x/4 J, where x is in meters. At x ! 5.0 m the particle has a kinetic energy of 2.0 J. (a) What is the mechanical energy of the system? (b) Make

206

CHAPTE R 8 POTE NTIAL E N E RGY AN D CONSE RVATION OF E N E RGY

a plot of U(x) as a function of x for 0 ( x ( 10 m, and on the same graph draw the line that represents the mechanical energy of the system. Use part (b) to determine (c) the least value of x the particle can reach and (d) the greatest value of x the particle can reach. Use part (b) to determine (e) the maximum kinetic energy of the particle and (f) the value of x at which it occurs. (g) Determine an expression in newtons and meters for F(x) as a function of x. (h) For what (finite) value of x does F(x) ! 0? Module 8-4 Work Done on a System by an External Force •42 A worker pushed a 27 kg block 9.2 m along a level floor at constant speed with a force directed 32# below the horizontal. If the coefficient of kinetic friction between block and floor was 0.20, what were (a) the work done by the worker’s force and (b) the increase in thermal energy of the block – floor system? •43 A collie drags its bed box across a floor by applying a horizontal force of 8.0 N. The kinetic frictional force acting on the box has magnitude 5.0 N. As the box is dragged through 0.70 m along the way, what are (a) the work done by the collie’s applied force and (b) the increase in thermal energy of the bed and floor? ••44 A horizontal force of magnitude 35.0 N pushes a block of mass 4.00 kg across a floor where the coefficient of kinetic friction is 0.600. (a) How much work is done by that applied force on the block – floor system when the block slides through a displacement of 3.00 m across the floor? (b) During that displacement, the thermal energy of the block increases by 40.0 J. What is the increase in thermal energy of the floor? (c) What is the increase in the kinetic energy of the block? ••45 SSM A rope is used to pull a 3.57 kg block at constant speed 4.06 m along a horizontal floor. The force on the block from the rope is 7.68 N and directed 15.0# above the horizontal. What are (a) the work done by the rope’s force, (b) the increase in thermal energy of the block – floor system, and (c) the coefficient of kinetic friction between the block and floor? Module 8-5 Conservation of Energy •46 An outfielder throws a baseball with an initial speed of 81.8 mi/h. Just before an infielder catches the ball at the same level, the ball’s speed is 110 ft/s. In foot-pounds, by how much is the mechanical energy of the ball – Earth system reduced because of air drag? (The weight of a baseball is 9.0 oz.) •47 A 75 g Frisbee is thrown from a point 1.1 m above the ground with a speed of 12 m/s. When it has reached a height of 2.1 m, its speed is 10.5 m/s. What was the reduction in Emec of the Frisbee – Earth system because of air drag? •48 In Fig. 8-51, a block slides down an incline. As it moves from A point A to point B, which are 5.0 m : apart, force F acts on the block, with B magnitude 2.0 N and directed down the incline.The magnitude of the fricFigure 8-51 Problems 48 tional force acting on the block is and 71. 10 N. If the kinetic energy of the block increases by 35 J between A and B, how much work is done on the block by the gravitational force as the block moves from A to B? •49 SSM ILW A 25 kg bear slides, from rest, 12 m down a lodgepole pine tree, moving with a speed of 5.6 m/s just before hitting the ground. (a) What change occurs in the gravitational

potential energy of the bear – Earth system during the slide? (b) What is the kinetic energy of the bear just before hitting the ground? (c) What is the average frictional force that acts on the sliding bear? •50 A 60 kg skier leaves the end of a ski-jump ramp with a velocity of 24 m/s directed 25# above the horizontal. Suppose that as a result of air drag the skier returns to the ground with a speed of 22 m/s, landing 14 m vertically below the end of the ramp. From the launch to the return to the ground, by how much is the mechanical energy of the skier – Earth system reduced because of air drag? •51 During a rockslide, a 520 kg rock slides from rest down a hillside that is 500 m long and 300 m high. The coefficient of kinetic friction between the rock and the hill surface is 0.25. (a) If the gravitational potential energy U of the rock – Earth system is zero at the bottom of the hill, what is the value of U just before the slide? (b) How much energy is transferred to thermal energy during the slide? (c) What is the kinetic energy of the rock as it reaches the bottom of the hill? (d) What is its speed then? ••52 A large fake cookie sliding on a horizontal surface is attached to one end of a horizontal spring with spring constant k ! 400 N/m; the other end of the spring is fixed in place. The cookie has a kinetic energy of 20.0 J as it passes through the spring’s equilibrium position. As the cookie slides, a frictional force of magnitude 10.0 N acts on it. (a) How far will the cookie slide from the equilibrium position before coming momentarily to rest? (b) What will be the kinetic energy of the cookie as it slides back through the equilibrium position? ••53 In Fig. 8-52, a 3.5 kg block is accelerated from rest by a compressed spring of spring constant 640 N/m. The No friction D block leaves the spring at the (µk ) spring’s relaxed length and then travels over a horizontal Figure 8-52 Problem 53. floor with a coefficient of kinetic friction mk ! 0.25. The frictional force stops the block in distance D ! 7.8 m. What are (a) the increase in the thermal energy of the block – floor system, (b) the maximum kinetic energy of the block, and (c) the original compression distance of the spring? ••54 A child whose weight is 267 N slides down a 6.1 m playground slide that makes an angle of 20# with the horizontal. The coefficient of kinetic friction between slide and child is 0.10. (a) How much energy is transferred to thermal energy? (b) If she starts at the top with a speed of 0.457 m/s, what is her speed at the bottom? ••55 ILW In Fig. 8-53, a block of mass m ! 2.5 kg slides head on into a spring of spring constant k ! 320 N/m. When the block stops, it has compressed the spring by 7.5 cm. The coefficient x of kinetic friction between block 0 and floor is 0.25. While the block Figure 8-53 Problem 55. is in contact with the spring and being brought to rest, what are (a) the work done by the spring force and (b) the increase in thermal energy of the block – floor system? (c) What is the block’s speed just as it reaches the spring? ••56 You push a 2.0 kg block against a horizontal spring, compressing the spring by 15 cm. Then you release the block, and the

PROB LE M S

spring sends it sliding across a tabletop. It stops 75 cm from where you released it. The spring constant is 200 N/m. What is the block – table coefficient of kinetic friction? ••57 In Fig. 8-54, a block slides along a track from one level to a higher level after passing through an intermediate valley. The track is frictionless until the block reaches the higher level. There a frictional force stops the block in a distance d. The block’s initial speed v0 is 6.0 m/s, the height difference h is 1.1 m, and mk is 0.60. Find d. d

µ=0

v0

h

µk

Figure 8-54 Problem 57. ••58 A cookie jar is moving up a 40# incline. At a point 55 cm from the bottom of the incline (measured along the incline), the jar has a speed of 1.4 m/s. The coefficient of kinetic friction between jar and incline is 0.15. (a) How much farther up the incline will the jar move? (b) How fast will it be going when it has slid back to the bottom of the incline? (c) Do the answers to (a) and (b) increase, decrease, or remain the same if we decrease the coefficient of kinetic friction (but do not change the given speed or location)? ••59 A stone with a weight of 5.29 N is launched vertically from ground level with an initial speed of 20.0 m/s, and the air drag on it is 0.265 N throughout the flight. What are (a) the maximum height reached by the stone and (b) its speed just before it hits the ground? ••60 A 4.0 kg bundle starts up a 30# incline with 128 J of kinetic energy. How far will it slide up the incline if the coefficient of kinetic friction between bundle and incline is 0.30? ••61 When a click beetle is upside down on its back, it jumps upward by suddenly arching its back, transferring energy stored in a muscle to mechanical energy.This launching mechanism produces an audible click, giving the beetle its name. Videotape of a certain clickbeetle jump shows that a beetle of mass m ! 4.0 $ 10%6 kg moved directly upward by 0.77 mm during the launch and then to a maximum height of h ! 0.30 m. During the launch, what are the average magnitudes of (a) the external force on the beetle’s back from the floor and (b) the acceleration of the beetle in terms of g? •••62 In Fig. 8-55, a block slides along a path that is without friction until the block reaches the section of length L ! 0.75 m, which begins at height h ! 2.0 m on a ramp of angle u ! 30#. In that section, the coefficient of kinetic friction is 0.40. The block passes through point A with a speed of 8.0 m/s. If the block can reach point B (where the friction ends), what is its speed there, and if it cannot, what is its greatest height above A? B

L

h

θ

A

Figure 8-55 Problem 62.

207

•••63 The cable of the 1800 kg elevator cab in Fig. 8-56 snaps when the cab is at rest at the first floor, where the cab bottom is a distance d ! 3.7 m above a spring of spring constant k ! 0.15 MN/m. A safety device clamps the cab against guide rails so that a constant frictional force of 4.4 kN opposes the cab’s motion. (a) Find the d speed of the cab just before it hits the spring. (b) Find the maximum distance x k that the spring is compressed (the frictional force still acts during this compression). (c) Find the distance that the cab Figure 8-56 will bounce back up the shaft. (d) Using Problem 63. conservation of energy, find the approximate total distance that the cab will move before coming to rest. (Assume that the frictional force on the cab is negligible when the cab is stationary.) •••64 In Fig. 8-57, a block is released from rest at height d ! 40 cm and slides down a frictionless ramp and onto a first plateau, which has length d and where the coefficient of kinetic friction is 0.50. If the block is still moving, it then slides down a second frictionless ramp through height d/2 and onto a lower plateau, which has length d/2 and where the coefficient of kinetic friction is again 0.50. If the block is still moving, it then slides up a frictionless ramp until it (momentarily) stops. Where does the block stop? If its final stop is on a plateau, state which one and give the distance L from the left edge of that plateau. If the block reaches the ramp, give the height H above the lower plateau where it momentarily stops.

d

d d/2 d/2

Figure 8-57 Problem 64. •••65 A particle can slide along a A track with elevated ends and a flat h central part, as shown in Fig. 8-58. The flat part has length L ! 40 cm. L The curved portions of the track are frictionless, but for the flat part the Figure 8-58 Problem 65. coefficient of kinetic friction is mk ! 0.20. The particle is released from rest at point A, which is at height h ! L/2. How far from the left edge of the flat part does the particle finally stop? Additional Problems 66 A 3.2 kg sloth hangs 3.0 m above the ground. (a) What is the gravitational potential energy of the sloth – Earth system if we take the reference point y ! 0 to be at the ground? If the sloth drops to the ground and air drag on it is assumed to be negligible, what are the (b) kinetic energy and (c) speed of the sloth just before it reaches the ground?

208

CHAPTE R 8 POTE NTIAL E N E RGY AN D CONSE RVATION OF E N E RGY

67 SSM A spring (k ! 200 N/m) is fixed at the top of a frictionless plane d inclined at angle ' ! 40# (Fig. 8-59). A 1.0 kg block is projected up the plane, from an initial position that is distance d ! 0.60 m from the end of the relaxed spring, with an initial kinetic energy of θ 16 J. (a) What is the kinetic energy of the block at the instant it has comFigure 8-59 Problem 67. pressed the spring 0.20 m? (b) With what kinetic energy must the block be projected up the plane if it is to stop momentarily when it has compressed the spring by 0.40 m? 68 From the edge of a cliff, a 0.55 kg projectile is launched with an initial kinetic energy of 1550 J. The projectile’s maximum upward displacement from the launch point is &140 m. What are the (a) horizontal and (b) vertical components of its launch velocity? (c) At the instant the vertical component of its velocity is 65 m /s, what is its vertical displacement from the launch point? 69 SSM In Fig. 8-60, the pulley has negligible mass, and both it and the inclined plane are frictionless. Block A has a mass of 1.0 kg, block B has a mass of 2.0 kg, and angle u is 30#. If the blocks are released from rest with the connecting cord taut, what is their total kinetic energy when block B has fallen 25 cm?

73 SSM The temperature of a plastic cube is monitored while the cube is pushed 3.0 m across a floor at constant speed by a horizontal force of 15 N. The thermal energy of the cube increases by 20 J. What is the increase in the thermal energy of the floor along which the cube slides? 74 A skier weighing 600 N goes over a frictionless circular hill of radius R ! 20 m (Fig. 8-62). Assume that the effects of air resistance on the skier are negligible. As she comes up the hill, her speed is 8.0 m/s at point B, at angle u ! 20#. (a) What is her speed at the hilltop (point A) if she coasts without using her poles? (b) What minimum speed can she have at B and still coast to the hilltop? (c) Do the answers to these two questions increase, decrease, or remain the same if the skier weighs 700 N instead of 600 N? A

B

θ R

A

B

θ

Figure 8-60 Problem 69.

70 In Fig. 8-38, the string is L ! 120 cm long, has a ball attached to one end, and is fixed at its other end. A fixed peg is at point P. Released from rest, the ball swings down until the string catches on the peg; then the ball swings up, around the peg. If the ball is to swing completely around the peg, what value must distance d exceed? (Hint: The ball must still be moving at the top of its swing. Do you see why?) 71 SSM In Fig. 8-51, a block is sent sliding down a frictionless ramp. Its speeds at points A and B are 2.00 m/s and 2.60 m/s, respectively. Next, it is again sent sliding down the ramp, but this time its speed at point A is 4.00 m/s. What then is its speed at point B? 72 Two snowy peaks are at heights H ! 850 m and h ! 750 m above the valley between them. A ski run extends between the peaks, with a total length of 3.2 km and an average slope of ' ! 30# (Fig. 8-61). (a) A skier starts from rest at the top of the higher peak. At what speed will he arrive at the top of the lower peak if he coasts without using ski poles? Ignore friction. (b) Approximately what coefficient of kinetic friction

H

h

θ

between snow and skis would make him stop just at the top of the lower peak?

θ

Figure 8-61 Problem 72.

Figure 8-62 Problem 74. 75 SSM To form a pendulum, a 0.092 kg ball is attached to one end of a rod of length 0.62 m and negligible mass, and the other end of the rod is mounted on a pivot. The rod is rotated until it is straight up, and then it is released from rest so that it swings down around the pivot. When the ball reaches its lowest point, what are (a) its speed and (b) the tension in the rod? Next, the rod is rotated until it is horizontal, and then it is again released from rest. (c) At what angle from the vertical does the tension in the rod equal the weight of the ball? (d) If the mass of the ball is increased, does the answer to (c) increase, decrease, or remain the same? 76 We move a particle along an x axis, first outward from x ! 1.0 m to x ! 4.0 m and then back to x ! 1.0 m, while an external force acts on it. That force is directed along the x axis, and its x component can have different values for the outward trip and for the return trip. Here are the values (in newtons) for four situations, where x is in meters: Outward

Inward

(a) &3.0 (b) &5.0 (c) &2.0x (d) &3.0x2

%3.0 &5.0 %2.0x &3.0x2

Find the net work done on the particle by the external force for the round trip for each of the four situations. (e) For which, if any, is the external force conservative? 77 SSM A conservative force F(x) acts on a 2.0 kg particle that moves along an x axis. The potential energy U(x) associated with F(x) is graphed in Fig. 8-63. When the particle is at x ! 2.0 m, its

PROB LE M S

velocity is %1.5 m/s. What are the (a) magnitude and (b) direction of F(x) at this position? Between what positions on the (c) left and (d) right does the particle move? (e) What is the particle’s speed at x ! 7.0 m?

0

0

x (m) 10

5

15

U(x) ( J )

–5

81 A particle can move along only an x axis, where conservative forces act on it (Fig. 8-66 and the following table). The particle is released at x ! 5.00 m with a kinetic energy of K ! 14.0 J and a potential energy of U ! 0. If its motion is in the negative direction of the x axis, what are its (a) K and (b) U at x ! 2.00 m and its (c) K and (d) U at x ! 0? If its motion is in the positive direction of the x axis, what are its (e) K and (f) U at x ! 11.0 m, its (g) K and (h) U at x ! 12.0 m, and its (i) K and ( j) U at x ! 13.0 m? (k) Plot U(x) versus x for the range x ! 0 to x ! 13.0 m. F1

–10 0

–15

209

F2 2

F3 4

6

8

F4

10

12

x (m)

Figure 8-66 Problems 81 and 82.

–20

Next, the particle is released from rest at x ! 0. What are (l) its kinetic energy at x ! 5.0 m and (m) the maximum positive position xmax it reaches? (n) What does the particle do after it reaches xmax?

Figure 8-63 Problem 77. 78 At a certain factory, 300 kg crates are dropped vertically from a packing machine onto a conveyor belt moving at 1.20 m/s (Fig. 8-64). E GIL FRA (A motor maintains the belt’s constant speed.) The coefficient of kinetic friction between the belt and Figure 8-64 Problem 78. each crate is 0.400. After a short time, slipping between the belt and the crate ceases, and the crate then moves along with the belt. For the period of time during which the crate is being brought to rest relative to the belt, calculate, for a coordinate system at rest in the factory, (a) the kinetic energy supplied to the crate, (b) the magnitude of the kinetic frictional force acting on the crate, and (c) the energy supplied by the motor. (d) Explain why answers (a) and (c) differ. FRA

GIL

E

79 SSM A 1500 kg car begins sliding down a 5.0# inclined road with a speed of 30 km/h. The engine is turned off, and the only forces acting on the car are a net frictional force from the road and the gravitational force. After the car has traveled 50 m along the road, its speed is 40 km/h. (a) How much is the mechanical energy of the car reduced because of the net frictional force? (b) What is the magnitude of that net frictional force? 80 In Fig. 8-65, a 1400 kg block of granite is pulled up an incline at a constant speed of 1.34 m/s by a cable and winch. The indicated distances are d1 ! 40 m and d2 ! 30 m. The coefficient of kinetic friction between the block and the incline is 0.40. What is the power due to the force applied to the block by the cable?

d2 d1

Figure 8-65 Problem 80.

Range 0 to 2.00 m 2.00 m to 3.00 m 3.00 m to 8.00 m 8.00 m to 11.0 m 11.0 m to 12.0 m 12.0 m to 15.0 m

Force ! &(3.00 N)iˆ ! &(5.00 N)iˆ F!0 : F3 ! %(4.00 N)iˆ : F4 ! %(1.00 N)iˆ

: F1 : F2

F!0

82 For the arrangement of forces in Problem 81, a 2.00 kg particle is released at x ! 5.00 m with an initial velocity of 3.45 m/s in the negative direction of the x axis. (a) If the particle can reach x ! 0 m, what is its speed there, and if it cannot, what is its turning point? Suppose, instead, the particle is headed in the positive x direction when it is released at x ! 5.00 m at speed 3.45 m/s. (b) If the particle can reach x ! 13.0 m, what is its speed there, and if it cannot, what is its turning point? 83 SSM A 15 kg block is accelerated at 2.0 m/s2 along a horizontal frictionless surface, with the speed increasing from 10 m/s to 30 m/s. What are (a) the change in the block’s mechanical energy and (b) the average rate at which energy is transferred to the block? What is the instantaneous rate of that transfer when the block’s speed is (c) 10 m/s and (d) 30 m/s? 84 A certain spring is found not to conform to Hooke’s law. The force (in newtons) it exerts when stretched a distance x (in meters) is found to have magnitude 52.8x & 38.4x2 in the direction opposing the stretch. (a) Compute the work required to stretch the spring from x ! 0.500 m to x ! 1.00 m. (b) With one end of the spring fixed, a particle of mass 2.17 kg is attached to the other end of the spring when it is stretched by an amount x ! 1.00 m. If the particle is then released from rest, what is its speed at the instant the stretch in the spring is x ! 0.500 m? (c) Is the force exerted by the spring conservative or nonconservative? Explain. 85 SSM Each second, 1200 m3 of water passes over a waterfall 100 m high. Three-fourths of the kinetic energy gained by the water in falling is transferred to electrical energy by a hydroelectric generator. At what rate does the generator produce electrical energy? (The mass of 1 m3 of water is 1000 kg.)

210

CHAPTE R 8 POTE NTIAL E N E RGY AN D CONSE RVATION OF E N E RGY

86 In Fig. 8-67, a small block is sent through point A with a speed of 7.0 m/s. Its path is without friction until it reaches the section of length L ! 12 m, where the coefficient of kinetic friction is 0.70. The indicated heights are h1 ! 6.0 m and h2 ! 2.0 m. What are the speeds of the block at (a) point B and (b) point C? (c) Does the block reach point D? If so, what is its speed there; if not, how far through the section of friction does it travel? A

h1

C B

h2

D L

Figure 8-67 Problem 86. D 87 SSM A massless rigid rod of length L has a ball of mass m attached to one end (Fig. 8-68). The Pivot other end is pivoted in such a way point that the ball will move in a vertical A L C circle. First, assume that there is no Rod friction at the pivot. The system is launched downward from the hori- v0 zontal position A with initial speed v0. The ball just barely reaches point B D and then stops. (a) Derive an exFigure 8-68 Problem 87. pression for v0 in terms of L, m, and g. (b) What is the tension in the rod when the ball passes through B? (c) A little grit is placed on the pivot to increase the friction there. Then the ball just barely reaches C when launched from A with the same speed as before. What is the decrease in the mechanical energy during this motion? (d) What is the decrease in the mechanical energy by the time the ball finally comes to rest at B after several oscillations?

88 A 1.50 kg water balloon is shot straight up with an initial speed of 3.00 m/s. (a) What is the kinetic energy of the balloon just as it is launched? (b) How much work does the gravitational force do on the balloon during the balloon’s full ascent? (c) What is the change in the gravitational potential energy of the balloon – Earth system during the full ascent? (d) If the gravitational potential energy is taken to be zero at the launch point, what is its value when the balloon reaches its maximum height? (e) If, instead, the gravitational potential energy is taken to be zero at the maximum height, what is its value at the launch point? (f) What is the maximum height? 89 A 2.50 kg beverage can is thrown directly downward from a height of 4.00 m, with an initial speed of 3.00 m/s. The air drag on the can is negligible. What is the kinetic energy of the can (a) as it reaches the ground at the end of its fall and (b) when it is halfway to the ground? What are (c) the kinetic energy of the can and (d) the gravitational potential energy of the can – Earth system 0.200 s before the can reaches the ground? For the latter, take the reference point y ! 0 to be at the ground. 90 A constant horizontal force moves a 50 kg trunk 6.0 m up a 30# incline at constant speed. The coefficient of kinetic friction is 0.20. What are (a) the work done by the applied force and (b) the increase in the thermal energy of the trunk and incline?

91 Two blocks, of masses M !2.0 kg and 2M, are connected to a spring of spring constant k ! 200 N/m that has one end fixed, as shown in Fig. 8-69. The horizontal surface and the pulley are frictionM less, and the pulley has negligible mass. The blocks are released from rest with the spring relaxed. (a) What is the combined kinetic energy of the two blocks when the 2M hanging block has fallen 0.090 m? (b) What is the kinetic energy of the hanging block when it has Figure 8-69 Problem 91. fallen that 0.090 m? (c) What maximum distance does the hanging block fall before momentarily stopping? 92 A volcanic ash flow is moving across horizontal ground when it encounters a 10# upslope. The front of the flow then travels 920 m up the slope before stopping. Assume that the gases entrapped in the flow lift the flow and thus make the frictional force from the ground negligible; assume also that the mechanical energy of the front of the flow is conserved. What was the initial speed of the front of the flow? 93 A playground slide is in the form of an arc of a circle that has a radius of 12 m. The maximum height of the slide is h ! 4.0 m, and the ground is tangent to the circle (Fig. 8-70). A 25 kg child starts from rest at the top of the slide and has a speed of 6.2 m/s at the bottom. (a) What is the length of the slide? (b) What average frictional force acts on the child over this distance? If, instead of the ground, a vertical line through the top of the slide is tangent to the circle, what are (c) the length of the slide and (d) the average frictional force on the child?

h

Figure 8-70 Problem 93. 94 The luxury liner Queen Elizabeth 2 has a diesel-electric power plant with a maximum power of 92 MW at a cruising speed of 32.5 knots. What forward force is exerted on the ship at this speed? (1 knot ! 1.852 km/h.) 95 A factory worker accidentally releases a 180 kg crate that was being held at rest at the top of a ramp that is 3.7 m long and inclined at 39# to the horizontal. The coefficient of kinetic friction between the crate and the ramp, and between the crate and the horizontal factory floor, is 0.28. (a) How fast is the crate moving as it reaches the bottom of the ramp? (b) How far will it subsequently slide across the floor? (Assume that the crate’s kinetic energy does not change as it moves from the ramp onto the floor.) (c) Do the answers to (a) and (b) increase, decrease, or remain the same if we halve the mass of the crate? 96 If a 70 kg baseball player steals home by sliding into the plate with an initial speed of 10 m/s just as he hits the ground, (a) what

PROB LE M S

is the decrease in the player’s kinetic energy and (b) what is the increase in the thermal energy of his body and the ground along which he slides? 97 A 0.50 kg banana is thrown directly upward with an initial speed of 4.00 m/s and reaches a maximum height of 0.80 m. What change does air drag cause in the mechanical energy of the banana – Earth system during the ascent? 98 A metal tool is sharpened by being held against the rim of a wheel on a grinding machine by a force of 180 N. The frictional forces between the rim and the tool grind off small pieces of the tool. The wheel has a radius of 20.0 cm and rotates at 2.50 rev/s. The coefficient of kinetic friction between the wheel and the tool is 0.320. At what rate is energy being transferred from the motor driving the wheel to the thermal energy of the wheel and tool and to the kinetic energy of the material thrown from the tool? 99 A swimmer moves through the water at an average speed of 0.22 m/s. The average drag force is 110 N. What average power is required of the swimmer? 100 An automobile with passengers has weight 16 400 N and is moving at 113 km/h when the driver brakes, sliding to a stop. The frictional force on the wheels from the road has a magnitude of 8230 N. Find the stopping distance. 101 A 0.63 kg ball thrown directly upward with an initial speed of 14 m/s reaches a maximum height of 8.1 m. What is the change in the mechanical energy of the ball – Earth system during the ascent of the ball to that maximum height?

211

releases the spring, which pushes the ball through the muzzle. The ball leaves the spring just as it leaves the outer end of the muzzle. When the gun is inclined upward by 30# to the horizontal, a 57 g ball is shot to a maximum height of 1.83 m above the gun’s muzzle. Assume air drag on the ball is negligible. (a) At what speed does the spring launch the ball? (b) Assuming that friction on the ball within the gun can be neglected, find the spring’s initial compression distance. :

107 The only force acting on a particle is conservative force F . If the particle is at point A, the potential energy of the system associ: ated with F and the particle is 40 J. If the particle moves from point : A to point B, the work done on the particle by F is &25 J. What is the potential energy of the system with the particle at B? 108 In 1981, Daniel Goodwin climbed 443 m up the exterior of the Sears Building in Chicago using suction cups and metal clips. (a) Approximate his mass and then compute how much energy he had to transfer from biomechanical (internal) energy to the gravitational potential energy of the Earth – Goodwin system to lift himself to that height. (b) How much energy would he have had to transfer if he had, instead, taken the stairs inside the building (to the same height)? 109 A 60.0 kg circus performer slides 4.00 m down a pole to the circus floor, starting from rest. What is the kinetic energy of the performer as she reaches the floor if the frictional force on her from the pole (a) is negligible (she will be hurt) and (b) has a magnitude of 500 N?

102 The summit of Mount Everest is 8850 m above sea level. (a) How much energy would a 90 kg climber expend against the gravitational force on him in climbing to the summit from sea level? (b) How many candy bars, at 1.25 MJ per bar, would supply an energy equivalent to this? Your answer should suggest that work done against the gravitational force is a very small part of the energy expended in climbing a mountain.

110 A 5.0 kg block is projected at 5.0 m/s up a plane that is inclined at 30# with the horizontal. How far up along the plane does the block go (a) if the plane is frictionless and (b) if the coefficient of kinetic friction between the block and the plane is 0.40? (c) In the latter case, what is the increase in thermal energy of block and plane during the block’s ascent? (d) If the block then slides back down against the frictional force, what is the block’s speed when it reaches the original projection point?

103 A sprinter who weighs 670 N runs the first 7.0 m of a race in 1.6 s, starting from rest and accelerating uniformly. What are the sprinter’s (a) speed and (b) kinetic energy at the end of the 1.6 s? (c) What average power does the sprinter generate during the 1.6 s interval?

111 A 9.40 kg projectile is fired vertically upward. Air drag decreases the mechanical energy of the projectile–Earth system by 68.0 kJ during the projectile’s ascent. How much higher would the projectile have gone were air drag negligible?

104 A 20 kg object is acted on by a conservative force given by F ! %3.0x % 5.0x2, with F in newtons and x in meters. Take the potential energy associated with the force to be zero when the object is at x ! 0. (a) What is the potential energy of the system associated with the force when the object is at x ! 2.0 m? (b) If the object has a velocity of 4.0 m/s in the negative direction of the x axis when it is at x ! 5.0 m, what is its speed when it passes through the origin? (c) What are the answers to (a) and (b) if the potential energy of the system is taken to be %8.0 J when the object is at x ! 0?

112 A 70.0 kg man jumping from a window lands in an elevated fire rescue net 11.0 m below the window. He momentarily stops when he has stretched the net by 1.50 m. Assuming that mechanical energy is conserved during this process and that the net functions like an ideal spring, find the elastic potential energy of the net when it is stretched by 1.50 m. 113 A 30 g bullet moving a horizontal velocity of 500 m/s comes to a stop 12 cm within a solid wall. (a) What is the change in the bullet’s mechanical energy? (b) What is the magnitude of the average force from the wall stopping it?

105 A machine pulls a 40 kg trunk 2.0 m up a 40# ramp at constant velocity, with the machine’s force on the trunk directed parallel to the ramp. The coefficient of kinetic friction between the trunk and the ramp is 0.40. What are (a) the work done on the trunk by the machine’s force and (b) the increase in thermal energy of the trunk and the ramp?

114 A 1500 kg car starts from rest on a horizontal road and gains a speed of 72 km/h in 30 s. (a) What is its kinetic energy at the end of the 30 s? (b) What is the average power required of the car during the 30 s interval? (c) What is the instantaneous power at the end of the 30 s interval, assuming that the acceleration is constant?

106 The spring in the muzzle of a child’s spring gun has a spring constant of 700 N/m. To shoot a ball from the gun, first the spring is compressed and then the ball is placed on it. The gun’s trigger then

115 A 1.50 kg snowball is shot upward at an angle of 34.0# to the horizontal with an initial speed of 20.0 m/s. (a) What is its initial kinetic energy? (b) By how much does the gravitational potential

212

CHAPTE R 8 POTE NTIAL E N E RGY AN D CONSE RVATION OF E N E RGY

energy of the snowball – Earth system change as the snowball moves from the launch point to the point of maximum height? (c) What is that maximum height? 116 A 68 kg sky diver falls at a constant terminal speed of 59 m/s. (a) At what rate is the gravitational potential energy of the Earth – sky diver system being reduced? (b) At what rate is the system’s mechanical energy being reduced? 117 A 20 kg block on a horizontal surface is attached to a horizontal spring of spring constant k ! 4.0 kN/m. The block is pulled to the right so that the spring is stretched 10 cm beyond its relaxed length, and the block is then released from rest. The frictional force between the sliding block and the surface has a magnitude of 80 N. (a) What is the kinetic energy of the block when it has moved 2.0 cm from its point of release? (b) What is the kinetic energy of the block when it first slides back through the point at which the spring is relaxed? (c) What is the maximum kinetic energy attained by the block as it slides from its point of release to the point at which the spring is relaxed? 118 Resistance to the motion of an automobile consists of road friction, which is almost independent of speed, and air drag, which is proportional to speed-squared. For a certain car with a weight of 12 000 N, the total resistant force F is given by F ! 300 & 1.8v2, with F in newtons and v in meters per second. Calculate the power (in horsepower) required to accelerate the car at 0.92 m/s2 when the speed is 80 km/h. 119 SSM A 50 g ball is thrown from a window with an initial velocity of 8.0 m/s at an angle of 30# above the horizontal. Using energy methods, determine (a) the kinetic energy of the ball at the top of its flight and (b) its speed when it is 3.0 m below the window. Does the answer to (b) depend on either (c) the mass of the ball or (d) the initial angle? 120 A spring with a spring constant of 3200 N/m is initially stretched until the elastic potential energy of the spring is 1.44 J. (U ! 0 for the relaxed spring.) What is "U if the initial stretch is changed to (a) a stretch of 2.0 cm, (b) a compression of 2.0 cm, and (c) a compression of 4.0 cm? 121 A locomotive with a power capability of 1.5 MW can accelerate a train from a speed of 10 m/s to 25 m/s in 6.0 min. (a) Calculate the mass of the train. Find (b) the speed of the train and (c) the force accelerating the train as functions of time (in seconds) during the 6.0 min interval. (d) Find the distance moved by the train during the interval. 122 SSM A 0.42 kg shuffleboard disk is initially at rest when a player uses a cue to increase its speed to 4.2 m/s at constant acceleration. The acceleration takes place over a 2.0 m distance, at the end of which the cue loses contact with the disk. Then the disk slides an additional 12 m before stopping. Assume that the shuffleboard court is level and that the force of friction on the disk is constant. What is the increase in the thermal energy of the disk – court system (a) for that additional 12 m and (b) for the entire 14 m distance? (c) How much work is done on the disk by the cue? 123 A river descends 15 m through rapids. The speed of the water is 3.2 m/s upon entering the rapids and 13 m/s upon leaving. What percentage of the gravitational potential energy of the water–Earth system is transferred to kinetic energy during the descent? (Hint: Consider the descent of, say, 10 kg of water.)

124 The magnitude of the gravitational force between a particle of mass m1 and one of mass m2 is given by F(x) ! G

m1m2 , x2

where G is a constant and x is the distance between the particles. (a) What is the corresponding potential energy function U(x)? Assume that U(x) : 0 as x : * and that x is positive. (b) How much work is required to increase the separation of the particles from x ! x1 to x ! x1 & d? 125 Approximately 5.5 $ 106 kg of water falls 50 m over Niagara Falls each second. (a) What is the decrease in the gravitational potential energy of the water–Earth system each second? (b) If all this energy could be converted to electrical energy (it cannot be), at what rate would electrical energy be supplied? (The mass of 1 m3 of water is 1000 kg.) (c) If the electrical energy were sold at 1 cent/kW ) h, what would be the yearly income? 126 To make a pendulum, a 300 g ball is attached to one end of a string that has a length of 1.4 m and negligible mass. (The other end of the string is fixed.) The ball is pulled to one side until the string makes an angle of 30.0# with the vertical; then (with the string taut) the ball is released from rest. Find (a) the speed of the ball when the string makes an angle of 20.0# with the vertical and (b) the maximum speed of the ball. (c) What is the angle between the string and the vertical when the speed of the ball is one-third its maximum value? 127 In a circus act, a 60 kg clown is shot from a cannon with an initial velocity of 16 m/s at some unknown angle above the horizontal. A short time later the clown lands in a net that is 3.9 m vertically above the clown’s initial position. Disregard air drag. What is the kinetic energy of the clown as he lands in the net? 128 A 70 kg firefighter slides, from rest, 4.3 m down a vertical pole. (a) If the firefighter holds onto the pole lightly, so that the frictional force of the pole on her is negligible, what is her speed just before reaching the ground floor? (b) If the firefighter grasps the pole more firmly as she slides, so that the average frictional force of the pole on her is 500 N upward, what is her speed just before reaching the ground floor? 129 The surface of the continental United States has an area of about 8 $ 106 km2 and an average elevation of about 500 m (above sea level). The average yearly rainfall is 75 cm. The fraction of this rainwater that returns to the atmosphere by evaporation is 23 ; the rest eventually flows into the ocean. If the decrease in gravitational potential energy of the water–Earth system associated with that flow could be fully converted to electrical energy, what would be the average power? (The mass of 1 m3 of water is 1000 kg.) 130 A spring with spring constant k ! 200 N/m is suspended vertically with its upper end fixed to the ceiling and its lower end at position y ! 0. A block of weight 20 N is attached to the lower end, held still for a moment, and then released. What are (a) the kinetic energy K, (b) the change (from the initial value) in the gravitational potential energy "Ug, and (c) the change in the elastic potential energy "Ue of the spring–block system when the block is at y ! %5.0 cm? What are (d) K, (e) "Ug, and (f) "Ue when y ! %10 cm, (g) K, (h) "Ug, and (i) "Ue when y ! %15 cm, and (j) K, (k) "Ug, and (l) "Ue when y ! %20 cm?

213

PROB LE M S

8000

Force (N)

6000

134 Figure 8-73a shows a molecule consisting of two atoms of masses m and M (with m , M) and separation r. Figure 8-73b shows the potential energy U(r) of the molecule as a function of r. Describe the motion of the atoms (a) if the total mechanical energy E of the two-atom system is greater than zero (as is E1), and (b) if E is less than zero (as is E2). For E1 ! 1 $ 10%19 J and r ! 0.3 nm, find (c) the potential energy of the system, (d) the total kinetic energy of the atoms, and (e) the force (magnitude and direction) acting on each atom. For what values of r is the force (f) repulsive, (g) attractive, and (h) zero?

4 3 2 1 0

1

2

3 4 x (m)

5

6

Figure 8-72 Problem 133.

r M

m (a)

3 2 1 0 –1 –2 –3

E1 E2

0

0.1

0.2 0.3 r (nm) (b)

0.4

Figure 8-73 Problem 134.

135 Repeat Problem 83, but now with the block accelerated up a frictionless plane inclined at 5.0# to the horizontal.

4000

2000

0

133 Conservative force F(x) acts on a particle that moves along an x axis. Figure 8-72 shows how the potential energy U(x) associated with force F(x) varies with the position of the particle, (a) Plot F(x) for the range 0 + x + 6 m. (b) The mechanical energy E of the system is 4.0 J. Plot the kinetic energy K(x) of the particle directly on Fig. 8-72.

U(x) ( J )

132 The maximum force you can exert on an object with one of your back teeth is about 750 N. Suppose that as you gradually bite on a clump of licorice, the licorice resists compression by one of your teeth by acting like a spring for which k ! 2.5 $ 105 N/m. Find (a) the distance the licorice is compressed by your tooth and (b) the work the tooth does on the licorice during the compression. (c) Plot the magnitude of your force versus the compression distance. (d) If there is a potential energy associated with this compression, plot it versus compression distance. In the 1990s the pelvis of a particular Triceratops dinosaur was found to have deep bite marks. The shape of the marks suggested that they were made by a Tyrannosaurus rex dinosaur. To test the idea, researchers made a replica of a T. rex tooth from bronze and aluminum and then used a hydraulic press to gradually drive the replica into cow bone to the depth seen in the Triceratops bone. A graph of the force required versus depth of penetration is given in Fig. 8-71 for one trial; the required force increased with depth because, as the nearly conical tooth penetrated the bone, more of the tooth came in contact with the bone. (e) How much work was done by the hydraulic press—and thus presumably by the T. rex—in such a penetration? (f) Is there a potential energy associated with this penetration? (The large biting force and energy expenditure

attributed to the T. rex by this research suggest that the animal was a predator and not a scavenger.)

U(r), E (10–19 J)

131 Fasten one end of a vertical spring to a ceiling, attach a cabbage to the other end, and then slowly lower the cabbage until the upward force on it from the spring balances the gravitational force on it. Show that the loss of gravitational potential energy of the cabbage–Earth system equals twice the gain in the spring’s potential energy.

0

1

2

3 4 5 6 7 8 9 10 11 12 Penetration depth (mm)

Figure 8-71 Problem 132.

136 A spring with spring constant k ! 620 N/m is placed in a vertical orientation with its lower end supported by a horizontal surface. The upper end is depressed 25 cm, and a block with a weight of 50 N is placed (unattached) on the depressed spring. The system is then released from rest. Assume that the gravitational potential energy Ug of the block is zero at the release point (y ! 0) and calculate the kinetic energy K of the block for y equal to (a) 0, (b) 0.050 m, (c) 0.10 m, (d) 0.15 m, and (e) 0.20 m. Also, (f) how far above its point of release does the block rise?

C

H

A

P

T

E

R

9

Center of Mass and Linear Momentum 9-1 CENTER OF MASS Learning Objectives 9.03 For a two-dimensional or three-dimensional extended object with a uniform distribution of mass, determine the center of mass by (a) mentally dividing the object into simple geometric figures, each of which can be replaced by a particle at its center and (b) finding the center of mass of those particles.

After reading this module, you should be able to . . .

9.01 Given the positions of several particles along an axis or a plane, determine the location of their center of mass. 9.02 Locate the center of mass of an extended, symmetric object by using the symmetry.

Key Idea ● The center of mass of a system of n particles is defined to be the point whose coordinates are given by

xcom !

1 M

or

n

' mi xi , i!1

n

ycom !

1 M

' mi yi , i!1

: rcom

1 M

mi : ri , ' i!1

!

zcom !

1 M

n

' mi zi , i!1

n

where M is the total mass of the system.

What Is Physics? Every mechanical engineer who is hired as a courtroom expert witness to reconstruct a traffic accident uses physics. Every dance trainer who coaches a ballerina on how to leap uses physics. Indeed, analyzing complicated motion of any sort requires simplification via an understanding of physics. In this chapter we discuss how the complicated motion of a system of objects, such as a car or a ballerina, can be simplified if we determine a special point of the system — the center of mass of that system. Here is a quick example. If you toss a ball into the air without much spin on the ball (Fig. 9-1a), its motion is simple — it follows a parabolic path, as we discussed in Chapter 4, and the ball can be treated as a particle. If, instead, you flip a baseball bat into the air (Fig. 9-1b), its motion is more complicated. Because every part of the bat moves differently, along paths of many different shapes, you cannot represent the bat as a particle. Instead, it is a system of particles each of which follows its own path through the air. However, the bat has one special point — the center of mass — that does move in a simple parabolic path. The other parts of the bat move around the center of mass. (To locate the center of mass, balance the bat on an outstretched finger; the point is above your finger, on the bat’s central axis.) You cannot make a career of flipping baseball bats into the air, but you can make a career of advising long-jumpers or dancers on how to leap properly into the air while either moving their arms and legs or rotating their torso. Your starting point would be to determine the person’s center of mass because of its simple motion. 214

9-1 CE NTE R OF MASS

215

The Center of Mass We define the center of mass (com) of a system of particles (such as a person) in order to predict the possible motion of the system. Richard Megna/Fundamental Photographs

The center of mass of a system of particles is the point that moves as though (1) all of the system’s mass were concentrated there and (2) all external forces were applied there.

Here we discuss how to determine where the center of mass of a system of particles is located. We start with a system of only a few particles, and then we consider a system of a great many particles (a solid body, such as a baseball bat). Later in the chapter, we discuss how the center of mass of a system moves when external forces act on the system.

Systems of Particles Two Particles. Figure 9-2a shows two particles of masses m1 and m2 separated by distance d.We have arbitrarily chosen the origin of an x axis to coincide with the particle of mass m1.We define the position of the center of mass (com) of this two-particle system to be xcom !

m2 d. m1 & m2

(a)

(9-1)

Suppose, as an example, that m2 ! 0. Then there is only one particle, of mass m1, and the center of mass must lie at the position of that particle; Eq. 9-1 dutifully reduces to xcom ! 0. If m1 ! 0, there is again only one particle (of mass m2), and we have, as we expect, xcom ! d. If m1 ! m2, the center of mass should be halfway between the two particles; Eq. 9-1 reduces to xcom ! 12 d, again as we expect. Finally, Eq. 9-1 tells us that if neither m1 nor m2 is zero, xcom can have only values that lie between zero and d; that is, the center of mass must lie somewhere between the two particles. We are not required to place the origin of the coordinate system on one of the particles. Figure 9-2b shows a more generalized situation, in which the coordinate system has been shifted leftward. The position of the center of mass is now defined as

xcom !

m1x1 & m2 x2 . m1 & m2

(b)

Figure 9-1 (a) A ball tossed into the air follows a parabolic path. (b) The center of mass (black dot) of a baseball bat flipped into the air follows a parabolic path, but all other points of the bat follow more complicated curved paths.

(9-2)

Note that if we put x1 ! 0, then x2 becomes d and Eq. 9-2 reduces to Eq. 9-1, as it must. Note also that in spite of the shift of the coordinate system, the center

y

This is the center of mass of the two-particle system.

xcom m1

m2 com d

y xcom m2

m1

x

com x1

d x2

(a)

(b)

Figure 9-2 (a) Two particles of masses m1 and m2 are separated by distance d. The dot labeled com shows the position of the center of mass, calculated from Eq. 9-1. (b) The same as (a) except that the origin is located farther from the particles. The position of the center of mass is calculated from Eq. 9-2. The location of the center of mass with respect to the particles is the same in both cases.

x

Shifting the axis does not change the relative position of the com.

216

CHAPTE R 9 CE NTE R OF MASS AN D LI N EAR M OM E NTU M

of mass is still the same distance from each particle. The com is a property of the physical particles, not the coordinate system we happen to use. We can rewrite Eq. 9-2 as xcom !

m1x1 & m2 x2 , M

(9-3)

in which M is the total mass of the system. (Here, M ! m1 & m2.) Many Particles. We can extend this equation to a more general situation in which n particles are strung out along the x axis. Then the total mass is M ! m1 & m2 & ) ) ) & mn, and the location of the center of mass is xcom ! !

m1 x1 & m2 x2 & m3 x3 & ) ) ) & mn xn M 1 M

n

' mi xi . i!1

(9-4)

The subscript i is an index that takes on all integer values from 1 to n. Three Dimensions. If the particles are distributed in three dimensions, the center of mass must be identified by three coordinates. By extension of Eq. 9-4, they are xcom !

1 M

n

' mi xi, i!1

ycom !

1 M

n

' mi yi, i!1

zcom !

1 M

n

' mizi. i!1

(9-5)

We can also define the center of mass with the language of vectors. First recall that the position of a particle at coordinates xi, yi, and zi is given by a position vector (it points from the origin to the particle): r:i ! xi iˆ & yi ˆj & zi kˆ .

(9-6)

Here the index identifies the particle, and ˆi, ˆj, and kˆ are unit vectors pointing, respectively, in the positive direction of the x, y, and z axes. Similarly, the position of the center of mass of a system of particles is given by a position vector: ˆ rcom ! xcomˆi & ycomˆj & zcom k.

(9-7)

:

If you are a fan of concise notation, the three scalar equations of Eq. 9-5 can now be replaced by a single vector equation, rcom !

:

1 M

n

' miri,

(9-8)

:

i!1

where again M is the total mass of the system. You can check that this equation is correct by substituting Eqs. 9-6 and 9-7 into it, and then separating out the x, y, and z components. The scalar relations of Eq. 9-5 result.

Solid Bodies An ordinary object, such as a baseball bat, contains so many particles (atoms) that we can best treat it as a continuous distribution of matter. The “particles” then become differential mass elements dm, the sums of Eq. 9-5 become integrals, and the coordinates of the center of mass are defined as xcom !

1 M

"

x dm,

ycom !

1 M

"

y dm,

zcom !

1 M

"

z dm,

(9-9)

where M is now the mass of the object.The integrals effectively allow us to use Eq. 9-5 for a huge number of particles, an effort that otherwise would take many years. Evaluating these integrals for most common objects (such as a television set or a moose) would be difficult, so here we consider only uniform objects. Such objects have uniform density, or mass per unit volume; that is, the density r (Greek letter

217

9-1 CE NTE R OF MASS

rho) is the same for any given element of an object as for the whole object. From Eq. 1-8, we can write M dm (9-10) ! , r! dV V where dV is the volume occupied by a mass element dm, and V is the total volume of the object. Substituting dm ! (M/V) dV from Eq. 9-10 into Eq. 9-9 gives xcom !

1 V

"

x dV,

ycom !

1 V

"

y dV,

zcom !

1 V

"

(9-11)

z dV.

Symmetry as a Shortcut. You can bypass one or more of these integrals if an object has a point, a line, or a plane of symmetry. The center of mass of such an object then lies at that point, on that line, or in that plane. For example, the center of mass of a uniform sphere (which has a point of symmetry) is at the center of the sphere (which is the point of symmetry). The center of mass of a uniform cone (whose axis is a line of symmetry) lies on the axis of the cone. The center of mass of a banana (which has a plane of symmetry that splits it into two equal parts) lies somewhere in the plane of symmetry. The center of mass of an object need not lie within the object. There is no dough at the com of a doughnut, and no iron at the com of a horseshoe. Sample Problem 9.01 com of three particles Three particles of masses m1 ! 1.2 kg, m2 ! 2.5 kg, and m 3 ! 3.4 kg form an equilateral triangle of edge length a ! 140 cm. Where is the center of mass of this system? KEY IDEA We are dealing with particles instead of an extended solid body, so we can use Eq. 9-5 to locate their center of mass. The particles are in the plane of the equilateral triangle, so we need only the first two equations. Calculations: We can simplify the calculations by choosing the x and y axes so that one of the particles is located at the origin and the x axis coincides with one of the triangle’s

This is the position vector rcom for the com (it points from the origin to the com).

150 m3 100 a

0 m1

Mass (kg)

x (cm)

y (cm)

1 2 3

1.2 2.5 3.4

0 140 70

0 0 120

The total mass M of the system is 7.1 kg. From Eq. 9-5, the coordinates of the center of mass are xcom !

rcom 50 xcom 100

1 M

3

' mi xi ! i!1

m1 x1 & m2 x2 & m3 x3 M

(1.2 kg)(0) & (2.5 kg)(140 cm) & (3.4 kg)(70 cm) 7.1 kg

! 83 cm and ycom !

1 M

(Answer) 3

' mi yi ! i!1

m1y1 & m2 y2 & m3y3 M

(1.2 kg)(0) & (2.5 kg)(0) & (3.4 kg)(120 cm) 7.1 kg ! 58 cm. (Answer)

a

!

50

0

Particle

!

y

ycom

sides (Fig. 9-3). The three particles then have the following coordinates:

m2 150

x

Figure 9-3 Three particles form an equilateral triangle of edge length a. The center of mass is located by the position vector : r com.

In Fig. 9-3, the center of mass is located by the position vecr com, which has components xcom and ycom. If we had tor : chosen some other orientation of the coordinate system, these coordinates would be different but the location of the com relative to the particles would be the same.

Additional examples, video, and practice available at WileyPLUS

218

CHAPTE R 9 CE NTE R OF MASS AN D LI N EAR M OM E NTU M

Sample Problem 9.02 com of plate with missing piece This sample problem has lots of words to read, but they will allow you to calculate a com using easy algebra instead of challenging integral calculus. Figure 9-4a shows a uniform metal plate P of radius 2R from which a disk of radius R has been stamped out (removed) in an assembly line. The disk is shown in Fig. 9-4b. Using the xy coordinate system shown, locate the center of mass comP of the remaining plate. KEY IDEAS (1) Let us roughly locate the center of plate P by using symmetry. We note that the plate is symmetric about the x axis (we get the portion below that axis by rotating the upper portion about the axis). Thus, comP must be on the x axis. The plate (with the disk removed) is not symmetric about the y axis. However, because there is somewhat more mass on the right of the y axis, comP must be somewhat to the right of that axis. Thus, the location of comP should be roughly as indicated in Fig. 9-4a. (2) Plate P is an extended solid body, so in principle we can use Eqs. 9-11 to find the actual coordinates of the center of mass of plate P. Here we want the xy coordinates of the center of mass because the plate is thin and uniform. If it had any appreciable thickness, we would just say that the center of mass is midway across the thickness. Still, using Eqs. 9-11 would be challenging because we would need a function for the shape of the plate with its hole, and then we would need to integrate the function in two dimensions. (3) Here is a much easier way: In working with centers of mass, we can assume that the mass of a uniform object (as we have here) is concentrated in a particle at the object’s center of mass. Thus we can treat the object as a particle and avoid any two-dimensional integration. Calculations: First, put the stamped-out disk (call it disk S) back into place (Fig. 9-4c) to form the original composite plate (call it plate C). Because of its circular symmetry, the center of mass comS for disk S is at the center of S, at x ! %R (as shown). Similarly, the center of mass comC for composite plate C is at the center of C, at the origin (as shown). We then have the following:

Plate

Center of Mass

Location of com

Mass

P S C

comP comS comC

xP ! ? xS ! %R xC ! 0

mP mS mC ! mS & mP

Assume that mass mS of disk S is concentrated in a particle at xS ! %R, and mass mP is concentrated in a particle at xP (Fig. 9-4d). Next we use Eq. 9-2 to find the center of mass xS&P of the two-particle system: xS&P !

mS xS & mP xP . mS & mP

(9-12)

Next note that the combination of disk S and plate P is composite plate C. Thus, the position xS&P of comS&P must coincide with the position xC of comC, which is at the origin; so xS&P ! xC ! 0. Substituting this into Eq. 9-12, we get xP ! %xS

mS . mP

(9-13)

We can relate these masses to the face areas of S and P by noting that mass ! density $ volume ! density $ thickness $ area. Then

mS densityS thicknessS areaS ! $ $ . mP densityP thicknessP areaP

Because the plate is uniform, the densities and thicknesses are equal; we are left with areaS areaS mS ! ! mP areaP areaC % areaS !

pR2 1 ! . p(2R)2 % pR2 3

Substituting this and xS ! %R into Eq. 9-13, we have xP ! 13R.

(Answer)

Additional examples, video, and practice available at WileyPLUS

y

Checkpoint 1 The figure shows a uniform square plate from which four identical squares at the corners will be removed. (a) Where is the center of mass of the plate originally? Where is it after the removal of (b) square 1; (c) squares 1 and 2; (d) squares 1 and 3; (e) squares 1, 2, and 3; (f) all four squares? Answer in terms of quadrants, axes, or points (without calculation, of course).

1

2 x

4

3

9-1 CE NTE R OF MASS

A

y

2R R comP

x

Assume the plate's mass is concentrated as a particle at the plate's center of mass.

Plate P

(a)

Here too, assume the mass is concentrated as a particle at the center of mass.

y

Disk S x

comS (b)

y

Composite plate C=S+P

Here too.

comC

(c)

Here are those three particles. (d)

comS Disk particle

219

comC comP Plate particle

x

The com of the composite plate is the same as the com of the two pieces. Figure 9-4 (a) Plate P is a metal plate of radius 2R, with a circular hole of radius R. The center of mass of P is at point comP. (b) Disk S. (c) Disk S has been put back into place to form a composite plate C. The center of mass comS of disk S and the center of mass comC of plate C are shown. (d) The center of mass comS&P of the combination of S and P coincides with comC, which is at x ! 0.

220

CHAPTE R 9 CE NTE R OF MASS AN D LI N EAR M OM E NTU M

9-2 NEWTON’S SECOND LAW FOR A SYSTEM OF PARTICLES Learning Objectives After reading this module, you should be able to . . .

9.04 Apply Newton’s second law to a system of particles by relating the net force (of the forces acting on the particles) to the acceleration of the system’s center of mass. 9.05 Apply the constant-acceleration equations to the motion of the individual particles in a system and to the motion of the system’s center of mass. 9.06 Given the mass and velocity of the particles in a system, calculate the velocity of the system’s center of mass. 9.07 Given the mass and acceleration of the particles in a system, calculate the acceleration of the system’s center of mass.

9.08 Given the position of a system’s center of mass as a function of time, determine the velocity of the center of mass. 9.09 Given the velocity of a system’s center of mass as a function of time, determine the acceleration of the center of mass. 9.10 Calculate the change in the velocity of a com by integrating the com’s acceleration function with respect to time. 9.11 Calculate a com’s displacement by integrating the com’s velocity function with respect to time. 9.12 When the particles in a two-particle system move without the system’s com moving, relate the displacements of the particles and the velocities of the particles.

Key Idea ● The motion of the center of mass of any system of particles

is governed by Newton’s second law for a system of particles, which is : Fnet ! M : a com.

:

Here Fnet is the net force of all the external forces acting on the system, M is the total mass of the system, and : a com is the acceleration of the system’s center of mass.

Newton’s Second Law for a System of Particles Now that we know how to locate the center of mass of a system of particles, we discuss how external forces can move a center of mass. Let us start with a simple system of two billiard balls. If you roll a cue ball at a second billiard ball that is at rest, you expect that the two-ball system will continue to have some forward motion after impact. You would be surprised, for example, if both balls came back toward you or if both moved to the right or to the left. You already have an intuitive sense that something continues to move forward. What continues to move forward, its steady motion completely unaffected by the collision, is the center of mass of the two-ball system. If you focus on this point — which is always halfway between these bodies because they have identical masses — you can easily convince yourself by trial at a billiard table that this is so. No matter whether the collision is glancing, head-on, or somewhere in between, the center of mass continues to move forward, as if the collision had never occurred. Let us look into this center-of-mass motion in more detail. Motion of a System’s com. To do so, we replace the pair of billiard balls with a system of n particles of (possibly) different masses. We are interested not in the individual motions of these particles but only in the motion of the center of mass of the system. Although the center of mass is just a point, it moves like a particle whose mass is equal to the total mass of the system; we can assign a position, a velocity, and an acceleration to it. We state (and shall prove next) that the vector equation that governs the motion of the center of mass of such a system of particles is :

Fnet ! M : acom

(system of particles).

(9-14)

This equation is Newton’s second law for the motion of the center of mass of a system of particles. Note that its form is the same as the form of the equation

9-2 N EWTON’S SECON D L AW FOR A SYSTE M OF PARTICLES :

: (Fnet ! ma ) for the motion of a single particle. However, the three quantities that appear in Eq. 9-14 must be evaluated with some care:

:

1. Fnet is the net force of all external forces that act on the system. Forces on one part of the system from another part of the system (internal forces) are not included in Eq. 9-14. 2. M is the total mass of the system. We assume that no mass enters or leaves the system as it moves, so that M remains constant. The system is said to be closed. 3. : acom is the acceleration of the center of mass of the system. Equation 9-14 gives no information about the acceleration of any other point of the system. Equation 9-14 is equivalent to three equations involving the components of : Fnet and : acom along the three coordinate axes. These equations are Fnet, x ! Macom, x

Fnet, y ! Macom, y

Fnet, z ! Macom, z.

(9-15)

Billiard Balls. Now we can go back and examine the behavior of the billiard balls. Once the cue ball has begun to roll, no net external force acts on the (two: ball) system. Thus, because Fnet ! 0, Eq. 9-14 tells us that : acom ! 0 also. Because acceleration is the rate of change of velocity, we conclude that the velocity of the center of mass of the system of two balls does not change. When the two balls collide, the forces that come into play are internal forces, on one ball from the other. : Such forces do not contribute to the net force Fnet, which remains zero. Thus, the center of mass of the system, which was moving forward before the collision, must continue to move forward after the collision, with the same speed and in the same direction. Solid Body. Equation 9-14 applies not only to a system of particles but also to a solid body, such as the bat of Fig. 9-1b. In that case, M in Eq. 9-14 is the mass : of the bat and Fnet is the gravitational force on the bat. Equation 9-14 then tells us that : acom ! : g. In other words, the center of mass of the bat moves as if the bat : were a single particle of mass M, with force Fg acting on it. Exploding Bodies. Figure 9-5 shows another interesting case. Suppose that at a fireworks display, a rocket is launched on a parabolic path. At a certain point, it explodes into fragments. If the explosion had not occurred, the rocket would have continued along the trajectory shown in the figure. The forces of the explosion are internal to the system (at first the system is just the rocket, and later it is its fragments); that is, they are forces on parts of the system from other parts. If we ignore : air drag, the net external force Fnet acting on the system is the gravitational force on the system, regardless of whether the rocket explodes. Thus, from Eq. 9-14, the acceleration : acom of the center of mass of the fragments (while they are in flight) remains equal to : g. This means that the center of mass of the fragments follows the same parabolic trajectory that the rocket would have followed had it not exploded. Ballet Leap. When a ballet dancer leaps across the stage in a grand jeté, she raises her arms and stretches her legs out horizontally as soon as her feet leave the The internal forces of the explosion cannot change the path of the com.

Figure 9-5 A fireworks rocket explodes in flight. In the absence of air drag, the center of mass of the fragments would continue to follow the original parabolic path, until fragments began to hit the ground.

221

222

CHAPTE R 9 CE NTE R OF MASS AN D LI N EAR M OM E NTU M Path of head

Path of center of mass

Figure 9-6 A grand jeté. (Based on The Physics of Dance, by Kenneth Laws, Schirmer Books, 1984.)

stage (Fig. 9-6). These actions shift her center of mass upward through her body. Although the shifting center of mass faithfully follows a parabolic path across the stage, its movement relative to the body decreases the height that is attained by her head and torso, relative to that of a normal jump.The result is that the head and torso follow a nearly horizontal path, giving an illusion that the dancer is floating.

Proof of Equation 9-14 Now let us prove this important equation. From Eq. 9-8 we have, for a system of n particles, M: r com ! m1: r1 & m2: r2 & m3:r3 & ) ) ) & mnr:n , (9-16) in which M is the system’s total mass and : rcom is the vector locating the position of the system’s center of mass. Differentiating Eq. 9-16 with respect to time gives : : : : : Mv com ! m1v1 & m2v2 & m3v3 & ) ) ) & mnvn.

(9-17)

Here v:i (! d : ri /dt) is the velocity of the ith particle, and : r com /dt) is the v com (! d : velocity of the center of mass. Differentiating Eq. 9-17 with respect to time leads to M a:com ! m1a:1 & m2a:2 & m3a:3 & ) ) ) & mna:n.

(9-18)

Here a:i (! d : v i /dt) is the acceleration of the ith particle, and a:com (! d : v com /dt) is the acceleration of the center of mass. Although the center of mass is just a geometrical point, it has a position, a velocity, and an acceleration, as if it were a particle. : From Newton’s second law, mi a:i is equal to the resultant force Fi that acts on the ith particle. Thus, we can rewrite Eq. 9-18 as :

:

:

:

Ma:com ! F1 & F2 & F3 & ) ) ) & Fn.

(9-19)

Among the forces that contribute to the right side of Eq. 9-19 will be forces that the particles of the system exert on each other (internal forces) and forces exerted on the particles from outside the system (external forces). By Newton’s third law, the internal forces form third-law force pairs and cancel out in the sum that appears on the right side of Eq. 9-19. What remains is the vector sum of all the external forces that act on the system. Equation 9-19 then reduces to Eq. 9-14, the relation that we set out to prove.

223

9-2 N EWTON’S SECON D L AW FOR A SYSTE M OF PARTICLES

Checkpoint 2 Two skaters on frictionless ice hold opposite ends of a pole of negligible mass.An axis runs along it, with the origin at the center of mass of the two-skater system. One skater, Fred, weighs twice as much as the other skater, Ethel.Where do the skaters meet if (a) Fred pulls hand over hand along the pole so as to draw himself to Ethel, (b) Ethel pulls hand over hand to draw herself to Fred, and (c) both skaters pull hand over hand?

Sample Problem 9.03 Motion of the com of three particles If the particles in a system all move together, the com moves with them—no trouble there. But what happens when they move in different directions with different accelerations? Here is an example. The three particles in Fig. 9-7a are initially at rest. Each experiences an external force due to bodies outside the three-particle system. The directions are indicated, and the magnitudes are F1 ! 6.0 N, F2 ! 12 N, and F3 ! 14 N. What is the acceleration of the center of mass of the system, and in what direction does it move? KEY IDEAS The position of the center of mass is marked by a dot in the figure. We can treat the center of mass as if it were a real particle, with a mass equal to the system’s total mass M ! 16 kg. We can also treat the three external forces as if they act at the center of mass (Fig. 9-7b).

Along the y axis, we have F 1y & F 2y & F 3y acom, y ! M 0 & (12 N) sin 45# & 0 ! ! 0.530 m/s2. 16 kg From these components, we find that : a com has the magnitude acom ! 2(acom, x)2 & (acom, y)2

(Answer) ! 1.16 m/s2 % 1.2 m/s2 and the angle (from the positive direction of the x axis)

' ! tan%1

F1

or so

&

: a com

: F2

!

& : F1

: F3

4.0 kg

! M a com

&

: F2

M

&

: F3

.

45°

1 –2

0

–1

8.0 kg

com 1

2

3

4

5

x

–1

(9-21)

Equation 9-20 tells us that the acceleration : a com of the center of mass is in the same direction as the net external force : Fnet on the system (Fig. 9-7b). Because the particles are initially at rest, the center of mass must also be at rest. As the center of mass then begins to accelerate, it must move off in : the common direction of : a com and Fnet. We can evaluate the right side of Eq. 9-21 directly on a vector-capable calculator, or we can rewrite Eq. 9-21 in component form, find the components of : a com, and then find : a com. Along the x axis, we have acom, x

F2

2

–3

:

(Answer)

! 27#.

3

(9-20)

: F net ! Ma com : F1

acom, x y

Calculations: We can now apply Newton’s second law : : (F net ! ma ) to the center of mass, writing :

acom, y

F 1x & F 2x & F 3x ! M %6.0 N & (12 N) cos 45# & 14 N ! ! 1.03 m/s2. 16 kg

–2

The com of the system will move as if all the mass were there and the net force acted there.

–3

4.0 kg F3 (a)

y F2

Fnet

3 2 1 –3

–2

–1

0

M = 16 kg F1

θ

acom

com 1 2 (b)

F3 3

4

5

x

Figure 9-7 (a) Three particles, initially at rest in the positions shown, are acted on by the external forces shown. The center of mass (com) of the system is marked. (b) The forces are now transferred to the center of mass of the system, which behaves like a particle with a mass M equal to the total mass of the system. The net external force : acom of the center of mass are shown. Fnet and the acceleration :

Additional examples, video, and practice available at WileyPLUS

224

CHAPTE R 9 CE NTE R OF MASS AN D LI N EAR M OM E NTU M

9-3 LINEAR MOMENTUM Learning Objectives After reading this module, you should be able to . . .

9.13 Identify that momentum is a vector quantity and thus has both magnitude and direction and also components. 9.14 Calculate the (linear) momentum of a particle as the product of the particle’s mass and velocity. 9.15 Calculate the change in momentum (magnitude and direction) when a particle changes its speed and direction of travel.

9.16 Apply the relationship between a particle’s momentum and the (net) force acting on the particle. 9.17 Calculate the momentum of a system of particles as the product of the system’s total mass and its center-of-mass velocity. 9.18 Apply the relationship between a system’s center-ofmass momentum and the net force acting on the system.

Key Ideas ● For a single particle, we define a quantity

p called its linear

:

momentum as

terms of this momentum:

d: p . dt ● For a system of particles these relations become : dP : : P ! Mv:com and Fnet ! . dt :

Fnet !

: , p ! mv

:

which is a vector quantity that has the same direction as the particle’s velocity. We can write Newton’s second law in

Linear Momentum Here we discuss only a single particle instead of a system of particles, in order to define two important quantities. Then we shall extend those definitions to systems of many particles. The first definition concerns a familiar word — momentum — that has several meanings in everyday language but only a single precise meaning in physics and : engineering. The linear momentum of a particle is a vector quantity p that is defined as : : p ! mv

(linear momentum of a particle),

(9-22)

in which m is the mass of the particle and : v is its velocity. (The adjective linear is of: ten dropped, but it serves to distinguish p from angular momentum, which is introduced in Chapter 11 and which is associated with rotation.) Since m is always a : positive scalar quantity, Eq. 9-22 tells us that p and : v have the same direction. From Eq. 9-22, the SI unit for momentum is the kilogram-meter per second (kg )m/s). Force and Momentum. Newton expressed his second law of motion in terms of momentum: The time rate of change of the momentum of a particle is equal to the net force acting on the particle and is in the direction of that force.

In equation form this becomes :

F net !

dp: . dt

(9-23) :

In words, Eq. 9-23 says that the net external force Fnet on a particle changes the : particle’s linear momentum p . Conversely, the linear momentum can be : changed only by a net external force. If there is no net external force, p cannot change. As we shall see in Module 9-5, this last fact can be an extremely powerful tool in solving problems.

9-3 LI N EAR M OM E NTU M

Manipulating Eq. 9-23 by substituting for : p from Eq. 9-22 gives, for constant mass m, : d dp d: v : : : Fnet ! ! (mv )!m ! ma . dt dt dt :

:

: : Thus, the relations F net ! dp are equivalent expressions of /dt and F net ! ma Newton’s second law of motion for a particle.

Checkpoint 3 The figure gives the magnitude p of the linear momentum versus time t for a particle moving along an axis.A force directed along the axis acts on the particle. (a) Rank the four regions indicated according to the magnitude of the force, greatest first. (b) In which region is the particle slowing?

p 2 1

3 4

t

The Linear Momentum of a System of Particles Let’s extend the definition of linear momentum to a system of particles. Consider a system of n particles, each with its own mass, velocity, and linear momentum. The particles may interact with each other, and external forces may act on them. : The system as a whole has a total linear momentum P, which is defined to be the vector sum of the individual particles’ linear momenta. Thus, :

P!: p1 & : p2 & : p3 & ) ) ) & : pn : : : ! m1 v 1 & m2 v 2 & m3 v 3 & ) ) ) & mn: v n.

(9-24)

If we compare this equation with Eq. 9-17, we see that :

P ! M: v com

(linear momentum, system of particles),

(9-25)

which is another way to define the linear momentum of a system of particles: The linear momentum of a system of particles is equal to the product of the total mass M of the system and the velocity of the center of mass.

Force and Momentum. If we take the time derivative of Eq. 9-25 (the velocity can change but not the mass), we find :

d: v com dP !M ! M: a com. dt dt

(9-26)

Comparing Eqs. 9-14 and 9-26 allows us to write Newton’s second law for a system of particles in the equivalent form :

:

Fnet ! :

dP dt

(system of particles),

(9-27)

where F net is the net external force acting on the system. This equation is the gen: : eralization of the single-particle equation F net ! dp /dt to a system of many : particles. In words, the equation says that the net external force F net on a system : of particles changes the linear momentum P of the system. Conversely, the linear momentum can be changed only by a net external force. If there is no net exter: nal force, P cannot change. Again, this fact gives us an extremely powerful tool for solving problems.

225

226

CHAPTE R 9 CE NTE R OF MASS AN D LI N EAR M OM E NTU M

9-4 COLLISION AND IMPULSE Learning Objectives After reading this module, you should be able to . . .

9.23 Given force as a function of time, calculate the impulse (and thus also the momentum change) by integrating the function. 9.24 Given a graph of force versus time, calculate the impulse (and thus also the momentum change) by graphical integration. 9.25 In a continuous series of collisions by projectiles, calculate the average force on the target by relating it to the rate at which mass collides and to the velocity change experienced by each projectile.

9.19 Identify that impulse is a vector quantity and thus has both magnitude and direction and also components. 9.20 Apply the relationship between impulse and momentum change. 9.21 Apply the relationship between impulse, average force, and the time interval taken by the impulse. 9.22 Apply the constant-acceleration equations to relate impulse to average force.

Key Ideas ● Applying Newton’s second law in momentum form to a particle-like body involved in a collision leads to the impulse–linear momentum theorem:

● When a steady stream of bodies, each with mass m and

speed v, collides with a body whose position is fixed, the average force on the fixed body is n n "p ! % m "v, Favg ! % "t "t

:

: pf % : p i ! "p ! J,

:

: where : pf % : p i ! "p is the change in the body’s linear momen: : tum, and J is the impulse due to the force F(t) exerted on the body by the other body in the collision: :

J !

"

tf

ti

where n/"t is the rate at which the bodies collide with the fixed body, and "v is the change in velocity of each colliding body. This average force can also be written as

:

F(t) dt.

Favg ! %

:

● If Favg is the average magnitude of F(t) during the collision

"m "v, "t

where "m/"t is the rate at which mass collides with the fixed body. The change in velocity is "v ! %v if the bodies stop upon impact and "v ! %2v if they bounce directly backward with no change in their speed.

and "t is the duration of the collision, then for one-dimensional motion J ! Favg "t.

Collision and Impulse Photo by Harold E. Edgerton. © The Harold and Esther Edgerton Family Trust, courtesy of Palm Press, Inc.

:

The collision of a ball with a bat collapses part of the ball.

The momentum p of any particle-like body cannot change unless a net external force changes it. For example, we could push on the body to change its momentum. More dramatically, we could arrange for the body to collide with a baseball bat. In such a collision (or crash), the external force on the body is brief, has large magnitude, and suddenly changes the body’s momentum. Collisions occur commonly in our world, but before we get to them, we need to consider a simple collision in which a moving particle-like body (a projectile) collides with some other body (a target).

Single Collision Let the projectile be a ball and the target be a bat. The collision is brief, and the ball experiences a force that is great enough to slow, stop, or even reverse its motion. : Figure 9-8 depicts the collision at one instant. The ball experiences a force F(t) that : varies during the collision and changes the linear momentum p of the ball. That : change is related to the force by Newton’s second law written in the form F ! dp:/dt. By rearranging this second-law expression, we see that, in time interval dt, the change in the ball’s momentum is :

: dp ! F(t) dt.

(9-28)

227

9-4 COLLISION AN D I M PU LSE

We can find the net change in the ball’s momentum due to the collision if we integrate both sides of Eq. 9-28 from a time ti just before the collision to a time tf just after the collision:

"

tf

dp ! :

ti

"

tf

ti

:

(9-29)

F(t) dt.

: : : The left side of this equation gives us the change in momentum: p f % pi ! "p. The right side, which is a measure of both the magnitude and the duration of the : collision force, is called the impulse J of the collision:

:

J !

"

tf

ti

:

F (t) dt

F (t)

(impulse defined).

Bat

x

Ball :

Figure 9-8 Force F (t) acts on a ball as the ball and a bat collide.

(9-30)

Thus, the change in an object’s momentum is equal to the impulse on the object: :

: "p ! J

(linear momentum – impulse theorem).

(9-31)

This expression can also be written in the vector form :

: : p f % pi ! J

(9-32)

"px ! Jx

(9-33)

and in such component forms as

and

pfx % pix !

"

tf

ti

The impulse in the collision is equal to the area under the curve.

(9-34)

F x dt. :

F

:

Integrating the Force. If we have a function for F (t), we can evaluate J (and : thus the change in momentum) by integrating the function. If we have a plot of F : versus time t, we can evaluate J by finding the area between the curve and the t axis, such as in Fig. 9-9a. In many situations we do not know how the force varies with time but we do know the average magnitude Favg of the force and the duration "t (! tf % ti) of the collision. Then we can write the magnitude of the impulse as J ! Favg "t.

F(t) J

t

(9-35)

The average force is plotted versus time as in Fig. 9-9b. The area under that curve is equal to the area under the curve for the actual force F(t) in Fig. 9-9a because both areas are equal to impulse magnitude J. Instead of the ball, we could have focused on the bat in Fig. 9-8. At any instant, Newton’s third law tells us that the force on the bat has the same magnitude but the opposite direction as the force on the ball. From Eq. 9-30, this means that the impulse on the bat has the same magnitude but the opposite direction as the impulse on the ball.

ti

tf ∆t (a)

F

The average force gives the same area under the curve.

Favg J t

Checkpoint 4 A paratrooper whose chute fails to open lands in snow; he is hurt slightly. Had he landed on bare ground, the stopping time would have been 10 times shorter and the collision lethal. Does the presence of the snow increase, decrease, or leave unchanged the values of (a) the paratrooper’s change in momentum, (b) the impulse stopping the paratrooper, and (c) the force stopping the paratrooper?

Series of Collisions Now let’s consider the force on a body when it undergoes a series of identical, repeated collisions. For example, as a prank, we might adjust one of those machines that fire tennis balls to fire them at a rapid rate directly at a wall. Each collision would produce a force on the wall, but that is not the force we are seeking. We

ti

tf ∆t (b)

Figure 9-9 (a) The curve shows the magnitude of the time-varying force F(t) that acts on the ball in the collision of Fig. 9-8. The area under the curve is equal to the magni: tude of the impulse J on the ball in the collision. (b) The height of the rectangle represents the average force Favg acting on the ball over the time interval "t. The area within the rectangle is equal to the area under the curve in (a) and thus is also equal to the : magnitude of the impulse J in the collision.

228

CHAPTE R 9 CE NTE R OF MASS AN D LI N EAR M OM E NTU M

v Projectiles

Target

x

Figure 9-10 A steady stream of projectiles, with identical linear momenta, collides with a target, which is fixed in place. The average force Favg on the target is to the right and has a magnitude that depends on the rate at which the projectiles collide with the target or, equivalently, the rate at which mass collides with the target.

want the average force Favg on the wall during the bombardment — that is, the average force during a large number of collisions. In Fig. 9-10, a steady stream of projectile bodies, with identical mass m and linear momenta mv:, moves along an x axis and collides with a target body that is fixed in place. Let n be the number of projectiles that collide in a time interval "t. Because the motion is along only the x axis, we can use the components of the momenta along that axis. Thus, each projectile has initial momentum mv and undergoes a change "p in linear momentum because of the collision. The total change in linear momentum for n projectiles during interval "t is n "p. The : resulting impulse J on the target during "t is along the x axis and has the same magnitude of n "p but is in the opposite direction. We can write this relation in component form as (9-36)

J ! %n "p,

where the minus sign indicates that J and "p have opposite directions. Average Force. By rearranging Eq. 9-35 and substituting Eq. 9-36, we find the average force Favg acting on the target during the collisions: F avg !

J n n !% "p ! % m "v. "t "t "t

(9-37)

This equation gives us Favg in terms of n/"t, the rate at which the projectiles collide with the target, and "v, the change in the velocity of those projectiles. Velocity Change. If the projectiles stop upon impact, then in Eq. 9-37 we can substitute, for "v, "v ! vf % vi ! 0 % v ! %v,

(9-38)

where vi (! v) and vf (! 0) are the velocities before and after the collision, respectively. If, instead, the projectiles bounce (rebound) directly backward from the target with no change in speed, then vf ! %v and we can substitute (9-39)

"v ! vf % vi ! %v % v ! %2v.

In time interval "t, an amount of mass "m ! nm collides with the target. With this result, we can rewrite Eq. 9-37 as F avg ! %

"m "v. "t

(9-40)

This equation gives the average force Favg in terms of "m/"t, the rate at which mass collides with the target. Here again we can substitute for "v from Eq. 9-38 or 9-39 depending on what the projectiles do.

Checkpoint 5 The figure shows an overhead view of a ball bouncing from a vertical wall without any : change in its speed. Consider the change "p in the ball’s linear momentum. (a) Is "px positive, negative, or zero? (b) Is "py positive, negative, or zero? (c) What is the direc: tion of "p ? y

θ θ x

229

9-4 COLLISION AN D I M PU LSE

Sample Problem 9.04 Two-dimensional impulse, race car–wall collision Race car–wall collision. Figure 9-11a is an overhead view of the path taken by a race car driver as his car collides with the racetrack wall. Just before the collision, he is traveling at speed vi ! 70 m/s along a straight line at 30# from the wall. Just after the collision, he is traveling at speed vf ! 50 m/s along a straight line at 10# from the wall. His mass m is 80 kg. :

(a) What is the impulse J on the driver due to the collision?

Impulse: The impulse is then : J ! (%910iˆ % 3500 jˆ ) kg )m/s, which means the impulse magnitude is

J ! 2J 2x & J 2y ! 3616 kg )m/s % 3600 kg)m/s. :

The angle of J is given by

We can treat the driver as a particle-like body and thus apply : the physics of this module. However, we cannot calculate J directly from Eq. 9-30 because we do not know anything about : the force F (t) on the driver during the collision. That is, we do : not have a function of F (t) or a plot for it and thus cannot : : integrate to find J . However, we can find J from the change in : : the driver’s linear momentum p via Eq. 9-32 ( J ! : pf % : p i). Calculations: Figure 9-11b shows the driver’s momentum : pi before the collision (at angle 30# from the positive x direction) and his momentum : p f after the collision (at angle %10#). From : Eqs. 9-32 and 9-22 ( : p ! mv ), we can write :

: : : : J ! : pf % : p i ! mv f % m vi ! m(v f % v i).

(9-41)

Jy

(Answer) , Jx which a calculator evaluates as 75.4#. Recall that the physically correct result of an inverse tangent might be the displayed answer plus 180#. We can tell which is correct here : by drawing the components of J (Fig. 9-11c). We find that u is actually 75.4# & 180# ! 255.4#, which we can write as u ! %105#. (Answer) u ! tan%1

KEY IDEAS

(Answer)

(b) The collision lasts for 14 ms. What is the magnitude of the average force on the driver during the collision? KEY IDEA From Eq. 9-35 (J ! Favg "t), the magnitude Favg of the average force is the ratio of the impulse magnitude J to the duration "t of the collision.

We could evaluate the right side of this equation directly on a vector-capable calculator because we know m is 80 kg, : vf is 50 m/s at %10#, and : v i is 70 m/s at 30#. Instead, here we evaluate Eq. 9-41 in component form.

Calculations: We have J 3616 kg)m/s F avg ! ! "t 0.014 s ! 2.583 $ 105 N % 2.6 $ 105 N.

x component: Along the x axis we have

Using F ! ma with m ! 80 kg, you can show that the magnitude of the driver’s average acceleration during the collision is about 3.22 $ 10 3 m/s 2 ! 329g, which is fatal.

Jx ! m(vfx % vix) ! (80 kg)[(50 m/s) cos(%10#) % (70 m/s) cos 30#] ! %910 kg )m/s. y component: Along the y axis, Jy ! m(vfy % viy) ! (80 kg)[(50 m/s) sin(%10#) % (70 m/s) sin 30#] ! %3495 kg )m/s % %3500 kg )m/s.

Figure 9-11 (a) Overhead view of the path taken by a race car and its driver as the car slams into the racetrack wall. (b) The initial momentum : p i and final momentum : pf of the driver. (c) The : impulse J on the driver during the collision.

Wall 30°

Path

Surviving: Mechanical engineers attempt to reduce the chances of a fatality by designing and building racetrack walls with more “give,” so that a collision lasts longer. For example, if the collision here lasted 10 times longer and the other data remained the same, the magnitudes of the average force and average acceleration would be 10 times less and probably survivable.

The collision changes the y momentum.

y

pi

30°

x

pf

10°

The impulse on the car is equal to the change in the momentum.

y Jx x

x –105°

10° J

(a)

(Answer)

(b)

Additional examples, video, and practice available at WileyPLUS

Jy (c)

230

CHAPTE R 9 CE NTE R OF MASS AN D LI N EAR M OM E NTU M

9-5 CONSERVATION OF LINEAR MOMENTUM Learning Objectives After reading this module, you should be able to . . .

9.26 For an isolated system of particles, apply the conservation of linear momenta to relate the initial momenta of the particles to their momenta at a later instant.

9.27 Identify that the conservation of linear momentum can be done along an individual axis by using components along that axis, provided that there is no net external force component along that axis.

Key Ideas ● If a system is closed and isolated so that no net external

● This conservation of linear momentum can also be written

force acts on it, then the linear momentum P must be constant even if there are internal changes:

in terms of the system’s initial momentum and its momentum at some later instant:

:

:

P ! constant

:

:

Pi ! Pf

(closed, isolated system).

(closed, isolated system),

Conservation of Linear Momentum :

:

Suppose that the net external force F net (and thus the net impulse J ) acting on a system of particles is zero (the system is isolated) and that no particles leave or : enter the system (the system is closed). Putting F net ! 0 in Eq. 9-27 then yields : dP /dt ! 0, which means that :

P ! constant

(9-42)

(closed, isolated system).

In words, :

If no net external force acts on a system of particles, the total linear momentum P of the system cannot change.

This result is called the law of conservation of linear momentum and is an extremely powerful tool in solving problems. In the homework we usually write the law as :

:

P i ! Pf

(9-43)

(closed, isolated system).

In words, this equation says that, for a closed, isolated system, linear momentum ! #total at some initial time t $ # i

total linear momentum at some later time tf .

$

Caution: Momentum should not be confused with energy. In the sample problems of this module, momentum is conserved but energy is definitely not. Equations 9-42 and 9-43 are vector equations and, as such, each is equivalent to three equations corresponding to the conservation of linear momentum in three mutually perpendicular directions as in, say, an xyz coordinate system. Depending on the forces acting on a system, linear momentum might be conserved in one or two directions but not in all directions. However, If the component of the net external force on a closed system is zero along an axis, then the component of the linear momentum of the system along that axis cannot change.

In a homework problem, how can you know if linear momentum can be conserved along, say, an x axis? Check the force components along that axis. If the net of any such components is zero, then the conservation applies. As an example, suppose that you toss a grapefruit across a room. During its flight, the only external force act: ing on the grapefruit (which we take as the system) is the gravitational force F g, which is directed vertically downward. Thus, the vertical component of the linear

231

9-5 CONSE RVATION OF LI N EAR M OM E NTU M

momentum of the grapefruit changes, but since no horizontal external force acts on the grapefruit, the horizontal component of the linear momentum cannot change. Note that we focus on the external forces acting on a closed system. Although internal forces can change the linear momentum of portions of the system, they cannot change the total linear momentum of the entire system. For example, there are plenty of forces acting between the organs of your body, but they do not propel you across the room (thankfully). The sample problems in this module involve explosions that are either onedimensional (meaning that the motions before and after the explosion are along a single axis) or two-dimensional (meaning that they are in a plane containing two axes). In the following modules we consider collisions.

Checkpoint 6 An initially stationary device lying on a frictionless floor explodes into two pieces, which then slide across the floor, one of them in the positive x direction. (a) What is the sum of the momenta of the two pieces after the explosion? (b) Can the second piece move at an angle to the x axis? (c) What is the direction of the momentum of the second piece?

Sample Problem 9.05 One-dimensional explosion, relative velocity, space hauler One-dimensional explosion: Figure 9-12a shows a space hauler and cargo module, of total mass M, traveling along an x axis in deep space. They have an initial velocity v:i of magnitude 2100 km/h relative to the Sun. With a small explosion, the hauler ejects the cargo module, of mass 0.20M (Fig. 9-12b). The hauler then travels 500 km/h faster than the module along the x axis; that is, the relative speed vrel between the hauler and the module is 500 km/h.What then is the velocity v:HS of the hauler relative to the Sun? KEY IDEA Because the hauler – module system is closed and isolated, its total linear momentum is conserved; that is, :

:

Pi ! Pf ,

vi

vHS

vMS

Hauler

0.20M

Cargo module

0.80M

x (a)

x (b)

Figure 9-12 (a) A space hauler, with a cargo module, moving at initial velocity : v i . (b) The hauler has ejected the cargo module. Now the velocities relative to the Sun are : v HS for the v MS for the module and : hauler.

(9-44)

where the subscripts i and f refer to values before and after the ejection, respectively. (We need to be careful here: Although the momentum of the system does not change, the momenta of the hauler and module certainly do.) Calculations: Because the motion is along a single axis, we can write momenta and velocities in terms of their x components, using a sign to indicate direction. Before the ejection, we have Pi ! Mvi.

The explosive separation can change the momentum of the parts but not the momentum of the system.

(9-45)

Let vMS be the velocity of the ejected module relative to the Sun. The total linear momentum of the system after the ejection is then Pf ! (0.20M)vMS & (0.80M)vHS, (9-46) where the first term on the right is the linear momentum of the module and the second term is that of the hauler.

We can relate the vMS to the known velocities with

#

$ #

$ #

$

velocity of velocity of velocity of hauler relative ! hauler relative & module relative . to Sun to module to Sun

In symbols, this gives us vHS ! vrel & vMS (9-47) or vMS ! vHS % vrel. Substituting this expression for vMS into Eq. 9-46, and then substituting Eqs. 9-45 and 9-46 into Eq. 9-44, we find Mvi ! 0.20M(vHS % vrel) & 0.80MvHS, which gives us vHS ! vi & 0.20vrel, or vHS ! 2100 km/h & (0.20)(500 km/h) ! 2200 km/h. (Answer)

Additional examples, video, and practice available at WileyPLUS

232

CHAPTE R 9 CE NTE R OF MASS AN D LI N EAR M OM E NTU M

Sample Problem 9.06 Two-dimensional explosion, momentum, coconut Two-dimensional explosion: A firecracker placed inside a coconut of mass M, initially at rest on a frictionless floor, blows the coconut into three pieces that slide across the floor. An overhead view is shown in Fig. 9-13a. Piece C, with mass 0.30M, has final speed vfC ! 5.0 m/s.

Pix ! Pfx,

(9-49)

where Pix ! 0 because the coconut is initially at rest. To get Pfx, we find the x components of the final momenta, using the fact that piece A must have a mass of 0.50M (! M % 0.20M % 0.30M):

(a) What is the speed of piece B, with mass 0.20M? KEY IDEA First we need to see whether linear momentum is conserved. We note that (1) the coconut and its pieces form a closed system, (2) the explosion forces are internal to that system, and (3) no net external force acts on the system. Therefore, the linear momentum of the system is conserved. (We need to be careful here: Although the momentum of the system does not change, the momenta of the pieces certainly do.) Calculations: To get started, we superimpose an xy coordinate system as shown in Fig. 9-13b, with the negative direction of the x axis coinciding with the direction of : vfA. The x axis is at 80# with the direction of : vf C and 50# with the direction of : vf B. Linear momentum is conserved separately along each axis. Let’s use the y axis and write Piy ! Pfy,

Calculations: Linear momentum is also conserved along the x axis because there is no net external force acting on the coconut and pieces along that axis. Thus we have

pfA,x ! %0.50MvfA, pfB,x ! 0.20MvfB,x ! 0.20MvfB cos 50#, pfC,x ! 0.30MvfC,x ! 0.30MvfC cos 80#. Equation 9-49 for the conservation of momentum along the x axis can now be written as Pix ! Pfx ! pfA,x & pfB,x & pfC,x. Then, with vfC ! 5.0 m/s and vfB ! 9.64 m/s, we have 0 ! %0.50MvfA & 0.20M(9.64 m/s) cos 50# & 0.30M(5.0 m/s) cos 80#, from which we find vfA ! 3.0 m/s.

(Answer)

(9-48) The explosive separation can change the momentum of the parts but not the momentum of the system.

where subscript i refers to the initial value (before the ex: plosion), and subscript y refers to the y component of Pi : or Pf . The component Piy of the initial linear momentum is zero, because the coconut is initially at rest. To get an expression for Pfy, we find the y component of the final linear momentum of each piece, using the y-component version of Eq. 9-22 ( py ! mvy):

100°

vfC

vfA C

A

pfA,y ! 0,

130°

B

pfB,y ! %0.20MvfB,y ! %0.20MvfB sin 50#, pfC,y ! 0.30MvfC,y ! 0.30MvfC sin 80#.

vfB (a)

(Note that pfA,y ! 0 because of our nice choice of axes.) Equation 9-48 can now be written as Piy ! Pfy ! pfA,y & pfB,y & pfC,y. 0 ! 0 % 0.20MvfB sin 50# & (0.30M)(5.0 m/s) sin 80#, from which we find (b) What is the speed of piece A?

vfC

vfA

Then, with vfC ! 5.0 m/s, we have

vfB ! 9.64 m/s % 9.6 m/s.

y

(Answer)

Figure 9-13 Three pieces of an exploded coconut move off in three directions along a frictionless floor. (a) An overhead view of the event. (b) The same with a two-dimensional axis system imposed.

Additional examples, video, and practice available at WileyPLUS

C

A

80°

B 50° vfB (b)

x

9-6 M OM E NTU M AN D KI N ETIC E N E RGY I N COLLISIONS

233

9-6 MOMENTUM AND KINETIC ENERGY IN COLLISIONS Learning Objectives After reading this module, you should be able to . . .

9.28 Distinguish between elastic collisions, inelastic collisions, and completely inelastic collisions. 9.29 Identify a one-dimensional collision as one where the objects move along a single axis, both before and after the collision.

9.30 Apply the conservation of momentum for an isolated one-dimensional collision to relate the initial momenta of the objects to their momenta after the collision. 9.31 Identify that in an isolated system, the momentum and velocity of the center of mass are not changed even if the objects collide.

Key Ideas ● In an inelastic collision of two bodies, the kinetic energy of

the two-body system is not conserved. If the system is closed and isolated, the total linear momentum of the system must be conserved, which we can write in vector form as p 2i ! : p 1f & : p 2f , p 1i & :

:

where subscripts i and f refer to values just before and just after the collision, respectively. ● If the motion of the bodies is along a single axis, the collision

is one-dimensional and we can write the equation in terms of

velocity components along that axis: m1v1i & m2v2i ! m1v1f & m2v2f . ● If the bodies stick together, the collision is a completely

inelastic collision and the bodies have the same final velocity V (because they are stuck together). ● The center of mass of a closed, isolated system of two col-

liding bodies is not affected by a collision. In particular, the velocity : v com of the center of mass cannot be changed by the collision.

Momentum and Kinetic Energy in Collisions In Module 9-4, we considered the collision of two particle-like bodies but focused on only one of the bodies at a time. For the next several modules we switch our focus to the system itself, with the assumption that the system is closed and isolated. In Module 9-5, we discussed a rule about such a system: The total linear : momentum P of the system cannot change because there is no net external force to change it. This is a very powerful rule because it can allow us to determine the results of a collision without knowing the details of the collision (such as how much damage is done). We shall also be interested in the total kinetic energy of a system of two colliding bodies. If that total happens to be unchanged by the collision, then the kinetic energy of the system is conserved (it is the same before and after the collision). Such a collision is called an elastic collision. In everyday collisions of common bodies, such as two cars or a ball and a bat, some energy is always transferred from kinetic energy to other forms of energy, such as thermal energy or energy of sound. Thus, the kinetic energy of the system is not conserved. Such a collision is called an inelastic collision. However, in some situations, we can approximate a collision of common bodies as elastic. Suppose that you drop a Superball onto a hard floor. If the collision between the ball and floor (or Earth) were elastic, the ball would lose no kinetic energy because of the collision and would rebound to its original height. However, the actual rebound height is somewhat short, showing that at least some kinetic energy is lost in the collision and thus that the collision is somewhat inelastic. Still, we might choose to neglect that small loss of kinetic energy to approximate the collision as elastic. The inelastic collision of two bodies always involves a loss in the kinetic energy of the system. The greatest loss occurs if the bodies stick together, in which case the collision is called a completely inelastic collision. The collision of a baseball and a bat is inelastic. However, the collision of a wet putty ball and a bat is completely inelastic because the putty sticks to the bat.

234

CHAPTE R 9 CE NTE R OF MASS AN D LI N EAR M OM E NTU M

Inelastic Collisions in One Dimension

Here is the generic setup for an inelastic collision. Body 1 v1i

Before

One-Dimensional Inelastic Collision

Body 2 v2i x

m2

m1 v1f

After m1

Figure 9-14 shows two bodies just before and just after they have a onedimensional collision. The velocities before the collision (subscript i) and after the collision (subscript f ) are indicated. The two bodies form our system, which is closed and isolated. We can write the law of conservation of linear momentum for this two-body system as

v2f m2

#

x

Figure 9-14 Bodies 1 and 2 move along an x axis, before and after they have an inelastic collision.

:

:

$ #

total momentum Pf total momentum Pi ! , before the collision after the collision

$

which we can symbolize as : : : : p 1i & p2i ! p1f & p2f

(9-50)

(conservation of linear momentum).

Because the motion is one-dimensional, we can drop the overhead arrows for vectors and use only components along the axis, indicating direction with a sign. Thus, from p ! mv, we can rewrite Eq. 9-50 as m1v1i & m2v2i ! m1v1f & m2v2f.

(9-51)

If we know values for, say, the masses, the initial velocities, and one of the final velocities, we can find the other final velocity with Eq. 9-51.

One-Dimensional Completely Inelastic Collision Figure 9-15 shows two bodies before and after they have a completely inelastic collision (meaning they stick together). The body with mass m2 happens to be initially at rest (v2i ! 0). We can refer to that body as the target and to the incoming body as the projectile. After the collision, the stuck-together bodies move with velocity V. For this situation, we can rewrite Eq. 9-51 as m1v1i ! (m1 & m2)V or

V!

(9-52)

m1 v1i . m1 & m2

(9-53)

If we know values for, say, the masses and the initial velocity v1i of the projectile, we can find the final velocity V with Eq. 9-53. Note that V must be less than v1i because the mass ratio m1/(m1 & m2) must be less than unity.

Velocity of the Center of Mass In a closed, isolated system, the velocity : v com of the center of mass of the system cannot be changed by a collision because, with the system isolated, there is no net external force to change it. To get an expression for : v com, let us return to the In a completely inelastic collision, the bodies stick together. Before

Figure 9-15 A completely inelastic collision between two bodies. Before the collision, the body with mass m2 is at rest and the body with mass m1 moves directly toward it. After the collision, the stuck: together bodies move with the same velocity V.

v1i

m1 Projectile After

v2i = 0

x

m2 Target V x m1 + m2

9-6 M OM E NTU M AN D KI N ETIC E N E RGY I N COLLISIONS

The com of the two bodies is between them and moves at a constant velocity.

v1i

Here is the incoming projectile.

vcom

m1

x v2i = 0 m2

Here is the stationary target.

Collision!

Figure 9-16 Some freeze-frames of the two-body system in Fig. 9-15, which undergoes a completely inelastic collision. The system’s center of mass is shown in each freeze-frame. The velocity : v com of the center of mass is unaffected by the collision. Because the bodies stick : together after the collision, their common velocity V : must be equal to v com.

m1 + m2

The com moves at the same velocity even after the bodies stick together.

two-body system and one-dimensional collision of Fig. 9-14. From Eq. 9-25 : : (P ! M : v com), we can relate : v com to the total linear momentum P of that two-body system by writing :

: P ! M: v com ! (m1 & m2)v com.

(9-54)

:

The total linear momentum P is conserved during the collision; so it is given by either side of Eq. 9-50. Let us use the left side to write :

P!: p 1i & : p 2i.

(9-55)

:

Substituting this expression for P in Eq. 9-54 and solving for : v com give us :

: v com

!

: p 2i P p 1i & : ! . m1 & m2 m1 & m2

(9-56)

The right side of this equation is a constant, and : v com has that same constant value before and after the collision. For example, Fig. 9-16 shows, in a series of freeze-frames, the motion of the center of mass for the completely inelastic collision of Fig. 9-15. Body 2 is the target, and its initial linear momentum in Eq. 9-56 is : p 2i ! m2: v 2i ! 0. Body 1 is the projectile, and its initial linear momentum in Eq. 9-56 is : p 1i ! m1: v 1i. Note that as the series of freeze-frames progresses to and then beyond the collision, the center of mass moves at a constant velocity to the right. After the collision, the common final speed V of the bodies is equal to : v com because then the center of mass travels with the stuck-together bodies.

Checkpoint 7 Body 1 and body 2 are in a completely inelastic one-dimensional collision.What is their final momentum if their initial momenta are, respectively, (a) 10 kg )m/s and 0; (b) 10 kg )m/s and 4 kg )m/s; (c) 10 kg )m/s and %4 kg )m/s?

V = vcom

235

236

CHAPTE R 9 CE NTE R OF MASS AN D LI N EAR M OM E NTU M

Sample Problem 9.07 Conservation of momentum, ballistic pendulum Here is an example of a common technique in physics. We have a demonstration that cannot be worked out as a whole (we don’t have a workable equation for it). So, we break it up into steps that can be worked separately (we have equations for them). The ballistic pendulum was used to measure the speeds of bullets before electronic timing devices were developed. The version shown in Fig. 9-17 consists of a large block of wood of mass M ! 5.4 kg, hanging from two long cords. A bullet of mass m ! 9.5 g is fired into the block, coming quickly to rest. The block & bullet then swing upward, their center of mass rising a vertical distance h ! 6.3 cm before the pendulum comes momentarily to rest at the end of its arc. What is the speed of the bullet just prior to the collision?

system is conserved: energy energy $. # mechanical $ ! #mechanical at bottom at top

(This mechanical energy is not changed by the force of the cords on the block, because that force is always directed perpendicular to the block’s direction of travel.) Let’s take the block’s initial level as our reference level of zero gravitational potential energy. Then conservation of mechanical energy means that the system’s kinetic energy at the start of the swing must equal its gravitational potential energy at the highest point of the swing. Because the speed of the bullet and block at the start of the swing is the speed V immediately after the collision, we may write this conservation as 1 2 (m

KEY IDEAS We can see that the bullet’s speed v must determine the rise height h. However, we cannot use the conservation of mechanical energy to relate these two quantities because surely energy is transferred from mechanical energy to other forms (such as thermal energy and energy to break apart the wood) as the bullet penetrates the block. Nevertheless, we can split this complicated motion into two steps that we can separately analyze: (1) the bullet–block collision and (2) the bullet–block rise, during which mechanical energy is conserved. Reasoning step 1: Because the collision within the bullet – block system is so brief, we can make two important assumptions: (1) During the collision, the gravitational force on the block and the force on the block from the cords are still balanced. Thus, during the collision, the net external impulse on the bullet – block system is zero. Therefore, the system is isolated and its total linear momentum is conserved: total momentum total momentum ! . #before the collision$ #after the collision$

(9-59)

& M )V 2 ! (m & M )gh.

(9-60)

Combining steps: Substituting for V from Eq. 9-58 leads to v! !

m&M 22gh m

(9-61)

kg & 5.4 kg # 0.00950.0095 $ 2(2)(9.8 m/s )(0.063 m) kg 2

! 630 m/s.

(Answer)

The ballistic pendulum is a kind of “transformer,” exchanging the high speed of a light object (the bullet) for the low — and thus more easily measurable — speed of a massive object (the block). There are two events here. The bullet collides with the block. Then the bullet–block system swings upward by height h.

(9-57)

(2) The collision is one-dimensional in the sense that the direction of the bullet and block just after the collision is in the bullet’s original direction of motion. Because the collision is one-dimensional, the block is initially at rest, and the bullet sticks in the block, we use Eq. 9-53 to express the conservation of linear momentum. Replacing the symbols there with the corresponding symbols here, we have V!

m v. m&M

(9-58)

Reasoning step 2: As the bullet and block now swing up together, the mechanical energy of the bullet – block – Earth

Figure 9-17 A ballistic pendulum, used to measure the speeds of bullets.

Additional examples, video, and practice available at WileyPLUS

M

m v

h

237

9-7 E L ASTIC COLLISIONS I N ON E DI M E NSION

9-7 ELASTIC COLLISIONS IN ONE DIMENSION Learning Objectives After reading this module, you should be able to . . .

9.32 For isolated elastic collisions in one dimension, apply the conservation laws for both the total energy and the net momentum of the colliding bodies to relate the initial values to the values after the collision.

9.33 For a projectile hitting a stationary target, identify the resulting motion for the three general cases: equal masses, target more massive than projectile, projectile more massive than target.

Key Idea An elastic collision is a special type of collision in which the kinetic energy of a system of colliding bodies is conserved. If the system is closed and isolated, its linear momentum is also conserved. For a one-dimensional collision in which body 2 is a target and body 1 is an incoming projectile, conservation of kinetic energy and linear momentum



yield the following expressions for the velocities immediately after the collision: m1 % m2 v v1f ! m1 & m2 1i 2m1 v1i. and v2f ! m1 & m2

Elastic Collisions in One Dimension As we discussed in Module 9-6, everyday collisions are inelastic but we can approximate some of them as being elastic; that is, we can approximate that the total kinetic energy of the colliding bodies is conserved and is not transferred to other forms of energy: total kinetic energy kinetic energy ! . #total before the collision$ # after the collision $

(9-62)

This means: In an elastic collision, the kinetic energy of each colliding body may change, but the total kinetic energy of the system does not change.

For example, the collision of a cue ball with an object ball in a game of pool can be approximated as being an elastic collision. If the collision is head-on (the cue ball heads directly toward the object ball), the kinetic energy of the cue ball can be transferred almost entirely to the object ball. (Still, the collision transfers some of the energy to the sound you hear.)

Stationary Target Figure 9-18 shows two bodies before and after they have a one-dimensional collision, like a head-on collision between pool balls. A projectile body of mass m1 and initial velocity v1i moves toward a target body of mass m2 that is initially at rest (v2i ! 0). Let’s assume that this two-body system is closed and isolated. Then the net linear momentum of the system is conserved, and from Eq. 9-51 we can write that conservation as m1v1i ! m1v1f & m2v2f (linear momentum). (9-63) If the collision is also elastic, then the total kinetic energy is conserved and we can write that conservation as 1 2 2 m1v1i

!

1 2 2 m1v1f

&

1 2 2 m2v2f

(kinetic energy).

(9-64)

In each of these equations, the subscript i identifies the initial velocities and the subscript f the final velocities of the bodies. If we know the masses of the bodies and if we also know v1i , the initial velocity of body 1, the only unknown quantities are v1f and v2f , the final velocities of the two bodies. With two equations at our disposal, we should be able to find these two unknowns.

Before m1 Projectile

v1i

Here is the generic setup for an elastic collision with a stationary target. v2i = 0

x

m2 Target v1f

After m1

v2f m2

x

Figure 9-18 Body 1 moves along an x axis before having an elastic collision with body 2, which is initially at rest. Both bodies move along that axis after the collision.

238

CHAPTE R 9 CE NTE R OF MASS AN D LI N EAR M OM E NTU M

To do so, we rewrite Eq. 9-63 as and Eq. 9-64 as*

m1(v1i % v1f) ! m2v2f

(9-65)

m1(v1i % v1f)(v1i & v1f) ! m2v22f .

(9-66)

After dividing Eq. 9-66 by Eq. 9-65 and doing some more algebra, we obtain

and

v1f !

m1 % m2 v1i m1 & m2

(9-67)

v2f !

2m1 v1i . m1 & m2

(9-68)

Note that v2f is always positive (the initially stationary target body with mass m2 always moves forward). From Eq. 9-67 we see that v1f may be of either sign (the projectile body with mass m1 moves forward if m1 - m2 but rebounds if m1 + m2). Let us look at a few special situations. 1. Equal masses If m1 ! m2, Eqs. 9-67 and 9-68 reduce to v1f ! 0

and v2f ! v1i ,

which we might call a pool player’s result. It predicts that after a head-on collision of bodies with equal masses, body 1 (initially moving) stops dead in its tracks and body 2 (initially at rest) takes off with the initial speed of body 1. In head-on collisions, bodies of equal mass simply exchange velocities. This is true even if body 2 is not initially at rest. 2. A massive target In Fig. 9-18, a massive target means that m2 . m1. For example, we might fire a golf ball at a stationary cannonball. Equations 9-67 and 9-68 then reduce to 2m1 v1i . (9-69) v1f % %v1i and v2f % m2

#

$

This tells us that body 1 (the golf ball) simply bounces back along its incoming path, its speed essentially unchanged. Initially stationary body 2 (the cannonball) moves forward at a low speed, because the quantity in parentheses in Eq. 9-69 is much less than unity. All this is what we should expect. 3. A massive projectile This is the opposite case; that is, m1 . m2. This time, we fire a cannonball at a stationary golf ball. Equations 9-67 and 9-68 reduce to v1f % v1i and

v2f % 2v1i .

(9-70)

Equation 9-70 tells us that body 1 (the cannonball) simply keeps on going, scarcely slowed by the collision. Body 2 (the golf ball) charges ahead at twice the speed of the cannonball. Why twice the speed? Recall the collision described by Eq. 9-69, in which the velocity of the incident light body (the golf ball) changed from &v to %v, a velocity change of 2v. The same change in velocity (but now from zero to 2v) occurs in this example also.

Moving Target Now that we have examined the elastic collision of a projectile and a stationary target, let us examine the situation in which both bodies are moving before they undergo an elastic collision. For the situation of Fig. 9-19, the conservation of linear momentum is written as m1v1i & m2v2i ! m1v1f & m2v2f ,

(9-71)

*In this step, we use the identity a2 % b2 ! (a % b)(a & b). It reduces the amount of algebra needed to solve the simultaneous equations Eqs. 9-65 and 9-66.

239

9-7 E L ASTIC COLLISIONS I N ON E DI M E NSION

and the conservation of kinetic energy is written as 1 2 2 m1v1i

&

1 2 2 m2v2i

!

1 2 2 m1v1f

&

1 2 2 m2v2f .

(9-72)

To solve these simultaneous equations for v1f and v2f , we first rewrite Eq. 9-71 as m1(v1i % v1f) ! %m2(v2i % v2f),

and Eq. 9-72 as

v1i

(9-73)

v2i x

m1(v1i % v1f)(v1i & v1f) ! %m2(v2i % v2f)(v2i & v2f).

(9-74)

After dividing Eq. 9-74 by Eq. 9-73 and doing some more algebra, we obtain

and

Here is the generic setup for an elastic collision with a moving target.

v1f !

m1 % m2 2m2 v1i & v2i m1 & m2 m1 & m2

(9-75)

v2f !

2m1 m2 % m1 v1i & v2i . m1 & m2 m1 & m2

(9-76)

m1

m2

Figure 9-19 Two bodies headed for a onedimensional elastic collision.

Note that the assignment of subscripts 1 and 2 to the bodies is arbitrary. If we exchange those subscripts in Fig. 9-19 and in Eqs. 9-75 and 9-76, we end up with the same set of equations. Note also that if we set v2i ! 0, body 2 becomes a stationary target as in Fig. 9-18, and Eqs. 9-75 and 9-76 reduce to Eqs. 9-67 and 9-68, respectively.

Checkpoint 8 What is the final linear momentum of the target in Fig. 9-18 if the initial linear momentum of the projectile is 6 kg )m/s and the final linear momentum of the projectile is (a) 2 kg )m/s and (b) %2 kg )m/s? (c) What is the final kinetic energy of the target if the initial and final kinetic energies of the projectile are, respectively, 5 J and 2 J?

Sample Problem 9.08 Chain reaction of elastic collisions In Fig. 9-20a, block 1 approaches a line of two stationary blocks with a velocity of v1i ! 10 m/s. It collides with block 2, which then collides with block 3, which has mass m3 ! 6.0 kg. After the second collision, block 2 is again stationary and block 3 has velocity v3f ! 5.0 m/s (Fig. 9-20b).Assume that the collisions are elastic. What are the masses of blocks 1 and 2? What is the final velocity v1f of block 1? KEY IDEAS Because we assume that the collisions are elastic, we are to conserve mechanical energy (thus energy losses to sound, heating, and oscillations of the blocks are negligible). Because no external horizontal force acts on the blocks, we are to conserve linear momentum along the x axis. For these v1i (a)

m1 v1f

(b)

m2

m3

x v3f x

Figure 9-20 Block 1 collides with stationary block 2, which then collides with stationary block 3.

two reasons, we can apply Eqs. 9-67 and 9-68 to each of the collisions. Calculations: If we start with the first collision, we have too many unknowns to make any progress: we do not know the masses or the final velocities of the blocks. So, let’s start with the second collision in which block 2 stops because of its collision with block 3. Applying Eq. 9-67 to this collision, with changes in notation, we have m2 % m3 v2i, v2f ! m2 & m3 where v2i is the velocity of block 2 just before the collision and v2f is the velocity just afterward. Substituting v2f ! 0 (block 2 stops) and then m3 ! 6.0 kg gives us m2 ! m3 ! 6.00 kg.

(Answer)

With similar notation changes, we can rewrite Eq. 9-68 for the second collision as 2m2 v3f ! v , m2 & m3 2i where v3f is the final velocity of block 3. Substituting m2 ! m3 and the given v3f ! 5.0 m/s, we find v2i ! v3f ! 5.0 m/s.

240

CHAPTE R 9 CE NTE R OF MASS AN D LI N EAR M OM E NTU M

Next, let’s reconsider the first collision, but we have to be careful with the notation for block 2: its velocity v2f just after the first collision is the same as its velocity v2i (! 5.0 m/s) just before the second collision. Applying Eq. 9-68 to the first collision and using the given v1i ! 10 m/s, we have 2m1 v1i, m1 & m2 2m1 5.0 m/s ! (10 m/s), m1 & m2 v2f !

which leads to m1 ! 13m2 ! 13(6.0 kg) ! 2.0 kg.

(Answer)

Finally, applying Eq. 9-67 to the first collision with this result and the given v1i, we write v1f ! !

m1 % m2 v1i, m1 & m2 1 3 m2 1 3 m2

% m2 (10 m/s) ! %5.0 m/s. & m2

(Answer)

Additional examples, video, and practice available at WileyPLUS

9-8 COLLISIONS IN TWO DIMENSIONS Learning Objectives After reading this module, you should be able to . . .

9.34 For an isolated system in which a two-dimensional collision occurs, apply the conservation of momentum along each axis of a coordinate system to relate the momentum components along an axis before the collision to the momentum components along the same axis after the collision.

9.35 For an isolated system in which a two-dimensional elastic collision occurs, (a) apply the conservation of momentum along each axis of a coordinate system to relate the momentum components along an axis before the collision to the momentum components along the same axis after the collision and (b) apply the conservation of total kinetic energy to relate the kinetic energies before and after the collision.

Key Idea ● If two bodies collide and their motion is not along a single axis

(the collision is not head-on), the collision is two-dimensional. If the two-body system is closed and isolated, the law of conservation of momentum applies to the collision and can be written as :

:

:

In component form, the law gives two equations that describe the collision (one equation for each of the two dimensions). If the collision is also elastic (a special case), the conservation of kinetic energy during the collision gives a third equation:

:

K1i & K2i ! K1f & K2f .

P1i & P2i ! P1f & P2f .

Collisions in Two Dimensions A glancing collision that conserves both momentum and kinetic energy.

y v2f

:

m2 m1

v1i

When two bodies collide, the impulse between them determines the directions in which they then travel. In particular, when the collision is not head-on, the bodies do not end up traveling along their initial axis. For such two-dimensional collisions in a closed, isolated system, the total linear momentum must still be conserved:

θ2

:

:

:

P1i & P2i ! P1f & P2f . x

θ1

If the collision is also elastic (a special case), then the total kinetic energy is also conserved: K1i & K2i ! K1f & K2f .

v1f

Figure 9-21 An elastic collision between two bodies in which the collision is not headon. The body with mass m2 (the target) is initially at rest.

(9-77)

(9-78)

Equation 9-77 is often more useful for analyzing a two-dimensional collision if we write it in terms of components on an xy coordinate system. For example, Fig. 9-21 shows a glancing collision (it is not head-on) between a projectile body and a target body initially at rest. The impulses between the bodies have sent the bodies off at angles u1 and u2 to the x axis, along which the projectile initially traveled. In this situ-

9-9 SYSTE M S WITH VARYI NG MASS: A ROCKET

241

ation we would rewrite Eq. 9-77 for components along the x axis as m1v1i ! m1v1f cos u1 & m2v2f cos u2,

(9-79)

0 ! %m1v1f sin u1 & m2v2f sin u2.

(9-80)

and along the y axis as We can also write Eq. 9-78 (for the special case of an elastic collision) in terms of speeds: 1 1 1 2 2 2 (kinetic energy). (9-81) 2 m1v1i ! 2 m1v1f & 2 m2v2f Equations 9-79 to 9-81 contain seven variables: two masses, m1 and m2; three speeds, v1i , v1f , and v2f ; and two angles, u1 and u 2 . If we know any four of these quantities, we can solve the three equations for the remaining three quantities.

Checkpoint 9 In Fig. 9-21, suppose that the projectile has an initial momentum of 6 kg )m/s, a final x component of momentum of 4 kg )m/s, and a final y component of momentum of %3 kg )m/s. For the target, what then are (a) the final x component of momentum and (b) the final y component of momentum?

9-9 SYSTEMS WITH VARYING MASS: A ROCKET Learning Objectives After reading this module, you should be able to . . .

9.36 Apply the first rocket equation to relate the rate at which the rocket loses mass, the speed of the exhaust products relative to the rocket, the mass of the rocket, and the acceleration of the rocket.

9.37 Apply the second rocket equation to relate the change in the rocket’s speed to the relative speed of the exhaust products and the initial and final mass of the rocket. 9.38 For a moving system undergoing a change in mass at a given rate, relate that rate to the change in momentum.

Key Ideas ● In the absence of external forces a rocket accelerates at an

instantaneous rate given by Rvrel ! Ma

(first rocket equation),

in which M is the rocket’s instantaneous mass (including unexpended fuel), R is the fuel consumption rate, and vrel is

the fuel’s exhaust speed relative to the rocket. The term Rvrel is the thrust of the rocket engine. ● For a rocket with constant R and vrel, whose speed

changes from vi to vf when its mass changes from Mi to Mf , vf % vi ! vrel ln

Systems with Varying Mass: A Rocket So far, we have assumed that the total mass of the system remains constant. Sometimes, as in a rocket, it does not. Most of the mass of a rocket on its launching pad is fuel, all of which will eventually be burned and ejected from the nozzle of the rocket engine. We handle the variation of the mass of the rocket as the rocket accelerates by applying Newton’s second law, not to the rocket alone but to the rocket and its ejected combustion products taken together. The mass of this system does not change as the rocket accelerates.

Finding the Acceleration Assume that we are at rest relative to an inertial reference frame, watching a rocket accelerate through deep space with no gravitational or atmospheric drag

Mi Mf

(second rocket equation).

242

CHAPTE R 9 CE NTE R OF MASS AN D LI N EAR M OM E NTU M

forces acting on it. For this one-dimensional motion, let M be the mass of the rocket and v its velocity at an arbitrary time t (see Fig. 9-22a). Figure 9-22b shows how things stand a time interval dt later. The rocket now has velocity v & dv and mass M & dM, where the change in mass dM is a negative quantity. The exhaust products released by the rocket during interval dt have mass %dM and velocity U relative to our inertial reference frame. Conserve Momentum. Our system consists of the rocket and the exhaust products released during interval dt. The system is closed and isolated, so the linear momentum of the system must be conserved during dt; that is,

The ejection of mass from the rocket's rear increases the rocket's speed. System boundary M

v

P i ! Pf , x (a )

where the subscripts i and f indicate the values at the beginning and end of time interval dt. We can rewrite Eq. 9-82 as Mv ! %dM U & (M & dM)(v & dv),

System boundary M + dM

–dM

v + dv

U

x (b )

Figure 9-22 (a) An accelerating rocket of mass M at time t, as seen from an inertial reference frame. (b) The same but at time t & dt. The exhaust products released during interval dt are shown.

(9-82)

(9-83)

where the first term on the right is the linear momentum of the exhaust products released during interval dt and the second term is the linear momentum of the rocket at the end of interval dt. Use Relative Speed. We can simplify Eq. 9-83 by using the relative speed vrel between the rocket and the exhaust products, which is related to the velocities relative to the frame with of rocket velocity of rocket velocity of products . ! & #velocity relative to frame $ #relative to products$ # relative to frame $ In symbols, this means (v & dv) ! vrel & U, or

U ! v & dv % vrel.

(9-84)

Substituting this result for U into Eq. 9-83 yields, with a little algebra, %dM vrel ! M dv.

(9-85)

Dividing each side by dt gives us %

dv dM v !M . dt rel dt

(9-86)

We replace dM/dt (the rate at which the rocket loses mass) by %R, where R is the (positive) mass rate of fuel consumption, and we recognize that dv/dt is the acceleration of the rocket. With these changes, Eq. 9-86 becomes Rvrel ! Ma

(first rocket equation).

(9-87)

Equation 9-87 holds for the values at any given instant. Note the left side of Eq. 9-87 has the dimensions of force (kg/s )m/s ! kg )m/s2 ! N) and depends only on design characteristics of the rocket engine — namely, the rate R at which it consumes fuel mass and the speed vrel with which that mass is ejected relative to the rocket. We call this term Rvrel the thrust of the rocket engine and represent it with T. Newton’s second law emerges if we write Eq. 9-87 as T ! Ma, in which a is the acceleration of the rocket at the time that its mass is M.

Finding the Velocity How will the velocity of a rocket change as it consumes its fuel? From Eq. 9-85 we have dv ! %vrel

dM . M

R EVI EW & SU M MARY

Integrating leads to

"

vf

vi

dv ! %vrel

"

Mf

Mi

243

dM , M

in which Mi is the initial mass of the rocket and Mf its final mass. Evaluating the integrals then gives Mi Mf

vf % vi ! vrel ln

(9-88)

(second rocket equation)

for the increase in the speed of the rocket during the change in mass from Mi to Mf . (The symbol “ln” in Eq. 9-88 means the natural logarithm.) We see here the advantage of multistage rockets, in which Mf is reduced by discarding successive stages when their fuel is depleted. An ideal rocket would reach its destination with only its payload remaining. Sample Problem 9.09 Rocket engine, thrust, acceleration In all previous examples in this chapter, the mass of a system is constant (fixed as a certain number). Here is an example of a system (a rocket) that is losing mass. A rocket whose initial mass Mi is 850 kg consumes fuel at the rate R ! 2.3 kg/s. The speed vrel of the exhaust gases relative to the rocket engine is 2800 m/s.What thrust does the rocket engine provide?

rocket’s mass. However, M decreases and a increases as fuel is consumed. Because we want the initial value of a here, we must use the intial value Mi of the mass. Calculation: We find a!

KEY IDEA Thrust T is equal to the product of the fuel consumption rate R and the relative speed vrel at which exhaust gases are expelled, as given by Eq. 9-87. Calculation: Here we find T ! Rvrel ! (2.3 kg/s)(2800 m/s) ! 6440 N % 6400 N.

(Answer)

T 6440 N ! ! 7.6 m/s2. Mi 850 kg

(Answer)

To be launched from Earth’s surface, a rocket must have an initial acceleration greater than g ! 9.8 m/s2. That is, it must be greater than the gravitational acceleration at the surface. Put another way, the thrust T of the rocket engine must exceed the initial gravitational force on the rocket, which here has the magnitude Mi g, which gives us (850 kg)(9.8 m/s2) ! 8330 N.

(b) What is the initial acceleration of the rocket? KEY IDEA We can relate the thrust T of a rocket to the magnitude a of the resulting acceleration with T ! Ma, where M is the

Because the acceleration or thrust requirement is not met (here T ! 6400 N), our rocket could not be launched from Earth’s surface by itself; it would require another, more powerful, rocket.

Additional examples, video, and practice available at WileyPLUS

Review & Summary Center of Mass The center of mass of a system of n particles is

Newton’s Second Law for a System of Particles The

defined to be the point whose coordinates are given by 1 n 1 n 1 mi xi , ycom ! mi yi , zcom ! xcom ! ' ' M i!1 M i!1 M

motion of the center of mass of any system of particles is governed by Newton’s second law for a system of particles, which is

or

: rcom

!

1 M

n

' mi :r i ,

i!1

where M is the total mass of the system.

n

' mi zi ,

i!1

:

(9-5)

Fnet ! M : a com.

(9-8)

Here Fnet is the net force of all the external forces acting on the system, M is the total mass of the system, and : a com is the acceleration of the system’s center of mass.

:

(9-14)

244

CHAPTE R 9 CE NTE R OF MASS AN D LI N EAR M OM E NTU M

Linear Momentum and Newton’s Second Law For a single particle, we define a quantity : p called its linear momentum as : , p ! mv

and can write Newton’s second law in terms of this momentum: :

p 1i & : p 2i ! : p 1f & : p 2f ,

:

(9-22)

:

Fnet !

must be conserved (it is a constant), which we can write in vector form as

d: p . dt

(9-23)

For a system of particles these relations become

where subscripts i and f refer to values just before and just after the collision, respectively. If the motion of the bodies is along a single axis, the collision is one-dimensional and we can write Eq. 9-50 in terms of velocity components along that axis:

:

:

:

P ! Mv:com and

Fnet !

dP . dt

(9-25, 9-27)

Collision and Impulse

Applying Newton’s second law in momentum form to a particle-like body involved in a collision leads to the impulse – linear momentum theorem: :

: pf % : p i ! "p ! J,

:

(9-31, 9-32)

: where : pf % : is the change in the body’s linear momenp i ! "p : : tum, and J is the impulse due to the force F(t) exerted on the body by the other body in the collision: :

J !

"

tf

ti

:

(9-30)

F(t) dt. :

If Favg is the average magnitude of F(t) during the collision and "t is the duration of the collision, then for one-dimensional motion (9-35)

J ! Favg "t.

When a steady stream of bodies, each with mass m and speed v, collides with a body whose position is fixed, the average force on the fixed body is n n (9-37) "p ! % m "v, Favg ! % "t "t where n/"t is the rate at which the bodies collide with the fixed body, and "v is the change in velocity of each colliding body. This average force can also be written as Favg ! %

"m "v, "t

(9-40)

where "m/"t is the rate at which mass collides with the fixed body. In Eqs. 9-37 and 9-40, "v ! %v if the bodies stop upon impact and "v ! %2v if they bounce directly backward with no change in their speed.

Conservation of Linear Momentum

If a system is isolated : so that no net external force acts on it, the linear momentum P of the system remains constant: :

P ! constant

(closed, isolated system).

(9-42)

This can also be written as :

:

Pi ! Pf

(closed, isolated system),

(9-43)

(9-50)

m1v1i & m2v2i ! m1v1f & m2v2f .

(9-51)

If the bodies stick together, the collision is a completely inelastic collision and the bodies have the same final velocity V (because they are stuck together).

Motion of the Center of Mass

The center of mass of a closed, isolated system of two colliding bodies is not affected by a collision. In particular, the velocity : v com of the center of mass cannot be changed by the collision.

Elastic Collisions in One Dimension

An elastic collision is a special type of collision in which the kinetic energy of a system of colliding bodies is conserved. If the system is closed and isolated, its linear momentum is also conserved. For a onedimensional collision in which body 2 is a target and body 1 is an incoming projectile, conservation of kinetic energy and linear momentum yield the following expressions for the velocities immediately after the collision:

and

v1f !

m1 % m2 v1i m1 & m2

(9-67)

v2f !

2m1 v1i. m1 & m2

(9-68)

Collisions in Two Dimensions

If two bodies collide and their motion is not along a single axis (the collision is not head-on), the collision is two-dimensional. If the two-body system is closed and isolated, the law of conservation of momentum applies to the collision and can be written as :

:

:

:

P1i & P2i ! P1f & P2f .

(9-77)

In component form, the law gives two equations that describe the collision (one equation for each of the two dimensions). If the collision is also elastic (a special case), the conservation of kinetic energy during the collision gives a third equation: K1i & K2i ! K1f & K2f .

(9-78)

Variable-Mass Systems

In the absence of external forces a rocket accelerates at an instantaneous rate given by Rvrel ! Ma

(first rocket equation),

(9-87)

:

where the subscripts refer to the values of P at some initial time and at a later time. Equations 9-42 and 9-43 are equivalent statements of the law of conservation of linear momentum.

Inelastic Collision in One Dimension In an inelastic collision of two bodies, the kinetic energy of the two-body system is not conserved (it is not a constant). If the system is closed and isolated, the total linear momentum of the system

in which M is the rocket’s instantaneous mass (including unexpended fuel), R is the fuel consumption rate, and vrel is the fuel’s exhaust speed relative to the rocket. The term Rvrel is the thrust of the rocket engine. For a rocket with constant R and vrel, whose speed changes from vi to vf when its mass changes from Mi to Mf , vf % vi ! vrel ln

Mi Mf

(second rocket equation).

(9-88)

QU ESTIONS

245

Questions y 1 Figure 9-23 shows an overhead view of three particles on which external forces act.The magnitudes and 1 3 directions of the forces on two of the 5N x particles are indicated. What are the 2 magnitude and direction of the force 3N acting on the third particle if the cenFigure 9-23 Question 1. ter of mass of the three-particle system is (a) stationary, (b) moving at a constant velocity rightward, and (c) accelerating rightward?

boxes move over a frictionless confectioner’s counter. For each box, is its linear momentum conserved along the x axis and the y axis? 6 Figure 9-28 shows four groups of three or four identical particles that move parallel to either the x axis or the y axis, at identical speeds. Rank the groups according to center-of-mass speed, greatest first. y

2 Figure 9-24 shows an overy (m) head view of four particles of b a equal mass sliding over a frictionless surface at constant 2 velocity. The directions of the velocities are indicated; their x (m) –4 –2 2 4 magnitudes are equal. Consider pairing the particles. Which –2 c d pairs form a system with a center of mass that (a) is stationary, Figure 9-24 Question 2. (b) is stationary and at the origin, and (c) passes through the origin? 3 Consider a box that explodes into two pieces while moving with a constant positive velocity along an x axis. If one piece, with mass m1, ends up with positive velocity v:1, then the second piece, with mass m2, could end up with (a) a positive velocity v:2 (Fig. 9-25a), (b) a negative velocity v:2 (Fig. 9-25b), or (c) zero velocity (Fig. 9-25c). Rank those three possible results for the second piece according to the corresponding magnitude of v:1, greatest first. v2

v1

v2

v1

v1

(a)

(b)

y

x (a)

(b) y

y

x

(c)

x

(d)

Figure 9-28 Question 6. 7 A block slides along a frictionless floor and into a stationary second block with the same mass. Figure 9-29 shows four choices for a graph of the kinetic energies K of the blocks. (a) Determine which represent physically impossible situations. Of the others, which best represents (b) an elastic collision and (c) an inelastic collision? K

(c)

x

K

Figure 9-25 Question 3. 4 Figure 9-26 shows graphs of force magnitude versus time for a body involved in a collision. Rank the graphs according to the magnitude of the impulse on the body, greatest first. F

F

F

t

t

3t0 (b)

12t0

(c)

y

2Ν 2Ν (a)

8Ν x

y

60° 60°

3Ν 2Ν

t

(c)

5 The free-body diagrams in Fig. 9-27 give, from overhead views, the horizontal forces acting on three boxes of chocolates as the

3Ν 4Ν





60°



x

60° 6Ν

(b)

Figure 9-27 Question 5.

t

t

Figure 9-26 Question 4.

y

(b)

K

2F0 6t0 (a)



t

K

4F0 2F0

(a)

(c)





x

(d)

t

Figure 9-29 Question 7. 8 Figure 9-30 shows a snapshot of A B C 1 2 block 1 as it slides along an x axis on a frictionless floor, before it undergoes Figure 9-30 Question 8. an elastic collision with stationary block 2.The figure also shows three possible positions of the center of mass (com) of the two-block system at the time of the snapshot. (Point B is halfway between the centers of the two blocks.) Is block 1 stationary, moving forward, or moving backward after the collision if the com is located in the snapshot at (a) A, (b) B, and (c) C?

246

CHAPTE R 9 CE NTE R OF MASS AN D LI N EAR M OM E NTU M

9 Two bodies have undergone an x 4 5 elastic one-dimensional collision along an x axis. Figure 9-31 is a graph 1 6 of position versus time for those bodies and for their center of mass. 2 3 (a) Were both bodies initially moving, t or was one initially stationary? Which Figure 9-31 Question 9. line segment corresponds to the motion of the center of mass (b) before the collision and (c) after the collision? (d) Is the mass of the body that was moving faster before the collision greater than, less than, or equal to that of the other body?

the negative direction of that axis. Then the block explodes into two pieces that slide along the x axis. Assume the block and the two pieces form a closed, isolated system. Six choices for a graph of the momenta of the block and the pieces are given, all versus time t. Determine which choices represent physically impossible situations and explain why. 11 Block 1 with mass m1 slides x 1 2 along an x axis across a frictionless A floor and then undergoes an elastic B collision with a stationary block 2 with 3 mass m2. Figure 9-33 shows a plot of xc C position x versus time t of block 1 until the collision occurs at position xc and D 4 time tc. In which of the lettered regions 5 t on the graph will the plot be contintc ued (after the collision) if (a) m1 + m2 Figure 9-33 Question 11. and (b) m1 - m2? (c) Along which of the numbered dashed lines will the plot be continued if m1 ! m2?

10 Figure 9-32: A block on a horizontal floor is initially either stationary, sliding in the positive direction of an x axis, or sliding in p

p

t

p

t

t

x

(a)

(b)

(c)

p

p

p

t

(d)

t

(e)

x

12 Figure 9-34 shows four graphs of position versus time for two bodies and their center of mass. The two bodies form a closed, isolated system t t and undergo a completely inelastic, (1) (2) x one-dimensional collision on an x axis. x In graph 1, are (a) the two bodies and (b) the center of mass moving in the positive or negative direction of the x t t axis? (c) Which of the graphs corre(3) (4) spond to a physically impossible situFigure 9-34 Question 12. ation? Explain.

t

(f )

Figure 9-32 Question 10.

Problems Tutoring problem available (at instructor’s discretion) in WileyPLUS and WebAssign SSM • – •••

Worked-out solution available in Student Solutions Manual

WWW Worked-out solution is at

Number of dots indicates level of problem difficulty

ILW

http://www.wiley.com/college/halliday

Interactive solution is at

Additional information available in The Flying Circus of Physics and at flyingcircusofphysics.com

••3 Figure 9-36 shows a slab with dimensions d1 ! 11.0 cm, d2 ! 2.80 cm, and d3 ! 13.0 cm. Half the slab consists of aluminum (den-

2d 1 d2

3

m3 •2 Figure 9-35 shows a three-parys ticle system, with masses m1 ! 3.0 m2 kg, m2 ! 4.0 kg, and m3 ! 8.0 kg. The scales on the axes are set by m1 x (m) xs ! 2.0 m and ys ! 2.0 m. What are xs 0 (a) the x coordinate and (b) the y coordinate of the system’s center Figure 9-35 Problem 2. of mass? (c) If m3 is gradually increased, does the center of mass of the system shift toward or away from that particle, or does it remain stationary?

sity ! 2.70 g/cm3) and half consists of iron (density ! 7.85 g/cm3). What are (a) the x coordinate, (b) the y coordinate, and (c) the z coordinate of the slab’s center of mass?

d

Module 9-1 Center of Mass •1 A 2.00 kg particle has the xy coordinates (%1.20 m, 0.500 m), and a 4.00 kg particle has the xy coordinates (0.600 m, %0.750 m). Both lie on a horizontal plane. At what (a) x and (b) y coordinates must you place a 3.00 kg particle such that the center of mass of the three-particle system has the coory (m) dinates (%0.500 m, %0.700 m)?

z

Iron

Midpoint Aluminum

x

d1 d1

Figure 9-36 Problem 3.

y

PROB LE M S

••4 In Fig. 9-37, three uniform thin rods, each of length L ! 22 cm, form an inverted U. The vertical rods each have a mass of 14 g; the horizontal rod has a mass of 42 g. What are (a) the x coordinate and (b) the y coordinate of the system’s center of mass?

Module 9-2 Newton’s Second Law for a System of Particles •9 ILW A stone is dropped at t ! 0. A second stone, with twice the mass of the first, is dropped from the same point at t ! 100 ms. (a) How far below the release point is the center of mass of the two stones at t ! 300 ms? (Neither stone has yet reached the ground.) (b) How fast is the center of mass of the twostone system moving at that time?

y x

L L

••5 What are (a) the x coordinate and (b) the y coordinate of the center of mass for the uniform plate shown in Fig. 9-38 if L ! 5.0 cm?

L

•10 A 1000 kg automobile is at rest at a traffic signal. At the instant the light turns green, the automobile starts to move with a constant acceleration of 4.0 m/s2. At the same instant a 2000 kg truck, traveling at a constant speed of 8.0 m/s, overtakes and passes the automobile. (a) How far is the com of the automobile – truck system from the traffic light at t ! 3.0 s? (b) What is the speed of the com then?

Figure 9-37 Problem 4.

y 2L

4L

•11 A big olive (m ! 0.50 kg) lies at the origin of an xy coordinate system, and a big Brazil nut (M ! 1.5 kg) lies at the : point (1.0, 2.0) m. At t ! 0, a force Fo ! (2.0iˆ & 3.0jˆ ) N begins to : act on the olive, and a force Fn ! (%3.0iˆ % 2.0jˆ ) N begins to act on the nut. In unit-vector notation, what is the displacement of the center of mass of the olive – nut system at t ! 4.0 s, with respect to its position at t ! 0?

L 3L x 4L 2L

•12 Two skaters, one with mass 65 kg and the other with mass 40 kg, stand on an ice rink holding a pole of length 10 m and negligible mass. Starting from the ends of the pole, the skaters pull themselves along the pole until they meet. How far does the 40 kg skater move?

2L

Figure 9-38 Problem 5. ••6 Figure 9-39 shows a cubical box that has been constructed from uniform metal plate of negligible thickness. The box is open at the top and has edge length L ! 40 cm. Find (a) the x coordinate, (b) the y coordinate, and (c) the z coordinate of the center of mass of the box.

z

L

247

O

•••7 ILW In the ammonia (NH3) molecule of Fig. 9-40, three hydrogen (H) x atoms form an equilateral triangle, with Figure 9-39 Problem 6. the center of the triangle at distance d ! %11 9.40 $ 10 m from each hydrogen atom. The nitrogen (N) atom is at the y apex of a pyramid, with the three hydroN gen atoms forming the base. The nitrogen-to-hydrogen atomic mass ratio is L 13.9, and the nitrogen-to-hydrogen disx H tance is L ! 10.14 $ 10%11 m. What are H the (a) x and (b) y coordinates of the d molecule’s center of mass? H •••8 A uniform soda can of mass Figure 9-40 Problem 7. 0.140 kg is 12.0 cm tall and filled with 0.354 kg of soda (Fig. 9-41). Then small holes are drilled in the top and bottom (with negligible loss of metal) to drain the soda. What is the height h of the com of the can and contents (a) initially and (b) after the can loses all the soda? (c) What happens to h as the soda x drains out? (d) If x is the height of the remaining soda at any given instant, find x when the com reaches its lowest Figure 9-41 Problem 8. point.

y

••13 SSM A shell is shot with an initial velocity : v0 of 20 m/s, at an angle of '0 ! 60# with the horizontal. At the top of the trajectory, the shell explodes into two fragments of equal mass (Fig. 9-42). One fragment, whose speed immediately after the explosion is zero, falls vertically. How far from the gun does the other fragment land, assuming that the terrain is level and that air drag is negligible? Explosion

v0

θ0

Figure 9-42 Problem 13. ••14 In Figure 9-43, two particles are launched from the origin of the coordinate system at time t ! 0. Particle 1 of mass m1 ! 5.00 g is shot directly along the x axis on a frictionless floor, with constant speed 10.0 m/s. Particle 2 of mass m2 ! 3.00 g is shot with a velocity of magnitude 20.0 m/s, at an upward angle such that it always stays directly above particle 1. (a) What is the maximum height Hmax reached by the com of the two-particle system? In unit-vector notation, what are the (b) velocity and (c) acceleration of the com when the com reaches Hmax? y 2 1

Figure 9-43 Problem 14.

x

248

CHAPTE R 9 CE NTE R OF MASS AN D LI N EAR M OM E NTU M

••15 Figure 9-44 shows an arrangement with an air track, in which a cart is connected by a cord to a hanging block. The cart has mass m1 ! 0.600 kg, and its center is initially at xy coordinates (%0.500 m, 0 m); the block has mass m2 ! 0.400 kg, and its center is initially at xy coordinates (0, %0.100 m).The mass of the cord and pulley are negligible. The cart is released from rest, and both cart and block move until the cart hits the pulley. The friction between the cart and the air track and between the pulley and its axle is negligible. (a) In unit-vector notation, what is the acceleration of the center of mass of the cart–block system? (b) What is the velocity of the com as a function of time t? (c) Sketch the path taken by the com. (d) If the path is curved, determine whether it bulges upward to the right or downward to the left, and if it is straight, find the angle between it and the x axis. y m1 x

••21 A 0.30 kg softball has a velocity of 15 m/s at an angle of 35# below the horizontal just before making contact with the bat.What is the magnitude of the change in momentum of the ball while in contact with the bat if the ball leaves with a velocity of (a) 20 m/s, vertically downward, and (b) 20 m/s, horizontally back toward the pitcher? ••22 Figure 9-47 gives an overhead view of the path taken by a 0.165 kg cue ball as it bounces from a rail of a pool table. The ball’s initial speed is 2.00 m/s, and the angle u1 is 30.0#.The bounce reverses the y component of the ball’s velocity but does not alter the x component. What are (a) angle u2 and (b) the change in the ball’s linear momentum in unit-vector notation? (The fact that the ball rolls is irrelevant to the problem.)

y

θ1 θ 2

x

Figure 9-47 Problem 22.

Module 9-4 Collision and Impulse •23 Until his seventies, Henri LaMothe (Fig. 9-48) excited audiences by belly-flopping from a height of 12 m into 30 cm of water. Assuming that he stops just as he reaches the bottom of the water and estimating his mass, find the magnitude of the impulse on him from the water.

m2

Figure 9-44 Problem 15. •••16 Ricardo, of mass 80 kg, and Carmelita, who is lighter, are enjoying Lake Merced at dusk in a 30 kg canoe. When the canoe is at rest in the placid water, they exchange seats, which are 3.0 m apart and symmetrically located with respect to the canoe’s center. If the canoe moves 40 cm horizontally relative to a pier post, D what is Carmelita’s mass? (a) •••17 In Fig. 9-45a, a 4.5 kg dog stands on an 18 kg flatboat at distance D ! 6.1 m from the shore. It walks 2.4 m along the boat toward shore and then stops. Assuming no friction between the boat and the water, find how far the dog is then from the shore. (Hint: See Fig. 9-45b.)

Dog's displacement dd

Boat's displacement db (b)

Figure 9-45 Problem 17.

Module 9-3 Linear Momentum •18 A 0.70 kg ball moving horizontally at 5.0 m/s strikes a vertical wall and rebounds with speed 2.0 m/s. What is the magnitude of the change in its linear momentum? •19 ILW A 2100 kg truck traveling north at 41 km/h turns east and accelerates to 51 km/h. (a) What is the change in the truck’s kinetic energy? What are the (b) magnitude and (c) direction of the change in its momentum? p0

George Long/Getty Images, Inc.

p (kg • m/s)

••20 At time t ! 0, a ball is struck at ground level and sent over level ground.The momentum p versus t during the flight is given by Fig. 9-46 (with p0 ! 6.0 kg)m/s and p1 ! 4.0 kg)m/s ). At what initial angle is the ball launched? (Hint: Find a solution that does not require you to read the time of the low point of the plot.)

Figure 9-48 Problem 23. Belly-flopping into 30 cm of water.

p1

0

1

2 3 t (s)

4

Figure 9-46 Problem 20.

5

•24 In February 1955, a paratrooper fell 370 m from an airplane without being able to open his chute but happened to land in snow, suffering only minor injuries. Assume that his speed at impact was 56 m/s (terminal speed), that his mass (including gear) was 85 kg, and that the magnitude of the force on him from the

249

PROB LE M S

snow was at the survivable limit of 1.2 $ 105 N. What are (a) the minimum depth of snow that would have stopped him safely and (b) the magnitude of the impulse on him from the snow?

are the magnitudes of the (c) impulse and (d) average force (assuming the same stopping time)?

Fx (N)

•25 A 1.2 kg ball drops vertically onto a floor, hitting with a speed of 25 m/s. It rebounds with an initial speed of 10 m/s. (a) What impulse acts on the ball during the contact? (b) If the ball is in contact with the floor for 0.020 s, what is the magnitude of the average force on the floor from the ball?

••32 A 5.0 kg toy car can move along an x axis; Fig. 9-50 gives Fx of the force acting on the car, which begins at rest at time t ! 0. The scale on the Fx axis is set by Fxs ! 5.0 N. In : unit-vector notation, what is p at (a) t ! 4.0 s and (b) t ! 7.0 s, and (c) what is : v at t ! 9.0 s?

Fxs

•26 In a common but dangerous prank, a chair is pulled away as a person is moving downward to sit on it, causing the victim to land hard on the floor. Suppose the victim falls by 0.50 m, the mass that moves downward is 70 kg, and the collision on the floor lasts 0.082 s. What are the magnitudes of the (a) impulse and (b) average force acting on the victim from the floor during the collision? •27 SSM A force in the negative direction of an x axis is applied for 27 ms to a 0.40 kg ball initially moving at 14 m/s in the positive direction of the axis. The force varies in magnitude, and the impulse has magnitude 32.4 N )s. What are the ball’s (a) speed and (b) direction of travel just after the force is applied? What are (c) the average magnitude of the force and (d) the direction of the impulse on the ball? •28 In tae-kwon-do, a hand is slammed down onto a target at a speed of 13 m/s and comes to a stop during the 5.0 ms collision. Assume that during the impact the hand is independent of the arm and has a mass of 0.70 kg. What are the magnitudes of the (a) impulse and (b) average force on the hand from the target? •29 Suppose a gangster sprays Superman’s chest with 3 g bullets at the rate of 100 bullets/min, and the speed of each bullet is 500 m/s. Suppose too that the bullets rebound straight back with no change in speed. What is the magnitude of the average force on Superman’s chest? ••30 Two average forces. A steady stream of 0.250 kg snowballs is shot perpendicularly into a wall at a speed of 4.00 m/s. Each ball sticks to the wall. Figure 9-49 gives the magnitude F of the force on the wall as a function of time t for two of the snowball impacts. Impacts occur with a repetition time interval "tr ! 50.0 ms, last a duration time interval "td ! 10 ms, and produce isosceles triangles on the graph, with each impact reaching a force maximum Fmax ! 200 N. During each impact, what are the magnitudes of (a) the impulse and (b) the average force on the wall? (c) During a time interval of many impacts, what is the magnitude of the average force on the wall?

t (s) 2

4

Figure 9-50 Problem 32.

••33 Figure 9-51 shows a 0.300 kg baseball just before and just after θ1 it collides with a bat. Just before, the v2 v1 ball has velocity : v1 of magnitude y 12.0 m/s and angle u1 ! 35.0#. Just after, it is traveling directly upward x v2 of magnitude 10.0 with velocity : m/s. The duration of the collision is Figure 9-51 Problem 33. 2.00 ms. What are the (a) magnitude and (b) direction (relative to the positive direction of the x axis) of the impulse on the ball from the bat? What are the (c) magnitude and (d) direction of the average force on the ball from the bat? ••34 Basilisk lizards can run across the top of a water surface (Fig. 9-52). With each step, a lizard first slaps its foot against the water and then pushes it down into the water rapidly enough to form an air cavity around the top of the foot. To avoid having to pull the foot back up against water drag in order to complete the step, the lizard withdraws the foot before water can flow into the air cavity. If the lizard is not to sink, the average upward impulse on the lizard during this full action of slap, downward push, and withdrawal must match the downward impulse due to the gravitational force. Suppose the mass of a basilisk lizard is 90.0 g, the mass of each foot is 3.00 g, the speed of a foot as it slaps the water is 1.50 m/s, and the time for a single step is 0.600 s. (a) What is the magnitude of the impulse on the lizard during the slap? (Assume this impulse is directly upward.) (b) During the 0.600 s duration of a step, what is the downward impulse on the lizard due to the gravitational force? (c) Which action, the slap or the push, provides the primary support for the lizard, or are they approximately equal in their support?

Fmax t ∆td ∆tr

Figure 9-49 Problem 30. ••31 Jumping up before the elevator hits. After the cable snaps and the safety system fails, an elevator cab free-falls from a height of 36 m. During the collision at the bottom of the elevator shaft, a 90 kg passenger is stopped in 5.0 ms. (Assume that neither the passenger nor the cab rebounds.) What are the magnitudes of the (a) impulse and (b) average force on the passenger during the collision? If the passenger were to jump upward with a speed of 7.0 m/s relative to the cab floor just before the cab hits the bottom of the shaft, what

8

–Fxs

F

∆td

6

Stephen Dalton/Photo Researchers, Inc.

Figure 9-52 Problem 34. Lizard running across water.

250

CHAPTE R 9 CE NTE R OF MASS AN D LI N EAR M OM E NTU M

F (N)

••35 Figure 9-53 shows an Fmax approximate plot of force magnitude F versus time t during the collision of a 58 g Superball with a wall. The initial velocity of the ball is 34 m/s perpendicular to the wall; the ball rebounds di0 0 2 4 6 rectly back with approximately t (ms) the same speed, also perpendiFigure 9-53 Problem 35. cular to the wall. What is Fmax, the maximum magnitude of the force on the ball from the wall during the collision? ••36 A 0.25 kg puck is initially stationary on an ice surface with negligible friction. At time t ! 0, a horizontal force begins to : : move the puck. The force is given by F ! (12.0 % 3.00t 2)iˆ , with F in newtons and t in seconds, and it acts until its magnitude is zero. (a) What is the magnitude of the impulse on the puck from the force between t ! 0.500 s and t ! 1.25 s? (b) What is the change in momentum of the puck between t ! 0 and the instant at which F ! 0?

locity that the explosion gives the rest of the rocket. (2) Next, at time t ! 0.80 s, block R is shot to the right with a speed of 3.00 m/s relative to the velocity that block C then has. At t ! 2.80 s, what are (a) the velocity of block C and (b) the position of its center? ••42 An object, with mass m and speed v relative to an observer, explodes into two pieces, one three times as massive as the other; the explosion takes place in deep space. The less massive piece stops relative to the observer. How much kinetic energy is added to the system during the explosion, as measured in the observer’s reference frame? ••43 In the Olympiad of 708 B.C., some athletes competing in the standing long jump used handheld weights called halteres to lengthen their jumps (Fig. 9-56).The weights were swung up in front just before liftoff and then swung down and thrown backward during the flight. Suppose a modern 78 kg long jumper similarly uses two 5.50 kg halteres, throwing them horizontally to the rear at his maximum height such that their horizontal velocity is zero relative to the ground. Let his liftoff velocity be : v ! (9.5iˆ & 4.0jˆ) m/s with or without the halteres, and assume that he lands at the liftoff level. What distance would the use of the halteres add to his range?

••37 SSM A soccer player kicks a soccer ball of mass 0.45 kg that is initially at rest. The foot of the player is in contact with the ball for 3.0 $ 10%3 s, and the force of the kick is given by F(t) ! [(6.0 $ 10 6)t % (2.0 $ 10 9)t2] N for 0 ( t ( 3.0 $ 10%3 s, where t is in seconds. Find the magnitudes of (a) the impulse on the ball due to the kick, (b) the average force on the ball from the player’s foot during the period of contact, (c) the maximum force on the ball from the player’s foot during the period of contact, and (d) the ball’s velocity immediately after it loses contact with the player’s foot. y ••38 In the overhead view of Fig. v 9-54, a 300 g ball with a speed v of v x 6.0 m/s strikes a wall at an angle u of 30# and then rebounds with the θ θ same speed and angle. It is in contact with the wall for 10 ms. In unitFigure 9-54 Problem 38. vector notation, what are (a) the impulse on the ball from the wall and (b) the average force on the wall from the ball?

Module 9-5 Conservation of Linear Momentum •39 SSM A 91 kg man lying on a surface of negligible friction shoves a 68 g stone away from himself, giving it a speed of 4.0 m/s. What speed does the man acquire as a result? •40 A space vehicle is traveling at 4300 km/h relative to Earth when the exhausted rocket motor (mass 4m) is disengaged and sent backward with a speed of 82 km/h relative to the command module (mass m). What is the speed of the command module relative to Earth just after the separation? ••41 Figure 9-55 shows a two-ended “rocket” that is initially stationary on a frictionless floor, with its center at the origin of an x axis. The rocket consists of a central block C (of mass M ! 6.00 kg) and blocks L and R (each of mass m ! 2.00 kg) on the left and right sides. Small explosions can C L R shoot either of the side blocks away from block C and along the x axis. x Here is the sequence: (1) At time t ! 0 0, block L is shot to the left with a Figure 9-55 Problem 41. speed of 3.00 m/s relative to the ve-

Réunion des Musées Nationaux/ Art Resource

Figure 9-56 Problem 43. ••44 In Fig. 9-57, a stationary block explodes into two pieces L and R that slide across a frictionless floor and then into regions with friction, where they stop. Piece L, with a mass of 2.0 kg, encounters a coefficient of kinetic friction mL ! 0.40 and slides to a stop in distance dL ! 0.15 m. Piece R encounters a coefficient of kinetic friction mR ! 0.50 and slides to a stop in distance dR ! 0.25 m. What was the mass of the block? µL

µ=0

dL

µR dR

Figure 9-57 Problem 44. ••45 SSM WWW A 20.0 kg body is moving through space in the positive direction of an x axis with a speed of 200 m/s when, due to an internal explosion, it breaks into three parts. One part, with a mass of 10.0 kg, moves away from the point of explosion with a speed of 100 m/s in the positive y direction. A second part, with a mass of 4.00 kg, moves in the negative x direction with a speed of 500 m/s. (a) In unit-vector notation, what is the velocity of the third part? (b) How much energy is released in the explosion? Ignore effects due to the gravitational force. ••46 A 4.0 kg mess kit sliding on a frictionless surface explodes into two 2.0 kg parts: 3.0 m/s, due north, and 5.0 m/s, 30# north of east. What is the original speed of the mess kit?

251

PROB LE M S

••47 A vessel at rest at the origin of an xy coordinate system explodes into three pieces. Just after the explosion, one piece, of mass m, moves with velocity (%30 m/s) ˆi and a second piece, also of mass m, moves with velocity (%30 m/s) ˆj . The third piece has mass 3m. Just after the explosion, what are the (a) magnitude and (b) direction of the velocity of the third piece? •••48 Particle A and particle B are held together with a compressed spring between them. When they are released, the spring pushes them apart, and they then fly off in opposite directions, free of the spring. The mass of A is 2.00 times the mass of B, and the energy stored in the spring was 60 J. Assume that the spring has negligible mass and that all its stored energy is transferred to the particles. Once that transfer is complete, what are the kinetic energies of (a) particle A and (b) particle B? Module 9-6 Momentum and Kinetic Energy in Collisions •49 A bullet of mass 10 g strikes a ballistic pendulum of mass 2.0 kg. The center of mass of the pendulum rises a vertical distance of 12 cm. Assuming that the bullet remains embedded in the pendulum, calculate the bullet’s initial speed. •50 A 5.20 g bullet moving at 672 m/s strikes a 700 g wooden block at rest on a frictionless surface. The bullet emerges, traveling in the same direction with its speed reduced to 428 m/s. (a) What is the resulting speed of the block? (b) What is the speed of the bullet – block center of mass? ••51 In Fig. 9-58a, a 3.50 g bullet is fired horizontally at two blocks at rest on a frictionless table. The bullet passes through block 1 (mass 1.20 kg) and embeds itself in block 2 (mass 1.80 kg). The blocks end up with speeds v1 ! 0.630 m/s and v2 ! 1.40 m/s (Fig. 9-58b). Neglecting the material removed from block 1 by the bullet, find the speed of the bullet as it (a) leaves and (b) enters block 1. 1

2

collisions (CVC). (b) What percent of the original kinetic energy is lost if the car hits a 300 kg camel? (c) Generally, does the percent loss increase or decrease if the animal mass decreases? ••54 A completely inelastic collision occurs between two balls of wet putty that move directly toward each other along a vertical axis. Just before the collision, one ball, of mass 3.0 kg, is moving upward at 20 m/s and the other ball, of mass 2.0 kg, is moving downward at 12 m/s. How high do the combined two balls of putty rise above the collision point? (Neglect air drag.) ••55 ILW A 5.0 kg block with a speed of 3.0 m/s collides with a 10 kg block that has a speed of 2.0 m/s in the same direction. After the collision, the 10 kg block travels in the original direction with a speed of 2.5 m/s. (a) What is the velocity of the 5.0 kg block immediately after the collision? (b) By how much does the total kinetic energy of the system of two blocks change because of the collision? (c) Suppose, instead, that the 10 kg block ends up with a speed of 4.0 m/s. What then is the change in the total kinetic energy? (d) Account for the result you obtained in (c). ••56 In the “before” part of Fig. 9-60, car A (mass 1100 kg) is stopped at a traffic light when it is rear-ended by car B (mass 1400 kg). Both cars then slide with locked wheels until the frictional force from the slick road (with a low mk of 0.13) stops them, at distances dA ! 8.2 m and dB ! 6.1 m. What are the speeds of (a) car A and (b) car B at the start of the sliding, just after the collision? (c) Assuming that linear momentum is conserved during the collision, find the speed of car B just before the collision. (d) Explain why this assumption may be invalid. v0

Before A

Frictionless dA

(a) v1

v2

After A

dB B

Figure 9-60 Problem 56.

(b)

Figure 9-58 Problem 51. ••52 In Fig. 9-59, a 10 g bullet moving directly upward at 1000 m/s strikes and passes through the center of mass of a 5.0 kg block initially at rest. The bullet emerges from the block moving directly upward at 400 m/s. To what maximum height does the block then rise above its initial position?

B

Bullet

Figure 9-59 Problem 52.

••53 In Anchorage, collisions of a vehicle with a moose are so common that they are referred to with the abbreviation MVC. Suppose a 1000 kg car slides into a stationary 500 kg moose on a very slippery road, with the moose being thrown through the windshield (a common MVC result). (a) What percent of the original kinetic energy is lost in the collision to other forms of energy? A similar danger occurs in Saudi Arabia because of camel–vehicle

••57 In Fig. 9-61, a ball of mass vi M m m ! 60 g is shot with speed vi ! 22 m/s into the barrel of a spring gun of mass M ! 240 g initially at rest on a Figure 9-61 Problem 57. frictionless surface. The ball sticks in the barrel at the point of maximum compression of the spring. Assume that the increase in thermal energy due to friction between the ball and the barrel is negligible. (a) What is the speed of the spring gun after the ball stops in the barrel? (b) What fraction of the initial kinetic energy of the ball is stored in the spring? •••58 In Fig. 9-62, block 2 (mass 1.0 v1 1 2 kg) is at rest on a frictionless surface and touching the end of an unstretched spring of spring constant Figure 9-62 Problem 58. 200 N/m. The other end of the spring is fixed to a wall. Block 1 (mass 2.0 kg), traveling at speed v1 ! 4.0 m/s, collides with block 2, and the two blocks stick together.When the blocks momentarily stop, by what distance is the spring compressed?

252

CHAPTE R 9 CE NTE R OF MASS AN D LI N EAR M OM E NTU M

•••59 ILW In Fig. 9-63, block 1 (mass 2.0 kg) is moving rightward at 10 m/s and block 2 (mass 5.0 kg) is moving rightward at 3.0 m/s. The surface is frictionless, and a spring with a spring constant of 1120 N/m is fixed to block 2. When the blocks collide, the compression of the spring is maximum at the instant the blocks have the same velocity. Find the maximum compression. 1

2

Figure 9-63 Problem 59. Module 9-7 Elastic Collisions in One Dimension •60 In Fig. 9-64, block A (mass 1.6 vAi vBi kg) slides into block B (mass 2.4 kg), along a frictionless surface. The directions of three velocities before (i) and after ( f ) the collision are indicated; the corresponding speeds are vAi ! vAf = ? vBf 5.5 m/s, vBi ! 2.5 m/s, and vBf ! 4.9 m/s. What are the (a) speed and (b) direction (left or right) of velocity : v Af ? (c) Is the collision elastic? Figure 9-64 Problem 60. •61 SSM A cart with mass 340 g moving on a frictionless linear air track at an initial speed of 1.2 m/s undergoes an elastic collision with an initially stationary cart of unknown mass. After the collision, the first cart continues in its original direction at 0.66 m/s. (a) What is the mass of the second cart? (b) What is its speed after impact? (c) What is the speed of the twocart center of mass? •62 Two titanium spheres approach each other head-on with the same speed and collide elastically. After the collision, one of the spheres, whose mass is 300 g, remains at rest. (a) What is the mass of the other sphere? (b) What is the speed of the two-sphere center of mass if the initial speed of each sphere is 2.00 m/s? ••63 Block 1 of mass m1 slides along a frictionless floor and into a one-dimensional elastic collision with stationary block 2 of mass m2 ! 3m1. Prior to the collision, the center of mass of the twoblock system had a speed of 3.00 m/s. Afterward, what are the speeds of (a) the center of mass and (b) block 2? ••64 A steel ball of mass 0.500 kg is fastened to a cord that is 70.0 cm long and fixed at the far end.The ball is then released when the cord is horizontal (Fig. 9-65). At the bottom of its path, the ball strikes a 2.50 kg steel block initially at rest on a frictionless surface. The collision is elastic. Find (a) the speed of the ball and (b) the speed of the block, both just after the collision.

Figure 9-65 Problem 64.

••65 SSM A body of mass 2.0 kg makes an elastic collision with another body at rest and continues to move in the original direction but with one-fourth of its original speed. (a) What is the mass of the other body? (b) What is the speed of the two-body center of mass if the initial speed of the 2.0 kg body was 4.0 m/s? ••66 Block 1, with mass m1 and speed 4.0 m/s, slides along an x axis on a frictionless floor and then undergoes a one-dimensional elastic collision with stationary block 2, with mass m2 ! 0.40m1. The two blocks then slide into a region where the coefficient of kinetic

friction is 0.50; there they stop. How far into that region do (a) block 1 and (b) block 2 slide? ••67 In Fig. 9-66, particle 1 of mass m1 ! 0.30 kg slides rightward along 1 2 an x axis on a frictionless floor with a x (cm) speed of 2.0 m/s.When it reaches x ! 0 xw 0, it undergoes a one-dimensional Figure 9-66 Problem 67. elastic collision with stationary particle 2 of mass m2 ! 0.40 kg.When particle 2 then reaches a wall at xw ! 70 cm, it bounces from the wall with no loss of speed. At what position on the x axis does particle 2 then collide with particle 1? ••68 In Fig. 9-67, block 1 of mass m1 slides from rest along a frictionless ramp from height h ! 2.50 m and then collides with stationary block 2, which has mass m2 ! 2.00m1. After the collision, block 2 slides into a region where the coefficient of kinetic friction mk is 0.500 and comes to a stop in distance d within that region. What is the value of distance d if the collision is (a) elastic and (b) completely inelastic? 1

h

Frictionless 2

µk

Figure 9-67 Problem 68. •••69 A small ball of mass m is aligned above a larger ball of mass M ! 0.63 kg (with a slight separation, as with the baseball and basketball of Fig. 9-68a), and the two are dropped simultaneously from a height of h ! 1.8 m. (Assume the radius of each ball is negligible relative to h.) (a) If the larger ball rebounds elastically from the floor and then the small ball rebounds elastically from the larger ball, what value of m results in the larger ball stopping when it collides with the small ball? (b) What height does the small ball then reach (Fig. 9-68b)?

Baseball

Basketball

(a) Before

(b) After

Figure 9-68 Problem 69.

•••70 In Fig. 9-69, puck 1 of mass m1 ! 0.20 kg is sent sliding across a frictionless lab bench, to undergo a one-dimensional elastic collision with stationary puck 2. Puck 2 then slides off the bench and lands a distance d from the base of the bench. Puck 1 rebounds from the collision and slides off the opposite edge of the bench, landing a distance 2d from the base of the bench. What is the mass of puck 2? (Hint: Be careful with signs.) 1

2

2d

d

Figure 9-69 Problem 70.

PROB LE M S

Module 9-8 Collisions in Two Dimensions ••71 ILW In Fig. 9-21, projectile particle 1 is an alpha particle and target particle 2 is an oxygen nucleus. The alpha particle is scattered at angle u1 ! 64.0# and the oxygen nucleus recoils with speed 1.20 $ 105 m/s and at angle u2 ! 51.0#. In atomic mass units, the mass of the alpha particle is 4.00 u and the mass of the oxygen nucleus is 16.0 u. What are the (a) final and (b) initial speeds of the alpha particle? ••72 Ball B, moving in the positive direction of an x axis at speed v, collides with stationary ball A at the origin. A and B have different masses. After the collision, B moves in the negative direction of the y axis at speed v/2. (a) In what direction does A move? (b) Show that the speed of A cannot be determined from the given information. ••73 After a completely inelastic collision, two objects of the same mass and same initial speed move away together at half their initial speed. Find the angle between the initial velocities of the objects. ••74 Two 2.0 kg bodies, A and B, collide. The velocities before the collision are : vA ! (15iˆ & 30jˆ ) m/s and : vB ! (%10iˆ & 5.0jˆ ) m/s.After the collision, : v/A ! (%5.0iˆ & 20jˆ ) m/s. What are (a) the final velocity of B and (b) the change in the total kinetic energy (including sign)? ••75 A projectile proton with a speed of 500 m/s collides elastically with a target proton initially at rest. The two protons then move along perpendicular paths, with the projectile path at 60# from the original direction. After the collision, what are the speeds of (a) the target proton and (b) the projectile proton? Module 9-9 Systems with Varying Mass: A Rocket •76 A 6090 kg space probe moving nose-first toward Jupiter at 105 m/s relative to the Sun fires its rocket engine, ejecting 80.0 kg of exhaust at a speed of 253 m/s relative to the space probe. What is the final velocity of the probe? •77 SSM In Fig. 9-70, two long barges are moving in the same direction in still water, one with a speed of 10 km/h and the other with a speed of 20 km/h. While they are passing each other, coal is shoveled from the slower to the faster one at a rate of 1000 kg/min. How much additional force must be provided by the driving engines of (a) the faster barge and (b) the slower barge if neither is to change speed? Assume that the shoveling is always perfectly sideways and that the frictional forces between the barges and the water do not depend on the mass of the barges.

Figure 9-70 Problem 77. •78 Consider a rocket that is in deep space and at rest relative to an inertial reference frame. The rocket’s engine is to be fired for a

253

certain interval. What must be the rocket’s mass ratio (ratio of initial to final mass) over that interval if the rocket’s original speed relative to the inertial frame is to be equal to (a) the exhaust speed (speed of the exhaust products relative to the rocket) and (b) 2.0 times the exhaust speed? •79 SSM ILW A rocket that is in deep space and initially at rest relative to an inertial reference frame has a mass of 2.55 $ 105 kg, of which 1.81 $ 105 kg is fuel. The rocket engine is then fired for 250 s while fuel is consumed at the rate of 480 kg/s. The speed of the exhaust products relative to the rocket is 3.27 km/s. (a) What is the rocket’s thrust? After the 250 s firing, what are (b) the mass and (c) the speed of the rocket? Additional Problems 80 An object is tracked by a radar station and determined to have a position vector given by r: ! (3500 % 160t)ˆi & 2700 ˆj & 300 kˆ, with r: in meters and t in seconds. The radar station’s x axis points east, its y axis north, and its z axis vertically up. If the object is a 250 kg meteorological missile, what are (a) its linear momentum, (b) its direction of motion, and (c) the net force on it? 81 The last stage of a rocket, which is traveling at a speed of 7600 m/s, consists of two parts that are clamped together: a rocket case with a mass of 290.0 kg and a payload capsule with a mass of 150.0 kg. When the clamp is released, a compressed spring causes the two parts to separate with a relative speed of 910.0 m/s. What are the speeds of (a) the rocket case and (b) the payload after they have separated? Assume that all velocities are along the same line. Find the total kinetic energy of the two parts (c) before and (d) after they separate. (e) Account for the difference. N 82 Pancake collapse of a tall building. In the section of a tall M building shown in Fig. 9-71a, the inL frastructure of any given floor K d must support the weight W of all K higher floors. Normally the infraJ structure is constructed with a I safety factor s so that it can withstand an even greater downward (a) (b) force of sW. If, however, the support Figure 9-71 Problem 82. columns between K and L suddenly collapse and allow the higher floors to free-fall together onto floor K (Fig. 9-71b), the force in the collision can exceed sW and, after a brief pause, cause K to collapse onto floor J, which collapses on floor I, and so on until the ground is reached. Assume that the floors are separated by d ! 4.0 m and have the same mass. Also assume that when the floors above K free-fall onto K, the collision lasts 1.5 ms. Under these simplified conditions, what value must the safety factor s exceed to prevent pancake collapse of the building?

83 “Relative” is an important L R word. In Fig. 9-72, block L of mass mL ! 1.00 kg and block R of mass mR ! 0.500 kg are held in place with Figure 9-72 Problem 83. a compressed spring between them. When the blocks are released, the spring sends them sliding across a frictionless floor. (The spring has negligible mass and falls to the floor after the blocks leave it.) (a) If the spring gives block L a release speed of 1.20 m/s relative to the floor, how far does block R travel in the next 0.800 s? (b) If, instead, the spring gives block L a release speed of 1.20 m/s relative to the velocity that the spring gives block R, how far does block R travel in the next 0.800 s?

254

CHAPTE R 9 CE NTE R OF MASS AN D LI N EAR M OM E NTU M

84 Figure 9-73 shows an overhead 1 A 2 view of two particles sliding at constant velocity over a frictionless surface. The B θ θ particles have the same mass and the x same initial speed v ! 4.00 m/s, and they θ θ C collide where their paths intersect. An x axis is arranged to bisect the angle be3 4 D tween their incoming paths, such that u ! 40.0#. The region to the right of the Figure 9-73 Problem 84. collision is divided into four lettered sections by the x axis and four numbered dashed lines. In what region or along what line do the particles travel if the collision is (a) completely inelastic, (b) elastic, and (c) inelastic? What are their final speeds if the collision is (d) completely inelastic and (e) elastic? 85 Speed deamplifier. In Fig. 9-74, block 1 of mass m1 slides along 1 2 3 an x axis on a frictionless floor at x speed 4.00 m/s. Then it undergoes a Figure 9-74 Problem 85. one-dimensional elastic collision with stationary block 2 of mass m2 ! 2.00m1. Next, block 2 undergoes a one-dimensional elastic collision with stationary block 3 of mass m3 ! 2.00m2. (a) What then is the speed of block 3? Are (b) the speed, (c) the kinetic energy, and (d) the momentum of block 3 greater than, less than, or the same as the initial values for block 1? 86 Speed amplifier. In Fig. 9-75, block 1 of mass m1 slides along an x 1 2 3 axis on a frictionless floor with a x speed of v1i ! 4.00 m/s. Then it underFigure 9-75 Problem 86. goes a one-dimensional elastic collision with stationary block 2 of mass m2 ! 0.500m1. Next, block 2 undergoes a one-dimensional elastic collision with stationary block 3 of mass m3 ! 0.500m2. (a) What then is the speed of block 3? Are (b) the speed, (c) the kinetic energy, and (d) the momentum of block 3 greater than, less than, or the same as the initial values for block 1? 87 A ball having a mass of 150 g strikes a wall with a speed of 5.2 m/s and rebounds with only 50% of its initial kinetic energy. (a) What is the speed of the ball immediately after rebounding? (b) What is the magnitude of the impulse on the wall from the ball? (c) If the ball is in contact with the wall for 7.6 ms, what is the magnitude of the average force on the ball from the wall during this time interval? 88 A spacecraft is separated into two parts by detonating the explosive bolts that hold them together. The masses of the parts are 1200 kg and 1800 kg; the magnitude of the impulse on each part from the bolts is 300 N )s. With what relative speed do the two parts separate because of the detonation? 89 SSM A 1400 kg car moving at 5.3 m/s is initially traveling north along the positive direction of a y axis. After completing a 90# right-hand turn in 4.6 s, the inattentive operator drives into a tree, which stops the car in 350 ms. In unit-vector notation, what is the impulse on the car (a) due to the turn and (b) due to the collision? What is the magnitude of the average force that acts on the car (c) during the turn and (d) during the collision? (e) What is the direction of the average force during the turn? 90 ILW A certain radioactive (parent) nucleus transforms to a different (daughter) nucleus by emitting an electron and a neutrino. The parent nucleus was at rest at the origin of an xy coordinate system. The electron moves away from the origin with linear momentum (%1.2 $ 10%22 kg )m/s)ˆi ; the neutrino moves away from the

origin with linear momentum (%6.4 $ 10%23 kg )m/s)ˆj . What are the (a) magnitude and (b) direction of the linear momentum of the daughter nucleus? (c) If the daughter nucleus has a mass of 5.8 $ 10%26 kg, what is its kinetic energy? 91 A 75 kg man rides on a 39 kg cart moving at a velocity of 2.3 m/s. He jumps off with zero horizontal velocity relative to the ground. What is the resulting change in the cart’s velocity, including sign? 92 Two blocks of masses 1.0 kg and 3.0 kg are connected by a spring and rest on a frictionless surface. They are given velocities toward each other such that the 1.0 kg block travels initially at 1.7 m/s toward the center of mass, which remains at rest. What is the initial speed of the other block? 93 SSM A railroad freight car of mass 3.18 $ 104 kg collides with a stationary caboose car. They couple together, and 27.0% of the initial kinetic energy is transferred to thermal energy, sound, vibrations, and so on. Find the mass of the caboose. 94 An old Chrysler with mass 2400 kg is moving along a straight stretch of road at 80 km/h. It is followed by a Ford with mass 1600 kg moving at 60 km/h. How fast is the center of mass of the two cars moving? 95 SSM In the arrangement of Fig. 9-21, billiard ball 1 moving at a speed of 2.2 m/s undergoes a glancing collision with identical billiard ball 2 that is at rest. After the collision, ball 2 moves at speed 1.1 m/s, at an angle of u2 ! 60#. What are (a) the magnitude and (b) the direction of the velocity of ball 1 after the collision? (c) Do the given data suggest the collision is elastic or inelastic? 96 A rocket is moving away from the solar system at a speed of 6.0 $ 10 3 m/s. It fires its engine, which ejects exhaust with a speed of 3.0 $ 10 3 m/s relative to the rocket. The mass of the rocket at this time is 4.0 $ 10 4 kg, and its acceleration is 2.0 m/s2. (a) What is the thrust of the engine? (b) At what rate, in kilograms per second, is exhaust ejected during the firing? 2 97 The three balls in the v0 x overhead view of Fig. 9-76 are 1 identical. Balls 2 and 3 touch 3 each other and are aligned perFigure 9-76 Problem 97. pendicular to the path of ball 1. The velocity of ball 1 has magnitude v0 ! 10 m/s and is directed at the contact point of balls 1 and 2. After the collision, what are the (a) speed and (b) direction of the velocity of ball 2, the (c) speed and (d) direction of the velocity of ball 3, and the (e) speed and (f) direction of the velocity of ball 1? (Hint: With friction absent, each impulse is directed along the line connecting the centers of the colliding balls, normal to the colliding surfaces.) 98 A 0.15 kg ball hits a wall with a velocity of (5.00 m/s)ˆi & (6.50 m/s)ˆj & (4.00 m/s) kˆ. It rebounds from the wall with a velocity of (2.00 m/s)ˆi & (3.50 m/s)ˆj & (%3.20 m/s)kˆ. What are (a) the change in the ball’s momentum, (b) the impulse on the ball, and (c) the impulse on the wall?

99 In Fig. 9-77, two identical containers of sugar are connected by a cord that passes over a frictionless pulley. The cord and pulley have negligible mass, each container and its sugar together have a mass of 500 g, the centers of the containers are separated by 50 mm, and the containers are held fixed at the same height. What is the horizontal distance between the center of container 1 and the center of mass of the two-container system (a) initially and

1

2

Figure 9-77 Problem 99.

PROB LE M S

(b) after 20 g of sugar is transferred from container 1 to container 2? After the transfer and after the containers are released, (c) in what direction and (d) at what acceleration magnitude does the center of mass move? 100 In a game of pool, the cue ball strikes another ball of the same mass and initially at rest. After the collision, the cue ball moves at 3.50 m/s along a line making an angle of 22.0# with the cue ball’s original direction of motion, and the second ball has a speed of 2.00 m/s. Find (a) the angle between the direction of motion of the second ball and the original direction of motion of the cue ball and (b) the original speed of the cue ball. (c) Is kinetic energy (of the centers of mass, don’t consider the rotation) conserved? 101 In Fig. 9-78, a 3.2 kg box of running shoes slides on a horizontal frictionless table and collides with a h 2.0 kg box of ballet slippers initially at rest on the edge of the table, at height h ! 0.40 m. The speed of the Figure 9-78 Problem 101. 3.2 kg box is 3.0 m/s just before the collision. If the two boxes stick together because of packing tape on their sides, what is their kinetic energy just before they strike the floor? 102 In Fig. 9-79, an 80 kg man is on a ladder hanging from a balloon that has a total mass of 320 kg (including the basket passenger). The balloon is initially stationary relative to the ground. If the man on the ladder begins to climb at 2.5 m/s relative to the ladder, (a) in what direction and (b) at what speed does the balloon move? (c) If the man then stops climbing, what is the speed of the balloon?

boat will initially touch the dock, as in Fig. 9-81; the boat can slide through the water without significant resistance; both the car and the boat can be approximated as uniform in their mass distribution. Determine what the width of the gap will be just as the car is about to make the jump. 105 SSM A 3.0 kg object moving at 8.0 m/s in the positive direction of an x axis has a one-dimensional elastic collision with an object of mass M, initially at rest. After the collision the object of mass M has a velocity of 6.0 m/s in the positive direction of the axis. What is mass M? 106 A 2140 kg railroad flatcar, which can move with negligible friction, is motionless next to a platform. A 242 kg sumo wrestler runs at 5.3 m/s along the platform (parallel to the track) and then jumps onto the flatcar. What is the speed of the flatcar if he then (a) stands on it, (b) runs at 5.3 m/s relative to it in his original direction, and (c) turns and runs at 5.3 m/s relative to the flatcar opposite his original direction? 107 SSM A 6100 kg rocket is set for vertical firing from the ground. If the exhaust speed is 1200 m/s, how much gas must be ejected each second if the thrust (a) is to equal the magnitude of the gravitational force on the rocket and (b) is to give the rocket an initial upward acceleration of 21 m/s2? 108 A 500.0 kg module is attached to a 400.0 kg shuttle craft, which moves at 1000 m/s relative to the stationary main spaceship. Then a small explosion sends the module backward with speed 100.0 m/s relative to the new speed of the shuttle craft. As measured by someone on the main spaceship, by what fraction did the kinetic energy of the module and shuttle craft increase because of the explosion? 109 SSM (a) How far is the center of mass of the Earth – Moon system from the center of Earth? (Appendix C gives the masses of Earth and the Moon and the distance between the two.) (b) What percentage of Earth’s radius is that distance?

103 In Fig. 9-80, block 1 of mass m1 ! 6.6 kg is at rest on a long frictionless table that is up against a wall. Block 2 of mass m2 is placed Figure 9-79 between block 1 and the wall and sent sliding Problem 102. to the left, toward block 1, with constant speed v2i . Find the value of m2 for which both blocks move with the same velocity after block 2 has collided once with block 1 and once with the wall. Assume all collisions are elastic (the collision with the wall does not change the speed of block 2). v2i 1

255

2

Figure 9-80 Problem 103. 104 The script for an action movie calls for a small race car (of mass 1500 kg and length 3.0 m) to accelerate along a flattop boat (of mass 4000 kg and length 14 m), from one end of the boat to the other, where the car will then jump Dock Boat the gap between the boat and a somewhat lower dock. You are the Figure 9-81 Problem 104. technical advisor for the movie. The A45678SF

110 A 140 g ball with speed 7.8 m/s strikes a wall perpendicularly and rebounds in the opposite direction with the same speed. The collision lasts 3.80 ms. What are the magnitudes of the (a) impulse and (b) average force on the wall from the ball during the elastic collision? 111 SSM A rocket sled with a mass of 2900 kg moves at 250 m/s on a set of rails. At a certain point, a scoop on the sled dips into a trough of water located between the tracks and scoops water into an empty tank on the sled. By applying the principle of conservation of linear momentum, determine the speed of the sled after 920 kg of water has been scooped up. Ignore any retarding force on the scoop. 112 SSM A pellet gun fires ten 2.0 g pellets per second with a speed of 500 m/s. The pellets are stopped by a rigid wall. What are (a) the magnitude of the momentum of each pellet, (b) the kinetic energy of each pellet, and (c) the magnitude of the average force on the wall from the stream of pellets? (d) If each pellet is in contact with the wall for 0.60 ms, what is the magnitude of the average force on the wall from each pellet during contact? (e) Why is this average force so different from the average force calculated in (c)? 113 A railroad car moves under a grain elevator at a constant speed of 3.20 m/s. Grain drops into the car at the rate of 540 kg/min. What is the magnitude of the force needed to keep the car moving at constant speed if friction is negligible?

256

CHAPTE R 9 CE NTE R OF MASS AN D LI N EAR M OM E NTU M

114 Figure 9-82 shows a uniform square plate of edge length 6d ! 6.0 m from which a square piece of edge length 2d has been removed. What are (a) the x coordinate and (b) the y coordinate of the center of mass of the remaining piece? y

3d

2d d 0

2d

3d 3d

Figure 9-82 Problem 114.

x

d

3d

115 SSM At time t ! 0, force ! (%4.00iˆ & 5.00jˆ ) N acts on an initially stationary particle of mass 2.00 $ 10%3 kg and force : F2 ! (2.00iˆ % 4.00jˆ ) N acts on an initially stationary particle of mass 4.00 $ 10%3 kg. From time t ! 0 to t ! 2.00 ms, what are the (a) magnitude and (b) angle (relative to the positive direction of the x axis) of the displacement of the center of mass of the twoparticle system? (c) What is the kinetic energy of the center of mass at t ! 2.00 ms? : F1

116 Two particles P and Q are released from rest 1.0 m apart. P has a mass of 0.10 kg, and Q a mass of 0.30 kg. P and Q attract each other with a constant force of 1.0 $ 10%2 N. No external forces act on the system. (a) What is the speed of the center of mass of P and Q when the separation is 0.50 m? (b) At what distance from P’s original position do the particles collide? 117 A collision occurs between a 2.00 kg particle traveling with velocity : v 1 ! (%4.00 m/s)iˆ & (%5.00 m/s)jˆ and a 4.00 kg particle traveling with velocity : v 2 ! (6.00 m/s)iˆ &(%2.00 m/s)jˆ. The collision connects the two particles. What then is their velocity in (a) unit-vector notation and as a (b) magnitude and (c) angle? 118 In the two-sphere arrangement of Fig. 9-20, assume that sphere 1 has a mass of 50 g and an initial height of h1 ! 9.0 cm, and that sphere 2 has a mass of 85 g. After sphere 1 is released and collides elastically with sphere 2, what height is reached by (a) sphere 1 and (b) sphere 2? After the next (elastic) collision, what height is reached by (c) sphere 1 and (d) sphere 2? (Hint: Do not use rounded-off values.) 119 In Fig. 9-83, block 1 slides along 1 2 an x axis on a frictionless floor with a x speed of 0.75 m/s. When it reaches sta- –1.50 m 0 tionary block 2, the two blocks undergo an elastic collision. The following table Figure 9-83 Problem 119. gives the mass and length of the (uniform) blocks and also the locations of their centers at time t ! 0. Where is the center of mass of the two-block system located (a) at t ! 0, (b) when the two blocks first touch, and (c) at t ! 4.0 s? Block

Mass (kg)

Length (cm)

Center at t ! 0

1 2

0.25 0.50

5.0 6.0

x ! %1.50 m x!0

120 A body is traveling at 2.0 m/s along the positive direction of an x axis; no net force acts on the body. An internal explosion sepa-

rates the body into two parts, each of 4.0 kg, and increases the total kinetic energy by 16 J. The forward part continues to move in the original direction of motion. What are the speeds of (a) the rear part and (b) the forward part? 121 An electron undergoes a one-dimensional elastic collision with an initially stationary hydrogen atom. What percentage of the electron’s initial kinetic energy is transferred to kinetic energy of the hydrogen atom? (The mass of the hydrogen atom is 1840 times the mass of the electron.) 122 A man (weighing 915 N) stands on a long railroad flatcar (weighing 2415 N) as it rolls at 18.2 m/s in the positive direction of an x axis, with negligible friction. Then the man runs along the flatcar in the negative x direction at 4.00 m/s relative to the flatcar. What is the resulting increase in the speed of the flatcar? 123 An unmanned space probe (of mass m and speed v relative to the Sun) approaches the planet Jupiter (of mass M and speed VJ relative to the Sun) as shown in Fig. 9-84. The spacecraft rounds the planet and departs in the opposite direction. What is its speed (in kilometers per second), relative to the Sun, after this slingshot encounter, which can be analyzed as a collision? Assume v ! 10.5 km/s and VJ ! 13.0 km/s (the orbital speed of Jupiter).The mass of Jupiter is very much greater than the mass of the spacecraft (M . m).

VJ

M

m v

Figure 9-84 Problem 123. 124 A 0.550 kg ball falls directly down onto concrete, hitting it with a speed of 12.0 m/s and rebounding directly upward with a speed of 3.00 m/s. Extend a y axis upward. In unit-vector notation, what are (a) the change in the ball’s momentum, (b) the impulse on the ball, and (c) the impulse on the concrete? 125 An atomic nucleus at rest at the origin of an xy coordinate system transforms into three particles. Particle 1, mass 16.7 $ 10%27 kg, moves away from the origin at velocity (6.00 $ 106 m/s)iˆ; particle 2, mass 8.35 $ 10%27 kg, moves away at velocity (%8.00 $ 106 m/s)jˆ. (a) In unit-vector notation, what is the linear momentum of the third particle, mass 11.7 $ 10%27 kg? (b) How much kinetic energy appears in this transformation? 126 Particle 1 of mass 200 g and speed 3.00 m/s undergoes a onedimensional collision with stationary particle 2 of mass 400 g. What is the magnitude of the impulse on particle 1 if the collision is (a) elastic and (b) completely inelastic? 127 During a lunar mission, it is necessary to increase the speed of a spacecraft by 2.2 m/s when it is moving at 400 m/s relative to the Moon. The speed of the exhaust products from the rocket engine is 1000 m/s relative to the spacecraft. What fraction of the initial mass of the spacecraft must be burned and ejected to accomplish the speed increase? 128 A cue stick strikes a stationary pool ball, with an average force of 32 N over a time of 14 ms. If the ball has mass 0.20 kg, what speed does it have just after impact?

C

H

A

P

T

E

R

1

0

Rotation 10-1 ROTATIONAL VARIABLES Learning Objectives After reading this module, you should be able to . . .

10.01 Identify that if all parts of a body rotate around a fixed axis locked together, the body is a rigid body. (This chapter is about the motion of such bodies.) 10.02 Identify that the angular position of a rotating rigid body is the angle that an internal reference line makes with a fixed, external reference line. 10.03 Apply the relationship between angular displacement and the initial and final angular positions. 10.04 Apply the relationship between average angular velocity, angular displacement, and the time interval for that displacement. 10.05 Apply the relationship between average angular acceleration, change in angular velocity, and the time interval for that change. 10.06 Identify that counterclockwise motion is in the positive direction and clockwise motion is in the negative direction. 10.07 Given angular position as a function of time, calculate the instantaneous angular velocity at any particular time and the average angular velocity between any two particular times.

10.08 Given a graph of angular position versus time, determine the instantaneous angular velocity at a particular time and the average angular velocity between any two particular times. 10.09 Identify instantaneous angular speed as the magnitude of the instantaneous angular velocity. 10.10 Given angular velocity as a function of time, calculate the instantaneous angular acceleration at any particular time and the average angular acceleration between any two particular times. 10.11 Given a graph of angular velocity versus time, determine the instantaneous angular acceleration at any particular time and the average angular acceleration between any two particular times. 10.12 Calculate a body’s change in angular velocity by integrating its angular acceleration function with respect to time. 10.13 Calculate a body’s change in angular position by integrating its angular velocity function with respect to time.

Key Ideas ● To describe the rotation of a rigid body about a fixed axis,

called the rotation axis, we assume a reference line is fixed in the body, perpendicular to that axis and rotating with the body. We measure the angular position u of this line relative to a fixed direction. When u is measured in radians, s (radian measure), u! r where s is the arc length of a circular path of radius r and angle u.

vavg !

"u . "t

● Radian measure is related to angle measure in revolutions

The (instantaneous) angular velocity v of the body is du . v! dt Both vavg and v are vectors, with directions given by a right-hand rule. They are positive for counterclockwise rotation and negative for clockwise rotation. The magnitude of the body’s angular velocity is the angular speed.

and degrees by

● If the angular velocity of a body changes from v1 to v2 in a

1 rev ! 360# ! 2p rad. ● A body that rotates about a rotation axis, changing its angu-

lar position from u1 to u2, undergoes an angular displacement "u ! u2 % u1, where "u is positive for counterclockwise rotation and negative for clockwise rotation. ● If a body rotates through an angular displacement "u in a

time interval "t, its average angular velocity vavg is

time interval "t ! t2 % t1, the average angular acceleration aavg of the body is aavg !

v 2 % v1 "v ! . t2 % t1 "t

The (instantaneous) angular acceleration a of the body is dv . a! dt Both aavg and a are vectors. 257

CHAPTE R 10 ROTATION

What Is Physics? As we have discussed, one focus of physics is motion. However, so far we have examined only the motion of translation, in which an object moves along a straight or curved line, as in Fig. 10-1a. We now turn to the motion of rotation, in which an object turns about an axis, as in Fig. 10-1b. You see rotation in nearly every machine, you use it every time you open a beverage can with a pull tab, and you pay to experience it every time you go to an amusement park. Rotation is the key to many fun activities, such as hitting a long drive in golf (the ball needs to rotate in order for the air to keep it aloft longer) and throwing a curveball in baseball (the ball needs to rotate in order for the air to push it left or right). Rotation is also the key to more serious matters, such as metal failure in aging airplanes. We begin our discussion of rotation by defining the variables for the motion, just as we did for translation in Chapter 2. As we shall see, the variables for rotation are analogous to those for one-dimensional motion and, as in Chapter 2, an important special situation is where the acceleration (here the rotational acceleration) is constant. We shall also see that Newton’s second law can be written for rotational motion, but we must use a new quantity called torque instead of just force. Work and the work–kinetic energy theorem can also be applied to rotational motion, but we must use a new quantity called rotational inertia instead of just mass. In short, much of what we have discussed so far can be applied to rotational motion with, perhaps, a few changes. Caution: In spite of this repetition of physics ideas, many students find this and the next chapter very challenging. Instructors have a variety of reasons as to why, but two reasons stand out: (1) There are a lot of symbols (with Greek

Elsa/Getty Images, Inc.

Mike Segar/Reuters/Landov LLC

258

(a)

(b)

Figure 10-1 Figure skater Sasha Cohen in motion of (a) pure translation in a fixed direction and (b) pure rotation about a vertical axis.

259

10-1 ROTATIONAL VAR IAB LES z Body This reference line is part of the body

Rotation axis

and perpendicular to the rotation axis. We use it to measure the rotation of the body relative to a fixed direction.

Reference line y O

x

Figure 10-2 A rigid body of arbitrary shape in pure rotation about the z axis of a coordinate system. The position of the reference line with respect to the rigid body is arbitrary, but it is perpendicular to the rotation axis. It is fixed in the body and rotates with the body.

letters) to sort out. (2) Although you are very familiar with linear motion (you can get across the room and down the road just fine), you are probably very unfamiliar with rotation (and that is one reason why you are willing to pay so much for amusement park rides). If a homework problem looks like a foreign language to you, see if translating it into the one-dimensional linear motion of Chapter 2 helps. For example, if you are to find, say, an angular distance, temporarily delete the word angular and see if you can work the problem with the Chapter 2 notation and ideas.

Rotational Variables We wish to examine the rotation of a rigid body about a fixed axis. A rigid body is a body that can rotate with all its parts locked together and without any change in its shape. A fixed axis means that the rotation occurs about an axis that does not move. Thus, we shall not examine an object like the Sun, because the parts of the Sun (a ball of gas) are not locked together. We also shall not examine an object like a bowling ball rolling along a lane, because the ball rotates about a moving axis (the ball’s motion is a mixture of rotation and translation). Figure 10-2 shows a rigid body of arbitrary shape in rotation about a fixed axis, called the axis of rotation or the rotation axis. In pure rotation (angular motion), every point of the body moves in a circle whose center lies on the axis of rotation, and every point moves through the same angle during a particular time interval. In pure translation (linear motion), every point of the body moves in a straight line, and every point moves through the same linear distance during a particular time interval. We deal now — one at a time — with the angular equivalents of the linear quantities position, displacement, velocity, and acceleration.

The body has rotated counterclockwise y by angle θ . This is the positive direction. e

nc re e f e Re lin

s r

θ

Angular Position Figure 10-2 shows a reference line, fixed in the body, perpendicular to the rotation axis and rotating with the body. The angular position of this line is the angle of the line relative to a fixed direction, which we take as the zero angular position. In Fig. 10-3, the angular position u is measured relative to the positive direction of the x axis. From geometry, we know that u is given by u!

s r

(radian measure).

(10-1)

Here s is the length of a circular arc that extends from the x axis (the zero angular position) to the reference line, and r is the radius of the circle.

x

Rotation axis

This dot means that the rotation axis is out toward you. Figure 10-3 The rotating rigid body of Fig. 10-2 in cross section, viewed from above. The plane of the cross section is perpendicular to the rotation axis, which now extends out of the page, toward you. In this position of the body, the reference line makes an angle u with the x axis.

260

CHAPTE R 10 ROTATION

An angle defined in this way is measured in radians (rad) rather than in revolutions (rev) or degrees. The radian, being the ratio of two lengths, is a pure number and thus has no dimension. Because the circumference of a circle of radius r is 2pr, there are 2p radians in a complete circle: 1 rev ! 360# ! and thus

2pr ! 2p rad, r

1 rad ! 57.3# ! 0.159 rev.

(10-2) (10-3)

We do not reset u to zero with each complete rotation of the reference line about the rotation axis. If the reference line completes two revolutions from the zero angular position, then the angular position u of the line is u ! 4p rad. For pure translation along an x axis, we can know all there is to know about a moving body if we know x(t), its position as a function of time. Similarly, for pure rotation, we can know all there is to know about a rotating body if we know u(t), the angular position of the body’s reference line as a function of time.

Angular Displacement If the body of Fig. 10-3 rotates about the rotation axis as in Fig. 10-4, changing the angular position of the reference line from u1 to u2, the body undergoes an angular displacement "u given by "u ! u2 % u1.

(10-4)

This definition of angular displacement holds not only for the rigid body as a whole but also for every particle within that body. Clocks Are Negative. If a body is in translational motion along an x axis, its displacement "x is either positive or negative, depending on whether the body is moving in the positive or negative direction of the axis. Similarly, the angular displacement "u of a rotating body is either positive or negative, according to the following rule: An angular displacement in the counterclockwise direction is positive, and one in the clockwise direction is negative.

The phrase “clocks are negative” can help you remember this rule (they certainly are negative when their alarms sound off early in the morning).

Checkpoint 1 A disk can rotate about its central axis like a merry-go-round. Which of the following pairs of values for its initial and final angular positions, respectively, give a negative angular displacement: (a) %3 rad, &5 rad, (b) %3 rad, %7 rad, (c) 7 rad, %3 rad?

Angular Velocity Suppose that our rotating body is at angular position u1 at time t1 and at angular position u2 at time t2 as in Fig. 10-4. We define the average angular velocity of the body in the time interval "t from t 1 to t 2 to be vavg !

u2 % u1 "u , ! t 2 % t1 "t

(10-5)

where "u is the angular displacement during "t (v is the lowercase omega).

10-1 ROTATIONAL VAR IAB LES y

Reference line At t2 ∆θ

O

θ1 θ2

At t1

This change in the angle of the reference line (which is part of the body) is equal to the angular displacement of the body itself during this time interval.

x

Rotation axis

Figure 10-4 The reference line of the rigid body of Figs. 10-2 and 10-3 is at angular position u1 at time t1 and at angular position u2 at a later time t2. The quantity "u (! u2 % u1) is the angular displacement that occurs during the interval "t (! t2 % t1). The body itself is not shown.

The (instantaneous) angular velocity v, with which we shall be most concerned, is the limit of the ratio in Eq. 10-5 as "t approaches zero. Thus, v ! lim

"t:0

"u du ! . "t dt

(10-6)

If we know u(t), we can find the angular velocity v by differentiation. Equations 10-5 and 10-6 hold not only for the rotating rigid body as a whole but also for every particle of that body because the particles are all locked together. The unit of angular velocity is commonly the radian per second (rad/s) or the revolution per second (rev/s). Another measure of angular velocity was used during at least the first three decades of rock: Music was produced by vinyl (phonograph) records that were played on turntables at “3313 rpm” or “45 rpm,” meaning at 3313 rev/min or 45 rev/min. If a particle moves in translation along an x axis, its linear velocity v is either positive or negative, depending on its direction along the axis. Similarly, the angular velocity v of a rotating rigid body is either positive or negative, depending on whether the body is rotating counterclockwise (positive) or clockwise (negative). (“Clocks are negative” still works.) The magnitude of an angular velocity is called the angular speed, which is also represented with v.

Angular Acceleration If the angular velocity of a rotating body is not constant, then the body has an angular acceleration. Let v 2 and v 1 be its angular velocities at times t 2 and t 1, respectively. The average angular acceleration of the rotating body in the interval from t1 to t2 is defined as aavg !

v2 % v1 "v ! , t2 % t1 "t

(10-7)

in which "v is the change in the angular velocity that occurs during the time interval "t. The (instantaneous) angular acceleration a, with which we shall be most concerned, is the limit of this quantity as "t approaches zero. Thus, a ! lim

"t:0

"v dv ! . "t dt

(10-8)

As the name suggests, this is the angular acceleration of the body at a given instant. Equations 10-7 and 10-8 also hold for every particle of that body. The unit of angular acceleration is commonly the radian per second-squared (rad/s2) or the revolution per second-squared (rev/s2).

261

262

CHAPTE R 10 ROTATION

Sample Problem 10.01 Angular velocity derived from angular position The disk in Fig. 10-5a is rotating about its central axis like a merry-go-round. The angular position u(t) of a reference line on the disk is given by

Calculations: To sketch the disk and its reference line at a particular time, we need to determine u for that time. To do so, we substitute the time into Eq. 10-9. For t ! %2.0 s, we get

u ! %1.00 % 0.600t & 0.250t 2,

u ! %1.00 % (0.600)(%2.0) & (0.250)(%2.0)2

(10-9)

with t in seconds, u in radians, and the zero angular position as indicated in the figure. (If you like, you can translate all this into Chapter 2 notation by momentarily dropping the word “angular” from “angular position” and replacing the symbol u with the symbol x. What you then have is an equation that gives the position as a function of time, for the onedimensional motion of Chapter 2.)

! 1.2 rad ! 1.2 rad

360# ! 69#. 20 rad

This means that at t ! %2.0 s the reference line on the disk is rotated counterclockwise from the zero position by angle 1.2 rad ! 69# (counterclockwise because u is positive). Sketch 1 in Fig. 10-5b shows this position of the reference line. Similarly, for t ! 0, we find u ! %1.00 rad ! %57#, which means that the reference line is rotated clockwise from the zero angular position by 1.0 rad, or 57#, as shown in sketch 3. For t ! 4.0 s, we find u ! 0.60 rad ! 34# (sketch 5). Drawing sketches for when the curve crosses the t axis is easy, because then u ! 0 and the reference line is momentarily aligned with the zero angular position (sketches 2 and 4).

(a) Graph the angular position of the disk versus time from t ! %3.0 s to t ! 5.4 s. Sketch the disk and its angular position reference line at t ! %2.0 s, 0 s, and 4.0 s, and when the curve crosses the t axis. KEY IDEA The angular position of the disk is the angular position u(t) of its reference line, which is given by Eq. 10-9 as a function of time t. So we graph Eq. 10-9; the result is shown in Fig. 10-5b.

(b) At what time tmin does u(t) reach the minimum value shown in Fig. 10-5b? What is that minimum value?

A This is a plot of the angle of the disk versus time.

θ (rad) Rotation axis Reference line

2

Zero angular position (a)

0

The angular position of the disk is the angle between these two lines.

–2

t (s)

–2

0

2

4

6

(b)

(1)

At t = −2 s, the disk is at a positive (counterclockwise) angle. So, a positive θ value is plotted.

(2)

Now, the disk is at a zero angle.

(3)

Now, it is at a negative (clockwise) angle. So, a negative θ value is plotted.

(4)

It has reversed its rotation and is again at a zero angle.

(5)

Now, it is back at a positive angle.

Figure 10-5 (a) A rotating disk. (b) A plot of the disk’s angular position u(t). Five sketches indicate the angular position of the reference line on the disk for five points on the curve. (c) A plot of the disk’s angular velocity v(t). Positive values of v correspond to counterclockwise rotation, and negative values to clockwise rotation.

10-1 ROTATIONAL VAR IAB LES

KEY IDEA To find the extreme value (here the minimum) of a function, we take the first derivative of the function and set the result to zero.

t ! %3.0 s to t ! 6.0 s. Sketch the disk and indicate the direction of turning and the sign of v at t ! %2.0 s, 4.0 s, and tmin. KEY IDEA From Eq. 10-6, the angular velocity v is equal to du/dt as given in Eq. 10-10. So, we have

Calculations: The first derivative of u(t) is du ! %0.600 & 0.500t. dt

(10-10)

Setting this to zero and solving for t give us the time at which u(t) is minimum: tmin ! 1.20 s.

(Answer)

To get the minimum value of u, we next substitute tmin into Eq. 10-9, finding u ! %1.36 rad % %77.9#. (Answer) This minimum of u(t) (the bottom of the curve in Fig. 10-5b) corresponds to the maximum clockwise rotation of the disk from the zero angular position, somewhat more than is shown in sketch 3. (c) Graph the angular velocity v of the disk versus time from

v ! %0.600 & 0.500t.

This is a plot of the angular velocity of the disk versus time.

2 0

–2

t (s)

–2

negative ω

0

2

zero ω (c)

4

positive ω

The angular velocity is initially negative and slowing, then momentarily zero during reversal, and then positive and increasing.

6

(10-11)

The graph of this function v(t) is shown in Fig. 10-5c. Because the function is linear, the plot is a straight line. The slope is 0.500 rad/s 2 and the intercept with the vertical axis (not shown) is %0.600 rad/s. Calculations: To sketch the disk at t ! %2.0 s, we substitute that value into Eq. 10-11, obtaining v ! %1.6 rad/s.

(Answer)

The minus sign here tells us that at t ! %2.0 s, the disk is turning clockwise (as indicated by the left-hand sketch in Fig. 10-5c). Substituting t ! 4.0 s into Eq. 10-11 gives us v ! 1.4 rad/s.

ω (rad/s)

263

(Answer)

The implied plus sign tells us that now the disk is turning counterclockwise (the right-hand sketch in Fig. 10-5c). For tmin, we already know that du/dt ! 0. So, we must also have v ! 0. That is, the disk momentarily stops when the reference line reaches the minimum value of u in Fig. 10-5b, as suggested by the center sketch in Fig. 10-5c. On the graph of v versus t in Fig. 10-5c, this momentary stop is the zero point where the plot changes from the negative clockwise motion to the positive counterclockwise motion. (d) Use the results in parts (a) through (c) to describe the motion of the disk from t ! %3.0 s to t ! 6.0 s. Description: When we first observe the disk at t ! %3.0 s, it has a positive angular position and is turning clockwise but slowing. It stops at angular position u ! %1.36 rad and then begins to turn counterclockwise, with its angular position eventually becoming positive again.

Additional examples, video, and practice available at WileyPLUS

264

CHAPTE R 10 ROTATION

Sample Problem 10.02 Angular velocity derived from angular acceleration A child’s top is spun with angular acceleration 3

a ! 5t % 4t, with t in seconds and a in radians per second-squared. At t ! 0, the top has angular velocity 5 rad/s, and a reference line on it is at angular position u ! 2 rad. (a) Obtain an expression for the angular velocity v(t) of the top.That is, find an expression that explicitly indicates how the angular velocity depends on time. (We can tell that there is such a dependence because the top is undergoing an angular acceleration, which means that its angular velocity is changing.) KEY IDEA By definition, a(t) is the derivative of v(t) with respect to time. Thus, we can find v(t) by integrating a(t) with respect to time. Calculations: Equation 10-8 tells us

"

so

dv ! a dt, dv !

v!

"

"

v ! 54 t 4 % 2t 2 & 5.

(Answer)

(b) Obtain an expression for the angular position u(t) of the top. KEY IDEA By definition, v(t) is the derivative of u(t) with respect to time. Therefore, we can find u(t) by integrating v(t) with respect to time. Calculations: Since Eq. 10-6 tells us that

u!

a dt.

(5t % 4t) dt !

so C ! 5 rad/s. Then

we can write

"

v dt !

"

du ! v dt, (54 t 4 % 2t 2 & 5) dt

! 14 t 5 % 23 t 3 & 5t & C/

From this we find

3

To evaluate the constant of integration C, we note that v ! 5 rad/s at t ! 0. Substituting these values in our expression for v yields 5 rad/s ! 0 % 0 & C ,

5 4 4t

%

4 2 2t

& C.

! 14 t 5 % 23 t 3 & 5t & 2,

(Answer)

where C/ has been evaluated by noting that u ! 2 rad at t ! 0.

Additional examples, video, and practice available at WileyPLUS

Are Angular Quantities Vectors? We can describe the position, velocity, and acceleration of a single particle by means of vectors. If the particle is confined to a straight line, however, we do not really need vector notation. Such a particle has only two directions available to it, and we can indicate these directions with plus and minus signs. In the same way, a rigid body rotating about a fixed axis can rotate only clockwise or counterclockwise as seen along the axis, and again we can select between the two directions by means of plus and minus signs. The question arises: “Can we treat the angular displacement, velocity, and acceleration of a rotating body as vectors?” The answer is a qualified “yes” (see the caution below, in connection with angular displacements). Angular Velocities. Consider the angular velocity. Figure 10-6a shows a vinyl record rotating on a turntable. The record has a constant angular speed v (! 3313 rev/min) in the clockwise direction. We can represent its angular ve: locity as a vector v pointing along the axis of rotation, as in Fig. 10-6b. Here’s how: We choose the length of this vector according to some convenient scale, for example, with 1 cm corresponding to 10 rev/min. Then we establish a direc: tion for the vector v by using a right-hand rule, as Fig. 10-6c shows: Curl your right hand about the rotating record, your fingers pointing in the direction of rotation. Your extended thumb will then point in the direction of the angular velocity vector. If the record were to rotate in the opposite sense, the right-

265

10-1 ROTATIONAL VAR IAB LES

Axis

z

Axis

z

Axis

z

Spindle

ω

This right-hand rule establishes the direction of the (a) angular velocity vector.

(b)

ω

(c)

Figure 10-6 (a) A record rotating about a vertical axis that coincides with the axis of the spindle. (b) The angular velocity of the rotating record can be represented by the vector : , lying along the axis and pointing down, as shown. (c) We establish the direction of the v angular velocity vector as downward by using a right-hand rule. When the fingers of the right hand curl around the record and point the way it is moving, the extended thumb : points in the direction of v .

y

y

PHY

P HYS ICS

z

z

y

y

The order of the rotations makes a big difference in the result.

PHYSICS

PHYSICS

x

z

x

SICS

x

PHYSICS

x

z y

y PH YS IC

PHYSICS

S

PHYSICS

hand rule would tell you that the angular velocity vector then points in the opposite direction. It is not easy to get used to representing angular quantities as vectors. We instinctively expect that something should be moving along the direction of a vector. That is not the case here. Instead, something (the rigid body) is rotating around the direction of the vector. In the world of pure rotation, a vector defines an axis of rotation, not a direction in which something moves. Nonetheless, the vector also defines the motion. Furthermore, it obeys all the rules for vector : manipulation discussed in Chapter 3. The angular acceleration a is another vector, and it too obeys those rules. In this chapter we consider only rotations that are about a fixed axis. For such situations, we need not consider vectors — we can represent angular velocity with v and angular acceleration with a, and we can indicate direction with an implied plus sign for counterclockwise or an explicit minus sign for clockwise. Angular Displacements. Now for the caution: Angular displacements (unless they are very small) cannot be treated as vectors. Why not? We can certainly give them both magnitude and direction, as we did for the angular velocity vector in Fig. 10-6. However, to be represented as a vector, a quantity must also obey the rules of vector addition, one of which says that if you add two vectors, the order in which you add them does not matter. Angular displacements fail this test. Figure 10-7 gives an example. An initially horizontal book is given two 90# angular displacements, first in the order of Fig. 10-7a and then in the order of Fig. 10-7b. Although the two angular displacements are identical, their order is not, and the book ends up with different orientations. Here’s another example. Hold your right arm downward, palm toward your thigh. Keeping your wrist rigid, (1) lift the arm forward until it is horizontal, (2) move it horizontally until it points toward the right, and (3) then bring it down to your side. Your palm faces forward. If you start over, but reverse the steps, which way does your palm end up facing? From either example, we must conclude that the addition of two angular displacements depends on their order and they cannot be vectors.

x

x z

z (a)

(b)

Figure 10-7 (a) From its initial position, at the top, the book is given two successive 90# rotations, first about the (horizontal) x axis and then about the (vertical) y axis. (b) The book is given the same rotations, but in the reverse order.

266

CHAPTE R 10 ROTATION

10-2 ROTATION WITH CONSTANT ANGULAR ACCELERATION Learning Objective After reading this module, you should be able to . . .

10.14 For constant angular acceleration, apply the relationships between angular position, angular displacement,

angular velocity, angular acceleration, and elapsed time (Table 10-1).

Key Idea ● Constant angular acceleration (a ! constant) is an important special case of rotational motion. The appropriate kinematic equations are v ! v0 & at,

u % u0 ! v0t & 12 at 2, v2 ! v 20 & 2a(u % u0), u % u0 ! 12 (v0 & v)t, u % u0 ! vt % 12 at 2.

Rotation with Constant Angular Acceleration In pure translation, motion with a constant linear acceleration (for example, that of a falling body) is an important special case. In Table 2-1, we displayed a series of equations that hold for such motion. In pure rotation, the case of constant angular acceleration is also important, and a parallel set of equations holds for this case also. We shall not derive them here, but simply write them from the corresponding linear equations, substituting equivalent angular quantities for the linear ones. This is done in Table 10-1, which lists both sets of equations (Eqs. 2-11 and 2-15 to 2-18; 10-12 to 10-16). Recall that Eqs. 2-11 and 2-15 are basic equations for constant linear acceleration — the other equations in the Linear list can be derived from them. Similarly, Eqs. 10-12 and 10-13 are the basic equations for constant angular acceleration, and the other equations in the Angular list can be derived from them. To solve a simple problem involving constant angular acceleration, you can usually use an equation from the Angular list (if you have the list). Choose an equation for which the only unknown variable will be the variable requested in the problem. A better plan is to remember only Eqs. 10-12 and 10-13, and then solve them as simultaneous equations whenever needed.

Checkpoint 2 In four situations, a rotating body has angular position u(t) given by (a) u ! 3t % 4, (b) u ! %5t 3 & 4t 2 & 6, (c) u ! 2/t 2 % 4/t, and (d) u ! 5t 2 % 3. To which situations do the angular equations of Table 10-1 apply?

Table 10-1 Equations of Motion for Constant Linear Acceleration and for Constant Angular Acceleration Equation Number

Linear Equation

(2-11) (2-15) (2-16) (2-17) (2-18)

v ! v0 & at x % x0 ! v0 t & 12 at 2 v2 ! v20 & 2a(x % x0) x % x0 ! 12(v0 & v)t x % x0 ! vt % 12 at 2

Missing Variable x % x0 v t a v0

u % u0 v t a v0

Angular Equation

Equation Number

v ! v0 & at u % u0 ! v0t & 12 at 2 v2 ! v20 & 2a(u % u0) u % u0 ! 12(v0 & v)t u % u0 ! vt % 12at 2

(10-12) (10-13) (10-14) (10-15) (10-16)

267

10-2 ROTATION WITH CONSTANT ANG U L AR ACCE LE RATION

Sample Problem 10.03 Constant angular acceleration, grindstone A grindstone (Fig. 10-8) rotates at constant angular acceleration a ! 0.35 rad/s2. At time t ! 0, it has an angular velocity of v0 ! %4.6 rad/s and a reference line on it is horizontal, at the angular position u0 ! 0.

(We converted 5.0 rev to 10p rad to keep the units consistent.) Solving this quadratic equation for t, we find t ! 32 s.

(Answer)

KEY IDEA

Now notice something a bit strange. We first see the wheel when it is rotating in the negative direction and through the u ! 0 orientation. Yet, we just found out that 32 s later it is at the positive orientation of u ! 5.0 rev. What happened in that time interval so that it could be at a positive orientation?

The angular acceleration is constant, so we can use the rotation equations of Table 10-1. We choose Eq. 10-13,

(b) Describe the grindstone’s rotation between t ! 0 and t ! 32 s.

u % u0 ! v0 t & 12 at 2,

Description: The wheel is initially rotating in the negative (clockwise) direction with angular velocity v0 ! %4.6 rad/s, but its angular acceleration a is positive. This initial opposition of the signs of angular velocity and angular acceleration means that the wheel slows in its rotation in the negative direction, stops, and then reverses to rotate in the positive direction. After the reference line comes back through its initial orientation of u ! 0, the wheel turns an additional 5.0 rev by time t ! 32 s.

(a) At what time after t ! 0 is the reference line at the angular position u ! 5.0 rev?

because the only unknown variable it contains is the desired time t. Calculations: Substituting known values and setting u0 ! 0 and u ! 5.0 rev ! 10p rad give us 10p rad ! (%4.6 rad/s)t & 12 (0.35 rad/s2)t 2. We measure rotation by using this reference line. Clockwise = negative Counterclockwise = positive Axis

Zero angular position

Reference line

Figure 10-8 A grindstone. At t ! 0 the reference line (which we imagine to be marked on the stone) is horizontal.

(c) At what time t does the grindstone momentarily stop? Calculation: We again go to the table of equations for constant angular acceleration, and again we need an equation that contains only the desired unknown variable t. However, now the equation must also contain the variable v, so that we can set it to 0 and then solve for the corresponding time t. We choose Eq. 10-12, which yields t!

v % v0 0 % (%4.6 rad/s) ! 13 s. ! a 0.35 rad/s2

(Answer)

Sample Problem 10.04 Constant angular acceleration, riding a Rotor While you are operating a Rotor (a large, vertical, rotating cylinder found in amusement parks), you spot a passenger in acute distress and decrease the angular velocity of the cylinder from 3.40 rad/s to 2.00 rad/s in 20.0 rev, at constant angular acceleration. (The passenger is obviously more of a “translation person” than a “rotation person.”) (a) What is the constant angular acceleration during this decrease in angular speed?

rad/s, the angular displacement is u % u0 ! 20.0 rev, and the angular velocity at the end of that displacement is v ! 2.00 rad/s. In addition to the angular acceleration a that we want, both basic equations also contain time t, which we do not necessarily want. To eliminate the unknown t, we use Eq. 10-12 to write v % v0 t! , a which we then substitute into Eq. 10-13 to write

KEY IDEA Because the cylinder’s angular acceleration is constant, we can relate it to the angular velocity and angular displacement via the basic equations for constant angular acceleration (Eqs. 10-12 and 10-13). Calculations: Let’s first do a quick check to see if we can solve the basic equations. The initial angular velocity is v0 ! 3.40

u % u0 ! v0

# v %a v $ & a# v %a v $ . 0

1 2

0

2

Solving for a, substituting known data, and converting 20 rev to 125.7 rad, we find a!

(2.00 rad/s)2 % (3.40 rad/s)2 v 2 % v 20 ! 2(u % u0) 2(125.7 rad)

! %0.0301 rad/s2.

(Answer)

268

CHAPTE R 10 ROTATION

(b) How much time did the speed decrease take? Calculation: Now that we know a, we can use Eq. 10-12 to solve for t:

t!

2.00 rad/s % 3.40 rad/s v % v0 ! a %0.0301 rad/s2

! 46.5 s.

(Answer)

Additional examples, video, and practice available at WileyPLUS

10-3 RELATING THE LINEAR AND ANGULAR VARIABLES Learning Objectives After reading this module, you should be able to . . .

10.15 For a rigid body rotating about a fixed axis, relate the angular variables of the body (angular position, angular velocity, and angular acceleration) and the linear variables of a particle on the body (position, velocity, and acceleration) at any given radius.

10.16 Distinguish between tangential acceleration and radial acceleration, and draw a vector for each in a sketch of a particle on a body rotating about an axis, for both an increase in angular speed and a decrease.

Key Ideas ● A point in a rigid rotating body, at a perpendicular distance

r from the rotation axis, moves in a circle with radius r. If the body rotates through an angle u, the point moves along an arc with length s given by s ! ur

(radian measure),

where u is in radians. : ● The linear velocity v of the point is tangent to the circle; the point’s linear speed v is given by v ! vr

(radian measure),

where v is the angular speed (in radians per second) of the body, and thus also the point.

● The linear acceleration a

of the point has both tangential and radial components. The tangential component is at ! ar (radian measure), where a is the magnitude of the angular acceleration (in radians per second-squared) of the body. The radial component of : a is v2 ar ! ! v2r (radian measure). r ● If the point moves in uniform circular motion, the period T of the motion for the point and the body is 2p 2pr T! ! (radian measure). v v :

Relating the Linear and Angular Variables In Module 4-5, we discussed uniform circular motion, in which a particle travels at constant linear speed v along a circle and around an axis of rotation. When a rigid body, such as a merry-go-round, rotates around an axis, each particle in the body moves in its own circle around that axis. Since the body is rigid, all the particles make one revolution in the same amount of time; that is, they all have the same angular speed v. However, the farther a particle is from the axis, the greater the circumference of its circle is, and so the faster its linear speed v must be. You can notice this on a merry-go-round. You turn with the same angular speed v regardless of your distance from the center, but your linear speed v increases noticeably if you move to the outside edge of the merry-go-round. We often need to relate the linear variables s, v, and a for a particular point in a rotating body to the angular variables u, v, and a for that body. The two sets of variables are related by r, the perpendicular distance of the point from the rotation axis. This perpendicular distance is the distance between the point and the rotation axis, measured along a perpendicular to the axis. It is also the radius r of the circle traveled by the point around the axis of rotation.

10-3 R E L ATI NG TH E LI N EAR AN D ANG U L AR VAR IAB LES

The Position If a reference line on a rigid body rotates through an angle u, a point within the body at a position r from the rotation axis moves a distance s along a circular arc, where s is given by Eq. 10-1: s ! ur

(radian measure).

(10-17)

This is the first of our linear – angular relations. Caution: The angle u here must be measured in radians because Eq. 10-17 is itself the definition of angular measure in radians.

The Speed

The velocity vector is always tangent to this circle around the rotation axis.

y Circle traveled by P

269

v

P

r

x Rotation axis

Differentiating Eq. 10-17 with respect to time — with r held constant — leads to du ds ! r. dt dt However, ds/dt is the linear speed (the magnitude of the linear velocity) of the point in question, and du/dt is the angular speed v of the rotating body. So v ! vr

(radian measure).

2pr . v

(10-19)

This equation tells us that the time for one revolution is the distance 2pr traveled in one revolution divided by the speed at which that distance is traveled. Substituting for v from Eq. 10-18 and canceling r, we find also that T!

2p v

(radian measure).

(10-20)

This equivalent equation says that the time for one revolution is the angular distance 2p rad traveled in one revolution divided by the angular speed (or rate) at which that angle is traveled.

The Acceleration Differentiating Eq. 10-18 with respect to time — again with r held constant — leads to dv dv ! r. (10-21) dt dt Here we run up against a complication. In Eq. 10-21, dv/dt represents only the part of the linear acceleration that is responsible for changes in the magnitude v v . Like : of the linear velocity : v , that part of the linear acceleration is tangent to the path of the point in question. We call it the tangential component at of the linear acceleration of the point, and we write at ! ar

(radian measure),

y

(10-18)

Caution: The angular speed v must be expressed in radian measure. Equation 10-18 tells us that since all points within the rigid body have the same angular speed v, points with greater radius r have greater linear speed v. Figure 10-9a reminds us that the linear velocity is always tangent to the circular path of the point in question. If the angular speed v of the rigid body is constant, then Eq. 10-18 tells us that the linear speed v of any point within it is also constant. Thus, each point within the body undergoes uniform circular motion. The period of revolution T for the motion of each point and for the rigid body itself is given by Eq. 4-35: T!

(a)

(10-22)

The acceleration always has a radial (centripetal) component and may have a tangential component. at ar

P x

Rotation axis

(b)

Figure 10-9 The rotating rigid body of Fig. 10-2, shown in cross section viewed from above. Every point of the body (such as P) moves in a circle around the rotation axis. (a) The linear velocity : v of every point is tangent to the circle in which the point moves. (b) The linear acceleration : a of the point has (in general) two components: tangential at and radial ar.

270

CHAPTE R 10 ROTATION

where a ! dv/dt. Caution: The angular acceleration a in Eq. 10-22 must be expressed in radian measure. In addition, as Eq. 4-34 tells us, a particle (or point) moving in a circular path has a radial component of linear acceleration, ar ! v2/r (directed radially inward), that is responsible for changes in the direction of the linear velocity : v . By substituting for v from Eq. 10-18, we can write this component as ar !

v2 ! v2 r r

(10-23)

(radian measure).

Thus, as Fig. 10-9b shows, the linear acceleration of a point on a rotating rigid body has, in general, two components. The radially inward component ar (given by Eq. 10-23) is present whenever the angular velocity of the body is not zero. The tangential component at (given by Eq. 10-22) is present whenever the angular acceleration is not zero.

Checkpoint 3 A cockroach rides the rim of a rotating merry-go-round. If the angular speed of this system (merry-go-round & cockroach) is constant, does the cockroach have (a) radial acceleration and (b) tangential acceleration? If v is decreasing, does the cockroach have (c) radial acceleration and (d) tangential acceleration?

Sample Problem 10.05 Designing The Giant Ring, a large-scale amusement park ride We are given the job of designing a large horizontal ring that will rotate around a vertical axis and that will have a radius of r ! 33.1 m (matching that of Beijing’s The Great Observation Wheel, the largest Ferris wheel in the world). Passengers will enter through a door in the outer wall of the ring and then stand next to that wall (Fig. 10-10a). We decide that for the time interval t ! 0 to t ! 2.30 s, the angular position u(t) of a reference line on the ring will be given by u ! ct3,

(10-24)

with c ! 6.39 $ 10%2 rad/s3. After t ! 2.30 s, the angular speed will be held constant until the end of the ride. Once the ring begins to rotate, the floor of the ring will drop away from the riders but the riders will not fall—indeed, they feel as though they are pinned to the wall. For the time t ! 2.20 s, let’s determine a rider’s angular speed v, linear speed v, angular acceleration a, tangential acceleration at, radial acceleration ar, and acceleration : a. KEY IDEAS (1) The angular speed v is given by Eq. 10-6 (v ! du/dt). (2) The linear speed v (along the circular path) is related to the angular speed (around the rotation axis) by Eq. 10-18 (v ! vr). (3) The angular acceleration a is given by Eq. 10-8 (a ! dv/dt). (4) The tangential acceleration at (along the circular path) is related to the angular acceleration (around the rotation axis) by Eq. 10-22 (at ! ar). (5) The radial acceleration ar is given Eq. 10-23 (ar ! v2r). (6) The tangential

a

Figure 10-10 (a) Overhead view of a passenger ready to ride The Giant Ring. (b) The radial and tangential acceleration components of the (full) acceleration.

at

u ar (a)

(b)

and radial accelerations are the (perpendicular) components of the (full) acceleration : a. Calculations: Let’s go through the steps. We first find the angular velocity by taking the time derivative of the given angular position function and then substituting the given time of t ! 2.20 s: d du (ct3) ! 3ct2 ! dt dt ! 3(6.39 $ 10%2 rad/s3)(2.20 s)2 ! 0.928 rad/s.

v!

(10-25) (Answer)

From Eq. 10-18, the linear speed just then is v ! vr ! 3ct2r ! 3(6.39 $ 10%2 rad/s3)(2.20 s)2(33.1 m) ! 30.7 m/s.

(10-26) (Answer)

271

10-4 KI N ETIC E N E RGY OF ROTATION

Although this is fast (111 km/h or 68.7 mi/h), such speeds are common in amusement parks and not alarming because (as mentioned in Chapter 2) your body reacts to accelerations but not to velocities. (It is an accelerometer, not a speedometer.) From Eq. 10-26 we see that the linear speed is increasing as the square of the time (but this increase will cut off at t ! 2.30 s). Next, let’s tackle the angular acceleration by taking the time derivative of Eq. 10-25: d dv a! (3ct2) ! 6ct ! dt dt ! 6(6.39 $ 10%2 rad/s3)(2.20 s) ! 0.843 rad/s2. (Answer) The tangential acceleration then follows from Eq. 10-22: at ! ar ! 6ctr

(10-27)

! 6(6.39 $ 10%2 rad/s3)(2.20 s)(33.1 m) ! 27.91 m/s2 % 27.9 m/s2,

(Answer)

or 2.8g (which is reasonable and a bit exciting). Equation 10-27 tells us that the tangential acceleration is increasing with time (but it will cut off at t ! 2.30 s). From Eq. 10-23, we write the radial acceleration as ar ! v2r. Substituting from Eq. 10-25 leads us to ar ! (3ct2)2r ! 9c2t4r ! 9(6.39 $ 10

%2

(10-28) 3 2

4

rad/s ) (2.20 s) (33.1 m)

! 28.49 m/s % 28.5 m/s2, 2

(Answer)

or 2.9g (which is also reasonable and a bit exciting).

The radial and tangential accelerations are perpendicular to each other and form the components of the rider’s acceleration : a (Fig. 10-10b). The magnitude of : a is given by a ! 2a 2r & a 2t

(10-29)

% 39.9 m/s2,

(Answer)

! 2(28.49 m/s2)2 & (27.91 m/s2)2

or 4.1g (which is really exciting!). All these values are acceptable. To find the orientation of : a, we can calculate the angle u shown in Fig. 10-10b: a tan u ! t . ar However, instead of substituting our numerical results, let’s use the algebraic results from Eqs. 10-27 and 10-28: u ! tan%1

# 9c6ctrt r $ ! tan # 3ct2 $. 2 4

%1

3

(10-30)

The big advantage of solving for the angle algebraically is that we can then see that the angle (1) does not depend on the ring’s radius and (2) decreases as t goes from 0 to 2.20 s. That is, the acceleration vector : a swings toward being radially inward because the radial acceleration (which depends on t4) quickly dominates over the tangential acceleration (which depends on only t).At our given time t ! 2.20 s, we have u ! tan %1

2 ! 44.4#. (Answer) 3(6.39 $ 10 rad/s3)(2.20 s)3 %2

Additional examples, video, and practice available at WileyPLUS

10-4 KINETIC ENERGY OF ROTATION Learning Objectives After reading this module, you should be able to . . .

10.17 Find the rotational inertia of a particle about a point. 10.18 Find the total rotational inertia of many particles moving around the same fixed axis.

10.19 Calculate the rotational kinetic energy of a body in terms of its rotational inertia and its angular speed.

Key Idea ● The kinetic energy K of a rigid body rotating about a fixed axis is given by K ! 12Iv2 (radian measure),

in which I is the rotational inertia of the body, defined as I ! ' mir 2i for a system of discrete particles.

Kinetic Energy of Rotation The rapidly rotating blade of a table saw certainly has kinetic energy due to that rotation. How can we express the energy? We cannot apply the familiar formula K ! 12 mv 2 to the saw as a whole because that would give us the kinetic energy only of the saw’s center of mass, which is zero.

272

CHAPTE R 10 ROTATION

Instead, we shall treat the table saw (and any other rotating rigid body) as a collection of particles with different speeds. We can then add up the kinetic energies of all the particles to find the kinetic energy of the body as a whole. In this way we obtain, for the kinetic energy of a rotating body,

Rod is easy to rotate this way. (a)

K ! 12 m1v 21 & 12 m2v22 & 12 m3v 23 & ) ) ) ! ' 12 miv 2i ,

Rotation axis

Harder this way. (b)

Figure 10-11 A long rod is much easier to rotate about (a) its central (longitudinal) axis than about (b) an axis through its center and perpendicular to its length. The reason for the difference is that the mass is distributed closer to the rotation axis in (a) than in (b).

(10-31)

in which mi is the mass of the ith particle and vi is its speed. The sum is taken over all the particles in the body. The problem with Eq. 10-31 is that vi is not the same for all particles. We solve this problem by substituting for v from Eq. 10-18 (v ! vr), so that we have K ! ' 12 mi(vri)2 ! 12 #' mi r 2i $v2,

(10-32)

in which v is the same for all particles. The quantity in parentheses on the right side of Eq. 10-32 tells us how the mass of the rotating body is distributed about its axis of rotation. We call that quantity the rotational inertia (or moment of inertia) I of the body with respect to the axis of rotation. It is a constant for a particular rigid body and a particular rotation axis. (Caution: That axis must always be specified if the value of I is to be meaningful.) We may now write I ! ' mi r 2i

(rotational inertia)

(10-33)

and substitute into Eq. 10-32, obtaining K ! 12 I12

(radian measure)

(10-34)

as the expression we seek. Because we have used the relation v ! vr in deriving Eq. 10-34, v must be expressed in radian measure. The SI unit for I is the kilogram – square meter (kg )m2). The Plan. If we have a few particles and a specified rotation axis, we find mr2 for each particle and then add the results as in Eq. 10-33 to get the total rotational inertia I. If we want the total rotational kinetic energy, we can then substitute that I into Eq. 10-34. That is the plan for a few particles, but suppose we have a huge number of particles such as in a rod. In the next module we shall see how to handle such continuous bodies and do the calculation in only a few minutes. Equation 10-34, which gives the kinetic energy of a rigid body in pure rotation, is the angular equivalent of the formula K ! 12 Mv2com, which gives the kinetic energy of a rigid body in pure translation. In both formulas there is a factor of 12. Where mass M appears in one equation, I (which involves both mass and its distribution) appears in the other. Finally, each equation contains as a factor the square of a speed — translational or rotational as appropriate. The kinetic energies of translation and of rotation are not different kinds of energy. They are both kinetic energy, expressed in ways that are appropriate to the motion at hand. We noted previously that the rotational inertia of a rotating body involves not only its mass but also how that mass is distributed. Here is an example that you can literally feel. Rotate a long, fairly heavy rod (a pole, a length of lumber, or something similar), first around its central (longitudinal) axis (Fig. 10-11a) and then around an axis perpendicular to the rod and through the center (Fig. 10-11b). Both rotations involve the very same mass, but the first rotation is much easier than the second. The reason is that the mass is distributed much closer to the rotation axis in the first rotation. As a result, the rotational inertia of the rod is much smaller in Fig. 10-11a than in Fig. 10-11b. In general, smaller rotational inertia means easier rotation.

10-5 CALCU L ATI NG TH E ROTATIONAL I N E RTIA

Checkpoint 4 The figure shows three small spheres that rotate about a vertical axis. The perpendicular distance between the axis and the center of each sphere is given. Rank the three spheres according to their rotational inertia about that axis, greatest first.

1m Rotation axis

273

36 kg

2m

9 kg

3m 4 kg

10-5 CALCULATING THE ROTATIONAL INERTIA Learning Objectives After reading this module, you should be able to . . .

10.20 Determine the rotational inertia of a body if it is given in Table 10-2. 10.21 Calculate the rotational inertia of a body by integration over the mass elements of the body.

10.22 Apply the parallel-axis theorem for a rotation axis that is displaced from a parallel axis through the center of mass of a body.

Key Ideas ● I is the rotational inertia of the body, defined as

I!' for a system of discrete particles and defined as I!

"

● The parallel-axis theorem relates the rotational inertia I of a

mir 2i

body about any axis to that of the same body about a parallel axis through the center of mass:

r 2 dm

I ! Icom & Mh2.

for a body with continuously distributed mass. The r and ri in these expressions represent the perpendicular distance from the axis of rotation to each mass element in the body, and the integration is carried out over the entire body so as to include every mass element.

Here h is the perpendicular distance between the two axes, and Icom is the rotational inertia of the body about the axis through the com. We can describe h as being the distance the actual rotation axis has been shifted from the rotation axis through the com.

Calculating the Rotational Inertia If a rigid body consists of a few particles, we can calculate its rotational inertia about a given rotation axis with Eq. 10-33 (I ! ' mi r 2i ); that is, we can find the product mr 2 for each particle and then sum the products. (Recall that r is the perpendicular distance a particle is from the given rotation axis.) If a rigid body consists of a great many adjacent particles (it is continuous, like a Frisbee), using Eq. 10-33 would require a computer. Thus, instead, we replace the sum in Eq. 10-33 with an integral and define the rotational inertia of the body as I!

"

r 2 dm

(rotational inertia, continuous body).

(10-35)

Table 10-2 gives the results of such integration for nine common body shapes and the indicated axes of rotation.

Parallel-Axis Theorem Suppose we want to find the rotational inertia I of a body of mass M about a given axis. In principle, we can always find I with the integration of Eq. 10-35. However, there is a neat shortcut if we happen to already know the rotational inertia Icom of the body about a parallel axis that extends through the body’s center of mass. Let h be the perpendicular distance between the given axis and the axis

274

CHAPTE R 10 ROTATION

Table 10-2 Some Rotational Inertias Axis

Axis Hoop about central axis

R

Axis

Annular cylinder (or ring) about central axis

R1

Solid cylinder (or disk) about central axis

R2

L R

(a )

I = MR2

(b )

I = _12_M ( R 12 + R 22 ) Axis

Axis

(c )

I = _12_M R 2 Axis

Thin rod about axis through center perpendicular to length

Solid cylinder (or disk) about central diameter

Solid sphere about any diameter 2R

L

L R I = _14_M R 2 +

1 __ 12

(d)

ML2

I=

(e )

1 __ 2 12M L

Axis

Axis

Axis Thin spherical shell about any diameter

2R

(f )

I = _25_M R 2

Hoop about any diameter

R

Slab about perpendicular axis through center b a

(g )

I = _23_M R 2

We need to relate the rotational inertia around the axis at P to that around the axis at the com.

I ! Icom & Mh2

P h a

r

(i )

(parallel-axis theorem).

(10-36)

Think of the distance h as being the distance we have shifted the rotation axis from being through the com. This equation is known as the parallel-axis theorem. We shall now prove it.

dm Rotation axis through P

1 __ I = 12 M ( a 2 + b 2)

through the center of mass (remember these two axes must be parallel). Then the rotational inertia I about the given axis is

y

com O

(h )

I = _12_M R 2

y–b

Proof of the Parallel-Axis Theorem x–a

b x

Rotation axis through center of mass

Let O be the center of mass of the arbitrarily shaped body shown in cross section in Fig. 10-12. Place the origin of the coordinates at O. Consider an axis through O perpendicular to the plane of the figure, and another axis through point P parallel to the first axis. Let the x and y coordinates of P be a and b. Let dm be a mass element with the general coordinates x and y. The rotational inertia of the body about the axis through P is then, from Eq. 10-35, I!

Figure 10-12 A rigid body in cross section, with its center of mass at O. The parallelaxis theorem (Eq. 10-36) relates the rotational inertia of the body about an axis through O to that about a parallel axis through a point such as P, a distance h from the body’s center of mass.

"

r 2 dm !

which we can rearrange as I!

"

(x2 & y2) dm % 2a

"

"

[(x % a)2 & ( y % b)2] dm,

x dm % 2b

"

y dm &

"

(a 2 & b2) dm.

(10-37)

From the definition of the center of mass (Eq. 9-9), the middle two integrals of Eq. 10-37 give the coordinates of the center of mass (multiplied by a constant)

275

10-5 CALCU L ATI NG TH E ROTATIONAL I N E RTIA

and thus must each be zero. Because x2 & y2 is equal to R2, where R is the distance from O to dm, the first integral is simply Icom, the rotational inertia of the body about an axis through its center of mass. Inspection of Fig. 10-12 shows that the last term in Eq. 10-37 is Mh2, where M is the body’s total mass. Thus, Eq. 10-37 reduces to Eq. 10-36, which is the relation that we set out to prove.

Checkpoint 5 The figure shows a book-like object (one side is longer than the other) and four choices of rotation axes, all perpendicular to the face of the object. Rank the choices according to the rotational inertia of the object about the axis, greatest first.

(1)

(2)

(3) (4)

Sample Problem 10.06 Rotational inertia of a two-particle system Figure 10-13a shows a rigid body consisting of two particles of mass m connected by a rod of length L and negligible mass. (a) What is the rotational inertia Icom about an axis through the center of mass,perpendicular to the rod as shown? KEY IDEA

left and L for the particle on the right. Now Eq. 10-33 gives us I ! m(0)2 & mL2 ! mL2. (Answer) Second technique: Because we already know Icom about an axis through the center of mass and because the axis here is parallel to that “com axis,” we can apply the parallel-axis theorem (Eq. 10-36). We find

Because we have only two particles with mass, we can find the body’s rotational inertia Icom by using Eq. 10-33 rather than by integration. That is, we find the rotational inertia of each particle and then just add the results.

I ! Icom & Mh2 ! 12 mL2 & (2m)(12 L)2 ! mL2.

Calculations: For the two particles, each at perpendicular distance 21 L from the rotation axis, we have I ! ' mir 2i ! (m)(12 L)2 & (m)(12 L)2 ! 12 mL2.

Rotation axis through center of mass m _1_ 2L

First technique: We calculate I as in part (a), except here the perpendicular distance ri is zero for the particle on the

_1_ 2L

(a)

(b) What is the rotational inertia I of the body about an axis through the left end of the rod and parallel to the first axis (Fig. 10-13b)?

This situation is simple enough that we can find I using either of two techniques. The first is similar to the one used in part (a). The other, more powerful one is to apply the parallel-axis theorem.

m

com

(Answer)

KEY IDEAS

(Answer)

Here the rotation axis is through the com.

Rotation axis through end of rod m

com

m

L (b)

Here it has been shifted from the com without changing the orientation. We can use the parallel-axis theorem.

Figure 10-13 A rigid body consisting of two particles of mass m joined by a rod of negligible mass.

Additional examples, video, and practice available at WileyPLUS

276

CHAPTE R 10 ROTATION

Sample Problem 10.07 Rotational inertia of a uniform rod, integration Figure 10-14 shows a thin, uniform rod of mass M and length L, on an x axis with the origin at the rod’s center. (a) What is the rotational inertia of the rod about the perpendicular rotation axis through the center?

We can now substitute this result for dm and x for r in Eq. 10-38.Then we integrate from end to end of the rod (from x ! %L/2 to x ! L/2) to include all the elements.We find I!

KEY IDEAS (1) The rod consists of a huge number of particles at a great many different distances from the rotation axis. We certainly don’t want to sum their rotational inertias individually. So, we first write a general expression for the rotational inertia of a mass element dm at distance r from the rotation axis: r2 dm. (2) Then we sum all such rotational inertias by integrating the expression (rather than adding them up one by one). From Eq. 10-35, we write I!

"

r 2 dm.

(10-38)

(3) Because the rod is uniform and the rotation axis is at the center, we are actually calculating the rotational inertia Icom about the center of mass. Calculations: We want to integrate with respect to coordinate x (not mass m as indicated in the integral), so we must relate the mass dm of an element of the rod to its length dx along the rod. (Such an element is shown in Fig. 10-14.) Because the rod is uniform, the ratio of mass to length is the same for all the elements and for the rod as a whole.Thus, we can write

or

element’s mass dm rod’s mass M ! element’s length dx rod’s length L M dm ! dx. L

A We want the rotational inertia.

!

x!&L/2

x!%L/2

M 3L

x2

() x3

# ML $ dx M 3L

&L/2

!

%L/2

(Answer)

(b) What is the rod’s rotational inertia I about a new rotation axis that is perpendicular to the rod and through the left end? KEY IDEAS We can find I by shifting the origin of the x axis to the left end of the rod and then integrating from x ! 0 to x ! L. However, here we shall use a more powerful (and easier) technique by applying the parallel-axis theorem (Eq. 10-36), in which we shift the rotation axis without changing its orientation. Calculations: If we place the axis at the rod’s end so that it is parallel to the axis through the center of mass, then we can use the parallel-axis theorem (Eq. 10-36). We know from part (a) that Icom is 121 ML2. From Fig. 10-14, the perpendicular distance h between the new rotation axis and the center of mass is 21 L. Equation 10-36 then gives us I ! Icom & Mh2 ! 121 ML2 & (M)( 21 L)2 (Answer)

First, pick any tiny element and write its rotational inertia as x2 dm.

L x = − __ 2 Leftmost

Rotation axis dx x

x dm

Rotation axis x

Figure 10-14 A uniform rod of length L and mass M. An element of mass dm and length dx is represented.

3

Actually, this result holds for any axis through the left or right end that is perpendicular to the rod.

M

L __ 2

3

1 2 12 ML .

x L __ 2

(# L2 $ % #% L2 $ )

! 13 ML2.

Rotation axis com

!

"

L x = __ 2 Rightmost

Additional examples, video, and practice available at WileyPLUS

Then, using integration, add up the rotational inertias for all of the elements, from leftmost to rightmost.

10-6 TORQU E

277

Large machine components that undergo prolonged, highspeed rotation are first examined for the possibility of failure in a spin test system. In this system, a component is spun up (brought up to high speed) while inside a cylindrical arrangement of lead bricks and containment liner, all within a steel shell that is closed by a lid clamped into place. If the rotation causes the component to shatter, the soft lead bricks are supposed to catch the pieces for later analysis. In 1985, Test Devices, Inc. (www.testdevices.com) was spin testing a sample of a solid steel rotor (a disk) of mass M ! 272 kg and radius R ! 38.0 cm. When the sample reached an angular speed v of 14 000 rev/min, the test engineers heard a dull thump from the test system, which was located one floor down and one room over from them. Investigating, they found that lead bricks had been thrown out in the hallway leading to the test room, a door to the room had been hurled into the adjacent parking lot, one lead brick had shot from the test site through the wall of a neighbor’s kitchen, the structural beams of the test building had been damaged, the concrete floor beneath the spin chamber had been shoved downward by about 0.5 cm, and the 900 kg lid had been blown upward through the ceiling and had then crashed back onto the test equipment (Fig. 10-15). The exploding pieces had not penetrated the room of the test engineers only by luck. How much energy was released in the explosion of the rotor?

Courtesy Test Devices, Inc.

Sample Problem 10.08 Rotational kinetic energy, spin test explosion Figure 10-15 Some of the destruction caused by the explosion of a rapidly rotating steel disk.

KEY IDEA The released energy was equal to the rotational kinetic energy K of the rotor just as it reached the angular speed of 14 000 rev/min. Calculations: We can find K with Eq. 10-34 (K ! 12 Iv2), but first we need an expression for the rotational inertia I. Because the rotor was a disk that rotated like a merry-go-round, I is given in Table 10-2c (I ! 12 MR2). Thus, I ! 12 MR 2 ! 12 (272 kg)(0.38 m)2 ! 19.64 kg )m2. The angular speed of the rotor was

# 160mins $

v ! (14 000 rev/min)(2p rad/rev) ! 1.466 $ 10 3 rad/s.

Then, with Eq. 10-34, we find the (huge) energy release: K ! 12 Iv2 ! 12(19.64 kg)m2)(1.466 $ 10 3 rad/s)2 ! 2.1 $ 107 J.

(Answer)

Additional examples, video, and practice available at WileyPLUS

10-6 TORQUE Learning Objectives After reading this module, you should be able to . . .

10.23 Identify that a torque on a body involves a force and a position vector, which extends from a rotation axis to the point where the force is applied. 10.24 Calculate the torque by using (a) the angle between the position vector and the force vector, (b) the line of action and the moment arm of the force, and (c) the force component perpendicular to the position vector.

10.25 Identify that a rotation axis must always be specified to calculate a torque. 10.26 Identify that a torque is assigned a positive or negative sign depending on the direction it tends to make the body rotate about a specified rotation axis: “clocks are negative.” 10.27 When more than one torque acts on a body about a rotation axis, calculate the net torque.

Key Ideas ● Torque is a turning or twisting action on a body about a :

:

rotation axis due to a force F . If F is exerted at a point given by the position vector :r relative to the axis, then the magnitude of the torque is t ! rFt ! r"F ! rF sin f, :

where Ft is the component of F perpendicular to :r and : f is the angle between :r and F . The quantity r" is the

perpendicular distance between the rotation axis and : an extended line running through the F vector. This line : is called the line of action of F , and r" is called the : moment arm of F . Similarly, r is the moment arm of Ft. ● The SI unit of torque is the newton-meter (N )m). A torque t is positive if it tends to rotate a body at rest counterclockwise and negative if it tends to rotate the body clockwise.

278

CHAPTE R 10 ROTATION

Torque F

φ P Rotation axis

r

O (a)

The torque due to this force causes rotation around this axis (which extends out toward you).

F

φ F r

Ft

P Rotation axis

r

O (b)

But actually only the tangential component of the force causes the rotation.

A doorknob is located as far as possible from the door’s hinge line for a good reason. If you want to open a heavy door, you must certainly apply a force, but that is not enough. Where you apply that force and in what direction you push are also important. If you apply your force nearer to the hinge line than the knob, or at any angle other than 90# to the plane of the door, you must use a greater force than if you apply the force at the knob and perpendicular to the door’s plane. Figure 10-16a shows a cross section of a body that is free to rotate about an : axis passing through O and perpendicular to the cross section. A force F is applied at point P, whose position relative to O is defined by a position vector :r . : The directions of vectors F and :r make an angle f with each other. (For simplicity, we consider only forces that have no component parallel to the rotation axis; : thus, F is in the plane of the page.) : To determine how F results in a rotation of the body around the rotation : axis, we resolve F into two components (Fig. 10-16b). One component, called the radial component Fr, points along :r . This component does not cause rotation, because it acts along a line that extends through O. (If you pull on a door parallel to the plane of the door, you do not rotate the door.) The other compo: nent of F , called the tangential component Ft, is perpendicular to :r and has magnitude Ft ! F sin f. This component does cause rotation. : Calculating Torques. The ability of F to rotate the body depends not only on the magnitude of its tangential component Ft, but also on just how far from O the force is applied. To include both these factors, we define a quantity called torque t as the product of the two factors and write it as t ! (r)(F sin f). Two equivalent ways of computing the torque are and

F

φ

(c)

Rotation axis

O Moment arm of F

φ

r

P Line of action of F

r

You calculate the same torque by using this moment arm distance and the full force magnitude. :

Figure 10-16 (a) A force F acts on a rigid body, with a rotation axis perpendicular to the page. The torque can be found with (a) angle f, (b) tangential force component Ft, or (c) moment arm r".

(10-39)

t ! (r)(F sin f) ! rFt

(10-40)

t ! (r sin f)(F) ! r"F,

(10-41)

where r" is the perpendicular distance between the rotation axis at O and an extended : line running through the vector F (Fig. 10-16c). This extended line is called the line : : of action of F , and r" is called the moment arm of F . Figure 10-16b shows that we can describe r, the magnitude of :r , as being the moment arm of the force component Ft. Torque, which comes from the Latin word meaning “to twist,” may be loosely : identified as the turning or twisting action of the force F . When you apply a force to an object — such as a screwdriver or torque wrench — with the purpose of turning that object, you are applying a torque. The SI unit of torque is the newtonmeter (N )m). Caution: The newton-meter is also the unit of work. Torque and work, however, are quite different quantities and must not be confused. Work is often expressed in joules (1 J ! 1 N )m), but torque never is. Clocks Are Negative. In Chapter 11 we shall use vector notation for torques, but here, with rotation around a single axis, we use only an algebraic sign. If a torque would cause counterclockwise rotation, it is positive. If it would cause clockwise rotation, it is negative. (The phrase “clocks are negative” from Module 10-1 still works.) Torques obey the superposition principle that we discussed in Chapter 5 for forces: When several torques act on a body, the net torque (or resultant torque) is the sum of the individual torques. The symbol for net torque is tnet.

Checkpoint 6 The figure shows an overhead view of a meter stick that can pivot about the dot at the position marked 20 (for 20 cm). All five forces on the stick are horizontal and have the same magnitude. Rank the forces according to the magnitude of the torque they produce, greatest first.

0

20

Pivot point 40

100 F5

F1

F2

F3

F4

279

10-7 N EWTON’S SECON D L AW FOR ROTATION

10-7 NEWTON’S SECOND LAW FOR ROTATION Learning Objective After reading this module, you should be able to . . .

10.28 Apply Newton’s second law for rotation to relate the net torque on a body to the body’s rotational inertia and

rotational acceleration, all calculated relative to a specified rotation axis.

Key Idea ● The rotational analog of Newton’s second law is

tnet ! Ia, where tnet is the net torque acting on a particle or rigid body,

I is the rotational inertia of the particle or body about the rotation axis, and a is the resulting angular acceleration about that axis.

Newton’s Second Law for Rotation A torque can cause rotation of a rigid body, as when you use a torque to rotate a door. Here we want to relate the net torque tnet on a rigid body to the angular acceleration a that torque causes about a rotation axis. We do so by analogy with Newton’s second law (Fnet ! ma) for the acceleration a of a body of mass m due to a net force Fnet along a coordinate axis. We replace Fnet with tnet, m with I, and a with a in radian measure, writing tnet ! Ia (Newton’s second law for rotation). (10-42)

Proof of Equation 10-42 We prove Eq. 10-42 by first considering the simple situation shown in Fig. 10-17. The rigid body there consists of a particle of mass m on one end of a massless rod of length r. The rod can move only by rotating about its other end, around a rotation axis (an axle) that is perpendicular to the plane of the page. Thus, the particle can move only in a circular path that has the rotation axis at its center. : A force F acts on the particle. However, because the particle can move only along the circular path, only the tangential component Ft of the force (the component that is tangent to the circular path) can accelerate the particle along the path. We can relate Ft to the particle’s tangential acceleration at along the path with Newton’s second law, writing Ft ! ma t. The torque acting on the particle is, from Eq. 10-40,

The torque due to the tangential component of the force causes an angular acceleration around the rotation axis.

t ! Ft r ! ma t r.

y

From Eq. 10-22 (at ! ar) we can write this as t ! m(ar)r ! (mr 2)a.

(radian measure).

(radian measure),

m r

(10-44)

Rod

θ

If more than one force is applied to the particle, Eq. 10-44 becomes tnet ! Ia

Fr

φ

The quantity in parentheses on the right is the rotational inertia of the particle about the rotation axis (see Eq. 10-33, but here we have only a single particle). Thus, using I for the rotational inertia, Eq. 10-43 reduces to t ! Ia

F

Ft

(10-43)

O

(10-45)

which we set out to prove. We can extend this equation to any rigid body rotating about a fixed axis, because any such body can always be analyzed as an assembly of single particles.

x

Rotation axis

Figure 10-17 A simple rigid body, free to rotate about an axis through O, consists of a particle of mass m fastened to the end of a rod of length r and negligible mass. An : applied force F causes the body to rotate.

280

CHAPTE R 10 ROTATION

Checkpoint 7 The figure shows an overhead view of a meter stick that can pivot about the point indicated, which is : : : to the left of the stick’s midpoint.Two horizontal forces, F1 and F2, are applied to the stick. Only F1 is : shown. Force F2 is perpendicular to the stick and is applied at the right end. If the stick is not to turn, : (a) what should be the direction of F2, and (b) should F2 be greater than, less than, or equal to F1?

Pivot point F1

Sample Problem 10.09 Using Newton’s second law for rotation in a basic judo hip throw To throw an 80 kg opponent with a basic judo hip throw, you : intend to pull his uniform with a force F and a moment arm d1 ! 0.30 m from a pivot point (rotation axis) on your right hip (Fig. 10-18). You wish to rotate him about the pivot point with an angular acceleration a of %6.0 rad/s2—that is, with an angular acceleration that is clockwise in the figure. Assume that his rotational inertia I relative to the pivot point is 15 kg )m2. :

(a) What must the magnitude of F be if, before you throw him, you bend your opponent forward to bring his center of mass to your hip (Fig. 10-18a)?

Moment arm d1 of your pull Opponent's center of mass

Fg Pivot on hip

Moment arm d2 of gravitational force on opponent

F Moment arm d1 of your pull

Fg

F

KEY IDEA :

We can relate your pull F on your opponent to the given angular acceleration a via Newton’s second law for rotation (tnet ! Ia). Calculations: As his feet leave the floor, we can assume that : : only three forces act on him: your pull F , a force N on him from you at the pivot point (this force is not indicated in Fig. : 10-18), and the gravitational force Fg.To use tnet ! Ia, we need the corresponding three torques, each about the pivot point. : From Eq. 10-41 (t ! r" F), the torque due to your pull F is equal to %d1F, where d1 is the moment arm r" and the sign indicates the clockwise rotation this torque tends to : : cause. The torque due to N is zero, because N acts at the pivot point and thus has moment arm r" ! 0. : : To evaluate the torque due to Fg, we can assume that Fg acts at your opponent’s center of mass. With the center of : mass at the pivot point, Fg has moment arm r" ! 0 and thus : the torque due to Fg is zero. So, the only torque on your op: ponent is due to your pull F , and we can write tnet ! Ia as %d1F ! Ia.

We then find F!

(a )

(b )

Figure 10-18 A judo hip throw (a) correctly executed and (b) incorrectly executed.

KEY IDEA :

Because the moment arm for Fg is no longer zero, the torque : due to Fg is now equal to d2mg and is positive because the torque attempts counterclockwise rotation. Calculations: Now we write tnet ! Ia as %d1F & d2mg ! Ia,

which gives

Ia d mg & 2 . d1 d1 From (a), we know that the first term on the right is equal to 300 N. Substituting this and the given data, we have F!%

F ! 300 N &

! 613.6 N % 610 N.

%Ia %(15 kg)m2)(%6.0 rad/s2) ! d1 0.30 m

! 300 N.

(Answer) :

(0.12 m)(80 kg)(9.8 m/s2) 0.30 m

(b) What must the magnitude of F be if your opponent : remains upright before you throw him, so that Fg has a moment arm d2 ! 0.12 m (Fig. 10-18b)?

(Answer)

The results indicate that you will have to pull much harder if you do not initially bend your opponent to bring his center of mass to your hip. A good judo fighter knows this lesson from physics. Indeed, physics is the basis of most of the martial arts, figured out after countless hours of trial and error over the centuries.

Additional examples, video, and practice available at WileyPLUS

281

10-7 N EWTON’S SECON D L AW FOR ROTATION

Sample Problem 10.10 Newton’s second law, rotation, torque, disk Figure 10-19a shows a uniform disk, with mass M ! 2.5 kg and radius R ! 20 cm, mounted on a fixed horizontal axle. A block with mass m ! 1.2 kg hangs from a massless cord that is wrapped around the rim of the disk. Find the acceleration of the falling block, the angular acceleration of the disk, and the tension in the cord. The cord does not slip, and there is no friction at the axle.

M M

R O

T

(c) y T

KEY IDEAS

m

(1) Taking the block as a system, we can relate its acceleration a to the forces acting on it with Newton’s second law : ( F net ! m: a ). (2) Taking the disk as a system, we can relate its angular acceleration a to the torque acting on it with Newton’s second law for rotation (tnet ! Ia). (3) To combine the motions of block and disk, we use the fact that the linear acceleration a of the block and the (tangential) linear acceleration at of the disk rim are equal. (To avoid confusion about signs, let’s work with acceleration magnitudes and explicit algebraic signs.) Forces on block: The forces are shown in the block’s free: body diagram in Fig. 10-19b: The force from the cord is T , : and the gravitational force is Fg, of magnitude mg. We can now write Newton’s second law for components along a vertical y axis (Fnet,y ! may) as T % mg ! m(%a),

Torque on disk: Previously, when we got stuck on the y axis, we switched to the x axis. Here, we switch to the rotation of the disk and use Newton’s second law in angular form. To calculate the torques and the rotational inertia I, we take the rotation axis to be perpendicular to the disk and through its center, at point O in Fig. 10-19c. The torques are then given by Eq. 10-40 (t ! rFt). The gravitational force on the disk and the force on the disk from the axle both act at the center of the disk and thus at distance : r ! 0, so their torques are zero.The force T on the disk due to the cord acts at distance r ! R and is tangent to the rim of the disk. Therefore, its torque is %RT, negative because the torque rotates the disk clockwise from rest. Let a be the magnitude of the negative (clockwise) angular acceleration. From Table 10-2c, the rotational inertia I of the disk is 12 MR 2. Thus we can write the general equation tnet ! Ia as %RT !

1 2 2 MR (%a).

(10-47)

m Fg

(a)

(b)

These two forces determine the block's (linear) acceleration. We need to relate those two accelerations.

Figure 10-19 (a) The falling block causes the disk to rotate. (b) A free-body diagram for the block. (c) An incomplete free-body diagram for the disk.

This equation seems useless because it has two unknowns, a and T, neither of which is the desired a. However, mustering physics courage, we can make it useful with this fact: Because the cord does not slip, the magnitude a of the block’s linear acceleration and the magnitude at of the (tangential) linear acceleration of the rim of the disk are equal. Then, by Eq. 10-22 (at ! ar) we see that here a ! a /R. Substituting this in Eq. 10-47 yields T ! 12 Ma.

(10-46)

where a is the magnitude of the acceleration (down the y axis). However, we cannot solve this equation for a because it also contains the unknown T.

The torque due to the cord's pull on the rim causes an angular acceleration of the disk.

(10-48)

Combining results: Combining Eqs. 10-46 and 10-48 leads to a!g

2m (2)(1.2 kg) ! (9.8 m/s2) M & 2m 2.5 kg & (2)(1.2 kg)

! 4.8 m/s2.

(Answer)

We then use Eq. 10-48 to find T: T ! 12 Ma ! 12(2.5 kg)(4.8 m/s2) (Answer) ! 6.0 N. As we should expect, acceleration a of the falling block is less than g, and tension T in the cord (! 6.0 N) is less than the gravitational force on the hanging block (! mg ! 11.8 N). We see also that a and T depend on the mass of the disk but not on its radius. As a check, we note that the formulas derived above predict a ! g and T ! 0 for the case of a massless disk (M ! 0). This is what we would expect; the block simply falls as a free body. From Eq. 10-22, the magnitude of the angular acceleration of the disk is a!

a 4.8 m/s2 ! ! 24 rad/s2. R 0.20 m

Additional examples, video, and practice available at WileyPLUS

(Answer)

282

CHAPTE R 10 ROTATION

10-8 WORK AND ROTATIONAL KINETIC ENERGY Learning Objectives After reading this module, you should be able to . . .

10.29 Calculate the work done by a torque acting on a rotating body by integrating the torque with respect to the angle of rotation. 10.30 Apply the work–kinetic energy theorem to relate the work done by a torque to the resulting change in the rotational kinetic energy of the body.

10.31 Calculate the work done by a constant torque by relating the work to the angle through which the body rotates. 10.32 Calculate the power of a torque by finding the rate at which work is done. 10.33 Calculate the power of a torque at any given instant by relating it to the torque and the angular velocity at that instant.

Key Ideas ● The equations used for calculating work and power in rotational motion correspond to equations used for translational motion and are

"

uf

W!

t du

ui

and

P!

● When t is constant, the integral reduces to

W ! t(uf % ui). ● The form of the work – kinetic energy theorem used for

rotating bodies is

dW ! tv. dt

"K ! Kf % Ki ! 12 Iv2f % 12 2v2i ! W.

Work and Rotational Kinetic Energy As we discussed in Chapter 7, when a force F causes a rigid body of mass m to accelerate along a coordinate axis, the force does work W on the body. Thus, the body’s kinetic energy (K ! 12 mv 2) can change. Suppose it is the only energy of the body that changes. Then we relate the change "K in kinetic energy to the work W with the work – kinetic energy theorem (Eq. 7-10), writing "K ! Kf % Ki ! 12 mv 2f % 12 mv 2i ! W

(10-49)

(work – kinetic energy theorem).

For motion confined to an x axis, we can calculate the work with Eq. 7-32, W!

"

xf

xi

F dx

(work, one-dimensional motion).

(10-50)

This reduces to W ! Fd when F is constant and the body’s displacement is d. The rate at which the work is done is the power, which we can find with Eqs. 7-43 and 7-48, dW (10-51) P! ! Fv (power, one-dimensional motion). dt Now let us consider a rotational situation that is similar. When a torque accelerates a rigid body in rotation about a fixed axis, the torque does work W on the body. Therefore, the body’s rotational kinetic energy (K ! 12 I12) can change. Suppose that it is the only energy of the body that changes. Then we can still relate the change "K in kinetic energy to the work W with the work – kinetic energy theorem, except now the kinetic energy is a rotational kinetic energy: "K ! Kf % Ki ! 12 Iv2f % 122v2i ! W

(work – kinetic energy theorem).

(10-52)

Here, I is the rotational inertia of the body about the fixed axis and vi and vf are the angular speeds of the body before and after the work is done.

10-8 WOR K AN D ROTATIONAL KI N ETIC E N E RGY

Also, we can calculate the work with a rotational equivalent of Eq. 10-50, W!

"

uf

t du

(10-53)

(work, rotation about fixed axis),

ui

where t is the torque doing the work W, and ui and uf are the body’s angular positions before and after the work is done, respectively. When t is constant, Eq. 10-53 reduces to W ! t(uf % ui)

(10-54)

(work, constant torque).

The rate at which the work is done is the power, which we can find with the rotational equivalent of Eq. 10-51, P!

dW ! tv dt

(10-55)

(power, rotation about fixed axis).

Table 10-3 summarizes the equations that apply to the rotation of a rigid body about a fixed axis and the corresponding equations for translational motion.

Proof of Eqs. 10-52 through 10-55 :

Let us again consider the situation of Fig. 10-17, in which force F rotates a rigid body consisting of a single particle of mass m fastened to the end of a massless : rod. During the rotation, force F does work on the body. Let us assume that the : only energy of the body that is changed by F is the kinetic energy. Then we can apply the work – kinetic energy theorem of Eq. 10-49: (10-56)

"K ! Kf % Ki ! W. Using K ! 12 mv 2 and Eq. 10-18 (v ! vr), we can rewrite Eq. 10-56 as "K ! 12 mr 2v2f % 12 mr 2v2i ! W.

(10-57)

From Eq. 10-33, the rotational inertia for this one-particle body is I ! mr 2. Substituting this into Eq. 10-57 yields "K ! 12 Iv2f % 12 2v2i ! W, which is Eq. 10-52. We derived it for a rigid body with one particle, but it holds for any rigid body rotated about a fixed axis. We next relate the work W done on the body in Fig. 10-17 to the torque t : on the body due to force F . When the particle moves a distance ds along its Table 10-3 Some Corresponding Relations for Translational and Rotational Motion Pure Translation (Fixed Direction) Position Velocity Acceleration Mass Newton’s second law Work Kinetic energy Power (constant force) Work – kinetic energy theorem

x v ! dx/dt a ! dv/dt m Fnet ! ma W ! * F dx K ! 12 mv2 P ! Fv W ! "K

Pure Rotation (Fixed Axis) Angular position Angular velocity Angular acceleration Rotational inertia Newton’s second law Work Kinetic energy Power (constant torque) Work – kinetic energy theorem

u v ! du/dt a ! dv/dt I tnet ! Ia W ! * t du K ! 12 Iv2 P ! tv W ! "K

283

284

CHAPTE R 10 ROTATION

circular path, only the tangential component Ft of the force accelerates the particle along the path. Therefore, only Ft does work on the particle. We write that work dW as Ft ds. However, we can replace ds with r du, where du is the angle through which the particle moves. Thus we have dW ! Ft r du.

(10-58)

From Eq. 10-40, we see that the product Ft r is equal to the torque t, so we can rewrite Eq. 10-58 as dW ! t du. (10-59) The work done during a finite angular displacement from ui to uf is then W!

"

uf

t du,

ui

which is Eq. 10-53. It holds for any rigid body rotating about a fixed axis. Equation 10-54 comes directly from Eq. 10-53. We can find the power P for rotational motion from Eq. 10-59: P! which is Eq. 10-55.

dW du !t ! tv, dt dt

Sample Problem 10.11 Work, rotational kinetic energy, torque, disk Let the disk in Fig. 10-19 start from rest at time t ! 0 and also let the tension in the massless cord be 6.0 N and the angular acceleration of the disk be %24 rad/s2. What is its rotational kinetic energy K at t ! 2.5 s?

K ! Ki & W ! 0 & W ! W.

KEY IDEA We can find K with Eq. 10-34 (K ! 12 Iv2). We already know that I ! 12 MR2, but we do not yet know v at t ! 2.5 s. However, because the angular acceleration a has the constant value of %24 rad/s2, we can apply the equations for constant angular acceleration in Table 10-1. Calculations: Because we want v and know a and v0 (! 0), we use Eq. 10-12: v ! v0 & at ! 0 & at ! at. Substituting v ! at and I ! 12 MR 2 into Eq. 10-34, we find K!

Calculations: First, we relate the change in the kinetic energy of the disk to the net work W done on the disk, using the work–kinetic energy theorem of Eq. 10-52 (Kf % Ki ! W). With K substituted for Kf and 0 for Ki, we get

1 1 1 1 2 2 2 2 2 Iv ! 2 (2 MR )(at) ! 4 M(Rat) 1 2 4 (2.5 kg)[(0.20 m)(%24 rad/s )(2.5

! ! 90 J.

s)]2 (Answer)

KEY IDEA We can also get this answer by finding the disk’s kinetic energy from the work done on the disk.

(10-60)

Next we want to find the work W. We can relate W to the torques acting on the disk with Eq. 10-53 or 10-54. The only torque causing angular acceleration and doing work is : the torque due to force T on the disk from the cord, which is equal to %TR. Because a is constant, this torque also must be constant. Thus, we can use Eq. 10-54 to write W ! t(uf % ui) ! %TR(uf % ui).

(10-61)

Because a is constant, we can use Eq. 10-13 to find uf % ui . With vi ! 0, we have uf % ui ! vit & 12at 2 ! 0 & 12 at 2 ! 12at 2. Now we substitute this into Eq. 10-61 and then substitute the result into Eq. 10-60. Inserting the given values T ! 6.0 N and a ! %24 rad/s2, we have K ! W ! %TR(uf % ui) ! %TR(12at 2) ! %12TRat 2 ! %12 (6.0 N)(0.20 m)(%24 rad/s2)(2.5 s)2 ! 90 J.

Additional examples, video, and practice available at WileyPLUS

(Answer)

R EVI EW & SU M MARY

285

Review & Summary Angular Position To describe the rotation of a rigid body about

a fixed axis, called the rotation axis, we assume a reference line is fixed in the body, perpendicular to that axis and rotating with the body.We measure the angular position u of this line relative to a fixed direction.When u is measured in radians, s (radian measure), (10-1) u! r where s is the arc length of a circular path of radius r and angle u. Radian measure is related to angle measure in revolutions and degrees by 1 rev ! 360# ! 2p rad. (10-2)

Angular Displacement A body that rotates about a rotation axis, changing its angular position from u1 to u2, undergoes an angular displacement "u ! u2 % u1, (10-4)

where "u is positive for counterclockwise rotation and negative for clockwise rotation.

Angular Velocity and Speed If a body rotates through an

angular displacement "u in a time interval "t, its average angular velocity vavg is "u (10-5) vavg ! . "t The (instantaneous) angular velocity v of the body is du . v! dt

(10-6)

Angular Acceleration If the angular velocity of a body

changes from v1 to v2 in a time interval "t ! t2 % t1, the average angular acceleration aavg of the body is v 2 % v1 "v ! . t2 % t1 "t

(10-7)

The (instantaneous) angular acceleration a of the body is dv . a! dt Both aavg and a are vectors.

(10-8)

u % u0 ! v0t &

v2 ! v20 & 2a(u % u0),

at ! ar

(10-22)

(radian measure),

where a is the magnitude of the angular acceleration (in radians per second-squared) of the body. The radial component of : a is ar !

v2 ! v2r r

(10-23)

(radian measure).

If the point moves in uniform circular motion, the period T of the motion for the point and the body is T!

2pr 2p ! v v

(10-19, 10-20)

(radian measure).

Rotational Kinetic Energy and Rotational Inertia The kiK ! 12Iv2

(10-34)

(radian measure),

in which I is the rotational inertia of the body, defined as I ! ' mir 2i

(10-33)

for a system of discrete particles and defined as I!

"

r 2 dm

(10-35)

for a body with continuously distributed mass. The r and ri in these expressions represent the perpendicular distance from the axis of rotation to each mass element in the body, and the integration is carried out over the entire body so as to include every mass element. the rotational inertia I of a body about any axis to that of the same body about a parallel axis through the center of mass: I ! Icom & Mh2.

portant special case of rotational motion. The appropriate kinematic equations, given in Table 10-1, are 1 2 2 at ,

(10-18)

(radian measure),

The Parallel-Axis Theorem The parallel-axis theorem relates

The Kinematic Equations for Constant Angular Acceleration Constant angular acceleration (a ! constant) is an im-

v ! v0 & at,

v ! vr

where v is the angular speed (in radians per second) of the body. a of the point has both tangential and The linear acceleration : radial components. The tangential component is

netic energy K of a rigid body rotating about a fixed axis is given by

Both vavg and v are vectors, with directions given by the right-hand rule of Fig. 10-6. They are positive for counterclockwise rotation and negative for clockwise rotation. The magnitude of the body’s angular velocity is the angular speed.

aavg !

moves in a circle with radius r. If the body rotates through an angle u, the point moves along an arc with length s given by s ! ur (radian measure), (10-17) where u is in radians. The linear velocity : v of the point is tangent to the circle; the point’s linear speed v is given by

(10-36)

Here h is the perpendicular distance between the two axes, and Icom is the rotational inertia of the body about the axis through the com. We can describe h as being the distance the actual rotation axis has been shifted from the rotation axis through the com.

(10-12)

Torque Torque is a turning or twisting action on a body about a ro-

(10-13)

tation axis due to a force F . If F is exerted at a point given by the position vector : r relative to the axis, then the magnitude of the torque is

(10-14)

u % u0 ! 12 (v0 & v)t,

(10-15)

u % u0 ! vt % 12 at 2.

(10-16)

Linear and Angular Variables Related A point in a rigid rotating body, at a perpendicular distance r from the rotation axis,

:

:

t ! rFt ! r"F ! rF sin f, :

(10-40, 10-41, 10-39)

where Ft is the component of F perpendicular to : r and f is the an: gle between : r and F . The quantity r" is the perpendicular distance between the rotation axis and an extended line running through : : the F vector. This line is called the line of action of F , and r" is : called the moment arm of F . Similarly, r is the moment arm of Ft.

286

CHAPTE R 10 ROTATION

The SI unit of torque is the newton-meter (N )m). A torque t is positive if it tends to rotate a body at rest counterclockwise and negative if it tends to rotate the body clockwise.

Newton’s Second Law in Angular Form The rotational analog of Newton’s second law is (10-45)

tnet ! Ia,

where tnet is the net torque acting on a particle or rigid body, I is the rotational inertia of the particle or body about the rotation axis, and a is the resulting angular acceleration about that axis.

Work and Rotational Kinetic Energy The equations used

equations used for translational motion and are

"

uf

W!

t du

(10-53)

ui

and

P!

dW ! tv. dt

(10-55)

When t is constant, Eq. 10-53 reduces to W ! t(uf % ui).

(10-54)

The form of the work – kinetic energy theorem used for rotating bodies is "K ! Kf % Ki ! 12 Iv2f % 12 2v2i ! W.

for calculating work and power in rotational motion correspond to

(10-52)

Questions ω

1 Figure 10-20 is a graph of the angular velocity versus time for a disk rotating like a merry-go-round. For a point on the disk rim, rank the instants a, b, c, and d according to the magnitude of the (a) tangential and (b) radial acceleration, greatest first.

a

b

c

d

t

Figure 10-20 Question 1.

2 Figure 10-21 shows plots of anguθ 90° lar position u versus time t for three 1 cases in which a disk is rotated like a merry-go-round. In each case, the ro3 tation direction changes at a certain t 0 angular position uchange. (a) For each 2 case, determine whether uchange is clockwise or counterclockwise from –90° u ! 0, or whether it is at u ! 0. For Figure 10-21 Question 2. each case, determine (b) whether v is zero before, after, or at t ! 0 and (c) whether a is positive, negative, or zero. 3 A force is applied to the rim of a disk that can rotate like a merry-go-round, so as to change its angular velocity. Its initial and final angular velocities, respectively, for four situations are: (a) %2 rad/s, 5 rad/s; (b) 2 rad/s, 5 rad/s; (c) %2 rad/s, %5 rad/s; and (d) 2 rad/s, %5 rad/s. Rank the situations according to the work done by the torque due to the force, greatest first. 4 Figure 10-22b is a graph of the angular position of the rotating disk of Fig. 10-22a. Is the angular velocity of the disk positive, negative, or zero at (a) t ! 1 s, (b) t ! 2 s, and (c) t ! 3 s? (d) Is the angular acceleration positive or negative?

(a)

2

3

t (s)

(b)

Figure 10-22 Question 4. :

F1

θ

F2

Figure 10-23 Question 5. F4 F5

F3

F1

P F2

Figure 10-24 Question 6.

7 Figure 10-25a is an overhead view of a horizontal bar that can pivot; two horizontal forces act on the : bar, but it is stationary. If the angle between the bar and F 2 is now decreased from 90# and the bar is still not to turn, should F2 be made larger, made smaller, or left the same? F1

Pivot point

F2

F2 Pivot point φ F1 (b)

Figure 10-25 Questions 7 and 8.

1

:

6 In the overhead view of Fig. 10-24, five forces of the same magnitude act on a strange merry-go-round; it is a square that can rotate about point P, at midlength along one of the edges. Rank the forces according to the magnitude of the torque they create about point P, greatest first.

(a)

θ (rad)

Rotation axis

angles during the rotation, which is counterclockwise and at a constant rate. However, we are to decrease the : angle u of F1 without changing the : magnitude of F1 . (a) To keep the angular speed constant, should we increase, decrease, or maintain the mag: : nitude of F 2? Do forces (b) F1 and (c) : F 2 tend to rotate the disk clockwise or counterclockwise?

5 In Fig. 10-23, two forces F1 and F2 act on a disk that turns about its center like a merry-go-round. The forces maintain the indicated

8 Figure 10-25b shows an overhead view of a horizontal bar that : : is rotated about the pivot point by two horizontal forces, F1 and F 2, : with F 2 at angle f to the bar. Rank the following values of f according to the magnitude of the angular acceleration of the bar, greatest a first: 90#, 70#, and 110#. 9 Figure 10-26 shows a uniform metal plate that had been square before 25% of it was snipped off. Three lettered points are indicated. Rank them according to the rotational inertia of the plate around a perpendicular axis through them, greatest first.

b c

Figure 10-26 Question 9.

PROB LE M S

11 Figure 10-28a shows a meter stick, half wood and half steel, : that is pivoted at the wood end at O.A force F is applied to the steel end at a. In Fig. 10-28b, the stick is reversed and pivoted at the steel end at O/, and the same force is applied at the wood end at a/. Is the resulting angular acceleration of Fig. 10-28a greater than, less than, or the same as that of Fig. 10-28b?

10 Figure 10-27 shows three flat disks (of the same radius) that can rotate about their centers like merry-go-rounds. Each disk consists of the same two materials, one denser than the other (density is mass per unit volume). In disks 1 and 3, the denser material forms the outer half of the disk area. In disk 2, it forms the inner half of the disk area. Forces with identical magnitudes are applied tangentially to the disk, either at the outer edge or at the interface of the two materials, as shown. Rank the disks according to (a) the torque about the disk center, (b) the rotational inertia about the disk center, and (c) the angular acceleration of the disk, greatest first. F

Denser Disk 1

F

Lighter Disk 2

287

a

O

Figure 10-28 Question 11.

F



a´ F

F (a)

(b)

12 Figure 10-29 shows three disks, each with a uniform distribution of mass. The radii R and masses M are indicated. Each disk can rotate 2m 3m around its central axis (perpendicular R: 1 m M: 26 kg 7 kg 3 kg to the disk face and through the center). Rank the disks according to their (a) (b) (c) rotational inertias calculated about Figure 10-29 Question 12. their central axes, greatest first.

Denser Disk 3

Figure 10-27 Question 10.

Problems Tutoring problem available (at instructor’s discretion) in WileyPLUS and WebAssign SSM • – •••

Worked-out solution available in Student Solutions Manual

WWW Worked-out solution is at

Number of dots indicates level of problem difficulty

ILW

Interactive solution is at

http://www.wiley.com/college/halliday

Additional information available in The Flying Circus of Physics and at flyingcircusofphysics.com

••3 When a slice of buttered toast is accidentally pushed over the edge of a counter, it rotates as it falls. If the distance to the floor is 76 cm and for rotation less than 1 rev, what are the (a) smallest and (b) largest angular speeds that cause the toast to hit and then topple to be butter-side down?

through the wheel without hitting any of the spokes. Assume that the arrow and the spokes are very thin. (a) What minimum speed must the arrow have? (b) Does it matter where between the axle and rim of the wheel you aim? If so, what is the best location? •••8 The angular acceleration of a Figure 10-30 Problem 7. wheel is a ! 6.0t4 % 4.0t2, with a in radians per second-squared and t in seconds. At time t ! 0, the wheel has an angular velocity of &2.0 rad/s and an angular position of &1.0 rad. Write expressions for (a) the angular velocity (rad/s) and (b) the angular position (rad) as functions of time (s).

••4 The angular position of a point on a rotating wheel is given by u ! 2.0 & 4.0t 2 & 2.0t 3, where u is in radians and t is in seconds. At t ! 0, what are (a) the point’s angular position and (b) its angular velocity? (c) What is its angular velocity at t ! 4.0 s? (d) Calculate its angular acceleration at t ! 2.0 s. (e) Is its angular acceleration constant?

Module 10-2 Rotation with Constant Angular Acceleration •9 A drum rotates around its central axis at an angular velocity of 12.60 rad/s. If the drum then slows at a constant rate of 4.20 rad/s2, (a) how much time does it take and (b) through what angle does it rotate in coming to rest?

••5 ILW A diver makes 2.5 revolutions on the way from a 10-m-high platform to the water. Assuming zero initial vertical velocity, find the average angular velocity during the dive.

•10 Starting from rest, a disk rotates about its central axis with constant angular acceleration. In 5.0 s, it rotates 25 rad. During that time, what are the magnitudes of (a) the angular acceleration and (b) the average angular velocity? (c) What is the instantaneous angular velocity of the disk at the end of the 5.0 s? (d) With the angular acceleration unchanged, through what additional angle will the disk turn during the next 5.0 s? •11 A disk, initially rotating at 120 rad/s, is slowed down with a constant angular acceleration of magnitude 4.0 rad/s2. (a) How much time does the disk take to stop? (b) Through what angle does the disk rotate during that time? •12 The angular speed of an automobile engine is increased at a constant rate from 1200 rev/min to 3000 rev/min in 12 s. (a) What is

Module 10-1 Rotational Variables •1 A good baseball pitcher can throw a baseball toward home plate at 85 mi/h with a spin of 1800 rev/min. How many revolutions does the baseball make on its way to home plate? For simplicity, assume that the 60 ft path is a straight line. •2 What is the angular speed of (a) the second hand, (b) the minute hand, and (c) the hour hand of a smoothly running analog watch? Answer in radians per second.

••6 The angular position of a point on the rim of a rotating wheel is given by u ! 4.0t % 3.0t 2 & t 3, where u is in radians and t is in seconds. What are the angular velocities at (a) t ! 2.0 s and (b) t ! 4.0 s? (c) What is the average angular acceleration for the time interval that begins at t ! 2.0 s and ends at t ! 4.0 s? What are the instantaneous angular accelerations at (d) the beginning and (e) the end of this time interval? •••7 The wheel in Fig. 10-30 has eight equally spaced spokes and a radius of 30 cm. It is mounted on a fixed axle and is spinning at 2.5 rev/s. You want to shoot a 20-cm-long arrow parallel to this axle and

288

CHAPTE R 10 ROTATION

its angular acceleration in revolutions per minute-squared? (b) How many revolutions does the engine make during this 12 s interval? ••13 ILW A flywheel turns through 40 rev as it slows from an angular speed of 1.5 rad/s to a stop. (a) Assuming a constant angular acceleration, find the time for it to come to rest. (b) What is its angular acceleration? (c) How much time is required for it to complete the first 20 of the 40 revolutions? ••14 A disk rotates about its central axis starting from rest and accelerates with constant angular acceleration. At one time it is rotating at 10 rev/s; 60 revolutions later, its angular speed is 15 rev/s. Calculate (a) the angular acceleration, (b) the time required to complete the 60 revolutions, (c) the time required to reach the 10 rev/s angular speed, and (d) the number of revolutions from rest until the time the disk reaches the 10 rev/s angular speed. ••15 SSM Starting from rest, a wheel has constant a = 3.0 rad/s2. During a certain 4.0 s interval, it turns through 120 rad. How much time did it take to reach that 4.0 s interval? ••16 A merry-go-round rotates from rest with an angular acceleration of 1.50 rad/s2. How long does it take to rotate through (a) the first 2.00 rev and (b) the next 2.00 rev? ••17 At t ! 0, a flywheel has an angular velocity of 4.7 rad/s, a constant angular acceleration of %0.25 rad/s2, and a reference line at u0 ! 0. (a) Through what maximum angle umax will the reference line turn in the positive direction? What are the (b) first and (c) second times the reference line will be at u ! 12umax? At what (d) negative time and (e) positive time will the reference line be at u ! 10.5 rad? (f) Graph u versus t, and indicate your answers. •••18 A pulsar is a rapidly rotating neutron star that emits a radio beam the way a lighthouse emits a light beam. We receive a radio pulse for each rotation of the star. The period T of rotation is found by measuring the time between pulses. The pulsar in the Crab nebula has a period of rotation of T ! 0.033 s that is increasing at the rate of 1.26 $ 10%5 s/y. (a) What is the pulsar’s angular acceleration a? (b) If a is constant, how many years from now will the pulsar stop rotating? (c) The pulsar originated in a supernova explosion seen in the year 1054. Assuming constant a, find the initial T. Module 10-3 Relating the Linear and Angular Variables •19 What are the magnitudes of (a) the angular velocity, (b) the radial acceleration, and (c) the tangential acceleration of a spaceship taking a circular turn of radius 3220 km at a speed of 29 000 km/h? •20 An object rotates about a fixed axis, and the angular position of a reference line on the object is given by u ! 0.40e2t, where u is in radians and t is in seconds. Consider a point on the object that is 4.0 cm from the axis of rotation. At t ! 0, what are the magnitudes of the point’s (a) tangential component of acceleration and (b) radial component of acceleration? •21 Between 1911 and 1990, the top of the leaning bell tower at Pisa, Italy, moved toward the south at an average rate of 1.2 mm/y. The tower is 55 m tall. In radians per second, what is the average angular speed of the tower’s top about its base? •22 An astronaut is tested in a centrifuge with radius 10 m and rotating according to u ! 0.30t2. At t ! 5.0 s, what are the magnitudes of the (a) angular velocity, (b) linear velocity, (c) tangential acceleration, and (d) radial acceleration? •23 SSM WWW A flywheel with a diameter of 1.20 m is rotating at an angular speed of 200 rev/min. (a) What is the angular speed of the flywheel in radians per second? (b) What is the linear speed of a point on the rim of the flywheel? (c) What constant angular ac-

celeration (in revolutions per minute-squared) will increase the wheel’s angular speed to 1000 rev/min in 60.0 s? (d) How many revolutions does the wheel make during that 60.0 s? •24 A vinyl record is played by rotating the record so that an approximately circular groove in the vinyl slides under a stylus. Bumps in the groove run into the stylus, causing it to oscillate. The equipment converts those oscillations to electrical signals and then to sound. Suppose that a record turns at the rate of 3313 rev/min, the groove being played is at a radius of 10.0 cm, and the bumps in the groove are uniformly separated by 1.75 mm. At what rate (hits per second) do the bumps hit the stylus? ••25 SSM (a) What is the angular speed v about the polar axis of a point on Earth’s surface at latitude 40# N? (Earth rotates about that axis.) (b) What is the linear speed v of the point? What are (c) v and (d) v for a point at the equator? ••26 The flywheel of a steam engine runs with a constant angular velocity of 150 rev/min.When steam is shut off, the friction of the bearings and of the air stops the wheel in 2.2 h. (a) What is the constant angular acceleration, in revolutions per minute-squared, of the wheel during the slowdown? (b) How many revolutions does the wheel make before stopping? (c) At the instant the flywheel is turning at 75 rev/min, what is the tangential component of the linear acceleration of a flywheel particle that is 50 cm from the axis of rotation? (d) What is the magnitude of the net linear acceleration of the particle in (c)? ••27 A seed is on a turntable rotating at 3313 rev/min, 6.0 cm from the rotation axis. What are (a) the seed’s acceleration and (b) the least coefficient of static friction to avoid slippage? (c) If the turntable had undergone constant angular acceleration from rest in 0.25 s, what is the least coefficient to avoid slippage? ••28 In Fig. 10-31, wheel A of radius rA rA ! 10 cm is coupled by belt B to rC wheel C of radius rC ! 25 cm. The anA C gular speed of wheel A is increased from rest at a constant rate of B 1.6 rad/s2. Find the time needed for Figure 10-31 Problem 28. wheel C to reach an angular speed of 100 rev/min, assuming the belt does not slip. (Hint: If the belt does not slip, the linear speeds at the two rims must be equal.) ••29 Figure 10-32 shows an early method of measuring the speed of light that makes use of a rotating slotted wheel. A beam of

L

Light beam Light source

Mirror perpendicular to light beam

Rotating slotted wheel

Figure 10-32 Problem 29.

PROB LE M S

light passes through one of the slots at the outside edge of the wheel, travels to a distant mirror, and returns to the wheel just in time to pass through the next slot in the wheel. One such slotted wheel has a radius of 5.0 cm and 500 slots around its edge. Measurements taken when the mirror is L ! 500 m from the wheel indicate a speed of light of 3.0 $ 105 km/s. (a) What is the (constant) angular speed of the wheel? (b) What is the linear speed of a point on the edge of the wheel? ••30 A gyroscope flywheel of radius 2.83 cm is accelerated from rest at 14.2 rad/s2 until its angular speed is 2760 rev/min. (a) What is the tangential acceleration of a point on the rim of the flywheel during this spin-up process? (b) What is the radial acceleration of this point when the flywheel is spinning at full speed? (c) Through what distance does a point on the rim move during the spin-up? ••31 A disk, with a radius of 0.25 m, is to be rotated like a merrygo-round through 800 rad, starting from rest, gaining angular speed at the constant rate a1 through the first 400 rad and then losing angular speed at the constant rate %a1 until it is again at rest.The magnitude of the centripetal acceleration of any portion of the disk is not to exceed 400 m/s2. (a) What is the least time required for the rotation? (b) What is the corresponding value of a1? ••32 A car starts from rest and moves around a circular track of radius 30.0 m. Its speed increases at the constant rate of 0.500 m/s2. (a) What is the magnitude of its net linear acceleration 15.0 s later? (b) What angle does this net acceleration vector make with the car’s velocity at this time? Module 10-4 Kinetic Energy of Rotation •33 SSM Calculate the rota- ◊ (rad/s) tional inertia of a wheel that has ωs a kinetic energy of 24 400 J when rotating at 602 rev/min. •34 Figure 10-33 gives angular speed versus time for a thin rod that rotates around one end. The scale on the v axis is t (s) 0 0 1 2 3 4 5 6 set by vs ! 6.0 rad/s. (a) What is the magnitude of the rod’s anFigure 10-33 Problem 34. gular acceleration? (b) At t ! 4.0 s, the rod has a rotational kinetic energy of 1.60 J. What is its kinetic energy at t ! 0? Module 10-5 Calculating the Rotational Inertia •35 SSM Two uniform solid cylinders, each rotating about its central (longitudinal) axis at 235 rad/s, have the same mass of 1.25 kg but differ in radius.What is the rotational kinetic energy of (a) the smaller cylinder, of radius 0.25 m, and (b) the larger cylinder, of radius 0.75 m? Figure 10-34a shows a disk that can rotate about an axis at IB

Axis h

I (kg • m2)

•36

IA (a)

0

0.1 h (m) (b)

Figure 10-34 Problem 36.

0.2

289

a radial distance h from the center of the disk. Figure 10-34b gives the rotational inertia I of the disk about the axis as a function of that distance h, from the center out to the edge of the disk. The scale on the I axis is set by IA ! 0.050 kg)m2 and IB ! 0.150 kg)m2. What is the mass of the disk? •37 SSM Calculate the rotational inertia of a meter stick, with mass 0.56 kg, about an axis perpendicular to the stick and located at the 20 cm mark. (Treat the stick as a thin rod.) •38 Figure 10-35 shows three L 0.0100 kg particles that have been Axis glued to a rod of length L ! 6.00 cm O m m m and negligible mass. The assembly d d d can rotate around a perpendicular axis through point O at the left end. Figure 10-35 Problems If we remove one particle (that is, 38 and 62. 33% of the mass), by what percentage does the rotational inertia of the assembly around the rotation axis decrease when that removed particle is (a) the innermost one and (b) the outermost one? ••39 Trucks can be run on energy stored in a rotating flywheel, with an electric motor getting the flywheel up to its top speed of 200p rad/s. Suppose that one such flywheel is a solid, uniform cylinder with a mass of 500 kg and a radius of 1.0 m. (a) What is the kinetic energy of the flywheel after charging? (b) If the truck uses an average power of 8.0 kW, for how many minutes can it operate between chargings? ••40 Figure 10-36 shows an arrangement of 15 identical disks that have been glued together in a rod-like shape of length L ! 1.0000 m and (total) mass M ! 100.0 mg. The disks are uniform, and the disk arrangement can rotate about a perpendicular axis through its central disk at point O. (a) What is the rotational inertia of the arrangement about that axis? (b) If we approximated the arrangement as being a uniform rod of mass M and length L, what percentage error would we make in using the formula in Table 10-2e to calculate the rotational inertia? L O

Figure 10-36 Problem 40. ••41 In Fig. 10-37, two particles, ω m each with mass m ! 0.85 kg, are fasd tened to each other, and to a rotation M axis at O, by two thin rods, each with m d length d ! 5.6 cm and mass M ! M 1.2 kg. The combination rotates Rotation axis around the rotation axis with the an- O gular speed v ! 0.30 rad/s. Measured Figure 10-37 Problem 41. about O, what are the combination’s (a) rotational inertia and (b) kinetic energy? ••42 The masses and coordinates of four particles are as follows: 50 g, x ! 2.0 cm, y ! 2.0 cm; 25 g, x ! 0, y ! 4.0 cm; 25 g, x ! %3.0 cm, y ! %3.0 cm; 30 g, x ! %2.0 cm, y ! 4.0 cm. What are the rotational inertias of this collection about the (a) x, (b) y, and (c) z axes? (d) Suppose that we symbolize the answers to (a) and (b) as A and B, respectively. Then what is the answer to (c) in terms of A and B?

290

CHAPTE R 10 ROTATION

••43 SSM WWW The uniform solid Rotation block in Fig. 10-38 has mass 0.172 kg axis and edge lengths a ! 3.5 cm, b ! 8.4 cm, and c ! 1.4 cm. Calculate its rotational inertia about an axis through one corner and perpendicular to the large faces.

b

c

a

••44 Four identical particles of Figure 10-38 Problem 43. mass 0.50 kg each are placed at the vertices of a 2.0 m $ 2.0 m square and held there by four massless rods, which form the sides of the square. What is the rotational inertia of this rigid body about an axis that (a) passes through the midpoints of opposite sides and lies in the plane of the square, (b) passes through the midpoint of one of the sides and is perpendicular to the plane of the square, and (c) lies in the plane of the square and passes through two diagonally opposite particles? Module 10-6 Torque •45 SSM ILW The body in Fig. 10-39 is pivoted at O, and two forces act on it as shown. If r1 ! 1.30 m, r2 ! 2.15 m, F1 ! 4.20 N, F2 ! 4.90 N, u1 ! 75.0#, and u2 ! 60.0#, what is the net torque about the pivot? •46 The body in Fig. 10-40 is pivoted at O. Three forces act on it: FA ! 10 N at point A, 8.0 m from O; FB ! 16 N at B, 4.0 m from O; and FC ! 19 N at C, 3.0 m from O. What is the net torque about O?

rest, block 2 falls 75.0 cm in 5.00 s without the cord slipping on the pulley. (a) What is the magnitude of the acceleration of the blocks? What are (b) tension T2 and (c) tension T1? (d) What is the magnitude of the pulley’s angular acceleration? (e) What is its rotational inertia? ••52 In Fig. 10-42, a cylinder having a mass of 2.0 kg can rotate about its central axis through point O. Forces are applied as shown: F1 ! 6.0 N, F2 ! 4.0 N, F3 ! 2.0 N, and F4 ! 5.0 N. Also, r ! 5.0 cm and R ! 12 cm. Find the (a) magnitude and (b) direction of the angular acceleration of the cylinder. (During the rotation, the forces maintain their same angles relative to the cylinder.)

F1 F4 Rotation axis

θ2

θ1

r1

F1

r2

O

F2

Figure 10-42 Problem 52.

FA 135°

C 160°

FB

A

O

90° B

•47 SSM A small ball of mass 0.75 kg is attached to one end Figure 10-40 Problem 46. of a 1.25-m-long massless rod, and the other end of the rod is hung from a pivot. When the resulting pendulum is 30# from the vertical, what is the magnitude of the gravitational torque calculated about the pivot? •48 The length of a bicycle pedal arm is 0.152 m, and a downward force of 111 N is applied to the pedal by the rider. What is the magnitude of the torque about the pedal arm’s pivot when the arm is at angle (a) 30#, (b) 90#, and (c) 180# with the vertical? Module 10-7 Newton’s Second Law for Rotation •49 SSM ILW During the launch from a board, a diver’s angular speed about her center of mass changes from zero to 6.20 rad/s in 220 ms. Her rotational inertia about her center of mass is 12.0 kg )m2. During the launch, what are the magnitudes of (a) her average angular acceleration and (b) the average external torque on her from the board? •50 If a 32.0 N )m torque on a wheel causes angular acceleration 25.0 rad/s2, what is the wheel’s rotational inertia?

R

T1

O r

F2

Figure 10-39 Problem 45.

FC

R

T2

••51 In Fig. 10-41, block 1 has mass m1 ! 460 g, block 2 has mass m2 ! 500 g, m m2 1 and the pulley, which is mounted on a horizontal axle with negligible friction, has Figure 10-41 radius R ! 5.00 cm. When released from Problems 51 and 83.

F3

••53 Figure 10-43 shows a uniform disk that can rotate around its center like a merry-go-round. The disk has a radius of 2.00 cm and a mass of 20.0 grams and is ini- F2 tially at rest. Starting at time t ! 0, two forces are to be applied tangentially to the rim as indicated, so that at time t ! 1.25 s the disk has an angular velocity of 250 : rad/s counterclockwise. Force F1 has a magnitude of 0.100 N. What is magnitude F2?

F1

Figure 10-43 Problem 53.

••54 In a judo foot-sweep Fa move, you sweep your opponent’s left foot out from under him while com pulling on his gi (uniform) toward that side. As a result, your oppoFg nent rotates around his right foot h and onto the mat. Figure 10-44 shows a simplified diagram of your opponent as you face him, with his left foot swept out. The rotational axis is through point O. : O The gravitational force Fg on him effectively acts at his center of d mass, which is a horizontal disFigure 10-44 Problem 54. tance d ! 28 cm from point O. His mass is 70 kg, and his rotational inertia about point O is 65 kg )m2. What is the magnitude of his initial : angular acceleration about point O if your pull Fa on his gi is (a) negligible and (b) horizontal with a magnitude of 300 N and applied at height h ! 1.4 m? ••55 In Fig. 10-45a, an irregularly shaped plastic plate with uniform thickness and density (mass per unit volume) is to be rotated around an axle that is perpendicular to the plate face and through point O. The rotational inertia of the plate about

PROB LE M S

that axle is measured with the following method. A circular disk of mass 0.500 kg and radius 2.00 cm is glued to the plate, with its center aligned with point O (Fig. 10-45b). A string is wrapped around the edge of the disk the way a string is wrapped around a top. Then the string is pulled for 5.00 s. As a result, the disk and plate are rotated by a constant force of 0.400 N that is applied by the string tangentially to the edge of the disk. The resulting angular speed is 114 rad/s. What is the rotational inertia of the plate about the axle?

Plate Axle

O

(a) Disk String (b)

Figure 10-45 Problem 55.

••56 Figure 10-46 shows particles 1 and 2, each of mass L1 L2 m, fixed to the ends of a rigid massless rod of length L1 & 2 L2, with L1 ! 20 cm and L2 ! 1 Figure 10-46 Problem 56. 80 cm. The rod is held horizontally on the fulcrum and then released. What are the magnitudes of the initial accelerations of (a) particle 1 and (b) particle 2? •••57 A pulley, with a rotational inertia of 1.0 $ 10%3 kg )m2 about its axle and a radius of 10 cm, is acted on by a force applied tangentially at its rim.The force magnitude varies in time as F ! 0.50t & 0.30t2, with F in newtons and t in seconds. The pulley is initially at rest. At t ! 3.0 s what are its (a) angular acceleration and (b) angular speed? Module 10-8 Work and Rotational Kinetic Energy •58 (a) If R ! 12 cm, M ! 400 g, and m ! 50 g in Fig. 10-19, find the speed of the block after it has descended 50 cm starting from rest. Solve the problem using energy conservation principles. (b) Repeat (a) with R ! 5.0 cm. •59 An automobile crankshaft transfers energy from the engine to the axle at the rate of 100 hp (! 74.6 kW) when rotating at a speed of 1800 rev/min. What torque (in newton-meters) does the crankshaft deliver? •60 A thin rod of length 0.75 m and mass 0.42 kg is suspended freely from one end. It is pulled to one side and then allowed to swing like a pendulum, passing through its lowest position with angular speed 4.0 rad/s. Neglecting friction and air resistance, find (a) the rod’s kinetic energy at its lowest position and (b) how far above that position the center of mass rises. •61 A 32.0 kg wheel, essentially a thin hoop with radius 1.20 m, is rotating at 280 rev/min. It must be brought to a stop in 15.0 s. (a) How much work must be done to stop it? (b) What is the required average power? ••62 In Fig. 10-35, three 0.0100 kg particles have been glued to a rod of length L ! 6.00 cm and negligible mass and can rotate around a perpendicular axis through point O at one end. How much work is required to change the rotational rate (a) from 0 to 20.0 rad/s, (b) from 20.0 rad/s to 40.0 rad/s, and (c) from 40.0 rad/s to 60.0 rad/s? (d) What is the slope of a plot of the assembly’s kinetic energy (in joules) versus the square of its rotation rate (in radianssquared per second-squared)? ••63 SSM ILW A meter stick is held vertically with one end on the floor and is then allowed to fall. Find the speed of the other end just before it hits the floor, assuming that the end on the floor does not slip. (Hint: Consider the stick to be a thin rod and use the conservation of energy principle.)

291

••64 A uniform cylinder of radius 10 cm and mass 20 kg is mounted so as to rotate freely about a horizontal axis that is parallel to and 5.0 cm from the central longitudinal axis of the cylinder. (a) What is the rotational inertia of the cylinder about the axis of rotation? (b) If the cylinder is released from rest with its central longitudinal axis at the same height as the axis about which the cylinder rotates, what is the angular speed of the cylinder as it passes through its lowest position? •••65 A tall, cylindrical chimney falls over when its base is ruptured. Treat the chimney as a thin rod of length 55.0 m. At the instant it makes an angle of 35.0# with the vertical as it falls, what are (a) the radial acceleration of the top, and (b) the tangential acceleration of the top. (Hint: Use energy considerations, not a torque.) (c) At what angle u is the tangential acceleration equal to g? •••66 A uniform spherical shell of mass M ! 4.5 kg and radius R ! 8.5 cm can rotate about a vertical axis on frictionless bearings (Fig. 10-47). A massless cord passes around the equator of the shell, over a pulley of rotational inertia I ! 3.0 $ 10%3 kg )m2 and radius r ! 5.0 cm, and is attached to a small object of mass m ! 0.60 kg. There is no friction on the pulley’s axle; the cord does not slip on the pulley. What is the speed of the object when it has fallen 82 cm after being released from rest? Use energy considerations. M, R I, r

Figure 10-47 Problem 66.

m

•••67 Figure 10-48 shows a rigid assembly of a thin hoop (of mass m and raHoop dius R ! 0.150 m) and a thin radial rod (of mass m and length L ! 2.00R). The assembly is upright, but if we give it a Rod slight nudge, it will rotate around a horiRotation zontal axis in the plane of the rod and axis hoop, through the lower end of the rod. Figure 10-48 Problem 67. Assuming that the energy given to the assembly in such a nudge is negligible, what would be the assembly’s angular speed about the rotation axis when it passes through the upside-down (inverted) orientation? Additional Problems 68 Two uniform solid spheres have the same mass of 1.65 kg, but one has a radius of 0.226 m and the other has a radius of 0.854 m. Each can rotate about an axis through its center. (a) What is the magnitude t of the torque required to bring the smaller sphere from rest to an angular speed of 317 rad/s in 15.5 s? (b) What is the magnitude F of the force that must be applied tangentially at the sphere’s equator to give that torque? What are the corresponding values of (c) t and O (d) F for the larger sphere? 69 In Fig. 10-49, a small disk of radius r ! 2.00 cm has been glued to the edge of a larger disk of radius R ! 4.00 cm so that

Figure 10-49 Problem 69.

292

CHAPTE R 10 ROTATION

the disks lie in the same plane. The disks can be rotated around a perpendicular axis through point O at the center of the larger disk. The disks both have a uniform density (mass per unit volume) of 1.40 $ 103 kg/m3 and a uniform thickness of 5.00 mm. What is the rotational inertia of the two-disk assembly about the rotation axis through O? 70 A wheel, starting from rest, rotates with a constant angular acceleration of 2.00 rad/s2. During a certain 3.00 s interval, it turns through 90.0 rad. (a) What is the angular velocity of the wheel at the start of the 3.00 s interval? (b) How long has the wheel been turning before the start of the 3.00 s interval? 71 SSM In Fig. 10-50, two 6.20 kg T2 blocks are connected by a massless string over a pulley of radius 2.40 cm and rotational inertia 7.40 $ 10%4 T1 kg )m2. The string does not slip on the pulley; it is not known whether there is friction between the table and the sliding block; the pulley’s axis is Figure 10-50 Problem 71. frictionless. When this system is released from rest, the pulley turns through 0.130 rad in 91.0 ms and the acceleration of the blocks is constant. What are (a) the magnitude of the pulley’s angular acceleration, (b) the magnitude of either block’s acceleration, (c) string tension T1, and (d) string tension T2? 72 Attached to each end of a thin steel rod of length 1.20 m and mass 6.40 kg is a small ball of mass 1.06 kg. The rod is constrained to rotate in a horizontal plane about a vertical axis through its midpoint. At a certain instant, it is rotating at 39.0 rev/s. Because of friction, it slows to a stop in 32.0 s.Assuming a constant retarding torque due to friction, compute (a) the angular acceleration, (b) the retarding torque, (c) the total energy transferred from mechanical energy to thermal energy by friction, and (d) the number of revolutions rotated during the 32.0 s. (e) Now suppose that the retarding torque is known not to be constant. If any of the quantities (a), (b), (c), and (d) can still be computed without additional information, give its value. 73 A uniform helicopter rotor blade is 7.80 m long, has a mass of 110 kg, and is attached to the rotor axle by a single bolt. (a) What is the magnitude of the force on the bolt from the axle when the rotor is turning at 320 rev/min? (Hint: For this calculation the blade can be considered to be a point mass at its center of mass. Why?) (b) Calculate the torque that must be applied to the rotor to bring it to full speed from rest in 6.70 s. Ignore air resistance. (The blade cannot be considered to be a point mass for this calculation. Why not? Assume the mass distribution of a uniform thin rod.) (c) How much work does the torque do on the blade in order for the blade to reach a speed of 320 rev/min? 74 Racing disks. Figure 10-51 shows two disks that can rotate about their centers like a merry-go-round. At time t ! 0, the reference lines of the two disks have the same orientation. Disk A Disk B Disk A is already rotating, with a conFigure 10-51 Problem 74. stant angular velocity of 9.5 rad/s. Disk B has been stationary but now begins to rotate at a constant angular acceleration of 2.2 rad/s2. (a) At what time t will the reference lines of the two disks momentarily have the same angular displacement u? (b) Will that time t be the first time since t ! 0 that the reference lines are momentarily aligned? 75 A high-wire walker always attempts to keep his center of mass over the wire (or rope). He normally carries a long, heavy pole

to help: If he leans, say, to his right (his com moves to the right) and is in danger of rotating around the wire, he moves the pole to his left (its com moves to the left) to slow the rotation and allow himself time to adjust his balance. Assume that the walker has a mass of 70.0 kg and a rotational inertia of 15.0 kg)m2 about the wire.What is the magnitude of his angular acceleration about the wire if his com is 5.0 cm to the right of the wire and (a) he carries no pole and (b) the 14.0 kg pole he carries has its com 10 cm to the left of the wire? 76 Starting from rest at t ! 0, a wheel undergoes a constant angular acceleration. When t ! 2.0 s, the angular velocity of the wheel is 5.0 rad/s. The acceleration continues until t ! 20 s, when it abruptly ceases. Through what angle does the wheel rotate in the interval t ! 0 to t ! 40 s? 77 SSM A record turntable rotating at 3313 rev/min slows down and stops in 30 s after the motor is turned off. (a) Find its (constant) angular acceleration in revolutions per minute-squared. (b) How many revolutions does it make in this time? 78 A rigid body is made of three L identical thin rods, each with length L L ! 0.600 m, fastened together in the form of a letter H (Fig. 10-52). The L body is free to rotate about a horizontal axis that runs along the length Figure 10-52 Problem 78. of one of the legs of the H. The body is allowed to fall from rest from a position in which the plane of the H is horizontal. What is the angular speed of the body when the plane of the H is vertical? 79 SSM (a) Show that the rotational inertia of a solid cylinder of mass M and radius R about its central axis is equal to the rotational inertia of a thin hoop of mass M and radius R/22 about its central axis. (b) Show that the rotational inertia I of any given body of mass M about any given axis is equal to the rotational inertia of an equivalent hoop about that axis, if the hoop has the same mass M and a radius k given by I k! . AM

The radius k of the equivalent hoop is called the radius of gyration of the given body.

80 A disk rotates at constant angular acceleration, from angular position u1 ! 10.0 rad to angular position u2 ! 70.0 rad in 6.00 s. Its angular velocity at u2 is 15.0 rad/s. (a) What was its angular velocity at u1? (b) What is the angular acceleration? (c) At what angular position was the disk initially at rest? (d) Graph u versus time t and angular speed v versus t for the disk, from the beginning of the motion (let t ! 0 then). 81 The thin uniform rod in Fig. 10-53 has length 2.0 m and can pivot about a horizontal, frictionless pin through one end. It is released from rest at angle u ! 40# above the horizontal. Use the principle of conservation of energy to determine the angular speed of the rod as it passes through the horizontal position.

θ Pin

Figure 10-53 Problem 81.

82 George Washington Gale Ferris, Jr., a civil engineering graduate from Rensselaer Polytechnic Institute, built the original Ferris wheel for the 1893 World’s Columbian Exposition in Chicago. The wheel, an astounding engineering construction at the time, carried 36 wooden cars, each holding up to 60 passengers, around a circle 76 m in diameter. The cars were loaded 6 at a time, and once all 36 cars were full, the wheel made a complete

293

PROB LE M S

rotation at constant angular speed in about 2 min. Estimate the amount of work that was required of the machinery to rotate the passengers alone.

the circle in which the pedals rotate to be 0.40 m, and determine the magnitude of the maximum torque he exerts about the rotation axis of the pedals.

83 In Fig. 10-41, two blocks, of mass m1 ! 400 g and m2 ! 600 g, are connected by a massless cord that is wrapped around a uniform disk of mass M ! 500 g and radius R ! 12.0 cm. The disk can rotate without friction about a fixed horizontal axis through its center; the cord cannot slip on the disk. The system is released from rest. Find (a) the magnitude of the acceleration of the blocks, (b) the tension T1 in the cord at the left, and (c) the tension T2 in the cord at the right.

90 The flywheel of an engine is rotating at 25.0 rad/s. When the engine is turned off, the flywheel slows at a constant rate and stops in 20.0 s. Calculate (a) the angular acceleration of the flywheel, (b) the angle through which the flywheel rotates in stopping, and (c) the number of revolutions made by the flywheel in stopping.

84 At 7!14 A.M. on June 30, 1908, a huge explosion occurred above remote central Siberia, at latitude 61# N and longitude 102# E; the fireball thus created was the brightest flash seen by anyone before nuclear weapons. The Tunguska Event, which according to one chance witness “covered an enormous part of the sky,” was probably the explosion of a stony asteroid about 140 m wide. (a) Considering only Earth’s rotation, determine how much later the asteroid would have had to arrive to put the explosion above Helsinki at longitude 25# E. This would have obliterated the city. (b) If the asteroid had, instead, been a metallic asteroid, it could have reached Earth’s surface. How much later would such an asteroid have had to arrive to put the impact in the Atlantic Ocean at longitude 20# W? (The resulting tsunamis would have wiped out coastal civilization on both sides of the Atlantic.) 85 A golf ball is launched at an angle of 20# to the horizontal, with a speed of 60 m/s and a rotation rate of 90 rad/s. Neglecting air drag, determine the number of revolutions the ball makes by the time it reaches maximum height. 86 Figure 10-54 shows a flat construction of two circular rings that have a common center and are held together by three rods of negligible mass. The construction, which is initially at rest, can rotate around the common center (like a merryFigure 10-54 go-round), where another rod of negligible mass Problem 86. lies. The mass, inner radius, and outer radius of the rings are given in the following table. A tangential force of magnitude 12.0 N is applied to the outer edge of the outer ring for 0.300 s. What is the change in the angular speed of the construction during the time interval? Ring

Mass (kg)

Inner Radius (m)

Outer Radius (m)

1 2

0.120 0.240

0.0160 0.0900

0.0450 0.1400

87 In Fig. 10-55, a wheel of radius 0.20 m is mounted on a frictionless horizontal axle. A massless cord is wrapped around the wheel and attached to a 2.0 kg box that slides on θ a frictionless surface inclined at anFigure 10-55 Problem 87. gle u ! 20# with the horizontal. The box accelerates down the surface at 2.0 m/s2. What is the rotational inertia of the wheel about the axle? 88 A thin spherical shell has a radius of 1.90 m. An applied torque of 960 N )m gives the shell an angular acceleration of 6.20 rad/s2 about an axis through the center of the shell. What are (a) the rotational inertia of the shell about that axis and (b) the mass of the shell? 89 A bicyclist of mass 70 kg puts all his mass on each downwardmoving pedal as he pedals up a steep road. Take the diameter of

91 SSM In Fig. 10-19a, a wheel of radius 0.20 m is mounted on a frictionless horizontal axis. The rotational inertia of the wheel about the axis is 0.40 kg )m2. A massless cord wrapped around the wheel’s circumference is attached to a 6.0 kg box. The system is released from rest.When the box has a kinetic energy of 6.0 J, what are (a) the wheel’s rotational kinetic energy and (b) the distance the box has fallen? 92 Our Sun is 2.3 $ 10 4 ly (light-years) from the center of our Milky Way galaxy and is moving in a circle around that center at a speed of 250 km/s. (a) How long does it take the Sun to make one revolution about the galactic center? (b) How many revolutions has the Sun completed since it was formed about 4.5 $ 10 9 years ago? 93 SSM A wheel of radius 0.20 m P is mounted on a frictionless horizontal axis. The rotational inertia of the wheel about the axis is 0.050 kg )m2. A massless cord wrapped around Figure 10-56 Problem 93. the wheel is attached to a 2.0 kg block that slides on a horizontal frictionless surface. If a horizontal force of magnitude P ! 3.0 N is applied to the block as shown in Fig. 10-56, what is the magnitude of the angular acceleration of the wheel? Assume the cord does not slip on the wheel. 94 If an airplane propeller rotates at 2000 rev/min while the airplane flies at a speed of 480 km/h relative to the ground, what is the linear speed of a point on the tip of the propeller, at radius 1.5 m, as seen by (a) the pilot and (b) an observer on the ground? The plane’s velocity is parallel to the propeller’s axis of rotation. 95 The rigid body shown in Fig. 10-57 consists of three particles connected by massless rods. It is to be rotated about an axis perpendicular to its plane through point P. If M ! 0.40 kg, a ! 30 cm, and b ! 50 cm, how much work is required to take the body from rest to an angular speed of 5.0 rad/s?

M

b

b

a 2M

a P

2M

96 Beverage engineering. The pull Figure 10-57 Problem 95. tab was a major advance in the engineering design of beverage containers. The tab pivots on a central bolt in the can’s top. When you pull upward on one end of the tab, the other end presses downward on a portion of the can’s top that has been scored. If you pull upward with a 10 N force, what force magnitude acts on the scored section? (You will need to examine a can with a pull tab.) 97 Figure 10-58 shows a propeller blade that rotates at 2000 rev/min about a perpendicular axis at point B. Point A is at the outer tip of the blade, at radial distance A B 1.50 m. (a) What is the difference in the magnitudes a of the centripetal acceleration of point A and of a point at radial distance Figure 10-58 0.150 m? (b) Find the slope of a plot of a Problem 97. versus radial distance along the blade.

294

CHAPTE R 10 ROTATION

98 A yo-yo-shaped device Rigid mount Fapp mounted on a horizontal frictionless axis is used to lift a 30 kg box as shown in Fig. 10-59. The Hub outer radius R of the device is 0.50 m, and the radius r of the r hub is 0.20 m. When a constant : horizontal force Fapp of magniR tude 140 N is applied to a rope wrapped around the outside of Yo-yo-shaped Rope device the device, the box, which is suspended from a rope wrapped around the hub, has an upward acceleration of magnitude 0.80 Figure 10-59 Problem 98. m/s2. What is the rotational inertia of the device about its axis of rotation? 99 A small ball with mass 1.30 kg is mounted on one end of a rod 0.780 m long and of negligible mass. The system rotates in a horizontal circle about the other end of the rod at 5010 rev/min. (a) Calculate the rotational inertia of the system about the axis of rotation. (b) There is an air drag of 2.30 $ 10%2 N on the ball, directed opposite its motion. What torque must be applied to the system to keep it rotating at constant speed? 100 Two thin rods (each of mass 0.20 kg) are joined together to form a rigid body as shown in Fig. 10-60. One of the rods has length L1 ! 0.40 m, and the other has length L2 ! 0.50 m. What is the rotational inertia of this rigid body about (a) an axis that is perpendicular to the plane of the paper and passes through the center of the shorter rod and (b) an axis that is perpendicular to the plane of the paper and passes through the center of the longer rod?

L2

1L __ 2 1

1L __ 2 1

Figure 10-60 Problem 100.

101 In Fig. 10-61, four pulBelt 1 B leys are connected by two belts. Pulley A (radius 15 cm) B' A is the drive pulley, and it rotates at 10 rad/s. Pulley B (raDrive dius 10 cm) is connected by pulley belt 1 to pulley A. Pulley B/ Belt 2 (radius 5 cm) is concentric with pulley B and is rigidly attached C to it. Pulley C (radius 25 cm) is connected by belt 2 to pulley B/. Calculate (a) the linear speed of Figure 10-61 Problem 101. a point on belt 1, (b) the angular speed of pulley B, (c) the angular speed of pulley B/, (d) the linear speed of a point on belt 2, and (e) the angular speed of pulley C. (Hint: If the belt between two pulleys does not slip, the linear speeds at the rims of the two pulleys must be equal.) 102

The rigid object shown in Fig. 10-62 consists of three balls 2M L

θ θ

Figure 10-62 Problem 102.

L 2M

P

2L

M

and three connecting rods, with M ! 1.6 kg, L ! 0.60 m, and ' ! 30#. The balls may be treated as particles, and the connecting rods have negligible mass. Determine the rotational kinetic energy of the object if it has an angular speed of 1.2 rad/s about (a) an axis that passes through point P and is perpendicular to the plane of the figure and (b) an axis that passes through point P, is perpendicular to the rod of length 2L, and lies in the plane of the figure. 103 In Fig. 10-63, a thin uniform rod (mass 3.0 kg, length 4.0 m) rotates freely about a horizontal axis A that is perpendicular to the rod and passes through a point at distance d ! 1.0 m from the end of the rod. The kinetic energy of the rod as it passes through the vertical position is 20 J. (a) What is the rotational inertia of the rod about axis A? (b) What is the (linear) speed of the end B of the rod as the rod passes through the vertical position? (c) At what angle u will the rod momentarily stop in its upward swing? 104 Four particles, each of mass, 0.20 kg, are placed at the vertices of a square with sides of length 0.50 m. The particles are connected by rods of negligible mass. This rigid body can rotate in a vertical plane about a horizontal axis A that passes through one of the particles. The body is released from rest with rod AB horizontal (Fig. 10-64). (a) What is the rotational inertia of the body about axis A? (b) What is the angular speed of the body about axis A when rod AB swings through the vertical position?

d

A

θ

B

Figure 10-63 Problem 103.

Rotation axis A

B

Figure 10-64 Problem 104.

105 Cheetahs running at top speed have been reported at an astounding 114 km/h (about 71 mi/h) by observers driving alongside the animals. Imagine trying to measure a cheetah’s speed by keeping your vehicle abreast of the animal while also glancing at your speedometer, which is registering 114 km/h. You keep the vehicle a constant 8.0 m from the cheetah, but the noise of the vehicle causes the cheetah to continuously veer away from you along a circular path of radius 92 m. Thus, you travel along a circular path of radius 100 m. (a) What is the angular speed of you and the cheetah around the circular paths? (b) What is the linear speed of the cheetah along its path? (If you did not account for the circular motion, you would conclude erroneously that the cheetah’s speed is 114 km/h, and that type of error was apparently made in the published reports.) 106 A point on the rim of a 0.75-m-diameter grinding wheel changes speed at a constant rate from 12 m/s to 25 m/s in 6.2 s. What is the average angular acceleration of the wheel? 107 A pulley wheel that is 8.0 cm in diameter has a 5.6-m-long cord wrapped around its periphery. Starting from rest, the wheel is given a constant angular acceleration of 1.5 rad/s2. (a) Through what angle must the wheel turn for the cord to unwind completely? (b) How long will this take? 108 A vinyl record on a turntable rotates at 3313 rev/min. (a) What is its angular speed in radians per second? What is the linear speed of a point on the record (b) 15 cm and (c) 7.4 cm from the turntable axis?

C

H

A

P

T

E

R

1

1

Rolling, Torque, and Angular Momentum 11-1 ROLLING AS TRANSLATION AND ROTATION COMBINED Learning Objectives After reading this module, you should be able to . . .

11.01 Identify that smooth rolling can be considered as a combination of pure translation and pure rotation.

11.02 Apply the relationship between the center-of-mass speed and the angular speed of a body in smooth rolling.

Key Ideas ● For a wheel of radius R rolling smoothly,

vcom ! vR, where vcom is the linear speed of the wheel’s center of mass and v is the angular speed of the wheel about its center.

● The wheel may also be viewed as rotating instantaneously

about the point P of the “road” that is in contact with the wheel. The angular speed of the wheel about this point is the same as the angular speed of the wheel about its center.

What Is Physics? As we discussed in Chapter 10, physics includes the study of rotation. Arguably, the most important application of that physics is in the rolling motion of wheels and wheel-like objects. This applied physics has long been used. For example, when the prehistoric people of Easter Island moved their gigantic stone statues from the quarry and across the island, they dragged them over logs acting as rollers. Much later, when settlers moved westward across America in the 1800s, they rolled their possessions first by wagon and then later by train. Today, like it or not, the world is filled with cars, trucks, motorcycles, bicycles, and other rolling vehicles. The physics and engineering of rolling have been around for so long that you might think no fresh ideas remain to be developed. However, skateboards and inline skates were invented and engineered fairly recently, to become huge financial successes. Street luge is now catching on, and the self-righting Segway (Fig. 11-1) may change the way people move around in large cities. Applying the physics of rolling can still lead to surprises and rewards. Our starting point in exploring that physics is to simplify rolling motion.

Rolling as Translation and Rotation Combined Here we consider only objects that roll smoothly along a surface; that is, the objects roll without slipping or bouncing on the surface. Figure 11-2 shows how complicated smooth rolling motion can be: Although the center of the object moves in a straight line parallel to the surface, a point on the rim certainly does not. However, we can study this motion by treating it as a combination of translation of the center of mass and rotation of the rest of the object around that center.

Justin Sullivan/Getty Images, Inc.

Figure 11-1 The self-righting Segway Human Transporter. 295

296

CHAPTE R 11 ROLLI NG, TORQU E, AN D ANG U L AR M OM E NTU M

Figure 11-2 A time-exposure photograph of a rolling disk. Small lights have been attached to the disk, one at its center and one at its edge. The latter traces out a curve called a cycloid. Richard Megna/Fundamental Photographs

O

vcom

vcom

θ

O

s P

P s

Figure 11-3 The center of mass O of a rolling vcom wheel moves a distance s at velocity : while the wheel rotates through angle u. The point P at which the wheel makes contact with the surface over which the wheel rolls also moves a distance s.

To see how we do this, pretend you are standing on a sidewalk watching the bicycle wheel of Fig. 11-3 as it rolls along a street. As shown, you see the center of mass O of the wheel move forward at constant speed vcom. The point P on the street where the wheel makes contact with the street surface also moves forward at speed vcom, so that P always remains directly below O. During a time interval t, you see both O and P move forward by a distance s. The bicycle rider sees the wheel rotate through an angle u about the center of the wheel, with the point of the wheel that was touching the street at the beginning of t moving through arc length s. Equation 10-17 relates the arc length s to the rotation angle u: (11-1)

s ! uR,

where R is the radius of the wheel. The linear speed vcom of the center of the wheel (the center of mass of this uniform wheel) is ds/dt. The angular speed v of the wheel about its center is du/dt. Thus, differentiating Eq. 11-1 with respect to time (with R held constant) gives us vcom ! vR

(11-2)

(smooth rolling motion).

A Combination. Figure 11-4 shows that the rolling motion of a wheel is a combination of purely translational and purely rotational motions. Figure 11-4a shows the purely rotational motion (as if the rotation axis through the center were stationary): Every point on the wheel rotates about the center with angular speed v. (This is the type of motion we considered in Chapter 10.) Every point on the outside edge of the wheel has linear speed vcom given by Eq. 11-2. Figure 11-4b shows the purely translational motion (as if the wheel did not rotate at all): Every point on the wheel moves to the right with speed vcom. The combination of Figs. 11-4a and 11-4b yields the actual rolling motion of the wheel, Fig. 11-4c. Note that in this combination of motions, the portion of the wheel at the bottom (at point P) is stationary and the portion at the top ( a ) Pure rotation

v = vcom T

ω

O P

v = –vcom

+

( b ) Pure translation

=

( c ) Rolling motion v = 2vcom

v = vcom T

T

vcom

O

ω P

v = vcom

vcom O P

v = –vcom + vcom = 0

Figure 11-4 Rolling motion of a wheel as a combination of purely rotational motion and purely translational motion. (a) The purely rotational motion: All points on the wheel move with the same angular speed v. Points on the outside edge of the wheel all move v of two such points, at top (T) with the same linear speed v ! vcom. The linear velocities : and bottom (P) of the wheel, are shown. (b) The purely translational motion: All points on the wheel move to the right with the same linear velocity v:com. (c) The rolling motion of the wheel is the combination of (a) and (b).

11-1 ROLLI NG AS TRANSL ATION AN D ROTATION COM B I N E D

Figure 11-5 A photograph of a rolling bicycle wheel. The spokes near the wheel’s top are more blurred than those near the bottom because the top ones are moving faster, as Fig. 11-4c shows.

297

Courtesy Alice Halliday

(at point T ) is moving at speed 2vcom, faster than any other portion of the wheel. These results are demonstrated in Fig. 11-5, which is a time exposure of a rolling bicycle wheel. You can tell that the wheel is moving faster near its top than near its bottom because the spokes are more blurred at the top than at the bottom. The motion of any round body rolling smoothly over a surface can be separated into purely rotational and purely translational motions, as in Figs. 11-4a and 11-4b.

Rolling as Pure Rotation Figure 11-6 suggests another way to look at the rolling motion of a wheel — namely, as pure rotation about an axis that always extends through the point where the wheel contacts the street as the wheel moves. We consider the rolling motion to be pure rotation about an axis passing through point P in Fig. 11-4c and perpendicular to the plane of the figure. The vectors in Fig. 11-6 then represent the instantaneous velocities of points on the rolling wheel. Question: What angular speed about this new axis will a stationary observer assign to a rolling bicycle wheel? Answer: The same v that the rider assigns to the wheel as she or he observes it in pure rotation about an axis through its center of mass. To verify this answer, let us use it to calculate the linear speed of the top of the rolling wheel from the point of view of a stationary observer. If we call the wheel’s radius R, the top is a distance 2R from the axis through P in Fig. 11-6, so the linear speed at the top should be (using Eq. 11-2) vtop ! (v)(2R) ! 2(vR) ! 2vcom, in exact agreement with Fig. 11-4c. You can similarly verify the linear speeds shown for the portions of the wheel at points O and P in Fig. 11-4c.

Checkpoint 1 The rear wheel on a clown’s bicycle has twice the radius of the front wheel. (a) When the bicycle is moving, is the linear speed at the very top of the rear wheel greater than, less than, or the same as that of the very top of the front wheel? (b) Is the angular speed of the rear wheel greater than, less than, or the same as that of the front wheel?

T

O

Rotation axis at P

Figure 11-6 Rolling can be viewed as pure rotation, with angular speed v, about an axis that always extends through P. The vectors show the instantaneous linear velocities of selected points on the rolling wheel. You can obtain the vectors by combining the translational and rotational motions as in Fig. 11-4.

298

CHAPTE R 11 ROLLI NG, TORQU E, AN D ANG U L AR M OM E NTU M

11-2 FORCES AND KINETIC ENERGY OF ROLLING Learning Objectives After reading this module, you should be able to . . .

11.03 Calculate the kinetic energy of a body in smooth rolling as the sum of the translational kinetic energy of the center of mass and the rotational kinetic energy around the center of mass. 11.04 Apply the relationship between the work done on a smoothly rolling object and the change in its kinetic energy. 11.05 For smooth rolling (and thus no sliding), conserve mechanical energy to relate initial energy values to the values at a later point.

11.06 Draw a free-body diagram of an accelerating body that is smoothly rolling on a horizontal surface or up or down a ramp. 11.07 Apply the relationship between the center-of-mass acceleration and the angular acceleration. 11.08 For smooth rolling of an object up or down a ramp, apply the relationship between the object’s acceleration, its rotational inertia, and the angle of the ramp.

Key Ideas ● A smoothly rolling wheel has kinetic energy

K!

1 2 2 Icomv

&

angular acceleration a about the center with

1 2 2 3v com,

acom ! aR.

where Icom is the rotational inertia of the wheel about its center of mass and M is the mass of the wheel.

● If the wheel rolls smoothly down a ramp of angle u, its

acceleration along an x axis extending up the ramp is

● If the wheel is being accelerated but is still rolling smoothly,

acom, x ! %

the acceleration of the center of mass acom is related to the :

g sin u . 1 & Icom /MR2

The Kinetic Energy of Rolling Let us now calculate the kinetic energy of the rolling wheel as measured by the stationary observer. If we view the rolling as pure rotation about an axis through P in Fig. 11-6, then from Eq. 10-34 we have K ! 12IPv 2,

(11-3)

in which v is the angular speed of the wheel and IP is the rotational inertia of the wheel about the axis through P. From the parallel-axis theorem of Eq. 10-36 (I ! Icom & Mh2), we have IP ! Icom & MR2, (11-4) in which M is the mass of the wheel, Icom is its rotational inertia about an axis through its center of mass, and R (the wheel’s radius) is the perpendicular distance h. Substituting Eq. 11-4 into Eq. 11-3, we obtain K ! 12 Icomv 2 & 12 3R 2v 2, and using the relation vcom ! vR (Eq. 11-2) yields K ! 12 Icomv 2 & 12 3v 2com.

(11-5)

We can interpret the term 12 Icomv 2 as the kinetic energy associated with the rotation of the wheel about an axis through its center of mass (Fig. 11-4a), and the term 12Mv2com as the kinetic energy associated with the translational motion of the wheel’s center of mass (Fig. 11-4b). Thus, we have the following rule: A rolling object has two types of kinetic energy: a rotational kinetic energy (12 Icomv 2) due to its rotation about its center of mass and a translational kinetic energy (12Mv2com) due to translation of its center of mass.

11-2 FORCES AN D KI N ETIC E N E RGY OF ROLLI NG

299

The Forces of Rolling Friction and Rolling If a wheel rolls at constant speed, as in Fig. 11-3, it has no tendency to slide at the point of contact P, and thus no frictional force acts there. However, if a net force acts on the rolling wheel to speed it up or to slow it, then that net force causes acceleration : acom of the center of mass along the direction of travel. It also causes the wheel to rotate faster or slower, which means it causes an angular acceleration a. These accelerations tend to make the wheel slide at P. Thus, a frictional force must act on the wheel at P to oppose that tendency. : If the wheel does not slide, the force is a static frictional force f s and the motion is smooth rolling. We can then relate the magnitudes of the linear acceleration : acom and the angular acceleration a by differentiating Eq. 11-2 with respect to time (with R held constant). On the left side, dvcom/dt is acom, and on the right side dv/dt is a. So, for smooth rolling we have acom ! aR

(smooth rolling motion).

(11-6)

If the wheel does slide when the net force acts on it, the frictional force that : acts at P in Fig. 11-3 is a kinetic frictional force f k. The motion then is not smooth rolling, and Eq. 11-6 does not apply to the motion. In this chapter we discuss only smooth rolling motion. Figure 11-7 shows an example in which a wheel is being made to rotate faster while rolling to the right along a flat surface, as on a bicycle at the start of a race. The faster rotation tends to make the bottom of the wheel slide to the left at point P. A frictional force at P, directed to the right, opposes this tendency to : slide. If the wheel does not slide, that frictional force is a static frictional force f s (as shown), the motion is smooth rolling, and Eq. 11-6 applies to the motion. (Without friction, bicycle races would be stationary and very boring.) If the wheel in Fig. 11-7 were made to rotate slower, as on a slowing bicycle, we would change the figure in two ways: The directions of the center-of: mass acceleration : acom and the frictional force f s at point P would now be to the left.

Rolling Down a Ramp Figure 11-8 shows a round uniform body of mass M and radius R rolling smoothly down a ramp at angle u, along an x axis.We want to find an expression for the body’s

FN

Forces Fg sin θ and fs determine the linear acceleration down the ramp.

Forces FN and Fg cos θ merely balance.

Fg sin θ

R fs

x

P

θ

The torque due to fs determines the angular acceleration around the com.

θ Fg cos θ Fg

Figure 11-8 A round uniform body of radius R rolls down a ramp. The forces that act on it : : : are the gravitational force Fg , a normal force FN , and a frictional force fs pointing up the : ramp. (For clarity, vector FN has been shifted in the direction it points until its tail is at the center of the body.)

acom P

fs

Figure 11-7 A wheel rolls horizontally without sliding while accelerating with linear acceleration : acom, as on a bicycle at the start : of a race. A static frictional force f s acts on the wheel at P, opposing its tendency to slide.

300

CHAPTE R 11 ROLLI NG, TORQU E, AN D ANG U L AR M OM E NTU M

acceleration acom,x down the ramp. We do this by using Newton’s second law in both its linear version (Fnet ! Ma) and its angular version (tnet ! Ia). We start by drawing the forces on the body as shown in Fig. 11-8: :

1. The gravitational force Fg on the body is directed downward. The tail of the vector is placed at the center of mass of the body. The component along the ramp is Fg sin u, which is equal to Mg sin u. : 2. A normal force FN is perpendicular to the ramp. It acts at the point of contact P, but in Fig. 11-8 the vector has been shifted along its direction until its tail is at the body’s center of mass. :

3. A static frictional force f s acts at the point of contact P and is directed up the ramp. (Do you see why? If the body were to slide at P, it would slide down the ramp.Thus, the frictional force opposing the sliding must be up the ramp.) We can write Newton’s second law for components along the x axis in Fig. 11-8 (Fnet,x ! max) as f s % Mg sin u ! Macom,x. (11-7) This equation contains two unknowns, f s and acom,x. (We should not assume that f s is at its maximum value f s,max. All we know is that the value of f s is just right for the body to roll smoothly down the ramp, without sliding.) We now wish to apply Newton’s second law in angular form to the body’s rotation about its center of mass. First, we shall use Eq. 10-41 (t ! r"F ) to write the : torques on the body about that point. The frictional force f s has moment arm R and thus produces a torque Rfs, which is positive because it tends to rotate the : : body counterclockwise in Fig. 11-8. Forces Fg and FN have zero moment arms about the center of mass and thus produce zero torques. So we can write the angular form of Newton’s second law (tnet ! Ia) about an axis through the body’s center of mass as Rf s ! Icoma. (11-8) This equation contains two unknowns, f s and a. Because the body is rolling smoothly, we can use Eq. 11-6 (acom ! aR) to relate the unknowns acom,x and a. But we must be cautious because here acom,x is negative (in the negative direction of the x axis) and a is positive (counterclockwise). Thus we substitute %acom,x/R for a in Eq. 11-8.Then, solving for f s, we obtain acom,x . R2 Substituting the right side of Eq. 11-9 for fs in Eq. 11-7, we then find f s ! %Icom

acom,x ! %

g sin u . 1 & Icom /MR2

(11-9)

(11-10)

We can use this equation to find the linear acceleration acom,x of any body rolling along an incline of angle u with the horizontal. Note that the pull by the gravitational force causes the body to come down the ramp, but it is the frictional force that causes the body to rotate and thus roll. If you eliminate the friction (by, say, making the ramp slick with ice or grease) or arrange for Mg sin u to exceed fs,max, then you eliminate the smooth rolling and the body slides down the ramp.

Checkpoint 2 Disks A and B are identical and roll across a floor with equal speeds.Then disk A rolls up an incline, reaching a maximum height h, and disk B moves up an incline that is identical except that it is frictionless. Is the maximum height reached by disk B greater than, less than, or equal to h?

301

11-3 TH E YO-YO

Sample Problem 11.01 Ball rolling down a ramp A uniform ball, of mass M ! 6.00 kg and radius R, rolls smoothly from rest down a ramp at angle u ! 30.0# (Fig. 11-8).

Doing so, substituting 25MR2 for Icom (from Table 10-2f), and then solving for vcom give us

(a) The ball descends a vertical height h ! 1.20 m to reach the bottom of the ramp.What is its speed at the bottom?

vcom ! 2(107)gh ! 2(107)(9.8 m/s2)(1.20 m)

! 4.10 m/s. (Answer) Note that the answer does not depend on M or R.

KEY IDEAS The mechanical energy E of the ball – Earth system is conserved as the ball rolls down the ramp. The reason is that the only force doing work on the ball is the gravitational force, a conservative force. The normal force on the ball from the ramp does zero work because it is perpendicular to the ball’s path. The frictional force on the ball from the ramp does not transfer any energy to thermal energy because the ball does not slide (it rolls smoothly). Thus, we conserve mechanical energy (Ef ! Ei): Kf & Uf ! Ki & Ui,

(11-11)

where subscripts f and i refer to the final values (at the bottom) and initial values (at rest), respectively. The gravitational potential energy is initially Ui ! Mgh (where M is the ball’s mass) and finally Uf ! 0. The kinetic energy is initially Ki ! 0. For the final kinetic energy Kf, we need an additional idea: Because the ball rolls, the kinetic energy involves both translation and rotation, so we include them both by using the right side of Eq. 11-5. Calculations: Substituting into Eq. 11-11 gives us 2 ) & 0 ! 0 & Mgh, (12 Icomv 2 & 123v com

(11-12)

where Icom is the ball’s rotational inertia about an axis through its center of mass, vcom is the requested speed at the bottom, and v is the angular speed there. Because the ball rolls smoothly, we can use Eq. 11-2 to substitute vcom/R for v to reduce the unknowns in Eq. 11-12.

(b) What are the magnitude and direction of the frictional force on the ball as it rolls down the ramp? KEY IDEA Because the ball rolls smoothly, Eq. 11-9 gives the frictional force on the ball. Calculations: Before we can use Eq. 11-9, we need the ball’s acceleration acom,x from Eq. 11-10: acom,x ! % !%

g sin u g sin u !% 1 & Icom /MR 2 1 & 25MR2 /MR2 (9.8 m/s2) sin 30.0# ! %3.50 m/s2. 1 & 25

Note that we needed neither mass M nor radius R to find acom,x. Thus, any size ball with any uniform mass would have this smoothly rolling acceleration down a 30.0# ramp. We can now solve Eq. 11-9 as acom,x a ! %25MR2 com,x ! % 25Macom,x f s ! %Icom R2 R2 ! %25(6.00 kg)(%3.50 m/s2) ! 8.40 N.

(Answer)

Note that we needed mass M but not radius R. Thus, the frictional force on any 6.00 kg ball rolling smoothly down a 30.0# ramp would be 8.40 N regardless of the ball’s radius but would be larger for a larger mass.

Additional examples, video, and practice available at WileyPLUS

11-3 THE YO-YO Learning Objectives After reading this module, you should be able to . . .

11.09 Draw a free-body diagram of a yo-yo moving up or down its string. 11.10 Identify that a yo-yo is effectively an object that rolls smoothly up or down a ramp with an incline angle of 90#.

11.11 For a yo-yo moving up or down its string, apply the relationship between the yo-yo’s acceleration and its rotational inertia. 11.12 Determine the tension in a yo-yo’s string as the yo-yo moves up or down its string.

Key Idea ● A yo-yo, which travels vertically up or down a string, can be treated as a wheel rolling along an inclined plane at angle u ! 90#.

302

CHAPTE R 11 ROLLI NG, TORQU E, AN D ANG U L AR M OM E NTU M

The Yo-Yo A yo-yo is a physics lab that you can fit in your pocket. If a yo-yo rolls down its string for a distance h, it loses potential energy in amount mgh but gains kinetic energy in both translational (12Mv 2com) and rotational (12 Icomv 2) forms. As it climbs back up, it loses kinetic energy and regains potential energy. In a modern yo-yo, the string is not tied to the axle but is looped around it. When the yo-yo “hits” the bottom of its string, an upward force on the axle from the string stops the descent. The yo-yo then spins, axle inside loop, with only rotational kinetic energy. The yo-yo keeps spinning (“sleeping”) until you “wake it” by jerking on the string, causing the string to catch on the axle and the yo-yo to climb back up. The rotational kinetic energy of the yo-yo at the bottom of its string (and thus the sleeping time) can be considerably increased by throwing the yo-yo downward so that it starts down the string with initial speeds vcom and v instead of rolling down from rest. To find an expression for the linear acceleration acom of a yo-yo rolling down a string, we could use Newton’s second law (in linear and angular forms) just as we did for the body rolling down a ramp in Fig. 11-8. The analysis is the same except for the following:

T

R

R0 R0

Fg (a)

(b)

Figure 11-9 (a) A yo-yo, shown in cross section. The string, of assumed negligible thickness, is wound around an axle of radius R0. (b) A free-body diagram for the falling yo-yo. Only the axle is shown.

1. Instead of rolling down a ramp at angle u with the horizontal, the yo-yo rolls down a string at angle u ! 90# with the horizontal. 2. Instead of rolling on its outer surface at radius R, the yo-yo rolls on an axle of radius R0 (Fig. 11-9a). :

3. Instead of being slowed by frictional force f s , the yo-yo is slowed by the force : T on it from the string (Fig. 11-9b). The analysis would again lead us to Eq. 11-10. Therefore, let us just change the notation in Eq. 11-10 and set u ! 90# to write the linear acceleration as acom ! %

g , 1 & Icom /MR20

(11-13)

where Icom is the yo-yo’s rotational inertia about its center and M is its mass. A yo-yo has the same downward acceleration when it is climbing back up.

11-4 TORQUE REVISITED Learning Objectives After reading this module, you should be able to . . .

11.13 Identify that torque is a vector quantity. 11.14 Identify that the point about which a torque is calculated must always be specified. 11.15 Calculate the torque due to a force on a particle by taking the cross product of the particle’s position vector

and the force vector, in either unit-vector notation or magnitude-angle notation. 11.16 Use the right-hand rule for cross products to find the direction of a torque vector.

Key Ideas ● In three dimensions, torque

t: is a vector quantity defined relative to a fixed point (usually an origin); it is :

t ! r $ F, :

:

:

where F is a force applied to a particle and :r is a position vector locating the particle relative to the fixed point.

● The magnitude of

t: is given by

t ! rF sin f ! rF" ! r"F, : where f is the angle between F and :r , F" is the component : : of F perpendicular to :r , and r" is the moment arm of F . ● The direction of

products.

t: is given by the right-hand rule for cross

303

11-4 TORQU E R EVISITE D z

z

Cross r into F. Torque τ is in the positive z direction.

τ (= r × F) y

O

O

φ

F (redrawn, with tail at origin) F

r

r A φ (a)

z

F

A

φ

y

r

F (c) x

:

y

O

φ

(b) x

x

τ

r

φ

Line of action of F

A

Figure 11-10 Defining torque. (a) A force F , lying in an xy plane, acts on a particle at point A. (b) This force produces a torque : t: (! : r $ F ) on the particle with respect to the origin O. By the right-hand rule for vector (cross) products, the torque vector points in the positive direction of z. Its magnitude is given by rF" in (b) and by r"F in (c).

Torque Revisited In Chapter 10 we defined torque t for a rigid body that can rotate around a fixed axis. We now expand the definition of torque to apply it to an individual particle that moves along any path relative to a fixed point (rather than a fixed axis). The path need no longer be a circle, and we must write the torque as a vector t: that may have any direction. We can calculate the magnitude of the torque with a formula and determine its direction with the right-hand rule for cross products. : Figure 11-10a shows such a particle at point A in an xy plane. A single force F in that plane acts on the particle, and the particle’s position relative to the origin O is given by position vector : r . The torque t: acting on the particle relative to the fixed point O is a vector quantity defined as :

t!: r $ F

:

(11-14)

(torque defined).

We can evaluate the vector (or cross) product in this definition of t: by using : the rules in Module 3-3. To find the direction of t:, we slide the vector F (without changing its direction) until its tail is at the origin O, so that the two vectors in the vector product are tail to tail as in Fig. 11-10b. We then use the right-hand rule in Fig. 3-19a, sweeping the fingers of the right hand from : r (the first vector in the : product) into F (the second vector). The outstretched right thumb then gives the direction of t:. In Fig. 11-10b, it is in the positive direction of the z axis. To determine the magnitude of t:, we apply the general result of Eq. 3-27 (c ! ab sin f), finding t ! rF sin f, (11-15) :

where f is the smaller angle between the directions of : r and F when the vectors are tail to tail. From Fig. 11-10b, we see that Eq. 11-15 can be rewritten as (11-16)

t ! rF", :

where F" (!F sin f) is the component of F perpendicular to :r . From Fig. 11-10c, we see that Eq. 11-15 can also be rewritten as (11-17)

t ! r"F, :

where r" (! r sin f) is the moment arm of F (the perpendicular distance : between O and the line of action of F ).

Checkpoint 3 The position vector : r of a particle points along the positive direction of a z axis. If the torque on the particle is (a) zero, (b) in the negative direction of x, and (c) in the negative direction of y, in what direction is the force causing the torque?

F

304

CHAPTE R 11 ROLLI NG, TORQU E, AN D ANG U L AR M OM E NTU M

Sample Problem 11.02 Torque on a particle due to a force In Fig. 11-11a, three forces, each of magnitude 2.0 N, act on a particle. The particle is in the xz plane at point A given by position vector : r , where r ! 3.0 m and u ! 30#. What is the torque, about the origin O, due to each force? KEY IDEA

t1 ! rF1 sin f1 ! (3.0 m)(2.0 N)(sin 150#) ! 3.0 N )m,

Because the three force vectors do not lie in a plane, we must use cross products, with magnitudes given by Eq. 11-15 (t ! rF sin f) and directions given by the right-hand rule. Calculations: Because we want the torques with respect to the origin O, the vector : r required for each cross product is the given position vector. To determine the angle f between : r and each force, we shift the force vectors of Fig. 1111a, each in turn, so that their tails are at the origin. Figures 11-11b, c, and d, which are direct views of the xz plane, show : : : the shifted force vectors F1, F2, and F3, respectively. (Note how much easier the angles between the force vectors and

A

z

θ = 30°

x

(a)

A

(b)

F1 r

θ

F2

O

t2 ! rF2 sin f2 ! (3.0 m)(2.0 N)(sin 120#) ! 5.2 N )m, and

t3 ! rF3 sin f3 ! (3.0 m)(2.0 N)(sin 90#) ! 6.0 N )m.

z

φ 1 = 150° O

F1

y

r

x

x

φ 2 = 120°

(c)

x

F1

F1

Cross r into F1. Torque τ 1 is into the figure (negative y). z

z

r

θ = 30°

z

r

z

x

(Answer)

Next, we use the right-hand rule, placing the fingers of : the right hand so as to rotate : r into F through the smaller of the two angles between their directions. The thumb points in the direction of the torque.Thus t:1 is directed into the page in Fig. 11-11b; t:2 is directed out of the page in Fig. 11-11c; and t:3 is directed as shown in Fig. 11-11d. All three torque vectors are shown in Fig. 11-11e.

z r

F3

the position vector are to see.) In Fig. 11-11d, the angle be: tween the directions of : r and F3 is 90# and the symbol # : means F3 is directed into the page. (For out of the page, we would use $.) Now, applying Eq. 11-15, we find

r

O

r

x

x

F2

Cross r into F2. Torque τ 2 is out of the figure (positive y).

F2

F2

z z

z

r x

r

F3

θ = 30° τ3

(d)

z

θ

O

τ1

r

x

τ3

Cross r into F3. Torque τ 3 is in the xz plane.

x

τ3

(e) x

τ2 θ

The three torques.

τ3

Figure 11-11 (a) A particle at point A is acted on by three forces, each parallel to a coordinate axis.The angle f (used in finding torque) is shown : : : : (b) for F1 and (c) for F2 . (d) Torque t:3 is perpendicular to both : r and F3 (force F3 is directed into the plane of the figure). (e) The torques.

Additional examples, video, and practice available at WileyPLUS

y

305

11-5 ANG U L AR M OM E NTU M

11-5 ANGULAR MOMENTUM Learning Objectives After reading this module, you should be able to . . .

11.17 Identify that angular momentum is a vector quantity. 11.18 Identify that the fixed point about which an angular momentum is calculated must always be specified. 11.19 Calculate the angular momentum of a particle by taking the cross product of the particle’s position vector and its

momentum vector, in either unit-vector notation or magnitude-angle notation. 11.20 Use the right-hand rule for cross products to find the direction of an angular momentum vector.

Key Ideas :

● The angular momentum !

of a particle with linear momentum p , mass m, and linear velocity v: is a vector quantity defined relative to a fixed point (usually an origin) as :

:

: ! ! :r $ p ! m(r: $ v:).

:

● The magnitude of !

is given by ! ! rmv sin f ! rp" ! rmv" ! r" p ! r" mv,

where f is the angle between :r and p:, p" and v" are the components of p: and v: perpendicular to :r , and r" is the perpendicular distance between the fixed point and the extension of p:. :

● The direction of !

is given by the right-hand rule: Position your right hand so that the fingers are in the direction of :r . Then rotate them around the palm to be in the direction of p:. : Your outstretched thumb gives the direction of ! .

Angular Momentum Recall that the concept of linear momentum p: and the principle of conservation of linear momentum are extremely powerful tools. They allow us to predict the outcome of, say, a collision of two cars without knowing the details of the collision. Here we begin a discussion of the angular counterpart of p: , winding up in Module 11-8 with the angular counterpart of the conservation principle, which can lead to beautiful (almost magical) feats in ballet, fancy diving, ice skating, and many other activities. Figure 11-12 shows a particle of mass m with linear momentum p: (! mv:) as : it passes through point A in an xy plane. The angular momentum ! of this particle with respect to the origin O is a vector quantity defined as :

! ! r $ p ! m(r $ v) :

:

:

:

(angular momentum defined),

z ! (=

where f is the smaller angle between :r and p: when these two vectors are tail

p (redrawn, with tail at origin)

O

φ

A x

y

φ

p

r

φ p

(a)

(11-18)

where : r is the position vector of the particle with respect to O. As the particle : moves relative to O in the direction of its momentum p (! mv:) , position vector : r rotates around O. Note carefully that to have angular momentum about O, the particle does not itself have to rotate around O. Comparison of Eqs. 11-14 and 11-18 shows that angular momentum bears the same relation to linear momentum that torque does to force. The SI unit of angular momentum is the kilogrammeter-squared per second (kg )m2/s), equivalent to the joule-second (J )s). : Direction. To find the direction of the angular momentum vector ! in Fig. 11: 12, we slide the vector p until its tail is at the origin O. Then we use the right-hand rule for vector products, sweeping the fingers from : r into p: . The outstretched : thumb then shows that the direction of ! is in the positive direction of the z axis in Fig. 11-12.This positive direction is consistent with the counterclockwise rotation of : position vector : r about the z axis, as the particle moves. (A negative direction of ! : would be consistent with a clockwise rotation of r about the z axis.) : Magnitude. To find the magnitude of ! , we use the general result of Eq. 3-27 to write (11-19) ! ! rmv sin f,

r × p)

z

!

y

O r

r

φ

Extension of p x

A

p

(b)

Figure 11-12 Defining angular momentum.A particle passing through point A has linear momentum p: (! mv:), with the vector p: lying in an xy plane.The particle has angular : : momentum ! (! :r $ p ) with respect to the origin O. By the right-hand rule, the angular momentum vector points in the positive : direction of z. (a) The magnitude of ! is given by:! ! rp" ! rmv". (b) The magnitude of ! is also given by ! ! r" p ! r"mv.

306

CHAPTE R 11 ROLLI NG, TORQU E, AN D ANG U L AR M OM E NTU M

to tail. From Fig. 11-12a, we see that Eq. 11-19 can be rewritten as ! ! rp" ! rmv",

(11-20)

: where p" is the component of p perpendicular to : r and v" is the component : : of v perpendicular to r . From Fig. 11-12b, we see that Eq. 11-19 can also be rewritten as (11-21) ! ! r" p ! r"mv, : where r" is the perpendicular distance between O and the extension of p . Important. Note two features here: (1) angular momentum has meaning only with respect to a specified origin and (2) its direction is always perpendicu: lar to the plane formed by the position and linear momentum vectors : . r and p

Checkpoint 4 In part a of the figure, particles 1 and 2 move around point O in circles with radii 2 m and 4 m. In part b, particles 3 and 4 travel along straight lines at perpendicular distances of 4 m and 2 m from point O. Particle 5 moves directly away from O. All five particles have the same mass and the same constant speed. (a) Rank the particles according to the magnitudes of their angular momentum about point O, greatest first. (b) Which particles have negative angular momentum about point O?

1

3

O

O

5 4

2 (a)

(b)

Sample Problem 11.03 Angular momentum of a two-particle system Figure 11-13 shows an overhead view of two particles moving at constant momentum along horizontal paths. Particle 1, with momentum magnitude p1 ! 5.0 kg )m/s, has position vector : r1 and will pass 2.0 m from point O. Particle 2, with momentum magnitude p2 ! 2.0 kg )m/s, has position vector : r 2 and will pass 4.0 m from point O. What are the magnitude and direction of : the net angular momentum L about point O of the twoparticle system?

p1

p2

Figure 11-13 Two particles pass near point O.

r⊥2 r2

r⊥1

r1

O

around O as particle 1 moves. Thus, the angular momentum vector for particle 1 is : r1

KEY IDEA

!1 ! &10 kg)m2/s.

:

To find L, we can first find the individual angular momenta : : ! 1 and ! 2 and then add them. To evaluate their magnitudes, we can use any one of Eqs. 11-18 through 11-21. However, Eq. 11-21 is easiest, because we are given the perpendicular distances r"1 (! 2.0 m) and r"2 (! 4.0 m) and the momentum magnitudes p1 and p2. Calculations: For particle 1, Eq. 11-21 yields !1 ! r"1 p1 ! (2.0 m)(5.0 kg )m/s) ! 10 kg)m2/s. :

To find the direction of vector ! 1, we use Eq. 11-18 and the : right-hand rule for vector products. For : r1 $ p 1, the vector product is out of the page, perpendicular to the plane of Fig. 11-13. This is the positive direction, consistent with the counterclockwise rotation of the particle’s position vector

:

Similarly, the magnitude of ! 2 is !2 ! r"2 p2 ! (4.0 m)(2.0 kg )m/s) ! 8.0 kg)m2/s, : and the vector product : r2 $ p 2 is into the page, which is the negative direction, consistent with the clockwise rotation of : r 2 around O as particle 2 moves. Thus, the angular momentum vector for particle 2 is

!2 ! %8.0 kg)m2/s. The net angular momentum for the two-particle system is L ! !1 & !2 ! &10 kg)m2/s & (%8.0 kg)m2/s) ! &2.0 kg)m2/s.

(Answer)

The plus sign means that the system’s net angular momentum about point O is out of the page.

Additional examples, video, and practice available at WileyPLUS

11-6 N EWTON’S SECON D L AW I N ANG U L AR FOR M

307

11-6 NEWTON’S SECOND LAW IN ANGULAR FORM Learning Objective After reading this module, you should be able to . . .

11.21 Apply Newton’s second law in angular form to relate the torque acting on a particle to the resulting rate of change of the particle’s angular momentum, all relative to a specified point.

Key Idea ● Newton’s second law for a particle can be written in angular form as :

t:net !

d! , dt

:

where t:net is the net torque acting on the particle and ! is the angular momentum of the particle.

Newton’s Second Law in Angular Form Newton’s second law written in the form :

F net !

dp: dt

(11-22)

(single particle)

expresses the close relation between force and linear momentum for a single particle. We have seen enough of the parallelism between linear and angular quantities to be pretty sure that there is also a close relation between torque and angular momentum. Guided by Eq. 11-22, we can even guess that it must be :

t:net !

d! dt

(11-23)

(single particle).

Equation 11-23 is indeed an angular form of Newton’s second law for a single particle: The (vector) sum of all the torques acting on a particle is equal to the time rate of change of the angular momentum of that particle.

Equation 11-23 has no meaning unless the torques t: and the angular momentum : ! are defined with respect to the same point, usually the origin of the coordinate system being used.

Proof of Equation 11-23 We start with Eq. 11-18, the definition of the angular momentum of a particle: :

! ! m(r: $ : v), where :r is the position vector of the particle and v: is the velocity of the particle. Differentiating* each side with respect to time t yields :

#

$

dr: d! dv: !m : r$ & $ v: . dt dt dt

(11-24)

However, dv:/dt is the acceleration : a of the particle, and dr:/dt is its velocity v: . Thus, we can rewrite Eq. 11-24 as :

d! ! m(r: $ : a & v: $ v:). dt *In differentiating a vector product, be sure not to change the order of the two quantities (here : r and : v ) that form that product. (See Eq. 3-25.)

308

CHAPTE R 11 ROLLI NG, TORQU E, AN D ANG U L AR M OM E NTU M

Now v: $ v: ! 0 (the vector product of any vector with itself is zero because the angle between the two vectors is necessarily zero). Thus, the last term of this expression is eliminated and we then have :

d! : ! m(r: $ : a) ! :r $ ma . dt :

: : We now use Newton’s second law (Fnet ! ma with its equal, the ) to replace ma vector sum of the forces that act on the particle, obtaining

:

d! : : ! :r $ Fnet ! '(r: $ F ). dt

(11-25) :

Here the symbol ' indicates that we must sum the vector products :r $ F for all the forces. However, from Eq. 11-14, we know that each one of those vector products is the torque associated with one of the forces.Therefore, Eq. 11-25 tells us that :

d! . dt This is Eq. 11-23, the relation that we set out to prove. t:net !

Checkpoint 5 F3 The figure shows the position vector : r of a particle F2 at a certain instant, and four choices for the direction of a force that is to accelerate the particle. All r four choices lie in the xy plane. (a) Rank the F4 F1 choices according to the magnitude of the time rate : of change (d! /dt) they produce in the angular momentum of the particle about point O, greatest first. (b) Which choice results in a negative rate of change about O?

y

O

x

Sample Problem 11.04 Torque and the time derivative of angular momentum Figure 11-14a shows a freeze-frame of a 0.500 kg particle moving along a straight line with a position vector given by : r ! (%2.00t 2 % t)iˆ & 5.00jˆ,

on a particle and the angular momentum of the particle are calculated around the same point, then the torque is re: lated to angular momentum by Eq. 11-23 (t: ! d!/dt).

with :r in meters and t in seconds, starting at t ! 0. The position vector points from the origin to the particle. In unit-vector : notation, find expressions for the angular momentum ! of the particle and the torque t: acting on the particle, both with respect to (or about) the origin. Justify their algebraic signs in terms of the particle’s motion.

Calculations: In order to use Eq. 11-18 to find the angular momentum about the origin, we first must find an expression for the particle’s velocity by taking a time derivative of its position vector. Following Eq. 4-10 ( v: ! dr:/dt), we write

KEY IDEAS (1) The point about which an angular momentum of a particle is to be calculated must always be specified. Here it is : the origin. (2) The angular momentum ! of a particle is : : : : given by Eq. 11-18 (! ! r $ p ! m(r $ : v )). (3) The sign associated with a particle’s angular momentum is set by the sense of rotation of the particle’s position vector (around the rotation axis) as the particle moves: clockwise is negative and counterclockwise is positive. (4) If the torque acting

d ((%2.00t 2 % t)iˆ & 5.00jˆ ) dt ! (%4.00t % 1.00)iˆ,

v: !

with v: in meters per second. Next, let’s take the cross product of :r and v: using the template for cross products displayed in Eq. 3-27: : ˆ. a: $ b ! (aybz % byaz)iˆ & (azbx % bzax)jˆ & (axby % bxay)k :

Here the generic a: is :r and the generic b is v:. However, because we really don’t want to do more work than needed, let’s first just think about our substitutions into

309

11-6 N EWTON’S SECON D L AW I N ANG U L AR FOR M

y (m) v

y (m) t !2s

5

t !1s

t !0

r1

r2

r0

x (m)

x (m) "10

"3

(a)

(b) y

y

y

Both angular momentum and torque point out of figure, in the positive z direction.

v r x (c)

the generic cross product. Because :r lacks any z component and because v: lacks any y or z component, the only nonzero ˆ. term in the generic cross product is the very last one (%bxay)k So, let’s cut to the (mathematical) chase by writing : ˆ ! (20.0t # 5.00)k ˆ m2/s. r $ v ! %(%4.00t % 1.00)(5.00)k

:

Note that, as always, the cross product produces a vector that is perpendicular to the original vectors. To finish up Eq. 11-18, we multiply by the mass, finding : ! ! (0.500 kg)[(20.0t # 5.00)kˆ m2/s]

(Answer)

The torque about the origin then immediately follows from Eq. 11-23: :

t

(d)

Figure 11-14 (a) A particle moving in a straight line, shown at time t ! 0. (b) The position vector at t ! 0, 1.00 s, and 2.00 s. (c) The first step in applying the right-hand rule for cross products. (d) The second step. (e) The angular momentum vector and the torque vector are along the z axis, which extends out of the plane of the figure.

ˆ kg " m2/s. ! (10.0t # 2.50)k

!

x

d ˆ kg"m2/s (10.0t # 2.50)k dt ˆ kg "m2/s2 ! 10.0k ˆ N"m, (Answer) ! 10.0k

t!

which is in the positive direction of the z axis. : Our result for ! tells us that the angular momentum is in the positive direction of the z axis. To make sense of that positive result in terms of the rotation of the position vector,

x

(e)

let’s evaluate that vector for several times: t ! 0,

:

t ! 1.00 s,

:

t ! 2.00 s,

:

r0 !

5.00jˆ m

r1 ! %3.00iˆ # 5.00jˆ m r2 ! %10.0iˆ # 5.00jˆ m

By drawing these results as in Fig. 11-14b, we see that :r rotates counterclockwise in order to keep up with the particle. That is the positive direction of rotation. Thus, even though the particle is moving in a straight line, it is still moving counterclockwise around the origin and thus has a positive angular momentum. : We can also make sense of the direction of ! by applying : the right-hand rule for cross products (here r $ : v, or, if you like, mr: $ : v, which gives the same direction). For any moment during the particle’s motion, the fingers of the right hand are first extended in the direction of the first vector in the cross product ( :r ) as indicated in Fig. 11-14c. The orientation of the hand (on the page or viewing screen) is then adjusted so that the fingers can be comfortably rotated about the palm to be in the direction of the second vector in the cross product (v:) as indicated in Fig. 11-14d. The outstretched thumb then points in the direction of the result of the cross product. As indicated in Fig. 11-14e, the vector is in the positive direction of the z axis (which is directly out of the plane of the figure), consistent with our previous result. Figure 11-14e also indicates the direction of : t , which is also in the positive direction of the z axis because the angular momentum is in that direction and is increasing in magnitude.

Additional examples, video, and practice available at WileyPLUS

310

CHAPTE R 11 ROLLI NG, TORQU E, AN D ANG U L AR M OM E NTU M

11-7 ANGULAR MOMENTUM OF A RIGID BODY Learning Objectives After reading this module, you should be able to . . .

11.22 For a system of particles, apply Newton’s second law in angular form to relate the net torque acting on the system to the rate of the resulting change in the system’s angular momentum.

11.23 Apply the relationship between the angular momentum of a rigid body rotating around a fixed axis and the body’s rotational inertia and angular speed around that axis. 11.24 If two rigid bodies rotate about the same axis, calculate their total angular momentum.

Key Ideas :

● The angular momentum L

of a system of particles is the vector sum of the angular momenta of the individual particles: :

:

:

:

L ! !1 # !2 # " " " # !n !

n

the torques due to interactions of the particles of the system with particles external to the system): :

dL t net ! (system of particles). dt ● For a rigid body rotating about a fixed axis, the component of its angular momentum parallel to the rotation axis is L ! Iv (rigid body, fixed axis). :

' ! i. i!1 :

● The time rate of change of this angular momentum is equal to the net external torque on the system (the vector sum of

The Angular Momentum of a System of Particles Now we turn our attention to the angular momentum of a system of particles with : respect to an origin. The total angular momentum L of the system is the (vector) : sum of the angular momenta ! of the individual particles (here with label i): :

:

:

:

:

L ! !1 # !2 # !3 # " " " # !n !

n

' ! i. i!1 :

(11-26)

With time, the angular momenta of individual particles may change because of interactions between the particles or with the outside. We can find the resulting : change in L by taking the time derivative of Eq. 11-26. Thus, :

:

n d!i dL ! ' . dt i!1 dt

(11-27)

:

From Eq. 11-23, we see that d! i /dt is equal to the net torque :& net,i on the ith particle. We can rewrite Eq. 11-27 as :

n dL ! ' &:net,i. dt i!1

(11-28) :

That is, the rate of change of the system’s angular momentum L is equal to the vector sum of the torques on its individual particles. Those torques include internal torques (due to forces between the particles) and external torques (due to forces on the particles from bodies external to the system). However, the forces between the particles always come in third-law force pairs so their torques sum to : zero. Thus, the only torques that can change the total angular momentum L of the system are the external torques acting on the system. Net External Torque. Let &:net represent the net external torque, the vector sum of all external torques on all particles in the system. Then we can write Eq. 11-28 as :

&:net !

dL dt

(system of particles),

(11-29)

311

11-7 ANG U L AR M OM E NTU M OF A R IG I D BODY

which is Newton’s second law in angular form. It says:

z

The net external torque &:net acting on a system of particles is equal to the time : rate of change of the system’s total angular momentum L . :

:

Equation 11-29 is analogous to Fnet ! dP/dt (Eq. 9-27) but requires extra caution: Torques and the system’s angular momentum must be measured relative to the same origin. If the center of mass of the system is not accelerating relative to an inertial frame, that origin can be any point. However, if it is accelerating, then it must be the origin. For example, consider a wheel as the system of particles. If it is rotating about an axis that is fixed relative to the ground, then the origin for applying Eq. 11-29 can be any point that is stationary relative to the ground. However, if it is rotating about an axis that is accelerating (such as when it rolls down a ramp), then the origin can be only at its center of mass.

r

i

∆ mi

pi ri

θ y

O x (a)

The Angular Momentum of a Rigid Body Rotating About a Fixed Axis

z

θ

We next evaluate the angular momentum of a system of particles that form a rigid body that rotates about a fixed axis. Figure 11-15a shows such a body. The fixed axis of rotation is a z axis, and the body rotates about it with constant angular speed v. We wish to find the angular momentum of the body about that axis. We can find the angular momentum by summing the z components of the angular momenta of the mass elements in the body. In Fig. 11-15a, a typical mass element, of mass 'mi, moves around the z axis in a circular path. The position of the mass element is located relative to the origin O by position vector :r i. The radius of the mass element’s circular path is r"i, the perpendicular distance between the element and the z axis. : The magnitude of the angular momentum ! i of this mass element, with respect to O, is given by Eq. 11-19: !i ! (r i)( pi)(sin 90() ! (r i)('mi vi), where pi and vi are the linear momentum and linear speed of the mass element, : and 90( is the angle between :ri and p:i . The angular momentum vector ! i for the mass element in Fig. 11-15a is shown in Fig. 11-15b; its direction must be perpendicular to those of :ri and p:i . : The z Components. We are interested in the component of ! i that is parallel to the rotation axis, here the z axis. That z component is !iz ! !i sin u ! (r i sin u)('mi vi) ! r"i 'mi vi. The z component of the angular momentum for the rotating rigid body as a whole is found by adding up the contributions of all the mass elements that make up the body. Thus, because v ! vr", we may write Lz !

n

n

n

' !iz ! i!1 ' 'mi vir"i ! i!1 ' 'mi(vr"i)r"i i!1

# ' 'm r $. n

!v

i!1

2 i "i

(11-30)

We can remove v from the summation here because it has the same value for all points of the rotating rigid body. 2 The quantity ' 'mi r"i in Eq. 11-30 is the rotational inertia I of the body about the fixed axis (see Eq. 10-33). Thus Eq. 11-30 reduces to L ! Iv

(rigid body, fixed axis).

(11-31)

!i !iz

x

θ y

O (b)

Figure 11-15 (a) A rigid body rotates about a z axis with angular speed v. A mass element of mass 'mi within the body moves about the z axis in a circle with radius r"i . The mass element has linear momentum : p i, and it is located relative to the origin O by position vector :ri . Here the mass element is shown when r"i is parallel to the : x axis. (b) The angular momentum ! i, with respect to O, of the mass element in (a).The z component !iz is also shown.

312

CHAPTE R 11 ROLLI NG, TORQU E, AN D ANG U L AR M OM E NTU M

Table 11-1 More Corresponding Variables and Relations for Translational and Rotational Motiona

Translational Force Linear momentum Linear momentumb Linear momentumb Newton’s second lawb Conservation lawd

:

F p : P (! ' : p i) : P ! Mv:com : dP : Fnet ! dt : P ! a constant :

Rotational Torque Angular momentum Angular momentumb Angular momentumc Newton’s second lawb Conservation lawd

:

& (! : r $ F) ! (! : r $: p) : : L (! ' ! i) L ! Iv : dL &:net ! dt : L ! a constant :

:

a

See also Table 10-3. For systems of particles, including rigid bodies. c For a rigid body about a fixed axis, with L being the component along that axis. d For a closed, isolated system. b

We have dropped the subscript z, but you must remember that the angular momentum defined by Eq. 11-31 is the angular momentum about the rotation axis. Also, I in that equation is the rotational inertia about that same axis. Table 11-1, which supplements Table 10-3, extends our list of corresponding linear and angular relations.

Checkpoint 6 In the figure, a disk, a hoop, and a solid sphere Disk Hoop Sphere are made to spin about fixed central axes (like a F F F top) by means of strings wrapped around them, with the strings producing the same constant tangential force : F on all three objects. The three objects have the same mass and radius, and they are initially stationary. Rank the objects according to (a) their angular momentum about their central axes and (b) their angular speed, greatest first, when the strings have been pulled for a certain time t.

11-8 CONSERVATION OF ANGULAR MOMENTUM Learning Objective After reading this module, you should be able to . . .

11.25 When no external net torque acts on a system along a specified axis, apply the conservation of angular momentum to relate the initial angular momentum value along that axis to the value at a later instant.

Key Idea ● The angular momentum

:

L of a system remains constant if the net external torque acting on the system is zero: :

L ! a constant or

: Li

!

: Lf

(isolated system)

(isolated system).

This is the law of conservation of angular momentum.

Conservation of Angular Momentum So far we have discussed two powerful conservation laws, the conservation of energy and the conservation of linear momentum. Now we meet a third law of this type, involving the conservation of angular momentum. We start from

11-8 CONSE RVATION OF ANG U L AR M OM E NTU M

313

:

Eq. 11-29 (t:net ! dL /dt), which is Newton’s second law in angular form. If no : net external torque acts on the system, this equation becomes dL/dt ! 0, or :

L ! a constant

(11-32)

(isolated system).

This result, called the law of conservation of angular momentum, can also be written as angular momentum net angular momentum , ! #netat some initial time t $ # at some later time t $ i

or

:

:

Li ! Lf

f

(isolated system).

(11-33)

Equations 11-32 and 11-33 tell us: :

If the net external torque acting on a system is zero, the angular momentum L of the system remains constant, no matter what changes take place within the system.

Equations 11-32 and 11-33 are vector equations; as such, they are equivalent to three component equations corresponding to the conservation of angular momentum in three mutually perpendicular directions. Depending on the torques acting on a system, the angular momentum of the system might be conserved in only one or two directions but not in all directions:

L

ωi

If the component of the net external torque on a system along a certain axis is zero, then the component of the angular momentum of the system along that axis cannot change, no matter what changes take place within the system.

Ii

This is a powerful statement: In this situation we are concerned with only the initial and final states of the system; we do not need to consider any intermediate state. We can apply this law to the isolated body in Fig. 11-15, which rotates around the z axis. Suppose that the initially rigid body somehow redistributes its mass relative to that rotation axis, changing its rotational inertia about that axis. Equations 11-32 and 11-33 state that the angular momentum of the body cannot change. Substituting Eq. 11-31 (for the angular momentum along the rotational axis) into Eq. 11-33, we write this conservation law as Iivi ! If vf.

Rotation axis (a)

L

(11-34)

Here the subscripts refer to the values of the rotational inertia I and angular speed v before and after the redistribution of mass. Like the other two conservation laws that we have discussed, Eqs. 11-32 and 11-33 hold beyond the limitations of Newtonian mechanics. They hold for particles whose speeds approach that of light (where the theory of special relativity reigns), and they remain true in the world of subatomic particles (where quantum physics reigns). No exceptions to the law of conservation of angular momentum have ever been found. We now discuss four examples involving this law. 1. The spinning volunteer Figure 11-16 shows a student seated on a stool that can rotate freely about a vertical axis. The student, who has been set into rotation at a modest initial angular speed vi, holds two dumbbells in his : outstretched hands. His angular momentum vector L lies along the vertical rotation axis, pointing upward. The instructor now asks the student to pull in his arms; this action reduces his rotational inertia from its initial value Ii to a smaller value If because he moves mass closer to the rotation axis. His rate of rotation increases markedly,

ωf

If

(b)

Figure 11-16 (a) The student has a relatively large rotational inertia about the rotation axis and a relatively small angular speed. (b) By decreasing his rotational inertia, the student automatically increases his angular : speed. The angular momentum L of the rotating system remains unchanged.

314

CHAPTE R 11 ROLLI NG, TORQU E, AN D ANG U L AR M OM E NTU M

L

Her angular momentum is fixed but she can still control her spin rate. L

Figure 11-17 The diver’s angular momentum : L is constant throughout the dive, being represented by the tail # of an arrow that is perpendicular to the plane of the figure. Note also that her center of mass (see the dots) follows a parabolic path.

from vi to vf.The student can then slow down by extending his arms once more, moving the dumbbells outward. No net external torque acts on the system consisting of the student, stool, and dumbbells. Thus, the angular momentum of that system about the rotation axis must remain constant, no matter how the student maneuvers the dumbbells. In Fig. 11-16a, the student’s angular speed vi is relatively low and his rotational inertia Ii is relatively high. According to Eq. 11-34, his angular speed in Fig. 11-16b must be greater to compensate for the decreased If. 2. The springboard diver Figure 11-17 shows a diver doing a forward one-and-ahalf-somersault dive. As you should expect, her center of mass follows a para: bolic path. She leaves the springboard with a definite angular momentum L about an axis through her center of mass, represented by a vector pointing into the plane of Fig. 11-17, perpendicular to the page. When she is in the air, no net external torque acts on her about her center of mass, so her angular momentum about her center of mass cannot change. By pulling her arms and legs into the closed tuck position, she can considerably reduce her rotational inertia about the same axis and thus, according to Eq. 11-34, considerably increase her angular speed. Pulling out of the tuck position (into the open layout position) at the end of the dive increases her rotational inertia and thus slows her rotation rate so she can enter the water with little splash. Even in a more complicated dive involving both twisting and somersaulting, the angular momentum of the diver must be conserved, in both magnitude and direction, throughout the dive. 3. Long jump When an athlete takes off from the ground in a running long jump, the forces on the launching foot give the athlete an angular momentum with a forward rotation around a horizontal axis. Such rotation would not allow the jumper to land properly: In the landing, the legs should be together and extended forward at an angle so that the heels mark the sand at the greatest distance. Once airborne, the angular momentum cannot change (it is conserved) because no external torque acts to change it. However, the jumper can shift most of the angular momentum to the arms by rotating them in windmill fashion (Fig. 11-18). Then the body remains upright and in the proper orientation for landing.

Figure 11-18 Windmill motion of the arms during a long jump helps maintain body orientation for a proper landing.

4. Tour jeté In a tour jeté, a ballet performer leaps with a small twisting motion on the floor with one foot while holding the other leg perpendicular to the body (Fig. 11-19a). The angular speed is so small that it may not be perceptible

Figure 11-19 (a) Initial phase of a tour jeté: large rotational inertia and small angular speed. (b) Later phase: smaller rotational inertia and larger angular speed.

θ

(a)

(b)

11-8 CONSE RVATION OF ANG U L AR M OM E NTU M

315

to the audience. As the performer ascends, the outstretched leg is brought down and the other leg is brought up, with both ending up at angle u to the body (Fig. 11-19b). The motion is graceful, but it also serves to increase the rotation because bringing in the initially outstretched leg decreases the performer’s rotational inertia. Since no external torque acts on the airborne performer, the angular momentum cannot change. Thus, with a decrease in rotational inertia, the angular speed must increase. When the jump is well executed, the performer seems to suddenly begin to spin and rotates 180( before the initial leg orientations are reversed in preparation for the landing. Once a leg is again outstretched, the rotation seems to vanish.

Checkpoint 7 A rhinoceros beetle rides the rim of a small disk that rotates like a merry-go-round. If the beetle crawls toward the center of the disk, do the following (each relative to the central axis) increase, decrease, or remain the same for the beetle–disk system: (a) rotational inertia, (b) angular momentum, and (c) angular speed?

Sample Problem 11.05 Conservation of angular momentum, rotating wheel demo Figure 11-20a shows a student, again sitting on a stool that can rotate freely about a vertical axis. The student, initially at rest, is holding a bicycle wheel whose rim is loaded with lead and whose rotational inertia Iwh about its central axis is 1.2 kg "m2. (The rim contains lead in order to make the value of Iwh substantial.) The wheel is rotating at an angular speed vwh of 3.9 rev/s; as seen from overhead, the rotation is counterclockwise. The axis of the wheel is vertical, and the angular : momentum Lwh of the wheel points vertically upward. The student now inverts the wheel (Fig. 11-20b) so that, as seen from overhead, it is rotating clockwise. Its angular : momentum is now %Lwh. The inversion results in the student, the stool, and the wheel’s center rotating together as a composite rigid body about the stool’s rotation axis, with rotational inertia Ib ! 6.8 kg "m2. (The fact that the wheel is also rotating about its center does not affect the mass distribution of this composite body; thus, Ib has the same value whether or not the wheel rotates.) With what angular speed vb and in what direction does the composite body rotate after the inversion of the wheel? KEY IDEAS 1. The angular speed vb we seek is related to the final angu: lar momentum Lb of the composite body about the stool’s rotation axis by Eq. 11-31 (L ! Iv). 2. The initial angular speed vwh of the wheel is related to the : angular momentum Lwh of the wheel’s rotation about its center by the same equation. : : 3. The vector addition of Lb and Lwh gives the total angular : momentum Ltot of the system of the student, stool, and wheel. 4. As the wheel is inverted, no net external torque acts on

Lb –Lwh

Lwh

ω wh

ω wh

ωb

(a)

(b)

=

+

Lwh

Lb

Initial

Final

–Lwh

The student now has angular momentum, and the net of these two vectors equals the initial vector.

(c)

Figure 11-20 (a) A student holds a bicycle wheel rotating around a vertical axis. (b) The student inverts the wheel, setting himself into rotation. (c) The net angular momentum of the system must remain the same in spite of the inversion.

:

that system to change Ltot about any vertical axis. (Torques due to forces between the student and the wheel as the student inverts the wheel are internal to the system.) So, the system’s total angular momentum is conserved about any vertical axis, including the rotation axis through the stool.

316

CHAPTE R 11 ROLLI NG, TORQU E, AN D ANG U L AR M OM E NTU M

:

Calculations: The conservation of Ltot is represented with vectors in Fig. 11-20c. We can also write this conservation in terms of components along a vertical axis as Lb,f # Lwh,f ! Lb,i # Lwh,i,

Using Eq. 11-31, we next substitute Ibvb for Lb,f and Iwhvwh for Lwh,i and solve for vb, finding

(11-35)

where i and f refer to the initial state (before inversion of the wheel) and the final state (after inversion). Because inversion of the wheel inverted the angular momentum vector of the wheel’s rotation, we substitute %Lwh,i for Lwh,f. Then, if we set Lb,i ! 0 (because the student, the stool, and the wheel’s center were initially at rest), Eq. 11-35 yields Lb,f ! 2Lwh,i.

)b !

!

2Iwh vwh Ib (2)(1.2 kg "m2)(3.9 rev/s) ! 1.4 rev/s. 6.8 kg"m2

(Answer)

This positive result tells us that the student rotates counterclockwise about the stool axis as seen from overhead. If the student wishes to stop rotating, he has only to invert the wheel once more.

Sample Problem 11.06 Conservation of angular momentum, cockroach on disk In Fig. 11-21, a cockroach with mass m rides on a disk of mass 6.00m and radius R. The disk rotates like a merry-go-round around its central axis at angular speed vi ! 1.50 rad/s. The cockroach is initially at radius r ! 0.800R, but then it crawls out to the rim of the disk. Treat the cockroach as a particle. What then is the angular speed? KEY IDEAS (1) The cockroach’s crawl changes the mass distribution (and thus the rotational inertia) of the cockroach–disk system. (2) The angular momentum of the system does not change because there is no external torque to change it. (The forces and torques due to the cockroach’s crawl are internal to the system.) (3) The magnitude of the angular momentum of a rigid body or a particle is given by Eq. 11-31 (L ! Iv). Calculations: We want to find the final angular speed. Our key is to equate the final angular momentum Lf to the initial angular momentum Li, because both involve angular speed. They also involve rotational inertia I. So, let’s start by finding the rotational inertia of the system of cockroach and disk before and after the crawl. ωi

The rotational inertia of a disk rotating about its central axis is given by Table 10-2c as 12MR 2. Substituting 6.00m for the mass M, our disk here has rotational inertia (11-36) Id ! 3.00mR2. (We don’t have values for m and R, but we shall continue with physics courage.) From Eq. 10-33, we know that the rotational inertia of the cockroach (a particle) is equal to mr 2. Substituting the cockroach’s initial radius (r ! 0.800R) and final radius (r ! R), we find that its initial rotational inertia about the rotation axis is Ici ! 0.64mR2

(11-37)

and its final rotational inertia about the rotation axis is Icf ! mR2.

(11-38)

So, the cockroach–disk system initially has the rotational inertia (11-39) Ii ! Id # Ici ! 3.64mR2, and finally has the rotational inertia If ! Id # Icf ! 4.00mR2.

(11-40)

Next, we use Eq. 11-31 (L ! Iv) to write the fact that the system’s final angular momentum Lf is equal to the system’s initial angular momentum Li: If vf ! Iivi

R

r Rotation axis

or

2

4.00mR vf ! 3.64mR2(1.50 rad/s).

After canceling the unknowns m and R, we come to vf ! 1.37 rad/s.

Figure 11-21 A cockroach rides at radius r on a disk rotating like a merry-go-round.

(Answer)

Note that v decreased because part of the mass moved outward, thus increasing that system’s rotational inertia.

Additional examples, video, and practice available at WileyPLUS

317

11-9 PR ECESSION OF A GYROSCOPE

11-9 PRECESSION OF A GYROSCOPE Learning Objectives After reading this module, you should be able to . . .

11.26 Identify that the gravitational force acting on a spinning gyroscope causes the spin angular momentum vector (and thus the gyroscope) to rotate about the vertical axis in a motion called precession.

11.27 Calculate the precession rate of a gyroscope. 11.28 Identify that a gyroscope’s precession rate is independent of the gyroscope’s mass.

Key Idea ● A spinning gyroscope can precess about a vertical axis through its support at the rate

Mgr , Iv where M is the gyroscope’s mass, r is the moment arm, I is the rotational inertia, and v is the spin rate. *!

Precession of a Gyroscope

z

A simple gyroscope consists of a wheel fixed to a shaft and free to spin about the axis of the shaft. If one end of the shaft of a nonspinning gyroscope is placed on a support as in Fig. 11-22a and the gyroscope is released, the gyroscope falls by rotating downward about the tip of the support. Since the fall involves rotation, it is governed by Newton’s second law in angular form, which is given by Eq. 11-29: :

dL (11-41) t: ! . dt This equation tells us that the torque causing the downward rotation (the fall) : changes the angular momentum L of the gyroscope from its initial value of zero. The torque &: is due to the gravitational force Mg: acting at the gyroscope’s center of mass, which we take to be at the center of the wheel.The moment arm relative to the support tip, located at O in Fig. 11-22a, is :r .The magnitude of t: is t ! Mgr sin 90( ! Mgr

(11-43)

where I is the rotational moment of the gyroscope about its shaft and v is the an: gular speed at which the wheel spins about the shaft. The vector L points along : : the shaft, as in Fig. 11-22b. Since L is parallel to r , torque t: must be : perpendicular to L .

y

τ x Support

Mg

(a) z

ω

O

r

L

(11-42)

(because the angle between Mg: and :r is 90(), and its direction is as shown in Fig. 11-22a. A rapidly spinning gyroscope behaves differently. Assume it is released with the shaft angled slightly upward. It first rotates slightly downward but then, while it is still spinning about its shaft, it begins to rotate horizontally about a vertical axis through support point O in a motion called precession. Why Not Just Fall Over? Why does the spinning gyroscope stay aloft instead of falling over like the nonspinning gyroscope? The clue is that when the spinning gyroscope is released, the torque due to Mg: must change not an initial angular momentum of zero but rather some already existing nonzero angular momentum due to the spin. To see how this nonzero initial angular momentum leads to precession, we first : consider the angular momentum L of the gyroscope due to its spin. To simplify the situation, we assume the spin rate is so rapid that the angular momentum due to pre: cession is negligible relative to L. We also assume the shaft is horizontal when pre: cession begins, as in Fig. 11-22b.The magnitude of L is given by Eq. 11-31: L ! Iv,

O

r

y dL τ = ___ dt

x Mg

(b)

Circular path taken by head of L vector

L(t) x

z O dφ

L(t + dt)

y

dL ___ dt (c)

Figure 11-22 (a) A nonspinning gyroscope falls by rotating in an xz plane because of torque t:. (b) A rapidly spinning gyroscope, : with angular momentum L, precesses around the z axis. Its precessional motion is : in the xy plane. (c) The change dL/dt in : angular momentum leads to a rotation of L about O.

318

CHAPTE R 11 ROLLI NG, TORQU E, AN D ANG U L AR M OM E NTU M :

According to Eq. 11-41, torque t: causes an incremental change dL in the angular momentum of the gyroscope in an incremental time interval dt; that is, :

(11-44)

dL ! t: dt. :

However, for a rapidly spinning gyroscope, the magnitude of L is fixed by : Eq. 11-43. Thus the torque can change only the direction of L, not its magnitude. : From Eq. 11-44 we see that the direction of dL is in the direction of t:, per: : pendicular to L . The only way that L can be changed in the direction of t: : without the magnitude L being changed is for L to rotate around the z axis as : : shown in Fig. 11-22c. L maintains its magnitude, the head of the L vector follows : : a circular path, and t is always tangent to that path. Since L must always point along the shaft, the shaft must rotate about the z axis in the direction of t:. Thus we have precession. Because the spinning gyroscope must obey Newton’s law in angular form in response to any change in its initial angular momentum, it must precess instead of merely toppling over. Precession. We can find the precession rate * by first using Eqs. 11-44 and : 11-42 to get the magnitude of dL : (11-45)

dL ! t dt ! Mgr dt. :

As L changes by an incremental amount in an incremental time interval dt, the shaft : and L precess around the z axis through incremental angle df. (In Fig. 11-22c, angle df is exaggerated for clarity.) With the aid of Eqs. 11-43 and 11-45, we find that df is given by Mgr dt dL ! . df ! L Iv Dividing this expression by dt and setting the rate * ! df/dt, we obtain *!

Mgr Iv

(11-46)

(precession rate).

This result is valid under the assumption that the spin rate v is rapid. Note that * decreases as v is increased. Note also that there would be no precession if the gravitational force Mg: did not act on the gyroscope, but because I is a function of M, mass cancels from Eq. 11-46; thus * is independent of the mass. Equation 11-46 also applies if the shaft of a spinning gyroscope is at an angle to the horizontal. It holds as well for a spinning top, which is essentially a spinning gyroscope at an angle to the horizontal.

Review & Summary Rolling Bodies For a wheel of radius R rolling smoothly, vcom ! vR,

(11-2)

where vcom is the linear speed of the wheel’s center of mass and v is the angular speed of the wheel about its center. The wheel may also be viewed as rotating instantaneously about the point P of the “road” that is in contact with the wheel. The angular speed of the wheel about this point is the same as the angular speed of the wheel about its center. The rolling wheel has kinetic energy K!

1 2 2 Icomv

#

1 2 2 +v com,

(11-5)

where Icom is the rotational inertia of the wheel about its center of mass and M is the mass of the wheel. If the wheel is being accelerated but is still rolling smoothly, the acceleration of the center of mass : acom is related to the angular acceleration a about the center with acom ! aR.

(11-6)

If the wheel rolls smoothly down a ramp of angle u, its acceleration along an x axis extending up the ramp is acom, x ! %

g sin u . 1 # Icom /MR2

(11-10)

Torque as a Vector In three dimensions, torque t: is a vector quantity defined relative to a fixed point (usually an origin); it is :

t: ! :r $ F ,

(11-14)

:

where F is a force applied to a particle and r is a position vector locating the particle relative to the fixed point.The magnitude of t: is :

t ! rF sin f ! rF" ! r"F, :

(11-15, 11-16, 11-17) :

where f is the angle between F and r , F" is the component of F : perpendicular to :r , and r" is the moment arm of F . The direction : of t is given by the right-hand rule. :

319

QU ESTIONS :

Angular Momentum of a Particle The angular momentum !

of a particle with linear momentum p, mass m, and linear velocity v is a vector quantity defined relative to a fixed point (usually an origin) as :

:

:

The time rate of change of this angular momentum is equal to the net external torque on the system (the vector sum of the torques due to interactions with particles external to the system): :

(11-18)

: ! ! :r $ p ! m(r: $ v:).

t:net !

:

The magnitude of ! is given by (11-19)

Angular Momentum of a Rigid Body For a rigid body

(11-20)

! r" p ! r" mv,

(11-21)

rotating about a fixed axis, the component of its angular momentum parallel to the rotation axis is

where f is the angle between :r and p:, p" and v" are the components of p: and v: perpendicular to :r , and r" is the perpendicular distance between the fixed point and the extension of p: . The direc: tion of ! is given by the right-hand rule for cross products.

L ! Iv

law for a particle can be written in angular form as :

d! , (11-23) dt : : where t net is the net torque acting on the particle and ! is the angular momentum of the particle. t:net !

Angular Momentum of a System of Particles The angu:

lar momentum L of a system of particles is the vector sum of the angular momenta of the individual particles: :

:

L ! !1 # !2 # " " " # !n !

n

' ! i. i!1 :

(11-26)

(11-31)

(rigid body, fixed axis).

Conservation of Angular Momentum The angular mo:

mentum L of a system remains constant if the net external torque acting on the system is zero:

Newton’s Second Law in Angular Form Newton’s second

:

(11-29)

(system of particles).

! rp" ! rmv"

! ! rmv sin f

:

dL dt

:

L ! a constant or

: Li

!

: Lf

(11-32)

(isolated system)

(11-33)

(isolated system).

This is the law of conservation of angular momentum.

Precession of a Gyroscope A spinning gyroscope can precess about a vertical axis through its support at the rate *!

Mgr , Iv

(11-46)

where M is the gyroscope’s mass, r is the moment arm, I is the rotational inertia, and v is the spin rate.

Questions 1 Figure 11-23 shows three particles of the same mass and the same constant speed moving as indicated by the velocity vectors. Points a, b, c, and d form a square, with point e at the center. Rank the points according to the magnitude of the net angular momentum of the three-particle system when measured about the points, greatest first.

a

b

:

above the point of contact), and (c) force F3 (the line of action passes to the right of the point of contact)?

e

4 The position vector :r of a particle relative to a certain point : has a magnitude of 3 m, and the force F on the particle has a mag: nitude of 4 N. What is the angle between the directions of :r and F if the magnitude of the associated torque equals (a) zero and (b) 12 N "m? y

c

d

Figure 11-23 Question 1. y A

3

5 In Fig. 11-26, three forces of the same magnitude are applied to a par: ticle at the origin ( F1 acts directly into the plane of the figure). Rank the forces according to the magnitudes of the torques they create about (a) point P1, (b) point P2, and (c) point P3, greatest first.

P3

2 Figure 11-24 shows two particles A and B at xyz coordinates 1 (1 m, 1 m, 0) and (1 m, 0, 1 m). 2 Acting on each particle are three 4 x numbered forces, all of the same magnitude and each directed paral5 lel to an axis. (a) Which of the z B 6 forces produce a torque about the Figure 11-24 Question 2. origin that is directed parallel to y? (b) Rank the forces according to F3 the magnitudes of the torques they F2 produce on the particles about the origin, greatest first.

6 The angular momenta !(t) of a Figure 11-26 Question 5. particle in four situations are (1) ! ! 3t # 4; (2) ! ! %6t 2; (3) ! ! 2; (4) ! ! 4/t. In which situation is the net torque on the particle (a) zero, (b) positive and constant, (c) negative and increasing in magnitude (t , 0), and (d) negative and decreasing in magnitude (t , 0)?

3 What happens to the initially stationary yo-yo in Fig. 11-25 if you pull it : via its string with (a) force F2 (the line of action passes through the point of contact on the table, as indicated), : (b) force F1 (the line of action passes

7 A rhinoceros beetle rides the rim of a horizontal disk rotating counterclockwise like a merry-go-round. If the beetle then walks along the rim in the direction of the rotation, will the magnitudes of the following quantities (each measured about the rotation axis) increase, decrease, or remain the same (the disk is still rotating in the counterclockwise direction): (a) the angular momentum of the

F1

Figure 11-25 Question 3.

F2

P2 P1

F3

x

F1

320

CHAPTE R 11 ROLLI NG, TORQU E, AN D ANG U L AR M OM E NTU M

beetle – disk system, (b) the angular momentum and angular velocity of the beetle, and (c) the angular momentum and angular velocity of the disk? (d) What are your answers if the beetle walks in the direction opposite the rotation?

ing to the magnitude of the angular momentum of the particle measured about them, greatest first. y c (1, 3)

1 8 Figure 11-27 shows an overhead view of a rectangular slab that can O 2 spin like a merry-go-round about its 7 3 center at O. Also shown are seven 4 5 6 paths along which wads of bubble gum can be thrown (all with the Figure 11-27 Question 8. same speed and mass) to stick onto the stationary slab. (a) Rank the paths according to the angular speed that the slab (and gum) will have after the gum sticks, greatest first. (b) For which paths will the angular momentum of the slab (and gum) about O be negative from L the view of Fig. 11-27?

9 Figure 11-28 gives the angular momentum magnitude L of a wheel versus time t. Rank the four lettered time intervals according to the magnitude of the torque acting on the wheel, greatest first.

A

B

C

D

(–3, 1)

a

e

v

(9, 1) x

(–1, –2)

d (4, –1)

b

Figure 11-29 Question 10. 11 A cannonball and a marble roll smoothly from rest down an incline. Is the cannonball’s (a) time to the bottom and (b) translational kinetic energy at the bottom more than, less than, or the same as the marble’s? 12 A solid brass cylinder and a solid wood cylinder have the same radius and mass (the wood cylinder is longer). Released together from rest, they roll down an incline. (a) Which cylinder reaches the bottom first, or do they tie? (b) The wood cylinder is then shortened to match the length of the brass cylinder, and the brass cylinder is drilled out along its long (central) axis to match the mass of the wood cylinder.Which cylinder now wins the race, or do they tie?

t

Figure 11-28 Question 9.

10 Figure 11-29 shows a particle moving at constant velocity v: and five points with their xy coordinates. Rank the points accord-

Problems Tutoring problem available (at instructor’s discretion) in WileyPLUS and WebAssign SSM • – •••

Worked-out solution available in Student Solutions Manual

WWW Worked-out solution is at

Number of dots indicates level of problem difficulty

ILW

Interactive solution is at

http://www.wiley.com/college/halliday

Additional information available in The Flying Circus of Physics and at flyingcircusofphysics.com

•2 An automobile traveling at 80.0 km/h has tires of 75.0 cm diameter. (a) What is the angular speed of the tires about their axles? (b) If the car is brought to a stop uniformly in 30.0 complete turns of the tires (without skidding), what is the magnitude of the angular acceleration of the wheels? (c) How far does the car move during the braking? Module 11-2 Forces and Kinetic Energy of Rolling •3 SSM A 140 kg hoop rolls along a horizontal floor so that the hoop’s center of mass has a speed of 0.150 m/s. How much work must be done on the hoop to stop it? •4 A uniform solid sphere rolls down an incline. (a) What must be the incline angle if the linear acceleration of the center of the sphere is to have a magnitude of 0.10g? (b) If a frictionless block were to slide down the incline at that angle, would its acceleration magnitude be more than, less than, or equal to 0.10g? Why?

•5 ILW A 1000 kg car has four 10 kg wheels. When the car is moving, what fraction of its total kinetic energy is due to rotation of the wheels about their axles? Assume vs that the wheels are uniform disks of the same mass and size.Why do you not need to know the radius of the wheels? ••6 Figure 11-30 gives the speed v versus time t for a 0.500 kg object of radius 6.00 cm that rolls smoothly down a 30( ramp. The scale on the velocity axis is set by vs ! 4.0 m/s. What is the rotational inertia of the object?

v (m/s)

Module 11-1 Rolling as Translation and Rotation Combined •1 A car travels at 80 km/h on a level road in the positive direction of an x axis. Each tire has a diameter of 66 cm. Relative to a woman riding in the car and in unit-vector notation, what are the velocity v: at the (a) center, (b) top, and (c) bottom of the tire and the magnitude a of the acceleration at the (d) center, (e) top, and (f) bottom of each tire? Relative to a hitchhiker sitting next to the road and in unit-vector notation, what are the velocity v: at the (g) center, (h) top, and (i) bottom of the tire and the magnitude a of the acceleration at the (j) center, (k) top, and (l) bottom of each tire?

••7 ILW In Fig. 11-31, a solid cylinder of radius 10 cm and mass 12 kg starts from rest and rolls without slipping a distance L ! 6.0 m down a roof that is inclined at angle u ! 30(. (a) What is the angular speed of the cylinder about its center as it leaves the roof? (b) The roof’s edge is at height H ! 5.0 m. How far horizontally from the roof’s edge does the cylinder hit the level ground?

0

0.2

0.4 0.6 t (s)

0.8

Figure 11-30 Problem 6.

L

θ

H

Figure 11-31 Problem 7.

1

PROB LE M S

••8 Figure 11-32 shows the po- U (J) tential energy U(x) of a solid Us ball that can roll along an x axis. The scale on the U axis is set by Us ! 100 J. The ball is uniform, rolls smoothly, and has a mass of 0.400 kg. It is released at x ! 7.0 m headed in the negative direction of the x axis with a mechanical 0 2 4 6 8 10 12 14 x (m) energy of 75 J. (a) If the ball can reach x ! 0 m, what is its speed Figure 11-32 Problem 8. there, and if it cannot, what is its turning point? Suppose, instead, it is headed in the positive direction of the x axis when it is released at x ! 7.0 m with 75 J. (b) If the ball can reach x ! 13 m, what is its speed there, and if it cannot, what is its turning point? ••9 In Fig. 11-33, a solid ball rolls smoothly from rest (starting at H height H ! 6.0 m) until it leaves the h horizontal section at the end of the A track, at height h ! 2.0 m. How far Figure 11-33 Problem 9. horizontally from point A does the ball hit the floor? ••10 A hollow sphere of radius 0.15 m, with rotational inertia I ! 0.040 kg "m2 about a line through its center of mass, rolls without slipping up a surface inclined at 30( to the horizontal. At a certain initial position, the sphere’s total kinetic energy is 20 J. (a) How much of this initial kinetic energy is rotational? (b) What is the speed of the center of mass of the sphere at the initial position? When the sphere has moved 1.0 m up the incline from its initial position, what are (c) its total kinetic energy and (d) the speed of its center of mass? ••11 In Fig. 11-34, a constant hori: zontal force Fapp of magnitude 10 N is Fapp applied to a wheel of mass 10 kg and radius 0.30 m. The wheel rolls smoothly on the horizontal surface, x and the acceleration of its center of 2 mass has magnitude 0.60 m/s . (a) In Figure 11-34 Problem 11. unit-vector notation, what is the frictional force on the wheel? (b) What is the rotational inertia of the wheel about the rotation axis through its center of mass? ••12 In Fig. 11-35, a solid brass ball of mass 0.280 g will roll smoothly along a loop-the-loop track when released from rest along h R Q the straight section. The circular loop has radius R ! 14.0 cm, and the ball has radius r - R. (a) What is h if the ball is on the verge of leaving Figure 11-35 Problem 12. the track when it reaches the top of the loop? If the ball is released at height h ! 6.00R, what are the (b) magnitude and (c) direction of the horizontal force component acting h on the ball at point Q? •••13 Nonuniform ball. In Fig. 1136, a ball of mass M and radius R

rolls smoothly from rest down a ramp and onto a circular loop of radius 0.48 m. The initial height of the ball is h ! 0.36 m. At the loop bottom, the magnitude of the normal force on the ball is 2.00Mg. The ball consists of an outer spherical shell (of a certain uniform density) that is glued to a central sphere (of a different uniform density). The rotational inertia of the ball can be expressed in the general form I ! bMR2, but b is not 0.4 as it is for a ball of uniform density. Determine b. •••14 In Fig. 11-37, a small, solid, uniform ball is to be shot from point P so that it rolls smoothly along a horizontal path, up along a ramp, and onto a plateau. Then it leaves the plateau horizontally to land on a game board, at a horizontal distance d from the right edge of the plateau. The vertical heights are h1 ! 5.00 cm and h2 ! 1.60 cm. With what speed must the ball be shot at point P for it to land at d ! 6.00 cm? h2 h1

d

Ball P

Figure 11-37 Problem 14. •••15 A bowler throws a bowling ball of radius R ! 11 cm vcom fk along a lane. The ball (Fig. 11-38) x slides on the lane with initial speed Figure 11-38 Problem 15. vcom,0 ! 8.5 m/s and initial angular speed v0 ! 0. The coefficient of kinetic friction between the ball and the lane is 0.21. The kinetic fric: tional force f k acting on the ball causes a linear acceleration of the ball while producing a torque that causes an angular acceleration of the ball. When speed vcom has decreased enough and angular speed v has increased enough, the ball stops sliding and then rolls smoothly. (a) What then is vcom in terms of v? During the sliding, what are the ball’s (b) linear acceleration and (c) angular acceleration? (d) How long does the ball slide? (e) How far does the ball slide? (f) What is the linear speed of the ball when smooth rolling begins? •••16 Nonuniform cylindrical object. In Fig. 11-39, a cylindrical object of mass M and radius R rolls smoothly from rest down a ramp and onto a horizontal section. From there it rolls off the ramp and onto the floor, landing a horizontal distance d ! 0.506 m from the end of the ramp. The initial height of the object is H ! 0.90 m; the end of the ramp is at height h ! 0.10 m. The object consists of an outer cylindrical shell (of a certain uniform density) that is glued to a central cylinder (of a different uniform density). The rotational inertia of the object can be expressed in the general form I ! bMR2, but b is not 0.5 as it is for a cylinder of uniform density. Determine b.

H

h

d

Figure 11-36 Problem 13.

321

Figure 11-39 Problem 16.

322

CHAPTE R 11 ROLLI NG, TORQU E, AN D ANG U L AR M OM E NTU M

Module 11-3 The Yo-Yo •17 SSM A yo-yo has a rotational inertia of 950 g "cm2 and a mass of 120 g. Its axle radius is 3.2 mm, and its string is 120 cm long. The yo-yo rolls from rest down to the end of the string. (a) What is the magnitude of its linear acceleration? (b) How long does it take to reach the end of the string? As it reaches the end of the string, what are its (c) linear speed, (d) translational kinetic energy, (e) rotational kinetic energy, and (f) angular speed? •18 In 1980, over San Francisco Bay, a large yo-yo was released from a crane. The 116 kg yo-yo consisted of two uniform disks of radius 32 cm connected by an axle of radius 3.2 cm. What was the magnitude of the acceleration of the yo-yo during (a) its fall and (b) its rise? (c) What was the tension in the cord on which it rolled? (d) Was that tension near the cord’s limit of 52 kN? Suppose you build a scaled-up version of the yo-yo (same shape and materials but larger). (e) Will the magnitude of your yo-yo’s acceleration as it falls be greater than, less than, or the same as that of the San Francisco yo-yo? (f) How about the tension in the cord? Module 11-4 Torque Revisited •19 In unit-vector notation, what is the net torque about the origin on a flea located at coordinates (0, %4.0 m, 5.0 m) when forces : : F1 ! (3.0 N)kˆ and F2 ! (%2.0 N)jˆ act on the flea? •20 A plum is located at coordinates (%2.0 m, 0, 4.0 m). In unitvector notation, what is the torque about the origin on the plum if : that torque is due to a force F whose only component is (a) Fx ! 6.0 N, (b) Fx ! %6.0 N, (c) Fz ! 6.0 N, and (d) Fz ! %6.0 N? •21 In unit-vector notation, what is the torque about the origin on a particle located at coordinates (0, %4.0 m, 3.0 m) if that torque is : due to (a):force F1 with components F1x ! 2.0 N, F1y ! F1z ! 0, and (b) force F2 with components F2x ! 0, F2y ! 2.0 N, F2z ! 4.0 N? ••22 A particle moves through an xyz coordinate system while a force acts on the particle. When the particle has the position vector :r ! (2.00 m)iˆ % (3.00 m)jˆ # (2.00 m)kˆ , the force is given : by F ! Fxiˆ # (7.00 N)jˆ % (6.00 N)kˆ and the corresponding torque about the origin is t: ! (4.00 N"m)iˆ # (2.00 N"m)jˆ % (1.00 N"m)kˆ. Determine Fx. : ••23 Force F ! (2.0 N)iˆ % (3.0 N)kˆ acts on a pebble with position : vector r ! (0.50 m)jˆ % (2.0 m)kˆ relative to the origin. In unit-vector notation, what is the resulting torque on the pebble about (a) the origin and (b) the point (2.0 m, 0, %3.0 m)? ••24 In unit-vector notation, what is the torque about the origin on a jar of jalapeño peppers located at coordinates (3.0 m, %2.0 m, : 4.0 m) due to (a) force F1 ! (3.0 N)iˆ % (4.0 N)jˆ # (5.0 N)kˆ , (b) : force F2 ! (%3.0 N)iˆ % (4.0 N)jˆ % (5.0 N)kˆ , and (c) the vector : : sum of F1 and F2? (d) Repeat part (c) for the torque about the point with coordinates (3.0 m, 2.0 m, 4.0 m). : ••25 SSM Force F ! (%8.0 N)iˆ # (6.0 N)jˆ acts on a particle with : position vector r ! (3.0 m)iˆ # (4.0 m)jˆ.What y are (a) the torque on the particle about the origin, in unit-vector notation, and (b) F θ the angle between the directions of :r 3 : and F ? Module 11-5 Angular Momentum •26 At the instant of Fig. 11-40, a 2.0 kg particle P has a position vector :r of magnitude 3.0 m and angle u1 ! 45( and a velocity vector v: of magnitude 4.0 m/s and angle : u2 ! 30(. Force F, of magnitude 2.0 N and

v

θ2

P

θ1

r

O

Figure 11-40 Problem 26.

x

angle u3 ! 30(, acts on P. All three vectors lie in the xy plane. About the origin, what are the (a) magnitude and (b) direction of the angular momentum of P and the (c) magnitude and (d) direction of the torque acting on P? : •27 SSM At one instant, force F ! 4.0jˆ N acts on a 0.25 kg object : that has position vector r ! (2.0iˆ % 2.0kˆ ) m and velocity vector v: ! (%5.0iˆ # 5.0kˆ ) m/s. About the origin and in unit-vector notation, what are (a) the object’s angular momentum and (b) the torque acting on the object? •28 A 2.0 kg particle-like object moves in a plane with velocity components vx ! 30 m/s and vy ! 60 m/s as it passes through the point with (x, y) coordinates of (3.0, %4.0) m. Just then, in unitvector notation, what is its angular momentum relative to (a) the origin and (b) the point located at (%2.0, %2.0) m? •29 ILW In the instant of Fig. 11-41, P1 v1 two particles move in an xy plane. y Particle P1 has mass 6.5 kg and x d1 v2 speed v1 ! 2.2 m/s, and it is at distance d1 ! 1.5 m from point O. d2 Particle P2 has mass 3.1 kg and speed O P2 v2 ! 3.6 m/s, and it is at distance d2 ! Figure 11-41 Problem 29. 2.8 m from point O. What are the (a) magnitude and (b) direction of the net angular momentum of the two particles about O? ••30 At the instant the displacement of a 2.00 kg object relative : to the origin is d ! (2.00 m)iˆ # (4.00 m) jˆ % (3.00 m)kˆ , its veloc: ity is v ! %(6.00 m/s)iˆ # (3.00 m/s)jˆ # (3.00 m/s)kˆ and it is sub: ject to a force F ! (6.00 N)iˆ % (8.00 N)jˆ # (4.00 N)kˆ. Find (a) the acceleration of the object, (b) the angular momentum of the object about the origin, (c) the torque about the origin acting on the object, and (d) the angle between the velocity of the object and the force acting on the object. y

••31 In Fig. 11-42, a 0.400 kg ball is shot directly upward at initial speed 40.0 Ball P m/s. What is its angular momentum x about P, 2.00 m horizontally from the launch point, when the ball is (a) at Figure 11-42 Problem 31. maximum height and (b) halfway back to the ground? What is the torque on the ball about P due to the gravitational force when the ball is (c) at maximum height and (d) halfway back to the ground? Module 11-6 Newton’s Second Law in Angular Form •32 A particle is acted on by two torques about the origin: &:1 has a magnitude of 2.0 N "m and is directed in the positive direction of the x axis, and t:2 has a magnitude of 4.0 N "m and is directed in the negative direction of the y axis. In unit-vector : : notation, find d ! /dt, where ! is the angular momentum of the particle about the origin. •33 SSM WWW ILW At time t ! 0, a 3.0 kg particle with velocity v: ! (5.0 m/s)iˆ % (6.0 m/s)jˆ is at x ! 3.0 m, y ! 8.0 m. It is pulled by a 7.0 N force in the negative x direction.About the origin, what are (a) the particle’s angular momentum, (b) the torque acting on the particle, and (c) the rate at which the angular momentum is changing? •34 A particle is to move in an xy plane, clockwise around the origin as seen from the positive side of the z axis. In unit-vector notation, what torque acts on the particle if the magnitude of its angular momentum about the origin is (a) 4.0 kg "m2/s, (b) 4.0t 2 kg "m2/s, (c) 4.0 2t kg"m2/s, and (d) 4.0/t2 kg "m2/s?

PROB LE M S

••35 At time t, the vector : r ! 4.0t 2 iˆ % (2.0t # 6.0t 2)jˆ gives the position of a 3.0 kg particle relative to the origin of an xy coordinate r is in meters and t is in seconds). (a) Find an expression for system (: the torque acting on the particle relative to the origin. (b) Is the magnitude of the particle’s angular momentum relative to the origin increasing, decreasing, or unchanging? Module 11-7 Angular Momentum of a Rigid Body •36 Figure 11-43 shows three rotating, uniform disks that are coupled by belts. One belt runs around the rims of disks A and C.Another belt runs around a central hub on disk A and the rim of disk B. The belts move smoothly without slippage on the rims and hub. Disk A has radius R; its hub has radius 0.5000R; disk B has radius 0.2500R; and disk C has radius 2.000R. Disks B and C have the same density (mass per unit volume) and thickness. What is the ratio of the magnitude of the angular momentum of disk C to that of disk B?

rotation of 2.5 s. Assuming R ! 0.50 m and m ! 2.0 kg, calculate (a) the structure’s rotational inertia about the axis of rotation and (b) its angular momentum about that axis. ••42 Figure 11-46 gives the torque t that acts on an initially stationary disk that can rotate about its center like a merry-go-round. The scale on the t axis is set by ts ! 4.0 N"m.What is the angular momentum of the disk about the rotation axis at times (a) t ! 7.0 s and (b) t ! 20 s?

τ (N • m)

τs

0

Belt

323

4

8

12

16

t (s) 20

Belt B C

A

Figure 11-46 Problem 42.

Figure 11-43 Problem 36. m

•37 In Fig. 11-44, three particles ω m of mass m ! 23 g are fastened to d three rods of length d ! 12 cm and m d negligible mass. The rigid assembly rotates around point O at the angud O lar speed v ! 0.85 rad/s. About O, what are (a) the rotational inertia Figure 11-44 Problem 37. of the assembly, (b) the magnitude of the angular momentum of the middle particle, and (c) the magnitude of the angular momentum of the asssembly? •38 A sanding disk with rotational inertia 1.2 $ 10 %3 kg "m2 is attached to an electric drill whose motor delivers a torque of magnitude 16 N "m about the central axis of the disk. About that axis and with the torque applied for 33 ms, what is the magnitude of the (a) angular momentum and (b) angular velocity of the disk? •39 SSM The angular momentum of a flywheel having a rotational inertia of 0.140 kg " m2 about its central axis decreases from 3.00 to 0.800 kg "m2/s in 1.50 s. (a) What is the magnitude of the average torque acting on the flywheel about its central axis during this period? (b) Assuming a constant angular acceleration, through what angle does the flywheel turn? (c) How much work is done on the wheel? (d) What is the average power of the flywheel? ••40 A disk with a rotational inertia of 7.00 kg"m2 rotates like a merry-go-round while undergoing a time-dependent torque given by t ! (5.00 # 2.00t) N"m. At time t ! 1.00 s, its angular momenRotation axis tum is 5.00 kg"m2/s. What is its angular momentum at t ! 3.00 s? ••41 Figure 11-45 shows a rigid structure consisting of a circular hoop of radius R and mass m, and a square made of four thin bars, each of length R and mass m. The rigid structure rotates at a constant speed about a vertical axis, with a period of

R

2R

Figure 11-45 Problem 41.

Module 11-8 Conservation of Angular Momentum •43 In Fig. 11-47, two skaters, each of mass 50 kg, approach each other along parallel paths separated by 3.0 m. They have opposite velocities of 1.4 m/s each. One skater carries one end of a long pole of negligible mass, and the other skater grabs the Figure 11-47 Problem 43. other end as she passes. The skaters then rotate around the center of the pole. Assume that the friction between skates and ice is negligible. What are (a) the radius of the circle, (b) the angular speed of the skaters, and (c) the kinetic energy of the two-skater system? Next, the skaters pull along the pole until they are separated by 1.0 m. What then are (d) their angular speed and (e) the kinetic energy of the system? (f) What provided the energy for the increased kinetic energy? •44 A Texas cockroach of mass 0.17 kg runs counterclockwise around the rim of a lazy Susan (a circular disk mounted on a vertical axle) that has radius 15 cm, rotational inertia 5.0 $ 10 %3 kg "m2, and frictionless bearings. The cockroach’s speed (relative to the ground) is 2.0 m/s, and the lazy Susan turns clockwise with angular speed v0 ! 2.8 rad/s. The cockroach finds a bread crumb on the rim and, of course, stops. (a) What is the angular speed of the lazy Susan after the cockroach stops? (b) Is mechanical energy conserved as it stops? •45 SSM WWW A man stands on a platform that is rotating (without friction) with an angular speed of 1.2 rev/s; his arms are outstretched and he holds a brick in each hand. The rotational inertia of the system consisting of the man, bricks, and platform about the central vertical axis of the platform is 6.0 kg "m2. If by moving the bricks the man decreases the rotational inertia of the system to 2.0 kg "m2, what are (a) the resulting angular speed of the platform and (b) the ratio of the new kinetic energy of the system to the original kinetic energy? (c) What source provided the added kinetic energy? •46 The rotational inertia of a collapsing spinning star drops to 13 its initial value. What is the ratio of the new rotational kinetic energy to the initial rotational kinetic energy?

324

CHAPTE R 11 ROLLI NG, TORQU E, AN D ANG U L AR M OM E NTU M

•47 SSM A track is mounted on a large wheel that is free to turn with negligible friction about a vertical axis (Fig. 11-48). A toy train of mass m is placed on the track and, with the Figure 11-48 Problem 47. system initially at rest, the train’s electrical power is turned on. The train reaches speed 0.15 m/s with respect to the track. What is the wheel’s angular speed if its mass is 1.1m and its radius is 0.43 m? (Treat it as a hoop, and neglect the mass of the spokes and hub.) ωb ω (rad/s)

•48 A Texas cockroach walks from the center of a circular disk (that rotates like a merry-go-round without external torques) out to the edge at radius R. The angular speed of the cockroach–disk system for the walk is given in Fig. 11-49 (va = 5.0 rad/s and vb = 6.0 rad/s). After reaching R, what fraction of the rotational inertia of the disk does the cockroach have?

ωa 0

Radial distance

R

Figure 11-49 Problem 48.

•49 Two disks are mounted (like a merry-go-round) on lowfriction bearings on the same axle and can be brought together so that they couple and rotate as one unit. The first disk, with rotational inertia 3.30 kg "m2 about its central axis, is set spinning counterclockwise at 450 rev/min. The second disk, with rotational inertia 6.60 kg "m2 about its central axis, is set spinning counterclockwise at 900 rev/min. They then couple together. (a) What is their angular speed after coupling? If instead the second disk is set spinning clockwise at 900 rev/min, what are their (b) angular speed and (c) direction of rotation after they couple together? •50 The rotor of an electric motor has rotational inertia Im ! 2.0 $ 10 %3 kg " m2 about its central axis. The motor is used to change the orientation of the space probe in which it is mounted. The motor axis is mounted along the central axis of the probe; the probe has rotational inertia Ip ! 12 kg "m2 about this axis. Calculate the number of revolutions of the rotor required to turn the probe through 30( about its central axis. •51 SSM ILW A wheel is rotating freely at angular speed 800 rev/min on a shaft whose rotational inertia is negligible. A second wheel, initially at rest and with twice the rotational inertia of the first, is suddenly coupled to the same shaft. (a) What is the angular speed of the resultant combination of the shaft and two wheels? (b) What fraction of the original rotational kinetic energy is lost? ••52 A cockroach of mass m lies on the rim of a uniform disk of mass 4.00m that can rotate freely about its center like a merry-goround. Initially the cockroach and disk rotate together with an angular velocity of 0.260 rad/s. Then the cockroach walks halfway to the center of the disk. (a) What then is the angular velocity of the cockroach – disk system? (b) What is the ratio K/K0 of the new kinetic energy of the system to its initial kinetic energy? (c) What accounts for the change in the kinetic energy? ••53 In Fig. 11-50 (an overhead view), a uniform thin rod of length 0.500 m and mass 4.00 kg can rotate in a horizontal plane about a vertical axis through its center. The rod is at rest when a 3.00 g bullet traveling in the rotation plane is fired into one end of the rod. In the view from

Axis

θ

Figure 11-50 Problem 53.

above, the bullet’s path makes angle u ! 60.0( with the rod (Fig. 1150). If the bullet lodges in the rod and the angular velocity of the rod is 10 rad/s immediately after the collision, what is the bullet’s speed just before impact? ••54 Figure 11-51 shows an overhead view of a ring that can rotate about its center like a merrygo-round. Its outer radius R2 is R1 0.800 m, its inner radius R1 is R2/2.00, R2 its mass M is 8.00 kg, and the mass of the crossbars at its center is negligible. It initially rotates at an anguFigure 11-51 Problem 54. lar speed of 8.00 rad/s with a cat of mass m ! M/4.00 on its outer edge, at radius R2. By how much does the cat increase the kinetic energy of the cat – ring system if the cat crawls to the inner edge, at radius R1? ••55 A horizontal vinyl record of mass 0.10 kg and radius 0.10 m rotates freely about a vertical axis through its center with an angular speed of 4.7 rad/s and a rotational inertia of 5.0 $ 10 %4 kg "m2. Putty of mass 0.020 kg drops vertically onto the record from above and sticks to the edge of the record. What is the angular speed of the record immediately afterwards? ••56 In a long jump, an athlete leaves the ground with an initial angular momentum that tends to rotate her body forward, threatening to ruin her landing. To counter this tendency, she rotates her outstretched arms to “take up” the angular momentum (Fig. 1118). In 0.700 s, one arm sweeps through 0.500 rev and the other arm sweeps through 1.000 rev. Treat each arm as a thin rod of mass 4.0 kg and length 0.60 m, rotating around one end. In the athlete’s reference frame, what is the magnitude of the total angular momentum of the arms around the common rotation axis through the shoulders? ••57 A uniform disk of mass 10m and radius 3.0r can rotate freely about its fixed center like a merry-go-round. A smaller uniform disk of mass m and radius r lies on top of the larger disk, concentric with it. Initially the two disks rotate together with an angular velocity of 20 rad/s.Then a slight disturbance causes the smaller disk to slide outward across the larger disk, until the outer edge of the smaller disk catches on the outer edge of the larger disk. Afterward, the two disks again rotate together (without further sliding). (a) What then is their angular velocity about the center of the larger disk? (b) What is the ratio K/K0 of the new kinetic energy of the two-disk system to the system’s initial kinetic energy? ••58 A horizontal platform in the shape of a circular disk rotates on a frictionless bearing about a vertical axle through the center of the disk. The platform has a mass of 150 kg, a radius of 2.0 m, and a rotational inertia of 300 kg "m2 about the axis of rotation. A 60 kg student walks slowly from the rim of the platform toward the center. If the angular speed of the system is 1.5 rad/s when the student starts at the rim, what is the angular speed when she is 0.50 m from the center? ••59 Figure 11-52 is an overhead view of a thin uniform rod of length Rotation 0.800 m and mass M rotating horizonaxis tally at angular speed 20.0 rad/s about Figure 11-52 Problem 59. an axis through its center. A particle of mass M/3.00 initially attached to one end is ejected from the rod and travels along a path that is perpendicular to the rod at the instant of ejection. If the particle’s speed vp is 6.00 m/s greater than the speed of the rod end just after ejection, what is the value of vp?

PROB LE M S

••60 In Fig. 11-53, a 1.0 g bullet is fired A into a 0.50 kg block attached to the end of a 0.60 m nonuniform rod of mass 0.50 kg. The block – rod – bullet system Rod then rotates in the plane of the figure, about a fixed axis at A. The rotational inertia of the rod alone about that axis at A is 0.060 kg "m2. Treat the block as a particle. (a) What then is the rotational Block inertia of the block – rod – bullet system Bullet about point A? (b) If the angular speed of the system about A just after impact Figure 11-53 Problem 60. is 4.5 rad/s, what is the bullet’s speed Rotation axis just before impact? ••61 The uniform rod (length 0.60 m, mass 1.0 kg) in Fig. 11-54 rotates in the plane of the figure about an axis through one end, with a rotational inertia of 0.12 kg "m2. As the rod swings through its lowest position, it collides with a 0.20 kg putty wad that sticks to the end of the rod. If the rod’s angular speed just before collision is 2.4 rad/s, what is the angular speed of the rod – putty system immediately after collision?

Rod

Figure 11-54 Problem 61.

•••62 During a jump to his partner, an aerialist is to make a quadruple somersault lasting a time t ! 1.87 s. For the first and last quarter-revolution, he is in the extended orientation shown in Fig. 11-55, with rotational inertia I1 ! 19.9 kg "m2 around his center of mass (the dot). During the rest of the flight he is in a tight tuck, with rotational inertia I2 ! 3.93 kg "m2. What must be his angular speed v 2 around his center of mass during the tuck? ω2

Tuck I2

Parabolic path of aerialist

ω1

325

tangent to the outer edge of the merry-go-round, as shown. What is the angular speed of the merry-go-round just after the ball is caught? •••64 A ballerina begins a tour jeté (Fig. 11-19a) with angular speed vi and a rotational inertia consisting of two parts: Ileg ! 1.44 kg"m2 for her leg extended outward at angle u ! 90.0( to her body and Itrunk ! 0.660 kg"m2 for the rest of her body (primarily her trunk). Near her maximum height she holds both legs at angle u ! 30.0( to her body and has angular speed vf (Fig. 11-19b). Assuming that Itrunk has not changed, what is the ratio vf /vi? •••65 SSM WWW Two 2.00 kg balls are attached to the ends of a thin rod of length 50.0 cm and negliPutty wad gible mass.The rod is free to rotate in a vertical plane without friction Rotation axis about a horizontal axis through its center. With the rod initially horizontal (Fig. 11-57), a 50.0 g wad of wet Figure 11-57 Problem 65. putty drops onto one of the balls, hitting it with a speed of 3.00 m/s and then sticking to it. (a) What is the angular speed of the system just after the putty wad hits? (b) What is the ratio of the kinetic energy of the system after the collision to that of the putty wad just before? (c) Through what angle will the system rotate before it momentarily stops? •••66 In Fig. 11-58, a small 50 g block slides down a frictionless surface through height h ! 20 cm and then sticks to a uniform rod of mass 100 g and length 40 cm.The rod pivots about point O through angle u before momentarily stopping. Find u.

O

θ

h •••67 Figure 11-59 is an overhead view of a thin uniform rod of length 0.600 m and mass M rotating Figure 11-58 Problem 66. horizontally at 80.0 rad/s counterclockwise about an axis through its center. A particle of mass M/3.00 and traveling horizontally at speed 40.0 m/s hits the rod and sticks. The particle’s path is perpendicular to the rod at the instant of the hit, at a distance d from the rod’s center. (a) At what value of d are rod and particle stationary after the hit? (b) In which direction do rod and particle rotate if d is greater than this value?

ω1

I1

Rotation axis

I1

Release

Particle d

Catch

Figure 11-59 Problem 67.

Figure 11-55 Problem 62. •••63 In Fig. 11-56, a 30 kg child stands on the edge of a stationary merry-go-round of radius 2.0 m. The rotational inertia of the merrygo-round about its rotation axis is 150 kg "m2. The child catches a ball of mass 1.0 kg thrown by a friend. Just before the ball is caught, it has a horizontal velocity v: of magnitude 12 m/s, at angle f ! 37( with a line

Ball v

Child

φ

Figure 11-56 Problem 63.

Module 11-9 Precession of a Gyroscope ••68 A top spins at 30 rev/s about an axis that makes an angle of 30( with the vertical. The mass of the top is 0.50 kg, its rotational inertia about its central axis is 5.0 $ 10 %4 kg "m2, and its center of mass is 4.0 cm from the pivot point. If the spin is clockwise from an overhead view, what are the (a) precession rate and (b) direction of the precession as viewed from overhead? ••69 A certain gyroscope consists of a uniform disk with a 50 cm radius mounted at the center of an axle that is 11 cm long and of negligible mass. The axle is horizontal and supported at one end. If the spin rate is 1000 rev/min, what is the precession rate?

326

CHAPTE R 11 ROLLI NG, TORQU E, AN D ANG U L AR M OM E NTU M

Additional Problems 70 A uniform solid ball rolls smoothly along a floor, then up a ramp inclined at 15.0(. It momentarily stops when it has rolled 1.50 m along the ramp. What was its initial speed? 71 SSM In Fig. 11-60, a constant Fapp : horizontal force Fapp of magnitude 12 Fishing line N is applied to a uniform solid cylinder by fishing line wrapped around the cylinder. The mass of the cylinder x is 10 kg, its radius is 0.10 m, and the cylinder rolls smoothly on the horiFigure 11-60 Problem 71. zontal surface. (a) What is the magnitude of the acceleration of the center of mass of the cylinder? (b) What is the magnitude of the angular acceleration of the cylinder about the center of mass? (c) In unit-vector notation, what is the frictional force acting on the cylinder? 72 A thin-walled pipe rolls along the floor. What is the ratio of its translational kinetic energy to its rotational kinetic energy about the central axis parallel to its length? 73 SSM A 3.0 kg toy car moves along an x axis with a velocity given by : v ! %2.0t 3iˆ m/s, with t in seconds. For t , 0, what are : (a) the angular momentum L of the car and (b) the torque : t on : the car, both calculated about the origin? What are (c) L and (d) : t : about the point (2.0 m, 5.0 m, 0)? What are (e) L and (f) : t about the point (2.0 m, %5.0 m, 0)? 74 A wheel rotates clockwise about its central axis with an angular momentum of 600 kg "m2/s. At time t ! 0, a torque of magnitude 50 N "m is applied to the wheel to reverse the rotation. At what time t is the angular speed zero? 75 SSM In a playground, there is a small merry-go-round of radius 1.20 m and mass 180 kg. Its radius of gyration (see Problem 79 of Chapter 10) is 91.0 cm. A child of mass 44.0 kg runs at a speed of 3.00 m/s along a path that is tangent to the rim of the initially stationary merry-go-round and then jumps on. Neglect friction between the bearings and the shaft of the merry-go-round. Calculate (a) the rotational inertia of the merry-go-round about its axis of rotation, (b) the magnitude of the angular momentum of the running child about the axis of rotation of the merry-go-round, and (c) the angular speed of the merry-go-round and child after the child has jumped onto the merry-go-round. 76 A uniform block of granite in the shape of a book has face dimensions of 20 cm and 15 cm and a thickness of 1.2 cm. The density (mass per unit volume) of granite is 2.64 g/cm3. The block rotates around an axis that is perpendicular to its face and halfway between its center and a corner. Its angular momentum about that axis is 0.104 kg "m2/s. What is its rotational kinetic energy about that axis?

79 Wheels A and B in Fig. 11-61 are connected by a belt that does not slip. The radius of B is 3.00 times the radius of A. What would be the ratio of the rotational inertias IA/IB if the two wheels had (a) the same angular momentum about their central axes and (b) the same rotational kinetic energy?

A B

Figure 11-61 Problem 79.

80 A 2.50 kg particle that is moving horizontally over a floor with velocity (%3.00 m/s)ˆj undergoes a completely inelastic collision with a 4.00 kg particle that is moving horizontally over the floor with velocity (4.50 m/s)iˆ. The collision occurs at xy coordinates (%0.500 m, %0.100 m). After the collision and in unit-vector notation, what is the angular momentum of the stuck-together particles with respect to the origin? 81 SSM A uniform wheel Wheel of mass 10.0 kg and radius Axle 0.400 m is mounted rigidly on a massless axle through its center (Fig. 11-62). The Groove radius of the axle is 0.200 m, and the rotational inertia θ of the wheel – axle combination about its central axis Figure 11-62 Problem 81. is 0.600 kg "m2. The wheel is initially at rest at the top of a surface that is inclined at angle u ! 30.0( with the horizontal; the axle rests on the surface while the wheel extends into a groove in the surface without touching the surface. Once released, the axle rolls down along the surface smoothly and without slipping. When the wheel – axle combination has moved down the surface by 2.00 m, what are (a) its rotational kinetic energy and (b) its translational kinetic energy? 82 A uniform rod rotates in a horizontal plane about a vertical axis through one end.The rod is 6.00 m long, weighs 10.0 N, and rotates at 240 rev/min. Calculate (a) its rotational inertia about the axis of rotation and (b) the magnitude of its angular momentum about that axis. 83 A solid sphere of weight 36.0 N rolls up an incline at an angle of 30.0(. At the bottom of the incline the center of mass of the sphere has a translational speed of 4.90 m/s. (a) What is the kinetic energy of the sphere at the bottom of the incline? (b) How far does the sphere travel up along the incline? (c) Does the answer to (b) depend on the sphere’s mass? 84 Suppose that the yo-yo in Problem 17, instead of rolling from rest, is thrown so that its initial speed down the string is 1.3 m/s. (a) How long does the yo-yo take to reach the end of the string? As it reaches the end of the string, what are its (b) total kinetic energy, (c) linear speed, (d) translational kinetic energy, (e) angular speed, and (f) rotational kinetic energy?

77 SSM Two particles, each of mass 2.90 $ 10 %4 kg and speed 5.46 m/s, travel in opposite directions along parallel lines separated by 4.20 cm. (a) What is the magnitude L of the angular momentum of the two-particle system around a point midway between the two lines? (b) Is the value different for a different location of the point? If the direction of either particle is reversed, what are the answers for (c) part (a) and (d) part (b)?

85 A girl of mass M stands on the rim of a frictionless merrygo-round of radius R and rotational inertia I that is not moving. She throws a rock of mass m horizontally in a direction that is tangent to the outer edge of the merry-go-round. The speed of the rock, relative to the ground, is v. Afterward, what are (a) the angular speed of the merry-go-round and (b) the linear speed of the girl?

78 A wheel of radius 0.250 m, moving initially at 43.0 m/s, rolls to a stop in 225 m. Calculate the magnitudes of its (a) linear acceleration and (b) angular acceleration. (c) Its rotational inertia is 0.155 kg "m2 about its central axis. Find the magnitude of the torque about the central axis due to friction on the wheel.

86 A body of radius R and mass m is rolling smoothly with speed v on a horizontal surface. It then rolls up a hill to a maximum height h. (a) If h ! 3v2/4g, what is the body’s rotational inertia about the rotational axis through its center of mass? (b) What might the body be?

C

H

A

P

T

E

R

1

2

Equilibrium and Elasticity 12-1 EQUILIBRIUM Learning Objectives After reading this module, you should be able to . . .

12.01 Distinguish between equilibrium and static equilibrium. 12.02 Specify the four conditions for static equilibrium.

12.03 Explain center of gravity and how it relates to center of mass. 12.04 For a given distribution of particles, calculate the coordinates of the center of gravity and the center of mass.

Key Ideas ● A rigid body at rest is said to be in static equilibrium. For

such a body, the vector sum of the external forces acting on it is zero: :

F net ! 0

(balance of forces).

If all the forces lie in the xy plane, this vector equation is equivalent to two component equations: Fnet,x ! 0

and

Fnet,y ! 0

(balance of forces).

● Static equilibrium also implies that the vector sum of the

external torques acting on the body about any point is zero, or t:net ! 0

If the forces lie in the xy plane, all torque vectors are parallel to the z axis, and the balance-of-torques equation is equivalent to the single component equation tnet,z ! 0

(balance of torques).

● The gravitational force acts individually on each element of

a body. The net effect of all individual actions may be found by : imagining an equivalent total gravitational force Fg acting at the center of gravity. If the gravitational acceleration g: is the same for all the elements of the body, the center of gravity is at the center of mass.

(balance of torques).

What Is Physics? Human constructions are supposed to be stable in spite of the forces that act on them. A building, for example, should be stable in spite of the gravitational force and wind forces on it, and a bridge should be stable in spite of the gravitational force pulling it downward and the repeated jolting it receives from cars and trucks. One focus of physics is on what allows an object to be stable in spite of any forces acting on it. In this chapter we examine the two main aspects of stability: the equilibrium of the forces and torques acting on rigid objects and the elasticity of nonrigid objects, a property that governs how such objects can deform. When this physics is done correctly, it is the subject of countless articles in physics and engineering journals; when it is done incorrectly, it is the subject of countless articles in newspapers and legal journals.

Equilibrium Consider these objects: (1) a book resting on a table, (2) a hockey puck sliding with constant velocity across a frictionless surface, (3) the rotating blades of a ceiling fan, and (4) the wheel of a bicycle that is traveling along a straight path at constant speed. For each of these four objects, 327

328

CHAPTE R 12 EQU I LI B R I U M AN D E L ASTICITY :

1. The linear momentum P of its center of mass is constant. : 2. Its angular momentum L about its center of mass, or about any other point, is also constant. We say that such objects are in equilibrium. The two requirements for equilibrium are then :

:

P ! a constant and L ! a constant.

Kanwarjit Singh Boparai/Shutterstock

Figure 12-1 A balancing rock. Although its perch seems precarious, the rock is in static equilibrium.

Our concern in this chapter is with situations in which the constants in Eq. 12-1 are zero; that is, we are concerned largely with objects that are not moving in any way — either in translation or in rotation — in the reference frame from which we observe them. Such objects are in static equilibrium. Of the four objects mentioned near the beginning of this module, only one — the book resting on the table — is in static equilibrium. The balancing rock of Fig. 12-1 is another example of an object that, for the present at least, is in static equilibrium. It shares this property with countless other structures, such as cathedrals, houses, filing cabinets, and taco stands, that remain stationary over time. As we discussed in Module 8-3, if a body returns to a state of static equilibrium after having been displaced from that state by a force, the body is said to be in stable static equilibrium. A marble placed at the bottom of a hemispherical bowl is an example. However, if a small force can displace the body and end the equilibrium, the body is in unstable static equilibrium. A Domino. For example, suppose we balance a domino with the domino’s center of mass vertically above the supporting edge, as in Fig. 12-2a. The torque : about the supporting edge due to the gravitational force Fg on the domino is : zero because the line of action of Fg is through that edge. Thus, the domino is in equilibrium. Of course, even a slight force on it due to some chance distur: bance ends the equilibrium. As the line of action of Fg moves to one side of : the supporting edge (as in Fig. 12-2b), the torque due to Fg increases the rotation of the domino. Therefore, the domino in Fig. 12-2a is in unstable static equilibrium. The domino in Fig. 12-2c is not quite as unstable. To topple this domino, a force would have to rotate it through and then beyond the balance position of Fig. 12-2a, in which the center of mass is above a supporting edge. A slight force will not topple this domino, but a vigorous flick of the finger against the domino certainly will. (If we arrange a chain of such upright dominos, a finger flick against the first can cause the whole chain to fall.) A Block. The child’s square block in Fig. 12-2d is even more stable because its center of mass would have to be moved even farther to get it to pass above a supporting edge. A flick of the finger may not topple the block. (This is why you To tip the block, the center of mass must pass over the supporting edge.

Figure 12-2 (a) A domino balanced on one edge, with its center of mass vertically above : that edge.The gravitational force Fg on the domino is directed through the supporting edge. (b) If the domino is rotated even slightly : from the balanced orientation, then Fg causes a torque that increases the rotation. (c) A domino upright on a narrow side is somewhat more stable than the domino in (a). (d) A square block is even more stable.

(12-1)

com Fg

(a)

Fg

Fg Supporting edge (b)

(c)

B Fg

(d)

12-1 EQU I LI B R I U M

329

never see a chain of toppling square blocks.) The worker in Fig. 12-3 is like both the domino and the square block: Parallel to the beam, his stance is wide and he is stable; perpendicular to the beam, his stance is narrow and he is unstable (and at the mercy of a chance gust of wind). The analysis of static equilibrium is very important in engineering practice. The design engineer must isolate and identify all the external forces and torques that may act on a structure and, by good design and wise choice of materials, ensure that the structure will remain stable under these loads. Such analysis is necessary to ensure, for example, that bridges do not collapse under their traffic and wind loads and that the landing gear of aircraft will function after the shock of rough landings.

The Requirements of Equilibrium The translational motion of a body is governed by Newton’s second law in its linear momentum form, given by Eq. 9-27 as :

: Fnet

dP ! . dt

(12-2) :

If the body is in translational equilibrium — that is, if P is a constant — then : dP/dt ! 0 and we must have :

Fnet ! 0

(12-3)

(balance of forces).

The rotational motion of a body is governed by Newton’s second law in its angular momentum form, given by Eq. 11-29 as :

t:net !

dL . dt

(12-4) :

:

If the body is in rotational equilibrium — that is, if L is a constant — then dL/dt ! 0 and we must have t:net ! 0

(balance of torques).

(12-5)

Thus, the two requirements for a body to be in equilibrium are as follows: 1. The vector sum of all the external forces that act on the body must be zero. 2. The vector sum of all external torques that act on the body, measured about any possible point, must also be zero.

These requirements obviously hold for static equilibrium. They also hold for the : : more general equilibrium in which P and L are constant but not zero. Equations 12-3 and 12-5, as vector equations, are each equivalent to three independent component equations, one for each direction of the coordinate axes: Balance of forces

Balance of torques

Fnet,x ! 0 Fnet,y ! 0 Fnet,z ! 0

tnet,x ! 0 tnet,y ! 0 tnet,z ! 0

(12-6)

The Main Equations. We shall simplify matters by considering only situations in which the forces that act on the body lie in the xy plane. This means that the only torques that can act on the body must tend to cause rotation around an axis parallel to

Robert Brenner/PhotoEdit

Figure 12-3 A construction worker balanced on a steel beam is in static equilibrium but is more stable parallel to the beam than perpendicular to it.

330

CHAPTE R 12 EQU I LI B R I U M AN D E L ASTICITY

the z axis.With this assumption, we eliminate one force equation and two torque equations from Eqs.12-6,leaving Fnet,x ! 0

(balance of forces),

(12-7)

Fnet,y ! 0

(balance of forces),

(12-8)

tnet,z ! 0

(balance of torques).

(12-9)

Here, tnet,z is the net torque that the external forces produce either about the z axis or about any axis parallel to it. A hockey puck sliding at constant velocity over ice satisfies Eqs. 12-7, 12-8, and 12-9 and is thus in equilibrium but not in static equilibrium. For static equilib: rium, the linear momentum P of the puck must be not only constant but also zero; the puck must be at rest on the ice. Thus, there is another requirement for static equilibrium: :

3. The linear momentum P of the body must be zero.

Checkpoint 1 The figure gives six overhead views of a uniform rod on which two or more forces act perpendicularly to the rod. If the magnitudes of the forces are adjusted properly (but kept nonzero), in which situations can the rod be in static equilibrium?

(a)

(b)

(c)

(d)

(e)

(f)

The Center of Gravity The gravitational force on an extended body is the vector sum of the gravitational forces acting on the individual elements (the atoms) of the body. Instead of considering all those individual elements, we can say that :

The gravitational force Fg on a body effectively acts at a single point, called the center of gravity (cog) of the body.

Here the word “effectively” means that if the gravitational forces on the individual : elements were somehow turned off and the gravitational force Fg at the center of gravity were turned on, the net force and the net torque (about any point) acting on the body would not change. : Until now, we have assumed that the gravitational force Fg acts at the center of mass (com) of the body. This is equivalent to assuming that the center of grav: ity is at the center of mass. Recall that, for a body of mass M, the force Fg is equal : : to Mg, where g is the acceleration that the force would produce if the body were

12-1 EQU I LI B R I U M

331

y

to fall freely. In the proof that follows, we show that If : g is the same for all elements of a body, then the body’s center of gravity (cog) is coincident with the body’s center of mass (com).

This is approximately true for everyday objects because g varies only a little along Earth’s surface and decreases in magnitude only slightly with altitude. Thus, for objects like a mouse or a moose, we have been justified in assuming that the gravitational force acts at the center of mass. After the following proof, we shall resume that assumption.

mi Fgi

:

O Moment arm

xi

Line of action x

(a)

y

Proof First, we consider the individual elements of the body. Figure 12-4a shows an extended body, of mass M, and one of its elements, of mass mi. A gravitational : force Fgi acts on each such element and is equal to mi: g i. The subscript on : gi : means gi is the gravitational acceleration at the location of the element i (it can be different for other elements). : For the body in Fig. 12-4a, each force Fgi acting on an element produces a torque ti on the element about the origin O, with a moment arm xi. Using Eq. 1041 (t ! r"F ) as a guide, we can write each torque ti as ti ! xiFgi.

(12-10)

The net torque on all the elements of the body is then tnet ! 'ti ! 'xiFgi.

(12-11)

Next, we consider the body as a whole. Figure 12-4b shows the gravitational : force Fg acting at the body’s center of gravity. This force produces a torque t on the body about O, with moment arm xcog. Again using Eq. 10-41, we can write this torque as t ! xcogFg. (12-12) :

The gravitational force Fg on the body is equal to the sum of the gravitational : forces Fgi on all its elements, so we can substitute 'Fgi for Fg in Eq. 12-12 to write t ! xcog 'Fgi.

(12-13)

: Fg

Now recall that the torque due to force acting at the center of gravity : is equal to the net torque due to all the forces Fgi acting on all the elements of the body. (That is how we defined the center of gravity.) Thus, t in Eq. 12-13 is equal to tnet in Eq. 12-11. Putting those two equations together, we can write xcog 'Fgi ! ' xiFgi. Substituting mi gi for Fgi gives us xcog 'mi gi ! ' xi mi gi.

(12-14)

Now here is a key idea: If the accelerations gi at all the locations of the elements are the same, we can cancel gi from this equation to write xcog 'mi ! ' xi mi.

(12-15)

The sum 'mi of the masses of all the elements is the mass M of the body. Therefore, we can rewrite Eq. 12-15 as xcog !

1 M

' xi mi.

(12-16)

cog Fg xcog O Moment arm (b)

x Line of action

Figure 12-4 (a) An element of mass mi in an : extended body. The gravitational force Fgi on the element has moment arm xi about the origin O of the coordinate system. (b) : The gravitational force Fg on a body is said to act at the center of gravity (cog) of the : body. Here Fg has moment arm xcog about origin O.

332

CHAPTE R 12 EQU I LI B R I U M AN D E L ASTICITY

The right side of this equation gives the coordinate xcom of the body’s center of mass (Eq. 9-4). We now have what we sought to prove. If the acceleration of gravity is the same at all locations of the elements in a body, then the coordinates of the body’s com and cog are identical: xcog ! xcom.

(12-17)

12-2 SOME EXAMPLES OF STATIC EQUILIBRIUM Learning Objectives After reading this module, you should be able to . . .

origin (about which to calculate torques) can simplify the calculations by eliminating one or more unknown forces from the torque equation.

12.05 Apply the force and torque conditions for static equilibrium. 12.06 Identify that a wise choice about the placement of the

Key Ideas ● A rigid body at rest is said to be in static equilibrium. For

such a body, the vector sum of the external forces acting on it is zero: :

F net ! 0

(balance of forces).

If all the forces lie in the xy plane, this vector equation is equivalent to two component equations: Fnet,x ! 0

and

Fnet,y ! 0

● Static equilibrium also implies that the vector sum of the

external torques acting on the body about any point is zero, or t:net ! 0

(balance of torques).

If the forces lie in the xy plane, all torque vectors are parallel to the z axis, and the balance-of-torques equation is equivalent to the single component equation tnet,z ! 0

(balance of forces).

(balance of torques).

Some Examples of Static Equilibrium Here we examine several sample problems involving static equilibrium. In each, we select a system of one or more objects to which we apply the equations of equilibrium (Eqs. 12-7, 12-8, and 12-9). The forces involved in the equilibrium are all in the xy plane, which means that the torques involved are parallel to the z axis. Thus, in applying Eq. 12-9, the balance of torques, we select an axis parallel to the z axis about which to calculate the torques. Although Eq. 12-9 is satisfied for any such choice of axis, you will see that certain choices simplify the application of Eq. 12-9 by eliminating one or more unknown force terms.

Checkpoint 2 The figure gives an overhead view of a uniform rod in static equilibrium. (a) Can you : : find the magnitudes of unknown forces F1 and F2 by balancing the forces? (b) If you : wish to find the magnitude of force F2 by using a balance of torques equation, where : should you place a rotation axis to eliminate F1 from the equation? (c) The magnitude : : of F2 turns out to be 65 N. What then is the magnitude of F1? 20 N

4d

2d

10 N

d

F1

d

30 N

F2

333

12-2 SOM E EXAM PLES OF STATIC EQU I LI B R I U M

System

Sample Problem 12.01 Balancing a horizontal beam L

L 4

In Fig. 12-5a, a uniform beam, of length L and mass m ! 1.8 kg, is at rest on two scales. A uniform block, with mass M ! 2.7 kg, is at rest on the beam, with its center a distance L/4 from the beam’s left end. What do the scales read?

Block

M

m

Beam

Scale

Scale

KEY IDEAS The first steps in the solution of any problem about static equilibrium are these: Clearly define the system to be analyzed and then draw a free-body diagram of it, indicating all the forces on the system. Here, let us choose the system as the beam and block taken together.Then the forces on the system are shown in the free-body diagram of Fig. 12-5b. (Choosing the system takes experience, and often there can be more than one good choice.) Because the system is in static equilibrium, we can apply the balance of forces equations (Eqs. 12-7 and 12-8) and the balance of torques equation (Eq. 12-9) to it. Calculations: The normal forces on the beam from the scales : : are Fl on the left and Fr on the right. The scale readings that we want are equal to the magnitudes of those forces. The gravita: tional force Fg,beam on the beam acts at the beam’s center of : mass and is equal to mg:. Similarly, the gravitational force Fg,block on the block acts at the block’s center of mass and is equal to : . However, to simplify Fig. 12-5b, the block is repreMg sented by a dot within the boundary of the beam and vector : Fg,block is drawn with its tail on that dot. (This shift of the : vector Fg,block along its line of action does not alter the : torque due to Fg,block about any axis perpendicular to the figure.) The forces have no x components, so Eq. 12-7 (Fnet, x ! 0) provides no information. For the y components, Eq. 12-8 (Fnet,y ! 0) gives us Fl # Fr % Mg % mg ! 0.

(12-18)

This equation contains two unknowns, the forces Fl and F r, so we also need to use Eq. 12-9, the balance of torques equation. We can apply it to any rotation axis perpendicular to the plane of Fig. 12-5. Let us choose a rotation axis through the left end of the beam. We shall also use our general rule for assigning signs to torques: If a torque would cause an initially stationary body to rotate clockwise about the rotation axis, the torque is negative. If the rotation would be counterclockwise, the torque is positive. Finally, we shall write the torques in the form r"F, : : where the moment arm r" is 0 for Fl, L/4 for Mg , L/2 for mg:, : and L for Fr. We now can write the balancing equation (tnet,z ! 0) as (0)(Fl) % (L/4)(Mg) % (L/2)(mg) # (L)(Fr) ! 0,

(a) y Fl

The vertical forces balance but that is not enough. L 2

Fr

L 4

x Block

Beam Fg,beam = mg

We must also balance torques, with a wise choice of rotation axis.

Fg,block = Mg (b)

Figure 12-5 (a) A beam of mass m supports a block of mass M. (b) A free-body diagram, showing the forces that act on the system beam # block.

which gives us Fr ! 14 Mg # 12 mg ! 14(2.7 kg)(9.8 m/s2) # 12(1.8 kg)(9.8 m/s2) ! 15.44 N % 15 N.

(Answer)

Now, solving Eq. 12-18 for Fl and substituting this result, we find Fl ! (M # m)g % Fr ! (2.7 kg # 1.8 kg)(9.8 m/s2) % 15.44 N ! 28.66 N % 29 N.

(Answer)

Notice the strategy in the solution: When we wrote an equation for the balance of force components, we got stuck with two unknowns. If we had written an equation for the balance of torques around some arbitrary axis, we would have again gotten stuck with those two unknowns. However, because we chose the axis to pass through the point of application of one of : the unknown forces, here Fl, we did not get stuck. Our choice neatly eliminated that force from the torque equation, allowing us to solve for the other unknown force magnitude Fr.Then we returned to the equation for the balance of force components to find the remaining unknown force magnitude.

Additional examples, video, and practice available at WileyPLUS

334

CHAPTE R 12 EQU I LI B R I U M AN D E L ASTICITY

Sample Problem 12.02 Balancing a leaning boom Figure 12-6a shows a safe (mass M ! 430 kg) hanging by a rope (negligible mass) from a boom (a ! 1.9 m and b ! 2.5 m) that consists of a uniform hinged beam (m ! 85 kg) and horizontal cable (negligible mass). (a) What is the tension Tc in the cable? In other words, what is : the magnitude of the force Tc on the beam from the cable? KEY IDEAS The system here is the beam alone, and the forces on it are shown in the free-body diagram of Fig. 12-6b. The force : from the cable is Tc. The gravitational force on the beam acts at the beam’s center of mass (at the beam’s center) and is represented by its equivalent mg:. The vertical component : of the force on the beam from the hinge is Fv, and the hori: zontal component of the force from the hinge is Fh. The : force from the rope supporting the safe is Tr. Because beam, : rope, and safe are stationary, the magnitude of Tr is equal to the weight of the safe: Tr ! Mg. We place the origin O of an xy coordinate system at the hinge. Because the system is in static equilibrium, the balancing equations apply to it. Calculations: Let us start with Eq. 12-9 (tnet,z ! 0). Note : that we are asked for the magnitude of force Tc and not of : : forces Fh and Fv acting at the hinge, at point O. To eliminate : : Fh and Fv from the torque calculation, we should calculate torques about an axis that is perpendicular to the figure at : : point O. Then Fh and Fv will have moment arms of zero. The : : lines of action for Tc, Tr, and mg: are dashed in Fig. 12-6b. The corresponding moment arms are a, b, and b/2. Writing torques in the form of r"F and using our rule about signs for torques, the balancing equation tnet,z ! 0 becomes (a)(Tc) % (b)(Tr) % ( 12b)(mg) ! 0.

(12-19)

Substituting Mg for Tr and solving for Tc, we find that Tc !

gb(M # a

Figure 12-6 (a) A heavy safe is hung from a boom consisting of a horizontal steel cable and a uniform beam. (b) A free-body diagram for the beam.

b Cable θ

(b) Find the magnitude F of the net force on the beam from the hinge. KEY IDEA Now we want the horizontal component Fh and vertical component Fv so that we can combine them to get the

Rope

Hinge

(a)

M

y

Tc θ

Tr Beam mg

Here is the wise choice of rotation axis.

Fv O

(b)

x

Fh

magnitude F of the net force. Because we know Tc, we apply the force balancing equations to the beam. Calculations: For the horizontal balance, we can rewrite Fnet, x ! 0 as Fh % Tc ! 0, (12-20) and so

Fh ! Tc ! 6093 N.

For the vertical balance, we write Fnet,y ! 0 as

1 2 m)

(9.8 m/s2)(2.5 m)(430 kg # 85/2 kg) ! 1.9 m ! 6093 N % 6100 N. (Answer)

Beam com

m

a

Fv % mg % Tr ! 0. Substituting Mg for Tr and solving for Fv, we find that Fv ! (m # M)g ! (85 kg # 430 kg)(9.8 m/s2) ! 5047 N. From the Pythagorean theorem, we now have F ! 2F h2 # F v2

! 2(6093 N)2 # (5047 N)2 % 7900 N.

(Answer)

Note that F is substantially greater than either the combined weights of the safe and the beam, 5000 N, or the tension in the horizontal wire, 6100 N.

Additional examples, video, and practice available at WileyPLUS

12-2 SOM E EXAM PLES OF STATIC EQU I LI B R I U M

335

Sample Problem 12.03 Balancing a leaning ladder In Fig. 12-7a, a ladder of length L ! 12 m and mass m ! 45 kg leans against a slick wall (that is, there is no friction between the ladder and the wall). The ladder’s upper end is at height h ! 9.3 m above the pavement on which the lower end is supported (the pavement is not frictionless). The ladder’s center of mass is L/3 from the lower end, along the length of the ladder. A firefighter of mass M ! 72 kg climbs the ladder until her center of mass is L/2 from the lower end. What then are the magnitudes of the forces on the ladder from the wall and the pavement?

line extending through the force vector), so that its tail is on the dot. (The shift does not alter a torque due to Mg: about any axis perpendicular to the figure.Thus, the shift does not affect the torque balancing equation that we shall be using.) The only force on the ladder from the wall is the hori: zontal force Fw (there cannot be a frictional force along a frictionless wall, so there is no vertical force on the ladder : from the wall). The force Fp on the ladder from the pave: ment has two components: a horizontal component Fpx that : is a static frictional force and a vertical component Fpy that is a normal force. To apply the balancing equations, let’s start with the torque balancing of Eq. 12-9 (tnet,z ! 0). To choose an axis about which to calculate the torques, note that we have : : unknown forces (Fw and Fp) at the two ends of the ladder. To : eliminate, say, Fp from the calculation, we place the axis at point O, perpendicular to the figure (Fig. 12-7b). We also place the origin of an xy coordinate system at O. We can find torques about O with any of Eqs. 10-39 through 10-41, but Eq. 10-41 (t ! r"F) is easiest to use here. Making a wise choice about the placement of the origin can make our torque calculation much easier. : To find the moment arm r" of the horizontal force F w from the wall, we draw a line of action through that vector

KEY IDEAS First, we choose our system as being the firefighter and ladder, together, and then we draw the free-body diagram of Fig. 12-7b to show the forces acting on the system. Because the system is in static equilibrium, the balancing equations for both forces and torques (Eqs. 12-7 through 12-9) can be applied to it. Calculations: In Fig. 12-7b, the firefighter is represented with a dot within the boundary of the ladder. The gravitational force on her is represented with its equivalent expression Mg:, and that vector has been shifted along its line of action (the

Frictionless

A

y System

Here are all the forces.

Fw

L Firefighter com

h

Firefighter

Ladder com

Fpy Ladder

Mg mg Fpx

a (a)

(b)

O

x

a/3 a/2

Figure 12-7 (a) A firefighter climbs halfway up a ladder that is leaning against a frictionless wall. The pavement beneath the ladder is not frictionless. (b) A free-body diagram, showing the forces that act on the firefighter # ladder system. The origin O of a coordinate system is : : placed at the point of application of the unknown force Fp (whose vector components Fpx and : Fpy are shown). (Figure 12-7 continues on following page.)

336

CHAPTE R 12 EQU I LI B R I U M AN D E L ASTICITY

y

y

y

Fw

Fpy

y

This moment arm is perpendicular to the line of action. h Mg mg

(c)

Fpx

O

x

O

O

x

Here too.

Choosing the rotation axis here eliminates the torques due to these forces.

Here too.

y

x

a/3 y

Fw

These vertical forces balance.

These horizontal forces balance. Figure 12-7 (Continued from previous page) (c) Calculating the torques. (d) Balancing the forces. In WileyPLUS, this figure is available as an animation with voiceover.

Fpy Mg mg

Fpx

(d )

(it is the horizontal dashed line shown in Fig. 12-7c). Then r" is the perpendicular distance between O and the line of action. In Fig. 12-7c, r" extends along the y axis and is equal to the height h. We similarly draw lines of action for the gravita: tional force vectors Mg: and mg and see that their moment arms extend along the x axis. For the distance a shown in Fig. 12-7a, the moment arms are a/2 (the firefighter is halfway up the ladder) and a/3 (the ladder’s center of mass is one-third:of the way up the ladder), respectively.The moment arms for Fpx : and Fpy are zero because the forces act at the origin. Now, with torques written in the form r"F, the balancing equation tnet,z ! 0 becomes %(h)(Fw) # (a/2)(Mg) # (a/3)(mg) # (0)(Fpx) # (0)(Fpy) ! 0. (12-21) (A positive torque corresponds to counterclockwise rotation and a negative torque corresponds to clockwise rotation.) Using the Pythagorean theorem for the right triangle made by the ladder in Fig. 11-7a, we find that a ! 1L2 % h2 ! 7.58 m.

O

x

a/2

O

x

O

x

Then Eq. 12-21 gives us Fw ! !

ga(M/2 # m/3) h (9.8 m/s2)(7.58 m)(72/2 kg # 45/3 kg) 9.3 m

! 407 N % 410 N.

(Answer)

Now we need to use the force balancing equations and Fig. 12-7d. The equation Fnet, x ! 0 gives us Fw % Fpx ! 0, so

Fpx ! Fw ! 410 N.

(Answer)

The equation Fnet,y ! 0 gives us Fpy % Mg % mg ! 0, so

Fpy ! (M # m)g ! (72 kg # 45 kg)(9.8 m/s2) ! 1146.6 N % 1100 N.

Additional examples, video, and practice available at WileyPLUS

(Answer)

12-2 SOM E EXAM PLES OF STATIC EQU I LI B R I U M

337

Sample Problem 12.04 Balancing the leaning Tower of Pisa Let’s assume that the Tower of Pisa is a uniform hollow cylinder of radius R ! 9.8 m and height h ! 60 m. The center of mass is located at height h/2, along the cylinder’s central axis. In Fig. 12-8a, the cylinder is upright. In Fig. 12-8b, it leans rightward (toward the tower’s southern wall) by u ! 5.5(, which shifts the com by a distance d. Let’s assume that the ground exerts only two forces on : the tower. A normal force FNL acts on the left (northern) : wall, and a normal force FNR acts on the right (southern) wall. By what percent does the magnitude FNR increase because of the leaning?

θ __ 1 2h

O

O Ground (a)

(b)

com

KEY IDEA

mg

Because the tower is still standing, it is in equilibrium and thus the sum of torques calculated around any point must be zero. Calculations: Because we want to calculate FNR on the right side and do not know or want FNL on the left side, we use a pivot point on the left side to calculate torques. The forces on the upright tower are represented in Fig. 12-8c. The gravitational force mg:, taken to act at the com, has a vertical line of action and a moment arm of R (the perpendicular distance from the pivot to the line of action). About the pivot, the torque associated with this force would tend to create clockwise rotation and thus is nega: tive. The normal force FNR on the southern wall also has a vertical line of action, and its moment arm is 2R. About the pivot, the torque associated with this force would tend to create counterclockwise rotation and thus is positive. We now can write the torque-balancing equation (tnet,z ! 0) as %(R)(mg) # (2R)(FNR) ! 0, which yields

d

com h

FNR ! 12 mg.

We should have been able to guess this result: With the center of mass located on the central axis (the cylinder’s line of symmetry), the right side supports half the cylinder’s weight. In Fig. 12-8b, the com is shifted rightward by distance d ! 12h tan u. The only change in the balance of torques equation is that the moment arm for the gravitational force is now R # d and the normal force at the right has a new magnitude F.NR (Fig. 12-8d). Thus, we write %(R # d)(mg) # (2R)(F.NR ) ! 0,

mg

FNR

O

FNR ‘

O

R

R+d

2R

2R (d)

(c)

Figure 12-8 A cylinder modeling the Tower of Pisa: (a) upright and (b) leaning, with the center of mass shifted rightward. The forces and moment arms to find torques about a pivot at point O for the cylinder (c) upright and (d) leaning.

which gives us F.NR !

(R # d) mg. 2R

Dividing this new result for the normal force at the right by the original result and then substituting for d, we obtain F.NR R#d d 0.5h tan u ! !1# !1# . FNR R R R Substituting the values of h ! 60 m, R ! 9.8 m, and u ! 5.5( leads to F.NR ! 1.29. FNR Thus, our simple model predicts that, although the tilt is modest, the normal force on the tower’s southern wall has increased by about 30%. One danger to the tower is that the force may cause the southern wall to buckle and explode outward. The cause of the leaning is the compressible soil beneath the tower, which worsened with each rainfall. Recently engineers have stabilized the tower and partially reversed the leaning by installing a drainage system.

Additional examples, video, and practice available at WileyPLUS

338

CHAPTE R 12 EQU I LI B R I U M AN D E L ASTICITY

12-3 ELASTICITY Learning Objectives After reading this module, you should be able to . . .

12.07 Explain what an indeterminate situation is. 12.08 For tension and compression, apply the equation that relates stress to strain and Young’s modulus. 12.09 Distinguish between yield strength and ultimate strength.

12.10 For shearing, apply the equation that relates stress to strain and the shear modulus. 12.11 For hydraulic stress, apply the equation that relates fluid pressure to strain and the bulk modulus.

Key Ideas ● Three elastic moduli are used to describe the elastic behav-

ior (deformations) of objects as they respond to forces that act on them. The strain (fractional change in length) is linearly related to the applied stress (force per unit area) by the proper modulus, according to the general stress–strain relation stress ! modulus $ strain. ● When an object is under tension or compression, the

stress–strain relation is written as

● When an object is under a shearing stress, the stress–strain

relation is written as F 'x !G , A L where 'x/L is the shearing strain of the object, 'x is the displacement of one end of the object in the direction of the : applied force F , and G is the shear modulus of the object. The stress is F/A. ● When an object undergoes hydraulic compression due to a

F 'L !E , A L where 'L/L is the tensile or compressive strain of the object, : F is the magnitude of the applied force F causing the strain, : A is the cross-sectional area over which F is applied (perpendicular to A), and E is the Young’s modulus for the object. The stress is F/A.

stress exerted by a surrounding fluid, the stress–strain relation is written as 'V p!B , V where p is the pressure (hydraulic stress) on the object due to the fluid, 'V/V (the strain) is the absolute value of the fractional change in the object’s volume due to that pressure, and B is the bulk modulus of the object.

Indeterminate Structures For the problems of this chapter, we have only three independent equations at our disposal, usually two balance of forces equations and one balance-of-torques equation about a given rotation axis. Thus, if a problem has more than three unknowns, we cannot solve it. Consider an unsymmetrically loaded car. What are the forces—all different— on the four tires? Again, we cannot find them because we have only three independent equations. Similarly, we can solve an equilibrium problem for a table with three legs but not for one with four legs. Problems like these, in which there are more unknowns than equations, are called indeterminate. Yet solutions to indeterminate problems exist in the real world. If you rest the tires of the car on four platform scales, each scale will register a definite reading, the sum of the readings being the weight of the car. What is eluding us in our efforts to find the individual forces by solving equations? The problem is that we have assumed — without making a great point of it — that the bodies to which we apply the equations of static equilibrium are perfectly rigid. By this we mean that they do not deform when forces are applied to them. Strictly, there are no such bodies. The tires of the car, for example, deform easily under load until the car settles into a position of static equilibrium. We have all had experience with a wobbly restaurant table, which we usually level by putting folded paper under one of the legs. If a big enough elephant sat on such a table, however, you may be sure that if the table did not collapse, it

339

12-3 E L ASTICITY

would deform just like the tires of a car. Its legs would all touch the floor, the forces acting upward on the table legs would all assume definite (and different) values as in Fig. 12-9, and the table would no longer wobble. Of course, we (and the elephant) would be thrown out onto the street but, in principle, how do we find the individual values of those forces acting on the legs in this or similar situations where there is deformation? To solve such indeterminate equilibrium problems, we must supplement equilibrium equations with some knowledge of elasticity, the branch of physics and engineering that describes how real bodies deform when forces are applied to them.

com

Checkpoint 3 A horizontal uniform bar of weight 10 N is to hang from a ceiling by two wires that : : exert upward forces F1 and F2 on the bar. The figure shows four arrangements for the wires. Which arrangements, if any, are indeterminate (so that we cannot solve for nu: : merical values of F1 and F2)? F1

F2

F1

d

10 N

d

d/2

10 N (c)

F2

10 N

(a)

F1

d

F4

F2 F1

Fg

F3

Figure 12-9 The table is an indeterminate structure. The four forces on the table legs differ from one another in magnitude and cannot be found from the laws of static equilibrium alone.

(b)

F2

F1

F2

10 N (d)

Elasticity When a large number of atoms come together to form a metallic solid, such as an iron nail, they settle into equilibrium positions in a three-dimensional lattice, a repetitive arrangement in which each atom is a well-defined equilibrium distance from its nearest neighbors. The atoms are held together by interatomic forces that are modeled as tiny springs in Fig. 12-10. The lattice is remarkably rigid, which is another way of saying that the “interatomic springs” are extremely stiff. It is for this reason that we perceive many ordinary objects, such as metal ladders, tables, and spoons, as perfectly rigid. Of course, some ordinary objects, such as garden hoses or rubber gloves, do not strike us as rigid at all. The atoms that make up these objects do not form a rigid lattice like that of Fig. 12-10 but are aligned in long, flexible molecular chains, each chain being only loosely bound to its neighbors. All real “rigid” bodies are to some extent elastic, which means that we can change their dimensions slightly by pulling, pushing, twisting, or compressing them. To get a feeling for the orders of magnitude involved, consider a vertical steel rod 1 m long and 1 cm in diameter attached to a factory ceiling. If you hang a subcompact car from the free end of such a rod, the rod will stretch but only by about 0.5 mm, or 0.05%. Furthermore, the rod will return to its original length when the car is removed. If you hang two cars from the rod, the rod will be permanently stretched and will not recover its original length when you remove the load. If you hang three cars from the rod, the rod will break. Just before rupture, the elongation of the

Figure 12-10 The atoms of a metallic solid are distributed on a repetitive three-dimensional lattice.The springs represent interatomic forces.

340

CHAPTE R 12 EQU I LI B R I U M AN D E L ASTICITY

F

∆x

V F ∆V

L

L + ∆L

L

(b)

(a) F

L

Figure 12-12 A test specimen used to determine a stress–strain curve such as that of Fig. 12-13.The change 'L that occurs in a certain length L is measured in a tensile stress–strain test.

F

(c)

Figure 12-11 (a) A cylinder subject to tensile stress stretches by an amount 'L. (b) A cylinder subject to shearing stress deforms by an amount 'x, somewhat like a pack of playing cards would. (c) A solid sphere subject to uniform hydraulic stress from a fluid shrinks in volume by an amount 'V. All the deformations shown are greatly exaggerated.

rod will be less than 0.2%. Although deformations of this size seem small, they are important in engineering practice. (Whether a wing under load will stay on an airplane is obviously important.) Three Ways. Figure 12-11 shows three ways in which a solid might change its dimensions when forces act on it. In Fig. 12-11a, a cylinder is stretched. In Fig. 12-11b, a cylinder is deformed by a force perpendicular to its long axis, much as we might deform a pack of cards or a book. In Fig. 12-11c, a solid object placed in a fluid under high pressure is compressed uniformly on all sides. What the three deformation types have in common is that a stress, or deforming force per unit area, produces a strain, or unit deformation. In Fig. 12-11, tensile stress (associated with stretching) is illustrated in (a), shearing stress in (b), and hydraulic stress in (c). The stresses and the strains take different forms in the three situations of Fig. 12-11, but — over the range of engineering usefulness — stress and strain are proportional to each other. The constant of proportionality is called a modulus of elasticity, so that stress ! modulus $ strain.

Ultimate strength

Stress (F/A)

Yield strength

0

Rupture

Range of permanent deformation Linear (elastic) range

Strain (∆L/L)

Figure 12-13 A stress – strain curve for a steel test specimen such as that of Fig. 12-12. The specimen deforms permanently when the stress is equal to the yield strength of the specimen’s material. It ruptures when the stress is equal to the ultimate strength of the material.

(12-22)

In a standard test of tensile properties, the tensile stress on a test cylinder (like that in Fig. 12-12) is slowly increased from zero to the point at which the cylinder fractures, and the strain is carefully measured and plotted. The result is a graph of stress versus strain like that in Fig. 12-13. For a substantial range of applied stresses, the stress – strain relation is linear, and the specimen recovers its original dimensions when the stress is removed; it is here that Eq. 12-22 applies. If the stress is increased beyond the yield strength Sy of the specimen, the specimen becomes permanently deformed. If the stress continues to increase, the specimen eventually ruptures, at a stress called the ultimate strength Su.

Tension and Compression For simple tension or compression, the stress on the object is defined as F/A, where F is the magnitude of the force applied perpendicularly to an area A on the object. The strain, or unit deformation, is then the dimensionless quantity 'L/L, the fractional (or sometimes percentage) change in a length of the specimen. If the specimen is a long rod and the stress does not exceed the yield strength, then not only the entire rod but also every section of it experiences the same strain when a given stress is applied. Because the strain is dimensionless, the modulus in Eq. 12-22 has the same dimensions as the stress — namely, force per unit area.

12-3 E L ASTICITY

341

The modulus for tensile and compressive stresses is called the Young’s modulus and is represented in engineering practice by the symbol E. Equation 12-22 becomes 'L F (12-23) !E . A L The strain 'L/L in a specimen can often be measured conveniently with a strain gage (Fig. 12-14), which can be attached directly to operating machinery with an adhesive. Its electrical properties are dependent on the strain it undergoes. Although the Young’s modulus for an object may be almost the same for tension and compression, the object’s ultimate strength may well be different for the two types of stress. Concrete, for example, is very strong in compression but is so weak in tension that it is almost never used in that manner.Table 12-1 shows the Young’s modulus and other elastic properties for some materials of engineering interest.

Shearing In the case of shearing, the stress is also a force per unit area, but the force vector lies in the plane of the area rather than perpendicular to it. The strain is the dimensionless ratio 'x/L, with the quantities defined as shown in Fig. 12-11b. The corresponding modulus, which is given the symbol G in engineering practice, is called the shear modulus. For shearing, Eq. 12-22 is written as 'x F (12-24) !G . A L Shearing occurs in rotating shafts under load and in bone fractures due to bending.

Hydraulic Stress In Fig. 12-11c, the stress is the fluid pressure p on the object, and, as you will see in Chapter 14, pressure is a force per unit area. The strain is 'V/V, where V is the original volume of the specimen and 'V is the absolute value of the change in volume. The corresponding modulus, with symbol B, is called the bulk modulus of the material. The object is said to be under hydraulic compression, and the pressure can be called the hydraulic stress. For this situation, we write Eq. 12-22 as 'V (12-25) p!B . V The bulk modulus is 2.2 $ 109 N/m2 for water and 1.6 $ 1011 N/m2 for steel. The pressure at the bottom of the Pacific Ocean, at its average depth of about 4000 m, is 4.0 $ 107 N/m2. The fractional compression 'V/V of a volume of water due to this pressure is 1.8%; that for a steel object is only about 0.025%. In general, solids — with their rigid atomic lattices — are less compressible than liquids, in which the atoms or molecules are less tightly coupled to their neighbors. Table 12-1 Some Elastic Properties of Selected Materials of Engineering Interest

Material Steela Aluminum Glass Concretec Woodd Bone Polystyrene

Density r (kg/m3)

Young’s Modulus E (109 N/m2)

Ultimate Strength Su (106 N/m2)

Yield Strength Sy (106 N/m2)

7860 2710 2190 2320 525 1900 1050

200 70 65 30 13 9b 3

400 110 50b 40b 50b 170b 48

250 95 — — — — —

a

b

c

d

Structural steel (ASTM-A36). High strength

In compression. Douglas fir.

Courtesy Micro Measurements, a Division of Vishay Precision Group, Raleigh, NC

Figure 12-14 A strain gage of overall dimensions 9.8 mm by 4.6 mm. The gage is fastened with adhesive to the object whose strain is to be measured; it experiences the same strain as the object. The electrical resistance of the gage varies with the strain, permitting strains up to 3% to be measured.

342

CHAPTE R 12 EQU I LI B R I U M AN D E L ASTICITY

Sample Problem 12.05 Stress and strain of elongated rod One end of a steel rod of radius R ! 9.5 mm and length L ! 81 cm is held in a vise. A force of magnitude F ! 62 kN is then applied perpendicularly to the end face (uniformly across the area) at the other end, pulling directly away from the vise. What are the stress on the rod and the elongation 'L and strain of the rod? KEY IDEAS (1) Because the force is perpendicular to the end face and uniform, the stress is the ratio of the magnitude F of the force to the area A. The ratio is the left side of Eq. 12 -23. (2) The elongation 'L is related to the stress and Young’s modulus E by Eq. 12 -23 (F/A ! E 'L/L). (3) Strain is the ratio of the elongation to the initial length L.

stress !

F F 6.2 $ 104 N ! ! A pR2 (p)(9.5 $ 10 %3 m)2

! 2.2 $ 108 N/m2.

(Answer)

The yield strength for structural steel is 2.5 $ 108 N/m2, so this rod is dangerously close to its yield strength. We find the value of Young’s modulus for steel in Table 12 -1. Then from Eq. 12 -23 we find the elongation: 'L !

(2.2 $ 108 N/m2)(0.81 m) (F/A)L ! E 2.0 $ 1011 N/m2

! 8.9 $ 10%4 m ! 0.89 mm. For the strain, we have

(Answer)

8.9 $ 10 %4 m 'L ! L 0.81 m ! 1.1 $ 10%3 ! 0.11%.

Calculations: To find the stress, we write

(Answer)

Sample Problem 12.06 Balancing a wobbly table A table has three legs that are 1.00 m in length and a fourth leg that is longer by d ! 0.50 mm, so that the table wobbles slightly. A steel cylinder with mass M ! 290 kg is placed on the table (which has a mass much less than M) so that all four legs are compressed but unbuckled and the table is level but no longer wobbles. The legs are wooden cylinders with cross-sectional area A ! 1.0 cm2; Young’s modulus is E ! 1.3 $ 1010 N/m2. What are the magnitudes of the forces on the legs from the floor? KEY IDEAS We take the table plus steel cylinder as our system. The situation is like that in Fig. 12-9, except now we have a steel cylinder on the table. If the tabletop remains level, the legs must be compressed in the following ways: Each of the short legs must be compressed by the same amount (call it 'L3) and thus by the same force of magnitude F3. The single long leg must be compressed by a larger amount 'L4 and thus by a force with a larger magnitude F4. In other words, for a level tabletop, we must have 'L4 ! 'L3 # d.

(12-26)

From Eq. 12-23, we can relate a change in length to the force causing the change with 'L ! FL/AE, where L is the original length of a leg.We can use this relation to replace 'L4 and 'L3 in Eq. 12-26. However, note that we can approximate the original length L as being the same for all four legs. Calculations: Making those replacements and that approxi-

mation gives us

FL F4L ! 3 # d. (12-27) AE AE We cannot solve this equation because it has two unknowns, F4 and F3. To get a second equation containing F4 and F3, we can use a vertical y axis and then write the balance of vertical forces (Fnet,y ! 0) as 3F3 # F4 % Mg ! 0, (12-28) where Mg is equal to the magnitude of the gravitational force : on the system. (Three legs have force F3 on them.) To solve the simultaneous equations 12-27 and 12-28 for, say, F3, we first use Eq. 12-28 to find that F4 ! Mg % 3F3. Substituting that into Eq. 12-27 then yields, after some algebra, Mg dAE F3 ! % 4 4L !

(290 kg)(9.8 m/s2) 4 (5.0 $ 10 %4 m)(10 %4 m2)(1.3 $ 1010 N/m2) % (4)(1.00 m)

! 548 N % 5.5 $ 102 N.

(Answer)

From Eq. 12-28 we then find F4 ! Mg % 3F3 ! (290 kg)(9.8 m/s2) % 3(548 N) % 1.2 kN.

(Answer)

You can show that the three short legs are each compressed by 0.42 mm and the single long leg by 0.92 mm.

Additional examples, video, and practice available at WileyPLUS

QU ESTIONS

343

Review & Summary Static Equilibrium A rigid body at rest is said to be in static equilibrium. For such a body, the vector sum of the external forces acting on it is zero: :

F net ! 0

and Fnet,y ! 0

(12-7, 12-8)

(balance of forces).

Static equilibrium also implies that the vector sum of the external torques acting on the body about any point is zero, or t:net ! 0

(12-5)

(balance of torques).

If the forces lie in the xy plane, all torque vectors are parallel to the z axis, and Eq. 12-5 is equivalent to the single component equation tnet,z ! 0

or compression, Eq. 12-22 is written as F 'L !E , A L

(12-3)

(balance of forces).

If all the forces lie in the xy plane, this vector equation is equivalent to two component equations: Fnet,x ! 0

Tension and Compression When an object is under tension

where 'L/L is the tensile or compressive strain of the object, F is : the magnitude of the applied force F causing the strain, A is the : cross-sectional area over which F is applied (perpendicular to A, as in Fig. 12-11a), and E is the Young’s modulus for the object. The stress is F/A.

Shearing When an object is under a shearing stress, Eq. 12-22 is written as F 'x !G , A L

(12-9)

(balance of torques).

Center of Gravity The gravitational force acts individually on each element of a body. The net effect of all individual actions may : be found by imagining an equivalent total gravitational force Fg acting at the center of gravity. If the gravitational acceleration g: is the same for all the elements of the body, the center of gravity is at the center of mass.

Elastic Moduli Three elastic moduli are used to describe the elastic behavior (deformations) of objects as they respond to forces that act on them. The strain (fractional change in length) is linearly related to the applied stress (force per unit area) by the proper modulus, according to the general relation stress ! modulus $ strain.

(12-22)

(12-23)

(12-24)

where 'x/L is the shearing strain of the object, 'x is the displacement of one end of the object in the direction of the ap: plied force F (as in Fig. 12-11b), and G is the shear modulus of the object. The stress is F/A.

Hydraulic Stress When an object undergoes hydraulic compression due to a stress exerted by a surrounding fluid, Eq. 12-22 is written as 'V p!B (12-25) , V where p is the pressure (hydraulic stress) on the object due to the fluid, 'V/V (the strain) is the absolute value of the fractional change in the object’s volume due to that pressure, and B is the bulk modulus of the object.

Questions 1 Figure 12-15 shows three situations in which the same horizontal rod is supported by a hinge on a wall at one end and a cord at its other end. Without written calculation, rank the situations according to the magnitudes of (a) the force on the rod from the cord, (b) the vertical force on the rod from the hinge, and (c) the horizontal force on the rod from the hinge, greatest first.

50°

compared to that of the safe.(a) Rank the positions according to the force on post A due to the safe, greatest compression first, greatest tension last, and indicate where, if anywhere, the force is zero. (b) Rank them according to the force on post B. 3 Figure 12-17 shows four overhead views of rotating uniform disks that are sliding across a frictionless floor. Three forces, of magnitude F, 2F, or 3F, act on each disk, either at the rim, at the center, or halfway between rim and center. The force vectors rotate along with the disks, and, in the “snapshots” of Fig. 12-17, point left or right. Which disks are in equilibrium?

50° F

(1)

(2)

1 A

(a) 2

3

4

5

B

Figure 12-16 Question 2.

6

F (b)

F

F

2F

3F

2F

(3)

Figure 12-15 Question 1. 2 In Fig. 12-16, a rigid beam is attached to two posts that are fastened to a floor. A small but heavy safe is placed at the six positions indicated, in turn. Assume that the mass of the beam is negligible

F

F

2F F (c)

2F (d)

Figure 12-17 Question 3. 4 A ladder leans against a frictionless wall but is prevented from falling because of friction between it and the ground. Suppose you shift the base of the ladder toward the wall. Determine whether the following become larger, smaller, or stay the same (in

344

CHAPTE R 12 EQU I LI B R I U M AN D E L ASTICITY

magnitude): (a) the normal force on the ladder from the ground, (b) the force on the ladder from the wall, (c) the static frictional force on the ladder from the ground, and (d) the maximum value fs,max of the static frictional force. 5 Figure 12-18 shows a mobile of toy penguins hanging from a ceiling. Each crossbar is horizontal, has negligible mass, and extends three times as far to the right of the wire supporting it as to the left. Penguin 1 has mass m1 ! 48 kg. What are the masses of (a) penguin 2, (b) penguin 3, and (c) penguin 4?

1

2 3

4

Figure 12-18 Question 5. 6 Figure 12-19 shows an overhead C view of a uniform stick on which four forces act. Suppose we choose a rotation axis through point O, calO A B culate the torques about that axis Figure 12-19 Question 6. due to the forces, and find that these torques balance. Will the torques balance if, instead, the rotation axis is chosen to be at (a) point A (on the stick), (b) point B (on line with the stick), or (c) point C (off to one side of the stick)? (d) Suppose, instead, that we find that the torques about point O do not balance. Is there another point about which the torques will balance? 7 In Fig. 12-20, a stationary 5 kg rod AC is held against a wall by a rope and friction between rod D and wall. The uniform rod is 1 m long, and angle u ! 30(. (a) If you are to find the magnitude of : θ the force T on the rod from the rope with a single equation, at what labeled point should a rota- A B C tion axis be placed? With that choice of axis and counterclockwise torques positive, what is the Figure 12-20 sign of (b) the torque tw due to the rod’s weight Question 7. and (c) the torque tr due to the pull on the rod by the rope? (d) Is the magnitude of tr greater than, less than, or equal to the magnitude of tw? 8 Three piñatas hang from the (stationary) assembly of massless pulleys and cords seen in Fig. 12-21. One long cord runs from the ceiling at the right to the lower pulley at the left, looping halfway around all the pulleys. Several shorter cords suspend pulleys from the ceiling or piñatas from the pulleys. The weights (in newtons) of two piñatas are given. (a) What is the weight of the third piñata? (Hint: A cord that loops halfway around a pulley pulls on the pulley with a net force that is twice the tension in the cord.) (b)

What is the tension in the short cord labeled with T ?

Fa

9 In Fig. 12-22, a vertical rod is hinged at its lower end and attached to a cable at its upper end.A horizon: tal force F a is to be applied to the rod as shown. If the point at which the force is applied is moved up the rod, does the tension in the cable increase, decrease, or remain the same?

Figure 12-22 Question 9.

10 Figure 12-23 shows a horizonA B tal block that is suspended by two wires, A and B, which are identical except for their original lengths. The com center of mass of the block is closer to wire B than to wire A. (a) Figure 12-23 Question 10. Measuring torques about the block’s center of mass, state whether the magnitude of the torque due to wire A is greater than, less than, or equal to the magnitude of the torque due to wire B. (b) Which wire exerts more force on the block? (c) If the wires are now equal in length, which one was originally shorter (before the block was suspended)? 11 The table gives the initial lengths of three rods and the changes in their lengths when forces are applied to their ends to put them under strain. Rank the rods according to their strain, greatest first. Initial Length

Change in Length

2L0 4L0 10L0

'L0 2'L0 4'L0

Rod A Rod B Rod C

12 A physical therapist gone wild has constructed the (stationary) assembly of massless pulleys and cords seen in Fig. 12-24. One long cord wraps around all the pulleys, and shorter cords suspend pulleys from the ceiling or weights from the pulleys. Except for one, the weights (in newtons) are indicated. (a) What is that last weight? (Hint: When a cord loops halfway around a pulley as here, it pulls on the pulley with a net force that is twice the tension in the cord.) (b) What is the tension in the short cord labeled T? T

T

6

10

20 17

Figure 12-21 Question 8.

5

34

15

Figure 12-24 Question 12.

23

345

PROB LE M S

Problems Tutoring problem available (at instructor’s discretion) in WileyPLUS and WebAssign SSM • – •••

Worked-out solution available in Student Solutions Manual

WWW Worked-out solution is at

Number of dots indicates level of problem difficulty

ILW

Interactive solution is at

http://www.wiley.com/college/halliday

Additional information available in The Flying Circus of Physics and at flyingcircusofphysics.com

Module 12-1 Equilibrium •1 Because g varies so little over the extent of y most structures, any structure’s center of gravity 3 4 effectively coincides with its center of mass. Here is a fictitious example where g varies more significantly. Figure 12-25 shows an array of 2 5 six particles, each with mass m, fixed to the edge of a rigid structure of negligible mass. The dis1 6 x tance between adjacent particles along the edge Figure 12-25 is 2.00 m. The following table gives the value of g Problem 1. (m/s2) at each particle’s location. Using the coordinate system shown, find (a) the x coordinate xcom and (b) the y coordinate ycom of the center of mass of the six-particle system. Then find (c) the x coordinate xcog and (d) the y coordinate ycog of the center of gravity of the six-particle system.

Particle

g

Particle

g

1 2 3

8.00 7.80 7.60

4 5 6

7.40 7.60 7.80

•4 An archer’s bow is drawn at its midpoint until the tension in the string is equal to the force exerted by the archer. What is the angle between the two halves of the string?

•8 A physics Brady Bunch, whose weights in newtons are indicated in Fig. 12-27, is balanced on a seesaw. What is the number of the person who causes the largest torque about the rotation axis at fulcrum f directed (a) out of the page and (b) into the page? 1

2

3

4

5

6

7

8

560

440

330

220 newtons

1

2

3

4 meters

f

Module 12-2 Some Examples of Static Equilibrium •2 An automobile with a mass of 1360 kg has 3.05 m between the front and rear axles. Its center of gravity is located 1.78 m behind the front axle. With the automobile on level ground, determine the magnitude of the force from the ground on (a) each front wheel (assuming equal forces on the front wheels) and (b) each rear wheel (assuming equal forces on the rear wheels). •3 SSM WWW In Fig. 12-26, a uniform sphere of mass m ! 0.85 kg and radius r ! 4.2 cm is held in place by a massless rope attached to a frictionless wall a distance L ! 8.0 cm above the center of the sphere. Find (a) the tension in the rope and (b) the force on the sphere from the wall.

•7 A 75 kg window cleaner uses a 10 kg ladder that is 5.0 m long. He places one end on the ground 2.5 m from a wall, rests the upper end against a cracked window, and climbs the ladder. He is 3.0 m up along the ladder when the window breaks. Neglect friction between the ladder and window and assume that the base of the ladder does not slip. When the window is on the verge of breaking, what are (a) the magnitude of the force on the window from the ladder, (b) the magnitude of the force on the ladder from the ground, and (c) the angle (relative to the horizontal) of that force on the ladder?

L

r

Figure 12-26 •5 ILW A rope of negligible mass is stretched Problem 3. horizontally between two supports that are 3.44 m apart. When an object of weight 3160 N is hung at the center of the rope, the rope is observed to sag by 35.0 cm. What is the tension in the rope? •6 A scaffold of mass 60 kg and length 5.0 m is supported in a horizontal position by a vertical cable at each end. A window washer of mass 80 kg stands at a point 1.5 m from one end. What is the tension in (a) the nearer cable and (b) the farther cable?

220

330

440

4

3

2

560 1

0

Figure 12-27 Problem 8. •9 SSM A meter stick balances horizontally on a knife-edge at the 50.0 cm mark. With two 5.00 g coins stacked over the 12.0 cm mark, the stick is found to balance at the 45.5 cm mark. What is the mass of the meter stick? •10 The system in Fig. 12-28 is in equilibrium, with the string in the center exactly horizontal. Block A weighs 40 N, block B weighs 50 N, and angle f is 35°. Find (a) tension T1, (b) tension T2, (c) tension T3, and (d) angle u.

T1

φ

θ

T3

T2 A

B

•11 SSM Figure 12-29 shows a diver of weight 580 N standing at the Figure 12-28 Problem 10. end of a diving board with a length of L ! 4.5 m and negligible mass. The board is fixed to two pedestals L (supports) that are separated by distance d ! 1.5 m. Of the forces acting on the board, what are the (a) magnitude and (b) direction (up or down) of the force from the left pedestal d and the (c) magnitude and (d) direction (up or down) of the force from the right pedestal? (e) Which Figure 12-29 Problem 11. pedestal (left or right) is being stretched, and (f) which pedestal is being compressed?

346

CHAPTE R 12 EQU I LI B R I U M AN D E L ASTICITY

•12 In Fig. 12-30, trying to get his car out of mud, a man ties one end of a rope around the front bumper and the other end tightly around a utility pole 18 m away. He then pushes sideways on the rope at its midpoint with a force of 550 N, displacing the center of the rope 0.30 m, but the car barely moves. What is the magnitude of the force on the car from the rope? (The rope stretches somewhat.) F

Figure 12-30 Problem 12. •13 Figure 12-31 shows the anatomical structures in the Calf muscle lower leg and foot that are involved in standing on tipLower leg bones toe, with the heel raised slightly off the floor so that the foot effectively contacts B A the floor only at point P. Assume distance a ! 5.0 cm, P distance b ! 15 cm, and the a b person’s weight W ! 900 N. Figure 12-31 Problem 13. Of the forces acting on the foot, what are the (a) magnitude and (b) direction (up or down) of the force at point A from the calf muscle and the (c) magnitude and (d) direction (up or down) of the force at point B from the lower leg bones? •14 In Fig. 12-32, a horizontal scaffold, of length 2.00 m and uniform mass 50.0 kg, is suspended Figure 12-32 Problem 14. from a building by two cables. The scaffold has dozens of paint cans stacked on it at various points. The total mass of the paint cans is 75.0 kg. The tension in the cable at the right is 722 N. How far horizontally from that cable is the center of mass of the system of paint cans? :

:

y F1

•18 In Fig. 12-35, horizontal scaffold 2, with uniform mass m2 ! 30.0 kg and length L2 ! 2.00 m, hangs from horizontal scaffold 1, with uniform mass m1 ! 50.0 kg. A 20.0 kg box of nails lies on scaffold 2, centered at distance d ! 0.500 m from the left end. What is the tension T in the cable indicated? •19 To crack a certain nut in a nutcracker, forces with magnitudes of at least 40 N must act on its shell from both sides. For the nutcracker of Fig. 12-36, with distances L ! 12 cm and d ! 2.6 cm, what are the force components F" (perpendicular to the handles) corresponding to that 40 N?

Cable D Beam

Figure 12-34 Problem 17.

T=?

1 2 d

d

d

L2

Figure 12-35 Problem 18.

F⊥

L

d

F⊥

Figure 12-36 Problem 19. •20 A bowler holds a bowling ball (M ! 7.2 kg) in the palm of his hand (Fig. 12 -37). His upper arm is vertical; his lower arm (1.8 kg) is horizontal. What is the magnitude of (a) the force of the biceps muscle on the lower arm and (b) the force between the bony structures at the elbow contact point?

Biceps

:

•15 ILW Forces F1, F2, and F3 act on the structure of Fig. 12-33, shown in an overhead view. We wish to put the structure in equilibrium by applying a fourth force, at a point such as P. The fourth : : force has vector components Fh and Fv. We are given that a ! 2.0 m, b ! 3.0 m, c ! 1.0 m, F1 ! 20 N, F2 ! 10 N, and F3 ! 5.0 N. Find (a) Fh, (b) Fv, and (c) d.

c

•17 In Fig. 12-34, a uniform beam of weight 500 N and length 3.0 m is suspended horizontally. On the left it is hinged to a wall; on the right it is supported by a cable bolted to the wall at distance D above the beam. The least tension that will snap the cable is 1200 N. (a) What value of D corresponds to that tension? (b) To prevent the cable from snapping, should D be increased or decreased from that value?

b

O

F2 P

Elbow contact point

4.0 cm 15 cm 33 cm

Lower arm (forearm plus hand) center of mass

Figure 12-37 Problem 20.

a x

Fh Fv

M

a

d F3

Figure 12-33 Problem 15. •16 A uniform cubical crate is 0.750 m on each side and weighs 500 N. It rests on a floor with one edge against a very small, fixed obstruction. At what least height above the floor must a horizontal force of magnitude 350 N be applied to the crate to tip it?

••21 ILW The system in Fig. 12-38 is Strut in equilibrium. A concrete block of mass 225 kg hangs from the end of T the uniform strut of mass 45.0 kg. A cable runs from the ground, over θ φ the top of the strut, and down to the block, holding the block in place. Hinge For angles f ! 30.0° and u ! 45.0°, Figure 12-38 Problem 21. find (a) the tension T in the cable and the (b) horizontal and (c) vertical components of the force on the strut from the hinge.

347

PROB LE M S

••22 In Fig. 12-39, a 55 kg w rock climber is in a lie-back climb along a fissure, with hands pulling on one side of the fissure and feet pressed against the opposite side. d h com The fissure has width w ! 0.20 m, and the center of mass of the climber is a horizontal distance d ! 0.40 m from the fissure. The coefficient of static friction between hands and Figure 12-39 Problem 22. rock is m1 ! 0.40, and between boots and rock it is m2 ! 1.2. (a) What is the least horizontal pull by the hands and push by the feet that will keep the climber stable? (b) For the horizontal pull of (a), what must be the vertical distance h between hands and feet? If the climber encounters wet rock, so that m1 and m2 are reduced, what happens to (c) the answer to (a) and (d) the answer to (b)? ••23 In Fig. 12-40, one end of a uniform beam of weight 222 N is hinged to a wall; the other end is supported by a wire that makes angles u ! 30.0° with both wall and beam. Find (a) the tension in the wire and the (b) horizontal and (c) vertical components of the force of the hinge on the beam.

feet – ground contact point. If he is on the verge of sliding, what is the coefficient of static friction between feet and ground?

y

L ••27 In Fig. 12-44, a 15 kg block is held in place via a pulley system. The com d person’s upper arm is vertical; the forearm is at angle u ! 30° with the horizonx tal. Forearm and hand together have a mass of 2.0 kg, with a center of mass at disa tance d1 ! 15 cm from the contact point of Figure 12-43 Problem 26. the forearm bone and the upper-arm bone (humerus). The triceps muscle pulls vertically upward on the forearm at distance d2 ! 2.5 cm behind that contact point. Distance d3 is 35 cm. What are the (a) magnitude and (b) direction (up or down) of the force on the forearm from the triceps muscle and the (c) magnitude and (d) direction (up or down) of the force on the forearm from the humerus?

θ Triceps Hinge

com

θ Humerus

••24 In Fig. 12 -41, a climber with a weight of 533.8 N is held by a belay rope connected to her climbing harness and belay device; the force of the rope on her has a line of action through her center of mass. The indicated angles are u ! 40.0° and f ! 30.0°. If her feet are on the verge of sliding on the vertical wall, what is the coefficient of static friction between her climbing shoes and the wall?

Figure 12-40 Problem 23.

φ

θ

••25 SSM WWW In Fig. 12-42, what : magnitude of (constant) force F applied horizontally at the axle of the wheel is necessary to raise the wheel over a step obstacle of height h ! 3.00 cm? The wheel’s radius is r ! 6.00 cm, and its mass is m ! 0.800 kg.

Figure 12-41 Problem 24.

r F h

Figure 12-42 Problem 25. ••26 In Fig. 12 -43, a climber leans out against a vertical ice wall that has negligible friction. Distance a is 0.914 m and distance L is 2.10 m. His center of mass is distance d ! 0.940 m from the

θ d1

Figure 12-44 Problem 27.

d3 d2

••28 In Fig. 12-45, suppose the length L of the uniform bar is 3.00 m and its weight is 200 N. Also, let the block’s weight W ! 300 N and the angle u ! 30.0°. The wire can withstand a maximum tension of 500 N. (a) What is the maximum possible distance x A before the wire breaks? With the block placed at this maximum x, what are the (b) horizontal and (c) vertical components of the force on the bar from the hinge at A? ••29 A door has a height of 2.1 m along a y axis that extends vertically upward and a width of 0.91 m along an x axis that extends outward from the hinged edge of the door.A hinge 0.30 m from the top and a hinge 0.30 m from the bottom each support half the door’s mass, which is 27 kg. In unit-vector notation, what are the forces on the door at (a) the top hinge and (b) the bottom hinge? ••30 In Fig. 12-46, a 50.0 kg uniform square sign, of edge length L ! 2.00 m, is hung from a horizontal rod of length dh ! 3.00 m and negligible mass. A cable is attached to the end of the rod

Block

C

x θ

com

B

L

Figure 12-45 Problems 28 and 34.

Cable

dv

Hinge Rod L L dh

Figure 12-46

Problem 30.

348

CHAPTE R 12 EQU I LI B R I U M AN D E L ASTICITY

and to a point on the wall at distance dv ! 4.00 m above the point where the rod is hinged to the wall. (a) What is the tension in the cable? What are the (b) magnitude and (c) direction (left or right) of the horizontal component of the force on the rod from the wall, and the (d) magnitude and (e) direction (up or down) of the vertical component of this force? ••31 In Fig. 12-47, a nonuniform bar is suspended φ at rest in a horizontal position θ by two massless cords. One L cord makes the angle u ! com 36.9° with the vertical; the x other makes the angle f ! 53.1° with the vertical. If the Figure 12-47 Problem 31. length L of the bar is 6.10 m, compute the distance x from the left end of the bar to its center of mass.

••34 In Fig. 12-45, a thin horizontal bar AB of negligible weight and length L is hinged to a vertical wall at A and supported at B by a thin wire BC that makes an angle u with the horizontal. A block of weight W can be moved anywhere along the bar; its position is defined by the distance x from the wall to its center of mass. As a function of x, find (a) the tension in the wire, and the (b) horizontal and (c) vertical components of the force on the bar from the hinge at A.

••32 In Fig. 12-48, the driver of a car on a horizontal road makes an emergency stop by applying the brakes so that all four wheels lock and skid along the road. The coefficient of kinetic friction between tires and road is 0.40. The separation between the front and rear axles is L ! 4.2 m, and the center of mass of the car is located at distance d ! 1.8 m behind the front axle and distance h ! 0.75 m above the road. The car weighs 11 kN. Find the magnitude of (a) the braking acceleration of the car, (b) the normal force on each rear wheel, (c) the normal force on each front wheel, (d) the braking force on each rear wheel, and (e) the braking force on each front wheel. (Hint: Although the car is not in translational equilibrium, it is in rotational equilibrium.)

h d

Figure 12-48 Problem 32.

L

••33 Figure 12-49a shows a vertical uniform beam of length L : that is hinged at its lower end. A horizontal force Fa is applied to Fa Cable y

θ

the beam at distance y from the lower end. The beam remains vertical because of a cable attached at the upper end, at angle u with the horizontal. Figure 12-49b gives the tension T in the cable as a function of the position of the applied force given as a fraction y/L of the beam length. The scale of the T axis is set by Ts ! 600 N. Figure 12-49c gives the magnitude Fh of the horizontal force on the beam from the hinge, also as a function of y/L. Evaluate (a) angle u : and (b) the magnitude of Fa.

L

••35 SSM WWW A cubical box is filled with sand and weighs 890 N. We wish to “roll” the box by pushing horizontally on one of the upper edges. (a) What minimum force is required? (b) What minimum coefficient of static friction between box and floor is required? (c) If there is a more efficient way to roll the box, find the smallest possible force that would have to be applied directly to the box to roll it. (Hint: At the onset of tipping, where is the normal force located?) ••36 Figure 12-50 shows a 70 kg climber y hanging by only the crimp hold of one hand on x the edge of a shallow horizontal ledge in a rock wall. (The fingers are pressed down to gain purchase.) Her feet touch the rock wall at distance H ! 2.0 m directly below her crimped fingers but do not provide any support. Her center of mass is distance a ! 0.20 m from the wall. Assume that the force from com the ledge supporting her fingers is equally shared by the four fingers. What are the values of the (a) horizontal component Fh and (b) vertical component Fv of the force on each fingertip? ••37 In Fig. 12-51, a uniform plank, with a length L of 6.10 m and a weight of 445 N, rests on the ground and against a frictionless roller at the top of a wall of height h ! 3.05 m.The plank remains in equilibrium for any value of u / 70° but slips if u 0 70°. Find the coefficient of static friction between the plank and the ground.

(a) Ts

T (N)

Fh (N)

240 L

120

Roller

h 0

0.2 0.4 0.6 0.8 y/L (b)

1

0

Figure 12-49 Problem 33.

0.2

0.4 0.6 0.8 y/L (c)

1

θ

Figure 12-51 Problem 37.

Fv Fh

H

a

Figure 12-50 Problem 36.

PROB LE M S

be 0.53. How far (in percent) up the ladder must the firefighter go to put the ladder on the verge of sliding?

LA

Bolt

B

d

Module 12-3 Elasticity •43 SSM ILW A horizontal aluminum rod 4.8 cm in diameter projects 5.3 cm from a wall. A 1200 kg object is suspended from the end of the rod. The shear modulus of aluminum is 3.0 $ 1010 N/m2. Neglecting the rod’s mass, find (a) the shear stress on the rod and (b) the vertical deflection of the end of the rod.

x

A

•44 Figure 12-55 shows the stress – strain curve for a material. The scale of the stress axis is set by s ! 300, in units of 106 N/m2. What are (a) the Young’s modulus and (b) the approximate yield strength for this material?

Figure 12-52 Problem 38.

C

B

E

•••40 Figure 12-54a shows a horizontal uniform beam of mass mb and Figure 12-53 Problem 39. length L that is supported on the left by a hinge attached to a wall and on the right by a cable at angle u with the horizontal. A package of mass mp is positioned on the beam at a distance x from the left end. The total mass is mb # mp ! 61.22 kg. Figure 12-54b gives the tension T in the cable as a function of the package’s position given as a fraction x/L of the beam length. The scale of the T axis is set by Ta ! 500 N and Tb ! 700 N. Evaluate (a) angle u, (b) mass mb, and (c) mass mp.

Cable

θ L

T (N)

Tb

x

Ta

0

0.2

(a)

0.4 0.6 x/L

0.8

(b)

Figure 12-54 Problem 40. •••41 A crate, in the form of a cube with edge lengths of 1.2 m, contains a piece of machinery; the center of mass of the crate and its contents is located 0.30 m above the crate’s geometrical center. The crate rests on a ramp that makes an angle u with the horizontal. As u is increased from zero, an angle will be reached at which the crate will either tip over or start to slide down the ramp. If the coefficient of static friction ms between ramp and crate is 0.60, (a) does the crate tip or slide and (b) at what angle u does this occur? If ms ! 0.70, (c) does the crate tip or slide and (d) at what angle u does this occur? (Hint: At the onset of tipping, where is the normal force located?) •••42 In Fig. 12-7 and the associated sample problem, let the coefficient of static friction ms between the ladder and the pavement

s

0 0

0.002

0.004

Strain ••45 In Fig. 12-56, a lead brick rests horizontally on cylinders A and B. Figure 12-55 Problem 44. The areas of the top faces of the cylinders are related by AA ! 2AB; the Young’s moduli of the cylinders dA dB are related by EA ! 2EB. The cylinders had identical lengths before the brick was placed on them. What com of brick fraction of the brick’s mass is supported (a) by cylinder A and (b) by cylinder B? The horizontal distances A B between the center of mass of the Figure 12-56 Problem 45. brick and the centerlines of the cylinders are dA for cylinder A and dB for cylinder B. (c) What is the ratio dA/dB?

D

A

Stress (106 N/m2)

•••39 For the stepladder shown in Fig. 12-53, sides AC and CE are each 2.44 m long and hinged at C. Bar BD is a tie-rod 0.762 m long, halfway up. A man weighing 854 N climbs 1.80 m along the ladder. Assuming that the floor is frictionless and neglecting the mass of the ladder, find (a) the tension in the tie-rod and the magnitudes of the forces on the ladder from the floor at (b) A and (c) E. (Hint: Isolate parts of the ladder in applying the equilibrium conditions.)

y

1

••46 Figure 12-57 shows an approximate plot of stress versus strain for a spider-web thread, out to the point of breaking at a strain of 2.00. The vertical axis scale is set by values a ! 0.12 GN/m2, b ! 0.30 GN/m2, and c ! 0.80 GN/m2. Assume that the thread has an initial length of 0.80 cm, an initial cross-sectional area of 8.0 $ 10%12 m2, and (during stretching) a constant volume. The strain on the thread is the ratio of the change in the thread’s length to that initial length, and the stress on the thread is the ratio of the collision force to that initial cross-sectional area. Assume that the work done on the thread by the collision force is given by the area under the curve on the graph. Assume also that when the single thread snares a flying insect, the insect’s kinetic energy is transferred to the stretching of the thread. (a) How much kinetic energy would put the thread on the verge of breaking? What is the kinetic energy of (b) a fruit fly of mass 6.00 mg and speed 1.70 m/s and (c) a bumble bee of mass 0.388 g and speed 0.420 m/s? Would (d) the fruit fly and (e) the bumble bee break the thread? c Stress (GN/m2)

••38 In Fig. 12-52, uniform beams A and B are attached to a wall with hinges and loosely bolted together (there is no torque of one on the other). Beam A has length LA ! 2.40 m and mass 54.0 kg; beam B has mass 68.0 kg. The two hinge points are separated by distance d ! 1.80 m. In unit-vector notation, what is the force on (a) beam A due to its hinge, (b) beam A due to the bolt, (c) beam B due to its hinge, and (d) beam B due to the bolt?

349

b a 0

1.0 1.4 Strain

2.0

Figure 12-57 Problem 46.

350

CHAPTE R 12 EQU I LI B R I U M AN D E L ASTICITY

••47 A tunnel of length L ! 150 m, height H ! 7.2 m, and width 5.8 m (with a flat roof) is to be constructed at distance d ! 60 m beneath the ground. (See Fig. 12-58.) The tunnel roof is to be supported entirely by square steel columns, each with a cross-sectional area of 960 cm2. The mass of 1.0 cm3 of the ground material is 2.8 g. (a) What is the total weight of the ground material the columns must support? (b) How many columns are needed to keep the compressive stress on each column at one-half its ultimate strength?

are forced against rigid walls at distances rA ! 7.0 cm and rB ! 4.0 cm from the axle. Initially the stoppers touch the walls without being : compressed. Then force F of magnitude 220 N is applied perpendicular to the rod at a distance R ! 5.0 cm from the axle. Find the magnitude of the force compressing (a) stopper A and (b) stopper B. F

R

L Stopper A

rA

rB

Stopper B

Axle

d H

Figure 12-62 Problem 51. Additional Problems 52 After a fall, a 95 kg rock climber finds himself dangling from the end of a rope that had been 15 m long and 9.6 mm in diameter but has stretched by 2.8 cm. For the rope, calculate (a) the strain, (b) the stress, and (c) the Young’s modulus.

Figure 12-58 Problem 47. Stress (107 N/m2)

s ••48 Figure 12-59 shows the stress versus strain plot for an aluminum wire that is stretched by a machine pulling in opposite directions at the two ends of the wire. The scale of the stress axis is set by s ! 7.0, in units of 0 1.0 107 N/m2. The wire has an initial Strain (10–3) length of 0.800 m and an initial Figure 12-59 Problem 48. cross-sectional area of 2.00 $ 10%6 m2. How much work does the force from the machine do on the wire to produce a strain of 1.00 $ 10%3?

••49 In Fig. 12-60, a 103 kg uniform log hangs by two steel wires, A and B, both of radius 1.20 mm. Initially, wire A was 2.50 m long and 2.00 mm shorter than wire B. The log is now horizontal. What are the magnitudes of the forces on it from (a) wire A and (b) wire B? (c) What is the ratio dA/dB?

Wire A

dA

dB

Wire B

com

Figure 12-60 Problem 49.

•••50 Figure 12-61 represents an insect caught at the midpoint of a spider-web thread. The thread breaks under a stress of 8.20 $ 108 N/m2 and a strain of 2.00. Initially, it was horizontal and had a length of 2.00 cm and a Figure 12-61 Problem 50. cross-sectional area of 8.00 $ 10%12 m2. As the thread was stretched under the weight of the insect, its volume remained constant. If the weight of the insect puts the thread on the verge of breaking, what is the insect’s mass? (A spider’s web is built to break if a potentially harmful insect, such as a bumble bee, becomes snared in the web.) •••51 Figure 12-62 is an overhead view of a rigid rod that turns about a vertical axle until the identical rubber stoppers A and B

53 SSM In Fig. 12-63, a rectangular slab of slate rests on a bedrock surL face inclined at angle u ! 26°. The slab has length L ! 43 m, thickness T T ! 2.5 m, and width W ! 12 m, and θ 1.0 cm3 of it has a mass of 3.2 g. The coefficient of static friction between Figure 12-63 Problem 53. slab and bedrock is 0.39. (a) Calculate the component of the gravitational force on the slab parallel to the bedrock surface. (b) Calculate the magnitude of the static frictional force on the slab. By comparing (a) and (b), you can see that the slab is in danger of sliding. This is prevented only by chance protrusions of bedrock. (c) To stabilize the slab, bolts are to be driven perpendicular to the bedrock surface (two bolts are shown). If each bolt has a crosssectional area of 6.4 cm2 and will snap under a shearing stress of 3.6 $ 108 N/m2, what is the minimum number of bolts needed? Assume that the bolts do not affect the normal force. 54 A uniform ladder whose length is 5.0 m and whose weight is 400 N leans against a frictionless vertical wall. The coefficient of static friction between the level ground and the foot of the ladder is 0.46. What is the greatest distance the foot of the ladder can be placed from the base of the wall without the ladder immediately slipping? 55 SSM In Fig. 12-64, block A (mass 10 kg) is in equilibrium, but it would slip if block B (mass 5.0 kg) were any heavier. For angle u ! 30°, what is the coefficient of static friction between block A and the surface below it?

θ

A

B

Figure 12-64 Problem 55.

56 Figure 12-65a shows a uniform ramp between two buildings that allows for motion between the buildings due to strong winds.

351

PROB LE M S

At its left end, it is hinged to the building wall; at its right end, it has a roller that can roll along the building wall. There is no vertical force on the roller from the building, only a horizontal force with magnitude Fh. The horizontal distance between the buildings is D ! 4.00 m. The rise of the ramp is h ! 0.490 m. A man walks across the ramp from the left. Figure 12-65b gives Fh as a function of the horizontal distance x of the man from the building at the left. The scale of the Fh axis is set by a ! 20 kN and b ! 25 kN. What are the masses of (a) the ramp and (b) the man? b

h

Fh (kN)

D x

a

0

2 x (m)

(a)

4

(b)

Figure 12-65 Problem 56. 57 In Fig. 12-66, a 10 kg sphere is supported on a frictionless plane inclined at angle u ! 45° from the horizontal. Angle f is 25°. Calculate the tension in the cable.

Cable

φ

58 In Fig. 12-67a, a uniform 40.0 kg beam is centered over two rollers. Vertical lines across the beam mark off equal lengths.Two of the lines are centered over the rollers; a 10.0 kg package of tamales is centered over roller B. What are the magnitudes of the forces on the beam from (a) roller A and (b) roller B? The beam is then rolled to the left until the right-hand end is centered over roller B (Fig. 12-67b). What now are the magnitudes of the forces on the beam from (c) roller A and (d) roller B? Next, the beam is rolled to the right. Assume that it has a length of 0.800 m. (e) What horizontal distance between the package and roller B puts the beam on the verge of losing contact with roller A? 59 SSM In Fig. 12-68, an 817 kg construction bucket is suspended by a cable A that is attached at O to two other cables B and C, making angles u1 ! 51.0° and u2 ! 66.0° with the horizontal. Find the tensions in (a) cable A, (b) cable B, and (c) cable C. (Hint: To avoid solving two equations in two unknowns, position the axes as shown in the figure.)

θ

Figure 12-66 Problem 57.

A

B (a)

(b)

Figure 12-67 Problem 58.

x

y

B

θ1

C O

θ2

A

Figure 12-68 Problem 59.

60 In Fig. 12-69, a package of mass m hangs from a short cord that is tied to the wall via cord 1 and to the ceiling via cord 2. Cord 1 is at angle f ! 40° with the horizontal; cord 2 is at angle u. (a) For what value of u is the tension in cord 2 minimized? (b) In terms of mg, what is the minimum tension in cord 2?

1

2

φ

θ

Figure 12-69 Problem 60. : 61 ILW The force F in Fig. 12-70 keeps the 6.40 kg block and the pulleys in T equilibrium. The pulleys have negligible mass and friction. Calculate the tension T in the upper cable. (Hint: When a cable wraps halfway around a pulley as here, the magnitude of its net force on the pulley is twice the tension in the cable.) 62 A mine elevator is supported by a single steel cable 2.5 cm in diameter. The total mass of the elevator cage and occupants is 670 kg. By how much does the cable stretch when the elevator hangs by (a) 12 m of cable and (b) 362 m of cable? (Neglect the mass of the cable.)

F

Figure 12-70 Problem 61.

63 Four bricks of length L, identical and uniform, are stacked on top of one another (Fig. 12-71) in such a way that part of each extends beyond the one beneath. Find, in terms of L, the maximum values of (a) a1, (b) a2, (c) a3, (d) a4, and (e) h, such that the stack is a4 in equilibrium, Figure 12-71 on the verge of Problem 63. falling. 64 In Fig. 12-72, two identical, uniform, and frictionless spheres, each of mass m, rest in a rigid rectangular container. A line connecting their centers is at 45° to the horizontal. Find the magnitudes of the forces on the spheres from (a) the bottom of the container, (b) the left side of the container, (c) the right side of the container, and (d) each other. (Hint: The force of one sphere on the other is directed along the center – center line.) 65 In Fig. 12-73, a uniform beam with a weight of 60 N and a length of 3.2 m is hinged at its lower end, and : a horizontal force F of magnitude 50 N acts at its upper end. The beam is held vertical by a cable that makes angle u ! 25° with the ground and is attached to the beam at height h ! 2.0 m. What are (a) the tension in the cable and (b) the force on the beam from the hinge in unit-vector notation?

L a1 a2 a3

h

45°

Figure 12-72 Problem 64. F y x

h

θ

Figure 12-73 Problem 65.

352

CHAPTE R 12 EQU I LI B R I U M AN D E L ASTICITY

66 A uniform beam is 5.0 m long and has a mass of 53 kg. In Fig. 1274, the beam is supported in a horizontal position by a hinge and a cable, with angle u ! 60°. In unit-vector notation, what is the force on the beam from the hinge?

Cable

y x

θ Beam

Figure 12-74 Problem 66.

67 A solid copper cube has an edge length of 85.5 cm. How much stress must be applied to the cube to reduce the edge length to 85.0 cm? The bulk modulus of copper is 1.4 $ 1011 N/m2. P

68 A construction worker attempts to lift a uniform beam off the floor and raise it to a vertical position. The beam is 2.50 m long and weighs 500 N. At a certain instant the d worker holds the beam momentarily at rest with one end at distance d ! 1.50 m above the floor, as shown in : Fig. 12-75, by exerting a force P on the beam, perpendicular to the Figure 12-75 Problem 68. beam. (a) What is the magnitude P? (b) What is the magnitude of the (net) force of the floor on the beam? (c) What is the minimum value the coefficient of static friction between beam and floor can have in order for the beam not to slip at this instant? 69 SSM In Fig. 12-76, a uniform rod of mass m is hinged to a building at its lower end, while its upper end is held in place by a rope attached to the wall. If angle u1 ! 60°, what value must angle u2 have so that the tension in the rope is equal to mg/2?

θ2

Rope

θ1 Rod

70 A 73 kg man stands on a level bridge of length L. He is at distance L/4 from one end. The Figure 12-76 bridge is uniform and weighs 2.7 kN. What are the magnitudes of the vertical forces on the bridge from Problem 69. its supports at (a) the end farther from him and (b) the nearer end? 71 SSM A uniform cube of side length 8.0 cm rests on a horizontal floor. The coefficient of static friction between cube and floor is : m. A horizontal pull P is applied perpendicular to one of the vertical faces of the cube, at a distance 7.0 cm above the floor on the : vertical midline of the cube face. The magnitude of P is gradually increased. During that increase, for what values of m will the cube eventually (a) begin to slide and (b) begin to tip? (Hint: At the onset of tipping, where is the normal force located?) 72 The system in Fig. 12-77 is in equilibrium.The angles are u1 ! 60° and u2 ! 20°, and the ball has mass M ! 2.0 kg. What is the tension in (a) string ab and (b) string bc? c

θ1

b M

θ2 a

Figure 12-77 Problem 72.

73 SSM A uniform ladder is 10 m y long and weighs 200 N. In Fig. 12-78, the ladder leans against x a vertical, frictionless wall at height h ! 8.0 m above the ground. A : horizontal force F is applied to the h ladder at distance d ! 2.0 m from its base (measured along the ladF der). (a) If force magnitude F ! 50 N, what is the force of the ground d on the ladder, in unit-vector notation? (b) If F ! 150 N, what is the Figure 12-78 Problem 73. force of the ground on the ladder, also in unit-vector notation? (c) Suppose the coefficient of static friction between the ladder and the ground is 0.38; for what minimum value of the force magnitude F will the base of the ladder just barely start to move toward the wall? 74 A pan balance is made up of a rigid, massless rod with a hanging pan attached at each end. The rod is supported at and free to rotate about a point not at its center. It is balanced by unequal masses placed in the two pans. When an unknown mass m is placed in the left pan, it is balanced by a mass m1 placed in the right pan; when the mass m is placed in the right pan, it is balanced by a mass m2 in the left pan. Show that m ! 1m1m2.

75 The rigid square frame in G A B Fig. 12-79 consists of the four side bars T T AB, BC, CD, and DA plus two diagonal bars AC and BD, which pass each other freely at E. By means of the turnE buckle G, bar AB is put under tension, as if its ends were subject to horizontal, : D C outward forces T of magnitude 535 N. Figure 12-79 Problem 75. (a) Which of the other bars are in tension? What are the magnitudes of (b) the forces causing the tension in those bars and (c) the forces causing compression in the other bars? (Hint: Symmetry considerations can lead to considerable simplification in this problem.) 76 A gymnast with mass 46.0 kg stands on the end of a uniform balance beam as shown in Fig. 12-80. The beam is 5.00 m long and has a mass of 250 kg (excluding the mass of the two supports). Each support is 0.540 m from its end of the beam. In unit-vector notation, what are the forces on the beam due to (a) support 1 and (b) support 2?

y x 1

2

Figure 12-80 Problem 76.

77 Figure 12-81 shows a 300 kg Ceiling cylinder that is horizontal. Three steel wires support the cylinder 1 2 3 from a ceiling. Wires 1 and 3 are attached at the ends of the cylinder, and wire 2 is attached at the cenFigure 12-81 Problem 77. ter. The wires each have a crosssectional area of 2.00 $ 10%6 m2. Initially (before the cylinder was put in place) wires 1 and 3 were 2.0000 m long and wire 2 was 6.00 mm longer than that. Now (with the cylinder in place) all three wires have been stretched. What is the tension in (a) wire 1 and (b) wire 2?

353

PROB LE M S

78 In Fig. 12-82, a uniform beam of length 12.0 m is supported by a horizontal cable and a hinge at angle u ! 50.0°. The tension in the cable is 400 N. In unit-vector notation, what are (a) the gravitational force on the beam and (b) the force on the beam from the hinge?

Cable

θ y x

79 Four bricks of length L, identical and uniform, are stacked on a table in two ways, as shown in Figure 12-82 Problem 78. Fig. 12-83 (compare with Problem 63). We seek to maximize the overhang distance h in both arrangements. Find the optimum distances a1, a2, b1, and b2, and calculate h for the two arrangements. L

b1

L

a2

b1

a1 b2

h

(a)

b1 h

(b)

Figure 12-83 Problem 79. 80 A cylindrical aluminum rod, with an initial length of 0.8000 m and radius 1000.0 mm, is clamped in place at one end and then stretched by a machine pulling parallel to its length at its other end. Assuming that the rod’s density (mass per unit volume) does not change, find the force magnitude that is required of the machine to decrease the radius to 999.9 mm. (The yield strength is not exceeded.) 81 A beam of length L is carried by three men, one man at one end and the other two supporting the beam between them on a crosspiece placed so that the load of the beam is equally divided among the three men. How far from the beam’s free end is the crosspiece placed? (Neglect the mass of the crosspiece.) 82 If the (square) beam in Fig. 12-6a and the associated sample problem is of Douglas fir, what must be its thickness to keep the compressive stress on it to 16 of its ultimate strength? 83 Figure 12-84 shows a stationary arrangement of two crayon boxes and three cords. Box A has a mass of 11.0 kg and is on a ramp at angle u ! 30.0°; box B has a mass of 7.00 kg and hangs on a cord. The cord connected to box A is parallel to the ramp, which is frictionless. (a) What is the tension in the upper cord, and (b) what angle does that cord make with the horizontal? y

the loop with the rope hanging vertically when the child’s father pulls on the child with a horizontal force and displaces the child to one side. Just before the child is released from rest, the rope makes an angle of 15( with the vertical and the tension in the rope is 280 N. (a) How much does the child weigh? (b) What is the magnitude of the (horizontal) force of the father on the child just before the child is released? (c) If the maximum horizontal force the father can exert on the child is 93 N, what is the maximum angle with the vertical the rope can make while the father is pulling horizontally? 85 Figure 12-85a shows details of a finger in the crimp hold of the climber in Fig. 12-50. A tendon that runs from muscles in the forearm is attached to the far bone in the finger. Along the way, the tendon runs through several guiding sheaths called pulleys. The A2 pulley is attached to the first finger bone; the A4 pulley is attached to the second finger bone. To pull the finger toward the palm, the forearm muscles pull the tendon through the pulleys, much like strings on a marionette can be pulled to move parts of the marionette. Figure 12-85b is a simplified diagram of the sec: ond finger bone, which has length d. The tendon’s pull Ft on the bone acts at the point where the tendon enters the A4 pulley, at distance d/3 along the bone. If the force components on each of the four crimped fingers in Fig. 12-50 are Fh ! 13.4 N and Fv ! : 162.4 N, what is the magnitude of Ft ? The result is probably tolerable, but if the climber hangs by only one or two fingers, the A2 and A4 pulleys can be ruptured, a common ailment among rock climbers. Fc First finger bone

Ft Second finger bone

A2 A4

d/3

45° 10°

Fv

Far bone

Tendon

Attached (a)

80°

Fh

(b)

Figure 12-85 Problem 85. 86 A trap door in a ceiling is 0.91 m square, has a mass of 11 kg, and is hinged along one side, with a catch at the opposite side. If the center of gravity of the door is 10 cm toward the hinged side from the door’s center, what are the magnitudes of the forces exerted by the door on (a) the catch and (b) the hinge? :

x A B

θ

Figure 12-84 Problem 83. 84 A makeshift swing is constructed by making a loop in one end of a rope and tying the other end to a tree limb. A child is sitting in

87 A particle is acted on by forces given, in newtons, by F1 ! : 8.40 iˆ % 5.70 jˆ and F2 ! 16.0 iˆ # 4.10 jˆ. (a) What are the x component : and (b) y component of the force F 3 that balances the sum of these : forces? (c) What angle does F 3 have relative to the #x axis? 88 The leaning Tower of Pisa is 59.1 m high and 7.44 m in diameter. The top of the tower is displaced 4.01 m from the vertical. Treat the tower as a uniform, circular cylinder. (a) What additional displacement, measured at the top, would bring the tower to the verge of toppling? (b) What angle would the tower then make with the vertical?

C

H

A

P

T

E

R

1

3

Gravitation 13-1 NEWTON’S LAW OF GRAVITATION Learning Objectives After reading this module, you should be able to . . .

13.01 Apply Newton’s law of gravitation to relate the gravitational force between two particles to their masses and their separation. 13.02 Identify that a uniform spherical shell of matter attracts a particle that is outside the shell as if all the shell’s mass were concentrated as a particle at its center.

13.03 Draw a free-body diagram to indicate the gravitational force on a particle due to another particle or a uniform, spherical distribution of matter.

Key Ideas ● Any particle in the universe attracts any other particle with a

gravitational force whose magnitude is m1m2 F!G (Newton’s law of gravitation), r2 where m1 and m2 are the masses of the particles, r is their separation, and G (! 6.67 $ 10%11 N "m2/kg2) is the gravitational constant.

● The gravitational force between extended bodies is found

by adding (integrating) the individual forces on individual particles within the bodies. However, if either of the bodies is a uniform spherical shell or a spherically symmetric solid, the net gravitational force it exerts on an external object may be computed as if all the mass of the shell or body were located at its center.

What Is Physics? One of the long-standing goals of physics is to understand the gravitational force—the force that holds you to Earth, holds the Moon in orbit around Earth, and holds Earth in orbit around the Sun. It also reaches out through the whole of our Milky Way galaxy, holding together the billions and billions of stars in the Galaxy and the countless molecules and dust particles between stars. We are located somewhat near the edge of this disk-shaped collection of stars and other matter, 2.6 $ 104 light-years (2.5 $ 1020 m) from the galactic center, around which we slowly revolve. The gravitational force also reaches across intergalactic space, holding together the Local Group of galaxies, which includes, in addition to the Milky Way, the Andromeda Galaxy (Fig. 13-1) at a distance of 2.3 $ 106 light-years away from Earth, plus several closer dwarf galaxies, such as the Large Magellanic Cloud. The Local Group is part of the Local Supercluster of galaxies that is being drawn by the gravitational force toward an exceptionally massive region of space called the Great Attractor. This region appears to be about 3.0 $ 108 light-years from Earth, on the opposite side of the Milky Way. And the gravitational force is even more far-reaching because it attempts to hold together the entire universe, which is expanding. 354

13-1 N EWTON’S L AW OF G RAVITATION

355

This force is also responsible for some of the most mysterious structures in the universe: black holes. When a star considerably larger than our Sun burns out, the gravitational force between all its particles can cause the star to collapse in on itself and thereby to form a black hole. The gravitational force at the surface of such a collapsed star is so strong that neither particles nor light can escape from the surface (thus the term “black hole”). Any star coming too near a black hole can be ripped apart by the strong gravitational force and pulled into the hole. Enough captures like this yields a supermassive black hole. Such mysterious monsters appear to be common in the universe. Indeed, such a monster lurks at the center of our Milky Way galaxy—the black hole there, called Sagittarius A*, has a mass of about 3.7 $ 106 solar masses. The gravitational force near this black hole is so strong that it causes orbiting stars to whip around the black hole, completing an orbit in as little as 15.2 y. Although the gravitational force is still not fully understood, the starting point in our understanding of it lies in the law of gravitation of Isaac Newton.

Newton’s Law of Gravitation Before we get to the equations, let’s just think for a moment about something that we take for granted. We are held to the ground just about right, not so strongly that we have to crawl to get to school (though an occasional exam may leave you crawling home) and not so lightly that we bump our heads on the ceiling when we take a step. It is also just about right so that we are held to the ground but not to each other (that would be awkward in any classroom) or to the objects around us (the phrase “catching a bus” would then take on a new meaning). The attraction obviously depends on how much “stuff” there is in ourselves and other objects: Earth has lots of “stuff” and produces a big attraction but another person has less “stuff” and produces a smaller (even negligible) attraction. Moreover, this “stuff” always attracts other “stuff,” never repelling it (or a hard sneeze could put us into orbit). In the past people obviously knew that they were being pulled downward (especially if they tripped and fell over), but they figured that the downward force was unique to Earth and unrelated to the apparent movement of astronomical bodies across the sky. But in 1665, the 23-year-old Isaac Newton recognized that this force is responsible for holding the Moon in its orbit. Indeed he showed that every body in the universe attracts every other body. This tendency of bodies to move toward one another is called gravitation, and the “stuff” that is involved is the mass of each body. If the myth were true that a falling apple inspired Newton to his law of gravitation, then the attraction is between the mass of the apple and the mass of Earth. It is appreciable because the mass of Earth is so large, but even then it is only about 0.8 N. The attraction between two people standing near each other on a bus is (thankfully) much less (less than 1 mN) and imperceptible. The gravitational attraction between extended objects such as two people can be difficult to calculate. Here we shall focus on Newton’s force law between two particles (which have no size). Let the masses be m1 and m2 and r be their separation. Then the magnitude of the gravitational force acting on each due to the presence of the other is given by F!G

m1m2 r2

(Newton’s law of gravitation).

(13-1)

G is the gravitational constant: G ! 6.67 $ 10%11 N "m2/kg2 ! 6.67 $ 10%11 m3/kg "s2.

(13-2)

Courtesy NASA

Figure 13-1 The Andromeda Galaxy. Located 2.3 $ 106 light-years from us, and faintly visible to the naked eye, it is very similar to our home galaxy, the Milky Way.

356

CHAPTE R 13 G RAVITATION :

This is the pull on particle 1 due to particle 2. F

1

In Fig. 13-2a, F is the gravitational force acting on particle 1 (mass m1) due to particle 2 (mass m2). The force is directed toward particle 2 and is said to be an attractive force because particle 1 is attracted toward particle 2. The magnitude : of the force is given by Eq. 13-1. We can describe F as being in the positive direction of an r axis extending radially from particle 1 through particle 2 (Fig. 13-2b). : We can also describe F by using a radial unit vector rˆ (a dimensionless vector of magnitude 1) that is directed away from particle 1 along the r axis (Fig. 13-2c). From Eq. 13-1, the force on particle 1 is then

2

r

(a)

m1m2 (13-3) rˆ . r2 The gravitational force on particle 2 due to particle 1 has the same magnitude as the force on particle 1 but the opposite direction. These two forces form a third-law force pair, and we can speak of the gravitational force between the two particles as having a magnitude given by Eq. 13-1. This force between two particles is not altered by other objects, even if they are located between the particles. Put another way, no object can shield either particle from the gravitational force due to the other particle. The strength of the gravitational force — that is, how strongly two particles with given masses at a given separation attract each other — depends on the value of the gravitational constant G. If G — by some miracle — were suddenly multiplied by a factor of 10, you would be crushed to the floor by Earth’s attraction. If G were divided by this factor, Earth’s attraction would be so weak that you could jump over a building. Nonparticles. Although Newton’s law of gravitation applies strictly to particles, we can also apply it to real objects as long as the sizes of the objects are small relative to the distance between them. The Moon and Earth are far enough apart so that, to a good approximation, we can treat them both as particles—but what about an apple and Earth? From the point of view of the apple, the broad and level Earth, stretching out to the horizon beneath the apple, certainly does not look like a particle. Newton solved the apple – Earth problem with the shell theorem: :

Draw the vector with its tail on particle 1 to show the pulling.

(b)

1

F !G

2

r

2

r

F

A unit vector points along the radial axis. 1 ˆr (c)

:

Figure 13-2 (a) The gravitational force F on particle 1 due to particle 2 is an attractive force because particle 1 is at: tracted to particle 2. (b) Force F is directed along a radial coordinate axis r extending from particle 1 through par: ticle 2. (c) F is in the direction of a unit vector rˆ along the r axis.

A uniform spherical shell of matter attracts a particle that is outside the shell as if all the shell’s mass were concentrated at its center.

F = 0.80 N F = 0.80 N

Figure 13-3 The apple pulls up on Earth just as hard as Earth pulls down on the apple.

Earth can be thought of as a nest of such shells, one within another and each shell attracting a particle outside Earth’s surface as if the mass of that shell were at the center of the shell. Thus, from the apple’s point of view, Earth does behave like a particle, one that is located at the center of Earth and has a mass equal to that of Earth. Third-Law Force Pair. Suppose that, as in Fig. 13-3, Earth pulls down on an apple with a force of magnitude 0.80 N. The apple must then pull up on Earth with a force of magnitude 0.80 N, which we take to act at the center of Earth. In the language of Chapter 5, these forces form a force pair in Newton’s third law. Although they are matched in magnitude, they produce different accelerations when the apple is released. The acceleration of the apple is about 9.8 m/s2, the familiar acceleration of a falling body near Earth’s surface. The acceleration of Earth, however, measured in a reference frame attached to the center of mass of the apple – Earth system, is only about 1 $ 10%25 m/s2.

Checkpoint 1 A particle is to be placed, in turn, outside four objects, each of mass m: (1) a large uniform solid sphere, (2) a large uniform spherical shell, (3) a small uniform solid sphere, and (4) a small uniform shell. In each situation, the distance between the particle and the center of the object is d. Rank the objects according to the magnitude of the gravitational force they exert on the particle, greatest first.

13-2 G RAVITATION AN D TH E PR I NCI PLE OF SU PE R POSITION

357

13-2 GRAVITATION AND THE PRINCIPLE OF SUPERPOSITION Learning Objectives After reading this module, you should be able to . . .

13.04 If more than one gravitational force acts on a particle, draw a free-body diagram showing those forces, with the tails of the force vectors anchored on the particle.

13.05 If more than one gravitational force acts on a particle, find the net force by adding the individual forces as vectors.

Key Ideas ● Gravitational forces obey the principle of superposition;

gravitational force F 1 on a particle from an extended body is found by first dividing the body into units of differential : mass dm, each of which produces a differential force dF on the particle, and then integrating over all those units to find the sum of those forces:

that is, if n particles interact, the net force on a particle labeled particle 1 is the sum of the forces on it from all the other particles taken one at a time: :

F1,net !

n

:

● The

: F 1,net

' F1i, i!2 :

:

F1 !

:

in which the sum is a vector sum of the forces F 1i on particle 1 from particles 2, 3, . . . , n.

Gravitation and the Principle of Superposition Given a group of particles, we find the net (or resultant) gravitational force on any one of them from the others by using the principle of superposition. This is a general principle that says a net effect is the sum of the individual effects. Here, the principle means that we first compute the individual gravitational forces that act on our selected particle due to each of the other particles. We then find the net force by adding these forces vectorially, just as we have done when adding forces in earlier chapters. Let’s look at two important points in that last (probably quickly read) sentence. (1) Forces are vectors and can be in different directions, and thus we must add them as vectors, taking into account their directions. (If two people pull on you in the opposite direction, their net force on you is clearly different than if they pull in the same direction.) (2) We add the individual forces. Think how impossible the world would be if the net force depended on some multiplying factor that varied from force to force depending on the situation, or if the presence of one force somehow amplified the magnitude of another force. No, thankfully, the world requires only simple vector addition of the forces. For n interacting particles, we can write the principle of superposition for the gravitational forces on particle 1 as :

:

:

:

:

:

F 1,net ! F 12 # F 13 # F 14 # F 15 # " " " # F 1n.

(13-4)

:

Here F 1,net is the net force on particle 1 due to the other particles and, for exam: ple, F 13 is the force on particle 1 from particle 3. We can express this equation more compactly as a vector sum: :

F1,net !

n

' F1i. i!2 :

(13-5)

Real Objects. What about the gravitational force on a particle from a real (extended) object? This force is found by dividing the object into parts small enough to treat as particles and then using Eq. 13-5 to find the vector sum of the forces on the particle from all the parts. In the limiting case, we can divide the extended object : into differential parts each of mass dm and each producing a differential force dF

"

:

dF.

358

CHAPTE R 13 G RAVITATION

Sample Problem 13.01 Net gravitational force, 2D, three particles Figure 13-4a shows an arrangement of three particles, particle 1 of mass m1 ! 6.0 kg and particles 2 and 3 of mass m2 ! m3 ! 4.0 kg, and distance a ! 2.0 cm. What is the net gravi: tational force F 1,net on particle 1 due to the other particles?

:

Force F 12 is directed in the positive direction of the y axis (Fig. : 13-4b) and has only the y component F12. Similarly, F 13 is directed in the negative direction of the x axis and has only the x component %F13 (Fig. 13-4c). (Note something important: We draw the force diagrams with the tail of a force vector anchored on the particle experiencing the force. Drawing them in other ways invites errors, especially on exams.) : To find the net force F 1,net on particle 1, we must add the two forces as vectors (Figs. 13-4d and e). We can do so on a vector-capable calculator. However, here we note that : %F13 and F12 are actually the x and y components of F 1,net. Therefore, we can use Eq. 3-6 to find first the magnitude and : then the direction of F 1,net. The magnitude is

KEY IDEAS (1) Because we have particles, the magnitude of the gravitational force on particle 1 due to either of the other particles is given by Eq. 13-1 (F ! Gm1m2/r2). (2) The direction of either gravitational force on particle 1 is toward the particle responsible for it. (3) Because the forces are not along a single axis, we cannot simply add or subtract their magnitudes or their components to get the net force. Instead, we must add them as vectors.

F1,net ! 2(F12)2 # (%F13)2

:

Calculations: From Eq. 13-1, the magnitude of the force F 12 on particle 1 from particle 2 is Gm1m2 (13-7) F12 ! a2

! 2(4.00 $ 10%6 N)2 # (%1.00 $ 10%6 N)2

! 4.1 $ 10%6 N.

(Answer)

Relative to the positive direction of the x axis, Eq. 3-6 gives : the direction of F 1,net as

(6.67 $ 10%11 m3/ kg "s2)(6.0 kg)(4.0 kg) ! (0.020 m)2 %6 ! 4.00 $ 10 N.

F12 4.00 $ 10%6 N ! tan%1 ! %76(. %F13 %1.00 $ 10%6 N

1 ! tan%1

:

Similarly, the magnitude of force F 13 on particle 1 from particle 3 is Gm1m3 (13-8) F13 ! (2a)2

Is this a reasonable direction (Fig. 13-4f)? No, because the : : direction of F 1,net must be between the directions of F 12 and : F 13. Recall from Chapter 3 that a calculator displays only one of the two possible answers to a tan%1 function. We find the other answer by adding 180°:

(6.67 $ 10%11 m3/ kg"s2)(6.0 kg)(4.0 kg) (0.040 m)2 %6 ! 1.00 $ 10 N. !

%76° # 180° ! 104°, which is a reasonable direction for

: F 1,net

(Answer) (Fig. 13-4g).

Additional examples, video, and practice available at WileyPLUS

on the particle. In this limit, the sum of Eq. 13-5 becomes an integral and we have :

F1 !

"

:

dF,

(13-6)

in which the integral is taken over the entire extended object and we drop the subscript “net.” If the extended object is a uniform sphere or a spherical shell, we can avoid the integration of Eq. 13-6 by assuming that the object’s mass is concentrated at the object’s center and using Eq. 13-1.

Checkpoint 2 The figure shows four arrangements of three particles of equal masses. (a) Rank the arrangements according to the magnitude of the net gravitational force on the particle labeled m, greatest first. (b) In arrangement 2, is the direction of the net force closer to the line of length d or to the line of length D?

D

D

d m

m (1)

d

(2)

d

D

m

m

D

d (3)

(4)

13-3 G RAVITATION N EAR EARTH’S SU R FACE

We want the forces (pulls) on particle 1, not the forces on the other particles. m3

2a

This is the force (pull) on particle 1 due to particle 2.

y m2

F12

x

m1

y

m1

F12

F13

m1

x

(c)

y

y

m1

F13

(d )

x

F1,net

x

This is one way to show the net force on particle 1. Note the head-to-tail arrangement.

x

m1

(b)

F13 y

F12

A

y This is the force (pull) on particle 1 due to particle 3.

y

a

(a)

F1,net

359

104° x

x –76°

(e)

This is another way, also a head-to-tail arrangement.

x

(f )

A calculator's inverse tangent can give this for the angle.

(g)

But this is the correct angle.

Figure 13-4 (a) An arrangement of three particles. The force on particle 1 due to (b) particle 2 and (c) particle 3. (d)–(g) Ways to combine the forces to get the net force magnitude and orientation. In WileyPLUS, this figure is available as an animation with voiceover.

13-3 GRAVITATION NEAR EARTH’S SURFACE Learning Objectives After reading this module, you should be able to . . .

13.06 Distinguish between the free-fall acceleration and the gravitational acceleration.

13.07 Calculate the gravitational acceleration near but outside a uniform, spherical astronomical body. 13.08 Distinguish between measured weight and the magnitude of the gravitational force.

Key Ideas ● The

gravitational acceleration ag of a particle (of mass m) is due solely to the gravitational force acting on it. When the particle is at distance r from the center of a uniform, spherical body of mass M, the magnitude F of the gravitational force on the particle is given by Eq. 13-1. Thus, by Newton’s second law, F ! mag, which gives

GM . r2 ● Because Earth’s mass is not distributed uniformly, because the planet is not perfectly spherical, and because it rotates, the actual free-fall acceleration g: of a particle near Earth differs slightly from the gravitational acceleration a:g, and the particle’s weight (equal to mg) differs from the magnitude of the gravitational force on it. ag !

360

CHAPTE R 13 G RAVITATION

Table 13-1 Variation of ag with Altitude Altitude (km) 0 8.8

9.83 9.80

36.6

9.71

400

8.70

35 700

Altitude Example

ag (m/s2)

0.225

Mean Earth surface Mt. Everest Highest crewed balloon Space shuttle orbit Communications satellite

Gravitation Near Earth’s Surface Let us assume that Earth is a uniform sphere of mass M. The magnitude of the gravitational force from Earth on a particle of mass m, located outside Earth a distance r from Earth’s center, is then given by Eq. 13-1 as Mm . (13-9) r2 If the particle is released, it will fall toward the center of Earth, as a result of the : gravitational force F , with an acceleration we shall call the gravitational accelera: tion a g. Newton’s second law tells us that magnitudes F and ag are related by F!G

F ! mag.

(13-10)

Now, substituting F from Eq. 13-9 into Eq. 13-10 and solving for ag, we find GM . (13-11) r2 Table 13-1 shows values of ag computed for various altitudes above Earth’s surface. Notice that ag is significant even at 400 km. Since Module 5-1, we have assumed that Earth is an inertial frame by neglecting its rotation. This simplification has allowed us to assume that the free-fall acceleration g of a particle is the same as the particle’s gravitational acceleration (which we now call ag). Furthermore, we assumed that g has the constant value 9.8 m/s2 any place on Earth’s surface. However, any g value measured at a given location will differ from the ag value calculated with Eq. 13-11 for that location for three reasons: (1) Earth’s mass is not distributed uniformly, (2) Earth is not a perfect sphere, and (3) Earth rotates. Moreover, because g differs from ag, the same three reasons mean that the measured weight mg of a particle differs from the magnitude of the gravitational force on the particle as given by Eq. 13-9. Let us now examine those reasons. ag !

12 10

Inner core

8

Outer core

Surface

Density (103 kg/m3)

14

6 4

Mantle

2 0

0

1

2

3

4

5

6

7

Distance from center (106 m)

Figure 13-5 The density of Earth as a function of distance from the center. The limits of the solid inner core, the largely liquid outer core, and the solid mantle are shown, but the crust of Earth is too thin to show clearly on this plot.

1. Earth’s mass is not uniformly distributed. The density (mass per unit volume) of Earth varies radially as shown in Fig. 13-5, and the density of the crust (outer section) varies from region to region over Earth’s surface. Thus, g varies from region to region over the surface. 2. Earth is not a sphere. Earth is approximately an ellipsoid, flattened at the poles and bulging at the equator. Its equatorial radius (from its center point out to the equator) is greater than its polar radius (from its center point out to either north or south pole) by 21 km. Thus, a point at the poles is closer to the dense core of Earth than is a point on the equator. This is one reason the free-fall acceleration g increases if you were to measure it while moving at sea level from the equator toward the north or south pole. As you move, you are actually getting closer to the center of Earth and thus, by Newton’s law of gravitation, g increases. 3. Earth is rotating. The rotation axis runs through the north and south poles of Earth. An object located on Earth’s surface anywhere except at those poles must rotate in a circle about the rotation axis and thus must have a centripetal acceleration directed toward the center of the circle. This centripetal acceleration requires a centripetal net force that is also directed toward that center. To see how Earth’s rotation causes g to differ from ag, let us analyze a simple situation in which a crate of mass m is on a scale at the equator. Figure 13-6a shows this situation as viewed from a point in space above the north pole. Figure 13-6b, a free-body diagram for the crate, shows the two forces on the crate, both acting along a radial r axis that extends from Earth’s center. The : normal force FN on the crate from the scale is directed outward, in the positive direction of the r axis. The gravitational force, represented with its equivalent a g, is directed inward. Because it travels in a circle about the center of Earth m:

13-3 G RAVITATION N EAR EARTH’S SU R FACE

Two forces act on this crate.

Crate Scale

R

North pole

The normal force is upward. Crate

r FN a

The gravitational force is downward.

mag

The net force is toward the center. So, the crate's acceleration is too.

(b)

(a)

Figure 13-6 (a) A crate sitting on a scale at Earth’s equator, as seen by an observer positioned on Earth’s rotation axis at some point above the north pole. (b) A free-body diagram for the crate, with a radial r axis extending from Earth’s center. The gravitational force on the crate is represented with its equivalent m a:g. The normal force on the crate : from the scale is F N. Because of Earth’s rotation, the crate has a centripetal acceleration a: that is directed toward Earth’s center.

a directed toward as Earth turns, the crate has a centripetal acceleration : Earth’s center. From Eq. 10-23 (ar ! v 2r), we know this acceleration is equal to v 2R, where v is Earth’s angular speed and R is the circle’s radius (approximately Earth’s radius). Thus, we can write Newton’s second law for forces along the r axis (Fnet,r ! mar) as FN % mag ! m(%v 2R).

(13-12)

The magnitude FN of the normal force is equal to the weight mg read on the scale. With mg substituted for FN, Eq. 13-12 gives us which says

mg ! mag % m(v 2R),

(13-13)

magnitude of mass times ! % . #measured weight $ #gravitational force$ #centripetal acceleration$ Thus, the measured weight is less than the magnitude of the gravitational force on the crate, because of Earth’s rotation. Acceleration Difference. To find a corresponding expression for g and ag, we cancel m from Eq. 13-13 to write g ! ag % v2R, (13-14) which says free-fall centripetal . % #acceleration $ ! #gravitational acceleration $ #acceleration$ Thus, the measured free-fall acceleration is less than the gravitational acceleration because of Earth’s rotation. Equator. The difference between accelerations g and ag is equal to v 2R and is greatest on the equator (for one reason, the radius of the circle traveled by the crate is greatest there). To find the difference, we can use Eq. 10-5 (v ! 'u/'t) and Earth’s radius R ! 6.37 $ 106 m. For one rotation of Earth, u is 2p rad and the time period 't is about 24 h. Using these values (and converting hours to seconds), we find that g is less than ag by only about 0.034 m/s2 (small compared to 9.8 m/s2). Therefore, neglecting the difference in accelerations g and ag is often justified. Similarly, neglecting the difference between weight and the magnitude of the gravitational force is also often justified.

361

362

CHAPTE R 13 G RAVITATION

Sample Problem 13.02 Difference in acceleration at head and feet (a) An astronaut whose height h is 1.70 m floats “feet down” in an orbiting space shuttle at distance r ! 6.77 $ 106 m away from the center of Earth. What is the difference between the gravitational acceleration at her feet and at her head? KEY IDEAS We can approximate Earth as a uniform sphere of mass ME. Then, from Eq. 13-11, the gravitational acceleration at any distance r from the center of Earth is GME (13-15) . r2 We might simply apply this equation twice, first with r ! 6.77 $ 106 m for the location of the feet and then with r ! 6.77 $ 106 m # 1.70 m for the location of the head. However, a calculator may give us the same value for ag twice, and thus a difference of zero, because h is so much smaller than r. Here’s a more promising approach: Because we have a differential change dr in r between the astronaut’s feet and head, we should differentiate Eq. 13-15 with respect to r. ag !

Calculations: The differentiation gives us GME (13-16) dag ! %2 dr, r3 where dag is the differential change in the gravitational acceleration due to the differential change dr in r. For the astronaut, dr ! h and r ! 6.77 $ 106 m. Substituting data into Eq. 13-16, we find dag ! %2

(6.67 $ 10%11 m3/ kg "s2)(5.98 $ 1024 kg) (1.70 m) (6.77 $ 106 m)3

(Answer)

! %4.37 $ 10%6 m/s2,

where the ME value is taken from Appendix C. This result means that the gravitational acceleration of the astronaut’s feet toward Earth is slightly greater than the gravitational acceleration of her head toward Earth. This difference in acceleration (often called a tidal effect) tends to stretch her body, but the difference is so small that she would never even sense the stretching, much less suffer pain from it. (b) If the astronaut is now “feet down” at the same orbital radius r ! 6.77 $ 106 m about a black hole of mass Mh ! 1.99 $ 1031 kg (10 times our Sun’s mass), what is the difference between the gravitational acceleration at her feet and at her head? The black hole has a mathematical surface (event horizon) of radius Rh ! 2.95 $ 104 m. Nothing, not even light, can escape from that surface or anywhere inside it. Note that the astronaut is well outside the surface (at r ! 229Rh). Calculations: We again have a differential change dr in r between the astronaut’s feet and head, so we can again use Eq. 13-16. However, now we substitute Mh ! 1.99 $ 1031 kg for ME. We find dag ! %2

(6.67 $ 10%11 m3/ kg "s2)(1.99 $ 1031 kg) (1.70 m) (6.77 $ 106 m)3

(Answer)

! %14.5 m/s2.

This means that the gravitational acceleration of the astronaut’s feet toward the black hole is noticeably larger than that of her head. The resulting tendency to stretch her body would be bearable but quite painful. If she drifted closer to the black hole, the stretching tendency would increase drastically.

Additional examples, video, and practice available at WileyPLUS

13-4 GRAVITATION INSIDE EARTH Learning Objectives After reading this module, you should be able to . . .

13.09 Identify that a uniform shell of matter exerts no net gravitational force on a particle located inside it.

13.10 Calculate the gravitational force that is exerted on a particle at a given radius inside a nonrotating uniform sphere of matter.

Key Ideas ● A uniform shell of matter exerts no net gravitational force on a particle located inside it. ● The gravitational force

:

F on a particle inside a uniform solid sphere, at a distance r from the center, is due only to mass Mins in an “inside sphere” with that radius r: M ins ! 43p r 3r !

M 3 r, R3

where r is the solid sphere’s density, R is its radius, and M is its mass. We can assign this inside mass to be that of a particle at the center of the solid sphere and then apply Newton’s law of gravitation for particles. We find that the magnitude of the force acting on mass m is F!

GmM r. R3

13-4 G RAVITATION I NSI DE EARTH

363

Gravitation Inside Earth Newton’s shell theorem can also be applied to a situation in which a particle is located inside a uniform shell, to show the following: A uniform shell of matter exerts no net gravitational force on a particle located inside it.

Caution: This statement does not mean that the gravitational forces on the particle from the various elements of the shell magically disappear. Rather, it means that the sum of the force vectors on the particle from all the elements is zero. If Earth’s mass were uniformly distributed, the gravitational force acting on a particle would be a maximum at Earth’s surface and would decrease as the particle moved outward, away from the planet. If the particle were to move inward, perhaps down a deep mine shaft, the gravitational force would change for two reasons. (1) It would tend to increase because the particle would be moving closer to the center of Earth. (2) It would tend to decrease because the thickening shell of material lying outside the particle’s radial position would not exert any net force on the particle. To find an expression for the gravitational force inside a uniform Earth, let’s use the plot in Pole to Pole, an early science fiction story by George Griffith. Three explorers attempt to travel by capsule through a naturally formed (and, of course, fictional) tunnel directly from the south pole to the north pole. Figure 13-7 shows the capsule (mass m) when it has fallen to a distance r from Earth’s center. At that moment, the net gravitational force on the capsule is due to the mass Mins inside the sphere with radius r (the mass enclosed by the dashed outline), not the mass in the outer spherical shell (outside the dashed outline). Moreover, we can assume that the inside mass Mins is concentrated as a particle at Earth’s center. Thus, we can write Eq. 13-1, for the magnitude of the gravitational force on the capsule, as F!

GmMins . r2

(13-17)

Because we assume a uniform density r, we can write this inside mass in terms of Earth’s total mass M and its radius R: density ! r!

total mass inside mass ! , inside volume total volume Mins 4 3 3 pr

!

M 4 3 3 pR

.

Solving for Mins we find Mins ! 43pr3r !

M 3 r. R3

(13-18)

Substituting the second expression for Mins into Eq. 13-17 gives us the magnitude of the gravitational force on the capsule as a function of the capsule’s distance r from Earth’s center: F!

GmM r. R3

(13-19)

According to Griffith’s story, as the capsule approaches Earth’s center, the gravitational force on the explorers becomes alarmingly large and, exactly at the center, it suddenly but only momentarily disappears. From Eq. 13-19 we see that, in fact, the force magnitude decreases linearly as the capsule approaches the center, until it is zero at the center.At least Griffith got the zero-at-the-center detail correct.

Mins r

m

Figure 13-7 A capsule of mass m falls from rest through a tunnel that connects Earth’s south and north poles.When the capsule is at distance r from Earth’s center, the portion of Earth’s mass that is contained in a sphere of that radius is Mins.

364

CHAPTE R 13 G RAVITATION :

Equation 13-19 can also be written in terms of the force vector F and the capsule’s position vector :r along a radial axis extending from Earth’s center. Letting K represent the collection of constants in Eq. 13-19, we can rewrite the force in vector form as :

F ! %K :r ,

(13-20) :

in which we have inserted a minus sign to indicate that F and :r have opposite : : directions. Equation 13-20 has the form of Hooke’s law (Eq. 7-20, F ! %k d ). Thus, under the idealized conditions of the story, the capsule would oscillate like a block on a spring, with the center of the oscillation at Earth’s center. After the capsule had fallen from the south pole to Earth’s center, it would travel from the center to the north pole (as Griffith said) and then back again, repeating the cycle forever. For the real Earth, which certainly has a nonuniform distribution of mass (Fig. 13-5), the force on the capsule would initially increase as the capsule descends. The force would then reach a maximum at a certain depth, and only then would it begin to decrease as the capsule further descends.

13-5 GRAVITATIONAL POTENTIAL ENERGY Learning Objectives After reading this module, you should be able to . . .

13.11 Calculate the gravitational potential energy of a system of particles (or uniform spheres that can be treated as particles). 13.12 Identify that if a particle moves from an initial point to a final point while experiencing a gravitational force, the work done by that force (and thus the change in gravitational potential energy) is independent of which path is taken. 13.13 Using the gravitational force on a particle near an astronomical body (or some second body that is fixed in

place), calculate the work done by the force when the body moves. 13.14 Apply the conservation of mechanical energy (including gravitational potential energy) to a particle moving relative to an astronomical body (or some second body that is fixed in place). 13.15 Explain the energy requirements for a particle to escape from an astronomical body (usually assumed to be a uniform sphere). 13.16 Calculate the escape speed of a particle in leaving an astronomical body.

Key Ideas ● The gravitational potential energy U(r) of a system of two particles, with masses M and m and separated by a distance r, is the negative of the work that would be done by the gravitational force of either particle acting on the other if the separation between the particles were changed from infinite (very large) to r. This energy is GMm U!% (gravitational potential energy). r ● If a system contains more than two particles, its total gravitational potential energy U is the sum of the terms rep-

resenting the potential energies of all the pairs. As an example, for three particles, of masses m1, m2, and m3,

# Gmr m

$

Gm1m3 Gm2m3 # . r r23 12 13 ● An object will escape the gravitational pull of an astronomical body of mass M and radius R (that is, it will reach an infinite distance) if the object’s speed near the body’s surface is at least equal to the escape speed, given by 2GM v! . A R U!%

1

2

#

Gravitational Potential Energy In Module 8-1, we discussed the gravitational potential energy of a particle – Earth system. We were careful to keep the particle near Earth’s surface, so that we could regard the gravitational force as constant. We then chose some reference configuration of the system as having a gravitational potential energy of zero. Often, in this configuration the particle was on Earth’s surface. For particles not

13-5 G RAVITATIONAL POTE NTIAL E N E RGY

on Earth’s surface, the gravitational potential energy decreased when the separation between the particle and Earth decreased. Here, we broaden our view and consider the gravitational potential energy U of two particles, of masses m and M, separated by a distance r. We again choose a reference configuration with U equal to zero. However, to simplify the equations, the separation distance r in the reference configuration is now large enough to be approximated as infinite. As before, the gravitational potential energy decreases when the separation decreases. Since U ! 0 for r ! 2, the potential energy is negative for any finite separation and becomes progressively more negative as the particles move closer together. With these facts in mind and as we shall justify next, we take the gravitational potential energy of the two-particle system to be U!%

GMm r

(gravitational potential energy).

r13

#

$

r23 r12

(13-21)

(13-22)

r dr

F

Let us shoot a baseball directly away from Earth along the path in Fig. 13-9. We want to find an expression for the gravitational potential energy U of the ball at point P along its path, at radial distance R from Earth’s center. To do so, we first find the work W done on the ball by the gravitational force as the ball travels from point P to a great (infinite) distance from Earth. Because the gravitational : force F (r) is a variable force (its magnitude depends on r), we must use the techniques of Module 7-5 to find the work. In vector notation, we can write

"

2

:

Work is done as the baseball moves upward.

P R

M

(13-23)

F(r) " d r:.

R :

The integral contains the scalar (or dot) product of the force F (r) and the differential displacement vector d : r along the ball’s path.We can expand that product as :

F (r) "d : r ! F(r) dr cos f, :

m2

Figure 13-8 A system consisting of three particles. The gravitational potential energy of the system is the sum of the gravitational potential energies of all three pairs of particles.

Proof of Equation 13-21

W!

Here too.

Here too.

Note that U(r) approaches zero as r approaches infinity and that for any finite value of r, the value of U(r) is negative. Language. The potential energy given by Eq. 13-21 is a property of the system of two particles rather than of either particle alone. There is no way to divide this energy and say that so much belongs to one particle and so much to the other. However, if M 3 m, as is true for Earth (mass M) and a baseball (mass m), we often speak of “the potential energy of the baseball.” We can get away with this because, when a baseball moves in the vicinity of Earth, changes in the potential energy of the baseball – Earth system appear almost entirely as changes in the kinetic energy of the baseball, since changes in the kinetic energy of Earth are too small to be measured. Similarly, in Module 13-7 we shall speak of “the potential energy of an artificial satellite” orbiting Earth, because the satellite’s mass is so much smaller than Earth’s mass. When we speak of the potential energy of bodies of comparable mass, however, we have to be careful to treat them as a system. Multiple Particles. If our system contains more than two particles, we consider each pair of particles in turn, calculate the gravitational potential energy of that pair with Eq. 13-21 as if the other particles were not there, and then algebraically sum the results. Applying Eq. 13-21 to each of the three pairs of Fig. 13-8, for example, gives the potential energy of the system as Gm1m2 Gm1m3 Gm2m3 . U!% # # r12 r13 r23

m3

This pair has potential energy.

m1

365

(13-24)

where f is the angle between the directions of F (r) and d r . When we substitute :

Figure 13-9 A baseball is shot directly away from Earth, through point P at radial distance R from Earth’s center. The gravita: tional force F on the ball and a differential displacement vector d :r are shown, both directed along a radial r axis.

366

CHAPTE R 13 G RAVITATION

180° for f and Eq. 13-1 for F(r), Eq. 13-24 becomes

F

G

:

E

F(r) " dr: ! %

D

GMm dr, r2

where M is Earth’s mass and m is the mass of the ball. Substituting this into Eq. 13-23 and integrating give us B

C

A

W ! %GMm Actual path from A to G is irrelevant.

Earth

Figure 13-10 Near Earth, a baseball is moved from point A to point G along a path consisting of radial lengths and circular arcs.

!0%

"

2

R

1 dr ! r2

(

GMm r

)

2 R

GMm GMm !% , R R

(13-25)

where W is the work required to move the ball from point P (at distance R) to infinity. Equation 8-1 ('U ! %W) tells us that we can also write that work in terms of potential energies as U2 % U ! %W. Because the potential energy U2 at infinity is zero, U is the potential energy at P, and W is given by Eq. 13-25, this equation becomes GMm . R Switching R to r gives us Eq. 13-21, which we set out to prove. U!W!%

Path Independence In Fig. 13-10, we move a baseball from point A to point G along a path consisting of three radial lengths and three circular arcs (centered on Earth). We are inter: ested in the total work W done by Earth’s gravitational force F on the ball as it moves from A to G. The work done along each circular arc is zero, because the : direction of F is perpendicular to the arc at every point. Thus, W is the sum of : only the works done by F along the three radial lengths. Now, suppose we mentally shrink the arcs to zero. We would then be moving the ball directly from A to G along a single radial length. Does that change W? No. Because no work was done along the arcs, eliminating them does not change the work. The path taken from A to G now is clearly different, but the work done : by F is the same. We discussed such a result in a general way in Module 8-1. Here is the point: The gravitational force is a conservative force. Thus, the work done by the gravitational force on a particle moving from an initial point i to a final point f is independent of the path taken between the points. From Eq. 8-1, the change 'U in the gravitational potential energy from point i to point f is given by 'U ! Uf % Ui ! %W.

(13-26)

Since the work W done by a conservative force is independent of the actual path taken, the change 'U in gravitational potential energy is also independent of the path taken.

Potential Energy and Force In the proof of Eq. 13-21, we derived the potential energy function U(r) from the : force function F (r). We should be able to go the other way — that is, to start from the potential energy function and derive the force function. Guided by Eq. 8-22 (F(x) ! %dU(x)/dx), we can write d dU !% dr dr GMm !% . r2

F!%

#% GMm r $ (13-27)

13-5 G RAVITATIONAL POTE NTIAL E N E RGY

This is Newton’s law of gravitation (Eq. 13-1). The minus sign indicates that the force on mass m points radially inward, toward mass M.

Escape Speed If you fire a projectile upward, usually it will slow, stop momentarily, and return to Earth. There is, however, a certain minimum initial speed that will cause it to move upward forever, theoretically coming to rest only at infinity. This minimum initial speed is called the (Earth) escape speed. Consider a projectile of mass m, leaving the surface of a planet (or some other astronomical body or system) with escape speed v. The projectile has a kinetic energy K given by 12 mv2 and a potential energy U given by Eq. 13-21: U!%

GMm , R

in which M is the mass of the planet and R is its radius. When the projectile reaches infinity, it stops and thus has no kinetic energy. It also has no potential energy because an infinite separation between two bodies is our zero-potential-energy configuration. Its total energy at infinity is therefore zero. From the principle of conservation of energy, its total energy at the planet’s surface must also have been zero, and so

#

K # U ! 12 mv2 # % This yields

v!

GMm R

$ ! 0.

2GM . A R

(13-28)

Note that v does not depend on the direction in which a projectile is fired from a planet. However, attaining that speed is easier if the projectile is fired in the direction the launch site is moving as the planet rotates about its axis. For example, rockets are launched eastward at Cape Canaveral to take advantage of the Cape’s eastward speed of 1500 km/h due to Earth’s rotation. Equation 13-28 can be applied to find the escape speed of a projectile from any astronomical body, provided we substitute the mass of the body for M and the radius of the body for R. Table 13-2 shows some escape speeds. Table 13-2 Some Escape Speeds Body

Mass (kg)

Radius (m)

Ceresa Earth’s moona Earth Jupiter Sun Sirius Bb Neutron starc

1.17 $ 1021 7.36 $ 1022 5.98 $ 1024 1.90 $ 1027 1.99 $ 1030 2 $ 1030 2 $ 1030

3.8 $ 105 1.74 $ 106 6.37 $ 106 7.15 $ 107 6.96 $ 108 1 $ 107 1 $ 104

Escape Speed (km/s) 0.64 2.38 11.2 59.5 618 5200 2 $ 105

a

The most massive of the asteroids. A white dwarf (a star in a final stage of evolution) that is a companion of the bright star Sirius. c The collapsed core of a star that remains after that star has exploded in a supernova event. b

Checkpoint 3 You move a ball of mass m away from a sphere of mass M. (a) Does the gravitational potential energy of the system of ball and sphere increase or decrease? (b) Is positive work or negative work done by the gravitational force between the ball and the sphere?

367

368

CHAPTE R 13 G RAVITATION

Sample Problem 13.03 Asteroid falling from space, mechanical energy An asteroid, headed directly toward Earth, has a speed of 12 km/s relative to the planet when the asteroid is 10 Earth radii from Earth’s center. Neglecting the effects of Earth’s atmosphere on the asteroid, find the asteroid’s speed vf when it reaches Earth’s surface. KEY IDEAS

tially at distance 10RE and finally at distance RE, where RE is Earth’s radius (6.37 $ 106 m). Substituting Eq. 13-21 for U and 12 mv2 for K, we rewrite Eq. 13-29 as 1 2 2 mv f

%

GMm GMm ! 12 mv 2i % . RE 10RE

Rearranging and substituting known values, we find

Because we are to neglect the effects of the atmosphere on the asteroid, the mechanical energy of the asteroid – Earth system is conserved during the fall. Thus, the final mechanical energy (when the asteroid reaches Earth’s surface) is equal to the initial mechanical energy. With kinetic energy K and gravitational potential energy U, we can write this as Kf # Uf ! Ki # Ui.

v2f ! v2i #

Calculations: Let m represent the asteroid’s mass and M represent Earth’s mass (5.98 $ 1024 kg). The asteroid is ini-

#1 % 101 $

! (12 $ 103 m/s)2 #

(13-29)

Also, if we assume the system is isolated, the system’s linear momentum must be conserved during the fall. Therefore, the momentum change of the asteroid and that of Earth must be equal in magnitude and opposite in sign. However, because Earth’s mass is so much greater than the asteroid’s mass, the change in Earth’s speed is negligible relative to the change in the asteroid’s speed. So, the change in Earth’s kinetic energy is also negligible.Thus, we can assume that the kinetic energies in Eq. 13-29 are those of the asteroid alone.

2GM RE

2(6.67 $ 10 %11 m3/kg "s2)(5.98 $ 1024 kg) 0.9 6.37 $ 106 m

! 2.567 $ 108 m2/s2, and

vf ! 1.60 $ 104 m/s ! 16 km/s.

(Answer)

At this speed, the asteroid would not have to be particularly large to do considerable damage at impact. If it were only 5 m across, the impact could release about as much energy as the nuclear explosion at Hiroshima. Alarmingly, about 500 million asteroids of this size are near Earth’s orbit, and in 1994 one of them apparently penetrated Earth’s atmosphere and exploded 20 km above the South Pacific (setting off nuclear-explosion warnings on six military satellites).

Additional examples, video, and practice available at WileyPLUS

13-6 PLANETS AND SATELLITES: KEPLER’S LAWS Learning Objectives After reading this module, you should be able to . . .

13.17 Identify Kepler’s three laws. 13.18 Identify which of Kepler’s laws is equivalent to the law of conservation of angular momentum. 13.19 On a sketch of an elliptical orbit, identify the semimajor axis, the eccentricity, the perihelion, the aphelion, and the focal points.

13.20 For an elliptical orbit, apply the relationships between the semimajor axis, the eccentricity, the perihelion, and the aphelion. 13.21 For an orbiting natural or artificial satellite, apply Kepler’s relationship between the orbital period and radius and the mass of the astronomical body being orbited.

Key Ideas ● The motion of satellites, both natural and artificial, is governed by Kepler’s laws: 1. The law of orbits. All planets move in elliptical orbits with the Sun at one focus. 2. The law of areas. A line joining any planet to the Sun sweeps out equal areas in equal time intervals. (This statement is equivalent to conservation of angular momentum.)

3. The law of periods. The square of the period T of any planet is proportional to the cube of the semimajor axis a of its orbit. For circular orbits with radius r, 44 2 3 r (law of periods), T2 ! GM where M is the mass of the attracting body—the Sun in the case of the solar system. For elliptical planetary orbits, the semimajor axis a is substituted for r.

#

$

369

13-6 PL AN ETS AN D SATE LLITES: KE PLE R’S L AWS

Planets and Satellites: Kepler’s Laws The motions of the planets, as they seemingly wander against the background of the stars, have been a puzzle since the dawn of history. The “loop-the-loop” motion of Mars, shown in Fig. 13-11, was particularly baffling. Johannes Kepler (1571 – 1630), after a lifetime of study, worked out the empirical laws that govern these motions. Tycho Brahe (1546 – 1601), the last of the great astronomers to make observations without the help of a telescope, compiled the extensive data from which Kepler was able to derive the three laws of planetary motion that now bear Kepler’s name. Later, Newton (1642 – 1727) showed that his law of gravitation leads to Kepler’s laws. In this section we discuss each of Kepler’s three laws. Although here we apply the laws to planets orbiting the Sun, they hold equally well for satellites, either natural or artificial, orbiting Earth or any other massive central body. 1. THE LAW OF ORBITS:All planets move in elliptical orbits,with the Sun at one focus.

Figure 13-12 shows a planet of mass m moving in such an orbit around the Sun, whose mass is M. We assume that M 3 m, so that the center of mass of the planet – Sun system is approximately at the center of the Sun. The orbit in Fig. 13-12 is described by giving its semimajor axis a and its eccentricity e, the latter defined so that ea is the distance from the center of the ellipse to either focus F or F9. An eccentricity of zero corresponds to a circle, in which the two foci merge to a single central point. The eccentricities of the planetary orbits are not large; so if the orbits are drawn to scale, they look circular. The eccentricity of the ellipse of Fig. 13-12, which has been exaggerated for clarity, is 0.74. The eccentricity of Earth’s orbit is only 0.0167. 2. THE LAW OF AREAS: A line that connects a planet to the Sun sweeps out equal areas in the plane of the planet’s orbit in equal time intervals; that is, the rate dA/dt at which it sweeps out area A is constant.

Qualitatively, this second law tells us that the planet will move most slowly when it is farthest from the Sun and most rapidly when it is nearest to the Sun. As it turns out, Kepler’s second law is totally equivalent to the law of conservation of angular momentum. Let us prove it. The area of the shaded wedge in Fig. 13-13a closely approximates the area swept out in time 't by a line connecting the Sun and the planet, which are separated by distance r. The area 'A of the wedge is approximately the area of

The planet sweeps out this area. r ∆θ ∆θ Sun M

(a)

θ

r

These are the two momentum components.

∆A Sun

p⊥

p

r

pr

θ

M

(b)

Figure 13-13 (a) In time 't, the line r connecting the planet to the Sun moves through an : angle 'u, sweeping out an area 'A (shaded). (b) The linear momentum p of the planet : and the components of p .

October 14 June 6 July 26

September 4

Figure 13-11 The path seen from Earth for the planet Mars as it moved against a background of the constellation Capricorn during 1971.The planet’s position on four days is marked. Both Mars and Earth are moving in orbits around the Sun so that we see the position of Mars relative to us; this relative motion sometimes results in an apparent loop in the path of Mars.

Ra

Rp

m r M

θ

F

ea

a

ea

F'

The Sun is at one of the two focal points.

Figure 13-12 A planet of mass m moving in an elliptical orbit around the Sun. The Sun, of mass M, is at one focus F of the ellipse. The other focus is F9, which is located in empty space. The semimajor axis a of the ellipse, the perihelion (nearest the Sun) distance Rp, and the aphelion (farthest from the Sun) distance Ra are also shown.

370

CHAPTE R 13 G RAVITATION

a triangle with base r 'u and height r. Since the area of a triangle is one-half of the base times the height, 'A % 12 r 2'u. This expression for 'A becomes more exact as 't (hence 'u) approaches zero. The instantaneous rate at which area is being swept out is then dA du (13-30) ! 12 r 2 ! 12 r 2v, dt dt in which v is the angular speed of the line connecting Sun and planet, as the line rotates around the Sun. : Figure 13-13b shows the linear momentum p of the planet, along with the radial : and perpendicular components of p . From Eq. 11-20 (L ! rp"), the magnitude of : the angular momentum L of the planet about the Sun is given by the product of r : and p", the component of p perpendicular to r. Here, for a planet of mass m, L ! rp" ! (r)(mv") ! (r)(mvr) ! mr 2v,

(13-31)

where we have replaced v" with its equivalent vr (Eq. 10-18). Eliminating r2v between Eqs. 13-30 and 13-31 leads to dA L ! . dt 2m

(13-32)

If dA/dt is constant, as Kepler said it is, then Eq. 13-32 means that L must also be constant — angular momentum is conserved. Kepler’s second law is indeed equivalent to the law of conservation of angular momentum. 3. THE LAW OF PERIODS: The square of the period of any planet is proportional to the cube of the semimajor axis of its orbit.

m r

To see this, consider the circular orbit of Fig. 13-14, with radius r (the radius of a circle is equivalent to the semimajor axis of an ellipse). Applying Newton’s second law (F ! ma) to the orbiting planet in Fig. 13-14 yields

θ M

Figure 13-14 A planet of mass m moving around the Sun in a circular orbit of radius r.

Table 13-3 Kepler’s Law of Periods for the Solar System

Planet Mercury Venus Earth Mars Jupiter Saturn Uranus Neptune Pluto

Semimajor Axis a (1010 m) 5.79 10.8 15.0 22.8 77.8 143 287 450 590

Period T (y) 0.241 0.615 1.00 1.88 11.9 29.5 84.0 165 248

T 2/a3 (10%34 y2/m3) 2.99 3.00 2.96 2.98 3.01 2.98 2.98 2.99 2.99

GMm ! (m)(v 2r). r2

(13-33)

Here we have substituted from Eq. 13-1 for the force magnitude F and used Eq. 10-23 to substitute v2r for the centripetal acceleration. If we now use Eq. 10-20 to replace v with 2p/T, where T is the period of the motion, we obtain Kepler’s third law: T2 !

4p # GM $r 2

3

(law of periods).

(13-34)

The quantity in parentheses is a constant that depends only on the mass M of the central body about which the planet orbits. Equation 13-34 holds also for elliptical orbits, provided we replace r with a, the semimajor axis of the ellipse. This law predicts that the ratio T 2/a3 has essentially the same value for every planetary orbit around a given massive body. Table 13-3 shows how well it holds for the orbits of the planets of the solar system.

Checkpoint 4 Satellite 1 is in a certain circular orbit around a planet, while satellite 2 is in a larger circular orbit. Which satellite has (a) the longer period and (b) the greater speed?

13-7 SATE LLITES: OR B ITS AN D E N E RGY

371

Sample Problem 13.04 Kepler’s law of periods, Comet Halley Comet Halley orbits the Sun with a period of 76 years and, in 1986, had a distance of closest approach to the Sun, its perihelion distance Rp, of 8.9 $ 1010 m. Table 13-3 shows that this is between the orbits of Mercury and Venus.

Ra ! 2a % Rp ! (2)(2.7 $ 1012 m) % 8.9 $ 1010 m ! 5.3 $ 1012 m.

(Answer)

(a) What is the comet’s farthest distance from the Sun, which is called its aphelion distance Ra?

Table 13-3 shows that this is a little less than the semimajor axis of the orbit of Pluto. Thus, the comet does not get farther from the Sun than Pluto.

KEY IDEAS

(b) What is the eccentricity e of the orbit of comet Halley?

From Fig. 13-12, we see that Ra # Rp ! 2a, where a is the semimajor axis of the orbit. Thus, we can find Ra if we first find a. We can relate a to the given period via the law of periods (Eq. 13-34) if we simply substitute the semimajor axis a for r.

KEY IDEA

Calculations: Making that substitution and then solving for a, we have

Calculation: We have

a!

#

GMT 2 4p 2

$

1/3

.

(13-35)

If we substitute the mass M of the Sun, 1.99 $ 1030 kg, and the period T of the comet, 76 years or 2.4 $ 109 s, into Eq. 13-35, we find that a ! 2.7 $ 1012 m. Now we have

We can relate e, a, and Rp via Fig. 13-12, in which we see that ea ! a % Rp. a % Rp Rp !1% a a 10 8.9 $ 10 m !1% ! 0.97. 2.7 $ 1012 m

e!

(13-36) (Answer)

This tells us that, with an eccentricity approaching unity, this orbit must be a long thin ellipse.

Additional examples, video, and practice available at WileyPLUS

13-7 SATELLITES: ORBITS AND ENERGY Learning Objectives After reading this module, you should be able to . . .

13.22 For a satellite in a circular orbit around an astronomical body, calculate the gravitational potential energy, the kinetic energy, and the total energy.

13.23 For a satellite in an elliptical orbit, calculate the total energy.

Key Ideas ● When a planet or satellite with mass m moves in a circular orbit with radius r, its potential energy U and kinetic energy K are given by GMm GMm U!% and K ! . r 2r

The mechanical energy E ! K # U is then GMm E!% . 2r For an elliptical orbit of semimajor axis a, GMm E!% . 2a

Satellites: Orbits and Energy As a satellite orbits Earth in an elliptical path, both its speed, which fixes its kinetic energy K, and its distance from the center of Earth, which fixes its gravitational potential energy U, fluctuate with fixed periods. However, the mechanical energy E of the satellite remains constant. (Since the satellite’s mass is so much smaller than Earth’s mass, we assign U and E for the Earth–satellite system to the satellite alone.)

372

CHAPTE R 13 G RAVITATION e=0

The potential energy of the system is given by Eq. 13-21:

0.5

U!%

0.8 0.9

M

Figure 13-15 Four orbits with different eccentricities e about an object of mass M. All four orbits have the same semimajor axis a and thus correspond to the same total mechanical energy E.

GMm r

(with U ! 0 for infinite separation). Here r is the radius of the satellite’s orbit, assumed for the time being to be circular, and M and m are the masses of Earth and the satellite, respectively. To find the kinetic energy of a satellite in a circular orbit, we write Newton’s second law (F ! ma) as GMm v2 (13-37) !m , 2 r r where v2/r is the centripetal acceleration of the satellite. Then, from Eq. 13-37, the kinetic energy is GMm (13-38) K ! 12 mv2 ! , 2r which shows us that for a satellite in a circular orbit, U (circular orbit). 2 The total mechanical energy of the orbiting satellite is

(13-39)

K!%

This is a plot of a satellite's energies versus orbit radius.

E!K#U!

Energy

or K(r)

The kinetic energy is positive. r

0 E(r) U(r)

E!%

(13-40)

(circular orbit).

This tells us that for a satellite in a circular orbit, the total energy E is the negative of the kinetic energy K: E ! %K (circular orbit). (13-41) For a satellite in an elliptical orbit of semimajor axis a, we can substitute a for r in Eq. 13-40 to find the mechanical energy:

The potential energy and total energy are negative.

Figure 13-16 The variation of kinetic energy K, potential energy U, and total energy E with radius r for a satellite in a circular orbit. For any value of r, the values of U and E are negative, the value of K is positive, and E ! %K. As r : 2, all three energy curves approach a value of zero.

GMm 2r

GMm GMm % 2r r

E!%

GMm 2a

(13-42)

(elliptical orbit).

Equation 13-42 tells us that the total energy of an orbiting satellite depends only on the semimajor axis of its orbit and not on its eccentricity e. For example, four orbits with the same semimajor axis are shown in Fig. 13-15; the same satellite would have the same total mechanical energy E in all four orbits. Figure 13-16 shows the variation of K, U, and E with r for a satellite moving in a circular orbit about a massive central body. Note that as r is increased, the kinetic energy (and thus also the orbital speed) decreases.

Checkpoint 5 In the figure here, a space shuttle is initially in a circular orbit of radius r about Earth. At point P, the pilot briefly fires a forward-pointing thruster to decrease the shuttle’s kinetic energy K and mechanical energy E. (a) Which of the dashed elliptical orbits shown in the figure will the shuttle then take? (b) Is the orbital period T of the shuttle (the time to return to P) then greater than, less than, or the same as in the circular orbit?

2

1

P r

373

13-7 SATE LLITES: OR B ITS AN D E N E RGY

Sample Problem 13.05 Mechanical energy of orbiting bowling ball A playful astronaut releases a bowling ball, of mass m ! 7.20 kg, into circular orbit about Earth at an altitude h of 350 km.

KEY IDEA

(a) What is the mechanical energy E of the ball in its orbit?

On the launchpad, the ball is not in orbit and thus Eq. 13-40 does not apply. Instead, we must find E0 ! K0 # U0, where K0 is the ball’s kinetic energy and U0 is the gravitational potential energy of the ball – Earth system.

KEY IDEA

Calculations: To find U0, we use Eq. 13-21 to write

We can get E from the orbital energy, given by Eq. 13-40 (E ! %GMm/2r), if we first find the orbital radius r. (It is not simply the given altitude.) Calculations: The orbital radius must be r ! R # h ! 6370 km # 350 km ! 6.72 $ 106 m, in which R is the radius of Earth. Then, from Eq. 13-40 with Earth mass M ! 5.98 $ 1024 kg, the mechanical energy is GMm 2r (6.67 $ 10%11 N"m2/kg2)(5.98 $ 1024 kg)(7.20 kg) !% (2)(6.72 $ 106 m) 8 ! %2.14 $ 10 J ! %214 MJ. (Answer)

E!%

(b) What is the mechanical energy E0 of the ball on the launchpad at the Kennedy Space Center (before launch)? From there to the orbit, what is the change 'E in the ball’s mechanical energy?

GMm R (6.67 $ 10%11 N"m2/kg2)(5.98 $ 1024 kg)(7.20 kg) !% 6.37 $ 106 m ! %4.51 $ 108 J ! %451 MJ.

U0 ! %

The kinetic energy K0 of the ball is due to the ball’s motion with Earth’s rotation. You can show that K0 is less than 1 MJ, which is negligible relative to U0. Thus, the mechanical energy of the ball on the launchpad is E0 ! K0 # U0 % 0 % 451 MJ ! %451 MJ.

(Answer)

The increase in the mechanical energy of the ball from launchpad to orbit is 'E ! E % E0 ! (%214 MJ) % (%451 MJ) ! 237 MJ.

(Answer)

This is worth a few dollars at your utility company. Obviously the high cost of placing objects into orbit is not due to their required mechanical energy.

Sample Problem 13.06 Transforming a circular orbit into an elliptical orbit A spaceship of mass m ! 4.50 $ 103 kg is in a circular Earth orbit of radius r ! 8.00 $ 106 m and period T0 ! 118.6 min ! 7.119 $ 103 s when a thruster is fired in the forward direction to decrease the speed to 96.0% of the original speed. What is the period T of the resulting elliptical orbit (Fig. 13-17)? KEY IDEAS (1) The orbit of an elliptical orbit is related to the semimajor axis a by Kepler’s third law, written as Eq. 13-34 (T 2 ! 4p2r3/GM) but with a replacing r. (2) The semimajor axis a is related to the total mechanical energy E of the ship by Eq. 13-42 (E ! %GMm/2a), in which Earth’s mass is M ! 5.98 $ 1024 kg. (3) The potential energy of the ship at radius r from Earth’s center is given by Eq. 13-21 (U ! %GMm/r). Calculations: Looking over the Key Ideas, we see that we need to calculate the total energy E to find the semimajor axis a, so that we can then determine the period of the elliptical orbit. Let’s start with the kinetic energy, calculating it just after the thruster is fired. The speed v just then is 96% of the initial speed v0, which was equal to the ratio of the circumfer-

P M

r

Figure 13-17 At point P a thruster is fired, changing a ship’s orbit from circular to elliptical.

ence of the initial circular orbit to the initial period of the orbit. Thus, just after the thruster is fired, the kinetic energy is

# 2pr T $

K ! 12mv 2 ! 12m(0.96v0)2 ! 12m(0.96)2 ! 12(4.50 $ 103 kg)(0.96)2 ! 1.0338 $ 1011 J.

#

2

0

2p (8.00 $ 106 m) 7.119 $ 103 s

$

2

374

CHAPTE R 13 G RAVITATION

Just after the thruster is fired, the ship is still at orbital radius r, and thus its gravitational potential energy is

OK, one more step to go. We substitute a for r in Eq. 13-34 and then solve for the period T, substituting our result for a:

GMm r (6.67 $ 10 %11 N"m2/kg 2)(5.98 $ 1024 kg)(4.50 $ 103 kg) !% 8.00 $ 106 m ! %2.2436 $ 1011 J.

# $ 4p (7.418 $ 10 m) !# (6.67 $ 10 N"m /kg )(5.98 $ 10

U!%

We can now find the semimajor axis by rearranging Eq. 13-42, substituting a for r, and then substituting in our energy results: GMm GMm !% a!% 2E 2(K # U) !%

(6.67 $ 10 %11 N"m2/kg 2)(5.98 $ 1024 kg)(4.50 $ 103 kg) 2(1.0338 $ 1011 J % 2.2436 $ 1011 J)

! 7.418 $ 106 m.

T!

4p2a 3 GM

1/2

2

6

2

%11

3

2

! 6.356 $ 103 s ! 106 min.

$

1/2

24

kg)

(Answer)

This is the period of the elliptical orbit that the ship takes after the thruster is fired. It is less than the period T0 for the circular orbit for two reasons. (1) The orbital path length is now less. (2) The elliptical path takes the ship closer to Earth everywhere except at the point of firing (Fig. 13-17). The resulting decrease in gravitational potential energy increases the kinetic energy and thus also the speed of the ship.

Additional examples, video, and practice available at WileyPLUS

13-8 EINSTEIN AND GRAVITATION Learning Objectives After reading this module, you should be able to . . .

13.24 Explain Einstein’s principle of equivalence.

13.25 Identify Einstein’s model for gravitation as being due to the curvature of spacetime.

Key Idea ● Einstein pointed out that gravitation and acceleration are equivalent. This principle of equivalence led him to a theory of gravitation (the general theory of relativity) that explains gravitational effects in terms of a curvature of space.

Einstein and Gravitation Principle of Equivalence Albert Einstein once said: “I was . . . in the patent office at Bern when all of a sudden a thought occurred to me: ‘If a person falls freely, he will not feel his own weight.’ I was startled. This simple thought made a deep impression on me. It impelled me toward a theory of gravitation.” Thus Einstein tells us how he began to form his general theory of relativity. The fundamental postulate of this theory about gravitation (the gravitating of objects toward each other) is called the principle of equivalence, which says that gravitation and acceleration are equivalent. If a physicist were locked up in a small box as in Fig. 13-18, he would not be able to tell whether the box was at Figure 13-18 (a) A physicist in a box resting on Earth sees a cantaloupe falling with acceleration a ! 9.8 m/s2. (b) If he and the box accelerate in deep space at 9.8 m/s2, the cantaloupe has the same acceleration relative to him. It is not possible, by doing experiments within the box, for the physicist to tell which situation he is in. For example, the platform scale on which he stands reads the same weight in both situations. (a)

a

a

(b)

13-8 E I NSTE I N AN D G RAVITATION

rest on Earth (and subject only to Earth’s gravitational force), as in Fig. 13-18a, or accelerating through interstellar space at 9.8 m/s2 (and subject only to the force producing that acceleration), as in Fig. 13-18b. In both situations he would feel the same and would read the same value for his weight on a scale. Moreover, if he watched an object fall past him, the object would have the same acceleration relative to him in both situations.

Curvature of Space We have thus far explained gravitation as due to a force between masses. Einstein showed that, instead, gravitation is due to a curvature of space that is caused by the masses. (As is discussed later in this book, space and time are entangled, so the curvature of which Einstein spoke is really a curvature of spacetime, the combined four dimensions of our universe.) Picturing how space (such as vacuum) can have curvature is difficult. An analogy might help: Suppose that from orbit we watch a race in which two boats begin on Earth’s equator with a separation of 20 km and head due south (Fig. 13-19a). To the sailors, the boats travel along flat, parallel paths. However, with time the boats draw together until, nearer the south pole, they touch. The sailors in the boats can interpret this drawing together in terms of a force acting on the boats. Looking on from space, however, we can see that the boats draw together simply because of the curvature of Earth’s surface. We can see this because we are viewing the race from “outside” that surface. Figure 13-19b shows a similar race: Two horizontally separated apples are dropped from the same height above Earth. Although the apples may appear to travel along parallel paths, they actually move toward each other because they both fall toward Earth’s center. We can interpret the motion of the apples in terms of the gravitational force on the apples from Earth. We can also interpret the motion in terms of a curvature of the space near Earth, a curvature due to the presence of Earth’s mass. This time we cannot see the curvature because we cannot get “outside” the curved space, as we got “outside” the curved Earth in the boat example. However, we can depict the curvature with a drawing like Fig. 13-19c; there the apples would move along a surface that curves toward Earth because of Earth’s mass. When light passes near Earth, the path of the light bends slightly because of the curvature of space there, an effect called gravitational lensing. When light passes a more massive structure, like a galaxy or a black hole having large mass, its path can be bent more. If such a massive structure is between us and a quasar (an extremely bright, extremely distant source of light), the light from the quasar

N

Curved space near Earth

Parallel paths Flat space far from Earth

Equator C

Converging paths (a)

S

(b)

S

(c)

Figure 13-19 (a) Two objects moving along lines of longitude toward the south pole converge because of the curvature of Earth’s surface. (b) Two objects falling freely near Earth move along lines that converge toward the center of Earth because of the curvature of space near Earth. (c) Far from Earth (and other masses), space is flat and parallel paths remain parallel. Close to Earth, the parallel paths begin to converge because space is curved by Earth’s mass.

Earth

375

CHAPTE R 13 G RAVITATION

Paths of light from quasar

Figure 13-20 (a) Light from a distant quasar follows curved paths around a galaxy or a large black hole because the mass of the galaxy or black hole has curved the adjacent space. If the light is detected, it appears to have originated along the backward extensions of the final paths (dashed lines). (b) The Einstein ring known as MG1131#0456 on the computer screen of a telescope. The source of the light (actually, radio waves, which are a form of invisible light) is far behind the large, unseen galaxy that produces the ring; a portion of the source appears as the two bright spots seen along the ring.

Courtesy National Radio Astronomy Observatory

376

Apparent quasar directions Galaxy or large black hole

Final paths

Earth detector (a)

(b)

can bend around the massive structure and toward us (Fig. 13-20a). Then, because the light seems to be coming to us from a number of slightly different directions in the sky, we see the same quasar in all those different directions. In some situations, the quasars we see blend together to form a giant luminous arc, which is called an Einstein ring (Fig. 13-20b). Should we attribute gravitation to the curvature of spacetime due to the presence of masses or to a force between masses? Or should we attribute it to the actions of a type of fundamental particle called a graviton, as conjectured in some modern physics theories? Although our theories about gravitation have been enormously successful in describing everything from falling apples to planetary and stellar motions, we still do not fully understand it on either the cosmological scale or the quantum physics scale.

Review & Summary The Law of Gravitation Any particle in the universe attracts any other particle with a gravitational force whose magnitude is F!G

m1m2 r2

(Newton’s law of gravitation),

(13-1)

particle from an extended body is found by dividing the body into units of differential mass dm, each of which produces a differential : force dF on the particle, and then integrating to find the sum of those forces: :

F1 !

where m1 and m2 are the masses of the particles, r is their separation, and G (! 6.67 $ 10%11 N "m2/kg2) is the gravitational constant.

Gravitational Behavior of Uniform Spherical Shells The gravitational force between extended bodies is found by adding (integrating) the individual forces on individual particles within the bodies. However, if either of the bodies is a uniform spherical shell or a spherically symmetric solid, the net gravitational force it exerts on an external object may be computed as if all the mass of the shell or body were located at its center.

Superposition Gravitational forces obey the principle of su: F 1,net

perposition; that is, if n particles interact, the net force on a particle labeled particle 1 is the sum of the forces on it from all the other particles taken one at a time: :

F 1,net !

n

'

i!2

: F 1i,

(13-5) :

in which the sum is a vector sum of the forces F 1i on particle : 1 from particles 2, 3, . . . , n. The gravitational force F 1 on a

"

:

dF.

(13-6)

Gravitational Acceleration The gravitational acceleration ag

of a particle (of mass m) is due solely to the gravitational force acting on it. When the particle is at distance r from the center of a uniform, spherical body of mass M, the magnitude F of the gravitational force on the particle is given by Eq. 13-1.Thus, by Newton’s second law, which gives

F ! mag, ag !

GM . r2

(13-10) (13-11)

Free-Fall Acceleration and Weight Because Earth’s mass is not distributed uniformly, because the planet is not perfectly spherical, and because it rotates, the actual free-fall acceleration g: of a particle near Earth differs slightly from the gravitational acceleration a:g, and the particle’s weight (equal to mg) differs from the magnitude of the gravitational force on it as calculated by Newton’s law of gravitation (Eq. 13-1).

QU ESTIONS

377

Gravitation Within a Spherical Shell A uniform shell of

Kepler’s Laws The motion of satellites, both natural and artifi-

matter exerts no net gravitational force on a particle located inside it. This means that if a particle is located inside a uniform solid sphere at distance r from its center, the gravitational force exerted on the particle is due only to the mass that lies inside a sphere of radius r (the inside sphere). The force magnitude is given by

cial, is governed by these laws:

F!

GmM r, R3

(13-19)

where M is the sphere’s mass and R is its radius.

Gravitational Potential Energy The gravitational potential energy U(r) of a system of two particles, with masses M and m and separated by a distance r, is the negative of the work that would be done by the gravitational force of either particle acting on the other if the separation between the particles were changed from infinite (very large) to r.This energy is U!%

GMm r

(gravitational potential energy).

(13-21)

Potential Energy of a System If a system contains more than two particles, its total gravitational potential energy U is the sum of the terms representing the potential energies of all the pairs. As an example, for three particles, of masses m1, m2, and m3, U!%

# Gmr m 1

2

#

12

$

Gm1m3 Gm2m3 # . r13 r23

(13-22)

Escape Speed An object will escape the gravitational pull of an astronomical body of mass M and radius R (that is, it will reach an infinite distance) if the object’s speed near the body’s surface is at least equal to the escape speed, given by v!

2GM . A R

(13-28)

1. The law of orbits. All planets move in elliptical orbits with the Sun at one focus. 2. The law of areas. A line joining any planet to the Sun sweeps out equal areas in equal time intervals. (This statement is equivalent to conservation of angular momentum.) 3. The law of periods. The square of the period T of any planet is proportional to the cube of the semimajor axis a of its orbit. For circular orbits with radius r, T2 !

4p # GM $r 2

3

(law of periods),

(13-34)

where M is the mass of the attracting body — the Sun in the case of the solar system. For elliptical planetary orbits, the semimajor axis a is substituted for r.

Energy in Planetary Motion When a planet or satellite with mass m moves in a circular orbit with radius r, its potential energy U and kinetic energy K are given by U!%

GMm r

and K !

GMm . 2r

(13-21, 13-38)

The mechanical energy E ! K # U is then GMm . 2r For an elliptical orbit of semimajor axis a, E!%

E!%

GMm . 2a

(13-40)

(13-42)

Einstein’s View of Gravitation Einstein pointed out that gravi-

tation and acceleration are equivalent. This principle of equivalence led him to a theory of gravitation (the general theory of relativity) that explains gravitational effects in terms of a curvature of space.

Questions 1 In Fig. 13-21, a central particle of mass M is surrounded by a square array of other particles, separated by either distance d or distance d/2 along the perimeter of the square. What are the magnitude and direction of the net gravitational force on the central particle due to the other particles?

2M

M

5M

7M 3M

M 7M

5M 4M

4M

M

2M

Figure 13-21 Question 1. 2 Figure 13-22 shows three arrangements of the same identical particles, with three of them placed on a circle of radius 0.20 m and the fourth one placed at the center of the circle. (a) Rank the arrangements according to the magnitude of (a) (b) (c) the net gravitational force on the Figure 13-22 Question 2. central particle due to the other three particles, greatest first. (b) Rank them according to the gravitational potential energy of the four-particle system, least negative first. 3 In Fig. 13-23, a central particle is surrounded by two circular

rings of particles, at radii r and R, with R , r. All the particles have mass m. What are the magnitude and direction of the net gravitational force on the central particle due to the particles in the rings? 4 In Fig. 13-24, two particles, of masses m and 2m, are fixed in place on an axis. (a) Where on the axis can a third particle of mass 3m be placed Figure 13-23 Question 3. (other than at infinity) so that the net gravitational force on it from the first two particles is zero: to the left of the first two particles, to their m 2m right, between them but closer to the more massive particle, or beFigure 13-24 Question 4. tween them but closer to the less massive particle? (b) Does the answer change if the third particle has, instead, a mass of 16m? (c) Is there a point off the axis (other than infinity) at which the net force on the third particle would be zero?

378

CHAPTE R 13 G RAVITATION

5 Figure 13-25 shows three situations involving a point particle P with mass m and a spherical shell with a uniformly distributed mass M. The radii of the shells are given. Rank the situations according to the magnitude of the gravitational force on particle P due to the shell, greatest first. P P

P

d R

2R

(a)

R/2

(b)

(c)

y

A

C

B

Figure 13-26 Question 6. 7 Rank the four systems of equalmass particles shown in Checkpoint 2 according to the absolute value of the gravitational potential energy of the system, greatest first.

x

A

Figure 13-28 Question 9.

24 h

48 h

b

d

f

c

e

12 In Fig. 13-31, a particle of mass m (which is not shown) is to be moved from an infinite distance to one of the three possible locations a, b, and c. Two other particles, of masses m and 2m, are already fixed in place on the axis, as shown. Rank the three possible locations according to the work done by the net gravitational force on the moving particle due to the fixed particles, greatest first.

1 2 3 4 R4

d

Figure 13-30 Question 11.

ag

R2 R3

d

16 h

a

8 Figure 13-27 gives the gravitational acceleration ag for four planets as a function of the radial distance r from the center of the planet, starting at the surface of the planet (at radius R1, R2, R3, or R4). Plots 1 and 2 coincide for r / R2;plots 3 and 4 coincide for r / R4.Rank the four planets according to (a) mass and (b) mass per unit volume,greatest first.

R1

C

θ

θ

11 Figure 13-30 shows three uniform spherical planets that are identical in size and mass. The periods of rotation T for the planets are given, and six lettered points are indicated — three points are on the equators of the planets and three points are on the north poles. Rank the points according to the value of the free-fall acceleration g at them, greatest first.

x

d

d

B

10 Figure 13-29 shows six paths by 1 b 5 which a rocket orbiting a moon might 2 move from point a to point b. Rank the 3 4 paths according to (a) the correspon6 a ding change in the gravitational potential energy of the rocket–moon system and (b) the net work done on the rocket by the gravitational force from Figure 13-29 Question 10. the moon, greatest first.

Figure 13-25 Question 5. 6 In Fig. 13-26, three particles are fixed in place. The mass of B is greater than the mass of C. Can a fourth particle (particle D) be placed somewhere so that the net gravitational force on particle A from particles B, C, and D is zero? If so, in which quadrant should it be placed and which axis should it be near?

y

9 Figure 13-28 shows three particles initially fixed in place, with B and C identical and positioned symmetrically about the y axis, at distance d from A. (a) In what direction : is the net gravitational force Fnet on A? (b) If we move C directly away : from the origin, does Fnet change in direction? If so, how and what is the limit of the change?

d

r a

d 2m

d b

d m

c

Figure 13-31 Question 12.

Figure 13-27 Question 8.

Problems Tutoring problem available (at instructor’s discretion) in WileyPLUS and WebAssign SSM • – •••

Worked-out solution available in Student Solutions Manual

WWW Worked-out solution is at

Number of dots indicates level of problem difficulty

ILW

Interactive solution is at

http://www.wiley.com/college/halliday

Additional information available in The Flying Circus of Physics and at flyingcircusofphysics.com

Module 13-1 Newton’s Law of Gravitation •1 ILW A mass M is split into two parts, m and M % m, which are then separated by a certain distance. What ratio m/M maximizes the magnitude of the gravitational force between the parts?

•2 Moon effect. Some people believe that the Moon controls their activities. If the Moon moves from being directly on the opposite side of Earth from you to being directly overhead, by what percent does (a) the Moon’s gravitational pull on you

379

PROB LE M S

increase and (b) your weight (as measured on a scale) decrease? Assume that the Earth – Moon (center-to-center) distance is 3.82 $ 108 m and Earth’s radius is 6.37 $ 106 m.

tance d, at what (a) x coordinate and (b) y coordinate should particle D be placed so that the net gravitational force on particle A from particles B, C, and D is zero?

•3 SSM What must the separation be between a 5.2 kg particle and a 2.4 kg particle for their gravitational attraction to have a magnitude of 2.3 $ 10%12 N?

••11 As seen in Fig. 13-36, two spheres of mass m and a third sphere of mass M form an equilateral triangle, and a fourth sphere of mass m4 is at the center of the triangle. The net gravitational force on that central sphere from the three other spheres is zero. (a) What is M in terms of m? (b) If we double the value of m4, what then is the magnitude of the net gravitational force on the central sphere?

•5 Miniature black holes. Left over from the big-bang beginning of the universe, tiny black holes might still wander through the universe. If one with a mass of 1 $ 1011 kg (and a radius of only 1 $ 10%16 m) reached Earth, at what distance from your head would its gravitational pull on you match that of Earth’s? Module 13-2 Gravitation and the Principle of Superposition m2 •6 In Fig. 13-32, a square of edge length m 1 20.0 cm is formed by four spheres of masses y m1 ! 5.00 g, m2 ! 3.00 g, m3 ! 1.00 g, and m4 ! 5.00 g. In unit-vector notation, what is x the net gravitational force from them on a m5 central sphere with mass m5 ! 2.50 g? •7 One dimension. In Fig. 13-33, two point particles are fixed on an x axis sepa- m 3 m4 rated by distance d. Particle A has mass mA Figure 13-32 and particle B has mass 3.00mA. A third Problem 6. particle C, of mass 75.0mA, is to be placed y on the x axis and near particles A and B. In terms of distance d, at what x coordinate d x should C be placed so that the net gravitaA B tional force on particle A from particles B and C is zero? Figure 13-33 Problem 7. •8 In Fig. 13-34, three 5.00 kg spheres are located at distances d1 ! 0.300 m and d2 ! 0.400 m.What are the (a) magnitude and (b) direction (relative to the positive direction of the x axis) of the net gravitational force on sphere B due to spheres A and C? y A d1 B

C

x

d2

Figure 13-34 Problem 8.

m

m

Figure 13-36 Problem 11.

••12 In Fig. 13-37a, particle A is fixed in place at x !%0.20 m on the x axis and particle B, with a mass of 1.0 kg, is fixed in place at the origin. Particle C (not shown) can be moved along the x axis, between particle B and x ! 2. Figure 13-37b shows the x component Fnet,x of the net gravitational force on particle B due to particles A and C, as a function of position x of particle C. The plot actually extends to the right, approaching an asymptote of %4.17 $ 10%10 N as x : 2. What are the masses of (a) particle A and (b) particle C?

y

A

x

B

0

0

0.2

0.4

0.6

0.8

x (m) (b)

(a)

Figure 13-37 Problem 12. ••13 Figure 13-38 shows a spherical d hollow inside a lead sphere of radius R ! 4.00 cm; the surface of the hollow passes through the center of the R m sphere and “touches” the right side of the sphere. The mass of the Figure 13-38 Problem 13. sphere before hollowing was M ! 2.95 kg. With what gravitational force does the hollowed-out lead sphere attract a small sphere of mass m ! 0.431 kg that lies at a distance d ! 9.00 cm from the center of the lead sphere, on the straight line connecting the centers of the spheres and of the hollow? y

•9 SSM WWW We want to position a space probe along a line that extends directly toward the Sun in order to monitor solar flares. How far from Earth’s center is the point on the y line where the Sun’s gravitational pull on the probe balances Earth’s pull? ••10 Two dimensions. In Fig. 13-35, three point particles are fixed in place in an xy plane. Particle A has mass mA, particle B has mass 2.00mA, and particle C has mass 3.00mA. A fourth particle D, with mass 4.00mA, is to be placed near the other three particles. In terms of dis-

m4

Fnet,x

•4 The Sun and Earth each exert a gravitational force on the Moon. What is the ratio FSun /FEarth of these two forces? (The average Sun – Moon distance is equal to the Sun – Earth distance.)

M

B d 1.5d C

A

x

Figure 13-35 Problem 10.

••14 Three point particles are fixed in position in an xy plane. Two of B them, particle A of mass 6.00 g and pardAB ticle B of mass 12.0 g, are shown in Fig. θ 13-39, with a separation of dAB ! 0.500 x A m at angle u ! 30°. Particle C, with mass 8.00 g, is not shown. The net gravitational force acting on particle A due to Figure 13-39 Problem 14. particles B and C is 2.77 $ 10%14 N at an angle of %163.8° from the positive direction of the x axis. What are (a) the x coordinate and (b) the y coordinate of particle C? •••15 Three dimensions. Three point particles are fixed in place in an xyz coordinate system. Particle A, at the origin, has mass mA.

CHAPTE R 13 G RAVITATION

Particle B, at xyz coordinates (2.00d, 1.00d, 2.00d), has mass 2.00mA, and particle C, at coordinates (%1.00d, 2.00d, %3.00d), has mass 3.00mA. A fourth particle D, with mass 4.00mA, is to be placed near the other particles. In terms of distance d, at what (a) x, (b) y, and (c) z coordinate should D be placed so that the net gravitational force on A from B, C, and D is zero? r

dm •••16 In Fig. 13-40, a particle of mass m1 ! 0.67 kg is a dis- m1 dr tance d ! 23 cm from one end of d L a uniform rod with length L ! Figure 13-40 Problem 16. 3.0 m and mass M ! 5.0 kg. What is the magnitude of the gravita: tional force F on the particle from the rod?

Module 13-3 Gravitation Near Earth’s Surface •17 (a) What will an object weigh on the Moon’s surface if it weighs 100 N on Earth’s surface? (b) How many Earth radii must this same object be from the center of Earth if it is to weigh the same as it does on the Moon? •18 Mountain pull. A large mountain can slightly affect the direction of “down” as determined by a plumb line. Assume that we can model a mountain as a sphere of radius R ! 2.00 km and density (mass per unit volume) 2.6 $ 103 kg/m3. Assume also that we hang a 0.50 m plumb line at a distance of 3R from the sphere’s center and such that the sphere pulls horizontally on the lower end. How far would the lower end move toward the sphere? •19 SSM At what altitude above Earth’s surface would the gravitational acceleration be 4.9 m/s2? •20 Mile-high building. In 1956, Frank Lloyd Wright proposed the construction of a mile-high building in Chicago. Suppose the building had been constructed. Ignoring Earth’s rotation, find the change in your weight if you were to ride an elevator from the street level, where you weigh 600 N, to the top of the building.

shells, when the particle is located at radial distance (a) a, (b) b, and (c) c. ••25 A solid sphere has a uniformly distributed mass of 1.0 $ 104 kg and a radius of 1.0 m. What is the magnitude of the gravitational force due to the sphere on a particle of mass m when the particle is located at a distance of (a) 1.5 m and (b) 0.50 m from the center of the sphere? (c) Write a general expression for the magnitude of the gravitational force on the particle at a distance r 5 1.0 m from the center of the sphere. ••26 A uniform solid sphere of radius R produces a gravitational acceleration of ag on its surface. At what distance from the sphere’s center are there points (a) inside and (b) outside the sphere where the gravitational acceleration is ag /3? ••27 Figure 13-42 shows, not to scale, a cross section through the interior of Earth. Rather than being uniform throughout, Earth is divided into three zones: an outer crust, a mantle, and an inner core. The dimensions of these zones and the masses contained within them are shown on the figure. Earth has a total mass of 5.98 $ 1024 kg and a radius of 6370 km. Ignore rotation and assume that Earth is spherical. (a) Calculate ag at the surface. (b) Suppose that a bore hole (the Mohole) is driven to the crust – mantle interface at a depth of 25.0 km; what would be the value of ag at the bottom of the hole? (c) Suppose that Earth were a uniform sphere with the same total mass and size. What would be the value of ag at a depth of 25.0 km? (Precise measurements of ag are sensitive probes of the interior structure of Earth, although results can be clouded by local variations in mass distribution.) 6345 km 25 km

Core, 1.93

••21 ILW Certain neutron stars (extremely dense stars) are believed to be rotating at about 1 rev/s. If such a star has a radius of 20 km, what must be its minimum mass so that material on its surface remains in place during the rapid rotation? ••22 The radius Rh and mass Mh of a black hole are related by Rh ! 2GMh/c2, where c is the speed of light. Assume that the gravitational acceleration ag of an object at a distance ro ! 1.001Rh from the center of a black hole is given by Eq. 13-11 (it is, for large black holes). (a) In terms of Mh, find ag at ro. (b) Does ag at ro increase or decrease as Mh increases? (c) What is ag at ro for a very large black hole whose mass is 1.55 $ 1012 times the solar mass of 1.99 $ 1030 kg? (d) If an astronaut of height 1.70 m is at ro with her feet down, what is the difference in gravitational acceleration between her head and feet? (e) Is the tendency to stretch the astronaut severe? ••23 One model for a certain planet has a core of radius R and mass M surrounded by an outer shell of inner radius R, outer radius 2R, and mass 4M. If M ! 4.1 $ 1024 kg and R ! 6.0 $ 106 m, what is the gravitational a acceleration of a particle at points (a) R and (b) 3R from the center of the planet? Module 13-4 Gravitation Inside Earth •24 Two concentric spherical shells with uniformly distributed masses M1 and M2 are situated as shown in Fig. 13-41. Find the magnitude of the net gravitational force on a particle of mass m, due to the

M1

c

b

M2

Figure 13-41 Problem 24.

Mantle, 4.01 Crust, 3.94

1024 kg 1024 kg 1022 kg

3490 km

Figure 13-42 Problem 27. ••28 Assume a planet is a uniform sphere of radius R that (somehow) has a narrow radial tunnel through its center (Fig. 13-7). Also assume we can position an apple anywhere along the tunnel or outside the sphere. Let FR be the magnitude of the gravitational force on the apple when it is located at the planet’s surface. How far from the surface is there a point Rs r 0 where the magnitude is 12 FR if we move the apple (a) away –1 from the planet and (b) into –2 the tunnel? Module 13-5 Gravitational Potential Energy •29 Figure 13-43 gives the potential energy function U(r) of a projectile, plotted outward from

U (109 J)

380

–3 –4 –5

Figure 13-43 Problems 29 and 34.

PROB LE M S

•30 In Problem 1, what ratio m/M gives the least gravitational potential energy for the system? •31 SSM The mean diameters of Mars and Earth are 6.9 $ 103 km and 1.3 $ 104 km, respectively. The mass of Mars is 0.11 times Earth’s mass. (a) What is the ratio of the mean density (mass per unit volume) of Mars to that of Earth? (b) What is the value of the gravitational acceleration on Mars? (c) What is the escape speed on Mars? •32 (a) What is the gravitational potential energy of the twoparticle system in Problem 3? If you triple the separation between the particles, how much work is done (b) by the gravitational force between the particles and (c) by you? •33 What multiple of the energy needed to escape from Earth gives the energy needed to escape from (a) the Moon and (b) Jupiter? •34 Figure 13-43 gives the potential energy function U(r) of a projectile, plotted outward from the surface of a planet of radius Rs. If the projectile is launched radially outward from the surface with a mechanical energy of %2.0 $ 109 J, what are (a) its kinetic energy at radius r ! 1.25Rs and (b) its turning point (see Module 8-3) in terms of Rs? d

••35 Figure 13-44 shows four particles, each of mass 20.0 g, that form a square with an edge length of d ! 0.600 m. If d is reduced to 0.200 m, what is the change in the gravitational potential energy of the four-particle system?

d

••36 Zero, a hypothetical planet, has a mass Figure 13-44 of 5.0 $ 10 23 kg, a radius of 3.0 $ 106 m, and no Problem 35. atmosphere. A 10 kg space probe is to be launched vertically from its surface. (a) If the probe is launched with an initial energy of 5.0 $ 107 J, what will be its kinetic energy when it is 4.0 $ 106 m from the center of Zero? (b) If the probe is to achieve a maximum distance of 8.0 $ 106 m from the center of Zero, with what initial kinetic energy must it be launched from the surface of Zero? ••37 The three spheres in Fig. 13-45, with masses mA ! 80 g, mB ! 10 g, and mC ! 20 g, have their centers on a common line, with L ! 12 cm and d ! 4.0 cm. You move sphere B along the line until its center-to-center separation from C is d ! 4.0 cm. How much work is done on sphere B (a) by you and (b) by the net gravitational force on B due to spheres A and C? L d

A

d

B

C

Figure 13-45 Problem 37. ••38 In deep space, sphere A of mass 20 kg is located at the origin of an x axis and sphere B of mass 10 kg is located on the axis at x ! 0.80 m. Sphere B is released from rest while sphere A is held at the origin. (a) What is the gravitational potential energy of the twosphere system just as B is released? (b) What is the kinetic energy of B when it has moved 0.20 m toward A?

••39 SSM (a) What is the escape speed on a spherical asteroid whose radius is 500 km and whose gravitational acceleration at the surface is 3.0 m/s2? (b) How far from the surface will a particle go if it leaves the asteroid’s surface with a radial speed of 1000 m/s? (c) With what speed will an object hit the asteroid if it is dropped from 1000 km above the surface? ••40 A projectile is shot directly away from Earth’s surface. Neglect the rotation of Earth. What multiple of Earth’s radius RE gives the radial distance a projectile reaches if (a) its initial speed is 0.500 of the escape speed from Earth and (b) its initial kinetic energy is 0.500 of the kinetic energy required to escape Earth? (c) What is the least initial mechanical energy required at launch if the projectile is to escape Earth? ••41 SSM Two neutron stars are separated by a distance of 1.0 $ 1010 m. They each have a mass of 1.0 $ 1030 kg and a radius of 1.0 $ 105 m. They are initially at rest with respect to each other. As measured from that rest frame, how fast are they moving when (a) their separation has decreased to one-half its initial value and (b) they are about to collide? ••42 Figure 13-46a shows a particle A that can be moved along a y axis from an infinite distance to the origin. That origin lies at the midpoint between particles B and C, which have identical masses, and the y axis is a perpendicular bisector between them. Distance D is 0.3057 m. Figure 13-46b shows the potential energy U of the three-particle system as a function of the position of particle A along the y axis. The curve actually extends rightward and approaches an asymptote of %2.7 $ 10%11 J as y : 2. What are the masses of (a) particles B and C and (b) particle A? y (m) 0 0.5

1

1.5

2

–1 U (10–10 J)

the surface of a planet of radius Rs. What least kinetic energy is required of a projectile launched at the surface if the projectile is to “escape” the planet?

381

y A B

C D

D

x

–2

–3

(a)

(b)

Figure 13-46 Problem 42. Module 13-6 Planets and Satellites: Kepler’s Laws •43 (a) What linear speed must an Earth satellite have to be in a circular orbit at an altitude of 160 km above Earth’s surface? (b) What is the period of revolution? •44 A satellite is put in a circular orbit about Earth with a radius equal to one-half the radius of the Moon’s orbit. What is its period of revolution in lunar months? (A lunar month is the period of revolution of the Moon.) •45 The Martian satellite Phobos travels in an approximately circular orbit of radius 9.4 $ 106 m with a period of 7 h 39 min. Calculate the mass of Mars from this information. •46 The first known collision between space debris and a functioning satellite occurred in 1996: At an altitude of 700 km, a yearold French spy satellite was hit by a piece of an Ariane rocket. A stabilizing boom on the satellite was demolished, and the satellite

382

CHAPTE R 13 G RAVITATION

was sent spinning out of control. Just before the collision and in kilometers per hour, what was the speed of the rocket piece relative to the satellite if both were in circular orbits and the collision was (a) head-on and (b) along perpendicular paths?

(a) Plot log a (y axis) against log T (x axis) and show that you get a straight line. (b) Measure the slope of the line and compare it with the value that you expect from Kepler’s third law. (c) Find the mass of Jupiter from the intercept of this line with the y axis.

•47 SSM WWW The Sun, which is 2.2 $ 1020 m from the center of the Milky Way galaxy, revolves around that center once every 2.5 $ 108 years. Assuming each star in the Galaxy has a mass equal to the Sun’s mass of 2.0 $ 1030 kg, the stars are distributed uniformly in a sphere about the galactic center, and the Sun is at the edge of that sphere, estimate the number of stars in the Galaxy.

••56 In 1993 the spacecraft Galileo sent an image (Fig. 13-48) of asteroid 243 Ida and a tiny orbiting moon (now known as Dactyl), the first confirmed example of an asteroid – moon system. In the image, the moon, which is 1.5 km wide, is 100 km from the center of the asteroid, which is 55 km long.Assume the moon’s orbit is circular with a period of 27 h. (a) What is the mass of the asteroid? (b) The volume of the asteroid, measured from the Galileo images, is 14 100 km3. What is the density (mass per unit volume) of the asteroid?

•48 The mean distance of Mars from the Sun is 1.52 times that of Earth from the Sun. From Kepler’s law of periods, calculate the number of years required for Mars to make one revolution around the Sun; compare your answer with the value given in Appendix C. •49 A comet that was seen in April 574 by Chinese astronomers on a day known by them as the Woo Woo day was spotted again in May 1994. Assume the time between observations is the period of the Woo Woo day comet and its eccentricity is 0.9932. What are (a) the semimajor axis of the comet’s orbit and (b) its greatest distance from the Sun in terms of the mean orbital radius RP of Pluto? •50 An orbiting satellite stays over a certain spot on the equator of (rotating) Earth. What is the altitude of the orbit (called a geosynchronous orbit)? •51 SSM A satellite, moving in an elliptical orbit, is 360 km above Earth’s surface at its farthest point and 180 km above at its closest point. Calculate (a) the semimajor axis and (b) the eccentricity of the orbit.

Courtesy NASA

Figure 13-48 Problem 56. A tiny moon (at right) orbits asteroid 243 Ida. ••57 ILW In a certain binary-star system, each star has the same mass as our Sun, and they revolve about their center of mass. The distance between them is the same as the distance between Earth and the Sun. What is their period of revolution in years?

••53 A 20 kg satellite has a circular orbit with a period of 2.4 h and a radius of 8.0 $ 106 m around a planet of unknown mass. If the magnitude of the gravitational acceleration on the surface of the planet is 8.0 m/s2, what is the radius of the planet?

•••58 The presence of an unseen planet orbiting a distant star can sometimes be inferred from the motion of the star as we see it. As the star and planet orbit the center of mass of the star – planet system, the star moves toward and away from us with what is called the line of sight velocity, a motion that can be detected. Figure 13-49 shows a graph of the line of sight velocity versus time for the star 14 Herculis. The star’s mass is believed to be 0.90 of the mass of our Sun. Assume that only one planet orbits the star and that our view is along the plane of the orbit. Then approximate (a) the planet’s mass in terms of Jupiter’s mass mJ and (b) the planet’s orbital radius in terms of Earth’s orbital radius rE.

••54 Hunting a black hole. Observations of the light from a certain star indicate that it is part of a binary (two-star) system.This visible star has orbital speed v ! 270 km/s, orbital period T ! 1.70 days, m 1 r1 and approximate mass m1 ! 6Ms, where Ms is the Sun’s mass, 1.99 $ 1030 kg. Assume that the visible star and its companion star, which is dark and unseen, are both in circular orbits (Fig. 13-47). What integer Figure 13-47 multiple of Ms gives the approximate mass m2 of the dark star?

O

r2

m2

Problem 54.

••55 In 1610, Galileo used his telescope to discover four moons around Jupiter, with these mean orbital radii a and periods T: Name

a (108 m)

T (days)

Io Europa Ganymede Callisto

4.22 6.71 10.7 18.8

1.77 3.55 7.16 16.7

Line of sight velocity (m/s)

•52 The Sun’s center is at one focus of Earth’s orbit. How far from this focus is the other focus, (a) in meters and (b) in terms of the solar radius, 6.96 $ 108 m? The eccentricity is 0.0167, and the semimajor axis is 1.50 $ 1011 m.

70

0

–70

1500 days Time

Figure 13-49 Problem 58. •••59 Three identical stars of mass M form an equilateral triangle that rotates around the triangle’s center as the stars move in a common circle about that center. The triangle has edge length L. What is the speed of the stars?

PROB LE M S

Module 13-7 Satellites: Orbits and Energy •60 In Fig. 13-50, two satellites, A and B, both of mass m ! 125 kg, move in the r same circular orbit of radius r ! 7.87 $ 106 m around Earth but in opposite senses of A B rotation and therefore on a collision Earth course. (a) Find the total mechanical energy EA # EB of the two satellites # Earth system before the collision. (b) If Figure 13-50 the collision is completely inelastic so that Problem 60. the wreckage remains as one piece of tangled material (mass ! 2m), find the total mechanical energy immediately after the collision. (c) Just after the collision, is the wreckage falling directly toward Earth’s center or orbiting around Earth? •61 (a) At what height above Earth’s surface is the energy required to lift a satellite to that height equal to the kinetic energy required for the satellite to be in orbit at that height? (b) For greater heights, which is greater, the energy for lifting or the kinetic energy for orbiting? •62 Two Earth satellites, A and B, each of mass m, are to be launched into circular orbits about Earth’s center. Satellite A is to orbit at an altitude of 6370 km. Satellite B is to orbit at an altitude of 19 110 km. The radius of Earth RE is 6370 km. (a) What is the ratio of the potential energy of satellite B to that of satellite A, in orbit? (b) What is the ratio of the kinetic energy of satellite B to that of satellite A, in orbit? (c) Which satellite has the greater total energy if each has a mass of 14.6 kg? (d) By how much? •63 SSM WWW An asteroid, whose mass is 2.0 $ 10%4 times the mass of Earth, revolves in a circular orbit around the Sun at a distance that is twice Earth’s distance from the Sun. (a) Calculate the period of revolution of the asteroid in years. (b) What is the ratio of the kinetic energy of the asteroid to the kinetic energy of Earth? •64 A satellite orbits a planet of unknown mass in a circle of radius 2.0 $ 107 m. The magnitude of the gravitational force on the satellite from the planet is F ! 80 N. (a) What is the kinetic energy of the satellite in this orbit? (b) What would F be if the orbit radius were increased to 3.0 $ 107 m? ••65 A satellite is in a circular Earth orbit of radius r. The area A enclosed by the orbit depends on r2 because A ! pr2. Determine how the following properties of the satellite depend on r: (a) period, (b) kinetic energy, (c) angular momentum, and (d) speed. ••66 One way to attack a satellite in Earth orbit is to launch a swarm of pellets in the same orbit as the satellite but in the opposite direction. Suppose a satellite in a circular orbit 500 km above Earth’s surface collides with a pellet having mass 4.0 g. (a) What is the kinetic energy of the pellet in the reference frame of the satellite just before the collision? (b) What is the ratio of this kinetic energy to the kinetic energy of a 4.0 g bullet from a modern army rifle with a muzzle speed of 950 m/s? •••67 What are (a) the speed and (b) the period of a 220 kg satellite in an approximately circular orbit 640 km above the surface of Earth? Suppose the satellite loses mechanical energy at the average rate of 1.4 $ 105 J per orbital revolution. Adopting the reasonable approximation that the satellite’s orbit becomes a “circle of slowly diminishing radius,” determine the satellite’s (c) altitude, (d) speed, and (e) period at the end of its 1500th revolution. (f) What

383

is the magnitude of the average retarding force on the satellite? Is angular momentum around Earth’s center conserved for (g) the satellite and (h) the satellite – Earth system (assuming that system is isolated)? •••68 Two small spaceships, each with mass m ! 2000 kg, are in the circular Earth orbit of Fig. 13-51, at an altitude h of 400 km. Igor, the commander of one of the ships, arrives at any fixed point in the orbit 90 s ahead of Picard, the commander of the other ship. What are the (a) period T0 and (b) speed v0 of the ships? At point P in R Fig. 13-51, Picard fires an instantaP neous burst in the forward direction, M r reducing his ship’s speed by 1.00%. After this burst, he follows the elliptical orbit shown dashed in the figure. What are the (c) kinetic Figure 13-51 Problem 68. energy and (d) potential energy of his ship immediately after the burst? In Picard’s new elliptical orbit, what are (e) the total energy E, (f) the semimajor axis a, and (g) the orbital period T? (h) How much earlier than Igor will Picard return to P? Module 13-8 Einstein and Gravitation •69 In Fig. 13-18b, the scale on which the 60 kg physicist stands reads 220 N. How long will the cantaloupe take to reach the floor if the physicist drops it (from rest relative to himself) at a height of 2.1 m above the floor? Additional Problems 70 The radius Rh of a black hole is the radius of a mathematical sphere, called the event horizon, that is centered on the black hole. Information from events inside the event horizon cannot reach the outside world. According to Einstein’s general theory of relativity, Rh ! 2GM/c2, where M is the mass of the black hole and c is the speed of light. Suppose that you wish to study a black hole near it, at a radial distance of 50Rh. However, you do not want the difference in gravitational acceleration between your feet and your head to exceed 10 m/s2 when you are feet down (or head down) toward the black hole. (a) As a multiple of our Sun’s mass MS, approximately what is the limit to the mass of the black hole you can tolerate at the given radial distance? (You need to estimate your height.) (b) Is the limit an upper limit (you can tolerate smaller masses) or a lower limit (you can tolerate larger masses)? 71 Several planets (Jupiter, Saturn, m Uranus) are encircled by rings, perhaps x R composed of material that failed to form a satellite. In addition, many galaxies contain ring-like structures. Consider a homogeneous thin ring of mass M and M outer radius R (Fig. 13-52). (a) What gravitational attraction does it exert on Figure 13-52 a particle of mass m located on the Problem 71. ring’s central axis a distance x from the ring center? (b) Suppose the particle falls from rest as a result of the attraction of the ring of matter. What is the speed with which it passes through the center of the ring? 72 A typical neutron star may have a mass equal to that of the Sun but a radius of only 10 km. (a) What is the gravitational acceleration at the surface of such a star? (b) How fast would an object be

384

CHAPTE R 13 G RAVITATION

moving if it fell from rest through a distance of 1.0 m on such a star? (Assume the star does not rotate.) 73 Figure 13-53 is a graph of the kinetic energy K of an asteroid versus its distance r from Earth’s center, as the asteroid falls directly in toward that center. (a) What is the (approximate) mass of the asteroid? (b) What is its speed at r ! 1.945 $ 107 m?

K (109 J)

T!

3p , A Gr

where r is the uniform density (mass per unit volume) of the spherical planet. (b) Calculate the rotation period assuming a density of 3.0 g/cm3, typical of many planets, satellites, and asteroids. No astronomical object has ever been found to be spinning with a period shorter than that determined by this analysis.

3

81 SSM In a double-star system, two stars of mass 3.0 $ 1030 kg each rotate about the system’s center of mass at radius 1.0 $ 1011 m. (a) What is their common angular speed? (b) If a meteoroid passes through the system’s center of mass perpendicular to their orbital plane, what minimum speed must it have at the center of mass if it is to escape to “infinity” from the two-star system?

2

1 1.75

80 The fastest possible rate of rotation of a planet is that for which the gravitational force on material at the equator just barely provides the centripetal force needed for the rotation. (Why?) (a) Show that the corresponding shortest period of rotation is

1.85 r (107 m)

1.95

Figure 13-53 Problem 73. 74 The mysterious visitor that appears in the enchanting story The Little Prince was said to come from a planet that “was scarcely any larger than a house!” Assume that the mass per unit volume of the planet is about that of Earth and that the planet does not appreciably spin. Approximate (a) the free-fall acceleration on the planet’s surface and (b) the escape speed from the planet. 75 ILW The masses and coordinates of three spheres are as follows: 20 kg, x ! 0.50 m, y ! 1.0 m; 40 kg, x ! %1.0 m, y ! %1.0 m; 60 kg, x ! 0 m, y ! %0.50 m. What is the magnitude of the gravitational force on a 20 kg sphere located at the origin due to these three spheres? 76 SSM A very early, simple satellite consisted of an inflated spherical aluminum balloon 30 m in diameter and of mass 20 kg. Suppose a meteor having a mass of 7.0 kg passes within 3.0 m of the surface of the satellite. What is the magnitude of the gravitational force on the meteor from the satellite at the closest approach? 77 Four uniform spheres, with masses mA ! 40 kg, mB ! 35 kg, mC ! 200 kg, and mD ! 50 kg, have (x, y) coordinates of (0, 50 cm), (0, 0), (%80 cm, 0), and (40 cm, 0), respectively. In unit-vector notation, what is the net gravitational force on sphere B due to the other spheres? 78 (a) In Problem 77, remove sphere A and calculate the gravitational potential energy of the remaining three-particle system. (b) If A is then put back in place, is the potential energy of the four-particle system more or less than that of the system in (a)? (c) In (a), is the work done by you to remove A positive or negative? (d) In (b), is the work done by you to replace A positive or negative? 79 SSM A certain triple-star system consists of two stars, each of mass m, revolving in the same circular orbit of radius r around a central star of mass M m (Fig. 13-54). The two orbiting stars are always at opposite ends of a diameter of the orbit. Derive an expression for the period of revolution of the stars.

r

M

Figure 13-54 Problem 79.

m

82 A satellite is in elliptical orbit with a period of 8.00 $ 104 s about a planet of mass 7.00 $ 1024 kg. At aphelion, at radius 4.5 $ 107 m, the satellite’s angular speed is 7.158 $ 10%5 rad/s. What is its angular speed at perihelion? 83 SSM In a shuttle craft of mass m ! 3000 kg, Captain Janeway orbits a planet of mass M ! 9.50 $ 1025 kg, in a circular orbit of radius r ! 4.20 $ 107 m. What are (a) the period of the orbit and (b) the speed of the shuttle craft? Janeway briefly fires a forwardpointing thruster, reducing her speed by 2.00%. Just then, what are (c) the speed, (d) the kinetic energy, (e) the gravitational potential energy, and (f) the mechanical energy of the shuttle craft? (g) What is the semimajor axis of the elliptical orbit now taken by the craft? (h) What is the difference between the period of the original circular orbit and that of the new elliptical orbit? (i) Which orbit has the smaller period? 84 Consider a pulsar, a collapsed star of extremely high density, with a mass M equal to that of the Sun (1.98 $ 1030 kg), a radius R of only 12 km, and a rotational period T of 0.041 s. By what percentage does the free-fall acceleration g differ from the gravitational acceleration ag at the equator of this spherical star? 85 ILW A projectile is fired vertically from Earth’s surface with an initial speed of 10 km/s. Neglecting air drag, how far above the surface of Earth will it go? 86 An object lying on Earth’s equator is accelerated (a) toward the center of Earth because Earth rotates, (b) toward the Sun because Earth revolves around the Sun in an almost circular orbit, and (c) toward the center of our galaxy because the Sun moves around the galactic center. For the latter, the period is 2.5 $ 108 y and the radius is 2.2 $ 1020 m. Calculate these three accelerations as multiples of g ! 9.8 m/s2. 87 (a) If the legendary apple of Newton could be released from rest at a height of 2 m from the surface of a neutron star with a mass 1.5 times that of our Sun and a radius of 20 km, what would be the apple’s speed when it reached the surface of the star? (b) If the apple could rest on the surface of the star, what would be the approximate difference between the gravitational acceleration at the top and at the bottom of the apple? (Choose a reasonable size for an apple; the answer indicates that an apple would never survive near a neutron star.)

385

PROB LE M S

88 With what speed would mail pass through the center of Earth if falling in a tunnel through the center? 89 SSM The orbit of Earth around the Sun is almost circular: The closest and farthest distances are 1.47 $ 108 km and 1.52 $ 108 km respectively. Determine the corresponding variations in (a) total energy, (b) gravitational potential energy, (c) kinetic energy, and (d) orbital speed. (Hint: Use conservation of energy and conservation of angular momentum.) 90 A 50 kg satellite circles planet Cruton every 6.0 h. The magnitude of the gravitational force exerted on the satellite by Cruton is 80 N. (a) What is the radius of the orbit? (b) What is the kinetic energy of the satellite? (c) What is the mass of planet Cruton? 91 We watch two identical astronomical bodies A and B, each of mass m, fall toward each other from rest because of the gravitational force on each from the other. Their initial center-to-center separation is Ri. Assume that we are in an inertial reference frame that is stationary with respect to the center of mass of this twobody system. Use the principle of conservation of mechanical energy (Kf # Uf ! Ki # Ui) to find the following when the centerto-center separation is 0.5Ri : (a) the total kinetic energy of the system, (b) the kinetic energy of each body, (c) the speed of each body relative to us, and (d) the speed of body B relative to body A. Next assume that we are in a reference frame attached to body A (we ride on the body). Now we see body B fall from rest toward us. From this reference frame, again use Kf # Uf ! Ki # Ui to find the following when the center-to-center separation is 0.5Ri : (e) the kinetic energy of body B and (f) the speed of body B relative to body A. (g) Why are the answers to (d) and (f) different? Which answer is correct? 92 A 150.0 kg rocket moving radially outward from Earth has a speed of 3.70 km/s when its engine shuts off 200 km above Earth’s surface. (a) Assuming negligible air drag acts on the rocket, find the rocket’s kinetic energy when the rocket is 1000 km above Earth’s surface. (b) What maximum height above the surface is reached by the rocket? 93 Planet Roton, with a mass of 7.0 $ 1024 kg and a radius of 1600 km, gravitationally attracts a meteorite that is initially at rest relative to the planet, at a distance great enough to take as infinite. The meteorite falls toward the planet. Assuming the planet is airless, find the speed of the meteorite when it reaches the planet’s surface. 94 Two 20 kg spheres are fixed in place on a y axis, one at y ! 0.40 m and the other at y ! %0.40 m. A 10 kg ball is then released from rest at a point on the x axis that is at a great distance (effectively infinite) from the spheres. If the only forces acting on the ball are the gravitational forces from the spheres, then when the ball reaches the (x, y) point (0.30 m, 0), what are (a) its kinetic energy and (b) the net force on it from the spheres, in unit-vector notation? 95 Sphere A with mass 80 kg is located at the origin of an xy coordinate system; sphere B with mass 60 kg is located at coordinates (0.25 m, 0); sphere C with mass 0.20 kg is located in the first quadrant 0.20 m from A and 0.15 m from B. In unit-vector notation, what is the gravitational force on C due to A and B? 96 In his 1865 science fiction novel From the Earth to the Moon, Jules Verne described how three astronauts are shot to the Moon by means of a huge gun. According to Verne, the aluminum capsule containing the astronauts is accelerated by ignition of

nitrocellulose to a speed of 11 km/s along the gun barrel’s length of 220 m. (a) In g units, what is the average acceleration of the capsule and astronauts in the gun barrel? (b) Is that acceleration tolerable or deadly to the astronauts? A modern version of such gun-launched spacecraft (although without passengers) has been proposed. In this modern version, called the SHARP (Super High Altitude Research Project) gun, ignition of methane and air shoves a piston along the gun’s tube, compressing hydrogen gas that then launches a rocket. During this launch, the rocket moves 3.5 km and reaches a speed of 7.0 km/s. Once launched, the rocket can be fired to gain additional speed. (c) In g units, what would be the average acceleration of the rocket within the launcher? (d) How much additional speed is needed (via the rocket engine) if the rocket is to orbit Earth at an altitude of 700 km? 97 An object of mass m is initially held in place at radial distance r ! 3RE from the center of Earth, where RE is the radius of Earth. Let ME be the mass of Earth. A force is applied to the object to move it to a radial distance r ! 4RE, where it again is held in place. Calculate the work done by the applied force during the move by integrating the force magnitude. 98 To alleviate the traffic congestion between two cities such as Boston and Washington, D.C., engineers have proposed building a rail tunnel along a chord line connecting the cities (Fig. 13-55). A train, unpropelled by any engine and starting from rest, would fall through the first half of the tunnel and then move up the second half. Assuming Earth is a uniform sphere and ignoring air drag and friction, find the city-to-city travel time. Train

Figure 13-55 Problem 98. 99 A thin rod with mass M ! 5.00 kg is bent in a semicircle of radius R ! 0.650 m (Fig. 13-56). (a) What is its gravitational force (both magnitude and direction on a particle with mass m ! 3.0 $ 10%3 kg at P, the center of curvature? (b) What would be the force on the particle if the rod were a complete circle? 100 In Fig. 13-57, identical blocks with identical masses m ! 2.00 kg hang from strings of different lengths on a balance at Earth’s surface. The strings have negligible mass and differ in length by h ! 5.00 cm. Assume Earth is spherical with a uniform density r ! 5.50 g/cm3. What is the difference in the weight of the blocks due to one being closer to Earth than the other?

R

Figure 13-56 Problem 99.

h

Figure 13-57 Problem 100.

101 A spaceship is on a straight-line path between Earth and the Moon. At what distance from Earth is the net gravitational force on the spaceship zero?

C

H

A

P

T

E

R

1

4

Fluids 14-1 FLUIDS, DENSITY, AND PRESSURE Learning Objectives After reading this module, you should be able to . . .

14.01 Distinguish fluids from solids. 14.02 When mass is uniformly distributed, relate density to mass and volume.

14.03 Apply the relationship between hydrostatic pressure, force, and the surface area over which that force acts.

Key Ideas ● The density r of any material is defined as the material’s

mass per unit volume: 'm . 'V Usually, where a material sample is much larger than atomic dimensions, we can write this as r!

m . V ● A fluid is a substance that can flow; it conforms to the boundaries of its container because it cannot withstand r!

shearing stress. It can, however, exert a force perpendicular to its surface. That force is described in terms of pressure p: 'F , 'A in which 'F is the force acting on a surface element of area 'A. If the force is uniform over a flat area, this can be written as F p! . A ● The force resulting from fluid pressure at a particular point in a fluid has the same magnitude in all directions. p!

What Is Physics? The physics of fluids is the basis of hydraulic engineering, a branch of engineering that is applied in a great many fields. A nuclear engineer might study the fluid flow in the hydraulic system of an aging nuclear reactor, while a medical engineer might study the blood flow in the arteries of an aging patient. An environmental engineer might be concerned about the drainage from waste sites or the efficient irrigation of farmlands. A naval engineer might be concerned with the dangers faced by a deep-sea diver or with the possibility of a crew escaping from a downed submarine. An aeronautical engineer might design the hydraulic systems controlling the wing flaps that allow a jet airplane to land. Hydraulic engineering is also applied in many Broadway and Las Vegas shows, where huge sets are quickly put up and brought down by hydraulic systems. Before we can study any such application of the physics of fluids, we must first answer the question “What is a fluid?”

What Is a Fluid? A fluid, in contrast to a solid, is a substance that can flow. Fluids conform to the boundaries of any container in which we put them. They do so because a fluid cannot sustain a force that is tangential to its surface. (In the more formal language of Module 12-3, a fluid is a substance that flows because it cannot 386

14-1 FLU I DS, DE NSITY, AN D PR ESSU R E

withstand a shearing stress. It can, however, exert a force in the direction perpendicular to its surface.) Some materials, such as pitch, take a long time to conform to the boundaries of a container, but they do so eventually; thus, we classify even those materials as fluids. You may wonder why we lump liquids and gases together and call them fluids. After all (you may say), liquid water is as different from steam as it is from ice. Actually, it is not. Ice, like other crystalline solids, has its constituent atoms organized in a fairly rigid three-dimensional array called a crystalline lattice. In neither steam nor liquid water, however, is there any such orderly long-range arrangement.

Density and Pressure When we discuss rigid bodies, we are concerned with particular lumps of matter, such as wooden blocks, baseballs, or metal rods. Physical quantities that we find useful, and in whose terms we express Newton’s laws, are mass and force. We might speak, for example, of a 3.6 kg block acted on by a 25 N force. With fluids, we are more interested in the extended substance and in properties that can vary from point to point in that substance. It is more useful to speak of density and pressure than of mass and force.

Density To find the density r of a fluid at any point, we isolate a small volume element 'V around that point and measure the mass 'm of the fluid contained within that element. The density is then r!

'm . 'V

387

Table 14-1 Some Densities Density (kg/m3)

Material or Object

Interstellar space Best laboratory vacuum Air: 20(C and 1 atm pressure 20(C and 50 atm Styrofoam Ice Water: 20(C and 1 atm 20(C and 50 atm Seawater: 20(C and 1 atm Whole blood Iron Mercury (the metal, not the planet) Earth: average core crust Sun: average core White dwarf star (core) Uranium nucleus Neutron star (core)

10%20 10%17 1.21 60.5 1 $ 102 0.917 $ 103 0.998 $ 103 1.000 $ 103 1.024 $ 103 1.060 $ 103 7.9 $ 103 13.6 $ 103 5.5 $ 103 9.5 $ 103 2.8 $ 103 1.4 $ 103 1.6 $ 105 1010 3 $ 1017 1018

(14-1)

In theory, the density at any point in a fluid is the limit of this ratio as the volume element 'V at that point is made smaller and smaller. In practice, we assume that a fluid sample is large relative to atomic dimensions and thus is “smooth” (with uniform density), rather than “lumpy” with atoms. This assumption allows us to write the density in terms of the mass m and volume V of the sample: m (uniform density). (14-2) r! V

Pressure sensor

Density is a scalar property; its SI unit is the kilogram per cubic meter. Table 14-1 shows the densities of some substances and the average densities of some objects. Note that the density of a gas (see Air in the table) varies considerably with pressure, but the density of a liquid (see Water) does not; that is, gases are readily compressible but liquids are not.

(a)

∆F

Pressure

∆A

Let a small pressure-sensing device be suspended inside a fluid-filled vessel, as in Fig. 14-1a. The sensor (Fig. 14-1b) consists of a piston of surface area 'A riding in a close-fitting cylinder and resting against a spring. A readout arrangement allows us to record the amount by which the (calibrated) spring is compressed by the surrounding fluid, thus indicating the magnitude 'F of the force that acts normal to the piston. We define the pressure on the piston as p!

'F . 'A

(14-3)

In theory, the pressure at any point in the fluid is the limit of this ratio as the surface area 'A of the piston, centered on that point, is made smaller and smaller. However, if the force is uniform over a flat area A (it is evenly distributed over every point of

Vacuum

(b)

Figure 14-1 (a) A fluid-filled vessel containing a small pressure sensor, shown in (b). The pressure is measured by the relative position of the movable piston in the sensor.

388

CHAPTE R 14 FLU I DS

Table 14-2 Some Pressures

the area), we can write Eq. 14-3 as Pressure (Pa)

Center of the Sun Center of Earth Highest sustained laboratory pressure Deepest ocean trench (bottom) Spike heels on a dance floor Automobile tirea Atmosphere at sea level Normal blood systolic pressurea,b Best laboratory vacuum

p!

16

2 $ 10 4 $ 1011 1.5 $ 1010 1.1 $ 108 106 2 $ 105 1.0 $ 105 1.6 $ 104 10%12

F A

(14-4)

(pressure of uniform force on flat area),

where F is the magnitude of the normal force on area A. We find by experiment that at a given point in a fluid at rest, the pressure p defined by Eq. 14-4 has the same value no matter how the pressure sensor is oriented. Pressure is a scalar, having no directional properties. It is true that the force acting on the piston of our pressure sensor is a vector quantity, but Eq. 14-4 involves only the magnitude of that force, a scalar quantity. The SI unit of pressure is the newton per square meter, which is given a special name, the pascal (Pa). In metric countries, tire pressure gauges are calibrated in kilopascals. The pascal is related to some other common (non-SI) pressure units as follows: 1 atm ! 1.01 $ 105 Pa ! 760 torr ! 14.7 lb/in.2.

a

Pressure in excess of atmospheric pressure. b Equivalent to 120 torr on the physician’s pressure gauge.

The atmosphere (atm) is, as the name suggests, the approximate average pressure of the atmosphere at sea level. The torr (named for Evangelista Torricelli, who invented the mercury barometer in 1674) was formerly called the millimeter of mercury (mm Hg). The pound per square inch is often abbreviated psi. Table 14-2 shows some pressures.

Sample Problem 14.01 Atmospheric pressure and force A living room has floor dimensions of 3.5 m and 4.2 m and a height of 2.4 m. (a) What does the air in the room weigh when the air pressure is 1.0 atm? KEY IDEAS (1) The air’s weight is equal to mg, where m is its mass. (2) Mass m is related to the air density r and the air volume V by Eq. 14-2 (r ! m/V). Calculation: Putting the two ideas together and taking the density of air at 1.0 atm from Table 14-1, we find mg ! (rV)g ! (1.21 kg/m3)(3.5 m $ 4.2 m $ 2.4 m)(9.8 m/s2) ! 418 N % 420 N.

(Answer)

This is the weight of about 110 cans of Pepsi.

(b) What is the magnitude of the atmosphere’s downward force on the top of your head, which we take to have an area of 0.040 m2? KEY IDEA When the fluid pressure p on a surface of area A is uniform, the fluid force on the surface can be obtained from Eq. 14-4 (p ! F/A). Calculation: Although air pressure varies daily, we can approximate that p ! 1.0 atm. Then Eq. 14-4 gives F ! pA ! (1.0 atm) ! 4.0 $ 103 N.

$ 10 N/m # 1.01 1.0 $(0.040 m ) atm 5

2

2

(Answer)

This large force is equal to the weight of the air column from the top of your head to the top of the atmosphere.

Additional examples, video, and practice available at WileyPLUS

14-2 FLUIDS AT REST Learning Objectives After reading this module, you should be able to . . .

14.04 Apply the relationship between the hydrostatic pressure, fluid density, and the height above or below a reference level.

14.05 Distinguish between total pressure (absolute pressure) and gauge pressure.

14-2 FLU I DS AT R EST

389

Key Ideas ● Pressure in a fluid at rest varies with vertical position y. For y measured positive upward, p2 ! p1 # rg(y1 % y2).

where p is the pressure in the sample. ● The pressure in a fluid is the same for all points at the same level.

If h is the depth of a fluid sample below some reference level at which the pressure is p0, this equation becomes p ! p0 # rgh,

● Gauge pressure is the difference between the actual pressure (or absolute pressure) at a point and the atmospheric pressure.

Fluids at Rest Figure 14-2a shows a tank of water — or other liquid — open to the atmosphere. As every diver knows, the pressure increases with depth below the air – water interface. The diver’s depth gauge, in fact, is a pressure sensor much like that of Fig. 14-1b. As every mountaineer knows, the pressure decreases with altitude as one ascends into the atmosphere. The pressures encountered by the diver and the mountaineer are usually called hydrostatic pressures, because they are due to fluids that are static (at rest). Here we want to find an expression for hydrostatic pressure as a function of depth or altitude. Let us look first at the increase in pressure with depth below the water’s surface. We set up a vertical y axis in the tank, with its origin at the air – water interface and the positive direction upward. We next consider a water sample con-

Three forces act on this sample of water. y Air

A

This downward force is due to the water pressure pushing on the top surface.

y

y= 0

y= 0

Water y1

y1

Level 1, p1

y2

y2 Sample

F1

(b)

(a)

This upward force is due to the water pressure pushing on the bottom surface. F2

Gravity pulls downward on the sample. y

y

F2

y= 0

y= 0

y1

y1

y2

y2

The three forces balance.

Sample

(c)

Level 2, p2 (d)

mg

mg

(e)

F1

Figure 14-2 (a) A tank of water in which a sample of water is contained in an imaginary cylinder of horizontal base area A. : : (b)–(d) Force F1 acts at the top surface of the cylinder; force F2 acts at the bottom surface of the cylinder; the gravitational : force on the water in the cylinder is represented by mg. (e) A free-body diagram of the water sample. In WileyPLUS, this figure is available as an animation with voiceover.

390

CHAPTE R 14 FLU I DS

tained in an imaginary right circular cylinder of horizontal base (or face) area A, such that y1 and y2 (both of which are negative numbers) are the depths below the surface of the upper and lower cylinder faces, respectively. Figure 14-2e is a free-body diagram for the water in the cylinder. The water is in static equilibrium; that is, it is stationary and the forces on it balance. Three : forces act on it vertically: Force F1 acts at the top surface of the cylinder and is : due to the water above the cylinder (Fig. 14-2b). Force F2 acts at the bottom surface of the cylinder and is due to the water just below the cylinder (Fig. 14-2c). The gravitational force on the water is m: g, where m is the mass of the water in the cylinder (Fig. 14-2d). The balance of these forces is written as (14-5)

F2 ! F1 # mg. To involve pressures, we use Eq. 14-4 to write F1 ! p1 A and

F2 ! p2 A.

(14-6)

The mass m of the water in the cylinder is, from Eq. 14-2, m ! rV, where the cylinder’s volume V is the product of its face area A and its height y1 % y2. Thus, m is equal to rA(y1 % y2). Substituting this and Eq. 14-6 into Eq. 14-5, we find p2 A ! p1 A # rAg(y1 % y2) or

p2 ! p1 # rg(y1 % y2).

(14-7)

This equation can be used to find pressure both in a liquid (as a function of depth) and in the atmosphere (as a function of altitude or height). For the former, suppose we seek the pressure p at a depth h below the liquid surface. Then we choose level 1 to be the surface, level 2 to be a distance h below it (as in Fig. 14-3), and p0 to represent the atmospheric pressure on the surface.We then substitute y1 ! 0, p1 ! p0 and y2 ! %h, p2 ! p into Eq. 14-7, which becomes p ! p0 # rgh

(pressure at depth h).

(14-8)

Note that the pressure at a given depth in the liquid depends on that depth but not on any horizontal dimension. The pressure at a point in a fluid in static equilibrium depends on the depth of that point but not on any horizontal dimension of the fluid or its container.

y Air p0

Level 1

y= 0

Liquid h p

Level 2

Figure 14-3 The pressure p increases with depth h below the liquid surface according to Eq. 14-8.

Thus, Eq. 14-8 holds no matter what the shape of the container. If the bottom surface of the container is at depth h, then Eq. 14-8 gives the pressure p there. In Eq. 14-8, p is said to be the total pressure, or absolute pressure, at level 2. To see why, note in Fig. 14-3 that the pressure p at level 2 consists of two contributions: (1) p0, the pressure due to the atmosphere, which bears down on the liquid, and (2) rgh, the pressure due to the liquid above level 2, which bears down on level 2. In general, the difference between an absolute pressure and an atmospheric pressure is called the gauge pressure (because we use a gauge to measure this pressure difference). For Fig. 14-3, the gauge pressure is rgh. Equation 14-7 also holds above the liquid surface: It gives the atmospheric pressure at a given distance above level 1 in terms of the atmospheric pressure p1 at level 1 (assuming that the atmospheric density is uniform over that distance). For example, to find the atmospheric pressure at a distance d above level 1 in Fig. 14-3, we substitute y1 ! 0, Then with r ! rair, we obtain

p1 ! p0 and

y2 ! d,

p ! p0 % rair gd.

p2 ! p.

391

14-2 FLU I DS AT R EST

Checkpoint 1 The figure shows four containers of olive oil. Rank them according to the pressure at depth h, greatest first.

h

(a)

(b)

(c)

(d)

Sample Problem 14.02 Gauge pressure on a scuba diver A novice scuba diver practicing in a swimming pool takes enough air from his tank to fully expand his lungs before abandoning the tank at depth L and swimming to the surface, failing to exhale during his ascent. At the surface, the difference 'p between the external pressure on him and the air pressure in his lungs is 9.3 kPa. From what depth does he start? What potentially lethal danger does he face? KEY IDEA The pressure at depth h in a liquid of density r is given by Eq. 14-8 (p ! p0 # rgh), where the gauge pressure rgh is added to the atmospheric pressure p0. Calculations: Here, when the diver fills his lungs at depth L, the external pressure on him (and thus the air pressure within his lungs) is greater than normal and given by Eq. 14-8 as p ! p0 # rgL, where r is the water’s density (998 kg/m3, Table 14-1). As he

ascends, the external pressure on him decreases, until it is atmospheric pressure p0 at the surface. His blood pressure also decreases, until it is normal. However, because he does not exhale, the air pressure in his lungs remains at the value it had at depth L. At the surface, the pressure difference 'p is 'p ! p % p0 ! rgL, so

L!

9300 Pa 'p ! 6g (998 kg/m3)(9.8 m/s2)

! 0.95 m.

This is not deep! Yet, the pressure difference of 9.3 kPa (about 9% of atmospheric pressure) is sufficient to rupture the diver’s lungs and force air from them into the depressurized blood, which then carries the air to the heart, killing the diver. If the diver follows instructions and gradually exhales as he ascends, he allows the pressure in his lungs to equalize with the external pressure, and then there is no danger.

Sample Problem 14.03 Balancing of pressure in a U-tube The U-tube in Fig. 14-4 contains two liquids in static equilibrium: Water of density rw (! 998 kg/m3) is in the right arm, and oil of unknown density rx is in the left. Measurement gives l ! 135 mm and d ! 12.3 mm. What is the density of the oil?

Oil

This much oil balances...

KEY IDEAS (1) The pressure pint at the level of the oil – water interface in the left arm depends on the density rx and height of the oil above the interface. (2) The water in the right arm at the same level must be at the same pressure pint. The reason is that, because the water is in static equilibrium, pressures at points in the water at the same level must be the same. Calculations: In the right arm, the interface is a distance l below the free surface of the water, and we have, from Eq. 14-8, pint ! p0 # rwgl (right arm). In the left arm, the interface is a distance l # d below the free surface of the oil, and we have, again from Eq. 14-8, pint ! p0 # rx g(l # d) (left arm).

(Answer)

d

Water

l

... this much water.

Interface

Figure 14-4 The oil in the left arm stands higher than the water.

Equating these two expressions and solving for the unknown density yield rx ! rw

l 135 mm ! (998 kg/m3) l#d 135 mm # 12.3 mm

! 915 kg/m3.

(Answer)

Note that the answer does not depend on the atmospheric pressure p0 or the free-fall acceleration g.

Additional examples, video, and practice available at WileyPLUS

392

CHAPTE R 14 FLU I DS

14-3 MEASURING PRESSURE Learning Objectives After reading this module, you should be able to . . .

14.06 Describe how a barometer can measure atmospheric pressure.

14.07 Describe how an open-tube manometer can measure the gauge pressure of a gas.

Key Ideas ● A mercury barometer can be used to measure atmospheric pressure.

● An open-tube manometer can be used to measure the gauge pressure of a confined gas.

Measuring Pressure The Mercury Barometer y p≈0

p≈0

Level 2

h h p0

Figure 14-5a shows a very basic mercury barometer, a device used to measure the pressure of the atmosphere. The long glass tube is filled with mercury and inverted with its open end in a dish of mercury, as the figure shows. The space above the mercury column contains only mercury vapor, whose pressure is so small at ordinary temperatures that it can be neglected. We can use Eq. 14-7 to find the atmospheric pressure p0 in terms of the height h of the mercury column.We choose level 1 of Fig. 14-2 to be that of the air – mercury interface and level 2 to be that of the top of the mercury column, as labeled in Fig. 14-5a.We then substitute

p0

Level 1

y1 ! 0,

(a)

(b)

Figure 14-5 (a) A mercury barometer. (b) Another mercury barometer. The distance h is the same in both cases.

p0

Level 1

p1 ! p0

and

y2 ! h,

p2 ! 0

into Eq. 14-7, finding that p0 ! rgh, (14-9) where r is the density of the mercury. For a given pressure, the height h of the mercury column does not depend on the cross-sectional area of the vertical tube. The fanciful mercury barometer of Fig. 14-5b gives the same reading as that of Fig. 14-5a; all that counts is the vertical distance h between the mercury levels. Equation 14-9 shows that, for a given pressure, the height of the column of mercury depends on the value of g at the location of the barometer and on the density of mercury, which varies with temperature. The height of the column (in millimeters) is numerically equal to the pressure (in torr) only if the barometer is at a place where g has its accepted standard value of 9.80665 m/s2 and the temperature of the mercury is 0°C. If these conditions do not prevail (and they rarely do), small corrections must be made before the height of the mercury column can be transformed into a pressure.

The Open-Tube Manometer h pg Level 2 Tank Manometer

Figure 14-6 An open-tube manometer, connected to measure the gauge pressure of the gas in the tank on the left. The right arm of the U-tube is open to the atmosphere.

An open-tube manometer (Fig. 14-6) measures the gauge pressure pg of a gas. It consists of a U-tube containing a liquid, with one end of the tube connected to the vessel whose gauge pressure we wish to measure and the other end open to the atmosphere. We can use Eq. 14-7 to find the gauge pressure in terms of the height h shown in Fig. 14-6. Let us choose levels 1 and 2 as shown in Fig. 14-6. With y1 ! 0,

p1 ! p0

and

y2 ! %h, p2 ! p

substituted into Eq. 14-7, we find that pg ! p % p0 ! rgh,

(14-10)

where r is the liquid’s density. The gauge pressure pg is directly proportional to h.

393

14-4 PASCAL’S PR I NCI PLE

The gauge pressure can be positive or negative, depending on whether p , p0 or p 0 p0. In inflated tires or the human circulatory system, the (absolute) pressure is greater than atmospheric pressure, so the gauge pressure is a positive quantity, sometimes called the overpressure. If you suck on a straw to pull fluid up the straw, the (absolute) pressure in your lungs is actually less than atmospheric pressure.The gauge pressure in your lungs is then a negative quantity.

14-4 PASCAL’S PRINCIPLE Learning Objectives After reading this module, you should be able to . . .

14.08 Identify Pascal’s principle. 14.09 For a hydraulic lift, apply the relationship between the

input area and displacement and the output area and displacement.

Key Idea ● Pascal’s principle states that a change in the pressure applied to an enclosed fluid is transmitted undiminished to every portion of the fluid and to the walls of the containing vessel.

Pascal’s Principle

Lead shot Piston

When you squeeze one end of a tube to get toothpaste out the other end, you are watching Pascal’s principle in action. This principle is also the basis for the Heimlich maneuver, in which a sharp pressure increase properly applied to the abdomen is transmitted to the throat, forcefully ejecting food lodged there. The principle was first stated clearly in 1652 by Blaise Pascal (for whom the unit of pressure is named):

pext Liquid P

A change in the pressure applied to an enclosed incompressible fluid is transmitted undiminished to every portion of the fluid and to the walls of its container.

Demonstrating Pascal’s Principle Consider the case in which the incompressible fluid is a liquid contained in a tall cylinder, as in Fig. 14-7. The cylinder is fitted with a piston on which a container of lead shot rests. The atmosphere, container, and shot exert pressure pext on the piston and thus on the liquid. The pressure p at any point P in the liquid is then p ! pext # rgh.

h p

Figure 14-7 Lead shot (small balls of lead) loaded onto the piston create a pressure pext at the top of the enclosed (incompressible) liquid. If pext is increased, by adding more lead shot, the pressure increases by the same amount at all points within the liquid.

... a large output force.

(14-11)

Let us add a little more lead shot to the container to increase pext by an amount 'pext. The quantities r, g, and h in Eq. 14-11 are unchanged, so the pressure change at P is 'p ! 'pext. (14-12)

A small input force produces ...

Fo Ao

Input Fi

This pressure change is independent of h, so it must hold for all points within the liquid, as Pascal’s principle states.

Ai

Pascal’s Principle and the Hydraulic Lever

di

Figure 14-8 shows how Pascal’s principle can be made the basis of a hydraulic lever. In operation, let an external force of magnitude Fi be directed downward on the lefthand (or input) piston, whose surface area is Ai. An incompressible liquid in the device then produces an upward force of magnitude Fo on the right-hand (or output) piston, whose surface area is Ao. To keep the system in equilibrium, there must be a downward force of magnitude Fo on the output piston from an external load (not

Output

do

Oil

Figure 14-8 A hydraulic arrangement that : can be used to magnify a force Fi.The work done is, however, not magnified and is the same for both the input and output forces.

394

CHAPTE R 14 FLU I DS :

:

shown). The force Fi applied on the left and the downward force Fo from the load on the right produce a change 'p in the pressure of the liquid that is given by 'p ! so

Fi Fo ! , Ai Ao

Fo ! Fi

Ao . Ai

(14-13)

Equation 14-13 shows that the output force Fo on the load must be greater than the input force Fi if Ao , Ai, as is the case in Fig. 14-8. If we move the input piston downward a distance di, the output piston moves upward a distance do, such that the same volume V of the incompressible liquid is displaced at both pistons. Then which we can write as

V ! Ai di ! Aodo, do ! di

Ai . Ao

(14-14)

This shows that, if Ao , Ai (as in Fig. 14-8), the output piston moves a smaller distance than the input piston moves. From Eqs. 14-13 and 14-14 we can write the output work as

#

W ! Fo do ! Fi

Ao Ai

$#d AA $ ! F d , i

i

o

i

i

(14-15)

which shows that the work W done on the input piston by the applied force is equal to the work W done by the output piston in lifting the load placed on it. The advantage of a hydraulic lever is this: With a hydraulic lever, a given force applied over a given distance can be transformed to a greater force applied over a smaller distance.

The product of force and distance remains unchanged so that the same work is done. However, there is often tremendous advantage in being able to exert the larger force. Most of us, for example, cannot lift an automobile directly but can with a hydraulic jack, even though we have to pump the handle farther than the automobile rises and in a series of small strokes.

14-5 ARCHIMEDES’ PRINCIPLE Learning Objectives After reading this module, you should be able to . . .

14.10 Describe Archimedes’ principle. 14.11 Apply the relationship between the buoyant force on a body and the mass of the fluid displaced by the body. 14.12 For a floating body, relate the buoyant force to the gravitational force.

14.13 For a floating body, relate the gravitational force to the mass of the fluid displaced by the body. 14.14 Distinguish between apparent weight and actual weight. 14.15 Calculate the apparent weight of a body that is fully or partially submerged.

Key Ideas ● Archimedes’ principle states that when a body is fully or partially submerged in a fluid, the fluid pushes upward with a buoyant force with magnitude Fb ! mf g, where mf is the mass of the fluid that has been pushed out of the way by the body.

● When a body floats in a fluid, the magnitude Fb of the

(upward) buoyant force on the body is equal to the magnitude Fg of the (downward) gravitational force on the body. ● The apparent weight of a body on which a buoyant force acts is related to its actual weight by weightapp ! weight % Fb.

14-5 ARCH I M E DES’ PR I NCI PLE

Archimedes’ Principle Figure 14-9 shows a student in a swimming pool, manipulating a very thin plastic sack (of negligible mass) that is filled with water. She finds that the sack and its contained water are in static equilibrium, tending neither to rise nor to sink. : The downward gravitational force Fg on the contained water must be balanced by a net upward force from the water surrounding the sack. : This net upward force is a buoyant force Fb . It exists because the pressure in the surrounding water increases with depth below the surface. Thus, the pressure near the bottom of the sack is greater than the pressure near the top, which means the forces on the sack due to this pressure are greater in magnitude near the bottom of the sack than near the top. Some of the forces are represented in Fig. 14-10a, where the space occupied by the sack has been left empty. Note that the force vectors drawn near the bottom of that space (with upward components) have longer lengths than those drawn near the top of the sack (with downward components). If we vectorially add all the forces on the sack from the water, the horizontal components cancel and the vertical components add to yield the upward buoyant : : force Fb on the sack. (Force Fb is shown to the right of the pool in Fig. 14-10a.) : Because the sack of water is in static equilibrium, the magnitude of Fb is equal to : the magnitude mf g of the gravitational force Fg on the sack of water: Fb ! mf g. (Subscript f refers to fluid, here the water.) In words, the magnitude of the buoyant force is equal to the weight of the water in the sack. In Fig. 14-10b, we have replaced the sack of water with a stone that exactly fills the hole in Fig. 14-10a. The stone is said to displace the water, meaning that the stone occupies space that would otherwise be occupied by water.We have changed nothing about the shape of the hole, so the forces at the hole’s surface must be the same as when the water-filled sack was in place. Thus, the same upward buoyant force that acted on the water-filled sack now acts on the stone; that is, the magnitude Fb of the buoyant force is equal to mf g, the weight of the water displaced by the stone. Unlike the water-filled sack, the stone is not in static equilibrium. The down: ward gravitational force Fg on the stone is greater in magnitude than the upward buoyant force (Fig. 14-10b). The stone thus accelerates downward, sinking. Let us next exactly fill the hole in Fig. 14-10a with a block of lightweight wood, as in Fig. 14-10c. Again, nothing has changed about the forces at the hole’s surface, so the magnitude Fb of the buoyant force is still equal to mf g, the weight

Fb

(a)

Figure 14-10 (a) The water surrounding the hole in the water produces a net upward buoyant force on whatever fills the hole. (b) For a stone of the same volume as the hole, the gravitational force exceeds the buoyant force in magnitude. (c) For a lump of wood of the same volume, the gravitational force is less than the buoyant force in magnitude.

The upward buoyant force on this sack of water equals the weight of the water.

Figure 14-9 A thin-walled plastic sack of water is in static equilibrium in the pool.The gravitational force on the sack must be balanced by a net upward force on it from the surrounding water.

Fb Stone

The buoyant force is due to the pressure of the surrounding water.

Fg

The net force is downward, so the stone accelerates downward.

(b)

Fb Wood Fg

(c)

395

The net force is upward, so the wood accelerates upward.

396

CHAPTE R 14 FLU I DS

of the displaced water. Like the stone, the block is not in static equilibrium. : However, this time the gravitational force Fg is lesser in magnitude than the buoyant force (as shown to the right of the pool), and so the block accelerates upward, rising to the top surface of the water. Our results with the sack, stone, and block apply to all fluids and are summarized in Archimedes’ principle: :

When a body is fully or partially submerged in a fluid, a buoyant force Fb from the surrounding fluid acts on the body. The force is directed upward and has a magnitude equal to the weight mf g of the fluid that has been displaced by the body.

The buoyant force on a body in a fluid has the magnitude Fb ! mf g

(buoyant force),

(14-16)

where mf is the mass of the fluid that is displaced by the body.

Floating When we release a block of lightweight wood just above the water in a pool, the block moves into the water because the gravitational force on it pulls it downward. As the block displaces more and more water, the magnitude Fb of the upward buoyant force acting on it increases. Eventually, Fb is large enough to equal the magnitude Fg of the downward gravitational force on the block, and the block comes to rest. The block is then in static equilibrium and is said to be floating in the water. In general, When a body floats in a fluid, the magnitude Fb of the buoyant force on the body is equal to the magnitude Fg of the gravitational force on the body.

We can write this statement as Fb ! Fg

(floating).

(14-17)

From Eq. 14-16, we know that Fb ! mf g. Thus, When a body floats in a fluid, the magnitude Fg of the gravitational force on the body is equal to the weight mf g of the fluid that has been displaced by the body.

We can write this statement as Fg ! mf g

(floating).

(14-18)

In other words, a floating body displaces its own weight of fluid.

Apparent Weight in a Fluid If we place a stone on a scale that is calibrated to measure weight, then the reading on the scale is the stone’s weight. However, if we do this underwater, the upward buoyant force on the stone from the water decreases the reading. That reading is then an apparent weight. In general, an apparent weight is related to the actual weight of a body and the buoyant force on the body by actual magnitude of #apparent weight $ ! #weight$ % #buoyant force$, which we can write as weightapp ! weight % Fb

(apparent weight).

(14-19)

397

14-5 ARCH I M E DES’ PR I NCI PLE

If, in some test of strength, you had to lift a heavy stone, you could do it more easily with the stone underwater. Then your applied force would need to exceed only the stone’s apparent weight, not its larger actual weight. The magnitude of the buoyant force on a floating body is equal to the body’s weight. Equation 14-19 thus tells us that a floating body has an apparent weight of zero — the body would produce a reading of zero on a scale. For example, when astronauts prepare to perform a complex task in space, they practice the task floating underwater, where their suits are adjusted to give them an apparent weight of zero.

Checkpoint 2 A penguin floats first in a fluid of density r0, then in a fluid of density 0.95r0, and then in a fluid of density 1.1r0. (a) Rank the densities according to the magnitude of the buoyant force on the penguin, greatest first. (b) Rank the densities according to the amount of fluid displaced by the penguin, greatest first.

Sample Problem 14.04 Floating, buoyancy, and density In Fig. 14-11, a block of density r ! 800 kg/m3 floats face down in a fluid of density rf ! 1200 kg/m3. The block has height H ! 6.0 cm.

Floating means means Floating that the the buoyant buoyant that force matches matches the the force gravitational force. force. gravitational

(a) By what depth h is the block submerged?

Figure 14-11 Block of height H floats in a fluid, to a depth of h.

KEY IDEAS (1) Floating requires that the upward buoyant force on the block match the downward gravitational force on the block. (2) The buoyant force is equal to the weight mf g of the fluid displaced by the submerged portion of the block. Calculations: From Eq. 14-16, we know that the buoyant force has the magnitude Fb ! mf g, where mf is the mass of the fluid displaced by the block’s submerged volume Vf . From Eq. 14-2 (r ! m/V), we know that the mass of the displaced fluid is mf ! rfVf . We don’t know Vf but if we symbolize the block’s face length as L and its width as W, then from Fig. 14-11 we see that the submerged volume must be Vf ! LWh. If we now combine our three expressions, we find that the upward buoyant force has magnitude Fb ! mf g ! rfVf g ! rf LWhg.

(14-20)

Similarly, we can write the magnitude Fg of the gravitational force on the block, first in terms of the block’s mass m, then in terms of the block’s density r and (full) volume V, and then in terms of the block’s dimensions L, W, and H (the full height): Fg ! mg ! rVg ! rf LWHg.

(14-21)

The floating block is stationary. Thus, writing Newton’s second law for components along a vertical y axis with the positive direction upward (Fnet,y ! may), we have Fb % Fg ! m(0),

H h

or from Eqs. 14-20 and 14-21, rfLWhg % rLWHg ! 0, which gives us 800 kg/m3 r (6.0 cm) h! r H! f 1200 kg/m3 ! 4.0 cm.

(Answer)

(b) If the block is held fully submerged and then released, what is the magnitude of its acceleration? Calculations: The gravitational force on the block is the same but now, with the block fully submerged, the volume of the displaced water is V ! LWH. (The full height of the block is used.) This means that the value of Fb is now larger, and the block will no longer be stationary but will accelerate upward. Now Newton’s second law yields Fb % Fg ! ma, or

rfLWHg % rLWHg ! rLWHa,

where we inserted rLWH for the mass m of the block. Solving for a leads to rf 1200 kg/m3 % 1 (9.8 m/s2) a! r %1 g! 800 kg/m3

#

! 4.9 m/s 2.

Additional examples, video, and practice available at WileyPLUS

$ #

$

(Answer)

398

CHAPTE R 14 FLU I DS

14-6 THE EQUATION OF CONTINUITY Learning Objectives After reading this module, you should be able to . . .

14.16 Describe steady flow, incompressible flow, nonviscous flow, and irrotational flow. 14.17 Explain the term streamline. 14.18 Apply the equation of continuity to relate the

cross-sectional area and flow speed at one point in a tube to those quantities at a different point. 14.19 Identify and calculate volume flow rate. 14.20 Identify and calculate mass flow rate.

Key Ideas ● An ideal fluid is incompressible and lacks viscosity, and its flow is steady and irrotational. ● A streamline is the path followed by an individual fluid particle.

in which RV is the volume flow rate, A is the cross-sectional area of the tube of flow at any point, and v is the speed of the fluid at that point.

● A tube

● The mass flow rate Rm is

of flow is a bundle of streamlines.

● The flow within any tube of flow obeys the equation of continuity:

Rm ! rRV ! rAv ! a constant.

RV ! Av ! a constant,

Ideal Fluids in Motion The motion of real fluids is very complicated and not yet fully understood. Instead, we shall discuss the motion of an ideal fluid, which is simpler to handle mathematically and yet provides useful results. Here are four assumptions that we make about our ideal fluid; they all are concerned with flow: 1. Steady flow In steady (or laminar) flow, the velocity of the moving fluid at any fixed point does not change with time. The gentle flow of water near the center of a quiet stream is steady; the flow in a chain of rapids is not. Figure 14-12 shows a transition from steady flow to nonsteady (or nonlaminar or turbulent) flow for a rising stream of smoke. The speed of the smoke particles increases as they rise and, at a certain critical speed, the flow changes from steady to nonsteady.

Will McIntyre/Photo Researchers, Inc.

Figure 14-12 At a certain point, the rising flow of smoke and heated gas changes from steady to turbulent.

2. Incompressible flow We assume, as for fluids at rest, that our ideal fluid is incompressible; that is, its density has a constant, uniform value. 3. Nonviscous flow Roughly speaking, the viscosity of a fluid is a measure of how resistive the fluid is to flow. For example, thick honey is more resistive to flow than water, and so honey is said to be more viscous than water. Viscosity is the fluid analog of friction between solids; both are mechanisms by which the kinetic energy of moving objects can be transferred to thermal energy. In the absence of friction, a block could glide at constant speed along a horizontal surface. In the same way, an object moving through a nonviscous fluid would experience no viscous drag force—that is, no resistive force due to viscosity; it could move at constant speed through the fluid. The British scientist Lord Rayleigh noted that in an ideal fluid a ship’s propeller would not work, but, on the other hand, in an ideal fluid a ship (once set into motion) would not need a propeller! 4. Irrotational flow Although it need not concern us further, we also assume that the flow is irrotational. To test for this property, let a tiny grain of dust move with the fluid. Although this test body may (or may not) move in a circular path, in irrotational flow the test body will not rotate about an axis through its own center of mass. For a loose analogy, the motion of a Ferris wheel is rotational; that of its passengers is irrotational. We can make the flow of a fluid visible by adding a tracer. This might be a dye injected into many points across a liquid stream (Fig. 14-13) or smoke

14-6 TH E EQUATION OF CONTI N U ITY

399

Figure 14-13 The steady flow of a fluid around a cylinder, as revealed by a dye tracer that was injected into the fluid upstream of the cylinder. Courtesy D. H. Peregrine, University of Bristol

particles added to a gas flow (Fig. 14-12). Each bit of a tracer follows a streamline, which is the path that a tiny element of the fluid would take as the fluid flows. Recall from Chapter 4 that the velocity of a particle is always tangent to the path taken by the particle. Here the particle is the fluid element, and its velocity : v is always tangent to a streamline (Fig. 14-14). For this reason, two streamlines can never intersect; if they did, then an element arriving at their intersection would have two different velocities simultaneously — an impossibility.

v

Streamline

Fluid element

Figure 14-14 A fluid element traces out a streamline as it moves. The velocity vector of the element is tangent to the streamline at every point.

The Equation of Continuity You may have noticed that you can increase the speed of the water emerging from a garden hose by partially closing the hose opening with your thumb. Apparently the speed v of the water depends on the cross-sectional area A through which the water flows. Here we wish to derive an expression that relates v and A for the steady flow of an ideal fluid through a tube with varying cross section, like that in Fig. 14-15. The flow there is toward the right, and the tube segment shown (part of a longer tube) has length L. The fluid has speeds v1 at the left end of the segment and v2 at the right end. The tube has cross-sectional areas A1 at the left end and A2 at the right end. Suppose that in a time interval 't a volume 'V of fluid enters the tube segment at its left end (that volume is colored purple in Fig. 14-15). Then, because the fluid is incompressible, an identical volume 'V must emerge from the right end of the segment (it is colored green in Fig. 14-15). L

The volume flow per second here must match ... v1

A2

v2

A1

(a) Time t

Figure 14-15 Fluid flows from left to right at a steady rate through a tube segment of length L. The fluid’s speed is v1 at the left side and v2 at the right side. The tube’s cross-sectional area is A1 at the left side and A2 at the right side. From time t in (a) to time t # 't in (b), the amount of fluid shown in purple enters at the left side and the equal amount of fluid shown in green emerges at the right side.

L

(b) Time t + ∆t

... the volume flow per second here.

400

CHAPTE R 14 FLU I DS

e

v

(a) Time t e

v

∆x

We can use this common volume 'V to relate the speeds and areas. To do so, we first consider Fig. 14-16, which shows a side view of a tube of uniform cross-sectional area A. In Fig. 14-16a, a fluid element e is about to pass through the dashed line drawn across the tube width. The element’s speed is v, so during a time interval 't, the element moves along the tube a distance 'x ! v 't. The volume 'V of fluid that has passed through the dashed line in that time interval 't is (14-22)

'V ! A 'x ! Av 't.

(b) Time t + ∆t

Figure 14-16 Fluid flows at a constant speed v through a tube. (a) At time t, fluid element e is about to pass the dashed line. (b) At time t # 't, element e is a distance 'x ! v 't from the dashed line.

Applying Eq. 14-22 to both the left and right ends of the tube segment in Fig. 14-15, we have 'V ! A1v1 't ! A2v2 't or

A1v1 ! A2v2

(equation of continuity).

(14-23)

This relation between speed and cross-sectional area is called the equation of continuity for the flow of an ideal fluid. It tells us that the flow speed increases when we decrease the cross-sectional area through which the fluid flows. Equation 14-23 applies not only to an actual tube but also to any so-called tube of flow, or imaginary tube whose boundary consists of streamlines. Such a tube acts like a real tube because no fluid element can cross a streamline; thus, all the fluid within a tube of flow must remain within its boundary. Figure 14-17 shows a tube of flow in which the cross-sectional area increases from area A1 to area A2 along the flow direction. From Eq. 14-23 we know that, with the increase in area, the speed must decrease, as is indicated by the greater spacing between streamlines at the right in Fig. 14-17. Similarly, you can see that in Fig. 14-13 the speed of the flow is greatest just above and just below the cylinder. We can rewrite Eq. 14-23 as RV ! Av ! a constant

The volume flow per second here must match ... A2 A1

... the volume flow per second here.

Figure 14-17 A tube of flow is defined by the streamlines that form the boundary of the tube.The volume flow rate must be the same for all cross sections of the tube of flow.

(volume flow rate, equation of continuity),

(14-24)

in which RV is the volume flow rate of the fluid (volume past a given point per unit time). Its SI unit is the cubic meter per second (m3/s). If the density r of the fluid is uniform, we can multiply Eq. 14-24 by that density to get the mass flow rate Rm (mass per unit time): Rm ! rRV ! rAv ! a constant

(mass flow rate).

(14-25)

The SI unit of mass flow rate is the kilogram per second (kg/s). Equation 14-25 says that the mass that flows into the tube segment of Fig. 14-15 each second must be equal to the mass that flows out of that segment each second.

Checkpoint 3 The figure shows a pipe and gives the volume flow rate (in cm3/s) and the direction of flow for all but one section. What are the volume flow 4 rate and the direction of flow for that section?

2

5

8

6 4

401

14-7 B E R NOU LLI’S EQUATION

Sample Problem 14.05 A water stream narrows as it falls Figure 14-18 shows how the stream of water emerging from a faucet “necks down” as it falls. This change in the horizontal cross-sectional area is characteristic of any laminar (nonturbulant) falling stream because the gravitational force increases the speed of the stream. Here the indicated crosssectional areas are A0 ! 1.2 cm2 and A ! 0.35 cm2. The two levels are separated by a vertical distance h ! 45 mm. What is the volume flow rate from the tap?

KEY IDEA The volume flow rate through the higher cross section must be the same as that through the lower cross section. Calculations: From Eq. 14-24, we have (14-26)

A0v0 ! Av,

where v0 and v are the water speeds at the levels corresponding to A0 and A. From Eq. 2-16 we can also write, because the water is falling freely with acceleration g, v2 ! v 20 # 2gh.

The volume flow per second here must match ...

Eliminating v between Eqs. 14-26 and 14-27 and solving for v0, we obtain v0 !

A0 h

! A

... the volume flow per second here.

Figure 14-18 As water falls from a tap, its speed increases. Because the volume flow rate must be the same at all horizontal cross sections of the stream, the stream must “neck down” (narrow).

(14-27)

2ghA2 A A20 % A2

(2)(9.8 m/s2)(0.045 m)(0.35 cm2)2 (1.2 cm2)2 % (0.35 cm2)2 A

! 0.286 m/s ! 28.6 cm/s.

From Eq. 14-24, the volume flow rate RV is then RV ! A0v0 ! (1.2 cm2)(28.6 cm/s) ! 34 cm3/s.

(Answer)

Additional examples, video, and practice available at WileyPLUS

14-7 BERNOULLI’S EQUATION Learning Objectives After reading this module, you should be able to . . .

14.21 Calculate the kinetic energy density in terms of a fluid’s density and flow speed. 14.22 Identify the fluid pressure as being a type of energy density. 14.23 Calculate the gravitational potential energy density.

14.24 Apply Bernoulli’s equation to relate the total energy density at one point on a streamline to the value at another point. 14.25 Identify that Bernoulli's equation is a statement of the conservation of energy.

Key Idea ● Applying the principle of conservation of mechanical energy to the flow of an ideal fluid leads to Bernoulli’s equation:

p # 12rv2 # rgy ! a constant along any tube of flow.

Bernoulli’s Equation Figure 14-19 represents a tube through which an ideal fluid is flowing at a steady rate. In a time interval 't, suppose that a volume of fluid 'V, colored purple in Fig. 14-19, enters the tube at the left (or input) end and an identical volume,

402

CHAPTE R 14 FLU I DS

y

colored green in Fig. 14-19, emerges at the right (or output) end. The emerging volume must be the same as the entering volume because the fluid is incompressible, with an assumed constant density r. Let y1, v1, and p1 be the elevation, speed, and pressure of the fluid entering at the left, and y2, v2, and p2 be the corresponding quantities for the fluid emerging at the right. By applying the principle of conservation of energy to the fluid, we shall show that these quantities are related by

L

v1 p1

t

y1 Input

p1 # 12 rv21 # rgy1 ! p2 # 12 rv22 # rgy2. x

(a) y

In general, the term 12rv2 is called the fluid’s kinetic energy density (kinetic energy per unit volume). We can also write Eq. 14-28 as p # 12 rv2 # rgy ! a constant

v2

(14-28)

(Bernoulli’s equation).

(14-29)

Equations 14-28 and 14-29 are equivalent forms of Bernoulli’s equation, after Daniel Bernoulli, who studied fluid flow in the 1700s.* Like the equation of continuity (Eq. 14-24), Bernoulli’s equation is not a new principle but simply the reformulation of a familiar principle in a form more suitable to fluid mechanics. As a check, let us apply Bernoulli’s equation to fluids at rest, by putting v1 ! v2 ! 0 in Eq. 14-28. The result is Eq. 14-7:

p2 Output y2 t + ∆t x (b)

Figure 14-19 Fluid flows at a steady rate through a length L of a tube, from the input end at the left to the output end at the right. From time t in (a) to time t # 't in (b), the amount of fluid shown in purple enters the input end and the equal amount shown in green emerges from the output end.

p2 ! p1 # rg(y1 % y2). A major prediction of Bernoulli’s equation emerges if we take y to be a constant (y ! 0, say) so that the fluid does not change elevation as it flows. Equation 14-28 then becomes p1 # 12 rv21 ! p2 # 12 rv22, (14-30) which tells us that: If the speed of a fluid element increases as the element travels along a horizontal streamline, the pressure of the fluid must decrease, and conversely.

Put another way, where the streamlines are relatively close together (where the velocity is relatively great), the pressure is relatively low, and conversely. The link between a change in speed and a change in pressure makes sense if you consider a fluid element that travels through a tube of various widths. Recall that the element’s speed in the narrower regions is fast and its speed in the wider regions is slow. By Newton’s second law, forces (or pressures) must cause the changes in speed (the accelerations). When the element nears a narrow region, the higher pressure behind it accelerates it so that it then has a greater speed in the narrow region. When it nears a wide region, the higher pressure ahead of it decelerates it so that it then has a lesser speed in the wide region. Bernoulli’s equation is strictly valid only to the extent that the fluid is ideal. If viscous forces are present, thermal energy will be involved, which here we neglect.

Proof of Bernoulli’s Equation Let us take as our system the entire volume of the (ideal) fluid shown in Fig. 14-19. We shall apply the principle of conservation of energy to this system as it moves from its initial state (Fig. 14-19a) to its final state (Fig. 14-19b). The fluid lying between the two vertical planes separated by a distance L in Fig. 14-19 does not change its properties during this process; we need be concerned only with changes that take place at the input and output ends. *For irrotational flow (which we assume), the constant in Eq. 14-29 has the same value for all points within the tube of flow; the points do not have to lie along the same streamline. Similarly, the points 1 and 2 in Eq. 14-28 can lie anywhere within the tube of flow.

403

14-7 B E R NOU LLI’S EQUATION

First, we apply energy conservation in the form of the work – kinetic energy theorem, W ! 'K, (14-31) which tells us that the change in the kinetic energy of our system must equal the net work done on the system. The change in kinetic energy results from the change in speed between the ends of the tube and is 'K ! 12 'm v 22 % 12 'm v 21 ! 12 r 'V(v 22 % v 21),

(14-32)

in which 'm (! r 'V) is the mass of the fluid that enters at the input end and leaves at the output end during a small time interval 't. The work done on the system arises from two sources. The work Wg done by the gravitational force ('m g: ) on the fluid of mass 'm during the vertical lift of the mass from the input level to the output level is Wg ! %'m g(y2 % y1) ! %rg 'V(y2 % y1).

(14-33)

This work is negative because the upward displacement and the downward gravitational force have opposite directions. Work must also be done on the system (at the input end) to push the entering fluid into the tube and by the system (at the output end) to push forward the fluid that is located ahead of the emerging fluid. In general, the work done by a force of magnitude F, acting on a fluid sample contained in a tube of area A to move the fluid through a distance 'x, is F 'x ! ( pA)('x) ! p(A 'x) ! p 'V. The work done on the system is then p1 'V, and the work done by the system is %p2 'V. Their sum Wp is Wp ! %p2 'V # p1 'V ! %( p2 % p1) 'V.

(14-34)

The work – kinetic energy theorem of Eq. 14-31 now becomes W ! Wg # Wp ! 'K. Substituting from Eqs. 14-32, 14-33, and 14-34 yields %rg 'V(y2 % y1) % 'V(p2 % p1) ! 12 r 'V(v 22 % v 21). This, after a slight rearrangement, matches Eq. 14-28, which we set out to prove.

Checkpoint 4 Water flows smoothly through the pipe shown in the figure, descending in the process. Rank the four numbered sections of pipe according to (a) the volume flow rate RV through them, (b) the flow speed v through them, and (c) the water pressure p within them, greatest first.

1 Flow

2 3

4

Sample Problem 14.06 Bernoulli principle of fluid through a narrowing pipe Ethanol of density r ! 791 kg/m3 flows smoothly through a horizontal pipe that tapers (as in Fig. 14-15) in crosssectional area from A1 ! 1.20 $ 10 %3 m2 to A2 ! A1/2.

The pressure difference between the wide and narrow sections of pipe is 4120 Pa. What is the volume flow rate RV of the ethanol?

404

CHAPTE R 14 FLU I DS

KEY IDEAS (1) Because the fluid flowing through the wide section of pipe must entirely pass through the narrow section, the volume flow rate RV must be the same in the two sections. Thus, from Eq. 14-24, RV ! v1A1 ! v2A2. (14-35) However, with two unknown speeds, we cannot evaluate this equation for RV. (2) Because the flow is smooth, we can apply Bernoulli’s equation. From Eq. 14-28, we can write p1 # 12 rv21 # rgy ! p2 # 12 rv22 # rgy,

(14-36)

where subscripts 1 and 2 refer to the wide and narrow sections of pipe, respectively, and y is their common elevation. This equation hardly seems to help because it does not contain the desired RV and it contains the unknown speeds v1 and v2. Calculations: There is a neat way to make Eq. 14-36 work for us: First, we can use Eq. 14-35 and the fact that A2 ! A1/2 to write v1 !

RV A1

and v2 !

RV 2RV . ! A2 A1

(14-37)

Then we can substitute these expressions into Eq. 14-36 to eliminate the unknown speeds and introduce the desired volume flow rate. Doing this and solving for RV yield RV ! A1

A

2( p1 % p2) . 3r

(14-38)

We still have a decision to make: We know that the pressure difference between the two sections is 4120 Pa, but does that mean that p1 % p2 is 4120 Pa or %4120 Pa? We could guess the former is true, or otherwise the square root in Eq. 14-38 would give us an imaginary number. However, let’s try some reasoning. From Eq. 14-35 we see that speed v2 in the narrow section (small A2) must be greater than speed v1 in the wider section (larger A1). Recall that if the speed of a fluid increases as the fluid travels along a horizontal path (as here), the pressure of the fluid must decrease. Thus, p1 is greater than p2, and p1 % p2 ! 4120 Pa. Inserting this and known data into Eq. 14-38 gives RV ! 1.20 $ 10%3 m2

(2)(4120 Pa) A (3)(791 kg/m3)

! 2.24 $ 10%3 m3/s.

Sample Problem 14.07 Bernoulli principle for a leaky water tank

(Answer)

p0

In the old West, a desperado fires a bullet into an open water tank (Fig. 14-20), creating a hole a distance h below the water surface.What is the speed v of the water exiting the tank?

p0

h y=0

KEY IDEAS (1) This situation is essentially that of water moving (downward) with speed v0 through a wide pipe (the tank) of crosssectional area A and then moving (horizontally) with speed v through a narrow pipe (the hole) of cross-sectional area a. (2) Because the water flowing through the wide pipe must entirely pass through the narrow pipe, the volume flow rate RV must be the same in the two “pipes.” (3) We can also relate v to v0 (and to h) through Bernoulli’s equation (Eq. 14-28). Calculations: From Eq. 14-24, and thus

RV ! av ! Av0 a v. v0 ! A

Because a - A, we see that v0 - v. To apply Bernoulli’s equation, we take the level of the hole as our reference level for measuring elevations (and thus gravitational potential energy). Noting that the pressure at the top of the tank and at the bullet hole is the atmospheric pressure p0 (because both places are exposed to the atmosphere), we write Eq. 14-28 as p0 # 12 rv20 # rgh ! p0 # 12 rv 2 # rg(0).

(14-39)

Figure 14-20 Water pours through a hole in a water tank, at a distance h below the water surface. The pressure at the water surface and at the hole is atmospheric pressure p0.

(Here the top of the tank is represented by the left side of the equation and the hole by the right side. The zero on the right indicates that the hole is at our reference level.) Before we solve Eq. 14-39 for v, we can use our result that v0 - v to simplify it: We assume that v20, and thus the term 1 2 2 rv 0 in Eq. 14-39, is negligible relative to the other terms, and we drop it. Solving the remaining equation for v then yields v ! 12gh.

(Answer)

This is the same speed that an object would have when falling a height h from rest.

Additional examples, video, and practice available at WileyPLUS

QU ESTIONS

Review & Summary

Pascal’s Principle A change in the pressure applied to an en-

Density The density r of any material is defined as the material’s mass per unit volume: r!

'm . 'V

(14-1)

m . V

(14-2)

Fluid Pressure

A fluid is a substance that can flow; it conforms to the boundaries of its container because it cannot withstand shearing stress. It can, however, exert a force perpendicular to its surface. That force is described in terms of pressure p: p!

'F , 'A

(14-3)

in which 'F is the force acting on a surface element of area 'A. If the force is uniform over a flat area, Eq. 14-3 can be written as p!

closed fluid is transmitted undiminished to every portion of the fluid and to the walls of the containing vessel.

Archimedes’ Principle When a body is fully or partially sub:

Usually, where a material sample is much larger than atomic dimensions, we can write Eq. 14-1 as r!

405

F . A

(14-4)

The force resulting from fluid pressure at a particular point in a fluid has the same magnitude in all directions. Gauge pressure is the difference between the actual pressure (or absolute pressure) at a point and the atmospheric pressure.

Pressure Variation with Height and Depth Pressure in a fluid at rest varies with vertical position y. For y measured positive upward, p2 ! p1 # rg(y1 % y2).

(14-7)

The pressure in a fluid is the same for all points at the same level. If h is the depth of a fluid sample below some reference level at which the pressure is p0, then the pressure in the sample is (14-8)

p ! p0 # rgh.

merged in a fluid, a buoyant force Fb from the surrounding fluid acts on the body. The force is directed upward and has a magnitude given by Fb ! mf g, (14-16) where mf is the mass of the fluid that has been displaced by the body (that is, the fluid that has been pushed out of the way by the body). When a body floats in a fluid, the magnitude Fb of the (upward) buoyant force on the body is equal to the magnitude Fg of the (downward) gravitational force on the body. The apparent weight of a body on which a buoyant force acts is related to its actual weight by weightapp ! weight % Fb.

(14-19)

Flow of Ideal Fluids An ideal fluid is incompressible and lacks viscosity, and its flow is steady and irrotational. A streamline is the path followed by an individual fluid particle. A tube of flow is a bundle of streamlines. The flow within any tube of flow obeys the equation of continuity: RV ! Av ! a constant,

(14-24)

in which RV is the volume flow rate, A is the cross-sectional area of the tube of flow at any point, and v is the speed of the fluid at that point. The mass flow rate Rm is Rm ! rRV ! rAv ! a constant.

(14-25)

Bernoulli’s Equation Applying the principle of conservation of mechanical energy to the flow of an ideal fluid leads to Bernoulli’s equation along any tube of flow: p # 12 rv2 # rgy ! a constant.

(14-29)

Questions 1 We fully submerge an irregular 3 kg lump of material in a certain fluid. The fluid that would have been in the space now occupied by the lump has a mass of 2 kg. (a) When we release the lump, does it move upward, move downward, or remain in place? (b) If we next fully submerge the lump in a less dense fluid and again release it, what does it do? 2 Figure 14-21 shows four situations in which a red liquid and a gray liquid are in a U-tube. In one situation the liquids cannot be in static equilibrium. (a) Which situation is that? (b) For the other three sit-

uations, assume static equilibrium. For each of them, is the density of the red liquid greater than, less than, or equal to the density of the gray liquid? 3 A boat with an anchor on board floats in a swimming pool that is somewhat wider than the boat. Does the pool water level move up, move down, or remain the same if the anchor is (a) dropped into the water or (b) thrown onto the surrounding ground? (c) Does the water level in the pool move upward, move downward, or remain the same if, instead, a cork is dropped from the boat into the water, where it floats? a c

(1)

(2)

(3)

Figure 14-21 Question 2.

(4)

4 Figure 14-22 shows a tank filled d with water. Five horizontal floors and ceilings are indicated; all have the same area and are located at b distances L, 2L, or 3L below the top of the tank. Rank them accorde ing to the force on them due to the Figure 14-22 Question 4. water, greatest first.

406

CHAPTE R 14 FLU I DS

Spout 5 The teapot effect. Water poured slowly from a teapot spout a can double back under the spout for b a considerable distance (held there by atmospheric pressure) before Water c d detaching and falling. In Fig. 14-23, flow the four points are at the top or botFigure 14-23 Question 5. tom of the water layers, inside or outside. Rank those four points according to the gauge pressure in the water there, most positive first.

water flows smoothly toward the right. The radii of the pipe sections are indicated. In which arrangements is the net work done on a unit volume of water moving from the leftmost section to the rightmost section (a) zero, (b) positive, and (c) negative? 8 A rectangular block is pushed Wapp face-down into three liquids, in turn. The apparent weight Wapp of a the block versus depth h in the three liquids is plotted in Fig. 14-26. b Rank the liquids according to their h weight per unit volume, greatest c first. Figure 14-26 Question 8. 9 Water flows smoothly in a hor-

6 Figure 14-24 shows three identical open-top containers filled to the brim with water; toy ducks float in two of them. Rank the containers and contents according to their weight, greatest first.

(a)

(b)

izontal pipe. Figure 14-27 shows the kinetic energy K of a water el- K ement as it moves along an x axis that runs along the pipe. Rank the x three lettered sections of the pipe A B C according to the pipe radius, greatFigure 14-27 Question 9. est first.

(c)

Figure 14-24 Question 6. 7 Figure 14-25 shows four arrangements of pipes through which

2.00R

R

2.00R

3.00R

(1)

2.00R

10 We have three containers with different liquids. The gauge pressure pg versus depth h is plotted in Fig. 14-28 for the liquids. In each container, we will fully submerge a rigid plastic bead. Rank the plots according to the magnitude of the buoyant force on the bead, greatest first.

R

pg

(2)

a

b c

2.00R

R

3.00R

R

(3)

3.00R

R

(4)

h

Figure 14-28 Question 10.

Figure 14-25 Question 7.

Problems Tutoring problem available (at instructor’s discretion) in WileyPLUS and WebAssign SSM • – •••

Worked-out solution available in Student Solutions Manual

WWW Worked-out solution is at

Number of dots indicates level of problem difficulty

ILW

Interactive solution is at

http://www.wiley.com/college/halliday

Additional information available in The Flying Circus of Physics and at flyingcircusofphysics.com

Module 14-1 Fluids, Density, and Pressure •1 ILW A fish maintains its depth in fresh water by adjusting the air content of porous bone or air sacs to make its average density the same as that of the water. Suppose that with its air sacs collapsed, a fish has a density of 1.08 g/cm3. To what fraction of its expanded body volume must the fish inflate the air sacs to reduce its density to that of water? •2 A partially evacuated airtight container has a tight-fitting lid of surface area 77 m2 and negligible mass. If the force required to remove the lid is 480 N and the atmospheric pressure is 1.0 $ 10 5 Pa, what is the internal air pressure? •3 SSM Find the pressure increase in the fluid in a syringe when a nurse applies a force of 42 N to the syringe’s circular piston, which has a radius of 1.1 cm.

•4 Three liquids that will not mix are poured into a cylindrical container. The volumes and densities of the liquids are 0.50 L, 2.6 g/cm3; 0.25 L, 1.0 g/cm3; and 0.40 L, 0.80 g/cm3. What is the force on the bottom of the container due to these liquids? One liter ! 1 L ! 1000 cm3. (Ignore the contribution due to the atmosphere.) •5 SSM An office window has dimensions 3.4 m by 2.1 m. As a result of the passage of a storm, the outside air pressure drops to 0.96 atm, but inside the pressure is held at 1.0 atm. What net force pushes out on the window? •6 You inflate the front tires on your car to 28 psi. Later, you measure your blood pressure, obtaining a reading of 120/80, the readings being in mm Hg. In metric countries (which is to say, most of the world), these pressures are customarily reported in kilopascals (kPa). In kilopascals, what are (a) your tire pressure and (b) your blood pressure?

407

PROB LE M S

••7 In 1654 Otto von Guericke, inventor of the air pump, gave a demonstration before the nobleF F men of the Holy Roman Empire in R which two teams of eight horses could not pull apart two evacuated brass hemispheres. (a) Assuming Figure 14-29 Problem 7. the hemispheres have (strong) thin walls, so that R in Fig. 14-29 may be considered both the inside : and outside radius, show that the force F required to pull apart the hemispheres has magnitude F ! pR2 'p, where 'p is the difference between the pressures outside and inside the sphere. (b) Taking R as 30 cm, the inside pressure as 0.10 atm, and the outside pressure as 1.00 atm, find the force magnitude the teams of horses would have had to exert to pull apart the hemispheres. (c) Explain why one team of horses could have proved the point just as well if the hemispheres were attached to a sturdy wall. Module 14-2 Fluids at Rest •8 The bends during flight. Anyone who scuba dives is advised not to fly within the next 24 h because the air mixture for diving can introduce nitrogen to the bloodstream. Without allowing the nitrogen to come out of solution slowly, any sudden air-pressure reduction (such as during airplane ascent) can result in the nitrogen forming bubbles in the blood, creating the bends, which can be painful and even fatal. Military special operation forces are especially at risk. What is the change in pressure on such a special-op soldier who must scuba dive at a depth of 20 m in seawater one day and parachute at an altitude of 7.6 km the next day? Assume that the average air density within the altitude range is 0.87 kg/m3. •9 Blood pressure in Argentinosaurus. (a) If this longnecked, gigantic sauropod had a head height of 21 m and a heart height of 9.0 m, what (hydrostatic) gauge pressure in its blood was required at the heart such that the blood pressure at the brain was 80 torr (just enough to perfuse the brain with blood)? Assume the blood had a density of 1.06 $ 103 kg/m3. (b) What was the blood pressure (in torr or mm Hg) at the feet? •10 The plastic tube in Fig. 14-30 has a cross-sectional area of 5.00 cm2. The tube is filled with water until the short arm (of length d ! 0.800 m) is full. Then the short arm is sealed and more water is gradually poured into the long arm. If the seal will pop off when the force on it exceeds 9.80 N, what total height of water in the long arm will put the seal on the verge of popping?

d

Figure 14-30 Problems 10 and 81.

•11 Giraffe bending to drink. In a giraffe with its head 2.0 m above its heart, and its heart 2.0 m above its feet, the (hydrostatic) gauge pressure in the blood at its heart is 250 torr.Assume that the giraffe stands upright and the blood density is 1.06 $ 103 kg/m3. In torr (or mm Hg), find the (gauge) blood pressure (a) at the brain (the pressure is enough to perfuse the brain with blood, to keep the giraffe from fainting) and (b) at the feet (the pressure must be countered by tight-fitting skin acting like a pressure stocking). (c) If the giraffe were to lower its head to drink from a pond without splaying its legs and moving slowly, what would be the increase in the blood pressure in the brain? (Such action would probably be lethal.) •12 The maximum depth dmax that a diver can snorkel is set by the density of the water and the fact that human lungs can func-

tion against a maximum pressure difference (between inside and outside the chest cavity) of 0.050 atm. What is the difference in dmax for fresh water and the water of the Dead Sea (the saltiest natural water in the world, with a density of 1.5 $ 10 3 kg/m3)? •13 At a depth of 10.9 km, the Challenger Deep in the Marianas Trench of the Pacific Ocean is the deepest site in any ocean. Yet, in 1960, Donald Walsh and Jacques Piccard reached the Challenger Deep in the bathyscaph Trieste. Assuming that seawater has a uniform density of 1024 kg/m3, approximate the hydrostatic pressure (in atmospheres) that the Trieste had to withstand. (Even a slight defect in the Trieste structure would have been disastrous.) •14 Calculate the hydrostatic difference in blood pressure between the brain and the foot in a person of height 1.83 m. The density of blood is 1.06 $ 10 3 kg/m3. •15 What gauge pressure must a machine produce in order to suck mud of density 1800 kg/m3 up a tube by a height of 1.5 m? •16 Snorkeling by humans and elephants. When a person snorkels, the lungs are connected directly to the atmosphere through the snorkel tube and thus are at atmospheric pressure. In atmospheres, what is the difference 'p between this internal air pressure Figure 14-31 Problem 16. and the water pressure against the body if the length of the snorkel tube is (a) 20 cm (standard situation) and (b) 4.0 m (probably lethal situation)? In the latter, the pressure difference causes blood vessels on the walls of the lungs to rupture, releasing blood into the lungs. As depicted in Fig. 14-31, an elephant can safely snorkel through its trunk while swimming with its lungs 4.0 m below the water surface because the membrane around its lungs contains connective tissue that holds and protects the blood vessels, preventing rupturing. •17 SSM Crew members attempt to escape from a damaged submarine 100 m below the surface. What force must be applied to a pop-out hatch, which is 1.2 m by 0.60 m, to push it out at that depth? Assume that the density of the ocean water is 1024 kg/m3 and the internal air pressure is at 1.00 atm. •18 In Fig. 14-32, an open tube of length L ! 1.8 m and cross-sectional area A ! 4.6 cm2 is fixed to the top of a cylindrical barrel of diameter D ! 1.2 m and height H ! 1.8 m. The barrel and tube are filled with water (to the top of the tube). Calculate the ratio of the hydrostatic force on the bottom of the barrel to the gravitational force on the water contained in the barrel. Why is that ratio not equal to 1.0? (You need not consider the atmospheric pressure.) ••19 A large aquarium of height 5.00 m is filled with fresh water to a depth of 2.00 m. One wall of the aquarium consists of thick plastic 8.00 m wide. By how much does the total force on that wall increase if the aquarium is next filled to a depth of 4.00 m?

A

L

FRESH

WATER SUITABLE FOR DRINKING

D

Figure 14-32 Problem 18.

H

408

CHAPTE R 14 FLU I DS

d ••20 The L-shaped fish tank shown in Fig. 14-33 is filled with water and is open at d 2d the top. If d ! 5.0 m, what is the (total) force exerted by the water (a) on face A and (b) on face B? 3d A ••21 SSM Two identical cylindrical vesd B sels with their bases at the same level each d 3 contain a liquid of density 1.30 $ 10 d kg/m3. The area of each base is 4.00 cm2, 2d but in one vessel the liquid height is 0.854 Figure 14-33 m and in the other it is 1.560 m. Find the Problem 20. work done by the gravitational force in equalizing the levels when the two vessels are connected.

••22 g-LOC in dogfights.When a pilot takes a tight turn at high speed in a modern fighter airplane, the blood pressure at the brain level decreases, blood no longer perfuses the brain, and the blood in the brain drains. If the heart maintains the (hydrostatic) gauge pressure in the aorta at 120 torr (or mm Hg) when the pilot undergoes a horizontal centripetal acceleration of 4g, what is the blood pressure (in torr) at the brain, 30 cm radially inward from the heart? The perfusion in the brain is small enough that the vision switches to black and white and narrows to “tunnel vision” and the pilot can undergo g-LOC (“ginduced loss of consciousness”). Blood density is 1.06 $ 103 kg/m3. ••23 In analyzing certain geoMountain logical features, it is often approH priate to assume that the pressure at some horizontal level of compensation, deep inside Earth, is the Continent T 3 same over a large region and is 2.9 g/cm equal to the pressure due to the Mantle Root gravitational force on the overlyD 3.3 g/cm3 y ing material. Thus, the pressure on the level of compensation is given Compensation by the fluid pressure formula. This level b a model requires, for one thing, that mountains have roots of continenFigure 14-34 Problem 23. tal rock extending into the denser mantle (Fig. 14-34). Consider a mountain of height H ! 6.0 km on a continent of thickness T ! 32 km. The continental rock has a density of 2.9 g /cm3, and beneath this rock the mantle has a density of 3.3 g /cm3. Calculate the depth D of the root. (Hint: Set the pressure at points a and b equal; the depth y of the level of compensation will cancel out.) •••24 In Fig. 14-35, water stands W at depth D ! 35.0 m behind the vertical upstream face of a dam of width W ! 314 m. Find (a) the net D horizontal force on the dam from the gauge pressure of the water and O (b) the net torque due to that force Figure 14-35 Problem 24. about a horizontal line through O parallel to the (long) width of the dam.This torque tends to rotate the dam around that line, which would cause the dam to fail. (c) Find the moment arm of the torque. Module 14-3 Measuring Pressure •25 In one observation, the column in a mercury barometer (as is shown in Fig. 14-5a) has a measured height h of 740.35 mm. The temperature is %5.0(C, at which temperature the density of mercury r is 1.3608 $ 104 kg/m3.The free-fall acceleration g at the site of the barom-

eter is 9.7835 m/s2.What is the atmospheric pressure at that site in pascals and in torr (which is the common unit for barometer readings)? •26 To suck lemonade of density 1000 kg/m3 up a straw to a maximum height of 4.0 cm, what minimum gauge pressure (in atmospheres) must you produce in your lungs? ••27 SSM What would be the height of the atmosphere if the air density (a) were uniform and (b) decreased linearly to zero with height? Assume that at sea level the air pressure is 1.0 atm and the air density is 1.3 kg/m3. Module 14-4 Pascal’s Principle •28 A piston of cross-sectional area a is used in a hydraulic press to F f exert a small force of magnitude f on A the enclosed liquid. A connecting a pipe leads to a larger piston of crosssectional area A (Fig. 14-36). (a) What force magnitude F will the larger piston sustain without moving? (b) If Figure 14-36 Problem 28. the piston diameters are 3.80 cm and 53.0 cm, what force magnitude on the small piston will balance a 20.0 kN force on the large piston? Beam ••29 In Fig. 14-37, a spring of spring Container constant 3.00 $ 10 4 N/m is between a Spring rigid beam and the output piston of a hydraulic lever. An empty container with negligible mass sits on the input piston.The input piston has area Ai, and the output piston has area 18.0Ai. Initially the spring is at its rest length. Figure 14-37 Problem 29. How many kilograms of sand must be (slowly) poured into the container to compress the spring by 5.00 cm?

Module 14-5 Archimedes’ Principle •30 A 5.00 kg object is released from rest while fully submerged in a liquid. The liquid displaced by the submerged object has a mass of 3.00 kg. How far and in what direction does the object move in 0.200 s, assuming that it moves freely and that the drag force on it from the liquid is negligible? •31 SSM A block of wood floats in fresh water with two-thirds of its volume V submerged and in oil with 0.90V submerged. Find the density of (a) the wood and (b) the oil. •32 In Fig. 14-38, a cube of edge length L ! 0.600 m and mass 450 kg is suspended by a rope in an open L/2 tank of liquid of density 1030 kg/m3. Find (a) the magnitude of the total downward force on the top of the L cube from the liquid and the atmosphere, assuming atmospheric pressure is 1.00 atm, (b) the magnitude Figure 14-38 Problem 32. of the total upward force on the bottom of the cube, and (c) the tension in the rope. (d) Calculate the magnitude of the buoyant force on the cube using Archimedes’ principle. What relation exists among all these quantities? •33 SSM An iron anchor of density 7870 kg/m3 appears 200 N lighter in water than in air. (a) What is the volume of the anchor? (b) How much does it weigh in air? •34

A boat floating in fresh water displaces water weighing

PROB LE M S

35.6 kN. (a) What is the weight of the water this boat displaces when floating in salt water of density 1.10 $ 10 3 kg/m3? (b) What is the difference between the volume of fresh water displaced and the volume of salt water displaced?

••36 In Fig. 14-39a, a rectangular block is gradually pushed face-down into a liquid. The block has height d; on the bottom and top the face area is A ! 5.67 cm2. Figure 14-39b gives the apparent weight Wapp of the block as a function of the depth h of its lower face. The scale on the vertical axis is set by Ws ! 0.20 N. What is the density of the liquid?

Wapp (N)

•35 Three children, each of weight 356 N, make a log raft by lashing together logs of diameter 0.30 m and length 1.80 m. How many logs will be needed to keep them afloat in fresh water? Take the density of the logs to be 800 kg/m3.

d (a)

Ws

0

1 h (cm)

2

(b)

Figure 14-39 Problem 36.

••37 ILW A hollow spherical iron shell floats almost completely submerged in water.The outer diameter is 60.0 cm, and the density of iron is 7.87 g/cm3. Find the inner diameter. Ks

K (J)

••38 A small solid ball is released from rest while fully submerged in a liquid and then its kinetic energy is measured when it has moved 4.0 cm in the liquid. Figure 14-40 gives 3 0 1 2 the results after many liquids are used: ρ liq (g/cm3) The kinetic energy K is plotted versus the liquid density rliq, and Ks ! 1.60 J Figure 14-40 Problem 38. sets the scale on the vertical axis. What are (a) the density and (b) the volume of the ball? ••39 SSM WWW A hollow sphere of inner radius 8.0 cm and outer radius 9.0 cm floats half-submerged in a liquid of density 800 kg/m3. (a) What is the mass of the sphere? (b) Calculate the density of the material of which the sphere is made. ••40 Lurking alligators. An alligator waits for prey by floating with only the top of its head exposed, so that the prey cannot easily see it. Figure 14-41 Problem 40. One way it can adjust the extent of sinking is by controlling the size of its lungs. Another way may be by swallowing stones (gastrolithes) that then reside in the stomach. Figure 14-41 shows a highly simplified model (a “rhombohedron gater”) of mass 130 kg that roams with its head partially exposed. The top head surface has area 0.20 m2. If the alligator were to swallow stones with a total mass of 1.0% of its body mass (a typical amount), how far would it sink? ••41 What fraction of the volume of an iceberg (density 917 kg/m3) would be visible if the iceberg floats (a) in the ocean (salt water, density 1024 kg/m3) and (b) in a river (fresh water, density 1000 kg/m3)? (When salt water freezes to form ice, the salt is excluded. So, an iceberg could provide fresh water to a community.) ••42 A flotation device is in the shape of a right cylinder, with a height of 0.500 m and a face area of 4.00 m2 on top and bottom, and its density is 0.400 times that of fresh water. It is initially held fully submerged in fresh water, with its top face at the water surface.Then

409

it is allowed to ascend gradually until it begins to float. How much work does the buoyant force do on the device during the ascent? ••43 When researchers find a reasonably complete fossil of a dinosaur, they can determine the mass and weight of the living dinosaur with a scale model sculpted from plastic and based on the dimensions of the fossil bones. The scale of the model is 1/20; that is, lengths are 1/20 actual length, areas are (1/20)2 actual Figure 14-42 Problem 43. areas, and volumes are (1/20)3 actual volumes. First, the model is suspended from one arm of a balance and weights are added to the other arm until equilibrium is reached. Then the model is fully submerged in water and enough weights are removed from the second arm to reestablish equilibrium (Fig. 14-42). For a model of a particular T. rex fossil, 637.76 g had to be removed to reestablish equilibrium. What was the volume of (a) the model and (b) the actual T. rex? (c) If the density of T. rex was approximately the density of water, what was its mass? ••44 A wood block (mass 3.67 kg, density 600 kg/m3) is fitted with lead (density 1.14 $ 104 kg/m3) so that it floats in water with 0.900 of its volume submerged. Find the lead mass if the lead is fitted to the block’s (a) top and (b) bottom. ••45 An iron casting containing a number of cavities weighs 6000 N in air and 4000 N in water. What is the total cavity volume in the casting? The density of solid iron is 7.87 g/cm3. ••46 Suppose that you release a small ball from rest at a depth of 0.600 m below the surface in a pool of water. If the density of the ball is 0.300 that of water and if the drag force on the ball from the water is negligible, how high above the water surface will the ball shoot as it emerges from the water? (Neglect any transfer of energy to the splashing and waves produced by the emerging ball.) ••47 The volume of air space in the passenger compartment of an 1800 kg car is 5.00 m3. The volume of the motor and front wheels is 0.750 m3, and the volume of the rear wheels, gas tank, and trunk is 0.800 m3 ; water cannot enter these two regions. The car rolls into a lake. (a) At first, no water enters the passenger compartment. How much of the car, in cubic meters, is below the water surface with the car floating (Fig. 14-43)? (b) As water slowly enters, the car sinks. How many cubic meters of water are in the car as it disappears below the water surface? (The car, with a heavy load in the trunk, remains horizontal.)

Figure 14-43 Problem 47. •••48 Figure 14-44 shows an iron ball suspended by thread of negligible mass from an upright cylinder that floats partially submerged in water. The cylinder has a height of 6.00 cm, a face area of 12.0 cm2 on the top and bottom, and a density of 0.30 g/cm3, and 2.00 cm of its height is above Figure 14-44 the water surface. What is the radius of the iron Problem 48. ball?

410

CHAPTE R 14 FLU I DS

Module 14-6 The Equation of Continuity •49 Canal effect. Figure 14-45 b a i shows an anchored barge that extends across a canal by distance vi d ! 30 m and into the water by disD d tance b ! 12 m. The canal has a width D ! 55 m, a water depth H ! 14 m, and a uniform water-flow h speed vi ! 1.5 m/s. Assume that the vi b flow around the barge is uniform. As H the water passes the bow, the water level undergoes a dramatic dip Figure 14-45 Problem 49. known as the canal effect. If the dip has depth h ! 0.80 m, what is the water speed alongside the boat through the vertical cross sections at (a) point a and (b) point b? The erosion due to the speed increase is a common concern to hydraulic engineers. A

B

•50 Figure 14-46 shows two ? sections of an old pipe system that runs through a hill, with dA dB D distances dA ! dB ! 30 m and D ! 110 m. On each side of Figure 14-46 Problem 50. the hill, the pipe radius is 2.00 cm. However, the radius of the pipe inside the hill is no longer known.To determine it, hydraulic engineers first establish that water flows through the left and right sections at 2.50 m/s. Then they release a dye in the water at point A and find that it takes 88.8 s to reach point B. What is the average radius of the pipe within the hill? •51 SSM A garden hose with an internal diameter of 1.9 cm is connected to a (stationary) lawn sprinkler that consists merely of a container with 24 holes, each 0.13 cm in diameter. If the water in the hose has a speed of 0.91 m/s, at what speed does it leave the sprinkler holes? •52 Two streams merge to form a river. One stream has a width of 8.2 m, depth of 3.4 m, and current speed of 2.3 m/s. The other stream is 6.8 m wide and 3.2 m deep, and flows at 2.6 m/s. If the river has width 10.5 m and speed 2.9 m/s, what is its depth? ••53 SSM Water is pumped steadily out of a flooded basement at 5.0 m/s through a hose of radius 1.0 cm, passing through a window 3.0 m above the waterline. What is the pump’s power? ••54 The water flowing through a 1.9 cm (inside diameter) pipe flows out through three 1.3 cm pipes. (a) If the flow rates in the three smaller pipes are 26, 19, and 11 L/min, what is the flow rate in the 1.9 cm pipe? (b) What is the ratio of the speed in the 1.9 cm pipe to that in the pipe carrying 26 L/min? Module 14-7 Bernoulli’s Equation •55 How much work is done by pressure in forcing 1.4 m3 of water through a pipe having an internal diameter of 13 mm if the difference in pressure at the two ends of the pipe is 1.0 atm? •56 Suppose that two tanks, 1 and 2, each with a large opening at the top, contain different liquids. A small hole is made in the side of each tank at the same depth h below the liquid surface, but the hole in tank 1 has half the cross-sectional area of the hole in tank 2. (a) What is the ratio r1/r2 of the densities of the liquids if the mass flow rate is the same for the two holes? (b) What is the ratio RV1/RV2 of the volume flow rates from the two tanks? (c) At one instant, the liquid in tank 1 is 12.0 cm above the hole. If the tanks are to have equal volume flow rates, what height above the hole must the liquid in tank 2 be just then?

•57 SSM A cylindrical tank with a large diameter is filled with water to a depth D ! 0.30 m. A hole of cross-sectional area A ! 6.5 cm2 in the bottom of the tank allows water to drain out. (a) What is the drainage rate in cubic meters per second? (b) At what distance below the bottom of the tank is the cross-sectional area of the stream equal to one-half the area of the hole? •58 The intake in Fig. 14-47 has cross-sectional area of 0.74 m2 and water flow at 0.40 m/s. At the outlet, distance D ! 180 m below the intake, the cross-sectional area is smaller than at the intake and the water flows out at 9.5 m/s into equipment. What is the pressure difference between inlet and outlet?

Reservoir

D

Generator building

Intake

Outlet

Figure 14-47 Problem 58.

•59 SSM Water is moving with a speed of 5.0 m/s through a pipe with a cross-sectional area of 4.0 cm2. The water gradually descends 10 m as the pipe cross-sectional area increases to 8.0 cm2. (a) What is the speed at the lower level? (b) If the pressure at the upper level is 1.5 $ 10 5 Pa, what is the pressure at the lower level? •60 Models of torpedoes are sometimes tested in a horizontal pipe of flowing water, much as a wind tunnel is used to test model airplanes. Consider a circular pipe of internal diameter 25.0 cm and a torpedo model aligned along the long axis of the pipe.The model has a 5.00 cm diameter and is to be tested with water flowing past it at 2.50 m/s. (a) With what speed must the water flow in the part of the pipe that is unconstricted by the model? (b) What will the pressure difference be between the constricted and unconstricted parts of the pipe? •61 ILW A water pipe having a 2.5 cm inside diameter carries water into the basement of a house at a speed of 0.90 m/s and a pressure of 170 kPa. If the pipe tapers to 1.2 cm and rises to the second floor 7.6 m above the input point, what are the (a) speed and (b) water pressure at the second floor? ••62 A pitot tube (Fig. 14-48) is used to determine the airspeed of an airplane. It consists of an outer tube with a number of small holes B (four are shown) that allow air into the tube; that tube is connected to one arm of a U-tube. The other arm of the U-tube is connected to hole A at the front end of the device, which points in the direction the plane is headed. At A the air becomes stagnant so that vA ! 0. At B, however, the speed of the air presumably equals the airspeed v of the plane. (a) Use Bernoulli’s equation to show that 2rgh , v! A rair

where r is the density of the liquid in the U-tube and h is the difference in the liquid levels in that tube. (b) Suppose that the tube contains alcohol and the level difference h is 26.0 cm. What is the plane’s speed relative to the air? The density of the air is 1.03 kg/m3 and that of alcohol is 810 kg/m3. v B

ρair Hole A

Figure 14-48 Problems 62 and 63.

Air

B

Liquid

h

ρ

411

PROB LE M S

••64 In Fig. 14-49, water flows v1 v2 through a horizontal pipe and then out d2 d1 into the atmosphere at a speed v1 ! 15 m/s. The diameters of the left and right Figure 14-49 Problem 64. sections of the pipe are 5.0 cm and 3.0 cm. (a) What volume of water flows into the atmosphere during a 10 min period? In the left section of the pipe, what are (b) the speed v2 and (c) the gauge pressure? ••65 SSM WWW A venturi meter is used to measure the flow speed of a fluid in a pipe. The meter is connected between two sections of the pipe (Fig. 14-50); the cross-sectional area A of the entrance and exit of the meter matches the pipe’s cross-sectional area. Between the entrance and exit, the fluid flows from the pipe with speed V and then through a narrow “throat” of crosssectional area a with speed v. A manometer connects the wider portion of the meter to the narrower portion. The change in the fluid’s speed is accompanied by a change 'p in the fluid’s pressure, which causes a height difference h of the liquid in the two arms of the manometer. (Here 'p means pressure in the throat minus pressure in the pipe.) (a) By applying Bernoulli’s equation and the equation of continuity to points 1 and 2 in Fig. 14-50, show that V!

2a 2 'p , A 6(a 2 % A2)

where r is the density of the fluid. (b) Suppose that the fluid is fresh water, that the cross-sectional areas are 64 cm2 in the pipe and 32 cm2 in the throat, and that the pressure is 55 kPa in the pipe and 41 kPa in the throat. What is the rate of water flow in cubic meters per second? Meter entrance

v a

A V

Pipe

Meter exit

Venturi meter A 2

1

Pipe

h Manometer

Figure 14-50 Problems 65 and 66. ••66 Consider the venturi tube of Problem 65 and Fig. 14-50 without the manometer. Let A equal 5a. Suppose the pressure p1 at A is 2.0 atm. Compute the values of (a) the speed V at A and (b) the speed v at a that make the pressure p2 at a equal to zero. (c) Compute the corresponding volume flow rate if the diameter at A is 5.0 cm. The phenomenon that occurs at a when p2 falls to nearly zero is known as cavitation.The water vaporizes into small bubbles. ••67 ILW In Fig. 14-51, the fresh water behind a reservoir dam has depth D ! 15 m. A horizontal pipe 4.0 cm in diameter passes through the dam at depth d ! 6.0 m. A plug secures the pipe

opening. (a) Find the magnitude of the frictional force between plug and pipe wall. (b) The plug is removed. What water volume exits the pipe in 3.0 h? ••68 Fresh water flows horizontally from pipe section 1 of cross-sectional area A1 into pipe section 2 of cross-sectional area A2. Figure 14-52 gives a plot of the pressure difference p2 % p1 versus the inverse area squared A1%2 that would be expected for a volume flow rate of a certain value if the water flow were laminar under all circumstances. The scale on the vertical axis is set by ∆ps ! 300 kN/m2. For the conditions of the figure, what are the values of (a) A2 and (b) the volume flow rate?

d D

Figure 14-51 Problem 67. p2 – p1 (kN/m2)

••63 A pitot tube (see Problem 62) on a high-altitude aircraft measures a differential pressure of 180 Pa. What is the aircraft’s airspeed if the density of the air is 0.031 kg/m3?

∆ps 0

16

–∆ps

32

A1–2 (m–4)

Figure 14-52 Problem 68. ••69 A liquid of density 900 kg/m3 flows through a horizontal pipe that has a cross-sectional area of 1.90 $ 10 %2 m2 in region A and a cross-sectional area of 9.50 $ 10 %2 m2 in region B. The pressure difference between the two regions is 7.20 $ 10 3 Pa. What are (a) the volume flow rate and (b) the mass flow rate? ••70 In Fig. 14-53, water flows steadily from the left pipe section (radius r1 ! 2.00R), through the midr1 R r3 dle section (radius R), and into the right section (radius r3 ! 3.00R). The Figure 14-53 Problem 70. speed of the water in the middle section is 0.500 m/s. What is the net work done on 0.400 m3 of the water as it moves from the left section to the right section? ••71 Figure 14-54 shows a stream of water flowing through a hole at depth h ! 10 cm in a tank holding water to height H ! 40 cm. (a) At what distance x does the stream strike the floor? (b) At what depth should a second hole be made to give the same value of x? (c) At what depth should a hole be made to maximize x?

h

v

H

x

Figure 14-54 Problem 71. •••72 A very simplified schematic of the rain drainage system for a home is shown in Fig. 14-55. Rain falling on the slanted roof runs off into gutters around the roof edge; it then drains through downspouts (only one is shown) into a main drainage pipe M below the basement, which carries the water to an even larger pipe below the street. In Fig. 14-55, a floor drain in the basement is w also connected to drainage pipe M. Suppose the following apply: (1) the downspouts have height h1 ! 11 m, (2) the floor drain has height h2 ! 1.2 m, (3) pipe M has radius 3.0 cm, (4) the house has side width w ! 30 m and front length L ! 60 m, (5) all

h1 h2

Floor drain

M

Figure 14-55 Problem 72.

412

CHAPTE R 14 FLU I DS

the water striking the roof goes through pipe M, (6) the initial speed of the water in a downspout is negligible, and (7) the wind speed is negligible (the rain falls vertically). At what rainfall rate, in centimeters per hour, will water from pipe M reach the height of the floor drain and threaten to flood the basement? Additional Problems 73 About one-third of the body of a person floating in the Dead Sea will be above the waterline. Assuming that the human body density is 0.98 g/cm3, find the density of the water in the Dead Sea. (Why is it so much greater than 1.0 g/cm3?) 74 A simple open U-tube contains mercury. When 11.2 cm of water is poured into the right arm of the tube, how high above its initial level does the mercury rise in the left arm? 75 If a bubble in sparkling water accelerates upward at the rate of 0.225 m/s2 and has a radius of 0.500 mm, what is its mass? Assume that the drag force on the bubble is negligible. 76 Suppose that your body has a uniform density of 0.95 times that of water. (a) If you float in a swimming pool, what fraction of your body’s volume is above the water surface? Quicksand is a fluid produced when water is forced up into sand, moving the sand grains away from one another so they are no longer locked together by friction. Pools of quicksand can form when water drains underground from hills into valleys where there are sand pockets. (b) If you float in a deep pool of quicksand that has a density 1.6 times that of water, what fraction of your body’s volume is above the quicksand surface? (c) Are you unable to breathe? 77 A glass ball of radius 2.00 cm sits at the bottom of a container of milk that has a density of 1.03 g/cm3. The normal force on the ball from the container’s lower surface has magnitude 9.48 $ 10 %2 N. What is the mass of the ball? 78 Caught in an avalanche, a skier is fully submerged in flowing snow of density 96 kg/m3. Assume that the average density of the skier, clothing, and skiing equipment is 1020 kg/m3. What percentage of the gravitational force on the skier is offset by the buoyant force from the snow? 79 An object hangs from a spring balance. The balance registers 30 N in air, 20 N when this object is immersed in water, and 24 N when the object is immersed in another liquid of unknown density. What is the density of that other liquid? 80 In an experiment, a rectangular block with height h is allowed to float in four separate liquids. In the first liquid, which is water, it floats fully submerged. In liquids A, B, and C, it floats with heights h/2, 2h/3, and h/4 above the liquid surface, respectively. What are the relative densities (the densities relative to that of water) of (a) A, (b) B, and (c) C? 81 SSM Figure 14-30 shows a modified U-tube: the right arm is shorter than the left arm. The open end of the right arm is height d ! 10.0 cm above the laboratory bench. The radius throughout the tube is 1.50 cm. Water is gradually poured into the open end of the left arm until the water begins to flow out the open end of the right arm. Then a liquid of density 0.80 g/cm3 is gradually added to the left arm until its height in that arm is 8.0 cm (it does not mix with the water). How much water flows out of the right arm? 82 What is the acceleration of a rising hot-air balloon if the ratio of the air density outside the balloon to that inside is 1.39? Neglect the mass of the balloon fabric and the basket.

B 83 Figure 14-56 shows a siphon, which is a device for h1 removing liquid from a container. Tube ABC must initially be filled, but once this has been done, liquid will flow through the tube until the d liquid surface in the container is A level with the tube opening at A. The liquid has density 1000 kg/m3 and negligible viscosity. The dish2 tances shown are h1 ! 25 cm, d ! 12 cm, and h2 ! 40 cm. (a) With what speed does the liquid emerge C from the tube at C? (b) If the atFigure 14-56 Problem 83. mospheric pressure is 1.0 $ 10 5 Pa, what is the pressure in the liquid at the topmost point B? (c) Theoretically, what is the greatest possible height h1 that a siphon can lift water?

84 When you cough, you expel air at high speed through the trachea and upper bronchi so that the air will remove excess mucus lining the pathway.You produce the high speed by this procedure:You breathe in a large amount of air, trap it by closing the glottis (the narrow opening in the larynx), increase the air pressure by contracting the lungs, partially collapse the trachea and upper bronchi to narrow the pathway, and then expel the air through the pathway by suddenly reopening the glottis. Assume that during the expulsion the volume flow rate is 7.0 $ 10 %3 m3/s. What multiple of 343 m/s (the speed of sound vs) is the airspeed through the trachea if the trachea diameter (a) remains its normal value of 14 mm and (b) contracts to 5.2 mm? 85 A tin can has a total volume of 1200 cm3 and a mass of 130 g. How many grams of lead shot of density 11.4 g/cm3 could it carry without sinking in water? 86 The tension in a string holding a solid block below the surface of a liquid (of density greater than the block) is T0 when the container (Fig. 14-57) is at rest. When the container is given an upward acceleration of 0.250g, what multiple of T0 gives the tension in the string?

Figure 14-57 Problem 86.

87 What is the minimum area (in square meters) of the top surface of an ice slab 0.441 m thick floating on fresh water that will hold up a 938 kg automobile? Take the densities of ice and fresh water to be 917 kg/m3 and 998 kg/m3, respectively. 88 A 8.60 kg sphere of radius 6.22 cm is at a depth of 2.22 km in seawater that has an average density of 1025 kg/m3. What are the (a) gauge pressure, (b) total pressure, and (c) corresponding total force compressing the sphere’s surface? What are (d) the magnitude of the buoyant force on the sphere and (e) the magnitude of the sphere’s acceleration if it is free to move? Take atmospheric pressure to be 1.01 $ 105 Pa. 89 (a) For seawater of density 1.03 g/cm3, find the weight of water on top of a submarine at a depth of 255 m if the horizontal cross-sectional hull area is 2200.0 m2. (b) In atmospheres, what water pressure would a diver experience at this depth? 90 The sewage outlet of a house constructed on a slope is 6.59 m below street level. If the sewer is 2.16 m below street level, find the minimum pressure difference that must be created by the sewage pump to transfer waste of average density 1000.00 kg/m3 from outlet to sewer.

C

H

A

P

T

E

R

1

5

Oscillations 15-1 SIMPLE HARMONIC MOTION Learning Objectives After reading this module, you should be able to . . .

15.01 Distinguish simple harmonic motion from other types of periodic motion. 15.02 For a simple harmonic oscillator, apply the relationship between position x and time t to calculate either if given a value for the other. 15.03 Relate period T, frequency f, and angular frequency v. 15.04 Identify (displacement) amplitude xm, phase constant (or phase angle) f, and phase vt # f. 15.05 Sketch a graph of the oscillator’s position x versus time t, identifying amplitude xm and period T. 15.06 From a graph of position versus time, velocity versus time, or acceleration versus time, determine the amplitude of the plot and the value of the phase constant f. 15.07 On a graph of position x versus time t describe the effects of changing period T, frequency f, amplitude xm, or phase constant f. 15.08 Identify the phase constant f that corresponds to the starting time (t ! 0) being set when a particle in SHM is at an extreme point or passing through the center point. 15.09 Given an oscillator’s position x(t) as a function of time, find its velocity v(t) as a function of time, identify the velocity amplitude vm in the result, and calculate the velocity at any given time.

15.10 Sketch a graph of an oscillator’s velocity v versus time t, identifying the velocity amplitude vm. 15.11 Apply the relationship between velocity amplitude vm, angular frequency v, and (displacement) amplitude xm. 15.12 Given an oscillator’s velocity v(t) as a function of time, calculate its acceleration a(t) as a function of time, identify the acceleration amplitude am in the result, and calculate the acceleration at any given time. 15.13 Sketch a graph of an oscillator’s acceleration a versus time t, identifying the acceleration amplitude am. 15.14 Identify that for a simple harmonic oscillator the acceleration a at any instant is always given by the product of a negative constant and the displacement x just then. 15.15 For any given instant in an oscillation, apply the relationship between acceleration a, angular frequency v, and displacement x. 15.16 Given data about the position x and velocity v at one instant, determine the phase vt # f and phase constant f. 15.17 For a spring–block oscillator, apply the relationships between spring constant k and mass m and either period T or angular frequency v. 15.18 Apply Hooke’s law to relate the force F on a simple harmonic oscillator at any instant to the displacement x of the oscillator at that instant.

Key Ideas ● The frequency f of periodic, or oscillatory, motion is the

number of oscillations per second. In the SI system, it is measured in hertz: 1 Hz ! 1 s%1. ● The period T is the time required for one complete oscillation, or cycle. It is related to the frequency by T ! 1/f. ● In simple harmonic motion (SHM), the displacement x(t) of a particle from its equilibrium position is described by the equation x ! xm cos(vt # f)

(displacement),

in which xm is the amplitude of the displacement, vt # f is the phase of the motion, and f is the phase constant. The angular frequency v is related to the period and frequency of the motion by v ! 2p/T ! 2pf. ● Differentiating x(t) leads to equations for the particle’s SHM velocity and acceleration as functions of time:

and

v ! %vxm sin(vt # f) a ! %v2xm cos(vt # f)

(velocity) (acceleration).

In the velocity function, the positive quantity vxm is the velocity amplitude vm. In the acceleration function, the positive quantity v 2xm is the acceleration amplitude am. ● A particle with mass m that moves under the influence of a Hooke’s law restoring force given by F ! %kx is a linear simple harmonic oscillator with v! and

k Am

T ! 2p

m A k

(angular frequency)

(period).

413

414

CHAPTE R 15 OSCI LL ATIONS

What Is Physics? Our world is filled with oscillations in which objects move back and forth repeatedly. Many oscillations are merely amusing or annoying, but many others are dangerous or financially important. Here are a few examples: When a bat hits a baseball, the bat may oscillate enough to sting the batter’s hands or even to break apart. When wind blows past a power line, the line may oscillate (“gallop” in electrical engineering terms) so severely that it rips apart, shutting off the power supply to a community. When an airplane is in flight, the turbulence of the air flowing past the wings makes them oscillate, eventually leading to metal fatigue and even failure. When a train travels around a curve, its wheels oscillate horizontally (“hunt” in mechanical engineering terms) as they are forced to turn in new directions (you can hear the oscillations). When an earthquake occurs near a city, buildings may be set oscillating so severely that they are shaken apart. When an arrow is shot from a bow, the feathers at the end of the arrow manage to snake around the bow staff without hitting it because the arrow oscillates. When a coin drops into a metal collection plate, the coin oscillates with such a familiar ring that the coin’s denomination can be determined from the sound. When a rodeo cowboy rides a bull, the cowboy oscillates wildly as the bull jumps and turns (at least the cowboy hopes to be oscillating). The study and control of oscillations are two of the primary goals of both physics and engineering. In this chapter we discuss a basic type of oscillation called simple harmonic motion. Heads Up. This material is quite challenging to most students. One reason is that there is a truckload of definitions and symbols to sort out, but the main reason is that we need to relate an object’s oscillations (something that we can see or even experience) to the equations and graphs for the oscillations. Relating the real, visible motion to the abstraction of an equation or graph requires a lot of hard work.

Simple Harmonic Motion x –xm

0

+xm

Figure 15-1 A particle repeatedly oscillates left and right along an x axis, between extreme points xm and %xm.

Figure 15-1 shows a particle that is oscillating about the origin of an x axis, repeatedly going left and right by identical amounts. The frequency f of the oscillation is the number of times per second that it completes a full oscillation (a cycle) and has the unit of hertz (abbreviated Hz), where 1 hertz ! 1 Hz ! 1 oscillation per second ! 1 s%1.

(15-1)

The time for one full cycle is the period T of the oscillation, which is T!

1 . f

(15-2)

Any motion that repeats at regular intervals is called periodic motion or harmonic motion. However, here we are interested in a particular type of periodic motion called simple harmonic motion (SHM). Such motion is a sinusoidal function of time t. That is, it can be written as a sine or a cosine of time t. Here we arbitrarily choose the cosine function and write the displacement (or position) of the particle in Fig. 15-1 as x(t) ! xm cos(vt # f)

(displacement),

(15-3)

in which xm, v, and f are quantities that we shall define. Freeze-Frames. Let’s take some freeze-frames of the motion and then arrange them one after another down the page (Fig. 15-2a). Our first freeze-frame is at t ! 0 when the particle is at its rightmost position on the x axis. We label that coordinate as xm (the subscript means maximum); it is the symbol in front of the cosine

415

15-1 SI M PLE HAR M ON IC M OTION

A The speed is zero at the extreme points.

A particle oscillates left and right in simple harmonic motion. –xm

0

+xm

–xm

t=0

t=0

t = T/4

t = T/4

The speed is greatest at the midpoint. 0

+xm

v

v

t = T/2

t = T/2

t = 3T/4

t = 3T/4

t=T

t=T 0

–xm

v

v

+xm

–xm

(a)

0

+xm

(b)

Rotating the figure reveals that the motion forms a cosine function. xm Displacement

x

0

xm 0

T

Displacement

x

(c)

0

T/2

T

Time (t)

–xm

(d)

–xm

This is a graph of the motion, with the period T indicated.

The speed is zero at extreme points.

xm 0 –xm

Time (t)

The speed is greatest at x = 0.

(e)

Figure 15-2 (a) A sequence of “freeze-frames” (taken at equal time intervals) showing the position of a particle as it oscillates back and forth about the origin of an x axis, between the limits #xm and %xm. (b) The vector arrows are scaled to indicate the speed of the particle.The speed is maximum when the particle is at the origin and zero when it is at 7xm. If the time t is chosen to be zero when the particle is at #xm, then the particle returns to #xm at t ! T, where T is the period of the motion.The motion is then repeated. (c) Rotating the figure reveals the motion forms a cosine function of time, as shown in (d). (e) The speed (the slope) changes.

416

CHAPTE R 15 OSCI LL ATIONS

Displacement at time t Phase

x(t) = xm cos(ω t + φ ) Amplitude

Time

Angular frequency

Phase constant or phase angle

Figure 15-3 A handy guide to the quantities in Eq. 15-3 for simple harmonic motion.

function in Eq. 15-3. In the next freeze-frame, the particle is a bit to the left of xm. It continues to move in the negative direction of x until it reaches the leftmost position, at coordinate %xm. Thereafter, as time takes us down the page through more freeze-frames, the particle moves back to xm and thereafter repeatedly oscillates between xm and %xm. In Eq. 15-3, the cosine function itself oscillates between #1 and %l. The value of xm determines how far the particle moves in its oscillations and is called the amplitude of the oscillations (as labeled in the handy guide of Fig. 15-3). Figure 15-2b indicates the velocity of the particle with respect to time, in the series of freeze-frames. We’ll get to a function for the velocity soon, but for now just notice that the particle comes to a momentary stop at the extreme points and has its greatest speed (longest velocity vector) as it passes through the center point. Mentally rotate Fig. 15-2a counterclockwise by 90(, so that the freeze-frames then progress rightward with time. We set time t ! 0 when the particle is at xm. The particle is back at xm at time t ! T (the period of the oscillation), when it starts the next cycle of oscillation. If we filled in lots of the intermediate freezeframes and drew a line through the particle positions, we would have the cosine curve shown in Fig. 15-2d. What we already noted about the speed is displayed in Fig. 15-2e. What we have in the whole of Fig. 15-2 is a transformation of what we can see (the reality of an oscillating particle) into the abstraction of a graph. (In WileyPLUS the transformation of Fig. 15-2 is available as an animation with voiceover.) Equation 15-3 is a concise way to capture the motion in the abstraction of an equation. More Quantities. The handy guide of Fig. 15-3 defines more quantities about the motion. The argument of the cosine function is called the phase of the motion. As it varies with time, the value of the cosine function varies. The constant f is called the phase angle or phase constant. It is in the argument only because we want to use Eq. 15-3 to describe the motion regardless of where the particle is in its oscillation when we happen to set the clock time to 0. In Fig. 15-2, we set t ! 0 when the particle is at xm. For that choice, Eq. 15-3 works just fine if we also set f ! 0. However, if we set t ! 0 when the particle happens to be at some other location, we need a different value of f. A few values are indicated in Fig. 15-4. For example, suppose the particle is at its leftmost position when we happen to start the clock at t ! 0. Then Eq. 15-3 describes the motion if f ! p rad. To check, substitute t ! 0 and f ! p rad into Eq. 15-3. See, it gives x ! %xm just then. Now check the other examples in Fig. 15-4. The quantity v in Eq. 15-3 is the angular frequency of the motion. To relate it to the frequency f and the period T, let’s first note that the position x(t) of the particle must (by definition) return to its initial value at the end of a period. That is, if x(t) is the position at some chosen time t, then the particle must return to that same position at time t # T. Let’s use Eq. 15-3 to express this condition, but let’s also just set f ! 0 to get it out of the way. Returning to the same position can then be written as xm cos vt ! xm cos v(t # T).

(15-4)

The cosine function first repeats itself when its argument (the phase, remember) has increased by 2p rad. So, Eq. 15-4 tells us that v(t # T) ! vt # 2p 3 2

p rad

–xm

p rad

1 p 2

0

or 0

rad +xm

Figure 15-4 Values of f corresponding to the position of the particle at time t ! 0.

vT ! 2p.

Thus, from Eq. 15-2 the angular frequency is v!

2p ! 2pf. 8

The SI unit of angular frequency is the radian per second.

(15-5)

15-1 SI M PLE HAR M ON IC M OTION

x'm xm t

0

The amplitudes are the same, but the frequencies and periods are different.

x Displacement

Displacement

x

The amplitudes are different, but the frequency and period are the same.

–xm –x'm

T

xm 0

t

–xm T'

(b)

Figure 15-5 In all three cases, the blue curve is obtained from Eq. 15-3 with f ! 0. (a) The red curve differs from the blue curve only in that the red-curve amplitude x.m is greater (the red-curve extremes of displacement are higher and lower). (b) The red curve differs from the blue curve only in that the red-curve period is T. ! T/2 (the red curve is compressed horizontally). (c) The red curve differs from the blue curve only in that for the red curve f ! %p/4 rad rather than zero (the negative value of f shifts the red curve to the right).

x Displacement

(a)

(c)

xm

_ φ=–π 4

–xm

t

φ=0

This zero gives a regular cosine curve.

Checkpoint 1 A particle undergoing simple harmonic oscillation of period T (like that in Fig. 15-2) is at %xm at time t ! 0. Is it at %xm, at #xm, at 0, between %xm and 0, or between 0 and #xm when (a) t ! 2.00T, (b) t ! 3.50T, and (c) t ! 5.25T?

The Velocity of SHM We briefly discussed velocity as shown in Fig. 15-2b, finding that it varies in magnitude and direction as the particle moves between the extreme points (where the speed is momentarily zero) and through the central point (where the speed is maximum). To find the velocity v(t) as a function of time, let’s take a time derivative of the position function x(t) in Eq. 15-3: v(t) !

dx(t) d ! [xm cos(vt # f)] dt dt

v(t) ! %vxm sin(vt # f)

(velocity).

This negative value shifts the cosine curve rightward.

0

We’ve had a lot of quantities here, quantities that we could experimentally change to see the effects on the particle’s SHM. Figure 15-5 gives some examples. The curves in Fig. 15-5a show the effect of changing the amplitude. Both curves have the same period. (See how the “peaks” line up?) And both are for f ! 0. (See how the maxima of the curves both occur at t ! 0?) In Fig. 15-5b, the two curves have the same amplitude xm but one has twice the period as the other (and thus half the frequency as the other). Figure 15-5c is probably more difficult to understand. The curves have the same amplitude and same period but one is shifted relative to the other because of the different f values. See how the one with f ! 0 is just a regular cosine curve? The one with the negative f is shifted rightward from it.That is a general result: negative f values shift the regular cosine curve rightward and positive f values shift it leftward. (Try this on a graphing calculator.)

or

T'

(15-6)

The velocity depends on time because the sine function varies with time, between the values of #1 and %1. The quantities in front of the sine function

417

418

CHAPTE R 15 OSCI LL ATIONS

Displacement

x +xm 0 –xm

Acceleration

Velocity

+ω xm

t

Extreme values here mean ...

t

zero values here and ...

t

extreme values here.

T v

(a)

0 –ω xm +ω 2xm

a

(b)

0 –ω 2xm (c)

Figure 15-6 (a) The displacement x(t) of a particle oscillating in SHM with phase angle f equal to zero. The period T marks one complete oscillation. (b) The velocity v(t) of the particle. (c) The acceleration a(t) of the particle.

determine the extent of the variation in the velocity, between #vxm and %vxm. We say that vxm is the velocity amplitude vm of the velocity variation. When the particle is moving rightward through x ! 0, its velocity is positive and the magnitude is at this greatest value. When it is moving leftward through x ! 0, its velocity is negative and the magnitude is again at this greatest value. This variation with time (a negative sine function) is displayed in the graph of Fig. 15-6b for a phase constant of f ! 0, which corresponds to the cosine function for the displacement versus time shown in Fig. 15-6a. Recall that we use a cosine function for x(t) regardless of the particle’s position at t ! 0. We simply choose an appropriate value of f so that Eq. 15-3 gives us the correct position at t ! 0. That decision about the cosine function leads us to a negative sine function for the velocity in Eq. 15-6, and the value of f now gives the correct velocity at t ! 0.

The Acceleration of SHM Let’s go one more step by differentiating the velocity function of Eq. 15-6 with respect to time to get the acceleration function of the particle in simple harmonic motion: a(t) !

d dv(t) ! [%vxm sin(vt # f)] dt dt

a(t) ! %v 2xm cos(vt # f)

or

(acceleration).

(15-7)

We are back to a cosine function but with a minus sign out front. We know the drill by now. The acceleration varies because the cosine function varies with time, between #1 and %1. The variation in the magnitude of the acceleration is set by the acceleration amplitude am, which is the product v2xm that multiplies the cosine function. Figure 15-6c displays Eq. 15-7 for a phase constant f ! 0, consistent with Figs. 15-6a and 15-6b. Note that the acceleration magnitude is zero when the cosine is zero, which is when the particle is at x ! 0. And the acceleration magnitude is maximum when the cosine magnitude is maximum, which is when the particle is at an extreme point, where it has been slowed to a stop so that its motion can be reversed. Indeed, comparing Eqs. 15-3 and 15-7 we see an extremely neat relationship: a(t) ! %v 2x(t).

(15-8)

This is the hallmark of SHM: (1) The particle’s acceleration is always opposite its displacement (hence the minus sign) and (2) the two quantities are always related by a constant (v 2). If you ever see such a relationship in an oscillating situation (such as with, say, the current in an electrical circuit, or the rise and fall of water in a tidal bay), you can immediately say that the motion is SHM and immediately identify the angular frequency v of the motion. In a nutshell: In SHM, the acceleration a is proportional to the displacement x but opposite in sign, and the two quantities are related by the square of the angular frequency v.

Checkpoint 2 Which of the following relationships between a particle’s acceleration a and its position x indicates simple harmonic oscillation: (a) a ! 3x2, (b) a ! 5x, (c) a ! %4x, (d) a ! %2/x? For the SHM, what is the angular frequency (assume the unit of rad/s)?

419

15-1 SI M PLE HAR M ON IC M OTION

The Force Law for Simple Harmonic Motion

k

Now that we have an expression for the acceleration in terms of the displacement in Eq. 15-8, we can apply Newton’s second law to describe the force responsible for SHM: F ! ma ! m(%v 2x) ! %(mv 2)x. (15-9) The minus sign means that the direction of the force on the particle is opposite the direction of the displacement of the particle.That is, in SHM the force is a restoring force in the sense that it fights against the displacement, attempting to restore the particle to the center point at x ! 0. We’ve seen the general form of Eq. 15-9 back in Chapter 8 when we discussed a block on a spring as in Fig. 15-7.There we wrote Hooke’s law, F ! %kx,

(15-10)

for the force acting on the block. Comparing Eqs. 15-9 and 15-10, we can now relate the spring constant k (a measure of the stiffness of the spring) to the mass of the block and the resulting angular frequency of the SHM: k ! mv 2.

(15-11)

Equation 15-10 is another way to write the hallmark equation for SHM. Simple harmonic motion is the motion of a particle when the force acting on it is proportional to the particle’s displacement but in the opposite direction.

The block–spring system of Fig. 15-7 is called a linear simple harmonic oscillator (linear oscillator, for short), where linear indicates that F is proportional to x to the first power (and not to some other power). If you ever see a situation in which the force in an oscillation is always proportional to the displacement but in the opposite direction, you can immediately say that the oscillation is SHM. You can also immediately identify the associated spring constant k. If you know the oscillating mass, you can then determine the angular frequency of the motion by rewriting Eq. 15-11 as v!

k Am

(angular frequency).

(15-12)

(This is usually more important than the value of k.) Further, you can determine the period of the motion by combining Eqs. 15-5 and 15-12 to write T ! 2p

m A k

(period).

(15-13)

Let’s make a bit of physical sense of Eqs. 15-12 and 15-13. Can you see that a stiff spring (large k) tends to produce a large v (rapid oscillations) and thus a small period T ? Can you also see that a large mass m tends to result in a small v (sluggish oscillations) and thus a large period T ? Every oscillating system, be it a diving board or a violin string, has some element of “springiness” and some element of “inertia” or mass. In Fig. 15-7, these elements are separated: The springiness is entirely in the spring, which we assume to be massless, and the inertia is entirely in the block, which we assume to be rigid. In a violin string, however, the two elements are both within the string.

Checkpoint 3 Which of the following relationships between the force F on a particle and the particle’s position x gives SHM: (a) F ! %5x, (b) F ! %400x2, (c) F ! 10x, (d) F ! 3x2?

m x –xm

x=0

+xm

Figure 15-7 A linear simple harmonic oscillator. The surface is frictionless. Like the particle of Fig. 15-2, the block moves in simple harmonic motion once it has been either pulled or pushed away from the x ! 0 position and released. Its displacement is then given by Eq. 15-3.

420

CHAPTE R 15 OSCI LL ATIONS

Sample Problem 15.01 Block–spring SHM, amplitude, acceleration, phase constant A block whose mass m is 680 g is fastened to a spring whose spring constant k is 65 N/m. The block is pulled a distance x ! 11 cm from its equilibrium position at x ! 0 on a frictionless surface and released from rest at t ! 0.

This maximum speed occurs when the oscillating block is rushing through the origin; compare Figs. 15-6a and 15-6b, where you can see that the speed is a maximum whenever x ! 0.

(a) What are the angular frequency, the frequency, and the period of the resulting motion?

(d) What is the magnitude am of the maximum acceleration of the block?

KEY IDEA

KEY IDEA

The block–spring system forms a linear simple harmonic oscillator, with the block undergoing SHM.

The magnitude am of the maximum acceleration is the acceleration amplitude v2xm in Eq. 15-7.

Calculations: The angular frequency is given by Eq. 15-12:

Calculation: So, we have

v!

65 N/m k ! ! 9.78 rad/s A 0.68 kg Am

% 9.8 rad/s.

am ! v2xm ! (9.78 rad/s)2(0.11 m) ! 11 m/s2. (Answer)

The frequency follows from Eq. 15-5, which yields f!

9.78 rad/s v ! ! 1.56 Hz % 1.6 Hz. (Answer) 2p 2p rad

The period follows from Eq. 15-2, which yields T!

1 1 ! ! 0.64 s ! 640 ms. f 1.56 Hz

(Answer)

KEY IDEA With no friction involved, the mechanical energy of the spring – block system is conserved. Reasoning: The block is released from rest 11 cm from its equilibrium position, with zero kinetic energy and the elastic potential energy of the system at a maximum. Thus, the block will have zero kinetic energy whenever it is again 11 cm from its equilibrium position, which means it will never be farther than 11 cm from that position. Its maximum displacement is 11 cm: (Answer)

(c) What is the maximum speed vm of the oscillating block, and where is the block when it has this speed? KEY IDEA The maximum speed vm is the velocity amplitude vxm in Eq. 15-6.

Calculations: Equation 15-3 gives the displacement of the block as a function of time. We know that at time t ! 0, the block is located at x ! xm. Substituting these initial conditions, as they are called, into Eq. 15-3 and canceling xm give us 1 ! cos f. (15-14) Taking the inverse cosine then yields f ! 0 rad.

(Answer)

(Any angle that is an integer multiple of 2p rad also satisfies Eq. 15-14; we chose the smallest angle.) (f) What is the displacement function x(t) for the spring – block system? Calculation: The function x(t) is given in general form by Eq. 15-3. Substituting known quantities into that equation gives us x(t) ! xm cos(vt # f) ! (0.11 m) cos[(9.8 rad/s)t # 0]

Calculation: Thus, we have

! 0.11 cos(9.8t),

vm ! vxm ! (9.78 rad/s)(0.11 m) ! 1.1 m/s.

This maximum acceleration occurs when the block is at the ends of its path, where the block has been slowed to a stop so that its motion can be reversed. At those extreme points, the force acting on the block has its maximum magnitude; compare Figs. 15-6a and 15-6c, where you can see that the magnitudes of the displacement and acceleration are maximum at the same times, when the speed is zero, as you can see in Fig. 15-6b. (e) What is the phase constant f for the motion?

(b) What is the amplitude of the oscillation?

xm ! 11 cm.

(Answer)

(Answer)

where x is in meters and t is in seconds.

Additional examples, video, and practice available at WileyPLUS

(Answer)

15-2 E N E RGY I N SI M PLE HAR M ON IC M OTION

421

Sample Problem 15.02 Finding SHM phase constant from displacement and velocity At t ! 0, the displacement x(0) of the block in a linear oscillator like that of Fig. 15-7 is %8.50 cm. (Read x(0) as “x at time zero.”) The block’s velocity v(0) then is %0.920 m/s, and its acceleration a(0) is #47.0 m/s2.

Calculations: We know v and want f and xm. If we divide Eq. 15-16 by Eq. 15-15, we eliminate one of those unknowns and reduce the other to a single trig function: %vxm sin f v(0) ! ! %v tan f. x(0) xm cos f

(a) What is the angular frequency v of this system?

Solving for tan f, we find

KEY IDEA With the block in SHM, Eqs. 15-3, 15-6, and 15-7 give its displacement, velocity, and acceleration, respectively, and each contains v. Calculations: Let’s substitute t ! 0 into each to see whether we can solve any one of them for v. We find

and

x(0) ! xm cos f,

(15-15)

v(0) ! %vxm sin f,

(15-16)

a(0) ! %v2xm cos f.

(15-17)

In Eq. 15-15, v has disappeared. In Eqs. 15-16 and 15-17, we know values for the left sides, but we do not know xm and f. However, if we divide Eq. 15-17 by Eq. 15-15, we neatly eliminate both xm and f and can then solve for v as v!

A

%

a(0) 47.0 m/s2 ! % x(0) A %0.0850 m

! 23.5 rad/s.

(Answer)

(b) What are the phase constant f and amplitude xm?

tan f ! %

%0.920 m/s v(0) !% vx(0) (23.5 rad/s)(%0.0850 m)

! %0.461. This equation has two solutions: f ! %25( and

f ! 180( # (%25() ! 155(.

Normally only the first solution here is displayed by a calculator, but it may not be the physically possible solution. To choose the proper solution, we test them both by using them to compute values for the amplitude xm. From Eq. 15-15, we find that if f ! %25(, then xm !

%0.0850 m x(0) ! ! %0.094 m. cos f cos(%25()

We find similarly that if f ! 155(, then xm ! 0.094 m. Because the amplitude of SHM must be a positive constant, the correct phase constant and amplitude here are f ! 155( and

xm ! 0.094 m ! 9.4 cm. (Answer)

Additional examples, video, and practice available at WileyPLUS

15-2 ENERGY IN SIMPLE HARMONIC MOTION Learning Objectives After reading this module, you should be able to . . .

15.19 For a spring–block oscillator, calculate the kinetic energy and elastic potential energy at any given time. 15.20 Apply the conservation of energy to relate the total energy of a spring–block oscillator at one instant to the total energy at another instant.

15.21 Sketch a graph of the kinetic energy, potential energy, and total energy of a spring–block oscillator, first as a function of time and then as a function of the oscillator’s position. 15.22 For a spring–block oscillator, determine the block’s position when the total energy is entirely kinetic energy and when it is entirely potential energy.

Key Ideas ● A particle in simple harmonic motion has, at any time, kinetic energy K ! 12mv2 and potential energy U ! 12kx2. If no

friction is present, the mechanical energy E ! K # U remains constant even though K and U change.

Energy in Simple Harmonic Motion Let’s now examine the linear oscillator of Chapter 8, where we saw that the energy transfers back and forth between kinetic energy and potential energy, while the sum of the two—the mechanical energy E of the oscillator—remains constant. The

422

CHAPTE R 15 OSCI LL ATIONS

U(t) + K(t)

E Energy

U(t)

potential energy of a linear oscillator like that of Fig. 15-7 is associated entirely with the spring. Its value depends on how much the spring is stretched or compressed — that is, on x(t). We can use Eqs. 8-11 and 15-3 to find U(t) ! 12 kx2 ! 12kx2m cos2(vt # f).

K(t) 0

T/2

T (a)

t

As time changes, the energy shifts between the two types, but the total is constant.

Caution: A function written in the form cos2 A (as here) means (cos A)2 and is not the same as one written cos A2, which means cos(A2). The kinetic energy of the system of Fig. 15-7 is associated entirely with the block. Its value depends on how fast the block is moving — that is, on v(t). We can use Eq. 15-6 to find K(t) ! 12 mv2 ! 12 mv2x2m sin2(vt # f).

K(t) ! 12 mv2 ! 12kx2m sin2(vt # f).

Energy

U(x)

–xm

+xm (b)

(15-20)

The mechanical energy follows from Eqs. 15-18 and 15-20 and is

K(x)

0

(15-19)

If we use Eq. 15-12 to substitute k/m for v2, we can write Eq. 15-19 as

U(x) + K(x)

E

(15-18)

E!U#K ! 12kx2m cos2(vt # f) # 12kx2m sin2(vt # f)

x

As position changes, the energy shifts between the two types, but the total is constant.

Figure 15-8 (a) Potential energy U(t), kinetic energy K(t), and mechanical energy E as functions of time t for a linear harmonic oscillator. Note that all energies are positive and that the potential energy and the kinetic energy peak twice during every period. (b) Potential energy U(x), kinetic energy K(x), and mechanical energy E as functions of position x for a linear harmonic oscillator with amplitude xm. For x ! 0 the energy is all kinetic, and for x ! 7xm it is all potential.

! 12kx2m [cos2(vt # f) # sin2(vt # f)]. For any angle a, cos2 a # sin2 a ! 1. Thus, the quantity in the square brackets above is unity and we have E ! U # K ! 12 kx2m.

(15-21)

The mechanical energy of a linear oscillator is indeed constant and independent of time. The potential energy and kinetic energy of a linear oscillator are shown as functions of time t in Fig. 15-8a and as functions of displacement x in Fig. 15-8b. In any oscillating system, an element of springiness is needed to store the potential energy and an element of inertia is needed to store the kinetic energy.

Checkpoint 4 In Fig. 15-7, the block has a kinetic energy of 3 J and the spring has an elastic potential energy of 2 J when the block is at x ! #2.0 cm. (a) What is the kinetic energy when the block is at x ! 0? What is the elastic potential energy when the block is at (b) x ! %2.0 cm and (c) x ! %xm?

Sample Problem 15.03 SHM potential energy, kinetic energy, mass dampers Many tall buildings have mass dampers, which are anti-sway devices to prevent them from oscillating in a wind. The device might be a block oscillating at the end of a spring and on a lubricated track. If the building sways, say, eastward, the block also moves eastward but delayed enough so that when it finally moves, the building is then moving back westward. Thus, the motion of the oscillator is out of step with the motion of the building. Suppose the block has mass m ! 2.72 $ 105 kg and is designed to oscillate at frequency f ! 10.0 Hz and with amplitude xm ! 20.0 cm.

(a) What is the total mechanical energy E of the spring – block system? KEY IDEA The mechanical energy E (the sum of the kinetic energy K ! 12mv2 of the block and the potential energy U ! 12kx2 of the spring) is constant throughout the motion of the oscillator. Thus, we can evaluate E at any point during the motion. Calculations: Because we are given amplitude xm of the oscillations, let’s evaluate E when the block is at position x ! xm,

423

15-3 AN ANG U L AR SI M PLE HAR M ON IC OSCI LL ATOR

where it has velocity v ! 0. However, to evaluate U at that point, we first need to find the spring constant k. From Eq. 15-12 (v ! 2k/m) and Eq.15-5 (v ! 2pf ),we find 2

2

k ! mv ! m(2pf ) ! (2.72 $ 105 kg)(2p)2(10.0 Hz)2

(b) What is the block’s speed as it passes through the equilibrium point? Calculations: We want the speed at x ! 0, where the potential energy is U ! 12kx 2 ! 0 and the mechanical energy is entirely kinetic energy. So, we can write E ! K # U ! 12mv2 # 12kx2

! 1.073 $ 109 N/m.

2.147 $ 107 J ! 12(2.72 $ 105 kg)v2 # 0,

We can now evaluate E as E ! K # U ! 12mv2 # 12kx2 ! 0 # 12(1.073 $ 109 N/m)(0.20 m)2 ! 2.147 $ 107 J % 2.1 $ 107 J.

or (Answer)

v ! 12.6 m/s.

(Answer)

Because E is entirely kinetic energy, this is the maximum speed vm.

Additional examples, video, and practice available at WileyPLUS

15-3 AN ANGULAR SIMPLE HARMONIC OSCILLATOR Learning Objectives After reading this module, you should be able to . . .

15.23 Describe the motion of an angular simple harmonic oscillator. 15.24 For an angular simple harmonic oscillator, apply the relationship between the torque t and the angular displacement u (from equilibrium).

Key Idea

15.25 For an angular simple harmonic oscillator, apply the relationship between the period T (or frequency f ), the rotational inertia I, and the torsion constant k. 15.26 For an angular simple harmonic oscillator at any instant, apply the relationship between the angular acceleration a, the angular frequency v, and the angular displacement u.

● A torsion pendulum consists of an object suspended on a wire. When the wire is twisted and then released, the object oscillates in angular simple harmonic motion with a period given by I T ! 2p , k A

where I is the rotational inertia of the object about the axis of rotation and k is the torsion constant of the wire.

Fixed end

An Angular Simple Harmonic Oscillator Figure 15-9 shows an angular version of a simple harmonic oscillator; the element of springiness or elasticity is associated with the twisting of a suspension wire rather than the extension and compression of a spring as we previously had. The device is called a torsion pendulum, with torsion referring to the twisting. If we rotate the disk in Fig. 15-9 by some angular displacement u from its rest position (where the reference line is at u ! 0) and release it, it will oscillate about that position in angular simple harmonic motion. Rotating the disk through an angle u in either direction introduces a restoring torque given by t ! %ku.

(15-22)

Here k (Greek kappa) is a constant, called the torsion constant, that depends on the length, diameter, and material of the suspension wire. Comparison of Eq. 15-22 with Eq. 15-10 leads us to suspect that Eq. 15-22 is the angular form of Hooke’s law, and that we can transform Eq. 15-13, which gives the period of linear SHM, into an equation for the period of angular SHM: We replace the spring constant k in Eq. 15-13 with its equivalent, the constant

Suspension wire Reference line

– θm

0

+ θm

Figure 15-9 A torsion pendulum is an angular version of a linear simple harmonic oscillator. The disk oscillates in a horizontal plane; the reference line oscillates with angular amplitude um. The twist in the suspension wire stores potential energy as a spring does and provides the restoring torque.

424

CHAPTE R 15 OSCI LL ATIONS

k of Eq. 15-22, and we replace the mass m in Eq. 15-13 with its equivalent, the rotational inertia I of the oscillating disk. These replacements lead to T ! 2p

9 Ak

(15-23)

(torsion pendulum).

Sample Problem 15.04 Angular simple harmonic oscillator, rotational inertia, period Figure 15-10a shows a thin rod whose length L is 12.4 cm and whose mass m is 135 g, suspended at its midpoint from a long wire. Its period Ta of angular SHM is measured to be 2.53 s. An irregularly shaped object, which we call object X, is then hung from the same wire, as in Fig. 15-10b, and its period Tb is found to be 4.76 s. What is the rotational inertia of object X about its suspension axis?

Ia Ib and Tb ! 2p . Ak Ak The constant k, which is a property of the wire, is the same for both figures; only the periods and the rotational inertias differ. Let us square each of these equations, divide the second by the first, and solve the resulting equation for Ib.The result is Ta ! 2p

Ib ! Ia

KEY IDEA The rotational inertia of either the rod or object X is related to the measured period by Eq. 15-23.

Tb2 (4.76 s)2 %4 2 ! (1.73 $ 10 kg "m ) Ta2 (2.53 s)2

! 6.12 $ 10%4 kg "m2. Suspension wire

Calculations: In Table 10-2e, the rotational inertia of a thin rod about a perpendicular axis through its midpoint is given as 1 2 12mL .Thus, we have, for the rod in Fig. 15-10a, Ia ! 121 mL2 ! (121 )(0.135 kg)(0.124 m)2 ! 1.73 $ 10%4 kg "m2. Now let us write Eq. 15-23 twice, once for the rod and once for object X:

(Answer)

Figure 15-10 Two torsion pendulums, consisting of (a) a wire and a rod and (b) the same wire and an irregularly shaped object.

Rod L

(a)

(b) Object X

Additional examples, video, and practice available at WileyPLUS

15-4 PENDULUMS, CIRCULAR MOTION Learning Objectives After reading this module, you should be able to . . .

15.27 Describe the motion of an oscillating simple pendulum. 15.28 Draw a free-body diagram of a pendulum bob with the pendulum at angle u to the vertical. 15.29 For small-angle oscillations of a simple pendulum, relate the period T (or frequency f ) to the pendulum’s length L. 15.30 Distinguish between a simple pendulum and a physical pendulum. 15.31 For small-angle oscillations of a physical pendulum, relate the period T (or frequency f ) to the distance h between the pivot and the center of mass. 15.32 For an angular oscillating system, determine the angular frequency v from either an equation relating torque t and angular displacement u or an equation relating angular acceleration a and angular displacement u.

15.33 Distinguish between a pendulum’s angular frequency v (having to do with the rate at which cycles are completed) and its du/dt (the rate at which its angle with the vertical changes). 15.34 Given data about the angular position u and rate of change du/dt at one instant, determine the phase constant f and amplitude um. 15.35 Describe how the free-fall acceleration can be measured with a simple pendulum. 15.36 For a given physical pendulum, determine the location of the center of oscillation and identify the meaning of that phrase in terms of a simple pendulum. 15.37 Describe how simple harmonic motion is related to uniform circular motion.

15-4 PE N DU LU M S, CI RCU L AR M OTION

425

Key Ideas ● A simple pendulum consists of a rod of negligible mass that pivots about its upper end, with a particle (the bob) attached at its lower end. If the rod swings through only small angles, its motion is approximately simple harmonic motion with a period given by

T ! 2p

I A mgL

(simple pendulum),

where I is the particle’s rotational inertia about the pivot, m is the particle’s mass, and L is the rod’s length.

● A physical pendulum has a more complicated distribution of mass. For small angles of swinging, its motion is simple harmonic motion with a period given by I (physical pendulum), T ! 2p A mgh

where I is the pendulum’s rotational inertia about the pivot, m is the pendulum’s mass, and h is the distance between the pivot and the pendulum’s center of mass.

● Simple harmonic motion corresponds to the projection of uniform circular motion onto a diameter of the circle.

Pendulums We turn now to a class of simple harmonic oscillators in which the springiness is associated with the gravitational force rather than with the elastic properties of a twisted wire or a compressed or stretched spring.

The Simple Pendulum If an apple swings on a long thread, does it have simple harmonic motion? If so, what is the period T ? To answer, we consider a simple pendulum, which consists of a particle of mass m (called the bob of the pendulum) suspended from one end of an unstretchable, massless string of length L that is fixed at the other end, as in Fig. 15-11a. The bob is free to swing back and forth in the plane of the page, to the left and right of a vertical line through the pendulum’s pivot point. : The Restoring Torque. The forces acting on the bob are the force T from the : string and the gravitational force Fg, as shown in Fig. 15-11b, where the string makes : an angle u with the vertical. We resolve Fg into a radial component Fg cos u and a component Fg sin u that is tangent to the path taken by the bob.This tangential component produces a restoring torque about the pendulum’s pivot point because the component always acts opposite the displacement of the bob so as to bring the bob back toward its central location. That location is called the equilibrium position (u ! 0) because the pendulum would be at rest there were it not swinging. From Eq. 10-41 (t ! r"F), we can write this restoring torque as t ! %L(Fg sin u),

L

m

(a)

θ L T

(15-24)

where the minus sign indicates that the torque acts to reduce u and L is the moment arm of the force component Fg sin u about the pivot point. Substituting Eq. 15-24 into Eq. 10-44 (t ! Ia) and then substituting mg as the magnitude of Fg, we obtain %L(mg sin u) ! Ia,

Pivot point

s = Lθ

(15-25)

Fg sinθ

where I is the pendulum’s rotational inertia about the pivot point and a is its angular acceleration about that point. We can simplify Eq. 15-25 if we assume the angle u is small, for then we can approximate sin u with u (expressed in radian measure). (As an example, if u ! 5.00( ! 0.0873 rad, then sin u ! 0.0872, a difference of only about 0.1%.) With that approximation and some rearranging, we then have

This component brings the bob back to center.

a!%

mgL u. I

(15-26)

This equation is the angular equivalent of Eq. 15-8, the hallmark of SHM. It tells us that the angular acceleration a of the pendulum is proportional to the angular displacement u but opposite in sign. Thus, as the pendulum bob moves to the right, as in Fig. 15-11a, its acceleration to the left increases until the bob stops and

m

θ

Fg

Fg cos θ

This component merely pulls on the string.

(b)

Figure 15-11 (a) A simple pendulum. (b) The forces acting on the bob are the gravitational : : force Fg and the force T from the string. The tangential component Fg sin u of the gravitational force is a restoring force that tends to bring the pendulum back to its central position.

426

CHAPTE R 15 OSCI LL ATIONS

begins moving to the left. Then, when it is to the left of the equilibrium position, its acceleration to the right tends to return it to the right, and so on, as it swings back and forth in SHM. More precisely, the motion of a simple pendulum swinging through only small angles is approximately SHM. We can state this restriction to small angles another way: The angular amplitude um of the motion (the maximum angle of swing) must be small. Angular Frequency. Here is a neat trick. Because Eq. 15-26 has the same form as Eq. 15-8 for SHM, we can immediately identify the pendulum’s angular frequency as being the square root of the constants in front of the displacement: v!

A

mgL . I

In the homework problems you might see oscillating systems that do not seem to resemble pendulums. However, if you can relate the acceleration (linear or angular) to the displacement (linear or angular), you can then immediately identify the angular frequency as we have just done here. Period. Next, if we substitute this expression for v into Eq. 15-5 (v ! 2p/T), we see that the period of the pendulum may be written as T ! 2p

O

θ

h

Fg sin θ

This component brings the pendulum back to center.

C

Fg cos θ

θ

Fg

Figure 15-12 A physical pendulum. The restoring torque is hFg sin u. When u ! 0, center of mass C hangs directly below pivot point O.

I . A mgL

(15-27)

All the mass of a simple pendulum is concentrated in the mass m of the particlelike bob, which is at radius L from the pivot point. Thus, we can use Eq. 10-33 (I ! mr 2) to write I ! mL2 for the rotational inertia of the pendulum. Substituting this into Eq. 15-27 and simplifying then yield T ! 2p

L A g

(simple pendulum, small amplitude).

(15-28)

We assume small-angle swinging in this chapter.

The Physical Pendulum A real pendulum, usually called a physical pendulum, can have a complicated distribution of mass. Does it also undergo SHM? If so, what is its period? Figure 15-12 shows an arbitrary physical pendulum displaced to one side : by angle u. The gravitational force Fg acts at its center of mass C, at a distance h from the pivot point O. Comparison of Figs. 15-12 and 15-11b reveals only one important difference between an arbitrary physical pendulum and a simple pendulum. For a physical pendulum the restoring component Fg sin u of the gravitational force has a moment arm of distance h about the pivot point, rather than of string length L. In all other respects, an analysis of the physical pendulum would duplicate our analysis of the simple pendulum up through Eq. 15-27. Again (for small um), we would find that the motion is approximately SHM. If we replace L with h in Eq. 15-27, we can write the period as

T ! 2p

I A mgh

(physical pendulum, small amplitude).

(15-29)

As with the simple pendulum, I is the rotational inertia of the pendulum about O. However, now I is not simply mL2 (it depends on the shape of the physical pendulum), but it is still proportional to m. A physical pendulum will not swing if it pivots at its center of mass. Formally, this corresponds to putting h ! 0 in Eq. 15-29. That equation then predicts T : 2, which implies that such a pendulum will never complete one swing.

15-4 PE N DU LU M S, CI RCU L AR M OTION

427

Corresponding to any physical pendulum that oscillates about a given pivot point O with period T is a simple pendulum of length L0 with the same period T. We can find L0 with Eq. 15-28. The point along the physical pendulum at distance L0 from point O is called the center of oscillation of the physical pendulum for the given suspension point.

Measuring g We can use a physical pendulum to measure the free-fall acceleration g at a particular location on Earth’s surface. (Countless thousands of such measurements have been made during geophysical prospecting.) To analyze a simple case, take the pendulum to be a uniform rod of length L, suspended from one end. For such a pendulum, h in Eq. 15-29, the distance between the pivot point and the center of mass, is 12L. Table 10-2e tells us that the rotational inertia of this pendulum about a perpendicular axis through its center of mass is 121 mL2. From the parallel-axis theorem of Eq. 10-36 (I ! Icom # Mh2), we then find that the rotational inertia about a perpendicular axis through one end of the rod is I ! Icom # mh2 ! 121 mL2 # m(21L)2 ! 13mL2.

(15-30)

If we put h ! 21L and I ! 31mL2 in Eq. 15-29 and solve for g, we find g!

8p 2L . 3T 2

(15-31)

Thus, by measuring L and the period T, we can find the value of g at the pendulum’s location. (If precise measurements are to be made, a number of refinements are needed, such as swinging the pendulum in an evacuated chamber.)

Checkpoint 5 Three physical pendulums, of masses m0, 2m0, and 3m0, have the same shape and size and are suspended at the same point. Rank the masses according to the periods of the pendulums, greatest first.

Sample Problem 15.05 Physical pendulum, period and length O

In Fig. 15-13a, a meter stick swings about a pivot point at one end, at distance h from the stick’s center of mass. (a) What is the period of oscillation T ?

h L0

KEY IDEA The stick is not a simple pendulum because its mass is not concentrated in a bob at the end opposite the pivot point — so the stick is a physical pendulum. Calculations: The period for a physical pendulum is given by Eq. 15-29, for which we need the rotational inertia I of the stick about the pivot point. We can treat the stick as a uniform rod of length L and mass m. Then Eq. 15-30 tells us that I ! 13 mL2, and the distance h in Eq. 15-29 is 12L. Substituting these quantities into Eq. 15-29,

C P

(a)

(b)

Figure 15-13 (a) A meter stick suspended from one end as a physical pendulum. (b) A simple pendulum whose length L0 is chosen so that the periods of the two pendulums are equal. Point P on the pendulum of (a) marks the center of oscillation.

428

CHAPTE R 15 OSCI LL ATIONS

we find T ! 2p

1 2 I 3 mL ! 2p A mg(12L) A mgh

2L ! 2p A 3g

(2)(1.00 m) ! 1.64 s. ! 2p A (3)(9.8 m/s2)

(15-32)

lum (drawn in Fig. 15-13b) that has the same period as the physical pendulum (the stick) of Fig. 15-13a. Setting Eqs. 15-28 and 15-33 equal yields T ! 2p

(15-33)

L0 2L ! 2p . A g A 3g

(15-34)

You can see by inspection that (Answer)

L0 ! 32L !

Note the result is independent of the pendulum’s mass m. (b) What is the distance L0 between the pivot point O of the stick and the center of oscillation of the stick? Calculations: We want the length L0 of the simple pendu-

(15-35)

(23)(100

cm) ! 66.7 cm.

(Answer)

In Fig. 15-13a, point P marks this distance from suspension point O. Thus, point P is the stick’s center of oscillation for the given suspension point. Point P would be different for a different suspension choice.

Additional examples, video, and practice available at WileyPLUS

Simple Harmonic Motion and Uniform Circular Motion In 1610, Galileo, using his newly constructed telescope, discovered the four principal moons of Jupiter. Over weeks of observation, each moon seemed to him to be moving back and forth relative to the planet in what today we would call simple harmonic motion; the disk of the planet was the midpoint of the motion. The record of Galileo’s observations, written in his own hand, is actually still available. A. P. French of MIT used Galileo’s data to work out the position of the moon Callisto relative to Jupiter (actually, the angular distance from Jupiter as seen from Earth) and found that the data approximates the curve shown in Fig. 15-14. The curve strongly suggests Eq. 15-3, the displacement function for simple harmonic motion. A period of about 16.8 days can be measured from the plot, but it is a period of what exactly? After all, a moon cannot possibly be oscillating back and forth like a block on the end of a spring, and so why would Eq. 15-3 have anything to do with it? Actually, Callisto moves with essentially constant speed in an essentially circular orbit around Jupiter. Its true motion — far from being simple harmonic — is uniform circular motion along that orbit. What Galileo saw — and what you can see with a good pair of binoculars and a little patience — is the projection of this uniform circular motion on a line in the plane of the motion. We are led by Galileo’s remarkable observations to the conclusion that simple harmonic

Angle (arc minutes)

12

0

!12

0

10

20

30

40

t (days)

Figure 15-14 The angle between Jupiter and its moon Callisto as seen from Earth. Galileo’s 1610 measurements approximate this curve, which suggests simple harmonic motion. At Jupiter’s mean distance from Earth, 10 minutes of arc corresponds to about 2 $ 106 km. (Based on A. P. French, Newtonian Mechanics, W. W. Norton & Company, New York, 1971, p. 288.)

15-4 PE N DU LU M S, CI RCU L AR M OTION

y

P ´ is a particle moving in a circle.

y

xm ωt + φ

O

x(t)

P

y

v

ω xm ωt + φ ω 2xm

P'

P'

O

P is a projection moving in SHM. (a )

v(t) P

x

O

This relates the velocities of P and P ´. (b )

(c )

Figure 15-15 (a) A reference particle P. moving with uniform circular motion in a reference circle of radius xm. Its projection P on the x axis executes simple harmonic motion. (b) The projection of the velocity v: of the reference particle is the velocity of SHM. (c) The projec: tion of the radial acceleration a of the reference particle is the acceleration of SHM.

motion is uniform circular motion viewed edge-on. In more formal language: Simple harmonic motion is the projection of uniform circular motion on a diameter of the circle in which the circular motion occurs.

Figure 15-15a gives an example. It shows a reference particle P. moving in uniform circular motion with (constant) angular speed v in a reference circle. The radius xm of the circle is the magnitude of the particle’s position vector. At any time t, the angular position of the particle is vt # f, where f is its angular position at t ! 0. Position. The projection of particle P. onto the x axis is a point P, which we take to be a second particle. The projection of the position vector of particle P. onto the x axis gives the location x(t) of P. (Can you see the x component in the triangle in Fig. 15-5a?) Thus, we find x(t) ! xm cos(vt # f),

(15-36)

which is precisely Eq. 15-3. Our conclusion is correct. If reference particle P. moves in uniform circular motion, its projection particle P moves in simple harmonic motion along a diameter of the circle. Velocity. Figure 15-15b shows the velocity v: of the reference particle. From Eq. 10-18 (v ! vr), the magnitude of the velocity vector is vxm; its projection on the x axis is v(t) ! %vxm sin(vt # f),

(15-37)

which is exactly Eq. 15-6. The minus sign appears because the velocity component of P in Fig. 15-15b is directed to the left, in the negative direction of x. (The minus sign is consistent with the derivative of Eq. 15-36 with respect to time.) Acceleration. Figure 15-15c shows the radial acceleration : a of the reference particle. From Eq. 10-23 (ar ! v2 r), the magnitude of the radial acceleration vector is v2xm; its projection on the x axis is a(t) ! %v2xm cos(vt # f),

(15-38)

which is exactly Eq. 15-7. Thus, whether we look at the displacement, the velocity, or the acceleration, the projection of uniform circular motion is indeed simple harmonic motion.

P'

a

ωt + φ

ωt + φ

x

429

a(t) P

x

This relates the accelerations of P and P ´.

430

CHAPTE R 15 OSCI LL ATIONS

15-5 DAMPED SIMPLE HARMONIC MOTION Learning Objectives After reading this module, you should be able to . . .

15.38 Describe the motion of a damped simple harmonic oscillator and sketch a graph of the oscillator’s position as a function of time. 15.39 For any particular time, calculate the position of a damped simple harmonic oscillator. 15.40 Determine the amplitude of a damped simple harmonic oscillator at any given time.

15.41 Calculate the angular frequency of a damped simple harmonic oscillator in terms of the spring constant, the damping constant, and the mass, and approximate the angular frequency when the damping constant is small. 15.42 Apply the equation giving the (approximate) total energy of a damped simple harmonic oscillator as a function of time.

Key Ideas ● The mechanical energy E in a real oscillating system de-

creases during the oscillations because external forces, such as a drag force, inhibit the oscillations and transfer mechanical energy to thermal energy. The real oscillator and its motion are then said to be damped. ● If the damping force is given by

:

F d ! %b v:, where v: is the velocity of the oscillator and b is a damping constant, then the displacement of the oscillator is given by x(t) ! xm e%bt/2m cos(v.t # f),

x

Rigid support

Springiness, k

Mass m

Vane Damping, b

Figure 15-16 An idealized damped simple harmonic oscillator. A vane immersed in a liquid exerts a damping force on the block as the block oscillates parallel to the x axis.

where v., the angular frequency of the damped oscillator, is given by b2 k "# $ ! . Am 4m2

● If the damping constant is small (b - 1km), then v. % v, where v is the angular frequency of the undamped oscillator. For small b, the mechanical energy E of the oscillator is given by

E(t) % 12kx2m e!bt/m.

Damped Simple Harmonic Motion A pendulum will swing only briefly underwater, because the water exerts on the pendulum a drag force that quickly eliminates the motion. A pendulum swinging in air does better, but still the motion dies out eventually, because the air exerts a drag force on the pendulum (and friction acts at its support point), transferring energy from the pendulum’s motion. When the motion of an oscillator is reduced by an external force, the oscillator and its motion are said to be damped. An idealized example of a damped oscillator is shown in Fig. 15-16, where a block with mass m oscillates vertically on a spring with spring constant k. From the block, a rod extends to a vane (both assumed massless) that is submerged in a liquid. As the vane moves up and down, the liquid exerts an inhibiting drag force on it and thus on the entire oscillating system. With time, the mechanical energy of the block – spring system decreases, as energy is transferred to thermal energy of the liquid and vane. : Let us assume the liquid exerts a damping force Fd that is proportional to the : velocity v of the vane and block (an assumption that is accurate if the vane moves slowly). Then, for force and velocity components along the x axis in Fig. 15-16, we have Fd $ !bv,

(15-39)

where b is a damping constant that depends on the characteristics of both the vane and the liquid and has the SI unit of kilogram per second. The minus sign : indicates that F d opposes the motion. Damped Oscillations. The force on the block from the spring is Fs $ !kx. Let us assume that the gravitational force on the block is negligible relative to Fd and Fs. Then we can write Newton’s second law for components along the x axis (Fnet,x $ max) as !bv ! kx $ ma.

(15-40)

15-5 DAM PE D SI M PLE HAR M ON IC M OTION x xme–bt/2m

+xm

0

1

x(t)

2

–xm

3

4

5

6

t (s)

xme–bt/2m

Figure 15-17 The displacement function x(t) for the damped oscillator of Fig. 15-16. The amplitude, which is xm e!bt/2m, decreases exponentially with time.

Substituting dx/dt for v and d 2x/dt 2 for a and rearranging give us the differential equation m

dx d 2x %b % kx $ 0. dt 2 dt

(15-41)

The solution of this equation is x(t) $ xm e!bt/2m cos(v#t % f),

(15-42)

where xm is the amplitude and v# is the angular frequency of the damped oscillator. This angular frequency is given by

v# $

b2 k ! . Am 4m2

(15-43)

If b $ 0 (there is no damping), then Eq. 15-43 reduces to Eq. 15-12 (v $ 1k/m) for the angular frequency of an undamped oscillator, and Eq. 15-42 reduces to Eq. 15-3 for the displacement of an undamped oscillator. If the damping constant is small but not zero (so that b & 1km), then v# % v. Damped Energy. We can regard Eq. 15-42 as a cosine function whose amplitude, which is xm e!bt/2m, gradually decreases with time, as Fig. 15-17 suggests. For an undamped oscillator, the mechanical energy is constant and is given by Eq. 15-21 (E $ 1 2 2kx m). If the oscillator is damped, the mechanical energy is not constant but decreases with time. If the damping is small, we can find E(t) by replacing xm in Eq. 15-21 with xm e!bt/2m, the amplitude of the damped oscillations. By doing so, we find that E(t) % 12kx2m e!bt/m,

(15-44)

which tells us that, like the amplitude, the mechanical energy decreases exponentially with time.

Checkpoint 6 Here are three sets of values for the spring constant, damping constant, and mass for the damped oscillator of Fig. 15-16. Rank the sets according to the time required for the mechanical energy to decrease to one-fourth of its initial value, greatest first. Set 1 Set 2 Set 3

2k0 k0 3k0

b0 6b0 3b0

m0 4m0 m0

431

432

CHAPTE R 15 OSCI LL ATIONS

Sample Problem 15.06 Damped harmonic oscillator, time to decay, energy For the damped oscillator of Fig. 15-16, m $ 250 g, k $ 85 N/m, and b $ 70 g/s.

on the left side. Thus, 1

t$

(a) What is the period of the motion?

$ 5.0 s.

KEY IDEA Because b & 1km $ 4.6 kg/s, the period is approximately that of the undamped oscillator. Calculation: From Eq. 15-13, we then have T $ 2p

1

!(2)(0.25 kg)(ln 2) !2m ln 2 $ b 0.070 kg/s

m 0.25 kg $ 2p $ 0.34 s. A k A 85 N/m

(Answer)

Because T $ 0.34 s, this is about 15 periods of oscillation. (c) How long does it take for the mechanical energy to drop to one-half its initial value? KEY IDEA

(Answer)

(b) How long does it take for the amplitude of the damped oscillations to drop to half its initial value? KEY IDEA The amplitude at time t is displayed in Eq. 15-42 as xm e

.

!bt/2m

Calculations: The amplitude has the value xm at t $ 0. Thus, we must find the value of t for which

From Eq. 15-44, the mechanical energy at time t is 12kx2m e!bt/m. Calculations: The mechanical energy has the value at t $ 0. Thus, we must find the value of t for which

1 2 2 kx m

1 2 2 kxm

If we divide both sides of this equation by 12kx2m and solve for t as we did above, we find 1

t$

xm e!bt/2m $ 12 xm. Canceling xm and taking the natural logarithm of the equation that remains, we have ln 12 on the right side and ln(e!bt/2m) $ !bt/2m

e!bt/m $ 12(12kx2m).

1

!m ln 2 !(0.25 kg)(ln 2) $ $ 2.5 s. b 0.070 kg/s

(Answer)

This is exactly half the time we calculated in (b), or about 7.5 periods of oscillation. Figure 15-17 was drawn to illustrate this sample problem.

Additional examples, video, and practice available at WileyPLUS

15-6 FORCED OSCILLATIONS AND RESONANCE Learning Objectives After reading this module, you should be able to . . .

15.43 Distinguish between natural angular frequency v and driving angular frequency vd. 15.44 For a forced oscillator, sketch a graph of the oscillation amplitude versus the ratio vd/v of driving angular fre-

quency to natural angular frequency, identify the approximate location of resonance, and indicate the effect of increasing the damping constant. 15.45 For a given natural angular frequency v, identify the approximate driving angular frequency vd that gives resonance.

Key Ideas ● If an external driving force with angular frequency vd acts

on an oscillating system with natural angular frequency v, the system oscillates with angular frequency vd.

● The velocity amplitude vm of the system is greatest when

vd $ v, a condition called resonance. The amplitude xm of the system is (approximately) greatest under the same condition.

Forced Oscillations and Resonance A person swinging in a swing without anyone pushing it is an example of free oscillation. However, if someone pushes the swing periodically, the swing has

433

15-6 FORCE D OSCI LL ATIONS AN D R ESONANCE

x(t) $ xm cos(vd t % f),

(15-45)

where xm is the amplitude of the oscillations. How large the displacement amplitude xm is depends on a complicated function of vd and v. The velocity amplitude vm of the oscillations is easier to describe: it is greatest when vd $ v

(resonance),

b = 50 g/s (least damping) Amplitude

forced, or driven, oscillations. Two angular frequencies are associated with a system undergoing driven oscillations: (1) the natural angular frequency v of the system, which is the angular frequency at which it would oscillate if it were suddenly disturbed and then left to oscillate freely, and (2) the angular frequency vd of the external driving force causing the driven oscillations. We can use Fig. 15-16 to represent an idealized forced simple harmonic oscillator if we allow the structure marked “rigid support” to move up and down at a variable angular frequency vd. Such a forced oscillator oscillates at the angular frequency vd of the driving force, and its displacement x(t) is given by

0.6

b = 70 g/s b = 140 g/s

0.8

1.0 ωd/ω

1.2

1.4

Figure 15-18 The displacement amplitude xm of a forced oscillator varies as the angular frequency vd of the driving force is varied. The curves here correspond to three values of the damping constant b.

(15-46)

a condition called resonance. Equation 15-46 is also approximately the condition at which the displacement amplitude xm of the oscillations is greatest. Thus, if you push a swing at its natural angular frequency, the displacement and velocity amplitudes will increase to large values, a fact that children learn quickly by trial and error. If you push at other angular frequencies, either higher or lower, the displacement and velocity amplitudes will be smaller. Figure 15-18 shows how the displacement amplitude of an oscillator depends on the angular frequency vd of the driving force, for three values of the damping coefficient b. Note that for all three the amplitude is approximately greatest when vd /v $ 1 (the resonance condition of Eq. 15-46). The curves of Fig. 15-18 show that less damping gives a taller and narrower resonance peak. Examples. All mechanical structures have one or more natural angular frequencies, and if a structure is subjected to a strong external driving force that matches one of these angular frequencies, the resulting oscillations of the structure may rupture it. Thus, for example, aircraft designers must make sure that none of the natural angular frequencies at which a wing can oscillate matches the angular frequency of the engines in flight. A wing that flaps violently at certain engine speeds would obviously be dangerous. Resonance appears to be one reason buildings in Mexico City collapsed in September 1985 when a major earthquake (8.1 on the Richter scale) occurred on the western coast of Mexico. The seismic waves from the earthquake should have been too weak to cause extensive damage when they reached Mexico City about 400 km away. However, Mexico City is largely built on an ancient lake bed, where the soil is still soft with water. Although the amplitude of the seismic waves was small in the firmer ground en route to Mexico City, their amplitude substantially increased in the loose soil of the city. Acceleration amplitudes of the waves were as much as 0.20g, and the angular frequency was (surprisingly) concentrated around 3 rad/s. Not only was the ground severely oscillated, but many intermediate-height buildings had resonant angular frequencies of about 3 rad/s. Most of those buildings collapsed during the shaking (Fig. 15-19), while shorter buildings (with higher resonant angular frequencies) and taller buildings (with lower resonant angular frequencies) remained standing. During a 1989 earthquake in the San Francisco–Oakland area, a similar resonant oscillation collapsed part of a freeway, dropping an upper deck onto a lower deck. That section of the freeway had been constructed on a loosely structured mudfill.

John T. Barr/Getty Images, Inc.

Figure 15-19 In 1985, buildings of intermediate height collapsed in Mexico City as a result of an earthquake far from the city. Taller and shorter buildings remained standing.

434

CHAPTE R 15 OSCI LL ATIONS

Review & Summary Frequency The frequency f of periodic, or oscillatory, motion is the number of oscillations per second. In the SI system, it is measured in hertz: 1 hertz $ 1 Hz $ 1 oscillation per second $ 1 s!1.

(15-1)

Pendulums Examples of devices that undergo simple

harmonic motion are the torsion pendulum of Fig. 15-9, the simple pendulum of Fig. 15-11, and the physical pendulum of Fig. 15-12. Their periods of oscillation for small oscillations are, respectively, T $ 2p 2I/k

Period The period T is the time required for one complete oscil-

lation, or cycle. It is related to the frequency by T$

1 . f

the displacement x(t) of a particle from its equilibrium position is described by the equation x $ xm cos(vt % f)

(displacement),

(15-3)

in which xm is the amplitude of the displacement, vt % f is the phase of the motion, and f is the phase constant. The angular frequency v is related to the period and frequency of the motion by 2p v$ $ 2p f T

(angular frequency).

(15-5)

Differentiating Eq. 15-3 leads to equations for the particle’s SHM velocity and acceleration as functions of time: v $ !vxm sin(vt % f) and

2

a $ !v xm cos(vt % f)

(velocity)

(15-6)

(acceleration).

(15-7)

In Eq. 15-6, the positive quantity vxm is the velocity amplitude vm of the motion. In Eq. 15-7, the positive quantity v 2xm is the acceleration amplitude am of the motion.

The Linear Oscillator A particle with mass m that moves under the influence of a Hooke’s law restoring force given by F $ !kx exhibits simple harmonic motion with v$ and

k Am

(angular frequency)

m T $ 2p A k

(15-23)

(simple pendulum),

(15-28)

T $ 2p 2L/g

T $ 2p 2I/mgh

(15-2)

Simple Harmonic Motion In simple harmonic motion (SHM),

(torsion pendulum),

(physical pendulum).

(15-29)

Simple Harmonic Motion and Uniform Circular Motion Simple harmonic motion is the projection of uniform circular motion onto the diameter of the circle in which the circular motion occurs. Figure 15-15 shows that all parameters of circular motion (position, velocity, and acceleration) project to the corresponding values for simple harmonic motion.

Damped Harmonic Motion The mechanical energy E in a real oscillating system decreases during the oscillations because external forces, such as a drag force, inhibit the oscillations and transfer mechanical energy to thermal energy. The real oscillator and its motion : are then said to be damped. If the damping force is given by F d $ : : !b v , where v is the velocity of the oscillator and b is a damping constant, then the displacement of the oscillator is given by x(t) $ xm e!bt/2m cos(v#t % f),

(15-42)

where v#, the angular frequency of the damped oscillator, is given by k b2 v# $ ! . (15-43) Am 4m2

If the damping constant is small (b & 1km), then v# % v, where v is the angular frequency of the undamped oscillator. For small b, the mechanical energy E of the oscillator is given by E(t) % 12kx2m e!bt/m.

(15-12)

(15-44)

Forced Oscillations and Resonance If an external (period).

(15-13)

Such a system is called a linear simple harmonic oscillator.

Energy A particle in simple harmonic motion has, at any time,

kinetic energy K $ 12mv2 and potential energy U $ 12kx2. If no friction is present, the mechanical energy E $ K % U remains constant even though K and U change.

driving force with angular frequency vd acts on an oscillating system with natural angular frequency v, the system oscillates with angular frequency vd. The velocity amplitude vm of the system is greatest when vd $ v, (15-46) a condition called resonance. The amplitude xm of the system is (approximately) greatest under the same condition.

Questions 1 Which of the following describe f for the SHM of Fig. 15-20a: (a) !p ' f ' !p/2, (b) p ' f ' 3p/2,

and (d) point B? Is the speed of the particle increasing or decreasing at (e) point A and (f) point B? x

v B

(c) !3p/2 ' f ' !p? 2 The velocity v(t) of a particle undergoing SHM is graphed in Fig. 15-20b. Is the particle momentarily stationary, headed toward !xm, or headed toward %xm at (a) point A on the graph and (b) point B? Is the particle at !xm, at %xm, at 0, between !xm and 0, or between 0 and %xm when its velocity is represented by (c) point A

t

t A

(a)

Figure 15-20 Questions 1 and 2.

(b)

435

QU ESTIONS

3 The acceleration a(t) of a parti- a cle undergoing SHM is graphed in 2 Fig. 15-21. (a) Which of the labeled 1 3 points corresponds to the particle 4 8 at !xm? (b) At point 4, is the veloct ity of the particle positive, negative, 7 5 or zero? (c) At point 5, is the parti6 cle at !xm, at %xm, at 0, between !xm and 0, or between 0 and %xm? Figure 15-21 Question 3. 4 Which of the following relationships between the acceleration a and the displacement x of a particle involve SHM: (a) a $ 0.5x, (b) a $ 400x2, (c) a $ !20x, (d) a $ !3x2? 5 You are to complete Fig. 15-22a so that it is a plot of velocity v versus time t for the spring–block oscillator that is shown in Fig. 15-22b for t $ 0. (a) In Fig. 15-22a, at which lettered point or in what region between the points should the (vertical) v axis intersect the t axis? (For example, should it intersect at point A, or maybe in the region between points A and B?) (b) If the block’s velocity is given by v $ !vm sin(vt % f), what is the value of f? Make it positive, and if you cannot specify the value (such as %p/2 rad), then give a range of values (such as between 0 and p/2 rad).

A

B

C

D

E

t

(a) t=0

–xm

xm

0

x

(b)

Figure 15-22 Question 5.

K

Rank the plots according to (a) the corresponding spring constant and (b) the corresponding period of the oscillator, greatest first.

A B

9 Figure 15-26 shows three physical C pendulums consisting of identical uniform spheres of the same mass that x are rigidly connected by identical rods –x xm m of negligible mass. Each pendulum is Figure 15-25 Question 8. vertical and can pivot about suspension point O. Rank the pendulums acO cording to their period of oscillation, greatest first. 10 You are to build the oscillation O O transfer device shown in Fig. 15-27. It consists of two spring – block systems hanging from a flexible rod. When (b) (c) the spring of system 1 is stretched (a) and then released, the resulting SHM Figure 15-26 Question 9. of system 1 at frequency f1 oscillates the rod. The rod then exerts a driving force on system 2, at the same frequency f1. You can choose from four springs with spring constants k of 1600, 1500, 1400, and 1200 N/m, and four blocks with masses m of 800, 500, 400, and 200 kg. Mentally determine which spring should go with which block in each of the two systems to maximize the amplitude of oscillations in system 2. Rod

6 You are to complete Fig. 15-23a t B C D E so that it is a plot of acceleration a A versus time t for the spring – block oscillator that is shown in Fig. 15(a) 23b for t $ 0. (a) In Fig. 15-23a, at which lettered point or in what ret=0 gion between the points should the (vertical) a axis intersect the t axis? x (For example, should it intersect at xm –xm 0 point A, or maybe in the region be(b) tween points A and B?) (b) If the Figure 15-23 Question 6. block’s acceleration is given by a $ !am cos(vt % f), what is the value of f? Make it positive, and if you cannot specify the value (such as %p/2 rad), then give a range of values (such as between 0 and p/2). x

7 Figure 15-24 shows the x(t) curves for three experiments involving a 1 2 particular spring–box system oscillating in t SHM. Rank the curves 3 according to (a) the system’s angular frequency, (b) the spring’s potential energy at time t $ 0, (c) the box’s kinetic enFigure 15-24 Question 7. ergy at t $ 0, (d) the box’s speed at t $ 0, and (e) the box’s maximum kinetic energy, greatest first. 8 Figure 15-25 shows plots of the kinetic energy K versus position x for three harmonic oscillators that have the same mass.

System 1

System 2

Figure 15-27 Question 10. 11 In Fig. 15-28, a spring – block system is put into SHM in two experiments. In the first, the block is pulled from the equilibrium position d1 through a displacement d1 and then d2 released. In the second, it is pulled Figure 15-28 Question 11. from the equilibrium position through a greater displacement d2 and then released. Are the (a) amplitude, (b) period, (c) frequency, (d) maximum kinetic energy, and (e) maximum potential energy in the second experiment greater than, less than, or the same as those in the first experiment? 12 Figure 15-29 gives, for three situations, the displacements x(t) of a pair of simple harmonic oscillators (A and B) that are identical except for phase. For each pair, what phase shift (in radians and in degrees) is needed to shift the curve for A to coincide with the curve for B? Of the many possible answers, choose the shift with the smallest absolute magnitude. x

x

x A

B

A

B

t

t

t B

(a)

(b)

Figure 15-29 Question 12.

A (c)

436

CHAPTE R 15 OSCI LL ATIONS

Problems Tutoring problem available (at instructor’s discretion) in WileyPLUS and WebAssign SSM • – •••

Worked-out solution available in Student Solutions Manual

WWW Worked-out solution is at

Number of dots indicates level of problem difficulty

ILW

Interactive solution is at

http://www.wiley.com/college/halliday

Additional information available in The Flying Circus of Physics and at flyingcircusofphysics.com

Module 15-1 Simple Harmonic Motion •1 An object undergoing simple harmonic motion takes 0.25 s to travel from one point of zero velocity to the next such point. The distance between those points is 36 cm. Calculate the (a) period, (b) frequency, and (c) amplitude of the motion. •2 A 0.12 kg body undergoes simple harmonic motion of amplitude 8.5 cm and period 0.20 s. (a) What is the magnitude of the maximum force acting on it? (b) If the oscillations are produced by a spring, what is the spring constant? •3 What is the maximum acceleration of a platform that oscillates at amplitude 2.20 cm and frequency 6.60 Hz? •4 An automobile can be considered to be mounted on four identical springs as far as vertical oscillations are concerned.The springs of a certain car are adjusted so that the oscillations have a frequency of 3.00 Hz. (a) What is the spring constant of each spring if the mass of the car is 1450 kg and the mass is evenly distributed over the springs? (b) What will be the oscillation frequency if five passengers, averaging 73.0 kg each, ride in the car with an even distribution of mass? •5 SSM In an electric shaver, the blade moves back and forth over a distance of 2.0 mm in simple harmonic motion, with frequency 120 Hz. Find (a) the amplitude, (b) the maximum blade speed, and (c) the magnitude of the maximum blade acceleration. •6 A particle with a mass of 1.00 ( 10 kg is oscillating with simple harmonic motion with a period of 1.00 ( 10!5 s and a maximum speed of 1.00 ( 103 m/s. Calculate (a) the angular frequency and (b) the maximum displacement of the particle. !20

•7 SSM A loudspeaker produces a musical sound by means of the oscillation of a diaphragm whose amplitude is limited to 1.00 mm. (a) At what frequency is the magnitude a of the diaphragm’s acceleration equal to g? (b) For greater frequencies, is a greater than or less than g? •8 What is the phase constant for the harmonic oscillator with the position function x(t) given in Fig. 1530 if the position function has the form x $ xm cos(vt % f)? The vertical axis scale is set by xs $ 6.0 cm.

x (cm) xs

t

•9 The position function x $ (6.0 m) cos[(3p rad/s)t % p/3 rad] –xs gives the simple harmonic motion Figure 15-30 Problem 8. of a body. At t $ 2.0 s, what are the (a) displacement, (b) velocity, (c) acceleration, and (d) phase of the motion? Also, what are the (e) frequency and (f ) period of the motion?

are attached to a block of mass 0.245 kg. What is the frequency of oscillation on the frictionless floor? •12 What is the phase constant for the harmonic oscillator with the velocity function v(t) given in Fig. 15-32 if the position function x(t) has the form x $ xm cos(vt % f)? The vertical axis scale is set by vs $ 4.0 cm/s.

v (cm/s) vs t –vs

•13 SSM An oscillator consists of a Figure 15-32 Problem 12. block of mass 0.500 kg connected to a spring. When set into oscillation with amplitude 35.0 cm, the oscillator repeats its motion every 0.500 s. Find the (a) period, (b) frequency, (c) angular frequency, (d) spring constant, (e) maximum speed, and (f) magnitude of the maximum force on the block from the spring. ••14 A simple harmonic oscillator consists of a block of mass 2.00 kg attached to a spring of spring constant 100 N/m. When t $ 1.00 s, the position and velocity of the block are x $ 0.129 m and v $ 3.415 m/s. (a) What is the amplitude of the oscillations? What were the (b) position and (c) velocity of the block at t $ 0 s? ••15 SSM Two particles oscillate in simple harmonic motion along a common straight-line segment of length A. Each particle has a period of 1.5 s, but they differ in phase by p/6 rad. (a) How far apart are they (in terms of A) 0.50 s after the lagging particle leaves one end of the path? (b) Are they then moving in the same direction, toward each other, or away from each other? ••16 Two particles execute simple harmonic motion of the same amplitude and frequency along close parallel lines. They pass each other moving in opposite directions each time their displacement is half their amplitude. What is their phase difference? ••17 ILW An oscillator consists of a block attached to a spring (k $ 400 N/m). At some time t, the position (measured from the system’s equilibrium location), velocity, and acceleration of the block are x $ 0.100 m, v $ !13.6 m/s, and a $ !123 m/s2. Calculate (a) the frequency of oscillation, (b) the mass of the block, and (c) the amplitude of the motion. ••18 At a certain harbor, the tides cause the ocean surface to rise and fall a distance d (from highest level to lowest level) in simple harmonic motion, with a period of 12.5 h. How long does it take for the water to fall a distance 0.250d from its highest level?

•10 An oscillating block – spring system takes 0.75 s to begin repeating its motion. Find (a) the period, (b) the frequency in hertz, and m (c) the angular frequency in radians per second.

••19 A block rides on a piston (a squat cylindrical piece) that is moving vertically with simple harmonic motion. (a) If the SHM has period 1.0 s, at what amplitude of motion will the block and piston separate? (b) If the piston has an amplitude of 5.0 cm, what is the maximum frequency for which the block and piston will be in contact continuously?

•11 In Fig. 15-31, two identical springs of spring constant 7580 N/m

••20 Figure 15-33a is a partial graph of the position function x(t) for a simple harmonic oscillator with an angular frequency of

Figure 15-31 Problems 11 and 21.

PROB LE M S

1.20 rad/s; Fig. 15-33b is a partial graph of the corresponding velocity function v(t). The vertical axis scales are set by xs $ 5.0 cm and vs $ 5.0 cm/s. What is the phase constant of the SHM if the position function x(t) is in the general form x $ xm cos(vt % f)? ••21 ILW In Fig. 15-31, two springs are attached to a block that can oscillate over a frictionless floor. If the left spring is removed, the block oscillates at a frequency of 30 Hz. If, instead, the spring on the right is removed, the block oscillates at a frequency of 45 Hz. At what frequency does the block oscillate with both springs attached?

x (cm) xs

t

–xs (a) v (cm/s) vs

t

–vs (b)

Figure 15-33 Problem 20.

••22 Figure 15-34 shows block 1 of mass 0.200 kg slid2 k 1 ing to the right over a frictionless elevated surface at a h speed of 8.00 m/s. The block undergoes an elastic collision d with stationary block 2, which is attached to a spring of spring Figure 15-34 Problem 22. constant 1208.5 N/m. (Assume that the spring does not affect the collision.) After the collision, block 2 oscillates in SHM with a period of 0.140 s, and block 1 slides off the opposite end of the elevated surface, landing a distance d from the base of that surface after falling height h $ 4.90 m. What is the value of d? ••23 SSM WWW A block is on a horizontal surface (a shake table) that is moving back and forth horizontally with simple harmonic motion of frequency 2.0 Hz. The coefficient of static friction between block and surface is 0.50. How great can the amplitude of the SHM be if the block is not to slip along the surface? •••24 In Fig. 15-35, two springs are joined and connected to a block of mass 0.245 kg that is set oscillating over a frictionless floor. The springs each have spring constant k $ 6430 N/m. What is the frequency of the oscillations?

m

k

k

Figure 15-35 Problem 24.

k

•••25 In Fig. 15-36, a block weighing 14.0 N, which can slide without friction on an incline at angle u $ 40.0), is connected to the top of the incline by a massless spring of unstretched length 0.450 θ m and spring constant 120 N/m. (a) Figure 15-36 Problem 25. How far from the top of the incline is the block’s equilibrium point? (b) If the block is pulled slightly down the incline and released, what is the period of the resulting oscillations? •••26

In Fig. 15-37, two blocks (m $ 1.8 kg and M $ 10 kg) and

437

a spring (k $ 200 N/m) are arm ranged on a horizontal, frictionless k surface. The coefficient of static M friction between the two blocks is 0.40. What amplitude of simple harmonic motion of the spring – blocks Figure 15-37 Problem 26. system puts the smaller block on the verge of slipping over the larger block? Module 15-2 Energy in Simple Harmonic Motion •27 SSM When the displacement in SHM is one-half the amplitude xm, what fraction of the total energy is (a) kinetic energy and (b) potential energy? (c) At what displacement, in terms of the amplitude, is the energy of the system half kinetic energy and half potential energy? U (J)

•28 Figure 15-38 gives the oneUs dimensional potential energy well for a 2.0 kg particle (the function U(x) has the form bx 2 and the vertical axis scale is set by Us $ 2.0 J). (a) If the particle passes through the equilibrium position with a velocity of 85 cm/s, will it be turned –20 –10 0 10 20 back before it reaches x $ 15 cm? x (cm) (b) If yes, at what position, and if Figure 15-38 Problem 28. no, what is the speed of the particle at x $ 15 cm? •29 SSM Find the mechanical energy of a block – spring system with a spring constant of 1.3 N/cm and an amplitude of 2.4 cm. •30 An oscillating block – spring system has a mechanical energy of 1.00 J, an amplitude of 10.0 cm, and a maximum speed of 1.20 m/s. Find (a) the spring constant, (b) the mass of the block, and (c) the frequency of oscillation. •31 ILW A 5.00 kg object on a horizontal frictionless surface is attached to a spring with k $ 1000 N/m. The object is displaced from equilibrium 50.0 cm horizontally and given an initial velocity of 10.0 m/s back toward the equilibrium position. What are (a) the motion’s frequency, (b) the initial potential energy of the block – spring system, (c) the initial kinetic energy, and (d) the motion’s amplitude? •32 Figure 15-39 shows the kinetic energy K of a simple harmonic oscillator versus its position x. The vertical axis scale is set by Ks $ 4.0 J. What is the spring constant?

K (J) Ks

••33 A block of mass M $ 5.4 kg, at rest on a horizontal –12 –8 –4 0 4 8 12 frictionless table, is attached to a x (cm) rigid support by a spring of conFigure 15-39 Problem 32. stant k $ 6000 N/m. A bullet of : mass m $ 9.5 g and velocity v of magnitude 630 m/s strikes and is v embedded in the block (Fig. 15k 40). Assuming the compression of M m the spring is negligible until the bullet is embedded, determine (a) Figure 15-40 Problem 33. the speed of the block immediately after the collision and (b) the amplitude of the resulting simple harmonic motion.

438

CHAPTE R 15 OSCI LL ATIONS

••34 In Fig. 15-41, block 2 of k mass 2.0 kg oscillates on the end of a 1 2 spring in SHM with a period of 20 ms. The block’s position is given by Figure 15-41 Problem 34. x $ (1.0 cm) cos(vt % p/2). Block 1 of mass 4.0 kg slides toward block 2 with a velocity of magnitude 6.0 m/s, directed along the spring’s length. The two blocks undergo a completely inelastic collision at time t $ 5.0 ms. (The duration of the collision is much less than the period of motion.) What is the amplitude of the SHM after the collision? ••35 A 10 g particle undergoes SHM with an amplitude of 2.0 mm, a maximum acceleration of magnitude 8.0 ( 103 m/s2, and an unknown phase constant f. What are (a) the period of the motion, (b) the maximum speed of the particle, and (c) the total mechanical energy of the oscillator? What is the magnitude of the force on the particle when the particle is at (d) its maximum displacement and (e) half its maximum displacement? ••36 If the phase angle for a block – spring system in SHM is p/6 rad and the block’s position is given by x $ xm cos(vt % f), what is the ratio of the kinetic energy to the potential energy at time t $ 0? •••37 A massless spring hangs from the ceiling with a small object attached to its lower end. The object is initially held at rest in a position yi such that the spring is at its rest length. The object is then released from yi and oscillates up and down, with its lowest position being 10 cm below yi. (a) What is the frequency of the oscillation? (b) What is the speed of the object when it is 8.0 cm below the initial position? (c) An object of mass 300 g is attached to the first object, after which the system oscillates with half the original frequency. What is the mass of the first object? (d) How far below yi is the new equilibrium (rest) position with both objects attached to the spring? Module 15-3 An Angular Simple Harmonic Oscillator •38 A 95 kg solid sphere with a 15 cm radius is suspended by a vertical wire. A torque of 0.20 N *m is required to rotate the sphere through an angle of 0.85 rad and then maintain that orientation. What is the period of the oscillations that result when the sphere is then released? ••39 SSM WWW The balance wheel of an old-fashioned watch oscillates with angular amplitude p rad and period 0.500 s. Find (a) the maximum angular speed of the wheel, (b) the angular speed at displacement p/2 rad, and (c) the magnitude of the angular acceleration at displacement p/4 rad. Module 15-4 Pendulums, Circular Motion •40 ILW A physical pendulum consists of a meter stick that is pivoted at a small hole drilled through the stick a distance d from the 50 cm mark. The period of oscillation is 2.5 s. Find d. •41 SSM In Fig. 15-42, the pendulum consists of a uniform disk with radius r $ 10.0 cm and mass 500 g attached to a uniform rod with length L $ 500 mm and mass 270 g. (a) Calculate the rotational inertia of the pendulum about the pivot point. (b) What is the distance between the pivot point and

the center of mass of the pendulum? (c) Calculate the period of oscillation. •42 Suppose that a simple pendulum consists of a small 60.0 g bob at the end of a cord of negligible mass. If the angle u between the cord and the vertical is given by u $ (0.0800 rad) cos[(4.43 rad/s)t % f], what are (a) the pendulum’s length and (b) its maximum kinetic energy? •43 (a) If the physical pendulum of Fig. 15-13 and the associated sample problem is inverted and suspended at point P, what is its period of oscillation? (b) Is the period now greater than, less than, or equal to its previous value? •44 A physical pendulum consists of two meter-long sticks joined together as shown in Fig. 15-43. What is the pendulum’s period of oscillation about a pin inserted through point A at the center of the horizontal stick?

•45 A performer seated on a trapeze is Figure 15-43 swinging back and forth with a period of 8.85 s. Problem 44. If she stands up, thus raising the center of mass of the trapeze % performer system by 35.0 cm, what will be the new period of the system? Treat trapeze % performer as a simple pendulum. •46 A physical pendulum has a center of oscillation at distance 2L/3 from its point of suspension. Show that the distance between the point of suspension and the center of oscillation for a physical pendulum of any form is I/mh, where I and h have the meanings in Eq. 15-29 and m is the mass of the pendulum. •47 In Fig. 15-44, a physical pendulum consists of a uniform solid disk (of radius R $ 2.35 cm) supported in a vertical plane by a pivot located a distance d $ 1.75 cm from the center of the disk. The disk is displaced by a small angle and released. What is the period of the resulting simple harmonic motion?

Pivot

r

Figure 15-42 Problem 41.

d

R

Figure 15-44 ••48 A rectangular block, with face Problem 47. lengths a $ 35 cm and b $ 45 cm, is to be suspended on a thin horizontal rod running through a narrow hole in the block. The block is then to be set swinging about the rod like a pendulum, through small angles so that it is in SHM. Figure 15-45 shows one possible position of the hole, at distance r from the block’s center, along a line connecting the center with a corner. (a) Plot the period versus distance r along that line such that the minimum in the a curve is apparent. (b) For what value of r does that minimum occur? There is a line of points around the block’s r center for which the period of swinging has the same minimum value. (c) What shape does that line make? b

L

A

••49 The angle of the pendulum of Fig. 15-11b is given by u $ um cos[(4.44 rad/s)t % f]. If at t $ 0, u $ 0.040 rad and du/dt $ !0.200 Figure 15-45 Problem 48. rad/s, what are (a) the phase constant f and (b) the maximum angle um? (Hint: Don’t confuse the rate du/dt at which u changes with the v of the SHM.)

PROB LE M S

••50 A thin uniform rod (mass $ 0.50 kg) swings about an axis that passes through one end of the rod and is perpendicular to the plane of the swing. The rod swings with a period of 1.5 s L/2 and an angular amplitude of 10). O (a) What is the length of the rod? x (b) What is the maximum kinetic energy of the rod as it swings? ••51 In Fig. 15-46, a stick of length L $ 1.85 m oscillates as a physical pendulum. (a) What value of distance x between the stick’s center of mass and its pivot point O gives the least period? (b) What is that least period?

L/2

Figure 15-46 Problem 51.

••52 The 3.00 kg cube in Fig. 15-47 has edge lengths d $ 6.00 cm and is mounted on an axle through its center. A spring (k $ 1200 N/m) connects the cube’s upper corner to a rigid wall. Initially the spring is at its rest length. If the cube is rotated 3) and released, what is the period of the resulting SHM?

d d

439

sen to minimize the period and then L is increased, does the period increase, decrease, or remain the same? (c) If, instead, m is increased without L increasing, does the period increase, decrease, or remain the same? •••56 In Fig. 15-50, a 2.50 kg disk of diameter D $ 42.0 cm is supL ported by a rod of length L $ 76.0 cm and negligible mass that is pivoted at its end. (a) With the massless torsion spring unconnected, what is the period of oscillation? (b) With the torsion spring connected, the rod D is vertical at equilibrium. What is the torsion constant of the spring if the period of oscillation has been de- Figure 15-50 Problem 56. creased by 0.500 s? Module 15-5 Damped Simple Harmonic Motion •57 The amplitude of a lightly damped oscillator decreases by 3.0% during each cycle. What percentage of the mechanical energy of the oscillator is lost in each cycle?

Figure 15-47 ••53 SSM ILW In the overhead view of Fig. 15Problem 52. 48, a long uniform rod of mass 0.600 kg is free to rotate in a horizontal plane about a Wall vertical axis through its center. A spring with force constant k $ 1850 N/m is connected horizontally be- k tween one end of the rod and a Rotation axis fixed wall. When the rod is in equiFigure 15-48 Problem 53. librium, it is parallel to the wall. What is the period of the small oscillations that result when the rod is rotated slightly and released?

•58 For the damped oscillator system shown in Fig. 15-16, with m $ 250 g, k $ 85 N/m, and b $ 70 g/s, what is the ratio of the oscillation amplitude at the end of 20 cycles to the initial oscillation amplitude?

••54 In Fig. 15-49a, a metal plate is mounted on an axle through its center of mass. A spring with k $ 2000 N/m connects a wall with a point on the rim a distance r $ 2.5 cm from the center of mass. Initially the spring is at its rest length. If the plate is rotated by 7) and released, it rotates about the axle in SHM, with its angular position given by Fig. 15-49b.The horizontal axis scale is set by ts $ 20 ms.What is the rotational inertia of the plate about its center of mass?

••60 The suspension system of a 2000 kg automobile “sags” 10 cm when the chassis is placed on it. Also, the oscillation amplitude decreases by 50% each cycle. Estimate the values of (a) the spring constant k and (b) the damping constant b for the spring and shock absorber system of one wheel, assuming each wheel supports 500 kg.

θ (deg) 8 4 r

0

0

ts

t (ms)

–4 –8 (a)

(b)

Figure 15-49 Problem 54. •••55 A pendulum is formed by pivoting a long thin rod about a point on the rod. In a series of experiments, the period is measured as a function of the distance x between the pivot point and the rod’s center. (a) If the rod’s length is L $ 2.20 m and its mass is m $ 22.1 g, what is the minimum period? (b) If x is cho-

•59 SSM WWW For the damped oscillator system shown in Fig. 15-16, the block has a mass of 1.50 kg and the spring constant is 8.00 N/m. The damping force is given by !b(dx/dt), where b $ 230 g/s. The block is pulled down 12.0 cm and released. (a) Calculate the time required for the amplitude of the resulting oscillations to fall to one-third of its initial value. (b) How many oscillations are made by the block in this time?

Module 15-6 Forced Oscillations and Resonance •61 For Eq. 15-45, suppose the amplitude xm is given by Fm xm $ , [m2(v 2d ! v 2)2 % b2v d2]1/2 where Fm is the (constant) amplitude of the external oscillating force exerted on the spring by the rigid support in Fig. 15-16. At resonance, what are the (a) amplitude and (b) velocity amplitude of the oscillating object? •62 Hanging from a horizontal beam are nine simple pendulums of the following lengths: (a) 0.10, (b) 0.30, (c) 0.40, (d) 0.80, (e) 1.2, (f) 2.8, (g) 3.5, (h) 5.0, and (i) 6.2 m. Suppose the beam undergoes horizontal oscillations with angular frequencies in the range from 2.00 rad/s to 4.00 rad/s. Which of the pendulums will be (strongly) set in motion? ••63 A 1000 kg car carrying four 82 kg people travels over a “washboard” dirt road with corrugations 4.0 m apart. The car bounces with maximum amplitude when its speed is 16 km/h. When the car stops, and the people get out, by how much does the car body rise on its suspension?

440

CHAPTE R 15 OSCI LL ATIONS

Additional Problems 64 Although California is known for earthquakes, it has large regions dotted with precariously balanced rocks that would be easily toppled by even a mild earthquake. Apparently no major earthquakes have occurred in those regions. If an earthquake were to put such a rock into sinusoidal oscillation (parallel to the ground) with a frequency of 2.2 Hz, an oscillation amplitude of 1.0 cm would cause the rock to topple. What would be the magnitude of the maximum acceleration of the oscillation, in terms of g?

is hung from its end. (a) Calculate the spring constant. This block is then displaced an additional 5.0 cm downward and released from rest. Find the (b) period, (c) frequency, (d) amplitude, and (e) maximum speed of the resulting SHM.

65 A loudspeaker diaphragm is oscillating in simple harmonic motion with a frequency of 440 Hz and a maximum displacement of 0.75 mm. What are the (a) angular frequency, (b) maximum speed, and (c) magnitude of the maximum acceleration?

75 A 4.00 kg block is suspended from a spring with k $ 500 N/m. A 50.0 g bullet is fired into the block from directly below with a speed of 150 m/s and becomes embedded in the block. (a) Find the amplitude of the resulting SHM. (b) What percentage of the original kinetic energy of the bullet is transferred to mechanical energy of the oscillator?

66 A uniform spring with k $ 8600 N/m is cut into pieces 1 and 2 of unstretched lengths L1 $ 7.0 cm and L2 $ 10 cm. What are (a) k1 and (b) k2? A block attached to the original spring as in Fig. 15-7 oscillates at 200 Hz. What is the oscillation frequency of the block attached to (c) piece 1 and (d) piece 2? 67 In Fig. 15-51, three 10 000 kg ore cars are held at rest on a mine railway using a cable that is parallel to the rails, which are inclined at angle u $ 30). The cable stretches 15 cm just before the coupling between the two lower cars breaks, detaching the lowest car. Assuming that the cable obeys Hooke’s law, find the (a) frequency and (b) amplitude of the resulting oscillations of the remaining two cars.

Car that breaks free

θ

77 Figure 15-53 gives the position of a 20 g block oscillating in SHM on the end of a spring. The horizontal axis scale is set by ts $ 40.0 ms. What are (a) the maximum kinetic energy of the block and (b) the number of times per second that maximum is reached? (Hint: Measuring a slope will probably not be very accurate. Find another approach.)

4

Figure 15-51 Problem 67.

69 SSM In the engine of a locomotive, a cylindrical piece known as a piston oscillates in SHM in a cylinder head (cylindrical chamber) with an angular frequency of 180 rev/min. Its stroke (twice the amplitude) is 0.76 m. What is its maximum speed? 70 A wheel is free to rotate about its fixed axle. A spring is attached to one of its spokes a distance r from the axle, as shown in Fig. 15-52. (a) Assuming that the wheel is a hoop of mass m and radius R, what is the angular frequency v of k small oscillations of this system in terms of m, R, r, and the spring conR r stant k? What is v if (b) r $ R and (c) r $ 0? 71 A 50.0 g stone is attached to the bottom of a vertical spring and set vibrating. If the maximum speed Figure 15-52 Problem 70. of the stone is 15.0 cm/s and the period is 0.500 s, find the (a) spring constant of the spring, (b) amplitude of the motion, and (c) frequency of oscillation. 72 A uniform circular disk whose radius R is 12.6 cm is suspended as a physical pendulum from a point on its rim. (a) What is its period? (b) At what radial distance r ' R is there a pivot point that gives the same period? SSM

76 A 55.0 g block oscillates in SHM on the end of a spring with k $ 1500 N/m according to x $ xm cos(vt % f). How long does the block take to move from position %0.800xm to (a) position %0.600xm and (b) position !0.800xm?

x (cm) 8

68 A 2.00 kg block hangs from a spring.A 300 g body hung below the block stretches the spring 2.00 cm farther. (a) What is the spring constant? (b) If the 300 g body is removed and the block is set into oscillation,find the period of the motion.

73

74 A massless spring with spring constant 19 N/m hangs vertically. A body of mass 0.20 kg is attached to its free end and then released. Assume that the spring was unstretched before the body was released. Find (a) how far below the initial position the body descends, and the (b) frequency and (c) amplitude of the resulting SHM.

A vertical spring stretches 9.6 cm when a 1.3 kg block

0

t (ms)

ts

–4 –8

Figure 15-53 Problems 77 and 78. 78 Figure 15-53 gives the position x(t) of a block oscillating in SHM on the end of a spring (ts $ 40.0 ms).What are (a) the speed and (b) the magnitude of the radial acceleration of a particle in the corresponding uniform circular motion? 79 Figure 15-54 shows the kinetic energy K of a simple pendulum versus its angle u from the vertical. The vertical axis scale is set by Ks $ 10.0 mJ. The pendulum bob has mass 0.200 kg. What is the length of the pendulum?

K (mJ) Ks

0 50 100 80 A block is in SHM on the end –100 –50 θ (mrad) of a spring, with position given by Figure 15-54 Problem 79. x $ xm cos(vt % f). If f $ p/5 rad, then at t $ 0 what percentage of the total mechanical energy is potential energy?

81 A simple harmonic oscillator consists of a 0.50 kg block attached to a spring. The block slides back and forth along a straight line on a frictionless surface with equilibrium point x $ 0. At t $ 0 the block is at x $ 0 and moving in the positive x direction. A graph : of the magnitude of the net force F on the block as a function of its

441

PROB LE M S

position is shown in Fig. 15-55. The vertical scale is set by Fs $ 75.0 N. What are (a) the amplitude and (b) the period of the motion, (c) the magnitude of the maximum acceleration, and (d) the maximum kinetic energy?

F (N) Fs 0.30 –0.30

x (m)

–Fs

82 A simple pendulum of Figure 15-55 Problem 81. length 20 cm and mass 5.0 g is suspended in a race car traveling with constant speed 70 m/s around a circle of radius 50 m. If the pendulum undergoes small oscillations in a radial direction about its equilibrium position, what is the frequency of oscillation? 83 The scale of a spring balance that reads from 0 to 15.0 kg is 12.0 cm long. A package suspended from the balance is found to oscillate vertically with a frequency of 2.00 Hz. (a) What is the spring constant? (b) How much does the package weigh? 84 A 0.10 kg block oscillates back and forth along a straight line on a frictionless horizontal surface. Its displacement from the origin is given by x $ (10 cm) cos[(10 rad/s)t % p/2 rad].

90 A particle executes linear SHM with frequency 0.25 Hz about the point x $ 0. At t $ 0, it has displacement x $ 0.37 cm and zero velocity. For the motion, determine the (a) period, (b) angular frequency, (c) amplitude, (d) displacement x(t), (e) velocity v(t), (f) maximum speed, (g) magnitude of the maximum acceleration, (h) displacement at t $ 3.0 s, and (i) speed at t $ 3.0 s. 91 SSM What is the frequency of a simple pendulum 2.0 m long (a) in a room, (b) in an elevator accelerating upward at a rate of 2.0 m/s2, and (c) in free fall? 92 A grandfather clock has a pendulum that consists of a thin brass disk of radius r $ 15.00 cm and mass 1.000 kg that is attached to a long thin rod of negligible mass. The pendulum swings freely about an axis perpendicular to the rod and through the end of the rod opposite the disk, as shown in Fig. 15-56. If the pendulum is to have a period of 2.000 s for small oscillations at a place where g $ 9.800 m/s2, what must be the rod length L to the nearest tenth of a millimeter?

Rotation axis

L

r

(a) What is the oscillation frequency? (b) What is the maximum speed acquired by the block? (c) At what value of x does this occur? (d) What is the magnitude of the maximum acceleration of the block? (e) At what value of x does this occur? (f ) What force, applied to the block by the spring, results in the given oscillation?

93 A 4.00 kg block hangs from a Figure 15-56 Problem 92. spring, extending it 16.0 cm from its unstretched position. (a) What is the spring constant? (b) The block is removed, and a 0.500 kg body is hung from the same spring. If the spring is then stretched and released, what is its period of oscillation?

85 The end point of a spring oscillates with a period of 2.0 s when a block with mass m is attached to it. When this mass is increased by 2.0 kg, the period is found to be 3.0 s. Find m.

94 What is the phase constant for SMH with a(t) given in Fig. 15-57 if the position function x(t) has the form x $ xm cos(vt % f) and as $ 4.0 m/s2?

88 A block weighing 20 N oscillates at one end of a vertical spring for which k $ 100 N/m; the other end of the spring is attached to a ceiling. At a certain instant the spring is stretched 0.30 m beyond its relaxed length (the length when no object is attached) and the block has zero velocity. (a) What is the net force on the block at this instant? What are the (b) amplitude and (c) period of the resulting simple harmonic motion? (d) What is the maximum kinetic energy of the block as it oscillates? 89 A 3.0 kg particle is in simple harmonic motion in one dimension and moves according to the equation x $ (5.0 m) cos[(p/3 rad/s)t ! p/4 rad], with t in seconds. (a) At what value of x is the potential energy of the particle equal to half the total energy? (b) How long does the particle take to move to this position x from the equilibrium position?

96 A spider can tell when its web has captured, say, a fly because the fly’s thrashing causes the web threads to oscillate. A spider can even determine the size of the fly by the frequency of the oscillations. Assume that a fly oscillates on the capτs ture thread on which it is caught like a block on a spring. What is the ratio of oscillation frequency for a 0 0.10 0.20 fly with mass m to a fly with θ (rad) mass 2.5m?

τ (10–3 N ! m)

87 A flat uniform circular disk has a mass of 3.00 kg and a radius of 70.0 cm. It is suspended in a horizontal plane by a vertical wire attached to its center. If the disk is rotated 2.50 rad about the wire, a torque of 0.0600 N *m is required to maintain that orientation. Calculate (a) the rotational inertia of the disk about the wire, (b) the torsion constant, and (c) the angular frequency of this torsion pendulum when it is set oscillating.

t

95 An engineer has an odd-shaped 10 kg object and needs to find its rotational inertia about an axis through its –as center of mass.The object is supported on a wire stretched along the desired Figure 15-57 Problem 94. axis. The wire has a torsion constant k $ 0.50 N *m. If this torsion pendulum oscillates through 20 cycles in 50 s, what is the rotational inertia of the object?

97 A torsion pendulum consists of a metal disk with a wire running through its center and soldered in place. The wire is mounted vertically on clamps and pulled taut. Figure 15-58a gives the magnitude t of the torque

(a)

0.2

θ (rad)

86 The tip of one prong of a tuning fork undergoes SHM of frequency 1000 Hz and amplitude 0.40 mm. For this tip, what is the magnitude of the (a) maximum acceleration, (b) maximum velocity, (c) acceleration at tip displacement 0.20 mm, and (d) velocity at tip displacement 0.20 mm?

a (m/s2) as

0

0

ts

–0.2 (b)

Figure 15-58 Problem 97.

t (s)

442

CHAPTE R 15 OSCI LL ATIONS

needed to rotate the disk about its center (and thus twist the wire) versus the rotation angle u. The vertical axis scale is set by ts $ 4.0 ( 10!3 N *m. The disk is rotated to u $ 0.200 rad and then released. Figure 15-58b shows the resulting oscillation in terms of angular position u versus time t. The horizontal axis scale is set by ts $ 0.40 s. (a) What is the rotational inertia of the disk about its center? (b) What is the maximum angular speed du/dt of the disk? (Caution: Do not confuse the (constant) angular frequency of the SHM with the (varying) angular speed of the rotating disk, even though they usually have the same symbol v. Hint: The potential energy U of a torsion pendulum is equal to 21 ku 2, analogous to U $ 12 kx2 for a spring.) 98 When a 20 N can is hung from the bottom of a vertical spring, it causes the spring to stretch 20 cm. (a) What is the spring constant? (b) This spring is now placed horizontally on a frictionless table. One end of it is held fixed, and the other end is attached to a 5.0 N can.The can is then moved (stretching the spring) and released from rest. What is the period of the resulting oscillation? 99 For a simple pendulum, find the angular amplitude um at which the restoring torque required for simple harmonic motion deviates from the actual restoring torque by 1.0%. (See “Trigonometric Expansions” in Appendix E.) 100 In Fig. 15-59, a solid cylinder M attached to a horizontal spring (k $ k 3.00 N/m) rolls without slipping along a horizontal surface. If the system is released from rest when the Figure 15-59 Problem 100. spring is stretched by 0.250 m, find (a) the translational kinetic energy and (b) the rotational kinetic energy of the cylinder as it passes through the equilibrium position. (c) Show that under these conditions the cylinder’s center of mass executes simple harmonic motion with period 3M T $ 2p , A 2k

where M is the cylinder mass. (Hint: Find the time derivative of the total mechanical energy.) 101 SSM A 1.2 kg block sliding on a horizontal frictionless surface is attached to a horizontal spring with k $ 480 N/m. Let x be the displacement of the block from the position at which the spring is unstretched. At t $ 0 the block passes through x $ 0 with a speed of 5.2 m/s in the positive x direction. What are the (a) frequency and (b) amplitude of the block’s motion? (c) Write an expression for x as a function of time. 102 A simple harmonic oscillator consists of an 0.80 kg block attached to a spring (k $ 200 N/m). The block slides on a horizontal frictionless surface about the equilibrium point x $ 0 with a total mechanical energy of 4.0 J. (a) What is the amplitude of the oscillation? (b) How many oscillations does the block complete in 10 s? (c) What is the maximum kinetic energy attained by the block? (d) What is the speed of the block at x $ 0.15 m? 103 A block sliding on a horizontal frictionless surface is attached to a horizontal spring with a spring constant of 600 N/m. The block executes SHM about its equilibrium position with a period of 0.40 s and an amplitude of 0.20 m. As the block slides through its equilibrium position, a 0.50 kg putty wad is dropped

vertically onto the block. If the putty wad sticks to the block, determine (a) the new period of the motion and (b) the new amplitude of the motion. 104 A damped harmonic oscillator consists of a block (m $ 2.00 kg), a spring (k $ 10.0 N/m), and a damping force (F $ !bv). Initially, it oscillates with an amplitude of 25.0 cm; because of the damping, the amplitude falls to three-fourths of this initial value at the completion of four oscillations. (a) What is the value of b? (b) How much energy has been “lost” during these four oscillations? 105 A block weighing 10.0 N is attached to the lower end of a vertical spring (k $ 200.0 N/m), the other end of which is attached to a ceiling. The block oscillates vertically and has a kinetic energy of 2.00 J as it passes through the point at which the spring is unstretched. (a) What is the period of the oscillation? (b) Use the law of conservation of energy to determine the maximum distance the block moves both above and below the point at which the spring is unstretched. (These are not necessarily the same.) (c) What is the amplitude of the oscillation? (d) What is the maximum kinetic energy of the block as it oscillates? 106 A simple harmonic oscillator consists of a block attached to a spring with k $ 200 N/m. The block slides on a frictionless surface, with equilibrium point x $ 0 and amplitude 0.20 m. A graph of the block’s velocity v as a function of time t is shown in Fig. 15-60. The horizontal scale is set by ts $ 0.20 s. What are (a) the period of the SHM, (b) the block’s mass, (c) its displacement at t $ 0, (d) its acceleration at t $ 0.10 s, and (e) its maximum kinetic energy? v (m/s) 2π –2π

ts 0

t (s)

Figure 15-60 Problem 106. 107 The vibration frequencies of atoms in solids at normal temperatures are of the order of 1013 Hz. Imagine the atoms to be connected to one another by springs. Suppose that a single silver atom in a solid vibrates with this frequency and that all the other atoms are at rest. Compute the effective spring constant. One mole of silver (6.02 ( 10 23 atoms) has a mass of 108 g. 108 Figure 15-61 shows that if we hang a block on the end of a spring with spring constant k, the spring is stretched by distance h $ 2.0 cm. If we pull down on the block a short distance and then release it, it oscillates vertically with a certain frequency. What length must a simple pendulum have to swing with that frequency?

k k h

Figure 15-61 Problem 108.

PROB LE M S

109 The physical pendulum in Fig. 15-62 has two possible pivot points A and B. Point A has a fixed position but B is adjustable along the length of the pendulum as indicated by the scaling. When suspended from A, the pendulum has a period of T $ 1.80 s. The pendulum is then suspended from B, which is moved until the pendulum again has that period. What is the distance L between A and B?

A

L com

B 110 A common device for entertaining a toddler is a jump seat that hangs from the horizontal portion of a doorframe via elastic cords (Fig. 15-63). Assume that only one cord is on each Figure 15-62 Problem 109. side in spite of the more realistic arrangement shown. When a child is placed in the seat, they both descend by a distance ds as the cords stretch (treat them as springs). Then the seat is pulled down an extra distance dm and released, so that the child oscillates vertically, like a block on the end of a spring. Suppose you are the safety engineer for the manufacturer of the seat. You do not want the magnitude of the child’s acceleration to exceed 0.20g for fear of hurting the child’s neck. If dm $ 10 cm, what value of ds corresponds to that acceleration magnitude?

443

112 In Fig. 15-64, a 2500 kg demolition ball swings from the end of a crane. The length of the swinging segment of cable is 17 m. (a) Find the period of the swinging, assuming that the system can be treated as a simple pendulum. (b) Does the period depend on the ball’s mass? 113 The cenFigure 15-64 Problem 112. ter of oscillation of a physical pendulum has this interesting property: If an impulse (assumed horizontal and in the plane of oscillation) acts at the center of oscillation, no oscillations are felt at the point of support. Baseball players (and players of many other sports) know that unless the ball hits the bat at this point (called the “sweet spot” by athletes), the oscillations due to the impact will sting their hands. To prove this property, let the stick in Fig. 15-13a simulate a baseball bat. Suppose that a hori: zontal force F (due to impact with the ball) acts toward the right at P, the center of oscillation. The batter is assumed to hold the bat at O, the pivot point of the stick. (a) What acceleration does the : point O undergo as a result of F ? (b) What angular acceleration is : produced by F about the center of mass of the stick? (c) As a result of the angular acceleration in (b), what linear acceleration does point O undergo? (d) Considering the magnitudes and directions of the accelerations in (a) and (c), convince yourself that P is indeed the “sweet spot.” 114 A (hypothetical) large slingshot is stretched 2.30 m to launch a 170 g projectile with speed sufficient to escape from Earth (11.2 km/s). Assume the elastic bands of the slingshot obey Hooke’s law. (a) What is the spring constant of the device if all the elastic potential energy is converted to kinetic energy? (b) Assume that an average person can exert a force of 490 N. How many people are required to stretch the elastic bands?

Figure 15-63 Problem 110.

115 What is the length of a simple pendulum whose full swing from left to right and then back again takes 3.2 s?

111 A 2.0 kg block executes SHM while attached to a horizontal spring of spring constant 200 N/m. The maximum speed of the block as it slides on a horizontal frictionless surface is 3.0 m/s. What are (a) the amplitude of the block’s motion, (b) the magnitude of its maximum acceleration, and (c) the magnitude of its minimum acceleration? (d) How long does the block take to complete 7.0 cycles of its motion?

116 A 2.0 kg block is attached to the end of a spring with a spring constant of 350 N/m and forced to oscillate by an applied force F $ (15 N) sin(vdt), where vd $ 35 rad/s. The damping constant is b $ 15 kg/s. At t $ 0, the block is at rest with the spring at its rest length. (a) Use numerical integration to plot the displacement of the block for the first 1.0 s. Use the motion near the end of the 1.0 s interval to estimate the amplitude, period, and angular frequency. Repeat the calculation for (b) vd $ 1k/m and (c) vd $ 20 rad/s.

C

H

A

P

T

E

R

1

6

Waves—I 16-1 TRANSVERSE WAVES Learning Objectives After reading this module, you should be able to . . .

16.01 Identify the three main types of waves. 16.02 Distinguish between transverse waves and longitudinal waves. 16.03 Given a displacement function for a traverse wave, determine amplitude ym, angular wave number k, angular frequency v, phase constant f, and direction of travel, and calculate the phase kx ! vt " f and the displacement at any given time and position. 16.04 Given a displacement function for a traverse wave, calculate the time between two given displacements. 16.05 Sketch a graph of a transverse wave as a function of position, identifying amplitude ym, wavelength l, where the slope is greatest, where it is zero, and where the string elements have positive velocity, negative velocity, and zero velocity. 16.06 Given a graph of displacement versus time for a transverse wave, determine amplitude ym and period T.

16.07 Describe the effect on a transverse wave of changing phase constant f. 16.08 Apply the relation between the wave speed v, the distance traveled by the wave, and the time required for that travel. 16.09 Apply the relationships between wave speed v, angular frequency v, angular wave number k, wavelength l, period T, and frequency f. 16.10 Describe the motion of a string element as a transverse wave moves through its location, and identify when its transverse speed is zero and when it is maximum. 16.11 Calculate the transverse velocity u(t) of a string element as a transverse wave moves through its location. 16.12 Calculate the transverse acceleration a(t) of a string element as a transverse wave moves through its location. 16.13 Given a graph of displacement, transverse velocity, or transverse acceleration, determine the phase constant f.

Key Ideas ● Mechanical waves can exist only in material media and are

governed by Newton’s laws. Transverse mechanical waves, like those on a stretched string, are waves in which the particles of the medium oscillate perpendicular to the wave’s direction of travel. Waves in which the particles of the medium oscillate parallel to the wave’s direction of travel are longitudinal waves. ● A sinusoidal wave moving in the positive direction of an

x axis has the mathematical form y(x, t) # ym sin(kx $ vt), where ym is the amplitude (magnitude of the maximum displacement) of the wave, k is the angular wave number, v is the angular frequency, and kx $ vt is the phase. The wavelength l is related to k by k#

444

2p . l

● The period T and frequency f of the wave are related to v by

1 v #f# . 2p T ● The wave speed v (the speed of the wave along the string) is

related to these other parameters by v#

v l # # lf. k T

● Any function of the form

y(x, t) # h(kx ! vt) can represent a traveling wave with a wave speed as given above and a wave shape given by the mathematical form of h. The plus sign denotes a wave traveling in the negative direction of the x axis, and the minus sign a wave traveling in the positive direction.

16-1 TRANSVE RSE WAVES

445

What Is Physics? One of the primary subjects of physics is waves. To see how important waves are in the modern world, just consider the music industry. Every piece of music you hear, from some retro-punk band playing in a campus dive to the most eloquent concerto playing on the web, depends on performers producing waves and your detecting those waves. In between production and detection, the information carried by the waves might need to be transmitted (as in a live performance on the web) or recorded and then reproduced (as with CDs, DVDs, or the other devices currently being developed in engineering labs worldwide). The financial importance of controlling music waves is staggering, and the rewards to engineers who develop new control techniques can be rich. This chapter focuses on waves traveling along a stretched string, such as on a guitar. The next chapter focuses on sound waves, such as those produced by a guitar string being played. Before we do all this, though, our first job is to classify the countless waves of the everyday world into basic types.

Types of Waves Waves are of three main types: 1. Mechanical waves. These waves are most familiar because we encounter them almost constantly; common examples include water waves, sound waves, and seismic waves. All these waves have two central features: They are governed by Newton’s laws, and they can exist only within a material medium, such as water, air, and rock. 2. Electromagnetic waves. These waves are less familiar, but you use them constantly; common examples include visible and ultraviolet light, radio and television waves, microwaves, x rays, and radar waves. These waves require no material medium to exist. Light waves from stars, for example, travel through the vacuum of space to reach us. All electromagnetic waves travel through a vacuum at the same speed c # 299 792 458 m/s. 3. Matter waves. Although these waves are commonly used in modern technology, they are probably very unfamiliar to you. These waves are associated with electrons, protons, and other fundamental particles, and even atoms and molecules. Because we commonly think of these particles as constituting matter, such waves are called matter waves.

y Pulse

v

x

(a) y

Sinusoidal wave v

Much of what we discuss in this chapter applies to waves of all kinds. However, for specific examples we shall refer to mechanical waves.

x

Transverse and Longitudinal Waves A wave sent along a stretched, taut string is the simplest mechanical wave. If you give one end of a stretched string a single up-and-down jerk, a wave in the form of a single pulse travels along the string. This pulse and its motion can occur because the string is under tension. When you pull your end of the string upward, it begins to pull upward on the adjacent section of the string via tension between the two sections. As the adjacent section moves upward, it begins to pull the next section upward, and so on. Meanwhile, you have pulled down on your end of the string. As each section moves upward in turn, it begins to be pulled back downward by neighboring sections that are already on the way down. The net result is that a distortion in the string’s shape (a pulse, as in Fig. 16-1a) moves along the string at some velocity v: .

(b)

Figure 16-1 (a) A single pulse is sent along a stretched string. A typical string element (marked with a dot) moves up once and then down as the pulse passes. The element’s motion is perpendicular to the wave’s direction of travel, so the pulse is a transverse wave. (b) A sinusoidal wave is sent along the string. A typical string element moves up and down continuously as the wave passes. This too is a transverse wave.

446

CHAPTE R 16 WAVES—I v

Air

Figure 16-2 A sound wave is set up in an airfilled pipe by moving a piston back and forth. Because the oscillations of an element of the air (represented by the dot) are parallel to the direction in which the wave travels, the wave is a longitudinal wave.

If you move your hand up and down in continuous simple harmonic motion, a v . Because the motion of your continuous wave travels along the string at velocity : hand is a sinusoidal function of time, the wave has a sinusoidal shape at any given instant, as in Fig. 16-1b; that is, the wave has the shape of a sine curve or a cosine curve. We consider here only an “ideal” string, in which no friction-like forces within the string cause the wave to die out as it travels along the string. In addition, we assume that the string is so long that we need not consider a wave rebounding from the far end. One way to study the waves of Fig. 16-1 is to monitor the wave forms (shapes of the waves) as they move to the right. Alternatively, we could monitor the motion of an element of the string as the element oscillates up and down while a wave passes through it. We would find that the displacement of every such oscillating string element is perpendicular to the direction of travel of the wave, as indicated in Fig. 16-1b. This motion is said to be transverse, and the wave is said to be a transverse wave. Longitudinal Waves. Figure 16-2 shows how a sound wave can be produced by a piston in a long, air-filled pipe. If you suddenly move the piston rightward and then leftward, you can send a pulse of sound along the pipe. The rightward motion of the piston moves the elements of air next to it rightward, changing the air pressure there. The increased air pressure then pushes rightward on the elements of air somewhat farther along the pipe. Moving the piston leftward reduces the air pressure next to it. As a result, first the elements nearest the piston and then farther elements move leftward. Thus, the motion of the air and the change in air pressure travel rightward along the pipe as a pulse. If you push and pull on the piston in simple harmonic motion, as is being done in Fig. 16-2, a sinusoidal wave travels along the pipe. Because the motion of the elements of air is parallel to the direction of the wave’s travel, the motion is said to be longitudinal, and the wave is said to be a longitudinal wave. In this chapter we focus on transverse waves, and string waves in particular; in Chapter 17 we focus on longitudinal waves, and sound waves in particular. Both a transverse wave and a longitudinal wave are said to be traveling waves because they both travel from one point to another, as from one end of the string to the other end in Fig. 16-1 and from one end of the pipe to the other end in Fig. 16-2. Note that it is the wave that moves from end to end, not the material (string or air) through which the wave moves.

Wavelength and Frequency To completely describe a wave on a string (and the motion of any element along its length), we need a function that gives the shape of the wave. This means that we need a relation in the form y # h(x, t), (16-1) in which y is the transverse displacement of any string element as a function h of the time t and the position x of the element along the string. In general, a sinusoidal shape like the wave in Fig. 16-1b can be described with h being either a sine or cosine function; both give the same general shape for the wave. In this chapter we use the sine function. Sinusoidal Function. Imagine a sinusoidal wave like that of Fig. 16-1b traveling in the positive direction of an x axis. As the wave sweeps through succeeding elements (that is, very short sections) of the string, the elements oscillate parallel to the y axis.At time t, the displacement y of the element located at position x is given by y(x, t) # ym sin(kx $ vt).

(16-2)

Because this equation is written in terms of position x, it can be used to find the displacements of all the elements of the string as a function of time. Thus, it can tell us the shape of the wave at any given time.

16-1 TRANSVE RSE WAVES

The names of the quantities in Eq. 16-2 are displayed in Fig. 16-3 and defined next. Before we discuss them, however, let us examine Fig. 16-4, which shows five “snapshots” of a sinusoidal wave traveling in the positive direction of an x axis. The movement of the wave is indicated by the rightward progress of the short arrow pointing to a high point of the wave. From snapshot to snapshot, the short arrow moves to the right with the wave shape, but the string moves only parallel to the y axis. To see that, let us follow the motion of the red-dyed string element at x # 0. In the first snapshot (Fig. 16-4a), this element is at displacement y # 0. In the next snapshot, it is at its extreme downward displacement because a valley (or extreme low point) of the wave is passing through it. It then moves back up through y # 0. In the fourth snapshot, it is at its extreme upward displacement because a peak (or extreme high point) of the wave is passing through it. In the fifth snapshot, it is again at y # 0, having completed one full oscillation.

Amplitude Displacement

447

Oscillating term Phase

y(x,t) = ym sin (kx – ω t) Angular Time wave number Position Angular frequency

Figure 16-3 The names of the quantities in Eq. 16-2, for a transverse sinusoidal wave.

Amplitude and Phase The amplitude ym of a wave, such as that in Fig. 16-4 , is the magnitude of the maximum displacement of the elements from their equilibrium positions as the wave passes through them. (The subscript m stands for maximum.) Because ym is a magnitude, it is always a positive quantity, even if it is measured downward instead of upward as drawn in Fig. 16-4a. The phase of the wave is the argument kx $ vt of the sine in Eq. 16-2. As the wave sweeps through a string element at a particular position x, the phase changes linearly with time t. This means that the sine also changes, oscillating between "1 and $1. Its extreme positive value ("1) corresponds to a peak of the wave moving through the element; at that instant the value of y at position x is ym. Its extreme negative value ($1) corresponds to a valley of the wave moving through the element; at that instant the value of y at position x is $ym. Thus, the sine function and the time-dependent phase of a wave correspond to the oscillation of a string element, and the amplitude of the wave determines the extremes of the element’s displacement. Caution: When evaluating the phase, rounding off the numbers before you evaluate the sine function can throw of the calculation considerably.

Watch this spot in this series of snapshots. y ym

x1

(a)

x λ

y x

Wavelength and Angular Wave Number The wavelength l of a wave is the distance (parallel to the direction of the wave’s travel) between repetitions of the shape of the wave (or wave shape). A typical wavelength is marked in Fig. 16-4a, which is a snapshot of the wave at time t # 0. At that time, Eq. 16-2 gives, for the description of the wave shape, y(x, 0) # ym sin kx.

2p l

(angular wave number).

x (c) y x

(16-4)

A sine function begins to repeat itself when its angle (or argument) is increased by 2p rad, so in Eq. 16-4 we must have kl # 2p, or k#

y

(16-3)

By definition, the displacement y is the same at both ends of this wavelength — that is, at x # x1 and x # x1 " l. Thus, by Eq. 16-3, ym sin kx1 # ym sin k(x1 " l) # ym sin(kx1 " kl).

(b)

(d) y x

(16-5)

We call k the angular wave number of the wave; its SI unit is the radian per meter, or the inverse meter. (Note that the symbol k here does not represent a spring constant as previously.) Notice that the wave in Fig. 16-4 moves to the right by 14l from one snapshot to the next. Thus, by the fifth snapshot, it has moved to the right by 1l.

(e)

Figure 16-4 Five “snapshots” of a string wave traveling in the positive direction of an x axis. The amplitude ym is indicated. A typical wavelength l, measured from an arbitrary position x1, is also indicated.

448

CHAPTE R 16 WAVES—I

Period, Angular Frequency, and Frequency

This is a graph, not a snapshot.

y t1

ym

t

Figure 16-5 shows a graph of the displacement y of Eq. 16-2 versus time t at a certain position along the string, taken to be x # 0. If you were to monitor the string, you would see that the single element of the string at that position moves up and down in simple harmonic motion given by Eq. 16-2 with x # 0: y(0, t) # ym sin($vt) # $ym sin vt

T

Figure 16-5 A graph of the displacement of the string element at x # 0 as a function of time, as the sinusoidal wave of Fig. 16-4 passes through the element. The amplitude ym is indicated. A typical period T, measured from an arbitrary time t1, is also indicated.

(x # 0).

(16-6)

Here we have made use of the fact that sin($a) # $sin a, where a is any angle. Figure 16-5 is a graph of this equation, with displacement plotted versus time; it does not show the shape of the wave. (Figure 16-4 shows the shape and is a picture of reality; Fig. 16-5 is a graph and thus an abstraction.) We define the period of oscillation T of a wave to be the time any string element takes to move through one full oscillation. A typical period is marked on the graph of Fig. 16-5. Applying Eq. 16-6 to both ends of this time interval and equating the results yield $ym sin vt1 # $ym sin v(t1 " T ) # $ym sin(vt1 " vT ).

(16-7)

This can be true only if vT # 2p, or if v#

2p T

(16-8)

(angular frequency).

We call v the angular frequency of the wave; its SI unit is the radian per second. Look back at the five snapshots of a traveling wave in Fig. 16-4. The time between snapshots is 14 T. Thus, by the fifth snapshot, every string element has made one full oscillation. The frequency f of a wave is defined as 1/T and is related to the angular frequency v by f#

y

The effect of the phase constant φ is to shift the wave.

x

(a)

1 v # T 2p

(16-9)

(frequency).

Like the frequency of simple harmonic motion in Chapter 15, this frequency f is a number of oscillations per unit time — here, the number made by a string element as the wave moves through it. As in Chapter 15, f is usually measured in hertz or its multiples, such as kilohertz.

Checkpoint 1

y

The figure is a composite of three snapshots, each of a wave traveling along a particular string. The phases for the waves are given by (a) 2x $ 4t, (b) 4x $ 8t, and (c) 8x $ 16t. Which phase corresponds to which wave in the figure?

1

2

3 x

y x

(b)

Figure 16-6 A sinusoidal traveling wave at t # 0 with a phase constant f of (a) 0 and (b) p/5 rad.

Phase Constant When a sinusoidal traveling wave is given by the wave function of Eq. 16-2, the wave near x # 0 looks like Fig. 16-6a when t # 0. Note that at x # 0, the displacement is y # 0 and the slope is at its maximum positive value. We can generalize Eq. 16-2 by inserting a phase constant f in the wave function: y # ym sin(kx $ vt " f).

(16-10)

16-1 TRANSVE RSE WAVES

449

The value of f can be chosen so that the function gives some other displacement and slope at x # 0 when t # 0. For example, a choice of f # "p/5 rad gives the displacement and slope shown in Fig. 16-6b when t # 0. The wave is still sinusoidal with the same values of ym, k, and v, but it is now shifted from what you see in Fig. 16-6a (where f # 0). Note also the direction of the shift. A positive value of f shifts the curve in the negative direction of the x axis; a negative value shifts the curve in the positive direction.

The Speed of a Traveling Wave

y

Figure 16-7 shows two snapshots of the wave of Eq. 16-2, taken a small time interval %t apart. The wave is traveling in the positive direction of x (to the right in Fig. 16-7), the entire wave pattern moving a distance %x in that direction during the interval %t. The ratio %x/%t (or, in the differential limit, dx/dt) is the wave speed v. How can we find its value? As the wave in Fig. 16-7 moves, each point of the moving wave form, such as point A marked on a peak, retains its displacement y. (Points on the string do not retain their displacement, but points on the wave form do.) If point A retains its displacement as it moves, the phase in Eq. 16-2 giving it that displacement must remain a constant:

A

kx $ vt # a constant.

(16-11)

Note that although this argument is constant, both x and t are changing. In fact, as t increases, x must also, to keep the argument constant. This confirms that the wave pattern is moving in the positive direction of x. To find the wave speed v, we take the derivative of Eq. 16-11, getting dx $v#0 dt v dx #v# . dt k

k or

(16-12)

Using Eq. 16-5 (k # 2p/l) and Eq. 16-8 (v # 2p/T ), we can rewrite the wave speed as v#

l v # # lf k T

(wave speed).

(16-13)

The equation v # l/T tells us that the wave speed is one wavelength per period; the wave moves a distance of one wavelength in one period of oscillation. Equation 16-2 describes a wave moving in the positive direction of x. We can find the equation of a wave traveling in the opposite direction by replacing t in Eq. 16-2 with $t. This corresponds to the condition kx " vt # a constant,

(16-14)

which (compare Eq. 16-11) requires that x decrease with time. Thus, a wave traveling in the negative direction of x is described by the equation y(x, t) # ym sin(kx " vt).

(16-15)

If you analyze the wave of Eq. 16-15 as we have just done for the wave of Eq. 16-2, you will find for its velocity dx v (16-16) #$ . dt k The minus sign (compare Eq. 16-12) verifies that the wave is indeed moving in the negative direction of x and justifies our switching the sign of the time variable.

∆x v

x Wave at t = ∆t Wave at t = 0

Figure 16-7 Two snapshots of the wave of Fig. 16-4, at time t # 0 and then at time t # %t. As the wave moves to the right at velocity v:, the entire curve shifts a distance %x during %t. Point A “rides” with the wave form, but the string elements move only up and down.

450

CHAPTE R 16 WAVES—I

Consider now a wave of arbitrary shape, given by (16-17)

y(x, t) # h(kx ! vt),

where h represents any function, the sine function being one possibility. Our previous analysis shows that all waves in which the variables x and t enter into the combination kx ! vt are traveling waves. Furthermore, all traveling waves must be of the form of Eq. 16-17. Thus, y(x, t) # 1ax " bt represents a possible (though perhaps physically a little bizarre) traveling wave. The function y(x, t) # sin(ax2 $ bt), on the other hand, does not represent a traveling wave.

Checkpoint 2 Here are the equations of three waves: (1) y(x, t) # 2 sin(4x $ 2t), (2) y(x, t) # sin(3x $ 4t), (3) y(x, t) # 2 sin(3x $ 3t). Rank the waves according to their (a) wave speed and (b) maximum speed perpendicular to the wave’s direction of travel (the transverse speed), greatest first.

Sample Problem 16.01 Determining the quantities in an equation for a transverse wave A transverse wave traveling along an x axis has the form given by y # ym sin(kx ! vt " f). (16-18) Figure 16-8a gives the displacements of string elements as a function of x, all at time t # 0. Figure 16-8b gives the displacements of the element at x # 0 as a function of t. Find the values of the quantities shown in Eq. 16-18, including the correct choice of sign. KEY IDEAS (1) Figure 16-8a is effectively a snapshot of reality (something that we can see), showing us motion spread out over the x axis. From it we can determine the wavelength l of the wave along that axis, and then we can find the angular wave number k (# 2p/l) in Eq. 16-18. (2) Figure 16-8b is an ab-

straction, showing us motion spread out over time. From it we can determine the period T of the string element in its SHM and thus also of the wave itself. From T we can then find angular frequency v (# 2p/T) in Eq. 16-18. (3) The phase constant f is set by the displacement of the string at x # 0 and t # 0. Amplitude: From either Fig. 16-8a or 16-8b we see that the maximum displacement is 3.0 mm. Thus, the wave’s amplitude xm # 3.0 mm. Wavelength: In Fig. 16-8a, the wavelength l is the distance along the x axis between repetitions in the pattern. The easiest way to measure l is to find the distance from one crossing point to the next crossing point where the string has the same slope. Visually we can roughly measure that distance with the scale on the axis. Instead, we can lay the edge of a

y (mm)

y (mm)

2

2

–8

–4

0

4

8

x (mm)

–20

0

10

20

t (ms)

–2

–2 (a)

–10

(b)

–3

Figure 16-8 (a) A snapshot of the displacement y versus position x along a string, at time t # 0. (b) A graph of displacement y versus time t for the string element at x # 0.

16-1 TRANSVE RSE WAVES

paper sheet on the graph, mark those crossing points, slide the sheet to align the left-hand mark with the origin, and then read off the location of the right-hand mark. Either way we find l # 10 mm. From Eq. 16-5, we then have 2p 2p # # 200p rad/m. l 0.010 m

k#

Period: The period T is the time interval that a string element’s SHM takes to begin repeating itself. In Fig. 16-8b, T is the distance along the t axis from one crossing point to the next crossing point where the plot has the same slope. Measuring the distance visually or with the aid of a sheet of paper, we find T # 20 ms. From Eq. 16-8, we then have v#

451

crease (mentally slide the curve slightly rightward). If, instead, the wave is moving leftward, then just after the snapshot, the depth at x # 0 should decrease. Now let’s check the graph in Fig. 16-8b. It tells us that just after t # 0, the depth increases.Thus, the wave is moving rightward, in the positive direction of x, and we choose the minus sign in Eq. 16-18. Phase constant: The value of f is set by the conditions at x # 0 at the instant t # 0. From either figure we see that at that location and time, y # $2.0 mm. Substituting these three values and also ym # 3.0 mm into Eq. 16-18 gives us $2.0 mm # (3.0 mm) sin(0 " 0 " f) f # sin$1($23) # $0.73 rad.

or

Note that this is consistent with the rule that on a plot of y versus x, a negative phase constant shifts the normal sine function rightward, which is what we see in Fig. 16-8a.

2p 2p # # 100p rad/s. T 0.020 s

Direction of travel: To find the direction, we apply a bit of reasoning to the figures. In the snapshot at t # 0 given in Fig. 16-8a, note that if the wave is moving rightward, then just after the snapshot, the depth of the wave at x # 0 should in-

Equation: Now we can fill out Eq. 16-18: y # (3.0 mm) sin(200px $ l00pt $ 0.73 rad),

(Answer)

with x in meters and t in seconds.

Sample Problem 16.02 Transverse velocity and transverse acceleration of a string element Next, substituting numerical values but suppressing the units, which are SI, we write

A wave traveling along a string is described by y(x, t) = (0.00327 m) sin(72.1x $ 2.72t), in which the numerical constants are in SI units (72.1 rad/m and 2.72 rad/s).

u # ($2.72)(0.00327) cos[(72.1)(0.225) $ (2.72)(18.9)] # 0.00720 m/s # 7.20 mm/s.

(Answer)

(a) What is the transverse velocity u of the string element at x # 22.5 cm at time t # 18.9 s? (This velocity, which is associated with the transverse oscillation of a string element, is parallel to the y axis. Don’t confuse it with v, the constant velocity at which the wave form moves along the x axis.)

Thus, at t # 18.9 s our string element is moving in the positive direction of y with a speed of 7.20 mm/s. (Caution: In evaluating the cosine function, we keep all the significant figures in the argument or the calculation can be off considerably. For example, round off the numbers to two significant figures and then see what you get for u.)

KEY IDEAS

(b) What is the transverse acceleration ay of our string element at t # 18.9 s?

The transverse velocity u is the rate at which the displacement y of the element is changing. In general, that displacement is given by

KEY IDEA

y(x, t) = ym sin(kx $ vt).

(16-19)

For an element at a certain location x, we find the rate of change of y by taking the derivative of Eq. 16-19 with respect to t while treating x as a constant. A derivative taken while one (or more) of the variables is treated as a constant is called a partial derivative and is represented by a symbol such as &/&t rather than d/dt. Calculations: Here we have u#

&y # $vym cos(kx $ vt). &t

(16-20)

The transverse acceleration ay is the rate at which the element’s transverse velocity is changing. Calculations: From Eq. 16-20, again treating x as a constant but allowing t to vary, we find ay #

&u # $v2ym sin (kx $ vt). &t

(16-21)

Substituting numerical values but suppressing the units, which are SI, we have ay 5 2(2.72)2(0.00327) sin[(72.1)(0.225) $ (2.72)(18.9)] # $0.0142 m/s2 # $14.2 mm/s2.

(Answer)

452

CHAPTE R 16 WAVES—I

From part (a) we learn that at t # 18.9 s our string element is moving in the positive direction of y, and here we learn that

it is slowing because its acceleration is in the opposite direction of u.

Additional examples, video, and practice available at WileyPLUS

16-2 WAVE SPEED ON A STRETCHED STRING Learning Objectives After reading this module, you should be able to . . .

16.14 Calculate the linear density m of a uniform string in terms of the total mass and total length.

l6.15 Apply the relationship between wave speed v, tension t, and linear density m.

Key Ideas ● The speed of a wave on a stretched string is set by properties of the string, not properties of the wave such as frequency or amplitude.

● The speed of a wave on a string with tension t and linear

density m is v#

t . m A

Wave Speed on a Stretched String The speed of a wave is related to the wave’s wavelength and frequency by Eq. 16-13, but it is set by the properties of the medium. If a wave is to travel through a medium such as water, air, steel, or a stretched string, it must cause the particles of that medium to oscillate as it passes, which requires both mass (for kinetic energy) and elasticity (for potential energy). Thus, the mass and elasticity determine how fast the wave can travel. Here, we find that dependency in two ways.

Dimensional Analysis In dimensional analysis we carefully examine the dimensions of all the physical quantities that enter into a given situation to determine the quantities they produce. In this case, we examine mass and elasticity to find a speed v, which has the dimension of length divided by time, or LT $1. For the mass, we use the mass of a string element, which is the mass m of the string divided by the length l of the string. We call this ratio the linear density m of the string. Thus, m # m/l, its dimension being mass divided by length, ML$1. You cannot send a wave along a string unless the string is under tension, which means that it has been stretched and pulled taut by forces at its two ends. The tension t in the string is equal to the common magnitude of those two forces. As a wave travels along the string, it displaces elements of the string by causing additional stretching, with adjacent sections of string pulling on each other because of the tension. Thus, we can associate the tension in the string with the stretching (elasticity) of the string. The tension and the stretching forces it produces have the dimension of a force — namely, MLT $2 (from F # ma). We need to combine m (dimension ML$1) and t (dimension MLT $2) to get v (dimension LT $1). A little juggling of various combinations suggests v#C

t , Am

(16-22)

in which C is a dimensionless constant that cannot be determined with dimensional analysis. In our second approach to determining wave speed, you will see that Eq. 16-22 is indeed correct and that C # 1.

16-2 WAVE SPE E D ON A STR ETCH E D STR I NG

Derivation from Newton’s Second Law

∆l

Instead of the sinusoidal wave of Fig. 16-1b, let us consider a single symmetrical pulse such as that of Fig. 16-9, moving from left to right along a string with speed v. For convenience, we choose a reference frame in which the pulse remains stationary; that is, we run along with the pulse, keeping it constantly in view. In this frame, the string appears to move past us, from right to left in Fig. 16-9, with speed v. Consider a small string element of length %l within the pulse, an element that forms an arc of a circle of radius R and subtending an angle 2u at the center of that circle. A force (: with a magnitude equal to the tension in the string pulls tangentially on this element at each end. The horizontal components of these : forces cancel, but the vertical components add to form a radial restoring force F . In magnitude, %l (force), (16-23) R where we have approximated sin u as u for the small angles u in Fig. 16-9. From that figure, we have also used 2u # %l/R. The mass of the element is given by F # 2(t sin u) % t(2u) # t

%m # m %l

(mass),

(16-24)

where m is the string’s linear density. At the moment shown in Fig. 16-9, the string element %l is moving in an arc of a circle. Thus, it has a centripetal acceleration toward the center of that circle, given by v2 (acceleration). (16-25) R Equations 16-23, 16-24, and 16-25 contain the elements of Newton’s second law. Combining them in the form a#

force # mass ' acceleration t %l v2 # (m %l) . R R Solving this equation for the speed v yields gives

v#

t m A

453

(speed),

(16-26)

in exact agreement with Eq. 16-22 if the constant C in that equation is given the value unity. Equation 16-26 gives the speed of the pulse in Fig. 16-9 and the speed of any other wave on the same string under the same tension. Equation 16-26 tells us: The speed of a wave along a stretched ideal string depends only on the tension and linear density of the string and not on the frequency of the wave.

The frequency of the wave is fixed entirely by whatever generates the wave (for example, the person in Fig. 16-1b). The wavelength of the wave is then fixed by Eq. 16-13 in the form l # v/f.

Checkpoint 3 You send a traveling wave along a particular string by oscillating one end. If you increase the frequency of the oscillations, do (a) the speed of the wave and (b) the wavelength of the wave increase, decrease, or remain the same? If, instead, you increase the tension in the string, do (c) the speed of the wave and (d) the wavelength of the wave increase, decrease, or remain the same?

τ

τ θ

θ v

R

O

Figure 16-9 A symmetrical pulse, viewed from a reference frame in which the pulse is stationary and the string appears to move right to left with speed v. We find speed v by applying Newton’s second law to a string element of length %l, located at the top of the pulse.

454

CHAPTE R 16 WAVES—I

16-3 ENERGY AND POWER OF A WAVE TRAVELING ALONG A STRING Learning Objective After reading this module, you should be able to . . .

16.16 Calculate the average rate at which energy is transported by a transverse wave.

Key Idea ● The average power of, or average rate at which energy is transmitted by, a sinusoidal wave on a stretched string is

given by

Pavg # 12mvv2y2m.

Energy and Power of a Wave Traveling Along a String When we set up a wave on a stretched string, we provide energy for the motion of the string. As the wave moves away from us, it transports that energy as both kinetic energy and elastic potential energy. Let us consider each form in turn.

Kinetic Energy A string element of mass dm, oscillating transversely in simple harmonic motion as the wave passes through it, has kinetic energy associated with its transverse velocity u:. When the element is rushing through its y # 0 position (element b in Fig. 16-10), its transverse velocity — and thus its kinetic energy — is a maximum. When the element is at its extreme position y # ym (as is element a), its transverse velocity — and thus its kinetic energy — is zero.

y

λ ym

Elastic Potential Energy

a

v

b x

0

dx

dx

Figure 16-10 A snapshot of a traveling wave on a string at time t # 0. String element a is at displacement y # ym, and string element b is at displacement y # 0. The kinetic energy of the string element at each position depends on the transverse velocity of the element. The potential energy depends on the amount by which the string element is stretched as the wave passes through it.

To send a sinusoidal wave along a previously straight string, the wave must necessarily stretch the string. As a string element of length dx oscillates transversely, its length must increase and decrease in a periodic way if the string element is to fit the sinusoidal wave form. Elastic potential energy is associatzed with these length changes, just as for a spring. When the string element is at its y # ym position (element a in Fig. 16-10), its length has its normal undisturbed value dx, so its elastic potential energy is zero. However, when the element is rushing through its y # 0 position, it has maximum stretch and thus maximum elastic potential energy.

Energy Transport The oscillating string element thus has both its maximum kinetic energy and its maximum elastic potential energy at y # 0. In the snapshot of Fig. 16-10, the regions of the string at maximum displacement have no energy, and the regions at zero displacement have maximum energy. As the wave travels along the string, forces due to the tension in the string continuously do work to transfer energy from regions with energy to regions with no energy. As in Fig. 16-1b, let’s set up a wave on a string stretched along a horizontal x axis such that Eq. 16-2 applies. As we oscillate one end of the string, we continuously provide energy for the motion and stretching of the string — as the string sections oscillate perpendicularly to the x axis, they have kinetic energy and elastic potential energy. As the wave moves into sections that were previously at rest, energy is transferred into those new sections. Thus, we say that the wave transports the energy along the string.

The Rate of Energy Transmission The kinetic energy dK associated with a string element of mass dm is given by dK # 12 dm u2,

(16-27)

455

16-3 E N E RGY AN D POWE R OF A WAVE TRAVE LI NG ALONG A STR I NG

where u is the transverse speed of the oscillating string element. To find u, we differentiate Eq. 16-2 with respect to time while holding x constant: ≠y # $vym cos(kx $ vt). ≠t Using this relation and putting dm # m dx, we rewrite Eq. 16-27 as u#

dK # 12(m dx)($vym)2 cos2(kx $ vt).

(16-28)

(16-29)

Dividing Eq. 16-29 by dt gives the rate at which kinetic energy passes through a string element, and thus the rate at which kinetic energy is carried along by the wave. The dx/dt that then appears on the right of Eq. 16-29 is the wave speed v, so dK # 12)v*2y2m cos2(kx $ * t). dt

(16-30)

The average rate at which kinetic energy is transported is

# dK dt $

avg

# 12mvv2y2m [cos2(kx $ vt)]avg # 14mvv2y2m.

(16-31)

Here we have taken the average over an integer number of wavelengths and have used the fact that the average value of the square of a cosine function over an integer number of periods is 21. Elastic potential energy is also carried along with the wave, and at the same average rate given by Eq. 16-31. Although we shall not examine the proof, you should recall that, in an oscillating system such as a pendulum or a spring – block system, the average kinetic energy and the average potential energy are equal. The average power, which is the average rate at which energy of both kinds is transmitted by the wave, is then Pavg # 2 or, from Eq. 16-31, Pavg # 12mvv2y2m

# dK dt $

(16-32)

avg

(average power).

(16-33)

The factors m and v in this equation depend on the material and tension of the string. The factors v and ym depend on the process that generates the wave. The dependence of the average power of a wave on the square of its amplitude and also on the square of its angular frequency is a general result, true for waves of all types. Sample Problem 16.03 Average power of a transverse wave A string has linear density m # 525 g/m and is under tension t # 45 N. We send a sinusoidal wave with frequency f # 120 Hz and amplitude ym # 8.5 mm along the string. At what average rate does the wave transport energy? KEY IDEA The average rate of energy transport is the average power Pavg as given by Eq. 16-33. Calculations: To use Eq. 16-33, we first must calculate

angular frequency v and wave speed v. From Eq. 16-9, v # 2pf # (2p)(120 Hz) # 754 rad/s. From Eq. 16-26 we have v#

( 45 N # # 9.26 m/s. A) A 0.525 kg/m

Equation 16-33 then yields Pavg # 12mvv2y2m

# (12)(0.525 kg/m)(9.26 m/s)(754 rad/s)2(0.0085 m)2 % 100 W.

Additional examples, video, and practice available at WileyPLUS

(Answer)

456

CHAPTE R 16 WAVES—I

16-4 THE WAVE EQUATION Learning Objective After reading this module, you should be able to . . .

16.17 For the equation giving a string-element displacement as a function of position x and time t, apply the relationship

between the second derivative with respect to x and the second derivative with respect to t.

Key Idea ● The general differential equation that governs the travel of waves

of all types is

2

2

&y 1 &y # 2 . 2 &x v &t2

Here the waves travel along an x axis and oscillate parallel to the y axis, and they move with speed v, in either the positive x direction or the negative x direction.

The Wave Equation As a wave passes through any element on a stretched string, the element moves perpendicularly to the wave’s direction of travel (we are dealing with a transverse wave). By applying Newton’s second law to the element’s motion, we can derive a general differential equation, called the wave equation, that governs the travel of waves of any type. Figure 16-11a shows a snapshot of a string element of mass dm and length ! as a wave travels along a string of linear density m that is stretched along a horizontal x axis. Let us assume that the wave amplitude is small so that the element : can be tilted only slightly from the x axis as the wave passes. The force F 2 on the right end of the element has a magnitude equal to tension t in the string and is : directed slightly upward. The force F 1 on the left end of the element also has a magnitude equal to the tension t but is directed slightly downward. Because of the slight curvature of the element, these two forces are not simply in opposite direction so that they cancel. Instead, they combine to produce a net force that causes the element to have an upward acceleration ay. Newton’s second law written for y components (Fnet,y # may) gives us F2y $ F1y # dm ay.

(16-34)

Let’s analyze this equation in parts, first the mass dm, then the acceleration component ay, then the individual force components F2y and F1y, and then finally the net force that is on the left side of Eq. 16-34. Mass. The element’s mass dm can be written in terms of the string’s linear density m and the element’s length ! as dm # m!. Because the element can have only a slight tilt, ! % dx (Fig. 16-11a) and we have the approximation (16-35)

dm # m dx. y y

ay

F2

F2 F1 (a)

! dx

Tangent line F2y

F2x x

(b)

x

Figure 16-11 (a) A string element as a sinusoidal transverse wave travels on a stretched string. : : Forces F 1 and F 2 act at the left and right ends, producing acceleration a: having a vertical component ay. (b) The force at the element’s right end is directed along a tangent to the element’s right side.

16-4 TH E WAVE EQUATION

Acceleration. The acceleration ay in Eq. 16-34 is the second derivative of the displacement y with respect to time: d2y ay # 2 . (16-36) dt :

Forces. Figure 16-11b shows that F 2 is tangent to the string at the right end of the string element. Thus we can relate the components of the force to the string slope S2 at the right end as F 2y # S2 . (16-37) F 2x We can also relate the components to the magnitude F2 (# t) with

or

2 2 " F 2y F2 # 2F 2x

2 2 " F 2y . t # 2F 2x

(16-38)

However, because we assume that the element is only slightly tilted, F2y + F2x and therefore we can rewrite Eq. 16-38 as t # F2x.

(16-39)

Substituting this into Eq. 16-37 and solving for F2y yield F2y # tS2.

(16-40)

Similar analysis at the left end of the string element gives us F1y # tS1.

(16-41)

Net Force. We can now substitute Eqs. 16-35, 16-36, 16-40, and 16-41 into Eq. 16-34 to write d 2y tS2 $ tS1 # (m dx) 2 , dt or

m d 2y S2 $ S1 # . dx t dt2

(16-42)

Because the string element is short, slopes S2 and S1 differ by only a differential amount dS, where S is the slope at any point: S#

dy . dx

(16-43)

First replacing S2 $ S1 in Eq. 16-42 with dS and then using Eq. 16-43 to substitute dy/dx for S, we find dS m d2y , # dx t dt2 d(dy/dx) m d2y , # dx t dt2 and

≠ 2y m ≠2y . # ≠x2 t ≠t2

(16-44)

In the last step, we switched to the notation of partial derivatives because on the left we differentiate only with respect to x and on the right we differentiate only with respect to t. Finally, substituting from Eq. 16-26 (v # 1t /m), we find ≠2y 1 ≠2y # 2 2 ≠x v ≠t2

(wave equation).

(16-45)

This is the general differential equation that governs the travel of waves of all types.

457

458

CHAPTE R 16 WAVES—I

16-5 INTERFERENCE OF WAVES Learning Objectives After reading this module, you should be able to . . .

16.18 Apply the principle of superposition to show that two overlapping waves add algebraically to give a resultant (or net) wave. 16.19 For two transverse waves with the same amplitude and wavelength and that travel together, find the displacement equation for the resultant wave and calculate the amplitude in terms of the individual wave amplitude and the phase difference.

16.20 Describe how the phase difference between two transverse waves (with the same amplitude and wavelength) can result in fully constructive interference, fully destructive interference, and intermediate interference. 16.21 With the phase difference between two interfering waves expressed in terms of wavelengths, quickly determine the type of interference the waves have.

Key Ideas ● When two or more waves traverse the same medium, the displacement of any particle of the medium is the sum of the displacements that the individual waves would give it, an effect known as the principle of superposition for waves. ● Two sinusoidal waves on the same string exhibit

interference, adding or canceling according to the principle of superposition. If the two are traveling in the same direction and have the same amplitude ym and

When two waves overlap, we see the resultant wave, not the individual waves.

frequency (hence the same wavelength) but differ in phase by a phase constant f, the result is a single wave with this same frequency: y,(x, t) # [2ym cos 12f] sin(kx $ vt " 12f). If f # 0, the waves are exactly in phase and their interference is fully constructive; if f # p rad, they are exactly out of phase and their interference is fully destructive.

The Principle of Superposition for Waves It often happens that two or more waves pass simultaneously through the same region. When we listen to a concert, for example, sound waves from many instruments fall simultaneously on our eardrums. The electrons in the antennas of our radio and television receivers are set in motion by the net effect of many electromagnetic waves from many different broadcasting centers. The water of a lake or harbor may be churned up by waves in the wakes of many boats. Suppose that two waves travel simultaneously along the same stretched string. Let y1(x, t) and y2(x, t) be the displacements that the string would experience if each wave traveled alone. The displacement of the string when the waves overlap is then the algebraic sum y,(x, t) # y1(x, t) " y2(x, t).

(16-46)

This summation of displacements along the string means that Overlapping waves algebraically add to produce a resultant wave (or net wave).

Figure 16-12 A series of snapshots that show two pulses traveling in opposite directions along a stretched string. The superposition principle applies as the pulses move through each other.

This is another example of the principle of superposition, which says that when several effects occur simultaneously, their net effect is the sum of the individual effects. (We should be thankful that only a simple sum is needed. If two effects somehow amplified each other, the resulting nonlinear world would be very difficult to manage and understand.) Figure 16-12 shows a sequence of snapshots of two pulses traveling in opposite directions on the same stretched string. When the pulses overlap, the resultant pulse is their sum. Moreover, Overlapping waves do not in any way alter the travel of each other.

16-5 I NTE R FE R E NCE OF WAVES

459

Interference of Waves Suppose we send two sinusoidal waves of the same wavelength and amplitude in the same direction along a stretched string. The superposition principle applies. What resultant wave does it predict for the string? The resultant wave depends on the extent to which the waves are in phase (in step) with respect to each other — that is, how much one wave form is shifted from the other wave form. If the waves are exactly in phase (so that the peaks and valleys of one are exactly aligned with those of the other), they combine to double the displacement of either wave acting alone. If they are exactly out of phase (the peaks of one are exactly aligned with the valleys of the other), they combine to cancel everywhere, and the string remains straight. We call this phenomenon of combining waves interference, and the waves are said to interfere. (These terms refer only to the wave displacements; the travel of the waves is unaffected.) Let one wave traveling along a stretched string be given by y1(x, t) # ym sin(kx $ vt)

(16-47)

and another, shifted from the first, by y2(x, t) # ym sin(kx $ vt " f).

(16-48)

These waves have the same angular frequency v (and thus the same frequency f ), the same angular wave number k (and thus the same wavelength l), and the same amplitude ym. They both travel in the positive direction of the x axis, with the same speed, given by Eq. 16-26. They differ only by a constant angle f, the phase constant. These waves are said to be out of phase by f or to have a phase difference of f, or one wave is said to be phase-shifted from the other by f. From the principle of superposition (Eq. 16-46), the resultant wave is the algebraic sum of the two interfering waves and has displacement y,(x, t) # y1(x, t) " y2(x, t) # ym sin(kx $ vt) " ym sin(kx $ vt " f).

(16-49)

In Appendix E we see that we can write the sum of the sines of two angles a and b as sin a " sin b # 2 sin 21(a " b) cos 12(a $ b).

(16-50)

Applying this relation to Eq. 16-49 leads to y,(x, t) # [2ym cos 21f] sin(kx $ vt " 12f).

(16-51)

As Fig. 16-13 shows, the resultant wave is also a sinusoidal wave traveling in the direction of increasing x. It is the only wave you would actually see on the string (you would not see the two interfering waves of Eqs. 16-47 and 16-48). If two sinusoidal waves of the same amplitude and wavelength travel in the same direction along a stretched string, they interfere to produce a resultant sinusoidal wave traveling in that direction.

The resultant wave differs from the interfering waves in two respects: (1) its phase constant is 21f, and (2) its amplitude y,m is the magnitude of the quantity in the brackets in Eq. 16-51: y,m # |2ym cos 12f| (amplitude). (16-52) If f # 0 rad (or 0-), the two interfering waves are exactly in phase and Eq. 16-51 reduces to y,(x, t) # 2ym sin(kx $ vt)

(f # 0).

(16-53)

Displacement 1 1 __ y'(x,t) = [2ym cos __ 2 φ ] sin(kx – ω t + 2 φ )

Magnitude gives amplitude

Oscillating term

Figure 16-13 The resultant wave of Eq. 16-51, due to the interference of two sinusoidal transverse waves, is also a sinusoidal transverse wave, with an amplitude and an oscillating term.

460

CHAPTE R 16 WAVES—I

Being exactly in phase, the waves produce a large resultant wave.

Being exactly out of phase, they produce a flat string.

y

y y1(x, t) and y2(x, t)

y y1(x, t)

y1(x, t)

y2(x, t)

x

x

φ = _23_π rad

φ = π rad

(a)

(b)

y

y2(x, t)

x

φ=0

Figure 16-14 Two identical sinusoidal waves, y1(x, t) and y2(x, t), travel along a string in the positive direction of an x axis. They interfere to give a resultant wave y,(x, t). The resultant wave is what is actually seen on the string. The phase difference f between the two interfering waves is (a) 0 rad or 0-, (b) p rad or 180-, and (c) 23p rad or 120-. The corresponding resultant waves are shown in (d), (e), and ( f ).

This is an intermediate situation, with an intermediate result.

(c)

y

y

y'(x, t)

y'(x, t) x

y'(x, t)

(d)

x

(e)

x

(f )

The two waves are shown in Fig. 16-14a, and the resultant wave is plotted in Fig. 16-14d. Note from both that plot and Eq. 16-53 that the amplitude of the resultant wave is twice the amplitude of either interfering wave. That is the greatest amplitude the resultant wave can have, because the cosine term in Eqs. 16-51 and 16-52 has its greatest value (unity) when f # 0. Interference that produces the greatest possible amplitude is called fully constructive interference. If f # p rad (or 180-), the interfering waves are exactly out of phase as in Fig. 16-14b. Then cos 21f becomes cos p/2 # 0, and the amplitude of the resultant wave as given by Eq. 16-52 is zero. We then have, for all values of x and t, y,(x, t) # 0

(f # p rad).

(16-54)

The resultant wave is plotted in Fig. 16-14e. Although we sent two waves along the string, we see no motion of the string. This type of interference is called fully destructive interference. Because a sinusoidal wave repeats its shape every 2p rad, a phase difference of f # 2p rad (or 360-) corresponds to a shift of one wave relative to the other wave by a distance equivalent to one wavelength. Thus, phase differences can be described in terms of wavelengths as well as angles. For example, in Fig. 16-14b the waves may be said to be 0.50 wavelength out of phase. Table 16-1 shows some other examples of phase differences and the interference they produce. Note that when interference is neither fully constructive nor fully destructive, it is called intermediate interference. The amplitude of the resultant wave is then intermediate between 0 and 2ym. For example, from Table 16-1, if the interfering waves have a phase difference of 120- (f # 23p rad # 0.33 wavelength), then the resultant wave has an amplitude of ym, the same as that of the interfering waves (see Figs. 16-14c and f ). Two waves with the same wavelength are in phase if their phase difference is zero or any integer number of wavelengths. Thus, the integer part of any phase difference expressed in wavelengths may be discarded. For example, a phase difference of 0.40 wavelength (an intermediate interference, close to fully destructive interference) is equivalent in every way to one of 2.40 wavelengths,

461

16-5 I NTE R FE R E NCE OF WAVES

Table 16-1 Phase Difference and Resulting Interference Typesa Phase Difference, in Degrees

Radians

Wavelengths 0

Amplitude of Resultant Wave 2ym

Type of Interference

0

0

120

2 3p

0.33

ym

Intermediate

180

p

0.50

0

Fully destructive

240

4 3p

0.67

ym

Intermediate

360

2p

1.00

2ym

865

15.1

2.40

0.60ym

Fully constructive

Fully constructive Intermediate

a

The phase difference is between two otherwise identical waves, with amplitude ym, moving in the same direction.

and so the simpler of the two numbers can be used in computations. Thus, by looking at only the decimal number and comparing it to 0, 0.5, or 1.0 wavelength, you can quickly tell what type of interference two waves have.

Checkpoint 4 Here are four possible phase differences between two identical waves, expressed in wavelengths: 0.20, 0.45, 0.60, and 0.80. Rank them according to the amplitude of the resultant wave, greatest first.

Sample Problem 16.04 Interference of two waves, same direction, same amplitude Two identical sinusoidal waves, moving in the same direction along a stretched string, interfere with each other. The amplitude ym of each wave is 9.8 mm, and the phase difference f between them is 100-. (a) What is the amplitude y,m of the resultant wave due to the interference, and what is the type of this interference? KEY IDEA These are identical sinusoidal waves traveling in the same direction along a string, so they interfere to produce a sinusoidal traveling wave. Calculations: Because they are identical, the waves have the same amplitude. Thus, the amplitude y,m of the resultant wave is given by Eq. 16-52: y,m # |2ym cos 12f| # |(2)(9.8 mm) cos(100-/2)| # 13 mm. (Answer) We can tell that the interference is intermediate in two ways. The phase difference is between 0 and 180-, and, correspondingly, the amplitude y,m is between 0 and 2ym (# 19.6 mm).

(b) What phase difference, in radians and wavelengths, will give the resultant wave an amplitude of 4.9 mm? Calculations: Now we are given y,m and seek f. From Eq. 16-52, y,m # |2ym cos 21f|, we now have 4.9 mm # (2)(9.8 mm) cos 12f, which gives us (with a calculator in the radian mode) 4.9 mm (2)(9.8 mm) # !2.636 rad % !2.6 rad.

f # 2 cos$1

(Answer)

There are two solutions because we can obtain the same resultant wave by letting the first wave lead (travel ahead of) or lag (travel behind) the second wave by 2.6 rad. In wavelengths, the phase difference is f !2.636 rad # 2p rad /wavelength 2p rad /wavelength # !0.42 wavelength.

Additional examples, video, and practice available at WileyPLUS

(Answer)

462

CHAPTE R 16 WAVES—I

16-6 PHASORS Learning Objectives After reading this module, you should be able to . . .

16.22 Using sketches, explain how a phasor can represent the oscillations of a string element as a wave travels through its location. 16.23 Sketch a phasor diagram for two overlapping waves traveling together on a string, indicating their amplitudes and phase difference on the sketch.

16.24 By using phasors, find the resultant wave of two transverse waves traveling together along a string, calculating the amplitude and phase and writing out the displacement equation, and then displaying all three phasors in a phasor diagram that shows the amplitudes, the leading or lagging, and the relative phases.

Key Idea ● A wave y(x, t) can be represented with a phasor. This is a vector that has a magnitude equal to the amplitude ym of the wave and that rotates about an origin with an angular speed

equal to the angular frequency v of the wave. The projection of the rotating phasor on a vertical axis gives the displacement y of a point along the wave’s travel.

Phasors Adding two waves as discussed in the preceding module is strictly limited to waves with identical amplitudes. If we have such waves, that technique is easy enough to use, but we need a more general technique that can be applied to any waves, whether or not they have the same amplitudes. One neat way is to use phasors to represent the waves. Although this may seem bizarre at first, it is essentially a graphical technique that uses the vector addition rules of Chapter 3 instead of messy trig additions. A phasor is a vector that rotates around its tail, which is pivoted at the origin of a coordinate system. The magnitude of the vector is equal to the amplitude ym of the wave that it represents. The angular speed of the rotation is equal to the angular frequency v of the wave. For example, the wave y1(x, t) # ym1 sin(kx $ vt)

(16-55)

is represented by the phasor shown in Figs. 16-15a to d. The magnitude of the phasor is the amplitude ym1 of the wave. As the phasor rotates around the origin at angular speed v, its projection y1 on the vertical axis varies sinusoidally, from a maximum of ym1 through zero to a minimum of $ym1 and then back to ym1. This variation corresponds to the sinusoidal variation in the displacement y1 of any point along the string as the wave passes through that point. (All this is shown as an animation with voiceover in WileyPLUS.) When two waves travel along the same string in the same direction, we can represent them and their resultant wave in a phasor diagram. The phasors in Fig. 16-15e represent the wave of Eq. 16-55 and a second wave given by y2(x, t) # ym2 sin(kx $ vt " f).

(16-56)

This second wave is phase-shifted from the first wave by phase constant f. Because the phasors rotate at the same angular speed v, the angle between the two phasors is always f. If f is a positive quantity, then the phasor for wave 2 lags the phasor for wave 1 as they rotate, as drawn in Fig. 16-15e. If f is a negative quantity, then the phasor for wave 2 leads the phasor for wave 1. Because waves y1 and y2 have the same angular wave number k and angular frequency v, we know from Eqs. 16-51 and 16-52 that their resultant is of the form y,(x, t) # y,m sin(kx $ vt " b),

(16-57)

463

16-6 PHASORS

A

This projection matches this displacement of the dot as the wave moves through it.

Zero projection, zero displacement

y

y

ω y1

ym1

x

x

y1 = 0

ω

(a)

(b)

Maximum negative projection

The next crest is about to move through the dot.

y

y

ω

y1

ym1 x y1

x

y1 = ym1

ω (c)

(d)

This is a snapshot of the two phasors for two waves.

Adding the two phasors as vectors gives the resultant phasor of the resultant wave.

Wave 2, delayed by φ radians These are the y2 projections of the two phasors. y1 (e)

ym2

φ

ym1

ω

Wave 1

ω

This is the projection of the resultant phasor.

ym2 y'

y2

y1

y'm

β

φ

ym1

(f )

Figure 16-15 (a)–(d) A phasor of magnitude ym1 rotating about an origin at angular speed v represents a sinusoidal wave. The phasor’s projection y1 on the vertical axis represents the displacement of a point through which the wave passes. (e) A second phasor, also of angular speed v but of magnitude ym2 and rotating at a constant angle f from the first phasor, represents a second wave, with a phase constant f. (f ) The resultant wave is represented by the vector sum y,m of the two phasors.

464

CHAPTE R 16 WAVES—I

where y,m is the amplitude of the resultant wave and b is its phase constant. To find the values of y,m and b, we would have to sum the two combining waves, as we did to obtain Eq. 16-51. To do this on a phasor diagram, we vectorially add the two phasors at any instant during their rotation, as in Fig. 16-15f where phasor ym2 has been shifted to the head of phasor ym1. The magnitude of the vector sum equals the amplitude y,m in Eq. 16-57. The angle between the vector sum and the phasor for y1 equals the phase constant b in Eq. 16-57. Note that, in contrast to the method of Module 16-5: We can use phasors to combine waves even if their amplitudes are different.

Sample Problem 16.05 Interference of two waves, same direction, phasors, any amplitudes Two sinusoidal waves y1(x, t) and y2(x, t) have the same wavelength and travel together in the same direction along a string. Their amplitudes are ym1 # 4.0 mm and ym2 # 3.0 mm, and their phase constants are 0 and p/3 rad, respectively. What are the amplitude y,m and phase constant b of the resultant wave? Write the resultant wave in the form of Eq. 16-57. KEY IDEAS (1) The two waves have a number of properties in common: Because they travel along the same string, they must have the same speed v, as set by the tension and linear density of the string according to Eq. 16-26. With the same wavelength l, they have the same angular wave number k (# 2p/l). Also, because they have the same wave number k and speed v, they must have the same angular frequency v (# kv). (2) The waves (call them waves 1 and 2) can be represented by phasors rotating at the same angular speed v about an origin. Because the phase constant for wave 2 is greater than that for wave 1 by p/3, phasor 2 must lag phasor 1 by p/3 rad in their clockwise rotation, as shown in Fig. 16-16a. The resultant wave due to the interference of waves 1 and 2 can then be represented by a phasor that is the vector sum of phasors 1 and 2. Calculations: To simplify the vector summation, we drew phasors 1 and 2 in Fig. 16-16a at the instant when phasor 1 lies along the horizontal axis. We then drew lagging phasor 2 at positive angle p/3 rad. In Fig. 16-16b we shifted phasor 2 so its tail is at the head of phasor 1. Then we can draw the phasor y,m of the resultant wave from the tail of phasor 1 to the head of phasor 2. The phase constant b is the angle phasor y,m makes with phasor 1. To find values for y,m and b, we can sum phasors 1 and 2 as vectors on a vector-capable calculator. However, here

we shall sum them by components. (They are called horizontal and vertical components, because the symbols x and y are already used for the waves themselves.) For the horizontal components we have y,mh # ym1 cos 0 " ym2 cos p/3 # 4.0 mm " (3.0 mm) cos p/3 # 5.50 mm. For the vertical components we have y,mv # ym1 sin 0 " ym2 sin p/3 # 0 " (3.0 mm) sin p/3 # 2.60 mm. Thus, the resultant wave has an amplitude of y,m # 2(5.50 mm)2 " (2.60 mm)2 # 6.1 mm (Answer) and a phase constant of 2.60 mm b # tan$1 (Answer) # 0.44 rad. 5.50 mm From Fig. 16-16b, phase constant b is a positive angle relative to phasor 1. Thus, the resultant wave lags wave 1 in their travel by phase constant b # "0.44 rad. From Eq. 16-57, we can write the resultant wave as y,(x, t) # (6.1 mm) sin(kx $ vt " 0.44 rad).

(Answer)

Add the phasors as vectors. ym2

π/3 ym1 (a)

y'm

y'

β ym1

ym2

π/3

(b)

Figure 16-16 (a) Two phasors of magnitudes ym1 and ym2 and with phase difference p/3. (b) Vector addition of these phasors at any instant during their rotation gives the magnitude y,m of the phasor for the resultant wave.

Additional examples, video, and practice available at WileyPLUS

16-7 STAN DI NG WAVES AN D R ESONANCE

465

16-7 STANDING WAVES AND RESONANCE Learning Objectives After reading this module, you should be able to . . .

16.25 For two overlapping waves (same amplitude and wavelength) that are traveling in opposite directions, sketch snapshots of the resultant wave, indicating nodes and antinodes. 16.26 For two overlapping waves (same amplitude and wavelength) that are traveling in opposite directions, find the displacement equation for the resultant wave and calculate the amplitude in terms of the individual wave amplitude. 16.27 Describe the SHM of a string element at an antinode of a standing wave.

16.28 For a string element at an antinode of a standing wave, write equations for the displacement, transverse velocity, and transverse acceleration as functions of time. 16.29 Distinguish between “hard” and “soft” reflections of string waves at a boundary. 16.30 Describe resonance on a string tied taut between two supports, and sketch the first several standing wave patterns, indicating nodes and antinodes. 16.31 In terms of string length, determine the wavelengths required for the first several harmonics on a string under tension. 16.32 For any given harmonic, apply the relationship between frequency, wave speed, and string length.

Key Ideas ● The interference of two identical sinusoidal waves moving in opposite directions produces standing waves. For a string with fixed ends, the standing wave is given by

y,(x, t) # [2ym sin kx] cos vt. Standing waves are characterized by fixed locations of zero displacement called nodes and fixed locations of maximum displacement called antinodes. ● Standing waves on a string can be set up by reflection of traveling waves from the ends of the string. If an end is fixed, it must be the position of a node. This limits the frequencies at

which standing waves will occur on a given string. Each possible frequency is a resonant frequency, and the corresponding standing wave pattern is an oscillation mode. For a stretched string of length L with fixed ends, the resonant frequencies are v v f# #n , for n # 1, 2, 3, . . . . l 2L The oscillation mode corresponding to n # 1 is called the fundamental mode or the first harmonic; the mode corresponding to n # 2 is the second harmonic; and so on.

Standing Waves In Module 16-5, we discussed two sinusoidal waves of the same wavelength and amplitude traveling in the same direction along a stretched string. What if they travel in opposite directions? We can again find the resultant wave by applying the superposition principle. Figure 16-17 suggests the situation graphically. It shows the two combining waves, one traveling to the left in Fig. 16-17a, the other to the right in Fig. 16-17b. Figure 16-17c shows their sum, obtained by applying the superposition Figure 16-17 (a) Five snapshots of a wave traveling to the left, at the times t indicated below part (c) (T is the period of oscillation). (b) Five snapshots of a wave identical to that in (a) but traveling to the right, at the same times t. (c) Corresponding snapshots for the superposition of the two waves on the same string.At t # 0, 12T, and T, fully constructive interference occurs because of the alignment of peaks with peaks and valleys with valleys.At t # 14T and 34T, fully destructive interference occurs because of the alignment of peaks with valleys. Some points (the nodes, marked with dots) never oscillate; some points (the antinodes) oscillate the most.

As the waves move through each other, some points never move and some move the most. (a )

(b )

(c )

x

t =0

x

t =

1 T 4

x

t =

1 T 2

x

t =

3 T 4

x

t =T

466

CHAPTE R 16 WAVES—I

principle graphically.The outstanding feature of the resultant wave is that there are places along the string, called nodes, where the string never moves. Four such nodes are marked by dots in Fig. 16-17c. Halfway between adjacent nodes are antinodes, where the amplitude of the resultant wave is a maximum. Wave patterns such as that of Fig. 16-17c are called standing waves because the wave patterns do not move left or right; the locations of the maxima and minima do not change. If two sinusoidal waves of the same amplitude and wavelength travel in opposite directions along a stretched string, their interference with each other produces a standing wave.

To analyze a standing wave, we represent the two waves with the equations y1(x, t) # ym sin(kx $ vt) (16-58) and

y2(x, t) # ym sin(kx " vt).

(16-59)

The principle of superposition gives, for the combined wave, y,(x, t) # y1(x, t) " y2(x, t) # ym sin(kx $ vt) " ym sin(kx " vt). Applying the trigonometric relation of Eq. 16-50 leads to Fig. 16-18 and y,(x, t) # [2ym sin kx] cos vt.

(16-60)

This equation does not describe a traveling wave because it is not of the form of Eq. 16-17. Instead, it describes a standing wave. The quantity 2ym sin kx in the brackets of Eq. 16-60 can be viewed as the amplitude of oscillation of the string element that is located at position x. However, since an amplitude is always positive and sin kx can be negative, we take the absolute value of the quantity 2ym sin kx to be the amplitude at x. In a traveling sinusoidal wave, the amplitude of the wave is the same for all string elements. That is not true for a standing wave, in which the amplitude varies with position. In the standing wave of Eq. 16-60, for example, the amplitude is zero for values of kx that give sin kx # 0. Those values are for n # 0, 1, 2, . . . .

kx # np,

(16-61)

Substituting k # 2p/l in this equation and rearranging, we get x#n

l , 2

for n # 0, 1, 2, . . .

(nodes),

(16-62)

as the positions of zero amplitude — the nodes — for the standing wave of Eq. 16-60. Note that adjacent nodes are separated by l/2, half a wavelength. The amplitude of the standing wave of Eq. 16-60 has a maximum value of 2ym, which occurs for values of kx that give | sin kx | # 1. Those values are kx # 12p, 32p, 52p, . . . # (n " 12)p, Displacement y'(x,t) = [2ym sin kx]cos ω t Magnitude Oscillating gives term amplitude at position x

Figure 16-18 The resultant wave of Eq. 16-60 is a standing wave and is due to the interference of two sinusoidal waves of the same amplitude and wavelength that travel in opposite directions.

for n # 0, 1, 2, . . . .

(16-63)

Substituting k # 2p/l in Eq. 16-63 and rearranging, we get

#

x# n"

1 2

$ l2 ,

for n # 0, 1, 2, . . .

(antinodes),

(16-64)

as the positions of maximum amplitude — the antinodes — of the standing wave of Eq. 16-60. Antinodes are separated by l/2 and are halfway between nodes.

Reflections at a Boundary We can set up a standing wave in a stretched string by allowing a traveling wave to be reflected from the far end of the string so that the wave travels back

16-7 STAN DI NG WAVES AN D R ESONANCE

through itself. The incident (original) wave and the reflected wave can then be described by Eqs. 16-58 and 16-59, respectively, and they can combine to form a pattern of standing waves. In Fig. 16-19, we use a single pulse to show how such reflections take place. In Fig. 16-19a, the string is fixed at its left end.When the pulse arrives at that end, it exerts an upward force on the support (the wall). By Newton’s third law, the support exerts an opposite force of equal magnitude on the string. This second force generates a pulse at the support, which travels back along the string in the direction opposite that of the incident pulse. In a “hard” reflection of this kind, there must be a node at the support because the string is fixed there. The reflected and incident pulses must have opposite signs, so as to cancel each other at that point. In Fig. 16-19b, the left end of the string is fastened to a light ring that is free to slide without friction along a rod. When the incident pulse arrives, the ring moves up the rod. As the ring moves, it pulls on the string, stretching the string and producing a reflected pulse with the same sign and amplitude as the incident pulse. Thus, in such a “soft” reflection, the incident and reflected pulses reinforce each other, creating an antinode at the end of the string; the maximum displacement of the ring is twice the amplitude of either of these two pulses.

467

There are two ways a pulse can reflect from the end of a string.

Checkpoint 5 Two waves with the same amplitude and wavelength interfere in three different situations to produce resultant waves with the following equations: (1) y,(x, t) # 4 sin(5x $ 4t) (2) y,(x, t) # 4 sin(5x) cos(4t) (3) y,(x, t) # 4 sin(5x " 4t) In which situation are the two combining waves traveling (a) toward positive x, (b) toward negative x, and (c) in opposite directions?

Standing Waves and Resonance Consider a string, such as a guitar string, that is stretched between two clamps. Suppose we send a continuous sinusoidal wave of a certain frequency along the string, say, toward the right. When the wave reaches the right end, it reflects and begins to travel back to the left. That left-going wave then overlaps the wave that is still traveling to the right. When the left-going wave reaches the left end, it reflects again and the newly reflected wave begins to travel to the right, overlapping the left-going and right-going waves. In short, we very soon have many overlapping traveling waves, which interfere with one another. For certain frequencies, the interference produces a standing wave pattern (or oscillation mode) with nodes and large antinodes like those in Fig. 16-20. Such a standing wave is said to be produced at resonance, and the string is said to resonate at these certain frequencies, called resonant frequencies. If the string

Richard Megna/Fundamental Photographs

Figure 16-20 Stroboscopic photographs reveal (imperfect) standing wave patterns on a string being made to oscillate by an oscillator at the left end. The patterns occur at certain frequencies of oscillation.

(a)

(b)

Figuer 16-19 (a) A pulse incident from the right is reflected at the left end of the string, which is tied to a wall. Note that the reflected pulse is inverted from the incident pulse. (b) Here the left end of the string is tied to a ring that can slide without friction up and down the rod. Now the pulse is not inverted by the reflection.

468

CHAPTE R 16 WAVES—I

L

(a)

L= λ 2

(b)

First harmonic

L = λ = 2λ 2

(c)

Second harmonic

L = 3λ 2

Third harmonic

Figure 16-21 A string, stretched between two clamps, is made to oscillate in standing wave patterns. (a) The simplest possible pattern consists of one loop, which refers to the composite shape formed by the string in its extreme displacements (the solid and dashed lines). (b) The next simplest pattern has two loops. (c) The next has three loops.

is oscillated at some frequency other than a resonant frequency, a standing wave is not set up. Then the interference of the right-going and left-going traveling waves results in only small, temporary (perhaps even imperceptible) oscillations of the string. Let a string be stretched between two clamps separated by a fixed distance L. To find expressions for the resonant frequencies of the string, we note that a node must exist at each of its ends, because each end is fixed and cannot oscillate. The simplest pattern that meets this key requirement is that in Fig. 16-21a, which shows the string at both its extreme displacements (one solid and one dashed, together forming a single “loop”). There is only one antinode, which is at the center of the string. Note that half a wavelength spans the length L, which we take to be the string’s length. Thus, for this pattern, l/2 # L. This condition tells us that if the left-going and right-going traveling waves are to set up this pattern by their interference, they must have the wavelength l # 2L. A second simple pattern meeting the requirement of nodes at the fixed ends is shown in Fig. 16-21b. This pattern has three nodes and two antinodes and is said to be a two-loop pattern. For the left-going and right-going waves to set it up, they must have a wavelength l # L. A third pattern is shown in Fig. 16-21c. It has four nodes, three antinodes, and three loops, and the wavelength is l # 23L. We could continue this progression by drawing increasingly more complicated patterns. In each step of the progression, the pattern would have one more node and one more antinode than the preceding step, and an additional l/2 would be fitted into the distance L. Thus, a standing wave can be set up on a string of length L by a wave with a wavelength equal to one of the values

.#

2L , n

for n # 1, 2, 3, . . . .

(16-65)

The resonant frequencies that correspond to these wavelengths follow from Eq. 16-13: f#

Courtesy Thomas D. Rossing, Northern Illinois University

Figure 16-22 One of many possible standing wave patterns for a kettledrum head, made visible by dark powder sprinkled on the drumhead. As the head is set into oscillation at a single frequency by a mechanical oscillator at the upper left of the photograph, the powder collects at the nodes, which are circles and straight lines in this two-dimensional example.

v v #n , . 2L

for n # 1, 2, 3, . . . .

(16-66)

Here v is the speed of traveling waves on the string. Equation 16-66 tells us that the resonant frequencies are integer multiples of the lowest resonant frequency, f # v/2L, which corresponds to n # 1. The oscillation mode with that lowest frequency is called the fundamental mode or the first harmonic. The second harmonic is the oscillation mode with n # 2, the third harmonic is that with n # 3, and so on. The frequencies associated with these modes are often labeled f1, f2, f3, and so on. The collection of all possible oscillation modes is called the harmonic series, and n is called the harmonic number of the nth harmonic. For a given string under a given tension, each resonant frequency corresponds to a particular oscillation pattern. Thus, if the frequency is in the audible range, you can hear the shape of the string. Resonance can also occur in two dimensions (such as on the surface of the kettledrum in Fig. 16-22) and in three dimensions (such as in the wind-induced swaying and twisting of a tall building).

Checkpoint 6 In the following series of resonant frequencies, one frequency (lower than 400 Hz) is missing: 150, 225, 300, 375 Hz. (a) What is the missing frequency? (b) What is the frequency of the seventh harmonic?

469

16-7 STAN DI NG WAVES AN D R ESONANCE

Sample Problem 16.06 Resonance of transverse waves, standing waves, harmonics Figure 16-23 shows resonant oscillation of a string of mass m # 2.500 g and length L # 0.800 m and that is under tension t # 325.0 N. What is the wavelength l of the transverse waves producing the standing wave pattern, and what is the harmonic number n? What is the frequency f of the transverse waves and of the oscillations of the moving string elements? What is the maximum magnitude of the transverse velocity um of the element oscillating at coordinate x # 0.180 m? At what point during the element’s oscillation is the transverse velocity maximum?

(1) The traverse waves that produce a standing wave pattern must have a wavelength such that an integer number n of half-wavelengths fit into the length L of the string. (2) The frequency of those waves and of the oscillations of the string elements is given by Eq. 16-66 (f # nv/2L). (3) The displacement of a string element as a function of position x and time t is given by Eq. 16-60: y,(x, t) # [2ym sin kx] cos vt.

(16-67)

Wavelength and harmonic number: In Fig. 16-23, the solid line, which is effectively a snapshot (or freeze-frame) of the oscillations, reveals that 2 full wavelengths fit into the length L # 0.800 m of the string.Thus, we have 2l # L,

.#

L . 2

(16-68)

0.800 m (Answer) # 0.400 m. 2 By counting the number of loops (or half-wavelengths) in Fig. 16-23, we see that the harmonic number is #

n # 4.

(Answer)

We also find n # 4 by comparing Eqs. 16-68 and 16-65 (l # 2L/n). Thus, the string is oscillating in its fourth harmonic. Frequency: We can get the frequency f of the transverse waves from Eq. 16-13 (v # lf ) if we first find the speed v of the waves. That speed is given by Eq. 16-26, but we must substitute m/L for the unknown linear density m.We obtain v# #

8.00 mm

x (m) 0.800

0

Figure 16-23 Resonant oscillation of a string under tension.

# 806.2 Hz % 806 Hz.

(Answer)

Note that we get the same answer by substituting into Eq. 16-66:

KEY IDEAS

or

y

t t tL # # A m/L A m Am

(325 N)(0.800 m) # 322.49 m/s. A 2.50 ' 10$3 kg

After rearranging Eq. 16-13, we write f#

v 322.49 m/s # l 0.400 m

f#n

322.49 m/s v #4 2L 2(0.800 m)

# 806 Hz.

(Answer)

Now note that this 806 Hz is not only the frequency of the waves producing the fourth harmonic but also it is said to be the fourth harmonic, as in the statement, “The fourth harmonic of this oscillating string is 806 Hz.” It is also the frequency of the string elements as they oscillate vertically in the figure in simple harmonic motion, just as a block on a vertical spring would oscillate in simple harmonic motion. Finally, it is also the frequency of the sound you would hear as the oscillating string periodically pushes against the air. Transverse velocity: The displacement y, of the string element located at coordinate x is given by Eq. 16-67 as a function of time t. The term cos vt contains the dependence on time and thus provides the “motion” of the standing wave. The term 2ym sin kx sets the extent of the motion — that is, the amplitude. The greatest amplitude occurs at an antinode, where sin kx is "1 or $1 and thus the greatest amplitude is 2ym. From Fig. 16-23, we see that 2ym # 4.00 mm, which tells us that ym # 2.00 mm. We want the transverse velocity — the velocity of a string element parallel to the y axis. To find it, we take the time derivative of Eq. 16-67: u(x, t) #

&y, & # [(2ym sin kx) cos vt] &t &t

# [$2ym* sin kx] sin *t.

(16-69)

Here the term sin vt provides the variation with time and the term $2ymv sin kx provides the extent of that variation. We want the absolute magnitude of that extent: um # ! $2ymv sin kx !. To evaluate this for the element at x # 0.180 m, we first note that ym # 2.00 mm, k # 2p/l # 2p/(0.400 m), and v # 2pf # 2p (806.2 Hz). Then the maximum speed of the element at x # 0.180 m is

470

CHAPTE R 16 WAVES—I

!

um # $2(2.00 ' 10$3 m)(2p)(806.2 Hz) 2p (0.180 m)$ ! # 0.400 m

' sin

# 6.26 m/s.

(Answer)

To determine when the string element has this maximum speed, we could investigate Eq. 16-69. However, a little thought can save a lot of work. The element is undergoing SHM and must come to a momentary stop at its extreme upward position and extreme downward position. It has the greatest speed as it zips through the midpoint of its oscillation, just as a block does in a block – spring oscillator.

Additional examples, video, and practice available at WileyPLUS

Review & Summary Transverse and Longitudinal Waves Mechanical waves can exist only in material media and are governed by Newton’s laws. Transverse mechanical waves, like those on a stretched string, are waves in which the particles of the medium oscillate perpendicular to the wave’s direction of travel. Waves in which the particles of the medium oscillate parallel to the wave’s direction of travel are longitudinal waves.

Sinusoidal Waves A sinusoidal wave moving in the positive direction of an x axis has the mathematical form y(x, t) # ym sin(kx $ vt),

(16-2)

where ym is the amplitude of the wave, k is the angular wave number, v is the angular frequency, and kx $ vt is the phase. The wavelength l is related to k by 2/ (16-5) k# . . The period T and frequency f of the wave are related to v by

* 1 #f# . 2/ T

(16-9)

Finally, the wave speed v is related to these other parameters by

* . # # .f. v# k T

(16-13)

Superposition of Waves When two or more waves traverse the same medium, the displacement of any particle of the medium is the sum of the displacements that the individual waves would give it.

Interference of Waves Two sinusoidal waves on the same string exhibit interference, adding or canceling according to the principle of superposition. If the two are traveling in the same direction and have the same amplitude ym and frequency (hence the same wavelength) but differ in phase by a phase constant f, the result is a single wave with this same frequency: y,(x, t) # [2ym cos 12f] sin(kx $ vt " 12f).

If f # 0, the waves are exactly in phase and their interference is fully constructive; if f # p rad, they are exactly out of phase and their interference is fully destructive.

Phasors A wave y(x, t) can be represented with a phasor. This is a vector that has a magnitude equal to the amplitude ym of the wave and that rotates about an origin with an angular speed equal to the angular frequency v of the wave. The projection of the rotating phasor on a vertical axis gives the displacement y of a point along the wave’s travel.

Standing Waves The interference of two identical sinusoidal waves moving in opposite directions produces standing waves. For a string with fixed ends, the standing wave is given by y,(x, t) # [2ym sin kx] cos vt.

Equation of a Traveling Wave Any function of the form y(x, t) # h(kx ! vt)

(16-17)

can represent a traveling wave with a wave speed given by Eq. 16-13 and a wave shape given by the mathematical form of h. The plus sign denotes a wave traveling in the negative direction of the x axis, and the minus sign a wave traveling in the positive direction.

Wave Speed on Stretched String The speed of a wave on a stretched string is set by properties of the string. The speed on a string with tension t and linear density m is v#

t . Am

(16-26)

Power The average power of, or average rate at which energy is transmitted by, a sinusoidal wave on a stretched string is given by Pavg # 12mvv2y2m.

(16-33)

(16-51)

(16-60)

Standing waves are characterized by fixed locations of zero displacement called nodes and fixed locations of maximum displacement called antinodes.

Resonance Standing waves on a string can be set up by reflection of traveling waves from the ends of the string. If an end is fixed, it must be the position of a node. This limits the frequencies at which standing waves will occur on a given string. Each possible frequency is a resonant frequency, and the corresponding standing wave pattern is an oscillation mode. For a stretched string of length L with fixed ends, the resonant frequencies are f#

v v #n , . 2L

for n # 1, 2, 3, . . . .

(16-66)

The oscillation mode corresponding to n # 1 is called the fundamental mode or the first harmonic; the mode corresponding to n # 2 is the second harmonic; and so on.

QU ESTIONS

471

Questions 6 The amplitudes and phase differences for four pairs of waves of equal wavelengths are (a) 2 mm, 6 mm, and p rad; (b) 3 mm, 5 mm, and p rad; (c) 7 mm, 9 mm, and p rad; (d) 2 mm, 2 mm, and 0 rad. Each pair travels in the same direction along the same string. Without written calculation, rank the four pairs according to the amplitude of their resultant wave, greatest first. (Hint: Construct phasor diagrams.)

1 The following four waves are sent along strings with the same linear densities (x is in meters and t is in seconds). Rank the waves according to (a) their wave speed and (b) the tension in the strings along which they travel, greatest first: (1) y1 # (3 mm) sin(x $ 3t), (3) y3 # (1 mm) sin(4x $ t), (2) y2 # (6 mm) sin(2x $ t), (4) y4 # (2 mm) sin(x $ 2t). 2 In Fig. 16-24, wave 1 consists of a rectangular peak of height 4 units and width d, and a rectangular valley of depth 2 units and width d. The wave travels rightward along an x axis. Choices 2, 3, and 4 are similar waves, with the same heights, depths, and widths, that will travel leftward along that axis and through wave 1. Right-going wave 1 and one of the left-going waves will interfere as they pass through each other. With which left-going wave will the interference give, for an instant, (a) the deepest valley,(b) a flat line,and (c) a flat peak 2d wide?

7 A sinusoidal wave is sent along a cord under tension, transporting energy at the average rate of Pavg,1. Two waves, identical to that first one, are then to be sent along the cord with a phase difference f of either 0, 0.2 wavelength, or 0.5 wavelength. (a) With only mental calculation, rank those choices of f according to the average rate at which the waves will transport energy, greatest first. (b) For the first choice of f, what is the average rate in terms of Pavg,1? 8 (a) If a standing wave on a string is given by y,(t) # (3 mm) sin(5x) cos(4t),

(1)

is there a node or an antinode of the oscillations of the string at x # 0? (b) If the standing wave is given by

(2)

y,(t) # (3 mm) sin(5x " p/2) cos(4t), is there a node or an antinode at x # 0?

(3)

9 Strings A and B have identical lengths and linear densities, but string B is under greater tension than string A. Figure 16-27 shows four situations, (a) through (d), in which standing wave patterns exist on the two strings. In which situations is there the possibility that strings A and B are oscillating at the same resonant frequency?

(4)

Figure 16-24 Question 2. 3 Figure 16-25a gives a snapshot of a wave traveling in the direction of positive x along a string under tension. Four string elements are indicated by the lettered points. For each of those elements, determine whether, at the instant of the snapshot, the element is moving upward or downward or is momentarily at rest. (Hint: Imagine the wave as it moves through the four string elements, as if you were watching a video of the wave as it traveled rightward.) Figure 16-25b gives the displacement of a string element located at, say, x # 0 as a function of time. At the lettered times, is the element moving upward or downward or is it momentarily at rest? y

String A (a)

(b)

y a

d

b

c

String B

(c) f g

x

e

t

h

(d)

Figure 16-27 Question 9. (a)

(b)

Figure 16-25 Question 3. 4 Figure 16-26 shows three waves that are separately sent along a string that is stretched under a certain tension along an x axis. Rank the waves according to their (a) wavelengths, (b) speeds, and (c) angular frequencies, greatest first.

y 3 x 2 1

5 If you start with two sinusoidal waves of the same amplitude traveling in phase on a string and then Figure 16-26 Question 4. somehow phase-shift one of them by 5.4 wavelengths, what type of interference will occur on the string?

10 If you set up the seventh harmonic on a string, (a) how many nodes are present, and (b) is there a node, antinode, or some intermediate state at the midpoint? If you next set up the sixth harmonic, (c) is its resonant wavelength longer or shorter than that for the seventh harmonic, and (d) is the resonant frequency higher or lower? 11 Figure 16-28 shows phasor diagrams for three (a) (b) (c) situations in which two Figure 16-28 Question 11. waves travel along the same string. All six waves have the same amplitude. Rank the situations according to the amplitude of the net wave on the string, greatest first.

472

CHAPTE R 16 WAVES—I

Problems Tutoring problem available (at instructor’s discretion) in WileyPLUS and WebAssign SSM • – •••

Worked-out solution available in Student Solutions Manual

WWW Worked-out solution is at

Number of dots indicates level of problem difficulty

ILW

Interactive solution is at

http://www.wiley.com/college/halliday

Additional information available in The Flying Circus of Physics and at flyingcircusofphysics.com

Module 16-1 Transverse Waves •1 If a wave y(x, t) # (6.0 mm) sin(kx " (600 rad/s)t " f) travels along a string, how much time does any given point on the string take to move between displacements y # "2.0 mm and y # $2.0 mm? •2 A human wave. During v sporting events within large, densely packed stadiums, spectators will send a wave (or pulse) around the stadium (Fig. 16-29). As the wave w reaches a group of spectators, they stand with a cheer and then sit. At Figure 16-29 Problem 2. any instant, the width w of the wave is the distance from the leading edge (people are just about to stand) to the trailing edge (people have just sat down). Suppose a human wave travels a distance of 853 seats around a stadium in 39 s, with spectators requiring about 1.8 s to respond to the wave’s passage by standing and then sitting.What are (a) the wave speed v (in seats per second) and (b) width w (in number of seats)? •3 A wave has an angular frequency of 110 rad/s and a wavelength of 1.80 m. Calculate (a) the angular wave number and (b) the speed of the wave. •4 A sand scorpion can detect the motion of a nearby beetle (its prey) by the waves the motion sends along the sand surface (Fig. 16-30). The waves are of two types: transverse waves traveling at vt # 50 m/s and longitudinal waves traveling at vl # 150 m/s. If a sudden motion sends out such waves, a scorpion can tell the distance of the beetle from the difference %t in the arrival times of the waves at its leg nearest the beetle. If %t # 4.0 ms, what is the beetle’s distance?

vl

vl

d vt

vt

Beetle

•5 A sinusoidal wave travels along Figure 16-30 Problem 4. a string. The time for a particular point to move from maximum displacement to zero is 0.170 s. What are the (a) period and (b) frequency? (c) The wavelength is 1.40 m; what is the wave speed? Slope

••6 A sinusoidal wave travels 0.2 along a string under tension. Figure 16-31 gives the slopes x (m) 0 0 xs along the string at time t # 0. The scale of the x axis is set by xs # –0.2 0.80 m. What is the amplitude of Figure 16-31 Problem 6. the wave? ••7 A transverse sinusoidal wave is moving along a string in the positive direction of an x axis with a speed of 80 m/s. At t # 0, the string particle at x # 0 has a transverse displacement of 4.0 cm from its equilibrium position and is not moving. The maximum

transverse speed of the string particle at x # 0 is 16 m/s. (a) What is the frequency of the wave? (b) What is the wavelength of the wave? If y(x, t) # ym sin(kx ! vt " f) is the form of the wave equation, what are (c) ym, (d) k, (e) v, (f) f, and (g) the correct choice of sign in front of v? u (m/s)

••8 Figure 16-32 shows the transus verse velocity u versus time t of the point on a string at x # 0, as a wave t passes through it. The scale on the vertical axis is set by us # 4.0 m/s. The wave has the generic form y(x, t) # –us ym sin(kx $ vt " f). What then is f? (Caution: A calculator does not always Figure 16-32 Problem 8. give the proper inverse trig function, so check your answer by substituting it and an assumed value of v into y(x, t) and then plotting the function.) ••9 A sinusoidal wave movd ing along a string is shown y A twice in Fig. 16-33, as crest A travels in the positive direction of an x axis by distance x H d # 6.0 cm in 4.0 ms. The tick marks along the axis are separated by 10 cm; height H # 6.00 mm. The equation Figure 16-33 Problem 9. for the wave is in the form y(x, t) # ym sin(kx ! vt), so what are (a) ym, (b) k, (c) v, and (d) the correct choice of sign in front of v? ••10 The equation of a transverse wave traveling along a very long string is y # 6.0 sin(0.020px " 4.0pt), where x and y are expressed in centimeters and t is in seconds. Determine (a) the amplitude, (b) the wavelength, (c) the frequency, (d) the speed, (e) the direction of propagation of the wave, and (f) the maximum transverse speed of a particle in the string. (g) What is the transverse displacement at x # 3.5 cm when t # 0.26 s? y (cm) y

s ••11 A sinusoidal transverse wave t (s) of wavelength 20 cm travels along a 0 10 string in the positive direction of an –ys x axis. The displacement y of the Figure 16-34 Problem 11. string particle at x # 0 is given in Fig. 16-34 as a function of time t. The scale of the vertical axis is set by ys # 4.0 cm. The wave equation is to be in the form y(x, t) # ym sin(kx ! vt " f). (a) At t # 0, is a plot of y versus x in the shape of a positive sine function or a negative sine function? What are (b) ym, (c) k, (d) v, (e) f, (f) the sign in front of v, and (g) the speed of the wave? (h) What is the transverse velocity of the particle at x # 0 when t # 5.0 s?

••12 The function y(x, t) # (15.0 cm) cos(px $ 15pt), with x in meters and t in seconds, describes a wave on a taut string. What is

473

PROB LE M S

Module 16-2 Wave Speed on a Stretched String •14 The equation of a transverse wave on a string is y # (2.0 mm) sin[(20 m$1)x $ (600 s$1)t]. The tension in the string is 15 N. (a) What is the wave speed? (b) Find the linear density of this string in grams per meter. •15 SSM WWW A stretched string has a mass per unit length of 5.00 g/cm and a tension of 10.0 N. A sinusoidal wave on this string has an amplitude of 0.12 mm and a frequency of 100 Hz and is traveling in the negative direction of an x axis. If the wave equation is of the form y(x, t) # ym sin(kx ! vt), what are (a) ym, (b) k, (c) v, and (d) the correct choice of sign in front of v? •16 The speed of a transverse wave on a string is 170 m/s when the string tension is 120 N. To what value must the tension be changed to raise the wave speed to 180 m/s? •17 The linear density of a string is 1.6 ' 10 $4 kg/m. A transverse wave on the string is described by the equation y # (0.021 m) sin[(2.0 m )x " (30 s )t]. $1

$1

What are (a) the wave speed and (b) the tension in the string? •18 The heaviest and lightest strings on a certain violin have linear densities of 3.0 and 0.29 g/m. What is the ratio of the diameter of the heaviest string to that of the lightest string, assuming that the strings are of the same material? •19 SSM What is the speed of a transverse wave in a rope of length 2.00 m and mass 60.0 g under a tension of 500 N? •20 The tension in a wire clamped at both ends is doubled without appreciably changing the wire’s length between the clamps. What is the ratio of the new to the old wave speed for transverse waves traveling along this wire? ••21 ILW A 100 g wire is held under a tension of 250 N with one end at x # 0 and the other at x # 10.0 m. At time t # 0, pulse 1 is sent along the wire from the end at x # 10.0 m. At time t # 30.0 ms, pulse 2 is sent along the wire from the end at x # 0. At what position x do the pulses begin to meet? ••22 A sinusoidal wave is traveling on a string with speed 40 cm/s. The displacement of the particles of the string at x # 10 cm varies with time according to y # (5.0 cm) sin[1.0 $ (4.0 s$1)t]. The linear density of the string is 4.0 g/cm. What y (cm) are (a) the frequency and (b) the wavelength of the wave? If the wave ys equation is of the form y(x, t) # ym sin(kx ! vt), what are (c) ym, (d) k, (e) v, and (f ) the correct choice of 0 sign in front of v? (g) What is the ten20 40 sion in the string? ••23 SSM ILW A sinusoidal trans- –ys verse wave is traveling along a string in x (cm) the negative direction of an x axis. Figure 16-35 Problem 23. Figure 16-35 shows a plot of the dis-

String 2

•••24 In Fig. 16-36a, string 1 has a linear density of 3.00 g/m, and string 2 has a linear density of 5.00 g/m. They are under tension due to the hanging block of mass M # 500 g. Calculate the wave speed on (a) string 1 and (b) string 2. (Hint: When a string loops halfway around a pulley, it pulls on the pulley with a net force that is twice the tension in the string.) Next the block is divided into two blocks (with M1 " M2 # M) and the apparatus is rearranged as shown in Fig. 16-36b. Find (c) M1 and (d) M2 such that the wave speeds in the two strings are equal.

Knot M (a) String 1 String 2

M1

M2

•••25 A uniform rope of mass m (b) and length L hangs from a ceiling. (a) Show that the speed of a transFigure 16-36 Problem 24. verse wave on the rope is a function of y, the distance from the lower end, and is given by v # 1gy. (b) Show that the time a transverse wave takes to travel the length of the rope is given by t # 2 1L/g. Module 16-3 Energy and Power of a Wave Traveling Along a String •26 A string along which waves can travel is 2.70 m long and has a mass of 260 g. The tension in the string is 36.0 N. What must be the frequency of traveling waves of amplitude 7.70 mm for the average power to be 85.0 W?

••27 A sinusoidal wave is sent along a string with a linear density of 2.0 g/m. As it travels, the kinetic energies of the mass elements along the string vary. Figure 16-37a gives the rate dK/dt at which kinetic energy passes through the string elements at a particular instant, plotted as a function of distance x along the string. Figure 16-37b is similar except that it gives the rate at which kinetic energy passes through a particular mass element (at a particular location), plotted as a function of time t. For both figures, the scale on the vertical (rate) axis is set by Rs # 10 W. What is the amplitude of the wave? Rs

Rs dK/dt (W)

••13 ILW A sinusoidal wave of frequency 500 Hz has a speed of 350 m/s. (a) How far apart are two points that differ in phase by p/3 rad? (b) What is the phase difference between two displacements at a certain point at times 1.00 ms apart?

placement as a function of position at time t # 0; the scale of the y axis is set by ys # 4.0 cm. The string tension is 3.6 N, and its linear density is 25 g/m. Find the (a) amplitude, (b) wavelength, (c) wave speed, and (d) period of the wave. (e) Find the maximum transverse speed of a particle in the string. If the wave is of the form y(x, t) # ym sin(kx ! vt " f), what are (f ) k, (g) v, (h) f, and (i) the correct choice of sign in front of v? String 1

dK/dt (W)

the transverse speed for a point on the string at an instant when that point has the displacement y # "12.0 cm?

0

0.1 x (m) (a)

0.2

0

Figure 16-37 Problem 27.

1 t (ms) (b)

2

474

CHAPTE R 16 WAVES—I

Module 16-4 The Wave Equation •28 Use the wave equation to find the speed of a wave given by y(x, t) # (3.00 mm) sin[(4.00 m$1)x $ (7.00 s$1)t]. ••29

Use the wave equation to find the speed of a wave given by 0.5

y(x, t) # (2.00 mm)[(20 m )x $ (4.0 s )t] . $1

$1

•••30 Use the wave equation to find the speed of a wave given in terms of the general function h(x, t): y(x, t) # (4.00 mm) h[(30 m$1)x " (6.0 s$1)t]. Module 16-5 Interference of Waves •31 SSM Two identical traveling waves, moving in the same direction, are out of phase by p/2 rad. What is the amplitude of the resultant wave in terms of the common amplitude ym of the two combining waves? •32 What phase difference between two identical traveling waves, moving in the same direction along a stretched string, results in the combined wave having an amplitude 1.50 times that of the common amplitude of the two combining waves? Express your answer in (a) degrees, (b) radians, and (c) wavelengths. y ••33 Two sinusoidal waves with the same amplitude of 9.00 mm and the same wavelength travel together x along a string that is stretched along H an x axis. Their resultant wave is shown twice in Fig. 16-38, as valley A A travels in the negative direction of d the x axis by distance d # 56.0 cm in 8.0 ms. The tick marks along the axis Figure 16-38 Problem 33. are separated by 10 cm, and height H is 8.0 mm. Let the equation for one wave be of the form y(x, t) # ym sin(kx ! vt " f1), where f1 # 0 and you must choose the correct sign in front of v. For the equation for the other wave, what are (a) ym, (b) k, (c) v, (d) f2, and (e) the sign in front of v?

•••34 A sinusoidal wave of angular frequency 1200 rad/s and amplitude 3.00 mm is sent along a cord with linear density 2.00 g/m and tension 1200 N. (a) What is the average rate at which energy is transported by the wave to the opposite end of the cord? (b) If, simultaneously, an identical wave travels along an adjacent, identical cord, what is the total average rate at which energy is transported to the opposite ends of the two cords by the waves? If, instead, those two waves are sent along the same cord simultaneously, what is the total average rate at which they transport energy when their phase difference is (c) 0, (d) 0.4p rad, and (e) p rad? Module 16-6 Phasors •35 SSM Two sinusoidal waves of the same frequency travel in the same direction along a string. If ym1 # 3.0 cm, ym2 # 4.0 cm, f1 # 0, and f2 # p/2 rad, what is the amplitude of the resultant wave? ••36 Four waves are to be sent along the same string, in the same direction: y1(x, t) # (4.00 mm) sin(2px $ 400pt) y2(x, t) # (4.00 mm) sin(2px $ 400pt " 0.7p) y3(x, t) # (4.00 mm) sin(2px $ 400pt " p) y4(x, t) # (4.00 mm) sin(2px $ 400pt " 1.7p). What is the amplitude of the resultant wave?

••37

These two waves travel along the same string: y1(x, t) # (4.60 mm) sin(2px $ 400pt) y2(x, t) # (5.60 mm) sin(2px $ 400pt " 0.80p rad).

What are (a) the amplitude and (b) the phase angle (relative to wave 1) of the resultant wave? (c) If a third wave of amplitude 5.00 mm is also to be sent along the string in the same direction as the first two waves, what should be its phase angle in order to maximize the amplitude of the new resultant wave? ••38 Two sinusoidal waves of the same frequency are to be sent in the same direction along a taut string. One wave has an amplitude of 5.0 mm, the other 8.0 mm. (a) What phase difference f1 between the two waves results in the smallest amplitude of the resultant wave? (b) What is that smallest amplitude? (c) What phase difference f 2 results in the largest amplitude of the resultant wave? (d) What is that largest amplitude? (e) What is the resultant amplitude if the phase angle is (f1 $ f2)/2? ••39 Two sinusoidal waves of the same period, with amplitudes of 5.0 and 7.0 mm, travel in the same direction along a stretched string; they produce a resultant wave with an amplitude of 9.0 mm. The phase constant of the 5.0 mm wave is 0. What is the phase constant of the 7.0 mm wave? Module 16-7 Standing Waves and Resonance •40 Two sinusoidal waves with identical wavelengths and amplitudes travel in opposite directions along a string with a speed of 10 cm/s. If the time interval between instants when the string is flat is 0.50 s, what is the wavelength of the waves? •41 SSM A string fixed at both ends is 8.40 m long and has a mass of 0.120 kg. It is subjected to a tension of 96.0 N and set oscillating. (a) What is the speed of the waves on the string? (b) What is the longest possible wavelength for a standing wave? (c) Give the frequency of that wave. •42 A string under tension ti oscillates in the third harmonic at frequency f3, and the waves on the string have wavelength l3. If the tension is increased to tf # 4ti and the string is again made to oscillate in the third harmonic, what then are (a) the frequency of oscillation in terms of f3 and (b) the wavelength of the waves in terms of l3? •43 SSM WWW What are (a) the lowest frequency, (b) the second lowest frequency, and (c) the third lowest frequency for standing waves on a wire that is 10.0 m long, has a mass of 100 g, and is stretched under a tension of 250 N? •44 A 125 cm length of string has mass 2.00 g and tension 7.00 N. (a) What is the wave speed for this string? (b) What is the lowest resonant frequency of this string? •45 SSM ILW A string that is stretched between fixed supports separated by 75.0 cm has resonant frequencies of 420 and 315 Hz, with no intermediate resonant frequencies. What are (a) the lowest resonant frequency and (b) the wave speed? •46 String A is stretched between two clamps separated by distance L. String B, with the same linear density and under the same tension as string A, is stretched between two clamps separated by distance 4L. Consider the first eight harmonics of string B. For which of these eight harmonics of B (if any) does the frequency match the frequency of (a) A’s first harmonic, (b) A’s second harmonic, and (c) A’s third harmonic? •47 One of the harmonic frequencies for a particular string under tension is 325 Hz. The next higher harmonic frequency is 390 Hz.

475

PROB LE M S

What harmonic frequency is next higher after the harmonic frequency 195 Hz? •48 If a transmission line in a cold climate collects ice, the increased diameter tends to cause vortex formation in a passing wind. The air pressure variations in the vortexes tend to cause the line to oscillate (gallop), especially if the frequency of the variations matches a resonant frequency of the line. In long lines, the resonant frequencies are so close that almost any wind speed can set up a resonant mode vigorous enough to pull down support towers or cause the line to short out with an adjacent line. If a transmission line has a length of 347 m, a linear density of 3.35 kg/m, and a tension of 65.2 MN, what are (a) the frequency of the fundamental mode and (b) the frequency difference between successive modes? •49 ILW A nylon guitar string has a D linear density of 7.20 g/m and is under a tension of 150 N.The fixed supports are distance D # 90.0 cm apart. The string Figure 16-39 Problem 49. is oscillating in the standing wave pattern shown in Fig. 16-39. Calculate the (a) speed, (b) wavelength, and (c) frequency of the traveling waves whose superposition gives this standing wave.

y (cm)

ys ••50 For a particular transverse standing wave on a long string, one of the antinodes is at x # 0 and an adjacent node is at x # 0.10 m. The t (s) 0 0.5 1.5 2.0 displacement y(t) of the string particle at x # 0 is shown in Fig. 16-40, where the scale of the y axis is set by –ys ys # 4.0 cm. When t # 0.50 s, what is the displacement of the string particle Figure 16-40 Problem 50. at (a) x # 0.20 m and (b) x # 0.30 m? What is the transverse velocity of the string particle at x # 0.20 m at (c) t # 0.50 s and (d) t # 1.0 s? (e) Sketch the standing wave at t # 0.50 s for the range x # 0 to x # 0.40 m.

••51 SSM WWW Two waves are generated on a string of length 3.0 m to produce a three-loop standing wave with an amplitude of 1.0 cm. The wave speed is 100 m/s. Let the equation for one of the waves be of the form y(x, t) # ym sin(kx " vt). In the equation for the other wave, what are (a) ym, (b) k, (c) v, and (d) the sign in front of v? ••52 A rope, under a tension of 200 N and fixed at both ends, oscillates in a second-harmonic standing wave pattern. The displacement of the rope is given by y # (0.10 m)(sin px/2) sin 12pt, where x # 0 at one end of the rope, x is in meters, and t is in seconds. What are (a) the length of the rope, (b) the speed of the waves on the rope, and (c) the mass of the rope? (d) If the rope oscillates in a third-harmonic standing wave pattern, what will be the period of oscillation? ••53

A string oscillates according to the equation

(# p3 cm $x ) cos[(40p s

y, # (0.50 cm) sin

$1

)t].

$1

What are the (a) amplitude and (b) speed of the two waves (identical except for direction of travel) whose superposition gives this oscillation? (c) What is the distance between nodes? (d) What is the transverse speed of a particle of the string at the position x # 1.5 cm when t # 98 s?

A y ••54 Two sinusoidal waves with the same amplitude and wavelength travel through each other along a string that is H x stretched along an x axis. Their resultant wave is shown twice in Fig. 16-41, as the antinode A A travels from an extreme upward displacement to an exFigure 16-41 Problem 54. treme downward displacement in 6.0 ms. The tick marks along the axis are separated by 10 cm; height H is 1.80 cm. Let the equation for one of the two waves be of the form y(x, t) # ym sin(kx " vt). In the equation for the other wave, what are (a) ym, (b) k, (c) v, and (d) the sign in front of v?

••55 The following two waves are sent in opposite directions on a horizontal string so as to create a standing wave in a vertical plane: y1(x, t) # (6.00 mm) sin(4.00px $ 400pt) y2(x, t) # (6.00 mm) sin(4.00px " 400pt), with x in meters and t in seconds. An antinode is located at point A. In the time interval that point takes to move from maximum upward displacement to maximum downward displacement, how far does each wave move along the string? ••56

A standing wave pattern on a string is described by y(x, t) # 0.040 (sin 5px)(cos 40pt),

where x and y are in meters and t is in seconds. For x 0 0, what is the location of the node with the (a) smallest, (b) second smallest, and (c) third smallest value of x? (d) What is the period of the oscillatory motion of any (nonnode) point? What are the (e) speed and (f ) amplitude of the two traveling waves that interfere to produce this wave? For t 0 0, what are the (g) first, (h) second, and (i) third time that all points on the string have zero transverse velocity? ••57 A generator at one end of a very long string creates a wave given by p [(2.00 m$1)x " (8.00 s$1)t], y # (6.0 cm) cos 2 and a generator at the other end creates the wave p y # (6.0 cm) cos [(2.00 m$1)x $ (8.00 s$1)t]. 2 Calculate the (a) frequency, (b) wavelength, and (c) speed of each wave. For x 0 0, what is the location of the node having the (d) smallest, (e) second smallest, and (f) third smallest value of x? For x 0 0, what is the location of the antinode having the (g) smallest, (h) second smallest, and (i) third smallest value of x? ••58 In Fig. 16-42, a string, tied to a sinusoidal oscillator at P and running over a support at Q, is stretched by a block of mass m. Separation L # 1.20 m, linear density m # 1.6 g/m, and the oscillator Oscillator

P

Q L m

Figure 16-42 Problems 58 and 60.

CHAPTE R 16 WAVES—I

frequency f # 120 Hz. The amplitude of the motion at P is small enough for that point to be considered a node. A node also exists at Q. (a) What mass m allows the oscillator to set up the fourth harmonic on the string? (b) What standing wave mode, if any, can be set up if m # 1.00 kg? L1

L2

•••59 In Fig. 16-43, Steel Aluminum an aluminum wire, of length L1 # 60.0 cm, cross-sectional area 1.00 m ' 10 $2 cm2, and density 2.60 g/cm3, is joined to a steel wire, of density Figure 16-43 Problem 59. 7.80 g/cm3 and the same cross-sectional area. The compound wire, loaded with a block of mass m # 10.0 kg, is arranged so that the distance L2 from the joint to the supporting pulley is 86.6 cm. Transverse waves are set up on the wire by an external source of variable frequency; a node is located at the pulley. (a) Find the lowest frequency that generates a standing wave having the joint as one of the nodes. (b) How many nodes are observed at this frequency? •••60 In Fig. 16-42, a string, tied to a sinusoidal oscillator at P and running over a support at Q, is stretched by a block of mass m. The separation L between P and Q is 1.20 m, and the frequency f of the oscillator is fixed at 120 Hz. The amplitude of the motion at P is small enough for that point to be considered a node. A node also exists at Q. A standing wave appears when the mass of the hanging block is 286.1 g or 447.0 g, but not for any intermediate mass. What is the linear density of the string? Additional Problems 61 In an experiment on standing waves, a string 90 cm long is attached to the prong of an electrically driven tuning fork that oscillates perpendicular to the length of the string at a frequency of 60 Hz. The mass of the string is 0.044 kg. What tension must the string be under (weights are attached to the other end) if it is to oscillate in four loops? 62 A sinusoidal transverse wave traveling in the positive direction of an x axis has an amplitude of 2.0 cm, a wavelength of 10 cm, and a frequency of 400 Hz. If the wave equation is of the form y(x, t) # ym sin(kx ! vt), what are (a) ym, (b) k, (c) v, and (d) the correct choice of sign in front of v? What are (e) the maximum transverse speed of a point on the cord and (f) the speed of the wave? 63 A wave has a speed of 240 m/s and a wavelength of 3.2 m. What are the (a) frequency and (b) period of the wave? 64 The equation of a transverse wave traveling along a string is y # 0.15 sin(0.79x $ 13t), in which x and y are in meters and t is in seconds. (a) What is the displacement y at x # 2.3 m, t # 0.16 s? A second wave is to be added to the first wave to produce standing waves on the string. If the second wave is of the form y(x, t) # ym sin(kx ! vt), what are (b) ym, (c) k, (d) v, and (e) the correct choice of sign in front of v for this second wave? (f) What is the displacement of the resultant standing wave at x # 2.3 m, t # 0.16 s? 65

The equation of a transverse wave traveling along a string is y # (2.0 mm) sin[(20 m$1)x $ (600 s$1)t].

Find the (a) amplitude, (b) frequency, (c) velocity (including

sign), and (d) wavelength of the wave. (e) Find the maximum transverse speed of a particle in the string. 66 Figure 16-44 shows the disy (mm) ys placement y versus time t of the point on a string at x # 0, as a wave passes through that point. t The scale of the y axis is set by ys # 6.0 mm. The wave is given –ys by y(x, t) # ym sin(kx $ vt " f). What is f? (Caution: A calculator Figure 16-44 Problem 66. does not always give the proper inverse trig function, so check your answer by substituting it and an assumed value of v into y(x, t) and then plotting the function.) 67 Two sinusoidal waves, identical except for phase, travel in the same direction along a string, producing the net wave y,(x, t) # (3.0 mm) sin(20x $ 4.0t " 0.820 rad), with x in meters and t in seconds. What are (a) the wavelength l of the two waves, (b) the phase difference between them, and (c) their amplitude ym? 68 A single pulse, given by h(x $ 5.0t), is shown in Fig. 16-45 for t # 0. The scale of the vertical hs axis is set by hs # 2. Here x is in t=0 centimeters and t is in seconds. What are the (a) speed and (b) di0 rection of travel of the pulse? (c) 0 1 2 3 4 5 Plot h(x $ 5t) as a function of x for x t # 2 s. (d) Plot h(x $ 5t) as a funcFigure 16-45 Problem 68. tion of t for x # 10 cm. h(x)

476

69 SSM Three sinusoidal waves of the same frequency travel along a string in the positive direction of an x axis. Their amplitudes are y1, y1/2, and y1/3, and their phase constants are 0, p/2, and p, respectively. What are the (a) amplitude and (b) phase constant of the resultant wave? (c) Plot the wave form of the resultant wave at t # 0, and discuss its behavior as t increases. 2

ay (m/s ) 70 Figure 16-46 shows as transverse acceleration ay versus time t of the point on a string at x # 0, as a wave in the form of t y(x, t) # y m sin(kx $ vt " f) passes through that point. The –as scale of the vertical axis is set 2 by as # 400 m/s . What is f? Figure 16-46 Problem 70. (Caution: A calculator does not always give the proper inverse trig function, so check your answer by substituting it and an assumed value of v into y(x, t) and then plotting the function.)

71 A transverse sinusoidal wave is generated at one end of a long, horizontal string by a bar that moves up and down through a distance of 1.00 cm. The motion is continuous and is repeated regularly 120 times per second. The string has linear density 120 g/m and is kept under a tension of 90.0 N. Find the maximum value of (a) the transverse speed u and (b) the transverse component of the tension t. (c) Show that the two maximum values calculated above occur at the same phase values for the wave. What is the transverse displacement y of the string at these phases? (d) What is the maximum rate of energy transfer along the string? (e) What is the transverse displacement y when this maximum transfer occurs? (f ) What is the minimum rate of energy transfer along the

PROB LE M S

string? (g) What is the transverse displacement y when this minimum transfer occurs? y's

y' (mm)

72 Two sinusoidal 120 Hz waves, of the same frequency and amplitude, are to be sent in 0 the positive direction of an x axis 10 20 that is directed along a cord un–y's der tension. The waves can be Shift distance (cm) sent in phase, or they can be phase-shifted. Figure 16-47 Figure 16-47 Problem 72. shows the amplitude y, of the resulting wave versus the distance of the shift (how far one wave is shifted from the other wave). The scale of the vertical axis is set by y,s # 6.0 mm. If the equations for the two waves are of the form y(x, t) # ym sin(kx ! vt), what are (a) ym, (b) k, (c) v, and (d) the correct choice of sign in front of v? 73 At time t # 0 and at position x # 0 m along a string, a traveling sinusoidal wave with an angular frequency of 440 rad/s has displacement y # "4.5 mm and transverse velocity u # $0.75 m/s. If the wave has the general form y(x, t) # ym sin(kx $ vt " f), what is phase constant f? 74 Energy is transmitted at rate P1 by a wave of frequency f1 on a string under tension t1. What is the new energy transmission rate P2 in terms of P1 (a) if the tension is increased to t2 # 4t1 and (b) if, instead, the frequency is decreased to f2 # f1/2? 75 (a) What is the fastest transverse wave that can be sent along a steel wire? For safety reasons, the maximum tensile stress to which steel wires should be subjected is 7.00 ' 10 8 N/m2. The density of steel is 7800 kg/m3. (b) Does your answer depend on the diameter of the wire? 76 A standing wave results from the sum of two transverse traveling waves given by y1 # 0.050 cos(px $ 4pt) and

y2 # 0.050 cos(px " 4pt),

where x, y1, and y2 are in meters and t is in seconds. (a) What is the smallest positive value of x that corresponds to a node? Beginning at t # 0, what is the value of the (b) first, (c) second, and (d) third time the particle at x # 0 has zero velocity? 77 SSM The type of rubber band used inside some baseballs and golf balls obeys Hooke’s law over a wide range of elongation of the band. A segment of this material has an unstretched length ! and a mass m. When a force F is applied, the band stretches an additional length %!. (a) What is the speed (in terms of m, %!, and the spring constant k) of transverse waves on this stretched rubber band? (b) Using your answer to (a), show that the time required for a transverse pulse to travel the length of the rubber band is proportional to 1/1%! if %! + ! and is constant if %! 1 ! . 78 The speed of electromagnetic waves (which include visible light, radio, and x rays) in vacuum is 3.0 ' 10 8 m/s. (a) Wavelengths of visible light waves range from about 400 nm in the violet to about 700 nm in the red. What is the range of frequencies of these waves? (b) The range of frequencies for shortwave radio (for example, FM radio and VHF television) is 1.5 to 300 MHz. What is the corresponding wavelength range? (c) X-ray wavelengths range from about 5.0 nm to about 1.0 ' 10 $2 nm. What is the frequency range for x rays?

477

79 SSM A 1.50 m wire has a mass of 8.70 g and is under a tension of 120 N. The wire is held rigidly at both ends and set into oscillation. (a) What is the speed of waves on the wire? What is the wavelength of the waves that produce (b) one-loop and (c) twoloop standing waves? What is the frequency of the waves that produce (d) one-loop and (e) two-loop standing waves? 80 When played in a certain manner, the lowest resonant frequency of a certain violin string is concert A (440 Hz). What is the frequency of the (a) second and (b) third harmonic of the string? 81 A sinusoidal transverse wave traveling in the negative direction of an x axis has an amplitude of 1.00 cm, a frequency of 550 Hz, and a speed of 330 m/s. If the wave equation is of the form y(x, t) # ym sin(kx ! vt), what are (a) ym, (b) v, (c) k, and (d) the correct choice of sign in front of v? 82 Two sinusoidal waves of the same wavelength travel in the same direction along a stretched string. For wave 1, ym # 3.0 mm and f # 0; for wave 2, ym # 5.0 mm and f # 70-. What are the (a) amplitude and (b) phase constant of the resultant wave? 83 SSM A sinusoidal transverse wave of amplitude ym and wavelength l travels on a stretched cord. (a) Find the ratio of the maximum particle speed (the speed with which a single particle in the cord moves transverse to the wave) to the wave speed. (b) Does this ratio depend on the material of which the cord is made? 84 Oscillation of a 600 Hz tuning fork sets up standing waves in a string clamped at both ends. The wave speed for the string is 400 m/s. The standing wave has four loops and an amplitude of 2.0 mm. (a) What is the length of the string? (b) Write an equation for the displacement of the string as a function of position and time. 85 A 120 cm length of string is stretched between fixed supports. What are the (a) longest, (b) second longest, and (c) third longest wavelength for waves traveling on the string if standing waves are to be set up? (d) Sketch those standing waves. 86 (a) Write an equation describing a sinusoidal transverse wave traveling on a cord in the positive direction of a y axis with an angular wave number of 60 cm$1, a period of 0.20 s, and an amplitude of 3.0 mm. Take the transverse direction to be the z direction. (b) What is the maximum transverse speed of a point on the cord? 87 A wave on a string is described by y(x, t) # 15.0 sin(px/8 $ 4pt), where x and y are in centimeters and t is in seconds. (a) What is the transverse speed for a point on the string at x # 6.00 cm when t # 0.250 s? (b) What is the maximum transverse speed of any point on the string? (c) What is the magnitude of the transverse acceleration for a point on the string at x # 6.00 cm when t # 0.250 s? (d) What is the magnitude of the maximum transverse acceleration for any point on the string? 88 Body armor. When a high-speed projectile such as a bullet or bomb fragment strikes modern body armor, the fabric of the armor stops the projectile and prevents penetration by quickly spreading the projectile’s energy over a large area. This spreading is done by longitudinal and transverse pulses that move radially from the impact point, where the projectile pushes a cone-shaped dent into the fabric. The longitudinal pulse, racing along the fibers of the fabric at speed vl ahead of the denting, causes the fibers to thin and stretch, with material flowing radially inward into the dent. One such radial fiber is shown in Fig. 16-48a. Part of the projectile’s energy goes into this motion and stretching. The transverse

478

CHAPTE R 16 WAVES—I

pulse, moving at a slower speed vt , is due to the denting. As the projectile increases the dent’s depth, the dent increases in radius, causing the material in the fibers to move in the same direction as the projectile (perpendicular to the transverse pulse’s direction of travel). The rest of the projectile’s energy goes into this motion. All the energy that does not eventually go into permanently deforming the fibers ends up as thermal energy. Figure 16-48b is a graph of speed v versus time t for a bullet of mass 10.2 g fired from a .38 Special revolver directly into body armor. The scales of the vertical and horizontal axes are set by vs # 300 m/s and ts # 40.0 ms. Take vl # 2000 m/s, and assume that the half-angle u of the conical dent is 60-. At the end of the collision, what are the radii of (a) the thinned region and (b) the dent (assuming that the person wearing the armor remains stationary)? v

Radius reached by longitudinal pulse

Radius reached by transverse pulse

θ

vl

vt

Bullet

vt

vl

92

Two waves, y1 # (2.50 mm) sin[(25.1 rad/m)x $ (440 rad/s)t]

and y2 # (1.50 mm) sin[(25.1 rad/m)x " (440 rad/s)t], travel along a stretched string. (a) Plot the resultant wave as a function of t for x # 0, l/8, l/4, 3l/8, and l/2, where l is the wavelength. The graphs should extend from t # 0 to a little over one period. (b) The resultant wave is the superposition of a standing wave and a traveling wave. In which direction does the traveling wave move? (c) How can you change the original waves so the resultant wave is the superposition of standing and traveling waves with the same amplitudes as before but with the traveling wave moving in the opposite direction? Next, use your graphs to find the place at which the oscillation amplitude is (d) maximum and (e) minimum. (f) How is the maximum amplitude related to the amplitudes of the original two waves? (g) How is the minimum amplitude related to the amplitudes of the original two waves? 93

A traveling wave on a string is described by

( # 0.40t " 80x $ ),

(a)

y # 2.0 sin 2p

vs

v (m/s)

where x and y are in centimeters and t is in seconds. (a) For t # 0, plot y as a function of x for 0 2 x 2 160 cm. (b) Repeat (a) for t # 0.05 s and t # 0.10 s. From your graphs, determine (c) the wave speed and (d) the direction in which the wave is traveling.

0 t (µ s) (b)

ts

Figure 16-48 Problem 88. 89 Two waves are described by y1 # 0.30 sin[p (5x $ 200t)] and

moving upward in the positive direction of a y axis with a transverse velocity of 5.0 m/s. What are (a) the amplitude of the motion of that point and (b) the tension in the rope? (c) Write the standing wave equation for the fundamental mode.

y2 # 0.30 sin[p (5x $ 200t) " p/3],

where y1, y2, and x are in meters and t is in seconds. When these two waves are combined, a traveling wave is produced. What are the (a) amplitude, (b) wave speed, and (c) wavelength of that traveling wave? u (cm/s)

90 A certain transverse sinuus soidal wave of wavelength 20 cm t (s) 0 is moving in the positive direc1 2 3 4 tion of an x axis. The transverse –us velocity of the particle at x # 0 as a function of time is shown in Figure 16-49 Problem 90. Fig. 16-49, where the scale of the vertical axis is set by us # 5.0 cm/s. What are the (a) wave speed, (b) amplitude, and (c) frequency? (d) Sketch the wave between x # 0 and x # 20 cm at t # 2.0 s. 91 SSM In a demonstration, a 1.2 kg horizontal rope is fixed in place at its two ends (x # 0 and x # 2.0 m) and made to oscillate up and down in the fundamental mode, at frequency 5.0 Hz. At t # 0, the point at x # 1.0 m has zero displacement and is

94 In Fig. 16-50, a circular loop of string is set spinning about the center point in a place with negligible gravity. The radius is 4.00 cm and the tangential speed of a string segment is 5.00 cm/s. The string is plucked. At what speed do transverse waves move along the string? (Hint: Apply Newton’s second law to a small, but finite, section of the string.)

Figure 16-50 Problem 94.

95 A continuous traveling wave with amplitude A is incident on a boundary. The continuous reflection, with a smaller amplitude B, travels back through the incoming wave. The resulting interference pattern is displayed in Fig. 16-51. The standing wave ratio is defined to be A"B SWR # . A$B The reflection coefficient R is the ratio of the power of the reflected wave to the Amax Amax Amin power of the incoming wave and is thus proportional to the ratio (B/A)2. What is the Figure 16-51 Problem 95. SWR for (a) total reflection and (b) no reflection? (c) For SWR # 1.50, what is R expressed as a percentage? 96 Consider a loop in the standing wave created by two waves (amplitude 5.00 mm and frequency 120 Hz) traveling in opposite directions along a string with length 2.25 m and mass 125 g and under tension 40 N. At what rate does energy enter the loop from (a) each side and (b) both sides? (c) What is the maximum kinetic energy of the string in the loop during its oscillation?

C

H

A

P

T

E

R

1

7

Waves—II 17-1 SPEED OF SOUND Learning Objectives After reading this module, you should be able to . . .

17.01 Distinguish between a longitudinal wave and a transverse wave. 17.02 Explain wavefronts and rays. 17.03 Apply the relationship between the speed of sound

through a material, the material’s bulk modulus, and the material’s density. 17.04 Apply the relationship between the speed of sound, the distance traveled by a sound wave, and the time required to travel that distance.

Key Idea ● Sound waves are longitudinal mechanical waves that can travel through solids, liquids, or gases. The speed v of a sound wave in a medium having bulk modulus B and density r is

B (speed of sound). A 3 In air at 20-C, the speed of sound is 343 m/s. v#

What Is Physics? The physics of sound waves is the basis of countless studies in the research journals of many fields. Here are just a few examples. Some physiologists are concerned with how speech is produced, how speech impairment might be corrected, how hearing loss can be alleviated, and even how snoring is produced. Some acoustic engineers are concerned with improving the acoustics of cathedrals and concert halls, with reducing noise near freeways and road construction, and with reproducing music by speaker systems. Some aviation engineers are concerned with the shock waves produced by supersonic aircraft and the aircraft noise produced in communities near an airport. Some medical researchers are concerned with how noises produced by the heart and lungs can signal a medical problem in a patient. Some paleontologists are concerned with how a dinosaur’s fossil might reveal the dinosaur’s vocalizations. Some military engineers are concerned with how the sounds of sniper fire might allow a soldier to pinpoint the sniper’s location, and, on the gentler side, some biologists are concerned with how a cat purrs. To begin our discussion of the physics of sound, we must first answer the question “What are sound waves?”

Sound Waves As we saw in Chapter 16, mechanical waves are waves that require a material medium to exist. There are two types of mechanical waves: Transverse waves involve oscillations perpendicular to the direction in which the wave travels; longitudinal waves involve oscillations parallel to the direction of wave travel. In this book, a sound wave is defined roughly as any longitudinal wave. Seismic prospecting teams use such waves to probe Earth’s crust for oil. Ships 479

480

CHAPTE R 17 WAVES—I I

Mauro Fermariello/SPL/Photo Researchers, Inc.

Figure 17-1 A loggerhead turtle is being checked with ultrasound (which has a frequency above your hearing range); an image of its interior is being produced on a monitor off to the right.

carry sound-ranging gear (sonar) to detect underwater obstacles. Submarines use sound waves to stalk other submarines, largely by listening for the characteristic noises produced by the propulsion system. Figure 17-1 suggests how sound waves can be used to explore the soft tissues of an animal or human body. In this chapter we shall focus on sound waves that travel through the air and that are audible to people. Figure 17-2 illustrates several ideas that we shall use in our discussions. Point S represents a tiny sound source, called a point source, that emits sound waves in all directions. The wavefronts and rays indicate the direction of travel and the spread of the sound waves. Wavefronts are surfaces over which the oscillations due to the sound wave have the same value; such surfaces are represented by whole or partial circles in a two-dimensional drawing for a point source. Rays are directed lines perpendicular to the wavefronts that indicate the direction of travel of the wavefronts. The short double arrows superimposed on the rays of Fig. 17-2 indicate that the longitudinal oscillations of the air are parallel to the rays. Near a point source like that of Fig. 17-2, the wavefronts are spherical and spread out in three dimensions, and there the waves are said to be spherical. As the wavefronts move outward and their radii become larger, their curvature decreases. Far from the source, we approximate the wavefronts as planes (or lines on two-dimensional drawings), and the waves are said to be planar.

The Speed of Sound The speed of any mechanical wave, transverse or longitudinal, depends on both an inertial property of the medium (to store kinetic energy) and an elastic property of the medium (to store potential energy). Thus, we can generalize Eq. 16-26, which gives the speed of a transverse wave along a stretched string, by writing v#

Wavefronts

Ray

Ray

Figure 17-2 A sound wave travels from a point source S through a three-dimensional medium. The wavefronts form spheres centered on S; the rays are radial to S. The short, double-headed arrows indicate that elements of the medium oscillate parallel to the rays.

(17-1)

where (for transverse waves) t is the tension in the string and m is the string’s linear density. If the medium is air and the wave is longitudinal, we can guess that the inertial property, corresponding to m, is the volume density r of air. What shall we put for the elastic property? In a stretched string, potential energy is associated with the periodic stretching of the string elements as the wave passes through them. As a sound wave passes through air, potential energy is associated with periodic compressions and expansions of small volume elements of the air. The property that determines the extent to which an element of a medium changes in volume when the pressure (force per unit area) on it changes is the bulk modulus B, defined (from Eq. 12-25) as B#$

S

elastic property t # , A inertial property Am

%p %V/V

(definition of bulk modulus).

(17-2)

Here %V/V is the fractional change in volume produced by a change in pressure %p. As explained in Module 14-1, the SI unit for pressure is the newton per square meter, which is given a special name, the pascal (Pa). From Eq. 17-2 we see that the unit for B is also the pascal. The signs of %p and %V are always opposite: When we increase the pressure on an element (%p is positive), its volume decreases (%V is negative). We include a minus sign in Eq. 17-2 so that B is always a positive quantity. Now substituting B for t and r for m in Eq. 17-1 yields v#

B A 3

(speed of sound)

(17-3)

17-1 SPE E D OF SOU N D

as the speed of sound in a medium with bulk modulus B and density r. Table 17-1 lists the speed of sound in various media. The density of water is almost 1000 times greater than the density of air. If this were the only relevant factor, we would expect from Eq. 17-3 that the speed of sound in water would be considerably less than the speed of sound in air. However, Table 17-1 shows us that the reverse is true. We conclude (again from Eq. 17-3) that the bulk modulus of water must be more than 1000 times greater than that of air. This is indeed the case. Water is much more incompressible than air, which (see Eq. 17-2) is another way of saying that its bulk modulus is much greater.

Formal Derivation of Eq. 17-3 We now derive Eq. 17-3 by direct application of Newton’s laws. Let a single pulse in which air is compressed travel (from right to left) with speed v through the air in a long tube, like that in Fig. 16-2. Let us run along with the pulse at that speed, so that the pulse appears to stand still in our reference frame. Figure 17-3a shows the situation as it is viewed from that frame. The pulse is standing still, and air is moving at speed v through it from left to right. Let the pressure of the undisturbed air be p and the pressure inside the pulse be p " %p, where %p is positive due to the compression. Consider an element of air of thickness %x and face area A, moving toward the pulse at speed v. As this element enters the pulse, the leading face of the element encounters a region of higher pressure, which slows the element to speed v " %v, in which %v is negative. This slowing is complete when the rear face of the element reaches the pulse, which requires time interval %x (17-4) . %t # v

Table 17-1 The Speed of Sounda Medium Gases Air (0-C) Air (20-C) Helium Hydrogen Liquids Water (0-C) Water (20-C) Seawaterb Solids Aluminum Steel Granite a

At 20-C and 3.5% salinity.

F # pA $ ( p " %p)A (17-5)

(net force).

The minus sign indicates that the net force on the air element is directed to the left in Fig. 17-3b. The volume of the element is A %x, so with the aid of Eq. 17-4, we can write its mass as %m # r %V # rA %x # rAv %t

(17-6)

(mass).

The average acceleration of the element during %t is a#

Moving air (fluid element)

(17-7)

(acceleration).

p + ∆ p, v + ∆v

v

A p, v

%v %t

∆x

Pulse

p, v

pA

∆x

(a)

Figure 17-3 A compression pulse is sent from right to left down a long air-filled tube. The reference frame of the figure is chosen so that the pulse is at rest and the air moves from left to right. (a) An element of air of width %x moves toward the pulse with speed v. (b) The leading face of the element enters the pulse. The forces acting on the leading and trailing faces (due to air pressure) are shown.

Speed (m/s) 331 343 965 1284 1402 1482 1522 6420 5941 6000

At 0-C and 1 atm pressure, except where noted.

b

Let us apply Newton’s second law to the element. During %t, the average force on the element’s trailing face is pA toward the right, and the average force on the leading face is ( p " %p)A toward the left (Fig. 17-3b). Therefore, the average net force on the element during %t is # $%p A

481

(p + ∆ p)A (b)

482

CHAPTE R 17 WAVES—I I

Thus, from Newton’s second law (F # ma), we have, from Eqs. 17-5, 17-6, and 17-7, %v (17-8) , $%p A # (3Av %t) %t which we can write as %p (17-9) 3v2 # $ . %v/v The air that occupies a volume V (# Av %t) outside the pulse is compressed by an amount %V (# A %v %t) as it enters the pulse. Thus, %V A %v %t %v # # . V Av %t v

(17-10)

Substituting Eq. 17-10 and then Eq. 17-2 into Eq. 17-9 leads to

3 v2 # $

%p %p #$ # B. %v/v %V/V

(17-11)

Solving for v yields Eq. 17-3 for the speed of the air toward the right in Fig. 17-3, and thus for the actual speed of the pulse toward the left.

17-2 TRAVELING SOUND WAVES Learning Objectives After reading this module, you should be able to . . .

17.05 For any particular time and position, calculate the displacement s(x, t) of an element of air as a sound wave travels through its location. 17.06 Given a displacement function s(x, t) for a sound wave, calculate the time between two given displacements. 17.07 Apply the relationships between wave speed v, angular frequency v, angular wave number k, wavelength l, period T, and frequency f. 17.08 Sketch a graph of the displacement s(x) of an element of air as a function of position, and identify the amplitude sm and wavelength l. 17.09 For any particular time and position, calculate the pres-

sure variation %p (variation from atmospheric pressure) of an element of air as a sound wave travels through its location. 17.10 Sketch a graph of the pressure variation %p(x) of an element as a function of position, and identify the amplitude %pm and wavelength l. 17.11 Apply the relationship between pressure-variation amplitude %pm and displacement amplitude sm. 17.12 Given a graph of position s versus time for a sound wave, determine the amplitude sm and the period T. 17.13 Given a graph of pressure variation %p versus time for a sound wave, determine the amplitude %pm and the period T.

Key Ideas ● A sound wave causes a longitudinal displacement s of a mass element in a medium as given by

● The sound wave also causes a pressure change %p of the medium from the equilibrium pressure:

%p # %pm sin(kx $ vt),

s # sm cos(kx $ vt), where sm is the displacement amplitude (maximum displacement) from equilibrium, k # 2p/l, and v # 2pf, l and f being the wavelength and frequency, respectively, of the sound wave.

where the pressure amplitude is %pm # (vrv)sm.

Traveling Sound Waves Here we examine the displacements and pressure variations associated with a sinusoidal sound wave traveling through air. Figure 17-4a displays such a wave traveling rightward through a long air-filled tube. Recall from Chapter 16 that we can produce such a wave by sinusoidally moving a piston at the left end of

17-2 TRAVE LI NG SOU N D WAVES

Compression

Figure 17-4 (a) A sound wave, traveling through a long air-filled tube with speed v, consists of a moving, periodic pattern of expansions and compressions of the air. The wave is shown at an arbitrary instant. (b) A horizontally expanded view of a short piece of the tube. As the wave passes, an air element of thickness %x oscillates left and right in simple harmonic motion about its equilibrium position. At the instant shown in (b), the element happens to be displaced a distance s to the right of its equilibrium position. Its maximum displacement, either right or left, is sm.

483

λ v

Expansion

(a)

∆x

The element oscillates left and right as the wave moves through it.

Oscillating fluid element

s sm

x

sm

(b)

Equilibrium position

the tube (as in Fig. 16-2). The piston’s rightward motion moves the element of air next to the piston face and compresses that air; the piston’s leftward motion allows the element of air to move back to the left and the pressure to decrease. As each element of air pushes on the next element in turn, the right – left motion of the air and the change in its pressure travel along the tube as a sound wave. Consider the thin element of air of thickness %x shown in Fig. 17-4b. As the wave travels through this portion of the tube, the element of air oscillates left and right in simple harmonic motion about its equilibrium position. Thus, the oscillations of each air element due to the traveling sound wave are like those of a string element due to a transverse wave, except that the air element oscillates longitudinally rather than transversely. Because string elements oscillate parallel to the y axis, we write their displacements in the form y(x, t). Similarly, because air elements oscillate parallel to the x axis, we could write their displacements in the confusing form x(x, t), but we shall use s(x, t) instead. Displacement. To show that the displacements s(x, t) are sinusoidal functions of x and t, we can use either a sine function or a cosine function. In this chapter we use a cosine function, writing s(x, t) # sm cos(kx $ vt).

(17-12)

Figure 17-5a labels the various parts of this equation. In it, sm is the displacement amplitude — that is, the maximum displacement of the air element to either side of its equilibrium position (see Fig. 17-4b). The angular wave number k, angular frequency v, frequency f, wavelength l, speed v, and period T for a sound (longitudinal) wave are defined and interrelated exactly as for a transverse wave, except that l is now the distance (again along the direction of travel) in which the pattern of compression and expansion due to the wave begins to repeat itself (see Fig. 17-4a). (We assume sm is much less than l.) Pressure. As the wave moves, the air pressure at any position x in Fig. 17-4a varies sinusoidally, as we prove next. To describe this variation we write %p(x, t) # %pm sin(kx $ vt).

(17-13)

Figure 17-5b labels the various parts of this equation. A negative value of %p in Eq. 17-13 corresponds to an expansion of the air, and a positive value to a compression. Here %pm is the pressure amplitude, which is the maximum increase or decrease in pressure due to the wave; %pm is normally very much less than the pressure p present when there is no wave. As we shall prove, the pressure ampli-

Displacement (a)

s(x,t) = sm cos(kx – ωt)

Displacement amplitude (b)

Oscillating term

∆p(x,t) = ∆pm sin(kx – ωt) Pressure amplitude Pressure variation

Figure 17-5 (a) The displacement function and (b) the pressure-variation function of a traveling sound wave consist of an amplitude and an oscillating term.

Displacement ( µ m)

484

CHAPTE R 17 WAVES—I I

10 0

20

tude %pm is related to the displacement amplitude sm in Eq. 17-12 by

t=0

sm 40

60

x (cm) 80

–10

Pressure variation (Pa)

(a )

30 20 10 0 –10 –20 –30

∆ pm 20

%pm # (vrv)sm .

(17-14)

Figure 17-6 shows plots of Eqs. 17-12 and 17-13 at t # 0; with time, the two curves would move rightward along the horizontal axes. Note that the displacement and pressure variation are p/2 rad (or 90-) out of phase. Thus, for example, the pressure variation %p at any point along the wave is zero when the displacement there is a maximum.

t=0 40

60

x (cm) 80

Checkpoint 1 When the oscillating air element in Fig. 17-4b is moving rightward through the point of zero displacement, is the pressure in the element at its equilibrium value, just beginning to increase, or just beginning to decrease?

(b)

Figure 17-6 (a) A plot of the displacement function (Eq. 17-12) for t # 0. (b) A similar plot of the pressure-variation function (Eq. 17-13). Both plots are for a 1000 Hz sound wave whose pressure amplitude is at the threshold of pain.

Derivation of Eqs. 17-13 and 17-14 Figure 17-4b shows an oscillating element of air of cross-sectional area A and thickness %x, with its center displaced from its equilibrium position by distance s. From Eq. 17-2 we can write, for the pressure variation in the displaced element, %p # $B

%V . V

(17-15)

The quantity V in Eq. 17-15 is the volume of the element, given by V # A %x.

(17-16)

The quantity %V in Eq. 17-15 is the change in volume that occurs when the element is displaced. This volume change comes about because the displacements of the two faces of the element are not quite the same, differing by some amount %s. Thus, we can write the change in volume as %V # A %s.

(17-17)

Substituting Eqs. 17-16 and 17-17 into Eq. 17-15 and passing to the differential limit yield &s %s (17-18) # $B . %p # $B %x &x The symbols & indicate that the derivative in Eq. 17-18 is a partial derivative, which tells us how s changes with x when the time t is fixed. From Eq. 17-12 we then have, treating t as a constant, & &s # [s cos(kx $ vt)] # $ksm sin(kx $ vt). &x &x m Substituting this quantity for the partial derivative in Eq. 17-18 yields %p # Bksm sin(kx $ vt). This tells us that the pressure varies as a sinusoidal function of time and that the amplitude of the variation is equal to the terms in front of the sine function. Setting %pm # Bksm, this yields Eq. 17-13, which we set out to prove. Using Eq. 17-3, we can now write %pm # (Bk)sm # (v2rk)sm. Equation 17-14, which we also wanted to prove, follows at once if we substitute v/v for k from Eq. 16-12.

17-3 I NTE R FE R E NCE

485

Sample Problem 17.01 Pressure amplitude, displacement amplitude The maximum pressure amplitude %pm that the human ear can tolerate in loud sounds is about 28 Pa (which is very much less than the normal air pressure of about 105 Pa). What is the displacement amplitude sm for such a sound in air of density r # 1.21 kg/m3, at a frequency of 1000 Hz and a speed of 343 m/s? KEY IDEA The displacement amplitude sm of a sound wave is related to the pressure amplitude %pm of the wave according to Eq. 17-14. Calculations: Solving that equation for sm yields sm #

%pm %pm # . v3* v3(2/ f )

Substituting known data then gives us

28 Pa (343 m/s)(1.21 kg/m3)(2/)(1000 Hz) # 1.1 ' 10$5 m # 11 mm. (Answer)

sm #

That is only about one-seventh the thickness of a book page. Obviously, the displacement amplitude of even the loudest sound that the ear can tolerate is very small. Temporary exposure to such loud sound produces temporary hearing loss, probably due to a decrease in blood supply to the inner ear. Prolonged exposure produces permanent damage. The pressure amplitude %pm for the faintest detectable sound at 1000 Hz is 2.8 ' 10$5 Pa. Proceeding as above leads to sm # 1.1 ' 10$11 m or 11 pm, which is about onetenth the radius of a typical atom. The ear is indeed a sensitive detector of sound waves.

Additional examples, video, and practice available at WileyPLUS

17-3 INTERFERENCE Learning Objectives After reading this module, you should be able to . . .

17.14 If two waves with the same wavelength begin in phase but reach a common point by traveling along different paths, calculate their phase difference f at that point by relating the path length difference %L to the wavelength l. 17.15 Given the phase difference between two sound

waves with the same amplitude, wavelength, and travel direction, determine the type of interference between the waves (fully destructive interference, fully constructive interference, or indeterminate interference). 17.16 Convert a phase difference between radians, degrees, and number of wavelengths.

Key Ideas ● The interference of two sound waves with identical wave-

lengths passing through a common point depends on their phase difference f there. If the sound waves were emitted in phase and are traveling in approximately the same direction, f is given by %L f# 2p, l where %L is their path length difference. ● Fully constructive interference occurs when f is an integer

multiple of 2p,

f # m(2p), for m # 0, 1, 2, . . . , and, equivalently, when %L is related to wavelength l by %L # 0, 1, 2, . . . . l ● Fully destructive interference occurs when f is an odd multiple of p, f # (2m " 1)p, for m # 0, 1, 2, . . . , and

Interference Like transverse waves, sound waves can undergo interference. In fact, we can write equations for the interference as we did in Module 16-5 for transverse waves. Suppose two sound waves with the same amplitude and wavelength are traveling in the positive direction of an x axis with a phase difference of f. We can express the waves in the form of Eqs. 16-47 and 16-48 but, to be consistent with Eq. 17-12, we use cosine functions instead of sine functions: s1(x, t) # sm cos(kx $ vt)

%L # 0.5, 1.5, 2.5, . . . . l

486

CHAPTE R 17 WAVES—I I

P

L1

and s2(x, t) # sm cos(kx $ vt " f).

S1 L2 S2

The interference at P depends on the difference in the path lengths to reach P.

These waves overlap and interfere. From Eq. 16-51, we can write the resultant wave as s, # [2sm cos 12f] cos(kx $ vt " 12f). As we saw with transverse waves, the resultant wave is itself a traveling wave. Its amplitude is the magnitude

(a)

P

If the difference is equal to, say, 2.0 λ, then the waves arrive exactly in phase. This is how transverse waves would look. (b)

P

If the difference is equal to, say, 2.5 λ, then the waves arrive exactly out of phase. This is how transverse waves would look. (c)

Figure 17-7 (a) Two point sources S1 and S2 emit spherical sound waves in phase. The rays indicate that the waves pass through a common point P. The waves (represented with transverse waves) arrive at P (b) exactly in phase and (c) exactly out of phase.

s,m # !2sm cos 12f!.

(17-19)

As with transverse waves, the value of f determines what type of interference the individual waves undergo. One way to control f is to send the waves along paths with different lengths. Figure 17-7a shows how we can set up such a situation: Two point sources S1 and S2 emit sound waves that are in phase and of identical wavelength l. Thus, the sources themselves are said to be in phase; that is, as the waves emerge from the sources, their displacements are always identical. We are interested in the waves that then travel through point P in Fig. 17-7a. We assume that the distance to P is much greater than the distance between the sources so that we can approximate the waves as traveling in the same direction at P. If the waves traveled along paths with identical lengths to reach point P, they would be in phase there. As with transverse waves, this means that they would undergo fully constructive interference there. However, in Fig. 17-7a, path L2 traveled by the wave from S2 is longer than path L1 traveled by the wave from S1. The difference in path lengths means that the waves may not be in phase at point P. In other words, their phase difference f at P depends on their path length difference %L # |L2 $ L1|. To relate phase difference f to path length difference %L, we recall (from Module 16-1) that a phase difference of 2p rad corresponds to one wavelength. Thus, we can write the proportion %L f , # 2p l

(17-20)

from which f#

%L 2p. l

(17-21)

Fully constructive interference occurs when f is zero, 2p, or any integer multiple of 2p. We can write this condition as f # m(2p),

for m # 0, 1, 2, . . .

(fully constructive interference).

(17-22)

From Eq. 17-21, this occurs when the ratio %L/l is %L # 0, 1, 2, . . . l

(fully constructive interference).

(17-23)

For example, if the path length difference %L # |L2 $ L1| in Fig. 17-7a is equal to 2l, then %L/l # 2 and the waves undergo fully constructive interference at point P (Fig. 17-7b). The interference is fully constructive because the wave from S2 is phase-shifted relative to the wave from S1 by 2l, putting the two waves exactly in phase at P. Fully destructive interference occurs when f is an odd multiple of p: f # (2m " 1)p,

for m # 0, 1, 2, . . .

(fully destructive interference).

(17-24)

487

17-3 I NTE R FE R E NCE

From Eq. 17-21, this occurs when the ratio %L/l is %L # 0.5, 1.5, 2.5, . . . l

(fully destructive interference).

(17-25)

For example, if the path length difference %L # |L2 $ L1| in Fig. 17-7a is equal to 2.5l, then %L/l # 2.5 and the waves undergo fully destructive interference at point P (Fig. 17-7c). The interference is fully destructive because the wave from S2 is phase-shifted relative to the wave from S1 by 2.5 wavelengths, which puts the two waves exactly out of phase at P. Of course, two waves could produce intermediate interference as, say, when %L/l # 1.2. This would be closer to fully constructive interference (%L/l # 1.0) than to fully destructive interference (%L/l # 1.5).

Sample Problem 17.02 Interference points along a big circle In Fig. 17-8a, two point sources S1 and S2, which are in phase and separated by distance D # 1.5l, emit identical sound waves of wavelength l.

Reasoning: The wave from S1 travels the extra distance D (# 1.5l) to reach P2.Thus, the path length difference is %L # 1.5l.

(a) What is the path length difference of the waves from S1 and S2 at point P1, which lies on the perpendicular bisector of distance D, at a distance greater than D from the sources (Fig. 17-8b)? (That is, what is the difference in the distance from source S1 to point P1 and the distance from source S2 to P1?) What type of interference occurs at P1?

From Eq. 17-25, this means that the waves are exactly out of phase at P2 and undergo fully destructive interference there. (c) Figure 17-8d shows a circle with a radius much greater than D, centered on the midpoint between sources S1 and S2. What is the number of points N around this circle at which the interference is fully constructive? (That is, at how many points do the waves arrive exactly in phase?)

Reasoning: Because the waves travel identical distances to reach P1, their path length difference is %L # 0.

(Answer)

Reasoning: Starting at point a, let’s move clockwise along the circle to point d. As we move, path length difference %L increases and so the type of interference changes. From (a), we know that is %L # 0l at point a. From (b), we know that %L # 1.5l at point d. Thus, there must be

From Eq. 17-23, this means that the waves undergo fully constructive interference at P1 because they start in phase at the sources and reach P1 in phase. (b) What are the path length difference and type of interference at point P2 in Fig. 17-8c?

S1

S1 D/2 D/2 S2

(a)

S2

(b)

A

The difference in these path lengths equals 0.

S1

L1

D L2

(Answer)

P1

The difference in these path lengths is D, which equals 1.5 λ .

D

S1

S2

Thus, the waves arrive exactly in phase and undergo fully constructive interference. (c)

P2

Thus, the waves arrive exactly out of phase and undergo fully destructive interference.

a 0λ

S2 d (d )

1.5λ

Figure 17-8 (a) Two point sources S1 and S2, separated by distance D, emit spherical sound waves in phase. (b) The waves travel equal distances to reach point P1. (c) Point P2 is on the line extending through S1 and S2. (d) We move around a large circle. (Figure continues)

488

CHAPTE R 17 WAVES—I I

1.0λ

c

(e)

1.0λ S1

0λ b

The difference in these path 1.0λ lengths equals 1.0λ .

1.5λ

We find six points of fully constructive interference.

Maximum phase difference

1.0λ

a 0λ

1.0λ

S2 1.0λ

Thus, the waves arrive exactly in phase and undergo fully constructive interference.

d

1.5λ

S1



1.0λ

Zero phase difference

(f )



S2 1.0λ

1.0λ (g)

Zero phase difference

Maximum phase difference

Figure 17-8 (continued) (e) Another point of fully constructive interference. ( f ) Using symmetry to determine other points. (g) The six points of fully constructive interference.

one point between a and d at which %L # l (Fig. 17-8e). From Eq. 17-23, fully constructive interference occurs at that point. Also, there can be no other point along the way from point a to point d at which fully constructive interference occurs, because there is no other integer than 1 between 0 at point a and 1.5 at point d.

We can now use symmetry to locate other points of fully constructive or destructive interference (Fig. 17-8f ). Symmetry about line cd gives us point b, at which %L # 0l. Also, there are three more points at which %L # l. In all (Fig. 17-8g) we have N # 6.

(Answer)

Additional examples, video, and practice available at WileyPLUS

17-4 INTENSITY AND SOUND LEVEL Learning Objectives After reading this module, you should be able to . . .

17.17 Calculate the sound intensity I at a surface as the ratio of the power P to the surface area A. 17.18 Apply the relationship between the sound intensity I and the displacement amplitude sm of the sound wave. 17.19 Identify an isotropic point source of sound. 17.20 For an isotropic point source, apply the relationship involving the emitting power Ps, the distance r to a detector, and the sound intensity I at the detector.

17.21 Apply the relationship between the sound level b, the sound intensity I, and the standard reference intensity I0. 17.22 Evaluate a logarithm function (log) and an antilogarithm function (log$1). 17.23 Relate the change in a sound level to the change in sound intensity.

Key Ideas ● The intensity I of a sound wave at a surface is the average rate per unit area at which energy is transferred by the wave through or onto the surface:

I#

P , A

where P is the time rate of energy transfer (power) of the sound wave and A is the area of the surface intercepting the sound. The intensity I is related to the displacement amplitude sm of the sound wave by I # 123vv 2s 2m .

● The intensity at a distance r from a point source that

emits sound waves of power Ps equally in all directions (isotropically) is Ps I# . 4pr 2 ● The sound level b in decibels (dB) is defined as b # (10 dB) log

I , I0

where I0 (# 10$12 W/m2) is a reference intensity level to which all intensities are compared. For every factor-of-10 increase in intensity, 10 dB is added to the sound level.

17-4 I NTE NSITY AN D SOU N D LEVE L

489

Intensity and Sound Level If you have ever tried to sleep while someone played loud music nearby, you are well aware that there is more to sound than frequency, wavelength, and speed. There is also intensity. The intensity I of a sound wave at a surface is the average rate per unit area at which energy is transferred by the wave through or onto the surface. We can write this as P , (17-26) I# A where P is the time rate of energy transfer (the power) of the sound wave and A is the area of the surface intercepting the sound. As we shall derive shortly, the intensity I is related to the displacement amplitude sm of the sound wave by I # 12 rvv 2s 2m.

(17-27)

Intensity can be measured on a detector. Loudness is a perception, something that you sense. The two can differ because your perception depends on factors such as the sensitivity of your hearing mechanism to various frequencies.

Variation of Intensity with Distance How intensity varies with distance from a real sound source is often complex. Some real sources (like loudspeakers) may transmit sound only in particular directions, and the environment usually produces echoes (reflected sound waves) that overlap the direct sound waves. In some situations, however, we can ignore echoes and assume that the sound source is a point source that emits the sound isotropically — that is, with equal intensity in all directions. The wavefronts spreading from such an isotropic point source S at a particular instant are shown in Fig. 17-9. Let us assume that the mechanical energy of the sound waves is conserved as they spread from this source. Let us also center an imaginary sphere of radius r on the source, as shown in Fig. 17-9. All the energy emitted by the source must pass through the surface of the sphere. Thus, the time rate at which energy is transferred through the surface by the sound waves must equal the time rate at which energy is emitted by the source (that is, the power Ps of the source). From Eq. 17-26, the intensity I at the sphere must then be

I#

Ps , 4pr 2

(17-28)

where 4pr 2 is the area of the sphere. Equation 17-28 tells us that the intensity of sound from an isotropic point source decreases with the square of the distance r from the source.

Checkpoint 2 The figure indicates three small patches 1, 2, and 3 that lie on the surfaces of two imaginary spheres; the spheres are centered on an isotropic point source S of sound. The rates at which energy is transmitted through the three patches by the sound waves are equal. Rank the patches according to (a) the intensity of the sound on them and (b) their area, greatest first.

3 S 2 1

r S

Figure 17-9 A point source S emits sound waves uniformly in all directions. The waves pass through an imaginary sphere of radius r that is centered on S.

490

CHAPTE R 17 WAVES—I I

The Decibel Scale The displacement amplitude at the human ear ranges from about 10$5 m for the loudest tolerable sound to about 10$11 m for the faintest detectable sound, a ratio of 106. From Eq. 17-27 we see that the intensity of a sound varies as the square of its amplitude, so the ratio of intensities at these two limits of the human auditory system is 1012. Humans can hear over an enormous range of intensities. We deal with such an enormous range of values by using logarithms. Consider the relation y # log x, in which x and y are variables. It is a property of this equation that if we multiply x by 10, then y increases by 1. To see this, we write y, # log(10x) # log 10 " log x # 1 " y. © Ben Rose

Sound can cause the wall of a drinking glass to oscillate. If the sound produces a standing wave of oscillations and if the intensity of the sound is large enough, the glass will shatter.

Similarly, if we multiply x by 1012, y increases by only 12. Thus, instead of speaking of the intensity I of a sound wave, it is much more convenient to speak of its sound level b, defined as b # (10 dB) log

I . I0

(17-29)

Here dB is the abbreviation for decibel, the unit of sound level, a name that was chosen to recognize the work of Alexander Graham Bell. I0 in Eq. 17-29 is a standard reference intensity (# 10$12 W/m2), chosen because it is near the lower limit of the human range of hearing. For I # I0, Eq. 17-29 gives b # 10 log 1 # 0, so our standard reference level corresponds to zero decibels. Then b increases by 10 dB every time the sound intensity increases by an order of magnitude (a factor of 10). Thus, b # 40 corresponds to an intensity that is 104 times the standard reference level. Table 17-2 lists the sound levels for a variety of environments.

Derivation of Eq. 17-27 Consider, in Fig. 17-4a, a thin slice of air of thickness dx, area A, and mass dm, oscillating back and forth as the sound wave of Eq. 17-12 passes through it. The kinetic energy dK of the slice of air is dK # 12 dm v2s .

(17-30)

Here vs is not the speed of the wave but the speed of the oscillating element of air, obtained from Eq. 17-12 as vs #

&s # $vsm sin(kx $ vt). &t

Using this relation and putting dm # rA dx allow us to rewrite Eq. 17-30 as Table 17-2 Some Sound Levels (dB) Hearing threshold Rustle of leaves Conversation Rock concert Pain threshold Jet engine

0 10 60 110 120 130

dK # 21(rA dx)($vsm)2 sin2(kx $ vt).

(17-31)

Dividing Eq. 17-31 by dt gives the rate at which kinetic energy moves along with the wave. As we saw in Chapter 16 for transverse waves, dx/dt is the wave speed v, so we have dK # 12rAvv2s2m sin2(kx $ vt). dt

(17-32)

17-4 I NTE NSITY AN D SOU N D LEVE L

491

The average rate at which kinetic energy is transported is

# dK dt $

avg

# 12 rAvv2s 2m[sin2(kx $ vt)]avg # 14 rAvv2s 2m .

(17-33)

To obtain this equation, we have used the fact that the average value of the square of a sine (or a cosine) function over one full oscillation is 12. We assume that potential energy is carried along with the wave at this same average rate. The wave intensity I, which is the average rate per unit area at which energy of both kinds is transmitted by the wave, is then, from Eq. 17-33, I#

2(dK/dt)avg A

# 12 rvv2s 2m ,

which is Eq. 17-27, the equation we set out to derive.

Sample Problem 17.03 Intensity change with distance, cylindrical sound wave An electric spark jumps along a straight line of length L # 10 m, emitting a pulse of sound that travels radially outward from the spark. (The spark is said to be a line source of sound.) The power of this acoustic emission is Ps # 1.6 ' 104 W.

Calculations: Putting these ideas together and noting that the area of the cylindrical surface is A # 2prL, we have

(a) What is the intensity I of the sound when it reaches a distance r # 12 m from the spark?

This tells us that the intensity of the sound from a line source decreases with distance r (and not with the square of distance r as for a point source). Substituting the given data, we find

KEY IDEAS (1) Let us center an imaginary cylinder of radius r # 12 m and length L # 10 m (open at both ends) on the spark, as shown in Fig. 17-10. Then the intensity I at the cylindrical surface is the ratio P/A, where P is the time rate at which sound energy passes through the surface and A is the surface area. (2) We assume that the principle of conservation of energy applies to the sound energy. This means that the rate P at which energy is transferred through the cylinder must equal the rate Ps at which energy is emitted by the source. Spark

L

I#

I#

(17-34)

1.6 ' 104 W 2p(12 m)(10 m)

# 21.2 W/m2 % 21 W/m2.

(Answer)

(b) At what time rate Pd is sound energy intercepted by an acoustic detector of area Ad # 2.0 cm2, aimed at the spark and located a distance r # 12 m from the spark? Calculations: We know that the intensity of sound at the detector is the ratio of the energy transfer rate Pd there to the detector’s area Ad: I#

r

Figure 17-10 A spark along a straight line of length L emits sound waves radially outward. The waves pass through an imaginary cylinder of radius r and length L that is centered on the spark.

Ps P . # A 2prL

Pd . Ad

(17-35)

We can imagine that the detector lies on the cylindrical surface of (a). Then the sound intensity at the detector is the intensity I (# 21.2 W/m2) at the cylindrical surface. Solving Eq. 17-35 for Pd gives us Pd # (21.2 W/m2)(2.0 ' 10$4 m2) # 4.2 mW.

Additional examples, video, and practice available at WileyPLUS

(Answer)

492

CHAPTE R 17 WAVES—I I

Sample Problem 17.04 Decibels, sound level, change in intensity Many veteran rockers suffer from acute hearing damage because of the high sound levels they endured for years. Many rockers now wear special earplugs to protect their hearing during performances (Fig. 17-11). If an earplug decreases the sound level of the sound waves by 20 dB, what is the ratio of the final intensity If of the waves to their initial intensity Ii? KEY IDEA For both the final and initial waves, the sound level b is related to the intensity by the definition of sound level in Eq. 17-29. Calculations: For the final waves we have If bf # (10 dB) log , I0 and for the initial waves we have I bi # (10 dB) log i . I0 The difference in the sound levels is If I (17-36) $ log i . bf $ bi # (10 dB) log I0 I0 Using the identity a c ad log $ log # log , b d bc we can rewrite Eq. 17-36 as If bf $ bi # (10 dB) log . (17-37) Ii Rearranging and then substituting the given decrease in

#

$

Figure 17-11 Lars Ulrich of Metallica is an advocate for the organization HEAR (Hearing Education and Awareness for Rockers), which warns about the damage high sound levels can have on hearing. Tim Mosenfelder/Getty Images, Inc.

sound level as bf $ bi # $20 dB, we find bf $ bi If $20 dB # # $2.0. log # Ii 10 dB 10 dB We next take the antilog of the far left and far right sides of this equation. (Although the antilog 10$2.0 can be evaluated mentally, you could use a calculator by keying in 10ˆ-2.0 or using the 10x key.) We find If # log$1 ($2.0) # 0.010. (Answer) Ii Thus, the earplug reduces the intensity of the sound waves to 0.010 of their initial intensity (two orders of magnitude).

Additional examples, video, and practice available at WileyPLUS

17-5 SOURCES OF MUSICAL SOUND Learning Objectives After reading this module, you should be able to . . .

17.24 Using standing wave patterns for string waves, sketch the standing wave patterns for the first several acoustical harmonics of a pipe with only one open end and with two open ends. 17.25 For a standing wave of sound, relate the distance between nodes and the wavelength.

17.26 Identify which type of pipe has even harmonics. 17.27 For any given harmonic and for a pipe with only one open end or with two open ends, apply the relationships between the pipe length L, the speed of sound v, the wavelength l, the harmonic frequency f, and the harmonic number n.

Key Ideas ● Standing sound wave patterns can be set up in pipes (that

is, resonance can be set up) if sound of the proper wavelength is introduced in the pipe. ● A pipe open at both ends will resonate at frequencies v nv f# , n # 1, 2, 3, . . . , # . 2L

where v is the speed of sound in the air in the pipe. ● For a pipe closed at one end and open at the other, the resonant frequencies are f#

v nv , # . 4L

n # 1, 3, 5, . . . .

17-5 SOU RCES OF M USICAL SOU N D

493

Sources of Musical Sound Musical sounds can be set up by oscillating strings (guitar, piano, violin), membranes (kettledrum, snare drum), air columns (flute, oboe, pipe organ, and the didgeridoo of Fig. 17-12), wooden blocks or steel bars (marimba, xylophone), and many other oscillating bodies. Most common instruments involve more than a single oscillating part. Recall from Chapter 16 that standing waves can be set up on a stretched string that is fixed at both ends. They arise because waves traveling along the string are reflected back onto the string at each end. If the wavelength of the waves is suitably matched to the length of the string, the superposition of waves traveling in opposite directions produces a standing wave pattern (or oscillation mode). The wavelength required of the waves for such a match is one that corresponds to a resonant frequency of the string. The advantage of setting up standing waves is that the string then oscillates with a large, sustained amplitude, pushing back and forth against the surrounding air and thus generating a noticeable sound wave with the same frequency as the oscillations of the string. This production of sound is of obvious importance to, say, a guitarist. Sound Waves. We can set up standing waves of sound in an air-filled pipe in a similar way. As sound waves travel through the air in the pipe, they are reflected at each end and travel back through the pipe. (The reflection occurs even if an end is open, but the reflection is not as complete as when the end is closed.) If the wavelength of the sound waves is suitably matched to the length of the pipe, the superposition of waves traveling in opposite directions through the pipe sets up a standing wave pattern. The wavelength required of the sound waves for such a match is one that corresponds to a resonant frequency of the pipe. The advantage of such a standing wave is that the air in the pipe oscillates with a large, sustained amplitude, emitting at any open end a sound wave that has the same frequency as the oscillations in the pipe. This emission of sound is of obvious importance to, say, an organist. Many other aspects of standing sound wave patterns are similar to those of string waves: The closed end of a pipe is like the fixed end of a string in that there must be a node (zero displacement) there, and the open end of a pipe is like the end of a string attached to a freely moving ring, as in Fig. 16-19b, in that there must be an antinode there. (Actually, the antinode for the open end of a pipe is located slightly beyond the end, but we shall not dwell on that detail.) Two Open Ends. The simplest standing wave pattern that can be set up in a pipe with two open ends is shown in Fig. 17-13a. There is an antinode across each Antinodes (maximum oscillation) occur at the open ends.

L λ = 2L

(a)

(b)

A

N

A

First harmonic

Figure 17-13 (a) The simplest standing wave pattern of displacement for (longitudinal) sound waves in a pipe with both ends open has an antinode (A) across each end and a node (N) across the middle. (The longitudinal displacements represented by the double arrows are greatly exaggerated.) (b) The corresponding standing wave pattern for (transverse) string waves.

Alamy

Figure 17-12 The air column within a didgeridoo (“a pipe”) oscillates when the instrument is played.

494

CHAPTE R 17 WAVES—I I λ = 4L

n=1 L

First Third

Second

λ = 4L/5

n=5

λ = 2L/3

n=3

Fifth

Third n=4 Fourth

λ = 4L/3

n=3

λ = 2L/2 = L

n=2

(a )

λ = 4L/7

n=7

λ = 2L/4 = L/2

Seventh

Two open ends— any harmonic

(b)

One open end— only odd harmonics

Figure 17-14 Standing wave patterns for string waves superimposed on pipes to represent standing sound wave patterns in the pipes. (a) With both ends of the pipe open, any harmonic can be set up in the pipe. (b) With only one end open, only odd harmonics can be set up.

open end, as required. There is also a node across the middle of the pipe. An easier way of representing this standing longitudinal sound wave is shown in Fig. 17-13b — by drawing it as a standing transverse string wave. The standing wave pattern of Fig. 17-13a is called the fundamental mode or first harmonic. For it to be set up, the sound waves in a pipe of length L must have a wavelength given by L # l/2, so that l # 2L. Several more standing sound wave patterns for a pipe with two open ends are shown in Fig. 17-14a using string wave representations. The second harmonic requires sound waves of wavelength l # L, the third harmonic requires wavelength l # 2L/3, and so on. More generally, the resonant frequencies for a pipe of length L with two open ends correspond to the wavelengths l#

2L , n

for n # 1, 2, 3, . . . ,

(17-38)

where n is called the harmonic number. Letting v be the speed of sound, we write the resonant frequencies for a pipe with two open ends as f#

nv v # , l 2L

for n # 1, 2, 3, . . .

(pipe, two open ends).

(17-39)

One Open End. Figure 17-14b shows (using string wave representations) some of the standing sound wave patterns that can be set up in a pipe with only one open end. As required, across the open end there is an antinode and across the closed end there is a node. The simplest pattern requires sound waves having a wavelength given by L # l/4, so that l # 4L. The next simplest pattern requires a wavelength given by L # 3l/4, so that l # 4L/3, and so on. More generally, the resonant frequencies for a pipe of length L with only one open end correspond to the wavelengths 4L for n # 1, 3, 5, . . . , (17-40) , n in which the harmonic number n must be an odd number. The resonant frequencies are then given by l#

f#

nv v # , l 4L

for n # 1, 3, 5, . . .

(pipe, one open end).

(17-41)

Note again that only odd harmonics can exist in a pipe with one open end. For example, the second harmonic, with n # 2, cannot be set up in such a pipe. Note also that for such a pipe the adjective in a phrase such as “the third harmonic” still refers to the harmonic number n (and not to, say, the third possible harmonic). Finally note that Eqs. 17-38 and 17-39 for two open ends contain the

17-5 SOU RCES OF M USICAL SOU N D

495

Bass saxophone Baritone saxophone Tenor saxophone Alto saxophone Soprano saxophone

Figure 17-15 The saxophone and violin families, showing the relations between instrument length and frequency range. The frequency range of each instrument is indicated by a horizontal bar along a frequency scale suggested by the keyboard at the bottom; the frequency increases toward the right.

Violin Viola Cello Bass

A B C D E F G A B C D E F G A B C D E F G A B C D E F G A B C D E F G A B C D E F G A B C D E F G A B C

number 2 and any integer value of n, but Eqs. 17-40 and 17-41 for one open end contain the number 4 and only odd values of n. Length. The length of a musical instrument reflects the range of frequencies over which the instrument is designed to function, and smaller length implies higher frequencies, as we can tell from Eq. 16-66 for string instruments and Eqs. 1739 and 17-41 for instruments with air columns. Figure 17-15, for example, shows the saxophone and violin families, with their frequency ranges suggested by the piano keyboard. Note that, for every instrument, there is overlap with its higher- and lower-frequency neighbors. Net Wave. In any oscillating system that gives rise to a musical sound, whether it is a violin string or the air in an organ pipe, the fundamental and one or more of the higher harmonics are usually generated simultaneously. Thus, you hear them together — that is, superimposed as a net wave. When different instruments are played at the same note, they produce the same fundamental frequency but different intensities for the higher harmonics. For example, the fourth harmonic of middle C might be relatively loud on one instrument and relatively quiet or even missing on another. Thus, because different instruments produce different net waves, they sound different to you even when they are played at the same note. That would be the case for the two net waves shown in Fig. 17-16, which were produced at the same note by different instruments. If you heard only the fundamentals, the music would not be musical.

(a)

(b)

Time

Figure 17-16 The wave forms produced by (a) a flute and (b) an oboe when played at the same note, with the same first harmonic frequency.

Checkpoint 3 Pipe A, with length L, and pipe B, with length 2L, both have two open ends. Which harmonic of pipe B has the same frequency as the fundamental of pipe A?

Sample Problem 17.05 Resonance between pipes of different lengths Pipe A is open at both ends and has length LA # 0.343 m. We want to place it near three other pipes in which standing waves have been set up, so that the sound can set up a standing wave in pipe A. Those other three pipes are each closed at one end and have lengths LB # 0.500LA, LC # 0.250LA, and LD # 2.00LA. For each of these three pipes, which of their harmonics can excite a harmonic in pipe A?

KEY IDEAS (1) The sound from one pipe can set up a standing wave in another pipe only if the harmonic frequencies match. (2) Equation 17-39 gives the harmonic frequencies in a pipe with two open ends (a symmetric pipe) as f # nv/2L, for n # 1, 2, 3, . . . , that is, for any positive integer. (3) Equation

496

CHAPTE R 17 WAVES—I I

17-41 gives the harmonic frequencies in a pipe with only one open end (an asymmetric pipe) as f # nv/4L, for n # 1, 3, 5, . . . , that is, for only odd positive integers. Pipe A: Let’s first find the resonant frequencies of symmetric pipe A (with two open ends) by evaluating Eq. 17-39: fA #

nAv n (343 m/s) # A 2LA 2(0.343 m)

The first six harmonic frequencies are shown in the top plot in Fig. 17-17. Pipe B: Next let’s find the resonant frequencies of asymmetric pipe B (with only one open end) by evaluating Eq. 17-41, being careful to use only odd integers for the harmonic numbers: nB(343 m/s) n Bv nBv # # 4LB 4(0.500LA) 2(0.343 m)

# nB(500 Hz) # nB(0.500 kHz),

nC (343 m/s) n Cv nCv # # 4LC 4(0.250LA) 0.343 m/s

# nC (1000 Hz) # nC (1.00 kHz),

for nB # 1, 3, 5, . . . .

n (343 m/s) nDv nDv # # D 4LD 4(2LA) 8(0.343 m/s)

fD #

# nD (125 Hz) # nD (0.125 kHz),

for nD # 1, 3, 5, . . . .

As shown in Fig. 17-17, none of these frequencies match a harmonic frequency of A. (Can you see that we would get a match if nD # 4nA? But that is impossible because 4nA cannot yield an odd integer, as required of nD.) Thus D cannot set up a standing wave in A. 0

0.50

1.0

1.5

2.0

2.5

3.0 kHz

1

2

3

4

5

6

1

with nB # 1, 3, 5, . . . . (Answer)

3

For example, as shown in Fig. 17-17, if we set up the fifth harmonic in pipe B and bring the pipe close to pipe A, the fifth harmonic will then be set up in pipe A. However, no harmonic in B can set up an even harmonic in A. Pipe C: Let’s continue with pipe C (with only one end) by writing Eq. 17-41 as

with nC # 1, 3, 5, . . . . (Answer)

Pipe D: Finally, let’s check D with our same procedure:

Comparing our two results, we see that we get a match for each choice of nB: fA = fB for nA # nB

for nC # 1, 3, 5, . . . .

From this we see that C can excite some of the harmonics of A but only those with harmonic numbers nA that are twice an odd integer: fA # fC for nA # 2nC,

# nA(500 Hz) # nA(0.50 kHz), for nA # 1, 2, 3, . . . .

fB #

fC #

5

1

1

3

5

7

9

nA

nB

3

nC

11 13 15 17 19 21 23

nD

Figure 17-17 Harmonic frequencies of four pipes.

Additional examples, video, and practice available at WileyPLUS

17-6 BEATS Learning Objectives After reading this module, you should be able to . . .

17.28 Explain how beats are produced. 17.29 Add the displacement equations for two sound waves of the same amplitude and slightly different angular frequencies to find the displacement equation of the resultant wave and identify the time-varying amplitude.

17.30 Apply the relationship between the beat frequency and the frequencies of two sound waves that have the same amplitude when the frequencies (or, equivalently, the angular frequencies) differ by a small amount.

Key Idea ● Beats arise when two waves having slightly different frequencies, f1 and f2, are detected together. The beat frequency is

fbeat # f 1 $ f 2.

17-6 B EATS

Beats If we listen, a few minutes apart, to two sounds whose frequencies are, say, 552 and 564 Hz, most of us cannot tell one from the other because the frequencies are so close to each other. However, if the sounds reach our ears simultaneously, what we hear is a sound whose frequency is 558 Hz, the average of the two combining frequencies. We also hear a striking variation in the intensity of this sound — it increases and decreases in slow, wavering beats that repeat at a frequency of 12 Hz, the difference between the two combining frequencies. Figure 17-18 shows this beat phenomenon. Let the time-dependent variations of the displacements due to two sound waves of equal amplitude sm be s1 # sm cos v1t

and

s2 # sm cos v 2t,

(17-42)

where v1 4 v2. From the superposition principle, the resultant displacement is the sum of the individual displacements: s # s1 " s2 # sm(cos v1t " cos v 2t). Using the trigonometric identity (see Appendix E) cos a " cos b # 2 cos[12(a $ b)] cos[12(a " b)] allows us to write the resultant displacement as s # 2 sm cos[ 12(v1 $ v 2)t] cos[ 12(v1 " v 2)t].

(17-43)

v, # 21(v1 $ v 2)

(17-44)

If we write and

v # 12(v 1 " v 2),

we can then write Eq. 17-43 as s(t) # [2sm cos v,t] cos vt.

(17-45)

We now assume that the angular frequencies v1 and v 2 of the combining waves are almost equal, which means that v 1 v, in Eq. 17-44. We can then regard Eq. 17-45 as a cosine function whose angular frequency is v and whose amplitude (which is not constant but varies with angular frequency v,) is the absolute value of the quantity in the brackets. A maximum amplitude will occur whenever cos v,t in Eq. 17-45 has the value "1 or $1, which happens twice in each repetition of the cosine function. Because cos v,t has angular frequency v,, the angular frequency vbeat at which beats occur is vbeat # 2v,. Then, with the aid of Eq. 17-44, we can write the beat angular frequency as vbeat # 2v, # (2)(12)(v1 $ v2) # v1 $ v 2. Because v # 2pf, we can recast this as fbeat # f1 $ f2

(beat frequency).

(17-46)

Musicians use the beat phenomenon in tuning instruments. If an instrument is sounded against a standard frequency (for example, the note called “concert A” played on an orchestra’s first oboe) and tuned until the beat disappears, the instrument is in tune with that standard. In musical Vienna, concert A (440 Hz) is available as a convenient telephone service for the city’s many musicians. Figure 17-18 (a, b) The pressure variations %p of two sound waves as they would be detected separately. The frequencies of the waves are nearly equal. (c) The resultant pressure variation if the two waves are detected simultaneously.

(a)

(b) Time

(c)

497

498

CHAPTE R 17 WAVES—I I

Sample Problem 17.06 Beat frequencies and penguins finding one another When an emperor penguin returns from a search for food, how can it find its mate among the thousands of penguins huddled together for warmth in the harsh Antarctic weather? It is not by sight, because penguins all look alike, even to a penguin. The answer lies in the way penguins vocalize. Most birds vocalize by using only one side of their two-sided vocal organ, called the syrinx. Emperor penguins, however, vocalize by using both sides simultaneously. Each side sets up acoustic standing waves in the bird’s throat and mouth, much like in a pipe with two open ends. Suppose that the frequency of the first harmonic produced by side A is fA1 # 432 Hz and the frequency of the first harmonic produced by side B is fB1 # 371 Hz. What is the beat frequency between those two first-harmonic frequencies and between the two second-harmonic frequencies?

Because the standing waves in the penguin are effectively in a pipe with two open ends, the resonant frequencies are given by Eq. 17-39 ( f # nv/2L), in which L is the (unknown) length of the effective pipe. The first-harmonic frequency is f1 # v/2L, and the second-harmonic frequency is f2 # 2v/2L. Comparing these two frequencies, we see that, in general, f2 # 2f1. For the penguin, the second harmonic of side A has frequency fA2 # 2fA1 and the second harmonic of side B has frequency fB2 # 2fB1. Using Eq. 17-46 with frequencies fA2 and fB2, we find that the corresponding beat frequency associated with the second harmonics is fbeat,2 # fA2 $ fB2 # 2fA1 $ 2fB1 # 2(432 Hz) $ 2(371 Hz) # 122 Hz.

KEY IDEA The beat frequency between two frequencies is their difference, as given by Eq. 17-46 ( fbeat # f1 $ f2). Calculations: For the two first-harmonic frequencies fA1 and fB1, the beat frequency is fbeat,1 # fA1 $ fB1 # 432 Hz $ 371 Hz # 61 Hz. (Answer)

(Answer)

Experiments indicate that penguins can perceive such large beat frequencies. (Humans cannot hear a beat frequency any higher than about 12 Hz — we perceive the two separate frequencies.) Thus, a penguin’s cry can be rich with different harmonics and different beat frequencies, allowing the voice to be recognized even among the voices of thousands of other, closely huddled penguins.

Additional examples, video, and practice available at WileyPLUS

17-7 THE DOPPLER EFFECT Learning Objectives After reading this module, you should be able to . . .

17.31 Identify that the Doppler effect is the shift in the detected frequency from the frequency emitted by a sound source due to the relative motion between the source and the detector. 17.32 Identify that in calculating the Doppler shift in sound, the speeds are measured relative to the medium (such as air or water), which may be moving. 17.33 Calculate the shift in sound frequency for (a) a source

moving either directly toward or away from a stationary detector, (b) a detector moving either directly toward or away from a stationary source, and (c) both source and detector moving either directly toward each other or directly away from each other. 17.34 Identify that for relative motion between a sound source and a sound detector, motion toward tends to shift the frequency up and motion away tends to shift it down.

Key Ideas ● The Doppler effect is a change in the observed frequency of a wave when the source or the detector moves relative to the transmitting medium (such as air). For sound the observed frequency f, is given in terms of the source frequency f by

f, # f

v ! vD v ! vS

(general Doppler effect),

where vD is the speed of the detector relative to the medium, vS is that of the source, and v is the speed of sound in the medium. ● The signs are chosen such that f, tends to be greater for relative motion toward (one of the objects moves toward the other) and less for motion away.

17-7 TH E DOPPLE R E FFECT

The Doppler Effect A police car is parked by the side of the highway, sounding its 1000 Hz siren. If you are also parked by the highway, you will hear that same frequency. However, if there is relative motion between you and the police car, either toward or away from each other, you will hear a different frequency. For example, if you are driving toward the police car at 120 km/h (about 75 mi/h), you will hear a higher frequency (1096 Hz, an increase of 96 Hz). If you are driving away from the police car at that same speed, you will hear a lower frequency (904 Hz, a decrease of 96 Hz). These motion-related frequency changes are examples of the Doppler effect. The effect was proposed (although not fully worked out) in 1842 by Austrian physicist Johann Christian Doppler. It was tested experimentally in 1845 by Buys Ballot in Holland, “using a locomotive drawing an open car with several trumpeters.” The Doppler effect holds not only for sound waves but also for electromagnetic waves, including microwaves, radio waves, and visible light. Here, however, we shall consider only sound waves, and we shall take as a reference frame the body of air through which these waves travel. This means that we shall measure the speeds of a source S of sound waves and a detector D of those waves relative to that body of air. (Unless otherwise stated, the body of air is stationary relative to the ground, so the speeds can also be measured relative to the ground.) We shall assume that S and D move either directly toward or directly away from each other, at speeds less than the speed of sound. General Equation. If either the detector or the source is moving, or both are moving, the emitted frequency f and the detected frequency f , are related by f, # f

v ! vD v ! vS

(general Doppler effect),

(17-47)

where v is the speed of sound through the air, vD is the detector’s speed relative to the air, and vS is the source’s speed relative to the air. The choice of plus or minus signs is set by this rule: When the motion of detector or source is toward the other, the sign on its speed must give an upward shift in frequency. When the motion of detector or source is away from the other, the sign on its speed must give a downward shift in frequency.

In short, toward means shift up, and away means shift down. Here are some examples of the rule. If the detector moves toward the source, use the plus sign in the numerator of Eq. 17-47 to get a shift up in the frequency. If it moves away, use the minus sign in the numerator to get a shift down. If it is stationary, substitute 0 for vD. If the source moves toward the detector, use the minus sign in the denominator of Eq. 17-47 to get a shift up in the frequency. If it moves away, use the plus sign in the denominator to get a shift down. If the source is stationary, substitute 0 for vS. Next, we derive equations for the Doppler effect for the following two specific situations and then derive Eq. 17-47 for the general situation. 1. When the detector moves relative to the air and the source is stationary relative to the air, the motion changes the frequency at which the detector intercepts wavefronts and thus changes the detected frequency of the sound wave. 2. When the source moves relative to the air and the detector is stationary relative to the air, the motion changes the wavelength of the sound wave and thus changes the detected frequency (recall that frequency is related to wavelength).

499

500

CHAPTE R 17 WAVES—I I

Shift up: The detector moves toward the source.

Figure 17-19 A stationary source of sound S emits spherical wavefronts, shown one wavelength apart, that expand outward at speed v. A sound detector D, represented by an ear, moves with velocity v:D toward the source. The detector senses a higher frequency because of its motion. v

vD

vS = 0

S

v

λ

D

x

v

Detector Moving, Source Stationary In Fig. 17-19, a detector D (represented by an ear) is moving at speed vD toward a stationary source S that emits spherical wavefronts, of wavelength l and frequency f, moving at the speed v of sound in air. The wavefronts are drawn one wavelength apart. The frequency detected by detector D is the rate at which D intercepts wavefronts (or individual wavelengths). If D were stationary, that rate would be f, but since D is moving into the wavefronts, the rate of interception is greater, and thus the detected frequency f, is greater than f. Let us for the moment consider the situation in which D is stationary (Fig. 17-20). In time t, the wavefronts move to the right a distance vt. The number of wavelengths in that distance vt is the number of wavelengths intercepted by D in time t, and that number is vt/l. The rate at which D intercepts wavelengths, which is the frequency f detected by D, is

(a ) D vt

v

(b )

λ

Figure 17-20 The wavefronts of Fig. 17-19, assumed planar, (a) reach and (b) pass a stationary detector D; they move a distance vt to the right in time t.

v vD

f#

(a )

vDt

vt

v vt/l # . t l

(17-48)

In this situation, with D stationary, there is no Doppler effect — the frequency detected by D is the frequency emitted by S. Now let us again consider the situation in which D moves in the direction opposite the wavefront velocity (Fig. 17-21). In time t, the wavefronts move to the right a distance vt as previously, but now D moves to the left a distance vDt. Thus, in this time t, the distance moved by the wavefronts relative to D is vt " vDt. The number of wavelengths in this relative distance vt " vDt is the number of wavelengths intercepted by D in time t and is (vt " vDt)/l. The rate at which D intercepts wavelengths in this situation is the frequency f,, given by f, #

D

(b )

λ

v " vD (vt " vDt)/. . # t .

(17-49)

From Eq. 17-48, we have l # v/f. Then Eq. 17-49 becomes v

f, #

vD

λ

Figure 17-21 Wavefronts traveling to the right (a) reach and (b) pass detector D, which moves in the opposite direction. In time t, the wavefronts move a distance vt to the right and D moves a distance vDt to the left.

v " vD v " vD . #f v/f v

(17-50)

Note that in Eq. 17-50, f, > f unless vD # 0 (the detector is stationary). Similarly, we can find the frequency detected by D if D moves away from the source. In this situation, the wavefronts move a distance vt $ vDt relative to D in time t, and f, is given by f,# f

v $ vD . v

(17-51)

In Eq. 17-51, f, < f unless vD # 0. We can summarize Eqs. 17-50 and 17-51 with f,# f

v ! vD v

(detector moving, source stationary).

(17-52)

17-7 TH E DOPPLE R E FFECT

Shift up: The source moves toward the detector. W1

Figure 17-22 A detector D is stationary, and a source S is moving toward it at speed vS. Wavefront W1 was emitted when the source was at S1, wavefront W7 when it was at S7. At the moment depicted, the source is at S. The detector senses a higher frequency because the moving source, chasing its own wavefronts, emits a reduced wavelength l, in the direction of its motion.

W2 W7

S1

vS

S7

vD = 0

x

S λ'

D

Source Moving, Detector Stationary Let detector D be stationary with respect to the body of air, and let source S move toward D at speed vS (Fig. 17-22). The motion of S changes the wavelength of the sound waves it emits and thus the frequency detected by D. To see this change, let T (# 1/f ) be the time between the emission of any pair of successive wavefronts W1 and W2. During T, wavefront W1 moves a distance vT and the source moves a distance vST. At the end of T, wavefront W2 is emitted. In the direction in which S moves, the distance between W1 and W2, which is the wavelength l, of the waves moving in that direction, is vT $ vST. If D detects those waves, it detects frequency f, given by v v v f, # # # ., vT $ vST v/f $ vS /f #f

v . v $ vS

(17-53)

Note that f, must be greater than f unless vS # 0. In the direction opposite that taken by S, the wavelength l, of the waves is again the distance between successive waves but now that distance is vT " vST. If D detects those waves, it detects frequency f, given by v . (17-54) f, # f v " vS Now f, must be less than f unless vS # 0. We can summarize Eqs. 17-53 and 17-54 with v (source moving, detector stationary). f, # f v ! vS

(17-55)

General Doppler Effect Equation We can now derive the general Doppler effect equation by replacing f in Eq. 17-55 (the source frequency) with f, of Eq. 17-52 (the frequency associated with motion of the detector). That simple replacement gives us Eq. 17-47 for the general Doppler effect. That general equation holds not only when both detector and source are moving but also in the two specific situations we just discussed. For the situation in which the detector is moving and the source is stationary, substitution of vS # 0 into Eq. 17-47 gives us Eq. 17-52, which we previously found. For the situation in which the source is moving and the detector is stationary, substitution of vD # 0 into Eq. 17-47 gives us Eq. 17-55, which we previously found. Thus, Eq. 17-47 is the equation to remember.

501

502

CHAPTE R 17 WAVES—I I

Checkpoint 4

Source

The figure indicates the directions of motion of a sound source and a detector for six situations in stationary air. For each situation, is the detected frequency greater than or less than the emitted frequency, or can’t we tell without more information about the actual speeds?

Detector

Source

Detector

(a) 99:

• 0 speed

(d ) ;99

;99

(b) ;99

• 0 speed

(e) 99:

;99

(f ) ;99

99:

(c) 99:

99:

Sample Problem 17.07 Double Doppler shift in the echoes used by bats Bats navigate and search out prey by emitting, and then detecting reflections of, ultrasonic waves, which are sound waves with frequencies greater than can be heard by a human. Suppose a bat emits ultrasound at frequency fbe # 82.52 kHz while flying with velocity : vb # (9.00 m/s)iˆ as it chases a moth that flies with velocity : vm # (8.00 m/s)iˆ. What frequency fmd does the moth detect? What frequency fbd does the bat detect in the returning echo from the moth? KEY IDEAS The frequency is shifted by the relative motion of the bat and moth. Because they move along a single axis, the shifted frequency is given by Eq. 17-47. Motion toward tends to shift the frequency up, and motion away tends to shift it down. Detection by moth: The general Doppler equation is v ! vD . f, # f v ! vS

(17-56)

Here, the detected frequency f, that we want to find is the frequency fmd detected by the moth. On the right side, the emitted frequency f is the bat’s emission frequency fbe # 82.52 kHz, the speed of sound is v # 343 m/s, the speed vD of the detector is the moth’s speed vm # 8.00 m/s, and the speed vS of the source is the bat’s speed vb # 9.00 m/s. The decisions about the plus and minus signs can be tricky. Think in terms of toward and away. We have the speed of the moth (the detector) in the numerator of Eq. 17 -56. The moth moves away from the bat, which tends to lower the detected frequency. Because the speed is in the

numerator, we choose the minus sign to meet that tendency (the numerator becomes smaller). These reasoning steps are shown in Table 17 -3. We have the speed of the bat in the denominator of Eq. 17 -56. The bat moves toward the moth, which tends to increase the detected frequency. Because the speed is in the denominator, we choose the minus sign to meet that tendency (the denominator becomes smaller). With these substitutions and decisions, we have fmd # fbe

v $ vm v $ vb

# (82.52 kHz)

343 m/s $ 8.00 m/s 343 m/s $ 9.00 m/s

# 82.767 kHz % 82.8 kHz.

(Answer)

Detection of echo by bat: In the echo back to the bat, the moth acts as a source of sound, emitting at the frequency fmd we just calculated. So now the moth is the source (moving away) and the bat is the detector (moving toward). The reasoning steps are shown in Table 17 -3. To find the frequency fbd detected by the bat, we write Eq. 17 -56 as fbd # fmd

v " vb v " vm

343 m/s " 9.00 m/s 343 m/s " 8.00 m/s # 83.00 kHz % 83.0 kHz. # (82.767 kHz)

(Answer)

Some moths evade bats by “jamming” the detection system with ultrasonic clicks.

Table 17-3 Echo Back to Bat

Bat to Moth Detector

Source

Detector

Source

moth speed vD # vm away shift down numerator minus

bat speed vS # vb toward shift up denominator minus

bat speed vD # vb toward shift up numerator plus

moth speed vS # vm away shift down denominator plus

Additional examples, video, and practice available at WileyPLUS

503

17-8 SU PE RSON IC SPE E DS, SHOCK WAVES

17-8 SUPERSONIC SPEEDS, SHOCK WAVES Learning Objectives After reading this module, you should be able to . . .

17.35 Sketch the bunching of wavefronts for a sound source traveling at the speed of sound or faster. 17.36 Calculate the Mach number for a sound source exceeding the speed of sound.

17.37 For a sound source exceeding the speed of sound, apply the relationship between the Mach cone angle, the speed of sound, and the speed of the source.

Key Idea ● If the speed of a source relative to the medium exceeds the speed of sound in the medium, the Doppler equation no longer applies. In such a case, shock waves result. The half-angle u of the Mach cone is given by

sin 5 #

v vS

(Mach cone angle).

Supersonic Speeds, Shock Waves If a source is moving toward a stationary detector at a speed vS equal to the speed of sound v, Eqs. 17-47 and 17-55 predict that the detected frequency f, will be infinitely great. This means that the source is moving so fast that it keeps pace with its own spherical wavefronts (Fig. 17-23a). What happens when vS > v? For such supersonic speeds, Eqs. 17-47 and 17-55 no longer apply. Figure 17-23b depicts the spherical wavefronts that originated at various positions of the source. The radius of any wavefront is vt, where t is the time that has elapsed since the source emitted that wavefront. Note that all the wavefronts bunch along a V-shaped envelope in this two-dimensional drawing. The wavefronts actually extend in three dimensions, and the bunching actually forms a cone called the Mach cone. A shock wave exists along the surface of this cone, because the bunching of wavefronts causes an abrupt rise and fall of air pressure as the surface passes through any point. From Fig. 17-23b, we see that the half-angle u of the cone (the Mach cone angle) is given by sin 5 #

v vt # vS t vS

(Mach cone angle).

W1 vt

S1

Surface of Mach cone W6 S6

vS

θ S

(17-57)

(b)

Figure 17-24 Shock waves produced by the wings of a Navy FA 18 jet. The shock waves are visible because the sudden decrease in air pressure in them caused water molecules in the air to condense, forming a fog.

x

(a)

The ratio vS /v is the Mach number. If a plane flies at Mach 2.3, its speed is 2.3 times the speed of sound in the air through which the plane is flying. The shock wave generated by a supersonic aircraft (Fig. 17-24)

U.S. Navy photo by Ensign John Gay

vS

S

vSt

Figure 17-23 (a) A source of sound S moves at speed vS equal to the speed of sound and thus as fast as the wavefronts it generates. (b) A source S moves at speed vS faster than the speed of sound and thus faster than the wavefronts. When the source was at position S1 it generated wavefront W1, and at position S6 it generated W6. All the spherical wavefronts expand at the speed of sound v and bunch along the surface of a cone called the Mach cone, forming a shock wave. The surface of the cone has half-angle u and is tangent to all the wavefronts.

x

504

CHAPTE R 17 WAVES—I I

or projectile produces a burst of sound, called a sonic boom, in which the air pressure first suddenly increases and then suddenly decreases below normal before returning to normal. Part of the sound that is heard when a rifle is fired is the sonic boom produced by the bullet. When a long bull whip is snapped, its tip is moving faster than sound and produces a small sonic boom — the crack of the whip.

Review & Summary Sound Waves Sound waves are longitudinal mechanical waves that can travel through solids, liquids, or gases. The speed v of a sound wave in a medium having bulk modulus B and density r is B v# A 3

(speed of sound).

(17-3)

In air at 20-C, the speed of sound is 343 m/s. A sound wave causes a longitudinal displacement s of a mass element in a medium as given by s # sm cos(kx $ vt),

(17-12)

where sm is the displacement amplitude (maximum displacement) from equilibrium, k # 2p/l, and v # 2pf, l and f being the wavelength and frequency of the sound wave. The wave also causes a pressure change %p from the equilibrium pressure: %p # %pm sin(kx $ vt),

(17-13)

where the pressure amplitude is %pm # (vrv)sm.

(17-14)

Interference The interference of two sound waves with identical wavelengths passing through a common point depends on their phase difference f there. If the sound waves were emitted in phase and are traveling in approximately the same direction, f is given by f#

%L 2p, l

(17-21)

where %L is their path length difference (the difference in the distances traveled by the waves to reach the common point). Fully constructive interference occurs when f is an integer multiple of 2p, for m # 0, 1, 2, . . . ,

f # m(2p),

(17-22)

(17-23)

Fully destructive interference occurs when f is an odd multiple of p, f # (2m " 1)p,

for m # 0, 1, 2, . . . ,

(17-24)

and, equivalently, when %L is related to l by %L # 0.5, 1.5, 2.5, . . . . l

I # 123vv 2s 2m .

(17-27)

The intensity at a distance r from a point source that emits sound waves of power Ps is Ps I# . (17-28) 4pr 2

Sound Level in Decibels The sound level b in decibels (dB) is defined as b # (10 dB) log

I , I0

(17-29)

where I0 (# 10$12 W/m2) is a reference intensity level to which all intensities are compared. For every factor-of-10 increase in intensity, 10 dB is added to the sound level.

Standing Wave Patterns in Pipes Standing sound wave patterns can be set up in pipes. A pipe open at both ends will resonate at frequencies nv v , n # 1, 2, 3, . . . , (17-39) # f# . 2L where v is the speed of sound in the air in the pipe. For a pipe closed at one end and open at the other, the resonant frequencies are v nv , n # 1, 3, 5, . . . . (17-41) f# # . 4L

Beats Beats arise when two waves having slightly different frequencies, f1 and f2, are detected together. The beat frequency is fbeat # f 1 $ f 2.

(17-46)

The Doppler Effect The Doppler effect is a change in the

and, equivalently, when %L is related to wavelength l by %L # 0, 1, 2, . . . . l

and A is the area of the surface intercepting the sound. The intensity I is related to the displacement amplitude sm of the sound wave by

(17-25)

Sound Intensity The intensity I of a sound wave at a surface is the average rate per unit area at which energy is transferred by the wave through or onto the surface: P I# , (17-26) A where P is the time rate of energy transfer (power) of the sound wave

observed frequency of a wave when the source or the detector moves relative to the transmitting medium (such as air). For sound the observed frequency f, is given in terms of the source frequency f by v ! vD f, # f (general Doppler effect), (17-47) v ! vS where vD is the speed of the detector relative to the medium, vS is that of the source, and v is the speed of sound in the medium. The signs are chosen such that f, tends to be greater for motion toward and less for motion away.

Shock Wave If the speed of a source relative to the medium exceeds the speed of sound in the medium, the Doppler equation no longer applies. In such a case, shock waves result. The half-angle u of the Mach cone is given by v sin u # (Mach cone angle). (17-57) vS

QU ESTIONS

505

Questions 1 In a first experiment, a sinusoidal sound wave is sent through a long tube of air, transporting energy at the average rate of Pavg,1. In a second experiment, two other sound waves, identical to the first one, are to be sent simultaneously through the tube with a phase difference f of either 0, 0.2 wavelength, or 0.5 wavelength between the waves. (a) With only mental calculation, rank those choices of f according to the average rate at which the waves will transport energy, greatest first. (b) For the first choice of f, what is the average rate in terms of Pavg,1? 2 In Fig. 17-25, two point sources S1 L1 S1 and S2, which are in phase, emit P identical sound waves of wave- S2 L2 length 2.0 m. In terms of waveFigure 17-25 Question 2. lengths, what is the phase difference between the waves arriving at point P if (a) L1 # 38 m and L2 # 34 m, and (b) L1 # 39 m and L2 # 36 m? (c) Assuming that the source separation is much smaller than L1 and L2, what type of interference occurs at P in situations (a) and (b)? 3 In Fig. 17-26, three long tubes (A, B, and C) are filled with different gases under different pressures. The ratio of the bulk modulus to the density is indicated for each gas in terms of a basic value B0 /r0. Each tube has a piston at its left end that can send a sound pulse through the tube (as in Fig. 16-2). The three pulses are sent simultaneously. Rank the tubes according to the time of arrival of the pulses at the open right ends of the tubes, earliest first.

L

L

L

16B 0/ρ 0

6 Pipe A has length L and one open end. Pipe B has length 2L and two open ends. Which harmonics of pipe B have a frequency that matches a resonant frequency of pipe A? 7 Figure 17-28 shows a moving sound source S that emits at a certain frequency, and four stationary sound detectors. Rank the detectors according to the frequency of the sound they detect from the source, greatest first. 3 4 2

Figure 17-28 Question 7. 8 A friend rides, in turn, the rims of three fast merry-go-rounds while holding a sound source that emits isotropically at a certain frequency. You stand far from each merry-go-round. The frequency you hear for each of your friend’s three rides varies as the merry-goround rotates. The variations in frequency for the three rides are given by the three curves in Fig. 17-29. Rank the curves according to (a) the linear speed v of the sound source, (b) the angular speeds v of the merry-go-rounds, and (c) the radii r of the merry-go-rounds, greatest first.

L

A

1

S

f 3

4B 0/ρ 0

B0/ρ 0

1

5 In Fig. 17-27, pipe A is made to oscillate in its third harmonic by a small internal sound source. Sound emitted at the right end happens to resonate four nearby pipes, each with only one open end (they are not drawn to scale). Pipe B oscillates in its lowest harmonic, pipe C in its second lowest harmonic, pipe D in its third lowest harmonic, and pipe E in its fourth lowest harmonic. Without computation, rank all five pipes according to their length, greatest first. (Hint: Draw the standing waves to scale and then draw the pipes to scale.) B

9 For a particular tube, here are four of the six harmonic frequencies below 1000 Hz: 300, 600, 750, and 900 Hz. What two frequencies are missing from the list? 10 Figure 17-30 shows a stretched string of length L and pipes a, b, c, and d of lengths L, 2L, L/2, and L/2, respectively. The string’s tension is adjusted until the speed of waves on the string equals the speed of sound waves in the air. The fundamental mode of oscillation is then set up on the string. In which pipe will the sound produced by the string cause resonance, and what oscillation mode will that sound set up? L b

a

c

d

C

Figure 17-30 Question 10.

D

11 You are given four tuning forks. The fork with the lowest frequency oscillates at 500 Hz. By striking two tuning forks at a time, you can produce the following beat frequencies, 1, 2, 3, 5, 7, and 8 Hz. What are the possible frequencies of the other three forks? (There are two sets of answers.)

E

Figure 17-27 Question 5.

2

Figure 17-29 Question 8.

C

Figure 17-26 Question 3. 4 The sixth harmonic is set up in a pipe. (a) How many open ends does the pipe have (it has at least one)? (b) Is there a node, antinode, or some intermediate state at the midpoint?

A

t

L

B

506

CHAPTE R 17 WAVES—I I

Problems Tutoring problem available (at instructor’s discretion) in WileyPLUS and WebAssign SSM • – •••

Worked-out solution available in Student Solutions Manual

WWW Worked-out solution is at

Number of dots indicates level of problem difficulty

ILW

Interactive solution is at

http://www.wiley.com/college/halliday

Additional information available in The Flying Circus of Physics and at flyingcircusofphysics.com

Where needed in the problems, use speed of sound in air # 343 m/s and

density of air # 1.21 kg/m3

unless otherwise specified. Module 17-1 Speed of Sound •1 Two spectators at a soccer game see, and a moment later hear, the ball being kicked on the playing field. The time delay for spectator A is 0.23 s, and for spectator B it is 0.12 s. Sight lines from the two spectators to the player kicking the ball meet at an angle of 90-. How far are (a) spectator A and (b) spectator B from the player? (c) How far are the spectators from each other? •2 What is the bulk modulus of oxygen if 32.0 g of oxygen occupies 22.4 L and the speed of sound in the oxygen is 317 m/s? •3 When the door of the Chapel of the Mausoleum in Hamilton, Scotland, is slammed shut, the last echo heard by someone standing just inside the door reportedly comes 15 s later. (a) If that echo were due to a single reflection off a wall opposite the door, how far from the door is the wall? (b) If, instead, the wall is 25.7 m away, how many reflections (back and forth) occur? •4 A column of soldiers, marching at 120 paces per minute, keep in step with the beat of a drummer at the head of the column. The soldiers in the rear end of the column are striding forward with the left foot when the drummer is advancing with the right foot. What is the approximate length of the column? ••5 SSM ILW Earthquakes generate sound waves inside Earth. Unlike a gas, Earth can experience both transverse (S) and longitudinal (P) sound waves. Typically, the speed of S waves is about 4.5 km/s, and that of P waves 8.0 km/s. A seismograph records P and S waves from an earthquake. The first P waves arrive 3.0 min before the first S waves. If the waves travel in a straight line, how far away did the earthquake occur? ••6 A man strikes one end of a thin rod with a hammer. The speed of sound in the rod is 15 times the speed of sound in air. A woman, at the other end with her ear close to the rod, hears the sound of the blow twice with a 0.12 s interval between; one sound comes through the rod and the other comes through the air alongside the rod. If the speed of sound in air is 343 m/s, what is the length of the rod? ••7 SSM WWW A stone is dropped into a well. The splash is heard 3.00 s later. What is the depth of the well? ••8 Hot chocolate effect. Tap a metal spoon inside a mug of water and note the frequency fi you hear. Then add a spoonful of powder (say, chocolate mix or instant coffee) and tap again as you stir the powder. The frequency you hear has a lower value fs because the tiny air bubbles released by the powder change the water’s bulk modulus. As the bubbles reach the water surface and disappear, the frequency gradually shifts back to its initial value. During the effect, the bubbles don’t appreciably change the water’s density or volume or the sound’s wavelength.

Rather, they change the value of dV/dp — that is, the differential change in volume due to the differential change in the pressure caused by the sound wave in the water. If fs /fi # 0.333, what is the ratio (dV/dp)s /(dV/dp)i? Module 17-2 Traveling Sound Waves •9 If the form of a sound wave traveling through air is s(x, t) # (6.0 nm) cos(kx " (3000 rad/s)t " f), how much time does any given air molecule along the path take to move between displacements s # "2.0 nm and s # $2.0 nm? •10 Underwater illusion. One clue used by your brain to determine Wavefronts the direction of a source of sound is the time delay %t between the arrival d of the sound at the ear closer to the θ source and the arrival at the farther θ ear. Assume that the source is distant D R L so that a wavefront from it is approxFigure 17-31 Problem 10. imately planar when it reaches you, and let D represent the separation between your ears. (a) If the source is located at angle u in front of you (Fig. 17-31), what is %t in terms of D and the speed of sound v in air? (b) If you are submerged in water and the sound source is directly to your right, what is %t in terms of D and the speed of sound vw in water? (c) Based on the time-delay clue, your brain interprets the submerged sound to arrive at an angle u from the forward direction. Evaluate u for fresh water at 20-C. •11 SSM Diagnostic ultrasound of frequency 4.50 MHz is used to examine tumors in soft tissue. (a) What is the wavelength in air of such a sound wave? (b) If the speed of sound in tissue is 1500 m/s, what is the wavelength of this wave in tissue? •12 The pressure in a traveling sound wave is given by the equation %p # (1.50 Pa) sin p[(0.900 m$1) x $ (315 s$1)t]. Find the (a) pressure amplitude, (b) frequency, (c) wavelength, and (d) speed of the wave. ••13 A sound wave of the form s # sm cos(kx $ vt " f) travels at 343 m/s through air in a long horizontal tube. At one instant, air molecule A at x # 2.000 m is at its maximum positive displace∆p (mPa) ment of 6.00 nm and air molecule B at x # 2.070 m is at a pos∆ps itive displacement of 2.00 nm. All the molecules between A and B are at intermediate dis- –1 1 2 placements. What is the fre–∆ps quency of the wave? ••14 Figure 17-32 shows the output from a pressure monitor mounted at a point along the

t (ms)

Figure 17-32 Problem 14.

507

PROB LE M S

path taken by a sound wave of a single frequency traveling at 343 m/s through air with a uniform density of 1.21 kg/m3. The vertical axis scale is set by %ps # 4.0 mPa. If the displacement function of the wave is s(x, t) # sm cos(kx $ vt), what are (a) sm, (b) k, and (c) v? The air is then cooled so that its density is 1.35 kg/m3 and the speed of a sound wave through it is 320 m/s. The sound source again emits the sound wave at the same frequency and same pressure amplitude. What now are (d) sm, (e) k, and (f ) v? ••15 A handclap on stage in an amphitheater sends out sound waves that scatter from terraces of width w # 0.75 m (Fig. 17-33). The sound returns to the stage as a periodic series of pulses, one from each terrace; the parade of pulses sounds like a played note. (a) Assuming that all the rays in Fig. 17-33 are horizontal, find the frequency at which the pulses return (that is, the frequency of the perceived note). (b) If the width w of the terraces were smaller, would the frequency be higher or lower?

ql.What are the (a) smallest and (b) second smallest values of q that put A and B exactly out of phase with each other after the reflections? S

S

1 2 ••19 Figure 17-35 shows two D isotropic point sources of sound, S1 Figure 17-35 and S2. The sources emit waves in Problems 19 and 105. phase at wavelength 0.50 m; they are separated by D # 1.75 m. If we move a sound detector along a large circle centered at the midpoint between the sources, at how many points do waves arrive at the detector (a) exactly in phase and (b) exactly out of phase?

••20 Figure 17-36 shows four isotropic point sources of sound that are uniformly spaced on an x axis. The sources emit sound at the same wavelength l and same amplitude sm, and they emit in phase. A point P is shown on the x axis. Assume that as the sound waves travel to P, the decrease in their amplitude is negligible. What multiple of sm is the amplitude of the net wave at P if distance d in the figure is (a) l/4, (b) l/2, and (c) l? S2

S1 d

S3 d

S4 P

d

Figure 17-36 Problem 20.

w Terrace

Figure 17-33 Problem 15. Module 17-3 Interference •16 Two sound waves, from two different sources with the same frequency, 540 Hz, travel in the same direction at 330 m/s. The sources are in phase. What is the phase difference of the waves at a point that is 4.40 m from one source and 4.00 m from the other? ••17 ILW Two loud speakers are located 3.35 m apart on an outdoor stage. A listener is 18.3 m from one and 19.5 m from the other. During the sound check, a signal generator drives the two speakers in phase with the same amplitude and frequency. The transmitted frequency is swept through the audible range (20 Hz to 20 kHz). (a) What is the lowest frequency fmin,1 that gives minimum signal (destructive interference) at the listener’s location? By what number must fmin,1 be multiplied to get (b) the second lowest frequency fmin,2 that gives minimum signal and (c) the third lowest frequency fmin,3 that gives minimum signal? (d) What is the lowest frequency fmax,1 that gives maximum signal (constructive interference) at the listener’s location? By what number must fmax,1 be multiplied to get (e) the second lowest frequency fmax,2 that gives maximum signal and (f) the third lowest frequency fmax,3 that gives maximum signal? ••18 In Fig. 17-34, sound waves A and B, both of wavelength l, are initially in phase and traveling rightward, as indicated by the two rays. Wave A is reflected from four surfaces but ends up traveling in its original direction.Wave B ends in that direction after reflecting from two surfaces. Let distance L in the figure be expressed as a multiple q of l: L #

L A L B L

Figure 17-34 Problem 18.

••21 SSM In Fig. 17-37, two speakers separated by distance d1 # 2.00 m are in phase. Assume the amplitudes of Speakers the sound waves from the speakers d 1 are approximately the same at the lisListener tener’s ear at distance d2 # 3.75 m directly in front of one speaker. d2 Consider the full audible range for Figure 17-37 Problem 21. normal hearing, 20 Hz to 20 kHz. (a) What is the lowest frequency fmin,1 that gives minimum signal (destructive interference) at the listener’s ear? By what number must fmin,1 be multiplied to get (b) the second lowest frequency fmin,2 that gives minimum signal and (c) the third lowest frequency fmin,3 that gives minimum signal? (d) What is the lowest frequency fmax,1 that gives maximum signal (constructive interference) at the listener’s ear? By what number must fmax,1 be multiplied to get (e) the second lowest frequency fmax,2 that gives maximum signal and (f ) the third lowest frequency fmax,3 that gives maximum signal? ••22 In Fig. 17-38, sound with a r 40.0 cm wavelength travels rightward from a source and through a Source Detector tube that consists of a straight porFigure 17-38 Problem 22. tion and a half-circle. Part of the sound wave travels through the halfcircle and then rejoins the rest of the y wave, which goes directly through the straight portion. This rejoining results in interference. What is the S1 P smallest radius r that results in an inx tensity minimum at the detector? •••23 Figure 17-39 shows two point sources S1 and S2 that emit sound of wavelength l # 2.00 m. The emissions are isotropic and in phase, and the separation between

d

S2

Figure 17-39 Problem 23.

508

CHAPTE R 17 WAVES—I I

the sources is d # 16.0 m. At any point P on the x axis, the wave from S1 and the wave from S2 interfere. When P is very far away (x % 6), what are (a) the phase difference between the arriving waves from S1 and S2 and (b) the type of interference they produce? Now move point P along the x axis toward S1. (c) Does the phase difference between the waves increase or decrease? At what distance x do the waves have a phase difference of (d) 0.50l, (e) 1.00l, and (f ) 1.50l?

••34 Two atmospheric sound sources A and B emit isotropically at constant power. The sound levels b of their emissions are plotted in Fig. 17-40 versus the radial distance r from the sources. The vertical axis scale is set by b1 # 85.0 dB and b2 # 65.0 dB. What are (a) the ratio of the larger power to the smaller power and (b) the sound level difference at r # 10 m?

β1

•25 A sound wave of frequency 300 Hz has an intensity of 1.00 mW/m2. What is the amplitude of the air oscillations caused by this wave? •26 A 1.0 W point source emits sound waves isotropically. Assuming that the energy of the waves is conserved, find the intensity (a) 1.0 m from the source and (b) 2.5 m from the source. •27 SSM WWW A certain sound source is increased in sound level by 30.0 dB. By what multiple is (a) its intensity increased and (b) its pressure amplitude increased? •28 Two sounds differ in sound level by 1.00 dB. What is the ratio of the greater intensity to the smaller intensity? •29 SSM A point source emits sound waves isotropically. The intensity of the waves 2.50 m from the source is 1.91 ' 10$4 W/m2. Assuming that the energy of the waves is conserved, find the power of the source. •30 The source of a sound wave has a power of 1.00 mW. If it is a point source, (a) what is the intensity 3.00 m away and (b) what is the sound level in decibels at that distance? •31 When you “crack” a knuckle, you suddenly widen the knuckle cavity, allowing more volume for the synovial fluid inside it and causing a gas bubble suddenly to appear in the fluid. The sudden production of the bubble, called “cavitation,” produces a sound pulse — the cracking sound. Assume that the sound is transmitted uniformly in all directions and that it fully passes from the knuckle interior to the outside. If the pulse has a sound level of 62 dB at your ear, estimate the rate at which energy is produced by the cavitation. •32 Approximately a third of people with normal hearing have ears that continuously emit a low-intensity sound outward through the ear canal. A person with such spontaneous otoacoustic emission is rarely aware of the sound, except perhaps in a noisefree environment, but occasionally the emission is loud enough to be heard by someone else nearby. In one observation, the sound wave had a frequency of 1665 Hz and a pressure amplitude of 1.13 ' 10$3 Pa. What were (a) the displacement amplitude and (b) the intensity of the wave emitted by the ear? •33 Male Rana catesbeiana bullfrogs are known for their loud mating call. The call is emitted not by the frog’s mouth but by its eardrums, which lie on the surface of the head. And, surprisingly, the sound has nothing to do with the frog’s inflated throat. If the emitted sound has a frequency of 260 Hz and a sound level of 85 dB (near the eardrum), what is the amplitude of the eardrum’s oscillation? The air density is 1.21 kg/m3.

A

β (dB)

Module 17-4 Intensity and Sound Level •24 Suppose that the sound level of a conversation is initially at an angry 70 dB and then drops to a soothing 50 dB. Assuming that the frequency of the sound is 500 Hz, determine the (a) initial and (b) final sound intensities and the (c) initial and (d) final sound wave amplitudes.

B

β2

Figure 17-40 Problem 34.

100

500 r (m)

1000

••35 A point source emits 30.0 W of sound isotropically. A small microphone intercepts the sound in an area of 0.750 cm2, 200 m from the source. Calculate (a) the sound intensity there and (b) the power intercepted by the microphone. ••36 Party hearing. As the number of people at a party increases, you must raise your voice for a listener to hear you against the background noise of the other partygoers. However, once you reach the level of yelling, the only way you can be heard is if you move closer to your listener, into the listener’s “personal space.” Model the situation by replacing you with an isotropic point source of fixed power P and replacing your listener with a point that absorbs part of your sound waves. These points are initially separated by ri # 1.20 m. If the background noise increases by %b # 5 dB, the sound level at your listener must also increase. What separation rf is then required? •••37 A sound source sends a sinusoidal sound wave of angular frequency 3000 rad/s and amplitude 12.0 nm through a tube of air. The internal radius of the tube is 2.00 cm. (a) What is the average rate at which energy (the sum of the kinetic and potential energies) is transported to the opposite end of the tube? (b) If, simultaneously, an identical wave travels along an adjacent, identical tube, what is the total average rate at which energy is transported to the opposite ends of the two tubes by the waves? If, instead, those two waves are sent along the same tube simultaneously, what is the total average rate at which they transport energy when their phase difference is (c) 0, (d) 0.40p rad, and (e) p rad? Module 17-5 Sources of Musical Sound •38 The water level in a vertical glass tube 1.00 m long can be adjusted to any position in the tube. A tuning fork vibrating at 686 Hz is held just over the open top end of the tube, to set up a standing wave of sound in the air-filled top portion of the tube. (That airfilled top portion acts as a tube with one end closed and the other end open.) (a) For how many different positions of the water level will sound from the fork set up resonance in the tube’s air-filled portion? What are the (b) least and (c) second least water heights in the tube for resonance to occur? •39 SSM ILW (a) Find the speed of waves on a violin string of mass 800 mg and length 22.0 cm if the fundamental frequency is 920 Hz. (b) What is the tension in the string? For the fundamental, what is the wavelength of (c) the waves on the string and (d) the sound waves emitted by the string?

PROB LE M S

509

•40 Organ pipe A, with both ends open, has a fundamental frequency of 300 Hz. The third harmonic of organ pipe B, with one end open, has the same frequency as the second harmonic of pipe A. How long are (a) pipe A and (b) pipe B?

one open end is 1080 Hz. The next-highest harmonic frequency is 1320 Hz. (c) What harmonic frequency is next highest after the harmonic frequency 600 Hz? (d) What is the number of this nexthighest harmonic?

•41 A violin string 15.0 cm long and fixed at both ends oscillates in its n # 1 mode. The speed of waves on the string is 250 m/s, and the speed of sound in air is 348 m/s. What are the (a) frequency and (b) wavelength of the emitted sound wave?

••49 SSM A violin string 30.0 cm long with linear density 0.650 g/m is placed near a loudspeaker that is fed by an audio oscillator of variable frequency. It is found that the string is set into oscillation only at the frequencies 880 and 1320 Hz as the frequency of the oscillator is varied over the range 500 – 1500 Hz. What is the tension in the string?

•42 A sound wave in a fluid medium is reflected at a barrier so that a standing wave is formed. The distance between nodes is 3.8 cm, and the speed of propagation is 1500 m/s. Find the frequency of the sound wave. •43 SSM In Fig. 17-41, S is a small loudspeaker driven by an audio oscillator with a frequency that is varied from 1000 Hz to 2000 Hz, and D is a cylindrical pipe with two open ends and a length of 45.7 cm. The speed of sound in the air-filled pipe is 344 m/s. (a) At how many frequencies does the sound from the loudspeaker set up resonance in the pipe? What are the (b) lowest and (c) second lowest frequencies at which resonance occurs?

S D

Figure 17-41 Problem 43.

•44 The crest of a Parasaurolophus dinosaur skull is shaped somewhat like a trombone and contains a nasal passage in the form of a long, bent tube open at both ends. The dinosaur may have used the passage to produce sound by setting up the fundamental mode in it. (a) If the nasal passage in a certain Parasaurolophus fossil is 2.0 m long, what frequency would have been produced? (b) If that dinosaur could be recreated (as in Jurassic Park), would a person with a hearing range of 60 Hz to 20 kHz be able to hear that fundamental mode and, if so, would the sound be high or low frequency? Fossil skulls that contain shorter nasal passages are thought to be those of the female Parasaurolophus. (c) Would that make the female’s fundamental frequency higher or lower than the male’s? •45 In pipe A, the ratio of a particular harmonic frequency to the next lower harmonic frequency is 1.2. In pipe B, the ratio of a particular harmonic frequency to the next lower harmonic frequency is 1.4. How many open ends are in (a) pipe A and (b) pipe B? ••46 Pipe A, which is 1.20 m long and open at both ends, oscillates at its third lowest harmonic frequency. It is filled with air for which the speed of sound is 343 m/s. Pipe B, which is closed at one end, oscillates at its second lowest harmonic frequency. This frequency of B happens to match the frequency of A. An x axis extends along the interior of B, with x # 0 at the closed end. (a) How many nodes are along that axis? What are the (b) smallest and (c) second smallest value of x locating those nodes? (d) What is the fundamental frequency of B? ••47 A well with vertical sides and water at the bottom resonates at 7.00 Hz and at no lower frequency. The air-filled portion of the well acts as a tube with one closed end (at the bottom) and one open end (at the top). The air in the well has a density of 1.10 kg/m3 and a bulk modulus of 1.33 ' 105 Pa. How far down in the well is the water surface? ••48 One of the harmonic frequencies of tube A with two open ends is 325 Hz. The next-highest harmonic frequency is 390 Hz. (a) What harmonic frequency is next highest after the harmonic frequency 195 Hz? (b) What is the number of this next-highest harmonic? One of the harmonic frequencies of tube B with only

••50 A tube 1.20 m long is closed at one end. A stretched wire is placed near the open end. The wire is 0.330 m long and has a mass of 9.60 g. It is fixed at both ends and oscillates in its fundamental mode. By resonance, it sets the air column in the tube into oscillation at that column’s fundamental frequency. Find (a) that frequency and (b) the tension in the wire. Module 17-6 Beats •51 The A string of a violin is a little too tightly stretched. Beats at 4.00 per second are heard when the string is sounded together with a tuning fork that is oscillating accurately at concert A (440 Hz). What is the period of the violin string oscillation? •52 A tuning fork of unknown frequency makes 3.00 beats per second with a standard fork of frequency 384 Hz. The beat frequency decreases when a small piece of wax is put on a prong of the first fork. What is the frequency of this fork? ••53 SSM Two identical piano wires have a fundamental frequency of 600 Hz when kept under the same tension. What fractional increase in the tension of one wire will lead to the occurrence of 6.0 beats/s when both wires oscillate simultaneously? ••54 You have five tuning forks that oscillate at close but different resonant frequencies. What are the (a) maximum and (b) minimum number of different beat frequencies you can produce by sounding the forks two at a time, depending on how the resonant frequencies differ? Module 17-7 The Doppler Effect •55 ILW A whistle of frequency 540 Hz moves in a circle of radius 60.0 cm at an angular speed of 15.0 rad/s. What are the (a) lowest and (b) highest frequencies heard by a listener a long distance away, at rest with respect to the center of the circle? •56 An ambulance with a siren emitting a whine at 1600 Hz overtakes and passes a cyclist pedaling a bike at 2.44 m/s. After being passed, the cyclist hears a frequency of 1590 Hz. How fast is the ambulance moving? •57 A state trooper chases a speeder along a straight road; both vehicles move at 160 km/h. The siren on the trooper’s vehicle produces sound at a frequency of 500 Hz. What is the Doppler shift in the frequency heard by the speeder? ••58 A sound source A and a reflecting surface B move directly toward each other. Relative to the air, the speed of source A is 29.9 m/s, the speed of surface B is 65.8 m/s, and the speed of sound is 329 m/s. The source emits waves at frequency 1200 Hz as measured in the source frame. In the reflector frame, what are the (a) frequency and (b) wavelength of the arriving sound waves? In the source frame, what are the (c) frequency and (d) wavelength of the sound waves reflected back to the source?

510

CHAPTE R 17 WAVES—I I

••59 In Fig. 17-42, a French submarine and a U.S. submarine move toward each other during maneuvers in motionless water in the North Atlantic. The French sub moves at speed vF # 50.00 km/h, and the U.S. sub at vUS # 70.00 km/h. The French sub sends out a sonar signal (sound wave in water) at 1.000 ' 103 Hz. Sonar waves travel at 5470 km/h. (a) What is the signal’s frequency as detected by the U.S. sub? (b) What frequency is detected by the French sub in the signal reflected back to it by the U.S. sub?

French

vF

v US

U.S.

Figure 17-42 Problem 59. ••60 A stationary motion detector sends sound waves of frequency 0.150 MHz toward a truck approaching at a speed of 45.0 m/s. What is the frequency of the waves reflected back to the detector? ••61 A bat is flitting about in a cave, navigating via ultrasonic bleeps. Assume that the sound emission frequency of the bat is 39 000 Hz. During one fast swoop directly toward a flat wall surface, the bat is moving at 0.025 times the speed of sound in air. What frequency does the bat hear reflected off the wall? ••62 Figure 17-43 shows four tubes with lengths 1.0 m or 2.0 m, with one or two open ends as drawn. The third harmonic is set up in each tube, and some of the sound that escapes from them is detected by detector D, which moves directly away from the tubes. In terms of the speed of sound v, 1 what speed must the detector 2 have such that the detected 3 D frequency of the sound from (a) tube 1, (b) tube 2, (c) tube 4 3, and (d) tube 4 is equal to the Figure 17-43 Problem 62. tube’s fundamental frequency? ••63 ILW An acoustic burglar alarm consists of a source emitting waves of frequency 28.0 kHz. What is the beat frequency between the source waves and the waves reflected from an intruder walking at an average speed of 0.950 m/s directly away from the alarm? ••64 A stationary detector measures the frequency of a sound source that first moves at constant velocity directly toward the detector and then (after passing the detector) directly away from it. The emitted frequency is f. During the approach the detected frequency is f,app and during the recession it is f,rec. If ( f,app $ f,rec)/f # 0.500, what is the ratio vs /v of the speed of the source to the speed of sound?

locomotive whistle emits sound at frequency 500.0 Hz. The air is still. (a) What frequency does the uncle hear? (b) What frequency does the girl hear? A wind begins to blow from the east at 10.00 m/s. (c) What frequency does the uncle now hear? (d) What frequency does the girl now hear? Module 17-8 Supersonic Speeds, Shock Waves •68 The shock wave off the cockpit of the FA 18 in Fig. 17-24 has an angle of about 60-. The airplane was traveling at about 1350 km/h when the photograph was taken. Approximately what was the speed of sound at the airplane’s altitude? ••69 SSM A jet plane passes over you at a height of 5000 m and a speed of Mach 1.5. (a) Find the Mach cone angle (the sound speed is 331 m/s). (b) How long after the jet passes directly overhead does the shock wave reach you? ••70 A plane flies at 1.25 times the speed of sound. Its sonic boom reaches a man on the ground 1.00 min after the plane passes directly overhead. What is the altitude of the plane? Assume the speed of sound to be 330 m/s. Additional Problems 71 At a distance of 10 km, a 100 Hz horn, assumed to be an isotropic point source, is barely audible. At what distance would it begin to cause pain? 72 A bullet is fired with a speed of 685 m/s. Find the angle made by the shock cone with the line of motion of the bullet. 73 A sperm whale (Fig. 17-44a) vocalizes by producing a series of clicks. Actually, the whale makes only a single sound near the front of its head to start the series. Part of that sound then emerges from the head into the water to become the first click of the series. The rest of the sound travels backward through the spermaceti sac (a body of fat), reflects from the frontal sac (an air layer), and then travels forward through the spermaceti sac. When it reaches the distal sac (another air layer) at the front of the head, some of the sound escapes into the water to form the second click, and the rest is sent back through the spermaceti sac (and ends up forming later clicks). Figure 17-44b shows a strip-chart recording of a series of clicks. A unit time interval of 1.0 ms is indicated on the chart. Assuming that the speed of sound in the spermaceti sac is 1372 m/s, find the length of the spermaceti sac. From such a calculation, marine scientists estimate the length of a whale from its click series. Spermaceti sac Distal sac

Frontal sac

•••65 A 2000 Hz siren and a civil defense official are both at rest with respect to the ground. What frequency does the official hear if the wind is blowing at 12 m/s (a) from source to official and (b) from official to source? •••66 Two trains are traveling toward each other at 30.5 m/s relative to the ground. One train is blowing a whistle at 500 Hz. (a) What frequency is heard on the other train in still air? (b) What frequency is heard on the other train if the wind is blowing at 30.5 m/s toward the whistle and away from the listener? (c) What frequency is heard if the wind direction is reversed? •••67 SSM WWW A girl is sitting near the open window of a train that is moving at a velocity of 10.00 m/s to the east. The girl’s uncle stands near the tracks and watches the train move away. The

1.0 ms

Figure 17-44 Problem 73.

PROB LE M S

74 The average density of Earth’s crust 10 km beneath the continents is 2.7 g/cm3. The speed of longitudinal seismic waves at that depth, found by timing their arrival from distant earthquakes, is 5.4 km/s. Find the bulk modulus of Earth’s crust at that depth. For comparison, the bulk modulus of steel is about 16 ' 1010 Pa. 75 A certain loudspeaker system emits sound isotropically with a frequency of 2000 Hz and an intensity of 0.960 mW/m2 at a distance of 6.10 m. Assume that there are no reflections. (a) What is the intensity at 30.0 m? At 6.10 m, what are (b) the displacement amplitude and (c) the pressure amplitude? 76 Find the ratios (greater to smaller) of the (a) intensities, (b) pressure amplitudes, and (c) particle displacement amplitudes for two sounds whose sound levels differ by 37 dB. 77 In Fig. 17-45, sound waves A and B, both of wavelength l, are initially in phase and traveling rightA ward, as indicated by the two rays. Wave A is reflected from four surL faces but ends up traveling in its original direction. What multiple of L wavelength l is the smallest value of distance L in the figure that puts A and B exactly out of phase with each B other after the reflections? 78 A trumpet player on a moving Figure 17-45 Problem 77. railroad flatcar moves toward a second trumpet player standing alongside the track while both play a 440 Hz note. The sound waves heard by a stationary observer between the two players have a beat frequency of 4.0 beats/s. What is the flatcar’s speed? 79 In Fig. 17-46, sound of wavelength 0.850 m is emitted isotropically by point source S. Sound ray 1 extends directly to detector D, at distance L # 10.0 m. Sound ray 2 extends to D via a reflection (effectively, a “bouncing”) of the sound at a flat surface. That reflection occurs on a perpendicular bisector to the SD line, at distance d from the line. Assume that the reflection shifts the sound wave by 0.500l. For what least value of d (other than zero) do the direct sound and the reflected sound arrive at D (a) exactly out of phase and (b) exactly in phase?

Ray 2

d

S L __ 2

Ray 1

D L __ 2

Figure 17-46 Problem 79. 80 A detector initially moves at constant velocity directly toward a stationary sound source and then (after passing it) directly from it. The emitted frequency is f. During the approach the detected frequency is f,app and during the recession it is f,rec. If the frequencies are related by (f,app $ f,rec)/f # 0.500, what is the ratio vD /v of the speed of the detector to the speed of sound? 81 SSM (a) If two sound waves, one in air and one in (fresh) water, are equal in intensity and angular frequency, what is the ratio of the pressure amplitude of the wave in water to that of the wave in air? Assume the water and the air are at 20-C. (See Table 14-1.) (b) If the pressure amplitudes are equal instead, what is the ratio of the intensities of the waves?

511

82 A continuous sinusoidal longitudinal wave is sent along a very long coiled spring from an attached oscillating source. The wave travels in the negative direction of an x axis; the source frequency is 25 Hz; at any instant the distance between successive points of maximum expansion in the spring is 24 cm; the maximum longitudinal displacement of a spring particle is 0.30 cm; and the particle at x # 0 has zero displacement at time t # 0. If the wave is written in the form s(x, t) # sm cos(kx ! vt), what are (a) sm, (b) k, (c) v, (d) the wave speed, and (e) the correct choice of sign in front of v? Incident ultrasound

83 SSM Ultrasound, which consists θ of sound waves with frequencies above the human audible range, can Artery be used to produce an image of the interior of a human body. Moreover, ultrasound can be used to measure Figure 17-47 Problem 83. the speed of the blood in the body; it does so by comparing the frequency of the ultrasound sent into the body with the frequency of the ultrasound reflected back to the body’s surface by the blood. As the blood pulses, this detected frequency varies. Suppose that an ultrasound image of the arm of a patient shows an artery that is angled at u # 20- to the ultrasound’s line of travel (Fig. 17-47). Suppose also that the frequency of the ultrasound reflected by the blood in the artery is increased by a maximum of 5495 Hz from the original ultrasound frequency of 5.000 000 MHz. (a) In Fig. 17-47, is the direction of the blood flow rightward or leftward? (b) The speed of sound in the human arm is 1540 m/s. What is the maximum speed of the blood? (Hint: The Doppler effect is caused by the component of the blood’s velocity along the ultrasound’s direction of travel.) (c) If angle u were greater, would the reflected frequency be greater or less? 84 The speed of sound in a certain metal is vm. One end of a long pipe of that metal of length L is struck a hard blow. A listener at the other end hears two sounds, one from the wave that travels along the pipe’s metal wall and the other from the wave that travels through the air inside the pipe. (a) If v is the speed of sound in air, what is the time interval %t between the arrivals of the two sounds at the listener’s ear? (b) If %t # 1.00 s and the metal is steel, what is the length L? 85 An avalanche of sand along some rare desert sand dunes can produce a booming that is loud enough to be heard 10 km away. The booming apparently results from a periodic oscillation of the sliding layer of sand — the layer’s thickness expands and contracts. If the emitted frequency is 90 Hz, what are (a) the period of the thickness oscillation and (b) the wavelength of the sound? 86 A sound source moves along an x axis, between detectors A and B. The wavelength of the sound detected at A is 0.500 that of the sound detected at B. What is the ratio 7s /7 of the speed of the source to the speed of sound? 87 SSM A siren emitting a sound of frequency 1000 Hz moves away from you toward the face of a cliff at a speed of 10 m/s. Take the speed of sound in air as 330 m/s. (a) What is the frequency of the sound you hear coming directly from the siren? (b) What is the frequency of the sound you hear reflected off the cliff? (c) What is the beat frequency between the two sounds? Is it perceptible (less than 20 Hz)? 88 At a certain point, two waves produce pressure variations given by %p1 # %pm sin vt and %p2 # %pm sin(vt $ f).At this point,

512

CHAPTE R 17 WAVES—I I

what is the ratio %pr /%pm, where %pr is the pressure amplitude of the resultant wave, if f is (a) 0, (b) p/2, (c) p/3, and (d) p/4? 89 Two sound waves with an amplitude of 12 nm and a wavelength of 35 cm travel in the same direction through a long tube, with a phase difference of p/3 rad. What are the (a) amplitude and (b) wavelength of the net sound wave produced by their interference? If, instead, the sound waves travel through the tube in opposite directions, what are the (c) amplitude and (d) wavelength of the net wave? 90 A sinusoidal sound wave moves at 343 m/s through air in the positive direction of an x axis. At one instant during the oscillations, air molecule A is at its maximum displacement in the negative direction of the axis while air molecule B is at its equilibrium position. The separation between those molecules is 15.0 cm, and the molecules between A and B have intermediate displacements in the negative direction of the axis. (a) What is the frequency of the sound wave? In a similar arrangement but for a different sinusoidal sound wave, at one instant air molecule C is at its maximum displacement in the positive direction while molecule D is at its maximum displacement in the negative direction. The separation between the molecules is again 15.0 cm, and the molecules between C and D have intermediate displacements. (b) What is the frequency of the sound wave? 91 Two identical tuning forks can oscillate at 440 Hz. A person is located somewhere on the line between them. Calculate the beat frequency as measured by this individual if (a) she is standing still and the tuning forks move in the same direction along the line at 3.00 m/s, and (b) the tuning forks are stationary and the listener moves along the line at 3.00 m/s. 92 You can estimate your distance from a lightning stroke by counting the seconds between the flash you see and the thunder you later hear. By what integer should you divide the number of seconds to get the distance in kilometers? S 93 SSM Figure 17-48 shows an airfilled, acoustic interferometer, used to demonstrate the interference of A B sound waves. Sound source S is an oscillating diaphragm; D is a sound detector, such as the ear or a microD phone. Path SBD can be varied in length, but path SAD is fixed. At D, Figure 17-48 Problem 93. the sound wave coming along path SBD interferes with that coming along path SAD. In one demonstration, the sound intensity at D has a minimum value of 100 units at one position of the movable arm and continuously climbs to a maximum value of 900 units when that arm is shifted by 1.65 cm. Find (a) the frequency of the sound emitted by the source and (b) the ratio of the amplitude at D of the SAD wave to that of the SBD wave. (c) How can it happen that these waves have different amplitudes, considering that they originate at the same source?

94 On July 10, 1996, a granite block broke away from a wall in Yosemite Valley and, as it began to slide down the wall, was launched into projectile motion. Seismic waves produced by its impact with the ground triggered seismographs as far away as 200 km. Later measurements indicated that the block had a mass between 7.3 ' 107 kg and 1.7 ' 108 kg and that it landed 500 m vertically below the launch point and 30 m horizontally from it.

(The launch angle is not known.) (a) Estimate the block’s kinetic energy just before it landed. Consider two types of seismic waves that spread from the impact point — a hemispherical body wave traveled through the ground in an expanding hemisphere and a cylindrical surface wave traveled along the ground in an expanding shallow vertical cylinder (Fig. 17-49). Assume that the impact lasted 0.50 s, the vertical cylinder had a depth d of 5.0 m, and each wave type received 20% of the energy the block had just before impact. Neglecting any mechanical energy loss the waves experienced as they traveled, determine the intensities of (b) the body wave and (c) the surface wave when they reached a seismograph 200 km away. (d) On the basis of these results, which wave is more easily detected on a distant seismograph? Cylindrical wave

Impact d

Hemispherical wave

Figure 17-49 Problem 94. 95 SSM The sound intensity is 0.0080 W/m2 at a distance of 10 m from an isotropic point source of sound. (a) What is the power of the source? (b) What is the sound intensity 5.0 m from the source? (c) What is the sound level 10 m from the source? 96 Four sound waves are to be sent through the same tube of air, in the same direction: s1(x, t) # (9.00 nm) cos(2px $ 700pt) s2(x, t) # (9.00 nm) cos(2px $ 700pt " 0.7p) s3(x, t) # (9.00 nm) cos(2px $ 700pt " p) s4(x, t) # (9.00 nm) cos(2px $ 700pt " 1.7p). What is the amplitude of the resultant wave? (Hint: Use a phasor diagram to simplify the problem.) 97 Straight line AB connects two point sources that are 5.00 m apart, emit 300 Hz sound waves of the same amplitude, and emit exactly out of phase. (a) What is the shortest distance between the midpoint of AB and a point on AB where the interfering waves cause maximum oscillation of the air molecules? What are the (b) second and (c) third shortest distances? 98 A point source that is stationary on an x axis emits a sinusoidal sound wave at a frequency of 686 Hz and speed 343 m/s. The wave travels radially outward from the source, causing air molecules to oscillate radially inward and outward. Let us define a wavefront as a line that connects points where the air molecules have the maximum, radially outward displacement. At any given instant, the wavefronts are concentric circles that are centered on the source. (a) Along x, what is the adjacent wavefront separation? Next, the source moves along x at a speed of 110 m/s. Along x, what are the wavefront separations (b) in front of and (c) behind the source? 99 You are standing at a distance D from an isotropic point source of sound. You walk 50.0 m toward the source and observe that the intensity of the sound has doubled. Calculate the distance D.

PROB LE M S

100 Pipe A has only one open end; pipe B is four times as long and has two open ends. Of the lowest 10 harmonic numbers nB of pipe B, what are the (a) smallest, (b) second smallest, and (c) third smallest values at which a harmonic frequency of B matches one of the harmonic frequencies of A? 101 A pipe 0.60 m long and closed at one end is filled with an unknown gas. The third lowest harmonic frequency for the pipe is 750 Hz. (a) What is the speed of sound in the unknown gas? (b) What is the fundamental frequency for this pipe when it is filled with the unknown gas? 102 A sound wave travels out uniformly in all directions from a point source. (a) Justify the following expression for the displacement s of the transmitting medium at any distance r from the source: s#

b sin k(r $ vt), r

where b is a constant. Consider the speed, direction of propagation, periodicity, and intensity of the wave. (b) What is the dimension of the constant b?

plunger P is provided at the other end of the tube, and the tube is filled with a gas. The rod is made to oscillate longitudinally at frequency f to produce sound waves inside the gas, and the location of the plunger is adjusted until a standing sound wave pattern is set up inside the tube. Once the standing wave is set up, the motion of the gas molecules causes the cork filings to collect in a pattern of ridges at the displacement nodes. If f # 4.46 ' 103 Hz and the separation between ridges is 9.20 cm, what is the speed of sound in the gas? 108 A source S and a detector D of radio waves are a distance d apart on level ground (Fig. 17-52). Radio waves of wavelength l reach D either along a straight path or by reflecting (bouncing) from a certain layer in the atmosphere. When the layer is at height H, the two waves reaching D are exactly in phase. If the layer gradually rises, the phase difference between the two waves gradually shifts, until they are exactly out of phase when the layer is at height H " h. Express l in terms of d, h, and H.

h

103 A police car is chasing a speeding Porsche 911. Assume that the Porsche’s maximum speed is 80.0 m/s and the police car’s is 54.0 m/s. At the moment both cars reach their maximum speed, what frequency will the Porsche driver hear if the frequency of the police car’s siren is 440 Hz? Take the speed of sound in air to be 340 m/s. 104 Suppose a spherical loudspeaker emits sound isotropically at 10 W into a room with completely absorbent walls, floor, and ceiling (an anechoic chamber). (a) What is the intensity of the sound at distance d # 3.0 m from the center of the source? (b) What is the ratio of the wave amplitude at d # 4.0 m to that at d # 3.0 m? 105 In Fig. 17-35, S1 and S2 are two isotropic point sources of sound. They emit waves in phase at wavelength 0.50 m; they are separated by D # 1.60 m. If we move a sound detector along a large circle centered at the midpoint between the sources, at how many points do waves arrive at the detector (a) exactly in phase and (b) exactly out of phase? 106 Figure 17-50 shows a transmitter and receiver of waves contained in a single instrument. It is used to measure the speed u of a target object (idealized as a flat plate) that is moving directly toward the unit, by analyzing the waves reflected from the target. What is u if the emitted frequency is 18.0 kHz and the detected frequency (of the returning waves) is 22.2 kHz?

513

H S

D

d/2

d/2

Figure 17-52 Problem 108. 109 In Fig. 17-53, a point source S of sound waves lies near a reflecting wall AB. A sound detector D intercepts sound ray R1 traveling directly from S. It also intercepts sound ray R2 that reflects from the wall such that the angle of incidence ui is equal to the angle of reflection ur. Assume that the reflection of sound by the wall causes a phase shift of 0.500l. If the distances are d1 # 2.50 m, d2 # 20.0 m, and d3 # 12.5 m, what are the (a) lowest and (b) second lowest frequency at which R1 and R2 are in phase at D? A

d1

d2

S ui ur

fr

u

R1 B

fs Target

Figure 17-50 Problem 106. 107 Kundt’s method for measuring the speed of sound. In Fig. 17-51, a rod R is clamped at its center; a disk D at its end projects into a glass tube that has cork filings spread over its interior. A

R

D

d

Figure 17-51 Problem 107.

P

d3

R2

D

Figure 17-53 Problem 109. 110 A person on a railroad car blows a trumpet note at 440 Hz. The car is moving toward a wall at 20.0 m/s. Find the sound frequency (a) at the wall and (b) reflected back to the trumpeter. 111 A listener at rest (with respect to the air and the ground) hears a signal of frequency f1 from a source moving toward him with a velocity of 15 m/s, due east. If the listener then moves toward the approaching source with a velocity of 25 m/s, due west, he hears a frequency f2 that differs from f1 by 37 Hz. What is the frequency of the source? (Take the speed of sound in air to be 340 m/s.)

C

H

A

P

T

E

R

1

8

Temperature, Heat, and the First Law of Thermodynamics 18-1 TEMPERATURE Learning Objectives After reading this module, you should be able to . . .

18.01 Identify the lowest temperature as 0 on the Kelvin scale (absolute zero). 18.02 Explain the zeroth law of thermodynamics. 18.03 Explain the conditions for the triple-point temperature.

Key Ideas ● Temperature is an SI base quantity related to our sense of hot and cold. It is measured with a thermometer, which contains a working substance with a measurable property, such as length or pressure, that changes in a regular way as the substance becomes hotter or colder. ● When a thermometer and some other object are placed in contact with each other, they eventually reach thermal equilibrium. The reading of the thermometer is then taken to be the temperature of the other object. The process provides consistent and useful temperature measurements because of the zeroth law of thermodynamics: If bodies A and B are each in thermal equilibrium with a third body C (the thermometer), then A and B are in thermal equilibrium with each other.

18.04 Explain the conditions for measuring a temperature with a constant-volume gas thermometer. 18.05 For a constant-volume gas thermometer, relate the pressure and temperature of the gas in some given state to the pressure and temperature at the triple point. ● In the SI system, temperature is measured on the Kelvin

scale, which is based on the triple point of water (273.16 K). Other temperatures are then defined by use of a constantvolume gas thermometer, in which a sample of gas is maintained at constant volume so its pressure is proportional to its temperature. We define the temperature T as measured with a gas thermometer to be T # (273.16 K)

# lim pp $. gas:0

3

Here T is in kelvins, and p3 and p are the pressures of the gas at 273.16 K and the measured temperature, respectively.

What Is Physics? One of the principal branches of physics and engineering is thermodynamics, which is the study and application of the thermal energy (often called the internal energy) of systems. One of the central concepts of thermodynamics is temperature. Since childhood, you have been developing a working knowledge of thermal energy and temperature. For example, you know to be cautious with hot foods and hot stoves and to store perishable foods in cool or cold compartments. You also know how to control the temperature inside home and car, and how to protect yourself from wind chill and heat stroke. Examples of how thermodynamics figures into everyday engineering and science are countless. Automobile engineers are concerned with the heating of a car engine, such as during a NASCAR race. Food engineers are concerned both with the proper heating of foods, such as pizzas being microwaved, and with the proper cooling of foods, such as TV dinners being quickly frozen at a processing plant. Geologists are concerned with the transfer of thermal energy in an El Niño event and in the gradual warming of ice expanses in the Arctic and Antarctic. 514

18-1 TE M PE RATU R E

Agricultural engineers are concerned with the weather conditions that determine whether the agriculture of a country thrives or vanishes. Medical engineers are concerned with how a patient’s temperature might distinguish between a benign viral infection and a cancerous growth. The starting point in our discussion of thermodynamics is the concept of temperature and how it is measured.

108

Temperature

106

The Zeroth Law of Thermodynamics The properties of many bodies change as we alter their temperature, perhaps by moving them from a refrigerator to a warm oven. To give a few examples: As their temperature increases, the volume of a liquid increases, a metal rod grows a little longer, and the electrical resistance of a wire increases, as does the pressure exerted by a confined gas. We can use any one of these properties as the basis of an instrument that will help us pin down the concept of temperature. Figure 18-2 shows such an instrument. Any resourceful engineer could design and construct it, using any one of the properties listed above. The instrument is fitted with a digital readout display and has the following properties: If you heat it (say, with a Bunsen burner), the displayed number starts to increase; if you then put it into a refrigerator, the displayed number starts to decrease. The instrument is not calibrated in any way, and the numbers have (as yet) no physical meaning. The device is a thermoscope but not (as yet) a thermometer. Suppose that, as in Fig. 18-3a, we put the thermoscope (which we shall call body T) into intimate contact with another body (body A). The entire system is confined within a thick-walled insulating box. The numbers displayed by the thermoscope roll by until, eventually, they come to rest (let us say the reading is “137.04”) and no further change takes place. In fact, we suppose that every measurable property of body T and of body A has assumed a stable, unchanging value. Then we say that the two bodies are in thermal equilibrium with each other. Even though the displayed readings for body T have not been calibrated, we conclude that bodies T and A must be at the same (unknown) temperature. Suppose that we next put body T into intimate contact with body B (Fig. 18-3b) and find that the two bodies come to thermal equilibrium at the same reading of the thermoscope. Then bodies T and B must be at the same (still unknown) temperature. If we now put bodies A and B into intimate contact (Fig. 18-3c), are they immediately in thermal equilibrium with each other? Experimentally, we find that they are. The experimental fact shown in Fig. 18-3 is summed up in the zeroth law of thermodynamics: If bodies A and B are each in thermal equilibrium with a third body T, then A and B are in thermal equilibrium with each other.

In less formal language, the message of the zeroth law is: “Every body has a property called temperature. When two bodies are in thermal equilibrium, their temperatures are equal. And vice versa.” We can now make our thermoscope

Temperature (K)

Temperature is one of the seven SI base quantities. Physicists measure temperature on the Kelvin scale, which is marked in units called kelvins. Although the temperature of a body apparently has no upper limit, it does have a lower limit; this limiting low temperature is taken as the zero of the Kelvin temperature scale. Room temperature is about 290 kelvins, or 290 K as we write it, above this absolute zero. Figure 18-1 shows a wide range of temperatures. When the universe began 13.7 billion years ago, its temperature was about 1039 K. As the universe expanded it cooled, and it has now reached an average temperature of about 3 K.We on Earth are a little warmer than that because we happen to live near a star.Without our Sun, we too would be at 3 K (or, rather, we could not exist).

1039

104 102 100

515

Universe just after beginning Highest laboratory temperature Center of the Sun

Surface of the Sun Tungsten melts Water freezes Universe today Boiling helium-3

10–2

10–9

Record low temperature

Figure 18-1 Some temperatures on the Kelvin scale. Temperature T # 0 corresponds to 10 $6 and cannot be plotted on this logarithmic scale.

Thermally sensitive element

Figure 18-2 A thermoscope. The numbers increase when the device is heated and decrease when it is cooled. The thermally sensitive element could be — among many possibilities — a coil of wire whose electrical resistance is measured and displayed.

516

CHAPTE R 18 TE M PE RATU R E, H EAT, AN D TH E FI RST L AW OF TH E R M ODYNAM ICS

Figure 18-3 (a) Body T (a thermoscope) and body A are in thermal equilibrium. (Body S is a thermally insulating screen.) (b) Body T and body B are also in thermal equilibrium, at the same reading of the thermoscope. (c) If (a) and (b) are true, the zeroth law of thermodynamics states that body A and body B are also in thermal equilibrium.

Gas thermometer bulb

T

S

A

B

B

A

(a)

T

T

S

(b)

A

B

(c)

(the third body T ) into a thermometer, confident that its readings will have physical meaning. All we have to do is calibrate it. We use the zeroth law constantly in the laboratory. If we want to know whether the liquids in two beakers are at the same temperature, we measure the temperature of each with a thermometer. We do not need to bring the two liquids into intimate contact and observe whether they are or are not in thermal equilibrium. The zeroth law, which has been called a logical afterthought, came to light only in the 1930s, long after the first and second laws of thermodynamics had been discovered and numbered. Because the concept of temperature is fundamental to those two laws, the law that establishes temperature as a valid concept should have the lowest number — hence the zero.

Vapor

Ice Water

Figure 18-4 A triple-point cell, in which solid ice, liquid water, and water vapor coexist in thermal equilibrium. By international agreement, the temperature of this mixture has been defined to be 273.16 K. The bulb of a constant-volume gas thermometer is shown inserted into the well of the cell.

Scale Gas-filled bulb

Measuring Temperature Here we first define and measure temperatures on the Kelvin scale. Then we calibrate a thermoscope so as to make it a thermometer.

The Triple Point of Water To set up a temperature scale, we pick some reproducible thermal phenomenon and, quite arbitrarily, assign a certain Kelvin temperature to its environment; that is, we select a standard fixed point and give it a standard fixed-point temperature. We could, for example, select the freezing point or the boiling point of water but, for technical reasons, we select instead the triple point of water. Liquid water, solid ice, and water vapor (gaseous water) can coexist, in thermal equilibrium, at only one set of values of pressure and temperature. Figure 18-4 shows a triple-point cell, in which this so-called triple point of water can be achieved in the laboratory. By international agreement, the triple point of water has been assigned a value of 273.16 K as the standard fixed-point temperature for the calibration of thermometers; that is, T3 # 273.16 K

0

h T R

(triple-point temperature),

(18-1)

in which the subscript 3 means “triple point.” This agreement also sets the size of the kelvin as 1/273.16 of the difference between the triple-point temperature of water and absolute zero. Note that we do not use a degree mark in reporting Kelvin temperatures. It is 300 K (not 300-K), and it is read “300 kelvins” (not “300 degrees Kelvin”). The usual SI prefixes apply. Thus, 0.0035 K is 3.5 mK. No distinction in nomenclature is made between Kelvin temperatures and temperature differences, so we can write, “the boiling point of sulfur is 717.8 K” and “the temperature of this water bath was raised by 8.5 K.”

The Constant-Volume Gas Thermometer Figure 18-5 A constant-volume gas thermometer, its bulb immersed in a liquid whose temperature T is to be measured.

The standard thermometer, against which all other thermometers are calibrated, is based on the pressure of a gas in a fixed volume. Figure 18-5 shows such a constant-volume gas thermometer; it consists of a gas-filled bulb connected by a tube to a mercury manometer. By raising and lowering reservoir R, the mercury

18-1 TE M PE RATU R E

373.50 Temperature (K)

Figure 18-6 Temperatures measured by a constant-volume gas thermometer, with its bulb immersed in boiling water. For temperature calculations using Eq. 18-5, pressure p3 was measured at the triple point of water.Three different gases in the thermometer bulb gave generally different results at different gas pressures, but as the amount of gas was decreased (decreasing p3), all three curves converged to 373.125 K.

373.40

N2

373.125 K

373.30 373.20

H2

373.10

He

0

20

40

60 p3 (kPa)

80

level in the left arm of the U-tube can always be brought to the zero of the scale to keep the gas volume constant (variations in the gas volume can affect temperature measurements). The temperature of any body in thermal contact with the bulb (such as the liquid surrounding the bulb in Fig. 18-5) is then defined to be (18-2)

T # Cp,

in which p is the pressure exerted by the gas and C is a constant. From Eq. 14-10, the pressure p is p # p0 $ rgh, (18-3) in which p0 is the atmospheric pressure, r is the density of the mercury in the manometer, and h is the measured difference between the mercury levels in the two arms of the tube.* (The minus sign is used in Eq. 18-3 because pressure p is measured above the level at which the pressure is p0.) If we next put the bulb in a triple-point cell (Fig. 18-4), the temperature now being measured is T3 # Cp3, (18-4) in which p3 is the gas pressure now. Eliminating C between Eqs. 18-2 and 18-4 gives us the temperature as T # T3

# pp $ # (273.16 K) # pp $ 3

(provisional).

(18-5)

3

We still have a problem with this thermometer. If we use it to measure, say, the boiling point of water, we find that different gases in the bulb give slightly different results. However, as we use smaller and smaller amounts of gas to fill the bulb, the readings converge nicely to a single temperature, no matter what gas we use. Figure 18-6 shows this convergence for three gases. Thus the recipe for measuring a temperature with a gas thermometer is T # (273.16 K)

# lim pp $. gas :0

(18-6)

3

The recipe instructs us to measure an unknown temperature T as follows: Fill the thermometer bulb with an arbitrary amount of any gas (for example, nitrogen) and measure p3 (using a triple-point cell) and p, the gas pressure at the temperature being measured. (Keep the gas volume the same.) Calculate the ratio p/p3. Then repeat both measurements with a smaller amount of gas in the bulb, and again calculate this ratio. Continue this way, using smaller and smaller amounts of gas, until you can extrapolate to the ratio p/p3 that you would find if there were approximately no gas in the bulb. Calculate the temperature T by substituting that extrapolated ratio into Eq. 18-6. (The temperature is called the ideal gas temperature.) *For pressure units, we shall use units introduced in Module 14-1. The SI unit for pressure is the newton per square meter, which is called the pascal (Pa). The pascal is related to other common pressure units by 1 atm # 1.01 ' 10 5 Pa # 760 torr # 14.7 lb/in.2.

100

120

517

518

CHAPTE R 18 TE M PE RATU R E, H EAT, AN D TH E FI RST L AW OF TH E R M ODYNAM ICS

18-2 THE CELSIUS AND FAHRENHEIT SCALES Learning Objectives After reading this module, you should be able to . . .

18.06 Convert a temperature between any two (linear) temperature scales, including the Celsius, Fahrenheit, and Kelvin scales.

18.07 Identify that a change of one degree is the same on the Celsius and Kelvin scales.

Key Idea ● The Celsius temperature scale is defined by

TC # T $ 273.15-,

with T in kelvins. The Fahrenheit temperature scale is defined by TF # 95TC " 32-.

The Celsius and Fahrenheit Scales So far, we have discussed only the Kelvin scale, used in basic scientific work. In nearly all countries of the world, the Celsius scale (formerly called the centigrade scale) is the scale of choice for popular and commercial use and much scientific use. Celsius temperatures are measured in degrees, and the Celsius degree has the same size as the kelvin. However, the zero of the Celsius scale is shifted to a more convenient value than absolute zero. If TC represents a Celsius temperature and T a Kelvin temperature, then TC # T $ 273.15-.

(18-7)

In expressing temperatures on the Celsius scale, the degree symbol is commonly used. Thus, we write 20.00-C for a Celsius reading but 293.15 K for a Kelvin reading. The Fahrenheit scale, used in the United States, employs a smaller degree than the Celsius scale and a different zero of temperature. You can easily verify both these differences by examining an ordinary room thermometer on which both scales are marked.The relation between the Celsius and Fahrenheit scales is T F # 95T C " 32-,

(18-8)

where TF is Fahrenheit temperature. Converting between these two scales can be done easily by remembering a few corresponding points, such as the freezing and boiling points of water (Table 18-1). Figure 18-7 compares the Kelvin, Celsius, and Fahrenheit scales. Triple point of water

Absolute zero

273.16 K

0K

0.01°C

–273.15°C

32.02°F

–459.67°F

Figure 18-7 The Kelvin, Celsius, and Fahrenheit temperature scales compared.

Table 18-1 Some Corresponding Temperatures Temperature

-C

-F

Boiling point of watera Normal body temperature Accepted comfort level Freezing point of watera Zero of Fahrenheit scale Scales coincide

100 37.0 20 0 % $18 $40

212 98.6 68 32 0 $40

a Strictly, the boiling point of water on the Celsius scale is 99.975-C, and the freezing point is 0.00-C.Thus, there is slightly less than 100 Cbetween those two points.

519

18-2 TH E CE LSI US AN D FAH R E N H E IT SCALES

We use the letters C and F to distinguish measurements and degrees on the two scales. Thus, 0-C # 32-F means that 0- on the Celsius scale measures the same temperature as 32- on the Fahrenheit scale, whereas 5 C- # 9 Fmeans that a temperature difference of 5 Celsius degrees (note the degree symbol appears after C) is equivalent to a temperature difference of 9 Fahrenheit degrees.

Checkpoint 1 The figure here shows three linear temperature scales with the freezing and boiling points of water indicated. (a) Rank the degrees on these scales by size, greatest first. (b) Rank the following temperatures, highest first: 50-X, 50-W, and 50-Y.

70°X

120°W

90°Y

–20°X

30°W

0°Y

Boiling point

Freezing point

Sample Problem 18.01 Conversion between two temperature scales Suppose you come across old scientific notes that describe a temperature scale called Z on which the boiling point of water is 65.0-Z and the freezing point is $14.0-Z. To what temperature on the Fahrenheit scale would a temperature of T # $98.0-Z correspond? Assume that the Z scale is linear; that is, the size of a Z degree is the same everywhere on the Z scale. KEY IDEA A conversion factor between two (linear) temperature scales can be calculated by using two known (benchmark) temperatures, such as the boiling and freezing points of water. The number of degrees between the known temperatures on one scale is equivalent to the number of degrees between them on the other scale. Calculations: We begin by relating the given temperature T to either known temperature on the Z scale. Since T # $98.0-Z is closer to the freezing point ($14.0-Z) than to the boiling point (65.0-Z), we use the freezing point. Then we note that the T we seek is below this point by $14.0-Z $ ($98.0-Z) # 84.0 Z- (Fig. 18-8). (Read this difference as “84.0 Z degrees.”) Next, we set up a conversion factor between the Z and Fahrenheit scales to convert this difference. To do so, we use both known temperatures on the Z scale and the

Z 65.0°Z

F Boil

79.0 Z° –14.0°Z

212°F 180 F°

Freeze

32°F

84.0 Z° T = –98.0°Z

T=?

Figure 18-8 An unknown temperature scale compared with the Fahrenheit temperature scale.

corresponding temperatures on the Fahrenheit scale. On the Z scale, the difference between the boiling and freezing points is 65.0-Z $ ($14.0-Z) # 79.0 Z-. On the Fahrenheit scale, it is 212-F $ 32.0-F # 180 F-. Thus, a temperature difference of 79.0 Z- is equivalent to a temperature difference of 180 F- (Fig. 18-8), and we can use the ratio (180 F-)/(79.0 Z-) as our conversion factor. Now, since T is below the freezing point by 84.0 Z-, it must also be below the freezing point by (84.0 Z-)

180 F# 191 F-. 79.0 Z-

Because the freezing point is at 32.0-F, this means that T # 32.0-F $ 191 F- # $159-F.

Additional examples, video, and practice available at WileyPLUS

(Answer)

520

CHAPTE R 18 TE M PE RATU R E, H EAT, AN D TH E FI RST L AW OF TH E R M ODYNAM ICS

18-3 THERMAL EXPANSION Learning Objectives After reading this module, you should be able to . . .

18.08 For one-dimensional thermal expansion, apply the relationship between the temperature change %T, the length change %L, the initial length L, and the coefficient of linear expansion a. 18.09 For two-dimensional thermal expansion, use one-

dimensional thermal expansion to find the change in area. 18.10 For three-dimensional thermal expansion, apply the relationship between the temperature change %T, the volume change %V, the initial volume V, and the coefficient of volume expansion b.

Key Ideas ● All objects change size with changes in temperature. For a temperature change %T, a change %L in any linear dimension L is given by %L # La %T,

in which a is the coefficient of linear expansion.

● The change %V in the volume V of a solid or liquid is

%V # Vb %T. Here b # 3a is the material’s coefficient of volume expansion.

Thermal Expansion

Hugh Thomas/BWP Media/Getty Images, Inc.

Figure 18-9 When a Concorde flew faster than the speed of sound, thermal expansion due to the rubbing by passing air increased the aircraft’s length by about 12.5 cm. (The temperature increased to about 128-C at the aircraft nose and about 90-C at the tail, and cabin windows were noticeably warm to the touch.)

You can often loosen a tight metal jar lid by holding it under a stream of hot water. Both the metal of the lid and the glass of the jar expand as the hot water adds energy to their atoms. (With the added energy, the atoms can move a bit farther from one another than usual, against the spring-like interatomic forces that hold every solid together.) However, because the atoms in the metal move farther apart than those in the glass, the lid expands more than the jar and thus is loosened. Such thermal expansion of materials with an increase in temperature must be anticipated in many common situations. When a bridge is subject to large seasonal changes in temperature, for example, sections of the bridge are separated by expansion slots so that the sections have room to expand on hot days without the bridge buckling. When a dental cavity is filled, the filling material must have the same thermal expansion properties as the surrounding tooth; otherwise, consuming cold ice cream and then hot coffee would be very painful. When the Concorde aircraft (Fig. 18-9) was built, the design had to allow for the thermal expansion of the fuselage during supersonic flight because of frictional heating by the passing air. The thermal expansion properties of some materials can be put to common use. Thermometers and thermostats may be based on the differences in expansion between the components of a bimetal strip (Fig. 18-10). Also, the familiar liquid-inglass thermometers are based on the fact that liquids such as mercury and alcohol expand to a different (greater) extent than their glass containers.

Linear Expansion Figure 18-10 (a) A bimetal strip, consisting of a strip of brass and a strip of steel welded together, at temperature T0. (b) The strip bends as shown at temperatures above this reference temperature. Below the reference temperature the strip bends the other way. Many thermostats operate on this principle, making and breaking an electrical contact as the temperature rises and falls.

If the temperature of a metal rod of length L is raised by an amount %T, its length is found to increase by an amount (18-9)

%L # La %T,

Brass Steel T = T0 (a)

Different amounts of expansion or contraction can produce bending.

T > T0 (b)

18-3 TH E R MAL EXPANSION

Table 18-2 Some Coefficients of

in which a is a constant called the coefficient of linear expansion. The coefficient a has the unit “per degree” or “per kelvin” and depends on the material. Although a varies somewhat with temperature, for most practical purposes it can be taken as constant for a particular material. Table 18-2 shows some coefficients of linear expansion. Note that the unit C- there could be replaced with the unit K. The thermal expansion of a solid is like photographic enlargement except it is in three dimensions. Figure 18-11b shows the (exaggerated) thermal expansion of a steel ruler. Equation 18-9 applies to every linear dimension of the ruler, including its edge, thickness, diagonals, and the diameters of the circle etched on it and the circular hole cut in it. If the disk cut from that hole originally fits snugly in the hole, it will continue to fit snugly if it undergoes the same temperature increase as the ruler.

Linear Expansiona Substance

If all dimensions of a solid expand with temperature, the volume of that solid must also expand. For liquids, volume expansion is the only meaningful expansion parameter. If the temperature of a solid or liquid whose volume is V is increased by an amount %T, the increase in volume is found to be

51 29 23 19 17 12 11 9 3.2 1.2 0.7 0.5

a

Room temperature values except for the listing for ice.

b

This alloy was designed to have a low coefficient of expansion. The word is a shortened form of “invariable.”

(18-10)

where b is the coefficient of volume expansion of the solid or liquid. The coefficients of volume expansion and linear expansion for a solid are related by b # 3a.

a (10 $6/C-)

Ice (at 0-C) Lead Aluminum Brass Copper Concrete Steel Glass (ordinary) Glass (Pyrex) Diamond Invarb Fused quartz

Volume Expansion

%V # Vb %T,

(18-11)

The most common liquid, water, does not behave like other liquids. Above about 4-C, water expands as the temperature rises, as we would expect. Between 0 and about 4-C, however, water contracts with increasing temperature. Thus, at about 4-C, the density of water passes through a maximum. At all other temperatures, the density of water is less than this maximum value. This behavior of water is the reason lakes freeze from the top down rather than from the bottom up. As water on the surface is cooled from, say, 10-C toward the freezing point, it becomes denser (“heavier”) than lower water and sinks to the bottom. Below 4-C, however, further cooling makes the water then on the surface less dense (“lighter”) than the lower water, so it stays on the surface until it freezes. Thus the surface freezes while the lower water is still liquid. If lakes froze from the bottom up, the ice so formed would tend not to melt completely during the summer, because it would be insulated by the water above. After a few years, many bodies of open water in the temperate zones of Earth would be frozen solid all year round—and aquatic life could not exist. Figure 18-11 The same steel ruler at two different temperatures. When it expands, the scale, the numbers, the thickness, and the diameters of the circle and circular hole are all increased by the same factor. (The expansion has been exaggerated for clarity.)

1

2

3

4

(a )

1

2

3

5

6

7

Circle Circular hole 4

5

6

7

(b )

Checkpoint 2 The figure here shows four rectangular metal plates, with sides of L, 2L, or 3L.They are all made of the same material, and their temperature is to be increased by the same amount. Rank the plates according to the expected increase in (a) their vertical heights and (b) their areas, greatest first.

(1)

521

(2)

(3)

(4)

522

CHAPTE R 18 TE M PE RATU R E, H EAT, AN D TH E FI RST L AW OF TH E R M ODYNAM ICS

Sample Problem 18.02 Thermal expansion of a volume On a hot day in Las Vegas, an oil trucker loaded 37 000 L of diesel fuel. He encountered cold weather on the way to Payson, Utah, where the temperature was 23.0 K lower than in Las Vegas, and where he delivered his entire load. How many liters did he deliver? The coefficient of volume expansion for diesel fuel is 9.50 ' 10 $4/C-, and the coefficient of linear expansion for his steel truck tank is 11 ' 10 $6/C-. KEY IDEA The volume of the diesel fuel depends directly on the temperature. Thus, because the temperature decreased, the

volume of the fuel did also, as given by Eq. 18-10 (%V # Vb %T). Calculations: We find %V# (37 000 L)(9.50 ' 10 $4/C-)($23.0 K) # $808 L. Thus, the amount delivered was Vdel # V " %V # 37 000 L $ 808 L # 36 190 L. (Answer) Note that the thermal expansion of the steel tank has nothing to do with the problem. Question: Who paid for the “missing” diesel fuel?

Additional examples, video, and practice available at WileyPLUS

18-4 ABSORPTION OF HEAT Learning Objectives After reading this module, you should be able to . . .

18.11 Identify that thermal energy is associated with the random motions of the microscopic bodies in an object. 18.12 Identify that heat Q is the amount of transferred energy (either to or from an object’s thermal energy) due to a temperature difference between the object and its environment. 18.13 Convert energy units between various measurement systems. 18.14 Convert between mechanical or electrical energy and thermal energy. 18.15 For a temperature change %T of a substance, relate the change to the heat transfer Q and the substance’s heat capacity C. 18.16 For a temperature change %T of a substance, relate the

change to the heat transfer Q and the substance’s specific heat c and mass m. 18.17 Identify the three phases of matter. 18.18 For a phase change of a substance, relate the heat transfer Q, the heat of transformation L, and the amount of mass m transformed. 18.19 Identify that if a heat transfer Q takes a substance across a phase-change temperature, the transfer must be calculated in steps: (a) a temperature change to reach the phase-change temperature, (b) the phase change, and then (c) any temperature change that moves the substance away from the phase-change temperature.

Key Ideas ● Heat Q is energy that is transferred between a system and its environment because of a temperature difference between them. It can be measured in joules (J), calories (cal), kilocalories (Cal or kcal), or British thermal units (Btu), with

1 cal # 3.968 ' 10$3 Btu # 4.1868 J. ● If heat Q is absorbed by an object, the object’s temperature change Tf $ Ti is related to Q by

Q # C(Tf $ Ti), in which C is the heat capacity of the object. If the object has mass m, then Q # cm(Tf $ Ti), where c is the specific heat of the material making up the object.

● The molar specific heat of a material is the heat capacity per mole, which means per 6.02 ' 1023 elementary units of the material. ● Heat absorbed by a material may change the material’s physical state —for example, from solid to liquid or from liquid to gas. The amount of energy required per unit mass to change the state (but not the temperature) of a particular material is its heat of transformation L. Thus,

Q # Lm. ● The heat of vaporization LV is the amount of energy per unit mass that must be added to vaporize a liquid or that must be removed to condense a gas. ● The heat of fusion LF is the amount of energy per unit mass that must be added to melt a solid or that must be removed to freeze a liquid.

18-4 ABSOR PTION OF H EAT

Temperature and Heat If you take a can of cola from the refrigerator and leave it on the kitchen table, its temperature will rise—rapidly at first but then more slowly—until the temperature of the cola equals that of the room (the two are then in thermal equilibrium). In the same way, the temperature of a cup of hot coffee, left sitting on the table, will fall until it also reaches room temperature. In generalizing this situation, we describe the cola or the coffee as a system (with temperature TS) and the relevant parts of the kitchen as the environment (with temperature TE) of that system. Our observation is that if TS is not equal to TE, then TS will change (TE can also change some) until the two temperatures are equal and thus thermal equilibrium is reached. Such a change in temperature is due to a change in the thermal energy of the system because of a transfer of energy between the system and the system’s environment. (Recall that thermal energy is an internal energy that consists of the kinetic and potential energies associated with the random motions of the atoms, molecules, and other microscopic bodies within an object.) The transferred energy is called heat and is symbolized Q. Heat is positive when energy is transferred to a system’s thermal energy from its environment (we say that heat is absorbed by the system). Heat is negative when energy is transferred from a system’s thermal energy to its environment (we say that heat is released or lost by the system). This transfer of energy is shown in Fig. 18-12. In the situation of Fig. 18-12a, in which TS 4 TE, energy is transferred from the system to the environment, so Q is negative. In Fig. 18-12b, in which TS # TE, there is no such transfer, Q is zero, and heat is neither released nor absorbed. In Fig. 18-12c, in which TS 8 TE, the transfer is to the system from the environment; so Q is positive.

Environment

The system has a higher temperature, so ... (a )

TE

Q TS > TE

Q0

Figure 18-12 If the temperature of a system exceeds that of its environment as in (a), heat Q is lost by the system to the environment until thermal equilibrium (b) is established. (c) If the temperature of the system is below that of the environment, heat is absorbed by the system until thermal equilibrium is established.

523

524

CHAPTE R 18 TE M PE RATU R E, H EAT, AN D TH E FI RST L AW OF TH E R M ODYNAM ICS

We are led then to this definition of heat: Heat is the energy transferred between a system and its environment because of a temperature difference that exists between them.

Language. Recall that energy can also be transferred between a system and its environment as work W via a force acting on a system. Heat and work, unlike temperature, pressure, and volume, are not intrinsic properties of a system. They have meaning only as they describe the transfer of energy into or out of a system. Similarly, the phrase “a $600 transfer” has meaning if it describes the transfer to or from an account, not what is in the account, because the account holds money, not a transfer. Units. Before scientists realized that heat is transferred energy, heat was measured in terms of its ability to raise the temperature of water. Thus, the calorie (cal) was defined as the amount of heat that would raise the temperature of 1 g of water from 14.5-C to 15.5-C. In the British system, the corresponding unit of heat was the British thermal unit (Btu), defined as the amount of heat that would raise the temperature of 1 lb of water from 63-F to 64-F. In 1948, the scientific community decided that since heat (like work) is transferred energy, the SI unit for heat should be the one we use for energy — namely, the joule. The calorie is now defined to be 4.1868 J (exactly), with no reference to the heating of water. (The “calorie” used in nutrition, sometimes called the Calorie (Cal), is really a kilocalorie.) The relations among the various heat units are 1 cal # 3.968 ' 10 $3 Btu # 4.1868 J.

(18-12)

The Absorption of Heat by Solids and Liquids Heat Capacity The heat capacity C of an object is the proportionality constant between the heat Q that the object absorbs or loses and the resulting temperature change %T of the object; that is, Q # C %T # C(Tf $ Ti),

(18-13)

in which Ti and Tf are the initial and final temperatures of the object. Heat capacity C has the unit of energy per degree or energy per kelvin. The heat capacity C of, say, a marble slab used in a bun warmer might be 179 cal/C-, which we can also write as 179 cal/K or as 749 J/K. The word “capacity” in this context is really misleading in that it suggests analogy with the capacity of a bucket to hold water. That analogy is false, and you should not think of the object as “containing” heat or being limited in its ability to absorb heat. Heat transfer can proceed without limit as long as the necessary temperature difference is maintained.The object may, of course, melt or vaporize during the process.

Specific Heat Two objects made of the same material — say, marble — will have heat capacities proportional to their masses. It is therefore convenient to define a “heat capacity per unit mass” or specific heat c that refers not to an object but to a unit mass of the material of which the object is made. Equation 18-13 then becomes Q # cm %T # cm(Tf $ Ti).

(18-14)

Through experiment we would find that although the heat capacity of a particular marble slab might be 179 cal/C- (or 749 J/K), the specific heat of marble itself (in that slab or in any other marble object) is 0.21 cal/g 9C- (or 880 J/kg 9K).

525

18-4 ABSOR PTION OF H EAT

From the way the calorie and the British thermal unit were initially defined, the specific heat of water is c # 1 cal/g 9C- # 1 Btu/lb 9F- # 4186.8 J/kg 9K.

(18-15)

Table 18-3 shows the specific heats of some substances at room temperature. Note that the value for water is relatively high. The specific heat of any substance actually depends somewhat on temperature, but the values in Table 18-3 apply reasonably well in a range of temperatures near room temperature.

Checkpoint 3 A certain amount of heat Q will warm 1 g of material A by 3 C- and 1 g of material B by 4 C-. Which material has the greater specific heat?

Molar Specific Heat In many instances the most convenient unit for specifying the amount of a substance is the mole (mol), where 1 mol # 6.02 ' 10 23 elementary units of any substance. Thus 1 mol of aluminum means 6.02 ' 10 23 atoms (the atom is the elementary unit), and 1 mol of aluminum oxide means 6.02 ' 10 23 molecules (the molecule is the elementary unit of the compound). When quantities are expressed in moles, specific heats must also involve moles (rather than a mass unit); they are then called molar specific heats. Table 18-3 shows the values for some elemental solids (each consisting of a single element) at room temperature.

An Important Point In determining and then using the specific heat of any substance, we need to know the conditions under which energy is transferred as heat. For solids and liquids, we usually assume that the sample is under constant pressure (usually atmospheric) during the transfer. It is also conceivable that the sample is held at constant volume while the heat is absorbed. This means that thermal expansion of the sample is prevented by applying external pressure. For solids and liquids, this is very hard to arrange experimentally, but the effect can be calculated, and it turns out that the specific heats under constant pressure and constant volume for any solid or liquid differ usually by no more than a few percent. Gases, as you will see, have quite different values for their specific heats under constant-pressure conditions and under constant-volume conditions.

Heats of Transformation When energy is absorbed as heat by a solid or liquid, the temperature of the sample does not necessarily rise. Instead, the sample may change from one phase, or state, to another. Matter can exist in three common states: In the solid state, the molecules of a sample are locked into a fairly rigid structure by their mutual attraction. In the liquid state, the molecules have more energy and move about more. They may form brief clusters, but the sample does not have a rigid structure and can flow or settle into a container. In the gas, or vapor, state, the molecules have even more energy, are free of one another, and can fill up the full volume of a container. Melting. To melt a solid means to change it from the solid state to the liquid state. The process requires energy because the molecules of the solid must be freed from their rigid structure. Melting an ice cube to form liquid water is a common example. To freeze a liquid to form a solid is the reverse of melting and requires that energy be removed from the liquid, so that the molecules can settle into a rigid structure.

Table 18-3 Some Specific Heats

and Molar Specific Heats at Room Temperature

Specific Heat Substance Elemental Solids Lead Tungsten Silver Copper Aluminum Other Solids Brass Granite Glass Ice ($10-C) Liquids Mercury Ethyl alcohol Seawater Water

Molar Specific Heat

cal

J

J

g 9K

kg 9K

mol 9K

0.0305 0.0321 0.0564 0.0923 0.215

128 134 236 386 900

26.5 24.8 25.5 24.5 24.4

0.092 0.19 0.20 0.530

380 790 840 2220

0.033

140

0.58 0.93 1.00

2430 3900 4187

526

CHAPTE R 18 TE M PE RATU R E, H EAT, AN D TH E FI RST L AW OF TH E R M ODYNAM ICS

Table 18-4 Some Heats of Transformation Melting Substance Hydrogen Oxygen Mercury Water Lead Silver Copper

Melting Point (K) 14.0 54.8 234 273 601 1235 1356

Boiling

Heat of Fusion LF (kJ/kg)

Boiling Point (K)

58.0 13.9 11.4 333 23.2 105 207

20.3 90.2 630 373 2017 2323 2868

Heat of Vaporization LV (kJ/kg) 455 213 296 2256 858 2336 4730

Vaporizing. To vaporize a liquid means to change it from the liquid state to the vapor (gas) state. This process, like melting, requires energy because the molecules must be freed from their clusters. Boiling liquid water to transfer it to water vapor (or steam — a gas of individual water molecules) is a common example. Condensing a gas to form a liquid is the reverse of vaporizing; it requires that energy be removed from the gas, so that the molecules can cluster instead of flying away from one another. The amount of energy per unit mass that must be transferred as heat when a sample completely undergoes a phase change is called the heat of transformation L. Thus, when a sample of mass m completely undergoes a phase change, the total energy transferred is Q # Lm.

(18-16)

When the phase change is from liquid to gas (then the sample must absorb heat) or from gas to liquid (then the sample must release heat), the heat of transformation is called the heat of vaporization LV. For water at its normal boiling or condensation temperature, LV # 539 cal/g # 40.7 kJ/mol # 2256 kJ/kg.

(18-17)

When the phase change is from solid to liquid (then the sample must absorb heat) or from liquid to solid (then the sample must release heat), the heat of transformation is called the heat of fusion LF. For water at its normal freezing or melting temperature, LF # 79.5 cal/g # 6.01 kJ/mol # 333 kJ/kg. (18-18) Table 18-4 shows the heats of transformation for some substances. Sample Problem 18.03 Hot slug in water, coming to equilibrium A copper slug whose mass mc is 75 g is heated in a laboratory oven to a temperature T of 312°C. The slug is then dropped into a glass beaker containing a mass mw # 220 g of water. The heat capacity Cb of the beaker is 45 cal/K. The initial temperature Ti of the water and the beaker is 12°C. Assuming that the slug, beaker, and water are an isolated system and the water does not vaporize, find the final temperature Tf of the system at thermal equilibrium.

can occur. (2) Because nothing in the system undergoes a phase change, the thermal energy transfers can only change the temperatures.

KEY IDEAS

Because the total energy of the system cannot change, the sum of these three energy transfers is zero:

(1) Because the system is isolated, the system’s total energy cannot change and only internal transfers of thermal energy

Calculations: To relate the transfers to the temperature changes, we can use Eqs. 18-13 and 18-14 to write for the water: Qw # cwmw(Tf $ Ti); for the beaker: Qb # Cb(Tf $ Ti); for the copper: Qc # ccmc(Tf $ T ).

Qw " Qb " Qc # 0.

(18-19) (18-20) (18-21)

(18-22)

527

18-4 ABSOR PTION OF H EAT

Substituting Eqs. 18-19 through 18-21 into Eq. 18-22 yields

and the denominator to be

cwmw(Tf $ Ti) " Cb(Tf $ Ti) " ccmc(Tf $ T) # 0.

(1.00 cal/g 9K)(220 g) " 45 cal/K

(18-23)

Temperatures are contained in Eq. 18-23 only as differences. Thus, because the differences on the Celsius and Kelvin scales are identical, we can use either of those scales in this equation. Solving it for Tf , we obtain cc mcT " CbTi " cw mwTi . Tf # cw mw " Cb " cc mc Using Celsius temperatures and taking values for cc and cw from Table 18-3, we find the numerator to be (0.0923 cal/g 9K)(75 g)(312-C) " (45 cal/K)(12-C) " (1.00 cal/g 9K)(220 g)(12-C) # 5339.8 cal,

" (0.0923 cal/g 9K)(75 g) # 271.9 cal/C-. We then have 5339.8 cal # 19.6-C % 20-C. 271.9 cal/C-

Tf #

(Answer)

From the given data you can show that Qw % 1670 cal,

Qb % 342 cal,

Qc % $2020 cal.

Apart from rounding errors, the algebraic sum of these three heat transfers is indeed zero, as required by the conservation of energy (Eq. 18-22).

Sample Problem 18.04 Heat to change temperature and state (a) How much heat must be absorbed by ice of mass m # 720 g at $10-C to take it to the liquid state at 15-C? KEY IDEAS The heating process is accomplished in three steps: (1) The ice cannot melt at a temperature below the freezing point — so initially, any energy transferred to the ice as heat can only increase the temperature of the ice, until 0-C is reached. (2) The temperature then cannot increase until all the ice melts — so any energy transferred to the ice as heat now can only change ice to liquid water, until all the ice melts. (3) Now the energy transferred to the liquid water as heat can only increase the temperature of the liquid water. Warming the ice: The heat Q1 needed to take the ice from the initial Ti # $10-C to the final Tf # 0-C (so that the ice can then melt) is given by Eq. 18-14 (Q # cm %T ). Using the specific heat of ice cice in Table 18-3 gives us Q1 # cicem(Tf $ Ti) # (2220 J/kg 9K)(0.720 kg)[0-C $ ($10-C)] # 15 984 J % 15.98 kJ. Melting the ice: The heat Q2 needed to melt all the ice is given by Eq. 18-16 (Q # Lm). Here L is the heat of fusion LF, with the value given in Eq. 18-18 and Table 18-4. We find Q2 # LF m # (333 kJ/kg)(0.720 kg) % 239.8 kJ. Warming the liquid: The heat Q3 needed to increase the temperature of the water from the initial value Ti # 0-C to the final value Tf # 15-C is given by Eq. 18-14 (with the specific heat of liquid water cliq):

Q3 # cliqm(Tf $ Ti) # (4186.8 J/kg 9K)(0.720 kg)(15-C $ 0°C) # 45 217 J % 45.22 kJ. Total: The total required heat Qtot is the sum of the amounts required in the three steps: Qtot # Q1 " Q2 " Q3 # 15.98 kJ " 239.8 kJ " 45.22 kJ % 300 kJ. (Answer) Note that most of the energy goes into melting the ice rather than raising the temperature. (b) If we supply the ice with a total energy of only 210 kJ (as heat), what are the final state and temperature of the water? KEY IDEA From step 1, we know that 15.98 kJ is needed to raise the temperature of the ice to the melting point. The remaining heat Qrem is then 210 kJ $ 15.98 kJ, or about 194 kJ. From step 2, we can see that this amount of heat is insufficient to melt all the ice. Because the melting of the ice is incomplete, we must end up with a mixture of ice and liquid; the temperature of the mixture must be the freezing point, 0-C. Calculations: We can find the mass m of ice that is melted by the available energy Qrem by using Eq. 18-16 with LF : m#

Qrem 194 kJ # # 0.583 kg % 580 g. LF 333 kJ/kg

Thus, the mass of the ice that remains is 720 g $ 580 g, or 140 g, and we have 580 g water and 140 g ice, at 0-C.

Additional examples, video, and practice available at WileyPLUS

(Answer)

528

CHAPTE R 18 TE M PE RATU R E, H EAT, AN D TH E FI RST L AW OF TH E R M ODYNAM ICS

18-5 THE FIRST LAW OF THERMODYNAMICS Learning Objectives After reading this module, you should be able to . . .

18.20 If an enclosed gas expands or contracts, calculate the work W done by the gas by integrating the gas pressure with respect to the volume of the enclosure. 18.21 Identify the algebraic sign of work W associated with expansion and contraction of a gas. 18.22 Given a p-V graph of pressure versus volume for a process, identify the starting point (the initial state) and the final point (the final state) and calculate the work by using graphical integration. 18.23 On a p-V graph of pressure versus volume for a gas, identify the algebraic sign of the work associated with a right-going process and a left-going process. 18.24 Apply the first law of thermodynamics to relate the change in the internal energy %Eint of a gas, the energy Q transferred as heat to or from the gas, and the work W done on or by the gas.

18.25 Identify the algebraic sign of a heat transfer Q that is associated with a transfer to a gas and a transfer from the gas. 18.26 Identify that the internal energy %Eint of a gas tends to increase if the heat transfer is to the gas, and it tends to decrease if the gas does work on its environment. 18.27 Identify that in an adiabatic process with a gas, there is no heat transfer Q with the environment. 18.28 Identify that in a constant-volume process with a gas, there is no work W done by the gas. 18.29 Identify that in a cyclical process with a gas, there is no net change in the internal energy %Eint. 18.30 Identify that in a free expansion with a gas, the heat transfer Q, work done W, and change in internal energy %Eint are each zero.

Key Ideas ● A gas may exchange energy with its surroundings through work. The amount of work W done by a gas as it expands or contracts from an initial volume Vi to a final volume Vf is given by

W#

"

dW #

"

Vf

Vi

p dV.

The integration is necessary because the pressure p may vary during the volume change. ● The principle of conservation of energy for a thermody-

namic process is expressed in the first law of thermodynamics, which may assume either of the forms %Eint # Eint,f $ Eint,i # Q $ W or

dEint # dQ $ dW

(first law)

(first law).

Eint represents the internal energy of the material, which depends only on the material’s state (temperature,

pressure, and volume). Q represents the energy exchanged as heat between the system and its surroundings; Q is positive if the system absorbs heat and negative if the system loses heat. W is the work done by the system; W is positive if the system expands against an external force from the surroundings and negative if the system contracts because of an external force. ● Q and W are path dependent; %Eint

is path independent.

● The first law of thermodynamics finds application in several special cases:

adiabatic processes:

Q # 0,

%Eint # $W

constant-volume processes:

W # 0,

%Eint # Q

cyclical processes: free expansions:

%Eint # 0,

Q#W

Q # W # %Eint # 0

A Closer Look at Heat and Work Here we look in some detail at how energy can be transferred as heat and work between a system and its environment. Let us take as our system a gas confined to a cylinder with a movable piston, as in Fig. 18-13. The upward force on the piston due to the pressure of the confined gas is equal to the weight of lead shot loaded onto the top of the piston. The walls of the cylinder are made of insulating material that does not allow any transfer of energy as heat. The bottom of the cylinder rests on a reservoir for thermal energy, a thermal reservoir (perhaps a hot plate) whose temperature T you can control by turning a knob. The system (the gas) starts from an initial state i, described by a pressure pi, a volume Vi, and a temperature Ti. You want to change the system to a final state f, described by a pressure pf, a volume Vf , and a temperature Tf . The procedure by which you change the system from its initial state to its final state is called a thermodynamic process. During such a process, energy may be trans-

18-5 TH E FI RST L AW OF TH E R M ODYNAM ICS

ferred into the system from the thermal reservoir (positive heat) or vice versa (negative heat). Also, work can be done by the system to raise the loaded piston (positive work) or lower it (negative work). We assume that all such changes occur slowly, with the result that the system is always in (approximate) thermal equilibrium (every part is always in thermal equilibrium). Suppose that you remove a few lead shot from the piston of Fig. 18-13, allowing the gas to push the piston and remaining shot upward through a differential dis: placement d : s with an upward force F . Since the displacement is tiny, we can as: : sume that F is constant during the displacement. Then F has a magnitude that is equal to pA, where p is the pressure of the gas and A is the face area of the piston. The differential work dW done by the gas during the displacement is

Lead shot

W

Q

(18-24)

in which dV is the differential change in the volume of the gas due to the movement of the piston. When you have removed enough shot to allow the gas to change its volume from Vi to Vf , the total work done by the gas is

"

dW #

"

Vi

(18-25)

p dV.

During the volume change, the pressure and temperature may also change. To evaluate Eq. 18-25 directly, we would need to know how pressure varies with volume for the actual process by which the system changes from state i to state f. One Path. There are actually many ways to take the gas from state i to state f. One way is shown in Fig. 18-14a, which is a plot of the pressure of the gas versus its volume and which is called a p-V diagram. In Fig. 18-14a, the curve indicates that the

Process

W >0

(b ) 0

It still goes from i to f, but now it does less work. i

f

W >0 (c ) 0

Volume

f

Volume

Cycling clockwise yields a positive net work.

Moving from f to i, it does negative work.

Pressure

h

i

f d

c (d ) 0

a W >0

We can control how much work it does. g

i

f

Volume

Figure 18-13 A gas is confined to a cylinder with a movable piston. Heat Q can be added to or withdrawn from the gas by regulating the temperature T of the adjustable thermal reservoir. Work W can be done by the gas by raising or lowering the piston.

Volume

Pressure

(a ) 0

Pressure

Pressure

i

Control knob

We control the heat transfer by adjusting the temperature.

It still goes from i to f, but now it does more work.

Gas moves from i to f, doing positive work.

Pressure

Figure 18-14 (a) The shaded area represents the work W done by a system as it goes from an initial state i to a final state f. Work W is positive because the system’s volume increases. (b) W is still positive, but now greater. (c) W is still positive, but now smaller. (d) W can be even smaller (path icdf ) or larger (path ighf). (e) Here the system goes from state f to state i as the gas is compressed to less volume by an external force. The work W done by the system is now negative. (f ) The net work Wnet done by the system during a complete cycle is represented by the shaded area.

T Thermal reservoir

Pressure

W#

Vf

The gas does work on this piston.

Insulation

:

dW # F "d : s # (pA)(ds) # p(A ds) # p dV,

529

i

W 0

f

f (f ) 0

Volume

A

530

CHAPTE R 18 TE M PE RATU R E, H EAT, AN D TH E FI RST L AW OF TH E R M ODYNAM ICS

pressure decreases as the volume increases. The integral in Eq. 18-25 (and thus the work W done by the gas) is represented by the shaded area under the curve between points i and f. Regardless of what exactly we do to take the gas along the curve, that work is positive, due to the fact that the gas increases its volume by forcing the piston upward. Another Path. Another way to get from state i to state f is shown in Fig. 18-14b. There the change takes place in two steps — the first from state i to state a, and the second from state a to state f. Step ia of this process is carried out at constant pressure, which means that you leave undisturbed the lead shot that ride on top of the piston in Fig. 18-13. You cause the volume to increase (from Vi to Vf) by slowly turning up the temperature control knob, raising the temperature of the gas to some higher value Ta. (Increasing the temperature increases the force from the gas on the piston, moving it upward.) During this step, positive work is done by the expanding gas (to lift the loaded piston) and heat is absorbed by the system from the thermal reservoir (in response to the arbitrarily small temperature differences that you create as you turn up the temperature). This heat is positive because it is added to the system. Step af of the process of Fig. 18-14b is carried out at constant volume, so you must wedge the piston, preventing it from moving. Then as you use the control knob to decrease the temperature, you find that the pressure drops from pa to its final value pf . During this step, heat is lost by the system to the thermal reservoir. For the overall process iaf, the work W, which is positive and is carried out only during step ia, is represented by the shaded area under the curve. Energy is transferred as heat during both steps ia and af, with a net energy transfer Q. Reversed Steps. Figure 18-14c shows a process in which the previous two steps are carried out in reverse order. The work W in this case is smaller than for Fig. 18-14b, as is the net heat absorbed. Figure 18-14d suggests that you can make the work done by the gas as small as you want (by following a path like icdf ) or as large as you want (by following a path like ighf ). To sum up: A system can be taken from a given initial state to a given final state by an infinite number of processes. Heat may or may not be involved, and in general, the work W and the heat Q will have different values for different processes. We say that heat and work are path-dependent quantities. Negative Work. Figure 18-14e shows an example in which negative work is done by a system as some external force compresses the system, reducing its volume. The absolute value of the work done is still equal to the area beneath the curve, but because the gas is compressed, the work done by the gas is negative. Cycle. Figure 18-14f shows a thermodynamic cycle in which the system is taken from some initial state i to some other state f and then back to i. The net work done by the system during the cycle is the sum of the positive work done during the expansion and the negative work done during the compression. In Fig. 18-14f , the net work is positive because the area under the expansion curve (i to f ) is greater than the area under the compression curve (f to i).

Checkpoint 4 The p-V diagram here shows six curved paths (connected by vertical paths) that can be followed by a gas. Which two of the curved paths should be part of a closed cycle (those curved paths plus connecting vertical paths) if the net work done by the gas during the cycle is to be at its maximum positive value?

p b d f

a c e V

18-5 TH E FI RST L AW OF TH E R M ODYNAM ICS

The First Law of Thermodynamics You have just seen that when a system changes from a given initial state to a given final state, both the work W and the heat Q depend on the nature of the process. Experimentally, however, we find a surprising thing. The quantity Q 2 W is the same for all processes. It depends only on the initial and final states and does not depend at all on how the system gets from one to the other. All other combinations of Q and W, including Q alone, W alone, Q " W, and Q $ 2W, are path dependent; only the quantity Q $ W is not. The quantity Q $ W must represent a change in some intrinsic property of the system. We call this property the internal energy Eint and we write %Eint # Eint,f $ Eint,i # Q $ W

(18-26)

(first law).

Equation 18-26 is the first law of thermodynamics. If the thermodynamic system undergoes only a differential change, we can write the first law as* dEint # dQ $ dW

(18-27)

(first law).

The internal energy Eint of a system tends to increase if energy is added as heat Q and tends to decrease if energy is lost as work W done by the system.

In Chapter 8, we discussed the principle of energy conservation as it applies to isolated systems — that is, to systems in which no energy enters or leaves the system. The first law of thermodynamics is an extension of that principle to systems that are not isolated. In such cases, energy may be transferred into or out of the system as either work W or heat Q. In our statement of the first law of thermodynamics above, we assume that there are no changes in the kinetic energy or the potential energy of the system as a whole; that is, %K # %U # 0. Rules. Before this chapter, the term work and the symbol W always meant the work done on a system. However, starting with Eq. 18-24 and continuing through the next two chapters about thermodynamics, we focus on the work done by a system, such as the gas in Fig. 18-13. The work done on a system is always the negative of the work done by the system, so if we rewrite Eq. 18-26 in terms of the work Won done on the system, we have %Eint # Q " Won. This tells us the following: The internal energy of a system tends to increase if heat is absorbed by the system or if positive work is done on the system. Conversely, the internal energy tends to decrease if heat is lost by the system or if negative work is done on the system.

Checkpoint 5 The figure here shows four paths on a p-V diagram along which a gas can be taken from state i to state f. Rank the paths according to (a) the change %Eint in the internal energy of the gas, (b) the work W done by the gas, and (c) the magnitude of the energy transferred as heat Q between the gas and its environment, greatest first.

p 4 3

i 2 1

f V

*Here dQ and dW, unlike dEint, are not true differentials; that is, there are no such functions as Q(p, V) and W(p, V) that depend only on the state of the system. The quantities dQ and dW are called inexact differentials and are usually represented by the symbols d¯Q and d¯W. For our purposes, we can treat them simply as infinitesimally small energy transfers.

531

532

CHAPTE R 18 TE M PE RATU R E, H EAT, AN D TH E FI RST L AW OF TH E R M ODYNAM ICS

We slowly remove lead shot, allowing an expansion without any heat transfer. Lead shot

Some Special Cases of the First Law of Thermodynamics Here are four thermodynamic processes as summarized in Table 18-5. 1. Adiabatic processes. An adiabatic process is one that occurs so rapidly or occurs in a system that is so well insulated that no transfer of energy as heat occurs between the system and its environment. Putting Q # 0 in the first law (Eq. 18-26) yields %Eint # $W

(adiabatic process).

(18-28)

This tells us that if work is done by the system (that is, if W is positive), the internal energy of the system decreases by the amount of work. Conversely, if work is done on the system (that is, if W is negative), the internal energy of the system increases by that amount. Figure 18-15 shows an idealized adiabatic process. Heat cannot enter or leave the system because of the insulation. Thus, the only way energy can be transferred between the system and its environment is by work. If we remove shot from the piston and allow the gas to expand, the work done by the system (the gas) is positive and the internal energy of the gas decreases. If, instead, we add shot and compress the gas, the work done by the system is negative and the internal energy of the gas increases.

W

Insulation

Figure 18-15 An adiabatic expansion can be carried out by slowly removing lead shot from the top of the piston. Adding lead shot reverses the process at any stage.

2. Constant-volume processes. If the volume of a system (such as a gas) is held constant, that system can do no work. Putting W # 0 in the first law (Eq. 18-26) yields %Eint # Q

Q#W

(18-30)

(cyclical process).

Thus, the net work done during the process must exactly equal the net amount of energy transferred as heat; the store of internal energy of the system remains unchanged. Cyclical processes form a closed loop on a p-V plot, as shown in Fig. 18-14f. We discuss such processes in detail in Chapter 20. 4. Free expansions. These are adiabatic processes in which no transfer of heat occurs between the system and its environment and no work is done on or by the system. Thus, Q # W # 0, and the first law requires that %Eint # 0

Insulation

Figure 18-16 The initial stage of a freeexpansion process. After the stopcock is opened, the gas fills both chambers and eventually reaches an equilibrium state.

(18-29)

Thus, if heat is absorbed by a system (that is, if Q is positive), the internal energy of the system increases. Conversely, if heat is lost during the process (that is, if Q is negative), the internal energy of the system must decrease. 3. Cyclical processes. There are processes in which, after certain interchanges of heat and work, the system is restored to its initial state. In that case, no intrinsic property of the system — including its internal energy — can possibly change. Putting %Eint # 0 in the first law (Eq. 18-26) yields

Stopcock

Vacuum

(constant-volume process).

(18-31)

(free expansion).

Figure 18-16 shows how such an expansion can be carried out. A gas, which is in thermal equilibrium within itself, is initially confined by a closed stopcock to one half of an insulated double chamber; the other half is evacuated. The stopcock is opened, and the gas expands freely to fill both halves of the chamber. No heat is Table 18-5 The First Law of Thermodynamics: Four Special Cases The Law: %Eint # Q $ W (Eq. 18-26) Process Adiabatic Constant volume Closed cycle Free expansion

Restriction

Consequence

Q#0 W#0 %Eint # 0 Q#W#0

%Eint # $W %Eint # Q Q#W %Eint # 0

18-5 TH E FI RST L AW OF TH E R M ODYNAM ICS

533

transferred to or from the gas because of the insulation. No work is done by the gas because it rushes into a vacuum and thus does not meet any pressure. A free expansion differs from all other processes we have considered because it cannot be done slowly and in a controlled way. As a result, at any given instant during the sudden expansion, the gas is not in thermal equilibrium and its pressure is not uniform. Thus, although we can plot the initial and final states on a p-V diagram, we cannot plot the expansion itself.

Checkpoint 6

p

For one complete cycle as shown in the p-V diagram here, are (a) %Eint for the gas and (b) the net energy transferred as heat Q positive, negative, or zero?

V

Sample Problem 18.05 First law of thermodynamics: work, heat, internal energy change Let 1.00 kg of liquid water at 100-C be converted to steam at 100-C by boiling at standard atmospheric pressure (which is 1.00 atm or 1.01 ' 10 5 Pa) in the arrangement of Fig. 18-17. The volume of that water changes from an initial value of 1.00 ' 10 $3 m3 as a liquid to 1.671 m3 as steam. (a) How much work is done by the system during this process?

%Eint # Q $ W # 2256 kJ $ 169 kJ

(1) The system must do positive work because the volume increases. (2) We calculate the work W done by integrating the pressure with respect to the volume (Eq. 18-25). Calculation: Because here the pressure is constant at 1.01 ' 10 5 Pa, we can take p outside the integral. Thus,

"

Vf

Vi

p dV # p

The change in the system’s internal energy is related to the heat (here, this is energy transferred into the system) and the work (here, this is energy transferred out of the system) by the first law of thermodynamics (Eq. 18-26). Calculation: We write the first law as

KEY IDEAS

W#

KEY IDEA

"

Vf

Vi

dV # p(Vf $ Vi )

# (1.01 ' 10 5 Pa)(1.671 m3 $ 1.00 ' 10 $3 m3) # 1.69 ' 10 5 J # 169 kJ.

% 2090 kJ # 2.09 MJ.

(Answer)

This quantity is positive, indicating that the internal energy of the system has increased during the boiling process. The added energy goes into separating the H2O molecules, which strongly attract one another in the liquid state. We see that, when water is boiled, about 7.5% (# 169 kJ/2260 kJ) of the heat goes into the work of pushing back the atmosphere. The rest of the heat goes into the internal energy of the system.

(Answer)

(b) How much energy is transferred as heat during the process?

Lead shot

KEY IDEA Because the heat causes only a phase change and not a change in temperature, it is given fully by Eq. 18-16 (Q # Lm). Calculation: Because the change is from liquid to gaseous phase, L is the heat of vaporization LV, with the value given in Eq. 18-17 and Table 18-4. We find Q # LVm # (2256 kJ/kg)(1.00 kg) # 2256 kJ % 2260 kJ.

(Answer)

(c) What is the change in the system’s internal energy during the process?

Figure 18-17 Water boiling at constant pressure. Energy is transferred from the thermal reservoir as heat until the liquid water has changed completely into steam. Work is done by the expanding gas as it lifts the loaded piston.

Additional examples, video, and practice available at WileyPLUS

W Steam

Insulation

Liquid water Q Thermal reservoir

T Control knob

534

CHAPTE R 18 TE M PE RATU R E, H EAT, AN D TH E FI RST L AW OF TH E R M ODYNAM ICS

18-6 HEAT TRANSFER MECHANISMS Learning Objectives After reading this module, you should be able to . . .

18.31 For thermal conduction through a layer, apply the relationship between the energy-transfer rate Pcond and the layer’s area A, thermal conductivity k, thickness L, and temperature difference %T (between its two sides). 18.32 For a composite slab (two or more layers) that has reached the steady state in which temperatures are no longer changing, identify that (by the conservation of energy) the rates of thermal conduction Pcond through the layers must be equal. 18.33 For thermal conduction through a layer, apply the relationship between thermal resistance R, thickness L, and thermal conductivity k. 18.34 Identify that thermal energy can be transferred by

convection, in which a warmer fluid (gas or liquid) tends to rise in a cooler fluid. 18.35 In the emission of thermal radiation by an object, apply the relationship between the energy-transfer rate Prad and the object’s surface area A, emissivity ´, and surface temperature T (in kelvins). 18.36 In the absorption of thermal radiation by an object, apply the relationship between the energy-transfer rate Pabs and the object’s surface area A and emissivity ´, and the environmental temperature T (in kelvins). 18.37 Calculate the net energy-transfer rate Pnet of an object emitting radiation to its environment and absorbing radiation from that environment.

Key Ideas ● The rate Pcond at which energy is conducted through a slab

for which one face is maintained at the higher temperature TH and the other face is maintained at the lower temperature TC is Pcond #

Q TH $ TC # kA . t L

Here each face of the slab has area A, the length of the slab (the distance between the faces) is L, and k is the thermal conductivity of the material. ● Convection occurs when temperature differences cause an energy transfer by motion within a fluid.

● Radiation is an energy transfer via the emission of electromagnetic energy. The rate Prad at which an object emits energy via thermal radiation is Prad # s´AT 4,

where s (# 5.6704 ' 10 $8 W/m2 9K4) is the Stefan – Boltzmann constant, ´ is the emissivity of the object’s surface, A is its surface area, and T is its surface temperature (in kelvins). The rate Pabs at which an object absorbs energy via thermal radiation from its environment, which is at the uniform temperature Tenv (in kelvins), is Pabs # s´AT 4env.

Heat Transfer Mechanisms We assume a steady transfer of energy as heat. L Hot reservoir at TH

k

Cold reservoir at TC

Q

TH > TC

Figure 18-18 Thermal conduction. Energy is transferred as heat from a reservoir at temperature TH to a cooler reservoir at temperature TC through a conducting slab of thickness L and thermal conductivity k.

We have discussed the transfer of energy as heat between a system and its environment, but we have not yet described how that transfer takes place. There are three transfer mechanisms: conduction, convection, and radiation. Let’s next examine these mechanisms in turn.

Conduction If you leave the end of a metal poker in a fire for enough time, its handle will get hot. Energy is transferred from the fire to the handle by (thermal) conduction along the length of the poker. The vibration amplitudes of the atoms and electrons of the metal at the fire end of the poker become relatively large because of the high temperature of their environment. These increased vibrational amplitudes, and thus the associated energy, are passed along the poker, from atom to atom, during collisions between adjacent atoms. In this way, a region of rising temperature extends itself along the poker to the handle. Consider a slab of face area A and thickness L, whose faces are maintained at temperatures TH and TC by a hot reservoir and a cold reservoir, as in Fig. 18-18. Let Q be the energy that is transferred as heat through the slab, from its hot face to its cold face, in time t. Experiment shows that the conduction rate Pcond (the

18-6 H EAT TRANSFE R M ECHAN ISM S

amount of energy transferred per unit time) is Pcond #

T $ TC Q # kA H , t L

535

Table 18-6 Some Thermal Conductivities (18-32)

in which k, called the thermal conductivity, is a constant that depends on the material of which the slab is made. A material that readily transfers energy by conduction is a good thermal conductor and has a high value of k. Table 18-6 gives the thermal conductivities of some common metals, gases, and building materials.

Thermal Resistance to Conduction (R-Value) If you are interested in insulating your house or in keeping cola cans cold on a picnic, you are more concerned with poor heat conductors than with good ones. For this reason, the concept of thermal resistance R has been introduced into engineering practice. The R-value of a slab of thickness L is defined as L (18-33) . R# k The lower the thermal conductivity of the material of which a slab is made, the higher the R-value of the slab; so something that has a high R-value is a poor thermal conductor and thus a good thermal insulator. Note that R is a property attributed to a slab of a specified thickness, not to a material. The commonly used unit for R (which, in the United States at least, is almost never stated) is the square foot – Fahrenheit degree – hour per British thermal unit (ft 2 9F-9h/Btu). (Now you know why the unit is rarely stated.)

Substance

k (W/m 9K)

Metals Stainless steel Lead Iron Brass Aluminum Copper Silver

14 35 67 109 235 401 428

Gases Air (dry) Helium Hydrogen

0.026 0.15 0.18

Building Materials Polyurethane foam Rock wool Fiberglass White pine Window glass

0.024 0.043 0.048 0.11 1.0

Conduction Through a Composite Slab Figure 18-19 shows a composite slab, consisting of two materials having different thicknesses L1 and L2 and different thermal conductivities k1 and k2. The temperatures of the outer surfaces of the slab are TH and TC. Each face of the slab has area A. Let us derive an expression for the conduction rate through the slab under the assumption that the transfer is a steady-state process; that is, the temperatures everywhere in the slab and the rate of energy transfer do not change with time. In the steady state, the conduction rates through the two materials must be equal. This is the same as saying that the energy transferred through one material in a certain time must be equal to that transferred through the other material in the same time. If this were not true, temperatures in the slab would be changing and we would not have a steady-state situation. Letting TX be the temperature of the interface between the two materials, we can now use Eq. 18-32 to write Pcond #

k2A(TH $ TX) k1A(TX $ TC) # . L2 L1

(18-34)

Hot reservoir at TH

L2

L1

k2

k1

Cold reservoir at TC

Solving Eq. 18-34 for TX yields, after a little algebra, TX #

k1L2TC " k2L1TH . k1L2 " k2L1

Q

(18-35) TX

Substituting this expression for TX into either equality of Eq. 18-34 yields Pcond #

A(TH $ TC) . L1/k1 " L2/k2

(18-36)

We can extend Eq. 18-36 to apply to any number n of materials making up a slab: A(TH $ TC) Pcond # (18-37) . ' (L/k) The summation sign in the denominator tells us to add the values of L/k for all the materials.

The energy transfer per second here ...

... equals the energy transfer per second here.

Figure 18-19 Heat is transferred at a steady rate through a composite slab made up of two different materials with different thicknesses and different thermal conductivities. The steady-state temperature at the interface of the two materials is TX.

536

CHAPTE R 18 TE M PE RATU R E, H EAT, AN D TH E FI RST L AW OF TH E R M ODYNAM ICS

Checkpoint 7 The figure shows the face and 25°C 15°C 10°C –5.0°C –10°C interface temperatures of a coma b c d posite slab consisting of four materials, of identical thicknesses, through which the heat transfer is steady. Rank the materials according to their thermal conductivities, greatest first.

Convection

Edward Kinsman/Photo Researchers, Inc.

Figure 18-20 A false-color thermogram reveals the rate at which energy is radiated by a cat.The rate is color-coded, with white and red indicating the greatest radiation rate.The nose is cool.

When you look at the flame of a candle or a match, you are watching thermal energy being transported upward by convection. Such energy transfer occurs when a fluid, such as air or water, comes in contact with an object whose temperature is higher than that of the fluid. The temperature of the part of the fluid that is in contact with the hot object increases, and (in most cases) that fluid expands and thus becomes less dense. Because this expanded fluid is now lighter than the surrounding cooler fluid, buoyant forces cause it to rise. Some of the surrounding cooler fluid then flows so as to take the place of the rising warmer fluid, and the process can then continue. Convection is part of many natural processes. Atmospheric convection plays a fundamental role in determining global climate patterns and daily weather variations. Glider pilots and birds alike seek rising thermals (convection currents of warm air) that keep them aloft. Huge energy transfers take place within the oceans by the same process. Finally, energy is transported to the surface of the Sun from the nuclear furnace at its core by enormous cells of convection, in which hot gas rises to the surface along the cell core and cooler gas around the core descends below the surface.

Radiation The third method by which an object and its environment can exchange energy as heat is via electromagnetic waves (visible light is one kind of electromagnetic wave). Energy transferred in this way is often called thermal radiation to distinguish it from electromagnetic signals (as in, say, television broadcasts) and from nuclear radiation (energy and particles emitted by nuclei). (To “radiate” generally means to emit.) When you stand in front of a big fire, you are warmed by absorbing thermal radiation from the fire; that is, your thermal energy increases as the fire’s thermal energy decreases. No medium is required for heat transfer via radiation — the radiation can travel through vacuum from, say, the Sun to you. The rate Prad at which an object emits energy via electromagnetic radiation depends on the object’s surface area A and the temperature T of that area in kelvins and is given by Prad # s´AT 4.

(18-38)

Here s # 5.6704 ' 10 $8 W/m2 9K4 is called the Stefan – Boltzmann constant after Josef Stefan (who discovered Eq. 18-38 experimentally in 1879) and Ludwig Boltzmann (who derived it theoretically soon after). The symbol ´ represents the emissivity of the object’s surface, which has a value between 0 and 1, depending on the composition of the surface. A surface with the maximum emissivity of 1.0 is said to be a blackbody radiator, but such a surface is an ideal limit and does not occur in nature. Note again that the temperature in Eq. 18-38 must be in kelvins so that a temperature of absolute zero corresponds to no radiation. Note also that every object whose temperature is above 0 K — including you — emits thermal radiation. (See Fig. 18-20.)

537

18-6 H EAT TRANSFE R M ECHAN ISM S

The rate Pabs at which an object absorbs energy via thermal radiation from its environment, which we take to be at uniform temperature Tenv (in kelvins), is Pabs # s´AT 4env.

(18-39)

The emissivity ´ in Eq. 18-39 is the same as that in Eq. 18-38.An idealized blackbody radiator, with ´ # 1, will absorb all the radiated energy it intercepts (rather than sending a portion back away from itself through reflection or scattering). Because an object both emits and absorbs thermal radiation, its net rate Pnet of energy exchange due to thermal radiation is Pnet # Pabs $ Prad # s´A(T 4env $ T 4).

(18-40)

Pnet is positive if net energy is being absorbed via radiation and negative if it is being lost via radiation. Thermal radiation is involved in the numerous medical cases of a dead rattlesnake striking a hand reaching toward it. Pits between each eye and nostril of a rattlesnake (Fig. 18-21) serve as sensors of thermal radiation.When, say, a mouse moves close to a rattlesnake’s head, the thermal radiation from the mouse triggers these sensors, causing a reflex action in which the snake strikes the mouse with its fangs and injects its venom. The thermal radiation from a reaching hand can cause the same reflex action even if the snake has been dead for as long as 30 min because the snake’s nervous system continues to function. As one snake expert advised, if you must remove a recently killed rattlesnake, use a long stick rather than your hand.

© David A. Northcott/Corbis Images

Figure 18-21 A rattlesnake’s face has thermal radiation detectors, allowing the snake to strike at an animal even in complete darkness.

Sample Problem 18.06 Thermal conduction through a layered wall Figure 18-22 shows the cross section of a wall made of white pine of thickness La and brick of thickness Ld (# 2.0La), sandwiching two layers of unknown material with identical thicknesses and thermal conductivities. The thermal conductivity of the pine is ka and that of the brick is kd (# 5.0ka). The face area A of the wall is unknown. Thermal conduction through the wall has reached the steady state; the only known interface temperatures are T1 # 25-C, T2 # 20-C, and T5 # $10-C. What is interface temperature T4?

KEY IDEAS (1) Temperature T4 helps determine the rate Pd at which energy is conducted through the brick, as given by Eq. 18-32. However, we lack enough data to solve Eq. 18-32 for T4. (2) Because the conduction is steady, the conduction rate Pd through the brick must equal the conduction rate Pa through the pine.That gets us going. Calculations: From Eq. 18-32 and Fig. 18-22, we can write Pa # kaA

T1

Indoors

T2

T3

T4

T5

ka

kb

kc

kd

La

Lb

Lc

Ld

(a )

(b )

(c )

(d )

Figure 18-22 Steady-state heat transfer through a wall.

and

Pd # kdA

T4 $ T5 . Ld

Setting Pa # Pd and solving for T4 yield T4 #

Outdoors

The energy transfer per second is the same in each layer.

T1 $ T2 La

kaLd (T $ T2) " T5. kdLa 1

Letting Ld # 2.0La and kd # 5.0ka, and inserting the known temperatures, we find T4 #

ka(2.0La) (25-C $ 20-C) " ($10-C) (5.0ka)La

# $8.0-C.

Additional examples, video, and practice available at WileyPLUS

(Answer)

538

CHAPTE R 18 TE M PE RATU R E, H EAT, AN D TH E FI RST L AW OF TH E R M ODYNAM ICS

Sample Problem 18.07 Thermal radiation by a skunk cabbage can melt surrounding snow Unlike most other plants, a skunk cabbage can regulate its internal temperature (set at T # 22-C) by altering the rate at which it produces energy. If it becomes covered with snow, it can increase that production so that its thermal radiation melts the snow in order to re-expose the plant to sunlight. Let’s model a skunk cabbage with a cylinder of height h # 5.0 cm and radius R # 1.5 cm and assume it is surrounded by a snow wall at temperature Tenv # $3.0-C (Fig. 18-23). If the emissivity ´ is 0.80, what is the net rate of energy exchange via thermal radiation between the plant’s curved side and the snow?

KEY IDEAS (1) In a steady-state situation, a surface with area A, emissivity ´, and temperature T loses energy to thermal radiation at the rate given by Eq. 18-38 (Prad # s´AT 4). (2) Simultaneously, it gains energy by thermal radiation from its environment at temperature Tenv at the rate given by Eq. 18-39 (Penv # s´AT 4env). Calculations: To find the net rate of energy exchange, we subtract Eq. 18-38 from Eq. 18-39 to write Pnet # Pabs $ Prad # s´A(T 4env $ T 4).

R h

We need the area of the curved surface of the cylinder, which is A # h(2pR). We also need the temperatures in kelvins: Tenv # 273 K $ 3 K # 270 K and T # 273 K " 22 K # 295 K. Substituting in Eq. 18-41 for A and then substituting known values in SI units (which are not displayed here), we find Pnet # (5.67 ' 10$8)(0.80)(0.050)(2p)(0.015)(2704 $ 2954) # $0.48 W.

Figure 18-23 Model of skunk cabbage that has melted snow to uncover itself.

(18-41)

(Answer)

Thus, the plant has a net loss of energy via thermal radiation of 0.48 W. The plant’s energy production rate is comparable to that of a hummingbird in flight.

Additional examples, video, and practice available at WileyPLUS

Review & Summary Temperature; Thermometers Temperature is an SI base quantity related to our sense of hot and cold. It is measured with a thermometer, which contains a working substance with a measurable property, such as length or pressure, that changes in a regular way as the substance becomes hotter or colder.

Zeroth Law of Thermodynamics

When a thermometer and some other object are placed in contact with each other, they eventually reach thermal equilibrium.The reading of the thermometer is then taken to be the temperature of the other object. The process provides consistent and useful temperature measurements because of the zeroth law of thermodynamics: If bodies A and B are each in thermal equilibrium with a third body C (the thermometer), then A and B are in thermal equilibrium with each other.

The Kelvin Temperature Scale In the SI system, temperature is measured on the Kelvin scale, which is based on the triple point of water (273.16 K). Other temperatures are then defined by

use of a constant-volume gas thermometer, in which a sample of gas is maintained at constant volume so its pressure is proportional to its temperature. We define the temperature T as measured with a gas thermometer to be T # (273.16 K)

# lim pp $. gas:0

(18-6)

3

Here T is in kelvins, and p3 and p are the pressures of the gas at 273.16 K and the measured temperature, respectively.

Celsius and Fahrenheit Scales The Celsius temperature scale is defined by TC # T $ 273.15-,

(18-7)

with T in kelvins.The Fahrenheit temperature scale is defined by TF # 95TC " 32-.

(18-8)

R EVI EW & SU M MARY

Thermal Expansion All objects change size with changes in temperature. For a temperature change %T, a change %L in any linear dimension L is given by

539

The integration is necessary because the pressure p may vary during the volume change.

First Law of Thermodynamics The principle of conser%L # La %T,

(18-9)

in which a is the coefficient of linear expansion. The change %V in the volume V of a solid or liquid is %V # Vb %T.

(18-10)

Here b # 3a is the material’s coefficient of volume expansion.

Heat Heat Q is energy that is transferred between a system and its environment because of a temperature difference between them. It can be measured in joules (J), calories (cal), kilocalories (Cal or kcal), or British thermal units (Btu), with 1 cal # 3.968 ' 10 $3 Btu # 4.1868 J.

(18-12)

Heat Capacity and Specific Heat If heat Q is absorbed by an object, the object’s temperature change Tf $ Ti is related to Q by

vation of energy for a thermodynamic process is expressed in the first law of thermodynamics, which may assume either of the forms %Eint # Eint, f $ Eint,i # Q $ W or

dEint # dQ $ dW

(first law)

(18-26) (18-27)

(first law).

Eint represents the internal energy of the material, which depends only on the material’s state (temperature, pressure, and volume). Q represents the energy exchanged as heat between the system and its surroundings; Q is positive if the system absorbs heat and negative if the system loses heat. W is the work done by the system; W is positive if the system expands against an external force from the surroundings and negative if the system contracts because of an external force. Q and W are path dependent; %Eint is path independent.

Applications of the First Law The first law of thermodynamics finds application in several special cases:

Q # C(Tf $ Ti),

(18-13) adiabatic processes:

in which C is the heat capacity of the object. If the object has mass m, then Q # cm(Tf $ Ti),

(18-14)

Heat of Transformation Matter can exist in three common states: solid, liquid, and vapor. Heat absorbed by a material may change the material’s physical state—for example, from solid to liquid or from liquid to gas.The amount of energy required per unit mass to change the state (but not the temperature) of a particular material is its heat of transformation L.Thus, (18-16)

The heat of vaporization LV is the amount of energy per unit mass that must be added to vaporize a liquid or that must be removed to condense a gas. The heat of fusion LF is the amount of energy per unit mass that must be added to melt a solid or that must be removed to freeze a liquid.

Work Associated with Volume Change A gas may exchange energy with its surroundings through work. The amount of work W done by a gas as it expands or contracts from an initial volume Vi to a final volume Vf is given by

W#

"

dW #

"

Vf

Vi

p dV.

cyclical processes: free expansions:

where c is the specific heat of the material making up the object. The molar specific heat of a material is the heat capacity per mole, which means per 6.02 ' 10 23 elementary units of the material.

Q # Lm.

constant-volume processes:

(18-25)

Q # 0,

%Eint # $W

W # 0, %Eint # Q %Eint # 0,

Q#W

Q # W # %Eint # 0

Conduction, Convection, and Radiation The rate Pcond at which energy is conducted through a slab for which one face is maintained at the higher temperature TH and the other face is maintained at the lower temperature TC is Pcond #

Q TH $ TC # kA t L

(18-32)

Here each face of the slab has area A, the length of the slab (the distance between the faces) is L, and k is the thermal conductivity of the material. Convection occurs when temperature differences cause an energy transfer by motion within a fluid. Radiation is an energy transfer via the emission of electromagnetic energy. The rate Prad at which an object emits energy via thermal radiation is Prad # s´AT 4,

(18-38)

where s (# 5.6704 ' 10 $8 W/m2 9K4) is the Stefan – Boltzmann constant, ´ is the emissivity of the object’s surface, A is its surface area, and T is its surface temperature (in kelvins). The rate Pabs at which an object absorbs energy via thermal radiation from its environment, which is at the uniform temperature Tenv (in kelvins), is Pabs # s´AT 4env.

(18-39)

540

CHAPTE R 18 TE M PE RATU R E, H EAT, AN D TH E FI RST L AW OF TH E R M ODYNAM ICS

Questions 1 The initial length L, change in temperature %T, and change in length %L of four rods are given in the following table. Rank the rods according to their coefficients of thermal expansion, greatest first. Rod

L (m)

%T (C-)

%L (m)

a b c d

2 1 2 4

10 20 10 5

4 ' 10 $4 4 ' 10 $4 8 ' 10 $4 4 ' 10 $4

2 Figure 18-24 shows three linear temperature scales, with the freezing 150° and boiling points of water indicated. X Rank the three scales according to the size of one degree on them, great–50° est first.

120°

60°

Y

Z

–140°

20°

Figure 18-24 Question 2. 3 Materials A, B, and C are solids that are at their melting temperatures. Material A requires 200 J to melt 4 kg, material B requires 300 J to melt 5 kg, and material C requires 300 J to melt 6 kg. Rank the materials according to their heats of fusion, greatest first. 4 A sample A of liquid water and a sample B of ice, of identical mass, are placed in a thermally insulated container and allowed to come to thermal equilibrium. Figure 18-25a is a sketch of the temperature T of the samples versus time t. (a) Is the equilibrium temperature above, below, or at the freezing point of water? (b) In reaching equilibrium, does the liquid partly freeze, fully freeze, or undergo no freezing? (c) Does the ice partly melt, fully melt, or undergo no melting? 5 Question 4 continued: Graphs b through f of Fig. 18-25 are additional sketches of T versus t, of which one or more are impossible to produce. (a) Which is impossible and why? (b) In the possible ones, is the equilibrium temperature above, below, or at the freezing point of water? (c) As the possible situations reach equilibrium, does the liquid partly freeze, fully freeze, or undergo no freezing? Does the ice partly melt, fully melt, or undergo no melting? T

(a)

T

t

(b)

T

T

t

(c)

T

t T

6 Figure 18-26 shows three different arrange1 2 3 1 3 2 3 1 2 ments of materials 1, 2, and 3 to form a wall. The thermal conductivities are k1 4 (a) (b) (c) k2 4 k3.The left side of the Figure 18-26 Question 6. wall is 20 C- higher than the right side. Rank the arrangements according to (a) the (steady state) rate of energy conduction through the wall and (b) the temperature difference across material 1, greatest first. 7 Figure 18-27 shows p p two closed cycles on p-V diagrams for a gas. The three parts of cycle 1 are of the same length V V and shape as those of cy(1) (2) cle 2. For each cycle, should the cycle be traFigure 18-27 Questions 7 and 8. versed clockwise or counterclockwise if (a) the net work W done by the gas is to be positive and (b) the net energy transferred by the gas as heat Q is to be positive? 8 For which cycle in Fig. 18-27, traversed clockwise, is (a) W greater and (b) Q greater? 9 Three different materials of T identical mass are placed one at a 1 time in a special freezer that can ex2 tract energy from a material at a certain constant rate. During the 3 cooling process, each material begins t in the liquid state and ends in the Figure 18-28 Question 9. solid state; Fig. 18-28 shows the temperature T versus time t. (a) For material 1, is the specific heat for the liquid state greater than or less than that for the solid state? Rank the materials according to (b) freezingpoint temperature, (c) specific heat in the liquid state, (d) specific heat in the solid state, and (e) heat of fusion, all greatest first. 10 A solid cube of edge length r, a solid sphere of radius r, and a solid hemisphere of radius r, all made of the same material, are maintained at temperature 300 K in an environment at temperature 350 K. Rank the objects according to the net rate at which thermal radiation is exchanged with the environment, greatest first. 11 A hot object is dropped into a thermally insulated container of water, and the object and water are then allowed to come to thermal equilibrium. The experiment is repeated twice, with different hot objects. All three objects have the same mass and initial temperature, and the mass and initial temperature of the water are the same in the three experiments. For each of the experiments, Fig. 18-29 gives graphs of the temperatures T of the object and the water versus time t. Rank the graphs according to the specific heats of the objects, greatest first. T

(d)

t

(e)

t

(f )

Figure 18-25 Questions 4 and 5.

t

(a)

T

t

(b)

T

t

Figure 18-29 Question 11.

(c)

t

PROB LE M S

541

Problems Tutoring problem available (at instructor’s discretion) in WileyPLUS and WebAssign SSM • – •••

Worked-out solution available in Student Solutions Manual

WWW Worked-out solution is at

Number of dots indicates level of problem difficulty

ILW

Interactive solution is at

http://www.wiley.com/college/halliday

Additional information available in The Flying Circus of Physics and at flyingcircusofphysics.com

Module 18-1 Temperature •1 Suppose the temperature of a gas is 373.15 K when it is at the boiling point of water. What then is the limiting value of the ratio of the pressure of the gas at that boiling point to its pressure at the triple point of water? (Assume the volume of the gas is the same at both temperatures.) •2 Two constant-volume gas thermometers are assembled, one with nitrogen and the other with hydrogen. Both contain enough gas so that p3 # 80 kPa. (a) What is the difference between the pressures in the two thermometers if both bulbs are in boiling water? (Hint: See Fig. 18-6.) (b) Which gas is at higher pressure? •3 A gas thermometer is constructed of two gas-containing bulbs, each in a water bath, as shown in Fig. 18-30.The pressure difference between the two bulbs is measured by a mercury manometer as shown. Figure 18-30 Problem 3. Appropriate reservoirs, not shown in the diagram, maintain constant gas volume in the two bulbs. There is no difference in pressure when both baths are at the triple point of water. The pressure difference is 120 torr when one bath is at the triple point and the other is at the boiling point of water. It is 90.0 torr when one bath is at the triple point and the other is at an unknown temperature to be measured.What is the unknown temperature? Module 18-2 The Celsius and Fahrenheit Scales •4 (a) In 1964, the temperature in the Siberian village of Oymyakon reached $71-C. What temperature is this on the Fahrenheit scale? (b) The highest officially recorded temperature in the continental United States was 134-F in Death Valley, California. What is this temperature on the Celsius scale? •5 At what temperature is the Fahrenheit scale reading equal to (a) twice that of the Celsius scale and (b) half that of the Celsius scale? ••6 On a linear X temperature scale, water freezes at $125.0-X and boils at 375.0-X. On a linear Y temperature scale, water freezes at $70.00-Y and boils at $30.00-Y. A temperature of 50.00-Y corresponds to what temperature on the X scale? ••7 ILW Suppose that on a linear temperature scale X, water boils at $53.5-X and freezes at $170-X. What is a temperature of 340 K on the X scale? (Approximate water’s boiling point as 373 K.) Module 18-3 Thermal Expansion •8 At 20-C, a brass cube has edge length 30 cm. What is the increase in the surface area when it is heated from 20-C to 75-C? •9 ILW A circular hole in an aluminum plate is 2.725 cm in diameter at 0.000-C. What is its diameter when the temperature of the plate is raised to 100.0-C? •10 An aluminum flagpole is 33 m high. By how much does its length increase as the temperature increases by 15 C-? •11 What is the volume of a lead ball at 30.00-C if the ball’s volume at 60.00-C is 50.00 cm3?

•12 An aluminum-alloy rod has a length of 10.000 cm at 20.000-C and a length of 10.015 cm at the boiling point of water. (a) What is the length of the rod at the freezing point of water? (b) What is the temperature if the length of the rod is 10.009 cm? •13 SSM Find the change in volume of an aluminum sphere with an initial radius of 10 cm when the sphere is heated from 0.0-C to 100-C. ••14 When the temperature of a copper coin is raised by 100 C-, its diameter increases by 0.18%. To two significant figures, give the percent increase in (a) the area of a face, (b) the thickness, (c) the volume, and (d) the mass of the coin. (e) Calculate the coefficient of linear expansion of the coin. ••15 ILW A steel rod is 3.000 cm in diameter at 25.00-C.A brass ring has an interior diameter of 2.992 cm at 25.00-C. At what common temperature will the ring just slide onto the rod? ••16 When the temperature of a metal cylinder is raised from 0.0-C to 100-C, its length increases by 0.23%. (a) Find the percent change in density. (b) What is the metal? Use Table 18-2. ••17 SSM WWW An aluminum cup of 100 cm3 capacity is completely filled with glycerin at 22-C. How much glycerin, if any, will spill out of the cup if the temperature of both the cup and the glycerin is increased to 28-C? (The coefficient of volume expansion of glycerin is 5.1 ' 10 $4/C-.) ••18 At 20-C, a rod is exactly 20.05 cm long on a steel ruler. Both are placed in an oven at 270-C, where the rod now measures 20.11 cm on the same ruler. What is the coefficient of linear expansion for the material of which the rod is made? ••19 A vertical glass tube of length L # 1.280 000 m is half filled with a liquid at 20.000 000-C. How much will the height of the liquid column change when the tube and liquid are heated to 30.000 000-C? Use coefficients aglass # 1.000 000 ' 10 $5/K and bliquid # 4.000 000 ' 10 $5/K. ••20 In a certain experiment, a Radioactive Electric heater source small radioactive source must move at selected, extremely slow speeds. This motion is accomClamp d plished by fastening the source to Figure 18-31 Problem 20. one end of an aluminum rod and heating the central section of the rod in a controlled way. If the effective heated section of the rod in Fig. 18-31 has length d # 2.00 cm, at what constant rate must the temperature of the rod be changed if the source is to move at a constant speed of 100 nm/s? •••21 SSM ILW As a result of a temperature rise of 32 C-, a bar with a crack at its center buckles upward (Fig. 18-32). The fixed distance L0 is 3.77 m and the coefficient of linear expansion of the bar is 25 ' 10 $6/C-. Find the rise x of the center.

L0

x L0

Figure 18-32 Problem 21.

CHAPTE R 18 TE M PE RATU R E, H EAT, AN D TH E FI RST L AW OF TH E R M ODYNAM ICS

•23 SSM A small electric immersion heater is used to heat 100 g of water for a cup of instant coffee. The heater is labeled “200 watts” (it converts electrical energy to thermal energy at this rate). Calculate the time required to bring all this water from 23.0-C to 100-C, ignoring any heat losses. •24 A certain substance has a mass per mole of 50.0 g/mol. When 314 J is added as heat to a 30.0 g sample, the sample’s temperature rises from 25.0-C to 45.0-C. What are the (a) specific heat and (b) molar specific heat of this substance? (c) How many moles are in the sample? •25 A certain diet doctor encourages people to diet by drinking ice water. His theory is that the body must burn off enough fat to raise the temperature of the water from 0.00-C to the body temperature of 37.0-C. How many liters of ice water would have to be consumed to burn off 454 g (about 1 lb) of fat, assuming that burning this much fat requires 3500 Cal be transferred to the ice water? Why is it not advisable to follow this diet? (One liter # 10 3 cm3. The density of water is 1.00 g/cm3.) •26 What mass of butter, which has a usable energy content of 6.0 Cal/g (# 6000 cal/g), would be equivalent to the change in gravitational potential energy of a 73.0 kg man who ascends from sea level to the top of Mt. Everest, at elevation 8.84 km? Assume that the average g for the ascent is 9.80 m/s2. •27 SSM Calculate the minimum amount of energy, in joules, required to completely melt 130 g of silver initially at 15.0-C. •28 How much water remains unfrozen after 50.2 kJ is transferred as heat from 260 g of liquid water initially at its freezing point? ••29 In a solar water heater, energy from the Sun is gathered by water that circulates through tubes in a rooftop collector. The solar radiation enters the collector through a transparent cover and warms the water in the tubes; this water is pumped into a holding tank. Assume that the efficiency of the overall system is 20% (that is, 80% of the incident solar energy is lost from the system). What collector area is necessary to raise the temperature of 200 L of water in the tank from 20-C to 40°C in 1.0 h when the intensity of incident sunlight is 700 W/m2?

T (K)

••30 A 0.400 kg sample is placed in a cooling apparatus that removes energy as heat at a constant rate. Figure 18-33 gives 300 the temperature T of the sample versus time t; the horizontal scale is set by ts # 80.0 min. 270 The sample freezes during the energy removal. The specific 250 heat of the sample in its initial liquid phase is 3000 J/kg 9K. 0 ts What are (a) the sample’s heat t (min) of fusion and (b) its specific Figure 18-33 Problem 30. heat in the frozen phase?

••31 ILW What mass of steam at 100-C must be mixed with 150 g of ice at its melting point, in a thermally insulated container, to produce liquid water at 50-C? ••32 The specific heat of a substance varies with temperature according to the function c # 0.20 " 0.14T " 0.023T 2, with T in -C and c in cal/g 9K. Find the energy required to raise the temperature of 2.0 g of this substance from 5.0-C to 15-C. ••33 Nonmetric version: (a) How long does a 2.0 ' 10 5 Btu/h water heater take to raise the temperature of 40 gal of water from 70-F to 100°F? Metric version: (b) How long does a 59 kW water heater take to raise the temperature of 150 L of water from 21-C to 38-C? ••34 Samples A and B are at different initial temperatures when they are placed in a thermally insulated container and allowed to come to thermal equilibrium. Figure 18-34a gives their temperatures T versus time t. Sample A has a mass of 5.0 kg; sample B has a mass of 1.5 kg. Figure 18-34b is a general plot for the material of sample B. It shows the temperature change %T that the material undergoes when energy is transferred to it as heat Q. The change %T is plotted versus the energy Q per unit mass of the material, and the scale of the vertical axis is set by %Ts # 4.0 C-. What is the specific heat of sample A? ∆Ts

100

A

∆T (C°)

Module 18-4 Absorption of Heat •22 One way to keep the contents of a garage from becoming too cold on a night when a severe subfreezing temperature is forecast is to put a tub of water in the garage. If the mass of the water is 125 kg and its initial temperature is 20-C, (a) how much energy must the water transfer to its surroundings in order to freeze completely and (b) what is the lowest possible temperature of the water and its surroundings until that happens?

T (°C)

542

60 B 20

0

10 t (min)

20

0

(a)

8 Q /m (kJ/kg)

16

(b)

Figure 18-34 Problem 34. ••35 An insulated Thermos contains 130 cm3 of hot coffee at 80.0-C. You put in a 12.0 g ice cube at its melting point to cool the coffee. By how many degrees has your coffee cooled once the ice has melted and equilibrium is reached? Treat the coffee as though it were pure water and neglect energy exchanges with the environment. ••36 A 150 g copper bowl contains 220 g of water, both at 20.0-C.A very hot 300 g copper cylinder is dropped into the water, causing the water to boil, with 5.00 g being converted to steam. The final temperature of the system is 100-C. Neglect energy transfers with the environment. (a) How much energy (in calories) is transferred to the water as heat? (b) How much to the bowl? (c) What is the original temperature of the cylinder? ••37 A person makes a quantity of iced tea by mixing 500 g of hot tea (essentially water) with an equal mass of ice at its melting point. Assume the mixture has negligible energy exchanges with its environment. If the tea’s initial temperature is Ti # 90-C, when thermal equilibrium is reached what are (a) the mixture’s temperature Tf and (b) the remaining mass mf of ice? If Ti # 70-C, when thermal equilibrium is reached what are (c) Tf and (d) mf? ••38 A 0.530 kg sample of liquid water and a sample of ice are placed in a thermally insulated container. The container also contains a device that transfers energy as heat from the liquid water to the ice at a constant rate P, until thermal equilibrium is

543

PROB LE M S

reached. The temperatures T of the liquid water and the ice are given in Fig. 18-35 as functions of time t; the horizontal scale is set by ts # 80.0 min. (a) What is rate P? (b) What is the initial mass of the ice in the container? (c) When thermal equilibrium is reached, what is the mass of the ice produced in this process? 40

state C, and then back to A, as shown in the p-V diagram of Fig. 1838a. The vertical scale is set by ps # 40 Pa, and the horizontal scale is set by Vs # 4.0 m3. (a) – (g) Complete the table in Fig. 18-38b by inserting a plus sign, a minus sign, or a zero in each indicated cell. (h) What is the net work done by the system as it moves once through the cycle ABCA?

Pressure (Pa)

0 –20 0

Figure 18-35 Problem 38.

(a)

••39 Ethyl alcohol has a boiling point of 78.0-C, a freezing point of $114-C, a heat of vaporization of 879 kJ/kg, a heat of fusion of 109 kJ/kg, and a specific heat of 2.43 kJ/kg9K. How much energy must be removed from 0.510 kg of ethyl alcohol that is initially a gas at 78.0-C so that it becomes a solid at $114-C? ••40 Calculate the specific heat of a metal from the following data. A container made of the metal has a mass of 3.6 kg and contains 14 kg of water. A 1.8 kg piece of the metal initially at a temperature of 180-C is dropped into the water. The container and water initially have a temperature of 16.0-C, and the final temperature of the entire (insulated) system is 18.0-C. •••41 SSM WWW (a) Two 50 g ice cubes are dropped into 200 g of water in a thermally insulated container. If the water is initially at 25-C, and the ice comes directly from a freezer at $15-C, what is the final temperature at thermal equilibrium? (b) What is the final temperature if only one ice cube is used?

•44 A thermodynamic system is taken from state A to state B to

Al

B

A

0

Volume (m3)

D

Figure 18-36 Problem 42.

Vs

(b)

A

B (a)

B

C

C

A (e)

+

(b)

+

(c)

(d)

(f)

(g)

Figure 18-38 Problem 44. •45 SSM ILW A gas within a closed chamber undergoes the cycle shown in the p-V diagram of Fig. 18-39. The horizontal scale is set by Vs # 4.0 m3. Calculate the net energy added to the system as heat during one complete cycle.

40 C

30

B

20 10

A

•46 Suppose 200 J of work is 0 Vs done on a system and 70.0 cal is Volume (m3) extracted from the system as heat. In the sense of the first law Figure 18-39 Problem 45. of thermodynamics, what are the values (including algebraic signs) of (a) W, (b) Q, and (c) %Eint? ••47 SSM WWW When a system is taken from state i to state f along path iaf in Fig. 18-40, Q # 50 cal and W # 20 cal. Along path ibf, Q # 36 cal. (a) What is W along path ibf? (b) If W # $13 cal for the return path fi, what is Q for this path? (c) If Eint,i # 10 cal, what is Eint, f? If Eint,b # 22 cal, what is Q for (d) path ib and (e) path bf ? Pressure

Cu

0

a

f

i

b Volume

Figure 18-40 Problem 47.

C

0

B

V0

4.0V0 3

Volume (m )

Figure 18-37 Problem 43.

••48 As a gas is held within a closed chamber, it passes through the cycle shown in Fig. 18-41. Determine the energy transferred by the system as heat during constant-pressure process CA if the energy added as heat QAB during constant-volume process AB is 20.0 J, no energy is transferred as heat during adiabatic process BC, and the net work done during the cycle is 15.0 J.

B

Pressure

A

p0 Pressure (Pa)

Module 18-5 The First Law of Thermodynamics •43 In Fig. 18-37, a gas sample expands from V0 to 4.0V0 while its pressure decreases from p0 to p0 /4.0. If V0 # 1.0 m3 and p0 # 40 Pa, how much work is done by the gas if its pressure changes with volume via (a) path A, (b) path B, and (c) path C?

d

W ∆Eint

Q

ts

t (min)

•••42 A 20.0 g copper ring at 0.000-C has an inner diameter of D # 2.54000 cm. An aluminum sphere at 100.0-C has a diameter of d # 2.545 08 cm. The sphere is put on top of the ring (Fig. 18-36), and the two are allowed to come to thermal equilibrium, with no heat lost to the surroundings. The sphere just passes through the ring at the equilibrium temperature. What is the mass of the sphere?

C

ps

Pressure (N/m2)

T (°C)

20

A

0

C

Volume

Figure 18-41 Problem 48.

544

CHAPTE R 18 TE M PE RATU R E, H EAT, AN D TH E FI RST L AW OF TH E R M ODYNAM ICS

••49 Figure 18-42 represents a closed cycle for a gas (the figure is not drawn to scale). The change in the internal energy of the gas as it moves from a to c along the path abc is $200 J. As it moves from c to d, 180 J must be transferred to it as heat. An additional transfer of 80 J to it as heat is needed as it moves from d to a. How much work is done on the gas as it moves from c to d?

p

••50 A lab sample of gas is taken through cycle abca shown in the p-V diagram of Fig. 18-43. The net work done is "1.2 J. Along path ab, the change in the internal energy is "3.0 J and the magnitude of the work done is 5.0 J. Along path ca, the energy transferred to the gas as heat is "2.5 J. How much energy is transferred as heat along (a) path ab and (b) path bc?

p

a b c

d V

Figure 18-42 Problem 49.

a

••57 (a) What is the rate of energy loss in watts per square meter through a glass window 3.0 mm thick if the outside temperature is $20-F and the inside temperature is "72-F? (b) A storm window having the same thickness of glass is installed parallel to the first window, with an air gap of 7.5 cm between the two windows. What now is the rate of energy loss if conduction is the only important energy-loss mechanism?

b

c V

Figure 18-43 Problem 50.

Module 18-6 Heat Transfer Mechanisms •51 A sphere of radius 0.500 m, temperature 27.0-C, and emissivity 0.850 is located in an environment of temperature 77.0-C. At what rate does the sphere (a) emit and (b) absorb thermal radiation? (c) What is the sphere’s net rate of energy exchange? •52 The ceiling of a single-family dwelling in a cold climate should have an R-value of 30. To give such insulation, how thick would a layer of (a) polyurethane foam and (b) silver have to be? •53 SSM Consider the slab shown in Fig. 18-18. Suppose that L # 25.0 cm, A # 90.0 cm2, and the material is copper. If TH # 125-C, TC # 10.0-C, and a steady state is reached, find the conduction rate through the slab. •54 If you were to walk briefly in space without a spacesuit while far from the Sun (as an astronaut does in the movie 2001, A Space Odyssey), you would feel the cold of space — while you radiated energy, you would absorb almost none from your environment. (a) At what rate would you lose energy? (b) How much energy would you lose in 30 s? Assume that your emissivity is 0.90, and estimate other data needed in the calculations. •55 ILW A cylindrical copper rod of length 1.2 m and cross-sectional area 4.8 cm2 is insulated along its side. The ends are held at a temperature difference of 100 C- by having one end in a water – ice mixture and the other in a mixture of boiling water and steam. At what rate (a) is energy conducted by the rod and (b) does the ice melt? ••56 The giant hornet Vespa mandarinia japonica preys on Japanese bees. However, if one of the hornets attempts to invade

Figure 18-44 Problem 56.

© Dr. Masato Ono, Tamagawa University

a beehive, several hundred of the bees quickly form a compact ball around the hornet to stop it. They don’t sting, bite, crush, or suffocate it. Rather they overheat it by quickly raising their body temperatures from the normal 35-C to 47-C or 48-C, which is lethal to the hornet but not to the bees (Fig. 18-44). Assume the following: 500 bees form a ball of radius R # 2.0 cm for a time t # 20 min, the primary loss of energy by the ball is by thermal radiation, the ball’s surface has emissivity ´ # 0.80, and the ball has a uniform temperature. On average, how much additional energy must each bee produce during the 20 min to maintain 47-C?

••58 A solid cylinder of radius r1 # 2.5 cm, length h1 # 5.0 cm, emissivity 0.85, and temperature 30-C is suspended in an environment of temperature 50-C. (a) What is the cylinder’s net thermal radiation transfer rate P1? (b) If the cylinder is stretched until its radius is r2 # 0.50 cm, its net thermal radiation transfer rate becomes P2. What is the ratio P2 /P1? ••59 In Fig. 18-45a, two identical rectangular rods of metal are welded end to end, with a temperature of T1 # 0-C on the left side and a temperature of T2 # 100-C on the right side. In 2.0 min, 10 J is conducted at a constant rate from the right side to the left side. How much time would be required to conduct 10 J if the rods were welded side to side as in Fig. 18-45b? ••60 Figure 18-46 shows the cross section of a wall made of three layers. The layer thicknesses are L1, L2 # 0.700L1, and L3 # 0.350L1. The thermal conductivities are k1, k2 # 0.900k1, and k3 # 0.800k1. The temperatures at the left side and right side of the wall are TH # 30.0-C and TC # $15.0-C, respectively. Thermal conduction is steady. (a) What is the temperature difference %T2 across layer 2 (between the left and right sides of the layer)? If k2 were, instead, equal to 1.1k1, (b) would the rate at which energy is conducted through the wall be greater than, less than, or the same as previously, and (c) what would be the value of %T2?

T2

T1 (a) T2

T1 (b)

Figure 18-45 Problem 59.

k1

k2

k3

TH

TC L1

L2 L3

Figure 18-46 Problem 60. Air Ice Water

••61 SSM A 5.0 cm slab has formed on an outdoor tank of water (Fig. 18-47). The air is at $10-C. Find the rate of ice formation (centimeters per hour). The ice has thermal conFigure 18-47 Problem 61. ductivity 0.0040 cal/s 9cm 9C- and 3 density 0.92 g/cm . Assume there is no energy transfer through the walls or bottom.

PROB LE M S

••62 Leidenfrost effect. A Water drop water drop will last about 1 s on a h hot skillet with a temperature L Skillet between 100-C and about 200-C. However, if the skillet is much hotFigure 18-48 Problem 62. ter, the drop can last several minutes, an effect named after an early investigator. The longer lifetime is due to the support of a thin layer of air and water vapor that separates the drop from the metal (by distance L in Fig. 18-48). Let L # 0.100 mm, and assume that the drop is flat with height h # 1.50 mm and bottom face area A # 4.00 ' 10$6 m2. Also assume that the skillet has a constant temperature Ts # 300-C and the drop has a temperature of 100-C. Water has density r # 1000 kg/m3, and the supporting layer has thermal conductivity k # 0.026 W/m9K. (a) At what rate is energy conducted from the skillet to the drop through the drop’s bottom surface? (b) If conduction is the primary way energy moves from the skillet to the drop, how long will the drop last? ••63 Figure 18-49 shows (in cross section) a wall consisting of four layers, with thermal conductivities k1 # 0.060 W/m 9K, k3 # 0.040 W/m 9K, and k4 # 0.12 W/m 9K (k2 is not known). The layer thicknesses are L1 # 1.5 cm, L3 # 2.8 cm, and L4 # 3.5 cm (L2 is not known). The known temperatures are T1 # 30-C, T12 # 25-C, and T4 # $10-C. Energy transfer through the wall is steady. What is interface temperature T34?

545

a huddled cylinder with top surface area Na and height h, the cylinder radiates at the rate Ph. If N # 1000, (a) what is the value of the fraction Ph/NPr and (b) by what percentage does huddling reduce the total radiation loss? ••65 Ice has formed on a shallow pond, and a steady state has been reached, with the air above the ice at $5.0-C and the bottom of the pond at 4.0-C. If the total depth of ice " water is 1.4 m, how thick is the ice? (Assume that the thermal conductivities of ice and water are 0.40 and 0.12 cal/m 9C-9s, respectively.) •••66 Evaporative cooling of beverages. A cold beverage can be kept cold even on a warm day if it is slipped into a porous ceramic container that has been soaked in water. Assume that energy lost to evaporation matches the net energy gained via the radiation exchange through the top and side surfaces. The container and beverage have temperature T # 15-C, the environment has temperature Tenv # 32-C, and the container is a cylinder with radius r # 2.2 cm and height 10 cm. Approximate the emissivity as ´ # 1, and neglect other energy exchanges. At what rate dm/dt is the container losing water mass?

Figure 18-49 Problem 63.

Additional Problems 67 In the extrusion of cold chocolate from a tube, work is done on the chocolate by the pressure applied by a ram forcing the chocolate through the tube. The work per unit mass of extruded chocolate is equal to p/r, where p is the difference between the applied pressure and the pressure where the chocolate emerges from the tube, and r is the density of the chocolate. Rather than increasing the temperature of the chocolate, this work melts cocoa fats in the chocolate. These fats have a heat of fusion of 150 kJ/kg. Assume that all of the work goes into that melting and that these fats make up 30% of the chocolate’s mass. What percentage of the fats melt during the extrusion if p # 5.5 MPa and r # 1200 kg/m3?

••64 Penguin huddling. To withstand the harsh weather of the Antarctic, emperor penguins huddle in groups (Fig. 18-50). Assume that a penguin is a circular cylinder with a top surface area a # 0.34 m2 and height h # 1.1 m. Let Pr be the rate at which an individual penguin radiates energy to the environment (through the top and the sides); thus NPr is the rate at which N identical, wellseparated penguins radiate. If the penguins huddle closely to form

68 Icebergs in the North Atlantic present hazards to shipping, causing the lengths of shipping routes to be increased by about 30% during the iceberg season. Attempts to destroy icebergs include planting explosives, bombing, torpedoing, shelling, ramming, and coating with black soot. Suppose that direct melting of the iceberg, by placing heat sources in the ice, is tried. How much energy as heat is required to melt 10% of an iceberg that has a mass of 200 000 metric tons? (Use 1 metric ton # 1000 kg.)

T1

k1

k2

T12

T23

L1

L2

k3

k4 T34

L3

T4

L4

69 Figure 18-51 displays a closed cycle for a gas. The change in internal energy along path ca is $160 J. The energy transferred to the gas as heat is 200 J along path ab, and 40 J along path bc. How much work is done by the gas along (a) path abc and (b) path ab?

p

c

a

b V

T (°C)

70 In a certain solar house, energy from the Sun is stored in barrels filled with water. In a particular winter stretch of five Ts cloudy days, 1.00 ' 10 6 kcal is needed to maintain the inside of the house at 22.0-C. Assuming that the water in the barrels is at 50.0-C and that the water has a density of 1.00 ' 10 3 kg/m3, what volume of water is required?

Figure 18-51 Problem 69.

Alain Torterotot/Peter Arnold/Photolibrary

Figure 18-50 Problem 64.

71 A 0.300 kg sample is placed in a cooling apparatus that removes energy as heat at a constant rate of 2.81 W. Figure 18-52 gives the temperature T of the sam-

0 t (min)

ts

Figure 18-52 Problem 71.

CHAPTE R 18 TE M PE RATU R E, H EAT, AN D TH E FI RST L AW OF TH E R M ODYNAM ICS

ple versus time t. The temperature scale is set by Ts # 30-C and the time scale is set by ts # 20 min.What is the specific heat of the sample? 72 The average rate at which energy is conducted outward through the ground surface in North America is 54.0 mW/m2, and the average thermal conductivity of the near-surface rocks is 2.50 W/m 9K. Assuming a surface temperature of 10.0-C, find the temperature at a depth of 35.0 km (near the base of the crust). Ignore the heat generated by the presence of radioactive elements. 73 What is the volume increase of an aluminum cube 5.00 cm on an edge when heated from 10.0-C to 60.0-C?

Tf (K)

74 In a series of experiments, Tfs block B is to be placed in a thermally insulated container with block A, which has the same mass as block B. In each experiment, block B is initially at a certain temperature TB, but temperature TA of 0 block A is changed from experiTA1 TA2 ment to experiment. Let Tf repreTA (K) sent the final temperature of the two blocks when they reach thermal Figure 18-53 Problem 74. equilibrium in any of the experiments. Figure 18-53 gives temperature Tf versus the initial temperature TA for a range of possible values of TA, from TA1 # 0 K to TA2 # 500 K. The vertical axis scale is set by Tfs # 400 K. What are (a) temperature TB and (b) the ratio cB/cA of the specific heats of the blocks? 75 Figure 18-54 displays a closed p c cycle for a gas. From c to b, 40 J is transferred from the gas as heat. From b to a, 130 J is transferred from the gas as heat, and the magnitude of the work done by the gas a b is 80 J. From a to c, 400 J is transferred to the gas as heat. What is V the work done by the gas from a to Figure 18-54 Problem 75. c? (Hint: You need to supply the plus and minus signs for the given data.) 76 Three equal-length straight rods, of aluminum, Invar, and steel, all at 20.0-C, form an equilateral triangle with hinge pins at the vertices. At what temperature will the angle opposite the Invar rod be 59.95-? See Appendix E for needed trigonometric formulas and Table 18-2 for needed data. 77 SSM The temperature of a 0.700 kg cube of ice is decreased to $150-C. Then energy is gradually transferred to the cube as heat while it is otherwise thermally isolated from its environment. The Tr total transfer is 0.6993 MJ. Assume the value of cice given in Table 18-3 Energy transfer is valid for temperatures from $150-C to 0-C. What is the final temperature of the water? L 78 Icicles. Liquid water coats an active (growing) icicle and extends up a short, narrow tube along the central axis (Fig. 18-55). Because the water–ice interface must have a temperature of 0-C, the water in the tube cannot lose energy through the

Liquid coating (0°C) Liquid water (0°C)

Figure 18-55 Problem 78.

sides of the icicle or down through the tip because there is no temperature change in those directions. It can lose energy and freeze only by sending energy up (through distance L) to the top of the icicle, where the temperature Tr can be below 0-C. Take L # 0.12 m and Tr # $5-C. Assume that the central tube and the upward conduction path both have cross-sectional area A. In terms of A, what rate is (a) energy conducted upward and (b) mass converted from liquid to ice at the top of the central tube? (c) At what rate does the top of the tube move downward because of water freezing there? The thermal conductivity of ice is 0.400 W/m9K, and the density of liquid water is 1000 kg/m3. 79 SSM A sample of gas expands from an initial pressure and volume of 10 Pa and 1.0 m3 to a final volume of 2.0 m3. During the expansion, the pressure and volume are related by the equation p # aV 2, where a # 10 N/m8. Determine the work done by the gas during this expansion. 80 Figure 18-56a shows a cylinder containing gas and closed by a movable piston.The cylinder is kept submerged in an ice – water mixture. The piston is quickly pushed down from position 1 to position 2 and then held at position 2 until the gas is again at the temperature of the ice – water mixture; it then is slowly raised back to position 1. Figure 18-56b is a p-V diagram for the process. If 100 g of ice is melted during the cycle, how much work has been done on the gas?

1

2

Pressure

546

Start

Ice and water V2

V1 Volume (b)

(a)

Figure 18-56 Problem 80. 81 SSM A sample of gas underp goes a transition from an initial state a to a final state b by three different 3pi/2 paths (processes), as shown in the p2 V diagram in Fig. 18-57, where Vb # a 1 pi b 5.00Vi. The energy transferred to 3 the gas as heat in process 1 is 10piVi. pi/2 In terms of piVi, what are (a) the energy transferred to the gas as heat in V process 2 and (b) the change in inVi Vb ternal energy that the gas undergoes Figure 18-57 Problem 81. in process 3? 82 A copper rod, an aluminum rod, and a brass rod, each of 6.00 m length and 1.00 cm diameter, are placed end to end with the aluminum rod between the other two. The free end of the copper rod is maintained at water’s boiling point, and the free end of the brass rod is maintained at water’s freezing point. What is the steady-state temperature of (a) the copper – aluminum junction and (b) the aluminum – brass junction? 83 SSM The temperature of a Pyrex disk is changed from 10.0-C to 60.0-C. Its initial radius is 8.00 cm; its initial thickness is 0.500 cm. Take these data as being exact. What is the change in the volume of the disk? (See Table 18-2.)

547

PROB LE M S

84 (a) Calculate the rate at which body heat is conducted through the clothing of a skier in a steady-state process, given the following data: the body surface area is 1.8 m2, and the clothing is 1.0 cm thick; the skin surface temperature is 33-C and the outer surface of the clothing is at 1.0-C; the thermal conductivity of the clothing is 0.040 W/m 9K. (b) If, after a fall, the skier’s clothes became soaked with water of thermal conductivity 0.60 W/m 9K, by how much is the rate of conduction multiplied? 85 SSM A 2.50 kg lump of aluminum is heated to 92.0-C and then dropped into 8.00 kg of water at 5.00-C. Assuming that the lump – water system is thermally isolated, what is the system’s equilibrium temperature? 86 A glass window pane is exactly 20 cm by 30 cm at 10-C. By how much has its area increased when its temperature is 40-C, assuming that it can expand freely? 87 A recruit can join the semi-secret “300 F” club at the Amundsen – Scott South Pole Station only when the outside temperature is below $70-C. On such a day, the recruit first basks in a hot sauna and then runs outside wearing only shoes. (This is, of course, extremely dangerous, but the rite is effectively a protest against the constant danger of the cold.) Assume that upon stepping out of the sauna, the recruit’s skin temperature is 102-F and the walls, ceiling, and floor of the sauna room have a temperature of 30-C. Estimate the recruit’s surface area, and take the skin emissivity to be 0.80. (a) What is the approximate net rate Pnet at which the recruit loses energy via thermal radiation exchanges with the room? Next, assume that when outdoors, half the recruit’s surface area exchanges thermal radiation with the sky at a temperature of $25-C and the other half exchanges thermal radiation with the snow and ground at a temperature of $80-C. What is the approximate net rate at which the recruit loses energy via thermal radiation exchanges with (b) the sky and (c) the snow and ground? 88 A steel rod at 25.0-C is bolted at both ends and then cooled. At what temperature will it rupture? Use Table 12-1. 89 An athlete needs to lose weight and decides to do it by “pumping iron.” (a) How many times must an 80.0 kg weight be lifted a distance of 1.00 m in order to burn off 1.00 lb of fat, assuming that that much fat is equivalent to 3500 Cal? (b) If the weight is lifted once every 2.00 s, how long does the task take? 90 Soon after Earth was formed, heat released by the decay of radioactive elements raised the average internal temperature from 300 to 3000 K, at about which value it remains today. Assuming an average coefficient of volume expansion of 3.0 ' 10 $5 K$1, by how much has the radius of Earth increased since the planet was formed? 91 It is possible to melt ice by rubbing one block of it against another. How much work, in joules, would you have to do to get 1.00 g of ice to melt? 92 A rectangular plate of glass initially has the dimensions 0.200 m by 0.300 m. The coefficient of linear expansion for the glass is 9.00 ' 10 $6/K. What is the change in the plate’s area if its temperature is increased by 20.0 K? 93 Suppose that you intercept 5.0 ' 10 $3 of the energy radiated by a hot sphere that has a radius of 0.020 m, an emissivity of 0.80, and a surface temperature of 500 K. How much energy do you intercept in 2.0 min? 94 A thermometer of mass 0.0550 kg and of specific heat 0.837 kJ/kg 9K reads 15.0-C. It is then completely immersed in

0.300 kg of water, and it comes to the same final temperature as the water. If the thermometer then reads 44.4-C, what was the temperature of the water before insertion of the thermometer? 95 A sample of gas expands from V1 # 1.0 m3 and p1 # 40 Pa to V2 # 4.0 m3 and p2 # 10 Pa along path B in the p-V diagram in Fig. 18-58. It is then compressed back to V1 along either path A or path C. Compute the net work done by the gas for the complete cycle along (a) path BA and (b) path BC.

p A

p1

B C

p2 0

V1

V2

V

96 Figure 18-59 shows a composite Figure 18-58 Problem 95. bar of length L # L1 " L2 and consisting of two materials. One material has length L1 and coeffiL2 L1 cient of linear expansion a1; the other has length L2 and coefficient of linear expanL sion a2. (a) What is the coeffiFigure 18-59 Problem 96. cient of linear expansion a for the composite bar? For a particular composite bar, L is 52.4 cm, material 1 is steel, and material 2 is brass. If a # 1.3 ' 10$5/C-, what are the lengths (b) L1 and (c) L2? 97 On finding your stove out of order, you decide to boil the water for a cup of tea by shaking it in a thermos flask. Suppose that you use tap water at 19-C, the water falls 32 cm each shake, and you make 27 shakes each minute. Neglecting any loss of thermal energy by the flask, how long (in minutes) must you shake the flask until the water reaches 100-C? 98 The p-V diagram in Fig. 18-60 shows two paths along which a sample of gas can be taken from state a to state b, where Vb # 3.0V1. Path 1 requires that energy equal to 5.0p1V1 be transferred to the gas as heat. Path 2 requires that energy equal to 5.5p1V1 be transferred to the gas as heat. What is the ratio p2/p1?

p

p2

p1

2

a V1

1

b Vb

V

99 A cube of edge length 6.0 ' 10$6 m, Figure 18-60 Problem 98. emissivity 0.75, and temperature $100-C floats in an environment at $150-C.What is the cube’s net thermal radiation transfer rate? 100 A flow calorimeter is a device used to measure the specific heat of a liquid. Energy is added as heat at a known rate to a stream of the liquid as it passes through the calorimeter at a known rate. Measurement of the resulting temperature difference between the inflow and the outflow points of the liquid stream enables us to compute the specific heat of the liquid. Suppose a liquid of density 0.85 g/cm3 flows through a calorimeter at the rate of 8.0 cm3/s. When energy is added at the rate of 250 W by means of an electric heating coil, a temperature difference of 15 C- is established in steady-state conditions between the inflow and the outflow points. What is the specific heat of the liquid? 101 An object of mass 6.00 kg falls through a height of 50.0 m and, by means of a mechanical linkage, rotates a paddle wheel that stirs 0.600 kg of water. Assume that the initial gravitational potential energy of the object is fully transferred to thermal energy of the water, which is initially at 15.0-C. What is the temperature rise of the water?

548

CHAPTE R 18 TE M PE RATU R E, H EAT, AN D TH E FI RST L AW OF TH E R M ODYNAM ICS

102 The Pyrex glass mirror in a telescope has a diameter of 170 in. The temperature ranges from $16-C to 32-C on the location of the telescope. What is the maximum change in the diameter of the mirror, assuming that the glass can freely expand and contract? 103 The area A of a rectangular plate is ab # 1.4 m2. Its coefficient of linear expansion is a # 32 ' 10$6/C-. After a temperature rise %T # 89-C, side a is longer by %a and side b is longer by %b (Fig. 18-61). Neglecting the small quantity (%a %b)/ab, find %A.

a

%a b

%a %b %b

Figure 18-61 Problem 103.

104 Consider the liquid in a barometer whose coefficient of volume expansion is 6.6 ' 10$4/C-. Find the relative change in the liquid’s height if the temperature changes by 12 C- while the pressure remains constant. Neglect the expansion of the glass tube. 105 A pendulum clock with a pendulum made of brass is designed to keep accurate time at 23-C. Assume it is a simple pendulum consisting of a bob at one end of a brass rod of negligible mass that is pivoted about the other end. If the clock operates at 0.0-C, (a) does it run too fast or too slow, and (b) what is the magnitude of its error in seconds per hour? 106 A room is lighted by four 100 W incandescent lightbulbs. (The power of 100 W is the rate at which a bulb converts electrical energy to heat and the energy of visible light.) Assuming that 73% of the energy is converted to heat, how much heat does the room receive in 6.9 h? 107 An energetic athlete can use up all the energy from a diet of 4000 Cal/day. If he were to use up this energy at a steady rate, what is the ratio of the rate of energy use compared to that of a 100 W bulb? (The power of 100 W is the rate at which the bulb converts electrical energy to heat and the energy of visible light.) 108 A 1700 kg Buick moving at 83 km/h brakes to a stop, at uniform deceleration and without skidding, over a distance of 93 m. At what average rate is mechanical energy transferred to thermal energy in the brake system?

C

H

A

P

T

E

R

1

9

The Kinetic Theory of Gases 19-1 AVOGADRO’S NUMBER Learning Objectives After reading this module, you should be able to . . .

19.01 Identify Avogadro’s number NA. 19.02 Apply the relationship between the number of moles n, the number of molecules N, and Avogadro’s number NA.

19.03 Apply the relationships between the mass m of a sample, the molar mass M of the molecules in the sample, the number of moles n in the sample, and Avogadro’s number NA.

Key Ideas ● The kinetic theory of gases relates the macroscopic properties of gases (for example, pressure and temperature) to the microscopic properties of gas molecules (for example, speed and kinetic energy). ● One mole of a substance contains NA (Avogadro’s

number) elementary units (usually atoms or molecules), where NA is found experimentally to be NA # 6.02 ' 10 23 mol$1 (Avogadro’s number).

One molar mass M of any substance is the mass of one mole of the substance.

● A mole is related to the mass m of the individual molecules

of the substance by M # mNA. ● The

number of moles n contained in a sample of mass Msam, consisting of N molecules, is related to the molar mass M of the molecules and to Avogadro’s number NA as given by n#

N M Msam # sam # . NA M mN A

What Is Physics? One of the main subjects in thermodynamics is the physics of gases. A gas consists of atoms (either individually or bound together as molecules) that fill their container’s volume and exert pressure on the container’s walls. We can usually assign a temperature to such a contained gas. These three variables associated with a gas—volume, pressure, and temperature—are all a consequence of the motion of the atoms. The volume is a result of the freedom the atoms have to spread throughout the container, the pressure is a result of the collisions of the atoms with the container’s walls, and the temperature has to do with the kinetic energy of the atoms. The kinetic theory of gases, the focus of this chapter, relates the motion of the atoms to the volume, pressure, and temperature of the gas. Applications of the kinetic theory of gases are countless. Automobile engineers are concerned with the combustion of vaporized fuel (a gas) in the automobile engines. Food engineers are concerned with the production rate of the fermentation gas that causes bread to rise as it bakes. Beverage engineers are concerned with how gas can produce the head in a glass of beer or shoot a cork from a champagne bottle. Medical engineers and physiologists are concerned with calculating how long a scuba diver must pause during ascent to eliminate nitrogen gas from the bloodstream (to avoid the bends). Environmental scientists are concerned with how heat exchanges between the oceans and the atmosphere can affect weather conditions. The first step in our discussion of the kinetic theory of gases deals with measuring the amount of a gas present in a sample, for which we use Avogadro’s number. 549

550

CHAPTE R 19 TH E KI N ETIC TH EORY OF GASES

Avogadro’s Number When our thinking is slanted toward atoms and molecules, it makes sense to measure the sizes of our samples in moles. If we do so, we can be certain that we are comparing samples that contain the same number of atoms or molecules. The mole is one of the seven SI base units and is defined as follows: One mole is the number of atoms in a 12 g sample of carbon-12.

The obvious question now is: “How many atoms or molecules are there in a mole?” The answer is determined experimentally and, as you saw in Chapter 18, is NA # 6.02 ' 10 23 mol$1

(Avogadro’s number),

(19-1)

where mol$1 represents the inverse mole or “per mole,” and mol is the abbreviation for mole. The number NA is called Avogadro’s number after Italian scientist Amedeo Avogadro (1776 – 1856), who suggested that all gases occupying the same volume under the same conditions of temperature and pressure contain the same number of atoms or molecules. The number of moles n contained in a sample of any substance is equal to the ratio of the number of molecules N in the sample to the number of molecules NA in 1 mol: n#

N . NA

(19-2)

(Caution: The three symbols in this equation can easily be confused with one another, so you should sort them with their meanings now, before you end in “N-confusion.”) We can find the number of moles n in a sample from the mass Msam of the sample and either the molar mass M (the mass of 1 mol) or the molecular mass m (the mass of one molecule): n#

M sam M sam # . M mN A

(19-3)

In Eq. 19-3, we used the fact that the mass M of 1 mol is the product of the mass m of one molecule and the number of molecules NA in 1 mol: M # mNA.

(19-4)

19-2 IDEAL GASES Learning Objectives After reading this module, you should be able to . . .

19.04 Identify why an ideal gas is said to be ideal. 19.05 Apply either of the two forms of the ideal gas law, written in terms of the number of moles n or the number of molecules N. 19.06 Relate the ideal gas constant R and the Boltzmann constant k. 19.07 Identify that the temperature in the ideal gas law must be in kelvins. 19.08 Sketch p-V diagrams for a constant-temperature expansion of a gas and a constant-temperature contraction. 19.09 Identify the term isotherm.

19.10 Calculate the work done by a gas, including the algebraic sign, for an expansion and a contraction along an isotherm. 19.11 For an isothermal process, identify that the change in internal energy %E is zero and that the energy Q transferred as heat is equal to the work W done. 19.12 On a p-V diagram, sketch a constant-volume process and identify the amount of work done in terms of area on the diagram. 19.13 On a p-V diagram, sketch a constant-pressure process and determine the work done in terms of area on the diagram.

19-2 I DEAL GASES

551

Key Ideas ● An ideal gas is one for which the pressure p, volume V, and temperature T are related by pV # nRT (ideal gas law).

Here n is the number of moles of the gas present and R is a constant (8.31 J/mol 9K) called the gas constant. ● The ideal gas law can also be written as

where the Boltzmann constant k is R k# # 1.38 ' 10$23 J/K. NA ● The work done by an ideal gas during an isothermal (constant-temperature) change from volume Vi to volume Vf is W # nRT ln

pV # NkT,

Ideal Gases Our goal in this chapter is to explain the macroscopic properties of a gas — such as its pressure and its temperature — in terms of the behavior of the molecules that make it up. However, there is an immediate problem: which gas? Should it be hydrogen, oxygen, or methane, or perhaps uranium hexafluoride? They are all different. Experimenters have found, though, that if we confine 1 mol samples of various gases in boxes of identical volume and hold the gases at the same temperature, then their measured pressures are almost the same, and at lower densities the differences tend to disappear. Further experiments show that, at low enough densities, all real gases tend to obey the relation pV # nRT

(ideal gas law),

(19-5)

in which p is the absolute (not gauge) pressure, n is the number of moles of gas present, and T is the temperature in kelvins. The symbol R is a constant called the gas constant that has the same value for all gases — namely, R # 8.31 J/mol 9K.

(19-6)

Equation 19-5 is called the ideal gas law. Provided the gas density is low, this law holds for any single gas or for any mixture of different gases. (For a mixture, n is the total number of moles in the mixture.) We can rewrite Eq. 19-5 in an alternative form, in terms of a constant called the Boltzmann constant k, which is defined as k#

R 8.31 J/mol9K # # 1.38 ' 10 $23 J/K. NA 6.02 ' 10 23 mol $1

(19-7)

This allows us to write R # kNA . Then, with Eq. 19-2 (n # N/NA), we see that nR # Nk.

(19-8)

Substituting this into Eq. 19-5 gives a second expression for the ideal gas law: pV # NkT

(ideal gas law).

(19-9)

(Caution: Note the difference between the two expressions for the ideal gas law — Eq. 19-5 involves the number of moles n, and Eq. 19-9 involves the number of molecules N.) You may well ask, “What is an ideal gas, and what is so ‘ideal’ about it?” The answer lies in the simplicity of the law (Eqs. 19-5 and 19-9) that governs its macroscopic properties. Using this law — as you will see — we can deduce many properties of the ideal gas in a simple way. Although there is no such thing in nature as a truly ideal gas, all real gases approach the ideal state at low enough densities — that is, under conditions in which their molecules are far enough apart that they do not interact with one another. Thus, the ideal gas concept allows us to gain useful insights into the limiting behavior of real gases.

Vf Vi

(ideal gas, isothermal process).

552

CHAPTE R 19 TH E KI N ETIC TH EORY OF GASES

Figure 19-1 gives a dramatic example of the ideal gas law. A stainless-steel tank with a volume of 18 m3 was filled with steam at a temperature of 110-C through a valve at one end. The steam supply was then turned off and the valve closed, so that the steam was trapped inside the tank (Fig. 19-1a). Water from a fire hose was then poured onto the tank to rapidly cool it. Within less than a minute, the enormously sturdy tank was crushed (Fig. 19-1b), as if some giant invisible creature from a grade B science fiction movie had stepped on it during a rampage. Actually, it was the atmosphere that crushed the tank. As the tank was cooled by the water steam, the steam cooled and much of it condensed, which means that the number N of gas molecules and the temperature T of the gas inside the tank both decreased. Thus, the right side of Eq. 19-9 decreased, and because volume V was constant, the gas pressure p on the left side also decreased. The gas pressure decreased so much that the external atmospheric pressure was able to crush the tank’s steel wall. Figure 19-1 was staged, but this type of crushing sometimes occurs in industrial accidents (photos and videos can be found on the web).

(a)

Work Done by an Ideal Gas at Constant Temperature (b) Courtesy www.doctorslime.com

Figure 19-1 (a) Before and (b) after images of a large steel tank crushed by atmospheric pressure after internal steam cooled and condensed.

Suppose we put an ideal gas in a piston – cylinder arrangement like those in Chapter 18. Suppose also that we allow the gas to expand from an initial volume Vi to a final volume Vf while we keep the temperature T of the gas constant. Such a process, at constant temperature, is called an isothermal expansion (and the reverse is called an isothermal compression). On a p-V diagram, an isotherm is a curve that connects points that have the same temperature. Thus, it is a graph of pressure versus volume for a gas whose temperature T is held constant. For n moles of an ideal gas, it is a graph of the equation 1 1 (19-10) # (a constant) . V V Figure 19-2 shows three isotherms, each corresponding to a different (constant) value of T. (Note that the values of T for the isotherms increase upward to the right.) Superimposed on the middle isotherm is the path followed by a gas during an isothermal expansion from state i to state f at a constant temperature of 310 K. To find the work done by an ideal gas during an isothermal expansion, we start with Eq. 18-25, p # nRT

p

The expansion is along an isotherm (the gas has constant temperature).

"

Vf

W#

(19-11)

p dV.

Vi

This is a general expression for the work done during any change in volume of any gas. For an ideal gas, we can use Eq. 19-5 ( pV # nRT) to substitute for p, obtaining

i

W# T = 320 K f T = 310 K T = 300 K V

Figure 19-2 Three isotherms on a p-V diagram. The path shown along the middle isotherm represents an isothermal expansion of a gas from an initial state i to a final state f. The path from f to i along the isotherm would represent the reverse process — that is, an isothermal compression.

"

Vf

Vi

nRT dV. V

(19-12)

Because we are considering an isothermal expansion, T is constant, so we can move it in front of the integral sign to write W # nRT

"

Vf

Vi

( )

dV # nRT ln V V

Vf

.

(19-13)

Vi

By evaluating the expression in brackets at the limits and then using the relationship ln a $ ln b # ln(a/b), we find that W # nRT ln

Vf Vi

(ideal gas, isothermal process).

Recall that the symbol ln specifies a natural logarithm, which has base e.

(19-14)

19-2 I DEAL GASES

553

For an expansion, Vf is greater than Vi, so the ratio Vf /Vi in Eq. 19-14 is greater than unity. The natural logarithm of a quantity greater than unity is positive, and so the work W done by an ideal gas during an isothermal expansion is positive, as we expect. For a compression, Vf is less than Vi, so the ratio of volumes in Eq. 19-14 is less than unity. The natural logarithm in that equation — hence the work W — is negative, again as we expect.

Work Done at Constant Volume and at Constant Pressure Equation 19-14 does not give the work W done by an ideal gas during every thermodynamic process. Instead, it gives the work only for a process in which the temperature is held constant. If the temperature varies, then the symbol T in Eq. 19-12 cannot be moved in front of the integral symbol as in Eq. 19-13, and thus we do not end up with Eq. 19-14. However, we can always go back to Eq. 19-11 to find the work W done by an ideal gas (or any other gas) during any process, such as a constant-volume process and a constant-pressure process. If the volume of the gas is constant, then Eq. 19-11 yields W#0

(19-15)

(constant-volume process).

If, instead, the volume changes while the pressure p of the gas is held constant, then Eq. 19-11 becomes W # p(Vf $ Vi ) # p %V

(19-16)

(constant-pressure process).

Checkpoint 1 An ideal gas has an initial pressure of 3 pressure units and an initial volume of 4 volume units. The table gives the final pressure and volume of the gas (in those same units) in five processes. Which processes start and end on the same isotherm?

p V

a 12 1

b 6 2

c 5 7

d e 4 1 3 12

Sample Problem 19.01 Ideal gas and changes of temperature, volume, and pressure A cylinder contains 12 L of oxygen at 20-C and 15 atm. The temperature is raised to 35-C, and the volume is reduced to 8.5 L. What is the final pressure of the gas in atmospheres? Assume that the gas is ideal. KEY IDEA Because the gas is ideal, we can use the ideal gas law to relate its parameters, both in the initial state i and in the final state f.

Note here that if we converted the given initial and final volumes from liters to the proper units of cubic meters, the multiplying conversion factors would cancel out of Eq. 19-17. The same would be true for conversion factors that convert the pressures from atmospheres to the proper pascals. However, to convert the given temperatures to kelvins requires the addition of an amount that would not cancel and thus must be included. Hence, we must write

Calculations: From Eq. 19-5 we can write piVi # nRTi

and

pfVf # nRTf .

Dividing the second equation by the first equation and solving for pf yields piTf Vi . pf # (19-17) Ti Vf

Ti # (273 " 20) K # 293 K and

Tf # (273 " 35) K # 308 K.

Inserting the given data into Eq. 19-17 then yields pf #

(15 atm)(308 K)(12 L) # 22 atm. (293 K)(8.5 L)

Additional examples, video, and practice available at WileyPLUS

(Answer)

554

CHAPTE R 19 TH E KI N ETIC TH EORY OF GASES

Sample Problem 19.02 Work by an ideal gas One mole of oxygen (assume it to be an ideal gas) expands at a constant temperature T of 310 K from an initial volume Vi of 12 L to a final volume Vf of 19 L. How much work is done by the gas during the expansion? KEY IDEA

You can show that if the expansion is now reversed, with the gas undergoing an isothermal compression from 19 L to 12 L, the work done by the gas will be $1180 J. Thus, an external force would have to do 1180 J of work on the gas to compress it.

Generally we find the work by integrating the gas pressure with respect to the gas volume, using Eq. 19-11. However, because the gas here is ideal and the expansion is isothermal, that integration leads to Eq. 19-14.

3.0

Calculation: Therefore, we can write Vf W # nRT ln Vi

2.0

# 1180 J.

19 L 12 L (Answer)

The expansion is graphed in the p-V diagram of Fig. 19-3. The work done by the gas during the expansion is represented by the area beneath the curve if.

Figure 19-3 The shaded area represents the work done by 1 mol of oxygen in expanding from Vi to Vf at a temperature T of 310 K.

Pressure (atm)

# (1 mol)(8.31 J/mol9K)(310 K) ln

i

f 1.0

W Vi

0

T = 310 K

Vf 10 20 Volume (L)

30

Additional examples, video, and practice available at WileyPLUS

19-3 PRESSURE, TEMPERATURE, AND RMS SPEED Learning Objectives After reading this module, you should be able to . . .

19.14 Identify that the pressure on the interior walls of a gas container is due to the molecular collisions with the walls. 19.15 Relate the pressure on a container wall to the momentum of the gas molecules and the time intervals between their collisions with the wall. 19.16 For the molecules of an ideal gas, relate the root-

mean-square speed vrms and the average speed vavg. 19.17 Relate the pressure of an ideal gas to the rms speed vrms of the molecules. 19.18 For an ideal gas, apply the relationship between the gas temperature T and the rms speed vrms and molar mass M of the molecules.

Key Ideas ● In terms of the speed of the gas molecules, the pressure exerted by n moles of an ideal gas is

nMv 2rms , 3V # 2(v2)avg is the root-mean-square speed of the p#

where vrms

molecules, M is the molar mass, and V is the volume. ● The rms speed can be written in terms of the temperature as 3RT vrms # . A M

Pressure, Temperature, and RMS Speed Here is our first kinetic theory problem. Let n moles of an ideal gas be confined in a cubical box of volume V, as in Fig. 19-4. The walls of the box are held at temperature T. What is the connection between the pressure p exerted by the gas on the walls and the speeds of the molecules?

19-3 PR ESSU R E, TE M PE RATU R E, AN D R M S SPE E D

The molecules of gas in the box are moving in all directions and with various speeds, bumping into one another and bouncing from the walls of the box like balls in a racquetball court. We ignore (for the time being) collisions of the molecules with one another and consider only elastic collisions with the walls. Figure 19-4 shows a typical gas molecule, of mass m and velocity : v , that is about to collide with the shaded wall. Because we assume that any collision of a molecule with a wall is elastic, when this molecule collides with the shaded wall, the only component of its velocity that is changed is the x component, and that component is reversed. This means that the only change in the particle’s momentum is along the x axis, and that change is %px # ($mvx) $ (mvx) # $2mvx . Hence, the momentum %px delivered to the wall by the molecule during the collision is "2mvx . (Because in this book the symbol p represents both momentum and pressure, we must be careful to note that here p represents momentum and is a vector quantity.) The molecule of Fig. 19-4 will hit the shaded wall repeatedly. The time %t between collisions is the time the molecule takes to travel to the opposite wall and back again (a distance 2L) at speed vx . Thus, %t is equal to 2L/vx . (Note that this result holds even if the molecule bounces off any of the other walls along the way, because those walls are parallel to x and so cannot change vx .) Therefore, the average rate at which momentum is delivered to the shaded wall by this single molecule is %px 2mvx mv 2x . # # %t 2L/vx L :

: From Newton’s second law (F # dp /dt), the rate at which momentum is delivered to the wall is the force acting on that wall. To find the total force, we must add up the contributions of all the molecules that strike the wall, allowing for the possibility that they all have different speeds. Dividing the magnitude of the total force Fx by the area of the wall (# L2) then gives the pressure p on that wall, where now and in the rest of this discussion, p represents pressure. Thus, using the expression for %px /%t, we can write this pressure as

p# #

Fx mv 2x1/L " mv 2x2 /L " 9 9 9 " mv 2xN /L # L2 L2

# Lm $(v 3

2 x1

" v 2x2 " 9 9 9 " v 2xN),

(19-18)

where N is the number of molecules in the box. Since N # nNA, there are nNA terms in the second set of parentheses of Eq. 19-18. We can replace that quantity by nNA(v 2x)avg, where (v 2x)avg is the average value of the square of the x components of all the molecular speeds. Equation 19-18 then becomes p#

nmNA 2 (vx)avg . L3

However, mNA is the molar mass M of the gas (that is, the mass of 1 mol of the gas). Also, L3 is the volume of the box, so nM(v2x)avg (19-19) p# . V For any molecule, v 2 # v 2x " v 2y " v 2z. Because there are many molecules and because they are all moving in random directions, the average values of the squares of their velocity components are equal, so that v 2x # 13 v2. Thus, Eq. 19-19 becomes nM(v2)avg p# (19-20) . 3V

555

y

Normal to shaded wall

v

m L

x L

z

L

Figure 19-4 A cubical box of edge length L, containing n moles of an ideal gas. A molecule of mass m and velocity : v is about to collide with the shaded wall of area L2. A normal to that wall is shown.

556

CHAPTE R 19 TH E KI N ETIC TH EORY OF GASES

Table 19-1 Some RMS Speeds at Room Temperature (T # 300 K)a Molar Mass (10$3 kg/mol)

Gas Hydrogen (H2) Helium (He) Water vapor (H2O) Nitrogen (N2) Oxygen (O2) Carbon dioxide (CO2) Sulfur dioxide (SO2)

vrms (m/s)

2.02 4.0

1920 1370

18.0 28.0 32.0

645 517 483

44.0

412

64.1

342

a

For convenience, we often set room temperature equal to 300 K even though (at 27-C or 81-F) that represents a fairly warm room.

The square root of (v2)avg is a kind of average speed, called the root-meansquare speed of the molecules and symbolized by vrms. Its name describes it rather well: You square each speed, you find the mean (that is, the average) of all these squared speeds, and then you take the square root of that mean. With 2(v2)avg # vrms , we can then write Eq. 19-20 as p#

nMv 2rms . 3V

(19-21)

This tells us how the pressure of the gas (a purely macroscopic quantity) depends on the speed of the molecules (a purely microscopic quantity). We can turn Eq. 19-21 around and use it to calculate vrms. Combining Eq. 19-21 with the ideal gas law ( pV # nRT ) leads to vrms #

3RT . A M

(19-22)

Table 19-1 shows some rms speeds calculated from Eq. 19-22. The speeds are surprisingly high. For hydrogen molecules at room temperature (300 K), the rms speed is 1920 m/s, or 4300 mi/h — faster than a speeding bullet! On the surface of the Sun, where the temperature is 2 ' 106 K, the rms speed of hydrogen molecules would be 82 times greater than at room temperature were it not for the fact that at such high speeds, the molecules cannot survive collisions among themselves. Remember too that the rms speed is only a kind of average speed; many molecules move much faster than this, and some much slower. The speed of sound in a gas is closely related to the rms speed of the molecules of that gas. In a sound wave, the disturbance is passed on from molecule to molecule by means of collisions. The wave cannot move any faster than the “average” speed of the molecules. In fact, the speed of sound must be somewhat less than this “average” molecular speed because not all molecules are moving in exactly the same direction as the wave. As examples, at room temperature, the rms speeds of hydrogen and nitrogen molecules are 1920 m/s and 517 m/s, respectively. The speeds of sound in these two gases at this temperature are 1350 m/s and 350 m/s, respectively. A question often arises: If molecules move so fast, why does it take as long as a minute or so before you can smell perfume when someone opens a bottle across a room? The answer is that, as we shall discuss in Module 19-5, each perfume molecule may have a high speed but it moves away from the bottle only very slowly because its repeated collisions with other molecules prevent it from moving directly across the room to you.

Sample Problem 19.03 Average and rms values Calculation: We find this from

Here are five numbers: 5, 11, 32, 67, and 89. (a) What is the average value navg of these numbers? Calculation: We find this from navg

5 " 11 " 32 " 67 " 89 # # 40.8. 5

(b) What is the rms value nrms of these numbers?

(Answer)

52 " 112 " 32 2 " 672 " 892 A 5 # 52.1.

nrms #

(Answer)

The rms value is greater than the average value because the larger numbers — being squared — are relatively more important in forming the rms value.

Additional examples, video, and practice available at WileyPLUS

19-4 TRANSL ATIONAL KI N ETIC E N E RGY

557

19-4 TRANSLATIONAL KINETIC ENERGY Learning Objectives After reading this module, you should be able to . . .

19.19 For an ideal gas, relate the average kinetic energy of the molecules to their rms speed. 19.20 Apply the relationship between the average kinetic energy and the temperature of the gas.

19.21 Identify that a measurement of a gas temperature is effectively a measurement of the average kinetic energy of the gas molecules.

Key Ideas ● The average translational kinetic energy per molecule in an ideal gas is Kavg # 12mv2rms.

● The average translational kinetic energy is related to the

temperature of the gas: Kavg # 32kT.

Translational Kinetic Energy We again consider a single molecule of an ideal gas as it moves around in the box of Fig. 19-4, but we now assume that its speed changes when it collides with other molecules. Its translational kinetic energy at any instant is 12 mv 2. Its average translational kinetic energy over the time that we watch it is Kavg # (12 mv 2)avg # 12 m(v2)avg # 12 mv 2rms ,

(19-23)

in which we make the assumption that the average speed of the molecule during our observation is the same as the average speed of all the molecules at any given time. (Provided the total energy of the gas is not changing and provided we observe our molecule for long enough, this assumption is appropriate.) Substituting for vrms from Eq. 19-22 leads to Kavg # (12 m)

3RT . M

However, M/m, the molar mass divided by the mass of a molecule, is simply Avogadro’s number. Thus, 3RT . 2NA Using Eq. 19-7 (k # R/NA), we can then write Kavg #

Kavg # 32kT.

(19-24)

This equation tells us something unexpected: At a given temperature T, all ideal gas molecules—no matter what their mass—have the same average translational kinetic energy—namely, 32kT. When we measure the temperature of a gas, we are also measuring the average translational kinetic energy of its molecules.

Checkpoint 2 A gas mixture consists of molecules of types 1, 2, and 3, with molecular masses m1 4 m2 4 m3. Rank the three types according to (a) average kinetic energy and (b) rms speed, greatest first.

558

CHAPTE R 19 TH E KI N ETIC TH EORY OF GASES

19-5 MEAN FREE PATH Learning Objectives After reading this module, you should be able to . . .

19.22 Identify what is meant by mean free path. 19.23 Apply the relationship between the mean free path, the

diameter of the molecules, and the number of molecules per unit volume.

Key Idea ● The mean free path l of a gas molecule is its average path length between collisions and is given by

l#

1 , 12pd 2 N/V

where N/V is the number of molecules per unit volume and d is the molecular diameter.

Mean Free Path

Figure 19-5 A molecule traveling through a gas, colliding with other gas molecules in its path. Although the other molecules are shown as stationary, they are also moving in a similar fashion.

We continue to examine the motion of molecules in an ideal gas. Figure 19-5 shows the path of a typical molecule as it moves through the gas, changing both speed and direction abruptly as it collides elastically with other molecules. Between collisions, the molecule moves in a straight line at constant speed. Although the figure shows the other molecules as stationary, they are (of course) also moving. One useful parameter to describe this random motion is the mean free path l of the molecules. As its name implies, l is the average distance traversed by a molecule between collisions. We expect l to vary inversely with N/V, the number of molecules per unit volume (or density of molecules). The larger N/V is, the more collisions there should be and the smaller the mean free path. We also expect l to vary inversely with the size of the molecules — with their diameter d, say. (If the molecules were points, as we have assumed them to be, they would never collide and the mean free path would be infinite.) Thus, the larger the molecules are, the smaller the mean free path. We can even predict that l should vary (inversely) as the square of the molecular diameter because the cross section of a molecule — not its diameter — determines its effective target area. The expression for the mean free path does, in fact, turn out to be l#

1 12pd 2 N/V

(19-25)

(mean free path).

To justify Eq. 19-25, we focus attention on a single molecule and assume — as Fig. 19-5 suggests — that our molecule is traveling with a constant speed v and that all the other molecules are at rest. Later, we shall relax this assumption. We assume further that the molecules are spheres of diameter d. A collision will then take place if the centers of two molecules come within a distance d of each other, as in Fig. 19-6a. Another, more helpful way to look at the situation is Figure 19-6 (a) A collision occurs when the centers of two molecules come within a distance d of each other, d being the molecular diameter. (b) An equivalent but more convenient representation is to think of the moving molecule as having a radius d and all other molecules as being points. The condition for a collision is unchanged.

d

m

m

m

d

d

(a)

2d

m

(b)

19-5 M EAN FR E E PATH

2d v ∆t

Figure 19-7 In time %t the moving molecule effectively sweeps out a cylinder of length v %t and radius d.

to consider our single molecule to have a radius of d and all the other molecules to be points, as in Fig. 19-6b. This does not change our criterion for a collision. As our single molecule zigzags through the gas, it sweeps out a short cylinder of cross-sectional area pd 2 between successive collisions. If we watch this molecule for a time interval %t, it moves a distance v %t, where v is its assumed speed. Thus, if we align all the short cylinders swept out in interval %t, we form a composite cylinder (Fig. 19-7) of length v %t and volume (pd 2)(v %t). The number of collisions that occur in time %t is then equal to the number of (point) molecules that lie within this cylinder. Since N/V is the number of molecules per unit volume, the number of molecules in the cylinder is N/V times the volume of the cylinder, or (N/V )(pd 2v %t). This is also the number of collisions in time %t. The mean free path is the length of the path (and of the cylinder) divided by this number: length of path during %t v %t % number of collisions in %t pd 2v %t N/V 1 # . pd 2 N/V

l#

(19-26)

This equation is only approximate because it is based on the assumption that all the molecules except one are at rest. In fact, all the molecules are moving; when this is taken properly into account, Eq. 19-25 results. Note that it differs from the (approximate) Eq. 19-26 only by a factor of 1/12. The approximation in Eq. 19-26 involves the two v symbols we canceled. The v in the numerator is vavg, the mean speed of the molecules relative to the container. The v in the denominator is vrel, the mean speed of our single molecule relative to the other molecules, which are moving. It is this latter average speed that determines the number of collisions. A detailed calculation, taking into account the actual speed distribution of the molecules, gives vrel # 12 vavg and thus the factor 12. The mean free path of air molecules at sea level is about 0.1 mm. At an altitude of 100 km, the density of air has dropped to such an extent that the mean free path rises to about 16 cm. At 300 km, the mean free path is about 20 km. A problem faced by those who would study the physics and chemistry of the upper atmosphere in the laboratory is the unavailability of containers large enough to hold gas samples (of Freon, carbon dioxide, and ozone) that simulate upper atmospheric conditions.

Checkpoint 3 One mole of gas A, with molecular diameter 2d0 and average molecular speed v0, is placed inside a certain container. One mole of gas B, with molecular diameter d0 and average molecular speed 2v0 (the molecules of B are smaller but faster), is placed in an identical container.Which gas has the greater average collision rate within its container?

559

560

CHAPTE R 19 TH E KI N ETIC TH EORY OF GASES

Sample Problem 19.04 Mean free path, average speed, collision frequency (a) What is the mean free path l for oxygen molecules at temperature T # 300 K and pressure p # 1.0 atm? Assume that the molecular diameter is d # 290 pm and the gas is ideal.

collisions for any given molecule? At what rate does the molecule collide; that is, what is the frequency f of its collisions? KEY IDEAS

KEY IDEA Each oxygen molecule moves among other moving oxygen molecules in a zigzag path due to the resulting collisions. Thus, we use Eq. 19-25 for the mean free path. Calculation: We first need the number of molecules per unit volume, N/V. Because we assume the gas is ideal, we can use the ideal gas law of Eq. 19-9 (pV # NkT) to write N/V # p/kT. Substituting this into Eq. 19-25, we find 1 kT # l# 2 12pd N/V 12pd 2p #

(1.38 ' 10$23 J/K)(300 K)

12p (2.9 ' 10$10 m)2(1.01 ' 105 Pa)

# 1.1 ' 10$7 m. This is about 380 molecular diameters.

(Answer)

(b) Assume the average speed of the oxygen molecules is v # 450 m/s. What is the average time t between successive

(1) Between collisions, the molecule travels, on average, the mean free path l at speed v. (2) The average rate or frequency at which the collisions occur is the inverse of the time t between collisions. Calculations: From the first key idea, the average time between collisions is t#

l 1.1 ' 10$7 m distance # # speed v 450 m/s

# 2.44 ' 10$10 s % 0.24 ns.

(Answer) This tells us that, on average, any given oxygen molecule has less than a nanosecond between collisions. From the second key idea, the collision frequency is 1 1 (Answer) # # 4.1 ' 109 s$1. t 2.44 ' 10$10 s This tells us that, on average, any given oxygen molecule makes about 4 billion collisions per second. f#

Additional examples, video, and practice available at WileyPLUS

19-6 THE DISTRIBUTION OF MOLECULAR SPEEDS Learning Objectives After reading this module, you should be able to . . .

19.24 Explain how Maxwell’s speed distribution law is used to find the fraction of molecules with speeds in a certain speed range. 19.25 Sketch a graph of Maxwell’s speed distribution, showing the probability distribution versus speed and indicating the relative positions of the average speed vavg, the most probable speed vP, and the rms speed vrms.

19.26 Explain how Maxwell’s speed distribution is used to find the average speed, the rms speed, and the most probable speed. 19.27 For a given temperature T and molar mass M, calculate the average speed vavg, the most probable speed vP, and the rms speed vrms.

Key Ideas ● The Maxwell speed distribution P(v) is a function such that P(v) dv gives the fraction of molecules with speeds in the interval dv at speed v: 3/2 M 2 v2 e $Mv /2RT. P(v) # 4p 2pRT

#

vavg #

$

● Three measures of the distribution of speeds among the molecules of a gas are

vP # and

vrms #

8RT A pM

2RT A M

3RT A M

(average speed),

(most probable speed),

(rms speed).

19-6 TH E DISTR I B UTION OF M OLECU L AR SPE E DS

561

The Distribution of Molecular Speeds The root-mean-square speed vrms gives us a general idea of molecular speeds in a gas at a given temperature. We often want to know more. For example, what fraction of the molecules have speeds greater than the rms value? What fraction have speeds greater than twice the rms value? To answer such questions, we need to know how the possible values of speed are distributed among the molecules. Figure 19-8a shows this distribution for oxygen molecules at room temperature (T # 300 K); Fig. 19-8b compares it with the distribution at T # 80 K. In 1852, Scottish physicist James Clerk Maxwell first solved the problem of finding the speed distribution of gas molecules. His result, known as Maxwell’s speed distribution law, is P(v) # 4p

#

M 2pRT

$

3/2

v2 e $Mv

2 / 2RT

.

(19-27)

Here M is the molar mass of the gas, R is the gas constant, T is the gas temperature, and v is the molecular speed. It is this equation that is plotted in Fig. 19-8a, b. The quantity P(v) in Eq. 19-27 and Fig. 19-8 is a probability distribution function: For any speed v, the product P(v) dv (a dimensionless quantity) is the fraction of molecules with speeds in the interval dv centered on speed v. As Fig. 19-8a shows, this fraction is equal to the area of a strip with height P(v) and width dv. The total area under the distribution curve corresponds to the fraction of the molecules whose speeds lie between zero and infinity. All molecules fall into this category, so the value of this total area is unity; that is,

"

6

0

(19-28)

P(v) dv # 1.

The fraction (frac) of molecules with speeds in an interval of, say, v1 to v2 is then frac #

"

v2

(19-29)

P(v) dv.

v1

Average, RMS, and Most Probable Speeds In principle, we can find the average speed vavg of the molecules in a gas with the following procedure:We weight each value of v in the distribution; that is, we multiply it

2.0

4.0 T = 80 K

vP vrms 0

(a)

Area = P(v) dv

vavg

1.0

P(v) (10–3 s/m)

P(v) (10–3 s/m)

A

0

200

dv 400

600 800 Speed (m/s)

1000

1200

3.0

2.0 T = 300 K

Figure 19-8 (a) The Maxwell speed distribution for oxygen molecules at T # 300 K. The three characteristic speeds are marked. (b) The curves for 300 K and 80 K. Note that the molecules move more slowly at the lower temperature. Because these are probability distributions, the area under each curve has a numerical value of unity.

1.0

0 (b)

0

200

400

600 800 Speed (m/s)

1000

1200

562

CHAPTE R 19 TH E KI N ETIC TH EORY OF GASES

by the fraction P(v) dv of molecules with speeds in a differential interval dv centered on v.Then we add up all these values of v P(v) dv.The result is vavg. In practice, we do all this by evaluating vavg #

"

6

v P(v) dv.

(19-30)

0

Substituting for P(v) from Eq. 19-27 and using generic integral 20 from the list of integrals in Appendix E, we find vavg #

8RT A pM

(average speed).

(19-31)

Similarly, we can find the average of the square of the speeds (v2)avg with (v2)avg #

"

6

v2 P(v) dv.

(19-32)

0

Substituting for P(v) from Eq. 19-27 and using generic integral 16 from the list of integrals in Appendix E, we find 3RT (19-33) (v 2)avg # . M The square root of (v2)avg is the root-mean-square speed vrms. Thus, vrms #

3RT A M

(rms speed),

(19-34)

which agrees with Eq. 19-22. The most probable speed vP is the speed at which P(v) is maximum (see Fig. 19-8a). To calculate vP, we set dP/dv # 0 (the slope of the curve in Fig. 19-8a is zero at the maximum of the curve) and then solve for v. Doing so, we find vP #

2RT A M

(most probable speed).

(19-35)

A molecule is more likely to have speed vP than any other speed, but some molecules will have speeds that are many times vP. These molecules lie in the high-speed tail of a distribution curve like that in Fig. 19-8a. Such higher speed molecules make possible both rain and sunshine (without which we could not exist): Rain The speed distribution of water molecules in, say, a pond at summertime temperatures can be represented by a curve similar to that of Fig. 19-8a. Most of the molecules lack the energy to escape from the surface. However, a few of the molecules in the high-speed tail of the curve can do so. It is these water molecules that evaporate, making clouds and rain possible. As the fast water molecules leave the surface, carrying energy with them, the temperature of the remaining water is maintained by heat transfer from the surroundings. Other fast molecules—produced in particularly favorable collisions— quickly take the place of those that have left, and the speed distribution is maintained. Sunshine Let the distribution function of Eq. 19-27 now refer to protons in the core of the Sun. The Sun’s energy is supplied by a nuclear fusion process that starts with the merging of two protons. However, protons repel each other because of their electrical charges, and protons of average speed do not have enough kinetic energy to overcome the repulsion and get close enough to merge. Very fast protons with speeds in the high-speed tail of the distribution curve can do so, however, and for that reason the Sun can shine.

563

19-6 TH E DISTR I B UTION OF M OLECU L AR SPE E DS

Sample Problem 19.05 Speed distribution in a gas In oxygen (molar mass M = 0.0320 kg/mol) at room temperature (300 K), what fraction of the molecules have speeds in the interval 599 to 601 m/s? KEY IDEAS

frac # (4p)(A)(v2)(eB)(%v),

1. The speeds of the molecules are distributed over a wide range of values, with the distribution P(v) of Eq. 19-27. 2. The fraction of molecules with speeds in a differential interval dv is P(v) dv. 3. For a larger interval, the fraction is found by integrating P(v) over the interval. 4. However, the interval %v # 2 m/s here is small compared to the speed v # 600 m/s on which it is centered. Calculations: Because %v is small, we can avoid the integration by approximating the fraction as frac # P(v) %v # 4p

The total area under the plot of P(v) in Fig. 19-8a is the total fraction of molecules (unity), and the area of the thin gold strip (not to scale) is the fraction we seek. Let’s evaluate frac in parts:

M # 2pRT $

3/2

v 2 e $Mv

2/2RT

%v.

(19-36)

where A#

M # 2pRT $

3/2

0.0320 kg/mol # (2p)(8.31 J/mol9K)(300 K) $

3/2

#

# 2.92 ' 10$9 s3/m3 (0.0320 kg/mol)(600 m/s)2 Mv2 #$ 2RT (2)(8.31 J/mol 9K)(300 K) # $2.31. Substituting A and B into Eq. 19-36 yields and B # $

frac # (4p)(A)(v2)(eB)(%v) # (4p)(2.92 ' 10$9 s3/m3)(600 m/s)2(e$2.31)(2 m/s) # 2.62 ' 10$3 # 0.262%.

(Answer)

Sample Problem 19.06 Average speed, rms speed, most probable speed The molar mass M of oxygen is 0.0320 kg/mol.

Calculation: We end up with Eq. 19-34, which gives us

(a) What is the average speed vavg of oxygen gas molecules at T # 300 K? KEY IDEA

vrms #

3(8.31 J/mol 9K)(300 K) A 0.0320 kg/mol (Answer) # 483 m/s. This result, plotted in Fig. 19-8a, is greater than vavg because the greater speed values influence the calculation more when we integrate the v2 values than when we integrate the v values. #

To find the average speed, we must weight speed v with the distribution function P(v) of Eq. 19-27 and then integrate the resulting expression over the range of possible speeds (from zero to the limit of an infinite speed). Calculation: We end up with Eq. 19-31, which gives us 8RT vavg # A pM 8(8.31 J/mol 9K)(300 K) # p(0.0320 kg/mol) A # 445 m/s.

(Answer)

3RT A M

(c) What is the most probable speed vP at 300 K? KEY IDEA Speed vP corresponds to the maximum of the distribution function P(v), which we obtain by setting the derivative dP/dv # 0 and solving the result for v. Calculation: We end up with Eq. 19-35, which gives us

This result is plotted in Fig. 19-8a. (b) What is the root-mean-square speed vrms at 300 K?

vP #

KEY IDEA 2

2

To find vrms, we must first find (v )avg by weighting v with the distribution function P(v) of Eq. 19-27 and then integrating the expression over the range of possible speeds. Then we must take the square root of the result.

#

2RT A M

2(8.31 J/mol 9K)(300 K) A 0.0320 kg/mol

# 395 m/s.

This result is also plotted in Fig. 19-8a.

Additional examples, video, and practice available at WileyPLUS

(Answer)

564

CHAPTE R 19 TH E KI N ETIC TH EORY OF GASES

19-7 THE MOLAR SPECIFIC HEATS OF AN IDEAL GAS Learning Objectives After reading this module, you should be able to . . .

19.28 Identify that the internal energy of an ideal monatomic gas is the sum of the translational kinetic energies of its atoms. 19.29 Apply the relationship between the internal energy Eint of a monatomic ideal gas, the number of moles n, and the gas temperature T. 19.30 Distinguish between monatomic, diatomic, and polyatomic ideal gases. 19.31 For monatomic, diatomic, and polyatomic ideal gases, evaluate the molar specific heats for a constant-volume process and a constant-pressure process. 19.32 Calculate a molar specific heat at constant pressure Cp by adding R to the molar specific heat at constant volume CV, and explain why (physically) Cp is greater. 19.33 Identify that the energy transferred to an ideal gas as heat in a constant-volume process goes entirely into the internal energy (the random translational motion) but that

in a constant-pressure process energy also goes into the work done to expand the gas. 19.34 Identify that for a given change in temperature, the change in the internal energy of an ideal gas is the same for any process and is most easily calculated by assuming a constant-volume process. 19.35 For an ideal gas, apply the relationship between heat Q, number of moles n, and temperature change %T, using the appropriate molar specific heat. 19.36 Between two isotherms on a p-V diagram, sketch a constant-volume process and a constant-pressure process, and for each identify the work done in terms of area on the graph. 19.37 Calculate the work done by an ideal gas for a constantpressure process. 19.38 Identify that work is zero for constant volume.

Key Ideas ● The molar specific heat CV of a gas at constant volume is

defined as

CV #

%Eint Q # , n %T n %T

in which Q is the energy transferred as heat to or from a sample of n moles of the gas, %T is the resulting temperature change of the gas, and %Eint is the resulting change in the internal energy of the gas. ● For an ideal monatomic gas, CV #

3 2R

# 12.5 J/mol9K.

● The molar specific heat Cp of a gas at constant pressure is

defined to be

Q , n %T in which Q, n, and %T are defined as above. Cp is also given by Cp #

Cp # CV " R. ● For n moles of an ideal gas,

Eint # nCVT

(ideal gas).

● If n moles of a confined ideal gas undergo a temperature

change %T due to any process, the change in the internal energy of the gas is %Eint # nCV %T (ideal gas, any process).

The Molar Specific Heats of an Ideal Gas In this module, we want to derive from molecular considerations an expression for the internal energy Eint of an ideal gas. In other words, we want an expression for the energy associated with the random motions of the atoms or molecules in the gas.We shall then use that expression to derive the molar specific heats of an ideal gas.

Internal Energy Eint Let us first assume that our ideal gas is a monatomic gas (individual atoms rather than molecules), such as helium, neon, or argon. Let us also assume that the internal energy Eint is the sum of the translational kinetic energies of the atoms. (Quantum theory disallows rotational kinetic energy for individual atoms.) The average translational kinetic energy of a single atom depends only on the gas temperature and is given by Eq. 19-24 as Kavg # 32 kT. A sample of n moles of such a gas contains nNA atoms.The internal energy Eint of the sample is then Eint # (nNA)Kavg # (nNA)( 32kT ).

(19-37)

565

19-7 TH E M OL AR SPECI FIC H EATS OF AN I DEAL GAS

Using Eq. 19-7 (k # R/NA), we can rewrite this as Eint # 32 nRT

(monatomic ideal gas).

(19-38) Pin

Pin

The internal energy Eint of an ideal gas is a function of the gas temperature only; it does not depend on any other variable.

With Eq. 19-38 in hand, we are now able to derive an expression for the molar specific heat of an ideal gas. Actually, we shall derive two expressions. One is for the case in which the volume of the gas remains constant as energy is transferred to or from it as heat. The other is for the case in which the pressure of the gas remains constant as energy is transferred to or from it as heat. The symbols for these two molar specific heats are CV and Cp, respectively. (By convention, the capital letter C is used in both cases, even though CV and Cp represent types of specific heat and not heat capacities.)

Thermal reservoir

Molar Specific Heat at Constant Volume

p + ∆p

Q # nCV %T

(constant volume),

(19-39)

where CV is a constant called the molar specific heat at constant volume. Substituting this expression for Q into the first law of thermodynamics as given by Eq. 18-26 (%Eint # Q $ W ) yields %Eint # nCV %T $ W. (19-40)

(a)

Pressure

Figure 19-9a shows n moles of an ideal gas at pressure p and temperature T, confined to a cylinder of fixed volume V. This initial state i of the gas is marked on the p-V diagram of Fig. 19-9b. Suppose now that you add a small amount of energy to the gas as heat Q by slowly turning up the temperature of the thermal reservoir. The gas temperature rises a small amount to T " %T, and its pressure rises to p " %p, bringing the gas to final state f. In such experiments, we would find that the heat Q is related to the temperature change %T by

Q

T

The temperature increase is done without changing the volume.

f

p i

T + ∆T V

T

Volume (b)

Figure 19-9 (a) The temperature of an ideal gas is raised from T to T " %T in a constantvolume process. Heat is added, but no work is done. (b) The process on a p-V diagram.

With the volume held constant, the gas cannot expand and thus cannot do any work. Therefore, W # 0, and Eq. 19-40 gives us %Eint . n %T From Eq. 19-38, the change in internal energy must be CV #

%Eint # 32 nR %T.

(19-41) Table 19-2 Molar Specific Heats at Constant Volume

(19-42) Molecule

Substituting this result into Eq. 19-41 yields CV # 32R # 12.5 J/mol9K

(monatomic gas).

(19-43)

As Table 19-2 shows, this prediction of the kinetic theory (for ideal gases) agrees very well with experiment for real monatomic gases, the case that we have assumed. The (predicted and) experimental values of CV for diatomic gases (which have molecules with two atoms) and polyatomic gases (which have molecules with more than two atoms) are greater than those for monatomic gases for reasons that will be suggested in Module 19-8. Here we make the preliminary assumption that the CV values for diatomic and polyatomic gases are greater than for monatomic gases because the more complex molecules can rotate and thus have rotational kinetic energy. So, when Q is transferred to a diatomic or polyatomic gas, only part of it goes into the translational kinetic energy, increasing the

Monatomic

He Ar

Ideal Real

# 12.5 12.5 12.6

5 2R

Ideal Real

Polyatomic

3 2R

Ideal Real

Diatomic

CV (J/mol 9K)

Example

N2 O2

# 20.8 20.7 20.8

3R # 24.9 NH4 CO2

29.0 29.7

566

CHAPTE R 19 TH E KI N ETIC TH EORY OF GASES

The paths are different, but the change in the internal energy is the same.

f

Pressure

3

temperature. (For now we neglect the possibility of also putting energy into oscillations of the molecules.) We can now generalize Eq. 19-38 for the internal energy of any ideal gas by substituting CV for 32 R; we get Eint # nCVT

f 1

f 2

i

T + ∆T T

W

(19-44)

This equation applies not only to an ideal monatomic gas but also to diatomic and polyatomic ideal gases, provided the appropriate value of CV is used. Just as with Eq. 19-38, we see that the internal energy of a gas depends on the temperature of the gas but not on its pressure or density. When a confined ideal gas undergoes temperature change %T, then from either Eq. 19-41 or Eq. 19-44 the resulting change in its internal energy is

Volume

Figure 19-10 Three paths representing three different processes that take an ideal gas from an initial state i at temperature T to some final state f at temperature T " %T. The change %Eint in the internal energy of the gas is the same for these three processes and for any others that result in the same change of temperature.

(any ideal gas).

%Eint # nCV %T

(ideal gas, any process).

(19-45)

This equation tells us: A change in the internal energy Eint of a confined ideal gas depends on only the change in the temperature, not on what type of process produces the change.

As examples, consider the three paths between the two isotherms in the p-V diagram of Fig. 19-10. Path 1 represents a constant-volume process. Path 2 represents a constant-pressure process (we examine it next). Path 3 represents a process in which no heat is exchanged with the system’s environment (we discuss this in Module 19-9). Although the values of heat Q and work W associated with these three paths differ, as do pf and Vf , the values of %Eint associated with the three paths are identical and are all given by Eq. 19-45, because they all involve the same temperature change %T. Therefore, no matter what path is actually taken between T and T " %T, we can always use path 1 and Eq. 19-45 to compute %Eint easily.

Molar Specific Heat at Constant Pressure

Q

(a)

We now assume that the temperature of our ideal gas is increased by the same small amount %T as previously but now the necessary energy (heat Q) is added with the gas under constant pressure. An experiment for doing this is shown in Fig. 19-11a; the p-V diagram for the process is plotted in Fig. 19-11b. From such experiments we find that the heat Q is related to the temperature change %T by

T

Thermal reservoir

Q # nCp %T

Pressure

The temperature increase is done without changing the pressure. f

p i

T + ∆T p ∆V

V (b)

T V + ∆V

Volume

Figure 19-11 (a) The temperature of an ideal gas is raised from T to T " %T in a constantpressure process. Heat is added and work is done in lifting the loaded piston. (b) The process on a p-V diagram. The work p %V is given by the shaded area.

(constant pressure),

(19-46)

where Cp is a constant called the molar specific heat at constant pressure. This Cp is greater than the molar specific heat at constant volume CV, because energy must now be supplied not only to raise the temperature of the gas but also for the gas to do work — that is, to lift the weighted piston of Fig. 19-11a. To relate molar specific heats Cp and CV, we start with the first law of thermodynamics (Eq. 18-26): (19-47) %Eint # Q $ W. We next replace each term in Eq. 19-47. For %Eint, we substitute from Eq. 19-45. For Q, we substitute from Eq. 19-46. To replace W, we first note that since the pressure remains constant, Eq. 19-16 tells us that W # p %V. Then we note that, using the ideal gas equation (pV # nRT), we can write W # p %V # nR %T.

(19-48)

Making these substitutions in Eq. 19-47 and then dividing through by n %T, we find CV # Cp $ R

19-7 TH E M OL AR SPECI FIC H EATS OF AN I DEAL GAS

Monatomic

Diatomic

7 nR ∆T __ 2

Figure 19-12 The relative values of Q for a monatomic gas (left side) and a diatomic gas undergoing a constant-volume process (labeled “con V”) and a constant-pressure process (labeled “con p”). The transfer of the energy into work W and internal energy (%Eint) is noted.

Q @ con p W ∆Eint

W 5 nR ∆T __ 2

Q @ con p

W 3 nR ∆T __ 2

567

W ∆Eint

rotation trans

Q @ con V ∆Eint

trans

rotation trans

Q @ con V ∆Eint

trans

and then Cp # CV " R.

(19-49)

This prediction of kinetic theory agrees well with experiment, not only for monatomic gases but also for gases in general, as long as their density is low enough so that we may treat them as ideal. The left side of Fig. 19-12 shows the relative values of Q for a monatomic gas undergoing either a constant-volume process (Q # 32nR %T ) or a constantpressure process (Q # 52nR %T ). Note that for the latter, the value of Q is higher by the amount W, the work done by the gas in the expansion. Note also that for the constant-volume process, the energy added as Q goes entirely into the change in internal energy %Eint and for the constant-pressure process, the energy added as Q goes into both %Eint and the work W.

Checkpoint 4 The figure here shows five paths traversed by a gas on a p-V diagram. Rank the paths according to the change in internal energy of the gas, greatest first.

p 1 2 4

3

T3 5 T2 T1

V

Sample Problem 19.07 Monatomic gas, heat, internal energy, and work A bubble of 5.00 mol of helium is submerged at a certain depth in liquid water when the water (and thus the helium) undergoes a temperature increase %T of 20.0 C- at constant pressure. As a result, the bubble expands. The helium is monatomic and ideal. (a) How much energy is added to the helium as heat during the increase and expansion? KEY IDEA

constant pressure Cp and Eq. 19-46, Q # nCp %T,

(19-50)

to find Q. To evaluate Cp we go to Eq. 19-49, which tells us that for any ideal gas, Cp # CV " R. Then from Eq. 19-43, we know that for any monatomic gas (like the helium here), CV # 32R. Thus, Eq. 19-50 gives us Q # n(CV " R) %T # n(32R " R) %T # n(52R) %T

Heat Q is related to the temperature change %T by a molar specific heat of the gas.

# (5.00 mol)(2.5)(8.31 J/mol9K)(20.0 C-) (Answer) # 2077.5 J % 2080 J.

Calculations: Because the pressure p is held constant during the addition of energy, we use the molar specific heat at

(b) What is the change %Eint in the internal energy of the helium during the temperature increase?

568

CHAPTE R 19 TH E KI N ETIC TH EORY OF GASES

KEY IDEA Because the bubble expands, this is not a constant-volume process. However, the helium is nonetheless confined (to the bubble). Thus, the change %Eint is the same as would occur in a constant-volume process with the same temperature change %T. Calculation: We can now easily find the constant-volume change %Eint with Eq. 19-45: %Eint # nCV %T # n(32R) %T # (5.00 mol)(1.5)(8.31 J/mol9K)(20.0 C-) (Answer) # 1246.5 J % 1250 J. (c) How much work W is done by the helium as it expands against the pressure of the surrounding water during the temperature increase? KEY IDEAS The work done by any gas expanding against the pressure from its environment is given by Eq. 19-11, which tells us to in-

tegrate p dV. When the pressure is constant (as here), we can simplify that to W # p %V. When the gas is ideal (as here), we can use the ideal gas law (Eq. 19-5) to write p %V # nR %T. Calculation: We end up with W # nR %T # (5.00 mol)(8.31 J/mol 9K)(20.0 C-) # 831 J.

(Answer)

Another way: Because we happen to know Q and %Eint, we can work this problem another way: We can account for the energy changes of the gas with the first law of thermodynamics, writing W # Q $ %Eint # 2077.5 J $ 1246.5 J # 831 J. (Answer) The transfers: Let’s follow the energy. Of the 2077.5 J transferred to the helium as heat Q, 831 J goes into the work W required for the expansion and 1246.5 J goes into the internal energy Eint, which, for a monatomic gas, is entirely the kinetic energy of the atoms in their translational motion. These several results are suggested on the left side of Fig. 19-12.

Additional examples, video, and practice available at WileyPLUS

19-8 DEGREES OF FREEDOM AND MOLAR SPECIFIC HEATS Learning Objectives After reading this module, you should be able to . . .

19.39 Identify that a degree of freedom is associated with each way a gas can store energy (translation, rotation, and oscillation). 19.40 Identify that an energy of 12kT per molecule is associated with each degree of freedom. 19.41 Identify that a monatomic gas can have an internal energy consisting of only translational motion.

19.42 Identify that at low temperatures a diatomic gas has energy in only translational motion, at higher temperatures it also has energy in molecular rotation, and at even higher temperatures it can also have energy in molecular oscillations. 19.43 Calculate the molar specific heat for monatomic and diatomic ideal gases in a constant-volume process and a constant-pressure process.

Key Ideas ● We find CV by using the equipartition of energy theorem, which states that every degree of freedom of a molecule (that is, every independent way it can store energy) has associated with it—on average—an energy 12kT per molecule (# 12RT per mole). ● If f

is the number of degrees of freedom, then

Eint # ( f/2)nRT and CV #

# 2f $R # 4.16f J/mol9K.

● For monatomic gases f

# 3 (three translational degrees); for diatomic gases f # 5 (three translational and two rotational degrees).

Degrees of Freedom and Molar Specific Heats As Table 19-2 shows, the prediction that CV # 32R agrees with experiment for monatomic gases but fails for diatomic and polyatomic gases. Let us try to explain the discrepancy by considering the possibility that molecules with more than one atom can store internal energy in forms other than translational kinetic energy. Figure 19-13 shows common models of helium (a monatomic molecule, containing a single atom), oxygen (a diatomic molecule, containing two atoms), and

19-8 DEG R E ES OF FR E E DOM AN D M OL AR SPECI FIC H EATS

569

Table 19-3 Degrees of Freedom for Various Molecules Degrees of Freedom Molecule

Predicted Molar Specific Heats

Example

Translational

Rotational

Total ( f )

CV (Eq. 19-51)

Cp # CV " R

Monatomic

He

3

0

3

Diatomic

O2

3

2

5

3 2R 5 2R

5 2R 7 2R

Polyatomic

CH4

3

3

6

3R

4R

methane (a polyatomic molecule). From such models, we would assume that all three types of molecules can have translational motions (say, moving left – right and up – down) and rotational motions (spinning about an axis like a top). In addition, we would assume that the diatomic and polyatomic molecules can have oscillatory motions, with the atoms oscillating slightly toward and away from one another, as if attached to opposite ends of a spring. To keep account of the various ways in which energy can be stored in a gas, James Clerk Maxwell introduced the theorem of the equipartition of energy:

He (a) He

O

O (b) O2

Every kind of molecule has a certain number f of degrees of freedom, which are independent ways in which the molecule can store energy. Each such degree of freedom has associated with it—on average—an energy of 12kT per molecule (or 12RT per mole).

Let us apply the theorem to the translational and rotational motions of the molecules in Fig. 19-13. (We discuss oscillatory motion below.) For the translational motion, superimpose an xyz coordinate system on any gas. The molecules will, in general, have velocity components along all three axes. Thus, gas molecules of all types have three degrees of translational freedom (three ways to move in translation) and, on average, an associated energy of 3(12kT ) per molecule. For the rotational motion, imagine the origin of our xyz coordinate system at the center of each molecule in Fig. 19-13. In a gas, each molecule should be able to rotate with an angular velocity component along each of the three axes, so each gas should have three degrees of rotational freedom and, on average, an additional energy of 3(12kT ) per molecule. However, experiment shows this is true only for the polyatomic molecules.According to quantum theory, the physics dealing with the allowed motions and energies of molecules and atoms, a monatomic gas molecule does not rotate and so has no rotational energy (a single atom cannot rotate like a top).A diatomic molecule can rotate like a top only about axes perpendicular to the line connecting the atoms (the axes are shown in Fig. 19-13b) and not about that line itself. Therefore, a diatomic molecule can have only two degrees of rotational freedom and a rotational energy of only 2(12kT ) per molecule. To extend our analysis of molar specific heats (Cp and C in Module 19-7) to ideal diatomic and polyatomic gases, it is necessary to retrace the derivations of that analysis in detail. First, we replace Eq. 19-38 (Eint # 32nRT ) with Eint # ( f/2)nRT, where f is the number of degrees of freedom listed in Table 19-3. Doing so leads to the prediction CV #

# 2f $R # 4.16f J/mol9K,

H

C

H

H

H

(c) CH4

Figure 19-13 Models of molecules as used in kinetic theory: (a) helium, a typical monatomic molecule; (b) oxygen, a typical diatomic molecule; and (c) methane, a typical polyatomic molecule.The spheres represent atoms, and the lines between them represent bonds.Two rotation axes are shown for the oxygen molecule.

(19-51)

which agrees—as it must—with Eq. 19-43 for monatomic gases ( f # 3). As Table 19-2 shows, this prediction also agrees with experiment for diatomic gases ( f # 5), but it is too low for polyatomic gases ( f # 6 for molecules comparable to CH4). Sample Problem 19.08 Diatomic gas, heat, temperature, internal energy We transfer 1000 J as heat Q to a diatomic gas, allowing the gas to expand with the pressure held constant. The gas molecules

each rotate around an internal axis but do not oscillate. How much of the 1000 J goes into the increase of the gas’s internal

570

CHAPTE R 19 TH E KI N ETIC TH EORY OF GASES

energy? Of that amount, how much goes into %Ktran (the kinetic energy of the translational motion of the molecules) and %Krot (the kinetic energy of their rotational motion)? KEY IDEAS 1. The transfer of energy as heat Q to a gas under constant pressure is related to the resulting temperature increase %T via Eq. 19-46 (Q # nCp %T ). 2. Because the gas is diatomic with molecules undergoing rotation but not oscillation, the molar specific heat is, from Fig. 19-12 and Table 19-3, Cp # 72R. 3. The increase %Eint in the internal energy is the same as would occur with a constant-volume process resulting in the same %T. Thus, from Eq. 19-45, %Eint # nCV %T. From Fig. 19-12 and Table 19-3, we see that CV # 52R. 4. For the same n and %T, %Eint is greater for a diatomic gas than for a monatomic gas because additional energy is required for rotation. Increase in Eint: Let’s first get the temperature change %T due to the transfer of energy as heat. From Eq. 19-46, substituting 7 2 R for Cp, we have Q . (19-52) %T # 7 2 nR We next find %Eint from Eq. 19-45, substituting the molar specific heat CV (# 52R) for a constant-volume process and using the same %T. Because we are dealing with a diatomic gas, let’s call this change %Eint,dia. Equation 19-45 gives us

Q # nR $#

%Eint,dia # nCV %T # n52R

7 2

5 7Q

(Answer)

# 0.71428Q # 714.3 J.

In words, about 71% of the energy transferred to the gas goes into the internal energy. The rest goes into the work required to increase the volume of the gas, as the gas pushes the walls of its container outward. Increases in K: If we were to increase the temperature of a monatomic gas (with the same value of n) by the amount given in Eq. 19-52, the internal energy would change by a smaller amount, call it %Eint, mon, because rotational motion is not involved. To calculate that smaller amount, we still use Eq. 19-45 but now we substitute the value of CV for a monatomic gas — namely, CV # 32R. So, %Eint,mon # n32R %T. Substituting for %T from Eq. 19-52 leads us to

# nQR $ #

%Eint,mon # n32R

7 2

3 7Q

# 0.42857Q # 428.6 J. For the monatomic gas, all this energy would go into the kinetic energy of the translational motion of the atoms. The important point here is that for a diatomic gas with the same values of n and %T, the same amount of energy goes into the kinetic energy of the translational motion of the molecules. The rest of %Eint, dia (that is, the additional 285.7 J) goes into the rotational motion of the molecules. Thus, for the diatomic gas, %Ktrans # 428.6 J

and %Krot # 285.7 J.

(Answer)

Additional examples, video, and practice available at WileyPLUS

4 7/2 3

Oscillation

CV/R

5/2 2

Rotation 3/2

1 Translation 0 20

50

100

200 500 1000 2000 Temperature (K)

5000 10,000

Figure 19-14 CV /R versus temperature for (diatomic) hydrogen gas. Because rotational and oscillatory motions begin at certain energies, only translation is possible at very low temperatures.As the temperature increases, rotational motion can begin.At still higher temperatures, oscillatory motion can begin.

A Hint of Quantum Theory We can improve the agreement of kinetic theory with experiment by including the oscillations of the atoms in a gas of diatomic or polyatomic molecules. For example, the two atoms in the O2 molecule of Fig. 19-13b can oscillate toward and away from each other, with the interconnecting bond acting like a spring. However, experiment shows that such oscillations occur only at relatively high temperatures of the gas — the motion is “turned on” only when the gas molecules have relatively large energies. Rotational motion is also subject to such “turning on,” but at a lower temperature. Figure 19-14 is of help in seeing this turning on of rotational motion and oscillatory motion. The ratio CV /R for diatomic hydrogen gas (H2) is plotted there against temperature, with the temperature scale logarithmic to cover several orders of magnitude. Below about 80 K, we find that CV /R # 1.5. This result implies that only the three translational degrees of freedom of hydrogen are involved in the specific heat.

19-9 TH E ADIABATIC EXPANSION OF AN I DEAL GAS

571

As the temperature increases, the value of CV /R gradually increases to 2.5, implying that two additional degrees of freedom have become involved. Quantum theory shows that these two degrees of freedom are associated with the rotational motion of the hydrogen molecules and that this motion requires a certain minimum amount of energy. At very low temperatures (below 80 K), the molecules do not have enough energy to rotate. As the temperature increases from 80 K, first a few molecules and then more and more of them obtain enough energy to rotate, and the value of CV /R increases, until all of the molecules are rotating and CV /R # 2.5. Similarly, quantum theory shows that oscillatory motion of the molecules requires a certain (higher) minimum amount of energy. This minimum amount is not met until the molecules reach a temperature of about 1000 K, as shown in Fig. 19-14. As the temperature increases beyond 1000 K, more and more molecules have enough energy to oscillate and the value of CV /R increases, until all of the molecules are oscillating and CV /R # 3.5. (In Fig. 19-14, the plotted curve stops at 3200 K because there the atoms of a hydrogen molecule oscillate so much that they overwhelm their bond, and the molecule then dissociates into two separate atoms.) The turning on of the rotation and vibration of the diatomic and polyatomic molecules is due to the fact that the energies of these motions are quantized, that is, restricted to certain values. There is a lowest allowed value for each type of motion. Unless the thermal agitation of the surrounding molecules provides those lowest amounts, a molecule simply cannot rotate or vibrate.

19-9 THE ADIABATIC EXPANSION OF AN IDEAL GAS Learning Objectives After reading this module, you should be able to . . .

19.44 On a p-V diagram, sketch an adiabatic expansion (or contraction) and identify that there is no heat exchange Q with the environment. 19.45 Identify that in an adiabatic expansion, the gas does work on the environment, decreasing the gas’s internal energy, and that in an adiabatic contraction, work is done on the gas, increasing the internal energy. 19.46 In an adiabatic expansion or contraction, relate the initial pressure and volume to the final pressure and volume.

19.47 In an adiabatic expansion or contraction, relate the initial temperature and volume to the final temperature and volume. 19.48 Calculate the work done in an adiabatic process by integrating the pressure with respect to volume. 19.49 Identify that a free expansion of a gas into a vacuum is adiabatic but no work is done and thus, by the first law of thermodynamics, the internal energy and temperature of the gas do not change.

Key Ideas ● When an ideal gas undergoes a slow adiabatic volume change (a change for which Q # 0), g pV # a constant (adiabatic process),

in which g (# Cp /CV) is the ratio of molar specific heats for the gas. ● For a free expansion, pV # a constant.

The Adiabatic Expansion of an Ideal Gas We saw in Module 17-2 that sound waves are propagated through air and other gases as a series of compressions and expansions; these variations in the transmission medium take place so rapidly that there is no time for energy to be transferred from one part of the medium to another as heat. As we saw in Module 18-5, a process for which Q # 0 is an adiabatic process. We can ensure that Q # 0 either by carrying out the process very quickly (as in sound waves) or by doing it (at any rate) in a well-insulated container.

CHAPTE R 19 TH E KI N ETIC TH EORY OF GASES

Figure 19-15a shows our usual insulated cylinder, now containing an ideal gas and resting on an insulating stand. By removing mass from the piston, we can allow the gas to expand adiabatically. As the volume increases, both the pressure and the temperature drop. We shall prove next that the relation between the pressure and the volume during such an adiabatic process is g

pV # a constant

(19-53)

(adiabatic process),

in which g # Cp /CV, the ratio of the molar specific heats for the gas. On a p-V diagram such as that in Fig. 19-15b, the process occurs along a line (called an adiabat) that has the equation p # (a constant)/V g. Since the gas goes from an initial state i to a final state f, we can rewrite Eq. 19-53 as piV gi # pfV gf

(19-54)

(adiabatic process).

To write an equation for an adiabatic process in terms of T and V, we use the ideal gas equation ( pV # nRT ) to eliminate p from Eq. 19-53, finding V # nRT V $

:

# a constant.

Because n and R are constants, we can rewrite this in the alternative form TV

g$1

# a constant

(19-55)

(adiabatic process),

in which the constant is different from that in Eq. 19-53. When the gas goes from an initial state i to a final state f, we can rewrite Eq. 19-55 as g

g

TiV i $1# TfV f $1

(19-56)

(adiabatic process).

Understanding adiabatic processes allows you to understand why popping the cork on a cold bottle of champagne or the tab on a cold can of soda causes a slight fog to form at the opening of the container. At the top of any unopened carbonated drink sits a gas of carbon dioxide and water vapor. Because the pressure of that gas is much greater than atmospheric pressure, the gas expands out into the atmosphere when the container is opened. Thus, the gas volume increases, but that means the gas must do work pushing against the atmosphere. Because the expansion is rapid, it is adiabatic, and the only source of energy for the work is the internal energy of the gas. Because the internal energy decreases, We slowly remove lead shot, allowing an expansion without any heat transfer. W

Adiabat (Q = 0)

Pressure

572

i

f

Insulation

Isotherms: 700 K 500 K 300 K

Volume (a)

(b)

Figure 19-15 (a) The volume of an ideal gas is increased by removing mass from the piston. The process is adiabatic (Q # 0). (b) The process proceeds from i to f along an adiabat on a p-V diagram.

19-9 TH E ADIABATIC EXPANSION OF AN I DEAL GAS

the temperature of the gas also decreases and so does the number of water molecules that can remain as a vapor. So, lots of the water molecules condense into tiny drops of fog.

Proof of Eq. 19-53 Suppose that you remove some shot from the piston of Fig. 19-15a, allowing the ideal gas to push the piston and the remaining shot upward and thus to increase the volume by a differential amount dV. Since the volume change is tiny, we may assume that the pressure p of the gas on the piston is constant during the change. This assumption allows us to say that the work dW done by the gas during the volume increase is equal to p dV. From Eq. 18-27, the first law of thermodynamics can then be written as dEint # Q $ p dV.

(19-57)

Since the gas is thermally insulated (and thus the expansion is adiabatic), we substitute 0 for Q. Then we use Eq. 19-45 to substitute nCV dT for dEint. With these substitutions, and after some rearranging, we have n dT # $

# Cp $ dV.

(19-58)

V

Now from the ideal gas law ( pV # nRT ) we have p dV " V dp # nR dT.

(19-59)

Replacing R with its equal, Cp $ CV, in Eq. 19-59 yields n dT #

p dV " V dp . Cp $ CV

(19-60)

Equating Eqs. 19-58 and 19-60 and rearranging then give dp " p

# C $ dVV # 0. Cp

V

Replacing the ratio of the molar specific heats with g and integrating (see integral 5 in Appendix E) yield ln p " g ln V # a constant. Rewriting the left side as ln pV g and then taking the antilog of both sides, we find pV g # a constant.

(19-61)

Free Expansions Recall from Module 18-5 that a free expansion of a gas is an adiabatic process with no work or change in internal energy. Thus, a free expansion differs from the adiabatic process described by Eqs. 19-53 through 19-61, in which work is done and the internal energy changes. Those equations then do not apply to a free expansion, even though such an expansion is adiabatic. Also recall that in a free expansion, a gas is in equilibrium only at its initial and final points; thus, we can plot only those points, but not the expansion itself, on a p-V diagram. In addition, because %Eint # 0, the temperature of the final state must be that of the initial state. Thus, the initial and final points on a p-V diagram must be on the same isotherm, and instead of Eq. 19-56 we have Ti # Tf

(free expansion).

(19-62)

573

574

CHAPTE R 19 TH E KI N ETIC TH EORY OF GASES

If we next assume that the gas is ideal (so that pV # nRT ), then because there is no change in temperature, there can be no change in the product pV. Thus, instead of Eq. 19-53 a free expansion involves the relation piVi # pf Vf

(19-63)

(free expansion).

Sample Problem 19.09 Work done by a gas in an adiabatic expansion Initially an ideal diatomic gas has pressure pi # 2.00 ' 105 Pa and volume Vi # 4.00 ' 10$6 m3. How much work W does it do, and what is the change %Eint in its internal energy if it expands adiabatically to volume Vf # 8.00 ' 10$6 m3? Throughout the process, the molecules have rotation but not oscillation. KEY IDEA

expression into Eq. 19-64 and integrating lead us to W#

Calculations: We want to calculate the work by filling out this integration, W#

"

Vf

Vi

p dV,

Vf

Vi

1 g (19-65) piV gi # V $gpiV i . Vg The initial quantities are given constants but the pressure p is a function of the variable volume V. Substituting this p#

"

g

"

Vf

Vi

Vf

Vi

g g V $ piV i dV

g

V$ dV #

1 g piVi [V$g "1]VVf i $g " 1

1 g g "1 g "1 $ V$ ]. piV i [V $ f i $g " 1

(19-66)

Before we substitute in given data, we must determine the ratio g of molar specific heats for a gas of diatomic molecules with rotation but no oscillation. From Table 19-3 we find g#

7 R Cp # 25 # 1.4. CV 2R

(19-67)

We can now write the work done by the gas as the following (with volume in cubic meters and pressure in pascals): W#

(19-64)

but we first need an expression for the pressure as a function of volume (so that we integrate the expression with respect to volume). So, let’s rewrite Eq. 19-54 with indefinite symbols (dropping the subscripts f ) as

p dV #

# piVi #

(1) In an adiabatic expansion, no heat is exchanged between the gas and its environment, and the energy for the work done by the gas comes from the internal energy. (2) The final pressure and volume are related to the initial pressure and volume by Eq. 19-54 (piV gi # pfV gf ). (3) The work done by a gas in any process can be calculated by integrating the pressure with respect to the volume (the work is due to the gas pushing the walls of its container outward).

"

1 (2.00 ' 105)(4.00 ' 10$6)1.4 $1.4 " 1 ' [(8.00 ' 10$6)$1.4"1 $ (4.00 ' 10$6)$1.4"1]

# 0.48 J.

(Answer)

The first law of thermodynamics (Eq. 18-26) tells us that %Eint # Q $ W. Because Q # 0 in the adiabatic expansion, we see that %Eint # $0.48 J. (Answer) With this decrease in internal energy, the gas temperature must also decrease because of the expansion.

Sample Problem 19.10 Adiabatic expansion, free expansion Initially, 1 mol of oxygen (assumed to be an ideal gas) has temperature 310 K and volume 12 L. We will allow it to expand to volume 19 L. (a) What would be the final temperature if the gas expands adiabatically? Oxygen (O2) is diatomic and here has rotation but not oscillation. KEY IDEAS 1. When a gas expands against the pressure of its environment, it must do work.

2. When the process is adiabatic (no energy is transferred as heat), then the energy required for the work can come only from the internal energy of the gas. 3. Because the internal energy decreases, the temperature T must also decrease. Calculations: We can relate the initial and final temperatures and volumes with Eq. 19-56: g $1

g $1

(19-68) . Because the molecules are diatomic and have rotation but not oscillation, we can take the molar specific heats from TiVi

# TfVf

575

R EVI EW & SU M MARY

Table 19-3. Thus,

KEY IDEA g#

Cp

#

CV

7 2R 5 2R

# 1.40.

Solving Eq. 19-68 for Tf and inserting known data then yield g$1

Tf #

TiV i

#

g$1 Vf

# (310

Calculation: Thus, the temperature is

(310 K)(12 L)1.40$1 (19 L)1.40$1

0.40 K)(12 19 )

# 258 K.

The temperature does not change in a free expansion because there is nothing to change the kinetic energy of the molecules.

Tf # Ti # 310 K. (Answer)

(b) What would be the final temperature and pressure if, instead, the gas expands freely to the new volume, from an initial pressure of 2.0 Pa?

(Answer)

We find the new pressure using Eq. 19-63, which gives us pf # pi

Vi 12 L # (2.0 Pa) # 1.3 Pa. Vf 19 L

(Answer)

In this chapter we have discussed four special processes that an ideal gas can undergo. An example of each (for a monatomic ideal gas) is shown in Fig. 19-16, and some associated characteristics are given in Table 19-4, including two process names (isobaric and isochoric) that we have not used but that you might see in other courses.

Pressure

Problem-Solving Tactics A Graphical Summary of Four Gas Processes

i 4 f

Checkpoint 5

1 2 3

f f f

700 K 500 K 400 K

Volume

Rank paths 1, 2, and 3 in Fig. 19-16 according to the energy transfer to the gas as heat, greatest first.

Figure 19-16 A p-V diagram representing four special processes for an ideal monatomic gas.

Table 19-4 Four Special Processes Some Special Results Path in Fig. 19-16 1 2 3 4

Constant Quantity

Process Type

p T g g pV , TV $1 V

Isobaric Isothermal Adiabatic Isochoric

(%Eint # Q $ W and %Eint # nCV %T for all paths) Q # nCp %T; W # p %V Q # W # nRT ln(Vf /Vi ); %Eint # 0 Q # 0; W # $%Eint Q # %Eint # nCV %T; W # 0

Additional examples, video, and practice available at WileyPLUS

Review & Summary Kinetic Theory of Gases The kinetic theory of gases relates the macroscopic properties of gases (for example, pressure and temperature) to the microscopic properties of gas molecules (for example, speed and kinetic energy).

One molar mass M of any substance is the mass of one mole of the substance. It is related to the mass m of the individual molecules of the substance by M # mNA. (19-4)

Avogadro’s Number One mole of a substance contains

The number of moles n contained in a sample of mass Msam, consisting of N molecules, is given by

NA (Avogadro’s number) elementary units (usually atoms or molecules), where NA is found experimentally to be 23

NA # 6.02 ' 10 mol

$1

(Avogadro’s number).

(19-1)

n#

Msam Msam N # # . NA M mNA

(19-2, 19-3)

576

CHAPTE R 19 TH E KI N ETIC TH EORY OF GASES

Ideal Gas An ideal gas is one for which the pressure p, volume V, and temperature T are related by pV # nRT

vavg # (19-5)

(ideal gas law).

Here n is the number of moles of the gas present and R is a constant (8.31 J/mol 9K) called the gas constant. The ideal gas law can also be written as pV # NkT, (19-9) where the Boltzmann constant k is

by an ideal gas during an isothermal (constant-temperature) change from volume Vi to volume Vf is (ideal gas, isothermal process).

Vi

sure exerted by n moles of an ideal gas, in terms of the speed of its molecules, is nMv2rms (19-21) , p# 3V where vrms # 2(v2)avg is the root-mean-square speed of the molecules of the gas. With Eq. 19-5 this gives 3RT . A M

(19-22)

Temperature and Kinetic Energy The average translational kinetic energy Kavg per molecule of an ideal gas is Kavg # 32kT.

(19-24)

Mean Free Path The mean free path l of a gas molecule is its average path length between collisions and is given by l#

1 , 12pd 2 N/V

(19-25)

where N/V is the number of molecules per unit volume and d is the molecular diameter.

Maxwell Speed Distribution The Maxwell speed distribution P(v) is a function such that P(v) dv gives the fraction of molecules with speeds in the interval dv at speed v: P(v) # 4p

M # 2pRT $

3/2

(19-31)

(most probable speed),

(19-35)

Molar Specific Heats The molar specific heat CV of a gas at

v2 e $Mv

2/2RT

.

CV #

(19-27)

Three measures of the distribution of speeds among the molecules of

Q %Eint # , n %T n %T

(19-39, 19-41)

in which Q is the energy transferred as heat to or from a sample of n moles of the gas, %T is the resulting temperature change of the gas, and %Eint is the resulting change in the internal energy of the gas. For an ideal monatomic gas,

(19-14)

Pressure, Temperature, and Molecular Speed The pres-

vrms #

2RT A M

(average speed),

and the rms speed defined above in Eq. 19-22.

(19-7)

Work in an Isothermal Volume Change The work done

Vf

vP #

8RT A pM

constant volume is defined as

R k# # 1.38 ' 10$23 J/K. NA

W # nRT ln

a gas are

CV # 32R # 12.5 J/mol9K.

(19-43)

The molar specific heat Cp of a gas at constant pressure is defined to be Cp #

Q , n %T

(19-46)

in which Q, n, and %T are defined as above. Cp is also given by (19-49)

Cp # CV " R. For n moles of an ideal gas, Eint # nCVT

(19-44)

(ideal gas).

If n moles of a confined ideal gas undergo a temperature change %T due to any process, the change in the internal energy of the gas is %Eint # nCV %T

(19-45)

(ideal gas, any process).

Degrees of Freedom and CV The equipartition of energy

theorem states that every degree of freedom of a molecule has an energy 12kT per molecule (# 12RT per mole). If f is the number of degrees of freedom, then Eint # ( f/2)nRT and CV #

# 2f $R # 4.16f J/mol9K.

(19-51)

For monatomic gases f # 3 (three translational degrees); for diatomic gases f # 5 (three translational and two rotational degrees).

Adiabatic Process When an ideal gas undergoes an adiabatic volume change (a change for which Q # 0), g

pV # a constant

(19-53)

(adiabatic process),

in which g (# Cp /CV) is the ratio of molar specific heats for the gas. For a free expansion, however, pV # a constant.

Questions 1 For four situations for an a b c d ideal gas, the table gives the Q $50 "35 $15 "20 energy transferred to or from $50 "35 the gas as heat Q and either the W $40 "40 work W done by the gas or the Won work Won done on the gas, all in joules. Rank the four situations in terms of the temperature change of the gas, most positive first.

p

2 In the p-V diagram of Fig. 19-17, the gas does 5 J of work when taken along isotherm ab and 4 J when taken along adiabat bc. What is the change Figure 19-17 Question 2.

a b

c V

PROB LE M S

final points on a p-V diagram for the two gases. Which path goes with which process? (e) Are the molecules of the diatomic gas rotating?

in the internal energy of the gas when it is taken along the straight path from a to c? 3 For a temperature increase of %T1, a certain amount of an ideal gas requires 30 J when heated at constant volume and 50 J when heated at constant pressure. How much work is done by the gas in the second situation?

6 The dot in Fig. 19-18b represents the initial state of a gas, and the isotherm through the dot divides the p-V diagram into regions 1 and 2. For the following processes, determine whether the change %Eint in the internal energy of the gas is positive, negative, or zero: (a) the gas moves up along the isotherm, (b) it moves down along the isotherm, (c) it moves to anywhere in region 1, and (d) it moves to anywhere in region 2.

4 The dot in Fig. 19-18a represents the initial state of a gas, and the vertical line through the dot divides the p-V diagram into regions 1 and 2. For the following processes, determine whether the work W done by the gas is positive, negative, or zero: (a) the gas moves up along the vertical line, (b) it moves down along the vertical line, (c) it moves to anywhere in region 1, and (d) it moves to anywhere in region 2. p

p

1

8 The dot in Fig. 19-18c represents the initial state of a gas, and the adiabat through the dot divides the p-V diagram into regions 1 and 2. For the following processes, determine whether the corresponding heat Q is positive, negative, or zero: (a) the gas moves up along the adiabat, (b) it moves down along the adiabat, (c) it moves to anywhere in region 1, and (d) it moves to anywhere in region 2.

2

1

1

V (a)

7 (a) Rank the four paths of Fig. 19-16 according to the work done by the gas, greatest first. (b) Rank paths 1, 2, and 3 according to the change in the internal energy of the gas, most positive first and most negative last.

p 2

2

V

V

(b)

(c)

Figure 19-18 Questions 4, 6, and 8. 5 A certain amount of energy is to be transferred as heat to 1 mol of a monatomic gas (a) at constant pressure and (b) at constant volume, and to 1 mol of a diatomic gas (c) at constant pressure and (d) at constant volume. Figure 19-19 shows four paths from an initial point to four

9 An ideal diatomic gas, with molecular rotation but without any molecular oscillation, loses a certain amount of energy as heat Q. Is the resulting decrease in the internal energy of the gas greater if the loss occurs in a constant-volume process or in a constant-pressure process?

p 1 2

577

3

10 Does the temperature of an ideal gas increase, decrease, or stay the same during (a) an isothermal expansion, (b) an expansion at constant pressure, (c) an adiabatic expansion, and (d) an increase in pressure at constant volume?

4 V

Figure 19-19 Question 5.

Problems Tutoring problem available (at instructor’s discretion) in WileyPLUS and WebAssign SSM • – •••

Worked-out solution available in Student Solutions Manual

WWW Worked-out solution is at

Number of dots indicates level of problem difficulty

ILW

Interactive solution is at

http://www.wiley.com/college/halliday

Additional information available in The Flying Circus of Physics and at flyingcircusofphysics.com

Module 19-1 Avogadro’s Number •1 Find the mass in kilograms of 7.50 ' 10 24 atoms of arsenic, which has a molar mass of 74.9 g/mol. •2 Gold has a molar mass of 197 g/mol. (a) How many moles of gold are in a 2.50 g sample of pure gold? (b) How many atoms are in the sample? Module 19-2 Ideal Gases •3 SSM Oxygen gas having a volume of 1000 cm3 at 40.0-C and 1.01 ' 10 5 Pa expands until its volume is 1500 cm3 and its pressure is 1.06 ' 10 5 Pa. Find (a) the number of moles of oxygen present and (b) the final temperature of the sample.

•6 Water bottle in a hot car. In the American Southwest, the temperature in a closed car parked in sunlight during the summer can be high enough to burn flesh. Suppose a bottle of water at a refrigerator temperature of 5.00-C is opened, then closed, and then left in a closed car with an internal temperature of 75.0-C. Neglecting the thermal expansion of the water and the bottle, find the pressure in the air pocket trapped in the bottle. (The pressure can be enough to push the bottle cap past the threads that are intended to keep the bottle closed.) •7 Suppose 1.80 mol of an ideal gas is taken from a volume of 3.00 m3 to a volume of 1.50 m3 via an isothermal compression at 30-C. (a) How much energy is transferred as heat during the compression, and (b) is the transfer to or from the gas?

•4 A quantity of ideal gas at 10.0-C and 100 kPa occupies a volume of 2.50 m3. (a) How many moles of the gas are present? (b) If the pressure is now raised to 300 kPa and the temperature is raised to 30.0-C, how much volume does the gas occupy? Assume no leaks.

•8 Compute (a) the number of moles and (b) the number of molecules in 1.00 cm3 of an ideal gas at a pressure of 100 Pa and a temperature of 220 K.

•5 The best laboratory vacuum has a pressure of about 1.00 ' 10$18 atm, or 1.01 ' 10$13 Pa. How many gas molecules are there per cubic centimeter in such a vacuum at 293 K?

•9 An automobile tire has a volume of 1.64 ' 10$2 m3 and contains air at a gauge pressure (pressure above atmospheric pressure) of 165 kPa when the temperature is 0.00-C. What is the gauge

578

CHAPTE R 19 TH E KI N ETIC TH EORY OF GASES

pressure of the air in the tires when its temperature rises to 27.0-C and its volume increases to 1.67 ' 10$2 m3? Assume atmospheric pressure is 1.01 ' 10 5 Pa.

the vertical axis is set by Vfs # 0.30 m3, and the scale of the horizontal axis is set by Qs # 1200 J.

•10 A container encloses 2 mol of an ideal gas that has molar mass M1 and 0.5 mol of a second ideal gas that has molar mass M2 # 3M1. What fraction of the total pressure on the container wall is attributable to the second gas? (The kinetic theory explanation of pressure leads to the experimentally discovered law of partial pressures for a mixture of gases that do not react chemically: The total pressure exerted by the mixture is equal to the sum of the pressures that the several gases would exert separately if each were to occupy the vessel alone. The molecule–vessel collisions of one type would not be altered by the presence of another type.) ••11 SSM ILW WWW Air that initially occupies 0.140 m3 at a gauge pressure of 103.0 kPa is expanded isothermally to a pressure of 101.3 kPa and then cooled at constant pressure until it reaches its initial volume. Compute the work done by the air. (Gauge pressure is the difference between the actual pressure and atmospheric pressure.)

••13 A sample of an ideal gas is taken through the cyclic process abca shown in Fig. 19-20. The scale of the vertical axis is set by pb # 7.5 kPa and pac # 2.5 kPa. At point a, T # 200 K. (a) How many moles of gas are in the sample? What are (b) the temperature of the gas at point b, (c) the temperature of the gas at point c, and (d) the net energy added to the gas as heat during the cycle?

Pressure (kPa)

••12 Submarine rescue. When the U.S. submarine Squalus became disabled at a depth of 80 m, a cylindrical chamber was lowered from a ship to rescue the crew. The chamber had a radius of 1.00 m and a height of 4.00 m, was open at the bottom, and held two rescuers. It slid along a guide cable that a diver had attached to a hatch on the submarine. Once the chamber reached the hatch and clamped to the hull, the crew could escape into the chamber. During the descent, air was released from tanks to prevent water from flooding the chamber. Assume that the interior air pressure matched the water pressure at depth h as given by p0 " rgh, where p0 # 1.000 atm is the surface pressure and r # 1024 kg/m3 is the density of seawater. Assume a surface temperature of 20.0-C and a submerged water temperature of $30.0-C. (a) What is the air volume in the chamber at the surface? (b) If air had not been released from the tanks, what would have been the air volume in the chamber at depth h # 80.0 m? (c) How many moles of air were needed to be released to maintain the original air volume in the chamber? b

pb

pac

Vf (m3) 0

Figure 19-21 Problem 15.

Q (J)

Qs

•••16 An air bubble of volume 20 cm3 is at the bottom of a lake 40 m deep, where the temperature is 4.0-C. The bubble rises to the surface, which is at a temperature of 20-C. Take the temperature of the bubble’s air to be the same as that of the surrounding water. Just as the bubble reaches the surface, what is its volume? •••17 Container A in Fig. 19-22 holds an ideal gas at a pressure of 5.0 ' 10 5 Pa and a temperature of 300 K. It is connected by a thin tube (and a closed valve) to container B, A with four times the volume of A. B Container B holds the same ideal 5 gas at a pressure of 1.0 ' 10 Pa and Figure 19-22 Problem 17. a temperature of 400 K. The valve is opened to allow the pressures to equalize, but the temperature of each container is maintained. What then is the pressure? Module 19-3 Pressure, Temperature, and RMS Speed •18 The temperature and pressure in the Sun’s atmosphere are 2.00 ' 10 6 K and 0.0300 Pa. Calculate the rms speed of free electrons (mass 9.11 ' 10$31 kg) there, assuming they are an ideal gas. •19 (a) Compute the rms speed of a nitrogen molecule at 20.0-C. The molar mass of nitrogen molecules (N2) is given in Table 19-1. At what temperatures will the rms speed be (b) half that value and (c) twice that value? •20 Calculate the rms speed of helium atoms at 1000 K. See Appendix F for the molar mass of helium atoms.

a

c

1.0 3.0 Volume (m3)

Figure 19-20 Problem 13.

••14 In the temperature range 310 K to 330 K, the pressure p of a certain nonideal gas is related to volume V and temperature T by p # (24.9 J/K)

Vfs

T T2 $ (0.00662 J/K2) . V V

•21 SSM The lowest possible temperature in outer space is 2.7 K. What is the rms speed of hydrogen molecules at this temperature? (The molar mass is given in Table 19-1.) •22 Find the rms speed of argon atoms at 313 K. See Appendix F for the molar mass of argon atoms. ••23 A beam of hydrogen molecules (H2) is directed toward a wall, at an angle of 55- with the normal to the wall. Each molecule in the beam has a speed of 1.0 km/s and a mass of 3.3 ' 10$24 g. The beam strikes the wall over an area of 2.0 cm2, at the rate of 10 23 molecules per second.What is the beam’s pressure on the wall?

How much work is done by the gas if its temperature is raised from 315 K to 325 K while the pressure is held constant?

••24 At 273 K and 1.00 ' 10$2 atm, the density of a gas is 1.24 ' 10$5 g/cm3. (a) Find vrms for the gas molecules. (b) Find the molar mass of the gas and (c) identify the gas. See Table 19-1.

••15 Suppose 0.825 mol of an ideal gas undergoes an isothermal expansion as energy is added to it as heat Q. If Fig. 19-21 shows the final volume Vf versus Q, what is the gas temperature? The scale of

Module 19-4 Translational Kinetic Energy •25 Determine the average value of the translational kinetic energy of the molecules of an ideal gas at temperatures (a) 0.00-C

PROB LE M S

••27 Water standing in the open at 32.0-C evaporates because of the escape of some of the surface molecules. The heat of vaporization (539 cal/g) is approximately equal to ´n, where ´ is the average energy of the escaping molecules and n is the number of molecules per gram. (a) Find ´. (b) What is the ratio of ´ to the average kinetic energy of H2O molecules, assuming the latter is related to temperature in the same way as it is for gases? Module 19-5 Mean Free Path •28 At what frequency would the wavelength of sound in air be equal to the mean free path of oxygen molecules at 1.0 atm pressure and 0.00-C? The molecular diameter is 3.0 ' 10$8 cm. •29 SSM The atmospheric density at an altitude of 2500 km is about 1 molecule/cm3. (a) Assuming the molecular diameter of 2.0 ' 10$8 cm, find the mean free path predicted by Eq. 19-25. (b) Explain whether the predicted value is meaningful. •30 The mean free path of nitrogen molecules at 0.0-C and 1.0 atm is 0.80 ' 10$5 cm. At this temperature and pressure there are 2.7 ' 10 19 molecules/cm3. What is the molecular diameter? ••31 In a certain particle accelerator, protons travel around a circular path of diameter 23.0 m in an evacuated chamber, whose residual gas is at 295 K and 1.00 ' 10$6 torr pressure. (a) Calculate the number of gas molecules per cubic centimeter at this pressure. (b) What is the mean free path of the gas molecules if the molecular diameter is 2.00 ' 10$8 cm? ••32 At 20-C and 750 torr pressure, the mean free paths for argon gas (Ar) and nitrogen gas (N2) are lAr # 9.9 ' 10$6 cm and lN2 # 27.5 ' 10$6 cm. (a) Find the ratio of the diameter of an Ar atom to that of an N2 molecule. What is the mean free path of argon at (b) 20-C and 150 torr, and (c) $40-C and 750 torr? Module 19-6 The Distribution of Molecular Speeds •33 SSM The speeds of 10 molecules are 2.0, 3.0, 4.0, . . . , 11 km/s. What are their (a) average speed and (b) rms speed? •34 The speeds of 22 particles are as follows (Ni represents the number of particles that have speed vi): Ni vi (cm/s)

2 1.0

4 2.0

6 3.0

8 4.0

2 5.0

What are (a) vavg, (b) vrms, and (c) vP? •35 Ten particles are moving with the following speeds: four at 200 m/s, two at 500 m/s, and four at 600 m/s. Calculate their (a) average and (b) rms speeds. (c) Is vrms 4 vavg? ••36 The most probable speed of the molecules in a gas at temperature T2 is equal to the rms speed of the molecules at temperature T1. Find T2 /T1. a P(v)

••37 SSM WWW Figure 19-23 shows a hypothetical speed distribution for a sample of N gas particles (note that P(v) # 0 for speed v 4 2v0).What are the values of (a) av0, (b) vavg /v0, and (c) vrms/v0? (d) What fraction of the particles has a speed between 1.5v0 and 2.0v0?

0

v0 Speed

2v0

Figure 19-23 Problem 37.

0

vs

v (m/s)

Figure 19-24 Problem 38. ••39 At what temperature does the rms speed of (a) H2 (molecular hydrogen) and (b) O2 (molecular oxygen) equal the escape speed from Earth (Table 13-2)? At what temperature does the rms speed of (c) H2 and (d) O2 equal the escape speed from the Moon (where the gravitational acceleration at the surface has magnitude 0.16g)? Considering the answers to parts (a) and (b), should there be much (e) hydrogen and (f) oxygen high in Earth’s upper atmosphere, where the temperature is about 1000 K? ••40 Two containers are at the same temperature. The first contains gas with pressure p1, molecular mass m1, and rms speed vrms1. The second contains gas with pressure 2.0p1, molecular mass m2, and average speed vavg2 # 2.0vrms1. Find the mass ratio m1/m2. ••41 A hydrogen molecule (diameter 1.0 ' 10$8 cm), traveling at the rms speed, escapes from a 4000 K furnace into a chamber containing cold argon atoms (diameter 3.0 ' 10$8 cm) at a density of 4.0 ' 10 19 atoms/cm3. (a) What is the speed of the hydrogen molecule? (b) If it collides with an argon atom, what is the closest their centers can be, considering each as spherical? (c) What is the initial number of collisions per second experienced by the hydrogen molecule? (Hint: Assume that the argon atoms are stationary. Then the mean free path of the hydrogen molecule is given by Eq. 19-26 and not Eq. 19-25.) Module 19-7 The Molar Specific Heats of an Ideal Gas •42 What is the internal energy of 1.0 mol of an ideal monatomic gas at 273 K? ••43 The temperature of 3.00 mol of an ideal diatomic gas is increased by 40.0 C- without the pressure of the gas changing. The molecules in the gas rotate but do not oscillate. (a) How much energy is transferred to the gas as heat? (b) What is the change in the internal energy of the gas? (c) How much work is done by the gas? (d) By how much does the rotational kinetic energy of the gas increase? ••44 One mole of an ideal diatomic gas goes from a to c along the diagonal path in Fig. 19-25. The scale of the vertical axis is set by pab # 5.0 kPa and pc # 2.0 kPa, and the scale of the horizontal axis is set by Vbc # 4.0 m3 and Va # 2.0 m3. During the transition, (a) what is the change in internal energy of the gas, and (b) how

Pressure (kPa)

•26 What is the average translational kinetic energy of nitrogen molecules at 1600 K?

••38 Figure 19-24 gives the probability distribution for nitrogen gas. The scale of the horizontal axis is set by vs # 1200 m/s. What are the (a) gas temperature and (b) rms speed of the molecules?

P(v)

and (b) 100-C. What is the translational kinetic energy per mole of an ideal gas at (c) 0.00-C and (d) 100-C?

579

pab

pc

a

b

c Va Vbc Volume (m3)

Figure 19-25 Problem 44.

580

CHAPTE R 19 TH E KI N ETIC TH EORY OF GASES

much energy is added to the gas as heat? (c) How much heat is required if the gas goes from a to c along the indirect path abc? ••45 ILW The mass of a gas molecule can be computed from its specific heat at constant volume cV . (Note that this is not CV.) Take cV # 0.075 cal/g 9C- for argon and calculate (a) the mass of an argon atom and (b) the molar mass of argon. ••46 Under constant pressure, the temperature of 2.00 mol of an ideal monatomic gas is raised 15.0 K. What are (a) the work W done by the gas, (b) the energy transferred as heat Q, (c) the change %Eint in the internal energy of the gas, and (d) the change %K in the average kinetic energy per atom? ••47 The temperature of 2.00 mol of an ideal monatomic gas is raised 15.0 K at constant volume. What are (a) the work W done by the gas, (b) the energy transferred as heat Q, (c) the change %Eint in the internal energy of the gas, and (d) the change %K in the average kinetic energy per atom? ••48 When 20.9 J was added as heat to a particular ideal gas, the volume of the gas changed from 50.0 cm3 to 100 cm3 while the pressure remained at 1.00 atm. (a) By how much did the internal energy of the gas change? If the quantity of gas present was 2.00 ' 10$3 mol, find (b) Cp and (c) CV. ••49 SSM A container holds a mixture of three nonreacting gases: 2.40 mol of gas 1 with CV1 # 12.0 J/mol 9K, 1.50 mol of gas 2 with CV2 # 12.8 J/mol 9K, and 3.20 mol of gas 3 with CV3 # 20.0 J/mol 9K. What is CV of the mixture?

•56 Suppose 1.00 L of a gas with g # 1.30, initially at 273 K and 1.00 atm, is suddenly compressed adiabatically to half its initial volume. Find its final (a) pressure and (b) temperature. (c) If the gas is then cooled to 273 K at constant pressure, what is its final volume? ••57 The volume of an ideal gas is adiabatically reduced from 200 L to 74.3 L. The initial pressure and temperature are 1.00 atm and 300 K. The final pressure is 4.00 atm. (a) Is the gas monatomic, diatomic, or polyatomic? (b) What is the final temperature? (c) How many moles are in the gas? ••58 Opening champagne. In a bottle of champagne, the pocket of gas (primarily carbon dioxide) between the liquid and the cork is at pressure of pi # 5.00 atm. When the cork is pulled from the bottle, the gas undergoes an adiabatic expansion until its pressure matches the ambient air pressure of 1.00 atm. Assume that the ratio of the molar specific heats is : # 43. If the gas has initial temperature Ti # 5.00-C, what is its temperature at the end of the adiabatic expansion? ••59 Figure 19-26 shows two paths that may be taken by a gas from an initial point i to a final point f. Path 1 consists of an isothermal expansion (work is 50 J in magnitude), an adiabatic expansion (work is 40 J in magnitude), an isothermal compression (work is 30 J in magnitude), and then an adiabatic compression (work is 25 J in magnitude). What is the change in the internal energy of the gas if the gas goes from point i to point f along path 2? p

i

Module 19-8 Degrees of Freedom and Molar Specific Heats •50 We give 70 J as heat to a diatomic gas, which then expands at constant pressure. The gas molecules rotate but do not oscillate. By how much does the internal energy of the gas increase? •51 ILW When 1.0 mol of oxygen (O2) gas is heated at constant pressure starting at 0-C, how much energy must be added to the gas as heat to double its volume? (The molecules rotate but do not oscillate.) ••52 Suppose 12.0 g of oxygen (O2) gas is heated at constant atmospheric pressure from 25.0-C to 125-C. (a) How many moles of oxygen are present? (See Table 19-1 for the molar mass.) (b) How much energy is transferred to the oxygen as heat? (The molecules rotate but do not oscillate.) (c) What fraction of the heat is used to raise the internal energy of the oxygen? ••53 SSM WWW Suppose 4.00 mol of an ideal diatomic gas, with molecular rotation but not oscillation, experienced a temperature increase of 60.0 K under constant-pressure conditions. What are (a) the energy transferred as heat Q, (b) the change %Eint in internal energy of the gas, (c) the work W done by the gas, and (d) the change %K in the total translational kinetic energy of the gas? Module 19-9 The Adiabatic Expansion of an Ideal Gas •54 We know that for an adiabatic process pVg # a constant. Evaluate “a constant” for an adiabatic process involving exactly 2.0 mol of an ideal gas passing through the state having exactly p # 1.0 atm and T # 300 K. Assume a diatomic gas whose molecules rotate but do not oscillate. •55 A certain gas occupies a volume of 4.3 L at a pressure of 1.2 atm and a temperature of 310 K. It is compressed adiabatically to a volume of 0.76 L. Determine (a) the final pressure and (b) the final temperature, assuming the gas to be an ideal gas for which g # 1.4.

Path 1 f Path 2

Isothermal Adiabatic

Isothermal

V

Figure 19-26 Problem 59. ••60 Adiabatic wind. The normal airflow over the Rocky Mountains is west to east. The air loses much of its moisture content and is chilled as it climbs the western side of the mountains. When it descends on the eastern side, the increase in pressure toward lower altitudes causes the temperature to increase. The flow, then called a chinook wind, can rapidly raise the air temperature at the base of the mountains. Assume that the air pressure p depends on altitude y according to p # p0 exp ($ay), where p0 # 1.00 atm and a # 1.16 ' 10$4 m$1. Also assume that the ratio of the molar specific heats is g # 43. A parcel of air with an initial temperature of $5.00-C descends adiabatically from y1 # 4267 m to y # 1567 m. What is its temperature at the end of the descent? ••61 A gas is to be expanded from initial state i to final state f along either path 1 or path 2 on a p-V diagram. Path 1 consists of three steps: an isothermal expansion (work is 40 J in magnitude), an adiabatic expansion (work is 20 J in magnitude), and another isothermal expansion (work is 30 J in magnitude). Path 2 consists of two steps: a pressure reduction at constant volume and an expansion at constant pressure. What is the change in the internal energy of the gas along path 2? •••62 An ideal diatomic gas, with rotation but no oscillation, undergoes an adiabatic compression. Its initial pressure and volume are

PROB LE M S

•••63 Figure 19-27 shows a cycle unAdiabatic dergone by 1.00 mol of an ideal monatomic gas. The temperatures are 1 3 T1 # 300 K, T2 # 600 K, and T3 # 455 K. For 1 : 2, what are (a) heat Q, (b) the change in internal energy %Eint, Volume and (c) the work done W? For 2 : 3, Figure 19-27 Problem 63. what are (d) Q, (e) %Eint, and (f) W? For 3 : 1, what are (g) Q, (h) %Eint, and (i) W? For the full cycle, what are (j) Q, (k) %Eint, and (l) W? The initial pressure at point 1 is 1.00 atm (# 1.013 ' 10 5 Pa).What are the (m) volume and (n) pressure at point 2 and the (o) volume and (p) pressure at point 3? Additional Problems 64 Calculate the work done by an external agent during an isothermal compression of 1.00 mol of oxygen from a volume of 22.4 L at 0-C and 1.00 atm to a volume of 16.8 L. 65 An ideal gas undergoes an adiabatic compression from p # 1.0 atm, V # 1.0 ' 10 6 L, T # 0.0-C to p # 1.0 ' 10 5 atm, V # 1.0 ' 10 3 L. (a) Is the gas monatomic, diatomic, or polyatomic? (b) What is its final temperature? (c) How many moles of gas are present? What is the total translational kinetic energy per mole (d) before and (e) after the compression? (f) What is the ratio of the squares of the rms speeds before and after the compression? 66 An ideal gas consists of 1.50 mol of diatomic molecules that rotate but do not oscillate. The molecular diameter is 250 pm. The gas is expanded at a constant pressure of 1.50 ' 10 5 Pa, with a transfer of 200 J as heat.What is the change in the mean free path of the molecules? 67 An ideal monatomic gas initially has a temperature of 330 K and a pressure of 6.00 atm. It is to expand from volume 500 cm3 to volume 1500 cm3. If the expansion is isothermal, what are (a) the final pressure and (b) the work done by the gas? If, instead, the expansion is adiabatic, what are (c) the final pressure and (d) the work done by the gas? 68 In an interstellar gas cloud at 50.0 K, the pressure is 1.00 ' 10$8 Pa. Assuming that the molecular diameters of the gases in the cloud are all 20.0 nm, what is their mean free path? 69 SSM The envelope and basket of a hot-air balloon have a combined weight of 2.45 kN, and the envelope has a capacity (volume) of 2.18 ' 10 3 m3. When it is fully inflated, what should be the temperature of the enclosed air to give the balloon a lifting capacity (force) of 2.67 kN (in addition to the balloon’s weight)? Assume that the surrounding air, at 20.0-C, has a weight per unit volume of 11.9 N/m3 and a molecular mass of 0.028 kg/mol, and is at a pressure of 1.0 atm. 70 An ideal gas, at initial temperature T1 and initial volume 2.0 m3, is expanded adiabatically to a volume of 4.0 m3, then expanded isothermally to a volume of 10 m3, and then compressed adiabatically back to T1. What is its final volume? 71 SSM The temperature of 2.00 mol of an ideal monatomic gas is raised 15.0 K in an adiabatic process. What are (a) the work W done by the gas, (b) the energy transferred as heat Q, (c) the change %Eint in internal energy of the gas, and (d) the change %K in the average kinetic energy per atom?

72 At what temperature do atoms of helium gas have the same rms speed as molecules of hydrogen gas at 20.0-C? (The molar masses are given in Table 19-1.) 73 SSM At what frequency do molecules (diameter 290 pm) collide in (an ideal) oxygen gas (O2) at temperature 400 K and pressure 2.00 atm? 74 (a) What is the number of molecules per cubic meter in air at 20-C and at a pressure of 1.0 atm (# 1.01 ' 10 5 Pa)? (b) What is the mass of 1.0 m3 of this air? Assume that 75% of the molecules are nitrogen (N2) and 25% are oxygen (O2). 75 The temperature of 3.00 mol of a gas with CV # 6.00 cal/mol 9K is to be raised 50.0 K. If the process is at constant volume, what are (a) the energy transferred as heat Q, (b) the work W done by the gas, (c) the change %Eint in internal energy of the gas, and (d) the change %K in the total translational kinetic energy? If the process is at constant pressure, what are (e) Q, (f) W, (g) %Eint, and (h) %K? If the process is adiabatic, what are (i) Q, (j) W, (k) %Eint, and (l) %K? 76 During a compression at a constant pressure of 250 Pa, the volume of an ideal gas decreases from 0.80 m3 to 0.20 m3. The initial temperature is 360 K, and the gas loses 210 J as heat. What are (a) the change in the internal energy of the gas and (b) the final temperature of the gas? 77 SSM Figure 19-28 shows a hypothetical speed distribution for particles of a certain gas: P(v) # Cv2 for 0 8 v 2 v0 and P(v) # 0 for v 4 v0. Find (a) an expression for C in terms of v0, (b) the average speed of the particles, and (c) their rms speed.

P(v)

2 Pressure

1.20 atm and 0.200 m3. Its final pressure is 2.40 atm. How much work is done by the gas?

581

0

Speed

v0

Figure 19-28 Problem 77.

78 (a) An ideal gas initially at pressure p0 undergoes a free expansion until its volume is 3.00 times its initial volume. What then is the ratio of its pressure to p0? (b) The gas is next slowly and adiabatically compressed back to its original volume. The pressure after compression is (3.00)1/3 p0. Is the gas monatomic, diatomic, or polyatomic? (c) What is the ratio of the average kinetic energy per molecule in this final state to that in the initial state? 79 SSM An ideal gas undergoes isothermal compression from an initial volume of 4.00 m3 to a final volume of 3.00 m3. There is 3.50 mol of the gas, and its temperature is 10.0-C. (a) How much work is done by the gas? (b) How much energy is transferred as heat between the gas and its environment? 80 Oxygen (O2) gas at 273 K and 1.0 atm is confined to a cubical container 10 cm on a side. Calculate %Ug /Kavg, where %Ug is the change in the gravitational potential energy of an oxygen molecule falling the height of the box and Kavg is the molecule’s average translational kinetic energy. 81 An ideal gas is taken through a complete cycle in three steps: adiabatic expansion with work equal to 125 J, isothermal contraction at 325 K, and increase in pressure at constant volume. (a) Draw a p-V diagram for the three steps. (b) How much energy is transferred as heat in step 3, and (c) is it transferred to or from the gas? 82 (a) What is the volume occupied by 1.00 mol of an ideal gas at standard conditions — that is, 1.00 atm (# 1.01 ' 10 5 Pa) and 273 K? (b) Show that the number of molecules per cubic centimeter (the Loschmidt number) at standard conditions is 2.69 ' 10 9. 83

SSM

A sample of ideal gas expands from an initial pressure

582

CHAPTE R 19 TH E KI N ETIC TH EORY OF GASES

and volume of 32 atm and 1.0 L to a final volume of 4.0 L. The initial temperature is 300 K. If the gas is monatomic and the expansion isothermal, what are the (a) final pressure pf , (b) final temperature Tf , and (c) work W done by the gas? If the gas is monatomic and the expansion adiabatic, what are (d) pf , (e) Tf , and (f) W? If the gas is diatomic and the expansion adiabatic, what are (g) pf , (h) Tf , and (i) W?

until the water rises halfway up in the pipe (Fig. 19-30). What is the depth h of the lower end of the pipe? Assume that the temperature is the same everywhere and does not change.

84 An ideal gas with 3.00 mol is initially in state 1 with pressure p1 # 20.0 atm and volume V1 # 1500 cm3. First it is taken to state 2 with pressure p2 # 1.50p1 and volume V2 # 2.00V1. Then it is taken to state 3 with pressure p3 # 2.00p1 and volume V3 # 0.500V1. What is the temperature of the gas in (a) state 1 and (b) state 2? (c) What is the net change in internal energy from state 1 to state 3?

90 In a motorcycle engine, a piston is forced down toward Figure 19-30 Problem 89. the crankshaft when the fuel in the top of the piston’s cylinder undergoes combustion. The mixture of gaseous combustion products then expands adiabatically as the piston descends. Find the average power in (a) watts and (b) horsepower that is involved in this expansion when the engine is running at 4000 rpm, assuming that the gauge pressure immediately after combustion is 15 atm, the initial volume is 50 cm3, and the volume of the mixture at the bottom of the stroke is 250 cm3. Assume that the gases are diatomic and that the time involved in the expansion is one-half that of the total cycle.

85 A steel tank contains 300 g of ammonia gas (NH3) at a pressure of 1.35 ' 10 6 Pa and a temperature of 77-C. (a) What is the volume of the tank in liters? (b) Later the temperature is 22-C and the pressure is 8.7 ' 10 5 Pa. How many grams of gas have leaked out of the tank? 86 In an industrial process the volume of 25.0 mol of a monatomic ideal gas is reduced at a uniform rate from 0.616 m3 to 0.308 m3 in 2.00 h while its temperature is increased at a uniform rate from 27.0-C to 450-C. Throughout the process, the gas passes through thermodynamic equilibrium states. What are (a) the cumulative work done on the gas, (b) the cumulative energy absorbed by the gas as heat, and (c) the molar specific heat for the process? (Hint: To evaluate the integral for the work, you might use

"

a " bx bx aB $ bA dx # " ln(A " Bx), A " Bx B B2

L/2

h L/2

91 For adiabatic processes in an ideal gas, show that (a) the bulk modulus is given by dp B # $V # g p, dV where g # Cp/CV. (See Eq. 17-2.) (b) Then show that the speed of sound in the gas is vs #

gp gRT # , A r A M

an indefinite integral.) Suppose the process is replaced with a twostep process that reaches the same final state. In step 1, the gas volume is reduced at constant temperature, and in step 2 the temperature is increased at constant volume. For this process, what are (d) the cumulative work done on the gas, (e) the cumulative energy absorbed by the gas as heat, and (f) the molar specific heat for the process?

where r is the density, T is the temperature, and M is the molar mass. (See Eq. 17-3.)

87 Figure 19-29 shows a cycle consisting of five paths: AB is isothermal at 300 K, BC is adiabatic with work # 5.0 J, CD is at a constant pressure of 5 atm, DE is isothermal, and EA is adiabatic with a change in internal energy of 8.0 J. What is the change in internal energy of the gas along path CD?

93 The speed of sound in different gases at a certain temperature T depends on the molar mass of the gases. Show that

92 Air at 0.000-C and 1.00 atm pressure has a density of 1.29 ' 10$3 g/cm3, and the speed of sound is 331 m/s at that temperature. Compute the ratio g of the molar specific heats of air. (Hint: See Problem 91.)

M2 v1 # , v2 A M1

where v1 is the speed of sound in a gas of molar mass M1 and v2 is the speed of sound in a gas of molar mass M2. (Hint: See Problem 91.)

A

p

94 From the knowledge that CV, the molar specific heat at constant volume, for a gas in a container is 5.0R, calculate the ratio of the speed of sound in that gas to the rms speed of the molecules, for gas temperature T. (Hint: See Problem 91.)

B

E D

C

V

Figure 19-29 Problem 87. 88 An ideal gas initially at 300 K is compressed at a constant pressure of 25 N/m2 from a volume of 3.0 m3 to a volume of 1.8 m3. In the process, 75 J is lost by the gas as heat.What are (a) the change in internal energy of the gas and (b) the final temperature of the gas? 89 A pipe of length L # 25.0 m that is open at one end contains air at atmospheric pressure. It is thrust vertically into a freshwater lake

95 The molar mass of iodine is 127 g/mol. When sound at frequency 1000 Hz is introduced to a tube of iodine gas at 400 K, an internal acoustic standing wave is set up with nodes separated by 9.57 cm. What is g for the gas? (Hint: See Problem 91.) 96 For air near 0-C, by how much does the speed of sound increase for each increase in air temperature by 1 C-? (Hint: See Problem 91.) 97 Two containers are at the same temperature. The gas in the first container is at pressure p1 and has molecules with mass m1 and root-mean-square speed vrms1. The gas in the second is at pressure 2p1 and has molecules with mass m2 and average speed vavg2 # 2vrms1. Find the ratio m1/m2 of the masses of their molecules.

C

H

A

P

T

E

R

2

0

Entropy and the Second Law of Thermodynamics 20-1 ENTROPY Learning Objectives After reading this module, you should be able to . . .

20.01 Identify the second law of thermodynamics: If a process occurs in a closed system, the entropy of the system increases for irreversible processes and remains constant for reversible processes; it never decreases. 20.02 Identify that entropy is a state function (the value for a particular state of the system does not depend on how that state is reached). 20.03 Calculate the change in entropy for a process by integrating the inverse of the temperature (in kelvins) with respect to the heat Q transferred during the process. 20.04 For a phase change with a constant temperature process, apply the relationship between the entropy change %S, the total transferred heat Q, and the temperature T (in kelvins).

20.05 For a temperature change %T that is small relative to the temperature T, apply the relationship between the entropy change %S, the transferred heat Q, and the average temperature Tavg (in kelvins). 20.06 For an ideal gas, apply the relationship between the entropy change %S and the initial and final values of the pressure and volume. 20.07 Identify that if a process is an irreversible one, the integration for the entropy change must be done for a reversible process that takes the system between the same initial and final states as the irreversible process. 20.08 For stretched rubber, relate the elastic force to the rate at which the rubber’s entropy changes with the change in the stretching distance.

Key Ideas ● An irreversible process is one that cannot be reversed by means of small changes in the environment. The direction in which an irreversible process proceeds is set by the change in entropy %S of the system undergoing the process. Entropy S is a state property (or state function) of the system; that is, it depends only on the state of the system and not on the way in which the system reached that state. The entropy postulate states (in part): If an irreversible process occurs in a closed system, the entropy of the system always increases. ● The entropy change %S for an irreversible process that takes a system from an initial state i to a final state f is exactly equal to the entropy change %S for any reversible process that takes the system between those same two states. We can compute the latter (but not the former) with f dQ %S # Sf $ Si # . i T Here Q is the energy transferred as heat to or from the system during the process, and T is the temperature of the system in kelvins during the process.

"

● For a reversible isothermal process, the expression for an entropy change reduces to

Q . T ● When the temperature change %T of a system is small relative to the temperature (in kelvins) before and after the process, the entropy change can be approximated as Q , %S # Sf $ Si % Tavg %S # Sf $ Si #

where Tavg is the system’s average temperature during the process. ● When an ideal gas changes reversibly from an initial state with temperature Ti and volume Vi to a final state with temperature Tf and volume Vf , the change %S in the entropy of the gas is Vf Tf %S # Sf $ Si # nR ln " nCV ln . Vi Ti ● The second law of thermodynamics, which is an extension of the entropy postulate, states: If a process occurs in a closed system, the entropy of the system increases for irreversible processes and remains constant for reversible processes. It never decreases. In equation form, %S 0 0.

583

584

CHAPTE R 20 E NTROPY AN D TH E SECON D L AW OF TH E R M ODYNAM ICS

What Is Physics? Time has direction, the direction in which we age. We are accustomed to many one-way processes — that is, processes that can occur only in a certain sequence (the right way) and never in the reverse sequence (the wrong way). An egg is dropped onto a floor, a pizza is baked, a car is driven into a lamppost, large waves erode a sandy beach — these one-way processes are irreversible, meaning that they cannot be reversed by means of only small changes in their environment. One goal of physics is to understand why time has direction and why oneway processes are irreversible. Although this physics might seem disconnected from the practical issues of everyday life, it is in fact at the heart of any engine, such as a car engine, because it determines how well an engine can run. The key to understanding why one-way processes cannot be reversed involves a quantity known as entropy.

Irreversible Processes and Entropy The one-way character of irreversible processes is so pervasive that we take it for granted. If these processes were to occur spontaneously (on their own) in the wrong way, we would be astonished. Yet none of these wrong-way events would violate the law of conservation of energy. For example, if you were to wrap your hands around a cup of hot coffee, you would be astonished if your hands got cooler and the cup got warmer. That is obviously the wrong way for the energy transfer, but the total energy of the closed system (hands " cup of coffee) would be the same as the total energy if the process had run in the right way. For another example, if you popped a helium balloon, you would be astonished if, later, all the helium molecules were to gather together in the original shape of the balloon. That is obviously the wrong way for molecules to spread, but the total energy of the closed system (molecules " room) would be the same as for the right way. Thus, changes in energy within a closed system do not set the direction of irreversible processes. Rather, that direction is set by another property that we shall discuss in this chapter — the change in entropy %S of the system. The change in entropy of a system is defined later in this module, but we can here state its central property, often called the entropy postulate: If an irreversible process occurs in a closed system, the entropy S of the system always increases; it never decreases.

Entropy differs from energy in that entropy does not obey a conservation law. The energy of a closed system is conserved; it always remains constant. For irreversible processes, the entropy of a closed system always increases. Because of this property, the change in entropy is sometimes called “the arrow of time.” For example, we associate the explosion of a popcorn kernel with the forward direction of time and with an increase in entropy. The backward direction of time (a videotape run backwards) would correspond to the exploded popcorn reforming the original kernel. Because this backward process would result in an entropy decrease, it never happens. There are two equivalent ways to define the change in entropy of a system: (1) in terms of the system’s temperature and the energy the system gains or loses as heat, and (2) by counting the ways in which the atoms or molecules that make up the system can be arranged. We use the first approach in this module and the second in Module 20-4.

20-1 E NTROPY

Change in Entropy

"

f

i

dQ T

(change in entropy defined).

Vacuum

Insulation (a) Initial state i Irreversible process Stopcock open

(20-1)

Here Q is the energy transferred as heat to or from the system during the process, and T is the temperature of the system in kelvins. Thus, an entropy change depends not only on the energy transferred as heat but also on the temperature at which the transfer takes place. Because T is always positive, the sign of %S is the same as that of Q. We see from Eq. 20-1 that the SI unit for entropy and entropy change is the joule per kelvin. There is a problem, however, in applying Eq. 20-1 to the free expansion of Fig. 20-1. As the gas rushes to fill the entire container, the pressure, temperature, and volume of the gas fluctuate unpredictably. In other words, they do not have a sequence of well-defined equilibrium values during the intermediate stages of the change from initial state i to final state f. Thus, we cannot trace a pressure – volume path for the free expansion on the p-V plot of Fig. 20-2, and we cannot find a relation between Q and T that allows us to integrate as Eq. 20-1 requires. However, if entropy is truly a state property, the difference in entropy between states i and f must depend only on those states and not at all on the way the system went from one state to the other. Suppose, then, that we replace the irreversible free expansion of Fig. 20-1 with a reversible process that connects states i and f. With a reversible process we can trace a pressure – volume path on a p-V plot, and we can find a relation between Q and T that allows us to use Eq. 20-1 to obtain the entropy change. We saw in Module 19-9 that the temperature of an ideal gas does not change during a free expansion: Ti # Tf # T. Thus, points i and f in Fig. 20-2 must be on the same isotherm. A convenient replacement process is then a reversible isothermal expansion from state i to state f, which actually proceeds along that isotherm. Furthermore, because T is constant throughout a reversible isothermal expansion, the integral of Eq. 20-1 is greatly simplified. Figure 20-3 shows how to produce such a reversible isothermal expansion. We confine the gas to an insulated cylinder that rests on a thermal reservoir maintained at the temperature T. We begin by placing just enough lead shot on the movable piston so that the pressure and volume of the gas are those of the initial state i of Fig. 20-1a. We then remove shot slowly (piece by piece) until the pressure and volume of the gas are those of the final state f of Fig. 20-1b. The temperature of the gas does not change because the gas remains in thermal contact with the reservoir throughout the process. The reversible isothermal expansion of Fig. 20-3 is physically quite different from the irreversible free expansion of Fig. 20-1. However, both processes have the same initial state and the same final state and thus must have the same change in

(b) Final state f

Figure 20-1 The free expansion of an ideal gas. (a) The gas is confined to the left half of an insulated container by a closed stopcock. (b) When the stopcock is opened, the gas rushes to fill the entire container. This process is irreversible; that is, it does not occur in reverse, with the gas spontaneously collecting itself in the left half of the container.

i Pressure

%S # Sf $ Si #

Stopcock closed

System

Let’s approach this definition of change in entropy by looking again at a process that we described in Modules 18-5 and 19-9: the free expansion of an ideal gas. Figure 20-1a shows the gas in its initial equilibrium state i, confined by a closed stopcock to the left half of a thermally insulated container. If we open the stopcock, the gas rushes to fill the entire container, eventually reaching the final equilibrium state f shown in Fig. 20-1b. This is an irreversible process; all the molecules of the gas will never return to the left half of the container. The p-V plot of the process, in Fig. 20-2, shows the pressure and volume of the gas in its initial state i and final state f. Pressure and volume are state properties, properties that depend only on the state of the gas and not on how it reached that state. Other state properties are temperature and energy. We now assume that the gas has still another state property — its entropy. Furthermore, we define the change in entropy Sf $ Si of a system during a process that takes the system from an initial state i to a final state f as

585

f

Volume

Figure 20-2 A p-V diagram showing the initial state i and the final state f of the free expansion of Fig. 20-1. The intermediate states of the gas cannot be shown because they are not equilibrium states.

586

CHAPTE R 20 E NTROPY AN D TH E SECON D L AW OF TH E R M ODYNAM ICS

entropy. Because we removed the lead shot slowly, the intermediate states of the gas are equilibrium states, so we can plot them on a p-V diagram (Fig. 20-4). To apply Eq. 20-1 to the isothermal expansion, we take the constant temperature T outside the integral, obtaining

Insulation

Lead shot

%S # Sf $ Si #

1 T

"

f

i

dQ.

Because * dQ # Q, where Q is the total energy transferred as heat during the process, we have Q (change in entropy, isothermal process). (20-2) T To keep the temperature T of the gas constant during the isothermal expansion of Fig. 20-3, heat Q must have been energy transferred from the reservoir to the gas. Thus, Q is positive and the entropy of the gas increases during the isothermal process and during the free expansion of Fig. 20-1. To summarize: %S # Sf $ Si #

Q T Thermal reservoir

Control knob

(a) Initial state i

Reversible process

To find the entropy change for an irreversible process, replace that process with any reversible process that connects the same initial and final states. Calculate the entropy change for this reversible process with Eq. 20-1.

When the temperature change %T of a system is small relative to the temperature (in kelvins) before and after the process, the entropy change can be approximated as

Lead shot

%S # Sf $ Si %

Q , Tavg

(20-3)

where Tavg is the average temperature of the system in kelvins during the process.

Checkpoint 1 Water is heated on a stove. Rank the entropy changes of the water as its temperature rises (a) from 20-C to 30-C, (b) from 30-C to 35-C, and (c) from 80-C to 85-C, greatest first.

Entropy as a State Function

T (b) Final state f

Figure 20-3 The isothermal expansion of an ideal gas, done in a reversible way. The gas has the same initial state i and same final state f as in the irreversible process of Figs. 20-1 and 20-2.

dEint # dQ $ dW.

Isotherm

Pressure

i

We have assumed that entropy, like pressure, energy, and temperature, is a property of the state of a system and is independent of how that state is reached. That entropy is indeed a state function (as state properties are usually called) can be deduced only by experiment. However, we can prove it is a state function for the special and important case in which an ideal gas is taken through a reversible process. To make the process reversible, it is done slowly in a series of small steps, with the gas in an equilibrium state at the end of each step. For each small step, the energy transferred as heat to or from the gas is dQ, the work done by the gas is dW, and the change in internal energy is dEint. These are related by the first law of thermodynamics in differential form (Eq. 18-27):

f

T

Because the steps are reversible, with the gas in equilibrium states, we can use Eq. 18-24 to replace dW with p dV and Eq. 19-45 to replace dEint with nCV dT. Solving for dQ then leads to dQ # p dV " nCV dT.

Volume

Figure 20-4 A p-V diagram for the reversible isothermal expansion of Fig. 20-3. The intermediate states, which are now equilibrium states, are shown.

Using the ideal gas law, we replace p in this equation with nRT/V. Then we divide each term in the resulting equation by T, obtaining dQ dT dV # nR " nCV . T V T

587

20-1 E NTROPY

Now let us integrate each term of this equation between an arbitrary initial state i and an arbitrary final state f to get

"

f

i

dQ # T

"

f

i

nR

dV " V

"

f

i

nCV

dT . T

The quantity on the left is the entropy change %S (# Sf $ Si) defined by Eq. 20-1. Substituting this and integrating the quantities on the right yield %S # Sf $ Si # nR ln

Vf Tf " nCV ln . Vi Ti

(20-4)

Note that we did not have to specify a particular reversible process when we integrated. Therefore, the integration must hold for all reversible processes that take the gas from state i to state f. Thus, the change in entropy %S between the initial and final states of an ideal gas depends only on properties of the initial state (Vi and Ti) and properties of the final state (Vf and Tf); %S does not depend on how the gas changes between the two states.

Checkpoint 2 Pressure

a T2

An ideal gas has temperature T1 at the initial state i shown in the p-V diagram here. The gas has a higher temperature T2 at final states a and b, which it can reach along the paths shown. Is the entropy change along the path to state a larger than, smaller than, or the same as that along the path to state b?

i

T1 b

T2

Volume

Sample Problem 20.01 Entropy change of two blocks coming to thermal equilibrium Figure 20-5a shows two identical copper blocks of mass m # 1.5 kg: block L at temperature TiL # 60-C and block R at temperature TiR # 20-C. The blocks are in a thermally insulated box and are separated by an insulating shutter. When we lift the shutter, the blocks eventually come to the equilibrium temperature Tf # 40-C (Fig. 20-5b). What is the net entropy change of the two-block system during this irreversible process? The specific heat of copper is 386 J/kg 9K. KEY IDEA To calculate the entropy change, we must find a reversible process that takes the system from the initial state of Fig. 20-5a to the final state of Fig. 20-5b. We can calculate the net entropy change %Srev of the reversible process using Eq. 20-1, and then the entropy change for the irreversible process is equal to %Srev. Calculations: For the reversible process, we need a thermal reservoir whose temperature can be changed slowly (say, by turning a knob). We then take the blocks through the following two steps, illustrated in Fig. 20-6. Step 1: With the reservoir’s temperature set at 60-C, put block L on the reservoir. (Since block and reservoir are at the same temperature, they are already in thermal equilib-

Movable shutter Insulation Warm TiL

Cool TiR

Tf

Tf

L

R

L

R

Irreversible process

(a)

(b)

Figure 20-5 (a) In the initial state, two copper blocks L and R, identical except for their temperatures, are in an insulating box and are separated by an insulating shutter. (b) When the shutter is removed, the blocks exchange energy as heat and come to a final state, both with the same temperature Tf . Insulation R

L Q Reservoir (a) Step 1

Q

(b) Step 2

Figure 20-6 The blocks of Fig. 20-5 can proceed from their initial state to their final state in a reversible way if we use a reservoir with a controllable temperature (a) to extract heat reversibly from block L and (b) to add heat reversibly to block R.

588

CHAPTE R 20 E NTROPY AN D TH E SECON D L AW OF TH E R M ODYNAM ICS

rium.) Then slowly lower the temperature of the reservoir and the block to 40-C. As the block’s temperature changes by each increment dT during this process, energy dQ is transferred as heat from the block to the reservoir. Using Eq. 18-14, we can write this transferred energy as dQ # mc dT, where c is the specific heat of copper. According to Eq. 20-1, the entropy change %SL of block L during the full temperature change from initial temperature TiL (# 60-C # 333 K) to final temperature Tf (# 40-C # 313 K) is

"

f

%SL #

i

dQ # T

"

Tf

mc dT # mc T TiL

"

Tf

dT TiL T

Tf

%SL # (1.5 kg)(386 J/kg9K) ln # $35.86 J/K.

%SR # (1.5 kg)(386 J/kg9K) ln

313 K 293 K

# "38.23 J/K. The net entropy change %Srev of the two-block system undergoing this two-step reversible process is then %Srev # %SL " %SR # $35.86 J/K " 38.23 J/K # 2.4 J/K.

. TiL Inserting the given data yields # mc ln

put block R on the reservoir. Then slowly raise the temperature of the reservoir and the block to 40-C. With the same reasoning used to find %SL, you can show that the entropy change %SR of block R during this process is

313 K 333 K

Step 2: With the reservoir’s temperature now set at 20-C,

Thus, the net entropy change %Sirrev for the two-block system undergoing the actual irreversible process is %Sirrev # %Srev # 2.4 J/K.

(Answer)

This result is positive, in accordance with the entropy postulate.

Sample Problem 20.02 Entropy change of a free expansion of a gas Suppose 1.0 mol of nitrogen gas is confined to the left side of the container of Fig. 20-1a. You open the stopcock, and the volume of the gas doubles. What is the entropy change of the gas for this irreversible process? Treat the gas as ideal.

in which n is the number of moles of gas present. From Eq. 20-2 the entropy change for this reversible process in which the temperature is held constant is %Srev #

KEY IDEAS (1) We can determine the entropy change for the irreversible process by calculating it for a reversible process that provides the same change in volume. (2) The temperature of the gas does not change in the free expansion. Thus, the reversible process should be an isothermal expansion — namely, the one of Figs. 20-3 and 20-4. Calculations: From Table 19-4, the energy Q added as heat to the gas as it expands isothermally at temperature T from an initial volume Vi to a final volume Vf is Vf Q # nRT ln , Vi

nRT ln(Vf /Vi ) Vf Q # # nR ln . T T Vi

Substituting n # 1.00 mol and Vf /Vi # 2, we find %Srev # nR ln

Vf Vi

# (1.00 mol)(8.31 J/mol 9K)(ln 2)

# "5.76 J/K. Thus, the entropy change for the free expansion (and for all other processes that connect the initial and final states shown in Fig. 20-2) is %Sirrev # %Srev # "5.76 J/K.

(Answer)

Because %S is positive, the entropy increases, in accordance with the entropy postulate.

Additional examples, video, and practice available at WileyPLUS

The Second Law of Thermodynamics Here is a puzzle. In the process of going from (a) to (b) in Fig. 20-3, the entropy change of the gas (our system) is positive. However, because the process is reversible, we can also go from (b) to (a) by, say, gradually adding lead shot to the piston, to restore the initial gas volume. To maintain a constant temperature, we need to remove energy as heat, but that means Q is negative and thus the entropy change is also. Doesn’t this entropy decrease violate the entropy postulate: en-

20-1 E NTROPY

tropy always increases? No, because the postulate holds only for irreversible processes in closed systems. Here, the process is not irreverible and the system is not closed (because of the energy transferred to and from the reservoir as heat). However, if we include the reservoir, along with the gas, as part of the system, then we do have a closed system. Let’s check the change in entropy of the enlarged system gas " reservoir for the process that takes it from (b) to (a) in Fig. 20-3. During this reversible process, energy is transferred as heat from the gas to the reservoir — that is, from one part of the enlarged system to another. Let +Q+ represent the absolute value (or magnitude) of this heat. With Eq. 20-2, we can then calculate separately the entropy changes for the gas (which loses +Q+) and the reservoir (which gains +Q+). We get !Q! T !Q! . #" T

%Sgas # $ and

%Sres

The entropy change of the closed system is the sum of these two quantities: 0. With this result, we can modify the entropy postulate to include both reversible and irreversible processes: If a process occurs in a closed system, the entropy of the system increases for irreversible processes and remains constant for reversible processes. It never decreases.

Although entropy may decrease in part of a closed system, there will always be an equal or larger entropy increase in another part of the system, so that the entropy of the system as a whole never decreases. This fact is one form of the second law of thermodynamics and can be written as %S 0 0

(second law of thermodynamics),

(20-5)

where the greater-than sign applies to irreversible processes and the equals sign to reversible processes. Equation 20-5 applies only to closed systems. In the real world almost all processes are irreversible to some extent because of friction, turbulence, and other factors, so the entropy of real closed systems undergoing real processes always increases. Processes in which the system’s entropy remains constant are always idealizations.

Force Due to Entropy To understand why rubber resists being stretched, let’s write the first law of thermodynamics dE # dQ $ dW for a rubber band undergoing a small increase in length dx as we stretch it between our hands. The force from the rubber band has magnitude F, is directed inward, and does work dW # $F dx during length increase dx. From Eq. 20-2 (%S # Q/T ), small changes in Q and S at constant temperature are related by dS # dQ/T, or dQ # T dS. So, now we can rewrite the first law as dE # T dS " F dx.

(20-6)

To good approximation, the change dE in the internal energy of rubber is 0 if the total stretch of the rubber band is not very much. Substituting 0 for dE in Eq. 20-6 leads us to an expression for the force from the rubber band: F # $T

dS . dx

(20-7)

589

590

CHAPTE R 20 E NTROPY AN D TH E SECON D L AW OF TH E R M ODYNAM ICS

Coiled

(a)

Uncoiled (b)

F

F

Figure 20-7 A section of a rubber band (a) unstretched and (b) stretched, and a polymer within it (a) coiled and (b) uncoiled.

This tells us that F is proportional to the rate dS/dx at which the rubber band’s entropy changes during a small change dx in the rubber band’s length. Thus, you can feel the effect of entropy on your hands as you stretch a rubber band. To make sense of the relation between force and entropy, let’s consider a simple model of the rubber material. Rubber consists of cross-linked polymer chains (long molecules with cross links) that resemble three-dimensional zig-zags (Fig. 20-7). When the rubber band is at its rest length, the polymers are coiled up in a spaghetti-like arrangement. Because of the large disorder of the molecules, this rest state has a high value of entropy. When we stretch a rubber band, we uncoil many of those polymers, aligning them in the direction of stretch. Because the alignment decreases the disorder, the entropy of the stretched rubber band is less. That is, the change dS/dx in Eq. 20-7 is a negative quantity because the entropy decreases with stretching. Thus, the force on our hands from the rubber band is due to the tendency of the polymers to return to their former disordered state and higher value of entropy.

20-2 ENTROPY IN THE REAL WORLD: ENGINES Learning Objectives After reading this module, you should be able to . . .

20.09 Identify that a heat engine is a device that extracts energy from its environment in the form of heat and does useful work and that in an ideal heat engine, all processes are reversible, with no wasteful energy transfers. 20.10 Sketch a p-V diagram for the cycle of a Carnot engine, indicating the direction of cycling, the nature of the processes involved, the work done during each process (including algebraic sign), the net work done in the cycle, and the heat transferred during each process (including algebraic sign). 20.11 Sketch a Carnot cycle on a temperature–entropy diagram, indicating the heat transfers.

20.12 Determine the net entropy change around a Carnot cycle. 20.13 Calculate the efficiency ´C of a Carnot engine in terms of the heat transfers and also in terms of the temperatures of the reservoirs. 20.14 Identify that there are no perfect engines in which the energy transferred as heat Q from a high temperature reservoir goes entirely into the work W done by the engine. 20.15 Sketch a p-V diagram for the cycle of a Stirling engine, indicating the direction of cycling, the nature of the processes involved, the work done during each process (including algebraic sign), the net work done in the cycle, and the heat transfers during each process.

Key Ideas ● An engine is a device that, operating in a cycle, extracts energy as heat +QH+ from a high-temperature reservoir and does a certain amount of work +W+. The efficiency ´ of any engine is defined as !W ! energy we get # . ´# energy we pay for !QH ! ● In an ideal engine, all processes are reversible and no waste-

ful energy transfers occur due to, say, friction and turbulence. ● A Carnot engine is an ideal engine that follows the cycle of

Fig. 20-9. Its efficiency is ´C # 1 $

TL !QL ! # 1$ , !QH ! TH

in which TH and TL are the temperatures of the high- and lowtemperature reservoirs, respectively. Real engines always have an efficiency lower than that of a Carnot engine. Ideal engines that are not Carnot engines also have efficiencies lower than that of a Carnot engine. ● A perfect engine is an imaginary engine in which energy extracted as heat from the high-temperature reservoir is converted completely to work. Such an engine would violate the second law of thermodynamics, which can be restated as follows: No series of processes is possible whose sole result is the absorption of energy as heat from a thermal reservoir and the complete conversion of this energy to work.

Entropy in the Real World: Engines A heat engine, or more simply, an engine, is a device that extracts energy from its environment in the form of heat and does useful work. At the heart of every engine is a working substance. In a steam engine, the working substance is water,

591

20-2 E NTROPY I N TH E R EAL WOR LD: E NG I N ES

in both its vapor and its liquid form. In an automobile engine the working substance is a gasoline – air mixture. If an engine is to do work on a sustained basis, the working substance must operate in a cycle; that is, the working substance must pass through a closed series of thermodynamic processes, called strokes, returning again and again to each state in its cycle. Let us see what the laws of thermodynamics can tell us about the operation of engines.

Schematic of a Carnot engine TH QH Heat is absorbed.

A Carnot Engine We have seen that we can learn much about real gases by analyzing an ideal gas, which obeys the simple law pV # nRT. Although an ideal gas does not exist, any real gas approaches ideal behavior if its density is low enough. Similarly, we can study real engines by analyzing the behavior of an ideal engine. In an ideal engine, all processes are reversible and no wasteful energy transfers occur due to, say, friction and turbulence.

We shall focus on a particular ideal engine called a Carnot engine after the French scientist and engineer N. L. Sadi Carnot (pronounced “car-no”), who first proposed the engine’s concept in 1824. This ideal engine turns out to be the best (in principle) at using energy as heat to do useful work. Surprisingly, Carnot was able to analyze the performance of this engine before the first law of thermodynamics and the concept of entropy had been discovered. Figure 20-8 shows schematically the operation of a Carnot engine. During each cycle of the engine, the working substance absorbs energy +QH+ as heat from a thermal reservoir at constant temperature TH and discharges energy +QL+ as heat to a second thermal reservoir at a constant lower temperature TL. Figure 20-9 shows a p-V plot of the Carnot cycle — the cycle followed by the working substance. As indicated by the arrows, the cycle is traversed in the clockwise direction. Imagine the working substance to be a gas, confined to an insulating cylinder with a weighted, movable piston. The cylinder may be placed at will on either of the two thermal reservoirs, as in Fig. 20-6, or on an insulating slab. Figure 20-9a shows that, if we place the cylinder in contact with the hightemperature reservoir at temperature TH, heat +QH+ is transferred to the working substance from this reservoir as the gas undergoes an isothermal expansion from volume Va to volume Vb. Similarly, with the working substance in contact with the low-temperature reservoir at temperature TL, heat +QL+ is transferred from

Heat is lost.

Work is done by the engine.

QL TL

Figure 20-8 The elements of a Carnot engine. The two black arrowheads on the central loop suggest the working substance operating in a cycle, as if on a p-V plot. Energy +QH+ is transferred as heat from the high-temperature reservoir at temperature TH to the working substance. Energy +QL+ is transferred as heat from the working substance to the low-temperature reservoir at temperature TL. Work W is done by the engine (actually by the working substance) on something in the environment.

A Isothermal: heat is absorbed

Pressure

a

QH

b

Positive work is done.

a

Adiabatic: no heat

Pressure

Stages of a Carnot engine

Figure 20-9 A pressure – volume plot of the cycle followed by the working substance of the Carnot engine in Fig. 20-8. The cycle consists of two isothermal (ab and cd) and two adiabatic processes (bc and da). The shaded area enclosed by the cycle is equal to the work W per cycle done by the Carnot engine.

W

b

W

W TH

d c

0

0

Adiabatic: no heat (a)

c

TL

Volume

TH

QL

d

Volume

Negative work is done. (b)

TL

Isothermal: heat is lost

592

CHAPTE R 20 E NTROPY AN D TH E SECON D L AW OF TH E R M ODYNAM ICS

the working substance to the low-temperature reservoir as the gas undergoes an isothermal compression from volume Vc to volume Vd (Fig. 20-9b). In the engine of Fig. 20-8, we assume that heat transfers to or from the working substance can take place only during the isothermal processes ab and cd of Fig. 20-9. Therefore, processes bc and da in that figure, which connect the two isotherms at temperatures TH and TL, must be (reversible) adiabatic processes; that is, they must be processes in which no energy is transferred as heat. To ensure this, during processes bc and da the cylinder is placed on an insulating slab as the volume of the working substance is changed. During the processes ab and bc of Fig. 20-9a, the working substance is expanding and thus doing positive work as it raises the weighted piston. This work is represented in Fig. 20-9a by the area under curve abc. During the processes cd and da (Fig. 20-9b), the working substance is being compressed, which means that it is doing negative work on its environment or, equivalently, that its environment is doing work on it as the loaded piston descends. This work is represented by the area under curve cda. The net work per cycle, which is represented by W in both Figs. 20-8 and 20-9, is the difference between these two areas and is a positive quantity equal to the area enclosed by cycle abcda in Fig. 20-9. This work W is performed on some outside object, such as a load to be lifted. Equation 20-1 (%S # * dQ/T ) tells us that any energy transfer as heat must involve a change in entropy. To see this for a Carnot engine, we can plot the Carnot cycle on a temperature – entropy (T-S) diagram as in Fig. 20-10. The lettered points a, b, c, and d there correspond to the lettered points in the p-V diagram in Fig. 20-9. The two horizontal lines in Fig. 20-10 correspond to the two isothermal processes of the cycle. Process ab is the isothermal expansion of the cycle. As the working substance (reversibly) absorbs energy +QH+ as heat at constant temperature TH during the expansion, its entropy increases. Similarly, during the isothermal compression cd, the working substance (reversibly) loses energy +QL+ as heat at constant temperature TL, and its entropy decreases. The two vertical lines in Fig. 20-10 correspond to the two adiabatic processes of the Carnot cycle. Because no energy is transferred as heat during the two processes, the entropy of the working substance is constant during them. The Work To calculate the net work done by a Carnot engine during a cycle, let us apply Eq. 18-26, the first law of thermodynamics (%Eint # Q $ W ), to the working substance. That substance must return again and again to any arbitrarily selected state in the cycle. Thus, if X represents any state property of the working substance, such as pressure, temperature, volume, internal energy, or entropy, we must have %X # 0 for every cycle. It follows that %Eint # 0 for a complete cycle of the working substance. Recalling that Q in Eq. 18-26 is the net heat transfer per cycle and W is the net work, we can write the first law of thermodynamics for the Carnot cycle as W # !QH !$ !QL !.

Temperature T

QH a

b

d

c

TH

TL

QL Entropy S

Figure 20-10 The Carnot cycle of Fig. 20-9 plotted on a temperature – entropy diagram. During processes ab and cd the temperature remains constant. During processes bc and da the entropy remains constant.

(20-8)

Entropy Changes In a Carnot engine, there are two (and only two) reversible energy transfers as heat, and thus two changes in the entropy of the working substance — one at temperature TH and one at TL. The net entropy change per cycle is then !QH! !QL! $ . (20-9) %S # %SH " %SL # TH TL Here %SH is positive because energy +QH+ is added to the working substance as heat (an increase in entropy) and %SL is negative because energy +QL+ is removed from the working substance as heat (a decrease in entropy). Because entropy is a state function, we must have %S # 0 for a complete cycle. Putting %S # 0 in Eq. 20-9 requires that !QL! !QH! (20-10) # . TH TL Note that, because TH 4 TL, we must have +QH+ 4 +QL+; that is, more energy is

20-2 E NTROPY I N TH E R EAL WOR LD: E NG I N ES

extracted as heat from the high-temperature reservoir than is delivered to the low-temperature reservoir. We shall now derive an expression for the efficiency of a Carnot engine.

TH QH

Efficiency of a Carnot Engine The purpose of any engine is to transform as much of the extracted energy QH into work as possible. We measure its success in doing so by its thermal efficiency ´, defined as the work the engine does per cycle (“energy we get”) divided by the energy it absorbs as heat per cycle (“energy we pay for”): !W! energy we get ´# # energy we pay for !QH!

Perfect engine: total conversion of heat to work

W (= Q H)

QL=0 (efficiency, any engine).

(20-11)

For a Carnot engine we can substitute for W from Eq. 20-8 to write Eq. 20-11 as ´C #

593

!QH! $ !QL! !QL! #1$ . QH !QH!

Figure 20-11 The elements of a perfect engine — that is, one that converts heat QH from a high-temperature reservoir directly to work W with 100% efficiency.

(20-12)

Using Eq. 20-10 we can write this as ´C # 1 $

TL TH

(efficiency, Carnot engine),

(20-13)

where the temperatures TL and TH are in kelvins. Because TL 8 TH, the Carnot engine necessarily has a thermal efficiency less than unity — that is, less than 100%. This is indicated in Fig. 20-8, which shows that only part of the energy extracted as heat from the high-temperature reservoir is available to do work, and the rest is delivered to the low-temperature reservoir. We shall show in Module 20-3 that no real engine can have a thermal efficiency greater than that calculated from Eq. 20-13. Inventors continually try to improve engine efficiency by reducing the energy +QL+ that is “thrown away” during each cycle. The inventor’s dream is to produce the perfect engine, diagrammed in Fig. 20-11, in which +QL+ is reduced to zero and +QH+ is converted completely into work. Such an engine on an ocean liner, for example, could extract energy as heat from the water and use it to drive the propellers, with no fuel cost. An automobile fitted with such an engine could extract energy as heat from the surrounding air and use it to drive the car, again with no fuel cost. Alas, a perfect engine is only a dream: Inspection of Eq. 20-13 shows that we can achieve 100% engine efficiency (that is, ´ # 1) only if TL # 0 or TH : 6, impossible requirements. Instead, experience gives the following alternative version of the second law of thermodynamics, which says in short, there are no perfect engines: No series of processes is possible whose sole result is the transfer of energy as heat from a thermal reservoir and the complete conversion of this energy to work. © Richard Ustinich

To summarize: The thermal efficiency given by Eq. 20-13 applies only to Carnot engines. Real engines, in which the processes that form the engine cycle are not reversible, have lower efficiencies. If your car were powered by a Carnot engine, it would have an efficiency of about 55% according to Eq. 20-13; its actual efficiency is probably about 25%. A nuclear power plant (Fig. 20-12), taken in its entirety, is an engine. It extracts energy as heat from a reactor core, does work by means of a turbine, and discharges energy as heat to a nearby river. If the power plant operated as a Carnot engine, its efficiency would be about 40%; its actual efficiency is about 30%. In designing engines of any type, there is simply no way to beat the efficiency limitation imposed by Eq. 20-13.

Figure 20-12 The North Anna nuclear power plant near Charlottesville, Virginia, which generates electric energy at the rate of 900 MW. At the same time, by design, it discards energy into the nearby river at the rate of 2100 MW. This plant and all others like it throw away more energy than they deliver in useful form. They are real counterparts of the ideal engine of Fig. 20-8.

594

CHAPTE R 20 E NTROPY AN D TH E SECON D L AW OF TH E R M ODYNAM ICS

Stirling Engine

Stages of a Stirling engine

Pressure

a

QH

Q

d

W

b Q

QL

TH

c TL

Va

Vb Volume

Figure 20-13 A p-V plot for the working substance of an ideal Stirling engine, with the working substance assumed for convenience to be an ideal gas.

Equation 20-13 applies not to all ideal engines but only to those that can be represented as in Fig. 20-9— that is, to Carnot engines. For example, Fig. 20-13 shows the operating cycle of an ideal Stirling engine. Comparison with the Carnot cycle of Fig. 20-9 shows that each engine has isothermal heat transfers at temperatures TH and TL. However, the two isotherms of the Stirling engine cycle are connected, not by adiabatic processes as for the Carnot engine but by constant-volume processes. To increase the temperature of a gas at constant volume reversibly from TL to TH (process da of Fig. 20-13) requires a transfer of energy as heat to the working substance from a thermal reservoir whose temperature can be varied smoothly between those limits. Also, a reverse transfer is required in process bc. Thus, reversible heat transfers (and corresponding entropy changes) occur in all four of the processes that form the cycle of a Stirling engine, not just two processes as in a Carnot engine. Thus, the derivation that led to Eq. 20-13 does not apply to an ideal Stirling engine. More important, the efficiency of an ideal Stirling engine is lower than that of a Carnot engine operating between the same two temperatures. Real Stirling engines have even lower efficiencies. The Stirling engine was developed in 1816 by Robert Stirling. This engine, long neglected, is now being developed for use in automobiles and spacecraft. A Stirling engine delivering 5000 hp (3.7 MW) has been built. Because they are quiet, Stirling engines are used on some military submarines.

Checkpoint 3 Three Carnot engines operate between reservoir temperatures of (a) 400 and 500 K, (b) 600 and 800 K, and (c) 400 and 600 K. Rank the engines according to their thermal efficiencies, greatest first.

Sample Problem 20.03 Carnot engine, efficiency, power, entropy changes Imagine a Carnot engine that operates between the temperatures TH # 850 K and TL # 300 K. The engine performs 1200 J of work each cycle, which takes 0.25 s.

(c) How much energy +QH+ is extracted as heat from the high-temperature reservoir every cycle?

(a) What is the efficiency of this engine?

KEY IDEA

KEY IDEA The efficiency ´ of a Carnot engine depends only on the ratio TL/TH of the temperatures (in kelvins) of the thermal reservoirs to which it is connected. Calculation: Thus, from Eq. 20-13, we have 300 K TL #1$ # 0.647 % 65%. TH 850 K (b) What is the average power of this engine? ´ # 1$

(Answer)

Calculation: Here we have 1200 J W # # 1855 J. (Answer) !QH! # ´ 0.647 (d) How much energy +QL+ is delivered as heat to the lowtemperature reservoir every cycle? KEY IDEA

KEY IDEA The average power P of an engine is the ratio of the work W it does per cycle to the time t that each cycle takes. Calculation: For this Carnot engine, we find W 1200 J P# # # 4800 W # 4.8 kW. t 0.25 s

The efficiency ´ is the ratio of the work W that is done per cycle to the energy +QH+ that is extracted as heat from the high-temperature reservoir per cycle (´ # W/+QH+).

(Answer)

For a Carnot engine, the work W done per cycle is equal to the difference in the energy transfers as heat: +QH+ $ +QL+, as in Eq. 20-8. Calculation: Thus, we have +QL+ # +QH+ $ W # 1855 J $ 1200 J # 655 J.

(Answer)

20-3 R E FR IG E RATORS AN D R EAL E NG I N ES

(e) By how much does the entropy of the working substance change as a result of the energy transferred to it from the high-temperature reservoir? From it to the low-temperature reservoir?

595

entropy of the working substance is %SH #

QH 1855 J # # "2.18 J/K. TH 850 K

(Answer)

Similarly, for the negative transfer of energy QL to the low-temperature reservoir at TL, we have

KEY IDEA The entropy change %S during a transfer of energy as heat Q at constant temperature T is given by Eq. 20-2 (%S # Q/T). Calculations: Thus, for the positive transfer of energy QH from the high-temperature reservoir at TH, the change in the

%SL #

QL $655 J # # $2.18 J/K. TL 300 K

(Answer)

Note that the net entropy change of the working substance for one cycle is zero, as we discussed in deriving Eq. 20-10.

Sample Problem 20.04 Impossibly efficient engine An inventor claims to have constructed an engine that has an efficiency of 75% when operated between the boiling and freezing points of water. Is this possible?

Calculation: From Eq. 20-13, we find that the efficiency of a Carnot engine operating between the boiling and freezing points of water is ´#1$

KEY IDEA The efficiency of a real engine must be less than the efficiency of a Carnot engine operating between the same two temperatures.

TL (0 " 273) K #1$ # 0.268 % 27%. TH (100 " 273) K

Thus, for the given temperatures, the claimed efficiency of 75% for a real engine (with its irreversible processes and wasteful energy transfers) is impossible.

Additional examples, video, and practice available at WileyPLUS

20-3 REFRIGERATORS AND REAL ENGINES Learning Objectives After reading this module, you should be able to . . .

20.16 Identify that a refrigerator is a device that uses work to transfer energy from a low-temperature reservoir to a high-temperature reservoir, and that an ideal refrigerator is one that does this with reversible processes and no wasteful losses. 20.17 Sketch a p-V diagram for the cycle of a Carnot refrigerator, indicating the direction of cycling, the nature of the processes involved, the work done during each process (including algebraic sign), the net work done in the cycle,

Key Ideas ● A refrigerator is a device that, operating in a cycle, has work W done on it as it extracts energy +QL+ as heat from a low-temperature reservoir. The coefficient of performance K of a refrigerator is defined as !QL ! what we want . # K# what we pay for !W ! ● A Carnot refrigerator is a Carnot engine operating in reverse. Its coefficient of performance is

KC #

!QL ! TL . # !QH ! $ !QL ! TH $ TL

and the heat transferred during each process (including algebraic sign). 20.18 Apply the relationship between the coefficient of performance K and the heat exchanges with the reservoirs and the temperatures of the reservoirs. 20.19 Identify that there is no ideal refrigerator in which all of the energy extracted from the low-temperature reservoir is transferred to the high-temperature reservoir. 20.20 Identify that the efficiency of a real engine is less than that of the ideal Carnot engine. A perfect refrigerator is an entirely imaginary refrigerator in which energy extracted as heat from the low-temperature reservoir is somehow converted completely to heat discharged to the high-temperature reservoir without any need for work. ●

●A

perfect refrigerator would violate the second law of thermodynamics, which can be restated as follows: No series of processes is possible whose sole result is the transfer of energy as heat from a reservoir at a given temperature to a reservoir at a higher temperature (without work being involved).

596

CHAPTE R 20 E NTROPY AN D TH E SECON D L AW OF TH E R M ODYNAM ICS

Entropy in the Real World: Refrigerators

Schematic of a refrigerator TH

Heat is lost.

QH

W

Work is done on the engine.

Heat is absorbed.

QL TL

Figure 20-14 The elements of a refrigerator. The two black arrowheads on the central loop suggest the working substance operating in a cycle, as if on a p-V plot. Energy is transferred as heat QL to the working substance from the low-temperature reservoir. Energy is transferred as heat QH to the high-temperature reservoir from the working substance. Work W is done on the refrigerator (on the working substance) by something in the environment.

A refrigerator is a device that uses work in order to transfer energy from a lowtemperature reservoir to a high-temperature reservoir as the device continuously repeats a set series of thermodynamic processes. In a household refrigerator, for example, work is done by an electrical compressor to transfer energy from the food storage compartment (a low-temperature reservoir) to the room (a hightemperature reservoir). Air conditioners and heat pumps are also refrigerators. For an air conditioner, the low-temperature reservoir is the room that is to be cooled and the high-temperature reservoir is the warmer outdoors. A heat pump is an air conditioner that can be operated in reverse to heat a room; the room is the high-temperature reservoir, and heat is transferred to it from the cooler outdoors. Let us consider an ideal refrigerator: In an ideal refrigerator, all processes are reversible and no wasteful energy transfers occur as a result of, say, friction and turbulence.

Figure 20-14 shows the basic elements of an ideal refrigerator. Note that its operation is the reverse of how the Carnot engine of Fig. 20-8 operates. In other words, all the energy transfers, as either heat or work, are reversed from those of a Carnot engine. We can call such an ideal refrigerator a Carnot refrigerator. The designer of a refrigerator would like to extract as much energy +QL+ as possible from the low-temperature reservoir (what we want) for the least amount of work +W+ (what we pay for).A measure of the efficiency of a refrigerator, then, is K#

!QL! what we want # what we pay for !W!

(coefficient of performance, any refrigerator),

(20-14)

where K is called the coefficient of performance. For a Carnot refrigerator, the first law of thermodynamics gives +W+ # +QH+ $ +QL+, where +QH+ is the magnitude of the energy transferred as heat to the high-temperature reservoir. Equation 20-14 then becomes KC # Perfect refrigerator: total transfer of heat from cold to hot without any work TH Q

Q TL

Figure 20-15 The elements of a perfect refrigerator — that is, one that transfers energy from a low-temperature reservoir to a high-temperature reservoir without any input of work.

!QL! . !QH ! $ !QL!

(20-15)

Because a Carnot refrigerator is a Carnot engine operating in reverse, we can combine Eq. 20-10 with Eq. 20-15; after some algebra we find KC #

TL TH $ TL

(coefficient of performance, Carnot refrigerator).

(20-16)

For typical room air conditioners, K % 2.5. For household refrigerators, K % 5. Perversely, the value of K is higher the closer the temperatures of the two reservoirs are to each other. That is why heat pumps are more effective in temperate climates than in very cold climates. It would be nice to own a refrigerator that did not require some input of work — that is, one that would run without being plugged in. Figure 20-15 represents another “inventor’s dream,” a perfect refrigerator that transfers energy as heat Q from a cold reservoir to a warm reservoir without the need for work. Because the unit operates in cycles, the entropy of the working substance does not change during a complete cycle. The entropies of the two reservoirs, however, do change: The entropy change for the cold reservoir is $+Q+/TL, and that for the warm reservoir is "+Q+/TH. Thus, the net entropy change for the entire system is %S # $

!Q! !Q! " . TL TH

20-3 R E FR IG E RATORS AN D R EAL E NG I N ES

Because TH 4 TL, the right side of this equation is negative and thus the net change in entropy per cycle for the closed system refrigerator " reservoirs is also negative. Because such a decrease in entropy violates the second law of thermodynamics (Eq. 20-5), a perfect refrigerator does not exist. (If you want your refrigerator to operate, you must plug it in.) Here, then, is another way to state the second law of thermodynamics: No series of processes is possible whose sole result is the transfer of energy as heat from a reservoir at a given temperature to a reservoir at a higher temperature.

In short, there are no perfect refrigerators.

Checkpoint 4 You wish to increase the coefficient of performance of an ideal refrigerator. You can do so by (a) running the cold chamber at a slightly higher temperature, (b) running the cold chamber at a slightly lower temperature, (c) moving the unit to a slightly warmer room, or (d) moving it to a slightly cooler room. The magnitudes of the temperature changes are to be the same in all four cases. List the changes according to the resulting coefficients of performance, greatest first.

The Efficiencies of Real Engines Let ´C be the efficiency of a Carnot engine operating between two given temperatures. Here we prove that no real engine operating between those temperatures can have an efficiency greater than ;C. If it could, the engine would violate the second law of thermodynamics. Let us assume that an inventor, working in her garage, has constructed an engine X, which she claims has an efficiency ´X that is greater than ´C : ´X 4 ´C

(20-17)

(a claim).

Let us couple engine X to a Carnot refrigerator, as in Fig. 20-16a. We adjust the strokes of the Carnot refrigerator so that the work it requires per cycle is just equal to that provided by engine X. Thus, no (external) work is performed on or by the combination engine " refrigerator of Fig. 20-16a, which we take as our system. If Eq. 20-17 is true, from the definition of efficiency (Eq. 20-11), we must have !W! !W! 4 , !Q,H! !QH! where the prime refers to engine X and the right side of the inequality is the efficiency of the Carnot refrigerator when it operates as an engine. This inequality requires that !QH ! 4 !Q,H !. (20-18) TH

Figure 20-16 (a) Engine X drives a Carnot refrigerator. (b) If, as claimed, engine X is more efficient than a Carnot engine, then the combination shown in (a) is equivalent to the perfect refrigerator shown here. This violates the second law of thermodynamics, so we conclude that engine X cannot be more efficient than a Carnot engine.

Q'H

Engine X

QH

Carnot refrigerator

Q Perfect refrigerator

W Q'L

QL

Q

TL (a)

(b)

597

598

CHAPTE R 20 E NTROPY AN D TH E SECON D L AW OF TH E R M ODYNAM ICS

Because the work done by engine X is equal to the work done on the Carnot refrigerator, we have, from the first law of thermodynamics as given by Eq. 20-8, !QH! $ ! QL! # !Q,H! $ ! Q,L!, which we can write as !QH! $ ! Q,H! # !QL! $ ! Q,L! # Q.

(20-19)

Because of Eq. 20-18, the quantity Q in Eq. 20-19 must be positive. Comparison of Eq. 20-19 with Fig. 20-16 shows that the net effect of engine X and the Carnot refrigerator working in combination is to transfer energy Q as heat from a low-temperature reservoir to a high-temperature reservoir without the requirement of work. Thus, the combination acts like the perfect refrigerator of Fig. 20-15, whose existence is a violation of the second law of thermodynamics. Something must be wrong with one or more of our assumptions, and it can only be Eq. 20-17. We conclude that no real engine can have an efficiency greater than that of a Carnot engine when both engines work between the same two temperatures. At most, the real engine can have an efficiency equal to that of a Carnot engine. In that case, the real engine is a Carnot engine.

20-4 A STATISTICAL VIEW OF ENTROPY Learning Objectives After reading this module, you should be able to . . .

20.21 Explain what is meant by the configurations of a system of molecules. 20.22 Calculate the multiplicity of a given configuration. 20.23 Identify that all microstates are equally probable but

the configurations with more microstates are more probable than the other configurations. 20.24 Apply Boltzmann’s entropy equation to calculate the entropy associated with a multiplicity.

Key Ideas ● The entropy of a system can be defined in terms of the possible distributions of its molecules. For identical molecules, each possible distribution of molecules is called a microstate of the system. All equivalent microstates are grouped into a configuration of the system. The number of microstates in a configuration is the multiplicity W of the configuration. ● For a system of N molecules that may be distributed

between the two halves of a box, the multiplicity is given by N! W# , n1! n2! in which n1 is the number of molecules in one half of the box and n2 is the number in the other half. A basic assumption of statistical mechanics is that all the microstates are equally probable.

Thus, configurations with a large multiplicity occur most often. When N is very large (say, N # 1022 molecules or more), the molecules are nearly always in the configuration in which n1 # n2. ● The multiplicity W of a configuration of a system and the en-

tropy S of the system in that configuration are related by Boltzmann’s entropy equation: S # k ln W, where k # 1.38 ' 10$23 J/K is the Boltzmann constant. ● When N is very large (the usual case), we can approximate

ln N! with Stirling’s approximation: ln N! % N(ln N) $ N.

A Statistical View of Entropy In Chapter 19 we saw that the macroscopic properties of gases can be explained in terms of their microscopic, or molecular, behavior. Such explanations are part of a study called statistical mechanics. Here we shall focus our attention on a single problem, one involving the distribution of gas molecules between the two halves of an insulated box. This problem is reasonably simple to analyze, and it allows us to use statistical mechanics to calculate the entropy change for the free expansion of an ideal gas. You will see that statistical mechanics leads to the same entropy change as we would find using thermodynamics.

20-4 A STATISTICAL VI EW OF E NTROPY

Figure 20-17 shows a box that contains six identical (and thus indistinguishable) molecules of a gas. At any instant, a given molecule will be in either the left or the right half of the box; because the two halves have equal volumes, the molecule has the same likelihood, or probability, of being in either half. Table 20-1 shows the seven possible configurations of the six molecules, each configuration labeled with a Roman numeral. For example, in configuration I, all six molecules are in the left half of the box (n1 # 6) and none are in the right half (n2 # 0). We see that, in general, a given configuration can be achieved in a number of different ways. We call these different arrangements of the molecules microstates. Let us see how to calculate the number of microstates that correspond to a given configuration. Suppose we have N molecules, distributed with n1 molecules in one half of the box and n2 in the other. (Thus n1 " n2 # N.) Let us imagine that we distribute the molecules “by hand,” one at a time. If N # 6, we can select the first molecule in six independent ways; that is, we can pick any one of the six molecules. We can pick the second molecule in five ways, by picking any one of the remaining five molecules; and so on. The total number of ways in which we can select all six molecules is the product of these independent ways, or 6 ' 5 ' 4 ' 3 ' 2 ' 1 # 720. In mathematical shorthand we write this product as 6! # 720, where 6! is pronounced “six factorial.” Your hand calculator can probably calculate factorials. For later use you will need to know that 0! # 1. (Check this on your calculator.) However, because the molecules are indistinguishable, these 720 arrangements are not all different. In the case that n1 # 4 and n2 # 2 (which is configuration III in Table 20-1), for example, the order in which you put four molecules in one half of the box does not matter, because after you have put all four in, there is no way that you can tell the order in which you did so. The number of ways in which you can order the four molecules is 4! # 24. Similarly, the number of ways in which you can order two molecules for the other half of the box is simply 2! # 2. To get the number of different arrangements that lead to the (4, 2) split of configuration III, we must divide 720 by 24 and also by 2. We call the resulting quantity, which is the number of microstates that correspond to a given configuration, the multiplicity W of that configuration. Thus, for configuration III, WIII #

6! 720 # # 15. 4! 2! 24 ' 2

Thus, Table 20-1 tells us there are 15 independent microstates that correspond to configuration III. Note that, as the table also tells us, the total number of microstates for six molecules distributed over the seven configurations is 64. Extrapolating from six molecules to the general case of N molecules, we have W#

N! n1! n2!

(multiplicity of configuration).

(20-20)

Table 20-1 Six Molecules in a Box Configuration Label n1 n2 I II III IV V VI VII

6 5 4 3 2 1 0

0 1 2 3 4 5 6

Multiplicity W (number of microstates) 1 6 15 20 15 6 1 Total # 64

Calculation of W (Eq. 20-20) 6!/(6! 6!/(5! 6!/(4! 6!/(3! 6!/(2! 6!/(1! 6!/(0!

0!) # 1 1!) # 6 2!) # 15 3!) # 20 4!) # 15 5!) # 6 6!) # 1

Entropy 10$23 J/K (Eq. 20-21) 0 2.47 3.74 4.13 3.74 2.47 0

(a)

599

Insulation

(b)

Figure 20-17 An insulated box contains six gas molecules. Each molecule has the same probability of being in the left half of the box as in the right half. The arrangement in (a) corresponds to configuration III in Table 20-1, and that in (b) corresponds to configuration IV.

CHAPTE R 20 E NTROPY AN D TH E SECON D L AW OF TH E R M ODYNAM ICS

Number of microstates W

600

Central configuration peak

0 25 50 75 100% Percentage of molecules in left half

Figure 20-18 For a large number of molecules in a box, a plot of the number of microstates that require various percentages of the molecules to be in the left half of the box. Nearly all the microstates correspond to an approximately equal sharing of the molecules between the two halves of the box; those microstates form the central configuration peak on the plot. For N % 1022, the central configuration peak is much too narrow to be drawn on this plot.

You should verify the multiplicities for all the configurations in Table 20-1. The basic assumption of statistical mechanics is that All microstates are equally probable.

In other words, if we were to take a great many snapshots of the six molecules as they jostle around in the box of Fig. 20-17 and then count the number of times each microstate occurred, we would find that all 64 microstates would occur equally often. Thus the system will spend, on average, the same amount of time in each of the 64 microstates. Because all microstates are equally probable but different configurations have different numbers of microstates, the configurations are not all equally probable. In Table 20-1 configuration IV, with 20 microstates, is the most probable configuration, with a probability of 20/64 # 0.313. This result means that the system is in configuration IV 31.3% of the time. Configurations I and VII, in which all the molecules are in one half of the box, are the least probable, each with a probability of 1/64 # 0.016 or 1.6%. It is not surprising that the most probable configuration is the one in which the molecules are evenly divided between the two halves of the box, because that is what we expect at thermal equilibrium. However, it is surprising that there is any probability, however small, of finding all six molecules clustered in half of the box, with the other half empty. For large values of N there are extremely large numbers of microstates, but nearly all the microstates belong to the configuration in which the molecules are divided equally between the two halves of the box, as Fig. 20-18 indicates. Even though the measured temperature and pressure of the gas remain constant, the gas is churning away endlessly as its molecules “visit” all probable microstates with equal probability. However, because so few microstates lie outside the very narrow central configuration peak of Fig. 20-18, we might as well assume that the gas molecules are always divided equally between the two halves of the box. As we shall see, this is the configuration with the greatest entropy.

Sample Problem 20.05 Microstates and multiplicity Suppose that there are 100 indistinguishable molecules in the box of Fig. 20-17. How many microstates are associated with the configuration n1 # 50 and n2 # 50, and with the configuration n1 # 100 and n2 # 0? Interpret the results in terms of the relative probabilities of the two configurations. KEY IDEA The multiplicity W of a configuration of indistinguishable molecules in a closed box is the number of independent microstates with that configuration, as given by Eq. 20-20. Calculations: Thus, for the (n1, n2) configuration (50, 50), W#

100! N! # n 1! n 2! 50! 50!

9.33 ' 10157 (3.04 ' 1064)(3.04 ' 1064) # 1.01 ' 1029.

#

(Answer)

Similarly, for the configuration (100, 0), we have W#

100! 1 1 N! # # # # 1. (Answer) n 1! n 2! 100! 0! 0! 1

The meaning: Thus, a 50 – 50 distribution is more likely than a 100 – 0 distribution by the enormous factor of about 1 ' 1029. If you could count, at one per nanosecond, the number of microstates that correspond to the 50 – 50 distribution, it would take you about 3 ' 1012 years, which is about 200 times longer than the age of the universe. Keep in mind that the 100 molecules used in this sample problem is a very small number. Imagine what these calculated probabilities would be like for a mole of molecules, say about N # 1024. Thus, you need never worry about suddenly finding all the air molecules clustering in one corner of your room, with you gasping for air in another corner. So, you can breathe easy because of the physics of entropy.

Additional examples, video, and practice available at WileyPLUS

20-4 A STATISTICAL VI EW OF E NTROPY

601

Probability and Entropy In 1877, Austrian physicist Ludwig Boltzmann (the Boltzmann of Boltzmann’s constant k) derived a relationship between the entropy S of a configuration of a gas and the multiplicity W of that configuration. That relationship is S # k ln W

(20-21)

(Boltzmann’s entropy equation).

This famous formula is engraved on Boltzmann’s tombstone. It is natural that S and W should be related by a logarithmic function. The total entropy of two systems is the sum of their separate entropies. The probability of occurrence of two independent systems is the product of their separate probabilities. Because ln ab # ln a " ln b, the logarithm seems the logical way to connect these quantities. Table 20-1 displays the entropies of the configurations of the six-molecule system of Fig. 20-17, computed using Eq. 20-21. Configuration IV, which has the greatest multiplicity, also has the greatest entropy. When you use Eq. 20-20 to calculate W, your calculator may signal “OVERFLOW” if you try to find the factorial of a number greater than a few hundred. Instead, you can use Stirling’s approximation for ln N!: ln N! % N(ln N) $ N

(20-22)

(Stirling’s approximation).

The Stirling of this approximation was an English mathematician and not the Robert Stirling of engine fame.

Checkpoint 5 A box contains 1 mol of a gas. Consider two configurations: (a) each half of the box contains half the molecules and (b) each third of the box contains one-third of the molecules. Which configuration has more microstates?

Sample Problem 20.06 Entropy change of free expansion using microstates In Sample Problem 20.01, we showed that when n moles of an ideal gas doubles its volume in a free expansion, the entropy increase from the initial state i to the final state f is Sf $ Si # nR ln 2. Derive this increase in entropy by using statistical mechanics. KEY IDEA We can relate the entropy S of any given configuration of the molecules in the gas to the multiplicity W of microstates for that configuration, using Eq. 20-21 (S # k ln W ). Calculations: We are interested in two configurations: the final configuration f (with the molecules occupying the full volume of their container in Fig. 20-1b) and the initial configuration i (with the molecules occupying the left half of the container). Because the molecules are in a closed container, we can calculate the multiplicity W of their microstates with Eq. 20-20. Here we have N molecules in the n moles of the gas. Initially, with the molecules all in the left

half of the container, their (n1, n2) configuration is (N, 0). Then, Eq. 20-20 gives their multiplicity as Wi #

N! # 1. N! 0!

Finally, with the molecules spread through the full volume, their (n1, n2) configuration is (N/2, N/2). Then, Eq. 20-20 gives their multiplicity as Wf #

N! . (N/2)! (N/2)!

From Eq. 20-21, the initial and final entropies are and

Si # k ln Wi # k ln 1 # 0 Sf # k ln Wf # k ln(N!) $ 2k ln[(N/2)!]. (20-23)

In writing Eq. 20-23, we have used the relation a ln 2 # ln a $ 2 ln b. b

602

CHAPTE R 20 E NTROPY AN D TH E SECON D L AW OF TH E R M ODYNAM ICS

Now, applying Eq. 20-22 to evaluate Eq. 20-23, we find that

thus Sf $ Si # nR ln 2 $ 0

Sf # k ln(N!) $ 2k ln[(N/2)!]

# nR ln 2,

# k[N(ln N) $ N] $ 2k[(N/2) ln(N/2) $ (N/2)] # k[N(ln N) $ N $ N ln(N/2) " N] # k[N(ln N) $ N(ln N $ ln 2)] # Nk ln 2.

(20-24)

From Eq. 19-8 we can substitute nR for Nk, where R is the universal gas constant. Equation 20-24 then becomes Sf # nR ln 2. The change in entropy from the initial state to the final is

(Answer)

which is what we set out to show. In the first sample problem of this chapter we calculated this entropy increase for a free expansion with thermodynamics by finding an equivalent reversible process and calculating the entropy change for that process in terms of temperature and heat transfer. In this sample problem, we calculate the same increase in entropy with statistical mechanics using the fact that the system consists of molecules. In short, the two, very different approaches give the same answer.

Additional examples, video, and practice available at WileyPLUS

Review & Summary One-Way Processes An irreversible process is one that cannot be reversed by means of small changes in the environment. The direction in which an irreversible process proceeds is set by the change in entropy %S of the system undergoing the process. Entropy S is a state property (or state function) of the system; that is, it depends only on the state of the system and not on the way in which the system reached that state. The entropy postulate states (in part): If an irreversible process occurs in a closed system, the entropy of the system always increases.

Calculating Entropy Change The entropy change %S for an irreversible process that takes a system from an initial state i to a final state f is exactly equal to the entropy change %S for any reversible process that takes the system between those same two states. We can compute the latter (but not the former) with %S # Sf $ Si #

"

f

i

dQ . T

Q . T

(20-2)

When the temperature change %T of a system is small relative to the temperature (in kelvins) before and after the process, the entropy change can be approximated as %S # Sf $ Si %

Q , Tavg

(20-3)

where Tavg is the system’s average temperature during the process. When an ideal gas changes reversibly from an initial state with temperature Ti and volume Vi to a final state with temperature Tf and volume Vf , the change %S in the entropy of the gas is %S # Sf $ Si # nR ln

Vf Tf " nCV ln . Vi Ti

an extension of the entropy postulate, states: If a process occurs in a closed system, the entropy of the system increases for irreversible processes and remains constant for reversible processes. It never decreases. In equation form, %S 0 0.

(20-4)

(20-5)

Engines An engine is a device that, operating in a cycle, extracts

energy as heat +QH+ from a high-temperature reservoir and does a certain amount of work +W+. The efficiency ´ of any engine is defined as ´#

energy we get !W ! # . energy we pay for !QH !

(20-11)

In an ideal engine, all processes are reversible and no wasteful energy transfers occur due to, say, friction and turbulence. A Carnot engine is an ideal engine that follows the cycle of Fig. 20-9. Its efficiency is

(20-1)

Here Q is the energy transferred as heat to or from the system during the process, and T is the temperature of the system in kelvins during the process. For a reversible isothermal process, Eq. 20-1 reduces to %S # Sf $ Si #

The Second Law of Thermodynamics This law, which is

´C # 1 $

!QL ! TL #1$ , !QH ! TH

(20-12, 20-13)

in which TH and TL are the temperatures of the high- and lowtemperature reservoirs, respectively. Real engines always have an efficiency lower than that given by Eq. 20-13. Ideal engines that are not Carnot engines also have lower efficiencies. A perfect engine is an imaginary engine in which energy extracted as heat from the high-temperature reservoir is converted completely to work. Such an engine would violate the second law of thermodynamics, which can be restated as follows: No series of processes is possible whose sole result is the absorption of energy as heat from a thermal reservoir and the complete conversion of this energy to work.

Refrigerators A refrigerator is a device that, operating in a cycle, has work W done on it as it extracts energy +QL+ as heat from a low-temperature reservoir. The coefficient of performance K of a refrigerator is defined as K#

what we want !QL! . # what we pay for !W !

(20-14)

A Carnot refrigerator is a Carnot engine operating in reverse.

603

QU ESTIONS

For a Carnot refrigerator, Eq. 20-14 becomes KC #

!QL! TL # . !QH! $ !QL! TH $ TL

(20-15, 20-16)

A perfect refrigerator is an imaginary refrigerator in which energy extracted as heat from the low-temperature reservoir is converted completely to heat discharged to the high-temperature reservoir, without any need for work. Such a refrigerator would violate the second law of thermodynamics, which can be restated as follows: No series of processes is possible whose sole result is the transfer of energy as heat from a reservoir at a given temperature to a reservoir at a higher temperature.

Entropy from a Statistical View The entropy of a system can be defined in terms of the possible distributions of its molecules. For identical molecules, each possible distribution of molecules is called a microstate of the system.All equivalent microstates are grouped into

a configuration of the system. The number of microstates in a configuration is the multiplicity W of the configuration. For a system of N molecules that may be distributed between the two halves of a box, the multiplicity is given by W#

N! , n1! n2!

(20-20)

in which n1 is the number of molecules in one half of the box and n2 is the number in the other half. A basic assumption of statistical mechanics is that all the microstates are equally probable. Thus, configurations with a large multiplicity occur most often. The multiplicity W of a configuration of a system and the entropy S of the system in that configuration are related by Boltzmann’s entropy equation: S # k ln W,

(20-21)

where k # 1.38 ' 10$23 J/K is the Boltzmann constant.

Questions sion (a) isothermal, (b) isobaric (constant pressure), and (c) adiabatic? Explain your answers. (d) In which processes does the entropy of the gas decrease?

a Pressure

1 Point i in Fig. 20-19 represents the initial state of an ideal gas at temperature T. Taking algebraic signs into account, rank the entropy changes that the gas undergoes as it moves, successively and reversibly, from point i to points a, b, c, and d, greatest first.

d

b

i

T c

T + ∆T T – ∆T

Volume

2 In four experiments, blocks A Figure 20-19 Question 1. and B, starting at different initial temperatures, were brought together in an insulating box and allowed to reach a common final temperature. The entropy changes for the blocks in the four experiments had the following values (in joules per kelvin), but not necessarily in the order given. Determine which values for A go with which values for B. Block A B

Values 8 $3

5 $8

3 $5

9 $2

3 A gas, confined to an insulated cylinder, is compressed adiabatically to half its volume. Does the entropy of the gas increase, decrease, or remain unchanged during this process?

Temperature

4 An ideal monatomic gas at initial temperature T0 (in kelvins) expands from initial volume V0 to volume 2V0 2.5T0 B by each of the five processes indicated in 2.0T0 C the T-V diagram of Fig. 20-20. In which 1.5T0 D process is the expanT0

A

0.63T0

Figure 20-20 Question 4.

E F

V0 Volume

2V0

5 In four experiments, 2.5 mol of hydrogen gas undergoes reversible isothermal expansions, starting from the same volume but at different temperatures. The corresponding p-V plots are shown in Fig. 20-21. Rank the situations according to the change in the entropy of the gas, greatest first.

p

a b c d V

Figure 20-21 Question 5.

6 A box contains 100 atoms in a configuration that has 50 atoms in each half of the box. Suppose that you could count the different microstates associated with this configuration at the rate of 100 billion states per second, using a supercomputer. Without written calculation, guess how much computing time you would need: a day, a year, or much more than a year. 7 Does the entropy per cycle increase, decrease, or remain the same for (a) a Carnot engine, (b) a real engine, and (c) a perfect engine (which is, of course, impossible to build)? 8 Three Carnot engines operate between temperature limits of (a) 400 and 500 K, (b) 500 and 600 K, and (c) 400 and 600 K. Each engine extracts the same amount of energy per cycle from the high-temperature reservoir. Rank the magnitudes of the work done by the engines per cycle, greatest first. 9 An inventor claims to have invented four engines, each of which operates between constant-temperature reservoirs at 400 and 300 K. Data on each engine, per cycle of operation, are: engine A, QH # 200 J, QL # $175 J, and W # 40 J; engine B, QH # 500 J, QL # $200 J, and W # 400 J; engine C, QH # 600 J, QL # $200 J, and W # 400 J; engine D, QH # 100 J, QL # $90 J, and W # 10 J. Of the first and second laws of thermodynamics, which (if either) does each engine violate? 10 Does the entropy per cycle increase, decrease, or remain the same for (a) a Carnot refrigerator, (b) a real refrigerator, and (c) a perfect refrigerator (which is, of course, impossible to build)?

604

CHAPTE R 20 E NTROPY AN D TH E SECON D L AW OF TH E R M ODYNAM ICS

Problems Tutoring problem available (at instructor’s discretion) in WileyPLUS and WebAssign SSM • – •••

Worked-out solution available in Student Solutions Manual

WWW Worked-out solution is at

Number of dots indicates level of problem difficulty

ILW

Interactive solution is at

http://www.wiley.com/college/halliday

Additional information available in The Flying Circus of Physics and at flyingcircusofphysics.com

•2 An ideal gas undergoes a reversible isothermal expansion at 77.0-C, increasing its volume from 1.30 L to 3.40 L. The entropy change of the gas is 22.0 J/K. How many moles of gas are present? •3 ILW A 2.50 mol sample of an ideal gas expands reversibly and isothermally at 360 K until its volume is doubled. What is the increase in entropy of the gas? •4 How much energy must be transferred as heat for a reversible isothermal expansion of an ideal gas at 132-C if the entropy of the gas increases by 46.0 J/K? •5 ILW Find (a) the energy absorbed as heat and (b) the change in entropy of a 2.00 kg block of copper whose temperature is increased reversibly from 25.0-C to 100-C. The specific heat of copper is 386 J/kg 9K. •6 (a) What is the entropy change of a 12.0 g ice cube that melts completely in a bucket of water whose temperature is just above the freezing point of water? (b) What is the entropy change of a 5.00 g spoonful of water that evaporates completely on a hot plate whose temperature is slightly above the boiling point of water? ••7 ILW A 50.0 g block of copper whose temperature is 400 K is placed in an insulating box with a 100 g block of lead whose temperature is 200 K. (a) What is the equilibrium temperature of the twoblock system? (b) What is the change in the internal energy of the system between the initial state and the equilibrium state? (c) What is the change in the entropy of the system? (See Table 18-3.) ••8 At very low temperatures, the molar specific heat CV of many solids is approximately CV # AT 3, where A depends on the particular substance. For aluminum, A # 3.15 ' 10 $5 J/mol 9K4. Find the entropy change for 4.00 mol of aluminum when its temperature is raised from 5.00 K to 10.0 K.

gives the change in entropy %S of the block until thermal equilibrium is reached. The scale of the horizontal axis is set by Ta # 280 K and Tb # 380 K.What is the specific heat of the block? ••11 SSM WWW In an experiment, 200 g of aluminum (with a specific heat of 900 J/kg 9 K) at 100-C is mixed with 50.0 g of water at 20.0-C, with the mixture thermally isolated. (a) What is the equilibrium temperature? What are the entropy changes of (b) the aluminum, (c) the water, and (d) the aluminum – water system? ••12 A gas sample undergoes a reversible isothermal expansion. Figure 20-23 gives the change %S in entropy of the gas versus the final volume Vf of the gas. The scale of the vertical axis is set by %Ss # 64 J/K. How many moles are in the sample? ∆Ss ∆S (J/K)

Module 20-1 Entropy •1 SSM Suppose 4.00 mol of an ideal gas undergoes a reversible isothermal expansion from volume V1 to volume V2 # 2.00V1 at temperature T # 400 K. Find (a) the work done by the gas and (b) the entropy change of the gas. (c) If the expansion is reversible and adiabatic instead of isothermal, what is the entropy change of the gas?

0

0.8

1.6 2.4 Vf (m3)

3.2

4.0

Figure 20-23 Problem 12. ••13 In the irreversible process of Fig. 20-5, let the initial temperatures of the identical blocks L and R be 305.5 and 294.5 K, respectively, and let 215 J be the energy that must be transferred between the blocks in order to reach equilibrium. For the reversible processes of Fig. 20-6, what is %S for (a) block L, (b) its reservoir, (c) block R, (d) its reservoir, (e) the two-block system, and (f) the system of the two blocks and the two reservoirs?

••10 A 364 g block is put in contact with a thermal reservoir. The block is initially at a lower temperature than the reservoir. Assume that the consequent transfer of energy as heat from the reservoir to the block is reversible. Figure 20-22

••15 A mixture of 1773 g of water and 227 g of ice is in an initial equilibrium state at 0.000-C. The mixture is then, in a reversible process, brought to a second equilibrium state where the water – ice ratio, by mass, is 1.00!1.00 at 0.000-C. (a) Calculate the entropy change of the system during this process. (The heat of fusion for water is 333 kJ/kg.) (b) The system is then returned to the initial equilibrium state in an irreversible process (say, by using a Bunsen burner). Calculate the entropy change of the system during this process. (c) Are your answers consistent with the second law of thermodynamics?

∆S (J/K)

60 40 20 Ta

T (K)

Figure 20-22 Problem 10.

Tb

Pressure

••9 A 10 g ice cube at $10-C is placed in a lake whose temperature is 15-C. Calculate the change in entropy of the cube – lake system as the ice cube comes to thermal equilibrium with the lake. The specific heat of ice is 2220 J/kg 9K. (Hint: Will the ice cube affect the lake temperature?)

••14 (a) For 1.0 mol of a monatomic ideal gas taken through the cycle in Fig. 20-24, where V1 # 4.00V0, what is W/p0V0 as the gas goes from state a to state c along path abc? What is %Eint/p0V0 in going (b) from b to c and (c) through one full cycle? What is %S in going (d) from b to c and (e) through one full cycle?

c

2p0 p0

a

b V0

V1 Volume

Figure 20-24 Problem 14.

PROB LE M S

••16 An 8.0 g ice cube at $10-C is put into a Thermos flask containing 100 cm3 of water at 20-C. By how much has the entropy of the cube – water system changed when equilibrium is reached? The specific heat of ice is 2220 J/kg 9K.

Pressure

Isothermal

Adiabatic

Module 20-2 Entropy in the Real World: Engines •23 A Carnot engine whose low-temperature reservoir is at 17-C has an efficiency of 40%. By how much should the temperature of the high-temperature reservoir be increased to increase the efficiency to 50%?

2 3

V1

Volume

•24 A Carnot engine absorbs 52 kJ as heat and exhausts 36 kJ as heat in each cycle. Calculate (a) the engine’s efficiency and (b) the work done per cycle in kilojoules.

V23

Figure 20-25 Problem 17.

•25 A Carnot engine has an efficiency of 22.0%. It operates between constant-temperature reservoirs differing in temperature by 75.0 C-. What is the temperature of the (a) lower-temperature and (b) higher-temperature reservoir?

Ts

0

Entropy (J/K)

ice " original water. (a) What is the equilibrium temperature of the system? What are the entropy changes of the water that was originally the ice cube (b) as it melts and (c) as it warms to the equilibrium temperature? (d) What is the entropy change of the original water as it cools to the equilibrium temperature? (e) What is the net entropy change of the ice " original water system as it reaches the equilibrium temperature?

Ss

Figure 20-26 Problem 18.

•••19 Suppose 1.00 mol of a monatomic ideal gas is taken from initial pressure p1 and volume V1 through two steps: (1) an isothermal expansion to volume 2.00V1 and (2) a pressure increase to 2.00p1 at constant volume. What is Q/p1V1 for (a) step 1 and (b) step 2? What is W/p1V1 for (c) step 1 and (d) step 2? For the full process, what are (e) %Eint/p1V1 and (f) %S? The gas is returned to its initial state and again taken to the same final state but now through these two steps: (1) an isothermal compression to pressure 2.00p1 and (2) a volume increase to 2.00V1 at constant pressure. What is Q/p1V1 for (g) step 1 and (h) step 2? What is W/p1V1 for (i) step 1 and (j) step 2? For the full process, what are (k) %Eint/p1V1 and (l) %S? •••20 Expand 1.00 mol of an monatomic gas initially at 5.00 kPa and 600 K from initial volume Vi # 1.00 m3 to final volume Vf # 2.00 m3. At any instant during the expansion, the pressure p and volume V of the gas are related by p # 5.00 exp[(Vi $ V )/a], with p in kilopascals, Vi and V in cubic meters, and a # 1.00 m3. What are the final (a) pressure and (b) temperature of the gas? (c) How much work is done by the gas during the expansion? (d) What is %S for the expansion? (Hint: Use two simple reversible processes to find %S.) •••21 Energy can be removed from water as heat at and even below the normal freezing point (0.0-C at atmospheric pressure) without causing the water to freeze; the water is then said to be supercooled. Suppose a 1.00 g water drop is supercooled until its temperature is that of the surrounding air, which is at $5.00-C. The drop then suddenly and irreversibly freezes, transferring energy to the air as heat. What is the entropy change for the drop? (Hint: Use a three-step reversible process as if the water were taken through the normal freezing point.) The specific heat of ice is 2220 J/kg 9K. •••22 An insulated Thermos contains 130 g of water at 80.0-C. You put in a 12.0 g ice cube at 0-C to form a system of

•26 In a hypothetical nuclear fusion reactor, the fuel is deuterium gas at a temperature of 7 ' 108 K. If this gas could be used to operate a Carnot engine with TL # 100-C, what would be the engine’s efficiency? Take both temperatures to be exact and report your answer to seven significant figures. •27 SSM WWW A Carnot engine operates between 235-C and 115-C, absorbing 6.30 ' 104 J per cycle at the higher temperature. (a) What is the efficiency of the engine? (b) How much work per cycle is this engine capable of performing? ••28 In the first stage of a two-stage Carnot engine, energy is absorbed as heat Q1 at temperature T1, work W1 is done, and energy is expelled as heat Q2 at a lower temperature T2. The second stage absorbs that energy as heat Q2, does work W2, and expels energy as heat Q3 at a still lower temperature T3. Prove that the efficiency of the engine is (T1 $ T3)/T1. ••29 Figure 20-27 shows a rec b V, p versible cycle through which 1.00 mol of a monatomic ideal gas is taken. Assume that p # 2p0, V # 2V0, p0 # 1.01 ' 105 Pa, and V0 # 0.0225 m3. V0, p0 d a Calculate (a) the work done during the cycle, (b) the energy added as heat during stroke abc, and (c) the efficiency of the cycle. (d) What is Volume the efficiency of a Carnot engine opFigure 20-27 Problem 29. erating between the highest and lowest temperatures that occur in the cycle? (e) Is this greater than or less than the efficiency calculated in (c)? Pressure

••18 A 2.0 mol sample of an ideal monatomic gas undergoes the reversible process shown in Fig. 20-26. The scale of the vertical axis is set by Ts # 400.0 K and the scale of the horizontal axis is set by Ss # 20.0 J/K. (a) How much energy is absorbed as heat by the gas? (b) What is the change in the internal energy of the gas? (c) How much work is done by the gas?

1

Temperature (K)

••17 In Fig. 20-25, where V23 # 3.00V1, n moles of a diatomic ideal gas are taken through the cycle with the molecules rotating but not oscillating. What are (a) p2 /p1, (b) p3 /p1, and (c) T3 /T1? For path 1 : 2, what are (d) W/nRT1, (e) Q/nRT1, (f) %Eint/nRT1, and (g) %S/nR? For path 2 : 3, what are (h) W/nRT1, (i) Q/nRT1, (j) %Eint/nRT1, (k) %S/nR? For path 3 : 1, what are (l) W/nRT1, (m) Q/nRT1, (n) %Eint/nRT1, and (o) %S/nR?

605

••30 A 500 W Carnot engine operates between constanttemperature reservoirs at 100-C and 60.0-C.What is the rate at which energy is (a) taken in by the engine as heat and (b) exhausted by the engine as heat? ••31 The efficiency of a particular car engine is 25% when the engine does 8.2 kJ of work per cycle. Assume the process is reversible. What are (a) the energy the engine gains per cycle as heat Qgain from the fuel combustion and (b) the energy the engine loses per cycle as heat Qlost? If a tune-up increases the efficiency to 31%, what are (c) Qgain and (d) Qlost at the same work value?

606

CHAPTE R 20 E NTROPY AN D TH E SECON D L AW OF TH E R M ODYNAM ICS

••32 A Carnot engine is set up to produce a certain work W per cycle. In each cycle, energy in the form of heat QH is transferred to the working substance of the engine from the higher-temperature thermal reservoir, which is at an adjustable temperature TH. The lower-temperature thermal reservoir is maintained at temperature TL # 250 K. Figure 20-28 gives QH for a range of TH. The scale of the vertical axis is set by QHs # 6.0 kJ. If TH is set at 550 K, what is QH?

Q H (kJ) 0 250

300 TH (K)

Figure 20-28 Problem 32. ••33 SSM ILW Figure 20-29 shows a reversible cycle through which 1.00 mol of a monatomic ideal gas is taken. Volume Vc # 8.00Vb. Process bc is an adiabatic expansion, with pb # 10.0 atm and Vb # 1.00 ' 10$3 m3. For the cycle, find (a) the energy added to the gas as heat, (b) the energy leaving the gas as heat, (c) the net work done by the gas, and (d) the efficiency of the cycle.

•39 SSM A Carnot air conditioner takes energy from the thermal energy of a room at 70-F and transfers it as heat to the outdoors, which is at 96-F. For each joule of electric energy required to operate the air conditioner, how many joules are removed from the room?

350

•40 To make ice, a freezer that is a reverse Carnot engine extracts 42 kJ as heat at $15-C during each cycle, with coefficient of performance 5.7. The room temperature is 30.3-C. How much (a) energy per cycle is delivered as heat to the room and (b) work per cycle is required to run the freezer?

b

Pressure

pb

Adiabatic

••41 ILW An air conditioner operating between 93-F and 70-F is rated at 4000 Btu/h cooling capacity. Its coefficient of performance is 27% of that of a Carnot refrigerator operating between the same two temperatures.What horsepower is required of the air conditioner motor?

c

a Vb

Volume

Vc

Figure 20-29 Problem 33. ••34 An ideal gas (1.0 mol) is the working substance in an engine that operates on the cycle shown in Fig. 20-30. Processes BC and DA are reversible and adiabatic. (a) Is the gas monatomic, diatomic, or polyatomic? (b) What is the engine efficiency? B

Pressure

p0

D

p0/32 V0

2V0

•37 SSM A heat pump is used to heat a building. The external temperature is less than the internal temperature. The pump’s coefficient of performance is 3.8, and the heat pump delivers 7.54 MJ as heat to the building each hour. If the heat pump is a Carnot engine working in reverse, at what rate must work be done to run it? •38 The electric motor of a heat pump transfers energy as heat from the outdoors, which is at $5.0-C, to a room that is at 17-C. If the heat pump were a Carnot heat pump (a Carnot engine working in reverse), how much energy would be transferred as heat to the room for each joule of electric energy consumed?

Q Hs

A

J as heat (a) from a reservoir at 7.0-C to one at 27-C, (b) from a reservoir at $73-C to one at 27-C, (c) from a reservoir at $173-C to one at 27-C, and (d) from a reservoir at $223-C to one at 27-C?

C

8V0 Volume

Figure 20-30 Problem 34.

Pressure

Module 20-3 Refrigerators and Real Engines •36 How much work must be done by a Carnot refrigerator to transfer 1.0

2 Adiabatic 3

Spark 1

Intake

Adiabatic 4 V1

••43 Figure 20-32 represents a Carnot engine that works between temperatures T1 # 400 K and T2 # 150 K and drives a Carnot refrigerator that works between temperatures T3 # 325 K and T4 # 225 K. What is the ratio Q3 /Q1?

T1 T3

Q1

Q3

W Q4

Q2

T4 Refrigerator

16V0

•••35 The cycle in Fig. 20-31 represents the operation of a gasoline in- 3.00p 1 ternal combustion engine. Volume V3 # 4.00V1. Assume the gasoline – air intake mixture is an ideal p1 gas with g # 1.30. What are the ratios (a) T2 /T1, (b) T3 /T1, (c) T4 /T1, (d) p3 /p1, and (e) p4 /p1? (f) What is the engine efficiency?

••42 The motor in a refrigerator has a power of 200 W. If the freezing compartment is at 270 K and the outside air is at 300 K, and assuming the efficiency of a Carnot refrigerator, what is the maximum amount of energy that can be extracted as heat from the freezing compartment in 10.0 min?

Volume

Figure 20-31 Problem 35.

V3

T2 ••44 (a) During each cycle, a Carnot engine abEngine sorbs 750 J as heat from a Figure 20-32 Problem 43. high-temperature reservoir at 360 K, with the low-temperature reservoir at 280 K. How much work is done per cycle? (b) The engine is then made to work in reverse to function as a Carnot refrigerator between those same two reservoirs. During each cycle, how much work is required to remove 1200 J as heat from the low-temperature reservoir?

Module 20-4 A Statistical View of Entropy •45 Construct a table like Table 20-1 for eight molecules. ••46 A box contains N identical gas molecules equally divided between its two halves. For N # 50, what are (a) the multiplicity W of the central configuration, (b) the total number of microstates, and (c) the percentage of the time the system spends in the central configuration? For N # 100, what are (d) W of the central configura-

607

PROB LE M S

tion, (e) the total number of microstates, and (f) the percentage of the time the system spends in the central configuration? For N # 200, what are (g) W of the central configuration, (h) the total number of microstates, and (i) the percentage of the time the system spends in the central configuration? ( j) Does the time spent in the central configuration increase or decrease with an increase in N? •••47 SSM WWW A box contains N gas molecules. Consider the box to be divided into three equal parts. (a) By extension of Eq. 20-20, write a formula for the multiplicity of any given configuration. (b) Consider two configurations: configuration A with equal numbers of molecules in all three thirds of the box, and configuration B with equal numbers of molecules in each half of the box divided into two equal parts rather than three. What is the ratio WA/WB of the multiplicity of configuration A to that of configuration B? (c) Evaluate WA/WB for N # 100. (Because 100 is not evenly divisible by 3, put 34 molecules into one of the three box parts of configuration A and 33 in each of the other two parts.) Additional Problems 48 Four particles are in the insulated box of Fig. 20-17. What are (a) the least multiplicity, (b) the greatest multiplicity, (c) the least entropy, and (d) the greatest entropy of the four-particle system? 49 A cylindrical copper rod of length 1.50 m and radius 2.00 cm is insulated to prevent heat loss through its curved surface. One end is attached to a thermal reservoir fixed at 300-C; the other is attached to a thermal reservoir fixed at 30.0-C. What is the rate at which entropy increases for the rod – reservoirs system? 50 Suppose 0.550 mol of an ideal gas is isothermally and reversibly expanded in the four situations given below. What is the change in the entropy of the gas for each situation? Situation Temperature (K) Initial volume (cm3) Final volume (cm3)

(a)

(b)

(c)

(d)

250 0.200 0.800

350 0.200 0.800

400 0.300 1.20

450 0.300 1.20

51 SSM As a sample of nitrogen gas (N2) undergoes a temperature increase at constant volume, the distribution of molecular speeds increases. That is, the probability distribution function P(v) for the molecules spreads to higher speed values, as suggested in Fig. 19-8b. One way to report the spread in P(v) is to measure the difference %v between the most probable speed vP and the rms speed vrms. When P(v) spreads to higher speeds, %v increases. Assume that the gas is ideal and the N2 molecules rotate but do not oscillate. For 1.5 mol, an initial temperature of 250 K, and a final temperature of 500 K, what are (a) the initial difference %vi, (b) the final difference %vf , and (c) the entropy change %S for the gas? 52 Suppose 1.0 mol of a monatomic ideal gas initially at 10 L and 300 K is heated at constant volume to 600 K, allowed to expand isothermally to its initial pressure, and finally compressed at constant pressure to its original volume, pressure, and temperature. During the cycle, what are (a) the net energy entering the system (the gas) as heat and (b) the net work done by the gas? (c) What is the efficiency of the cycle? 53 Suppose that a deep shaft were drilled in Earth’s crust near one of the poles, where the surface temperature is $40-C, to a depth where the temperature is 800-C. (a) What is the theoretical limit to the efficiency of an engine operating between these

temperatures? (b) If all the energy released as heat into the lowtemperature reservoir were used to melt ice that was initially at $40-C, at what rate could liquid water at 0-C be produced by a 100 MW power plant (treat it as an engine)? The specific heat of ice is 2220 J/kg 9K; water’s heat of fusion is 333 kJ/kg. (Note that the engine can operate only between 0-C and 800-C in this case. Energy exhausted at $40-C cannot warm anything above $40-C.) 54 What is the entropy change for 3.20 mol of an ideal monatomic gas undergoing a reversible increase in temperature from 380 K to 425 K at constant volume? 55 A 600 g lump of copper at 80.0-C is placed in 70.0 g of water at 10.0-C in an insulated container. (See Table 18-3 for specific heats.) (a) What is the equilibrium temperature of the copper – water system? What entropy changes do (b) the copper, (c) the water, and (d) the copper – water system undergo in reaching the equilibrium temperature? F (N) 56 Figure 20-33 gives the force magnitude F versus stretch distance x for a rubber band, with the scale of the F axis set by Fs # 1.50 N and the scale of the x axis set by xs # 3.50 cm. The temperature is 2.00-C. When the rubber band is stretched by x # 1.70 cm, at what rate does the entropy of the rubber band change during a small additional stretch?

Fs

0

xs x (cm)

Figure 20-33 57 The temperature of 1.00 mol of a Problem 56. monatomic ideal gas is raised reversibly from 300 K to 400 K, with its volume kept constant. What is the entropy change of the gas? 58 Repeat Problem 57, with the pressure now kept constant. 59 SSM A 0.600 kg sample of water is initially ice at temperature $20-C. What is the sample’s entropy change if its temperature is increased to 40-C? 60 A three-step cycle is undergone by 3.4 mol of an ideal diatomic gas: (1) the temperature of the gas is increased from 200 K to 500 K at constant volume; (2) the gas is then isothermally expanded to its original pressure; (3) the gas is then contracted at constant pressure back to its original volume. Throughout the cycle, the molecules rotate but do not oscillate.What is the efficiency of the cycle? 61 An inventor has built an engine X and claims that its efficiency ´X is greater than the efficiency ´ of an ideal engine operating between the same two temperatures. Suppose you couple engine X to an ideal refrigerator (Fig. 20-34a) and adjust the cycle TH Q'H Engine X

QH

Ideal refrigerator

Q Perfect refrigerator

W Q'C

QC

Q

TC (a)

(b)

Figure 20-34 Problem 61.

CHAPTE R 20 E NTROPY AN D TH E SECON D L AW OF TH E R M ODYNAM ICS

of engine X so that the work per cycle it provides equals the work per cycle required by the ideal refrigerator. Treat this combination as a single unit and show that if the inventor’s claim were true (if ´X 4 ´), the combined unit would act as a perfect refrigerator (Fig. 20-34b), transferring energy as heat from the low-temperature reservoir to the high-temperature reservoir without the need for work. Temperature (K)

62 Suppose 2.00 mol of a di1 2 350 atomic gas is taken reversibly around the cycle shown in the TS diagram of Fig. 20-35, where S1 # 6.00 J/K and S2 # 8.00 J/K. 300 3 The molecules do not rotate or oscillate. What is the energy transferred as heat Q for (a) path 1 : 2, (b) path 2 : 3, and (c) the full cycle? (d) What is the work S1 S2 W for the isothermal process? Entropy (J/K) The volume V1 in state 1 is Figure 20-35 Problem 62. 0.200 m3. What is the volume in (e) state 2 and (f) state 3? What is the change %Eint for (g) path 1 : 2, (h) path 2 : 3, and (i) the full cycle? (Hint: (h) can be done with one or two lines of calculation using Module 19-7 or with a page of calculation using Module 19-9.) ( j) What is the work W for the adiabatic process? 63 A three-step cycle is undergone reversibly by 4.00 mol of an ideal gas: (1) an adiabatic expansion that gives the gas 2.00 times its initial volume, (2) a constant-volume process, (3) an isothermal compression back to the initial state of the gas. We do not know whether the gas is monatomic or diatomic; if it is diatomic, we do not know whether the molecules are rotating or oscillating.What are the entropy changes for (a) the cycle, (b) process 1, (c) process 3, and (d) process 2? 64 (a) A Carnot engine operates between a hot reservoir at 320 K and a cold one at 260 K. If the engine absorbs 500 J as heat per cycle at the hot reservoir, how much work per cycle does it deliver? (b) If the engine working in reverse functions as a refrigerator between the same two reservoirs, how much work per cycle must be supplied to remove 1000 J as heat from the cold reservoir? 65 A 2.00 mol diatomic gas initially at 300 K undergoes this cycle: It is (1) heated at constant volume to 800 K, (2) then allowed to expand isothermally to its initial pressure, (3) then compressed at constant pressure to its initial state. Assuming the gas molecules neither rotate nor oscillate, find (a) the net energy transferred as heat to the gas, (b) the net work done by the gas, and (c) the efficiency of the cycle. 66 An ideal refrigerator does 150 J of work to remove 560 J as heat from its cold compartment. (a) What is the refrigerator’s coefficient of performance? (b) How much heat per cycle is exhausted to the kitchen? 67 Suppose that 260 J is conducted from a constant-temperature reservoir at 400 K to one at (a) 100 K, (b) 200 K, (c) 300 K, and (d) 360 K. What is the net change in entropy %Snet of the reservoirs in each case? (e) As the temperature difference of the two reservoirs decreases, does %Snet increase, decrease, or remain the same? 68 An apparatus that liquefies helium is in a room maintained at 300 K. If the helium in the apparatus is at 4.0 K, what is the minimum ratio Qto /Qfrom, where Qto is the energy delivered as heat to the room and Qfrom is the energy removed as heat from the helium?

69 A brass rod is in thermal contact with a constant-temperature reservoir at 130-C at one end and a constant-temperature reservoir at 24.0-C at the other end. (a) Compute the total change in entropy of the rod – reservoirs system when 5030 J of energy is conducted through the rod, from one reservoir to the other. (b) Does the entropy of the rod change? 70 A 45.0 g block of tungsten at 30.0-C and a 25.0 g block of silver at $120-C are placed together in an insulated container. (See Table 18-3 for specific heats.) (a) What is the equilibrium temperature? What entropy changes do (b) the tungsten, (c) the silver, and (d) the tungsten – silver system undergo in reaching the equilibrium temperature? 71 A box contains N molecules. Consider two configurations: configuration A with an equal division of the molecules between the two halves of the box, and configuration B with 60.0% of the molecules in the left half of the box and 40.0% in the right half. For N # 50, what are (a) the multiplicity WA of configuration A, (b) the multiplicity WB of configuration B, and (c) the ratio fB/A of the time the system spends in configuration B to the time it spends in configuration A? For N # 100, what are (d) WA, (e) WB, and (f) fB/A? For N # 200, what are (g) WA, (h) WB, and (i) fB/A? ( j) With increasing N, does f increase, decrease, or remain the same? 72 Calculate the efficiency of a fossil-fuel power plant that consumes 380 metric tons of coal each hour to produce useful work at the rate of 750 MW. The heat of combustion of coal (the heat due to burning it) is 28 MJ/kg. 73 SSM A Carnot refrigerator extracts 35.0 kJ as heat during each cycle, operating with a coefficient of performance of 4.60. What are (a) the energy per cycle transferred as heat to the room and (b) the work done per cycle? 74 A Carnot engine whose high-temperature reservoir is at 400 K has an efficiency of 30.0%. By how much should the temperature of the low-temperature reservoir be changed to increase the efficiency to 40.0%? 75 SSM System A of three particles and system B of five particles are in insulated boxes like that in Fig. 20-17. What is the least multiplicity W of (a) system A and (b) system B? What is the greatest multiplicity W of (c) A and (d) B? What is the greatest entropy of (e) A and (f) B? 76 Figure 20-36 shows a Carnot cycle on a T-S diagram, with a scale set by Ss # 0.60 J/K. For a full cycle, find (a) the net heat transfer and (b) the net work done by the system.

400 Temperature (K)

608

300 200 100

77 Find the relation be0 tween the efficiency of a Entropy (J/K) reversible ideal heat engine and the coefficient of perFigure 20-36 Problem 76. formance of the reversible refrigerator obtained by running the engine backwards.

Ss

78 A Carnot engine has a power of 500 W. It operates between heat reservoirs at 100-C and 60.0-C. Calculate (a) the rate of heat input and (b) the rate of exhaust heat output. 79 In a real refrigerator, the low-temperature coils are at $13-C, and the compressed gas in the condenser is at 26-C. What is the theoretical coefficient of performance?

C

H

A

P

T

E

R

2

1

Coulomb’s Law 21-1 COULOMB’S LAW Learning Objectives After reading this module, you should be able to . . .

21.01 Distinguish between being electrically neutral, negatively charged, and positively charged and identify excess charge. 21.02 Distinguish between conductors, nonconductors (insulators), semiconductors, and superconductors. 21.03 Describe the electrical properties of the particles inside an atom. 21.04 Identify conduction electrons and explain their role in making a conducting object negatively or positively charged. 21.05 Identify what is meant by “electrically isolated” and by “grounding.” 21.06 Explain how a charged object can set up induced charge in a second object. 21.07 Identify that charges with the same electrical sign repel each other and those with opposite electrical signs attract each other. 21.08 For either of the particles in a pair of charged particles, draw a free-body diagram, showing the electrostatic force (Coulomb force) on it and anchoring the tail of the force vector on that particle. 21.09 For either of the particles in a pair of charged particles, apply Coulomb’s law to relate the magnitude of the electrostatic force, the charge magnitudes of the particles, and the separation between the particles.

21.10 Identify that Coulomb’s law applies only to (point-like) particles and objects that can be treated as particles. 21.11 If more than one force acts on a particle, find the net force by adding all the forces as vectors, not scalars. 21.12 Identify that a shell of uniform charge attracts or repels a charged particle that is outside the shell as if all the shell’s charge were concentrated as a particle at the shell’s center. 21.13 Identify that if a charged particle is located inside a shell of uniform charge, there is no net electrostatic force on the particle from the shell. 21.14 Identify that if excess charge is put on a spherical conductor, it spreads out uniformly over the external surface area. 21.15 Identify that if two identical spherical conductors touch or are connected by conducting wire, any excess charge will be shared equally. 21.16 Identify that a nonconducting object can have any given distribution of charge, including charge at interior points. 21.17 Identify current as the rate at which charge moves through a point. 21.18 For current through a point, apply the relationship between the current, a time interval, and the amount of charge that moves through the point in that time interval.

Key Ideas ● The strength of a particle’s electrical interaction with ob-

jects around it depends on its electric charge (usually represented as q), which can be either positive or negative. Particles with the same sign of charge repel each other, and particles with opposite signs of charge attract each other. ● An object with equal amounts of the two kinds of charge is

electrically neutral, whereas one with an imbalance is electrically charged and has an excess charge. ● Conductors are materials in which a significant number of

electrons are free to move. The charged particles in nonconductors (insulators) are not free to move. ● Electric current i is the rate dq/dt at which charge passes a

point: i#

dq . dt

● Coulomb’s law describes the electrostatic force (or electric

force) between two charged particles. If the particles have charges q1 and q2, are separated by distance r, and are at rest (or moving only slowly) relative to each other, then the magnitude of the force acting on each due to the other is given by F#

1 !q1! !q2! 4p´0 r2

(Coulomb’s law),

where ´0 # 8.85 ' 10$12 C2/N9m2 is the permittivity constant. The ratio 1/4p´0 is often replaced with the electrostatic constant (or Coulomb constant) k # 8.99 ' 109 N9m2/C2. ● The electrostatic force vector acting on a charged particle

due to a second charged particle is either directly toward the second particle (opposite signs of charge) or directly away from it (same sign of charge). ● If multiple electrostatic forces act on a particle, the net force

is the vector sum (not scalar sum) of the individual forces. 609

610

CHAPTE R 21 COU LOM B’S L AW

● Shell theorem 1: A charged particle outside a shell with charge

uniformly distributed on its surface is attracted or repelled as if the shell's charge were concentrated as a particle at its center. ● Shell theorem 2: A charged particle inside a shell with

charge uniformly distributed on its surface has no net force acting on it due to the shell. ● Charge on a conducting spherical shell spreads uniformly over the (external) surface.

What Is Physics?

F Glass Glass –F (a)

You are surrounded by devices that depend on the physics of electromagnetism, which is the combination of electric and magnetic phenomena. This physics is at the root of computers, television, radio, telecommunications, household lighting, and even the ability of food wrap to cling to a container. This physics is also the basis of the natural world. Not only does it hold together all the atoms and molecules in the world, it also produces lightning, auroras, and rainbows. The physics of electromagnetism was first studied by the early Greek philosophers, who discovered that if a piece of amber is rubbed and then brought near bits of straw, the straw will jump to the amber. We now know that the attraction between amber and straw is due to an electric force. The Greek philosophers also discovered that if a certain type of stone (a naturally occurring magnet) is brought near bits of iron, the iron will jump to the stone. We now know that the attraction between magnet and iron is due to a magnetic force. From these modest origins with the Greek philosophers, the sciences of electricity and magnetism developed separately for centuries—until 1820, in fact, when Hans Christian Oersted found a connection between them: an electric current in a wire can deflect a magnetic compass needle. Interestingly enough, Oersted made this discovery, a big surprise, while preparing a lecture demonstration for his physics students. The new science of electromagnetism was developed further by workers in many countries. One of the best was Michael Faraday, a truly gifted experimenter with a talent for physical intuition and visualization. That talent is attested to by the fact that his collected laboratory notebooks do not contain a single equation. In the mid-nineteenth century, James Clerk Maxwell put Faraday’s ideas into mathematical form, introduced many new ideas of his own, and put electromagnetism on a sound theoretical basis. Our discussion of electromagnetism is spread through the next 16 chapters. We begin with electrical phenomena, and our first step is to discuss the nature of electric charge and electric force.

Electric Charge

F

–F

Glass Plastic

(b)

Figure 21-1 (a) The two glass rods were each rubbed with a silk cloth and one was suspended by thread. When they are close to each other, they repel each other. (b) The plastic rod was rubbed with fur. When brought close to the glass rod, the rods attract each other.

Here are two demonstrations that seem to be magic, but our job here is to make sense of them. After rubbing a glass rod with a silk cloth (on a day when the humidity is low), we hang the rod by means of a thread tied around its center (Fig. 21-la). Then we rub a second glass rod with the silk cloth and bring it near the hanging rod. The hanging rod magically moves away. We can see that a force repels it from the second rod, but how? There is no contact with that rod, no breeze to push on it, and no sound wave to disturb it. In the second demonstration we replace the second rod with a plastic rod that has been rubbed with fur. This time, the hanging rod moves toward the nearby rod (Fig. 21-1b). Like the repulsion, this attraction occurs without any contact or obvious communication between the rods. In the next chapter we shall discuss how the hanging rod knows of the presence of the other rods, but in this chapter let’s focus on just the forces that are involved. In the first demonstration, the force on the hanging rod was repulsive, and

21-1 COU LOM B’S L AW

611

in the second, attractive. After a great many investigations, scientists figured out that the forces in these types of demonstrations are due to the electric charge that we set up on the rods when they are in contact with silk or fur. Electric charge is an intrinsic property of the fundamental particles that make up objects such as the rods, silk, and fur. That is, charge is a property that comes automatically with those particles wherever they exist. Two Types. There are two types of electric charge, named by the American scientist and statesman Benjamin Franklin as positive charge and negative charge. He could have called them anything (such as cherry and walnut), but using algebraic signs as names comes in handy when we add up charges to find the net charge. In most everyday objects, such as a mug, there are about equal numbers of negatively charged particles and positively charged particles, and so the net charge is zero, the charge is said to be balanced, and the object is said to be electrically neutral (or just neutral for short). Excess Charge. Normally you are approximately neutral. However, if you live in regions where the humidity is low, you know that the charge on your body can become slightly unbalanced when you walk across certain carpets. Either you gain negative charge from the carpet (at the points of contact between your shoes with the carpet) and become negatively charged, or you lose negative charge and become positively charged. Either way, the extra charge is said to be an excess charge.You probably don’t notice it until you reach for a door handle or another person. Then, if your excess charge is enough, a spark leaps between you and the other object, eliminating your excess charge. Such sparking can be annoying and even somewhat painful. Such charging and discharging does not happen in humid conditions because the water in the air neutralizes your excess charge about as fast as you acquire it. Two of the grand mysteries in physics are (1) why does the universe have particles with electric charge (what is it, really?) and (2) why does electric charge come in two types (and not, say, one type or three types). We just do not know. Nevertheless, with lots of experiments similar to our two demonstrations scientists discovered that Particles with the same sign of electrical charge repel each other, and particles with opposite signs attract each other.

In a moment we shall put this rule into quantitative form as Coulomb’s law of electrostatic force (or electric force) between charged particles. The term electrostatic is used to emphasize that, relative to each other, the charges are either stationary or moving only very slowly. Demos. Now let’s get back to the demonstrations to understand the motions of the rod as being something other than just magic. When we rub the glass rod with a silk cloth, a small amount of negative charge moves from the rod to the silk (a transfer like that between you and a carpet), leaving the rod with a small amount of excess positive charge. (Which way the negative charge moves is not obvious and requires a lot of experimentation.) We rub the silk over the rod to increase the number of contact points and thus the amount, still tiny, of transferred charge. We hang the rod from the thread so as to electrically isolate it from its surroundings (so that the surroundings cannot neutralize the rod by giving it enough negative charge to rebalance its charge). When we rub the second rod with the silk cloth, it too becomes positively charged. So when we bring it near the first rod, the two rods repel each other (Fig. 21-2a). Next, when we rub the plastic rod with fur, it gains excess negative charge from the fur. (Again, the transfer direction is learned through many experiments.) When we bring the plastic rod (with negative charge) near the hanging glass rod (with positive charge), the rods are attracted to each other (Fig. 21-2b). All this is subtle.You cannot see the charge or its transfer, only the results.

F +++ ++++++++ + +

Glass

+ + +++++ ++++++++ + +

Glass –F

(a)

+++ Glass ++++++++ + + –F F – –––––– Plastic –––––––– –

(b)

Figure 21-2 (a) Two charged rods of the same sign repel each other. (b) Two charged rods of opposite signs attract each other. Plus signs indicate a positive net charge, and minus signs indicate a negative net charge.

612

CHAPTE R 21 COU LOM B’S L AW

Conductors and Insulators

––––––– – – – + + + ++ Neutral copper +++ + + –F F – – – – –––––––– –– – Charged plastic

Figure 21-3 A neutral copper rod is electrically isolated from its surroundings by being suspended on a nonconducting thread. Either end of the copper rod will be attracted by a charged rod. Here, conduction electrons in the copper rod are repelled to the far end of that rod by the negative charge on the plastic rod. Then that negative charge attracts the remaining positive charge on the near end of the copper rod, rotating the copper rod to bring that near end closer to the plastic rod.

We can classify materials generally according to the ability of charge to move through them. Conductors are materials through which charge can move rather freely; examples include metals (such as copper in common lamp wire), the human body, and tap water. Nonconductors — also called insulators — are materials through which charge cannot move freely; examples include rubber (such as the insulation on common lamp wire), plastic, glass, and chemically pure water. Semiconductors are materials that are intermediate between conductors and insulators; examples include silicon and germanium in computer chips. Superconductors are materials that are perfect conductors, allowing charge to move without any hindrance. In these chapters we discuss only conductors and insulators. Conducting Path. Here is an example of how conduction can eliminate excess charge on an object. If you rub a copper rod with wool, charge is transferred from the wool to the rod. However, if you are holding the rod while also touching a faucet, you cannot charge the rod in spite of the transfer. The reason is that you, the rod, and the faucet are all conductors connected, via the plumbing, to Earth’s surface, which is a huge conductor. Because the excess charges put on the rod by the wool repel one another, they move away from one another by moving first through the rod, then through you, and then through the faucet and plumbing to reach Earth’s surface, where they can spread out.The process leaves the rod electrically neutral. In thus setting up a pathway of conductors between an object and Earth’s surface, we are said to ground the object, and in neutralizing the object (by eliminating an unbalanced positive or negative charge), we are said to discharge the object. If instead of holding the copper rod in your hand, you hold it by an insulating handle, you eliminate the conducting path to Earth, and the rod can then be charged by rubbing (the charge remains on the rod), as long as you do not touch it directly with your hand. Charged Particles. The properties of conductors and insulators are due to the structure and electrical nature of atoms. Atoms consist of positively charged protons, negatively charged electrons, and electrically neutral neutrons. The protons and neutrons are packed tightly together in a central nucleus. The charge of a single electron and that of a single proton have the same magnitude but are opposite in sign. Hence, an electrically neutral atom contains equal numbers of electrons and protons. Electrons are held near the nucleus because they have the electrical sign opposite that of the protons in the nucleus and thus are attracted to the nucleus. Were this not true, there would be no atoms and thus no you. When atoms of a conductor like copper come together to form the solid, some of their outermost (and so most loosely held) electrons become free to wander about within the solid, leaving behind positively charged atoms ( positive ions). We call the mobile electrons conduction electrons. There are few (if any) free electrons in a nonconductor. Induced Charge. The experiment of Fig. 21-3 demonstrates the mobility of charge in a conductor. A negatively charged plastic rod will attract either end of an isolated neutral copper rod. What happens is that many of the conduction electrons in the closer end of the copper rod are repelled by the negative charge on the plastic rod. Some of the conduction electrons move to the far end of the copper rod, leaving the near end depleted in electrons and thus with an unbalanced positive charge. This positive charge is attracted to the negative charge in the plastic rod. Although the copper rod is still neutral, it is said to have an induced charge, which means that some of its positive and negative charges have been separated due to the presence of a nearby charge. Similarly, if a positively charged glass rod is brought near one end of a neutral copper rod, induced charge is again set up in the neutral copper rod but now the near end gains conduction electrons, becomes negatively charged, and is attracted to the glass rod, while the far end is positively charged.

613

21-1 COU LOM B’S L AW

Note that only conduction electrons, with their negative charges, can move; positive ions are fixed in place. Thus, an object becomes positively charged only through the removal of negative charges.

A

N2

Blue Flashes from a Wintergreen LifeSaver Indirect evidence for the attraction of charges with opposite signs can be seen with a wintergreen LifeSaver (the candy shaped in the form of a marine lifesaver). If you adapt your eyes to darkness for about 15 minutes and then have a friend chomp on a piece of the candy in the darkness, you will see a faint blue flash from your friend’s mouth with each chomp. Whenever a chomp breaks a sugar crystal into pieces, each piece will probably end up with a different number of electrons. Suppose a crystal breaks into pieces A and B, with A ending up with more electrons on its surface than B (Fig. 21-4). This means that B has positive ions (atoms that lost electrons to A) on its surface. Because the electrons on A are strongly attracted to the positive ions on B, some of those electrons jump across the gap between the pieces. As A and B move away from each other, air (primarily nitrogen, N2) flows into the gap, and many of the jumping electrons collide with nitrogen molecules in the air, causing the molecules to emit ultraviolet light. You cannot see this type of light. However, the wintergreen molecules on the surfaces of the candy pieces absorb the ultraviolet light and then emit blue light, which you can see — it is the blue light coming from your friend’s mouth.

B

+ + + + + + +

Figure 21-4 Two pieces of a wintergreen LifeSaver candy as they fall away from each other. Electrons jumping from the negative surface of piece A to the positive surface of piece B collide with nitrogen (N2) molecules in the air.

Always draw the force vector with the tail on the particle.

Checkpoint 1 The figure shows five pairs of plates: A, B, and A C C D B D are charged plastic plates and C is an electrically neutral copper B A D A D plate.The electrostatic forces between the pairs of plates are shown for three of the pairs. For the remaining two pairs, do the plates repel or attract each other?

The forces push the particles apart.

(a )

Here too.

(b )

But here the forces pull the particles together.

(c )

Coulomb’s Law Now we come to the equation for Coulomb’s law, but first a caution. This equation works for only charged particles (and a few other things that can be treated as particles). For extended objects, with charge located in many different places, we need more powerful techniques. So, here we consider just charged particles and not, say, two charged cats. If two charged particles are brought near each other, they each exert an electrostatic force on the other. The direction of the force vectors depends on the signs of the charges. If the particles have the same sign of charge, they repel each other. That means that the force vector on each is directly away from the other particle (Figs. 21-5a and b). If we release the particles, they accelerate away from each other. If, instead, the particles have opposite signs of charge, they attract each other. That means that the force vector on each is directly toward the other particle (Fig. 21-5c). If we release the particles, they accelerate toward each other. The equation for the electrostatic forces acting on the particles is called Coulomb’s law after Charles-Augustin de Coulomb, whose experiments in 1785 led him to it. Let’s write the equation in vector form and in terms of the particles shown in Fig. 21-6, where particle 1 has charge q1 and particle 2 has charge q2. (These symbols can represent either positive or negative charge.) Let’s also focus on particle 1 and write the force acting on it in terms of a unit vector rˆ that points along a radial

– – – – – ––

Figure 21-5 Two charged particles repel each other if they have the same sign of charge, either (a) both positive or (b) both negative. (c) They attract each other if they have opposite signs of charge.

F

r q1

ˆr

q2

Figure 21-6 The electrostatic force on particle 1 can be described in terms of a unit vector rˆ along an axis through the two particles, radially away from particle 2.

614

CHAPTE R 21 COU LOM B’S L AW

axis extending through the two particles, radially away from particle 2. (As with other unit vectors, rˆ has a magnitude of exactly 1 and no unit; its purpose is to point, like a direction arrow on a street sign.) With these decisions, we write the electrostatic force as q1q2 rˆ (Coulomb’s law), (21-1) r2 where r is the separation between the particles and k is a positive constant called the electrostatic constant or the Coulomb constant. (We’ll discuss k below.) Let’s first check the direction of the force on particle 1 as given by Eq. 21-1. If q1 and q2 have the same sign, then the product q1q2 gives us a positive result. So, Eq. 21-1 tells us that the force on particle 1 is in the direction of rˆ. That checks, because particle 1 is being repelled from particle 2. Next, if q1 and q2 have opposite signs, the product q1q2 gives us a negative result. So, now Eq. 21-1 tells us that the force on particle 1 is in the direction opposite rˆ. That checks because particle 1 is being attracted toward particle 2. An Aside. Here is something that is very curious. The form of Eq. 21-1 is the same as that of Newton’s equation (Eq. 13-3) for the gravitational force between two particles with masses m1 and m2 and separation r: :

F #k

m 1m 2 rˆ (Newton’s law), (21-2) r2 where G is the gravitational constant. Although the two types of forces are wildly different, both equations describe inverse square laws (the 1/r2 dependences) that involve a product of a property of the interacting particles—the charge in one case and the mass in the other. However, the laws differ in that gravitational forces are always attractive but electrostatic forces may be either attractive or repulsive, depending on the signs of the charges. This difference arises from the fact that there is only one type of mass but two types of charge. Unit. The SI unit of charge is the coulomb. For practical reasons having to do with the accuracy of measurements, the coulomb unit is derived from the SI unit ampere for electric current i. We shall discuss current in detail in Chapter 26, but here let’s just note that current i is the rate dq/dt at which charge moves past a point or through a region: dq i# (electric current). (21-3) dt :

F #G

Rearranging Eq. 21-3 and replacing the symbols with their units (coulombs C, amperes A, and seconds s) we see that 1 C # (1 A)(1 s). Force Magnitude. For historical reasons (and because doing so simplifies many other formulas), the electrostatic constant k in Eq. 21-1 is often written as 1/4p´0. Then the magnitude of the electrostatic force in Coulomb’s law becomes F#

!q1!!q2! 1 4p´0 r2

(Coulomb’s law).

(21-4)

The constants in Eqs. 21-1 and 21-4 have the value k#

1 # 8.99 ' 10 9 N9m2/C2. 4p´0

(21-5)

The quantity ´0, called the permittivity constant, sometimes appears separately in equations and is ´0 # 8.85 ' 10 $12 C 2/N 9 m2. (21-6) Working a Problem. Note that the charge magnitudes appear in Eq. 21-4, which gives us the force magnitude. So, in working problems in this chapter, we use Eq. 21-4 to find the magnitude of a force on a chosen particle due to a second

21-1 COU LOM B’S L AW

particle and we separately determine the direction of the force by considering the charge signs of the two particles. Multiple Forces. As with all forces in this book, the electrostatic force obeys the principle of superposition. Suppose we have n charged particles near a chosen particle called particle 1; then the net force on particle 1 is given by the vector sum :

:

:

:

:

:

F1,net # F12 " F13 " F14 " F15 " 9 9 9 " F1n,

(21-7)

: example, F14

in which, for is the force on particle 1 due to the presence of particle 4. This equation is the key to many of the homework problems, so let’s state it in words. If you want to know the net force acting on a chosen charged particle that is surrounded by other charged particles, first clearly identify that chosen particle and then find the force on it due to each of the other particles. Draw those force vectors in a free-body diagram of the chosen particle, with the tails anchored on the particle. (That may sound trivial, but failing to do so easily leads to errors.) Then add all those forces as vectors according to the rules of Chapter 3, not as scalars. (You cannot just willy-nilly add up their magnitudes.) The result is the net force (or resultant force) acting on the particle. Although the vector nature of the forces makes the homework problems harder than if we simply had scalars, be thankful that Eq. 21-7 works. If two force vectors did not simply add but for some reason amplified each other, the world would be very difficult to understand and manage. Shell Theories. Analogous to the shell theories for the gravitational force (Module 13-1), we have two shell theories for the electrostatic force: Shell theory 1. A charged particle outside a shell with charge uniformly distributed on its surface is attracted or repelled as if the shell’s charge were concentrated as a particle at its center. Shell theory 2. A charged particle inside a shell with charge uniformly distributed on its surface has no net force acting on it due to the shell.

(In the first theory, we assume that the charge on the shell is much greater than the particle’s charge. Thus the presence of the particle has negligible effect on the distribution of charge on the shell.)

Spherical Conductors If excess charge is placed on a spherical shell that is made of conducting material, the excess charge spreads uniformly over the (external) surface. For example, if we place excess electrons on a spherical metal shell, those electrons repel one another and tend to move apart, spreading over the available surface until they are uniformly distributed. That arrangement maximizes the distances between all pairs of the excess electrons. According to the first shell theorem, the shell then will attract or repel an external charge as if all the excess charge on the shell were concentrated at its center. If we remove negative charge from a spherical metal shell, the resulting positive charge of the shell is also spread uniformly over the surface of the shell. For example, if we remove n electrons, there are then n sites of positive charge (sites missing an electron) that are spread uniformly over the shell. According to the first shell theorem, the shell will again attract or repel an external charge as if all the shell’s excess charge were concentrated at its center.

Checkpoint 2 The figure shows two protons e p p (symbol p) and one electron (symbol e) on an axis. On the central proton, what is the direction of (a) the force due to the electron, (b) the force due to the other proton, and (c) the net force?

615

616

CHAPTE R 21 COU LOM B’S L AW

Sample Problem 21.01 Finding the net force due to two other particles This sample problem actually contains three examples, to build from basic stuff to harder stuff. In each we have the same charged particle 1. First there is a single force acting on it (easy stuff). Then there are two forces, but they are just in opposite directions (not too bad). Then there are again two forces but they are in very different directions (ah, now we have to get serious about the fact that they are vectors). The key to all three examples is to draw the forces correctly before you reach for a calculator, otherwise you may be calculating nonsense on the calculator. (Figure 21-7 is available in WileyPLUS as an animation with voiceover.) (a) Figure 21-7a shows two positively charged particles fixed in place on an x axis. The charges are q1 # 1.60 ' 10 $19 C and q2 # 3.20 ' 10 $19 C, and the particle separation is R # 0.0200 m. What are the magnitude and direction of the electrostatic force : F12 on particle 1 from particle 2? KEY IDEAS Because both particles are positively charged, particle 1 is repelled by particle 2, with a force magnitude given by Eq. 21-4. : Thus, the direction of force F12 on particle 1 is away from particle 2, in the negative direction of the x axis, as indicated in the free-body diagram of Fig. 21-7b. Two particles: Using Eq. 21-4 with separation R substituted for r, we can write the magnitude F12 of this force as

1 !q1!!q2! 4p´0 R2

F12 #

# (8.99 ' 10 9 N9m2/C2) (1.60 ' 10 $19 C)(3.20 ' 10 $19 C) (0.0200 m)2

'

# 1.15 ' 10 $24 N. :

Thus, force F12 has the following magnitude and direction (relative to the positive direction of the x axis): 1.15 ' 10 $24 N and 180-. : F12

We can also write

in unit-vector notation as

# $(1.15 ' 10 $24 N)iˆ .

: F12

KEY IDEA The presence of particle 3 does not alter the electrostatic force : on particle 1 from particle 2.Thus, force F12 still acts on particle : 1. Similarly, the force F13 that acts on particle 1 due to particle 3 is not affected by the presence of particle 2. Because particles 1

y

q1

x

q1

x

It is pushed away from particle 2.

q2

x

q1

F12

F13

q2

x

(e)

This is still the particle of interest. x (d)

It is pulled toward particle 3. It is pushed away from particle 2.

This is the third arrangement.

θ

(c)

This is the particle of interest. (b)

q3

__ 3 4R

__ 3 4R

R (a)

F12

q4

This is the second arrangement. q2

(Answer)

(b) Figure 21-7c is identical to Fig. 21-7a except that particle 3 now lies on the x axis between particles 1 and 2. Particle 3 has charge q3 # $3.20 ' 10 $19 C and is at a distance 34 R from : particle 1.What is the net electrostatic force F1,net on particle 1 due to particles 2 and 3?

A This is the first arrangement.

(Answer)

y F12

θ

F14

This is still the particle of interest. x

It is pulled toward (f ) particle 4. It is pushed away from particle 2.

Figure 21-7 (a) Two charged particles of charges q1 and q2 are fixed in place on an x axis. (b) The free-body diagram for particle 1, showing the electrostatic force on it from particle 2. (c) Particle 3 included. (d) Free-body diagram for particle 1. (e) Particle 4 included. (f ) Free-body diagram for particle 1.

617

21-1 COU LOM B’S L AW

and 3 have charge of opposite signs, particle 1 is attracted : to particle 3.Thus, force F13 is directed toward particle 3, as indicated in the free-body diagram of Fig. 21-7d. :

Three particles: To find the magnitude of F13, we can rewrite Eq. 21-4 as F13 #

!q1!!q3! 1 4p´0 (34R)2 (1.60 ' 10 $19 C)(3.20 ' 10 $19 C) (34)2(0.0200 m)2

We can also write

in unit-vector notation:

: F13

# (2.05 ' 10

$24

: F14 # (F14 cos u)iˆ " (F14 sin u)jˆ .

N)iˆ .

: F1,net

: F12

The net force on particle 1 is the vector sum of : and F13; that is, from Eq. 21-7, we can write the net force : F1,net on particle 1 in unit-vector notation as : F1,net

#

: F12

"

: F13

Substituting 2.05 ' 10$24 N for F14 and 60- for u, this becomes : F14 # (1.025 ' 10 $24 N)iˆ " (1.775 ' 10 $24 N)jˆ.

Then we sum: :

:

:

F1,net # F12 " F14

# $(1.15 ' 10 $24 N)iˆ " (2.05 ' 10 $24 N)iˆ # (9.00 ' 10 $25 N)iˆ . (Answer) : Thus, F1,net

has the following magnitude and direction (relative to the positive direction of the x axis): 9.00 ' 10 $25 N and 0-.

(Answer)

(c) Figure 21-7e is identical to Fig. 21-7a except that particle 4 is now included. It has charge q4 # $3.20 ' 10 $19 C, is at a distance 34 R from particle 1, and lies on a line that makes an angle u # 60- with the x axis. What is the net electrostatic : force F1,net on particle 1 due to particles 2 and 4?

# $(1.15 ' 10 $24 N)iˆ " (1.025 ' 10 $24 N)iˆ " (1.775 ' 10 $24 N)jˆ % ($1.25 ' 10 $25 N)iˆ " (1.78 ' 10 $24 N)jˆ . (Answer) Method 3. Summing components axis by axis. The sum of the x components gives us F1,net,x # F12,x " F14,x # F12 " F14 cos 60# $1.15 ' 10 $24 N " (2.05 ' 10 $24 N)(cos 60-) # $1.25 ' 10 $25 N. The sum of the y components gives us

KEY IDEA : F1,net

: F12

The net force is the vector sum of and a new force : F14 acting on particle 1 due to particle 4. Because particles 1 and 4 have charge of opposite signs, particle 1 is attracted to : particle 4. Thus, force F14 on particle 1 is directed toward particle 4, at angle u # 60-, as indicated in the free-body diagram of Fig. 21-7f. Four particles: We can rewrite Eq. 21-4 as 1 !q1!!q4! 4p´0 (34R)2 # (8.99 ' 10 9 N9m2/C2) (1.60 ' 10 $19 C)(3.20 ' 10 $19 C) ' (34)2(0.0200 m)2 # 2.05 ' 10 $24 N.

F14 #

:

Method 2. Summing in unit-vector notation. First we : rewrite F14 as

# 2.05 ' 10 $24 N. : F13

:

Because the forces F12 and F14 are not directed along the same axis, we cannot sum simply by combining their magnitudes. Instead, we must add them as vectors, using one of the following methods. Method 1. Summing directly on a vector-capable calculator. : For F12, we enter the magnitude 1.15 ' 10$24 and the angle : 180-. For F14, we enter the magnitude 2.05 ' 10$24 and the angle 60-.Then we add the vectors.

# (8.99 ' 10 9 N9m2/C2) '

:

Then from Eq. 21-7, we can write the net force F1,net on particle 1 as : : : F1,net # F12 " F14.

F1,net,y # F12,y " F14,y # 0 " F14 sin 60# (2.05 ' 10 $24 N)(sin 60-) # 1.78 ' 10 $24 N. The net force

: F1,net

has the magnitude

2 2 F1,net # 2F 1,net,x " F 1,net,y # 1.78 ' 10 $24 N.

To find the direction of

: F1,net, we

u # tan$1

(Answer)

take

F1,net,y F1,net,x

# $86.0-. :

However, this is an unreasonable result because F1,net must : : have a direction between the directions of F12 and F14. To correct u, we add 180-, obtaining $86.0- " 180- # 94.0-. (Answer)

Additional examples, video, and practice available at WileyPLUS

618

CHAPTE R 21 COU LOM B’S L AW

Checkpoint 3 The figure here shows three arrangements of an electron e and two protons p.(a) Rank the arrangements according to the magnitude of the net electrostatic force on the electron due to the protons,largest first.(b) In situation c,is the angle between the net force on the electron and the line labeled d less than or more than 45-?

d e

d

p

p

p

D e

(a)

(c)

:

:

If F 1 is the force on the proton due to charge q1 and F 2 is the force on the proton due to charge q2, then the point we seek is : : where F 1 " F 2 # 0. Thus, :

:

F 1 # $ F 2.

Pushed away from q1, pulled toward q2.

y q2

q1

P

(a)

q2

x

F1

The forces cannot cancel (same direction).

(b) y

y q2 x

q1

q2

q1

F1

S

(c)

The forces cannot cancel (d) (one is definitely larger).

(21-8)

F2

q1

x

L

F2

p

(b)

y

KEY IDEA

p

d

p

Sample Problem 21.02 Equilibrium of two forces on a particle Figure 21-8a shows two particles fixed in place: a particle of charge q1 # "8q at the origin and a particle of charge q2 # $2q at x # L. At what point (other than infinitely far away) can a proton be placed so that it is in equilibrium (the net force on it is zero)? Is that equilibrium stable or unstable? (That is, if the proton is displaced, do the forces drive it back to the point of equilibrium or drive it farther away?)

D

e

D

F2

x R

F1

The forces can cancel, at the right distance.

This tells us that at the point we seek, the forces acting on the proton due to the other two particles must be of equal magnitudes, F1 # F2, (21-9)

Figure 21-8 (a) Two particles of charges q1 and q2 are fixed in place on an x axis, with separation L. (b) – (d) Three possible locations P, S, : and R for a proton. At each location, F1 is the force on the proton : from particle 1 and F2 is the force on the proton from particle 2.

and that the forces must have opposite directions.

Calculations: With Eq.21-4,we can now rewrite Eq.21-9:

Reasoning: Because a proton has a positive charge, the proton and the particle of charge q1 are of the same sign, and : force F 1 on the proton must point away from q1. Also, the proton and the particle of charge q2 are of opposite signs, so : force F 2 on the proton must point toward q2. “Away from q1” and “toward q2” can be in opposite directions only if the proton is located on the x axis. If the proton is on the x axis at any point between q1 and : : q2, such as point P in Fig. 21-8b, then F 1 and F 2 are in the same direction and not in opposite directions as required. If the proton is at any point on the x axis to the left of q1, : : such as point S in Fig. 21-8c, then F 1 and F 2 are in opposite : : directions. However, Eq. 21-4 tells us that F 1 and F 2 cannot have equal magnitudes there: F1 must be greater than F2, because F1 is produced by a closer charge (with lesser r) of greater magnitude (8q versus 2q). Finally, if the proton is at any point on the x axis to the : : right of q2, such as point R in Fig. 21-8d, then F 1 and F 2 are again in opposite directions. However, because now the charge of greater magnitude (q1) is farther away from the proton than the charge of lesser magnitude, there is a point at which F1 is equal to F2. Let x be the coordinate of this point, and let qp be the charge of the proton.

2qqp 1 1 8qqp # . 4p´0 x2 4p´0 (x $ L)2

(21-10)

(Note that only the charge magnitudes appear in Eq. 21-10. We already decided about the directions of the forces in drawing Fig. 21-8d and do not want to include any positive or negative signs here.) Rearranging Eq. 21-10 gives us

# x $x L $ # 14 . 2

After taking the square roots of both sides, we find 1 x$L # x 2 and

x # 2L.

(Answer)

The equilibrium at x # 2L is unstable; that is, if the proton is displaced leftward from point R, then F1 and F2 both increase but F2 increases more (because q2 is closer than q1), and a net force will drive the proton farther leftward. If the proton is displaced rightward, both F1 and F2 decrease but F2 decreases more, and a net force will then drive the proton farther rightward. In a stable equilibrium, if the proton is displaced slightly, it returns to the equilibrium position.

Additional examples, video, and practice available at WileyPLUS

619

21-2 CHARG E IS QUANTI Z E D

Sample Problem 21.03 Charge sharing by two identical conducting spheres In Fig. 21-9a, two identical, electrically isolated conducting spheres A and B are separated by a (center-to-center) distance a that is large compared to the spheres. Sphere A has a positive charge of "Q, and sphere B is electrically neutral. Initially, there is no electrostatic force between the spheres. (The large separation means there is no induced charge.) (a) Suppose the spheres are connected for a moment by a conducting wire. The wire is thin enough so that any net charge on it is negligible. What is the electrostatic force between the spheres after the wire is removed? KEY IDEAS (1) Because the spheres are identical, connecting them means that they end up with identical charges (same sign and same amount). (2) The initial sum of the charges (including the signs of the charges) must equal the final sum of the charges. Reasoning: When the spheres are wired together, the (negative) conduction electrons on B, which repel one another, have a way to move away from one another (along the wire to positively charged A, which attracts them—Fig. 21-9b). As B loses negative charge, it becomes positively charged, and as A gains negative charge, it becomes less positively charged. The transfer of charge stops when the charge on B has increased to "Q/2 and the charge on A has decreased to "Q/2, which occurs when $Q/2 has shifted from B to A. After the wire has been removed (Fig. 21-9c), we can assume that the charge on either sphere does not disturb the uniformity of the charge distribution on the other sphere, because the spheres are small relative to their separation.Thus, we can apply the first shell theorem to each sphere. By Eq. 21-4 with q1 # q2 # Q/2 and r # a,

B

+Q/2

q=0

+Q/2

+Q/2

–Q/2

a A

+Q

(a )

+Q/2

(b )

(c )

–Q/2 (d )

q=0

(e )

Figure 21-9 Two small conducting spheres A and B. (a) To start, sphere A is charged positively. (b) Negative charge is transferred from B to A through a connecting wire. (c) Both spheres are then charged positively. (d) Negative charge is transferred through a grounding wire to sphere A. (e) Sphere A is then neutral.

F#

1 (Q/2)(Q/2) 1 # 4p´0 a2 16p´0

# Qa $ . 2

(Answer)

The spheres, now positively charged, repel each other. (b) Next, suppose sphere A is grounded momentarily, and then the ground connection is removed. What now is the electrostatic force between the spheres? Reasoning: When we provide a conducting path between a charged object and the ground (which is a huge conductor), we neutralize the object. Were sphere A negatively charged, the mutual repulsion between the excess electrons would cause them to move from the sphere to the ground. However, because sphere A is positively charged, electrons with a total charge of $Q/2 move from the ground up onto the sphere (Fig. 21-9d), leaving the sphere with a charge of 0 (Fig. 21-9e). Thus, the electrostatic force is again zero.

Additional examples, video, and practice available at WileyPLUS

21-2 CHARGE IS QUANTIZED Learning Objectives After reading this module, you should be able to . . .

21.19 Identify the elementary charge.

21.20 Identify that the charge of a particle or object must be a positive or negative integer times the elementary charge.

Key Ideas ● Electric charge is quantized (restricted to certain values). ● The charge of a particle can be written as ne, where n is a

which is the magnitude of the charge of the electron and proton (% 1.602 ' 10$19 C).

positive or negative integer and e is the elementary charge,

Charge Is Quantized In Benjamin Franklin’s day, electric charge was thought to be a continuous fluid — an idea that was useful for many purposes. However, we now know that

620

CHAPTE R 21 COU LOM B’S L AW

fluids themselves, such as air and water, are not continuous but are made up of atoms and molecules; matter is discrete. Experiment shows that “electrical fluid” is also not continuous but is made up of multiples of a certain elementary charge. Any positive or negative charge q that can be detected can be written as q # ne,

n # !1, !2, !3, . . . ,

(21-11)

in which e, the elementary charge, has the approximate value e # 1.602 ' 10 $19 C.

Table 21-1 The Charges of Three Particles Particle

Symbol

Charge

Electron Proton Neutron

e or e$ p n

$e "e 0

(21-12)

The elementary charge e is one of the important constants of nature. The electron and proton both have a charge of magnitude e (Table 21-1). (Quarks, the constituent particles of protons and neutrons, have charges of !e/3 or !2e/3, but they apparently cannot be detected individually. For this and for historical reasons, we do not take their charges to be the elementary charge.) You often see phrases — such as “the charge on a sphere,” “the amount of charge transferred,” and “the charge carried by the electron” — that suggest that charge is a substance. (Indeed, such statements have already appeared in this chapter.) You should, however, keep in mind what is intended: Particles are the substance and charge happens to be one of their properties, just as mass is. When a physical quantity such as charge can have only discrete values rather than any value, we say that the quantity is quantized. It is possible, for example, to find a particle that has no charge at all or a charge of "10e or $6e, but not a particle with a charge of, say, 3.57e. The quantum of charge is small. In an ordinary 100 W lightbulb, for example, about 10 19 elementary charges enter the bulb every second and just as many leave. However, the graininess of electricity does not show up in such large-scale phenomena (the bulb does not flicker with each electron).

Checkpoint 4 Initially, sphere A has a charge of $50e and sphere B has a charge of "20e. The spheres are made of conducting material and are identical in size. If the spheres then touch, what is the resulting charge on sphere A?

Sample Problem 21.04 Mutual electric repulsion in a nucleus The nucleus in an iron atom has a radius of about 4.0 ' 10 $15 m and contains 26 protons. (a) What is the magnitude of the repulsive electrostatic force between two of the protons that are separated by 4.0 ' 10 $15 m? KEY IDEA The protons can be treated as charged particles, so the magnitude of the electrostatic force on one from the other is given by Coulomb’s law. Calculation: Table 21-1 tells us that the charge of a proton is "e. Thus, Eq. 21-4 gives us 1 e2 F# 4p´0 r2 (8.99 ' 10 9 N9m2/C2)(1.602 ' 10 $19 C)2 # (4.0 ' 10 $15 m)2 # 14 N. (Answer)

No explosion: This is a small force to be acting on a macroscopic object like a cantaloupe, but an enormous force to be acting on a proton. Such forces should explode the nucleus of any element but hydrogen (which has only one proton in its nucleus). However, they don’t, not even in nuclei with a great many protons. Therefore, there must be some enormous attractive force to counter this enormous repulsive electrostatic force. (b) What is the magnitude of the gravitational force between those same two protons? KEY IDEA Because the protons are particles, the magnitude of the gravitational force on one from the other is given by Newton’s equation for the gravitational force (Eq. 21-2). Calculation: With mp (# 1.67 ' 10 $27 kg) representing the

21-3 CHARG E IS CONSE RVE D

mass of a proton, Eq. 21-2 gives us F#G #

m2p r2

(6.67 ' 10 $11 N9m2/kg2)(1.67 ' 10 $27 kg)2 (4.0 ' 10 $15 m)2

# 1.2 ' 10 $35 N.

(Answer)

Weak versus strong: This result tells us that the (attractive) gravitational force is far too weak to counter the repulsive electrostatic forces between protons in a nucleus. Instead, the protons are bound together by an enormous force called

621

(aptly) the strong nuclear force — a force that acts between protons (and neutrons) when they are close together, as in a nucleus. Although the gravitational force is many times weaker than the electrostatic force, it is more important in largescale situations because it is always attractive.This means that it can collect many small bodies into huge bodies with huge masses, such as planets and stars, that then exert large gravitational forces. The electrostatic force, on the other hand, is repulsive for charges of the same sign, so it is unable to collect either positive charge or negative charge into large concentrations that would then exert large electrostatic forces.

Additional examples, video, and practice available at WileyPLUS

21-3 CHARGE IS CONSERVED Learning Objectives After reading this module, you should be able to . . .

21.21 Identify that in any isolated physical process, the net charge cannot change (the net charge is always conserved).

Key Ideas ● The net electric charge of any isolated system is always

conserved. ● If two charged particles undergo an annihilation process,

21.22 Identify an annihilation process of particles and a pair production of particles. 21.23 Identify mass number and atomic number in terms of the number of protons, neutrons, and electrons. they have opposite signs of charge. ● If two charged particles appear as a result of a pair production process, they have opposite signs of charge.

Charge Is Conserved If you rub a glass rod with silk, a positive charge appears on the rod. Measurement shows that a negative charge of equal magnitude appears on the silk. This suggests that rubbing does not create charge but only transfers it from one body to another, upsetting the electrical neutrality of each body during the process. This hypothesis of conservation of charge, first put forward by Benjamin Franklin, has stood up under close examination, both for large-scale charged bodies and for atoms, nuclei, and elementary particles. No exceptions have ever been found. Thus, we add electric charge to our list of quantities — including energy and both linear momentum and angular momentum — that obey a conservation law. Important examples of the conservation of charge occur in the radioactive decay of nuclei, in which a nucleus transforms into (becomes) a different type of nucleus. For example, a uranium-238 nucleus (238U) transforms into a thorium234 nucleus (234Th) by emitting an alpha particle. Because that particle has the same makeup as a helium-4 nucleus, it has the symbol 4He. The number used in the name of a nucleus and as a superscript in the symbol for the nucleus is called the mass number and is the total number of the protons and neutrons in the nucleus. For example, the total number in 238U is 238. The number of protons in a nucleus is the atomic number Z, which is listed for all the elements in Appendix F. From that list we find that in the decay 238

U : 234Th " 4He,

(21-13)

622

Courtesy Lawrence Berkeley Laboratory

CHAPTE R 21 COU LOM B’S L AW

the parent nucleus 238U contains 92 protons (a charge of "92e), the daughter nucleus 234Th contains 90 protons (a charge of "90e), and the emitted alpha particle 4He contains 2 protons (a charge of "2e). We see that the total charge is "92e before and after the decay; thus, charge is conserved. (The total number of protons and neutrons is also conserved: 238 before the decay and 234 " 4 # 238 after the decay.) Another example of charge conservation occurs when an electron e$ (charge $e) and its antiparticle, the positron e" (charge "e), undergo an annihilation process, transforming into two gamma rays (high-energy light): e$ " e" : g " g e

e

(annihilation).

(21-14)

In applying the conservation-of-charge principle, we must add the charges algebraically, with due regard for their signs. In the annihilation process of Eq. 21-14 then, the net charge of the system is zero both before and after the event. Charge is conserved. In pair production, the converse of annihilation, charge is also conserved. In this process a gamma ray transforms into an electron and a positron: g : e $ " e"

Figure 21-10 A photograph of trails of bubbles left in a bubble chamber by an electron and a positron. The pair of particles was produced by a gamma ray that entered the chamber directly from the bottom. Being electrically neutral, the gamma ray did not generate a telltale trail of bubbles along its path, as the electron and positron did.

(pair production).

(21-15)

Figure 21-10 shows such a pair-production event that occurred in a bubble chamber. (This is a device in which a liquid is suddenly made hotter than its boiling point. If a charged particle passes through it, tiny vapor bubbles form along the particle’s trail.) A gamma ray entered the chamber from the bottom and at one point transformed into an electron and a positron. Because those new particles were charged and moving, each left a trail of bubbles. (The trails were curved because a magnetic field had been set up in the chamber.) The gamma ray, being electrically neutral, left no trail. Still, you can tell exactly where it underwent pair production — at the tip of the curved V, which is where the trails of the electron and positron begin.

Review & Summary Electric Charge The strength of a particle’s electrical interaction with objects around it depends on its electric charge (usually represented as q), which can be either positive or negative. Particles with the same sign of charge repel each other, and particles with opposite signs of charge attract each other. An object with equal amounts of the two kinds of charge is electrically neutral, whereas one with an imbalance is electrically charged and has an excess charge. Conductors are materials in which a significant number of electrons are free to move. The charged particles in nonconductors (insulators) are not free to move. Electric current i is the rate dq/dt at which charge passes a point: dq i# (electric current). (21-3) dt

Coulomb’s Law Coulomb’s law describes the electrostatic force (or electric force) between two charged particles. If the particles have charges q1 and q2, are separated by distance r, and are at rest (or moving only slowly) relative to each other, then the magnitude of the force acting on each due to the other is given by 1 !q1! !q2! F# 4p´0 r2

(Coulomb’s law),

(21-4)

where ´0 # 8.85 ' 10$12 C2/N9m2 is the permittivity constant. The ratio 1/4p´0 is often replaced with the electrostatic constant (or Coulomb constant) k # 8.99 ' 109 N9m2/C2.

The electrostatic force vector acting on a charged particle due to a second charged particle is either directly toward the second particle (opposite signs of charge) or directly away from it (same sign of charge).As with other types of forces, if multiple electrostatic forces act on a particle, the net force is the vector sum (not scalar sum) of the individual forces. The two shell theories for electrostatics are Shell theorem 1: A charged particle outside a shell with charge uniformly distributed on its surface is attracted or repelled as if the shell’s charge were concentrated as a particle at its center. Shell theorem 2: A charged particle inside a shell with charge uniformly distributed on its surface has no net force acting on it due to the shell. Charge on a conducting spherical shell spreads uniformly over the (external) surface.

The Elementary Charge Electric charge is quantized (restricted to certain values). The charge of a particle can be written as ne, where n is a positive or negative integer and e is the elementary charge, which is the magnitude of the charge of the electron and proton (% 1.602 ' 10 $19 C).

Conservation of Charge The net electric charge of any isolated system is always conserved.

623

QU ESTIONS

Questions 1 Figure 21-11 shows (1) –e –e +e four situations in which five charged particles are (2) +e +e +e evenly spaced along an axis. The charge values (3) are indicated except for –e –e +e the central particle, which has the same charge in all (4) –e +e +e four situations. Rank the situations according to Figure 21-11 Question 1. the magnitude of the net electrostatic force on the central particle, greatest first.

–e –e +e

ground connection is removed and (b) the ground connection is first removed and then the ball is taken away? 7 Figure 21-16 shows three situations involving a charged particle and a uniformly charged spherical shell. The charges are given, and the radii of the shells are indicated. Rank the situations according to the magnitude of the force on the particle due to the presence of the shell, greatest first.

–e

+2q +6q

2 Figure 21-12 shows three pairs of identical spheres that are to be touched together and then separated. The initial charges on them are indicated. Rank the pairs according to (a) the magnitude of the charge transferred during touching and (b) the charge left on the positively charged sphere, greatest first.

+6e

–4e

0

(1)

+2e

–12e

(2)

–q

d R –4Q

R/2 +8Q

(b)

(c)

+5Q

2R

(a)

Figure 21-16 Question 7. 8 Figure 21-17 shows four arrangements of charged particles. Rank the arrangements according to the magnitude of the net electrostatic force on the particle with charge "Q, greatest first.

+14e

(3)

Figure 21-12 Question 2.

p

p

3 Figure 21-13 shows four situations in which charged particles are fixed in place on an axis. In which situations is there a point to the left of the particles where an electron will be in equilibrium?

d

d

+Q +q

–3q

–q

(a)

+3q

p

2d (a)

+3q

+Q

e

2d (b)

(b)

–q

–3q

(c)

+q (d)

Figure 21-13 Question 3.

+Q

4 Figure 21-14 shows two charged –3q –q particles on an axis. The charges are free to move. However, a third Figure 21-14 Question 4. charged particle can be placed at a certain point such that all three particles are then in equilibrium. (a) Is +4q that point to the left of the first two particles, to their right, or between +2q –2q them? (b) Should the third particle be –2q positively or negatively charged? (c) +q Is the equilibrium stable or unstable? 5 In Fig. 21-15, a central particle of charge $q is surrounded by two circular rings of charged particles. What are the magnitude and direction of the net electrostatic force on the central particle due to the other particles? (Hint: Consider symmetry.)

r

–7q

–7q

–2q

d

d

+Q

e

2d (d)

Figure 21-17 Question 8. 9 Figure 21-18 shows four situations in which particles of charge "q or $q are fixed in place. In each situation, the partiy

y –q

+q

+q

–2q

+q

x

+q

+q

(1)

(2)

y

y

x

–q

+4q

+q

Figure 21-15 Question 5.

6 A positively charged ball is brought close to an electrically neutral isolated conductor. The conductor is then grounded while the ball is kept close. Is the conductor charged positively, charged negatively, or neutral if (a) the ball is first taken away and then the

e

p

2d (c)

R

+q

e

+q

–q

x

+q

–q (4)

(3)

Figure 21-18 Question 9.

x

624

CHAPTE R 21 COU LOM B’S L AW

ducting container that is grounded by a wire. The bubbles initially have the same charge. Bubble A bumps into the container’s ceiling and then into bubble B. Then bubble B bumps into bubble C, which then drifts to the container’s floor. When bubble C reaches the floor, a charge of $3e is transferred upward through the wire, from the ground to the container, as indicated. (a) What was the initial charge of each bubble? When (b) bubble A and (c) bubble B reach the floor, what is the charge transfer through the wire? (d) During this whole process, what is the total charge transfer through the wire?

cles on the x axis are equidistant from the y axis. First, consider the middle particle in situation 1; the middle particle experiences an electrostatic force from each of the other two particles. (a) Are the magnitudes F of those forces the same or different? (b) Is the magnitude of the net force on the middle particle equal to, greater than, or less than 2F ? (c) Do the x components of the two forces add or cancel? (d) Do their y components add or cancel? (e) Is the direction of the net force on the middle particle that of the canceling components or the adding components? (f) What is the direction of that net force? Now consider the remaining situations: What is the direction of the net force on the middle particle in (g) situation 2, (h) situation 3, and (i) situation 4? (In each situation, consider the symmetry of the charge distribution and determine the canceling components and the adding components.) 10 In Fig. 21-19, a central particle of charge $2q is +2q surrounded by a square array of charged particles, separated –5q by either distance d or d/2 along the perimeter of the square. What are the magni- +3q tude and direction of the net electrostatic force on the central particle due to the other –3q particles? (Hint: Consideration of symmetry can greatly re- +4q duce the amount of work required here.)

–7q

+4q

12 Figure 21-21 shows four situations in which a central proton is partially surrounded by protons or electrons fixed in place along a half-circle. The angles u are identical; the angles f are also. (a) In each situation, what is the direction of the net force on the central proton due to the other particles? (b) Rank the four situations according to the magnitude of that net force on the central proton, greatest first.

–3q –2q

y p

p

p u

u

f

+2q

p

f

Figure 21-19 Question 10. 11 Figure 21-20 shows three identical conducting bubbles A, B, and C floating in a con-

f

x

p

p

f

u p

p

p

e

(1)

(2)

y

y e

e

x

u

p

A

e

p

p

B

e

p

p

–5q

–7q

y

e u

Grounded conducting container

f

u p

f

f

x

p

f

x

C u p

e

!3e

u e

e

p

(3)

e (4)

Figure 21-21 Question 12.

Figure 21-20 Question 11.

Problems Tutoring problem available (at instructor’s discretion) in WileyPLUS and WebAssign SSM • – •••

Worked-out solution available in Student Solutions Manual

WWW Worked-out solution is at

Number of dots indicates level of problem difficulty

ILW

Interactive solution is at

http://www.wiley.com/college/halliday

Additional information available in The Flying Circus of Physics and at flyingcircusofphysics.com

Module 21-1 Coulomb’s Law •1 SSM ILW Of the charge Q initially on a tiny sphere, a portion q is to be transferred to a second, nearby sphere. Both spheres

can be treated as particles and are fixed with a certain separation. For what value of q/Q will the electrostatic force between the two spheres be maximized?

PROB LE M S

–F

1

2

F

1 3

(a )

1

2

(b )

2 3

–F'

1

(c )

2

F'

(d )

Figure 21-22 Problem 2. •3 SSM What must be the distance between point charge q1 # 26.0 mC and point charge q2 # $47.0 mC for the electrostatic force between them to have a magnitude of 5.70 N? •4 In the return stroke of a typical lightning bolt, a current of 2.5 ' 10 4 A exists for 20 ms. How much charge is transferred in this event? •5 A particle of charge "3.00 ' 10 $6 C is 12.0 cm distant from a second particle of charge $1.50 ' 10 $6 C. Calculate the magnitude of the electrostatic force between the particles. •6 ILW Two equally charged particles are held 3.2 ' 10 $3 m apart and then released from rest. The initial acceleration of the first particle is observed to be 7.0 m/s2 and that of the second to be 9.0 m/s2. If the mass of the first particle is 6.3 ' 10 $7 kg, what are (a) the mass of the second particle and (b) the magnitude of the charge of each particle? ••7 In Fig. 21-23, three charged L 12 L 23 x particles lie on an x axis. Particles 1 2 3 1 and 2 are fixed in place. Particle 3 is free to move, but the net elec- Figure 21-23 Problems 7 and 40. trostatic force on it from particles 1 and 2 happens to be zero. If L23 # L12, what is the ratio q1/q2? ••8 In Fig. 21-24, three identical conducting spheres initially have the following charges: sphere A, 4Q; sphere B, $6Q; and sphere C, 0. Spheres A and B are fixed in place, with a center-to-center separation that is much larger than the spheres. Two experiments are conducted. In experiment 1, d sphere C is touched to sphere A A B and then (separately) to sphere B, and then it is removed. In experiment 2, starting with the same initial states, the procedure is re- C versed: Sphere C is touched to sphere B and then (separately) to Figure 21-24 sphere A, and then it is removed. Problems 8 and 65. What is the ratio of the electro-

static force between A and B at the end of experiment 2 to that at the end of experiment 1? ••9 SSM WWW Two identical conducting spheres, fixed in place, attract each other with an electrostatic force of 0.108 N when their center-to-center separation is 50.0 cm. The spheres are then connected by a thin conducting wire. When the wire is removed, the spheres repel each other with an electrostatic force of 0.0360 N. Of the initial charges on the spheres, with a positive net charge, what was (a) the negative charge on one of them and (b) the positive charge on the other? ••10 In Fig. 21-25, four particles form a square. The charges are q1 # q4 # Q and q2 # q3 # q. (a) y What is Q/q if the net electrostatic a 1 2 force on particles 1 and 4 is zero? (b) Is there any value of q that makes the net electrostatic force on each of the four particles zero? Explain. a a ••11 ILW In Fig. 21-25, the particles have charges q1 # $q2 # 100 nC and q3 # $q4 # 200 nC, and distance a # 5.0 cm. What are the (a) x and (b) y components of the net electrostatic force on particle 3?

3

a

4

x

Figure 21-25 Problems 10, 11, and 70.

••12 Two particles are fixed on an x axis. Particle 1 of charge 40 mC is located at x # $2.0 cm; particle 2 of charge Q is located at x # 3.0 cm. Particle 3 of charge magnitude 20 mC is released from rest on the y axis at y # 2.0 cm. What is the value of Q if the initial acceleration of particle 3 is in the positive direction of (a) the x axis and (b) the y axis? y ••13 In Fig. 21-26, particle 1 of 1 2 x charge "1.0 mC and particle 2 of charge L $3.0 mC are held at separation L # 10.0 cm on an x axis. If particle 3 of unFigure 21-26 Problems 13, known charge q3 is to be located such 19, 30, 58, and 67. that the net electrostatic force on it from particles 1 and 2 is zero, what must be the (a) x and (b) y coordinates of particle 3? ••14 Three particles are fixed on an x axis. Particle 1 of charge q1 is at x # $a, and particle 2 of charge q2 is at x # "a. If their net electrostatic force on particle 3 of charge "Q is to be zero, what must be the ratio q1/q2 when particle 3 is at (a) x # "0.500a and (b) x # "1.50a? ••15 The charges and coordinates of two charged particles held fixed in an xy plane are q1 # "3.0 mC, x1 # 3.5 cm, y1 # 0.50 cm, and q2 # $4.0 mC, x2 # $2.0 cm, y2 # 1.5 cm. Find the (a) magnitude and (b) direction of the electrostatic force on particle 2 due to particle 1. At what (c) x and (d) y coordinates should a third particle of charge q3 # "4.0 mC be placed such that the net electrostatic force on particle 2 due to particles 1 and 3 is zero? ••16 In Fig. 21-27a, particle 1 (of charge q1) and particle 2 (of charge q2) are fixed in place on an x axis, 8.00 cm apart. Particle 3 (of y 1

2 (a)

x

F (10–23 N)

•2 Identical isolated conducting spheres 1 and 2 have equal charges and are separated by a distance that is large compared with their diameters (Fig. 21-22a). The electrostatic force acting on : sphere 2 due to sphere 1 is F . Suppose now that a third identical sphere 3, having an insulating handle and initially neutral, is touched first to sphere 1 (Fig. 21-22b), then to sphere 2 (Fig. 21-22c), and finally removed (Fig. 21-22d). The electrostatic force that now acts on sphere 2 has magnitude F,.What is the ratio F,/F?

625

1 0

0

xs

–1 (b)

Figure 21-27 Problem 16.

x (cm)

626

CHAPTE R 21 COU LOM B’S L AW

charge q3 # "8.00 ' 10 $19 C) is to be placed on the line between par: ticles 1 and 2 so that they produce a net electrostatic force F3,net on it. Figure 21-27b gives the x component of that force versus the coordinate x at which particle 3 is placed.The scale of the x axis is set by xs # 8.0 cm.What are (a) the sign of charge q1 and (b) the ratio q2 /q1? ••17 In Fig. 21-28a, particles 1 and 2 have charge 20.0 mC each and are held at separation distance d # 1.50 m. (a) What is the magnitude of the electrostatic force on particle 1 due to particle 2? In Fig. 21-28b, particle 3 of charge 20.0 mC is positioned so as to complete an equilateral triangle. (b) What is the magnitude of the net electrostatic force on particle 1 due to particles 2 and 3?

1

d 3

d d

2 (a )

(b )

Figure 21-28 Problem 17.

x ••18 In Fig. 21-29a, three positively A BC charged particles are fixed on an x (a) axis. Particles B and C are so close to each other that they can be conx A C sidered to be at the same distance B (b) from particle A. The net force on particle A due to particles B and Figure 21-29 Problem 18. C is 2.014 ' 10 $23 N in the negative direction of the x axis. In Fig. 2129b, particle B has been moved to the opposite side of A but is still at the same distance from it. The net force on A is now 2.877 ' 10 $24 N in the negative direction of the x axis. What is the ratio qC /qB?

••19 SSM WWW In Fig. 21-26, particle 1 of charge "q and particle 2 of charge "4.00q are held at separation L # 9.00 cm on an x axis. If particle 3 of charge q3 is to be located such that the three particles remain in place when released, what must be the (a) x and (b) y coordinates of particle 3, and (c) the ratio q3 /q? •••20 Figure 21-30a shows an arrangement of three charged particles separated by distance d. Particles A and C are fixed on the x axis, but particle B can be moved along a circle centered on particle A. During the movement, a radial line between A and B makes an angle u relative to the positive direction of the x axis (Fig. 21-30b). The curves in Fig. 21-30c give, for two situations, the magnitude Fnet of the net electrostatic force on particle A due to the other particles. That net force is given as a function of angle u and as a multiple of a basic amount F0. For example on curve 1, at u # 180-, we see that Fnet # 2F0. (a) For the situation corresponding to curve 1, what is the ratio of the charge of particle C to that of particle B (including sign)? (b) For the situation corresponding to curve 2, what is that ratio? 2 d

B (a) d

C

x Fnet

d A

1

B

θ

A

C

x

1

0 0°

(b)

Figure 21-30 Problem 20.

2

90° θ (c)

180°

•••21 A nonconducting spherical shell, with an inner radius of 4.0 cm and an outer radius of 6.0 cm, has charge spread nonuniformly through its volume between its inner and outer surfaces. The volume charge density r is the charge per unit volume, with the unit coulomb per cubic meter. For this shell r # b/r, where r is the distance in meters from the center of the shell and b # 3.0 mC/m2. What is the net charge in the shell? y •••22 Figure 21-31 shows an 3 arrangement of four charged particles, with angle u ! 30.0" and disθ x tance d ! 2.00 cm. Particle 2 has θ d D 1 2 charge q2 ! #8.00 $ 10 %19 C; par4 ticles 3 and 4 have charges q3 ! q4 ! %1.60 $ 10 %19 C. (a) What is distance D between the origin and Figure 21-31 Problem 22. particle 2 if the net electrostatic force on particle 1 due to the other particles is zero? (b) If particles 3 and 4 were moved closer to the x axis but maintained their symmetry about that axis, would the required value of D be greater than, less than, or the same as in part (a)? •••23 In Fig. 21-32, particles 1 and y 2 of charge q1 # q2 # "3.20 ' 10 $19 C 1 are on a y axis at distance d # 17.0 cm d from the origin. Particle 3 of charge x q3 # "6.40 ' 10 $19 C is moved gradu3 ally along the x axis from x # 0 to x # d "5.0 m. At what values of x will the 2 magnitude of the electrostatic force on the third particle from the other two Figure 21-32 Problem 23. particles be (a) minimum and (b) maximum? What are the (c) minimum and (d) maximum magnitudes? Module 21-2 Charge Is Quantized •24 Two tiny, spherical water drops, with identical charges of $1.00 ' 10 $16 C, have a center-to-center separation of 1.00 cm. (a) What is the magnitude of the electrostatic force acting between them? (b) How many excess electrons are on each drop, giving it its charge imbalance? •25 ILW How many electrons would have to be removed from a coin to leave it with a charge of #1.0 $ 10 %7 C? •26 What is the magnitude of the electrostatic force between a singly charged sodium ion (Na#, of charge #e) and an adjacent singly charged chlorine ion (Cl%, of charge %e) in a salt crystal if their separation is 2.82 $ 10 %10 m? •27 SSM The magnitude of the electrostatic force between two identical ions that are separated by a distance of 5.0 $ 10 %10 m is 3.7 $ 10 %9 N. (a) What is the charge of each ion? (b) How many electrons are “missing” from each ion (thus giving the ion its charge imbalance)? •28 A current of 0.300 A through your chest can send your heart into fibrillation, ruining the y normal rhythm of heartbeat and disrupting the flow of blood (and 4 thus oxygen) to your brain. If that x current persists for 2.00 min, how 1 3 many conduction electrons pass through your chest? ••29 In Fig. 21-33, particles 2 and 4, of charge %e, are fixed in place on a y axis, at y2 ! %10.0 cm

2

Figure 21-33 Problem 29.

627

PROB LE M S

and y4 ! 5.00 cm. Particles 1 and 3, of charge %e, can be moved along the x axis. Particle 5, of charge #e, is fixed at the origin. Initially particle 1 is at x1 ! %10.0 cm and particle 3 is at x3 ! 10.0 cm. (a) To what x value must particle 1 be moved to rotate the : direction of the net electric force Fnet on particle 5 by 30" counterclockwise? (b) With particle 1 fixed at its new position, to what x : value must you move particle 3 to rotate Fnet back to its original direction? ••30 In Fig. 21-26, particles 1 and 2 are fixed in place on an x axis, at a separation of L ! 8.00 cm. Their charges are q1 ! #e and q2 ! %27e. Particle 3 with charge q3 ! #4e is to be placed on the line between : particles 1 and 2, so that they produce a net electrostatic force F3,net on it. (a) At what coordinate should particle 3 be placed to minimize the magnitude of that force? (b) What is that minimum magnitude?

•••35 SSM In crystals of the salt cesium chloride, cesium ions Cs# form the eight corners of a cube and a chlorine ion Cl% is at the cube’s center (Fig. 21-36). The edge length of the cube is 0.40 nm. The Cs# ions are each deficient by one electron (and thus each has a charge of #e), and the Cl% ion has one excess electron (and thus has a charge of %e). (a) What is the magnitude of the net electrostatic force exerted on the Cl% ion by the eight Cs# ions at the corners of the cube? (b) If one of the Cs# ions is missing, the crystal is said to have a defect; what is the magnitude of the net electrostatic force exerted on the Cl% ion by the seven remaining Cs# ions?

Cl– Cs+

••31 ILW Earth’s atmosphere is constantly bombarded by cosmic ray protons that originate somewhere in space. If the protons all passed through the atmosphere, each square meter of Earth’s surface would intercept protons at the average rate of 1500 protons per second. What would be the electric current intercepted by the total surface area of the planet? ••32 Figure 21-34a shows charged particles 1 and 2 that are fixed in place on an x axis. Particle 1 has a charge with a magnitude of |q1| ! 8.00e. Particle 3 of charge q3 ! #8.00e is initially on the x axis near particle 2. Then particle 3 is gradually moved in the positive direction of the x axis. As a result, the magnitude of the net : electrostatic force F2,net on particle 2 due to particles 1 and 3 changes. Figure 21-34b gives the x component of that net force as a function of the position x of particle 3. The scale of the x axis is set by xs ! 0.80 m. The plot has an asymptote of F2,net ! 1.5 $ 10 %25 N as x : &. As a multiple of e and including the sign, what is the charge q2 of particle 2? 2

1

2

3

(a)

x

F2, net (10–25 N)

y 1

0

x (m) xs

0

–1

(b)

Figure 21-34 Problem 32. ••33 Calculate the number of coulombs of positive charge in 250 cm3 of (neutral) water. (Hint: A hydrogen atom contains one proton; an oxygen atom contains eight protons.) y •••34 Figure 21-35 shows 3 –q electrons 1 and 2 on an x axis 1 2 θ and charged ions 3 and 4 of idenx –e –e θ tical charge %q and at identical –q angles u. Electron 2 is free to 4 move; the other three particles R R are fixed in place at horizontal distances R from electron 2 and Figure 21-35 Problem 34. are intended to hold electron 2 in place. For physically possible values of q ' 5e, what are the (a) smallest, (b) second smallest, and (c) third smallest values of u for which electron 2 is held in place?

0.40 nm

Figure 21-36 Problem 35. Module 21-3 Charge Is Conserved •36 Electrons and positrons are produced by the nuclear transformations of protons and neutrons known as beta decay. (a) If a proton transforms into a neutron, is an electron or a positron produced? (b) If a neutron transforms into a proton, is an electron or a positron produced? •37 SSM Identify X in the following nuclear reactions: (a) 1H # Be : X # n; (b) 12C # 1H : X; (c) 15N # 1H : 4He # X. Appendix F will help.

9

Additional Problems 38 Figure 21-37 shows four iden- A B C tical conducting spheres that are actually well separated from one another. Sphere W (with an initial charge of W zero) is touched to sphere A and then Figure 21-37 Problem 38. they are separated. Next, sphere W is touched to sphere B (with an initial charge of %32e) and then they are separated. Finally, sphere W is touched to sphere C (with an initial charge of #48e), and then they are separated. The final charge on sphere W is #18e. What was the initial charge on sphere A? y

39 SSM In Fig. 21-38, particle 1 of charge #4e is above a floor by distance d1 ! 2.00 mm and particle 2 of charge #6e is on the floor, at distance d2 ! 6.00 mm horizontally from particle 1. What is the x component of the electrostatic force on particle 2 due to particle 1?

1 d1

2

x

d2

Figure 21-38 Problem 39.

40 In Fig. 21-23, particles 1 and 2 are fixed in place, but particle 3 is free to move. If the net electrostatic force on particle 3 due to particles 1 and 2 is zero and L23 ! 2.00L12, what is the ratio q1/q2? 41 (a) What equal positive charges would have to be placed on Earth and on the Moon to neutralize their gravitational attraction? (b) Why don’t you need to know the lunar distance to solve this problem? (c) How many kilograms of hydrogen ions (that is, protons) would be needed to provide the positive charge calculated in (a)?

628

CHAPTE R 21 COU LOM B’S L AW

42 In Fig. 21-39, two tiny conducting balls of identical mass m and identical charge q hang from nonconducting threads of length L. Assume that u is so small that tan u can be replaced by its approximate equal, sin u. (a) Show that x!

What value should h have so that the rod exerts no vertical force on the bearing when the rod is horizontal and balanced? θθ

L

x +q

L

Rod Bearing

+2q

h

# 2p´q Lmg $ 2

L

1/3

+Q

W

0

gives the equilibrium separation x of the balls. (b) If L ! 120 cm, m ! 10 g, q and x ! 5.0 cm, what is |q|?

q x

43 (a) Explain what happens to the Figure 21-39 balls of Problem 42 if one of them is Problems 42 and 43. discharged (loses its charge q to, say, the ground). (b) Find the new equilibrium separation x, using the given values of L and m and the computed value of |q|. 44 SSM How far apart must two protons be if the magnitude of the electrostatic force acting on either one due to the other is equal to the magnitude of the gravitational force on a proton at Earth’s surface? 45 How many megacoulombs of positive charge are in 1.00 mol of neutral molecular-hydrogen gas (H2)? 46 In Fig. 21-40, four particles are d d d x fixed along an x axis, separated by 1 2 3 4 distances d ! 2.00 cm. The charges Figure 21-40 Problem 46. are q1 ! #2e, q2 ! %e, q3 ! #e, and q4 ! #4e, with e ! 1.60 $ 10 %19 C. In unit-vector notation, what is the net electrostatic force on (a) particle 1 and (b) particle 2 due to the other particles? 47 Point charges of #6.0 mC and %4.0 mC are placed on an x axis, at x ! 8.0 m and x ! 16 m, respectively. What charge must be placed at x ! 24 m so that any charge placed at the origin would experience no electrostatic force? 48 In Fig. 21-41, three identical conA ducting spheres form an equilateral trid d angle of side length d ! 20.0 cm. The sphere radii are much smaller than d, and the sphere charges are qA ! %2.00 B C nC, qB ! %4.00 nC, and qC ! #8.00 nC. d (a) What is the magnitude of the electroFigure 21-41 static force between spheres A and C? Problem 48. The following steps are then taken: A and B are connected by a thin wire and then disconnected; B is grounded by the wire, and the wire is then removed; B and C are connected by the wire and then disconnected. What now are the magnitudes of the electrostatic force (b) between spheres A and C and (c) between spheres B and C?

+Q

Figure 21-42 Problem 50. 51 A charged nonconducting rod, with a length of 2.00 m and a cross-sectional area of 4.00 cm2, lies along the positive side of an x axis with one end at the origin. The volume charge density r is charge per unit volume in coulombs per cubic meter. How many excess electrons are on the rod if r is (a) uniform, with a value of %4.00 mC/m3, and (b) nonuniform, with a value given by r ! bx2, where b ! %2.00 mC/m5? 52 A particle of charge Q is fixed at the origin of an xy coordinate system. At t ! 0 a particle (m ! 0.800 g, q ! 4.00 mC) is located on the x axis at x ! 20.0 cm, moving with a speed of 50.0 m/s in the positive y direction. For what value of Q will the moving particle execute circular motion? (Neglect the gravitational force on the particle.) 53 What would be the magnitude of the electrostatic force between two 1.00 C point charges separated by a distance of (a) 1.00 m and (b) 1.00 km if such point charges existed (they do not) and this configuration could be set up? 54 A charge of 6.0 mC is to be split into two parts that are then separated by 3.0 mm. What is the maximum possible magnitude of the electrostatic force between those two parts? 55 Of the charge Q on a tiny sphere, a fraction a is to be transferred to a second, nearby sphere. The spheres can be treated as particles. (a) What value of a maximizes the magnitude F of the electrostatic force between the two spheres? What are the (b) smaller and (c) larger values of a that put F at half the maximum magnitude?

49 A neutron consists of one “up” quark of charge #2e/3 and two “down” quarks each having charge %e/3. If we assume that the down quarks are 2.6 $ 10 %15 m apart inside the neutron, what is the magnitude of the electrostatic force between them?

56 If a cat repeatedly rubs against your cotton slacks on a dry day, the charge transfer between the cat hair and the cotton can leave you with an excess charge of %2.00 mC. (a) How many electrons are transferred between you and the cat? You will gradually discharge via the floor, but if instead of waiting, you immediately reach toward a faucet, a painful spark can suddenly appear as your fingers near the faucet. (b) In that spark, do electrons flow from you to the faucet or vice versa? (c) Just before the spark appears, do you induce positive or negative charge in the faucet? (d) If, instead, the cat reaches a paw toward the faucet, which way do electrons flow in the resulting spark? (e) If you stroke a cat with a bare hand on a dry day, you should take care not to bring your fingers near the cat’s nose or you will hurt it with a spark. Considering that cat hair is an insulator, explain how the spark can appear.

50 Figure 21-42 shows a long, nonconducting, massless rod of length L, pivoted at its center and balanced with a block of weight W at a distance x from the left end. At the left and right ends of the rod are attached small conducting spheres with positive charges q and 2q, respectively. A distance h directly beneath each of these spheres is a fixed sphere with positive charge Q. (a) Find the distance x when the rod is horizontal and balanced. (b)

57 We know that the negative charge on the electron and the positive charge on the proton are equal. Suppose, however, that these magnitudes differ from each other by 0.00010%. With what force would two copper coins, placed 1.0 m apart, repel each other? Assume that each coin contains 3 $ 10 22 copper atoms. (Hint: A neutral copper atom contains 29 protons and 29 electrons.) What do you conclude?

PROB LE M S

629

58 In Fig. 21-26, particle 1 of charge %80.0 mC and particle 2 of charge #40.0 mC are held at separation L ! 20.0 cm on an x axis. In unit-vector notation, what is the net electrostatic force on particle 3, of charge q3 ! 20.0 mC, if particle 3 is placed at (a) x ! 40.0 cm and (b) x ! 80.0 cm? What should be the (c) x and (d) y coordinates of particle 3 if the net electrostatic force on it due to particles 1 and 2 is zero?

65 The initial charges on the three identical metal spheres in Fig. 21-24 are the following: sphere A, Q; sphere B, %Q/4; and sphere C, Q/2, where Q ! 2.00 $ 10 %14 C. Spheres A and B are fixed in place, with a center-to-center separation of d ! 1.20 m, which is much larger than the spheres. Sphere C is touched first to sphere A and then to sphere B and is then removed. What then is the magnitude of the electrostatic force between spheres A and B?

59 What is the total charge in coulombs of 75.0 kg of electrons?

66 An electron is in a vacuum near Earth’s surface and located at y ! 0 on a vertical y axis. At what value of y should a second electron be placed such that its electrostatic force on the first electron balances the gravitational force on the first electron?

60 In Fig. 21-43, six charged particles surround particle 7 at radial distances of either d ! 1.0 cm or 2d, as drawn. The charges are q1 ! #2e, q2 ! #4e, q3 ! #e, q4 ! #4e, q5 ! #2e, q6 ! #8e, q7 ! #6e, with e ! 1.60 $ 10 %19 C. What is the magnitude of the net electrostatic force on particle 7? y 2

1

7

3

68 Two engineering students, John with a mass of 90 kg and Mary with a mass of 45 kg, are 30 m apart. Suppose each has a 0.01% imbalance in the amount of positive and negative charge, one student being positive and the other negative. Find the order of magnitude of the electrostatic force of attraction between them by replacing each student with a sphere of water having the same mass as the student.

x

4

5 6

Figure 21-43 Problem 60. 61 Three charged particles form a triangle: particle 1 with charge Q1 ! 80.0 nC is at xy coordinates (0, 3.00 mm), particle 2 with charge Q2 is at (0, %3.00 mm), and particle 3 with charge q ! 18.0 nC is at (4.00 mm, 0). In unit-vector notation, what is the electrostatic force on particle 3 due to the other two particles if Q2 is equal to (a) 80.0 nC and (b) %80.0 nC? 62 SSM In Fig. 21-44, what are the (a) magnitude and (b) direction of the net electrostatic force on particle 4 due to the other three particles? All four particles are fixed in the xy plane, and q1 ! %3.20 $ 10 %19 C, q2 ! #3.20 $ 10 %19 C, q3 ! #6.40 $ 10 %19 C, q4 ! #3.20 $ 10 %19 C, u1 ! 35.0", d1 ! 3.00 cm, and d2 ! d3 ! 2.00 cm. y 2 d2 4

θ1

d3

3

67 SSM In Fig. 21-26, particle 1 of charge %5.00q and particle 2 of charge #2.00q are held at separation L on an x axis. If particle 3 of unknown charge q3 is to be located such that the net electrostatic force on it from particles 1 and 2 is zero, what must be the (a) x and (b) y coordinates of particle 3?

x

d1 1

Figure 21-44 Problem 62. 63 Two point charges of 30 nC and %40 nC are held fixed on an x axis, at the origin and at x ! 72 cm, respectively. A particle with a charge of 42 mC is released from rest at x ! 28 cm. If the initial acceleration of the particle has a magnitude of 100 km/s2, what is the particle’s mass? 64 Two small, positively charged spheres have a combined charge of 5.0 $ 10 %5 C. If each sphere is repelled from the other by an electrostatic force of 1.0 N when the spheres are 2.0 m apart, what is the charge on the sphere with the smaller charge?

69 In the radioactive decay of Eq. 21-13, a 238U nucleus transforms to 234Th and an ejected 4He. (These are nuclei, not atoms, and thus electrons are not involved.) When the separation between 234Th and 4 He is 9.0 $ 10 %15 m, what are the magnitudes of (a) the electrostatic force between them and (b) the acceleration of the 4He particle? 70 In Fig. 21-25, four particles form a square. The charges are q1 ! #Q, q2 ! q3 ! q, and q4 ! %2.00Q. What is q/Q if the net electrostatic force on particle 1 is zero? 71 In a spherical metal shell of radius R, an electron is shot from the center directly toward a tiny hole in the shell, through which it escapes. The shell is negatively charged with a surface charge density (charge per unit area) of 6.90 $ 10%13 C/m2. What is the magnitude of the electron’s acceleration when it reaches radial distances (a) r ! 0.500R and (b) 2.00R? 72 An electron is projected with an initial speed vi ! 3.2 $ 105 m/s directly toward a very distant proton that is at rest. Because the proton mass is large relative to the electron mass, assume that the proton remains at rest. By calculating the work done on the electron by the electrostatic force, determine the distance between the two particles when the electron instantaneously has speed 2vi. 73 In an early model of the hydrogen atom (the Bohr model), the electron orbits the proton in uniformly circular motion. The radius of the circle is restricted (quantized) to certain values given by r ! n2a0,

for n ! 1, 2, 3, . . . ,

where a0 ! 52.92 pm. What is the speed of the electron if it orbits in (a) the smallest allowed orbit and (b) the second smallest orbit? (c) If the electron moves to larger orbits, does its speed increase, decrease, or stay the same? 74 A 100 W lamp has a steady current of 0.83 A in its filament. How long is required for 1 mol of electrons to pass through the lamp? 75 The charges of an electron and a positron are %e and #e. The mass of each is 9.11 $ 10%31 kg. What is the ratio of the electrical force to the gravitational force between an electron and a positron?

C

H

A

P

T

E

R

2

2

Electric Fields 22-1 THE ELECTRIC FIELD Learning Objectives After reading this module, you should be able to . . .

22.01 Identify that at every point in the space surrounding : a charged particle, the particle sets up an electric field E, which is a vector quantity and thus has both magnitude and direction. : 22.02 Identify how an electric field E can be used to explain : how a charged particle can exert an electrostatic force F

on a second charged particle even though there is no contact between the particles. 22.03 Explain how a small positive test charge is used (in principle) to measure the electric field at any given point. 22.04 Explain electric field lines, including where they originate and terminate and what their spacing represents.

Key Ideas ● A charged particle sets up an electric field (a vector quantity)

in the surrounding space. If a second charged particle is located in that space, an electrostatic force acts on it due to the magnitude and direction of the field at its location. :

● The electric field E at any point is defined in terms of the elec:

trostatic force F that would be exerted on a positive test charge q0 placed there: : F : E! . q0

● Electric field lines help us visualize the direction and

magnitude of electric fields. The electric field vector at any point is tangent to the field line through that point. The density of field lines in that region is proportional to the magnitude of the electric field there. Thus, closer field lines represent a stronger field. ● Electric field lines originate on positive charges and

terminate on negative charges. So, a field line extending from a positive charge must end on a negative charge.

What Is Physics?

+

+

q1

q2

Figure 22-1 How does charged particle 2 push on charged particle 1 when they have no contact? 630

Figure 22-1 shows two positively charged particles. From the preceding chapter we know that an electrostatic force acts on particle 1 due to the presence of particle 2. We also know the force direction and, given some data, we can calculate the force magnitude. However, here is a leftover nagging question. How does particle 1 “know” of the presence of particle 2? That is, since the particles do not touch, how can particle 2 push on particle 1—how can there be such an action at a distance? One purpose of physics is to record observations about our world, such as the magnitude and direction of the push on particle 1. Another purpose is to provide an explanation of what is recorded. Our purpose in this chapter is to provide such an explanation to this nagging question about electric force at a distance. The explanation that we shall examine here is this: Particle 2 sets up an electric field at all points in the surrounding space, even if the space is a vacuum. If we place particle 1 at any point in that space, particle 1 knows of the presence of particle 2 because it is affected by the electric field particle 2 has already set up at that point. Thus, particle 2 pushes on particle 1 not by touching it as you would push on a coffee mug by making contact. Instead, particle 2 pushes by means of the electric field it has set up.

22-1 TH E E LECTR IC FI E LD

Our goals in this chapter are to (1) define electric field, (2) discuss how to calculate it for various arrangements of charged particles and objects, and (3) discuss how an electric field can affect a charged particle (as in making it move).

F

+

+ + + Test charge q0 + + at point P + + + + + + Charged

The Electric Field A lot of different fields are used in science and engineering. For example, a temperature field for an auditorium is the distribution of temperatures we would find by measuring the temperature at many points within the auditorium. Similarly, we could define a pressure field in a swimming pool. Such fields are examples of scalar fields because temperature and pressure are scalar quantities, having only magnitudes and not directions. In contrast, an electric field is a vector field because it is responsible for conveying the information for a force, which involves both magnitude and direction. : This field consists of a distribution of electric field vectors E, one for each point : in the space around a charged object. In principle, we can define E at some point near the charged object, such as point P in Fig. 22-2a, with this procedure: At P, we place a particle with a small positive charge q0, called a test charge because we use it to test the field. (We want the charge to be small so that it does not disturb : the object’s charge distribution.) We then measure the electrostatic force F that acts on the test charge. The electric field at that point is then :

F E! q0 :

(electric field).

(22-1)

631

object

(a)

The rod sets up an electric field, which can create a force on the test charge.

E P

+ + + + + + + + + ++

Electric field at point P

(b)

Figure 22-2 (a) A positive test charge q0 placed at point P near a charged object. An : electrostatic force F acts on the test charge. : (b) The electric field E at point P produced by the charged object.

Because the test charge is positive, the two vectors in Eq. 22-1 are in the same : : direction, so the direction of E is the direction we measure for F .The magnitude of : E at point P is F/q0. As shown in Fig. 22-2b, we always represent an electric field with an arrow with its tail anchored on the point where the measurement is made. (This may sound trivial, but drawing the vectors any other way usually results in errors. Also, another common error is to mix up the terms force and field because they both start with the letter f. Electric force is a push or pull. Electric field is an abstract property set up by a charged object.) From Eq. 22-1, we see that the SI unit for the electric field is the newton per coulomb (N/C). We can shift the test charge around to various other points, to measure the electric fields there, so that we can figure out the distribution of the electric field set up by the charged object. That field exists independent of the test charge. It is something that a charged object sets up in the surrounding space (even vacuum), independent of whether we happen to come along to measure it. For the next several modules, we determine the field around charged particles and various charged objects. First, however, let’s examine a way of visualizing electric fields.

– – –



– – –



F

+

Positive test charge

(a)

– – –



– – –



E

Electric Field Lines Look at the space in the room around you. Can you visualize a field of vectors throughout that space—vectors with different magnitudes and directions? As impossible as that seems, Michael Faraday, who introduced the idea of electric fields in the 19th century, found a way. He envisioned lines, now called electric field lines, in the space around any given charged particle or object. Figure 22-3 gives an example in which a sphere is uniformly covered with negative charge. If we place a positive test charge at any point near the sphere (Fig. 22-3a), we find that an electrostatic force pulls on it toward the center of the sphere. Thus at every point around the sphere, an electric field vector points radially inward toward the sphere. We can represent this electric field with

Electric field lines (b) :

Figure 22-3 (a) The electrostatic force F acting on a positive test charge near a sphere of uniform negative charge. : (b) The electric field vector E at the location of the test charge, and the electric field lines in the space near the sphere. The field lines extend toward the negatively charged sphere. (They originate on distant positive charges.)

632

CHAPTE R 22 E LECTR IC FI E LDS

+ + + +

+ + + +

+ + + +

+ + +

Positive test charge

+ F

+

+ + + +

(a)

+ + + +

+ + + +

+ + + +

(b)

E

+ + + + + + + + + (c)

Figure 22-4 (a) The force on a positive test charge near a very large, nonconducting sheet : with uniform positive charge on one side. (b) The electric field vector E at the test charge’s location, and the nearby electric field lines, extending away from the sheet. (c) Side view.

electric field lines as in Fig. 22-3b. At any point, such as the one shown, the direction of the field line through the point matches the direction of the electric vector at that point. The rules for drawing electric fields lines are these: (1) At any point, the electric field vector must be tangent to the electric field line through that point and in the same direction. (This is easy to see in Fig. 22-3 where the lines are straight, but we’ll see some curved lines soon.) (2) In a plane perpendicular to the field lines, the relative density of the lines represents the relative magnitude of the field there, with greater density for greater magnitude. If the sphere in Fig. 22-3 were uniformly covered with positive charge, the electric field vectors at all points around it would be radially outward and thus so would the electric field lines. So, we have the following rule: Electric field lines extend away from positive charge (where they originate) and toward negative charge (where they terminate).

+ E

+

Figure 22-5 Field lines for two particles with equal positive charge. Doesn’t the pattern itself suggest that the particles repel each other?

In Fig. 22-3b, they originate on distant positive charges that are not shown. For another example, Fig. 22-4a shows part of an infinitely large, nonconducting sheet (or plane) with a uniform distribution of positive charge on one side. If we place a positive test charge at any point near the sheet (on either side), we find that the electrostatic force on the particle is outward and perpendicular to the sheet. The perpendicular orientation is reasonable because any force component that is, say, upward is balanced out by an equal component that is downward. That leaves only outward, and thus the electric field vectors and the electric field lines must also be outward and perpendicular to the sheet, as shown in Figs. 22-4b and c. Because the charge on the sheet is uniform, the field vectors and the field lines are also. Such a field is a uniform electric field, meaning that the electric field has the same magnitude and direction at every point within the field. (This is a lot easier to work with than a nonuniform field, where there is variation from point to point.) Of course, there is no such thing as an infinitely large sheet. That is just a way of saying that we are measuring the field at points close to the sheet relative to the size of the sheet and that we are not near an edge. Figure 22-5 shows the field lines for two particles with equal positive charges. Now the field lines are curved, but the rules still hold: (1) the electric field vector at any given point must be tangent to the field line at that point and in the same direction, as shown for one vector, and (2) a closer spacing means a larger field magnitude. To imagine the full three-dimensional pattern of field lines around the particles, mentally rotate the pattern in Fig. 22-5 around the axis of symmetry, which is a vertical line through both particles.

22-2 TH E E LECTR IC FI E LD DU E TO A CHARG E D PARTICLE

633

22-2 THE ELECTRIC FIELD DUE TO A CHARGED PARTICLE Learning Objectives After reading this module, you should be able to . . .

magnitude E, the charge magnitude !q!, and the distance r between the point and the particle. 22.08 Identify that the equation given here for the magnitude of an electric field applies only to a particle, not an extended object. 22.09 If more than one electric field is set up at a point, draw each electric field vector and then find the net electric field by adding the individual electric fields as vectors (not as scalars).

22.05 In a sketch, draw a charged particle, indicate its sign, pick a nearby point, and then draw the electric field vector : E at that point, with its tail anchored on the point. 22.06 For a given point in the electric field of a charged particle, : identify the direction of the field vector E when the particle is positively charged and when it is negatively charged. 22.07 For a given point in the electric field of a charged particle, apply the relationship between the field

Key Ideas

:

● The magnitude of the electric field E

set up by a particle with charge q at distance r from the particle is !q! 1 . E! 4p´0 r 2

by a negatively charged particle all point directly toward the particle. ● If more than one charged particle sets up an electric field at a point, the net electric field is the vector sum of the individual electric fields—electric fields obey the superposition principle.

● The electric field vectors set up by a positively charged particle all point directly away from the particle. Those set up

The Electric Field Due to a Point Charge To find the electric field due to a charged particle (often called a point charge), we place a positive test charge at any point near the particle, at distance r. From Coulomb’s law (Eq.21-4),the force on the test charge due to the particle with charge q is :

F !

1 qq0 rˆ . 4p´0 r2

:

As previously, the direction of F is directly away from the particle if q is positive (because q0 is positive) and directly toward it if q is negative. From Eq. 22-1, we can now write the electric field set up by the particle (at the location of the test charge) as :

q F 1 E! ! rˆ q0 4p´0 r 2 :

(charged particle).

(22-2)

:

Let’s think through the directions again. The direction of E matches that of the force on the positive test charge: directly away from the point charge if q is positive and directly toward it if q is negative. So, if given another charged particle, we can immediately determine the directions of the electric field vectors near it by just looking at the sign of the charge q. We can find the magnitude at any given distance r by converting Eq. 22-2 to a magnitude form: E!

!q! 1 4p´0 r2

(charged particle).

+

(22-3)

We write !q! to avoid the danger of getting a negative E when q is negative, and then thinking the negative sign has something to do with direction. Equation 22-3 gives magnitude E only. We must think about the direction separately. Figure 22-6 gives a number of electric field vectors at points around a positively charged particle, but be careful. Each vector represents the vector

Figure 22-6 The electric field vectors at various points around a positive point charge.

634

CHAPTE R 22 E LECTR IC FI E LDS

quantity at the point where the tail of the arrow is anchored. The vector is not something that stretches from a “here” to a “there” as with a displacement vector. In general, if several electric fields are set up at a given point by several charged particles, we can find the net field by placing a positive test particle at the point and then writing out the force acting on it due to each particle, such : as F01 due to particle 1. Forces obey the principle of superposition, so we just add the forces as vectors: :

:

:

:

F0 ! F01 # F02 # ( ( ( # F0n. To change over to electric field, we repeatedly use Eq. 22-1 for each of the individual forces: :

:

E!

:

:

:

F0 F01 F02 F0n ! # # ((( # q0 q0 q0 q0 :

:

:

(22-4)

! E1 # E2 # ( ( ( # En.

This tells us that electric fields also obey the principle of superposition. If you want the net electric field at a given point due to several particles, find the electric field : due to each particle (such as E1 due to particle 1) and then sum the fields as vectors. (As with electrostatic forces, you cannot just willy-nilly add up the magnitudes.) This addition of fields is the subject of many of the homework problems.

Checkpoint 1 The figure here shows a proton p and an electron e on an x axis.What is the direction of the electric field due to the electron at (a) point x S and (b) point R? What is the direction of the S p e R net electric field at (c) point R and (d) point S?

Sample Problem 22.01 Net electric field due to three charged particles y

Figure 22-7a shows three particles with charges q1 ! #2Q, q2 ! %2Q, and q3 ! %4Q, each a distance d from the origin. : What net electric field E is produced at the origin?

q3

q1 d

d

30°

KEY IDEA

Find the net field at this empty point.

: : E1, E2,

Charges q1, q2, and q3 produce electric field vectors : and E3, respectively, at the origin, and the net electric field : : : : is the vector sum E ! E1 # E2 # E3. To find this sum, we first must find the magnitudes and orientations of the three field vectors.

q2

y

y E3

:

Field away

1 2Q . 4p´0 d2 :

(b)

and

30° 30° E2

E1

E3

Field toward

30° 30°

x

Field toward

x E1 + E2

(c)

:

Similarly, we find the magnitudes of E2 and E3 to be 1 2Q E2 ! 4p´0 d2

x

(a)

Magnitudes and directions: To find the magnitude of E1, which is due to q1, we use Eq. 22-3, substituting d for r and 2Q for q and obtaining E1 !

30° 30° d

1 4Q E3 ! . 4p´0 d2

Figure 22-7 (a) Three particles with charges q1, q2, and q3 are at the : same distance d from the origin. (b) The electric field vectors E1, : : E2, and E3, at the origin due to the three particles. (c) The electric : : : field vector E3 and the vector sum E1 # E2 at the origin.

22-3 TH E E LECTR IC FI E LD DU E TO A DI POLE

We next must find the orientations of the three electric field vectors at the origin. Because q1 is a positive charge, the field vector it produces points directly away from it, and because q2 and q3 are both negative, the field vectors they produce point directly toward each of them. Thus, the three electric fields produced at the origin by the three charged particles are oriented as in Fig. 22-7b. (Caution: Note that we have placed the tails of the vectors at the point where the fields are to be evaluated; doing so decreases the chance of error. Error becomes very probable if the tails of the field vectors are placed on the particles creating the fields.) Adding the fields: We can now add the fields vectorially just as we added force vectors in Chapter 21. However, here we can use symmetry to simplify the procedure. From Fig. 22-7b, : : we see that electric fields E1 and E2 have the same direction. Hence, their vector sum has that direction and has the magnitude

635

1 2Q 1 2Q # 2 4p´0 d 4p´0 d2 1 4Q ! , 4p´0 d2

E1 # E2 !

:

which happens to equal the magnitude of field E3. : We must now combine two vectors, E3 and the vector : : sum E1 # E2, that have the same magnitude and that are oriented symmetrically about the x axis, as shown in Fig. 22-7c. From the symmetry of Fig. 22-7c, we realize that the equal y components of our two vectors cancel (one is upward and the other is downward) and the equal x components add : (both are rightward). Thus, the net electric field E at the origin is in the positive direction of the x axis and has the magnitude E ! 2E3x ! 2E3 cos 30" ! (2)

1 4Q 6.93Q (0.866) ! . 2 4p´0 d 4p´0d2

(Answer)

Additional examples, video, and practice available at WileyPLUS

22-3 THE ELECTRIC FIELD DUE TO A DIPOLE Learning Objectives After reading this module, you should be able to . . .

22.10 Draw an electric dipole, identifying the charges (sizes and signs), dipole axis, and direction of the electric dipole moment. 22.11 Identify the direction of the electric field at any given point along the dipole axis, including between the charges. 22.12 Outline how the equation for the electric field due to an electric dipole is derived from the equations for the electric field due to the individual charged particles that form the dipole. 22.13 For a single charged particle and an electric dipole, compare the rate at which the electric field magnitude

decreases with increase in distance. That is, identify which drops off faster. 22.14 For an electric dipole, apply the relationship between the magnitude p of the dipole moment, the separation d between the charges, and the magnitude q of either of the charges. 22.15 For any distant point along a dipole axis, apply the relationship between the electric field magnitude E, the distance z from the center of the dipole, and either the dipole moment magnitude p or the product of charge magnitude q and charge separation d.

Key Ideas ● An electric dipole consists of two particles with charges of equal magnitude q but opposite signs, separated by a small distance d. ● The electric dipole moment

:

p has magnitude qd and points from the negative charge to the positive charge. ● The magnitude of the electric field set up by an electric dipole at a distant point on the dipole axis (which runs through both particles) can be written in terms of either the product qd or the magnitude p of the dipole moment:

E!

p 1 qd 1 , ! 3 2p´0 z 2p´0 z3

where z is the distance between the point and the center of the dipole. of the 1/z3 dependence, the field magnitude of an electric dipole decreases more rapidly with distance than the field magnitude of either of the individual charges forming the dipole, which depends on 1/r2.

● Because

636

CHAPTE R 22 E LECTR IC FI E LDS

The Electric Field Due to an Electric Dipole +

E



Figure 22-8 The pattern of electric field lines around an electric dipole, with an electric : field vector E shown at one point (tangent to the field line through that point).

Figure 22-8 shows the pattern of electric field lines for two particles that have the same charge magnitude q but opposite signs, a very common and important arrangement known as an electric dipole. The particles are separated by distance d and lie along the dipole axis, an axis of symmetry around which you can imagine rotating the pattern in Fig. 22-8. Let’s label that axis as a z axis. Here : we restrict our interest to the magnitude and direction of the electric field E at an arbitrary point P along the dipole axis, at distance z from the dipole’s midpoint. Figure 22-9a shows the electric fields set up at P by each particle. The nearer particle with charge #q sets up field E(#) in the positive direction of the z axis (directly away from the particle). The farther particle with charge %q sets up a smaller field E(%) in the negative direction (directly toward the particle). We want the net field at P, as given by Eq. 22-4. However, because the field vectors are along the same axis, let’s simply indicate the vector directions with plus and minus signs, as we commonly do with forces along a single axis. Then we can write the magnitude of the net field at P as E ! E(#) % E(%) 1 q 1 q % 2 2 4p´0 r(#) 4p´0 r(%) q q % . ! 1 2 4p´0(z % 2d) 4p´0(z # 12d)2 !

(22-5)

After a little algebra, we can rewrite this equation as z

E! E(+)

q 4p´ 0z2

##

1

d 1% 2z

$

2

%

1

#

d 1# 2z

$$

(22-6)

.

2

After forming a common denominator and multiplying its terms, we come to P

E!

E(–)

r(+) z r(–)

+

Up here the +q field dominates.

+

+q

q 4p´0z2

2d/z d 1% 2z

# # $$



–q

Dipole center

– Down here the –q field dominates.

(a)

(b)

Figure 22-9 (a) An electric dipole. The elec: : tric field vectors E(#) and E(%) at point P on the dipole axis result from the dipole’s two charges. Point P is at distances r(#) and r(%) from the individual charges that make up the dipole. (b) The dipole moment p: of the dipole points from the negative charge to the positive charge.

!

q 2p´0z3

d

# # $$ 1%

d 2z

2 2

.

(22-7)

We are usually interested in the electrical effect of a dipole only at distances that are large compared with the dimensions of the dipole — that is, at distances such that z ) d.At such large distances, we have d/2z * 1 in Eq. 22-7.Thus, in our approximation, we can neglect the d/2z term in the denominator, which leaves us with E!

p

d

2 2

1 qd . 2p´0 z3

(22-8)

The product qd, which involves the two intrinsic properties q and d of the dipole, is the magnitude p of a vector quantity known as the electric dipole moment : of the dipole. (The unit of p: is the coulomb-meter.) Thus, we can write Eq. 22-8 as p E!

1 p 2p´0 z3

(electric dipole).

(22-9)

The direction of p: is taken to be from the negative to the positive end of the dipole, as indicated in Fig. 22-9b. We can use the direction of p: to specify the orientation of a dipole. Equation 22-9 shows that, if we measure the electric field of a dipole only at distant points, we can never find q and d separately; instead, we can find only their product. The field at distant points would be unchanged if, for example, q

637

22-3 TH E E LECTR IC FI E LD DU E TO A DI POLE

were doubled and d simultaneously halved. Although Eq. 22-9 holds only for distant points along the dipole axis, it turns out that E for a dipole varies as 1/r 3 for all distant points, regardless of whether they lie on the dipole axis; here r is the distance between the point in question and the dipole center. Inspection of Fig. 22-9 and of the field lines in Fig. 22-8 shows that the direc: tion of E for distant points on the dipole axis is always the direction of the dipole moment vector p: . This is true whether point P in Fig. 22-9a is on the upper or the lower part of the dipole axis. Inspection of Eq. 22-9 shows that if you double the distance of a point from a dipole, the electric field at the point drops by a factor of 8. If you double the distance from a single point charge, however (see Eq. 22-3), the electric field drops only by a factor of 4. Thus the electric field of a dipole decreases more rapidly with distance than does the electric field of a single charge. The physical reason for this rapid decrease in electric field for a dipole is that from distant points a dipole looks like two particles that almost — but not quite — coincide. Thus, because they have charges of equal magnitude but opposite signs, their electric fields at distant points almost — but not quite — cancel each other.

Sample Problem 22.02 Electric dipole and atmospheric sprites Sprites (Fig. 22-10a) are huge flashes that occur far above a large thunderstorm. They were seen for decades by pilots flying at night, but they were so brief and dim that most pilots figured they were just illusions. Then in the 1990s sprites were captured on video. They are still not well understood but are believed to be produced when especially powerful lightning occurs between the ground and storm clouds, particularly when the lightning transfers a huge amount of negative charge %q from the ground to the base of the clouds (Fig. 22-10b). Just after such a transfer, the ground has a complicated distribution of positive charge. However, we can model the electric field due to the charges in the clouds and the ground by assuming a vertical electric dipole that has charge %q at cloud height h and charge #q at below-ground depth h (Fig. 22-10c). If q ! 200 C and h ! 6.0 km, what is the magnitude of the dipole’s electric field at altitude z1 ! 30 km somewhat above the clouds and altitude z2 ! 60 km somewhat above the stratosphere?

KEY IDEA We can approximate the magnitude E of an electric dipole’s electric field on the dipole axis with Eq.22-8. Calculations: We write that equation as E!

1 q(2h) , 2p´0 z3

where 2h is the separation between %q and #q in Fig. 22-10c. For the electric field at altitude z1 ! 30 km, we find E!

1 (200 C)(2)(6.0 $ 103 m) 2p´0 (30 $ 103 m)3

! 1.6 $ 103 N/C.

(Answer)

Similarly, for altitude z2 ! 60 km, we find E ! 2.0 $ 10 2 N/C. (Answer) As we discuss in Module 22-6, when the magnitude of

z –q Cloud

h

Charge transfer (a)

Courtesy NASA

(b)

– – – – – – Ground

h (c)

+q

Figure 22-10 (a) Photograph of a sprite. (b) Lightning in which a large amount of negative charge is transferred from ground to cloud base. (c) The cloud – ground system modeled as a vertical electric dipole.

638

CHAPTE R 22 E LECTR IC FI E LDS

an electric field exceeds a certain critical value Ec, the field can pull electrons out of atoms (ionize the atoms), and then the freed electrons can run into other atoms, causing those atoms to emit light. The value of Ec depends on the density of the air in which the electric field exists. At altitude z2 ! 60 km the density of the air is so low that

E ! 2.0 $ 102 N/C exceeds Ec, and thus light is emitted by the atoms in the air. That light forms sprites. Lower down, just above the clouds at z1 ! 30 km, the density of the air is much higher, E ! 1.6 $ 103 N/C does not exceed Ec, and no light is emitted. Hence, sprites occur only far above storm clouds.

Additional examples, video, and practice available at WileyPLUS

22-4 THE ELECTRIC FIELD DUE TO A LINE OF CHARGE Learning Objectives After reading this module, you should be able to . . .

22.16 For a uniform distribution of charge, find the linear charge density l for charge along a line, the surface charge density s for charge on a surface, and the volume charge density r for charge in a volume. 22.17 For charge that is distributed uniformly along a line, find the net electric field at a given point near the line by

splitting the distribution up into charge elements dq and : then summing (by integration) the electric field vectors dE set up at the point by each element. 22.18 Explain how symmetry can be used to simplify the calculation of the electric field at a point near a line of uniformly distributed charge.

Key Ideas ● The equation for the electric field set up by a particle does not apply to an extended object with charge (said to have a continuous charge distribution). ● To find the electric field of an extended object at a point, we first consider the electric field set up by a charge element dq in the object, where the element is small enough for us to apply

the equation for a particle. Then we sum, via integration, com: ponents of the electric fields dE from all the charge elements. :

● Because the individual electric fields dE

have different magnitudes and point in different directions, we first see if symmetry allows us to cancel out any of the components of the fields, to simplify the integration.

The Electric Field Due to a Line of Charge So far we have dealt with only charged particles, a single particle or a simple collection of them. We now turn to a much more challenging situation in which a thin (approximately one-dimensional) object such as a rod or ring is charged with a huge number of particles, more than we could ever even count. In the next module, we consider two-dimensional objects, such as a disk with charge spread over a surface. In the next chapter we tackle three-dimensional objects, such as a sphere with charge spread through a volume. Heads Up. Many students consider this module to be the most difficult in the book for a variety of reasons. There are lots of steps to take, a lot of vector features to keep track of, and after all that, we set up and then solve an integral. The worst part, however, is that the procedure can be different for different arrangements of the charge. Here, as we focus on a particular arrangement (a charged ring), be aware of the general approach, so that you can tackle other arrangements in the homework (such as rods and partial circles). Figure 22-11 shows a thin ring of radius R with a uniform distribution of positive charge along its circumference. It is made of plastic, which means that the charge is fixed in place. The ring is surrounded by a pattern of electric field lines, but here we restrict our interest to an arbitrary point P on the central axis (the axis through the ring’s center and perpendicular to the plane of the ring), at distance z from the center point. The charge of an extended object is often conveyed in terms of a charge density rather than the total charge. For a line of charge, we use the linear charge

639

22-4 TH E E LECTR IC FI E LD DU E TO A LI N E OF CHARG E

density l (the charge per unit length), with the SI unit of coulomb per meter. Table 22-1 shows the other charge densities that we shall be using for charged surfaces and volumes. First Big Problem. So far, we have an equation for the electric field of a particle. (We can combine the field of several particles as we did for the electric dipole to generate a special equation, but we are still basically using Eq. 22-3). Now take a look at the ring in Fig. 22-11. That clearly is not a particle and so Eq. 22-3 does not apply. So what do we do? The answer is to mentally divide the ring into differential elements of charge that are so small that we can treat them as though they are particles. Then we can apply Eq. 22-3. Second Big Problem. We now know to apply Eq. 22-3 to each charge element dq (the front d emphasizes that the charge is very small) and can write an : expression for its contribution of electric field dE (the front d emphasizes that the contribution is very small). However, each such contributed field vector at P is in its own direction. How can we add them to get the net field at P? The answer is to split the vectors into components and then separately sum one set of components and then the other set. However, first we check to see if one set simply all cancels out. (Canceling out components saves lots of work.) Third Big Problem. There is a huge number of dq elements in the ring and : thus a huge number of dE components to add up, even if we can cancel out one set of components. How can we add up more components than we could even count? The answer is to add them by means of integration. Do It. Let’s do all this (but again, be aware of the general procedure, not just the fine details). We arbitrarily pick the charge element shown in Fig. 22-11. Let ds be the arc length of that (or any other) dq element. Then in terms of the linear density l (the charge per unit length), we have dq ! l ds.

Table 22-1 Some Measures of Electric Charge Name

Symbol

Charge Linear charge density Surface charge density Volume charge density

SI Unit

q

C

l

C/m

s

C/m2

r

C/m3

(22-10)

An Element’s Field. This charge element sets up the differential electric : field dE at P, at distance r from the element, as shown in Fig. 22-11. (Yes, we are introducing a new symbol that is not given in the problem statement, but soon we shall replace it with “legal symbols.”) Next we rewrite the field equation for a particle (Eq. 22-3) in terms of our new symbols dE and dq, but then we replace dq using Eq. 22-10. The field magnitude due to the charge element is

z

dE

θ

dq l ds 1 1 ! . dE ! 2 4p´0 r 4p´0 r 2

(22-11)

P

Notice that the illegal symbol r is the hypotenuse of the right triangle displayed in Fig. 22-11. Thus, we can replace r by rewriting Eq. 22-11 as l ds 1 dE ! . 4p´0 (z2 # R2)

θ

(22-12)

Because every charge element has the same charge and the same distance from point P, Eq. 22-12 gives the field magnitude contributed by each of them. : Figure 22-11 also tells us that each contributed dE leans at angle u to the central axis (the z axis) and thus has components perpendicular and parallel to that axis. Canceling Components. Now comes the neat part, where we eliminate one set of those components. In Fig. 22-11, consider the charge element on the opposite side of the ring. It too contributes the field magnitude dE but the field vector leans at angle u in the opposite direction from the vector from our first charge

z

r

+ + + + + +

ds

+

+

+ + + + +

+

+

+

R +

+ + + + +

+

+

+

+ + + + +

Figure 22-11 A ring of uniform positive charge. A differential element of charge occupies a length ds (greatly exaggerated for clarity).This element sets up an electric field : dE at point P.

640

CHAPTE R 22 E LECTR IC FI E LDS

z dE cos u dE

dE

u

u

Figure 22-12 The electric fields set up at P by a charge element and its symmetric partner (on the opposite side of the ring). The components perpendicular to the z axis cancel; the parallel components add.

element, as indicated in the side view of Fig. 22-12. Thus the two perpendicular components cancel. All around the ring, this cancelation occurs for every charge element and its symmetric partner on the opposite side of the ring. So we can neglect all the perpendicular components. Adding Components. We have another big win here. All the remaining components are in the positive direction of the z axis, so we can just add them up as scalars. Thus we can already tell the direction of the net electric field at P: directly away from the ring. From Fig. 22-12, we see that the parallel components each have magnitude dE cos u, but u is another illegal symbol. We can replace cos u with legal symbols by again using the right triangle in Fig. 22-11 to write z z (22-13) cos u ! ! 2 . r (z # R2)1/2 Multiplying Eq. 22-12 by Eq. 22-13 gives us the parallel field component from each charge element: dE cos u !

zl 1 ds. 2 4p+0 (z # R2)3/2

(22-14)

Integrating. Because we must sum a huge number of these components, each small, we set up an integral that moves along the ring, from element to element, from a starting point (call it s ! 0) through the full circumference (s ! 2pR). Only the quantity s varies as we go through the elements; the other symbols in Eq. 22-14 remain the same, so we move them outside the integral.We find E! !

"

dE cos u !

zl 4p´0(z # R2)3/2 2

"

2pR

0

ds

zl(2pR) . 4p´0(z2 # R2)3/2

(22-15)

This is a fine answer, but we can also switch to the total charge by using l ! q/(2pR): E!

qz 4p´0(z2 # R2)3/2

(charged ring).

(22-16)

If the charge on the ring is negative, instead of positive as we have assumed, the magnitude of the field at P is still given by Eq. 22-16. However, the electric field vector then points toward the ring instead of away from it. Let us check Eq. 22-16 for a point on the central axis that is so far away that z ) R. For such a point, the expression z2 # R2 in Eq. 22-16 can be approximated as z2, and Eq. 22-16 becomes E!

q 1 4p´0 z2

(charged ring at large distance).

(22-17)

This is a reasonable result because from a large distance, the ring “looks like” a point charge. If we replace z with r in Eq. 22-17, we indeed do have the magnitude of the electric field due to a point charge, as given by Eq. 22-3. Let us next check Eq. 22-16 for a point at the center of the ring — that is, for z ! 0. At that point, Eq. 22-16 tells us that E ! 0. This is a reasonable result because if we were to place a test charge at the center of the ring, there would be no net electrostatic force acting on it; the force due to any element of the ring would be canceled by the force due to the element on the opposite side of the ring. By Eq. 22-1, if the force at the center of the ring were zero, the electric field there would also have to be zero.

641

22-4 TH E E LECTR IC FI E LD DU E TO A LI N E OF CHARG E

Sample Problem 22.03 Electric field of a charged circular rod :

Figure 22-13a shows a plastic rod with a uniform charge %Q. It is bent in a 120° circular arc of radius r and symmetrically paced across an x axis with the origin at the center of curvature P of the rod. In terms of Q and r, what is the elec: tric field E due to the rod at point P?

rewrite Eq. 22-3 to express the magnitude of dE as dE !

:

Symmetric partner: Our element has a symmetrically located (mirror image) element ds, in the bottom half of the : rod. The electric field dE, set up at P by ds, also has the magnitude given by Eq. 22-19, but the field vector points toward ds, as shown in Fig. 22-13d. If we resolve the electric field vectors of ds and ds, into x and y components as shown in Figs. 22-13e and f, we see that their y components cancel (because they have equal magnitudes and are in opposite directions). We also see that their x components have equal magnitudes and are in the same direction.

Because the rod has a continuous charge distribution, we must find an expression for the electric fields due to differential elements of the rod and then sum those fields via calculus. An element: Consider a differential element having arc length ds and located at an angle u above the x axis (Figs. 22-13b and c). If we let l represent the linear charge density of the rod, our element ds has a differential charge of magnitude (22-18)

Summing: Thus, to find the electric field set up by the rod, we need sum (via integration) only the x components of the differential electric fields set up by all the differential elements of the rod. From Fig. 22-13f and Eq. 22-19, we can write

The element’s field: Our element produces a differential : electric field dE at point P, which is a distance r from the element. Treating the element as a point charge, we can

This negatively charged rod is obviously not a particle. y

But we can treat this element as a particle.

A

Here is the field the element creates.

y

Plastic rod of charge –Q

y

y ds

ds

ds

dE

dE 60° P

r

60°

x

θ

x

P

(22-19)

The direction of dE is toward ds because charge dq is negative.

KEY IDEA

dq ! l ds.

1 dq 1 l ds ! . 4p´0 r2 4p´0 r2

x

P

P

θ θ

x

dE' Symmetric element ds' (b)

(a)

Figure 22-13 Available in WileyPLUS as an animation with voiceover. (a) A plastic rod of charge %Q is a circular section of radius r and central angle 120"; point P is the center of curvature of the rod. (b)–(c) A differential element in the top half of the rod, at an angle u to the x axis and of arc length ds, sets up a differential : electric field dE at P. (d) An element ds,, symmetric to ds about the x axis, sets up a field : dE, at P with the same magnitude. (e)–(f ) The field components. (g) Arc length ds makes an angle du about point P.

These y components just cancel, so neglect them.

These x components add. Our job is to add all such components.

y

Here is the field created by the symmetric element, same size and angle.

y ds

y

ds

dEy dE

dE

θ θ

θ θ

P

(d)

(c)

x

dE'

P

dEx

x

We use this to relate the element’s arc length to the angle that it subtends.

ds dθ

r x

P

dE' Symmetric element ds' (e)

Symmetric element ds' (f )

(g )

642

CHAPTE R 22 E LECTR IC FI E LDS

the component dEx set up by ds as dE x ! dE cos u !

l 1 cos u ds. 4p´0 r2

(22-20)

Equation 22-20 has two variables, u and s. Before we can integrate it, we must eliminate one variable. We do so by replacing ds, using the relation ds ! r du, in which du is the angle at P that includes arc length ds (Fig. 22-13g). With this replacement, we can integrate Eq. 22-20 over the angle made by the rod at P, from u ! %60" to u ! 60"; that will give us the field magnitude at P: E! !

"

l 4p´0r

"

60"

dE x !

"

%60"

60"

(If we had reversed the limits on the integration, we would have gotten the same result but with a minus sign. Since the : integration gives only the magnitude of E , we would then have discarded the minus sign.) Charge density: To evaluate l, we note that the full rod subtends an angle of 120" and so is one-third of a full circle. Its arc length is then 2pr/3, and its linear charge density must be

Substituting this into Eq. 22-21 and simplifying give us

1 l cos u r du 4p´0 r2

cos u du !

%60"

l [sin 60" % sin(%60")] 4p´0r

!

1.73l . 4p´0r

E!

( )

l sin u 4p´0r

!

Q 0.477Q charge ! ! . length 2pr/3 r

l!

60"

!

%60"

(1.73)(0.477Q) 4p´0r2 0.83Q . 4p´0r2

(Answer)

:

(22-21)

The direction of E is toward the rod, along the axis of symmetry : of the charge distribution. We can write E in unit-vector notation as 0.83Q ˆ : E! i. 4p´0r2

Problem-Solving Tactics A Field Guide for Lines of Charge :

Here is a generic guide for finding the electric field E produced at a point P by a line of uniform charge, either circular or straight. The general strategy is to pick out an element : dq of the charge, find dE due to that element, and integrate : dE over the entire line of charge. Step 1. If the line of charge is circular, let ds be the arc length of an element of the distribution. If the line is straight, run an x axis along it and let dx be the length of an element. Mark the element on a sketch. Step 2. Relate the charge dq of the element to the length of the element with either dq ! l ds or dq ! l dx. Consider dq and l to be positive, even if the charge is actually negative. (The sign of the charge is used in the next step.) :

Step 3. Express the field dE produced at P by dq with Eq. 22-3, replacing q in that equation with either l ds or l dx. If the charge on the line is positive, then at P draw a : vector dE that points directly away from dq. If the charge is negative, draw the vector pointing directly toward dq. Step 4. Always look for any symmetry in the situation. If P is on an axis of symmetry of the charge distribution, : resolve the field dE produced by dq into components that are perpendicular and parallel to the axis of symmetry. Then consider a second element dq, that is located symmetrically to dq about the line of symmetry. At P : draw the vector dE, that this symmetrical element pro-

duces and resolve it into components. One of the components produced by dq is a canceling component; it is canceled by the corresponding component produced by dq, and needs no further attention. The other component produced by dq is an adding component; it adds to the corresponding component produced by dq,. Add the adding components of all the elements via integration. Step 5. Here are four general types of uniform charge distributions, with strategies for the integral of step 4. Ring, with point P on (central) axis of symmetry, as in Fig. 22-11. In the expression for dE, replace r 2 with z2 # R2, as in Eq. 22-12. Express the adding component : of dE in terms of u. That introduces cos u, but u is identical for all elements and thus is not a variable. Replace cos u as in Eq. 22-13. Integrate over s, around the circumference of the ring. Circular arc, with point P at the center of curvature, as in Fig. 22-13. Express the adding component of : dE in terms of u. That introduces either sin u or cos u. Reduce the resulting two variables s and u to one, u, by replacing ds with r du. Integrate over u from one end of the arc to the other end. Straight line, with point P on an extension of the line, as in Fig. 22-14a. In the expression for dE, replace r with x. Integrate over x, from end to end of the line of charge.

643

22-5 TH E E LECTR IC FI E LD DU E TO A CHARG E D DISK

Straight line, with point P at perpendicular distance y from the line of charge, as in Fig. 22-14b. In the expression for dE, replace r with an expression involving x and y. If P is on the perpendicular bisector of the line of : charge, find an expression for the adding component of dE. That will introduce either sin u or cos u. Reduce the resulting two variables x and u to one, x, by replacing the trigonometric function with an expression (its definition) involving x and y. Integrate over x from end to end of the line of charge. If P is not on a line of symmetry, as in Fig. 22-14c, set up an integral to sum the components dEx, and integrate over x to find Ex. Also set up an integral to sum the components dEy, and integrate over x again to find Ey. Use the components Ex and Ey in the usual way to : find the magnitude E and the orientation of E. Step 6. One arrangement of the integration limits gives a positive result.The reverse gives the same result with a mi-

nus sign; discard the minus sign. If the result is to be stated in terms of the total charge Q of the distribution, replace l with Q/L, in which L is the length of the distribution. + + + + + + + + + x

P

(a) P

P

y

y

+ + + + + + + + + x (b)

+ + + + + + + + + x (c)

Figure 22-14 (a) Point P is on an extension of the line of charge. (b) P is on a line of symmetry of the line of charge, at perpendicular distance y from that line. (c) Same as (b) except that P is not on a line of symmetry.

Additional examples, video, and practice available at WileyPLUS

y

Checkpoint 2

y

–Q

The figure here shows three nonconducting rods, one circular and two straight. Each has a uniform charge of magnitude Q along its top half and another along its bottom half. For each rod, what is the direction of the net electric field at point P?

y

+Q P

Px +Q (a)

+Q P

x

+Q (b)

x

–Q (c)

22-5 THE ELECTRIC FIELD DUE TO A CHARGED DISK Learning Objectives After reading this module, you should be able to . . .

22.19 Sketch a disk with uniform charge and indicate the direction of the electric field at a point on the central axis if the charge is positive and if it is negative. 22.20 Explain how the equation for the electric field on the central axis of a uniformly charged ring can be used to find

the equation for the electric field on the central axis of a uniformly charged disk. 22.21 For a point on the central axis of a uniformly charged disk, apply the relationship between the surface charge density s, the disk radius R, and the distance z to that point.

Key Idea ● On the central axis through a uniformly charged disk,

E!

#

s z 1% 2 2´0 2z # R2

$

gives the electric field magnitude. Here z is the distance along the axis from the center of the disk, R is the radius of the disk, and s is the surface charge density.

The Electric Field Due to a Charged Disk Now we switch from a line of charge to a surface of charge by examining the electric field of a circular plastic disk, with a radius R and a uniform surface charge density s (charge per unit area, Table 22-1) on its top surface. The disk sets up a

644

CHAPTE R 22 E LECTR IC FI E LDS

dE P

z

dr

r

R

pattern of electric field lines around it, but here we restrict our attention to the electric field at an arbitrary point P on the central axis, at distance z from the center of the disk, as indicated in Fig. 22-15. We could proceed as in the preceding module but set up a two-dimensional integral to include all of the field contributions from the two-dimensional distribution of charge on the top surface. However, we can save a lot of work with a neat shortcut using our earlier work with the field on the central axis of a thin ring. We superimpose a ring on the disk as shown in Fig. 22-15, at an arbitrary radius r ' R. The ring is so thin that we can treat the charge on it as a charge element dq. To find its small contribution dE to the electric field at point P, we rewrite Eq. 22-16 in terms of the ring’s charge dq and radius r: dE !

Figure 22-15 A disk of radius R and uniform positive charge. The ring shown has radius r and radial width dr. It sets up a differential : electric field dE at point P on its central axis.

dq z . 4p´0(z2 # r2)3/2

(22-22)

The ring’s field points in the positive direction of the z axis. To find the total field at P, we are going to integrate Eq. 22-22 from the center of the disk at r ! 0 out to the rim at r ! R so that we sum all the dE contributions (by sweeping our arbitrary ring over the entire disk surface). However, that means we want to integrate with respect to a variable radius r of the ring. We get dr into the expression by substituting for dq in Eq. 22-22. Because the ring is so thin, call its thickness dr. Then its surface area dA is the product of its circumference 2pr and thickness dr. So, in terms of the surface charge density s, we have dq ! s dA ! s (2pr dr).

(22-23)

After substituting this into Eq. 22-22 and simplifying slightly, we can sum all the dE contributions with E!

"

dE !

sz 4´0

"

R

0

(z2 # r2)%3/2(2r) dr,

(22-24)

where we have pulled the constants (including z) out of the integral. To solve this integral, we cast it in the form " Xm dX by setting X ! (z2 # r 2), m ! %32 , and dX ! (2r) dr. For the recast integral we have

and so Eq. 22-24 becomes

"

Xm dX ! sz 4´0

E!

Xm#1 , m#1

( (z #%r ) ) . 2

2 %1/2

1 2

R

(22-25)

0

Taking the limits in Eq. 22-25 and rearranging, we find E!

s 2´0

#1 %

z 2z2 # R2

$

(charged disk)

(22-26)

as the magnitude of the electric field produced by a flat, circular, charged disk at points on its central axis. (In carrying out the integration, we assumed that z - 0.) If we let R : ` while keeping z finite, the second term in the parentheses in Eq. 22-26 approaches zero, and this equation reduces to E!

s 2´0

(infinite sheet).

(22-27)

This is the electric field produced by an infinite sheet of uniform charge located on one side of a nonconductor such as plastic. The electric field lines for such a situation are shown in Fig. 22-4. We also get Eq. 22-27 if we let z : 0 in Eq. 22-26 while keeping R finite. This shows that at points very close to the disk, the electric field set up by the disk is the same as if the disk were infinite in extent.

22-6 A POI NT CHARG E I N AN E LECTR IC FI E LD

645

22-6 A POINT CHARGE IN AN ELECTRIC FIELD Learning Objectives After reading this module, you should be able to . . .

22.22 For a charged particle placed in an external electric field (a field due to other charged objects), apply the rela: tionship between the electric field E at that point, the parti: cle’s charge q, and the electrostatic force F that acts on the particle, and identify the relative directions of the force

and the field when the particle is positively charged and negatively charged. 22.23 Explain Millikan’s procedure of measuring the elementary charge. 22.24 Explain the general mechanism of ink-jet printing.

Key Ideas ● If a particle with charge q is placed in an external electric : : field E , an electrostatic force F acts on the particle: :

:

F ! qE .

● If charge q is positive, the force vector is in the same direc-

tion as the field vector. If charge q is negative, the force vector is in the opposite direction (the minus sign in the equation reverses the force vector from the field vector).

A Point Charge in an Electric Field In the preceding four modules we worked at the first of our two tasks: given a charge distribution, to find the electric field it produces in the surrounding space. Here we begin the second task: to determine what happens to a charged particle when it is in an electric field set up by other stationary or slowly moving charges. What happens is that an electrostatic force acts on the particle, as given by :

:

(22-28)

F ! qE , :

in which q is the charge of the particle (including its sign) and E is the electric field that other charges have produced at the location of the particle. (The field is not the field set up by the particle itself; to distinguish the two fields, the field acting on the particle in Eq. 22-28 is often called the external field. A charged particle or object is not affected by its own electric field.) Equation 22-28 tells us :

The electrostatic force F acting on a charged particle located in an external electric : : field E has the direction of E if the charge q of the particle is positive and has the opposite direction if q is negative.

Measuring the Elementary Charge Equation 22-28 played a role in the measurement of the elementary charge e by American physicist Robert A. Millikan in 1910 – 1913. Figure 22-16 is a representation of his apparatus. When tiny oil drops are sprayed into chamber A, some of them become charged, either positively or negatively, in the process. Consider a drop that drifts downward through the small hole in plate P1 and into chamber C. Let us assume that this drop has a negative charge q. If switch S in Fig. 22-16 is open as shown, battery B has no electrical effect on chamber C. If the switch is closed (the connection between chamber C and the positive terminal of the battery is then complete), the battery causes an excess positive charge on conducting plate P1 and an excess negative charge on conduct: ing plate P2. The charged plates set up a downward-directed electric field E in chamber C. According to Eq. 22-28, this field exerts an electrostatic force on any charged drop that happens to be in the chamber and affects its motion. In particular, our negatively charged drop will tend to drift upward. By timing the motion of oil drops with the switch opened and with it closed and thus determining the effect of the charge q, Millikan discovered that the

Oil spray

A Insulating chamber wall

P1 Oil drop

C Microscope

P2 S

+

B



Figure 22-16 The Millikan oil-drop apparatus for measuring the elementary charge e. When a charged oil drop drifted into chamber C through the hole in plate P1, its motion could be controlled by closing and opening switch S and thereby setting up or eliminating an electric field in chamber C. The microscope was used to view the drop, to permit timing of its motion.

646

CHAPTE R 22 E LECTR IC FI E LDS Input signals Deflecting plate E

G

C

Deflecting plate

Figure 22-17 Ink-jet printer. Drops shot from generator G receive a charge in charging unit : C. An input signal from a computer controls the charge and thus the effect of field E on where the drop lands on the paper.

values of q were always given by q ! ne,

for n ! 0, .1, .2, .3, . . . ,

(22-29)

in which e turned out to be the fundamental constant we call the elementary charge, 1.60 $ 10%19 C. Millikan’s experiment is convincing proof that charge is quantized, and he earned the 1923 Nobel Prize in physics in part for this work. Modern measurements of the elementary charge rely on a variety of interlocking experiments, all more precise than the pioneering experiment of Millikan.

Ink-Jet Printing The need for high-quality, high-speed printing has caused a search for an alternative to impact printing, such as occurs in a standard typewriter. Building up letters by squirting tiny drops of ink at the paper is one such alternative. Figure 22-17 shows a negatively charged drop moving between two conduct: ing deflecting plates, between which a uniform, downward-directed electric field E has been set up. The drop is deflected upward according to Eq. 22-28 and then : strikes the paper at a position that is determined by the magnitudes of E and the charge q of the drop. In practice, E is held constant and the position of the drop is determined by the charge q delivered to the drop in the charging unit, through which the drop must pass before entering the deflecting system. The charging unit, in turn, is activated by electronic signals that encode the material to be printed.

Electrical Breakdown and Sparking If the magnitude of an electric field in air exceeds a certain critical value Ec, the air undergoes electrical breakdown, a process whereby the field removes electrons from the atoms in the air. The air then begins to conduct electric current because the freed electrons are propelled into motion by the field. As they move, they collide with any atoms in their path, causing those atoms to emit light. We can see the paths, commonly called sparks, taken by the freed electrons because of that emitted light. Figure 22-18 shows sparks above charged metal wires where the electric fields due to the wires cause electrical breakdown of the air.

Checkpoint 3

Adam Hart-Davis/Photo Researchers, Inc.

Figure 22-18 The metal wires are so charged that the electric fields they produce in the surrounding space cause the air there to undergo electrical breakdown.

(a) In the figure, what is the direction of the electrostatic force on the electron due to the external electric field shown? (b) In which direction will the electron accelerate if it is moving parallel to the y axis before it encounters the external field? (c) If, instead, the electron is initially moving rightward, will its speed increase, decrease, or remain constant?

y E

e

x

647

22-7 A DI POLE I N AN E LECTR IC FI E LD

Sample Problem 22.04 Motion of a charged particle in an electric field y Figure 22-19 shows the deflecting plates of an ink-jet Plate printer, with superimposed coordinate axes. An ink drop m,Q with a mass m of 1.3 $ 10%10 E x=L kg and a negative charge of 0 Plate magnitude Q ! 1.5 $ 10%13 C enters the region between the plates, initially moving Figure 22-19 An ink drop of along the x axis with speed mass m and charge magnitude vx ! 18 m/s. The length L of Q is deflected in the electric each plate is 1.6 cm. The field of an ink-jet printer. plates are charged and thus produce an electric field at all : points between them. Assume that field E is downward directed, is uniform, and has a magnitude of 1.4 $ 106 N/C. What is the vertical deflection of the drop at the far edge of the plates? (The gravitational force on the drop is small relative to the electrostatic force acting on the drop and can be neglected.)

magnitude QE acts upward on the charged drop. Thus, as the drop travels parallel to the x axis at constant speed vx, it accelerates upward with some constant acceleration ay. x

Calculations: Applying Newton’s second law (F ! ma) for components along the y axis, we find that ay !

The drop is negatively charged and the electric field is directed downward. From Eq. 22-28, a constant electrostatic force of

(22-30)

Let t represent the time required for the drop to pass through the region between the plates. During t the vertical and horizontal displacements of the drop are y ! 12 ay t2

and

L ! vx t,

(22-31)

respectively. Eliminating t between these two equations and substituting Eq. 22-30 for ay, we find y! !

KEY IDEA

F QE ! . m m

QEL2 2mv2x (1.5 $ 10%13 C)(1.4 $ 106 N/C)(1.6 $ 10%2 m)2 (2)(1.3 $ 10%10 kg)(18 m/s)2

! 6.4 $ 10%4 m ! 0.64 mm.

(Answer)

Additional examples, video, and practice available at WileyPLUS

22-7 A DIPOLE IN AN ELECTRIC FIELD Learning Objectives After reading this module, you should be able to . . .

22.25 On a sketch of an electric dipole in an external electric field, indicate the direction of the field, the direction of the dipole moment, the direction of the electrostatic forces on the two ends of the dipole, and the direction in which those forces tend to rotate the dipole, and identify the value of the net force on the dipole. 22.26 Calculate the torque on an electric dipole in an external electric field by evaluating a cross product of the dipole moment vector and the electric field vector, in magnitudeangle notation and unit-vector notation.

22.27 For an electric dipole in an external electric field, relate the potential energy of the dipole to the work done by a torque as the dipole rotates in the electric field. 22.28 For an electric dipole in an external electric field, calculate the potential energy by taking a dot product of the dipole moment vector and the electric field vector, in magnitudeangle notation and unit-vector notation. 22.29 For an electric dipole in an external electric field, identify the angles for the minimum and maximum potential energies and the angles for the minimum and maximum torque magnitudes.

Key Ideas ● The torque on an electric dipole of dipole moment p

when placed in an external electric field E is given by a cross product: :

:

:

: $ E. t: ! p ● A potential energy U is associated with the orientation of the dipole moment in the field, as given by a dot product:

:

: U ! %p " E.

● If the dipole orientation changes, the work done by the

electric field is W ! %/U. If the change in orientation is due to an external agent, the work done by the agent is Wa ! %W.

648

CHAPTE R 22 E LECTR IC FI E LDS

A Dipole in an Electric Field

Positive side Hydrogen

Hydrogen p

105° Oxygen

Negative side

Figure 22-20 A molecule of H2O, showing the three nuclei (represented by dots) and the regions in which the electrons can be lo: cated.The electric dipole moment p points from the (negative) oxygen side to the (positive) hydrogen side of the molecule.

We have defined the electric dipole moment p: of an electric dipole to be a vector that points from the negative to the positive end of the dipole.As you will see, the behavior : of a dipole in a uniform external electric field E can be described completely in terms : of the two vectors E and p:, with no need of any details about the dipole’s structure. A molecule of water (H2O) is an electric dipole; Fig. 22-20 shows why. There the black dots represent the oxygen nucleus (having eight protons) and the two hydrogen nuclei (having one proton each). The colored enclosed areas represent the regions in which electrons can be located around the nuclei. In a water molecule, the two hydrogen atoms and the oxygen atom do not lie on a straight line but form an angle of about 105", as shown in Fig. 22-20. As a result, the molecule has a definite “oxygen side” and “hydrogen side.” Moreover, the 10 electrons of the molecule tend to remain closer to the oxygen nucleus than to the hydrogen nuclei. This makes the oxygen side of the molecule slightly more negative than the hydrogen side and creates an electric dipole moment p: that points along the symmetry axis of the molecule as shown. If the water molecule is placed in an external electric field, it behaves as would be expected of the more abstract electric dipole of Fig. 22-9. To examine this behavior, we now consider such an abstract dipole in a uniform : external electric field E , as shown in Fig. 22-21a. We assume that the dipole is a rigid structure that consists of two centers of opposite charge, each of magnitude q, sepa: rated by a distance d.The dipole moment p: makes an angle u with field E . Electrostatic forces act on the charged ends of the dipole. Because the electric field is uniform, those forces act in opposite directions (as shown in Fig. 22-21a) and with the same magnitude F ! qE. Thus, because the field is uniform, the net force on the dipole from the field is zero and the center of mass of the dipole does not move. However, the forces on the charged ends do produce a net torque t: on the dipole about its center of mass. The center of mass lies on the line connecting the charged ends, at some distance x from one end and thus a distance d % x from the other end. From Eq. 10-39 (t ! rF sin f), we can write the magnitude of the net torque t: as t ! Fx sin u # F(d % x) sin u ! Fd sin u.

(22-32)

We can also write the magnitude of t: in terms of the magnitudes of the electric field E and the dipole moment p ! qd. To do so, we substitute qE for F and p/q for d in Eq. 22-32, finding that the magnitude of t: is t ! pE sin u.

(22-33)

We can generalize this equation to vector form as :

t: ! p: $ E

(torque on a dipole).

(22-34)

:

: Vectors p and E are shown in Fig. 22-21b. The torque acting on a dipole tends to : : rotate p (hence the dipole) into the direction of field E , thereby reducing u. In Fig. 22-21, such rotation is clockwise. As we discussed in Chapter 10, we can represent a torque that gives rise to a clockwise rotation by including a minus sign with the magnitude of the torque. With that notation, the torque of Fig. 22-21 is

t ! %pE sin u.

(22-35)

Potential Energy of an Electric Dipole Potential energy can be associated with the orientation of an electric dipole in an electric field. The dipole has its least potential energy when it is in its equi: librium orientation, which is when its moment p: is lined up with the field E : (then t: ! p: $ E ! 0). It has greater potential energy in all other orientations. Thus the dipole is like a pendulum, which has its least gravitational potential

649

22-7 A DI POLE I N AN E LECTR IC FI E LD

energy in its equilibrium orientation — at its lowest point. To rotate the dipole or the pendulum to any other orientation requires work by some external agent. In any situation involving potential energy, we are free to define the zeropotential-energy configuration in an arbitrary way because only differences in potential energy have physical meaning. The expression for the potential energy of an electric dipole in an external electric field is simplest if we choose the potential energy to be zero when the angle u in Fig. 22-21 is 90". We then can find the potential energy U of the dipole at any other value of u with Eq. 8-1 (/U ! %W) by calculating the work W done by the field on the dipole when the dipole is rotated to that value of u from 90". With the aid of Eq. 10-53 (W ! *t du) and Eq. 22-35, we find that the potential energy U at any angle u is

"

U ! %W ! %

u

90"

t du !

"

u

90"

θ com E

–q

–F

(a)

The dipole is being torqued into alignment. p

τ

Evaluating the integral leads to

F

p

(22-36)

pE sin u du.

+q

d

θ

E (b)

U ! %pE cos u.

(22-37)

We can generalize this equation to vector form as :

U ! %p: " E

(22-38)

(potential energy of a dipole).

Equations 22-37 and 22-38 show us that the potential energy of the dipole is least : (U ! %pE) when 0 ! 0 ( p: and E are in the same direction); the potential energy is : greatest (U ! pE) when u ! 180" ( p: and E are in opposite directions). When a dipole rotates from an initial orientation ui to another orientation uf , the work W done on the dipole by the electric field is W ! %/U ! %(Uf % Ui ),

(22-39)

where Uf and Ui are calculated with Eq. 22-38. If the change in orientation is caused by an applied torque (commonly said to be due to an external agent), then the work Wa done on the dipole by the applied torque is the negative of the work done on the dipole by the field; that is, Wa ! %W ! (Uf % Ui ).

(22-40)

Microwave Cooking Food can be warmed and cooked in a microwave oven if the food contains water because water molecules are electric dipoles. When you turn on the oven, the mi: crowave source sets up a rapidly oscillating electric field E within the oven and : thus also within the food. From Eq. 22-34, we see that any electric field E pro: : : duces a torque on an electric dipole moment p to align p with E. Because the : oven’s E oscillates, the water molecules continuously flip-flop in a frustrated at: tempt to align with E. Energy is transferred from the electric field to the thermal energy of the water (and thus of the food) where three water molecules happened to have bonded together to form a group. The flip-flop breaks some of the bonds. When the molecules reform the bonds, energy is transferred to the random motion of the group and then to the surrounding molecules. Soon, the thermal energy of the water is enough to cook the food.

Checkpoint 4 The figure shows four orientations of an electric dipole in an external electric field. Rank the orientations according to (a) the magnitude of the torque on the dipole and (b) the potential energy of the dipole, greatest first.

(1) +

(3)

+

+ (2) θ

θ

θ

θ

E

+ (4)

Figure 22-21 (a) An electric dipole in a : uniform external electric field E . Two centers of equal but opposite charge are separated by distance d. The line between them represents their rigid connection. (b) Field : E causes a torque t: on the dipole. The direction of t: is into the page, as represented by the symbol #.

650

CHAPTE R 22 E LECTR IC FI E LDS

Sample Problem 22.05 Torque and energy of an electric dipole in an electric field A neutral water molecule (H2O) in its vapor state has an electric dipole moment of magnitude 6.2 $ 10%30 C (m. (a) How far apart are the molecule’s centers of positive and negative charge?

KEY IDEA The torque on a dipole is maximum when the angle u be: tween p: and E is 90". Calculation: Substituting u ! 90" in Eq. 22-33 yields

KEY IDEA A molecule’s dipole moment depends on the magnitude q of the molecule’s positive or negative charge and the charge separation d.

t ! pE sin u ! (6.2 $ 10%30 C (m)(1.5 $ 104 N/C)(sin 90") (Answer) ! 9.3 $ 10%26 N(m.

Calculations: There are 10 electrons and 10 protons in a neutral water molecule; so the magnitude of its dipole moment is p ! qd ! (10e)(d),

(c) How much work must an external agent do to rotate this molecule by 180" in this field, starting from its fully aligned position, for which u ! 0?

in which d is the separation we are seeking and e is the elementary charge. Thus,

KEY IDEA

6.2 $ 10%30 C (m p d! ! 10e (10)(1.60 $ 10%19 C) ! 3.9 $ 10%12 m ! 3.9 pm.

(Answer)

This distance is not only small, but it is also actually smaller than the radius of a hydrogen atom. (b) If the molecule is placed in an electric field of 1.5 $ 104 N/C, what maximum torque can the field exert on it? (Such a field can easily be set up in the laboratory.)

The work done by an external agent (by means of a torque applied to the molecule) is equal to the change in the molecule’s potential energy due to the change in orientation. Calculation: From Eq. 22-40, we find Wa ! U180" % U 0 ! (%pE cos 180") % (%pE cos 0) ! 2pE ! (2)(6.2 $ 10%30 C (m)(1.5 $ 104 N/C) ! 1.9 $ 10%25 J. (Answer)

Additional examples, video, and practice available at WileyPLUS

Review & Summary Electric Field

To explain the electrostatic force between two charges, we assume that each charge sets up an electric field in the space around it. The force acting on each charge is then due to the electric field set up at its location by the other charge. :

Definition of Electric Field The electric field E at any point :

is defined in terms of the electrostatic force F that would be exerted on a positive test charge q0 placed there: :

F E! . q0 :

(22-1)

Electric Field Lines Electric field lines provide a means for visualizing the direction and magnitude of electric fields. The electric field vector at any point is tangent to a field line through that point. The density of field lines in any region is proportional to the magnitude of the electric field in that region. Field lines originate on positive charges and terminate on negative charges.

Field Due to a Point Charge The magnitude of the electric :

field E set up by a point charge q at a distance r from the charge is E!

1 !q! . 4p´0 r 2

(22-3)

:

The direction of E is away from the point charge if the charge is positive and toward it if the charge is negative.

Field Due to an Electric Dipole An electric dipole consists of two particles with charges of equal magnitude q but opposite sign, separated by a small distance d. Their electric dipole moment : has magnitude qd and points from the negative charge to the p positive charge. The magnitude of the electric field set up by the dipole at a distant point on the dipole axis (which runs through both charges) is 1 p (22-9) E! , 2p´0 z3 where z is the distance between the point and the center of the dipole.

Field Due to a Continuous Charge Distribution The electric field due to a continuous charge distribution is found by treating charge elements as point charges and then summing, via integration, the electric field vectors produced by all the charge elements to find the net vector.

651

QU ESTIONS

Field Due to a Charged Disk The electric field magnitude at a point on the central axis through a uniformly charged disk is given by s z (22-26) E! 1% , 2´0 2z2 # R2

#

$

where z is the distance along the axis from the center of the disk, R is the radius of the disk, and s is the surface charge density.

Force on a Point Charge in an Electric Field When a :

point charge q is placed in an external electric field E , the electro: static force F that acts on the point charge is :

:

F ! qE .

(22-28)

:

:

Force F has the same direction as E if q is positive and the opposite direction if q is negative.

Dipole in an Electric Field When an electric dipole of dipole :

: moment p is placed in an electric field E , the field exerts a torque t: on the dipole: : : (22-34) t: ! p $ E.

The dipole has a potential energy U associated with its orientation in the field: : : (22-38) U ! %p " E. : This potential energy is defined to be zero when p is perpendicular : : : to E ; it is least (U ! %pE) when p is aligned with E and greatest : : (U ! pE) when p is directed opposite E .

Questions 1 Figure 22-22 shows three arrangements of electric field lines. In each arrangement, a proton is released from rest at point A and is then accelerated through point B by the electric field. Points A and B have equal separations in the three arrangements. Rank the arrangements according to the linear momentum of the proton at point B, greatest first.

A

B

(a)

A

B

A

B

(b)

(c)

Figure 22-22 Question 1. 2 Figure 22-23 shows two +6q square arrays of charged particles. The squares, which are centered on point P, are misaligned.The particles are separated by either d or d/2 –2q along the perimeters of the squares. What are the magnitude and direction of the net electric field at P?

–2q

+2q

–3q –q +2q

+3q

P

–q

–q

–3q

3 In Fig. 22-24, two particles +3q +6q –2q of charge %q are arranged symmetrically about the y Figure 22-23 Question 2. axis; each produces an electric field at point P on that axis. (a) Are the magnitudes of the fields at P equal? (b) Is each electric field directed toward or away from the charge producing it? (c) Is the magnitude of the net electric field at P equal to the sum of the magnitudes E of the two field vecy tors (is it equal to 2E )? (d) Do the x P components of those two field vectors add or cancel? (e) Do their y components add or cancel? (f ) Is –q –q the direction of the net field at P x that of the canceling components or d d the adding components? (g) What is Figure 22-24 Question 3. the direction of the net field?

4 Figure 22-25 shows four situations in which four charged particles are evenly spaced to the left and right of a central point. The charge values are indicated. Rank the situations accord ing to the magnitude of the net electric field at the central point, greatest first.

(1)

(2)

(3)

(4)

+e

–e

–e

+e

+e

+e

–e

–e

–e

+e

+e

+e

–e

–e

+e

–e

5 Figure 22-26 shows two d d d d charged particles fixed in Figure 22-25 Question 4. place on an axis. (a) Where on the axis (other than at an infinite distance) is there a point at which +q –3q their net electric field is zero: between the charges, to their left, or to Figure 22-26 Question 5. their right? (b) Is there a point of zero net electric field anywhere off the axis (other than at an infinite distance)? 6 In Fig. 22-27, two identical circuP1 P2 P3 lar nonconducting rings are centered on the same line with their planes Ring A Ring B perpendicular to the line. Each ring has charge that is uniformly distribFigure 22-27 Question 6. uted along its circumference. The rings each produce electric fields at points along the line. For three situations, the charges on rings A and B are, respectively, (1) q0 and q0, (2) %q0 and %q0, and (3) %q0 and q0. Rank the situations according to the magnitude of the net electric field at (a) point P1 midway between the rings, (b) point P2 at the center of ring B, and (c) point P3 to the right of ring B, greatest first. 7 The potential energies associated with four orientations of an electric dipole in an electric field are (1) %5U0, (2) %7U0, (3) 3U0, and (4) 5U0, where U0 is positive. Rank the orientations according : to (a) the angle between the electric dipole moment p and the elec: tric field E and (b) the magnitude of the torque on the electric dipole, greatest first. 8 (a) In Checkpoint 4, if the dipole rotates from orientation 1 to orientation 2, is the work done on the dipole by the field positive, negative, or zero? (b) If, instead, the dipole rotates from orientation 1 to orientation 4, is the work done by the field more than, less than, or the same as in (a)?

652

CHAPTE R 22 E LECTR IC FI E LDS

9 Figure 22-28 shows two disks and a flat ring, each with the same uniform charge Q. Rank the objects according to the magnitude of the electric field they create at points P (which are at the same vertical heights), greatest first. P

P

P

R

R

2R

(a)

(b)

curvature (at the origin). In Figs. 22-30b, c, and d, more circular rods, each with identical uniform charges #Q, are added until the circle is complete. A fifth arrangement (which would be labeled e) is like that in d except the rod in the fourth quadrant has charge %Q. Rank the five arrangements according to the magnitude of the electric field at the center of curvature, greatest first. 12 When three electric dipoles are near each other, they each experience the electric field of the other two, and the three-dipole system has a certain potential energy. Figure 22-31 shows two arrangements in which three electric dipoles are side by side. Each dipole has the same magnitude of electric dipole moment, and the spacings between adjacent dipoles are identical. In which arrangement is the potential energy of the three-dipole system greater?

2R

(c)

Figure 22-28 Question 9. 10 In Fig. 22-29, an electron e travels through a small hole in plate A and then toward plate B. A uniform electric field in the region between e +q2 the plates then slows the electron without deflecting it. (a) What is the +q1 –q3 direction of the field? (b) Four other n particles similarly travel through A B small holes in either plate A or plate Figure 22-29 Question 10. B and then into the region between the plates. Three have charges #q1, #q2, and %q3. The fourth (labeled n) is a neutron, which is electrically neutral. Does the speed of each of those four other particles increase, decrease, or remain the same in the region between the plates?

(a)

Figure 22-31 Question 12. 13 Figure 22-32 shows three rods, each with the same charge Q spread uniformly along its length. Rods a (of length L) and b (of length L/2) are straight, and points P are aligned with their midpoints. Rod c (of length L/2) forms a complete circle about point P. Rank the rods according to the magnitude of the electric field they create at points P, greatest first.

11 In Fig. 22-30a, a circular plastic rod with uniform charge #Q produces an electric field of magnitude E at the center of y

(b)

y

P

P

P

L

L/2

L/2

(a)

(b)

(c)

Figure 22-32 Question 13. x (a)

x (b)

y

y

14 Figure 22-33 shows five protons that are launched in a uni: form electric field E ; the magnitude and direction of the launch velocities are indicated. Rank the protons according to the magnitude of their accelerations due to the field, greatest first. E

x

(c)

x

a 10 m/s

(d)

Figure 22-30 Question 11.

3 m/s

5 m/s

b

d

c

7 m/s e

16 m/s

Figure 22-33 Question 14.

Problems Tutoring problem available (at instructor’s discretion) in WileyPLUS and WebAssign SSM • – •••

Worked-out solution available in Student Solutions Manual Number of dots indicates level of problem difficulty

WWW Worked-out solution is at ILW

Interactive solution is at

http://www.wiley.com/college/halliday

Additional information available in The Flying Circus of Physics and at flyingcircusofphysics.com

Module 22-1 The Electric Field •1 Sketch qualitatively the electric field lines both between and outside two concentric conducting spherical shells when a uniform

positive charge q1 is on the inner shell and a uniform negative charge %q2 is on the outer. Consider the cases q1 1 q2, q1 ! q2, and q1 2 q2.

653

PROB LE M S

B

••10 Figure 22-38a shows two charged particles fixed in place on an x axis with separation L. The ratio q1/q2 of their charge magnitudes is 4.00. Figure 22-38b shows the x component Enet,x of their net electric field along the x axis just to the right of particle 2. The x axis scale is set by xs ! 30.0 cm. (a) At what value of x 1 0 is Enet,x maximum? (b) If particle 2 has charge %q2 ! %3e, what is the value of that maximum?

A

Figure 22-34 Problem 2.

Module 22-2 The Electric Field Due to a Charged Particle •3 SSM The nucleus of a plutonium-239 atom contains 94 protons. Assume that the nucleus is a sphere with radius 6.64 fm and with the charge of the protons uniformly spread through the sphere. At the surface of the nucleus, what are the (a) magnitude and (b) direction (radially inward or outward) of the electric field produced by the protons? •4 Two charged particles are attached to an x axis: Particle 1 of charge %2.00 $ 10%7 C is at position x ! 6.00 cm and particle 2 of charge #2.00 $ 10%7 C is at position x ! 21.0 cm. Midway between the particles, what is their net electric field in unit-vector notation? •5 SSM A charged particle produces an electric field with a magnitude of 2.0 N/C at a point that is 50 cm away from the particle. What is the magnitude of the particle’s charge? •6 What is the magnitude of a point charge that would create an electric field of 1.00 N/C at points 1.00 y m away? q1

••7 SSM ILW WWW In Fig. 22-35, the four particles form a square of edge length a ! 5.00 cm and have charges q1 ! #10.0 nC, q2 ! %20.0 nC, q3 ! #20.0 nC, and q4 ! %10.0 nC. In unitvector notation, what net electric field do the particles produce at the square’s center?

q2

a q4

q3 a

••8 In Fig. 22-36, the four particles are fixed in place and have charges q1 ! q2 ! #5e, q3 ! #3e, and q4 ! %12e. Distance d ! 5.0 mm. What is the magnitude of the net electric field at point P due to the particles?

x

Figure 22-35 Problem 7.

d q1 d

d

q4

q3

••9 Figure 22-37 shows two P charged particles on an x axis: %q ! d q2 %19 %3.20 $ 10 C at x ! %3.00 m and %19 q ! 3.20 $ 10 C at x ! #3.00 m. What are the (a) magnitude and Figure 22-36 Problem 8. (b) direction (relative to the positive direction of the x axis) of the net electric field produced at point P at y ! 4.00 m? y P

y +q1

–q2 L

x

Enet,x (10–8 N/C)

•2 In Fig. 22-34 the electric field lines on the left have twice the separation of those on the right. (a) If the magnitude of the field at A is 40 N/C, what is the magnitude of the force on a proton at A?(b) What is the magnitude of the field at B?

2 0

xs

0

–2 –4 x (cm) (b)

(a)

Figure 22-38 Problem 10. ••11 SSM Two charged particles are fixed to an x axis: Particle 1 of charge q1 ! 2.1 $ 10%8 C is at position x ! 20 cm and particle 2 of charge q2 ! %4.00q1 is at position x ! 70 cm. At what coordinate on the axis (other than at infinity) is the net electric field produced by the two particles equal to zero? y ••12 Figure 22-39 shows an uneven arrangement of electrons (e) p and protons (p) on a circular arc of e radius r ! 2.00 cm, with angles θ2 p u1 ! 30.0", u2 ! 50.0", u3 ! 30.0", and p θ3 θ u4 ! 20.0". What are the (a) magniθ4 1 e x tude and (b) direction (relative to the positive direction of the x axis) of the net electric field produced at the cenFigure 22-39 Problem 12. ter of the arc? z

••13 Figure 22-40 shows a proton p (p) on the central axis through a disk with a uniform charge density due to excess electrons. The disk is seen from es ec es an edge-on view. Three of those elecR R trons are shown: electron ec at the disk center and electrons es at oppoFigure 22-40 Problem 13. site sides of the disk, at radius R from the center. The proton is initially at distance z ! R ! 2.00 cm from the disk. At that location, what are : the magnitudes of (a) the electric field Ec due to electron ec and (b) : the net electric field Es,net due to electrons es? The proton is then moved to z ! R/10.0. What then are the magnitudes of : : (c) Ec and (d) Es,net at the proton’s location? (e) From (a) and (c) we see that as the proton gets nearer to the disk, the magnitude of : Ec increases, as expected. Why does the : y magnitude of Es,net from the two side electrons decrease, as we see from (b) and (d)? x q

–q

q

Figure 22-37 Problem 9.

x

q

1 2 ••14 In Fig. 22-41, particle 1 of charge L q1 ! %5.00q and particle 2 of charge q2 ! #2.00q are fixed to an x axis. (a) As a Figure 22-41 Problem 14. multiple of distance L, at what coordinate on the axis is the net electric field of the particles zero? (b) Sketch the net electric field lines between and around the particles.

654

CHAPTE R 22 E LECTR IC FI E LDS

••15 In Fig. 22-42, the three particles are fixed in place and have charges q1 ! q2 ! #e and q3 ! #2e. Distance a ! 6.00 mm. What are the (a) magnitude and (b) direction of the net electric field at point P due to the particles?

y

••19 Figure 22-45 shows an electric dipole. What are the (a) magnitude and (b) direction (relative to the positive direction of the x axis) of the dipole’s electric field at point P, located at distance r ) d?

1 a

P

+q 2

3

•••16 Figure 22-43 shows a plastic ring of radius R ! 50.0 cm. Two small charged beads are on the ring: Bead 1 of charge #2.00 mC is fixed in place at the left side; bead 2 of charge #6.00 mC can be moved along the ring. The two Ring beads produce a net electric field of magnitude E at the center of the ring. At what (a) positive and (b) negative value of angle u should Bead 1 bead 2 be positioned such that E ! 2.00 $ 105 N/C?

x

–q

x

Ring Bead 1

θ

x

0

90°

180° θ

Ey (104 N/C)

Ex (104 N/C)

Exs

θ 180° (c)

–Exs

Eys

Figure 22-44 Problem 17. Module 22-3 The Electric Field Due to a Dipole ••18 The electric field of an electric dipole along the dipole axis is approximated by Eqs. 22-8 and 22-9. If a binomial expansion is made of Eq. 22-7, what is the next term in the expression for the dipole’s electric field along the dipole axis? That is, what is Enext in the expression E!

••20 Equations 22-8 and 22-9 are approximations of the magnitude of the electric field of an electric dipole, at points along the dipole axis. Consider a point P on that axis at distance z ! 5.00d from the dipole center (d is the separation distance between the particles of the dipole). Let Eappr be the magnitude of the field at point P as approximated by Eqs. 22-8 and 22-9. Let Eact be the actual magnitude.What is the ratio Eappr /Eact? •••21 SSM Electric quadrupole. Figure 22-46 shows a generic electric quadrupole. It consists of two dipoles with dipole moments that are equal in magnitude but opposite in direction. Show that the value of E on the axis of the quadrupole for a point P a distance z from its center (assume z ) d) is given by

1 qd # Enext? 2p´0 z3

z d

–+

d

– –

+

–q –q

+q –p

P

+q +p

Figure 22-46 Problem 21.

3Q , 4p´0z4

in which Q (! 2qd 2) is known as the quadrupole moment of the charge distribution.

(a)

(b)



E!

R

P

Figure 22-45 Problem 19.

Bead 2

y

90°

x

d/2

y

θ

y

r

Figure 22-42 Problem 15.

•••17 Two charged beads are on the plastic ring in Fig. 22-44a. Bead Figure 22-43 Problem 16. 2, which is not shown, is fixed in place on the ring, which has radius R ! 60.0 cm. Bead 1, which is not fixed in place, is initially on the x axis at angle u ! 0". It is then moved to the opposite side, at angle u ! 180", through the first and second quadrants of the xy coordinate system. Figure 22-44b gives the x component of the net electric field produced at the origin by the two beads as a function of u, and Fig. 22-44c gives the y component of that net electric field. The vertical axis scales are set by Exs ! 5.0 $ 104 N/C and Eys ! %9.0 $ 104 N/C. (a) At what angle u is bead 2 located? What are the charges of (b) bead 1 and (c) bead 2?

0

d/2

a

R

+

Module 22-4 The Electric Field Due to a Line of Charge •22 Density, density, density. (a) A charge %300e is uniformly distributed along a circular arc of radius 4.00 cm, which subtends an angle of 40". What is the linear charge density along the arc? (b) A charge %300e is uniformly distributed over one face of a circular disk of radius 2.00 cm. What is the surface charge density over that face? (c) A charge %300e is uniformly distributed over the surface of a sphere of radius 2.00 cm. What is the surface charge density over that surface? (d) A charge %300e is uniformly spread through the volume of a sphere of radius 2.00 cm. What is the volume charge density in that sphere? •23 Figure 22-47 shows two parallel nonconducting rings with their central axes along a common line. Ring 1 has uniform charge q1 and radius R; ring 2 has uniform charge q2 and the same radius R. The rings are separated by distance d ! 3.00R. The net electric field at point P on the common line, at distance R from ring 1, is zero.What is the ratio q1/q2?

Ring 1

q1

Ring 2

q2

P R

R R

d

Figure 22-47 Problem 23.

••24 A thin nonconducting rod with a uniform distribution of positive charge Q is bent into a complete circle of radius R

655

PROB LE M S

(Fig. 22-48). The central perpendicular axis through the ring is a z axis, with the origin at the center of the ring. What is the magnitude of the electric field due to the rod at (a) z ! 0 and (b) z ! &? (c) In terms of R, at what positive value of z is that magnitude maximum? (d) If R ! 2.00 cm and Q ! 4.00 mC, what is the maximum magnitude? ••25 Figure 22-49 shows three circular arcs centered on the origin of a coordinate system. On each arc, the uniformly distributed charge is given in terms of Q ! 2.00 mC. The radii are given in terms of R ! 10.0 cm. What are the (a) magnitude and (b) direction (relative to the positive x direction) of the net electric field at the origin due to the arcs?

z

••30 Figure 22-53 shows two concentric rings, of radii R and R, ! 3.00R, that lie on the same plane. Point P lies on the central z axis, at distance D ! 2.00R from the center of the rings. The smaller ring has uniformly distributed charge #Q. In terms of Q, what is the uniformly distributed charge on the larger ring if the net electric field at P is zero?

R

Figure 22-48 Problem 24. y +9Q

3R –4Q

2R

+Q

R x

Figure 22-49 Problem 25. y

ILW In Fig. 22-50, a thin glass rod ••26 forms a semicircle of radius r ! 5.00 cm. Charge is uniformly distributed along the rod, with #q ! 4.50 pC in the upper half and %q ! %4.50 pC in the lower half.What are the (a) magnitude and (b) direction (relative to the positive direction of the x axis) of the electric : field E at P, the center of the semicircle?

••27 In Fig. 22-51, two curved plastic rods, one of charge #q and the other of charge %q, form a circle of radius R ! 8.50 cm in an xy plane. The x axis passes through both of the connecting points, and the charge is distributed uniformly on both rods. If q ! 15.0 pC, what are the (a) magnitude and (b) direction (relative to the positive direction of the x axis) of : the electric field E produced at P, the center of the circle?

+q

r

P

x

–q

Figure 22-50 Problem 26. y +q P

x

–q

Figure 22-51 Problem 27.

••28 Charge is uniformly distributed around a ring of radius R ! 2.40 cm, and the resulting electric field magnitude E is measured along the ring’s central axis (perpendicular to the plane of the ring). At what distance from the ring’s center is E maximum? ••29 Figure 22-52a shows a nonconducting rod with a uniformly distributed charge #Q.The rod forms a half-circle with radius R and produces an electric field of magnitude Earc at its center of curvature P. If the arc is collapsed to a point at distance R from P (Fig. 22-52b), by what factor is the magnitude of the electric field at P multiplied?

+Q

+Q

P

R

R (a)

P

(b)

Figure 22-52 Problem 29.

z P D

R

R'

Figure 22-53 Problem 30. ••31 SSM ILW WWW In Fig. 22-54, a nonconducting rod of length L ! 8.15 cm has a charge %q ! %4.23 fC –q P uniformly distributed along its length. – – – – x (a) What is the linear charge density a L of the rod? What are the (b) magnitude and (c) direction (relative to Figure 22-54 Problem 31. the positive direction of the x axis) of the electric field produced at point P, at distance a ! 12.0 cm from the rod? What is the electric field magnitude produced at distance a !50 m by (d) the rod and (e) a particle of charge %q ! %4.23 fC that we use to replace the rod? (At that distance, the rod y “looks” like a particle.) P •••32 In Fig. 22-55, positive charge q ! 7.81 pC is spread uniformly along a thin nonconducting rod of length L ! 14.5 cm. What are the (a) magnitude and (b) direction (relative to the positive direction of the x axis) of the electric field produced at point P, at distance R ! 6.00 cm from the rod along its perpendicular bisector?

R

+ + + + + + + + +

x

L

Figure 22-55 Problem 32.

•••33 In Fig. 22-56, a “semi+ + + + infinite” nonconducting rod (that is, infinite in one direction only) has uniform linear charge density l. R : Show that the electric field Ep at point P makes an angle of 45" with the rod P and that this result is independent of the distance R. (Hint: Separately find Figure 22-56 Problem 33. : the component of Ep parallel to the rod and the component perpendicular to the rod.) Module 22-5 The Electric Field Due to a Charged Disk •34 A disk of radius 2.5 cm has a surface charge density of 5.3 mC/m2 on its upper face. What is the magnitude of the electric field produced by the disk at a point on its central axis at distance z ! 12 cm from the disk? •35 SSM WWW At what distance along the central perpendicular axis of a uniformly charged plastic disk of radius 0.600 m is the magnitude of the electric field equal to one-half the magnitude of the field at the center of the surface of the disk? ••36 A circular plastic disk with radius R ! 2.00 cm has a uniformly distributed charge Q ! #(2.00 $ 106)e on one face. A circular ring of width 30 mm is centered on that face, with the center of that width at radius r ! 0.50 cm. In coulombs, what charge is contained within the width of the ring?

656

CHAPTE R 22 E LECTR IC FI E LDS

z z ••37 Suppose you design an apparatus in which a uniformly charged P P disk of radius R is to produce an electric field. The field magnitude is most important along the central perpendicular axis of the disk, at a point P at distance 2.00R from the disk (Fig. 22-57a). Cost analysis suggests that you switch to a ring of the (a) (b) same outer radius R but with inner radius R/2.00 (Fig. 22-57b). Assume Figure 22-57 Problem 37. that the ring will have the same surface charge density as the original disk. If you switch to the ring, by what percentage will you decrease the electric field magnitude at P?

••38 Figure 22-58a shows a circular disk that is uniformly charged. The central z axis is perpendicular to the disk face, with the origin at the disk. Figure 22-58b gives the magnitude of the electric field along that axis in terms of the maximum magnitude Em at the disk surface. The z axis scale is set by zs ! 8.0 cm.What is the radius of the disk?

•44 An alpha particle (the nucleus of a helium atom) has a mass of 6.64 $ 10%27 kg and a charge of #2e. What are the (a) magnitude and (b) direction of the electric field that will balance the gravitational force on the particle? •45 ILW An electron on the axis of an electric dipole is 25 nm from the center of the dipole. What is the magnitude of the electrostatic force on the electron if the dipole moment is 3.6 $ 10%29 C (m? Assume that 25 nm is much larger than the separation of the charged particles that form the dipole. •46 An electron is accelerated eastward at 1.80 $ 109 m/s2 by an electric field. Determine the field (a) magnitude and (b) direction. •47 SSM Beams of high-speed protons can be produced in “guns” using electric fields to accelerate the protons. (a) What acceleration would a proton experience if the gun’s electric field were 2.00 $ 104 N/C? (b) What speed would the proton attain if the field accelerated the proton through a distance e of 1.00 cm?

Figure 22-58 Problem 38.

••48 In Fig. 22-59, an electron (e) is to be released from rest on the central axis of a uniformly charged disk of radius R. The surface charge density on the disk is #4.00 mC/m2. What is the magnitude of Figure 22-59 the electron’s initial acceleration if it is Problem 48. released at a distance (a) R, (b) R/100, and (c) R/1000 from the center of the disk? (d) Why does the acceleration magnitude increase only slightly as the release point is moved closer to the disk?

Module 22-6 A Point Charge in an Electric Field •39 In Millikan’s experiment, an oil drop of radius 1.64 mm and density 0.851 g/cm3 is suspended in chamber C (Fig. 22-16) when a downward electric field of 1.92 $ 105 N/C is applied. Find the charge on the drop, in terms of e.

••49 A 10.0 g block with a charge of #8.00 $ 10%5 C is placed in : an electric field E ! (3000iˆ % 600jˆ ) N/C. What are the (a) magnitude and (b) direction (relative to the positive direction of the x axis) of the electrostatic force on the block? If the block is released from rest at the origin at time t ! 0, what are its (c) x and (d) y coordinates at t ! 3.00 s?

z

Em 0.5Em 0

(a)

z (cm) (b)

zs

•40 An electron with a speed of 5.00 $ 108 cm/s enters an electric field of magnitude 1.00 $ 103 N/C, traveling along a field line in the direction that retards its motion. (a) How far will the electron travel in the field before stopping momentarily, and (b) how much time will have elapsed? (c) If the region containing the electric field is 8.00 mm long (too short for the electron to stop within it), what fraction of the electron’s initial kinetic energy will be lost in that region? •41 SSM A charged cloud system produces an electric field in the air near Earth’s surface. A particle of charge %2.0 $ 10%9 C is acted on by a downward electrostatic force of 3.0 $ 10%6 N when placed in this field. (a) What is the magnitude of the electric field? What are the (b) magnitude and (c) direction of the electrostatic : force Fel on the proton placed in this field? (d) What is the magni: tude of the gravitational force Fg on the proton? (e) What is the ratio Fel /Fg in this case? •42 Humid air breaks down (its molecules become ionized) in an electric field of 3.0 $ 106 N/C. In that field, what is the magnitude of the electrostatic force on (a) an electron and (b) an ion with a single electron missing? •43 SSM An electron is released from rest in a uniform electric field of magnitude 2.00 $ 104 N/C. Calculate the acceleration of the electron. (Ignore gravitation.)

••50 At some instant the velocity components of an electron moving between two charged parallel plates are vx ! 1.5 $ 105 m/s and vy ! 3.0 $ 103 m/s. Suppose the electric field between the : plates is uniform and given by E ! (120 N/C)jˆ. In unit-vector notation, what are (a) the electron’s acceleration in that field and (b) the electron’s velocity when its x coordinate has changed by 2.0 cm? ••51 Assume that a honeybee is a sphere of diameter 1.000 cm with a charge of #45.0 pC uniformly spread over its surface. Assume also that a spherical pollen grain of diameter 40.0 mm is electrically held on the surface of the bee because the bee’s charge induces a charge of %1.00 pC on the near side of the grain and a charge of #1.00 pC on the far side. (a) What is the magnitude of the net electrostatic force on the grain due to the bee? Next, assume that the bee brings the grain to a distance of 1.000 mm from the tip of a flower’s stigma and that the tip is a particle of charge %45.0 pC. (b) What is the magnitude of the net electrostatic force on the grain due to the stigma? (c) Does the grain remain on the bee or does it move to the stigma? ••52 An electron enters a region of uniform electric field with an initial velocity of 40 km/s in the same direction as the electric field, which has magnitude E ! 50 N/C. (a) What is the speed of the electron 1.5 ns after entering this region? (b) How far does the electron travel during the 1.5 ns interval?

657

PROB LE M S

••53 Two large parallel copper Positive Negative plate p plate plates are 5.0 cm apart and have a e uniform electric field between them as depicted in Fig. 22-60. An electron is released from the negative E plate at the same time that a proton is released from the positive plate. Neglect the force of the particles on Figure 22-60 Problem 53. each other and find their distance from the positive plate when they pass each other. (Does it surprise you that you need not know the electric field to solve this problem?) y Detecting ••54 In Fig. 22-61, an electron is E v0 screen shot at an initial speed of θ0 v0 ! 2.00 $ 106 m/s, at angle u0 ! x 40.0" from an x axis. It moves through a uniform electric field : E ! (5.00 N/C)jˆ. A screen for deFigure 22-61 Problem 54. tecting electrons is positioned parallel to the y axis, at distance x ! 3.00 m. In unit-vector notation, what is the velocity of the electron when it hits the screen?

••55 ILW A uniform electric field exists in a region between two oppositely charged plates. An electron is released from rest at the surface of the negatively charged plate and strikes the surface of the opposite plate, 2.0 cm away, in a time 1.5 $ 10%8 s. (a) What is the speed of the electron as it strikes the second plate? (b) What is : the magnitude of the electric field E ? Module 22-7 A Dipole in an Electric Field •56 An electric dipole consists of charges #2e and %2e separated by 0.78 nm. It is in an electric field of strength 3.4 $ 106 N/C. Calculate the magnitude of the torque on the dipole when the dipole moment is (a) parallel to, (b) perpendicular to, and (c) antiparallel to the electric field.

U (10–28 J)

•57 SSM An electric dipole consisting of charges of magnitude 1.50 nC separated by 6.20 mm is in an electric field of strength 1100 N/C. What are (a) the magnitude of the electric dipole moment and (b) the difference between the potential energies for dipole orienta: tions parallel and antiparallel to E ? ••58 A certain electric dipole is Us : placed in a uniform electric field E of magnitude 20 N/C. Figure 22-62 gives the potential energy U of the dipole θ 0 : versus the angle u between E and the : dipole moment p . The vertical axis scale is set by Us ! 100 $ 10%28 J. What –Us : is the magnitude of p ? Figure 22-62 Problem 58. ••59 How much work is required to turn an electric dipole 180" in a uniform electric field of magnitude E ! 46.0 N/C if the dipole moment has a magnitude of p ! 3.02 $ 10%25 C (m and the initial angle is 64"?

! (10–28 N • m)

••60 A certain electric dipole is placed : in a uniform electric field E of magnitude !s 40 N/C. Figure 22-63 gives the magnitude t of the torque on the dipole versus the : angle u between field E and the dipole : moment p . The vertical axis scale is set by θ 0 ts ! 100 $ 10%28 N(m. What is the mag: nitude of p ? Figure 22-63 Problem 60.

••61 Find an expression for the oscillation frequency of an elec: tric dipole of dipole moment p and rotational inertia I for small amplitudes of oscillation about its equilibrium position in a uniform electric field of magnitude E. Additional Problems 62 (a) What is the magnitude of an electron’s acceleration in a uniform electric field of magnitude 1.40 $ 106 N/C? (b) How long would the electron take, starting from rest, to attain one-tenth the speed of light? (c) How far would it travel in that time? 63 A spherical water drop 1.20 mm in diameter is suspended in calm air due to a downward-directed atmospheric electric field of magnitude E ! 462 N/C. (a) What is the magnitude of the gravitational force on the drop? (b) How many excess electrons does it have? 64 Three particles, each with positive charge Q, form an equilateral triangle, with each side of length d.What is the magnitude of the electric field produced by the particles at the midpoint of any side? 65 In Fig. 22-64a, a particle of charge #Q produces an electric field of magnitude Epart at point P, at distance R from the particle. In Fig. 22-64b, that same amount of charge is spread uniformly along a circular arc that has radius R and subtends an angle u. R +Q θ/2 +Q P The charge on the arc produces an electric field θ/2 P R of magnitude Earc at its center of curvature P. For what (a) (b) value of u does Earc ! 0.500Epart? (Hint: You will Figure 22-64 Problem 65. probably resort to a graphical solution.) 66 A proton and an electron form two corners of an equilateral triangle of side length 2.0 $ 10%6 m. What is the magnitude of the net electric field these two particles produce at the third corner? 67 A charge (uniform linear density ! 9.0 nC/m) lies on a string that is stretched along an x axis from x ! 0 to x ! 3.0 m. Determine the magnitude of the electric field at x ! 4.0 m on the x axis. 68 In Fig. 22-65, eight particles form a square in which distance q2 q3 d ! 2.0 cm. The charges are q1 ! #3e, q 1 d d q2 ! #e, q3 ! %5e, q4 ! %2e, q5 ! #3e, q6 ! #e, q7 ! %5e, and q8 ! #e. In unitd d y vector notation, what is the net electric field at the square’s center? 69 Two particles, each with a charge of magnitude 12 nC, are at two of the vertices of an equilateral triangle with edge length 2.0 m. What is the magnitude of the electric field at the third vertex if (a) both charges are positive and (b) one charge is positive and the other is negative?

q8

q4

x

d

d d

q7

d q6

q5

Figure 22-65 Problem 68.

70 The following table gives the charge seen by Millikan at different times on a single drop in his experiment. From the data, calculate the elementary charge e. 6.563 $ 10%19 C 8.204 $ 10%19 C 11.50 $ 10%19 C

13.13 $ 10%19 C 16.48 $ 10%19 C 18.08 $ 10%19 C

19.71 $ 10%19 C 22.89 $ 10%19 C 26.13 $ 10%19 C

658

CHAPTE R 22 E LECTR IC FI E LDS

71 A charge of 20 nC is uniformly distributed along a straight rod of length 4.0 m that is bent into a circular arc with a radius of 2.0 m. What is the magnitude of the electric field at the center of curvature of the arc? 72 An electron is constrained to the central axis of the ring of charge of radius R in Fig. 22-11, with z ! R. Show that the electrostatic force on the electron can cause it to oscillate through the ring center with an angular frequency

v"

eq , A 4p´0 mR3

where q is the ring’s charge and m is the electron’s mass. 73 SSM The electric field in an xy plane produced by a positively charged particle is 7.2(4.0iˆ & 3.0jˆ ) N/C at the point (3.0, 3.0) cm and 100iˆ N/C at the point (2.0, 0) cm. What are the (a) x and (b) y coordinates of the particle? (c) What is the charge of the particle? 74 (a) What total (excess) charge q must the disk in Fig. 22-15 have for the electric field on the surface of the disk at its center to have magnitude 3.0 # 106 N/C, the E value at which air breaks down electrically, producing sparks? Take the disk radius as 2.5 cm. (b) Suppose each surface atom has an effective cross-sectional area of 0.015 nm2. How many atoms are needed to make up the disk surface? (c) The charge calculated in (a) results from some of the surface atoms having one excess electron. What fraction of these atoms must be so charged? 75 In Fig. 22-66, particle 1 (of charge &1.00 mC), particle 2 (of charge &1.00 mC), and particle 3 (of charge Q) form an equilateral triangle of edge length a. For what value of Q (both sign and magnitude) does the net electric field produced by the particles at the center of the triangle vanish?

y

3 a

x

a

1

2 a

Figure 22-66 Problems 75 and 86.

76 In Fig. 22-67, an electric dipole swings from an initial orientation i (ui " 20.0$) to a final orientation f (uf " 20.0$) in a uniform : external electric field E . The electric dipole moment is 1.60 # 10%27 C 'm; the field magnitude is 3.00 # 106 N/C. What is the change in the dipole’s potential energy?

E pf

θf θi pi

77 A particle of charge %q1 is at the origin of an x axis. (a) At what location on the axis should a particle of charge %4q1 be placed so that the Figure 22-67 net electric field is zero at x " 2.0 mm on the Problem 76. axis? (b) If, instead, a particle of charge &4q1 is placed at that location, what is the direction (relative to the positive direction of the x axis) of the net electric field at x " 2.0 mm? 78 Two particles, each of positive charge q, are fixed in place on a y axis, one at y " d and the other at y " %d. (a) Write an expression that gives the magnitude E of the net electric field at points on the x axis given by x " ad. (b) Graph E versus a for the range 0 ( a ( 4. From the graph, determine the values of a that give (c) the maximum value of E and (d) half the maximum value of E. 79 A clock face has negative point charges %q, %2q, %3q, . . . , %12q fixed at the positions of the corresponding numerals. The clock hands do not perturb the net field due to the point charges. At

what time does the hour hand point in the same direction as the electric field vector at the center of the dial? (Hint: Use symmetry.) 80 Calculate the electric dipole moment of an electron and a proton 4.30 nm apart. :

81 An electric field E with an average magnitude of about 150 N/C points downward in the atmosphere near Earth’s surface. We wish to “float” a sulfur sphere weighing 4.4 N in this field by charging the sphere. (a) What charge (both sign and magnitude) must be used? (b) Why is the experiment impractical? 82 A circular rod has a radius of curvature R " 9.00 cm and a uniformly distributed positive charge Q " 6.25 pC and subtends an angle u " 2.40 rad. What is the magnitude of the electric field that Q produces at the center of curvature? 83

SSM

An electric dipole with dipole moment : p " (3.00iˆ & 4.00jˆ )(1.24 # 10%30 C'm)

: is in an electric field E " (4000 N/C)iˆ . (a) What is the potential energy of the electric dipole? (b) What is the torque acting on it? (c) If an external agent turns the dipole until its electric dipole moment is : p " (%4.00iˆ & 3.00jˆ )(1.24 # 10%30 C 'm),

how much work is done by the agent? 84 In Fig. 22-68, a uniform, upward d : E electric field E of magnitude 2.00 # v0 3 10 N/C has been set up between two θ horizontal plates by charging the L lower plate positively and the upper plate negatively. The plates have Figure 22-68 Problem 84. length L " 10.0 cm and separation d " 2.00 cm. An electron is then shot between the plates from the left edge of the lower plate. The initial velocity : v0 of the electron makes an angle u " 45.0$ with the lower plate and has a magnitude of 6.00 # 106 m/s. (a) Will the electron strike one of the plates? (b) If so, which plate and how far horizontally from the left edge will the electron strike? 85 For the data of Problem 70, assume that the charge q on the drop is given by q " ne, where n is an integer and e is the elementary charge. (a) Find n for each given value of q. (b) Do a linear regression fit of the values of q versus the values of n and then use that fit to find e. 86 In Fig. 22-66, particle 1 (of charge &2.00 pC), particle 2 (of charge %2.00 pC), and particle 3 (of charge &5.00 pC) form an equilateral triangle of edge length a " 9.50 cm. (a) Relative to the positive direction of the x axis, determine the direction of : the force F3 on particle 3 due to the other particles by sketching electric field lines of the other particles. (b) Calculate the mag: nitude of F3. 87 In Fig. 22-69, particle 1 of charge q1 " 1.00 pC and particle 2 of charge q2 " %2.00 pC are fixed at a distance d " 5.00 cm apart. In unit-vector notation, what is the net electric field at points (a) A, (b) B, and (c) C? (d) Sketch the electric field lines.

A

d

+ 1

d 2

B

d 2

– 2

Figure 22-69 Problem 87.

d

C

x

C

H

A

P

T

E

R

2

3

Gauss’ Law 23-1 ELECTRIC FLUX Learning Objectives After reading this module, you should be able to . . .

23.01 Identify that Gauss’ law relates the electric field at points on a closed surface (real or imaginary, said to be a Gaussian surface) to the net charge enclosed by that surface. 23.02 Identify that the amount of electric field piercing a surface (not skimming along the surface) is the electric flux ) through the surface. 23.03 Identify that an area vector for a flat surface is a vector that is perpendicular to the surface and that has a magnitude equal to the area of the surface. 23.04 Identify that any surface can be divided into area elements (patch elements) that are each small enough and : flat enough for an area vector dA to be assigned to it, with the vector perpendicular to the element and having a magnitude equal to the area of the element.

23.05 Calculate the flux ) through a surface by integrating the : dot product of the electric field vector E and the area vec: tor dA (for patch elements) over the surface, in magnitudeangle notation and unit-vector notation. 23.06 For a closed surface, explain the algebraic signs associated with inward flux and outward flux. 23.07 Calculate the net flux ) through a closed surface, algebraic sign included, by integrating the dot product of the : : electric field vector E and the area vector dA (for patch elements) over the full surface. 23.08 Determine whether a closed surface can be broken up into parts (such as the sides of a cube) to simplify the integration that yields the net flux through the surface.

Key Ideas

"

● The electric flux ) through a surface is the amount of electric :

● The area vector dA for an area element (patch element) on

a surface is a vector that is perpendicular to the element and has a magnitude equal to the area dA of the element. ● The electric flux d) through a patch element with area : vector dA is given by a dot product: :

:

(total flux),

where the integration is carried out over the surface. ● The net flux through a closed surface (which is used in

Gauss’ law) is given by )"

:

d) " E " dA. ● The total flux through a surface is given by

:

) " E " dA

field that pierces the surface.

,

:

:

E " dA

(net flux),

where the integration is carried out over the entire surface.

What Is Physics? In the preceding chapter we found the electric field at points near extended charged objects, such as rods. Our technique was labor-intensive: We split the : charge distribution up into charge elements dq, found the field dE due to an element, and resolved the vector into components. Then we determined whether the components from all the elements would end up canceling or adding. Finally we summed the adding components by integrating over all the elements, with several changes in notation along the way. One of the primary goals of physics is to find simple ways of solving such labor-intensive problems. One of the main tools in reaching this goal is the use of symmetry. In this chapter we discuss a beautiful relationship between charge and 659

660

CHAPTE R 23 GAUSS’ L AW

Gaussian surface

E

Field line

Figure 23-1 Electric field vectors and field lines pierce an imaginary, spherical Gaussian surface that encloses a particle with charge &Q.

electric field that allows us, in certain symmetric situations, to find the electric field of an extended charged object with a few lines of algebra. The relationship is called Gauss’ law, which was developed by German mathematician and physicist Carl Friedrich Gauss (1777–1855). Let’s first take a quick look at some simple examples that give the spirit of Gauss’ law. Figure 23-1 shows a particle with charge &Q that is surrounded by an imaginary concentric sphere.At points on the sphere (said to be a Gaussian surface), the electric field vectors have a moderate magnitude (given by E " kQ/r2) and point radially away from the particle (because it is positively charged). The electric field lines are also outward and have a moderate density (which, recall, is related to the field magnitude).We say that the field vectors and the field lines pierce the surface. Figure 23-2 is similar except that the enclosed particle has charge &2Q. Because the enclosed charge is now twice as much, the magnitude of the field vectors piercing outward through the (same) Gaussian surface is twice as much as in Fig. 23-1, and the density of the field lines is also twice as much. That sentence, in a nutshell, is Gauss’ law. Guass’ law relates the electric field at points on a (closed) Gaussian surface to the net charge enclosed by that surface.

Figure 23-2 Now the enclosed particle has charge &2Q.

Let’s check this with a third example with a particle that is also enclosed by the same spherical Gaussian surface (a Gaussian sphere, if you like, or even the catchy G-sphere) as shown in Fig. 23-3. What is the amount and sign of the enclosed charge? Well, from the inward piercing we see immediately that the charge must be negative. From the fact that the density of field lines is half that of Fig. 23-1, we also see that the charge must be 0.5Q. (Using Gauss’ law is like being able to tell what is inside a gift box by looking at the wrapping paper on the box.) The problems in this chapter are of two types. Sometimes we know the charge and we use Gauss’ law to find the field at some point. Sometimes we know the field on a Gaussian surface and we use Gauss’ law to find the charge enclosed by the surface. However, we cannot do all this by simply comparing the density of field lines in a drawing as we just did. We need a quantitative way of determining how much electric field pierces a surface. That measure is called the electric flux.

Electric Flux Figure 23-3 Can you tell what the enclosed charge is now?

Flat Surface, Uniform Field. We begin with a flat surface with area A in a uni: : form electric field E . Figure 23-4a shows one of the electric field vectors E piercing a small square patch with area *A (where * indicates “small”). Actually, only the x component (with magnitude Ex " E cos u in Fig. 23-4b) pierces the patch. The y component merely skims along the surface (no piercing in that) and does not come into play in Gauss’ law. The amount of electric field piercing the patch is defined to be the electric flux %& through it: *) " (E cos u) *A. y

E

Figure 23-4 (a) An electric field vector pierces a small square patch on a flat surface. (b) Only the x component actually pierces the patch; the y component skims across it. (c) The area vector of the patch is perpendicular to the patch, with a magnitude equal to the patch’s area.

y

y

u

Ey

x

Ex

!A

x

x !A

A

(a )

(b )

(c )

!A

661

23-1 E LECTR IC FLUX

There is another way to write the right side of this statement so that we have only : : the piercing component of E .We define an area vector *A that is perpendicular to the patch and that has a magnitude equal to the area *A of the patch (Fig. 23-4c). Then we can write :

:

*) " E " *A, :

and the dot product automatically gives us the component of E that is parallel to : *A and thus piercing the patch. To find the total flux ) through the surface in Fig. 23-4, we sum the flux through every patch on the surface: ) " ' E " *A. :

:

(23-1)

However, because we do not want to sum hundreds (or more) flux values, we transform the summation into an integral by shrinking the patches from small squares with area *A to patch elements (or area elements) with area dA.The total flux is then )"

"

:

:

E " dA

(total flux).

(23-2)

Now we can find the total flux by integrating the dot product over the full surface. Dot Product. We can evaluate the dot product inside the integral by writing the : : two vectors in unit-vector notation. For example, in Fig. 23-4, dA " dAˆi and E might be, say, (4ˆi " 4ˆj ) N/C. Instead, we can evaluate the dot product in magnitude-angle notation: E cos u dA. When the electric field is uniform and the surface is flat, the product E cos u is a constant and comes outside the integral. The remaining "dA is just an instruction to sum the areas of all the patch elements to get the total area, but we already know that the total area is A. So the total flux in this simple situation is ) " (E cos u)A

(uniform field, flat surface).

(23-3)

Closed Surface. To use Gauss’ law to relate flux and charge, we need a closed surface. Let’s use the closed surface in Fig. 23-5 that sits in a nonuniform electric field. (Don’t worry.The homework problems involve less complex surfaces.) As before, we first consider the flux through small square patches. However, now we are interested in not only the piercing components of the field but also on whether the piercing is inward or outward (just as we did with Figs. 23-1 through 23-3). Directions. To keep track of the piercing direction, we again use an area vec: tor *A that is perpendicular to a patch, but now we always draw it pointing outward from the surface (away from the interior). Then if a field vector pierces outward, it and the area vector are in the same direction, the angle is u " 0, and cos u " 1. : : Thus, the dot product E ' *A is positive and so is the flux. Conversely, if a field vector pierces inward, the angle is u " 180$ and cos u " %1. Thus, the dot product is negative and so is the flux. If a field vector skims the surface (no piercing), the dot product is zero (because cos 90$ " 0) and so is the flux. Figure 23-5 gives some general examples and here is a summary: An inward piercing field is negative flux. An outward piercing field is positive flux. A skimming field is zero flux.

Net Flux. In principle, to find the net flux through the surface in Fig. 23-5, we find the flux at every patch and then sum the results (with the algebraic signs included). However, we are not about to do that much work. Instead, we shrink the : squares to patch elements with area vectors dA and then integrate: )"

,

:

:

E " dA

(net flux).

(23-4)

Gaussian surface

1

3 2

∆A θ

∆A

E

θ

1 Φ0 E 2 ∆A Φ=0

Pierce outward: positive flux

Skim: zero flux Figure 23-5 A Gaussian surface of arbitrary shape immersed in an electric field. The surface is divided into small squares of area : !A. The electric field vectors E and the : area vectors !A for three representative squares, marked 1, 2, and 3, are shown.

662

CHAPTE R 23 GAUSS’ L AW

The loop on the integral sign indicates that we must integrate over the entire closed surface, to get the net flux through the surface (as in Fig. 23-5, flux might enter on one side and leave on another side). Keep in mind that we want to determine the net flux through a surface because that is what Gauss’ law relates to the charge enclosed by the surface. (The law is coming up next.) Note that flux is a scalar (yes, we talk about field vectors but flux is the amount of piercing field, not a vector itself). The SI unit of flux is the newton–square-meter per coulomb (N#m2/C). y

Checkpoint 1 The figure here shows a Gaussian cube of face area A : immersed in a uniform electric field E that has the positive direction of the z axis. In terms of E and A, what is the flux through (a) the front face (which is in the xy plane), (b) the rear face, (c) the top face, and (d) the whole cube?

A x E

z

Sample Problem 23.01 Flux through a closed cylinder, uniform field Figure 23-6 shows a Gaussian surface in the form of a closed cylinder (a Gaussian cylinder or G-cylinder) of : radius R. It lies in a uniform electric field E with the cylinder’s central axis (along the length of the cylinder) parallel to the field. What is the net flux & of the electric field through the cylinder?

"

:

:

E " dA $

a

"

Calculations: We break the integral of Eq. 23-4 into three terms: integrals over the left cylinder cap a, the curved cylindrical surface b, and the right cap c:

$

a

:

:

E " dA :

:

:

E " dA $

c

We can find the net flux & with Eq. 23-4 by integrating the : : dot product E " dA over the cylinder’s surface. However, we cannot write out functions so that we can do that with one integral. Instead, we need to be a bit clever: We break up the surface into sections with which we can actually evaluate an integral.

, "

E(cos 180%) dA $ 'E

:

E " dA "

" b

:

:

E " dA "

"

:

:

E " dA.

"

E(cos 0) dA $ EA.

"

:

:

E " dA $

b

"

E(cos 90%) dA $ 0.

Substituting these results into Eq. 23-5 leads us to & $ 'EA " 0 " EA $ 0.

(Answer)

The net flux is zero because the field lines that represent the electric field all pass entirely through the Gaussian surface, from the left to the right.

dA

θ

c

:

dA $ 'EA,

Finally, for the cylindrical surface, where the angle u is 90% at all points,

(23-5)

Pick a patch element on the left cap. Its area vector dA must be perpendicular to the patch and pointing away from the interior of the cylinder. In Fig. 23-6, that means the angle between it and the field piercing the patch is 180%. Also, note that the electric field through the end cap is uniform and thus E can be pulled out of the integration. So, we can write the flux through the left cap as

"

where " dA gives the cap’s area A ($ pR2). Similarly, for the right cap, where u $ 0 for all points,

KEY IDEAS

&$

"

dA

θ a

Gaussian surface

E

b E

E c

dA

Figure 23-6 A cylindrical Gaussian surface, closed by end caps, is immersed in a uniform electric field. The cylinder axis is parallel to the field direction.

Additional examples, video, and practice available at WileyPLUS

663

23-1 E LECTR IC FLUX

Sample Problem 23.02 Flux through a closed cube, nonuniform field : A nonuniform electric field given by E $ 3.0xiˆ " 4.0jˆ pierces the Gaussian cube shown in Fig. 23-7a. (E is in newtons per coulomb and x is in meters.) What is the electric flux through the right face, the left face, and the top face? (We consider the other faces in another sample problem.)

KEY IDEA We can find the flux & through the surface by integrating : : the scalar product E " dA over each face. :

Right face: An area vector A is always perpendicular to its surface and always points away from the interior of a : Gaussian surface. Thus, the vector dA for any patch element (small section) on the right face of the cube must point in the positive direction of the x axis. An example of such an element is shown in Figs. 23-7b and c, but we would have an identical vector for any other choice of a patch element on that face. The most convenient way to express the vector is in unit-vector notation, : dA $ dAiˆ . From Eq. 23-4, the flux &r through the right face is then

y Gaussian

surface

z

(a )

$

:

(3.0xiˆ " 4.0jˆ ) " (dAiˆ )

[(3.0x)(dA)iˆ " iˆ " (4.0)(dA)jˆ " iˆ ] (3.0x dA " 0) $ 3.0

&r $ 3.0

"

"

x dA.

(3.0) dA $ 9.0

"

&r $ (9.0 N/C)(4.0 m2) $ 36 N #m2/C.

dA

dA

(Answer)

The element area vector (for a patch element) is perpendicular to the surface and outward.

dA

x

x dA z (b )

Figure 23-7 (a) A Gaussian cube with one edge on the x axis lies within a nonuniform electric field that depends on the value of x. (b) Each patch element has an outward vector that is perpendicular to the area. (c) Right face: the x component of the field pierces the area and produces positive (outward) flux. The y component does not pierce the area and thus does not produce any flux. (Figure continues on following page)

dA.

The integral " dA merely gives us the area A $ 4.0 m2 of the right face, so

dA

The x component depends on the value of x.

"

:

E " dA $

We are about to integrate over the right face, but we note that x has the same value everywhere on that face — namely, x $ 3.0 m. This means we can substitute that constant value for x. This can be a confusing argument. Although x is certainly a variable as we move left to right across the figure, because the right face is perpendicular to the x axis, every point on the face has the same x coordinate. (The y and z coordinates do not matter in our integral.) Thus, we have

y

Ey E

x = 1.0 m x = 3.0 m

$

" " "

Left face: We repeat this procedure for the left face. However,

The y component is a constant.

Ex

&r $

y

Ey

The y component of the field skims the surface and gives no flux. The dot product is just zero. Ex dA

z (c )

x

The x component of the field pierces the surface and gives outward flux. The dot product is positive.

A

664

CHAPTE R 23 GAUSS’ L AW

y

Ey

Ex

dA

The y component of the field skims the surface and gives no flux. The dot product is just zero.

The y component of the field pierces the surface and gives outward flux. The dot product is positive.

dA

y

The x component of the field skims the surface and gives no flux. The dot product is just zero.

Ey Ex

x

x

The x component of the field pierces the surface and gives inward flux. The dot product is negative.

z (d )

(e )

z

Figure 23-7 (Continued from previous page) (d) Left face: the x component of the field produces negative (inward) flux. (e) Top face: the y component of the field produces positive (outward) flux.

:

two factors change. (1) The element area vector dA points in : the negative direction of the x axis, and thus dA $ 'dAiˆ (Fig. 23-7d). (2) On the left face, x $ 1.0 m. With these changes, we find that the flux &l through the left face is &l $ '12 N #m2/C.

&t $ $

(Answer)

:

Top face: Now dA points in the positive direction of the y : axis, and thus dA $ dAjˆ (Fig. 23-7e). The flux &t is

$

" " "

(3.0xiˆ " 4.0jˆ ) " (dAjˆ ) [(3.0x)(dA)iˆ " jˆ " (4.0)(dA)jˆ " jˆ ] (0 " 4.0 dA) $ 4.0

"

dA

$ 16 N # m2/C.

(Answer)

Additional examples, video, and practice available at WileyPLUS

23-2 GAUSS’ LAW

Learning Objectives

After reading this module, you should be able to . . .

23.09 Apply Gauss’ law to relate the net flux & through a closed surface to the net enclosed charge qenc. 23.10 Identify how the algebraic sign of the net enclosed charge corresponds to the direction (inward or outward) of the net flux through a Gaussian surface. 23.11 Identify that charge outside a Gaussian surface makes

no contribution to the net flux through the closed surface. 23.12 Derive the expression for the magnitude of the electric field of a charged particle by using Gauss’ law. 23.13 Identify that for a charged particle or uniformly charged sphere, Gauss’ law is applied with a Gaussian surface that is a concentric sphere.

Key Ideas ● Gauss’ law relates the net flux & penetrating a closed sur-

face to the net charge qenc enclosed by the surface: ´0& $ qenc

● Gauss’ law can also be written in terms of the electric field

piercing the enclosing Gaussian surface: ´0

(Gauss’ law).

,

:

:

E " dA $ qenc

(Gauss’ law).

Gauss’ Law Gauss’ law relates the net flux & of an electric field through a closed surface (a Gaussian surface) to the net charge qenc that is enclosed by that surface. It tells us that ´0& $ qenc

(Gauss’ law).

(23-6)

665

23-2 GAUSS’ L AW

By substituting Eq. 23-4, the definition of flux, we can also write Gauss’ law as ´0

,

:

:

E " dA $ qenc

(23-7)

(Gauss’ law).

S1

Equations 23-6 and 23-7 hold only when the net charge is located in a vacuum or (what is the same for most practical purposes) in air. In Chapter 25, we modify Gauss’ law to include situations in which a material such as mica, oil, or glass is present. In Eqs. 23-6 and 23-7, the net charge qenc is the algebraic sum of all the enclosed positive and negative charges, and it can be positive, negative, or zero. We include the sign, rather than just use the magnitude of the enclosed charge, because the sign tells us something about the net flux through the Gaussian surface: If qenc is positive, the net flux is outward; if qenc is negative, the net flux is inward. Charge outside the surface, no matter how large or how close it may be, is not included in the term qenc in Gauss’ law. The exact form and location of the charges inside the Gaussian surface are also of no concern; the only things that matter on the right side of Eqs. 23-6 and 23-7 are the magnitude and sign of the net enclosed : charge.The quantity E on the left side of Eq. 23-7, however, is the electric field resulting from all charges, both those inside and those outside the Gaussian surface. This statement may seem to be inconsistent, but keep this in mind: The electric field due to a charge outside the Gaussian surface contributes zero net flux through the surface, because as many field lines due to that charge enter the surface as leave it. Let us apply these ideas to Fig. 23-8, which shows two particles, with charges equal in magnitude but opposite in sign, and the field lines describing the electric fields the particles set up in the surrounding space. Four Gaussian surfaces are also shown, in cross section. Let us consider each in turn. Surface S1. The electric field is outward for all points on this surface. Thus, the flux of the electric field through this surface is positive, and so is the net charge within the surface, as Gauss’ law requires. (That is, in Eq. 23-6, if & is positive, qenc must be also.) Surface S2. The electric field is inward for all points on this surface. Thus, the flux of the electric field through this surface is negative and so is the enclosed charge, as Gauss’ law requires. Surface S3. This surface encloses no charge, and thus qenc $ 0. Gauss’ law (Eq. 23-6) requires that the net flux of the electric field through this surface be zero. That is reasonable because all the field lines pass entirely through the surface, entering it at the top and leaving at the bottom. Surface S4. This surface encloses no net charge, because the enclosed positive and negative charges have equal magnitudes. Gauss’ law requires that the net flux of the electric field through this surface be zero. That is reasonable because there are as many field lines leaving surface S4 as entering it. What would happen if we were to bring an enormous charge Q up close to surface S4 in Fig. 23-8? The pattern of the field lines would certainly change, but the net flux for each of the four Gaussian surfaces would not change. Thus, the value of Q would not enter Gauss’ law in any way, because Q lies outside all four of the Gaussian surfaces that we are considering. 5

Checkpoint 2 The figure shows three situations in which a Gaussian cube sits in an electric field.The arrows and the values indicate the directions of the field lines and the magnitudes (in N ?m2/C) of the flux through the six sides of each cube. (The lighter arrows are for the hidden faces.) In which situation does the cube enclose (a) a positive net charge, (b) a negative net charge, and (c) zero net charge?

+ S4

S3

S2



Figure 23-8 Two charges, equal in magnitude but opposite in sign, and the field lines that represent their net electric field. Four Gaussian surfaces are shown in cross section. Surface S1 encloses the positive charge. Surface S2 encloses the negative charge. Surface S3 encloses no charge. Surface S4 encloses both charges and thus no net charge.

10

3

7

6

2

3

3

8

5 4

2 (1)

7

(2)

5

7

4 6

(3)

5

666

CHAPTE R 23 GAUSS’ L AW

Gauss’ Law and Coulomb’s Law Gaussian surface

r q

E

+

Figure 23-9 A spherical Gaussian surface centered on a particle with charge q.

One of the situations in which we can apply Gauss’ law is in finding the electric field of a charged particle. That field has spherical symmetry (the field depends on the distance r from the particle but not the direction). So, to make use of that symmetry, we enclose the particle in a Gaussian sphere that is centered on the particle, as shown in Fig. 23-9 for a particle with positive charge q. Then the electric field has the same magnitude E at any point on the sphere (all points are at the same distance r). That feature will simplify the integration. The drill here is the same as previously. Pick a patch element on the surface and : draw its area vector dA perpendicular to the patch and directed outward. From the : symmetry of the situation, we know that the electric field E at the patch is also radi: ally outward and thus at angle u $ 0 with dA. So, we rewrite Gauss’ law as ´0

,

:

:

E " dA $ ´0

,

E dA $ qenc.

(23-8)

Here qenc $ q. Because the field magnitude E is the same at every patch element, E can be pulled outside the integral: ´0E

,

(23-9)

dA $ q.

The remaining integral is just an instruction to sum all the areas of the patch elements on the sphere, but we already know that the total area is 4pr 2. Substituting this, we have ´0E(4pr 2) $ q or

E$

q 1 . 4p´0 r2

(23-10)

This is exactly Eq. 22-3, which we found using Coulomb’s law.

Checkpoint 3 There is a certain net flux &i through a Gaussian sphere of radius r enclosing an isolated charged particle. Suppose the enclosing Gaussian surface is changed to (a) a larger Gaussian sphere, (b) a Gaussian cube with edge length equal to r, and (c) a Gaussian cube with edge length equal to 2r. In each case, is the net flux through the new Gaussian surface greater than, less than, or equal to &i?

Sample Problem 23.03 Using Gauss’ law to find the electric field Figure 23-10a shows, in cross section, a plastic, spherical shell with uniform charge Q $ '16e and radius R $ 10 cm. A particle with charge q $ "5e is at the center.What is the electric field (magnitude and direction) at (a) point P1 at radial distance r1 $ 6.00 cm and (b) point P2 at radial distance r2 $ 12.0 cm? KEY IDEAS (1) Because the situation in Fig. 23-10a has spherical symmetry, we can apply Gauss’ law (Eq. 23-7) to find the electric field at a point if we use a Gaussian surface in the form of a sphere concentric with the particle and shell. (2) To find the electric field at a point, we put that point on a Gaussian surface (so that the : : E we want is the E in the dot product inside the integral in Gauss’ law). (3) Gauss’ law relates the net electric flux through a closed surface to the net enclosed charge. Any external charge is not included.

Calculations: To find the field at point P1, we construct a Gaussian sphere with P1 on its surface and thus with a radius of r1. Because the charge enclosed by the Gaussian sphere is positive, the electric flux through the surface must be positive : and thus outward. So, the electric field E pierces the surface outward and, because of the spherical symmetry, must be radially outward, as drawn in Fig. 23-10b. That figure does not include the plastic shell because the shell is not enclosed by the Gaussian sphere. Consider a patch element on the sphere at P1. Its area vec: tor dA is radially outward (it must always be outward from a : : Gaussian surface). Thus the angle u between E and dA is zero. We can now rewrite the left side of Eq. 23-7 (Gauss’ law) as ´0

,

:

:

E " dA $ ´0

,

E cos 0 dA $ ´0

,

E dA $ ´0 E

,

dA,

667

23-2 GAUSS’ L AW

r1 Q

E

q P1

q

The only charge enclosed by the Gaussian surface through P1 is that of the particle. Solving for E and substituting qenc $ 5e and r $ r1 $ 6.00 ( 10'2 m, we find that the magnitude of the electric field at P1 is qenc E$ 4p´0r2 5(1.60 ( 10 '19 C) $ 4p(8.85 ( 10 '12 C2/N#m2)(0.0600 m)2

dA

P2 (b )

r1 r2

(a )

$ 2.00 ( 10'6 N/C.

r2

Q q

E

dA

(c )

Figure 23-10 (a) A charged plastic spherical shell encloses a charged particle. (b) To find the electric field at P1, arrange for the point to be on a Gaussian sphere. The electric field pierces outward. The area vector for the patch element is outward. (c) P2 : : is on a Gaussian sphere, E is inward, and dA is still outward.

To find the electric field at P2, we follow the same procedure by constructing a Gaussian sphere with P2 on its surface.This time, however, the net charge enclosed by the sphere is qenc $ q " Q $ 5e " ('16e) $ '11e. Because the net charge is negative, the electric field vectors on the sphere’s : surface pierce inward (Fig. 23-10c), the angle u between E : and dA is 180%, and the dot product is E (cos 180%) dA $ 'E dA. Now solving Gauss’ law for E and substituting r $ r2 $ 12.00 ( 10'2 m and the new qenc, we find 'qenc E$ 4p´0r2 $

where in the last step we pull the field magnitude E out of the integral because it is the same at all points on the Gaussian sphere and thus is a constant. The remaining integral is simply an instruction for us to sum the areas of all the patch elements on the sphere, but we already know that the surface area of a sphere is 4pr2. Substituting these results, Eq. 23-7 for Gauss’ law gives us ´0E4pr2 $ qenc.

(Answer)

' ['11(1.60 ( 10 '19 C)] 4p(8.85 ( 10 '12 C2/N#m2)(0.120 m)2

$ 1.10 ( 10'6 N/C.

(Answer)

Note how different the calculations would have been if we had put P1 or P2 on the surface of a Gaussian cube instead of mimicking the spherical symmetry with a Gaussian sphere. Then angle u and magnitude E would have varied considerably over the surface of the cube and evaluation of the integral in Gauss’ law would have been difficult.

Sample Problem 23.04 Using Gauss’ law to find the enclosed charge What is the net charge enclosed by the Gaussian cube of Sample Problem 23.02? KEY IDEA The net charge enclosed by a (real or mathematical) closed surface is related to the total electric flux through the surface by Gauss’ law as given by Eq. 23-6 (´0& $ qenc). Flux: To use Eq. 23-6, we need to know the flux through all six faces of the cube. We already know the flux through the right face (&r $ 36 N #m2/C), the left face (&l $ '12 N ?m2/C), and the top face (&t $ 16 N ?m2/C). For the bottom face, our calculation is just like that for : the top face except that the element area vector dA is now directed downward along the y axis (recall, it must be outward from the Gaussian enclosure). Thus, we have

: dA $ 'dAjˆ , and we find &b $ '16 N ?m2/C. : ˆ , and for the back face, For the front face we have dA $ dAk : ˆ dA $ 'dAk . When we take the dot product of the given elec: tric field E $ 3.0xiˆ " 4.0 ˆj with either of these expressions for : dA, we get 0 and thus there is no flux through those faces. We can now find the total flux through the six sides of the cube:

& $ (36 ' 12 " 16 ' 16 " 0 " 0) N#m2/C $ 24 N#m2/C. Enclosed charge: Next, we use Gauss’ law to find the charge qenc enclosed by the cube: qenc $ )0& $ (8.85 ( 10'12 C2/N#m2)(24 N#m2/C) $ 2.1 ( 10'10 C. (Answer) Thus, the cube encloses a net positive charge.

Additional examples, video, and practice available at WileyPLUS

668

CHAPTE R 23 GAUSS’ L AW

23-3 A CHARGED ISOLATED CONDUCTOR Learning Objectives After reading this module, you should be able to . . .

23.14 Apply the relationship between surface charge density s and the area over which the charge is uniformly spread. 23.15 Identify that if excess charge (positive or negative) is placed on an isolated conductor, that charge moves to the surface and none is in the interior. 23.16 Identify the value of the electric field inside an isolated conductor. 23.17 For a conductor with a cavity that contains a charged

object, determine the charge on the cavity wall and on the external surface. 23.18 Explain how Gauss’ law is used to find the electric field magnitude E near an isolated conducting surface with a uniform surface charge density s. 23.19 For a uniformly charged conducting surface, apply the relationship between the charge density s and the electric field magnitude E at points near the conductor, and identify the direction of the field vectors.

Key Ideas ● An excess charge on an isolated conductor is located

entirely on the outer surface of the conductor. ● The internal electric field of a charged, isolated conductor is zero, and the external field (at nearby points) is perpendicu-

lar to the surface and has a magnitude that depends on the surface charge density s: s E$ . ´0

A Charged Isolated Conductor Gauss’ law permits us to prove an important theorem about conductors: If an excess charge is placed on an isolated conductor, that amount of charge will move entirely to the surface of the conductor. None of the excess charge will be found within the body of the conductor.

Copper surface Gaussian surface (a )

Gaussian surface Copper surface (b )

Figure 23-11 (a) A lump of copper with a charge q hangs from an insulating thread. A Gaussian surface is placed within the metal, just inside the actual surface. (b) The lump of copper now has a cavity within it. A Gaussian surface lies within the metal, close to the cavity surface.

This might seem reasonable, considering that charges with the same sign repel one another. You might imagine that, by moving to the surface, the added charges are getting as far away from one another as they can. We turn to Gauss’ law for verification of this speculation. Figure 23-11a shows, in cross section, an isolated lump of copper hanging from an insulating thread and having an excess charge q. We place a Gaussian surface just inside the actual surface of the conductor. The electric field inside this conductor must be zero. If this were not so, the field would exert forces on the conduction (free) electrons, which are always present in a conductor, and thus current would always exist within a conductor. (That is, charge would flow from place to place within the conductor.) Of course, there is no such perpetual current in an isolated conductor, and so the internal electric field is zero. (An internal electric field does appear as a conductor is being charged. However, the added charge quickly distributes itself in such a way that the net internal electric field — the vector sum of the electric fields due to all the charges, both inside and outside — is zero. The movement of charge then ceases, because the net force on each charge is zero; the charges are then in electrostatic equilibrium.) : If E is zero everywhere inside our copper conductor, it must be zero for all points on the Gaussian surface because that surface, though close to the surface of the conductor, is definitely inside the conductor. This means that the flux through the Gaussian surface must be zero. Gauss’ law then tells us that the net charge inside the Gaussian surface must also be zero. Then because the excess charge is not inside the Gaussian surface, it must be outside that surface, which means it must lie on the actual surface of the conductor.

669

23-3 A CHARG E D ISOL ATE D CON DUCTOR

An Isolated Conductor with a Cavity Figure 23-11b shows the same hanging conductor, but now with a cavity that is totally within the conductor. It is perhaps reasonable to suppose that when we scoop out the electrically neutral material to form the cavity, we do not change the distribution of charge or the pattern of the electric field that exists in Fig. 23-11a. Again, we must turn to Gauss’ law for a quantitative proof. We draw a Gaussian surface surrounding the cavity, close to its surface but in: side the conducting body. Because E $ 0 inside the conductor, there can be no flux through this new Gaussian surface. Therefore, from Gauss’ law, that surface can enclose no net charge. We conclude that there is no net charge on the cavity walls; all the excess charge remains on the outer surface of the conductor, as in Fig. 23-11a.

The Conductor Removed Suppose that, by some magic, the excess charges could be “frozen” into position on the conductor’s surface, perhaps by embedding them in a thin plastic coating, and suppose that then the conductor could be removed completely. This is equivalent to enlarging the cavity of Fig. 23-11b until it consumes the entire conductor, leaving only the charges. The electric field would not change at all; it would remain zero inside the thin shell of charge and would remain unchanged for all external points. This shows us that the electric field is set up by the charges and not by the conductor. The conductor simply provides an initial pathway for the charges to take up their positions.

The External Electric Field You have seen that the excess charge on an isolated conductor moves entirely to the conductor’s surface. However, unless the conductor is spherical, the charge does not distribute itself uniformly. Put another way, the surface charge density s (charge per unit area) varies over the surface of any nonspherical conductor. Generally, this variation makes the determination of the electric field set up by the surface charges very difficult. However, the electric field just outside the surface of a conductor is easy to determine using Gauss’ law. To do this, we consider a section of the surface that is small enough to permit us to neglect any curvature and thus to take the section to be flat. We then imagine a tiny cylindrical Gaussian surface to be partially embedded in the section as shown in Fig. 23-12: One end cap is fully inside the conductor, the other is fully outside, and the cylinder is perpendicular to the conductor’s surface. : The electric field E at and just outside the conductor’s surface must also be perpendicular to that surface. If it were not, then it would have a component along the conductor’s surface that would exert forces on the surface charges, causing them to move. However, such motion would violate our implicit as: sumption that we are dealing with electrostatic equilibrium. Therefore, E is perpendicular to the conductor’s surface. We now sum the flux through the Gaussian surface. There is no flux through the internal end cap, because the electric field within the conductor is zero. There is no flux through the curved surface of the cylinder, because internally (in the conductor) there is no electric field and externally the electric field is parallel to the curved portion of the Gaussian surface. The only flux through the Gaussian : surface is that through the external end cap, where E is perpendicular to the plane of the cap. We assume that the cap area A is small enough that the field magnitude E is constant over the cap. Then the flux through the cap is EA, and that is the net flux & through the Gaussian surface. The charge qenc enclosed by the Gaussian surface lies on the conductor’s surface in an area A. (Think of the cylinder as a cookie cutter.) If s is the charge per unit area, then qenc is equal to sA. When we substitute sA for qenc and EA for &,

+ + + + +

E=0

+ + + + +

+ + + + +

+ + + + +

+ + + + +

+ + + + +

+ + + + +

A E

There is flux only (a ) through the external end face.

+ + + + + + + + +

A E

(b )

Figure 23-12 (a) Perspective view and (b) side view of a tiny portion of a large, isolated conductor with excess positive charge on its surface. A (closed) cylindrical Gaussian surface, embedded perpendicularly in the conductor, encloses some of the charge. Electric field lines pierce the external end cap of the cylinder, but not the internal end cap. The external end cap has area A and : area vector A.

670

CHAPTE R 23 GAUSS’ L AW

Gauss’ law (Eq. 23-6) becomes ´0EA $ sA, from which we find E$

s ´0

(conducting surface).

(23-11)

Thus, the magnitude of the electric field just outside a conductor is proportional to the surface charge density on the conductor. The sign of the charge gives us the direction of the field. If the charge on the conductor is positive, the electric field is directed away from the conductor as in Fig. 23-12. It is directed toward the conductor if the charge is negative. The field lines in Fig. 23-12 must terminate on negative charges somewhere in the environment. If we bring those charges near the conductor, the charge density at any given location on the conductor’s surface changes, and so does the magnitude of the electric field. However, the relation between s and E is still given by Eq. 23-11.

Sample Problem 23.05 Spherical metal shell, electric field and enclosed charge Figure 23-13a shows a cross section of a spherical metal shell of inner radius R. A particle with a charge of '5.0 mC is located at a distance R/2 from the center of the shell. If the shell is electrically neutral, what are the (induced) charges on its inner and outer surfaces? Are those charges uniformly distributed? What is the field pattern inside and outside the shell?

Gaussian surface

R R/2

KEY IDEAS

_

_ + + _ + + _+ + _ +

+

_ +

_ +

_ _ +

_+

_

+_ + _

_

Figure 23-13b shows a cross section of a spherical Gaussian surface within the metal, just outside the inner wall of the shell. The electric field must be zero inside the metal (and thus on the Gaussian surface inside the metal). This means that the electric flux through the Gaussian surface must also be zero. Gauss’ law then tells us that the net charge enclosed by the Gaussian surface must be zero.

Figure 23-13 (a) A negatively charged particle is located within a spherical metal shell that is electrically neutral. (b) As a result, positive charge is nonuniformly distributed on the inner wall of the shell, and an equal amount of negative charge is uniformly distributed on the outer wall.

Reasoning: With a particle of charge '5.0 mC within the shell, a charge of "5.0 mC must lie on the inner wall of the shell in order that the net enclosed charge be zero. If the particle were centered, this positive charge would be uniformly distributed along the inner wall. However, since the particle is off-center, the distribution of positive charge is skewed, as suggested by Fig. 23-13b, because the positive charge tends to collect on the section of the inner wall nearest the (negative) particle. Because the shell is electrically neutral, its inner wall can have a charge of "5.0 mC only if electrons, with a total charge of '5.0 mC, leave the inner wall and move to the outer wall. There they spread out uniformly, as is also suggested by Fig. 23-13b. This distribution of negative charge is

uniform because the shell is spherical and because the skewed distribution of positive charge on the inner wall cannot produce an electric field in the shell to affect the distribution of charge on the outer wall. Furthermore, these negative charges repel one another. The field lines inside and outside the shell are shown approximately in Fig. 23-13b. All the field lines intersect the shell and the particle perpendicularly. Inside the shell the pattern of field lines is skewed because of the skew of the positive charge distribution. Outside the shell the pattern is the same as if the particle were centered and the shell were missing. In fact, this would be true no matter where inside the shell the particle happened to be located.

(a )

Additional examples, video, and practice available at WileyPLUS

(b )

23-4 APPLYI NG GAUSS’ L AW: CYLI N DR ICAL SYM M ETRY

671

23-4 APPLYING GAUSS’ LAW: CYLINDRICAL SYMMETRY Learning Objectives After reading this module, you should be able to . . .

23.20 Explain how Gauss’ law is used to derive the electric field magnitude outside a line of charge or a cylindrical surface (such as a plastic rod) with a uniform linear charge density l. 23.21 Apply the relationship between linear charge density l

on a cylindrical surface and the electric field magnitude E at radial distance r from the central axis. 23.22 Explain how Gauss’ law can be used to find the electric field magnitude inside a cylindrical nonconducting surface (such as a plastic rod) with a uniform volume charge density r.

Key Idea ● The electric field at a point near an infinite line of charge (or charged rod) with uniform linear charge density l is perpendicular

to the line and has magnitude E$

l 2p´0r

(line of charge),

where r is the perpendicular distance from the line to the point.

Applying Gauss’ Law: Cylindrical Symmetry Figure 23-14 shows a section of an infinitely long cylindrical plastic rod with a uniform charge density l. We want to find an expression for the electric field magnitude E at radius r from the central axis of the rod, outside the rod. We could do : that using the approach of Chapter 22 (charge element dq, field vector dE , etc.). However, Gauss’ law gives a much faster and easier (and prettier) approach. The charge distribution and the field have cylindrical symmetry. To find the field at radius r, we enclose a section of the rod with a concentric Gaussian cylinder of radius r and height h. (If you want the field at a certain point, put a Gaussian surface through that point.) We can now apply Gauss’ law to relate the charge enclosed by the cylinder and the net flux through the cylinder’s surface. First note that because of the symmetry, the electric field at any point must be radially outward (the charge is positive). That means that at any point on the end caps, the field only skims the surface and does not pierce it. So, the flux through each end cap is zero. To find the flux through the cylinder’s curved surface, first note that for any : patch element on the surface, the area vector dA is radially outward (away from the interior of the Gaussian surface) and thus in the same direction as the field piercing the patch.The dot product in Gauss’ law is then simply E dA cos 0 $ E dA, and we can pull E out of the integral. The remaining integral is just the instruction to sum the areas of all patch elements on the cylinder’s curved surface, but we already know that the total area is the product of the cylinder’s height h and circumference 2pr. The net flux through the cylinder is then & $ EA cos u $ E(2prh)cos 0 $ E(2prh). On the other side of Gauss’s law we have the charge qenc enclosed by the cylinder. Because the linear charge density (charge per unit length, remember) is uniform, the enclosed charge is lh. Thus, Gauss’ law, ´0& $ qenc, reduces to yielding

´0E(2prh) $ lh, E$

l 2p´0r

(line of charge).

(23-12)

This is the electric field due to an infinitely long, straight line of charge, at a point : that is a radial distance r from the line. The direction of E is radially outward

h

+ λ + 2π r + +r + Gaussian + surface + + + E + + + + There is flux only + + through the + curved surface.

Figure 23-14 A Gaussian surface in the form of a closed cylinder surrounds a section of a very long, uniformly charged, cylindrical plastic rod.

672

CHAPTE R 23 GAUSS’ L AW

from the line of charge if the charge is positive, and radially inward if it is negative. Equation 23-12 also approximates the field of a finite line of charge at points that are not too near the ends (compared with the distance from the line). If the rod has a uniform volume charge density r, we could use a similar procedure to find the electric field magnitude inside the rod. We would just shrink the Gaussian cylinder shown in Fig. 23-14 until it is inside the rod. The charge qenc enclosed by the cylinder would then be proportional to the volume of the rod enclosed by the cylinder because the charge density is uniform. Sample Problem 23.06 Gauss’ law and an upward streamer in a lightning storm Upward streamer in a lightning storm. The woman in Fig. 2315 was standing on a lookout platform high in the Sequoia National Park when a large storm cloud moved overhead. Some of the conduction electrons in her body were driven into the ground by the cloud’s negatively charged base (Fig. 23-16a), leaving her positively charged. You can tell she was highly charged because her hair strands repelled one another and extended away from her along the electric field lines produced by the charge Courtesy NOAA on her. Figure 23-15 This woman has Lightning did not strike become positively charged by the woman, but she was in an overhead storm cloud. extreme danger because that electric field was on the verge of causing electrical breakdown in the surrounding air. Such a breakdown would have occurred along a path extending away from her in what is called an upward streamer. An upward streamer is dangerous because the resulting ionization of molecules in the air suddenly frees a tremendous number of electrons from those molecules. Had the woman in Fig. 23-15 developed an upward streamer, the free electrons in the air would have moved to neutralize her (Fig. 23-16b), producing a large, perhaps fatal, charge flow through her body. That charge flow is dangerous because it could have interfered with or even stopped her breathing (which is obviously necessary for oxygen) and the steady beat of her heart (which is obviously necessary for the blood flow that carries the oxygen). The charge flow could also have caused burns. Let’s model her body as a narrow vertical cylinder of height L $ 1.8 m and radius R $ 0.10 m (Fig. 23-16c). Assume that charge Q was uniformly distributed along the cylinder and that electrical breakdown would have occurred if the electric

field magnitude along her body had exceeded the critical value Ec $ 2.4 MN/C. What value of Q would have put the air along her body on the verge of breakdown? KEY IDEA Because R * L, we can approximate the charge distribution as a long line of charge. Further, because we assume that the charge is uniformly distributed along this line, we can approximate the magnitude of the electric field along the side of her body with Eq. 23-12 (E $ l/2p´0 r). Calculations: Substituting the critical value Ec for E, the cylinder radius R for radial distance r, and the ratio Q/L for linear charge density l, we have Ec $ or

Q/L , 2p´0R

Q $ 2p´0RLEc.

Substituting given data then gives us Q $ (2p)(8.85 ( 10 '12 C 2 / N#m2)(0.10 m) ( (1.8 m)(2.4 ( 10 6 N/C) $ 2.402 ( 10 '5 C % 24 mC.

(Answer)

e R Upward streamer

+Q

e

L

e (a)

(b)

(c)

Figure 23-16 (a) Some of the conduction electrons in the woman’s body are driven into the ground, leaving her positively charged. (b) An upward streamer develops if the air undergoes electrical breakdown, which provides a path for electrons freed from molecules in the air to move to the woman. (c) A cylinder represents the woman.

Additional examples, video, and practice available at WileyPLUS

673

23-5 APPLYI NG GAUSS’ L AW: PL ANAR SYM M ETRY

23-5 APPLYING GAUSS’ LAW: PLANAR SYMMETRY Learning Objectives After reading this module, you should be able to . . .

23.23 Apply Gauss’ law to derive the electric field magnitude E near a large, flat, nonconducting surface with a uniform surface charge density s. 23.24 For points near a large, flat nonconducting surface with a uniform charge density s, apply the relationship be-

tween the charge density and the electric field magnitude E and also specify the direction of the field. 23.25 For points near two large, flat, parallel, conducting surfaces with a uniform charge density s, apply the relationship between the charge density and the electric field magnitude E and also specify the direction of the field.

Key Ideas

● The external electric field just outside the surface of an iso-

● The electric field due to an infinite nonconducting sheet with uniform surface charge density s is perpendicular to the plane of the sheet and has magnitude

E$

s 2´0

lated charged conductor with surface charge density s is perpendicular to the surface and has magnitude s (external, charged conductor). E$ ´0 Inside the conductor, the electric field is zero.

(nonconducting sheet of charge).

Applying Gauss’ Law: Planar Symmetry Nonconducting Sheet Figure 23-17 shows a portion of a thin, infinite, nonconducting sheet with a uniform (positive) surface charge density s. A sheet of thin plastic wrap, uniformly : charged on one side, can serve as a simple model. Let us find the electric field E a distance r in front of the sheet. A useful Gaussian surface is a closed cylinder with end caps of area A, : arranged to pierce the sheet perpendicularly as shown. From symmetry, E must be perpendicular to the sheet and hence to the end caps. Furthermore, since the : charge is positive, E is directed away from the sheet, and thus the electric field lines pierce the two Gaussian end caps in an outward direction. Because the field lines do not pierce the curved surface, there is no flux through this portion of the : : Gaussian surface. Thus E " dA is simply E dA; then Gauss’ law, ´0 becomes

,

:

:

E " dA $ qenc,

´0(EA " EA) $ sA,

where sA is the charge enclosed by the Gaussian surface. This gives E$

s 2´0

(23-13)

(sheet of charge).

Since we are considering an infinite sheet with uniform charge density, this result holds for any point at a finite distance from the sheet. Equation 23-13 agrees with Eq. 22-27, which we found by integration of electric field components.

Figure 23-17 (a) Perspective view and (b) side view of a portion of a very large, thin plastic sheet, uniformly charged on one side to surface charge density s. A closed cylindrical Gaussian surface passes through the sheet and is perpendicular to it.

+ E+

+ (a)

+ + + +

+ + + + + + + +

+ + + + + + + +

+ + + +

+ + + +

+ + + +

+ + + +

+ +

r

+

+ + +

+ + + +

+

σ

+ +

+ Gaussian + + surface + + A +E +

+

+

+

There is flux only through the two end faces.

A E

+ + + + + + + + + + + (b)

A E

674

σ1

CHAPTE R 23 GAUSS’ L AW

σ1

σ1

+ +

– – – –

+ + E

+ +

E

E

– –

+ +

– –

+ +

– –

(a)

(b)

E=0

σ1

+ + + + + + + + + +

2σ 1

E

– – – – – – – – – –

E

E=0

(c)

Figure 23-18 (a) A thin, very large conducting plate with excess positive charge. (b) An identical plate with excess negative charge. (c) The two plates arranged so they are parallel and close.

Two Conducting Plates Figure 23-18a shows a cross section of a thin, infinite conducting plate with excess positive charge. From Module 23-3 we know that this excess charge lies on the surface of the plate. Since the plate is thin and very large, we can assume that essentially all the excess charge is on the two large faces of the plate. If there is no external electric field to force the positive charge into some particular distribution, it will spread out on the two faces with a uniform surface charge density of magnitude s1. From Eq. 23-11 we know that just outside the plate this charge sets up an electric field of magnitude E $ s1/´0. Because the excess charge is positive, the field is directed away from the plate. Figure 23-18b shows an identical plate with excess negative charge having the same magnitude of surface charge density s1. The only difference is that now the electric field is directed toward the plate. Suppose we arrange for the plates of Figs. 23-18a and b to be close to each other and parallel (Fig. 23-18c). Since the plates are conductors, when we bring them into this arrangement, the excess charge on one plate attracts the excess charge on the other plate, and all the excess charge moves onto the inner faces of the plates as in Fig. 23-18c. With twice as much charge now on each inner face, the new surface charge density (call it s) on each inner face is twice s1. Thus, the electric field at any point between the plates has the magnitude E$

2s1 s $ . ´0 ´0

(23-14)

This field is directed away from the positively charged plate and toward the negatively charged plate. Since no excess charge is left on the outer faces, the electric field to the left and right of the plates is zero. Because the charges moved when we brought the plates close to each other, the charge distribution of the two-plate system is not merely the sum of the charge distributions of the individual plates. One reason why we discuss seemingly unrealistic situations, such as the field set up by an infinite sheet of charge, is that analyses for “infinite” situations yield good approximations to many real-world problems. Thus, Eq. 23-13 holds well for a finite nonconducting sheet as long as we are dealing with points close to the sheet and not too near its edges. Equation 23-14 holds well for a pair of finite conducting plates as long as we consider points that are not too close to their edges. The trouble with the edges is that near an edge we can no longer use planar symmetry to find expressions for the fields. In fact, the field lines there are curved (said to be an edge effect or fringing), and the fields can be very difficult to express algebraically.

Sample Problem 23.07 Electric field near two parallel nonconducting sheets with charge Figure 23-19a shows portions of two large, parallel, nonconducting sheets, each with a fixed uniform charge on one side. The magnitudes of the surface charge densities are s(") $ 6.8 mC/m2 for the positively charged sheet and s(') $ 4.3 mC/m2 for the negatively charged sheet. : Find the electric field E (a) to the left of the sheets, (b) between the sheets, and (c) to the right of the sheets. KEY IDEA With the charges fixed in place (they are on nonconductors), we can find the electric field of the sheets in Fig. 23-19a by (1) finding the field of each sheet as if that sheet were isolated and (2) algebraically adding the fields of the isolated sheets

+ σ (+)+ + + + + + + + + + + + + (a) +

– σ (–) – – – – – – – – –

+ + E(+) + + + + + + L + + E(–) + + + + (b) + + Figure 23-19 (a) Two large, paral+ + lel sheets, uniformly charged on + + one side. (b) The individual + EL + electric fields resulting from the + + two charged sheets. (c) The net + + field due to both charged + + sheets, found by superposition. + (c) +

E(+) B E(–)

EB

– – E(+) – – – R – – E(–) – – – – – – – – ER – – – – –

675

23-6 APPLYI NG GAUSS’ L AW: SPH E R ICAL SYM M ETRY

via the superposition principle. (We can add the fields algebraically because they are parallel to each other.) :

Calculations: At any point, the electric field E(") due to the positive sheet is directed away from the sheet and, from Eq. 23-13, has the magnitude E(") $

s(") 2)0

$

6.8 ( 10 '6 C/m2 (2)(8.85 ( 10 '12 C2/N#m2)

$ 3.84 ( 10 5 N/C. :

Similarly, at any point, the electric field E(') due to the negative sheet is directed toward that sheet and has the magnitude 4.3 ( 10 '6 C/m2 2´0 (2)(8.85 ( 10 '12 C2/N#m2) $ 2.43 ( 10 5 N/C.

E(') $

s(')

$

Figure 23-19b shows the fields set up by the sheets to the left of the sheets (L),between them (B),and to their right (R). The resultant fields in these three regions follow from the superposition principle.To the left, the field magnitude is EL $ E(") ' E(') $ 3.84 ( 10 5 N/C ' 2.43 ( 10 5 N/C $ 1.4 ( 10 5 N/C. (Answer) :

Because E(") is larger than E('), the net electric field EL in this region is directed to the left, as Fig. 23-19c shows.To the right of the sheets, the net electric field has the same magnitude but is directed to the right, as Fig. 23-19c shows. Between the sheets, the two fields add and we have EB $ E(") " E(') $ 3.84 ( 10 5 N/C " 2.43 ( 10 5 N/C $ 6.3 ( 10 5 N/C. (Answer) :

The electric field EB is directed to the right. Additional examples, video, and practice available at WileyPLUS

23-6 APPLYING GAUSS’ LAW: SPHERICAL SYMMETRY Learning Objectives After reading this module, you should be able to . . .

23.26 Identify that a shell of uniform charge attracts or repels a charged particle that is outside the shell as if all the shell’s charge is concentrated at the center of the shell. 23.27 Identify that if a charged particle is enclosed by a shell of uniform charge, there is no electrostatic force on the particle from the shell. 23.28 For a point outside a spherical shell with uniform

charge, apply the relationship between the electric field magnitude E, the charge q on the shell, and the distance r from the shell’s center. 23.29 Identify the magnitude of the electric field for points enclosed by a spherical shell with uniform charge. 23.30 For a uniform spherical charge distribution (a uniform ball of charge), determine the magnitude and direction of the electric field at interior and exterior points.

Key Ideas ● Outside a spherical shell of uniform charge q, the electric

● Inside the shell, the field due to the shell is zero.

field due to the shell is radial (inward or outward, depending on the sign of the charge) and has the magnitude

● Inside a sphere with a uniform volume charge density, the

E$

1 q 4p´0 r2

(outside spherical shell),

where r is the distance to the point of measurement from the center of the shell. The field is the same as though all of the charge is concentrated as a particle at the center of the shell.

field is radial and has the magnitude E$

q 1 r 4p´0 R3

(inside sphere of charge),

where q is the total charge, R is the sphere’s radius, and r is the radial distance from the center of the sphere to the point of measurement.

Applying Gauss’ Law: Spherical Symmetry Here we use Gauss’ law to prove the two shell theorems presented without proof in Module 21-1: A shell of uniform charge attracts or repels a charged particle that is outside the shell as if all the shell’s charge were concentrated at the center of the shell.

676

CHAPTE R 23 GAUSS’ L AW

S2 q

R r

Figure 23-20 shows a charged spherical shell of total charge q and radius R and two concentric spherical Gaussian surfaces, S1 and S2. If we followed the procedure of Module 23-2 as we applied Gauss’ law to surface S2, for which r - R, we would find that

S1

E$

Figure 23-20 A thin, uniformly charged, spherical shell with total charge q, in cross section. Two Gaussian surfaces S1 and S2 are also shown in cross section. Surface S2 encloses the shell, and S1 encloses only the empty interior of the shell.

Enclosed charge is q

R r

(a) Enclosed charge is q'

Gaussian surface

E$0

(b)

(spherical shell, field at r , R),

(23-16)

If a charged particle is located inside a shell of uniform charge, there is no electrostatic force on the particle from the shell.

Any spherically symmetric charge distribution, such as that of Fig. 23-21, can be constructed with a nest of concentric spherical shells. For purposes of applying the two shell theorems, the volume charge density r should have a single value for each shell but need not be the same from shell to shell. Thus, for the charge distribution as a whole, r can vary, but only with r, the radial distance from the center. We can then examine the effect of the charge distribution “shell by shell.” In Fig. 23-21a, the entire charge lies within a Gaussian surface with r + R. The charge produces an electric field on the Gaussian surface as if the charge were that of a particle located at the center, and Eq. 23-15 holds. Figure 23-21b shows a Gaussian surface with r , R. To find the electric field at points on this Gaussian surface, we separately consider the charge inside it and the charge outside it. From Eq. 23-16, the outside charge does not set up a field on the Gaussian surface. From Eq. 23-15, the inside charge sets up a field as though it is concentrated at the center. Letting q. represent that enclosed charge, we can then rewrite Eq. 23-15 as E$

1 q. 4p´0 r2

(spherical distribution, field at r / R).

(23-17)

If the full charge q enclosed within radius R is uniform, then q. enclosed within radius r in Fig. 23-21b is proportional to q: enclosed by #charge sphere of radius r $ full charge $ full volume volume enclosed by # sphere of radius r $

The flux through the surface depends on only the enclosed charge.

Figure 23-21 The dots represent a spherically symmetric distribution of charge of radius R, whose volume charge density r is a function only of distance from the center. The charged object is not a conductor, and therefore the charge is assumed to be fixed in position. A concentric spherical Gaussian surface with r + R is shown in (a). A similar Gaussian surface with r , R is shown in (b).

(23-15)

because this Gaussian surface encloses no charge. Thus, if a charged particle were enclosed by the shell, the shell would exert no net electrostatic force on the particle. This proves the second shell theorem.

R r

(spherical shell, field at r - R).

This field is the same as one set up by a particle with charge q at the center of the shell of charge. Thus, the force produced by a shell of charge q on a charged particle placed outside the shell is the same as if all the shell’s charge is concentrated as a particle at the shell’s center. This proves the first shell theorem. Applying Gauss’ law to surface S1, for which r , R, leads directly to

Gaussian surface

ρ

q 1 4p´0 r2

q.

or

4 3 3 pr

$

This gives us q. $ q

q 4 3 3 pR

.

r3 . R3

(23-18) (23-19)

Substituting this into Eq. 23-17 yields E$

# 4p´q R $r 0

3

(uniform charge, field at r / R).

(23-20)

677

QU ESTIONS

Checkpoint 4 The figure shows two large, parallel, nonconducting sheets with identical (positive) uniform surface charge densities, and a sphere with a uniform (positive) volume charge density. Rank the four numbered points according to the magnitude of the net electric field there, greatest first.

+ + + + + +

1 2

+

3

d

d

4

d

d

d

+ + + + + +

Review & Summary Gauss’ Law Gauss’ law and Coulomb’s law are different ways of describing the relation between charge and electric field in static situations. Gauss’ law is ´0& $ qenc

,

:

:

E " dA

(electric flux through a Gaussian surface).

(23-4)

Coulomb’s law can be derived from Gauss’ law. cases, symmetry arguments, we can derive several important results in electrostatic situations. Among these are: 1. An excess charge on an isolated conductor is located entirely on the outer surface of the conductor. 2. The external electric field near the surface of a charged conductor is perpendicular to the surface and has a magnitude that depends on the surface charge density s : s ´0

l 2p´0r

(line of charge),

(23-12)

where r is the perpendicular distance from the line of charge to the point. 4. The electric field due to an infinite nonconducting sheet with uniform surface charge density s is perpendicular to the plane of the sheet and has magnitude s (sheet of charge). (23-13) 2´0 5. The electric field outside a spherical shell of charge with radius R and total charge q is directed radially and has magnitude E$

Applications of Gauss’ Law Using Gauss’ law and, in some

E$

E$

(23-6)

(Gauss’ law),

in which qenc is the net charge inside an imaginary closed surface (a Gaussian surface) and & is the net flux of the electric field through the surface: &$

with uniform linear charge density l is perpendicular to the line of charge and has magnitude

(23-11)

(conducting surface).

Within the conductor, E $ 0. 3. The electric field at any point due to an infinite line of charge

E$

1 q 4p´0 r2

(23-15)

(spherical shell, for r - R).

Here r is the distance from the center of the shell to the point at which E is measured. (The charge behaves, for external points, as if it were all located at the center of the sphere.) The field inside a uniform spherical shell of charge is exactly zero: E$0

(23-16)

(spherical shell, for r , R).

6. The electric field inside a uniform sphere of charge is directed radially and has magnitude E$

# 4p´q R $ r. 0

(23-20)

3

Questions 1 A surface has the area vector A $ (2iˆ " 3jˆ ) m2. What is the flux of a uniform electric field through the area if the field is : : (a) E $ 4iˆ N/C and (b) E $ 4kˆ N/C? :

2 Figure 23-22 shows, in cross section, three solid cylinders, each of length L and uniform charge Q. Concentric with each cylinder is a cylindrical Gaussian surface, with all three surfaces having the same radius. Rank the Gaussian surfaces according to the electric field at any point on the surface, greatest first.

3 Figure 23-23 shows, in cross section, a central metal ball, two spherical metal shells, and three spherical Gaussian surfaces of radii R, 2R, and 3R, all with the same center. The uniform charges on the three objects are: ball, Q; smaller shell, 3Q; larger shell, 5Q. Rank the Gaussian surfaces according to the magnitude of the electric field at any point on the surface, greatest first. 3R

Shell Cylinder Gaussian surface

R (a)

(b)

Figure 23-22 Question 2.

(c)

Figure 23-23 Question 3.

Gaussian surface

2R

678

CHAPTE R 23 GAUSS’ L AW

4 Figure 23-24 shows, in cross section, two Gaussian spheres and two Gaussian cubes that are centered on a positively charged particle. (a) +q Rank the net flux through the four Gaussian surfaces, greatest first. (b) a c Rank the magnitudes of the electric b fields on the surfaces, greatest first, d and indicate whether the magnitudes are uniform or variable along each surface. Figure 23-24 Question 4. 5 In Fig. 23-25, an electron is released between two infinite nonconducting sheets that are horizontal and have uniform surface charge densities s(") and s('), as indicated.The electron is subjected to the following three situations involving surface charge densities and sheet separations. Rank the magnitudes of the electron’s acceleration,greatest first. Situation

s(")

s(')

Separation

1 2 3

"4s "7s "3s

'4s 's '5s

d 4d 9d

+

+

+

+

σ(+) + +

+







– – σ(–)

P

(a)

P

P

P

(b)

(c)

(d)

Figure 23-27 Question 8. 9 A small charged ball lies within the hollow of a metallic spherical shell of radius R. For three situations, the net charges on the ball and shell, respectively, are (1) "4q, 0; (2) '6q, "10q; (3) "16q, '12q. Rank the situations according to the charge on (a) the inner surface of the shell and (b) the outer surface, most positive first. 10 Rank the situations of Question 9 according to the magnitude of the electric field (a) halfway through the shell and (b) at a point 2R from the center of the shell, greatest first. 11 Figure 23-28 shows a section of three long charged cylinders centered on the same axis. Central cylinder A has a uniform charge qA $ "3q0. What uniform charges qB and qC should be on cylinders B and C so that (if possible) the net electric field is zero at (a) point 1, (b) point 2, and (c) point 3?

e –

8 Figure 23-27 shows four solid spheres, each with charge Q uniformly distributed through its volume. (a) Rank the spheres according to their volume charge density, greatest first. The figure also shows a point P for each sphere, all at the same distance from the center of the sphere. (b) Rank the spheres according to the magnitude of the electric field they produce at point P, greatest first.



Figure 23-25 Question 5. 6 Three infinite nonconducting sheets, with uniform positive surface charge densities s, 2s, and 3s, are arranged to be parallel like the two sheets in Fig. 23-19a. What is their order, from left to : right, if the electric field E produced by the arrangement has magnitude E $ 0 in one region and E $ 2s/´0 in another region? 7 Figure 23-26 shows four situations in which four very long rods extend into and out of the page (we see only their cross sections). The value below each cross section gives that particular rod’s uniform charge density in microcoulombs per meter. The rods are separated by either d or 2d as drawn, and a central point is shown midway between the inner rods. Rank the situations according to the magnitude of the net electric field at that central point, greatest first. (a)

(b)

(c)

(d )

+3

+2

–2

–3

+2

–4

–4

+2

AB

3

C

2 1

Figure 23-28 Question 11. 12 Figure 23-29 shows four Gaussian surfaces consisting of identical cylindrical midsections but different end caps. The surfaces are in a : uniform electric field E that is directed parallel to the central axis of each cylindrical midsection. The end caps have these shapes: S1, convex hemispheres; S2, concave hemispheres; S3, cones; S4, flat disks. Rank the surfaces according to (a) the net electric flux through them and (b) the electric flux through the top end caps, greatest first.

E +8

–2

+2

+8

–6

+5

+5

–6

Figure 23-26 Question 7.

S1

S2

S3

Figure 23-29 Question 12.

S4

PROB LE M S

679

Problems Tutoring problem available (at instructor’s discretion) in WileyPLUS and WebAssign SSM • – •••

Worked-out solution available in Student Solutions Manual

WWW Worked-out solution is at

Number of dots indicates level of problem difficulty

ILW

Interactive solution is at

http://www.wiley.com/college/halliday

Additional information available in The Flying Circus of Physics and at flyingcircusofphysics.com

Module 23-1 Electric Flux •1 SSM The square surface shown in Fig. 23-30 measures 3.2 mm on Normal each side. It is immersed in a uniform electric field with magnitude θ E $ 1800 N/C and with field lines at an angle of u $ 35° with a normal to the surface, as shown. Take that normal to be directed “outward,” as though the surface were one face of a box. Calculate the electric flux through the surface. Figure 23-30 Problem 1. ••2 An electric field given by : z E $ 4.0iˆ ' 3.0(y2 " 2.0)jˆ pierces a Gaussian cube of edge length 2.0 m and positioned as shown in Fig. 23-7. (The magnitude E is in newtons per coulomb and the position x is in meters.) What is the electric flux through the (a) top face, (b) bottom face, (c) left face, and (d) back face? (e) What is the net electric flux through the cube? x ••3 The cube in Fig. 23-31 has edge length 1.40 m and is oriented as shown Figure 23-31 Problems 3, in a region of uniform electric field. Find 6, and 9. the electric flux through the right face if the electric field, in newtons per coulomb, is given by (a) 6.00iˆ , (b) '2.00jˆ , and (c) '3.00iˆ " 4.00kˆ . (d) What is the total flux through the cube for each field? Module 23-2 Gauss’ Law •4 In Fig. 23-32, a butterfly net is in a uniform electric field of magnitude E $ 3.0 mN/C. The rim, a circle of radius a $ 11 cm, is aligned perpendicular to the field. The net contains no net charge. Find the electric flux through the netting.

a

•7 A particle of charge 1.8 mC is at the center of a Gaussian cube 55 cm on edge. What is the net electric flux through the surface? ••8 When a shower is turned on in a closed bathroom, the splashing of the water on the bare tub can fill the room’s air with negatively charged ions and produce an electric field in the air as great as 1000 N/C. Consider a bathroom with dimensions 2.5 m ( 3.0 m ( 2.0 m. Along the ceiling, floor, and four walls, approximate the electric field in the air as being directed perpendicular to the surface and as having a uniform magnitude of 600 N/C.Also, treat those surfaces as forming a closed Gaussian surface around the room’s air. What are (a) the volume charge density r and (b) the number of excess elementary charges e per cubic meter in the room’s air?

y

••9 ILW Fig. 23-31 shows a Gaussian surface in the shape of a cube with edge length 1.40 m. What are (a) the net flux & through the surface and (b) the net charge qenc enclosed by the surface if : E $(3.00yjˆ ) N/C, with y in meters? What : z are (c) & and (d) qenc if E $ ['4.00iˆ " (6.00 " 3.00y)jˆ ] N/C? ••10 Figure 23-34 shows a closed Gaussian surface in the shape of a cube of edge length 2.00 m. It lies in a region where the nonuni: form electric field is given by E $ (3.00x " x ˆ ˆ ˆ 4.00)i " 6.00j " 7.00k N/C, with x in meters. What is the net charge contained by the cube?

y

Figure 23-34 Problem 10.

••11 Figure 23-35 shows a closed Gaussian surface in the shape of a cube of edge length 2.00 m, with one corner at x1 $ 5.00 m, y1 $ 4.00 m.The cube lies in a region where the electric field vector is given by : E $ '3.00iˆ ' 4.00y 2jˆ " 3.00kˆ N/C, with y in meters.What is the net charge contained by the cube? z y1

Figure 23-32 Problem 4.

•5 In Fig. 23-33, a proton is a distance d/2 directly above the center of a square of side d. What is the magnitude of the electric flux through the square? (Hint: Think of the square as one face of a cube with edge d.)

+ d/2 d

Figure 23-33 Problem 5.

: : E $ '34kˆ N/C, and on the bottom face it is E $ " 20kˆ N/C. Determine the net charge contained within the cube.

d

•6 At each point on the surface of the cube shown in Fig. 23-31, the electric field is parallel to the z axis. The length of each edge of the cube is 3.0 m. On the top face of the cube the field is

y

x1 x

Figure 23-35 Problem 11. ••12 Figure 23-36 shows two nonconducting spherical shells fixed in place. Shell 1 has uniform surface charge density "6.0 mC/m2 on its outer surface and radius 3.0 cm; shell 2 has uniform surface charge density "4.0 mC/m2 on its outer surface and radius 2.0 cm; the shell centers are separated by L $ 10 cm. In unit-vector notation, what is the net electric field at x $ 2.0 cm?

y Shell 1 Shell 2 x L

Figure 23-36 Problem 12.

680

CHAPTE R 23 GAUSS’ L AW

••13 SSM The electric field in a certain region of Earth’s atmosphere is directed vertically down. At an altitude of 300 m the field has magnitude 60.0 N/C; at an altitude of 200 m, the magnitude is 100 N/C. Find the net amount of charge contained in a cube 100 m on edge, with horizontal faces at altitudes of 200 and 300 m. ••14 Flux and nonconducting shells. A charged particle is suspended at the center of two concentric spherical shells that are very thin and made of nonconducting material. Figure 23-37a shows a cross section. Figure 23-37b gives the net flux & through a Gaussian sphere centered on the particle, as a function of the radius r of the sphere. The scale of the vertical axis is set by &s $ 5.0 ( 105 N ?m2/C. (a) What is the charge of the central particle? What are the net charges of (b) shell A and (c) shell B?

Φ (105 N • m2/C)

•20 Flux and conducting shells. A charged particle is held at the center of two concentric conducting spherical shells. Figure 23-39a shows a cross section. Figure 23-39b gives the net flux & through a Gaussian sphere centered on the particle, as a function of the radius r of the sphere. The scale of the vertical axis is set by &s $ 5.0 ( 105 N ?m2/C. What are (a) the charge of the central particle and the net charges of (b) shell A and (c) shell B? B A

Φs

r

0 – Φs

B A

Φ(105 N • m2/C)

–2 Φs Φs

(a)

r

0

(b)

Figure 23-39 Problem 20.

••21 An isolated conductor has net charge "10 ( 10 '6 C and a cavity with a particle of charge q $ "3.0 ( 10 '6 C.What is the charge on (a) the cavity wall and (b) the outer surface?

–Φs (b)

Figure 23-37 Problem 14. ••15 A particle of charge "q is placed at one corner of a Gaussian cube. What multiple of q/´0 gives the flux through (a) each cube face forming that corner and (b) each of the other cube faces? •••16 The box-like Gaussian surface shown in Fig. 23-38 encloses a net charge of "24.0´0 C and lies in an electric field given : by E $ [(10.0 " 2.00x)iˆ ' 3.00jˆ " bzkˆ ] N/C, with x and z in meters and b a constant. The bottom face is in the xz plane; the top face is in the horizontal plane passing through y2 $ 1.00 m. For x1 $ 1.00 m, x2 $ 4.00 m, z1 $ 1.00 m, and z2 $ 3.00 m, what is b? y y2 x2 z1

z1

z2

Figure 23-38 Problem 16.

z

x

z2 x1

x2

Module 23-3 A Charged Isolated Conductor •17 SSM A uniformly charged conducting sphere of 1.2 m diameter has surface charge density 8.1 mC/m2. Find (a) the net charge on the sphere and (b) the total electric flux leaving the surface.

Module 23-4 Applying Gauss’ Law: Cylindrical Symmetry •22 An electron is released 9.0 cm from a very long nonconducting rod with a uniform 6.0 mC/m. What is the magnitude of the electron’s initial acceleration? •23 (a) The drum of a photocopying machine has a length of 42 cm and a diameter of 12 cm. The electric field just above the drum’s surface is 2.3 ( 105 N/C. What is the total charge on the drum? (b) The manufacturer wishes to produce a desktop version of the machine. This requires reducing the drum length to 28 cm and the diameter to 8.0 cm. The electric field at the drum surface must not change. What must be the charge on this new drum? •24 Figure 23-40 shows a section of a long, thin-walled metal tube of radius R $ 3.00 cm, with a charge per unit length of l $ 2.00 ( 10 '8 C/m. What is the magnitude E of the electric field at radial distance (a) r $ R/2.00 and (b) r $ 2.00R? (c) Graph E versus r for the range r $ 0 to 2.00R.

+

+ +

+ + + +

+

+

+ + + +

R

+ + + + +

+ +

+

Es

•18 The electric field just above the surface of the charged conducting drum of a photocopying machine has a magnitude E of 2.3 ( 10 5 N/C. What is the surface charge density on the drum? •19 Space vehicles traveling through Earth’s radiation belts can intercept a significant number of electrons. The resulting charge buildup can damage electronic components and disrupt operations. Suppose a spherical metal satellite 1.3 m in diameter accumulates 2.4 mC of charge in one orbital revolution. (a) Find the resulting surface charge density. (b) Calculate the magnitude of the electric field just outside the surface of the satellite, due to the surface charge.

+

+

+

•25 SSM An infinite line of charge produces a field of magnitude 4.5 ( Figure 23-40 Problem 24. 10 4 N/C at distance 2.0 m. Find the linear charge density. ••26 Figure 23-41a shows a narrow charged solid cylinder that is coaxial with a larger charged cylindrical shell. Both are noncon-

(a)

E (103 N/C)

(a)

(b)

0

–Es

1

2

3

r (cm)

Figure 23-41 Problem 26.

4

5

6

681

PROB LE M S

ducting and thin and have uniform surface charge densities on their outer surfaces. Figure 23-41b gives the radial component E of the electric field versus radial distance r from the common axis, and Es $ 3.0 ( 103 N/C.What is the shell’s linear charge density?

the plates have excess surface charge densities of opposite signs and magnitude 7.00 ( 10 '22 C/m2. In unit-vector notation, what is the electric field at points (a) to the left of the plates, (b) to the right of them, and (c) between them?

••27 A long, straight wire has fixed negative charge with a linear charge density of magnitude 3.6 nC/m. The wire is to be enclosed by a coaxial, thin-walled nonconducting cylindrical shell of radius 1.5 cm. The shell is to have positive charge on its outside surface with a surface charge density s that makes the net external electric field zero. Calculate s.

•34 In Fig. 23-45, a small circular hole of radius R $ 1.80 cm has been cut in the middle of an infinite, flat, nonconducting surface that has uniform charge density s $ 4.50 pC/m2. A z axis, with its origin at the hole’s center, is perpendicular to the surface. In unitvector notation, what is the electric field at point P at z $ 2.56 cm? (Hint: See Eq. 22-26 and use superposition.)

••28 A charge of uniform linear density 2.0 nC/m is distributed along a long, thin, nonconducting rod. The rod is coaxial with a long conducting cylindrical shell (inner radius $ 5.0 cm, outer radius $ 10 cm). The net charge on the shell is zero. (a) What is the magnitude of the electric field 15 cm from the axis of the shell? What is the surface charge density on the (b) inner and (c) outer surface of the shell? ••29 SSM WWW Figure 23-42 is a section of a conducting rod of radius R1 $ 1.30 mm and length L $ 11.00 m inside a thin-walled coaxial conducting cylindrical shell of R1 radius R2 $ 10.0R1 and the (same) length L. The net charge on the rod R2 is Q1 $ "3.40 ( 10 '12 C; that on Q1 the shell is Q2 $ '2.00Q1. What are the (a) magnitude E and (b) diQ2 rection (radially inward or outFigure 23-42 Problem 29. ward) of the electric field at radial distance r $ 2.00R2? What are (c) E and (d) the direction at r $ 5.00R1? What is the charge on the (e) interior and (f) exterior surface of the shell?

z P

+ + + + + + + + + + + + + + + + + + + + + + + + + + + + + + + + + + R + + + + + + + + + + + + + + + + + + + + + + + + + + + + + + + + + + Figure 23-45 Problem 34. •35 Figure 23-46a shows three plastic sheets that are large, parallel, and uniformly charged. Figure 23-46b gives the component of the net electric field along an x axis through the sheets. The scale of the vertical axis is set by Es $ 6.0 ( 105 N/C.What is the ratio of the charge density on sheet 3 to that on sheet 2? 1

2

3

x

(a)

y

Module 23-5 Applying Gauss’ Law: Planar Symmetry •33 In Fig. 23-44, two large, thin metal plates are parallel and close to each other. On their inner faces,

– – – – –

+ + + + +

Figure 23-44 Problem 33.

x

E (105 N/C)

x

0 (b)

Figure 23-46 Problem 35. •36 Figure 23-47 shows cross sections through two large, parallel, nonconducting sheets with identical distributions of positive charge with surface charge density s $ 1.77 ( 10 '22 C/m2. : In unit-vector notation, what is E at points (a) above the sheets, (b) between them, and (c) below them?

y x + + + + + + + + + + +

•••32 A long, nonconducting, solid cylinder of radius 4.0 cm has a nonuniform volume charge density r that is a function of radial distance r from the cylinder axis: r $ – + Ar 2. For A $ 2.5 mC/m5, what is the – + magnitude of the electric field at – + (a) r $ 3.0 cm and (b) r $ 5.0 cm? – +

Es

+ + + + + + + + + + +

••30 In Fig. 23-43, short sections of two very long parallel lines of Line 2 charge are shown, fixed in place, Line 1 separated by L $ 8.0 cm. The uniform linear charge densities are x "6.0 mC/m for line 1 and '2.0 mC/m for line 2. Where along the x axis shown is the net electric field L/2 L/2 from the two lines zero? Figure 23-43 Problem 30. ••31 ILW Two long, charged, thin-walled, concentric cylindrical shells have radii of 3.0 and 6.0 cm. The charge per unit length is 5.0 ( 10 '6 C/m on the inner shell and '7.0 ( 10 '6 C/m on the outer shell. What are the (a) magnitude E and (b) direction (radially inward or outward) of the electric field at radial distance r $ 4.0 cm? What are (c) E and (d) the direction at r $ 8.0 cm?

Figure 23-47 Problem 36.

•37 SSM WWW A square metal plate of edge length 8.0 cm and negligible thickness has a total charge of 6.0 ( 10 '6 C. (a) Estimate the magnitude E of the electric field just off the center of the plate (at, say, a distance of 0.50 mm from the center) by assuming that the charge is spread uniformly over the two faces of the plate. (b) Estimate E at a distance of 30 m (large relative to the plate size) by assuming that the plate is a charged particle.

CHAPTE R 23 GAUSS’ L AW

v (105 m/s)

vs

–e

0

9 3

6

12

+ + + + + + + + –vs (a)

t (ps) (b)

Figure 23-48 Problem 38. ••39 SSM In Fig. 23-49, a small, nonconducting ball of mass m $ 1.0 mg and charge q $ 2.0 ( 10 '8 C (distributed uniformly through its volume) hangs from an insulating thread that makes an angle u $ 30° with a vertical, uniformly charged nonconducting sheet (shown in cross section). Considering the gravitational force on the ball and assuming the sheet extends far vertically and into and out of the page, calculate the surface charge density s of the sheet.

+ +σ + + +θ + + + + + + + +

m, q

••40 Figure 23-50 shows a very large nonconducting sheet that has a uniform surface charge density Figure 23-49 of s $ '2.00 mC/m2; it also shows a particle of Problem 39. charge Q $ 6.00 mC, at distance d from the sheet. Both are fixed in place. If d $ 0.200 m, y at what (a) positive and (b) negative σ coordinate on the x axis (other than in: finity) is the net electric field Enet of x the sheet and particle zero? (c) If d $ Q d 0.800 m, at what coordinate on the x : axis is Enet $ 0? Figure 23-50 Problem 40. ••41 An electron is shot directly toward the center of a large metal plate that has surface charge density '2.0 ( 10 '6 C/m2. If the initial kinetic energy of the electron is 1.60 ( 10 '17 J and if the electron is to stop (due to electrostatic repulsion from the plate) just as it reaches the plate, how far from the plate must the launch point be? ••42 Two large metal plates of area 1.0 m2 face each other, 5.0 cm apart, with equal charge magnitudes ! q ! but opposite signs. The field magnitude E between them (neglect fringing) is 55 N/C. Find ! q !. •••43 Figure 23-51 shows a cross section through a very large nonconducting slab of thickness d $ 9.40 mm and uniform volume charge density r $ 5.80 fC/m3. The origin of an x axis is at the slab’s center. What is the magnitude of the slab’s electric field at an x coordinate of (a) 0, (b) 2.00 mm, (c) 4.70 mm, and (d) 26.0 mm?

d/2 x

0 d

Figure 23-51 Problem 43.

Module 23-6 Applying Gauss’ Law: Spherical Symmetry Es •44 Figure 23-52 gives the magnitude of the electric field inside and outside a sphere with a positive charge distributed uniformly throughout its volume.The scale of the vertical axis is 4 0 2 set by Es $ 5.0 ( 107 N/C. What is the r (cm) charge on the sphere? Figure 23-52 Problem 44. •45 Two charged concentric spherical shells have radii 10.0 cm and 15.0 cm. The charge on the inner shell is 4.00 ( 10 '8 C, and that on the outer shell is 2.00 ( 10 '8 C. Find the electric field (a) at r $ 12.0 cm and (b) at r $ 20.0 cm. •46 Assume that a ball of charged particles has a uniformly distributed negative charge density except for a narrow radial tunnel through its center, from the surface on one side to the surface on the opposite side. Also assume that we can position a proton anywhere along the tunnel or outside the ball. Let FR be the magnitude of the electrostatic force on the proton when it is located at the ball’s surface, at radius R. As a multiple of R, how far from the surface is there a point where the force magnitude is 0.50FR if we move the proton (a) away from the ball and (b) into the tunnel? •47 SSM An unknown charge sits on a conducting solid sphere of radius 10 cm. If the electric field 15 cm from the center of the sphere has the magnitude 3.0 ( 10 3 N/C and is directed radially inward, what is the net charge on the sphere? ••48 A charged particle is held at the center of a spherical shell. Figure 23-53 gives the magnitude E of the electric field versus radial distance r. The scale of the vertical axis is set by Es $ 10.0 ( 107 N/C. Approximately, what is the net charge on the shell? E (107 N/C)

••38 In Fig. 23-48a, an electron is shot directly away from a uniformly charged plastic sheet, at speed vs $ 2.0 ( 10 5 m/s. The sheet is nonconducting, flat, and very large. Figure 23-48b gives the electron’s vertical velocity component v versus time t until the return to the launch point.What is the sheet’s surface charge density?

Es

E (107 N/C)

682

0

1

2

3

4

5

r (cm)

Figure 23-53 Problem 48. ••49 In Fig. 23-54, a solid sphere of radius a $ 2.00 cm is concentric with a spherical conducting shell of inner radius b $ 2.00a and outer radius c $ 2.40a. The sphere has a net uniform charge q1 $ "5.00 fC; the shell has a net charge q2 $ 'q1. What is the magnitude of the electric field at radial distances (a) r $ 0, (b) r $ a/2.00, (c) r $ a, (d) r $ 1.50a, (e) r $ 2.30a, and (f) r $ 3.50a? What is the net charge on the (g) inner and (h) outer surface of the shell?

a

b c

Figure 23-54 Problem 49.

PROB LE M S

••50 Figure 23-55 shows two nonconducting spherical shells fixed in place on an x axis. Shell 1 has uniform surface charge density "4.0 mC/m2 on its outer surface and radius 0.50 cm, and shell 2 has uniform surface charge density '2.0 mC/m2 on its outer surface and radius 2.0 cm; the centers are separated by L $ 6.0 cm. Other than at x $ 0, where on the x axis is the net electric field equal to zero? ••51 SSM WWW In Fig. 23-56, a nonconducting spherical shell of inner radius a $ 2.00 cm and outer radius b $ 2.40 cm has (within its thickness) a positive volume charge density r $ A/r, where A is a constant and r is the distance from the center of the shell. In addition, a small ball of charge q $ 45.0 fC is located at that center. What value should A have if the electric field in the shell (a / r / b) is to be uniform? ••52 Figure 23-57 shows a spherical shell with uniform volume charge density r $ 1.84 nC/m3, inner radius a $ 10.0 cm, and outer radius b $ 2.00a. What is the magnitude of the electric field at radial distances (a) r $ 0; (b) r $ a/2.00, (c) r $ a, (d) r $ 1.50a, (e) r $ b, and (f) r $ 3.00b?

y Shell 1

Shell 2 x

683

57 A thin-walled metal spherical shell has radius 25.0 cm and charge 2.00 ( 10 '7 C. Find E for a point (a) inside the shell, (b) just outside it, and (c) 3.00 m from the center. 58 A uniform surface charge of density 8.0 nC/m2 is distributed over the entire xy plane. What is the electric flux through a spherical Gaussian surface centered on the origin and having a radius of 5.0 cm?

L

Figure 23-55 Problem 50.

q

+

b a

Figure 23-56 Problem 51. + + + + + + + + + + + + + + + + + + + + + + + + + + + + + + + + + + + + + + + a + + + + + + + + + + + + + + + + + + + + + + + b + + + + +

Figure 23-57 Problem 52. •••53 ILW The volume charge density of a solid nonconducting sphere of radius R $ 5.60 cm varies with radial distance r as given by r $ (14.1 pC/m3)r/R. (a) What is the sphere’s total charge? What is the field magnitude E at (b) r $ 0, (c) r $ R/2.00, and (d) r $ R? (e) Graph E versus r. •••54 Figure 23-58 shows, in cross R R section, two solid spheres with uniP formly distributed charge through1 2 out their volumes. Each has radius R. Point P lies on a line connecting Figure 23-58 Problem 54. the centers of the spheres, at radial distance R/2.00 from the center of sphere 1. If the net electric field at point P is zero, what is the ratio q2/q1 of the total charges?

59 Charge of uniform volume density r $ 1.2 nC/m3 fills an infinite slab between x $ '5.0 cm and x $ "5.0 cm. What is the magnitude of the electric field at any point with the coordinate (a) x $ 4.0 cm and (b) x $ 6.0 cm? 60 The chocolate crumb mystery. Explosions ignited by electrostatic discharges (sparks) constitute a serious danger in facilities handling grain or powder. Such an explosion occurred in chocolate crumb powder at a biscuit factory in the 1970s. Workers usually emptied newly delivered sacks of the powder into a loading bin, from which it was blown through electrically grounded plastic pipes to a silo for storage. Somewhere along this route, two conditions for an explosion were met: (1) The magnitude of an electric field became 3.0 ( 10 6 N/C or greater, so that electrical breakdown and thus sparking could occur. (2) The energy of a spark was 150 mJ or greater so that it could ignite the powder explosively. Let us check for the first condition in the powder flow through the plastic pipes. Suppose a stream of negatively charged powder was blown through a cylindrical pipe of radius R $ 5.0 cm. Assume that the powder and its charge were spread uniformly through the pipe with a volume charge density r. (a) Using Gauss’ law, find an ex: pression for the magnitude of the electric field E in the pipe as a function of radial distance r from the pipe center. (b) Does E in: crease or decrease with increasing r? (c) Is E directed radially inward or outward? (d) For r $ 1.1 ( 10 '3 C/m3 (a typical value at the factory), find the maximum E and determine where that maximum field occurs. (e) Could sparking occur, and if so, where? (The story continues with Problem 70 in Chapter 24.) 61 SSM A thin-walled metal spherical shell of radius a has a charge qa. Concentric with it is a thin-walled metal spherical shell of radius b + a and charge qb. Find the electric field at points a distance r from the common center, where (a) r , a, (b) a , r , b, and (c) r + b. (d) Discuss the criterion you would use to determine how the charges are distributed on the inner and outer surfaces of the shells. 62 A particle of charge q $ 1.0 ( 10 '7 C is at the center of a spherical cavity of radius 3.0 cm in a chunk of metal. Find the electric field (a) 1.5 cm from the cavity center and (b) anyplace in the metal. 63 A proton at speed v $ 3.00 ( 10 5 m/s orbits at radius r $ 1.00 cm outside a charged sphere. Find the sphere’s charge.

•••55 A charge distribution that is spherically symmetric but not uniform radially produces an electric field of magnitude E $ Kr 4, directed radially outward from the center of the sphere. Here r is the radial distance from that center, and K is a constant. What is the volume density r of the charge distribution?

64 Equation 23-11 (E $ s/´0) gives the electric field at points near a charged conducting surface. Apply this equation to a conducting sphere of radius r and charge q, and show that the electric field outside the sphere is the same as the field of a charged particle located at the center of the sphere.

Additional Problems : 56 The electric field in a particular space is E $ (x " 2)iˆ N/C, with x in meters. Consider a cylindrical Gaussian surface of radius 20 cm that is coaxial with the x axis. One end of the cylinder is at x $ 0. (a) What is the magnitude of the electric flux through the other end of the cylinder at x $ 2.0 m? (b) What net charge is enclosed within the cylinder?

65 Charge Q is uniformly distributed in a sphere of radius R. (a) What fraction of the charge is contained within the radius r $ R/2.00? (b) What is the ratio of the electric field magnitude at r $ R/2.00 to that on the surface of the sphere? 66 A charged particle causes an electric flux of '750 N ?m2/C to pass through a spherical Gaussian surface of 10.0 cm radius centered on the charge. (a) If the radius of the Gaussian surface were

CHAPTE R 23 GAUSS’ L AW

doubled, how much flux would pass through the surface? (b) What is the charge of the particle? 67 SSM The electric field at point P just outside the outer surface of a hollow spherical conductor of inner radius 10 cm and outer radius 20 cm has magnitude 450 N/C and is directed outward. When a particle of unknown charge Q is introduced into the center of the sphere, the electric field at P is still directed outward but is now 180 N/C. (a) What was the net charge enclosed by the outer surface before Q was introduced? (b) What is charge Q? After Q is introduced, what is the charge on the (c) inner and (d) outer surface of the conductor? 68 The net electric flux through each face of a die (singular of dice) has a magnitude in units of 10 3 N ?m2/C that is exactly equal to the number of spots N on the face (1 through 6). The flux is inward for N odd and outward for N even. What is the net charge inside the die? 69 Figure 23-59 shows, in cross section, three infinitely large nonconducting sheets on which charge is uniformly spread. The surface charge densities are s1 $ "2.00 mC/m2, s2 $ "4.00 mC/m2, and s3 $ '5.00 mC/m2, and distance L $ 1.50 cm. In unitvector notation,what is the net electric field at point P?

P

L/2

σ3 y x

2L

σ2 L

σ1

Figure 23-59 Problem 69. 70 Charge of uniform volume density r $ 3.2 mC/m3 fills a nonconducting solid sphere of radius 5.0 cm. What is the magnitude of the electric field (a) 3.5 cm and (b) 8.0 cm from the sphere’s center? 71 A Gaussian surface in the form of a hemisphere of radius R $ 5.68 cm lies in a uniform electric field of magnitude E $ 2.50 N/C. The surface encloses no net charge. At the (flat) base of the surface, the field is perpendicular to the surface and directed into the surface. What is the flux through (a) the base and (b) the curved portion of the surface? 72 What net charge is enclosed by the Gaussian cube of Problem 2? 73 A nonconducting solid sphere has a uniform volume charge density r. Let : r be the vector from the center of the sphere to a general point P within the sphere. (a) Show that a : the electric field at P is given by E $ rr:/3´0. (Note that the result is independent of the radius of the sphere.) (b) A spherical cavity is hollowed out of the sphere, as shown in Fig. 23Figure 23-60 60. Using superposition concepts, show that Problem 73. the electric field at all points within the cavity : is uniform and equal to E $ ra:/3´0, where a: is the position vector from the center of the sphere to the center of the cavity. 74 A uniform charge density of 500 nC/m3 is distributed throughout a spherical volume of radius 6.00 cm. Consider a cubical Gaussian surface with its center at the center of the sphere. What is the electric flux through this cubical surface if its edge length is (a) 4.00 cm and (b) 14.0 cm? 75 Figure 23-61 shows a Geiger counter, a device used to detect ionizing radiation, which causes ionization of atoms. A thin, posi-

Particle tively charged central wire is surSignal rounded by a concentric, circular, conducting cylindrical shell with an equal – – negative charge, creating a strong ra– – dial electric field. The shell contains a – – – – low-pressure inert gas. A particle of ra– – – – diation entering the device through – the shell wall ionizes a few of the gas – – – e – atoms. The resulting free electrons (e) – – – are drawn to the positive wire. – – Charged – However, the electric field is so intense – wire – – – that, between collisions with gas – atoms, the free electrons gain energy – sufficient to ionize these atoms also. More free electrons are thereby cre- Charged ated, and the process is repeated until cylindrical shell the electrons reach the wire. The re- Figure 23-61 Problem 75. sulting “avalanche” of electrons is collected by the wire, generating a signal that is used to record the passage of the original particle of radiation. Suppose that the radius of the central wire is 25 mm, the inner radius of the shell 1.4 cm, and the length of the shell 16 cm. If the electric field at the shell’s inner wall is 2.9 ( 10 4 N/C, what is the total positive charge on the central wire? ++++ ++++ ++++

684

76 Charge is distributed uniformly throughout the volume of an infinitely long solid cylinder of radius R. (a) Show that, at a distance r , R from the cylinder axis, rr E$ , 2´0 where r is the volume charge density. (b) Write an expression for E when r + R. 77 SSM A spherical conducting shell has a charge of '14 mC on its outer surface and a charged particle in its hollow. If the net charge on the shell is '10 mC, what is the charge (a) on the inner surface of the shell and (b) of the particle? 78 A charge of 6.00 pC is spread uniformly throughout the volume of a sphere of radius r $ 4.00 cm. What is the magnitude of the electric field at a radial distance of (a) 6.00 cm and (b) 3.00 cm? 79 Water in an irrigation ditch of width w $ 3.22 m and depth d $ 1.04 m flows with a speed of 0.207 m/s. The mass flux of the flowing water through an imaginary surface is the product of the water’s density (1000 kg/m3) and its volume flux through that surface. Find the mass flux through the following imaginary surfaces: (a) a surface of area wd, entirely in the water, perpendicular to the flow; (b) a surface with area 3wd/2, of which wd is in the water, perpendicular to the flow; (c) a surface of area wd/2, entirely in the water, perpendicular to the flow; (d) a surface of area wd, half in the water and half out, perpendicular to the flow; (e) a surface of area wd, entirely in the water, with its normal 34.0° from the direction of flow. 80 Charge of uniform surface density 8.00 nC/m2 is distributed over an entire xy plane; charge of uniform surface density 3.00 nC/m2 is distributed over the parallel plane defined by z $ 2.00 m. Determine the magnitude of the electric field at any point having a z coordinate of (a) 1.00 m and (b) 3.00 m. 81 A spherical ball of charged particles has a uniform charge density. In terms of the ball’s radius R, at what radial distances (a) inside and (b) outside the ball is the magnitude of the ball’s electric field equal to 14 of the maximum magnitude of that field?

C

H

A

P

T

E

R

2

4

Electric Potential 24-1 ELECTRIC POTENTIAL Learning Objectives After reading this module, you should be able to . . .

24.01 Identify that the electric force is conservative and thus has an associated potential energy. 24.02 Identify that at every point in a charged object’s electric field, the object sets up an electric potential V, which is a scalar quantity that can be positive or negative depending on the sign of the object’s charge. 24.03 For a charged particle placed at a point in an object’s electric field, apply the relationship between the object’s electric potential V at that point, the particle’s charge q, and the potential energy U of the particle–object system. 24.04 Convert energies between units of joules and electron-volts. 24.05 If a charged particle moves from an initial point to a final point in an electric field, apply the relationships

between the change !V in the potential, the particle’s charge q, the change !U in the potential energy, and the work W done by the electric force. 24.06 If a charged particle moves between two given points in the electric field of a charged object, identify that the amount of work done by the electric force is path independent. 24.07 If a charged particle moves through a change !V in electric potential without an applied force acting on it, relate !V and the change !K in the particle’s kinetic energy. 24.08 If a charged particle moves through a change !V in electric potential while an applied force acts on it, relate !V, the change !K in the particle’s kinetic energy, and the work Wapp done by the applied force.

Key Ideas !U $ q !V $ q(Vf ' Vi).

● The electric potential V at a point P in the electric field of a

charged object is 'W0 U V$ $ , q0 q0 where W0 is the work that would be done by the electric force on a positive test charge q0 were it brought from an infinite distance to P, and U is the electric potential energy that would then be stored in the test charge–object system. ● If a particle with charge q is placed at a point where the

electric potential of a charged object is V, the electric potential energy U of the particle–object system is U $ qV. ● If the particle moves through a potential difference !V, the change in the electric potential energy is

● If a particle moves through a change !V in electric

potential without an applied force acting on it, applying the conservation of mechanical energy gives the change in kinetic energy as !K $ 'q !V. ● If, instead, an applied force acts on the particle, doing work

Wapp, the change in kinetic energy is !K $ 'q !V " Wapp. ● In the special case when !K $ 0, the work of an applied

force involves only the motion of the particle through a potential difference: Wapp $ q !V.

What Is Physics? One goal of physics is to identify basic forces in our world, such as the electric force we discussed in Chapter 21. A related goal is to determine whether a force is conservative—that is, whether a potential energy can be associated with it. The motivation for associating a potential energy with a force is that we can then 685

686

CHAPTE R 24 E LECTR IC POTE NTIAL

+

+

q1

q2

Figure 24-1 Particle 1 is located at point P in the electric field of particle 2.

apply the principle of the conservation of mechanical energy to closed systems involving the force. This extremely powerful principle allows us to calculate the results of experiments for which force calculations alone would be very difficult. Experimentally, physicists and engineers discovered that the electric force is conservative and thus has an associated electric potential energy. In this chapter we first define this type of potential energy and then put it to use. For a quick taste, let’s return to the situation we considered in Chapter 22: In Figure 24-1, particle 1 with positive charge q1 is located at point P near particle 2 with positive charge q2. In Chapter 22 we explained how particle 2 is able : to push on particle 1 without any contact. To account for the force F (which is a : vector quantity), we defined an electric field E (also a vector quantity) that is set up at P by particle 2. That field exists regardless of whether particle 1 is at P. If we choose to place particle 1 there, the push on it is due to charge q1 and that : pre-existing field E . Here is a related problem. If we release particle 1 at P, it begins to move and thus has kinetic energy. Energy cannot appear by magic, so from where does it come? It comes from the electric potential energy U associated with the force between the two particles in the arrangement of Fig. 24-1. To account for the potential energy U (which is a scalar quantity), we define an electric potential V (also a scalar quantity) that is set up at P by particle 2. The electric potential exists regardless of whether particle 1 is at P. If we choose to place particle 1 there, the potential energy of the two-particle system is then due to charge q1 and that preexisting electric potential V. Our goals in this chapter are to (1) define electric potential, (2) discuss how to calculate it for various arrangements of charged particles and objects, and (3) discuss how electric potential V is related to electric potential energy U.

Electric Potential and Electric Potential Energy +

+ + + Test charge q0 + + at point P + + + + + + Charged object

(a)

The rod sets up an electric potential, which determines the potential energy. P

+ + + + + + + + + ++

Electric potential V at point P

(b)

Figure 24-2 (a) A test charge has been brought in from infinity to point P in the electric field of the rod. (b) We define an electric potential V at P based on the potential energy of the configuration in (a).

We are going to define the electric potential (or potential for short) in terms of electric potential energy, so our first job is to figure out how to measure that potential energy. Back in Chapter 8, we measured gravitational potential energy U of an object by (1) assigning U $ 0 for a reference configuration (such as the object at table level) and (2) then calculating the work W the gravitational force does if the object is moved up or down from that level. We then defined the potential energy as being U $ 'W

(potential energy).

(24-1)

Let’s follow the same procedure with our new conservative force, the electric force. In Fig. 24-2a, we want to find the potential energy U associated with a positive test charge q0 located at point P in the electric field of a charged rod. First, we need a reference configuration for which U $ 0. A reasonable choice is for the test charge to be infinitely far from the rod, because then there is no interaction with the rod. Next, we bring the test charge in from infinity to point P to form the configuration of Fig. 24-2a. Along the way, we calculate the work done by the electric force on the test charge. The potential energy of the final configuration is then given by Eq. 24-1, where W is now the work done by the electric force. Let’s use the notation W0 to emphasize that the test charge is brought in from infinity. The work and thus the potential energy can be positive or negative depending on the sign of the rod’s charge. Next, we define the electric potential V at P in terms of the work done by the electric force and the resulting potential energy: V$

'W0 U $ q0 q0

(electric potential).

(24-2)

24-1 E LECTR IC POTE NTIAL

That is, the electric potential is the amount of electric potential energy per unit charge when a positive test charge is brought in from infinity. The rod sets up this potential V at P regardless of whether the test charge (or anything else) happens to be there (Fig. 24-2b). From Eq. 24-2 we see that V is a scalar quantity (because there is no direction associated with potential energy or charge) and can be positive or negative (because potential energy and charge have signs). Repeating this procedure we find that an electric potential is set up at every point in the rod’s electric field. In fact, every charged object sets up electric potential V at points throughout its electric field. If we happen to place a particle with, say, charge q at a point where we know the pre-existing V, we can immediately find the potential energy of the configuration: (electric potential energy) $ (particle’s charge) or

potential energy $, # electricunit charge

U $ qV,

(24-3)

where q can be positive or negative. Two Cautions. (1) The (now very old) decision to call V a potential was unfortunate because the term is easily confused with potential energy. Yes, the two quantities are related (that is the point here) but they are very different and not interchangeable. (2) Electric potential is a scalar, not a vector. (When you come to the homework problems, you will rejoice on this point.) Language. A potential energy is a property of a system (or configuration) of objects, but sometimes we can get away with assigning it to a single object. For example, the gravitational potential energy of a baseball hit to outfield is actually a potential energy of the baseball–Earth system (because it is associated with the force between the baseball and Earth). However, because only the baseball noticeably moves (its motion does not noticeably affect Earth), we might assign the gravitational potential energy to it alone. In a similar way, if a charged particle is placed in an electric field and has no noticeable effect on the field (or the charged object that sets up the field), we usually assign the electric potential energy to the particle alone. Units. The SI unit for potential that follows from Eq. 24-2 is the joule per coulomb. This combination occurs so often that a special unit, the volt (abbreviated V), is used to represent it. Thus, 1 volt $ 1 joule per coulomb. With two unit conversions, we can now switch the unit for electric field from newtons per coulomb to a more conventional unit: 1 N/C $

V 1J #1 NC $ # 11J/C $ # 1 N#m $

$ 1 V/m. The conversion factor in the second set of parentheses comes from our definition of volt given above; that in the third set of parentheses is derived from the definition of the joule. From now on, we shall express values of the electric field in volts per meter rather than in newtons per coulomb.

Motion Through an Electric Field Change in Electric Potential. If we move from an initial point i to a second point f in the electric field of a charged object, the electric potential changes by !V $ Vf ' Vi.

687

688

CHAPTE R 24 E LECTR IC POTE NTIAL

If we move a particle with charge q from i to f, then, from Eq. 24-3, the potential energy of the system changes by !U $ q !V $ q(Vf ' Vi).

(24-4)

The change can be positive or negative, depending on the signs of q and !V. It can also be zero, if there is no change in potential from i to f (the points have the same value of potential). Because the electric force is conservative, the change in potential energy !U between i and f is the same for all paths between those points (it is path independent). Work by the Field. We can relate the potential energy change !U to the work W done by the electric force as the particle moves from i to f by applying the general relation for a conservative force (Eq. 8-1): W $ '!U

(work, conservative force).

(24-5)

Next, we can relate that work to the change in the potential by substituting from Eq. 24-4: W $ '!U $ 'q !V $ 'q(Vf ' Vi). (24-6) Up until now, we have always attributed work to a force but here can also say that W is the work done on the particle by the electric field (because it, of course, produces the force). The work can be positive, negative, or zero. Because !U between any two points is path independent, so is the work W done by the field. (If you need to calculate work for a difficult path, switch to an easier path—you get the same result.) Conservation of Energy. If a charged particle moves through an electric field with no force acting on it other than the electric force due to the field, then the mechanical energy is conserved. Let’s assume that we can assign the electric potential energy to the particle alone. Then we can write the conservation of mechanical energy of the particle that moves from point i to point f as Ui " Ki $ Uf " Kf , or

!K $ '!U.

(24-7) (24-8)

Substituting Eq. 24-4, we find a very useful equation for the change in the particle’s kinetic energy as a result of the particle moving through a potential difference: !K $ 'q !V $ 'q(Vf ' Vi).

(24-9)

Work by an Applied Force. If some force in addition to the electric force acts on the particle, we say that the additional force is an applied force or external force, which is often attributed to an external agent. Such an applied force can do work on the particle, but the force may not be conservative and thus, in general, we cannot associate a potential energy with it. We account for that work Wapp by modifying Eq. 24-7: (initial energy) " (work by applied force) $ (final energy) or

Ui " Ki " Wapp $ Uf " Kf .

(24-10)

Rearranging and substituting from Eq. 24-4, we can also write this as !K $ '!U " Wapp $ 'q !V " Wapp.

(24-11)

The work by the applied force can be positive, negative, or zero, and thus the energy of the system can increase, decrease, or remain the same. In the special case where the particle is stationary before and after the move, the kinetic energy terms in Eqs. 24-10 and 24-11 are zero and we have Wapp $ q !V

(for Ki $ Kf).

(24-12)

In this special case, the work Wapp involves the motion of the particle through the potential difference !V and not a change in the particle’s kinetic energy.

689

24-1 E LECTR IC POTE NTIAL

By comparing Eqs. 24-6 and 24-12, we see that in this special case, the work by the applied force is the negative of the work by the field: Wapp $ 'W

(for Ki $ Kf).

(24-13)

Electron-volts. In atomic and subatomic physics, energy measures in the SI unit of joules often require awkward powers of ten. A more convenient (but nonSI unit) is the electron-volt (eV), which is defined to be equal to the work required to move a single elementary charge e (such as that of an electron or proton) through a potential difference !V of exactly one volt. From Eq. 24-6, we see that the magnitude of this work is q !V.Thus, 1 eV $ e(1 V) $ (1.602 ( 10'19 C)(1 J/C) $ 1.602 ( 10'19 J.

(24-14)

Checkpoint 1 E

In the figure, we move a proton from point i to point f in a uniform electric field. Is positive or negative work done by (a) the electric field and (b) our force? (c) Does the electric potential energy increase or decrease? (d) Does the proton move to a point of higher or lower electric potential?

f

+

i

Sample Problem 24.01 Work and potential energy in an electric field Electrons are continually being knocked out of air molecules in the atmosphere by cosmic-ray particles coming in from space. Once released, each electron experiences an : : electric force F due to the electric field E that is produced in the atmosphere by charged particles already on Earth. Near Earth’s surface the electric field has the magnitude E $ 150 N/C and is directed downward. What is the change !U in the electric potential energy of a released electron when the electric force causes it to move vertically upward through a distance d $ 520 m (Fig. 24-3)? Through what potential change does the electron move? KEY IDEAS

E

d

F e



Figure 24-3 An electron in the atmosphere is moved upward through : : : displacement d by an electric force F due to an electric field E . :

:

where u is the angle between the directions of E and d . : : The field E is directed downward and the displacement d is directed upward; so u $ 180%. We can now evaluate the work as W $ ('1.6 ( 10'19 C)(150 N/C)(520 m) cos 180%

(1) The change !U in the electric potential energy of the electron is related to the work W done on the electron by the electric field. Equation 24-5 (W $ '!U) gives the relation. : (2) The work done by a constant force F on a particle under: going a displacement d is : :

W $ F " d. (3) The electric force and the electric field are related by the : : force equation F $ qE, where here q is the charge of an electron ($ '1.6 ( 10'19 C). Calculations: Substituting the force equation into the work equation and taking the dot product yield :

:

W $ qE " d $ qEd cos u,

$ 1.2 ( 10'14 J. Equation 24-5 then yields !U $ 'W $ '1.2 ( 10'14 J.

(Answer)

This result tells us that during the 520 m ascent, the electric potential energy of the electron decreases by 1.2 ( 10'14 J. To find the change in electric potential, we apply Eq. 24-4: !V $

'1.2 ( 10'14 J !U $ 'q '1.6 ( 10'19 C

$ 4.5 ( 104 V $ 45 kV.

(Answer)

This tells us that the electric force does work to move the electron to a higher potential.

Additional examples, video, and practice available at WileyPLUS

690

CHAPTE R 24 E LECTR IC POTE NTIAL

24-2 EQUIPOTENTIAL SURFACES AND THE ELECTRIC FIELD Learning Objectives After reading this module, you should be able to . . .

24.09 Identify an equipotential surface and describe how it is related to the direction of the associated electric field. 24.10 Given an electric field as a function of position, calculate the change in potential !V from an initial point to a final point by choosing a path between the points and : integrating the dot product of the field E and a length element d : s along the path.

24.11 For a uniform electric field, relate the field magnitude E and the separation !x and potential difference !V between adjacent equipotential lines. 24.12 Given a graph of electric field E versus position along an axis, calculate the change in potential !V from an initial point to a final point by graphical integration. 24.13 Explain the use of a zero-potential location.

Key Ideas ● The points on an equipotential surface all have the same electric potential. The work done on a test charge in moving it from one such surface to another is independent of the locations of the initial and final points on these surfaces and of the : path that joins the points. The electric field E is always directed perpendicularly to corresponding equipotential surfaces. ● The electric potential difference between two points i and f is

Vf ' Vi $ '

"

f

i

:

E " d: s,

where the integral is taken over any path connecting the points. If the integration is difficult along any particular path,

we can choose a different path along which the integration might be easier. ● If we choose Vi $ 0, we have, for the potential at a particular point, V$'

"

f

i

:

E " d: s.

● In a uniform field of magnitude E, the change in potential

from a higher equipotential surface to a lower one, separated by distance !x, is !V $ 'E !x.

Equipotential Surfaces Adjacent points that have the same electric potential form an equipotential surface, which can be either an imaginary surface or a real, physical surface. No net work W is done on a charged particle by an electric field when the particle moves between two points i and f on the same equipotential surface. This follows from Eq. 24-6, which tells us that W must be zero if Vf $ Vi . Because of the path independence of work (and thus of potential energy and potential), W $ 0 for any path connecting points i and f on a given equipotential surface regardless of whether that path lies entirely on that surface. Figure 24-4 shows a family of equipotential surfaces associated with the electric field due to some distribution of charges. The work done by the electric field on a charged particle as the particle moves from one end to the other of paths Equal work is done along these paths between the same surfaces.

No work is done along this path on an equipotential surface.

V1 III

I

Figure 24-4 Portions of four equipotential surfaces at electric potentials V1 $ 100 V, V2 $ 80 V, V3 $ 60 V, and V4 $ 40 V. Four paths along which a test charge may move are shown. Two electric field lines are also indicated.

IV

V2 V3

II

No work is done along this path that returns to the same surface.

V4

24-2 EQU I POTE NTIAL SU R FACES AN D TH E E LECTR IC FI E LD

I and II is zero because each of these paths begins and ends on the same equipotential surface and thus there is no net change in potential. The work done as the charged particle moves from one end to the other of paths III and IV is not zero but has the same value for both these paths because the initial and final potentials are identical for the two paths; that is, paths III and IV connect the same pair of equipotential surfaces. From symmetry, the equipotential surfaces produced by a charged particle or a spherically symmetrical charge distribution are a family of concentric spheres. For a uniform electric field, the surfaces are a family of planes perpendicular to the field lines. In fact, equipotential surfaces are always perpendicular to electric : : field lines and thus to E , which is always tangent to these lines. If E were not perpendicular to an equipotential surface, it would have a component lying along that surface. This component would then do work on a charged particle as it moved along the surface. However, by Eq. 24-6 work cannot be done if the : surface is truly an equipotential surface; the only possible conclusion is that E must be everywhere perpendicular to the surface. Figure 24-5 shows electric field lines and cross sections of the equipotential surfaces for a uniform electric field and for the field associated with a charged particle and with an electric dipole.

Equipotential surface Field line

(a)

Calculating the Potential from the Field We can calculate the potential difference between any two points i and f in an : electric field if we know the electric field vector E all along any path connecting those points. To make the calculation, we find the work done on a positive test charge by the field as the charge moves from i to f, and then use Eq. 24-6. Consider an arbitrary electric field, represented by the field lines in Fig. 24-6, and a positive test charge q0 that moves along the path shown from point i to : point f. At any point on the path, an electric force q0 E acts on the charge as it moves through a differential displacement d : s . From Chapter 7, we know that the : differential work dW done on a particle by a force F during a displacement d : s is given by the dot product of the force and the displacement: :

+

(b)

(24-15)

dW $ F " d : s. :

691

+

:

For the situation of Fig. 24-6, F $ q0 E and Eq. 24-15 becomes :

(24-16)

dW $ q0E " d : s.

To find the total work W done on the particle by the field as the particle moves from point i to point f, we sum — via integration — the differential works done on the charge as it moves through all the displacements d : s along the path: W $ q0

"

f

:

(24-17)

E " d: s.

i

(c)

If we substitute the total work W from Eq. 24-17 into Eq. 24-6, we find

"

Vf ' Vi $ '

f

i

:

E " d: s.

(24-18)

Path

Field line

i

Figure 24-6 A test charge q0 moves from point i to point f along the path shown in a nonuniform electric field. During a displacement d s:, : an electric force q0 E acts on the test charge. This force points in the direction of the field line at the location of the test charge.

q0

+

ds f q0E

Figure 24-5 Electric field lines (purple) and cross sections of equipotential surfaces (gold) for (a) a uniform electric field, (b) the field due to a charged particle, and (c) the field due to an electric dipole.

692

CHAPTE R 24 E LECTR IC POTE NTIAL Path

i

Figure 24-7 We move between points i and f, between adjacent equipotential lines in a : uniform electric field E , parallel to a field line.

f

x

Field line Higher potential

E !x

Lower potential

Thus, the potential difference Vf ' Vi between any two points i and f in an electric field is equal to the negative of the line integral (meaning the integral along a : particular path) of E " d : s from i to f. However, because the electric force is conservative, all paths (whether easy or difficult to use) yield the same result. Equation 24-18 allows us to calculate the difference in potential between any two points in the field. If we set potential Vi $ 0, then Eq. 24-18 becomes

"

V$'

f

i

:

E " d: s,

(24-19)

in which we have dropped the subscript f on Vf . Equation 24-19 gives us the potential V at any point f in the electric field relative to the zero potential at point i. If we let point i be at infinity, then Eq. 24-19 gives us the potential V at any point f relative to the zero potential at infinity. Uniform Field. Let’s apply Eq. 24-18 for a uniform field as shown in Fig. 24-7. We start at point i on an equipotential line with potential Vi and move to point f on an equipotential line with a lower potential Vf. The separation between the two equipotential lines is !x. Let’s also move along a path that is parallel to the : electric field E (and thus perpendicular to the equipotential lines). The angle be: tween E and d : s in Eq. 24-18 is zero, and the dot product gives us :

s $ E ds cos 0 $ E ds. E " d: Because E is constant for a uniform field, Eq. 24-18 becomes Vf ' Vi $ 'E

"

f

i

ds.

(24-20)

The integral is simply an instruction for us to add all the displacement elements ds from i to f, but we already know that the sum is length !x. Thus we can write the change in potential Vf ' Vi in this uniform field as !V $ 'E !x

(uniform field).

(24-21)

This is the change in voltage !V between two equipotential lines in a uniform field of magnitude E, separated by distance !x. If we move in the direction of the field by distance !x, the potential decreases. In the opposite direction, it increases. The electric field vector points from higher potential toward lower potential.

1

Checkpoint 2

2

The figure here shows a family of parallel equipotential surfaces (in cross section) and five paths along which we shall move an electron from one surface to another. (a) What is the direction of the electric field associated with the surfaces? (b) For each path, is the work we do positive, negative, or zero? (c) Rank the paths according to the work we do, greatest first.

3

4 5

90 V

80 V

70 V

60 V

50 V

40 V

693

24-2 EQU I POTE NTIAL SU R FACES AN D TH E E LECTR IC FI E LD

Sample Problem 24.02 Finding the potential change from the electric field (a) Figure 24-8a shows two points i and f in a uniform elec: tric field E . The points lie on the same electric field line (not shown) and are separated by a distance d. Find the potential difference Vf ' Vi by moving a positive test charge q0 from i to f along the path shown, which is parallel to the field direction.

:

ment d : s of the test charge is perpendicular to E . Thus, the : : angle u between E and d : s is 90%, and the dot product E " d : s is 0. Equation 24-18 then tells us that points i and c are at the same potential: Vc ' Vi $ 0. Ah, we should have seen this coming. The points are on the same equipotential surface, which is perpendicular to the electric field lines. For line cf we have u $ 45% and, from Eq. 24-18,

"

KEY IDEA

Vf ' Vi $ '

We can find the potential difference between any two points : s along a path conin an electric field by integrating E " d : necting those two points according to Eq. 24-18. Calculations: We have actually already done the calculation for such a path in the direction of an electric field line in a uniform field when we derived Eq. 24-21.With slight changes in notation, Eq. 24-21 gives us

(b) Now find the potential difference Vf ' Vi by moving the positive test charge q0 from i to f along the path icf shown in Fig. 24-8b. Calculations: The Key Idea of (a) applies here too, except now we move the test charge along a path that consists of two lines: ic and cf. At all points along line ic, the displace-

The electric field points from higher potential to lower potential.

i Higher potential

c

:

"

f

E " d: s $'

$ 'E(cos 45%)

"

f

c

E(cos 45%) ds

c

ds.

The integral in this equation is just the length of line cf; from Fig. 24-8b, that length is d/cos 45%. Thus, Vf ' Vi $ 'E(cos 45%)

(Answer)

Vf ' Vi $ 'Ed.

f

d $ 'Ed. cos 45%

This is the same result we obtained in (a), as it must be; the potential difference between two points does not depend on the path connecting them. Moral: When you want to find the potential difference between two points by moving a test charge between them, you can save time and work by choosing a path that simplifies the use of Eq. 24-18.

The field is perpendicular to this ic path, so there is no change in the potential.

i

q0

+

ds

c 45°

E q0 ds d

q0

+ ds

d

+

45°

E

f

E

The field has a component along this cf path, so there is a change in the potential.

f Lower potential

(a)

(Answer)

(b)

Figure 24-8 (a) A test charge q0 moves in a straight line from point i to point f, along the direction of a uniform external electric field. (b) Charge q0 moves along path icf in the same electric field.

Additional examples, video, and practice available at WileyPLUS

694

CHAPTE R 24 E LECTR IC POTE NTIAL

24-3 POTENTIAL DUE TO A CHARGED PARTICLE Learning Objectives After reading this module, you should be able to . . .

24.14 For a given point in the electric field of a charged particle, apply the relationship between the electric potential V, the charge of the particle q, and the distance r from the particle. 24.15 Identify the correlation between the algebraic signs of the potential set up by a particle and the charge of the particle. 24.16 For points outside or on the surface of a spherically

symmetric charge distribution, calculate the electric potential as if all the charge is concentrated as a particle at the center of the sphere. 24.17 Calculate the net potential at any given point due to several charged particles, identifying that algebraic addition is used, not vector addition. 24.18 Draw equipotential lines for a charged particle.

Key Ideas ● The electric potential due to a single charged particle at a distance r from that charged particle is

1 q , V$ 4p´0 r where V has the same sign as q.

● The potential due to a collection of charged particles is n

' Vi $ i$1

V$

1 4p´0

n

qi . ri

' i$1

Thus, the potential is the algebraic sum of the individual potentials, with no consideration of directions.

Potential Due to a Charged Particle To find the potential of the charged particle, we move this test charge out to infinity. E q0

We now use Eq. 24-18 to derive, for the space around a charged particle, an expression for the electric potential V relative to the zero potential at infinity. Consider a point P at distance R from a fixed particle of positive charge q (Fig. 24-9). To use Eq. 24-18, we imagine that we move a positive test charge q0 from point P to infinity. Because the path we take does not matter, let us choose the simplest one — a line that extends radially from the fixed particle through P to infinity. To use Eq. 24-18, we must evaluate the dot product

ds

:

(24-22)

E " d: s $ E cos 1 ds.

+ :

P r

The electric field E in Fig. 24-9 is directed radially outward from the fixed particle.Thus, the differential displacement d : s of the test particle along its path has : the same direction as E . That means that in Eq. 24-22, angle u $ 0 and cos u $ 1. Because the path is radial, let us write ds as dr.Then, substituting the limits R and 0, we can write Eq. 24-18 as

"

Vf ' Vi $ '

R

0

R

(24-23)

E dr.

Next, we set Vf $ 0 (at 0) and Vi $ V (at R). Then, for the magnitude of the electric field at the site of the test charge, we substitute from Eq. 22-3: E$

+ q

q 1 . 4p´0 r 2

(24-24)

With these changes, Eq. 24-23 then gives us Figure 24-9 The particle with positive charge : q produces an electric field E and an electric potential V at point P. We find the potential by moving a test charge q0 from P to infinity. The test charge is shown at distance r from the particle, during differential displacement d : s.

0'V$' $'

q 4p´0

"

0

R

1 q . 4p´0 R

1 q dr $ r2 4p´0

( 1r )

0 R

(24-25)

24-3 POTE NTIAL DU E TO A CHARG E D PARTICLE

Solving for V and switching R to r, we then have V$

695

V(r)

1 q 4p´0 r

(24-26)

as the electric potential V due to a particle of charge q at any radial distance r from the particle. Although we have derived Eq. 24-26 for a positively charged particle, the derivation holds also for a negatively charged particle, in which case, q is a negative quantity. Note that the sign of V is the same as the sign of q: y

A positively charged particle produces a positive electric potential. A negatively charged particle produces a negative electric potential.

Figure 24-10 shows a computer-generated plot of Eq. 24-26 for a positively charged particle; the magnitude of V is plotted vertically. Note that the magnitude increases as r : 0. In fact, according to Eq. 24-26, V is infinite at r $ 0, although Fig. 24-10 shows a finite, smoothed-off value there. Equation 24-26 also gives the electric potential either outside or on the external surface of a spherically symmetric charge distribution. We can prove this by using one of the shell theorems of Modules 21-1 and 23-6 to replace the actual spherical charge distribution with an equal charge concentrated at its center. Then the derivation leading to Eq. 24-26 follows, provided we do not consider a point within the actual distribution.

Potential Due to a Group of Charged Particles We can find the net electric potential at a point due to a group of charged particles with the help of the superposition principle. Using Eq. 24-26 with the plus or minus sign of the charge included, we calculate separately the potential resulting from each charge at the given point. Then we sum the potentials. Thus, for n charges, the net potential is V$

n

' Vi i$1

$

1 4p´0

n

' i$1

qi ri

(n charged particles).

(24-27)

Here qi is the value of the ith charge and ri is the radial distance of the given point from the ith charge. The sum in Eq. 24-27 is an algebraic sum, not a vector sum like the sum that would be used to calculate the electric field resulting from a group of charged particles. Herein lies an important computational advantage of potential over electric field: It is a lot easier to sum several scalar quantities than to sum several vector quantities whose directions and components must be considered.

Checkpoint 3 The figure here shows three arrangements of two protons. Rank the arrangements according to the net electric potential produced at point P by the protons, greatest first. d

D

D

P

P d

d

D P

(a)

(b)

(c)

x

Figure 24-10 A computer-generated plot of the electric potential V(r) due to a positively charged particle located at the origin of an xy plane. The potentials at points in the xy plane are plotted vertically. (Curved lines have been added to help you visualize the plot.) The infinite value of V predicted by Eq. 24-26 for r $ 0 is not plotted.

696

CHAPTE R 24 E LECTR IC POTE NTIAL

Sample Problem 24.03 Net potential of several charged particles What is the electric potential at point P, located at the center of the square of charged particles shown in Fig. 24-11a? The distance d is 1.3 m, and the charges are q1 $ "12 nC,

q3 $ "31 nC,

q2 $ '24 nC,

q4 $ "17 nC.

q1

q2

d

P

q2

P

d V = 350 V

q3

4

' Vi $ i$1

1 4p´0

# qr

1

"

$

q2 q3 q4 . " " r r r

The distance r is d/1 2, which is 0.919 m, and the sum of the charges is

The electric potential V at point P is the algebraic sum of the electric potentials contributed by the four particles.

d

Calculations: From Eq. 24-27, we have V$

KEY IDEA

q1

(Because electric potential is a scalar, the orientations of the particles do not matter.)

d

q4

(a)

q4

q3 (b)

Figure 24-11 (a) Four charged particles. (b) The closed curve is a (roughly drawn) cross section of the equipotential surface that contains point P.

q1 " q2 " q3 " q4 $ (12 ' 24 " 31 " 17) ( 10'9 C $ 36 ( 10'9 C. Thus,

(8.99 ( 109 N#m2/C2)(36 ( 10'9 C) 0.919 m % 350 V. (Answer)

V$

Close to any of the three positively charged particles in Fig. 24-11a, the potential has very large positive values. Close to the single negative charge, the potential has very large negative values. Therefore, there must be points within the square that have the same intermediate potential as that at point P. The curve in Fig. 24-11b shows the intersection of the plane of the figure with the equipotential surface that contains point P.

Sample Problem 24.04 Potential is not a vector, orientation is irrelevant (a) In Fig. 24-12a, 12 electrons (of charge 'e) are equally spaced and fixed around a circle of radius R. Relative to V $ 0 at infinity, what are the electric potential and electric field at the center C of the circle due to these electrons? KEY IDEAS (1) The electric potential V at C is the algebraic sum of the electric potentials contributed by all the electrons. Because Potential is a scalar and orientation is irrelevant.

electric potential is a scalar, the orientations of the electrons do not matter. (2) The electric field at C is a vector quantity and thus the orientation of the electrons is important. Calculations: Because the electrons all have the same negative charge 'e and are all the same distance R from C, Eq. 24-27 gives us e 1 (Answer) (24-28) . V $ '12 4p´0 R Because of the symmetry of the arrangement in Fig. 24-12a, the electric field vector at C due to any given electron is canceled by the field vector due to the electron that is diametrically opposite it. Thus, at C, :

E $ 0. R R

C

120°

(a)

C

(b)

Figure 24-12 (a) Twelve electrons uniformly spaced around a circle. (b) The electrons nonuniformly spaced along an arc of the original circle.

(Answer)

(b) The electrons are moved along the circle until they are nonuniformly spaced over a 120% arc (Fig. 24-12b). At C, find the electric potential and describe the electric field. Reasoning: The potential is still given by Eq. 24-28, because the distance between C and each electron is unchanged and orientation is irrelevant. The electric field is no longer zero, however, because the arrangement is no longer symmetric. A net field is now directed toward the charge distribution.

Additional examples, video, and practice available at WileyPLUS

24-4 POTE NTIAL DU E TO AN E LECTR IC DI POLE

697

24-4 POTENTIAL DUE TO AN ELECTRIC DIPOLE Learning Objectives After reading this module, you should be able to . . .

24.19 Calculate the potential V at any given point due to an electric dipole, in terms of the magnitude p of the dipole moment or the product of the charge separation d and the magnitude q of either charge.

24.20 For an electric dipole, identify the locations of positive potential, negative potential, and zero potential. 24.21 Compare the decrease in potential with increasing distance for a single charged particle and an electric dipole.

Key Idea ● At a distance r from an electric dipole with dipole moment magnitude p

V$

$ qd, the electric potential of the dipole is

1 p cos u 4p´0 r2

for r 2 d; the angle u lies between the dipole moment vector and a line extending from the dipole midpoint to the point of measurement. z

Potential Due to an Electric Dipole

P

Now let us apply Eq. 24-27 to an electric dipole to find the potential at an arbitrary point P in Fig. 24-13a. At P, the positively charged particle (at distance r(")) sets up potential V(") and the negatively charged particle (at distance r(')) sets up potential V('). Then the net potential at P is given by Eq. 24-27 as V$ $

2

' Vi $ V(") " V(') $ i$1

1 4p´0

# rq

"

(")

'q r(')

$ (24-29)

Naturally occurring dipoles — such as those possessed by many molecules — are quite small; so we are usually interested only in points that are relatively far from the dipole, such that r 2 d, where d is the distance between the charges and r is the distance from the dipole’s midpoint to P. In that case, we can approximate the two lines to P as being parallel and their length difference as being the leg of a right triangle with hypotenuse d (Fig. 24-13b). Also, that difference is so small that the product of the lengths is approximately r 2. Thus, and

r(–) – r(+) –q

(a)

z

r(+)

r(')r(") % r 2.

+q

q d cos 1 , 4p´0 r2

1 p cos u 4p´0 r2

(electric dipole),

r(–)

+

d

θ

where u is measured from the dipole axis as shown in Fig. 24-13a. We can now write V as V$

+

d O

If we substitute these quantities into Eq. 24-29, we can approximate V to be V$

r(–)

r +q

θ

q r(') ' r(") . 4p´0 r(')r(")

r(') ' r(") % d cos u

r(+)

(24-30)

in which p ($ qd) is the magnitude of the electric dipole moment p: defined in Module 22-3. The vector p: is directed along the dipole axis, from the negative to the positive charge. (Thus, u is measured from the direction of p:.) We use this vector to report the orientation of an electric dipole.

(b)

r(–) – r(+)

–q

Figure 24-13 (a) Point P is a distance r from the midpoint O of a dipole. The line OP makes an angle u with the dipole axis. (b) If P is far from the dipole, the lines of lengths r(") and r(') are approximately parallel to the line of length r, and the dashed black line is approximately perpendicular to the line of length r(').

698

CHAPTE R 24 E LECTR IC POTE NTIAL

The electric field shifts the positive and negative charges, creating a dipole. E

+

p

(a)

+

Checkpoint 4 Suppose that three points are set at equal (large) distances r from the center of the dipole in Fig. 24-13: Point a is on the dipole axis above the positive charge, point b is on the axis below the negative charge, and point c is on a perpendicular bisector through the line connecting the two charges. Rank the points according to the electric potential of the dipole there, greatest (most positive) first.

Induced Dipole Moment

(b)

Figure 24-14 (a) An atom, showing the positively charged nucleus (green) and the negatively charged electrons (gold shading). The centers of positive and negative charge coincide. (b) If the atom is : placed in an external electric field E , the electron orbits are distorted so that the centers of positive and negative charge no longer coincide. An induced dipole moment p: appears. The distortion is greatly exaggerated here.

Many molecules, such as water, have permanent electric dipole moments. In other molecules (called nonpolar molecules) and in every isolated atom, the centers of the positive and negative charges coincide (Fig. 24-14a) and thus no dipole moment is set up. However, if we place an atom or a nonpolar molecule in an external electric field, the field distorts the electron orbits and separates the centers of positive and negative charge (Fig. 24-14b). Because the electrons are negatively charged, they tend to be shifted in a direction opposite the field. This shift sets up a dipole moment p: that points in the direction of the field. This dipole moment is said to be induced by the field, and the atom or molecule is then said to be polarized by the field (that is, it has a positive side and a negative side). When the field is removed, the induced dipole moment and the polarization disappear.

24-5 POTENTIAL DUE TO A CONTINUOUS CHARGE DISTRIBUTION Learning Objective After reading this module, you should be able to . . .

24.22 For charge that is distributed uniformly along a line or over a surface, find the net potential at a given point by splitting the distribution up into charge elements and summing (by integration) the potential due to each one.

Key Ideas ● For a continuous distribution of charge (over an extended object), the potential is found by (1) dividing the distribution into charge elements dq that can be treated as particles and then (2) summing the potential due to each element by integrating over the full distribution:

1 V$ 4p)0

"

dq . r

● In order to carry out the integration, dq is replaced with the

product of either a linear charge density l and a length element (such as dx), or a surface charge density s and area element (such as dx dy). ● In some cases where the charge is symmetrically distrib-

uted, a two-dimensional integration can be reduced to a onedimensional integration.

Potential Due to a Continuous Charge Distribution When a charge distribution q is continuous (as on a uniformly charged thin rod or disk), we cannot use the summation of Eq. 24-27 to find the potential V at a point P. Instead, we must choose a differential element of charge dq, determine the potential dV at P due to dq, and then integrate over the entire charge distribution. Let us again take the zero of potential to be at infinity. If we treat the element of charge dq as a particle, then we can use Eq. 24-26 to express the potential dV at point P due to dq: dV $

1 dq 4p´0 r

(positive or negative dq).

(24-31)

Here r is the distance between P and dq. To find the total potential V at P, we

24-5 POTE NTIAL DU E TO A CONTI N UOUS CHARG E DISTR I B UTION

699

integrate to sum the potentials due to all the charge elements: V$

"

dV $

1 4p´0

"

dq . r

(24-32)

The integral must be taken over the entire charge distribution. Note that because the electric potential is a scalar, there are no vector components to consider in Eq. 24-32. We now examine two continuous charge distributions, a line and a disk.

Line of Charge In Fig. 24-15a, a thin nonconducting rod of length L has a positive charge of uniform linear density l. Let us determine the electric potential V due to the rod at point P, a perpendicular distance d from the left end of the rod. We consider a differential element dx of the rod as shown in Fig. 24-15b. This (or any other) element of the rod has a differential charge of (24-33)

dq $ l dx.

This element produces an electric potential dV at point P, which is a distance r $ (x2 " d 2)1/2 from the element (Fig. 24-15c). Treating the element as a point charge, we can use Eq. 24-31 to write the potential dV as dV $

1 dq 1 l dx $ . 2 4p´0 r 4p´0 (x " d 2)1/2

(24-34)

A P

This charged rod is obviously not a particle.

P

d

P

But we can treat this element as a particle.

d x

(b)

P

Our job is to add the potentials due to all d=r the elements.

x

(c)

P d

r

x

Here is the leftmost element. (d )

dx x

(a)

x=0

r

x

dx

L

d

Here is how to find distance r from the element.

Here is the rightmost element.

x=L

x

(e )

Figure 24-15 (a) A thin, uniformly charged rod produces an electric potential V at point P. (b) An element can be treated as a particle. (c) The potential at P due to the element depends on the distance r. We need to sum the potentials due to all the elements, from the left side (d) to the right side (e).

700

CHAPTE R 24 E LECTR IC POTE NTIAL

Since the charge on the rod is positive and we have taken V $ 0 at infinity, we know from Module 24-3 that dV in Eq. 24-34 must be positive. We now find the total potential V produced by the rod at point P by integrating Eq. 24-34 along the length of the rod, from x $ 0 to x $ L (Figs. 24-15d and e), using integral 17 in Appendix E. We find V$

$

$

$

"

"

L

dV $

0

1 l dx 2 4p´0 (x " d 2)1/2

l 4p´0

"

l 4p´0

(#

l 4p´0

(ln#L " (L " d ) $ ' ln d).

L

0

dx (x " d 2)1/2 2

ln x " (x2 " d 2)1/2

$)

L 0

2

2 1/2

We can simplify this result by using the general relation ln A ' ln B $ ln(A/B). We then find L " (L2 " d 2)1/2 l (24-35) ln . V$ 4p´0 d

(

)

Because V is the sum of positive values of dV, it too is positive, consistent with the logarithm being positive for an argument greater than 1.

Charged Disk In Module 22-5, we calculated the magnitude of the electric field at points on the central axis of a plastic disk of radius R that has a uniform charge density s on one surface. Here we derive an expression for V(z), the electric potential at any point on the central axis. Because we have a circular distribution of charge on the disk, we could start with a differential element that occupies angle du and radial distance dr. We would then need to set up a two-dimensional integration. However, let’s do something easier. In Fig. 24-16, consider a differential element consisting of a flat ring of radius R. and radial width dR.. Its charge has magnitude

P

dq $ s(2pR.)(dR.), r

z Every charge element

in the ring contributes to the potential at P.

in which (2pR.)(dR.) is the upper surface area of the ring. All parts of this charged element are the same distance r from point P on the disk’s axis. With the aid of Fig. 24-16, we can use Eq. 24-31 to write the contribution of this ring to the electric potential at P as dV $

R' R

dR'

Figure 24-16 A plastic disk of radius R, charged on its top surface to a uniform surface charge density s. We wish to find the potential V at point P on the central axis of the disk.

1 s(2pR.)(dR.) 1 dq . $ 4p´0 r 4p´0 2z2 " R.2

(24-36)

We find the net potential at P by adding (via integration) the contributions of all the rings from R. $ 0 to R. $ R: V$

"

dV $

s 2´0

"

R

0

R. dR. 2

2

2z " R.

$

s (2z2 " R2 ' z). 2´0

(24-37)

Note that the variable in the second integral of Eq. 24-37 is R. and not z, which remains constant while the integration over the surface of the disk is carried out. (Note also that, in evaluating the integral, we have assumed that z - 0.)

701

24-6 CALCU L ATI NG TH E FI E LD FROM TH E POTE NTIAL

24-6 CALCULATING THE FIELD FROM THE POTENTIAL Learning Objectives After reading this module, you should be able to . . .

24.23 Given an electric potential as a function of position along an axis, find the electric field along that axis. 24.24 Given a graph of electric potential versus position along an axis, determine the electric field along the axis. 24.25 For a uniform electric field, relate the field magnitude E

and the separation !x and potential difference !V between adjacent equipotential lines. 24.26 Relate the direction of the electric field and the directions in which the potential decreases and increases.

Key Ideas ● The component of

:

E in any direction is the negative of the rate at which the potential changes with distance in that direction: 3V Es $ ' . 3s : ● The x, y, and z components of E may be found from Ex $ '

3V ; 3x

Ey $ '

3V ; 3y

Ez $ '

3V . 3z

:

When E is uniform, all this reduces to !V , !s where s is perpendicular to the equipotential surfaces. E$'

● The electric field is zero parallel to an equipotential

surface.

Calculating the Field from the Potential In Module 24-2, you saw how to find the potential at a point f if you know the electric field along a path from a reference point to point f. In this module, we propose to go the other way — that is, to find the electric field when we know the potential. As Fig. 24-5 shows, solving this problem graphically is easy: If we know the potential V at all points near an assembly of charges, we can draw in a family of equipotential surfaces. The electric field lines, sketched perpendicular : to those surfaces, reveal the variation of E . What we are seeking here is the mathematical equivalent of this graphical procedure. Figure 24-17 shows cross sections of a family of closely spaced equipotential surfaces, the potential difference between each pair of adjacent surfaces : being dV. As the figure suggests, the field E at any point P is perpendicular to the equipotential surface through P. Suppose that a positive test charge q0 moves through a displacement d : s from one equipotential surface to the adjacent surface. From Eq. 24-6, we see that the work the electric field does on the test charge during the move is 'q0 dV. From Eq. 24-16 and Fig. 24-17, we see that the work done by the electric field may : also be written as the scalar product (q0E) " d : s , or q0 E(cos u) ds. Equating these two expressions for the work yields 'q0 dV $ q0 E(cos u) ds, or

E cos u $ '

dV . ds

(24-38) (24-39)

:

Since E cos u is the component of E in the direction of d : s , Eq. 24-39 becomes Es $ '

3V . 3s

(24-40)

We have added a subscript to E and switched to the partial derivative symbols to emphasize that Eq. 24-40 involves only the variation of V along a specified axis : (here called the s axis) and only the component of E along that axis. In words, Eq. 24-40 (which is essentially the reverse operation of Eq. 24-18) states:

E q0

P

+

θ

s

ds

Two equipotential surfaces

Figure 24-17 A test charge q0 moves a distance d : s from one equipotential surface to another. (The separation between the surfaces has been exaggerated for clarity.) The displacement d : s makes an angle : u with the direction of the electric field E .

702

CHAPTE R 24 E LECTR IC POTE NTIAL

:

The component of E in any direction is the negative of the rate at which the electric potential changes with distance in that direction.

If we take the s axis to be, in turn, the x, y, and z axes, we find that the x, y, and : z components of E at any point are Ex $ '

3V ; 3x

Ey $ '

3V ; 3y

Ez $ '

3V . 3z

(24-41)

Thus, if we know V for all points in the region around a charge distribution — that : is, if we know the function V(x, y, z) — we can find the components of E , and thus : E itself, at any point by taking partial derivatives. : For the simple situation in which the electric field E is uniform, Eq. 24-40 becomes !V E$' (24-42) , !s where s is perpendicular to the equipotential surfaces. The component of the electric field is zero in any direction parallel to the equipotential surfaces because there is no change in potential along the surfaces.

Checkpoint 5 The figure shows three pairs of parallel plates with the same separation, and the electric potential of –50 V +150 V –20 V +200 V –200 V –400 V each plate. The electric field between the (1) (2) (3) plates is uniform and perpendicular to the plates. (a) Rank the pairs according to the magnitude of the electric field between the plates, greatest first. (b) For which pair is the electric field pointing rightward? (c) If an electron is released midway between the third pair of plates, does it remain there, move rightward at constant speed, move leftward at constant speed, accelerate rightward, or accelerate leftward?

Sample Problem 24.05 Finding the field from the potential The electric potential at any point on the central axis of a uniformly charged disk is given by Eq. 24-37, V$

s ( 2z2 " R2 ' z). 2´0

Starting with this expression, derive an expression for the electric field at any point on the axis of the disk. KEY IDEAS

:

about that axis. Thus, we want the component Ez of E in the direction of z. This component is the negative of the rate at which the electric potential changes with distance z. Calculation: Thus, from the last of Eqs. 24-41, we can write Ez $ ' $

:

We want the electric field E as a function of distance z along : the axis of the disk. For any value of z, the direction of E must be along that axis because the disk has circular symmetry

s d 3V $' (2z2 " R 2 ' z) 3z 2´0 dz

s 2´0

#1 '

z 2z2 " R2

$.

(Answer)

This is the same expression that we derived in Module 22-5 by integration, using Coulomb’s law.

Additional examples, video, and practice available at WileyPLUS

703

24-7 E LECTR IC POTE NTIAL E N E RGY OF A SYSTE M OF CHARG E D PARTICLES

24-7 ELECTRIC POTENTIAL ENERGY OF A SYSTEM OF CHARGED PARTICLES Learning Objectives After reading this module, you should be able to . . .

24.27 Identify that the total potential energy of a system of charged particles is equal to the work an applied force must do to assemble the system, starting with the particles infinitely far apart. 24.28 Calculate the potential energy of a pair of charged particles. 24.29 Identify that if a system has more than two charged parti-

cles, then the system’s total potential energy is equal to the sum of the potential energies of every pair of the particles. 24.30 Apply the principle of the conservation of mechanical energy to a system of charged particles. 24.31 Calculate the escape speed of a charged particle from a system of charged particles (the minimum initial speed required to move infinitely far from the system).

Key Idea ● The electric potential energy of a system of charged particles is equal to the work needed to assemble the system with the particles initially at rest and infinitely distant from each other. For two particles at separation r, 1 q1q2 U$W$ . 4p´0 r

Electric Potential Energy of a System of Charged Particles In this module we are going to calculate the potential energy of a system of two charged particles and then briefly discuss how to expand the result to a system of more than two particles. Our starting point is to examine the work we must do (as an external agent) to bring together two charged particles that are initially infinitely far apart and that end up near each other and stationary. If the two particles have the same sign of charge, we must fight against their mutual repulsion. Our work is then positive and results in a positive potential energy for the final two-particle system. If, instead, the two particles have opposite signs of charge, our job is easy because of the mutual attraction of the particles. Our work is then negative and results in a negative potential energy for the system. Let’s follow this procedure to build the two-particle system in Fig. 24-18, where particle 1 (with positive charge q1) and particle 2 (with positive charge q2) have separation r. Although both particles are positively charged, our result will apply also to situations where they are both negatively charged or have different signs. We start with particle 2 fixed in place and particle 1 infinitely far away, with an initial potential energy Ui for the two-particle system. Next we bring particle 1 to its final position, and then the system’s potential energy is Uf . Our work changes the system’s potential energy by !U $ Uf ' Ui. With Eq. 24-4 (!U $ q(Vf ' Vi)), we can relate !U to the change in potential through which we move particle 1: Uf ' Ui $ q1(Vf ' Vi).

(24-43)

Let’s evaluate these terms. The initial potential energy is Ui $ 0 because the particles are in the reference configuration (as discussed in Module 24-1). The two potentials in Eq. 24-43 are due to particle 2 and are given by Eq. 24-26: 1 q2 (24-44) . 4p´0 r This tells us that when particle 1 is initially at distance r $ 0, the potential at its location is Vi $ 0. When we move it to the final position at distance r, the potential at its location is 1 q2 Vf $ (24-45) . 4p´0 r V$

q1

+

r

q2

Figure 24-18 Two charges held a fixed distance r apart.

+

704

CHAPTE R 24 E LECTR IC POTE NTIAL

Substituting these results into Eq. 24-43 and dropping the subscript f, we find that the final configuration has a potential energy of U$

1 q1q2 4p´0 r

(two-particle system).

(24-46)

Equation 24-46 includes the signs of the two charges. If the two charges have the same sign, U is positive. If they have opposite signs, U is negative. If we next bring in a third particle, with charge q3, we repeat our calculation, starting with particle 3 at an infinite distance and then bringing it to a final position at distance r31 from particle 1 and distance r32 from particle 2. At the final position, the potential Vf at the location of particle 3 is the algebraic sum of the potential V1 due to particle 1 and the potential V2 of particle 2. When we work out the algebra, we find that The total potential energy of a system of particles is the sum of the potential energies for every pair of particles in the system.

This result applies to a system for any given number of particles. Now that we have an expression for the potential energy of a system of particles, we can apply the principle of the conservation of energy to the system as expressed in Eq. 24-10. For example, if the system consists of many particles, we might consider the kinetic energy (and the associated escape speed) required of one of the particles to escape from the rest of the particles.

Sample Problem 24.06 Potential energy of a system of three charged particles Figure 24-19 shows three charged particles held in fixed positions by forces that are not shown. What is the electric potential energy U of this system of charges? Assume that d $ 12 cm and that q1 $ "q, q2 $ '4q,

q2

d

and q3 $ "2q,

in which q $ 150 nC.

+

q1

KEY IDEA The potential energy U of the system is equal to the work we must do to assemble the system, bringing in each charge from an infinite distance. Calculations: Let’s mentally build the system of Fig. 24-19, starting with one of the charges, say q1, in place and the others at infinity. Then we bring another one, say q2, in from infinity and put it in place. From Eq. 24-46 with d substituted for r, the potential energy U12 associated with the pair of charges q1 and q2 is U12 $

1 q1q2 . 4p´0 d

We then bring the last charge q3 in from infinity and put it in

d

d

Energy is associated with each pair of particles.

+

q3

Figure 24-19 Three charges are fixed at the vertices of an equilateral triangle. What is the electric potential energy of the system?

place. The work that we must do in this last step is equal to the sum of the work we must do to bring q3 near q1 and the work we must do to bring it near q2. From Eq. 24-46, with d substituted for r, that sum is 1 q2q3 1 q1q3 W13 " W23 $ U13 " U23 $ " . 4p´0 d 4p´0 d The total potential energy U of the three-charge system is the sum of the potential energies associated with the three pairs of charges. This sum (which is actually independent of the order in which the charges are brought together) is

24-7 E LECTR IC POTE NTIAL E N E RGY OF A SYSTE M OF CHARG E D PARTICLES

U $ U12 " U13 " U23 $

1 4p´0

("q)("2q) ('4q)("2q) " " # ("q)('4q) $ d d d

$'

10q2 4p´0d

$'

(8.99 ( 109 N#m2/C 2)(10)(150 ( 10'9 C)2 0.12 m

$ '1.7 ( 10'2 J $ '17 mJ.

(Answer)

705

The negative potential energy means that negative work would have to be done to assemble this structure, starting with the three charges infinitely separated and at rest. Put another way, an external agent would have to do 17 mJ of positive work to disassemble the structure completely, ending with the three charges infinitely far apart. The lesson here is this: If you are given an assembly of charged particles, you can find the potential energy of the assembly by finding the potential of every possible pair of the particles and then summing the results.

Sample Problem 24.07 Conservation of mechanical energy with electric potential energy An alpha particle (two protons, two neutrons) moves into a stationary gold atom (79 protons, 118 neutrons), passing through the electron region that surrounds the gold nucleus like a shell and headed directly toward the nucleus (Fig. 24-20). The alpha particle slows until it momentarily stops when its center is at radial distance r $ 9.23 fm from the nuclear center. Then it moves back along its incoming path. (Because the gold nucleus is much more massive than the alpha particle, we can assume the gold nucleus does not move.) What was the kinetic energy Ki of the alpha particle when it was initially far away (hence external to the gold atom)? Assume that the only force acting between the alpha particle and the gold nucleus is the (electrostatic) Coulomb force and treat each as a single charged particle. KEY IDEA During the entire process, the mechanical energy of the alpha particle " gold atom system is conserved. Reasoning: When the alpha particle is outside the atom, the system’s initial electric potential energy Ui is zero because the atom has an equal number of electrons and protons, which produce a net electric field of zero. However, once the alpha particle passes through the electron region surrounding the nucleus on its way to the nucleus, the electric field due to the electrons goes to zero. The reason is that the electrons act like a closed spherical shell of uniform negative charge and, as discussed in Module 23-6, such a shell produces zero electric field in the space it encloses. The alpha particle still experiences the electric field of the protons in the nucleus, which produces a repulsive force on the protons within the alpha particle.

r Alpha particle Gold nucleus

Figure 24-20 An alpha particle, traveling head-on toward the center of a gold nucleus, comes to a momentary stop (at which time all its kinetic energy has been transferred to electric potential energy) and then reverses its path.

As the incoming alpha particle is slowed by this repulsive force, its kinetic energy is transferred to electric potential energy of the system. The transfer is complete when the alpha particle momentarily stops and the kinetic energy is Kf $ 0. Calculations: The principle of conservation of mechanical energy tells us that Ki " Ui $ Kf " Uf .

(24-47)

We know two values: Ui $ 0 and Kf $ 0. We also know that the potential energy Uf at the stopping point is given by the right side of Eq. 24-46, with q1 $ 2e, q2 $ 79e (in which e is the elementary charge, 1.60 ( 10'19 C), and r $ 9.23 fm. Thus, we can rewrite Eq. 24-47 as Ki $ $

1 (2e)(79e) 4p´0 9.23 fm (8.99 ( 109 N#m2/C 2)(158)(1.60 ( 10'19 C)2 9.23 ( 10'15 m

$ 3.94 ( 10'12 J $ 24.6 MeV.

Additional examples, video, and practice available at WileyPLUS

(Answer)

706

CHAPTE R 24 E LECTR IC POTE NTIAL

24-8 POTENTIAL OF A CHARGED ISOLATED CONDUCTOR Learning Objectives After reading this module, you should be able to . . .

24.32 Identify that an excess charge placed on an isolated conductor (or connected isolated conductors) will distribute itself on the surface of the conductor so that all points of the conductor come to the same potential. 24.33 For an isolated spherical conducting shell, sketch graphs of the potential and the electric field magnitude versus distance from the center, both inside and outside the shell. 24.34 For an isolated spherical conducting shell, identify that internally the electric field is zero and the electric potential

has the same value as the surface and that externally the electric field and the electric potential have values as though all of the shell’s charge is concentrated as a particle at its center. 24.35 For an isolated cylindrical conducting shell, identify that internally the electric field is zero and the electric potential has the same value as the surface and that externally the electric field and the electric potential have values as though all of the cylinder’s charge is concentrated as a line of charge on the central axis.

Key Ideas ● An excess charge placed on a conductor will, in the equilibrium state, be located entirely on the outer surface of the conductor. ● The entire conductor, including interior points, is at a uniform potential.

electric field, then at every internal point, the electric field due to the charge cancels the external electric field that otherwise would have been there. ● Also, the net electric field at every point on the surface is perpendicular to the surface.

● If an isolated charged conductor is placed in an external

Potential of a Charged Isolated Conductor :

In Module 23-3, we concluded that E $ 0 for all points inside an isolated conductor. We then used Gauss’ law to prove that an excess charge placed on an isolated conductor lies entirely on its surface. (This is true even if the conductor has an empty internal cavity.) Here we use the first of these facts to prove an extension of the second:

V (kV)

12 8 4 0 0

1

2 r (m)

3

4

(a)

An excess charge placed on an isolated conductor will distribute itself on the surface of that conductor so that all points of the conductor—whether on the surface or inside—come to the same potential. This is true even if the conductor has an internal cavity and even if that cavity contains a net charge.

Our proof follows directly from Eq. 24-18, which is

"

E (kV/m)

12

Vf ' Vi $ '

8

:

4 0 0

1

2 r (m)

3

4

(b)

Figure 24-21 (a) A plot of V(r) both inside and outside a charged spherical shell of radius 1.0 m. (b) A plot of E(r) for the same shell.

f

i

:

E " ds:.

Since E $ 0 for all points within a conductor, it follows directly that Vf $ Vi for all possible pairs of points i and f in the conductor. Figure 24-21a is a plot of potential against radial distance r from the center for an isolated spherical conducting shell of 1.0 m radius, having a charge of 1.0 mC. For points outside the shell, we can calculate V(r) from Eq. 24-26 because the charge q behaves for such external points as if it were concentrated at the center of the shell. That equation holds right up to the surface of the shell. Now let us push a small test charge through the shell — assuming a small hole exists — to its center. No extra work is needed to do this because no net electric force acts on the test charge once it is inside the shell. Thus, the potential at all points inside the shell has the same value as that on the surface, as Fig. 24-21a shows.

R EVI EW & SU M MARY

707

Figure 24-21b shows the variation of electric field with radial distance for the same shell. Note that E $ 0 everywhere inside the shell. The curves of Fig. 24-21b can be derived from the curve of Fig. 24-21a by differentiating with respect to r, using Eq. 24-40 (recall that the derivative of any constant is zero). The curve of Fig. 24-21a can be derived from the curves of Fig. 24-21b by integrating with respect to r, using Eq. 24-19.

Spark Discharge from a Charged Conductor On nonspherical conductors, a surface charge does not distribute itself uniformly over the surface of the conductor. At sharp points or sharp edges, the surface charge density — and thus the external electric field, which is proportional to it — may reach very high values. The air around such sharp points or edges may become ionized, producing the corona discharge that golfers and mountaineers see on the tips of bushes, golf clubs, and rock hammers when thunderstorms threaten. Such corona discharges, like hair that stands on end, are often the precursors of lightning strikes. In such circumstances, it is wise to enclose yourself in a cavity inside a conducting shell, where the electric field is guaranteed to be zero. A car (unless it is a convertible or made with a plastic body) is almost ideal (Fig. 24-22).

Isolated Conductor in an External Electric Field If an isolated conductor is placed in an external electric field, as in Fig. 24-23, all points of the conductor still come to a single potential regardless of whether the conductor has an excess charge. The free conduction electrons distribute themselves on the surface in such a way that the electric field they produce at interior points cancels the external electric field that would otherwise be there. Furthermore, the electron distribution causes the net electric field at all points on the surface to be perpendicular to the surface. If the conductor in Fig. 24-23 could be somehow removed, leaving the surface charges frozen in place, the internal and external electric field would remain absolutely unchanged.

Figure 24-23 An uncharged conductor is suspended in an external electric field. The free electrons in the conductor distribute themselves on the surface as shown, so as to reduce the net electric field inside the conductor to zero and make the net field at the surface perpendicular to the surface.

Courtesy Westinghouse Electric Corporation

Figure 24-22 A large spark jumps to a car’s body and then exits by moving across the insulating left front tire (note the flash there), leaving the person inside unharmed.

–– – – –– –– –– E – – – – –

+

=0 +

+

+

+ +++ + + ++ + + +

Review & Summary Electric Potential The electric potential V at a point P in the electric field of a charged object is V$

'W0 U $ , q0 q0

(24-2)

If the particle moves through a potential difference !V, the change in the electric potential energy is !U $ q !V $ q(Vf ' Vi).

(24-4)

Mechanical Energy If a particle moves through a change !V

where W0 is the work that would be done by the electric force on a positive test charge were it brought from an infinite distance to P, and U is the potential energy that would then be stored in the test charge–object system.

in electric potential without an applied force acting on it, applying the conservation of mechanical energy gives the change in kinetic energy as !K $ 'q !V. (24-9)

Electric Potential Energy If a particle with charge q is

If, instead, an applied force acts on the particle, doing work Wapp, the change in kinetic energy is

placed at a point where the electric potential of a charged object is V, the electric potential energy U of the particle–object system is

!K $ 'q !V " Wapp.

U $ qV.

(24-3)

(24-11)

In the special case when !K $ 0, the work of an applied force

708

CHAPTE R 24 E LECTR IC POTE NTIAL

involves only the motion of the particle through a potential difference: (24-12)

Wapp $ q !V (for Ki $ Kf).

Equipotential Surfaces

The points on an equipotential surface all have the same electric potential. The work done on a test charge in moving it from one such surface to another is independent of the locations of the initial and final points on these surfaces and of : the path that joins the points. The electric field E is always directed perpendicularly to corresponding equipotential surfaces.

V$

1 p cos u 4p´0 r2

(24-30)

for r 2 d; the angle u is defined in Fig. 24-13.

Potential Due to a Continuous Charge Distribution For a continuous distribution of charge, Eq. 24-27 becomes V$

"

1 4p´0

dq , r

(24-32)

in which the integral is taken over the entire distribution.

:

Finding V from E The electric potential difference between two points i and f is Vf ' Vi $ '

"

f

i

:

(24-18)

E " d: s,

where the integral is taken over any path connecting the points. If the integration is difficult along any particular path, we can choose a different path along which the integration might be easier. If we choose Vi $ 0, we have, for the potential at a particular point, V$'

"

f

i

:

:

Calculating E from V The component of E in any direction is the negative of the rate at which the potential changes with distance in that direction: 3V (24-40) Es $ ' . 3s :

The x, y, and z components of E may be found from Ex $ '

3V ; 3x

Ey $ '

3V ; 3y

Ez $ '

3V . 3z

(24-41)

:

:

(24-19)

E " d: s.

In the special case of a uniform field of magnitude E, the potential change between two adjacent (parallel) equipotential lines separated by distance !x is

When E is uniform, Eq. 24-40 reduces to !V , !s where s is perpendicular to the equipotential surfaces.

(24-42)

E$'

(24-21)

Electric Potential Energy of a System of Charged Particles The electric potential energy of a system of charged

Potential Due to a Charged Particle The electric potential

particles is equal to the work needed to assemble the system with the particles initially at rest and infinitely distant from each other. For two particles at separation r,

!V $ 'E !x.

due to a single charged particle at a distance r from that particle is V$

1 q , 4p´0 r

(24-26)

where V has the same sign as q. The potential due to a collection of charged particles is n

1 V $ ' Vi $ 4p´ 0 i$1

n

'

i$1

qi . ri

(24-27)

Potential Due to an Electric Dipole At a distance r from an electric dipole with dipole moment magnitude p $ qd, the electric potential of the dipole is

U$W$

1 q1q2 . 4p´0 r

(24-46)

Potential of a Charged Conductor An excess charge placed on a conductor will, in the equilibrium state, be located entirely on the outer surface of the conductor.The charge will distribute itself so that the following occur: (1) The entire conductor, including interior points, is at a uniform potential. (2) At every internal point, the electric field due to the charge cancels the external electric field that otherwise would have been there. (3) The net electric field at every point on the surface is perpendicular to the surface.

Questions 1 Figure 24-24 shows eight parti- –4q cles that form a square, with distance d between adjacent particles. What is the net electric potential at point P at the center of the square if we take the electric potential to +5q be zero at infinity? 2 Figure 24-25 shows three sets of cross sections of equipotential surfaces in uniform electric fields; all three cover the same size region of space. The electric potential is indi-

–q

–2q

+q d

P

–2q

cated for each equipotential surface. (a) Rank the arrangements according to the magnitude of the electric field present in the region, greatest first. (b) In which is the electric field directed down the page?

–5q

+4q

Figure 24-24 Question 1.

20 V 40 60 80 100 (1)

–140 V

–10 V

–120

–30

–100 (2)

Figure 24-25 Question 2.

–50 (3)

QU ESTIONS

3 Figure 24-26 shows four pairs of charged particles. For each pair, let V $ 0 at infinity and consider Vnet at points on the x axis. For which pairs is there a point at which Vnet $ 0 (a) between the particles and : (b) to the right of the particles? (c) At such a point is Enet due to the particles equal to zero? (d) For each pair, are there off-axis points (other than at infinity) where Vnet $ 0? –2q

x

+6q

+3q

(1)

x

–4q (2)

+12q

x

+q

–6q

(3)

x

–2q (4)

Figure 24-26 Questions 3 and 9. 4 Figure 24-27 gives the electric V potential V as a function of x. (a) Rank the five regions according to the magnitude of the x component of the electric field within 1 2 3 4 5 x them, greatest first. What is the direction of the field along the x axis Figure 24-27 Question 4. in (b) region 2 and (c) region 4? 5 Figure 24-28 shows three paths 3 along which we can move the positively charged sphere A closer to +A B + 2 positively charged sphere B, which 1 is held fixed in place. (a) Would sphere A be moved to a higher or lower electric potential? Is the work Figure 24-28 Question 5. done (b) by our force and (c) by the electric field due to B positive, negative, or zero? (d) Rank the paths according to the work our force does, greatest first. 6 Figure 24-29 shows four arrangements of charged particles, all the same distance from the origin. Rank the situations according to the net electric potential at the origin, most positive first. Take the potential to be zero at infinity. +2q

–2q

–2q –2q

–9q

–4q +2q

+2q –2q

(a)

(b)

(c)

(d)

7 Figure 24-30 shows a system of three charged particles. If you move the particle of charge "q from point A to point D, are the following quantities positive, negative, or zero: (a) the change in the electric potential energy of the three-particle system, (b) the work done by the net electric force on the particle you moved (that is, the net force due to the other two particles), and (c) the work done by your force? (d) What are the answers to (a) through (c) if, instead, the particle is moved from B to C? d A

d +Q

d B

d C

Figure 24-30 Questions 7 and 8.

9 Figure 24-26 shows four pairs of charged particles with identical separations. (a) Rank the pairs according to their electric potential energy (that is, the energy of the two-particle system), greatest (most positive) first. (b) For each pair, if the separation between the particles is increased, Q + R P does the potential energy of the pair increase or decrease? (a ) 10 (a) In Fig. 24-31a, what is the Q 40°(full angle) potential at point P due to charge R P Q at distance R from P? Set V $ 0 at infinity. (b) In Fig. 24-31b, the same charge Q has been spread (b ) uniformly over a circular arc of radius R and central angle 40%. What Q is the potential at point P, the center of curvature of the arc? (c) In Fig. 24-31c, the same charge Q has R been spread uniformly over a circle P of radius R. What is the potential at point P, the center of the circle? (d) Rank the three situations according to the magnitude of the electric field that is set up at P, (c ) greatest first. Figure 24-31 Question 10. 11 Figure 24-32 shows a thin, uniformly charged rod and three points at the same distance d from the rod. Rank the magnitude of the electric potential the rod produces at those three points, greatest first. y a

b d

L/2

c

L/2

x

d

–7q

Figure 24-29 Question 6.

+q

8 In the situation of Question 7, is the work done by your force positive, negative, or zero if the particle is moved (a) from A to B, (b) from A to C, and (c) from B to D? (d) Rank those moves according to the magnitude of the work done by your force, greatest first.

Figure 24-32 Question 11.

–q

–3q

709

A

B

100 V

d +Q

12 In Fig. 24-33, a particle is to be released at rest at point A and then is to be accelerated directly through point B by an electric field. The potential difference between points A and B is 100 V. Which point should be at higher electric potential if the particle is (a) an electron, (b) a proton, and (c) an alpha particle (a nucleus of two protons and two neutrons)? (d) Rank the kinetic energies of the particles at point B, greatest first.

D

Figure 24-33 Question 12.

710

CHAPTE R 24 E LECTR IC POTE NTIAL

Problems Tutoring problem available (at instructor’s discretion) in WileyPLUS and WebAssign SSM • – •••

Worked-out solution available in Student Solutions Manual

WWW Worked-out solution is at

Number of dots indicates level of problem difficulty

ILW

Interactive solution is at

http://www.wiley.com/college/halliday

Additional information available in The Flying Circus of Physics and at flyingcircusofphysics.com

Module 24-1 Electric Potential •1 SSM A particular 12 V car battery can send a total charge of 84 A #h (ampere-hours) through a circuit, from one terminal to the other. (a) How many coulombs of charge does this represent? (Hint: See Eq. 21-3.) (b) If this entire charge undergoes a change in electric potential of 12 V, how much energy is involved?

••9 An infinite nonconducting sheet has a surface charge density s $ "5.80 pC/m2. (a) How much work is done by the electric field due to the sheet if a particle of charge q $ "1.60 ( 10'19 C is moved from the sheet to a point P at distance d $ 3.56 cm from the sheet? (b) If the electric potential V is defined to be zero on the sheet, what is V at P?

•2 The electric potential difference between the ground and a cloud in a particular thunderstorm is 1.2 ( 109 V. In the unit electron-volts, what is the magnitude of the change in the electric potential energy of an electron that moves between the ground and the cloud?

•••10 Two uniformly charged, infinite, nonconducting planes are parallel to a yz plane and positioned at x $ '50 cm and x $ "50 cm. The charge densities on the planes are '50 nC/m2 and "25 nC/m2, respectively. What is the magnitude of the potential difference between the origin and the point on the x axis at x $ "80 cm? (Hint: Use Gauss’ law.)

•3 Suppose that in a lightning flash the potential difference between a cloud and the ground is 1.0 ( 109 V and the quantity of charge transferred is 30 C. (a) What is the change in energy of that transferred charge? (b) If all the energy released could be used to accelerate a 1000 kg car from rest, what would be its final speed? Module 24-2 Equipotential Surfaces and the Electric Field •4 Two large, parallel, conducting plates are 12 cm apart and have charges of equal magnitude and opposite sign on their facing surfaces. An electric force of 3.9 ( 10'15 N acts on an electron placed anywhere between the two plates. (Neglect fringing.) (a) Find the electric field at the position of the electron. (b) What is the potential difference between the plates? •5 SSM An infinite nonconducting sheet has a surface charge density s $ 0.10 mC/m2 on one side. How far apart are equipotential surfaces whose potentials differ by 50 V? •6 When an electron moves from Electric A to B along an electric field line in field line Fig. 24-34, the electric field does '19 3.94 ( 10 J of work on it. What are the electric potential differences B (a) VB ' VA, (b) VC ' VA, and (c) VC ' VB?

A

Module 24-3 Potential Due to a Charged Particle •12 As a space shuttle moves through the dilute ionized gas of Earth’s ionosphere, the shuttle’s potential is typically changed by '1.0 V during one revolution. Assuming the shuttle is a sphere of radius 10 m, estimate the amount of charge it collects. •13 What are (a) the charge and (b) the charge density on the surface of a conducting sphere of radius 0.15 m whose potential is 200 V (with V $ 0 at infinity)? •14 Consider a particle with charge q $ 1.0 mC, point A at distance d1 $ 2.0 m from q, and point B at distance d2 $ 1.0 m. (a) If A and B are diametrically opposite each other, as in Fig. 24-36a, what is the electric potential difference VA ' VB? (b) What is that electric potential difference if A and B are located as in Fig. 24-36b? B d2

C

••7 The electric field in a region of Equipotentials space has the components Ey $ Figure 24-34 Problem 6. Ez $ 0 and Ex $ (4.00 N/C)x. Point A is on the y axis at y $3.00 m, and point B is on the x axis at x $ 4.00 m. What is the potential difference VB ' VA? ••8 A graph of the x component of the electric field as a function of x in a region of space is shown in Fig. 24-35. The scale of the vertical axis is set by Exs $ 20.0 N/C. The y and z components of the electric field are zero in this region. If the electric potential at the origin is 10 V, (a) what is the electric Exs potential at x $ 2.0 m, (b) what is the greatest positive value of the elec0 1 2 3 4 5 6 tric potential for points on the x axis –E for which 0 / x / 6.0 m, and (c) for xs x (m) what value of x is the electric potential zero? Figure 24-35 Problem 8. Ex (N/C)

•••11 A nonconducting sphere has radius R $ 2.31 cm and uniformly distributed charge q $ "3.50 fC. Take the electric potential at the sphere’s center to be V0 $ 0. What is V at radial distance (a) r $ 1.45 cm and (b) r $ R. (Hint: See Module 23-6.)

B

d2

+

d1

q

A

+

d1

q

(a )

A

(b )

Figure 24-36 Problem 14. ••15 SSM ILW A spherical drop of water carrying a charge of 30 pC has a potential of 500 V at its surface (with V $ 0 at infinity). (a) What is the radius of the drop? (b) If two such drops of the same charge and radius combine to form a single spherical drop, what is the potential at the surface +2q1 +4q 2 –3q1 of the new drop? ••16 Figure 24-37 shows a rectangular array of charged particles fixed in place, with distance a $ 39.0 cm and the charges shown as integer multiples of q1 $ 3.40 pC and q2 $ 6.00 pC. With V $ 0 at infinity, what

a

a

a

a a

–q1

a +4q 2

+2q1

Figure 24-37 Problem 16.

711

PROB LE M S

is the net electric potential at the rectangle’s center? (Hint: Thoughtful examination of the arrangement can reduce the calculation.)

–q

+ +q

an electric dipole, on the positive side of the dipole. (The origin of z is at the center of the dipole.) The particle is then moved along a circular path around the dipole center until it is at coordinate z $ '20 nm, on the negative side of the dipole axis. Figure 24-41b gives the work Wa done by the force moving the particle versus the angle u that locates the particle relative to the positive direction of the z axis. The scale of the vertical axis is set by Was $ 4.0 ( 10'30 J. What is the magnitude of the dipole moment?

d

d

–q

d

P

••17 In Fig. 24-38, what is the net electric potential at point P due to the four particles if V $ 0 at infinity, q $ 5.00 fC, and d $ 4.00 cm?

d +q +

••18 Two charged particles are Figure 24-38 Problem 17. shown in Fig. 24-39a. Particle 1, with charge q1, is fixed in place at distance d. Particle 2, with charge q2, can be moved along the x axis. Figure 24-39b gives the net electric potential V at the origin due to the two particles as a function of the x coordinate of particle 2. The scale of the x axis is set by xs $ 16.0 cm. The plot has an asymptote of V $ 5.76 ( 10'7 V as x : 0. What is q2 in terms of e? 4

d 1

2

x

V (10–7 V)

y

0

xs

Module 24-5 Potential Due to a Continuous Charge Distribution •23 (a) Figure 24-42a shows a nonconducting rod of length L $ 6.00 cm and uniform linear charge density l $ "3.68 pC/m. Assume that the electric potential is defined to be V $ 0 at infinity. What is V at point P at distance d $ 8.00 cm along the rod’s perpendicular bisector? (b) Figure 24-42b shows an identical rod except that one half is now negatively charged. Both halves have a linear charge density of magnitude 3.68 pC/m.With V $ 0 at infinity, what is V at P? P

x (cm)

P

d

d

–10 (a)

L/2

Figure 24-39 Problem 18. ••19 In Fig. 24-40, particles with the charges q1 $ "5e and q2 $ '15e are fixed in place with a separation of d $ 24.0 cm. With electric potential defined to be V $ 0 at infinity, what are the finite (a) positive and (b) negative values of x at which the net electric potential on the x axis is zero?

L/2

q2

q1

x

d

Figure 24-40 Problems 19 and 20.

z

(b )

•24 In Fig. 24-43, a plastic rod having a uniformly distributed charge Q $ '25.6 pC has been bent into a circular arc of radius R $ 3.71 cm and central angle f $ 120%. With V $ 0 at infinity, what is the electric potential at P, the center of curvature of the rod?

Wa (10–30 J)

+e 0

θ

Q

φ

P

R

•25 A plastic rod has been bent into a circle of radius R $ 8.20 cm. It has a charge Q1 $ "4.20 pC uniformly distributed along onequarter of its circumference and a charge Figure 24-43 Q2 $ '6Q1 uniformly distributed along the Problem 24. rest of the circumference (Fig. 24-44). With V $ 0 at infinity, what is the electric potential at (a) the center C of the circle and (b) point P, on the central axis of the circle at distance D $ 6.71 cm from the center? P D

Q2

θ

L/2

Figure 24-42 Problem 23.

Module 24-4 Potential Due to an Electric Dipole •21 ILW The ammonia molecule NH3 has a permanent electric dipole moment equal to 1.47 D, where 1 D $ 1 debye unit $ 3.34 ( 10'30 C #m. Calculate the electric potential due to an ammonia molecule at a point 52.0 nm away along the axis of the dipole. (Set V $ 0 at infinity.)

(a)

L/2 (a )

y

••20 Two particles, of charges q1 and q2, are separated by distance d in Fig. 24-40. The net electric field due to the particles is zero at x $ d/4. With V $ 0 at infinity, locate (in terms of d) any point on the x axis (other than at infinity) at which the electric potential due to the two particles is zero.

+ –

+++++++ –––––––

+ + + + + + + + + + + + +

(b)

R

C Q1

– Was

(b)

Figure 24-41 Problem 22. ••22 In Fig. 24-41a, a particle of elementary charge "e is initially at coordinate z $ 20 nm on the dipole axis (here a z axis) through

Figure 24-44 Problem 25. ••26 Figure 24-45 shows a thin rod with a uniform charge density of 2.00 mC/m. Evaluate the electric potential at point P if d $ D $ L/4.00. Assume that the potential is zero at infinity.

P d

Rod D

L

Figure 24-45 Problem 26.

x

CHAPTE R 24 E LECTR IC POTE NTIAL y (cm) 4.0

••27 In Fig. 24-46, three thin plastic rods form quarter-circles with a common center of curvature at the origin. The uniform charges on the three rods are Q1 $ "30 nC, Q2 $ "3.0Q1, and Q3 $ '8.0Q1. What is the net electric potential at the origin due to the rods?

Q2

•••32 A nonuniform linear charge distribution given by l $ bx, where b is a constant, is located along an x axis from x $ 0 to x $ 0.20 m. If b $ 20 nC/m2 and V $ 0 at infinity, what is the electric potential at (a) the origin and (b) the point y $ 0.15 m on the y axis?

Q3

2.0 1.0

x (cm)

Q1

Figure 24-46 Problem 27.

••28 Figure 24-47 shows a thin plastic rod of length L $ 12.0 cm and uniform positive charge Q $ 56.1 fC lying on an x axis.With V $ 0 at infinity, find the electric potential at point P1 on the axis, at distance d $ 2.50 cm from the rod.

y P2 D P1 d

+ + + + + + + + + + x L

••29 In Fig. 24-48, what is the net electric potential at the origin due Figure 24-47 Problems 28, 33, to the circular arc of charge Q1 $ 38, and 40. "7.21 pC and the two particles of charges Q2 $ 4.00Q1 and Q3 $ '2.00Q1? The arc’s center of curvature is at the origin and its radius is R $ 2.00 m; the angle indicated is u $ 20.0%. y Q2 Q1

R

2.00R θ

x

•35 The electric potential at points in an xy plane is given by V $ (2.0 V/m2)x2 ' (3.0 V/m2)y2. In unit-vector notation, what is the electric field at the point (3.0 m, 2.0 m)? •36 The electric potential V in the space between two flat parallel plates 1 and 2 is given (in volts) by V $ 1500x2, where x (in meters) is the perpendicular distance from plate 1. At x $ 1.3 cm, (a) what is the magnitude of the electric field and (b) is the field directed toward or away from plate 1?

••38 Figure 24-47 shows a thin plastic rod of length L $ 13.5 cm and uniform charge 43.6 fC. (a) In terms of distance d, find an expression for the electric potential at point P1. (b) Next, substitute variable x for d and find an expression for the magnitude of the component Ex of the electric field at P1. (c) What is the direction of Ex relative to the positive direction of the x axis? (d) What is the value of Ex at P1 for x $ d $ 6.20 cm? (e) From the symmetry in Fig. 24-47, determine Ey at P1.

Q3

Figure 24-48 Problem 29. ••30 The smiling face of Fig. 2449 consists of three items:

••39 An electron is placed in an xy plane where the electric potential depends on x and y as shown, for the coordinate axes, in Fig. 24-51 (the potential does not depend on z). The scale of the vertical axis is set by Vs $ 500 V. In unit-vector notation, what is the electric force on the electron?

1. a thin rod of charge '3.0 mC that forms a full circle of radius 6.0 cm; 2. a second thin rod of charge 2.0 mC that forms a circular arc of radius 4.0 cm, subtending an angle of 90% about the center of the full circle;

What is the net electric potential at the center?

Module 24-6 Calculating the Field from the Potential •34 Two large parallel metal plates are 1.5 cm apart and have charges of equal magnitudes but opposite signs on their facing surfaces. Take the potential of the negative plate to be zero. If the potential halfway between the plates is then "5.0 V, what is the electric field in the region between the plates?

••37 SSM What is the magnitude of the electric field at the point (3.00iˆ ' 2.00jˆ " 4.00kˆ ) m if the electric potential in the region is given by V $ 2.00xyz2, where V is in volts and coordinates x, y, and z are in meters?

R

0

Figure 24-49 Problem 30. V (V)

3. an electric dipole with a dipole moment that is perpendicular to a radial line and has a magnitude of 1.28 ( 10'21 C #m.

•••33 The thin plastic rod shown in Fig. 24-47 has length L $ 12.0 cm and a nonuniform linear charge density l $ cx, where c $ 28.9 pC/m2. With V $ 0 at infinity, find the electric potential at point P1 on the axis, at distance d $ 3.00 cm from one end.

P

0.2

0.4 V (V)

712

!Vs

0

D

••31 SSM WWW A plastic disk of R radius R $ 64.0 cm is charged on one side with a uniform surface Figure 24-50 Problem 31. charge density s $ 7.73 fC/m2, and then three quadrants of the disk are removed. The remaining quadrant is shown in Fig. 24-50. With V $ 0 at infinity, what is the potential due to the remaining quadrant at point P, which is on the central axis of the original disk at distance D $ 25.9 cm from the original center?

Vs

!2Vs x (m)

0.2

0.4

y (m)

Figure 24-51 Problem 39. •••40 The thin plastic rod of length L $ 10.0 cm in Fig. 24-47 has a nonuniform linear charge density l $ cx, where c $ 49.9 pC/m2. (a) With V $ 0 at infinity, find the electric potential at point P2 on the y axis at y $ D $ 3.56 cm. (b) Find the electric field component Ey at P2. (c) Why cannot the field component Ex at P2 be found using the result of (a)?

713

PROB LE M S

•42 (a) What is the electric potential energy of two electrons separated by 2.00 nm? (b) If the separation increases, does the potential energy increase or decrease? +q

•43 SSM ILW WWW How much work is required to set up the arrangement of Fig. 24-52 if q $ 2.30 pC, a $ 64.0 cm, and the particles are initially infinitely far apart and at rest?

+

a

•44 In Fig. 24-53, seven charged particles are fixed in place to form a square with an edge length of 4.0 cm. How much work must we do to bring a particle of charge "6e initially at rest from an infinite distance to the center of the square? y

–e

–2e

–q

a a

+

+q

Figure 24-52 Problem 43.

–3e x

+2e +3e

–q a

+3e

+e

Figure 24-53 Problem 44. ••45 ILW A particle of charge q is fixed at point P, and a second particle of mass m and the same charge q is initially held a distance r1 from P. The second particle is then released. Determine its speed when it is a distance r2 from P. Let q $ 3.1 mC, m $ 20 mg, r1 $ 0.90 mm, and r2 $ 2.5 mm. ••46 A charge of '9.0 nC is uniformly distributed around a thin plastic ring lying in a yz plane with the ring center at the origin. A '6.0 pC particle is located on the x axis at x $ 3.0 m. For a ring radius of 1.5 m, how much work must an external force do on the particle to move it to the origin? ••47 What is the escape speed for an electron initially at rest on the surface of a sphere with a radius of 1.0 cm and a uniformly distributed charge of 1.6 ( 10'15 C? That is, what initial speed must the electron have in order to reach an infinite distance from the sphere and have zero kinetic energy when it gets there? ••48 A thin, spherical, conducting shell of radius R is mounted on an isolating support and charged to a potential of '125 V. An electron is then fired directly toward the center of the shell, from point P at distance r from the center of the shell (r 2 R). What initial speed v0 is needed for the electron to just reach the shell before reversing direction?

infinity to the indicated point near two fixed particles of charges q1 $ "4e and q2 $ 'q1/2? Distance d $ 1.40 cm, u1 $ 43%, and u2 $ 60%.

q1

Q

+

2.00d

θ1

+

θ2 d

••51 In the rectangle of Fig. 24q2 55, the sides have lengths 5.0 cm and 15 cm, q1 $ '5.0 mC, and q2 $ "2.0 ∞ mC. With V $ 0 at infinity, what is the Figure 24-54 Problem 50. electric potential at (a) corner A and (b) corner B? (c) How much work is q1 A required to move a charge q3 $ "3.0 mC from B to A along a diagonal of the rectangle? (d) Does this work in+q crease or decrease the electric po- B 2 tential energy of the three-charge system? Is more, less, or the same Figure 24-55 Problem 51. work required if q3 is moved along a path that is (e) inside the rectangle but not on a diagonal and (f) outside the rectangle? ••52 Figure 24-56a shows an electron moving along an electric dipole axis toward the negative side of the dipole. The dipole is fixed in place. The electron was initially very far from the dipole, with kinetic energy 100 eV. Figure 24-56b gives the kinetic energy K of the electron versus its distance r from the dipole center. The scale of the horizontal axis is set by rs $ 0.10 m. What is the magnitude of the dipole moment?

+ –

–e (a)

100 K (eV)

Module 24-7 Electric Potential Energy of a System of Charged Particles •41 A particle of charge "7.5 mC is released from rest at the point x $ 60 cm on an x axis. The particle begins to move due to the presence of a charge Q that remains fixed at the origin. What is the kinetic energy of the particle at the instant it has moved 40 cm if (a) Q $ "20 mC and (b) Q $ '20 mC?

50 0 r (m) (b)

rs

Figure 24-56 Problem 52.

••53 Two tiny metal spheres A and B, mass mA $ 5.00 g and mB $ 10.0 g, have equal positive charge q $ 5.00 mC. The spheres are connected by a massless nonconducting string of length d $ 1.00 m, which is much greater than the radii of the spheres. (a) What is the electric potential energy of the system? (b) Suppose you cut the string.At that instant, what is the acceleration of each sphere? (c) A long time after you cut the string, what is the speed of each sphere? ••54 A positron (charge "e, V (V) mass equal to the electron mass) is 7 V s moving at 1.0 ( 10 m/s in the positive direction of an x axis when, at x $ 0, it encounters an electric field directed along the x axis.The electric potential V associated with the field x (cm) 20 50 is given in Fig. 24-57.The scale of the 0 vertical axis is set by Vs $ 500.0 V. Figure 24-57 Problem 54. (a) Does the positron emerge from the field at x $ 0 (which means its motion is reversed) or at x $ 0.50 m (which means its motion is not reversed)? (b) What is its speed when it emerges?

••49 Two electrons are fixed 2.0 cm apart. Another electron is shot from infinity and stops midway between the two. What is its initial speed?

••55 An electron is projected with an initial speed of 3.2 ( 10 5 m/s directly toward a proton that is fixed in place. If the electron is initially a great distance from the proton, at what distance from the proton is the speed of the electron instantaneously equal to twice the initial value?

••50 In Fig. 24-54, how much work must we do to bring a particle, of charge Q $ "16e and initially at rest, along the dashed line from

••56 Particle 1 (with a charge of "5.0 mC) and particle 2 (with a charge of "3.0 mC) are fixed in place with separation d $ 4.0 cm

714

CHAPTE R 24 E LECTR IC POTE NTIAL

on the x axis shown in Fig. 24-58a. Particle 3 can be moved along the x axis to the right of particle 2. Figure 24-58b gives the electric potential energy U of the three-particle system as a function of the x coordinate of particle 3. The scale of the vertical axis is set by Us $ 5.0 J. What is the charge of particle 3?

the circle. (b) With that addition of the electron to the system of 12 charged particles, what is the change in the electric potential energy of the system? –e

–e

y

1

2

3

x

U (J)

Us

d

0 –Us

Q 10

(a)

x (cm)

20

R

(b)

(a)

Figure 24-58 Problem 56.

Figure 24-61 Problem 60.

••57 SSM Identical 50 mC charges are fixed on an x axis at x ! "3.0 m. A particle of charge q ! #15 mC is then released from rest at a point on the positive part of the y axis. Due to the symmetry of the situation, the particle moves along the y axis and has kinetic energy 1.2 J as it passes through the point x ! 0, y ! 4.0 m. (a) What is the kinetic energy of the particle as it passes through the origin? (b) At what negative value of y will the particle momentarily stop?

V (V)

Vs ••58 Proton in a well. Figure 24-59 shows electric potential V along an x axis. The scale of the verx (cm) tical axis is set by 0 1 2 3 4 5 6 7 Vs ! 10.0 V. A proFigure 24-59 Problem 58. ton is to be released at x ! 3.5 cm with initial kinetic energy 4.00 eV. (a) If it is initially moving in the negative direction of the axis, does it reach a turning point (if so, what is the x coordinate of that point) or does it escape from the plotted region (if so, what is its speed at x ! 0)? (b) If it is initially moving in the positive direction of the axis, does it reach a turning point (if so, what is the x coordinate of that point) or does it escape from the plotted region (if so, what is its speed at x ! 6.0 cm)? What are the (c) magnitude F and (d) direction (positive or negative direction of the x axis) of the electric force on the proton if the proton moves just to the left of x ! 3.0 cm? What are (e) F and (f) the direction if the proton moves just to the right of x ! 5.0 cm?

••59 In Fig. 24-60, a charged particle (either an electron or a proton) is moving rightward between two parallel charged plates separated by distance d ! 2.00 mm. The plate potentials are V1 ! #70.0 V and V2 ! #50.0 V. The particle is slowing from an initial speed of 90.0 km/s at the left V1 plate. (a) Is the particle an electron or a proton? (b) What is its speed just as it reaches plate 2?

(b)

d

•••61 Suppose N electrons can be placed in either of two configurations. In configuration 1, they are all placed on the circumference of a narrow ring of radius R and are uniformly distributed so that the distance between adjacent electrons is the same everywhere. In configuration 2, N # 1 electrons are uniformly distributed on the ring and one electron is placed in the center of the ring. (a) What is the smallest value of N for which the second configuration is less energetic than the first? (b) For that value of N, consider any one circumference electron — call it e0. How many other circumference electrons are closer to e0 than the central electron is? Module 24-8 Potential of a Charged Isolated Conductor •62 Sphere 1 with radius R1 has positive charge q. Sphere 2 with radius 2.00R1 is far from sphere 1 and initially uncharged. After the separated spheres are connected with a wire thin enough to retain only negligible charge, (a) is potential V1 of sphere 1 greater than, less than, or equal to potential V2 of sphere 2? What fraction of q ends up on (b) sphere 1 and (c) sphere 2? (d) What is the ratio s1/s2 of the surface charge densities of the spheres? •63 SSM WWW Two metal spheres, each of radius 3.0 cm, have a center-to-center separation of 2.0 m. Sphere 1 has charge %1.0 $ 10#8 C; sphere 2 has charge #3.0 $ 10#8 C. Assume that the separation is large enough for us to say that the charge on each sphere is uniformly distributed (the spheres do not affect each other). With V ! 0 at infinity, calculate (a) the potential at the point halfway between the centers and the potential on the surface of (b) sphere 1 and (c) sphere 2. •64 A hollow metal sphere has a potential of %400 V with respect to ground (defined to be at V ! 0) and a charge of 5.0 $ 10#9 C. Find the electric potential at the center of the sphere. •65 SSM What is the excess charge on a conducting sphere of radius r ! 0.15 m if the potential of the sphere is 1500 V and V ! 0 at infinity?

V2

Figure 24-60 Problem 59.

••60 In Fig. 24-61a, we move an electron from an infinite distance to a point at distance R ! 8.00 cm from a tiny charged ball. The move requires work W ! 2.16 $ 10#13 J by us. (a) What is the charge Q on the ball? In Fig. 24-61b, the ball has been sliced up and the slices spread out so that an equal amount of charge is at the hour positions on a circular clock face of radius R ! 8.00 cm. Now the electron is brought from an infinite distance to the center of

••66 Two isolated, concentric, conducting spherical shells have radii R1 ! 0.500 m and R2 ! 1.00 m, uniform charges q1 ! %2.00 mC and q2 ! %1.00 mC, and negligible thicknesses. What is the magnitude of the electric field E at radial distance (a) r ! 4.00 m, (b) r ! 0.700 m, and (c) r ! 0.200 m? With V ! 0 at infinity, what is V at (d) r ! 4.00 m, (e) r ! 1.00 m, (f) r ! 0.700 m, (g) r ! 0.500 m, (h) r ! 0.200 m, and (i) r ! 0? ( j) Sketch E(r) and V(r). ••67 A metal sphere of radius 15 cm has a net charge of 3.0 $ 10#8 C. (a) What is the electric field at the sphere’s surface? (b) If V ! 0 at infinity, what is the electric potential at the sphere’s surface? (c) At what distance from the sphere’s surface has the electric potential decreased by 500 V?

715

PROB LE M S

Additional Problems 68 Here are the charges and coordinates of two charged particles located in an xy plane: q1 ! %3.00 $ 10#6 C, x ! %3.50 cm, y ! %0.500 cm and q2 !#4.00 $ 10#6 C, x ! #2.00 cm, y ! %1.50 cm. How much work must be done to locate these charges at their given positions, starting from infinite separation?

pC, q2 ! #2.00q1, q3 ! %3.00q1. With V ! 0 at infinity, what is the net electric potential of the arcs at the common center of curvature? 79 An electron is released from rest on the axis of an electric dipole that has charge e and charge separation d ! 20 pm and that is fixed in place. The release point is on the positive side of the dipole, at distance 7.0d from the dipole center. What is the electron’s speed when it reaches a point 5.0d from the dipole center?

69 SSM A long, solid, conducting cylinder has a radius of 2.0 cm. The electric field at the surface of the cylinder is 160 N/C, directed radially outward. Let A, B, and C be points that are 1.0 cm, 2.0 cm, and 5.0 cm, respectively, from the central axis of the cylinder. What are (a) the magnitude of the electric field at C and the electric potential differences (b) VB #VC and (c) VA #VB?

80 Figure 24-64 shows a ring of outer radius R ! 13.0 cm, inner radius r ! 0.200R, and uniform surface charge density s ! 6.20 pC/m2. With V ! 0 at infinity, find the electric potential at point P on the central axis of the ring, at distance z ! 2.00R from the center of the ring.

70 The chocolate crumb mystery. This story begins with Problem 60 in Chapter 23. (a) From the answer to part (a) of that problem, find an expression for the electric potential as a function of the radial distance r from the center of the pipe. (The electric potential is zero on the grounded pipe wall.) (b) For the typical volume charge density r ! #1.1 $ 10#3 C/m3, what is the difference in the electric potential between the pipe’s center and its inside wall? (The story continues with Problem 60 in Chapter 25.)

73 (a) If an isolated conducting sphere 10 cm in radius has a net charge of 4.0 mC and if V ! 0 at infinity, what is the potential on the surface of the sphere? (b) Can this situation actually occur, given that the air around the sphere undergoes electrical q3 breakdown when the field exceeds 3.0 MV/m? a

q1

b

Figure 24-62 75 An electric field of approximately Problem 74. 100 V/m is often observed near the surface of Earth. If this were the field over the entire surface, what would be the electric potential of a point on the surface? (Set V ! 0 at infinity.)

78 Figure 24-63 shows three circular, nonconducting arcs of radius R ! 8.50 cm.The charges on the arcs are q1 ! 4.52

45.0°

45.0°

x

R

q2

q3

Figure 24-63 Problem 78.

r

82 (a) If Earth had a uniform surface charge density of 1.0 electron/m2 (a very artificial assumption), what would its potential be? (Set V ! 0 at infinity.) What would be the (b) magnitude and (c) direction (radially inward or outward) of the electric field due to Earth just outside its surface? d1 83 In Fig. 24-66, point P is at distance P d1 ! 4.00 m from particle 1 (q1 ! #2e) q 1 and distance d2 ! 2.00 m from particle d2 2 (q2 ! %2e), with both particles fixed in place. (a) With V ! 0 at infinity, what q2 is V at P? If we bring a particle of charge q3 ! %2e from infinity to P, Figure 24-66 Problem 83. (b) how much work do we do and (c) what is the potential energy of the three-particle system?

76 A Gaussian sphere of radius 4.00 cm is centered on a ball that has a radius of 1.00 cm and a uniform charge distribution. The total (net) y electric flux through the surface of the Gaussian sphere is %5.60 $ 10 4 N &m2/C. q1 What is the electric potential 12.0 cm from the center of the ball? 77 In a Millikan oil-drop experiment (Module 22-6), a uniform electric field of 1.92 $ 10 5 N/C is maintained in the region between two plates separated by 1.50 cm. Find the potential difference between the plates.

R

σ

V (V)

72 The magnitude E of an electric field depends on the radial distance r according to E ! A/r 4, where A is a constant with the unit volt – cubic meter. As a multiple of A, what is the magnitude of the electric potential difference between r ! 2.00 m and r ! 3.00 m?

a

z

Figure 24-64 Problem 80. 81 Electron in a well. Figure 2465 shows electric potential Vs V along an x axis. The scale of the vertical axis is set by Vs ! 8.0 V. An electron is to be released at x ! 4.5 cm with initial kinetic enx (cm) 0 1 2 3 4 5 6 7 ergy 3.00 eV. (a) If it is initially moving in the negaFigure 24-65 Problem 81. tive direction of the axis, does it reach a turning point (if so, what is the x coordinate of that point) or does it escape from the plotted region (if so, what is its speed at x ! 0)? (b) If it is initially moving in the positive direction of the axis, does it reach a turning point (if so, what is the x coordinate of that point) or does it escape from the plotted region (if so, what is its speed at x ! 7.0 cm)? What are the (c) magnitude F and (d) direction (positive or negative direction of the x axis) of the electric force on the electron if the electron moves just to the left of x ! 4.0 cm? What are (e) F and (f) the direction if it moves just to the right of x ! 5.0 cm?

71 SSM Starting from Eq. 24-30, derive an expression for the electric field due to a dipole at a point on the dipole axis.

74 Three particles, charge q1 ! %10 mC, q2 ! #20 mC, and q3 ! %30 mC, are positioned at the vertices of an isosceles triangle as shown in Fig. 24-62. If a ! 10 cm and b ! 6.0 cm, how much work must an external agent do to exchange the positions of (a) q1 q2 and q3 and, instead, (b) q1 and q2?

P

84 A solid conducting sphere of radius 3.0 cm has a charge of 30 nC distributed uniformly over its surface. Let A be a point 1.0 cm from the center of the sphere, S be a point on the surface of the sphere, ∞ and B be a point 5.0 cm from the center of the sphere. What are the electric +2e potential differences (a) VS #VB and (b) VA #VB? 85 In Fig. 24-67, we move a particle of charge %2e in from infinity to the x axis. How much work do we do? Distance D is 4.00 m.

+2e

+e D

x D

Figure 24-67 Problem 85.

716

CHAPTE R 24 E LECTR IC POTE NTIAL

y 86 Figure 24-68 shows a hemiP sphere with a charge of 4.00 mC distributed uniformly through its volume. The hemisphere lies on an xy x plane the way half a grapefruit might Figure 24-68 Problem 86. lie face down on a kitchen table. Point P is located on the plane, along a radial line from the hemisphere’s center of curvature, at radial distance 15 cm. What is the electric potential at point P due to the hemisphere?

96 A charge q is distributed uniformly throughout a spherical volume of radius R. Let V ! 0 at infinity. What are (a) V at radial distance r ) R and (b) the potential difference between points at r ! R and the point at r ! 0? 97 SSM A solid copper sphere whose radius is 1.0 cm has a very thin surface coating of nickel. Some of the nickel atoms are radioactive, each atom emitting an electron as it decays. Half of these electrons enter the copper sphere, each depositing 100 keV of energy there. The other half of the electrons escape, each carrying away a charge #e.The nickel coating has an activity of 3.70 $ 10 8 radioactive decays per second. The sphere is hung from a long, nonconducting string and isolated from its surroundings. (a) How long will it take for the potential of the sphere to increase by 1000 V? (b) How long will it take for the temperature of the sphere to increase by 5.0 K due to the energy deposited by the electrons? The heat capacity of the sphere is 14 J/K. 98 In Fig. 24-71, a metal sphere with charge q ! 5.00 mC and radius Q r ! 3.00 cm is concentric with a larger metal sphere with charge Q ! 15.0 mC and radius R ! 6.00 cm. (a) r q What is the potential difference between the spheres? If we connect the R spheres with a wire, what then is the charge on (b) the smaller sphere and (c) the larger sphere?

87 SSM Three %0.12 C charges form an equilateral triangle 1.7 m on a side. Using energy supplied at the rate of 0.83 kW, how many days would be required to move one of the charges to the midpoint of the line joining the other two charges? 88 Two charges q ! %2.0 mC are fixed a distance d ! 2.0 cm apart (Fig. 24-69). (a) With V ! 0 at infinity, what is the electric potential at point C? (b) You bring a third charge q ! %2.0 mC from infinity to C. How much work must you do? (c) What is the potential energy U of the three-charge configuration when the third charge is in place?

C d/2

+

d/2

q

+

d/2

q

Figure 24-69 Problem 88.

89 Initially two electrons are fixed in place with a separation of 2.00 mm. How much work must we do to bring a third electron in from infinity to complete an equilateral triangle? 90 A particle of positive charge Q is fixed at point P. A second particle of mass m and negative charge #q moves at constant speed in a circle of radius r1, centered at P. Derive an expression for the work W that must be done by an external agent on the second particle to increase the radius of the circle of motion to r2. 91 Two charged, parallel, flat conducting surfaces are spaced d ! 1.00 cm apart and produce a potential difference 'V ! 625 V between them. An electron is projected from one surface directly toward the second. What is the initial speed of the electron if it stops just at the second surface? +q –q –q 1

92 In Fig. 24-70, point P is at the center of the rectangle. With V ! 0 at infinity, q1 ! 5.00 fC, q2 ! 2.00 fC, d q3 ! 3.00 fC, and d ! 2.54 cm, what is the net electric potential at P due to +q 3 the six charged particles?

d

2

d

–q 2

3

d

P d

Isolating 99 (a) Using Eq. 24-32, show that stand the electric potential at a point on the central axis of a thin ring (of charge q and radius R) and at disFigure 24-71 Problem 98. tance z from the ring is q 1 . V! 4p´0 2z2 % R2

d

+q 1

93 SSM A uniform charge of %16.0 Figure 24-70 Problem 92. mC is on a thin circular ring lying in an xy plane and centered on the origin. The ring’s radius is 3.00 cm. If point A is at the origin and point B is on the z axis at z ! 4.00 cm, what is VB # VA? 94 Consider a particle with charge q ! 1.50 $ 10#8 C, and take V ! 0 at infinity. (a) What are the shape and dimensions of an equipotential surface having a potential of 30.0 V due to q alone? (b) Are surfaces whose potentials differ by a constant amount (1.0 V, say) evenly spaced? 95 SSM A thick spherical shell of charge Q and uniform volume charge density r is bounded by radii r1 and r2 ( r1. With V ! 0 at infinity, find the electric potential V as a function of distance r from the center of the distribution, considering regions (a) r ( r2, (b) r2 ( r ( r1, and (c) r ) r1. (d) Do these solutions agree with each other at r ! r2 and r ! r1? (Hint: See Module 23-6.)

(b) From this result, derive an expression for the electric field magnitude E at points on the ring’s axis; compare your result with the calculation of E in Module 22-4. 100 An alpha particle (which has two protons) is sent directly toward a target nucleus containing 92 protons. The alpha particle has an initial kinetic energy of 0.48 pJ. What is the least center-to-center distance the alpha particle will be from the target nucleus, assuming the nucleus does not move? 101 In the quark model of fundamental particles, a proton is composed of three quarks: two “up” quarks, each having charge %2e/3, and one “down” quark, having charge #e/3. Suppose that the three quarks are equidistant from one another. Take that separation distance to be 1.32 $ 10#15 m and calculate the electric potential energy of the system of (a) only the two up quarks and (b) all three quarks. 102 A charge of 1.50 $ 10#8 C lies on an isolated metal sphere of radius 16.0 cm. With V ! 0 at infinity, what is the electric potential at points on the sphere’s surface? 103 In Fig. 24-72, two particles of 1 2 P x charges q1 and q2 are fixed to an x axis. If a third particle, of charge d 1.5d %6.0 mC, is brought from an infinite Figure 24-72 Problem 103. distance to point P, the three-particle system has the same electric potential energy as the original two-particle system. What is the charge ratio q1/q2?

C

H

A

P

T

E

R

2

5

Capacitance 25-1 CAPACITANCE Learning Objectives After reading this module, you should be able to . . .

25.01 Sketch a schematic diagram of a circuit with a parallelplate capacitor, a battery, and an open or closed switch. 25.02 In a circuit with a battery, an open switch, and an uncharged capacitor, explain what happens to the conduction electrons when the switch is closed.

25.03 For a capacitor, apply the relationship between the magnitude of charge q on either plate (“the charge on the capacitor”), the potential difference V between the plates (“the potential across the capacitor”), and the capacitance C of the capacitor.

Key Ideas ● A capacitor consists of two isolated conductors (the plates) with charges %q and #q. Its capacitance C is defined from q ! CV,

where V is the potential difference between the plates.

● When a circuit with a battery, an open switch, and an

uncharged capacitor is completed by closing the switch, conduction electrons shift, leaving the capacitor plates with opposite charges.

What Is Physics? One goal of physics is to provide the basic science for practical devices designed by engineers. The focus of this chapter is on one extremely common example—the capacitor, a device in which electrical energy can be stored. For example, the batteries in a camera store energy in the photoflash unit by charging a capacitor. The batteries can supply energy at only a modest rate, too slowly for the photoflash unit to emit a flash of light. However, once the capacitor is charged, it can supply energy at a much greater rate when the photoflash unit is triggered—enough energy to allow the unit to emit a burst of bright light. The physics of capacitors can be generalized to other devices and to any situation involving electric fields. For example, Earth’s atmospheric electric field is modeled by meteorologists as being produced by a huge spherical capacitor that partially discharges via lightning. The charge that skis collect as they slide along snow can be modeled as being stored in a capacitor that frequently discharges as sparks (which can be seen by nighttime skiers on dry snow). The first step in our discussion of capacitors is to determine how much charge can be stored. This “how much” is called capacitance.

Paul Silvermann/Fundamental Photographs

Figure 25-1 An assortment of capacitors.

Capacitance Figure 25-1 shows some of the many sizes and shapes of capacitors. Figure 25-2 shows the basic elements of any capacitor — two isolated conductors of any

+q

–q

Figure 25-2 Two conductors, isolated electrically from each other and from their surroundings, form a capacitor. When the capacitor is charged, the charges on the conductors, or plates as they are called, have the same magnitude q but opposite signs. 717

718

CHAPTE R 25 CAPACITANCE

Figure 25-3 (a) A parallel-plate capacitor, made up of two plates of area A separated by a distance d. The charges on the facing plate surfaces have the same magnitude q but opposite signs. (b) As the field lines show, the electric field due to the charged plates is uniform in the central region between the plates. The field is not uniform at the edges of the plates, as indicated by the “fringing” of the field lines there.

Area A

V

Electric field lines

d Top side of bottom plate has charge –q

Bottom side of top plate has charge +q

A

–q (b)

(a )

shape. No matter what their geometry, flat or not, we call these conductors plates. Figure 25-3a shows a less general but more conventional arrangement, called a parallel-plate capacitor, consisting of two parallel conducting plates of area A separated by a distance d. The symbol we use to represent a capacitor (%&) is based on the structure of a parallel-plate capacitor but is used for capacitors of all geometries. We assume for the time being that no material medium (such as glass or plastic) is present in the region between the plates. In Module 25-5, we shall remove this restriction. When a capacitor is charged, its plates have charges of equal magnitudes but opposite signs: %q and #q. However, we refer to the charge of a capacitor as being q, the absolute value of these charges on the plates. (Note that q is not the net charge on the capacitor, which is zero.) Because the plates are conductors, they are equipotential surfaces; all points on a plate are at the same electric potential. Moreover, there is a potential difference between the two plates. For historical reasons, we represent the absolute value of this potential difference with V rather than with the 'V we used in previous notation. The charge q and the potential difference V for a capacitor are proportional to each other; that is, q ! CV.

h

+

B

l C

1 farad ! 1 F ! 1 coulomb per volt ! 1 C/V.

S (a )

(25-2)

As you will see, the farad is a very large unit. Submultiples of the farad, such as the microfarad (1 mF ! 10#6 F) and the picofarad (1 pF ! 10#12 F), are more convenient units in practice.

Charging a Capacitor

Terminal

C l

h V S

Terminal

(25-1)

The proportionality constant C is called the capacitance of the capacitor. Its value depends only on the geometry of the plates and not on their charge or potential difference. The capacitance is a measure of how much charge must be put on the plates to produce a certain potential difference between them: The greater the capacitance, the more charge is required. The SI unit of capacitance that follows from Eq. 25-1 is the coulomb per volt. This unit occurs so often that it is given a special name, the farad (F):



+ B–

+q

(b)

Figure 25-4 (a) Battery B, switch S, and plates h and l of capacitor C, connected in a circuit. (b) A schematic diagram with the circuit elements represented by their symbols.

One way to charge a capacitor is to place it in an electric circuit with a battery. An electric circuit is a path through which charge can flow. A battery is a device that maintains a certain potential difference between its terminals (points at which charge can enter or leave the battery) by means of internal electrochemical reactions in which electric forces can move internal charge. In Fig. 25-4a, a battery B, a switch S, an uncharged capacitor C, and interconnecting wires form a circuit. The same circuit is shown in the schematic diagram of Fig. 25-4b, in which the symbols for a battery, a switch, and a capacitor represent those devices. The battery maintains potential difference V between its terminals. The terminal of higher potential is labeled % and is often called the positive terminal; the terminal of lower potential is labeled # and is often called the negative terminal.

25-2 CALCU L ATI NG TH E CAPACITANCE

719

The circuit shown in Figs. 25-4a and b is said to be incomplete because switch S is open; that is, the switch does not electrically connect the wires attached to it. When the switch is closed, electrically connecting those wires, the circuit is complete and charge can then flow through the switch and the wires. As we discussed in Chapter 21, the charge that can flow through a conductor, such as a wire, is that of electrons. When the circuit of Fig. 25-4 is completed, electrons are driven through the wires by an electric field that the battery sets up in the wires. The field drives electrons from capacitor plate h to the positive terminal of the battery; thus, plate h, losing electrons, becomes positively charged. The field drives just as many electrons from the negative terminal of the battery to capacitor plate l; thus, plate l, gaining electrons, becomes negatively charged just as much as plate h, losing electrons, becomes positively charged. Initially, when the plates are uncharged, the potential difference between them is zero. As the plates become oppositely charged, that potential difference increases until it equals the potential difference V between the terminals of the battery. Then plate h and the positive terminal of the battery are at the same potential, and there is no longer an electric field in the wire between them. Similarly, plate l and the negative terminal reach the same potential, and there is then no electric field in the wire between them. Thus, with the field zero, there is no further drive of electrons. The capacitor is then said to be fully charged, with a potential difference V and charge q that are related by Eq. 25-1. In this book we assume that during the charging of a capacitor and afterward, charge cannot pass from one plate to the other across the gap separating them. Also, we assume that a capacitor can retain (or store) charge indefinitely, until it is put into a circuit where it can be discharged.

Checkpoint 1 Does the capacitance C of a capacitor increase, decrease, or remain the same (a) when the charge q on it is doubled and (b) when the potential difference V across it is tripled?

25-2 CALCULATING THE CAPACITANCE Learning Objectives After reading this module, you should be able to . . .

25.04 Explain how Gauss’ law is used to find the capacitance of a parallel-plate capacitor. 25.05 For a parallel-plate capacitor, a cylindrical capacitor, a spherical capacitor, and an isolated sphere, calculate the capacitance.

Key Ideas ● We generally determine the capacitance of a particular capacitor configuration by (1) assuming a charge q to have : been placed on the plates, (2) finding the electric field E due to this charge, (3) evaluating the potential difference V between the plates, and (4) calculating C from q ! CV. Some results are the following: ● A parallel-plate capacitor with flat parallel plates of area A

and spacing d has capacitance ´A C! 0 . d ● A cylindrical capacitor (two long coaxial cylinders) of length

L and radii a and b has capacitance C ! 2p´0

L . ln(ba)

● A spherical capacitor with concentric spherical plates of

radii a and b has capacitance C ! 4p´0

ab . b#a

● An isolated sphere of radius R has capacitance

C ! 4p´0R.

720

CHAPTE R 25 CAPACITANCE

Calculating the Capacitance Our goal here is to calculate the capacitance of a capacitor once we know its geometry. Because we shall consider a number of different geometries, it seems wise to develop a general plan to simplify the work. In brief our plan is as follows: : (1) Assume a charge q on the plates; (2) calculate the electric field E between : the plates in terms of this charge, using Gauss’ law; (3) knowing E , calculate the potential difference V between the plates from Eq. 24-18; (4) calculate C from Eq. 25-1. Before we start, we can simplify the calculation of both the electric field and the potential difference by making certain assumptions. We discuss each in turn.

Calculating the Electric Field :

To relate the electric field E between the plates of a capacitor to the charge q on either plate, we shall use Gauss’ law: ´0

,

:

:

(25-3)

E " dA ! q. :

:

Here q is the charge enclosed by a Gaussian surface and - E " dA is the net electric flux through that surface. In all cases that we shall consider, the Gaussian : surface will be such that whenever there is an electric flux through it, E will have : : a uniform magnitude E and the vectors E and dA will be parallel. Equation 25-3 then reduces to q ! ´0EA

(special case of Eq. 25-3),

(25-4)

in which A is the area of that part of the Gaussian surface through which there is a flux. For convenience, we shall always draw the Gaussian surface in such a way that it completely encloses the charge on the positive plate; see Fig. 25-5 for an example.

Calculating the Potential Difference In the notation of Chapter 24 (Eq. 24-18), the potential difference between : the plates of a capacitor is related to the field E by Vf # Vi ! #

We use Gauss’ law to relate q and E. Then we integrate the E to get the potential difference. + + + + + + + + + + +q d A –q – – – – – – – – – –

Gaussian surface

Path of integration

Figure 25-5 A charged parallel-plate capacitor. A Gaussian surface encloses the charge on the positive plate. The integration of Eq. 25-6 is taken along a path extending directly from the negative plate to the positive plate.

"

f

i

:

E " ds:,

(25-5)

in which the integral is to be evaluated along any path that starts on one plate and ends on the other. We shall always choose a path that follows an electric field line, from the negative plate to the positive plate. For this path, the vectors : : s will have opposite directions; so the dot product E " ds: will be equal E and d : to #E ds. Thus, the right side of Eq. 25-5 will then be positive. Letting V represent the difference Vf # Vi , we can then recast Eq. 25-5 as V!

"

%

E ds

(special case of Eq. 25-5),

(25-6)

#

in which the # and % remind us that our path of integration starts on the negative plate and ends on the positive plate. We are now ready to apply Eqs. 25-4 and 25-6 to some particular cases.

A Parallel-Plate Capacitor We assume, as Fig. 25-5 suggests, that the plates of our parallel-plate capacitor are so large and so close together that we can neglect the fringing of the electric field

721

25-2 CALCU L ATI NG TH E CAPACITANCE :

at the edges of the plates, taking E to be constant throughout the region between the plates. We draw a Gaussian surface that encloses just the charge q on the positive plate, as in Fig. 25-5. From Eq. 25-4 we can then write (25-7)

q ! ´0EA, where A is the area of the plate. Equation 25-6 yields V!

"

%

"

d

E ds ! E

0

#

(25-8)

ds ! Ed.

In Eq. 25-8, E can be placed outside the integral because it is a constant; the second integral then is simply the plate separation d. If we now substitute q from Eq. 25-7 and V from Eq. 25-8 into the relation q ! CV (Eq. 25-1), we find C!

+0 A d

(25-9)

(parallel-plate capacitor).

Thus, the capacitance does indeed depend only on geometrical factors — namely, the plate area A and the plate separation d. Note that C increases as we increase area A or decrease separation d. As an aside, we point out that Eq. 25-9 suggests one of our reasons for writing the electrostatic constant in Coulomb’s law in the form 1/4p´0. If we had not done so, Eq. 25-9 — which is used more often in engineering practice than Coulomb’s law — would have been less simple in form. We note further that Eq. 25-9 permits us to express the permittivity constant ´0 in a unit more appropriate for use in problems involving capacitors; namely, ´0 ! 8.85 $ 10#12 F/m ! 8.85 pF/m.

(25-10)

We have previously expressed this constant as ´0 ! 8.85 $ 10#12 C 2/N &m2.

(25-11) Total charge +q

A Cylindrical Capacitor Figure 25-6 shows, in cross section, a cylindrical capacitor of length L formed by two coaxial cylinders of radii a and b. We assume that L * b so that we can neglect the fringing of the electric field that occurs at the ends of the cylinders. Each plate contains a charge of magnitude q. As a Gaussian surface, we choose a cylinder of length L and radius r, closed by end caps and placed as is shown in Fig. 25-6. It is coaxial with the cylinders and encloses the central cylinder and thus also the charge q on that cylinder. Equation 25-4 then relates that charge and the field magnitude E as q ! ´0EA ! ´0E(2prL),

q . 2p´0Lr

(25-12)

Substitution of this result into Eq. 25-6 yields V!

"

%

#

E ds ! #

q 2p´0L

"

a

b





– –

– –



+ ++ + + + b + + a + + + r ++ + + ++ +

– –

– –



in which 2prL is the area of the curved part of the Gaussian surface. There is no flux through the end caps. Solving for E yields E!



Total charge –q

# $

dr b q , ! ln r 2p´0L a

(25-13)

where we have used the fact that here ds ! #dr (we integrated radially inward).

– –



Path of integration



– Gaussian surface

Figure 25-6 A cross section of a long cylindrical capacitor, showing a cylindrical Gaussian surface of radius r (that encloses the positive plate) and the radial path of integration along which Eq. 25-6 is to be applied. This figure also serves to illustrate a spherical capacitor in a cross section through its center.

722

CHAPTE R 25 CAPACITANCE

From the relation C ! q/V, we then have L ln(b/a)

C ! 2p´0

(25-14)

(cylindrical capacitor).

We see that the capacitance of a cylindrical capacitor, like that of a parallel-plate capacitor, depends only on geometrical factors, in this case the length L and the two radii b and a.

A Spherical Capacitor Figure 25-6 can also serve as a central cross section of a capacitor that consists of two concentric spherical shells, of radii a and b. As a Gaussian surface we draw a sphere of radius r concentric with the two shells; then Eq. 25-4 yields q ! ´0EA ! ´0E(4pr 2), in which 4pr2 is the area of the spherical Gaussian surface. We solve this equation for E, obtaining q 1 , (25-15) E! 4p´0 r2 which we recognize as the expression for the electric field due to a uniform spherical charge distribution (Eq. 23-15). If we substitute this expression into Eq. 25-6, we find V!

"

%

E ds ! #

#

q 4p´0

"

a

b

dr q ! r2 4p´0

# 1a # b1 $ ! 4p´q

0

b#a , (25-16) ab

where again we have substituted #dr for ds. If we now substitute Eq. 25-16 into Eq. 25-1 and solve for C, we find C ! 4p´0

ab b#a

(spherical capacitor).

(25-17)

An Isolated Sphere We can assign a capacitance to a single isolated spherical conductor of radius R by assuming that the “missing plate” is a conducting sphere of infinite radius. After all, the field lines that leave the surface of a positively charged isolated conductor must end somewhere; the walls of the room in which the conductor is housed can serve effectively as our sphere of infinite radius. To find the capacitance of the conductor, we first rewrite Eq. 25-17 as a . C ! 4p´0 1 # a/b If we then let b : , and substitute R for a, we find C ! 4p´0R

(isolated sphere).

(25-18)

Note that this formula and the others we have derived for capacitance (Eqs. 25-9, 25-14, and 25-17) involve the constant ´0 multiplied by a quantity that has the dimensions of a length.

Checkpoint 2 For capacitors charged by the same battery, does the charge stored by the capacitor increase, decrease, or remain the same in each of the following situations? (a) The plate separation of a parallel-plate capacitor is increased. (b) The radius of the inner cylinder of a cylindrical capacitor is increased. (c) The radius of the outer spherical shell of a spherical capacitor is increased.

723

25-3 CAPACITORS I N PARALLE L AN D I N SE R I ES

Sample Problem 25.01 Charging the plates in a parallel-plate capacitor In Fig. 25-7a, switch S is closed to connect the uncharged capacitor of capacitance C ! 0.25 mF to the battery of potential difference V ! 12 V. The lower capacitor plate has thickness L ! 0.50 cm and face area A ! 2.0 $ 10#4 m2, and it consists of copper, in which the density of conduction electrons is n ! 8.49 $ 1028 electrons/m3. From what depth d within the plate (Fig. 25-7b) must electrons move to the plate face as the capacitor becomes charged?

magnitude that collects there is q ! CV ! (0.25 $ 10#6 F)(12 V) ! 3.0 $ 10#6 C. Dividing this result by e gives us the number N of conduction electrons that come up to the face: N!

3.0 $ 10 #6 C q ! e 1.602 $ 10 #19 C

! 1.873 $ 1013 electrons.

KEY IDEA The charge collected on the plate is related to the capacitance and the potential difference across the capacitor by Eq. 25-1 (q ! CV). Calculations: Because the lower plate is connected to the negative terminal of the battery, conduction electrons move up to the face of the plate. From Eq. 25-1, the total charge Figure 25-7 (a) A battery and capacitor circuit. (b) The lower capacitor plate.

S C







(a)







These electrons come from a volume that is the product of the face area A and the depth d we seek. Thus, from the density of conduction electrons (number per volume), we can write N , n! Ad or d!

! 1.1 $ 10#12 m ! 1.1 pm.

d

(b)

N 1.873 $ 1013 electrons ! #4 An (2.0 $ 10 m2)(8.49 $ 1028 electrons/m3) (Answer)

We commonly say that electrons move from the battery to the negative face but, actually, the battery sets up an electric field in the wires and plate such that electrons very close to the plate face move up to the negative face.

Additional examples, video, and practice available at WileyPLUS

25-3 CAPACITORS IN PARALLEL AND IN SERIES Learning Objectives After reading this module, you should be able to . . .

25.06 Sketch schematic diagrams for a battery and (a) three capacitors in parallel and (b) three capacitors in series. 25.07 Identify that capacitors in parallel have the same potential difference, which is the same value that their equivalent capacitor has. 25.08 Calculate the equivalent of parallel capacitors. 25.09 Identify that the total charge stored on parallel capacitors is the sum of the charges stored on the individual capacitors. 25.10 Identify that capacitors in series have the same charge, which is the same value that their equivalent capacitor has. 25.11 Calculate the equivalent of series capacitors. 25.12 Identify that the potential applied to capacitors in series is equal to the sum of the potentials across the individual capacitors.

25.13 For a circuit with a battery and some capacitors in parallel and some in series, simplify the circuit in steps by finding equivalent capacitors, until the charge and potential on the final equivalent capacitor can be determined, and then reverse the steps to find the charge and potential on the individual capacitors. 25.14 For a circuit with a battery, an open switch, and one or more uncharged capacitors, determine the amount of charge that moves through a point in the circuit when the switch is closed. 25.15 When a charged capacitor is connected in parallel to one or more uncharged capacitors, determine the charge and potential difference on each capacitor when equilibrium is reached.

Key Idea ● The equivalent capacitances Ceq of combinations of individual

capacitors connected in parallel and in series can be found from Ceq !

n

' Cj j!1

(n capacitors in parallel)

and

n 1 1 ! ' Ceq C j!1 j

(n capacitors in series).

Equivalent capacitances can be used to calculate the capacitances of more complicated series – parallel combinations.

724

CHAPTE R 25 CAPACITANCE

Capacitors in Parallel and in Series

Terminal + B– (a)

+q 3 V

–q 3 C 3

V

+q 2 –q 2 C 2

V

+q 1

V –q 1 C 1

Terminal

Parallel capacitors and their equivalent have the same V (“par-V”). + B–

+q V

–q Ceq

(b)

Figure 25-8 (a) Three capacitors connected in parallel to battery B. The battery maintains potential difference V across its terminals and thus across each capacitor. (b) The equivalent capacitor, with capacitance Ceq, replaces the parallel combination.

When there is a combination of capacitors in a circuit, we can sometimes replace that combination with an equivalent capacitor — that is, a single capacitor that has the same capacitance as the actual combination of capacitors. With such a replacement, we can simplify the circuit, affording easier solutions for unknown quantities of the circuit. Here we discuss two basic combinations of capacitors that allow such a replacement.

Capacitors in Parallel Figure 25-8a shows an electric circuit in which three capacitors are connected in parallel to battery B. This description has little to do with how the capacitor plates are drawn. Rather, “in parallel” means that the capacitors are directly wired together at one plate and directly wired together at the other plate, and that the same potential difference V is applied across the two groups of wired-together plates. Thus, each capacitor has the same potential difference V, which produces charge on the capacitor. (In Fig. 25-8a, the applied potential V is maintained by the battery.) In general: When a potential difference V is applied across several capacitors connected in parallel, that potential difference V is applied across each capacitor.The total charge q stored on the capacitors is the sum of the charges stored on all the capacitors.

When we analyze a circuit of capacitors in parallel, we can simplify it with this mental replacement: Capacitors connected in parallel can be replaced with an equivalent capacitor that has the same total charge q and the same potential difference V as the actual capacitors.

(You might remember this result with the nonsense word “par-V,” which is close to “party,” to mean “capacitors in parallel have the same V.”) Figure 25-8b shows the equivalent capacitor (with equivalent capacitance Ceq) that has replaced the three capacitors (with actual capacitances C1, C2, and C3) of Fig. 25-8a. To derive an expression for Ceq in Fig. 25-8b, we first use Eq. 25-1 to find the charge on each actual capacitor: q1 ! C1V,

q2 ! C2V,

and q3 ! C3V.

The total charge on the parallel combination of Fig. 25-8a is then q ! q1 % q2 % q3 ! (C1 % C2 % C3)V. The equivalent capacitance, with the same total charge q and applied potential difference V as the combination, is then q Ceq ! ! C1 % C2 % C3, V a result that we can easily extend to any number n of capacitors, as Ceq !

n

' Cj j!1

(n capacitors in parallel).

(25-19)

Thus, to find the equivalent capacitance of a parallel combination, we simply add the individual capacitances.

Capacitors in Series Figure 25-9a shows three capacitors connected in series to battery B.This description has little to do with how the capacitors are drawn. Rather,“in series” means that the capacitors are wired serially, one after the other, and that a potential difference V is

25-3 CAPACITORS I N PARALLE L AN D I N SE R I ES

applied across the two ends of the series. (In Fig. 25-9a, this potential difference V is maintained by battery B.) The potential differences that then exist across the capacitors in the series produce identical charges q on them. When a potential difference V is applied across several capacitors connected in series, the capacitors have identical charge q. The sum of the potential differences across all the capacitors is equal to the applied potential difference V.

We can explain how the capacitors end up with identical charge by following a chain reaction of events, in which the charging of each capacitor causes the charging of the next capacitor.We start with capacitor 3 and work upward to capacitor 1.When the battery is first connected to the series of capacitors, it produces charge #q on the bottom plate of capacitor 3. That charge then repels negative charge from the top plate of capacitor 3 (leaving it with charge %q). The repelled negative charge moves to the bottom plate of capacitor 2 (giving it charge #q). That charge on the bottom plate of capacitor 2 then repels negative charge from the top plate of capacitor 2 (leaving it with charge %q) to the bottom plate of capacitor 1 (giving it charge #q). Finally, the charge on the bottom plate of capacitor 1 helps move negative charge from the top plate of capacitor 1 to the battery, leaving that top plate with charge %q. Here are two important points about capacitors in series: 1. When charge is shifted from one capacitor to another in a series of capacitors, it can move along only one route, such as from capacitor 3 to capacitor 2 in Fig. 25-9a. If there are additional routes, the capacitors are not in series. 2. The battery directly produces charges on only the two plates to which it is connected (the bottom plate of capacitor 3 and the top plate of capacitor 1 in Fig. 25-9a). Charges that are produced on the other plates are due merely to the shifting of charge already there. For example, in Fig. 25-9a, the part of the circuit enclosed by dashed lines is electrically isolated from the rest of the circuit. Thus, its charge can only be redistributed. When we analyze a circuit of capacitors in series, we can simplify it with this mental replacement: Capacitors that are connected in series can be replaced with an equivalent capacitor that has the same charge q and the same total potential difference V as the actual series capacitors.

(You might remember this with the nonsense word “seri-q” to mean “capacitors in series have the same q.”) Figure 25-9b shows the equivalent capacitor (with equivalent capacitance Ceq) that has replaced the three actual capacitors (with actual capacitances C1, C2, and C3) of Fig. 25-9a. To derive an expression for Ceq in Fig. 25-9b, we first use Eq. 25-1 to find the potential difference of each actual capacitor: q q q , V2 ! , and V3 ! . V1 ! C1 C2 C3 The total potential difference V due to the battery is the sum V ! V1 % V2 % V3 ! q

# C1

1

%

The equivalent capacitance is then q 1 , ! Ceq ! V 1/C1 % 1/C2 % 1/C3 or

$

1 1 % . C2 C3

1 1 1 1 ! % % . Ceq C1 C2 C3

725

Terminal +q V1

+ B–

–q C 1 +q

V2

V

–q C 2 +q

V3

–q C 3

Terminal (a)

+ B–

Series capacitors and their equivalent have the same q (“seri-q”).

+q V

–q Ceq (b)

Figure 25-9 (a) Three capacitors connected in series to battery B. The battery maintains potential difference V between the top and bottom plates of the series combination. (b) The equivalent capacitor, with capacitance Ceq, replaces the series combination.

726

CHAPTE R 25 CAPACITANCE

We can easily extend this to any number n of capacitors as n 1 1 ! ' Ceq C j!1 j

(25-20)

(n capacitors in series).

Using Eq. 25-20 you can show that the equivalent capacitance of a series of capacitances is always less than the least capacitance in the series.

Checkpoint 3 A battery of potential V stores charge q on a combination of two identical capacitors. What are the potential difference across and the charge on either capacitor if the capacitors are (a) in parallel and (b) in series?

Sample Problem 25.02 Capacitors in parallel and in series (a) Find the equivalent capacitance for the combination of capacitances shown in Fig. 25-10a, across which potential difference V is applied. Assume C1 ! 12.0 mF,

C2 ! 5.30 mF,

parallel can be replaced with their equivalent capacitor. Therefore, we should first check whether any of the capacitors in Fig. 25-10a are in parallel or series. Finding equivalent capacitance: Capacitors 1 and 3 are connected one after the other, but are they in series? No. The potential V that is applied to the capacitors produces charge on the bottom plate of capacitor 3. That charge causes charge to shift from the top plate of capacitor 3. However, note that the shifting charge can move to the bot-

and C3 ! 4.50 mF.

KEY IDEA Any capacitors connected in series can be replaced with their equivalent capacitor, and any capacitors connected in

A

We first reduce the circuit to a single capacitor.

The equivalent of parallel capacitors is larger.

Next, we work backwards to the desired capacitor.

A

A C1 = 12.0 µ F V

B

(a)

Series capacitors and their equivalent have the same q (“seri-q”). q 12 = 44.6 µC C 12 = 17.3 µF q3 = 44.6 µ C C3 = 4.50 µ F (f )

Applying q = CV yields the charge. q 123 = 44.6 µ C

C 12 = 17.3 µF

C2 = 5.30 µ F C3 = 4.50 µ F

12.5 V

The equivalent of series capacitors is smaller.

B

V

V

C 123 = 3.57 µF

12.5 V

C3 = 4.50 µ F (b)

Applying V = q/C yields the potential difference. q 12 = 44.6 µC V 12 = C 12 = 17.3 µF 2.58 V 12.5 V q 3 = 44.6 µ C V3 = C3 = 4.50 µ F 9.92 V (g )

C 123 = 3.57 µF

(d )

(c )

Parallel capacitors and their equivalent have the same V (“par-V”).

C1 = V1 = V2 = C2 = 12.0 µF 2.58 V 5.30 µ F 2.58 V 12.5 V

q3 = 44.6 µ C C3 = V3 = 4.50 µ F 9.92 V (h)

V 123 = 12.5 V

12.5 V

C 123 = V 123 = 3.57 µF 12.5 V

(e )

Applying q = CV yields the charge. q2 = q1 = 31.0 µC 13.7 µ C V1 = C1 = V2 = C2 = 12.0 µ F 2.58 V 5.30 µ F 2.58 V q3 = 12.5 V 44.6 µ C V3 = C3 = 4.50 µ F 9.92 V (i )

Figure 25-10 (a)–(d) Three capacitors are reduced to one equivalent capacitor. (e)–(i) Working backwards to get the charges.

25-3 CAPACITORS I N PARALLE L AN D I N SE R I ES

tom plates of both capacitor 1 and capacitor 2. Because there is more than one route for the shifting charge, capacitor 3 is not in series with capacitor 1 (or capacitor 2). Any time you think you might have two capacitors in series, apply this check about the shifting charge. Are capacitor 1 and capacitor 2 in parallel? Yes. Their top plates are directly wired together and their bottom plates are directly wired together, and electric potential is applied between the top-plate pair and the bottom-plate pair. Thus, capacitor 1 and capacitor 2 are in parallel, and Eq. 25-19 tells us that their equivalent capacitance C12 is C12 ! C1 % C2 ! 12.0 mF % 5.30 mF ! 17.3 mF. In Fig. 25-10b, we have replaced capacitors 1 and 2 with their equivalent capacitor, called capacitor 12 (say “one two” and not “twelve”). (The connections at points A and B are exactly the same in Figs. 25-10a and b.) Is capacitor 12 in series with capacitor 3? Again applying the test for series capacitances, we see that the charge that shifts from the top plate of capacitor 3 must entirely go to the bottom plate of capacitor 12. Thus, capacitor 12 and capacitor 3 are in series, and we can replace them with their equivalent C123 (“one two three”), as shown in Fig. 25-10c. From Eq. 25-20, we have

from which 1 ! 3.57 mF. 0.280 mF#1

KEY IDEAS We now need to work backwards from the equivalent capacitance to get the charge on a particular capacitor. We have two techniques for such “backwards work”: (1) Seri-q: Series capacitors have the same charge as their equivalent capacitor. (2) Par-V: Parallel capacitors have the same potential difference as their equivalent capacitor. Working backwards: To get the charge q1 on capacitor 1, we work backwards to that capacitor, starting with the equivalent capacitor 123. Because the given potential difference V (! 12.5 V) is applied across the actual combination of three capacitors in Fig. 25-10a, it is also applied across C123 in Figs. 25-10d and e. Thus, Eq. 25-1 (q ! CV) gives us q123 ! C123V ! (3.57 mF)(12.5 V) ! 44.6 mC. The series capacitors 12 and 3 in Fig. 25-10b each have the same charge as their equivalent capacitor 123 (Fig. 25-10f ). Thus, capacitor 12 has charge q12 ! q123 ! 44.6 mC. From Eq. 25-1 and Fig. 25-10g, the potential difference across capacitor 12 must be q12 44.6 mC ! ! 2.58 V. C12 17.3 mF

The parallel capacitors 1 and 2 each have the same potential difference as their equivalent capacitor 12 (Fig. 25-10h). Thus, capacitor 1 has potential difference V1 ! V12 ! 2.58 V, and, from Eq. 25-1 and Fig. 25-10i, the charge on capacitor 1 must be

1 1 % ! 0.280 mF#1, ! 17.3 mF 4.50 mF

C123 !

(b) The potential difference applied to the input terminals in Fig. 25-10a is V ! 12.5 V. What is the charge on C1?

V12 !

1 1 1 ! % C123 C12 C3

727

(Answer)

q1 ! C1V1 ! (12.0 mF)(2.58 V) ! 31.0 mC.

(Answer)

Sample Problem 25.03 One capacitor charging up another capacitor Capacitor 1, with C1 ! 3.55 mF, is charged to a potential difference V0 ! 6.30 V, using a 6.30 V battery. The battery is then removed, and the capacitor is connected as in Fig. 25-11 to an uncharged capacitor 2, with C2 ! 8.95 mF. When switch S is closed, charge flows between the capacitors. Find the charge on each capacitor when equilibrium is reached.

As the electric potential across capacitor 1 decreases, that across capacitor 2 increases. Equilibrium is reached when the two potentials are equal because, with no potential difference between connected plates of the capacitors, there

After the switch is closed, charge is transferred until the potential differences match.

KEY IDEAS The situation here differs from the previous example because here an applied electric potential is not maintained across a combination of capacitors by a battery or some other source. Here, just after switch S is closed, the only applied electric potential is that of capacitor 1 on capacitor 2, and that potential is decreasing. Thus, the capacitors in Fig. 25-11 are not connected in series; and although they are drawn parallel, in this situation they are not in parallel.

Figure 25-11 A potential difference V0 is applied to capacitor 1 and the charging battery is removed. Switch S is then closed so that the charge on capacitor 1 is shared with capacitor 2.

S

q0 C1

C2

728

CHAPTE R 25 CAPACITANCE

is no electric field within the connecting wires to move conduction electrons. The initial charge on capacitor 1 is then shared between the two capacitors.

Because the total charge cannot magically change, the total after the transfer must be

Calculations: Initially, when capacitor 1 is connected to the battery, the charge it acquires is, from Eq. 25-1,

thus

q0 ! C1V0 ! (3.55 $ 10#6 F)(6.30 V) ! 22.365 $ 10#6 C. When switch S in Fig. 25-11 is closed and capacitor 1 begins to charge capacitor 2, the electric potential and charge on capacitor 1 decrease and those on capacitor 2 increase until V1 ! V2

(equilibrium).

q1 % q2 ! q0

(charge conservation);

q2 ! q0 # q1.

We can now rewrite the second equilibrium equation as q0 # q1 q1 ! . C1 C2 Solving this for q1 and substituting given data, we find q1 ! 6.35 mC.

(Answer)

The rest of the initial charge (q0 ! 22.365 mC) must be on capacitor 2:

From Eq. 25-1, we can rewrite this as q1 q2 (equilibrium). ! C1 C2

q2 ! 16.0 mC.

(Answer)

Additional examples, video, and practice available at WileyPLUS

25-4 ENERGY STORED IN AN ELECTRIC FIELD Learning Objectives After reading this module, you should be able to . . .

25.16 Explain how the work required to charge a capacitor results in the potential energy of the capacitor. 25.17 For a capacitor, apply the relationship between the potential energy U, the capacitance C, and the potential difference V. 25.18 For a capacitor, apply the relationship between the

potential energy, the internal volume, and the internal energy density. 25.19 For any electric field, apply the relationship between the potential energy density u in the field and the field’s magnitude E. 25.20 Explain the danger of sparks in airborne dust.

Key Ideas ● The electric potential energy U of a charged capacitor,

U!

2

q ! 12CV 2, 2c

is equal to the work required to charge the capacitor. This en: ergy can be associated with the capacitor’s electric field E .

● Every electric field, in a capacitor or from any other source, has an associated stored energy. In vacuum, the energy density u (potential energy per unit volume) in a field of magnitude E is

u ! 12´0E2.

Energy Stored in an Electric Field Work must be done by an external agent to charge a capacitor. We can imagine doing the work ourselves by transferring electrons from one plate to the other, one by one. As the charges build, so does the electric field between the plates, which opposes the continued transfer. So, greater amounts of work are required. Actually, a battery does all this for us, at the expense of its stored chemical energy. We visualize the work as being stored as electric potential energy in the electric field between the plates.

25-4 E N E RGY STOR E D I N AN E LECTR IC FI E LD

Suppose that, at a given instant, a charge q- has been transferred from one plate of a capacitor to the other. The potential difference V- between the plates at that instant will be q-/C. If an extra increment of charge dq- is then transferred, the increment of work required will be, from Eq. 24-6, qdq-. C The work required to bring the total capacitor charge up to a final value q is dW ! V- dq- !

W!

"

dW !

1 C

"

q

0

q- dq- !

q2 . 2C

This work is stored as potential energy U in the capacitor, so that U!

q2 2C

(potential energy).

(25-21)

From Eq. 25-1, we can also write this as U ! 12 CV 2

(potential energy).

(25-22)

Equations 25-21 and 25-22 hold no matter what the geometry of the capacitor is. To gain some physical insight into energy storage, consider two parallelplate capacitors that are identical except that capacitor 1 has twice the plate separation of capacitor 2. Then capacitor 1 has twice the volume between its plates and also, from Eq. 25-9, half the capacitance of capacitor 2. Equation 254 tells us that if both capacitors have the same charge q, the electric fields between their plates are identical. And Eq. 25-21 tells us that capacitor 1 has twice the stored potential energy of capacitor 2. Thus, of two otherwise identical capacitors with the same charge and same electric field, the one with twice the volume between its plates has twice the stored potential energy. Arguments like this tend to verify our earlier assumption: The potential energy of a charged capacitor may be viewed as being stored in the electric field between its plates.

Explosions in Airborne Dust As we discussed in Module 24-8, making contact with certain materials, such as clothing, carpets, and even playground slides, can leave you with a significant electrical potential. You might become painfully aware of that potential if a spark leaps between you and a grounded object, such as a faucet. In many industries involving the production and transport of powder, such as in the cosmetic and food industries, such a spark can be disastrous. Although the powder in bulk may not burn at all, when individual powder grains are airborne and thus surrounded by oxygen, they can burn so fiercely that a cloud of the grains burns as an explosion. Safety engineers cannot eliminate all possible sources of sparks in the powder industries. Instead, they attempt to keep the amount of energy available in the sparks below the threshold value Ut (% 150 mJ) typically required to ignite airborne grains. Suppose a person becomes charged by contact with various surfaces as he walks through an airborne powder.We can roughly model the person as a spherical capacitor of radius R ! 1.8 m. From Eq. 25-18 (C ! 4p´ 0R) and Eq. 25-22 (U ! 12CV 2), we see that the energy .f the capacitor is U ! 12 (4p´ 0R)V 2.

729

730

CHAPTE R 25 CAPACITANCE

From this we see that the threshold energy corresponds to a potential of V!

2Ut 2(150 $ 10 #3 J) ! A 4p´0R A 4p(8.85 $ 10#12 C 2/N&m2)(1.8 m)

! 3.9 $ 104 V.

Safety engineers attempt to keep the potential of the personnel below this level by “bleeding” off the charge through, say, a conducting floor.

Energy Density In a parallel-plate capacitor, neglecting fringing, the electric field has the same value at all points between the plates. Thus, the energy density u — that is, the potential energy per unit volume between the plates — should also be uniform. We can find u by dividing the total potential energy by the volume Ad of the space between the plates. Using Eq. 25-22, we obtain u!

U CV 2 ! . Ad 2Ad

(25-23)

With Eq. 25-9 (C ! ´0 A/d), this result becomes u ! 12 ´0

# Vd $ . 2

(25-24)

However, from Eq. 24-42 (E ! #'V/'s), V/d equals the electric field magnitude E; so u ! 12 ´0E2

(energy density).

(25-25)

Although we derived this result for the special case of an electric field of a : parallel-plate capacitor, it holds for any electric field. If an electric field E exists at any point in space, that site has an electric potential energy with a density (amount per unit volume) given by Eq. 25-25.

Sample Problem 25.04 Potential energy and energy density of an electric field An isolated conducting sphere whose radius R is 6.85 cm has a charge q ! 1.25 nC. (a) How much potential energy is stored in the electric field of this charged conductor? KEY IDEAS

(b) What is the energy density at the surface of the sphere? KEY IDEA The density u of the energy stored in an electric field depends on the magnitude E of the field, according to Eq. 25-25 (u ! 12´0E2).

(1) An isolated sphere has capacitance given by Eq. 25-18 (C ! 4p´0R). (2) The energy U stored in a capacitor depends on the capacitor’s charge q and capacitance C according to Eq. 25-21 (U ! q2/2C).

Calculations: Here we must first find E at the surface of the sphere, as given by Eq. 23-15:

Calculation: Substituting C ! 4p´0R into Eq. 25-21 gives us

The energy density is then

q2 q2 ! U! 2C 8p´0R !

u ! 12´0E2 !

(1.25 $ 10#9 C)2 (8p)(8.85 $ 10#12 F/m)(0.0685 m)

! 1.03 $ 10#7 J ! 103 nJ.

E!

! (Answer)

1 q . 4p´0 R2

q2 32p 2´0R4

(1.25 $ 10#9 C)2 (32p 2)(8.85 $ 10#12 C2/N&m2)(0.0685 m)4

! 2.54 $ 10#5 J/m3 ! 25.4 mJ/m3.

Additional examples, video, and practice available at WileyPLUS

(Answer)

25-5 CAPACITOR WITH A DI E LECTR IC

731

25-5 CAPACITOR WITH A DIELECTRIC Learning Objectives After reading this module, you should be able to . . .

25.21 Identify that capacitance is increased if the space between the plates is filled with a dielectric material. 25.22 For a capacitor, calculate the capacitance with and without a dielectric. 25.23 For a region filled with a dielectric material with a given dielectric constant k, identify that all electrostatic equations containing the permittivity constant ´0 are modified by multiplying that constant by the dielectric constant to get k´0.

25.24 Name some of the common dielectrics. 25.25 In adding a dielectric to a charged capacitor, distinguish the results for a capacitor (a) connected to a battery and (b) not connected to a battery. 25.26 Distinguish polar dielectrics from nonpolar dielectrics. 25.27 In adding a dielectric to a charged capacitor, explain what happens to the electric field between the plates in terms of what happens to the atoms in the dielectric.

Key Ideas ● If the space between the plates of a capacitor is completely

filled with a dielectric material, the capacitance C in vacuum (or, effectively, in air) is multiplied by the material’s dielectric constant k, which is a number greater than 1. ● In a region that is completely filled by a dielectric, all

electrostatic equations containing the permittivity constant ´0 must be modified by replacing ´0 with k´0.

● When a dielectric material is placed in an external electric field, it develops an internal electric field that is oriented opposite the external field, thus reducing the magnitude of the electric field inside the material. ● When a dielectric material is placed in a capacitor with a fixed amount of charge on the surface, the net electric field between the plates is decreased.

Capacitor with a Dielectric If you fill the space between the plates of a capacitor with a dielectric, which is an insulating material such as mineral oil or plastic, what happens to the capacitance? Michael Faraday — to whom the whole concept of capacitance is largely due and for whom the SI unit of capacitance is named — first looked into this matter in 1837. Using simple equipment much like that shown in Fig. 25-12, he found that the capacitance increased by a numerical factor k, which he called

The Royal Institute, England/Bridgeman Art Library/NY

Figure 25-12 The simple electrostatic apparatus used by Faraday. An assembled apparatus (second from left) forms a spherical capacitor consisting of a central brass ball and a concentric brass shell. Faraday placed dielectric materials in the space between the ball and the shell.

732

CHAPTE R 25 CAPACITANCE

the dielectric constant of the insulating material. Table 25-1 shows some dielectric materials and their dielectric constants. The dielectric constant of a vacuum is unity by definition. Because air is mostly empty space, its measured dielectric constant is only slightly greater than unity. Even common paper can significantly increase the capacitance of a capacitor, and some materials, such as strontium titanate, can increase the capacitance by more than two orders of magnitude. Another effect of the introduction of a dielectric is to limit the potential difference that can be applied between the plates to a certain value Vmax, called the breakdown potential. If this value is substantially exceeded, the dielectric material will break down and form a conducting path between the plates. Every dielectric material has a characteristic dielectric strength, which is the maximum value of the electric field that it can tolerate without breakdown. A few such values are listed in Table 25-1. As we discussed just after Eq. 25-18, the capacitance of any capacitor can be written in the form

Table 25-1 Some Properties of Dielectricsa

Material

Dielectric Constant k

Air (1 atm) 1.00054 Polystyrene 2.6 Paper 3.5 Transformer oil 4.5 Pyrex 4.7 Ruby mica 5.4 Porcelain 6.5 Silicon 12 Germanium 16 Ethanol 25 Water (20°C) 80.4 Water (25°C) 78.5 Titania ceramic 130 Strontium titanate 310

Dielectric Strength (kV/mm) 3 24 16 14

(25-26)

C ! ´0",

in which " has the dimension of length. For example, " ! A/d for a parallel-plate capacitor. Faraday’s discovery was that, with a dielectric completely filling the space between the plates, Eq. 25-26 becomes

8

For a vacuum, k ! unity.

C ! k´0" ! kCair,

(25-27)

a

Measured at room temperature, except for the water.

where Cair is the value of the capacitance with only air between the plates. For example, if we fill a capacitor with strontium titanate, with a dielectric constant of 310, we multiply the capacitance by 310. Figure 25-13 provides some insight into Faraday’s experiments. In Fig. 25-13a the battery ensures that the potential difference V between the plates will remain constant. When a dielectric slab is inserted between the plates, the charge q on the plates increases by a factor of k; the additional charge is delivered to the capacitor plates by the battery. In Fig. 25-13b there is no battery, and therefore the charge q must remain constant when the dielectric slab is inserted; then the potential difference V between the plates decreases by a factor of k. Both these observations are consistent (through the relation q ! CV) with the increase in capacitance caused by the dielectric. Comparison of Eqs. 25-26 and 25-27 suggests that the effect of a dielectric can be summed up in more general terms: In a region completely filled by a dielectric material of dielectric constant k, all electrostatic equations containing the permittivity constant +0 are to be modified by replacing ´0 with k´0.

+ + + +

+

– – – –



B

++++++++ κ ––––––––

V = a constant (a)

+

+ + + +



– – – –

B

+ + + + κ – – – –

0

0 VOLTS

VOLTS

q = a constant (b)

Figure 25-13 (a) If the potential difference between the plates of a capacitor is maintained, as by battery B, the effect of a dielectric is to increase the charge on the plates. (b) If the charge on the capacitor plates is maintained, as in this case, the effect of a dielectric is to reduce the potential difference between the plates.The scale shown is that of a potentiometer, a device used to measure potential difference (here, between the plates). A capacitor cannot discharge through a potentiometer.

25-5 CAPACITOR WITH A DI E LECTR IC

733

Thus, the magnitude of the electric field produced by a point charge inside a dielectric is given by this modified form of Eq. 23-15: E!

q 1 . 4pk´0 r2

(25-28)

Also, the expression for the electric field just outside an isolated conductor immersed in a dielectric (see Eq. 23-11) becomes E!

s . k´0

(25-29)

Because k is always greater than unity, both these equations show that for a fixed distribution of charges, the effect of a dielectric is to weaken the electric field that would otherwise be present.

Sample Problem 25.05 Work and energy when a dielectric is inserted into a capacitor A parallel-plate capacitor whose capacitance C is 13.5 pF is charged by a battery to a potential difference V ! 12.5 V between its plates. The charging battery is now disconnected, and a porcelain slab (k ! 6.50) is slipped between the plates. (a) What is the potential energy of the capacitor before the slab is inserted? KEY IDEA

Because the battery has been disconnected, the charge on the capacitor cannot change when the dielectric is inserted. However, the potential does change. Calculations: Thus, we must now use Eq. 25-21 to write the final potential energy Uf , but now that the slab is within the capacitor, the capacitance is kC. We then have Ui 1055 pJ q2 ! ! 2kC k 6.50 ! 162 pJ % 160 pJ.

Uf !

We can relate the potential energy Ui of the capacitor to the capacitance C and either the potential V (with Eq. 25-22) or the charge q (with Eq. 25-21): Ui ! 12CV 2 !

q2 . 2C

Calculation: Because we are given the initial potential V (! 12.5 V), we use Eq. 25-22 to find the initial stored energy: 1 2 2 CV

KEY IDEA

1 2 (13.5 #9

Ui ! ! ! 1.055 $ 10

2

$ 10 F)(12.5 V) J ! 1055 pJ % 1100 pJ. (Answer) #12

(b) What is the potential energy of the capacitor–slab device after the slab is inserted?

(Answer)

When the slab is introduced, the potential energy decreases by a factor of k. The “missing” energy, in principle, would be apparent to the person who introduced the slab. The capacitor would exert a tiny tug on the slab and would do work on it, in amount W ! Ui # Uf ! (1055 # 162) pJ ! 893 pJ. If the slab were allowed to slide between the plates with no restraint and if there were no friction, the slab would oscillate back and forth between the plates with a (constant) mechanical energy of 893 pJ, and this system energy would transfer back and forth between kinetic energy of the moving slab and potential energy stored in the electric field.

Additional examples, video, and practice available at WileyPLUS

Dielectrics: An Atomic View What happens, in atomic and molecular terms, when we put a dielectric in an electric field? There are two possibilities, depending on the type of molecule: 1. Polar dielectrics. The molecules of some dielectrics, like water, have permanent electric dipole moments. In such materials (called polar dielectrics), the

734

CHAPTE R 25 CAPACITANCE





+



– + –

+

+ – +





+

+

+





+

+



+

(b)

Figure 25-14 (a) Molecules with a permanent electric dipole moment, showing their random orientation in the absence of an external electric field. (b) An electric field is applied, producing partial alignment of the dipoles.Thermal agitation prevents complete alignment.

Figure 25-15a shows a nonpolar dielectric slab with no external electric field : applied. In Fig. 25-15b, an electric field E0 is applied via a capacitor, whose plates are charged as shown. The result is a slight separation of the centers of the positive and negative charge distributions within the slab, producing positive charge on one face of the slab (due to the positive ends of dipoles there) and negative charge on the opposite face (due to the negative ends of dipoles there). The slab as a whole remains electrically neutral and — within the slab — there is no excess charge in any volume element. Figure 25-15c shows that the induced surface charges on the faces produce an : : electric field E- in the direction opposite that of the applied electric field E0. The : : : resultant field E inside the dielectric (the vector sum of fields E0 and E-) has the : direction of E0 but is smaller in magnitude. : Both the field E- produced by the surface charges in Fig. 25-15c and the electric field produced by the permanent electric dipoles in Fig. 25-14 act in the : same way — they oppose the applied field E . Thus, the effect of both polar and nonpolar dielectrics is to weaken any applied field within them, as between the plates of a capacitor. The initial electric field inside this nonpolar dielectric slab is zero.

E0 = 0

The applied field aligns the atomic dipole moments.

+ + + + + + + +



+

(a)

–+

–+

–+

–+

–+

–+

–+

–+

–+

–+

–+

–+

(a)

Figure 25-15 (a) A nonpolar dielectric slab. The circles represent the electrically neutral atoms within the slab. (b) An electric field is applied via charged capacitor plates; the field slightly stretches the atoms, separating the centers of positive and negative charge. (c) The separation produces surface charges on the slab faces. These : charges set up a field E-, which opposes the : : applied field E0. The resultant field E in: side the dielectric (the vector sum of E0 : : and E-) has the same direction as E0 but a smaller magnitude.

E0

(b)

The field of the aligned atoms is opposite the applied field.

+ + + + + + + +

+

+ +





+

+



+

+

p









+

electric dipoles tend to line up with an external electric field as in Fig. 25-14. Because the molecules are continuously jostling each other as a result of their random thermal motion, this alignment is not complete, but it becomes more complete as the magnitude of the applied field is increased (or as the temperature, and thus the jostling, are decreased). The alignment of the electric dipoles produces an electric field that is directed opposite the applied field and is smaller in magnitude. 2. Nonpolar dielectrics. Regardless of whether they have permanent electric dipole moments, molecules acquire dipole moments by induction when placed in an external electric field. In Module 24-4 (see Fig. 24-14), we saw that this occurs because the external field tends to “stretch” the molecules, slightly separating the centers of negative and positive charge.

– – –

+ E

E' E0 (c)

+ +

– – – – – – – –

+





+



+

– – – – – – – –



25-6 DI E LECTR ICS AN D GAUSS’ L AW

735

25-6 DIELECTRICS AND GAUSS’ LAW Learning Objectives After reading this module, you should be able to . . .

25.28 In a capacitor with a dielectric, distinguish free charge from induced charge. 25.29 When a dielectric partially or fully fills the space in a

capacitor, find the free charge, the induced charge, the electric field between the plates (if there is a gap, there is more than one field value), and the potential between the plates.

Key Ideas ● Inserting a dielectric into a capacitor causes induced charge to appear on the faces of the dielectric and weakens the electric field between the plates.

generalized to

● The induced charge is less than the free charge on the plates.

where q is the free charge. Any induced surface charge is accounted for by including the dielectric constant k inside the integral.

´0

● When a dielectric is present, Gauss’ law may be

Dielectrics and Gauss’ Law In our discussion of Gauss’ law in Chapter 23, we assumed that the charges existed in a vacuum. Here we shall see how to modify and generalize that law if dielectric materials, such as those listed in Table 25-1, are present. Figure 25-16 shows a parallel-plate capacitor of plate area A, both with and without a dielectric. We assume that the charge q on the plates is the same in both situations. Note that the field between the plates induces charges on the faces of the dielectric by one of the methods described in Module 25-5. For the situation of Fig. 25-16a, without a dielectric, we can find the electric : field E0 between the plates as we did in Fig. 25-5: We enclose the charge %q on the top plate with a Gaussian surface and then apply Gauss’ law. Letting E0 represent the magnitude of the field, we find ´0

,

:

:

E " dA ! ´0EA ! q,

(25-30)

q . ´0A

(25-31)

or

E0 !

In Fig. 25-16b, with the dielectric in place, we can find the electric field between the plates (and within the dielectric) by using the same Gaussian surface. However, now the surface encloses two types of charge: It still encloses charge %q on the top plate, but it now also encloses the induced charge #q- on the top face of the dielectric. The charge on the conducting plate is said to be free charge because it can move if we change the electric potential of the plate; the induced charge on the surface of the dielectric is not free charge because it cannot move from that surface. +q

Gaussian surface

Gaussian surface

+ + + + + + + + + +

+ + + + + + + + + + – – – – –

E0

+q –q

– – – – – – – – – –

E

κ

+ + + + + – – – – – – – – – –

–q' +q'

–q (a)

(b)

Figure 25-16 A parallel-plate capacitor (a) without and (b) with a dielectric slab inserted. The charge q on the plates is assumed to be the same in both cases.

,

:

:

kE " dA ! q,

736

CHAPTE R 25 CAPACITANCE

The net charge enclosed by the Gaussian surface in Fig. 25-16b is q # q-, so Gauss’ law now gives ´0

,

:

:

E " dA ! ´0EA ! q # q-,

(25-32)

q # q. ´0A

(25-33)

or

E!

The effect of the dielectric is to weaken the original field E0 by a factor of k; so we may write E0 q ! . k k´0 A

E!

(25-34)

Comparison of Eqs. 25-33 and 25-34 shows that q # q- !

q . k

(25-35)

Equation 25-35 shows correctly that the magnitude q- of the induced surface charge is less than that of the free charge q and is zero if no dielectric is present (because then k ! 1 in Eq. 25-35). By substituting for q # q- from Eq. 25-35 in Eq. 25-32, we can write Gauss’ law in the form ´0

,

:

:

kE " dA ! q

(25-36)

(Gauss’ law with dielectric).

This equation, although derived for a parallel-plate capacitor, is true generally and is the most general form in which Gauss’ law can be written. Note: :

:

:

1. The flux integral now involves kE, not just E. (The vector ´0kE is sometimes : called the electric displacement D, so that Eq. 25-36 can be written in the form : : - D " dA ! q.) 2. The charge q enclosed by the Gaussian surface is now taken to be the free charge only. The induced surface charge is deliberately ignored on the right side of Eq. 25-36, having been taken fully into account by introducing the dielectric constant k on the left side. 3. Equation 25-36 differs from Eq. 23-7, our original statement of Gauss’ law, only in that +0 in the latter equation has been replaced by k+0. We keep k inside the integral of Eq. 25-36 to allow for cases in which k is not constant over the entire Gaussian surface.

Sample Problem 25.06 Dielectric partially filling the gap in a capacitor Figure 25-17 shows a parallel-plate capacitor of plate area A and plate separation d. A potential difference V0 is applied between the plates by connecting a battery between them. The battery is then disconnected, and a dielectric slab of thickness b and dielectric constant k is placed between the plates as shown. Assume A ! 115 cm2, d ! 1.24 cm, V0 ! 85.5 V, b ! 0.780 cm, and k ! 2.61. (a) What is the capacitance C0 before the dielectric slab is inserted?

+q Gaussian surface I

+ + + + + + + +

–q'









+q'

+

+ κ +

+

Gaussian surface II

b

d

– – – – – – – – –q

Figure 25-17 A parallel-plate capacitor containing a dielectric slab that only partially fills the space between the plates.

737

25-6 DI E LECTR ICS AN D GAUSS’ L AW

Calculation: From Eq. 25-9 we have

KEY IDEA

+0 A (8.85 $ 10#12 F/m)(115 $ 10#4 m2) ! d 1.24 $ 10#2 m ! 8.21 $ 10#12 F ! 8.21 pF. (Answer)

C0 !

Now we apply Gauss’ law in the form of Eq. 25-36 to Gaussian surface II in Fig. 25-17. Calculations: Only the free charge #q is in Eq. 25-36, so

(b) What free charge appears on the plates? ´0

Calculation: From Eq. 25-1, q ! C0V0 ! (8.21 $ 10#12 F)(85.5 V) ! 7.02 $ 10#10 C ! 702 pC. (Answer) Because the battery was disconnected before the slab was inserted, the free charge is unchanged. (c) What is the electric field E0 in the gaps between the plates and the dielectric slab? KEY IDEA

Calculations: That surface passes through the gap, and so it encloses only the free charge on the upper capacitor plate. Electric field pierces only the bottom of the Gaussian surface. : : Because there the area vector dA and the field vector E0 are both directed downward, the dot product in Eq. 25-36 becomes :

:

E0 " dA ! E0 dA cos 0/ ! E0 dA.

,

dA ! q.

The integration now simply gives the surface area A of the plate. Thus, we obtain

(25-37)

E0 6.90 kV/m q ! ! ´0kA k 2.61

! 2.64 kV/m.

(Answer)

(e) What is the potential difference V between the plates after the slab has been introduced? KEY IDEA We find V by integrating along a straight line directly from the bottom plate to the top plate.

V!

"

%

#

E ds ! E0(d # b) % E1b

! (6900 V/m)(0.0124 m # 0.00780 m)

´0kE0 A ! q,

% (2640 V/m)(0.00780 m)

q . E0 ! ´0kA

(Answer) ! 52.3 V. This is less than the original potential difference of 85.5 V.

We must put k ! 1 here because Gaussian surface I does not pass through the dielectric. Thus, we have 7.02 $ 10#10 C q ! E0 ! #12 ´0kA (8.85 $ 10 F/m)(1)(115 $ 10#4 m2) ! 6900 V/m ! 6.90 kV/m.

:

Calculation: Within the dielectric, the path length is b and the electric field is E1. Within the two gaps above and below the dielectric, the total path length is d # b and the electric field is E0. Equation 25-6 then yields

Equation 25-36 then becomes ´0kE0

:

kE1 " dA ! #´0kE1A ! #q.

The first minus sign in this equation comes from the dot : : product E1 " dA along the top of the Gaussian surface be: cause now the field vector E1 is directed downward and the : area vector dA (which, as always, points outward from the interior of a closed Gaussian surface) is directed upward. With 180/ between the vectors, the dot product is negative. Now k ! 2.61. Thus, Eq. 25-37 gives us E1 !

We need to apply Gauss’ law, in the form of Eq. 25-36, to Gaussian surface I in Fig. 25-17.

or

,

(Answer)

Note that the value of E0 does not change when the slab is introduced because the amount of charge enclosed by Gaussian surface I in Fig. 25-17 does not change. (d) What is the electric field E1 in the dielectric slab?

(f) What is the capacitance with the slab in place? KEY IDEA The capacitance C is related to q and V via Eq. 25-1. Calculation: Taking q from (b) and V from (e), we have C!

7.02 $ 10#10 C q ! V 52.3 V

! 1.34 $ 10#11 F ! 13.4 pF.

(Answer)

This is greater than the original capacitance of 8.21 pF.

Additional examples, video, and practice available at WileyPLUS

738

CHAPTE R 25 CAPACITANCE

Review & Summary Capacitor; Capacitance A capacitor consists of two isolated

Equivalent capacitances can be used to calculate the capacitances of more complicated series – parallel combinations.

(25-1)

Potential Energy and Energy Density The electric poten-

conductors (the plates) with charges %q and #q. Its capacitance C is defined from q ! CV, where V is the potential difference between the plates.

Determining

L (25-14) . ln(b/a) A spherical capacitor with concentric spherical plates of radii a and b has capacitance ab (25-17) C ! 4p´0 . b#a An isolated sphere of radius R has capacitance C ! 2p´0

C ! 4p´0R.

(25-18)

Capacitors in Parallel and in Series The equivalent

capacitances Ceq of combinations of individual capacitors connected in parallel and in series can be found from

and

U!

Capacitance We generally determine the

capacitance of a particular capacitor configuration by (1) assuming a charge q to have been placed on the plates, (2) finding the electric field : E due to this charge, (3) evaluating the potential difference V, and (4) calculating C from Eq. 25-1. Some specific results are the following: A parallel-plate capacitor with flat parallel plates of area A and spacing d has capacitance ´A (25-9) C! 0 . d A cylindrical capacitor (two long coaxial cylinders) of length L and radii a and b has capacitance

Ceq !

tial energy U of a charged capacitor,

n

' Cj

(n capacitors in parallel)

(25-19)

(n capacitors in series).

(25-20)

j!1 n

1 1 ! ' Ceq j!1 Cj

q2 ! 12 CV 2, 2C

(25-21, 25-22)

is equal to the work required to charge the capacitor. This energy : can be associated with the capacitor’s electric field E. By extension we can associate stored energy with any electric field. In vacuum, the energy density u, or potential energy per unit volume, within an electric field of magnitude E is given by u ! 12´0 E 2.

(25-25)

Capacitance with a Dielectric If the space between the plates of a capacitor is completely filled with a dielectric material, the capacitance C is increased by a factor k, called the dielectric constant, which is characteristic of the material. In a region that is completely filled by a dielectric, all electrostatic equations containing +0 must be modified by replacing +0 with k+0. The effects of adding a dielectric can be understood physically in terms of the action of an electric field on the permanent or induced electric dipoles in the dielectric slab. The result is the formation of induced charges on the surfaces of the dielectric, which results in a weakening of the field within the dielectric for a given amount of free charge on the plates.

Gauss’ Law with a Dielectric When a dielectric is present, Gauss’ law may be generalized to ´0

,

:

:

(25-36)

kE " dA ! q.

Here q is the free charge; any induced surface charge is accounted for by including the dielectric constant k inside the integral.

Questions 1 Figure 25-18 shows plots of charge versus potential difference for three parallel-plate capacitors that have the plate areas and separations given in the table. Which plot goes with which capacitor?

q

3 (a) In Fig. 25-19a, are capacitors 1 and 3 in series? (b) In the same a C3

b c V

+ –

Figure 25-18 Question 1.

Capacitor

Area

Separation

1 2 3

A 2A A

d d 2d

2 What is Ceq of three capacitors, each of capacitance C, if they are connected to a battery (a) in series with one another and (b) in parallel? (c) In which arrangement is there more charge on the equivalent capacitance?

C1

C2

+ –

C3

(a)

C2

C1

(b) C1

C1

(c)

C3 + –

+ –

C2

C3 (d)

C2

Figure 25-19 Question 3.

PROB LE M S

8 Figure 25-22 shows an open C C C switch, a battery of potential difference V, a current-measuring meter A A, and three identical uncharged + – capacitors of capacitance C. When V the switch is closed and the circuit Figure 25-22 Question 8. reaches equilibrium, what are (a) the potential difference across each capacitor and (b) the charge on the left plate of each capacitor? (c) During charging, what net charge passes through the meter?

figure, are capacitors 1 and 2 in parallel? (c) Rank the equivalent capacitances of the four circuits shown in Fig. 25-19, greatest first. 4 Figure 25-20 shows three circuits, each consisting of a switch and two capacitors, initially charged as indicated (top plate positive). After the switches have been closed, in which circuit (if any) will the charge on the left-hand capacitor (a) increase, (b) decrease, and (c) remain the same?

6q

3q

6q

3q

6q

3q

2C

C

3C

C

2C

2C

(1)

(2)

9 A parallel-plate capacitor is connected to a battery of electric potential difference V. If the plate separation is decreased, do the following quantities increase, decrease, or remain the same: (a) the capacitor’s capacitance, (b) the potential difference across the capacitor, (c) the charge on the capacitor, (d) the energy stored by the capacitor, (e) the magnitude of the electric field between the plates, and (f ) the energy density of that electric field?

(3)

Figure 25-20 Question 4. 5 Initially, a single capacitance C1 is wired to a battery. Then capacitance C2 is added in parallel. Are (a) the potential difference across C1 and (b) the charge q1 on C1 now more than, less than, or the same as previously? (c) Is the equivalent capacitance C12 of C1 and C2 more than, less than, or equal to C1? (d) Is the charge stored on C1 and C2 together more than, less than, or equal to the charge stored previously on C1?

10 When a dielectric slab is inserted between the plates of one of the two identical capacitors in Fig. 25-23, do the following properties of that capacitor increase, decrease, or remain the same: (a) capacitance, (b) charge, (c) potential difference, and (d) potential energy? (e) How about the same properties of the other capacitor?

6 Repeat Question 5 for C2 added in series rather than in parallel. 7 For each circuit in Fig. 25-21, are the capacitors connected in series, in parallel, or in neither mode? – + + –

– + (a)

(b)

739

C

κ

+ –

B

C

Figure 25-23 Question 10.

11 You are to connect capacitances C1 and C2, with C1 ( C2, to a battery, first individually, then in series, and then in parallel. Rank those arrangements according to the amount of charge stored, greatest first.

(c)

Figure 25-21 Question 7.

Problems Tutoring problem available (at instructor’s discretion) in WileyPLUS and WebAssign SSM • – •••

Worked-out solution available in Student Solutions Manual

WWW Worked-out solution is at

Number of dots indicates level of problem difficulty

ILW

Interactive solution is at

http://www.wiley.com/college/halliday

Additional information available in The Flying Circus of Physics and at flyingcircusofphysics.com

Module 25-1 Capacitance •1 The two metal objects in Fig. 25-24 have net charges of %70 pC and #70 pC, which result in a 20 V potential difference between them. (a) What is the capacitance of the system? (b) If the charges are changed to %200 pC and #200 pC, what does the capacitance become? (c) What does the potential difference become?

Module 25-2 Calculating the Capacitance •3 SSM A parallel-plate capacitor has circular plates of 8.20 cm radius and 1.30 mm separation. (a) Calculate the capacitance. (b) Find the charge for a potential difference of 120 V. •4 The plates of a spherical capacitor have radii 38.0 mm and 40.0 mm. (a) Calculate the capacitance. (b) What must be the plate area of a parallel-plate capacitor with the same plate separation and capacitance? •5 What is the capacitance of a drop that results when two mercury spheres, each of radius R ! 2.00 mm, merge?

Figure 25-24 Problem 1. S

•2 The capacitor in Fig. 25-25 has a capacitance of 25 mF and is initially + C uncharged. The battery provides a – potential difference of 120 V. After switch S is closed, how much charge will pass through it? Figure 25-25 Problem 2.

•6 You have two flat metal plates, each of area 1.00 m2, with which to construct a parallel-plate capacitor. (a) If the capacitance of the device is to be 1.00 F, what must be the separation between the plates? (b) Could this capacitor actually be constructed? •7 If an uncharged parallel-plate capacitor (capacitance C) is connected to a battery, one plate becomes negatively charged as

740

CHAPTE R 25 CAPACITANCE

electrons move to the plate face (area d (pm) A). In Fig. 25-26, the depth d from which the electrons come in the plate in a par- ds ticular capacitor is plotted against a range of values for the potential difference V of the battery. The density of conduction electrons in the copper plates is 8.49 $ 10 28 electrons/m3. The vertical Vs 0 scale is set by ds ! 1.00 pm, and the horiV (V) zontal scale is set by Vs ! 20.0 V. What is Figure 25-26 Problem 7. the ratio C/A? Module 25-3 Capacitors in Parallel and in Series •8 How many 1.00 mF capacitors must be connected in parallel to store a charge of 1.00 C with a potential of 110 V across the capacitors? •9 Each of the uncharged capacitors in Fig. 25-27 has a capacitance of 25.0 mF. A potential difference of V ! 4200 V is established when the switch is closed. How many coulombs of charge then pass through meter A?

capacitor drops to 35 V, what is the capacitance of this second capacitor? ••14 In Fig. 25-30, the battery has a potential difference of V ! 10.0 V and the five capacitors each have a capacitance of 10.0 mF. What is the charge on (a) capacitor 1 and (b) capacitor 2?

C2

C1 + –

V

••15 In Fig. 25-31, a 20.0 V batFigure 25-30 Problem 14. tery is connected across capacitors of capacitances C1 ! C6 ! 3.00 mF and C3 ! C5 ! 2.00C2 ! 2.00C4 ! 4.00 mF. What are (a) the equivalent capacitance Ceq of the capacitors and (b) the charge stored by Ceq? What are (c) V1 and (d) q1 of capacitor 1, (e) V2 and (f) q2 of capacitor 2, and (g) V3 and (h) q3 of capacitor 3?

A C

V

C

C

C5

C4

C3

+ V –

Figure 25-27 Problem 9. •10 In Fig. 25-28, find the equivalent capacitance of the combination. Assume that C1 is 10.0 mF, C2 is 5.00 mF, and C3 is 4.00 mF.

C6

C2

C1

Figure 25-31 Problem 15.

C3

V C2

Figure 25-28 Problems 10 and 34. •11 ILW In Fig. 25-29, find the equivalent capacitance of the combination. Assume that C1 ! 10.0 mF, C2 ! 5.00 mF, and C3 ! 4.00 mF. C2 C1

V C3

Figure 25-29 Problems 11, 17, and 38. ••12 Two parallel-plate capacitors, 6.0 mF each, are connected in parallel to a 10 V battery. One of the capacitors is then squeezed so that its plate separation is 50.0% of its initial value. Because of the squeezing, (a) how much additional charge is transferred to the capacitors by the battery and (b) what is the increase in the total charge stored on the capacitors? ••13 SSM ILW A 100 pF capacitor is charged to a potential difference of 50 V, and the charging battery is disconnected. The capacitor is then connected in parallel with a second (initially uncharged) capacitor. If the potential difference across the first

••16 Plot 1 in Fig. 25-32a gives the charge q that can be stored on capacitor 1 versus the electric potential V set up across it. The vertical scale is set by qs ! 16.0 mC, and the horizontal scale is set by Vs ! 2.0 V. Plots 2 and 3 are similar plots for capacitors 2 and 3, respectively. Figure 25-32b shows a circuit with those three capacitors and a 6.0 V battery. What is the charge stored on capacitor 2 in that circuit? qs

1 2

q (µ C)

C1

3 0 V (V)

V

C1

C2

C3

Vs

(a)

(b)

Figure 25-32 Problem 16.

••17 In Fig. 25-29, a potential difference of V ! 100.0 V is applied across a capacitor arrangement with capacitances C1 ! 10.0 mF, C2 ! 5.00 mF, and C3 ! 4.00 mF. If capacitor 3 undergoes electrical breakdown so that it becomes equivalent to conducting wire, what is the increase in (a) the charge on capacitor 1 and (b) the potential difference across capacitor 1? ••18 Figure 25-33 shows a circuit section of four air-filled capacitors that is connected to a larger circuit.The graph below the section shows the electric potential V(x) as a function of position x along the lower part of the section, through capacitor 4. Similarly, the graph above the section shows the electric potential V(x) as a function of position x along the upper part of the section, through capacitors 1, 2, and 3.

741

PROB LE M S

V

2V

5V

x

1

2

3

V (V)

4

Figure 25-33 Problem 18.

12 x

••19 In Fig. 25-34, the battery has potential difference V ! 9.0 V, C2 ! 3.0 mF, C4 ! 4.0 mF, and all the capacitors are initially uncharged. When switch S is closed, a total charge of 12 mC passes through point a and a total charge of 8.0 mC passes through point b. What are (a) C1 and (b) C3? S V

C1

C2

a

b C3

C4

Figure 25-34 Problem 19. ••20 Figure 25-35 shows a variable “air gap” capacitor for manual tuning. Alternate plates A are connected together; one group of plates is fixed in posid tion, and the other group is A capable of rotation. Consider a capacitor of n ! 8 plates of alFigure 25-35 Problem 20. ternating polarity, each plate 2 having area A ! 1.25 cm and separated from adjacent plates by distance d ! 3.40 mm. What is the maximum capacitance of the device?

••23 The capacitors in Fig. 25-38 are iniS a C2 tially uncharged. The capacitances are C1 c V C1 ! 4.0 mF, C2 ! 8.0 mF, and C3 ! 12 mF, C3 b and the battery’s potential difference is d V ! 12 V. When switch S is closed, how many electrons travel through (a) point a, Figure 25-38 Problem 23. (b) point b, (c) point c, and (d) point d? In the figure, do the electrons travel up or down through (e) point b and (f) point c? ••24 Figure 25-39 represents two air-filled cylindrical capacitors connected in series across a battery with potential V ! 10 V. Capacitor 1 has an inner plate radius of 5.0 mm, an outer plate radius of 1.5 cm, and a length of 5.0 cm. Capacitor 2 has an inner plate radius of 2.5 mm, an outer plate radius of 1.0 cm, and a length of 9.0 cm.The outer plate of capacitor 2 is a conducting P organic membrane that can be stretched, C1 and the capacitor can be inflated to inV crease the plate separation. If the outer C2 plate radius is increased to 2.5 cm by inflation, (a) how many electrons move Figure 25-39 Problem 24. through point P and (b) do they move toward or away from the battery? ••25 In Fig. 25-40, two parallel-plate C1 C2 capacitors (with air between the plates) are connected to a battery. Capacitor 1 has a plate area of 1.5 cm2 and an electric Figure 25-40 field (between its plates) of magnitude Problem 25. 2000 V/m. Capacitor 2 has a plate area of 0.70 cm2 and an electric field of magnitude 1500 V/m. What is the total charge on the two capacitors? •••26 Capacitor 3 in Fig. 25-41a is a variable capacitor (its capacitance C3 can be varied). Figure 25-41b gives the electric potential V1 across capacitor 1 versus C3. The horizontal scale is set by C3s = 12.0 mF. Electric potential V1 approaches an asymptote of 10 V as C3 : ,. What are (a) the electric potential V across the battery, (b) C1, and (c) C2? 10 8

C1 V1 (V)

Capacitor 3 has a capacitance of 0.80 mF. What are the capacitances of (a) capacitor 1 and (b) capacitor 2?

V C2

C3

a

S1 ••21 SSM WWW In Fig. 25-36, the capacitances are C1 ! 1.0 mF and + + + + –– –– C C2 C2 ! 3.0 mF, and both capacitors are – – – – 1 ++ ++ S2 charged to a potential difference of b V ! 100 V but with opposite polarFigure 25-36 Problem 21. ity as shown. Switches S1 and S2 are now closed. (a) What is now the potential difference between points a and b? What now is the charge on capacitor (b) 1 and (c) 2?

••22 In Fig. 25-37, V ! 10 V, C1 ! 10 S mF, and C2 ! C3 ! 20 mF. Switch S is first thrown to the left side until capacC1 C2 C3 itor 1 reaches equilibrium. Then the V switch is thrown to the right. When equilibrium is again reached, how Figure 25-37 Problem 22. much charge is on capacitor 1?

6 4 2 0

C 3s

C 3 (µ F)

(a)

(b)

Figure 25-41 Problem 26. •••27 Figure 25-42 shows a 12.0 V battery and four uncharged capacitors of capacitances C1 ! 1.00 mF, C2 ! 2.00 mF, C3 ! 3.00 mF, and C4 ! 4.00 mF. If only switch S1 is closed, what is the charge on (a) capacitor 1, (b) capacitor 2, (c) capacitor 3, and (d) capacitor 4? If both switches are closed, what is the charge on (e) capacitor 1, (f) capacitor 2, (g) capacitor 3, and (h) capacitor 4?

C1

C3

S2 C2 + – B

C4 S1

Figure 25-42 Problem 27.

742

CHAPTE R 25 CAPACITANCE

•••28 Figure 25-43 displays a 12.0 V battery and 3 uncharged capaciS C2 tors of capacitances C1 ! 4.00 mF, + V 0 C2 ! 6.00 mF, and C3 ! 3.00 mF. The – C1 C3 switch is thrown to the left side until capacitor 1 is fully charged. Then the switch is thrown to the right. What is Figure 25-43 Problem 28. the final charge on (a) capacitor 1, (b) capacitor 2, and (c) capacitor 3? Module 25-4 Energy Stored in an Electric Field •29 What capacitance is required to store an energy of 10 kW &h at a potential difference of 1000 V? •30 How much energy is stored in 1.00 m3 of air due to the “fair weather” electric field of magnitude 150 V/m? •31 SSM A 2.0 mF capacitor and a 4.0 mF capacitor are connected in parallel across a 300 V potential difference. Calculate the total energy stored in the capacitors. •32 A parallel-plate air-filled capacitor having area 40 cm2 and plate spacing 1.0 mm is charged to a potential difference of 600 V. Find (a) the capacitance, (b) the magnitude of the charge on each plate, (c) the stored energy, (d) the electric field between the plates, and (e) the energy density between the plates. ••33 A charged isolated metal sphere of diameter 10 cm has a potential of 8000 V relative to V ! 0 at infinity. Calculate the energy density in the electric field near the surface of the sphere. ••34 In Fig. 25-28, a potential difference V ! 100 V is applied across a capacitor arrangement with capacitances C1 ! 10.0 mF, C2 ! 5.00 mF, and C3 ! 4.00 mF. What are (a) charge q3, (b) potential difference V3, and (c) stored energy U3 for capacitor 3, (d) q1, (e) V1, and (f) U1 for capacitor 1, and (g) q2, (h) V2, and (i) U2 for capacitor 2? ••35 Assume that a stationary electron is a point of charge. What is the energy density u of its electric field at radial distances (a) r ! 1.00 mm, (b) r ! 1.00 mm, (c) r ! 1.00 nm, and (d) r ! 1.00 pm? (e) What is u in the limit as r : 0? Venting port

••36 As a safety engineer, you must evaluate the practice of + – storing flammable conducting liq- –– ++ –– –– –– –– + – h – – – uids in nonconducting containers. – + + + + + + + + + – – – – – – – – The company supplying a certain r liquid has been using a squat, cylindrical plastic container of radius Figure 25-44 Problem 36. r ! 0.20 m and filling it to height h ! 10 cm, which is not the container’s full interior height (Fig. 25-44). Your investigation reveals that during handling at the company, the exterior surface of the container commonly acquires a negative charge density of magnitude 2.0 mC/m2 (approximately uniform). Because the liquid is a conducting material, the charge on the container induces charge separation within the liquid. (a) How much negative charge is induced in the center of the liquid’s bulk? (b) Assume the capacitance of the central portion of the liquid relative to ground is 35 pF. What is the potential energy associated with the negative charge in that effective capacitor? (c) If a spark occurs between the ground and the central portion of the liquid (through the venting port), the potential energy can be fed into the spark. The minimum spark energy needed to ignite the liquid is 10 mJ. In this situation, can a spark ignite the liquid?

••37 SSM ILW WWW The parallel plates in a capacitor, with a plate area of 8.50 cm2 and an air-filled separation of 3.00 mm, are charged by a 6.00 V battery. They are then disconnected from the battery and pulled apart (without discharge) to a separation of 8.00 mm. Neglecting fringing, find (a) the potential difference between the plates, (b) the initial stored energy, (c) the final stored energy, and (d) the work required to separate the plates. ••38 In Fig. 25-29, a potential difference V ! 100 V is applied across a capacitor arrangement with capacitances C1 ! 10.0 mF, C2 ! 5.00 mF, and C3 ! 15.0 mF. What are (a) charge q3, (b) potential difference V3, and (c) stored energy U3 for capacitor 3, (d) q1, (e) V1, and (f) U1 for capacitor 1, and (g) q2, (h) V2, and (i) U2 for capacitor 2? A

B

••39 In Fig. 25-45, C1 ! 10.0 C1 C2 C3 mF, C2 ! 20.0 mF, and C3 ! Figure 25-45 Problem 39. 25.0 mF. If no capacitor can withstand a potential difference of more than 100 V without failure, what are (a) the magnitude of the maximum potential difference that can exist between points A and B and (b) the maximum energy that can be stored in the three-capacitor arrangement? Module 25-5 Capacitor with a Dielectric •40 An air-filled parallel-plate capacitor has a capacitance of 1.3 pF. The separation of the plates is doubled, and wax is inserted between them. The new capacitance is 2.6 pF. Find the dielectric constant of the wax. •41 SSM A coaxial cable used in a transmission line has an inner radius of 0.10 mm and an outer radius of 0.60 mm. Calculate the capacitance per meter for the cable. Assume that the space between the conductors is filled with polystyrene. •42 A parallel-plate air-filled capacitor has a capacitance of 50 pF. (a) If each of its plates has an area of 0.35 m2, what is the separation? (b) If the region between the plates is now filled with material having k ! 5.6, what is the capacitance? •43 Given a 7.4 pF air-filled capacitor, you are asked to convert it to a capacitor that can store up to 7.4 mJ with a maximum potential difference of 652 V. Which dielectric in Table 25-1 should you use to fill the gap in the capacitor if you do not allow for a margin of error? ••44 You are asked to construct a capacitor having a capacitance near 1 nF and a breakdown potential in excess of 10 000 V. You think of using the sides of a tall Pyrex drinking glass as a dielectric, lining the inside and outside curved surfaces with aluminum foil to act as the plates. The glass is 15 cm tall with an inner radius of 3.6 cm and an outer radius of 3.8 cm. What are the (a) capacitance and (b) breakdown potential of this capacitor? ••45 A certain parallel-plate capacitor is filled with a dielectric for which k ! 5.5. The area of each plate is 0.034 m2, and the plates are separated by 2.0 mm. The capacitor will fail (short out and burn up) if the electric field between the plates exceeds 200 kN/C. What is the maximum energy that can be stored in the capacitor? ••46 In Fig. 25-46, how much charge is stored on the parallel-plate capacitors by the 12.0 V battery? One is filled C1 C2 with air, and the other is filled with a di- V electric for which k ! 3.00; both capacitors have a plate area of 5.00 $ 10#3 m2 and a plate separation of 2.00 mm. Figure 25-46 Problem 46.

743

PROB LE M S

••47 SSM ILW A certain substance has a dielectric constant of 2.8 and a dielectric strength of 18 MV/m. If it is used as the dielectric material in a parallel-plate capacitor, what minimum area should the plates of the capacitor have to obtain a capacitance of 7.0 $ 10#2 mF and to ensure that the capacitor will be able to withstand a potential difference of 4.0 kV? ••48 Figure 25-47 shows a parallelplate capacitor with a plate area A ! 5.56 cm2 and separation d ! 5.56 mm. The left half of the gap is filled with material of dielectric constant k1 ! 7.00; the right half is filled with material of dielectric constant k2 ! 12.0. What is the capacitance?

A/2

κ1

material. (b) Determine the magnitude of the charge induced on each dielectric surface.

A/2

κ2

d

Additional Problems 56 In Fig. 25-50, the battery potential difference V is 10.0 V and each of the seven capacitors has capacitance 10.0 mF. What is the charge on (a) capacitor 1 and (b) capacitor 2?

Figure 25-47 Problem 48.

••49 Figure 25-48 shows a parallel-plate capacitor with a plate area A ! 7.89 cm2 and plate separation d ! 4.62 mm. The top half of the gap is filled with material of dielectric constant k1 ! 11.0; the bottom half is filled with material of dielectric constant k2 ! 12.0. What is the capacitance?

κ2 κ1

d

Figure 25-48 Problem 49.

••50 Figure 25-49 shows a parallelplate capacitor of plate area A ! 10.5 A/2 A/2 cm2 and plate separation 2d ! 7.12 mm. The left half of the gap is filled with maκ2 d κ1 terial of dielectric constant k1 ! 21.0; 2d κ3 d the top of the right half is filled with material of dielectric constant k2 ! 42.0; the bottom of the right half is filled with material of dielectric constant k3 ! Figure 25-49 Problem 50. 58.0.What is the capacitance? Module 25-6 Dielectrics and Gauss’ Law •51 SSM WWW A parallel-plate capacitor has a capacitance of 100 pF, a plate area of 100 cm2, and a mica dielectric (k ! 5.4) completely filling the space between the plates. At 50 V potential difference, calculate (a) the electric field magnitude E in the mica, (b) the magnitude of the free charge on the plates, and (c) the magnitude of the induced surface charge on the mica. •52 For the arrangement of Fig. 25-17, suppose that the battery remains connected while the dielectric slab is being introduced. Calculate (a) the capacitance, (b) the charge on the capacitor plates, (c) the electric field in the gap, and (d) the electric field in the slab, after the slab is in place. ••53 A parallel-plate capacitor has plates of area 0.12 m2 and a separation of 1.2 cm. A battery charges the plates to a potential difference of 120 V and is then disconnected. A dielectric slab of thickness 4.0 mm and dielectric constant 4.8 is then placed symmetrically between the plates. (a) What is the capacitance before the slab is inserted? (b) What is the capacitance with the slab in place? What is the free charge q (c) before and (d) after the slab is inserted? What is the magnitude of the electric field (e) in the space between the plates and dielectric and (f) in the dielectric itself? (g) With the slab in place, what is the potential difference across the plates? (h) How much external work is involved in inserting the slab? ••54 Two parallel plates of area 100 cm2 are given charges of equal magnitudes 8.9 $ 10#7 C but opposite signs. The electric field within the dielectric material filling the space between the plates is 1.4 $ 106 V/m. (a) Calculate the dielectric constant of the

••55 The space between two concentric conducting spherical shells of radii b ! 1.70 cm and a ! 1.20 cm is filled with a substance of dielectric constant k ! 23.5. A potential difference V ! 73.0 V is applied across the inner and outer shells. Determine (a) the caC1 pacitance of the device, (b) the free charge q on – + the inner shell, and (c) the charge q- induced V along the surface of the inner shell. C2

Figure 25-50 Problem 56.

57 SSM In Fig. 25-51, V ! 9.0 V, C1 ! C2 ! 30 mF, and C3 ! C4 ! 15 mF. What is the charge on capacitor 4? C1 V

C2

C4 C3

Figure 25-51 Problem 57. C

58 (a) If C ! 50 mF in Fig. 25-52, what A is the equivalent capacitance between points A and B? (Hint: First imagine 2C that a battery is connected between those two points.) (b) Repeat for points B A and D. 59 In Fig. 25-53, V ! 12 V, C1 ! C4 ! 2.0 mF, C2 ! 4.0 mF, and C3 ! 1.0 mF. What is the charge on capacitor 4?

4C D 6C

Figure 25-52 Problem 58.

60 The chocolate crumb mystery. C1 C2 This story begins with Problem 60 in Chapter 23. As part of the investigation V of the biscuit factory explosion, the electric potentials of the workers were C3 C4 measured as they emptied sacks of chocolate crumb powder into the loading bin, stirring up a cloud of the powder Figure 25-53 Problem 59. around themselves. Each worker had an electric potential of about 7.0 kV relative to the ground, which was taken as zero potential. (a) Assuming that each worker was effectively a capacitor with a typical capacitance of 200 pF, find the energy stored in that effective capacitor. If a single spark between the worker and any conducting object connected to the ground neutralized the worker, that energy would be transferred to the spark. According to measurements, a spark that could ignite a cloud of chocolate crumb powder, and thus set off an explosion, had to have an energy of at least 150 mJ. (b) Could a spark from a worker have set off an explosion in the cloud of powder in the loading bin? (The story continues with Problem 60 in P S C1 Chapter 26.) C2 C4 V 61 Figure 25-54 shows capacitor 1 (C1 ! 8.00 mF), capacitor 2 (C2 ! 6.00 mF), and capacitor 3 (C3 !

C3

Figure 25-54 Problem 61.

744

CHAPTE R 25 CAPACITANCE

8.00 mF) connected to a 12.0 V battery. When switch S is closed so as to connect uncharged capacitor 4 (C4 ! 6.00 mF), (a) how much charge passes through point P from the battery and (b) how much charge shows up on capacitor 4? (c) Explain the discrepancy in those two results. 62 Two air-filled, parallel-plate capacitors are to be connected to a 10 V battery, first individually, then in series, and then in parallel. In those arrangements, the energy stored in the capacitors turns out to be, listed least to greatest: 75 mJ, 100 mJ, 300 mJ, and 400 mJ. Of the two capacitors, what is the (a) smaller and (b) greater capacitance? 63 Two parallel-plate capacitors, 6.0 mF each, are connected in series to a 10 V battery. One of the capacitors is then squeezed so that its plate separation is halved. Because of the squeezing, (a) how much additional charge is transferred to the capacitors by the battery and (b) what is the increase in the total charge stored on the capacitors (the charge on the positive plate of one capacitor plus the charge on the positive plate of the other capacitor)? 64 In Fig. 25-55, V ! 12 V, C1! C5 ! C6 ! 6.0 mF, and C2 ! C3 ! C4! 4.0 mF. What are (a) the net charge stored on the capacitors and (b) the V charge on capacitor 4? 65 SSM In Fig. 25-56, the parallel-plate capacitor of plate area 2.00 $ 10#2 m2 is filled with two dielectric slabs, each with thickness 2.00 mm. One slab has dielectric constant 3.00, and the other, 4.00. How much charge does the 7.00 V battery store on the capacitor? 66 A cylindrical capacitor has radii a and b as in Fig. 25-6. Show that half the stored electric potential energy lies within a cylinder whose radius is r ! 1ab.

C1 C2

C5

C3

C4

C6

Figure 25-55 Problem 64.

V

Figure 25-56 Problem 65.

67 A capacitor of capacitance C1 ! 6.00 mF is connected in series with a capacitor of capacitance C2 ! 4.00 mF, and a potential difference of 200 V is applied across the pair. (a) Calculate the equivalent capacitance. What are (b) charge q1 and (c) potential difference V1 on capacitor 1 and (d) q2 and (e) V2 on capacitor 2? 68 Repeat Problem 67 for the same two capacitors but with them now connected in parallel.

69 A certain capacitor is charged to a potential difference V. If you wish to increase its stored energy by 10%, by what percentage should you increase V? 70 A slab of copper of thickness b ! 2.00 mm is thrust into a paralleld plate capacitor of plate area A ! 2.40 b cm2 and plate separation d ! 5.00 mm, as shown in Fig. 25-57; the slab is exactly halfway between the plates. Figure 25-57 (a) What is the capacitance after the Problems 70 and 71. slab is introduced? (b) If a charge q ! 3.40 mC is maintained on the plates, what is the ratio of the stored energy before to that after the slab is inserted? (c) How much work is done on the slab as it is inserted? (d) Is the slab sucked in or must it be pushed in?

71 Repeat Problem 70, assuming that a potential difference V ! 85.0 V, rather than the charge, is held constant. 72 A potential difference of 300 V is applied to a series connection of two capacitors of capacitances C1 ! 2.00 mF and C2 ! 8.00 mF. What are (a) charge q1 and (b) potential difference V1 on capacitor 1 and (c) q2 and (d) V2 on capacitor 2? The charged capacitors are then disconnected from each other and from the battery. Then the capacitors are reconnected with plates of the same signs wired together (the battery is not used). What now are (e) q1, (f) V1, (g) q2, and (h) V2? Suppose, instead, the capacitors charged in part (a) are reconnected with plates of opposite signs wired together. What now are (i) q1, (j) V1, (k) q2, and (l) V2? 73 Figure 25-58 shows a fourcapacitor arrangement that is connected to a larger circuit at points A and B. The capacitances are C1 ! 10 mF and C2 ! C3 ! C4 ! 20 mF. The charge on capacitor 1 is 30 mC. What is the magnitude of the potential difference VA # VB?

C1

C3

A

B C2

C4

Figure 25-58 Problem 73.

74 You have two plates of copper, a sheet of mica (thickness ! 0.10 mm, k ! 5.4), a sheet of glass (thickness ! 2.0 mm, k ! 7.0), and a slab of paraffin (thickness ! 1.0 cm, k ! 2.0). To make a parallel-plate capacitor with the largest C, which sheet should you place between the copper plates? 75 A capacitor of unknown capacitance C is charged to 100 V and connected across an initially uncharged 60 mF capacitor. If the final potential difference across the 60 mF capacitor is 40 V, what is C? 76 A 10 V battery is connected to a series of n capacitors, each of capacitance 2.0 mF. If the total stored energy is 25 mJ, what is n? 77 SSM In Fig. 25-59, two parallelplate capacitors A and B are con+ – nected in parallel across a 600 V B A battery. Each plate has area 80.0 cm2; the plate separations are 3.00 mm. Figure 25-59 Problem 77. Capacitor A is filled with air; capacitor B is filled with a dielectric of dielectric constant k ! 2.60. Find the magnitude of the electric field within (a) the dielectric of capacitor B and (b) the air of capacitor A. What are the free charge densities s on the higher-potential plate of (c) capacitor A and (d) capacitor B? (e) What is the induced charge density s- on the top surface of the dielectric? 78 You have many 2.0 mF capacitors, each capable of withstanding 200 V without undergoing electrical breakdown (in which they conduct charge instead of storing it). How would you assemble a combination having an equivalent capacitance of (a) 0.40 mF and (b) 1.2 mF, each combination capable of withstanding 1000 V? 79 A parallel-plate capacitor has charge q and plate area A. (a) By finding the work needed to increase the plate separation from x to x % dx, determine the force between the plates. (Hint: See Eq. 8-22.) (b) Then show that the force per unit area (the electrostatic stress) acting on either plate is equal to the energy density ´0E2/2 between the plates. 80 A capacitor is charged until its stored energy is 4.00 J. A second capacitor is then connected to it in parallel. (a) If the charge distributes equally, what is the total energy stored in the electric fields? (b) Where did the missing energy go?

C

H

A

P

T

E

R

2

6

Current and Resistance 26-1 ELECTRIC CURRENT Learning Objectives After reading this module, you should be able to . . .

26.01 Apply the definition of current as the rate at which charge moves through a point, including solving for the amount of charge that passes the point in a given time interval. 26.02 Identify that current is normally due to the motion of conduction electrons that are driven by electric fields (such as those set up in a wire by a battery).

26.03 Identify a junction in a circuit and apply the fact that (due to conservation of charge) the total current into a junction must equal the total current out of the junction. 26.04 Explain how current arrows are drawn in a schematic diagram of a circuit, and identify that the arrows are not vectors.

Key Ideas ● An electric current i in a conductor is defined by

dq i! , dt where dq is the amount of positive charge that passes in time dt.

● By

convention, the direction of electric current is taken as the direction in which positive charge carriers would move even though (normally) only conduction electrons can move.

What Is Physics? In the last five chapters we discussed electrostatics — the physics of stationary charges. In this and the next chapter, we discuss the physics of electric currents — that is, charges in motion. Examples of electric currents abound and involve many professions. Meteorologists are concerned with lightning and with the less dramatic slow flow of charge through the atmosphere. Biologists, physiologists, and engineers working in medical technology are concerned with the nerve currents that control muscles and especially with how those currents can be reestablished after spinal cord injuries. Electrical engineers are concerned with countless electrical systems, such as power systems, lightning protection systems, information storage systems, and music systems. Space engineers monitor and study the flow of charged particles from our Sun because that flow can wipe out telecommunication systems in orbit and even power transmission systems on the ground. In addition to such scholarly work, almost every aspect of daily life now depends on information carried by electric currents, from stock trades to ATM transfers and from video entertainment to social networking. In this chapter we discuss the basic physics of electric currents and why they can be established in some materials but not in others. We begin with the meaning of electric current. 745

746

CHAPTE R 26 CU R R E NT AN D R ESISTANCE

Electric Current Although an electric current is a stream of moving charges, not all moving charges constitute an electric current. If there is to be an electric current through a given surface, there must be a net flow of charge through that surface. Two examples clarify our meaning.

(a ) i i

i

Battery i +

i

– (b )

Figure 26-1 (a) A loop of copper in electrostatic equilibrium.The entire loop is at a single potential, and the electric field is zero at all points inside the copper. (b) Adding a battery imposes an electric potential difference between the ends of the loop that are connected to the terminals of the battery. The battery thus produces an electric field within the loop, from terminal to terminal, and the field causes charges to move around the loop.This movement of charges is a current i.

1. The free electrons (conduction electrons) in an isolated length of copper wire are in random motion at speeds of the order of 106 m/s. If you pass a hypothetical plane through such a wire, conduction electrons pass through it in both directions at the rate of many billions per second—but there is no net transport of charge and thus no current through the wire. However, if you connect the ends of the wire to a battery, you slightly bias the flow in one direction, with the result that there now is a net transport of charge and thus an electric current through the wire. 2. The flow of water through a garden hose represents the directed flow of positive charge (the protons in the water molecules) at a rate of perhaps several million coulombs per second. There is no net transport of charge, however, because there is a parallel flow of negative charge (the electrons in the water molecules) of exactly the same amount moving in exactly the same direction. In this chapter we restrict ourselves largely to the study — within the framework of classical physics — of steady currents of conduction electrons moving through metallic conductors such as copper wires. As Fig. 26-1a reminds us, any isolated conducting loop — regardless of whether it has an excess charge — is all at the same potential. No electric field can exist within it or along its surface. Although conduction electrons are available, no net electric force acts on them and thus there is no current. If, as in Fig. 26-1b, we insert a battery in the loop, the conducting loop is no longer at a single potential. Electric fields act inside the material making up the loop, exerting forces on the conduction electrons, causing them to move and thus establishing a current. After a very short time, the electron flow reaches a constant value and the current is in its steady state (it does not vary with time). Figure 26-2 shows a section of a conductor, part of a conducting loop in which current has been established. If charge dq passes through a hypothetical plane (such as aa-) in time dt, then the current i through that plane is defined as i!

dq dt

(definition of current).

(26-1)

We can find the charge that passes through the plane in a time interval extending from 0 to t by integration: q!

The current is the same in any cross section. a

b

i

c i c'

a'

b'

Figure 26-2 The current i through the conductor has the same value at planes aa-, bb-, and cc-.

"

dq !

"

t

0

i dt,

(26-2)

in which the current i may vary with time. Under steady-state conditions, the current is the same for planes aa-, bb-, and cc- and indeed for all planes that pass completely through the conductor, no matter what their location or orientation. This follows from the fact that charge is conserved. Under the steady-state conditions assumed here, an electron must pass through plane aa- for every electron that passes through plane cc-. In the same way, if we have a steady flow of water through a garden hose, a drop of water must leave the nozzle for every drop that enters the hose at the other end. The amount of water in the hose is a conserved quantity. The SI unit for current is the coulomb per second, or the ampere (A), which is an SI base unit: 1 ampere ! 1 A ! 1 coulomb per second ! 1 C/s. The formal definition of the ampere is discussed in Chapter 29.

747

26-1 E LECTR IC CU R R E NT

Current, as defined by Eq. 26-1, is a scalar because both charge and time in that equation are scalars. Yet, as in Fig. 26-1b, we often represent a current with an arrow to indicate that charge is moving. Such arrows are not vectors, however, and they do not require vector addition. Figure 26-3a shows a conductor with current i0 splitting at a junction into two branches. Because charge is conserved, the magnitudes of the currents in the branches must add to yield the magnitude of the current in the original conductor, so that i0 ! i1 % i2.

The current into the junction must equal the current out (charge is conserved).

i1

i0 a

(26-3)

i2

As Fig. 26-3b suggests, bending or reorienting the wires in space does not change the validity of Eq. 26-3. Current arrows show only a direction (or sense) of flow along a conductor, not a direction in space.

(a )

The Directions of Currents

i1

In Fig. 26-1b we drew the current arrows in the direction in which positively charged particles would be forced to move through the loop by the electric field. Such positive charge carriers, as they are often called, would move away from the positive battery terminal and toward the negative terminal. Actually, the charge carriers in the copper loop of Fig. 26-1b are electrons and thus are negatively charged.The electric field forces them to move in the direction opposite the current arrows, from the negative terminal to the positive terminal. For historical reasons, however, we use the following convention: A current arrow is drawn in the direction in which positive charge carriers would move, even if the actual charge carriers are negative and move in the opposite direction.

i0 a i2

(b )

Figure 26-3 The relation i0 ! i1 % i2 is true at junction a no matter what the orientation in space of the three wires. Currents are scalars, not vectors.

We can use this convention because in most situations, the assumed motion of positive charge carriers in one direction has the same effect as the actual motion of negative charge carriers in the opposite direction. (When the effect is not the same, we shall drop the convention and describe the actual motion.)

Checkpoint 1 The figure here shows a portion of a circuit. What are the magnitude and direction of the current i in the lower right-hand wire?

1A 2A

2A

2A 3A

4A i

Sample Problem 26.01 Current is the rate at which charge passes a point Water flows through a garden hose at a volume flow rate dV/dt of 450 cm3/s. What is the current of negative charge?

Calculations: We can write the current in terms of the number of molecules that pass through such a plane per second as

KEY IDEAS The current i of negative charge is due to the electrons in the water molecules moving through the hose. The current is the rate at which that negative charge passes through any plane that cuts completely across the hose.

i!

or

#

charge per electron

$#

electrons per molecule

i ! (e)(10)

$#

dN . dt

molecules per second

$

748

CHAPTE R 26 CU R R E NT AN D R ESISTANCE

We substitute 10 electrons per molecule because a water (H2O) molecule contains 8 electrons in the single oxygen atom and 1 electron in each of the two hydrogen atoms. We can express the rate dN/dt in terms of the given volume flow rate dV/dt by first writing

#

$ #

$# $ # $# $

molecules molecules per per ! second mole

mass $ per unit volume

moles per unit mass

volume per . second

“Molecules per mole” is Avogadro’s number NA. “Moles per unit mass” is the inverse of the mass per mole, which is the molar mass M of water. “Mass per unit volume” is the (mass) density rmass of water. The volume per second is the volume flow rate dV/dt.Thus, we have

# $ # $

dN 1 dV ! NA rmass dt M dt

NArmass dV ! . M dt

Substituting this into the equation for i, we find i ! 10eNAM #1rmass

dV . dt

We know that Avogadro’s number NA is 6.02 $ 10 23 molecules/mol, or 6.02 $ 10 23 mol#1, and from Table 15-1 we know that the density of water rmass under normal conditions is 1000 kg/m3. We can get the molar mass of water from the molar masses listed in Appendix F (in grams per mole): We add the molar mass of oxygen (16 g/mol) to twice the molar mass of hydrogen (1 g/mol), obtaining 18 g/mol ! 0.018 kg/mol. So, the current of negative charge due to the electrons in the water is i ! (10)(1.6 $ 10 #19 C)(6.02 $ 10 23 mol#1) $ (0.018 kg/mol)#1(1000 kg/m3)(450 $ 10 #6 m3/s) ! 2.41 $ 10 7 C/s ! 2.41 $ 10 7 A ! 24.1 MA. (Answer) This current of negative charge is exactly compensated by a current of positive charge associated with the nuclei of the three atoms that make up the water molecule. Thus, there is no net flow of charge through the hose.

Additional examples, video, and practice available at WileyPLUS

26-2 CURRENT DENSITY Learning Objectives After reading this module, you should be able to . . .

26.05 Identify a current density and a current density vector. 26.06 For current through an area element on a cross section through a conductor (such as a wire), identify the element’s : area vector dA . 26.07 Find the current through a cross section of a conductor by integrating the dot product of the current density : : vector J and the element area vector dA over the full cross section. 26.08 For the case where current is uniformly spread over a cross section in a conductor, apply the relationship

between the current i, the current density magnitude J, and the area A. 26.09 Identify streamlines. 26.10 Explain the motion of conduction electrons in terms of their drift speed. 26.11 Distinguish the drift speeds of conduction electrons from their random-motion speeds, including relative magnitudes. 26.12 Identify carrier charge density n. 26.13 Apply the relationship between current density J, charge carrier density n, and charge carrier drift speed vd.

Key Ideas ● Current i (a scalar quantity) is related to current density

(a vector quantity) by i! :

"

:

:

J

:

J " dA,

where dA is a vector perpendicular to a surface element of area dA and the integral is taken over any surface : cutting across the conductor. The current density J has the same direction as the velocity of the moving charges if

they are positive and the opposite direction if they are negative. : ● When an electric field E is established in a conductor, the charge carriers (assumed positive) acquire a drift speed vd in : the direction of E . ● The drift velocity vd :

is related to the current density by :

J ! (ne)v:d,

where ne is the carrier charge density.

26-2 CU R R E NT DE NSITY

749

Current Density Sometimes we are interested in the current i in a particular conductor. At other times we take a localized view and study the flow of charge through a cross section of the conductor at a particular point. To describe this flow, we can use the : current density J , which has the same direction as the velocity of the moving charges if they are positive and the opposite direction if they are negative. For each element of the cross section, the magnitude J is equal to the current per unit area through that element. We can write the amount of current through the ele: : : ment as J " dA, where dA is the area vector of the element, perpendicular to the element. The total current through the surface is then i!

"

:

:

J " dA.

(26-4) :

:

If the current is uniform across the surface and parallel to dA, then J is also uni: form and parallel to dA. Then Eq. 26-4 becomes i!

"

J dA ! J

"

dA ! JA,

i (26-5) , A where A is the total area of the surface. From Eq. 26-4 or 26-5 we see that the SI unit for current density is the ampere per square meter (A/m2). In Chapter 22 we saw that we can represent an electric field with electric field lines. Figure 26-4 shows how current density can be represented with a similar set of lines, which we can call streamlines. The current, which is toward the right in Fig. 26-4, makes a transition from the wider conductor at the left to the narrower conductor at the right. Because charge is conserved during the transition, the amount of charge and thus the amount of current cannot change. However, the current density does change — it is greater in the narrower conductor. The spacing of the streamlines suggests this increase in current density; streamlines that are closer together imply greater current density. so

J!

Drift Speed When a conductor does not have a current through it, its conduction electrons move randomly, with no net motion in any direction. When the conductor does have a current through it, these electrons actually still move randomly, but now they tend to drift with a drift speed vd in the direction opposite that of the applied electric field that causes the current. The drift speed is tiny compared with the speeds in the random motion. For example, in the copper conductors of household wiring, electron drift speeds are perhaps 10 #5 or 10 #4 m/s, whereas the random-motion speeds are around 10 6 m/s. We can use Fig. 26-5 to relate the drift speed vd of the conduction electrons in a current through a wire to the magnitude J of the current density in the wire. For Figure 26-5 Positive charge carriers drift at speed vd in the direction : of the applied electric field E. By convention, the direction of the : current density J and the sense of the current arrow are drawn in that same direction.

Current is said to be due to positive charges that are propelled by the electric field. i L

+

+

+

+ vd

E J

+

i

Figure 26-4 Streamlines representing current density in the flow of charge through a constricted conductor.

750

CHAPTE R 26 CU R R E NT AN D R ESISTANCE

convenience, Fig. 26-5 shows the equivalent drift of positive charge carriers in the : direction of the applied electric field E. Let us assume that these charge carriers all move with the same drift speed vd and that the current density J is uniform across the wire’s cross-sectional area A. The number of charge carriers in a length L of the wire is nAL, where n is the number of carriers per unit volume. The total charge of the carriers in the length L, each with charge e, is then q ! (nAL)e. Because the carriers all move along the wire with speed vd, this total charge moves through any cross section of the wire in the time interval t!

L . vd

Equation 26-1 tells us that the current i is the time rate of transfer of charge across a cross section, so here we have i!

nALe q ! nAevd. ! t L/vd

(26-6)

Solving for vd and recalling Eq. 26-5 (J ! i/A), we obtain vd !

i J ! nAe ne

or, extended to vector form, :

: J ! (ne)v d.

(26-7)

Here the product ne, whose SI unit is the coulomb per cubic meter (C/m3), is the carrier charge density. For positive carriers, ne is positive and Eq. 26-7 predicts : : that J and : vd have the same direction. For negative carriers, ne is negative and J : and vd have opposite directions.

Checkpoint 2 The figure shows conduction electrons moving leftward in a wire. Are the following leftward or rightward: (a) the current i, (b) the current density : : J , (c) the electric field E in the wire?

Sample Problem 26.02 Current density, uniform and nonuniform (a) The current density in a cylindrical wire of radius R ! 2.0 mm is uniform across a cross section of the wire and is J ! 2.0 $ 10 5 A/m2.What is the current through the outer portion of the wire between radial distances R/2 and R (Fig. 26-6a)? KEY IDEA Because the current density is uniform across the cross section, the current density J, the current i, and the crosssectional area A are related by Eq. 26-5 (J ! i/A). Calculations: We want only the current through a reduced cross-sectional area A- of the wire (rather than the entire

area), where A- ! pR2 # p !

# R2 $ ! p # 3R4 $ 2

2

3p (0.0020 m)2 ! 9.424 $ 10 #6 m2. 4

So, we rewrite Eq. 26-5 as i ! JAand then substitute the data to find i ! (2.0 $ 10 5 A/m2)(9.424 $ 10 #6 m2) ! 1.9 A.

(Answer)

751

26-2 CU R R E NT DE NSITY

(b) Suppose, instead, that the current density through a cross section varies with radial distance r as J ! ar2, in which a ! 3.0 $ 10 11 A/m4 and r is in meters. What now is the current through the same outer portion of the wire? KEY IDEA Because the current density is not uniform across a cross : : section of the wire, we must resort to Eq. 26-4 (i ! " J " dA) and integrate the current density over the portion of the wire from r ! R/2 to r ! R.

We need to replace the differential area dA with something we can actually integrate between the limits r ! R/2 and r ! R. The simplest replacement (because J is given as a function of r) is the area 2pr dr of a thin ring of circumference 2pr and width dr (Fig. 26-6b). We can then integrate with r as the variable of integration. Equation 26-4 then gives us i!

:

:

J " dA ! J dA cos 0 ! J dA.

We want the current in the area between these two radii.

R/2

:

R/2

! 2pa !

:

J " dA !

R

!

:

Calculations: The current density vector J (along the : wire’s length) and the differential area vector dA (perpendicular to a cross section of the wire) have the same direction. Thus,

" "

"

J dA

ar 2 2pr dr ! 2pa

( ) r4 4

R R/2

!

pa 2

"

R/2

(

r 3 dr

R4 #

R4 16

) ! 1532 paR

4

15 p (3.0 $ 10 11 A/m4)(0.0020 m)4 ! 7.1 A. 32 (Answer)

If the current is nonuniform, we start with a ring that is so thin that we can approximate the current density as being uniform within it.

A Its area is the product of the circumference and the width.

R

(a)

R

dr

(c)

(b)

The current within the ring is the product of the current density and the ring’s area. Our job is to sum the current in all rings from this smallest one ...

Figure 26-6 (a) Cross section of a wire of radius R. If the current density is uniform, the current is just the product of the current density and the area. (b)–(e) If the current is nonuniform, we must first find the current through a thin ring and then sum (via integration) the currents in all such rings in the given area.

... to this largest one.

R

R/2

(d)

Additional examples, video, and practice available at WileyPLUS

(e)

752

CHAPTE R 26 CU R R E NT AN D R ESISTANCE

Sample Problem 26.03 In a current, the conduction electrons move very slowly What is the drift speed of the conduction electrons in a copper wire with radius r ! 900 mm when it has a uniform current i ! 17 mA? Assume that each copper atom contributes one conduction electron to the current and that the current density is uniform across the wire’s cross section.

:

1. The drift speed vd is related to the current density J and the number n of conduction electrons per unit volume according to Eq. 26-7, which we can write as J ! nevd. 2. Because the current density is uniform, its magnitude J is related to the given current i and wire size by Eq. 26-5 (J ! i/A, where A is the cross-sectional area of the wire). 3. Because we assume one conduction electron per atom, the number n of conduction electrons per unit volume is the same as the number of atoms per unit volume. Calculations: Let us start with the third idea by writing

#

atoms per unit volume

n!

$ # $# !

atoms per mole

moles per unit mass

$#

$

mass per unit . volume

The number of atoms per mole is just Avogadro’s number NA (! 6.02 $ 10 23 mol#1). Moles per unit mass is the inverse of the mass per mole, which here is the molar mass M of copper. The mass per unit volume is the (mass) density rmass of copper. Thus,

# M1 $r

n ! NA

mass

!

NArmass . M

(6.02 $ 10 23 mol#1)(8.96 $ 10 3 kg/m3) 63.54 $ 10 #3 kg/mol

! 8.49 $ 10 28 electrons/m3 or

KEY IDEAS

n!

Taking copper’s molar mass M and density rmass from Appendix F, we then have (with some conversions of units)

n ! 8.49 $ 10 28 m#3.

Next let us combine the first two key ideas by writing i ! nevd. A Substituting for A with pr 2 (! 2.54 $ 10 #6 m2) and solving for vd, we then find i vd ! ne(pr 2) 17 $ 10 #3 A ! (8.49 $ 10 28 m#3)(1.6 $ 10 #19 C)(2.54 $ 10 #6 m2) ! 4.9 $ 10 #7 m/s,

(Answer)

which is only 1.8 mm/h, slower than a sluggish snail. Lights are fast: You may well ask: “If the electrons drift so slowly, why do the room lights turn on so quickly when I throw the switch?” Confusion on this point results from not distinguishing between the drift speed of the electrons and the speed at which changes in the electric field configuration travel along wires. This latter speed is nearly that of light; electrons everywhere in the wire begin drifting almost at once, including into the lightbulbs. Similarly, when you open the valve on your garden hose with the hose full of water, a pressure wave travels along the hose at the speed of sound in water. The speed at which the water itself moves through the hose — measured perhaps with a dye marker — is much slower.

Additional examples, video, and practice available at WileyPLUS

26-3 RESISTANCE AND RESISTIVITY Learning Objectives After reading this module, you should be able to . . .

26.14 Apply the relationship between the potential difference V applied across an object, the object’s resistance R, and the resulting current i through the object, between the application points. 26.15 Identify a resistor. 26.16 Apply the relationship between the electric field magnitude E set up at a point in a given material, the material’s resistivity r, and the resulting current density magnitude J at that point. 26.17 For a uniform electric field set up in a wire, apply the relationship between the electric field magnitude E,

the potential difference V between the two ends, and the wire’s length L. 26.18 Apply the relationship between resistivity r and conductivity s. 26.19 Apply the relationship between an object’s resistance R, the resistivity of its material r, its length L, and its crosssectional area A. 26.20 Apply the equation that approximately gives a conductor’s resistivity r as a function of temperature T. 26.21 Sketch a graph of resistivity r versus temperature T for a metal.

26-3 R ESISTANCE AN D R ESISTIVITY

753

Key Ideas ● The resistance R of a conductor is defined as

● The resistance R of a conducting wire of length L and

V , R! i where V is the potential difference across the conductor and i is the current.

uniform cross section is

● The resistivity r and conductivity s of a material are related by

● The resistivity r for most materials changes with tempera-

E 1 ! , s J where E is the magnitude of the applied electric field and J is the magnitude of the current density. r!

● The electric field and current density are related to the resistivity by : : E ! rJ.

L , A where A is the cross-sectional area. R!r

ture. For many materials, including metals, the relation between r and temperature T is approximated by the equation r # r0 ! r0a(T # T0). Here T0 is a reference temperature, r0 is the resistivity at T0, and a is the temperature coefficient of resistivity for the material.

Resistance and Resistivity

R!

V i

(definition of R).

(26-8)

The SI unit for resistance that follows from Eq. 26-8 is the volt per ampere.This combination occurs so often that we give it a special name, the ohm (symbol 0); that is, 1 ohm ! 1 0 ! 1 volt per ampere ! 1 V/A.

(26-9)

A conductor whose function in a circuit is to provide a specified resistance is called a resistor (see Fig. 26-7). In a circuit diagram, we represent a resistor and a resistance with the symbol . If we write Eq. 26-8 as i!

V , R

we see that, for a given V, the greater the resistance, the smaller the current. The resistance of a conductor depends on the manner in which the potential difference is applied to it. Figure 26-8, for example, shows a given potential difference applied in two different ways to the same conductor. As the current density streamlines suggest, the currents in the two cases — hence the measured resistances — will be different. Unless otherwise stated, we shall assume that any given potential difference is applied as in Fig. 26-8b.

(a )

(b )

Figure 26-8 Two ways of applying a potential difference to a conducting rod. The gray connectors are assumed to have negligible resistance. When they are arranged as in (a) in a small region at each rod end, the measured resistance is larger than when they are arranged as in (b) to cover the entire rod end.

The Image Works

If we apply the same potential difference between the ends of geometrically similar rods of copper and of glass, very different currents result. The characteristic of the conductor that enters here is its electrical resistance. We determine the resistance between any two points of a conductor by applying a potential difference V between those points and measuring the current i that results. The resistance R is then

Figure 26-7 An assortment of resistors. The circular bands are color-coding marks that identify the value of the resistance.

754

CHAPTE R 26 CU R R E NT AN D R ESISTANCE

Table 26-1 Resistivities of Some Materials at Room Temperature (20/C)

Temperature Coefficient of Resistivity, a (K#1)

Material

Resistivity, r (0&m)

Silver Copper Gold Aluminum Manganina Tungsten Iron Platinum

Typical Metals 1.62 $ 10 #8 4.1 $ 10 #3 1.69 $ 10 #8 4.3 $ 10 #3 2.35 $ 10 #8 4.0 $ 10 #3 2.75 $ 10 #8 4.4 $ 10 #3 4.82 $ 10 #8 0.002 $ 10 #3 5.25 $ 10 #8 4.5 $ 10 #3 9.68 $ 10 #8 6.5 $ 10 #3 10.6 $ 10 #8 3.9 $ 10 #3 Typical Semiconductors

Silicon, pure Silicon, n-typeb Silicon, p-typec Glass Fused quartz

2.5 $ 10 3

#70 $ 10 #3

8.7 $ 10 #4

As we have done several times in other connections, we often wish to take a general view and deal not with particular objects but with materials. Here we do so by focusing not on the potential difference V across a particular resistor but on : the electric field E at a point in a resistive material. Instead of dealing with the : current i through the resistor, we deal with the current density J at the point in question. Instead of the resistance R of an object, we deal with the resistivity r of the material: r!

E J

(definition of r).

(26-10)

(Compare this equation with Eq. 26-8.) If we combine the SI units of E and J according to Eq. 26-10, we get, for the unit of r, the ohm-meter (0&m): V/m V unit (E) ! ! m ! 0&m. unit (J) A/m2 A (Do not confuse the ohm-meter, the unit of resistivity, with the ohmmeter, which is an instrument that measures resistance.) Table 26-1 lists the resistivities of some materials. We can write Eq. 26-10 in vector form as :

:

E ! rJ.

2.8 $ 10 #3

(26-11)

Equations 26-10 and 26-11 hold only for isotropic materials — materials whose electrical properties are the same in all directions. We often speak of the conductivity s of a material. This is simply the reciprocal of its resistivity, so

Typical Insulators 10 10 #10 14 .10 16

a

An alloy specifically designed to have a small value of a. b Pure silicon doped with phosphorus impurities to a charge carrier density of 10 23 m#3. c Pure silicon doped with aluminum impurities to a charge carrier density of 10 23 m#3.

s!

1 r

(definition of s).

(26-12)

The SI unit of conductivity is the reciprocal ohm-meter, (0&m)#1. The unit name mhos per meter is sometimes used (mho is ohm backwards). The definition of s allows us to write Eq. 26-11 in the alternative form :

:

J ! sE.

(26-13)

Calculating Resistance from Resistivity We have just made an important distinction: Resistance is a property of an object. Resistivity is a property of a material.

Current is driven by a potential difference. L i

i

A V

Figure 26-9 A potential difference V is applied between the ends of a wire of length L and cross section A, establishing a current i.

If we know the resistivity of a substance such as copper, we can calculate the resistance of a length of wire made of that substance. Let A be the cross-sectional area of the wire, let L be its length, and let a potential difference V exist between its ends (Fig. 26-9). If the streamlines representing the current density are uniform throughout the wire, the electric field and the current density will be constant for all points within the wire and, from Eqs. 24-42 and 26-5, will have the values E ! V/L and

J ! i/A.

(26-14)

We can then combine Eqs. 26-10 and 26-14 to write r!

E V/L ! . J i/A

(26-15)

26-3 R ESISTANCE AN D R ESISTIVITY

However, V/i is the resistance R, which allows us to recast Eq. 26-15 as R!r

L . A

(26-16)

Equation 26-16 can be applied only to a homogeneous isotropic conductor of uniform cross section, with the potential difference applied as in Fig. 26-8b. The macroscopic quantities V, i, and R are of greatest interest when we are making electrical measurements on specific conductors. They are the quantities that we read directly on meters. We turn to the microscopic quantities E, J, and r when we are interested in the fundamental electrical properties of materials.

Checkpoint 3 The figure here shows three cylindrical copper conductors along with their face areas and lengths. Rank them according to the current through them, greatest first, when the same potential difference V is placed across their lengths. L A

A _ 2 (a)

1.5L

A _ 2

L/2

(c)

(b)

Variation with Temperature The values of most physical properties vary with temperature, and resistivity is no exception. Figure 26-10, for example, shows the variation of this property for copper over a wide temperature range. The relation between temperature and resistivity for copper — and for metals in general — is fairly linear over a rather broad temperature range. For such linear relations we can write an empirical approximation that is good enough for most engineering purposes: r # r0 ! r0a(T #T0).

(26-17)

Here T0 is a selected reference temperature and r0 is the resistivity at that temperature. Usually T0 ! 293 K (room temperature), for which r0 ! 1.69 $ 10 #8 0&m for copper. Because temperature enters Eq. 26-17 only as a difference, it does not matter whether you use the Celsius or Kelvin scale in that equation because the sizes of degrees on these scales are identical. The quantity a in Eq. 26-17, called the temperature coefficient of resistivity, is chosen so that the equation gives good agreement with experiment for temperatures in the chosen range. Some values of a for metals are listed in Table 26-1. Room temperature

Resistivity (10–8 Ω .m)

10 8 6 4 2 0

0

Resistivity can depend on temperature.

(T0, ρ0)

200 400 600 800 1000 1200 Temperature (K)

Figure 26-10 The resistivity of copper as a function of temperature. The dot on the curve marks a convenient reference point at temperature T0 ! 293 K and resistivity r0 ! 1.69 $ 10 #8 0&m.

755

756

CHAPTE R 26 CU R R E NT AN D R ESISTANCE

Sample Problem 26.04 A material has resistivity, a block of the material has resistance A rectangular block of iron has dimensions 1.2 cm $ 1.2 cm $ 15 cm. A potential difference is to be applied to the block between parallel sides and in such a way that those sides are equipotential surfaces (as in Fig. 26-8b). What is the resistance of the block if the two parallel sides are (1) the square ends (with dimensions 1.2 cm $ 1.2 cm) and (2) two rectangular sides (with dimensions 1.2 cm $ 15 cm)?

Calculations: For arrangement 1, we have L ! 15 cm ! 0.15 m and A ! (1.2 cm)2 ! 1.44 $ 10 #4 m2. Substituting into Eq. 26-16 with the resistivity r from Table 26-1, we then find that for arrangement 1, rL (9.68 $ 10 #8 0&m)(0.15 m) ! A 1.44 $ 10 #4 m2 (Answer) ! 1.0 $ 10 #4 0 ! 100 m0.

R!

KEY IDEA The resistance R of an object depends on how the electric potential is applied to the object. In particular, it depends on the ratio L/A, according to Eq. 26-16 (R ! rL/A), where A is the area of the surfaces to which the potential difference is applied and L is the distance between those surfaces.

Similarly, for arrangement 2, with distance L ! 1.2 cm and area A ! (1.2 cm)(15 cm), we obtain R!

(9.68 $ 10 #8 0&m)(1.2 $ 10 #2 m) rL ! A 1.80 $ 10 #3 m2

! 6.5 $ 10 #7 0 ! 0.65 m0.

(Answer)

Additional examples, video, and practice available at WileyPLUS

26-4 OHM’S LAW Learning Objectives After reading this module, you should be able to . . .

26.22 Distinguish between an object that obeys Ohm’s law and one that does not. 26.23 Distinguish between a material that obeys Ohm’s law and one that does not. 26.24 Describe the general motion of a conduction electron in a current.

26.25 For the conduction electrons in a conductor, explain the relationship between the mean free time t, the effective speed, and the thermal (random) motion. 26.26 Apply the relationship between resistivity r, number density n of conduction electrons, and the mean free time t of the electrons.

Key Ideas ● A given device (conductor, resistor, or any other electrical device) obeys Ohm’s law if its resistance R (! V/i) is independent of the applied potential difference V. ● A given material obeys Ohm’s law if its resistivity r (! E/J)

is independent of the magnitude and direction of the applied : electric field E . ● The assumption that the conduction electrons in a metal are free to move like the molecules in a gas leads to an

expression for the resistivity of a metal: m r! 2 . e nt Here n is the number of free electrons per unit volume and t is the mean time between the collisions of an electron with the atoms of the metal. ● Metals obey Ohm’s law because the mean free time t is approximately independent of the magnitude E of any electric field applied to a metal.

Ohm’s Law As we just discussed, a resistor is a conductor with a specified resistance. It has that same resistance no matter what the magnitude and direction (polarity) of the applied potential difference are. Other conducting devices, however, might have resistances that change with the applied potential difference.

26-4 OH M’S L AW

Ohm’s law is an assertion that the current through a device is always directly proportional to the potential difference applied to the device.

(This assertion is correct only in certain situations; still, for historical reasons, the term “law” is used.) The device of Fig. 26-11b — which turns out to be a 1000 0 resistor — obeys Ohm’s law. The device of Fig. 26-11c — which is called a pn junction diode — does not. A conducting device obeys Ohm’s law when the resistance of the device is independent of the magnitude and polarity of the applied potential difference.

It is often contended that V ! iR is a statement of Ohm’s law. That is not true! This equation is the defining equation for resistance, and it applies to all conducting devices, whether they obey Ohm’s law or not. If we measure the potential difference V across, and the current i through, any device, even a pn junction diode, we can find its resistance at that value of V as R ! V/i. The essence of Ohm’s law, however, is that a plot of i versus V is linear; that is, R is independent of V.We can gener: : alize this for conducting materials by using Eq. 26-11 (E ! r J ): A conducting material obeys Ohm’s law when the resistivity of the material is independent of the magnitude and direction of the applied electric field.

All homogeneous materials, whether they are conductors like copper or semiconductors like pure silicon or silicon containing special impurities, obey Ohm’s law within some range of values of the electric field. If the field is too strong, however, there are departures from Ohm’s law in all cases.

Checkpoint 4 The following table gives the current i (in amperes) through two devices for several values of potential difference V (in volts). From these data, determine which device does not obey Ohm’s law.

Device 1

Device 2

V

i

V

i

2.00 3.00 4.00

4.50 6.75 9.00

2.00 3.00 4.00

1.50 2.20 2.80

V + i

?

– i

Current (mA)

(a )

+2 0 –2 –4

Current (mA)

Figure 26-11a shows how to distinguish such devices. A potential difference V is applied across the device being tested, and the resulting current i through the device is measured as V is varied in both magnitude and polarity. The polarity of V is arbitrarily taken to be positive when the left terminal of the device is at a higher potential than the right terminal. The direction of the resulting current (from left to right) is arbitrarily assigned a plus sign. The reverse polarity of V (with the right terminal at a higher potential) is then negative; the current it causes is assigned a minus sign. Figure 26-11b is a plot of i versus V for one device. This plot is a straight line passing through the origin, so the ratio i/V (which is the slope of the straight line) is the same for all values of V. This means that the resistance R ! V/i of the device is independent of the magnitude and polarity of the applied potential difference V. Figure 26-11c is a plot for another conducting device. Current can exist in this device only when the polarity of V is positive and the applied potential difference is more than about 1.5 V. When current does exist, the relation between i and V is not linear; it depends on the value of the applied potential difference V. We distinguish between the two types of device by saying that one obeys Ohm’s law and the other does not.

757

–2 0 +2 +4 Potential difference (V) (b )

+4 +2 0 –2

–4

–2 0 +2 +4 Potential difference (V) (c )

Figure 26-11 (a) A potential difference V is applied to the terminals of a device, establishing a current i. (b) A plot of current i versus applied potential difference V when the device is a 1000 0 resistor. (c) A plot when the device is a semiconducting pn junction diode.

758

CHAPTE R 26 CU R R E NT AN D R ESISTANCE

A Microscopic View of Ohm’s Law To find out why particular materials obey Ohm’s law, we must look into the details of the conduction process at the atomic level. Here we consider only conduction in metals, such as copper. We base our analysis on the free-electron model, in which we assume that the conduction electrons in the metal are free to move throughout the volume of a sample, like the molecules of a gas in a closed container. We also assume that the electrons collide not with one another but only with atoms of the metal. According to classical physics, the electrons should have a Maxwellian speed distribution somewhat like that of the molecules in a gas (Module 19-6), and thus the average electron speed should depend on the temperature. The motions of electrons are, however, governed not by the laws of classical physics but by those of quantum physics. As it turns out, an assumption that is much closer to the quantum reality is that conduction electrons in a metal move with a single effective speed veff, and this speed is essentially independent of the temperature. For copper, veff % 1.6 $ 10 6 m/s. When we apply an electric field to a metal sample, the electrons modify their random motions slightly and drift very slowly — in a direction opposite that of the field — with an average drift speed vd. The drift speed in a typical metallic conductor is about 5 $ 10 #7 m/s, less than the effective speed (1.6 $ 10 6 m/s) by many orders of magnitude. Figure 26-12 suggests the relation between these two speeds. The gray lines show a possible random path for an electron in the absence of an applied field; the electron proceeds from A to B, making six collisions along the way. The green lines show how the same events might occur when an electric : field E is applied. We see that the electron drifts steadily to the right, ending at Brather than at B. Figure 26-12 was drawn with the assumption that vd % 0.02veff. However, because the actual value is more like vd % (10 #13)veff, the drift displayed in the figure is greatly exaggerated. : The motion of conduction electrons in an electric field E is thus a combina: tion of the motion due to random collisions and that due to E. When we consider all the free electrons, their random motions average to zero and make no contribution to the drift speed. Thus, the drift speed is due only to the effect of the electric field on the electrons. If an electron of mass m is placed in an electric field of magnitude E, the electron will experience an acceleration given by Newton’s second law: eE F (26-18) ! . m m After a typical collision, each electron will — so to speak — completely lose its memory of its previous drift velocity, starting fresh and moving off in a random direction. In the average time t between collisions, the average electron will acquire a drift speed of vd ! at. Moreover, if we measure the drift speeds of all the electrons at any instant, we will find that their average drift speed is also at. Thus, at any instant, on average, the electrons will have drift speed vd ! at.Then Eq. 26-18 gives us a!

vd ! at !

eEt . m

(26-19)

Figure 26-12 The gray lines show an electron moving from A to B, making six collisions en route. The green lines show what the electron’s path might be in the : presence of an applied electric field E . Note the steady : drift in the direction of #E . (Actually, the green lines should be slightly curved, to represent the parabolic paths followed by the electrons between collisions, under the influence of an electric field.)

B

A

E

B'

759

26-4 OH M’S L AW :

Combining this result with Eq. 26-7 ( J ! nev:d), in magnitude form, yields vd ! which we can write as E! :

eEt J ! , ne m

(26-20)

# emnt $ J.

(26-21)

2

:

Comparing this with Eq. 26-11 (E ! r J ), in magnitude form, leads to m r! 2 . e nt

(26-22)

Equation 26-22 may be taken as a statement that metals obey Ohm’s law if we can show that, for metals, their resistivity r is a constant, independent of the : strength of the applied electric field E. Let’s consider the quantities in Eq. 26-22. We can reasonably assume that n, the number of conduction electrons per volume, is independent of the field, and m and e are constants. Thus, we only need to convince ourselves that t, the average time (or mean free time) between collisions, is a constant, independent of the strength of the applied electric field. Indeed, t can be considered to be a constant because the drift speed vd caused by the field is so much smaller than the effective speed veff that the electron speed — and thus t— is hardly affected by the field. Thus, because the right side of Eq. 26-22 is independent of the field magnitude, metals obey Ohm’s law.

Sample Problem 26.05 Mean free time and mean free distance (a) What is the mean free time t between collisions for the conduction electrons in copper?

Using these results and substituting for the electron mass m, we then have t!

KEY IDEAS

9.1 $ 10 #31 kg ! 2.5 $ 10 #14 s. (Answer) 3.67 $ 10 #17 kg/s

The mean free time t of copper is approximately constant, and in particular does not depend on any electric field that might be applied to a sample of the copper. Thus, we need not consider any particular value of applied electric field. However, because the resistivity r displayed by copper under an electric field depends on t, we can find the mean free time t from Eq. 26-22 (r ! m/e2nt).

(b) The mean free path l of the conduction electrons in a conductor is the average distance traveled by an electron between collisions. (This definition parallels that in Module 19-5 for the mean free path of molecules in a gas.) What is l for the conduction electrons in copper, assuming that their effective speed veff is 1.6 $ 10 6 m/s?

Calculations: That equation gives us m t! . ne 2r

KEY IDEA (26-23)

The number of conduction electrons per unit volume in copper is 8.49 $ 10 28 m#3.We take the value of r from Table 26-1. The denominator then becomes (8.49 $ 10 28 m#3)(1.6 $ 10 #19 C)2(1.69 $ 10 #8 0&m) ! 3.67 $ 10 #17 C 2 &0/m2 ! 3.67 $ 10 #17 kg/s, where we converted units as C 2 &0 C 2 &J/C kg&m2/s2 kg C 2 &V ! ! ! . ! 2 2 2 2 m m &A m &C/s m /s s

The distance d any particle travels in a certain time t at a constant speed v is d ! vt. Calculation: For the electrons in copper, this gives us (26-24)

l ! vefft 6

! (1.6 $ 10 m/s)(2.5 $ 10 ! 4.0 $ 10

#8

m ! 40 nm.

#14

s) (Answer)

This is about 150 times the distance between nearestneighbor atoms in a copper lattice. Thus, on the average, each conduction electron passes many copper atoms before finally hitting one.

Additional examples, video, and practice available at WileyPLUS

760

CHAPTE R 26 CU R R E NT AN D R ESISTANCE

26-5 POWER, SEMICONDUCTORS, SUPERCONDUCTORS Learning Objectives After reading this module, you should be able to . . .

26.27 Explain how conduction electrons in a circuit lose energy in a resistive device. 26.28 Identify that power is the rate at which energy is transferred from one type to another. 26.29 For a resistive device, apply the relationships between power P, current i, voltage V, and resistance R.

26.30 For a battery, apply the relationship between power P, current i, and potential difference V. 26.31 Apply the conservation of energy to a circuit with a battery and a resistive device to relate the energy transfers in the circuit. 26.32 Distinguish conductors, semiconductors, and superconductors.

Key Ideas ● The power P, or rate of energy transfer, in an electrical device across which a potential difference V is maintained is

P ! iV. ● If the device is a resistor, the power can also be written as

V2 P!iR! . R 2

● In a resistor, electric potential energy is converted to internal

thermal energy via collisions between charge carriers and atoms. ● Semiconductors are materials that have few conduction electrons but can become conductors when they are doped with other atoms that contribute charge carriers. ● Superconductors are materials that lose all electrical resis-

tance. Most such materials require very low temperatures, but some become superconducting at temperatures as high as room temperature.

Power in Electric Circuits Figure 26-13 shows a circuit consisting of a battery B that is connected by wires, which we assume have negligible resistance, to an unspecified conducting device. The device might be a resistor, a storage battery (a rechargeable battery), a motor, or some other electrical device. The battery maintains a potential difference of magnitude V across its own terminals and thus (because of the wires) across the terminals of the unspecified device, with a greater potential at terminal a of the device than at terminal b. Because there is an external conducting path between the two terminals of the battery, and because the potential differences set up by the battery are maintained, a steady current i is produced in the circuit, directed from terminal a to terminal b. The amount of charge dq that moves between those terminals in time interval dt is equal to i dt. This charge dq moves through a decrease in potential of magnitude V, and thus its electric potential energy decreases in magnitude by the amount

The battery at the left supplies energy to the conduction electrons that form the current.

dU ! dq V ! i dt V.

i i

i a

+ B –

(26-25)

The principle of conservation of energy tells us that the decrease in electric potential energy from a to b is accompanied by a transfer of energy to some other form. The power P associated with that transfer is the rate of transfer dU/dt, which is given by Eq. 26-25 as

?

P ! iV

b

i

i i

Figure 26-13 A battery B sets up a current i in a circuit containing an unspecified conducting device.

(rate of electrical energy transfer).

(26-26)

Moreover, this power P is also the rate at which energy is transferred from the battery to the unspecified device. If that device is a motor connected to a mechanical load, the energy is transferred as work done on the load. If the device is a storage battery that is being charged, the energy is transferred to stored chemical energy in the storage battery. If the device is a resistor, the energy is transferred to internal thermal energy, tending to increase the resistor’s temperature.

26-5 POWE R, SE M ICON DUCTORS, SU PE RCON DUCTORS

761

The unit of power that follows from Eq. 26-26 is the volt-ampere (V &A). We can write it as

# CJ $ #1 Cs $ ! 1 Js ! 1 W.

1 V&A ! 1

As an electron moves through a resistor at constant drift speed, its average kinetic energy remains constant and its lost electric potential energy appears as thermal energy in the resistor and the surroundings. On a microscopic scale this energy transfer is due to collisions between the electron and the molecules of the resistor, which leads to an increase in the temperature of the resistor lattice. The mechanical energy thus transferred to thermal energy is dissipated (lost) because the transfer cannot be reversed. For a resistor or some other device with resistance R, we can combine Eqs. 26-8 (R ! V/i) and 26-26 to obtain, for the rate of electrical energy dissipation due to a resistance, either P ! i 2R or

P!

V2 R

(resistive dissipation)

(resistive dissipation).

(26-27) (26-28)

Caution: We must be careful to distinguish these two equations from Eq. 26-26: P ! iV applies to electrical energy transfers of all kinds; P ! i2R and P ! V 2/R apply only to the transfer of electric potential energy to thermal energy in a device with resistance.

Checkpoint 5 A potential difference V is connected across a device with resistance R, causing current i through the device. Rank the following variations according to the change in the rate at which electrical energy is converted to thermal energy due to the resistance, greatest change first: (a) V is doubled with R unchanged, (b) i is doubled with R unchanged, (c) R is doubled with V unchanged, (d) R is doubled with i unchanged.

Sample Problem 26.06 Rate of energy dissipation in a wire carrying current You are given a length of uniform heating wire made of a nickel – chromium – iron alloy called Nichrome; it has a resistance R of 72 0. At what rate is energy dissipated in each of the following situations? (1) A potential difference of 120 V is applied across the full length of the wire. (2) The wire is cut in half, and a potential difference of 120 V is applied across the length of each half.

(120 V)2 V2 (Answer) ! ! 200 W. R 72 0 In situation 2, the resistance of each half of the wire is (72 0)/2, or 36 0. Thus, the dissipation rate for each half is

KEY IDEA

and that for the two halves is P ! 2P- ! 800 W.

Current in a resistive material produces a transfer of mechanical energy to thermal energy; the rate of transfer (dissipation) is given by Eqs. 26-26 to 26-28. Calculations: Because we know the potential V and resistance R, we use Eq. 26-28, which yields, for situation 1,

P!

P- !

(120 V)2 ! 400 W, 36 0 (Answer)

This is four times the dissipation rate of the full length of wire. Thus, you might conclude that you could buy a heating coil, cut it in half, and reconnect it to obtain four times the heat output. Why is this unwise? (What would happen to the amount of current in the coil?)

Additional examples, video, and practice available at WileyPLUS

762

CHAPTE R 26 CU R R E NT AN D R ESISTANCE

Semiconductors Semiconducting devices are at the heart of the microelectronic revolution that ushered in the information age. Table 26-2 compares the properties of silicon — a typical semiconductor — and copper — a typical metallic conductor. We see that silicon has many fewer charge carriers, a much higher resistivity, and a temperature coefficient of resistivity that is both large and negative. Thus, although the resistivity of copper increases with increasing temperature, that of pure silicon decreases. Pure silicon has such a high resistivity that it is effectively an insulator and thus not of much direct use in microelectronic circuits. However, its resistivity can be greatly reduced in a controlled way by adding minute amounts of specific “impurity” atoms in a process called doping. Table 26-1 gives typical values of resistivity for silicon before and after doping with two different impurities. We can roughly explain the differences in resistivity (and thus in conductivity) between semiconductors, insulators, and metallic conductors in terms of the energies of their electrons. (We need quantum physics to explain in more detail.) In a metallic conductor such as copper wire, most of the electrons are firmly locked in place within the atoms; much energy would be required to free them so they could move and participate in an electric current. However, there are also some electrons that, roughly speaking, are only loosely held in place and that require only little energy to become free. Thermal energy can supply that energy, as can an electric field applied across the conductor. The field would not only free these loosely held electrons but would also propel them along the wire; thus, the field would drive a current through the conductor. In an insulator, significantly greater energy is required to free electrons so they can move through the material. Thermal energy cannot supply enough energy, and neither can any reasonable electric field applied to the insulator. Thus, no electrons are available to move through the insulator, and hence no current occurs even with an applied electric field. A semiconductor is like an insulator except that the energy required to free some electrons is not quite so great. More important, doping can supply electrons or positive charge carriers that are very loosely held within the material and thus are easy to get moving. Moreover, by controlling the doping of a semiconductor, we can control the density of charge carriers that can participate in a current and thereby can control some of its electrical properties. Most semiconducting devices, such as transistors and junction diodes, are fabricated by the selective doping of different regions of the silicon with impurity atoms of different kinds. Let us now look again at Eq. 26-22 for the resistivity of a conductor: m r! 2 , (26-29) e nt where n is the number of charge carriers per unit volume and t is the mean time between collisions of the charge carriers. The equation also applies to semiconductors. Let’s consider how n and t change as the temperature is increased. In a conductor, n is large but very nearly constant with any change in temperature. The increase of resistivity with temperature for metals (Fig. 26-10) is due to an increase in the collision rate of the charge carriers, which shows up in Eq. 26-29 as a decrease in t, the mean time between collisions. Table 26-2 Some Electrical Properties of Copper and Silicon Property Type of material Charge carrier density, m#3 Resistivity, 0&m Temperature coefficient of resistivity, K#1

Copper

Silicon

Metal 8.49 $ 10 28 1.69 $ 10 #8 %4.3 $ 10 #3

Semiconductor 1 $ 10 16 2.5 $ 10 3 #70 $ 10 #3

R EVI EW & SU M MARY

Superconductors In 1911, Dutch physicist Kamerlingh Onnes discovered that the resistivity of mercury absolutely disappears at temperatures below about 4 K (Fig. 26-14). This phenomenon of superconductivity is of vast potential importance in technology because it means that charge can flow through a superconducting conductor without losing its energy to thermal energy. Currents created in a superconducting ring, for example, have persisted for several years without loss; the electrons making up the current require a force and a source of energy at start-up time but not thereafter. Prior to 1986, the technological development of superconductivity was throttled by the cost of producing the extremely low temperatures required to achieve the effect. In 1986, however, new ceramic materials were discovered that become superconducting at considerably higher (and thus cheaper to produce) temperatures. Practical application of superconducting devices at room temperature may eventually become commonplace. Superconductivity is a phenomenon much different from conductivity. In fact, the best of the normal conductors, such as silver and copper, cannot become superconducting at any temperature, and the new ceramic superconductors are actually good insulators when they are not at low enough temperatures to be in a superconducting state. One explanation for superconductivity is that the electrons that make up the current move in coordinated pairs. One of the electrons in a pair may electrically distort the molecular structure of the superconducting material as it moves through, creating nearby a short-lived concentration of positive charge. The other electron in the pair may then be attracted toward this positive charge. According to the theory, such coordination between electrons would prevent them from colliding with the molecules of the material and thus would eliminate electrical resistance. The theory worked well to explain the pre-1986, lower temperature superconductors, but new theories appear to be needed for the newer, higher temperature superconductors.

Resistance (Ω)

In a semiconductor, n is small but increases very rapidly with temperature as the increased thermal agitation makes more charge carriers available. This causes a decrease of resistivity with increasing temperature, as indicated by the negative temperature coefficient of resistivity for silicon in Table 26-2. The same increase in collision rate that we noted for metals also occurs for semiconductors, but its effect is swamped by the rapid increase in the number of charge carriers.

763

0.16 0.08 0 0

2 4 6 Temperature (K)

Figure 26-14 The resistance of mercury drops to zero at a temperature of about 4 K.

Courtesy Shoji Tonaka/International Superconductivity Technology Center, Tokyo, Japan

A disk-shaped magnet is levitated above a superconducting material that has been cooled by liquid nitrogen. The goldfish is along for the ride.

Review & Summary Current An electric current i in a conductor is defined by

Drift Speed of the Charge Carriers When an electric field :

dq i! . (26-1) dt Here dq is the amount of (positive) charge that passes in time dt through a hypothetical surface that cuts across the conductor. By convention, the direction of electric current is taken as the direction in which positive charge carriers would move. The SI unit of electric current is the ampere (A): 1 A ! 1 C/s.

E is established in a conductor, the charge carriers (assumed posi: tive) acquire a drift speed vd in the direction of E ; the velocity v:d is related to the current density by

Current Density Current (a scalar) is related to current

is defined as

:

density J (a vector) by

i! :

"

:

J ! (ne)v:d,

(26-7)

where ne is the carrier charge density.

Resistance of a Conductor The resistance R of a conductor V (definition of R), (26-8) i where V is the potential difference across the conductor and i is the current.The SI unit of resistance is the ohm (0): 1 0 ! 1 V/A. Similar equations define the resistivity r and conductivity s of a material: R!

:

:

J " dA,

(26-4)

where dA is a vector perpendicular to a surface element of area dA and the integral is taken over any surface cutting across the conduc: tor. J has the same direction as the velocity of the moving charges if they are positive and the opposite direction if they are negative.

r!

E 1 ! s J

(definitions of r and s),

(26-12, 26-10)

764

CHAPTE R 26 CU R R E NT AN D R ESISTANCE

where E is the magnitude of the applied electric field. The SI unit of resistivity is the ohm-meter (0&m). Equation 26-10 corresponds to the vector equation :

:

E ! rJ.

(26-11)

The resistance R of a conducting wire of length L and uniform cross section is L , R!r (26-16) A where A is the cross-sectional area.

possible to derive an expression for the resistivity of a metal: r!

m . e nt

(26-22)

2

Here n is the number of free electrons per unit volume and t is the mean time between the collisions of an electron with the atoms of the metal. We can explain why metals obey Ohm’s law by pointing out that t is essentially independent of the magnitude E of any electric field applied to a metal.

Power The power P, or rate of energy transfer, in an electrical

Change of r with Temperature The resistivity r for most

device across which a potential difference V is maintained is

materials changes with temperature. For many materials, including metals, the relation between r and temperature T is approximated by the equation

P ! iV (rate of electrical energy transfer).

r # r0 ! r0a(T # T0).

(26-17)

Resistive Dissipation If the device is a resistor, we can write Eq. 26-26 as P ! i2R !

Here T0 is a reference temperature, r0 is the resistivity at T0, and a is the temperature coefficient of resistivity for the material.

Ohm’s Law A given device (conductor, resistor, or any other electrical device) obeys Ohm’s law if its resistance R, defined by Eq. 26-8 as V/i, is independent of the applied potential difference V. A given material obeys Ohm’s law if its resistivity, defined by Eq. 26-10, is independent of the magnitude and direction of the ap: plied electric field E.

Resistivity of a Metal By assuming that the conduction electrons in a metal are free to move like the molecules of a gas, it is

(26-26)

V2 R

(resistive dissipation).

(26-27, 26-28)

In a resistor, electric potential energy is converted to internal thermal energy via collisions between charge carriers and atoms.

Semiconductors Semiconductors are materials that have few conduction electrons but can become conductors when they are doped with other atoms that contribute charge carriers.

Superconductors Superconductors are materials that lose all electrical resistance at low temperatures. Some materials are superconducting at surprisingly high temperatures.

Questions 1 Figure 26-15 shows cross sections through three long conductors of the same length and material, with square cross sections of edge lengths as shown. Conductor B fits snugly within conductor A, and conductor C fits snugly within conductor B. Rank the following according to their end-to-end resistances, greatest first: the individual conductors and the combinations of A % B (B inside A), B % C (C inside B), and A % B % C (B inside A inside C). √3 l

√2 l

l

C

B

A

Figure 26-15 Question 1. 2 Figure 26-16 shows cross sections through three wires of identical length and material; the sides are given in millimeters. Rank the wires according to their resistance (measured end to end along each wire’s length), greatest first.

3 Figure 26-17 shows a rectangular 3L solid conductor of edge lengths L, 2L, 2L and 3L. A potential difference V is to be applied uniformly between pairs of opposite faces of the conductor as in L Fig. 26-8b. (The potential difference is Figure 26-17 Question 3. applied between the entire face on one side and the entire face on the other side.) First V is applied between the left–right faces, then between the top–bottom faces, and then between the front–back faces. Rank those pairs, greatest first, according to the following (within the conductor): (a) the magnitude of the electric field, (b) the current density, (c) the current, and (d) the drift speed of the electrons. 4 Figure 26-18 shows plots of the current i through a certain cross section of a wire over four different time periods. Rank the periods according to the net charge that passes through the cross section during the period, greatest first. i a b

4 6 5

4 (a)

(b)

2

Figure 26-16 Question 2.

c

d t

0

3 (c)

Figure 26-18 Question 4.

765

PROB LE M S

5 Figure 26-19 shows four situations in which positive and negative charges move horizontally and gives the rate at which each charge moves. Rank the situations according to the effective current through the regions, greatest first. 7 C/s

3 C/s

2 C/s

+ 4 C/s

(a)

(b)

6 C/s

+ 5 C/s

+

(d)

Figure 26-19 Question 5. 6 In Fig. 26-20, a wire that carries a current consists of three sections with different radii. Rank the sections according to the following quantities, greatest first: (a) current, (b) magnitude of current density, and (c) magnitude of electric field.

2R 0

R0

1.5R 0

A

B

C

Length

Diameter

Potential Difference

1 2 3

L 2L 3L

3d d 2d

V 2V 2V

9 Figure 26-22 gives the drift speed vd vd of conduction electrons in a copper wire versus position x along the wire. The wire consists of three sections x that differ in radius. Rank the three A B C sections according to the following Figure 26-22 Question 9. quantities, greatest first: (a) radius, (b) number of conduction electrons per cubic meter, (c) magnitude of electric field, (d) conductivity.

1 C/s

(c)

Rod

10 Three wires, of the same diameter, are connected in turn between two points maintained at a constant potential difference. Their resistivities and lengths are r and L (wire A), 1.2r and 1.2L (wire B), and 0.9r and L (wire C). Rank the wires according to the rate at which energy is transferred to thermal energy within them, greatest first.

Figure 26-20 Question 6. 7 Figure 26-21 gives the electric potential V(x) versus position x V along a copper wire carrying current. The wire consists of three sections that differ in radius. Rank the three sections according to the magnitude of the (a) electric field and x (b) current density, greatest first. A B C 8 The following table gives the Figure 26-21 Question 7. lengths of three copper rods, their diameters, and the potential differences between their ends. Rank the rods according to (a) the magnitude of the electric field within them, (b) the current density within them, and (c) the drift speed of electrons through them, greatest first.

a

11 Figure 26-23 gives, for three wires of radius R, the current denb sity J(r) versus radius r, as meas- J c ured from the center of a circular cross section through the wire. The r R wires are all made from the same Figure 26-23 Question 11. material. Rank the wires according to the magnitude of the electric field (a) at the center, (b) halfway to the surface, and (c) at the surface, greatest first.

Problems Tutoring problem available (at instructor’s discretion) in WileyPLUS and WebAssign SSM • – •••

Worked-out solution available in Student Solutions Manual

WWW Worked-out solution is at

Number of dots indicates level of problem difficulty

ILW

Interactive solution is at

http://www.wiley.com/college/halliday

Additional information available in The Flying Circus of Physics and at flyingcircusofphysics.com

••2 An isolated conducting sphere has a 10 cm radius. One wire carries a current of 1.000 002 0 A into it. Another wire carries a current of 1.000 000 0 A out of it. How long would it take for the sphere to increase in potential by 1000 V? ••3 A charged belt, 50 cm wide, travels at 30 m/s between a source of charge and a sphere.The belt carries charge into the sphere at a rate corresponding to 100 mA. Compute the surface charge density on the belt. Module 26-2 Current Density •4 The (United States) National Electric Code, which sets maximum safe currents for insulated copper wires of various diameters, is given (in part) in the table. Plot the safe current density as a function of diameter. Which wire gauge has the maximum safe current density? (“Gauge” is a way of identifying wire diameters, and 1 mil ! 10 #3 in.)

Gauge Diameter, mils Safe current, A

4 204 70

6 162 50

8 129 35

10 102 25

12 81 20

14 64 15

16 51 6

18 40 3

•5 SSM WWW A beam contains 2.0 $ 10 8 doubly charged positive ions per cubic centimeter, all of which are moving north with a speed of 1.0 $ 10 5 m/s. What are the (a) magnitude and (b) direc: tion of the current density J ? (c) What additional quantity do you need to calculate the total current i in this ion beam? •6 A certain cylindrical wire carries current. We draw a circle of radius r around its central axis in Fig. 26-24a to determine the current i within the circle. Figure 2624b shows current i as a function of r 2. The vertical scale is set by is ! 4.0 mA, and the

r

(a)

i (mA)

Module 26-1 Electric Current •1 During the 4.0 min a 5.0 A current is set up in a wire, how many (a) coulombs and (b) electrons pass through any cross section across the wire’s width?

is

0

2

2

r (mm ) (b)

Figure 26-24 Problem 6.

r s2

766

CHAPTE R 26 CU R R E NT AN D R ESISTANCE

horizontal scale is set by r 2s ! 4.0 mm2. (a) Is the current density uniform? (b) If so, what is its magnitude? •7 A fuse in an electric circuit is a wire that is designed to melt, and thereby open the circuit, if the current exceeds a predetermined value. Suppose that the material to be used in a fuse melts when the current density rises to 440 A/cm2. What diameter of cylindrical wire should be used to make a fuse that will limit the current to 0.50 A? •8 A small but measurable current of 1.2 $ 10 #10 A exists in a copper wire whose diameter is 2.5 mm. The number of charge carriers per unit volume is 8.49 $ 10 28 m#3. Assuming the current is uniform, calculate the (a) current density and (b) electron drift speed.

•17 A wire of Nichrome (a nickel – chromium – iron alloy commonly used in heating elements) is 1.0 m long and 1.0 mm2 in cross-sectional area. It carries a current of 4.0 A when a 2.0 V potential difference is applied between its ends. Calculate the conductivity s of Nichrome. •18 A wire 4.00 m long and 6.00 mm in diameter has a resistance of 15.0 m0. A potential difference of 23.0 V is applied between the ends. (a) What is the current in the wire? (b) What is the magnitude of the current density? (c) Calculate the resistivity of the wire material. (d) Using Table 26-1, identify the material. •19 SSM What is the resistivity of a wire of 1.0 mm diameter, 2.0 m length, and 50 m0 resistance?

••9 The magnitude J(r) of the current density in a certain cylindrical wire is given as a function of radial distance from the center of the wire’s cross section as J(r) ! Br, where r is in meters, J is in amperes per square meter, and B ! 2.00 $ 10 5 A/m3. This function applies out to the wire’s radius of 2.00 mm. How much current is contained within the width of a thin ring concentric with the wire if the ring has a radial width of 10.0 mm and is at a radial distance of 1.20 mm?

•20 A certain wire has a resistance R. What is the resistance of a second wire, made of the same material, that is half as long and has half the diameter?

••10 The magnitude J of the current density in a certain lab wire with a circular cross section of radius R ! 2.00 mm is given by J ! (3.00 $ 10 8)r 2, with J in amperes per square meter and radial distance r in meters. What is the current through the outer section bounded by r ! 0.900R and r ! R?

••22 Kiting during a storm. The legend that Benjamin Franklin flew a kite as a storm approached is only a legend—he was neither stupid nor suicidal. Suppose a kite string of radius 2.00 mm extends directly upward by 0.800 km and is coated with a 0.500 mm layer of water having resistivity 150 0&m. If the potential difference between the two ends of the string is 160 MV, what is the current through the water layer? The danger is not this current but the chance that the string draws a lightning strike, which can have a current as large as 500 000 A (way beyond just being lethal).

••12 Near Earth, the density of protons in the solar wind (a stream of particles from the Sun) is 8.70 cm#3, and their speed is 470 km/s. (a) Find the current density of these protons. (b) If Earth’s magnetic field did not deflect the protons, what total current would Earth receive? ILW How long does it take electrons to get from a car ••13 battery to the starting motor? Assume the current is 300 A and the electrons travel through a copper wire with cross-sectional area 0.21 cm2 and length 0.85 m. The number of charge carriers per unit volume is 8.49 $ 10 28 m#3.

Module 26-3 Resistance and Resistivity •14 A human being can be electrocuted if a current as small as 50 mA passes near the heart. An electrician working with sweaty hands makes good contact with the two conductors he is holding, one in each hand. If his resistance is 2000 0, what might the fatal voltage be? •15 SSM A coil is formed by winding 250 turns of insulated 16-gauge copper wire (diameter ! 1.3 mm) in a single layer on a cylindrical form of radius 12 cm. What is the resistance of the coil? Neglect the thickness of the insulation. (Use Table 26-1.) •16 Copper and aluminum are being considered for a high-voltage transmission line that must carry a current of 60.0 A. The resistance per unit length is to be 0.150 0/km. The densities of copper and aluminum are 8960 and 2600 kg/m3, respectively. Compute (a) the magnitude J of the current density and (b) the mass per unit length l for a copper cable and (c) J and (d) l for an aluminum cable.

••23 When 115 V is applied across a wire that is 10 m long and has a 0.30 mm radius, the magnitude of the current density is 1.4 $ 10 8 A/m2. Find the resistivity of the wire. ••24 Figure 26-25a gives the magnitude E(x) of the electric fields that have been set up by a battery along a resistive rod of length 9.00 mm (Fig. 26-25b). The vertical scale is set by Es ! 4.00 $ 103 V/m. The rod consists of three sections of the same material but with different radii. (The schematic diagram of Fig. 26-25b does not indicate the different radii.) The radius of section 3 is 2.00 mm. What is the radius of (a) section 1 and (b) section 2? Es E (103 V/m)

••11 What is the current in a wire of radius R ! 3.40 mm if the magnitude of the current density is given by (a) Ja ! J0r/R and (b) Jb ! J0(1 # r/R), in which r is the radial distance and J0 ! 5.50 $ 10 4 A/m2? (c) Which function maximizes the current density near the wire’s surface?

••21 ILW A common flashlight bulb is rated at 0.30 A and 2.9 V (the values of the current and voltage under operating conditions). If the resistance of the tungsten bulb filament at room temperature (20/C) is 1.1 0, what is the temperature of the filament when the bulb is on?

(a)

x=0 V 1 0

2 3

3 6

x = 9 mm 9

(b)

x (mm)

Figure 26-25 Problem 24. ••25 SSM ILW A wire with a resistance of 6.0 0 is drawn out through a die so that its new length is three times its original length. Find the resistance of the longer wire, assuming that the resistivity and density of the material are unchanged. ••26 In Fig. 26-26a, a 9.00 V battery is connected to a resistive strip that consists of three sections with the same cross-sectional areas but different conductivities. Figure 26-26b gives the electric

PROB LE M S

potential V(x) versus position x along the strip. The horizontal scale is set by xs ! 8.00 mm. Section 3 has conductivity 3.00 $ 10 7 (0&m)#1. What is the conductivity of section (a) 1 and (b) 2?

V (V)

x=0 V x = xs

8 6 4

1 2

2 0

(a)

x (mm)

(b)

3

xs

Figure 26-26 Problem 26. ••27 SSM WWW Two conductors are made of the same material and have the same length. Conductor A is a solid wire of diameter 1.0 mm. Conductor B is a hollow tube of outside diameter 2.0 mm and inside diameter 1.0 mm. What is the resistance ratio RA/RB, measured between their ends? Vs V (µ V)

••28 Figure 26-27 gives the electric potential V(x) along a copper wire carrying uniform current, from a point of higher potential Vs ! 12.0 mV at x ! 0 to a point of zero potential at xs ! 3.00 m. The wire has a radius of 2.00 mm. What is the current in the wire?

0 x (m)

xs

••29 A potential difference of Figure 26-27 Problem 28. 3.00 nV is set up across a 2.00 cm length of copper wire that has a radius of 2.00 mm. How much charge drifts through a cross section in 3.00 ms? ••30 If the gauge number of a wire is increased by 6, the diameter is halved; if a gauge number is increased by 1, the diameter decreases by the factor 21/6 (see the table in Problem 4). Knowing this, and knowing that 1000 ft of 10-gauge copper wire has a resistance of approximately 1.00 0, estimate the resistance of 25 ft of 22-gauge copper wire. ••31 An electrical cable consists of 125 strands of fine wire, each having 2.65 m0 resistance. The same potential difference is applied between the ends of all the strands and results in a total current of 0.750 A. (a) What is the current in each strand? (b) What is the applied potential difference? (c) What is the resistance of the cable? ••32 Earth’s lower atmosphere contains negative and positive ions that are produced by radioactive elements in the soil and cosmic rays from space. In a certain region, the atmospheric

+ +

E

+ +

+

+ +

Figure 26-28 Problem 32.

767

electric field strength is 120 V/m and the field is directed vertically down. This field causes singly charged positive ions, at a density of 620 cm#3, to drift downward and singly charged negative ions, at a density of 550 cm#3, to drift upward (Fig. 26-28). The measured conductivity of the air in that region is 2.70 $ 10 #14 (0&m)#1. Calculate (a) the magnitude of the current density and (b) the ion drift speed, assumed to be the same for positive and negative ions. ••33 A block in the shape of a rectangular solid has a crosssectional area of 3.50 cm2 across its width, a front-to-rear length of 15.8 cm, and a resistance of 935 0. The block’s material contains 5.33 $ 10 22 conduction electrons/m3. A potential difference of 35.8 V is maintained between its front and rear faces. (a) What is the current in the block? (b) If the current density is uniform, what is its magnitude? What are (c) the drift velocity of the conduction electrons and (d) the magL nitude of the electric field in the D1 D2 block? (2) •••34 Figure 26-29 shows wire (1) section 1 of diameter D1 ! 4.00R and wire section 2 of diameter D2 ! Figure 26-29 Problem 34. 2.00R, connected by a tapered section. The wire is copper and carries a current. Assume that the current is uniformly distributed across any cross-sectional area through the wire’s width. The electric potential change V along the length L ! 2.00 m shown in section 2 is 10.0 mV. The number of charge carriers per unit volume is 8.49 $ 10 28 m#3. What is the drift speed of the conduction electrons in section 1?

•••35 In Fig. 26-30, current is set up through a truncated right circular cone of resistivity 731 0 & m, left radius a ! 2.00 mm, right radius b ! 2.30 mm, and length L ! 1.94 cm. Assume that the current density is uniform across any cross section taken perpendicular to the length. What is the resistance of the cone? L i

i a

b

Figure 26-30 Problem 35. •••36 Swimming during a storm. Figure 26-31 shows a swimmer at distance D ! 35.0 m from a lightning strike to the water, with current I ! 78 kA. The water has ∆r resistivity 30 0&m, the width of the swimmer along a radial line from the strike is 0.70 m, and his resistD ance across that width is 4.00 k0. Figure 26-31 Problem 36. Assume that the current spreads through the water over a hemisphere centered on the strike point. What is the current through the swimmer? Module 26-4 Ohm’s Law ••37 Show that, according to the free-electron model of electrical conduction in metals and classical physics, the resistivity of metals should be proportional to 1T, where T is the temperature in kelvins. (See Eq. 19-31.)

CHAPTE R 26 CU R R E NT AN D R ESISTANCE

Module 26-5 Power, Semiconductors, Superconductors •38 In Fig. 26-32a, a 20 0 resistor is connected to a battery. Figure 26-32b shows the increase of thermal energy Eth in the resistor as a function of time t. The vertical scale is set by Eth,s ! 2.50 mJ, and the horizontal scale is set by ts ! 4.0 s. What is the electric potential across the battery?

E th (mJ)

E th,s

R

0 t (s)

(a)

ts

(b)

Figure 26-32 Problem 38. •39 A certain brand of hot-dog cooker works by applying a potential difference of 120 V across opposite ends of a hot dog and allowing it to cook by means of the thermal energy produced. The current is 10.0 A, and the energy required to cook one hot dog is 60.0 kJ. If the rate at which energy is supplied is unchanged, how long will it take to cook three hot dogs simultaneously? •40 Thermal energy is produced in a resistor at a rate of 100 W when the current is 3.00 A. What is the resistance? •41 SSM A 120 V potential difference is applied to a space heater whose resistance is 14 0 when hot. (a) At what rate is electrical energy transferred to thermal energy? (b) What is the cost for 5.0 h at US$0.05/kW &h? •42 In Fig. 26-33, a battery of potential difference V ! 12 V is connected to a resistive V R strip of resistance R ! 6.0 0. When an electron moves through the strip from one end to the other, (a) in which direction in the figFigure 26-33 ure does the electron move, (b) how much Problem 42. work is done on the electron by the electric field in the strip, and (c) how much energy is transferred to the thermal energy of the strip by the electron? •43 ILW An unknown resistor is connected between the terminals of a 3.00 V battery. Energy is dissipated in the resistor at the rate of 0.540 W. The same resistor is then connected between the terminals of a 1.50 V battery. At what rate is energy now dissipated? •44 A student kept his 9.0 V, 7.0 W radio turned on at full volume from 9:00 P.M. until 2:00 A.M. How much charge went through it? •45 SSM ILW A 1250 W radiant heater is constructed to operate at 115 V. (a) What is the current in the heater when the unit is operating? (b) What is the resistance of the heating coil? (c) How much thermal energy is produced in 1.0 h? ••46 A copper wire of cross-sectional area 2.00 $ 10 #6 m2 and length 4.00 m has a current of 2.00 A uniformly distributed across that area. (a) What is the magnitude of the electric field along the wire? (b) How much electrical energy is transferred to thermal energy in 30 min? ••47 A heating element is made by maintaining a potential difference of 75.0 V across the length of a Nichrome wire that

has a 2.60 $ 10 #6 m2 cross section. Nichrome has a resistivity of 5.00 $ 10 #7 0&m. (a) If the element dissipates 5000 W, what is its length? (b) If 100 V is used to obtain the same dissipation rate, what should the length be? ••48 Exploding shoes. The rain-soaked shoes of a person may explode if ground current from nearby lightning vaporizes the water. The sudden conversion of water to water vapor causes a dramatic expansion that can rip apart shoes. Water has density 1000 kg/m3 and requires 2256 kJ/kg to be vaporized. If horizontal current lasts 2.00 ms and encounters water with resistivity 150 0 & m, length 12.0 cm, and vertical cross-sectional area 15 $ 10#5 m2, what average current is required to vaporize the water? ••49 A 100 W lightbulb is plugged into a standard 120 V outlet. (a) How much does it cost per 31-day month to leave the light turned on continuously? Assume electrical energy costs US$0.06/kW &h. (b) What is the resistance of the bulb? (c) What is the current in the bulb? ••50 The current through the battery and resistors 1 and 2 in Fig. 26-34a is 2.00 A. Energy is transferred from the current to thermal energy Eth in both resistors. Curves 1 and 2 in Fig. 26-34b give that thermal energy Eth for resistors 1 and 2, respectively, as a function of time t. The vertical scale is set by Eth,s ! 40.0 mJ, and the horizontal scale is set by ts ! 5.00 s. What is the power of the battery? E th,s

R1 R2 (a)

1

E th (mJ)

768

2

0 (b)

t (s)

ts

Figure 26-34 Problem 50. C SSM WWW Wire C and ••51 D wire D are made from different materials and have length LC ! LD ! 1.0 m. The resistivity and diameLC LD ter of wire C are 2.0 $ 10 #6 0&m 1 2 3 and 1.00 mm, and those of wire D Figure 26-35 Problem 51. are 1.0 $ 10 #6 0&m and 0.50 mm. The wires are joined as shown in Fig. 26-35, and a current of 2.0 A is set up in them. What is the electric potential difference between (a) points 1 and 2 and (b) points 2 and 3? What is the rate at which energy is dissipated between (c) points 1 and 2 and (d) points 2 and 3?

••52 The current-density magnitude in a certain circular wire is J ! (2.75 $ 10 10 A/m4)r 2, where r is the radial distance out to the wire’s radius of 3.00 mm. The potential applied to the wire (end to end) is 60.0 V. How much energy is converted to thermal energy in 1.00 h? ••53 A 120 V potential difference is applied to a space heater that dissipates 500 W during operation. (a) What is its resistance during operation? (b) At what rate do electrons flow through any cross section of the heater element?

PROB LE M S

•••54 Figure 26-36a shows a rod (a) x (m) 0 1.0 of resistive material. The resistance dx per unit length of the rod increases (b) in the positive direction of the x axis. At any position x along the rod, the resistance dR of a narrow (differen- (c) V tial) section of width dx is given by dR ! 5.00x dx, where dR is in ohms Figure 26-36 Problem 54. and x is in meters. Figure 26-36b shows such a narrow section. You are to slice off a length of the rod between x ! 0 and some position x ! L and then connect that length to a battery with potential difference V ! 5.0 V (Fig. 26-36c). You want the current in the length to transfer energy to thermal energy at the rate of 200 W. At what position x ! L should you cut the rod? Additional Problems 55 SSM A Nichrome heater dissipates 500 W when the applied potential difference is 110 V and the wire temperature is 8008C. What would be the dissipation rate if the wire temperature were held at 2008C by immersing the wire in a bath of cooling oil? The applied potential difference remains the same, and a for Nichrome at 8008C is 4.0 $ 10 #4 K#1. 56 A potential difference of 1.20 V will be applied to a 33.0 m length of 18-gauge copper wire (diameter ! 0.0400 in.). Calculate (a) the current, (b) the magnitude of the current density, (c) the magnitude of the electric field within the wire, and (d) the rate at which thermal energy will appear in the wire. 57 An 18.0 W device has 9.00 V across it. How much charge goes through the device in 4.00 h? 58 An aluminum rod with a square cross section is 1.3 m long and 5.2 mm on edge. (a) What is the resistance between its ends? (b) What must be the diameter of a cylindrical copper rod of length 1.3 m if its resistance is to be the same as that of the aluminum rod? 59 A cylindrical metal rod is 1.60 m long and 5.50 mm in diameter. The resistance between its two ends (at 20°C) is 1.09 $ 10 #3 0. (a) What is the material? (b) A round disk, 2.00 cm in diameter and 1.00 mm thick, is formed of the same material. What is the resistance between the round faces, assuming that each face is an equipotential surface? 60 The chocolate crumb mystery. This story begins with Problem 60 in Chapter 23 and continues through Chapters 24 and 25. The chocolate crumb powder moved to the silo through a pipe of radius R with uniform speed v and uniform charge density r. (a) Find an expression for the current i (the rate at which charge on the powder moved) through a perpendicular cross section of the pipe. (b) Evaluate i for the conditions at the factory: pipe radius R ! 5.0 cm, speed v ! 2.0 m/s, and charge density r ! 1.1 $ 10 #3 C/m3. If the powder were to flow through a change V in electric potential, its energy could be transferred to a spark at the rate P ! iV. (c) Could there be such a transfer within the pipe due to the radial potential difference discussed in Problem 70 of Chapter 24? As the powder flowed from the pipe into the silo, the electric potential of the powder changed. The magnitude of that change was at least equal to the radial potential difference within the pipe (as evaluated in Problem 70 of Chapter 24). (d) Assuming that value for the potential difference and using the current found in (b) above, find the rate at which energy could have been transferred from the powder to a spark as the powder exited the pipe. (e) If a spark did occur at the exit and lasted for 0.20 s (a reasonable expectation), how much energy would have been transferred to the spark? Recall

769

from Problem 60 in Chapter 23 that a minimum energy transfer of 150 mJ is needed to cause an explosion. (f) Where did the powder explosion most likely occur: in the powder cloud at the unloading bin (Problem 60 of Chapter 25), within the pipe, or at the exit of the pipe into the silo? 61 SSM A steady beam of alpha particles (q ! %2e) traveling with constant kinetic energy 20 MeV carries a current of 0.25 mA. (a) If the beam is directed perpendicular to a flat surface, how many alpha particles strike the surface in 3.0 s? (b) At any instant, how many alpha particles are there in a given 20 cm length of the beam? (c) Through what potential difference is it necessary to accelerate each alpha particle from rest to bring it to an energy of 20 MeV? 62 A resistor with a potential difference of 200 V across it transfers electrical energy to thermal energy at the rate of 3000 W. What is the resistance of the resistor? 63 A 2.0 kW heater element from a dryer has a length of 80 cm. If a 10 cm section is removed, what power is used by the now shortened element at 120 V? 64 A cylindrical resistor of radius 5.0 mm and length 2.0 cm is made of material that has a resistivity of 3.5 $ 10 #5 0&m. What are (a) the magnitude of the current density and (b) the potential difference when the energy dissipation rate in the resistor is 1.0 W? 65 A potential difference V is applied to a wire of cross-sectional area A, length L, and resistivity r. You want to change the applied potential difference and stretch the wire so that the energy dissipation rate is multiplied by 30.0 and the current is multiplied by 4.00. Assuming the wire’s density does not change, what are (a) the ratio of the new length to L and (b) the ratio of the new cross-sectional area to A? 66 The headlights of a moving car require about 10 A from the 12 V alternator, which is driven by the engine. Assume the alternator is 80% efficient (its output electrical power is 80% of its input mechanical power), and calculate the horsepower the engine must supply to run the lights. 67 A 500 W heating unit is designed to operate with an applied potential difference of 115 V. (a) By what percentage will its heat output drop if the applied potential difference drops to 110 V? Assume no change in resistance. (b) If you took the variation of resistance with temperature into account, would the actual drop in heat output be larger or smaller than that calculated in (a)? 68 The copper windings of a motor have a resistance of 50 0 at 20°C when the motor is idle. After the motor has run for several hours, the resistance rises to 58 0. What is the temperature of the windings now? Ignore changes in the dimensions of the windings. (Use Table 26-1.) 69 How much electrical energy is transferred to thermal energy in 2.00 h by an electrical resistance of 400 0 when the potential applied across it is 90.0 V? 70 A caterpillar of length 4.0 cm crawls in the direction of electron drift along a 5.2-mm-diameter bare copper wire that carries a uniform current of 12 A. (a) What is the potential difference between the two ends of the caterpillar? (b) Is its tail positive or negative relative to its head? (c) How much time does the caterpillar take to crawl 1.0 cm if it crawls at the drift speed of the electrons in the wire? (The number of charge carriers per unit volume is 8.49 $ 10 28 m#3.) 71 SSM (a) At what temperature would the resistance of a copper conductor be double its resistance at 20.08C? (Use 20.08C as the reference point in Eq. 26-17; compare your answer with

770

CHAPTE R 26 CU R R E NT AN D R ESISTANCE

Fig. 26-10.) (b) Does this same “doubling temperature” hold for all copper conductors, regardless of shape or size? 72 A steel trolley-car rail has a cross-sectional area of 56.0 cm2. What is the resistance of 10.0 km of rail? The resistivity of the steel is 3.00 $ 10 #7 0&m. 73 A coil of current-carrying Nichrome wire is immersed in a liquid. (Nichrome is a nickel – chromium – iron alloy commonly used in heating elements.) When the potential difference across the coil is 12 V and the current through the coil is 5.2 A, the liquid evaporates at the steady rate of 21 mg/s. Calculate the heat of vaporization of the liquid (see Module 18-4). 74 The current density in a wire is uniform and has magnitude 2.0 $ 10 6 A/m2, the wire’s length is 5.0 m, and the density of conduction electrons is 8.49 $ 10 28 m#3. How long does an electron take (on the average) to travel the length of the wire? 75 A certain x-ray tube operates at a current of 7.00 mA and a potential difference of 80.0 kV. What is its power in watts? 76 A current is established in a gas discharge tube when a sufficiently high potential difference is applied across the two electrodes in the tube. The gas ionizes; electrons move toward the positive terminal and singly charged positive ions toward the negative terminal. (a) What is the current in a hydrogen discharge tube in which 3.1 $ 10 18 electrons and 1.1 $ 10 18 protons move past a crosssectional area of the tube each second? (b) Is the direction of the : current density J toward or away from the negative terminal? 77 In Fig. 26-37, a resistance coil, wired to an external battery, is placed inside a thermally insulated cylinder fitted with a frictionless piston and containing an ideal gas. A current i ! 240 mA flows through the coil, which has a resistance R ! 550 0. At what speed v must the piston, of mass m ! 12 kg, move upward in – order that the temperature of the gas + remains unchanged?

v

m

R

i

78 An insulating belt moves at speed 30 m/s and has a width of 50 cm. It carries Figure 26-37 Problem 77. charge into an experimental device at a rate corresponding to 100 mA. What is the surface charge density on the belt?

79 In a hypothetical fusion research lab, high temperature helium gas is completely ionized and each helium atom is separated into two free electrons and the remaining positively charged nucleus, which is called an alpha particle. An applied electric field causes the alpha particles to drift to the east at 25.0 m/s while the electrons drift to the west at 88.0 m/s. The alpha particle density is 2.80 $ 1015 cm#3. What are (a) the net current density and (b) the current direction? 80 When a metal rod is heated, not only its resistance but also its length and cross-sectional area change. The relation R ! rL/A suggests that all three factors should be taken into account in measuring r at various temperatures. If the temperature changes by 1.0 C°, what percentage changes in (a) L, (b) A, and (c) R occur for a copper conductor? (d) What conclusion do you draw? The coefficient of linear expansion is 1.70 $ 10#5 K#1. 81 A beam of 16 MeV deuterons from a cyclotron strikes a copper block. The beam is equivalent to current of 15 mA. (a) At what rate do deuterons strike the block? (b) At what rate is thermal energy produced in the block? 82 A linear accelerator produces a pulsed beam of electrons. The pulse current is 0.50 A, and the pulse duration is 0.10 ms. (a) How many electrons are accelerated per pulse? (b) What is the average current for a machine operating at 500 pulses/s? If the electrons are accelerated to an energy of 50 MeV, what are the (c) average power and (d) peak power of the accelerator? 83 An electric immersion heater normally takes 100 min to bring cold water in a well-insulated container to a certain temperature, after which a thermostat switches the heater off. One day the line voltage is reduced by 6.00% because of a laboratory overload. How long does heating the water now take? Assume that the resistance of the heating element does not change. 84 A 400 W immersion heater is placed in a pot containing 2.00 L of water at 20/C. (a) How long will the water take to rise to the boiling temperature, assuming that 80% of the available energy is absorbed by the water? (b) How much longer is required to evaporate half of the water? 85 A 30 mF capacitor is connected across a programmed power supply. During the interval from t ! 0 to t ! 3.00 s the output voltage of the supply is given by V(t) ! 6.00 % 4.00t # 2.00t2 volts. At t ! 0.500 s find (a) the charge on the capacitor, (b) the current into the capacitor, and (c) the power output from the power supply.

C

H

A

P

T

E

R

2

7

Circuits 27-1 SINGLE-LOOP CIRCUITS Learning Objectives After reading this module, you should be able to . . .

27.01 Identify the action of an emf source in terms of the work it does. 27.02 For an ideal battery, apply the relationship between the emf, the current, and the power (rate of energy transfer). 27.03 Draw a schematic diagram for a single-loop circuit containing a battery and three resistors. 27.04 Apply the loop rule to write a loop equation that relates the potential differences of the circuit elements around a (complete) loop. 27.05 Apply the resistance rule in crossing through a resistor. 27.06 Apply the emf rule in crossing through an emf. 27.07 Identify that resistors in series have the same current, which is the same value that their equivalent resistor has. 27.08 Calculate the equivalent of series resistors. 27.09 Identify that a potential applied to resistors wired in

series is equal to the sum of the potentials across the individual resistors. 27.10 Calculate the potential difference between any two points in a circuit. 27.11 Distinguish a real battery from an ideal battery and, in a circuit diagram, replace a real battery with an ideal battery and an explicitly shown resistance. 27.12 With a real battery in a circuit, calculate the potential difference between its terminals for current in the direction of the emf and in the opposite direction. 27.13 Identify what is meant by grounding a circuit, and draw a schematic diagram for such a connection. 27.14 Identify that grounding a circuit does not affect the current in a circuit. 27.15 Calculate the dissipation rate of energy in a real battery. 27.16 Calculate the net rate of energy transfer in a real battery for current in the direction of the emf and in the opposite direction.

Key Ideas ● An emf device does work on charges to maintain a potential dif-

ference between its output terminals. If dW is the work the device does to force positive charge dq from the negative to the positive terminal, then the emf (work per unit charge) of the device is dW (definition of #). #! dq ● An ideal emf device is one that lacks any internal resistance. The potential difference between its terminals is equal to the emf. ● A real emf device has internal resistance. The potential

difference between its terminals is equal to the emf only if there is no current through the device. ● The change in potential in traversing a resistance R in the direction of the current is #iR; in the opposite direction it is %iR (resistance rule). ● The change in potential in traversing an ideal emf device in

the direction of the emf arrow is %#; in the opposite direction it is ## (emf rule). ● Conservation of energy leads to the loop rule: Loop Rule. The algebraic sum of the changes in potential encoun-

tered in a complete traversal of any loop of a circuit must be zero. Conservation of charge leads to the junction rule (Chapter 26): Junction Rule. The sum of the currents entering any junction must be equal to the sum of the currents leaving that junction. ● When a real battery of emf # and internal resistance r does work on the charge carriers in a current i through the battery, the rate P of energy transfer to the charge carriers is

P ! iV, where V is the potential across the terminals of the battery. ● The rate Pr at which energy is dissipated as thermal energy in the battery is P ! i 2r. r

● The rate Pemf at which the chemical energy in the battery

changes is

Pemf ! i#.

● When resistances are in series, they have the same current. The equivalent resistance that can replace a series combination of resistances is

Req !

n

' Rj j!1

(n resistances in series).

771

772

CHAPTE R 27 CI RCU ITS

What Is Physics? You are surrounded by electric circuits. You might take pride in the number of electrical devices you own and might even carry a mental list of the devices you wish you owned. Every one of those devices, as well as the electrical grid that powers your home, depends on modern electrical engineering. We cannot easily estimate the current financial worth of electrical engineering and its products, but we can be certain that the financial worth continues to grow yearly as more and more tasks are handled electrically. Radios are now tuned electronically instead of manually. Messages are now sent by email instead of through the postal system. Research journals are now read on a computer instead of in a library building, and research papers are now copied and filed electronically instead of photocopied and tucked into a filing cabinet. Indeed, you may be reading an electronic version of this book. The basic science of electrical engineering is physics. In this chapter we cover the physics of electric circuits that are combinations of resistors and batteries (and, in Module 27-4, capacitors). We restrict our discussion to circuits through which charge flows in one direction, which are called either directcurrent circuits or DC circuits. We begin with the question: How can you get charges to flow?

“Pumping” Charges

Courtesy Southern California Edison Company

The world’s largest battery energy storage plant (dismantled in 1996) connected over 8000 large lead-acid batteries in 8 strings at 1000 V each with a capability of 10 MW of power for 4 hours. Charged up at night, the batteries were then put to use during peak power demands on the electrical system.

If you want to make charge carriers flow through a resistor, you must establish a potential difference between the ends of the device. One way to do this is to connect each end of the resistor to one plate of a charged capacitor. The trouble with this scheme is that the flow of charge acts to discharge the capacitor, quickly bringing the plates to the same potential. When that happens, there is no longer an electric field in the resistor, and thus the flow of charge stops. To produce a steady flow of charge, you need a “charge pump,” a device that — by doing work on the charge carriers — maintains a potential difference between a pair of terminals. We call such a device an emf device, and the device is said to provide an emf #, which means that it does work on charge carriers. An emf device is sometimes called a seat of emf. The term emf comes from the outdated phrase electromotive force, which was adopted before scientists clearly understood the function of an emf device. In Chapter 26, we discussed the motion of charge carriers through a circuit in terms of the electric field set up in the circuit — the field produces forces that move the charge carriers. In this chapter we take a different approach: We discuss the motion of the charge carriers in terms of the required energy — an emf device supplies the energy for the motion via the work it does. A common emf device is the battery, used to power a wide variety of machines from wristwatches to submarines. The emf device that most influences our daily lives, however, is the electric generator, which, by means of electrical connections (wires) from a generating plant, creates a potential difference in our homes and workplaces. The emf devices known as solar cells, long familiar as the wing-like panels on spacecraft, also dot the countryside for domestic applications. Less familiar emf devices are the fuel cells that powered the space shuttles and the thermopiles that provide onboard electrical power for some spacecraft and for remote stations in Antarctica and elsewhere. An emf device does not have to be an instrument — living systems, ranging from electric eels and human beings to plants, have physiological emf devices. Although the devices we have listed differ widely in their modes of operation, they all perform the same basic function — they do work on charge carriers and thus maintain a potential difference between their terminals.

773

27-1 SI NG LE-LOOP CI RCU ITS

Work, Energy, and Emf

a

i

Figure 27-1 shows an emf device (consider it to be a battery) that is part of a simple circuit containing a single resistance R (the symbol for resistance and a resistor is ). The emf device keeps one of its terminals (called the positive terminal and often labeled %) at a higher electric potential than the other terminal (called the negative terminal and labeled #). We can represent the emf of the device with an arrow that points from the negative terminal toward the positive terminal as in Fig. 27-1. A small circle on the tail of the emf arrow distinguishes it from the arrows that indicate current direction. When an emf device is not connected to a circuit, the internal chemistry of the device does not cause any net flow of charge carriers within it. However, when it is connected to a circuit as in Fig. 27-1, its internal chemistry causes a net flow of positive charge carriers from the negative terminal to the positive terminal, in the direction of the emf arrow. This flow is part of the current that is set up around the circuit in that same direction (clockwise in Fig. 27-1). Within the emf device, positive charge carriers move from a region of low electric potential and thus low electric potential energy (at the negative terminal) to a region of higher electric potential and higher electric potential energy (at the positive terminal). This motion is just the opposite of what the electric field between the terminals (which is directed from the positive terminal toward the negative terminal) would cause the charge carriers to do. Thus, there must be some source of energy within the device, enabling it to do work on the charges by forcing them to move as they do. The energy source may be chemical, as in a battery or a fuel cell. It may involve mechanical forces, as in an electric generator. Temperature differences may supply the energy, as in a thermopile; or the Sun may supply it, as in a solar cell. Let us now analyze the circuit of Fig. 27-1 from the point of view of work and energy transfers. In any time interval dt, a charge dq passes through any cross section of this circuit, such as aa-. This same amount of charge must enter the emf device at its low-potential end and leave at its high-potential end. The device must do an amount of work dW on the charge dq to force it to move in this way. We define the emf of the emf device in terms of this work: #!

dW dq

a' +

i

R –

i Figure 27-1 A simple electric circuit, in which a device of emf # does work on the charge carriers and maintains a steady current i in a resistor of resistance R.

A

i

– + A

M

R

i

B

(definition of #).

– +

(27-1)

In words, the emf of an emf device is the work per unit charge that the device does in moving charge from its low-potential terminal to its high-potential terminal. The SI unit for emf is the joule per coulomb; in Chapter 24 we defined that unit as the volt. An ideal emf device is one that lacks any internal resistance to the internal movement of charge from terminal to terminal. The potential difference between the terminals of an ideal emf device is equal to the emf of the device. For example, an ideal battery with an emf of 12.0 V always has a potential difference of 12.0 V between its terminals. A real emf device, such as any real battery, has internal resistance to the internal movement of charge. When a real emf device is not connected to a circuit, and thus does not have current through it, the potential difference between its terminals is equal to its emf. However, when that device has current through it, the potential difference between its terminals differs from its emf. We shall discuss such real batteries near the end of this module. When an emf device is connected to a circuit, the device transfers energy to the charge carriers passing through it. This energy can then be transferred from the charge carriers to other devices in the circuit, for example, to light a bulb. Figure 27-2a shows a circuit containing two ideal rechargeable (storage) batteries A and B, a resistance R, and an electric motor M that can lift an object by using

i

B

m (a) Work done by motor on mass m Thermal energy produced by resistance R

Chemical energy lost by B

Chemical energy stored in A (b)

Figure 27-2 (a) In the circuit, #B ( #A; so battery B determines the direction of the current. (b) The energy transfers in the circuit.

774

CHAPTE R 27 CI RCU ITS

energy it obtains from charge carriers in the circuit. Note that the batteries are connected so that they tend to send charges around the circuit in opposite directions. The actual direction of the current in the circuit is determined by the battery with the larger emf, which happens to be battery B, so the chemical energy within battery B is decreasing as energy is transferred to the charge carriers passing through it. However, the chemical energy within battery A is increasing because the current in it is directed from the positive terminal to the negative terminal. Thus, battery B is charging battery A. Battery B is also providing energy to motor M and energy that is being dissipated by resistance R. Figure 27-2b shows all three energy transfers from battery B; each decreases that battery’s chemical energy.

Calculating the Current in a Single-Loop Circuit We discuss here two equivalent ways to calculate the current in the simple singleloop circuit of Fig. 27-3; one method is based on energy conservation considerations, and the other on the concept of potential. The circuit consists of an ideal battery B with emf #, a resistor of resistance R, and two connecting wires. (Unless otherwise indicated, we assume that wires in circuits have negligible resistance. Their function, then, is merely to provide pathways along which charge carriers can move.)

Energy Method Equation 26-27 (P ! i 2R) tells us that in a time interval dt an amount of energy given by i2R dt will appear in the resistor of Fig. 27-3 as thermal energy. As noted in Module 26-5, this energy is said to be dissipated. (Because we assume the wires to have negligible resistance, no thermal energy will appear in them.) During the same interval, a charge dq ! i dt will have moved through battery B, and the work that the battery will have done on this charge, according to Eq. 27-1, is dW ! # dq ! #i dt. From the principle of conservation of energy, the work done by the (ideal) battery must equal the thermal energy that appears in the resistor: #i dt ! i2R dt. This gives us # ! iR.

The battery drives current through the resistor, from high potential to low potential. i + –

B a

Higher potential R

i Lower potential

i Figure 27-3 A single-loop circuit in which a resistance R is connected across an ideal battery B with emf #. The resulting current i is the same throughout the circuit.

The emf # is the energy per unit charge transferred to the moving charges by the battery. The quantity iR is the energy per unit charge transferred from the moving charges to thermal energy within the resistor. Therefore, this equation means that the energy per unit charge transferred to the moving charges is equal to the energy per unit charge transferred from them. Solving for i, we find i!

# . R

(27-2)

Potential Method Suppose we start at any point in the circuit of Fig. 27-3 and mentally proceed around the circuit in either direction, adding algebraically the potential differences that we encounter. Then when we return to our starting point, we must also have returned to our starting potential. Before actually doing so, we shall formalize this idea in a statement that holds not only for single-loop circuits such as that of Fig. 27-3 but also for any complete loop in a multiloop circuit, as we shall discuss in Module 27-2:

27-1 SI NG LE-LOOP CI RCU ITS

LOOP RULE: The algebraic sum of the changes in potential encountered in a complete traversal of any loop of a circuit must be zero.

This is often referred to as Kirchhoff’s loop rule (or Kirchhoff’s voltage law), after German physicist Gustav Robert Kirchhoff. This rule is equivalent to saying that each point on a mountain has only one elevation above sea level. If you start from any point and return to it after walking around the mountain, the algebraic sum of the changes in elevation that you encounter must be zero. In Fig. 27-3, let us start at point a, whose potential is Va, and mentally walk clockwise around the circuit until we are back at a, keeping track of potential changes as we move. Our starting point is at the low-potential terminal of the battery. Because the battery is ideal, the potential difference between its terminals is equal to #. When we pass through the battery to the high-potential terminal, the change in potential is %#. As we walk along the top wire to the top end of the resistor, there is no potential change because the wire has negligible resistance; it is at the same potential as the high-potential terminal of the battery. So too is the top end of the resistor. When we pass through the resistor, however, the potential changes according to Eq. 26-8 (which we can rewrite as V ! iR). Moreover, the potential must decrease because we are moving from the higher potential side of the resistor. Thus, the change in potential is #iR. We return to point a by moving along the bottom wire. Because this wire also has negligible resistance, we again find no potential change. Back at point a, the potential is again Va. Because we traversed a complete loop, our initial potential, as modified for potential changes along the way, must be equal to our final potential; that is, Va % # # iR ! Va. The value of Va cancels from this equation, which becomes # # iR ! 0. Solving this equation for i gives us the same result, i ! #/R, as the energy method (Eq. 27-2). If we apply the loop rule to a complete counterclockwise walk around the circuit, the rule gives us ## % iR ! 0 and we again find that i ! #/R. Thus, you may mentally circle a loop in either direction to apply the loop rule. To prepare for circuits more complex than that of Fig. 27-3, let us set down two rules for finding potential differences as we move around a loop: RESISTANCE RULE: For a move through a resistance in the direction of the current, the change in potential is #iR; in the opposite direction it is %iR.

EMF RULE: For a move through an ideal emf device in the direction of the emf arrow, the change in potential is %#; in the opposite direction it is ##.

Checkpoint 1

i

The figure shows the current i in a single-loop circuit with a battery B and a resistance R (and wires of negligible resistance). (a) Should the emf arrow at B be a b c R B drawn pointing leftward or rightward? At points a, b, and c, rank (b) the magnitude of the current, (c) the electric potential, and (d) the electric potential energy of the charge carriers, greatest first.

775

776

CHAPTE R 27 CI RCU ITS

Other Single-Loop Circuits Next we extend the simple circuit of Fig. 27-3 in two ways.

Internal Resistance Figure 27-4a shows a real battery, with internal resistance r, wired to an external resistor of resistance R. The internal resistance of the battery is the electrical resistance of the conducting materials of the battery and thus is an unremovable feature of the battery. In Fig. 27-4a, however, the battery is drawn as if it could be separated into an ideal battery with emf # and a resistor of resistance r. The order in which the symbols for these separated parts are drawn does not matter. i

r

i a

R

i

Potential (V)

a

b +



Real battery

b r

a

R Vb

ir

iR

Va Emf device

i (a )

i

Va

Resistor (b )

Figure 27-4 (a) A single-loop circuit containing a real battery having internal resistance r and emf #. (b) The same circuit, now spread out in a line. The potentials encountered in traversing the circuit clockwise from a are also shown. The potential Va is arbitrarily assigned a value of zero, and other potentials in the circuit are graphed relative to Va. i

If we apply the loop rule clockwise beginning at point a, the changes in potential give us

b R1 + –

# # ir # iR ! 0. i

R2

Solving for the current, we find

R3

i!

a i (a )

Series resistors and their equivalent have the same current (“ser-i”).

b

+ –

R eq

i

a (b )

Figure 27-5 (a) Three resistors are connected in series between points a and b. (b) An equivalent circuit, with the three resistors replaced with their equivalent resistance Req.

(27-3)

# . R%r

(27-4)

Note that this equation reduces to Eq. 27-2 if the battery is ideal — that is, if r ! 0. Figure 27-4b shows graphically the changes in electric potential around the circuit. (To better link Fig. 27-4b with the closed circuit in Fig. 27-4a, imagine curling the graph into a cylinder with point a at the left overlapping point a at the right.) Note how traversing the circuit is like walking around a (potential) mountain back to your starting point — you return to the starting elevation. In this book, when a battery is not described as real or if no internal resistance is indicated, you can generally assume that it is ideal — but, of course, in the real world batteries are always real and have internal resistance.

Resistances in Series Figure 27-5a shows three resistances connected in series to an ideal battery with emf #. This description has little to do with how the resistances are drawn. Rather, “in series” means that the resistances are wired one after another and that a potential difference V is applied across the two ends of the series. In Fig. 27-5a, the resistances are connected one after another between a and b, and a potential difference is maintained across a and b by the battery. The potential differences that then exist across the resistances in the series produce identical currents i in them. In general,

27-1 SI NG LE-LOOP CI RCU ITS

777

When a potential difference V is applied across resistances connected in series, the resistances have identical currents i. The sum of the potential differences across the resistances is equal to the applied potential difference V.

Note that charge moving through the series resistances can move along only a single route. If there are additional routes, so that the currents in different resistances are different, the resistances are not connected in series. Resistances connected in series can be replaced with an equivalent resistance Req that has the same current i and the same total potential difference V as the actual resistances.

You might remember that Req and all the actual series resistances have the same current i with the nonsense word “ser-i.” Figure 27-5b shows the equivalent resistance Req that can replace the three resistances of Fig. 27-5a. To derive an expression for Req in Fig. 27-5b, we apply the loop rule to both circuits. For Fig. 27-5a, starting at a and going clockwise around the circuit, we find # # iR1 # iR2 # iR3 ! 0, or

i!

# . R1 % R2 % R3

(27-5)

For Fig. 27-5b, with the three resistances replaced with a single equivalent resistance Req, we find # # iReq ! 0, or

i!

# . Req

(27-6)

Comparison of Eqs. 27-5 and 27-6 shows that Req ! R1 % R2 % R3. The extension to n resistances is straightforward and is Req !

n

' Rj j!1

(n resistances in series).

(27-7)

Note that when resistances are in series, their equivalent resistance is greater than any of the individual resistances.

Checkpoint 2 In Fig. 27-5a, if R1 ( R2 ( R3, rank the three resistances according to (a) the current through them and (b) the potential difference across them, greatest first.

The internal resistance reduces the potential difference between the terminals.

Potential Difference Between Two Points We often want to find the potential difference between two points in a circuit. For example, in Fig. 27-6, what is the potential difference Vb # Va between points a and b? To find out, let’s start at point a (at potential Va) and move through the battery to point b (at potential Vb) while keeping track of the potential changes we encounter. When we pass through the battery’s emf, the potential increases by #. When we pass through the battery’s internal resistance r, we move in the direction of the current and thus the potential decreases by ir. We are then at the

b +

i r = 2.0 Ω

R = 4.0 Ω

= 12 V a –

i

Figure 27-6 Points a and b, which are at the terminals of a real battery, differ in potential.

778

CHAPTE R 27 CI RCU ITS

potential of point b and we have Va % # # ir ! Vb, or

Vb # Va ! # # ir.

(27-8)

To evaluate this expression, we need the current i. Note that the circuit is the same as in Fig. 27-4a, for which Eq. 27-4 gives the current as i!

# . R%r

(27-9)

Substituting this equation into Eq. 27-8 gives us Vb # Va ! # #

# r R%r

# R. R%r Now substituting the data given in Fig. 27-6, we have !

Vb # Va !

12 V 4.0 0 ! 8.0 V. 4.0 0 % 2.0 0

(27-10)

(27-11)

Suppose, instead, we move from a to b counterclockwise, passing through resistor R rather than through the battery. Because we move opposite the current, the potential increases by iR. Thus, Va % iR ! Vb or

Vb # Va ! iR.

(27-12)

Substituting for i from Eq. 27-9, we again find Eq. 27-10. Hence, substitution of the data in Fig. 27-6 yields the same result, Vb # Va ! 8.0 V. In general, To find the potential between any two points in a circuit, start at one point and traverse the circuit to the other point, following any path, and add algebraically the changes in potential you encounter.

Potential Difference Across a Real Battery In Fig. 27-6, points a and b are located at the terminals of the battery. Thus, the potential difference Vb # Va is the terminal-to-terminal potential difference V across the battery. From Eq. 27-8, we see that V ! # # ir.

(27-13)

If the internal resistance r of the battery in Fig. 27-6 were zero, Eq. 27-13 tells us that V would be equal to the emf # of the battery — namely, 12 V. However, because r ! 2.0 0, Eq. 27-13 tells us that V is less than #. From Eq. 27-11, we know that V is only 8.0 V. Note that the result depends on the value of the current through the battery. If the same battery were in a different circuit and had a different current through it, V would have some other value.

Grounding a Circuit Figure 27-7a shows the same circuit as Fig. 27-6 except that here point a is directly connected to ground, as indicated by the common symbol . Grounding a circuit usually means connecting the circuit to a conducting path to Earth’s surface (actually to the electrically conducting moist dirt and rock below ground). Here, such a connection means only that the potential is defined to be zero at the grounding point in the circuit. Thus in Fig. 27-7a, the potential at a is defined to be Va ! 0. Equation 27-11 then tells us that the potential at b is Vb ! 8.0 V.

27-1 SI NG LE-LOOP CI RCU ITS

b +

i R = 4.0 Ω r = 2.0 Ω

= 12 V a –

i

b +

r = 2.0 Ω

i

R = 4.0 Ω

= 12 V a –

Ground is taken to be zero potential.

(a)

i (b)

Figure 27-7 (a) Point a is directly connected to ground. (b) Point b is directly connected to ground.

Figure 27-7b is the same circuit except that point b is now directly connected to ground. Thus, the potential there is defined to be Vb ! 0. Equation 27-11 now tells us that the potential at a is Va ! #8.0 V.

Power, Potential, and Emf When a battery or some other type of emf device does work on the charge carriers to establish a current i, the device transfers energy from its source of energy (such as the chemical source in a battery) to the charge carriers. Because a real emf device has an internal resistance r, it also transfers energy to internal thermal energy via resistive dissipation (Module 26-5). Let us relate these transfers. The net rate P of energy transfer from the emf device to the charge carriers is given by Eq. 26-26: P ! iV, (27-14) where V is the potential across the terminals of the emf device. From Eq. 27-13, we can substitute V ! # # ir into Eq. 27-14 to find P ! i(# # ir) ! i# # i 2r.

(27-15)

From Eq. 26-27, we recognize the term i2r in Eq. 27-15 as the rate Pr of energy transfer to thermal energy within the emf device: Pr ! i 2r

(internal dissipation rate).

(27-16)

Then the term i# in Eq. 27-15 must be the rate Pemf at which the emf device transfers energy both to the charge carriers and to internal thermal energy. Thus, Pemf ! i#

(power of emf device).

(27-17)

If a battery is being recharged, with a “wrong way” current through it, the energy transfer is then from the charge carriers to the battery — both to the battery’s chemical energy and to the energy dissipated in the internal resistance r. The rate of change of the chemical energy is given by Eq. 27-17, the rate of dissipation is given by Eq. 27-16, and the rate at which the carriers supply energy is given by Eq. 27-14.

Checkpoint 3 A battery has an emf of 12 V and an internal resistance of 2 0. Is the terminal-toterminal potential difference greater than, less than, or equal to 12 V if the current in the battery is (a) from the negative to the positive terminal, (b) from the positive to the negative terminal, and (c) zero?

779

780

CHAPTE R 27 CI RCU ITS

Sample Problem 27.01 Single-loop circuit with two real batteries The emfs and resistances in the circuit of Fig. 27-8a have the following values: #1 ! 4.4 V, r1 ! 2.3 0,

#2 ! 2.1 V,

r2 ! 1.8 0,

R ! 5.5 0.

(a) What is the current i in the circuit?

two batteries. Because #1 is greater than #2, battery 1 controls the direction of i, so the direction is clockwise. Let us then apply the loop rule by going counterclockwise — against the current — and starting at point a. (These decisions about where to start and which way you go are arbitrary but, once made, you must be consistent with decisions about the plus and minus signs.) We find

KEY IDEA

##1 % ir1 % iR % ir2 % #2 ! 0.

We can get an expression involving the current i in this single-loop circuit by applying the loop rule, in which we sum the potential changes around the full loop. Calculations: Although knowing the direction of i is not necessary, we can easily determine it from the emfs of the a i 1

Battery 1

i!

2

r1 b

Battery 2

r2

i R

Check that this equation also results if we apply the loop rule clockwise or start at some point other than a. Also, take the time to compare this equation term by term with Fig. 27-8b, which shows the potential changes graphically (with the potential at point a arbitrarily taken to be zero). Solving the above loop equation for the current i, we obtain 4.4 V # 2.1 V #1 # #2 ! R % r1 % r2 5.5 0 % 2.3 0 % 1.8 0

! 0.2396 A % 240 mA.

c

(Answer)

(b) What is the potential difference between the terminals of battery 1 in Fig. 27-8a?

(a)

KEY IDEA i a

0

r1

1

b

R

c

Va

Potential (V)

a Va

–1 –2

2

r2

2 1

ir2

iR

Vb ir1

–4

Calculations: Let us start at point b (effectively the negative terminal of battery 1) and travel clockwise through battery 1 to point a (effectively the positive terminal), keeping track of potential changes. We find that

Vc

= 4.4 V

–3

= 2.1 V

We need to sum the potential differences between points a and b.

Vb # ir1 % #1 ! Va, which gives us Va # Vb ! #ir1 % # 1

–5

! #(0.2396 A)(2.3 0) % 4.4 V Battery 1

Resistor

Battery 2

(b)

Figure 27-8 (a) A single-loop circuit containing two real batteries and a resistor. The batteries oppose each other; that is, they tend to send current in opposite directions through the resistor. (b) A graph of the potentials, counterclockwise from point a, with the potential at a arbitrarily taken to be zero. (To better link the circuit with the graph, mentally cut the circuit at a and then unfold the left side of the circuit toward the left and the right side of the circuit toward the right.)

! %3.84 V % 3.8 V,

(Answer)

which is less than the emf of the battery. You can verify this result by starting at point b in Fig. 27-8a and traversing the circuit counterclockwise to point a. We learn two points here. (1) The potential difference between two points in a circuit is independent of the path we choose to go from one to the other. (2) When the current in the battery is in the “proper” direction, the terminal-to-terminal potential difference is low, that is, lower than the stated emf for the battery that you might find printed on the battery.

Additional examples, video, and practice available at WileyPLUS

781

27-2 M U LTI LOOP CI RCU ITS

27-2 MULTILOOP CIRCUITS Learning Objectives After reading this module, you should be able to . . .

27.17 Apply the junction rule. 27.18 Draw a schematic diagram for a battery and three parallel resistors and distinguish it from a diagram with a battery and three series resistors. 27.19 Identify that resistors in parallel have the same potential difference, which is the same value that their equivalent resistor has. 27.20 Calculate the resistance of the equivalent resistor of several resistors in parallel. 27.21 Identify that the total current through parallel resistors is the sum of the currents through the individual resistors. 27.22 For a circuit with a battery and some resistors in parallel and some in series, simplify the circuit in steps by finding

equivalent resistors, until the current through the battery can be determined, and then reverse the steps to find the currents and potential differences of the individual resistors. 27.23 If a circuit cannot be simplified by using equivalent resistors, identify the several loops in the circuit, choose names and directions for the currents in the branches, set up loop equations for the various loops, and solve these simultaneous equations for the unknown currents. 27.24 In a circuit with identical real batteries in series, replace them with a single ideal battery and a single resistor. 27.25 In a circuit with identical real batteries in parallel, replace them with a single ideal battery and a single resistor.

Key Idea ● When resistances are in parallel, they have the same potential difference. The equivalent resistance that can replace a parallel combination of resistances is given by n 1 1 ! ' Req j!1 Rj

(n resistances in parallel).

Multiloop Circuits Figure 27-9 shows a circuit containing more than one loop. For simplicity, we assume the batteries are ideal. There are two junctions in this circuit, at b and d, and there are three branches connecting these junctions. The branches are the left branch (bad), the right branch (bcd), and the central branch (bd). What are the currents in the three branches? We arbitrarily label the currents, using a different subscript for each branch. Current i1 has the same value everywhere in branch bad, i2 has the same value everywhere in branch bcd, and i3 is the current through branch bd. The directions of the currents are assumed arbitrarily. Consider junction d for a moment: Charge comes into that junction via incoming currents i1 and i3, and it leaves via outgoing current i2. Because there is no variation in the charge at the junction, the total incoming current must equal the total outgoing current: i1 % i3 ! i2.

The current into the junction must equal the current out (charge is conserved).

(27-18)

i1

JUNCTION RULE: The sum of the currents entering any junction must be equal to the sum of the currents leaving that junction.

This rule is often called Kirchhoff’s junction rule (or Kirchhoff’s current law). It is simply a statement of the conservation of charge for a steady flow of charge — there is neither a buildup nor a depletion of charge at a junction. Thus, our basic tools for solving complex circuits are the loop rule (based on the conservation of energy) and the junction rule (based on the conservation of charge).

1

a

You can easily check that applying this condition to junction b leads to exactly the same equation. Equation 27-18 thus suggests a general principle:

b

+ –

R1

2

– +

i3

R3

R2

c

i2

d

Figure 27-9 A multiloop circuit consisting of three branches: left-hand branch bad, righthand branch bcd, and central branch bd. The circuit also consists of three loops: lefthand loop badb, right-hand loop bcdb, and big loop badcb.

782

CHAPTE R 27 CI RCU ITS

Equation 27-18 is a single equation involving three unknowns. To solve the circuit completely (that is, to find all three currents), we need two more equations involving those same unknowns. We obtain them by applying the loop rule twice. In the circuit of Fig. 27-9, we have three loops from which to choose: the left-hand loop (badb), the right-hand loop (bcdb), and the big loop (badcb). Which two loops we choose does not matter — let’s choose the left-hand loop and the righthand loop. If we traverse the left-hand loop in a counterclockwise direction from point b, the loop rule gives us #1 # i1R1 % i3R3 ! 0.

(27-19)

If we traverse the right-hand loop in a counterclockwise direction from point b, the loop rule gives us #i3 R3 # i2R2 # #2 ! 0.

(27-20)

We now have three equations (Eqs. 27-18, 27-19, and 27-20) in the three unknown currents, and they can be solved by a variety of techniques. If we had applied the loop rule to the big loop, we would have obtained (moving counterclockwise from b) the equation #1 # i1R1 # i2R2 # #2 ! 0. However, this is merely the sum of Eqs. 27-19 and 27-20.

Resistances in Parallel Figure 27-10a shows three resistances connected in parallel to an ideal battery of emf #. The term “in parallel” means that the resistances are directly wired together on one side and directly wired together on the other side, and that a potential difference V is applied across the pair of connected sides. Thus, all three resistances have the same potential difference V across them, producing a current through each. In general, When a potential difference V is applied across resistances connected in parallel, the resistances all have that same potential difference V.

In Fig. 27-10a, the applied potential difference V is maintained by the battery. In Fig. 27-10b, the three parallel resistances have been replaced with an equivalent resistance Req.

Parallel resistors and their equivalent have the same potential difference (“par-V”). i

a

+ –

R1

i

b

i2 + i3

i1 R 2

i2 + i3 (a )

i

i2 R 3

i3

a

+ –

i

R eq

i

b (b )

Figure 27-10 (a) Three resistors connected in parallel across points a and b. (b) An equivalent circuit, with the three resistors replaced with their equivalent resistance Req.

27-2 M U LTI LOOP CI RCU ITS

783

Resistances connected in parallel can be replaced with an equivalent resistance Req that has the same potential difference V and the same total current i as the actual resistances.

You might remember that Req and all the actual parallel resistances have the same potential difference V with the nonsense word “par-V.” To derive an expression for Req in Fig. 27-10b, we first write the current in each actual resistance in Fig. 27-10a as i1 !

V , R1

V , R2

i2 !

and i3 !

V , R3

where V is the potential difference between a and b. If we apply the junction rule at point a in Fig. 27-10a and then substitute these values, we find i ! i1 % i2 % i3 ! V

# R1

%

1

$

1 1 % . R2 R3

(27-21)

If we replaced the parallel combination with the equivalent resistance Req (Fig. 27-10b), we would have V . (27-22) i! Req Comparing Eqs. 27-21 and 27-22 leads to 1 1 1 1 ! % % . Req R1 R2 R3

(27-23)

Extending this result to the case of n resistances, we have n 1 1 ! ' Req j!1 Rj

(n resistances in parallel).

(27-24)

For the case of two resistances, the equivalent resistance is their product divided by their sum; that is, Req !

R1R2 . R1 % R2

(27-25)

Note that when two or more resistances are connected in parallel, the equivalent resistance is smaller than any of the combining resistances.Table 27-1 summarizes the equivalence relations for resistors and capacitors in series and in parallel. Table 27-1 Series and Parallel Resistors and Capacitors Series

Parallel Resistors

Req !

n

' Rj

Eq. 27-7

j!1

Same current through all resistors

n 1 1 ! ' Eq. 27-24 Req j!1 Rj Same potential difference across all resistors

Series

Parallel

Capacitors n n 1 1 Ceq ! ' Cj Eq. 25-19 ! ' Eq. 25-20 Ceq j!1 Cj j!1 Same charge on all Same potential difference capacitors across all capacitors

Checkpoint 4 A battery, with potential V across it, is connected to a combination of two identical resistors and then has current i through it. What are the potential difference across and the current through either resistor if the resistors are (a) in series and (b) in parallel?

784

CHAPTE R 27 CI RCU ITS

Sample Problem 27.02 Resistors in parallel and in series Figure 27-11a shows a multiloop circuit containing one ideal battery and four resistances with the following values: R1 ! 20 0,

R2 ! 20 0,

R3 ! 30 0,

R4 ! 8.0 0.

# ! 12 V,

We can now redraw the circuit as in Fig. 27-11c; note that the current through R23 must be i1 because charge that moves through R1 and R4 must also move through R23. For this simple one-loop circuit, the loop rule (applied clockwise from point a as in Fig. 27-11d) yields

(a) What is the current through the battery?

Substituting the given data, we find

KEY IDEA Noting that the current through the battery must also be the current through R1, we see that we might find the current by applying the loop rule to a loop that includes R1 because the current would be included in the potential difference across R1. Incorrect method: Either the left-hand loop or the big loop should do. Noting that the emf arrow of the battery points upward, so the current the battery supplies is clockwise, we might apply the loop rule to the left-hand loop, clockwise from point a. With i being the current through the battery, we would get %# # iR1 # iR2 # iR4 ! 0

(incorrect).

However, this equation is incorrect because it assumes that R1, R2, and R4 all have the same current i. Resistances R1 and R4 do have the same current, because the current passing through R4 must pass through the battery and then through R1 with no change in value. However, that current splits at junction point b — only part passes through R2, the rest through R3. Dead-end method: To distinguish the several currents in the circuit, we must label them individually as in Fig. 27-11b. Then, circling clockwise from a, we can write the loop rule for the left-hand loop as %# # i1R1 # i2R2 # i1R4 ! 0. Unfortunately, this equation contains two unknowns, i1 and i2; we would need at least one more equation to find them. Successful method: A much easier option is to simplify the circuit of Fig. 27-11b by finding equivalent resistances. Note carefully that R1 and R2 are not in series and thus cannot be replaced with an equivalent resistance. However, R2 and R3 are in parallel, so we can use either Eq. 27-24 or Eq. 27-25 to find their equivalent resistance R23. From the latter, R23 !

%# # i1R1 # i1R23 # i1R4 ! 0.

R2R3 (20 0)(30 0) ! ! 12 0. R2 % R3 50 0

12 V # i1(20 0) # i1(12 0) # i1(8.0 0) ! 0, which gives us i1 !

12 V ! 0.30 A. 40 0

(Answer)

(b) What is the current i2 through R2? KEY IDEAS (1) we must now work backward from the equivalent circuit of Fig. 27-11d, where R23 has replaced R2 and R3. (2) Because R2 and R3 are in parallel, they both have the same potential difference across them as R23. Working backward: We know that the current through R23 is i1 ! 0.30 A. Thus, we can use Eq. 26-8 (R ! V/i) and Fig. 27-11e to find the potential difference V23 across R23. Setting R23 ! 12 0 from (a), we write Eq. 26-8 as V23 ! i1R23 ! (0.30 A)(12 0) ! 3.6 V. The potential difference across R2 is thus also 3.6 V (Fig. 27-11f), so the current i2 in R2 must be, by Eq. 26-8 and Fig. 27-11g, i2 !

V2 3.6 V ! ! 0.18 A. R2 20 0

(Answer)

(c) What is the current i3 through R3? KEY IDEAS We can answer by using either of two techniques: (1) Apply Eq. 26-8 as we just did. (2) Use the junction rule, which tells us that at point b in Fig. 27-11b, the incoming current i1 and the outgoing currents i2 and i3 are related by i1 ! i2 % i3. Calculation: Rearranging this junction-rule result yields the result displayed in Fig. 27-11g: i3 ! i1 # i2 ! 0.30 A # 0.18 A ! 0.12 A.

Additional examples, video, and practice available at WileyPLUS

(Answer)

27-2 M U LTI LOOP CI RCU ITS

A

The equivalent of parallel resistors is smaller. i1

b R1

R1

R3

+ –

b

+ –

R2 R4

i3

i1

R3

R 1 = 20 Ω

c

a

i1 R 23 = 12 Ω R 4 = 8.0 Ω

R4

a

c

a

c

i1 (a )

i1 (b )

(c )

Applying the loop rule yields the current.

Applying V = iR yields the potential difference.

i 1 = 0.30 A

i 1 = 0.30 A

b

R 1 = 20 Ω = 12 V

i 1 = 0.30 A R 23 = 12 Ω

+ –

= 12 V

+ –

c

a

i 1 = 0.30 A

c i 1 = 0.30 A

(d )

(e )

Parallel resistors and their equivalent have the same V (“par-V”). b

R 1 = 20 Ω + –

R 4 = 8.0 Ω

Applying i = V/R yields the current.

i3

i 1 = 0.30 A

R 3 = 30 Ω V 3 = 3.6 V

R 1 = 20 Ω

i2 R 2 = 20 Ω

V 2 = 3.6 V

= 12 V

+ –

b

i 3 = 0.12 A R 3 = 30 Ω V 3 = 3.6 V i 2 = 0.18 A R 2 = 20 Ω

V 2 = 3.6 V R 4 = 8.0 Ω

c i 1 = 0.30 A

i 1 = 0.30 A R 23 = 12 Ω

V 23 = 3.6 V R 4 = 8.0 Ω

a

i 1 = 0.30 A

b

R 1 = 20 Ω

R 4 = 8.0 Ω

= 12 V

b

+ –

i2

R2

(f )

c i 1 = 0.30 A

785

(g)

Figure 27-11 (a) A circuit with an ideal battery. (b) Label the currents. (c) Replacing the parallel resistors with their equivalent. (d)–(g) Working backward to find the currents through the parallel resistors.

786

CHAPTE R 27 CI RCU ITS

Sample Problem 27.03 Many real batteries in series and in parallel in an electric fish Electric fish can generate current with biological emf cells called electroplaques. In the South American eel they are arranged in 140 rows, each row stretching horizontally along the body and each containing 5000 cells, as suggested by Fig. 27-12a. Each electroplaque has an emf # of 0.15 V and an internal resistance r of 0.25 0. The water surrounding the eel completes a circuit between the two ends of the electroplaque array, one end at the head of the animal and the other near the tail.

The total resistance Rrow along a row is the sum of the internal resistances of the 5000 electroplaques: Rrow ! 5000r ! (5000)(0.25 0) ! 1250 0. We can now represent each of the 140 identical rows as having a single emf #row and a single resistance Rrow (Fig. 27-12b). In Fig. 27-12b, the emf between point a and point b on any row is #row ! 750 V. Because the rows are identical and because they are all connected together at the left in Fig. 27-12b, all points b in that figure are at the same electric potential. Thus, we can consider them to be connected so that there is only a single point b. The emf between point a and this single point b is #row ! 750 V, so we can draw the circuit as shown in Fig. 27-12c. Between points b and c in Fig. 27-12c are 140 resistances Rrow ! 1250 0, all in parallel. The equivalent resistance Req of this combination is given by Eq. 27-24 as

(a) If the surrounding water has resistance Rw ! 800 0, how much current can the eel produce in the water? KEY IDEA We can simplify the circuit of Fig. 27-12a by replacing combinations of emfs and internal resistances with equivalent emfs and resistances.

140 1 1 1 ! ' ! 140 , Req R R j!1 j row

Calculations: We first consider a single row. The total emf #row along a row of 5000 electroplaques is the sum of the emfs: #row ! 5000# ! (5000)(0.15 V) ! 750 V.

or

Req !

1250 0 Rrow ! ! 8.93 0. 140 140

First, reduce each row to one emf and one resistance. Electroplaque

+ –

+ –

Points with the same potential can be taken as though connected.

+ –

r 5000 electroplaques per row

750 V row

+ –

+ –

Rrow

+ –

+ –

b

row

140 rows

Rrow

+ –

b

a + –

+ –

+ – Rw

(a )

Replace the parallel resistances with their equivalent.

Rrow

row = 750 V + – b a

+ – (b )

Rrow

Emfs in parallel act as a single emf.

c row

a

c Rrow

Rrow b Rw

i

row

+ – b

Req

c

Rw

Rw (c )

(d )

i

Figure 27-12 (a) A model of the electric circuit of an eel in water. Along each of 140 rows extending from the head to the tail of the eel, there are 5000 electroplaques.The surrounding water has resistance Rw. (b) The emf #row and resistance Rrow of each row. (c) The emf between points a and b is #row. Between points b and c are 140 parallel resistances Rrow. (d) The simplified circuit.

787

27-2 M U LTI LOOP CI RCU ITS

Replacing the parallel combination with Req, we obtain the simplified circuit of Fig. 27-12d. Applying the loop rule to this circuit counterclockwise from point b, we have #row # iRw # iReq ! 0. Solving for i and substituting the known data, we find # row 750 V i! ! Rw % Req 800 0 % 8.93 0 ! 0.927 A % 0.93 A.

(Answer)

If the head or tail of the eel is near a fish, some of this current could pass along a narrow path through the fish, stunning or killing it.

(b) How much current irow travels through each row of Fig. 27-12a? KEY IDEA Because the rows are identical, the current into and out of the eel is evenly divided among them. Calculation: Thus, we write 0.927 A i ! ! 6.6 $ 10 #3 A. (Answer) 140 140 Thus, the current through each row is small, so that the eel need not stun or kill itself when it stuns or kills a fish. irow !

Sample Problem 27.04 Multiloop circuit and simultaneous loop equations i1

Figure 27-13 shows a circuit whose elements have the following values: # 1 ! 3.0 V, # 2 ! 6.0 V, R1 ! 2.0 0, R2 ! 4.0 0. The three batteries are ideal batteries. Find the magnitude and direction of the current in each of the three branches. KEY IDEAS It is not worthwhile to try to simplify this circuit, because no two resistors are in parallel, and the resistors that are in series (those in the right branch or those in the left branch) present no problem. So, our plan is to apply the junction and loop rules. Junction rule: Using arbitrarily chosen directions for the currents as shown in Fig. 27-13, we apply the junction rule at point a by writing i3 ! i1 % i2. (27-26) An application of the junction rule at junction b gives only the same equation, so we next apply the loop rule to any two of the three loops of the circuit. Left-hand loop: We first arbitrarily choose the left-hand loop, arbitrarily start at point b, and arbitrarily traverse the loop in the clockwise direction, obtaining where we have used (i1 % i2) instead of i3 in the middle branch. Substituting the given data and simplifying yield (27-27)

Right-hand loop: For our second application of the loop rule, we arbitrarily choose to traverse the right-hand loop counterclockwise from point b, finding #i2R1 % #2 # i2R1 # (i1 % i2)R2 # #2 ! 0.

Figure 27-13 A multiloop circuit with three ideal batteries and five resistances.

R2 + –

R1 i1

b

R1 2

i2

Combining equations: We now have a system of two equations (Eqs. 27-27 and 27-28) in two unknowns (i1 and i2) to solve either “by hand” (which is easy enough here) or with a “math package.” (One solution technique is Cramer’s rule, given in Appendix E.) We find i1 ! #0.50 A.

(27-29)

(The minus sign signals that our arbitrary choice of direction for i1 in Fig. 27-13 is wrong, but we must wait to correct it.) Substituting i1 ! #0.50 A into Eq. 27-28 and solving for i2 then give us i2 ! 0.25 A. (Answer) i3 ! i1 % i2 ! #0.50 A % 0.25 A ! #0.25 A. The positive answer we obtained for i2 signals that our choice of direction for that current is correct. However, the negative answers for i1 and i3 indicate that our choices for those currents are wrong.Thus, as a last step here, we correct the answers by reversing the arrows for i1 and i3 in Fig. 27-13 and then writing i1 ! 0.50 A and i3 ! 0.25 A.

Substituting the given data and simplifying yield i1(4.0 0) % i2(8.0 0) ! 0.

1

+ –

+ –

R1

i3

With Eq. 27-26 we then find that

#i1R1 % #1 # i1R1 # (i1 % i2)R2 # #2 ! 0,

i1(8.0 0) % i2(4.0 0) ! #3.0 V.

R1

i2

a

(27-28)

(Answer)

Caution: Always make any such correction as the last step and not before calculating all the currents.

Additional examples, video, and practice available at WileyPLUS

2

788

CHAPTE R 27 CI RCU ITS

27-3 THE AMMETER AND THE VOLTMETER Learning Objective After reading this module, you should be able to . . .

27.26 Explain the use of an ammeter and a voltmeter, includ-

Key Idea ● Here are three measurement instruments used with circuits: An ammeter measures current. A voltmeter measures

R2

r

c a A

voltage (potential differences). A multimeter can be used to measure current, voltage, or resistance.

The Ammeter and the Voltmeter

i + –

ing the resistance required of each in order not to affect the measured quantities.

V

R1 b

i

d

Figure 27-14 A single-loop circuit, showing how to connect an ammeter (A) and a voltmeter (V).

An instrument used to measure currents is called an ammeter. To measure the current in a wire, you usually have to break or cut the wire and insert the ammeter so that the current to be measured passes through the meter. (In Fig. 27-14, ammeter A is set up to measure current i.) It is essential that the resistance RA of the ammeter be very much smaller than other resistances in the circuit. Otherwise, the very presence of the meter will change the current to be measured. A meter used to measure potential differences is called a voltmeter. To find the potential difference between any two points in the circuit, the voltmeter terminals are connected between those points without breaking or cutting the wire. (In Fig. 27-14, voltmeter V is set up to measure the voltage across R1.) It is essential that the resistance RV of a voltmeter be very much larger than the resistance of any circuit element across which the voltmeter is connected. Otherwise, the meter alters the potential difference that is to be measured. Often a single meter is packaged so that, by means of a switch, it can be made to serve as either an ammeter or a voltmeter — and usually also as an ohmmeter, designed to measure the resistance of any element connected between its terminals. Such a versatile unit is called a multimeter.

27-4 RC CIRCUITS Learning Objectives After reading this module, you should be able to . . .

27.27 Draw schematic diagrams of charging and discharging RC circuits. 27.28 Write the loop equation (a differential equation) for a charging RC circuit. 27.29 Write the loop equation (a differential equation) for a discharging RC circuit. 27.30 For a capacitor in a charging or discharging RC circuit, apply the relationship giving the charge as a function of time.

27.31 From the function giving the charge as a function of time in a charging or discharging RC circuit, find the capacitor’s potential difference as a function of time. 27.32 In a charging or discharging RC circuit, find the resistor’s current and potential difference as functions of time. 27.33 Calculate the capacitive time constant t. 27.34 For a charging RC circuit and a discharging RC circuit, determine the capacitor’s charge and potential difference at the start of the process and then a long time later.

Key Ideas ● When an emf # is applied to a resistance R and capacitance C

in series, the charge on the capacitor increases according to q ! C #(1 # e

)

#t/RC

● During the charging, the current is

# $e # R

#t/RC

charge on the capacitor decays according to

(charging a capacitor),

in which C # ! q0 is the equilibrium (final) charge and RC ! t is the capacitive time constant of the circuit. dq ! i! dt

● When a capacitor discharges through a resistance R, the

(charging a capacitor).

q ! q0e#t/RC

(discharging a capacitor).

● During the discharging, the current is

i!

# $

q0 dq !# e #t/RC dt RC

(discharging a capacitor).

789

27-4 RC CI RCU ITS

RC Circuits

a S

In preceding modules we dealt only with circuits in which the currents did not vary with time. Here we begin a discussion of time-varying currents.

b

R

+ –

C

Charging a Capacitor The capacitor of capacitance C in Fig. 27-15 is initially uncharged. To charge it, we close switch S on point a. This completes an RC series circuit consisting of the capacitor, an ideal battery of emf #, and a resistance R. From Module 25-1, we already know that as soon as the circuit is complete, charge begins to flow (current exists) between a capacitor plate and a battery terminal on each side of the capacitor. This current increases the charge q on the plates and the potential difference VC (! q/C) across the capacitor. When that potential difference equals the potential difference across the battery (which here is equal to the emf #), the current is zero. From Eq. 25-1 (q ! CV), the equilibrium (final) charge on the then fully charged capacitor is equal to C #. Here we want to examine the charging process. In particular we want to know how the charge q(t) on the capacitor plates, the potential difference VC(t) across the capacitor, and the current i(t) in the circuit vary with time during the charging process. We begin by applying the loop rule to the circuit, traversing it clockwise from the negative terminal of the battery. We find

Figure 27-15 When switch S is closed on a, the capacitor is charged through the resistor. When the switch is afterward closed on b, the capacitor discharges through the resistor.

q (27-30) ! 0. C The last term on the left side represents the potential difference across the capacitor. The term is negative because the capacitor’s top plate, which is connected to the battery’s positive terminal, is at a higher potential than the lower plate. Thus, there is a drop in potential as we move down through the capacitor. We cannot immediately solve Eq. 27-30 because it contains two variables, i and q. However, those variables are not independent but are related by # # iR #

dq . dt

(27-31) q (µC)

Substituting this for i in Eq. 27-30 and rearranging, we find R

q dq % !# dt C

(charging equation).

(charging a capacitor).

(27-33)

(Here e is the exponential base, 2.718 . . . , and not the elementary charge.) Note that Eq. 27-33 does indeed satisfy our required initial condition, because at t ! 0 the term e#t/RC is unity; so the equation gives q ! 0. Note also that as t goes to infinity (that is, a long time later), the term e#t/RC goes to zero; so the equation gives the proper value for the full (equilibrium) charge on the capacitor — namely, q ! C #. A plot of q(t) for the charging process is given in Fig. 27-16a. The derivative of q(t) is the current i(t) charging the capacitor: i!

dq ! dt

# R# $e

#t/RC

(charging a capacitor).

(27-34)

C

8 4

(27-32)

This differential equation describes the time variation of the charge q on the capacitor in Fig. 27-15. To solve it, we need to find the function q(t) that satisfies this equation and also satisfies the condition that the capacitor be initially uncharged; that is, q ! 0 at t ! 0. We shall soon show that the solution to Eq. 27-32 is q ! C #(1 # e#t/RC )

12

0

i (mA)

i!

The capacitor’s charge grows as the resistor’s current dies out.

2

6

4 6 8 Time (ms) (a)

10

/R

4 2 0

2

4 6 8 Time (ms)

10

(b)

Figure 27-16 (a) A plot of Eq. 27-33, which shows the buildup of charge on the capacitor of Fig. 27-15. (b) A plot of Eq. 27-34, which shows the decline of the charging current in the circuit of Fig. 27-15. The curves are plotted for R ! 2000 0, C ! 1 mF, and # ! 10 V; the small triangles represent successive intervals of one time constant t.

790

CHAPTE R 27 CI RCU ITS

A plot of i(t) for the charging process is given in Fig. 27-16b. Note that the current has the initial value #/R and that it decreases to zero as the capacitor becomes fully charged. A capacitor that is being charged initially acts like ordinary connecting wire relative to the charging current. A long time later, it acts like a broken wire.

By combining Eq. 25-1 (q ! CV ) and Eq. 27-33, we find that the potential difference VC(t) across the capacitor during the charging process is VC !

q ! #(1 # e #t/RC ) C

(charging a capacitor).

(27-35)

This tells us that VC ! 0 at t ! 0 and that VC ! # when the capacitor becomes fully charged as t : ,.

The Time Constant The product RC that appears in Eqs. 27-33, 27-34, and 27-35 has the dimensions of time (both because the argument of an exponential must be dimensionless and because, in fact, 1.0 0 $ 1.0 F ! 1.0 s). The product RC is called the capacitive time constant of the circuit and is represented with the symbol t: t ! RC

(time constant).

(27-36)

From Eq. 27-33, we can now see that at time t ! t (! RC), the charge on the initially uncharged capacitor of Fig. 27-15 has increased from zero to q ! C #(1 # e#1) ! 0.63C #.

(27-37)

In words, during the first time constant t the charge has increased from zero to 63% of its final value C #. In Fig. 27-16, the small triangles along the time axes mark successive intervals of one time constant during the charging of the capacitor. The charging times for RC circuits are often stated in terms of t. For example, a circuit with t ! 1 ms charges quickly while one with t ! 100 s charges much more slowly,

Discharging a Capacitor Assume now that the capacitor of Fig. 27-15 is fully charged to a potential V0 equal to the emf # of the battery. At a new time t ! 0, switch S is thrown from a to b so that the capacitor can discharge through resistance R. How do the charge q(t) on the capacitor and the current i(t) through the discharge loop of capacitor and resistance now vary with time? The differential equation describing q(t) is like Eq. 27-32 except that now, with no battery in the discharge loop, # ! 0. Thus, R

q dq % !0 dt C

(discharging equation).

(27-38)

The solution to this differential equation is q ! q0e#t/RC

(discharging a capacitor),

(27-39)

where q0 (! CV0) is the initial charge on the capacitor. You can verify by substitution that Eq. 27-39 is indeed a solution of Eq. 27-38.

27-4 RC CI RCU ITS

Equation 27-39 tells us that q decreases exponentially with time, at a rate that is set by the capacitive time constant t ! RC. At time t ! t, the capacitor’s charge has been reduced to q0e#1, or about 37% of the initial value. Note that a greater t means a greater discharge time. Differentiating Eq. 27-39 gives us the current i(t): i!

# $

q0 dq !# e #t/RC dt RC

(27-40)

(discharging a capacitor).

This tells us that the current also decreases exponentially with time, at a rate set by t. The initial current i0 is equal to q0 /RC. Note that you can find i0 by simply applying the loop rule to the circuit at t ! 0; just then the capacitor’s initial potential V0 is connected across the resistance R, so the current must be i0 ! V0 /R ! (q0 /C)/R ! q0 /RC. The minus sign in Eq. 27-40 can be ignored; it merely means that the capacitor’s charge q is decreasing.

Derivation of Eq. 27-33 To solve Eq. 27-32, we first rewrite it as q # dq % ! . dt RC R

(27-41)

The general solution to this differential equation is of the form q ! qp % Ke#at,

(27-42)

where qp is a particular solution of the differential equation, K is a constant to be evaluated from the initial conditions, and a ! 1/RC is the coefficient of q in Eq. 27-41. To find qp, we set dq/dt ! 0 in Eq. 27-41 (corresponding to the final condition of no further charging), let q ! qp, and solve, obtaining (27-43)

qp ! C #. To evaluate K, we first substitute this into Eq. 27-42 to get q ! C # % Ke#at. Then substituting the initial conditions q ! 0 and t ! 0 yields 0 ! C # % K,

or K ! #C #. Finally, with the values of qp, a, and K inserted, Eq. 27-42 becomes q ! C # # C #e#t/RC, which, with a slight modification, is Eq. 27-33.

Checkpoint 5 The table gives four sets of values for the circuit elements in Fig. 27-15. Rank the sets according to (a) the initial current (as the switch is closed on a) and (b) the time required for the current to decrease to half its initial value, greatest first.

# (V) R (0) C (mF)

1

2

3

4

12 2 3

12 3 2

10 10 0.5

10 5 2

791

792

CHAPTE R 27 CI RCU ITS

Sample Problem 27.05 Discharging an RC circuit to avoid a fire in a race car pit stop As a car rolls along pavement, electrons move from the pavement first onto the tires and then onto the car body. The car stores this excess charge and the associated electric potential energy as if the car body were one plate of a capacitor and the pavement were the other plate (Fig. 27-17a). When the car stops, it discharges its excess charge and energy through the tires, just as a capacitor can discharge through a resistor. If a conducting object comes within a few centimeters of the car before the car is discharged, the remaining energy can be suddenly transferred to a spark between the car and the object. Suppose the conducting object is a fuel dispenser. The spark will not ignite the fuel and cause a fire if the spark energy is less than the critical value Ufire ! 50 mJ. When the car of Fig. 27-17a stops at time t ! 0, the car – ground potential difference is V0 ! 30 kV. The car – ground capacitance is C ! 500 pF, and the resistance of each tire is Rtire ! 100 G0. How much time does the car take to discharge through the tires to drop below the critical value Ufire? KEY IDEAS (1) At any time t, a capacitor’s stored electric potential energy U is related to its stored charge q according to Eq. 25-21 (U ! q2/2C). (2) While a capacitor is discharging, the charge decreases with time according to Eq. 27-39 (q ! q0e#t/RC). Calculations: We can treat the tires as resistors that are connected to one another at their tops via the car body and at their bottoms via the pavement. Figure 27-17b shows how the four resistors are connected in parallel across the car’s capacitance, and Fig. 27-17c shows their equivalent resistance R. From Eq. 27-24, R is given by 1 1 1 1 1 % % % , ! R Rtire Rtire Rtire Rtire or

R!

100 $ 10 9 0 Rtire ! ! 25 $ 10 9 0. 4 4

(27-44)

#t/RC 2

q (q0e ) ! 2C 2C q20 #2t/RC . ! e 2C

U!

RPM

MDOG

3N

WNFR

ULTRA

6

True Vales

Schtuff

Bomman

PST4 XP3I

THRU

MOTEL

Tire resistance

Effective capacitance

(a)

R tire

R tire

C

R tire

R tire

(b)

C

R

(c)

Figure 27-17 (a) A charged car and the U pavement acts like a capacitor that can discharge through the tires. (b) The 100 GΩ effective circuit of the car – pavement capacitor, with four tire resistances Rtire 10 GΩ connected in parallel. (c) The equivalent Ufire resistance R of the tires. (d) The electric 0.94 9.4 potential energy U in the car – pavement t (s) capacitor decreases during discharge. (d)

or

e #2t/RC !

2U . CV 20

(27-46)

Taking the natural logarithms of both sides, we obtain #

or

#

$

2U 2t ! ln , RC CV 20

t!#

#

$

2U RC . ln 2 CV 20

(27-47)

Substituting the given data, we find that the time the car takes to discharge to the energy level Ufire ! 50 mJ is

When the car stops, it discharges its excess charge and energy through R. We now use our two Key Ideas to analyze the discharge. Substituting Eq. 27-39 into Eq. 25-21 gives 2

DRIVE

t!#

(25 $ 10 9 0)(500 $ 10 #12 F) 2

$ ln

#

2(50 $ 10 #3 J) (500 $ 10 #12 F)(30 $ 10 3 V)2

! 9.4 s. (27-45)

From Eq. 25-1 (q ! CV ), we can relate the initial charge q0 on the car to the given initial potential difference V0: q0 ! CV0. Substituting this equation into Eq. 27-45 brings us to (CV0)2 #2t/RC CV 20 #2t/RC U! ! , e e 2C 2

$ (Answer)

Fire or no fire: This car requires at least 9.4 s before fuel can be brought safely near it. A pit crew cannot wait that long. So the tires include some type of conducting material (such as carbon black) to lower the tire resistance and thus increase the discharge rate. Figure 27-17d shows the stored energy U versus time t for tire resistances of R ! 100 G0 (our value) and R ! 10 G0. Note how much more rapidly a car discharges to level Ufire with the lower R value.

Additional examples, video, and practice available at WileyPLUS

793

QU ESTIONS

Review & Summary Emf An emf device does work on charges to maintain a potential difference between its output terminals. If dW is the work the device does to force positive charge dq from the negative to the positive terminal, then the emf (work per unit charge) of the device is dW (definition of #). (27-1) #! dq The volt is the SI unit of emf as well as of potential difference.An ideal emf device is one that lacks any internal resistance. The potential difference between its terminals is equal to the emf. A real emf device has internal resistance. The potential difference between its terminals is equal to the emf only if there is no current through the device.

where V is the potential across the terminals of the battery. The rate Pr at which energy is dissipated as thermal energy in the battery is Pr ! i 2r.

The rate Pemf at which the chemical energy in the battery changes is

Loop Rule. The algebraic sum of the changes in potential encountered in a complete traversal of any loop of a circuit must be zero. Conservation of charge gives us the junction rule: Junction Rule. The sum of the currents entering any junction must be equal to the sum of the currents leaving that junction.

Single-Loop Circuits The current in a single-loop circuit containing a single resistance R and an emf device with emf # and internal resistance r is # (27-4) , i! R%r which reduces to i ! #/R for an ideal emf device with r ! 0.

Series Resistances When resistances are in series, they have the same current. The equivalent resistance that can replace a series combination of resistances is Req !

(27-14)

P ! iV,

n

' Rj j!1

(27-7)

(n resistances in series).

Parallel Resistances When resistances are in parallel, they have the same potential difference. The equivalent resistance that can replace a parallel combination of resistances is given by n 1 1 ! ' Req j!1 Rj

(27-24)

(n resistances in parallel).

RC Circuits When an emf # is applied to a resistance R and capacitance C in series, as in Fig. 27-15 with the switch at a, the charge on the capacitor increases according to q ! C #(1 # e#t/RC) (charging a capacitor),

(27-33)

in which C # ! q0 is the equilibrium (final) charge and RC ! t is the capacitive time constant of the circuit. During the charging, the current is i!

dq ! dt

# R# $e

#t/RC

(27-34)

(charging a capacitor).

When a capacitor discharges through a resistance R, the charge on the capacitor decays according to (27-39)

q ! q0e#t/RC (discharging a capacitor).

Power When a real battery of emf # and internal resistance r does work on the charge carriers in a current i through the battery, the rate P of energy transfer to the charge carriers is

(27-17)

Pemf ! i#.

Analyzing Circuits The change in potential in traversing a resistance R in the direction of the current is #iR; in the opposite direction it is %iR (resistance rule). The change in potential in traversing an ideal emf device in the direction of the emf arrow is %#; in the opposite direction it is ## (emf rule). Conservation of energy leads to the loop rule:

(27-16)

During the discharging, the current is i!

# $

q0 dq !# e #t/RC dt RC

(discharging a capacitor). (27-40)

Questions 1 (a) In Fig. 27-18a, with R1 ( R2, is the potential difference R3

+ –

R1

R2 – +

R3 (a)

R1

R2

(b) R1 – +

R3 R1 (c)

+ –

R2

R3 (d) R2

Figure 27-18 Questions 1 and 2.

across R2 more than, less than, or equal to that across R1? (b) Is the current through resistor R2 more than, less than, or equal to that through resistor R1? 2 (a) In Fig. 27-18a, are resistors R1 and R3 in series? (b) Are resistors R1 and R2 in parallel? (c) Rank the equivalent resistances of the four circuits shown in Fig. 27-18, greatest first. 3 You are to connect resistors R1 and R2, with R1 ( R2, to a battery, first individually, then in series, and then in parallel. Rank those arrangements according to the amount of current through the battery, greatest first. 4 In Fig. 27-19, a circuit consists of a battery and two uniform resistors, and the section lying along an x axis is divided into five segments of equal lengths. (a) Assume that R1 ! R2 and rank the segments according to the magnitude of the average electric

+



R1 a

b

R2 c

d

x e

Figure 27-19 Question 4.

794

CHAPTE R 27 CI RCU ITS

field in them, greatest first. (b) Now assume that R1 ( R2 and then again rank the segments. (c) What is the direction of the electric field along the x axis? 5 For each circuit in Fig. 27-20, are the resistors connected in series, in parallel, or neither? –

+ –

(a)

C

+ –

+

(b)

(c)

Figure 27-20 Question 5. 6 Res-monster maze. In Fig. 27-21, all the resistors have a resistance of 4.0 0 and all the (ideal) batteries have an emf of 4.0 V. What is the current through resistor R? (If you can find the proper loop through this maze, you can answer the question with a few seconds of mental calculation.)

Figure 27-22 Question 10. 11 Initially, a single resistor R1 is wired to a battery. Then resistor R2 is added in parallel. Are (a) the potential difference across R1 and (b) the current i1 through R1 now more than, less than, or the same as previously? (c) Is the equivalent resistance R12 of R1 and R2 more than, less than, or equal to R1? (d) Is the total current through R1 and R2 together more than, less than, or equal to the current through R1 previously? 12 After the switch in Fig. 27-15 is closed on point a, there is current i through resistance R. Figure 27-23 gives that current for four sets of values of R and capacitance C: (1) R0 and C0, (2) 2R0 and C0, (3) R0 and 2C0, (4) 2R0 and 2C0. Which set goes with which curve? i

R c

d

a b

Figure 27-23 Question 12.

t

Figure 27-21 Question 6. 7 A resistor R1 is wired to a battery, then resistor R2 is added in series. Are (a) the potential difference across R1 and (b) the current i1 through R1 now more than, less than, or the same as previously? (c) Is the equivalent resistance R12 of R1 and R2 more than, less than, or equal to R1?

13 Figure 27-24 shows three sections of circuit that are to be connected in turn to the same battery via a switch as in Fig. 27-15. The resistors are all identical, as are the capacitors. Rank the sections according to (a) the final (equilibrium) charge on the capacitor and (b) the time required for the capacitor to reach 50% of its final charge, greatest first.

8 What is the equivalent resistance of three resistors, each of resistance R, if they are connected to an ideal battery (a) in series with one another and (b) in parallel with one another? (c) Is the potential difference across the series arrangement greater than, less than, or equal to that across the parallel arrangement? 9 Two resistors are wired to a battery. (a) In which arrangement, parallel or series, are the potential differences across each resistor and across the equivalent resistance all equal? (b) In which arrangement are the currents through each resistor and through the equivalent resistance all equal? 10 Cap-monster maze. In Fig. 27-22, all the capacitors have a capacitance of 6.0 mF, and all the batteries have an emf of 10 V. What is the charge on capacitor C? (If you can find the proper loop through this maze, you can answer the question with a few seconds of mental calculation.)

(1)

(2)

(3)

Figure 27-24 Question 13.

PROB LE M S

795

Problems Tutoring problem available (at instructor’s discretion) in WileyPLUS and WebAssign SSM • – •••

Worked-out solution available in Student Solutions Manual

WWW Worked-out solution is at

Number of dots indicates level of problem difficulty

ILW

Interactive solution is at

http://www.wiley.com/college/halliday

Additional information available in The Flying Circus of Physics and at flyingcircusofphysics.com

Module 27-1 Single-Loop Circuits •1 SSM WWW In Fig. 27-25, the ideal batteries have emfs #1 ! 12 V and #2 ! 6.0 V. What are (a) the current, the dissipation rate in (b) resistor 1 (4.0 0) and (c) resistor 2 (8.0 0), and the energy transfer rate in (d) battery 1 and (e) battery 2? Is energy being supplied or absorbed by (f) battery 1 and (g) battery 2?

•6 A standard flashlight battery can deliver about 2.0 W &h of energy before it runs down. (a) If a battery costs US$0.80, what is the cost of operating a 100 W lamp for 8.0 h using batteries? (b) What is the cost if energy is provided at the rate of US$0.06 per kilowatt-hour?

R1 + –

R2

2 1

•7 A wire of resistance 5.0 0 is connected to a battery whose emf # is 2.0 V and whose internal resistance is 1.0 0. In 2.0 min, how much energy is (a) transferred from chemical form in the battery, (b) dissipated as thermal energy in the wire, and (c) dissipated as thermal energy in the battery?

– +

Figure 27-25 Problem 1.

•2 In Fig. 27-26, the ideal batteries Q have emfs #1 ! 150 V and #2 ! 50 V and the resistances are R1 ! 3.0 0 and – R2 ! 2.0 0. If the potential at P is 100 V, + what is it at Q?

R1 – +

2

1

R2

P •3 ILW A car battery with a 12 V emf and an internal resistance of 0.040 0 is Figure 27-26 Problem 2. being charged with a current of 50 A. What are (a) the potential difference V across the terminals, (b) the rate Pr of energy dissipation inside the battery, and (c) the rate Pemf of energy conversion to chemical form? When the battery is used to supply 50 A to the starter motor, what are (d) V and (e) Pr?

•4 Figure 27-27 shows a circuit of four resistors that are connected to a larger circuit. The graph below the circuit shows the electric potential V(x) as a function of position x along the lower branch of the circuit, through resistor 4; the potential VA is 12.0 V. The graph above the circuit shows the electric potential V(x) versus position x along the upper branch of the circuit, through resistors 1, 2, and 3; the potential differences are 'VB ! 2.00 V and 'VC ! 5.00 V. Resistor 3 has a resistance of 200 0. What is the resistance of (a) resistor 1 and (b) resistor 2? V

∆VB

∆VC

x

0

1

2

3

V (V)

VA 0

•9 (a) In electron-volts, how much work does an ideal battery with a 12.0 V emf do on an electron that passes through the battery from the positive to the negative terminal? (b) If 3.40 $ 10 18 electrons pass through each second, what is the power of the battery in watts? ••10 (a) In Fig. 27-28, what value must R have if the current in the circuit is to be 1.0 mA? Take #1 ! 2.0 V, #2 ! 3.0 V, and r1 ! r2 ! 3.0 0. (b) What is the rate at which thermal energy appears in R?

1

+ –

+ –

r1

2

r2 R

••11 SSM In Fig. 27-29, circuit secFigure 27-28 Problem 10. tion AB absorbs energy at a rate of 50 W when current i ! 1.0 A i through it is in the indicated X direction. Resistance R ! 2.0 0. (a) A B R What is the potential difference between A and B? Emf device X lacks Figure 27-29 Problem 11. internal resistance. (b) What is its emf? (c) Is point B connected to the positive terminal of X or to the negative terminal? Wire 1

4

Figure 27-27 Problem 4.

•8 A certain car battery with a 12.0 V emf has an initial charge of 120 A &h. Assuming that the potential across the terminals stays constant until the battery is completely discharged, for how many hours can it deliver energy at the rate of 100 W?

x

•5 A 5.0 A current is set up in a circuit for 6.0 min by a rechargeable battery with a 6.0 V emf. By how much is the chemical energy of the battery reduced?

••12 Figure 27-30 shows a resistor of resistance R ! 6.00 0 connected to an ideal battery of emf # ! 12.0 V by means of two copper R wires. Each wire has length 20.0 cm and radius 1.00 mm. In dealing with such circuits in this chapter, we generally neglect the potential Wire 2 differences along the wires and the transfer of Figure 27-30 energy to thermal energy in them. Check Problem 12. the validity of this neglect for the circuit of Fig. 27-30: What is the potential difference across (a) the resistor and (b) each of the two sections of wire? At what rate is energy lost to thermal energy in (c) the resistor and (d) each section of wire? ••13 A 10-km-long underground cable extends east to west and consists of two parallel wires, each of which has resistance 13 0/km. An electrical short develops at distance x from the west end when

796

CHAPTE R 27 CI RCU ITS

a conducting path of resistance R connects the wires (Fig. 27-31). The resistance of the wires and the short is then 100 0 when measured from the east end and 200 0 when measured from the west end. What are (a) x and (b) R?

Conducting path East

West

x

Figure 27-31 Problem 13.

••14 In Fig. 27-32a, both batteries have emf # ! 1.20 V and the external resistance R is a variable resistor. Figure 27-32b gives the electric potentials V between the terminals of each battery as functions of R: Curve 1 corresponds to battery 1, and curve 2 corresponds to battery 2. The horizontal scale is set by Rs ! 0.20 0. What is the internal resistance of (a) battery 1 and (b) battery 2? 1

1 2

+ –

R

V (V)

0.5 + –

Rs

–0.3 (a)

R (Ω) (b)

Figure 27-32 Problem 14. ••15 ILW The current in a single-loop circuit with one resistance R is 5.0 A. When an additional resistance of 2.0 0 is inserted in series with R, the current drops to 4.0 A. What is R? •••16 A solar cell generates a potential difference of 0.10 V when a 500 0 resistor is connected across it, and a potential difference of 0.15 V when a 1000 0 resistor is substituted. What are the (a) internal resistance and (b) emf of the solar cell? (c) The area of the cell is 5.0 cm2, and the rate per unit area at which it receives energy from light is 2.0 mW/cm2. What is the efficiency of the cell for converting light energy to thermal energy in the 1000 0 external resistor? •••17 SSM In Fig. 27-33, battery 1 has emf #1 ! 12.0 V and internal resistance r1 ! 0.016 0 and battery 2 has emf #2 ! 12.0 V and internal resistance r2 ! 0.012 0. The batteries are connected in series with an external resistance R. (a) What R value makes the terminal-to-terminal potential difference of one of the batteries zero? (b) Which battery is that?

1

+ r – 1

2

+ r – 2

•22 Figure 27-34 shows five 5.00 0 resistors. Find the equivalent resistance between points (a) F and H and (b) F and G. (Hint: For each pair of points, imagine that a battery is connected across the pair.)

R

R R H

F

•23 In Fig. 27-35, R1 ! 100 0, R2 ! 50 0, and the ideal batteries have emfs #1 ! 6.0 V, #2 ! 5.0 V, and #3 ! 4.0 V. Find (a) the current in re- a sistor 1, (b) the current in resistor 2, and (c) the potential difference between points a and b.

R

R G

Figure 27-34 Problem 22. + – R2

1

+ –

b

+ – 3

2

R1

•24 In Fig. 27-36, R1 ! R2 ! 4.00 0 and R3 ! 2.50 0. Find the equivalent Figure 27-35 Problem 23. resistance between points D and E. (Hint: Imagine that a battery is conR1 D nected across those points.)

2 0

•21 Four 18.0 0 resistors are connected in parallel across a 25.0 V ideal battery. What is the current through the battery?

R

Figure 27-33 Problem 17.

Module 27-2 Multiloop Circuits •18 In Fig. 27-9, what is the potential difference Vd # Vc between points d and c if #1 ! 4.0 V, #2 ! 1.0 V, R1 ! R2 ! 10 0, and R3 ! 5.0 0, and the battery is ideal? •19 A total resistance of 3.00 0 is to be produced by connecting an unknown resistance to a 12.0 0 resistance. (a) What must be the value of the unknown resistance, and (b) should it be connected in series or in parallel? •20 When resistors 1 and 2 are connected in series, the equivalent resistance is 16.0 0. When they are connected in parallel, the equivalent resistance is 3.0 0. What are (a) the smaller resistance and (b) the larger resistance of these two resistors?

•25 SSM Nine copper wires of length l and diameter d are connected in parallel to form a single composite conductor of resistance R. What must be the diameter D of a single copper wire of length l if it is to have the same resistance? ••26 Figure 27-37 shows a battery connected across a uniform resistor R0. A sliding contact can move across the resistor from x ! 0 at the left to x ! 10 cm at the right. Moving the contact changes how much resistance is to the left of the contact and how much is to the right. Find the rate at which energy is dissipated in resistor R as a function of x. Plot the function for # ! 50 V, R ! 2000 0, and R0 ! 100 0.

R2

R3

E

Figure 27-36 Problem 24. R Sliding contact

x R0 + –

Figure 27-37 Problem 26. Lightning current

I d

••27 Side flash. Figure 27-38 indicates one reason no one should stand under a tree during a lightning h storm. If lightning comes down the side of the tree, a portion can jump over to the person, especially if the current on the tree reaches a dry reFigure 27-38 Problem 27. gion on the bark and thereafter must travel through air to reach the ground. In the figure, part of the lightning jumps through distance d in air and then travels through the person (who has negligible resistance relative to that of air because of the highly conducting salty fluids within the body). The rest of the current travels through air alongside the tree, for a distance h. If d/h ! 0.400 and the total current is I ! 5000 A, what is the current through the person? ••28 The ideal battery in Fig. 27-39a has emf # ! 6.0 V. Plot 1 in Fig. 27-39b gives the electric potential difference V that can appear across resistor 1 versus the current i in that resistor when the resistor

797

PROB LE M S

is individually tested by putting a variable potential across it. The scale of the V axis is set by Vs ! 18.0 V, and the scale of the i axis is set by is ! 3.00 m1. Plots 2 and 3 are similar plots for resistors 2 and 3, respectively, when they are individually tested by putting a variable potential across them. What is the current in resistor 2 in the circuit of Fig. 27-39a?

battery is opposite the direction of that battery’s emf. What are (a) emf #1, (b) resistance R1, and (c) resistance R2? ••33 In Fig. 27-44, the current in resistance 6 is i6 ! 1.40 A and the resistances are R1 ! R2 ! R3 ! 2.00 0, R4 ! 16.0 0, R5 ! 8.00 0, and R6 ! 4.00 0. What is the emf of the ideal battery?

Vs

+ –

R3

+ –

R1

V (V)

1

R2

R1

R2

R5

R3

i6

R4

2

Figure 27-44 Problem 33.

3

(a) 0 (b)

is

i (mA)

Figure 27-39 Problem 28.

••34 The resistances in Figs. 27-45a and b are all 6.0 0, and the batteries are ideal 12 V batteries. (a) When switch S in Fig. 27-45a is closed, what is the change in the electric potential V1 across resistor 1, or does V1 remain the same? (b) When switch S in Fig. 27-45b is closed, what is the change in V1 across resistor 1, or does V1 remain the same? S

+ –

••29 In Fig. 27-40, R1 ! 6.00 0, R2 ! 18.0 0, and the ideal battery has emf # ! 12.0 V. What are the (a) size and (b) direction (left or right) of current i1? (c) How much energy is dissipated by all four resistors in 1.00 min? ••30 In Fig. 27-41, the ideal batteries have emfs #1 ! 10.0 V and #2 ! 0.500#1, and the resistances are each 4.00 0. What is the current in (a) resistance 2 and (b) resistance 3?

R1

+ –

R2 R2

i1 R2

R2

R1 1

R3

2

+ –

2

(a)

+ –

Current (A)

••32 Both batteries in Fig. 27-43a are ideal. Emf #1 of battery 1 has a + fixed value, but emf #2 of battery 2 1 – + can be varied between 1.0 V and 10 2 – V. The plots in Fig. 27-43b give the V2 V1 currents through the two batteries as a function of #2. The vertical scale is Figure 27-42 Problem 31. set by is ! 0.20 A. You must decide which plot corresponds to which battery, but for both plots, a negative current occurs when the direction of the current through the

R1

R2

1

is 0

5

–is

(b)

Figure 27-43 Problem 32.

R2

R3

+ –

R1

R2

(b)

Figure 27-45 Problem 34.

Figure 27-40 Problem 29.

+ –

R1

S

(a)

••31 SSM In Fig. 27-42, the ideal batteries have emfs #1 ! 5.0 V and #2 ! 12 V, the resistances are Figure 27-41 Problems 30, 41, each 2.0 0, and the potential is deand 88. fined to be zero at the grounded point of the circuit. What are potentials (a) V1 and (b) V2 at the indicated points?

+ –

R6

2

(V)

10

A

••35 In Fig. 27-46, # ! 12.0 V, R1 ! 2000 0, R2 ! 3000 0, and R3 ! 4000 0. What are the potential differences (a) VA # VB, (b) VB # VC, (c) VC # VD, and (d) VA # VC?

R2 C + –

••36 In Fig. 27-47, #1 ! 6.00 V, #2 ! 12.0 V, R1 ! 100 0, R2 ! 200 0, and R3 ! 300 0. One point of the circuit is grounded (V ! 0). What are the (a) size and (b) direction (up or down) of the current through resistance 1, the (c) size and (d) direction (left or right) of the current through resistance 2, and the (e) size and (f) direction of the current through resistance 3? (g) What is the electric potential at point A?

B

R3 R1

R2 D

Figure 27-46 Problem 35. A R2

+ –

1

R3 R1

2

+ –

Figure 27-47 Problem 36.

••37 In Fig. 27-48, the resistances are R1 ! 2.00 0, R2 ! 5.00 0, and the battery is ideal. What value of R3 maximizes the dissipation rate in resistance 3? ••38 Figure 27-49 shows a section of a circuit. The resistances are R1 ! 2.0 0, R2 ! 4.0 0, and R3 ! 6.0 0, and the indicated current is i ! 6.0 A. The electric potential difference between points A and B that connect the section to the rest of the circuit is VA # VB ! 78 V. (a) Is the device represented by “Box” absorbing or providing energy to the circuit, and (b) at what rate?

R1

R1 R2

R3

+ –

Figure 27-48 Problems 37 and 98. R2 R1 A

Box

R3 i

Figure 27-49 Problem 38.

B

CHAPTE R 27 CI RCU ITS

••39 In Fig. 27-50, two batteries with an emf # ! 12.0 V and an internal resistance r ! 0.300 0 are connected in parallel across a resistance R. (a) For what value of R is the dissipation rate in the resistor a maximum? (b) What is that maximum? ••40 Two identical batteries of emf # ! 12.0 V and internal resistance r ! 0.200 0 are to be connected to an external resistance R, either in parallel (Fig. 27-50) or in series (Fig. 27-51). If R ! 2.00r, what is the current i in the external resistance in the (a) parallel and (b) series arrangements? (c) For which arrangement is i greater? If R ! r/2.00, what is i in the external resist+ – ance in the (d) parallel arrangement r and (e) series arrangement? (f) For which arrangement is i greater now?

through the battery as a function of R3. The horizontal scale is set by R3s ! 20 0. The curve has an asymptote of 2.0 mA as R3 : ,. What are (a) resistance R1 and (b) resistance R2?

+ – r

6 + –

+ –

r R

Figure 27-50 Problems 39 and 40.

2 R 3s

r

R

R 3 (Ω) (b)

Figure 27-55 Problem 46.

+ –

R

R n resistors in parallel

Figure 27-52 Problem 42.

••43 You are given a number of 10 0 resistors, each capable of dissipating only 1.0 W without being destroyed. What is the minimum number of such resistors that you need to combine in series or in parallel to make a 10 0 resistance that is capable of dissipating at R1 least 5.0 W? R 4

4

(a)

R

+ –

R3

0

••41 In Fig. 27-41, #1 ! 3.00 V, #2 ! 1.00 V, R1 ! 4.00 0, R2 ! 2.00 0, R3 ! Figure 27-51 Problem 40. 5.00 0, and both batteries are ideal. What is the rate at which energy is dissipated in (a) R1, (b) R2, and (c) R3? What is the power of (d) battery 1 and (e) battery 2? ••42 In Fig. 27-52, an array of n parallel resistors is connected in series to a resistor and an ideal battery. All the resistors have the same resistance. If an identical resistor were added in parallel to the parallel array, the current through the battery would change by 1.25%. What is the value of n?

R1 R2

i (mA)

798

•••47 SSM A copper wire of radius a ! 0.250 mm has an aluminum jacket of outer radius b ! 0.380 mm. There is a current i ! 2.00 A in the composite wire. Using Table 26-1, calculate the current in (a) the copper and (b) the aluminum. (c) If a potential difference V ! 12.0 V between the ends maintains the current, what is the length of the composite wire? •••48 In Fig. 27-53, the resistors have the values R1 ! 7.00 0, R2 ! 12.0 0, and R3 ! 4.00 0, and the ideal battery’s emf is # ! 24.0 V. For what value of R4 will the rate at which the battery transfers energy to the resistors equal (a) 60.0 W, (b) the maximum possible rate Pmax, and (c) the minimum possible rate Pmin? What are (d) Pmax and (e) Pmin? Module 27-3 The Ammeter and the Voltmeter ••49 ILW (a) In Fig. 27-56, what current does the ammeter read if # ! 5.0 V (ideal battery), R1 ! 2.0 0, R2 ! 4.0 0, and R3 ! 6.0 0? (b) The ammeter and battery are now interchanged. Show that the ammeter reading is unchanged. ••50 In Fig. 27-57, R1 ! 2.00R, the ammeter resistance is zero, and the battery is ideal. What multiple of #/R gives the current in the ammeter?

A

+ –

R2 R3

R1

Figure 27-56 Problem 49.

R1 + –

R A

R

R

••45 ILW In Fig. 27-54, the resistances + 3 – are R1 ! 1.0 0 and R2 ! 2.0 0, R2 + and the ideal batteries have emfs 1 – R1 + #1 ! 2.0 V and #2 ! #3 ! 4.0 V. What 2 R1 – are the (a) size and (b) direction (up or down) of the current in battery 1, b the (c) size and (d) direction of the Figure 27-54 Problem 45. current in battery 2, and the (e) size and (f) direction of the current in battery 3? (g) What is the potential difference Va # Vb?

••51 In Fig. 27-58, a voltmeter of resistance RV ! 300 0 and an amFigure 27-57 Problem 50. meter of resistance RA ! 3.00 0 are being used to measure a resistance A R in a circuit that also contains a reR sistance R0 ! 100 0 and an ideal battery with an emf of # ! 12.0 V. V Resistance R is given by R ! V/i, R0 where V is the potential across R and i is the ammeter reading. The + – voltmeter reading is V-, which is V plus the potential difference Figure 27-58 Problem 51. across the ammeter. Thus, the ratio of the two meter readings is not R but only an apparent resistance R- ! V-/i. If R ! 85.0 0, what are (a) the ammeter reading, (b) the voltmeter reading, and (c) R-? (d) If RA is decreased, does the difference between R- and R increase, decrease, or remain the same?

••46 In Fig. 27-55a, resistor 3 is a variable resistor and the ideal battery has emf # ! 12 V. Figure 27-55b gives the current i

••52 A simple ohmmeter is made by connecting a 1.50 V flashlight battery in series with a resistance R and an ammeter that

••44 In Fig. 27-53, R1 ! 100 0, R2 ! R3 ! 50.0 0, R4 ! 75.0 0, and the ideal battery has emf # ! 6.00 V. (a) What is the equivalent resistance? What is i in (b) resistance 1, (c) resistance 2, (d) resistance 3, and (e) resistance 4?

R2

R3

Figure 27-53 Problems 44 and 48. R1

a

R1

799

PROB LE M S 0–1.00

reads from 0 to 1.00 mA, as shown in mA Fig. 27-59. Resistance R is adjusted – so that when the clip leads are + shorted together, the meter deflects R to its full-scale value of 1.00 mA. What external resistance across the Figure 27-59 Problem 52. leads results in a deflection of (a) 10.0%, (b) 50.0%, and (c) 90.0% of full scale? (d) If the ammeter has a resistance of 20.0 0 and the internal resistance of the battery is negligible, what is the value of R? ••53 In Fig. 27-14, assume that # ! 3.0 V, r ! 100 0, R1 ! 250 0, and R2 ! 300 0. If the voltmeter resistance RV is 5.0 k0, what percent error does it introduce into the measurement of the potential difference across R1? Ignore the presence of the ammeter. Lights A

S Starting motor

S

V

+

••54 When the lights of a car are switched on, an ammeter in series with them reads 10.0 A and a voltmeter connected across them reads 12.0 V (Fig. 27-60). When the electric starting motor is turned on, the ammeter reading drops to 8.00 A and the lights dim somewhat. If the internal resistance of the battery is 0.0500 0 and that of the ammeter is negligible, what are (a) the emf of the battery and (b) the current through the starting motor when the lights are on?



r

••55 In Fig. 27-61, Rs is to be adjusted in Figure 27-60 value by moving the sliding contact across Problem 54. it until points a and b are brought to the same potential. (One tests for this condition by momentarily connecting a sensitive ammeter bea tween a and b; if these points are at the same potential, the ammeter R2 R1 will not deflect.) Show that when this adjustment is made, the followSliding contact ing relation holds: Rx ! RsR2/R1. An unknown resistance (Rx) can be measured in terms of a standard Rs Rx (Rs) using this device, which is called a Wheatstone bridge. b ••56 In Fig. 27-62, a voltmeter of R0 + – resistance RV ! 300 0 and an ammeter of resistance RA ! 3.00 0 Figure 27-61 Problem 55. are being used to measure a resistance R in a circuit that also conA tains a resistance R0 ! 100 0 and R an ideal battery of emf # ! 12.0 V. Resistance R is given by R ! V/i, V where V is the voltmeter reading and i is the current in resistance R. R0 However, the ammeter reading is + – not i but rather i-, which is i plus the current through the voltmeter. Figure 27-62 Problem 56. Thus, the ratio of the two meter readings is not R but only an apparent resistance R- ! V/i-. If R ! 85.0 0, what are (a) the ammeter reading, (b) the voltmeter reading, and (c) R-? (d) If RV is increased, does the difference between R- and R increase, decrease, or remain the same?

Module 27-4 RC Circuits •57 Switch S in Fig. 27-63 is closed at time t ! 0, to begin charging an initially uncharged capacitor of capacitance C ! 15.0 mF through a resistor of resistance R ! 20.0 0. At what time is the potential across the capacitor equal to that across the resistor?

S + –

R

C

Figure 27-63 Problems 57 and 96.

•58 In an RC series circuit, emf # ! 12.0 V, resistance R ! 1.40 M0, and capacitance C ! 1.80 mF. (a) Calculate the time constant. (b) Find the maximum charge that will appear on the capacitor during charging. (c) How long does it take for the charge to build up to 16.0 mC? •59 SSM What multiple of the time constant t gives the time taken by an initially uncharged capacitor in an RC series circuit to be charged to 99.0% of its final charge? •60 A capacitor with initial charge q0 is discharged through a resistor. What multiple of the time constant t gives the time the capacitor takes to lose (a) the first one-third of its charge and (b) two-thirds of its charge? •61 ILW A 15.0 k0 resistor and a capacitor are connected in series, and then a 12.0 V potential difference is suddenly applied across them. The potential difference across the capacitor rises to 5.00 V in 1.30 ms. (a) Calculate the time constant of the circuit. (b) Find the capacitance of the capacitor. R ••62 Figure 27-64 shows the circuit of a flashing lamp, like those attached to barrels at highway construction sites. + The fluorescent lamp L (of negligible – C L capacitance) is connected in parallel across the capacitor C of an RC circuit. There is a current through the lamp Figure 27-64 only when the potential difference Problem 62. across it reaches the breakdown voltage VL; then the capacitor discharges completely through the lamp and the lamp flashes briefly. For a lamp with breakdown voltage VL ! 72.0 V, wired to a 95.0 V ideal battery and a 0.150 mF capacitor, what resistance R is needed for two flashes per second?

••63 SSM WWW In the circuit of Fig. S R1 27-65, # ! 1.2 kV, C ! 6.5 mF, R1 ! R3 R2 ! R3 ! 0.73 M0. With C completely + R uncharged, switch S is suddenly closed – 2 C (at t ! 0). At t ! 0, what are (a) current i1 in resistor 1, (b) current i2 in resistor 2, and (c) current i3 in resistor 3? At t ! , Figure 27-65 Problem 63. (that is, after many time constants), what are (d) i1, (e) i2, and (f) i3? What is the potential difference V2 across resistor 2 at (g) t ! 0 and (h) t ! ,? (i) Sketch V2 versus t between these two extreme times. ••64 A capacitor with an initial potential difference of 100 V is discharged through a resistor when a switch between them is closed at t ! 0.At t ! 10.0 s, the potential difference across the capacitor is 1.00 V. (a) C What is the time constant of the + R2 – circuit? (b) What is the potential differR1 ence across the capacitor at t ! 17.0 s? ••65 In Fig. 27-66, R1 ! 10.0 k0, R2 ! 15.0 k0, C ! 0.400 mF, and the

Figure 27-66 Problems 65 and 99.

800

CHAPTE R 27 CI RCU ITS

ideal battery has emf # ! 20.0 V. First, the switch is closed a long time so that the steady state is reached. Then the switch is opened at time t ! 0. What is the current in resistor 2 at t ! 4.00 ms? ••66 Figure 27-67 displays two circuits with a charged capacitor that C2 R1 R2 is to be discharged through a resis- C 1 tor when a switch is closed. In Fig. 27-67a, R1 ! 20.0 0 and C1 ! 5.00 (a) (b) mF. In Fig. 27-67b, R2 ! 10.0 0 and C2 ! 8.00 mF. The ratio of the initial Figure 27-67 Problem 66. charges on the two capacitors is q02/q01 ! 1.50. At time t ! 0, both switches are closed. At what time t do the two capacitors have the same charge?

73 SSM Wires A and B, having equal lengths of 40.0 m and equal diameters of 2.60 mm, are connected in series. A potential difference of 60.0 V is applied between the ends of the composite wire. The resistances are RA ! 0.127 0 and RB ! 0.729 0. For wire A, what are (a) magnitude J of the current density and (b) potential difference V? (c) Of what type material is wire A made (see Table 26-1)? For wire B, what are (d) J and (e) V? (f) Of what type material is B made? 74 What are the (a) size and (b) direction (up or down) of current i in Fig. 27-71, where all resistances are 4.0 0 and all batteries are ideal and have an emf of 10 V? (Hint: This can be answered using only mental calculation.)

••67 The potential difference between the plates of a leaky (meaning that charge leaks from one plate to the other) 2.0 mF capacitor drops to one-fourth its initial value in 2.0 s. What is the equivalent resistance between the capacitor plates? ••68 A 1.0 mF capacitor with an initial stored energy of 0.50 J is discharged through a 1.0 M0 resistor. (a) What is the initial charge on the capacitor? (b) What is the current through the resistor when the discharge starts? Find an expression that gives, as a function of time t, (c) the potential difference VC across the capacitor, (d) the potential difference VR across the resistor, and (e) the rate at which thermal energy is produced in the resistor.

i

•••69 A 3.00 M0 resistor and a 1.00 mF capacitor are connected in series with an ideal battery of emf # ! 4.00 V.At 1.00 s after the connection is made, what is the rate at which (a) the charge of the capacitor is increasing, (b) energy is being stored in the capacitor, (c) thermal energy is appearing in the resistor, and (d) energy is being delivered by the battery? Figure 27-71 Problem 74.

Additional Problems 70 Each of the six real batteries in Fig. 27-68 has an emf of 20 V and a resistance of 4.0 0. (a) What is the current through the (external) resistance R ! 4.0 0? (b) What is the potential difference across each battery? (c) What is the power of each battery? (d) At what rate does each battery transfer energy to internal thermal energy? 71 In Fig. 27-69, R1 ! 20.0 0, R2 ! 10.0 0, and the ideal battery has emf a # ! 120 V. What is the current at point a if we close (a) only switch S1, (b) only switches S1 and S2, and (c) all – + three switches?

R

Figure 27-68 Problem 70.

S1

S2

S3

R1

R1

R1

72 In Fig. 27-70, the ideal battery has R1 R2 R2 emf # ! 30.0 V, and the resistances are R1 ! R2 ! 14 0, R3 ! R4 ! Figure 27-69 Problem 71. R5 ! 6.0 0, R6 ! 2.0 0, and R7 ! 1.5 0.What are currents (a) i2, (b) i4, (c) i1, (d) i3, and (e) i5? i2 R1

i4

R2

R7 i5

+ –

Figure 27-70 Problem 72.

i3

R3

R4

i1

R5

R6

75 Suppose that, while you are sitting in a chair, charge separation between your clothing and the chair puts you at a potential of 200 V, with the capacitance between you and the chair at 150 pF. When you stand up, the increased separation between your body and the chair decreases the capacitance to 10 pF. (a) What then is the potential of your body? That potential is reduced over time, as the charge on you drains through your body and shoes (you are a capacitor discharging through a resistance). Assume that the resistance along that route is 300 G0. If you touch an electrical component while your potential is greater than 100 V, you could ruin the component. (b) How long must you wait until your potential reaches the safe level of 100 V? If you wear a conducting wrist strap that is connected to ground, your potential does not increase as much when you stand up; you also discharge more rapidly because the resistance through the grounding connection is much less than through your body and shoes. (c) Suppose that when you stand up, your potential is 1400 V and the chair-to-you capacitance is 10 pF. What resistance in that wrist-strap grounding connection will allow you to discharge to 100 V in 0.30 s, which is less time than you would need to reach for, say, your computer? 76 In Fig. 27-72, the ideal batteries have emfs #1 ! 20.0 V, #2 ! 10.0 V, and #3 ! 5.00 V, and the resistances are each 2.00 0. What are the (a) size and (b) direction (left or right) of current i1? (c) Does battery 1 supply or absorb energy, and (d) what is its power? (e) Does battery 2 supply or absorb energy, and (f) what is

801

PROB LE M S

its power? (g) Does battery 3 supply or absorb energy, and (h) what is its power? – + 1

+ –

+ –

3

2

i1

Figure 27-72 Problem 76.

77 SSM A temperature-stable resistor is made by connecting a resistor made of silicon in series with one made of iron. If the required total resistance is 1000 0 in a wide temperature range around 20/C, what should be the resistance of the (a) silicon resistor and (b) iron resistor? (See Table 26-1.) 78 In Fig. 27-14, assume that # ! 5.0 V, r ! 2.0 0, R1 ! 5.0 0, and R2 ! 4.0 0. If the ammeter resistance RA is 0.10 0, what percent error does it introduce into the measurement of the current? Assume that the voltmeter is not present. 79 SSM An initially uncharged capacitor C is fully charged by a device of constant emf # connected in series with a resistor R. (a) Show that the final energy stored in the capacitor is half the energy supplied by the emf device. (b) By direct integration of i2R over the charging time, show that the thermal energy dissipated by the resistor is also half the energy supplied by the emf device. 80 In Fig. 27-73, R1 ! 5.00 0, R2 ! 10.0 0, R3 ! 15.0 0, C1 ! 5.00 mF, C2 ! 10.0 mF, and the ideal battery has emf # ! 20.0 V.Assuming that the circuit is in the steady state, what is the total energy stored in the two capacitors? 81 In Fig. 27-5a, find the potential difference across R2 if # ! 12 V, R1 ! 3.0 0, R2 ! 4.0 0, and R3 ! 5.0 0.

C1

+ –

C2

R3

Figure 27-73 Problem 80.

82 In Fig. 27-8a, calculate the potential difference between a and c by considering a path that contains R, r1, and #1. 83 SSM A controller on an electronic arcade game consists of a variable resistor connected across the plates of a 0.220 mF capacitor. The capacitor is charged to 5.00 V, then discharged through the resistor. The time for the potential difference across the plates to decrease to 0.800 V is measured by a Rindicator clock inside the game. If the range of Indicator discharge times that can be handled effectively is from 10.0 ms to 6.00 ms, Tank what should be the (a) lower value + unit 12 V and (b) higher value of the resist- – ance range of the resistor? 84 An automobile gasoline gauge is shown schematically in Fig. 27-74. The indicator (on the dashboard) has a resistance of 10 0. The tank

Connected through chassis

85 SSM The starting motor of a car is turning too slowly, and the mechanic has to decide whether to replace the motor, the cable, or the battery. The car’s manual says that the 12 V battery should have no more than 0.020 0 internal resistance, the motor no more than 0.200 0 resistance, and the cable no more than 0.040 0 resistance. The mechanic turns on the motor and measures 11.4 V across the battery, 3.0 V across the cable, and a current of 50 A. Which part is defective? 86 Two resistors R1 and R2 may be connected either in series or in parallel across an ideal battery with emf #. We desire the rate of energy dissipation of the parallel combination to be five times that of the series combination. If R1 ! 100 0, what are the (a) smaller and (b) larger of the two values of R2 that result in that dissipation rate? 87 The circuit of Fig. 27-75 shows a capacitor, two ideal batteries, two resistors, and a switch S. Initially S has been open for a long time. If it is then closed for a long time, what is the change in the charge on the capacitor? Assume C ! 10 mF, #1 ! 1.0 V, #2 ! 3.0 V, R1 ! 0.20 0, and R2 ! 0.40 0.

Rtank

Figure 27-74 Problem 84.

S

– +

1

C

R2

Figure 27-75 Problem 87. 2.0R

88 In Fig. 27-41, R1 ! 10.0 0, R2 ! 20.0 0, and the ideal batteries have emfs #1 ! 20.0 V and #2 ! 50.0 V. What 4.0R value of R3 results in no current through battery 1?

B R

6.0R 3.0R

A

Figure 27-76 Problem 89.

90 (a) In Fig. 27-4a, show that the rate at which energy is dissipated in R as thermal energy is a maximum when R ! r. (b) Show that this maximum power is P ! #2/4r. 91 In Fig. 27-77, the ideal batteries have emfs #1 ! 12.0 V and #2 ! 4.00 V, and the resistances are each 4.00 0. What are the (a) size and (b) direction (up or down) of i1 and the (c) size and (d) direction of i2? (e) Does battery 1 supply or absorb energy, and (f) what is its energy transfer rate? (g) Does battery 2 supply or absorb energy, and (h) what is its energy transfer rate? 92 Figure 27-78 shows a portion of a circuit through which there is a current i1 I ! 6.00 A. The resistances are R1 ! R2 ! 2.00R3 ! 2.00R4 ! 4.00 0. What is the current i1 through resistor 1? 93 Thermal energy is to be generated in a 0.10 0 resistor at the rate of

– +

2

R1

89 In Fig. 27-76, R ! 10 0. What is the equivalent resistance between points A and B? (Hint: This circuit section might look simpler if you first assume that points A and B are connected to a battery.)

R2

R1

unit is a float connected to a variable resistor whose resistance varies linearly with the volume of gasoline. The resistance is 140 0 when the tank is empty and 20 0 when the tank is full. Find the current in the circuit when the tank is (a) empty, (b) half-full, and (c) full. Treat the battery as ideal.

+ –

1

i2

i1

– +

2

Figure 27-77 Problem 91. I

R3 R1

R2

R4

I

Figure 27-78 Problem 92.

802

CHAPTE R 27 CI RCU ITS

10 W by connecting the resistor to a battery whose emf is 1.5 V. (a) What potential difference must exist across the resistor? (b) What must be the internal resistance of the battery? 94 Figure 27-79 shows three 20.0 0 C resistors. Find the equivalent resistB ance between points (a) A and B, (b) A and C, and (c) B and C. (Hint: A Imagine that a battery is connected Figure 27-79 Problem 94. between a given pair of points.) 95 A 120 V power line is protected by a 15 A fuse. What is the maximum number of 500 W lamps that can be simultaneously operated in parallel on this line without “blowing” the fuse because of an excess of current? 96 Figure 27-63 shows an ideal battery of emf # ! 12 V, a resistor of resistance R ! 4.0 0, and an uncharged capacitor of capacitance C ! 4.0 mF. After switch S is closed, what is the current through the resistor when the charge on the capacitor is 8.0 mC? 97 SSM A group of N identical batteries of emf # and internal resistance r may be connected all in series (Fig. 27-80a) or all in parallel (Fig. 27-80b) and then across a resistor R. Show that both arrangements give the same current in R if R ! r.

+ –

+ – r

R (a) N batteries in parallel + –

+ – r

+ – r

R

r

(b)

Figure 27-80 Problem 97. 98 SSM In Fig. 27-48, R1 ! R2 ! 10.0 0, and the ideal battery has emf # ! 12.0 V. (a) What value of R3 maximizes the rate at which the battery supplies energy and (b) what is that maximum rate? 99 SSM In Fig. 27-66, the ideal battery has emf # ! 30 V, the resistances are R1 ! 20 k0 and R2 ! 10 k0, and the capacitor is uncharged. When the switch is closed at time t ! 0, what is the current in (a) resistance 1 and (b) resistance 2? (c) A long time later, what is the current in resistance 2? 100 In Fig. 27-81, the ideal batteries have emfs #1 ! 20.0 V, #2 ! 10.0 V,

R3 R2

R4

1

– + 2

– +

R5

102 The following table gives the + – electric potential difference VT across the terminals of a battery as a Figure 27-82 Problem 101. function of current i being drawn from the battery. (a) Write an equation that represents the relationship between VT and i. Enter the data into your graphing calculator and perform a linear regression fit of VT versus i. From the parameters of the fit, find (b) the battery’s emf and (c) its internal resistance. 50.0 10.7

75.0 9.00

100 7.70

125 6.00

103 In Fig. 27-83, #1 ! 6.00 V, #2 ! 12.0 V, R1 ! 200 0, and R2 ! 100 0. What are the (a) size and (b) direction (up or down) of the current through resistance 1, the (c) size and (d) direction of the current through resistance 2, and the (e) size and (f) direction of the current through battery 2?

r

r

R1

101 In Fig. 27-82, an ideal battery of emf # ! 12.0 V is connected to a network of resistances R1 ! 6.00 0, R2 ! 12.0 0, R3 ! 4.00 0, R4 ! 3.00 0, and R5 ! 5.00 0. What is the potential difference across resistance 5?

i(A): VT(V):

N batteries in series + –

#3 ! 5.00 V, and #4 ! 5.00 V, and the resistances are each 2.00 0. What are the (a) size and (b) direction (left or right) of current i1 and the (c) size and (d) direction of current i2? (This can be answered with only mental calculation.) (e) At what rate is energy being transferred in battery 4, and (f) is the energy being supplied or absorbed by the battery?

150 4.80

175 3.00

R2

– +

R1

1

105 In Fig. 27-84, R1 ! R2 ! 2.0 0, R3 ! 4.0 0, R4 ! 3.0 0, R5 ! 1.0 0, and R6 ! R7 ! R8 ! 8.0 0, and the ideal batteries have emfs #1 ! 16 V and #2 ! 8.0 V. What are the (a) size and (b) direction (up or down) of current i1 and the (c) size and (d) direction of current i2? What is the energy transfer rate in (e) battery 1 and (f) battery 2? Is energy being supplied or absorbed in (g) battery 1 and (h) battery 2?

R2

R3

R4 i1

R1

4

R6 i2

R8

1

+ –

R7

i1

Figure 27-81 Problem 100.

+ –

104 A three-way 120 V lamp bulb that contains two filaments is rated for 100-200-300 W. One filament burns out. Afterward, the bulb operates at the same intensity (dissipates energy at the same rate) on its lowest as on its highest switch positions but does not operate at all on the middle position. (a) How are the two filaments wired to the three switch positions? What are the (b) smaller and (c) larger values of the filament resistances?

3

i2

2

Figure 27-83 Problem 103.

– +

+ –

200 1.70

R5

2

– +

Figure 27-84 Problem 105.

C

H

A

P

T

E

R

2

8

Magnetic Fields 28-1 MAGNETIC FIELDS AND THE DEFINITION OF B

:

Learning Objectives After reading this module, you should be able to . . .

28.01 Distinguish an electromagnet from a permanent magnet. 28.02 Identify that a magnetic field is a vector quantity and thus has both magnitude and direction. 28.03 Explain how a magnetic field can be defined in terms of what happens to a charged particle moving through the field. 28.04 For a charged particle moving through a uniform magnetic field, apply the relationship between force magnitude FB, charge q, speed v, field magnitude B, and the angle f between the directions of the velocity vector v: and the : magnetic field vector B . 28.05 For a charged particle sent through a uniform magnetic field, find the direction of the magnetic force : FB by (1) applying the right-hand rule to find the direction

Key Ideas ● When a charged particle moves through a magnetic field

a magnetic force acts on the particle as given by :

:

B,

:

FB ! q(v: $ B ), where q is the particle’s charge (sign included) and v is the particle’s velocity.

:

of the cross product v: $ B and (2) determining what effect the charge q has on the direction. : 28.06 Find the magnetic force FB acting on a moving charged : particle by evaluating the cross product q(v: $ B ) in unit-vector notation and magnitude-angle notation. : 28.07 Identify that the magnetic force vector FB must always be perpendicular to both the velocity vector v: and the : magnetic field vector B . 28.08 Identify the effect of the magnetic force on the particle’s speed and kinetic energy. 28.09 Identify a magnet as being a magnetic dipole. 28.10 Identify that opposite magnetic poles attract each other and like magnetic poles repel each other. 28.11 Explain magnetic field lines, including where they originate and terminate and what their spacing represents. :

● The magnitude of

:

● The right-hand rule for cross products gives the direction

:

of v: $ B . The sign of q then determines whether FB is : in the same direction as v: $ B or in the opposite direction. :

FB is given by FB ! !q!vB sin f, :

where f is the angle between v: and B .

What Is Physics? As we have discussed, one major goal of physics is the study of how an electric field can produce an electric force on a charged object. A closely related goal is the study of how a magnetic field can produce a magnetic force on a (moving) charged particle or on a magnetic object such as a magnet. You may already have a hint of what a magnetic field is if you have ever attached a note to a refrigerator door with a small magnet or accidentally erased a credit card by moving it near a magnet. The magnet acts on the door or credit card via its magnetic field. The applications of magnetic fields and magnetic forces are countless and changing rapidly every year. Here are just a few examples. For decades, the entertainment industry depended on the magnetic recording of music and images on audiotape and videotape. Although digital technology has largely replaced 803

804

CHAPTE R 28 MAG N ETIC FI E LDS

magnetic recording, the industry still depends on the magnets that control CD and DVD players and computer hard drives; magnets also drive the speaker cones in headphones, TVs, computers, and telephones. A modern car comes equipped with dozens of magnets because they are required in the motors for engine ignition, automatic window control, sunroof control, and windshield wiper control. Most security alarm systems, doorbells, and automatic door latches employ magnets. In short, you are surrounded by magnets. The science of magnetic fields is physics; the application of magnetic fields is engineering. Both the science and the application begin with the question “What produces a magnetic field?”

What Produces a Magnetic Field? :

Digital Vision/Getty Images, Inc.

Figure 28-1 Using an electromagnet to collect and transport scrap metal at a steel mill.

Because an electric field E is produced by an electric charge, we might reason: ably expect that a magnetic field B is produced by a magnetic charge. Although individual magnetic charges (called magnetic monopoles) are predicted by certain theories, their existence has not been confirmed. How then are magnetic fields produced? There are two ways. One way is to use moving electrically charged particles, such as a current in a wire, to make an electromagnet. The current produces a magnetic field that can be used, for example, to control a computer hard drive or to sort scrap metal (Fig. 28-1). In Chapter 29, we discuss the magnetic field due to a current. The other way to produce a magnetic field is by means of elementary particles such as electrons because these particles have an intrinsic magnetic field around them. That is, the magnetic field is a basic characteristic of each particle just as mass and electric charge (or lack of charge) are basic characteristics. As we discuss in Chapter 32, the magnetic fields of the electrons in certain materials add together to give a net magnetic field around the material. Such addition is the reason why a permanent magnet, the type used to hang refrigerator notes, has a permanent magnetic field. In other materials, the magnetic fields of the electrons cancel out, giving no net magnetic field surrounding the material. Such cancellation is the reason you do not have a permanent field around your body, which is good because otherwise you might be slammed up against a refrigerator door every time you passed one. : Our first job in this chapter is to define the magnetic field B . We do so by using the experimental fact that when a charged particle moves through a : magnetic field, a magnetic force FB acts on the particle.

:

The Definition of B

:

We determined the electric field E at a point by putting a test particle of charge : q at rest at that point and measuring the electric force FE acting on the particle. : We then defined E as :

:

E!

FE . q

(28-1) :

If a magnetic monopole were available, we could define B in a similar way. : Because such particles have not been found, we must define B in another way, : in terms of the magnetic force FB exerted on a moving electrically charged test particle. Moving Charged Particle. In principle, we do this by firing a charged parti: cle through the point at which B is to be defined, using various directions and : speeds for the particle and determining the force FB that acts on the particle at that point. After many such trials we would find that when the particle’s velocity

:

28-1 MAG N ETIC FI E LDS AN D TH E DE FI N ITION OF B :

v: is along a particular axis through the point, force FB is zero. For all other direc: tions of v: , the magnitude of FB is always proportional to v sin f, where f is the angle between the zero-force axis and the direction of v: . Furthermore, the direc: tion of FB is always perpendicular to the direction of v: . (These results suggest that a cross product is involved.) : The Field. We can then define a magnetic field B to be a vector quantity that is directed along the zero-force axis. We can next measure the magnitude of : FB when v: is directed perpendicular to that axis and then define the magnitude : of B in terms of that force magnitude: B!

FB , !q!v

where q is the charge of the particle. We can summarize all these results with the following vector equation: :

:

FB ! qv: $ B;

(28-2)

: FB

that is, the force on the particle is equal to the charge q times the cross product : of its velocity v: and the field B (all measured in the same reference frame). : Using Eq. 3-24 for the cross product, we can write the magnitude of FB as FB ! !q!vB sin f,

(28-3) :

where f is the angle between the directions of velocity v: and magnetic field B .

Finding the Magnetic Force on a Particle :

Equation 28-3 tells us that the magnitude of the force FB acting on a particle in a magnetic field is proportional to the charge q and speed v of the particle. Thus, the force is equal to zero if the charge is zero or if the particle is stationary. : Equation 28-3 also tells us that the magnitude of the force is zero if v: and B are either parallel (f ! 0/) or antiparallel (f ! 180/), and the force is at its maximum : when v: and B are perpendicular to each other. : Directions. Equation 28-2 tells us all this plus the direction of FB. From : : Module 3-3, we know that the cross product v $ B in Eq. 28-2 is a vector that is : perpendicular to the two vectors v: and B. The right-hand rule (Figs. 28-2a through c) tells us that the thumb of the right hand points in the direction : : of v: $ B when the fingers sweep v: into B . If q is positive, then (by Eq. 28-2) the : : : force FB has the same sign as v $ B and thus must be in the same direction; that : is, for positive q, FB is directed along the thumb (Fig. 28-2d). If q is negative, then

Force on positive particle

Cross v into B to get the new vector v ' B .

v 'B

v 'B

v

Force on negative particle

v 'B FB

B

B

(a)

B

(b)

B

(c)

B

(d ) :

Figure 28-2 (a)–(c) The right-hand rule (in which v: is swept into B through the smaller an: gle f between them) gives the direction of v: $ B as the direction of the thumb. (d) If q is : : : : positive, then the direction of FB ! qv $ B is in the direction of v: $ B. (e) If q is negative, : : then the direction of FB is opposite that of v: $ B.

FB (e )

805

806

CHAPTE R 28 MAG N ETIC FI E LDS :

:

the force FB and cross product v: $ B have opposite signs and thus must be in : opposite directions. For negative q, FB is directed opposite the thumb (Fig. 28-2e). Heads up: Neglect of this effect of negative q is a very common error on exams. Regardless of the sign of the charge, however,

e

e

:

v through a The force FB acting on a charged particle moving with velocity : : : magnetic field B is always perpendicular to v: and B .

e

:

Lawrence Berkeley Laboratory/Photo Researchers, Inc.

Figure 28-3 The tracks of two electrons (e#) and a positron (e%) in a bubble chamber that is immersed in a uniform magnetic field that is directed out of the plane of the page.

:

Thus, FB never has a component parallel to v: . This means that FB cannot change the particle’s speed v (and thus it cannot change the particle’s kinetic energy). The force can change only the direction of v: (and thus the direction of travel); : only in this sense can FB accelerate the particle. To develop a feeling for Eq. 28-2, consider Fig. 28-3, which shows some tracks left by charged particles moving rapidly through a bubble chamber. The chamber, which is filled with liquid hydrogen, is immersed in a strong uniform magnetic field that is directed out of the plane of the figure. An incoming gamma ray particle — which leaves no track because it is uncharged — transforms into an electron (spiral track marked e#) and a positron (track marked e%) while it knocks an electron out of a hydrogen atom (long track marked e#). Check with Eq. 28-2 and Fig. 28-2 that the three tracks made by these two negative particles and one positive particle curve in the proper directions. : Unit. The SI unit for B that follows from Eqs. 28-2 and 28-3 is the newton per coulomb-meter per second. For convenience, this is called the tesla (T):

Table 28-1 Some Approximate

1 tesla ! 1 T ! 1

Magnetic Fields

At surface of neutron star Near big electromagnet Near small bar magnet At Earth’s surface In interstellar space Smallest value in magnetically shielded room

108 T 1.5 T 10#2 T 10#4 T 10#10 T

newton . (coulomb)(meter/second)

Recalling that a coulomb per second is an ampere, we have 1T!1

newton N !1 . (coulomb/second)(meter) A&m

(28-4)

:

An earlier (non-SI) unit for B , still in common use, is the gauss (G), and 1 tesla ! 104 gauss.

10#14 T

(28-5)

Table 28-1 lists the magnetic fields that occur in a few situations. Note that Earth’s magnetic field near the planet’s surface is about 10#4 T (! 100 mT or 1 G).

Checkpoint 1 The figure shows three situations in which a charged particle with velocity v: travels through a uniform : magnetic field B . In each situation, what is the direction of the magnetic : force FB on the particle?

y

y

y B

B v

+

z

z B (a)

x

x

x v

(b)

z

v

(c)

Magnetic Field Lines We can represent magnetic fields with field lines, as we did for electric fields. Similar rules apply: (1) the direction of the tangent to a magnetic field line at : any point gives the direction of B at that point, and (2) the spacing of the lines : represents the magnitude of B — the magnetic field is stronger where the lines are closer together, and conversely.

:

28-1 MAG N ETIC FI E LDS AN D TH E DE FI N ITION OF B

Figure 28-4a shows how the magnetic field near a bar magnet (a permanent magnet in the shape of a bar) can be represented by magnetic field lines. The lines all pass through the magnet, and they all form closed loops (even those that are not shown closed in the figure). The external magnetic effects of a bar magnet are strongest near its ends, where the field lines are most closely spaced. Thus, the magnetic field of the bar magnet in Fig. 28-4b collects the iron filings mainly near the two ends of the magnet. Two Poles. The (closed) field lines enter one end of a magnet and exit the other end. The end of a magnet from which the field lines emerge is called the north pole of the magnet; the other end, where field lines enter the magnet, is called the south pole. Because a magnet has two poles, it is said to be a magnetic dipole. The magnets we use to fix notes on refrigerators are short bar magnets. Figure 28-5 shows two other common shapes for magnets: a horseshoe magnet and a magnet that has been bent around into the shape of a C so that the pole faces are facing each other. (The magnetic field between the pole faces can then be approximately uniform.) Regardless of the shape of the magnets, if we place two of them near each other we find:

807

N S

(a)

Opposite magnetic poles attract each other, and like magnetic poles repel each other.

When you hold two magnets near each other with your hands, this attraction or repulsion seems almost magical because there is no contact between the two to visibly justify the pulling or pushing. As we did with the electrostatic force between two charged particles, we explain this noncontact force in terms of a field that you cannot see, here the magnetic field. Earth has a magnetic field that is produced in its core by still unknown mechanisms. On Earth’s surface, we can detect this magnetic field with a compass, which is essentially a slender bar magnet on a low-friction pivot. This bar magnet, or this needle, turns because its north-pole end is attracted toward the Arctic region of Earth. Thus, the south pole of Earth’s magnetic field must be located toward the Arctic. Logically, we then should call the pole there a south pole. However, because we call that direction north, we are trapped into the statement that Earth has a geomagnetic north pole in that direction. With more careful measurement we would find that in the Northern Hemisphere, the magnetic field lines of Earth generally point down into Earth and toward the Arctic. In the Southern Hemisphere, they generally point up out of Earth and away from the Antarctic — that is, away from Earth’s geomagnetic south pole.

N N

S S

(a)

The field lines run from the north pole to the south pole.

(b)

Figure 28-5 (a) A horseshoe magnet and (b) a C-shaped magnet. (Only some of the external field lines are shown.)

(b) Courtesy Dr. Richard Cannon, Southeast Missouri State University, Cape Girardeau

Figure 28-4 (a) The magnetic field lines for a bar magnet. (b) A “cow magnet” — a bar magnet that is intended to be slipped down into the rumen of a cow to prevent accidentally ingested bits of scrap iron from reaching the cow’s intestines. The iron filings at its ends reveal the magnetic field lines.

808

CHAPTE R 28 MAG N ETIC FI E LDS

Sample Problem 28.01 Magnetic force on a moving charged particle :

A uniform magnetic field B, with magnitude 1.2 mT, is directed vertically upward throughout the volume of a laboratory chamber. A proton with kinetic energy 5.3 MeV enters the chamber, moving horizontally from south to north. What magnetic deflecting force acts on the proton as it enters the chamber? The proton mass is 1.67 $ 10#27 kg. (Neglect Earth’s magnetic field.) KEY IDEAS Because the proton is charged and moving through a mag: netic field, a magnetic force FB can act on it. Because the initial direction of the proton’s velocity is not along a magnetic : field line, FB is not simply zero. :

Magnitude: To find the magnitude of FB, we can use Eq. 28-3 (FB ! !q!vB sin f) provided we first find the proton’s speed v. We can find v from the given kinetic energy because K ! 12 mv 2. Solving for v, we obtain v!

a!

6.1 $ 10#15 N FB ! ! 3.7 $ 1012 m/s2. m 1.67 $ 10#27 kg :

Direction: To find the direction of FB, we use the fact that : : FB has the direction of the cross product qv: $ B. Because : the charge q is positive, FB must have the same direction as : v: $ B , which can be determined with the right-hand rule for cross products (as in Fig. 28-2d). We know that v: is di: rected horizontally from south to north and B is directed vertically up. The right-hand rule shows us that the deflect: ing force FB must be directed horizontally from west to east, as Fig. 28-6 shows. (The array of dots in the figure represents a magnetic field directed out of the plane of the figure. An array of Xs would have represented a magnetic field directed into that plane.) If the charge of the particle were negative, the magnetic deflecting force would be directed in the opposite direction — that is, horizontally from east to west. This is predicted automatically by Eq. 28-2 if we substitute a negative value for q.

2K (2)(5.3 MeV)(1.60 $ 10#13 J/MeV) ! A m A 1.67 $ 10#27 kg

N v

Path of proton

7

! 3.2 $ 10 m/s.

B

Equation 28-3 then yields FB ! !q!vB sin f ! (1.60 $ 10#19 C)(3.2 $ 107 m/s) $ (1.2 $ 10#3 T)(sin 90/) ! 6.1 $ 10#15 N.

W

+

FB

E

S

(Answer)

This may seem like a small force, but it acts on a particle of small mass, producing a large acceleration; namely,

Figure 28-6 An overhead view of a proton moving from south to v in a chamber. A magnetic field is directed north with velocity : vertically upward in the chamber, as represented by the array of dots (which resemble the tips of arrows). The proton is deflected toward the east.

Additional examples, video, and practice available at WileyPLUS

28-2 CROSSED FIELDS: DISCOVERY OF THE ELECTRON Learning Objectives After reading this module, you should be able to . . .

28.12 Describe the experiment of J. J. Thomson. 28.13 For a charged particle moving through a magnetic field and an electric field, determine the net force on the particle in both magnitude-angle notation and unit-vector notation.

28.14 In situations where the magnetic force and electric force on a particle are in opposite directions, determine the speeds at which the forces cancel, the magnetic force dominates, and the electric force dominates.

Key Ideas ● If a charged particle moves through a region containing both an electric field and a magnetic field, it can be affected by both an electric force and a magnetic force.

● If the fields are perpendicular to each other, they are said to

be crossed fields.

● If the forces are in opposite directions, a particular speed

will result in no deflection of the particle.

28-2 CROSSE D FI E LDS: DISCOVE RY OF TH E E LECTRON

809

Crossed Fields: Discovery of the Electron :

:

Both an electric field E and a magnetic field B can produce a force on a charged particle. When the two fields are perpendicular to each other, they are said to be crossed fields. Here we shall examine what happens to charged particles — namely, electrons — as they move through crossed fields. We use as our example the experiment that led to the discovery of the electron in 1897 by J. J. Thomson at Cambridge University. Two Forces. Figure 28-7 shows a modern, simplified version of Thomson’s experimental apparatus — a cathode ray tube (which is like the picture tube in an old-type television set). Charged particles (which we now know as electrons) are emitted by a hot filament at the rear of the evacuated tube and are accelerated by an applied potential difference V. After they pass through a slit in screen C, they : : form a narrow beam. They then pass through a region of crossed E and B fields, headed toward a fluorescent screen S, where they produce a spot of light (on a television screen the spot is part of the picture). The forces on the charged particles in the crossed-fields region can deflect them from the center of the screen. By controlling the magnitudes and directions of the fields, Thomson could thus control where the spot of light appeared on the screen. Recall that the force on a negatively charged particle due to an electric field is directed opposite the field. Thus, for the arrangement of Fig. 28-7, electrons are forced up the page by electric : : field E and down the page by magnetic field B ; that is, the forces are in opposition. Thomson’s procedure was equivalent to the following series of steps. 1. Set E ! 0 and B ! 0 and note the position of the spot on screen S due to the undeflected beam. : 2. Turn on E and measure the resulting beam deflection. :

:

3. Maintaining E , now turn on B and adjust its value until the beam returns to the undeflected position. (With the forces in opposition, they can be made to cancel.) We discussed the deflection of a charged particle moving through an electric : field E between two plates (step 2 here) in Sample Problem 22.04. We found that the deflection of the particle at the far end of the plates is y!

!q! EL2 , 2mv 2

(28-6)

where v is the particle’s speed, m its mass, and q its charge, and L is the length of the plates. We can apply this same equation to the beam of electrons in Fig. 28-7; if need be, we can calculate the deflection by measuring the deflection of the beam on screen S and then working back to calculate the deflection y at the end of the plates. (Because the direction of the deflection is set by the sign of the particle’s charge, Thomson was able to show that the particles that were lighting up his screen were negatively charged.)

+

Figure 28-7 A modern version of J. J. Thomson’s apparatus for measuring the ratio of mass to charge for the electron. An : electric field E is established by connecting a battery across the deflecting-plate termi: nals. The magnetic field B is set up by means of a current in a system of coils (not shown). The magnetic field shown is into the plane of the figure, as represented by the array of Xs (which resemble the feathered ends of arrows).

E

B

Spot of light

Filament Screen C Screen S

– Glass envelope

V To vacuum pump

810

CHAPTE R 28 MAG N ETIC FI E LDS

Canceling Forces. When the two fields in Fig. 28-7 are adjusted so that the two deflecting forces cancel (step 3), we have from Eqs. 28-1 and 28-3 !q!E ! !q!vB sin(90/) ! 2q2vB or

v!

E B

(28-7)

(opposite forces canceling).

Thus, the crossed fields allow us to measure the speed of the charged particles passing through them. Substituting Eq. 28-7 for v in Eq. 28-6 and rearranging yield m B2L2 ! , !q! 2yE

(28-8)

in which all quantities on the right can be measured.Thus, the crossed fields allow us to measure the ratio m/!q! of the particles moving through Thomson’s apparatus. (Caution: Equation 28-7 applies only when the electric and magnetic forces are in opposite directions.You might see other situations in the homework problems.) Thomson claimed that these particles are found in all matter. He also claimed that they are lighter than the lightest known atom (hydrogen) by a factor of more than 1000. (The exact ratio proved later to be 1836.15.) His m/!q! measurement, coupled with the boldness of his two claims, is considered to be the “discovery of the electron.”

Checkpoint 2 The figure shows four directions for the velocity vector v: of a positively charged particle moving : through a uniform electric field E (directed out of the page and represented with an encircled dot) : and a uniform magnetic field B . (a) Rank directions 1, 2, and 3 according to the magnitude of the net force on the particle, greatest first. (b) Of all four directions, which might result in a net force of zero?

2 v

B 1

v

+

E

v

3

v

4

28-3 CROSSED FIELDS: THE HALL EFFECT Learning Objectives After reading this module, you should be able to . . .

28.15 Describe the Hall effect for a metal strip carrying current, explaining how the electric field is set up and what limits its magnitude. 28.16 For a conducting strip in a Hall-effect situation, draw the vectors for the magnetic field and electric field. For the conduction electrons, draw the vectors for the velocity, magnetic force, and electric force. 28.17 Apply the relationship between the Hall potential

difference V, the electric field magnitude E, and the width of the strip d. 28.18 Apply the relationship between charge-carrier number density n, magnetic field magnitude B, current i, and Hall-effect potential difference V. 28.19 Apply the Hall-effect results to a conducting object moving through a uniform magnetic field, identifying the width across which a Hall-effect potential difference V is set up and calculating V.

Key Ideas ● When a uniform magnetic field B is applied to a conducting

strip carrying current i, with the field perpendicular to the direction of the current, a Hall-effect potential difference V is set up across the strip. ● The electric force

:

FE on the charge carriers is then balanced : by the magnetic force FB on them. ● The number density n of the charge carriers can then be determined from

Bi , Vle : where l is the thickness of the strip (parallel to B ). n!

When a conductor moves through a uniform magnetic field : B at speed v, the Hall-effect potential difference V across it is V ! vBd, : where d is the width perpendicular to both velocity v: and field B . ●

811

28-3 CROSSE D FI E LDS: TH E HALL E FFECT

Crossed Fields: The Hall Effect

i

V ! Ed.

(28-9)

By connecting a voltmeter across the width, we can measure the potential difference between the two edges of the strip. Moreover, the voltmeter can tell us which edge is at higher potential. For the situation of Fig. 28-8b, we would find that the left edge is at higher potential, which is consistent with our assumption that the charge carriers are negatively charged. For a moment, let us make the opposite assumption, that the charge carriers in current i are positively charged (Fig. 28-8c). Convince yourself that as these charge : carriers move from top to bottom in the strip, they are pushed to the right edge by FB and thus that the right edge is at higher potential. Because that last statement is contradicted by our voltmeter reading, the charge carriers must be negatively charged. Number Density. Now for the quantitative part. When the electric and magnetic forces are in balance (Fig. 28-8b), Eqs. 28-1 and 28-3 give us eE ! evdB.

(28-10)

From Eq. 26-7, the drift speed vd is i J ! , (28-11) ne neA in which J (! i/A) is the current density in the strip, A is the cross-sectional area of the strip, and n is the number density of charge carriers (number per unit volume). In Eq. 28-10, substituting for E with Eq. 28-9 and substituting for vd with Eq. 28-11, we obtain vd !

n!

Bi , Vle

(28-12)

d + ×

×

×

×

×

× vd

×

×

×

×

F ×B ×

×

×

×

× ×

×

E

× × B + × ×

×

×

×

×

× +× vd

×

×

×

×

×

+ FE +

Low

×

High

× B ×

×

F ×B ×

×

×

i

i

(a )

(b )

×

i

+

E ×

×

×

× +

B ×

×

×

×

×

×+ ×

× ×

FE

+

×

High

Low

As we just discussed, a beam of electrons in a vacuum can be deflected by a magnetic field. Can the drifting conduction electrons in a copper wire also be deflected by a magnetic field? In 1879, Edwin H. Hall, then a 24-year-old graduate student at the Johns Hopkins University, showed that they can. This Hall effect allows us to find out whether the charge carriers in a conductor are positively or negatively charged. Beyond that, we can measure the number of such carriers per unit volume of the conductor. Figure 28-8a shows a copper strip of width d, carrying a current i whose conventional direction is from the top of the figure to the bottom. The charge carriers are electrons and, as we know, they drift (with drift speed vd) in the opposite direction, from bottom to top. At the instant shown in Fig. 28-8a, : an external magnetic field B , pointing into the plane of the figure, has just : been turned on. From Eq. 28-2 we see that a magnetic deflecting force FB will act on each drifting electron, pushing it toward the right edge of the strip. As time goes on, electrons move to the right, mostly piling up on the right edge of the strip, leaving uncompensated positive charges in fixed positions at the left edge. The separation of positive charges on the left edge and negative charges : on the right edge produces an electric field E within the strip, pointing from left : to right in Fig. 28-8b. This field exerts an electric force FE on each electron, tending to push it to the left. Thus, this electric force on the electrons, which opposes the magnetic force on them, begins to build up. Equilibrium. An equilibrium quickly develops in which the electric force on each electron has increased enough to match the magnetic force. When this hap: : pens, as Fig. 28-8b shows, the force due to B and the force due to E are in balance. The drifting electrons then move along the strip toward the top of the page at velocity v:d with no further collection of electrons on the right edge of the strip and : thus no further increase in the electric field E . A Hall potential difference V is associated with the electric field across strip width d. From Eq. 24-21, the magnitude of that potential difference is

i

+ FB × × × + × v × × d

i (c )

Figure 28-8 A strip of copper carrying a cur: rent i is immersed in a magnetic field B. (a) The situation immediately after the magnetic field is turned on. The curved path that will then be taken by an electron is shown. (b) The situation at equilibrium, which quickly follows. Note that negative charges pile up on the right side of the strip, leaving uncompensated positive charges on the left. Thus, the left side is at a higher potential than the right side. (c) For the same current direction, if the charge carriers were positively charged, they would pile up on the right side, and the right side would be at the higher potential.

812

CHAPTE R 28 MAG N ETIC FI E LDS

in which l (! A/d) is the thickness of the strip. With this equation we can find n from measurable quantities. Drift Speed. It is also possible to use the Hall effect to measure directly the drift speed vd of the charge carriers, which you may recall is of the order of centimeters per hour. In this clever experiment, the metal strip is moved mechanically through the magnetic field in a direction opposite that of the drift velocity of the charge carriers. The speed of the moving strip is then adjusted until the Hall potential difference vanishes. At this condition, with no Hall effect, the velocity of the charge carriers with respect to the laboratory frame must be zero, so the velocity of the strip must be equal in magnitude but opposite the direction of the velocity of the negative charge carriers. Moving Conductor. When a conductor begins to move at speed v through a magnetic field, its conduction electrons do also. They are then like the moving : conduction electrons in the current in Figs. 28-8a and b, and an electric field E and potential difference V are quickly set up. As with the current, equilibrium of the electric and magnetic forces is established, but we now write that condition in terms of the conductor’s speed v instead of the drift speed vd in a current as we did in Eq. 28-10: eE ! evB. Substituting for E with Eq. 28-9, we find that the potential difference is (28-13)

V ! vBd.

Such a motion-caused circuit potential difference can be of serious concern in some situations, such as when a conductor in an orbiting satellite moves through Earth’s magnetic field. However, if a conducting line (said to be an electrodynamic tether) dangles from the satellite, the potential produced along the line might be used to maneuver the satellite.

Sample Problem 28.02 Potential difference set up across a moving conductor Figure 28-9a shows a solid metal cube, of edge length d ! 1.5 cm, moving in the positive y direction at a constant velocity v: of magnitude 4.0 m/s. The cube moves through a : uniform magnetic field B of magnitude 0.050 T in the positive z direction. (a) Which cube face is at a lower electric potential and which is at a higher electric potential because of the motion through the field? KEY IDEA :

Because the cube is moving through a magnetic field B , a : magnetic force FB acts on its charged particles, including its conduction electrons. Reasoning: When the cube first begins to move through the magnetic field, its electrons do also. Because each electron has charge q and is moving through a magnetic field : with velocity v:, the magnetic force FB acting on the electron is given by Eq. 28-2. Because q is negative, the direction of : : FB is opposite the cross product v: $ B , which is in the posi-

:

tive direction of the x axis (Fig. 28-9b). Thus, FB acts in the negative direction of the x axis, toward the left face of the cube (Fig. 28-9c). Most of the electrons are fixed in place in the atoms of the cube. However, because the cube is a metal, it contains conduction electrons that are free to move. Some of : those conduction electrons are deflected by FB to the left cube face, making that face negatively charged and leaving the right face positively charged (Fig. 28-9d). This : charge separation produces an electric field E directed from the positively charged right face to the negatively charged left face (Fig. 28-9e). Thus, the left face is at a lower electric potential, and the right face is at a higher electric potential. (b) What is the potential difference between the faces of higher and lower electric potential? KEY IDEAS :

1. The electric field E created by the charge separation : : produces an electric force FE ! qE on each electron

28-3 CROSSE D FI E LDS: TH E HALL E FFECT

This is the crossproduct result.

y

y

v

d

z

y

d

FB

(b)

(a)

" " "

v !B

x

x (c)

This is the resulting electric field.

y

y " " "

! ! !

E

(e)

x

More migration creates a greater electric field.

FB

FE

x

(f )

" " " " "

x

The forces now balance. No more electrons move to the left face. y

y

E

! ! ! (d)

B

The weak electric field creates a weak electric force.

A

Electrons are forced to the left face, leaving the right face positive. y

v !B

x d

This is the magnetic force on an electron.

813

E

! ! ! ! !

FB

x

FE

x

(h)

(g)

Figure 28-9 (a) A solid metal cube moves at constant velocity through a uniform magnetic field. (b)–(d) In these front views, the magnetic force acting on an electron forces the electron to the left face, making that face negative and leaving the opposite face positive. (e)–(f) The resulting weak electric field creates a weak electric force on the next electron, but it too is forced to the left face. Now (g) the electric field is stronger and (h) the electric force matches the magnetic force.

(Fig. 28-9f ). Because q is negative, this force is directed : opposite the field E — that is, rightward. Thus on each : : electron, FE acts toward the right and FB acts toward the left. 2. When the cube had just begun to move through the magnetic field and the charge separation had just begun, the : magnitude of E began to increase from zero. Thus, the : magnitude of FE also began to increase from zero and was : initially smaller than the magnitude of FB. During this early stage, the net force on any electron was dominated : by FB, which continuously moved additional electrons to the left cube face, increasing the charge separation between the left and right cube faces (Fig. 28-9g). 3. However, as the charge separation increased, eventually magnitude FE became equal to magnitude FB (Fig. 28-9h). Because the forces were in opposite directions, the net force on any electron was then zero, and no additional electrons were moved to the left cube face. Thus, the : magnitude of FE could not increase further, and the electrons were then in equilibrium.

Calculations: We seek the potential difference V between the left and right cube faces after equilibrium was reached (which occurred quickly). We can obtain V with Eq. 28-9 (V ! Ed ) provided we first find the magnitude E of the electric field at equilibrium. We can do so with the equation for the balance of forces (FE ! FB). For FE, we substitute !q!E, and then for FB, we substitute !q!vB sin f from Eq. 28-3. From Fig. 28-9a, we see that the angle f between velocity vector v: and magnetic field vector : B is 90"; thus sin f ! 1 and FE ! FB yields !q!E ! !q!vB sin 90" ! !q!vB. This gives us E ! vB; so V ! Ed becomes V ! vBd. Substituting known values tells us that the potential difference between the left and right cube faces is V ! (4.0 m/s)(0.050 T)(0.015 m) ! 0.0030 V ! 3.0 mV.

Additional examples, video, and practice available at WileyPLUS

(Answer)

814

CHAPTE R 28 MAG N ETIC FI E LDS

28-4 A CIRCULATING CHARGED PARTICLE Learning Objectives After reading this module, you should be able to . . .

28.20 For a charged particle moving through a uniform magnetic field, identify under what conditions it will travel in a straight line, in a circular path, and in a helical path. 28.21 For a charged particle in uniform circular motion due to a magnetic force, start with Newton’s second law and derive an expression for the orbital radius r in terms of the field magnitude B and the particle’s mass m, charge magnitude q, and speed v. 28.22 For a charged particle moving along a circular path in a magnetic field, calculate and relate speed, centripetal force, centripetal acceleration, radius, period, frequency, and angular frequency, and identify which of the quantities do not depend on speed. 28.23 For a positive particle and a negative particle moving

along a circular path in a uniform magnetic field, sketch the path and indicate the magnetic field vector, the velocity vector, the result of the cross product of the velocity and field vectors, and the magnetic force vector. 28.24 For a charged particle moving in a helical path in a magnetic field, sketch the path and indicate the magnetic field, the pitch, the radius of curvature, the velocity component parallel to the field, and the velocity component perpendicular to the field. 28.25 For helical motion in a magnetic field, apply the relationship between the radius of curvature and one of the velocity components. 28.26 For helical motion in a magnetic field, identify pitch p and relate it to one of the velocity components.

Key Ideas !q! moving with velocity v perpendicular to a uniform magnetic : field B will travel in a circle. ● Applying Newton’s second law to the circular motion yields ● A charged particle with mass m and charge magnitude :

mv 2 !q!vB ! , r from which we find the radius r of the circle to be

mv . !q!B ● The frequency of revolution f, the angular frequency v, and the period of the motion T are given by r!

f!

1 !q!B v ! ! . 2p T 2pm

● If the velocity of the particle has a component parallel to the mag:

netic field, the particle moves in a helical path about field vector B.

A Circulating Charged Particle If a particle moves in a circle at constant speed, we can be sure that the net force acting on the particle is constant in magnitude and points toward the center of the circle, always perpendicular to the particle’s velocity. Think of a stone tied to a string and whirled in a circle on a smooth horizontal surface, or of a satellite moving in a circular orbit around Earth. In the first case, the tension in the string provides the necessary force and centripetal acceleration. In the second case, Earth’s gravitational attraction provides the force and acceleration. Figure 28-10 shows another example: A beam of electrons is projected into a chamber by an electron gun G. The electrons enter in the plane of the page with : speed v and then move in a region of uniform magnetic field B directed out of : : that plane. As a result, a magnetic force FB ! qv: $ B continuously deflects the : electrons, and because v: and B are always perpendicular to each other, this deflection causes the electrons to follow a circular path. The path is visible in the photo because atoms of gas in the chamber emit light when some of the circulating electrons collide with them. We would like to determine the parameters that characterize the circular motion of these electrons, or of any particle of charge magnitude #q# and mass m : moving perpendicular to a uniform magnetic field B at speed v. From Eq. 28-3, the force acting on the particle has a magnitude of !q! vB. From Newton’s second : law (F ! ma: ) applied to uniform circular motion (Eq. 6-18), F!m

v2 , r

(28-14)

28-4 A CI RCU L ATI NG CHARG E D PARTICLE

v B FB

G

Courtesy Jearl Walker

Figure 28-10 Electrons circulating in a chamber containing gas at low pressure (their path : is the glowing circle). A uniform magnetic field B , pointing directly out of the plane of : the page, fills the chamber. Note the radially directed magnetic force FB; for circular motion to : occur, FB must point toward the center of the circle. Use the right-hand rule for cross products to : : : confirm that FB ! qv: $ B gives FB the proper direction. (Don’t forget the sign of q.)

we have !q !vB !

mv 2 . r

(28-15)

Solving for r, we find the radius of the circular path as r!

mv !q!B

(radius).

(28-16)

The period T (the time for one full revolution) is equal to the circumference divided by the speed: T!

2p 2pr ! v v

mv 2pm ! !q!B !q!B

(period).

(28-17)

The frequency f (the number of revolutions per unit time) is f!

!q!B 1 ! T 2pm

(frequency).

(28-18)

The angular frequency v of the motion is then v ! 2p f !

!q!B m

(angular frequency).

(28-19)

The quantities T, f, and v do not depend on the speed of the particle (provided the speed is much less than the speed of light). Fast particles move in large circles and slow ones in small circles, but all particles with the same charge-to-mass ratio !q!/m take the same time T (the period) to complete one round trip. Using Eq. 28-2, : you can show that if you are looking in the direction of B , the direction of rotation for a positive particle is always counterclockwise, and the direction for a negative particle is always clockwise.

815

816

CHAPTE R 28 MAG N ETIC FI E LDS

The velocity component perpendicular to the field causes circling, which is stretched upward by the parallel component.

p B

v

φ

B

v

v⎪⎪

φ

+

v⊥

v⎪⎪

B

φ

v⊥

q (a )

Figure 28-11 (a) A charged particle moves in : a uniform magnetic field B , the particle’s : velocity v making an angle f with the field direction. (b) The particle follows a helical path of radius r and pitch p. (c) A charged particle spiraling in a nonuniform magnetic field. (The particle can become trapped in this magnetic bottle, spiraling back and forth between the strong field regions at either end.) Note that the magnetic force vectors at the left and right sides have a component pointing toward the center of the figure.

B

Particle

q

+

Spiral path B

F

FB

FB

r (c )

(b )

Helical Paths If the velocity of a charged particle has a component parallel to the (uniform) magnetic field, the particle will move in a helical path about the direction of the field vector. Figure 28-11a, for example, shows the velocity vector v: of such a particle : resolved into two components, one parallel to B and one perpendicular to it: v, ! v cos f

and

(28-20)

v" ! v sin f.

The parallel component determines the pitch p of the helix — that is, the distance between adjacent turns (Fig. 28-11b). The perpendicular component determines the radius of the helix and is the quantity to be substituted for v in Eq. 28-16. Figure 28-11c shows a charged particle spiraling in a nonuniform magnetic field. The more closely spaced field lines at the left and right sides indicate that the magnetic field is stronger there. When the field at an end is strong enough, the particle “reflects” from that end.

Checkpoint 3 The figure here shows the circular paths of two particles that travel : at the same speed in a uniform magnetic field B , which is directed into the page. One particle is a proton; the other is an electron (which is less massive). (a) Which particle follows the smaller circle, and (b) does that particle travel clockwise or counterclockwise?

B

Sample Problem 28.03 Helical motion of a charged particle in a magnetic field An electron with a kinetic energy of 22.5 eV moves into a : region of uniform magnetic field B of magnitude 4.55 $ : 10%4 T. The angle between the directions of B and the electron’s velocity v: is 65.5°. What is the pitch of the helical path taken by the electron? KEY IDEAS (1) The pitch p is the distance the electron travels parallel to : the magnetic field B during one period T of circulation. (2) The period T is given by Eq. 28-17 for any nonzero angle : between v:and B .

Calculations: Using Eqs. 28-20 and 28-17, we find p ! v,T ! (v cos f)

2p m . ! q !B

(28-21)

Calculating the electron’s speed v from its kinetic energy, we find that v ! 2.81 $ 106 m/s, and so Eq. 28-21 gives us p ! (2.81 $ 106 m/s)(cos 65.5") $

2p (9.11 $ 10%31 kg) (1.60 $ 10%19 C)(4.55 $ 10%4 T)

! 9.16 cm.

Additional examples, video, and practice available at WileyPLUS

(Answer)

817

28-5 CYCLOTRONS AN D SYNCH ROTRONS

Sample Problem 28.04 Uniform circular motion of a charged particle in a magnetic field Figure 28-12 shows the essentials of a mass spectrometer, which can be used to measure the mass of an ion; an ion of mass m (to be measured) and charge q is produced in source S. The initially stationary ion is accelerated by the electric field due to a potential difference V. The ion leaves S and en: ters a separator chamber in which a uniform magnetic field B is perpendicular to the path of the ion. A wide detector lines : the bottom wall of the chamber, and the B causes the ion to move in a semicircle and thus strike the detector. Suppose that B ! 80.000 mT, V ! 1000.0 V, and ions of charge q ! &1.6022 $ 10%19 C strike the detector at a point that lies at x ! 1.6254 m. What is the mass m of the individual ions, in atomic mass units (Eq. 1-7: 1 u ! 1.6605 $ 10%27 kg)?

Figure 28-12 A positive ion is accelerated from its source S by a potential difference V, enters a chamber of uniform : magnetic field B , travels through a semicircle of radius r, – and strikes a detector V + at a distance x.

KEY IDEAS

we get

(1) Because the (uniform) magnetic field causes the (charged) ion to follow a circular path, we can relate the ion’s mass m to the path’s radius r with Eq. 28-16 (r ! mv/ !q!B). From Fig. 28-12 we see that r ! x/2 (the radius is half the diameter). From the problem statement, we know the magnitude B of the magnetic field. However, we lack the ion’s speed v in the magnetic field after the ion has been accelerated due to the potential difference V. (2) To relate v and V, we use the fact that mechanical energy (Emec ! K & U ) is conserved during the acceleration. Finding speed: When the ion emerges from the source, its kinetic energy is approximately zero. At the end of the acceleration, its kinetic energy is 12mv 2. Also, during the acceleration, the positive ion moves through a change in potential of %V. Thus, because the ion has positive charge q, its potential energy changes by %qV. If we now write the conservation of mechanical energy as

B r

Detector

x +q

S

1 2 2 mv

or

v!

% qV ! 0

2qV . A m

(28-22)

Finding mass: Substituting this value for v into Eq. 28-16 gives us m 2qV 1 2mV mv ! . r! ! qB qB A m B A q Thus,

x ! 2r !

2mV 2 . B A q

Solving this for m and substituting the given data yield m! !

B2qx 2 8V (0.080000 T)2(1.6022 $ 10%19 C)(1.6254 m)2 8(1000.0 V)

! 3.3863 $ 10%25 kg ! 203.93 u.

'K & 'U ! 0,

(Answer)

Additional examples, video, and practice available at WileyPLUS

28-5 CYCLOTRONS AND SYNCHROTRONS Learning Objectives After reading this module, you should be able to . . .

28.27 Describe how a cyclotron works, and in a sketch indicate a particle’s path and the regions where the kinetic energy is increased. 28.28 Identify the resonance condition.

28.29 For a cyclotron, apply the relationship between the particle’s mass and charge, the magnetic field, and the frequency of circling. 28.30 Distinguish between a cyclotron and a synchrotron.

Key Ideas In a cyclotron, charged particles are accelerated by electric forces as they circle in a magnetic field.



● A synchrotron is needed for particles accelerated to nearly

the speed of light.

818

CHAPTE R 28 MAG N ETIC FI E LDS

The protons spiral outward in a cyclotron, picking up energy in the gap. Dee

Dee

S Beam

Deflector plate Oscillator

Figure 28-13 The elements of a cyclotron, showing the particle source S and the dees. A uniform magnetic field is directed up from the plane of the page. Circulating protons spiral outward within the hollow dees, gaining energy every time they cross the gap between the dees.

Cyclotrons and Synchrotrons Beams of high-energy particles, such as high-energy electrons and protons, have been enormously useful in probing atoms and nuclei to reveal the fundamental structure of matter. Such beams were instrumental in the discovery that atomic nuclei consist of protons and neutrons and in the discovery that protons and neutrons consist of quarks and gluons. Because electrons and protons are charged, they can be accelerated to the required high energy if they move through large potential differences. The required acceleration distance is reasonable for electrons (low mass) but unreasonable for protons (greater mass). A clever solution to this problem is first to let protons and other massive particles move through a modest potential difference (so that they gain a modest amount of energy) and then use a magnetic field to cause them to circle back and move through a modest potential difference again. If this procedure is repeated thousands of times, the particles end up with a very large energy. Here we discuss two accelerators that employ a magnetic field to repeatedly bring particles back to an accelerating region, where they gain more and more energy until they finally emerge as a high-energy beam.

The Cyclotron Figure 28-13 is a top view of the region of a cyclotron in which the particles (protons, say) circulate. The two hollow D-shaped objects (each open on its straight edge) are made of sheet copper. These dees, as they are called, are part of an electrical oscillator that alternates the electric potential difference across the gap between the dees. The electrical signs of the dees are alternated so that the electric field in the gap alternates in direction, first toward one dee and then toward the other dee, back and forth. The dees are immersed in a large magnetic field directed out of the plane of the page. The magnitude B of this field is set via a control on the electromagnet producing the field. Suppose that a proton, injected by source S at the center of the cyclotron in Fig. 28-13, initially moves toward a negatively charged dee. It will accelerate toward this dee and enter it. Once inside, it is shielded from electric fields by the copper walls of the dee; that is, the electric field does not enter the dee. The magnetic field, however, is not screened by the (nonmagnetic) copper dee, so the proton moves in a circular path whose radius, which depends on its speed, is given by Eq. 28-16 (r ! mv/ !q!B). Let us assume that at the instant the proton emerges into the center gap from the first dee, the potential difference between the dees is reversed. Thus, the proton again faces a negatively charged dee and is again accelerated. This process continues, the circulating proton always being in step with the oscillations of the dee potential, until the proton has spiraled out to the edge of the dee system. There a deflector plate sends it out through a portal. Frequency. The key to the operation of the cyclotron is that the frequency f at which the proton circulates in the magnetic field (and that does not depend on its speed) must be equal to the fixed frequency fosc of the electrical oscillator, or f ! fosc

(resonance condition).

(28-23)

This resonance condition says that, if the energy of the circulating proton is to increase, energy must be fed to it at a frequency fosc that is equal to the natural frequency f at which the proton circulates in the magnetic field. Combining Eqs. 28-18 ( f ! !q!B/2pm) and 28-23 allows us to write the resonance condition as !q!B ! 2pmfosc. (28-24) The oscillator (we assume) is designed to work at a single fixed frequency fosc. We

819

28-5 CYCLOTRONS AN D SYNCH ROTRONS

then “tune” the cyclotron by varying B until Eq. 28-24 is satisfied, and then many protons circulate through the magnetic field, to emerge as a beam.

The Proton Synchrotron At proton energies above 50 MeV, the conventional cyclotron begins to fail because one of the assumptions of its design — that the frequency of revolution of a charged particle circulating in a magnetic field is independent of the particle’s speed — is true only for speeds that are much less than the speed of light. At greater proton speeds (above about 10% of the speed of light), we must treat the problem relativistically.According to relativity theory, as the speed of a circulating proton approaches that of light, the proton’s frequency of revolution decreases steadily. Thus, the proton gets out of step with the cyclotron’s oscillator — whose frequency remains fixed at f osc — and eventually the energy of the still circulating proton stops increasing. There is another problem. For a 500 GeV proton in a magnetic field of 1.5 T, the path radius is 1.1 km. The corresponding magnet for a conventional cyclotron of the proper size would be impossibly expensive, the area of its pole faces being about 4 $ 106 m2. The proton synchrotron is designed to meet these two difficulties. The magnetic field B and the oscillator frequency fosc, instead of having fixed values as in the conventional cyclotron, are made to vary with time during the accelerating cycle. When this is done properly, (1) the frequency of the circulating protons remains in step with the oscillator at all times, and (2) the protons follow a circular — not a spiral — path. Thus, the magnet need extend only along that circular path, not over some 4 $ 106 m2. The circular path, however, still must be large if high energies are to be achieved.

Sample Problem 28.05 Accelerating a charged particle in a cyclotron Suppose a cyclotron is operated at an oscillator frequency of 12 MHz and has a dee radius R ! 53 cm. (a) What is the magnitude of the magnetic field needed for deuterons to be accelerated in the cyclotron? The deuteron mass is m ! 3.34 $ 10%27 kg (twice the proton mass). KEY IDEA For a given oscillator frequency fosc, the magnetic field magnitude B required to accelerate any particle in a cyclotron depends on the ratio m/ !q! of mass to charge for the particle, according to Eq. 28-24 ( !q!B ! 2pmfosc). Calculation: For deuterons and the oscillator frequency fosc ! 12 MHz, we find 2pmfosc (2p)(3.34 $ 10%27 kg)(12 $ 106 s%1) B! ! !q! 1.60 $ 10 %19 C ! 1.57 T % 1.6 T.

(Answer)

Note that, to accelerate protons, B would have to be reduced by a factor of 2, provided the oscillator frequency remained fixed at 12 MHz. (b) What is the resulting kinetic energy of the deuterons?

KEY IDEAS (1) The kinetic energy (12 mv 2) of a deuteron exiting the cyclotron is equal to the kinetic energy it had just before exiting, when it was traveling in a circular path with a radius approximately equal to the radius R of the cyclotron dees. (2) We can find the speed v of the deuteron in that circular path with Eq. 28-16 (r ! mv/#q#B). Calculations: Solving that equation for v, substituting R for r, and then substituting known data, we find v!

R !q! B (0.53 m)(1.60 $ 10%19 C)(1.57 T) ! m 3.34 $ 10%27 kg

! 3.99 $ 107 m/s. This speed corresponds to a kinetic energy of K ! 12 mv 2 ! 12 (3.34 $ 10%27 kg)(3.99 $ 107 m/s)2 ! 2.7 $ 10%12 J, or about 17 MeV.

Additional examples, video, and practice available at WileyPLUS

(Answer)

820

CHAPTE R 28 MAG N ETIC FI E LDS

28-6 MAGNETIC FORCE ON A CURRENT-CARRYING WIRE Learning Objectives After reading this module, you should be able to . . .

28.31 For the situation where a current is perpendicular to a magnetic field, sketch the current, the direction of the magnetic field, and the direction of the magnetic force on the current (or wire carrying the current). 28.32 For a current in a magnetic field, apply the relationship between the magnetic force magnitude FB, the current i, the length of the wire L, and the angle f between the : : length vector L and the field vector B . 28.33 Apply the right-hand rule for cross products to find

the direction of the magnetic force on a current in a magnetic field. 28.34 For a current in a magnetic field, calculate the : magnetic force FB with a cross product of the length : : vector L and the field vector B , in magnitude-angle and unit-vector notations. 28.35 Describe the procedure for calculating the force on a current-carrying wire in a magnetic field if the wire is not straight or if the field is not uniform.

Key Ideas ● A straight wire carrying a current i in a uniform magnetic

field experiences a sideways force : : : FB ! iL $ B. ● The force acting on a current element i

B

i

B

B

i=0

(a)

i

(b)

:

:

:

dFB ! i dL $ B. :

dL in a magnetic

● The direction of the length vector

:

:

L or dL is that of the

current i.

Magnetic Force on a Current-Carrying Wire

A force acts on a current through a B field.

i

field is

i

(c)

Figure 28-14 A flexible wire passes between the pole faces of a magnet (only the farther pole face is shown). (a) Without current in the wire, the wire is straight. (b) With upward current, the wire is deflected rightward. (c) With downward current, the deflection is leftward. The connections for getting the current into the wire at one end and out of it at the other end are not shown.

We have already seen (in connection with the Hall effect) that a magnetic field exerts a sideways force on electrons moving in a wire. This force must then be transmitted to the wire itself, because the conduction electrons cannot escape sideways out of the wire. In Fig. 28-14a, a vertical wire, carrying no current and fixed in place at both ends, extends through the gap between the vertical pole faces of a magnet. The magnetic field between the faces is directed outward from the page. In Fig. 28-14b, a current is sent upward through the wire; the wire deflects to the right. In Fig. 28-14c, we reverse the direction of the current and the wire deflects to the left. Figure 28-15 shows what happens inside the wire of Fig. 28-14b. We see one of the conduction electrons, drifting downward with an assumed drift speed vd. : Equation 28-3, in which we must put f ! 90", tells us that a force FB of magnitude evdB must act on each such electron. From Eq. 28-2 we see that this force must be directed to the right. We expect then that the wire as a whole will experience a force to the right, in agreement with Fig. 28-14b. If, in Fig. 28-15, we were to reverse either the direction of the magnetic field or the direction of the current, the force on the wire would reverse, being directed now to the left. Note too that it does not matter whether we consider negative charges B

FB

L i

Figure 28-15 A close-up view of a section of the wire of Fig. 28-14b. The current direction is upward, which means that electrons drift downward. A magnetic field that emerges from the plane of the page causes the electrons and the wire to be deflected to the right.

vd x

x

28-6 MAG N ETIC FORCE ON A CU R R E NT-CAR RYI NG WI R E

821

drifting downward in the wire (the actual case) or positive charges drifting upward. The direction of the deflecting force on the wire is the same. We are safe then in dealing with a current of positive charge, as we usually do in dealing with circuits. Find the Force. Consider a length L of the wire in Fig. 28-15. All the conduction electrons in this section of wire will drift past plane xx in Fig. 28-15 in a time t ! L/vd. Thus, in that time a charge given by q ! it ! i

L vd

will pass through that plane. Substituting this into Eq. 28-3 yields FB ! qvd B sin f ! or

iL v B sin 90" vd d (28-25)

FB ! iLB.

Note that this equation gives the magnetic force that acts on a length L of straight : wire carrying a current i and immersed in a uniform magnetic field B that is perpendicular to the wire. If the magnetic field is not perpendicular to the wire, as in Fig. 28-16, the magnetic force is given by a generalization of Eq. 28-25: :

:

:

FB ! iL $ B

(force on a current).

(28-26)

:

Here L is a length vector that has magnitude L and is directed along the wire segment in the direction of the (conventional) current. The force magnitude FB is FB ! iLB sin f,

(28-27) :

:

:

where f is the angle between the directions of L and B . The direction of FB is : : that of the cross product L $ B because we take current i to be a positive quan: tity. Equation 28-26 tells us that FB is always perpendicular to the plane defined : : by vectors L and B , as indicated in Fig. 28-16. Equation 28-26 is equivalent to Eq. 28-2 in that either can be taken as the : : defining equation for B . In practice, we define B from Eq. 28-26 because it is much easier to measure the magnetic force acting on a wire than that on a single moving charge. Crooked Wire. If a wire is not straight or the field is not uniform, we can imagine the wire broken up into small straight segments and apply Eq. 28-26 to each segment. The force on the wire as a whole is then the vector sum of all the forces on the segments that make it up. In the differential limit, we can write :

:

:

(28-28)

dFB ! i dL $ B,

and we can find the resultant force on any given arrangement of currents by integrating Eq. 28-28 over that arrangement. In using Eq. 28-28, bear in mind that there is no such thing as an isolated current-carrying wire segment of length dL. There must always be a way to introduce the current into the segment at one end and take it out at the other end.

Checkpoint 4

y

The figure shows a current i through a wire in a uni: form magnetic field B , as well as the magnetic force : FB acting on the wire. The field is oriented so that the force is maximum. In what direction is the field?

i x z

FB

The force is perpendicular to both the field and the length. FB

i

φ

B

L

Figure 28-16 A wire carrying current i makes : an angle f with magnetic field B . The wire has length L in the field and length vector : L (in the direction of the current). A mag: : : netic force FB ! iL $ B acts on the wire.

822

CHAPTE R 28 MAG N ETIC FI E LDS

Sample Problem 28.06 Magnetic force on a wire carrying current A straight, horizontal length of copper wire has a current i ! 28 A through it. What are the magnitude and direction : of the minimum magnetic field B needed to suspend the wire — that is, to balance the gravitational force on it? The linear density (mass per unit length) of the wire is 46.6 g/m.

FB L

Figure 28-17 A wire (shown in cross section) carrying current out of the page.

B mg

KEY IDEAS :

(1) Because the wire carries a current, a magnetic force FB : can act on the wire if we place it in a magnetic field B . To : balance the downward gravitational force Fg on the wire, we : want FB to be directed upward (Fig. 28-17). (2) The direction : : of FB is related to the directions of B and the wire’s length : : : : vector L by Eq. 28-26 (FB ! iL $ B ).

:

where mg is the magnitude of Fg and m is the mass of the : wire. We also want the minimal field magnitude B for FB to : balance Fg. Thus, we need to maximize sin f in Eq. 28-29. To : do so, we set f ! 90", thereby arranging for B to be perpendicular to the wire. We then have sin f ! 1, so Eq. 28-29 yields B!

:

Calculations: Because L is directed horizontally (and the current is taken to be positive), Eq. 28-26 and the right: hand rule for cross products tell us that B must be horizontal and rightward (in Fig. 28-17) to give the required : upward FB. : The magnitude of FB is FB ! iLB sin f (Eq. 28-27). : : Because we want FB to balance Fg, we want iLB sin f ! mg,

(28-29)

(m/L)g mg ! . iL sin f i

(28-30)

We write the result this way because we know m/L, the linear density of the wire. Substituting known data then gives us (46.6 $ 10%3 kg/m)(9.8 m/s2) 28 A ! 1.6 $ 10%2 T.

B!

(Answer)

This is about 160 times the strength of Earth’s magnetic field.

Additional examples, video, and practice available at WileyPLUS

28-7 TORQUE ON A CURRENT LOOP Learning Objectives After reading this module, you should be able to . . .

28.36 Sketch a rectangular loop of current in a magnetic field, indicating the magnetic forces on the four sides, the direction of the current, the normal vector n:, and the direction in which a torque from the forces tends to rotate the loop.

28.37 For a current-carrying coil in a magnetic field, apply the relationship between the torque magnitude t, the number of turns N, the area of each turn A, the current i, the magnetic field magnitude B, and the angle u between : the normal vector n: and the magnetic field vector B.

Key Ideas ● Various magnetic forces act on the sections of a currentcarrying coil lying in a uniform external magnetic field, but the net force is zero. ● The net torque acting on the coil has a magnitude given by t ! NiAB sin u,

where N is the number of turns in the coil, A is the area of each turn, i is the current, B is the field magnitude, and u is : the angle between the magnetic field B and the normal vector to the coil n:.

Torque on a Current Loop Much of the world’s work is done by electric motors. The forces behind this work are the magnetic forces that we studied in the preceding section — that is, the forces that a magnetic field exerts on a wire that carries a current.

28-7 TORQU E ON A CU R R E NT LOOP

Figure 28-18 shows a simple motor, consisting of a single current-carrying : : : loop immersed in a magnetic field B. The two magnetic forces F and %F produce a torque on the loop, tending to rotate it about its central axis. Although many essential details have been omitted, the figure does suggest how the action of a magnetic field on a current loop produces rotary motion. Let us analyze that action. Figure 28-19a shows a rectangular loop of sides a and b, carrying current : i through uniform magnetic field B . We place the loop in the field so that its long sides, labeled 1 and 3, are perpendicular to the field direction (which is into the page), but its short sides, labeled 2 and 4, are not. Wires to lead the current into and out of the loop are needed but, for simplicity, are not shown. To define the orientation of the loop in the magnetic field, we use a normal vector n: that is perpendicular to the plane of the loop. Figure 28-19b shows a right-hand rule for finding the direction of n: . Point or curl the fingers of your right hand in the direction of the current at any point on the loop. Your extended thumb then points in the direction of the normal vector n:. In Fig. 28-19c, the normal vector of the loop is shown at an arbitrary angle : u to the direction of the magnetic field B . We wish to find the net force and net torque acting on the loop in this orientation. Net Torque. The net force on the loop is the vector sum of the forces acting : on its four sides. For side 2 the vector L in Eq. 28-26 points in the direction of the : : current and has magnitude b.The angle between L and B for side 2 (see Fig. 28-19c) is 90" % u. Thus, the magnitude of the force acting on this side is F2 ! ibB sin(90" % u) ! ibB cos u.

823

F

i

N

i

S

B

–F

Figure 28-18 The elements of an electric motor.A rectangular loop of wire, carrying a current and free to rotate about a fixed axis, is placed in a magnetic field. Magnetic forces on the wire produce a torque that rotates it.A commutator (not shown) reverses the direction of the current every half-revolution so that the torque always acts in the same direction.

(28-31)

: : You can show that the force F4 acting on side 4 has the same magnitude as F 2 but : : the opposite direction. Thus, F2 and F4 cancel out exactly. Their net force is zero

and, because their common line of action is through the center of the loop, their net torque is also zero. : : The situation is different for sides 1 and 3. For them, L is perpendicular to B , : : so the forces F1 and F3 have the common magnitude iaB. Because these two forces have opposite directions, they do not tend to move the loop up or down. However, as Fig. 28-19c shows, these two forces do not share the same line of action; so they do produce a net torque. The torque tends to rotate the loop so : as to align its normal vector n: with the direction of the magnetic field B. That torque has moment arm (b/2) sin u about the central axis of the loop. The magni: : tude t( of the torque due to forces F1 and F3 is then (see Fig. 28-19c)

#

$ #

$

b b sin u & iaB sin u ! iabB sin u. 2 2

t( ! iaB

(28-32)

Coil. Suppose we replace the single loop of current with a coil of N loops, or turns. Further, suppose that the turns are wound tightly enough that they can be

F1

F1

Side 1 i

i

Side 1

F4

Side 4

Figure 28-19 A rectangular loop, of length a and width b and carrying a current i, is located in a uniform magnetic field.A torque t : acts to align the normal vector n with the direction of the field. (a) The loop as seen by looking in the direction of the magnetic field. (b) A perspective of the loop showing how the right-hand rule gives the direction : of n , which is perpendicular to the plane of the loop. (c) A side view of the loop, from side 2.The loop rotates as indicated.

n

Side 1

Side 2

τ

F2

i

b

Side 2

Side 4

(a)

F3 a

θ

b

Side 3

Side 2 Side 3

Rotation B

(b)

(c)

n

B F3

824

CHAPTE R 28 MAG N ETIC FI E LDS

approximated as all having the same dimensions and lying in a plane. Then the turns form a flat coil, and a torque t( with the magnitude given in Eq. 28-32 acts on each of them. The total torque on the coil then has magnitude t ! Nt( ! NiabB sin u ! (NiA)B sin u,

(28-33)

in which A (! ab) is the area enclosed by the coil. The quantities in parentheses (NiA) are grouped together because they are all properties of the coil: its number of turns, its area, and the current it carries. Equation 28-33 holds for all flat coils, no matter what their shape, provided the magnetic field is uniform. For example, for the common circular coil, with radius r, we have t !(Nipr2)B sin u.

(28-34)

Normal Vector. Instead of focusing on the motion of the coil, it is simpler to keep track of the vector n: , which is normal to the plane of the coil. Equation 28-33 tells us that a current-carrying flat coil placed in a magnetic field will tend to rotate so that n: has the same direction as the field. In a motor, the current in the coil is reversed as n: begins to line up with the field direction, so that a torque continues to rotate the coil. This automatic reversal of the current is done via a commutator that electrically connects the rotating coil with the stationary contacts on the wires that supply the current from some source.

28-8 THE MAGNETIC DIPOLE MOMENT Learning Objectives After reading this module, you should be able to . . .

28.38 Identify that a current-carrying coil is a magnetic dipole with a magnetic dipole moment m: that has the direction of the normal vector n:, as given by a right-hand rule. 28.39 For a current-carrying coil, apply the relationship between the magnitude m of the magnetic dipole moment, the number of turns N, the area A of each turn, and the current i. 28.40 On a sketch of a current-carrying coil, draw the direction of the current, and then use a right-hand rule to determine the direction of the magnetic dipole moment vector m:. 28.41 For a magnetic dipole in an external magnetic field, apply the relationship between the torque magnitude t, the dipole moment magnitude m, the magnetic field magnitude B, and the angle u between the dipole moment vector m: : and the magnetic field vector B. 28.42 Identify the convention of assigning a plus or minus sign to a torque according to the direction of rotation. 28.43 Calculate the torque on a magnetic dipole by evaluating a cross product of the dipole moment vector m: and the

:

external magnetic field vector B, in magnitude-angle notation and unit-vector notation. 28.44 For a magnetic dipole in an external magnetic field, identify the dipole orientations at which the torque magnitude is minimum and maximum. 28.45 For a magnetic dipole in an external magnetic field, apply the relationship between the orientation energy U, the dipole moment magnitude m, the external magnetic field magnitude B, and the angle u between the dipole : moment vector m: and the magnetic field vector B. 28.46 Calculate the orientation energy U by taking a dot product of the dipole moment vector m: and the external magnetic : field vector B, in magnitude-angle and unit-vector notations. 28.47 Identify the orientations of a magnetic dipole in an external magnetic field that give the minimum and maximum orientation energies. 28.48 For a magnetic dipole in a magnetic field, relate the orientation energy U to the work Wa done by an external torque as the dipole rotates in the magnetic field.

Key Ideas A coil (of area A and N turns, carrying current i) in a uni: form magnetic field B will experience a torque t: given by



field is :

: ! B. U(u) ! %m

:

: t: ! m $ B. : Here m is the magnetic dipole moment of the coil, with magnitude m ! NiA and direction given by the righthand rule.



The orientation energy of a magnetic dipole in a magnetic

If an external agent rotates a magnetic dipole from an initial orientation ui to some other orientation uf and the dipole is stationary both initially and finally, the work Wa done on the dipole by the agent is ●

Wa ! 'U ! Uf % Ui.

825

28-8 TH E MAG N ETIC DI POLE M OM E NT

The Magnetic Dipole Moment As we have just discussed, a torque acts to rotate a current-carrying coil placed in a magnetic field. In that sense, the coil behaves like a bar magnet placed in the magnetic field. Thus, like a bar magnet, a current-carrying coil is said to be a magnetic dipole. Moreover, to account for the torque on the coil due to the magnetic : : field, we assign a magnetic dipole moment m to the coil. The direction of m is that : of the normal vector n to the plane of the coil and thus is given by the same righthand rule shown in Fig. 28-19. That is, grasp the coil with the fingers of your right hand in the direction of current i; the outstretched thumb of that hand gives the : : direction of m . The magnitude of m is given by m ! NiA

(magnetic moment),

(28-35)

in which N is the number of turns in the coil, i is the current through the coil, and A is the area enclosed by each turn of the coil. From this equation, with i in : amperes and A in square meters, we see that the unit of m is the ampere – square 2 meter (A !m ). : Torque. Using m , we can rewrite Eq. 28-33 for the torque on the coil due to a magnetic field as t ! mB sin u,

(28-36) :

: in which u is the angle between the vectors m and B. We can generalize this to the vector relation

:

: t !m $ B,

:

(28-37)

which reminds us very much of the corresponding equation for the torque exerted by an electric field on an electric dipole — namely, Eq. 22-34: :

t: ! p: $ E. In each case the torque due to the field — either magnetic or electric — is equal to the vector product of the corresponding dipole moment and the field vector. Energy. A magnetic dipole in an external magnetic field has an energy that depends on the dipole’s orientation in the field. For electric dipoles we have shown (Eq. 22-38) that :

U(u ) ! %p: ! E . In strict analogy, we can write for the magnetic case :

: U(u ) ! %m ! B.

(28-38)

In each case the energy due to the field is equal to the negative of the scalar product of the corresponding dipole moment and the field vector. A magnetic dipole has its lowest energy (! %mB cos 0 ! %mB) when its di: pole moment m is lined up with the magnetic field (Fig. 28-20). It has its highest : energy (! %mB cos 180" ! &mB) when m is directed opposite the field. From : : Eq. 28-38, with U in joules and B in teslas, we see that the unit of m can be the joule per tesla (J/T) instead of the ampere – square meter as suggested by Eq. 28-35. Work. If an applied torque (due to “an external agent”) rotates a magnetic dipole from an initial orientation ui to another orientation uf , then work Wa is done on the dipole by the applied torque. If the dipole is stationary before and after the change in its orientation, then work Wa is Wa ! Uf % Ui , where Uf and Ui are calculated with Eq. 28-38.

(28-39)

The magnetic moment vector attempts to align with the magnetic field. B

i

µ

i

µ Highest energy

Lowest energy

Figure 28-20 The orientations of highest and lowest energy of a magnetic dipole (here a coil carrying current) in an external mag: netic field B. The direction of the current i gives the direction of the magnetic : dipole moment m via the right-hand rule : shown for n in Fig. 28-19b.

826

CHAPTE R 28 MAG N ETIC FI E LDS

Table 28-2 Some Magnetic Dipole Moments

Small bar magnet Earth Proton Electron

5 J/T 8.0 $ 1022 J/T 1.4 $ 10%26 J/T 9.3 $ 10%24 J/T

So far, we have identified only a current-carrying coil and a permanent magnet as a magnetic dipole. However, a rotating sphere of charge is also a magnetic dipole, as is Earth itself (approximately). Finally, most subatomic particles, including the electron, the proton, and the neutron, have magnetic dipole moments. As you will see in Chapter 32, all these quantities can be viewed as current loops. For comparison, some approximate magnetic dipole moments are shown in Table 28-2. Language. Some instructors refer to U in Eq. 28-38 as a potential energy and relate it to work done by the magnetic field when the orientation of the dipole changes. Here we shall avoid the debate and say that U is an energy associated with the dipole orientation.

Checkpoint 5 : The figure shows four orientations, at angle u, of a magnetic dipole moment m in a magnetic field. Rank the orientations according to (a) the magnitude of the torque on the dipole and (b) the orientation energy of the dipole, greatest first.

1

4

µ

µ

θ θ

θ θ

µ

µ

2 B 3

Sample Problem 28.07 Rotating a magnetic dipole in a magnetic field Figure 28-21 shows a circular coil with 250 turns, an area A of 2.52 $ 10%4 m2, and a current of 100 mA. The coil is at rest in a uniform magnetic field of magnitude B ! 0.85 T, with : : its magnetic dipole moment m initially aligned with B. (a) In Fig. 28-21, what is the direction of the current in the coil? Right-hand rule: Imagine cupping the coil with your right hand so that your right thumb is outstretched in the direc: tion of m . The direction in which your fingers curl around the coil is the direction of the current in the coil. Thus, in the wires on the near side of the coil — those we see in Fig. 28-21— the current is from top to bottom. (b) How much work would the torque applied by an external agent have to do on the coil to rotate it 90" from its ini-

:

: tial orientation, so that m is perpendicular to B and the coil is again at rest?

KEY IDEA The work Wa done by the applied torque would be equal to the change in the coil’s orientation energy due to its change in orientation. Calculations: From Eq. 28-39 (Wa ! Uf % Ui), we find Wa ! U(90") % U(0") ! %mB cos 90" % (%mB cos 0") ! 0 & mB ! mB. Substituting for m from Eq. 28-35 (m ! NiA), we find that Wa ! (NiA)B

µ

! (250)(100 $ 10%6 A)(2.52 $ 10%4 m2)(0.85 T) B

Figure 28-21 A side view of a circular coil carrying a current and oriented so that its magnetic dipole moment is aligned with magnetic : field B.

! 5.355 $ 10%6 J % 5.4 m J.

(Answer)

Similarly, we can show that to change the orientation by another 90°, so that the dipole moment is opposite the field, another 5.4 mJ is required.

Additional examples, video, and practice available at WileyPLUS

827

QU ESTIONS

Review & Summary :

:

Magnetic Field B A magnetic field B is defined in terms of the : FB

force acting on a test particle with charge q moving through the field with velocity v: : :

:

(28-2)

FB ! qv: $ B. :

The SI unit for B is the tesla (T): 1 T ! 1 N/(A )m) ! 104 gauss.

The Hall Effect When a conducting strip carrying a current i is :

placed in a uniform magnetic field B , some charge carriers (with charge e) build up on one side of the conductor, creating a potential difference V across the strip. The polarities of the sides indicate the sign of the charge carriers.

Magnetic Force on a Current-Carrying Wire A straight wire carrying a current i in a uniform magnetic field experiences a sideways force : : : (28-26) FB ! iL $ B. :

The force acting on a current element i dL in a magnetic field is :

mv2 , r

!q!vB !

(28-15)

:

:

Torque on a Current-Carrying Coil A coil (of area A and N :

turns, carrying current i) in a uniform magnetic field B will experience a torque *: given by :

(28-37)

: t: ! m $ B.

Here m is the magnetic dipole moment of the coil, with magnitude m ! NiA and direction given by the right-hand rule. :

Orientation Energy of a Magnetic Dipole The orienta:

(28-16)

The frequency of revolution f, the angular frequency v, and the period of the motion T are given by v 1 !q!B ! ! . 2p T 2p m

(28-28)

tion energy of a magnetic dipole in a magnetic field is

from which we find the radius r of the circle to be mv . r! !q!B

f!

:

The direction of the length vector L or dL is that of the current i.

A Charged Particle Circulating in a Magnetic Field A

charged particle with mass m and charge magnitude !q! moving with : velocity v: perpendicular to a uniform magnetic field B will travel in a circle.Applying Newton’s second law to the circular motion yields

:

dFB ! i dL $ B.

(28-38)

: U(u) ! %m ! B.

If an external agent rotates a magnetic dipole from an initial orientation ui to some other orientation uf and the dipole is stationary both initially and finally, the work Wa done on the dipole by the agent is

(28-19, 28-18, 28-17)

Wa ! 'U ! Uf % Ui.

(28-39)

Questions 1 Figure 28-22 shows three situations in which a positively charged : particle moves at velocity v: through a uniform magnetic field B : and experiences a magnetic force FB. In each situation, determine whether the orientations of the vectors are physically reasonable. v

FB

B FB (a)

B

v v

B (b)

FB (c)

Figure 28-22 Question 1. Wire 2 Figure 28-23 shows a wire that carries current to the right through a uni- i 1 3 form magnetic field. It also shows four 2 4 choices for the direction of that field. Choices for B (a) Rank the choices according to the magnitude of the electric potential Figure 28-23 Question 2. difference that would be set up across the width of the wire, greatest first. (b) For which choice is the top side of the wire at higher potential than the bottom side of the wire?

3 Figure 28-24 shows a metallic, rectangular solid that is to move : at a certain speed v through the uniform magnetic field B . The dimensions of the solid are multiples of d, as shown. You have six choices for the direction of the velocity: parallel to x, y, or z in ei-

ther the positive or negative direction. (a) Rank the six choices according to the potential difference set up across the solid, greatest first. (b) For which choice is the front face at lower potential?

y B 2d d

x

4 Figure 28-25 shows the path of a z 3d particle through six regions of uniform magnetic field, where the path is either Figure 28-24 Question 3. a half-circle or a quarter-circle. Upon leaving the last region, the particle travels between two charged, parallel plates and is deflected toward the plate of higher potential. What is the direction of the magnetic field in each of the six regions? e

d

c a

f

b

Figure 28-25 Question 4.

828

CHAPTE R 28 MAG N ETIC FI E LDS

5 In Module 28-2, we discussed a charged particle moving : : through crossed fields with the forces FE and FB in opposition. We found that the particle moves in a straight line (that is, neither force dominates the motion) if its speed is given by Eq. 28-7 (v ! E/B). Which of the two forces dominates if the speed of the particle is (a) v + E/B and (b) v , E/B? 6 Figure 28-26 shows crossed uniform electric and magnetic : : fields E and B and, at a certain instant, the velocity vectors of the 10 charged particles listed in Table 28-3. (The vectors are not drawn to scale.) The speeds given in the table are either less than or greater than E/B (see Question 5). Which particles will move out of the page toward you after the instant shown in Fig. 28-26? B

7

4

3

5

e

d

a

g

Figure 28-29 Question 10.

Table 28-4 Question 10

Figure 28-26 Question 6.

Table 28-3 Question 6 Particle

Charge

Speed

Particle

Charge

Speed

1 2 3 4 5

& & & & %

Less Greater Less Greater Less

6 7 8 9 10

% & & % %

Greater Less Greater Less Greater

7 Figure 28-27 shows the path of an electron that passes through two regions containing uniform magnetic fields of magnitudes B1 and B2. Its path in each region is a half-circle. (a) Which field is stronger? (b) What is the direction of each field? (c) Is the time spent by the : electron in the B1 region greater than, less than, or the same as the time spent : in the B2 region?

f

c

j

10

9

h

b

2 6

k

i

8 E

1

particle in the table? (The direction of the magnetic field can be determined by means of one of the paths, which is unique.)

B1

Particle

Mass

Charge

Speed

1 2 3 4 5 6 7 8 9 10 11

2m m m/2 3m 2m m m m 2m m 3m

q 2q q 3q q %q %4q %q %2q %2q 0

v v 2v 3v 2v 2v v v 3v 8v 3v

B2

Figure 28-27 Question 7.

8 Figure 28-28 shows the path of an electron in a region of uniform magnetic field. The path consists of two straight sections, each between a pair of uniformly charged plates, and two half-circles. Which plate is at the higher electric potential in (a) Figure 28-28 Question 8. the top pair of plates and (b) the bottom pair? (c) What is the direction of the magnetic field? 9 (a) In Checkpoint 5, if the dipole moment m: is rotated from orientation 2 to orientation 1 by an external agent, is the work done on the dipole by the agent positive, negative, or zero? (b) Rank the work done on the dipole by the agent for these three rotations, greatest first: 2 : 1, 2 : 4, 2 : 3. 10 Particle roundabout. Figure 28-29 shows 11 paths through a region of uniform magnetic field. One path is a straight line; the rest are half-circles. Table 28-4 gives the masses, charges, and speeds of 11 particles that take these paths through the field in the directions shown. Which path in the figure corresponds to which

11 In Fig. 28-30, a charged particle : enters a uniform magnetic field B B with speed v0, moves through a halfcircle in time T0, and then leaves the field. (a) Is the charge positive or negative? (b) Is the final speed of the particle greater than, less than, Figure 28-30 Question 11. or equal to v0? (c) If the initial speed : had been 0.5v0, would the time spent in field B have been greater than, less than, or equal to T0? (d) Would the path have been a half-circle, more than a half-circle, or less than a half-circle? 12 Figure 28-31 gives snapshots for three situations in which a positively charged particle passes through a uniform magnetic field : B . The velocities v: of the particle differ in orientation in the three snapshots but not in magnitude. Rank the situations according to (a) the period, (b) the frequency, and (c) the pitch of the particle’s motion, greatest first.

B

B v

v (1)

B v

(2)

Figure 28-31 Question 12.

(3)

829

PROB LE M S

Problems Tutoring problem available (at instructor’s discretion) in WileyPLUS and WebAssign SSM • – •••

Worked-out solution available in Student Solutions Manual

WWW Worked-out solution is at

Number of dots indicates level of problem difficulty

ILW

Interactive solution is at

http://www.wiley.com/college/halliday

Additional information available in The Flying Circus of Physics and at flyingcircusofphysics.com :

Module 28-1 Magnetic Fields and the Definition of B •1 SSM ILW A proton traveling at 23.0° with respect to the direction of a magnetic field of strength 2.60 mT experiences a magnetic force of 6.50 $ 10%17 N. Calculate (a) the proton’s speed and (b) its kinetic energy in electron-volts.

V2 ! 100 V. The lower plate is at the lower potential. Neglect fringing and assume that the electron’s velocity vector is perpendicular to the electric field vector between the plates. In unit-vector notation, what uniform magnetic field allows the electron to travel in a straight line in the gap?

•2 A particle of mass 10 g and charge 80 mC moves through a uniform magnetic field, in a region where the free-fall acceleration is %9.8jˆ m/s2. The velocity of the particle is a constant 20iˆ km/s, which is perpendicular to the magnetic field. What, then, is the magnetic field?

••10 A proton travels through uniform magnetic and electric : fields. The magnetic field is B ! %2.50iˆ mT. At one instant the velocity of the proton is v: ! 2000jˆ m/s. At that instant and in unit-vector notation, what is the net force acting on the proton if the electric field is (a) 4.00kˆ V/m, (b) %4.00kˆ V/m, and (c) 4.00iˆ V/m?

An electron that has an instantaneous velocity of v: ! (2.0 $ 106 m/s)iˆ & (3.0 $ 106 m/s)jˆ

: is moving through the uniform magnetic field B ! (0.030 T)iˆ % (0.15 T)jˆ. (a) Find the force on the electron due to the magnetic field. (b) Repeat your calculation for a proton having the same velocity.

•4 An alpha particle travels at a velocity v: of magnitude 550 m/s : through a uniform magnetic field B of magnitude 0.045 T. (An alpha particle has a charge of &3.2 $ 10%19 C and a mass of 6.6 $ : 10%27 kg.) The angle between v: and B is 52°. What is the magni: tude of (a) the force FB acting on the particle due to the field and : (b) the acceleration of the particle due to FB? (c) Does the speed of the particle increase, decrease, or remain the same? ••5 An electron moves through a uniform magnetic field given : by B ! Bxiˆ & (3.0Bx)jˆ . At a particular instant, the electron has velocity v: ! (2.0 iˆ & 4.0 ˆj ) m/s and the magnetic force acting on it is (6.4 $ 10%19 N)kˆ. Find Bx. ••6 A proton moves through a uniform magnetic field given by : B ! (10iˆ % 20jˆ & 30kˆ ) mT. At time t1, the proton has a velocity given by v: ! vxiˆ & vyjˆ & (2.0 km/s)kˆ and the magnetic force on : the proton is FB ! (4.0 $ 10%17 N)iˆ & (2.0 $ 10%17 N)jˆ . At that instant, what are (a) vx and (b) vy? Module 28-2 Crossed Fields: Discovery of the Electron •7 An electron has an initial velocity of (12.0jˆ & 15.0kˆ ) km/s and a constant acceleration of (2.00 $ 1012 m/s2)iˆ in a region in which : uniform electric and magnetic fields are present. If B ! (400 mT)iˆ , : find the electric field E . •8 An electric field of 1.50 kV/m and a perpendicular magnetic field of 0.400 T act on a moving electron to produce no net force. What is the electron’s speed? •9 ILW In Fig. 28-32, an electron accelerated from rest through potential difference V1 ! 1.00 kV enters the gap between two parallel plates having separation d ! 20.0 mm and potential difference V1

y x

d

Figure 28-32 Problem 9.

V2

••11 An ion source is producing 6Li ions, which have charge &e and mass 9.99 $ 10%27 kg. The ions are accelerated by a potential difference of 10 kV and pass horizontally into a region in which there is a uniform vertical magnetic field of magnitude B ! 1.2 T. Calculate the strength of the smallest electric field, to be set up over the same region, that will allow the 6Li ions to pass through undeflected. •••12 At time t1, an electron is 2 sent along the positive direction of an x axis, through both an electric 1 : : field E and a magnetic field B , with : E directed parallel to the y axis. 0 Figure 28-33 gives the y component 0 vs Fnet,y of the net force on the electron –1 due to the two fields, as a function of the electron’s speed v at time t1. The –2 scale of the velocity axis is set by v (m/s) vs ! 100.0 m/s. The x and z compoFigure 28-33 Problem 12. nents of the net force are zero at t1. Assuming Bx ! 0, find (a) the magni: tude E and (b) B in unit-vector notation. Fnet,y (10–19 N)

•3

Module 28-3 Crossed Fields: The Hall Effect •13 A strip of copper 150 mm thick and 4.5 mm wide is placed in : : a uniform magnetic field B of magnitude 0.65 T, with B perpendicular to the strip. A current i ! 23 A is then sent through the strip such that a Hall potential difference V appears across the width of the strip. Calculate V. (The number of charge carriers per unit volume for copper is 8.47 $ 1028 electrons/m3.) •14 A metal strip 6.50 cm long, 0.850 cm wide, and 0.760 mm thick moves with constant velocity v: through a uniform magnetic field B ! 1.20 mT directed perpendicular to the strip, as shown in Fig. 28-34. A potential difference of 3.90 mV is measured between points x and y across the strip. Calculate the speed v.

v

x

y

B

••15 A conducting rectangular solid of dimensions dx ! 5.00 m, dy ! Figure 28-34 Problem 14. 3.00 m, and dz ! 2.00 m moves with a : ˆ constant velocity v ! (20.0 m/s)i through a uniform magnetic field

830

CHAPTE R 28 MAG N ETIC FI E LDS

:

y

•••16 Figure 28-35 shows a metallic block, with its faces parallel to coordinate axes. The block is in a unidy form magnetic field of magnitude x 0.020 T. One edge length of the block dz is 25 cm; the block is not drawn to dx scale. The block is moved at 3.0 m/s z parallel to each axis, in turn, and the Figure 28-35 Problems 15 resulting potential difference V that and 16. appears across the block is measured. With the motion parallel to the y axis, V ! 12 mV; with the motion parallel to the z axis, V ! 18 mV; with the motion parallel to the x axis, V ! 0. What are the block lengths (a) dx, (b) dy, and (c) dz? Module 28-4 A Circulating Charged Particle •17 An alpha particle can be produced in certain radioactive decays of nuclei and consists of two protons and two neutrons. The particle has a charge of q ! &2e and a mass of 4.00 u, where u is the atomic mass unit, with 1 u ! 1.661 $ 10%27 kg. Suppose an alpha particle travels in a circular path of radius 4.50 cm in a uniform magnetic field with B ! 1.20 T. Calculate (a) its speed, (b) its period of revolution, (c) its kinetic energy, and (d) the potential difference through which it would have to be accelerated to achieve this energy. •18 In Fig. 28-36, a particle moves along a circle in a region of uniform magnetic field of magnitude B ! 4.00 mT. The particle is either a proton or an electron (you must decide which). It experiences a magnetic force of magnitude 3.20 $ 10%15 N. What are (a) the particle’s speed, (b) the radius of the circle, and (c) the period of the motion?

rs

0

V 1/2

s •21 SSM An electron of kinetic enV 1/2 (V1/2) ergy 1.20 keV circles in a plane perpendicular to a uniform magnetic Figure 28-38 Problem 20. field. The orbit radius is 25.0 cm. Find (a) the electron’s speed, (b) the magnetic field magnitude, (c) the circling frequency, and (d) the period of the motion.

•22 In a nuclear experiment a proton with kinetic energy 1.0 MeV moves in a circular path in a uniform magnetic field. What energy must (a) an alpha particle (q ! &2e, m ! 4.0 u) and (b) a deuteron (q ! &e, m ! 2.0 u) have if they are to circulate in the same circular path? •23 What uniform magnetic field, applied perpendicular to a beam of electrons moving at 1.30 $ 106 m/s, is required to make the electrons travel in a circular arc of radius 0.350 m? •24 An electron is accelerated from rest by a potential difference of 350 V. It then enters a uniform magnetic field of magnitude 200 mT with its velocity perpendicular to the field. Calculate (a) the speed of the electron and (b) the radius of its path in the magnetic field. •25 (a) Find the frequency of revolution of an electron with an energy of 100 eV in a uniform magnetic field of magnitude 35.0 mT. (b) Calculate the radius of the path of this electron if its velocity is perpendicular to the magnetic field.

B

Figure 28-36 Problem 18.

•19 A certain particle is sent into a uniform magnetic field, with the particle’s velocity vector perpendicular to the direction of the field. Figure 28-37 gives the period T of the particle’s motion versus the inverse of the field magnitude B. The vertical axis scale is set by Ts ! 40.0 ns, and the horizontal axis scale is set by %1 B%1 s ! 5.0 T . What is the ratio m/q of the particle’s mass to the magnitude of its charge?

••26 In Fig. 28-39, a charged particle moves into a region of uniform : magnetic field B , goes through half a circle, and then exits that region. B The particle is either a proton or an electron (you must decide which). Figure 28-39 Problem 26. It spends 130 ns in the region. : (a) What is the magnitude of B ? (b) If the particle is sent back through the magnetic field (along the same initial path) but with 2.00 times its previous kinetic energy, how much time does it spend in the field during this trip? ••27 A mass spectrometer (Fig. 28-12) is used to separate uranium ions of mass 3.92 $ 10%25 kg and charge 3.20 $ 10%19 C from related species. The ions are accelerated through a potential difference of 100 kV and then pass into a uniform magnetic field, where they are bent in a path of radius 1.00 m. After traveling through 180° and passing through a slit of width 1.00 mm and height 1.00 cm, they are collected in a cup. (a) What is the magnitude of the (perpendicular) magnetic field in the separator? If the machine is used to separate out 100 mg of material per hour, calculate (b) the current of the desired ions in the machine and (c) the thermal energy produced in the cup in 1.00 h.

T (ns)

Ts

0

undergoes uniform circular motion. Figure 28-38 gives the radius r of that motion versus V1/2. The vertical axis scale is set by rs ! 3.0 mm, and the horizontal axis scale is set by V s1/2 ! 40.0 V 1/2. What is the magnitude of the magnetic field?

r (mm)

B ! (30.0 mT)jˆ (Fig. 28-35). What are the resulting (a) electric field within the solid, in unit-vector notation, and (b) potential difference across the solid?

B s–1

Figure 28-37 Problem 19.

••28 A particle undergoes uniform circular motion of radius 26.1 mm in a uniform magnetic field. The magnetic force on the particle has a magnitude of 1.60 $ 10%17 N. What is the kinetic energy of the particle?

•20 An electron is accelerated from rest through potential difference V and then enters a region of uniform magnetic field, where it

••29 An electron follows a helical path in a uniform magnetic field of magnitude 0.300 T. The pitch of the path is 6.00 mm, and the

B –1

–1

(T )

PROB LE M S

magnitude of the magnetic force on the electron is 2.00 $ 10%15 N. What is the electron’s speed? ••30 In Fig. 28-40, an electron with an B1 initial kinetic energy of 4.0 keV enters region 1 at time t ! 0. That region contains a Region 1 uniform magnetic field directed into the page, with magnitude 0.010 T. The electron goes through a half-circle and then exits Region 2 region 1, headed toward region 2 across a B2 gap of 25.0 cm. There is an electric potential difference 'V ! 2000 V across the Figure 28-40 gap, with a polarity such that the electron’s Problem 30. speed increases uniformly as it traverses the gap. Region 2 contains a uniform magnetic field directed out of the page, with magnitude 0.020 T. The electron goes through a halfcircle and then leaves region 2. At what time t does it leave? ••31 A particular type of fundamental particle decays by transforming into an electron e% and a positron e&. Suppose the decay: ing particle is at rest in a uniform magnetic field B of magnitude 3.53 mT and the e% and e& move away from the decay point in : paths lying in a plane perpendicular to B . How long after the decay % & do the e and e collide? ••32 A source injects an electron of speed v ! 1.5 $ 107 m/s into a uniform magnetic field of magnitude B ! 1.0 $ 10%3 T. The velocity of the electron makes an angle u ! 10° with the direction of the magnetic field. Find the distance d from the point of injection at which the electron next crosses the field line that passes through the injection point. ••33 SSM WWW A positron with kinetic energy 2.00 keV is pro: jected into a uniform magnetic field B of magnitude 0.100 T, with : its velocity vector making an angle of 89.0° with B . Find (a) the period, (b) the pitch p, and (c) the radius r of its helical path. ••34 An electron follows a helical path in a uniform magnetic : field given by B ! (20iˆ % 50jˆ % 30kˆ ) mT. At time t ! 0, the electron’s velocity is given by v: ! (20iˆ % 30jˆ & 50kˆ ) m/s. (a) What is : the angle f between v: and B ? The electron’s velocity changes with time. Do (b) its speed and (c) the angle f change with time? (d) What is the radius of the helical path? Module 28-5 Cyclotrons and Synchrotrons ••35 A proton circulates in a cyclotron, beginning approximately at rest at the center. Whenever it passes through the gap between dees, the electric potential difference between the dees is 200 V. (a) By how much does its kinetic energy increase with each passage through the gap? (b) What is its kinetic energy as it completes 100 passes through the gap? Let r100 be the radius of the proton’s circular path as it completes those 100 passes and enters a dee, and let r101 be its next radius, as it enters a dee the next time. (c) By what percentage does the radius increase when it changes from r100 to r101? That is, what is percentage increase !

r101 % r100 100%? r100

••36 A cyclotron with dee radius 53.0 cm is operated at an oscillator frequency of 12.0 MHz to accelerate protons. (a) What magnitude B of magnetic field is required to achieve resonance? (b) At that field magnitude, what is the kinetic energy of a proton emerging from the cyclotron? Suppose, instead, that B ! 1.57 T. (c) What oscillator frequency is required to achieve resonance now? (d) At that frequency, what is the kinetic energy of an emerging proton?

831

••37 Estimate the total path length traveled by a deuteron in a cyclotron of radius 53 cm and operating frequency 12 MHz during the (entire) acceleration process. Assume that the accelerating potential between the dees is 80 kV. ••38 In a certain cyclotron a proton moves in a circle of radius 0.500 m. The magnitude of the magnetic field is 1.20 T. (a) What is the oscillator frequency? (b) What is the kinetic energy of the proton, in electron-volts? Module 28-6 Magnetic Force on a Current-Carrying Wire •39 SSM A horizontal power line carries a current of 5000 A from south to north. Earth’s magnetic field (60.0 mT) is directed toward the north and inclined downward at 70.0° to the horizontal. Find the (a) magnitude and (b) direction of the magnetic force on 100 m of the line due to Earth’s field. •40 A wire 1.80 m long carries a current of 13.0 A and makes an angle of 35.0° with a uniform magnetic field of magnitude B ! 1.50 T. Calculate the magnetic force on the wire. •41 ILW A 13.0 g wire of length L ! 62.0 cm is suspended by a pair of flexible leads in a uniform magnetic field of magnitude 0.440 T (Fig. 28-41). What are the (a) magnitude and (b) direction (left or right) of the current required to remove the tension in the supporting leads? •42 The bent wire shown in Fig. 2842 lies in a uniform magnetic field. Each straight section is 2.0 m long and makes an angle of u ! 60° with the x axis, and the wire carries a current of 2.0 A. What is the net magnetic force on the wire in unit-vector notation if the magnetic field is given by (a) 4.0kˆ T and (b) 4.0iˆ T?

L

Figure 28-41 Problem 41. y

θ

i

x

θ

i

•43 A single-turn current loop, carFigure 28-42 Problem 42. rying a current of 4.00 A, is in the shape of a right triangle with sides 50.0, 120, and 130 cm. The loop is in a uniform magnetic field of magnitude 75.0 mT whose direction is parallel to the current in the 130 cm side of the loop. What is the magnitude of the magnetic force on (a) the 130 cm side, (b) the 50.0 cm side, and (c) the 120 cm side? (d) What is the magnitude of the net force on the loop? ••44 Figure 28-43 shows a wire ring of radius a ! 1.8 cm that is peri pendicular to the general direction a of a radially symmetric, diverging magnetic field. The magnetic field at B the ring is everywhere of the same θ magnitude B ! 3.4 mT, and its Figure 28-43 Problem 44. direction at the ring everywhere makes an angle u ! 20° with a normal to the plane of the ring. The twisted lead wires have no effect on the problem. Find the magnitude of the force the field exerts on the ring if the ring carries a current i ! 4.6 mA. ••45 A wire 50.0 cm long carries a 0.500 A current in the posi: tive direction of an x axis through a magnetic field B ! (3.00 mT)jˆ & (10.0 mT)kˆ . In unit-vector notation, what is the magnetic force on the wire?

832

CHAPTE R 28 MAG N ETIC FI E LDS

••46 In Fig. 28-44, a metal wire of mass m ! 24.1 mg can slide with negligible friction on two horizontal parallel rails separated by distance d ! 2.56 cm. The track lies in a vertical uniform magnetic field of magnitude 56.3 mT. At time t ! 0, device G is connected to the rails, producing a constant current i ! 9.13 mA in the wire and rails (even as the wire moves). At t ! 61.1 ms, what are the wire’s (a) speed and (b) direction of motion (left or right)?

B

B

m

i

G

d i i

Figure 28-44 Problem 46. •••47 A 1.0 kg copper rod rests on two horizontal rails 1.0 m apart and carries a current of 50 A from one rail to the other. The coefficient of static friction between rod and rails is 0.60. What are the (a) magnitude and (b) angle (relative to the vertical) of the smallest magnetic field that puts the rod on the verge of sliding? •••48 A long, rigid conductor, lying along an x axis, carries a current of 5.0 A in the negative x direction. A magnetic field : : : B is present, given by B ! 3.0iˆ & 8.0x 2jˆ , with x in meters and B in milliteslas. Find, in unit-vector notation, the force on the 2.0 m segment of the conductor that lies between x ! 1.0 m and x ! 3.0 m. Module 28-7 Torque on a Current Loop •49 SSM Figure 28-45 shows a recy tangular 20-turn coil of wire, of dimensions 10 cm by 5.0 cm. It carries a current of 0.10 A and is hinged Hinge along one long side. It is mounted in line the xy plane, at angle u ! 30" to the direction of a uniform magnetic field of magnitude 0.50 T. In unit-vector notation, what is the torque acting on the coil about the hinge line? z

i

x

θ B

••50 An electron moves in a circle Figure 28-45 Problem 49. of radius r ! 5.29 $ 10%11 m with speed 2.19 $ 106 m/s. Treat the circular path as a current loop with a constant current equal to the ratio of the electron’s charge magnitude to the period of the motion. If the circle lies in a uniform magnetic field of magnitude B ! 7.10 mT, what is the maximum possible magnitude of the torque produced on the loop by the field? ••51 Figure 28-46 shows a wood cylinder of mass m ! 0.250 kg and length L ! 0.100 m, with N ! 10.0 turns of wire wrapped around it longitudinally, so that the B plane of the wire coil contains the long central axis of the cylinder. The L m cylinder is released on a plane ini clined at an angle u to the horizontal, with the plane of the coil parallel to the incline plane. If there is a vertical uniform magnetic field of magnitude 0.500 T, what is the least current i θ through the coil that keeps the cylinder from rolling down the plane? Figure 28-46 Problem 51.

••52 In Fig. 28-47, a rectangular loop carrying current lies in the plane of a uniform magnetic field of magnitude 0.040 T. The loop consists of a single turn of flexible conducting wire that is wrapped around a flexible mount such that the dimensions of the x rectangle can be changed. (The total length Figure 28-47 of the wire is not changed.) As edge length x Problem 52. is varied from approximately zero to its maximum value of approximately 4.0 cm, the magnitude t of the torque on the loop changes. The maximum value of t is 4.80 $ 10%8 N )m. What is the current in the loop? ••53 Prove that the relation t ! NiAB sin u holds not only for the rectangular loop of Fig. 28-19 but also for a closed loop of any shape. (Hint: Replace the loop of arbitrary shape with an assembly of adjacent long, thin, approximately rectangular loops that are nearly equivalent to the loop of arbitrary shape as far as the distribution of current is concerned.) Module 28-8 The Magnetic Dipole Moment •54 A magnetic dipole with a dipole moment of magnitude 0.020 J/T is released from rest in a uniform magnetic field of magnitude 52 mT. The rotation of the dipole due to the magnetic force on it is unimpeded. When the dipole rotates through the orientation where its dipole moment is aligned with the magnetic field, its kinetic energy is 0.80 mJ. (a) What is the initial angle between the dipole moment and the magnetic field? (b) What is the angle when the dipole is next (momentarily) at rest? •55 SSM Two concentric, circular wire loops, of radii r1 ! 20.0 cm and r2 ! 30.0 cm, are located in an xy plane; each carries a clockwise current of 7.00 A (Fig. 28-48). (a) Find the magnitude of the net magnetic dipole moment of the system. (b) Repeat for reversed current in the inner loop.

y i i r2 x r1

•56 A circular wire loop of radius 15.0 cm carries a current of 2.60 A. It is placed so that the normal to its plane makes an angle of 41.0° with a Figure 28-48 Problem 55. uniform magnetic field of magnitude 12.0 T. (a) Calculate the magnitude of the magnetic dipole moment of the loop. (b) What is the magnitude of the torque acting on the loop? •57 SSM A circular coil of 160 turns has a radius of 1.90 cm. (a) Calculate the current that results in a magnetic dipole moment of magnitude 2.30 A )m2. (b) Find the maximum magnitude of the torque that the coil, carrying this current, can experience in a uniform 35.0 mT magnetic field. •58 The magnetic dipole moment of Earth has magnitude 8.00 $ 1022 J/T. Assume that this is produced by charges flowing in Earth’s molten outer core. If the radius of their circular path is 3500 km, calculate the current they produce. •59 A current loop, carrying a current of 5.0 A, is in the shape of a right triangle with sides 30, 40, and 50 cm. The loop is in a uniform magnetic field of magnitude 80 mT whose direction is parallel to the current in the 50 cm side of the loop. Find the magnitude of (a) the magnetic dipole moment of the loop and (b) the torque on the loop.

PROB LE M S

••60 Figure 28-49 shows a current E loop ABCDEFA carrying a current i ! D i 5.00 A. The sides of the loop are parallel C to the coordinate axes shown, with z AB ! 20.0 cm, BC ! 30.0 cm, and FA ! 10.0 cm. In unit-vector notation, what is F the magnetic dipole moment of this i loop? (Hint: Imagine equal and oppoB A y site currents i in the line segment AD; then treat the two rectangular loops x ABCDA and ADEFA.) Figure 28-49 Problem 60. ••61 SSM The coil in Fig. 28-50 carries current i ! 2.00 A in the direction indicated, is parallel to an xz plane, has 3.00 turns and an area of 4.00 $ 10%3 m2, and lies in a : uniform magnetic field B ! (2.00iˆ % 3.00jˆ % 4.00kˆ ) mT. What are (a) the orientation energy of the coil in the magnetic field and (b) the torque (in unit-vector notation) on the coil due to the magnetic field? y

x i

z

Figure 28-50 Problem 61.

µ net (10–5 A • m2)

••62 In Fig. 28-51a, two concentric coils, lying in the same plane, carry currents in opposite directions. The current in the larger coil 1 is fixed. Current i2 in coil 2 can be varied. Figure 28-51b gives the net magnetic moment of the two-coil system as a function of i2. The vertical axis scale is set by mnet,s ! 2.0 $ 10 %5 A)m2, and the horizontal axis scale is set by i2s ! 10.0 mA. If the current in coil 2 is then reversed, what is the magnitude of the net magnetic moment of the two-coil system when i2 ! 7.0 mA?

2 1 (a)

µ net,s

0 – µ net,s

i 2s

0

i 2 (mA) (b)

Figure 28-51 Problem 62. ••63 A circular loop of wire having a radius of 8.0 cm carries a current of 0.20 A. A vector of unit length and parallel to the dipole : moment m of the loop is given by 0.60iˆ % 0.80jˆ. (This unit vector gives the orientation of the magnetic dipole moment vector.) If the : loop is located in a uniform magnetic field given by B ! (0.25 T)iˆ & (0.30 T)kˆ , find (a) the torque on the loop (in unit-vector notation) and (b) the orientation energy of the loop. ••64 Figure 28-52 gives the orientation energy U of a magnetic : dipole in an external magnetic field B , as a function of angle f : between the directions of B and the dipole moment. The vertical axis scale is set by Us ! 2.0 $ 10 %4 J. The dipole can be rotated about an axle with negligible friction in order to change f. Counterclockwise rotation from f ! 0 yields positive values of f,

833

and clockwise rotations yield negative values. The dipole is to be released at angle f ! 0 with a rotational kinetic energy of 6.7 $ 10%4 J, so that it rotates counterclockwise. To what maximum value of f will it rotate? (In the language of Module 8-3, what value f is the turning point in the potential well of Fig. 28-52?) U (10–4 J)

Us –180°

–Us

φ 180°

Figure 28-52 Problem 64.

••65 SSM ILW A wire of length 25.0 cm carrying a current of 4.51 mA is to be formed into a circular coil and placed in a uniform : magnetic field B of magnitude 5.71 mT. If the torque on the coil : from the field is maximized, what are (a) the angle between B and the coil’s magnetic dipole moment and (b) the number of turns in the coil? (c) What is the magnitude of that maximum torque? Additional Problems 66 A proton of charge &e and mass m enters a uniform magnetic : field B ! Biˆ with an initial velocity : v ! v0xiˆ & v0yjˆ . Find an expression in unit-vector notation for its velocity v: at any later time t. 67 A stationary circular wall clock has a face with a radius of 15 cm. Six turns of wire are wound around its perimeter; the wire carries a current of 2.0 A in the clockwise direction. The clock is located where there is a constant, uniform external magnetic field of magnitude 70 mT (but the clock still keeps perfect time). At exactly 1:00 P.M., the hour hand of the clock points in the direction of the external magnetic field. (a) After how many minutes will the minute hand point in the direction of the torque on the winding due to the magnetic field? (b) Find the torque magnitude. 68 A wire lying along a y axis from y ! 0 to y ! 0.250 m carries a current of 2.00 mA in the negative direction of the axis. The wire fully lies in a nonuniform magnetic field that is given by : B ! (0.300 T/m)yiˆ & (0.400 T/m)yjˆ . In unit-vector notation, what is the magnetic force on the wire? 69 Atom 1 of mass 35 u and atom 2 of mass 37 u are both singly ionized with a charge of &e. After being introduced into a mass spectrometer (Fig. 28-12) and accelerated from rest through a potential difference V ! 7.3 kV, each ion follows a circular path in a uniform magnetic field of magnitude B ! 0.50 T. What is the distance 'x between the points where the ions strike the detector? 70 An electron with kinetic energy 2.5 keV moving along the positive direction of an x axis enters a region in which a uniform electric field of magnitude 10 kV/m is in the negative direction of : the y axis. A uniform magnetic field B is to be set up to keep the : electron moving along the x axis, and the direction of B is to be : chosen to minimize the required magnitude of B . In unit-vector : notation, what B should be set up? 71 Physicist S. A. Goudsmit devised a method for measuring the mass of heavy ions by timing their period of revolution in a known magnetic field. A singly charged ion of iodine makes 7.00 rev in a 45.0 mT field in 1.29 ms. Calculate its mass in atomic mass units.

834

CHAPTE R 28 MAG N ETIC FI E LDS

72 A beam of electrons whose kinetic energy is K emerges from a thin-foil “window” at the end of an accelerator tube. A metal plate at distance d from this window is perpendicular to the direction of the emerging beam (Fig. 28-53). (a) Show that we can prevent the beam from hitting the plate if we apply a uniform magnetic field such that B-

the strip is

Foil window

E B ! , EC ner

Electron beam

where r is the resistivity of the material and n is the number density of the charge carriers. (b) Compute this ratio numerically for Problem 13. (See Table 26-1.)

Plate

Tube

d

Figure 28-53 Problem 72.

2 mK , A e2 d2

in which m and e are the electron mass and charge. (b) How should : B be oriented? 73 SSM At time t ! 0, an electron with kinetic energy 12 keV moves through x ! 0 in the positive direction of an x axis that is : parallel to the horizontal component of Earth’s magnetic field B . The field’s vertical component is downward and has magnitude 55.0 mT. (a) What is the magnitude of the electron’s acceleration : due to B ? (b) What is the electron’s distance from the x axis when the electron reaches coordinate x ! 20 cm? 74 A particle with charge 2.0 C moves through a uniform magnetic field. At one instant the velocity of the particle is (2.0iˆ & 4.0jˆ & 6.0kˆ ) m/s and the magnetic force on the particle is (4.0iˆ % 20jˆ & 12kˆ ) N. The x and y components of the magnetic : field are equal. What is B ? 75 A proton, a deuteron (q ! &e, m ! 2.0 u), and an alpha particle (q ! &2e, m ! 4.0 u) all having the same kinetic energy enter a : : region of uniform magnetic field B , moving perpendicular to B . What is the ratio of (a) the radius rd of the deuteron path to the radius rp of the proton path and (b) the radius ra of the alpha particle path to rp? 76 Bainbridge’s mass spectrometer, S1 shown in Fig. 28-54, separates ions S2 having the same velocity. The ions, after entering through slits, S1 and S2, – + pass through a velocity selector comB Plate posed of an electric field produced by P P' the charged plates P and P(, and a : r magnetic field B perpendicular to the B' electric field and the ion path. The ions that then pass undeviated : : through the crossed E and B fields Figure 28-54 Problem 76. enter into a region where a second : magnetic field B ( exists, where they are made to follow circular paths. A photographic plate (or a modern detector) registers their arrival. Show that, for the ions, q/m ! E/rBB(, where r is the radius of the circular orbit. 77 SSM In Fig. 28-55, an electron moves at speed v ! 100 m/s along an x axis through uniform electric and : magnetic fields. The magnetic field B is directed into the page and has magnitude 5.00 T. In unit-vector notation, what is the electric field?

y B v

x

Figure 28-55 Problem 77.

78 (a) In Fig. 28-8, show that the ratio of the Hall electric field magnitude E to the magnitude EC of the electric field responsible for moving charge (the current) along the length of

79 SSM A proton, a deuteron (q ! &e, m ! 2.0 u), and an alpha particle (q ! &2e, m ! 4.0 u) are accelerated through the same potential difference and then enter the same region of uniform : : magnetic field B , moving perpendicular to B . What is the ratio of (a) the proton’s kinetic energy Kp to the alpha particle’s kinetic energy Ka and (b) the deuteron’s kinetic energy Kd to Ka? If the radius of the proton’s circular path is 10 cm, what is the radius of (c) the deuteron’s path and (d) the alpha particle’s path? 80 An electron is moving at 7.20 $ 106 m/s in a magnetic field of strength 83.0 mT. What is the (a) maximum and (b) minimum magnitude of the force acting on the electron due to the field? (c) At one point the electron has an acceleration of magnitude 4.90 $ 1014 m/s2. What is the angle between the electron’s velocity and the magnetic field? 81 A 5.0 mC particle moves through a region containing the uniform magnetic field %20iˆ mT and the uniform electric field 300jˆ V/m. At a certain instant the velocity of the particle is (17iˆ % 11jˆ & 7.0kˆ ) km/s. At that instant and in unit-vector notation, what is the net electromagnetic force (the sum of the electric and magnetic forces) on the particle? 82 In a Hall-effect experiment, a current of 3.0 A sent lengthwise through a conductor 1.0 cm wide, 4.0 cm long, and 10 mm thick produces a transverse (across the width) Hall potential difference of 10 mV when a magnetic field of 1.5 T is passed perpendicularly through the thickness of the conductor. From these data, find (a) the drift velocity of the charge carriers and (b) the number density of charge carriers. (c) Show on a diagram the polarity of the Hall potential difference with assumed current and magnetic field directions, assuming also that the charge carriers are electrons. 83 SSM A particle of mass 6.0 g moves at 4.0 km/s in an xy plane, in a region with a uniform magnetic field given by 5.0iˆ mT. At one instant, when the particle’s velocity is directed 37° counterclockwise from the positive direction of the x axis, the magnetic force on the particle is 0.48kˆ N. What is the particle’s charge? 84 A wire lying along an x axis from x ! 0 to x ! 1.00 m carries a current of 3.00 A in the positive x direction. The wire is : immersed in a nonuniform magnetic field that is given by B ! (4.00 T/m2)x2iˆ % (0.600 T/m2)x 2jˆ . In unit-vector notation, what is the magnetic force on the wire? 85 At one instant, v: ! (%2.00iˆ & 4.00jˆ % 6.00kˆ ) m/s is the ve: locity of a proton in a uniform magnetic field B ! (2.00iˆ % 4.00jˆ & 8.00kˆ ) mT. At that instant, what are (a) the magnetic force : F acting on the proton, in unit-vector notation, (b) the angle : : between v: and F , and (c) the angle between v: and B ? 86 An electron has velocity : v ! (32iˆ & 40jˆ ) km/s as it enters a : ˆ uniform magnetic field B ! 60i mT. What are (a) the radius of the helical path taken by the electron and (b) the pitch of that path? (c) To an observer looking into the magnetic field region from the entrance point of the electron, does the electron spiral clockwise or counterclockwise as it moves?

PROB LE M S

87 Figure 28-56 shows a homopolar generator, which has a solid conducting disk as rotor and which is rotated by a motor (not shown). Conducting brushes connect this emf device to a circuit through which the device drives current. The device can produce a greater emf than wire loop rotors because they can spin at a much higher angular speed without rupturing. The disk has radius R ! 0.250 m and rotation frequency f ! 4000 Hz, and the device is in a uniform magnetic field of magnitude B ! 60.0 mT that is perpendicular to the disk. As the disk is rotated, conduction electrons along the conducting path (dashed line) are forced to move through the magnetic field. (a) For the indicated rotation, is the magnetic force on those electrons up or down in the figure? (b) Is the magnitude of that force greater at the rim or near the center of the disk? (c) What is the work per unit charge done by that force in moving charge along the radial line, between the rim and the center? (d) What, then, is the emf of the device? (e) If the current is 50.0 A, what is the power at which electrical energy is being produced?

Brush

i

Conducting path i Brush

B

Figure 28-56 Problem 87. 88 In Fig. 28-57, the two ends of a U-shaped wire of mass m ! 10.0 g and length L ! 20.0 cm are immersed in mercury (which is a conductor). The wire is in a uniform field of magnitude B ! 0.100 T. A switch (unshown) is rapidly closed and then reopened, sending a pulse of current through the wire, which causes the wire to jump upward. If jump height h ! 3.00 m, how much charge was in the pulse? Assume that the duration of the pulse is much less than the time of flight. Consider the definition of impulse (Eq. 9-30) and its

835

relationship with momentum (Eq. 9-31). Also consider the relationship between charge and current (Eq. 26-2). B

L

m

i Hg

Figure 28-57 Problem 88. 89 In Fig. 28-58, an electron of mass m, charge %e, and low (negligible) speed enters the region between V d two plates of potential difference V and plate separation d, initially headed directly toward the top Figure 28-58 Problem 89. plate. A uniform magnetic field of magnitude B is normal to the plane of the figure. Find the minimum value of B such that the electron will not strike the top plate. 90 A particle of charge q moves in a circle of radius r with speed v. Treating the circular path as a current loop with an average current, find the maximum torque exerted on the loop by a uniform field of magnitude B. 91 In a Hall-effect experiment, express the number density of charge carriers in terms of the Hall-effect electric field magnitude E, the current density magnitude J, and the magnetic field magnitude B. 92 An electron that is moving through a uniform magnetic field has velocity v: ! (40 km/s)iˆ & (35 km/s)jˆ when it experiences : a force F ! %(4.2 fN)iˆ & (4.8 fN)jˆ due to the magnetic field. If : Bx ! 0, calculate the magnetic field B .

C

H

A

P

T

E

R

2

9

Magnetic Fields Due to Currents 29-1 MAGNETIC FIELD DUE TO A CURRENT Learning Objectives After reading this module, you should be able to . . .

29.01 Sketch a current-length element in a wire and indicate the direction of the magnetic field that it sets up at a given point near the wire. 29.02 For a given point near a wire and a given current-element in the wire, determine the magnitude and direction of the magnetic field due to that element. 29.03 Identify the magnitude of the magnetic field set up by a current-length element at a point in line with the direction of that element. 29.04 For a point to one side of a long straight wire carrying current, apply the relationship between the magnetic field magnitude, the current, and the distance to the point. 29.05 For a point to one side of a long straight wire carrying

current, use a right-hand rule to determine the direction of the field vector. 29.06 Identify that around a long straight wire carrying current, the magnetic field lines form circles. 29.07 For a point to one side of the end of a semi-infinite wire carrying current, apply the relationship between the magnetic field magnitude, the current, and the distance to the point. 29.08 For the center of curvature of a circular arc of wire carrying current, apply the relationship between the magnetic field magnitude, the current, the radius of curvature, and the angle subtended by the arc (in radians). 29.09 For a point to one side of a short straight wire carrying current, integrate the Biot–Savart law to find the magnetic field set up at the point by the current.

Key Ideas ● The magnetic field set up by a current-carrying conductor can

be found from the Biot–Savart law. This law asserts that the : contribution dB to the field produced by a current-length ele: ment i ds at a point P located a distance r from the current element is m 0 i ds: $ rˆ : (Biot – Savart law). dB ! r2 4p Here rˆ is a unit vector that points from the element toward P. The quantity m0, called the permeability constant, has the value 4p $ 10 %7 T )m/A % 1.26 $ 10 %6 T )m/A.

● For a long straight wire carrying a current i, the Biot–Savart

law gives, for the magnitude of the magnetic field at a perpendicular distance R from the wire, B!

m0i 2pR

(long straight wire).

● The magnitude of the magnetic field at the center of a circular arc, of radius R and central angle f (in radians), carrying current i, is m 0 if B! (at center of circular arc). 4pR

What Is Physics? One basic observation of physics is that a moving charged particle produces a magnetic field around itself. Thus a current of moving charged particles produces a magnetic field around the current. This feature of electromagnetism, which is the combined study of electric and magnetic effects, came as a surprise to the people who discovered it. Surprise or not, this feature has become enormously important in everyday life because it is the basis of countless electromagnetic devices. For example, a magnetic field is produced in maglev trains and other devices used to lift heavy loads. Our first step in this chapter is to find the magnetic field due to the current in a very small section of current-carrying wire. Then we shall find the magnetic field due to the entire wire for several different arrangements of the wire. 836

29-1 MAG N ETIC FI E LD DU E TO A CU R R E NT

Calculating the Magnetic Field Due to a Current Figure 29-1 shows a wire of arbitrary shape carrying a current i. We want to find : the magnetic field B at a nearby point P. We first mentally divide the wire into differential elements ds and then define for each element a length vector ds: that has length ds and whose direction is the direction of the current in ds. We can then define a differential current-length element to be i ds: ; we wish to calculate : the field dB produced at P by a typical current-length element. From experiment we find that magnetic fields, like electric fields, can be superimposed to find a net : field. Thus, we can calculate the net field B at P by summing, via integration, the : contributions dB from all the current-length elements. However, this summation is more challenging than the process associated with electric fields because of a complexity; whereas a charge element dq producing an electric field is a scalar, a current-length element i ds: producing a magnetic field is a vector, being the product of a scalar and a vector. : Magnitude. The magnitude of the field dB produced at point P at distance r by a current-length element i ds: turns out to be dB !

m 0 i ds sin u , 4p r2

837

This element of current creates a magnetic field at P, into the page. ids ds

i

θ

ˆr

r P

d B (into page)

Current distribution

Figure 29-1 A current-length element i ds: : produces a differential magnetic field dB at point P. The green $ (the tail of an arrow) : at the dot for point P indicates that dB is directed into the page there.

(29-1)

where u is the angle between the directions of ds: and rˆ , a unit vector that points from ds toward P. Symbol m 0 is a constant, called the permeability constant, whose value is defined to be exactly m 0 ! 4p $ 10 %7 T )m/A % 1.26 $ 10 %6 T )m/A.

(29-2)

:

Direction. The direction of dB, shown as being into the page in Fig. 29-1, is that of the cross product ds: $ rˆ. We can therefore write Eq. 29-1 in vector form as :

dB !

m 0 i ds: $ rˆ r2 4p

(Biot – Savart law).

(29-3)

This vector equation and its scalar form, Eq. 29-1, are known as the law of Biot and Savart (rhymes with “Leo and bazaar”). The law, which is experimentally deduced, is an inverse-square law. We shall use this law to calculate the net : magnetic field B produced at a point by various distributions of current. Here is one easy distribution: If current in a wire is either directly toward or directly away from a point P of measurement, can you see from Eq. 29-1 that the magnetic field at P from the current is simply zero (the angle u is either 0" for toward or 180° for away, and both result in sin u ! 0)?

Magnetic Field Due to a Current in a Long Straight Wire Shortly we shall use the law of Biot and Savart to prove that the magnitude of the magnetic field at a perpendicular distance R from a long (infinite) straight wire carrying a current i is given by B!

m0i 2pR

(long straight wire).

The magnetic field vector at any point is tangent to a circle. Wire with current into the page

(29-4)

The field magnitude B in Eq. 29-4 depends only on the current and the perpendicular distance R of the point from the wire. We shall show in our derivation : that the field lines of B form concentric circles around the wire, as Fig. 29-2 shows

Figure 29-2 The magnetic field lines produced by a current in a long straight wire form concentric circles around the wire. Here the current is into the page, as indicated by the $.

B B

838

CHAPTE R 29 MAG N ETIC FI E LDS DU E TO CU R R E NTS

Courtesy Education Development Center

Figure 29-3 Iron filings that have been sprinkled onto cardboard collect in concentric circles when current is sent through the central wire.The alignment, which is along magnetic field lines, is caused by the magnetic field produced by the current.

B

r

:

Figure 29-4 The magnetic field vector B is perpendicular to the radial line extending from a long straight wire with current, but which of the two perpendicular vectors is it?

and as the iron filings in Fig. 29-3 suggest. The increase in the spacing of the lines in Fig. 29-2 with increasing distance from the wire represents the 1/R decrease in : : the magnitude of B predicted by Eq. 29-4. The lengths of the two vectors B in the figure also show the 1/R decrease. Directions. Plugging values into Eq. 29-4 to find the field magnitude B at a given radius is easy. What is difficult for many students is finding the direction of : a field vector B at a given point. The field lines form circles around a long straight wire, and the field vector at any point on a circle must be tangent to the circle. That means it must be perpendicular to a radial line extending to the point from the wire. But there are two possible directions for that perpendicular vector, as shown in Fig. 29-4. One is correct for current into the figure, and the other is correct for current out of the figure. How can you tell which is which? Here is a simple right-hand rule for telling which vector is correct: Curled–straight right-hand rule: Grasp the element in your right hand with your extended thumb pointing in the direction of the current. Your fingers will then naturally curl around in the direction of the magnetic field lines due to that element.

The result of applying this right-hand rule to the current in the straight wire of Fig. 29-2 is shown in a side view in Fig. 29-5a. To determine the direction of the : magnetic field B set up at any particular point by this current, mentally wrap your right hand around the wire with your thumb in the direction of the current. Let your fingertips pass through the point; their direction is then the direction of the : magnetic field at that point. In the view of Fig. 29-2, B at any point is tangent to a magnetic field line; in the view of Fig. 29-5, it is perpendicular to a dashed radial line connecting the point and the current. Figure 29-5 A right-hand rule gives the direction of the magnetic field due to a current in a wire. (a) The situation of Fig. 29-2, seen : from the side.The magnetic field B at any point to the left of the wire is perpendicular to the dashed radial line and directed into the page, in the direction of the fingertips, as indicated by the $. (b) If the current is re: versed, B at any point to the left is still perpendicular to the dashed radial line but now is directed out of the page, as indicated by the dot.

i

B

B i (a )

(b )

The thumb is in the current’s direction. The fingers reveal the field vector’s direction, which is tangent to a circle.

839

29-1 MAG N ETIC FI E LD DU E TO A CU R R E NT

Proof of Equation 29-4

This element of current creates a magnetic field at P, into the page.

Figure 29-6, which is just like Fig. 29-1 except that now the wire is straight and of : infinite length, illustrates the task at hand. We seek the field B at point P, a perpendicular distance R from the wire. The magnitude of the differential magnetic field produced at P by the current-length element i ds: located a distance r from P is given by Eq. 29-1: m 0 i ds sin u dB ! . 4p r2

ds

θ

ˆr

s

r dB

:

The direction of dB in Fig. 29-6 is that of the vector ds: $ rˆ — namely, directly into the page. : Note that dB at point P has this same direction for all the current-length elements into which the wire can be divided. Thus, we can find the magnitude of the magnetic field produced at P by the current-length elements in the upper half of the infinitely long wire by integrating dB in Eq. 29-1 from 0 to .. Now consider a current-length element in the lower half of the wire, one that is as far below P as ds: is above P. By Eq. 29-3, the magnetic field produced at P by this current-length element has the same magnitude and direction as that from element i ds: in Fig. 29-6. Further, the magnetic field produced by the lower half of the wire is exactly the same as that produced by the upper half. To find the : magnitude of the total magnetic field B at P, we need only multiply the result of our integration by 2. We get B!2

"

.

0

dB !

m0i 2p

"

sin u ds . r2

.

0

R

P

i

Figure 29-6 Calculating the magnetic field produced by a current i in a long straight : wire. The field dB at P associated with the current-length element i ds: is directed into the page, as shown.

(29-5)

The variables u, s, and r in this equation are not independent; Fig. 29-6 shows that they are related by r ! 2s 2 & R2 and

sin u ! sin(p % u) !

1s 2

R . & R2

With these substitutions and integral 19 in Appendix E, Eq. 29-5 becomes m 0i B! 2p !

m 0i 2pR

"

R ds 2 (s & R2)3/2

(

s 2 (s & R2)1/2

.

0

R C

)

. 0

!

m 0i , 2pR

(29-6)

as we wanted. Note that the magnetic field at P due to either the lower half or the upper half of the infinite wire in Fig. 29-6 is half this value; that is, m 0i B! 4pR

φ

i

C

ds

r

ˆr (a)

(b)

i

B

C (semi-infinite straight wire).

(29-7)

Magnetic Field Due to a Current in a Circular Arc of Wire To find the magnetic field produced at a point by a current in a curved wire, we would again use Eq. 29-1 to write the magnitude of the field produced by a single current-length element, and we would again integrate to find the net field produced by all the current-length elements. That integration can be difficult, depending on the shape of the wire; it is fairly straightforward, however, when the wire is a circular arc and the point is the center of curvature. Figure 29-7a shows such an arc-shaped wire with central angle f, radius R, and center C, carrying current i. At C, each current-length element i ds: of the wire produces a magnetic field of magnitude dB given by Eq. 29-1. Moreover, as Fig. 29-7b shows, no matter where the element is located on the wire, the angle u

(c)

The right-hand rule reveals the field’s direction at the center. Figure 29-7 (a) A wire in the shape of a circular arc with center C carries current i. (b) For any element of wire along the arc, the angle between the directions of ds: and rˆ is 90°. (c) Determining the direction of the magnetic field at the center C due to the current in the wire; the field is out of the page, in the direction of the fingertips, as indicated by the colored dot at C.

840

CHAPTE R 29 MAG N ETIC FI E LDS DU E TO CU R R E NTS

between the vectors ds: and rˆ is 90°; also, r ! R. Thus, by substituting R for r and 90° for u in Eq. 29-1, we obtain dB !

m 0 i ds sin 90" m 0 i ds ! . 4p R2 4p R2

(29-8)

The field at C due to each current-length element in the arc has this magnitude. : Directions. How about the direction of the differential field dB set up by an element? From above we know that the vector must be perpendicular to a radial line extending through point C from the element, either into the plane of Fig. 29-7a or out of it. To tell which direction is correct, we use the right-hand rule for any of the elements, as shown in Fig. 29-7c. Grasping the wire with the thumb in the direction of the current and bringing the fingers into the region near C, we : see that the vector dB due to any of the differential elements is out of the plane of the figure, not into it. Total Field. To find the total field at C due to all the elements on the arc, : we need to add all the differential field vectors dB. However, because the vectors are all in the same direction, we do not need to find components. We just sum the magnitudes dB as given by Eq. 29-8. Since we have a vast number of those magnitudes, we sum via integration. We want the result to indicate how the total field depends on the angle f of the arc (rather than the arc length). So, in Eq. 29-8 we switch from ds to df by using the identity ds ! R df. The summation by integration then becomes B!

"

dB !

"

f

0

m 0 iR df m 0i ! 2 4p R 4pR

"

f

0

df.

Integrating, we find that B!

m 0 if 4pR

(at center of circular arc).

(29-9)

Heads Up. Note that this equation gives us the magnetic field only at the center of curvature of a circular arc of current. When you insert data into the equation, you must be careful to express f in radians rather than degrees. For example, to find the magnitude of the magnetic field at the center of a full circle of current, you would substitute 2p rad for f in Eq. 29-9, finding B!

m 0i m 0 i(2p) ! 4pR 2R

(at center of full circle).

(29-10)

Sample Problem 29.01 Magnetic field at the center of a circular arc of current The wire in Fig. 29-8a carries a current i and consists of a circular arc of radius R and central angle p/2 rad, and two straight sections whose extensions intersect the center C of : the arc. What magnetic field B (magnitude and direction) does the current produce at C?

straight section at the left, (2) the straight section at the right, and (3) the circular arc. Straight sections: For any current-length element in section 1, the angle u between ds: and rˆ is zero (Fig. 29-8b); so Eq. 29-1 gives us

KEY IDEAS :

We can find the magnetic field B at point C by applying the Biot – Savart law of Eq. 29-3 to the wire, point by point along the full length of the wire. However, the application of : Eq. 29-3 can be simplified by evaluating B separately for the three distinguishable sections of the wire — namely, (1) the

dB1 !

m 0 i ds sin u m i ds sin 0 ! 0 ! 0. 2 4p r 4p r2

Thus, the current along the entire length of straight section 1 contributes no magnetic field at C: B1 ! 0.

841

29-1 MAG N ETIC FI E LD DU E TO A CU R R E NT

Current directly toward or away from C does not create any field there. i

Figure 29-8 (a) A wire consists of two straight sections (1 and 2) and a circular arc (3), and carries current i. (b) For a current-length element in section 1, the angle between ds: and rˆ is zero. (c) Determining the di: rection of magnetic field B3 at C due to the current in the circular arc; the field is into the page there.

i

1

2

3

(a )

B2 ! 0. Circular arc: Application of the Biot – Savart law to evaluate the magnetic field at the center of a circular arc leads to Eq. 29-9 (B ! m 0 if/4pR). Here the central angle f of the arc is p/2 rad.Thus from Eq. 29-9, the magnitude of the magnetic : field B3 at the arc’s center C is m 0i m 0 i(p/2) ! . 4pR 8R :

To find the direction of B3, we apply the right-hand rule displayed in Fig. 29-5. Mentally grasp the circular arc with your right hand as in Fig. 29-8c, with your thumb in the

i

ˆr

ds

r

R

The same situation prevails in straight section 2, where the angle u between ds: and rˆ for any current-length element is 180°. Thus,

B3 !

i

C

C

(b )

C

(c )

B3

direction of the current. The direction in which your fingers curl around the wire indicates the direction of the magnetic field lines around the wire. They form circles around the wire, coming out of the page above the arc and going into the page inside the arc. In the region of point C (inside the : arc), your fingertips point into the plane of the page. Thus, B3 is directed into that plane. Net field: Generally, we combine multiple magnetic fields as vectors. Here, however, only the circular arc produces a magnetic field at point C. Thus, we can write the magnitude : of the net field B as B ! B1 & B2 & B3 ! 0 & 0 &

m 0i m 0i ! . 8R 8R

:

(Answer)

:

The direction of B is the direction of B3 — namely, into the plane of Fig. 29-8.

Sample Problem 29.02 Magnetic field off to the side of two long straight currents

R i1

B R

d

B2 i2

B1

(a)

KEY IDEAS :

(1) The net magnetic field B at point P is the vector sum of the magnetic fields due to the currents in the two wires. (2) We can find the magnetic field due to any current by applying the Biot – Savart law to the current. For points near the current in a long straight wire, that law leads to Eq. 29-4. Finding the vectors: In Fig. 29-9a, point P is distance R from both currents i1 and i2. Thus, Eq. 29-4 tells us that at : : point P those currents produce magnetic fields B1 and B2 with magnitudes B1 !

y

P

Figure 29-9a shows two long parallel wires carrying currents i1 and i2 in opposite directions. What are the magnitude and direction of the net magnetic field at point P? Assume the following values: i1 ! 15 A, i2 ! 32 A, and d ! 5.3 cm.

m 0 i1 2pR

and

B2 !

m 0 i2 . 2pR

In the right triangle of Fig. 29-9a, note that the base angles (between sides R and d) are both 45°. This allows us to write

φ

45°

The two currents create magnetic fields that must be added as vectors to get the net field.

45°

x

P

i1

d (b)

i2

Figure 29-9 (a) Two wires carry currents i1 and i2 in opposite directions (out of and into the page). Note the right angle at P. (b) The separate : : : fields B1 and B2 are combined vectorially to yield the net field B.

cos 45° ! R/d and replace R with d cos 45°. Then the field magnitudes B1 and B2 become B1 !

m 0 i1 2pd cos 45"

and B2 !

m 0 i2 . 2pd cos 45"

842

CHAPTE R 29 MAG N ETIC FI E LDS DU E TO CU R R E NTS

:

:

We want to combine B1 and B2 to find their vector sum, : : which is the net field B at P. To find the directions of B1 and : B2, we apply the right-hand rule of Fig. 29-5 to each current in Fig. 29-9a. For wire 1, with current out of the page, we mentally grasp the wire with the right hand, with the thumb pointing out of the page. Then the curled fingers indicate that the field lines run counterclockwise. In particular, in the region of point P, they are directed upward to the left. Recall that the magnetic field at a point near a long, straight current-carrying wire must be directed perpendicular to a : radial line between the point and the current. Thus, B1 must be directed upward to the left as drawn in Fig. 29-9b. (Note : carefully the perpendicular symbol between vector B1 and the line connecting point P and wire 1.) Repeating this analysis for the current in wire 2, we find : that B2 is directed upward to the right as drawn in Fig. 29-9b. : B1

: B2

Adding the vectors: We can now vectorially add and : to find the net magnetic field B at point P, either by using a vector-capable calculator or by resolving the vectors into : components and then combining the components of B.

:

However, in Fig. 29-9b, there is a third method: Because B1 : and B2 are perpendicular to each other, they form the legs : of a right triangle, with B as the hypotenuse. So, B ! 2B21 & B22 !

m0 2i21 & i22 2pd(cos 45")

(4p $ 10%7 T )m /A)2(15 A)2 & (32 A)2 (2p)(5.3 $ 10%2 m)(cos 45") (Answer) ! 1.89 $ 10%4 T % 190 mT. : : The angle f between the directions of B and B2 in Fig. 29-9b follows from B1 f ! tan%1 , B2 which, with B1 and B2 as given above, yields !

f ! tan%1

i1 15 A ! tan%1 ! 25". i2 32 A

:

The angle between B and the x axis shown in Fig. 29-9b is then f & 45° ! 25° & 45° ! 70°.

(Answer)

Additional examples, video, and practice available at WileyPLUS

29-2 FORCE BETWEEN TWO PARALLEL CURRENTS Learning Objectives After reading this module, you should be able to . . .

29.10 Given two parallel or antiparallel currents, find the magnetic field of the first current at the location of the second current and then find the force acting on that second current.

29.11 Identify that parallel currents attract each other, and antiparallel currents repel each other. 29.12 Describe how a rail gun works.

Key Ideas ● Parallel wires carrying currents in the same direction attract

each other, whereas parallel wires carrying currents in opposite directions repel each other. The magnitude of the force on a length L of either wire is

m 0 Liaib , 2pd where d is the wire separation, and ia and ib are the currents in the wires. F ba ! ib LBa sin 90" !

Force Between Two Parallel Currents Two long parallel wires carrying currents exert forces on each other. Figure 29-10 shows two such wires, separated by a distance d and carrying currents ia and ib. Let us analyze the forces on these wires due to each other. We seek first the force on wire b in Fig. 29-10 due to the current in wire a. : That current produces a magnetic field Ba, and it is this magnetic field that actually causes the force we seek. To find the force, then, we need the magnitude and : : direction of the field Ba at the site of wire b. The magnitude of Ba at every point of wire b is, from Eq. 29-4, m i Ba ! 0 a . (29-11) 2pd

843

29-2 FORCE B ETWE E N TWO PARALLE L CU R R E NTS :

The curled – straight right-hand rule tells us that the direction of Ba at wire b is down, as Fig. 29-10 shows. Now that we have the field, we can find the force it pro: duces on wire b. Equation 28-26 tells us that the force Fba on a length L of wire b : due to the external magnetic field Ba is : Fba :

:

! ib L $

: Ba,

The field due to a at the position of b creates a force on b.

(29-12) :

: Ba

where L is the length vector of the wire. In Fig. 29-10, vectors L and are perpendicular to each other, and so with Eq. 29-11, we can write m 0 Lia ib (29-13) Fba ! ib LBa sin 90" ! . 2pd : : : The direction of Fba is the direction of the cross product L $ Ba. Applying : : : the right-hand rule for cross products to L and Ba in Fig. 29-10, we see that Fba is directly toward wire a, as shown. The general procedure for finding the force on a current-carrying wire is this:

a

d b

Fba L L

ia ib

Ba (due to ia )

Figure 29-10 Two parallel wires carrying currents in the same direction attract each : other. Ba is the magnetic field at wire b pro: duced by the current in wire a. Fba is the resulting force acting on wire b because it : carries current in Ba.

To find the force on a current-carrying wire due to a second current-carrying wire, first find the field due to the second wire at the site of the first wire. Then find the force on the first wire due to that field.

We could now use this procedure to compute the force on wire a due to the current in wire b. We would find that the force is directly toward wire b; hence, the two wires with parallel currents attract each other. Similarly, if the two currents were antiparallel, we could show that the two wires repel each other. Thus,

Projectile Conducting fuse i i

Parallel currents attract each other, and antiparallel currents repel each other.

Conducting rail

The force acting between currents in parallel wires is the basis for the definition of the ampere, which is one of the seven SI base units. The definition, adopted in 1946, is this: The ampere is that constant current which, if maintained in two straight, parallel conductors of infinite length, of negligible circular cross section, and placed 1 m apart in vacuum, would produce on each of these conductors a force of magnitude 2 $ 10 %7 newton per meter of wire length.

(a) F

Conducting gas

Rail Gun The basics of a rail gun are shown in Fig. 29-11a. A large current is sent out along one of two parallel conducting rails, across a conducting “fuse” (such as a narrow piece of copper) between the rails, and then back to the current source along the second rail. The projectile to be fired lies on the far side of the fuse and fits loosely between the rails. Immediately after the current begins, the fuse element melts and vaporizes, creating a conducting gas between the rails where the fuse had been. The curled – straight right-hand rule of Fig. 29-5 reveals that the currents in the rails of Fig. 29-11a produce magnetic fields that are directed downward between the : : rails.The net magnetic field B exerts a force F on the gas due to the current i through the gas (Fig. 29-11b). With Eq. 29-12 and the right-hand rule for cross products, we : find that F points outward along the rails.As the gas is forced outward along the rails, it pushes the projectile, accelerating it by as much as 5 $ 10 6g, and then launches it with a speed of 10 km/s, all within 1 ms. Someday rail guns may be used to launch materials into space from mining operations on the Moon or an asteroid.

i i

B

i

(b)

Figure 29-11 (a) A rail gun, as a current i is set up in it. The current rapidly causes the conducting fuse to vaporize. (b) The cur: rent produces a magnetic field B between : the rails, and the field causes a force F to act on the conducting gas, which is part of the current path. The gas propels the projectile along the rails, launching it.

Checkpoint 1 The figure here shows three long, straight, parallel, equally spaced wires with identical currents either into or out of the page. Rank the wires according to the magnitude of the force on each due to the currents in the other two wires, greatest first.

a

b

c

844

CHAPTE R 29 MAG N ETIC FI E LDS DU E TO CU R R E NTS

29-3 AMPERE’S LAW Learning Objectives After reading this module, you should be able to . . .

29.13 Apply Ampere’s law to a loop that encircles current. 29.14 With Ampere’s law, use a right-hand rule for determining the algebraic sign of an encircled current. 29.15 For more than one current within an Amperian loop, determine the net current to be used in Ampere’s law.

29.16 Apply Ampere’s law to a long straight wire with current, to find the magnetic field magnitude inside and outside the wire, identifying that only the current encircled by the Amperian loop matters.

Key Idea ● Ampere’s law states that

,

:

B ! ds: ! m 0 ienc

(Ampere’s law).

The line integral in this equation is evaluated around a closed loop called an Amperian loop. The current i on the right side is the net current encircled by the loop.

Ampere’s Law We can find the net electric field due to any distribution of charges by first writ: ing the differential electric field dE due to a charge element and then summing : the contributions of dE from all the elements. However, if the distribution is complicated, we may have to use a computer. Recall, however, that if the distribution has planar, cylindrical, or spherical symmetry, we can apply Gauss’ law to find the net electric field with considerably less effort. Similarly, we can find the net magnetic field due to any distribution of currents : by first writing the differential magnetic field dB (Eq. 29-3) due to a current-length : element and then summing the contributions of dB from all the elements.Again we may have to use a computer for a complicated distribution. However, if the distribution has some symmetry, we may be able to apply Ampere’s law to find the magnetic field with considerably less effort. This law, which can be derived from the Biot – Savart law, has traditionally been credited to André-Marie Ampère (1775 – 1836), for whom the SI unit of current is named. However, the law actually was advanced by English physicist James Clerk Maxwell. Ampere’s law is

,

:

B ! ds: ! m0 ienc

(29-14)

(Ampere’s law). :

The loop on the integral sign means that the scalar (dot) product B ! ds: is to be integrated around a closed loop, called an Amperian loop. The current ienc is the net current encircled by that closed loop. : To see the meaning of the scalar product B ! ds: and its integral, let us first apply Ampere’s law to the general situation of Fig. 29-12. The figure shows cross sections of three long straight wires that carry currents i1, i2, and i3 either directly into or directly out of the page. An arbitrary Amperian loop lying in the plane of the page encircles two of the currents but not the third. The counterclockwise direction marked on the loop indicates the arbitrarily chosen direction of integration for Eq. 29-14. To apply Ampere’s law, we mentally divide the loop into differential vector elements ds: that are everywhere directed along the tangent to the loop in the direction of integration. Assume that at the location of the element ds: : shown in Fig. 29-12, the net magnetic field due to the three currents is B. Because the wires are perpendicular to the page, we know that the magnetic

845

29-3 AM PE R E’S L AW

field at ds: due to each current is in the plane of Fig. 29-12; thus, their net mag: netic field B at ds: must also be in that plane. However, we do not know the ori: : entation of B within the plane. In Fig. 29-12, B is arbitrarily drawn at an angle u : : to the direction of ds . The scalar product B ! ds: on the left side of Eq. 29-14 is equal to B cos u ds. Thus, Ampere’s law can be written as

,

:

B ! ds: !

,

B cos u ds ! m 0 ienc. :

(29-15)

We can now interpret the scalar product B ! ds as being the product of a length ds of the Amperian loop and the field component B cos u tangent to the loop. Then we can interpret the integration as being the summation of all such products around the entire loop. Signs. When we can actually perform this integration, we do not need to : : know the direction of B before integrating. Instead, we arbitrarily assume B to be generally in the direction of integration (as in Fig. 29-12). Then we use the following curled – straight right-hand rule to assign a plus sign or a minus sign to each of the currents that make up the net encircled current ienc: :

Curl your right hand around the Amperian loop, with the fingers pointing in the direction of integration. A current through the loop in the general direction of your outstretched thumb is assigned a plus sign, and a current generally in the opposite direction is assigned a minus sign.

Only the currents encircled by the loop are used in Ampere’s law. Amperian loop

i1

i3 i2

ienc ! i1 % i2. (Current i3 is not encircled by the loop.) We can then rewrite Eq. 29-15 as

,

B cos u ds ! m 0(i1 % i2).

(29-16)

You might wonder why, since current i3 contributes to the magnetic-field magnitude B on the left side of Eq. 29-16, it is not needed on the right side. The answer is that the contributions of current i3 to the magnetic field cancel out because the integration in Eq. 29-16 is made around the full loop. In contrast, the contributions of an encircled current to the magnetic field do not cancel out. We cannot solve Eq. 29-16 for the magnitude B of the magnetic field because for the situation of Fig. 29-12 we do not have enough information to simplify and solve the integral. However, we do know the outcome of the integration; it must be equal to m0(i1 % i2), the value of which is set by the net current passing through the loop. We shall now apply Ampere’s law to two situations in which symmetry does allow us to simplify and solve the integral, hence to find the magnetic field.

Magnetic Field Outside a Long Straight Wire with Current Figure 29-14 shows a long straight wire that carries current i directly out of the : page. Equation 29-4 tells us that the magnetic field B produced by the current has the same magnitude at all points that are the same distance r from the wire; that is, : the field B has cylindrical symmetry about the wire. We can take advantage of that symmetry to simplify the integral in Ampere’s law (Eqs. 29-14 and 29-15) if we encircle the wire with a concentric circular Amperian loop of radius r, as in Fig. 29-14. The magnetic field then has the same magnitude B at every point on the loop. We shall integrate counterclockwise, so that ds: has the direction shown in Fig. 29-14.

θ

B

Direction of integration

Figure 29-12 Ampere’s law applied to an arbitrary Amperian loop that encircles two long straight wires but excludes a third wire. Note the directions of the currents.

This is how to assign a sign to a current used in Ampere’s law. +i 1

:

Finally, we solve Eq. 29-15 for the magnitude of B. If B turns out positive, then : the direction we assumed for B is correct. If it turns out negative, we neglect the : minus sign and redraw B in the opposite direction. Net Current. In Fig. 29-13 we apply the curled – straight right-hand rule for Ampere’s law to the situation of Fig. 29-12. With the indicated counterclockwise direction of integration, the net current encircled by the loop is

ds

Direction of integration

–i2

Figure 29-13 A right-hand rule for Ampere’s law, to determine the signs for currents encircled by an Amperian loop. The situation is that of Fig. 29-12.

All of the current is encircled and thus all is used in Ampere’s law.

Wire surface

Amperian loop

r i

B ds

(θ = 0)

Figure 29-14 Using Ampere’s law to find the magnetic field that a current i produces outside a long straight wire of circular cross section. The Amperian loop is a concentric circle that lies outside the wire.

846

CHAPTE R 29 MAG N ETIC FI E LDS DU E TO CU R R E NTS :

We can further simplify the quantity B cos u in Eq. 29-15 by noting that B is : tangent to the loop at every point along the loop, as is ds:. Thus, B and ds: are either parallel or antiparallel at each point of the loop, and we shall arbitrarily : assume the former. Then at every point the angle u between ds: and B is 0°, so cos u ! cos 0° ! 1. The integral in Eq. 29-15 then becomes

,

:

B ! ds: !

,

B cos u ds ! B

,

ds ! B(2pr).

Note that - ds is the summation of all the line segment lengths ds around the circular loop; that is, it simply gives the circumference 2pr of the loop. Our right-hand rule gives us a plus sign for the current of Fig. 29-14. The right side of Ampere’s law becomes &m0 i, and we then have B(2pr) ! m 0i m0i or B! (outside straight wire). (29-17) 2pr With a slight change in notation, this is Eq. 29-4, which we derived earlier — with considerably more effort — using the law of Biot and Savart. In addition, because : the magnitude B turned out positive, we know that the correct direction of B must be the one shown in Fig. 29-14.

Magnetic Field Inside a Long Straight Wire with Current

Only the current encircled by the loop is used in Ampere’s law. B ds

Wire surface

r i

Figure 29-15 shows the cross section of a long straight wire of radius R that carries a uniformly distributed current i directly out of the page. Because the current is uniformly distributed over a cross section of the wire, the magnetic : field B produced by the current must be cylindrically symmetrical. Thus, to find the magnetic field at points inside the wire, we can again use an Amperian loop of : radius r, as shown in Fig. 29-15, where now r + R. Symmetry again suggests that B is tangent to the loop, as shown; so the left side of Ampere’s law again yields

,

R Amperian loop

Figure 29-15 Using Ampere’s law to find the magnetic field that a current i produces inside a long straight wire of circular cross section. The current is uniformly distributed over the cross section of the wire and emerges from the page. An Amperian loop is drawn inside the wire.

:

B ! ds: ! B

,

(29-18)

ds ! B(2pr).

Because the current is uniformly distributed, the current ienc encircled by the loop is proportional to the area encircled by the loop; that is, ienc ! i

p r2 . pR2

(29-19)

Our right-hand rule tells us that ienc gets a plus sign. Then Ampere’s law gives us B(2pr) ! m 0 i or

B!

m i # 2pR $r 0

2

p r2 pR2 (29-20)

(inside straight wire).

Thus, inside the wire, the magnitude B of the magnetic field is proportional to r , is zero at the center, and is maximum at r ! R (the surface). Note that Eqs. 29-17 and 29-20 give the same value for B at the surface.

Checkpoint 2 The figure here shows three equal currents i (two parallel and one antiparallel) and four Amperian loops. Rank the : loops according to the magnitude of - B ! ds: along each, greatest first.

i

i a b

c

d i

29-3 AM PE R E’S L AW

847

Sample Problem 29.03 Ampere’s law to find the field inside a long cylinder of current Figure 29-16a shows the cross section of a long conducting cylinder with inner radius a ! 2.0 cm and outer radius b ! 4.0 cm. The cylinder carries a current out of the page, and the magnitude of the current density in the cross section is given by J ! cr2, with c ! 3.0 $ 10 6 A/m4 and r in meters. : What is the magnetic field B at the dot in Fig. 29-16a, which is at radius r ! 3.0 cm from the central axis of the cylinder?

Next, we must compute the current ienc that is encircled by the Amperian loop. However, we cannot set up a proportionality as in Eq. 29-19, because here the current is not uniformly distributed. Instead, we must integrate the current density magnitude from the cylinder’s inner radius a to the loop radius r, using the steps shown in Figs. 29-16c through h. Calculations: We write the integral as

KEY IDEAS

ienc !

:

The point at which we want to evaluate B is inside the material of the conducting cylinder, between its inner and outer radii. We note that the current distribution has cylindrical symmetry (it is the same all around the cross section for any given radius). Thus, the symmetry allows us to use Ampere’s : law to find B at the point. We first draw the Amperian loop shown in Fig. 29-16b. The loop is concentric with the cylinder and has radius r ! 3.0 cm because we want to evaluate : B at that distance from the cylinder’s central axis.

We want the magnetic field at the dot at radius r.

So, we put a concentric Amperian loop through the dot.

r

r

a

"

J dA !

! 2pc

!

"

r

a

"

r

cr 2(2p r dr)

a

r 3 dr ! 2pc

( r4 ) 4

r a

pc(r 4 % a 4) . 2

Note that in these steps we took the differential area dA to be the area of the thin ring in Figs. 29-16d–f and then

We need to find the current in the area encircled by the loop.

We start with a ring that is so thin that we can approximate the current density as being uniform within it.

Amperian loop

b

(b)

(a)

Its area dA is the product of the ring’s circumference and the width dr.

The current within the ring is the product of the current density J and the ring’s area dA.

(c)

(d)

Our job is to sum the currents in all rings from this smallest one ...

... to this largest one, which has the same radius as the Amperian loop.

dr r

a dA (e)

(f )

(g )

(h)

Figure 29-16 (a)–(b) To find the magnetic field at a point within this conducting cylinder, we use a concentric Amperian loop through the point.We then need the current encircled by the loop. (c)–(h) Because the current density is nonuniform, we start with a thin ring and then sum (via integration) the currents in all such rings in the encircled area.

A

848

CHAPTE R 29 MAG N ETIC FI E LDS DU E TO CU R R E NTS

replaced it with its equivalent, the product of the ring’s circumference 2pr and its thickness dr. For the Amperian loop, the direction of integration indicated in Fig. 29-16b is (arbitrarily) clockwise. Applying the right-hand rule for Ampere’s law to that loop, we find that we should take ienc as negative because the current is directed out of the page but our thumb is directed into the page. We next evaluate the left side of Ampere’s law as we did in Fig. 29-15, and we again obtain Eq. 29-18. Then Ampere’s law,

,

gives us

B!% !%

m 0c 4 (r % a 4) 4r (4p $ 10 %7 T )m/A)(3.0 $ 10 6 A /m4) 4(0.030 m)

$ [(0.030 m)4 % (0.020 m)4] ! %2.0 $ 10 %5 T. :

Thus, the magnetic field B at a point 3.0 cm from the central axis has magnitude

:

B ! ds: ! m 0 ienc ,

B(2pr) ! %

Solving for B and substituting known data yield

B ! 2.0 $ 10 %5 T m 0pc 4 (r % a 4). 2

(Answer)

and forms magnetic field lines that are directed opposite our direction of integration, hence counterclockwise in Fig. 29-16b.

Additional examples, video, and practice available at WileyPLUS

29-4 SOLENOIDS AND TOROIDS Learning Objectives After reading this module, you should be able to . . .

29.17 Describe a solenoid and a toroid and sketch their magnetic field lines. 29.18 Explain how Ampere’s law is used to find the magnetic field inside a solenoid. 29.19 Apply the relationship between a solenoid’s internal magnetic field B, the current i, and the number of turns per

unit length n of the solenoid. 29.20 Explain how Ampere’s law is used to find the magnetic field inside a toroid. 29.21 Apply the relationship between a toroid’s internal magnetic field B, the current i, the radius r, and the total number of turns N.

Key Ideas ● Inside a long solenoid carrying current i, at points not near its ends, the magnitude B of the magnetic field is

B ! m 0 in

● At a point inside a toroid, the magnitude B of the magnetic

field is

m 0 iN 1 (toroid), 2p r where r is the distance from the center of the toroid to the point. B!

(ideal solenoid),

where n is the number of turns per unit length.

Solenoids and Toroids Magnetic Field of a Solenoid i

i

Figure 29-17 A solenoid carrying current i.

We now turn our attention to another situation in which Ampere’s law proves useful. It concerns the magnetic field produced by the current in a long, tightly wound helical coil of wire. Such a coil is called a solenoid (Fig. 29-17). We assume that the length of the solenoid is much greater than the diameter. Figure 29-18 shows a section through a portion of a “stretched-out” solenoid. The solenoid’s magnetic field is the vector sum of the fields produced by the individual turns (windings) that make up the solenoid. For points very

849

29-4 SOLE NOI DS AN D TOROI DS

P

Figure 29-18 A vertical cross section through the central axis of a “stretched-out” solenoid. The back portions of five turns are shown, as are the magnetic field lines due to a current through the solenoid. Each turn produces circular magnetic field lines near itself. Near the solenoid’s axis, the field lines combine into a net magnetic field that is directed along the axis. The closely spaced field lines there indicate a strong magnetic field. Outside the solenoid the field lines are widely spaced; the field there is very weak.

close to a turn, the wire behaves magnetically almost like a long straight wire, : and the lines of B there are almost concentric circles. Figure 29-18 suggests that the field tends to cancel between adjacent turns. It also suggests that, at : points inside the solenoid and reasonably far from the wire, B is approximately parallel to the (central) solenoid axis. In the limiting case of an ideal solenoid, which is infinitely long and consists of tightly packed (close-packed) turns of square wire, the field inside the coil is uniform and parallel to the solenoid axis. At points above the solenoid, such as P in Fig. 29-18, the magnetic field set up by the upper parts of the solenoid turns (these upper turns are marked $) is directed to the left (as drawn near P) and tends to cancel the field set up at P by the lower parts of the turns (these lower turns are marked #), which is directed to the right (not drawn). In the limiting case of an ideal solenoid, the magnetic field outside the solenoid is zero. Taking the external field to be zero is an excellent assumption for a real solenoid if its length is much greater than its diameter and if we consider external points such as point P that are not at either end of the solenoid. The direction of the magnetic field along the solenoid axis is given by a curled – straight right-hand rule: Grasp the solenoid with your right hand so that your fingers follow the direction of the current in the windings; your extended right thumb then points in the direction of the axial magnetic field. : Figure 29-19 shows the lines of B for a real solenoid. The spacing of these lines in the central region shows that the field inside the coil is fairly strong and uniform over the cross section of the coil. The external field, however, is relatively weak. Ampere’s Law. Let us now apply Ampere’s law,

,

:

:

:

B ! ds: !

"

b

a

:

B ! ds: &

"

c

b

:

B ! ds: &

"

d

c

P1

Figure 29-19 Magnetic field lines for a real solenoid of finite length. The field is strong and uniform at interior points such as P1 but relatively weak at external points such as P2.

d

(29-21)

B ! ds ! m 0 ienc , :

to the ideal solenoid of Fig. 29-20, where B is uniform within the solenoid and : zero outside it, using the rectangular Amperian loop abcda. We write - B ! ds: as the sum of four integrals, one for each loop segment:

,

P2

:

B ! ds: &

"

a

d

:

B ! ds:.

(29-22)

B

a

h

c

i

b

Figure 29-20 Application of Ampere’s law to a section of a long ideal solenoid carrying a current i. The Amperian loop is the rectangle abcda.

850

CHAPTE R 29 MAG N ETIC FI E LDS DU E TO CU R R E NTS

The first integral on the right of Eq. 29-22 is Bh, where B is the magnitude of : the uniform field B inside the solenoid and h is the (arbitrary) length of the segment from a to b. The second and fourth integrals are zero because for every : element ds of these segments, B either is perpendicular to ds or is zero, and thus : : the product B ! d s is zero. The third integral, which is taken along a segment that lies outside the solenoid, is zero because B ! 0 at all external points. Thus, : - B ! ds: for the entire rectangular loop has the value Bh. Net Current. The net current ienc encircled by the rectangular Amperian loop in Fig. 29-20 is not the same as the current i in the solenoid windings because the windings pass more than once through this loop. Let n be the number of turns per unit length of the solenoid; then the loop encloses nh turns and ienc ! i(nh). Ampere’s law then gives us Bh ! m0inh or

B ! m 0 in

(ideal solenoid).

(29-23)

Although we derived Eq. 29-23 for an infinitely long ideal solenoid, it holds quite well for actual solenoids if we apply it only at interior points and well away from the solenoid ends. Equation 29-23 is consistent with the experimental fact that the magnetic field magnitude B within a solenoid does not depend on the diameter or the length of the solenoid and that B is uniform over the solenoidal cross section. A solenoid thus provides a practical way to set up a known uniform magnetic field for experimentation, just as a parallel-plate capacitor provides a practical way to set up a known uniform electric field.

Magnetic Field of a Toroid

i (a) Amperian loop

i

Figure 29-21a shows a toroid, which we may describe as a (hollow) solenoid that has been curved until its two ends meet, forming a sort of hollow bracelet. What : magnetic field B is set up inside the toroid (inside the hollow of the bracelet)? We can find out from Ampere’s law and the symmetry of the bracelet. : From the symmetry, we see that the lines of B form concentric circles inside the toroid, directed as shown in Fig. 29-21b. Let us choose a concentric circle of radius r as an Amperian loop and traverse it in the clockwise direction. Ampere’s law (Eq. 29-14) yields (B)(2pr) ! m0iN, where i is the current in the toroid windings (and is positive for those windings enclosed by the Amperian loop) and N is the total number of turns. This gives

r

B! B (b)

Figure 29-21 (a) A toroid carrying a current i. (b) A horizontal cross section of the toroid. The interior magnetic field (inside the bracelet-shaped tube) can be found by applying Ampere’s law with the Amperian loop shown.

m 0 iN 1 2p r

(toroid).

(29-24)

In contrast to the situation for a solenoid, B is not constant over the cross section of a toroid. It is easy to show, with Ampere’s law, that B ! 0 for points outside an ideal toroid (as if the toroid were made from an ideal solenoid). The direction of the magnetic field within a toroid follows from our curled – straight right-hand rule: Grasp the toroid with the fingers of your right hand curled in the direction of the current in the windings; your extended right thumb points in the direction of the magnetic field.

851

29-5 A CU R R E NT-CAR RYI NG COI L AS A MAG N ETIC DI POLE

Sample Problem 29.04 The field inside a solenoid (a long coil of current) A solenoid has length L ! 1.23 m and inner diameter d ! 3.55 cm, and it carries a current i ! 5.57 A. It consists of five close-packed layers, each with 850 turns along length L. What is B at its center? KEY IDEA The magnitude B of the magnetic field along the solenoid’s central axis is related to the solenoid’s current i and number of turns per unit length n by Eq. 29-23 (B ! m0 in).

Calculation: Because B does not depend on the diameter of the windings, the value of n for five identical layers is simply five times the value for each layer. Equation 29-23 then tells us B ! m0 in ! (4p $ 10 %7 T )m /A)(5.57 A) ! 2.42 $ 10 %2 T ! 24.2 mT.

5 $ 850 turns 1.23 m (Answer)

To a good approximation, this is the field magnitude throughout most of the solenoid.

Additional examples, video, and practice available at WileyPLUS

29-5 A CURRENT-CARRYING COIL AS A MAGNETIC DIPOLE Learning Objectives After reading this module, you should be able to . . .

29.22 Sketch the magnetic field lines of a flat coil that is carrying current. 29.23 For a current-carrying coil, apply the relationship between the dipole moment magnitude m and the coil’s

current i, number of turns N, and area per turn A. 29.24 For a point along the central axis, apply the relationship between the magnetic field magnitude B, the magnetic moment m, and the distance z from the center of the coil.

Key Idea ● The magnetic field produced by a current-carrying coil, which is a magnetic dipole, at a point P located a distance z along the coil’s perpendicular central axis is parallel to the axis and is given by :

B(z) !

: m0 m , 2p z3

: where m is the dipole moment of the coil. This equation applies only when z is much greater than the dimensions of the coil.

A Current-Carrying Coil as a Magnetic Dipole So far we have examined the magnetic fields produced by current in a long straight wire, a solenoid, and a toroid. We turn our attention here to the field produced by a coil carrying a current. You saw in Module 28-8 that such a coil : behaves as a magnetic dipole in that, if we place it in an external magnetic field B, a torque t: given by :

: t: ! m $B

(29-25)

: acts on it. Here m is the magnetic dipole moment of the coil and has the magnitude NiA, where N is the number of turns, i is the current in each turn, and A is the area enclosed by each turn. (Caution: Don’t confuse the magnetic dipole : moment m with the permeability constant m0.) : Recall that the direction of m is given by a curled – straight right-hand rule: Grasp the coil so that the fingers of your right hand curl around it in the direction of the current; your extended thumb then points in the direction of the dipole : moment m .

852

CHAPTE R 29 MAG N ETIC FI E LDS DU E TO CU R R E NTS

N

µ

i

i S B

Figure 29-22 A current loop produces a magnetic field like that of a bar magnet and thus has : associated north and south poles.The magnetic dipole moment m of the loop, its direction given by a curled – straight right-hand rule, points from the south pole to the north pole, in : the direction of the field B within the loop.

Magnetic Field of a Coil We turn now to the other aspect of a current-carrying coil as a magnetic dipole. What magnetic field does it produce at a point in the surrounding space? The problem does not have enough symmetry to make Ampere’s law useful; so we must turn to the law of Biot and Savart. For simplicity, we first consider only a coil with a single circular loop and only points on its perpendicular central axis, which we take to be a z axis. We shall show that the magnitude of the magnetic field at such points is B(z) !

m 0 iR2 , 2(R2 & z2)3/2

(29-26)

in which R is the radius of the circular loop and z is the distance of the point in question from the center of the loop. Furthermore, the direction of the mag: : netic field B is the same as the direction of the magnetic dipole moment m of the loop. Large z. For axial points far from the loop, we have z / R in Eq. 29-26. With that approximation, the equation reduces to B(z) %

m 0 iR2 . 2z3

Recalling that pR2 is the area A of the loop and extending our result to include a coil of N turns, we can write this equation as B(z) !

m 0 NiA . 2p z3

:

: Further, because B and m have the same direction, we can write the equation in vector form, substituting from the identity m ! NiA:

:

B(z) !

: m0 m 2p z3

(current-carrying coil).

(29-27)

Thus, we have two ways in which we can regard a current-carrying coil as a magnetic dipole: (1) it experiences a torque when we place it in an external magnetic field; (2) it generates its own intrinsic magnetic field, given, for distant points along its axis, by Eq. 29-27. Figure 29-22 shows the magnetic field of : a current loop; one side of the loop acts as a north pole (in the direction of m )

29-5 A CU R R E NT-CAR RYI NG COI L AS A MAG N ETIC DI POLE

853

and the other side as a south pole, as suggested by the lightly drawn magnet in the figure. If we were to place a current-carrying coil in an external magnetic field, it would tend to rotate just like a bar magnet would.

Checkpoint 3 The figure here shows four arrangements of circular loops of radius r or 2r, centered on vertical axes (perpendicular to the loops) and carrying identical currents in the directions indicated. Rank the arrangements according to the magnitude of the net magnetic field at the dot, midway between the loops on the central axis, greatest first.

(a)

(b)

(c)

(d)

Proof of Equation 29-26

dB

Figure 29-23 shows the back half of a circular loop of radius R carrying a current i. Consider a point P on the central axis of the loop, a distance z from its plane. Let us apply the law of Biot and Savart to a differential element ds of the loop, located at the left side of the loop. The length vector ds: for this element points perpendicularly out of the page. The angle u between ds: and rˆ in Fig. 29-23 is 90°; the plane formed by these two vectors is perpendicular to the plane of the page and contains both rˆ and ds:. From the law of Biot and Savart (and the right-hand : rule), the differential field dB produced at point P by the current in this element is perpendicular to this plane and thus is directed in the plane of the figure, perpendicular to rˆ, as indicated in Fig. 29-23. : Let us resolve dB into two components: dB, along the axis of the loop and dB" perpendicular to this axis. From the symmetry, the vector sum of all the perpendicular components dB" due to all the loop elements ds is zero. This leaves only the axial (parallel) components dB, and we have B!

"

dB,.

For the element ds: in Fig. 29-23, the law of Biot and Savart (Eq. 29-1) tells us that the magnetic field at distance r is dB !

m 0 i ds sin 90" . 4p r2

We also have dB, ! dB cos a. Combining these two relations, we obtain dB, !

m 0 i cos a ds . 4pr 2

(29-28)

Figure 29-23 shows that r and a are related to each other. Let us express each in terms of the variable z, the distance between point P and the center of the loop. The relations are r ! 2R2 & z2 (29-29)

d B n1

(a)

(b)

If the indexes match, there is no direction change.

n2 < n1

If the next index is greater, the ray is bent toward the normal.

(c)

If the next index is less, the ray is bent away from the normal.

Figure 33-17 Refraction of light traveling from a medium with an index of refraction n1 into a medium with an index of refraction n2 . (a) The beam does not bend when n2 ! n1; the refracted light then travels in the undeflected direction (the dotted line), which is the same as the direction of the incident beam. The beam bends (b) toward the normal when n2 $ n1 and (c) away from the normal when n2 % n1.

993

33-5 R E FLECTION AN D R E FRACTION

Refraction cannot bend a beam so much that the refracted ray is on the same side of the normal as the incident ray.

Chromatic Dispersion The index of refraction n encountered by light in any medium except vacuum depends on the wavelength of the light. The dependence of n on wavelength implies that when a light beam consists of rays of different wavelengths, the rays will be refracted at different angles by a surface; that is, the light will be spread out by the refraction. This spreading of light is called chromatic dispersion, in which “chromatic” refers to the colors associated with the individual wavelengths and “dispersion” refers to the spreading of the light according to its wavelengths or colors. The refractions of Figs. 33-16 and 33-17 do not show chromatic dispersion because the beams are monochromatic (of a single wavelength or color). Generally, the index of refraction of a given medium is greater for a shorter wavelength (corresponding to, say, blue light) than for a longer wavelength (say, red light). As an example, Fig. 33-18 shows how the index of refraction of fused quartz depends on the wavelength of light. Such dependence means that when a beam made up of waves of both blue and red light is refracted through a surface, such as from air into quartz or vice versa, the blue component (the ray corresponding to the wave of blue light) bends more than the red component. A beam of white light consists of components of all (or nearly all) the colors in the visible spectrum with approximately uniform intensities. When you see such a beam, you perceive white rather than the individual colors. In Fig. 33-19a, a beam of white light in air is incident on a glass surface. (Because the pages of this book are white, a beam of white light is represented with a gray ray here. Also, a beam of monochromatic light is generally represented with a red ray.) Of the refracted light in Fig. 33-19a, only the red and blue components are shown. Because the blue component is bent more than the red component, the angle of refraction u2b for the blue component is smaller than the angle of refraction u2r for the red component. (Remember, angles are measured relative to the normal.) In Fig. 33-19b, a ray of white light in glass is incident on a glass – air interface. Again, the blue component is bent more than the red component, but now u2b is greater than u2r .

Incident white light

Normal

θ1

Figure 33-19 Chromatic dispersion of white light. The blue component is bent more than the red component. (a) Passing from air to glass, the blue component ends up with the smaller angle of refraction. (b) Passing from glass to air, the blue component ends up with the greater angle of refraction. Each dotted line represents the direction in which the light would continue to travel if it were not bent by the refraction.

1.48 Index of refraction

2. If n2 is greater than n1, then u2 is less than u1. In this case, refraction bends the light beam away from the undeflected direction and toward the normal, as in Fig. 33-17b. 3. If n2 is less than n1, then u2 is greater than u1. In this case, refraction bends the light beam away from the undeflected direction and away from the normal, as in Fig. 33-17c.

1.47

1.46

1.45 300

400 500 600 700 Wavelength (nm)

Figure 33-18 The index of refraction as a function of wavelength for fused quartz. The graph indicates that a beam of shortwavelength light, for which the index of refraction is higher, is bent more upon entering or leaving quartz than a beam of long-wavelength light.

Reflected white light

Normal Incident white light

θ1 Air n1 Glass n2

Reflected white light

θ1

θ1

Glass n1 Air n2

θ 2b

θ 2r

θ 2r Refracted light (a)

800

Blue is always bent more than red.

θ 2b (b)

Refracted light

994

CHAPTE R 33 E LECTROMAG N ETIC WAVES

To increase the color separation, we can use a solid glass prism with a triangular cross section, as in Fig. 33-20a. The dispersion at the first surface (on the left in Figs. 33-20a, b) is then enhanced by the dispersion at the second surface.

Rainbows

Courtesy Bausch & Lomb

(a)

White light

(b)

Figure 33-20 (a) A triangular prism separating white light into its component colors. (b) Chromatic dispersion occurs at the first surface and is increased at the second surface.

The most charming example of chromatic dispersion is a rainbow. When sunlight (which consists of all visible colors) is intercepted by a falling raindrop, some of the light refracts into the drop, reflects once from the drop’s inner surface, and then refracts out of the drop. Figure 33-21a shows the situation when the Sun is on the horizon at the left (and thus when the rays of sunlight are horizontal). The first refraction separates the sunlight into its component colors, and the second refraction increases the separation. (Only the red and blue rays are shown in the figure.) If many falling drops are brightly illuminated, you can see the separated colors they produce when the drops are at an angle of 42° from the direction of the antisolar point A, the point directly opposite the Sun in your view. To locate the drops, face away from the Sun and point both arms directly away from the Sun, toward the shadow of your head. Then move your right arm directly up, directly rightward, or in any intermediate direction until the angle between your arms is 42". If illuminated drops happen to be in the direction of your right arm, you see color in that direction. Because any drop at an angle of 42" in any direction from A can contribute to the rainbow, the rainbow is always a 42" circular arc around A (Fig. 33-21b) and the top of a rainbow is never more than 42" above the horizon. When the Sun is above the horizon, the direction of A is below the horizon, and only a shorter, lower rainbow arc is possible (Fig. 33-21c). Because rainbows formed in this way involve one reflection of light inside each drop, they are often called primary rainbows. A secondary rainbow involves two reflections inside a drop, as shown in Fig. 33-21d. Colors appear in the secondary rainbow at an angle of 52" from the direction of A. A secondary rainbow Water drops

Sunlight

Rainbow 42° (a)

O

A

42°

42°

(b)

42° A

Water drops

Figure 33-21 (a) The separation of colors when sunlight refracts into and out of falling raindrops leads to a primary rainbow. The antisolar point A is on the horizon at the right. The rainbow colors appear at an angle of 42° from the direction of A. (b) Drops at 42° from A in any direction can contribute to the rainbow. (c) The rainbow arc when the Sun is higher (and thus A is lower). (d) The separation of colors leading to a secondary rainbow.

Sunlight

Rainbow

42° (c)

42° A

O (d)

52°

A

995

33-5 R E FLECTION AN D R E FRACTION

is wider and dimmer than a primary rainbow and thus is more difficult to see. Also, the order of colors in a secondary rainbow is reversed from the order in a primary rainbow, as you can see by comparing parts a and d of Fig. 33-21. Rainbows involving three or four reflections occur in the direction of the Sun and cannot be seen against the glare of sunshine in that part of the sky but have been photographed with special techniques.

Checkpoint 5 Which of the three drawings here (if any) show physically possible refraction?

n = 1.6

n = 1.5 n = 1.8

n = 1.4

n = 1.6

n = 1.6

(a)

(c)

(b)

Sample Problem 33.03 Reflection and refraction of a monochromatic beam (a) In Fig. 33-22a, a beam of monochromatic light reflects and refracts at point A on the interface between material 1 with index of refraction n1 ! 1.33 and material 2 with index of refraction n2 ! 1.77. The incident beam makes an angle of 50° with the interface. What is the angle of reflection at point A? What is the angle of refraction there?

θ1 θ'1 50° A

θ2

KEY IDEAS

A

n2

θ2

θ 2 θ' 2

n2

(1) The angle of reflection is equal to the angle of incidence, and both angles are measured relative to the normal to the surface at the point of reflection. (2) When light reaches the interface between two materials with different indexes of refraction (call them n1 and n2), part of the light can be refracted by the interface according to Snell’s law, Eq. 33-40: (33-42)

n2 sin u2 ! n1 sin u1,

where both angles are measured relative to the normal at the point of refraction. Calculations: In Fig. 33-22a, the normal at point A is drawn as a dashed line through the point. Note that the angle of incidence u1 is not the given 50" but is 90" & 50" ! 40". Thus, the angle of reflection is u#1 ! u1 ! 40".

(Answer)

The light that passes from material 1 into material 2 undergoes refraction at point A on the interface between the two materials. Again we measure angles between light rays and a normal, here at the point of refraction. Thus, in Fig. 33-22a, the angle of refraction is the angle marked u2. Solving Eq. 33-42 for u2 gives us u2 ! sin&1

n1

# nn

1 2

$

sin u1 ! sin&1

! 28.88" % 29".

sin 40"$ # 1.33 1.77 (Answer)

B

n3

θ3

Air (a)

(b)

Figure 33-22 (a) Light reflects and refracts at point A on the interface between materials 1 and 2. (b) The light that passes through material 2 reflects and refracts at point B on the interface between materials 2 and 3 (air). Each dashed line is a normal. Each dotted line gives the incident direction of travel.

This result means that the beam swings toward the normal (it was at 40° to the normal and is now at 29°). The reason is that when the light travels across the interface, it moves into a material with a greater index of refraction. Caution: Note that the beam does not swing through the normal so that it appears on the left side of Fig. 33-22a. (b) The light that enters material 2 at point A then reaches point B on the interface between material 2 and material 3, which is air, as shown in Fig. 33-22b. The interface through B is parallel to that through A. At B, some of the light reflects and the rest enters the air. What is the angle of reflection? What is the angle of refraction into the air? Calculations: We first need to relate one of the angles at

996

CHAPTE R 33 E LECTROMAG N ETIC WAVES

point B with a known angle at point A. Because the interface through point B is parallel to that through point A, the incident angle at B must be equal to the angle of refraction u2, as shown in Fig. 33-22b. Then for reflection, we again use the law of reflection. Thus, the angle of reflection at B is u#2 ! u 2 ! 28.88" % 29".

(Answer)

Next, the light that passes from material 2 into the air undergoes refraction at point B, with refraction angle u3. Thus, we again apply Snell’s law of refraction, but this time

we write Eq. 33-40 as n3 sin u3 ! n2 sin u2.

(33-43)

Solving for u3 then leads to u3 ! sin&1

# nn

2 3

$

sin u2 ! sin&1

! 58.75" % 59".

sin 28.88"$ # 1.77 1.00 (Answer)

Thus, the beam swings away from the normal (it was at 29° to the normal and is now at 59°) because it moves into a material (air) with a lower index of refraction.

Additional examples, video, and practice available at WileyPLUS

33-6 TOTAL INTERNAL REFLECTION Learning Objectives After reading this module, you should be able to . . .

33.45 With sketches, explain total internal reflection and include the angle of incidence, the critical angle, and the relative values of the indexes of refraction on the two sides of the interface.

33.46 Identify the angle of refraction for incidence at a critical angle. 33.47 For a given pair of indexes of refraction, calculate the critical angle.

Key Idea

● A wave encountering a boundary across which the index of refraction decreases will experience total internal reflection if the angle of incidence exceeds a critical angle uc, where n2 uc ! sin&1 (critical angle). n1

Total Internal Reflection Figure 33-23a shows rays of monochromatic light from a point source S in glass incident on the interface between the glass and air. For ray a, which is perpendicular to the interface, part of the light reflects at the interface and the rest travels through it with no change in direction. For rays b through e, which have progressively larger angles of incidence at the interface, there are also both reflection and refraction at the interface. As the angle of incidence increases, the angle of refraction increases; for ray e it is 90°, which means that the refracted ray points directly along the interface. The angle of incidence giving this situation is called the critical angle uc . For angles of incidence larger than uc , such as for rays f and g, there is no refracted ray and all the light is reflected; this effect is called total internal reflection because all the light remains inside the glass. To find uc , we use Eq. 33-40; we arbitrarily associate subscript 1 with the glass and subscript 2 with the air, and then we substitute uc for u1 and 90° for u2, which leads to n1 sin uc ! n2 sin 90°, (33-44)

33-7 POL AR I ZATION BY R E FLECTION

997

Critical case a b

c

d

e

g

f

Air Glass

θc

(a)

If the next index is lower and the incident angle is large enough, the light can be trapped inside.

S

(b) Ken Kay/Fundamental Photographs

Figure 33-23 (a) Total internal reflection of light from a point source S in glass occurs for all angles of incidence greater than the critical angle uc. At the critical angle, the refracted ray points along the air – glass interface. (b) A source in a tank of water.

which gives us uc ! sin&1

n2 n1

(33-45)

(critical angle).

Because the sine of an angle cannot exceed unity, n2 cannot exceed n1 in this equation. This restriction tells us that total internal reflection cannot occur when the incident light is in the medium of lower index of refraction. If source S were in the air in Fig. 33-23a, all its rays that are incident on the air – glass interface (including f and g) would be both reflected and refracted at the interface. Total internal reflection has found many applications in medical technology. For example, a physician can view the interior of an artery of a patient by running two thin bundles of optical fibers through the chest wall and into an artery (Fig. 33-24). Light introduced at the outer end of one bundle undergoes repeated total internal reflection within the fibers so that, even though the bundle provides a curved path, most of the light ends up exiting the other end and illuminating the interior of the artery. Some of the light reflected from the interior then comes back up the second bundle in a similar way, to be detected and converted to an image on a monitor’s screen for the physician to view. The physician can then perform a surgical procedure, such as the placement of a stent.

©Laurent/Phototake

Figure 33-24 An endoscope used to inspect an artery.

33-7 POLARIZATION BY REFLECTION Learning Objectives After reading this module, you should be able to . . .

33.48 With sketches, explain how unpolarized light can be converted to polarized light by reflection from an interface. 33.49 Identify Brewster’s angle.

Key Idea

● A reflected wave will be fully polarized, with its

33.50 Apply the relationship between Brewster’s angle and the indexes of refraction on the two sides of an interface. 33.51 Explain the function of polarizing sunglasses.

:

E vectors perpendicular to the plane of incidence, if it strikes a boundary at the

Brewster angle uB, where u B ! tan&1

n2 n1

(Brewster angle).

998

CHAPTE R 33 E LECTROMAG N ETIC WAVES Incident unpolarized ray θB

Reflected ray

θB Air Glass

n = 1.5

θr

Refracted ray

Component perpendicular to page Component parallel to page

Figure 33-25 A ray of unpolarized light in air is incident on a glass surface at the Brewster angle uB. The electric fields along that ray have been resolved into components perpendicular to the page (the plane of incidence, reflection, and refraction) and components parallel to the page. The reflected light consists only of components perpendicular to the page and is thus polarized in that direction. The refracted light consists of the original components parallel to the page and weaker components perpendicular to the page; this light is partially polarized.

Polarization by Reflection You can vary the glare you see in sunlight that has been reflected from, say, water by looking through a polarizing sheet (such as a polarizing sunglass lens) and then rotating the sheet’s polarizing axis around your line of sight. You can do so because any light that is reflected from a surface is either fully or partially polarized by the reflection. Figure 33-25 shows a ray of unpolarized light incident on a glass surface. Let us resolve the electric field vectors of the light into two components. The perpendicular components are perpendicular to the plane of incidence and thus also to the page in Fig. 33-25; these components are represented with dots (as if we see the tips of the vectors). The parallel components are parallel to the plane of incidence and the page; they are represented with double-headed arrows. Because the light is unpolarized, these two components are of equal magnitude. In general, the reflected light also has both components but with unequal magnitudes. This means that the reflected light is partially polarized — the electric fields oscillating along one direction have greater amplitudes than those oscillating along other directions. However, when the light is incident at a particular incident angle, called the Brewster angle uB, the reflected light has only perpendicular components, as shown in Fig. 33-25. The reflected light is then fully polarized perpendicular to the plane of incidence.The parallel components of the incident light do not disappear but (along with perpendicular components) refract into the glass. Polarizing Sunglasses. Glass, water, and the other dielectric materials discussed in Module 25-5 can partially and fully polarize light by reflection. When you intercept sunlight reflected from such a surface, you see a bright spot (the glare) on the surface where the reflection takes place. If the surface is horizontal as in Fig. 33-25, the reflected light is partially or fully polarized horizontally. To eliminate such glare from horizontal surfaces, the lenses in polarizing sunglasses are mounted with their polarizing direction vertical.

Brewster’s Law For light incident at the Brewster angle uB, we find experimentally that the reflected and refracted rays are perpendicular to each other. Because the reflected ray is reflected at the angle uB in Fig. 33-25 and the refracted ray is at an angle ur, we have u B ' ur ! 90°. (33-46) These two angles can also be related with Eq. 33-40. Arbitrarily assigning subscript 1 in Eq. 33-40 to the material through which the incident and reflected rays travel, we have, from that equation, n1 sin uB ! n2 sin ur .

(33-47)

Combining these equations leads to n1 sin uB ! n2 sin(90° & uB) ! n2 cos uB,

(33-48)

which gives us u B ! tan&1

n2 n1

(Brewster angle).

(33-49)

(Note carefully that the subscripts in Eq. 33-49 are not arbitrary because of our decision as to their meanings.) If the incident and reflected rays travel in air, we can approximate n1 as unity and let n represent n2 in order to write Eq. 33-49 as uB ! tan&1 n

(Brewster’s law).

(33-50)

This simplified version of Eq. 33-49 is known as Brewster’s law. Like uB, it is named after Sir David Brewster, who found both experimentally in 1812.

R EVI EW & SU M MARY

999

(total reflection back along path).

(33-35)

Review & Summary Electromagnetic Waves An electromagnetic wave consists of oscillating electric and magnetic fields. The various possible frequencies of electromagnetic waves form a spectrum, a small part of which is visible light. An electromagnetic wave traveling along an x : : axis has an electric field E and a magnetic field B with magnitudes that depend on x and t: E ! Em sin(kx & vt) and

B ! Bm sin(kx & vt),

(33-1, 33-2) :

pr !

2I c

Polarization Electromagnetic waves are polarized if their electric field vectors are all in a single plane, called the plane of oscillation. From a head-on view, the field vectors oscillate parallel to a single axis perpendicular to the path taken by the waves. Light waves from common sources are not polarized; that is, they are unpolarized, or polarized randomly. From a head-on view, the vectors oscillate parallel to every possible axis that is perpendicular to the path taken by the waves.

:

where Em and Bm are the amplitudes of E and B . The oscillating electric field induces the magnetic field, and the oscillating magnetic field induces the electric field. The speed of any electromagnetic wave in vacuum is c, which can be written as c!

and

E 1 ! , m B 1 0´ 0

(33-5, 33-3)

where E and B are the simultaneous (but nonzero) magnitudes of the two fields.

Polarizing Sheets When a polarizing sheet is placed in the path of light, only electric field components of the light parallel to the sheet’s polarizing direction are transmitted by the sheet; components perpendicular to the polarizing direction are absorbed. The light that emerges from a polarizing sheet is polarized parallel to the polarizing direction of the sheet. If the original light is initially unpolarized, the transmitted intensity I is half the original intensity I0 : I ! 12 I0.

Energy Flow The rate per unit area at which energy is transported via an electromagnetic wave is given by the Poynting : vector S : :

S!

: 1 : E ( B. m0

(33-19)

If the original light is initially polarized, the transmitted intensity depends on the angle u between the polarization direction of the original light (the axis along which the fields oscillate) and the polarizing direction of the sheet:

:

The direction of S (and thus of the wave’s travel and the energy : : transport) is perpendicular to the directions of both E and B. The time-averaged rate per unit area at which energy is transported is Savg, which is called the intensity I of the wave: I!

1 E 2rms , cm0

(33-26)

in which Erms ! Em /1 2. A point source of electromagnetic waves emits the waves isotropically — that is, with equal intensity in all directions. The intensity of the waves at distance r from a point source of power Ps is I!

Ps . 4pr 2

(33-27)

Radiation Pressure When a surface intercepts electromagnetic radiation, a force and a pressure are exerted on the surface. If the radiation is totally absorbed by the surface, the force is F!

IA c

(total absorption),

(33-32)

in which I is the intensity of the radiation and A is the area of the surface perpendicular to the path of the radiation. If the radiation is totally reflected back along its original path, the force is F!

2IA c

(total reflection back along path).

(33-33)

The radiation pressure pr is the force per unit area: pr !

I c

(total absorption)

(33-34)

(33-36)

I ! I0 cos2 u.

(33-38)

Geometrical Optics Geometrical optics is an approximate treatment of light in which light waves are represented as straightline rays. Reflection and Refraction When a light ray encounters a boundary between two transparent media, a reflected ray and a refracted ray generally appear. Both rays remain in the plane of incidence. The angle of reflection is equal to the angle of incidence, and the angle of refraction is related to the angle of incidence by Snell’s law, n2 sin u 2 ! n1 sin u1 (refraction),

(33-40)

where n1 and n2 are the indexes of refraction of the media in which the incident and refracted rays travel.

Total Internal Reflection A wave encountering a boundary across which the index of refraction decreases will experience total internal reflection if the angle of incidence exceeds a critical angle uc, where uc ! sin&1

n2 n1

(critical angle).

(33-45)

Polarization by Reflection A reflected wave will be fully :

polarized, with its E vectors perpendicular to the plane of incidence, if the incident, unpolarized wave strikes a boundary at the Brewster angle uB, where u B ! tan&1

n2 n1

(Brewster angle).

(33-49)

1000

CHAPTE R 33 E LECTROMAG N ETIC WAVES

Questions 1 If the magnetic field of a light wave oscillates parallel to a y axis and is given by By ! Bm sin(kz & vt), (a) in what direction does the wave travel and (b) parallel to which axis does the associated electric field oscillate? I a

2 Suppose we rotate the second b sheet in Fig. 33-15a, starting with the c d polarization direction aligned with θ the y axis (u ! 0) and ending with it Figure 33-26 Question 2. aligned with the x axis (u ! 90°). Which of the four curves in Fig. 33-26 best shows the intensity of the light through the three-sheet system during this 90° rotation? 3 (a) Figure 33-27 shows light reaching a polarizing sheet whose polarizing direction is parallel to a y axis. We shall rotate the sheet 40° clockwise about the light’s indicated line of travel. During this rotation, does the fraction of the initial light intensity passed by the sheet increase, decrease, or remain the same if the light is (a) initially unpolarized, (b) initially polarized parallel to the x axis, and (c) initially polarized parallel to the y axis?

y

x

Figure 33-27 Question 3.

B

7 Figure 33-30 shows rays of monochromatic light propagating through three materials a, b, and c. Rank the materials according to the index of refraction, greatest first.

A n1

Figure 33-28 Question 4.

d

a

Figure 33-31 Question 8. Air 1.3

A

1.5

B

Air

Air

A

A

n2

n3

Figure 33-33 Question 10.

x

11 Each part of Fig. 33-34 shows light that refracts through an interface between two materials. The incident ray (shown gray in the figure) consists of red and blue light. The approximate index of refraction for visible light is indicated for each material. Which of the three parts show physically possible refraction? (Hint: First consider the refraction in general, regardless of the color, and then consider how red and blue light refract differently.)

1 n = 1.5

2 3 4 5

b

c

8 Figure 33-31 shows the multiple Figure 33-30 Question 7. reflections of a light ray along a glass corridor where the walls are either parallel or perpendicular to one another. If the angle of incidence at point a is 30°, what are

n = 1.3

n = 1.3

n = 1.5

n = 1.3

(a)

Figure 33-29 Question 6.

a

e b

10 The leftmost block in Fig. 33-33 depicts total internal reflection for light 1.4 C inside a material with an index of re1.3 D fraction n1 when air is outside the mateAir rial. A light ray reaching point A from anywhere within the shaded region at Figure 33-32 Question 9. the left (such as the ray shown) fully reflects at that point and ends up in the shaded region at the right. The other blocks show similar situations for two other materials. Rank the indexes of refraction of the three materials, greatest first.

E

5 In the arrangement of Fig. 33-15a, start with light that is initially polarized parallel to the x axis, and write the ratio of its final intensity I3 to its initial intensity I0 as I3/I0 ! A cosn u. What are A, n, and u if we rotate the polarizing direction of the first sheet (a) 60" counterclockwise and (b) 90° clockwise from what is shown? y 6 In Fig. 33-29, unpolarized light is sent into a system of five polarizing sheets. Their polarizing directions, measured counterclockwise from the positive direction of the y axis, are the following: sheet 1, 35"; sheet 2, 0"; sheet 3, 0"; sheet 4, 110"; sheet 5, 45". Sheet 3 is then rotated 180° counterclockwise about the light ray. During that rotation, at what angles (measured counterclockwise from the y axis) is the transmission of light through the system eliminated?

c

9 Figure 33-32 shows four long horizontal layers A – D of different materials, with air above and below them. The index of refraction of each material is given. Rays of light are sent into the left end of each layer as shown. In which layer is there the possibility of totally trapping the light in that layer so that, after many reflections, all the light reaches the right end of the layer?

Air

4 Figure 33-28 shows the electric and magnetic fields of an electromagnetic wave at a certain instant. Is the wave traveling into the page or out of the page?

f

the angles of reflection of the light ray at points b, c, d, e, and f ?

n = 1.5

(b)

(c)

Figure 33-34 Question 11. 12 In Fig. 33-35, light travels from material a, through three layers of other materials with surfaces parallel to one another, and then back into another layer of material a. The refractions (but not the associated reflections) at the surfaces are shown. Rank the materials according to index of refraction, greatest first. (Hint: The parallel arrangement of the surfaces allows comparison.)

a

b

d

a

c

Figure 33-35 Question 12.

1001

PROB LE M S

Problems Tutoring problem available (at instructor’s discretion) in WileyPLUS and WebAssign SSM • – •••

Worked-out solution available in Student Solutions Manual

WWW Worked-out solution is at

Number of dots indicates level of problem difficulty

ILW

Interactive solution is at

http://www.wiley.com/college/halliday

Additional information available in The Flying Circus of Physics and at flyingcircusofphysics.com

Module 33-1 Electromagnetic Waves •1 A certain helium – neon laser emits red light in a narrow band of wavelengths centered at 632.8 nm and with a “wavelength width” (such as on the scale of Fig. 33-1) of 0.0100 nm. What is the corresponding “frequency width” for the emission? •2 Project Seafarer was an ambitious program to construct an enormous antenna, buried underground on a site about 10 000 km2 in area. Its purpose was to transmit signals to submarines while they were deeply submerged. If the effective wavelength were 1.0 ( 104 Earth radii, what would be the (a) frequency and (b) period of the radiations emitted? Ordinarily, electromagnetic radiations do not penetrate very far into conductors such as seawater, and so normal signals cannot reach the submarines. •3 From Fig. 33-2, approximate the (a) smaller and (b) larger wavelength at which the eye of a standard observer has half the eye’s maximum sensitivity. What are the (c) wavelength, (d) frequency, and (e) period of the light at which the eye is the most sensitive? •4 About how far apart must you hold your hands for them to be separated by 1.0 nano-light-second (the distance light travels in 1.0 ns)? •5 SSM What inductance must be connected to a 17 pF capacitor in an oscillator capable of generating 550 nm (i.e., visible) electromagnetic waves? Comment on your answer.

••13 Sunlight just outside Earth’s atmosphere has an intensity of 1.40 kW/m2. Calculate (a) Em and (b) Bm for sunlight there, assuming it to be a plane wave. ••14 An isotropic point source emits light at wavelength 500 nm, at the rate of 200 W. A light detector is positioned 400 m from the source. What is the maximum rate )B/)t at which the magnetic component of the light changes with time at the detector’s location? ••15 An airplane flying at a distance of 10 km from a radio transmitter receives a signal of intensity 10 mW/m2. What is the amplitude of the (a) electric and (b) magnetic component of the signal at the airplane? (c) If the transmitter radiates uniformly over a hemisphere, what is the transmission power? ••16 Frank D. Drake, an investigator in the SETI (Search for Extra-Terrestrial Intelligence) program, once said that the large radio telescope in Arecibo, Puerto Rico (Fig. 33-36), “can detect a signal which lays down on the entire surface of the earth a power of only one picowatt.” (a) What is the power that would be received by the Arecibo antenna for such a signal? The antenna diameter is 300 m. (b) What would be the power of an isotropic source at the center of our galaxy that could provide such a signal? The galactic center is 2.2 ( 104 ly away. A light-year is the distance light travels in one year.

•6 What is the wavelength of the electromagnetic wave emitted by the oscillator – antenna system of Fig. 33-3 if L ! 0.253 mH and C ! 25.0 pF? Module 33-2 Energy Transport and the Poynting Vector •7 What is the intensity of a traveling plane electromagnetic wave if Bm is 1.0 ( 10&4 T? •8 Assume (unrealistically) that a TV station acts as a point source broadcasting isotropically at 1.0 MW. What is the intensity of the transmitted signal reaching Proxima Centauri, the star nearest our solar system, 4.3 ly away? (An alien civilization at that distance might be able to watch X Files.) A light-year (ly) is the distance light travels in one year. •9 ILW Some neodymium – glass lasers can provide 100 TW of power in 1.0 ns pulses at a wavelength of 0.26 mm. How much energy is contained in a single pulse?

Courtesy SRI International, USRA, UMET

Figure 33-36 Problem 16. Radio telescope at Arecibo.

•11 ILW A plane electromagnetic wave traveling in the positive direction of an x axis in vacuum has components Ex ! Ey ! 0 and Ez ! (2.0 V/m) cos[(p ( 1015 s&1)(t & x/c)]. (a) What is the amplitude of the magnetic field component? (b) Parallel to which axis does the magnetic field oscillate? (c) When the electric field component is in the positive direction of the z axis at a certain point P, what is the direction of the magnetic field component there?

••17 The maximum electric field 10 m from an isotropic point source of light is 2.0 V/m. What are Is (a) the maximum value of the magnetic field and (b) the average intensity of the light there? (c) What is the power of the source?

•12 In a plane radio wave the maximum value of the electric field component is 5.00 V/m. Calculate (a) the maximum value of the magnetic field component and (b) the wave intensity.

I (W/m2)

•10 A plane electromagnetic wave has a maximum electric field magnitude of 3.20 ( 10&4 V/m. Find the magnetic field amplitude.

••18 The intensity I of light from an isotropic point source is determined as a function of distance r from the source. Figure 33-37 gives

0 r

–2

–2

r s–2

(m )

Figure 33-37 Problem 18.

1002

CHAPTE R 33 E LECTROMAG N ETIC WAVES

intensity I versus the inverse square r&2 of that distance. The vertical axis scale is set by Is ! 200 W/m2, and the horizontal axis scale is &2 set by r &2 s ! 8.0 m . What is the power of the source? Module 33-3 Radiation Pressure •19 SSM High-power lasers are used to compress a plasma (a gas of charged particles) by radiation pressure. A laser generating radiation pulses with peak power 1.5 ( 103 MW is focused onto 1.0 mm2 of high-electron-density plasma. Find the pressure exerted on the plasma if the plasma reflects all the light beams directly back along their paths. •20 Radiation from the Sun reaching Earth (just outside the atmosphere) has an intensity of 1.4 kW/m2. (a) Assuming that Earth (and its atmosphere) behaves like a flat disk perpendicular to the Sun’s rays and that all the incident energy is absorbed, calculate the force on Earth due to radiation pressure. (b) For comparison, calculate the force due to the Sun’s gravitational attraction. •21 ILW What is the radiation pressure 1.5 m away from a 500 W lightbulb? Assume that the surface on which the pressure is exerted faces the bulb and is perfectly absorbing and that the bulb radiates uniformly in all directions. •22 A black, totally absorbing piece of cardboard of area A ! 2.0 cm2 intercepts light with an intensity of 10 W/m2 from a camera strobe light. What radiation pressure is produced on the cardboard by the light? ••23 Someone plans to float a small, totally absorbing sphere 0.500 m above an isotropic point source of light, so that the upward radiation force from the light matches the downward gravitational force on the sphere. The sphere’s density is 19.0 g/cm3, and its radius is 2.00 mm. (a) What power would be required of the light source? (b) Even if such a source were made, why would the support of the sphere be unstable? ••24 It has been proposed that a spaceship might be propelled in the solar system by radiation pressure, using a large sail made of foil. How large must the surface area of the sail be if the radiation force is to be equal in magnitude to the Sun’s gravitational attraction? Assume that the mass of the ship ' sail is 1500 kg, that the sail is perfectly reflecting, and that the sail is oriented perpendicular to the Sun’s rays. See Appendix C for needed data. (With a larger sail, the ship is continuously driven away from the Sun.) ••25 SSM Prove, for a plane electromagnetic wave that is normally incident on a flat surface, that the radiation pressure on the surface is equal to the energy density in the incident beam. (This relation between pressure and energy density holds no matter what fraction of the incident energy is reflected.) ••26 In Fig. 33-38, a laser beam of power 4.60 W and diameter D ! 2.60 mm is directed upward at one circular face (of diameter d % 2.60 mm) of a perfectly reflecting cylinder. The cylinder is levitated because the upward radiation force matches the downward gravitational force. If the cylinder’s density is 1.20 g/cm3, what is its height H?

the y axis. What are the (a) frequency, (b) angular frequency, and (c) angular wave number of the wave? (d) What is the amplitude of the magnetic field component? (e) Parallel to which axis does the magnetic field oscillate? (f) What is the time-averaged rate of energy flow in watts per square meter associated with this wave? The wave uniformly illuminates a surface of area 2.0 m2. If the surface totally absorbs the wave, what are (g) the rate at which momentum is transferred to the surface and (h) the radiation pressure on the surface? ••28 The average intensity of the solar radiation that strikes normally on a surface just outside Earth’s atmosphere is 1.4 kW/m2. (a) What radiation pressure pr is exerted on this surface, assuming complete absorption? (b) For comparison, find the ratio of pr to Earth’s sea-level atmospheric pressure, which is 1.0 ( 105 Pa. ••29 SSM A small spaceship with a mass of only 1.5 ( 103 kg (including an astronaut) is drifting in outer space with negligible gravitational forces acting on it. If the astronaut turns on a 10 kW laser beam, what speed will the ship attain in 1.0 day because of the momentum carried away by the beam? ••30 A small laser emits light at power 5.00 mW and wavelength 633 nm. The laser beam is focused (narrowed) until its diameter matches the 1266 nm diameter of a sphere placed in its path. The sphere is perfectly absorbing and has density 5.00 ( 10 3 kg/m3. What are (a) the beam intensity at the sphere’s location, (b) the radiation pressure on the sphere, (c) the magnitude of the corresponding force, and (d) the magnitude of the acceleration that force alone would give the sphere? •••31 As a comet swings around the Sun, ice on the comet’s surface 1 Dust tail vaporizes, releasing trapped dust particles and ions. The ions, because Sun 3 they are electrically charged, are 2 forced by the electrically charged Comet solar wind into a straight ion tail Ion tail that points radially away from the Sun (Fig. 33-39). The (electrically Figure 33-39 Problem 31. neutral) dust particles are pushed radially outward from the Sun by the radiation force on them from sunlight. Assume that the dust particles are spherical, have density 3.5 ( 103 kg/m3, and are totally absorbing. (a) What radius must a particle have in order to follow a straight path, like path 2 in the figure? (b) If its radius is larger, does its path curve away from the Sun (like path 1) or toward the Sun (like path 3)? Module 33-4 Polarization •32 In Fig. 33-40, initially unpolarized light is sent into a system of three polarizing sheets whose polarizing directions make angles

D

θ1 A

y x

H

••27 SSM WWW A plane electromagFigure 33-38 netic wave, with wavelength 3.0 m, travels Problem 26. in vacuum in the positive direction of an x axis. The electric field, of amplitude 300 V/m, oscillates parallel to

θ3 θ2

Figure 33-40 Problems 32 and 33.

PROB LE M S

of u1 ! u2 ! u3 ! 50" with the direction of the y axis. What percentage of the initial intensity is transmitted by the system? (Hint: Be careful with the angles.) •33 SSM In Fig. 33-40, initially unpolarized light is sent into a system of three polarizing sheets whose polarizing directions make angles of u1 ! 40°, u2 ! 20°, and u3 ! 40° with the direction of the y axis. What percentage of the light’s initial intensity is transmitted by the system? (Hint: Be careful with the angles.) y

•34 In Fig. 33-41, a beam of unpolarized light, with intensity 43 W/m2, is sent into a system of two polarizing sheets with polarizing directions at angles u1 ! 70° and u2 ! 90° to the y axis. What is the intensity of the light transmitted by the system?

θ1

x

θ2

•35 ILW In Fig. 33-41, a beam of light, with Figure 33-41 intensity 43 W/m2 and polarization parallel Problems 34, 35, to a y axis, is sent into a system of two and 42. polarizing sheets with polarizing directions at angles of u1 ! 70" and u2 ! 90" to the y axis. What is the intensity of the light transmitted by the two-sheet system? ••36 At a beach the light is generally partially polarized due to reflections off sand and water. At a particular beach on a particular day near sundown, the horizontal component of the electric field vector is 2.3 times the vertical component. A standing sunbather puts on polarizing sunglasses; the glasses eliminate the horizontal field component. (a) What fraction of the light intensity received before the glasses were put on now reaches the sunbather’s eyes? (b) The sunbather, still wearing the glasses, lies on his side. What fraction of the light intensity received before the glasses were put on now reaches his eyes? ••37 SSM WWW We want to rotate the direction of polarization of a beam of polarized light through 90° by sending the beam through one or more polarizing sheets. (a) What is the minimum number of sheets required? (b) What is the minimum number of sheets required if the transmitted intensity is to be more than 60% of the original intensity? ••38 In Fig. 33-42, unpolarized light is sent into a system of three polarizing sheets. The angles u1, u2, and u3 of the polarizing directions are measured counterclockwise from the positive direction of the y axis (they are not drawn to scale). Angles u1 and u3 are fixed, but angle u2 can be varied. Figure 33-43 gives the intensity of the light emerging from sheet 3 as a function of u2. (The scale of the intensity axis is not indicated.) What percentage of the light’s initial intensity is transmitted by the system when u2 ! 30°?

θ2

x I

θ3

Figure 33-42 Problems 38, 40, and 44.

0 0°

••39 Unpolarized light of intensity 10 mW/m2 is sent into a polarizing sheet as in Fig. 33-11. What are (a) the amplitude of the electric field component of the transmitted light and (b) the radiation pressure on the sheet due to its absorbing some of the light? ••40 In Fig. 33-42, unpolarized light is sent into a system of three polarizing sheets. The angles u1, u2, and u3 of the polarizing directions are measured counterclockwise from the positive direction of the y axis (they are not drawn to scale). Angles u1 and u3 are fixed, but angle u2 can be varied. Figure 33-44 gives the intensity of the light emerging from sheet 3 as a function of u2. (The scale of the intensity axis is not indicated.) What percentage of the light’s initial intensity is transmitted by the three-sheet system when u2 ! 90°? I

0 0°

60°

120°

180°

θ2

Figure 33-44 Problem 40. ••41 A beam of polarized light is sent into a system of two polarizing sheets. Relative to the polarization direction of that incident light, the polarizing directions of the sheets are at angles u for the first sheet and 90° for the second sheet. If 0.10 of the incident intensity is transmitted by the two sheets, what is u? ••42 In Fig. 33-41, unpolarized light is sent into a system of two polarizing sheets. The angles u1 and u2 of the polarizing directions of the sheets are measured counterclockwise from the positive direction of the y axis (they are not drawn to scale in the figure). Angle u1 is fixed but angle u2 can be varied. Figure 33-45 gives the intensity of the light emerging from sheet 2 as a function of u2. (The scale of the intensity axis is not indicated.) What percentage of the light’s initial intensity is transmitted by the two-sheet system when u2 ! 90°? I

0 90°

θ2

180°

Figure 33-45 Problem 42. ••43 A beam of partially polarized light can be considered to be a mixture of polarized and unpolarized light. Suppose we send such a beam through a polarizing filter and then rotate the filter through 360° while keeping it perpendicular to the beam. If the transmitted intensity varies by a factor of 5.0 during the rotation, what fraction of the intensity of the original beam is associated with the beam’s polarized light?

y

θ1

1003

90°

180°

Figure 33-43 Problem 38.

θ2

••44 In Fig. 33-42, unpolarized light is sent into a system of three polarizing sheets, which transmits 0.0500 of the initial light intensity. The polarizing directions of the first and third sheets are at angles u1 ! 0° and u3 ! 90°. What are the (a) smaller and (b) larger possible values of angle u2 (% 90°) for the polarizing direction of sheet 2?

1004

CHAPTE R 33 E LECTROMAG N ETIC WAVES

Module 33-5 Reflection and Refraction •45 When the rectangular metal tank in Fig. 33-46 is filled to the top with an unknown liquid, observer O O, with eyes level with the top of the tank, can just see corner E. A ray that refracts toward O at the top surface of the liquid is shown. If D ! 85.0 cm and L ! 1.10 m, what is the index of refraction of the liquid?

•49 Figure 33-49 shows light reflecting from two perpendicular reflecting surfaces A and B. Find the angle between the incoming ray i and the outgoing ray r#.

Normal to liquid surface

D

E

L

•46 In Fig. 33-47a, a light ray in Figure 33-46 Problem 45. an underlying material is incident at angle u1 on a boundary with water, and some of the light refracts into the water. There are two choices of underlying material. For each, the angle of refraction u2 versus the incident angle u1 is given in Fig. 33-47b. The horizontal axis scale is set by u1s ! 90°. Without calculation, determine whether the index of refraction of (a) material 1 and (b) material 2 is greater or less than the index of water (n ! 1.33). What is the index of refraction of (c) material 1 and (d) material 2?

i θ r

A

r'

B

••50 In Fig. 33-50a, a beam of light Figure 33-49 Problem 49. in material 1 is incident on a boundary at an angle u1 ! 40°. Some of the light travels through material 2, and then some of it emerges into material 3. The two boundaries between the three materials are parallel. The final direction of the beam depends, in part, on the index of refraction n3 of the third material. Figure 33-50b gives the angle of refraction u3 in that material versus n3 for a range of possible n3 values. The vertical axis scale is set by u3a ! 30.0° and u3b ! 50.0°. (a) What is the index of refraction of material 1, or is the index impossible to calculate without more information? (b) What is the index of refraction of material 2, or is the index impossible to calculate without more information? (c) If u1 is changed to 70° and the index of refraction of material 3 is 2.4, what is u3?

1

2

3

θ 3b

θ3

θ2 90° 1

Water

θ1 2

45°

θ 3a

1.5

1.7

(a)

θ 1s

0° (a)

Figure 33-47 Problem 46. •47 Light in vacuum is incident on the surface of a glass slab. In the vacuum the beam makes an angle of 32.0° with the normal to the surface, while in the glass it makes an angle of 21.0° with the normal. What is the index of refraction of the glass? •48 In Fig. 33-48a, a light ray in water is incident at angle u1 on a boundary with an underlying material, into which some of the light refracts. There are two choices of underlying material. For each, the angle of refraction u2 versus the incident angle u1 is given in Fig. 33-48b. The vertical axis scale is set by u2s ! 90°. Without calculation, determine whether the index of refraction of (a) material 1 and (b) material 2 is greater or less than the index of water (n ! 1.33). What is the index of refraction of (c) material 1 and (d) material 2?

θ 2s

Figure 33-50 Problem 50. ••51 In Fig. 33-51, light is incident at angle u1 ! 40.1" on a boundary between two transparent materials. Some of the light travels down through the next three layers of transparent materials, while some of it reflects upward and then escapes into the air. If n1 ! 1.30, n2 ! 1.40, n3 ! 1.32, and n4 ! 1.45, what is the value of (a) u5 in the air and (b) u4 in the bottom material?

1 1



45° (b)

Figure 33-48 Problem 48.

2

θ5 θ1

Air

n1 n2 n3 n4

θ4

••52 In Fig. 33-52a, a beam of light Figure 33-51 Problem 51. in material 1 is incident on a boundary at an angle of u1 = 30°. The extent of refraction of the light into material 2 depends, in part, on the index of refraction n2 of material 2. Figure 33-52b gives the angle of refraction u2 versus n2 for a range of possible n2 values. The vertical axis scale is set by u2a ! 20.0° and u2b ! 40.0°. (a) What is the index of refraction of

θ2

Water

(a)

(b)

θ1

(b)

n3

1.9

2

θ 2b

θ2

θ1 θ 2a 1.3

θ1 90°

1.5

(a)

1.7 (b)

Figure 33-52 Problem 52.

1.9

n2

1005

PROB LE M S

material 1? (b) If the incident angle is changed to 60° and material 2 has n2 ! 2.4, then what is angle u2? ••53 SSM WWW ILW In Fig. 33-53, a ray is incident on one face of a triangular glass prism in air. The angle of incidence u is chosen so that the emerging ray also makes the same angle u with the normal to the other face. Show that the index of refraction n of the glass prism is given by n!

sin 12(c ' f) sin

1 2f

φ

θ

θ

Figure 33-53 Problems 53 and 64. ••54 Dispersion in a window pane. In Fig. 33-54, a beam of white light is incident at angle u ! 50" on a common window pane (shown in cross section). For the pane’s type of glass, the θ index of refraction for visible light ranges from 1.524 at the blue end of the spectrum to 1.509 at the red end. The two sides of the pane are parallel. What is the angular spread of the colors in the Figure 33-54 beam (a) when the light enters the pane and Problem 54. (b) when it emerges from the opposite side? (Hint: When you look at an object through a window pane, are the colors in the light from the object dispersed as shown in, say, Fig. 33-20?) SSM In Fig. 33-55, a 2.00••55 m-long vertical pole extends from the bottom of a swimming pool to a point 50.0 cm above the water. Sunlight is incident at angle u ! 55.0°. What is the length of the shadow of the pole on the level bottom of the pool?

Module 33-6 Total Internal Reflection •57 A point source of light is 80.0 cm below the surface of a body of water. Find the diameter of the circle at the surface through which light emerges from the water. •58 The index of refraction of benzene is 1.8. What is the critical angle for a light ray traveling in benzene toward a flat layer of air above the benzene?

,

where f is the vertex angle of the prism and c is the deviation angle, the total angle through which the beam is turned in passing through the prism. (Under these conditions the deviation angle c has the smallest possible value, which is called the angle of minimum deviation.)

ψ

(a) point A and (b) point B? (This angular difference in the light emerging at, say, point A would be the rainbow’s angular width.)

Blocked sunrays

θ

••56 Rainbows from square Figure 33-55 Problem 55. drops. Suppose that, on some surreal world, raindrops had a square cross section and always fell with one face horizontal. Figure 33-56 shows such a falling drop, with a θ white beam of sunlight incident at u ! 70.0° at point P. The part of the light that enters the P drop then travels to point A, where some of it refracts out into the air and the rest reflects. A That reflected light then travels to point B, where again some of the light refracts out into B the air and the rest reflects. What is the difference in the angles of the red light (n ! 1.331) Figure 33-56 and the blue light (n ! 1.343) that emerge at Problem 56.

••59 SSM ILW In Fig. 33-57, a ray of light is perpendicular to the face ab of a glass prism (n ! 1.52). Find the largest value for the angle f so that the ray is totally reflected at face ac if the prism is immersed (a) in air and (b) in water.

a

φ

b

c

Figure 33-57 Problem 59.

••60 In Fig. 33-58, light from ray A refracts from material 1 (n1 ! 1.60) n3 into a thin layer of material 2 (n2 ! n2 1.80), crosses that layer, and is then θB incident at the critical angle on the interface between materials 2 and 3 θA n1 (n3 ! 1.30). (a) What is the value of incident angle uA? (b) If uA is Figure 33-58 Problem 60. decreased, does part of the light refract into material 3? Light from ray B refracts from material 1 into the thin layer, crosses that layer, and is then incident at the critical angle on the interface between materials 2 and 3. (c) What is the value of incident angle uB? (d) If uB is decreased, does part of the light refract into material 3? ••61 In Fig. 33-59, light initially in material 1 refracts into material 2, crosses that material, and is then incident at the critical angle on the interface between materials 2 and 3. The indexes of refraction are n1 ! 1.60, n2 ! 1.40, and n3 ! 1.20. (a) What is angle u? (b) If u is increased, is there refraction of light into material 3?

θ

n1

n3

n2

••62 A catfish is 2.00 m below the surface of a smooth lake. Figure 33-59 Problem 61. (a) What is the diameter of the circle on the surface through which the fish can see the world outside the water? (b) If the fish descends, does the diameter of the circle increase, decrease, or remain the same? Air

••63 In Fig. 33-60, light enters a 90° triangular prism at point P with inciθ dent angle u, and then some of it P Q refracts at point Q with an angle of refraction of 90°. (a) What is the index of refraction of the prism in Figure 33-60 Problem 63. terms of u? (b) What, numerically, is the maximum value that the index of refraction can have? Does light emerge at Q if the incident angle at P is (c) increased slightly and (d) decreased slightly?

1006

CHAPTE R 33 E LECTROMAG N ETIC WAVES

••64 Suppose the prism of Fig. 33-53 has apex angle f ! 60.0° and index of refraction n ! 1.60. (a) What is the smallest angle of incidence u for which a ray can enter the left face of the prism and exit the right face? (b) What angle of incidence u is required for the ray to exit the prism with an identical angle u for its refraction, as it does in Fig. 33-53? ••65 Figure 33-61 depicts a simn2 plistic optical fiber: a plastic core A (n1 ! 1.58) is surrounded by a plastic sheath (n2 ! 1.53). A light ray is incin1 dent on one end of the fiber at angle θ u. The ray is to undergo total internal reflection at point A, where it enFigure 33-61 Problem 65. counters the core – s h e a t h boundary. (Thus there is no loss of light through that boundary.) What is the maximum value of u that allows total internal reflection at A? ••66 In Fig. 33-62, a light ray in W air is incident at angle u1 on a block 2 of transparent plastic with an index of refraction of 1.56. The dimenH sions indicated are H ! 2.00 cm 1 3 and W ! 3.00 cm. The light passes through the block to one of its sides and there undergoes reflection (in- θ 1 4 side the block) and possibly refraction (out into the air). This is Figure 33-62 Problem 66. the point of first reflection. The reflected light then passes through the block to another of its sides — a point of second reflection. If u1 ! 40°, on which side is the point of (a) first reflection and (b) second reflection? If there is refraction at the point of (c) first reflection and (d) second reflection, give the angle of refraction; if not, answer “none.” If u1 ! 70°, on which side is the point of (e) first reflection and (f) second reflection? If there is refraction at the point of (g) first reflection and (h) second reflection, give the angle of refraction; if not, answer “none.” ••67 In the ray diagram of Fig. 33-63, where the angles are not drawn to scale, the ray is incident at the critical angle on the interface between materials 2 and 3. Angle f ! 60.0°, and two of the indexes of refraction are n1 ! 1.70 and n2 ! 1.60. Find (a) index of refraction n3 and (b) angle u. (c) If u is decreased, does light refract into material 3?

•69 SSM Light that is traveling in water (with an index of refraction of 1.33) is incident on a plate of glass (with index of refraction 1.53). At what angle of incidence does the reflected light end up fully polarized? ••70 In Fig. 33-64, a light ray in air is incident on a flat layer of material 2 that has an index of refraction n2 ! 1.5. Beneath material 2 is material 3 with an index of refraction n3. The ray is incident on the air–material 2 interface at the Brewster angle for that interface. The ray of light refracted into material 3 happens to be incident on the material 2 – material 3 interface at the Brewster angle for that interface. What is the value of n3? θ1 Air

θ2

n2 n3

Figure 33-64 Problem 70. Additional Problems 71 SSM (a) How long does it take a radio signal to travel 150 km from a transmitter to a receiving antenna? (b) We see a full Moon by reflected sunlight. How much earlier did the light that enters our eye leave the Sun? The Earth – Moon and Earth – Sun distances are 3.8 ( 105 km and 1.5 ( 108 km, respectively. (c) What is the round-trip travel time for light between Earth and a spaceship orbiting Saturn, 1.3 ( 109 km distant? (d) The Crab nebula, which is about 6500 light-years (ly) distant, is thought to be the result of a supernova explosion recorded by Chinese astronomers in A.D. 1054. In approximately what year did the explosion actually occur? (When we look into the night sky, we are effectively looking back in time.) 72 An electromagnetic wave with frequency 4.00 ( 1014 Hz travels through vacuum in the positive direction of an x axis. The wave has its electric field oscillating parallel to the y axis, with an amplitude Em. At time t ! 0, the electric field at point P on the x axis has a value of 'Em /4 and is decreasing with time. What is the distance along the x axis from point P to the first point with E ! 0 if we search in (a) the negative direction and (b) the positive direction of the x axis? 73

SSM

The electric component of a beam of polarized light is Ey ! (5.00 V/m) sin[(1.00 ( 106 m&1)z ' vt].

n2 n3

(a) Write an expression for the magnetic field component of the wave, including a value for v. What are the (b) wavelength, (c) period, and (d) intensity of this light? (e) Parallel to which axis does the magnetic field oscillate? (f) In which region of the electromagnetic spectrum is this wave?

Figure 33-63 Problem 67.

74 A particle in the solar system is under the combined influence of the Sun’s gravitational attraction and the radiation force due to the Sun’s rays. Assume that the particle is a sphere of density 1.0 ( 103 kg/m3 and that all the incident light is absorbed. (a) Show that, if its radius is less than some critical radius R, the particle will be blown out of the solar system. (b) Calculate the critical radius.

φ θ n1

Module 33-7 Polarization by Reflection •68 (a) At what angle of incidence will the light reflected from water be completely polarized? (b) Does this angle depend on the wavelength of the light?

PROB LE M S

75 SSM In Fig. 33-65, a light ray enters a glass slab at point A at incident angle u1 ! 45.0° and then undergoes total internal reflection at point B. (The reflection at A is not shown.) What minimum value for the index of refraction of the glass can be inferred from this information?

θ1 Air

Incident ray

A

Glass

76 In Fig. 33-66, unpolarized B light with an intensity of 25 W/m2 is sent into a system of four polarizing sheets with polarizing directions at Figure 33-65 Problem 75. angles u1 ! 40°, u2 ! 20°, u3 ! 20°, and u4 ! 30°. What is the intensity of the light that emerges from the system? y

θ1

x

1007

for (c) red light and (d) blue light. (e) If these colors form the inner and outer edges of a rainbow (Fig. 33-21a), what is the angular width of the rainbow? 78 The primary rainbow described in Problem 77 is the type commonly seen in regions where rainbows appear. It is produced by light reflecting once inside the drops. Rarer is the secondary rainbow described in Module 33-5, produced by light reflecting twice inside the drops (Fig. 33-68a). (a) Show that the angular deviation of light entering and then leaving a spherical water drop is udev ! (180°)k ' 2ui & 2(k ' 1)ur , where k is the number of internal reflections. Using the procedure of Problem 77, find the angle of minimum deviation for (b) red light and (c) blue light in a secondary rainbow. (d) What is the angular width of that rainbow (Fig. 33-21d)? The tertiary rainbow depends on three internal reflections (Fig. 33-68b). It probably occurs but, as noted in Module 33-5, cannot be seen with the eye because it is very faint and lies in the bright sky surrounding the Sun. What is the angle of minimum deviation for (e) the red light and (f) the blue light in this rainbow? (g) What is the rainbow’s angular width?

θ2 θ3

θ4

(a)

(b)

Figure 33-66 Problem 76.

Figure 33-68 Problem 78.

77 Rainbow. Figure 33-67 shows a light ray entering and then leaving a falling, spherical raindrop after one internal reflection (see Fig. 33-21a). The final direction of travel is deviated (turned) from the initial direction of travel by angular deviation udev. (a) Show that udev is

79 SSM (a) Prove that a ray of light incident on the surface of a sheet of plate glass of thickness t emerges from the opposite face parallel to its initial direction but displaced sideways, as in Fig. 33-69. (b) Show that, for small angles of incidence u, this displacement is given by n&1 x ! tu , n

udev ! 180° ' 2ui & 4ur , where ui is the angle of incidence of the ray on the drop and ur is the angle of refraction of the ray within the drop. (b) Using Snell’s law, substitute for ur in terms of ui and the index of refraction n of the water. Then, on a graphing calculator or with a computer graphing package, graph udev versus ui for the range of possible ui values and for n ! 1.331 for red light (at one end of the visible spectrum) and n ! 1.333 for blue light (at the other end). The red-light curve and the blue-light curve have different minima, which means that there is a different angle of minimum deviation for each color. The light of Water drop any given color that leaves the drop θi at that color’s angle of minimum deviation is especially bright because Incident θr rays bunch up at that angle. Thus, the ray bright red light leaves the drop at one angle and the bright blue light leaves it at another angle. Determine the angle of miniFigure 33-67 Problem 77. mum deviation from the udev curve

where n is the index of refraction of the glass and u is measured in radians.

θ θ

x

t

Figure 33-69 Problem 79. 80 An electromagnetic wave is traveling in the negative direction of a y axis. At a particular position and time, the electric field is directed along the positive direction of the z axis and has a magnitude of 100 V/m. What are the (a) magnitude and (b) direction of the corresponding magnetic field?

1008

CHAPTE R 33 E LECTROMAG N ETIC WAVES

81 The magnetic component of a polarized wave of light is Bx ! (4.0 ( 10&6 T) sin[(1.57 ( 107 m&1)y ' vt]. (a) Parallel to which axis is the light polarized? What are the (b) frequency and (c) intensity of the light? 82 In Fig. 33-70, unpolarized light is sent into the system of three polarizing sheets, where the polarizing directions of the first and third sheets are at angles u1 ! 30° (counterclockwise) and u3 ! 30° (clockwise). What fraction of the initial light intensity emerges from the system?

θ1

at angle ub ! 20°. Relative to a normal at the last interface, at what angle do (a) ray a and (b) ray b emerge? (Hint: Solving the problem algebraically can save time.) If the air at the left and right sides in the figure were, instead, glass with index of refraction 1.5, at what angle would (c) ray a and (d) ray b emerge?

y x a

θ3

83 SSM A ray of white light traveling through fused quartz is incident at a quartz – air interface at angle u1. Figure 33-70 Problem 82. Assume that the index of refraction of quartz is n ! 1.456 at the red end of the visible range and n ! 1.470 at the blue end. If u1 is (a) 42.00°, (b) 43.10°, and (c) 44.00°, is the refracted light white, white dominated by the red end of the visible range, or white dominated by the blue end of the visible range, or is there no refracted light? 84 Three polarizing sheets are stacked. The first and third are crossed; the one between has its polarizing direction at 45.0° to the polarizing directions of the other two. What fraction of the intensity of an originally unpolarized beam is transmitted by the stack? 85 In a region of space where gravitational forces can be neglected, a sphere is accelerated by a uniform light beam of intensity 6.0 mW/m2. The sphere is totally absorbing and has a radius of 2.0 mm and a uniform density of 5.0 ( 10 3 kg/m3. What is the magnitude of the sphere’s acceleration due to the light? 86 An unpolarized beam of light is sent into a stack of four polarizing sheets, oriented so that the angle between the polarizing directions of adjacent sheets is 30°. What fraction of the incident intensity is transmitted by the system? 87 SSM During a test, a NATO surveillance radar system, operating at 12 GHz at 180 kW of power, attempts to detect an incoming stealth aircraft at 90 km. Assume that the radar beam is emitted uniformly over a hemisphere. (a) What is the intensity of the beam when the beam reaches the aircraft’s location? The aircraft reflects radar waves as though it has a cross-sectional area of only 0.22 m2. (b) What is the power of the aircraft’s reflection? Assume that the beam is reflected uniformly over a hemisphere. Back at the radar site, what are (c) the intensity, (d) the maximum value of the electric field vector, and (e) the rms value of the magnetic field of the reflected radar beam? 88 The magnetic component of an electromagnetic wave in vacuum has an amplitude of 85.8 nT and an angular wave number of 4.00 m&1. What are (a) the frequency of the wave, (b) the rms value of the electric component, and (c) the intensity of the light? 89 Calculate the (a) upper and (b) lower limit of the Brewster angle for white light incident on fused quartz. Assume that the wavelength limits of the light are 400 and 700 nm. 90 In Fig. 33-71, two light rays pass from air through five layers of transparent plastic and then back into air. The layers have parallel interfaces and unknown thicknesses; their indexes of refraction are n1 ! 1.7, n2 ! 1.6, n3 ! 1.5, n4 ! 1.4, and n5 ! 1.6. Ray b is incident

θb b

Air

n1

n2

n3

n4

n5

Air

Figure 33-71 Problem 90. 91 A helium – neon laser, radiating at 632.8 nm, has a power output Laser θ of 3.0 mW. The beam diverges (spreads) at angle u ! 0.17 mrad Figure 33-72 Problem 91. (Fig. 33-72). (a) What is the intensity of the beam 40 m from the laser? (b) What is the power of a point source providing that intensity at that distance? 92 In about A.D. 150, Claudius Ptolemy gave the following measured values for the angle of incidence u1 and the angle of refraction u2 for a light beam passing from air to water: u1

u2

u1

u2

10° 20° 30° 40°

8° 15°30# 22°30# 29°

50° 60° 70° 80°

35° 40°30# 45°30# 50°

Assuming these data are consistent with the law of refraction, use them to find the index of refraction of water. These data are interesting as perhaps the oldest recorded physical measurements. 93 A beam of initially unpolarized light is sent through two polarizing sheets placed one on top of the other. What must be the angle between the polarizing directions of the sheets if the intensity of the transmitted light is to be one-third the incident intensity? 94 In Fig. 33-73, a long, straight copper wire (diameter 2.50 mm and resistance 1.00 * per 300 m) carries a uniform current of 25.0 A in the positive x direction. For point P on the wire’s surface, calcu: late the magnitudes of (a) the electric field E, (b) the magnetic : : field B, and (c) the Poynting vector S , and (d) determine the direc: tion of S . y P

i

x

Figure 33-73 Problem 94.

PROB LE M S

95 Figure 33-74 shows a cylindrical resistor of length l, radius a, and resistivity r, carrying current i. (a) Show : that the Poynting vector S at the surface of the resistor is everywhere directed normal to the surface, as S shown. (b) Show that the rate P at S which energy flows into the resistor through its cylindrical surface, calculated by integrating the Poynting vector over this surface, is equal to the rate at which thermal energy is S produced:

"

:

edge length of 2.0 m and is located 3.0 ( 1011 m from the Sun’s center. What is the radiation force on the surface from the light rays?

i

103 The rms value of the electric field in a certain light wave is 0.200 V/m. What is the amplitude of the associated magnetic field? S S l

S

S

S

:

S + dA ! i2R,

:

where dA is an element of area on the cylindrical surface and R is the resistance.

i 2a

Figure 33-74 Problem 95. 96 A thin, totally absorbing sheet of mass m, face area A, and specific heat cs is fully illuminated by a perpendicular beam of a plane electromagnetic wave. The magnitude of the maximum electric field of the wave is Em. What is the rate dT/dt at which the sheet’s temperature increases due to the absorption of the wave? 97 Two polarizing sheets, one directly above the other, transmit p% of the initially unpolarized light that is perpendicularly incident on the top sheet. What is the angle between the polarizing directions of the two sheets? 98 A laser beam of intensity I reflects from a flat, totally reflecting surface of area A, with a normal at angle u with the beam. Write an expression for the beam’s radiation pressure pr(u) on the surface in terms of the beam’s pressure pr" when u ! 0". 99 A beam of intensity I reflects from a long, totally reflecting cylinder of radius R; the beam is y perpendicular to the central axis of x the cylinder and has a diameter θ1 larger than 2R. What is the beam’s θ2 force per unit length on the cylinder? 100 In Fig. 33-75, unpolarized light is sent into a system of three polarizing sheets, where the polarizing directions of the first and second sheets are at angles u1 ! 20" and u2 ! 40". What fraction of the initial light intensity emerges from the system?

θ1

102 A square, perfectly reflecting surface is oriented in space to be perpendicular to the light rays from the Sun. The surface has an

y x

θ2

θ3

Figure 33-76 Problem 101.

Sunray 104 In Fig. 33-77, an albatross θ glides at a constant 15 m/s horizontally above level ground, moving in a vertical plane that contains the h Sun. It glides toward a wall of height h ! 2.0 m, which it will just barely clear. At that time of day, the angle Figure 33-77 Problem 104. of the Sun relative to the ground is u ! 30". At what speed does the shadow of the albatross move (a) across the level ground and then (b) up the wall? Suppose that later a hawk happens to glide along the same path, also at 15 m/s. You see that when its shadow reaches the wall, the speed of the shadow noticeably increases. (c) Is the Sun now higher or lower in the sky than when the albatross flew by earlier? (d) If the speed of the hawk’s shadow on the wall is 45 m/s, what is the angle u of the Sun just then?

105 The magnetic component of a polarized wave of light is given by Bx ! (4.00 mT) sin [ky ' (2.00 ( 1015 s&1)t]. (a) In which direction does the wave travel, (b) parallel to which axis is it polarized, and (c) what is its intensity? (d) Write an expression for the electric field of the wave, including a value for the angular wave number. (e) What is the wavelength? (f) In which region of the electromagnetic spectrum is this electromagnetic wave? θ

106 In Fig. 33-78, where n1 ! 1.70, n1 n2 ! 1.50, and n3 ! 1.30, light refracts from material 1 into material 2. If it is incident at point A at the B n2 critical angle for the interface beA tween materials 2 and 3, what are n3 (a) the angle of refraction at point B and (b) the initial angle u? If, inFigure 33-78 Problem 106. stead, light is incident at B at the critical angle for the interface between materials 2 and 3, what are (c) the angle of refraction at point A and (d) the initial angle u? If, instead of all that, light is incident at point A at Brewster’s angle for the interface between materials 2 and 3, what are (e) the angle of refraction at point B and (f) the initial angle u? 107 When red light in vacuum is incident at the Brewster angle on a certain glass slab, the angle of refraction is 32.0".What are (a) the index of refraction of the glass and (b) the Brewster angle?

Figure 33-75 Problem 100.

101 In Fig. 33-76, unpolarized light is sent into a system of three polarizing sheets with polarizing directions at angles u1 ! 20", u2 ! 60", and u3 ! 40". What fraction of the initial light intensity emerges from the system?

1009

108 Start from Eqs. 33-11 and 33-17 and show that E(x, t) and B(x, t), the electric and magnetic field components of a plane traveling electromagnetic wave, must satisfy the “wave equations” )2E )2E ! c2 2 2 )t )x

and

)2B )2B ! c2 2 . 2 )t )x

109 SSM (a) Show that Eqs. 33-1 land 33-2 satisfy the wave equations displayed in Problem 108. (b) Show that any expressions of the form E ! Em f(kx , vt) and B ! Bm f(kx , vt), where f(kx , vt) denotes an arbitrary function, also satisfy these wave equations. 110 A point source of light emits isotropically with a power of 200 W. What is the force due to the light on a totally absorbing sphere of radius 2.0 cm at a distance of 20 m from the source?

C

H

A

P

T

E

R

3

4

Images 34-1 IMAGES AND PLANE MIRRORS Learning Objectives After reading this module, you should be able to . . .

34.01 Distinguish virtual images from real images. 34.02 Explain the common roadway mirage. 34.03 Sketch a ray diagram for the reflection of a point source of light by a plane mirror, indicating the object distance and image distance.

34.04 Using the proper algebraic sign, relate the object distance p to the image distance i. 34.05 Give an example of the apparent hallway that you can see in a mirror maze based on equilateral triangles.

Key Ideas ● An image is a reproduction of an object via light. If the

image can form on a surface, it is a real image and can exist even if no observer is present. If the image requires the visual system of an observer, it is a virtual image. ● A plane (flat) mirror can form a virtual image of a light source (said to be the object) by redirecting light rays emerging from the source. The image can be seen where backward

extensions of reflected rays pass through one another. The object’s distance p from the mirror is related to the (apparent) image distance i from the mirror by i ! &p

(plane mirror).

Object distance p is a positive quantity. Image distance i for a virtual image is a negative quantity.

What Is Physics? One goal of physics is to discover the basic laws governing light, such as the law of refraction. A broader goal is to put those laws to use, and perhaps the most important use is the production of images. The first photographic images, made in 1824, were only novelties, but our world now thrives on images. Huge industries are based on the production of images on television, computer, and theater screens. Images from satellites guide military strategists during times of conflict and environmental strategists during times of blight. Camera surveillance can make a subway system more secure, but it can also invade the privacy of unsuspecting citizens. Physiologists and medical engineers are still puzzled by how images are produced by the human eye and the visual cortex of the brain, but they have managed to create mental images in some sightless people by electrical stimulation of the brain’s visual cortex. Our first step in this chapter is to define and classify images. Then we examine several basic ways in which they can be produced.

Two Types of Image For you to see, say, a penguin, your eye must intercept some of the light rays spreading from the penguin and then redirect them onto the retina at the rear of the eye. Your visual system, starting with the retina and ending with the visual cortex at the rear of your brain, automatically and subconsciously processes the information provided by the light. That system identifies edges, orientations, 1010

1011

34-1 I MAG ES AN D PL AN E M I R RORS

textures, shapes, and colors and then rapidly brings to your consciousness an image (a reproduction derived from light) of the penguin; you perceive and recognize the penguin as being in the direction from which the light rays came and at the proper distance. Your visual system goes through this processing and recognition even if the light rays do not come directly from the penguin, but instead reflect toward you from a mirror or refract through the lenses in a pair of binoculars. However, you now see the penguin in the direction from which the light rays came after they reflected or refracted, and the distance you perceive may be quite different from the penguin’s true distance. For example, if the light rays have been reflected toward you from a standard flat mirror, the penguin appears to be behind the mirror because the rays you intercept come from that direction. Of course, the penguin is not back there. This type of image, which is called a virtual image, truly exists only within the brain but nevertheless is said to exist at the perceived location. A real image differs in that it can be formed on a surface, such as a card or a movie screen. You can see a real image (otherwise movie theaters would be empty), but the existence of the image does not depend on your seeing it and it is present even if you are not. Before we discuss real and virtual images in detail, let’s examine a natural virtual image.

A Common Mirage A common example of a virtual image is a pool of water that appears to lie on the road some distance ahead of you on a sunny day, but that you can never reach. The pool is a mirage (a type of illusion), formed by light rays coming from the low section of the sky in front of you (Fig. 34-1a). As the rays approach the road, they travel through progressively warmer air that has been heated by the road, which is usually relatively warm. With an increase in air temperature, the density of the air — and hence the index of refraction of the air — decreases slightly. Thus, as the rays descend, encountering progressively smaller indexes of refraction, they continuously bend toward the horizontal (Fig. 34-1b). Once a ray is horizontal, somewhat above the road’s surface, it still bends because the lower portion of each associated wavefront is in slightly warmer air and is moving slightly faster than the upper portion of the wavefront (Fig. 34-1c). This nonuniform motion of the wavefronts bends the ray upward. As the ray then ascends, it continues to bend upward through progressively greater indexes of refraction (Fig. 34-1d). If you intercept some of this light, your visual system automatically infers that it originated along a backward extension of the rays you have intercepted and, to make sense of the light, assumes that it came from the road surface. If the light happens to be bluish from blue sky, the mirage appears bluish, like water. Because the air is probably turbulent due to the heating, the mirage shimmies, as if water waves were present. The bluish coloring and the shimmy enhance the illusion of a pool of water, but you are actually seeing a virtual image of a low section of the sky. As you travel toward the illusionary pool, you no longer intercept the shallow refracted rays and the illusion disappears. Warm Light ray

Fast

Warmer Pool mirage (a)

Road

Road (b)

Warm Warmer

Faster (c)

Road (d)

Figure 34-1 (a) A ray from a low section of the sky refracts through air that is heated by a road (without reaching the road). An observer who intercepts the light perceives it to be from a pool of water on the road. (b) Bending (exaggerated) of a light ray descending across an imaginary boundary from warm air to warmer air. (c) Shifting of wavefronts and associated bending of a ray, which occur because the lower ends of wavefronts move faster in warmer air. (d) Bending of a ray ascending across an imaginary boundary to warm air from warmer air.

1012

CHAPTE R 34 I MAG ES

Plane Mirrors

In a plane mirror the light seems to come from an object on the other side. Mirror I

O

θ θ

p

i

Figure 34-2 A point source of light O, called the object, is a perpendicular distance p in front of a plane mirror. Light rays reaching the mirror from O reflect from the mirror. If your eye intercepts some of the reflected rays, you perceive a point source of light I to be behind the mirror, at a perpendicular distance i. The perceived source I is a virtual image of object O.

O

p

i

θ

b

θ θ

θ

I

a Mirror

Figure 34-3 Two rays from Fig. 34-2. Ray Oa makes an arbitrary angle u with the normal to the mirror surface. Ray Ob is perpendicular to the mirror. O

Mirror

A mirror is a surface that can reflect a beam of light in one direction instead of either scattering it widely in many directions or absorbing it. A shiny metal surface acts as a mirror; a concrete wall does not. In this module we examine the images that a plane mirror (a flat reflecting surface) can produce. Figure 34-2 shows a point source of light O, which we shall call the object, at a perpendicular distance p in front of a plane mirror. The light that is incident on the mirror is represented with rays spreading from O. The reflection of that light is represented with reflected rays spreading from the mirror. If we extend the reflected rays backward (behind the mirror), we find that the extensions intersect at a point that is a perpendicular distance i behind the mirror. If you look into the mirror of Fig. 34-2, your eyes intercept some of the reflected light. To make sense of what you see, you perceive a point source of light located at the point of intersection of the extensions. This point source is the image I of object O. It is called a point image because it is a point, and it is a virtual image because the rays do not actually pass through it. (As you will see, rays do pass through a point of intersection for a real image.) Ray Tracing. Figure 34-3 shows two rays selected from the many rays in Fig. 34-2. One reaches the mirror at point b, perpendicularly. The other reaches it at an arbitrary point a, with an angle of incidence u. The extensions of the two reflected rays are also shown. The right triangles aOba and aIba have a common side and three equal angles and are thus congruent (equal in size); so their horizontal sides have the same length. That is,

I

a

Ib ! Ob,

(34-1)

where Ib and Ob are the distances from the mirror to the image and the object, respectively. Equation 34-1 tells us that the image is as far behind the mirror as the object is in front of it. By convention (that is, to get our equations to work out), object distances p are taken to be positive quantities and image distances i for virtual images (as here) are taken to be negative quantities. Thus, Eq. 34-1 can be written as |i| ! p or as i ! &p

(plane mirror).

(34-2)

Only rays that are fairly close together can enter the eye after reflection at a mirror. For the eye position shown in Fig. 34-4, only a small portion of the mirror near point a (a portion smaller than the pupil of the eye) is useful in forming the image. To find this portion, close one eye and look at the mirror image of a small object such as the tip of a pencil. Then move your fingertip over the mirror surface until you cannot see the image. Only that small portion of the mirror under your fingertip produced the image.

Extended Objects In Fig. 34-5, an extended object O, represented by an upright arrow, is at perpendicular distance p in front of a plane mirror. Each small portion of the Figure 34-4 A “pencil” of rays from O enters the eye after reflection at the mirror. Only a small portion of the mirror near a is involved in this reflection. The light appears to originate at point I behind the mirror.

p O

i I

In a plane mirror the image is just as far from the mirror as the object.

Figure 34-5 An extended object O and its virtual image I in a plane mirror.

1013

34-1 I MAG ES AN D PL AN E M I R RORS

Figure 34-6 A maze of mirrors. Courtesy Adrian Fisher, www.mazemaker.com

A

object that faces the mirror acts like the point source O of Figs. 34-2 and 34-3. If you intercept the light reflected by the mirror, you perceive a virtual image I that is a composite of the virtual point images of all those portions of the object. This virtual image seems to be at (negative) distance i behind the mirror, with i and p related by Eq. 34-2. We can also locate the image of an extended object as we did for a point object in Fig. 34-2: we draw some of the rays that reach the mirror from the top of the object, draw the corresponding reflected rays, and then extend those reflected rays behind the mirror until they intersect to form an image of the top of the object. We then do the same for rays from the bottom of the object. As shown in Fig. 34-5, we find that virtual image I has the same orientation and height (measured parallel to the mirror) as object O.

(a)

O

B

B

(b)

O

Mirror Maze In a mirror maze (Fig. 34-6), each wall is covered, floor to ceiling, with a mirror. Walk through such a maze and what you see in most directions is a confusing montage of reflections. In some directions, however, you see a hallway that seems to offer a path through the maze.Take these hallways, though, and you soon learn, after smacking into mirror after mirror, that the hallways are largely an illusion. Figure 34-7a is an overhead view of a simple mirror maze in which differently painted floor sections form equilateral triangles (60° angles) and walls are covered with vertical mirrors. You look into the maze while standing at point O at the middle of the maze entrance. In most directions, you see a confusing jumble of images. However, you see something curious in the direction of the ray shown in Fig. 34-7a. That ray leaves the middle of mirror B and reflects to you at the middle of mirror A. (The reflection obeys the law of reflection, with the angle of incidence and the angle of reflection both equal to 30°.) To make sense of the origin of the ray reaching you, your brain automatically extends the ray backward. It appears to originate at a point lying behind mirror A. That is, you perceive a virtual image of B behind A, at a distance equal to the actual distance between A and B (Fig. 34-7b). Thus, when you face into the maze in this direction, you see B along an apparent straight hallway consisting of four triangular floor sections. This story is incomplete, however, because the ray reaching you does not originate at mirror B — it only reflects there. To find the origin, we continue to apply the law of reflection as we work backwards, reflection by reflection on the mirrors (Fig. 34-7c). We finally come to the origin of the ray: you! What you see when you look along the apparent hallway is a virtual image of yourself, at a distance of nine triangular floor sections from you (Fig. 34-7d).

(c)

O

A hallway seems to lie in front of you.

(d)

O

O

Figure 34-7 (a) Overhead view of a mirror maze. A ray from mirror B reaches you at O by reflecting from mirror A. (b) Mirror B appears to be behind A. (c) The ray reaching you comes from you. (d) You see a virtual image of yourself at the end of an apparent hallway. (Can you find a second apparent hallway extending away from point O?)

1014

CHAPTE R 34 I MAG ES

Checkpoint 1 In the figure you are in a system of two vertical parallel A mirrors A and B separated by distance d. A grinning 0.2d O gargoyle is perched at point O, a distance 0.2d from d mirror A. Each mirror produces a first (least deep) image of the gargoyle. Then each mirror produces a second image with the object being the first image in B the opposite mirror. Then each mirror produces a third image with the object being the second image in the opposite mirror, and so on—you might see hundreds of grinning gargoyle images. How deep behind mirror A are the first, second, and third images in mirror A?

34-2 SPHERICAL MIRRORS Learning Objectives After reading this module, you should be able to . . .

34.06 Distinguish a concave spherical mirror from a convex spherical mirror. 34.07 For concave and convex mirrors, sketch a ray diagram for the reflection of light rays that are initially parallel to the central axis, indicating how they form the focal points, and identifying which is real and which is virtual. 34.08 Distinguish a real focal point from a virtual focal point, identify which corresponds to which type of mirror, and identify the algebraic sign associated with each focal length. 34.09 Relate a focal length of a spherical mirror to the radius. 34.10 Identify the terms “inside the focal point” and “outside the focal point.” 34.11 For an object (a) inside and (b) outside the focal point of a concave mirror, sketch the reflections of at least two rays to find the image and identify the type and orientation of the image.

34.12 For a concave mirror, distinguish the locations and orientations of a real image and a virtual image. 34.13 For an object in front of a convex mirror, sketch the reflections of at least two rays to find the image and identify the type and orientation of the image. 34.14 Identify which type of mirror can produce both real and virtual images and which type can produce only virtual images. 34.15 Identify the algebraic signs of the image distance i for real images and virtual images. 34.16 For convex, concave, and plane mirrors, apply the relationship between the focal length f, object distance p, and image distance i. 34.17 Apply the relationships between lateral magnification m, image height h#, object height h, image distance i, and object distance p.

Key Ideas ● A spherical mirror is in the shape of a small section of a spherical surface and can be concave (the radius of curvature r is a positive quantity), convex (r is a negative quantity), or plane (flat, r is infinite). ● If parallel rays are sent into a (spherical) concave mirror parallel to the central axis, the reflected rays pass through a common point (a real focus F ) at a distance f (a positive quantity) from the mirror. If they are sent toward a (spherical) convex mirror, backward extensions of the reflected rays pass through a common point (a virtual focus F ) at a distance f (a negative quantity) from the mirror. ● A concave mirror can form a real image (if the object is outside the focal point) or a virtual image (if the object is inside the focal point).

● A convex mirror can form only a virtual image. ● The mirror equation relates an object distance p, the mir-

ror’s focal length f and radius of curvature r, and the image distance i: 1 1 2 1 ' ! ! . p i f r ● The magnitude of the lateral magnification m of an object is

the ratio of the image height h# to object height h, h# !m! ! , h and is related to the object distance p and image distance i by m!&

i . p

1015

34-2 SPH E R ICAL M I R RORS

Spherical Mirrors We turn now from images produced by plane mirrors to images produced by mirrors with curved surfaces. In particular, we consider spherical mirrors, which are simply mirrors in the shape of a small section of the surface of a sphere. A plane mirror is in fact a spherical mirror with an infinitely large radius of curvature and thus an approximately flat surface.

Making a Spherical Mirror We start with the plane mirror of Fig. 34-8a, which faces leftward toward an object O that is shown and an observer that is not shown. We make a concave mirror by curving the mirror’s surface so it is concave (“caved in”) as in Fig. 34-8b. Curving the surface in this way changes several characteristics of the mirror and the image it produces of the object:

O

I

1. The center of curvature C (the center of the sphere of which the mirror’s surface is part) was infinitely far from the plane mirror; it is now closer but still in front of the concave mirror. 2. The field of view — the extent of the scene that is reflected to the observer — was wide; it is now smaller. 3. The image of the object was as far behind the plane mirror as the object was in front; the image is farther behind the concave mirror; that is, |i| is greater. 4. The height of the image was equal to the height of the object; the height of the image is now greater. This feature is why many makeup mirrors and shaving mirrors are concave — they produce a larger image of a face. We can make a convex mirror by curving a plane mirror so its surface is convex (“flexed out”) as in Fig. 34-8c. Curving the surface in this way (1) moves the center of curvature C to behind the mirror and (2) increases the field of view. It also (3) moves the image of the object closer to the mirror and (4) shrinks it. Store surveillance mirrors are usually convex to take advantage of the increase in the field of view — more of the store can then be seen with a single mirror.

p

Bending the mirror this way shifts the image away.

I O

C

Central axis

Focal Points of Spherical Mirrors For a plane mirror, the magnitude of the image distance i is always equal to the object distance p. Before we can determine how these two distances are related for a spherical mirror, we must consider the reflection of light from an object O located an effectively infinite distance in front of a spherical mirror, on the mirror’s central axis. That axis extends through the center of curvature C and the center c of the mirror. Because of the great distance between the object and the mirror, the light waves spreading from the object are plane waves when they reach the mirror along the central axis. This means that the rays representing the light waves are all parallel to the central axis when they reach the mirror. Forming a Focus. When these parallel rays reach a concave mirror like that of Fig. 34-9a, those near the central axis are reflected through a common point F; two of these reflected rays are shown in the figure. If we placed a (small) card at F, a point image of the infinitely distant object O would appear on the card. (This would occur for any infinitely distant object.) Point F is called the focal point (or focus) of the mirror, and its distance from the center of the mirror c is the focal length f of the mirror. If we now substitute a convex mirror for the concave mirror, we find that the parallel rays are no longer reflected through a common point. Instead, they diverge as shown in Fig. 34-9b. However, if your eye intercepts some of the reflected light, you perceive the light as originating from a point source behind the mirror. This perceived source is located where extensions of the reflected rays pass through a common point (F in Fig. 34-9b). That point is the focal point (or

i (a)

c

p

r

i

(b)

Bending it this way shifts the image closer.

O I

c

p

i

Central axis

C

r (c)

Figure 34-8 (a) An object O forms a virtual image I in a plane mirror. (b) If the mirror is bent so that it becomes concave, the image moves farther away and becomes larger. (c) If the plane mirror is bent so that it becomes convex, the image moves closer and becomes smaller.

1016

CHAPTE R 34 I MAG ES

To find the focus, send in rays parallel to the central axis.

C

If you intercept the reflections, they seem to come from this point.

Real focus F

c

Central axis

Virtual focus F

c

C

Central axis

f

r

f

(a)

r (b)

Figure 34-9 (a) In a concave mirror, incident parallel light rays are brought to a real focus at F, on the same side of the mirror as the incident light rays. (b) In a convex mirror, incident parallel light rays seem to diverge from a virtual focus at F, on the side of the mirror opposite the light rays.

focus) F of the convex mirror, and its distance from the mirror surface is the focal length f of the mirror. If we placed a card at this focal point, an image of object O would not appear on the card; so this focal point is not like that of a concave mirror. Two Types. To distinguish the actual focal point of a concave mirror from the perceived focal point of a convex mirror, the former is said to be a real focal point and the latter is said to be a virtual focal point. Moreover, the focal length f of a concave mirror is taken to be a positive quantity, and that of a convex mirror a negative quantity. For mirrors of both types, the focal length f is related to the radius of curvature r of the mirror by f ! 12 r

(34-3)

(spherical mirror),

where r is positive for a concave mirror and negative for a convex mirror.

Images from Spherical Mirrors Inside. With the focal point of a spherical mirror defined, we can find the relation between image distance i and object distance p for concave and convex spherical mirrors. We begin by placing the object O inside the focal point of the concave mirror — that is, between the mirror and its focal point F (Fig. 34-10a).

Changing the location of the object relative to F changes the image. O

Virtual image I

O

O

I (i = –∞)

F

F I (i = +∞)

(a)

f

p

i

Parallel rays

p=f (b)

F

Real image I

i

Figure 34-10 (a) An object O inside the focal point of a concave mirror, and its virtual image I. (b) The object at the focal point F. (c) The object outside the focal point, and its real image I.

p (c)

f

34-2 SPH E R ICAL M I R RORS

An observer can then see a virtual image of O in the mirror: The image appears to be behind the mirror, and it has the same orientation as the object. If we now move the object away from the mirror until it is at the focal point, the image moves farther and farther back from the mirror until, when the object is at the focal point, the image is at infinity (Fig. 34-10b). The image is then ambiguous and imperceptible because neither the rays reflected by the mirror nor the ray extensions behind the mirror cross to form an image of O. Outside. If we next move the object outside the focal point — that is, farther away from the mirror than the focal point — the rays reflected by the mirror converge to form an inverted image of object O (Fig. 34-10c) in front of the mirror. That image moves in from infinity as we move the object farther outside F. If you were to hold a card at the position of the image, the image would show up on the card — the image is said to be focused on the card by the mirror. (The verb “focus,” which in this context means to produce an image, differs from the noun “focus,” which is another name for the focal point.) Because this image can actually appear on a surface, it is a real image — the rays actually intersect to create the image, regardless of whether an observer is present. The image distance i of a real image is a positive quantity, in contrast to that for a virtual image.We can now generalize about the location of images from spherical mirrors: Real images form on the side of a mirror where the object is, and virtual images form on the opposite side.

Main Equation. As we shall prove in Module 34-6, when light rays from an object make only small angles with the central axis of a spherical mirror, a simple equation relates the object distance p, the image distance i, and the focal length f : 1 1 1 ' ! p i f

(spherical mirror).

(34-4)

We assume such small angles in figures such as Fig. 34-10, but for clarity the rays are drawn with exaggerated angles. With that assumption, Eq. 34-4 applies to any concave, convex, or plane mirror. For a convex or plane mirror, only a virtual image can be formed, regardless of the object’s location on the central axis. As shown in the example of a convex mirror in Fig. 34-8c, the image is always on the opposite side of the mirror from the object and has the same orientation as the object. Magnification. The size of an object or image, as measured perpendicular to the mirror’s central axis, is called the object or image height. Let h represent the height of the object, and h# the height of the image. Then the ratio h#/h is called the lateral magnification m produced by the mirror. However, by convention, the lateral magnification always includes a plus sign when the image orientation is that of the object and a minus sign when the image orientation is opposite that of the object. For this reason, we write the formula for m as |m| !

h# h

(lateral magnification).

(34-5)

We shall soon prove that the lateral magnification can also be written as m!&

i p

(lateral magnification).

(34-6)

For a plane mirror, for which i ! &p, we have m ! '1. The magnification of 1 means that the image is the same size as the object. The plus sign means that

1017

1018

CHAPTE R 34 I MAG ES

Table 34-1 Your Organizing Table for Mirrors Image Mirror Type

Object Location

Plane

Location

Sign

Type

Orientation

of f

of r

of i

of m

Anywhere Inside F

Concave

Outside F

Convex

Anywhere

the image and the object have the same orientation. For the concave mirror of Fig. 34-10c, m % &1.5. Organizing Table. Equations 34-3 through 34-6 hold for all plane mirrors, concave spherical mirrors, and convex spherical mirrors. In addition to those equations, you have been asked to absorb a lot of information about these mirrors, and you should organize it for yourself by filling in Table 34-1. Under Image Location, note whether the image is on the same side of the mirror as the object or on the opposite side. Under Image Type, note whether the image is real or virtual. Under Image Orientation, note whether the image has the same orientation as the object or is inverted. Under Sign, give the sign of the quantity or fill in , if the sign is ambiguous. You will need this organization to tackle homework or a test.

Figure 34-11 (a, b) Four rays that may be drawn to find the image formed by a concave mirror. For the object position shown, the image is real, inverted, and smaller than the object. (c, d) Four similar rays for the case of a convex mirror. For a convex mirror, the image is always virtual, oriented like the object, and smaller than the object. [In (c), ray 2 is initially directed toward focal point F. In (d), ray 3 is initially directed toward center of curvature C.]

Locating Images by Drawing Rays Figures 34-11a and b show an object O in front of a concave mirror. We can graphically locate the image of any off-axis point of the object by drawing a ray diagram with any two of four special rays through the point:

Any two of these four rays will locate the image. b C

O

F

O

c

I

2

C a

d e

4

c

F I 3

1 (b)

(a) 1 3 2 O

I

c

F

C

O

(c)

Here too, any two rays 4 will locate the image.

C

I c

F

(d)

34-2 SPH E R ICAL M I R RORS

1019

1. A ray that is initially parallel to the central axis reflects through the focal point F (ray 1 in Fig. 34-11a). 2. A ray that reflects from the mirror after passing through the focal point emerges parallel to the central axis (ray 2 in Fig. 34-11a). 3. A ray that reflects from the mirror after passing through the center of curvature C returns along itself (ray 3 in Fig. 34-11b). 4. A ray that reflects from the mirror at point c is reflected symmetrically about that axis (ray 4 in Fig. 34-11b). The image of the point is at the intersection of the two special rays you choose. The image of the object can then be found by locating the images of two or more of its off-axis points (say, the point most off axis) and then sketching in the rest of the image. You need to modify the descriptions of the rays slightly to apply them to convex mirrors, as in Figs. 34-11c and d.

Proof of Equation 34-6 We are now in a position to derive Eq. 34-6 (m ! &i/p), the equation for the lateral magnification of an object reflected in a mirror. Consider ray 4 in Fig. 34-11b. It is reflected at point c so that the incident and reflected rays make equal angles with the axis of the mirror at that point. The two right triangles abc and dec in the figure are similar (have the same set of angles); so we can write cd de ! . ab ca The quantity on the left (apart from the question of sign) is the lateral magnification m produced by the mirror. Because we indicate an inverted image as a negative magnification, we symbolize this as &m. However, cd ! i and ca ! p; so we have m!&

i p

(magnification),

(34-7)

which is the relation we set out to prove.

Checkpoint 2 A Central American vampire bat, dozing on the central axis of a spherical mirror, is magnified by m ! &4. Is its image (a) real or virtual, (b) inverted or of the same orientation as the bat, and (c) on the same side of the mirror as the bat or on the opposite side?

Sample Problem 34.01 Image produced by a spherical mirror A tarantula of height h sits cautiously before a spherical mirror whose focal length has absolute value !f ! ! 40 cm. The image of the tarantula produced by the mirror has the same orientation as the tarantula and has height h# ! 0.20h.

(b) Is the mirror concave or convex, and what is its focal length f, sign included?

(a) Is the image real or virtual, and is it on the same side of the mirror as the tarantula or the opposite side?

We cannot tell the type of mirror from the type of image because both types of mirror can produce virtual images. Similarly, we cannot tell the type of mirror from the sign of the focal length f, as obtained from Eq. 34-3 or Eq. 34-4, because we lack enough information to use either equation. However, we can make use of the magnification information.

Reasoning: Because the image has the same orientation as the tarantula (the object), it must be virtual and on the opposite side of the mirror. (You can easily see this result if you have filled out Table 34-1.)

KEY IDEA

1020

CHAPTE R 34 I MAG ES

Calculations: From the given information, we know that the ratio of image height h# to object height h is 0.20. Thus, from Eq. 34-5 we have h# !m! ! ! 0.20. h Because the object and image have the same orientation, we know that m must be positive: m ! '0.20. Substituting this into Eq. 34-6 and solving for, say, i gives us i ! &0.20p,

which does not appear to be of help in finding f. However, it is helpful if we substitute it into Eq. 34-4. That equation gives us 1 1 1 1 1 1 ! ' ! ' ! (&5 ' 1), f i p &0.20p p p from which we find f ! &p/4. Now we have it: Because p is positive, f must be negative, which means that the mirror is convex with f ! &40 cm.

(Answer)

Additional examples, video, and practice available at WileyPLUS

34-3 SPHERICAL REFRACTING SURFACES Learning Objectives After reading this module, you should be able to . . .

34.18 Identify that the refraction of rays by a spherical surface can produce real images and virtual images of an object, depending on the indexes of refraction on the two sides, the surface’s radius of curvature r, and whether the object faces a concave or convex surface. 34.19 For a point object on the central axis of a spherical refracting surface, sketch the refraction of a ray in the six general arrangements and identify whether the image is real or virtual.

34.20 For a spherical refracting surface, identify what type of image appears on the same side as the object and what type appears on the opposite side. 34.21 For a spherical refracting surface, apply the relationship between the two indexes of refraction, the object distance p, the image distance i, and the radius of curvature r. 34.22 Identify the algebraic signs of the radius r for an object facing a concave refracting surface and a convex refracting surface.

Key Ideas ● A single spherical surface that refracts light can form an image. ● The object distance p, the image distance i, and the radius of curvature r of the surface are related by n1 n2 n2 & n1 ' ! , p i r

where n1 is the index of refraction of the material where the object is located and n2 is the index of refraction on the other side of the surface. ● If the surface faced by the object is convex, r is positive,

and if it is concave, r is negative. ● Images on the object’s side of the surface are virtual, and

images on the opposite side are real.

Spherical Refracting Surfaces We now turn from images formed by reflection to images formed by refraction through surfaces of transparent materials, such as glass. We shall consider only spherical surfaces, with radius of curvature r and center of curvature C. The light will be emitted by a point object O in a medium with index of refraction n1; it will refract through a spherical surface into a medium of index of refraction n2. Our concern is whether the light rays, after refracting through the surface, form a real image (no observer necessary) or a virtual image (assuming that an

34-3 SPH E R ICAL R E FRACTI NG SU R FACES

observer intercepts the rays). The answer depends on the relative values of n1 and n2 and on the geometry of the situation. Six possible results are shown in Fig. 34-12. In each part of the figure, the medium with the greater index of refraction is shaded, and object O is always in the medium with index of refraction n1, to the left of the refracting surface. In each part, a representative ray is shown refracting through the surface. (That ray and a ray along the central axis suffice to determine the position of the image in each case.) At the point of refraction of each ray, the normal to the refracting surface is a radial line through the center of curvature C. Because of the refraction, the ray bends toward the normal if it is entering a medium of greater index of refraction and away from the normal if it is entering a medium of lesser index of refraction. If the bending sends the ray toward the central axis, that ray and others (undrawn) form a real image on that axis. If the bending sends the ray away from the central axis, the ray cannot form a real image; however, backward extensions of it and other refracted rays can form a virtual image, provided (as with mirrors) some of those rays are intercepted by an observer. Real images I are formed (at image distance i) in parts a and b of Fig. 34-12, where the refraction directs the ray toward the central axis. Virtual images are formed in parts c and d, where the refraction directs the ray away from the central axis. Note, in these four parts, that real images are formed when the object is relatively far from the refracting surface and virtual images are formed when the object is nearer the refracting surface. In the final situations (Figs. 34-12e and f ), refraction always directs the ray away from the central axis and virtual images are always formed, regardless of the object distance. Note the following major difference from reflected images:

Dr. Paul A. Zahl/Photo Researchers, Inc.

This insect has been entombed in amber for about 25 million years. Because we view the insect through a curved refracting surface, the location of the image we see does not coincide with the location of the insect (see Fig. 34-12d).

Real images form on the side of a refracting surface that is opposite the object, and virtual images form on the same side as the object.

O

C n2

n1

I

O

C

I

n1

n2

r p

Virtual

Real

Real

i

Virtual O n1

r

C n2

(c)

p

(a)

I

i (b)

Virtual

Virtual O

IC

O n2

n1 (e)

I

C

n1

n2 (f )

Figure 34-12 Six possible ways in which an image can be formed by refraction through a spherical surface of radius r and center of curvature C. The surface separates a medium with index of refraction n1 from a medium with index of refraction n2. The point object O is always in the medium with n1, to the left of the surface. The material with the lesser index of refraction is unshaded (think of it as being air, and the other material as being glass). Real images are formed in (a) and (b); virtual images are formed in the other four situations.

1021

CO I n1

n2 (d)

1022

CHAPTE R 34 I MAG ES

In Module 34-6, we shall show that (for light rays making only small angles with the central axis) n2 n2 & n1 n1 ' ! . p i r

(34-8)

Just as with mirrors, the object distance p is positive, and the image distance i is positive for a real image and negative for a virtual image. However, to keep all the signs correct in Eq. 34-8, we must use the following rule for the sign of the radius of curvature r: When the object faces a convex refracting surface, the radius of curvature r is positive. When it faces a concave surface, r is negative.

Be careful: This is just the reverse of the sign convention we have for mirrors, which can be a slippery point in the heat of an exam.

Checkpoint 3 A bee is hovering in front of the concave spherical refracting surface of a glass sculpture. (a) Which part of Fig. 34-12 is like this situation? (b) Is the image produced by the surface real or virtual, and (c) is it on the same side as the bee or the opposite side?

Sample Problem 34.02 Image produced by a refracting surface A Jurassic mosquito is discovered embedded in a chunk of amber, which has index of refraction 1.6. One surface of the amber is spherically convex with radius of curvature 3.0 mm (Fig. 34-13). The mosquito’s head happens to be on the central axis of that surface and, when viewed along the axis, appears to be buried 5.0 mm into the amber. How deep is it really?

Calculations: Making these substitutions in Eq. 34-8, n1 n2 n2 & n1 ' ! , p i r yields

1.6 1.0 1.0 & 1.6 ' ! p &5.0 mm &3.0 mm

and

p ! 4.0 mm.

(Answer)

KEY IDEAS The head appears to be 5.0 mm into the amber only because the light rays that the observer intercepts are bent by refraction at the convex amber surface. The image distance i differs from the object distance p according to Eq. 34-8. To use that equation to find the object distance, we first note: 1. Because the object (the head) and its image are on the same side of the refracting surface, the image must be virtual and so i ! &5.0 mm. 2. Because the object is always taken to be in the medium of index of refraction n1, we must have n1 ! 1.6 and n2 ! 1.0. 3. Because the object faces a concave refracting surface, the radius of curvature r is negative, and so r ! &3.0 mm.

I

O C

i

p

r

Figure 34-13 A piece of amber with a mosquito from the Jurassic period, with the head buried at point O. The spherical refracting surface at the right end, with center of curvature C, provides an image I to an observer intercepting rays from the object at O.

Additional examples, video, and practice available at WileyPLUS

34-4 TH I N LE NSES

1023

34-4 THIN LENSES Learning Objectives After reading this module, you should be able to . . .

34.23 Distinguish converging lenses from diverging lenses. 34.24 For converging and diverging lenses, sketch a ray diagram for rays initially parallel to the central axis, indicating how they form focal points, and identifying which is real and which is virtual. 34.25 Distinguish a real focal point from a virtual focal point, identify which corresponds to which type of lens and under which circumstances, and identify the algebraic sign associated with each focal length. 34.26 For an object (a) inside and (b) outside the focal point of a converging lens, sketch at least two rays to find the image and identify the type and orientation of the image. 34.27 For a converging lens, distinguish the locations and orientations of a real image and a virtual image. 34.28 For an object in front of a diverging lens, sketch at least two rays to find the image and identify the type and orientation of the image. 34.29 Identify which type of lens can produce both real

and virtual images and which type can produce only virtual images. 34.30 Identify the algebraic sign of the image distance i for a real image and for a virtual image. 34.31 For converging and diverging lenses, apply the relationship between the focal length f, object distance p, and image distance i. 34.32 Apply the relationships between lateral magnification m, image height h#, object height h, image distance i, and object distance p. 34.33 Apply the lens maker’s equation to relate a focal length to the index of refraction of a lens (assumed to be in air) and the radii of curvature of the two sides of the lens. 34.34 For a multiple-lens system with the object in front of lens 1, find the image produced by lens 1 and then use it as the object for lens 2, and so on. 34.35 For a multiple-lens system, determine the overall magnification (of the final image) from the magnifications produced by each lens.

Key Ideas ● This module primarily considers thin lenses with symmetric, spherical surfaces. ● If parallel rays are sent through a converging lens parallel to the central axis, the refracted rays pass through a common point (a real focus F ) at a focal distance f (a positive quantity) from the lens. If they are sent through a diverging lens, backward extensions of the refracted rays pass through a common point (a virtual focus F ) at a focal distance f (a negative quantity) from the lens. ● A converging lens can form a real image (if the object is outside the focal point) or a virtual image (if the object is inside the focal point). ● A diverging lens can form only a virtual image. ● For an object in front of a lens, object distance p and

image distance i are related to the lens’s focal length f, index

of refraction n, and radii of curvature r1 and r2 by 1 1 1 1 1 ' ! ! (n & 1) & . p i f r1 r2

#

$

● The magnitude of the lateral magnification m of an object is

the ratio of the image height h# to object height h, h# , !m! ! h and is related to the object distance p and image distance i by i m!& . p ● For a system of lenses with a common central axis, the

image produced by the first lens acts as the object for the second lens, and so on, and the overall magnification is the product of the individual magnifications.

Thin Lenses A lens is a transparent object with two refracting surfaces whose central axes coincide. The common central axis is the central axis of the lens. When a lens is surrounded by air, light refracts from the air into the lens, crosses through the lens, and then refracts back into the air. Each refraction can change the direction of travel of the light. A lens that causes light rays initially parallel to the central axis to converge is (reasonably) called a converging lens. If, instead, it causes such rays to diverge, the lens is a diverging lens. When an object is placed in front of a lens of either type, light rays from the object that refract into and out of the lens can produce an image of the object.

1024

CHAPTE R 34 I MAG ES

Lens Equations. We shall consider only the special case of a thin lens — that is, a lens in which the thickest part is thin relative to the object distance p, the image distance i, and the radii of curvature r1 and r2 of the two surfaces of the lens. We shall also consider only light rays that make small angles with the central axis (they are exaggerated in the figures here). In Module 34-6 we shall prove that for such rays, a thin lens has a focal length f. Moreover, i and p are related to each other by 1 1 1 ! ' f p i

(34-9)

(thin lens),

which is the same as we had for mirrors. We shall also prove that when a thin lens with index of refraction n is surrounded by air, this focal length f is given by 1 ! (n & 1) f

Courtesy Matthew G. Wheeler

A fire is being started by focusing sunlight onto newspaper by means of a converging lens made of clear ice. The lens was made by melting both sides of a flat piece of ice into a convex shape in the shallow vessel (which has a curved bottom).

# r1

1

&

1 r2

$

(34-10)

(thin lens in air),

which is often called the lens maker’s equation. Here r1 is the radius of curvature of the lens surface nearer the object and r2 is that of the other surface. The signs of these radii are found with the rules in Module 34-3 for the radii of spherical refracting surfaces. If the lens is surrounded by some medium other than air (say, corn oil) with index of refraction nmedium, we replace n in Eq. 34-10 with n/nmedium. Keep in mind the basis of Eqs. 34-9 and 34-10: A lens can produce an image of an object only because the lens can bend light rays, but it can bend light rays only if its index of refraction differs from that of the surrounding medium.

Forming a Focus. Figure 34-14a shows a thin lens with convex refracting surfaces, or sides. When rays that are parallel to the central axis of the lens are sent through the lens, they refract twice, as is shown enlarged in Fig. 34-14b. This To find the focus, send in rays parallel to the central axis.

Figure 34-14 (a) Rays initially parallel to the central axis of a converging lens are made to converge to a real focal point F2 by the lens. The lens is thinner than drawn, with a width like that of the vertical line through it. We shall consider all the bending of rays as occurring at this central line. (b) An enlargement of the top part of the lens of (a); normals to the surfaces are shown dashed. Note that both refractions bend the ray downward, toward the central axis. (c) The same initially parallel rays are made to diverge by a diverging lens. Extensions of the diverging rays pass through a virtual focal point F2. (d) An enlargement of the top part of the lens of (c). Note that both refractions bend the ray upward, away from the central axis.

C2

The bending occurs only at the surfaces.

F1

F2

f

r2

C1

r1

(a)

(b)

If you intercept these rays, they seem to come from F2. C1

F2

F1

C2 Extension

r1

f

r2 (c)

(d)

34-4 TH I N LE NSES

double refraction causes the rays to converge and pass through a common point F2 at a distance f from the center of the lens. Hence, this lens is a converging lens; further, a real focal point (or focus) exists at F2 (because the rays really do pass through it), and the associated focal length is f. When rays parallel to the central axis are sent in the opposite direction through the lens, we find another real focal point at F1 on the other side of the lens. For a thin lens, these two focal points are equidistant from the lens. Signs, Signs, Signs. Because the focal points of a converging lens are real, we take the associated focal lengths f to be positive, just as we do with a real focus of a concave mirror. However, signs in optics can be tricky; so we had better check this in Eq. 34-10. The left side of that equation is positive if f is positive; how about the right side? We examine it term by term. Because the index of refraction n of glass or any other material is greater than 1, the term (n & 1) must be positive. Because the source of the light (which is the object) is at the left and faces the convex left side of the lens, the radius of curvature r1 of that side must be positive according to the sign rule for refracting surfaces. Similarly, because the object faces a concave right side of the lens, the radius of curvature r2 of that side must be negative according to that rule. Thus, the term (1/r1 & 1/r2) is positive, the whole right side of Eq. 34-10 is positive, and all the signs are consistent. Figure 34-14c shows a thin lens with concave sides. When rays that are parallel to the central axis of the lens are sent through this lens, they refract twice, as is shown enlarged in Fig. 34-14d; these rays diverge, never passing through any common point, and so this lens is a diverging lens. However, extensions of the rays do pass through a common point F2 at a distance f from the center of the lens. Hence, the lens has a virtual focal point at F2. (If your eye intercepts some of the diverging rays, you perceive a bright spot to be at F2, as if it is the source of the light.) Another virtual focus exists on the opposite side of the lens at F1, symmetrically placed if the lens is thin. Because the focal points of a diverging lens are virtual, we take the focal length f to be negative.

Converging lenses can give either type of image.

C1

O C2

F1

r2

(a)

I

Real images form on the side of a lens that is opposite the object, and virtual images form on the side where the object is.

The lateral magnification m produced by converging and diverging lenses is given by Eqs. 34-5 and 34-6, the same as for mirrors. You have been asked to absorb a lot of information in this module, and you should organize it for yourself by filling in Table 34-2 for thin symmetric lenses (both

I f

r1

p

i

O F1 p

Images from Thin Lenses We now consider the types of image formed by converging and diverging lenses. Figure 34-15a shows an object O outside the focal point F1 of a converging lens. The two rays drawn in the figure show that the lens forms a real, inverted image I of the object on the side of the lens opposite the object. When the object is placed inside the focal point F1, as in Fig. 34-15b, the lens forms a virtual image I on the same side of the lens as the object and with the same orientation. Hence, a converging lens can form either a real image or a virtual image, depending on whether the object is outside or inside the focal point, respectively. Figure 34-15c shows an object O in front of a diverging lens. Regardless of the object distance (regardless of whether O is inside or outside the virtual focal point), this lens produces a virtual image that is on the same side of the lens as the object and has the same orientation. As with mirrors, we take the image distance i to be positive when the image is real and negative when the image is virtual. However, the locations of real and virtual images from lenses are the reverse of those from mirrors:

1025

(b)

i

f

Diverging lenses can give only virtual images. O

(c)

C2

I

C1

p

r1

i

r2

Figure 34-15 (a) A real, inverted image I is formed by a converging lens when the object O is outside the focal point F1. (b) The image I is virtual and has the same orientation as O when O is inside the focal point. (c) A diverging lens forms a virtual image I, with the same orientation as the object O, whether O is inside or outside the focal point of the lens.

1026

CHAPTE R 34 I MAG ES

Table 34-2 Your Organizing Table for Thin Lenses Image Lens Type

Object Location

Location

Type

Sign Orientation

of f

of i

of m

Inside F

Converging

Outside F

Diverging

Anywhere

sides are convex or both sides are concave). Under Image Location note whether the image is on the same side of the lens as the object or on the opposite side. Under Image Type note whether the image is real or virtual. Under Image Orientation note whether the image has the same orientation as the object or is inverted.

Locating Images of Extended Objects by Drawing Rays Figure 34-16a shows an object O outside focal point F1 of a converging lens. We can graphically locate the image of any off-axis point on such an object (such as the tip of the arrow in Fig. 34-16a) by drawing a ray diagram with any two of three special rays through the point. These special rays, chosen from all those that pass through the lens to form the image, are the following: 1. A ray that is initially parallel to the central axis of the lens will pass through focal point F2 (ray 1 in Fig. 34-16a). 2. A ray that initially passes through focal point F1 will emerge from the lens parallel to the central axis (ray 2 in Fig. 34-16a). 3. A ray that is initially directed toward the center of the lens will emerge from the lens with no change in its direction (ray 3 in Fig. 34-16a) because the ray encounters the two sides of the lens where they are almost parallel. The image of the point is located where the rays intersect on the far side of the lens. The image of the object is found by locating the images of two or more of its points. Figure 34-16b shows how the extensions of the three special rays can be used to locate the image of an object placed inside focal point F1 of a converging lens. Note that the description of ray 2 requires modification (it is now a ray whose backward extension passes through F1). You need to modify the descriptions of rays 1 and 2 to use them to locate an image placed (anywhere) in front of a diverging lens. In Fig. 34-16c, for example, we find the point where ray 3 intersects the backward extensions of rays 1 and 2. In each figure, any two of the rays will locate the image.

1 O

3 F1

2

2 I

F2 F1

I

(a)

1

F2

O 3

(b)

1

Figure 34-16 Three special rays allow us to locate an image formed by a thin lens whether the object O is (a) outside or (b) inside the focal point of a converging lens, or (c) anywhere in front of a diverging lens.

2

O F2 I

F1 3

(c)

34-4 TH I N LE NSES

1027

Two-Lens Systems Here we consider an object sitting in front of a system of two lenses whose central axes coincide. Some of the possible two-lens systems are sketched in Fig. 34-17, but the figures are not drawn to scale. In each, the object sits to the left of lens 1 but can be inside or outside the focal point of the lens. Although tracing the light rays through any such two-lens system can be challenging, we can use the following simple two-step solution: Step 1 Neglecting lens 2, use Eq. 34-9 to locate the image I1 produced by lens 1. Determine whether the image is on the left or right side of the lens, whether it is real or virtual, and whether it has the same orientation as the object. Roughly sketch I1.The top part of Fig. 34-17a gives an example.

p1

Outside focal point

A

p1

I1

I1

Outside focal point

I2 is somewhere to the right of lens 2. Outside focal point

p2

I2 is somewhere to the right of lens 2.

(a)

p2 is negative.

(b)

p1

p1 I1 I1

Outside focal point

Inside focal point

I2 is somewhere to the right of lens 2.

I2 is somewhere to the left of lens 2. Outside focal point p2

(c)

Figure 34-17 Several sketches (not to scale) of a two-lens system in which an object sits to the left of lens 1. In step 1 of the solution, we consider lens 1 and ignore lens 2 (shown in dashes). In step 2, we consider lens 2 and ignore lens 1 (no longer shown). We want to find the final image, that is, the image produced by lens 2.

p2 (d)

p1

p1

I1

Outside focal point Outside focal point

I1

I2 is somewhere to the right of lens 2. I2 is somewhere to the left of lens 2.

p2 (e)

(f )

p2

1028

CHAPTE R 34 I MAG ES

Step 2 Neglecting lens 1, treat I1 as though it is the object for lens 2. Use Eq. 34-9 to locate the image I2 produced by lens 2. This is the final image of the system. Determine whether the image is on the left or right side of the lens, whether it is real or virtual, and whether it has the same orientation as the object for lens 2. Roughly sketch I2. The bottom part of Fig. 34-17a gives an example. Thus we treat the two-lens system with two single-lens calculations, using the normal decisions and rules for a single lens. The only exception to the procedure occurs if I1 lies to the right of lens 2 (past lens 2). We still treat it as the object for lens 2, but we take the object distance p2 as a negative number when we use Eq. 34-9 to find I2. Then, as in our other examples, if the image distance i2 is positive, the image is real and on the right side of the lens. An example is sketched in Fig. 34-17b. This same step-by-step analysis can be applied for any number of lenses. It can also be applied if a mirror is substituted for lens 2. The overall (or net) lateral magnification M of a system of lenses (or lenses and a mirror) is the product of the individual lateral magnifications as given by Eq. 34-7 (m ! &i/p). Thus, for a two-lens system, we have M ! m1m2.

(34-11)

If M is positive, the final image has the same orientation as the object (the one in front of lens 1). If M is negative, the final image is inverted from the object. In the situation where p2 is negative, such as in Fig. 34-17b, determining the orientation of the final image is probably easiest by examining the sign of M.

Checkpoint 4 A thin symmetric lens provides an image of a fingerprint with a magnification of '0.2 when the fingerprint is 1.0 cm farther from the lens than the focal point of the lens. What are the (a) type and (b) orientation of the image, and (c) what is the type of lens?

Sample Problem 34.03 Image produced by a thin symmetric lens A praying mantis preys along the central axis of a thin symmetric lens, 20 cm from the lens. The lateral magnification of the mantis provided by the lens is m ! &0.25, and the index of refraction of the lens material is 1.65.

The object must be outside the focal point (the only way a real image can be produced). Also, the image is inverted and on the side of the lens opposite the object. (That is how a converging lens makes a real image.)

(a) Determine the type of image produced by the lens, the type of lens, whether the object (mantis) is inside or outside the focal point, on which side of the lens the image appears, and whether the image is inverted.

(b) What are the two radii of curvature of the lens?

Reasoning: We can tell a lot about the lens and the image from the given value of m. From it and Eq. 34-6 (m ! &i/p), we see that i ! &mp ! 0.25p. Even without finishing the calculation, we can answer the questions. Because p is positive, i here must be positive. That means we have a real image, which means we have a converging lens (the only lens that can produce a real image).

KEY IDEAS 1. Because the lens is symmetric, r1 (for the surface nearer the object) and r2 have the same magnitude r. 2. Because the lens is a converging lens, the object faces a convex surface on the nearer side and so r1 ! 'r. Similarly, it faces a concave surface on the farther side; so r2 ! &r. 3. We can relate these radii of curvature to the focal length f via the lens maker’s equation, Eq. 34-10 (our only equation involving the radii of curvature of a lens). 4. We can relate f to the object distance p and image distance i via Eq. 34-9.

1029

34-4 TH I N LE NSES

Calculations: We know p, but we do not know i. Thus, our starting point is to finish the calculation for i in part (a); we obtain i ! (0.25)(20 cm) ! 5.0 cm.

Equation 34-10 then gives us

# r1 & r1 $ ! (n & 1) # 'r1 & &r1 $

1 ! (n & 1) f

1

2

or, with known values inserted,

Now Eq. 34-9 gives us 1 1 1 1 1 ! ' ! ' , f p i 20 cm 5.0 cm

2 1 ! (1.65 & 1) , 4.0 cm r which yields

from which we find f ! 4.0 cm.

r ! (0.65)(2)(4.0 cm) ! 5.2 cm.

(Answer)

Sample Problem 34.04 Image produced by a system of two thin lenses Figure 34-18a shows a jalapeño seed O1 that is placed in front of two thin symmetrical coaxial lenses 1 and 2, with focal lengths f1 ! '24 cm and f2 ! '9.0 cm, respectively, and with lens separation L ! 10 cm. The seed is 6.0 cm from lens 1. Where does the system of two lenses produce an image of the seed?

Lens 1

Lens 2

O1

KEY IDEA We could locate the image produced by the system of lenses by tracing light rays from the seed through the two lenses. However, we can, instead, calculate the location of that image by working through the system in steps, lens by lens. We begin with the lens closer to the seed. The image we seek is the final one — that is, image I2 produced by lens 2.

L

p1

(a)

Lens 1 I1

First, use the nearest lens to locate its image.

O1

Lens 1: Ignoring lens 2, we locate the image I1 produced by lens 1 by applying Eq. 34-9 to lens 1 alone: p1

1 1 1 ' ! . p1 i1 f1 The object O1 for lens 1 is the seed, which is 6.0 cm from the lens; thus, we substitute p1 ! '6.0 cm. Also substituting the given value of f1, we then have

i1 (b)

Lens 2: In the second step of our solution, we treat image I1 as an object O2 for the second lens and now ignore lens 1. We first note that this object O2 is outside the focal point

Then use that image as the object for the other lens. Lens 2

1 1 1 ' ! , '6.0 cm i1 '24 cm which yields i1 ! &8.0 cm. This tells us that image I1 is 8.0 cm from lens 1 and virtual. (We could have guessed that it is virtual by noting that the seed is inside the focal point of lens 1, that is, between the lens and its focal point.) Because I1 is virtual, it is on the same side of the lens as object O1 and has the same orientation as the seed, as shown in Fig. 34-18b.

L

f1

O2 I2

f2 p2

i2

(c)

Figure 34-18 (a) Seed O1 is distance p1 from a two-lens system with lens separation L. We use the arrow to orient the seed. (b) The image I1 produced by lens 1 alone. (c) Image I1 acts as object O2 for lens 2 alone, which produces the final image I2.

1030

CHAPTE R 34 I MAG ES

of lens 2. So the image I2 produced by lens 2 must be real, inverted, and on the side of the lens opposite O2. Let us see. The distance p2 between this object O2 and lens 2 is, from Fig. 34-18c, p2 ! L ' |i1 | ! 10 cm ' 8.0 cm ! 18 cm. Then Eq. 34-9, now written for lens 2, yields

1 1 1 ' ! . '18 cm i2 '9.0 cm Hence,

i2 ! '18 cm.

(Answer)

The plus sign confirms our guess: Image I2 produced by lens 2 is real, inverted, and on the side of lens 2 opposite O2, as shown in Fig. 34-18c. Thus, the image would appear on a card placed at its location.

Additional examples, video, and practice available at WileyPLUS

34-5 OPTICAL INSTRUMENTS Learning Objectives After reading this module, you should be able to . . .

34.36 Identify the near point in vision. 34.37 With sketches, explain the function of a simple magnifying lens. 34.38 Identify angular magnification. 34.39 Determine the angular magnification for an object at the focal point of a simple magnifying lens. 34.40 With a sketch, explain a compound microscope. 34.41 Identify that the overall magnification of a compound

microscope is due to the lateral magnification by the objective and the angular magnification by the eyepiece. 34.42 Calculate the overall magnification of a compound microscope. 34.43 With a sketch, explain a refracting telescope. 34.44 Calculate the angular magnification of a refracting telescope.

Key Ideas ● The angular magnification of a simple magnifying lens is

25 cm , mu ! f where f is the focal length of the lens and 25 cm is a reference value for the near point value.

where m is the lateral magnification of the objective, mu is the angular magnification of the eyepiece, s is the tube length, fob is the focal length of the objective, and fey is the focal length of the eyepiece. ● The angular magnification of a refracting telescope is

● The overall magnification of a compound microscope is

M ! mmu ! &

s 25 cm , fob fey

mu ! &

fob . fey

Optical Instruments The human eye is a remarkably effective organ, but its range can be extended in many ways by optical instruments such as eyeglasses, microscopes, and telescopes. Many such devices extend the scope of our vision beyond the visible range; satellite-borne infrared cameras and x-ray microscopes are just two examples. The mirror and thin-lens formulas can be applied only as approximations to most sophisticated optical instruments. The lenses in typical laboratory microscopes are by no means “thin.” In most optical instruments the lenses are compound lenses; that is, they are made of several components, the interfaces rarely being exactly spherical. Now we discuss three optical instruments, assuming, for simplicity, that the thin-lens formulas apply.

34-5 OPTICAL I NSTR U M E NTS O

O

h

h

Pn

θ

Pn

25 cm

(b)

(a)

To distant virtual image

I

O h

Pn

θ'

F1

f (c)

Figure 34-19 (a) An object O of height h placed at the near point of a human eye occupies angle u in the eye’s view. (b) The object is moved closer to increase the angle, but now the observer cannot bring the object into focus. (c) A converging lens is placed between the object and the eye, with the object just inside the focal point F1 of the lens. The image produced by the lens is then far enough away to be focused by the eye, and the image occupies a larger angle u9 than object O does in (a).

Simple Magnifying Lens The normal human eye can focus a sharp image of an object on the retina (at the rear of the eye) if the object is located anywhere from infinity to a certain point called the near point Pn. If you move the object closer to the eye than the near point, the perceived retinal image becomes fuzzy. The location of the near point normally varies with age, generally moving away from the person. To find your own near point, remove your glasses or contacts if you wear any, close one eye, and then bring this page closer to your open eye until it becomes indistinct. In what follows, we take the near point to be 25 cm from the eye, a bit more than the typical value for 20-year-olds. Figure 34-19a shows an object O placed at the near point Pn of an eye. The size of the image of the object produced on the retina depends on the angle u that the object occupies in the field of view from that eye. By moving the object closer to the eye, as in Fig. 34-19b, you can increase the angle and, hence, the possibility of distinguishing details of the object. However, because the object is then closer than the near point, it is no longer in focus; that is, the image is no longer clear. You can restore the clarity by looking at O through a converging lens, placed so that O is just inside the focal point F1 of the lens, which is at focal length f (Fig. 34-19c). What you then see is the virtual image of O produced by the lens. That image is farther away than the near point; thus, the eye can see it clearly. Moreover, the angle u# occupied by the virtual image is larger than the largest angle u that the object alone can occupy and still be seen clearly.The angular magnification mu (not to be confused with lateral magnification m) of what is seen is mu ! u#/u. In words, the angular magnification of a simple magnifying lens is a comparison of the angle occupied by the image the lens produces with the angle occupied by the object when the object is moved to the near point of the viewer.

1031

1032

CHAPTE R 34 I MAG ES

Eyepiece Objective O

F1'

F2

I

F1

Parallel rays

I'

To distant virtual image fob

s

fob

fey

Figure 34-20 A thin-lens representation of a compound microscope (not to scale). The objective produces a real image I of object O just inside the focal point F#1 of the eyepiece. Image I then acts as an object for the eyepiece, which produces a virtual final image I# that is seen by the observer. The objective has focal length fob; the eyepiece has focal length fey; and s is the tube length.

From Fig. 34-19, assuming that O is at the focal point of the lens, and approximating tan u as u and tan u# as u# for small angles, we have u % h/25 cm and

We then find that

mu %

25 cm f

u# % h/f.

(simple magnifier).

(34-12)

Compound Microscope Figure 34-20 shows a thin-lens version of a compound microscope. The instrument consists of an objective (the front lens) of focal length fob and an eyepiece (the lens near the eye) of focal length fey. It is used for viewing small objects that are very close to the objective. The object O to be viewed is placed just outside the first focal point F1 of the objective, close enough to F1 that we can approximate its distance p from the lens as being fob. The separation between the lenses is then adjusted so that the enlarged, inverted, real image I produced by the objective is located just inside the first focal point F#1 of the eyepiece. The tube length s shown in Fig. 34-20 is actually large relative to fob, and therefore we can approximate the distance i between the objective and the image I as being length s. From Eq. 34-6, and using our approximations for p and i, we can write the lateral magnification produced by the objective as m!&

s i !& . p fob

(34-13)

Because the image I is located just inside the focal point F#1 of the eyepiece, the eyepiece acts as a simple magnifying lens, and an observer sees a final (virtual, inverted) image I# through it. The overall magnification of the instrument is the product of the lateral magnification m produced by the objective, given by Eq. 34-13, and the angular magnification mu produced by the eyepiece, given by Eq. 34-12; that is, M ! mmu ! &

s fob

25 cm fey

(microscope).

(34-14)

Refracting Telescope Telescopes come in a variety of forms. The form we describe here is the simple refracting telescope that consists of an objective and an eyepiece; both are represented in Fig. 34-21 with simple lenses, although in practice, as is also true for most microscopes, each lens is actually a compound lens system.

1033

34-6 TH R E E PROOFS

The lens arrangements for telescopes and for microscopes are similar, but telescopes are designed to view large objects, such as galaxies, stars, and planets, at large distances, whereas microscopes are designed for just the opposite purpose. This difference requires that in the telescope of Fig. 34-21 the second focal point of the objective F2 coincide with the first focal point of the eyepiece F#1, whereas in the microscope of Fig. 34-20 these points are separated by the tube length s. In Fig. 34-21a, parallel rays from a distant object strike the objective, making an angle uob with the telescope axis and forming a real, inverted image I at the common focal point F2, F#1. This image I acts as an object for the eyepiece, through which an observer sees a distant (still inverted) virtual image I#. The rays defining the image make an angle uey with the telescope axis. The angular magnification mu of the telescope is uey /uob. From Fig. 34-21b, for rays close to the central axis, we can write uob ! h#/fob and uey % h#/fey, which gives us mu ! &

fob fey

(telescope),

Eyepiece Objective

θob

F2 , F1' I

Parallel rays from distant object

Parallel rays

To image I'

fey

fob

(a)

θey

θob h'

I

(34-15)

(b) where the minus sign indicates that I# is inverted. In words, the angular magnification of a telescope is a comparison of the angle occupied by the Figure 34-21 (a) A thin-lens representation of a refractimage the telescope produces with the angle occupied by the distant obing telescope. From rays that are approximately paralject as seen without the telescope. lel when they reach the objective, the objective proMagnification is only one of the design factors for an astronomical duces a real image I of a distant source of light (the telescope and is indeed easily achieved. A good telescope needs lightobject). (One end of the object is assumed to lie on gathering power, which determines how bright the image is.This is importhe central axis.) Image I, formed at the common tant for viewing faint objects such as distant galaxies and is accomplished focal points F2 and F#1, acts as an object for the eyeby making the objective diameter as large as possible. A telescope also piece, which produces a virtual final image I# at a needs resolving power, which is the ability to distinguish between two disgreat distance from the observer. The objective has focal length fob; the eyepiece has focal length fey. (b) tant objects (stars, say) whose angular separation is small. Field of view is Image I has height h# and takes up angle uob meaanother important design parameter. A telescope designed to look at sured from the objective and angle uey measured from galaxies (which occupy a tiny field of view) is much different from one the eyepiece. designed to track meteors (which move over a wide field of view). The telescope designer must also take into account the difference between real lenses and the ideal thin lenses we have discussed. A real lens with spherical surfaces does not form sharp images, a flaw called spherical aberration. Also, because refraction by the two surfaces of a real lens depends on wavelength, a real lens does not focus light of different wavelengths to the same point, a flaw called chromatic aberration. This brief discussion by no means exhausts the design parameters of astronomical telescopes — many others are involved. We could make a similar listing for any other high-performance optical instrument.

34-6 THREE PROOFS

a

The Spherical Mirror Formula (Eq. 34-4) Figure 34-22 shows a point object O placed on the central axis of a concave spherical mirror, outside its center of curvature C. A ray from O that makes an angle a with the axis intersects the axis at I after reflection from the mirror at a. A ray that leaves O along the axis is reflected back along itself at c and also passes through I. Thus, because both rays pass through that common point, I is the image of O; it is a real image because light actually passes through it. Let us find the image distance i.

α

C

O

p

β

θ θ γ Axis c I Mirror

r

i

Figure 34-22 A concave spherical mirror forms a real point image I by reflecting light rays from a point object O.

1034

CHAPTE R 34 I MAG ES

θ1

θ2

α n1

γ

β

c

O

A trigonometry theorem that is useful here tells us that an exterior angle of a triangle is equal to the sum of the two opposite interior angles. Applying this to triangles OaC and OaI in Fig. 34-22 yields

a

C Axis

I

n2 > n1

b ! a ' u and

If we eliminate u between these two equations, we find a ' g ! 2b.

r p

g ! a ' 2u.

i

Figure 34-23 A real point image I of a point object O is formed by refraction at a spherical convex surface between two media.

(34-16)

We can write angles a, b, and g, in radian measure, as ! ! ! ! ac ac ac ac .% /! ! , ! , cO p cC r

0%

and

! ! ac ac ! , cI i

(34-17)

where the overhead symbol means “arc.” Only the equation for b is exact, because ! is at C. However, the equations for a and g are the center of curvature of ac approximately correct if these angles are small enough (that is, for rays close to the central axis). Substituting Eqs. 34-17 into Eq. 34-16, using Eq. 34-3 to replace r with ! lead exactly to Eq. 34-4, the relation that we set out to prove. 2f, and canceling ac

The Refracting Surface Formula (Eq. 34-8) The incident ray from point object O in Fig. 34-23 that falls on point a of a spherical refracting surface is refracted there according to Eq. 33-40, n1 sin u1 ! n2 sin u 2. If a is small, u1 and u2 will also be small and we can replace the sines of these angles with the angles themselves. Thus, the equation above becomes n1u1 % n2u 2.

(34-18)

We again use the fact that an exterior angle of a triangle is equal to the sum of the two opposite interior angles. Applying this to triangles COa and ICa yields u1 ! a ' b and

b ! u2 ' g.

(34-19)

If we use Eqs. 34-19 to eliminate u1 and u2 from Eq. 34-18, we find n1a ' n2g ! (n2 & n1)b.

(34-20)

In radian measure the angles a, b, and g are

.%

! ac ; p

/!

! ac ; r

0%

! ac . i

(34-21)

Only the second of these equations is exact. The other two are approximate ! is a part. However, for because I and O are not the centers of circles of which ac a small enough (for rays close to the axis), the inaccuracies in Eqs. 34-21 are small. Substituting Eqs. 34-21 into Eq. 34-20 leads directly to Eq. 34-8, as we wanted.

The Thin-Lens Formulas (Eqs. 34-9 and 34-10) Our plan is to consider each lens surface as a separate refracting surface, and to use the image formed by the first surface as the object for the second. We start with the thick glass “lens” of length L in Fig. 34-24a whose left and right refracting surfaces are ground to radii r# and r-. A point object O# is placed near the left surface as shown. A ray leaving O# along the central axis is not deflected on entering or leaving the lens. A second ray leaving O# at an angle a with the central axis intersects the left surface at point a#, is refracted, and intersects the second (right) surface at point a-.

34-6 TH R E E PROOFS a" a'

α O' c'

Glass

c"

C"

α O' c'

I'

Axis C' Air

p'

a'

I"

n

n1 = 1.0

r'

r"

C'

p'

i"

Glass

Air

n2 = n

r'

i'

L

(b)

(a)

a"

Figure 34-24 (a) Two rays from point object O# form a real image I- after refracting through two spherical surfaces of a lens. The object faces a convex surface at the left side of the lens and a concave surface at the right side. The ray traveling through points a# and a- is actually close to the central axis through the lens. (b) The left side and (c) the right side of the lens in (a), shown separately.

c'

O"

i'

L (c)

n2 n2 & n1 n1 ' ! . p i r Putting n1 ! 1 for air and n2 ! n for lens glass and bearing in mind that the (virtual) image distance is negative (that is, i ! &i# in Fig. 34-24b), we obtain (34-22)

(Because the minus sign is explicit, i# will be a positive number.) Figure 34-24c shows the second surface again. Unless an observer at point a- were aware of the existence of the first surface, the observer would think that the light striking that point originated at point I# in Fig. 34-24b and that the region to the left of the surface was filled with glass as indicated. Thus, the (virtual) image I# formed by the first surface serves as a real object O- for the second surface. The distance of this object from the second surface is p- ! i# ' L.

(34-23)

To apply Eq. 34-8 to the second surface, we must insert n1 ! n and n2 ! 1 because the object now is effectively imbedded in glass. If we substitute with Eq. 34-23, then Eq. 34-8 becomes 1 1&n n ' ! . i# ' L ir-

(34-24)

Let us now assume that the thickness L of the “lens” in Fig. 34-24a is so small that we can neglect it in comparison with our other linear quantities (such as p#, i#, p-, i-, r#, and r-). In all that follows we make this thin-lens approximation. Putting L ! 0 in Eq. 34-24 and rearranging the right side lead to n 1 n&1 ' !& . i# ir-

Glass r"

p"

The ray is again refracted and crosses the axis at I-, which, being the intersection of two rays from O#, is the image of point O#, formed after refraction at two surfaces. Figure 34-24b shows that the first (left) surface also forms a virtual image of O# at I#. To locate I#, we use Eq. 34-8,

n n&1 1 & ! . p# i# r#

c"

C" n1 = n

(34-25)

n2 = 1.0

I"

Air i"

1035

1036

CHAPTE R 34 I MAG ES

Adding Eqs. 34-22 and 34-25 leads to 1 1 ' ! (n & 1) p# i-

# r#1 & r-1 $.

Finally, calling the original object distance simply p and the final image distance simply i leads to 1 1 ' ! (n & 1) p i

# r#1 & r-1 $,

(34-26)

which, with a small change in notation, is Eqs. 34-9 and 34-10.

Review & Summary Real and Virtual Images An image is a reproduction of an object via light. If the image can form on a surface, it is a real image and can exist even if no observer is present. If the image requires the visual system of an observer, it is a virtual image.

faces the object has a positive radius of curvature; a concave lens surface that faces the object has a negative radius of curvature. Real images form on the side of a lens that is opposite the object, and virtual images form on the same side as the object.

Image Formation Spherical mirrors, spherical refracting sur-

Lateral Magnification The lateral magnification m produced

faces, and thin lenses can form images of a source of light — the object — by redirecting rays emerging from the source. The image occurs where the redirected rays cross (forming a real image) or where backward extensions of those rays cross (forming a virtual image). If the rays are sufficiently close to the central axis through the spherical mirror, refracting surface, or thin lens, we have the following relations between the object distance p (which is positive) and the image distance i (which is positive for real images and negative for virtual images): 1. Spherical Mirror: 1 1 2 1 ' ! ! , p i f r

(34-4, 34-3)

where f is the mirror’s focal length and r is its radius of curvature. A plane mirror is a special case for which r : 1, so that p ! &i. Real images form on the side of a mirror where the object is located, and virtual images form on the opposite side. 2. Spherical Refracting Surface: n1 n2 n2 & n1 ' ! p i r

(single surface),

(34-8)

where n1 is the index of refraction of the material where the object is located, n2 is the index of refraction of the material on the other side of the refracting surface, and r is the radius of curvature of the surface. When the object faces a convex refracting surface, the radius r is positive. When it faces a concave surface, r is negative. Real images form on the side of a refracting surface that is opposite the object, and virtual images form on the same side as the object. 3. Thin Lens: 1 1 1 ' ! ! (n & 1) p i f

# r1 & r1 $, 1

(34-9, 34-10)

2

where f is the lens’s focal length, n is the index of refraction of the lens material, and r1 and r2 are the radii of curvature of the two sides of the lens, which are spherical surfaces. A convex lens surface that

by a spherical mirror or a thin lens is i . p

(34-6)

h# , h

(34-5)

m!& The magnitude of m is given by !m! !

where h and h# are the heights (measured perpendicular to the central axis) of the object and image, respectively.

Optical Instruments Three optical instruments that extend human vision are: 1. The simple magnifying lens, which produces an angular magnification mu given by mu !

25 cm , f

(34-12)

where f is the focal length of the magnifying lens. The distance of 25 cm is a traditionally chosen value that is a bit more than the typical near point for someone 20 years old. 2. The compound microscope, which produces an overall magnification M given by M ! mmu ! &

s 25 cm , fob fey

(34-14)

where m is the lateral magnification produced by the objective, mu is the angular magnification produced by the eyepiece, s is the tube length, and fob and fey are the focal lengths of the objective and eyepiece, respectively. 3. The refracting telescope, which produces an angular magnification mu given by mu ! &

fob . fey

(34-15)

1037

QU ESTIONS

Questions 1 Figure 34-25 shows a fish and a fish stalker in water. (a) Does the stalker see the fish in the general region of point a or point b? (b) Does the fish see the (wild) eyes of the stalker in the general region of point c or point d?

c d

a

b

2 In Fig. 34-26, stick figure O Figure 34-25 Question 1. stands in front of a spherical mirror that is mounted within the boxed reI3 I1 gion; the central axis through the O mirror is shown. The four stick figures I1 to I4 suggest general locaI2 I4 tions and orientations for the images that might be produced by the Figure 34-26 mirror. (The figures are only Questions 2 and 10. sketched in; neither their heights nor their distances from the mirror are drawn to scale.) (a) Which of the stick figures could not possibly represent images? Of the possible images, (b) which would be due to a concave mirror, (c) which would be due to a convex mirror, (d) which would be virtual, and (e) which would involve negative magnification? 3 Figure 34-27 is an overc head view of a mirror maze based on floor sections that are equilateral triangles. x Every wall within the maze is mirrored. If you stand at a entrance x, (a) which of the maze monsters a, b, and c hiding in the maze can you b see along the virtual hallFigure 34-27 Question 3. ways extending from entrance x; (b) how many times does each visible monster appear in a hallway; and (c) what is at the far end of a hallway? 4 A penguin waddles along the central axis of a concave mirror, from the focal point to an effectively infinite distance. (a) How does its image move? (b) Does the height of its image increase continuously, decrease continuously, or change in some more complicated manner? 5 When a T. rex pursues a jeep in the movie Jurassic Park, we see a reflected image of the T. rex via a side-view mirror, on which is printed the (then darkly humorous) warning: “Objects in mirror are closer than they appear.” Is the mirror flat, convex, or concave? 6 An object is placed against the center of a concave mirror and then moved along the central axis |i| until it is 5.0 m from the mirror. 1 1 During the motion, the distance |i| 2 between the mirror and the image it 3 produces is measured. The procedure is then repeated with a convex mirror and a plane mirror. Figure 34-28 gives the results versus object p distance p. Which curve corresponds to which mirror? (Curve 1 has two Figure 34-28 Questions segments.) 6 and 8.

7 The table details six variations of Lens 1 Lens 2 the basic arrangement of two thin lenses represented in Fig. 34-29. (The F1 F2 F2 points labeled F1 and F2 are the focal F1 points of lenses 1 and 2.) An object Figure 34-29 Question 7. is distance p1 to the left of lens 1, as in Fig. 34-18. (a) For which variations can we tell, without calculation, whether the final image (that due to lens 2) is to the left or right of lens 2 and whether it has the same orientation as the object? (b) For those “easy” variations, give the image location as “left” or “right” and the orientation as “same” or “inverted.” Variation

Lens 1

Lens 2

1 2 3 4 5 6

Converging Converging Diverging Diverging Diverging Diverging

Converging Converging Converging Converging Diverging Diverging

p1 % p1 $ p1 % p1 $ p1 % p1 $

!f1! !f1! !f1! !f1! !f1! !f1!

8 An object is placed against the center of a converging lens and then moved along the central axis until it is 5.0 m from the lens. During the motion, the distance !i! between the lens and the image it produces is measured. The procedure is then repeated with a diverging lens. Which of the curves in Fig. 34-28 best gives !i! versus the object distance p for these lenses? (Curve 1 consists of two segments. Curve 3 is straight.) 9 Figure 34-30 shows four thin lenses, all of the same material, with sides that either are flat or have a radius of curvature of magnitude 10 cm. (b) (c) (d) Without written calculation, rank the (a) Figure 34-30 Question 9. lenses according to the magnitude of the focal length, greatest first. 10 In Fig. 34-26, stick figure O stands in front of a thin, symmetric lens that is mounted within the boxed region; the central axis through the lens is shown. The four stick figures I1 to I4 suggest general locations and orientations for the images that might be produced by the lens. (The figures are only sketched in; neither their height nor their distance from the lens is drawn to scale.) (a) Which of the stick figures could not possibly represent images? Of the possible images, (b) which would be due to a converging lens, (c) which would be due to a diverging lens, (d) which would be virtual, and (e) which y would involve negative magnification? 11 Figure 34-31 shows a coordinate system in front of a flat mirror, with the x axis perpendicular to the mirror. Draw the image of the system in the mirror. (a) Which axis is reversed by the reflection? (b) If you face a mirror, is your image inverted (top for bottom)? (c) Is it reversed left and right (as commonly believed)? (d) What then is reversed?

x

z

Figure 34-31 Question 11.

1038

CHAPTE R 34 I MAG ES

Problems Tutoring problem available (at instructor’s discretion) in WileyPLUS and WebAssign SSM • – •••

Worked-out solution available in Student Solutions Manual

WWW Worked-out solution is at

Number of dots indicates level of problem difficulty

ILW

Interactive solution is at

http://www.wiley.com/college/halliday

Additional information available in The Flying Circus of Physics and at flyingcircusofphysics.com

•2 ILW A moth at about eye level is 10 cm in front of a plane mirror; you are behind the moth, 30 cm from the mirror. What is the distance between your eyes and the apparent position of the moth’s image in the mirror? ••3 In Fig. 34-32, an isotropic point source of light S is positioned at distance d from a d viewing screen A and the light intensity IP at point P (level with S) is M measured. Then a plane mirror M is placed behind S at distance d. By how much is IP multiplied by the presence of the mirror? ••4 Figure 34-33 shows an overhead view of a corridor with a plane mirror M mounted at one end. A burglar B sneaks along the corridor directly toward the center of the mirror. If d ! 3.0 m, how far from the mirror will she be when the security guard S can first see her in the mirror? WWW •••5 SSM Figure 34-34 shows a small lightbulb suspended at distance d1 ! 250 cm above the surface of the water in a swimming pool where the water depth is d2 ! 200 cm. The bottom of the pool is a large mirror. How far below the mir-

S

d

P

d

Figure 34-32 Problem 3. d

d S

B

Module 34-2 Spherical Mirrors •6 An object is moved along the central axis of a spherical mirror while the lateral magnification m of it is measured. Figure 34-35 gives m versus object distance p for the range pa ! 2.0 cm to pb ! 8.0 cm.What is m for p = 14.0 cm?

m

ror surface is the image of the bulb? (Hint: Assume that the rays are close to a vertical axis through the bulb, and use the small-angle approximation in which sin u % tan 4 u % u.) 2

0

pa

p (cm)

•7 A concave shaving mirror has a radius of curvature of 35.0 cm. It is positioned so that the (upright) image of a man’s face is 2.50 times the size of the face. How far is the mirror from the face? •8 An object is placed against the center of a spherical mirror and then moved 70 cm from it along the central axis as the image distance i is measured. Figure 34-36 gives i versus object distance p out to ps ! 40 cm. What is i for p ! 70 cm?

400

0

'18 '15 '8.0 '24 '12 '22 '10 '17

ps

–400

Mirror

Figure 34-34 Problem 5.

••17 through 29 22 SSM 23, 29 More mirrors. Object O stands on the central axis of a spherical or plane mirror. For this sit-

Figure 34-33 Problem 4.

d1

Table 34-3 Problems 9 through 16: Spherical Mirrors. See the setup for these problems.

9 10 11 12 13 14 15 16

0

p (cm) ••9 through 16 12 SSM 9, 11, 13 Spherical mirrors. Object O Figure 34-36 Problem 8. stands on the central axis of a spherical mirror. For this situation, each problem in Table 34-3 gives object distance ps (centimeters), the type of mirror, and then the distance (centimeters, without proper sign) between the focal point and the mirror. Find (a) the radius of curvature r (including sign), (b) the image distance i, and (c) the lateral magnification m. Also, determine whether the image is (d) real (R) or virtual (V), (e) inverted (I) from object O or noninverted (NI), and (f) on the same side of the mirror as O or on the opposite side.

d2

p

pb

Figure 34-35 Problem 6.

i (cm)

Module 34-1 Images and Plane Mirrors •1 You look through a camera toward an image of a hummingbird in a plane mirror.The camera is 4.30 m in front of the mirror. The bird is at camera level, 5.00 m to your right and 3.30 m from A the mirror. What is the distance between the camera M and the apparent position of the bird’s image in the mirror?

Mirror Concave, 12 Concave, 10 Convex, 10 Concave, 36 Concave, 18 Convex, 35 Convex, 8.0 Convex, 14

(a) r

(b) i

(c) m

(d) R/V

(e) I/NI

(f) Side

PROB LE M S

1039

Table 34-4 Problems 17 through 29: More Mirrors. See the setup for these problems. (a) Type 17 18 19 20 21 22 23 24 25 26 27 28 29

(b) f

Concave

(c) r

(d) p

20

'10 '24 &40

0.50

'40 '30

'20 20 30

(g) R/V

(h) I/NI

(i) Side

I

&0.70 '0.10 '0.20 &0.50 0.40

'60 '30 '60

20 &30

I Same

&15 '10

Convex

(f) m

&10

40

'1.0 4.0

m

uation, each problem in Table 34-4 refers to (a) the type of mirror, (b) the focal distance f, (c) the radius of curvature r, (d) the object distance p, (e) the image distance i, and (f) the lateral magnification m. (All distances are in cen1 timeters.) It also refers to whether (g) the image is real (R) or virtual (V), (h) inverted (I) or noninverted 0.5 (NI) from O, and (i) on the same side of the mirror as object O or on the opposite side. Fill in the missing information. Where only a sign is 0 ps missing, answer with the sign. p (cm) ••30 Figure 34-37 gives the latFigure 34-37 Problem 30. eral magnification m of an object versus the object distance p from a spherical mirror as the object is moved along the mirror’s central axis through a range of values for p. The horizontal scale is set by ps ! 10.0 cm. What is the magnification of the object when the object is 21 cm from the mirror? ••31 (a) A luminous point is moving at speed vO toward a spherical mirror with radius of curvature r, along the central axis of the mirror. Show that the image of this point is moving at speed vI ! &

(e) i

# 2p r& r $ v , 2

O

where p is the distance of the luminous point from the mirror at any given time. Now assume the mirror is concave, with r ! 15 cm,

and let vO ! 5.0 cm/s. Find vI when (b) p ! 30 cm (far outside the focal point), (c) p ! 8.0 cm ( just outside the focal point), and (d) p ! 10 mm (very near the mirror). Module 34-3 Spherical Refracting Surfaces ••32 through 38 37, 38 SSM 33, 35 Spherical refracting surfaces. An object O stands on the central axis of a spherical refracting surface. For this situation, each problem in Table 34-5 refers to the index of refraction n1 where the object is located, (a) the index of refraction n2 on the other side of the refracting surface, (b) the object distance p, (c) the radius of curvature r of the surface, and (d) the image distance i. (All distances are in centimeters.) Fill in the missing information, including whether the image is (e) real (R) or virtual (V) and (f) on the same side of the surface as object O or on the opposite side. ••39 In Fig. 34-38, a beam of parallel light rays from a laser is incident on a solid transparent sphere of index of refraction n. (a) If a point image is produced at the back of the sphere, what is the index of refraction of the sphere? (b) What index of refraction, if any, will produce a point image at the center of the sphere? Figure 34-38 Problem 39. ••40 A glass sphere has radius R ! 5.0 cm and index of refraction 1.6. A paperweight is constructed by slicing through the sphere along a plane that is 2.0 cm

Table 34-5 Problems 32 through 38: Spherical Refracting Surfaces. See the setup for these problems. n1 32 33 34 35 36 37 38

1.0 1.0 1.5 1.5 1.5 1.5 1.0

(a) n2

(b) p

(c) r

1.5 1.5

'10 '10 '100 '70

'30

1.0 1.0 1.0 1.5

&30 '30 &30

'10 '30

(d) i &13 '600 &7.5 &6.0 '600

(e) R/V

(f) Side

CHAPTE R 34 I MAG ES

from the center of the sphere, leaving height h ! 3.0 cm. The paperweight is placed on a table and viewed from directly above by an observer who is distance d ! 8.0 cm from the tabletop (Fig. 34-39). When viewed through the paperweight, how far away does the tabletop appear to be to the observer?

Observer d h R

6 4

400

i (cm)

•44 An object is placed against the center of a thin lens and then moved away from it along the central axis as the image distance i is measured. Figure 34-41 gives i versus object distance p out to ps ! 60 cm. What is the image distance when p ! 100 cm?

0

0

–400

ps

p (cm)

Figure 34-41 Problem 44. •45 You produce an image of the Sun on a screen, using a thin lens whose focal length is 20.0 cm. What is the diameter of the image? (See Appendix C for needed data on the Sun.) •46 An object is placed against the center of a thin lens and then moved 70 cm from it along the central axis as the image distance i

m

m

Figure 34-39 Problem 40. Module 34-4 Thin Lenses •41 A lens is made of glass having an index of refraction of 1.5. One side of the lens is flat, and the other is convex with a radius of curvature of 20 cm. (a) Find the focal length of the lens. (b) If an object is placed 40 cm in front of the lens, where is the image? 1 •42 Figure 34-40 gives the lateral magnification m of an object versus the object distance p from a lens as 0.5 the object is moved along the central axis of the lens through a range of values for p out to ps ! 20.0 cm. What is the magnification of the ob0 ps ject when the object is 35 cm from p (cm) the lens? Figure 34-40 Problem 42. •43 A movie camera with a (single) lens of focal length 75 mm takes a picture of a person standing 27 m away. If the person is 180 cm tall, what is the height of the image on the film?

is measured. Figure 34-42 gives i verp (cm) 0 sus object distance p out to ps ps ! 40 cm. What is the image distance when p ! 70 cm? –10 •47 SSM WWW A double-convex lens is to be made of glass with an –20 index of refraction of 1.5. One surFigure 34-42 Problem 46. face is to have twice the radius of curvature of the other and the focal length is to be 60 mm. What is the (a) smaller and (b) larger radius? •48 An object is moved along the central axis of a thin lens while the lateral magnification m is measured. Figure 34-43 gives m versus object distance p out to ps ! 8.0 cm. What is the magnification of the object when the object is 14.0 cm from the lens? i (cm)

1040

2 0

ps p (cm)

Figure 34-43 Problem 48. •49 SSM An illuminated slide is held 44 cm from a screen. How far from the slide must a lens of focal length 11 cm be placed (between the slide and the screen) to form an image of the slide’s picture on the screen? ••50 through 57 55, 57 SSM 53 Thin lenses. Object O stands on the central axis of a thin symmetric lens. For this situation, each problem in Table 34-6 gives object distance p (centimeters), the type of lens (C stands for converging and D for diverging), and then the distance (centimeters, without proper sign) between a focal point and the lens. Find (a) the image distance i and (b) the lateral magnification m of the object, including signs. Also, determine whether the image is (c) real (R) or virtual (V), (d) inverted (I) from object O or noninverted (NI), and (e) on the same side of the lens as object O or on the opposite side. ••58 through 67 61 SSM 59 Lenses with given radii. Object O stands in front of a thin lens, on the central axis. For this

Table 34-6 Problems 50 through 57: Thin Lenses. See the setup for these problems. p 50 51 52 53 54 55 56 57

'16 '12 '25 '8.0 '10 '22 '12 '45

Lens C, 4.0 C, 16 C, 35 D, 12 D, 6.0 D, 14 D, 31 C, 20

(a) i

(b) m

(c) R/V

(d) I/NI

(e) Side

PROB LE M S

1041

Table 34-7 Problems 58 through 67: Lenses with Given Radii. See the setup for these problems.

58 59 60 61 62 63 64 65 66 67

(a) i

p

n

r1

r2

'29 '75 '6.0 '24 '10 '35 '10 '10 '18 '60

1.65 1.55 1.70 1.50 1.50 1.70 1.50 1.50 1.60 1.50

'35 '30 '10 &15 '30 '42 &30 &30 &27 '35

1 &42 &12 &25 &30 '33 &60 '30 '24 &35

situation, each problem in Table 34-7 gives object distance p, index of refraction n of the lens, radius r1 of the nearer lens surface, and radius r2 of the farther lens surface. (All distances are in centimeters.) Find (a) the image distance i and (b) the lateral magnification m of the object, including signs. Also, determine whether the image is (c) real (R) or virtual (V), (d) inverted (I) from object O or noninverted (NI), and (e) on the same side of the lens as object O or on the opposite side. ••68 In Fig. 34-44, a real inverted O image I of an object O is formed by Lens a particular lens (not shown); the here Axis object – image separation is d ! 40.0 I cm, measured along the central axis d of the lens. The image is just half the size of the object. (a) What kind of Figure 34-44 Problem 68. lens must be used to produce this image? (b) How far from the object must the lens be placed? (c) What is the focal length of the lens?

(b) m

(c) R/V

(d) I/NI

(e) Side

verging (C) or diverging (D), (b) the focal distance f, (c) the object distance p, (d) the image distance i, and (e) the lateral magnification m. (All distances are in centimeters.) It also refers to whether (f) the image is real (R) or virtual (V), (g) inverted (I) or noninverted (NI) from O, and (h) on the same side of the lens as O or on the opposite side. Fill in the missing information, including the value of m when only an inequality is given. Where only a sign is missing, answer with the sign. ••80 through 87 80, 87 SSM WWW 83 Two-lens systems. In Fig. 34-45, stick figure O (the object) stands on the common central axis of two thin, symmetric lenses, which are mounted in the boxed regions. Lens 1 is mounted within the boxed region closer to O, which is at object distance p1. Lens 2 is mounted within the farther O

••69 through 79 76, 78 SSM 75, 77 More lenses. Object O stands on the central axis of a thin symmetric lens. For this situation, each problem in Table 34-8 refers to (a) the lens type, con-

1

2

d

Figure 34-45 Problems 80 through 87.

Table 34-8 Problems 69 through 79: More Lenses. See the setup for these problems. (a) Type 69 70 71 72 73 74 75 76 77 78 79

C

(b) f

(c) p

'10 20

'5.0 '8.0 '16 '16 '10 '20 '5.0 '5.0 '16 '10 '8.0

10 10 10

20

(d) i

(e) m

%1.0 '0.25 &0.25 &0.50 %1.0 $1.0 '1.25 0.50 $1.0

(f) R/V

(g) I/NI

(h) Side

NI

Same

NI

1042

CHAPTE R 34 I MAG ES

Table 34-9 Problems 80 through 87: Two-Lens Systems. See the setup for these problems.

80 81 82 83 84 85 86 87

p1

Lens 1

d

Lens 2

'10 '12 '8.0 '20 '15 '4.0 '12 '20

C, 15 C, 8.0 D, 6.0 C, 9.0 C, 12 C, 6.0 C, 8.0 D, 12

10 32 12 8.0 67 8.0 30 10

C, 8.0 C, 6.0 C, 6.0 C, 5.0 C, 10 D, 6.0 D, 8.0 D, 8.0

boxed region, at distance d. Each problem in Table 34-9 refers to a different combination of lenses and different values for distances, which are given in centimeters. The type of lens is indicated by C for converging and D for diverging; the number after C or D is the distance between a lens and either of its focal points (the proper sign of the focal distance is not indicated). Find (a) the image distance i2 for the image produced by lens 2 (the final image produced by the system) and (b) the overall lateral magnification M for the system, including signs. Also, determine whether the final image is (c) real (R) or virtual (V), (d) inverted (I) from object O or noninverted (NI), and (e) on the same side of lens 2 as object O or on the opposite side. Module 34-5 Optical Instruments •88 If the angular magnification of an astronomical telescope is 36 and the diameter of the objective is 75 mm, what is the minimum diameter of the eyepiece required to collect all the light entering the objective from a distant point source on the telescope axis? •89 SSM In a microscope of the type shown in Fig. 34-20, the focal length of the objective is 4.00 cm, and that of the eyepiece is 8.00 cm. The distance between the lenses is 25.0 cm. (a) What is the tube length s? (b) If image I in Fig. 34-20 is to be just inside focal point F#1, how far from the objective should the object be? What then are (c) the lateral magnification m of the objective, (d) the angular magnification mu of the eyepiece, and (e) the overall magnification M of the microscope? ••90 Figure 34-46a shows the basic structure of an old film camera. A lens can be moved forward or back to produce an image on film at the back of the camera. For a certain camera, with the distance i between the lens and the film set at f ! 5.0 cm, parallel light rays from a very distant object O converge to a point image on the film, as shown. The object is now brought closer, to a distance of p ! 100 cm, and the lens – film distance is adjusted so that an

(a) i2

(b) M

(c) R/V

(d) I/NI

(e) Side

inverted real image forms on the film (Fig. 34-46b). (a) What is the lens – film distance i now? (b) By how much was distance i changed? ••91 SSM Figure 34-47a shows the basic structure of a human eye. Light refracts into the eye through the cornea and is then further redirected by a lens whose shape (and thus ability to focus the light) is controlled by muscles. We can treat the cornea and eye lens as a single effective thin lens (Fig. 34-47b). A “normal” eye can focus parallel light rays from a distant object O to a point on the retina at the back of the eye, where processing of the visual information begins. As an object is brought close to the eye, however, the muscles must change the shape of the lens so that rays form an inverted real image on the retina (Fig. 34-47c). (a) Suppose that for the parallel rays of Figs. 34-47a and b, the focal length f of the effective thin lens of the eye is 2.50 cm. For an object at distance p ! 40.0 cm, what focal length f# of the effective lens is required for the object to be seen clearly? (b) Must the eye muscles increase or decrease the radii of curvature of the eye lens to give focal length f#? Lens Retina Muscle Cornea Light from distant object O

Effective lens

Retina I

I f (b)

(a) O I p

i (c)

O

Figure 34-47 Problem 91.

I

I

Film f

p

i (b)

(a)

Figure 34-46 Problem 90.

••92 An object is 10.0 mm from the objective of a certain compound microscope. The lenses are 300 mm apart, and the intermediate image is 50.0 mm from the eyepiece. What overall magnification is produced by the instrument? ••93 Someone with a near point Pn of 25 cm views a thimble through a simple magnifying lens of focal length 10 cm by placing

PROB LE M S

the lens near his eye. What is the angular magnification of the thimble if it is positioned so that its image appears at (a) Pn and (b) infinity? p (cm)

0

ps i (cm)

Additional Problems 94 An object is placed against the center of a spherical mirror and then moved 70 cm from it along the central axis as the image distance i is measured. Figure 34-48 gives i versus object distance p out to ps ! 40 cm. What is the image distance when the object is 70 cm from the mirror?

–10

1043

mirrors, you see multiple images of O. You can find them by drawing the reflection in each mirror of the angular region between the mirrors, as is done in Fig. 34-50b for the left-hand mirror. Then draw the reflection of the reflection. Continue this on the left and on the right until the reflections meet or overlap at the rear of the mirrors. Then you can count the number of images of O. How many images of O would you see if u is (a) 90°, (b) 45°, and (c) 60°? If u ! 120°, determine the (d) smallest and (e) largest number of images that can be seen, depending on your perspective and the location of O. (f ) In each situation, draw the image locations and orientations as in Fig. 34-50b.

–20

Figure 34-48 Problem 94.

95 through 100 95, 96, 99 Three-lens systems. In Fig. 34-49, stick figure O (the object) stands on the common central axis of three thin, symmetric lenses, which are mounted in the boxed regions. Lens 1 is mounted within the boxed region closest to O, which is at object distance p1. Lens 2 is mounted within the middle boxed region, at distance d12 from lens 1. Lens 3 is mounted in the farthest boxed region, at distance d23 from lens 2. Each problem in Table 34-10 refers to a different combination of lenses and different values for distances, which are given in centimeters. The type of lens is indicated by C for converging and D for diverging; the number after C or D is the distance between a lens and either of the focal points (the proper sign of the focal distance is not indicated). Find (a) the image distance i3 1 2 3 for the (final) image produced by lens 3 (the final image produced by O the system) and (b) the overall latd12 d23 eral magnification M for the system, Figure 34-49 Problems 95 including signs. Also, determine through 100. whether the final image is (c) real (R) or virtual (V), (d) inverted (I) from object O or noninverted (NI), and (e) on the same side of lens 3 as object O or on the opposite side. 101 SSM The formula 1/p ' 1/i ! 1/f is called the Gaussian form of the thin-lens formula. Another form of this formula, the Newtonian form, is obtained by considering the distance x from the object to the first focal point and the distance x# from the second focal point to the image. Show that xx# ! f 2 is the Newtonian form of the thin-lens formula. 102 Figure 34-50a is an overhead view of two vertical plane mirrors with an object O placed between them. If you look into the

θ θ

θ O

O (a)

(b)

Figure 34-50 Problem 102. 103 SSM Two thin lenses of focal lengths f1 and f2 are in contact and share the same central axis. Show that, in image formation, they are equivalent to a single thin lens for which the focal length is f ! f1 f2 /( f1 ' f2). 104 Two plane mirrors are placed parallel to each other and 40 cm apart. An object is placed 10 cm from one mirror. Determine the (a) smallest, (b) second smallest, (c) third smallest (occurs twice), and (d) fourth smallest distance between the object and image of the object. 105 In Fig. 34-51, a box is somewhere at the left, on the central axis Im of the thin converging lens. The image Im of the box produced by the plane mirror is 4.00 cm “inside” the mirror. The lens – mirror separation Figure 34-51 Problem 105. is 10.0 cm, and the focal length of the lens is 2.00 cm. (a) What is the distance between the box and the lens? Light reflected by the mirror travels back through the lens, which produces a final image of the box. (b) What is the distance between the lens and that final image?

Table 34-10 Problems 95 through 100: Three-Lens Systems. See the setup for these problems. p1 95 96 97 98 99 100

'12 '4.0 '18 '2.0 '8.0 '4.0

Lens 1

d12

Lens 2

d23

Lens 3

C, 8.0 D, 6.0 C, 6.0 C, 6.0 D, 8.0 C, 6.0

28 9.6 15 15 8.0 8.0

C, 6.0 C, 6.0 C, 3.0 C, 6.0 D, 16 D, 4.0

8.0 14 11 19 5.1 5.7

C, 6.0 C, 4.0 C, 3.0 C, 5.0 C, 8.0 D, 12

(a) i3

(b) M

(c) R/V

(d) I/NI

(e) Side

1044

CHAPTE R 34 I MAG ES

106 In Fig. 34-52, an object is placed in front of a converging lens at a distance equal to twice the focal length f1 of the lens. On the other side of the lens is a concave mirror of focal length f2 separated from the lens by a distance 2( f1 ' f2). Light from the object passes rightward through the lens, reflects from the mirror, passes leftward through the lens, and forms a final image of the object. What are (a) the distance between the lens and that final image and (b) the overall lateral magnification M of the object? Is the image (c) real or virtual (if it is virtual, it requires someone looking through the lens toward the mirror), (d) to the left or right of the lens, and (e) inverted or noninverted relative to the object? f1

110 A goldfish in a spherical fish bowl of radius R is at the level of the center C of the bowl and at distance R/2 from the glass (Fig. 34-55). What magnification of the fish is produced by the water in the bowl for a viewer looking along a line that includes the fish and the center, with the fish on the near side of the center? The index of refraction of the water is 1.33. Neglect the glass wall of the bowl. Assume the viewer looks with one eye. (Hint: Equation 34-5 holds, but Eq. 34-6 does not. You need to work with a ray diagram of the situation and assume that the rays are close to the observer’s line of sight — that is, they deviate from that line by only small angles.)

f2

O

C 2f1

2(f1 + f2)

R __ 2

Figure 34-52 Problem 106. R

107 SSM A fruit fly of height H sits in front of lens 1 on the central axis through the lens. The lens forms an image of the fly at a distance d ! 20 cm from the fly; the image has the fly’s orientation and height HI ! 2.0H. What are (a) the focal length f1 of the lens and (b) the object distance p1 of the fly? The fly then leaves lens 1 and sits in front of lens 2, which also forms an image at d ! 20 cm that has the same orientation as the fly, but now HI ! 0.50H. What are (c) f2 and (d) p2? 108 You grind the lenses shown in Fig. 34-53 from flat glass disks (n ! 1.5) using a machine that can grind a radius of curvature of either 40 cm or (2) (3) 60 cm. In a lens where either radius is (1) appropriate, you select the 40 cm radius. Then you hold each lens in sunshine to form an image of the Sun. What are the (a) focal length f and (b) image type (real or virtual) for (5) (6) (bi-convex) lens 1, (c) f and (d) image (4) type for (plane-convex) lens 2, (e) f Figure 34-53 and (f) image type for (meniscus conProblem 108. vex) lens 3, (g) f and (h) image type for (bi-concave) lens 4, (i) f and ( j) image type for (plane-concave) lens 5, and (k) f and (l) image type for (meniscus concave) lens 6? 109 In Fig. 34-54, a fish watcher at d2 d3 d1 point P watches a fish through a glass wall of a fish tank. The watcher is level with the fish; the index of re- P fraction of the glass is 8/5, and that Watcher of the water is 4/3. The distances are d1 ! 8.0 cm, d2 ! 3.0 cm, and d3 ! Wall 6.8 cm. (a) To the fish, how far away Figure 34-54 does the watcher appear to be? Problem 109. (Hint: The watcher is the object. Light from that object passes through the wall’s outside surface, which acts as a refracting surface. Find the image produced by that surface. Then treat that image as an object whose light passes through the wall’s inside surface, which acts as another refracting surface.) (b) To the watcher, how far away does the fish appear to be?

Figure 34-55 Problem 110. 111 Figure 34-56 shows a beam exd pander made with two coaxial con2 1 verging lenses of focal lengths f1 and f2 and separation d ! f1 ' f2. The de- W Wf i vice can expand a laser beam while keeping the light rays in the beam parallel to the central axis through Figure 34-56 Problem 111. the lenses. Suppose a uniform laser beam of width Wi ! 2.5 mm and intensity Ii ! 9.0 kW/m2 enters a beam expander for which f1 ! 12.5 cm and f2 ! 30.0 cm. What are (a) Wf and (b) If of the beam leaving the expander? (c) What value of d is needed for the beam expander if lens 1 is replaced with a diverging lens of focal length f1 ! &26.0 cm? 112 You look down at a coin that lies at the bottom of a pool of liquid of depth d and index of refraction n (Fig. 34-57). Because you view with two eyes, which intercept different rays of light from the coin, you perceive the coin to be where extensions of the intercepted rays cross, at depth da instead of d. Assuming that the intercepted rays in Fig. 34-57 are close to a vertical axis through the coin, show that da ! d/n. (Hint: Use the small-angle approximation sin u % tan u % u.)

To left eye

To right eye

da

Air n

d

Figure 34-57 Problem 112. 113 A pinhole camera has the hole a distance 12 cm from the film plane, which is a rectangle of height 8.0 cm and width 6.0 cm. How far from a painting of dimensions 50 cm by 50 cm should the camera be placed so as to get the largest complete image possible on the film plane? 114 Light travels from point A to point B via reflection at point O on the surface of a mirror. Without using calculus, show that length AOB is a minimum when the angle of incidence u is equal to the angle of reflection f. (Hint: Consider the image of A in the mirror.)

PROB LE M S

115 A point object is 10 cm away from a plane mirror, and the eye of an observer (with pupil diameter 5.0 mm) is 20 cm away. Assuming the eye and the object to be on the same line perpendicular to the mirror surface, find the area of the mirror used in observing the reflection of the point. (Hint: Adapt Fig. 34-4.) 116 Show that the distance between an object and its real image formed by a thin converging lens is always greater than or equal to four times the focal length of the lens. 117 A luminous object and a screen are a fixed distance D apart. (a) Show that a converging lens of focal length f, placed between object and screen, will form a real image on the screen for two lens positions that are separated by a distance d ! 2D(D & 4f ). (b) Show that D&d 2 D'd

#

$

gives the ratio of the two image sizes for these two positions of the lens. 118 An eraser of height 1.0 cm is placed 10.0 cm in front of a two-lens system. Lens 1 (nearer the eraser) has focal length f1 ! &15 cm, lens 2 has f2 ! 12 cm, and the lens separation is d ! 12 cm. For the image produced by lens 2, what are (a) the image distance i2 (including sign), (b) the image height, (c) the image type (real or virtual), and (d) the image orientation (inverted relative to the eraser or not inverted)? 119 A peanut is placed 40 cm in front of a two-lens system: lens 1 (nearer the peanut) has focal length f1 ! '20 cm, lens 2 has f2 ! &15 cm, and the lens separation is d ! 10 cm. For the image produced by lens 2, what are (a) the image distance i2 (including sign), (b) the image orientation (inverted relative to the peanut or not inverted), and (c) the image type (real or virtual)? (d) What is the net lateral magnification? 120 A coin is placed 20 cm in front of a two-lens system. Lens 1 (nearer the coin) has focal length f1 ! '10 cm, lens 2 has f2 ! '12.5 cm, and the lens separation is d ! 30 cm. For the image produced by lens 2, what are (a) the image distance i2 (including sign), (b) the overall lateral magnification, (c) the image type (real or virtual), and (d) the image orientation (inverted relative to the coin or not inverted)? 121 An object is 20 cm to the left of a thin diverging lens that has a 30 cm focal length. (a) What is the image distance i? (b) Draw a ray diagram showing the image position. 122 In Fig 34-58 a pinecone is at distance p1 ! 1.0 m in front of a lens of focal length f1 ! 0.50 m; a flat mirror is at distance d ! 2.0 m behind the lens. Light from the pinecone passes rightward through the lens, p1 d reflects from the mirror, passes leftFigure 34-58 Problem 122. ward through the lens, and forms a final image of the pinecone. What are (a) the distance between the lens and that image and (b) the overall lateral magnification of the pinecone? Is the image (c) real or virtual (if it is virtual, it requires someone looking through the lens toward the mirror), (d) to the left or right of the lens, and (e) inverted relative to the pinecone or not inverted? 123 One end of a long glass rod (n ! 1.5) is a convex surface of radius 6.0 cm. An object is located in air along the axis of the rod, at a distance of 10 cm from the convex end. (a) How far apart are the

1045

object and the image formed by the glass rod? (b) Within what range of distances from the end of the rod must the object be located in order to produce a virtual image? 124 A short straight object of length L lies along the central axis of a spherical mirror, a distance p from the mirror. (a) Show that its image in the mirror has a length L#, where

# p &f f $ .

L# ! L

2

(Hint: Locate the two ends of the object.) (b) Show that the longitudinal magnification m#(! L#/L) is equal to m2, where m is the lateral magnification. 125 Prove that if a plane mirror is rotated through an angle a, the reflected beam is rotated through an angle 2a. Show that this result is reasonable for a ! 45". 126 An object is 30.0 cm from a spherical mirror, along the mirror’s central axis. The mirror produces an inverted image with a lateral magnification of absolute value 0.500. What is the focal length of the mirror? 127 A concave mirror has a radius of curvature of 24 cm. How far is an object from the mirror if the image formed is (a) virtual and 3.0 times the size of the object, (b) real and 3.0 times the size of the object, and (c) real and 1/3 the size of the object? 128 A pepper seed is placed in front of a lens. The lateral magnification of the seed is '0.300. The absolute value of the lens’s focal length is 40.0 cm. How far from the lens is the image? 129 The equation 1/p ' 1/i ! 2/r for spherical mirrors is an approximation that is valid if the image is formed by rays that make only small angles with the central axis. In reality, many of the angles are large, which smears the image a little. You can determine how much. Refer to Fig. 34-22 and consider a ray that leaves a point source (the object) on the central axis and that makes an angle a with that axis. First, find the point of intersection of the ray with the mirror. If the coordinates of this intersection point are x and y and the origin is placed at the center of curvature, then y ! (x ' p & r) tan a and x2 ' y2 ! r2, where p is the object distance and r is the mirror’s radius of curvature. Next, use tan b ! y/x to find the angle b at the point of intersection, and then use a ' g ! 2b to find the value of g. Finally, use the relation tan g ! y/(x ' i & r) to find the distance i of the image. (a) Suppose r ! 12 cm and p ! 20 cm. For each of the following values of a, find the position of the image — that is, the position of the point where the reflected ray crosses the central axis: 0.500, 0.100, 0.0100 rad. Compare the results with those obtained with the equation 1/p ' 1/i ! 2/r. (b) Repeat the calculations for p ! 4.00 cm. 130 A small cup of green tea is positioned on the central axis of a spherical mirror. The lateral magnification of the cup is '0.250, and the distance between the mirror and its focal point is 2.00 cm. (a) What is the distance between the mirror and the image it produces? (b) Is the focal length positive or negative? (c) Is the image real or virtual? 131 A 20-mm-thick layer of water (n ! 1.33) floats on a 40-mm-thick layer of carbon tetrachloride (n ! 1.46) in a tank. A coin lies at the bottom of the tank. At what depth below the top water surface do you perceive the coin? (Hint: Use the result and assumptions of Problem 112 and work with a ray diagram.)

1046

CHAPTE R 34 I MAG ES

132 A millipede sits 1.0 m in front of the nearest part of the surface of a shiny sphere of diameter 0.70 m. (a) How far from the surface does the millipede’s image appear? (b) If the millipede’s height is 2.0 mm, what is the image height? (c) Is the image inverted? 133 (a) Show that if the object O in Fig. 34-19c is moved from focal point F1 toward the observer’s eye, the image moves in from infinity and the angle u# (and thus the angular magnification mu) increases. (b) If you continue this process, where is the image when mu has its maximum usable value? (You can then still increase mu, but the image will no longer be clear.) (c) Show that the maximum usable value of mu is 1 ' (25 cm)/f. (d) Show that in this situation the angular magnification is equal to the lateral magnification. 134 Isaac Newton, having convinced himself (erroneously as it turned out) that chromatic aberration is an inherent property of refracting telescopes, invented the reflecting teleM' F scope, shown schematically in Fig. 34-59. He presented his second model of this Eyepiece telescope, with a magnifying power of 38, to the Royal Society (of London), which still has it. In Fig. 34-59 incident light falls, closely parallel to the telescope axis, on the objective mirror M. After reflection from small mirror M# M (the figure is not to scale), the rays form a real, inverted image in the focal Objective plane (the plane perpendicular to mirror the line of sight, at focal point F). Figure 34-59 This image is then viewed through an Problem 134. eyepiece. (a) Show that the angular magnification mu for the device is given by Eq. 34-15: mu ! &fob /fey, where fob is the focal length of the objective mirror and fey is that of the eyepiece. (b) The 200 in. mirror in the reflecting telescope at Mt. Palomar in California has a focal length of 16.8 m. Estimate the size of the image formed by this mirror when the object is a meter stick 2.0 km away. Assume parallel incident rays. (c) The mirror of a different reflecting astronomical telescope has an effective radius of curvature of 10 m (“effective” because such mirrors are ground to a parabolic rather than a spherical shape, to eliminate spherical aberration defects). To give an angular magnification of 200, what must be the focal length of the eyepiece? 135 A narrow beam of parallel light rays is incident on a glass sphere from the left, directed toward the center of the sphere. (The

sphere is a lens but certainly not a thin lens.) Approximate the angle of incidence of the rays as 0", and assume that the index of refraction of the glass is n % 2.0. (a) In terms of n and the sphere radius r, what is the distance between the image produced by the sphere and the right side of the sphere? (b) Is the image to the left or right of that side? (Hint: Apply Eq. 34-8 to locate the image that is produced by refraction at the left side of the sphere; then use that image as the object for refraction at the right side of the sphere to locate the final image. In the second refraction, is the object distance p positive or negative?) 136 A corner reflector, much used in optical, microwave, and other applications, consists of three plane mirrors fastened together to form the corner of a cube. Show that after three reflections, an incident ray is returned with its direction exactly reversed. 137 A cheese enchilada is 4.00 cm in front of a converging lens. The magnification of the enchilada is &2.00. What is the focal length of the lens? 138 A grasshopper hops to a point on the central axis of a spherical mirror. The absolute magnitude of the mirror’s focal length is 40.0 cm, and the lateral magnification of the image produced by the mirror is '0.200. (a) Is the mirror convex or concave? (b) How far from the mirror is the grasshopper? 139 In Fig. 34-60, a sand grain is 3.00 cm from thin lens 1, on the central axis through the two symmetric lenses. 1 2 The distance between focal point and Figure 34-60 Problem 139. lens is 4.00 cm for both lenses; the lenses are separated by 8.00 cm. (a) What is the distance between lens 2 and the image it produces of the sand grain? Is that image (b) to the left or right of lens 2, (c) real or virtual, and (d) inverted relative to the sand grain or not inverted? 140 Suppose the farthest distance a person can see without visual aid is 50 cm. (a) What is the focal length of the corrective lens that will allow the person to see very far away? (b) Is the lens converging or diverging? (c) The power P of a lens (in diopters) is equal to 1/f, where f is in meters. What is P for the lens? 141 A simple magnifier of focal length f is placed near the eye of someone whose near point Pn is 25 cm. An object is positioned so that its image in the magnifier appears at Pn. (a) What is the angular magnification of the magnifier? (b) What is the angular magnification if the object is moved so that its image appears at infinity? For f ! 10 cm, evaluate the angular magnifications of (c) the situation in (a) and (d) the situation in (b). (Viewing an image at Pn requires effort by muscles in the eye, whereas viewing an image at infinity requires no such effort for many people.)

C

H

A

P

T

E

R

3

5

Interference 35-1 LIGHT AS A WAVE Learning Objectives After reading this module, you should be able to . . .

35.01 Using a sketch, explain Huygens’ principle. 35.02 With a few simple sketches, explain refraction in terms of the gradual change in the speed of a wavefront as it passes through an interface at an angle to the normal. 35.03 Apply the relationship between the speed of light in vacuum c, the speed of light in a material v, and the index of refraction of the material n. 35.04 Apply the relationship between a distance L in a material, the speed of light in that material, and the time required for a pulse of the light to travel through L. 35.05 Apply Snell’s law of refraction. 35.06 When light refracts through an interface, identify that the frequency does not change but the wavelength and effective speed do. 35.07 Apply the relationship between the wavelength in vacuum l, the wavelength ln in a material (the internal wavelength), and the index of refraction n of the material.

35.08 For light in a certain length of a material, calculate the number of internal wavelengths that fit into the length. 35.09 If two light waves travel through different materials with different indexes of refraction and then reach a common point, determine their phase difference and interpret the resulting interference in terms of maximum brightness, intermediate brightness, and darkness. 35.10 Apply the learning objectives of Module 17-3 (sound waves there, light waves here) to find the phase difference and interference of two waves that reach a common point after traveling paths of different lengths. 35.11 Given the initial phase difference between two waves with the same wavelength, determine their phase difference after they travel through different path lengths and through different indexes of refraction. 35.12 Identify that rainbows are examples of optical interference.

Key Ideas ● The three-dimensional transmission of waves, including light, may often be predicted by Huygens’ principle, which states that all points on a wavefront serve as point sources of spherical secondary wavelets. After a time t, the new position of the wavefront will be that of a surface tangent to these secondary wavelets.

● The wavelength ln of light in a medium depends on the

● The law of refraction can be derived from Huygens’

● Because of this dependency, the phase difference between

principle by assuming that the index of refraction of any medium is n ! c/v, in which v is the speed of light in the medium and c is the speed of light in vacuum.

index of refraction n of the medium: ln !

l , n

in which l is the wavelength in vacuum. two waves can change if they pass through different materials with different indexes of refraction.

What Is Physics? One of the major goals of physics is to understand the nature of light. This goal has been difficult to achieve (and has not yet fully been achieved) because light is complicated. However, this complication means that light offers many opportunities for applications, and some of the richest opportunities involve the interference of light waves — optical interference. Nature has long used optical interference for coloring. For example, the wings of a Morpho butterfly are a dull, uninspiring brown, as can be seen on the 1047

1048

CHAPTE R 35 I NTE R FE R E NCE

bottom wing surface, but the brown is hidden on the top surface by an arresting blue due to the interference of light reflecting from that surface (Fig. 351). Moreover, the top surface is color-shifting; if you change your perspective or if the wing moves, the tint of the color changes. Similar color shifting is used in the inks on many currencies to thwart counterfeiters, whose copy machines can duplicate color from only one perspective and therefore cannot duplicate any shift in color caused by a change in perspective. To understand the basic physics of optical interference, we must largely abandon the simplicity of geometrical optics (in which we describe light as rays) and return to the wave nature of light.

Light as a Wave Philippe Colombi/PhotoDisc/Getty Images, Inc.

The first convincing wave theory for light was in 1678 by Dutch physicist Christian Huygens. Mathematically simpler than the electromagnetic theory of Maxwell, it nicely explained reflection and refraction in terms of waves and gave physical meaning to the index of refraction. Huygens’ wave theory is based on a geometrical construction that allows us to tell where a given wavefront will be at any time in the future if we know its present position. Huygens’ principle is:

Figure 35-1 The blue of the top surface of a Morpho butterfly wing is due to optical interference and shifts in color as your viewing perspective changes.

b

d

All points on a wavefront serve as point sources of spherical secondary wavelets. After a time t, the new position of the wavefront will be that of a surface tangent to these secondary wavelets.

c ∆t

New position of wavefront at time t = ∆t

Wavefront at t=0

a

e

Figure 35-2 The propagation of a plane wave in vacuum, as portrayed by Huygens’ principle.

Refraction occurs at the surface, giving a new direction of travel.

Here is a simple example.At the left in Fig. 35-2, the present location of a wavefront of a plane wave traveling to the right in vacuum is represented by plane ab, perpendicular to the page. Where will the wavefront be at time 2t later? We let several points on plane ab (the dots) serve as sources of spherical secondary wavelets that are emitted at t ! 0. At time 2t, the radius of all these spherical wavelets will have grown to c 2t, where c is the speed of light in vacuum. We draw plane de tangent to these wavelets at time 2t. This plane represents the wavefront of the plane wave at time 2t; it is parallel to plane ab and a perpendicular distance c 2t from it.

The Law of Refraction We now use Huygens’ principle to derive the law of refraction, Eq. 33-40 (Snell’s law). Figure 35-3 shows three stages in the refraction of several wavefronts at a flat interface between air (medium 1) and glass (medium 2). We arbitrarily choose the wavefronts in the incident light beam to be separated by l1, the wavelength in medium 1. Let the speed of light in air be v1 and that in glass be v2. We assume that v2 % v1, which happens to be true.

e

λ1

Incident wave

θ1

(a)

v1

h

Air Glass (b)

θ1 λ2

λ1 θ2 g

c Refracted wave

λ2

v2

(c)

Figure 35-3 The refraction of a plane wave at an air – glass interface, as portrayed by Huygens’ principle. The wavelength in glass is smaller than that in air. For simplicity, the reflected wave is not shown. Parts (a) through (c) represent three successive stages of the refraction.

35-1 LIG HT AS A WAVE

Angle u1 in Fig. 35-3a is the angle between the wavefront and the interface; it has the same value as the angle between the normal to the wavefront (that is, the incident ray) and the normal to the interface. Thus, u1 is the angle of incidence. As the wave moves into the glass, a Huygens wavelet at point e in Fig. 35-3b will expand to pass through point c, at a distance of l1 from point e. The time interval required for this expansion is that distance divided by the speed of the wavelet, or l1/v1. Now note that in this same time interval, a Huygens wavelet at point h will expand to pass through point g, at the reduced speed v2 and with wavelength l2. Thus, this time interval must also be equal to l2/v2. By equating these times of travel, we obtain the relation v l1 ! 1, l2 v2

(35-1)

which shows that the wavelengths of light in two media are proportional to the speeds of light in those media. By Huygens’ principle, the refracted wavefront must be tangent to an arc of radius l2 centered on h, say at point g. The refracted wavefront must also be tangent to an arc of radius l1 centered on e, say at c. Then the refracted wavefront must be oriented as shown. Note that u2, the angle between the refracted wavefront and the interface, is actually the angle of refraction. For the right triangles hce and hcg in Fig. 35-3b we may write sin u1 !

l1 hc

(for triangle hce)

l2 (for triangle hcg). hc Dividing the first of these two equations by the second and using Eq. 35-1, we find and

sin u2 !

l1 v1 sin u1 ! ! . sin u2 l2 v2

(35-2)

We can define the index of refraction n for each medium as the ratio of the speed of light in vacuum to the speed of light v in the medium. Thus, c v

n!

(35-3)

(index of refraction).

In particular, for our two media, we have n1 !

c v1

and

n2 !

c . v2

We can now rewrite Eq. 35-2 as sin u1 c/n1 n2 ! ! sin u2 c/n2 n1 or

n1 sin u1 ! n2 sin u2

(35-4)

(law of refraction),

as introduced in Chapter 33.

Checkpoint 1 The figure shows a monochromatic ray of light traveling across parallel interfaces, from an original material a, through layers of materials b and c, and then back into material a. Rank the materials according to the speed of light in them, greatest first.

b a

c

a

1049

1050

CHAPTE R 35 I NTE R FE R E NCE

Wavelength and Index of Refraction We have now seen that the wavelength of light changes when the speed of the light changes, as happens when light crosses an interface from one medium into another. Further, the speed of light in any medium depends on the index of refraction of the medium, according to Eq. 35-3. Thus, the wavelength of light in any medium depends on the index of refraction of the medium. Let a certain monochromatic light have wavelength l and speed c in vacuum and wavelength ln and speed v in a medium with an index of refraction n. Now we can rewrite Eq. 35-1 as v (35-5) ln ! l . c Using Eq. 35-3 to substitute 1/n for v/c then yields ln !

l . n

(35-6)

This equation relates the wavelength of light in any medium to its wavelength in vacuum: A greater index of refraction means a smaller wavelength. Next, let fn represent the frequency of the light in a medium with index of refraction n. Then from the general relation of Eq. 16-13 (v ! lf ), we can write fn !

v . ln

Substituting Eqs. 35-3 and 35-6 then gives us fn ! The difference in indexes causes a phase shift between the rays. n2 n1 L

Figure 35-4 Two light rays travel through two media having different indexes of refraction.

c c/n ! ! f, l/n l

where f is the frequency of the light in vacuum. Thus, although the speed and wavelength of light in the medium are different from what they are in vacuum, the frequency of the light in the medium is the same as it is in vacuum. Phase Difference. The fact that the wavelength of light depends on the index of refraction via Eq. 35-6 is important in certain situations involving the interference of light waves. For example, in Fig. 35-4, the waves of the rays (that is, the waves represented by the rays) have identical wavelengths l and are initially in phase in air (n % 1). One of the waves travels through medium 1 of index of refraction n1 and length L. The other travels through medium 2 of index of refraction n2 and the same length L. When the waves leave the two media, they will have the same wavelength — their wavelength l in air. However, because their wavelengths differed in the two media, the two waves may no longer be in phase. The phase difference between two light waves can change if the waves travel through different materials having different indexes of refraction.

As we shall discuss soon, this change in the phase difference can determine how the light waves will interfere if they reach some common point. To find their new phase difference in terms of wavelengths, we first count the number N1 of wavelengths there are in the length L of medium 1. From Eq. 35-6, the wavelength in medium 1 is ln1 ! l/n1; so N1 !

L Ln1 ! . ln1 l

(35-7)

Similarly, we count the number N2 of wavelengths there are in the length L of medium 2, where the wavelength is ln2 ! l/n2: N2 !

L Ln2 ! . ln2 l

(35-8)

35-1 LIG HT AS A WAVE

To find the new phase difference between the waves, we subtract the smaller of N1 and N2 from the larger. Assuming n2 $ n1, we obtain N2 & N1 !

Ln1 L Ln2 & ! (n2 & n1). l l l

(35-9)

Suppose Eq. 35-9 tells us that the waves now have a phase difference of 45.6 wavelengths. That is equivalent to taking the initially in-phase waves and shifting one of them by 45.6 wavelengths. However, a shift of an integer number of wavelengths (such as 45) would put the waves back in phase; so it is only the decimal fraction (here, 0.6) that is important. A phase difference of 45.6 wavelengths is equivalent to an effective phase difference of 0.6 wavelength. A phase difference of 0.5 wavelength puts two waves exactly out of phase. If the waves had equal amplitudes and were to reach some common point, they would then undergo fully destructive interference, producing darkness at that point. With a phase difference of 0.0 or 1.0 wavelength, they would, instead, undergo fully constructive interference, resulting in brightness at the common point. Our phase difference of 0.6 wavelength is an intermediate situation but closer to fully destructive interference, and the waves would produce a dimly illuminated common point. We can also express phase difference in terms of radians and degrees, as we have done already. A phase difference of one wavelength is equivalent to phase differences of 2p rad and 360°. Path Length Difference. As we discussed with sound waves in Module 173, two waves that begin with some initial phase difference can end up with a different phase difference if they travel through paths with different lengths before coming back together. The key for the waves (whatever their type might be) is the path length difference 2L, or more to the point, how 2L compares to the wavelength l of the waves. From Eqs. 17-23 and 17-24, we know that, for light waves, fully constructive interference (maximum brightness) occurs when 2L ! 0, 1, 2, . . . l

(fully constructive interference),

(35-10)

and that fully destructive interference (darkness) occurs when 2L ! 0.5, 1.5, 2.5, . . . l

(fully destructive interference).

(35-11)

Intermediate values correspond to intermediate interference and thus also illumination.

Rainbows and Optical Interference In Module 33-5, we discussed how the colors of sunlight are separated into a rainbow when sunlight travels through falling raindrops. We dealt with a simplified situation in which a single ray of white light entered a drop. Actually, light waves pass into a drop along the entire side that faces the Sun. Here we cannot discuss the details of how these waves travel through the drop and then emerge, but we can see that different parts of an incoming wave will travel different paths within the drop. That means waves will emerge from the drop with different phases. Thus, we can see that at some angles the emerging light will be in phase and give constructive interference. The rainbow is the result of such constructive interference. For example, the red of the rainbow appears because waves of red light emerge in phase from each raindrop in the direction in which you see that part of the rainbow. The light waves that emerge in other directions from each raindrop have a range of different phases because they take a

1051

1052

CHAPTE R 35 I NTE R FE R E NCE

Primary rainbow Supernumeraries

Figure 35-5 A primary rainbow and the faint supernumeraries below it are due to optical interference.

range of different paths through each drop. This light is neither bright nor colorful, and so you do not notice it. If you are lucky and look carefully below a primary rainbow, you can see dimmer colored arcs called supernumeraries (Fig. 35-5). Like the main arcs of the rainbow, the supernumeraries are due to waves that emerge from each drop approximately in phase with one another to give constructive interference. If you are very lucky and look very carefully above a secondary rainbow, you might see even more (but even dimmer) supernumeraries. Keep in mind that both types of rainbows and both sets of supernumeraries are naturally occurring examples of optical interference and naturally occurring evidence that light consists of waves.

Checkpoint 2 The light waves of the rays in Fig. 35-4 have the same wavelength and amplitude and are initially in phase. (a) If 7.60 wavelengths fit within the length of the top material and 5.50 wavelengths fit within that of the bottom material, which material has the greater index of refraction? (b) If the rays are angled slightly so that they meet at the same point on a distant screen, will the interference there result in the brightest possible illumination, bright intermediate illumination, dark intermediate illumination, or darkness?

Sample Problem 35.01 Phase difference of two waves due to difference in refractive indexes In Fig. 35-4, the two light waves that are represented by the rays have wavelength 550.0 nm before entering media 1 and 2. They also have equal amplitudes and are in phase. Medium 1 is now just air, and medium 2 is a transparent plastic layer of index of refraction 1.600 and thickness 2.600 mm.

Thus, the phase difference of the emerging waves is 2.84 wavelengths. Because 1.0 wavelength is equivalent to 2p rad and 360°, you can show that this phase difference is equivalent to

(a) What is the phase difference of the emerging waves in wavelengths, radians, and degrees? What is their effective phase difference (in wavelengths)?

The effective phase difference is the decimal part of the actual phase difference expressed in wavelengths. Thus, we have

phase difference ! 17.8 rad % 1020°.

(Answer)

effective phase difference ! 0.84 wavelength. (Answer)

KEY IDEA The phase difference of two light waves can change if they travel through different media, with different indexes of refraction. The reason is that their wavelengths are different in the different media. We can calculate the change in phase difference by counting the number of wavelengths that fits into each medium and then subtracting those numbers. Calculations: When the path lengths of the waves in the two media are identical, Eq. 35-9 gives the result of the subtraction. Here we have n1 ! 1.000 (for the air), n2 ! 1.600, L ! 2.600 mm, and l ! 550.0 nm.Thus, Eq. 35-9 yields N2 & N1 ! !

L (n2 & n1) 3 2.600 ( 10&6 m (1.600 & 1.000) 5.500 ( 10&7 m

! 2.84.

(Answer)

You can show that this is equivalent to 5.3 rad and about 300°. Caution: We do not find the effective phase difference by taking the decimal part of the actual phase difference as expressed in radians or degrees. For example, we do not take 0.8 rad from the actual phase difference of 17.8 rad. (b) If the waves reached the same point on a distant screen, what type of interference would they produce? Reasoning: We need to compare the effective phase difference of the waves with the phase differences that give the extreme types of interference. Here the effective phase difference of 0.84 wavelength is between 0.5 wavelength (for fully destructive interference, or the darkest possible result) and 1.0 wavelength (for fully constructive interference, or the brightest possible result), but closer to 1.0 wavelength. Thus, the waves would produce intermediate interference that is closer to fully constructive interference — they would produce a relatively bright spot.

Additional examples, video, and practice available at WileyPLUS

35-2 YOU NG'S I NTE R FE R E NCE EXPE R I M E NT

1053

35-2 YOUNG’S INTERFERENCE EXPERIMENT Learning Objectives After reading this module, you should be able to . . .

35.13 Describe the diffraction of light by a narrow slit and the effect of narrowing the slit. 35.14 With sketches, describe the production of the interference pattern in a double-slit interference experiment using monochromatic light. 35.15 Identify that the phase difference between two waves can change if the waves travel along paths of different lengths, as in the case of Young’s experiment. 35.16 In a double-slit experiment, apply the relationship between the path length difference 2L and the wavelength l, and then interpret the result in terms of interference (maximum brightness, intermediate brightness, and darkness). 35.17 For a given point in a double-slit interference pattern, express the path length difference 2L of the rays reaching that point in terms of the slit separation d and the angle u to that point. 35.18 In a Young's experiment, apply the relationships between the slit separation d, the light wavelength l, and the

angles u to the minima (dark fringes) and to the maxima (bright fringes) in the interference pattern. 35.19 Sketch the double-slit interference pattern, identifying what lies at the center and what the various bright and dark fringes are called (such as “first side maximum” and “third order”). 35.20 Apply the relationship between the distance D between a double-slit screen and a viewing screen, the angle u to a point in the interference pattern, and the distance y to that point from the pattern’s center. 35.21 For a double-slit interference pattern, identify the effects of changing d or l and also identify what determines the angular limit to the pattern. 35.22 For a transparent material placed over one slit in a Young’s experiment, determine the thickness or index of refraction required to shift a given fringe to the center of the interference pattern.

Key Ideas ● In Young’s interference experiment, light passing through a single slit falls on two slits in a screen. The light leaving these slits

flares out (by diffraction), and interference occurs in the region beyond the screen. A fringe pattern, due to the interference, forms on a viewing screen. ● The conditions for maximum and minimum intensity are

d sin u ! ml,

for m ! 0, 1, 2, . . .

(maxima — bright fringes),

d sin u ! (m ' 12 )l,

for m ! 0, 1, 2, . . . (minima — dark fringes), where u is the angle the light path makes with a central axis and d is the slit separation.

Diffraction In this module we shall discuss the experiment that first proved that light is a wave. To prepare for that discussion, we must introduce the idea of diffraction of waves, a phenomenon that we explore much more fully in Chapter 36. Its essence is this: If a wave encounters a barrier that has an opening of dimensions similar to the wavelength, the part of the wave that passes through the opening will flare (spread) out — will diffract — into the region beyond the barrier. The flaring is consistent with the spreading of wavelets in the Huygens construction of Fig. 35-2. Diffraction occurs for waves of all types, not just light waves; Fig. 35-6 shows the diffraction of water waves traveling across the surface of water in a shallow tank. Similar diffraction of ocean waves through openings in a barrier can actually increase the erosion of a beach the barrier is intended to protect.

Figure 35-6 Waves produced by an oscillating paddle at the left flare out through an opening in a barrier along the water surface.

George Resch/Fundamental Photographs

1054

CHAPTE R 35 I NTE R FE R E NCE

A wave passing through a slit flares (diffracts). Incident wave λ

Diffracted wave

a (6.0λ)

(a )

Figure 35-7a shows the situation schematically for an incident plane wave of wavelength l encountering a slit that has width a ! 6.0l and extends into and out of the page. The part of the wave that passes through the slit flares out on the far side. Figures 35-7b (with a ! 3.0l) and 35-7c (a ! 1.5l) illustrate the main feature of diffraction: the narrower the slit, the greater the diffraction. Diffraction limits geometrical optics, in which we represent an electromagnetic wave with a ray. If we actually try to form a ray by sending light through a narrow slit, or through a series of narrow slits, diffraction will always defeat our effort because it always causes the light to spread. Indeed, the narrower we make the slits (in the hope of producing a narrower beam), the greater the spreading is. Thus, geometrical optics holds only when slits or other apertures that might be located in the path of light do not have dimensions comparable to or smaller than the wavelength of the light.

Screen

Young’s Interference Experiment λ a (3.0λ)

(b )

In 1801, Thomas Young experimentally proved that light is a wave, contrary to what most other scientists then thought. He did so by demonstrating that light undergoes interference, as do water waves, sound waves, and waves of all other types. In addition, he was able to measure the average wavelength of sunlight; his value, 570 nm, is impressively close to the modern accepted value of 555 nm. We shall here examine Young’s experiment as an example of the interference of light waves. Figure 35-8 gives the basic arrangement of Young’s experiment. Light from a distant monochromatic source illuminates slit S0 in screen A. The emerging light then spreads via diffraction to illuminate two slits S1 and S2 in screen B. Diffraction of the light by these two slits sends overlapping circular waves into

λ Max

a

Max

(1.5λ)

Max

Incident wave (c )

Figure 35-7 Diffraction represented schematically. For a given wavelength l, the diffraction is more pronounced the smaller the slit width a. The figures show the cases for (a) slit width a ! 6.0l, (b) slit width a ! 3.0l, and (c) slit width a ! 1.5l. In all three cases, the screen and the length of the slit extend well into and out of the page, perpendicular to it.

Max

S2

The waves emerging from the two slits overlap and form an interference pattern.

Max Max

S0

Max Max Max

S1

Max Max Max

A

B

C

Max

Figure 35-8 In Young’s interference experiment, incident monochromatic light is diffracted by slit S0, which then acts as a point source of light that emits semicircular wavefronts. As that light reaches screen B, it is diffracted by slits S1 and S2, which then act as two point sources of light. The light waves traveling from slits S1 and S2 overlap and undergo interference, forming an interference pattern of maxima and minima on viewing screen C. This figure is a cross section; the screens, slits, and interference pattern extend into and out of the page. Between screens B and C, the semicircular wavefronts centered on S2 depict the waves that would be there if only S2 were open. Similarly, those centered on S1 depict waves that would be there if only S1 were open.

35-2 YOU NG'S I NTE R FE R E NCE EXPE R I M E NT

1055

the region beyond screen B, where the waves from one slit interfere with the waves from the other slit. The “snapshot” of Fig. 35-8 depicts the interference of the overlapping waves. However, we cannot see evidence for the interference except where a viewing screen C intercepts the light. Where it does so, points of interference maxima form visible bright rows — called bright bands, bright fringes, or (loosely speaking) maxima — that extend across the screen (into and out of the page in Fig. 35-8). Dark regions — called dark bands, dark fringes, or (loosely speaking) minima — result from fully destructive interference and are visible between adjacent pairs of bright fringes. (Maxima and minima more properly refer to the center of a band.) The pattern of bright and dark fringes on the screen is called an interference pattern. Figure 35-9 is a photograph of part of the interference pattern that would be seen by an observer standing to the left of screen C in the arrangement of Fig. 35-8.

Locating the Fringes Light waves produce fringes in a Young’s double-slit interference experiment, as it is called, but what exactly determines the locations of the fringes? To answer, we shall use the arrangement in Fig. 35-10a.There, a plane wave of monochromatic light is incident on two slits S1 and S2 in screen B; the light diffracts through the slits and produces an interference pattern on screen C. We draw a central axis from the point halfway between the slits to screen C as a reference. We then pick, for discussion, an arbitrary point P on the screen, at angle u to the central axis. This point intercepts the wave of ray r1 from the bottom slit and the wave of ray r2 from the top slit. Path Length Difference. These waves are in phase when they pass through the two slits because there they are just portions of the same incident wave. However, once they have passed the slits, the two waves must travel different distances to reach P. We saw a similar situation in Module 17-3 with sound waves and concluded that

Courtesy Jearl Walker

Figure 35-9 A photograph of the interference pattern produced by the arrangement shown in Fig. 35-8, but with short slits. (The photograph is a front view of part of screen C.) The alternating maxima and minima are called interference fringes (because they resemble the decorative fringe sometimes used on clothing and rugs).

The phase difference between two waves can change if the waves travel paths of different lengths.

The change in phase difference is due to the path length difference 2L in the paths taken by the waves. Consider two waves initially exactly in phase, traveling along paths with a path length difference 2L, and then passing through some common point. When 2L is zero or an integer number of wavelengths, the waves arrive at the common point exactly in phase and they interfere fully constructively there. If that is true for the waves of rays r1 and r2 in Fig. 35-10, then D r2

P

Incident wave

r2

S2 S1

b

θ

r1

y

d

d S1 (b )

(a )

B

C

θ

S2

θ

r1

The ∆L shifts one wave from the other, which determines the interference.

θb Path length difference ∆L

Figure 35-10 (a) Waves from slits S1 and S2 (which extend into and out of the page) combine at P, an arbitrary point on screen C at distance y from the central axis. The angle u serves as a convenient locator for P. (b) For D 4 d, we can approximate rays r1 and r2 as being parallel, at angle u to the central axis.

1056

CHAPTE R 35 I NTE R FE R E NCE

point P is part of a bright fringe. When, instead, 2L is an odd multiple of half a wavelength, the waves arrive at the common point exactly out of phase and they interfere fully destructively there. If that is true for the waves of rays r1 and r2, then point P is part of a dark fringe. (And, of course, we can have intermediate situations of interference and thus intermediate illumination at P.) Thus, What appears at each point on the viewing screen in a Young’s double-slit interference experiment is determined by the path length difference 2L of the rays reaching that point.

Angle. We can specify where each bright fringe and each dark fringe is located on the screen by giving the angle u from the central axis to that fringe. To find u, we must relate it to 2L. We start with Fig. 35-10a by finding a point b along ray r1 such that the path length from b to P equals the path length from S2 to P. Then the path length difference 2L between the two rays is the distance from S1 to b. The relation between this S1-to-b distance and u is complicated, but we can simplify it considerably if we arrange for the distance D from the slits to the screen to be much greater than the slit separation d. Then we can approximate rays r1 and r2 as being parallel to each other and at angle u to the central axis (Fig. 35-10b). We can also approximate the triangle formed by S1, S2, and b as being a right triangle, and approximate the angle inside that triangle at S2 as being u. Then, for that triangle, sin u ! 2L/d and thus 2L ! d sin u

(path length difference).

(35-12)

For a bright fringe, we saw that 2L must be either zero or an integer number of wavelengths. Using Eq. 35-12, we can write this requirement as 2L ! d sin u ! (integer)(l),

(35-13)

or as d sin u ! ml,

for m ! 0, 1, 2, . . .

(maxima — bright fringes).

(35-14)

For a dark fringe, 2L must be an odd multiple of half a wavelength. Again using Eq. 35-12, we can write this requirement as or as

2L ! d sin u ! (odd number)(12 l), d sin u ! (m ' 12 )l,

for m ! 0, 1, 2, + + +

(minima — dark fringes).

(35-15) (35-16)

With Eqs. 35-14 and 35-16, we can find the angle u to any fringe and thus locate that fringe; further, we can use the values of m to label the fringes. For the value and label m ! 0, Eq. 35-14 tells us that a bright fringe is at u ! 0 and thus on the central axis. This central maximum is the point at which waves arriving from the two slits have a path length difference 2L ! 0, hence zero phase difference. For, say, m ! 2, Eq. 35-14 tells us that bright fringes are at the angle u ! sin&1

# 2ld $

above and below the central axis. Waves from the two slits arrive at these two fringes with 2L ! 2l and with a phase difference of two wavelengths. These fringes are said to be the second-order bright fringes (meaning m ! 2) or the second side maxima (the second maxima to the side of the central maximum), or

35-2 YOU NG'S I NTE R FE R E NCE EXPE R I M E NT

1057

they are described as being the second bright fringes from the central maximum. For m ! 1, Eq. 35-16 tells us that dark fringes are at the angle u ! sin&1

# 1.5l d $

above and below the central axis. Waves from the two slits arrive at these two fringes with 2L ! 1.5l and with a phase difference, in wavelengths, of 1.5. These fringes are called the second-order dark fringes or second minima because they are the second dark fringes to the side of the central axis. (The first dark fringes, or first minima, are at locations for which m ! 0 in Eq. 35-16.) Nearby Screen. We derived Eqs. 35-14 and 35-16 for the situation D 4 d. However, they also apply if we place a converging lens between the slits and the viewing screen and then move the viewing screen closer to the slits, to the focal point of the lens. (The screen is then said to be in the focal plane of the lens; that is, it is in the plane perpendicular to the central axis at the focal point.) One property of a converging lens is that it focuses all rays that are parallel to one another to the same point on its focal plane. Thus, the rays that now arrive at any point on the screen (in the focal plane) were exactly parallel (rather than approximately) when they left the slits. They are like the initially parallel rays in Fig. 34-14a that are directed to a point (the focal point) by a lens.

Checkpoint 3 In Fig. 35-10a, what are 2L (as a multiple of the wavelength) and the phase difference (in wavelengths) for the two rays if point P is (a) a third side maximum and (b) a third minimum?

Sample Problem 35.02 Double-slit interference pattern What is the distance on screen C in Fig. 35-10a between adjacent maxima near the center of the interference pattern? The wavelength l of the light is 546 nm, the slit separation d is 0.12 mm, and the slit – screen separation D is 55 cm. Assume that u in Fig. 35-10 is small enough to permit use of the approximations sin u % tan u % u, in which u is expressed in radian measure.

Calculations: If we equate our two expressions for angle u and then solve for ym, we find ym !

ym'1 !

ym . D (2) From Eq. 35-14, this angle u for the mth maximum is given by tan u % u !

ml sin u % u ! . d

(35-17)

For the next maximum as we move away from the pattern’s center, we have

KEY IDEAS (1) First, let us pick a maximum with a low value of m to ensure that it is near the center of the pattern. Then, from the geometry of Fig. 35-10a, the maximum’s vertical distance ym from the center of the pattern is related to its angle u from the central axis by

mlD . d

(m ' 1)lD . d

(35-18)

We find the distance between these adjacent maxima by subtracting Eq. 35-17 from Eq. 35-18: 2y ! ym'1 & ym ! !

lD d

(546 ( 10&9 m)(55 ( 10&2 m) 0.12 ( 10&3 m

! 2.50 ( 10&3 m % 2.5 mm.

(Answer)

As long as d and u in Fig. 35-10a are small, the separation of the interference fringes is independent of m; that is, the fringes are evenly spaced.

Additional examples, video, and practice available at WileyPLUS

1058

CHAPTE R 35 I NTE R FE R E NCE

Sample Problem 35.03 Double-slit interference pattern with plastic over one slit A double-slit interference pattern is produced on a screen, as in Fig. 35-10; the light is monochromatic at a wavelength of 600 nm. A strip of transparent plastic with index of refraction n ! 1.50 is to be placed over one of the slits. Its presence changes the interference between light waves from the two slits, causing the interference pattern to be shifted across the screen from the original pattern. Figure 35-11a shows the original locations of the central bright fringe (m ! 0) and the first bright fringes (m ! 1) above and below the central fringe. The purpose of the plastic is to shift the pattern upward so that the lower m ! 1 bright fringe is shifted to the center of the pattern. Should the plastic be placed over the top slit (as arbitrarily drawn in Fig. 3511b) or the bottom slit, and what thickness L should it have? KEY IDEA The interference at a point on the screen depends on the phase difference of the light rays arriving from the two slits. The light rays are in phase at the slits because they derive from the same wave, but their relative phase can shift on the way to the screen due to (1) a difference in the length of the paths they follow and (2) a difference in the number of their internal wavelengths ln in the materials through which they pass. The first condition applies to any off-center point, and the second condition applies when the plastic covers one of the slits. Path length difference: Figure 35-11a shows rays r1 and r2 along which waves from the two slits travel to reach the lower m ! 1 bright fringe. Those waves start in phase at the slits but arrive at the fringe with a phase difference of exactly 1 wavelength. To remind ourselves of this main characteristic of the fringe, let us call it the 1l fringe. The onewavelength phase difference is due to the one-wavelength path length difference between the rays reaching the fringe; that is, there is exactly one more wavelength along ray r2 than along r1. Figure 35-11b shows the 1l fringe shifted up to the center of the pattern with the plastic strip over the top slit (we still do not know whether the plastic should be there or over the bottom slit). The figure also shows the new orientations of rays r1 and r2 to reach that fringe. There still must be one more wavelength along r2 than along r1 (because they still produce the 1l fringe), but now the path length difference between those rays is zero, as we can tell from the geometry of Fig. 35-11b. However, r2 now passes through the plastic.

Internal wavelength: The wavelength l n of light in a material with index of refraction n is smaller than the wavelength in vacuum, as given by Eq. 35-6 (l n ! l/n). Here, this means that the wavelength of the light is smaller in the plastic than in the air. Thus, the ray that passes through the plastic will have more wavelengths along it than the ray that passes through only air — so we do get the one extra wavelength we need along ray r2 by placing the plastic over the top slit, as drawn in Fig. 35-11b. Thickness: To determine the required thickness L of the plastic, we first note that the waves are initially in phase and travel equal distances L through different materials (plastic and air). Because we know the phase difference and require L, we use Eq. 35-9, L (35-19) (n2 & n1). 3 We know that N2 & N1 is 1 for a phase difference of one wavelength, n2 is 1.50 for the plastic in front of the top slit, n1 is 1.00 for the air in front of the bottom slit, and l is 600 ( 10&9 m. Then Eq. 35-19 tells us that, to shift the lower m = 1 bright fringe up to the center of the interference pattern, the plastic must have the thickness N2 & N1 !

L!

(600 ( 10&9 m)(1) l(N2 & N1) ! n2 & n1 1.50 & 1.00

! 1.2 ( 10&6 m.

(Answer)

The difference in indexes causes a phase shift between the rays, moving the 1l fringe upward. m=1 r2

m=0

r1

m=1 1l fringe

(a)

r2 r1

1l fringe

(b)

Figure 35-11 (a) Arrangement for two-slit interference (not to scale). The locations of three bright fringes (or maxima) are indicated. (b) A strip of plastic covers the top slit. We want the 1l fringe to be at the center of the pattern.

Additional examples, video, and practice available at WileyPLUS

35-3 I NTE R FE R E NCE AN D DOU B LE-SLIT I NTE NSITY

1059

35-3 INTERFERENCE AND DOUBLE-SLIT INTENSITY Learning Objectives After reading this module, you should be able to . . .

35.23 Distinguish between coherent and incoherent light. 35.24 For two light waves arriving at a common point, write expressions for their electric field components as functions of time and a phase constant. 35.25 Identify that the phase difference between two waves determines their interference. 35.26 For a point in a double-slit interference pattern, calculate the intensity in terms of the phase difference of

the arriving waves and relate that phase difference to the angle u locating that point in the pattern. 35.27 Use a phasor diagram to find the resultant wave (amplitude and phase constant) of two or more light waves arriving at a common point and use that result to determine the intensity. 35.28 Apply the relationship between a light wave’s angular frequency v and the angular speed v of the phasor representing the wave.

Key Ideas ● If two light waves that meet at a point are to interfere perceptibly, the phase difference between them must remain constant with time; that is, the waves must be coherent. When two coherent waves meet, the resulting intensity may be found by using phasors.

● In Young’s interference experiment, two waves, each with intensity I0, yield a resultant wave of intensity I at the viewing screen, with

I ! 4I0 cos2 12 f,

Coherence For the interference pattern to appear on viewing screen C in Fig. 35-8, the light waves reaching any point P on the screen must have a phase difference that does not vary in time. That is the case in Fig. 35-8 because the waves passing through slits S1 and S2 are portions of the single light wave that illuminates the slits. Because the phase difference remains constant, the light from slits S1 and S2 is said to be completely coherent. Sunlight and Fingernails. Direct sunlight is partially coherent; that is, sunlight waves intercepted at two points have a constant phase difference only if the points are very close. If you look closely at your fingernail in bright sunlight, you can see a faint interference pattern called speckle that causes the nail to appear to be covered with specks. You see this effect because light waves scattering from very close points on the nail are sufficiently coherent to interfere with one another at your eye. The slits in a double-slit experiment, however, are not close enough, and in direct sunlight, the light at the slits would be incoherent. To get coherent light, we would have to send the sunlight through a single slit as in Fig. 35-8; because that single slit is small, light that passes through it is coherent. In addition, the smallness of the slit causes the coherent light to spread via diffraction to illuminate both slits in the double-slit experiment. Incoherent Sources. If we replace the double slits with two similar but independent monochromatic light sources, such as two fine incandescent wires, the phase difference between the waves emitted by the sources varies rapidly and randomly. (This occurs because the light is emitted by vast numbers of atoms in the wires, acting randomly and independently for extremely short times — of the order of nanoseconds.) As a result, at any given point on the viewing screen, the interference between the waves from the two sources varies rapidly and randomly between fully constructive and fully destructive. The eye (and most common optical detectors) cannot follow such changes, and no interference pattern can be seen. The fringes disappear, and the screen is seen as being uniformly illuminated.

where f !

2pd sin u. l

1060

CHAPTE R 35 I NTE R FE R E NCE

Coherent Source. A laser differs from common light sources in that its atoms emit light in a cooperative manner, thereby making the light coherent. Moreover, the light is almost monochromatic, is emitted in a thin beam with little spreading, and can be focused to a width that almost matches the wavelength of the light.

Intensity in Double-Slit Interference Equations 35-14 and 35-16 tell us how to locate the maxima and minima of the double-slit interference pattern on screen C of Fig. 35-10 as a function of the angle u in that figure. Here we wish to derive an expression for the intensity I of the fringes as a function of u. The light leaving the slits is in phase. However, let us assume that the light waves from the two slits are not in phase when they arrive at point P. Instead, the electric field components of those waves at point P are not in phase and vary with time as and

E1 ! E0 sin vt

(35-20)

E2 ! E0 sin(vt ' f),

(35-21)

where v is the angular frequency of the waves and f is the phase constant of wave E2. Note that the two waves have the same amplitude E0 and a phase difference of f. Because that phase difference does not vary, the waves are coherent. We shall show that these two waves will combine at P to produce an intensity I given by I ! 4I0 cos2 12 f,

(35-22)

2pd sin u. l

(35-23)

and that f!

In Eq. 35-22, I0 is the intensity of the light that arrives on the screen from one slit when the other slit is temporarily covered. We assume that the slits are so narrow in comparison to the wavelength that this single-slit intensity is essentially uniform over the region of the screen in which we wish to examine the fringes. Equations 35-22 and 35-23, which together tell us how the intensity I of the fringe pattern varies with the angle u in Fig. 35-10, necessarily contain information about the location of the maxima and minima. Let us see if we can extract that information to find equations about those locations. Maxima. Study of Eq. 35-22 shows that intensity maxima will occur when 1 2f

for m ! 0, 1, 2, . . . .

! mp,

(35-24)

If we put this result into Eq. 35-23, we find 2mp ! or

2pd sin u, l

d sin u ! ml,

for m ! 0, 1, 2, . . .

for m ! 0, 1, 2, . . .

(maxima),

(35-25)

which is exactly Eq. 35-14, the expression that we derived earlier for the locations of the maxima. Minima. The minima in the fringe pattern occur when 1 2f

! (m ' 12 )p,

for m ! 0, 1, 2, . . . .

(35-26)

If we combine this relation with Eq. 35-23, we are led at once to d sin u ! (m ' 12 )l,

for m ! 0, 1, 2, . . .

(minima),

(35-27)

35-3 I NTE R FE R E NCE AN D DOU B LE-SLIT I NTE NSITY

Intensity at screen

1061

4I 0 (two coherent sources) 2I 0 (two incoherent sources) I 0 (one source)







2 2 2.5

2π 1

1 2

π

1.5

0

0.5



0 0



1

0 0

1

π

0.5

4π 2

1 1



1.5

2 2

2.5

φ m, for maxima m, for minima ∆L/λ

Figure 35-12 A plot of Eq. 35-22, showing the intensity of a double-slit interference pattern as a function of the phase difference between the waves when they arrive from the two slits. I0 is the (uniform) intensity that would appear on the screen if one slit were covered.The average intensity of the fringe pattern is 2I0, and the maximum intensity (for coherent light) is 4I0.

which is just Eq. 35-16, the expression we derived earlier for the locations of the fringe minima. Figure 35-12, which is a plot of Eq. 35-22, shows the intensity of double-slit interference patterns as a function of the phase difference f between the waves at the screen. The horizontal solid line is I0, the (uniform) intensity on the screen when one of the slits is covered up. Note in Eq. 35-22 and the graph that the intensity I varies from zero at the fringe minima to 4I0 at the fringe maxima. If the waves from the two sources (slits) were incoherent, so that no enduring phase relation existed between them, there would be no fringe pattern and the intensity would have the uniform value 2I0 for all points on the screen; the horizontal dashed line in Fig. 35-12 shows this uniform value. Interference cannot create or destroy energy but merely redistributes it over the screen. Thus, the average intensity on the screen must be the same 2I0 regardless of whether the sources are coherent. This follows at once from Eq. 35-22; if we substitute 12, the average value of the cosine-squared function, this equation reduces to Iavg ! 2I0.

E2 E1

E ! 2(E 0 cos b) ! 2E 0 cos 12 f.

(35-28)

E0

φ ωt

Proof of Eqs. 35-22 and 35-23 We shall combine the electric field components E1 and E2, given by Eqs. 35-20 and 35-21, respectively, by the method of phasors as is discussed in Module 16-6. In Fig. 35-13a, the waves with components E1 and E2 are represented by phasors of magnitude E0 that rotate around the origin at angular speed v. The values of E1 and E2 at any time are the projections of the corresponding phasors on the vertical axis. Figure 35-13a shows the phasors and their projections at an arbitrary time t. Consistent with Eqs. 35-20 and 35-21, the phasor for E1 has a rotation angle vt and the phasor for E2 has a rotation angle vt ' f (it is phase-shifted ahead of E1). As each phasor rotates, its projection on the vertical axis varies with time in the same way that the sinusoidal functions of Eqs. 35-20 and 35-21 vary with time. To combine the field components E1 and E2 at any point P in Fig. 35-10, we add their phasors vectorially, as shown in Fig. 35-13b. The magnitude of the vector sum is the amplitude E of the resultant wave at point P, and that wave has a certain phase constant b. To find the amplitude E in Fig. 35-13b, we first note that the two angles marked b are equal because they are opposite equal-length sides of a triangle. From the theorem (for triangles) that an exterior angle (here f, as shown in Fig. 35-13b) is equal to the sum of the two opposite interior angles (here that sum is b ' b), we see that b ! 12 f. Thus, we have

ω

E0

(a )

Phasors that represent waves can be added to find the net wave. ω

E2

β E E1

β

E0

φ E0

ωt (b )

Figure 35-13 (a) Phasors representing, at time t, the electric field components given by Eqs. 35-20 and 35-21. Both phasors have magnitude E 0 and rotate with angular speed v. Their phase difference is f. (b) Vector addition of the two phasors gives the phasor representing the resultant wave, with amplitude E and phase constant b.

1062

CHAPTE R 35 I NTE R FE R E NCE

If we square each side of this relation, we obtain E 2 ! 4E 20 cos2 12 f.

(35-29)

Intensity. Now, from Eq. 33-24, we know that the intensity of an electromagnetic wave is proportional to the square of its amplitude. Therefore, the waves we are combining in Fig. 35-13b, whose amplitudes are E 0, each has an intensity I0 that is proportional to E 20 , and the resultant wave, with amplitude E, has an intensity I that is proportional to E 2. Thus, I E2 ! 2. I0 E0 Substituting Eq. 35-29 into this equation and rearranging then yield I ! 4I0 cos2 12 f, which is Eq. 35-22, which we set out to prove. We still must prove Eq. 35-23, which relates the phase difference f between the waves arriving at any point P on the screen of Fig. 35-10 to the angle u that serves as a locator of that point. The phase difference f in Eq. 35-21 is associated with the path length difference S1b in Fig. 35-10b. If S1b is 12 l, then f is p; if S1b is l, then f is 2p, and so on. This suggests phase length . #difference $ ! 2pl #path difference $

(35-30)

The path length difference S1b in Fig. 35-10b is d sin u (a leg of the right triangle); so Eq. 35-30 for the phase difference between the two waves arriving at point P on the screen becomes f!

2pd sin u, l

which is Eq. 35-23, the other equation that we set out to prove to relate f to the angle u that locates P.

Combining More Than Two Waves In a more general case, we might want to find the resultant of more than two sinusoidally varying waves at a point. Whatever the number of waves is, our general procedure is this: 1. Construct a series of phasors representing the waves to be combined. Draw them end to end, maintaining the proper phase relations between adjacent phasors. 2. Construct the vector sum of this array. The length of this vector sum gives the amplitude of the resultant phasor. The angle between the vector sum and the first phasor is the phase of the resultant with respect to this first phasor. The projection of this vector-sum phasor on the vertical axis gives the time variation of the resultant wave.

Checkpoint 4 Each of four pairs of light waves arrives at a certain point on a screen. The waves have the same wavelength. At the arrival point, their amplitudes and phase differences are (a) 2E0, 6E0, and p rad; (b) 3E0, 5E0, and p rad; (c) 9E0, 7E0, and 3p rad; (d) 2E0, 2E0, and 0 rad. Rank the four pairs according to the intensity of the light at the arrival point, greatest first. (Hint: Draw phasors.)

35-4 I NTE R FE R E NCE FROM TH I N FI LM S

1063

Sample Problem 35.04 Combining three light waves by using phasors Three light waves combine at a certain point where their electric field components are E1 ! E0 sin vt,

ER ! 2(2.37E0)2 ' (0.366E0)2 ! 2.4E0,

and a phase angle b relative to the phasor representing E1 of

E2 ! E0 sin(vt ' 60"), E3 ! E0 sin(vt & 30"). Find their resultant component E(t) at that point.

b ! tan&1

! 8.8". # 0.366E 2.37E $ 0

0

We can now write, for the resultant wave E(t), E ! ER sin(vt ' b) ! 2.4E0 sin(vt ' 8.8").

KEY IDEA The resultant wave is E(t) ! E1(t) ' E2(t) ' E3(t). We can use the method of phasors to find this sum, and we are free to evaluate the phasors at any time t.

Be careful to interpret the angle b correctly in Fig. 35-14: It is the constant angle between ER and the phasor representing E1 as the four phasors rotate as a single unit around the origin. The angle between ER and the horizontal axis in Fig. 35-14 does not remain equal to b.

Calculations: To simplify the solution, we choose t ! 0, for which the phasors representing the three waves are shown in Fig. 35-14. We can add these three phasors either directly on a vector-capable calculator or by components. For the component approach, we first write the sum of their horizontal components as

Phasors that represent waves can be added to find the net wave.

E0

' Eh ! E0 cos 0 ' E0 cos 60° ' E0 cos(&30°) ! 2.37E0. The sum of their vertical components, which is the value of E at t ! 0, is

' Ev ! E0 sin 0 ' E0 sin 60° ' E0 sin(&30°) ! 0.366E0. The resultant wave E(t) thus has an amplitude ER of

(Answer)

E

β

E0

60°

30°

E0

ER

Figure 35-14 Three phasors, representing waves with equal amplitudes E0 and with phase constants 0°, 60°, and &30°, shown at time t ! 0. The phasors combine to give a resultant phasor with magnitude ER, at angle b.

Additional examples, video, and practice available at WileyPLUS

35-4 INTERFERENCE FROM THIN FILMS Learning Objectives After reading this module, you should be able to . . .

35.29 Sketch the setup for thin-film interference, showing the incident ray and reflected rays (perpendicular to the film but drawn slightly slanted for clarity) and identifying the thickness and the three indexes of refraction. 35.30 Identify the condition in which a reflection can result in a phase shift, and give the value of that phase shift. 35.31 Identify the three factors that determine the interference of the reflected waves: reflection shifts, path length difference, and internal wavelength (set by the film's index of refraction). 35.32 For a thin film, use the reflection shifts and the desired result (the reflected waves are in phase or out of phase, or

the transmitted waves are in phase or out of phase) to determine and then apply the necessary equation relating the thickness L, the wavelength l (measured in air), and the index of refraction n of the film. 35.33 For a very thin film in air (with thickness much less than the wavelength of visible light), explain why the film is always dark. 35.34 At each end of a thin film in the form of a wedge, determine and then apply the necessary equation relating the thickness L, the wavelength l (measured in air), and the index of refraction n of the film, and then count the number of bright bands and dark bands across the film.

1064

CHAPTE R 35 I NTE R FE R E NCE

Key Ideas ● When light is incident on a thin transparent film, the light waves reflected from the front and back surfaces interfere. For near-normal incidence, the wavelength conditions for maximum and minimum intensity of the light reflected from a film in air are

2L ! (m '

1 2)

l , n2

for m ! 0, 1, 2, . . .

(maxima — bright film in air),

2L ! m

l , n2

for m ! 0, 1, 2, . . .

(minima — dark film in air),

n2

n3

r2 c

θ θ i

● If a film is sandwiched between media other than air, these

equations for bright and dark films may be interchanged, depending on the relative indexes of refraction. ● If the light incident at an interface between media with different indexes of refraction is in the medium with the smaller index of refraction, the reflection causes a phase change of p rad, or half a wavelength, in the reflected wave. Otherwise, there is no phase change due to the reflection. Refraction causes no phase shift.

Interference from Thin Films

The interference depends on the reflections and the n1 path lengths.

r1

where n2 is the index of refraction of the film, L is its thickness, and l is the wavelength of the light in air.

b

a L

Figure 35-15 Light waves, represented with ray i, are incident on a thin film of thickness L and index of refraction n2. Rays r1 and r2 represent light waves that have been reflected by the front and back surfaces of the film, respectively. (All three rays are actually nearly perpendicular to the film.) The interference of the waves of r1 and r2 with each other depends on their phase difference. The index of refraction n1 of the medium at the left can differ from the index of refraction n3 of the medium at the right, but for now we assume that both media are air, with n1 ! n3 ! 1.0, which is less than n2.

The colors on a sunlit soap bubble or an oil slick are caused by the interference of light waves reflected from the front and back surfaces of a thin transparent film. The thickness of the soap or oil film is typically of the order of magnitude of the wavelength of the (visible) light involved. (Greater thicknesses spoil the coherence of the light needed to produce the colors due to interference.) Figure 35-15 shows a thin transparent film of uniform thickness L and index of refraction n2, illuminated by bright light of wavelength l from a distant point source. For now, we assume that air lies on both sides of the film and thus that n1 ! n3 in Fig. 35-15. For simplicity, we also assume that the light rays are almost perpendicular to the film (u % 0). We are interested in whether the film is bright or dark to an observer viewing it almost perpendicularly. (Since the film is brightly illuminated, how could it possibly be dark? You will see.) The incident light, represented by ray i, intercepts the front (left) surface of the film at point a and undergoes both reflection and refraction there. The reflected ray r1 is intercepted by the observer’s eye. The refracted light crosses the film to point b on the back surface, where it undergoes both reflection and refraction. The light reflected at b crosses back through the film to point c, where it undergoes both reflection and refraction. The light refracted at c, represented by ray r2, is intercepted by the observer’s eye. If the light waves of rays r1 and r2 are exactly in phase at the eye, they produce an interference maximum and region ac on the film is bright to the observer. If they are exactly out of phase, they produce an interference minimum and region ac is dark to the observer, even though it is illuminated. If there is some intermediate phase difference, there are intermediate interference and brightness. The Key. Thus, the key to what the observer sees is the phase difference between the waves of rays r1 and r2. Both rays are derived from the same ray i, but the path involved in producing r2 involves light traveling twice across the film (a to b, and then b to c), whereas the path involved in producing r1 involves no travel through the film. Because u is about zero, we approximate the path length difference between the waves of r1 and r2 as 2L. However, to find the phase difference between the waves, we cannot just find the number of wavelengths l that is equivalent to a path length difference of 2L. This simple approach is impossible for two reasons: (1) the path length difference occurs in a medium other than air, and (2) reflections are involved, which can change the phase. The phase difference between two waves can change if one or both are reflected.

Let’s next discuss changes in phase that are caused by reflections.

35-4 I NTE R FE R E NCE FROM TH I N FI LM S

Reflection Phase Shifts Refraction at an interface never causes a phase change — but reflection can, depending on the indexes of refraction on the two sides of the interface. Figure 35-16 shows what happens when reflection causes a phase change, using as an example pulses on a denser string (along which pulse travel is relatively slow) and a lighter string (along which pulse travel is relatively fast). When a pulse traveling relatively slowly along the denser string in Fig. 35-16a reaches the interface with the lighter string, the pulse is partially transmitted and partially reflected, with no change in orientation. For light, this situation corresponds to the incident wave traveling in the medium of greater index of refraction n (recall that greater n means slower speed). In that case, the wave that is reflected at the interface does not undergo a change in phase; that is, its reflection phase shift is zero. When a pulse traveling more quickly along the lighter string in Fig. 35-16b reaches the interface with the denser string, the pulse is again partially transmitted and partially reflected. The transmitted pulse again has the same orientation as the incident pulse, but now the reflected pulse is inverted. For a sinusoidal wave, such an inversion involves a phase change of p rad, or half a wavelength. For light, this situation corresponds to the incident wave traveling in the medium of lesser index of refraction (with greater speed). In that case, the wave that is reflected at the interface undergoes a phase shift of p rad, or half a wavelength. We can summarize these results for light in terms of the index of refraction of the medium off which (or from which) the light reflects:

Reflection

Reflection phase shift

Off lower index Off higher index

0 0.5 wavelength

1065

Interface

Before After (a)

Before After (b)

Figure 35-16 Phase changes when a pulse is reflected at the interface between two stretched strings of different linear densities. The wave speed is greater in the lighter string. (a) The incident pulse is in the denser string. (b) The incident pulse is in the lighter string. Only here is there a phase change, and only in the reflected wave.

This might be remembered as “higher means half.”

Equations for Thin-Film Interference In this chapter we have now seen three ways in which the phase difference between two waves can change: 1. by reflection 2. by the waves traveling along paths of different lengths 3. by the waves traveling through media of different indexes of refraction When light reflects from a thin film, producing the waves of rays r1 and r2 shown in Fig. 35-15, all three ways are involved. Let us consider them one by one. Reflection Shift. We first reexamine the two reflections in Fig. 35-15. At point a on the front interface, the incident wave (in air) reflects from the medium having the higher of the two indexes of refraction; so the wave of reflected ray r1 has its phase shifted by 0.5 wavelength. At point b on the back interface, the incident wave reflects from the medium (air) having the lower of the two indexes of refraction; so the wave reflected there is not shifted in phase by the reflection, and thus neither is the portion of it that exits the film as ray r2. We can organize this information with the first line in Table 35-1, which refers to the simplified drawing in Fig. 35-17 for a thin film in air. So far, as a result of the reflection phase shifts, the waves of r1 and r2 have a phase difference of 0.5 wavelength and thus are exactly out of phase. Path Length Difference. Now we must consider the path length difference 2L that occurs because the wave of ray r2 crosses the film twice. (This difference

Air

n2

Air

r1 r2 i L

Figure 35-17 Reflections from a thin film in air.

1066

CHAPTE R 35 I NTE R FE R E NCE

Table 35-1 An Organizing Table for

Thin-Film Interference in Air (Fig. 35-17)a Reflection phase shifts Path length difference Index in which path length difference occurs a

In phase : Out of phasea:

r1

r2

0.5 wavelength

0

2L is shown on the second line in Table 35-1.) If the waves of r1 and r2 are to be exactly in phase so that they produce fully constructive interference, the path length 2L must cause an additional phase difference of 0.5, 1.5, 2.5, . . . wavelengths. Only then will the net phase difference be an integer number of wavelengths. Thus, for a bright film, we must have 2L !

2L

2L !

odd number l 2L ! ( 2 n2

(35-31)

l n2

odd number ( ln2 2

(in-phase waves).

(35-32)

If, instead, the waves are to be exactly out of phase so that there is fully destructive interference, the path length 2L must cause either no additional phase difference or a phase difference of 1, 2, 3, . . . wavelengths. Only then will the net phase difference be an odd number of half-wavelengths. For a dark film, we must have 2L ! integer ( wavelength

a

Valid for n2 $ n1 and n2 $ n3.

(in-phase waves).

The wavelength we need here is the wavelength ln2 of the light in the medium containing path length 2L — that is, in the medium with index of refraction n2. Thus, we can rewrite Eq. 35-31 as

n2

2L ! integer (

odd number ( wavelength 2

(out-of-phase waves).

(35-33)

where, again, the wavelength is the wavelength ln2 in the medium containing 2L. Thus, this time we have 2L ! integer ( ln2

(out-of-phase waves).

(35-34)

Now we can use Eq. 35-6 (ln ! l/n) to write the wavelength of the wave of ray r2 inside the film as ln2 !

l , n2

(35-35)

where l is the wavelength of the incident light in vacuum (and approximately also in air). Substituting Eq. 35-35 into Eq. 35-32 and replacing “odd number/2” with (m ' 12) give us 2L ! (m ' 12)

l , n2

for m ! 0, 1, 2, . . .

(maxima — bright film in air).

(35-36)

Similarly, with m replacing “integer,” Eq. 35-34 yields 2L ! m

l , n2

for m ! 0, 1, 2, . . .

(minima — dark film in air).

(35-37)

For a given film thickness L, Eqs. 35-36 and 35-37 tell us the wavelengths of light for which the film appears bright and dark, respectively, one wavelength for each value of m. Intermediate wavelengths give intermediate brightnesses. For a given wavelength l, Eqs. 35-36 and 35-37 tell us the thicknesses of the films that appear bright and dark in that light, respectively, one thickness for each value of m. Intermediate thicknesses give intermediate brightnesses. Heads Up. (1) For a thin film surrounded by air, Eq. 35-36 corresponds to bright reflections and Eq. 35-37 corresponds to no reflections. For transmissions, the roles of the equations are reversed (after all, if the light is brightly reflected, then it is not transmitted, and vice versa). (2) If we have a different set of values of the indexes of refraction, the roles of the equations may be reversed. For any given set of indexes, you must go through the thought process behind Table 35-1 and, in particular, determine the reflection shifts to see which equation applies to bright reflections and which applies to no reflections. (3) The index of refraction in the equations is that of the thin film, where the path length difference occurs.

1067

35-4 I NTE R FE R E NCE FROM TH I N FI LM S

Film Thickness Much Less Than l A special situation arises when a film is so thin that L is much less than l, say, L % 0.1l. Then the path length difference 2L can be neglected, and the phase difference between r1 and r2 is due only to reflection phase shifts. If the film of Fig. 35-17, where the reflections cause a phase difference of 0.5 wavelength, has thickness L % 0.1l, then r1 and r2 are exactly out of phase, and thus the film is dark, regardless of the wavelength and intensity of the light. This special situation corresponds to m ! 0 in Eq. 35-37. We shall count any thickness L % 0.1l as being the least thickness specified by Eq. 35-37 to make the film of Fig. 35-17 dark. (Every such thickness will correspond to m ! 0.) The next greater thickness that will make the film dark is that corresponding to m ! 1. In Fig. 35-18, bright white light illuminates a vertical soap film whose thickness increases from top to bottom. However, the top portion is so thin that it is dark. In the (somewhat thicker) middle we see fringes, or bands, whose color depends primarily on the wavelength at which reflected light undergoes fully constructive interference for a particular thickness. Toward the (thickest) bottom the fringes become progressively narrower and the colors begin to overlap and fade.

Richard Megna/Fundamental Photographs

Figure 35-18 The reflection of light from a soapy water film spanning a vertical loop. The top portion is so thin (due to gravitational slumping) that the light reflected there undergoes destructive interference, making that portion dark. Colored interference fringes, or bands, decorate the rest of the film but are marred by circulation of liquid within the film as the liquid is gradually pulled downward by gravitation.

Checkpoint 5 1.5 1.5 1.4 1.3 The figure shows four situations in which L 1.4 1.3 1.3 1.4 L light reflects perpen1.3 1.4 1.5 1.5 dicularly from a thin (1) (2) (3) (4) film of thickness L, with indexes of refraction as given. (a) For which situations does reflection at the film interfaces cause a zero phase difference for the two reflected rays? (b) For which situations will the film be dark if the path length difference 2L causes a phase difference of 0.5 wavelength?

Sample Problem 35.05 Thin-film interference of a water film in air White light, with a uniform intensity across the visible wavelength range of 400 to 690 nm, is perpendicularly incident on a water film, of index of refraction n2 ! 1.33 and thickness L ! 320 nm, that is suspended in air. At what wavelength l is the light reflected by the film brightest to an observer? KEY IDEA The reflected light from the film is brightest at the wavelengths l for which the reflected rays are in phase with one another. The equation relating these wavelengths l to the given film thickness L and film index of refraction n2 is either Eq. 35-36 or Eq. 35-37, depending on the reflection phase shifts for this particular film. Calculations: To determine which equation is needed, we should fill out an organizing table like Table 35-1. However, because there is air on both sides of the water film, the situation here is exactly like that in Fig. 35-17, and thus the table would be exactly like Table 35-1. Then from Table 35-1, we

see that the reflected rays are in phase (and thus the film is brightest) when 2L !

odd number l , ( 2 n2

which leads to Eq. 35-36: 2L ! (m ' 12 )

l . n2

Solving for l and substituting for L and n2, we find l!

2n2L (2)(1.33)(320 nm) 851 nm ! . 1 ! 1 m'2 m'2 m ' 12

For m ! 0, this gives us l ! 1700 nm, which is in the infrared region. For m ! 1, we find l ! 567 nm, which is yellow-green light, near the middle of the visible spectrum. For m ! 2, l ! 340 nm, which is in the ultraviolet region. Thus, the wavelength at which the light seen by the observer is brightest is

Additional examples, video, and practice available at WileyPLUS

l ! 567 nm.

(Answer)

1068

CHAPTE R 35 I NTE R FE R E NCE

Sample Problem 35.06 Thin-film interference of a coating on a glass lens In Fig. 35-19, a glass lens is coated on one side with a thin film of magnesium fluoride (MgF2) to reduce reflection from the lens surface. The index of refraction of MgF2 is 1.38; that of the glass is 1.50. What is the least coating thickness that eliminates (via interference) the reflections at the middle of the visible spectrum (l ! 550 nm)? Assume that the light is approximately perpendicular to the lens surface. KEY IDEA

Glass Air MgF2 n1 = 1.00 n2 = 1.38 n3 = 1.50 r2 r1

c

θ θ

b

a

i

Reflection is eliminated if the film thickness L is such that light waves reflected from the two film interfaces are exactly out of phase. The equation relating L to the given wavelength l and the index of refraction n2 of the thin film is either Eq. 35-36 or Eq. 35-37, depending on the reflection phase shifts at the interfaces. Calculations: To determine which equation is needed, we fill out an organizing table like Table 35-1. At the first interface, the incident light is in air, which has a lesser index of refraction than the MgF2 (the thin film). Thus, we fill in 0.5 wavelength under r1 in our organizing table (meaning that the waves of ray r1 are shifted by 0.5l at the first interface). At the second interface, the incident light is in the MgF2, which has a lesser index of refraction than the glass on the other side of the interface. Thus, we fill in 0.5 wavelength under r2 in our table. Because both reflections cause the same phase shift, they tend to put the waves of r1 and r2 in phase. Since we want those waves to be out of phase, their path length difference 2L must be an odd number of half-wavelengths: 2L !

odd number l . ( 2 n2

L

Both reflection phase shifts are 0.5 wavelength. So, only the path length difference determines the interference.

Figure 35-19 Unwanted reflections from glass can be suppressed (at a chosen wavelength) by coating the glass with a thin transparent film of magnesium fluoride of the properly chosen thickness.

This leads to Eq. 35-36 (for a bright film sandwiched in air but for a dark film in the arrangement here). Solving that equation for L then gives us the film thicknesses that will eliminate reflection from the lens and coating: L ! (m ' 12 )

l , 2n2

for m ! 0, 1, 2, . . . . (35-38)

We want the least thickness for the coating — that is, the smallest value of L. Thus, we choose m ! 0, the smallest possible value of m. Substituting it and the given data in Eq. 35-38, we obtain L!

l 550 nm ! ! 99.6 nm. 4n2 (4)(1.38)

(Answer)

Sample Problem 35.07 Thin-film interference of a transparent wedge Figure 35-20a shows a transparent plastic block with a thin wedge of air at the right. (The wedge thickness is exaggerated in the figure.) A broad beam of red light, with wavelength l ! 632.8 nm, is directed downward through the top of the block (at an incidence angle of 0°). Some of the light that passes into the plastic is reflected back up from the top and bottom surfaces of the wedge, which acts as a thin film (of air) with a thickness that varies uniformly and gradually from LL at the left-hand end to LR at the righthand end. (The plastic layers above and below the wedge of air are too thick to act as thin films.) An observer looking down on the block sees an interference pattern consisting of six dark fringes and five bright red fringes along the wedge. What is the change in thickness 2L (! LR & LL) along the wedge?

KEY IDEAS (1) The brightness at any point along the left–right length of the air wedge is due to the interference of the waves reflected at the top and bottom interfaces of the wedge. (2) The variation of brightness in the pattern of bright and dark fringes is due to the variation in the thickness of the wedge. In some regions, the thickness puts the reflected waves in phase and thus produces a bright reflection (a bright red fringe). In other regions, the thickness puts the reflected waves out of phase and thus produces no reflection (a dark fringe). Organizing the reflections: Because the observer sees more dark fringes than bright fringes, we can assume that a dark fringe is produced at both the left and right ends of

35-4 I NTE R FE R E NCE FROM TH I N FI LM S

1069

A r1

Overhead incident light i

Side view

LL

L

LR

(a)

(c)

r2 n1 n2 n3

mL + 2 mL + 4 Overhead view (b)

This dark fringe is due to fully destructive interference. So, the reflected rays must be out of phase. r1 Reflection shifts: 0 0.5 λ

i

r2

mL mL +1 mL + 3 mL + 5

Here too, the dark fringe means that the reflected waves are out of phase. r1

n1 plastic (higher index)

i

The path length LL n2 air (low index) difference (down n3 plastic and back up) is 2L.

LR

(d)

Total reflection shift = 0.5 wavelength. So, the reflections put the waves out of phase.

r2

The path length difference is 2L here too but the L is larger.

(f ) (e) Organizing Table

Reflection phase shifts Path length difference

r1

r2

0

0.5 wavelength

2L

Here again, the waves are already out of phase by the reflection shifts. So, the path length difference must be 2L = (integer)l/n2, but with the larger L.

We want the reflected waves to be out of phase. They already are out of phase because of the reflection shifts. So, we don’t want the path length difference 2L to change that. Thus, 2L = (integer)l/n2.

Figure 35-20 (a) Red light is incident on a thin, air-filled wedge in the side of a transparent plastic block. The thickness of the wedge is LL at the left end and LR at the right end. (b) The view from above the block: an interference pattern of six dark fringes and five bright red fringes lies over the region of the wedge. (c) A representation of the incident ray i, reflected rays r1 and r2, and thickness L of the wedge anywhere along the length of the wedge. The reflection rays at the (d) left and (f ) right ends of the wedge and (e) their organizing table.

1070

CHAPTE R 35 I NTE R FE R E NCE

the wedge. Thus, the interference pattern is that shown in Fig. 35-20b. We can represent the reflection of light at the top and bottom interfaces of the wedge, at any point along its length, with Fig. 35-20c, in which L is the wedge thickness at that point. Let us apply this figure to the left end of the wedge, where the reflections give a dark fringe. We know that, for a dark fringe, the waves of rays r1 and r2 in Fig. 35-20d must be out of phase. We also know that the equation relating the film thickness L to the light’s wavelength l and the film’s index of refraction n2 is either Eq. 35-36 or Eq. 35-37, depending on the reflection phase shifts. To determine which equation gives a dark fringe at the left end of the wedge, we should fill out an organizing table like Table 35-1, as shown in Fig. 35-20e. At the top interface of the wedge, the incident light is in the plastic, which has a greater n than the air beneath that interface. So, we fill in 0 under r1 in our organizing table. At the bottom interface of the wedge, the incident light is in air, which has a lesser n than the plastic beneath that interface. So we fill in 0.5 wavelength under r2. So, the phase difference due to the reflection shifts is 0.5 wavelength. Thus the reflections alone tend to put the waves of r1 and r2 out of phase. Reflections at left end (Fig. 35-20d): Because we see a dark fringe at the left end of the wedge, which the reflection phase shifts alone would produce, we don’t want the path length difference to alter that condition. So, the path length difference 2L at the left end must be given by 2L ! integer (

which leads to Eq. 35-37: 2L ! m

l , n2

for m ! 0, 1, 2, . . . .

(35-39)

Reflections at right end (Fig. 35-20f): Equation 35-39 holds not only for the left end of the wedge but also for any point along the wedge where a dark fringe is observed, including the right end, with a different integer value of m for each fringe. The least value of m is associated with the least thickness of the wedge where a dark fringe is observed. Progressively greater values of m are associated with progressively greater thicknesses of the wedge where a dark fringe is observed. Let mL be the value at the left end. Then the value at the right end must be mL ' 5 because, from Fig. 35-20b, the right end is located at the fifth dark fringe from the left end. Thickness difference: To find 2L, we first solve Eq. 35-39 twice — once for the thickness LL at the left end and once for the thickness LR at the right end: LL ! (mL)

l , 2n2

LR ! (mL ' 5)

l . 2n2

(35-40)

We can now subtract LL from LR and substitute n2 ! 1.00 for the air within the wedge and l = 632.8 ( 10&9 m: 2L ! LR & LL !

(mL ' 5)l m Ll 5 l & ! 2n2 2n2 2 n2

! 1.58 ( 10&6 m.

l , n2

(Answer)

Additional examples, video, and practice available at WileyPLUS

35-5 MICHELSON’S INTERFEROMETER Learning Objectives After reading this module, you should be able to . . .

35.35 With a sketch, explain how an interferometer works. 35.36 When a transparent material is inserted into one of the beams in an interferometer, apply the relationship between the phase change of the light (in terms of

wavelength) and the material’s thickness and index of refraction. 35.37 For an interferometer, apply the relationship between the distance a mirror is moved and the resulting fringe shift in the interference pattern.

Key Ideas ● In Michelson’s interferometer, a light wave is split into two beams that then recombine after traveling along different paths. ● The interference pattern they produce depends on the difference in the lengths of those paths and the indexes of refraction along the paths.

If a transparent material of index n and thickness L is in one path, the phase difference (in terms of wavelength) in the recombining beams is equal to 2L phase difference ! (n & 1), l where l is the wavelength of the light. ●

35-5 M ICH E LSON’S I NTE R FE ROM ETE R

Michelson’s Interferometer

Movable mirror

An interferometer is a device that can be used to measure lengths or changes in length with great accuracy by means of interference fringes. We describe the form originally devised and built by A. A. Michelson in 1881. Consider light that leaves point P on extended source S in Fig. 35-21 and encounters beam splitter M. A beam splitter is a mirror that transmits half the incident light and reflects the other half. In the figure we have assumed, for convenience, that this mirror possesses negligible thickness. At M the light thus divides into two waves. One proceeds by transmission toward mirror M1 at the end of one arm of the instrument; the other proceeds by reflection toward mirror M2 at the end of the other arm. The waves are entirely reflected at these mirrors and are sent back along their directions of incidence, each wave eventually entering telescope T. What the observer sees is a pattern of curved or approximately straight interference fringes; in the latter case the fringes resemble the stripes on a zebra. Mirror Shift. The path length difference for the two waves when they recombine at the telescope is 2d2 & 2d1, and anything that changes this path length difference will cause a change in the phase difference between these two waves at the eye. As an example, if mirror M2 is moved by a distance 12l, the path length difference is changed by l and the fringe pattern is shifted by one fringe (as if each dark stripe on a zebra had moved to where the adjacent dark stripe had been). Similarly, moving mirror M2 by 14l causes a shift by half a fringe (each dark zebra stripe shifts to where the adjacent white stripe had been). Insertion. A shift in the fringe pattern can also be caused by the insertion of a thin transparent material into the optical path of one of the mirrors — say, M1. If the material has thickness L and index of refraction n, then the number of wavelengths along the light’s to-and-fro path through the material is, from Eq. 35-7, Nm !

2L 2Ln ! . ln l

(35-41)

The number of wavelengths in the same thickness 2L of air before the insertion of the material is 2L (35-42) . Na ! l When the material is inserted, the light returned from mirror M1 undergoes a phase change (in terms of wavelengths) of Nm & Na !

2Ln 2L 2L & ! (n & 1). l l l

(35-43)

For each phase change of one wavelength, the fringe pattern is shifted by one fringe. Thus, by counting the number of fringes through which the material causes the pattern to shift, and substituting that number for Nm & Na in Eq. 35-43, you can determine the thickness L of the material in terms of l. Standard of Length. By such techniques the lengths of objects can be expressed in terms of the wavelengths of light. In Michelson’s day, the standard of length — the meter — was the distance between two fine scratches on a certain metal bar preserved at Sèvres, near Paris. Michelson showed, using his interferometer, that the standard meter was equivalent to 1 553 163.5 wavelengths of a certain monochromatic red light emitted from a light source containing cadmium. For this careful measurement, Michelson received the 1907 Nobel Prize in physics. His work laid the foundation for the eventual abandonment (in 1961) of the meter bar as a standard of length and for the redefinition of the meter in terms of the wavelength of light. By 1983, even this wavelength standard was not precise enough to meet the growing technical needs, and it was replaced with a new standard based on a defined value for the speed of light.

d2 P

1071

M2

Arm 2 M

S

d1 Arm 1

T

M1

The interference at the eye depends on the path length difference and the index of any inserted material.

Figure 35-21 Michelson’s interferometer, showing the path of light originating at point P of an extended source S. Mirror M splits the light into two beams, which reflect from mirrors M1 and M2 back to M and then to telescope T. In the telescope an observer sees a pattern of interference fringes.

1072

CHAPTE R 35 I NTE R FE R E NCE

Review & Summary Huygens’ Principle The three-dimensional transmission of waves, including light, may often be predicted by Huygens’ principle, which states that all points on a wavefront serve as point sources of spherical secondary wavelets. After a time t, the new position of the wavefront will be that of a surface tangent to these secondary wavelets. The law of refraction can be derived from Huygens’ principle by assuming that the index of refraction of any medium is n ! c/v, in which v is the speed of light in the medium and c is the speed of light in vacuum.

Wavelength and Index of Refraction The wavelength ln

of light in a medium depends on the index of refraction n of the medium: l ln ! , n

(35-6)

in which l is the wavelength in vacuum. Because of this dependency, the phase difference between two waves can change if they pass through different materials with different indexes of refraction.

Young’s Experiment In Young’s interference experiment, light passing through a single slit falls on two slits in a screen. The light leaving these slits flares out (by diffraction), and interference occurs in the region beyond the screen. A fringe pattern, due to the interference, forms on a viewing screen. The light intensity at any point on the viewing screen depends in part on the difference in the path lengths from the slits to that point. If this difference is an integer number of wavelengths, the waves interfere constructively and an intensity maximum results. If it is an odd number of half-wavelengths, there is destructive interference and an intensity minimum occurs. The conditions for maximum and minimum intensity are d sin u ! ml,

for m ! 0, 1, 2, . . .

(maxima — bright fringes),

d sin u ! (m '

1 2 )l,

(35-14)

for m ! 0, 1, 2, . . .

(minima — dark fringes),

(35-16)

where u is the angle the light path makes with a central axis and d is the slit separation.

Coherence If two light waves that meet at a point are to interfere perceptibly, the phase difference between them must remain constant with time; that is, the waves must be coherent. When two coherent waves meet, the resulting intensity may be found by using phasors. Intensity in Two-Slit Interference In Young’s interference experiment, two waves, each with intensity I 0, yield a resultant wave of intensity I at the viewing screen, with I ! 4I0 cos2 12 f,

where f !

2pd sin u. l

Equations 35-14 and 35-16, which identify the positions of the fringe maxima and minima, are contained within this relation.

Thin-Film Interference When light is incident on a thin transparent film, the light waves reflected from the front and back surfaces interfere. For near-normal incidence, the wavelength conditions for maximum and minimum intensity of the light reflected from a film in air are l , for m ! 0, 1, 2, . . . 2L ! (m ' 12 ) n2 (maxima — bright film in air), (35-36) l 2L ! m , for m ! 0, 1, 2, . . . n2 (minima — dark film in air), (35-37) where n2 is the index of refraction of the film, L is its thickness, and l is the wavelength of the light in air. If the light incident at an interface between media with different indexes of refraction is in the medium with the smaller index of refraction, the reflection causes a phase change of p rad, or half a wavelength, in the reflected wave. Otherwise, there is no phase change due to the reflection. Refraction causes no phase shift.

The Michelson Interferometer In Michelson’s interferometer a light wave is split into two beams that, after traversing paths of different lengths, are recombined so they interfere and form a fringe pattern. Varying the path length of one of the beams allows distances to be accurately expressed in terms of wavelengths of light, by counting the number of fringes through which the pattern shifts because of the change.

Questions 1 Does the spacing between fringes in a two-slit interference pattern increase, decrease, or stay the same if (a) the slit separation is increased, (b) the color of the light is switched from red to blue, and (c) the whole apparatus is submerged in cooking sherry? (d) If the slits are illuminated with white light, then at any side maximum, does the blue component or the red component peak closer to the central maximum? 2 (a) If you move from one bright fringe in a two-slit interference pattern to the next one farther out, (b) does the path length difference 2L increase or decrease and (c) by how much does it change, in wavelengths l? 3 Figure 35-22 shows two light rays that are initially exactly in phase and that reflect from several glass surfaces. Neglect the

(35-22, 35-23)

d

d

slight slant in the path of the light in the second arrangement. (a) What is d the path length difference of the rays? In wavelengths l, (b) what should that path length difference equal if the rays are to be exactly out of phase when they emerge, Figure 35-22 Question 3. and (c) what is the smallest value of d that will allow that final phase 1.60 a difference? 4 In Fig. 35-23, three pulses of light — a, b, and c — of the same wavelength are sent through layers

b

1.50

c

1.55

Figure 35-23 Question 4.

1073

QU ESTIONS

of plastic having the given indexes of refraction and along the paths indicated. Rank the pulses according to their travel time through the plastic layers, greatest first. 5 Is there an interference maximum, a minimum, an intermediate state closer to a maximum, or an intermediate state closer to a minimum at point P in Fig. 35-10 if the path length difference of the two rays is (a) 2.2l, (b) 3.5l, (c) 1.8l, and (d) 1.0l? For each situation, give the value of m associated with the maximum or minimum involved. 6 Figure 35-24a gives intensity I versus position x on the viewing screen for the central portion of a two-slit interference pattern. The other parts of the figure give phasor diagrams for the electric field components of the waves arriving at the screen from the two slits (as in Fig. 35-13a). Which numbered points on the screen best correspond to which phasor diagram? I Central fringe

1 2

3

4 (a)

11 Figure 35-28 shows four situations in which light reflects perpendicularly from a thin film of thickness L sandwiched between much thicker materials. The indexes of refraction are given. In which situations does Eq. 35-36 correspond to the reflections yielding maxima (that is, a bright film)?

L

x

5

10 Figure 35-27a shows the cross a section of a vertical thin film whose width increases downward because b gravitation causes slumping. Figure c 35-27b is a face-on view of the film, d showing four bright (red) interfer(b) ence fringes that result when the (a) film is illuminated with a perpendiFigure 35-27 Question 10. cular beam of red light. Points in the cross section corresponding to the bright fringes are labeled. In terms of the wavelength of the light inside the film, what is the difference in film thickness between (a) points a and b and (b) points b and d?

(b)

(d)

(c)

(a)

1.4

1.8

1.5

1.4

1.6

1.6

1.3

1.6

1.8

1.4

1.4

1.5

(b)

(c)

L

(d)

Figure 35-24 Question 6.

Figure 35-28 Question 11.

7 Figure 35-25 shows two sources S1 and S2 that emit radio waves of wavelength l in all directions. The sources are exactly in phase and are separated by a distance equal to 1.5l. The vertical broken line is the perpendicular bisector of the distance between the sources. (a) If we start at the indicated start point and travel along path 1, does the interference produce a maximum all along the path, a minimum all along the path, or alternating maxima and minima? Repeat for (b) path 2 (along an axis through the sources) and (c) path 3 (along a perpendicular to that axis).

12 Figure 35-29 shows the transmission of light through a thin film in air by a perpendicular beam (tilted in the figure for clarity). (a) Did ray r3 undergo a phase shift due to reflection? (b) In wavelengths, what is the reflection phase shift for ray r4? (c) If the film thickness is L, what is the path length difference between rays r3 and r4?

r4

Incident light

r3

Figure 35-29 Question 12. 1 S1 Start

3 S2 Start

2

Figure 35-25 Question 7. 8 Figure 35-26 shows two rays of light, of wavelength 600 nm, that reflect from glass surfaces separated by 150 nm. The rays are initially in phase. (a) What is the path length 150 nm difference of the rays? (b) When they have cleared the reflection reFigure 35-26 Question 8. gion, are the rays exactly in phase, exactly out of phase, or in some intermediate state? 9 Light travels along the length of a 1500-nm-long nanostructure. When a peak of the wave is at one end of the nanostructure, is there a peak or a valley at the other end if the wavelength is (a) 500 nm and (b) 1000 nm?

13 Figure 35-30 shows three situations in which two rays of sunlight penetrate slightly into and then scatter out of lunar soil. Assume that the rays are initially in phase. In which situation are the associated waves most likely to end up in phase? (Just as the Moon becomes full, its brightness suddenly peaks, becoming 25% greater than its brightness on the nights before and after, because at full Moon we intercept light waves that are scattered by lunar soil back toward the Sun and undergo constructive interference at our eyes. Before astronauts first landed on the Moon, NASA was concerned that backscatter of sunlight from the soil might blind the lunar astronauts if they did not have proper viewing shields on their helmets.)

(a)

(b)

Figure 35-30 Question 13.

(c)

1074

CHAPTE R 35 I NTE R FE R E NCE

Problems Tutoring problem available (at instructor’s discretion) in WileyPLUS and WebAssign Worked-out solution available in Student Solutions Manual

SSM • – •••

WWW Worked-out solution is at

Number of dots indicates level of problem difficulty

ILW

Interactive solution is at

http://www.wiley.com/college/halliday

Additional information available in The Flying Circus of Physics and at flyingcircusofphysics.com

Module 35-1 Light as a Wave •1 In Fig. 35-31, a light wave along L r2 ray r1 reflects once from a mirror and a light wave along ray r2 reflects twice r1 from that same mirror and once from a tiny mirror at distance L from the bigger mirror. (Neglect the slight tilt Figure 35-31 Problems 1 and 2. of the rays.) The waves have wavelength 620 nm and are initially in phase. (a) What is the smallest value of L that puts the final light waves exactly out of phase? (b) With the tiny mirror initially at that value of L, how far must it be moved away from the bigger mirror to again put the final waves out of phase? •2 In Fig. 35-31, a light wave along ray r1 reflects once from a mirror and a light wave along ray r2 reflects twice from that same mirror and once from a tiny mirror at distance L from the bigger mirror. (Neglect the slight tilt of the rays.) The waves have wavelength l and are initially exactly out of phase. What are the (a) smallest, (b) second smallest, and (c) third smallest values of L/l that result in the final waves being exactly in phase? •3 SSM In Fig. 35-4, assume that two waves of light in air, of wavelength 400 nm, are initially in phase. One travels through a glass layer of index of refraction n1 ! 1.60 and thickness L. The other travels through an equally thick plastic layer of index of refraction n2 ! 1.50. (a) What is the smallest value L should have if the waves are to end up with a phase difference of 5.65 rad? (b) If the waves arrive at some common point with the same amplitude, is their interference fully constructive, fully destructive, intermediate but closer to fully constructive, or intermediate but closer to fully destructive? •4 In Fig. 35-32a, a beam of light in material 1 is incident on a boundary at an angle of 30°. The extent to which the light is bent due to refraction depends, in part, on the index of refraction n2 of material 2. Figure 35-32b gives the angle of refraction u2 versus n2 for a range of possible n2 values, from na ! 1.30 to nb ! 1.90. What is the speed of light in material 1? θ2 40°

30°

n1 30°

n2

θ2 20° na (a)

nb

n2

(b)

Figure 35-32 Problem 4. •5 How much faster, in meters per second, does light travel in sapphire than in diamond? See Table 33-1. •6 The wavelength of yellow sodium light in air is 589 nm. (a) What is its frequency? (b) What is its wavelength in glass whose index of refraction is 1.52? (c) From the results of (a) and (b), find its speed in this glass.

•7 The speed of yellow light (from a sodium lamp) in a certain liquid is measured to be 1.92 ( 108 m/s. What is the index of refraction of this liquid for the light? •8 In Fig. 35-33, two light pulses are sent through layers of plastic with thicknesses of either L or 2L as shown and indexes of refraction n1 ! 1.55, n2 ! 1.70, n3 ! 1.60, n4 ! 1.45, n5 ! 1.59, n6 ! 1.65, and n7 ! 1.50. (a) Which pulse travels through the plastic in less time? (b) What multiple of L/c gives the times of the pulses?

L

L

L

L

Pulse n 1 2

n2

n3

n4

n6

n7

Pulse 1

n5

Figure 35-33 Problem 8. difference in the traversal

••9 In Fig. 35-4, assume that the two light waves, of wavelength 620 nm in air, are initially out of phase by p rad. The indexes of refraction of the media are n1 ! 1.45 and n2 ! 1.65. What are the (a) smallest and (b) second smallest value of L that will put the waves exactly in phase once they pass through Air n1 n3 Air the two media? ••10 In Fig. 35-34, a light ray is incident at angle u1 ! 50° on a series of θ1 five transparent layers with parallel boundaries. L1 L3 For layers 1 and 3, L1 ! Figure 35-34 Problem 10. 20 mm, L3 ! 25 mm, n1 ! 1.6, and n3 ! 1.45. (a) At what angle does the light emerge back into air at the right? (b) How much time does the light take to travel through layer 3? ••11 Suppose that the two waves in Fig. 35-4 have wavelength l ! 500 nm in air. What multiple of l gives their phase difference when they emerge if (a) n1 ! 1.50, n2 ! 1.60, and L ! 8.50 mm; (b) n1 ! 1.62, n2 ! 1.72, and L ! 8.50 mm; and (c) n1 ! 1.59, n2 ! 1.79, and L ! 3.25 mm? (d) Suppose that in each of these three situations the waves arrive at a common point (with the same amplitude) after emerging. Rank the situaL tions according to the brightness the waves produce at the common point. L ••12 In Fig. 35-35, two light rays go through different paths by reflecting Ray 1 from the various flat surfaces L shown. The light waves have a wavelength of 420.0 nm and are initially in phase. What are the (a) smallest and (b) second smallest value of distance L that will put the waves ex- Ray 2 actly out of phase as they emerge L from the region? ILW Two waves of light in ••13 air, of wavelength l ! 600.0 nm, are initially in phase. They then

Figure 35-35 Problems 12 and 98.

PROB LE M S

1075

L2

Module 35-2 Young’s Interference Experiment •14 In a double-slit arrangement the slits are separated by a distance equal to 100 times the wavelength of the light passing through the slits. (a) What is the angular separation in radians between the central maximum and an adjacent maximum? (b) What is the distance between these maxima on a screen 50.0 cm from the slits? •15 SSM A double-slit arrangement produces interference fringes for sodium light (l ! 589 nm) that have an angular separation of 3.50 ( 10&3 rad. For what wavelength would the angular separation be 10.0% greater? •16 A double-slit arrangement produces interference fringes for sodium light (l ! 589 nm) that are 0.20° apart. What is the angular separation if the arrangement is immersed in water (n ! 1.33)? SSM In Fig. 35-37, two radio•17 d frequency point sources S1 and S2, sepx arated by distance d ! 2.0 m, are radiS1 S2 ating in phase with l ! 0.50 m. A Figure 35-37 Problems 17 detector moves in a large circular path and 22. around the two sources in a plane containing them. How many maxima does it detect?

•18 In the two-slit experiment of Fig. 35-10, let angle u be 20.0°, the slit separation be 4.24 mm, and the wavelength be l ! 500 nm. (a) What multiple of l gives the phase difference between the waves of rays r1 and r2 when they arrive at point P on the distant screen? (b) What is the phase difference in radians? (c) Determine where in the interference pattern point P lies by giving the maximum or minimum on which it lies, or the maximum and minimum between which it lies. •19 SSM ILW Suppose that Young’s experiment is performed with blue-green light of wavelength 500 nm. The slits are 1.20 mm apart, and the viewing screen is 5.40 m from the slits. How far apart are the bright fringes near the center of the interference pattern? •20 Monochromatic green light, of wavelength 550 nm, illuminates two parallel narrow slits 7.70 mm apart. Calculate the angular deviation (u in Fig. 35-10) of the third-order (m ! 3) bright fringe (a) in radians and (b) in degrees. ••21 In a double-slit experiment, the distance between slits is 5.0 mm and the slits are 1.0 m from the screen. Two interference patterns can be seen on the screen: one due to light of wavelength 480 nm, and the other due to light of wavelength 600 nm. What is the separation on the screen between the third-order (m ! 3) bright fringes of the two interference patterns? ••22 In Fig. 35-37, two isotropic point sources S1 and S2 emit identical light waves in phase at wavelength l.The sources lie at separation d on an x axis, and a light detector is moved in a circle of large radius around the midpoint between them. It detects 30 points of zero intensity, including two on the x axis, one of them to the left of the sources and the other to the right of the sources.What is the value of d/l?

••23 In Fig. 35-38, sources A and B emit long-range radio waves of wavelength 400 m, with the phase of the emission from A ahead of that from source B by 90°. The distance rA from A to detector D is greater than the corresponding distance rB by 100 m. What is the phase difference of the waves at D?

D

rA rB A

B

Figure 35-38 Problem 23. 0

P x

••24 In Fig. 35-39, two isotropic point Screen sources S1 and S2 emit light in phase at D wavelength l and at the same amplitude. The sources are separated by distance 2d ! 6.00l. They lie on an axis S1 S2 that is parallel to an x axis, which runs along a viewing screen at distance D ! d d 20.0l. The origin lies on the perpendicuFigure 35-39 Problem 24. lar bisector between the sources. The figure shows two rays reaching point P on the screen, at position xP. (a) At what value of xP do the rays have the minimum possible phase difference? (b) What multiple of l gives that minimum phase difference? (c) At what value of xP do the rays have the maximum possible phase difference? What multiple of l gives (d) that maximum phase difference and (e) the phase difference when xP ! 6.00l? (f) When xP ! 6.00l, is the resulting intensity at point P maximum, minimum, intermediate but closer to maximum, or intermediate but closer to minimum? y ••25 In Fig. 35-40, two isotropic point sources of light (S1 and S2) are x S1 P separated by distance 2.70 mm along a y axis and emit in phase at wavelength S2 900 nm and at the same amplitude. A light detector is located at point P at Figure 35-40 Problems 25 coordinate xP on the x axis. What is and 28. the greatest value of xP at which the detected light is minimum due to destructive interference?

••26 In a double-slit experiment, the fourth-order maximum for a wavelength of 450 nm occurs at an angle of u ! 90°. (a) What range of wavelengths in the visible range (400 nm to 700 nm) are not present in the third-order maxima? To eliminate all visible light in the fourth-order maximum, (b) should the slit separation be increased or decreased and (c) what least change is needed? •••27 A thin flake of mica (n ! 1.58) is used to cover one slit of a double-slit interference arrangement. The central point on the viewing screen is now occupied by what had been the seventh bright side fringe (m ! 7). If l ! 550 nm, what is the thickness of the mica? •••28 Figure 35-40 6π shows two isotropic point sources of light 4π (S1 and S2) that emit in phase at wavelength 2π 400 nm and at the same amplitude. A detection point P is shown on an 0 xs x axis that extends x (10–7 m) through source S1. The Figure 35-41 Problem 28. phase difference f between the light arriving at point P from the two sources is to be measured as P is moved along the x axis from x ! 0 out to x ! '1.The results out to xs ! 10 ( 10&7 m are given in Fig. 35-41. On the way out to φ (rad)

both travel through a layer of plastic as n2 shown in Fig. 35-36, with L1 ! 4.00 mm, L2 ! 3.50 mm, n1 ! 1.40, and n2 ! 1.60. (a) What multiple of l gives their phase difference after they both have n1 emerged from the layers? (b) If the waves later arrive at some common L1 point with the same amplitude, is their interference fully constructive, fully deFigure 35-36 Problem 13. structive, intermediate but closer to fully constructive, or intermediate but closer to fully destructive?

1076

CHAPTE R 35 I NTE R FE R E NCE

'1, what is the greatest value of x at which the light arriving at P from S1 is exactly out of phase with the light arriving at P from S2? Module 35-3 Interference and Double-Slit Intensity •29 SSM Two waves of the same frequency have amplitudes 1.00 and 2.00. They interfere at a point where their phase difference is 60.0°. What is the resultant amplitude? •30 Find the sum y of the following quantities: y1 ! 10 sin vt and

y2 ! 8.0 sin(vt ' 30°).

••31 ILW Add the quantities y1 ! 10 sin vt, y2 ! 15 sin(vt ' 30"), and y3 ! 5.0 sin(vt & 45") using the phasor method. ••32 In the double-slit experiment of Fig. 35-10, the electric fields of the waves arriving at point P are given by E1 ! (2.00 mV/m) sin[(1.26 ( 1015)t] E2 ! (2.00 mV/m) sin[(1.26 ( 1015)t ' 39.6 rad], where time t is in seconds. (a) What is the amplitude of the resultant electric field at point P? (b) What is the ratio of the intensity IP at point P to the intensity Icen at the center of the interference pattern? (c) Describe where point P is in the interference pattern by giving the maximum or minimum on which it lies, or the maximum and minimum between which it lies. In a phasor diagram of the electric fields, (d) at what rate would the phasors rotate around the origin and (e) what is the angle between the phasors? ••33 Three electromagnetic waves travel through a certain point P along an x axis. They are polarized parallel to a y axis, with the following variations in their amplitudes. Find their resultant at P. E1 ! (10.0 mV/m) sin[(2.0 ( 1014 rad/s)t] E2 ! (5.00 mV/m) sin[(2.0 ( 1014 rad/s)t ' 45.0°] E3 ! (5.00 mV/m) sin[(2.0 ( 1014 rad/s)t & 45.0°] ••34 In the double-slit experiment of Fig. 35-10, the viewing screen is at distance D ! 4.00 m, point P lies at distance y ! 20.5 cm from the center of the pattern, the slit separation d is 4.50 mm, and the wavelength l is 580 nm. (a) Determine where point P is in the interference pattern by giving the maximum or minimum on which it lies, or the maximum and minimum between which it lies. (b) What is the ratio of the intensity IP at point P to the intensity Icen at the center of the pattern? Module 35-4 Interference from Thin Films •35 SSM We wish to coat flat glass (n ! 1.50) with a transparent material (n ! 1.25) so that reflection of light at wavelength 600 nm is eliminated by interference. What minimum thickness can the coating have to do this? •36 A 600-nm-thick soap film (n ! 1.40) in air is illuminated with white light in a direction perpendicular to the film. For how many different wavelengths in the 300 to 700 nm range is there (a) fully constructive interference and (b) fully destructive interference in the reflected light? •37 The rhinestones in costume jewelry are glass with index of refraction 1.50. To make them more reflective, they are often coated

with a layer of silicon monoxide of index of refraction 2.00. What is the minimum coating thickness needed to ensure that light of wavelength 560 nm and of perpendicular incidence will be reflected from the two surfaces of the coating with fully constructive interference? •38 White light is sent downward onto a horizontal thin film that is sandwiched between two materials. The indexes of refraction are 1.80 for the top material, 1.70 for the thin film, and 1.50 for the bottom material. The film thickness is 5.00 ( 10&7 m. Of the visible wavelengths (400 to 700 nm) that result in fully constructive interference at an observer above the film, which is the (a) longer and (b) shorter wavelength? The materials and film are then heated so that the film thickness increases. (c) Does the light resulting in fully constructive interference shift toward longer or shorter wavelengths? •39 ILW Light of wavelength 624 nm is incident perpendicularly on a soap film (n ! 1.33) suspended in air. What are the (a) least and (b) second least thicknesses of the film for which the reflections from the film undergo fully constructive interference? ••40 A thin film of acetone (n ! 1.25) coats a thick glass plate (n ! 1.50). White light is incident normal to the film. In the reflections, fully destructive interference occurs at 600 nm and fully constructive interference at 700 nm. Calculate the thickness of the acetone film. ••41 through 52 43, 51 SSM 47, 51 n1 n2 n3 Reflection by thin layers. In Fig. 35-42, light is incident perpendicularly on a thin r2 layer of material 2 that lies between r 1 (thicker) materials 1 and 3. (The rays are i tilted only for clarity.) The waves of rays r1 and r2 interfere, and here we consider L the type of interference to be either Figure 35-42 Problems 41 maximum (max) or minimum (min). For through 52. this situation, each problem in Table 352 refers to the indexes of refraction n1, n2, and n3, the type of interference, the thin-layer thickness L in nanometers, and the wavelength l in nanometers of the light as measured in air. Where l is missing, give the wavelength that is in the visible range. Where L is missing, give the second least thickness or the third least thickness as indicated.

Table 35-2 Problems 41 through 52: Reflection by Thin Layers. See the setup for these problems.

41 42 43 44 45 46 47 48 49 50 51 52

n1

n2

n3

Type

L

l

1.68 1.55 1.60 1.50 1.55 1.68 1.50 1.60 1.32 1.40 1.40 1.32

1.59 1.60 1.40 1.34 1.60 1.59 1.34 1.40 1.75 1.46 1.46 1.75

1.50 1.33 1.80 1.42 1.33 1.50 1.42 1.80 1.39 1.75 1.75 1.39

min max min max max min min max max min min max

2nd 285 200 2nd 3rd 415 380 2nd 3rd 2nd 210 325

342

587 612

632 382 482

PROB LE M S

••53 The reflection of perpendicularly incident white light by a soap film in air has an interference maximum at 600 nm and a minimum at 450 nm, with no minimum in between. If n ! 1.33 for the film, what is the film thickness, assumed uniform? ••54 A plane wave of monochromatic light is incident normally on a uniform thin film of oil that covers a glass plate. The wavelength of the source can be varied continuously. Fully destructive interference of the reflected light is observed for wavelengths of 500 and 700 nm and for no wavelengths in between. If the index of refraction of the oil is 1.30 and that of the glass is 1.50, find the thickness of the oil film. ••55 SSM WWW A disabled tanker leaks kerosene (n ! 1.20) into the Persian Gulf, creating a large slick on top of the water (n ! 1.30). (a) If you are looking straight down from an airplane, while the Sun is overhead, at a region of the slick where its thickness is 460 nm, for which wavelength(s) of visible light is the reflection brightest because of constructive interference? (b) If you are scuba diving directly under this same region of the slick, for which wavelength(s) of visible light is the transmitted intensity strongest? ••56 A thin film, with a thickness of 272.7 nm and with air on both sides, is illuminated with a beam of white light. The beam is perpendicular to the film and consists of the full range of wavelengths for the visible spectrum. In the light reflected by the film, light with a wavelength of 600.0 nm undergoes fully constructive interference. At what wavelength does the reflected light undergo fully destructive interference? (Hint: You must make a reasonable assumption about the index of refraction.) ••57 through 68 64, 65 SSM 59 n 1 n2 n3 Transmission through thin layers. In r4 Fig. 35-43, light is incident perpendicularly r3 i on a thin layer of material 2 that lies between (thicker) materials 1 and 3. (The rays L are tilted only for clarity.) Part of the light ends up in material 3 as ray r3 (the light Figure 35-43 does not reflect inside material 2) and r4 Problems 57 (the light reflects twice inside material 2). through 68. The waves of r3 and r4 interfere, and here we consider the type of interference to be either maximum (max) or minimum (min). For this situation, each problem in Table 35-3 refers to the indexes of refraction n1, n2, and n3, the type

1077

of interference, the thin-layer thickness L in nanometers, and the wavelength l in nanometers of the light as measured in air. Where l is missing, give the wavelength that is in the visible range. Where L is missing, give the second least thickness or the third least thickness as indicated. ••69 In Fig. 35-44, a broad beam of light of wavelength 630 nm is incident at 90° on a thin, wedge-shaped film with index of refraction 1.50. Transmission gives 10 bright and 9 dark fringes along the film’s length. What is the left-to-right change in film thickness?

Incident light

Figure 35-44 Problem 69.

••70 In Fig. 35-45, a broad beam of light of wavelength 620 nm is sent diIncident light rectly downward through the top plate of a pair of glass plates touching at the left end. The air between the plates acts as a thin film, and an interference pattern can be seen from above the plates. Initially, a dark fringe lies at the left end, a bright fringe lies at the right end, and nine dark fringes lie between Figure 35-45 Problems 70–74. those two end fringes. The plates are then very gradually squeezed together at a constant rate to decrease the angle between them. As a result, the fringe at the right side changes between being bright to being dark every 15.0 s. (a) At what rate is the spacing between the plates at the right end being changed? (b) By how much has the spacing there changed when both left and right ends have a dark fringe and there are five dark fringes between them? ••71 In Fig. 35-45, two microscope slides touch at one end and are separated at the other end. When light of wavelength 500 nm shines vertically down on the slides, an overhead observer sees an interference pattern on the slides with the dark fringes separated by 1.2 mm. What is the angle between the slides?

••72 In Fig. 35-45, a broad beam of monochromatic light is directed perpendicularly through two glass plates that are held together at one end to create a wedge of air between them. An observer intercepting light reflected from the wedge of air, which acts as a thin film, sees 4001 dark fringes along the length of the wedge. When the air Table 35-3 Problems 57 through 68: Transmission Through Thin Layers. between the plates is evacuated, only 4000 See the setup for these problems. dark fringes are seen. Calculate to six significant figures the index of refraction of air n1 n2 n3 Type L l from these data. 57 1.55 1.60 1.33 min 285 ••73 SSM In Fig. 35-45, a broad beam of 58 1.32 1.75 1.39 min 3rd 382 light of wavelength 683 nm is sent directly 59 1.68 1.59 1.50 max 415 downward through the top plate of a pair of 60 1.50 1.34 1.42 max 380 glass plates. The plates are 120 mm long, 61 1.32 1.75 1.39 min 325 touch at the left end, and are separated by 62 1.68 1.59 1.50 max 2nd 342 48.0 mm at the right end. The air between the plates acts as a thin film. How many bright 63 1.40 1.46 1.75 max 2nd 482 fringes will be seen by an observer looking 64 1.40 1.46 1.75 max 210 down through the top plate? 65 1.60 1.40 1.80 min 2nd 632 66 1.60 1.40 1.80 max 200 ••74 Two rectangular glass plates (n ! 1.60) are in contact along one edge and are 67 1.50 1.34 1.42 min 2nd 587 separated along the opposite edge 68 1.55 1.60 1.33 min 3rd 612 (Fig. 35-45). Light with a wavelength of 600

1078

CHAPTE R 35 I NTE R FE R E NCE

nm is incident perpendicularly onto the top plate. The air between the plates acts as a thin film. Nine dark fringes and eight bright fringes are observed from above the top plate. If the distance between the two plates along the separated edges is increased by 600 nm, how many dark fringes will there then be across the top plate? ••75 SSM ILW Figure 35-46a shows a lens with radius of curvature R lying on a flat glass plate and illuminated from above by light with wavelength l. Figure 35-46b (a Incident photograph taken from above light the lens) shows that circular inR terference fringes (known as Newton’s rings) appear, associated with the variable thickness d r of the air film between the lens Air Glass d and the plate. Find the radii r of the interference maxima assumGlass (a) ing r/R 5 1.

Module 35-5 Michelson’s Interferometer •79 If mirror M2 in a Michelson interferometer (Fig. 35-21) is moved through 0.233 mm, a shift of 792 bright fringes occurs. What is the wavelength of the light producing the fringe pattern? •80 A thin film with index of refraction n ! 1.40 is placed in one arm of a Michelson interferometer, perpendicular to the optical path. If this causes a shift of 7.0 bright fringes of the pattern produced by light of wavelength 589 nm, what is the film thickness? WWW ••81 SSM In Fig. 35-48, an airtight chamber of length d ! 5.0 cm is placed in one of the arms of a Michelson interferometer. (The glass window on each end of the chamber has negligible thickness.) Light of wavelength l ! 500 nm is used. Evacuating the air from the chamber causes a shift of 60 bright fringes. From these data and to six significant figures, find the index of refraction of air at atmospheric pressure.

Mirror

d Source

Mirror To vacuum pump

••82 The element sodium can emit light at two wavelengths, l1 ! 588.9950 nm and l2 ! 589.5924 nm. Figure 35-48 Problem 81. Light from sodium is being used in a Michelson interferometer (Fig. 35-21). Through what distance must mirror M2 be moved if the shift in the fringe pattern for one wavelength is to be 1.00 fringe more than the shift in the fringe pattern for the other wavelength? Figure 35-46 Problems 75–77. (b)

Courtesy Bausch & Lomb

••76 The lens in a Newton’s rings experiment (see Problem 75) has diameter 20 mm and radius of curvature R ! 5.0 m. For l ! 589 nm in air, how many bright rings are produced with the setup (a) in air and (b) immersed in water (n ! 1.33)? ••77 A Newton’s rings apparatus is to be used to determine the radius of curvature of a lens (see Fig. 35-46 and Problem 75). The radii of the nth and (n ' 20)th bright rings are found to be 0.162 and 0.368 cm, respectively, in light of wavelength 546 nm. Calculate the radius of curvature of the lower surface of the lens.

I

•••78 A thin film of liquid is held in a horizontal circular ring, with air on both sides of the film. A beam of light at wavelength 550 nm is directed perpendicularly onto the film, and the intensity I of its reflection is monitored. Figure 35-47 gives intensity I as a function of time t; the horizontal scale is set by ts ! 20.0 s. The intensity changes because of evaporation from the two sides of the film. Assume that the film is flat and has parallel sides, a radius of 1.80 cm, and an index of refraction of 1.40. Also assume that the film’s volume decreases at a constant rate. Find that rate.

0 t (s)

ts

Figure 35-47 Problem 78.

Additional Problems 83 Two light rays, initially in phase and with a wavelength of 500 nm, go through different paths by reflecting from the various mirrors shown in Fig. 35-49. (Such a reflection does not itself produce a phase shift.) (a) What least value of distance d will put the rays exactly out of phase when they emerge from the region? (Ignore the slight tilt of the path for ray 2.) (b) Repeat the question assuming that the entire apparatus is immersed in a protein solution with an index of refraction of 1.38.

d

d Ray 1

d d

Ray 2

Figure 35-49 Problem 83. y

Screen 84 In Figure 35-50, two isotropic point sources S1 and S2 emit light in P phase at wavelength l and at the same amplitude. The sources are separated by distance d ! 6.00l S1 S2 on an x axis. A viewing screen is at x distance D ! 20.0l from S2 and pard D allel to the y axis. The figure shows two rays reaching point P on Figure 35-50 Problem 84. the screen, at height yP. (a) At what value of yP do the rays have the minimum possible phase difference? (b) What multiple of l gives that minimum phase difference? (c) At what value of yP do the rays have the maximum possible phase difference? What multiple of l gives (d) that maximum phase difference and (e) the phase difference when yP ! d? (f) When yP ! d, is the resulting intensity at point P maximum, mini-

PROB LE M S

85 SSM A double-slit arrangement produces bright interference fringes for sodium light (a distinct yellow light at a wavelength of l ! 589 nm). The fringes are angularly separated by 0.30° near the center of the pattern. What is the angular fringe separation if the entire arrangement is immersed in water, which has an index of refraction of 1.33? 86 In Fig. 35-51a, the waves along rays 1 and 2 are initially in phase, with the same wavelength l in air. Ray 2 goes through a material with length L and index of refraction n. The rays are then reflected by mirrors to a common point P on a screen. Suppose that we can vary n from n ! 1.0 to n ! 2.5. Suppose also that, from n ! 1.0 to n ! ns ! 1.5, the intensity I of the light at point P varies with n as given in Fig. 35-51b. At what values of n greater than 1.4 is intensity I (a) maximum and (b) zero? (c) What multiple of l gives the phase difference between the rays at point P when n ! 2.0?

Ray 2

I

Screen

L n

P

Ray 1

n ns

1 (a)

(b)

Figure 35-51 Problems 86 and 87. I 87 SSM In Fig. 35-51a, the waves along rays 1 and 2 are initially in phase, with the same wavelength l in air. Ray 2 goes through a material with length L and index of refraction n. The rays are then reflected by mirrors to a common point P on a screen. Suppose that we Ls can vary L from 0 to 2400 nm. Suppose 0 L (nm) also that, from L ! 0 to Ls ! 900 nm, Figure 35-52 Problem 87. the intensity I of the light at point P varies with L as given in Fig. 35-52. At what values of L greater than Ls is intensity I (a) maximum and (b) zero? (c) What multiple of l gives the phase difference between ray 1 and ray 2 at common point P when L ! 1200 nm?

88 Light of wavelength 700.0 nm is sent along a route of length 2000 nm. The route is then filled with a medium having an index of refraction of 1.400. In degrees, by how much does the medium phase-shift the light? Give (a) the full shift and (b) the equivalent shift that has a value less than 360°.

Assuming that the lake width D is much greater than a and x, and that l 6 a, find an expression that gives the values of x for which the signal at the receiver is maximum. (Hint: Does the reflection cause a phase change?) y

90 In Fig. 35-54, two isotropic point P2 sources S1 and S2 emit light at wavelength l ! 400 nm. Source S1 is located at y ! 640 S1 nm; source S2 is located at y ! &640 nm. x At point P1 (at x ! 720 nm), the wave P1 S 2 from S2 arrives ahead of the wave from S1 by a phase difference of 0.600p rad. (a) Figure 35-54 What multiple of l gives the phase differProblem 90. ence between the waves from the two sources as the waves arrive at point P2, which is located at y ! 720 nm? (The figure is not drawn to scale.) (b) If the waves arrive at P2 with equal amplitudes, is the interference there fully constructive, fully destructive, intermediate but closer to fully constructive, or intermediate but closer to fully Shoreline θ2 destructive? 91 Ocean waves moving at a Shallow water speed of 4.0 m/s are approaching a beach at angle u1 ! 30° to the normal, as shown from above in Fig. 35-55. Suppose the water depth Deep water θ1 changes abruptly at a certain distance from the beach and the wave Figure 35-55 Problem 91. speed there drops to 3.0 m/s. (a) Close to the beach, what is the angle u2 between the direction of wave motion and the normal? (Assume the same law of refraction as for light.) (b) Explain why most waves come in normal to a shore even though at large distances they approach at a variety of angles. 92 Figure 35-56a shows two light rays that are initially in phase as they travel upward through a block of plastic, with wavelength 400 nm as measured in air. Light ray r1 exits directly into air. However, before light ray r2 exits into air, it travels through a liquid in a hollow cylinder within the plastic. Initially the height Lliq of the liquid is 40.0 mm, but then the liquid begins to evaporate. Let f be the phase difference between rays r1 and r2 once they both exit into the air. Figure 35-56b shows f versus the liquid’s height Lliq until the liquid disappears, with f given in terms of wavelength and the horizontal scale set by Ls ! 40.00 mm. What are (a) the index of refraction of the plastic and (b) the index of refraction of the liquid? r1

r2

60

89 SSM In Fig. 35-53, a microwave transmitter at height a above the water level of a wide lake transmits microwaves of wavelength l toward a receiver on the opposite shore, a distance x above the water level. The microwaves reflecting from the water interfere with the microwaves arriving directly from the transmitter.

L liq

Plastic

φ ( λ)

mum, intermediate but closer to maximum, or intermediate but closer to minimum?

20 0

(a) a

x

D

Figure 35-53 Problem 89.

1079

L liq (µ m)

Ls

(b)

Figure 35-56 Problem 92. 93 SSM If the distance between the first and tenth minima of a double-slit pattern is 18.0 mm and the slits are separated by 0.150 mm with the screen 50.0 cm from the slits, what is the wavelength of the light used?

1080

CHAPTE R 35 I NTE R FE R E NCE

n2 94 Figure 35-57 shows an optical fiber in which a central plastic core of index of refraction n1 ! n1 1.58 is surrounded by a plastic sheath of index of refraction n2 ! 1.53. Light can travel along difFigure 35-57 Problem 94. ferent paths within the central core, leading to different travel times through the fiber.This causes an initially short pulse of light to spread as it travels along the fiber, resulting in information loss. Consider light that travels directly along the central axis of the fiber and light that is repeatedly reflected at the critical angle along the core – sheath interface, reflecting from side to side as it travels down the central core. If the fiber length is 300 m, what is the difference in the travel times along these two routes?

95 SSM Two parallel slits are illuminated with monochromatic light of wavelength 500 nm. An interference pattern is formed on a screen some distance from the slits, and the fourth dark band is located 1.68 cm from the central bright band on the screen. (a) What is the path length difference corresponding to the fourth dark band? (b) What is the distance on the screen between the central bright band and the first bright band on either side of the central band? (Hint: The angle to the fourth dark band and the angle to the first bright band are small enough that tan u % sin u.) 96 A camera lens with index of refraction greater than 1.30 is coated with a thin transparent film of index of refraction 1.25 to eliminate by interference the reflection of light at wavelength l that is incident perpendicularly on the lens. What multiple of l gives the minimum film thickness needed? 97 SSM Light of wavelength l is used in a Michelson interferometer. Let x be the position of the movable mirror, with x ! 0 when the arms have equal lengths d2 ! d1.Write an expression for the intensity of the observed light as a function of x, letting Im be the maximum intensity. 98 In two experiments, light is to be sent along the two paths shown in Fig. 35-35 by reflecting it from the various flat surfaces shown. In the first experiment, rays 1 and 2 are initially in phase and have a wavelength of 620.0 nm. In the second experiment, rays 1 and 2 are initially in phase and have a wavelength of 496.0 nm. What least value of distance L is required such that the 620.0 nm waves emerge from the region exactly in phase but the 496.0 nm waves emerge exactly out of phase? 99 Figure 35-58 shows the design of a Texas arcade game. Four laser pistols are pointed toward the center of an array of plastic 1 n1 n2 n4 n5

2

4

n6

n7

n8 n9

Figure 35-58 Problem 99.

n3

3

layers where a clay armadillo is the target. The indexes of refraction of the layers are n1 ! 1.55, n2 ! 1.70, n3 ! 1.45, n4 ! 1.60, n5 ! 1.45, n6 ! 1.61, n7 ! 1.59, n8 ! 1.70, and n9 ! 1.60. The layer thicknesses are either 2.00 mm or 4.00 mm, as drawn. What is the travel time through the layers for the laser burst from (a) pistol 1, (b) pistol 2, (c) pistol 3, and (d) pistol 4? (e) If the pistols are fired simultaneously, which laser burst hits the target first? 100 A thin film suspended in air is 0.410 mm thick and is illuminated with white light incident perpendicularly on its surface. The index of refraction of the film is 1.50. At what wavelength will visible light that is reflected from the two surfaces of the film undergo fully constructive interference? 101 Find the slit separation of a double-slit arrangement that will produce interference fringes 0.018 rad apart on a distant screen when the light has wavelength l ! 589 nm. 102 In a phasor diagram for any point on the viewing screen for the two-slit experiment in Fig. 35-10, the resultant-wave phasor rotates 60.0° in 2.50 ( 10&16 s. What is the wavelength? 103 In Fig. 35-59, an oil drop (n 5 1.20) floats on the surface of water (n 5 1.33) and is viewed from overhead when illuminated by sunlight shining vertically downward and reflected vertically upward. (a) Are the outer (thinnest) regions of the drop bright or dark? The oil film displays several spectra of colors. (b) Move from the rim inward to the third blue band and, using a wavelength of 475 nm for blue light, determine the film thickness there. (c) If the oil thickness increases, why do the colors gradually fade and then disappear? Oil Water

Figure 35-59 Problem 103. 104 Lloyd’s Mirror. In Fig. 35-60, monoA chromatic light of wavelength l diffracts S h through a narrow slit S in an otherwise opaque screen. On the other side, a plane mirMirror ror is perpendicular to the screen and a distance h from the slit. A viewing screen A is a distance much greater than h. (Because it sits Figure 35-60 in a plane through the focal point of the lens, Problem 104. screen A is effectively very distant. The lens plays no other role in the experiment and can otherwise be neglected.) Light that travels from the slit directly to A interferes with light from the slit that reflects from the mirror to A. The reflection causes a halfwavelength phase shift. (a) Is the fringe that corresponds to a zero path length difference bright or dark? Find expresy sions (like Eqs. 35-14 and 35-16) that locate (b) the bright fringes and (c) the dark fringes r1 in the interference pattern. (Hint: Consider the image of S produced by the mirror as seen r2 from a point on the viewing screen, and then consider Young’s two-slit interference.) 105 The two point sources in Fig. 35-61 emit coherent waves. Show that all curves (such as the one shown), over which the phase difference for rays r1 and r2 is a constant, are hyperbolas. (Hint: A constant phase difference implies a constant difference in length between r1 and r2.)

S1

S2 d

Figure 35-61 Problem 105.

x

C

H

A

P

T

E

R

3

6

Diffraction 36-1 SINGLE-SLIT DIFFRACTION Learning Objectives After reading this module, you should be able to . . .

rectangular slit or object, the wavelength l, the angle u to 36.01 Describe the diffraction of light waves by a narrow opening any of the minima in the diffraction pattern, the distance to and an edge, and also describe the resulting interference pattern. a viewing screen, and the distance between a minimum 36.02 Describe an experiment that demonstrates the and the center of the pattern. Fresnel bright spot. 36.06 Sketch the diffraction pattern for monochromatic light, iden36.03 With a sketch, describe the arrangement for a tifying what lies at the center and what the various bright and single-slit diffraction experiment. dark fringes are called (such as “first minimum”). 36.04 With a sketch, explain how splitting a slit width into 36.07 Identify what happens to a diffraction pattern when equal zones leads to the equations giving the angles to the the wavelength of the light or the width of the diffracting minima in the diffraction pattern. aperture or object is varied. 36.05 Apply the relationships between width a of a thin,

Key Ideas ● When waves encounter an edge, an obstacle, or an aperture the size of which is comparable to the wavelength of the waves, those waves spread out as they travel and, as a result, undergo interference. This type of interference is called diffraction. ● Waves passing through a long narrow slit of width a

produce, on a viewing screen, a single-slit diffraction

pattern that includes a central maximum (bright fringe) and other maxima. They are separated by minima that are located relative to the central axis by angles u: a sin u ! ml,

for m ! 1, 2, 3, . . .

(minima).

● The maxima are located approximately halfway between minima.

What Is Physics? One focus of physics in the study of light is to understand and put to use the diffraction of light as it passes through a narrow slit or (as we shall discuss) past either a narrow obstacle or an edge.We touched on this phenomenon in Chapter 35 when we looked at how light flared—diffracted—through the slits in Young’s experiment. Diffraction through a given slit is more complicated than simple flaring, however, because the light also interferes with itself and produces an interference pattern. It is because of such complications that light is rich with application opportunities. Even though the diffraction of light as it passes through a slit or past an obstacle seems awfully academic, countless engineers and scientists make their living using this physics, and the total worth of diffraction applications worldwide is probably incalculable. Before we can discuss some of these applications, we first must discuss why diffraction is due to the wave nature of light.

Diffraction and the Wave Theory of Light In Chapter 35 we defined diffraction rather loosely as the flaring of light as it emerges from a narrow slit. More than just flaring occurs, however, because the 1081

1082

CHAPTE R 36 DI FFRACTION

Ken Kay/Fundamental Photographs

Figure 36-1 This diffraction pattern appeared on a viewing screen when light that had passed through a narrow vertical slit reached the screen. Diffraction caused the light to flare out perpendicular to the long sides of the slit. That flaring produced an interference pattern consisting of a broad central maximum plus less intense and narrower secondary (or side) maxima, with minima between them.

light produces an interference pattern called a diffraction pattern. For example, when monochromatic light from a distant source (or a laser) passes through a narrow slit and is then intercepted by a viewing screen, the light produces on the screen a diffraction pattern like that in Fig. 36-1. This pattern consists of a broad and intense (very bright) central maximum plus a number of narrower and less intense maxima (called secondary or side maxima) to both sides. In between the maxima are minima. Light flares into those dark regions, but the light waves cancel out one another. Such a pattern would be totally unexpected in geometrical optics: If light traveled in straight lines as rays, then the slit would allow some of those rays through to form a sharp rendition of the slit on the viewing screen instead of a pattern of bright and dark bands as we see in Fig. 36-1. As in Chapter 35, we must conclude that geometrical optics is only an approximation. Edges. Diffraction is not limited to situations in which light passes through a narrow opening (such as a slit or pinhole). It also occurs when light passes an edge, such as the edges of the razor blade whose diffraction pattern is shown in Fig. 36-2. Note the lines of maxima and minima that run approximately parallel to the edges, at both the inside edges of the blade and the outside edges. As the light passes, say, the vertical edge at the left, it flares left and right and undergoes interference, producing the pattern along the left edge. The rightmost portion of that pattern actually lies behind the blade, within what would be the blade’s shadow if geometrical optics prevailed. Floaters. You encounter a common example of diffraction when you look at a clear blue sky and see tiny specks and hairlike structures floating in your view. These floaters, as they are called, are produced when light passes the edges of tiny deposits in the vitreous humor, the transparent material filling most of the eyeball. What you are seeing when a floater is in your field of vision is the diffraction pattern produced on the retina by one of these deposits. If you sight through a pinhole in a piece of cardboard so as to make the light entering your eye approximately a plane wave, you can distinguish individual maxima and minima in the patterns. Cheerleaders. Diffraction is a wave effect. That is, it occurs because light is a wave and it occurs with other types of waves as well. For example, you have probably seen diffraction in action at football games. When a cheerleader near the playing field yells up at several thousand noisy fans, the yell can hardly be heard because the sound waves diffract when they pass through the narrow opening of the cheerleader’s mouth. This flaring leaves little of the waves traveling toward the fans in front of the cheerleader. To offset the diffraction, the cheerleader can yell through a megaphone. The sound waves then emerge from the much wider opening at the end of the megaphone. The flaring is thus reduced, and much more of the sound reaches the fans in front of the cheerleader.

The Fresnel Bright Spot

Ken Kay/Fundamental Photographs

Figure 36-2 The diffraction pattern produced by a razor blade in monochromatic light. Note the lines of alternating maximum and minimum intensity.

Diffraction finds a ready explanation in the wave theory of light. However, this theory, originally advanced in the late 1600s by Huygens and used 123 years later by Young to explain double-slit interference, was very slow in being adopted, largely because it ran counter to Newton’s theory that light was a stream of particles. Newton’s view was the prevailing view in French scientific circles of the early 19th century, when Augustin Fresnel was a young military engineer. Fresnel, who believed in the wave theory of light, submitted a paper to the French Academy of Sciences describing his experiments with light and his wave-theory explanations of them. In 1819, the Academy, dominated by supporters of Newton and thinking to challenge the wave point of view, organized a prize competition for an essay on the subject of diffraction. Fresnel won. The Newtonians, however, were not swayed. One of them, S. D. Poisson, pointed out the “strange result” that if Fresnel’s theories were correct, then light waves should flare into the shadow region of a sphere as they pass the edge of the sphere, producing a bright spot at the center of the shadow. The prize committee arranged a test of Poisson’s prediction and dis-

36-1 SI NG LE-SLIT DI FFRACTION

1083

covered that the predicted Fresnel bright spot, as we call it today, was indeed there (Fig. 36-3). Nothing builds confidence in a theory so much as having one of its unexpected and counterintuitive predictions verified by experiment.

Diffraction by a Single Slit: Locating the Minima Let us now examine the diffraction pattern of plane waves of light of wavelength l that are diffracted by a single long, narrow slit of width a in an otherwise opaque screen B, as shown in cross section in Fig. 36-4. (In that figure, the slit’s length extends into and out of the page, and the incoming wavefronts are parallel to screen B.) When the diffracted light reaches viewing screen C, waves from different points within the slit undergo interference and produce a diffraction pattern of bright and dark fringes (interference maxima and minima) on the screen. To locate the fringes, we shall use a procedure somewhat similar to the one we used to locate the fringes in a two-slit interference pattern. However, diffraction is more mathematically challenging, and here we shall be able to find equations for only the dark fringes. Before we do that, however, we can justify the central bright fringe seen in Fig. 36-1 by noting that the Huygens wavelets from all points in the slit travel about the same distance to reach the center of the pattern and thus are in phase there. As for the other bright fringes, we can say only that they are approximately halfway between adjacent dark fringes. Pairings. To find the dark fringes, we shall use a clever (and simplifying) strategy that involves pairing up all the rays coming through the slit and then finding what conditions cause the wavelets of the rays in each pair to cancel each other. We apply this strategy in Fig. 36-4 to locate the first dark fringe, at point P1. First, we mentally divide the slit into two zones of equal widths a/2. Then we extend to P1 a light ray r1 from the top point of the top zone and a light ray r2 from the top point of the bottom zone. We want the wavelets along these two rays to cancel each other when they arrive at P1. Then any similar pairing of rays from the two zones will give cancellation. A central axis is drawn from the center of the slit to screen C, and P1 is located at an angle u to that axis. Path Length Difference. The wavelets of the pair of rays r1 and r2 are in phase within the slit because they originate from the same wavefront passing through the slit, along the width of the slit. However, to produce the first dark fringe they must be out of phase by l/2 when they reach P1; this phase difference is due to their path length difference, with the path traveled by the wavelet of r2 to reach P1 being longer than the path traveled by the wavelet of r1. To display this path length difference, we find a point b on ray r2 such that the path length from b to P1 matches the path length of ray r1. Then the path length difference between the two rays is the distance from the center of the slit to b. When viewing screen C is near screen B, as in Fig. 36-4, the diffraction pattern on C is difficult to describe mathematically. However, we can simplify the mathematics considerably if we arrange for the screen separation D to be much larger than the slit width a. Then, as in Fig. 36-5, we can approximate rays r1 and r2 r1

θ a/2 θ

Figure 36-5 For D 4 a, we can approximate rays r1 and r2 as being parallel, at angle u to the central axis.

b

θ Path length difference

r2

This path length difference shifts one wave from the other, which determines the interference.

Courtesy Jearl Walker

Figure 36-3 A photograph of the diffraction pattern of a disk. Note the concentric diffraction rings and the Fresnel bright spot at the center of the pattern. This experiment is essentially identical to that arranged by the committee testing Fresnel’s theories, because both the sphere they used and the disk used here have a cross section with a circular edge.

This pair of rays cancel each other at P1. So do all such pairings. D Totally destructive interference r1 b

a/2

P1 r2

θ Central axis

P0

a/2

Incident wave

B

Viewing screen C

Figure 36-4 Waves from the top points of two zones of width a/2 undergo fully destructive interference at point P1 on viewing screen C.

1084

CHAPTE R 36 DI FFRACTION

D

P2

r1

P1

r2 r3

a/4 a/4

r4

θ

a/4

P0

These rays cancel at P2.

a/4 B

Incident wave

which gives us

To see the cancellation, group the rays into pairs.

r1

θ

r2

Path length difference between r1 and r2

r3 a/4

θ r4

a/4

θ θ

Path length difference between r3 and r4

(b)

Figure 36-6 (a) Waves from the top points of four zones of width a/4 undergo fully destructive interference at point P2. (b) For D 4 a, we can approximate rays r1, r2, r3, and r4 as being parallel, at angle u to the central axis.

l a sin u ! , 2 2 a sin u ! l

C (a)

a/4

as being parallel, at angle u to the central axis. We can also approximate the triangle formed by point b, the top point of the slit, and the center point of the slit as being a right triangle, and one of the angles inside that triangle as being u. The path length difference between rays r1 and r2 (which is still the distance from the center of the slit to point b) is then equal to (a/2) sin u. First Minimum. We can repeat this analysis for any other pair of rays originating at corresponding points in the two zones (say, at the midpoints of the zones) and extending to point P1. Each such pair of rays has the same path length difference (a/2) sin u. Setting this common path length difference equal to l/2 (our condition for the first dark fringe), we have

(first minimum).

(36-1)

Given slit width a and wavelength l, Eq. 36-1 tells us the angle u of the first dark fringe above and (by symmetry) below the central axis. Narrowing the Slit. Note that if we begin with a $ l and then narrow the slit while holding the wavelength constant, we increase the angle at which the first dark fringes appear; that is, the extent of the diffraction (the extent of the flaring and the width of the pattern) is greater for a narrower slit. When we have reduced the slit width to the wavelength (that is, a ! l), the angle of the first dark fringes is 90°. Since the first dark fringes mark the two edges of the central bright fringe, that bright fringe must then cover the entire viewing screen. Second Minimum. We find the second dark fringes above and below the central axis as we found the first dark fringes, except that we now divide the slit into four zones of equal widths a/4, as shown in Fig. 36-6a. We then extend rays r1, r2, r3, and r4 from the top points of the zones to point P2, the location of the second dark fringe above the central axis. To produce that fringe, the path length difference between r1 and r2, that between r2 and r3, and that between r3 and r4 must all be equal to l/2. For D 4 a, we can approximate these four rays as being parallel, at angle u to the central axis. To display their path length differences, we extend a perpendicular line through each adjacent pair of rays, as shown in Fig. 36-6b, to form a series of right triangles, each of which has a path length difference as one side. We see from the top triangle that the path length difference between r1 and r2 is (a/4) sin u. Similarly, from the bottom triangle, the path length difference between r3 and r4 is also (a/4) sin u. In fact, the path length difference for any two rays that originate at corresponding points in two adjacent zones is (a/4) sin u. Since in each such case the path length difference is equal to l/2, we have

which gives us

l a sin u ! , 4 2 a sin u ! 2l

(second minimum).

(36-2)

All Minima. We could now continue to locate dark fringes in the diffraction pattern by splitting up the slit into more zones of equal width. We would always choose an even number of zones so that the zones (and their waves) could be paired as we have been doing. We would find that the dark fringes above and below the central axis can be located with the general equation a sin u ! ml,

for m ! 1, 2, 3, . . .

(minima — dark fringes).

(36-3)

You can remember this result in the following way. Draw a triangle like the one in Fig. 36-5, but for the full slit width a, and note that the path length difference between the top and bottom rays equals a sin u. Thus, Eq. 36-3 says:

36-1 SI NG LE-SLIT DI FFRACTION

1085

In a single-slit diffraction experiment, dark fringes are produced where the path length differences (a sin u) between the top and bottom rays are equal to l, 2l, 3l, . . . .

This may seem to be wrong because the waves of those two particular rays will be exactly in phase with each other when their path length difference is an integer number of wavelengths. However, they each will still be part of a pair of waves that are exactly out of phase with each other; thus, each wave will be canceled by some other wave, resulting in darkness. (Two light waves that are exactly out of phase will always cancel each other, giving a net wave of zero, even if they happen to be exactly in phase with other light waves.) Using a Lens. Equations 36-1, 36-2, and 36-3 are derived for the case of D 4 a. However, they also apply if we place a converging lens between the slit and the viewing screen and then move the screen in so that it coincides with the focal plane of the lens. The lens ensures that rays which now reach any point on the screen are exactly parallel (rather than approximately) back at the slit.They are like the initially parallel rays of Fig. 34-14a that are directed to the focal point by a converging lens.

Checkpoint 1 We produce a diffraction pattern on a viewing screen by means of a long narrow slit illuminated by blue light. Does the pattern expand away from the bright center (the maxima and minima shift away from the center) or contract toward it if we (a) switch to yellow light or (b) decrease the slit width?

Sample Problem 36.01 Single-slit diffraction pattern with white light A slit of width a is illuminated by white light.

KEY IDEA

(a) For what value of a will the first minimum for red light of wavelength l ! 650 nm appear at u ! 15°?

The first side maximum for any wavelength is about halfway between the first and second minima for that wavelength.

KEY IDEA

Calculations: Those first and second minima can be located with Eq. 36-3 by setting m ! 1 and m ! 2, respectively. Thus, the first side maximum can be located approximately by setting m ! 1.5.Then Eq. 36-3 becomes

Diffraction occurs separately for each wavelength in the range of wavelengths passing through the slit, with the locations of the minima for each wavelength given by Eq. 36-3 (a sin u ! ml). Calculation: When we set m ! 1 (for the first minimum) and substitute the given values of u and l, Eq. 36-3 yields a!

ml (1)(650 nm) ! sin u sin 15"

! 2511 nm % 2.5 mm.

(Answer)

For the incident light to flare out that much (,15° to the first minima) the slit has to be very fine indeed — in this case, a mere four times the wavelength. For comparison, note that a fine human hair may be about 100 mm in diameter. (b) What is the wavelength l# of the light whose first side diffraction maximum is at 15°, thus coinciding with the first minimum for the red light?

a sin u ! 1.5l#. Solving for l# and substituting known data yield a sin u (2511 nm)(sin 15") ! 1.5 1.5 ! 430 nm.

l# !

(Answer)

Light of this wavelength is violet (far blue, near the shortwavelength limit of the human range of visible light). From the two equations we used, can you see that the first side maximum for light of wavelength 430 nm will always coincide with the first minimum for light of wavelength 650 nm, no matter what the slit width is? However, the angle u at which this overlap occurs does depend on slit width. If the slit is relatively narrow, the angle will be relatively large, and conversely.

Additional examples, video, and practice available at WileyPLUS

1086

CHAPTE R 36 DI FFRACTION

36-2 INTENSITY IN SINGLE-SLIT DIFFRACTION Learning Objectives After reading this module, you should be able to . . .

36.08 Divide a thin slit into multiple zones of equal width and write an expression for the phase difference of the wavelets from adjacent zones in terms of the angle u to a point on the viewing screen. 36.09 For single-slit diffraction, draw phasor diagrams for the central maximum and several of the minima and maxima off to one side, indicating the phase difference between adjacent phasors, explaining how the net electric field is calculated, and

identifying the corresponding part of the diffraction pattern. 36.10 Describe a diffraction pattern in terms of the net electric field at points in the pattern. 36.11 Evaluate a, the convenient connection between angle u to a point in a diffraction pattern and the intensity I at that point. 36.12 For a given point in a diffraction pattern, at a given angle, calculate the intensity I in terms of the intensity Im at the center of the pattern.

Key Idea ● The intensity of the diffraction pattern at any given angle u

is

where Im is the intensity at the center of the pattern and

# sina a $ ,

I(u) ! Im

2

a!

pa sin u. l

Intensity in Single-Slit Diffraction, Qualitatively In Module 36-1 we saw how to find the positions of the minima and the maxima in a single-slit diffraction pattern. Now we turn to a more general problem: find an expression for the intensity I of the pattern as a function of u, the angular position of a point on a viewing screen. To do this, we divide the slit of Fig. 36-4 into N zones of equal widths 2x small enough that we can assume each zone acts as a source of Huygens wavelets. We wish to superimpose the wavelets arriving at an arbitrary point P on the viewing screen, at angle u to the central axis, so that we can determine the amplitude Eu of the electric component of the resultant wave at P. The intensity of the light at P is then proportional to the square of that amplitude. To find Eu , we need the phase relationships among the arriving wavelets. The point here is that in general they have different phases because they travel different distances to reach P. The phase difference between wavelets from adjacent zones is given by phase length . #difference $ ! # 2pl $ #path difference $ For point P at angle u, the path length difference between wavelets from adjacent zones is 2x sin u. Thus, we can write the phase difference 2f between wavelets from adjacent zones as 2p (36-4) (2 x sin u). 2f ! l

# $

We assume that the wavelets arriving at P all have the same amplitude 2E. To find the amplitude Eu of the resultant wave at P, we add the amplitudes 2E via phasors. To do this, we construct a diagram of N phasors, one corresponding to the wavelet from each zone in the slit. Central Maximum. For point P0 at u ! 0 on the central axis of Fig. 36-4, Eq. 36-4 tells us that the phase difference 2f between the wavelets is zero; that is, the wavelets all arrive in phase. Figure 36-7a is the corresponding phasor diagram; adjacent phasors represent wavelets from adjacent zones and are arranged head to tail. Because there is zero phase difference between the wavelets, there is zero angle between each pair of adjacent phasors. The amplitude Eu of the net

36-2 I NTE NSITY I N SI NG LE-SLIT DI FFRACTION

θ

1087

Here, with an even larger phase difference, they add to give a small amplitude and thus a small intensity.



(d)

The last phasor is out of phase with the first phasor by 2π rad (full circle). Here, with a larger phase difference, the phasors add to give zero amplitude and thus a minimum in the pattern.

Eθ = 0 (c)



I

(b)

Here the phasors have a small phase difference and add to give a smaller amplitude and thus less intensity in the pattern.

⎧ ⎨ ⎩

Eθ (= Em) Phasor for top ray

Figure 36-7 Phasor diagrams for N ! 18 phasors, corresponding to the division of a single slit into 18 zones. Resultant amplitudes Eu are shown for (a) the central maximum at u ! 0, (b) a point on the screen lying at a small angle u to the central axis, (c) the first minimum, and (d) the first side maximum.

wave at P0 is the vector sum of these phasors. This arrangement of the phasors turns out to be the one that gives the greatest value for the amplitude E u. We call this value Em; that is, Em is the value of Eu for u ! 0. We next consider a point P that is at a small angle u to the central axis. Equation 36-4 now tells us that the phase difference 2f between wavelets from adjacent zones is no longer zero. Figure 36-7b shows the corresponding phasor diagram; as before, the phasors are arranged head to tail, but now there is an angle 2f between adjacent phasors. The amplitude Eu at this new point is still the vector sum of the phasors, but it is smaller than that in Fig. 36-7a, which means that the intensity of the light is less at this new point P than at P0. First Minimun. If we continue to increase u, the angle 2f between adjacent phasors increases, and eventually the chain of phasors curls completely around so that the head of the last phasor just reaches the tail of the first phasor (Fig. 36-7c). The ampli-

∆E

(a)

Phasor for bottom ray

The phasors from the 18 zones in the slit are in phase and add to give a maximum amplitude and thus the central maximum in the diffraction pattern.

A

1088

CHAPTE R 36 DI FFRACTION

tude Eu is now zero, which means that the intensity of the light is also zero. We have reached the first minimum, or dark fringe, in the diffraction pattern. The first and last phasors now have a phase difference of 2p rad, which means that the path length difference between the top and bottom rays through the slit equals one wavelength. Recall that this is the condition we determined for the first diffraction minimum. First Side Maximum. As we continue to increase u, the angle 2f between adjacent phasors continues to increase, the chain of phasors begins to wrap back on itself, and the resulting coil begins to shrink. Amplitude Eu now increases until it reaches a maximum value in the arrangement shown in Fig. 36-7d. This arrangement corresponds to the first side maximum in the diffraction pattern. Second Minimum. If we increase u a bit more, the resulting shrinkage of the coil decreases Eu , which means that the intensity also decreases. When u is increased enough, the head of the last phasor again meets the tail of the first phasor. We have then reached the second minimum. We could continue this qualitative method of determining the maxima and minima of the diffraction pattern but, instead, we shall now turn to a quantitative method.

Checkpoint 2 The figures represent, in smoother form (with more phasors) than Fig. 36-7, the phasor diagrams for two points of a diffraction pattern that are on opposite sides of a certain diffraction maximum. (a) Which maximum is it? (b) What is the approximate value of m (in Eq. 36-3) that corresponds to this maximum?

(a)

(b)

Intensity in Single-Slit Diffraction, Quantitatively Equation 36-3 tells us how to locate the minima of the single-slit diffraction pattern on screen C of Fig. 36-4 as a function of the angle u in that figure. Here we wish to derive an expression for the intensity I(u) of the pattern as a function of u. We state, and shall prove below, that the intensity is given by

where

I(u) ! Im

# sina a $ ,

(36-5)

a ! 12f !

pa sin u. l

(36-6)

2

The symbol a is just a convenient connection between the angle u that locates a point on the viewing screen and the light intensity I(u) at that point. The intensity Im is the greatest value of the intensities I(u) in the pattern and occurs at the central maximum (where u ! 0), and f is the phase difference (in radians) between the top and bottom rays from the slit of width a. Study of Eq. 36-5 shows that intensity minima will occur where a ! mp,

for m ! 1, 2, 3, . . . .

(36-7)

If we put this result into Eq. 36-6, we find mp ! or

a sin u ! ml,

pa sin u, l

for m ! 1, 2, 3, . . . ,

for m ! 1, 2, 3, . . .

(minima — dark fringes),

(36-8)

which is exactly Eq. 36-3, the expression that we derived earlier for the location of the minima.

1089

36-2 I NTE NSITY I N SI NG LE-SLIT DI FFRACTION

Plots. Figure 36-8 shows plots of the intensity of a single-slit diffraction pattern, calculated with Eqs. 36-5 and 36-6 for three slit widths: a ! l, a ! 5l, and a ! 10l. Note that as the slit width increases (relative to the wavelength), the width of the central diffraction maximum (the central hill-like region of the graphs) decreases; that is, the light undergoes less flaring by the slit. The secondary maxima also decrease in width (and become weaker). In the limit of slit width a being much greater than wavelength l, the secondary maxima due to the slit disappear; we then no longer have single-slit diffraction (but we still have diffraction due to the edges of the wide slit, like that produced by the edges of the razor blade in Fig. 36-2).

Relative intensity 1.0 0.8 0.6 0.2 20

15

10

Proof of Eqs. 36-5 and 36-6 To find an expression for the intensity at a point in the diffraction pattern, we need to divide the slit into many zones and then add the phasors corresponding to those zones, as we did in Fig. 36-7. The arc of phasors in Fig. 36-9 represents the wavelets that reach an arbitrary point P on the viewing screen of Fig. 36-4, corresponding to a particular small angle u.The amplitude Eu of the resultant wave at P is the vector sum of these phasors. If we divide the slit of Fig. 36-4 into infinitesimal zones of width 2x, the arc of phasors in Fig. 36-9 approaches the arc of a circle; we call its radius R as indicated in that figure. The length of the arc must be Em, the amplitude at the center of the diffraction pattern, because if we straightened out the arc we would have the phasor arrangement of Fig. 36-7a (shown lightly in Fig. 36-9). The angle f in the lower part of Fig. 36-9 is the difference in phase between the infinitesimal vectors at the left and right ends of arc Em. From the geometry, f is also the angle between the two radii marked R in Fig. 36-9. The dashed line in that figure, which bisects f, then forms two congruent right triangles. From either triangle we can write Eu (36-9) sin 12f ! . 2R In radian measure, f is (with Em considered to be a circular arc)

Solving this equation for R and substituting in Eq. 36-9 lead to Eu !

Em sin 127. 1 f 2

(36-10)

Intensity. In Module 33-2 we saw that the intensity of an electromagnetic wave is proportional to the square of the amplitude of its electric field. Here, this means that the maximum intensity Im (at the center of the pattern) is proportional to E 2m and the intensity I(u) at angle u is proportional to E u2. Thus, I(u) E u2 ! . Im E m2

0.6

I(u ) ! Im

#

$.

The second equation we wish to prove relates a to u. The phase difference f between the rays from the top and bottom of the entire slit may be related to a path length difference with Eq. 36-4; it tells us that f!

# $ (a sin u), 2p l

where a is the sum of the widths 2x of the infinitesimal zones. However, f ! 2a, so this equation reduces to Eq. 36-6.

15

20

a = 5λ

∆θ

0.4 0.2 20

15

10

5 0 5 θ (degrees) (b)

10

Relative intensity 1.0 0.8

15

20

a = 10λ

0.6 0.4 0.2 15

10

5 0 5 θ (degrees) (c)

10

15

20

Figure 36-8 The relative intensity in single-slit diffraction for three values of the ratio a/l. The wider the slit is, the narrower is the central diffraction maximum.

α α

(36-11)

2

10

0.8

φ

R

Substituting for Eu with Eq. 36-10 and then substituting a ! 12f, we are led to Eq. 36-5 for the intensity as a function of u: sin a a

5 0 5 θ (degrees) (a)

Relative intensity 1.0

20

Em . f! R

a =λ

0.4

R



Em

φ

Em

Figure 36-9 A construction used to calculate the intensity in single-slit diffraction. The situation shown corresponds to that of Fig. 36-7b.

1090

CHAPTE R 36 DI FFRACTION

Checkpoint 3 Two wavelengths, 650 and 430 nm, are used separately in a single-slit diffraction experiment.The figure shows the results as graphs of intensity I versus angle u for the two diffraction patterns. If both wavelengths are then used simultaneously, what color will be seen in the combined diffraction pattern at (a) angle A and (b) angle B?

I

0

A

θ

B

Sample Problem 36.02 Intensities of the maxima in a single-slit interference pattern Find the intensities of the first three secondary maxima (side maxima) in the single-slit diffraction pattern of Fig. 36-1, measured as a percentage of the intensity of the central maximum.

#

I sin a ! Im a

$ # 2

!

1 sin(m ' 2)p (m ' 12)p

$, 2

for m ! 1, 2, 3, . . . .

The first of the secondary maxima occurs for m ! 1, and its relative intensity is

KEY IDEAS The secondary maxima lie approximately halfway between the minima, whose angular locations are given by Eq. 36-7 (a ! mp). The locations of the secondary maxima are then given (approximately) by a ! (m ' 12 )p,

intensities at those maxima, we get

for m ! 1, 2, 3, . . . ,

with a in radian measure. We can relate the intensity I at any point in the diffraction pattern to the intensity Im of the central maximum via Eq. 36-5. Calculations: Substituting the approximate values of a for the secondary maxima into Eq. 36-5 to obtain the relative

I1 ! Im

#

1 sin(1 ' 2)p (1 ' 12)p

$ # 2

!

sin 1.5p 1.5p

! 4.50 ( 10&2 % 4.5%.

$

2

(Answer)

For m ! 2 and m ! 3 we find that I2 ! 1.6% and Im

I3 ! 0.83%. Im

(Answer)

As you can see from these results, successive secondary maxima decrease rapidly in intensity. Figure 36-1 was deliberately overexposed to reveal them.

Additional examples, video, and practice available at WileyPLUS

36-3 DIFFRACTION BY A CIRCULAR APERTURE Learning Objectives After reading this module, you should be able to . . .

36.13 Describe and sketch the diffraction pattern from a small circular aperture or obstacle. 36.14 For diffraction by a small circular aperture or obstacle, apply the relationships between the angle u to the first minimum, the wavelength l of the light, the diameter d of the aperture, the distance D to a viewing screen, and the distance y between the minimum and the center of the diffraction pattern. 36.15 By discussing the diffraction patterns of point objects,

explain how diffraction limits visual resolution of objects. 36.16 Identify that Rayleigh’s criterion for resolvability gives the (approximate) angle at which two point objects are just barely resolvable. 36.17 Apply the relationships between the angle uR in Rayleigh’s criterion, the wavelength l of the light, the diameter d of the aperture (for example, the diameter of the pupil of an eye), the angle u subtended by two distant point objects, and the distance L to those objects.

36-3 DI FFRACTION BY A CI RCU L AR APE RTU R E

1091

Key Ideas ● Diffraction by a circular aperture or a lens with diameter d produces a central maximum and concentric maxima and minima, with the first minimum at an angle u given by

l sin u ! 1.22 d

verge of resolvability if the central diffraction maximum of one is at the first minimum of the other. Their angular separation can then be no less than u R ! 1.22

(first minimum — circular aperture).

● Rayleigh’s criterion suggests that two objects are on the

l d

(Rayleigh’s criterion),

in which d is the diameter of the aperture through which the light passes.

Diffraction by a Circular Aperture Here we consider diffraction by a circular aperture — that is, a circular opening, such as a circular lens, through which light can pass. Figure 36-10 shows the image formed by light from a laser that was directed onto a circular aperture with a very small diameter. This image is not a point, as geometrical optics would suggest, but a circular disk surrounded by several progressively fainter secondary rings. Comparison with Fig. 36-1 leaves little doubt that we are dealing with a diffraction phenomenon. Here, however, the aperture is a circle of diameter d rather than a rectangular slit. The (complex) analysis of such patterns shows that the first minimum for the diffraction pattern of a circular aperture of diameter d is located by sin u ! 1.22

l d

(first minimum — circular aperture).

(36-12)

The angle u here is the angle from the central axis to any point on that (circular) minimum. Compare this with Eq. 36-1, sin u !

l a

(first minimum — single slit),

Courtesy Jearl Walker

Figure 36-10 The diffraction pattern of a circular aperture. Note the central maximum and the circular secondary maxima.The figure has been overexposed to bring out these secondary maxima, which are much less intense than the central maximum.

(36-13)

which locates the first minimum for a long narrow slit of width a. The main difference is the factor 1.22, which enters because of the circular shape of the aperture.

Resolvability

Figure 36-11 At the top, the images of two point sources (stars) formed by a converging lens. At the bottom, representations of the image intensities. In (a) the angular separation of the sources is too small for them to be distinguished, in (b) they can be marginally distinguished, and in (c) they are clearly distinguished. Rayleigh’s criterion is satisfied in (b), with the central maximum of one diffraction pattern coinciding with the first minimum of the other.

Courtesy Jearl Walker

The fact that lens images are diffraction patterns is important when we wish to resolve (distinguish) two distant point objects whose angular separation is small. Figure 36-11 shows, in three different cases, the visual appearance and corresponding intensity pattern for two distant point objects (stars, say) with small

(a)

(b)

(c)

1092

CHAPTE R 36 DI FFRACTION

angular separation. In Figure 36-11a, the objects are not resolved because of diffraction; that is, their diffraction patterns (mainly their central maxima) overlap so much that the two objects cannot be distinguished from a single point object. In Fig. 36-11b the objects are barely resolved, and in Fig. 36-11c they are fully resolved. In Fig. 36-11b the angular separation of the two point sources is such that the central maximum of the diffraction pattern of one source is centered on the first minimum of the diffraction pattern of the other, a condition called Rayleigh’s criterion for resolvability. From Eq. 36-12, two objects that are barely resolvable by this criterion must have an angular separation uR of uR ! sin&1

1.22l . d

Since the angles are small, we can replace sin uR with uR expressed in radians: uR ! 1.22

l d

(Rayleigh’s criterion).

(36-14)

Human Vision. Applying Rayleigh’s criterion for resolvability to human vision is only an approximation because visual resolvability depends on many factors, such as the relative brightness of the sources and their surroundings, turbulence in the air between the sources and the observer, and the functioning of the observer’s visual system. Experimental results show that the least angular separation that can actually be resolved by a person is generally somewhat greater than the value given by Eq. 36-14. However, for calculations here, we shall take Eq. 3614 as being a precise criterion: If the angular separation u between the sources is greater than uR, we can visually resolve the sources; if it is less, we cannot. Pointillism. Rayleigh’s criterion can explain the arresting illusions of color in the style of painting known as pointillism (Fig. 36-12). In this style, a painting is made not with brush strokes in the usual sense but rather with a myriad of small colored dots. One fascinating aspect of a pointillistic painting is that when you change your distance from it, the colors shift in subtle, almost subconscious ways. This color shifting has to do with whether you can resolve the colored dots. When you stand close enough to the painting, the angular separations u of adjacent dots are greater than uR and thus the dots can be seen individually. Their colors are the true colors of the paints used. However, when

Figure 36-12 The pointillistic painting The Seine at Herblay by Maximilien Luce consists of thousands of colored dots. With the viewer very close to the canvas, the dots and their true colors are visible. At normal viewing distances, the dots are irresolvable and thus blend.

Maximilien Luce, The Seine at Herblay, 1890. Musée d’Orsay, Paris, France. Photo by Erich Lessing/Art Resource

36-3 DI FFRACTION BY A CI RCU L AR APE RTU R E

1093

you stand far enough from the painting, the angular separations u are less than uR and the dots cannot be seen individually. The resulting blend of colors coming into your eye from any group of dots can then cause your brain to “make up” a color for that group — a color that may not actually exist in the group. In this way, a pointillistic painter uses your visual system to create the colors of the art. When we wish to use a lens instead of our visual system to resolve objects of small angular separation, it is desirable to make the diffraction pattern as small as possible. According to Eq. 36-14, this can be done either by increasing the lens diameter or by using light of a shorter wavelength. For this reason ultraviolet light is often used with microscopes because its wavelength is shorter than a visible light wavelength.

Checkpoint 4 Suppose that you can barely resolve two red dots because of diffraction by the pupil of your eye. If we increase the general illumination around you so that the pupil decreases in diameter, does the resolvability of the dots improve or diminish? Consider only diffraction. (You might experiment to check your answer.)

Sample Problem 36.03 Pointillistic paintings use the diffraction of your eye Figure 36-13a is a representation of the colored dots on a pointillistic painting. Assume that the average centerto-center separation of the dots is D ! 2.0 mm. Also assume that the diameter of the pupil of your eye is d ! 1.5 mm and that the least angular separation between dots you can resolve is set only by Rayleigh’s criterion. What is the least viewing distance from which you cannot distinguish any dots on the painting? KEY IDEA Consider any two adjacent dots that you can distinguish when you are close to the painting. As you move away, you continue to disting